本页面汇集了初版“悠闲数学娱乐论坛”(kkkkuingggg.5d6d.net)所有存档的帖子,纯文字,用于搜索。

方法:用浏览器的搜索功能定位到关键字,双击内容打开相应帖子。手机若双击无效可点击上方的链接。

thread-1-1-1.html: 初步支持简单 LaTeX 输入
kuing 1# 2011-9-25 20:46
基于 MathJax,大致与 $\LaTeX$ 基本用法差不多,支持部分简单环境的输入。 右键公式有选项可以显示具体代码。 目前有一个问题还没解决,就是还没做到将连续多个空格自动变成一个空格,待研究。
kuing 2# 2011-9-25 20:47
示例: 已知 $a,b,c$ 是实数,且 $a+b+c=1$,求 $4^a + 4^b + 4^{c^2 }$ 的最小值。 由均值不等式,有 \begin{align}   4^a + 4^b + 4^{c^2 } & \geqslant 2\sqrt {4^a \cdot 4^b } + 4^{c^2 }\\   & = 2^{1 - c} + 2^{1 - c} + 2^{2c^2 }  \\   & \geqslant 3\sqrt[3]{{2^{1 - c} \cdot 2^{1 - c} \cdot 2^{2c^2 } }}\\   & = 3 \cdot 2^{\frac{2}{3}\left( {c - \frac{1}{2}} \right)^2 + \frac{1}{2}}  \\   & \geqslant 3\sqrt 2 , \end{align} 当 $a = b = \frac{1}{4},c = \frac{1}{2}$ 时 $4^a + 4^b + 4^{c^2 } = 3\sqrt 2$,即能取等号,故最小值就是 $3\sqrt 2$。 空格 $\sqrt[4]{a^4}=|a|$,$a+b+        c=1$
kuing 3# 2011-9-25 21:15
更多功能有待研究中………… \[\sqrt{\frac{x^2+y^2}{2}}+\frac{2}{\frac{1}{x}+\frac{1}{y}} \geqslant \frac{x+y}{2}+\sqrt{xy},公式中文直接支持哈     哈\]
测试号 4# 2011-9-25 22:02
我也试试 $\sum\frac{a^2}{b^2+c^2}\ge\frac32$
测试号 5# 2011-9-25 22:03
行间公式再来 \[\sum\frac{a^2}{b^2+c^2}\ge\frac32\]
thread-10-1-1.html: k 似乎基本输入代码中少对“{”“}”
isea 1# 2011-9-26 14:10
本帖最后由 isea 于 2011-9-26 14:21 编辑 如数列$\displaystyle\{a_n\}$,$a_n=|n-k|+|n-2k|$,若对任意正整数$n,a_n\geqslant a_3=a_4$都成立,则实数k的取值范围为多少?
isea 2# 2011-9-26 14:17
本帖最后由 isea 于 2011-9-26 14:28 编辑 \begin{align} &a_3=|3-k|+|3-2k|\\ &a_4=|4-k|+|4-2k|\\ &|3-k|+|3-2k|=a_3=a_4=|4-k|+|4-2k| \end{align}
kuing 3# 2011-9-26 14:20
已经补上
isea 4# 2011-9-26 22:43
本帖最后由 isea 于 2011-9-26 22:45 编辑 \vec a $\vec a$
kuing 5# 2011-9-26 22:50
\vec a $\vec a$ isea 发表于 2011-9-26 22:43 嗯,不过这个只适用于单个字母。比较小
thread-100-1-3.html: 来自群的三角形最小边求参数最小值 $a^2+b^2>mc^2$
kuing 1# 2011-10-15 21:47
题目:设 $a$、$b$、$c$ 是三角形的三边长,$m$ 为正整数,只要 $a^2+b^2>mc^2$,就有 $c$ 是该三角形最短边,则 $m$ 的最小值是(  )。 这个题貌似很容易把逻辑搞乱了,下面玩玩。 我们不限 $m$ 为正整数,改成:求实数 $m$ 的范围,使得 对满足 $a^2+b^2>mc^2$ 的所有三角形三边 $a,b,c$ 都满足 $a\geqslant c \wedge b\geqslant c$。 这也等价于: 对满足 $a<c \vee b<c$ 的所有三角形三边 $a,b,c$ 都满足 $a^2+b^2\leqslant mc^2$。 故只要求出当 $c$ 不是最短边时 $\dfrac{a^2+b^2}{c^2}$ 的上确界,只要 $m$ 不小于这个上确界即可。 若 $c$ 为最大边,那么 \[\frac{a^2+b^2}{c^2}\leqslant \frac{c^2+c^2}{c^2}=2,\] 当 $a=b=c$ 时取等; 若 $c$ 不是最大边,由于也不是最短边,由对称性,不妨设 $a<c<b$,则由三边的条件,有 \[b<c+a<2c\implies \frac bc<2,\] 故 \[\frac{a^2+b^2}{c^2}< \frac{c^2+b^2}{c^2}=1+\left(\frac bc\right)^2<5,\] 下面证明这个 $5$ 就是上确界。取 $a=1-t, c=1+t, b=2-u, t, u>0$ 且 $t+u<1$,这显然满足 $a<c<b<c+a$,此时令 $t,u\to0$ 则显然 $\dfrac{a^2+b^2}{c^2}\to5$。 故此综上,当 $c$ 不是最短边时 $\dfrac{a^2+b^2}{c^2}$ 的上确界是 $5$。因此,$m$ 的取值范围是 $[5,+\infty)$。
kuing 2# 2011-10-15 22:56
后面分类有点多余,完全不必分类,只要不妨设 $c>a$ 后面照样。
nash 3# 2011-10-15 23:05
kuing威武
kuing 4# 2011-10-15 23:14
3# nash 咳,别说威武了,那天好像是你先在群里发的,我当时看上去应该没什么难,我没理会就让你们玩。刚才看回才发现我一开始差点把逻辑搞错了,呵呵,所以还是写了写。再说,现在还是有多余的东西,改良一下写成如下: 求实数 $m$ 的范围,使得对满足 $a^2+b^2>mc^2$ 的所有三角形三边 $a,b,c$ 都满足 $a\geqslant c \wedge b\geqslant c$。 等价于:对满足 $a<c \vee b<c$ 的所有三角形三边 $a,b,c$ 都满足 $a^2+b^2\leqslant mc^2$。 可见只要求出当 $c$ 不是最短边时 $\dfrac{a^2+b^2}{c^2}$ 的上确界,只要 $m$ 不小于这个上确界即可。 由 $a,b$ 的对称性,不妨设 $a\leqslant b$,则由 $c$ 不是最短边,必有 $a<c$。由三边的条件,有 \[b<c+a<2c\implies \frac bc<2,\] 故 \[\frac{a^2+b^2}{c^2}< \frac{c^2+b^2}{c^2}=1+\left(\frac bc\right)^2<5,\] 下面证明这个 $5$ 就是上确界。取 $a=1-t, c=1+t, b=2-u, t>0, 1\geqslant u>0$,这显然满足所设,此时令 $t,u\to0$ 则显然 $\dfrac{a^2+b^2}{c^2}\to5$,因此当 $c$ 不是最短边时 $\dfrac{a^2+b^2}{c^2}$ 的上确界是 $5$。 所以,$m$ 的取值范围是 $[5,+\infty)$。
yes94 5# 2013-3-31 14:18
4# kuing 当$m=5$时,$a^2+b^2=5c^2$有一个奇妙的垂直性质。
thread-1000-1-5.html: [组合] 正八面体的顶点染色
realnumber 1# 2012-12-31 09:23
本帖最后由 realnumber 于 2013-1-1 10:10 编辑 用红蓝两种颜色染色一个正八面体的顶点,要求每个顶点都染上色,有多少种染法?能旋转重合认为是同一种 做出后换3色,或4色或5色,或6色
realnumber 2# 2012-12-31 09:41
0红,  1种 1红5蓝,1种 2红4蓝,2种,(2红相对顶点与相邻顶点) 3红3蓝,2种,(有2个为一对顶点或3红为一个面) 4红2蓝,2,同上 5红       1 6红       1 合计有10种
realnumber 3# 2012-12-31 10:23
本帖最后由 realnumber 于 2012-12-31 22:14 编辑 1色1种; 2色2+8=10种; 3色57种如下: 6顶点同一种颜色3种, 6顶点两种颜色$8×C_3^2=24$种,8已在2楼得出. 6顶点三种颜色齐全合计30种.①1个顶点红2个黄3蓝合计5种,其中2黄为相对顶点1种,2黄为相邻顶点的有2种,1黄2红3蓝等都一样多,---合计$3A_3^3=18$种;②2红2黄2蓝,同色恰好都是相对的顶点,1种;有且仅有某1种颜色恰好是相对顶点,3种;同色都没有在相对顶点的,2种;---合计6种;③4红1黄1蓝,黄蓝在相对顶点1种,黄蓝顶点相邻1种,4蓝1红1黄等同样多,合计2×3=6种; 4色216种如下: 同一种颜色4种, 两种颜色$8×C_4^2=48$种, 三种颜色$30×C_4^3=120$, 四色齐全合计44种.①3红1黄1蓝1绿,3红任何2点都不在相对顶点有2种,3红有2个在相对顶点3种;3黄1红1蓝1绿等也一样多,合计有$5C_4^1=20$种;②2红2黄1蓝1绿,红黄都在相对的顶点,1种,合计$C_4^2=6$种,红黄仅1色在相对顶点,比如红,有2种,合计$2C_4^1C_3^1=24$种 5色740种如下: 同一种颜色5种, 两种颜色$8×C_5^2=80$种, 三种颜色$30×C_5^3=300$, 四色$44×C_5^4=220$种. 五种颜色齐全135种.2红其它1,2红在相对顶点,3种;2红在相邻顶点$A_4^4=24$种;合计$(24+3)C_5^1=135$种 6色2226种如下: 同一种颜色6种, 两种颜色$8×C_6^2=120$种, 三种颜色$30×C_6^3=600$, 点四色$44×C_6^4=660$种. 五色$135×C_6^5=810$种 6色齐全的30种.与红色相对顶点的颜色有$C_5^1$种,再接下来选定第三个顶点,与它相对的有$C_3^1$种,所以一共$15C_2^1=30$种 $n(n\ge{7})$色有$n+8C_n^2+30C_n^3+44C_n^4+135C_n^5+30C_n^6$种, -----ps,有错误通知我下,旋转不能重合的算不同,
abababa 4# 2012-12-31 17:53
2种的好算,请问有答案吗?一位网友说他只解用3种颜色和6种颜色涂的,说3种的有57种方法,6种的有2226种方法,感觉好多啊,具体怎么做他没说就下线了,楼主得出3种的方法了吗?是57吗?
realnumber 5# 2012-12-31 19:30
没答案的,想不到真有人也在玩啊,我会写在2楼的3~7色。看来直接求比如5色很难,但逐步从2色~7色几乎没什么难度了.
realnumber 6# 2012-12-31 21:21
正方体也应该可以这样出来吧,只需要耐心和细心
realnumber 7# 2012-12-31 21:47
1 波利亚计数定理(萧文强).pdf 有网友推荐看这个,新浪爱问上有免费下
abababa 8# 2012-12-31 21:50
1黄2红3蓝,2红为相邻顶点的有4种,我就找出来两种 第一种是3,4是红,2是黄 第二种是3,4是红,1是黄 还有哪两种?
realnumber 9# 2012-12-31 21:56
8# abababa 果然错了,已经在原贴修改
abababa 10# 2012-12-31 22:00
这样三种颜色就真有57种涂法啊,看来4楼那位网友的解答可能是正确的
abababa 11# 2012-12-31 22:06
4种往上我想不动了,不弄了 请问楼主知道用什么软件画立体几何图形好点吗?
realnumber 12# 2012-12-31 22:15
不知道,6种我检查了,是漏了,这下很有可能都对了
abababa 13# 2012-12-31 22:20
刚问了下4色和5色的,那位网友说是240种和800种,没说怎么算的,我是算不动了,楼主再看看?
realnumber 14# 2012-12-31 22:43
本帖最后由 realnumber 于 2013-1-6 09:44 编辑 如果这样,会比6色2226多很多.应该不会错了,如果6色是2226的话,因为我的算法,前面的影响后面. 正四面体. 依次是1~4色,再n色,有几种? 补充说明:旋转能重合的算同一种;旋转不能重合,但镜像对称的算不同;颜色可以用一部分,顶点必须都染色. 1色 就1种. 2色 2+3=5种. 3色 $3+3C_3^2+3=15$种. 4色 $4+3C_4^2+3C_4^3+2=36$种. n色($n\ge{5}$)$C_n^1+3C_n^2+3C_n^3+2C_n^2$种. 正方体这样做累,看来得看那个网友推荐的.
abababa 15# 2012-12-31 23:18
虽然我这么说没什么根据,但会不会有可能前边少算了,后边多算了呢,一加一减也有可能得数正确 但我没根据,4以上的我自己也想不动,完全是按网友给的答案来的,他就说个答案,可能是想让我动脑想想吧,但确实想不动了
realnumber 16# 2012-12-31 23:46
恩,还会不时来检验的,刚才给小孩洗澡去了
thread-1001-1-5.html: [几何] 来自人教群的一道平几
kuing 1# 2012-12-31 17:47
哎,进群又不自觉改群名片,还这样……怎么说,初中题也有难的,何况这道平几题我也没秒到,好不容易才想到了证明。就不能给点时间别人慢慢想?需要这样刷屏来催么?会的自然答你,不会的催也没用,甚至觉得厌烦,要是N年前,我干脆就不理了。不过我现在够闲,所以后来还是把下面略证发了……
isea 2# 2013-1-3 20:24
看图好熟悉,横着一看,哦,原来是这道经典竞赛题呀,经常拿来作为四点共圆的例子 http://bbs.pep.com.cn/forum.php? ... d=370978&page=1
kuing 3# 2013-1-3 20:29
2# isea 你的找贴能力也不错哇,初中区我去得少,挖不来
Gauss门徒 4# 2013-1-4 15:58
以前做过,平行四边形+员米定理
kuing 5# 2013-1-4 16:13
以前做过,平行四边形+员米定理 Gauss门徒 发表于 2013-1-4 15:58 员米定理?没听过的说……
yes94 6# 2013-1-4 19:14
和我一样用拼音,圆幂定理
kuing 7# 2013-1-4 19:35
原来如此……
thread-1002-1-1.html: 关于波浪符号
abababa 1# 2012-12-4 20:43
本帖最后由 abababa 于 2012-12-31 18:29 编辑 问一下,~怎么在两个美元号里打出来?比如1~10 $1~10$
kuing 2# 2012-12-4 20:54
2# abababa \sim
abababa 3# 2012-12-4 21:14
谢谢。 我看到\sim是相似的意思,也与那个波浪线用同一个表示吗?
kuing 4# 2012-12-4 21:38
4# abababa 1\sim10 $1\sim10$ 1\text{~}10 $1\text{~}10$
abababa 5# 2012-12-5 18:59
5# kuing 谢谢。 就是说这些符号没有固定的意义,只要最后结果看上去是那样就行吧,\sim不一定只代表相似,所有跟它样子一样的都用这个表示。
kuing 6# 2012-12-5 19:42
一般来说不产生歧义就行了,如果要专业点的话其实我也不清楚。 顺便说一下,真正的 LaTeX 里 \$1\text{~}10\$ 也不会出现波浪,而是要 \$1\text{\~{}}10\$(非公式环境里面直接 1\~{}10),但波浪并不居中而是在较上方,所以一般来说还是在公式环境里用 \sim 好了。
kuing 7# 2012-12-31 18:25
已分割
abababa 8# 2012-12-31 18:30
谢谢管理员分割
thread-1003-1-5.html: [不等式] 来自人教论坛的关于“IMO42-2”的“新推广”
kuing 1# 2012-12-31 19:42
来自:http://bbs.pep.com.cn/forum.php?mod=viewthread&tid=2637145 已知 $a$, $b$, $c$ 是正数,$n$ 是正整数,求证: \[\frac a{\sqrt[n]{a^n+(3^n-1)b^{\frac n2}c^{\frac n2}}}+\frac b{\sqrt[n]{b^n+(3^n-1)c^{\frac n2}a^{\frac n2}}}+\frac c{\sqrt[n]{c^n+(3^n-1)a^{\frac n2}b^{\frac n2}}}\geqslant1.\] 贴里说是新推广,其实这跟我09年做安振平的征解题时的一道题等价。 具体地,作置换 $a^{\frac n2}\to a$, $b^{\frac n2}\to b$, $c^{\frac n2}\to c$,那么原不等式等价于 \[\sqrt[n]{\frac{a^2}{a^2+(3^n-1)bc}}+\sqrt[n]{\frac{b^2}{b^2+(3^n-1)ca}}+\sqrt[n]{\frac{c^2}{c^2+(3^n-1)ab}}\geqslant1,\] 而这就是附件中的题目。
thread-1005-1-5.html: 请教2012清华大学保送生一题(第二问)
wenshengli 1# 2013-1-1 15:35

abababa 2# 2013-1-1 15:55
1# wenshengli 发一位网友的解答,取AB的中点C,取$OA, OB$中点$C_1, C_2$,则得四个三角形,从上到下从左到右分别标为1234号 将13号视为一组,24号视为一组,讨论这两组,对于13号,所有点的横坐标都小于1,对于24号,所有点的纵坐标都小于1 若要使13号的所有点的横坐标大于6,必须在这一组里有6个以上的点,若要使24号中点的纵坐标之和大于6,必须在这一组里有6个以上的点,从而总点数大于12,与已知11点矛盾
abababa 3# 2013-1-1 16:06
2# abababa 感觉这个解答有问题,要是点都在最右边就不能这么解释了
abababa 4# 2013-1-1 20:26
本帖最后由 abababa 于 2013-1-1 20:57 编辑 3# abababa 突然想到$x+y$不超过2,能不能从这方面入手呢? 先把它们的横坐标从小到大排序,$x_1$最小$x_{11}$最大,然后从小往大加,总有一个数使得它前边的数的和不大于6,但再加下一个就比6大 就是必然有一个$n$使得$\sum_{k=1}^{n}x_k \leqslant 6(n \leqslant 11)$,但$\sum_{k=1}^{n+1}x_k>6$ 这时候如果能证明从$n+1$开始有$\sum_{k=n+1}^{11}y_k \leqslant 6$就达到目的了 $\sum_{k=n+1}^{11}y_k+\sum_{k=n+1}^{11}x_k \leqslant 2*(11-(n+1)+1)=22-2n$ $\sum_{k=n+1}^{11}y_k \leqslant 22-2n-\sum_{k=n+1}^{11}x_k$,只要能证明$22-2n-\sum_{k=n+1}^{11}x_k \leqslant 6$就行了,就是证明$16-2n \leqslant \sum_{k=n+1}^{11}x_k$ 然后因为排序过,$x_{k+2}$肯定比$x_{k+1}$大,只要能证明$16-2n \leqslant (11-(n+1)+1)x_{n+1}$ 又有$6<\sum_{k=1}^{n+1}x_k \leqslant (n+1)(x_{n+1})$,所以$x_{n+1}>\frac{6}{n+1}$ 只要能证明$16-2n \leqslant  (11-(n+1)+1)\frac{6}{n+1}$,就是证明$n^2-10n+25 \geqslant 0$恒成立,这就好办了
wenshengli 5# 2013-1-1 22:42
4# abababa 谢谢!
thread-1006-1-5.html: [数列] 这个关于数列第三项相等的说法是对还是错[解决]
isea 1# 2013-1-1 21:40
本帖最后由 isea 于 2013-1-2 17:38 编辑 一个等比数列$\{a_n\}$和一个等差数列$\{b_n\}$中,如果$a_k=b_k\ne a_m=b_m$($k,m$都是正整数),则对任意的不同于$k,m$的正整数$n$都有$a_n\ne b_n$。 以上,这个命题是真是假?先谢。
kuing 2# 2013-1-1 21:57
公比是负的话可能会有第三个。 正的话正确
abababa 3# 2013-1-1 21:59
好像能用直线和指数函数的交点?
realnumber 4# 2013-1-1 22:00
如果加条件公比q>0,那么为真命题,指数函数(凹或凸)和直线从图形看,最多2公共点. 否则$a_n=(-2)^n$,$b_n=6n-8$,n=1,2,4,三项相等.
isea 5# 2013-1-2 17:37
多谢楼上三位指点! 偶,完全没考虑到公比为负的时候!!!受教了。
thread-1007-1-5.html: [不等式] 一个优美的不等式
reny 1# 2013-1-2 22:03
本帖最后由 reny 于 2013-1-2 22:07 编辑 设$x_1,x_2,x_3,x_4为正数,且\prod_{i=1}^{4}{x_i}=1,证明:\sum_{i=1}^{4}\frac{1}{1+x_i+x_i^2+x_i^3}\ge1$ PS、我准备用函数$y=1+e^x+e^{2x}+e^{3x}的凸凹性再结合Jensen$不等式来证明,可是这个函数是先上凸再下凸的,解决不了。 高手们,有没有好的方法呢?谢谢大家思考一下。
kuing 2# 2013-1-2 22:11
早前见过,除了半凹半凸定理之外暂时没什么办法。 不过印象中杨学枝有个证明,有空找找看。
reny 3# 2013-1-2 22:36
本帖最后由 reny 于 2013-1-2 22:37 编辑 2# kuing 杨学枝的解法在$P66$我看过,通过构造了$$\frac{1}{1+x_1+x_1^2+x_1^3}+\frac{1}{1+x_2+x_2^2+x_2^3}\ge\frac{1}{1+\sqrt{x_1^3x_2^3}}$$解决了,不过对这个构造的不等式的证明看起挺复杂的。
kuing 4# 2013-1-2 22:58
翻到了这个链接 http://www.artofproblemsolving.c ... p?f=52&t=314320
reny 5# 2013-1-3 11:04
4# kuing 半凹半凸定理对于证明本题是个好方法。
reny 6# 2013-1-3 23:16
请教一下,对于$a,b,c\in R^+,记s_1=a+b+c,s_2=ab+bc+ca,s_3=abc, 能否证明9s_3^2-4s_1s_3+1\ge0?$
thread-1008-1-5.html: 来自某教师群的一道求系数题
kuing 1# 2013-1-2 23:30
题目:已知 $n\in\mbb N^+$,若对任意实数 $x$,都有 $x^n=a_0+a_1(x-n)+a_2(x-n)^2+\cdots+a_n(x-n)^n$,则 $a_{n-1}$ 的值为(选项略) 广州kuing  23:13:08 显然 $a_n=1$,于是 $a_{n-1}$ 为 $(x-n)^n$ 展开后的 $x^{n-1}$ 的系数的相反数,即 $n^2$。 广州kuing  23:20:15 或者两边对 $x$ 求 $n-1$ 阶导数,可得 $n!x=(n-1)!a_{n-1}+n!a_n(x-n)$,再令 $x=n$,即得 $a_{n-1}=n^2$。
isea 2# 2013-1-3 16:13
求导,很赞的思路。 若从学生角度出发,将$x^n=(x-n+n)^n$按$C_n^r(x-n)^{n-r}n^{r}$展开,相对好理解一点
kuing 3# 2013-1-3 16:16
2# isea right
kuing 4# 2013-1-3 16:19
应该说你的方法更好,一下全出来了
isea 5# 2013-1-3 16:22
不敢当不敢当,实质就是思路一,再详细说了一下。 楼上看问题眼光又“准”确又“独”到 真的受益
thread-1009-1-5.html: [不等式] 昨晚人教群的不等式没时间写
kuing 1# 2013-1-3 14:27
若 $k>1/2$,则不等式成立,并且等号取不了。 \[k^a\leqslant \frac1{a+1}\left( \left( k+\frac12 \right)^{a+1}-\left( k-\frac12 \right)^{a+1} \right)\iff\frac1k\leqslant \frac1{a+1}\left( \left( 1+\frac1{2k} \right)^{a+1}-\left( 1-\frac1{2k} \right)^{a+1} \right),\] 令 $1/(2k)=t$,则由 $k>1/2$ 知 $t\in(0,1)$,则不等式等价于 \[\frac1{a+1}\bigl((1+t)^{a+1}-(1-t)^{a+1}\bigr)\geqslant 2t,\] 于是令 \begin{align*} f(t)&=\frac1{a+1}\bigl((1+t)^{a+1}-(1-t)^{a+1}\bigr)-2t, \\ f'(t)&=(1+t)^a+(1-t)^a-2, \\ f''(t)&=a\bigl((1+t)^{a-1}-(1-t)^{a-1}\bigr), \end{align*} 可见 $f(0)=f'(0)=0$ 且由 $a<0$ 可得 $f''(t)>0$,所以对任意 $t\in(0,1)$ 都有 $f(t)>0$,得证。 而对于某些特别的 $a$,则 $k$ 就可以取比 $1/2$ 更小的值,这时不等式不成立。 比如说当 $a=-2$ 时,原不等式为 \[\frac1{k^2}\leqslant \frac1{k-\frac12}-\frac1{k+\frac12},\] 上式对 $k\in(-1/2,1/2)$ 反向成立。
thread-101-1-9.html: [不等式] 请教两个Tran Quoc Anh 的题
pxchg1200 1# 2011-10-15 23:15
probelm 1: Let $ a,b,c \geq 0 $ prove that: \begin{align} (a+b)^{2}(b+c)^{2}(c+a)^{2}\geq \frac{64}{3}abc(a^{2}b+b^{2}c+c^{2}a) \end{align} Problem 2: With the same condition as problem 1,prove that: \begin{align} (a^{2}+b^{2}+c^{2})(a+b+c)\geq 3 \sqrt{3abc(a^{3}+b^{3}+c^{3})} \end{align} PS:谢绝 UVW 和 PQR, ABC method..
pxchg1200 2# 2011-10-20 16:14
怎么没人做啊? kuing呢?
kuing 3# 2011-10-20 16:15
2# pxchg1200 最近在谷底……
pxchg1200 4# 2011-10-24 14:56
昨天把第一题做出来了,AM-GM还要讨论。。 (有时间再贴答案)
kuing 5# 2011-10-24 17:59
我也做了下第一题,用老招。 Let $ a,b,c \geqslant 0 $, prove that: \[(a+b)^2(b+c)^2(c+a)^2\geqslant \frac{64}3abc(a^2b+b^2c+c^2a).\] 由轮换对称性,不妨设 $b$ 在 $a,c$ 之间,即 $(b-a)(b-c)\leqslant0$,由此可得 \[a^2b+b^2c+c^2a\leqslant b(a^2+c^2+ac),\] 于是由均值得 \[a^2b+b^2c+c^2a \leqslant \frac{b\bigl(b(c+a)+ca\bigr)\bigl((c+a)^2-ca\bigr)}{ab+bc+ca}\leqslant \frac{b(c+a)^2(a+b+c)^2}{4(ab+bc+ca)},\] 于是只要证 \[\frac{16ab^2c(a+b+c)^2}{3(ab+bc+ca)}\leqslant (a+b)^2(b+c)^2,\] 又由 \[ab+bc+ca\geqslant ab+bc+ca+\frac12(b-a)(b-c)=\frac{b(a+b+c)+3ca}2,\] 故只要证 \[\frac{32ab^2c(a+b+c)^2}{3b(a+b+c)+9ca}\leqslant (a+b)^2(b+c)^2,\] 或 \[\frac{32ca\bigl(b(a+b+c)\bigr)^2}{3b(a+b+c)+9ca}\leqslant \bigl(b(a+b+c)+ca\bigr)^2,\] 令 $p=ca$, $q=b(a+b+c)$,上式为 \[\frac{32pq^2}{9p+3q}\leqslant(p+q)^2\iff \frac{(p+3q)(3p-q)^2}{9p+3q}\geqslant0, \] 显然成立,故原不等式得证。
pxchg1200 6# 2011-10-24 22:22
本帖最后由 pxchg1200 于 2011-10-24 22:57 编辑 kuing 终于犀利了! 下面是我的证明: Case1 :\[ 3\sum{ab^{2}}\geq \sum{a^{2}b}+6abc \] By AM-GM gives: \[ \frac{64}{9}(3abc)(a^{2}b+b^{2}c+c^{2}a)\leq \frac{16}{9}(a^{2}b+b^{2}c+c^{2}a+3abc)^{2} \] Thus it suffice to check that: \[ 3(a+b)(b+c)(c+a)\geq 4(a^{2}b+b^{2}c+c^{2}a+3abc) \] \[ 3\sum{ab^{2}}\geq \sum{a^{2}b}+6abc \] Case 2: \[ 3\sum{ab^{2}}\leq \sum{a^{2}b}+6abc \] use the well-know inequality: \[ (x+y+z)^{2}\geq 3(xy+yz+xz) \] Let $x= ab^{2}, y=bc^{2}, z=ca^{2} $ Therefore: \[ \frac{64}{3}(a^{3}b^{2}c+b^{3}c^{2}a+c^{3}a^{2}b)\leq \frac{64}{9}(ab^{2}+bc^{2}+ca^{2})^{2} \] Then,it suffices to prove that: \[ 8(ab^{2}+bc^{2}+ca^{2})\leq 3(a+b)(b+c)(c+a) \] or \[ 3(a^{2}b+b^{2}c+c^{2}a)+6abc\geq 5(ab^{2}+bc^{2}+ca^{2}) \] but \[ 3(a^{2}b+b^{2}c+c^{2}a)+6abc\geq 3(3(ab^{2}+bc^{2}+ca^{2})-6abc) +6abc\geq 5(ab^{2}+bc^{2}+ca^{2}) \] Which reduce to \[ ab^{2}+bc^{2}+ca^{2}\geq 3abc \] Which is obvious by AM-GM Done!
kuing 7# 2011-10-24 22:37
6# pxchg1200 倒数第二个式子中间漏了 +6abc ,其余的如果你展开没错的话暂时没看出什么问题嗯。
pxchg1200 8# 2011-10-24 22:57
本帖最后由 pxchg1200 于 2011-10-24 22:58 编辑 7# kuing Thanks. I have fixed it 不过你那个AM-GM的方法我还真没见过啊,怎么想到配那个 $ ab+bc+ca$啊?
kuing 9# 2011-10-24 23:05
8# pxchg1200 都说是老招,是我想起当年证http://www.artofproblemsolving.com/blog/38749的时候的东西,然后才想到这样弄的。
pxchg1200 10# 2011-10-24 23:10
9# kuing 果然犀利!   学习了。。。 
thread-1010-1-5.html: [几何] 来自人教群的一道解几小题
kuing 1# 2013-1-3 15:26
如果没看错的话,题目是: 以 $P(x_0,y_0)$ 为圆心,以 $\displaystyle r=\frac35x_0+3$ 为半径,$P(x_0,y_0)$ 在椭圆 $\displaystyle \frac{x^2}{25}+\frac{y^2}{16}=1$ 上,则圆 $P$ 所覆盖面积为? 关键其实就在于观察到那个 $r$。 记左右焦点分别为 $F_1(-3,0)$, $F_2(3,0)$,则由焦半径公式,有 \begin{align*} \abs{PF_1}&=a+ex_0=5+\frac35x_0,\\ \abs{PF_2}&=a-ex_0=5-\frac35x_0, \end{align*} 从而有 \begin{align*} \abs{PF_1}-r&=5+\frac35x_0-3-\frac35x_0=2,\\ \abs{PF_2}+r&=5-\frac35x_0+3+\frac35x_0=8, \end{align*} 这也就是说,圆 $P$ 与以左焦点为圆心,半径为 $2$ 的圆外切,与以右焦点为圆心,半径为 $8$ 的圆内切,所以所面积就是 $8^2\pi-2^2\pi=60\pi$。
kuing 2# 2013-1-3 15:39
这道题还是蛮不错的,就是在题目的表达上不够清晰,不过这或者这只是抄题下来的人省略了点东西。
kuing 3# 2013-1-3 15:49
配图 PS、那两个圆和椭圆都切于左顶点只是因为数据特殊,如果将 $r$ 改一下就不一定相切了。
thread-1011-1-1.html: [几何] 平行,平行,还是平行
isea 1# 2013-1-3 17:19
本帖最后由 isea 于 2013-1-3 18:49 编辑 如图,平行四边形$ABCD$,若$AF \sslash CE$,则$BF \sslash DE$,简单之极。 哪怕$E,F$跑到边的延长线上,依然有若$AF \sslash CE$,则$BF \sslash DE$。 我们把平行四边形砍一刀,变成梯形,如图,依然有若$AF \sslash CE$,则$BF \sslash DE$,有点意思了,想“秒”可不易。 跑到延长线上,依然有若$AF \sslash CE$,则$BF \sslash DE$,其实,跑不跑到延长线上,其实一样,换换字母即可将外圈变成梯形,所以还是同一个东东。 要证明这个问题,怕是少不了辅助线了。 本帖引出问题,下回开一新回复,简单看看证法。
kuing 2# 2013-1-3 17:22
路过…… PS、平行符号我在本论坛上专门定义了个命令 \sslash,效果 $AB\sslash CD$,看上去应该好些。当然,如果觉得麻烦,直接打两斜杠 // ,不强制。
isea 3# 2013-1-3 17:44
本帖最后由 isea 于 2013-1-3 18:00 编辑 2# kuing 呵呵,\sslash ,复制过去了,这个当然需要统一语言了。 \parallel 要是这个就好记多了。 ================ 晕,parallel 两竖直线啊,原来是
kuing 4# 2013-1-3 17:49
3# isea \parallel 是 $\parallel$ 默认就有的了
isea 5# 2013-1-3 17:56
本帖最后由 isea 于 2013-1-3 19:34 编辑 纯平面几何的方法就不写了,大家直接看人教论坛zheyic的解答。 解答过程的配图为:(主要是字母顺序问题) 延长BA, CD交于K。 AD//BC => KA/KB=KD/KC ...(1) DE//BF => KE/KB=KD/KF ...(2) (1)/(2)得:KA/KE=KF/KC => CE//AF 原链接:http://bbs.pep.com.cn/forum.php?mod=viewthread&tid=2524769 ========================================================= 看来,将两腰延长是关键! 那将平行进行到底—— 同一平面内六点,$A,B,C$三点共线;$D,E,F$三点共线,若$AE \sslash BF,BD \sslash CE$,则$CF \sslash AD$。 (图为示意图,只需要两组三点共线,若有两组平行,则另一组亦平行) 这事实不够"平易近人"(指不太像考试题目),改改点的位置,可得图形 并重新编一个字母顺序,可得到: 题目: 点$O$为在三角形$ABC$内一点,过点$O$作$OD \sslash BC$交$AB$于$D$,过$D$作$DE \sslash AC$交$CO$于$E$,连结$BE$。 求证:$BE \sslash AO$。(以下说的题目就是指这个) 或者直接写成四边形的形式,对角线上的点,等等 如果,你动手或思考这些图形各自证明,会发现AB与DE直线的交点这些图形的共同“特征”,用这个交点,证法都一是致的。 当然,如,此楼的题目,还有平几其它证法。 这里给一种向量证法(其实是自己熟悉一下,哈哈) 记$AB$与$DE$的交点为$O$,则由$AE \sslash BF,BD \sslash CE$,设$\vv {OB}=\lambda \vv {OA},OE=\mu \vv {OD}$,则 $\vv {OC}=\mu \vv {OB}=\mu\lambda \vv {OA}$,$\vv {OF}=\lambda \vv {OE}=\lambda\mu \vv {OD}$,下略 (好处是,由共线直接写比例关系,顺其自然)
isea 6# 2013-1-3 19:44
本帖最后由 isea 于 2013-1-3 19:48 编辑 以此为引理,则可以统一证明下面几何题。 这里仅给出曾经在人教论坛里写的第一个证明。 对于第2问,在以上证明中,只需要相应将条件改动,则有对应等腰三角形,从而得到角分线。 后记 再次在看到第2个题,是在张景中写的绕来绕去的向量法见到的,2003年德国竞赛题,再者,早想整理一下这些个相关的零散的东西,再者,特别想有个第2题的面积证明,所以今天干脆来个总结,顺便在线写写latex代码。
isea 7# 2013-1-3 19:54
本帖最后由 isea 于 2013-1-3 20:06 编辑 顺便说一下,在人教论坛原帖讨论中,0.1说可以用Menelaus定理及同一法证明, 不过,偶在找有无面积法的过程中(虽然目前无果),倒是可以用Ceva定理直接搞定。 对三角形$CDB$中,$CG,DR,BQ$相交于$G$,则 $\dfrac{DP}{PB}\dfrac{BR}{RC}\dfrac{CQ}{QD}=1$, $\dfrac{DP}{PB}\dfrac{BE}{CD}\dfrac{BC}{FD}=1$, 又$BE=DF$,有$\dfrac{DP}{PB}\dfrac{BC}{CD}=1$, 即$\dfrac{DP}{PB}=\dfrac{CD}{BC}$,……
isea 8# 2013-5-28 13:30
本帖最后由 isea 于 2013-5-28 13:35 编辑 以此为引理,则可以统一证明下面几何题。 663 这里仅给出曾经在人教论坛里写的第一个证明。 对于第2 ... isea 发表于 2013-1-3 19:44 终于找到第2题的面积法了! 给个辅助线,先。 虚线为平行四边形边的垂直,现在没时间(过程虽然很短),回头有空再写过程。
isea 9# 2013-5-28 22:32
本帖最后由 isea 于 2013-5-28 22:37 编辑 面积法(来自 仁者无敌面积法,张景中,彭翕成) \begin{align*} 2(S_{\triangle BED} -  S_{\triangle BEG})  &= 2(S_{\triangle BFD} - S_{\triangle  GFD})  \\ BE(MN-MG)&=DF(PQ-GP)\\ BE\cdot GN&=DF \cdot GQ\\ BE&=DF\\ \therefore GN&=GQ\\ GN&\perp DC\\GQ&\perp CB\\ \therefore \angle BCG&=\angle DCG \end{align*}
thread-1012-1-5.html: [几何] 在人教论坛看到的一道具有几何性质的椭圆题
kuing 1# 2013-1-4 00:31
来源:http://bbs.pep.com.cn/forum.php?mod=viewthread&tid=2638857 原题目:已知椭圆方程为 $x^2/4+y^2=1$,它的左右顶点分别为 $A_1$, $A_2$,左右焦点分别为 $F_1$, $F_2$,椭圆上有一点 $P$,直线 $A_1P$ 与直线 $A_2P$ 分别交直线 $x=4$ 于 $M$、$N$ 两点。求证:以 $MN$ 为直径的圆与直线 $PF_2$ 相切。 原题的解法在原贴链接中已经有,这里不再贴出,发在这里当然是要研究一下别的东东。 看到如此具有几何性质的题,当然不能错过研究其一般的情况以及其几何证法。 由于最终作出来的线会比较多,下面一步步来。首先很容易证明如下的: $AB$ 是圆 $O$ 的直径,$C$ 为圆上异于 $A$, $B$ 的任一点,过 $B$ 作 $AB$ 的垂线,分别与 $C$ 处的切线及直线 $AC$ 交于 $D$, $E$,则 $D$ 为 $BE$ 的中点。 证明自然就略了,而通过压缩变换,可知在椭圆中也有相同结论,如图所示。        图开始要变复杂了,作出焦点三角形 $\triangle PF_1F_2$ 及其旁切圆,如图所示。 熟知这个旁切圆与直线 $F_1F_2$ 切于顶点 $B$,又熟知切线 $PD$ 正是 $\triangle PF_1F_2$ 的外角平分线,由此可见,点 $D$ 正是该旁切圆的圆心,再由上述中点的结论,可知该圆也过点 $E$。 继续画复杂点,我们再任作一条与直线 $BE$ 平行的直线,得交点后作各种垂线,得到如图所示 则由比例关系,有 \[\frac{DB}{D'B'}=\frac{DE}{D'E'}=\frac{DG}{D'G'}=\frac{DH}{D'H'}=\frac{PD}{PD'},\] 而 $DB=DE=DG=DH$,所以 \[D'B'=D'E'=D'G'=D'H',\] 这表明以 $B'E'$ 为直径的圆仍然与直线 $PF_1$ 和 $PF_2$ 相切,所以,原来那个题就成立了。 这样,我们就得到了更一般的命题,椭圆可以是一般的,直线只要与长轴垂直即可。
yuzi 2# 2013-1-4 08:39
哎呦呦!!!学习。。。。。
isea 3# 2013-1-4 09:11
本帖最后由 isea 于 2013-1-4 09:19 编辑 不得不标记一下! ====== 压缩得好啊,收获颇丰,这个推广实在赞! 不知道其它二次曲线是否也有类似的结论。
isea 4# 2013-1-4 09:25
最后实际是一个位似变换,P为位似中心……
kuing 5# 2013-1-4 10:27
最后实际是一个位似变换,P为位似中心…… isea 发表于 2013-1-4 09:25 是的,其实我思考的时候正是看出这一点,所以已经知道那条直线平移一下也会成立,从而转化到特殊情形,也就是旁切圆情形,然后就得到了上述的证明。所以说,思考的过程与写出来的过程往往相反。
kuing 6# 2013-1-4 13:50
不知道其它二次曲线是否也有类似的结论。 isea 发表于 2013-1-4 09:11 用几何画板画了一下,双曲线上同样成立,可惜双曲线无法用压缩变换成圆,不能像上面那样证了。 然而证明的方向仍然可以利用,也就是说可以类似地考察特殊情形——内切圆情形,然后再用位似推到一般。而只证那一步,用代数方法也会简单起来,于是代数+几何,就得到了更一般的命题以及并不复杂的证明。 等会写写
kuing 7# 2013-1-4 14:36
续楼上 双曲线 $x^2/a^2-y^2/b^2=1$, $a$, $b>0$,其上点 $P(x_0,y_0)$,则 \begin{align*} L_{PD}:&\frac{x_0x}{a^2}-\frac{y_0y}{b^2}=1, \\ L_{PA}:&y(x_0+a)-y_0(x+a)=0, \end{align*} 分别令 $x=a$,解得 $D$, $E$ 的坐标为 \[D\left( a,\frac{b^2}{y_0}\left( \frac{x_0}{a}-1 \right) \right),\quad E\left( a,\frac{2y_0a}{x_0+a} \right),\] 而 \[\frac{y_0a}{x_0+a}-\frac{b^2}{y_0}\left( \frac{x_0}a-1 \right)=\frac{y_0^2a^2-b^2(x_0^2-a^2)}{y_0a(x_0+a)}=0,\] 所以 $D$ 为 $BE$ 的中点。(前面椭圆的情形当然也可以这样证) 然后作焦点三角形及其内切圆,类似地可知 $D$ 就是内切圆圆心,且内切圆过点 $E$。 最后还是画得很复杂地 类似地得到 $D'B'=D'E'=D'G'=D'H'$,所以以 $B'E'$ 为直径的圆也与直线 $PF_1$, $PF_2$ 相切。 这样,椭圆和双曲线的情形无论是结论还是证法都可以统一起来了。
kuing 8# 2013-1-4 16:09
哈哈,原来双曲线的情形仍然可以纯平几。为此我们先证明如下命题: $\triangle ABC$ 的内切圆 $I$ 与 $AB$ 切于点 $D$,作直径 $DE$,直线 $CE$ 与 $AB$ 交于 $F$,则 $AF=BD$。           证明:作 $E$ 处的切线交 $CA$, $CB$ 于 $G$, $H$,则 $GH\sslash AB$。 设内切圆与 $CA$, $CB$ 切于 $J$, $K$,则由切线长相等知 $CG+GE=CJ=CK=CH+HE$,即 $CF$ 平分 $\triangle CGH$ 的周长。 由于 $GH\sslash AB$,所以 $CF$ 同样平分 $\triangle ABC$ 的周长,记 $\triangle ABC$ 的半周长为 $s$,仍然由切线长相等可得 $BD+AC=s=AF+AC$,即得 $AF=BD$。 有了这个命题,楼上前半部分要证的东西就显然成立了。这样一来,双曲线的情形也完全不用计算了。
kuing 9# 2013-1-4 17:12
继续!相应地,椭圆的情况也对应着这样的命题: $\triangle ABC$ 中,与 $BC$ 边相切的旁切圆 $I_A$ 与 $AB$ 的延长线切于点 $D$,作直径 $DE$,直线 $CE$ 与 $BA$ 的延长线交于点 $F$,则 $AF=BD$。          证明:作 $E$ 处的切线与 $AC$, $BC$ 的延长线交于 $G$, $H$,则 $\triangle ABC\sim\triangle GHC$。 记 $\triangle ABC$ 及 $\triangle GHC$ 的半周长分别为 $s$ 及 $s'$,则由相似可知 $AB:s=GH:s'$。 又由切线长相等,易知 $s=AD$, $s'=HE$,于是 $AB:AD=GH:HE=AB:BF$,得到 $AD=BF$,从而 $AF=BD$。 这样一来,对于椭圆的情形就不需要用到压缩变换了,可见,椭圆与双曲线的情形在几何证法上又统一了!
yuzi 10# 2013-1-4 17:17
高人。。。。
yes94 11# 2013-1-4 18:15
高人。。。。 yuzi 发表于 2013-1-4 17:17 神人!
isea 12# 2013-1-4 18:31
本帖最后由 isea 于 2013-1-4 20:44 编辑 果然啊果然,两道,典型竞赛命题! 后面这个实例更精彩。 进这帖的,绝对会有收获!这个发散太厉害了,比偶那个原来是 Pascal 定理的强N倍。 === 先表扬楼主,一会来看之间联系与变化 === 8楼其实就是第8届美国邀请赛试题,也有将内切圆,换成旁切圆的命题, 但这样联系到椭圆与双曲线,大娘上花轿——头一回。 四个字结尾:天衣无缝!
第一章 13# 2013-1-4 20:21
8# kuing 原来利用内切圆也可以作出分周线,我一直以为只有利用旁切圆呢。 话说,K真的厉害!
力工 14# 2013-1-4 20:47
9# kuing 膜拜!kuing太强大了。
第一章 15# 2013-1-4 23:24
话说,k的这一解,连直线的具体位置都可以不用;而且,作出来的圆跟两个焦半径所在的直线都相切。确实是妙。
thread-1013-1-5.html: [几何] 不平行,不平行,还是不平行
isea 1# 2013-1-4 11:36
本帖最后由 isea 于 2013-1-4 11:47 编辑 《平行,平行,还是平行》中,即,如图 若将平行看作在无穷远点相交,则,将平行改成相交,有,如图 那么,$P,Q,R$三点将共线!对,就是这样。 若将$P,Q,R$与$A,B,C$字母互换,便是大名鼎鼎的帕斯卡(Pascal)定理,(即,圆锥曲线的内接六边形其三条对边的交点共线,)二次曲线退化成直线的特殊情形。 难怪,在平面几何变换与几何证题里等,这类竞赛书中,直接拿用的。 退化后,就是上图,也有个名字,叫:帕普斯定理,具体: http://zh.wikipedia.org/wiki/帕普斯定理 http://zh.wikipedia.org/wiki/帕斯卡定理 角度不同,境界完全不同。
isea 2# 2013-1-4 20:59
本帖最后由 isea 于 2013-1-4 21:08 编辑 一是为了完整性,二是,我多时候见到的证明都是圆或者是两直线,很少见到统一证明的。 这里把单墫在平面几何小花中给出的证明帖出来。
thread-1014-1-5.html: [数论] AMC试题
Gauss门徒 1# 2013-1-4 16:24
本帖最后由 Gauss门徒 于 2013-1-4 16:26 编辑 定义\[n=\prod_{i=1}^{n}p_{i}^{e_{i}},f(n)=\prod_{i=1}^{n}(p_{i}+1)^{e_{i}-1},f_{m}(n)=f_{1}(f_{m-1}(n))\] 求所有小于等于$400$的自然数$n$使得数列$f_1(n),f_2(n),...f_m(n)$发散。
kuing 2# 2013-1-4 17:22
题都看不懂的说……
Gauss门徒 3# 2013-1-4 17:28
n=xxx的是n的标准分解式
yes94 4# 2013-1-4 19:35
n=xxx的是n的标准分解式 Gauss门徒 发表于 2013-1-4 17:28 标准分解式,就是质因数分解吧,只是题目那么吓人啊! 只有先取几个特殊的n,例如n=6,12,16,24,27之类的摸索一下,帮助理解题意
thread-1015-1-2.html: 原来今天就是传说中的201314
kuing 1# 2013-1-4 19:50
直到刚才收到某网友的消息才发现今天原来就是传说中的201314。 看来今天应该有更多的人正in the love river中,而我显然没这种福气咯。 不过今天我的状态还是蛮好的,在这个贴中,玩得很开心,那种玩推广的过程中的快gan以及得到统一结论和证法的成功感大概不比“in the love river”差吧…… 还是不扯太多了,继续玩题……
isea 2# 2013-1-6 11:49
本帖最后由 isea 于 2013-1-6 11:51 编辑 那帖,还有一个结论,就是切点为焦点时,圆心在准线上 我是从这个方向出发的,不过,最终又证那个中点,除了计算与压缩,几何构造似乎只有位似到过顶点 其次,把抛物线另一焦点看作是无穷远点的话,抛物线里也有类似的结果
kuing 3# 2013-1-6 13:49
2# isea 那抛物线的话就交给你玩吧
thread-1016-1-5.html: [数列] 数列求通项
老人与海 1# 2013-1-4 23:12
$已知数列首项a1=3,满足递推关系a(n+1)=a^2(n)-2,(n>=1),求数列a(n)的通项公式。$
第一章 2# 2013-1-4 23:16
貌似之前在群里讨论过。 设a(n)=b(n)+1/b(n) 得到b(n+1)=b(n)^2 继而两边取对数
kuing 3# 2013-1-4 23:17
http://www.pep.com.cn/rjwk/gzsxs ... 0110402_1031466.htm 例3.1.2
老人与海 4# 2013-1-5 23:51
2# 第一章 $假如后面是减1呢?好像这种方法不行。$
kuing 5# 2013-1-6 00:01
4# 老人与海 除了 -0 和 -2 之外还没发现能解的……
老人与海 6# 2013-1-6 00:35
那只有利用吴炜超的“双曲函数法”可以解决了.
thread-1017-1-1.html: 物理求助,好像要积分
yayaweha 1# 2013-1-4 23:34
如图,质量为M的足够长金属导轨abcd放在光滑的绝缘水平面上。一电阻不计,质量为m的导体棒PQ放置在导轨上,始终与导轨接触良好,PQbc构成矩形。棒与导轨间动摩擦因数为μ,棒左侧有两个固定于水平面的立柱。导轨bc段长为L,开始时PQ左侧导轨的总电阻为R,右侧导轨单位长度的电阻为R0。以ef为界,其左侧匀强磁场方向竖直向上,右侧匀强磁场水平向左,磁感应强度大小均为B。在t=0时,一水平向左的拉力F垂直作用在导轨的bc边上,使导轨由静止开始做匀加速直线运动,加速度为a。 (1)求回路中感应电动势及感应电流随时间变化的表达式; (2)经过多长时间拉力F达到最大值,拉力F的最大值为多少? (3)某过程中回路产生的焦耳热为Q,导轨克服摩擦力做功为W,求导轨动能的增加量。 那个摩擦力是变力,求它的功应该要积分,不会呀!求助
kuing 2# 2013-1-5 17:52
那个棒因电流而受到的力是向上还是向下?我突然忘了什么左手右手的判断方法
kuing 3# 2013-1-5 18:14
蒙吧,猜是向下的,这样就有 \begin{align*} E(t)&=BLat, \\ R(t)&=R+at^2R_0, \\ I(t)&=\frac{E(t)}{R(t)}, \\ f(t)&=\mu(mg+BI(t)L), \\ F(t)&=Ma+f(t), \end{align*} 如果蒙错了的话就将第四个式子的加改成减。
yayaweha 4# 2013-1-5 18:41
3# kuing 力左手,电右手
kuing 5# 2013-1-5 19:02
4# yayaweha 呃,那些手指和手心什么的代表方向也不记得了……你就告诉我那个力是向上还是向下就行了。 不过从结果来看应该是向下的,因为2#的式子各个代入后化简可得 \[F(t)=Ma+\mu mg+\frac{\mu B^2L^2a}{\frac Rt+aR_0t},\] 这样由均值就可求出 $F$ 的最大值,如果是向上的话不等号就反向了。 第三问吃完饭再看……
yayaweha 6# 2013-1-5 19:11
5# kuing 没错是向下的
yayaweha 7# 2013-1-5 19:36
我主要想问第三问,因为正压力等于安培力加重力,安培力是变力所以这个摩擦力是变力,变力做功应该要积分吧!
kuing 8# 2013-1-5 21:37
设该段过程为时间 $t_1$ 到 $t_2$,导轨的位移由 $S_1$ 变化到 $S_2$,则 \begin{align*} Q&=\int_{t_1}^{t_2}EI\rmd t=BL\int_{t_1}^{t_2}Iv\rmd t=BL\int_{S_1}^{S_2}I\rmd S, \\ W&=\int_{S_1}^{S_2}f\rmd S=\int_{S_1}^{S_2}\mu (mg+BIL)\rmd S=\mu mg(S_2-S_1)+Q, \end{align*} 所以 \[\Delta E_k=Ma(S_2-S_1)=\frac{Ma(W-Q)}{\mu mg}.\]
yayaweha 9# 2013-1-5 21:46
本帖最后由 yayaweha 于 2013-1-5 21:48 编辑 8# kuing Q前面是不是少了一个 \mu\
kuing 10# 2013-1-5 21:49
9# yayaweha 噢对,漏掉了,所以像我这种人考试就惨了…… \[W=\int_{S_1}^{S_2}f\rmd S=\int_{S_1}^{S_2}\mu (mg+BIL)\rmd S=\mu mg(S_2-S_1)+\mu Q,\]\[\Delta E_k=Ma(S_2-S_1)=\frac{Ma(W-\mu Q)}{\mu mg}.\]
yayaweha 11# 2013-1-5 23:38
发个标准答案 我觉得太不标准了 变力做功还用恒力做功的公式
yayaweha 12# 2013-1-5 23:41
如果是微元法 是不是就是积分?
kuing 13# 2013-1-5 23:59
发个标准答案 我觉得太不标准了 变力做功还用恒力做功的公式 yayaweha 发表于 2013-1-5 23:38 你是指这一步?:
yayaweha 14# 2013-1-6 00:03
本帖最后由 yayaweha 于 2013-1-6 00:07 编辑 对 就是这步
kuing 15# 2013-1-6 00:06
这个……我也不好说…… 微元法和积分大概本质都差不多,只不过微元法看起来初等一些。
thread-1018-1-1.html: 还是要求积分的
yayaweha 1# 2013-1-4 23:42
.图甲为一研究电磁感应的装置示意图,其中电流传感器(相当于一只理想的电流表)能将各时刻的电流数据实时送到计算机,经过计算机处理后在屏幕上显示电流I和时间t的关系图象.已知电阻R及杆的电阻r均为0.5Ω,杆的质量m及悬挂物块的质量m0均为0.1 kg,杆长L=1m.实验时,先断开S,取下细线调节轨道倾角,使杆(杆不是圆柱体,下滑时不滚动)恰好能沿轨道匀速下滑,然后固定轨道,闭合S,在导轨区域加一垂直轨道平面向下的匀强磁场,让杆在物块的牵引下从图示位置由静止开始释放,此时计算机屏幕上显示出如图乙所示的I—t图象(设杆在整个运动过程中与轨道始终垂直,且细线始终沿与轨道平行的方向拉杆,导轨的电阻忽略不计,细线与滑轮间的摩擦忽略不计,取g=10m/s2,试求: (1)匀强磁场的磁感应强度B; (2)0~0.2 s内通过电阻R的电荷量; (3)0~0.2s内电阻R上产生的焦耳热. 第二问能不能用积分做?
kuing 2# 2013-1-5 17:00
还是干脆把I-t表达式算出来吧 据条件可得 \begin{align*} F&=m_0g-BIL,\\ I&=\frac{BLv}{R+r},\\ \frac{\rmd v}{\rmd t}&=\frac Fm, \end{align*} 消去 $F$ 和 $v$ 得 \[\frac{R+r}{BL}\cdot \frac{\rmd I}{\rmd t}=\frac{m_0g-BIL}m,\] 将 $m_0=m=0.1$, $L=1$, $R=r=0.5$, $g=10$ 代入即 \[\frac{\rmd I}{B\rmd t}=10-10BI,\] 化为 \[\frac{\rmd{(BI)}}{1-BI}=10B^2\rmd t,\] 两边积分得 \[-\ln (1-BI)+C=10B^2t,\] 由 $t=0$ 时 $I=0$ 得 $C=0$,从而解得 \[I=\frac{1-e^{-10B^2t}}B,\] 由图知 $t\to\infty$ 时 $I\to1$,由此得 $B=1$,即 \[I=1-e^{-10t}.\] 表达式知道了应该干啥都行了吧
yayaweha 3# 2013-1-5 19:52
积分后得到的结果 怎么和数格子算面积得出来的答案相差胜远?
kuing 4# 2013-1-5 19:58
呃,大概是出题人或排版者没把图画准确……
yayaweha 5# 2013-1-5 20:00
其实这道题的做法是数格子,但我喜欢精确的
yayaweha 6# 2013-1-6 00:00
这个怎么化的
kuing 7# 2013-1-6 00:03
6# yayaweha 左边分母和右边交换一下然后两边乘B……
yayaweha 8# 2013-1-6 00:05
7# kuing dB与B不一样呀
kuing 9# 2013-1-6 00:07
B是常值,d(BI)=BdI
yayaweha 10# 2013-1-6 00:08
oh I see
yayaweha 11# 2013-1-6 23:24
第三问还是不会
kuing 12# 2013-1-6 23:29
算 $\int_0^{0.2}I^2R\rmd t$ 不就好了么?不会又要数格子吧?
yayaweha 13# 2013-1-8 23:34
把B=1带进去是 dI/(1-dI)=10dt变成   dI=10dt/(1+dt)这样求积分求出来的表达式怎么不一样
kuing 14# 2013-1-8 23:37
13# yayaweha 分母是 1-I
yayaweha 15# 2013-1-8 23:40
那个电流不是应该取极小量吗?
kuing 16# 2013-1-8 23:43
不知怎么给你说了……或者你要先熟悉一下微积分的东东
yayaweha 17# 2013-1-8 23:44
泛泛的讲一下就OK了
yayaweha 18# 2013-1-8 23:45
$$\frac{R+r}{BL}\cdot \frac{\rmd I}{\rmd t}=\frac{m_0g-BIL}m,$$ 这里面的I为什么不是极小量
kuing 19# 2013-1-8 23:54
不知怎么讲
yayaweha 20# 2013-1-8 23:56
你给些资料也可以
thread-1018-2-1.html:
kuing 21# 2013-1-8 23:56
看高等数学教材…… 我的高数就是看教材自学的……
kuing 22# 2013-1-9 00:08
讲微分方程的章节应该会有类似这些物理例子的
海盗船长 23# 2013-2-18 22:20
高中做过这个,也发现过这个问题。。估计出题人没用微积分验算自己乱画的图= =
thread-1019-1-5.html: [几何] 来自人教群的椭圆焦点三角形内角平分线长
kuing 1# 2013-1-5 15:19
教师-其妙(2360****)  14:21:18 问题:椭圆的短半轴长 $b$,$F_1$、$F_2$ 为其焦点,$A$ 是椭圆上一点。求证:$\triangle AF_1F_2$ 的过 $A$ 的内角平分线长 $L\leqslant b$ 其实利用内角平分线长公式($w_a=\frac{\sqrt{bc(b+c+a)(b+c-a)}}{b+c}$),可以发现其表达式极其简单 \begin{align*} L & =\frac{\sqrt{AF_1\cdot AF_2\cdot \bigl((AF_1+AF_2)^2-F_1F_2^2\bigr)}}{AF_1+AF_2} \\ & =\frac{\sqrt{AF_1\cdot AF_2\cdot \bigl((2a)^2-(2c)^2\bigr)}}{2a} \\ & =\frac ba\cdot\sqrt{AF_1\cdot AF_2}, \end{align*} 下略。
yes94 2# 2013-1-7 23:19
妙! 然后均值不等式就出来了!
kuing 3# 2013-1-7 23:23
2# yes94 还可以求下界。$\sqrt{AF_1\cdot AF_2}=\sqrt{(2a)^2-(AF_1-AF_2)^2}\Big/2>\sqrt{a^2-c^2}$……
yes94 4# 2013-1-7 23:34
本帖最后由 yes94 于 2013-1-7 23:38 编辑 sqrt(AF1*AF2)的下确界据说是b,但是三角形不能取到b,版主的这个配方太妙啦! 还可以用焦半径公式的乘积来证明
thread-1020-1-5.html: [不等式] 三角不等式
reny 1# 2013-1-5 20:19
本帖最后由 reny 于 2013-3-9 16:21 编辑 $已知a,b,c为正数,ab+bc+ca=1,证明 9a^2b^2c^2-4(a+b+c)abc+1\ge0. $ PS:其实嘛,这个不等式只是我证明下面这个不等式: $在锐角△ABC中,$ $$\color{red}{\sum \frac{\cos B\cos C}{\cos A}\ge 2\sum \cos^2A}$$ 最终用代数方法转化得到的. 仅就这个不等式,能提供其它方法证明吗? PS:类似地,在$△ABC中,$ $$\color{red}{\sum\frac{\sin B\sin C}{\sin A}\ge \frac{2\sqrt3}{3}\sum{\sin^2A}}$$
kuing 2# 2013-1-5 21:42
因式分解可能没什么办法,也没必要吧,齐次化再配方或者Schur分拆可能有得玩玩。
reny 3# 2013-1-5 21:54
本帖最后由 reny 于 2013-1-5 22:58 编辑 2# kuing 其中$a,b,c>0,就是准备证明上式大于等于0. $不知道咋办
reny 4# 2013-1-22 16:32
本帖最后由 reny 于 2013-1-22 16:38 编辑 1# reny 令$x=b+c>0,y=bc>0,则x^2\ge 4y,由ab+bc+ca=1有a=\frac{1-y}{x}$ \begin{align} 9a^2b^2c^2-4(a+b+c)abc+1&=(9b^2c^2-4bc)a^2-4bc(b+c)a+1\notag\\ &=(9y^2-4y)(\frac{1-y}{x})^2-4xy(\frac{1-y}{x})+1\notag\\ &=\frac{(y-1)^2(9y^2-4y)}{x^2}+(2y-1)^2 \end{align} $当y\ge\frac49时,(1)式显然大于0;$ $当0<y<\frac49时,(1)式\ge\frac14(y-1)^2(9y-4)+(2y-1)^2=\frac14y(3y-1)^2\ge0$ 综上,不等式成立,当且仅当$a=b=c=\frac{1}{\sqrt3}时取到等号.$
kuing 5# 2013-1-22 16:37
4# reny 不错,如果计算没错的话。
kuing 6# 2013-1-22 16:52
PS:其实嘛,这个不等式只是我证明下面这个不等式: $在锐角△ABC中,$ $$\color{red}{\sum \frac{cosBcosC}{cosA}\ge 2\sum cos^2A}$$ 最终用代数方法转化得到的. 仅就这个不等式,能提供其它方法证明吗? reny 发表于 2013-1-5 20:19 噢,这个之前在安振平的QQ空间里看到过,也写过简证,见 http://user.qzone.qq.com/363215694/blog/1316295244 不过可能不是Q好友进不去,我还是把我写的截上来
kuing 7# 2013-1-22 17:04
顺便指出,若 $\triangle ABC$ 为钝角三角形,那么必有 $\cos A\cos B\cos C<0$,所以此时两边除以 $\cos A\cos B\cos C$ 后 不等式应反向,也即当 $\triangle ABC$ 为钝角三角形时原不等式反向成立。
reny 8# 2013-1-22 18:10
6# kuing 这方法太漂亮了! 原来如此啊!  我做复杂了啊!!!
reny 9# 2013-1-22 19:18
本帖最后由 reny 于 2013-1-22 19:40 编辑 6# kuing 我想到一个类似的不等式: 在$△ABC中,$ $$\sum\frac{sinB sinC}{sinA}\ge \frac{2\sqrt3}{3}\sum{sin^2A}$$ 我用类似的代数方法证明这个不等式,显得挺简单,而证明前面那个过程挺复杂的。 这个不等式证明有没有其他个方法?
kuing 10# 2013-1-22 21:40
类似地,在$△ABC中,$ $$\color{red}{\sum\frac{sinB sinC}{sinA}\ge \frac{2\sqrt3}{3}\sum{sin^2A}}$$ reny 发表于 2013-1-22 19:18 PS、sinA 应写成 \sin A ,前面的 cos 也一样。 这个我刚才也想尝试用嵌入,可惜没成功。然后想用外森比克,也反了向,后来用更强的F-H不等式就搞定了。 由面积公式有 \begin{align*} \sum\frac{\sin B\sin C}{\sin A}\geqslant \frac{2\sqrt3}3\sum\sin ^2A&\iff2S\sum\frac1{a^2}\geqslant \frac{8\sqrt3S^2}3\sum\frac1{b^2c^2} \\ & \iff\frac{\sum b^2c^2}{\sum a^2}\geqslant \frac{4\sqrt3S}3, \end{align*} 据根 Finsler-Hadwiger 不等式 \[\sum a^2\geqslant 4\sqrt3S+\sum(a-b)^2,\] 可知只要证 \[ \frac{\sum b^2c^2}{\sum a^2}\geqslant \frac{\sum a^2-\sum (a-b)^2}3, \] 而 \begin{align*} 3\sum b^2c^2-\sum a^2\left( \sum a^2-\sum (a-b)^2 \right)&=\sum b^2c^2-\sum a^4+\sum a^2\sum (a-b)^2 \\ & =-\frac12\sum (a^2-b^2)^2+\sum a^2\sum (a-b)^2 \\ & =\frac12\sum \left( 2\sum{a^2}-(a+b)^2 \right)(a-b)^2 \\ & =\frac12\sum (2c^2+(a-b)^2)(a-b)^2\\ &\geqslant 0, \end{align*} 从而原不等式得证。
reny 11# 2013-1-22 21:59
本帖最后由 reny 于 2013-1-22 22:02 编辑 10# kuing 这个题我类比过来后,还是用换元,转化为代数形式,感觉挺容易的(用到陶平生曾经写的一个三角结构的系统)。 我就是挺纳闷,相同方法做前面那个(关于余弦)的不等式,就蛮复杂,结果一下就被你秒了。
reny 12# 2013-1-22 22:09
我也只会想到这个代数方法了,你说的F-H不等式又是第一次看到,厉害。
kuing 13# 2013-1-22 22:13
嗯,你说你是在几何不等式转化出来的时候我看1#那条件就猜到你用切系统了 PS、连乘符号你好像打反了
kuing 14# 2013-1-22 22:29
其实F-H不等式只是等价于 $xy+yz+zx\geqslant\sqrt{3xyz(x+y+z)}$,不是用切系统,是用内切圆系统。
reny 15# 2013-1-22 22:43
14# kuing 切系统在三角不等式证明中有时挺管用滴。 这么多陌生的不等式,很少用,所以也就记不得啦,向你学习吧!
thread-1021-1-5.html: [不等式] 回复352的问题
kuing 1# 2013-1-6 14:29
我直接目测是测不出来的,不过要是能换元搞搞还是比较容易推出来的。 令 $x/(x-1)=a$, $y/(y-1)=b$, $z/(z-1)=c$,解得 $x=a/(a-1)$, $y=b/(b-1)$, $z=c/(c-1)$,故 \[xyz=1\iff abc=(a-1)(b-1)(c-1)\iff 0=-ab-bc-ca+a+b+c-1,\] 所以 \begin{align*} \frac{x^2}{(x-1)^2}+\frac{y^2}{(y-1)^2}+\frac{z^2}{(z-1)^2}-1&=a^2+b^2+c^2-1\\ &=(a+b+c)^2-2(ab+bc+ca)-1\\ &=(a+b+c)^2-2(a+b+c-1)-1\\ &=(a+b+c-1)^2, \end{align*} 这样代回去就得到了那个神奇的配方。
reny 2# 2013-1-6 15:05
好想法! 转化不等式的形式真的有点不好想啊。
kuing 3# 2013-1-6 15:09
碰巧看Q空间又见一个类似的 \[\left( \frac{a}{b-c} \right)^2+\left( \frac{b}{c-a} \right)^2+\left( \frac{c}{a-b} \right)^2-2=\left( \frac{a}{b-c}+\frac{b}{c-a}+\frac{c}{a-b} \right)^2\] 这个曾经也研究过,只要注意到有恒等式 \[\frac{bc}{(a-b)(a-c)}+\frac{ca}{(b-c)(b-a)}+\frac{ab}{(c-a)(c-b)}=1,\] 看到分母这个样子,有没有想起拉格朗日cha值公式? 没错,当年我就利用拉格朗日cha值公式导出了一系列恒等式,上式就是其中之一,见 http://bbs.pep.com.cn/forum.php?mod=viewthread&tid=583379
thread-1022-1-5.html: [函数] R上f(f(f(x)))=x,求f(x)
realnumber 1# 2013-1-6 17:55
群里看到一题目: 是否存在R上函数f(x),使得f(x)≠x,且f(f(f(x)))=x,? 解答:f(x)=x,x≠1,2,3 然后f(1)=2,f(2)=3,f(3)=1;这样即使n次复合也有例子了 然后又加连续或又要周期3,...想不出了.
kuing 2# 2013-1-6 18:43
先记一个集合 $\displaystyle M=\left\{\frac x3 \biggm| x\in\mbb Z\right\}$,将 $f(x)$ 定义为 \[ f(x)=\begin{cases} x-\frac23, & x\in\mbb Z,\\ x+\frac13, & x\in M \setminus \mbb Z,\\ x, & x\in\mbb R \setminus M. \end{cases} \] 行不行? 注:$A \setminus B$ 表示集合差,即:$A \setminus B = \{x|x\in A~\text{且}~x\notin B\}$。
kuing 3# 2013-1-6 19:03
先记一个集合 $\displaystyle M=\left\{\frac x3 \biggm| x\in\mbb Z\right\}$,将 $f(x)$ 定义为 \[ f(x)=\begin{cases} x-\frac23, & x\in\mbb Z,\\ x+\frac13, & x\in M \setminus \mbb Z,\\ x, & x\in\mbb R \setminus M. \end{cases} \]kuing 发表于 2013-1-6 18:43 (1)若 $a\in\mbb Z$,则 \[f(a)=a-\frac23\in M \setminus \mbb Z,\] 故 \[f(f(a))=f\left(a-\frac23\right)=a-\frac13\in M \setminus \mbb Z,\] 所以 \[f(f(f(a)))=f\left(a-\frac13\right)=a;\] (2)若 $a\in M \setminus \mbb Z$,再记 \[M_1=\left\{\frac13+k\biggm| k\in\mbb Z\right\}, \quad M_2=\left\{\frac23+k\biggm| k\in\mbb Z\right\},\] 注意到 $M \setminus \mbb Z = M_1 \cup M_2$ 且 $M_1 \cap M_2=\varnothing$,故再分两小类: (2-1)若 $a\in M_1$,则 \[f(a)=a+\frac13\in M_2,\] 故 \[f(f(a))=f\left(a+\frac13\right)=a+\frac23\in \mbb Z,\] 所以 \[f(f(f(a)))=f\left(a+\frac23\right)=a;\] (2-2)若 $a\in M_2$,则 \[f(a)=a+\frac13\in \mbb Z,\] 故 \[f(f(a))=f\left(a+\frac13\right)=a-\frac13\in M_1,\] 所以 \[f(f(f(a)))=f\left(a-\frac13\right)=a;\] (3)若 $a\in\mbb R \setminus M$,则显然 $f(f(f(a)))=a$。 综上所述,该 $f(x)$ 对任意 $x\in\mbb R$ 都有 $f(f(f(x)))=x$。
kuing 4# 2013-1-6 19:59
n 次迭代应该也可以类似构造出来。
realnumber 5# 2013-1-6 20:06
有些看晕了,又对符号P/M不熟悉,意思猜得出, 分段函数第一条和第二条有没重复定义.-----我明天继续看,今天恐怕晕了
kuing 6# 2013-1-6 20:09
5# realnumber 那个是集合差,$A \setminus B = \{x|x\in A~\text{且}~x\notin B\}$,又或者看成补集 $\complement_AB$ 其实我才看到你后面说的还要加连续和周期……那样的话我这个也不连续也没周期,f(x)-x 倒是有周期。 还是不必再看3#了,我有空再研究过……
yes94 7# 2013-1-7 23:06
题目条件写完没有啊? 是否存在R上函数f(x),使得f(x)≠x,且f(f(f(x)))=x,? 令f(x)=1-1/x,满足f(x)≠x,且f(f(f(x)))=x。
realnumber 8# 2013-1-7 23:11
本帖最后由 realnumber 于 2013-1-7 23:13 编辑 "R"上的意思是定义域为R,还是定义域是R的子集? 题目我也是群里看来的,不清楚是否还有别的条件,可以推广,任意删减或添加,
yes94 9# 2013-1-7 23:13
哦,没注意定义域
yes94 10# 2013-1-8 00:42
本帖最后由 yes94 于 2013-1-8 00:43 编辑 是否存在R上函数f(x),使得f(x)≠x,且f(f(f(x)))=x,? 令f(x)为:当x不为0,1,2时,f(x)=1-1/x,;并且定义f(0)=1,f(1)=2,f(2)=0, 可以验证f(f(f(x)))=x(x不为0,1,2); f(f(f(0)))=0,f(f(f(1)))=1,f(f(f(2)))=2, 综上所述,在R上f(f(f(x)))=x。 不知对否?最近做题老是有失误
kuing 11# 2013-1-8 00:58
10# yes94 这个还f(x)恒不等于x,更有意思
yes94 12# 2013-1-10 21:17
不知道real觉得对不?找到原题没?
realnumber 13# 2013-1-10 22:27
dui对的,就是还是没连续
yes94 14# 2013-1-10 22:33
13# realnumber 还有连续的条件啊?
kuing 15# 2013-1-10 22:50
应该说是 kai 放 xing 题目,尽可能找出满足等式的而且有其他“特色”的函数出来。
realnumber 16# 2013-1-11 08:14
完全满足条件,又连续的估计没有,我下星期想想,周六教工系统有围棋比赛.
realnumber 17# 2013-1-11 08:30
估计是那个发言的学生发挥想象力自己加的,"连续,周期"什么的;不过挺好,正是这些刺激了想象力.
yes94 18# 2013-1-11 12:17
完全满足条件,又连续的估计没有,我下星期想想,周六教工系统有围棋比赛. realnumber 发表于 2013-1-11 08:14 安逸哦,还有围棋比赛,
realnumber 19# 2013-1-11 12:48
恩,否则也不会在论坛混了.
realnumber 20# 2013-1-12 20:32
本帖最后由 realnumber 于 2013-1-14 08:30 编辑 看来数学和围棋不可兼得,做题目兴奋,导致睡得少,围棋大输. 是否存在R上连续函数f(x),使得$f(x)≠x$,且$f(f(f(x)))=x$. 引理:$f(x)$是R连续函数,那么对于任意$m,f(a)<m<f(b)$,总存在$n,a<n<b$,有$m=f(n)$($a>{b}$也有类似结论). 高中阶段连续定义不便叙述,就按图象不加证明直接得到,其实就是介值定理. 证明:用反证法,假设存在,记$f(a)=b\ne{a},f(b)=c,f(c)=a$,(若$b=c$也容易得到矛盾)不妨假设$a>{b}>{c}$,如图,则存在$m$ $c<m<b$,有$f(m)=f(a)=b$,进一步得到$f(f(f(m)))=f(f(f(a)))$,即$m=a$,那么$c<m<b$与$a>{b}>{c}$矛盾.
thread-1022-2-5.html:
kuing 21# 2013-1-12 21:44
话说,这个“$f(x)\ne x$”的条件到底是不恒等还是恒不等?
realnumber 22# 2013-1-12 22:23
猜是不恒等.看来还是说明下好,省得读题目的人纠结.
kuing 23# 2013-1-12 22:33
不恒等为何设 $f(a)=b\ne a,f(b)=c\ne b,f(c)=a\ne c$? PS、不等号代码 \ne
yes94 24# 2013-1-12 23:33
23# kuing y=x作为一个函数整体,$f(x)\ne x$,指的是存在t,使$f(t)\ne t$,即允许f(x)有不动点? 第一次复制代码,随便玩一下,
realnumber 25# 2013-1-13 09:11
@yes94,检查下20楼证明有没问题?总觉得是不是太简单了,有没什么没考虑到?.
realnumber 26# 2013-1-13 15:38
23# kuing 好吧,我承认罗嗦了,我去修改就是.
kuing 27# 2013-1-13 16:21
26# realnumber 其实我不是说啰嗦,而是为何要三个都不等?
realnumber 28# 2013-1-14 08:28
果然,还是应该写一个啊,晕啊.
pxchg1200 29# 2013-1-14 08:48
27# kuing 其实如果连续的话,只有$f(x)=x$ 首先,我们可以证明$f(x)$是一个一一映射。证明很简单,大家自己思考下。 然后,连续的一一映射一定具备单调性的,所以,我们可以假设$f(x)$单调,这是发现$f(x)$只有可能单调递增,理由就是3层复合。
realnumber 30# 2013-1-14 08:52
29# pxchg1200 三层复合会导致增,不明白;20楼证明对吗?可能同你的意思.
pxchg1200 31# 2013-1-14 09:00
30# realnumber 你想想如果$f(x)$递减,那么$f(f(x))$就会递增,$f(f(f(x)))$会递减,这显然是不可能的
realnumber 32# 2013-1-14 09:08
明白了就是标题呢
kuing 33# 2013-1-14 10:09
噢,原来一定是一一映射, 那周期自然也不可能有了。 再搞搞别的要求?
thread-1023-1-5.html: [组合] 一道排列组合题
hflz01 1# 2013-1-6 22:13
见附图
kuing 2# 2013-1-6 23:08
这么难……用程序搞一下竟然有90个这么多,看来穷举也不易…… PS、纯文字无公式的题目就不必贴图了吧……
hflz01 3# 2013-1-7 22:39
无人问津?
kuing 4# 2013-1-7 22:42
3# hflz01 可能难度大吧,我计数弱,暂时也没什么好办法。
realnumber 5# 2013-1-7 23:24
今天太晚了,明天用逐步淘汰原理或递推数列的方法试下,别抱太大希望.
kuing 6# 2013-1-7 23:25
今天太晚了,明天用逐步淘汰原理或递推数列的方法试下,别抱太大希望. realnumber 发表于 2013-1-7 23:24 递推我昨晚想了好久没想出来 “逐步淘汰原理”第一次听……
realnumber 7# 2013-1-7 23:34
“逐步淘汰原理”就是容斥原理啦,百度.
kuing 8# 2013-1-7 23:36
7# realnumber 原来还有这么个别名……容斥我也想过,也无果……
realnumber 9# 2013-1-8 13:29
本帖最后由 realnumber 于 2013-1-11 08:36 编辑 先穷举法完成它,积累些经验 1,2,3,4,5,6排成一列,要求任意两个连续自然数不能相邻,问有几种排法? 考虑到1,6对称,同样对于2,5;3,4 以及1放第一个位置和第6个位置一样多 一.1放1号位置(或6号位置)       ①2在3号位置,                  (1)3在5号位置,4只能在2号位置,----2种               (2)3在6号位置,4只能在4号位置(在2号的话,没法放5,6了)---2种       ②2在4号位置,                  (1)3在2号位置,4可以在5或6号位置,----2种               (2)3在6号位置,4可以在2或3号位置-----2种       ③2在5号位置,                 (1)3在2号位置,4只能在4号位置,----1种               (2)3在3号位置,4只能在6号位置-----2种        ④2在6号位置               (1)3在第2号位置,----0种               (2)3在第3号位置,4只能在5号位置-----2种               (3)3在第4号位置,4只能在2号位置,----1种 这样一来"1放1号位置或6号位置"有28种             ---每一条数量倒不多,太烦琐,容易出错.暂停下这个办法,如有必要才来继续.. 二.1放2号位置(或5号位置)       ①2在第4号位置,       ②2在第5号位置,       ③2在第6号位置 三.1放3号位置(或4号位置)       ①2在第1号位置,       ②2在第5号位置,       ③2在第6号位置, 逐步淘汰原则successive sweep principle: 总数    $A_6^6=720$ 出现1对   $ 1,2;2,3;3,4;4,5;5,6,$这1对看作一个整体,$5A_2^2A_5^5=1200$ 出现2对,  $"123,234,345,456,321,432,543,654"$有$8A_4^4=192$              或$"12,34;12,45;12,56;23,45;23,56;34,56$有$6A_2^2A_2^2A_4^4=576$ 出现3对,$"1234;2345,3456,4321,5432,6543"$有$6A_3^3=36$            或$123,45;123,56;234,56;345,12;456,12;456,23$有$6\times 2\times 2A_3^3=144$            或$12;34;56$有$({A_2^2})^3A_3^3=48$ 出现4对,$"12345;23456;54321;65432"$有$4A_2^2=8$,还有"123,456;12,3456;1234,56;"有$24$种,合计$8+24=32$ 出现5对,$"123456;654321"$有$2$种. 所以本题结果是$720-1200+(192+576)-(36+144+48)+32-2=90$ 和kuing程序做出的$90$一样了.
Gauss门徒 10# 2013-1-8 16:06
9# realnumber 作考试的话,时间够么?
realnumber 11# 2013-1-8 16:19
恩,我不知道,也许一直的训练的一些牛B选手,也许没问题,我得想递推数列,如果可以,应该是最快的.
kuing 12# 2013-1-8 17:36
9# realnumber 真好耐心啊!
realnumber 13# 2013-1-10 18:22
本帖最后由 realnumber 于 2013-1-11 21:00 编辑 $n$个数据依次为$1,2,3,...,n-1,n$. 记 $ a_n $为$n$个数据没有2个相邻的排列数.比如$a_6=90,a_1=1,a_2=a_3=0,a_4=2$ 记 $ b_n $为$n$个数据有且仅有1对相邻的排列数.$b_1=0,b_2=2,b_3=4$ 记 $c_n$为$n$个数据中有且仅$n-1,n$相邻的排列数.$c_1=0,c_2=c_3=c_4=2,c_5=6$ 记 $d_n$表示有3个数据连在一起的个数,$d_3=2,d_4=4,d_5=10$ 那么有如下关系: $a_{n+1}=(n-1)a_n+b_n-c_n$,第$n+1$个数据插入$n$个互不相邻的有$n-1$个位置,插入有且仅有一个相邻的只有一个位置,或无法插入(因为$n-1,n$相邻了(kuing)). $c_n=2a_{n-1}+c_{n-1}$ $b_n=2(n-1)a_{n-1}+2b_{n-1}+d_{n-1}$1楼问题其实已经解决,就是递推还不够完善. $d_n=2(n-2)a_{n-2}+2b_{n-2}+d_{n-2}=?b_{n-1}+?$---就这条了,可能有错,没想明白.$a_5,a_7$可以通过四个递推得到,并得到kuing的程序验证. 楼主公布下答案和出处?--想不到已经过去了3天,也许会写文要引用下.
hflz01 14# 2013-1-15 22:26
没答案哦,不知同事从哪搞来的。
thread-1024-1-5.html: [组合] 集合元素个数,群里看到的
realnumber 1# 2013-1-7 16:12
虽然已经有答案,第一眼觉得不错,保留在这里 唐山齐老师: http://zhidao.baidu.com/question/492222198.html,搜到个解答 ____kuing edit in $\LaTeX$____ 给定整数 $n>1$,设 $a_1$, $a_2$, $\ldots$, $a_n$ 是互不相同的非负实数,记集合 \[A=\{a_i+b_i|1\leqslant i\leqslant j\leqslant n\}, B=\{a_ia_j|1\leqslant i\leqslant j\leqslant n\}.\] 求 $\frac{\abs A}{\abs B}$ 的最小值。这里,$\abs X$ 表示集合 $X$ 中元素的个数。
realnumber 2# 2013-1-7 21:32
本帖最后由 realnumber 于 2013-1-7 21:45 编辑 1楼连接中的解答 pighead520| 四级   首先:$|A|_{min}=2n-3$ 这是因为:为方便起见,将an做一个排序 即  $a_i>a_j>0, (i>j)$这样 $a_1+a_1<a_1+a_2<……<a_1+a_n<a_2+a_n<……<a_{n-1}+a_n<a_n+a_n$   这里共有2n-1个数      而且$|A|=2n-1$ 可以取到(递增的等差数列就能满足)|B|的最大值就是每两个数的乘积都不相同$|B|=\frac{n(n+1)}{2}$    所以$\frac{|A|}{|B|}$的最小值就是$\frac{4n-2}{n^2+n}$ 最好能加一个$|B|=\frac{n(n+1)}{2}$的实例,怎么构造呢?
realnumber 3# 2013-1-7 21:58
令等差数列为$a_n=1+n\sqrt{2}$ 若$a_pa_q=a_sa_t,p\ge{q},s\ge{t}$,那么代入通项公式有$p+q=s+t,pq=st$,可得$p=s,q=t$ 所以任意两项乘积互不相同
kuing 4# 2013-1-7 22:07
nice 话说最大值如何?|B| 最小似乎还能上面那个还小,因为可以有0
realnumber 5# 2013-1-7 22:13
本帖最后由 realnumber 于 2013-1-8 17:43 编辑 最大模仿2楼那个网友pighead520做法,{$b_k$}为递增,$1\ge{k}\ge{n-1}$,但另外加个$b_n=0$, 那么非零的有$b_1b_1<b_1b_2<b_1b_3<...<b_1b_{n-1}<b_2b_{n-1}<....<b_{n-1}b_{n-1}$包括0,共2n-2项,前$n-1$项为等比即符合题意. 和最大个数为$\frac{n(n+1)}{2}$,实例是$b_k=\sqrt{2}3^{k-1}$,$k=1,2,...,n-1$,$b_n=0$, p<q<n,s<t<n,若$b_p+b_q=b_s+b_t$,假定$p<s$,两边约去$\sqrt{2}3^p$后,矛盾==, 最大就是$\frac{n(n+1)}{4n-4}$
thread-1025-1-5.html: [不等式] 来自某教师群的一道绝对值不等式求参数范围
kuing 1# 2013-1-7 22:33
字有点小,而且后面漏了“范围”二字,我重新打一打: 若关于 $x$ 的不等式 $ax^2-\abs{x-1}+2a<0$ 的解集为空集,求 $a$ 的取值范围。 解:分离变量有 \[ax^2-\abs{x-1}+2a<0\iff a<\frac{\abs{x-1}}{x^2+2}=f(x),\] 下面求 $f(x)$ 的最大值,由均值不等式,有 \begin{align*} f(x)&=\frac{\abs{x-1}}{(x-1)^2+3+2(x-1)} \\ & \leqslant \frac{\abs{x-1}}{(x-1)^2+3-2\abs{x-1}} \\ & =\frac1{\abs{x-1}+\dfrac3{\abs{x-1}}-2} \\ & \leqslant \frac1{2\sqrt3-2}=\frac{\sqrt3+1}4, \end{align*} 当 $x=1-\sqrt3$ 时等号成立,故 $f(x)$ 的最大值就是 \[f(x)_{\max}=f\bigl(1-\sqrt3\bigr)=\frac{\sqrt3+1}4,\] 所以要原不等式解集为空,只需 $a\geqslant \frac{\sqrt3+1}4$。
yes94 2# 2013-1-7 22:36
一次就成功了,还不需要讨论去绝对值,放缩等号也能同时取到,妙!
yes94 3# 2013-1-7 23:48
显然a>0,故原不等式可化为x^2+2>=|x-1|/a恒成立,做出两个函数图像,发现x<1时相切时,可以得到1/a的最大值,于是可得到a的最小值
kuing 4# 2013-1-7 23:54
3# yes94 嗯,图象法也不错。
yes94 5# 2013-1-8 00:16
目前只能搞这种高考的了, 再来一种:
yes94 6# 2013-1-8 00:24
最后分母有理化失误,答案的分子是加号。 取等号条件同版主的,这里略去
thread-1026-1-1.html: U盘容量“自扩充”?!
戊概念·五 1# 2013-1-7 23:13
RT,该U盘的容量原本是8G,但某次接入电脑后显示。。。。。 弱问,这是发生了什么神奇的事? 还是、我这U盘有问题哦?!
kuing 2# 2013-1-7 23:20
不懂and不清楚…… PS、右上角修改日期也很牛……
thread-1027-1-5.html: [函数] 简单问题
yayaweha 1# 2013-1-7 23:15
n/(n+1)^2的原函数是什么
kuing 2# 2013-1-7 23:18
ln(n+1)+1/(n+1)
yayaweha 3# 2013-1-7 23:23
怎么推出来的?
kuing 4# 2013-1-7 23:24
n/(n+1)^2 = (n+1-1)/(n+1)^2 = 1/(n+1)-1/(n+1)^2
yayaweha 5# 2013-1-7 23:25
为什么想着把上面的n分离出来?
kuing 6# 2013-1-7 23:31
因为要化为最简分式,高数课本里一般都有讲的吧,有理积分什么的。
realnumber 7# 2013-1-8 08:21
百度"部分分式"
thread-1028-1-5.html: [函数] 群里看到的,似乎是很早的练习题了
realnumber 1# 2013-1-8 15:50
本帖最后由 realnumber 于 2013-1-8 18:38 编辑 若有$f(a)=a$,则$f(f(a))=f(a)=a$,说明 反之$f(f(b))=b$,因为单调增若$f(b)>b$,有$f(f(b)>f(b))=b$,若$f(b)<b$也矛盾,只能$f(b)=b$,所以B中元素一定在A内, ....那么A=B D错的,比如$f(x)=x+1$,那么$A=B=\varnothing$
yes94 2# 2013-1-8 17:59
据说还有一个C={x|f(f(x))=f(x)},求A、B、C的关系?
kuing 3# 2013-1-9 00:01
也等?
kuing 4# 2013-1-9 00:30
或者说, 等号两边的迭代次数随意但不同, 那集合也等?
realnumber 5# 2013-1-10 18:50
应该是的,很容易证明的.
thread-1029-1-5.html: [不等式] 半凹半凸定理没看懂
reny 1# 2013-1-8 17:26
本帖最后由 reny 于 2013-1-8 17:30 编辑 前面http://kkkkuingggg.5d6d.net/thread-1007-1-1.html请教过不等式,Kuing提及到半凹半凸定理,且链接了 http://www.artofproblemsolving.com/Forum/viewtopic.php?t=64933,大致看了对原题的解答,但是没看懂定理. 划红线的怎么来的?
kuing 2# 2013-1-8 17:44
或者你看懂这个先 http://kkkkuingggg.5d6d.net/thread-901-1-1.html
v6mm131 3# 2013-1-12 12:41
参看韩京俊那本书上有这个定理 (赵斌老师的)
thread-103-1-5.html: [数列] 2011联赛A卷一试10数列求通项
kuing 1# 2011-10-16 16:03
\[a_1=2t-3,t\in\mathbb{R},t\ne\pm1,a_{n+1}=\frac{(2t^{n+1}-3)a_{n}+2(t-1)t^{n}-1}{a_{n}+2t^{n}-1}.\] 两边加 $2t^{n+1}-1$ 整理为 \[a_{n+1}+2t^{n+1}-1=\frac{4(t^{n+1}-1)(a_{n}+t^{n})}{a_{n}+2t^{n}-1},\] 令 $b_{n}=a_{n}+t^{n}$,化为 \[b_{n+1}+t^{n+1}-1=\frac{4(t^{n+1}-1)b_{n}}{b_{n}+t^{n}-1}\iff\frac{b_{n+1}}{t^{n+1}-1}+1=\frac{4}{1+\frac{t^{n}-1}{b_{n}}},\] 令 $c_{n}=\dfrac{b_{n}}{t^{n}-1}$,化为 \[c_{n+1}+1=\frac{4}{1+\frac{1}{c_{n}}}\iff c_{n+1}=\frac{3c_{n}-1}{c_{n}+1}\iff c_{n+1}-1=\frac{2(c_{n}-1)}{c_{n}+1}\iff\frac{1}{c_{n+1}-1}=\frac{1}{c_{n}-1}+\frac{1}{2},\] 由 $a_1=2t-3$ 得 $c_1=3$,从而易得 \[c_n=\frac{n+2}{n},\] 回代,即得 \[a_{n}=b_{n}-t^{n}=c_{n}(t^{n}-1)-t^{n}=\frac{n+2}{n}(t^{n}-1)-t^{n}=\frac{2t^{n}-n-2}{n}.\] 第二问是 $t>0$ 时比较 $a_{n+1}$ 与 $a_n$,这简单,由均值就行了。故此这个题其实难在第一问……
Gauss门徒 2# 2013-1-5 13:27
先算几项找规律
hnsredfox_007 3# 2013-1-5 14:09
干啥子哟!不就是不动点法求通项嘛
hnsredfox_007 4# 2013-1-5 14:18
thread-1030-1-1.html: 本论坛的自定义命令[2013-3-2新增“度”的简化输入\du]
kuing 1# 2013-1-8 21:46
MathJax 的确不错,还能自定义命令。 目前本论坛已经自定义的命令有: \mbb 等价于 \mathbb,纯粹为了简写,用于特殊数集。例:\mbb R 显示 $\mbb R$ \riff 向右推出,等价于 \implies,纯粹为了简写。例:A\riff B 显示 $A\riff B$ \liff 向左推出,等价于 \impliedby,纯粹为了简写。例:A\liff B 显示 $A\liff B$ \sslash 斜的平行符号(默认的平行符号命令 \parallel 是直立的 $\parallel$),例:AB\sslash CD 显示 $AB\sslash CD$ \pqd 平行且等于(无论 MathJax 还是真 LaTeX 都无此符号),例:AB\pqd CD 显示 $AB\pqd CD$ \abs 绝对值(自动适应高度),用法是 \abs{xx}。 例:\abs{-2-2},\abs{\dfrac ab} 显示 $\abs{-2-2}$,$\abs{\dfrac ab}$ (试试 \(\verb"$|-2-2|$"\) 与 \(\verb"$\abs{-2-2}$"\) 看看有什么不同?) 注:带箭头的向量的模不建议用它,效果不太好看如 $\abs{\vv a}+\abs{\vv b}$,这是因为箭头太高。 \rmd 直立的微积算子 d,用法 \rmd{x}。例:\int_a^b f(x) \rmd{x} 显示 $\int_a^b f(x) \rmd{x}$ \vv 向量箭头,等价于 \overrightarrow,纯粹为了简写。例:\vv a,\vv{AB} 显示 $\vv a$,$\vv{AB}$ \veps 希腊字母 $\varepsilon$,等价于 \varepsilon,纯粹为了简化输入。 \du 角度,等价于 ^\circ,纯粹为了简化输入。例:30\du 显示 $30\du$ 以上这些大家已经可以直接用的了。 而除此之外,还可以在某一页面内临时自定义一个命令来用,此时该命令只应用于该页面,不影响全局。 比如,在某个贴子里需要多次输入相同的 $a_1+a_2+\cdots+a_n$,这时你可以定义临时命令来代替它,这样就可以简化输入。 具体的定义方法是: 在开头先输入一个 \(\verb"$\newcommand\asdf{a_1+a_2+\cdots+a_n}$"\) 这样后面只要用 \asdf 命令就可以得到 $a_1+a_2+\cdots+a_n$ 了。 那个命令名 asdf 你可以随意取,不过一定要纯英文,而且不能与已有命令重复。 进一步,如果某个贴子里需要多次输入形如 $a_1+a_2+\cdots+a_k$ 的式子,但最后下标不一定是 $k$,而可能是 $k+1$,$n$ 等等,这时你可以定义含一个参数的临时命令来代替它。具体方法类似: 在开头先输入一个 \(\verb"$\newcommand\fdsa[1]{a_1+a_2+\cdots+a_{#1}}$"\) 这样,用 \fdsa{k} 得 $a_1+a_2+\cdots+a_k$,用 \fdsa{n} 得 $a_1+a_2+\cdots+a_n$,用 \fdsa{k+1} 得 $a_1+a_2+\cdots+a_{k+1}$ 等等。 再进一步,要是不但最后下标不定,而且字母 $a$ 也不定呢?那就定义含两个参数的临时命令。 \(\verb"$\newcommand\aassdd[2]{{#1}_1+{#1}_2+\cdots+{#1}_{#2}}$"\) 这样,用 \aassdd{a}{n} 得 $a_1+a_2+\cdots+a_n$,用 \aassdd{x}{k} 得 $x_1+x_2+\cdots+x_k$,用 \aassdd{S}{k+p} 得 $S_1+S_2+\cdots+S_{k+p}$ 等等。 还可以定义更多个参数的命令,跟真 LaTeX 差不多。参数的个数应该是有上限的,不过一般也不会用到那么多了。 大家不妨测试测试,有更好的提议欢迎回贴提出。
thread-1031-1-5.html: [不等式] 来自某教师群的一道不等式
kuing 1# 2013-1-9 14:05
广州朱sir(1054******) 08:19:09 求助高手: 广州朱sir(1054******) 09:06:55 不知道出自哪里,学生问的 其实条件可以弱化,只要 $p>1$ 且 $k>1$ 即可。 这显然是以拉格朗日中值定理为背景的,而且可以给出另一边。 具体地,令 $f(x)=x^{-p+1}$, $x>0$,则 $f'(x)=(-p+1)x^{-p}$,由拉格朗日中值定理得 \[ \frac{f(k)-f(k-1)}{k-(k-1)}=f'(\xi ), \] 其中 $\xi \in (k-1,k)$,再由 $f''(x)=p(p-1)x^{-p-1}>0$ 得 \[ f'(k-1)<f'(\xi )<f'(k), \] 所以 \[ f'(k-1)<f(k)-f(k-1)<f'(k), \] 即 \[ (-p+1)(k-1)^{-p}<k^{-p+1}-(k-1)^{-p+1}<(-p+1)k^{-p}, \] 也即 \[ \frac{p-1}{(k-1)^p}>\frac1{(k-1)^{p-1}}-\frac1{k^{p-1}}>\frac{p-1}{k^p}. \]
kuing 2# 2013-1-9 14:45
顺便指出,该群里还提及到将原不等式变形为 \begin{align*} \frac{p-1}{k^p}<\frac1{(k-1)^{p-1}}-\frac1{k^{p-1}}&\iff\frac{p-1+k}{k^p}<\frac1{(k-1)^{p-1}} \\ & \iff (k-1)^{p-1}(p+k-1)<k^p \\ & \iff p(k-1)^{p-1}<k^p-(k-1)^p, \end{align*} 然后对右边因式分解再放缩。事实上,由于 $p$ 不一定是整数,所以并不能直接分解。 其实变到这里也可以用拉格朗日,还更容易看些。 那么,有没有中学生能理解的方法?当然有 \[ \frac{p-1}{k^p}<\frac1{(k-1)^{p-1}}-\frac1{k^{p-1}}\iff\frac{p-1}k<\left( \frac k{k-1} \right)^{p-1}-1, \] 令 $k=1/t$,由 $k>1$ 得 $t\in(0,1)$,于是上式等价于 \[ g(t)=\frac1{(1-t)^{p-1}}-1-(p-1)t>0, \] 求导得 \[ g'(t)=(p-1)\left( \frac1{(1-t)^p}-1 \right)>0, \] 从而 $g(t)>g(0)=0$,得证。
kuing 3# 2013-1-9 14:57
用同样的方法可以把另一边也证了。 \[ \frac{p-1}{(k-1)^p}>\frac1{(k-1)^{p-1}}-\frac1{k^{p-1}}\iff\frac{p-1}{k-1}>1-\left( \frac{k-1}k \right)^{p-1}, \] 令 $1/(k-1)=u$,由 $k>1$ 得 $u\in(0,+\infty)$,则上式等价于 \[ h(u)=(p-1)u-1+\frac1{(1-u)^{p-1}}>0, \] 求导得 \[ h'(u)=(p-1)\left( 1+\frac1{(1-u)^p} \right)>0, \] 所以 $h(u)>h(0)=0$。
thread-1032-1-5.html: 一个老链接的三角代换解法及初中解法
kuing 1# 2013-1-9 16:53
话说刚才群里提到了一道方程组的题: 我翻到了5年多以前的老链接: http://bbs.pep.com.cn/forum.php?mod=viewthread&tid=321820 均值解法在贴中已经有了,这里我顺便把三角解法发上来,其实是大概一年多前写的,不过没发到论坛上。 令 $x=\frac12\tan a$, $y=\frac12\tan b$, $z=\frac12\tan c$,其中 $a$, $b$, $c\in(-\pi/2,\pi/2)$,则方程组化为 \[\left\{\begin{aligned} \frac12\tan b&=\sin^2a,\\ \frac12\tan c&=\sin^2b,\\ \frac12\tan a&=\sin^2c, \end{aligned}\right.\] 三式相乘并去分母得 \[\sin a\sin b\sin c=8\sin^2a\sin^2b\sin^2c\cos a\cos b\cos c,\] 或 \[\sin a\sin b\sin c=\sin a\sin b\sin c\sin2a\sin2b\sin2c,\] 于是有 \[\sin a\sin b\sin c=0~\text{或}~\sin2a\sin2b\sin2c=1.\] 由前者显然解得 $a=b=c=0$,即 $x=y=z=0$ 为原方程组的一组解; 由后者得 $\sin2a$, $\sin2b$, $\sin2c$ 都为 $1$ 或一个为 $1$ 另外两个为 $-1$,若 $\sin2a=-1$ 则 $\tan a=-1<0$,又由原方程组显然 $x$, $y$, $z$ 为非负,所以 $\sin2a=-1$ 不满足方程组,另外两个也一样,因此此时只能 $\sin2a$, $\sin2b$, $\sin2c$ 都为 $1$,即得 $x=y=z=1/2$ 为原方程组的另一组解。
kuing 2# 2013-1-9 17:15
改均值解法改写一下,便得所谓初中解法: 当 $xyz\ne0$ 时,将三个等式倒数再配方,得 \[\left\{\begin{aligned} \frac1y&=1+\frac1{4x^2}=\left( 1-\frac1{2x} \right)^2+\frac1x, \\ \frac1x&=1+\frac1{4z^2}=\left( 1-\frac1{2z} \right)^2+\frac1z, \\ \frac1z&=1+\frac1{4y^2}=\left( 1-\frac1{2y} \right)^2+\frac1y, \end{aligned}\right.\] 相加得 \[\left( 1-\frac1{2x} \right)^2+\left( 1-\frac1{2y} \right)^2+\left( 1-\frac1{2z} \right)^2=0.\]
kuing 3# 2013-1-9 17:42
如果考虑复数范围的话,那除了最原始的代入消元法之外我就没什么法子了,消去 $y$, $z$ 化简最终可得 \[x (2 x-1)^2 (5696 x^6+1600 x^5+496 x^4+96 x^3+28 x^2+4 x+1)=0,\] 6个复数解
thread-1033-1-4.html: [不等式] 不等式证明
yayaweha 1# 2013-1-10 00:04
本帖最后由 yayaweha 于 2013-2-8 18:27 编辑 证明   $$\frac{2}{1+2}+\frac{2^2}{1+2^2}+.........+\frac{2^n}{1+2^n}>\frac{n^2}{n+1}$$
kuing 2# 2013-1-10 02:10
前面保留足够项,后面用 2^k/(1+2^k)=1-1/(1+2^k)>1-1/(k(k+1))
hnsredfox_007 3# 2013-1-10 08:29

hnsredfox_007 4# 2013-1-10 09:38
本帖最后由 hnsredfox_007 于 2013-1-10 19:44 编辑 由于$$\sum_{k=1}^n\frac{2^k+1}{2^k}=n+1-\frac{1}{2^{n+1}}< n+1,$$于是$$(n+1)\sum_{k=1}^n \frac{2^k}{2^k+1} > \sum_{k=1}^n \frac{2^k+1}{2^k}\sum_{k=1}^n\frac{2^k}{2^k+1}> \left(\sum_{k=1}^n 1\right)^2=n^2,$$ 即 $$\sum_{k=1}^n\frac{2^k}{2^k+1}> \frac{n^2}{n+1},$$ 也就是 $$\frac{2}{1+2}+\frac{2^2}{1+2^2}+\cdots+\frac{2^n}{1+2^n} >\frac{n^2}{n+1}.$$
kuing 5# 2013-1-10 16:25
牛方法 PS、行间公式中一般不需要 \dfrac 只需 \frac,除非繁分式等
hnsredfox_007 6# 2013-1-10 19:41
5# kuing 谢谢。不熟悉。慢慢学习中……
yes94 7# 2013-1-10 21:09

yes94 8# 2013-1-10 21:13
没来得及检查,
kuing 9# 2013-1-10 21:34
跟3#或4#实质一样
yes94 10# 2013-1-10 22:16
这个倒没注意和他的证法是不是一样, 只是我喜欢用权方和不等式书写, 当然所有解法最后实质都是一样的,因为殊途同归嘛! 下面这个解法呢? 实质上也用了那个当比数列求和,似乎无法逃避。
yayaweha 11# 2013-1-10 22:23
10# yes94 这个引理是怎么想的?
yes94 12# 2013-1-10 22:29
11# yayaweha 实质就是柯西不等式的证法之一,再改造一下就行了,并且让它披上神秘的面纱,
yayaweha 13# 2013-1-10 22:31
12# yes94 我还是不明白这引理
yayaweha 14# 2013-1-10 22:35
都是柯西不等式
yes94 15# 2013-1-10 23:11
本帖最后由 yes94 于 2013-1-10 23:18 编辑 14# yayaweha 那这次不用引理,也不用柯西,用看得懂的二元均值不等式:
第一章 16# 2013-2-7 20:31
这题好像不难啊,好像不用做得这么复杂。难道我想错了?
第一章 17# 2013-2-7 20:45
呵呵,看来是我小看了它了。 自己的证法有点小问题,没办法进行下去 。
yes94 18# 2013-2-8 09:54
还有其他证法,只是要从$n$要从某个数开始放缩,例如从$n\geqslant5$开始放缩。
第一章 19# 2013-2-8 10:01
嗯,昨天就是想通过比较对应项的大小去证明,不过有点小麻烦,就第四项的时候要做个说明。 也可以证明的。
yes94 20# 2013-2-8 10:34
19# 第一章 那就甩出来看看,
thread-1033-2-4.html:
yayaweha 21# 2013-2-8 14:32
本帖最后由 yayaweha 于 2013-2-8 18:28 编辑 来一道很经典的一个不等式,大家有什么解法都发上来吧! 求证:$$\frac{1}{n+1}+\frac{1}{n+2}+\cdots+\frac{1}{2n}<\frac{4}{5}$$
yayaweha 22# 2013-2-8 14:41
本帖最后由 yayaweha 于 2013-2-8 18:33 编辑 记$R_n=C_{n+1}+C_{n+2}+\cdots+C_{2n}$,     $$R_{n+1}-R_n=\frac{1}{2n+1}+\frac{1}{2n+2}-\frac{1}{n+1}$$ 所以数列$\{R_n\}$为増数列,所以$$R_n<\lim_{n\to\infty}R_n=\large\int_{0}^{1}\frac{1}{1+x}dx=ln(1+x)|_{0}^{1}=ln2<\frac{4}{5}$$
kuing 23# 2013-2-8 14:44
21# yayaweha 积分……柯西神马的……
yayaweha 24# 2013-2-8 14:48
22# yayaweha 谁解释一下为什么积分区间是0到1
kuing 25# 2013-2-8 15:21
24# yayaweha \[\sum_{k=1}^n\frac1{n+k}=\sum_{k=1}^n\frac1n\cdot\frac1{1+\frac kn}\] 注意 k 从 1 到 n 时 k/n 由 1/n 到 1,故当 n 无穷…… PS、ln -> \ln ,还有你那小于号为什么总是全角的?没切换到纯英文输入法下输入?
第一章 26# 2013-2-8 15:38
本帖最后由 第一章 于 2013-2-8 15:39 编辑 20# yes94 吃个午饭到现在才回来。 把证明打一下,大家帮忙看看。
第一章 27# 2013-2-8 15:44
来了
kuing 28# 2013-2-8 16:49
27# 第一章 其实我 2# 就是这个意思……
yayaweha 29# 2013-2-8 16:55
25# kuing 小于号怎么打,我不会
yayaweha 30# 2013-2-8 16:57
那个<我是用搜狗打的
kuing 31# 2013-2-8 18:20
30# yayaweha 直接在键盘里打啊,不用任何输入法……
yayaweha 32# 2013-2-8 18:27
我终于知道了
yayaweha 33# 2013-2-8 18:28
怎么看是半角还是全角
kuing 34# 2013-2-8 18:31
   $1<2$ 半角打出来的效果    $1<2$ 全角打出来的效果    细看就能看出区别
yayaweha 35# 2013-2-8 18:32
全角的大一点
yes94 36# 2013-2-8 18:34
21# yayaweha 方法多啊~! 可是,要过年了的嘛!
kuing 37# 2013-2-8 18:37
35# yayaweha 更重要的是mathjax将全角符号看成是普通的字符,所以不会对它与其他的字符之间作适当的距离调整。正常的二元关系符一般都会与两边的东西自动产生距离,上面的对比很清楚了。
yayaweha 38# 2013-2-8 18:40
本帖最后由 yayaweha 于 2013-2-8 18:51 编辑 问个东西 $p(x),q(x)是关于x的多项式,p的次数小于q,q(x)=(x-x_1)^{a_1}(x-x_2)^{a_2}\cdots(x-x_n)^{a_n}$ $那么\frac{p(x)}{q(x)}都能写成 \frac{常数}{(x-x_1)^{a_1}}+ \frac{常数}{(x-x1)^{{a_1}-1}}+\cdots+ \frac{常数}{(x-x_1)}+ \frac{常数}{(x-x_2)^{a_2}}+\cdots一直加到 \frac{常数}{(x-x_n)}$ 的形式,低次的待定系数即可求每个分子上的常数.              这是什么原理
yayaweha 39# 2013-2-8 21:07
38# yayaweha 这个系数怎么定
kuing 40# 2013-2-8 21:23
39# yayaweha 我也不是很清楚……估计要找本高等代数看看……我对这些也是大概知道一点点而已
thread-1033-3-4.html:
yes94 41# 2013-2-8 22:00
38# yayaweha 这个系数怎么定 yayaweha 发表于 2013-2-8 21:07 要么通分后,比较系数;要么赋值法建立若干方程组。 在《数学分析》(数学专业)或《高等数学》(非数学专业)关于有理函数的积分里有。 设$t=(x-x_1)^{-1}$,则$\dfrac{p(x)}{q(x)}=f(t)$,将$f(t)$按泰勒公式(亦叫马克劳林公式)展开即可,所以系数为常数。
realnumber 42# 2013-2-8 23:14
说原理,估计得看系统的,;要理解或应用的话,按特殊到一般做实验好了,先次数低点,分母因式2项开始....,你都已经自己写好等式了,...
thread-1034-1-5.html: [数论] 群里看到的,数列,数论
realnumber 1# 2013-1-10 17:51
本帖最后由 realnumber 于 2013-1-13 16:20 编辑 觉得特别,先留下,=会来考虑,虽然群里有人称已经解决了. 暂时看不进去,想不出再来看下面解答.粗略地看似乎是对的.
realnumber 2# 2013-1-13 16:27
本帖最后由 realnumber 于 2013-1-13 16:30 编辑 先尝试"利器"--"特殊到一般" 试了下$a_1=a_2=1$,$a_{n+2}=\frac{a_{n+1}+1}{a_n}$,前几项依次为1,1,2,3,2,1,2,3,2,1,很简单,后面还出现周期. $a_1=a_2=a_3=1$,$a_{n+3}=\frac{a_{n+1}a_{n+2}+1}{a_n}$,这个前几项依次为1,1,1,2,3,7,11,26看起来也成立,怎么证明得得继续想. $a_1=a_2=a_3=a_4=1$,$a_{n+4}=\frac{a_{n+1}a_{n+2}a_{n+3}+1}{a_n}$试了前几项,也成立. 猜是第2个解决的话,可以推广到1楼问题
yes94 3# 2013-1-13 21:40
本帖最后由 yes94 于 2013-1-13 21:44 编辑 先尝试"利器"--"特殊到一般" 试了下$a_1=a_2=1$,$a_{n+2}=\frac{a_{n+1}+1}{a_n}$,前几项依次为1,1,2,3,2,1,2,3,2,1,很简单,后面还出现周期. $a_1=a_2=a_3=1$,$a_{n+3}=\frac{a_{n+1}a_{n+2}+1}{a_n}$,这个前几项依 ... realnumber 发表于 2013-1-13 16:27 已知$a_1=a_2=a_3=1$,$a_{n+3}=\frac{a_{n+1}a_{n+2}+1}{a_n}$,这个是二阶线性递归数列,其通项公式为:                $a_n=mx^n+ny^n$ 由前四项可得$m,n,x,y$。
realnumber 4# 2013-1-13 21:55
3# yes94 看来我说错了.如果这个成立,也不知道如何推广到1楼问题.
thread-1035-1-5.html: [组合] 群里看到的排列组合一题(今天成都一诊的题)
realnumber 1# 2013-1-10 21:40
本帖最后由 realnumber 于 2013-1-11 08:11 编辑 有数学老师3名。语文老师2名,语文老师有一个是干部。英语老师3名,其中两个是干部。将这八个老师全分到三个区去,并且每个区必须有一个干部。a区,至少有一名数学老师,c区,必须有两名英语老师。问共多少分配方法? 1.语文干部在a区,两个英语干部去B,C有$C_2^1$,那么剩下的英语老师必须去C区.     a区有且只有1名数学$C_3^1$,另两位数学都有$C_2^1$种选择,余下的1位语文有三种选择.      $C_2^1C_3^1(C_2^1)^2C_3^1=72$       a区有且只有2名数学$C_3^2$,另1位数学有$C_2^1$种选择,余下的1位语文有三种选择.      ${C_2^1}{C_3^2}{C_2^1}{C_3^1}=36$      a区有且只有3名数学$C_3^2$,余下的1位语文有三种选择.      ${C_2^1}{C_3^3}{C_3^1}=6$ 2.语文干部去B区,和1一样多. 3.语文干部去C区,因为有2个英语干部去ab,那么c区需要2个英语没法满足,这条没有. 本题最后答案是$2(72+36+6)=228$.
thread-1036-1-1.html: 来自粉丝群的一道积分不等式$\int_0^1(f''(x))^2\rmd x\ge4$
kuing 1# 2013-1-11 00:01
题目:设函数 $f$ 在 $[0,1]$ 上有二阶连续导数,$f(0)=f(1)=f'(0)=0$,$f'(1)=1$,求证: \[\int_0^1{(f''(x))^2\rmd x}\geqslant 4,\] 并指出等号成立的条件。 待定一个 $g(x)$,由柯西不等式,有 \begin{align*} \int_0^1{(f''(x))^2\rmd x}\int_0^1{g(x)^2\rmd x}&\geqslant \left( \int_0^1{f''(x)g(x)\rmd x} \right)^2 \\ & =\left( \int_0^1{g(x)\rmd{(f'(x))}} \right)^2 \\ & =\left( g(x)f'(x)|_0^1-\int_0^1{f'(x)\rmd{(g(x))}} \right)^2 \\ & =\left( g(1)-\int_0^1{f'(x)g'(x)\rmd x} \right)^2 \\ & =\left( g(1)-\int_0^1{g'(x)\rmd{(f(x))}} \right)^2 \\ & =\left( g(1)-g'(x)f(x)|_0^1+\int_0^1{f(x)\rmd{(g'(x))}} \right)^2 \\ & =\left( g(1)+\int_0^1{f(x)g''(x)\rmd x} \right)^2, \end{align*} 为了把后面的积分弄掉,尝试令 $g(x)=x+k$,代入得到 \[\int_0^1{(f''(x))^2\rmd x}\geqslant \frac{(1+k)^2}{\int_0^1{(x+k)^2\rmd x}}=\frac{3(k+1)^2}{3k^2+3k+1},\] 令 \[\frac{3(k+1)^2}{3k^2+3k+1}=4,\] 解得 $k=-1/3$,所以当 $g(x)=x-1/3$ 时就得到了原不等式。等号成立的条件是 $f''(x)=p(x-1/3)$。 PS、事实上 $3(k+1)^2/(3k^2+3k+1)$ 的最大值就是 $4$。
q85669551 2# 2013-1-13 21:13
这个尝试真大胆..
kuing 3# 2013-1-14 00:04
2# q85669551 不是大胆,而是除了这样我已经没别的办法进行下去了……
pxchg1200 4# 2013-1-14 09:03
3# kuing Cauchy-Schwarz是明智的。
kuing 5# 2013-1-14 10:24
4# pxchg1200 CS在积分不等式中用着也那么爽
pxchg1200 6# 2013-1-14 11:51
5# kuing 是啊,很多好玩的东东呢。
thread-1037-1-5.html: [几何] 立几题,涉及旋转,
realnumber 1# 2013-1-11 11:08
本帖最后由 realnumber 于 2013-1-11 22:58 编辑 一个棱长均为a的正四面体模型,它的顶点O在桌面内,且该模型绕O点转动,记模型上最高点到桌面的距离为h,则h的取值范围是. 也是不等式群看到的, 由图象猜测是$[\frac{\sqrt{2}a}{2},a]$(南京支激扬(532###902)),左边是其中一个面平行桌面或就在桌面上,右边就是棱长. 有没有更严密的一般性办法,以及能否推广到其它多面体上? 试着向量,但考虑不够成熟,如下: 最高距离就在其余三顶点某一点取到(需要说明吗?但我不知道怎么说明.), 设O出发的三向量记为$\vv{a},\vv{b},\vv{c}$,不妨先设$a=1$,那么$\abs{\vv{a}}=\abs{\vv{b}}=\abs{\vv{c}}=\abs{\vv{a}-\vv{b}}=\abs{\vv{a}-\vv{c}}=\abs{\vv{c}-\vv{b}}=1$ 桌面的单位法向量为$\vv{n}=x\vv{a}+y\vv{b}+z\vv{c}$,由$\abs{\vv{n}}=1$,得到$x^2+y^2+z^2+xy+yz+xz=1$. 那么$\vv{a},\vv{b},\vv{c}$在$\vv{n}$上的射影绝对值依次为$\abs{x+0.5y+0.5z}$,$\abs{y+0.5x+0.5z}$,$\abs{z+0.5y+0.5x}$(kuing).由桌面上模型可得,这些绝对值可以直接去掉,因为都非负. 本题就是$f(x,y,z)=max\{x+0.5y+0.5z,y+0.5x+0.5z,z+0.5y+0.5x\}$,求$f(x,y,z)$的取值范围. 推广问题1:一个棱长均为a的正方体,它的一个顶点O在桌面内,且该模型绕O点转动,记模型上最高点到桌面的距离为h,则h的取值范围是. 同样,最高距离就在其余7顶点某一点取到. 设设O出发的三向量记为$\vv{a},\vv{b},\vv{c}$,不妨先设$a=1$,那么$\abs{\vv{a}}=\abs{\vv{b}}=\abs{\vv{c}}=1$, $\vv{a}\cdot\vv{b}=\vv{a}\cdot\vv{c}=\vv{c}\cdot\vv{b}=0$ 2.可能还是这样更一般,就任意一个四面体(不知道可不可以计算)或一个长方体.
realnumber 2# 2013-1-11 12:53
转发下,1楼支老师的想法
kuing 3# 2013-1-11 15:48
接1#: 令 $p=x+0.5y+0.5z$, $q=0.5x+y+0.5z$, $r=0.5x+0.5y+z$,则 $p$, $q$, $r\geqslant0$ 且 $3(p^2+q^2+r^2)-2(pq+qr+rp)=2$。 由对称性不妨设 $p\geqslant q\geqslant r\geqslant 0$,则问题等价于求 \[\frac{\sqrt2p}{\sqrt{3(p^2+q^2+r^2)-2(pq+qr+rp)}}\] 的取值范围。由齐次性令 $p+q+r=1$,则 $p\in[1/3,1]$,且 \[\frac{\sqrt2p}{\sqrt{3(p^2+q^2+r^2)-2(pq+qr+rp)}}=\frac{\sqrt2p}{\sqrt{3-8p(1-p)-8qr}},\] 而 \[ \max\{0,p(1-2p)\}\leqslant qr\leqslant\frac{(1-p)^2}4, \] 所以只需要分别求出以下两个函数在 $[1/3,1]$ 上的值域即可 \begin{align*} g(p)&=\frac{\sqrt2p}{\sqrt{3-8p(1-p)-8\max\{0,p(1-2p)\}}},\\ h(p)&=\frac{\sqrt2p}{\sqrt{3-8p(1-p)-2(1-p)^2}}, \end{align*} 不难求出为 $\bigl[\sqrt{1/2},\sqrt{3/4}\bigr]$ 和 $\bigl[\sqrt{2/3},1\bigr]$,因此所求的取值范围就是 $\bigl[\sqrt{1/2},1\bigr]$。
kuing 4# 2013-1-11 18:16
其实正方体的情形应该更简单,如果允许穿过桌面的话可能才有点玩头。 这也让我想起之前遇到过的一个问题,就是一个平面过正方体某个顶点,另外七个顶点到该平面的距离分别为1,2,3,4,5,6,7,求正方体棱长,结果发现不止一种情况。
realnumber 5# 2013-1-12 15:21
本帖最后由 realnumber 于 2013-1-13 23:25 编辑 你也说起过的换元,终于搞定.但愿不要再出错了,改成脱开图形的办法应该可以,但不想费那个脑筋了. 设$x+y=p,y+z=q,z+x=r$,那么已知条件即为$p^2+q^2+r^2=2,p+q\ge0,p+r\ge0,q+r\ge0$, 求$f(x,y,z)=max\{\frac{p+q}{2},\frac{p+r}{2},\frac{r+q}{2}\}$的取值范围. 容易得$2=p^2+q^2+r^2\ge{p^2+q^2}\ge{2(\frac{p+q}{2})^2}$,即$\frac{p+q}{2}\le 1$.所以$f(x,y,z)_{max}=1$. 又不妨设$r$最大,那么$f(x.y,z)=max\{\frac{p+r}{2},\frac{r+q}{2}\}$, 固定$r$,条件为$p^2+q^2=2-r^2,p+q\ge0$如图所示所以在$p=q$或$p=-q\ge0$或$q=-p\ge0$时即D或E或F三点,$f(x,y,z)$才取到最小值. 不妨设$p\ge0$也即问题为:已知:$2p^2+r^2=2,0\le{p}\le{r}$,求$f(x,y,z)=\frac{p+r}{2}$的最小值,如图,容易得$p=0,r=\sqrt{2}$取到,完.
kuing 6# 2013-1-12 15:35
还是整个换了方便……
realnumber 7# 2013-1-12 16:17
本帖最后由 realnumber 于 2013-1-12 16:41 编辑 要不先补成正方体,估计就是换元后的办法.可惜一般四面体没对应长方体,只是个平行六面体. 设四面体从O出发的三棱所在向量依次为$\vv{a},\vv{b},\vv{c}$, 并已知它们的长度依次为$a,b,c$和夹角$<\vv{a},\vv{b}>=\alpha,<\vv{b},\vv{c}>=\beta,<\vv{c},\vv{a}>=\gamma$. 桌面单位法向量为$\vv{n}=x\vv{a}+y\vv{b}+z\vv{c}$,那么由$\abs{\vv{n}}=1$可得$a^2x^2+b^2y^2+c^2z^2+2abxy\cos{\alpha}+2acxz\cos{\gamma}+2bcyz\cos{\beta}=1$ 求$f(x,y,z)=max\{ \vv{n}\cdot\vv{a},\vv{n}\cdot\vv{b},\vv{n}\cdot\vv{c}\}$的取值范围.似乎很复杂,不会解了,就晾着吧.
thread-1038-1-5.html: [几何] 来自人教群的两个圆的平几
kuing 1# 2013-1-12 20:52
(1)由 $\angle ACB=\angle AEB$,$\angle AFB=\angle ADB$ 得 $\triangle ACF\sim \triangle AED$,从而易得 $\angle CAE=\angle DAF$,所以 $CD$ 的中垂线正是 $\angle EAF$ 的角平分线。 设 $\angle EAF$ 的角平分线与 $\triangle AEF$ 的外接圆交于 $P'$,则由 $\angle EAP'=\angle FAP'$ 得 $P'E=P'F$,即 $P'$ 在 $EF$ 的中垂线上,因此 $P'$ 为 $CD$ 的中垂线与 $EF$ 的中垂线的共公点,所以这两条中垂线或相交或重合。如果重合,即中垂线与角平分线重合,则 $AE=AF$,从而可得 $\triangle ACF$ 和 $\triangle AED$ 为全等的等腰三角形,于是可得两圆半径相等,与条件矛盾,所以两条中垂线必然是相交。 (2)由(1)的证明过程知 $P$ 就是 $P'$,即 $P$ 在 $\triangle AEF$ 的外接圆上。由 $\triangle ACF\sim \triangle AED$ 得 \[CA^2=AC\cdot AD=AE\cdot AF,\] 所以要证原命题只需证明 \[AP^2=AE\cdot AF+EP^2,\] 由托勒密定理,有 \[AP\cdot EF=EP\cdot AF+FP\cdot AE=EP\cdot (AE+AF),\] 于是 \begin{align*} AP^2-AE\cdot AF-EP^2&=\frac{EP^2\cdot (AE+AF)^2}{EF^2}-AE\cdot AF-EP^2 \\ & =AE\cdot AF\cdot \left( \frac{2EP^2}{EF^2}-1 \right)+EP^2\cdot \left( \frac{AE^2+AF^2}{EF^2}-1 \right) \\ & =AE\cdot AF\cdot \frac{EP^2+FP^2-EF^2}{EF^2}+EP^2\cdot \frac{AE^2+AF^2-EF^2}{EF^2} \\ & =AE\cdot AF\cdot \frac{2EP\cdot FP\cdot \cos \angle EPF}{EF^2}+EP^2\cdot \frac{2AE\cdot AF\cdot \cos \angle EAF}{EF^2} \\ & =AE\cdot AF\cdot \frac{2EP^2\cdot \cos \angle EPF}{EF^2}-EP^2\cdot \frac{2AE\cdot AF\cdot \cos \angle EPF}{EF^2} \\ & =0, \end{align*} 得证。
kuing 2# 2013-1-12 21:33
感觉我这证得有点麻烦,不知有没有更简单的证法。 PS、那个“今日竞赛之几何”是何意?
力工 3# 2013-1-13 14:48
2# kuing 2013CmO第一天题。
kuing 4# 2013-1-13 15:48
3# 力工 噢?原来是CMO是这个时间……呵呵,那有官方答案没?
thread-1039-1-3.html: [数论] (网络上收集)无理数的一些小而精巧的证明
realnumber 1# 2013-1-13 09:35
本帖最后由 realnumber 于 2013-1-13 23:48 编辑 1.$\sqrt{2}$是无理数,数学科普类读物一般有,比如<数学是什么>柯朗,新浪爱问上免费下. 证明:假设$\sqrt{2}$是有理数,那么有$\sqrt{2}=\frac{m}{n}$,其中$m,n\in{Z}$,$m,n$互素,ps,用记号$(m,n)=1$表示. 可得$2n^2=m^2$,所以$m^2$为偶数,那么$m$也为偶数.令$m=2k,k\in{Z}$,那么有$n^2=2k^2$,可得$n$也是偶数,与$(m,n)=1$矛盾. 所以$\sqrt{2}$不是有理数,即为无理数.        这样的练习$\sqrt6,2^\frac{1}{3}$是无理数,你也会证明了吧. 2.$\log_{10}2$是无理数,<数论导引>的一个简单习题.   证明:假设$\log_{10}2=\frac{m}{n}$,其中$m,n\in{Z}$,$(m,n)=1$,即$2^n=10^m$,右边是5的倍数,左边不是,矛盾. 3.$0.12345678910111213.....$是无理数,见彭翕成与数学传播http://blog.sina.com.cn/s/blog_6029f03301017fun.html 4.判断命题真假.命题:若$a$是无理数,则$a^\sqrt{2}$也是无理数. 假设是真命题,考虑${\sqrt{2}}^\sqrt2$,若它是有理数,则与假设矛盾;若它是无理数,考虑${{(\sqrt{2}}^\sqrt2)}^\sqrt2=2$,也与假设矛盾.--这个证明有趣的地方在于${\sqrt{2}}^\sqrt2$是不是有理数依然没有证明(k12出现过,一位网友这么评论,印象深刻). 5.循环小数怎样化为分数(即有理数).利用等比数列求和公式,比如$0.3333..=0.3+0.03+0.003+..$或方程 设$0.3333..=x$,那么$10x=3+x$,那么$x=\frac{1}{3}$.(看起来挺不错,可惜这些不得不涉及无穷,极限) 6.判断整系数方程有没有有理根.刚刚在群里看到$6x^3+3x^2-3x+1=0$左边为什么不能分解(要求整系数因式) 若$6x^3+3x^2-3x+1=(ax+b)(cx^2+dx+e)=acx^3+...+be$,其中$a,b,c,d,e$都是整数,得到$ac=6,be=1$, 也就是说方程若有一个有理根$-\frac{b}{a}$,那么分子$b$是$be$的因数,分母$a$是$ac$的因数.本题只需要检验$±1,±\frac{1}{2},±\frac{1}{3},±\frac{1}{6}$. 7.五边形老师的2007竞赛培训1---有理数无理数http://www.aoshoo.com/bbs1/dispb ... ;ID=8200&skin=0 ps,奥数论坛假期可能会再度热闹起来,可以先收藏起来. 8.$\pi,e$的超越性见<数论导引>,新浪爱问上免费下,ps,中文都认识,意思全然不懂,远没到能看懂水平. 9.见<无理数的发现>.pdf新浪爱问.
Tesla35 2# 2013-3-17 22:34
mark
thread-104-1-9.html: [不等式] 2011联赛A卷一试4三角不等式$\cos^5x-\sin^5x <7(\sin^3x-\cos^3x)$
kuing 1# 2011-10-16 16:53
$\cos^5x-\sin^5x <7(\sin^3x-\cos^3x)$,$x\in[0,2\pi)$。 两边都因式分解一下,有 \begin{align*} \cos^5x-\sin^5x &=\frac{1}{8}(\cos x-\sin x)(\cos4x+4\sin2x+7),\\ \sin^3x-\cos^3x &=\frac{1}{2}(\sin x-\cos x)(\sin2x+2), \end{align*} 于是不等式等价于 \[(\sin x-\cos x)(\cos4x+32\sin2x+63)>0,\] 显然恒有 $\cos4x+32\sin2x+63>0$,所以不等式等价于 \[\sin x>\cos x,\] 解得 \[\frac{\pi}{4}<x<\frac{5\pi}{4}.\]
kuing 2# 2011-10-16 17:01
这个7也太大了吧,以致于剩下那项竟然是显然非负的,不会是抄错题吧?弄小一点,最好弄成负的,另一项还可以分解一下好点,现在这样太废了……
GAM 3# 2011-10-16 22:13
话说我直接移项就是函数了,之后就有sin>cos...
kuing 4# 2011-10-16 22:38
3# GAM 是的,我习惯性因式分解了,呵呵。所以还是那个7太大了,得是负的才有玩头
nash 5# 2011-10-22 22:05
因式分解化了一下 其实蛮简单的
thread-1040-1-1.html: 向量上的箭头能不能一样高?
abababa 1# 2013-1-13 11:35
$\vv{a}+\vv{b}+\vv{c}$ 我安装了latex,打向量的时候字母b上的箭头看起来比其它两个都高点,可我看有些书里向量上的箭头就是一样高的,是怎么做的呢?
kuing 2# 2013-1-13 12:55
因为那些书不是用 latex 排的…… 在 latex 里要一样高,你可以试试重定义一下 \vv ,让它在里面加个 \mathstrut(这是一个高度和深度相当于普通括号且宽度为0的盒子,纯粹就是撑高用的)
kuing 3# 2013-1-13 13:01
不过我觉得不一样高也没什么所谓,主要是 latex 默认的箭头不太好看,建议用 esvect 宏包。
abababa 4# 2013-1-13 17:36
3# kuing $\vv{\mathstrut a}+\vv{\mathstrut b}+\vv{\mathstrut c}$ 谢谢。但用了mathstrut,视觉上a,c上的箭头又比b上的箭头要高了,虽然实际是一样高。唉,还是用原来的吧。
kuing 5# 2013-1-13 18:04
呵呵,视觉误差吧。 其实也可以用 \vphantom{b} 产生与字母 $b$ 等高且宽度为 0 的盒子,我刚才试了一下发现 \mathstrut 的高度比 b 要略高一点点。
kuing 6# 2013-1-13 19:08
最后那里放大点看清楚些:
thread-1041-1-1.html: 怎么才能让标点符号占半格?
abababa 1# 2013-1-13 11:39
比如我打$AB$,$CD$,中间那个逗号是中文的,但是它看起来太宽了,怎么能让它显示成中文的逗号,又让它只占半个格呢?
kuing 2# 2013-1-13 12:40
xeCJK 宏包里有半角选项,详情见 xeCJK 宏包的说明文档。
abababa 3# 2013-1-13 17:10
本帖最后由 abababa 于 2013-1-13 17:12 编辑 2# kuing 谢谢,是开明式、全角式、半角式那些吗?我试了,但是全都变成窄的了,我只想让它在数学模式里变窄点,在普通的汉语行文里还是原样。 可能做不到这一点吧,它也认不出来我打的$AB$,$CD$中间的逗号是数学里的。
kuing 4# 2013-1-13 17:22
原来你的意思是这样。 我想唯一的办法大概是将半角逗号映射到全角逗号去并且用一个半格宽度的盒子装起它,然后你用 \$AB,CD\$ 这样就可以了…… 但我不会弄这种东东。
thread-1042-1-5.html: [函数] 有关极值问题(似乎是高三模拟试题)
realnumber 1# 2013-1-13 20:05
已知函数$f(x)=x^3+3ax^2+6bx$的两个极值$x_1,x_2$满足$x_1\in{[-1,0]}$,$x_2\in{[1,2]}$,求f$(x_2)$的最大值与最小值的和.
kuing 2# 2013-1-13 20:46
没想到你对这种题都有兴趣……我一般是懒得看的 不过既然发在这里还是看了一下,其实还蛮简单? 依题意可设 $f'(x)=3(x-x_1)(x-x_2)$,然后求不定积分并确定常数(也可以对原来给出的式子求导对比系数求出 $a$, $b$),易得 \[f(x)=x^3-\frac32(x_1+x_2)x^2+3x_1x_2x \riff f(x_2)=\frac12x_2^2(3x_1-x_2),\] 然后由所给的范围即得 \begin{align*} f(x_2)&\geqslant \frac12x_2^2(-3-x_2)\geqslant -10, \\ f(x_2)&\leqslant \frac12x_2^2(-x_2)\leqslant -\frac12. \end{align*} PS、\in 后面不需要花括号
q85669551 3# 2013-1-13 21:01
那群里那位初二的小朋友还蛮厉害的诶。
yes94 4# 2013-1-13 21:11
2# kuing 这种题我也不是很想看,学习一下代码输入, 已知函数$f(x)=x^3+3ax^2+6bx$的两个极值$x_1,x_2$满足$x_1\in[-1,0]$,$x_2\in[1,2]$,求$f(x_2)$的最大值与最小值的和.
zwl1972 5# 2013-1-13 21:16
本帖最后由 zwl1972 于 2013-1-13 21:22 编辑 2009年全国一卷压轴题改编,kuing解法很酷!
yes94 6# 2013-1-13 21:22
已知函数$f(x)=x^3+3ax^2+6bx$的两个极值$x_1,x_2$,满足$x_1\in[-1,0]$,$x_2\in[1,2]$,求$f(x_2)$的最大值与最小值的和. realnumber 发表于 2013-1-13 20:05 是2009全国卷Ⅰ理试题,
yes94 7# 2013-1-13 21:28
本帖最后由 yes94 于 2013-1-13 21:29 编辑 2009年全国一卷压轴题改编,kuing解法很酷! zwl1972 发表于 2013-1-13 21:16 我刚刚找到,发表了就看见你发表的了! 设函数$f(x)=x^3+3bx^2+3cx$的两个极值点$x_1,x_2$,且$x_1\in[-1,0]$,$x_2\in[1,2]$,(2)证明$f(x_2)$的最大值与最小值分别是$-10$和$-\frac12$
realnumber 8# 2013-1-13 21:29
本帖最后由 realnumber 于 2013-1-13 21:57 编辑 想不到你们这么快啊!!,只能发部分结果$h(a,b)=f(x_2)=(4b-2a^2)(-a+\sqrt{a^2-2b})-2ab$,可行域见附件 其实不太感兴趣,觉得一帮高手讨论得很热烈,是不是有什么问题,就发过来了.
kuing 9# 2013-1-13 22:00
我也没想到洗了个澡回来竟然多了那么多的回复感谢大家对本论坛的支持
kuing 10# 2013-1-13 22:17
想不到你们这么快啊!!,只能发部分结果$h(a,b)=f(x_2)=(4b-2a^2)(-a+\sqrt{a^2-2b})-2ab$,可行域见附件 其实不太感兴趣,觉得一帮高手讨论得很热烈,是不是有什么问题,就发过来了. realnumber 发表于 2013-1-13 21:29 这样做麻烦,选择一个合适的东东做变量很重要…… 话说,这题居然会有“一帮高手讨论得很热烈”?
realnumber 11# 2013-1-13 23:22
其实他们讨论前,韩安静老师就指出象你这样证明的大方向了.可能是思路,没有过程,都忽视.
kuing 12# 2013-1-13 23:27
难道是不等式群??
realnumber 13# 2013-1-13 23:30
恩,其实高手并不在乎无理式,严文兰老师的一个证明
yes94 14# 2013-1-13 23:52
13# realnumber 我把原高考题标答贴出来吧
pengcheng1130 15# 2013-1-17 21:59
严文兰做的结果和Kuing做的结果不一样,一个是$-10.5$,另一个$-10$.
realnumber 16# 2013-1-17 22:24
本帖最后由 realnumber 于 2013-1-17 22:25 编辑 15# pengcheng1130 严文兰老师的区域算错了一个点(他自己说过);不影响方法.
pengcheng1130 17# 2013-1-17 22:29
谢谢!
thread-1043-1-4.html: [不等式] 3元6次的怎么玩
pxchg1200 1# 2013-1-14 08:50
Let $a,b,c$ be real numbers,prove that: \[ 4(a^6+b^6+c^6)+5(a^5b+b^5c+c^5a)\geq 0 \]
pxchg1200 2# 2013-1-15 11:33
不知道上面那个和这个Le Hai的不等式有什么关系 Let $a,b,c$ be real numbers,with $a+b+c=0 $ and $a^2+b^2+c^2=3$,show that \[ a^5b+b^5c+c^5a\leq -3 \]
pxchg1200 3# 2013-1-17 19:23
本帖最后由 pxchg1200 于 2013-1-25 11:09 编辑 1# pxchg1200 经过很多尝试,我发现了下面更强不等式: \[  4(a^6+b^6+c^6)+5(a^5b+b^5c+c^5a)\ge\frac{(a+b+c)^6}{27} \] 但证明成了问题 BQ说这个也成立 \[ 4(a^6+b^6+c^6)+5(a^5b+b^5c+c^5a)\geq \frac{1}{3}(ab+bc+ca)^2(a+b+c)^2 \]
pxchg1200 4# 2013-1-19 18:07
本帖最后由 pxchg1200 于 2013-1-19 18:08 编辑 2# pxchg1200 :O :O :O :O
kuing 5# 2013-1-19 18:12
这样看上去难度不是一个级别……
pxchg1200 6# 2013-1-19 18:17
5# kuing There exist a nice CS proof for $ \sum{a^2b}$...
kuing 7# 2013-1-19 18:47
我是说看了4#发现1#和2#的相差很远,所以应该没什么联系,难度也不是一个级别……
pxchg1200 8# 2013-1-20 10:51
7# kuing 计算了下,按刚刚那样的替换 \[ \sum{a^4b}=\frac{15}{8}\sqrt{2}\cos{3t}-\frac{9}{8}\sqrt{6}\sin{3t} \] \[ \sum{a^3b}=-\frac{9}{4} \] ?! \[ \sum{a^2b}=\frac{3\sqrt{2}}{4}\cos{3t}-\frac{3\sqrt{6}}{4}\sin{3t} \] \[ \sum{a^6b}=\frac{63\sqrt{2}}{16}\cos{3t}-\frac{27\sqrt{6}}{16}\sin{3t} \]
pxchg1200 9# 2013-1-25 10:58
8# pxchg1200 严文兰老师给出了AM-GM proof   围观中。。。
kuing 10# 2013-1-25 12:15
9# pxchg1200 我更加感兴趣的是你那个加强式,因为有(1,1,1)取等
kuing 11# 2013-1-25 14:11
咦,原来你更新了3#(BQ's 加强)看来也是有(1,1,1)取等
pxchg1200 12# 2013-1-25 14:11
10# kuing 强配很难啊 三次方的有 $ \sum{a^3}$ $\sum{a^2b}$ $\sum{ab^2}$ $abc$ 要设系数的话有10个呢。。。
kuing 13# 2013-1-25 14:12
12# pxchg1200 嗯,所以我前几天也没待出来,暂且搁置……
pxchg1200 14# 2013-1-25 14:13
11# kuing BQ那个软件灌水dang,那个加强比$ \frac{1}{27}(a+b+c)^6$还弱。
pxchg1200 15# 2013-1-25 14:13
13# kuing 能否pqr?
kuing 16# 2013-1-25 15:42
15# pxchg1200 按我们之前搞的那些顶多处理到5次……
pxchg1200 17# 2013-1-25 17:45
本帖最后由 pxchg1200 于 2013-1-25 17:47 编辑 16# kuing 能不能改进下,让6次以上的都有用?   比如,所有的都可以变成 $F(a,b,c)+G(a,b,c)\cdot (a-b)(b-c)(c-a)$ 能否估计下 $ (a-b)(b-c)(c-a)$ ?
thread-1044-1-4.html: [不等式] 有AM-GM或CS proof的吗?(727)
pxchg1200 1# 2013-1-14 08:54
Let $a,b,c,d>0$,show that: \[ \frac{ab}{7a+2b+7c}+\frac{bc}{7b+2c+7d}+\frac{cd}{7c+2d+7a}+\frac{da}{7d+2a+7b}\leq \frac{a+b+c+d}{16} \] Ps:拆了一个晚上弄不出,估计是我的技术退化了。。。。
pxchg1200 2# 2013-1-25 11:02
本帖最后由 pxchg1200 于 2013-1-25 11:03 编辑 只上图,不说话。
thread-1045-1-5.html: [不等式] 最近做的有趣不等式
pxchg1200 1# 2013-1-14 08:58
1.设点$A(a,b),B(c,d)$在单位圆盘内运动,求证: \[\frac{1-a}{1-a^2-b^2}+\frac{1-c}{1-c^2-d^2}\geq\frac{2-a-c}{1-\left(\frac{a+c}{2}\right)^2-\left( \frac{b+d}{2}\right)^2} \]   2.$a,b,c\geq 0$求证: \[ (a^2+b^2+c^2)^5\geq 27(a^3b^2+b^3c^2+c^3a^2)^2 \] (proposed by tian27546)
pxchg1200 2# 2013-1-14 08:59
Both of them can be proved by Cauchy-Schwarz. :D
kuing 3# 2013-1-14 16:21
2. $a$, $b$, $c\geqslant 0$,求证 \[(a^2+b^2+c^2)^5\geqslant 27(a^3b^2+b^3c^2+c^3a^2)^2.\] CS想不到,只想到了下面这个麻烦的证法。 由轮换对称性,不妨设 $b$ 在 $a$, $c$ 之间,则有 \[c(ab+bc+ca)(a-b)(b-c)\geqslant 0 \iff a^3b^2+b^3c^2+c^3a^2\leqslant b^2(a^3+c^3+a^2c+ac^2-abc),\] 因此要证原不等式只需证 \[(a^2+b^2+c^2)^5\geqslant 27b^4(a^3+c^3+a^2c+ac^2-abc)^2,\] 由齐次性,不妨设 $b=1$,记 $p=a+c$, $q=ac$,则 $q\in[0,p^2/4]$,由 $b$ 在 $a$, $c$ 之间可知 $p\geqslant 1$ 以及上式右边括号内恒为正,所以上式可以两边取对数,整理后等价于 \begin{equation}\label{20130114z10degineqzyz} f(q)=5\ln (1+p^2-2q)-\ln 27-2\ln (p^3-2pq-q)\geqslant 0. \end{equation} 求导得 \[f'(q)=\frac{-10}{1+p^2-2q}+\frac{2(2p+1)}{p^3-2pq-q}=\frac{2(p+1)^2-6p^3+6(2p+1)q}{(1+p^2-2q)(p^3-2pq-q)},\] 则 \[f'(q)=0\iff q=\frac{3p^3-(p+1)^2}{3(2p+1)},\] 可见,无论 $p$ 取何值,$f(q)$ 取最小值时的 $q$ 必为 $0$ 或 $p^2/4$ 或 $(3p^3-(p+1)^2)/(3(2p+1))$,所以只要证明当 $q$ 取这三个值时 $f(q)$ 都是非负即可。经一系列的计算,可以得到 \begin{align*} f(0)&=5\ln (1+p^2)-\ln 27-6\ln p=g(p), \\ f\left( \frac{p^2}4 \right)&=5\ln (2+p^2)-\ln 54-4\ln p-2\ln (2p-1)=h(p), \\ f\left( \frac{3p^3-(p+1)^2}{3(2p+1)} \right)&=6\ln (p+1)+\ln \frac{3125}{729}-5\ln (2p+1)=k(p), \end{align*} 分别求导得 \begin{align*} g'(p)=\frac{2(2p^2-3)}{p(1+p^2)}&\riff g(p)\geqslant g\left( \sqrt{\frac32} \right)=\ln \frac{3125}{2916}>0, \\ h'(p)=\frac{2(p-2)(4p^2+5p-2)}{p(2p-1)(2+p^2)}&\riff h(p)\geqslant h(2)=0, \\ k'(p)=\frac{2(p-2)}{(p+1)(2p+1)}&\riff k(p)\geqslant k(2)=0, \end{align*} 所以式 \eqref{20130114z10degineqzyz} 成立,原不等式得证。
pxchg1200 4# 2013-1-14 16:46
证明: 我们先证明一个引理 \[ (a+b+c)^5\geq 27(ab+bc+ca)(a^2b+b^2c+c^2a) \] 我们可以假定$(b-a)(b-c)\leq 0 $,由此可得 \[ a^2b+b^2c+c^2a\leq b(a^2+c^2+ac) \] 由AM-GM, \[ 27(ab+bc+ca)(a^2b+b^2c+c^2a)\leq 27b(b(c+a)+ca)(a^2+c^2+ac) \] \[ =27b(b(c+a)+ca)((c+a)^2-ca)\leq \frac{27b(c+a)^2(a+b+c)^2}{4}\leq \frac{27(\frac{2(a+b+c)}{3})^3(a+b+c)^2}{8}=RHS \] 由此,引理得证。 用$a,b,c$代替$a^2,b^2,c^2$,不等式变为 \[ (a+b+c)^5\geq 27(a^{\frac{3}{2}}b+b^{\frac{3}{2}}c+c^{\frac{3}{2}}a)^{2} \] 由Cauchy-Schwarz不等式 \[ 27(a^{\frac{3}{2}}b+b^{\frac{3}{2}}c+c^{\frac{3}{2}}a)^{2}\leq 27(a^2b+b^2c+c^2a)(ab+bc+ca) \] 结论显然,不等式证毕。
kuing 5# 2013-1-14 16:50
4# pxchg1200 唉,那个引理我当年也证过,没想到可以用到这里来……其实都是五次方,本应想想的…… 学习了
kuing 6# 2013-1-14 18:10
第一个刚才想玩玩几何意义,没发现出什么,还好CS不难想 \begin{align*} \frac{1-a}{1-a^2-b^2}+\frac{1-c}{1-c^2-d^2}&=\frac1{1+a-\frac{b^2}{1-a}}+\frac1{1+c-\frac{d^2}{1-c}} \\ & \geqslant \frac4{2+a+c-\frac{b^2}{1-a}-\frac{d^2}{1-c}} \\ & \geqslant \frac4{2+a+c-\frac{(b+d)^2}{2-a-c}} \\ & =\frac{2-a-c}{1-\bigl(\frac{a+c}2\bigr)^2-\bigl(\frac{b+d}2\bigr)^2}. \end{align*}
pxchg1200 7# 2013-1-14 18:37
6# kuing Nice! Here is my proof. 证明:两边乘2后再同时减去2,不等式变为 \[ \frac{(1-a)^2+b^2}{1-a^2-b^2}+\frac{(1-c)^2+d^2}{1-c^2-d^2} \geq \frac{2-2a-2c+\frac{(a+c)^2}{2}+\frac{(b+d)^2}{2}}{1-\left(\frac{a+c}{2}\right)^2-\left( \frac{b+d}{2}\right)^2} \] 注意到: \[ 2-2a-2c+\frac{(a+c)^2}{2}=\frac{1}{2}(a^2+c^2+2ac-4a-4c+4)=\frac{1}{2}(2-a-c)^2 \] 所以,不等式变为 \[ \frac{(1-a)^2+b^2}{1-a^2-b^2}+\frac{(1-c)^2+d^2}{1-c^2-d^2} \geq \frac{\frac{1}{2}(2-a-c)^2+\frac{(b+d)^2}{2}}{1-\left(\frac{a+c}{2}\right)^2-\left( \frac{b+d}{2}\right)^2}\] 现在,运用Cauchy-Schwarz不等式: \[ \frac{(1-a)^2}{1-a^2-b^2}+\frac{(1-c)^2}{1-c^2-d^2} \geq \frac{(2-a-c)^2}{2-(a^2+b^2+c^2+d^2)}\geq \frac{\frac{1}{2}(2-a-c)^2}{1-\left(\frac{a+c}{2}\right)^2-\left( \frac{b+d}{2}\right)^2}\] \[ \frac{b^2}{1-a^2-b^2}+\frac{d^2}{1-c^2-d^2} \geq \frac{(b+d)^2}{2-(a^2+b^2+c^2+d^2)}\geq\frac{\frac{(b+d)^2}{2}}{1-\left(\frac{a+c}{2}\right)^2-\left( \frac{b+d}{2}\right)^2}\] 两式相加,不等式得证。
kuing 8# 2013-1-14 18:47
互相学习 第三步变出来的不等式挺漂亮
pxchg1200 9# 2013-1-15 08:50
8# kuing 你的也很牛啊!
thread-1046-1-5.html: [不等式] a=b=c and 4a=2b=c
pxchg1200 1# 2013-1-14 09:15
If are positive real numbers, then \[ \frac{a(a-b)}{17a^2+4ab+6b^2}+\frac{b(b-c)}{17b^2+4bc+6c^2}+\frac{c(c-a)}{17c^2+4ca+6a^2}\geq 0, \] with equality holds if $a=b=c$ and only if$4a=2b=c$ or or any cyclic permutation.
pxchg1200 2# 2013-1-15 10:24
不知道这题能否用我介绍过的CYH技术。
thread-1047-1-4.html: [不等式] kuing来CS吧
pxchg1200 1# 2013-1-14 09:17
Let $a,b,c$ be positive real numbers. Prove that : \[ \frac{bc}{3a^2+b^2+c^2}+\frac{ca}{3b^2+c^2+a^2}+\frac{ab}{3c^2+a^2+b^2}\le\frac{3}{5}. \] (Vasc and Pham Kim Hung, 2005)
pxchg1200 2# 2013-1-26 18:51
硬开后等价为 \[ 9\sum{a^6}-50\sum{a^3b^3}-15\sum{a^5(b+c)}+39\sum{a^4(b^2+c^2)}-5abc\sum{a^3}-20abc\sum{a^2(b+c)}+114a^2b^2c^2\geq 0 \]
zdyzhj 3# 2013-1-26 21:41
it is not easy to give a CS proof, but we can give another proof easily.
yes94 4# 2013-1-26 21:47
it is not easy to give a CS proof, but we can give another proof easily. zdyzhj 发表于 2013-1-26 21:41 牛啊
pxchg1200 5# 2013-1-26 22:36
3# zdyzhj true! Let $ x=\min{(x,y,z)}$ it can be write \[ (x^2+2y^2+2z^2)(3x-y-z)^2(x-y)(x-z)+[14x^4-19x^3(y+z)+25x^2(y^2+z^2)-17x^2yz-13x(y+z)(y^2+z^2)+9(y^4+z^4)+yz(y^2+z^2)+28y^2z^2](y-z)^2\geq 0\]
yes94 6# 2013-1-26 22:40
3# zdyzhj true! Let $ x=\min{(x,y,z)}$ it can be write \[ (x^2+2y^2+2z^2)(3x-y-z)^2(x-y)(x-z)+[14x^4-19x^3(y+z)+25x^2(y^2+z^2)-17x^2yz-13x(y+z)(y^2+z^2)+9(y^4+z^4)+yz(y^2+z^2)+28y^2z^2](y-z) ... pxchg1200 发表于 2013-1-26 22:36 机器?
kuing 7# 2013-1-26 22:56
5# pxchg1200    6# yes94 好久没见过这种分拆了,当年看褚小光他们经常用,我倒用得很少(对上一次用应该是在这里,灰常bao力),也不太熟悉中间的配凑技巧,反正不用软件辅助我是不敢玩这种分拆的…… 早前看过褚小光写的关于这种分拆的文章,一时找不到在哪了……
pxchg1200 8# 2013-1-28 08:42
7# kuing Well,Equality occurs when $ a=b=c$ or $a=2,b=c=3$ up to permutation.with the two euqality cases,I wonder there exist a excellent CS proof or not...
thread-1048-1-5.html: [不等式] 这个不知道放这里合适么
pxchg1200 1# 2013-1-14 09:22
设序列$\{x_{n}\}$满足$a_{m+n}\leq a_{m}+a_{n}$其中$m,n\in N^{+}$ 证明:\[ \sum_{k=1}^{n}{\frac{a_{k}}{k^2}}\geq \frac{a_{n}\ln{n}}{4n} \] (proposed by tian27546)
kuing 2# 2013-1-14 09:26
回复标题: 如果解决办法基本上不是初等的话…
pxchg1200 3# 2013-1-14 09:34
2# kuing 由于不会做,所以不知道用了啥东东呢。。
kuing 4# 2013-1-14 10:12
没有非负条件?
pxchg1200 5# 2013-1-14 10:15
5# kuing 没有啊
kuing 6# 2013-1-14 10:45
那如果全是正的成立,变成全是负不就反向了
pxchg1200 7# 2013-1-14 11:36
6# kuing 再次确认了下,题目没问题。
realnumber 8# 2013-1-17 16:07
本帖最后由 realnumber 于 2013-1-18 08:29 编辑 觉得kuing是对的,$a_n=-n$满足条件,但不等式反向 下面这样行不? 我就当作正项数列处理了,记\[f(a_1,a_2,...,a_n)= \sum_{k=1}^{n}{\frac{a_{k}}{k^2}}-\frac{a_{n}\ln{n}}{4n} \] 固定$a_1,...,a_{n-1}$,看作$a_n$的一次函数,由条件$a_n\le a_{n-1}+a_1$($a_n$的实际范围可能更小,比如$a_n\le a_{n-2}+a_2$), 只需要证明$a_n=a_{n-1}+a_1$以及$a_n=0$成立(一次函数,对应的图象是线段,只需要线段两端点代入,后者显然成立). 当$a_n=a_{n-1}+a_1$时,固定$a_1,...,a_{n-2}$,由条件$a_{n-1}\le a_{n-2}+a_1$ 只需要证明$a_{n-1}=a_{n-2}+a_1$以及$a_{n-1}=0$成立即可. ..... 如此本题只需要证明,当$a_n=a_{n-1}+a_1=a_{n-2}+2a_1=...=na_1$成立即可. 此时\[\sum_{k=1}^{n}{\frac{1}{k}}\geq \frac{\ln{n}}{4} \] 显然成立.
yes94 9# 2013-1-17 18:44
那个tian27546的题80%都暴难,根本都不敢看。
realnumber 10# 2013-1-17 18:51
是的,偶尔才能发现个简单的,上面如果加条件"正项数列" 8楼证明对吗?
thread-1049-1-1.html: [不等式] Tran Quoc Anh 不等式习题集
pxchg1200 1# 2013-1-14 10:22
Hmm..... nguoivn topic test.pdf (304.55 KB) 在这里上传下Tran Quoc Anh的一些不等式,大家有空来玩啊。题目有难的,也有简单的。 建议在这里专门讨论他的不等式 :D 答案可以跟下面,呵呵。 鉴于大家可能不认识 Tran Quoc Anh,这里发一个简介,摘自《Inequalities with Beautiful Solutions》。 附上图片一张:
pxchg1200 2# 2013-1-14 10:37
一中哥好像搞得差不多了,要多来玩哦。 :P
kuing 3# 2013-1-14 10:40
3# pxchg1200 他估计都忙不过来了,大概没空上论坛,更没空敲上来了。
kuing 4# 2013-1-14 12:27
重新补上了介绍,为方便阅读把刚才的无关贴子删掉了。 可以开始玩了。 还有哪些未解决的?
pxchg1200 5# 2013-1-14 16:47
前30道都已经解决了吧
pxchg1200 6# 2013-1-14 16:53
本帖最后由 pxchg1200 于 2013-1-14 17:02 编辑 1-4题都属于同一类问题,具体可以参考Potla同学的文章。 On Maximisation of prod(a-b).pdf (226.93 KB) kuing 接上。。。。
天书 7# 2013-1-14 17:34
跪求这本书的电子版《inequalities with beautiful solutions》
kuing 8# 2013-1-15 00:27
第 187 题: 由于这个第三个当年证过,所以顺手用同样的方法把前两个也搞掉先。 第一个的计算稍麻烦,如下: \begin{equation}\label{nguoivn187s1} 11(\sqrt[4]{a}+\sqrt[4]{b}+\sqrt[4]{c})+3\geqslant 12(ab+bc+ca). \end{equation} 不妨设 $c=\min \{a,b,c\}$,分两类讨论。 (i)$c<16/81$,则易得 \[ \sqrt[4]{c}>\frac{27}8c=\frac98c(a+b+c)>\frac{12}{11}c(a+b+c), \] 故 \[ 11(\sqrt[4]{a}+\sqrt[4]{b}+\sqrt[4]{c})+3\geqslant 11(\sqrt[4]{a}+\sqrt[4]{b})+12c(a+b+c)+3, \] 所以要证式 \eqref{nguoivn187s1} 只要证 \[ 11(\sqrt[4]{a}+\sqrt[4]{b})+3\geqslant 12ab, \] 令 $\sqrt[8]{ab}=u\in (0,\sqrt[4]{3/2})$,则由均值知只要证 \[ g(u)=22u+3-12u^8\geqslant 0, \] 求导得 $g'(u)=2(11-48u^7)$,所以 $g(u)$ 在 $(0,\sqrt[7]{11/48})$ 递增,在 $(\sqrt[7]{11/48},\sqrt[4]{3/2})$ 递减,所以 \[ g(u)\geqslant \min \left\{g(0),g\left(\sqrt[4]{\frac32}\right)\right\}, \] 而 $g(0)=3>0$,$g(\sqrt[4]{3/2})=11\sqrt[4]{24}-24$,又 $11^4-24^3=817>0$,所以 $g(\sqrt[4]{3/2})>0$,从而得到 $g(u)>0$,所以此时式 \eqref{nguoivn187s1} 成立; (ii)$c\geqslant 16/81$,则 $a$, $b$, $c\geqslant 16/81$,而 \begin{align*} \text{式}~\eqref{nguoivn187s1}&\iff 11(\sqrt[4]{a}+\sqrt[4]{b}+\sqrt[4]{c})+3\geqslant 6(a+b+c)^2-6(a^2+b^2+c^2) \\ & \iff\sum{(6a^2+11\sqrt[4]{a}-17)}\geqslant 0, \end{align*} 故只要证出如下不等式即可 \[ 6a^2+11\sqrt[4]{a}-17\geqslant \frac{59}4(a-1), \] 令 $a=t^4$,由 $a\geqslant 16/81$ 得 $t\geqslant 2/3$,上式化为 \[ f(t)=6t^8+11t-17-\frac{59}4(t^4-1)\geqslant 0, \] 求导得 $f'(t)=48t^7+11-59t^3$,则当 $2/3\leqslant t\leqslant 1$ 时,有 \[ f'(t)\leqslant 48t^6+11-59t^3=(t^3-1)(48t^3-11)\leqslant 0, \] 而当 $t\geqslant 1$ 时,有 \[ f'(t)\geqslant 48t^9+11-59t^3=(t^3-1)(48t^6+48t^3-11)\geqslant 0, \] 从而得到 $f(t)\geqslant f(1)=0$,所以此时式 \eqref{nguoivn187s1} 也成立。 综合(i)(ii),式 \eqref{nguoivn187s1} 获证。 第二个的计算简单些,如下: \begin{equation}\label{nguoivn187s2} 5(\sqrt{a}+\sqrt{b}+\sqrt{c})\geqslant 4(ab+bc+ca)+3. \end{equation} 不妨设 $c=\min \{a,b,c\}$,分两类讨论。 (i)$c<1/9$,则易得 $\sqrt{c}>3c=c(a+b+c)$,故 \[ 5(\sqrt{a}+\sqrt{b}+\sqrt{c})\geqslant 5(\sqrt{a}+\sqrt{b})+5c(a+b+c), \] 所以要证式 \eqref{nguoivn187s2} 只要证 \[ 5(\sqrt{a}+\sqrt{b})\geqslant 4ab+3, \] 两边平方等价于 \[ 25(a+b)+50\sqrt{ab}\geqslant (4ab+3)^2, \] 由 $c<1/9$,可得 $25(a+b)=75-25c>69$,所以只要证 \[ 69+50\sqrt{ab}\geqslant (4ab+3)^2, \] 因式分解为 \[ 2(3-2\sqrt{ab})(4ab\sqrt{ab}+6ab+15\sqrt{ab}+10)\geqslant 0, \] 显然成立,所以此时式 \eqref{nguoivn187s2} 成立; (ii)$c\geqslant 1/9$,则 $a$, $b$, $c\geqslant 1/9$,于是 \begin{align*} \text{式}~\eqref{nguoivn187s2}&\iff 5(\sqrt{a}+\sqrt{b}+\sqrt{c})\geqslant 2(a+b+c)^2-2(a^2+b^2+c^2)+3 \\ &\iff \sum{\left( 2a^2+5\sqrt{a}-7+\frac{13}2(1-a) \right)}\geqslant 0 \\ &\iff \sum{\left( \frac12(\sqrt{a}-1)^2(4a+8\sqrt{a}-1) \right)}\geqslant 0, \end{align*} 显然成立,所以此时式 \eqref{nguoivn187s2} 也成立。 综合(i)(ii),式 \eqref{nguoivn187s2} 获证。 第三个以前证过了,见《数学空间》2011 第 3 期 P29~30,例 3.2.4 : http://www.pep.com.cn/rjwk/gzsxs ... 0110516_1041458.htm PS、时间关系,没检查太多,希望没计算错。躲被窝去了
pxchg1200 9# 2013-1-15 08:32
本帖最后由 pxchg1200 于 2013-1-15 09:42 编辑 Nice! kuing. 第194题 Let $a,b,c$ be three real numbers and $k\in R$,Prove that: \[ \left( \frac{a}{4a-3b-c}\right)^{2}+\left( \frac{b}{4b-3c-a}\right)^2+\left( \frac{c}{4c-3a-b}\right)^2\geq \frac{10}{169}\] The general problem: \[ \sum_{cyc}{\left(\frac{a}{2ka-(k+1)b-(k-1)c}\right)^2}\geq \frac{2(1+k^2)}{(3k^2+1)^2} \] 证明:运用Cauchy-Schwarz 不等式 \[ \left[\sum_{cyc}{(10a-b-9c)^{2}(4a-3b-c)^2}\right]\cdot\left[\sum_{cyc}{\left(\frac{a}{4a-3b-c} \right)^{2}} \right]\geq \left(\sum{a(10a-b-9c)}\right)^{2}=100\left[\sum{a^2}-\sum{ab}\right]^2 \] 然而 \[ \left[\sum_{cyc}{(10a-b-9c)^{2}(4a-3b-c)^2}\right]=1690\left(\sum{a^2}-\sum{ab}\right)^2 \] 不等式显然得证。 The general one: 我们模仿陈计的方式来弄这个: 注意到\[ LHS-RHS=\left[\frac{\sum{(k^2-1)^2a^3+(3k^4+6k^2+8k-1)\sum{a^2b}+(3k^4+6k^2-8k-1)\sum{ab^2}-(21k^4+30k^2-3)abc}}{\prod{(2ka-(k+1)b-(k-1)c)}}\right]^{2}\geq 0 \] Done! ________________________________________________________________ 虽然说那个推广做出来了,但毕竟很暴力,由于不知道等号成立的条件,故难以CS。 下面证明一个类似的不等式,此题最早由越南人Nguyen Dinh Thi出的,后来被tian27546独立发现,并用在讲座中,但他给出的证明是配方。我们接下来用Cauchy-Schwarz来证明下这个精彩的不等式。 Let $a,b,c$ be distinct real numbers.Prove that for any real number $k$, we have: \[ \left(\frac{a-kb}{a-b} \right)^{2}+\left( \frac{b-kc}{b-c}\right)^{2}+\left(\frac{c-ka}{c-a} \right)^2\geq k^2+1 \] 证明:运用Cauchy-Schwarz不等式,我们有 \[ \left[\sum{\frac{(a-kb)^2}{(a-b)^2}} \right][\sum{(a-b)^2(a-kb+(k-1)c)^2}]\geq [\sum{(a-kb)(a-kb+(k-1)c}]^{2} \] 这时,只要证明 \[ [\sum{(a-kb)(a-kb+(k-1)c}]^{2}\geq (k^2+1)\sum{(a-b)^2[a-kb+(k-1)c]^{2}} \] 事实上,这个是个恒等式。 故不等式成立。 到这里,有人肯定要问,那个 $(a-kb+(k-1)c)^2 $怎么配的? 其实,这个是根据等号成立的条件来配的,这一类不等式具有2个不同的取等条件,所以我们可以正向使用待定系数的方式,以对应项成比例的原理来配,比如,我们先待定$ (a-b)^2(a+mb+nc)^2 $ 然后让对应项成比例,由于有2个等号成立条件,我们可以确定系数$m$和$n$。这样,后面的柯西估计就变得相当的精确了,最后甚至会变成恒等式。      对于上面那个不等式,我们可以对$k$赋予不同的值得到一些经典的不等式,如 $k=0$我们得到了 IMO  2008. $k=-1$,我们得到了另外一个熟悉的不等式 \[ \left(\frac{a+b}{a-b} \right)^2+\left(\frac{b+c}{b-c}\right)^2+\left(\frac{c+a}{c-a}\right)^2\geq 2 \] 同样的手段可以用于证明那个general one和下面的不等式,前提是当我们找到那2个取等条件。 \[ \frac{(a-b)^2}{(b-c)^2}+\frac{(b-c)^2}{(c-a)^2}+\frac{(c-a)^2}{(a-b)^2}\geq 5 \] Ps:这里配$ \sum{(3a-2b-c)^2(b-c)^2}$是合适的。 :D 鉴于这些不等式有着公共的内核,我有理由相信,我们甚至可以从 \[ \left(\frac{a-kb}{a-b} \right)^{2}+\left( \frac{b-kc}{b-c}\right)^{2}+\left(\frac{c-ka}{c-a} \right)^2\geq k^2+1 \] 通过适当的线性变换得到其他不等式,如作替换$a=kx+my+nz$等等,具体的情况留给大家去研究。 额,文采不好,希望我表述清楚了。
kuing 10# 2013-1-15 08:49
10# pxchg1200 看来这种技术你已经完全熟练了啊,我再睡一会晚点上来玩。
pxchg1200 11# 2013-1-15 10:08
第200题,(最后一题) 这题很难,证明属于Vo Quoc Ba Can 证明:设$x=|a+b|,y=|c+a|,z=|b+c|$,我们有$x+y+z\geq 6 $ \[ \frac{|a+b|}{4a+b^2+c^2}=\frac{3x}{4a(a+b+c)+3b^2+3c^2}=\frac{3x}{2x^2+2y^2+b^2+c^2}\leq \frac{3x}{2x^2+2y^2+\frac{z^2}{2}}=\frac{6x}{4x^2+4y^2+z^2}\leq \frac{x(x+y+z)}{4x^2+4y^2+z^2} \] 现在,只要证明 \[ \frac{4x(x+y+z)}{4x^2+4y^2+z^2}+\frac{4y(x+y+z)}{4y^2+4z^2+x^2}+\frac{4z(x+y+z)}{4z^2+4x^2+y^2}\leq 4 \] \[ \Leftrightarrow \sum{\frac{4xy-z^2}{4x^2+4y^2+z^2}}+\sum{\frac{(2x+z)^2}{4x^2+4y^2+z^2}}\leq 4 \] 运用Cauchy-Schwarz,我们有 \[ \frac{(2x+z)^2}{4x^2+4y^2+z^2}\leq \frac{(2x)^2}{2(2x^2+y^2)}+\frac{z^2}{2y^2+z^2} \] 马上看到 \[ \sum{\frac{(2x+z)^2}{4x^2+4y^2+z^2}}\leq 3 \] 剩下只要证明 \[ \sum{\frac{4xy-z^2}{4x^2+4y^2+z^2}}\leq 1 \] 由于 \[ \frac{4xy-z^2}{4x^2+4y^2+z^2}\leq \frac{2x^2+2y^2-z^2}{4x^2+4y^2+z^2}=\frac{1}{2}-\frac{3z^2}{2(4x^2+4y^2+z^2)} \] 所以,只要证明 \[ \frac{x^2}{x^2+4y^2+4z^2}+\frac{y^2}{y^2+4z^2+4x^2}+\frac{z^2}{z^2+4x^2+4y^2}\geq \frac{1}{3} \] 再次使用Cauchy-Schwarz,我们得到 \[ \sum{\frac{x^2}{x^2+4y^2+4z^2}}\geq \frac{(\sum{x^2})^2}{\sum{x^2(x^2+4y^2+4z^2)}} \] 且我们有 \[ 3(\sum{x^2})^2-\sum{x^2(x^2+4y^2+4z^2)}=2(\sum{x^4}-\sum{x^2y^2})\geq 0 \] 不等式显然成立。等号成立当且仅当$a=b=c=1$ ______________________________________________________________________________ 以上证明技巧性极强,具备很强的观赏性。Can神将Cauchy-Schwarz使用得出神入化正好验证了Vasile Cirtoaje的一句话The simpler and sharper, the more beautiful! :D
pxchg1200 12# 2013-1-15 10:21
第197题 再上一个Vo Quoc Ba Can的证明,让大家欣赏下他神一样的AM-GM和Cauchy-Schwarz技术。 证明: 设$x=\frac{a^2+b^2}{2},y=\frac{c^2+a^2}{2},z=\frac{b^2+c^2}{2}$ 由$a+b\leq 2\sqrt{x} $ \[ 2a^2+3b^2+3c^2=2(x+y+2z) \] 我们有 \[ \frac{a+b}{2a^2+3b^2+3c^2+8}\leq \frac{\sqrt{x}}{x+y+2z+4} \] 所以,只要证明 \[ \frac{\sqrt{x}}{x+y+2z+4}+\frac{\sqrt{y}}{y+z+2x+4}+\frac{\sqrt{z}}{z+x+2y+4}\leq \frac{3}{8}\] 同时运用AM-GM和Cauchy-Schwarz,我们得到 \[ \sum{\frac{x}{x+y+2z+4}}\leq \frac{1}{4}\sum{\sqrt{\frac{x}{x+y+2z}}}\leq \frac{1}{4}\sqrt{[\sum{x(x+z+2y)}][\sum{\frac{1}{(x+y+2z)(x+z+2y)}}]} =\frac{1}{2}\sqrt{\frac{(\sum{x^2}+3\sum{xy})(\sum{x})}{(x+y+2z)(y+z+2x)(z+x+2y)}}\] 化简后只要证明 \[ 9(x+y+2z)(y+z+2x)(z+x+2y)\geq 16\left(\sum{x^2}+3\sum{xy}\right)(x+y+z) \] 即 \[ (x+y)(x-y)^2+(y+z)(y-z)^2+(z+x)(z-x)^2\geq 0 \] 不等式得证。
pxchg1200 13# 2013-1-15 10:23
kuing继续玩啊。呵呵
kuing 14# 2013-1-15 18:40
看到上面有绝对值,我也玩一下绝对值。 第 181 题。 这里题目最好加一句“$a$, $b$, $c$ 不同时为0”。 首先可以证明 $a$, $b$, $c$ 的符号相同,这是因为已知等式可以写为 $(a+b-c)^2=4ab$ 等等。 再注意到当 $a$, $b$, $c$ 同时变为其相反数时原不等式不变,因此只要证明非负的情形即可。 这样,我们可以令 $a=x^2$, $b=y^2$, $c=z^2$,其中 $x$, $y$, $z\geqslant 0$ 且不同时为 $0$,那么由已知等式有 \begin{align*} 0&=2(ab+bc+ca)-a^2-b^2-c^2\\ &=2(x^2y^2+y^2z^2+z^2x^2)-x^4-y^4-z^4\\ &=(x+y+z)(x+y-z)(y+z-x)(z+x-y), \end{align*} 因此,由对称性,我们可以不妨设 $z=x+y$,于是,原不等式等价于 \[\frac{\abs{x^2-y^2}}{\sqrt{2x^2y^2+(x+y)^4}}+\frac{x(x+2y)}{\sqrt{2y^2(x+y)^2+x^4}}+\frac{y(y+2x)}{\sqrt{2x^2(x+y)^2+y^4}}\geqslant 2,\] 由均值不等式,有 \begin{align*} LHS&\geqslant \frac{\abs{x^2-y^2}}{\sqrt{\frac{xy(x+y)^2}2+(x+y)^4}}+\frac{x(x+2y)}{\sqrt{4y^2(x^2+y^2)+x^4}}+\frac{y(y+2x)}{\sqrt{4x^2(x^2+y^2)+y^4}} \\ & =\frac{\sqrt2\abs{x-y}}{\sqrt{xy+2(x+y)^2}}+\frac{x(x+2y)}{x^2+2y^2}+\frac{y(y+2x)}{y^2+2x^2} \\ & =\frac{\sqrt2\abs{x-y}}{\sqrt{xy+2(x+y)^2}}+\frac{2y(x-y)}{x^2+2y^2}+\frac{2x(y-x)}{y^2+2x^2}+2 \\ & =\frac{\sqrt2\abs{x-y}}{\sqrt{xy+2(x+y)^2}}-\frac{2(x^2-xy+y^2)(x-y)^2}{(x^2+2y^2)(y^2+2x^2)}+2, \end{align*} 所以,只要证明 \[\frac1{xy+2(x+y)^2}\geqslant \frac{2(x^2-xy+y^2)^2(x-y)^2}{(x^2+2y^2)^2(y^2+2x^2)^2},\] 上式去分母作差分解为 \[3xy(2x^6+2y^6+8xy(x-y)(x^3-y^3)+23x^3y^3)\geqslant 0,\] 显然成立,故原不等式得证。
kuing 15# 2013-1-16 15:59
再来绝对值。 第 159 题。 还是那句,题目最好加一句“$a$, $b$, $c$ 互不相同” 由均值不等式及绝对值不等式,有 \[ \left( \sum{\left| \frac{a^2+bc}{b-c} \right|} \right)^2\geqslant 3\sum{\left| \frac{(a^2+bc)(b^2+ca)}{(b-c)(c-a)} \right|}\geqslant 3\left| \sum{\frac{(a^2+bc)(b^2+ca)}{(b-c)(c-a)}} \right|, \] 于是只要证 \begin{equation}\label{nguoivn159jcs} \left| \sum{\frac{(a^2+bc)(b^2+ca)}{(b-c)(c-a)}} \right|\geqslant a^2+b^2+c^2, \end{equation} 我们将式 \eqref{nguoivn159jcs} 左边分子配凑一下 \[ (a^2+bc)(b^2+ca)=ab(3ab-a^2-b^2)+(a-b)^2\sum{ab}+abc\sum{a}, \] 那么 \begin{align} &\sum{\frac{(a^2+bc)(b^2+ca)}{(b-c)(c-a)}} \notag\\ ={}&\sum{\frac{ab(3ab-a^2-b^2)}{(b-c)(c-a)}}+\sum{ab}\sum{\frac{(a-b)^2}{(b-c)(c-a)}}+abc\sum{a}\sum{\frac1{(b-c)(c-a)}},\label{nguoivn159dhj} \end{align} 易见 \begin{align} \sum{\frac1{(b-c)(c-a)}}&=\frac{\sum{(a-b)}}{(a-b)(b-c)(c-a)}=0,\label{nguoivn159hj1}\\ \sum{\frac{(a-b)^2}{(b-c)(c-a)}}&=\frac{\sum{(a-b)^3}}{(a-b)(b-c)(c-a)}=3,\label{nguoivn159hj2} \end{align} 构造二次函数 \[ f(x)=4x^2-3(a+b+c)x+3(ab+bc+ca)-(a^2+b^2+c^2) \] 由于 $a$, $b$, $c$ 互不相同,由拉格朗 ri cha 值公式,$f(x)$ 可以写成 \begin{align*} f(x)&=\frac{(x-b)(x-c)}{(a-b)(a-c)}f(a)+\frac{(x-c)(x-a)}{(b-c)(b-a)}f(b)+\frac{(x-a)(x-b)}{(c-a)(c-b)}f(c) \\ & =\sum{\frac{(3ab-a^2-b^2)(x-a)(x-b)}{(c-a)(c-b)}}, \end{align*} 于是对比常数项,即得 \begin{equation}\label{nguoivn159hj3} \sum{\frac{ab(3ab-a^2-b^2)}{(b-c)(c-a)}}=a^2+b^2+c^2-3(ab+bc+ca), \end{equation} 将式 \eqref{nguoivn159hj1}, \eqref{nguoivn159hj2}, \eqref{nguoivn159hj3} 代入式 \eqref{nguoivn159dhj} 中,便得到 \[ \sum{\frac{(a^2+bc)(b^2+ca)}{(b-c)(c-a)}}=a^2+b^2+c^2, \] 可见,式 \eqref{nguoivn159jcs} 其实是恒等式!所以原不等式得证。
yes94 16# 2013-1-16 18:56
伤不起啊!
pxchg1200 17# 2013-1-16 20:58
15# kuing 181题 证明:注意到若$a=b=c=0$不等式显然不成立。 所以$ (a-b)^2+(b-c)^2>0 $, $(b-c)^2+(c-a)^2>0 $, $(c-a)^2+(a-b)^2>0$ 而 $ a^2+2bc=(a-b)^2+(a-c)^2 $ 所以,我们可以设 $ x=(b-c)^2, y=(a-c)^2, z=(a-b)^2 $ 这样,不等式变为 \[ \sqrt{\frac{x}{y+z}}+\sqrt{\frac{y}{x+z}}+\sqrt{\frac{z}{x+y}}\geq 2 \] 接下来不用我多说大家都会证明了。:D
kuing 18# 2013-1-16 21:13
18# pxchg1200 原来是这样变出来的。 我一开始的方向就跟你的不一样了,为了消去条件升高了次数,所以后面难免较繁
kuing 19# 2013-1-16 23:12
奇怪,为什么你那里明明是17#我回复你的时候变成了回复18#我自己
kuing 20# 2013-2-1 16:14
再顶一下
thread-1049-2-1.html:
pxchg1200 21# 2013-5-15 21:56
顶一下
pxchg1200 22# 2013-5-15 22:02
顺便做个简单的 第96 题 注意到 $$ \frac{a^3b^2}{a+b}=\frac{a^2b^2(a+b)-a^2b^3}{a+b}$$ 于是,不等式变成 \[ \sum_{cyc}{\frac{a^5+a^2b^3}{a+b}}\geq \sum{a^2b^2} \] 而 $$ \frac{a^5+a^2b^3}{a+b}=\frac{a^2(a^3+b^3)}{a+b}=a^2(a^2-ab+b^2) $$ 故不等式变成 \[ a^4+b^4+c^4\geq a^3b+b^3c+c^3a \] 显然。
yes94 23# 2013-5-15 22:05
23# pxchg1200 这些变形技巧太牛啦!
pxchg1200 24# 2013-5-15 22:09
再玩一个 证明 由 AM-GM, \[ \frac{a^{11}}{bc}+abc\geq 2a^6 \] 这样, \[ \sum{ \frac{a^{11}}{bc}}\geq 2\sum{a^6}-3abc \] 故只要证明 \[ 3\sum{a^6}+\frac{6}{a^2b^2c^2}\geq 9+6abc \] 而由AM-GM则有 \begin{equation}   2\sum{a^6}+\frac{6}{a^2b^2c^2}\geq 6a^2b^2c^2+\frac{6}{a^2b^2c^2}\geq 12 \end{equation} 同时又有 \begin{equation}   \sum{a^6}+3\geq 6abc \end{equation} 把上面2个相加,结论显然。
kuing 25# 2013-5-15 22:11
25# pxchg1200 竟然有这么弱的……形式粹纯吓人啊
pxchg1200 26# 2013-5-15 22:12
25# pxchg1200 再水一个 证明:不等式等价于 \[ (5a+b)(5b+c)(5c+a)\leq 27(a+b)(b+c)(c+a) \] 展开AM-GM显然。
pxchg1200 27# 2013-5-15 22:20
26# kuing 呵呵,有的很弱的 继续水 证明 不等式等价于 \[ 74\sum{a^4bc}-34abc\sum{a^2(b+c)}-78a^2b^2c^2+37\sum{a^4(b^2+c^2)}-54\sum{a^3b^3}\geq 0 \] 由米尔黑德定理 \[  68\sum{a^4bc}\geq 34abc\sum{a^2(b+c)} \] \[  27\sum{a^4(b^2+c^2)}\geq 54\sum{a^3b^3}\] 剩下的就AM-GM咯!
thread-105-1-9.html: [几何] 2011联赛A卷二试1平几
kuing 1# 2011-10-16 22:12
下面为方便书写,记 $AB=x,BC=y,CD=z,DA=w,AC=t,BD=u$,再记对角线 $AC$ 与 $BD$ 交于 $K$。我们找一个那两个角相等的等价条件。 (1)当 $K$ 与 $P$ 不重合时,由角分线定理及面积公式等,易得如下等价关系 \[\angle BPA=\angle DPA\iff \frac{PB}{PD}=\frac{KB}{KD}\iff \frac{PB}{PD}=\frac{S_{\triangle BAC}}{S_{\triangle DAC}}\iff \frac{PB}{PD}=\frac{BA\cdot BC}{DA\cdot DC},\] 对最后一式两边平方,再用中线长公式,等价于 \[\frac{2x^{2}+2y^{2}-t^{2}}{2w^{2}+2z^{2}-t^{2}}=\frac{x^{2}y^{2}}{z^{2}w^{2}},\] 再利用圆内接四边形对角线长公式 \begin{align} t^{2}&=\frac{(wx+yz)(xz+yw)}{xy+zw}, \\ u^{2}&=\frac{(xy+zw)(xz+yw)}{wx+yz}, \end{align} 代入后去分母因式分解整理,等价于 \[(wy-xz)(wx-yz)=0,\] 如果 $wx-yz=0$,那么将有 $S_{\triangle ABD}=S_{\triangle CBD}$,则必有 $AK=CK$,于是 $K$ 就与 $P$ 重合,与原设不符,所以此时最终等价式为 \[wy=xz;\] (2)当 $K$ 与 $P$ 重合时,即 $AC$、$BD$ 交于 $AC$ 的中点 $P$,又 \[\angle BPA=\angle DPA\iff \angle BKA=\angle DKA\iff AC\perp BD,\] 即 $BD$ 垂直平分 $AC$,即此时的等价式为 \[x=y~且~z=w.\] 同理,当 $K$ 与 $Q$ 不重合时,待证的 $\angle AQB=\angle CQB$ 也等价于 \[wy=xz,\] 当 $K$ 与 $Q$ 重合时,待证的 $\angle AQB=\angle CQB$ 等价于 \[w=x~且~y=z.\] 这样,若 $K$ 与 $P$、$Q$ 都不重合时,两等价式相同,命题显然成立;若 $K$ 与 $P$ 重合但不与 $Q$ 重合,那么有 $x=y$ 且 $z=w$,也满足 $\angle AQB=\angle CQB$ 的等价式 $wy-xz=0$,命题成立;若 $K$ 与 $Q$ 重合但不与 $P$ 重合,那么有 $wy-xz=0$,但由 $K$ 与 $Q$ 重合又有 $S_{\triangle BAC}=S_{\triangle DAC}\iff xy=zw$,由此得 $w=x$ 且 $y=z$,命题成立;当 $K$、$P$、$Q$ 都重合时,$AC$、$BD$ 必为两互相垂直的直径,命题也成立。 综上所述,原命题成立。
kuing 2# 2011-10-16 22:43
补上了 $K$ 与 $P$ 或 $Q$ 重合的特殊情形,这下应该够严格了吧?
pxchg1200 3# 2011-10-16 23:06
哇,kuing 还擅长平面几何啊! 厉害! 不过我想如果这个题贴到mathlink的话会被秒了吧。。
kuing 4# 2011-10-16 23:20
3# pxchg1200 呃。。我水皮……
isea 5# 2011-10-17 10:50
有没有纯几何法呢,两中点,很优美的题
isea 6# 2011-10-17 18:00
6# kuing 原来是高中竞赛呀,看了一下,整体使用的相似,不过,那个因为中点,就有弧相等,一眼还没看出来,晕,有空再研究
kuing 7# 2011-10-17 19:17
7# isea 是的,昨天的联赛。 弧相等那里是因为除了中点之外还有角相等,关于AC的中垂线对称了
thread-1050-1-5.html: [函数] 在人教论坛未有具体过程一个非周期函数的证明
isea 1# 2013-1-14 22:33
本帖最后由 isea 于 2013-1-14 23:45 编辑 原帖:http://bbs.pep.com.cn/forum.php?mod=viewthread&tid=2646542 原帖里,其实5楼的链接已经给出了一般情况,及严格的证明。 这里,来看看8楼$y=\sin x+\sin {\pi x}$这个具体的例子,用已有的三角函数知识如何来证明不是周期函数。 题目:求证:函数$y=\sin x+\sin {\pi x}$不是周期函数。 证明:反证法,假设此函数是周期函数,设$T$为其周期,则 $\sin (x+T)+\sin {\pi (x+T)}=\sin x+\sin {\pi x}…………(*)\\ \sin (x+T)-\sin x=\sin {\pi x}-\sin {\pi (x+T)}\\ 2\cos \dfrac{x+T+x}{2}\sin {\dfrac{x+T-x}{2}}=2\cos \dfrac{\pi x+\pi (x+T)}{2}\sin \dfrac{\pi x-\pi (x+T)}{2}…………(1)\\ $ 这里用到了差化积公式,其实就是正弦和与差公式的变式,即 $\sin (x+y)=\sin x\cos y+\cos x\sin y \\ \sin (x-y)=\sin x\cos y-\cos x\sin y$ 两式相减, $\sin (x+y)-\sin (x-y)=2\cos x\sin y$ 再代换成上式中(目标)角即可。 回到正题,将(1)继续化简一下 $\cos ({x}+\dfrac T2)\sin \dfrac{T}{2}=\cos (\pi x+\dfrac{\pi T}{2})\sin (-\dfrac{\pi T}{2})$ 令${x}+\dfrac T2=\dfrac{\pi}{2}$,则有 $0=\cos (\pi (\dfrac{\pi}{2}-\dfrac T2)+\dfrac{\pi T}{2})\sin (-\dfrac{\pi T}{2})\\ -\cos (\dfrac{\pi^2}{2})\sin (\dfrac{\pi T}{2})=0\\ \sin (\dfrac{\pi T}{2})=0\\ \dfrac{\pi T}{2}=k\pi\\ T=2k\\$ 代回(*)式,则 $\sin (x+2k)+\sin {\pi (x+2k)}=\sin x+\sin {\pi x}\\ \sin (x+2k)=\sin x\\$ 令$x=0$,则有 $\sin 2k=0\\ 2k=m\pi $ 亦是说 $\pi=\dfrac {2k}{m}$ 而这里$k$与$m$均为整数,从而矛盾……
kuing 2# 2013-1-14 22:39
看来你也是五笔?
isea 3# 2013-1-14 23:21
本帖最后由 isea 于 2013-1-14 23:50 编辑 是啊,把 周期 打成 同期 了。 这个错的有得“远”,被五笔同仁一眼看穿。 另外,分式等等其他式子混排,纯代码看着还是吃力……
isea 4# 2013-1-14 23:42
将主楼中的$\pi$换成$e$,最后的结果是$e=\dfrac {2k}{m}$…… 目测以为会多个$\pi$,就难办了,其实不然
thread-1051-1-5.html: [数列] 请教一个数列求通项的题,先谢谢了!
hongxian 1# 2013-1-15 10:45
本帖最后由 hongxian 于 2013-1-15 10:51 编辑 已知$\{a_n\}$满足:$a_n=2a_{n-1}^{2}-1$($n \in N^*$) (1)若$a_0=\frac{1}{3}$,求$\{a_n\}$的通项公式; (2)若$a_0=3$,求$\{a_n\}$的通项公式。
pxchg1200 2# 2013-1-15 10:51
本帖最后由 pxchg1200 于 2013-1-15 10:56 编辑 1# hongxian 不知道这样行不行。 设 $a_{0}=\frac{e^{-x}+e^{x}}{2}$ \[ a_{1}=2\left(\frac{e^{x}+e^{-x}}{2}\right)^{2}-1=\frac{e^{2x}+e^{-2x}}{2} \] ........ \[ a_{n}=\frac{e^{(n+1)x}+e^{-(n+1)x}}{2} \] 然后根据$a_{0}$的值把$x$解出来。
hongxian 3# 2013-1-15 11:02
2# pxchg1200 第二问可以,第一问好象不行!
pxchg1200 4# 2013-1-15 11:07
3# hongxian 怎么不行了,如果说方程无解了,那么请不要局限于实数范围,尝试在复数范围去解,再用Euler公式 \[ e^{ix}=\cos{x}+i\sin{x} \]
hongxian 5# 2013-1-15 11:23
4# pxchg1200 的确局限在实数范围内了!高手!
kuing 6# 2013-1-15 11:26
令 $a_n=\dfrac{b_n}2$ 转化为 http://www.pep.com.cn/rjwk/gzsxs ... 0110402_1031466.htm 例3.1.2 其实三角和双曲本身就是相通的,在复数范围看。
yes94 7# 2013-1-15 19:51
令 $a_n=\dfrac{b_n}2$ 转化为 http://www.pep.com.cn/rjwk/gzsxs ... 0110402_1031466.htm 例3.1.2 其实三角和双曲本身就是相通的,在复数范围看。 kuing 发表于 2013-1-15 11:26 双曲函数的二倍角公式和三角函数的也类似
thread-1052-1-2.html: [不等式] 昨晚人教群里提起的一道正整数不等式(实为早前安振平的某征解题)
kuing 1# 2013-1-15 11:17
当年好像一开始没想出来,后来不知为什么就丢下没理了。昨晚人教群里被再次提起,刚才想了一下其实还蛮简单?看来当年应该是心情欠佳了…… 令 \[f(a_1,a_2,\ldots,a_n)=a_1^3+a_2^3+\cdots+a_n^3-\frac{n(n+1)}2(a_1+a_2+\cdots+a_n),\] 其中各 $a_i$ 为互不相等的正整数。 首先有必要略为说明一下 $f(a_1,a_2,\ldots,a_n)$ 存在最小值,虽然比较显然,不过还是提一下比较好。 对于每个确定的 $n$,当任一 $a_i$ 充分大时,由于 $a_i^3$ 比 $a_i$ 高阶,所以 $f$ 也会充分大,而当所有 $a_i$ 都不充分大时,$f$ 取有限个值,其中必有最小者,所以最小值存在。 下面证明 $f(a_1,a_2,\ldots,a_n)$ 取最小值时必然是 $\{a_1,a_2,\ldots,a_n\}=\{1,2,\ldots,n\}$ 时。 若不然,则必存在某个 $a_k$ 使得 $a_k\geqslant n+1$,并且存在 $m$($1\leqslant m\leqslant n$)使得所有的 $a_i$ 都不等于 $m$,于是 \begin{align*} f(\ldots,a_k,\ldots)-f(\ldots,m,\ldots)&=a_k^3-m^3-\frac{n(n+1)}2(a_k-m)\\ &=(a_k-m)\left(a_k^2+a_km+m^2-\frac{n(n+1)}2\right)\\ &\geqslant (n+1-m)\left((n+1)^2+(n+1)m+m^2-\frac{n(n+1)}2\right)\\ &=(n+1-m)\left(\frac12(n+1)(n+2)+(n+1)m+m^2\right)\\ &> 0, \end{align*} 可见将 $a_k$ 变成 $m$ 后 $f$ 更小,所以仅当 $\{a_1,a_2,\ldots,a_n\}=\{1,2,\ldots,n\}$ 时 $f$ 取最小值,易见为 $0$,所以原不等式得证。
pxchg1200 2# 2013-1-15 11:48
1# kuing 有米有一招秒的办法?
kuing 3# 2013-1-15 11:53
呃,其实1#的几乎算是一招秒吧,只不过我为了严格起见所以写得有点哆嗦。呵呵。 你也来想想吧,连续变量的玩多了,来玩玩离散
yes94 4# 2013-1-15 19:38
本题可以加强为:                 猜测他是在某书上看到了这个不等式才给出的加强,实际上他的什么“23个优美不等式”,被很多人几下子就证明出来了。
yes94 5# 2013-1-15 19:42
本帖最后由 yes94 于 2013-1-15 19:45 编辑 本题可以加强为:                 741 猜测他是在某书上看到了这个不等式才给出的加强,实际上他的什么“23个优美不等式”,被很多人几下子就证明出来了。 yes94 发表于 2013-1-15 19:38 取等号的时候,就是一道常见的数列题: 已知正项数列{$a_n$}满足: ,求数列{$a_n$}的通项公式。    答案:$a_n=n$
kuing 6# 2013-1-21 20:31
嗯,1#的证法仍然可用。 记 \[g(a_1,a_2,\ldots,a_n)=a_1^3+a_2^3+\cdots+a_n^3-(a_1+a_2+\cdots+a_n)^2,\] $g$ 存在最小值就不哆嗦了,假设取最小值时 $\{a_1,a_2,\ldots,a_n\}\ne\{1,2,\ldots,n\}$,不妨设 $a_n=\max\{a_1,a_2,\ldots,a_n\}$,则必有 $a_n\geqslant n+1$,并且存在 $m$($1\leqslant m\leqslant n$)使得所有的 $a_i$ 都不等于 $m$,注意到 \begin{align*} a_1+a_2+\cdots +a_{n-1}&\leqslant a_n-1+a_n-2+\cdots +a_n-(n-1) \\ & =(n-1)a_n-\frac{n(n-1)}2, \end{align*} 于是 \begin{align*} g(\ldots ,a_n)-g(\ldots ,m)&=a_n^3-m^3-(a_n-m)(2a_1+2a_2+\cdots +2a_{n-1}+a_n+m) \\ & =(a_n-m)\bigl(a_n^2+(a_n+m)(m-1)-2(a_1+a_2+\cdots +a_{n-1})\bigr) \\ & \geqslant (a_n-m)\bigl(a_n^2+(a_n+m)(m-1)-2(n-1)a_n+n(n-1)\bigr) \\ & =(a_n-m)\bigl((a_n-n)(a_n-n+2)+n+(a_n+m)(m-1)\bigr) \\ & >0, \end{align*} 可见将 $a_n$ 变成 $m$ 后 $g$ 更小,所以仅当 $\{a_1,a_2,\ldots,a_n\}=\{1,2,\ldots,n\}$ 时 $g$ 取最小值,易见为 $0$,所以原不等式得证。 PS1、安振平较早期的那些征解题我大多看过,也秒过不少,后期就没怎么关注了,是偏易了点,比较适合大众。 PS2、用附件发图,引用的时候就会出现5#的情形……打代码就不会了(代码又一好处)。
realnumber 7# 2013-1-22 10:37
6# kuing "是偏易了点,比较适合大众" 怀疑是不是故意这样,所谓"曲高和寡",人多才热闹.
kuing 8# 2013-1-22 12:12
7# realnumber 那要问一下他本人才知道了……
Gauss门徒 9# 2013-4-22 15:47
直接数归嘛
kuing 10# 2013-4-22 16:14
9# Gauss门徒 没归出来……写写看……
yes94 11# 2013-4-22 23:09
10# kuing 还有那个加强呢?
Gauss门徒 12# 2013-4-23 00:59
本帖最后由 Gauss门徒 于 2013-4-24 14:06 编辑 10# kuing Let$(b_1,b_2,...b_{n})$ be a permutation of $(1,2,...,n)$ We assume that \[\sum_{i=1}^{k}(b_{i}+x_{i})^3+\sum_{j=k+1}^{n}b_{j}^3\geqslant \left(\sum_{1}^{n}n+\sum_{m=1}^{n} b_{m}\right)^2\] And then we apply induction for $k$.
yes94 13# 2013-4-23 12:43
12# Gauss门徒 好简略!还是英文,看不懂啊
kuing 14# 2013-4-24 14:01
Let$(b_1,b_2,...b_{n})$ be a permutation of $(1,2,...,n)$ We assume that \[\sum_{i=1}^{k}(b_{i}+x_{i})^3+\sum_{j=k+1}^{n}b_{j+1}^3\geqslant \left(\sum_{1}^{n}n+\sum_{m=1}^{n} b_{m}\right)^2\] And then we apply induction for $k$. Gauss门徒 发表于 2013-4-23 00:59 $x_i$ 是神马?第二项的下标为什么是 $j+1$ 而不是 $j$?
Gauss门徒 15# 2013-4-24 14:07
10# kuing 我修改好了,你再看看
yes94 16# 2013-4-24 14:18
15# Gauss门徒 搞得完整点嘛!要不然又要动笔想,好麻烦啊
kuing 17# 2013-4-24 18:31
一点都看不懂仍然……
yes94 18# 2013-4-25 20:56
什么时候我写一下那本书上的过程,不过看上去也够烦的
kuing 19# 2013-4-25 21:49
18# yes94 什么书?
yes94 20# 2013-4-26 00:08
19# kuing 叶军
thread-1052-2-2.html:
kuing 21# 2013-4-26 01:22
20# yes94 数学奥林匹克教程?
yes94 22# 2013-4-26 13:28
21# kuing 嗯,
kuing 23# 2013-4-26 14:22
22# yes94 哪页,我去瞧瞧
yes94 24# 2013-4-26 18:34
23# kuing 264,831
kuing 25# 2013-4-26 20:31
看到了,打错了个符号,牛笔的证法。
yes94 26# 2013-4-26 22:00
25# kuing 你的调整做法也很牛笔
Gauss门徒 27# 2013-4-28 15:59
本帖最后由 Gauss门徒 于 2013-5-1 16:29 编辑 16# yes94 其实我是证明那个加强,考虑恒等式$1^3+2^3+..+n^3=(1+2+...+n)^2  $然后在其中若干项添数,比如$(1+k1)^3+2^3+(3+k2)^3...\ge (1+2+3+k1+k2...)^2$然后对ki的个数归纳,貌似是正确的
Gauss门徒 28# 2013-4-29 17:18
23# kuing 喂,怎么没人回复啊!
yes94 29# 2013-4-29 17:22
28# Gauss门徒 看不懂啊
李斌斌755 30# 2013-4-30 02:59
29# yes94 门徒的贴 1 英语 2太难
yes94 31# 2013-4-30 13:56
29# yes94 门徒的贴 1 英语 2太难 李斌斌755 发表于 2013-4-30 02:59 3.简略(也是最重要的一条)
Gauss门徒 32# 2013-5-1 16:30
29# yes94 我又修改了,爪机latex不方便
yes94 33# 2013-5-1 19:43
32# Gauss门徒 意思明白了。 只是你没有过程证明你的想法,而我又没有笔。
Gauss门徒 34# 2013-5-2 16:09
33# yes94 我是写在纸上而且没错,只是让你们再检验一次
thread-1053-1-5.html: [不等式] 昨晚人教群里提起的另一道有顺序根式不等式
kuing 1# 2013-1-15 12:35
已知 $0<a\leqslant b\leqslant c$,求证 \[\sqrt{\frac{a}{a+2b}}+\sqrt{\frac{b}{b+2c}}+\sqrt{\frac{c}{c+2a}}\leqslant \sqrt3.\] 据说也是安振平的,不过我没什么印象,可能后期我没关注了。 由柯西不等式有 \begin{align*} \sum{\sqrt{\frac{a}{a+2b}}}&\leqslant \sqrt{\sum{(c+2a)}\cdot \sum{\frac{a}{(a+2b)(c+2a)}}} \\ & =\sqrt{3(a+b+c)\cdot \frac{\sum{a(b+2c)}}{(a+2b)(b+2c)(c+2a)}} \\ & =\sqrt{\frac{9(a+b+c)(ab+bc+ca)}{(a+2b)(b+2c)(c+2a)}}, \end{align*} 所以只要证 \[ 3(a+b+c)(ab+bc+ca)\leqslant (a+2b)(b+2c)(c+2a), \] 展开分解即为 \[ (a-b)(b-c)(c-a)\geqslant 0, \] 显然成立,故得证。
ccnu_chb_ycb 2# 2013-1-15 19:21
这个解答很妙!!!
thread-1054-1-5.html: [组合] 再来一道排列组合题
hflz01 1# 2013-1-15 21:55
如题
realnumber 2# 2013-1-15 22:27
本帖最后由 realnumber 于 2013-1-15 22:44 编辑 不知道我有没误会题目的意思, 先考虑白球放法,出现各箱子球的数目$3,0,0;2,1,0;1,1,1$,分别有$3+6+1=10$种. 红$4,0,0;3,1,0;2,2,0;2,1,1$,分别有$3+6+3+3=15$种. 黄$5,0,0;4,1,0;3,2,0;3,1,1;2,2,1$,分别有$3+6+6+3+3=21$种. 那么不同放法有$10\times15\times21$种. 如果是5个不同黄球,那么可以用逐步淘汰原则来做,每个盒子都要出现黄球$3^5-C_3^22^5+C_3^11^5$,黄球可以放一起$3^5.$
hflz01 3# 2013-1-16 09:43
楼上威武。3150是对的。见图,是参考解答,怎么考虑的呢?
hflz01 4# 2013-1-16 09:43
附件错了,重传一下。
realnumber 5# 2013-1-16 10:36
隔板法呢,就是###@@,对应着球数目是3,0,0, #@##@,对应1,2,0 所以就$C_5^2$
realnumber 6# 2013-1-16 12:26
本帖最后由 realnumber 于 2013-1-16 14:34 编辑 那么这样的问题其实也解决了 $1.n$个不同的球,放入$m$个不同箱子,允许出现空箱.$m^n$ $2.n$个不同的球,放入$m$个不同箱子,不允许出现空箱($n\ge{m}$).$m^n-C_m^{m-1}(m-1)^n+C_m^{m-2}(m-2)^n-...$ $3.n$个一样的球,放入$m$个不同箱子,允许出现空箱.$C_{n+m-1}^{m-1}$ $4.n$个一样的球,放入$m$个不同箱子,不允许出现空箱($n\ge{m}$).$C_{n-1}^{m-1}$ $5.n$个一样的球,放入$m$个一样箱子,允许出现空箱.? 等价于$x_1+x_2+...+x_m=n,(n\ge x_1\ge x_2 ...\ge x_m \ge 0)$有几组整数解。(好象可用递推数列或幂级数?) 就是整数分拆<组合数学>.pdf曹汝成 $6.n$个一样的球,放入$m$个一样箱子,不允许出现空箱($n\ge{m}$).即为$n-m$个一样的球,放入$m$个一样箱子,允许出现空箱.?
kuing 7# 2013-1-16 12:28
4# hflz01 一行小公式都要贴图
thread-1055-1-5.html: [数论] $2^k \equiv 1 \pmod{2013} $
realnumber 1# 2013-1-16 09:23
本帖最后由 realnumber 于 2013-1-16 15:01 编辑 找出最小正整数k,使得$2^k≡1\pmod{2013}$. $2013=3\times11\times61$
hnsredfox_007 2# 2013-1-16 10:43

realnumber 3# 2013-1-16 12:04
本帖最后由 realnumber 于 2013-1-16 12:15 编辑 最小需要怎么说明? $(2^{30}-1)(2^{30}+1)=1 \mod61$,总不能单独检验60以下都不行. $2^2=1\mod3$,$2^10=1\mod11$,分别穷举得到2,10最小,说明所求k是10 的倍数,但k=10,20,30,40,50为什么不行怎么说明? ps,数论其实也是弱项,可能在说外行话.
hnsredfox_007 4# 2013-1-16 13:39
3# realnumber
realnumber 5# 2013-1-16 14:00
恩,明白了,这样穷举并不吃力.还以为另有特定的方法.
hnsredfox_007 6# 2013-1-16 14:11
5# realnumber 或许有也不一定,我对这也不是很了解,知道一点点点点点点点皮毛
kuing 7# 2013-1-16 14:39
我也不了解,纯路过…… 顺便扯扯同余的输入。 可以写成  2^k \equiv 1 \mod 2013  ,效果:$2^k \equiv 1 \mod 2013$ 如果习惯用括号,也可以写成  2^k \equiv 1 \pmod{2013}  ,效果 $2^k \equiv 1 \pmod{2013}$ 。 括号就会自动添加,距离也会调节好。 但是 \pmod 后面的花括号不能少,除非是单个字符,否则括号会括乱,比如 \pmod{2013} 和 \pmod2013 分别显示 $\pmod{2013}$,$\pmod2013$
thread-1056-1-5.html: [不等式] 回复352的关于外森比克不等式的构图证明
kuing 1# 2013-1-16 14:32
三下五除二 23:46:41 这个是一本书里的,靠下面那个图形构造的,怎么比较三分之一和s的大小,我想半天没搞出来 不知是不是我想复杂了,照我看应该还涉及了费马点。 首先有这样一个的简单引理: 如左图,$\triangle ABC$ 为正三角形,$D$ 在绿色部分区域上(含边界),则 $S_{\triangle ABC}\geqslant3S_{\triangle BCD}$。 其证明见右图。 回到原题,作出如书上所说的图形后,再分两类讨论。 (1)如果 $\triangle ABC$ 的三个内角都小于 $120^\circ$,则其费马点 $F$ 在其内部,连结 $FA$,$FB$,$FC$: 则 $A$、$R$、$B$、$F$ 四点共圆;$B$、$P$、$C$、$F$ 四点共圆;$C$、$Q$、$A$、$F$ 四点共圆。 因此由引理知 $S_{\triangle ABR}\geqslant3S_{\triangle ABF}$;$S_{\triangle BCP}\geqslant3S_{\triangle BCF}$;$S_{\triangle CAQ}\geqslant3S_{\triangle CAF}$,三式相加即得; (2)如果 $\triangle ABC$ 某个内角大于或等于 $120^\circ$,则直接由引理可得。
thread-1057-1-5.html: [不等式] 粉丝群提到的一道三元轮换分式根式不等式
kuing 1# 2013-1-17 21:44
这里证明稍一般的情况,令 \[ f(t)=\sum{\sqrt{\frac{a^2+t}{b^2+t}}}, \] 我们将证明 $f(t)$ 单调递减。求导得 \[ f'(t)=\sum{\frac{b^2-a^2}{2(b^2+t)\sqrt{(a^2+t)(b^2+t)}}}, \] 于是 \begin{align*} f'(t)\leqslant 0&\iff\sum{\frac{\bigl(a^2+t-(b^2+t)\bigr)\sqrt{c^2+t}}{(b^2+t)}}\geqslant 0 \\ & \iff\sum{\frac{(a^2+t)\sqrt{c^2+t}}{(b^2+t)}}\geqslant \sum{\sqrt{c^2+t}}, \end{align*} 令 $\sqrt{a^2+t}=x$, $\sqrt{b^2+t}=y$, $\sqrt{c^2+t}=z$,则等价于证 \[ \sum{\frac{x^2z}{y^2}}\geqslant \sum x, \] 配凑系数,上式等价于 \[ \sum{\left( \frac6{13}\cdot \frac{x^2z}{y^2}+\frac5{13}\cdot \frac{y^2x}{z^2}+\frac2{13}\cdot \frac{z^2y}{x^2} \right)}\geqslant \sum x, \] 由加权均值即得 \begin{align*} \frac6{13}\cdot \frac{x^2z}{y^2}+\frac5{13}\cdot \frac{y^2x}{z^2}+\frac2{13}\cdot \frac{z^2y}{x^2}&\geqslant x^{2\times 6/13+5/13-2\times 2/13}y^{-2\times 6/13+2\times 5/13+2/13}z^{6/13-2\times 5/13+2\times 2/13}\\ &=x, \end{align*} 因此单调性得证,从而由 $f(0)\geqslant f(1)$ 即得原不等式。
thread-1058-1-5.html: 答案不一致的原因?
realnumber 1# 2013-1-18 09:38
本帖最后由 realnumber 于 2013-1-18 11:35 编辑
realnumber 2# 2013-1-18 11:35
本帖最后由 realnumber 于 2013-1-18 11:39 编辑 由$a_3^2+a_3a_4+1=0$,得到\[-d=2a_3+\frac{1}{a_3}\]由此可得$d\ge{2\sqrt{2}}$或$d\le{-2\sqrt{2}}$. 给定$d$和$a_3$,总可以得到一个$a_1$.
realnumber 3# 2013-1-18 12:08
不等式的几个基本性质,有些是双向的,可以用来解取值问题,有些是单向的,只用来证明。 原解法用后者求取值范围,有问题. 还有应该是有位老师说的a3,a4有关联, 次序也有些混乱,......
hnsredfox_007 4# 2013-1-18 13:58
$a_3$与$a_4$不是相互独立的,也就是$a_3$与$d$不是相互独立的,因此由最后“两个”“由”得$d$的范围,实际上扩大了范围,不是充要条件了
yes94 5# 2013-1-18 17:56
看似正确的推理,却得到错误的答案。 这话的反应的现实情况可以这样看待: 1、所得到范围是正确答案的必要而非充分条件。 2、并不是说这个范围真的就错了,而是这个范围不够精确而已。    我们通常所说的范围应该改成值域,或改成与值域具有相同意思的词汇。 3、这个范围还可以缩小,因为如果继续挖掘寻找隐含条件,将会把该范围进一步缩小成为正确答案。    当然,一开始就每步用充要条件解题,那么所得答案就必定是正确答案。可惜,每步都用充要条件,在有些时候是个美好的愿望而已。 4、正确的推理得到肯定不是错误答案,只是本题正确的推理还没有完成,还需要将正确的推理继续进行下去,直到所得答案是充要条件为止(只需证明该条件是充分的即可)。    以上就是学生常常因为自己的推理没错,答案却错了,而百思不得其解的原因。 一家之言,欢迎讨论
kuing 6# 2013-1-18 19:42
看似正确的推理,却得到错误的答案。 这话的反应的现实情况可以这样看待: 1、所得到范围是正确答案的必要而非充分条件。 2、并不是说这个范围真的就错了,而是这个范围不够精确而已。    我们通常所说的范围应该改成值域,或改成与值域具有相同意思的词汇。 3、这个范围还可以缩小,因为如果继续挖掘寻找隐含条件,将会把该范围进一步缩小成为正确答案。    当然,一开始就每步用充要条件解题,那么所得答案就必定是正确答案。可惜,每步都用充要条件,在有些时候是个美好的愿望而已。 4、正确的推理得到肯定不是错误答案,只是本题正确的推理还没有完成,还需要将正确的推理继续进行下去,直到所得答案是充要条件为止(只需证明该条件是充分的即可)。    以上就是学生常常因为自己的推理没错,答案却错了,而百思不得其解的原因。 一家之言,欢迎讨论 yes94 发表于 2013-1-18 17:56 的确是这样。 有时不止是学生,甚至是老师都很少考虑这些。 太少强调充要性、等价、同解变形这些东东了,像这种求取值范围的题解很少看到有验证充分性的习惯,往往不加思索地由不等式断言取值范围。
kuing 7# 2013-1-21 17:05
不过话说回来,如果每道取值范围的题都要说明充分性的话,可能会显得比较麻烦和哆嗦。 比如推理得到 $x\geqslant A$ 之后,要想说明 $x$ 的取值范围是 $[A,+\infty)$,说明 $x$ 能取 $A$ 够不够?不够,再说明 $x$ 能够无穷大够不够?仍然不够,还得说明是连续变化的,那就够了。
kuing 8# 2013-1-21 17:11
这个问题最近似乎成了热门? 352的博客中(http://blog.sina.com.cn/s/blog_68ef132301018kom.html)的标题写着“也谈问题214”,那就是他已经看了很多个人谈它了? 其实我觉得这跟线性划规问题那种直接用不等式加减计算到的范围会比画图得到的范围要大是差不多的道理,没想到成了热门(或许不是)
yes94 9# 2013-1-22 13:34
这个问题最近似乎成了热门? 352的博客中(http://blog.sina.com.cn/s/blog_68ef132301018kom.html)的标题写着“也谈问题214”,那就是他已经看了很多个人谈它了? 其实我觉得这跟线性划规问题那种直接用不等式加 ... kuing 发表于 2013-1-21 17:11 楼主是352?
kuing 10# 2013-1-22 13:40
9# yes94 嗯,你没看到那个博客的名字叫“maths352”么……
isea 11# 2013-1-22 14:11
本帖最后由 isea 于 2013-1-22 19:03 编辑 原来是数列问题啊,mark,先 有空时来看看 ============== 汗,大家已经弄得很明了……
thread-1059-1-1.html: 下载了,安装了,不知道怎么打开软件
o54ljh 1# 2013-1-18 09:38
下载了个CTEX套装,好像也安装了,但是不知道怎么打开。$\frac{n^{2}}{2}>\frac{n}{a}$$\frac{n^{2}}{2}>\frac{n}{a}$这是个什么效果?
kuing 2# 2013-1-18 12:53
打开 winedt 写代码,写好后编译。 PS、“o54ljh”这个ID好像在人教论坛见过,早前。
hnsredfox_007 3# 2013-1-18 14:01
能否将过程详细说一遍 打开winedt写好代码,然后另存?怎么编译?打开什么编译?怎么存成pdf格式? 我也一点儿不会
kuing 4# 2013-1-18 17:47
编译方式很多,如下图,要视情况而定。 而代码不仅仅是数学公式,因为它当你是在写文章的,而不是像这里只是打几个公式那么简单。 所以需要声明或加载一些东东,比如使用什么文档类 \documentclass{...} ,要不要加载宏包 \usepackage{...}(可省,但一般你省不了),文章开始了要 \(\verb"\begin{document}"\) ,结束要 \(\verb"\end{document}"\) ,这些是必不可少的。 附件中就是一个简单示例,就用第一个 PDFTeXify 编译就可以了,编译完自动出来 PDF。
kuing 5# 2013-1-18 20:15
附件中,第一行用的 ctexart 文档类对中文什么的已经设置好,否则还要加载相关宏包及设置,第二行是加载了三个数学上最常用宏包。 这个文档除了用 PDFTeXify 编译之外,还可以用 PDFLaTeX、LaTeX 来编译,但这两种通常要编译两次才能生成正确的公式编号引用,LaTeX 编译后还需要点 dvi2pdf 才能生成 PDF。
abababa 6# 2013-1-18 20:18
我其实是受一位网友的影响才用的latex,应该说是xelatex吧,这些我也分不出来。 那位网友用的是linux,我不会,他说让我用记事本写代码就行了,结果就一直用,现在还好,了解了不少东西。
hnsredfox_007 7# 2013-1-22 09:22
5# kuing 刚下载,看看!不知道好学不?
kuing 8# 2013-1-22 10:41
7# hnsredfox_007 能编译出PDF来吗?
hnsredfox_007 9# 2013-1-22 14:24
5# kuing 自动出来的pdf在哪里啊
kuing 10# 2013-1-22 14:26
9# hnsredfox_007 一般会在你所编译的文件的地方
hnsredfox_007 11# 2013-1-24 08:26
10# kuing 又试了N多次,结果还是不行,只有暂时放弃了。以后有机会再弄吧!谢谢你的多次帮忙哦
kuing 12# 2013-1-24 08:37
11# hnsredfox_007 有没有试试换成用PDFLaTeX编译? 或等网络好一点给你远程看看吧
hnsredfox_007 13# 2013-1-24 08:53
12# kuing 总之先谢谢你了。最近可能上网的机会少点,要陪着学生学习——准备应付自主招生笔试。
kuing 14# 2013-1-24 08:57
13# hnsredfox_007 噢…那些我不懂,呵呵…咦,话说楼主不见人了。
abababa 15# 2013-1-24 15:46
9# hnsredfox_007 一个网友教我的方法 比如写好了一个latex文件是my.tex,然后把它和批处理文件放在同一个文件夹里,双击那个build.bat,有让输入一个文件名,输入my,不用带上.tex,回车就出来了,在文件夹里就有my.pdf了。 如果又写好了一个latex文件是my2.tex,再双击build.bat,输入my2,文件夹里就出来my2.pdf了。 我把他给我写的传上来。 build.zip (511 Bytes)
kuing 16# 2013-1-24 15:54
15# abababa 这跟点击winedt里的按钮有什么区别……而且bat里还规定了xelatex?
abababa 17# 2013-1-24 15:56
本帖最后由 abababa 于 2013-1-24 15:58 编辑 16# kuing 哦,我是用记事本那么写的,所以就没用winedt,其实里面改的话也很容易,但又觉得xelatex就挺好用的,够用了。 其实我也不太会用,总是点开始运行然后进cmd,麻烦了,就问了网友应该怎么办,结果就给我这个,觉得还挺方便的。
kuing 18# 2013-1-24 16:06
17# abababa winedt 的按钮大概也是运行类似这样的命令行…… xelatex 有一个要注意的地方是编码问题,不过你用记事本写应该没什么问题,用 winedt 就有可能会乱码。
abababa 19# 2013-1-24 16:13
18# kuing 是有一次弄出来的pdf都是汉字,细看也有认识的,但连不成句,和我写在tex里的一点也不一样,那个就是乱码了吧。后来不知怎么就没出现了。
kuing 20# 2013-1-24 16:16
19# abababa 不好说,我对编码什么的还是有点迷糊……
thread-1059-2-1.html:
yes94 21# 2013-3-5 19:46
10# kuing 又试了N多次,结果还是不行,只有暂时放弃了。以后有机会再弄吧!谢谢你的多次帮忙哦 hnsredfox_007 发表于 2013-1-24 08:26 我也是试了n次,点了那几个pdftex试试,根本没看见那个pdf文件在哪儿?而且连反应都没有! 我下载你的那个asdffdsa.tex文件在桌面上,后来叫我存盘,生成一个什么.bak的文件了,又不能打开它!
yes94 22# 2013-3-5 19:51
21# yes94
yes94 23# 2013-3-5 19:58
22# yes94 abababa的方法还可以用,下面是编译出来的结果(编译提示了kuing的第13行有些小错误吧,可能记错了行数,可能是13行,但仍能编译出结果,只是有些乱码):
kuing 24# 2013-3-5 20:01
22# yes94 选了 pdflatex 之后,就直接点那里,不是点拉下三角形了
yes94 25# 2013-3-5 20:07
24# kuing 我晕!我还点下拉三角形了! 用abababa的方法不好,编译出来有小部分是乱码, 还是winedit编译出来的没乱码, 谢谢啦!
kuing 26# 2013-3-5 20:27
难道之前 hnsredfox_007 没编译出来也是因为这样?
hnsredfox_007 27# 2013-3-7 10:07
26# kuing 编辑tex,另存为0.tex在桌面上 点击换成“pdflatex” 点击此处 没有反应啊!pdf文件在哪里呢
hnsredfox_007 28# 2013-3-7 10:30
25# yes94 的确,好像不支持中文,含中文的都不行哦
kuing 29# 2013-3-7 11:21
那就不知怎么回事了……
yes94 30# 2013-3-7 14:04
28# hnsredfox_007 abababa的方法对中文好像不行 但winedt对中文是可行的
kuing 31# 2013-3-7 14:16
30# yes94 之所以会乱码,是因为他用的是xelatex编译,默认编码是utf-8,而winedt里默认的编码是gbk。 我将他那个bat改了下,改为用pdflatex编译,这样应该就可以用于我前面发的那个tex。 用法跟他讲的一样,见附件。 pdflatexbuild.bat (87 Bytes) PS、为保正正确生成公式引用编号,该bat里运行了两次pdflatex。
hnsredfox_007 32# 2013-3-7 18:48
31# kuing 都放在桌面上,运行结果:nothng.
kuing 33# 2013-3-7 18:50
看来得现场看看了
yes94 34# 2013-3-7 22:12
33# kuing 果然比abababa的那个好,测试成功!
yes94 35# 2013-3-7 22:19
本帖最后由 yes94 于 2013-3-7 22:20 编辑 kuing能不能帮个忙把这个word文档搞成latex的? 前提是花时间不多哈 要求不改变行距,字体,只将公式弄成漂亮的latex就行了 然后我编译成pdf,谢谢
kuing 36# 2013-3-7 23:30
实在闲得dan疼再说吧……
hnsredfox_007 37# 2013-3-8 09:36
好像是什么"yap"错误,但是卸载,重新下载,安装,还是不行啊
hnsredfox_007 38# 2013-3-8 20:06
如下图,安装过程中出现的问题,如何解决
kuing 39# 2013-3-8 20:22
38# hnsredfox_007 不清楚了,我没遇到过这种问题……不知怎么帮你了,要么你去 CTeX 论坛问下
abababa 40# 2013-3-8 21:42
38# hnsredfox_007 哎,刚把这个图给一位网友看,结果他回,windows的?重启重启换机器。 不过最后还是说了点,问是不是用了ghost的版本,最好是换原版的。要是不影响使用不换也行。
thread-1059-3-1.html:
hnsredfox_007 41# 2013-3-22 14:32
40# abababa 现在的机子不都是现成装好的吗?貌似都是ghost的!不知道怎么办……谢了
abababa 42# 2013-3-25 19:57
41# hnsredfox_007 据说是有些dll文件没注册,其实我也不大懂,都是听网友说的。
hnsredfox_007 43# 2013-3-26 13:45
42# abababa 谢谢! 唉,有东好西就是不能用,真郁闷……
abababa 44# 2013-3-27 19:58
43# hnsredfox_007 又问了下网友,他说让试试这里的那个软件。 http://www.fixpcdll.com/repair-d ... -yap.exe-error.html 安装,然后启动,扫描,然后点Fix All,看看能不能修复。 他说两个都让试试。查找“Download yap.exe Error Fix Tool Now”和“Run a Free Security Scan on yap.exe”
hnsredfox_007 45# 2013-3-27 21:14
44# abababa 谢谢,我试一试……
hnsredfox_007 46# 2013-3-28 18:42
44# abababa 抱歉啊,这个修复没有好,还是不能用哦
abababa 47# 2013-3-29 20:22
46# hnsredfox_007 哎,我也不怎么懂电脑。刚才问了下网友,他说不行换texlive试试。不过他下线了,我也不知道在哪下载。
abababa 48# 2013-3-30 14:39
46# hnsredfox_007 问了网友,给了这个网址 http://www.tug.org/texlive/acquire-netinstall.html 说是让下载install-tl.zip for Windows (24mb)的那个,试试看这个行不行? 网友说主要是系统和MikTex的yap出的错,不用MikTex就行了,哎,我也听不懂。
kuing 49# 2013-3-30 14:45
其实我总觉得 yap 并不是必须品……出错了不至于整个都用不了吧……顶多影响插图……
abababa 50# 2013-3-30 14:51
49# kuing 我也很少用插图功能,以前弄过,好像还挺麻烦的,后来听说了tikz,不过我也不会用,版主给的那个画箭头的现在只是照抄着那么画上了,原理到现在还没看。
hnsredfox_007 51# 2013-3-30 15:45
49# kuing 红线部分是灰色的吗?我的一直是啊
hnsredfox_007 52# 2013-3-30 15:45
48# abababa 正在试这个方法……
kuing 53# 2013-3-30 15:50
51# hnsredfox_007 编译前那堆是灰色是正常的,但是编译过后就会有些变化,你现在点那个PDFLaTeX会有什么反应?
hnsredfox_007 54# 2013-3-30 15:53
53# kuing 没有反应啊……还是那样
kuing 55# 2013-3-30 15:55
不是点小三角喔
hnsredfox_007 56# 2013-3-30 15:58
55# kuing 直接点的那个图标啊 但是什么也没有 就是安装过程中出现了“yap错误”,点击“确定”或“取消”,安装后结果就这样了
abababa 57# 2013-3-30 16:05
56# hnsredfox_007 试试kuing在31楼发的那个批处理呢?能不能编译?能的话说明大部分功能还是好用的吧。我在网友的建议下是用记事本写的,还没用过那个winedit编辑器呢。
hnsredfox_007 58# 2013-3-30 16:44
57# abababa 不行啊 没有pdf输出 15#那个英文可用,有pdf输出,但是对于中文就是乱码了
abababa 59# 2013-3-30 16:49
58# hnsredfox_007 你试试把那个1.tex文件用记事本打开,然后另存为,保存的时候那个保存按钮左边有一个编码,选UTF-8,然后编译那个新保存的试试呢?
hnsredfox_007 60# 2013-3-30 16:56
59# abababa 试过了不行,我可以死心了……谢谢大家啊 看来是电脑问题啊
thread-1059-4-1.html:
yes94 61# 2013-3-30 18:52
59# abababa 试过了不行,我可以死心了……谢谢大家啊 看来是电脑问题啊 hnsredfox_007 发表于 2013-3-30 16:56 把你的源文件传上来,我们试一试能否编译?
hnsredfox_007 62# 2013-3-31 20:04
61# yes94 今天不知道怎么一回事,点击了一下那个kuing的bat,输入后编译成功了!可以输出pdf格式了……但是我至今还是一头雾水……
kuing 63# 2013-3-31 20:20
62# hnsredfox_007 ………………
isea 64# 2013-3-31 22:02
查看编译文件 CTeX 下是 按 F8, 让其编译后并查看是F9 有些选项点鼠标不弹出(查看)编译后的PDF
李斌斌755 65# 2013-5-10 17:57
难度系数好大
isea 66# 2013-5-10 21:29
65# 李斌斌755 重装个完整的系统,一定什么都解决
kuing 67# 2013-5-11 21:23
66# isea 还不一定,不同的发行版都可能会有区别……
李斌斌755 68# 2013-5-11 21:55
软件更新太快。
thread-106-1-2.html: 与椭球面有关的问题
图图 1# 2011-10-17 00:54
本帖最后由 图图 于 2011-10-17 01:01 编辑 由椭球面S:$\dfrac{x^2}{a^2}+\dfrac{y^2}{b^2}+\dfrac{z^2}{c^2}=1$的中心引三条互相垂直的射线,分别交S于点$P_1$、$P_2$、$P_3$. 设$|\overrightarrow{OP_i}|=r_i$    (i=1,2,3). 证明 \[\frac{1}{r_1^2}+\frac{1}{r_2^2}+\frac{1}{r_3^2}=\frac{1}{a^2}+\frac{1}{b^2}+\frac{1}{c^2}\]
kuing 2# 2011-10-17 22:22
由椭球面S:$\dfrac{x^2}{a^2}+\dfrac{y^2}{b^2}+\dfrac{z^2}{c^2}=1$的中心引三条互相垂直的射线,分别交S于点$P_1$、$P_2$、$P_3$. 设$|\overrightarrow{OP_i}|=r_i$    (i=1,2,3). 证明 \[\frac{1}{r_1^2}+\frac{1}{r_2^2}+\frac{1}{r_3^2}=\frac{1}{a^2}+\frac{1}{b^2}+\frac{1}{c^2}\] 图图 发表于 2011-10-17 00:54 首先我们有如下两个事实: 一、上述命题的平面情形成立(即对平面椭圆或圆成立)。 这是高中证过的,这里从略; 二、若椭球与某平面相交,则交线为椭圆(此处暂且认为圆也为椭圆)。 设椭球方程 $ax^2+by^2+cz^2=1$ 与平面方程 $Ax+By+Cz+D=0$ 相交,不妨设 $C\ne0$,消去 $z$ 有 \[ C^2(ax^2+by^2)+c(D+Ax+By)^2=C^2, \]上式为交线在 $xOy$ 平面的投影方程,由于交线显然为封闭图形,且由 $C\ne0$ 知交线所在平面不与 $xOy$ 平面垂直,故此上式必为椭圆方程,而投影不改变二次曲线的类型,因此交线也必为椭圆。 有了这两个事实,就可以证明这空间推广了。 在空间直角坐标系中,对于过原点的三条互相垂直的射线,不妨记为 $L_1$、$L_2$、$L_3$,若固定其中一条 $L_1$,旋转另外两条 $L_2$、$L_3$(旋转过程中保持三条射线的相对位置不变,下同),那么 $L_2$、$L_3$ 恒在过原点且垂直于 $L_1$ 的平面内。由事实二,该平面与椭球的交线为一椭圆,该椭圆与 $L_2$、$L_3$ 相交于 $P_2$、$P_3$,那么在该平面内的情形正符合事实一,所以,在旋转过程中,$1/r_2^2+1/r_3^2$ 保持不变,而 $L_1$ 固定所以 $r_1$ 固定,即 $1/r_1^2+1/r_2^2+1/r_3^2$ 也不变。 又显然地,旋转过程中总可以将 $L_2$ 或 $L_3$ 旋转到其中一个坐标平面上,不妨将 $L_2$ 旋转到平面 $xOy$ 上,然后固定 $L_2$,旋转 $L_1$、$L_3$,类似地,也可以将 $L_1$ 或 $L_3$ 之一旋转到平面 $xOy$ 上,不妨将 $L_1$ 旋转到平面 $xOy$ 上,那么 $L_3$ 必然与 $z$ 轴重合,再固定 $L_3$,也必能将 $L_1$、$L_2$ 旋转到与 $x$、$y$ 坐标重合,此时 $P_1$、$P_2$、$P_3$ 就是椭球的三个顶点,即显然有 $1/r_1^2+1/r_2^2+1/r_3^2=1/a^2+1/b^2+1/c^2$,再根据上述过程的任意性及旋转对 $1/r_1^2+1/r_2^2+1/r_3^2$ 的不变性,可知原命题成立。
kuing 3# 2011-10-17 22:37
这种证法是不是有点非主流?
kuing 4# 2011-10-17 23:22
有没有代数证法?图图?
GAM 5# 2011-10-17 23:32
我不知道怎么表示,但感觉用球坐标系会简单点吧
kuing 6# 2011-10-17 23:44
5# GAM 嗯?可以的话大致写写过程哟
GAM 7# 2011-10-18 11:37
没想到那三条线的角怎么表示
kuing 8# 2011-10-19 01:51
7# GAM 空间中的角的情形比平面复杂很多的说。。。垂直那里我除了用数量积表示之外也没其他想法了。。。
GAM 9# 2011-10-19 11:40
本来以为用式子最后能贷出来,最后失败了。。。
tian27546 10# 2011-10-19 14:40
很简单 但是laxt不熟悉 利用矩阵正交变换之后相加就可以了
kuing 11# 2011-10-19 15:02
有空补矩阵去…………
图图 12# 2011-10-19 22:59
本帖最后由 图图 于 2011-10-19 23:19 编辑 这是参考答案 设$OP_i$与x,y,z轴的夹角的余弦值分别为$\lambda_i$,$\mu_i$,$\nu_i$(i=1,2,3),由于$OP_i$(i=1,2,3)两两垂直,可以把他们看成长方体中的边,而把三个坐标轴看成长方体中的对角线,从而有$\displaystyle\sum_{i=1}^{3}\lambda_i^2=\sum_{i=1}^{3}\mu_i^2=\sum_{i=1}^{3}\nu_i^2=1$,因为$\dfrac1{r_i^2}=\dfrac{\lambda_i^2}{a^2}+\dfrac{\mu_i^2}{b^2}+\dfrac{\nu_i^2}{c^2}$(i=1,2,3), 故$\dfrac1{r_1^2}+\dfrac1{r_2^2}+\dfrac1{r_3^2}=\dfrac1{a^2}\displaystyle\sum_{i=1}^{3}\lambda_i^2+\dfrac1{b^2}\sum_{i=1}^{3}\mu_i^2+\dfrac1{c^2}\sum_{i=1}^{3}\nu_i^2=\dfrac1{a^2}+\dfrac1{b^2}+\dfrac1{c^2}$
鱼儿 13# 2011-10-20 14:54
本帖最后由 鱼儿 于 2011-10-20 14:58 编辑 实际上,只要先证明如下命题即可(猜测教材在楼主所提的问题前会有这道题): 设$P$是椭球面S:$\dfrac{x^2}{a^2}+\dfrac{y^2}{b^2}+\dfrac{z^2}{c^2}=1$上任意一点,$\overrightarrow{OP}$的方向余弦为$\lambda$,$\mu$,$\nu$,$|\overrightarrow{OP}|=r$. 则有 \[\frac{\lambda^2}{a^2}+\frac{\mu^2}{b^2}+\frac{\nu^2}{c^2}=\frac{1}{r^2}\]
鱼儿 14# 2011-10-20 15:03
很简单 但是laxt不熟悉 利用矩阵正交变换之后相加就可以了 tian27546 发表于 2011-10-19 14:40 遗憾的是,在这道题被教材安排在“向量”这一章,在此之前还没有讲述“正交变换”的内容。
kuing 15# 2011-10-20 16:01
又见鱼儿,
thread-1060-1-3.html: 未解决问题收集贴,(不定期更新1楼连接)
realnumber 1# 2013-1-18 12:20
本帖最后由 realnumber 于 2013-3-29 09:34 编辑 期待高手出手的,解决后请给出同类的练习,碰到的问题,最好是推广问题,.... 1.开个数论的帖子   yizhong 2.开个多项式帖子    yizhong 3.翻人教群聊记录发现的一道题(幂指不等式)  kuing  17和24楼 4.16点问题---推广后呢?  realnumber  2楼 5.一道条件比较诡异的求最大值问题  yizhong 6.求通项公式  Chetion 7.开个帖子玩玩一个新函数  都市侠影 8.某群里看到的一道题       kuing 1楼 9.How to holder it?     pxchg1200 10.数月前在安zp空间里看到的一道不等式  kuing 要求其它证明 11.一个猜想   hflz01 12. 13. 14.最多可作几条大于90°的射线--以及怎么推广?  realnumber 15.一道数论题   nash  9楼 16. 17.求最大公约数的一道题,只需要思路    q85669551   不仅读不懂,也没清楚是否解决 18.任意抽取三个人,必有一道题的答案互不相同  realnumber 19.请高手指教一道意外得到的排列组合题  ccnu_chb_ycb 20. 21.来自人教群的数三角形[未解决]  kuing 22.越南人也叫着过年。。。    pxchg1200 23.MathUniverse的一个题     pxchg1200 24.ham-hap 不等式   pxchg1200 25.$\max\{1,ab,a+b\}\ge\frac{4}{9}(1+a)(1+b)$,怎么推广?    16楼kuing 26.来自粉丝群的不等式若干 1楼 天书 27.初等数论学习贴--不定方程 1楼 realnumber 28.来自粉丝群的一道幂指轮换最小值[未解决] 终于解决掉的 12.请问一个棋盘的问题   解答者  abababa的一网友  8楼 13.卡住了,关于e  解答者    爱好者-Salvation 16.一个三角不等式,未解决  解答者   hnsredfox_007  5楼 20.网友这次问数列  解答者  yizhong 4楼  realnumber  6楼 有未解决的,跟连接,我会加在1楼,要么一般2星期~1月后了.
kuing 2# 2013-1-18 12:55
呃?未解决问题?这几个不是都解决了的么
yayaweha 3# 2013-1-18 22:32
这样不如,在初等数学讨论中分几个板块
kuing 4# 2013-1-18 22:46
分版块我想就不必了,除非论坛真的热了起来
kuing 5# 2013-1-18 23:37
有空或者会整主题分类
abababa 6# 2013-1-26 09:31
我看有很多帖都加了分类,版主能不能弄个“未解决”的分类?让大家各自编辑自己帖子的分类就好了,这样由别人找还是太烦琐。
kuing 7# 2013-1-26 13:49
我看有很多帖都加了分类,版主能不能弄个“未解决”的分类?让大家各自编辑自己帖子的分类就好了,这样由别人找还是太烦琐。 abababa 发表于 2013-1-26 09:31 由于论坛功能有限,没有多重分类的功能,所以设置分类时也只能是按一种属性来分(比如说按是否被解决、按题目类型、按题目难度级别等等),是不应兼容的。 这些我几经考虑,最后还是觉得按题目类型来分比较好。 退一步的办法,也可以对你自己未解决的贴子在标题上直接输入[未解决]来标明,沉了就顶上来就行了。
hnsredfox_007 8# 2013-1-28 08:58
问题 16 可以更改为“已解决”了
abababa 9# 2013-2-4 23:34
9# realnumber 第12,请教一个棋盘的问题,已被一位网友解决
Tesla35 10# 2013-2-27 22:35
mark!
yes94 11# 2013-2-28 19:43
第六题:kuing在8楼说了, 此题已解决,删掉吧
realnumber 12# 2013-2-28 22:38
11# yes94 8楼没解答啊?,而且是海盗发的贴
yes94 13# 2013-2-28 23:39
12# realnumber http://kkkkuingggg.5d6d.net/thread-575-1-10.html 此贴8楼kuing
realnumber 14# 2013-3-1 08:02
13# yes94 我也觉得8楼结论对的,但是还是归到猜想,也需要证明.
yes94 15# 2013-3-2 14:13
14# realnumber 我以前看过一个链接,说这类题没法求通项的。(除非新定义一个什么函数)
abababa 16# 2013-3-8 21:36
添加一个题,http://kkkkuingggg.5d6d.net/thread-1207-1-1.html 标题:请教一个比较角度大小的问题。 想了几天没结果,绕来绕去就迷糊了。
realnumber 17# 2013-3-29 09:36
加了几个~~
thread-1061-1-1.html: 问主人几个问题,请恕冒昧!
math_dalin 1# 2013-1-18 15:00
1、您从接触latex到使用latex打印第一篇数学文稿用了多久的时间? 2、就现在而言,您打印一份标准的高考数学试卷(全国卷)需要多久的时间? 谢谢您的回复,祝您万事如意!
kuing 2# 2013-1-18 17:18
1、记不清了,只记得我用 latex 并不是“连续”的。早期并没有认真学,断断续续地用着。 第一篇东东肯定是很费时而且排得很烂,要各种改来改去的。可以说直到要搞人教网刊《数学空间》了,我才真正认真去学习它,才排版出较为像样的 latex 数学文稿,此前的那些都不值一提。 2、我没试过排版标准的数学试卷,不过我想,在模版精心设计好之后,应该还是比较快而且方便的。 速度方面大概不会比 word 慢很多(除非图形很多,因为我会用 tikz 画,用命令作出精确的矢量图,就会比较慢),但效果肯定是 word 比不了的。 PS、精美的数学试卷 latex 模版可以参考盖鹤麟的《LaTeX数学试卷排版指南》,有能力的话最好自己设计一个,不要全抄,因为那里的设计不一定适合你的要求,有的东东也过时了。
thread-1062-1-5.html: [不等式] Jack Garfunkel不等式
realnumber 1# 2013-1-19 13:28
四川陈钇帆(52#####75) 13:16:27 除了can的柯西,韩jj的对称求导,和某本书的巧妙证法 --转自不等式群.
kuing 2# 2013-1-19 13:33
怎么不把所知的那几个证法贴一贴……要不然重复了就……
realnumber 3# 2013-1-19 13:40
2# kuing 没看到他们发证明.
pxchg1200 4# 2013-1-21 09:34
2# kuing 这个就是传说中的 \[ \frac{a}{\sqrt{a+b}}+\frac{b}{\sqrt{b+c}}+\frac{c}{\sqrt{c+a}}\leq \frac{5}{4}\sqrt{a+b+c} \]
kuing 5# 2013-1-21 17:25
4# pxchg1200 oh,这个就见过几次,不过这跟1#的貌似不等价…… 及此发现1#的不等式似乎打少了条件,否则将 $b$ 固定为小于 1 的常数然后令 $a\to0$ 则左边无穷大。 要是1#的不等式加上 $a^2$, $b^2$, $c^2$ 能构成三角形,那应该就等价了。
thread-1063-1-1.html: 关于本版块的主题分类
kuing 1# 2013-1-19 16:02
目前暂时设定了本版块的主题分类共 6 类:数列、不等式、几何、函数、组合、数论。 这可以在版块左上角看到,并且可以点击列出相应的主题。 其中“几何”类包括了平面几何、立体几何以及解析几何,不再细分,因为解几题也常用平几方法,反之亦可。 而“组合”类则包括了排列组合计数原理的题目以及涉及这些计算的概率题等。 以后在发新贴时都可以选择这些分类,最多选一个,也可以不选,当然了,能选的话最好还是选一下。 如果一下子忘记了选分类,贴子已经发了出去,也可以在编辑贴子时重新选分类。 有时可能涉及不止一种类型的题,比如常见的“数列不等式”涉及了前两类,这时大概就要看情况而定了,看偏重哪一方面,如果实在不知选哪个类好,可以干脆不选。 刚才我对大部分主题都给简单地分类了一下,如果你觉得你的主题被分的类不对,你可以自己编辑贴子重选。 有什么别的提议可以跟贴提出。
kuing 2# 2013-1-19 20:25
大致分类得差不多了,挺累的说……有的真是不易分,还好可以不分……
abababa 3# 2013-2-14 11:48
其实觉得“未解决”也可以单分一类吧,感觉分类不一定非要并列关系。 我在此论坛的问题已经都解决了,呵呵,十分感谢网友们的帮助。
yes94 4# 2013-3-7 18:24
我觉得在几何一类里打个括号,内写(解几、平几、立几) [几何](解几、平几、立几)但是有太长!怎么办?
thread-1064-1-5.html: [不等式] 来自人教论坛跟贴提问的似曾相识的n元不等式
kuing 1# 2013-1-19 17:47
来自: http://bbs.pep.com.cn/forum.php? ... 120&pid=7718568 30楼 tyzhifubao123 发表于 2013-1-19 11:53 各位老师到我相册看看这道题咋解??? 把你的题贴出来: \[\sum{\frac{a_k}{1+a_{k+1}^2}}=\sum{\frac{a_k(1+a_{k+1}^2-a_{k+1}^2)}{1+a_{k+1}^2}}=2-\sum{\frac{a_ka_{k+1}^2}{1+a_{k+1}^2}}\geqslant2-\frac12\sum{a_ka_{k+1}},\] 由已知不等式 \[\left( \sum{a_k} \right)^2\geqslant 4\sum{a_ka_{k+1}} \riff 2-\frac12\sum{a_ka_{k+1}}\geqslant 2-\frac18\left( \sum{a_k} \right)^2=\frac32,\] 当 $a_1=a_2=\cdots =a_{n-2}=0$, $a_{n-1}=a_n=1$ 时取等。
yes94 2# 2013-1-19 22:39
本帖最后由 yes94 于 2013-1-19 23:18 编辑 我也觉得是似曾相识,07女子竞赛,韩京俊的书也有,
kuing 3# 2013-1-21 17:22
2# yes94 oh, 多谢出处dang
yes94 4# 2013-1-22 13:35
2# yes94 oh, 多谢出处dang kuing 发表于 2013-1-21 17:22 向你学的,
thread-1065-1-1.html: [不等式] 来自人教群的一道大题第三问对数平方和不等式(兼水母收集)
kuing 1# 2013-1-19 21:01
前两问就不做了,不好玩,只玩玩第三问,估计下面的证法跟参考答案不一样。 由柯西不等式有 \[ n(\ln^21+\ln^22+\cdots +\ln^2n)\geqslant (\ln 1+\ln 2+\cdots +\ln n)^2, \] 于是要证原不等式,只需证 \[ \ln 1+\ln 2+\cdots +\ln n>\frac{(n-1)^2}{2n}, \]即\[ \ln 1+\ln n+\ln 2+\ln (n-1)+\ln 3+\ln (n-2)+\cdots +\ln n+\ln 1>\frac{(n-1)^2}n, \]即\[ \ln n+\ln (2(n-1))+\ln (3(n-2))+\cdots +\ln n>\frac{(n-1)^2}n, \] 注意到对于任意 $1\leqslant k\leqslant n$ 都有 $k(n-k+1)\geqslant n$,由此得到 \[ \ln n+\ln (2(n-1))+\ln (3(n-2))+\cdots +\ln n\geqslant n\ln n, \] 因此,只要证 \[ \ln n>\left( \frac{n-1}n \right)^2, \] 令 $n=1/t$,由 $n\geqslant 2$ 知 $0<t<1$,由上式等价于 \[ g(t)=(1-t)^2+\ln t<0, \] 求导得 \[ g'(t)=-2(1-t)+\frac1t=t+t+\frac1t-2\geqslant t>0, \] 从而 $g(t)<g(1)=0$,所以原不等式得证。
yuzi 2# 2013-1-19 21:15
强。。。。学习。。。。学习无数次,学不会
yayaweha 3# 2013-1-19 22:06
KK来看看我当年问过的一题吧,不知道你有没有新颖的方法 $$\frac{1}{\ln2}+\frac{1}{\ln3}+\cdots+\frac{1}{\ln n}<\frac{1-f(n+1)}{\ln2\cdot\ln n}.$$ 其中$n\in\mathbf{N}^+,n\geqslant 2$,$f(x)=x-x\ln x$
yes94 4# 2013-1-19 22:09
2# yuzi 总看得会噻,
realnumber 5# 2013-1-22 09:38
本帖最后由 realnumber 于 2013-1-22 09:40 编辑 这个不等式其实是水母, 注意到$n\ge3$时,$\ln{n}\ge1$,那么左边就是大于$n-2$,那么就是要证明 \[n-2\ge\frac{(n-1)^4}{4n^3}\],只要证明$n\ge{n-1},4(n-2)\ge{n-1}$,都成立,再证一下,$n=2$,就ok. 而我要提的问题是左边有更好的近似估计吗?可以假定$n\ge{n_0}$什么的.
kuing 6# 2013-1-22 10:56
5# realnumber OMG,水母,果然够形象。
realnumber 7# 2013-1-27 09:47
本帖最后由 realnumber 于 2013-1-27 10:35 编辑 似乎又是类似一个 来自不等式群福建-周坤泷(27####3) 2013-1-26 23:10:40 因为$\ln{n}<n-1$,只需要证明\[\frac{1}{2^2}+\frac{2}{3^2}+\frac{3}{4^2}+...+\frac{n-1}{n^2}<\frac{2n^2-n-1}{4(n+1)}=\frac{n^2}{2(n+1)}-\frac{1}{4}\] \[\frac{2}{3^2}+\frac{3}{4^2}+...+\frac{n-1}{n^2}<\frac{n^2}{2(n+1)}\] \[\frac{1}{3^2}+\frac{1}{4^2}+...+\frac{1}{n^2}<\frac{n}{2(n+1)}\] \[\frac{1}{3^2}+\frac{1}{4^2}+...+\frac{1}{n^2}<\frac{1}{2}-\frac{1}{n}<\frac{n}{2(n+1)}=\frac{1}{2}-\frac{1}{2(n+1)}\]
realnumber 8# 2013-1-27 13:11
本帖最后由 realnumber 于 2013-1-27 14:44 编辑 3# yayaweha \[\frac{1}{\ln2}+\frac{1}{\ln3}+\cdots+\frac{1}{\ln n}<\frac{1-f(n+1)}{\ln2\cdot\ln n}.\] \[\frac{\ln2}{\ln2}+\frac{\ln2}{\ln3}+\cdots+\frac{\ln2}{\ln n}<1+1+\frac{1}{2}+\cdots+\frac{1}{2}=\frac{n+1}{2}\] 如此只需要证明\[\frac{n+1}{2}\ln{n}\le 1-f(n+1)=(n+1)\ln{(n+1)}-n\] 等价于\[n\le{(n+1)\ln{\frac{n+1}{\sqrt{n}}}}\] 而\[\ln{\frac{n+1}{\sqrt{n}}}>1\]--突然觉得这个不算水母,似乎谁也这么证明过,kuing?
kuing 9# 2013-1-27 22:43
8# realnumber 这样看也很水啊 看来以后遇数列不等式还是先从最水的方向放缩一下再说……
yayaweha 10# 2013-1-27 22:57
什么是水母
realnumber 11# 2013-1-28 08:18
10# yayaweha 百度的 幽浮水母(Moon Jellyfish) 一般市面上常见半透明的水母俗称 学名:Aurelia auriea,中名:海月水母。分布於世界各大洋温 水域到暖水域,为腔肠动物。中央是它的胃;伞缘四周的细丝是触手, 胃下4条彩带般的口腕为捕食工具,以浮游生物为食, 身体是胶质,含近98%的水份。 意思是题目设计得不够好.
realnumber 12# 2013-1-28 08:31
9# kuing 这里作个假设:一般的高中教师编的关于$\ln x$的不等式,就是利用常见的$\ln (1+x)\le x$,而这个一旦$x$离0比较远的话,比如数列,.. 而"$\ln x$有关的"利用积分又不是高中能解决的.
realnumber 13# 2013-1-29 11:52
筱弦月(53####48)  11:34:08 $(\ln 2-\ln 1)^2+(\ln 3-\ln 2)^2+……+(\ln(n+1)-\ln(n))^2<1$ 利用$\ln(n+1)-\ln(n)=\ln(1+\frac{1}{n})\le{\frac{1}{n}}$ 再利用这个$\frac{1}{n^2}<\frac{1}{n(n+1)}=\frac{1}{n}-\frac{1}{n+1}$
realnumber 14# 2013-3-4 08:28
http://kkkkuingggg.5d6d.net/thread-1200-1-1.html 把水母也放一起 求证:$\frac{1}{2^2}\ln 2^2+\frac{1}{3^2}\ln 3^2+\frac{1}{4^2}\ln 4^2+\cdots+\frac{1}{(n+1)^2}\ln (n+1)^2>\frac{n}{2(n+1)(n+2)}$ 证明:当$n\ge2$时,有$\ln (n+1)>1$ 那么本题,$n=1$单独验证. 当$n\ge2$时,$\frac{1}{2^2}\ln 2^2+\frac{1}{3^2}\ln 3^2+\frac{1}{4^2}\ln 4^2+\cdots+\frac{1}{(n+1)^2}\ln (n+1)^2>\frac{1}{4}+\frac{2}{3^2}+\frac{2}{4^2}+\cdots+\frac{2}{(n+1)^2}=\frac{1}{4}+\frac{2}{3}-\frac{2}{n+2}>\frac{1}{2(n+2)}>\frac{n}{2(n+1)(n+2)}$
yayaweha 15# 2013-3-5 13:00
本帖最后由 yayaweha 于 2013-3-5 13:02 编辑 直接证明$$\frac{1}{2}\ln 2>\frac{n}{2(n+1)(n+2)}$$就做完了
realnumber 16# 2013-3-5 13:44
15# yayaweha 哦,那不是水母中的水母了~~
kuing 17# 2013-3-5 13:50

realnumber 18# 2013-3-27 12:02
本帖最后由 realnumber 于 2013-3-27 13:51 编辑 又一弱弱的水题,可以保留前2项,放缩,后面都用$\ln n<n$. 如果知道$e^x>x^e,x>0$,本题更容易处理,变为求证:$\frac{1}{2^3}+\frac{1}{3^3}+\frac{1}{4^3}+\ldots +\frac{1}{n^3}<\frac{1}{2}$
kuing 19# 2013-3-27 13:24
改了一下标题
yes94 20# 2013-3-27 21:29
似乎又是类似一个795 来自不等式群福建-周坤泷(27####3) 2013-1-26 23:10:40 因为$\ln{n} realnumber 发表于 2013-1-27 09:47 这个水母似乎不够好吧?
thread-1065-2-1.html:
第一章 21# 2013-3-28 16:27
我也发一个水母的题目,以前汕头的二模题,最后一题: 已知正项数$\{a_n\}$的首项$a_1=m$,其中$0<m<1$,函数$f(x)=\frac{x}{1+2x}$ (1)若数列$\{a_n\}$满足$a_{n+1}=f(a_n)(n\ge1且n\in{N^*})$,证明$\{\frac{1}{a_n}\}$是等差数列,并求出数列$\{a_n\}$的通项公式; (2)若数列$\{a_n\}$满足$a_{n+1}\le{f(a_n)}(n\ge1且n\in{N^*})$,数列$\{b_n\}$满足$b_n=\frac{a_n}{2n+1}$,试证明$b_1+b_2+\dots+b_n<\frac{1}{2}$.
第一章 22# 2013-3-28 18:46
突然发现,这题并不是你们所说的那种水母,而是元芳说的“这里的水很深”。
isea 23# 2013-3-28 22:21
这个不等式其实是水母, 注意到$n\ge3$时,$\ln{n}\ge1$,那么左边就是大于$n-2$,那么就是要证明 \[n-2\ge\frac{(n-1)^4}{4n^3}\],只要证明$n\ge{n-1},4(n-2)\ge{n-1}$,都成立,再证一下,$n=2$,就ok. 而我要提的问题是 ... realnumber 发表于 2013-1-22 09:38 这个怎么说,解法  qiáng  暴了,题目弱暴了 但这这真不是一般人一下能看出来的,这是多少经验的积累,看穿了多少类似的题 9# kuing 这里作个假设:一般的高中教师编的关于$\ln x$的不等式,就是利用常见的$\ln (1+x)\le x$,而这个一旦$x$离0比较远的话,比如数列,.. 而"$\ln x$有关的"利用积分又不是高中能解决的. realnumber 发表于 2013-1-28 08:31 受用!! _______ 审核通过,尽量注意mingan词,见谅…… kuing
isea 24# 2013-3-28 22:27
本帖最后由 isea 于 2013-3-28 22:31 编辑 761 前两问就不做了,不好玩,只玩玩第三问,估计下面的证法跟参考答案不一样。 由柯西不等式有 \[ n(\ln^21+\ln^22+\cdots +\ln^2n)\geqslant (\ln 1+\ln 2+\cdots +\ln n)^2, \] 于是要证原不等式,只需 ... kuing 发表于 2013-1-19 21:01 这种证法,体现了熟能生巧,也受教啊 特别是5楼,更是“短”“平”“快” 12楼一语道破天机
isea 25# 2013-3-28 23:58
本帖最后由 isea 于 2013-3-29 00:03 编辑 似乎又是类似一个795 来自不等式群福建-周坤泷(27####3) 2013-1-26 23:10:40 因为$\ln{n} realnumber 发表于 2013-1-27 09:47 今天正好碰到这题,模拟高考卷的倒数第三数,第三问(层进),另一种综合法为: 题:若$n\ge2,n\in \mathbf{N^*}$,求证:$\dfrac {\ln 2}{2^2}+\dfrac{\ln 3}{3^2}+\cdots+\dfrac{\ln n}{n^2}<\dfrac{2n^2-n-1}{4(n+1)}$。 分析与证: 先需证$x>1,\ln x <x-1$,这里略去。于是$$\dfrac {\ln n^2}{n^2}<\dfrac {n^2-1}{n^2}=1-\dfrac 1{n^2} $$ 进一步 \begin{align*} 2\sum_{k=2}^{n}\dfrac{\ln k}{k^2}&=\sum_{k=2}^{n}\dfrac{\ln k^2}{k^2}\\ &<\sum_{k=2}^{n}(1-\dfrac 1{k^2})\\ &=n-1-\sum_{k=2}^{n}\dfrac {1}{k^2}\\ &=n-1-(\dfrac1{2^2}+\dfrac1{3^2}+\cdots+\dfrac1{n^2}) \end{align*} 易得 $\dfrac1{n^2}>\dfrac1{n(n+1)}$ ,于是 \begin{align*} 2\sum_{k=2}^{n}\dfrac{\ln k}{k^2}&<n-1-(\dfrac1{2\cdot3}+\dfrac1{3\cdot4}+\cdots+\dfrac1{n\cdot(n+1)})\\ &=n-1-(\dfrac12-\dfrac13+\dfrac13-\dfrac14+\cdots+\dfrac1n-\dfrac1{n+1})\\ &=n-1-(\dfrac12-\dfrac1{n+1})\\ &=n-\dfrac32+\dfrac1{n+1}\\ &=\dfrac{2n^2-n-1}{2(n+1)} \end{align*} 即\[\dfrac {\ln 2}{2^2}+\dfrac{\ln 3}{3^2}+\cdots+\dfrac{\ln n}{n^2}<\dfrac{2n^2-n-1}{4(n+1)}\]
isea 26# 2013-3-29 00:20
本帖最后由 isea 于 2013-3-29 00:25 编辑 3# yayaweha \[\frac{1}{\ln2}+\frac{1}{\ln3}+\cdots+\frac{1}{\ln n} realnumber 发表于 2013-1-27 13:11 广东,武汉等地喜欢出这样的 题:$n\in\mathbf{N}_+,n\geqslant 2,\dfrac{1}{\ln2}+\dfrac{1}{\ln3}+\cdots+\dfrac{1}{\ln n}<\dfrac{1-f(n+1)}{\ln2\cdot\ln n}.$ 这道除了欣赏,一时偶无他法—— 和21楼,先丢一起,回头看瞧瞧
第一章 27# 2013-3-30 23:27
21# 第一章 贴了很久,怎么没人鸟?只得自己打上来了 第(1)题自然是略; 第(2)题,放缩法, 解:$\frac{1}{a_{n+1}}\ge\frac{1+2a_n}{a_n}$ 可得$\frac{1}{a_{n+1}}-\frac{1}{a_n}\ge2$ 于是$\frac{1}{a_n}-\frac{1}{a_{n-1}}\ge2$,$\cdots$ $\frac{1}{a_2}-\frac{1}{a_1}\ge2$, 迭加,可得$\frac{1}{a_n}-\frac{1}{a_1}\ge2(n-1)$, $a_n\le\frac{1}{2(n-1)+\frac{1}{m}}<\frac{1}{2n-1}$, 于是$b_n<\frac{1}{(2n-1)(2n+1)}=\frac{1}{2}(\frac{1}{2n-1}-\frac{1}{2n+1})$ $b_1+b_2+\cdots+b_n<\frac{1}{2}$. 放缩成裂项.
第一章 28# 2013-5-10 18:59
继续水母收集,2013年成都三诊最后一题数列不等式: (3)求证:$\frac{3n^2+5n}{8n^2+24n+16}+\ln \sqrt{n+1}<1+\frac{1}{2}+\dots+\frac{1}{n},n\in N^*.$
第一章 29# 2013-5-10 19:20
把证明打了一下, 证明:该不等式化为$\frac{n(3n+5)}{8(n+1)(n+2)}+\ln \sqrt{n+1}<1+\frac{1}{2}+\dots+\frac{1}{n},$ 只需证$\frac{n(3n+5)}{8(n+1)(n+2)}+\ln \sqrt{n+1}-[\frac{(n-1)(3n+2)}{8n(n+1)}+\ln \sqrt{n}] <\frac{1}{n},$ 即证$\ln \frac{n+1}{n}<\frac{n+4}{n(n+2)}$ 即$\ln (1+\frac{1}{n})<\frac{1}{n}+\frac{2}{n(n+2)}$ 上式明显成立,故原题得证.
yayaweha 30# 2013-5-17 17:42
本帖最后由 yayaweha 于 2013-5-17 17:43 编辑 这道算水母吗? 设正实数$a_1,a_2.a_3\cdots ,a_n$满足$\sum_{k=1}^n a_k=1$ 求证:$$\sum_{k=1}^n ln(1+ \frac{1}{a_k^2})>\frac{2n^2}{n+2}$$
kuing 31# 2013-5-17 18:15
这道算水母吗? 设正实数$a_1,a_2.a_3\cdots ,a_n$满足$\sum_{k=1}^n a_k=1$ 求证:$$\sum_{k=1}^n ln(1+ \frac{1}{a_k^2})>\frac{2n^2}{n+2}$$ yayaweha 发表于 2013-5-17 17:42 这个我的想法是,当 $n\geqslant2$ 时 \begin{align*} \sum_{k=1}^n{\ln \left( 1+\frac1{a_k^2} \right)}&>\sum_{k=1}^n{\ln \frac1{a_k^2}} \\ & =-2\ln (a_1a_2\cdots a_n) \\ & \geqslant -2\ln \left( \frac{a_1+a_2+\cdots +a_n}n \right)^n \\ & =2n\ln n \\ & >2n \\ & >\frac{2n^2}{n+2}. \end{align*} 很随意地放了四次缩,chi度比较大,的确比较弱的不等式,应该有更简单的办法,不知你的想法如何?
零定义 32# 2013-5-18 18:07
31# kuing 木系n>=3么... kk牛笔!!! 俺先判断凹凸性,再用琴森...
kuing 33# 2013-5-18 18:09
32# 零定义 噢对,是应该 $n\geqslant3$ 才有 $\ln n>1$
零定义 34# 2013-5-18 18:12
33# kuing 俺大菜鸟,还是喜欢琴森多点,木用缩来缩去的...
kuing 35# 2013-5-18 18:23
34# 零定义 其实先去掉 1 是很自然的,因为 $1/a_k^2$ 比 1 大得多($n$ 越大差距越大),整个式子来说它是主部,那个 1 微不足道。而去掉之后合起来就简洁多了,后面也顺利成章。
零定义 36# 2013-5-18 18:35
35# kuing 嗯~看起来确实,但关键是俺木这习惯... 其实,对这水题,俺还是习惯琴森...
yayaweha 37# 2013-5-18 21:24
本帖最后由 yayaweha 于 2013-5-18 21:27 编辑 求导容易得到$lnx >2\frac{x-1}{x+1}$ 令$x=1+\frac{1}{a_k^2}$有$$\sum_{k=1}^n ln(1+ \frac{1}{a_k^2})>\sum_{k=1}^n \frac{2}{2a_k^2+1}$$ 由CS$$\sum_{k=1}^n \frac{2}{2a_k^2+1}>\frac{2n^2}{n+2\sum _{k=1}^n a_k^2}$$ $\sum _{k=1}^n a_k^2<1$ $$\sum_{k=1}^n \frac{2}{2a_k^2+1}>\frac{2n^2}{n+2\sum _{k=1}^n a_k^2}>\frac{2n^2}{n+2}$$
yayaweha 38# 2013-5-18 21:29
37# yayaweha 这个应该没问题吧?
零定义 39# 2013-5-19 01:18
水饺前把俺的愚见也贴贴...
kuing 40# 2013-5-19 01:29
38# yayaweha 没问题 不过由于第一步,看上去似乎不太水母的样子……
thread-1066-1-5.html: [不等式] 数学空间上的一道题
yayaweha 1# 2013-1-20 13:00
本帖最后由 yayaweha 于 2013-1-20 14:23 编辑 这道题用SOS怎么证明  如图
yes94 2# 2013-1-20 13:46
链接? http://kkkkuingggg.5d6d.net/viewthread.php?tid=649 看来你很认真啊
yayaweha 3# 2013-1-20 14:23
这是两道不一样的题
thread-1067-1-2.html: [不等式] 问题1:来看看这个较大中的最小值
isea 1# 2013-1-20 17:43
本帖最后由 isea 于 2013-1-20 19:01 编辑 若$a,b$均为正数,记$m=\max\left\{\dfrac{a^2+b^2}{b},\dfrac{1}{a}\right\}$中较大的,求$m$的最小值.
kuing 2# 2013-1-20 17:56
若$a,b$均为正数,记$m=max\{\dfrac{a^2+b^2}{b},\dfrac{1}{a}\}$中较大的,求$m$的最小值. isea 发表于 2013-1-20 17:43 $m=\max\left\{\dfrac{a^2+b^2}b,\dfrac1a\right\}$    m=\max\left\{\dfrac{a^2+b^2}b,\dfrac1a\right\} \[2m\geqslant \frac{a^2+b^2}b+\frac1a\geqslant2a+\frac1a\geqslant2\sqrt{2}.\] 或者 \[m^2\geqslant\frac{a^2+b^2}{ab}\geqslant2.\]
isea 3# 2013-1-20 18:09
本帖最后由 isea 于 2013-1-20 18:13 编辑 我叩,原来属于不等式类啊,我还以为是要用函数,讨论之,但两个字母,就傻了。 原来这样子被你秒了
yes94 4# 2013-1-20 18:42
这叫均值法,常用算数均值或几何均值,
isea 5# 2013-1-20 19:00
4# yes94 主要以前碰到全是一元的,基本全用图象解决,如 $\min\left\{x+1,2-x,2x-1\right\}$的最大值为_______. 这样的,从未想过不等式……
yes94 6# 2013-1-20 21:53
本帖最后由 yes94 于 2013-1-20 21:56 编辑 主要以前碰到全是一元的,基本全用图象解决,如 $\min\left\{x+1,2-x,2x-1\right\}$的最大值为_______. 这样的,从未想过不等式…… isea 发表于 2013-1-20 19:00 图像也可以搞,下面用加权均值: 设$m$=$\min\left\{x+1,2-x,2x-1\right\}$,则$m\leqslant 2-x$,$m\leqslant 2x-1$, 以上两式相加可得$m\leqslant x+1$, 于是只需要$m\leqslant 2-x$,$m\leqslant 2x-1$这两式子即可 显然$2m\leqslant 2(2-x)$,$m\leqslant 2x-1$,故 $3m\leqslant 2(2-x)+(2x-1)=3$, 所以,$m\leqslant 1$,即$\min\left\{x+1,2-x,2x-1\right\}$的最大值为$1$ 当且仅当$2-x=2x-1$,即$x=1$取等号。 输入很麻烦,慢慢来
yayaweha 7# 2013-4-24 22:50
这个除了画图,有其他方法吗? _____kuing edit in $\LaTeX$_____ 求 $\max\{\min\{x+1,x^2-x+1,-x+6\}\}$
第一章 8# 2013-4-24 22:54
1# isea 印象中人教版4-5出现过这个练习题。跟k的做法一样。
kuing 9# 2013-4-25 00:22
这个除了画图,有其他方法吗? _____kuing edit in $\LaTeX$_____ 求 $\max\{\min\{x+1,x^2-x+1,-x+6\}\}$ yayaweha 发表于 2013-4-24 22:50 这是一个坑 die 的解法: \[2\min\{x+1,x^2-x+1,-x+6\} \leqslant x+1+(-x+6)=7,\] 得到 \[\max\bigl\{\min\{x+1,x^2-x+1,-x+6\}\bigr\}\leqslant\frac72,\] 当 $x=5/2$ 时取等,故 $\max\bigl\{\min\{x+1,x^2-x+1,-x+6\}\bigr\}=7/2$。
yes94 10# 2013-4-25 12:28
9# kuing 权值为0
yayaweha 11# 2013-4-25 22:13
10# yes94 权值为零什么意思
yayaweha 12# 2013-4-25 22:14
9# kuing 为什么不用考虑那个二次函数
yes94 13# 2013-4-26 00:06
12# yayaweha 可以先从几何图像上获得答案,再用代数证明
kuing 14# 2013-4-26 00:16
所以坑die。。
yes94 15# 2013-4-26 00:20
14# kuing
thread-1068-1-5.html: [函数] 问题2:应该能和函数挂勾……
isea 1# 2013-1-20 17:49
本帖最后由 isea 于 2013-1-20 18:41 编辑 若$m,n$均为实数,且$m^3-3m^2+5m=1,n^3-3n^2+5n=5$,求$m+n$的值为_______. 强解如下,记$m+n=a,mn=b$, 将已知两式相加: $m^3+n^3-3(m^2+n^2)+5(m+n)=6\\ a(a^2-3b)-3(a^2-2b)+5a-6=0\\ a^3-3a^2+5a-6-3ab+6b=0\\ $ 强制分解因式: $a^3-3a^2+5a-6-3ab+6b=0\\ (a^3-2a^2)-(a^2-2a)+(3a-6)-(3ab-6b)=0\\ (a-2)(a^2-a+3-3b)=0\\ $ 再将$m+n=a,mn=b$反代回$a^2-a+3-3b$,配方说明此式不为零,从而$a=2$. 这方法太麻烦,看看大家有没好办法。
kuing 2# 2013-1-20 17:59
怎么中间会有全角括号?
isea 3# 2013-1-20 18:07
2# kuing 这都看出来了,厉害,极点中文下输入的,回车上编码形成的
kuing 4# 2013-1-20 18:12
http://bbs.pep.com.cn/thread-334853-1-1.html http://bbs.pep.com.cn/thread-388644-1-1.html http://bbs.pep.com.cn/thread-423694-1-1.html http://bbs.pep.com.cn/thread-500410-1-1.html http://bbs.pep.com.cn/thread-520012-1-1.html http://bbs.pep.com.cn/thread-303211-1-1.html
isea 5# 2013-1-20 18:14
4# kuing 果然是陈得不行的老题,在一张高三模拟卷上看到了。 多谢k
isea 6# 2013-1-20 18:51
本帖最后由 isea 于 2013-1-20 18:54 编辑 参考以上链接解法 其实就是说三次函数是中心对称的(就像二次函数是轴对称一样)。 设$f(x)=x^3-3x^2+5x$, 条件变为$f(m)=1,f(n)=5$. 对于$f(x)=x^3-3x^2+5x$有$f''(x)=6x-6$, 令$f''(x)=0$,有$x=1$, 从而$f(1)=1-3+5=3$, 即$(1,3)$为$f(x)=x^3-3x^2+5x$对称中心,从而 $\dfrac{f(2-m)+f(m)}2=3\\ f(2-m)=6-f(m)=5=f(n)\\ $ 又易得$f'(x)>0$,即$f(x)$在$R$上单调函数,于是 $2-m=n\\ m+n=2\\ $ 用点对称,痛快,痛快……
kuing 7# 2013-1-20 20:19
即将要出的第11期数学空间有提及此题……
yes94 8# 2013-1-20 22:07
即将要出的第11期数学空间有提及此题…… kuing 发表于 2013-1-20 20:19 期盼……
thread-1069-1-5.html: [几何] 来自人教群的椭圆内接过焦点平行四边形面积
kuing 1# 2013-1-21 15:17
如图,将椭圆拉伸成圆,则平行四边形变成矩形,两焦点位置不变,矩形仍然要过该两点。 设原来平行四边形的面积为 $S$,拉伸后变成的矩形的面积为 $S'$,则 $S:S' = b:a$。 注意到 $A'B'$ 的取值范围是 $[2b,2a)$,所以: (1)若 $2b\leqslant \sqrt2a$,则 $A'B'$ 在变化时能使矩形达到正方形,而圆内接四边形中以正方形面积最大,所以此时 $S'$ 最大值为 $2a^2$; (2)若 $2b>\sqrt2a$,则由 $S'=A'B'\cdot\sqrt{(2a)^2-(A'B')^2}$,可知此时 $S'$ 关于 $A'B'$ 递减,于是 $S'$ 的最大值为 $2b\cdot\sqrt{(2a)^2-(2b)^2}$,即 $4bc$。 还原为椭圆的情形,即:若 $e\geqslant1/\sqrt2$,则 $S_{\max}=2ab$;若 $e<1/\sqrt2$,则 $S_{\max}=4b^2e$。
kuing 2# 2013-1-21 16:46
爱好者-何万程(1785***)  16:43:58 那个椭圆问题,设过焦点的一边与椭圆相交于点(x1,y1),(x2,y2),那么四边形面积就是2c|y1-y2|,设直线方程y=k(x-c),后面用韦达定理很容易了
isea 3# 2013-1-22 23:33
本帖最后由 isea 于 2013-1-22 23:34 编辑 想一块去了,不过,我是从对角线出发的,在圆的情况下,显然对角线垂直时…… 我转过去链接 == 原来还有推广,厉害
thread-107-1-1.html: [数列] 来自群的1,2,2,3,3,3,4,4,4,4,...构造通项
kuing 1# 2011-10-17 17:15
教师-槑(8355***)  16:49:18 求通项公式: 1,2,2,3,3,3,4,4,4,4,5,5,5,5,5,…… 答: \[a_{n}=\left\lceil \frac{\sqrt{8n+1}-1}{2} \right\rceil .\] 想法很简单,构造一个函数,使得对应每串相同数的最后一个位置,而且单增,然后再向上取整就行了。 具体地,记最后一个 $k$ 在该数列中的第 $f(k)$ 项中,那么 $f(1)=1$,$f(2)=3$,$f(3)=6$ 等等,容易求出 \[f(k)=\frac{k(k+1)}{2},\] 故我们要构造的是使 $a_{k(k+1)/2}=k$ 的单增函数(数列),令 $n=k(k+1)/2$,反解出 \[k=\frac{\pm\sqrt{8n+1}-1}{2},\] 显然应取正者,然后向上取整,即得 \[a_{n}=\left\lceil \frac{\sqrt{8n+1}-1}{2} \right\rceil .\]
kuing 2# 2011-10-17 17:18
类似地,构造对应于数字串的第一个位置,然后向下取整也可以,得到另一表达式。 记第一个 $k$ 在该数列中的第 $g(k)$ 项中,那么 $g(1)=1$,$g(2)=2$,$g(3)=4$,$g(4)=7$ 等等,容易求出 \[g(k)=\frac{k(k-1)}{2}+1,\] 故我们要构造的是使 $b_{k(k-1)/2+1}=k$ 的单增函数(数列),令 $n=k(k-1)/2+1$,反解出 \[k=\frac{\pm\sqrt{8n-7}+1}{2},\] 显然应取正者,然后向下取整,即得 \[b_{n}=\left\lfloor \frac{\sqrt{8n-7}+1}{2} \right\rfloor .\]
①②③④⑤⑥⑦ 3# 2011-10-18 08:58
本帖最后由 ①②③④⑤⑥⑦ 于 2011-10-18 08:59 编辑 嗯,这玩意儿就在 $\sqrt{2n}$ 附近飘 \[a_n=Round(\sqrt{2n})=Round(\sqrt{2n-1})\] http://oeis.org/A002024
kuing 4# 2011-10-18 11:43
3# ①②③④⑤⑥⑦ 呃,Round 是什么 四舍五入? 还是五舍六入?
①②③④⑤⑥⑦ 5# 2011-10-18 13:07
4# kuing 四舍五入 有“五舎六入”?我咋没听说捏?只知道“四舎六入五单双”,其实是超过一半的入,不到一半的舍,正好一半的靠偶数,也就是恰好0.5才看单双,整数开根不会恰好是0.5的小数,这里的Round是四舍五入还是五单双都没啥区别
kuing 6# 2011-10-18 13:16
5# ①②③④⑤⑥⑦ 呃,其实我意思也就是指0.5是入还是不入,所以才随口说了五舍六入
①②③④⑤⑥⑦ 7# 2011-10-18 13:31
6# kuing 哦,那么这里没区别的,写Round因为印象中没有什么比较通用的符号,切换为取整不如直接用Round爽快 Excel、几何画板等的Round是四舍五入 Mathematica、VB等的Round,0.5是向偶数靠的
kuing 8# 2011-10-18 13:39
7# ①②③④⑤⑥⑦ 嗯,在Mathematica7里试了试的确如此 In[1]:= Table[Round[x + 0.5], {x, -6, 6}] Out[1]= {-6, -4, -4, -2, -2, 0, 0, 2, 2, 4, 4, 6, 6}
海盗船长 9# 2011-10-20 14:19
用Round函数表达式好简单
李斌斌755 10# 2013-5-6 20:39
这这这……解法!
huamahu 11# 2013-5-8 15:00
这解法太厉害了。
thread-1070-1-4.html: [不等式] 不等式菜鸟提问
guanmo 1# 2013-1-21 23:46
附件中的解法构造的过程是怎样的(分母16的“来历”)?
yes94 2# 2013-1-21 23:50
可以待定系数法,设那个16为t(做此题之前不知道t=16)。 再根据取等条件,结合已知条件得到方程组, 从而得到系数t=16
yes94 3# 2013-1-22 00:00
柯西:
isea 4# 2013-1-22 00:18
本帖最后由 isea 于 2013-1-22 00:21 编辑 凑配能力要求高,这样子 动笔算了下,消元直接代入求式,变成函数,巧的是通分化简后分子是2次,最直接判别式法就求出最小值$\dfrac14$ 若,直接求导,会出现4次方,能知道是导函数是单调的,但求零点卡住 =========== 楼上的亲切多了,虽然我看不懂,哈哈 据说柯西不等可以用向量证明,不知能不能构造向量证,哈哈
kuing 5# 2013-1-22 00:21
4# isea 相通的东西一定可以。
isea 6# 2013-1-22 00:47
本帖最后由 isea 于 2013-1-22 01:23 编辑 5# kuing 终于看懂yes94在3楼式子的意思了, 首先,向量不等式 \begin{align*} \vv m\cdot\vv n\le|m||n|\\ \end{align*} 成立是非常明显的。 令 \begin{align*}\vv m&=(\sqrt {x+2},\sqrt {y+1}),\\\vv n&=(\dfrac{x}{\sqrt{x+2}},\dfrac{y}{\sqrt{y+1}})\end{align*} 于是 \begin{align*} \vv m \cdot \vv n&=(\sqrt {x+2},\sqrt {y+1})\cdot(\dfrac{x}{\sqrt{x+2}},\dfrac{y}{\sqrt{y+1}})\\ &=x+y\\ |m||n|&=\sqrt {x+2+y+1}\sqrt{\dfrac{x^2}{x+2}+\dfrac{y^2}{y+1}}\\ \vv m\cdot\vv n&\le|m||n|\\ x+y&\le\sqrt {x+2+y+1}\sqrt{\dfrac{x^2}{x+2}+\dfrac{y^2}{y+1}}\\ (x+y)^2&\le (x+2+y+1)(\dfrac{x^2}{x+2}+\dfrac{y^2}{y+1})\\ \dfrac{(x+y)^2}{x+2+y+1}&\le \dfrac{x^2}{x+2}+\dfrac{y^2}{y+1}\\ \end{align*} 不等式的确“高深”,凑配一个柯西形式不知要废了多少脑细胞…… 收工,睡觉
kuing 7# 2013-1-22 01:39
6# isea 最后一行公式后面就不必再\\换行了
realnumber 8# 2013-1-22 08:45
其实和上面相同, 设$x+2=p,y+1=q$,问题就为$p+q=4,(p>2,q>1)$\[\frac{(p-2)^2}{p}+\frac{(q-1)^2}{q}=\frac{4}{p}+\frac{1}{q}-2=\frac{p+q}{4}(\frac{4}{p}+\frac{1}{q})-2\] 后面略.
yes94 9# 2013-1-22 13:18
本帖最后由 yes94 于 2013-1-22 13:23 编辑 还有人构造概率的期望和方差来证明的: $D(ξ)=E(ξ^2)-E^2(ξ)\ge0$
yes94 10# 2013-1-22 13:18
本帖最后由 yes94 于 2013-1-22 13:23 编辑 还有人构造概率的期望和方差来证明的: $D(ξ)=E(ξ^2)-E^2(ξ)\geqslant0$ 怎么发表的时候出现“未定义操作”,以为没发成功,再点了一次查看,却已发现发了两次, 遂将$\ge0$,改为$\geqslant0$,看一下效果如何
realnumber 11# 2013-1-22 13:34
10# yes94 你的意思是用来证明1楼?好象2个没什么关联....,
yes94 12# 2013-1-22 13:39
10# yes94 你的意思是用来证明1楼?好象2个没什么关联...., realnumber 发表于 2013-1-22 13:34 对,那个方差不等式可以用来证明本题,奈何打代码不是我强项,只有改天再打代码试一试。 另外还有人喜欢用,点到直线的距离,或者构造非负二次函数,判别式来证明,等等方法 实质就是柯西不等式的不同证法而已
kuing 13# 2013-1-22 13:39
还有人构造概率的期望和方差来证明的: $D(ξ)=E(ξ^2)-E^2(ξ)\geqslant0$ 怎么发表的时候出现“未定义操作”,以为没发成功,再点了一次查看,却已发现发了两次, 遂将$\ge0$,改为$\geqslant0$,看一下效果如何 ... yes94 发表于 2013-1-22 13:18 ξ 用 \xi 所有希腊字母都可以用代码打,不需要切换输入法
yes94 14# 2013-1-22 19:16
ξ 用 \xi 所有希腊字母都可以用代码打,不需要切换输入法 kuing 发表于 2013-1-22 13:39 谢谢! 找了下你置顶的例句里没有ξ,只有其他的希腊字母,最后跑到word里的符号里复制的,
kuing 15# 2013-1-22 19:36
14# yes94 oh,我去补充一下。
isea 16# 2013-1-22 21:26
15# kuing 别理yes94兄,这个是常识,其实,无视他,气气yes94
kuing 17# 2013-1-22 21:46
16# isea
yayaweha 18# 2013-1-25 17:41
借贴的名字问个问题,韩京俊那本《初等不等式的证明方法》第10页,3大于等于a+b+1/a^2+b^2+1求和 等价于 3大于等于a^3+b^3+1/a^6+b^6+1 怎么等价的,还有后面那个a^3+b^3大于等于ab(a+b)是怎么得来的?
yes94 19# 2013-1-25 18:28
借贴的名字问个问题,韩京俊那本《初等不等式的证明方法》第10页,$3\ge a+b+1/a^2+b^2+1$求和 等价于 $3\ge a^3+b^3+1/a^6+b^6+1 $怎么等价的,还有后面那个$a^3+b^3\ge ab(a+b)$是怎么得来的? yayaweha 发表于 2013-1-25 17:41 $a$、$b$的条件?正数? 那个$a^3+b^3\ge ab(a+b)$,当$a$、$b$是正数时, $a^3+b^3=(a+b)(a^2+b^2-ab)\ge (a+b)(2ab-ab)=ab(a+b)$
第一章 20# 2013-1-25 19:54
扬州市第一学期期中考,就有楼主的题目。 PS,方差模型怎么弄?考虑哪两个数的方差?
thread-1070-2-4.html:
yes94 21# 2013-1-25 20:00
扬州市第一学期期中考,就有楼主的题目。 PS,方差模型怎么弄?考虑哪两个数的方差? 第一章 发表于 2013-1-25 19:54 很不好输入,就算啦,看看刊物
第一章 22# 2013-1-25 21:06
晕死。我只想知道构造哪两个数的方差。不方便打字,方便上个图不? 另外,问问K,怎么我每次开电脑,上这个论坛,都得重新登录?
kuing 23# 2013-1-25 21:10
另外,问问K,怎么我每次开电脑,上这个论坛,都得重新登录? 第一章 发表于 2013-1-25 21:06 可能是你浏览器的cookies问题?我一般登录一次就能维持好久。
yayaweha 24# 2013-1-25 22:41
19# yes94 条件是abc=1
kuing 25# 2013-1-25 23:14
借贴的名字问个问题,韩京俊那本《初等不等式的证明方法》第10页,3大于等于a+b+1/a^2+b^2+1求和 等价于 3大于等于a^3+b^3+1/a^6+b^6+1 怎么等价的,还有后面那个a^3+b^3大于等于ab(a+b)是怎么得来的? yayaweha 发表于 2013-1-25 17:41 这就跟昨晚的这个 http://kkkkuingggg.5d6d.net/thread-1083-1-1.html 的刚开始那里一样
yes94 26# 2013-1-26 16:11
这就跟昨晚的这个 http://kkkkuingggg.5d6d.net/thread-1083-1-1.html 的刚开始那里一样 kuing 发表于 2013-1-25 23:14 就是,
thread-1071-1-1.html: 公式里字体更黑了
realnumber 1# 2013-1-22 10:28
看得更舒服了,以前确实有些淡
kuing 2# 2013-1-22 10:40
其实我没作任何改变, 应该是你的电脑有所改变?
realnumber 3# 2013-1-22 11:52
2# kuing 是的,又回到原状了,总觉得公式比普通的汉字,颜色淡了点,也许是我的电脑缘故.
kuing 4# 2013-1-22 12:05
3# realnumber 截几个图来瞧瞧
thread-1072-1-2.html: 5d6d
isea 1# 2013-1-22 14:12
这域名有什么特别意思么? 免费服务器还可以,表扬一下……
kuing 2# 2013-1-22 14:18
我的领地
thread-1073-1-5.html: [不等式] 再来一道不等式
guanmo 1# 2013-1-22 14:18
如附件,不用三角代换(正切),如何解决?
kuing 2# 2013-1-22 14:20
就在本论坛写过:http://kkkkuingggg.5d6d.net/thread-860-1-1.html
yes94 3# 2013-1-23 21:44
还可以柯西不等式放缩成关于c的函数
thread-1074-1-4.html: [几何] 三角形周长最大
reny 1# 2013-1-22 16:03
本帖最后由 reny 于 2013-1-23 19:58 编辑 已知椭圆方程:$\frac{x^2}{a^2}+\frac{y^2}{b^2}=1(a>b>0,且a,b为常数),左焦点为F_1,$直线方程: $y=kx+m(k\ge0),与椭圆交于A、B两点,分别固定k和m,△F_1 AB的周长什么时候最大?$ 这是一个一般性问题,似乎不好算. PS、什么时候$\color{green}{面积}$最大,仅从图像可能就做不出来了吧,估计真的有点复杂。
realnumber 2# 2013-1-22 16:07
1# reny 固定了k、m,那么看作a,b的函数?a,b趋于无穷,周长也应该是无上限的.
reny 3# 2013-1-22 16:11
2# realnumber a,b当然是常数呗
kuing 4# 2013-1-22 16:13
他说“分别固定k和m”,意思可能是:固定 m 的时候 k 取什么是周长最大,固定 k 的时候 m 取什么是周长最大?
reny 5# 2013-1-22 16:46
4# kuing 就是这个意思!
kuing 6# 2013-1-22 16:55
这样看来几何方法大概没机会了,用最基本的代数方法设直线联立用上韦达定理什么的估计也不太容易……我想潜水……
yes94 7# 2013-1-22 19:25
这样看来几何方法大概没机会了,用最基本的代数方法设直线联立用上韦达定理什么的估计也不太容易……我想潜水…… kuing 发表于 2013-1-22 16:55 k版不想做这种体力活的题目,只想用巧妙地解法,例如用性质,第一、第二定义、几何方法等等。 还有就是并不是所有题都是可解的,当可解时,并不一定答案都是漂亮的。 平时的所有题目都是可解的,当可解时,答案都是简单、好看的,并不是一个繁杂的结果。
isea 8# 2013-1-22 21:31
退一步,把椭圆看成圆,想像一下动态过程,要么无解,要么表达起来也麻烦
yes94 9# 2013-1-22 23:39
退一步,把椭圆看成圆,想像一下动态过程,要么无解,要么表达起来也麻烦 isea 发表于 2013-1-22 21:31 你的意思也就和7楼说的差不离
hnsredfox_007 10# 2013-1-23 08:42

realnumber 11# 2013-1-23 08:53
10# hnsredfox_007
hnsredfox_007 12# 2013-1-23 08:57
11# realnumber
kuing 13# 2013-1-23 10:06
10# hnsredfox_007 OMG! 有道理,原来那么简单。 这回老猫烧须了我
reny 14# 2013-1-23 11:19
10# hnsredfox_007 漂亮!
hnsredfox_007 15# 2013-1-23 11:23
其实如果你们画图的话,肯定比我先解出来,只是大家可能先入为主没动手罢了。
kuing 16# 2013-1-23 13:47
15# hnsredfox_007 的确没有画图, 也没怎么细想, 就潜水了,呵呵…
yes94 17# 2013-1-23 15:25
已知椭圆方程:$\frac{x^2}{a^2}+\frac{y^2}{b^2}=1(a>b>0,且a,b为常数),左焦点为F_1,$直线方程: $y=kx+m(k\ge0),与椭圆交于A、B两点,分别固定k和m,△F_1 AB的周长什么时候最大?$ 这是一个一般性问题,似乎不 ... reny 发表于 2013-1-22 16:03 首先敬佩火狐的功力深厚,没有人云亦云, 其次,还是有些遗憾,当$k=0$时,就无法取得等号,因为不构成三角形。 不过这已经无碍大局了,犹如宝玉里面的瑕疵算的了什么?    再次,有的随便提的问题不可解,有的问题解出来不是漂亮的结果,应该是对的。    只是恰巧本题运气太好了!似乎是给我的一个反例。    现在想了想楼主的问题是怎么得来的? 估计是从一道习题中来的灵感?那么就是,我说的,平时的习题的答案都基本是完美的! “平时的所有题目都是可解的,当可解时,答案都是简单、好看的,并不是一个繁杂的结果。 ”
reny 18# 2013-1-23 15:52
17# yes94 此题的来历是2012年四川卷的一个填空题 的确也是,$k=0时,周长的范围是(2a,4a)$,取不到边界。
yes94 19# 2013-1-23 16:00
17# yes94 此题的来历是2012年四川卷的一个填空题 的确也是,$k=0时,周长的范围是(2a,4a)$,取不到边界。 reny 发表于 2013-1-23 15:52 楼主的提问很有价值,这种深入的思考是很宝贵的, 不是那种随随便便就提问的那种,
isea 20# 2013-1-23 20:18
本帖最后由 isea 于 2013-1-23 20:21 编辑 看到这个结果,我突然想起一事,就是 若(点)光线从一个焦点出发经椭圆反射后(第一次)回后这个焦点, 请问:光走的路线是最长的还是最短的? 哈哈
thread-1074-2-4.html:
isea 21# 2013-1-23 20:27
继续,光学性质。 若双曲线与椭圆共焦点,光从一个(如左)焦点出发,经椭圆,双曲线反射,会回到这个焦点么? 若能,第一次回到这个焦点时,路程是多少?
realnumber 22# 2013-1-23 21:32
20# isea 与其它什么在比较长短?被椭圆反射不都是定值吗?
realnumber 23# 2013-1-23 21:33
21# isea 不是很明白,最好配个图形
yes94 24# 2013-1-23 21:37
21# isea 不是很明白,最好配个图形 realnumber 发表于 2013-1-23 21:33 她没叙述好,但是知道他的意思。 光线反射已经是确定的路线,那个路程是定值,谈最小值已经失去意义了。
isea 25# 2013-1-23 22:29
本帖最后由 isea 于 2013-1-23 22:35 编辑 10楼的结果是和光行路径一模一样的,10楼的结果是最大,而按光得最速原理,此路径应该是最小的。 矛盾 所以,我就哈哈笑了 这个哈哈一乐,并不否定主楼与10楼,就这是20楼的要说的意思。 ==== 而21楼,有椭圆有双曲的是这样的,(示意图),
yes94 26# 2013-1-23 22:54
10楼的结果是和光行路径一模一样的,10楼的结果是最大,而按光得最速原理,此路径应该是最小的。 矛盾 所以,我就哈哈笑了 这个哈哈一乐,并不否定主楼与10楼,就这是20楼的要说的意思。 ==== 而21 ... isea 发表于 2013-1-23 22:29 顺便说下,$k=0$或$m=0$均取不到最小值,因为无法构成三角形,只成为下确界。 你的那个20楼的题目,我给你发一个类似的题目来给你参考,(不是在考你的哈,你不会在论坛说气我吧? )
kuing 27# 2013-1-24 10:05
她没叙述好,但是知道他的意思。 光线反射已经是确定的路线,那个路程是定值,谈最小值已经失去意义了。 yes94 发表于 2013-1-23 21:37 她?他?
kuing 28# 2013-1-24 10:10
10楼的结果是和光行路径一模一样的,10楼的结果是最大,而按光得最速原理,此路径应该是最小的。 矛盾 所以,我就哈哈笑了 这个哈哈一乐,并不否定主楼与10楼,就这是20楼的要说的意思。 ==== 而21 ... isea 发表于 2013-1-23 22:29 那个什么光程原理说什么路径最短其实是一种误解,参考: http://zh.wikipedia.org/wiki/%E8%B4%B9%E9%A9%AC%E5%8E%9F%E7%90%86
kuing 29# 2013-1-24 10:12
你的那个20楼的题目,我给你发一个类似的题目来给你参考,(不是在考你的哈,你不会在论坛说气我吧? ) yes94 发表于 2013-1-23 22:54 OMG,真有这样的题目……
yes94 30# 2013-1-24 15:52
OMG,真有这样的题目…… kuing 发表于 2013-1-24 10:12
reny 31# 2013-1-24 20:47
1# reny 关于面积最值问题,的确复杂。仅固定k时,结果就不简洁.
yes94 32# 2013-1-24 21:08
31# reny 的确不够简洁,但是将$k$取适当的具体常数,将会使式子变得简洁些,所以如果出你的那个面积题,一定要给出具体数字,否则结果不漂亮 如果不给出$k$的具体数字,那么结果通常都不是漂亮的(也可能减少抽象,减轻学生负担)
thread-1075-1-1.html: [函数] 使函数$f(x)$在$[a,b]$上的值域为$[ka,kb]$
realnumber 1# 2013-1-22 17:23
本帖最后由 realnumber 于 2013-1-22 17:25 编辑 设$f(x)$是定义在$D$上的函数。若存在区间$[a,b]$是$D$的子集,使函数$f(x)$在$[a,b]$上的值域为$[ka,kb]$,   则称函数f(x)是k类函数。设函数$f(x)=x^3+2x^2+x$($x\le0$)是$k$类函数,则$k$的最小值是_____ http://zhidao.baidu.com/question/462247481.html
kuing 2# 2013-1-22 17:30
现在时兴这类题?像 http://kkkkuingggg.5d6d.net/thread-922-1-1.html
kuing 3# 2013-1-22 17:31
不过看上去1#的比2#的简单
reny 4# 2013-1-22 18:23
图像就是这样,要分好几种情况讨论吧。
yes94 5# 2013-1-22 19:17
用$y=kx$试一试
reny 6# 2013-1-22 19:36
4# reny 这个答案是$\frac1 9$吗?做了一下,有错误的话,希望指出一下。
realnumber 7# 2013-1-22 20:51
本帖最后由 realnumber 于 2013-1-23 21:37 编辑 6# reny 怎么简化呢?只简化了小部分,只好明天继续了---没完成,放弃...
yes94 8# 2013-1-23 22:02
用$y=kx$试一试 yes94 发表于 2013-1-22 19:17 用$y=kx$试一试,从图可以看出,$a=-\dfrac43$,$b=0$,依据那个关键的极小值点$x=-\dfrac13$,得到极小值$y=-\dfrac4{27}$,故得到$k=\dfrac19$,此时k得到最小值$k=\dfrac19$
realnumber 9# 2013-1-24 14:20
8# yes94 没明白,即使..,也觉得不严密.
yes94 10# 2013-1-24 15:46
8# yes94 没明白,即使..,也觉得不严密. realnumber 发表于 2013-1-24 14:20 我的意思是用这种方法思考,换一种方法书写。 很多时候思考和书写可以不一样。 例如先猜后证,就反映了思考的过程和书写完全不一样。
kuing 11# 2013-1-25 20:11
现在时兴这类题? kuing 发表于 2013-1-22 17:30 看来真是一种潮流,刚才又看到这种题
yes94 12# 2013-1-25 20:21
看来真是一种潮流,刚才又看到这种题 787 kuing 发表于 2013-1-25 20:11 考虑$y=x$,试试看
kuing 13# 2013-1-25 20:30
12# yes94 我倒没什么兴趣做……
yes94 14# 2013-1-25 20:35
12# yes94 我倒没什么兴趣做…… kuing 发表于 2013-1-25 20:30 我也不想,看图就知道方法了,这类题见得太多了
kuing 15# 2013-2-15 12:22
看来真是一种潮流,刚才又看到这种题 kuing 发表于 2013-1-25 20:11 又来了
yes94 16# 2013-2-15 13:30
15# kuing 又来了,那就归类在此帖,作为资料也好,
kuing 17# 2013-4-8 16:04
继续
hnsredfox_007 18# 2013-4-8 16:38
本帖最后由 hnsredfox_007 于 2013-4-8 21:07 编辑 17# kuing $分析:若存在,则必满足\begin{cases}f(m)&=\log_am=n^2\\f(n)&=\log_an=m^2\end{cases} \Longrightarrow \begin{cases}m=a^{n^2}\\ \log_an=\left(a^2\right)^{n^2}=\log_{a^2}n^2\end{cases}$ $表明m^2,n^2是方程$$\log_bx=b^x\left(b=a^2\right)的两个根. 余下过程见数学空间2011年第7期。$
kuing 19# 2013-4-8 16:44
18# hnsredfox_007 应该没问题,这样的话,根据已知结论,只能 $m=n$ 了……这样有点坑……
hnsredfox_007 20# 2013-4-8 17:04
19# kuing 不是吧 应该是有3个公共点那个结论吧
thread-1075-2-1.html:
kuing 21# 2013-4-8 17:14
20# hnsredfox_007 哦,对喔,可以三选二的我发傻了……居然想成了等价于有且仅有两交点,于是把三个交点的情形排除在外
AAAAA 22# 2013-5-29 15:39
yes94的思路很好!其实可以直接拿两个函数图象相交,求临界值!
yes94 23# 2013-5-29 21:26
四个A!炸弹!
kuing 24# 2013-5-29 21:41
明明是五个……
yes94 25# 2013-5-29 21:44
24# kuing 老眼昏花!
isea 26# 2013-5-29 23:01
24# kuing boom 后,A~一声~
thread-1076-1-4.html: [组合] 5人排队,3男2女,要求甲女不在首,乙丙两男生不相邻
isea 1# 2013-1-22 18:48
5人排队,3男2女,要求甲女不在首,乙丙2男生不相邻 共有多少有排法 确认一下,是60种么,最后结果,被弄晕了
kuing 2# 2013-1-22 18:53
题外话:这类题最好不要用男和女……以免……
isea 3# 2013-1-22 19:04
2# kuing 哎呀,数学排队而已 别想太多
kuing 4# 2013-1-22 19:24
3# isea 问题是经常要kunbangcharu……你们上课都没避免下么……
yes94 5# 2013-1-22 19:29
实际学生要笑的,尤其男生,而且有时候男生们在课堂也大笑,但老师常常假装不知道,过一段时间学生就平息了
kuing 6# 2013-1-22 19:55
其实这题似乎不必分男和女?直接相当于甲不在首,乙丙不相邻就行了吧
kuing 7# 2013-1-22 20:00
全部:5! 甲在首:4! 乙丙相邻:2*4! 甲在首且乙丙相邻:2*3! 所以所求为 5!-4!-2*4!+2*3!=60
isea 8# 2013-1-22 21:24
其实这题似乎不必分男和女?直接相当于甲不在首,乙丙不相邻就行了吧 kuing 发表于 2013-1-22 19:55 我随口说的,非正式,题目不够通畅 结果已经有了,就是60了。 来来接着歪楼,男学生女学生男女学生生男女……
yes94 9# 2013-1-22 23:27
我随口说的,非正式,题目不够通畅 结果已经有了,就是60了。 来来接着歪楼,男学生女学生男女学生生男女…… isea 发表于 2013-1-22 21:24 你那个是上联, 下联:南通州北通州南北通州通南北,
kuing 10# 2013-1-22 23:52
10# yes94 niubility
依然饭特稀 11# 2013-1-25 23:01
9# yes94 这个题可以这样考虑:先考虑一个约束条件乙丙不相邻有3!A(2,4)再减去甲女在乙丙不相邻时排首位的2!A(2,3)
thread-1077-1-5.html: [函数] 函数$g(x)=\dfrac1x+\ln x$,是否$\exists x_0>0$,使下式成立?
isea 1# 2013-1-22 22:32
本帖最后由 isea 于 2013-1-23 09:31 编辑 对这类问题,如何思考,如何找到切入点?请指点。 题:函数$g(x)=\dfrac1x+\ln x.$ 问:是否$\exists x_0>0$,对$\forall x>0$使得$|g(x)-g(x_0)|<\dfrac1x$成立? ====== 查了下,此题,一个源是,http://edu.people.com.cn/GB/116076/185875/14852237.html 2011年陕西高考数学(理)试题,最后一题最后一问。
realnumber 2# 2013-1-22 22:44
本帖最后由 realnumber 于 2013-1-22 22:59 编辑 去绝对值(分离变量)后是这个 \[\ln(x)<g(x_0)<\ln(x)+\frac{2}{x}\] 对任意$x$都成立,得到 \[max\{\ln(x) \}<g(x_0)<min\{\ln(x)+\frac{2}{x}\}\] 而这是做不到的.
isea 3# 2013-1-22 23:05
本帖最后由 isea 于 2013-1-22 23:19 编辑 去绝对值(分离变量)后是这个 \[\ln(x) realnumber 发表于 2013-1-22 22:44 对呀对呀! == 不过,细细想想,这个有问题,前后两个$x$在不等式里,任意取值时,取值应该是相同的。 这样前后两函数的最大值与最小值并不能同时取到,其次,当$x>1,$函数都单调增加。 似乎这样不可行。 == 这种问题的确很绕啊,再想想,正因为如此,从图象上说,不存在平行于$x$轴的直线在这两函数图象之间,从而直观上说,的确是不存在这样的$g(x_0)$。 很抽象,我理解理解,先
realnumber 4# 2013-1-22 23:13
3# isea (但是$x_0$已经固定住了,)2个不等式可以先看一个,都要成立.应该可以吧
kuing 5# 2013-1-23 00:44
这类题没什么兴趣的说…… PS、1#的减号似乎是全角的。
reny 6# 2013-1-23 00:48
2#中就取$x=e^{x_0}$,左边显然就不行了嘛。
第一章 7# 2013-1-23 08:29
本帖最后由 第一章 于 2013-1-23 08:35 编辑 当年这题的参考答案也确实是无解。 另外,对于这种恒成立的问题,考虑的是x取值的全体,只能说g(x_0)比左边全部要大,比右边全部要小。
都市侠影 8# 2013-1-23 09:56
这个还用看吗,你既然要对于任意 $x$ 都成立,那就当 $x\to\infty$ 来看,$g(x_0)$ 就必须是 $g(x)$ 在正无穷处的极限,显然这个极限是不存在的。
第一章 9# 2013-1-23 10:19
8# 都市侠影 确实是这样。不过高中混得久了,连极限都忘记了。 不过话说回来,用初等的方法说明无解也是很简单的
thread-1078-1-5.html: [不等式] 18次齐次化
realnumber 1# 2013-1-23 09:15
其实我也不太懂,纯粹是围观看热闹.18次冲击太大...
kuing 2# 2013-1-23 11:02
证明见《数学空间》2011 第 3 期 P29~30,例 3.2.4 : http://www.pep.com.cn/rjwk/gzsxs ... 0110516_1041458.htm 类似的几个见 http://kkkkuingggg.5d6d.net/view ... &page=1#pid5748 8#
thread-1079-1-4.html: [不等式] 一个循环不等式
reny 1# 2013-1-23 11:26
已知 $a$, $b$, $c$, $d$ 为正实数,证明 \begin{align*} (1)& \left(\frac a b+\frac b c+\frac c a\right)\left(\frac b a+\frac c b+\frac a c\right)\ge(a+b+c)\left(\frac 1 a+\frac 1 b+\frac 1 c\right);\\ (2)& \left(\frac a b+\frac b c+\frac c d+\frac d a\right)\left(\frac b a+\frac c b+\frac d c+\frac a d\right)\ge(a+b+c+d)\left(\frac 1 a+\frac 1 b+\frac 1 c+\frac 1 d\right). \end{align*}
goft 2# 2013-1-23 12:29
两次柯西就搞定了
reny 3# 2013-1-23 12:37
2# goft 怎么两次用cauchy,能详细点吗
goft 4# 2013-1-23 12:53
本帖最后由 goft 于 2013-1-23 15:11 编辑 太粗心了……
goft 5# 2013-1-23 13:08
晕搞错柯西了 另法: 设原式为AB>=CD 等价于AB>=3+A+B 等价于(A-2)(B-2)>=4 而A>=3,B>=3
reny 6# 2013-1-23 14:26
5# goft $AB\ge3+A+B等价于(A-2)(B-2)\ge4 ?$
goft 7# 2013-1-23 15:10
5# goft $AB\ge3+A+B等价于(A-2)(B-2)\ge4 ?$ reny 发表于 2013-1-23 14:26 又打错,$AB\ge3+A+B等价于(A-1)(B-1)\ge4 $
reny 8# 2013-1-23 16:15
7# goft 四元的话,这种方法,似乎不行吧。
kuing 9# 2013-1-24 10:17
昨天早上看明明是三元的,啥时候成了四元…… 提个建议,顶楼加题目的话最好不要把原来的题删掉,除非还没人回贴,不然前面几楼回的是原来的题,删掉了的话别人看贴就会莫名其妙了。 PS、三元的情况之前在这里也扯过http://kkkkuingggg.5d6d.net/thread-957-1-1.html楼主也回过贴给了加强。
reny 10# 2013-1-24 11:18
9# kuing 哦,这个贴原来见过,久了忘了。四元的应该就复杂很多吧。能不能推广到n元呢?
kuing 11# 2013-1-24 11:37
10# reny 四元也不是很复杂,等会写写,N元可能很难,5元我都没什么头绪。 PS、改了一下1#的输入。
kuing 12# 2013-1-24 12:14
\[(2)\left(\frac a b+\frac b c+\frac c d+\frac d a\right)\left(\frac b a+\frac c b+\frac d c+\frac a d\right)\ge(a+b+c+d)\left(\frac 1 a+\frac 1 b+\frac 1 c+\frac 1 d\right).\] 令 $x=a/b$, $y=b/c$, $z=c/d$, $w=d/a$,则 $x$, $y$, $z$, $w>0$ 且 $xyzw=1$,将原不等式右边展开,可以整理为 \begin{equation}\label{201301244yxhbdsjds1} \sum x\sum \frac1x\geqslant \sum x+\sum \frac1x+(x+z)(y+w)+4, \end{equation} 或 \begin{equation}\label{201301244yxhbdsjds2} \left( \sum x-1 \right)\left( \frac{x+z}{xz}+xz(y+w)-1 \right)-(x+z)(y+w)-5\geqslant 0, \end{equation} 固定 $x$, $z$,下面证明式 \eqref{201301244yxhbdsjds2} 左边关于 $y+w$ 单调递增,为此,记 $y+w=t$, $\sqrt{xz}=u$ 以及 \[ f(t)=(x+z+t-1)\left( \frac{x+z}{xz}+xzt-1 \right)-(x+z)t-5, \] 求导得 \begin{align*} f'(t)&=\frac{x+z}{xz}+xzt-1+xz(x+z+t-1)-x-z \\ & =2xzt+(x+z)\left( \frac1{xz}+xz-1 \right)-xz-1 \\ & \geqslant 2xzt+2\sqrt{xz}\left( \frac1{xz}+xz-1 \right)-xz-1 \\ & =2u^2t+2u\left( \frac1u-u \right)^2-(u-1)^2 \\ & =2u^2t+\frac{(u-1)^2\bigl( 2(1+u)^2-u \bigr)}u \\ & >0, \end{align*} 于是单调性得证,注意到当 $x$, $z$ 固定时 $yw$ 也是固定的,所以式 \eqref{201301244yxhbdsjds2} 左边取最小值时必有 $y=w$,再注意到式 \eqref{201301244yxhbdsjds1} 的对称性(同时交换 $x$, $y$ 及 $z$, $w$ 时式 \eqref{201301244yxhbdsjds1} 不变),当 $y$, $w$ 固定时式 \eqref{201301244yxhbdsjds2} 左边取最小值时也必有 $x=z$,从而要证式 \eqref{201301244yxhbdsjds1},就只要证 $x=z$ 且 $y=w$ 时即可,此时记 $x=z=v$,则 $y=w=1/v$,式 \eqref{201301244yxhbdsjds1} 化为 \[\left(2v+\frac2v\right)^2\geqslant 8-4v-\frac4v,\] 作差因式分解为 \[\frac{4(v-1)(v^3-1)}{v^2}\geqslant 0,\] 显然成立,故原不等式得证。
goft 13# 2013-1-24 20:41
本帖最后由 goft 于 2013-1-24 20:58 编辑 齐次可设a+b+c+d=4 证明:A>=C(均值不等式),B>=D(B>=A,将a+b+c+d=4,代入D中,然后排序不等式) 即得证。AB>=CD,
reny 14# 2013-1-24 20:50
13# goft 没看懂。怎么$AB\ge CD,就有A\ge C,B\ge D?$
yes94 15# 2013-1-24 21:17
13# goft 没看懂。怎么$AB\ge CD,就有A\ge C,B\ge D?$ reny 发表于 2013-1-24 20:50 他比较喜欢些思路,而且思路通常比较好。只是不喜欢具体的过程,所以理解起来较困难 他的意思是,欲证$AB\geqslant CD$,可以先证$A\geqslant C$,且$B\geqslant D$ 只是那个排序,因为不是强对称,只是弱对称(即仅仅轮换对称),如何用排序不等式啊?
kuing 16# 2013-1-24 21:24
在a+b+c+d=4下,A>=D、B>=D都不是恒成立的
thread-108-1-1.html: 白痴答题技巧
kuing 1# 2011-10-18 00:13
已知函数 $\displaystyle f(x)=\frac12x^3-x^2-\frac72x$,则 $f(-a^2)$ 与 $f(-1)$ 的大小关系为(  ) A. $f(-a^2)\leqslant f(-1)$ B. $f(-a^2)<f(-1)$ C. $f(-a^2)\geqslant f(-1)$ D. $f(-a^2)$ 与 $f(-1)$ 的大小关系不确定 这里不是讲这题怎么解,事实上这题极为简单。这里用所谓“白痴答题技巧”去做。 前提:把自己当成数学白痴。目的:选出最有可能正确的选项。 首先,这种题多数不会关系不确定,所以不看D。 A和C不等号反过来,多数是其中一个,而B选项则可能是为了凑够四选项而充数,正确的可能性不大,但看其方向,是与A相同但差一点点,看来A、B间还带一点陷阱,所以A的可能性最大!选A!
图图 2# 2011-10-18 00:18
这个方法老师讲过呢,貌似
kuing 3# 2011-10-18 00:22
2# 图图 看来是应试必学,呃,我刚才也是无聊才突然想这些东西, 唔,珍爱生命,远离…… 我闪
图图 4# 2011-10-18 00:27
老师讲这些还是会比较正经的...
kuing 5# 2011-10-18 00:30
4# 图图 说这些应该轻松一些比较好,因为本来就是灵活的东西……
nash 6# 2011-10-18 00:40
这个方法估计要把出题人气倒!!! 还以为你回带数字呢 a=1显然等号,这个出题人…
kuing 7# 2011-10-18 00:42
6# nash 前提是数学白痴,暂且不允许代数字,如果允许的话,能发现这个代数字那就更容易判断了。
isea 8# 2011-10-18 09:57
这题本来出得就很模糊
kuing 9# 2011-10-18 12:43

魔幻水果 10# 2011-10-18 13:59
你家才是白痴
kuing 11# 2011-10-18 14:03
楼上的表情…… 话说好像经常出现在人教如果没记错的话……
yuzi 12# 2011-10-18 20:20
水果现身,就这么一句话??
kuing 13# 2011-10-18 20:43
12# yuzi 你认识她喔?
魔幻水果 14# 2011-10-18 20:58
12# yuzi 鱼子蜀黍,我耐你~!
kuing 15# 2011-10-18 21:34
14# 魔幻水果 你们……
kuing 16# 2011-10-19 01:51
14# 魔幻水果 话说这头像是谁哟?
yuzi 17# 2011-10-19 09:33
14# 魔幻水果
kuing 18# 2011-10-19 12:04
嘿嘿
海盗船长 19# 2012-1-14 20:48

kuing 20# 2012-8-6 21:30
PS、“白痴答题法”这名字由sunjialong所创。
thread-108-2-1.html:
froglove 21# 2012-8-6 22:02
这帖......真好==
kuing 22# 2012-8-6 22:05
21# froglove
froglove 23# 2012-8-6 22:48
这东西真该给出题人看看.........囧囧的考试
kuing 24# 2012-8-6 22:49
叫口号……
froglove 25# 2012-8-6 23:07
口号.....免了吧...大家懂的.
李斌斌755 26# 2013-4-27 21:31
25# froglove 哪来的魔女。
李斌斌755 27# 2013-5-21 00:38

kuing 28# 2013-5-21 01:18
你还真的找。。。
realnumber 29# 2013-5-21 13:20
揣摩出题老师的心理?果然够滑头,用来读书够2的. 我也补充一个,解三角形题目可以用尺规猜答案或验证,立几公理那章节的题目可以用纸笔搭模型,--这个不算白痴了
thread-1080-1-4.html: [函数] 连云港高三期末试题
pengcheng1130 1# 2013-1-23 21:58
关于 $x$ 的不等式 $x^2-a x+2a <0$ 的解集为 $A$, 若集合 $A$ 中恰有两个整数,则实数 $a$ 的取值范围是多少?
realnumber 2# 2013-1-24 14:16
本帖最后由 realnumber 于 2013-1-24 14:18 编辑 用零点存在定理,配合$2$次函数图象$f(x)=x^2-ax+2a$(过定点$(2,4)$), 满足$f(k-1)\ge0$且$f(k)<0$且$f(k+1)<0$且$f(k+2)\ge0$,$k$取遍满足条件$k-1\ge2$或$k+2\le2$的所有整数. 取了k=3,4,发现问题被我处理复杂了. 修改如下:不等式作如下变形$x^2<a(x-2)$,令$x-2=t$($x,t$同为整数或同不为整数),得到$t^2+4t+4\le{at}$,注意到$t=0$不满足, 若$t>0$,则有\[h(t)=t+\frac{4}{t}+4\le a\] 即两个整数就是$t=2,3$,所以$h(3)<a\le h(1)$,即$\frac{25}{3}<a\le{9}$ 若$t<0$,你自己解吧,$t$一正一负自然不可能
yes94 3# 2013-1-24 16:18
直接用求根公式可以不? 两根的绝对值差介于$(1,3]$之间,再检查充分性
realnumber 4# 2013-1-24 18:21
3# yes94 期待...
yes94 5# 2013-1-24 18:43
3# yes94 期待... realnumber 发表于 2013-1-24 18:21 我问你可以不?你反而期待我,
pengcheng1130 6# 2013-1-24 18:56
答案似乎不对啊!!!
yes94 7# 2013-1-24 18:58
答案似乎不对啊!!! pengcheng1130 发表于 2013-1-24 18:56 给出答案嘛!
pengcheng1130 8# 2013-1-24 19:16
$-1\le a<-\frac{1}{3}$或$\frac{25}{3}<a\le 9$
yes94 9# 2013-1-24 19:27
8# pengcheng1130 $2$楼给似乎你提示了, 当$t<0$时,请你自行解决
pengcheng1130 10# 2013-1-24 20:44
谢谢,你的解法还是比较简单的啊!
realnumber 11# 2013-1-25 08:53
3# yes94 也许也可以,正好可以简化2楼的前面一个办法.
isea 12# 2013-1-25 22:48
本帖最后由 isea 于 2013-1-26 22:05 编辑 关于$x$的不等式$x^2-ax+2a<0$的解集为$A$,若集合$A$中恰有两个整数,则实数$a$的取值范围是_____ 设$m\in \mathbf{Z},m,m+1\in A,m-1,m+2\notin A,$设满足题设$a$的集合为$M$, 若$\exists a_0\in M,$使得 \begin{align*} x^2-a_0x+2a_0+t & \equiv (x-m)(x-m-1) ,t\in (0,2]\\ x^2-a_0x+2a_0+t & \equiv x^2-(2m+1)x+m^2+m) \\ &\begin{cases} a_0&=2m+1 \\ 2a_0+t&=m^2+m\end{cases}\\ 4m+2+t&=m^2+m\\ m^2-3m-2&=t\in (0,2],m\in \mathbf{Z}\\ m^2-3m-2&=1,2,m\in \mathbf{Z}\\ m&=-1,4\\ \end{align*} 此时,$a_0=-1,9$,临界最值吧(?) 于是$-1,0\in A,-2,1\notin A$或$4,5\in A,3,6\notin A,$以下具体计算略去,最后结果为 $$M=[-1,-\dfrac13)\cup(\dfrac{25}3,9]$$ PS:这题,入手难,很多时候导出的结果是不对的;间歇想了三五回,最后,在特殊情况下,先求出其整数根是关键,看来 PPS:对我,这题,真难,考场上第一次见,肯定在单位时间内搞不定 最后,终于看懂2楼realnumber的解法了是啥意思了 好吧,k说行间用得太多,去掉几个,哈哈 PPPS:现已修改部分叙述,解法不严密,后面回复,有考虑其顶点坐标轨迹,从数形两方面得到答案,详细过程不会了,最后期待此题更完善的办法
kuing 13# 2013-1-25 23:09
我终于知道为什么会自动变成了要shen核了……
isea 14# 2013-1-25 23:14
13# kuing 哈哈,发帖前有个预览就好了……
kuing 15# 2013-1-25 23:17
14# isea 我也想,不知怎么弄……
isea 16# 2013-1-25 23:38
14# isea 我也想,不知怎么弄…… kuing 发表于 2013-1-25 23:17 latex,排版太强了,我虽然只学到了一点点……
kuing 17# 2013-1-25 23:44
14# isea 我也想,不知怎么弄…… kuing 发表于 2013-1-25 23:17 或者说根本没种预览的功能,Discuz7设计者的原意只是想让我们用“去掉右上角‘源码’的勾”这样的法子来实现所谓的预览,这对一般的贴子是可行的,但是这里的特殊情况(MathJax 对代码的读取及显示公式的特点)就顾及不到了。 呵呵,扯太多了,还是回归正题吧。 PS、你上面那个行间公式用得太多了,有些不必
pengcheng1130 18# 2013-1-26 19:58
12楼的解法,存在$a_{0}$,$t$,怎么来的啊!
pengcheng1130 19# 2013-1-26 20:22
十二楼的解法似乎有点说不通啊!大家看看是不是有点问题?
isea 20# 2013-1-26 20:37
12楼的解法,存在$a_{0}$,$t$,怎么来的啊! pengcheng1130 发表于 2013-1-26 19:58 可先考虑一下这两个函数$f(x)=x^2-x,g(x)=x^2-x-2$,然后想像一下图象平移。
thread-1080-2-4.html:
pengcheng1130 21# 2013-1-26 20:41
我也在想考虑平移,但是如何断定就存在$a$!使得它们的对称轴一样呢?
isea 22# 2013-1-26 21:55
本帖最后由 isea 于 2013-1-26 22:17 编辑 我也在想考虑平移,但是如何断定就存在$a$!使得它们的对称轴一样呢? pengcheng1130 发表于 2013-1-26 20:41 问得有道理,败给这题了,哈哈。 说一下原解题思路吧,原始想的是,水平平移是$\mathbf{R}$, 总有一个位置使得$y=(x-m)(x-m-1),$或者$y=(x-m+1)(x-m-2),$至少一个成立, 这里的确默了其中至少有一个是应该边界而开始思考的。且认为,即便不存在这样的抛物线时也不碍事,因为想“夹出”是两个整数根,至少有个接近数值与方向。 从而, 原来的解法,写存在$a_0$是有bug的(随便手画了画示意图),应该是假设存在$a_0$,如果解出了$a_0$合题设,即肯定其存在性。 若是无解,则不存在(这种对称的抛物线),则要另想方法。 ================== 好吧,那就干脆“主观”到底——数型结合,至少得到正确结果是没问题的。 \begin{align*}y&=x^2-ax+2a\\ &=(x-\dfrac a2)^2-\dfrac{a^2}{4}+2a\\ \end{align*} 则顶点坐标$\left (\dfrac a2,-\dfrac{a^2}{4}+2a\right )$的轨迹是$y=-4x^2+4x$,借助网格,(手画)相对较精确的图,讨论这两个整数到底是哪两个,即可得到答案。
pengcheng1130 23# 2013-1-26 22:27
谢谢,代数解法有代数解法的魅力,数形结合有数形结合的魅力 有人喜欢纯字母的推理,有人喜欢图形秒杀,各有其乐! 就如同四色定理的证明,虽然是计算机给出了“证明”,但现在仍有数学家试图给出一个手工的证明!
yes94 24# 2013-1-26 22:42
谢谢,代数解法有代数解法的魅力,数形结合有数形结合的魅力 有人喜欢纯字母的推理,有人喜欢图形秒杀,各有其乐! 就如同四色定理的证明,虽然是计算机给出了“证明”,但现在仍有数学家试图给出一个手工的证明! pengcheng1130 发表于 2013-1-26 22:27 喜欢这个中肯的评价
jsxh2005 25# 2013-1-30 11:50
3# yes94 我是这样处理的,感觉这种问题没有一般方法啊。
yes94 26# 2013-2-3 15:42
本帖最后由 yes94 于 2013-2-5 23:41 编辑 25# jsxh2005 下面用最原始的观察法解决该问题(写了这么多的代码,代码却不给力,写错了? ): 令$f(x)=x^2-ax+2a$,则不等式$f(x)<0$,即$x^2<a(x-2)$,当$a>0$时,$x>2$,故此时整数解至少从$3$开始. 罗列出如下不等式:$f(3)=9-a<0$,$f(4)=16-2a<0$,$f(5)=25-3a<0$,$f(6)=36-4a<0$,    $f(7)=49-5a<0$,……,$f(n)=n^2-(n-2)a<0(n\geqslant 7)$, 以上不等式分别等价于:$a>9$,$a>8$,$a>\dfrac{25}{3}$,$a>9$,$a>\dfrac{49}{5}(>9)$,……,$a>\dfrac{n^2}{n-2}(>9)$,     注:这里可证明$g(n)=\dfrac{n^2}{n-2}$$(n\geqslant4)$是递增数列。下文的$g(n)=\dfrac{n^2}{n-2}$$(n\leqslant0)$也是递增数列 如果$a>9$,那么至少有四个整数解$3,4,5,6$满足不等式$f(x)=x^2-ax+2a<0$,矛盾于条件。 所以$a\leqslant 9$,于是$x=3$,$6$,$7$,$8$,……不是不等式$f(x)=x^2-ax+2a<0$的解,只余下$x=4$,$5$这两个整数了。 当$a\leqslant \dfrac{25}{3}$,只有$x=5$不是整数解。当$a>\dfrac{25}{3}$,恰有$x=4,5$是整数解     所以,$\dfrac{25}{3}<a\leqslant 9$ 对于$a<0$,可同样讨论,此时要求$x<2$,对$x=1$,$0$,$-1$,$-2$,$-3$,$-4$,……,赋值,发现 $f(1)=9+a<0$,$f(0)=2a<0$,$f(-1)=1+3a<0$,$f(-2)=4+4a<0$,$f(-3)=9+5a<0$,$f(-4)=16+6a<0$,$f(-5)=25+7a<0$,……,$f(n)=n^2-(n-2)a<0$,($n\leqslant 1,n\in Z$), 分别得到:$a<-9$,$a<0$,$a<-\dfrac{1}{3}$,$a<-1$,$a<-\dfrac{9}{5}$,$a<-\dfrac{8}{3}$,$a<-\dfrac{25}{7}$,……,$a<\dfrac{n^2}{n-2}$, 讨论过程略,得到$-1\leqslant a<-\dfrac{1}{3}$, 综上,$-1\leqslant a<-\dfrac{1}{3}$或$\dfrac{25}{3}<a\leqslant 9$
thread-1081-1-4.html: [不等式] BQ的一道非对称几何不等式
kuing 1# 2013-1-24 21:36
在 $\triangle ABC$ 中,分别记 $w_a$, $w_b$, $w_c$ 为 $a$, $b$, $c$ 边上的内角平分线,则 \[\frac{\sqrt3(2a+c)}2\geqslant 2w_b+w_c.\] 由齐次性,不妨设 $a+b+c=1$,由内角平分线长公式、柯西不等式及均值不等式,有 \begin{align*} 2w_b+w_c&=\frac{2\sqrt{ca(c+a-b)}}{c+a}+\frac{\sqrt{ab(a+b-c)}}{a+b} \\ & \leqslant \sqrt{\left( \frac{2(c+a-b)}{c+a}+\frac{a+b-c}{a+b} \right)\left( \frac{2ca}{c+a}+\frac{ab}{a+b} \right)} \\ & \leqslant \sqrt{\left( 3-\frac{2b}{1-b}-\frac{c}{1-c} \right)\left( \frac{c+a}2+\frac{a+b}4 \right)} \\ & \leqslant \sqrt{\left( 3-\frac{(2b+c)^2}{2b(1-b)+c(1-c)} \right)\frac{3-2b-c}4} \\ & =\sqrt{\left( 3-\frac{3(2b+c)^2}{3(2b+c)-(2+1)(2b^2+c^2)} \right)\frac{3-2b-c}4} \\ & \leqslant \sqrt{\left( 3-\frac{3(2b+c)^2}{3(2b+c)-(2b+c)^2} \right)\frac{3-2b-c}4} \\ & =\sqrt{\frac94-\frac32(2b+c)} \\ & =\sqrt{\frac32(2a+c)-\frac34} \\ & =\sqrt{\frac{3(2a+c)^2}4-\frac34(2a+c-1)^2} \\ & \leqslant \frac{\sqrt3(2a+c)}2. \end{align*} 蛮累的,弄了几个小时才证到,主要是对非对称的老鼠拉龟了,还好原不等式其实挺弱,以至于允许我大胆放缩那么多次也不过头。
yes94 2# 2013-1-24 22:29
1# kuing 放缩了5次! 居然还能取等号, 太奇迹了吧!
kuing 3# 2013-1-24 22:34
看得出 a=b=c 取等……放缩的时候系数迁就一下……
thread-1083-1-3.html: [不等式] 来自人教群的$abc=1$轮换不等式
kuing 1# 2013-1-25 00:41
话说我证得有点点bao力的。 首先那些平方是多余的(因 $a\leftrightarrow\sqrt a$ 条件不变),于是即证 \[ \sum \frac1{a+3b+2}\leqslant \frac12, \] 正因为次数不高,所以才不怕bao力将上式去分母展开,等价于 \[ 9\sum a^2c+3\sum a^2b-16\sum a-16+28abc\geqslant 0, \] 由 $abc=1$,上式整理为 \[ 3\left( 3\sum a^2c-4\sum a+3 \right)+3\sum a^2b-4\sum a+3\geqslant 0, \] 于是我们只要证明 \begin{align} 3\sum a^2c-4\sum a+3&\geqslant 0, \label{20130125lhdcxblzs1}\\ 3\sum a^2b-4\sum a+3&\geqslant 0, \label{20130125lhdcxblzs2} \end{align} 而这显然只需证其一即可,另一个是同理的。下面就证明式 \eqref{20130125lhdcxblzs2},令 $a=x/y$, $b=y/z$, $c=z/x$,其中 $x$, $y$, $z>0$,则 \begin{align*} 3\sum a^2b-4\sum a+3\geqslant 0&\iff3\sum\frac{x^2}{yz}-4\sum\frac xy+3\geqslant 0 \\ & \iff3\sum x^3-4\sum x^2z+3xyz\geqslant 0 \\ & \iff\frac16\sum(5x+13y-3z)(x-y)^2\geqslant 0, \end{align*} 由轮换对称性,不妨设 $y$ 在 $x$ 与 $z$ 之间,即 $(x-y)(y-z)\geqslant 0$,此时有 \[ (5z+13x-3y)(z-x)^2\geqslant (5z+13x-3y)\bigl((x-y)^2+(y-z)^2\bigr), \] 于是 \begin{align*} & \sum(5x+13y-3z)(x-y)^2 \\ \geqslant{}& (5x+13y-3z+5z+13x-3y)(x-y)^2+(5y+13z-3x+5z+13x-3y)(y-z)^2 \\ ={}& (18x+10y+2z)(x-y)^2+(10x+2y+18z)(y-z)^2 \\ \geqslant{}& 0, \end{align*} 从而式 \eqref{20130125lhdcxblzs2} 得证,所以原不等式成立。
yes94 2# 2013-1-25 19:05
本帖最后由 yes94 于 2013-1-25 19:08 编辑 借助于版主说的,“首先那些平方是多余的(因 $a\leftrightarrow\sqrt a$ 条件不变),于是即证 \[ \sum \frac1{a+3b+2}\leqslant \frac12, \] 被一大堆平方吓住了,……” 那不是可以改成: \[ \sum \frac1{a^3+3b^3+2}\leqslant \frac12, \] 或者改成: \[ \sum \frac1{a^n+3b^n+2}\leqslant \frac12, \] 例如,$n=\dfrac12$就变成: \[ \sum \frac1{\sqrt a+3\sqrt b+2}\leqslant \frac12, \] 嘻嘻,为了练习代码输入
kuing 3# 2013-1-25 19:48
2# yes94 写成那些当然也可以了,只是这样出题可能会被高手们BS...
yes94 4# 2013-1-25 19:59
2# yes94 写成那些当然也可以了,只是这样出题可能会被高手们BS... kuing 发表于 2013-1-25 19:48
yes94 5# 2013-1-27 22:55
原来这个也一样:
kuing 6# 2013-1-27 22:58
5# yes94 完全对称简单很多……
yes94 7# 2013-1-27 23:00
6# kuing 这个的确很简单, 我说“一样”指的是“条件一样”和“指数上升下降"一样,方法可不一样了,这道题对称,就简单得多
zdyzhj 8# 2013-1-28 08:37
本帖最后由 zdyzhj 于 2013-1-28 08:54 编辑 那些平方显然是炸蛋,糊弄人的,前几天整理过了,方法与这个稍有不同。还可以进一步简化证明。
realnumber 9# 2013-1-28 08:43
8# zdyzhj 也发上来,口气怎么跟不等式群的县长一样,总是说整理了什么,就不见过程.
yes94 10# 2013-1-28 15:23
8# zdyzhj 也发上来,口气怎么跟不等式群的县长一样,总是说整理了什么,就不见过程. realnumber 发表于 2013-1-28 08:43 县长驾到!有失远迎,罪过!
reny 11# 2013-3-9 16:10
5# yes94 争对(2),能否证明$$\dfrac1{\sqrt{1+a+b}}+\dfrac1{\sqrt{1+b+c}}+\dfrac1{\sqrt{1+c+a}}\leqslant \sqrt3$$
kuing 12# 2013-3-9 16:48
5# yes94 争对(2),能否证明$$\dfrac1{\sqrt{1+a+b}}+\dfrac1{\sqrt{1+b+c}}+\dfrac1{\sqrt{1+c+a}}\leqslant \sqrt3$$ reny 发表于 2013-3-9 16:10 不是弱于那个(2)么,柯西一下……
reny 13# 2013-3-9 17:16
本帖最后由 reny 于 2013-3-9 17:33 编辑 12# kuing 的确是! 问题变弱了. 如果是n次根号的话,用Holder解决. #5问题(1)变为$$\dfrac1{\sqrt[3]{1+2a}}+\dfrac1{\sqrt[3]{1+2b}}+\dfrac1{\sqrt[3]{1+2c}}\geqslant \sqrt[3]9$$,就不弱于(1)吧. Ps、顺便说下,我搜索曾经发的帖子,怎么只有最近两条
thread-1084-1-4.html: [几何] 双曲线题
guanmo 1# 2013-1-25 16:13
_____kuing edit in $\LaTeX$_____ 设 $F$ 为双曲线 $x^2/16-y^2/9=1$ 的左焦点,在 $x$ 轴上 $F$ 点的右侧有一点 $A$,以 $FA$ 为直径的圆与双曲线左、右两支在 $x$ 轴上方的交点分别为 $M$、$N$,则 $(\abs{FN}-\abs{FM})/\abs{FA}$ 的值为(  )
kuing 2# 2013-1-25 16:24
方不方便配个图…… PS、本主题分类可选“几何”
yes94 3# 2013-1-25 18:35
答案: $\dfrac45$,离心率的倒数:$\dfrac1e$
yes94 4# 2013-1-25 18:36
类似的还有椭圆,抛物线。
abababa 5# 2013-1-25 19:02
能不能用极坐标?两个曲线方程都好写
guanmo 6# 2013-1-25 23:06
4#yes94兄果然是高手,请提示一下先。ps:一直很好奇yes94兄的尊姓大名,能否透露一把啊?膜拜已久呀!
kuing 7# 2013-1-26 00:07
貌似 $\dfrac2e$ 用最常规的代数方法挺简单,可惜一时没想到几何方法,先闪了……
yes94 8# 2013-1-26 16:31
貌似 $\dfrac2e$ 用最常规的代数方法挺简单,可惜一时没想到几何方法,先闪了…… kuing 发表于 2013-1-26 00:07 哦? 其实这道题就是用的几何方法……
kuing 9# 2013-1-26 18:00
哦? 其实这道题就是用的几何方法…… yes94 发表于 2013-1-26 16:31 原来我前面看错题,以为是以FA为半径,原来是直径,咳,没配图,看文字就是容易看错……直径的话那就自然是1/e了 几何方法怎么玩?最近似乎对几何没了feel
yes94 10# 2013-1-26 21:44
题做多了,肯定不是每天都有感觉的,有时候就不是很想做了。
kuing 11# 2013-1-27 01:23
10# yes94 倒没有不想做, 只是好像状态差了,前两天那个椭圆的还差点误导了你们,幸亏fox007及时解决掉了。 你有空写写解法吧,或者找到链接也行(我突然觉得好像在哪里见过, 说不定你也在)
yes94 12# 2013-1-27 13:25
用射影定理,(作直角三角形的斜边上的高) 然后平方差,再用第二定义,作准线垂线,立刻可得
kuing 13# 2013-1-27 13:32
12# yes94 原来如此,懂了
abababa 14# 2013-1-27 17:09
几何法还是没大看懂,用极坐标挺容易的 $\rho=\frac{ep}{1+e\cos\theta}, \rho=2r\cos\theta$,消去$\cos\theta$得到$\rho^2+\frac{2r}{e}\rho-2rp=0$ 两根积是负的,两根一正一负,$|\rho_1|-|\rho_2|=|\rho_1+\rho_2|=\frac{2r}{e}$,$FA=2r$,最后就是$1/e$
yes94 15# 2013-1-27 18:04
14# abababa 圆的极坐标用得好! 双曲线的极坐标倒是熟知的。
thread-1085-1-1.html: 太晚了,88
isea 1# 2013-1-26 00:21
睡觉去了,你们玩儿~~
kuing 2# 2013-1-26 00:22
1# isea 呃,时间尚早…… 不过现在论坛在线似乎没第三个人了……或者说,还有几个游客……
realnumber 3# 2013-1-26 09:25
2# kuing 还早啊,一般都睡了几小时了~~~~
kuing 4# 2013-1-26 12:18
3# realnumber 嗯,我们有时差。。
realnumber 5# 2013-1-26 22:13
,睡觉去了,有所顾忌,孩子,健康,.... 4# kuing
realnumber 6# 2013-1-26 22:15
5# realnumber 曾经3个白天晚上晚三国志四,现在想来有啥好玩的,一开星际或博得,根本不想玩.不如科普书好看.
isea 7# 2013-1-26 22:27
我还以为是不如多睡觉睡觉呢 南边的时差也不至于这么大吧?
kuing 8# 2013-1-26 22:33
夜猫专用时区……
李斌斌755 9# 2013-5-5 02:29
冒个泡,吹下水,睡觉了。
李斌斌755 10# 2013-5-6 01:35
题没证明出……
isea 11# 2013-5-6 10:35
10# 李斌斌755 几何题呀?浩如烟海,抓些小花,自娱自乐,足已
李斌斌755 12# 2013-5-6 11:59
11# isea 海犹如磁场,我是铁
李斌斌755 13# 2013-5-13 01:40
明天上班,睡觉了。
thread-1086-1-2.html: 畢業論文
kuing 1# 2013-1-26 14:14
內涵版: @╰☆ヾo.海x
╰☆ヾo.海x 2# 2013-1-26 19:48
哈哈哈。。是让我心理有些安慰了。。  我第一篇论文总共也有改了7、8次。。。 但是这次第二篇论文老师没有责任给我们改你知道吗,所以都要靠自己。。还有课堂的理解。 主要是没有老师全部的批注的话,自己我很难发现问题在哪,可能改了3、5遍还是一团垃圾!!哎。。这是最悲剧的地方
kuing 3# 2013-1-26 19:56
2# ╰☆ヾo.海x 俺一点儿都不懂
thread-1087-1-4.html: [几何] 这道求角度的题人教初中区应该有链接,谁找找
kuing 1# 2013-1-26 15:27
如题,这道求角度的题人教初中区应该有链接,我不熟悉那里,没找着,谁来挖挖 如图,四边形 $ABCD$ 的对角线交于 $E$,$\angle BEC=72^\circ$,$\angle ABD=18^\circ$,$\angle ACD=30^\circ$,且 $BE=CE$,求 $\angle ADB$ 的度数。
isea 2# 2013-1-26 20:43
有差不多的,图象,没细看,15楼图8,19楼longnetpro有解,后亦有深入讨论 可供参考 http://bbs.pep.com.cn/forum.php? ... d=668772&extra=
kuing 3# 2013-1-26 20:58
2# isea 哇,好长的贴…… 话说用三角函数应该都可以搞,只不过我在想辅助线……
yes94 4# 2013-1-26 21:49

isea 5# 2013-1-26 22:22
这种题多半是初二的竞赛题,多数时候都有很巧的平几法。 或者是 高中联赛试题,这题图倒是很熟悉。 这种题从对称,等边,旋转,共圆方向与思考,一般有一种“组合”办法得到结果, 但偶真心怕这种题,太伤脑子了,等哪天有空的时候看看或者欣赏一下此题过程。 闪人
kuing 6# 2013-1-27 18:06
搞成这样了,有不少等腰三角形还有共圆,但是还是没推出来,除了用角元塞瓦定理之外……
kuing 7# 2013-1-27 18:35
话说用角元塞瓦定理感觉还有点liu氓…… \[\frac{\sin x}{\sin(54^\circ-x)}\cdot\frac{\sin54^\circ}{\sin42^\circ}\cdot\frac{\sin12^\circ}{\sin18^\circ}=1,\] 直接验证 $x=30^\circ$ 是其中一个解,再由图形知 $x$ 唯一,所以就只有 $x=30^\circ$ 了…… 而验证 $x=30^\circ$ 的时候有没有什么巧妙办法呢?我是直接将各个三角函数值代入验证的……
kuing 8# 2013-1-27 18:50
7# kuing oh,原来用那个用得比较少的三倍角公式就行了 \begin{align*} \frac{\sin30^\circ}{\sin24^\circ}\cdot\frac{\sin54^\circ}{\sin42^\circ}\cdot\frac{\sin12^\circ}{\sin18^\circ} &=\frac1{4\sin12^\circ\cos12^\circ}\cdot\frac{\cos36^\circ}{\sin42^\circ}\cdot\frac{\sin12^\circ}{\sin18^\circ}\\ &=\frac1{4\cos12^\circ}\cdot\frac{4\cos12^\circ\cos(60^\circ-12^\circ)\cos(60^\circ+12^\circ)}{\sin42^\circ}\cdot\frac1{\sin18^\circ}\\ &=\frac{\cos48^\circ\cos72^\circ}{\sin42^\circ\sin18^\circ}\\ &=1. \end{align*}
isea 9# 2013-2-17 23:52
本帖最后由 isea 于 2013-2-18 22:48 编辑 现补上一种纯几法 一种纯几何法,提示如下: 作$△ACD$的外接圆交$BD$于$F$(即作$∠FAC=42°$),则$∠AFD=30°$. 将△ABF沿直线BF折叠,得△A'BF,则正△AFA'. 则由角大小关系,可得$△ABA'≅△CBA'$(SAS). 于是$A' $为$△ACF$的外心,亦$A,F,C,D$四点共圆的圆心,下面你想怎么说,都有$30°$了…… ============================================ 真心不是人做的题,又像回到当初见到这种(相对容易的题)一样迷离了...... 这里的30度太TMD难用了,难怪当初只整理9题,舍弃哪些四边形中的求角的......
kuing 10# 2013-2-18 00:00
9# isea 反正我是放弃找那极其巧妙的平几证法了,虽然感觉美妙证法一定存在…… 能用塞瓦三角法解决大部分这类题我暂时已经满足……
╰☆ヾo.海x 11# 2013-2-18 05:34
本帖最后由 ╰☆ヾo.海x 于 2013-2-18 05:46 编辑 3# kuing 我用三角函数做了下然后列了2个方程还是不会解。。 啊法=角ABD,被他=角CAD...
kuing 12# 2013-2-18 11:34
3# kuing 我用三角函数做了下然后列了2个方程还是不会解。。 啊法=角ABD,被他=角CAD... ╰☆ヾo.海x 发表于 2013-2-18 05:34 应该是 啊法=角ADB 吧? 这个方法不错哇,只利用了等腰和正弦定理,列出来的方程比我的简单且易懂,nice one! 晚点再看看怎么解简单些。
kuing 13# 2013-2-18 12:04
12# kuing 消去 $\beta $,得 \begin{gather*} \frac{\sin 42^\circ}{\sin 84^\circ}=\frac{\sin \alpha }{\sin (54^\circ+108^\circ-\alpha )} \\ \frac1{2\cos 42^\circ}=\frac{\sin \alpha }{\sin (18^\circ+\alpha )} \\ \sin (18^\circ+\alpha )=2\cos (60^\circ-18^\circ)\sin \alpha \\ \sin 18^\circ\cos \alpha +\cos 18^\circ\sin \alpha =2(\cos 60^\circ\cos 18^\circ+\sin 60^\circ\sin 18^\circ)\sin \alpha \\ \cos \alpha =\sqrt3\sin \alpha \\ \alpha =30. \end{gather*} 这样,就得到了更简单的三角方法了,多谢海x!
isea 14# 2013-2-18 22:50
9# isea 反正我是放弃找那极其巧妙的平几证法了,虽然感觉美妙证法一定存在…… 能用塞瓦三角法解决大部分这类题我暂时已经满足…… kuing 发表于 2013-2-18 00:00 在9楼已几何法,这里也丢个图,供参考
kuing 15# 2013-2-18 22:56
14# isea 就等这种平几证法,终于被你找到了恭喜
isea 16# 2013-2-18 23:05
三个多星期,得闲无事时,就在纸上画画,又回头翻了翻了翻曾经整理的9道题,虽有些粗糙和笨拙,不过,基本也道出了题目关键所在。 想当初(时间飞快呀)在人教整理9题是以 角BAC=80度,角ABC=60度,角DAB=10度,角DBA=20度,求角ACD. 此题作结,因为感叹实在 longnetpro 解法中那个外心,实在妙不可言。 不曾想,对主楼的题为转化那个30度也来了个外心! 现如今 longnetpro 优美几何构造,还在深深的影响偶。 PS: 后来:zheyic 又提到本楼作结题目,(人教论坛源链接),南山菊,大脚丫子等又给出不几的解法,如南山菊的 而种种几何解法正验证了前面“胡言乱语”。
kuing 17# 2013-2-19 00:50
16# isea 你们都很牛……要是都多过来这边玩玩就好了
╰☆ヾo.海x 18# 2013-2-19 01:35
13# kuing 恩是ADB...谢酷儿给解方程了
kuing 19# 2013-2-19 01:46
18# ╰☆ヾo.海x 果然变厉害了
╰☆ヾo.海x 20# 2013-2-19 14:30
19# kuing 在这关键时刻我这个早晨已经吐了三次了。。还没吐干净还是不舒服。。
thread-1087-2-4.html:
kuing 21# 2013-2-19 14:51
20# ╰☆ヾo.海x 还上论坛。。好好休息吧
thread-1088-1-4.html: [不等式] 一个有意思的不等式
pxchg1200 1# 2013-1-26 18:19
本帖最后由 pxchg1200 于 2013-1-26 18:21 编辑 西西网友出了个有趣的不等式 小弄了下,走运找到个证明,希望看到更多犀利的证明。 :D Proof (1)by Holder inequality, \[ \left(\sum{\frac{x^4}{y^3}}\right)^{5}\left(\sum{x^{10}y^5}\right)\left(\sum{x^5y^5}\right)^{2}\geq (\sum{x^5})^{8} \] So,it's suffice to prove \[ (\sum{x^5})^8\geq 3^{5}\left(\sum{x^{10}y^5}\right)\left(\sum{x^5y^5}\right)^{2} \] We have the know result \[ (\sum{a})^2\geq 3\sum{ab}\] and \[ (\sum{a})^5\geq 27(ab+bc+ca)(a^2b+b^2c+c^2a) \] The result follows. Done! (2) After homogenous,just need to check \[ \left(\sum{\frac{x^3}{y^2}}\right)^5\geq 81(x^5+y^5+z^5) \] recalling the Well-known Vasile Cirtoaje inequality \[ \boxed{(a+b+c)^5\geq 81abc(a^2+b^2+c^2)} \] and use $ a=\frac{x^3}{y^2},b=\frac{y^3}{z^2},c=\frac{y^3}{x^2}$,it's suffices to prove \[ \frac{x^6}{y^4}+\frac{y^6}{z^4}+\frac{z^6}{x^4}\geq \frac{x^4}{yz}+\frac{y^4}{xz}+\frac{z^4}{xy} \] Now,Using AM-GM inequality \[26\frac{x^6}{y^4}+11\frac{y^6}{z^4}+\frac{z^6}{x^4}\geq 38\frac{x^4}{yz} \] \[ 26\frac{y^6}{z^4}+11\frac{z^6}{x^4}+\frac{x^6}{y^4}\geq 38\frac{y^4}{xz} \] \[26\frac{z^6}{x^4}+11\frac{x^6}{y^4}+\frac{y^6}{z^4}\geq 38\frac{z^4}{xy} \] sum its up ,the result follows. Done!
thread-1089-1-4.html: [数论] 存在无限个与m互素.有没简单办法
realnumber 1# 2013-1-27 10:54
本帖最后由 realnumber 于 2013-1-27 10:56 编辑 来自<初等数论100例>.pdf第2题,新浪爱问免费下 设$(a,b)=1,m>0$,则数列$\{a+bk\}中(k=0,1,2,3,..$),存在无限个数与$m$互素.
yes94 2# 2013-1-27 13:32
(Dirichlet): 若d>=0  a<>0 是2个互质的正整数,那么 a, a+d ,a+2d .... 包含无穷多个质数; 百度一搜索:张云华的关于质数的 http://blog.sina.com.cn/s/blog_630088e00100hlmu.html 原来是狄利克雷的定理。
yes94 3# 2013-1-27 13:38
本帖最后由 yes94 于 2013-1-27 13:39 编辑 $1$.http://www.doc88.com/p-909533383581.html $2$.素数等差数列 等差数列是数列的一种。在等差数列中,任何相邻两项的差相等。该差值称为公差。类似7、37、67、97、107、137、167、197。这样由素数组成的数列叫做等差素数数列。2004年,格林和陶哲轩证明存在任意长的素数等差数列。2004年4月18日,两人宣布:他们证明了“存在任意长度的素数等差数列”,也就是说,对于任意值K,存在K个成等差级数的素数。例如 K=3,有素数序列3,5,7 (每两个差2)……K=10,有素数序列 199,409,619,829,1039,1249,1459,1669,1879,2089 (每两个差210)。 只不过这是有限个素数的等差数列。
realnumber 4# 2013-1-27 14:38
3# yes94 也算吧,牛刀杀鸡.其实不希望看到运用这个定理,因为这个定理证明没看到,据说也不好懂. 希望有素数无限个类似巧妙的证明.
yes94 5# 2013-1-27 18:14
4# realnumber 陶哲轩! 所以,还是放弃了吧 反正已经被吓到了, 即便有了极简单的证明
realnumber 6# 2013-1-27 18:24
5# yes94 没要求素数,只要求互素,原题答案其实也不复杂,我觉得可能可以用更简单的,就发上来了,比如说可能用剩余系什么的.
realnumber 7# 2013-1-27 19:18
如果找到一个$(a+bk_0,m)=1$,则可以令$k=k_0+tm$,均符合要求. 若$k=0,1,2,...,m-1$,都不与$m$互素,则$k$不存在,所以要么在$k=0,1,2,...,m-1$找到一个,要么构造一个怎么结构,应该能解决问题.
realnumber 8# 2013-1-27 19:32
7# realnumber 令$(a,m)=c$,那么取$k_0=\frac{m}{c}$,翻了下答案,似乎有点接近.
thread-109-1-1.html: Mathematica里的取整函数
kuing 1# 2011-10-18 13:48

kuing 2# 2011-10-18 14:09
Floor 为向下取整,后不带参数时相当于高斯函数 Ceiling 为向上取整 IntegerPart 为整数部分,相应地,还有小数部分函数 FractionalPart Round 为取最近的整数,其中 0.5 向偶数靠
kuing 3# 2011-10-18 14:15
图象 注意这里的图象没法显示端点的情况,所以看上去第二个图象像是第一个的平移,其实上不是。
kuing 4# 2012-5-9 14:16
maple 对 round 函数的定义跟 mathematica 不一样 区别还是在于对 .5 的处理了 别的软件可能也会不一样,有空再试试
thread-1090-1-4.html: [数论] $n^9-n^3\equiv0 \mod{504}$
realnumber 1# 2013-1-27 19:41
求证:$n^9-n^3\equiv0  \mod{504}$. 来自<初等数论100例>.pdf第22题
yes94 2# 2013-1-27 20:07
1# realnumber $504=2^3\cdot3^2\cdot7$ 然后Euler函数即可以解决问题
realnumber 3# 2013-1-27 20:35
2# yes94 过程,这样怎么行,不会.
yes94 4# 2013-1-27 21:06
本帖最后由 yes94 于 2013-1-27 21:24 编辑 3# realnumber 试了一试,输入很麻烦,编辑检查也麻烦,做起来还也麻烦。 因为$\phi(7)=\phi(9)=6$,$\phi(8)=4$, (1)$n^6\equiv1$ $(\mod 7)$,故$n^9\equiv n^3$ $(\mod {7})$, (2)当$n\not\equiv 0,3,6$  $(\mod {9})$时,$n^6\equiv 1$ $(\mod {9})$,故$n^9\equiv n^3$ $(\mod {9})$;      当$n\equiv 0,3,6$ $(\mod {9})$时,$n^4\equiv 1 (\mod {9})$不成立! (3)还有$n^4\equiv1$ $(\mod {8})$还不一定成立,需要条件$n\not\equiv 0,2,4,6$  $(\mod {8})$ 即便成立,如何证这个$n^9\equiv n^3$ $(\mod {7})$?还麻烦呢 笨办法就是穷举$n\equiv 0,1,2,3,4,5,6,7$ $(\mod {8})$ 有没有好一点的办法?
realnumber 5# 2013-1-28 11:58
4# yes94 欧拉函数不会用,我本来是打算分解因式类的做法,万一不行,就用你也想到的笨办法,$\mod 7,8,9$都穷举,貌似计算量也不大,烦琐倒是的.
realnumber 6# 2013-1-28 19:30
$n^9-n^3=n^3(n^6-1)\equiv0 \mod7$, 费马小定理 $n^3(n-1)(n+1)(n^2-n+1)(n^2+n+1)\equiv0 \mod8$,$n$分奇数偶数讨论. $n^3(n-1)(n+1)((n+1)^2-3n)((n-1)^2+3n)\equiv0 \mod9$,$n$按$\mod3$分类. 而$7,8,9$显然两两互素.所以原题成立.
thread-1091-1-4.html: 这就是所谓创新题么
kuing 1# 2013-1-27 21:28
刚才人教群里看到的 第一次见这样的题,这就是所谓创新题?还很和谐呢…… 不过我还是解了一下,但是用了三倍角公式 群管-kuing  21:12:22 三倍角公式…… 群管-kuing  21:15:01 化简右边: \begin{align*} \sin2013^\circ\cdot\sin210^\circ&=(-\sin33^\circ)\cdot(-1/2)\\ &=2\sin11^\circ\sin(60^\circ+11^\circ)\sin(60^\circ-11^\circ)\\ &=2\sin11^\circ\sin71^\circ\sin49^\circ, \end{align*} 所以等式化为 $\sin(x+30^\circ)=\sin49^\circ$,然后观察发现 $x=101^\circ$ 符合……
kuing 2# 2013-1-27 21:37
说起来也巧,那个三倍角公式我用得很少,可是今天我用了两次……
yes94 3# 2013-1-27 21:41
说起来也巧,那个三倍角公式我用得很少,可是今天我用了两次…… kuing 发表于 2013-1-27 21:37 用的很爽哈? 这种概率太小了吧! 该去买彩票了!
kuing 4# 2013-1-27 22:17
3# yes94 运气守恒定律……这时买肯定不中……
yes94 5# 2013-1-27 22:21
3# yes94 运气守恒定律……这时买肯定不中…… kuing 发表于 2013-1-27 22:17
kuing 6# 2013-1-27 23:06
顺便说一下,其实1#的解答还差唯一性的说明,大概就是要把解集中能表示成月日组合的 $x$ 都写出来,然后看看还有没有重要节日的,不过那样实在太无聊,什么才算重要节日?运用集合定义,{重要节日} 是否是一个集合?……
yes94 7# 2013-1-27 23:35
6# kuing 根据重要程度,构成模糊集合——《模糊数学》
thread-1092-1-4.html: [不等式] 来自粉丝群的三个不等式
kuing 1# 2013-1-28 00:18
天书(1846******) 19:30:54 最近遗留的三个不等式: 时间关系明天再玩,第三题大概已经想好怎么玩了…… 第二题真是有点不太敢玩……1000……
pxchg1200 2# 2013-1-28 08:55
本帖最后由 pxchg1200 于 2013-1-28 08:58 编辑 1# kuing Well,I guess the We can kill the second one by Cauchy-Schwarz and AM-GM. :P Having a notice that Cauchy-Schwarz yield that \[ (a^4+b^4+c^4)(a^2b^2+b^2c^2+c^2a^2)\geq (a^3b+b^3c+c^3a)^2 \] That inspire us that: \[ (\sum{a^4})^{4}+4\times 250(\sum{a^2b^2})\geq 5\sqrt[5]{250^{4}\cdot(\sum{a^4})^{4}(\sum{a^2b^2})^{4}} \] then .... I am not sure it's correct or not
kuing 3# 2013-1-28 14:35
先把昨晚想好的第三题写写。 3: 设 $a$, $b$, $c$ 是三角形三边,求证 \[\sqrt{\frac a{b+c}}+\sqrt{\frac b{c+a}}+\sqrt{\frac c{a+b}}<1+\frac{2\sqrt3}3.\] 引理:记方程 $x^3+2x^2-2=0$ 的唯一正数根为 $\lambda(\approx0.839)$,则对任意 $x$, $y>0$, $x+y\leqslant\lambda$,成立如下不等式 \begin{equation}\label{20130128eytzyl} \sqrt{\frac{1-x}{1+x}}+\sqrt{\frac{1-y}{1+y}}<\sqrt{\frac{1-x-y}{1+x+y}}+1. \end{equation} 引理的证明: \begin{align*} \text{式}~\eqref{20130128eytzyl}&\iff \frac{1-x}{1+x}+\frac{1-y}{1+y}+2\sqrt{\frac{1-x}{1+x}}\sqrt{\frac{1-y}{1+y}}<\frac{1-x-y}{1+x+y}+1+2\sqrt{\frac{1-x-y}{1+x+y}}\\ &\iff\sqrt{\frac{1-x}{1+x}}\sqrt{\frac{1-y}{1+y}}-\sqrt{\frac{1-x-y}{1+x+y}}<\frac{x y (2+x+y)}{(1+x) (1+y) (1+x+y)}\\ &\iff\frac{1-x}{1+x}\cdot\frac{1-y}{1+y}-\frac{1-x-y}{1+x+y}<\frac{x y (2+x+y)}{(1+x) (1+y) (1+x+y)}\left(\sqrt{\frac{1-x}{1+x}}\sqrt{\frac{1-y}{1+y}}+\sqrt{\frac{1-x-y}{1+x+y}}\right)\\ &\iff\frac{2(x+y)}{2+x+y}<\sqrt{\frac{1-x}{1+x}}\sqrt{\frac{1-y}{1+y}}+\sqrt{\frac{1-x-y}{1+x+y}}, \end{align*} 根据“糖水不等式”有 \[\frac{1-x}{1+x}\cdot\frac{1-y}{1+y}=\frac{1-x-y+xy}{1+x+y+xy}>\frac{1-x-y}{1+x+y},\] 记 $x+y=t$,则只要证明 \[\frac t{2+t}\leqslant \sqrt{\frac{1-t}{1+t}},\] 两边平方整理等价于 \[\frac{2(t^3+2t^2-2)}{(t+1)(t+2)^2}\leqslant0,\] 由 $0<t\leqslant\lambda$ 可知 $t^3+2t^2-2\leqslant \lambda^3+2\lambda^2-2=0$,从而上式成立,引理得证。 回到原题,设 $a=y+z$, $b=z+x$, $c=x+y$, $x$, $y$, $z>0$,则原不等式等价于 \[\sqrt{\frac{y+z}{2x+y+z}}+\sqrt{\frac{z+x}{2y+z+x}}+\sqrt{\frac{x+y}{2z+x+y}}<1+\frac{2\sqrt3}3,\] 由齐次性及对称性不妨设 $x+y+z=1$ 且 $x\leqslant y\leqslant z$,则必有 $x+y\leqslant 2/3<\lambda$,上式等价于 \[\sqrt{\frac{1-x}{1+x}}+\sqrt{\frac{1-y}{1+y}}+\sqrt{\frac{1-z}{1+z}}<1+\frac{2\sqrt3}3,\] 由引理有 \[\sqrt{\frac{1-x}{1+x}}+\sqrt{\frac{1-y}{1+y}}<\sqrt{\frac{1-x-y}{1+x+y}}+1=\sqrt{\frac z{2-z}}+1,\] 所以只需证 \[\sqrt{\frac z{2-z}}+\sqrt{\frac{1-z}{1+z}}\leqslant \frac{2\sqrt3}3,\] 上式两边平方整理再平方整理最终等价于 \[\frac{4 (2 z-1)^4}{9 (z-2)^2 (z+1)^2}\geqslant 0,\] 所以原不等式得证。
kuing 4# 2013-1-28 16:06
Well,I guess the We can kill the second one by Cauchy-Schwarz and AM-GM. :P Having a notice that Cauchy-Schwarz yield that \[ (a^4+b^4+c^4)(a^2b^2+b^2c^2+c^2a^2)\geq (a^3b+b^3c+c^3a)^2 \] That inspire us that: \[ (\sum{a^4})^{4}+4\times 250(\sum{a^2b^2})\geq 5\sqrt[5]{250^{4}\cdot(\sum{a^4})^{4}(\sum{a^2b^2})^{4}} \] then .... I am not sure it's correct or not pxchg1200 发表于 2013-1-28 08:55 就差取等条件不知能不能满足了,最好举个取等例子出来验证下
kuing 5# 2013-1-28 17:10
才发现第一题原来也是双取等条件的,既然如此,又前天被 pxchg 提起了 SOS-Schur,那我也来试一把吧,还好这次的数据还算简单。 1: 已知 $a$, $b$, $c>0$, $abc=1$,求证 \[\prod\left(3-b-\frac1c\right)+(a+b+c)\left(\frac1a+\frac1b+\frac1c\right)\geqslant10.\] 令 $a=y/x$, $b=z/y$, $c=x/z$, $x$, $y$, $z>0$,则原不等式等价于 \[\prod\left(3-\frac zy-\frac zx\right)+\sum\frac yx\sum\frac xy\geqslant10,\] 由对称性,不妨设 $x=\max\{x,y,z\}$,上式去分母展开并按 $x$ 整理为 \begin{align} &yzx^4+(4y^3-7z^2y-7y^2z+4z^3)x^3+(-7z^3y-7zy^3+27y^2z^2)x^2 \notag\\ &+(z^4y-7z^3y^2-7z^2y^3+zy^4)x+4y^3z^3\geqslant0,\label{20130128shqddjs} \end{align} 注意到取等条件为 $x:y:z=1:1:1$ 或 $x:y:z=2:1:1$ 及其轮换,所以我们假设式 \eqref{20130128shqddjs} 能写成 \[f(x,y,z)(y+z-x)^2(x-y)(x-z)+g(x,y,z)(y-z)^2\geqslant0,\] 注意到 $x$ 的最高次项为 $yzx^4$,所以 $f(x,y,z)$ 只能是 $yz$,代入作差分解可求得 $g(x,y,z)$,再经整理后,最终得到式 \eqref{20130128shqddjs} 化为 \begin{align*} &yz(y+z-x)^2(x-y)(x-z)\\ &+\bigl(yz(xy-yz+zx)+xy(4x-z)(x-z)+xz(4x-y)(x-y)\bigr)(y-z)^2\geqslant0, \end{align*} 显然成立,故原不等式得证。
kuing 6# 2013-1-28 20:30
终究还是要玩一玩第二题,跟 pxchg 的结果似乎不一样…… 2: 设 $a$, $b$, $c\in\mbb R$, $a^3b+b^3c+c^3a=3$,求\[f(a,b,c)=\left(\sum a^4\right)^4+1000\sum a^2b^2\]的最小值。 记 $p=\sum a^4$, $q=\sum a^2b^2$,则由 Vasc 不等式有 \[p+2q=\left(\sum a^2\right)^2\geqslant3\sum a^3b=9,\] 于是 \[f(a,b,c)=p^4+1000q\geqslant p^4+500(9-p)=g(p),\] 求导得 \[g'(p)=4p^3-500=4(p^3-5^3),\] 可见当 $p=5$ 时 $g(p)$ 取最小值,从而 \[f(a,b,c)\geqslant g(5)=2625.\] 下面讨论是否能取等,由上述过程知,取等当且仅当满足 Vasc 不等式的取等条件并且 $p=5$。熟知 Vasc 不等式有两个取等条件,分别为 $a:b:c=1:1:1$ 以及 $a:b:c=\sin^2(4\pi/7):\sin^2(2\pi/7):\sin^2(\pi/7)$ 及其轮换。若取前者则有 $a=b=c=\pm1$ 显然不符合 $p=5$,所以只能取后者。不妨令 $a=\sin^2(4\pi/7)\cdot t$, $b=\sin^2(2\pi/7)\cdot t$, $c=\sin^2(\pi/7)\cdot t$,那么就要满足 \[\left\{\begin{aligned} t^4\left(\sin^6\frac{4\pi}7\sin^2\frac{2\pi}7+\sin^6\frac{2\pi}7\sin^2\frac{\pi}7+\sin^6\frac{\pi}7\sin^2\frac{4\pi}7\right)&=3,\\ t^4\left(\sin^8\frac{4\pi}7+\sin^8\frac{2\pi}7+\sin^8\frac{\pi}7\right)&=5, \end{aligned}\right.\] 事实上,可以计算出 \[\left\{\begin{aligned} \sin^6\frac{4\pi}7\sin^2\frac{2\pi}7+\sin^6\frac{2\pi}7\sin^2\frac{\pi}7+\sin^6\frac{\pi}7\sin^2\frac{4\pi}7&=\frac{147}{256},\\ \sin^8\frac{4\pi}7+\sin^8\frac{2\pi}7+\sin^8\frac{\pi}7&=\frac{245}{256}, \end{aligned}\right.\] 正好有 $147:245=3:5$,所以上述方程组存在实数解 \[t=\pm\frac4{\sqrt7},\] 因此,取等条件为 \[a=\pm\frac4{\sqrt7}\sin^2\frac{4\pi}7,b=\pm\frac4{\sqrt7}\sin^2\frac{2\pi}7,c=\pm\frac4{\sqrt7}\sin^2\frac{\pi}7\] 及其轮换,这样,$f(a,b,c)$ 的最小值的确为 $2625$。 可以说,有点意外,正好有那啥…… 果然不是瞎出的,而且出得很高明,至少我看不出怎么出的……
kuing 7# 2013-1-28 21:10
很想看看标答……@天书 同学看到的话回下
thread-1093-1-4.html: [不等式] (转发)解题群的一个不等式选择题,最大中最小
realnumber 1# 2013-1-28 10:35

goft 2# 2013-1-28 10:57
连加,再均值
realnumber 3# 2013-1-28 11:24
2# goft 得说明理由,以及过程,这样不够照顾新手的意思.
v6mm131 4# 2013-1-28 14:30
贴一下
hnsredfox_007 5# 2013-1-28 15:44
不妨设$$M=\max\left\{x^2+y^2,xy+z,\frac{1}{\sqrt[3]{x^2y^2z}}\right\},$$ 则 \begin{align*} M&\geqslant x^2+y^2,\\ M&\geqslant xy+z,\\ M&\geqslant \frac{1}{\sqrt[3]{x^2y^2z}}, \end{align*} 于是 \begin{align*} M+2M+3M &\geqslant x^2+y^2+2(xy+z)+\frac{3}{\sqrt[3]{x^2y^2z}}\\ &\geqslant 2xy+2xy+2z+\frac{3}{\sqrt[3]{x^2y^2z}} \\ &\geqslant 6\sqrt[3]{x^2y^2z}+\frac{3}{\sqrt[3]{x^2y^2z}}\geqslant 6\sqrt2, \end{align*} 即$$M\geqslant\sqrt2.$$ 显然等号成立条件是$$x=y=\frac{\sqrt[4]{2}}{2},z=\frac{\sqrt2}{2}.$$
yes94 6# 2013-1-28 15:56
5# hnsredfox_007 输入看不清
yes94 7# 2013-1-28 16:05
试试看: $M+2M+3M \geqslant x^2+y^2+2(xy+z)+\frac{3}{\sqrt[3]{x^2y^2z}}\\ \geqslant 2xy+2xy+2z+\frac{3}{\sqrt[3]{x^2y^2z}} \\ \geqslant 6\sqrt[3]{x^2y^2z}+\frac{3}{\sqrt[3]{x^2y^2z}}\geqslant 6\sqrt2$.
yes94 8# 2013-1-28 16:08
7# yes94 原来红狐用的是加权平均。
yes94 9# 2013-1-28 16:41
本帖最后由 yes94 于 2013-1-28 16:54 编辑 原先看到红狐有编辑过的信息:“本帖最后由 &&& 于 2013-1-28 &:& 编辑 ”,奇怪现在怎么没了编辑过的信息?怎么回事? 反倒觉得我6楼乱写的一样, 此贴http://kkkkuingggg.5d6d.net/thread-1067-1-2.html 已经提过这类问题的一般方法:均值法,常用算数均值或几何均值 红狐用了加权算术算数均值,那么为了不重复,我用加权几何均值: 代码书写是个困难事件,改上附件:
kuing 10# 2013-1-28 17:30
原先看到红狐有编辑过的信息:“本帖最后由 &&& 于 2013-1-28 &:& 编辑 ”,奇怪现在怎么没了编辑过的信息?怎么回事? yes94 发表于 2013-1-28 16:41 因为他原先的代码没写好,最后是我编辑了,这论坛有一个地方不好,就是管理员编辑过的话会连痕迹也没了。
yes94 11# 2013-1-28 17:55
10# kuing 哦,我是说呢怎么怪事出现了
kuing 12# 2013-1-28 18:03
11# yes94 嗯,容易引起误会,所以这是个不好的设置,但我也没办法修改它,只能尽量避免了。 如果不是他在Q上跟我说代码不知哪里错,我也不会出手编辑。
yes94 13# 2013-1-28 18:19
12# kuing 后来关注了一下qq群,才发现聊天记录
realnumber 14# 2013-1-28 19:00
特殊到一般 通过考察 1.$a>0,max\{a,\frac{1}{a}\}$的最小值. 2.$a>0,max\{4a,\frac{1}{a}\}$的最小值. 3.$a>0,max\{a,\frac{1}{a^2}\}$的最小值. 可以这样处理, 引理:$max\{a,b,c,d\}\ge \frac{2a+b+c+d}{5}$, 或更一般的$max\{a,b,c,d\}\ge \frac{pa+qb+rc+sd}{m}$,其中正整数$p,q,r,s$满足$p+q+r+s=m$. 由此,可以按2楼的要求直接搭配均值了.其实同5楼办法.
yes94 15# 2013-1-28 19:09
特殊到一般 通过考察 1.$a>0,max\{a,\frac{1}{a}\}$的最小值. 2.$a>0,max\{4a,\frac{1}{a}\}$的最小值. 3.$a>0,max\{a,\frac{1}{a^2}\}$的最小值. 可以这样处理, 引理:$max\{a,b,c,d\}\ge \frac{2a+b+c+d}{5}$ ... realnumber 发表于 2013-1-28 19:00 你那不就是加权均值么?
thread-1094-1-4.html: [不等式] 不等式证明,求助
yayaweha 1# 2013-1-28 17:06
本帖最后由 yayaweha 于 2013-1-28 17:12 编辑 第2问不等式证明
kuing 2# 2013-1-28 17:27
2011 广东理数 20,正好之前写过异于标答的证法,见附件
yes94 3# 2013-1-28 17:27
广东高考题? (1)的答案你要写出来噻!
yayaweha 4# 2013-1-28 17:34
大家玩过?
kuing 5# 2013-1-28 17:37
4# yayaweha 高考题肯定被玩过很多啦,我2#发的那个证法或许也早被发现过了……
yayaweha 6# 2013-1-28 17:46
为什么设b=2c呀?
kuing 7# 2013-1-28 18:00
6# yayaweha 粹纯为了化简得好看些,理论上可以不这样设。
yayaweha 8# 2013-1-28 18:06
这是一个小技巧。我发现这题用GM-HM很简单
kuing 9# 2013-1-28 18:09
标答是用均值的,你可以百度一下
thread-1095-1-10.html: [数列] 一道扬州市填空压轴题
pengcheng1130 1# 2013-1-28 21:59
本帖最后由 pengcheng1130 于 2013-1-28 22:05 编辑 扬州市2012~2013学年度第一学期期末检测高三数学试卷: 数列$\left\{ {{a}_{n}} \right\}$满足${{a}_{1}}>1$,${{a}_{n+1}}-1={{a}_{n}}({{a}_{n}}-1)$,  且$(n\in {{N}^{*}})$,$\dfrac{1}{{{a}_{1}}}+\dfrac{1}{{{a}_{2}}}+\cdots +\dfrac{1}{{{a}_{2012}}}=2$,   求${{a}_{2013}}-4{{a}_{1}}$的最小值.
kuing 2# 2013-1-28 22:11
又是这个FAQ条件……题穷了真是…… \[a_{n+1}-1=a_n(a_n-1) \iff \frac1{a_{n+1}-1}=\frac1{a_n-1}-\frac1{a_n} \iff \frac1{a_n}=\frac1{a_n-1}-\frac1{a_{n+1}-1},\] 所以 \[\frac1{a_1}+\frac1{a_2}+\cdots+\frac1{a_{2012}}=2 \iff \frac1{a_1-1}-\frac1{a_{2013}-1}=2,\] 解得 \[a_{2013}=\frac{2-a_1}{3-2a_1},\] \(\require{cancel}\) $\xcancel{这样看来没有最小值啊}$[更新:请看13#]
pengcheng1130 3# 2013-1-28 22:12
2# kuing FAQ条件指的是什么啊?
pengcheng1130 4# 2013-1-28 22:14
刚刚看了题目,题目没有出错啊!
kuing 5# 2013-1-28 22:18
2# kuing FAQ条件指的是什么啊? pengcheng1130 发表于 2013-1-28 22:12 就是那个 $a_{n+1}-1=a_n(a_n-1)$……在各种考题中见过不知多少次,我大概最早在06年见过,之后年年都拿来出题,后面那个倒数和的下标换个数字又是另一个年份的题了……
kuing 6# 2013-1-28 22:19
4# pengcheng1130 嗯,应该是我还没解完,忘了有范围限制,等下再补充……
pengcheng1130 7# 2013-1-28 22:23
谢谢!今天有点累!我休息了啊!
kuing 8# 2013-1-28 22:23
唔……我突然觉得这题的水很深,深到可能超出了出题人的想象。 数列的每一项由 $a_1$ 唯一确定,所以根据后面的等式,理论上可以将 $a_1$ 解出来,尽管可能不止一个解,但这不会是连续的变量,是离散的一些数值,$a_{2013}$ 也一样,也就是说,所求的式子 $a_{2013}-4a_1$ 只能取若干个固定的值(极可能只有一个,感觉上),我想这肯定是出题人并未曾考虑的事情。 更详细的……等我再慢慢想想……
pengcheng1130 9# 2013-1-28 22:27
有何高见,速速写来!
yes94 10# 2013-1-28 22:32
福州小江也问过啊, 题穷匕首现
pengcheng1130 11# 2013-1-28 22:33
好的啊,等你的高见!
转化与化归 12# 2013-1-28 23:00
我猜命题者的意图
kuing 13# 2013-1-28 23:01
唔……我突然觉得这题的水很深,深到可能超出了出题人的想象。 数列的每一项由 $a_1$ 唯一确定,所以根据后面的等式,理论上可以将 $a_1$ 解出来,尽管可能不止一个解,但这不会是连续的变量,是离散的一些数值,$a_{2013}$ 也一样,也就是说,所求的式子 $a_{2013}-4a_1$ 只能取若干个固定的值(极可能只有一个,感觉上),我想这肯定是出题人并未曾考虑的事情。 更详细的……等我再慢慢想想…… kuing 发表于 2013-1-28 22:23 真的只有一个 $a_1$。 为严格说明,有必要先证明一些东西,即如下引理。 引理:设 $f(x)=x^2-x+1$,记 $f(x)$ 的 $n$ 次迭代为 $f_n(x)$,即 $f_{n}(x)=\underbrace{f(f(f(\cdots f(x)\cdots )))}_{n~\text{个}~f}$,则对任意正整数 $n$,$f_n(x)$ 在 $(1,+\infty)$ 上都是严格递增且大于 $1$。 其实引理很显然,只不过还是想严格点证证。 用归纳法,当 $n=1$,$f(x)$ 显然在 $(1,+\infty)$ 上严格递增且大于 $1$,假设当 $n=k$ 时引理成立,注意到 $f_{k+1}(x)=f(f_k(x))$ 且 $f_n(x)$ 显然可导,则由复合函数求导法则知 \[f_{k+1}'(x)=f'(f_k(x))f'_k(x)=(2f_k(x)-1)f'_k(x),\] 从而当 $x\in(1,+\infty)$ 时由归纳假设可得 $f_{k+1}'(x)>0$,再者易知恒有 $f_n(1)=1$,所以当 $n=k+1$ 时引理也成立,从而由数学归纳法知引理获证。 回到原题,便有 \[\frac1{a_1}+\frac1{a_2}+\cdots +\frac1{a_{2012}}=\frac1{a_1}+\frac1{f(a_1)}+\frac1{f_2(a_1)}+\cdots +\frac1{f_{2011}(a_1)}=g(a_1),\] 由引理知 $g(a_1)$ 在 $(1,+\infty)$ 上严格递减,又显然 $g(1)=2012$ 且 $\lim_{a_1\to+\infty}g(a_1)=0$,从而方程 $g(a_1)=2$ 在 $(1,+\infty)$ 上有且只有一个解,这就证明了满足题目条件的 $a_1$ 有且只有一个。
kuing 14# 2013-1-28 23:09
至于 $a_1$ 是多少,估计没办法解出来,所以……又一经典错题出现。
pengcheng1130 15# 2013-1-29 10:23
学习了啊!!
realnumber 16# 2013-1-29 10:45
本帖最后由 realnumber 于 2013-1-29 10:47 编辑 虽然精确值没法解出来,由2楼等式,可以肯定$a_1≈1.5$,误差不会超过$100亿分之一$,无它,$a_n$增长速度实在快. 改为估计$a_{2003}$的近似值如何,也难得离谱,误差允许100万.
yes94 17# 2013-1-29 15:35
本帖最后由 yes94 于 2013-1-29 15:38 编辑 个人认为版主的想法是对的,$8$楼、$13$楼的分析很有道理。 $a_{2013}$的确可以表示成$a_1$的函数, 但是由方程 $\frac1{a_1}+\frac1{a_2}+\cdots +\frac1{a_{2012}}=2$, 知可以解出$a_1$的值,一般是少数几个解,即是离散的,不是连续的。 例如:由某年的竞赛题,当$a_1=2$时,可以证明 $\dfrac{1}{a_1}+\dfrac{1}{a_2}+\cdots +\dfrac{1}{a_{2012}}$<1, 这与条件$\dfrac{1}{a_1}+\dfrac{1}{a_2}+\cdots +\dfrac{1}{a_{2012}}=2$矛盾。 另外,当$a_1=2$时,$a_{2013}=\dfrac{2-a_1}{3-2a_1}$,故$a_{2013}=0$,由递推式可知,$a_n\equiv 1$($n\ge2014$),这与数列{${a_n}$}单调递增矛盾. 所以,$a_1$的值,不是连续的。 所以,咋一看,这个$a_1$要么不存在,要么存在且唯一 (版主已从正面的角度证明唯一,且$a_1\in(1,+\infty)$ )。 下面说明$a_1\in(1,\dfrac32)$ )。 由条件${a}_{1}>1$知,数列{${a_n}$}单调递增,故$a_n>1$。由 $\frac1{a_1}+\frac1{a_2}+\cdots+\frac1{a_{n}}=\frac1{a_1-1}-\frac1{a_{n+1}-1}$, 可得,$\frac1{a_1}+\frac1{a_2}+\cdots+\frac1{a_{n}}<\frac1{a_1-1}$ 由条件,并结合上式知,$2<\frac1{a_1-1}$,故$1<a_1<\frac32$, 或者像$2$楼或,$12$楼一样可解得,$a_{2013}=\dfrac{2-a_1}{3-2a_1}$, 数列{${a_n}$}单调递增,故$a_{2013}=\dfrac{2-a_1}{3-2a_1}>a_1>1$,也可得$1<a_1<\frac32$ 所以,即便$a_1≈1.5$,也要$a_1<\frac32$ 写了那么多的废话,本想放弃,因为辛苦写了那么多的代码,所以发出去看看代码显示效果
kuing 18# 2013-1-29 18:08
虽然精确值没法解出来,由2楼等式,可以肯定$a_1≈1.5$,误差不会超过$100亿分之一$,无它,$a_n$增长速度实在快. 改为估计$a_{2003}$的近似值如何,也难得离谱,误差允许100万. realnumber 发表于 2013-1-29 10:45 $a_{2003}$ 大概很难估计,来估计一下满足条件的 $a_1$ 大概是多少,看能不能简单证出你所说的“误差不会超过100亿分之一”。 为方便处理,令 $a_n=1+b_n$,则有 $b_{n+1}=b_n^2+b_n$,再记 $b_1=b$,由 $a_1>1$ 得 $b>0$,由前面的讨论知 $a_1<1.5$ 故 $b<0.5$。 显然 $b_n$ 是递增数列,故当 $k\geqslant 2$ 时有 \[ b_{k+1}-b_k=b_k^2>b^2, \] 故当 $n\geqslant 3$ 时,求和得 \[ b_n>b+\sum_{k=1}^{n-1}{b^2}=b+(n-1)b^2, \] 于是又有 \[ b_{k+1}-b_k=b_k^2>(b+(k-1)b^2)^2, \] 再求和得 \begin{align*} b_n&>b+\sum_{k=1}^{n-1}{(b+(k-1)b^2)^2} \\ & =b+\frac16(n-1)(2b^2n^2+(6-7b)bn+6b^2-12b+6)b^2, \end{align*} 再有 \[ b_{k+1}-b_k=b_k^2>\left( b+\frac16(k-1)(2b^2k^2+(6-7b)bk+6b^2-12b+6)b^2 \right)^2, \] 再求和 \begin{align*} b_n&>b+\sum_{k=1}^{n-1}{\left( b+\frac16(k-1)(2b^2k^2+(6-7b)bk+6b^2-12b+6)b^2 \right)^2} \\ & =\cdots, \end{align*} 这时求和出来的数据已经灰常复杂,这里就不写出来了,理论上一直这样求下去可以得到越来越强的下界式子。 我们就取第二次求和得到的那个式子 $b_n>b+\frac16(n-1)(2b^2n^2+(6-7b)bn+6b^2-12b+6)b^2$ 令 $n=2013$,得到 \[ b_{2013}>2712931506b^4+4046132b^3+2012b^2+b, \] 由前面的讨论知 \[ \frac1b-\frac1{b_{2013}}=2 \riff b_{2013}=\frac b{1-2b}, \] 代入化简,最终得到如下不等式 \[ 2712931506b^3-1352419621b^2-2021054b-1005>0, \] 解之得到 \[b>0.49999999852996940461\ldots,\] 所以 \[1.5>a_1>1.49999999852996940461\ldots,\] 只是精确到亿分之一,主要还是中间用的那个式子还是离实际值差很远,估计用第三次求和的那个可能够100亿分之一了。 有空再搞搞 $b_n$ 的上界式,待续……
realnumber 19# 2013-1-29 18:43
18# kuing 我是根据这个来猜测的\[\frac{1}{a_1-1}-\frac{1}{a_{2013}-1}=2\],因为如你分析,原等式左边是$a_1(a_1>1)$的单调增函数,有唯一解 而$a_1=1.4$时,由递推公式(也是$a_n(a_n>1)$的单调递增)依次计算$a_2=1.56,a_3≈1.87,a_4≈2.6,a_5≈5.3,a_6≈23,a_7≈543,a_8≈294496,..$,可估计$\frac{1}{a_{2013}}$几乎是0.更不用说$a_1=1.5$.
thread-1095-1-4.html: [数列] 一道扬州市填空压轴题
pengcheng1130 1# 2013-1-28 21:59
本帖最后由 pengcheng1130 于 2013-1-28 22:05 编辑 扬州市2012~2013学年度第一学期期末检测高三数学试卷: 数列$\left\{ {{a}_{n}} \right\}$满足${{a}_{1}}>1$,${{a}_{n+1}}-1={{a}_{n}}({{a}_{n}}-1)$,  且$(n\in {{N}^{*}})$,$\dfrac{1}{{{a}_{1}}}+\dfrac{1}{{{a}_{2}}}+\cdots +\dfrac{1}{{{a}_{2012}}}=2$,   求${{a}_{2013}}-4{{a}_{1}}$的最小值.
kuing 2# 2013-1-28 22:11
又是这个FAQ条件……题穷了真是…… \[a_{n+1}-1=a_n(a_n-1) \iff \frac1{a_{n+1}-1}=\frac1{a_n-1}-\frac1{a_n} \iff \frac1{a_n}=\frac1{a_n-1}-\frac1{a_{n+1}-1},\] 所以 \[\frac1{a_1}+\frac1{a_2}+\cdots+\frac1{a_{2012}}=2 \iff \frac1{a_1-1}-\frac1{a_{2013}-1}=2,\] 解得 \[a_{2013}=\frac{2-a_1}{3-2a_1},\] \(\require{cancel}\) $\xcancel{这样看来没有最小值啊}$[更新:请看13#]
pengcheng1130 3# 2013-1-28 22:12
2# kuing FAQ条件指的是什么啊?
pengcheng1130 4# 2013-1-28 22:14
刚刚看了题目,题目没有出错啊!
kuing 5# 2013-1-28 22:18
2# kuing FAQ条件指的是什么啊? pengcheng1130 发表于 2013-1-28 22:12 就是那个 $a_{n+1}-1=a_n(a_n-1)$……在各种考题中见过不知多少次,我大概最早在06年见过,之后年年都拿来出题,后面那个倒数和的下标换个数字又是另一个年份的题了……
kuing 6# 2013-1-28 22:19
4# pengcheng1130 嗯,应该是我还没解完,忘了有范围限制,等下再补充……
pengcheng1130 7# 2013-1-28 22:23
谢谢!今天有点累!我休息了啊!
kuing 8# 2013-1-28 22:23
唔……我突然觉得这题的水很深,深到可能超出了出题人的想象。 数列的每一项由 $a_1$ 唯一确定,所以根据后面的等式,理论上可以将 $a_1$ 解出来,尽管可能不止一个解,但这不会是连续的变量,是离散的一些数值,$a_{2013}$ 也一样,也就是说,所求的式子 $a_{2013}-4a_1$ 只能取若干个固定的值(极可能只有一个,感觉上),我想这肯定是出题人并未曾考虑的事情。 更详细的……等我再慢慢想想……
pengcheng1130 9# 2013-1-28 22:27
有何高见,速速写来!
yes94 10# 2013-1-28 22:32
福州小江也问过啊, 题穷匕首现
pengcheng1130 11# 2013-1-28 22:33
好的啊,等你的高见!
转化与化归 12# 2013-1-28 23:00
我猜命题者的意图
kuing 13# 2013-1-28 23:01
唔……我突然觉得这题的水很深,深到可能超出了出题人的想象。 数列的每一项由 $a_1$ 唯一确定,所以根据后面的等式,理论上可以将 $a_1$ 解出来,尽管可能不止一个解,但这不会是连续的变量,是离散的一些数值,$a_{2013}$ 也一样,也就是说,所求的式子 $a_{2013}-4a_1$ 只能取若干个固定的值(极可能只有一个,感觉上),我想这肯定是出题人并未曾考虑的事情。 更详细的……等我再慢慢想想…… kuing 发表于 2013-1-28 22:23 真的只有一个 $a_1$。 为严格说明,有必要先证明一些东西,即如下引理。 引理:设 $f(x)=x^2-x+1$,记 $f(x)$ 的 $n$ 次迭代为 $f_n(x)$,即 $f_{n}(x)=\underbrace{f(f(f(\cdots f(x)\cdots )))}_{n~\text{个}~f}$,则对任意正整数 $n$,$f_n(x)$ 在 $(1,+\infty)$ 上都是严格递增且大于 $1$。 其实引理很显然,只不过还是想严格点证证。 用归纳法,当 $n=1$,$f(x)$ 显然在 $(1,+\infty)$ 上严格递增且大于 $1$,假设当 $n=k$ 时引理成立,注意到 $f_{k+1}(x)=f(f_k(x))$ 且 $f_n(x)$ 显然可导,则由复合函数求导法则知 \[f_{k+1}'(x)=f'(f_k(x))f'_k(x)=(2f_k(x)-1)f'_k(x),\] 从而当 $x\in(1,+\infty)$ 时由归纳假设可得 $f_{k+1}'(x)>0$,再者易知恒有 $f_n(1)=1$,所以当 $n=k+1$ 时引理也成立,从而由数学归纳法知引理获证。 回到原题,便有 \[\frac1{a_1}+\frac1{a_2}+\cdots +\frac1{a_{2012}}=\frac1{a_1}+\frac1{f(a_1)}+\frac1{f_2(a_1)}+\cdots +\frac1{f_{2011}(a_1)}=g(a_1),\] 由引理知 $g(a_1)$ 在 $(1,+\infty)$ 上严格递减,又显然 $g(1)=2012$ 且 $\lim_{a_1\to+\infty}g(a_1)=0$,从而方程 $g(a_1)=2$ 在 $(1,+\infty)$ 上有且只有一个解,这就证明了满足题目条件的 $a_1$ 有且只有一个。
kuing 14# 2013-1-28 23:09
至于 $a_1$ 是多少,估计没办法解出来,所以……又一经典错题出现。
pengcheng1130 15# 2013-1-29 10:23
学习了啊!!
realnumber 16# 2013-1-29 10:45
本帖最后由 realnumber 于 2013-1-29 10:47 编辑 虽然精确值没法解出来,由2楼等式,可以肯定$a_1≈1.5$,误差不会超过$100亿分之一$,无它,$a_n$增长速度实在快. 改为估计$a_{2003}$的近似值如何,也难得离谱,误差允许100万.
yes94 17# 2013-1-29 15:35
本帖最后由 yes94 于 2013-1-29 15:38 编辑 个人认为版主的想法是对的,$8$楼、$13$楼的分析很有道理。 $a_{2013}$的确可以表示成$a_1$的函数, 但是由方程 $\frac1{a_1}+\frac1{a_2}+\cdots +\frac1{a_{2012}}=2$, 知可以解出$a_1$的值,一般是少数几个解,即是离散的,不是连续的。 例如:由某年的竞赛题,当$a_1=2$时,可以证明 $\dfrac{1}{a_1}+\dfrac{1}{a_2}+\cdots +\dfrac{1}{a_{2012}}$<1, 这与条件$\dfrac{1}{a_1}+\dfrac{1}{a_2}+\cdots +\dfrac{1}{a_{2012}}=2$矛盾。 另外,当$a_1=2$时,$a_{2013}=\dfrac{2-a_1}{3-2a_1}$,故$a_{2013}=0$,由递推式可知,$a_n\equiv 1$($n\ge2014$),这与数列{${a_n}$}单调递增矛盾. 所以,$a_1$的值,不是连续的。 所以,咋一看,这个$a_1$要么不存在,要么存在且唯一 (版主已从正面的角度证明唯一,且$a_1\in(1,+\infty)$ )。 下面说明$a_1\in(1,\dfrac32)$ )。 由条件${a}_{1}>1$知,数列{${a_n}$}单调递增,故$a_n>1$。由 $\frac1{a_1}+\frac1{a_2}+\cdots+\frac1{a_{n}}=\frac1{a_1-1}-\frac1{a_{n+1}-1}$, 可得,$\frac1{a_1}+\frac1{a_2}+\cdots+\frac1{a_{n}}<\frac1{a_1-1}$ 由条件,并结合上式知,$2<\frac1{a_1-1}$,故$1<a_1<\frac32$, 或者像$2$楼或,$12$楼一样可解得,$a_{2013}=\dfrac{2-a_1}{3-2a_1}$, 数列{${a_n}$}单调递增,故$a_{2013}=\dfrac{2-a_1}{3-2a_1}>a_1>1$,也可得$1<a_1<\frac32$ 所以,即便$a_1≈1.5$,也要$a_1<\frac32$ 写了那么多的废话,本想放弃,因为辛苦写了那么多的代码,所以发出去看看代码显示效果
kuing 18# 2013-1-29 18:08
虽然精确值没法解出来,由2楼等式,可以肯定$a_1≈1.5$,误差不会超过$100亿分之一$,无它,$a_n$增长速度实在快. 改为估计$a_{2003}$的近似值如何,也难得离谱,误差允许100万. realnumber 发表于 2013-1-29 10:45 $a_{2003}$ 大概很难估计,来估计一下满足条件的 $a_1$ 大概是多少,看能不能简单证出你所说的“误差不会超过100亿分之一”。 为方便处理,令 $a_n=1+b_n$,则有 $b_{n+1}=b_n^2+b_n$,再记 $b_1=b$,由 $a_1>1$ 得 $b>0$,由前面的讨论知 $a_1<1.5$ 故 $b<0.5$。 显然 $b_n$ 是递增数列,故当 $k\geqslant 2$ 时有 \[ b_{k+1}-b_k=b_k^2>b^2, \] 故当 $n\geqslant 3$ 时,求和得 \[ b_n>b+\sum_{k=1}^{n-1}{b^2}=b+(n-1)b^2, \] 于是又有 \[ b_{k+1}-b_k=b_k^2>(b+(k-1)b^2)^2, \] 再求和得 \begin{align*} b_n&>b+\sum_{k=1}^{n-1}{(b+(k-1)b^2)^2} \\ & =b+\frac16(n-1)(2b^2n^2+(6-7b)bn+6b^2-12b+6)b^2, \end{align*} 再有 \[ b_{k+1}-b_k=b_k^2>\left( b+\frac16(k-1)(2b^2k^2+(6-7b)bk+6b^2-12b+6)b^2 \right)^2, \] 再求和 \begin{align*} b_n&>b+\sum_{k=1}^{n-1}{\left( b+\frac16(k-1)(2b^2k^2+(6-7b)bk+6b^2-12b+6)b^2 \right)^2} \\ & =\cdots, \end{align*} 这时求和出来的数据已经灰常复杂,这里就不写出来了,理论上一直这样求下去可以得到越来越强的下界式子。 我们就取第二次求和得到的那个式子 $b_n>b+\frac16(n-1)(2b^2n^2+(6-7b)bn+6b^2-12b+6)b^2$ 令 $n=2013$,得到 \[ b_{2013}>2712931506b^4+4046132b^3+2012b^2+b, \] 由前面的讨论知 \[ \frac1b-\frac1{b_{2013}}=2 \riff b_{2013}=\frac b{1-2b}, \] 代入化简,最终得到如下不等式 \[ 2712931506b^3-1352419621b^2-2021054b-1005>0, \] 解之得到 \[b>0.49999999852996940461\ldots,\] 所以 \[1.5>a_1>1.49999999852996940461\ldots,\] 只是精确到亿分之一,主要还是中间用的那个式子还是离实际值差很远,估计用第三次求和的那个可能够100亿分之一了。 有空再搞搞 $b_n$ 的上界式,待续……
realnumber 19# 2013-1-29 18:43
18# kuing 我是根据这个来猜测的\[\frac{1}{a_1-1}-\frac{1}{a_{2013}-1}=2\],因为如你分析,原等式左边是$a_1(a_1>1)$的单调增函数,有唯一解 而$a_1=1.4$时,由递推公式(也是$a_n(a_n>1)$的单调递增)依次计算$a_2=1.56,a_3≈1.87,a_4≈2.6,a_5≈5.3,a_6≈23,a_7≈543,a_8≈294496,..$,可估计$\frac{1}{a_{2013}}$几乎是0.更不用说$a_1=1.5$.
thread-1097-1-4.html: [不等式] $\max\{1,ab,a+b\}\ge\frac{4}{9}(1+a)(1+b)$,怎么推广?
realnumber 1# 2013-1-29 11:11
本帖最后由 realnumber 于 2013-1-31 18:21 编辑 例4  任意$a,b\in R$,求证:$\max\{1,ab,a+b\}\ge\frac{4}{9}(1+a)(1+b)$. --mathlink上碰到的,做完了,总觉得没领会到什么,怎么推广?或.... 证明:假设存在$a,b\in R$,令$s=1+a,t=1+b.$ 有以下三式同时成立,\[1<\frac{4}{9}st---(1)\]\[s+t-2<\frac{4}{9}st---(2)\]\[(s-1)(t-1)<\frac{4}{9}st---(3)\] 由(1)(3),可得$s>0,t>0$,又$4st\le{(s+t)^2}$----(4). 由(1)(4),可解得$s+t>3$,由(2)(3)解得$s+t<6$. 由(2)(4),可解得$s+t<3或s+t>6$,矛盾.可见原不等式成立.
yes94 2# 2013-1-29 16:02
(4)? 还有,解答过程过于简略, 怎么推广?
realnumber 3# 2013-1-30 18:41
2# yes94 修改好了,其实很简单的一次二次不等式,不需要写过程的.
yes94 4# 2013-1-30 21:08
2# yes94 修改好了,其实很简单的一次二次不等式,不需要写过程的. realnumber 发表于 2013-1-30 18:41 你自己觉得简单罢了, 别人不拿草稿纸恐怕看不出来吧?
realnumber 5# 2013-1-31 13:01
4# yes94 我也是草稿上算的,口算太费力,
kuing 6# 2013-1-31 15:23
先试试3元? PS、max -> \max ;中间那三行公式可以试试用 align 环境,自动编号。
kuing 7# 2013-1-31 16:53
话说,那个 max 和 4/9 让我回想起这道题:http://kkkkuingggg.5d6d.net/thread-388-1-1.html
yes94 8# 2013-1-31 17:45
本帖最后由 yes94 于 2013-1-31 17:52 编辑 7# kuing 这么巧啊? $(a+b)^2\ge4ab$成立。 那不是可以推广为: $\min\{1,ab,a+b\}\leqslant \dfrac14(1+a)(1+b)$
kuing 9# 2013-1-31 18:24
7# kuing 这么巧啊? $(a+b)^2\ge4ab$成立。 那不是可以推广为: $\min\{1,ab,a+b\}\leqslant \dfrac14(1+a)(1+b)$ yes94 发表于 2013-1-31 17:45 应该是,这样看上去两道题说不定有关联……
kuing 10# 2013-1-31 18:36
如果那个贴的条件可以弱化一些(将“正系数”改弱些)就可以利用它来证明1#及8# 先煮饭……
yes94 11# 2013-1-31 19:06
10# kuing 好勤快!妹子最喜欢了!
kuing 12# 2013-2-1 14:39
那个贴子的一般系数情况已经证出,见:http://kkkkuingggg.5d6d.net/view ... &page=1#pid6360 现将该命题引用过来: 设 $a$, $b$, $c\in\mbb R$ 使得关于 $x$ 的方程 $ax^2+bx+c=0$ 有实根,则有 \begin{align*} \max\{a,b,c\}&\geqslant\frac49(a+b+c);\\ \min\{a,b,c\}&\leqslant\frac14(a+b+c). \end{align*} 由此命题知,因为方程 $(x+a)(x+b)=0$ 有实根,展开为 $x^2+(a+b)x+ab=0$,因此有 \begin{align*} \max\{1,a+b,ab\}&\geqslant\frac49(1+a+b+ab)=\frac49(1+a)(1+b),\\ \min\{1,a+b,ab\}&\leqslant\frac14(1+a+b+ab)=\frac14(1+a)(1+b), \end{align*} 这样就得到了 1# 及 8# 的结果。
yes94 13# 2013-2-1 15:02
没想到啊!这个帖子居然和我发的那个帖子有关,可是我就不认识了,即便认识,也不知道怎么办, 还是斑竹厉害!
realnumber 14# 2013-2-2 13:56
瞎蒙一个\[a,b,c\in R,\max{\{1,a+b+c,ab+bc+ca,abc\}}\ge\frac{3^3}{4^3}(1+a)(1+b)(1+c)\].
kuing 15# 2013-2-2 14:32
14# realnumber 前几天试三元的时候就猜这种形式,不过你这个系数肯定不正确。先猜 $a=b=c=t$ 取等,取 $t=1$ 知右边系数不能比 $3/8$ 大,我估计有可能就是 \[\max{\{1,a+b+c,ab+bc+ca,abc\}}\geqslant\frac38(1+a)(1+b)(1+c),\] 不过没证到
kuing 16# 2013-2-2 15:17
你那个猜的时候应该是取 $t=3$ 来猜的吧,也是很合理的想法,因为此时后面两个相等,不过可惜计算发现 $t=1$ 得到的必要条件的更佳。 其实也可以通过画 $\max\{1,3t,3t^2,t^3\}$ 与 $k(1+t)^3$ 的图来找更好的 $k$,用类似方法试了下四元,找到了如下猜测: \[\max\{1,a+b+c+d,ab+ac+ad+bc+bd+cd,abc+abd+acd+bcd,abcd\}\geqslant\frac{216}{625}(1+a)(1+b)(1+c)(1+d),\] 当然,暂时全都是猜测,估计要有突破性的证明方法才能搞定它们
thread-1098-1-1.html: [函数] "三角形函数"转自解题群
realnumber 1# 2013-1-29 11:45
_____kuing edit in $\LaTeX$_____ 已知函数 $f(x)$,若对给定的 $\triangle ABC$,它的三边的长 $a$, $b$, $c$ 均在函数 $f(x)$ 的定义域内,都有 $f(a)$, $f(b)$, $f(c)$ 也为某三角形的三边长,则称 $f(x)$ 是 $\triangle ABC$ 的“三角形函数”。下面给出四个命题: 1、 2、 3、若定义在 $(0,+\infty)$ 上的周期函数 $f_3(x)$ 的值域也是 $(0,+\infty)$,则 $f_3(x)$ 是任意三角形的“三角形函数”; 4、对锐角 $\triangle ABC$,它的三边长 $a$, $b$, $c\in\mbb N^+$,则 $f_4(x)=x^2+\ln x$($x>0$)是锐角 $\triangle ABC$ 的“三角形函数”。 以上命题正确的有_____
kuing 2# 2013-1-29 12:33
又换了个名字,以前曾叫“保三角形函数”,比如见http://bbs.pep.com.cn/thread-523458-1-1.html不过定义有不同,这里是存在性,链接里是任意性
kuing 3# 2013-1-29 14:40
证一下第四个命题。 (1)若 $\triangle ABC$ 为等腰三角形,不妨设 $a=b$,则由锐角三角形以及边长为正整数知 $a=b\geqslant 1$ 且 $1\leqslant c\leqslant \bigl[\sqrt2a\bigr]$($[x]$ 表示高斯函数,下同),此时 $f_4(a)$, $f_4(b)$, $f_4(c)$ 均为正数且 $f_4(a)=f_4(b)$,故要证 $f_4(x)$ 为其“三角形函数”只需证 \[2f_4(a)>f_4(c),\] 因 $f_4(x)$ 递增,故只需证 \[2f_4(a)>f_4\bigl(\bigl[\sqrt2a\bigr]\bigr),\] 即 \[2a^2+2\ln a>\bigl[\sqrt2a\bigr]^2+\ln\bigl[\sqrt2a\bigr],\] 若 $a=1$,则上式等价于 $2>1$ 显然成立,若 $a\geqslant 2$,则由 $\bigl[\sqrt2a\bigr]<\sqrt2a+1$ 且 $\bigl[\sqrt2a\bigr]^2<\bigl[\bigl(\sqrt2a\bigr)^2\bigr]=2a^2$,只需证 \[2\ln a\geqslant \ln\bigl(\sqrt2a+1\bigr),\] 即 \[a^2\geqslant\sqrt2a+1,\] 易证其对所有 $a\geqslant 2$ 成立,所以此时命题成立; (2)若 $\triangle ABC$ 不是等腰三角形,不妨设 $1\leqslant a<b<c$,则由边长为正整数可设 $b=a+t$, $c=a+t+u$,其中 $t$, $u\in\mbb N^+$,因为 $0<f_4(a)<f_4(b)<f_4(c)$,故要证 $f_4(x)$ 为其“三角形函数”只需证 \[f_4(a)+f_4(b)>f_4(c),\] 由锐角三角形知 $a^2+b^2>c^2$,所以只要证 \[\ln a+\ln b\geqslant\ln c \iff a(a+t)\geqslant a+t+u,\] 又由锐角三角形有 \[a^2+b^2>c^2\iff a^2>(c-b)(c+b)\iff a^2>u(2a+2t+u),\] 所以只要证 \[u(2a+2t+u)+at\geqslant a+t+u,\] 整理为 \[(2u+t-1)a+(u-1)(t+u)+tu\geqslant 0,\] 显然成立,所以此时命题也成立。 综合(1)(2),命题四获证。
kuing 4# 2013-1-29 14:56
前两个命题很简单就不说了,至于第三个命题,粉丝群里“地狱的死灵”已经说了,这里引用一下。 地狱的死灵(4040*****) 2013-1-29 10:03:15 设f(x)有周期T, 且f(T)=f(2T)=m>0, 由周期性f(x)在(T,2T]上的值域也是(0,+∞), 所以存在c∈(T,2T)使得f(c)>2m, 取a=T,b=2T 则a,b,c可构成三角形但f(a),f(b),f(c)不能构成三角形
yes94 5# 2013-1-29 16:03
答案?
地狱的死灵 6# 2013-1-29 19:10
总假设a为最短边,c为最长边。 若a=1,则b=c f(a)+f(b)-f(c)=1>0 若a>=2 lna+lnb=ln(ab)>=ln(2b)>=ln(a+b)>lnc, 且a^2+b^2>c^2……
kuing 7# 2013-1-29 19:38
6# 地狱的死灵 OMG!原来可以这么简单,好解……
kuing 8# 2013-1-29 19:44
又看到粉丝群里网友发的这个: 看来跟1#的是一文一理的考题……
yes94 9# 2013-1-29 22:01
姊妹题?
kuing 10# 2013-1-29 22:30
9# yes94 估计同是一地方考题的一文一理
yes94 11# 2013-1-29 22:52
2# kuing 为什么这里是存在性? 总觉得是任意性呢? 虽然是给定的三角形…… 但是给定的三角形并不确定是那种三角形啊? (像$6$楼,“若$a=1$,则$b=c$”,不一定给定三角形是等腰三角形啊?)
kuing 12# 2013-1-29 23:07
11# yes94 那是因为1#命题4中需要任意,所以才是任意。 像8#命题3就是存在性问题
yes94 13# 2013-1-29 23:23
12# kuing 也没仔细看3, 只看了下4, 也看了下题目定义
thread-1098-1-4.html: [函数] "三角形函数"转自解题群
realnumber 1# 2013-1-29 11:45
_____kuing edit in $\LaTeX$_____ 已知函数 $f(x)$,若对给定的 $\triangle ABC$,它的三边的长 $a$, $b$, $c$ 均在函数 $f(x)$ 的定义域内,都有 $f(a)$, $f(b)$, $f(c)$ 也为某三角形的三边长,则称 $f(x)$ 是 $\triangle ABC$ 的“三角形函数”。下面给出四个命题: 1、 2、 3、若定义在 $(0,+\infty)$ 上的周期函数 $f_3(x)$ 的值域也是 $(0,+\infty)$,则 $f_3(x)$ 是任意三角形的“三角形函数”; 4、对锐角 $\triangle ABC$,它的三边长 $a$, $b$, $c\in\mbb N^+$,则 $f_4(x)=x^2+\ln x$($x>0$)是锐角 $\triangle ABC$ 的“三角形函数”。 以上命题正确的有_____
kuing 2# 2013-1-29 12:33
又换了个名字,以前曾叫“保三角形函数”,比如见http://bbs.pep.com.cn/thread-523458-1-1.html不过定义有不同,这里是存在性,链接里是任意性
kuing 3# 2013-1-29 14:40
证一下第四个命题。 (1)若 $\triangle ABC$ 为等腰三角形,不妨设 $a=b$,则由锐角三角形以及边长为正整数知 $a=b\geqslant 1$ 且 $1\leqslant c\leqslant \bigl[\sqrt2a\bigr]$($[x]$ 表示高斯函数,下同),此时 $f_4(a)$, $f_4(b)$, $f_4(c)$ 均为正数且 $f_4(a)=f_4(b)$,故要证 $f_4(x)$ 为其“三角形函数”只需证 \[2f_4(a)>f_4(c),\] 因 $f_4(x)$ 递增,故只需证 \[2f_4(a)>f_4\bigl(\bigl[\sqrt2a\bigr]\bigr),\] 即 \[2a^2+2\ln a>\bigl[\sqrt2a\bigr]^2+\ln\bigl[\sqrt2a\bigr],\] 若 $a=1$,则上式等价于 $2>1$ 显然成立,若 $a\geqslant 2$,则由 $\bigl[\sqrt2a\bigr]<\sqrt2a+1$ 且 $\bigl[\sqrt2a\bigr]^2<\bigl[\bigl(\sqrt2a\bigr)^2\bigr]=2a^2$,只需证 \[2\ln a\geqslant \ln\bigl(\sqrt2a+1\bigr),\] 即 \[a^2\geqslant\sqrt2a+1,\] 易证其对所有 $a\geqslant 2$ 成立,所以此时命题成立; (2)若 $\triangle ABC$ 不是等腰三角形,不妨设 $1\leqslant a<b<c$,则由边长为正整数可设 $b=a+t$, $c=a+t+u$,其中 $t$, $u\in\mbb N^+$,因为 $0<f_4(a)<f_4(b)<f_4(c)$,故要证 $f_4(x)$ 为其“三角形函数”只需证 \[f_4(a)+f_4(b)>f_4(c),\] 由锐角三角形知 $a^2+b^2>c^2$,所以只要证 \[\ln a+\ln b\geqslant\ln c \iff a(a+t)\geqslant a+t+u,\] 又由锐角三角形有 \[a^2+b^2>c^2\iff a^2>(c-b)(c+b)\iff a^2>u(2a+2t+u),\] 所以只要证 \[u(2a+2t+u)+at\geqslant a+t+u,\] 整理为 \[(2u+t-1)a+(u-1)(t+u)+tu\geqslant 0,\] 显然成立,所以此时命题也成立。 综合(1)(2),命题四获证。
kuing 4# 2013-1-29 14:56
前两个命题很简单就不说了,至于第三个命题,粉丝群里“地狱的死灵”已经说了,这里引用一下。 地狱的死灵(4040*****) 2013-1-29 10:03:15 设f(x)有周期T, 且f(T)=f(2T)=m>0, 由周期性f(x)在(T,2T]上的值域也是(0,+∞), 所以存在c∈(T,2T)使得f(c)>2m, 取a=T,b=2T 则a,b,c可构成三角形但f(a),f(b),f(c)不能构成三角形
yes94 5# 2013-1-29 16:03
答案?
地狱的死灵 6# 2013-1-29 19:10
总假设a为最短边,c为最长边。 若a=1,则b=c f(a)+f(b)-f(c)=1>0 若a>=2 lna+lnb=ln(ab)>=ln(2b)>=ln(a+b)>lnc, 且a^2+b^2>c^2……
kuing 7# 2013-1-29 19:38
6# 地狱的死灵 OMG!原来可以这么简单,好解……
kuing 8# 2013-1-29 19:44
又看到粉丝群里网友发的这个: 看来跟1#的是一文一理的考题……
yes94 9# 2013-1-29 22:01
姊妹题?
kuing 10# 2013-1-29 22:30
9# yes94 估计同是一地方考题的一文一理
yes94 11# 2013-1-29 22:52
2# kuing 为什么这里是存在性? 总觉得是任意性呢? 虽然是给定的三角形…… 但是给定的三角形并不确定是那种三角形啊? (像$6$楼,“若$a=1$,则$b=c$”,不一定给定三角形是等腰三角形啊?)
kuing 12# 2013-1-29 23:07
11# yes94 那是因为1#命题4中需要任意,所以才是任意。 像8#命题3就是存在性问题
yes94 13# 2013-1-29 23:23
12# kuing 也没仔细看3, 只看了下4, 也看了下题目定义
thread-1099-1-1.html: [数论] 初等数论学习贴--不定方程
realnumber 1# 2013-1-29 12:02
本帖最后由 realnumber 于 2013-3-24 07:38 编辑 声明:本人只是业余爱好者,只是自己学习下,也不清楚碰到的问题难易,没解错就不错了,. 未解决的:7楼; 15楼 ;19楼;20楼 ;21楼,27楼,28楼,36楼 已经解答的:3楼,4楼,8楼,9楼,13楼,22楼(严文兰老师),23楼,24楼. \begin{array}{|c|c|c|c|c|c|c|c|c|} \hline\\\mod& 2  & 3  & 4  & 5  & 6  & 7  & 8  &  9 & 10 \\ \hline\\ 2^x &0 &-1,1&2,0,0&2,-1,-2,1&2,-2,&2,-3,1&2,4,0,0 &2,4,-1,-2,-4,1&2,4,-2,-4  \\ \hline\\ 3^x & 1 & 0 &-1,1  & -2,-1,2,1&3&3,2,-1,-3,-2,1& 3,1&3,0,0&3,-1,-3,1\\ \hline\\ 4^x &  & 1& 0 &-1,1&-2 &-3,2,1&4,0,0&4,-2,1&4,-4 \\ \hline\\ 5^x &  &  & 1 & 0&-1,1&-2,-3,-1,2,3,1&-3,1&-4,-2,-1,4,2,1&5 \\ \hline\\ 6^x &  &  &  & 1 & 0&-1,1&-2,4,0,0&-3,0,0&-4\\ \hline\\ 7^x &  &  &  & & 1&0&-1,1&-2,4,1 &-3,-1,3,1\\ \hline\\ 8^x & &  &  & &&1&0&-1,1&-2,4,2,-4 \\ \hline \end{array} ---突然发现求解时,基本重新算的,表格根本没看,鸡肋,好在可以放在这里唬下新手.
yes94 2# 2013-1-29 15:41
数论很难搞啊!很多未解决的题没有特定的模式,
realnumber 3# 2013-2-2 09:55
本帖最后由 realnumber 于 2013-2-3 12:53 编辑 $3^y-2^x=1,x,y\in Z_+$求证只有两组解(1,1),(3,2)----解答见6楼
realnumber 4# 2013-2-2 09:57
本帖最后由 realnumber 于 2013-2-5 09:45 编辑 $3^x+4^y=5^z$,除了$(2,2,2)$还有其它正整数解吗?并证明.---解答见12楼.
abababa 5# 2013-2-2 12:17
4# realnumber 问了一位网友,说已经证明了没有其它正整数解。证明没细讲,只讲了关键的先模3,再模8,呵呵,没看懂,大家帮看看这思路。
realnumber 6# 2013-2-2 13:33
本帖最后由 realnumber 于 2013-2-5 10:43 编辑 3# realnumber $3^y-2^x=1,x,y\in Z_+$求证只有两组解(1,1),(3,2) $x=1,2,3$单独检验后,$x\ge4$时,两边取$\mod4$,可得$(-1)^y\equiv1$,所以$y$为偶数,设$y=2t,$. 两边取$\mod3$,可得$-(-1)^x\equiv1$,所以$x$为奇数,设$x=2s+1,s\ge2$. 得到$9^t=2\times4^s+1$,考虑两边的个位数(即$\mod10$),可得$s,t$均为奇数,设$s=2m+1,t=2n+1,m\ge1,n\ge0$. 因此$9\times81^n=8\times16^m+1$,两边$\mod16$,可得$9=1$矛盾.所以原不定方程只有2组解. 多次取模的办法学习自这个帖子.
realnumber 7# 2013-2-2 13:37
求不定方程$y^2=x^3+1$的所有正整数解,来自Joseph2338.
realnumber 8# 2013-2-2 13:49
本帖最后由 realnumber 于 2013-2-5 09:22 编辑 4# realnumber 不怕数据大的话,似乎编n个方程,比如求$5^x+12^y=13^z$的所有正整数解.不知道方法是否都类似?---解答在17楼,至少2题都用了类似办法.
realnumber 9# 2013-2-3 12:33
本帖最后由 realnumber 于 2013-2-3 12:55 编辑 求方程$3^x=2^y-1$的所有正整数解.---解答见10楼.
realnumber 10# 2013-2-3 12:40
本帖最后由 realnumber 于 2013-2-5 09:46 编辑 9# realnumber 求方程$3^x=2^y-1$的所有正整数解. 依次检验$y=1,2,3,4,$显然$(1,2)$是一组解 当$y\ge 5$时,两边$\mod16$,右边是$-1$,左边是$3,9,11,1$矛盾.所以$(1,2)$是唯一一组解.
kuing 11# 2013-2-3 12:44
表格太长,这里放不下了……
realnumber 12# 2013-2-3 12:50
本帖最后由 realnumber 于 2013-2-5 11:40 编辑 4# realnumber 求$3^x+4^y=5^z$的所有正整数解. 两边$\mod4$,可得$(-1)^x\equiv1$,所以$x$为偶数,设$x=2r,r\in Z_+$. 两边$\mod3$,可得$z$为偶数,设$z=2t,t\in Z_+$. 所以$4^y=(5^t-3^r)(5^t+3^r)$,则存在$m,n\in N,m\ge n$,有 \begin{cases}5^t-3^r=2^n \\5^t+3^r=2^m \end{cases} 解得\begin{cases}5^t=2^{m-1}+2^{n-1} \\3^r=2^{m-1}-2^{n-1} \end{cases} 因为3,5为奇数,所以$n-1=0$,得$3^r=2^{m-1}-1$.-------(*) 依次检验$m-1=1,2,3,4,$显然$r=1,m=3$是一组解 当$m\ge 5$时,两边$\mod16$,右边是$-1$,左边是$3,9,11,1$矛盾.所以$(1,3)$是唯一一组解. 也即原方程$(2,2,2)$是唯一一组解.
realnumber 13# 2013-2-3 21:43
诸如这类不定方程$51x+33y=17$的整数解情况.$(51,33)=3$,而$17$不被3整除,所以无解. 求$5x+2xy-3y=4$的整数解 -----来自<不定方程的整数解和填数法>
yes94 14# 2013-2-3 21:54
13# realnumber 这类方程较简单,已有定法: 求$5x+2xy−3y=4$的整数解 解为:$(x,y)=(1,1)$或$(-2,-2)$
realnumber 15# 2013-2-3 22:53
14# yes94 这本书上还写者$x^3-y^2=7$,的一组解(32,181),(也没有说是否唯一),烦琐,没心情看下去,留着以后试试.
yes94 16# 2013-2-3 23:33
14# yes94 这本书上还写者$x^3-y^2=7$,的一组解(32,181),(也没有说是否唯一),烦琐,没心情看下去,留着以后试试. realnumber 发表于 2013-2-3 22:53 显然,$(x,y)=(2,1)$或$(2,-1)$也可以噻,
realnumber 17# 2013-2-5 08:55
本帖最后由 realnumber 于 2013-2-8 15:55 编辑 8# realnumber 求$5^x+12^y=13^z$的所有正整数解.      两边$\mod6$,得到$(-1)^x\equiv1$,所以$x$为偶数,设$x=2r,r\in Z^+$. 两边$\mod 10$,三项个位数依次为$5;2,4,6,8;3,9,7,1$,由此可得$y,z$同为奇数,或同为偶数. 1.若$y,z$同为偶数,设$y=2s,z=2t,s,t\in Z^+$. 则有,$25^r=(13^t-12^s)(13^t+12^s)$,则存在$m,n\in N,m\ge n$,有 \begin{cases}13^t-12^s=5^n \\13^t+12^s=5^m \end{cases} 解得$2\times 13^t=5^m+5^n$,$13$不是$5$的倍数,所以$n=0$, 即有$13^t-12^s=1$,(1,1)显然是它的一组解,以下证明是唯一一组解.-------(*) 当$s\ge2$时,取$\mod11$,得$2,4,-3,5,-1,-2,-4,3,-5,1\equiv2$可得$t=10k+1,k\in N$, 取$\mod 16$,即得$5,-3\equiv1$,矛盾. 2.若$y,z$同为奇数,设$y=2s+1,z=2t+1,s,t\in N$. 则有$25^r+12\times 144^s=13\times 169^t$ 取$\mod 16$,可得$(-7)^r+12\times 8^s=(-3)\times (-7)^t$,可得$s=0$, 此时有$25^r+12=13\times 169^t$,取$\mod 13$,得到$r$为偶数,设$r=2m,m\in Z^+$, 取$\mod 10$,得到$t$为奇数,设$t=2n+1,n\in N$ 所以有$625^m+12=13\times 169^{2n+1}$ 两边$\mod17$,可得$(-4)^m-5\equiv4$,即"$-4$或$-1$或$4$或$1$"$-5\equiv4$矛盾. 所以$(2,2,2)$是原题唯一一组解. $ps,17$是这样想到要去尝试的,$169^2-1$含有因子$17$,更进一步的原因,不知道;又取$\mod16$,还是觉得莫名其妙,虽然似乎也很关键. 果然经验累积多了,会有改观.
realnumber 18# 2013-2-5 09:24
14# yes94 书写不多的话,写写看~~~
realnumber 19# 2013-2-5 09:30
继续丢个,近期不想再去处理,想换个口味. 求$7^x+24^y=25^z$所有正整数解,
realnumber 20# 2013-2-5 09:34
本帖最后由 realnumber 于 2013-2-5 10:03 编辑 求所有正整数解$9^x+40^y=41^z$,哈哈,无穷无尽.哪个看得不爽的,解决终极的,设正常数$a,b,c,(a,b,c)=1,(a,b,c)$是一组勾股数, 求$a^x+b^y=c^z$的所有正整数解.
thread-1099-2-1.html:
realnumber 21# 2013-2-5 10:11
继续换个方式折腾 求所有正整数解,$x^2+4^y=25$的所有正整数解.或者更复杂点的,$x^2+4^y=5^z$的所有正整数解.
realnumber 22# 2013-2-5 10:24
本帖最后由 realnumber 于 2013-2-5 10:47 编辑 (转)严文兰老师的一个解答的网址.
realnumber 23# 2013-2-5 11:22
本帖最后由 realnumber 于 2013-2-8 15:50 编辑 22# realnumber 试下一直用取模的办法,---呼~~终于也完成,从楼上学了不少. 见楼上,设$x=2r,y=2s,z=2t,r,s,t\in Z^+$, 得到$21^y=(29^t-20^r)(29^t+20^r)$ 取$\mod7$,     $0\equiv (1-(-1)^r)(1+(-1)^r)$ 而$(1-(-1)^r),(1+(-1)^r)$有且仅有一项被$7$整除,------(1) 又由$(29^t-20^r,29^t+20^r)=(2\times 29^t,29^t+20^r)=1$ 1.当$r$为偶数时,$r=2m,m\in Z^+$, \begin{cases}29^t+20^{2m}=3^y \\ 29^t-20^{2m}=7^y\end{cases}    取$\mod8$,得到$t$为偶数,设$t=2n$,那么得到$(29^n-20^m)(29^n+20^m)=7^y$, 得到$7\mid 29^n-20^m,7\mid 29^n+20^m$而这与(1)矛盾. 所以$r$为偶数时,原方程无解. 2.当$r$为奇数时,$r=2m+1,m\in N$, \begin{cases}29^t+20^{2m+1}=49^s \\ 29^t-20^{2m+1}=9^s\end{cases} 取$\mod3,\mod5$后,得$s,t$都为奇数,再取$\mod16$,得到$m=0$. 即 \begin{cases}29^t+20=49^s \\ 29^t-20=9^s\end{cases} 相减,得到$40=49^s-9^s=(49-9)(49^{s-1}+\cdots+9^{s-1})$,因此$s=1$, 如此原方程只有解$(2,2,2).$
realnumber 24# 2013-2-6 16:49
本帖最后由 realnumber 于 2013-2-6 17:29 编辑 方程$x^2+y^2=6$有理解是否存在.问题来自张云华老师的博客--解答见2楼下.
realnumber 25# 2013-2-6 17:01
本帖最后由 realnumber 于 2013-2-6 17:41 编辑 24# realnumber 方程$x^2+y^2=6$有理解是否存在.问题来自张云华老师的博客 \[x=\frac{n}{m},y=\frac{t}{s},m,n,s,t\in Z,(m,n)=(s,t)=1\] 得到$(ns)^2+(mt)^2=6(ms)^2$,------(1) 注意到$m^2\mid 6(ms)^2,m^2\mid (mt)^2$,所以$m^2\mid (ns)^2$,又$(m,n)=1$,所以$m^2\mid s^2$.同样可以得到$s^2\mid m^2$,所以有$m^2=s^2$, 因此方程(1)等价于$n^2+t^2=6s^2,(n,s)=(t,s)=1.n,s,t\in Z$,-----(2) 若$3\mid n$由$3\mid 6s^2$,可得$3\mid t$,则$9\mid n^2,9\mid t^2$,所以$9\mid 6s^2$,则$3\mid s^2$,$3\mid s$ 那么与$(s,t)=1$,矛盾,所以$3\nmid n$且$3\nmid t$ 方程(2)两边取$\mod3$,得到$2\equiv0$矛盾,所以(2)无整数解,也即原方程无有理数解.
realnumber 26# 2013-2-6 20:44
卡塔兰定理
realnumber 27# 2013-2-6 20:53
本帖最后由 realnumber 于 2013-2-6 20:56 编辑 试证明$x^3+y^3=z^3$无正整数解.--来自<不定方程的整数解和填数法>,其实是费马大定理特例.
realnumber 28# 2013-2-6 20:59
试证明$x^5+y^5=z^5$无正整数解.
yes94 29# 2013-2-6 21:18
28# realnumber 无穷递降法? 它和最小数原理等价
realnumber 30# 2013-2-6 21:27
29# yes94 读大学上课时,好象听说过,现在基本忘了,你可以试试.我是先收集着,不知深浅,有兴趣时来想.
realnumber 31# 2013-2-6 22:01
本帖最后由 realnumber 于 2013-2-6 22:04 编辑 解答和题目来自<初等数论100例>第28题 求$x^3+y^3+z^3-3xyz=0$的所有整数解. $x^3+y^3+z^3-3xyz=0.5(x+y+z)((x-y)^2+(x-z)^2+(y-z)^2)$ 可以证明$x=y=z=t,t\in Z,x=u+v,y=-u,z=-v,u,v\in Z$是所有整数解.
yes94 32# 2013-2-7 15:01
31# realnumber 这个倒蛮简单的,以前用这个恒等式来证明, 当$a、b、c\in R^+$时,有不等式$a^3+b^3+c^3\geqslant 3abc$
kuing 33# 2013-2-7 15:03
32# yes94 总算有看得懂的了……
yes94 34# 2013-2-7 16:42
33# kuing 我也是,
realnumber 35# 2013-2-7 17:19
本帖最后由 realnumber 于 2013-2-7 17:20 编辑 34# yes94 上面证明基本思路很单一,就"取模",其实你俩能懂的,$\mod3$就是把自然数分成三类,$3k,3k+1,3k+2$.下面举例 25楼部分:$n^2+t^2=6s^2,(n,s)=(t,s)=1.n,s,t\in Z$,-----(2) 若$3\mid n$由$3\mid 6s^2$,可得$3\mid t$,则$9\mid n^2,9\mid t^2$,所以$9\mid 6s^2$,则$3\mid s^2$,$3\mid s$ 那么与$(s,t)=1$,矛盾,所以$3\nmid n$且$3\nmid t$ 方程(2)两边取$\mod3$,得到$2\equiv0$矛盾,所以(2)无整数解,也即原方程无有理数解. -----就是对$n=3k+1,n=3k+2;t=3l+1,t=3l+2,k,l\in Z$,你把这些代入等式(2)看看,左边被3除余2,右边是3的倍数.
realnumber 36# 2013-3-24 07:36
来自14楼末尾 初等方法求证:$1+x^4=2y^4$有唯一正整数解.(要求初等方法,搜索到的办法连符号都看不懂.)
realnumber 37# 2013-5-21 14:54
转自QQ群,有空再看
李斌斌755 38# 2013-5-21 15:04
曾经看过“模”知识,如啃天书,无奈放弃!
realnumber 39# 2013-5-21 15:34
38# 李斌斌755 不习惯记号而已,觉得开始难度就算术除法乘法相当,也就是说小学3,4年级.
thread-11-1-1.html: $\lim_{n\to+\infty}a_n$ 与 $\lim_{n\to+\infty}\frac{a_1+\cdots+a_n}{n}$
kuing 1# 2011-9-26 17:58
求证:若 $\displaystyle\lim_{n\to+\infty}a_n=a$ 则 $\displaystyle\lim_{n\to+\infty}\frac{a_1+\cdots+a_n}{n}=a$。 反之如何? 取 $a_n=(-1)^n$
海盗船长 2# 2011-9-29 17:10
反例 $a_{2n+1}=2a$ $a_{2n}=0$
kuing 3# 2011-9-29 17:26
2# 海盗船长 :D 这个更好
海盗船长 4# 2011-9-29 17:26
本帖最后由 海盗船长 于 2011-9-29 17:40 编辑 但可以证明 如果 $\displaystyle \lim_{n \to \infty} \frac{\sum_{i=1}^n a_{i}}{n}=a$ 就有 $\displaystyle \lim_{n \to \infty} \frac{a_{n}}{n}=0$
海盗船长 5# 2011-9-29 17:34
4# 海盗船长 能不能把字体弄大一点?
kuing 6# 2011-9-29 17:35
但可以证明 如果 $\lim_{n \to \infty} \frac{\sum_{i=1}^n a_{i}}{n}=a$ 就有 $\lim_{n \to \infty} \frac{x_{n}}{n}=0$ 海盗船长 发表于 2011-9-29 17:26 $x_n$ 是啥?
kuing 7# 2011-9-29 17:35
4# 海盗船长 能不能把字体弄大一点? 海盗船长 发表于 2011-9-29 17:34 你指的是公式还是什么?
海盗船长 8# 2011-9-29 17:35
哦,是$a_n$。。
海盗船长 9# 2011-9-29 17:36
7# kuing 公式
kuing 10# 2011-9-29 17:37
9# 海盗船长 可以在公式前加 \displaystyle,或者用行间公式模式。
海盗船长 11# 2011-9-29 17:39
哦,我来试试
尐蒓江华 12# 2011-10-21 00:10
可以利用O.Stolz定理  这个定理在微积分教程(菲赫金哥尔茨著作)P50
pxchg1200 13# 2011-10-21 22:42
12# 尐蒓江华 谁都知道啦,。。。。
秋风树林 14# 2011-10-28 23:41
我学的那本复旦版的数学分析上全都有。。。
icesheep 15# 2011-11-10 12:48
后者又被称为 a[n] 的 Cesaro 极限(区别于Cauchy极限)
叶文明 16# 2011-12-20 16:26
由stolz定理可证得。
thread-110-1-9.html: [不等式] 群里看到的简单根式不等式$\sum\sqrt{x/(3-2x)}>1$
kuing 1# 2011-10-18 15:27
证:依题意有 \[\sum{\sqrt{\frac{x}{3-2x}}}=\sum{\sqrt{\frac{x}{3(x+y+z)-2x}}}=\sum{\sqrt{\frac{x}{x+3y+3z}}},\] 由 Holder 不等式,有 \[\left( \sum{\sqrt{\frac{x}{x+3y+3z}}} \right)^{2}\sum{x^{2}(x+3y+3z)}\geqslant \left( \sum{x} \right)^{3},\] 所以要证原不等式只需证明 \[\left( \sum{x} \right)^{3}>\sum{x^{2}(x+3y+3z)},\] 上式展开为 $6xyz>0$ 显然成立,故得证。
kuing 2# 2011-10-18 19:00
顺便指出,当 $x,y\to0,z\to1$ 时原式 $\to1$,这说明 1 已经是下确界。 若将 $x,y,z\in\mathbb{R}^+$ 改为 $x,y,z\geqslant0$,那么取等条件就为 $(x,y,z) = (0,0,1)$。 另外,原式还有最大值 $\dfrac3{\sqrt7}$,有空再证,如果你有兴趣,一起玩。
zwl1972 3# 2011-10-18 20:55
本帖最后由 zwl1972 于 2011-10-18 21:03 编辑 上题启发编一简单题,大家(高手kuing除外)练练手 若$x,y,z\in \mathbb{R}^+$且$4x+4y+4z=9$,求证: \begin{equation*}     \sqrt{\frac{x}{3-2x}}+\sqrt{\frac{y}{3-2y}}+\sqrt{\frac{z}{3-2z}}\ge \frac{3}{\sqrt{2}} \end{equation*}
pxchg1200 4# 2011-10-18 23:52
上题启发编一简单题,大家(高手kuing除外)练练手 若$x,y,z\in \mathbb{R}^+$且$4x+4y+4z=9$,求证: \begin{equation*}     \sqrt{\frac{x}{3-2x}}+\sqrt{\frac{y}{3-2y}}+\sqrt{\frac{z}{3-2z}}\ge \frac{3}{\sq ... zwl1972 发表于 2011-10-18 20:55 proof: by Holder: \[ (\sum{ \sqrt{\frac{x}{3-2x}}})^{2}(\sum{x^{2}(3-2x)})\geq (x+y+z)^{3}   \] it's suffices to prove that: :\[ (x+y+z)^{3} \geq \frac{9}{2}(\frac{4}{3}\sum{x^{2}(y+z)}-\frac{2}{3}\sum{x^{3}})\] after expand gives \[ 8 \sum{x^{3}}+12xyz \geq 6\sum{x^{2}(y+z)}\] by Schur \[ 4\sum{x^{3}}+12xyz \geq 4 \sum{x^{2}(y+z)} \] and AM-GM \[ 4\sum{x^{3}}\geq 2 \sum{x^{2}(y+z)} \] sum it up,the result follows. Done!
kuing 5# 2011-10-18 23:53
4# pxchg1200 看看2#我说那最大值有什么好办法?
pxchg1200 6# 2011-10-18 23:55
什么时候取等号啊?那个最大值
kuing 7# 2011-10-18 23:59
6# pxchg1200 x=y=z=1/3 咯 我用凹凸性看了下是成立的(半凹半凸定理的思想,只要证二元相等时,这通过平方再平方是可以出来的),暂时没想到简单方法。
pxchg1200 8# 2011-10-19 00:03
你的切线法呢? 应该有用吧
kuing 9# 2011-10-19 00:05
8# pxchg1200 切不来,在另一边的可不是小区间……
pxchg1200 10# 2011-10-19 00:10
悲剧,Cauchy-Schwarz不会用啊!
zwl1972 11# 2011-10-19 15:49
本帖最后由 zwl1972 于 2011-10-19 22:07 编辑 proof: by Holder: \[ (\sum{ \sqrt{\frac{x}{3-2x}}})^{2}(\sum{x^{2}(3-2x)})\geq (x+y+z)^{3}   \] it's suffices to prove that: :\[ (x+y+z)^{3} \geq \frac{9}{2}(\frac{4}{3}\sum{x^{2}(y+z)}-\frac{2} ... pxchg1200 发表于 2011-10-18 23:52 若$x,y,z\in \mathbb{R}^+$且$4x+4y+4z=9$,求证: \begin{equation*}     \sqrt{\frac{x}{3-2x}}+\sqrt{\frac{y}{3-2y}}+\sqrt{\frac{z}{3-2z}}\ge \frac{3}{\sqrt{2}} \end{equation*} we kill it only a line: From GM-HM , $\sqrt{ab}\ge \dfrac{2}{\dfrac{1}{a}+\dfrac{1}{b}}$ ,so   \begin{equation*} \sum_{cyc}\sqrt{\frac{1}{2}\cdot\frac{x}{3-2x}}\ge \sum_{cyc}\frac{2}{2+\dfrac{3-2x}{x}}=\sum_{cyc}\frac{2x}{3}=\frac{3}{2} \end{equation*}
kuing 12# 2011-10-19 15:58
11# zwl1972 原来是为了凑取等条件才把和改成9/4 PS。刚才冒号和美元符号连起来变成了表情,我把原先默认的表情禁用了,现在可以显示了,以后不必担心这个连用问题
kuing 13# 2011-10-19 16:14
关于最大值,即 \[x,y,z\geqslant0,x+y+z=1,\sum{\sqrt{\frac{x}{x+3y+3z}}}\leqslant\frac3{\sqrt7}\] 某群有两个证法,有空慢慢看下 PS。由于下图出现真名,为免不必要的麻烦,作了打码处理,这里仅补充网名(希望没搞错):第一个是 tian27546 ,第二个是 srr345(或 德雷纳特)。 取等条件的确是有两个的,我昨天证两个相等时也得到了另一个取等条件,忘记说了 而 srr345 的证明似乎还没完,最后的系数并非恒非负。
pxchg1200 14# 2011-10-19 23:43
待定柯西强是强不过运算量还不是一般的大啊
kuing 15# 2011-10-19 23:47
14# pxchg1200 之前蔡是不是也很喜欢这样搞……
pxchg1200 16# 2011-10-20 16:07
本帖最后由 pxchg1200 于 2011-10-20 16:12 编辑 嗯,好像是的,不过最先看到的是Can这样搞了。然后都跟着这样了,听说还有待定二次式的,(见过陈计用了一次).不过运算量就更上一层楼了。。。 attachment: Let  $a,b,c \geq 0 $ prove that: \[ \frac{a}{a+\sqrt{a^{2}+3bc}}+\frac{b}{b+\sqrt{b^{2}+3ac}}+\frac{c}{c+\sqrt{c^{2}+3ab}}\leq 1 \] (Ji Chen 就是用待定二次柯西把这题秒了。。。 由于解法过于暴力,恕不转载 )
kuing 17# 2011-10-20 16:19
16# pxchg1200
pxchg1200 18# 2011-10-20 22:57
本帖最后由 pxchg1200 于 2011-10-20 23:00 编辑 17# kuing Ok,还是转了吧,看见kuing晕了 $ \sum{a^{2}\sqrt{a^{2}+3bc}}\leq a^{3}+b^{3}+c^{3}+3abc. $ $ \left(\sum{a^{2}\sqrt{a^{2}+3bc}}\right)^{2}\leq\sum{\frac{a^{2}}{b^{2}+c^{2}-bc+ca+ab}}\sum{a^{2}(a^{2}+3bc)(b^{2}+c^{2}-bc+ca+ab)} $ $ = (a^{3}+b^{3}+c^{3}+3abc)^{2}-\frac{a^{2}b^{2}c^{2}(a+b+c)^{2}(a^{2}+b^{2}+c^{2})(a^{2}+b^{2}+c^{2}-bc-ca-ab)}{\prod{(b^{2}+c^{2}-bc+ca+ab)}} $ $ \leq (a^{3}+b^{3}+c^{3}+3abc)^{2}. $ (黄不?暴力不?....)
kuing 19# 2011-10-20 23:00
18# pxchg1200 mathlinks里的所有旧贴,引用的话 + - = 会变成 \plus{} \minus{} \equal{} ,不知为什么。但是点击公式的话会显示正常。
pxchg1200 20# 2011-10-20 23:02
19# kuing 现在正常了。呵呵,不过那个计算。。。
thread-110-2-9.html:
kuing 21# 2011-10-20 23:06
20# pxchg1200 暂时无法理解的程度,只能得个看了……
thread-1100-1-4.html: [函数] 广东六校联考题
yayaweha 1# 2013-1-29 17:35
看其中F(x)=lnx+1/x
yayaweha 2# 2013-1-29 17:41
2013广东省高三六校第一次联考,第三问那个标准答案的讨论范围不好分析,不知道有没有直接求的方法
yes94 3# 2013-1-29 22:02
2# yayaweha 答案先贴出来看看,
yes94 4# 2013-1-29 22:08
3# yes94 $F(x)=lnx+\dfrac1x$的图像:
yes94 5# 2013-1-29 22:12
本帖最后由 yes94 于 2013-1-29 22:23 编辑 4# yes94 从图可以看出,$m\in(0,1)$是恒成立的,
yes94 6# 2013-1-29 22:21
本帖最后由 yes94 于 2013-1-29 22:24 编辑 5# yes94 显然,$F(x)$在$x\in(1,+\infty)$是单调递增的, 并且$\alpha+\beta=x_1+x_2$, 若$m\notin(0,1)$,则$\alpha$和$\beta$分居在$x_1和x_2$的两旁,则题目不等式反向, 故从正反两个方面得到,$m$的取值范围是$(0,1)$, 不知道对不对?现在是凭感觉做题了,不想打草稿了, 这就是草稿,现在修改第一次,
yayaweha 7# 2013-1-29 23:10
6# yes94 YES,I think it too. 但是如果是考试那该怎么说
kuing 8# 2013-1-29 23:15
同3#……
yayaweha 9# 2013-1-29 23:17
答案直接去百度文库搜就有了
thread-1101-1-4.html: [几何] 轨迹问题
Gauss门徒 1# 2013-1-30 01:31
已知非等腰$\Delta ABC$中有内点$P$满足$\angle PBA=\angle PCA$求证:$P$的轨迹是双曲线
yes94 2# 2013-1-30 14:18
差点以为那是什么布勃卡点之类的东东呢?
kuing 3# 2013-1-30 15:15
已知非等腰$\Delta ABC$中有内点$P$满足$\angle PBA=\angle PCA$求证:$P$的轨迹是双曲线 Gauss门徒 发表于 2013-1-30 01:31 应该说是双曲线的一部分吧。 不知下面这样证行不行?感觉不太严格,至少缺一些特殊情况的说明,因为用到了斜率,有时会不存在,还有引理中包含了退化情形,但这里都暂时忽略掉。 引理:与两定点连线斜率之积为正常数的点在一条双曲线(或退化的)上。(证明略) 总以水平方向为 $x$ 轴,不妨设 $C>B$,图形一开始时 $BC$ 水平放置,$B$ 在左,$C$ 在右且 $A$ 在上方。 设 $\angle PBA=\angle PCA=\alpha$,则 $PB$ 与 $PC$ 的斜率分别为 \begin{align*} k_{PB}&=\tan (B-\alpha ), \\ k_{PC}&=\tan (\alpha -C), \end{align*} 现将图形逆时针旋转 $\theta$ 角,旋转后 $PB$ 与 $PC$ 的斜率分别变为 \begin{align*} k_{PB}'&=\tan (B-\alpha +\theta ), \\ k_{PC}'&=\tan (\alpha -C+\theta ), \end{align*} 令 \[(B-\alpha +\theta )+(\alpha -C+\theta )=\frac\pi2 \iff \theta =\frac{\pi +2C-2B}4,\] 此时就有 \[k_{PB}'\cdot k_{PC}'=1,\] 可见此时 $P$ 的在一条双曲线上,而旋转不改变曲线形状,所以未旋转前的 $P$ 也在一条双曲线上。 PS1、主题分类可以选[几何] PS2、
kuing 4# 2013-1-30 15:49
差点以为那是什么布勃卡点之类的东东呢? yes94 发表于 2013-1-30 14:18 你是说 $\angle PAB=\angle PBC=\angle PCA$ 那个 Brocard 点?
yes94 5# 2013-1-30 15:50
4# kuing 对,
thread-1102-1-4.html: [不等式] 数列类型的Inequality
yayaweha 1# 2013-1-30 20:13
请教,如图最后那个不等式不知道如何分析如何下手
kuing 2# 2013-1-30 20:18
楼主对数列不等式似乎情有独钟哟…… 这是我相对弱的东东…… 对了,bn 通项如何
yayaweha 3# 2013-1-30 20:19
bn我求不出来
yayaweha 4# 2013-1-30 20:21
我高三,现在放寒假,最近在狂做题,这最后一题基本上都是这种类型,所以问的多
kuing 5# 2013-1-30 20:51
噢,原来 bn 那个递推式也就是那个FAQ递推,昨天在这里才用过,通项的确是求不出来的。
yes94 6# 2013-1-30 21:12
噢,原来 bn 那个递推式也就是那个FAQ递推,昨天在这里才用过,通项的确是求不出来的。 kuing 发表于 2013-1-30 20:51 那{$a_n$}通项呢? 不要又叫我百度哈? 问问题的老规矩……
yes94 7# 2013-1-30 21:14
6# yes94 对象回复错了,
yayaweha 8# 2013-1-30 21:15
6# yes94 且看
yes94 9# 2013-1-30 21:25
8# yayaweha 这就对了噻,等会儿版主就可能给你答案了
yes94 10# 2013-1-30 21:33
能证明$\dfrac12(2^n-\dfrac1{2^n})<lnb_{n+1}<2^n-\dfrac1{2^n}$就好了,
yayaweha 11# 2013-1-30 21:35
10# yes94 bn的范围确定是个关键
kuing 12# 2013-1-30 21:42
11# yayaweha 昨天我就在做 bn 的估计,也没得到什么好的结果
yayaweha 13# 2013-1-30 22:31
11# yayaweha 昨天我就在做 bn 的估计,也没得到什么好的结果 kuing 发表于 2013-1-30 21:42 我问的这题知道b2=e,我觉得这是个突破口
realnumber 14# 2013-1-31 09:37
$b_{n+1}=b_n(b_n+1)>{(b_n)}^2$-----(1) 那么$\ln(b_2)=1,\ln(b_3)>2,\ln(b_4)>2^2,....,\ln(b_n)>2^{n-2}$,又$\ln(b_1)>0$ 所以$\ln(b_1)+\ln(b_2)+...+\ln(b_n)>1+2+2^2+...+2^{n-2}=2^{n-1}-1>\frac{3}{2}(a_n-1)$. 又$b_{n+1}=b_n(b_n+1)<e{(b_n)}^2$------(2) 那么那么$\ln(b_2)=1,\ln(b_3)<2+1=2^2-1,\ln(b_4)<2^2+2+1=2^3-1,....,\ln(b_n)<2^{n-2}+2^{n-3}+...+1=2^{n-1}-1$,又$\ln(b_1)<1$. 所以所以$\ln(b_1)+\ln(b_2)+...+\ln(b_n)<1+1+(2^2-1)+...+(2^{n-1}-1)=2^n-n<3a_n-1$. --ps,(1)(2)其实放缩得很宽松,特别开始几项严重影响后面. 比如(2)可以修改为$b_{n+1}=b_n(b_n+1)<e^{0.5}{(b_n)}^2$------(2),诸如此类,计算会开始复杂.
kuing 15# 2013-1-31 11:53
14# realnumber 嗯,很松,所以这种不是我想要的估计
yes94 16# 2013-2-1 15:08
能证明$\dfrac12(2^n-\dfrac1{2^n})<lnb_{n+1}<2^n-\dfrac1{2^n}$就好了! yes94 发表于 2013-1-30 21:33 实际只需证明:$2^{n-1}<lnb_{n+1}<2^n-1$即可, 那么此时必定$\dfrac12(2^n-\dfrac1{2^n})<lnb_{n+1}<2^n-\dfrac1{2^n}$,于是由累加法立得。 所以此不等式较弱。
thread-1103-1-4.html: [不等式] 前晚天书在粉丝群里提及的一对几何不等式
kuing 1# 2013-1-31 00:54
天书(1846******) 0:14:54 sinA+sinB+sinC>=sin2A+sin2B+sin2C遇到个类似的:csc2A+csc2B+csc2C>=cscA+cscB+cscC 天书(1846******) 0:16:14 这俩不等式好养眼的说... 记录一下。 第一个我在《数学空间》2011第2期P14例2.2.10提过,等价于熟知的$\sin(A/2)\sin(B/2)\sin(C/2)\leqslant1/8$。 但是没发现到还有第二个的类似,放一起的确养眼了,不过显然第二个需要在锐角三角形的条件。 作角代换 $A\to(\pi-A)/2$,等价于任意三角形下的 \[\csc A+\csc B+\csc C\geqslant\sec\frac A2+\sec\frac B2+\sec\frac C2,\] 用内切圆代换很容易证明上式,就是不知道有没有不代数化的证法,像第一个那样,转化为熟知的东东? 时间关系先闪,明天再玩玩。
thread-1104-1-4.html: [不等式] 一系列不等式问题的抽象问题
ccnu_chb_ycb 1# 2013-1-31 18:29
最近看到很多从Nissbit不等式引致出来的一些不等式问题,便提出了一个一般问题,解决了一点点之后没有了思路,请指教 _____kuing edit in $\LaTeX$_____ 已知 $a,b,c,d,e,f,x,y,z\in\mbb R^+$,求 $f(x,y,z)=\dfrac x{ay+bz}+\dfrac y{cz+dx}+\dfrac z{ex+fy}$ 的最小值(如有可能,也希望解决最大值问题)。 现在通过代换 $X=\dfrac{bdx}c$, $Y=ay$, $Z=bz$ 已经解决了 $ace=bdf$ 时的最小值问题,其他的没有什么思绪,望有兴趣的参加讨论或者指点思路,感激不尽。
kuing 2# 2013-1-31 18:44
请主题分类一下……
yes94 3# 2013-1-31 19:10
?? 一般可以分母代换吧? 只是要看系数$a,b,c$的取值,有时候是求最大值,而不是求最小值,所以不能随便乱提问的。
ccnu_chb_ycb 4# 2013-1-31 19:29
2# kuing 恩恩  ,下不为例哟,一定注意了
ccnu_chb_ycb 5# 2013-1-31 19:31
3# yes94 主要是最近遇到了好几个这样的不等式问题,我就觉得这样的不等式是不是把一般问题解决掉,在最大值与最小值指甲确实有考虑到底那个可求(所以加了括号的后注),但是至于这个一般问题有没有结果我就真不知道了,只是这个问题感觉有些意思
yes94 6# 2013-1-31 19:31
2# kuing 恩恩  ,下不为例哟,一定注意了 ccnu_chb_ycb 发表于 2013-1-31 19:29 这个你找天书就行了,他会告诉你一般方法的
ccnu_chb_ycb 7# 2013-1-31 19:56
恩恩  我还是觉得这个问题没有什么问题,有一定的价值。我再想想,还是感谢哦
thread-1105-1-1.html: 一些自定义表情(全部来自网上&Q群收集)随时更新
kuing 1# 2013-1-31 20:42
默认修改
kuing 2# 2013-1-31 21:03
一些其他
kuing 3# 2013-1-31 21:16
大家也可以传一些
q85669551 4# 2013-1-31 21:23
mark  有空了下 。。
kuing 5# 2013-2-1 17:53
乜都有
kuing 6# 2013-2-15 13:51
继续     有没有人知道这系列表情叫什么名字?
╰☆ヾo.海x 7# 2013-2-22 23:48
顶!!!哈哈哈 看的我笑死了 哈哈
kuing 8# 2013-2-23 00:41
7# ╰☆ヾo.海x 嘿嘿 :D 又更新了几个
戊概念·五 9# 2013-2-24 19:12
凑个热闹,来个大尺码的:
kuing 10# 2013-2-24 19:19
9# 戊概念·五 那我也来两个
戊概念·五 11# 2013-2-24 19:23
10# kuing 浓眉大眼的美男子啊~
kuing 12# 2013-2-28 14:03
更新了一下6#,顺便加了一个问题。
戊概念·五 13# 2013-2-28 19:06
12# kuing 叫“红脸蛋儿”怎么样呢=。=
kuing 14# 2013-4-2 00:36

kuing 15# 2013-4-16 23:06
认不认得这个表情?:
isea 16# 2013-4-16 23:49
表情与opera无缘分
李斌斌755 17# 2013-4-30 18:48
有郁闷的表情吗?
kuing 18# 2013-4-30 20:40
17# 李斌斌755 在默认里挑一个吧……
李斌斌755 19# 2013-4-30 20:58
18# kuing 里面没有啊。
kuing 20# 2013-5-3 20:17
19# 李斌斌755 找个相近的呗, 什么的……
thread-1105-2-1.html:
kuing 21# 2013-5-3 20:17
更新了一下这个
李斌斌755 22# 2013-5-14 13:10
21# kuing 14号发表,显示3号
kuing 23# 2013-5-14 13:13
22# 李斌斌755 看1楼下面灰色字
李斌斌755 24# 2013-5-14 13:21
干脆设置顶。
kuing 25# 2013-5-16 18:30
这个 的效果比1#的那个好一些
kuing 26# 2013-5-20 22:21
thread-1106-1-4.html: [几何] (转)正方体空间轨迹问题
realnumber 1# 2013-1-31 21:39
本帖最后由 realnumber 于 2013-1-31 21:59 编辑 转自解题群
kuing 2# 2013-1-31 21:41
看上去好麻烦的题…… PS、为什么“体”字单独跑开了,排版真怪…… PS2、题目还是以几何为主,分类可以选[几何]
realnumber 3# 2013-1-31 21:59
本帖最后由 realnumber 于 2013-2-1 13:39 编辑 对着魔方看了下,觉得解的个数为0,2,无数个.-----看来想错了.
yes94 4# 2013-1-31 23:06
http://bbs.pep.com.cn/forum.php? ... &extra=page%3D1
kuing 5# 2013-1-31 23:12
4# yes94 眼都花……
yes94 6# 2013-2-1 15:03
鱼子都说看得要命了
thread-1107-1-4.html: [几何] 转一个解几定点问题,看还有什么好方法,谢谢了!
hongxian 1# 2013-2-1 14:50
已知点$A(-1,0)$,$B(1,-1)$和抛物线$C$:$y^2=4x$,$O$为坐标原点,过点$A$的动直线交抛物线$C$于$M$、$P$,直线$MB$交抛物线$C$于另一点$Q$, 证明:直线$PQ$恒过一个定点。 来自http://bbs.pep.com.cn/forum.php?mod=viewthread&tid=2659365
yes94 2# 2013-2-1 14:58
1# hongxian 图画的真好! 先特殊位置法,可以猜得定点的横坐标
kuing 3# 2013-2-1 15:46
还是极点极线背景……
第一章 4# 2013-2-1 16:21
现在做解析几何的题目,常常会想着如何设出点的坐标,利用向量共线或者垂直跳过“两点式”甚至是“韦达定理”。 有时候确实能减少计算。不过这个题,就算用向量,计算量也不小。
yes94 5# 2013-2-1 16:24
一般结果见 1998MO
kuing 6# 2013-2-1 16:25
抛物线可一般化为圆锥曲线,定点 B 只要在定点 A 对应的极线上即可
thread-1108-1-4.html: 除了列举法之外,能否把证明过程严密的写出来
转化与化归 1# 2013-2-1 17:57
除了列举法之外,能否把证明过程严密的写出来 ______kuing edit in $\LaTeX$______ 14. 将整数 $1,2,3,\ldots,25$ 填入如图所示的 5 行 5 列的表格中,使每一行的数字从左到右都成递增数列,则第三列各数之和的最小值为_____,最大值为_____。 \begin{array}{|c|c|c|c|c|} \hline\\  & & & & \\ \hline\\  & & & & \\ \hline\\  & & & & \\ \hline\\  & & & & \\ \hline\\  & & & & \\ \hline \end{array}
realnumber 2# 2013-2-1 18:43
本帖最后由 realnumber 于 2013-2-1 19:28 编辑 第3列所有数的和为$x$,第4列所有数的和为$x+s+5$,第5列所有数的和为$x+s+t+10$,其中$s,t$为非负整数. 那么后三列和$3x+2s+t+15\le 11+12+13+...+25$(不会超过最大的15个数的和),可得$x\le85$,而第3列依次为$11,14,17,20,23$正好取到,说明就是最大值$85$. 第3列所有数的和为$x$,第2列所有数的和为$x-s-5$,第1列所有数的和为$x-s-t-10$,其中$s,t$为非负整数. 那么前三列和$3x-2s-t-15\ge 1+2+3+...+15$(不会小于最小的..),可得$x\ge45$,而第3列依次为$3,6,9,12,15$正好取到,说明就是最小值$45$.
kuing 3# 2013-2-1 19:30
2# realnumber 顺便试试用代码画表,举最小最大值的例 \begin{array}{|c|c|c|c|c|} \hline\\ 1 & 2 & 3 & 16 & 17 \\ \hline\\ 4 & 5 & 6 & 18 & 19 \\ \hline\\ 7 & 8 & 6 & 20 & 21 \\ \hline\\ 10 & 11 & 12 & 22 & 23 \\ \hline\\ 13 & 14 & 15 & 24 & 25 \\ \hline \end{array} \begin{array}{|c|c|c|c|c|} \hline\\ 1 & 2 & 11 & 12 & 13 \\ \hline\\ 3 & 4 & 14 & 15 & 16 \\ \hline\\ 5 & 6 & 17 & 18 & 19 \\ \hline\\ 7 & 8 & 20 & 21 & 22 \\ \hline\\ 9 & 10 & 23 & 24 & 25 \\ \hline \end{array} 代码自己右键查看
kuing 4# 2013-2-1 19:33
3# kuing 奇怪,行距不定,看来 mathjax 对表格的处理还有待优化
转化与化归 5# 2013-2-1 19:45
2# realnumber 这个写法很好,实质就是增量代换。
转化与化归 6# 2013-2-1 19:47
3# kuing 第二个表如果把第5行和第1行对调,第4行和第2行对调。。。。。。更符合思考的顺序啊
thread-1109-1-4.html: [函数] 有没有解法3?
转化与化归 1# 2013-2-2 11:06
14. 已知 $f(x)=2mx+m^2+2$, $m\ne0$, $m\in\mbb R$, $x\in\mbb R$。若 $\abs{x_1}+\abs{x_2}=1$,则 $\dfrac{f(x_1)}{f(x_2)}$ 的取值范围是_________ 答案:$\left[ 1-\dfrac{\sqrt2}2,2+\sqrt2 \right]$ 有没有解法3?
kuing 2# 2013-2-2 11:33
解法2已经很好了 我前两天见过这道题,不过没什么兴趣就没往下想了……看了解法2才觉得其实我判断错了……
yes94 3# 2013-2-2 22:20
人教不知在哪里有这个类似的帖子
thread-111-1-1.html: 一行输出$\sin3k^\circ$值
kuing 1# 2011-10-18 16:27
TraditionalForm[Simplify[FunctionExpand[Table[Sin[k*Pi/60], {k, 0, 30}]]]] 输出的是 $\sin0^\circ, \sin3^\circ, \sin6^\circ, \sin9^\circ, \ldots , \sin90^\circ$ 的值。 将 Sin 换成 Cos 也就输出余弦的,而若换成 Tan,他会化成 sin/cos 来算,相当于将两个表各项值对应地比一下而已,并不会化简。
thread-1110-1-4.html: [几何] 来自粉丝群的平几,等腰三角形等线段
kuing 1# 2013-2-2 15:50
证: 在 $AF$ 延长线上取点 $G'$ 使 $AF=FG'$,连结 $G'B$, $G'C$, $G'D$, $G'E$, 由 $DE$, $AG'$ 互相平分知 $ADG'E$ 为平行四边形, 从而有 $BD=AE=DG'$, $CE=AD=EG'$ 且 $\angle BDG'=\angle BAC=\angle CEG'$, 所以 $\triangle BDG'$ 与 $\triangle CEG'$ 为相似的等腰三角形,于是有 \[\angle BG'D+\angle DG'E+\angle EG'C=\angle BG'D+\angle G'DB+\angle DBG'=180^\circ,\] 可见 $B$, $G'$, $C$ 三点共线,所以 $G'$ 与 $G$ 重合,即原命题得证。
kuing 2# 2013-2-2 17:05
其实我一开始想用梅氏定理,没成功……但总感觉可行……
isea 3# 2013-2-2 18:42
此题有个变式,是其证明中间过程。 http://bbs.pep.com.cn/forum.php?mod=viewthread&tid=668180 3楼,其实没什么看头,下面就是要说的 其次,用同一肯定搞定的。 另外,直接过$D$作$DH$平行于$AE$交$BC$于$H$,则$DH=DB=AE$,连$HA$则过$DE$中点。 从现在的眼光看,此题用向量是最简单的,丢楼下,有代码嘛……
isea 4# 2013-2-2 18:56
本帖最后由 isea 于 2013-2-2 19:08 编辑 $D,F,E$三点共线 ;$B,G,C$三点共线。 \begin{align*} 2\vv {AF}&=\vv {AD}+\vv {AE}\\ \\ 2\dfrac{AF}{AG}\vv {AG}&=\dfrac{AD}{AB}\vv {AB}+\dfrac{AE}{AC}\vv {AC}\\ \\ \dfrac{2AF}{AG}&=\dfrac{AD}{AB}+\dfrac{AE}{AC},AB=AC,AE=BD\\ \\ \dfrac{2AF}{AG}&=\dfrac{AD+AE}{AB}=\dfrac{AB}{AB}=1\\ \\ …… \end{align*}
kuing 5# 2013-2-2 19:02
今天论坛比较不稳定,建议在本地任意文本编辑器上打好再复制过来回贴,以免回贴时出错致白打……
isea 6# 2013-2-2 19:06
慢得人都快疯了,还以为偶的网出问题了 闪人了
kuing 7# 2013-2-2 19:17
6# isea I'm sorry... 整个5d6d都这样,时慢时快
kuing 8# 2013-2-2 22:57
向量用得 nice! 3#那链接又是一个大贴,isea,我建议你将你的大贴整理成文章,可以发表
第一章 9# 2013-2-2 23:49
证明△AEF与△GDF全等即可
kuing 10# 2013-2-2 23:50
9# 第一章 怎么证
第一章 11# 2013-2-2 23:51
9# 第一章
kuing 12# 2013-2-3 00:00
11# 第一章 oh,好像看懂了……也不错
yes94 13# 2013-2-3 00:10
其实我一开始想用梅氏定理,没成功……但总感觉可行…… kuing 发表于 2013-2-2 17:05 我和你的感觉一样,搞了很多个梅氏定理的等式,结果终于筛选出来了两个有用的!
kuing 14# 2013-2-3 00:13
13# yes94 那就是证出来了?写来瞧瞧
yes94 15# 2013-2-3 00:28
14# kuing 连结$DG$,并延长$ED$、$CB$交于点$H$,则由梅氏定理知, $\dfrac{DA}{AB}\cdot\dfrac{BG}{GH}\cdot\dfrac{HF}{FD}=1$, $\dfrac{EA}{AC}\cdot\dfrac{CG}{GH}\cdot\dfrac{HF}{FE}=1$, 因为$AB=AC$,$FD=FE$,故$DA\cdot BG=EA\cdot CG$,即$\dfrac{DA}{EA}=\dfrac{CG}{BG}$ 由于$EA=BD$,故$\dfrac{DA}{BD}=\dfrac{CG}{BG}$, 于是$DG//AC$, 以下就是显然的了。 不知道代码显示对不对,
kuing 16# 2013-2-3 00:40
15# yes94 原来如此,应该没问题,不错。 不过没我想象中简洁,我以为可以列几个那样的等式直接解出相等来……
yes94 17# 2013-2-3 00:44
16# kuing 我的目标和你一样,也是列出一大堆梅氏定理等式,然后选出有用的,解出线段相等(或者2倍关系) 却意外得到平行!与初衷背道而驰,
第一章 18# 2013-2-3 06:38
昨晚太困了,急着要睡觉。早上起来证了一下,可以只作两条辅助线。 PS,以前学会了代码的,不过一年没用了,忘记得差不多了。
kuing 19# 2013-2-3 08:06
18# 第一章 其实昨晚我就是这样看的。
yes94 20# 2013-2-4 13:53
本帖最后由 yes94 于 2013-2-4 14:10 编辑 今天回过来发现,这道题还真的很简单呢! 因为梅氏定理的一种证法是平行线方法,另一种是用面积法,而平行线方法第一章已经完成,所以试一试面积法是否可做,结果一试,完全一帆风顺的成功了! 设$\angle BAG=\alpha$,$\angle GAC=\beta$, 因为$F$为$DE$的中点,故$S_{\triangle ADF}=S_{\triangle AEF}$,于是$AD\sin\alpha=AE\sin\beta$,    即$\dfrac{\sin\alpha}{\sin\beta}=\dfrac{AE}{AD}=\dfrac{BD}{AD}$ (因为$AE=BD$)。 而$\dfrac{BG}{GC}=\dfrac{S_{\triangle ABG}}{S_{\triangle ACG}}=\dfrac{AB\sin\alpha}{AC\sin\beta}=\dfrac{\sin\alpha}{\sin\beta}=\dfrac{BD}{AD}$(因为$AB=AC$)。。 故$DG\sslash AC$,以下略。 终于编辑好了,那多了个}都没看出来,,谢谢
thread-1110-2-4.html:
kuing 21# 2013-2-4 14:03
20# yes94 后面多了个 } ,不过从代码已经看懂了。 PS、平行符号我在本论坛上专门定义了个命令 \sslash,效果 $AB\sslash CD$,看上去应该比 $AB//CD$ 好些。当然,如果觉得麻烦,直接打两斜杠 // ,不强制(从来不)。
thread-1111-1-4.html: [组合] 来自人教群的数三角形[未解决]
kuing 1# 2013-2-2 18:04
爱好者 bb226562  16:39:00 一个圆周上给定7个点,两两用线段相连,所得图形中三角形的个数至少多少个? 7个点那么多,我…… 话说,5个点以下的话个数确定(如果没数错的话5个点时35个三角形),因为至少六边形才能让对角线共点,所以6个点的时候有两种情况,且个数相差1,但我已经数不来了……还7个……我
kuing 2# 2013-2-2 18:20
突然发现这个贴的ID是1111…… 也就是本论坛第1111个贴了
kuing 3# 2013-2-2 18:37
7边形,对角线共点的话也只能是3条共1点,因为想要4条共1点至少要八边形了。 那么7边形的对角线最多能有多少个这样的共点?(猜测3)
realnumber 4# 2013-2-2 19:24
3# kuing 7个吓人啊,要是必须是7个点的其中三点为顶点.那还容易.
kuing 5# 2013-2-2 22:01
7边形,对角线共点的话也只能是3条共1点,因为想要4条共1点至少要八边形了。 那么7边形的对角线最多能有多少个这样的共点?(猜测3) kuing 发表于 2013-2-2 18:37 的确是3,不过证起来……表达真是好麻烦。 \(\newcommand\xxx{{\Large *}~\text{点}}\) 为方便讲述,当多边形三条对角线共点于多边形内,我们称所共的那个点叫 $\xxx$。 要证明的是:圆内接 7 边形的所有对角线最多构成 3 个 $\xxx$。 先构造 3 个 $\xxx$的情形。 然后证明不可能有 4 个 $\xxx$。假设有 4 个 $\xxx$,由于每个 $\xxx$需要 6 个顶点来构成,而现在只有 7 个顶点,所以必有 3 个顶点同时应用于那 4 个 $\xxx$的构成,记这 3 个顶点为 $A$, $B$, $C$,也就是说那 4 个 $\xxx$同在 $\triangle ABC$ 的边上。 我们用如下示意图来表示其中一种情况 当我们将那些短线延长,不难看出,它们会与圆相交于至少 6 个点(异于 $A$, $B$, $C$),所以不可能是由 7 边形构成。 其他情况类似均可以地得出不可能(很难表达,自己试着想想吧),于是结论成立。
kuing 6# 2013-2-2 22:13
每减少一个 $\xxx$,三角形个数将增加 1(应该没问题吧?),于是由楼上可知圆内接 7 边形及其所有对角线构成的三角形个数有四种可能,且是四个连续的数字,最少的情况就是楼上第一个图,至于是多少,我还是没数出来……
yes94 7# 2013-2-2 22:22
最怕组合计数了!
thread-1112-1-4.html: [数论] 3个不同的质因数
realnumber 1# 2013-2-2 21:07
fans  of 西神(14###048) 2013-01-28 21:57:11 证明:当正整数$n$充分大时.数$n,n+1,n+2,...,n+9$中至少有一个数,它的不同质因数个数不小于3 . 改成这样:当正整数n充分大时.数$n,n+1,n+2,...,n+9$中至少有一个数,它的不同质因数个数不小于4.(甚至5)可不可以呢?举反例或证明. 就假定$n>100$.
kuing 2# 2013-2-2 22:06
咦?这个你不是发过一次的么
yes94 3# 2013-2-2 22:17
今天还没发过帖子,拿分走人
kuing 4# 2013-2-2 22:19
3# yes94 这里的分数是完全没用的
yes94 5# 2013-2-2 22:21
4# kuing 回帖没有分啊?
kuing 6# 2013-2-2 22:26
5# yes94 有,但分多最多就多些太阳,级别名字变了,但是权限一样。
yes94 7# 2013-2-2 22:27
6# kuing 我又不要权限,
realnumber 8# 2013-2-2 22:28
5# yes94 我把那个帖子取出来了,原来那个打算专门学习取模,---有点脑子烧坏,
thread-1113-1-4.html: [函数] 只需要解答第三问(转自解题群)的(1)
realnumber 1# 2013-2-3 13:06
奇函数可以证明. ————kuing edit in $\LaTeX$———— 已知定义在 $\mbb R$ 上的函数 $f(x)$ 满足: 1、对任意的实数 $x$, $y$,有 $f(x+y+1)=f(x-y+1)-f(x)f(y)$ 2、$f(1)=2$ 3、$f(x)$ 在 $[0,1]$ 上为增函数 (I)求 $f(0)$ 及 $f(-1)$ (II)判断函数 $f(x)$ 的奇偶性,并证明 (III)(说明:请在(i)(ii)问中选择一问解答即可。若选择(i)并解答正确,满分6分,若选择(ii)并解答正确,满分4分) (i)设 $a$, $b$, $c$ 为周长不超过 $2$ 的三角形三边的长,求证:$f(a)$, $f(b)$, $f(c)$ 也是某个三角形三边的长 (ii)解不等式 $f(x)>1$
yes94 2# 2013-2-3 13:37
1# realnumber 既然只需解答第3问,那么把第一二问的简要答案写出来噻,或者他得到了哪些主要结果, 这样可以减轻解答者的负担,使他可以集中精力……
realnumber 3# 2013-2-3 13:45
2# yes94 那个人什么也没写啊,奇函数是容易的,[0,1]上增函数就直接用,第三问和这些关系也不大. 写些第三问相关的(其实这些容易得到,但愿别干扰你的思路),假设$a\ge b\ge c$,因为$a+b+c\le2$,所以$1> a\ge b\ge c\ge0$, 而由单调性,$f(a)\ge f(b)\ge f(c) \ge 0$,只需要证明$f(a)<f(b)+f(c)$,...
kuing 4# 2013-2-3 16:53
那个人什么也没写啊,奇函数是容易的,[0,1]上增函数就直接用,第三问和这些关系也不大. 写些第三问相关的(其实这些容易得到,但愿别干扰你的思路),假设$a\ge b\ge c$,因为$a+b+c\le2$,所以$1> a\ge b\ge c\ge0$, 而由单调性,$f(a)\ge f(b)\ge f(c) \ge 0$,只需要证明$f(a)<f(b)+f(c)$,... realnumber 发表于 2013-2-3 13:45 证更强式 $f(a)+f(b+c-a)\leqslant f(b)+f(c)$ 就行了,这个比较容易,注意到由条件可得 $f(x)+f(y)=f(1-(x-y)/2)f((x+y)/2)$。
realnumber 5# 2013-2-3 20:57
4# kuing 忧闷了,怎么看出来的呢?我还以为参考函数$y=2\sin{0.5\pi x}$,
kuing 6# 2013-2-3 21:15
易证 $\abs{f(x)}\leqslant2$,故当 $x$, $y\in[0,1]$ 时,有 $f(x)+f(y)=f(1-(x-y)/2)f((x+y)/2)\leqslant2f((x+y)/2)$,所以 $f(x)$ 在 $[0,1]$ 上是上凸函数……
yes94 7# 2013-2-3 21:19
基础性成果,都没人写,例如$f(0)=$?$ f(-1)$=?
kuing 8# 2013-2-3 21:22
7# yes94 1#不是已经补充说了已证奇函数么
yes94 9# 2013-2-3 21:31
8# kuing 浏览论坛,不一定时时要带着草稿纸啊?
realnumber 10# 2013-2-3 21:31
6# kuing 就需要上凸函数,得到这个不等式,却没看出来.
realnumber 11# 2013-2-4 17:52
本帖最后由 realnumber 于 2013-2-4 19:19 编辑 1楼发题苏老师其它小题的解答 见楼下
suwenyu333x 12# 2013-2-4 19:05

realnumber 13# 2013-2-4 19:18
12# suwenyu333x 欢迎苏老师,~~~
yes94 14# 2013-2-4 22:50
12# suwenyu333x 欢迎苏老师,~~~ realnumber 发表于 2013-2-4 19:18 欢迎苏老师!
kuing 15# 2013-2-6 14:22
莫非是出题的来了? PS、为了方便存档,编辑了一下1#,打成 latex
thread-1114-1-4.html: [函数] 求抽象函数解析式 (陈题)
isea 1# 2013-2-3 16:14
本帖最后由 isea 于 2013-2-3 16:17 编辑 题:已知定义域为$\mathbf{R}$的函数$f(x)$满足$f(f(x)-x^2+x)=f(x)-x^2+x$. 设有且仅有一个实数$x_0$ ,使得$f(x_0)=x_0$. 求函数$f(x)$的解析表达式. 被自己弄迷糊了……指点指点,谢,先
realnumber 2# 2013-2-3 16:27
好象是某年的重庆卷,这个会做 设$f(x)-x^2+x=t$,那么条件就是$f(t)=t$,若t有2个或以上不同值,那么与题目中接下去的一句话矛盾,可见$t$是某一唯一确定的常数, 也就有$f(x)=x^2-x+t$,其中t为某常数,又与题意,它与$y=x$相切,那么$t=1$, 所以$f(x)=x^2-x+1$
isea 3# 2013-2-3 16:48
现在就是在“$t$是某一唯一确定的常数”把自己给整得不明白了 不知有没“科谱”的? 感谢楼上realnumber,等有集中时间,特意来理解理解!
isea 4# 2013-2-3 20:19
那,也就是说:“设有且仅有一个实数$x_0$ ,使得$f(x_0)=x_0$.”这句话是指 $f(x)$的图象与$y=x$有且有一个交点? 按2楼结果,只能是$f(1)=1$,不能有$f(2)=2$这样的结果存在? 偶迷糊的是 “一个实数$x_0$”,是一个确定的常数,若$x_0=1$了,为什么就不能有$f(2)=2$这样的结果呢?
isea 5# 2013-2-3 20:38
按提示找到源了,2006年重庆卷理科第21题 标答案丢上来 哎,不管了就当$x_0$是方程$f(x)=x$的惟一解来理解算了,越想想头越疼疼
yes94 6# 2013-2-3 21:21
这个21题,对考生不公平
realnumber 7# 2013-2-3 21:34
4# isea 这个是已经条件,既然$x_0=1$,就取1一个.
thread-1115-1-1.html: [几何] 来自人教群的昨晚没时间写的解几共圆
kuing 1# 2013-2-4 12:52
几何意义不知道,只知道用二次曲线系会比较简单,或者说是一个背景结论。 设两直线 $y=k_1x+b_1$, $y=k_2x+b_2$ 与二次曲线 $Ax^2+By^2=1$($A\ne B$ 且 $A$, $B$ 至少一个为正)相交于四点,则过这四点的二次曲线系为 \[\lambda(k_1x-y+b_1)(k_2x-y+b_2)+\mu(Ax^2+By^2-1)=0,\] 其中 $\lambda$, $\mu$ 不同为 $0$。因此,该四点共圆当且仅当存在 $\lambda$, $\mu$ 使得上述方程为圆,将方程展开为 \[(\lambda k_1k_2+\mu A)x^2+(\lambda+\mu B)y^2-\lambda(k_1+k_2)xy+\cdots=0,\] 此方程为圆当且仅当 \[ \left\{\begin{aligned} &\lambda k_1k_2+\mu A=\lambda+\mu B\ne0,\\ &\lambda(k_1+k_2)=0, \end{aligned}\right. \] 若 $\lambda=0$,则 $\mu\ne0$,再由 $A\ne B$ 知上述方程组不可能成立,所以 $\lambda\ne0$,因此必须有 \[k_1+k_2=0,\] 反之,当 $k_1+k_2=0$ 时,方程组必有解。 所以,该四点共圆当且仅当两直线斜率互反。 回到原题,可知定点就是过 $A$ 且斜率为 $-1/2$ 的直线与椭圆的另一个交点,即 $(0,1)$。
yes94 2# 2013-2-4 13:18
1# kuing 设$B(0,1)$,则$k_{AB}=-\dfrac12$,而$k_{PQ}=\dfrac12$,于是$k_{AB}+k_{PQ}=0$。 我还往$PQ$中点$M$在定直线($OM$):$y=-\dfrac12x$上想呢!此时直线$OM$和直线$AB$平行,这有奥妙吗($OM//AB$)? 只是方程成为圆,是不是还需要其它条件……
kuing 3# 2013-2-4 13:24
2# yes94 那二次曲线至少过那四点,不会是虚圆也不会是一个点,所以只要系数满足那个方程组就行了。
yes94 4# 2013-2-4 13:40
3# kuing 明白了, 那担心是多余的,
yuzi 5# 2013-2-4 13:42
本帖最后由 yuzi 于 2013-2-4 13:45 编辑 嗯,曲线系确实简单,比标答和我的解法简单多了
yuzi 6# 2013-2-4 13:50
怎么想到曲线系了。。。我想不到
kuing 7# 2013-2-4 13:57
6# yuzi 印象中如果没记错的话是几年前见 ab1962 用过一次这种方法……不过题目应该不一样,好像是反过来的
yes94 8# 2013-2-4 14:14
嗯,曲线系确实简单,比标答和我的解法简单多了 yuzi 发表于 2013-2-4 13:42 鱼子的什么做法啊?标答呢?欣赏一下? 另外,那个$OM\sslash AB$有没有用?是碰巧?
kuing 9# 2013-2-4 14:34
8# yes94 是碰巧,换别的椭圆就没这关系了
yes94 10# 2013-2-4 15:02
关于椭圆上四点共圆的充要条件的一个资料,他用的是相交弦定理(直线的参数方程) 可以推广到圆锥曲线。 http://wenku.baidu.com/view/57b3 ... ight=10&count=5
yes94 11# 2013-2-4 15:14
本帖最后由 yes94 于 2013-2-5 15:53 编辑 9# kuing 哦,
hongxian 12# 2013-2-5 06:46
1# kuing 那是不是得事先知道定点一定在椭圆上呢?
kuing 13# 2013-2-5 07:47
12# hongxian 应该说是在知道那个共圆的充要条件下能够判断出在椭圆上。
yes94 14# 2013-2-5 15:53
本帖最后由 yes94 于 2013-2-5 15:56 编辑 椭圆$C$:$\dfrac{x^2}{a^2}+\dfrac{y^2}{b^2}=1$,$A(a,0)$,$B(0,b)$,不经过原点的直线$l$和椭圆$C$交于$P、Q$两点,且线段$PQ$的中点为$M$,若$A、B、P、Q$四点共圆,则$OM\sslash AB$ 这个结论对不对?
kuing 15# 2013-2-5 16:14
椭圆$C$:$\dfrac{x^2}{a^2}+\dfrac{y^2}{b^2}=1$,$A(a,0)$,$B(0,b)$,不经过原点的直线$l$和椭圆$C$交于$P、Q$两点,且线段$PQ$的中点为$M$,若$A、B、P、Q$四点共圆,则$OM\sslash AB$ 这个结论对不对? yes94 发表于 2013-2-5 15:53 由1#结论知 $k_{PQ}+k_{AB}=0$,由共轭直径神马之类的东东知 $k_{OM}\cdot k_{PQ}=-b^2/a^2$,消去 $k_{PQ}$ 即得 \[k_{OM}\cdot k_{AB}=\frac{b^2}{a^2},\] 因此 $\abs{k_{AB}}=b/a \riff OM\sslash AB$。
yes94 16# 2013-2-5 16:48
15# kuing 谢谢!
thread-1115-1-4.html: [几何] 来自人教群的昨晚没时间写的解几共圆
kuing 1# 2013-2-4 12:52
几何意义不知道,只知道用二次曲线系会比较简单,或者说是一个背景结论。 设两直线 $y=k_1x+b_1$, $y=k_2x+b_2$ 与二次曲线 $Ax^2+By^2=1$($A\ne B$ 且 $A$, $B$ 至少一个为正)相交于四点,则过这四点的二次曲线系为 \[\lambda(k_1x-y+b_1)(k_2x-y+b_2)+\mu(Ax^2+By^2-1)=0,\] 其中 $\lambda$, $\mu$ 不同为 $0$。因此,该四点共圆当且仅当存在 $\lambda$, $\mu$ 使得上述方程为圆,将方程展开为 \[(\lambda k_1k_2+\mu A)x^2+(\lambda+\mu B)y^2-\lambda(k_1+k_2)xy+\cdots=0,\] 此方程为圆当且仅当 \[ \left\{\begin{aligned} &\lambda k_1k_2+\mu A=\lambda+\mu B\ne0,\\ &\lambda(k_1+k_2)=0, \end{aligned}\right. \] 若 $\lambda=0$,则 $\mu\ne0$,再由 $A\ne B$ 知上述方程组不可能成立,所以 $\lambda\ne0$,因此必须有 \[k_1+k_2=0,\] 反之,当 $k_1+k_2=0$ 时,方程组必有解。 所以,该四点共圆当且仅当两直线斜率互反。 回到原题,可知定点就是过 $A$ 且斜率为 $-1/2$ 的直线与椭圆的另一个交点,即 $(0,1)$。
yes94 2# 2013-2-4 13:18
1# kuing 设$B(0,1)$,则$k_{AB}=-\dfrac12$,而$k_{PQ}=\dfrac12$,于是$k_{AB}+k_{PQ}=0$。 我还往$PQ$中点$M$在定直线($OM$):$y=-\dfrac12x$上想呢!此时直线$OM$和直线$AB$平行,这有奥妙吗($OM//AB$)? 只是方程成为圆,是不是还需要其它条件……
kuing 3# 2013-2-4 13:24
2# yes94 那二次曲线至少过那四点,不会是虚圆也不会是一个点,所以只要系数满足那个方程组就行了。
yes94 4# 2013-2-4 13:40
3# kuing 明白了, 那担心是多余的,
yuzi 5# 2013-2-4 13:42
本帖最后由 yuzi 于 2013-2-4 13:45 编辑 嗯,曲线系确实简单,比标答和我的解法简单多了
yuzi 6# 2013-2-4 13:50
怎么想到曲线系了。。。我想不到
kuing 7# 2013-2-4 13:57
6# yuzi 印象中如果没记错的话是几年前见 ab1962 用过一次这种方法……不过题目应该不一样,好像是反过来的
yes94 8# 2013-2-4 14:14
嗯,曲线系确实简单,比标答和我的解法简单多了 yuzi 发表于 2013-2-4 13:42 鱼子的什么做法啊?标答呢?欣赏一下? 另外,那个$OM\sslash AB$有没有用?是碰巧?
kuing 9# 2013-2-4 14:34
8# yes94 是碰巧,换别的椭圆就没这关系了
yes94 10# 2013-2-4 15:02
关于椭圆上四点共圆的充要条件的一个资料,他用的是相交弦定理(直线的参数方程) 可以推广到圆锥曲线。 http://wenku.baidu.com/view/57b3 ... ight=10&count=5
yes94 11# 2013-2-4 15:14
本帖最后由 yes94 于 2013-2-5 15:53 编辑 9# kuing 哦,
hongxian 12# 2013-2-5 06:46
1# kuing 那是不是得事先知道定点一定在椭圆上呢?
kuing 13# 2013-2-5 07:47
12# hongxian 应该说是在知道那个共圆的充要条件下能够判断出在椭圆上。
yes94 14# 2013-2-5 15:53
本帖最后由 yes94 于 2013-2-5 15:56 编辑 椭圆$C$:$\dfrac{x^2}{a^2}+\dfrac{y^2}{b^2}=1$,$A(a,0)$,$B(0,b)$,不经过原点的直线$l$和椭圆$C$交于$P、Q$两点,且线段$PQ$的中点为$M$,若$A、B、P、Q$四点共圆,则$OM\sslash AB$ 这个结论对不对?
kuing 15# 2013-2-5 16:14
椭圆$C$:$\dfrac{x^2}{a^2}+\dfrac{y^2}{b^2}=1$,$A(a,0)$,$B(0,b)$,不经过原点的直线$l$和椭圆$C$交于$P、Q$两点,且线段$PQ$的中点为$M$,若$A、B、P、Q$四点共圆,则$OM\sslash AB$ 这个结论对不对? yes94 发表于 2013-2-5 15:53 由1#结论知 $k_{PQ}+k_{AB}=0$,由共轭直径神马之类的东东知 $k_{OM}\cdot k_{PQ}=-b^2/a^2$,消去 $k_{PQ}$ 即得 \[k_{OM}\cdot k_{AB}=\frac{b^2}{a^2},\] 因此 $\abs{k_{AB}}=b/a \riff OM\sslash AB$。
yes94 16# 2013-2-5 16:48
15# kuing 谢谢!
thread-1116-1-4.html: [组合] 排列组合
guanmo 1# 2013-2-5 09:10
要将这 6 个灯笼撤下来,每次撤其中一列最下面的一个,则不同的撤法种数为? 答案为 $C_6^2\cdot C_4^2\cdot C_2^2=90$ 怎么解释?
guanmo 2# 2013-2-5 09:13
加图
kuing 3# 2013-2-5 09:41
等价于将它们排成一列且次序不变
guanmo 4# 2013-2-5 09:45
对,是的。我也理解了。考虑每列的两个,分步即可。
kuing 5# 2013-2-6 14:14
为了方便存档,编辑了一下1#,打成 latex
thread-1117-1-4.html: 麻烦一下大家看看我刚制作的平行四边形及平行且等于符号效果如何
kuing 1# 2013-2-5 17:34
麻烦一下大家看看我刚制作的平行四边形及平行且等于符号效果如何 \(\newcommand\tmpa{\mathord{\scriptstyle\raise.3ex{/}\kern-.35em\raise-.5ex{{-}\!{-}}\kern-.7em\raise1.1ex{{-}\!{-}}\kern-.35em\raise.3ex{/}}} \newcommand\tmpb{\stackrel{\,/\!/}{\raise-.5ex{=\!\!\!\!=}}}\) 测试一下:在 $\tmpa ABCD$ 中,$AB\tmpb CD$。 字大点测试一下:在 $\tmpa ABCD$ 中,$AB\tmpb CD$。
isea 2# 2013-2-5 22:27
嗯,的确是平行且相等
kuing 3# 2013-2-5 22:53
经群里多人的截图,以及我自己的亲测,发现平行且等于的效果还凑合,但是平行四边形就会有点问题,容易因不同浏览器而有不同的粗糙程度,而且更重要的一点是,如果平行四边形处于下标状态时,它仍然会像原来一样大,这是不合适的。 所以决定暂时不搞平行四边形。 而将平行且等于的代码定为 \pqd ,比如 AB\pqd CD 显示 $AB\pqd CD$。
kuing 4# 2013-2-5 22:55
欢迎试用 \[AB\pqd CD=X_{AB\pqd CD}^{AB\pqd CD}\]
hongxian 5# 2013-2-5 23:06
3# kuing 汉语拼音,好记!
kuing 6# 2013-2-5 23:28
5# hongxian 嗯,反正这不是国际标准符号,可能是中国发明的,所以直接用拼音好了,让你们好记些。
yes94 7# 2013-2-5 23:34
平且等, $OM\pqd PQ$,   ${AB\pqd CD}^{AB\pqd CD}$
kuing 8# 2013-2-7 16:12
5# hongxian 或者你会问,那为什么斜的平行符号又不类似地定成拼音?而用了相对复杂的不知什么英文 sslash ? 其实这是因为在真 LaTeX 里有宏包 stmaryrd 里面就提供了斜的平行符号命令就是 \sslash,所以……
thread-1118-1-4.html: [组合] 再来一个排列组合
guanmo 1# 2013-2-5 18:21
$x_1+2x_2+2x_3=100$,$x_i$ 是正整数,则方程的根有(  )组
kuing 2# 2013-2-5 18:37
怎么都传两个相同的图?
kuing 3# 2013-2-5 18:40
这种题一般情况的计算量是很大的,还好现在这个系数特殊,可以转化。 根据奇偶性,显然 $x_1$ 必定为偶数,于是可设 $x_1=2x_4$, $x_4\in\mbb N^+$,代入方程化为 $x_2+x_3+x_4=50$,然后就可以用隔板法,即 $C_{49}^2$。
yes94 4# 2013-2-5 21:02
3# kuing 那个正整数解的个数公式,
kuing 5# 2013-2-6 14:10
为了方便存档,编辑了一下1#,打成 latex
thread-1119-1-4.html: [不等式] $\sum a=3$,证明:$\sum a\sqrt{a+b}\geqslant3\sqrt2$
yes94 1# 2013-2-5 20:42
本帖最后由 yes94 于 2013-2-5 20:47 编辑 已知$a、b、c\in R^+,a+b+c=3$,求证:$\sum a\sqrt{a+b}\geqslant 3\sqrt2$
yes94 2# 2013-2-5 20:58
1# yes94 px的亲笔吧?
kuing 3# 2013-2-5 23:11
2# yes94 这个证明牛
kuing 4# 2013-2-6 00:56
已知$a、b、c\in R^+,a+b+c=3$,求证:$\sum a\sqrt{a+b}\geqslant 3\sqrt2$ yes94 发表于 2013-2-5 20:42 临闪前终于也搞到了一个证法。 两边平方并齐次化,有 \[ \sum a\sqrt{a+b}\geqslant 3\sqrt2\iff \sum a^2(a+b)+2\sum ab\sqrt{(a+b)(b+c)}\geqslant \frac23\left( \sum a \right)^3, \] 注意到 \begin{align*} 2\sqrt{(a+b)(b+c)}&=a+2b+c-\bigl(\sqrt{a+b}-\sqrt{b+c}\bigr)^2 \\ & =b+\sum a-\frac{(a-c)^2}{\bigl(\sqrt{a+b}+\sqrt{b+c}\bigr)^2} \\ & \geqslant b+\sum a-\frac{(a-c)^2}{\sum a}, \end{align*} 所以只要证 \begin{equation}\label{20130206lczyzs} \sum a^2(a+b)+\sum ab\left( b+\sum a-\frac{(a-c)^2}{\sum a} \right)\geqslant \frac23\left( \sum a \right)^3, \end{equation} 整理,有 \begin{align*} \text{式}~\eqref{20130206lczyzs} & \iff \sum a^3+\sum a^2b+\sum ca^2+\sum ab\sum a-\frac23\left( \sum a \right)^3\geqslant \frac{\sum ab(a-c)^2}{\sum a} \\ & \iff \left( \sum a^2+\sum ab-\frac23\left( \sum a \right)^2 \right)\sum a\geqslant \frac{\sum ab(a-c)^2}{\sum a} \\ & \iff \sum \left( \left( \sum a \right)^2-6ab \right)(a-c)^2\geqslant 0, \end{align*} 记 $S_c=\left( \sum a \right)^2-6ab$, $S_a=\left( \sum a \right)^2-6bc$, $S_b=\left( \sum a \right)^2-6ca$,则 \[ S_a+S_b+S_c=3\left( \sum a \right)^2-6\sum ab=3\sum a^2>0, \] 以及由 Schur 不等式有 \begin{align*} S_aS_b+S_bS_c+S_cS_a&=3\left( \sum a \right)^4-12\sum ab\left( \sum a \right)^2+36abc\sum a \\ & =3\left( \sum a \right)\left( \left( \sum a \right)^3-4\sum ab\sum a+12abc \right) \\ & \geqslant 9abc\sum a>0, \end{align*} 因此由 SOS 定理知 $\sum S_c(a-c)^2\geqslant 0$ 成立,从而原不等式得证。
kuing 5# 2013-2-16 17:17
刚才天书在人教群里给出以下相关链接 http://www.aoshoo.com/bbs1/dispbbs.asp?boardid=14&Id=24794
thread-112-1-1.html: 向量系列命令
kuing 1# 2011-10-18 16:31
In[1]:= a = {1, 2, 3};              b = {4, 5, 6}; In[3]:= a.b Out[3]= 32 In[4]:= Dot[a, b] Out[4]= 32 In[5]:= Cross[a, b] Out[5]= {-3, 6, -3} In[6]:= Norm[a]              Norm[b] Out[6]= Sqrt[14] Out[7]= Sqrt[77] In[8]:= VectorAngle[a, b] Out[8]= ArcCos[(16 Sqrt[2/11])/7]
thread-1120-1-4.html: 今天存档了本论坛所有贴子(Archiver大字体版+讨论区图片)
kuing 1# 2013-2-6 19:56
下载链接:http://www.kuaipan.cn/file/id_56999842425078121.htm 说明: 进入“贴子”文件夹中,打开任意一个 html 文件都可以看贴(其中 index.html 为进入首页),并且可以点击上方的链接实现跳转(不要点下面那个“查看完整版本……”否则就跳出去了)。 由于添加了本地 MathJax 支持,故完全不依赖网络,也就是断网也可以看贴并且显示公式。 若用 IE 浏览器打开文件时可能会弹出提示,显示限制了脚本等,此时请单击提示并选择“允许阻止的内容--确定”方可加载 MathJax。 本次存档还包括了数学讨论区的所有贴内的图片,但未能与贴子实现链接。 本次存档时间:2013-2-6 存档类型:Archiver版(纯文字(大字体)无图片)+数学讨论区所有图片 贴子存档率:100%         悠闲数学娱乐论坛(kkkkuingggg.5d6d.net)kuing
yes94 2# 2013-2-6 21:04
祝贺!
kuing 3# 2013-2-6 21:09
2# yes94 存了档,安心些……
yes94 4# 2013-2-6 21:14
3# kuing 对! 到时想找帖子的时候,如果断网,就找网盘就行了,先把网盘地址存起!
kuing 5# 2013-2-6 21:20
4# yes94 断网也能进网盘?
yes94 6# 2013-2-6 22:16
4# yes94 断网也能进网盘? kuing 发表于 2013-2-6 21:20 不是我断网,指的是万一论坛像人教一样,在斯…巴…达两…会…时关闭一会儿呢
kuing 7# 2013-2-6 22:19
oh,原来是这个意思……
yes94 8# 2013-2-6 22:50
结果发现就在群里也有,
kuing 9# 2013-2-7 14:46
8# yes94 嗯,两个群都发了共享,虽然没什么人下载……不过也正常……
yes94 10# 2013-2-7 14:56
9# kuing 需要时再下载,现在不是好好的么? 论坛用的很爽啊!
kuing 11# 2013-2-7 14:57
10# yes94 也是
thread-1121-1-4.html: [不等式] 狂问问题
yayaweha 1# 2013-2-7 07:44
本帖最后由 yayaweha 于 2013-2-7 08:27 编辑 第三问
yayaweha 2# 2013-2-7 07:45
an,bn
realnumber 3# 2013-2-7 11:37
本帖最后由 realnumber 于 2013-2-7 11:52 编辑 \[A_n=\frac{1}{2}\frac{3}{3-1}\frac{3^2}{3^2-1}\frac{3^3}{3^3-1}\cdots\frac{3^{n-1}}{3^{n-1}-1}\] \[A_n=\frac{1}{2}\frac{1}{1-\frac{1}{3}}\frac{1}{1-\frac{1}{3^2}}\frac{1}{1-\frac{1}{3^3}}\cdots\frac{1}{1-\frac{1}{3^{n-1}}}\] $(1-a)(1-b)=1-a-b+ab>1-a-b,a,b\in (0,1)$, 类似地$(1-a)(1-b)(1-c)>(1-a-b)(1-c)>1-a-b-c$ \[而(1-\frac{1}{3})(1-\frac{1}{3^2})(1-\frac{1}{3^3})\cdots(1-\frac{1}{3^{n-1}})>1-\frac{1}{3}-\frac{1}{3^2}\cdots-\frac{1}{3^{n-1}}=1-\frac{\frac{1}{3}(1-\frac{1}{3^n})}{1-\frac{1}{3}}>1-\frac{\frac{1}{3}}{1-\frac{1}{3}}=\frac{1}{2}\] 所以$0<A_n<1<\frac{\pi}{2}$,而$\frac{\pi}{2}>\pi-b_n>1$,$$所以..
yayaweha 4# 2013-2-7 12:05
这个能这样推广吗
yayaweha 5# 2013-2-7 12:10
4# yayaweha 好像用数学归纳法是可以说得通的
kuing 6# 2013-2-7 12:21
又玩伯努力
yayaweha 7# 2013-2-7 12:26
6# kuing 广义贝努力不等式
yes94 8# 2013-2-7 14:53
本帖最后由 yes94 于 2013-2-7 16:37 编辑 那我就不玩被努力,刚才搞错了,为了挽回面子,再试试看: 先给出一个引理: 引理  设$k\in N_+$,则有$1-\dfrac1{3^k}\geqslant\dfrac{1+\dfrac1{3^k}}{1+\dfrac1{3^{k-1}}}$,当且仅当$k=1$取等号。        这个引理也就是:$(1-\dfrac1{3^k})(1+\dfrac1{3^{k-1}})\geqslant{1+\dfrac1{3^k}}$,这很容易证明的,展开即得。 下面给出异于贝努利的放缩: $A_n=\dfrac{1}{2}\cdot\dfrac{3}{3-1}\cdot\dfrac{3^2}{3^2-1}\cdot\dfrac{3^3}{3^3-1}\cdots\dfrac{3^{n-1}}{3^{n-1}-1}$,故$A_n=\dfrac{1}{2}\cdot\dfrac{1}{1-\dfrac{1}{3}}\cdot\dfrac{1}{1-\dfrac{1}{3^2}}\cdot\dfrac{1}{1-\dfrac{1}{3^3}}\cdots\dfrac{1}{1-\dfrac{1}{3^{n-1}}}$ 由引理可得,$(1-\dfrac{1}{3})(1-\dfrac{1}{3^2})(1-\dfrac{1}{3^3})\cdots(1-\dfrac{1}{3^{n-1}})=\prod_{k=1}^{n-1}(1-\dfrac1{3^k})\geqslant\prod_{k=1}^{n-1} \dfrac{1+\dfrac1{3^k}}{1+\dfrac1{3^{k-1}}}=\dfrac{1+\dfrac1{3^{n-1}}}{1+1}>\dfrac12$ 于是,$0<A_n<1<\dfrac{\pi}{2}$,以下略。
yayaweha 9# 2013-2-7 15:09
8# yes94 $$A_n$$是增数列吧
kuing 10# 2013-2-7 15:15
才看了下原题,觉得……这道题拼凑得……实在是狼狈
yayaweha 11# 2013-2-7 15:17
第(3)咋一看被吓到了,其实不等式结构很清楚
realnumber 12# 2013-2-7 16:09
10# kuing 犀利 ```
yes94 13# 2013-2-7 16:28
自从$2006\cdot$江西开播以来,被努力就成为经典,被人重提了! 是不是考生不知道贝努利,就不能做了? 另外,在8楼已修改。
yayaweha 14# 2013-2-7 16:33
大家都用引理 ,看来平时有所积累
yayaweha 15# 2013-2-7 16:38
那我就不玩被努力,刚才搞错了,为了挽回面子,再试试看: 先给出一个引理: 引理  设$k\in N_+$,则有$1-\dfrac1{3^k}>\dfrac{1+\dfrac1{3^k}}{1+\dfrac1{3^{k-1}}}$        这个引理也就是:$(1-\dfrac1{3^k})( ... yes94 发表于 2013-2-7 14:53 本题的标准答案就是这样
yes94 16# 2013-2-7 16:39
14# yayaweha 这个引理,不是众所周知的,谈不上积累,是临时根据问题想出的, 那个被努力,才算知识的积累,
yes94 17# 2013-2-7 16:40
15# yayaweha 看来我的做法和标答思想基本一致,
yayaweha 18# 2013-2-7 16:41
16# yes94 佩服,这个引理是怎么想的?
yes94 19# 2013-2-7 16:49
18# yayaweha 这个不好说吧,打代码麻烦,要修改很多次,显示才正常, 和以前的那道题一样,http://kkkkuingggg.5d6d.net/thread-1033-1-4.html
yayaweha 20# 2013-2-7 16:52
19# yes94 不明白你说什么
thread-1121-2-4.html:
yayaweha 21# 2013-2-7 18:48
yes94大神,现身呀!
yes94 22# 2013-2-7 18:54
21# yayaweha 正在辛苦、痛苦的编辑、测试代码,好麻烦! 只可意会不好言传,例如,那个引理还可加强为:    引理$2$  设$k∈N_+$,$k\geqslant2$,则有$(1-\dfrac1{3^k})(\dfrac32+\dfrac1{3^{k-1}})\geqslant{\dfrac32+\dfrac1{3^k}}$,      仅将$8$楼的引理中的“$1$”改为“$\dfrac32$”而已。为了配合$15$楼,这个引理也可以改写为:        $\dfrac{3^k}{3^k-1}<\dfrac{3(3^k+2)}{3^{k+1}+2}$($k\geqslant2$),      只要展开即可得证(等价于$3^k>6$),证明从略。 于是$A_n=\dfrac{1}{2}\cdot\dfrac{3}{3-1}\cdot\dfrac{3^2}{3^2-1}\cdot\dfrac{3^3}{3^3-1}\cdots\dfrac{3^{n-1}}{3^{n-1}-1}=\dfrac{1}{2}\prod_{k=1}^{n-1}\dfrac{3^k}{3^k-1}=\dfrac{3}{4}\prod_{k=2}^{n-1}\dfrac{3^k}{3^k-1}$,      故$A_n<\dfrac{3}{4}\cdot3^{n-2}\cdot\dfrac{3^2+2}{3^n+2}=\dfrac{11\cdot3^{n-1}}{4(3^n+2)}=\dfrac{11\cdot3^{n-1}}{12\cdot3^{n-1}+8}<1$($n\geqslant2$)       当$n=1$时显然成立吧,从略。
yayaweha 23# 2013-2-7 19:33
为什么要测试代码?
yes94 24# 2013-2-7 19:54
23# yayaweha 要不然,帖子就会显示编辑多次的情况,
thread-1122-1-4.html: [不等式] [转]第三届全国数学奥林匹克命题比赛获奖不等式题
kuing 1# 2013-2-7 20:48
转载于:http://www.aoshoo.com/bbs1/dispbbs.asp?boardid=48&Id=24752 1.(一等奖,河南武爱民)已知$a,b,c>0$,且$ \frac{1}{a^2+2}+\frac{1}{b^2+2}+\frac{1}{c^2+2}=\frac{1}{3} $,求证\[\frac{1}{a}+\frac{1}{b}+\frac{1}{c}>1\] 2.(二等奖,黑龙江张利民)已知$a\in (0,\pi ),x,y\in [0,\pi]$,且$x+y=a$,试确定$ f(x,y) = x\sin y+y\sin x $的最大值、最小值. 3.(二等奖,广东杨志明)已知\(a,b,c\in\left({0,\sqrt 2 }\right) \),且满足$ a^2+b^2+c^2+abc = 4 $,证明\[\sum \frac{a^2}{b+c}\ge \frac{1}{4}(1+2abc)+\frac{1}{4}\sqrt{3(a^2+b^2+c^2)}\] 4.(二等奖,安徽邹守文)设$a,b,c>0$,证明\[ \left({a^{2012}+a^{2010}+3}\right)\left({b^{2012}+b^{2010}+3}\right)\left({c^{2012}+c^{2010}+3}\right)\ge 3(a+b+c)^2 \] 5.(二等奖,甘肃庞耀辉)设\(a_1,a_2,\cdots,a_n\)是\(1,2,\cdots,n\)的一个排列,证明\[ \sum\limits_{i = 1}^{n-2}{\frac{1}{{a_k^3+a_{k+1}^3+a_{k+2}^3 }}}\ge\frac{{4(n-2)}}{{3n^2 (n+1)^2 }} \] 6.(二等奖,四川蒋明斌)求使不等式\[ 1\le\frac{a}{{\sqrt{a^2+\lambda bc}}}+\frac{b}{{\sqrt{b^2+\lambda ca}}}+\frac{c}{{\sqrt{c^2+\lambda ab}}}\le 2 \]对任意正实数$a,b,c$都成立的正常数$ \lambda $的取值范围,并分别求出等号的成立条件. 7.(二等奖,陕西樊益武)设$a,b,c>0$,且$a+b+c=1$,证明\[ \frac{a}{{\sqrt{b^2+c}}}+\frac{b}{{\sqrt{c^2+a}}}+\frac{c}{{\sqrt{a^2+b}}}\ge\frac{3}{2} \] 8.(三等奖,湖北甘超一)已知\(x,y \in R\),证明 \[ \left({\frac{x}{{x+1}}}\right)^2+\left({\frac{y}{{y+1}}}\right)^2+\left({\frac{1}{{1-xy}}}\right)^2\ge 1 \] 9.(三等奖,陕西刘康宁,陈孝庚)设\(a,b,c,x,y,z\)是实数,且满足\(A=ax+by+cz,B=ay+bz+ca,C=az+bx+cy\),设\(\left| {A - B} \right| \ge 1,\left| {B - C} \right| \ge 1,\left| {C - A} \right| \ge 1\) ,求\(\left( {a^2 + b^2 + c^2 } \right)\left( {x^2 + y^2 + z^2 } \right)\)的最小可能值. 10.(三等奖,四川刑凯慧)已知非负实数$x,y,z$满足$x^2+y^2+z^2=1$,证明 \[\frac{x+y}{z^2(x+y)+x^3+y^3}+\frac{y+z}{x^2(y+z)+y^3+z^3}+\frac{z+x}{y^2(z+x)+z^3+x^3}\le\frac92\] 11.(三等奖,辽宁张雷)已知正实数$a,b,c$,满足$a+b+c=ab+bc+ca$,若\[\frac1{1+a}+\frac1{1+b}+\frac1{1+c}\ge k\]恒成立,求$k$的最大值. 12.(三等奖,四川姚先伟)设$x_i>0$,且$\sum_{i=1}^n\frac{x_i}{1+x_i}=1$,证明 \[\sum_{i=1}^n\frac1{x_i^3}\ge n(n-1)^3\] 13.(三等奖,四川宿晓阳)在$\triangle ABC$中,证明 \[\frac{\cos^2B}{\sin^2A}+\frac{\cos^2C}{\sin^2B}+\frac{\cos^2A}{\sin^2C}\ge1\] 14.(三等奖,安徽)已知$n$是正整数,证明 \[\frac{1^2C_n^1}{1}+\frac{2^2C_n^2}{4}+\frac{3^2C_n^3}{7}+\cdots+\frac{n^2C_n^n}{3n-2}\le2^{n-2}n\left[n-\frac{3(n-1)^2+1}{9(n-1)^2-1}\right]\] _________________________转载部分到此_________________________ 以下开始就不是转载的了。 话说,其实我比较好奇的是这样的命题比赛是按照怎样的标准来评比的呢? 个人感觉有的题目似乎水了点,就看了前面几个: 第 1 题换元后等价于一年多前的这道题:http://kkkkuingggg.5d6d.net/thread-110-1-1.html; 第 3 题很弱,易证 $LHS\geqslant3/2\geqslant RHS$,昨晚群里也有人写过了,而且条件中那个 $\sqrt2$ 的上限是多余的; 第 4 题贴里可能写错了,我估计是 $(a^{2012}-a^{2010}+3)(b^{2012}-b^{2010}+3)(c^{2012}-c^{2010}+3)\geqslant3(a+b+c)^2$ 才对,而这其实只是减弱了熟知的 $(a^2+2)(b^2+2)(c^2+2)\geqslant3(a+b+c)^2$ 而已,正数条件也是多余的; …… 待续
kuing 2# 2013-2-7 20:59
第 8 题等价于这道:http://kkkkuingggg.5d6d.net/thread-1021-1-1.html(2009IMO) 第 12 题很弱,见后面的回贴(6#、15#、16#、19#、20#、22#) 第 10 题等价于 $\sum\frac1{1-xy}\leqslant\frac92$,也算是一个比较多见的不等式,详细见23#。
yes94 3# 2013-2-7 21:08
以后k去命题比赛,以人教著名版主的名气和实力,保准一等奖。
kuing 4# 2013-2-7 21:12
3# yes94 我不懂得命题,只是好奇是怎么评的……
yes94 5# 2013-2-7 21:23
4# kuing 你的那些不等式的推广啊、加强啊之类的,也算是命题噻, 猜测是名气(过去在刊物经常发表文章,为编辑们所熟悉)和该题的命制过程,这些的综合吧。 还有就是,参赛人数少了,质量就不是很好,就只有那几个编辑们熟悉的老面孔获奖了, 如果参赛人很多,那么也常常只看那几个编辑们熟悉的老面孔,新人如果不是特别标新立异,很难脱颖而出(但不排除有脱颖而出的新人)。
kuing 6# 2013-2-7 21:37
第 12 题也有点弱,换元后那个函数是下凸的,所以切线法、琴生都行,柯西、各种平均值不等式这些也自然能秒了……
kuing 7# 2013-2-7 23:12
我个人比较喜欢的是第 11 题: 11.(三等奖,辽宁张雷)已知正实数 $a$, $b$, $c$,满足 $a+b+c=ab+bc+ca$,若\[\frac1{1+a}+\frac1{1+b}+\frac1{1+c}\geqslant k\]恒成立,求 $k$ 的最大值。 条件和不等式都很简洁漂亮,而且虽然不等式的次数低,但是却没有想象中那样简单,玩下去还会发现并不是最常规的取等。
isea 8# 2013-2-7 23:50
本帖最后由 isea 于 2013-2-7 23:52 编辑 3# yes94 我不懂得命题,只是好奇是怎么评的…… kuing 发表于 2013-2-7 21:12 先不说评不评的事,我觉得数学里,其实对题不应该说“难”与“不难”,应该说“熟悉”与“不熟悉” 只因kuing对这些东西太熟,所以才有此感 而这便是一种境界
yes94 9# 2013-2-8 09:51
先不说评不评的事,我觉得数学里,其实对题不应该说“难”与“不难”,应该说“熟悉”与“不熟悉” 只因kuing对这些东西太熟,所以才有此感 而这便是一种境界 isea 发表于 2013-2-7 23:50 对题不应该说“难”与“不难”,应该说“熟悉”与“不熟悉” 呵呵!那学生怎么经常说题很难啊?对大多学生来说,很多题他们都是不熟悉的,否则怎么会叫苦数学难学? 不过呢,得奖不一定是最难的题才得奖,如果一道题让参赛的选手都做不出来,那么这道题就不太应该获奖(当然IMO、国家IMO选拔赛等等除外)。
kuing 10# 2013-2-8 10:30
评选的人都应该对初等不等式很熟悉吧,至少像第四题那种应该很明显能看出来。不是说要最难才得奖但也不能太水吧,我觉得应该要完全原创,要改编至少要搞深一些,而不是像第四题和第八题那样对经典题目改编而且两步之内轻松还原。
yes94 11# 2013-2-8 10:36
10# kuing 这就是IC说的,能一两步就看出来,这便是一种境界!
kuing 12# 2013-2-8 10:47
11# yes94 我相信你也能看出
yes94 13# 2013-2-8 10:54
12# kuing 我不敢看哦,关于不等式,只会高考难度的,或略大于高考难度以及竞赛初赛难度的,
kuing 14# 2013-2-8 11:01
13# yes94 别被标题给吓了。。。我前面点的几题你应该能做啊。
yes94 15# 2013-2-8 12:06
本帖最后由 yes94 于 2013-2-8 12:10 编辑 第 12 题也有点弱,换元后那个函数是下凸的,所以切线法、琴生都行,柯西、各种平均值不等式这些也自然能秒了…… kuing 发表于 2013-2-7 21:37 第$12$题的确太弱!下面用切线法搞定它! 做换元,令$a_i=\dfrac{x_i}{1+x_i}>0$,则$x_i=\dfrac{a_i}{1-a_i},i=1,2,\cdots,n$, 于是本题等价于已知$a_i>0(i=1,2,\cdots,n)$,且$a_1+a_2+\cdots+a_n=1$,求证:$\sum_{i=1}^{n}(\dfrac{1-a_i}{a_i})^3\geqslant n(n-1)^3$。 令$f(x)=\dfrac{1-x}{x}=\dfrac1x-1$,显然$f(x)$是$y=\dfrac1x$向下平移一个单位而得,故在$(0,+\infty)$是下凸函数。易求导得在$x=\dfrac1n$出的切线方程为$y=-n^2x+2n-1$,并且从图像可以看出$f(x)$在该切线的上方,于是$f(x)=\dfrac{1-x}{x}=\dfrac1x-1\geqslant -n^2x+2n-1$,当且仅当$x=\dfrac1n$取等号。     (这里从图形上看出,免去了证明,代码输入麻烦,而且没有令$f(x)=(\dfrac{1-x}{x})^3$,这是因为可以使求导简单些)。 于是$\sum_{i=1}^{n}(\dfrac{1-a_i}{a_i})^3=\sum_{i=1}^{n}[f(a_i)]^3\geqslant\dfrac{[\sum_{i=1}^{n}f(a_i)]^3}{n^2}$         $\geqslant \dfrac{[\sum_{i=1}^{n}(-n^2a_i+2n-1)]^3}{n^2}=\dfrac{[-n^2+(2n-1)n]^3}{n^2}=n(n-1)^3$。 取等号略。
yes94 16# 2013-2-8 12:28
15# yes94 不等式$f(x)=\dfrac{1-x}{x}=\dfrac1x-1\geqslant -n^2x+2n-1(x>0)$很好证明的,下面补上证明: 上述不等式$\Longleftrightarrow \dfrac1x\geqslant -n^2x+2n\Longleftrightarrow 1\geqslant -n^2x^2+2nx\Longleftrightarrow (nx-1)^2\geqslant 0$,    这显然成立 ,当且仅当$x=\dfrac1n$取等号。
kuing 17# 2013-2-8 12:30
16# yes94 等价于可以用 \iff
yes94 18# 2013-2-8 12:35
17# kuing 我的很多代码几乎全部在你那个置顶输入帖子里复制的,所以缩写与不缩写,效果一样, ,等到熟悉了,就能领会缩写是什么意思了,现在对代码输入稍微有点感觉了!
yes94 19# 2013-2-8 12:57
18# yes94 第 $12$ 题还可以这样做(用两次权方和即可): $\sum_{i=1}^{n}(\dfrac{1-a_i}{a_i})^3=\sum_{i=1}^{n}(\dfrac{1}{a_i}-1)^3$$\geqslant \dfrac{[\sum_{i=1}^{n}(\dfrac1{a_i}-1)]^3}{n^2}$$\geqslant\dfrac{(\sum_{i=1}^{n}\dfrac{1}{a_i}-n)^3}{n^2}\geqslant \dfrac{(\dfrac{n^2}{\sum_{i=1}^{n}a_i}-n)^3}{n^2}=n(n-1)^3$。
yes94 20# 2013-2-8 13:27
本帖最后由 yes94 于 2013-2-8 13:30 编辑 做起劲了,第$12$题再来一个方法: 自娱自乐,      证法三:显然$\dfrac{1}{a_i}-1>0,n-1\geqslant0$,由均值不等式可知,$(\dfrac{1}{a_i}-1)^3+(n-1)^3+(n-1)^3\geqslant 3(n-1)^2(\dfrac{1}{a_i}-1)$, 对上式求和,并记$M=\sum_{i=1}^{n}(\dfrac{1-a_i}{a_i})^3=\sum_{i=1}^{n}(\dfrac{1}{a_i}-1)^3$,则 $M+2n(n-1)^3\geqslant 3(n-1)^2[\sum_{i=1}^{n}(\dfrac{1}{a_i}-1)]\geqslant 3(n-1)^2(\dfrac{n^2}{\sum_{i=1}^{n}a_i}-n)=3(n-1)^2(n^2-n)$,     于是,$M\geqslant 3(n-1)^2(n^2-n)-2n(n-1)^3=n(n-1)^3$。              Done!
thread-1122-2-4.html:
kuing 21# 2013-2-8 13:57
20# yes94 试下 11 吧,我觉得挺不错的题,有点玩头。(当然,也有可能是我想复杂了,晚点贴下我的解法)
yes94 22# 2013-2-8 14:22
21# kuing 第$11$题不敢玩,继续第$12$题,要玩就玩尽兴!:   方法四:已知条件可以变形为$\sum\dfrac1{1+\dfrac1{x_i}}=1$,令$\dfrac1{x_i}=y_i$,则第$12$题变为             $\sum\dfrac1{1+y_i}=1$,求证:$\sum y_i^3\geqslant n(n-1)^3$. 由于$1=\sum\dfrac1{1+y_i}\geqslant\dfrac{n^2}{n+\sum y_i}$,故$n+\sum y_i\geqslant n^2$,即$\sum y_i\geqslant n^2-n$, 于是,$\sum y_i^3\geqslant \dfrac{(\sum y_i)^3}{n^2}\geqslant \dfrac{(n^2-n)^3}{n^2}=n(n-1)^3$.       完.
kuing 23# 2013-2-8 14:36
又找到了一个一步改编的: 第 10 题,分母立方和分解,约去 x+y 等,再代入已知条件,等价于 $\sum\frac1{1-xy}\leqslant\frac92$,而这是多年前Vasc的不等式,也算是比较多见的了,比如说 http://www.artofproblemsolving.com/Forum/viewtopic.php?t=36873 http://www.artofproblemsolving.com/Forum/viewtopic.php?t=109029 http://bbs.pep.com.cn/thread-503121-1-1.html 等等……
kuing 24# 2013-2-8 15:11
现在来贴一下我对第 11 题的解法。 我个人比较喜欢的是第 11 题: 11.(三等奖,辽宁张雷)已知正实数 $a$, $b$, $c$,满足 $a+b+c=ab+bc+ca$,若\[\frac1{1+a}+\frac1{1+b}+\frac1{1+c}\geqslant k\]恒成立,求 $k$ 的最大值。 条件和不等式都很简洁漂亮,而且虽然不等式的次数低,但是却没有想象中那样简单,玩下去还会发现并不是最常规的取等。 记 \[f(a,b,c)=\frac1{1+a}+\frac1{1+b}+\frac1{1+c},\] 由对称性,不妨设 $a\geqslant b\geqslant c$,则 \[ (a+b+c)^2\geqslant 3(ab+bc+ca)=3(a+b+c)\riff a+b+c\geqslant 3\riff a+b\geqslant 2, \] 由柯西不等式得 \begin{align*} f(a,b,c)&=\frac{2+a+b}{1+a+b+ab}+\frac{a+b-1}{(a+b-1)(1+c)} \\ & \geqslant \frac{\bigl( \sqrt{2+a+b}+\sqrt{a+b-1} \bigr)^2}{1+a+b+ab+(a+b-1)(1+c)} \\ & =\frac{\bigl( \sqrt{2+a+b}+\sqrt{a+b-1} \bigr)^2}{3(a+b)}, \end{align*} 令 $u=1/(a+b)$,由 $a+b\geqslant 2$ 知 $0<u\leqslant 1/2$,即有 \[ f(a,b,c)\geqslant \frac13\bigl( \sqrt{2u+1}+\sqrt{1-u} \bigr)^2=g(u), \] 求导有 \[ g'(u)=\frac23\bigl( \sqrt{2u+1}+\sqrt{1-u} \bigr)\left( \frac1{\sqrt{2u+1}}-\frac1{2\sqrt{1-u}} \right), \] 由此易证当 $0<u\leqslant 1/2$ 时恒有 $g'(u)\geqslant 0$ 且等号仅当 $u=1/2$ 时成立,所以 \[ f(a,b,c)\geqslant g(u)>g(0)=\frac43. \] 另一方面,设 $\veps$ 为一个很小的正数,令 $b=c=1/2+\veps$,代入条件可解出 $a=(3+4\veps-4\veps^2)/(8\veps)$,则 \[f\left(\frac{3+4\veps-4\veps^2}{8\veps},\frac12+\veps,\frac12+\veps\right)=\frac4{3+2\veps}+\frac{8\veps}{3+12\veps-4\veps^2},\] 此时,若 $\veps\to0^+$,则显然 $f\to4/3$,由此可见,$4/3$ 就是 $f(a,b,c)$ 的下确界。 综上,$k$ 的最大值就是 $4/3$。
kuing 25# 2013-2-8 16:02
为方便观看,将转载的原贴的内容敲入了过来,虽然怀疑有输入错误,但这里也未作改动。 另外还更新了一下2#
kuing 26# 2013-2-8 16:44
第 13 题也可以 13.(三等奖,四川宿晓阳)在 $\triangle ABC$ 中,证明 \[\frac{\cos^2B}{\sin^2A}+\frac{\cos^2C}{\sin^2B}+\frac{\cos^2A}{\sin^2C}\geqslant1.\] 虽然弱于一个已知结论,但胜在形式够简洁且轮换,也不是那么容易看出来跟那个结论的关系。具体地,利用面积公式及余弦定理,易知原不等式等价于 \[ \sum \frac{b^2(c^2+a^2-b^2)^2}{a^2}\geqslant 16S^2, \] 由柯西不等式有 \[ \sum \frac{b^2(c^2+a^2-b^2)^2}{a^2}\sum \frac{a^2}{b^2}\geqslant \left( \sum (c^2+a^2-b^2) \right)^2=\left( \sum a^2 \right)^2, \] 所以只要证 \[ a^2+b^2+c^2\geqslant 4\sqrt{\frac{a^2}{b^2}+\frac{b^2}{c^2}+\frac{c^2}{a^2}}\cdot S, \] 这就是一个已知结论(LBQ的书的封面就见过,记忆尤深)。
kuing 27# 2013-2-8 16:58
第 7 题也不错 7.(二等奖,陕西樊益武)设 $a$, $b$, $c>0$,且 $a+b+c=1$,证明 \[ \frac a{\sqrt{b^2+c}}+\frac b{\sqrt{c^2+a}}+\frac c{\sqrt{a^2+b}}\geqslant\frac32. \] 形式简洁,轮换分式有根号,看上去有一定难度,可惜实际上又没想象中那么难,过程在原贴中“天书”已经写过了,这里就不重复了,只是顺便找两个链接,需要证的 $\sum a(b^{2}+c)\leqslant\frac49$ 之前在这里证过: http://bbs.pep.com.cn/forum.php? ... 364&pid=5438322(31#) http://bbs.pep.com.cn/thread-416393-1-1.html
kuing 28# 2013-2-8 18:18
第 9 题也甚是不错,原贴有个地方显然打错了($B=ay+bz+ca$ 应改为 $B=ay+bz+c\color{red}x$) 9.(三等奖,陕西刘康宁,陈孝庚)设 $a$, $b$, $c$, $x$, $y$, $z$ 是实数,且满足 $A=ax+by+cz$, $B=ay+bz+cx$, $C=az+bx+cy$,设 $\abs{A-B}\geqslant1$, $\abs{B-C}\geqslant1$, $\abs{C-A}\geqslant1$,求\[(a^2 + b^2 + c^2)(x^2 + y^2 + z^2)\]的最小可能值。 竟然也只是三等奖哎…… 由于 $A-B$, $B-C$, $C-A$ 三者中必有两者同号,不妨设 $A-B$ 与 $C-A$ 同号,则由条件可知 $(A-B)(C-A)\geqslant1$,于是 \[(B-C)^2=(A-B+C-A)^2\geqslant 4(A-B)(C-A)\geqslant 4,\] 由拉格朗日恒等式,有 \begin{align*} (a^2+b^2+c^2)(x^2+y^2+z^2)&=A^2+(ay-bx)^2+(bz-cy)^2+(cx-az)^2 \\ & \geqslant A^2+\frac13(ay-bx+bz-cy+cx-az)^2 \\ & =A^2+\frac13(B-C)^2 \\ & \geqslant \frac43, \end{align*} 而当 $a=0$, $b=1$, $c=-1$, $x=-2/3$, $y=z=1/3$ 时满足条件且 $(a^2+b^2+c^2)(x^2+y^2+z^2)=4/3$,所以所求的最小值就是 $4/3$。
yes94 29# 2013-2-8 23:08
本帖最后由 yes94 于 2013-2-8 23:09 编辑 又找到了一个一步改编的: 第 10 题,分母立方和分解,约去 x+y 等,再代入已知条件,等价于 $\sum\frac1{1-xy}\leqslant\frac92$,而这是多年前Vasc的不等式,也算是比较多见的了,比如说 http://www.artofprob ... kuing 发表于 2013-2-8 14:36 怪不得我试了两次都做不出来哦,都分别放缩后,得到关于$x^2$、$y^2$、$z^2$的三个式子相加,狂喜!以为能用切线法(只需令$a=x^2$,$b=y^2$,$c=z^2$),结果发现不等式两次都反向了!放缩过度。     该帖里zhaobin做得很好,得到了Vasc的赞扬(他说“Nice solution, zhaobin”),但是zhaobin也说不是他首先用的此法(他说“It is not from me ”)那么谁先用的?这个柯西逆用的太妙了!
kuing 30# 2013-2-8 23:12
29# yes94 是谁先用我就不清楚了,印象中我当年也没做出来……
yes94 31# 2013-2-8 23:29
30# kuing 结果安正平也知道你的那个帖子了(他没有像你一样引用原始文献,不过,似乎也许是他自己思考出来的呢?): http://blog.sina.com.cn/s/blog_5618e6650101hedx.html Vasc赞扬zhaobin后,给zhaobin一个“下马威”(不要邪恶的去理解哈,题目挑战古来早有之 ): http://www.artofproblemsolving.com/Forum/viewtopic.php?t=109029 他说:“Nice solution, zhaobin. However, I think it doesn't work for the sharper inequalitiy $ \dfrac {1}{5-6ab}+\dfrac {1}{5-6bc}+\dfrac {1}{5-6ca}\leqslant 1 $”
kuing 32# 2013-2-9 01:14
猛然发现第 2 题其实也非常容易 2.(二等奖,黑龙江张利民)已知 $a\in (0,\pi)$, $x$, $y\in [0,\pi]$,且 $x+y=a$,试确定 $f(x,y) = x\sin y+y\sin x $ 的最大值、最小值。 由 $x$, $y\in [0,\pi]$ 显然 $f(x,y)\geqslant0$,又 $f(0,a)=0$,所以最小值为 $0$; 由对称性,不妨设 $x\geqslant y\geqslant0$,由于 $x+y\leqslant\pi$,易得 $\sin x\geqslant\sin y$,因此由切比雪夫不等式得 \[f(x,y)\leqslant\frac12(x+y)(\sin x+\sin y)=(x+y)\sin\frac{x+y}2\cos\frac{x-y}2\leqslant a\sin\frac a2,\] 当且仅当 $x=y=a/2$ 时取等号,所以最大值就是 $a\sin(a/2)$。
thread-1123-1-4.html: [不等式] 来自粉丝群的三角形三边$5\abs{\prod(a^2-b^2)}$
kuing 1# 2013-2-9 17:16
昨晚凌晨 天书(1846******) 2013-2-9 1:48:18 $a$, $b$, $c$ 三角形三边长,证 \[(a^2+b^2)(b^2+c^2)(c^2+a^2)\ge5\abs{(a^2-b^2)(b^2-c^2)(c^2-a^2)}.\] 做不出来... 话说我很想知道这不等式是谁发现的?竟然有着不平凡的“取等”条件,之所以要加双引号,是因为实际上还是取不到,但若允许两边之和等于第三边的退化情形则可以取等。 若有两边相等则显然成立,下设三边都不相等,由对称性,不妨设 $a<b<c$,则可令 $b^2=a^2+t$, $c^2=a^2+t+u$, $t$, $u>0$,不等式化为 \[(2a^2+t)(2a^2+2t+u)(2a^2+t+u)\geqslant 5tu(t+u),\] 由构成三角形的条件,有 \[a^2>(c-b)^2=b^2+c^2-2bc=2a^2+2t+u-2\sqrt{(a^2+t)(a^2+t+u)},\] 解得 \[a^2>\frac13\bigl( 2\sqrt{t^2+tu+u^2}-2t-u \bigr),\] 为方便书写,记 $m=\sqrt{t^2+tu+u^2}$,则只要证 \[(4m-t-2u)(4m+2t+u)(4m-t+u)\geqslant 135tu(t+u),\] 展开并按 $m$ 整理为 \[4^3m^3-12(t^2+tu+u^2)m+(t-u)(2(t+u)^2+tu)\geqslant 135tu(t+u),\] 即 \[52m^3+(t-u)(2(t+u)^2+tu)\geqslant 135tu(t+u),\] 由齐次性,不妨设 $t+u=1$,于是可令 $tu=(1-v^2)/4$, $v\in [0,1)$,则 \begin{align*} m&=\sqrt{1-\frac{1-v^2}4}=\frac{\sqrt{3+v^2}}2, \\ t-u&\geqslant -\sqrt{(t-u)^2}=-v, \end{align*} 于是又只要证 \[\frac{13}2\sqrt{(3+v^2)^3}-v\left( 2+\frac{1-v^2}4 \right)\geqslant \frac{135(1-v^2)}4,\] 即 \[26^2(3+v^2)^3\geqslant \bigl( 135(1-v^2)+v(9-v^2) \bigr)^2,\] 作差等价于 \[27(5v^3-v^2-45v+1)^2\geqslant 0,\] 显然成立,故原不等式得证。 我承认最后一步用了软件分解,不然还真证不来…… 可以看出,如果能允许 $a+b=c$,那么解出 $5v^3-v^2-45v+1=0$ 的 $v$,就能够得到取等条件,这就是我觉得不平凡的地方。
pxchg1200 2# 2013-2-9 21:09
Well,We have \[ a^2b^2c^2\geq |(a^2-b^2)(b^2-c^2)(c^2-a^2)| \] hence just check \[ (a^2+b^2)(b^2+c^2)(c^2+a^2)\geq 5a^2b^2c^2 \] Or \[ (a^2+b^2)(b^2+c^2)(c^2+a^2)\geq 8a^2b^2c^2 \geq 5a^2b^2c^2 \] :D
kuing 3# 2013-2-9 21:16
Well,We have \[ a^2b^2c^2\geq |(a^2-b^2)(b^2-c^2)(c^2-a^2)| \] hence just check \[ (a^2+b^2)(b^2+c^2)(c^2+a^2)\geq 5a^2b^2c^2 \] Or \[ (a^2+b^2)(b^2+c^2)(c^2+a^2)\geq 8a^2b^2c^2 \geq 5a^2b^2c^2 \] :D pxchg1200 发表于 2013-2-9 21:09 不会吧,按我上面的证法,右边的 5 是最佳系数,怎么会 >=8>=5 …… 第一个式子应该不成立,找找反例先
pxchg1200 4# 2013-2-9 21:19
3# kuing Schur in Third degree ?
kuing 5# 2013-2-9 21:20
3# kuing 找到了 $a=2$, $b=5$, $c=6$, $a^2b^2c^2=3600$, $\abs{(a^2-b^2)(b^2-c^2)(c^2-a^2)}=7392$
kuing 6# 2013-2-9 21:21
3# kuing Schur in Third degree ? pxchg1200 发表于 2013-2-9 21:19 3 次 Schur 的等价式是 $abc\ge(a+b-c)(b+c-a)(c+a-b)$
pxchg1200 7# 2013-2-9 21:22
6# kuing   我居然记错了,我去。。。。
kuing 8# 2013-2-9 21:23
7# pxchg1200 看来你真不在状态……
pxchg1200 9# 2013-2-9 21:23
8# kuing 最近不行了,唉,啥都做不出来。。。
kuing 10# 2013-2-9 21:25
9# pxchg1200 好好过个年,然后就会恢复状态哩
pxchg1200 11# 2013-2-9 21:25
9# pxchg1200 不知道能不能用 $a=x+y$替换后再SOS-Schur,或者直接AM-GM
kuing 12# 2013-2-9 21:27
11# pxchg1200 次数比较高,不敢这样玩,而且经过1#的证明我估计很难这样弄,“取等”条件太奇怪……
pxchg1200 13# 2013-2-9 21:28
6# kuing 我想问下这句什么意思的。。。
kuing 14# 2013-2-9 21:43
13# pxchg1200 我可不懂Vietnam文
pxchg1200 15# 2013-2-9 22:02
14# kuing 唉,被Can骗了,他说那个成立的,然后我想都没想就用了。。。。 Damn it! f~~~ it~!
kuing 16# 2013-2-9 22:04
15# pxchg1200 会不会是前面的文字表达了一些其他限制条件? 你让他把那段文字翻译一下呗
pxchg1200 17# 2013-2-9 23:08
16# kuing tian27546的证明: 证明:不妨设$a>b>c,a=y+z,b=z+x,c=x+y,x<y<z$,则等价证明 \[\prod[(y+z)^2+(z+x)^2]-5\prod[(y+z)^2-(z+x)^2]>0\] 整理得等价证明 \begin{align*} &2x[x^5+x^4(8y-2z)+x^3(18y^2+12yz-7z^2)+x^2(6y^3+30y^2z+10z^2y+6z^3)+x(-7y^4+10y^3z+28y^2z^2\\ &+30z^3y+18z^4)-2y^5+12y^4z+30y^3z^2+10y^2z^3+12z^4y+8z^5]\\ &+2(z^6-2yz^5-7y^2z^4+6y^3z^3+18y^4z^2+8y^5z+y^6)>0 \end{align*} 注意$x<y<z$,扫描下中括号系数和符号易得正数\\ 又注意到 \[z^6-2yz^5-7y^2z^4+6y^3z^3+18y^4z^2+8y^5z+y^6=(z^3-yz^2-4zy^2-y^3)^2\] 故原不等式得证.
kuing 18# 2013-2-9 23:12
17# pxchg1200 好bao力……最后也出现了一个完全平方,看来很难逃掉了……
kuing 19# 2013-2-10 20:37
天书  19:58:10 这个是褚小光老师的
pxchg1200 20# 2013-2-12 10:11
19# kuing another proof,by MathUniverse WLOG we can suppose that $a\ge b\ge c$. Just for the first observation, let's make substitution: $(a,b,c)=(\sqrt{x},\sqrt{y},\sqrt{z})$. Condition is equivalent to: $\sqrt{x}\ge \sqrt{y}\ge \sqrt{z} \ge \sqrt{x}-\sqrt{y}$ and inequality becomes: $(x+y)(x+z)(y+z)\ge 5(x-y)(x-z)(y-z) \quad (1)$ Firstly, let's prove prove following statemet: LEMA: For given condition for $x,y,z$, there exist exactly one $d_0\in [0,z]$ such that: $\sqrt{y-d_0}+\sqrt{z-d_0}=\sqrt{x-d_0}$ and for $d< d_0$,  inequality: $\sqrt{y-d}+\sqrt{z-d}>\sqrt{x-d}$ holds. Proof: Let's define $f(d)=\sqrt{y-d}+\sqrt{z-d}-\sqrt{x-d}$, then it's obvious that $f'(d)=\frac{1}{\sqrt{x-d}}-\frac{1}{\sqrt{y-d}}-\frac{1}{\sqrt{z-d}}<0$ Since $f(0)=\sqrt{y}+\sqrt{z}-\sqrt{x}\ge 0$ and $f(z)=\sqrt{y-z}-\sqrt{x-z}\le 0$, we have proved the statement of lema. Now, back to inequality $(1)$. Let's denote $F(x,y,z)=(x+y)(x+z)(y+z)-5(x-y)(x-z)(y-z)$. It's easy to see that for $0\le d\le d_0$, following inequality holds: $F(x,y,z)>F(x-d,y-d,z-d)$ and condition of inequality is fulfilled. Hence, it's enough to prove: $F(x-d_0,y-d_0,z-d_0)\ge 0$ where $x-d_0=(\sqrt{y-d_0}+\sqrt{z-d_0})^2$. But this is equivalent to proving starting inequality in the case when $a=b+1$. (Because we can WLOG suppose that $c=1$). And this is equivalent to: $ b^6-2b^5-7b^4+6b^3+18b^2+8b+1 \ge 0$ $\Leftrightarrow (b^3-b^2-4b-1)^2 \ge 0$. Equality holds when $\frac{b}{c}= t_0$ and $a=b+c$ where $t_0$ is positive root of $t^3-t^2-4t-1=0$ with all permutations. $\blacksquare$
thread-1124-1-2.html:
kuing 1# 2013-2-9 21:03
注:第一个不是鸟
thread-1125-1-4.html: [不等式] 来自粉丝群的不等式若干
kuing 1# 2013-2-9 22:06
天书(1846******) 2013-2-9 9:24:37 最近卡死的不等式,都不是我这弱渣能做的: 一中(3175*****) 2013-2-9 9:25:30 小天书在做kuing论坛的那份200题的不等式么 天书(1846******) 2013-2-9 9:26:29 0 0  不是,就是群或者贴吧论坛看到的不等式 注:第 5 题加上条件 $\sum_{k=1}^n a_k^2=1$  (5#说的)
kuing 2# 2013-2-9 22:11
解决情况(随时更新) 第 1 题,未解决…… 第 2 题,刚才在 http://kkkkuingggg.5d6d.net/thread-1123-1-1.html 已经证出,虽然不太满意…… 第 3 题,那个好像叫“混合算术-几何平均值不等式”,百度到了相关文章,不过下不下来…… 第 4 题,见 3#…… 第 5 题,未解决…… 第 6 题,排序易证,注意到 $\sqrt[k]{1+x^k}$ 当 $x>0$ 时是单调增函数…… 第 7 题,见 6#…… 第 8 题,未解决……
kuing 3# 2013-2-9 22:42
第 4 题还是老方法玩增量代换了。 左边通分为 \[LHS=\frac{\abs{(x-y)(y-z)(z-x)}}{xyz},\] 故若 $x$, $y$, $z$ 有二者相等时不等式显然成立,下设它们均不相等,由对称性,不妨设 $x<y<z$,则可令 $y=x+t$, $z=x+t+u$, $t$, $u>0$,由条件有 \[z\leqslant kx\iff x+t+u\leqslant kx\iff x\geqslant \frac{t+u}{k-1},\] 于是 \begin{align*} LHS&=\frac{tu(t+u)}{x(x+t)(x+t+u)} \\ & \leqslant \frac{tu(t+u)}{\frac{t+u}{k-1}\left( \frac{t+u}{k-1}+t \right)\left( \frac{t+u}{k-1}+t+u \right)} \\ & =\frac{(k-1)^3}k\cdot \frac{tu}{(t+u)(u+kt)}\\ & \leqslant \frac{(k-1)^3}k\cdot \frac{tu}{\bigl(\sqrt{tu}+\sqrt{ktu}\bigr)^2}\\ & =\frac{\bigl(\sqrt k+1\bigr)\bigl(\sqrt k-1\bigr)^3}k, \end{align*} 在上述所设情况下,取等条件为 $x=(t+u)/(k-1)$ 且 $\sqrt t:\sqrt u=\sqrt u:\sqrt{kt} \iff u=\sqrt k\cdot t$,此时 \begin{align*} \frac yx&=1+\frac tx=1+\frac{t(k-1)}{t+\sqrt k\cdot t}=\sqrt k ,\\ \frac zx&=1+\frac{t+u}x=k, \end{align*} 所以取等条件即为 $x:y:z=1:\sqrt k:k$ 及其任意交换次序。
kuing 4# 2013-2-10 00:59
刚才楼上后面用了函数,麻烦了,现在更新了一下,用柯西简单多了。
天书 5# 2013-2-10 19:51
第五题加上 \sum_{k=1}^n a_k^2=1
kuing 6# 2013-2-11 23:08
第 7 题。 两边平方等价于 \[\sum a^2(1-bc)+2\sum ab\sqrt{(1-bc)(1-ca)}\geqslant \frac{23}{24}-\frac{15abc}8,\] 注意到 \begin{align*} & 2\sqrt{(1-bc)(1-ca)}-(2-c(a+b)) \\ ={}&\frac{4(1-bc)(1-ca)-(2-c(a+b))^2}{2\sqrt{(1-bc)(1-ca)}+2-c(a+b)} \\ ={}&\frac{-c^2(a-b)^2}{2\sqrt{((a+b+c)^2-bc)((a+b+c)^2-ca)}+2-c(a+b)} \\ \geqslant{}&\frac{-c^2(a-b)^2}{2(a^2+b^2+c^2)+ab+2bc+2ca+2-c(a+b)} \\ ={}&\frac{-c^2(a-b)^2}{4-3(ab+bc+ca)}, \end{align*} 所以只要证 \[\sum a^2(1-bc)+\sum ab\left( 2-c(a+b)-\frac{c^2(a-b)^2}{4-3(ab+bc+ca)} \right)\geqslant \frac{23}{24}-\frac{15abc}8,\] 上式展开为 \[\sum a^2-abc\sum a+2\sum ab-2abc\sum a-\frac{abc\sum c(a-b)^2}{4-3(ab+bc+ca)}\geqslant \frac{23}{24}-\frac{15abc}8,\] 由 $a+b+c=1$,记 $q=ab+bc+ca$, $r=abc$,则 $0<q\leqslant 1/3$, $0<r\leqslant q/9\leqslant 1/27$,上式化简等价于 \begin{equation}\label{20130211tsbdsrgt7zyz} \frac1{24}\geqslant \frac{r(q-9r)}{4-3q}+\frac98r. \end{equation} 若 $0<q\leqslant 1/4$,则 $0<r\leqslant 1/36$,故 \begin{align*} \frac1{24}-\frac{r(q-9r)}{4-3q}-\frac98r&\geqslant \frac1{24}-\frac{r(1-36r)}{13}-\frac98r \\ & \geqslant \frac1{24}-\frac{3r(1-36r)}8-\frac98r \\ & =\frac1{24}(18r-1)^2>0, \end{align*} 所以此时式 \eqref{20130211tsbdsrgt7zyz} 成立; 若 $1/4<q\leqslant 1/3$,则将式 \eqref{20130211tsbdsrgt7zyz} 整理为 \[\frac1{24r}+\frac{9r}{4-3q}\geqslant \frac q{4-3q}+\frac98,\] 由双勾函数性质易证左边关于 $r$ 递减,故由 Schur 不等式的等价形式 $r\geqslant (4q-1)/9$ 可知只需证 \[\frac9{24(4q-1)}+\frac{4q-1}{4-3q}\geqslant \frac q{4-3q}+\frac98,\] 作差分解等价于 \[\frac{(1-3q)(14-17q)}{2(4-3q)(4q-1)}\geqslant 0,\] 显然成立,故此时式 \eqref{20130211tsbdsrgt7zyz} 也成立。 综上,原不等式得证。
kuing 7# 2013-2-12 18:09
6# kuing 如果没计算错的话,用楼上这个方法可以将原不等式加强为 \[\sum a\sqrt{1-bc}\geqslant\sqrt{\frac{323}{324}-\frac{35abc}{12}}.\] 当然了,这肯定还不是最佳系数。
zdyzhj 8# 2013-2-14 01:07
本帖最后由 zdyzhj 于 2013-2-14 01:08 编辑 第7题的有理化不等式有错 \frac{-c^2(a-b)^2}{4-3(ab+bc+ca)}
kuing 9# 2013-2-14 01:20
第7题的有理化不等式有错 \frac{-c^2(a-b)^2}{4-3(ab+bc+ca)} zdyzhj 发表于 2013-2-14 01:07 哪一个等号(或不等号)错了?
realnumber 10# 2013-2-14 18:56
本帖最后由 realnumber 于 2013-2-15 17:36 编辑 只是1.的最大值,另一边还是没头绪,--呼~~~终于修改好了. 假设$x$最大, $(1+x+y^2)(1+y+z^2)(1+z+x^2)\le (1+x+y)(1+y+z)(1+z^2+x^2)$---(注:这个结论是通过几何画板画图猜的,如果不能肯定它正确,一定不会折腾下去 ,下面证明) 两边展开,即要证明\[((1+x)(1+y)+(1+x)z^2+(1+y)y^2+y^2z^2)(1+z+x^2)\le ((1+x)(1+y)+(1+x)z+(1+y)y+yz)(1+z^2+x^2)\] \[\iff (z-z^2)(1+x)(1+y)+zy(1+y)(y-z)\le (1+x^2)[(1+x)(z-z^2)+(1+y)(y-y^2)+yz(1-yz)]+z^3(y-y^2) \] \[ \liff (z-z^2)(1+x)(1+y)+zy(1+y)(y-z)\le (1+x)(z-z^2)+(1+y)(y-y^2)+yz(1-yz)---(*) \] \[ zy(1+y)(y-z)\le yz(1-yz)\iff y(1+y)\le 1+z,而x最大,y\le 0.5,所以成立,那么要证明(*),即要证明\] \[\liff (z-z^2)(1+x)(1+y)\le (1+x)(z-z^2)+(1+y)(y-y^2)\] \[\iff (z-z^2)(1+x)y\le (1+y)(y-y^2)\iff (z-z^2)(1+x)\le0.5\le 1-y^2,y,z都不超过0.5,所以成立.\] 记$(1+x+y)(1+y+z)(1+z^2+x^2)=(2+2y+xz)(1+x^2+z^2)=f(x)$ 以下先固定$y$,对$x$求导,$y'_x=0,z'_x=-1,x'_x=1$ $f'_x(x)=(z-x)(1+x^2+z^2)+(2+2y+xz)(2x-2z)=(x-z)(4+4y+2xz-1-x^2-z^2)>0$, 所以在$x=1-y,z=0$取得最大,此时$f(x)=2(1+y)(1+(1-y)^2),0\le y\le 1-x$,继续对y求导,得到$y=0$,最大, 即原不等式左边成立,在x=1,y=z=0最大(若x最大的话) ---奇怪,不知道按到哪个键了,一下子发了3,4个帖子,
thread-1126-1-4.html: [不等式] $\sum a\sqrt{a^2+bc}\geqslant\sqrt2\sum{ab}$
yayaweha 1# 2013-2-10 10:17
证明:对任意的非负实数a,b,c有 $$\sum a\sqrt{a^2+bc}\geqslant\sqrt2\sum{ab}$$
yayaweha 2# 2013-2-10 10:25
本帖最后由 yayaweha 于 2013-2-10 10:30 编辑 $$a\sqrt{a^2+bc}+b\sqrt{b^2+ca}+c\sqrt{c^2+ab}\geqslant\underline{\frac{1}{\sqrt2}(a(a+\sqrt{bc})+b(b+\sqrt{ca})+c(c+\sqrt{ab}))\geqslant\frac{1}{\sqrt2}(\sqrt{ab}(a+b)+\sqrt{bc}(b+c)+\sqrt{ca}(c+a)}\geqslant\sqrt2(ab+bc+ca)$$
yayaweha 3# 2013-2-10 10:31
加下划线的部分怎么变得?
kuing 4# 2013-2-10 10:33
3# yayaweha 那个应该是用了Schur不等式的 $\sum a^2(a-b)(a-c)\ge0$
yayaweha 5# 2013-2-10 10:52
4# kuing $$\sum a^2(a-b)(a-c)\ge0\iff\sum a^2(a^2+bc)\ge\sum a^3(b+c)$$ 还是看不出来
kuing 6# 2013-2-10 11:19
5# yayaweha $a\to\sqrt a$, $\sum a^3(b+c)=\sum ab(a^2+b^2)$
yayaweha 7# 2013-2-10 12:13
本帖最后由 yayaweha 于 2013-2-10 12:50 编辑 $\sum a^3(b+c)=\sum ab(a^2+b^2)$这个怎么一眼瞄到? 顺带问下,怎样用柯西不等式证明AM-GM
yayaweha 8# 2013-2-10 13:11
再比如这个又是怎么一眼看出来(知道能这样变)$$\sum x^2y-\sum xy^2=\prod(x-y)$$
yayaweha 9# 2013-2-10 14:53
证明:在$\Delta ABC$中有 $$\sum a^3-2\sum a^2(b+c)+9abc\le0$$
kuing 10# 2013-2-10 15:46
9# yayaweha 容易配成 $\sum(b+c-a)(a-b)(a-c)\ge0$
yayaweha 11# 2013-2-10 17:15
10# kuing 这怎么配,是看出来的?
kuing 12# 2013-2-10 17:19
11# yayaweha 我现在还在外面。。。
yayaweha 13# 2013-2-10 17:41
本帖最后由 yayaweha 于 2013-2-10 17:46 编辑 $$\sum a^3-\sum a^2(b+c)+3abc=\sum a(a-b)(a-c)$$ $$\sum a^2(b+c)-6abc=\sum (b+c)(a-b)(a-c)$$ $$\sum a^3-2\sum a^2(b+c)+9abc=\sum(a-(b+b))(a-b)(a-c)$$
kuing 14# 2013-2-10 17:46
13# yayaweha 嗯,我就是这样看的,分出一个Schur,剩下那个也简单。
yayaweha 15# 2013-2-10 17:48
我是看着你的结果反推的
kuing 16# 2013-2-10 18:22
$\sum a^3(b+c)=\sum ab(a^2+b^2)$这个怎么一眼瞄到? 顺带问下,怎样用柯西不等式证明AM-GM yayaweha 发表于 2013-2-10 12:13 重新组合一下而已……
yayaweha 17# 2013-2-10 18:24
16# kuing 柯西呢?
kuing 18# 2013-2-10 18:24
再比如这个又是怎么一眼看出来(知道能这样变)$$\sum x^2y-\sum xy^2=\prod(x-y)$$ yayaweha 发表于 2013-2-10 13:11 很早就知道了,至于当年是不是一眼看出就不清楚了,反正一直记得。 类似更高次的情形也会有这种因式,因为任意两元相等时原式为0
kuing 19# 2013-2-10 18:26
18# yayaweha 你是说证 n 元均值?没想过,一下也没想到。 刚到家,晚点再玩
yayaweha 20# 2013-2-11 18:00
18# kuing 高次的是怎样得?
thread-1126-2-4.html:
kuing 21# 2013-2-11 18:26
20# yayaweha 没研究过一般情况,只记得当年用软件分解 $\sum xy^n-\sum x^ny$ 是很有规律的,也没证明过。 设 $\sum xy^n-\sum x^ny=(x-y)(y-z)(z-x)\cdot P_n(x,y,z)$,记 $\sum$ 为循环求和,$\sum_{sym}$ 为对称求和 \begin{array}{|l|l|} \hline\\ n & P_n(x,y,z) \\ \hline\\ 2 & 1 \\ \hline\\ 3 & \sum x \\ \hline\\ 4 & \sum x^2+\sum xy \\ \hline\\ 5 & \sum x^3+\sum_{sym} x^2y + xyz \\ \hline\\ 6 & \sum x^4+\sum_{sym} x^3y + \sum x^2y^2 + xyz\sum x \\ \hline\\ 7 & \sum x^5+\sum_{sym} x^4y + \sum_{sym} x^3y^2 + xyz\sum x^2 + xyz\sum xy \\ \hline\\ 8 & \sum x^6+\sum_{sym} x^5y+\sum_{sym} x^4y^2 +\sum x^3y^3+ xyz\sum x^3 + xyz\sum_{sym} x^2y+x^2y^2z^2 \\ \hline\\ \vdots & \vdots \\ \hline \end{array}
yayaweha 22# 2013-2-11 18:50
20# yayaweha 没研究过一般情况,只记得当年用软件分解 $\sum xy^n-\sum x^ny$ 是很有规律的,也没证明过。 设 $\sum xy^n-\sum x^ny=(x-y)(y-z)(z-x)\cdot P_n(x,y,z)$,记 $\sum$ 为循环求和,$\sum_{sym}$  ... kuing 发表于 2013-2-11 18:26 好东西
kuing 23# 2013-2-11 19:18
续21# 大概可以归纳成这样,当 $n\geqslant5$, 若 $n$ 为奇数,令 $n=2k+1$,则 \[P_n=\sum x^{n-2}+\sum_{sym} x^{n-3}y+\sum_{sym} x^{n-4}y^2+\cdots+\sum_{sym} x^ky^{k-1}+xyzP_{n-3};\] 若 $n$ 为偶数,令 $n=2k$,则 \[P_n=\sum x^{n-2}+\sum_{sym} x^{n-3}y+\sum_{sym} x^{n-4}y^2+\cdots+\sum_{sym} x^ky^{k-2}+\sum x^{k-1}y^{k-1}+xyzP_{n-3}.\]
pxchg1200 24# 2013-2-12 10:12
1# yayaweha 这个是所谓的hjj书上的那个题么?
yayaweha 25# 2013-2-12 10:28
yes
thread-1127-1-4.html: [函数] 关于2013上海虹口一模数学23题第1小题另一种解法的辨析
longma 1# 2013-2-10 17:20

kuing 2# 2013-2-10 18:35
$f(x+2a)=f(x)$ 是 $f(x+a)=-f(x)$ 的必要而不充分条件,所以由前者得到的 $a$ 的集合也是一个必要而不充分条件,一定有多余的东西。
longma 3# 2013-2-10 18:59
2# kuing 分析的有理,多谢哈,祝新年快乐,合家安康!
kuing 4# 2013-2-10 19:01
3# longma 客气客气,同乐同乐
thread-1128-1-2.html: [几何] 如何判断空间法向量的方向
资本主义 1# 2013-2-11 15:22
空间法向量的题我会做,但是我不知道n=(X1,Y1,Z1) 的方向和它的正负号!
kuing 2# 2013-2-11 15:37
这类问题似乎也算是FAQ,当年在人教论坛收藏了几个贴子 http://bbs.pep.com.cn/thread-372369-1-1.html http://bbs.pep.com.cn/thread-263239-1-1.html http://bbs.pep.com.cn/thread-421574-1-1.html
kuing 3# 2013-2-11 16:27
话说收藏的那几个贴子其实一直没细看过,刚才看了看,也有点乱,有的图片也显示不出来,还是自己想了一下,其实用向量叉乘就可以很容易解决这些问题,因为叉乘就已经规定了方向,就不会出现现在那样的利用两个数量积为0而解出两个方向的法向量之后还要判断什么相等还是互补的麻烦事。 简单来说,先来看看平面上的四点,$A$, $B$, $P$, $Q$,其中 $P$, $Q$ 在直线 $\ell$ 上,且 $A$ 和 $B$ 在 $\ell$ 的异侧,由叉乘方向的规定可知,向量 $\vv m=\vv{PA}\times\vv{PB}$ 与 $\vv n=\vv{QA}\times\vv{QB}$ 的方向相反,即 $\bigl\langle\vv m,\vv n\bigr\rangle= 180^\circ$。 现在,以 $\ell$ 为界,将其中一边的半平面折起来,比如说将 $P$ 所在的那半平面折起,此时 $\vv m$ 也跟着绕 $\ell$ 旋转,而显然该半平面折起多少度,$\vv m$ 也旋转多少度,因此总有 $\bigl\langle\vv m,\vv n\bigr\rangle=\text{二面角}~P\text{-}AB\text{-}Q$。
kuing 4# 2013-2-11 16:39
注意一下,向量叉乘是有顺序讲究的,不满足交换律,$\vv a \times \vv b = -\vv b \times \vv a$,所以上面的顺序不要乱调,要调就两边同时调。 讲开又讲,一直不明白为什么高中教向量点乘但不教叉乘,明明是一对的……很多问题用叉乘解决很方便,点乘叉乘一起用更是有力的工具……
kuing 5# 2013-2-16 16:12
试试写个具体计算表达式出来,方便以后用。 $\newcommand{\relph}[1]{\mathrel{\phantom{#1}}}$ 接着3#,有 \[\text{二面角}~P\text{-}AB\text{-}Q=\bigl\langle\vv m,\vv n\bigr\rangle=\arccos\frac{\vv m\cdot\vv n}{\bigl|\vv m\bigr|\cdot\bigl|\vv n\bigr|},\] 设 $A(x_A,y_A,z_A)$, $B(x_B,y_B,z_B)$, $P(x_P,y_P,z_P)$, $Q(x_Q,y_Q,z_Q)$,则有 \begin{align*} \vv m\cdot\vv n&=\bigl(\vv{PA}\times\vv{PB}\bigr)\cdot\bigl(\vv{QA}\times\vv{QB}\bigr)\\ &=\begin{vmatrix} \vv{PA}\cdot\vv{QA} & \vv{PA}\cdot\vv{QB}\\ \vv{PB}\cdot\vv{QA} & \vv{PB}\cdot\vv{QB}\\ \end{vmatrix}\\ &=\sum_{x,y,z}(x_P-x_A)(x_Q-x_A)\sum_{x,y,z}(x_P-x_B)(x_Q-x_B)\\ &\relph{=}{}-\sum_{x,y,z}(x_P-x_A)(x_Q-x_B)\sum_{x,y,z}(x_P-x_B)(x_Q-x_A), \end{align*} 以及 \begin{align*} \bigl|\vv m\bigr|&=\sqrt{\begin{vmatrix} y_P-y_A & z_P-z_A\\ y_P-y_B & z_P-z_B \end{vmatrix}^2+\begin{vmatrix} z_P-z_A & x_P-x_A\\ z_P-z_B & x_P-x_B \end{vmatrix}^2+\begin{vmatrix} x_P-x_A & y_P-y_A\\ x_P-x_B & y_P-y_B \end{vmatrix}^2},\\ \bigl|\vv n\bigr|&=\sqrt{\begin{vmatrix} y_Q-y_A & z_Q-z_A\\ y_Q-y_B & z_Q-z_B \end{vmatrix}^2+\begin{vmatrix} z_Q-z_A & x_Q-x_A\\ z_Q-z_B & x_Q-x_B \end{vmatrix}^2+\begin{vmatrix} x_Q-x_A & y_Q-y_A\\ x_Q-x_B & y_Q-y_B \end{vmatrix}^2}, \end{align*} 代入……还是算了吧……
海盗船长 6# 2013-2-16 22:03
一般的题目画个图目测就行了
zwl1972 7# 2013-3-28 21:29
看向量坐标的符号,就可以知道它指向第几卦限,求二面角时,z坐标正指向上,负指向下,结合图形很容易知道法向量指向二面角的内外,若一里一外,向量角$~=~$二面角,否则为补角
isea 8# 2013-4-6 00:08
本帖最后由 isea 于 2013-4-6 00:11 编辑 原来这个帖子的内容谈的是如何判断二面角是钝角还是锐角 其实,这个玩意可能用直角(以其中一个法向量构造垂直平面)来判断 具体要用到四点共面(及推广):若 $\vv {OP} =x\vv {OA}+y\overrightarrow{OB}+z\overrightarrow{OC} ,P,A,B,C$共面$\iff x+y+z=1$ 由此$x+y+z>1,x+y+z<1$又有什么几何意义呢?从这个方面严格判断,相对好操作。
dualliot 9# 2013-4-10 11:41
我一直用外积,指向很清楚.右手系而已.
thread-1129-1-1.html: 数学空间
yayaweha 1# 2013-2-11 20:10
2011年的数学空间怎么看不到了?
kuing 2# 2013-2-11 20:57
嗯,我也发现那里没显示2011的链接了,不过这不归我管,等过完年再跟他们说说。 不过,呃,当时没下载PDF吗?
yayaweha 3# 2013-2-11 21:06
下载了,网站上没有觉得奇怪
kuing 4# 2013-2-11 21:15
呵呵,上面不重视,是这样的了。 经常拖得很慢,这次有新的看已经不错了,之前我还以为要过完年他们放完假了才出2013的呢,结果前几天出了我都觉得意外了。
kuing 5# 2013-2-26 21:59
昨天才有回应,现在应该OK了。
thread-113-1-1.html: 整数序列拟合(类似于找规律)
kuing 1# 2011-10-18 16:36
输入 FindSequenceFunction[{2, 3, 10, 15, 26, 35, 50, 63, 82}, n] 输出 -(-1)^n + n^2 不一定能找到,不过简单的一般可以 不过怎么说都好,能做到这样的算法,已经很强大了……
海盗船长 2# 2011-10-20 14:31
原来还有这个功能。。强大!
kuing 3# 2011-10-20 22:45
2# 海盗船长 其实我以前就说过
thread-1130-1-1.html: 求助:宽带薪酬计算过程
童光红 1# 2013-2-12 10:26
公司薪酬市场定位: 调查结束以后,将调查分析的结果和工作评价的结果结合起来,用最小二乘法来进行拟合,将评价点数设为X,市场薪酬水平设为Y,那么薪酬曲线的方程为:Y=bX+a,利用联合方程解出a,b :   有工作描述的内容,我们得到附表1(见插图1): 代入附表1值的数值得: a=-611 b=5.492 则 Y=5.492X-611 可得附表2(见插图2) 问: 1、数值得: a=-611 b=5.492 则 Y=5.492X-611    这个结果的计算过程是怎样的?能给予详细的解答吗?谢谢
童光红 2# 2013-2-12 10:36
本帖最后由 童光红 于 2013-2-12 10:49 编辑 说明,我已将上述数字代入如下方程式: a=[∑X²×∑y×∑(xy) ]/ [n∑x²•(∑x²)] b=[n∑(xy)-(∑x)( ∑y) ]/ [n∑x²-(∑x) ²_] 但公式中的数字如天文数字一样,连计算机都无法计算了。 谁能告诉上题的计算过程吗?我都憋了一个新年了, 不甚感激!
kuing 3# 2013-2-12 16:21
后两列的数据不全正确 再说,这些数据还远未到“天文数字”的程度,计算机怎么会算不了?
kuing 4# 2013-2-12 16:37
反正公式就是那样,而且既然有了合计,就不用一个个 x 和 y 代入计算,只要将那些合计的四个数据代入公式就行了,用计算机不会算不了的。 PS、你上面那里贴的公式乱码,这里我打打 $Y=bX+a$ \begin{align} b&=\frac{n\sum x_i y_i - \left(\sum x_i\right)\left(\sum y_i\right)}{n\sum x_i^2 - \left(\sum x_i\right)^2},\\ a&=\frac{\sum y_i}n-b\cdot\frac{\sum x_i}n, \end{align} 这样先算 $b$ 再算 $a$,简单些。
kuing 5# 2013-2-12 16:58
用 excel 算了一下,发现上面那个表根本就是坑人啊,大部分数据都是错的。 此表附件: asdf1.xls (14.5 KB)
童光红 6# 2013-2-13 10:42
我也是从别的网上的PPT中截图来的,可能有点错了
童光红 7# 2013-2-13 10:46
∑x与n∑x是不是一个意思啊? 请各位老师指点嘛
童光红 8# 2013-2-13 10:54
4# kuing 谢谢老师指点,祝老师新年快乐! 请问 ∑x与n∑x是不是一个意思啊?
kuing 9# 2013-2-13 12:28
8# 童光红 当然不是。 PS、我不是老师
童光红 10# 2013-2-13 15:19
5# kuing 请问老师,我应该如何将【插图2】表的数字代入你给出的公式中啊? 插图2
童光红 11# 2013-2-13 15:29
本帖最后由 童光红 于 2013-2-13 15:34 编辑 是不是这样代入? b=31732259-9781×43962/7015227-9781×9781     (1) a=43962/n-b·9781/n                                              (2) 我感觉算式(2)有点不对劲的,算出b得数后可以代入算式(1),那么n又是什么呢? 如果不是这样的,请问老师,我应该怎样代入方程式啊?或代入的过程是一个什么样子的算式? 我高中没读完,仅自学而已,现在处理员工宽带薪酬,被这种最小二乘法算式给卡住了,求老师百忙之际,给予点破和指明,谢谢,辛苦老师了,我太笨了
kuing 12# 2013-2-13 15:49
…… n 就是样本个数,现在不是有14种职位的数据么,所以 n=14 你第(1)式也没把 n 代入去 应该是 $\dfrac{14\times31732259 - 9781\times43962}{14\times7015227 - 9781^2}$
童光红 13# 2013-2-13 16:48
12# kuing 看明白了,终于看明白了,谢谢老师,谢谢老师!欧耶
yayaweha 14# 2013-2-13 18:32
本帖最后由 yayaweha 于 2013-2-13 18:38 编辑 8# kuing 当然不是。 PS、我不是老师 三人行必有我师焉
thread-1131-1-4.html: [不等式] 越南人也叫着过年。。。
pxchg1200 1# 2013-2-12 11:52
Let $ x,y,z\in R$, Show that: \[  2(x^2+y^2)(y^2+z^2)(z^2+x^2)+16x^2y^2z^2\geq (x-y)^2(y-z)^2(z-x)^2\]
thread-1132-1-4.html: [不等式] MathUniverse的一个题
pxchg1200 1# 2013-2-12 11:54
For $a,b,c \ge 0$ such that $a^2+b^2+c^2=3$, prove: \[ 3(a^3+b^3+c^3)+2abc \ge 11\] 我除了暴力和pqr以外想不到其他的好办法,有没有基本的证明?
thread-1133-1-4.html: [函数] 一道函数题
yayaweha 1# 2013-2-12 17:11
本帖最后由 yayaweha 于 2013-2-12 17:14 编辑 若不等式$\Large(1+\frac{1}{n})^{n+a}\ge e$对任意的$n\in N^*$都成立,求常数$a$的最小值。
kuing 2# 2013-2-12 17:18
好像在哪儿见过……有没有出处dǎng在……
yayaweha 3# 2013-2-12 17:20
本帖最后由 yayaweha 于 2013-2-12 17:26 编辑 出处很正规,高三考试题
kuing 4# 2013-2-12 18:41
3# yayaweha 那种很难想的所谓高中标准解法我如无意外是想不到的了,只懂用野猪在网刊写的那种,先求极限找必要条件再验证充分性……
kuing 5# 2013-2-12 18:49
4# kuing 具体简单来说就是取对数等价于 \[a\geqslant \frac1{\ln (1+1/n)}-n,\] 求极限有 \begin{align*} \lim_{n\to+\infty}\left( \frac1{\ln (1+1/n)}-n \right)&=\lim_{x\to0^+}\left( \frac1{\ln (1+x)}-\frac1x \right) \\ & =\lim_{x\to0^+}\frac{x-\ln (1+x)}{x\ln (1+x)} \\ & =\lim_{x\to0^+}\frac{1-1/(1+x)}{x/(1+x)+\ln (1+x)} \\ & =\lim_{x\to0^+}\frac1{1+\ln (1+x)^{1/x}+\ln (1+x)} \\ & =\frac12, \end{align*} 所以必须有 $a\geqslant1/2$,然后验证 $a=1/2$ 时原式递减,这是容易的。
yayaweha 6# 2013-2-12 19:35
5# kuing 额,你跟标准答案80%相似
Tesla35 7# 2013-2-12 19:51
这尼玛也是高中题了。。。。。。。。。。。。
yayaweha 8# 2013-2-12 20:10
7# Tesla35 嘿呀!2013广东六校联考
kuing 9# 2013-2-12 20:20
6# yayaweha really??!! 贴来看看
yayaweha 10# 2013-2-12 20:24
9# kuing http://wenku.baidu.com/view/d609370bde80d4d8d15a4f67.html 我考试时没做出来
kuing 11# 2013-2-12 20:28
10# yayaweha 差很远好吧……他那种是先找出充分条件,再证明必要性(也就是最后面那部分证明)……绕过了求极限,但是很麻烦很难想。
yayaweha 12# 2013-2-12 20:29
解法二用了咯必答求极限
kuing 13# 2013-2-12 20:31
12# yayaweha 那个解法二我想应该不是标答来的,标答会出现洛必达?
yayaweha 14# 2013-2-12 20:32
是呀是标答,我们老师都没讲
yes94 15# 2013-2-12 23:36
本帖最后由 yes94 于 2013-2-12 23:37 编辑 就是泰勒公式: $\ln(1+x)=x-\dfrac{x^2}{2}+0(x^2)$,或者干脆$\ln(1+x)$ ~ $x-\dfrac{x^2}{2}$ 那么,$\begin{align*} \lim_{n\to+\infty}\left( \frac1{\ln (1+1/n)}-n \right)&=\lim_{x\to0^+}\left( \frac1{\ln (1+x)}-\frac1x \right) \\ & =\frac12, \end{align*}$
kuing 16# 2013-2-12 23:41
15# yes94 习惯用洛了,展开式用得少…… PS、 是 O(x) 不是 0(x) 吧; ~ 可以用 \sim ,就不用断开写了; 用环境可以两边不加 $ 就会居中显示
realnumber 17# 2013-2-12 23:42
考试时,那么点时间算算都不够~~
yes94 18# 2013-2-13 13:46
16# kuing 重新打试一试: 泰勒公式: $\ln(1+x)=x-\dfrac{x^2}{2}+O(x^2)$,或者干脆$\ln(1+x)\sim x-\dfrac{x^2}{2}$ 那么,\begin{align*} \lim_{n\to+\infty}\left( \frac1{\ln (1+1/n)}-n \right)&=\lim_{x\to0^+}\left( \frac1{\ln (1+x)}-\frac1x \right) \\ & =\frac12, \end{align*} 但是那个“代码基本输入首页”没看见\sim符号? 如果在证明,$\dfrac12$是一个上界的话就好了,即证$\dfrac12>\dfrac1{\ln (1+1/n)}-n$, 那么$\dfrac12$就是一个上确界。 PS:一眼可以看出$a\leqslant0$是不行的,因为$(1+\dfrac1n)^n<e$。 第一感觉是$a\geqslant0$,结果是错的!所以几秒钟猜的答案不可靠啊! 此题原来背景是$\ln(1+x)=x-\dfrac{x^2}{2}+O(x^2)$,如果再难点,还可以搞个$O(x^3)$出来!
kuing 19# 2013-2-13 13:51
18# yes94 ... 但是那个“代码基本输入首页”没看见\sim符号? ... yes94 发表于 2013-2-13 13:46 嗯,忘记加,去更新一下先,顺便想起全等也没弄……
yayaweha 20# 2013-2-13 18:30
泰勒公式对高考有帮助吗?
thread-1133-2-4.html:
yayaweha 21# 2013-2-13 18:56
这一道高三模拟题,整道题一句话泰勒公式就完了
yayaweha 22# 2013-2-13 18:57
LOOK
yayaweha 23# 2013-2-13 18:59
本帖最后由 yayaweha 于 2013-2-13 19:11 编辑 22# yayaweha 看看我这样做对不对? $f(x)=ln(x+1)+\sqrt{x+1}-1$令$t=\sqrt{x+1},t\in(1\sqrt3)$ 又$lnt< t-1,t\in(1\sqrt3)$ 有$f(x)=ln(x+1)+\sqrt{x+1}-1<3t-3$ 即证$$3t-3<\frac{9t^2-9}{t^2+5}\iff t^2+5<3t+3\iff t^2-3t+2<0\iff t\in(1,2)$$ 又因为$$t\in(1,\sqrt3)\subsetneqq(1,2)$$所以不等式得证
kuing 24# 2013-2-13 21:29
23# yayaweha 跟这个 http://www.pep.com.cn/rjwk/gzsxs ... 0121107_1143036.htm 的思路2的证法大概是一样的
yayaweha 25# 2013-2-13 21:36
本帖最后由 yayaweha 于 2013-2-13 21:41 编辑 我的即去了ln也去了根号。
第一章 26# 2013-2-15 06:43
21# yayaweha 这题像是2012年广州的模拟题
thread-1134-1-4.html: [数论] 请教一个完全平方数的问题
abababa 1# 2013-2-14 09:11
把前2012个自然数,就是1到2012,按任意顺序排成一串,组成一个新数(例如:把前3个自然数按任意顺序排成一串,可以是123,132,231,213,312,321),求证这个新数不是完全平方数。
realnumber 2# 2013-2-14 11:25
先发个百度来得一个帖子,来自这里 能表示为某个整数的平方的数称为完全平方数,简称平方数。 例如: 0,1,4,9,16,25,36,49,64,81,100,121,144,169,196,225,256,289, 324,361,400,441,484,…      观察这些完全平方数,可以获得对它们的个位数、十位数、数字和等的规律性的认识。 一、平方数有以下性质:   【性质1】完全平方数的末位数只能是0,1,4,5,6,9。   【性质2】奇数的平方的个位数字为奇数,十位数字为偶数。 【性质3】如果完全平方数的十位数字是奇数,则它的个位数字一定是6;反之,如果完全平方数的个位数字是6,则它的十位数字一定是奇数。 推论1:如果一个数的十位数字是奇数,而个位数字不是6,那么这个数一定不是完全平方数。 推论2:如果一个完全平方数的个位数字不是6,则它的十位数字是偶数。 【性质4】(1)凡个位数字是5,但末两位数字不是25的自然数不是完全平方数; (2)末尾只有奇数个“0”的自然数(不包括0本身)不是完全平方数;      100,10000,1000000是完全平方数, 10,1000,100000等则不是完全平方数。 (3)个位数字为1,4,9而十位数字为奇数的自然数不是完全平方数。 需要说明的是:个位数字为1,4,9而十位数字为奇数的自然数一定不是完全平方数,如:11,31,51,74,99,211,454,879等一定不是完全平方数一定不是完全平方数。 但个位数字为1,4,9而十位数字为偶数的自然数不都是完全平方数。如:21,44,89不是完全平方数,但49,64,81是完全平方数。 【性质5】偶数的平方是4的倍数;奇数的平方是4的倍数加1。 这是因为   (2k+1)^2=4k(k+1)+1                    (2k)^2=4k^2 【性质6】奇数的平方是8n+1型;偶数的平方为8n或8n+4型。 【性质7】平方数的形式一定是下列两种之一:3k,3k+1。【注意:具备以上条件的不一定是完全平方数(如13,21,24,28等)】 【性质8】不能被5整除的数的平方为5k±1型,能被5整除的数的平方为5k型。 【性质9】平方数的形式具有下列形式之一:16m,16m+1,16m+4,16m+9。 除了上面关于个位数,十位数和余数的性质之外,还可研究完全平方数各位数字之和。 例如,256它的各位数字相加为2+5+6=13,13叫做256的各位数字和。如果再把13的各位数字相加:1+3=4,4也可以叫做256的各位数字的和。 下面我们提到的一个数的各位数字之和是指把它的各位数字相加,如果得到的数字之和不是一位数,就把所得的数字再相加,直到成为一位数为止。 关于完全平方数的数字和有下面的性质: 【性质10】完全平方数的各位数字之和只能是0,1,4,7,9。 证明   因为一个整数被9除只能是 9k,9k±1, 9k±2, 9k±3, 9k±4这几种形式,而 (9k)^2=9(9k^2)+0 (9k±1)^2=9(9k^2±2k)+1 (9k±2)^2=9(9k^2±4k)+4 (9k±3)^2=9(9k^2±6k)+9 (9k±4)^2=9(9k^2±8k+1)+7 除了以上几条性质以外,还有下列重要性质: 【性质11】a^2b为完全平方数的充要条件是b为完全平方数。 【性质12】如果质数p能整除a,但p^2不能整除a,则a不是完全平方数。 证明   由题设可知,a有质因子p,但无因子p^2,可知a分解成标准式时,p的次方为1,而完全平方数分解成标准式时,各质因子的次方均为偶数,可见a不是完全平方数。 【性质13】在两个相邻的整数的平方数之间的所有整数都不是完全平方数,即 【性质14】一个正整数n是完全平方数的充分必要条件是n有奇数个因子(包括1和n本身)。 【性质15】完全平方数的约数个数是奇数个。约数的个数为奇数个的自然数是完全平方数。 【性质16】若质数p整除完全平方数a,则p^2|a。 【性质17】任何四个连续整数的乘积加1,必定是一个平方数。 二、重要结论(不是完全平方数的特点) 1.个位数是2,3,7,8的整数一定不是完全平方数;   2.个位数和十位数都是奇数的整数一定不是完全平方数; 3.个位数是6,十位数是偶数的整数一定不是完全平方数; 4.形如3n+2型的整数一定不是完全平方数; 5.形如4n+2和4n+3型的整数一定不是完全平方数; 6.形如5n±2型的整数一定不是完全平方数; 7.形如8n+2, 8n+3, 8n+5, 8n+6,8n+7型的整数一定不是完全平方数; 8.数字和是2,3,5,6,8的整数一定不是完全平方数
kuing 3# 2013-2-14 11:32
2# realnumber 性质10?
realnumber 4# 2013-2-14 11:39
3# kuing 有可能,=我把各位数字加起来
abababa 5# 2013-2-14 11:42
刚才也问了一位网友,他给了很简单的证明,已经懂了,呵呵。 记此数为$S$,各位数字之和为$M$,则$3 \mid M$,$9 \nmid M$,所以$3 \mid S$,$9 \nmid S$,即$S$中只有奇数个因子3,所以不是完全平方数。
kuing 6# 2013-2-14 11:43
4# realnumber 他那里显然是写错了的, 不过证法也给出了正确的结论。
kuing 7# 2013-2-14 11:45
5# abababa 不错, 我也懂了。
realnumber 8# 2013-2-14 11:53
本帖最后由 realnumber 于 2013-2-14 11:56 编辑 4# realnumber 个位数$1+2+\cdots+9\equiv0\mod9$,只余下2011,1012,两个的个位数和为3 十位数,只余下2010,2011,2012,三个的十位数和为3 百位数,没有余下,正好出现几边"1+2+\cdots+9" 千位数,1000~1999,1千个1,2000~2012,13个2 所以各位数和($\mod9$),$3+3+1000+26\equiv6$,按性质10的证明,所以不是完全平方数.这样应该也可以.
abababa 9# 2013-2-14 11:55
8# realnumber 呵呵,这样就麻烦了,他又给我说了一下,各位数字和能被3整除,等价于$1+2+...+2012$能被3整除,这么就简单了。
realnumber 10# 2013-2-14 11:58
9# abababa 恩,这样好,
realnumber 11# 2013-2-14 12:00
本帖最后由 realnumber 于 2013-2-14 12:01 编辑 10# realnumber 也各有用处,万一这个不行的话,试另外一个.当然简单办法排在前面.
realnumber 12# 2013-2-14 12:14
提问:1,2,3,4,5,6,7,8,9任意排列成一个9位数.能否组成一个完全平方数?---先这个开始吧,解决后,看看能否用更"刁"的条件.
abababa 13# 2013-2-14 12:37
12# realnumber 呵呵,我不会,又问了网友,原来竟然有很多啊,存在又不唯一,感觉方法好像不好弄 他给了两个例子$11826^2=139854276$,$12363^2=152843769$
realnumber 14# 2013-2-14 12:45
13# abababa 我也不会,本来希望不是,;不过你朋友好牛,要不请他提些,后续的问题?
abababa 15# 2013-2-14 13:07
14# realnumber 到是提了一个,不过他自己也没解决 将1到n这n个自然数任意排成一列组成一个新数,n为何值时这些排列中至少有一个完全平方数,是不是存在简单的函数表达式
yes94 16# 2013-2-14 13:48
知道答案就简单了, 原来就是历史上的“弃九法”
kuing 17# 2013-2-14 15:04
16# yes94 第一次听……
thread-1135-1-2.html: 自我感言
童光红 1# 2013-2-14 10:32
本帖最后由 童光红 于 2013-2-14 10:33 编辑 读高二上学期辍学,现在深感【书到用时方恨少】啊 问好各位老师!
kuing 2# 2013-2-14 11:49
现在还要用【书】??
kuing 3# 2013-2-14 12:20
对了,为什么会有一个空图片?
thread-1136-1-4.html: [不等式] 来自粉丝群的类似Schur
kuing 1# 2013-2-14 22:17
天书(1846******)  21:08:08 a,b,c>=0,a+b+c<=1,证a+b+c+9abc>=4(ab+bc+ca) 固定 $b$, $c$,记 $f(a)=a+b+c+9abc-4(ab+bc+ca)$,注意到这是关于 $a$ 的一次函数,所以只要证明 $a$ 取最大及最小时都成立即可。 由条件知 $a$ 取最大即 $a+b+c=1$,齐次化后亦即是 Schur 不等式,成立; 而 $a$ 取最小即取到 $0$,此时 $f(0)=b+c-4bc\geqslant (b+c)^2-4bc=(b-c)^2\geqslant0$,也成立。
kuing 2# 2013-2-15 01:10
在相同的条件下,还有更强式 \[\sqrt[3]{(a+b+c)^5}+9abc\geqslant 4(ab+bc+ca).\] 证:为方便书写,记 $p=a+b+c$, $q=ab+bc+ca$,则由 Schur 不等式 $9abc\geqslant4pq-p^3$ 可知只需证 \[p^{5/3}+4pq-p^3\geqslant 4q,\]即\[p^{5/3}-p^3\geqslant 4q(1-p),\] 由条件 $p\leqslant 1$ 及均值不等式 $p^2\geqslant 3q$ 可知只需证 \[p^{5/3}-p^3\geqslant \frac43p^2(1-p),\] 作差分解为 \[\frac13p^{5/3}(p^{1/3}-1)^2(p^{2/3}+2p^{1/3}+3)\geqslant 0,\] 显然成立。 猜测:5/3 是最佳指数。 待续,时间关系明天再玩……
realnumber 3# 2013-2-15 09:34
介绍Schur不等式的 http://www.mathoe.com/dispbbs.asp?boardID=106&ID=35942
yes94 4# 2013-2-15 11:49
本帖最后由 yes94 于 2013-2-15 12:51 编辑 这次不打代码了,用附件(输入错误已修改): 不晓的对不?
kuing 5# 2013-2-15 11:58
4# yes94 还是尽量建议用代码吧,顺便改一下输入错误。
yes94 6# 2013-2-15 12:22
本帖最后由 yes94 于 2013-2-15 13:12 编辑 由4楼可以看出,
yes94 7# 2013-2-15 13:26
在相同的条件下,还有更强式 \[\sqrt[3]{(a+b+c)^5}+9abc\geqslant 4(ab+bc+ca).\] 证:为方便书写,记 $p=a+b+c$, $q=ab+bc+ca$,则由 Schur 不等式 $9abc\geqslant4pq-p^3$ 可知只需证 \[p^{5/3}+4pq-p^3\geq ... kuing 发表于 2013-2-15 01:10
kuing 8# 2013-2-15 14:28
7# yes94 不错,因为仅用了一次均值将两个元进行了调整,所以那种形式很强。 有一点点小可惜的是那种加强式没有了Schur不等式的原貌(这其实也是我直接往指数加强方面思考而没考虑添加其他项的原因,平时我是不太敢玩指数的,因为通常不易玩)
yes94 9# 2013-2-15 14:34
8# kuing 还是令$t=1$最好, Schur也存在,式子也更美观。
thread-1137-1-4.html: [不等式] $x+y+z=1$,求证:$\sum\dfrac{xy}{\sqrt{xy+yz}}\leqslant\dfrac{\sqrt2}2$
yes94 1# 2013-2-15 09:51
本帖最后由 yes94 于 2013-2-15 11:52 编辑 已知$x、y、z\in R^+$,且$x+y+z=1$,求证:$\sum\dfrac{xy}{\sqrt{xy+yz}}\leqslant\dfrac{\sqrt2}2$
kuing 2# 2013-2-15 11:42
http://bbs.pep.com.cn/forum.php?mod=viewthread&tid=502645
kuing 3# 2013-2-15 11:51
刚才论坛出现了 bug... 竟然我的回贴顶上了一楼……
yes94 4# 2013-2-15 11:53
3# kuing 我幸好抓拍了这个现象,要不然还真难理解呢!
pxchg1200 5# 2013-2-15 12:35

kuing 6# 2013-2-15 12:37
5# pxchg1200 what happen?
realnumber 7# 2013-2-15 12:43
6# kuing 你得解决他最近的2个帖子,我猜是这样,---
kuing 8# 2013-2-15 13:17
7# realnumber 太难,不会……
pxchg1200 9# 2013-2-15 17:54
本帖最后由 pxchg1200 于 2013-2-15 17:59 编辑 6# kuing There exist a prefect proof ! also,the followsing stronger inequality holds.
kuing 10# 2013-2-15 18:32
9# pxchg1200 oh,原来是证那个加强式,晚点再看,先煮饭喔
kuing 11# 2013-2-15 18:55
9# pxchg1200 中间那行字的是不是“由轮换对称性不妨设 $y$ 在 $x$, $z$ 之间”之类的意思?
kuing 12# 2013-2-15 19:05
的确 very nice!
yes94 13# 2013-2-15 19:26
9# pxchg1200 中间那行字的是不是“由轮换对称性不妨设 $y$ 在 $x$, $z$ 之间”之类的意思? kuing 发表于 2013-2-15 18:55 你会粤语? 错了,你会越语?
kuing 14# 2013-2-15 19:54
13# yes94 我猜的,应该是那种意思,才得到后面那个不等式
yes94 15# 2013-2-15 20:37
据说是蔡玉书先生在《数学通讯》(教师刊)2010年底4期的证法: 2006CMO
thread-1138-1-4.html: [不等式] $x+y+z=1$,求证:$\sum\dfrac{x}{\sqrt{y+z}}\geqslant\dfrac{\sqrt6}2$
yes94 1# 2013-2-15 10:06
已知$x、y、z\in R^+$,且$x+y+z=1$,求证:$\sum\dfrac{x}{\sqrt{y+z}}\geqslant\dfrac{\sqrt6}2$
realnumber 2# 2013-2-15 10:41
1# yes94 $f(x)=\frac{x}{\sqrt{1-x}}$是下凸函数,用琴生不等式或$x=\frac{1}{3}$处切线法都可以的,也许cauchy不等式也行.
yes94 3# 2013-2-15 11:56
2# realnumber 不错,切线法可以搞:
pxchg1200 4# 2013-2-15 18:02
There also  exist a proof with Holder inequality :D
yes94 5# 2013-2-15 18:39
There also  exist a proof with Holder inequality :D pxchg1200 发表于 2013-2-15 18:02 那我来试一试: 是不是这个?如果不是请给出解答,谢谢!
pxchg1200 6# 2013-2-20 19:01
5# yes94 There are many ways to apply Holder inequality :D \[ \left(\sum{\frac{x}{\sqrt{y+z}}} \right)^{2}\left(\sum{x(y+z)} \right)\geq (x+y+z)^{3} \] The result is easy :D
thread-1139-1-4.html: [不等式] 设$a,b,c\in R_+且abc=1,求证:\sum\frac{1}{1+a+b}\le 1$
yayaweha 1# 2013-2-15 13:55
本帖最后由 yayaweha 于 2013-2-15 13:58 编辑 设$a,b,c\in R_+且abc=1,求证:\frac{1}{1+a+b}+\frac{1}{1+b+c}+\frac{1}{1+c+a}\le 1$
kuing 2# 2013-2-15 14:18
http://bbs.pep.com.cn/forum.php?mod=viewthread&tid=511270
yes94 3# 2013-2-15 14:37
2# kuing 那个鱼儿从此就消失了! 两种很好的换元方法,构造的局部不等式都非常赏心悦目!
kuing 4# 2013-2-15 14:48
3# yes94 可能到别的地方玩去了……又或者要忙自己的事,没空玩了
yes94 5# 2013-2-15 21:12
把那边的解法贴过来(by kuing): 设$x,y,z$是正数,令 $a=\dfrac{x^2}{yz}$,$b=\dfrac{y^2}{zx}$, $c=\dfrac{z^2}{xy}$ 则$\dfrac1{a+b+1}=\dfrac{xyz}{x^3+y^3+xyz}\leqslant\dfrac{xyz}{x^2y+xy^2+xyz}=\dfrac z{x+y+z}$ 同理得,$\dfrac1{b+c+1}\leqslant\dfrac x{x+y+z}$, $\dfrac1{c+a+1}\leqslant\dfrac y{x+y+z}$ 相加即得证。
yes94 6# 2013-2-15 21:18
(By 鱼儿) 令$a=x^3$,$b=y^3$,$c=z^3$,其中$x,y,z>0$,则$xyz=1$, 由排序不等式得 $\dfrac1{a+b+1}=\dfrac1{x^3+y^3+1}\leqslant\dfrac1{x^2y+xy^2+1}=\dfrac z{x+y+z}$, 同理可得 $\dfrac1{b+c+1}\leqslant\dfrac x{x+y+z}$,$\dfrac1{c+a+1}\leqslant\dfrac y{x+y+z}$, 上述三式叠加即得 $\dfrac1{a+b+1}+\dfrac1{b+c+1}+\dfrac1{c+a+1}\leqslant1$.
kuing 7# 2013-2-15 22:37
相关的,或者说是更强的还有这个 http://bbs.pep.com.cn/forum.php?mod=viewthread&tid=396030 一个牛比的证明可参考pxchg的blog的这篇 http://pxchg1200.is-programmer.com/posts/37415.html
yes94 8# 2013-2-16 13:13
本帖最后由 yes94 于 2013-2-16 13:17 编辑 7# kuing 这种放缩成指数局部不等式好像看过多次了,只是极难想到啊!每看到一次除了赞叹也只有赞叹! 共同特点都是,放缩后相加后为 $1$! $\dfrac{a}{a+2}=\dfrac{a}{a+2(abc)^{\frac{1}{3}}}\geqslant \dfrac{a^{\frac{2}{3}}}{a^{\frac{2}{3}}+b^{\frac{2}{3}}+c^{\frac{2}{3}}}$ 于是,$\dfrac{a}{a+2}+\dfrac{b}{b+2}+\dfrac{c}{c+2}\geqslant1$
yayaweha 9# 2013-2-16 13:32
类似的
yes94 10# 2013-2-16 14:02
9# yayaweha 不错!
kuing 11# 2013-2-16 14:12
8# yes94 最牛的地方是在第一步,后面那种齐次构造算是常规方法之一了,那些指数可以通过待定获得。
yes94 12# 2013-2-16 14:18
11# kuing 嗯,这个的确非常难想到:$\dfrac{2}{a+2}-\dfrac{b}{ab+b+1}-\dfrac{1}{a+b+1}=\dfrac{a(b-1)^2}{(a+2)(ab+b+1)(a+b+1)}\geqslant0$ 这个还算看过:$\sum{\dfrac{b}{ab+b+1}}=1$
thread-114-1-1.html: mathematica用极限要注意某些细节
kuing 1# 2011-10-18 17:04
In[75]:= Limit[Abs[x]/x, x -> 1] Out[75]= 1 In[76]:= Limit[Abs[x]/x, x -> -1] Out[76]= -1 In[77]:= Limit[Abs[x]/x, x -> Infinity] Out[77]= 1 In[78]:= Limit[Abs[x]/x, x -> -Infinity] Out[78]= -1 In[79]:= Limit[Abs[x]/x, x -> 0] Out[79]= 1 In[80]:= Limit[Abs[x]/x, x -> -0] Out[80]= 1 In[81]:= Limit[Abs[x]/x, x -> 0, Direction -> 1] Out[81]= -1 说明: Abs[x] 是绝对值 Infinity 为无穷,是正无穷大的 本身 $\lim_{x\to0}|x|/x$ 的极限不存在,但 Limit[Abs[x]/x, x -> 0] 时当成了 $x\to0+$,所以输出 $-1$,然而,我们却不能用 x -> -0 来表示 $x\to0-$,用 x -> 0- 更会被认为是错误输入,要在后面用 Direction -> 1 才能表示左极限,相应地,Direction -> -1 是右极限。 注:以上测试在 Mathematica7 里进行。
kuing 2# 2011-10-18 17:08
猜猜以下这些会输出什么? Limit[Sin[x], x -> Infinity] Limit[Log[-x^2], x -> Infinity] Limit[Sqrt[-x^2], x -> Infinity]
①②③④⑤⑥⑦ 3# 2011-10-18 17:31
本帖最后由 ①②③④⑤⑥⑦ 于 2011-10-19 09:17 编辑 1# kuing 呃,在复数范围内工作的,$z\infty$ 就是 $z$ 方向的无穷大。不确定的无穷是 ComplexInfinity  。 DirectedInfinity[]函数,空参数就是 ComplexInfinity,DirectedInfinity[z] 就是 $z\infty$
kuing 4# 2011-10-18 17:36
3# ①②③④⑤⑥⑦ 怎样的情况下会出现 Indeterminate?
①②③④⑤⑥⑦ 5# 2011-10-19 09:16
4# kuing 弄错了,应该不会,Indeterminate似乎是计算中才会出现, Limit的结果,会有各种方向的无穷大,还有用上Inteval的 In[16]:= Limit[Exp[-(-2 + 2 I)/x], x -> 0] Out[16]= ComplexInfinity In[17]:= 0 Limit[Exp[-(-2 + 2 I)/x], x -> 0] During evaluation of In[17]:= Infinity::indet: Indeterminate expression 0 ComplexInfinity encountered. >> Out[17]= Indeterminate In[19]:= Limit[(-1)^x, x -> \[Infinity]] Out[19]= E^(2 I Interval[{0, \[Pi]}])
kuing 6# 2011-10-19 12:08
5# ①②③④⑤⑥⑦ 呃 0 Limit...... 还能这样玩。。。
kuing 7# 2012-1-9 23:32
原来三年前在人教就有人扯过,我也在场哩……不过没什么印象了都 http://bbs.pep.com.cn/thread-433336-1-1.html
thread-1140-1-1.html: 请问一个连加的写法
abababa 1# 2013-2-15 14:01
请问$x_1x_2x_3+x_2x_3x_4+...+x_{n-1}x_nx_1+x_nx_1x_2$这样的连加,有没有$\sum$这样的符号表示?要怎么写呢?
kuing 2# 2013-2-15 14:15
这种情况通常处理方法是另记 $x_{n+1}=x_1$, $x_{n+2}=x_2$,然后就可以用 $\displaystyle\sum_{k=1}^nx_kx_{k+1}x_{k+2}$ 表示它
abababa 3# 2013-2-20 19:51
2# kuing 谢谢。还以为有什么简单的表示法表示这类轮换的呢。
thread-1141-1-2.html: KK好勇呀!
yayaweha 1# 2013-2-15 17:28
look
yayaweha 2# 2013-2-15 17:28
KK当时最后一题做得怎么样?
kuing 3# 2013-2-15 17:31
2# yayaweha 结果是不太会…… 其实不是勇,反正都只是回去玩玩,不玩白不玩的……
yayaweha 4# 2013-2-15 17:32
07年的题?
kuing 5# 2013-2-15 17:35
4# yayaweha 嗯,我是2007毕的业。 最后那个好像是数列吧,是个二次分式的,当年还没怎么玩过……
yayaweha 6# 2013-2-15 17:38
this?
kuing 7# 2013-2-15 17:41
6# yayaweha 记不清了,反正只记得是会遇到二次分式的递推数列
yayaweha 8# 2013-2-15 17:43
你有没有想痛扁那张卷的感觉
kuing 9# 2013-2-15 17:46
8# yayaweha 没有,何必呢?要是还有这等火气就不会回去考了,明知考不上的,纯粹只是玩玩而已,反正不考白不考,算是给高中一个完结的句号吧。
kuing 10# 2013-2-15 18:33
讲开又讲,其实我高二后期就不想上学了,之所以读到高三,是因为当时还打算参加那个高中数学联赛试试看,结果也是不太会,那时还是很菜(其实现在也不一定行)。 之后直到高三下学期才没上学(为什么还要拖到下学期?其实是因为某妞,不过最终还是没泡到哎……),但是虽然没上学,大部分学科没及格过,最后竟然还毕了业,也能参考高考,所以就……
yes94 11# 2013-2-17 17:17
10# kuing 奇才啊!
╰☆ヾo.海x 12# 2013-2-17 17:56
10# kuing 咳咳。。大新闻啊!!!!!!哈哈原来酷儿有这样的经历。。其实大部分了解了,没想到还有泡妞那段。。
kuing 13# 2013-2-17 18:56
12# ╰☆ヾo.海x 旧闻了啊…… 都没泡到,所以往常就没提直接带过了……
isea 14# 2013-2-18 23:12
玩不等式那么“爽”的人不是奇人就奇了怪了!
kuing 15# 2013-2-18 23:21
有代价的……
thread-1142-1-1.html: [几何] 已知2平面,求交线方程
╰☆ヾo.海x 1# 2013-2-16 11:24
请问第三问怎么做?
kuing 2# 2013-2-16 12:21
暂时想到两种方法 第一种是求出该直线上两个点,比如分别令 x=0 及 y=0 之类解方程组就可以得到两点,然后用两点式方程; 第二种是求出该直线上一个点,然后用两平面法向量的叉乘求出直线的方向向量,然后就可以写出方程。
kuing 3# 2013-2-16 12:32
鉴于理论上还是初等内容,还是移过来这边了。 刷牙去了
╰☆ヾo.海x 4# 2013-2-16 12:44
3# kuing 谢啦
yes94 5# 2013-2-16 12:48
这里的$\overrightarrow{r}$,指的是$\overrightarrow{r}=(x,y,z)$? $\overrightarrow{i}$,$\overrightarrow{j}$,$\overrightarrow{k}$指的是空间直角坐标系$O-xyz$的单位方向向量?
╰☆ヾo.海x 6# 2013-2-16 12:50
2# kuing 为什么两平面法向量的叉乘就是直线的方向向量啊。。
╰☆ヾo.海x 7# 2013-2-16 12:51
5# yes94 恩
yes94 8# 2013-2-16 12:57
6# ╰☆ヾo.海x 叉乘的定义啊?代kuing回答 也可以将已知两个方程的右边的$1$和$8$都改成$0$,然后解这两个方程联立而成的方程组(可以令$x=1$,或者$y=1$,也可$z=1$),得到的解$(x,y,z)$就是交线的方向向量。
╰☆ヾo.海x 9# 2013-2-16 12:58
2# kuing 为什么两平面法向量的叉乘就是直线的方向向量啊。。 ╰☆ヾo.海x 发表于 2013-2-16 12:50 额我知道了。。。我怎么没意识到。。哎。。 叉乘的2个向量不一定要共起点哇,任意2个向量都可以叉乘啊? 哎我傻了
╰☆ヾo.海x 10# 2013-2-16 12:59
8# yes94 恩。。我当时就用的这个方法 呵呵
yes94 11# 2013-2-16 13:08
10# ╰☆ヾo.海x 这是哪来的题啊?竟然是外文的!
╰☆ヾo.海x 12# 2013-2-16 13:13
11# yes94 额,我们考试的题目。。我在外国。。哎现在水平超烂,必须加紧学习了。。
╰☆ヾo.海x 13# 2013-2-16 13:15
不是, 我再确认下。。 是不是任意2个向量都可以进行叉乘?
kuing 14# 2013-2-16 14:15
13# ╰☆ヾo.海x 是
kuing 15# 2013-2-16 14:20
话说,1#图中那个 i j k 上面那个 ^ 我竟然是不知道怎么打出来的
kuing 16# 2013-2-16 14:27
噢,想起来了,学线性回归方程那时候出现过这个 ^ ,那时还有人在论坛问过怎么念,记得有人说 hat,所以代码估计就是 \hat{...},试试 $\hat{i}$
yes94 17# 2013-2-16 14:32
16# kuing 我还以为是箭头$\vv i$呢,结果是$\hat i$,
realnumber 18# 2013-2-16 15:44
1楼配上中文就更好了,虽然百度可以查e文单词的,...
kuing 19# 2013-2-16 16:04
18# realnumber 其实也不用看那些英文,看标题,再看那几个公式,已经了解题意了……
海盗船长 20# 2013-2-16 22:04
方法2好用些
thread-1142-2-1.html:
╰☆ヾo.海x 21# 2013-5-27 00:39
还是不懂。。为啥两平面的交线满足任意x。。。。。。。
kuing 22# 2013-5-27 01:29
21# ╰☆ヾo.海x 看不懂你的不懂。。。
thread-1143-1-3.html: [不等式] 陈题征新解:已知$4x^2+5y^2=y$,求$x^2+y^2$的最大值。
yes94 1# 2013-2-17 14:25
陈题征新解, 三角代换就免了,数形结合也免了,消元法也免了。 不过,如果你愿意,以上这些方法也可以写出来的。 希望有使用 A-G不等式,柯西,甚至排序不等式的解答,等等。 已知$4x^2+5y^2=y$,求$x^2+y^2$的最大值。
yes94 2# 2013-2-21 14:39
1# yes94 我错了! 现在不加任何限制,此题该如何做? 回复量为0太不好了!
realnumber 3# 2013-2-21 20:46
2# yes94 没新想法,你不是会了上面几种的啊,刻意改成别的,似乎也不好改,不过也没动力去改.
yes94 4# 2013-2-21 21:29
3# realnumber 这是某网站的某同学的帖子,我转发而已,他只要求不用三角代换。
realnumber 5# 2013-2-23 00:14
其实你1楼说的消元法挺好的,正好强调注意2个变量x和y的取值范围.
yes94 6# 2013-2-24 20:53
5# realnumber 对, 转了一个简单的,却没得好几个人回复
kuing 7# 2013-2-24 20:58
没什么new idea,不知回什么好嘛……
yes94 8# 2013-2-25 21:41
本帖最后由 yes94 于 2013-2-25 21:45 编辑 我还是搞一个用纯粹不等式的解答吧,给这道题给一个结果:    由均值不等式可知,$\dfrac52y^2+\dfrac1{10}\geqslant y$,故$\dfrac52y^2\geqslant y-\dfrac1{10}$,当且仅当$y=\dfrac15$取等号。于是, $y=4x^2+5y^2=\dfrac52(x^2+y^2)+\dfrac32x^2+\dfrac52y^2\geqslant \dfrac52(x^2+y^2)+\dfrac52y^2\geqslant \dfrac52(x^2+y^2)+y-\dfrac1{10}$,     所以,$\dfrac1{10}\geqslant \dfrac52(x^2+y^2)$,即:$x^2+y^2\leqslant \dfrac1{25}$,当且仅当$x=0$,$y=\dfrac15$取等号。    于是$[x^2+y^2]_{\max}$=$\dfrac1{25}$。
yes94 9# 2013-3-4 10:43
8# yes94 再搞一种纯粹不等式的解答: $0=4x^2+5y^2-y=\dfrac52(x^2+y^2)+\dfrac32x^2+\dfrac52y^2-y=\dfrac52(x^2+y^2)+\dfrac32x^2+\dfrac52(y-\dfrac15)^2-\dfrac1{10}\geqslant\dfrac52(x^2+y^2)-\dfrac1{10}$, 即$x^2+y^2\leqslant\dfrac1{25}$,当且仅当$\dfrac32x^2=0$,$y-\dfrac15=0$取等号。
thread-1144-1-4.html: [几何] 圆
╰☆ヾo.海x 1# 2013-2-17 17:48
已知圆M: x^2+(y-2)^2=1, 点Q为x轴上一动点,过点Q作圆M的切线,切点为A,B. 1) 当|AB|=4(根号2)/3时,求MQ的方程; 2) 求动弦AB的中点P的轨迹方程。 额,还没研究代码。。。就将就看拉。。图也木有。。。
kuing 2# 2013-2-17 17:53
貌似反演……
╰☆ヾo.海x 3# 2013-2-17 17:54
2# kuing 。。。能不能再多几个字的解释。。
kuing 4# 2013-2-17 18:15
只做第二问啦: 下面证明 $C$ 是个定点,这是因为 \[MC\cdot MD=MP\cdot MQ=r^2,\] 然后就……
yes94 5# 2013-2-17 18:44
4# kuing 然后 $P$点就在以 $MC$为直径的圆上啦! (1)答案:$MQ$:$\dfrac{x}{\sqrt5}+\dfrac{y}{2}=1$ (2)答案:$x^2+(y-\dfrac74)^2=\dfrac1{16}$,有范围,在圆$M$:$x^2+(y-2)^2=1$的内部吧? 对不对?
kuing 6# 2013-2-17 18:45
取不了M而已
yes94 7# 2013-2-17 19:09
6# kuing     对,要挖掉$M$点。是不是可以这样认为: 由于$QA^2=QP\cdot QM$,故可以看做点$Q$对两圆的幂相等 (先默认$P$点、$M$点在某圆上,这个是不是有点说不过去了哈,嘻嘻),    于是,点$Q$在两圆的等幂轴上,由于点$Q$的轨迹为$x$轴即$y=0$,圆$M$:$ x^2+(y-2)^2-1=0$,即$x^2+y^2-4y+3=0$,于是$P$点在圆$x^2+y^2-\lambda y+3=0$上,代入$M$点坐标$(0,2)$,得到$\lambda=\dfrac72$,    所以,$P$点轨迹为$x^2+y^2-\dfrac72y+3=0$,    但是很无情的是,最后还是要去掉$M$点。
yes94 8# 2013-2-17 20:03
其实,最开始还是这样做的:   设$Q(t,0)$,则$P$点在$Q$的极线$AB$上:$tx-2(y-2)=1$   显然$P$点在直线$MQ$上:$\dfrac xt+\dfrac y2=1$上, 以上两式,消去$t$得:$P$点轨迹为$x^2+y^2−\frac72y+3=0$
kuing 9# 2013-2-17 21:56
8# yes94 还是别说极线了,海叉会晕的。 写出以 $MQ$ 为直径的方程,那么 $AB$ 为其公共弦,于是两圆相减即得 $AB$ 方程
thread-1145-1-4.html: 三角函数
╰☆ヾo.海x 1# 2013-2-17 17:49
本帖最后由 ╰☆ヾo.海x 于 2013-2-17 17:52 编辑 嘿嘿,发这里,谁高兴做的话要写下过程啊。。。或者指点下。。
yes94 2# 2013-2-17 18:13
1# ╰☆ヾo.海x (i)积分就可以啦!$y=sinx,x\in [0,\dfrac{\pi}{2}]$在直线$y=x$的下方,并且用内接三角形面积放缩。 老外的题还真有意思呢 (ii)还没看。
kuing 3# 2013-2-17 18:18
2# yes94 据说前半部分都是提示我一开始还没看懂呢…… PS、sin -> \sin ,公式里还是用半角逗号吧
海盗船长 4# 2013-2-17 18:23
第二个类似,也是积分考虑面积
海盗船长 5# 2013-2-17 18:23
$y=a^x$从0积到1
kuing 6# 2013-2-17 18:32
第2题其实常规求导也很好做
yes94 7# 2013-2-17 18:46
6# kuing 如果没有提示,一般都是先移项,再常规求导做,
╰☆ヾo.海x 8# 2013-2-19 01:51
怎么经常碰到积分。。哎,我这个坎还没起跨呢。。。
thread-1146-1-4.html: [不等式] 又一个三角不等式
ccnu_chb_ycb 1# 2013-2-17 19:54
请教这个三角不等式如何证明,若有兴趣也可以将类似的三角不等式问题进行接龙,谢谢
kuing 2# 2013-2-17 19:58
弱于这个结果:http://kkkkuingggg.5d6d.net/thread-1020-1-1.html
ccnu_chb_ycb 3# 2013-2-17 20:03
2# kuing 这个结果我用嵌入不等式证过,但是如何直接证明这个不等式?
kuing 4# 2013-2-17 20:07
也行,易证 \[\frac{\cos A\cos B}{\cos C}=\frac{\tan C}{\tan A+\tan B},\] 然后略。
ccnu_chb_ycb 5# 2013-2-17 20:14
这样的补充也许更容易理解些,还有其他的证法吗?其实以前记得也遇到过很多类似的问题,能否接龙一下,哈哈 4# kuing
ccnu_chb_ycb 6# 2013-2-17 20:23

kuing 7# 2013-2-17 20:52
ccnu_chb_ycb 发表于 2013-2-17 20:23 第一个错了,事实上可以证明对任意 $\triangle ABC$ 有 \[\sum\frac{\sin^2A\cos^2B}{\sin^2C}\geqslant\frac34.\] 用正弦、余弦定理及柯西不等式,有 \[\sum\frac{\sin^2A\cos^2B}{\sin^2C}=\sum\frac{(c^2+a^2-b^2)^2}{4c^4}\geqslant\frac{\left(\sum a^2(c^2+a^2-b^2)\right)^2}{4\sum c^4a^4} =\frac{\left(\sum a^4\right)^2}{4\sum c^4a^4}\geqslant\frac34.\]
kuing 8# 2013-2-17 21:08
第二和第三个都比较简单,展开 $\cos(B-C)$,除一下,利用三角恒等式 $\sum\cot A\cot B=1$,均易证。
ccnu_chb_ycb 9# 2013-2-17 21:16

kuing 10# 2013-2-17 21:25
9# ccnu_chb_ycb oh,又变成这类问题,没记错的话,这个对 $k\leqslant0$ 及 $k\geqslant\log_23$ 成立
kuing 11# 2013-2-17 21:47
$\sum(a/(b+c))^k$ 及 $\sum(a/(a+b))^k$ 问题的分界线都和 $\log_23$ 有关,有点神奇。 上次其中一个的链接给你发过了(记得否?) http://bbs.pep.com.cn/forum.php?mod=viewthread&tid=250615 这次的也是差不多的一个链接 http://bbs.pep.com.cn/forum.php?mod=viewthread&tid=251786 都是用了半凹半凸定理。
ccnu_chb_ycb 12# 2013-2-17 22:06
哈哈,这个必须记得的嘛,我再次好好的学习下这类问题
thread-1147-1-4.html: 三角函数证明
╰☆ヾo.海x 1# 2013-2-18 06:19
哎,第二步到第三步怎么来的?第三步到第四步左边那个2怎么没了。。然后第五步到第六步也不理解了。。你看第七步到第8步是。。为什么把2直接改成3啊。。 这个题好像就是我以前问过酷儿的。。 _________kuing edit in $\LaTeX$_________ 已知在 $\triangle ABC$ 中,三边 $a$, $b$, $c$ 所对的角分别是 $\angle A$, $\angle B$, $\angle C$,且 $a+c=2b$,求证 $\tan\frac A2\tan\frac C2=\frac13$。 证明: \begin{align*} & \because a+c=2b \\ & \therefore \sin A+\sin C=2\sin B\\ & \therefore 2\sin\left(\frac A2+\frac C2\right)\cos\left(\frac A2-\frac C2\right)=2\sin B\\ & \therefore \cos\frac B2\cos\left(\frac A2-\frac C2\right)=2\sin\frac B2\cos\frac B2\\ & \therefore \cos\left(\frac A2-\frac C2\right)=2\sin\frac B2\\ & \therefore \cos\left(\frac A2-\frac C2\right)-\cos\left(\frac A2+\frac C2\right)=\cos\left(\frac A2+\frac C2\right)\\ & \therefore 2\sin\frac A2\sin\frac C2=\cos\left(\frac A2+\frac C2\right)\\ & \therefore 3\sin\frac A2\sin\frac C2=\cos\left(\frac A2+\frac C2\right)\\ & \therefore 3\sin\frac A2\sin\frac C2=\cos\frac A2\cos\frac C2\\ & \therefore \tan\frac A2\tan\frac C2=\frac13. \end{align*}
kuing 2# 2013-2-19 13:05
\(\require{cancel}\) \begin{align*} & \because a+c=2b \\ & \therefore \sin A+\sin C=2\sin B\\ & \therefore 2\sin\left(\frac A2+\frac C2\right)\cos\left(\frac A2-\frac C2\right)=2\sin B \text{(和差化积)}\\ & \therefore \cos\frac B2\cos\left(\frac A2-\frac C2\right)=2\sin\frac B2\cos\frac B2 \text{(右边两倍角公式有个2约了左边的2)}\\ & \therefore \cos\left(\frac A2-\frac C2\right)=2\sin\frac B2\\ & \therefore \cos\left(\frac A2-\frac C2\right)-\cos\left(\frac A2+\frac C2\right)=\cos\left(\frac A2+\frac C2\right)\\ & \therefore 2\sin\frac A2\sin\frac C2=\cos\left(\frac A2+\frac C2\right) \text{(展开)}\\ & \xcancel{\therefore 3\sin\frac A2\sin\frac C2=\cos\left(\frac A2+\frac C2\right)}\\ & \therefore 3\sin\frac A2\sin\frac C2=\cos\frac A2\cos\frac C2 \text{(展开)}\\ & \therefore \tan\frac A2\tan\frac C2=\frac13. \end{align*}
╰☆ヾo.海x 3# 2013-2-19 14:28
2# kuing 酷儿你太可爱了哈哈哈 这么批注跟老师一样。。
kuing 4# 2013-2-19 14:53
3# ╰☆ヾo.海x 嘿嘿,看懂了吧?
yes94 5# 2013-2-19 14:54
4# kuing 很疑惑,那个大叉叉的代码是什么?
kuing 6# 2013-2-19 15:04
5# yes94 要加载"宏包", 现在爪机, 电脑再说
kuing 7# 2013-2-19 23:46
4# kuing 很疑惑,那个大叉叉的代码是什么? yes94 发表于 2013-2-19 14:54 在真正的 LaTeX 里,需要加载 cancel 宏包,然后就可以用 \cancel, \bcancel, \xcancel 以及 \cancelto 命令来实现这些效果,详细见 http://blog.sina.com.cn/s/blog_5e16f1770100ktrn.html 而在这里,MathJax 也提供了具有相同功能的拓展 cancel.js ,在需要用的时候先加载它,然后使用方法也一样。注意这里加载拓展的命令与 LaTeX 里不同,是 $\verb"\(\require{cancel}\)"$,放在同一页面的随便一个地方均可。
╰☆ヾo.海x 8# 2013-2-20 10:10
4# kuing 恩,明白了。。
yes94 9# 2013-2-20 14:20
7# kuing 看了一下大致明白了一些,就是还需要花大力气搞懂, 还是会最基本的就算了吧
thread-1148-1-1.html: bottema2009里面的三元轮换求和sgm
kuing 1# 2013-2-18 14:24
bottema2009 里定义了 sgm 函数,这是纯粹为了方便输入的,比如输入 sgm(a^2*b) 得到的是 a^2*b+b^2*c+c^2*a。 查了一下源代码,是这样定义的: sgm:=proc(expr)    local rap,ex2,ex3,ex:        rap:={a=b,b=c,c=a,A=B,B=C,C=A,x=y,y=z,z=x,ha=hb,hb=hc,hc=ha,Ra=Rb,Rb=Rc,Rc=Ra,          ra=rb,rb=rc,rc=ra,ma=mb,mb=mc,mc=ma,wa=wb,wb=wc,wc=wa,ka=kb,kb=kc,kc=ka,          HA=HB,HB=HC,HC=HA,IA=IB,IB=IC,          IC=IA,Ha=Hb,Hb=Hc,Hc=Ha,A=B,B=C,C=A,Ra=Rb,Rb=Rc,Rc=Ra,GA=GB,GB=GC,          GC=GA,JA=JB,JB=JC,JC=JA,ca=cb,cb=cc,cc=ca,Ja=Jb,Jb=Jc,Jc=Ja}:              ex2:=subs(rap,expr):    ex3:=subs(rap,ex2):    ex:=expr+ex2+ex3:    RETURN(ex) end: 复制代码 那个 rap 里很清楚地显示了这个函数适用于哪些字母。 但是里面只定义了求和,没有求积,也没有四元,所以可以仿照着定义一下,方便使用。 四元的情况的 rap 就不用定义那么多东西了,就弄 a,b,c,d 和 x,y,z,w 好了。 sgm4:=proc(expr)    local rap,ex2,ex3,ex4,ex:        rap:={a=b,b=c,c=d,d=a,x=y,y=z,z=w,w=x}:              ex2:=subs(rap,expr):    ex3:=subs(rap,ex2):    ex4:=subs(rap,ex3):    ex:=expr+ex2+ex3+ex4:    RETURN(ex) end: pro:=proc(expr)    local rap,ex2,ex3,ex:        rap:={a=b,b=c,c=a,A=B,B=C,C=A,x=y,y=z,z=x,ha=hb,hb=hc,hc=ha,Ra=Rb,Rb=Rc,Rc=Ra,          ra=rb,rb=rc,rc=ra,ma=mb,mb=mc,mc=ma,wa=wb,wb=wc,wc=wa,ka=kb,kb=kc,kc=ka,          HA=HB,HB=HC,HC=HA,IA=IB,IB=IC,          IC=IA,Ha=Hb,Hb=Hc,Hc=Ha,A=B,B=C,C=A,Ra=Rb,Rb=Rc,Rc=Ra,GA=GB,GB=GC,          GC=GA,JA=JB,JB=JC,JC=JA,ca=cb,cb=cc,cc=ca,Ja=Jb,Jb=Jc,Jc=Ja}:              ex2:=subs(rap,expr):    ex3:=subs(rap,ex2):    ex:=expr*ex2*ex3:    RETURN(ex) end: pro4:=proc(expr)    local rap,ex2,ex3,ex4,ex:        rap:={a=b,b=c,c=d,d=a,x=y,y=z,z=w,w=x}:              ex2:=subs(rap,expr):    ex3:=subs(rap,ex2):    ex4:=subs(rap,ex3):    ex:=expr*ex2*ex3*ex4:    RETURN(ex) end: 复制代码
thread-1149-1-4.html: [不等式] 2003美国竞赛不等式的推广
yes94 1# 2013-2-18 15:12
By azhenping:
kuing 2# 2013-2-18 15:26
切线、支撑线照用……
yes94 3# 2013-2-18 15:51
2# kuing
yes94 4# 2013-2-18 17:25
有没有柯西、均值不等式等等的方法啊?
pxchg1200 5# 2013-2-20 18:56
其实柯西也是和切线的本质差不多的。
yes94 6# 2013-2-20 20:03
其实柯西也是和切线的本质差不多的。 pxchg1200 发表于 2013-2-20 18:56 只是切线要令a+b+c=1,并且切线法得出的局部不等式一步就成立, 但柯西后一般要多步才证明完,就像旅游一个美景,缓步可以欣赏到其中的风景,匆匆而过就是走马观花,! 就看人们是着重点在于完成任务呢还是欣赏美景
thread-115-1-1.html: 求解数列通项命令RSolve的bug?
kuing 1# 2011-10-18 17:27
虽然我们知道 RSolve 并不是万能,对于一些可解的二次分式也解不出来,但一次分式的一般能解出。 但下面这个一次分式竟然也解不出: In[95]:= RSolve[{a[1] == 1/2, a[n + 1] == 1/(2 - a[n])}, a[n], n] During evaluation of In[95]:= RSolve::bvnul: For some branches of the general solution, the given boundary conditions lead to an empty solution. >> Out[95]= {} 若将 1/(2 - a[n]) 改成 1/(1 - a[n]) 就能解出,那个 2 变成 3、4、5 都没问题,但是变成 -2 也不行。算了一下,2 和 -2 的时候正好是不动点方程的根相同时,莫非就因为这样所以求不出?但事实上有重根也有办法,倒数法就行了,呃,这算不算 bug?
kuing 2# 2011-10-18 17:30
不知这在新版本 Mathematica8 里面有没有改进?
①②③④⑤⑥⑦ 3# 2011-10-19 13:11
2# kuing 没有改进。 方法问题吧,不清楚中间具体的变换方法,可能是漏考虑了,或者不care这些特殊例子。 从解函数方程来说,下面给出的那个解是不错的,虽然没包括所有情况,也许对于两个不动点重合的情形再给出更多的解更好一点。 对于p=2,用通式得到了 a[n]=1 这个么平凡的解,再去匹配其他的条件他就会宣告计算失败了。
kuing 4# 2011-10-20 21:44
3# ①②③④⑤⑥⑦ 竟然只得到了an=1。。。
thread-1150-1-4.html: [不等式] (转)群里看到一个不等式a+b+c=3
realnumber 1# 2013-2-18 18:10
本帖最后由 realnumber 于 2013-2-18 23:06 编辑 江苏-冯加明(12*****96)  18:02:23 $a,b,c\in [0,3],a+b+c=3$,求证:$(1+a^3)(1+b^3)(1+c^3)\ge (1+a^2)(1+b^2)(1+c^2)\ge (1+a)(1+b)(1+c)$.
realnumber 2# 2013-2-18 18:38
1# realnumber 原来是杨学枝老师的22个猜想14的特例.
kuing 3# 2013-2-18 20:35
只要证右边的就可以了 PS、麻烦主题分个类
kuing 4# 2013-2-18 21:11
哎,既然是被解决过的特例,我也懒得想妙法,既然齐次化是3元6次,用 Schur 分拆算了。 \[(1+a^2)(1+b^2)(1+c^2)\geqslant (1+a)(1+b)(1+c),\] 齐次化即 \[\prod\left(\frac{(a+b+c)^2}9+a^2\right)-\frac{(a+b+c)^3}{27}\prod\left(\frac{a+b+c}3+a\right)\geqslant0,\] Schur 分拆为 \[\frac2{243}\sum a^4(a-b)(a-c)+\frac5{243}\sum a^3(c+b)(a-b)(a-c)+\frac{26}{243}(a-b)^2(b-c)^2(c-a)^2+\frac{92}{243}\sum b^2c^2(a-b)(a-c)+\frac{10}{81}abc\sum a(a-b)(a-c)+\frac{14}{243}abc\sum (c+b)(a-b)(a-c)\geqslant 0,\] 显然成立。 刚才为什么说只要证右边?这是因为,令 $f(t)=(1+a^{t+1})/(1+a^t)$,求导有 \[f'(t)=\frac{a^t(a-1)\ln a}{(1+a^t)^2},\] 易见 $f'(t)\geqslant0$ 恒成立,即 $f(t)$ 递增。由此易得,对 $t\geqslant1$,都有 \[\frac{(1+a^{t+1})(1+b^{t+1})(1+c^{t+1})}{(1+a^t)(1+b^t)(1+c^t)}\geqslant\frac{(1+a^2)(1+b^2)(1+c^2)}{(1+a)(1+b)(1+c)},\] 所以,只要证明了上式右边大于等于1,那么原不等式左边自然成立,更高次也成立。 话说,多元情况不知能不能这样玩?
kuing 5# 2013-2-18 22:31
咦?多元的情形似乎对比较大的 $n$ 就不成立了,那个猜想到底是被证明了还是被否定了的? PS、顺便问下那堆猜想的最新情况
realnumber 6# 2013-2-18 23:02
本帖最后由 realnumber 于 2013-2-18 23:05 编辑 5# kuing 没多少留下了;14.杨老师修改了条件,附件其实可以百度到
kuing 7# 2013-2-18 23:08
6# realnumber 原来后来才加了条件……
realnumber 8# 2013-2-18 23:12
7# kuing 加条件,我记得发给你快要完成的.doc过,希望你补完的,不过在这种情况下,很快自己完成证明了.
kuing 9# 2013-2-18 23:14
哎,我一直没怎么关注过那堆猜想,落后了啊…… n 元你们解决过了,我还是玩玩低元,看有没有简单证法。
realnumber 10# 2013-2-18 23:28
9# kuing 开玩笑,落后? 至少比我好几个等级,我只会导数处理,别的常见不等式基本不会用,甚至高考难度地运用cauchy.
kuing 11# 2013-2-18 23:33
10# realnumber 我是指不了解最新信息…… 其实说到玩导数,我还不够你熟呢,所以我很少能搞定指数不等式,这方面你解决得比较多。
realnumber 12# 2013-2-18 23:42
11# kuing 空想想,要是都在同个论坛就好了,人多才势众,睡觉去了,
pxchg1200 13# 2013-2-20 18:50
4# kuing 好久没上了。 Proof: we may assume $a\ge b \ge c$. 1-CASE. $a\ge b \ge 1\ge c$. By CS \[(a^2+1)(b^2+1)(c^2+1)\ge (a+b)(b+c)(c+a)\] \[(a+b)(b+c)(c+a)-(a+1)(b+1)(c+1)=2(1-a)(1-b)(1-c)\ge 0\] 2-CASE. $a\ge 1\ge b\ge c$.By CS \[(a^2+1)(b^2+1)(c^2+1)\ge (a+b)(a+c)(bc+1)= \] \[=(a+b)(a+c)(b+c)+(a+b)(a+c)(1-c)(1-b)\] \[(a+b)(a+c)(bc+1)-(a+1)(b+1)(c+1)=\] \[(a+b)(a+c)(1-c)(1-b)+2(1-a)(1-b)(1-c)=\] \[=(1-b)(1-c)[(a+b)(a+c)-2a+2]=(1-b)(1-c)[(a-1)^2+1+ab+bc+ca]\ge 0\]
kuing 14# 2013-2-20 22:54
玩不玩根号 $a$, $b$, $c\geqslant0$, $a+b+c=3$,证 \[\bigl(1+a\sqrt a\bigr)\bigl(1+b\sqrt b\bigr)\bigl(1+c\sqrt c\bigr)\geqslant(1+a)(1+b)(1+c).\]
yes94 15# 2013-2-20 22:57
14# kuing 看你们玩,学习中
realnumber 16# 2013-2-21 16:29
$a$, $b$, $c\geqslant0$, $a+b+c=3$,证 \[\bigl(1+a^m)\bigl(1+b^m)\bigl(1+c^m)\geqslant(1+a)(1+b)(1+c).\] 几何画板实验结果,似乎$m\ge 1$都成立.
kuing 17# 2013-2-21 16:35
16# realnumber 几何画板很难显示出很精确的东西,你试试 m=1.01 时放大点来看
realnumber 18# 2013-2-21 16:46
17# kuing 果然不对了
kuing 19# 2013-2-21 16:51
18# realnumber 嗯,我是用 mathematica 画图来实验的,画出来比较清楚,最佳值现在也不确定,不过的确可以比较小,也就是说上面改成三次、四次根号应该都成立……
realnumber 20# 2013-2-21 16:54
19# kuing 这个最佳估计不容易,改成2元或四元,m最佳有可能变化
thread-1151-1-4.html: [函数] 三角
realnumber 1# 2013-2-18 18:40
本帖最后由 realnumber 于 2013-2-18 23:06 编辑 群里看到的 江苏-冯加明(12****96)
kuing 2# 2013-2-18 20:32
1# realnumber 最小值显然是0; 最大值方面,进一步知 $x$ 能且只能取 $[0,\pi/4]$,于是只要求 $x+2\cos x$ 的最大值,求导易知当 $x=\pi/6$ 时取最大值 $\pi/6+\sqrt3$。
yes94 3# 2013-2-18 22:56
本帖最后由 yes94 于 2013-2-18 22:59 编辑 本来不得已,是不想用图像法解答题的,这次用一用! 明显可行域如图阴影部分所示,作直线$y=-\dfrac12x+\dfrac z2$, 最小值$z_{\min}=0$,最大值和$y=\cos x$相切,对$y=\cos x$求导,令其导数$-\sin x=-\dfrac12$得,$x=\dfrac{\pi}6$,故可得$z_{\max}=\dfrac{\pi}6+\sqrt3$。
thread-1152-1-1.html: 旋度与散度
秋风树林 1# 2013-2-18 19:17
总之,三观尽毁。。。
kuing 2# 2013-2-18 19:37
1# 秋风树林 表示还没这些概念……lu过……
海盗船长 3# 2013-2-18 22:17
... adec散度为0 abc旋度为0
秋风树林 4# 2013-2-19 07:47
3# 海盗船长 其实a的旋度不为0...d的旋度为0 所以我三观都毁了...
秋风树林 5# 2013-2-19 09:03
总之还是把mathematica给出的结果发上来... a的旋度不为0倒是可以明显从图中得出 其实d的无旋性应该还只是一种可能...非一定这样
海盗船长 6# 2013-2-19 12:40
好吧,我把a看成匀强的了
海盗船长 7# 2013-2-19 12:43
其实这种只给图像没有解析式应该是不能判断的
thread-1153-1-4.html: [不等式] 安zp刚才在Q上发来的三元不等式
kuing 1# 2013-2-18 20:19
PS、不知是问?还是考? 不过挺简单,都没所谓了。 题目:设 $x$, $y$, $z>0$,求证 \[\sum \frac{yz(y+z)}{y(z+x)+z(x+y)}\geqslant \frac{x+y+z}2.\] 作倒代换 $x\to 1/x$, $y\to 1/y$, $z\to 1/z$,代入化简易知等价于 \[\sum \frac{x^2(y+z)}{2x+y+z}\geqslant \frac{xy+yz+zx}2,\] 由柯西不等式,有 \[\sum \frac{2x+y+z}{y+z}\sum \frac{x^2(y+z)}{2x+y+z}\geqslant(x+y+z)^2,\] 于是只要证 \[ 2(x+y+z)^2\geqslant (xy+yz+zx)\left( 3+2\sum \frac x{y+z} \right), \] 即 \[\frac{2(x+y+z)^2}{xy+yz+zx}-6\geqslant 2\sum \frac x{y+z}-3,\] 容易 SOS 为 \[\frac{\sum (x-y)^2}{xy+yz+zx}\geqslant \sum \frac{(x-y)^2}{(y+z)(z+x)},\] 显然成立。
yes94 2# 2013-2-18 21:24
1# kuing 有90%的把握是问,我这话在熟悉kuing的人中99.9%的人都会同意, 尽管他研究不等式也多年了,也颇有成果和造诣。
kuing 3# 2013-2-18 21:28
2# yes94 话说我写完还给他发了链接的……
yes94 4# 2013-2-18 21:57
3# kuing 呵呵!还给链接啊
kuing 5# 2013-2-18 22:03
4# yes94 当然要给的啊,不然他以为我不鸟他……
kuing 6# 2013-2-19 10:52
4# yes94 其实我3#的意思是不知他看到你的回复会有何感想。。。 不过昨晚我给链接后他也没有反应
yes94 7# 2013-2-19 14:23
呵呵!我这评价也没有贬低他吧? 任何高手,必定有自己做不出来的题目的,对吧?谁敢说自己打遍天下无敌手? 像宋庆老师、杨学枝老师等等若干不等式高手,也有不会做的不等式,(他们命名为猜想),这就是事实的嘛! 况且我还评价他颇有造诣和成果,没说错吧? 哈哈!
kuing 8# 2013-2-19 14:55
7# yes94 嗯。
yes94 9# 2013-2-19 14:58
8# kuing 那就好,希望他看见了不要多心哈!
guanmo 10# 2013-2-19 17:01
很有兴趣的是,yes94老师尊姓大名啊?膜拜已久哦!
thread-1154-1-4.html: [不等式] 人教群里看到的典型换元法换出来的不等式
kuing 1# 2013-2-18 23:50
PS、其实我前几天好像在哪里见过这道题,不过当时没怎么看,所以也不记得是谁命的题,不过应该不是刀客编的。 先作换元 $1/(x+1)=a \iff x=1/a-1$ 等,则 $a$, $b$, $c\in(0,1)$ 且 $a+b+c=2$,代入不等式右边并齐次化后就是极为熟悉的 Nessbit 不等式。 注意到由 $0<a$, $b$, $c<1$ 且 $a+b+c=2$ 易得 $a+b>c$ 等等,可见 $a$, $b$, $c$ 构成三角形,于是再令 $a=t+u$, $b=u+v$, $c=v+t$, $t$, $u$, $v>0$,则 $t+u+v=1$,代入左边的不等式并齐次化,等价于 \[\sum\sqrt{\frac{tu}{(u+v)(v+t)}}\leqslant\frac32,\] 这时用均值已经显然成立,不过若是了解内切圆代换,也可以知道上式其实也等价于极为熟悉的三角形不等式 \[\sum\sin\frac A2\leqslant\frac32.\] 这样你应该能知道这题是怎么编制的了。
thread-1155-1-1.html: 神奇。。的代码
╰☆ヾo.海x 1# 2013-2-19 01:58
我才发现点那些用代码打的东东,还会放大诶。。。。。好神奇。。。
kuing 2# 2013-2-19 02:03
1# ╰☆ヾo.海x 我好像是设置成默认双击公式就会放大。当然我是说电脑上,爪机上就不清楚了。
thread-1156-1-2.html: [数列] 请教:黑龙江省哈三中2012—2013学年度上学期高三学年期末考试数学试卷(理科)第12题
shidilin 1# 2013-2-19 15:39
标准答案是D,有啥简捷的解法?谢谢!
realnumber 2# 2013-2-19 16:52
本帖最后由 realnumber 于 2013-2-19 17:31 编辑 先做了试验(或者说特殊到一般) $a_1=1,a_2=2,a_3=\frac{1}{2},a_4=3,a_5=\frac{1}{3},a_6=\frac{3}{2},a_7=\frac{2}{3},a_8=4,a_9=\frac{1}{4}$ \[那么a_n=1+\frac{8}{11},a_{\frac{n}{2}}=\frac{8}{11},a_{\frac{n}{2}-1}=\frac{11}{8}=1+\frac{3}{8}\] \[a_{\frac{1}{2}(\frac{n}{2}-1)}=\frac{3}{8},a_{\frac{1}{2}(\frac{n}{2}-1)-1}=\frac{8}{3}=1+1+\frac{2}{3}\] 也即$\frac{1}{4}(\frac{1}{2}(\frac{n}{2}-1)-1)=7$ 选择题似乎足够了,解答题看来还要证明$a_m=\frac{2}{3},m=7$是唯一解.---- 不知道怎么做 似乎$a_n=1+\frac{8}{11}\riff a_{\frac{n}{2}}=\frac{8}{11}$都不够严密.----也不知道怎么做,n偶数奇数分类试下?
yuzi 3# 2013-2-19 21:30
本帖最后由 yuzi 于 2013-2-19 21:38 编辑 人教论坛里,战巡似乎解答过类似的题。。。。
yuzi 4# 2013-2-19 22:00
本帖最后由 yuzi 于 2013-2-19 22:30 编辑 首先$n\ge 2$时,$n$为偶数时,$a_n>1$;$n$为奇数时,$a_n<1$. 因为$a_n=\frac{19}{11}>1$,所以$n$为偶数。 则$a_{\frac{n}{2}}=\frac{8}{11}<1,a_{\frac{n-2}{2}}=\frac{11}{8}>1,a_{\frac{n-2}{4}}=\frac{3}{8}<1,$    $a_{\frac{n-6}{4}}=\frac{8}{3}>1,a_{\frac{n-6}{8}}=\frac{5}{3}>1,a_{\frac{n-6}{16}}=\frac{2}{3}<1,$    $a_{\frac{n-22}{16}}=\frac{3}{2}>1,a_{\frac{n-22}{32}}=\frac{1}{2}<1,a_{\frac{n-54}{32}}=2>1,$    $a_{\frac{n-54}{64}}=1$ 所以$\frac{n-54}{64}=1,n=118.$
shidilin 5# 2013-2-19 23:12
感谢诸位!
dualliot 6# 2013-4-9 08:12
本帖最后由 dualliot 于 2013-4-9 08:15 编辑 $\dfrac{19}{11}=1+\dfrac{1}{1+\dfrac{1}{2+\dfrac{1}{1+\dfrac{1}{1+1}}}}$ 若倒数,则下标加1;若前面加$n$,下标乘以$2^n$. 感谢Telsa的整理!正在遍历.
yes94 7# 2013-4-9 12:02
6# dualliot 买糕的! 原来背景是连分数?
Tesla35 8# 2013-4-9 12:15
6# dualliot 欢迎把那些链接里的未解之题,解答后回复于各贴之中
李斌斌755 9# 2013-4-9 14:34
8# Tesla35 打通关
dualliot 10# 2013-4-10 09:51
把Tesla35的id写错了.
thread-1157-1-2.html: 打字
realnumber 1# 2013-2-20 10:19
y=x^2+2 $y=x^2+1$
kuing 2# 2013-2-20 10:33
what ?
realnumber 3# 2013-2-20 10:36
给一个学生演示,可以来这里发贴,省得乘40分钟公共汽车到我家补课.
kuing 4# 2013-2-20 10:37
在这里上公开课??
realnumber 5# 2013-2-20 10:46
不是啊,是解答问题, 一个2流学校重点班的学生,有能力自学的.
realnumber 6# 2013-2-20 10:48
5# realnumber 也没一定要求她这么做,只是给她提供个信息. 又发现会上网的不少学生,并不知道有论坛,贴吧什么的,
kuing 7# 2013-2-20 10:48
5# realnumber oh,还以为在这里上课这么牛…… 不管什么流什么的都没所谓啦
realnumber 8# 2013-2-20 10:50
6# realnumber 至于我任教的学生,虽然也每届都介绍,来论坛的也就1,2个.三流普高,没那个基础与动力.
realnumber 9# 2013-2-20 10:53
8# realnumber 都快6,7届了,才那么1,2个 ,所以工作只是混饭吃的,还不如把重点放在培养自己孩子上.
kuing 10# 2013-2-20 11:01
所以我不搞教学
realnumber 11# 2013-2-20 11:09
我猜,你除非去搞竞赛,或重点中学; 否则没耐心讲解的,学生会认为跳步厉害.
kuing 12# 2013-2-20 11:12
11# realnumber 主要是没兴趣。再说,竞赛我也不太会。
isea 13# 2013-2-20 23:27
我就从来不出帖吧
╰☆ヾo.海x 14# 2013-2-23 00:34
10# kuing 但是还是要教我哦!!参见酷老师,受海x一拜
kuing 15# 2013-2-23 00:42
14# ╰☆ヾo.海x
戊概念·五 16# 2013-2-24 18:52
14# ╰☆ヾo.海x 当心你这一拜、磕到他聪明的脑袋~
thread-1158-1-4.html: 一个不等式,玩好后推广
realnumber 1# 2013-2-20 17:51
$(2n+1)^n \ge (2n)^n +(2n-1)^n,n\in N^*$ 这里看到
thread-1159-1-4.html: [不等式] ham-hap 不等式
pxchg1200 1# 2013-2-20 19:13
Let $a,b,c$ be postive numbers,show that: \[ \frac{(4a+b)(4c+a)}{4b^2+bc+c^2}+\frac{(4b+c)(4a+b)}{4c^2+ca+a^2}+\frac{(4c+a)(4b+c)}{4a^2+ab+b^2}\ge\frac{25}{2}. \] 要回学校了,假期就这样混完了。。。。   以后可能没多少时间上来玩了,各种麻烦事。
thread-116-1-9.html: [不等式] 比较a,b,c的大小关系
nash 1# 2011-10-18 17:59
[kuing edit]: 用代码写,自己编辑一下贴子看看是怎么打的,很简单。 已知 $a>0,a^2-2ab+c^2=0,bc>a^2$,则实数 $a,b,c$ 的大小关系为________。
kuing 2# 2011-10-18 18:14
首先显然 $b>0$,否则由 $a>0$ 知 $a^2-2ab+c^2>0$ 矛盾,故再由 $bc>a^2$ 知也有 $c>0$,即 $a,b,c$ 都是正数,下面可以放心用基本不等式。 \[b=\frac{a^2+c^2}{2a}\geqslant \frac{2ac}{2a}=c,\] 以及 \[bc>a^2=2ab-c^2\geqslant 2ab-bc \implies c>a,\] 由此也可见 $b,c$ 不能取等,所以 $b>c>a$
ludousxm 3# 2011-10-18 18:16
thread-1160-1-4.html: [数列] 一个数列填空题
转化与化归 1# 2013-2-20 19:56
一个数列填空题
realnumber 2# 2013-2-21 12:39
1# 转化与化归 奇怪$a_2$就有2个值.$a_2$是方程$x=\frac{2}{2(2+x)-2}$的根.即$x^2+x-1=0$,$a_3$就可能有四个,依次下去,8个,16个,..? 是不是题目出问题了.
yes94 3# 2013-2-21 14:32
2# realnumber 所以建议,以后发题人应该稍微提一下题目的来源,例如自编、某市模拟题、某年竞赛题等等。
转化与化归 4# 2013-2-21 17:37
学生做的自主招生辅导班的题,我们不妨加个条件a(n)>0
realnumber 5# 2013-2-23 00:15
4# 转化与化归 还是不会,实验都困难,$a_3$都要算不动了,... 有答案后,请贴下
kuing 6# 2013-2-23 13:40
5# realnumber 只怕到时贴的答案是说题目抄错了
yes94 7# 2013-2-23 15:34
5# realnumber 只怕到时贴的答案是说题目抄错了 kuing 发表于 2013-2-23 13:40 这种事件遇到好几起了,
转化与化归 8# 2013-2-24 08:33
这是一道学生问的题目,我只能保证从学生到我这儿肯定没有抄错,至于学生有没有抄错,我不能保证!但我会去核实一下!
kuing 9# 2013-2-24 08:36
8# 转化与化归 完全明白。
thread-1161-1-4.html: 求助:概率问题
雪落湖畔 1# 2013-2-20 19:57
怎么用概率证明,在胜率是0.5的条件下,久赌必输。有熟悉的朋友老师请帮忙
kuing 2# 2013-2-20 20:14
严格 0.5 的话不会久赌必输吧? 或者是考虑资金(赌本)的原因?
yes94 3# 2013-2-20 20:21
条件不严格 如果是两人对垒,怎么会久赌必输? 反而是久赌不输不赢。
realnumber 4# 2013-2-20 22:25
3# yes94 可能是这个意思,两个人一定会分出胜负,胜者与另外一个继续赌,再分出胜负,再继续与下一个,...也许是这样类似的意义上,久赌必输.
yes94 5# 2013-2-20 22:56
4# realnumber 那不是$n$场比赛连胜的概率为$\dfrac1{2^n}$?极限为零。
kuing 6# 2013-2-20 22:58
4# realnumber 应该不是这个意思
realnumber 7# 2013-2-21 09:29
百度的    中科院院士论证"十赌九输" 茅于轼:庄家久赌必赢 新加坡赌场管制局今年初对有关赌博业所使用的数学计算法进行研究后发现,人们常说的“十赌九输”是有一定科学依据的。   数学家、中国科学院院士张景中指出,在大量抛硬币的过程中,我们不难发现经常有N次连续出一面的情况发生,这种情况的科学说法即“概率波动论”,出现概率波动是概率发生的必然。因此,赌博过程中常常会出现连输N次的情况。   张景中表示,排除外界因素干扰,赌博游戏要想最终不输钱,只能建立在一个基础之上:无限次赌博。但由于每个人的精力有限,资本有限,不可能做到无限次赌博。这将导致下赌注的资本越来越少,也越来越难翻本。   著名经济学家茅于轼一针见血地指出,进赌场赌钱,输的机会当然比赢的机会大,否则赌场都会赔本关门。由“庄家理论”不难知道,赌客的总体收益率必为负。茅于轼进一步分析说,赌场庄家就是依靠精确计算的胜负概率之差来赚钱的。换言之,赌客赢钱的机会略低于输钱的机会,这个差数的平均值就成为赌场的收入,用以支付赌场的各种开支并赚取利润。赌博没有制胜的绝招,庄家没有,赌客也没有,只有久赌必输和久赌必赢的大势。略微有利于庄家的赌规加上大数定律,构成了赌场这个“攻不破的堡垒”的坚强基石
realnumber 8# 2013-2-21 09:35
依然百度  数学家组团赌博狂赚上百亿 只有数学家能利用漏洞 摘要:赌场在设定规则的时候,总要保证概率略微有利于庄家。据悉,目前“庞特俱乐部”中至少3名成员澳大利亚塔斯马尼亚岛的职业赌客大卫?瓦尔士、乔治?马马卡斯以及泽尔吉克?拉诺嘎杰克正在接受调查,并收到了巨额税单。赌场在设定规则的时候,总要保证概率略微有利于庄家。所以世人都说久赌必输。从长期来看,赌场和向赌场收税的ZF拿走了赌客的全部金钱。但凡事有例外,在规则足够复杂的时候,某些数学家的确能找到被庄家忽视的概率漏洞。
kuing 9# 2013-2-21 11:28
7# realnumber 嗯,我前面提到的“资金(赌本)的原因”大概也跟张的意思差不多,因为个体赌本少,相对来说du场就可以看成无限大,双方都可能连输N次,但你可能就在某次中输清光。
雪落湖畔 10# 2013-2-21 15:45
这个是指现实中的赌场中的概率问题,不是只有2个人的情况,貌似以前见过有人证明过的,现在找不到了。谢谢大家的参与
雪落湖畔 11# 2013-2-21 15:52
可以用数学方法证明不?
thread-1162-1-2.html: [几何] 请教一个求面积的题
abababa 1# 2013-2-20 20:00
矩形$ABCD$中,$E$,$F$两点分别在$BC$,$CD$上,$\triangle ABE$,$\triangle ECF$,$\triangle FDA$的面积分别是$4,3,5$,求$\triangle AEF$的面积。 列方程到是容易算,有没有其它方法呢?
yes94 2# 2013-2-20 20:14
本帖最后由 yes94 于 2013-2-20 20:18 编辑 1# abababa 面积为多少? 8?
abababa 3# 2013-2-20 20:28
2# yes94 是的,面积是8,我是设了长宽,然后解方程算出来的,有没有更好点的方法呢?
yes94 4# 2013-2-20 20:34
3# abababa 就是建立方程组,不必解出具体未知数,整体代换就得到了面积8.
abababa 5# 2013-2-20 20:39
4# yes94 是的,不用解出来长宽,只要乘积就可以了。我是想有没有作辅助线,然后通过图形的割补这类方法解呢?看这几个数都挺整的,答案也是整数
yes94 6# 2013-2-20 20:50
5# abababa 但边长不是有理数,是无理数。
yes94 7# 2013-2-20 20:54
6# yes94 哪里找的题啊?小学?初中?
abababa 8# 2013-2-20 21:19
7# yes94 初中题。 不一定都是无理数才行吧?把AD换成10,其它图中出现的$\sqrt{10}$都换成1也可以的
yes94 9# 2013-2-20 21:44
8# abababa 那我还猜错了,
李斌斌755 10# 2013-5-3 14:34
本帖最后由 李斌斌755 于 2013-5-3 14:37 编辑 还得求出$k$来才能作图 设$BE=a_1,EC=a_2,DF=b_1,FC=b_2,\dfrac{b_1}{b_2}=k\riff a_1(b_1+b_2)=\dfrac12[b_1(a_1+a_2)+a_2b_2]=4\riff\dfrac{a_1}{a_2}=\dfrac{k+1}{k+2}$ 又\[S_{ABCD}=2S_{ADF}\dfrac{b_1}{b_1+b_2}=2S_{ABE}\dfrac{a_1}{a_1+a_2}\\\riff5k^2+2k-7=0\riff k=1,k=-1.4\] 故$E$为$CD$的中点,由图作$FG\sslash AD$交$AB,AE$于$G,H$两点,作$EP\sslash AB$交$GF$于$P$。有\[S_{AGH}=S_{EPH}\riff S_{AEF}=S_{AGF}+S_{EPF}=5+3=8\]
thread-1163-1-2.html: 看到个只用一个转角公式原来魔方的,这个公式是……,但,还是无聊……
isea 1# 2013-2-20 23:07
本帖最后由 isea 于 2013-2-20 23:13 编辑 看到个只用一个转角公式原来魔方的,这个公式是(右手公式)U R U' L' U R' U' L;我怎么都觉得太无聊了,虽然含其对称的左手公式,但,还是无聊…… 虽然偶曾经用上公式的对称公式 + R U R U R U' R' U' R' (含左右手及对称公式)也做个这种无聊的事……这样想想,上面先对棱角的确避免偶的附加公式
kuing 2# 2013-2-20 23:10
魔方……N年前玩过,忘得差不多了……现在大概只会还原一层(这自己很容易研究出来,第二层开始就无能为力了)
isea 3# 2013-2-20 23:24
魔方这玩意,是这样,玩会了就不想了;最近搞了个四阶转了转,顺便又简单的看了看三阶高级玩法,的确有点意思 每个人玩魔方,哪怕是同个说明书,都有不同的地方
戊概念·五 4# 2013-2-24 18:49
2# kuing 当时拆开之后装回去了哦?!
thread-1164-1-4.html: [不等式] 昨晚粉丝群看到的六元不等式的反例(原来是杨学枝的某个猜想)
kuing 1# 2013-2-21 01:31
杨学枝猜想(2012.08.15)设 $a$, $b$, $c$, $x$, $y$, $z>0$,则 \[\frac1{\sqrt{ax(a+x)}}+\frac1{\sqrt{by(b+y)}}+\frac1{\sqrt{cz(c+z)}}\geqslant\frac3{\sqrt{abc+xyz}}.\] 反例 a=2, b=4, c=1, x=2, y=1, z=4 很可惜,很漂亮的不等式,没成立。 话说我也想了一晚,开头不知道是猜想,所以一直以为是正确的,后来整来整去就怀疑不正确了,但由于 bottema 对多元多根式一般会卡死,即使能出来反例大概也不容易手工验算,所以为了找这个好算的反例还是试出来的……
yes94 2# 2013-2-21 14:37
1# kuing 怎么没人把kuing发掘出来啊!
kuing 3# 2013-2-21 14:56
1# kuing 怎么没人把kuing发掘出来啊! yes94 发表于 2013-2-21 14:37 呃……否定了一个小猜想而已,说不定已经有人发现过,只是我不知道,我消息不太灵通 …… 讲开又讲,现在网络时代了,有论坛、Q这些平台大家交流都很容易,应该没什么发掘不发掘之说了吧
thread-1168-1-4.html: [函数] 转一个函数交点个数的问题
hongxian 1# 2013-2-22 10:01
1.已知函数$f(x)=\left\{ \begin{matrix}    {{\log }_{2}}(x+2)\ \ \ \ x<0  \\    \frac{1}{2}f(x-1)\ \ \ \ \ x\ge 0  \\ \end{matrix} \right.$若$y=f(x)$与$y={{\left( \frac{1}{3} \right)}^{x}}+a$的图象恰有3个不同的交点,求实数$a$的取值范围 原来估计是一个错题,不知有没有正解。关键是$a$可不可以大于0? ______kuing edit______ $f(x)=\begin{cases} \log_2(x+2) & x<0 \\ \frac{1}{2}f(x-1) & x\ge 0 \end{cases}$
kuing 2# 2013-2-22 11:41
爪机ing... 这类题也是一种潮流,看图说话吧。。。 PS. 你这代码是不是软件转出来的?
hongxian 3# 2013-2-22 11:57
2# kuing 火眼金睛,的确是从mathtype中转过来的。
kuing 4# 2013-2-22 12:01
3# hongxian 因为我爪机上,只能看代码,一看这么烂的代码就猜到是软件转的,而且那个距离也是手动打空格空过去对齐的,不是自动的。
kuing 5# 2013-2-22 14:32
空心点总是右端。 对任意 $n\in\mbb N$,$f(x)$ 在 $[n,n+1)$ 上递增,$f(n)=0$,$\lim_{x\to n^-}f(x)=1/2^n$。 记 $g(x)=1/3^x+a$,$h(x)=1/2^x$。 如果 $a=0$,右边无数个交点; 如果 $a<0$,则应 $g(1)\geqslant0$ 且 $g(2)<0$; 如果 $a>0$,则应 $g(3)<h(3)$ 且 $g(4)\geqslant h(4)$。 前两点可能比较容易想,看图说话也无妨,但最后那一点就不是太显然的事了,应该要严格证明,但是说起来有点麻烦,我也不知怎么说,大概就是由 $1/2^n-1/3^n$ 是递减的正数数列可得。
hongxian 6# 2013-2-22 15:55
5# kuing $1/2^n-1/3^n$为递减的正数列,一句话点醒了梦中人,刚好是前三个交点。
kuing 7# 2013-2-22 17:35
6# hongxian 嗯,你理解了就好,那我就不用再想怎么细说了…… PS、你说是转过来的,转自哪里?给个链接或出处方便以后整理。 PS2、我编辑了一下1#将那个分段函数的正确代码写上了,你自己可以看看,对比下原来的。
yes94 8# 2013-2-22 17:38
7# kuing 用啥子高级软件画的图? 以前不会几何画板,现在刚学会几何画板画函数图了,又有新的出现了, 真的是防不胜防、学不胜学啊
kuing 9# 2013-2-22 17:41
8# yes94 Mathematica
yes94 10# 2013-2-22 17:51
9# kuing 我自建了一个网页(网址导航),把我常用的网址收集起来,以后鼠标直接点就进来了。 你的网站是直接进“初等论坛”,这样的坏处就是其他分论坛(例如高等数学、灌水论坛、Mathematica 等)都略过了
hongxian 11# 2013-2-22 19:58
本帖最后由 hongxian 于 2013-2-22 20:51 编辑 7# kuing 1.转自http://bbs.pep.com.cn/forum.php? ... &extra=page%3D1 2.$\LaTeX$现在还没有仔细学,主要是数学还没有学好,下次尽量不用软件转码了!边学边用!
hongxian 12# 2013-2-22 20:46
本帖最后由 hongxian 于 2013-2-23 08:58 编辑 5# kuing 练习一下代码,试着说一下$a>0$的道理 设三根分别在$[k-1,k)$,$[k,k+1)$,$[k+1,k+2)$,$k \in N^*$则 $\begin{cases} \left(\frac{1}{2} \right)^{k-1} \leqslant \left(\frac{1}{3} \right)^{k-1} +a \\ \left(\frac{1}{2} \right)^{k} > \left(\frac{1}{3} \right)^{k} +a \\ \left(\frac{1}{2} \right)^{k+1} > \left(\frac{1}{3} \right)^{k+1} +a \\ \left(\frac{1}{2} \right)^{k+2} > \left(\frac{1}{3} \right)^{k+2} +a \\ \left(\frac{1}{2} \right)^{k+3} \leqslant \left(\frac{1}{3} \right)^{k+3} +a \end{cases} \Longrightarrow \begin{cases} a \geqslant \left(\frac{1}{2} \right)^{k-1}-\left(\frac{1}{3} \right)^{k-1}  \\ a<\left(\frac{1}{2} \right)^{k} -\left(\frac{1}{3} \right)^{k} \\ a<\left(\frac{1}{2} \right)^{k+1} - \left(\frac{1}{3} \right)^{k+1} \\ a<\left(\frac{1}{2} \right)^{k+2}- \left(\frac{1}{3} \right)^{k+2} \\ a \geqslant \left(\frac{1}{2} \right)^{k+3} - \left(\frac{1}{3} \right)^{k+3} \end{cases}$ 所以$max \left\{ \left(\frac{1}{2} \right)^{k-1}- \left(\frac{1}{3} \right)^{k-1} ,\left(\frac{1}{2} \right)^{k+3}- \left(\frac{1}{3} \right)^{k+3} \right\} \leqslant a < min \left\{ \left(\frac{1}{2} \right)^k- \left(\frac{1}{3} \right)^k , \left(\frac{1}{2} \right)^{k+1}- \left(\frac{1}{3} \right)^{k+1}, \left(\frac{1}{2} \right)^{k+2}- \left(\frac{1}{3} \right)^{k+2} \right\}$ 又因为$\left\{ \left(\frac{1}{2} \right)^n- \left(\frac{1}{3} \right)^n \right\}$,$n \in N^*$为正项递减数列, 所以$\left( \frac{1}{2} \right)^4- \left( \frac{1}{3} \right)^4 \leqslant a < \left(\frac{1}{2} \right)^3- \left(\frac{1}{3} \right)^3$
kuing 13# 2013-2-22 20:47
11# hongxian ??这个链接不对呀?
hongxian 14# 2013-2-22 20:50
13# kuing 这一个http://bbs.pep.com.cn/forum.php? ... &extra=page%3D1 刚才有可能复制错了!
kuing 15# 2013-2-22 20:55
14# hongxian 嗯,这个链接就对了。这样看来那个题的确要选 E ……
thread-1169-1-4.html: [不等式] 来自人教群的小题(04-05法国竞赛题)
kuing 1# 2013-2-24 10:50
老师冯加明(1217*****) 24. 已知 $\sum_{i=1}^n x_i^2=\sum_{i=1}^n y_i^2=1$,证明不等式 \[(x_1y_2-x_2y_1)^2\leqslant 2\left|1-\sum_{i=1}^n x_iy_i\right|.\] (2004-2005 年法国数学奥林匹克试题) 由拉格朗日恒等式以及柯西不等式得 \begin{align*} (x_1y_2-x_2y_1)^2&\leqslant \sum_{1\leqslant i<j\leqslant n}{(x_iy_j-x_jy_i)^2} \\ & =\sum x_i^2\sum y_i^2-\left( \sum x_iy_i \right)^2 \\ & =\left| 1-\sum x_iy_i \right|\cdot \left| 1+\sum x_iy_i \right| \\ & \leqslant \left| 1-\sum x_iy_i \right|\left( 1+\left| \sum x_iy_i \right| \right) \\ & \leqslant \left| 1-\sum x_iy_i \right|\left( 1+\sqrt{\sum x_i^2\sum y_i^2} \right) \\ & =2\left| 1-\sum x_iy_i \right|. \end{align*}
yes94 2# 2013-2-24 15:16
待证不等式左边居然只有一项,而不是循环和! 过分了噻
thread-117-1-9.html: 代码试练基地
nash 1# 2011-10-18 18:26
本帖最后由 nash 于 2011-10-22 18:17 编辑 换招牌啦
nash 2# 2011-10-18 18:33
我先来一个 已知函数f(x)=$\frac {1}{a}$-$\frac{1}{x}$,(a>0,x>0).若函数f(x)在[m,n]上的值域为[m,n],求a的取值范围 —————————————————— [kuing edit] 已知函数 $f(x)=\dfrac1a-\dfrac1x,(a>0,x>0)$。若函数 $f(x)$ 在 $[m,n]$ 上的值域为 $[m,n]$,求 $a$ 的取值范围。
nash 3# 2011-10-18 18:40
暴汗… 编辑分式都没搞定 —————————————————— [kuing edit] 已知 $t>0$,关于 $x$ 的方程为 $|x|+\sqrt{t-x^2}=\sqrt2$,则下列说法正确的是________ ①存在 $t$ 使得方程无解;②存在 $t$ 使得方程有一个解;③存在 $t$ 使得方程有二个解;④存在 $t$ 使得方程有三个解;⑤存在 $t$ 使得方程有四个解。
kuing 4# 2011-10-18 19:02
2# nash 可以整个公式一起啊,不用分开的
nash 5# 2011-10-19 00:23
这个贴搞的失败 题太简单… 大家无视吧
kuing 6# 2011-10-19 00:25
5# nash 个人认为:常在线人数还不多,搞这个还未是时候。
nash 7# 2011-10-19 00:30
期待中 以后有专门的某块内容,查找学习也比较方便
kuing 8# 2011-10-19 00:31
7# nash 这个…………再说吧,呵呵
cgj1982 9# 2011-10-22 08:09
\begin{align} 我来试验公式: f(x)&=\frac{1}{a}-\frac{1}{x}的定义域和值域都为[m,n],求实数a的范围。\\ f(x)&=\frac{1}{a}-\frac{1}{x}在区间(-\infty,0),(0,+\infty)单调增,\\ 所以分m\le{n}<0,0<m\le{n},m<0<n,三种情况 f(x)&=\frac{1}{a}-\frac{1}{x}与y=x有交点,\\ 也就是\frac{1}{a}-\frac{1}{x}=x\\ \frac{1}{a}=x+\frac{1}{x}\\ \end{align} 代码好累,记不住 —————————————————————————————— [kuing edit] $f(x)=\dfrac{1}{a}-\dfrac{1}{x}$ 的定义域和值域都为 $[m,n]$,求实数 $a$ 的范围。 $f(x)=\dfrac{1}{a}-\dfrac{1}{x}$ 在区间 $(-\infty,0),(0,+\infty)$ 单调增,所以分 $m\le{n}<0,0<m\le{n},m<0<n$,三种情况 $f(x)=\dfrac{1}{a}-\dfrac{1}{x}$ 与 $y=x$ 有交点,也就是 \[\frac{1}{a}-\frac{1}{x}=x \iff \frac{1}{a}=x+\frac{1}{x}\]
kuing 10# 2011-10-22 12:30
9# cgj1982 中文说明尽量放在公式代码外面。 这里不必用align环境,用普通的行内或行间公式模式即可。
kuing 11# 2011-10-22 12:36
9# cgj1982 按照你所写,我编辑了一下,你点击编辑贴子可以看看具体写法
kuing 12# 2011-10-22 22:15
既然变成了练代码基地。。。我修改了上面的某些贴
图图 13# 2011-10-24 14:01
$\cases{ax+by+c=0 \\ c^2=a^2+b^2}$ $f(x)=\cases{x+1&x>0 \\ x-1&x\le0}$
kuing 14# 2011-10-24 14:01
$\cases{ax+by+c=0 \\ c^2=a^2+b^2}\cdots\cdots\cdots\text{(cases,只能左对齐,行距较少)}$ $\left\{\begin{aligned}&ax+by+c=0 \\ &c^2=a^2+b^2\end{aligned}\right.\cdots\cdots\cdots\text{(aligned,左边用 &,左对齐)}$ $\left\{\begin{aligned}ax+by+c=0 \\ c^2=a^2+b^2\end{aligned}\right.\cdots\cdots\cdots\text{(aligned,不加 &,右对齐)}$ $\left\{\begin{aligned}ax+by+c&=0 \\ c^2&=a^2+b^2\end{aligned}\right.\cdots\cdots\cdots\text{(aligned,等号前加 &,等号对齐)}$
thread-1170-1-1.html: 压强
yayaweha 1# 2013-2-24 12:50
本帖最后由 yayaweha 于 2013-2-24 12:52 编辑 温度升高, 则单位时间撞击单位面积的分子数增加?(体积不变 面积不变)
yes94 2# 2013-2-24 17:18
温度升高, 则单位时间撞击单位面积的分子数增加?(体积不变 面积不变) yayaweha 发表于 2013-2-24 12:50 根据PV=nRT,对压强有无限制?
thread-1171-1-4.html: [数论] 请教一个关于方程整数根的题
abababa 1# 2013-2-24 19:47
已知方程$x^2+bx+c$有两个整数根$m_1,m_2$,方程$x^2+cx+b$有两个整数根$n_1,n_2$,且有$m_1m_2>0,n_1n_2>0$,求b和c
yes94 2# 2013-2-24 20:17
本帖最后由 yes94 于 2013-2-24 20:25 编辑 1# abababa $b=c=4$,$b=5,c=6$,
abababa 3# 2013-2-24 20:24
2# yes94 是其中一个解,还有两个是b=6,c=5和b=5,c=6,网友解的,告诉我答案了,没说过程,让我再想两天,呵呵,正在想
yes94 4# 2013-2-24 20:26
3# abababa 现在写过程不?
abababa 5# 2013-2-24 20:32
4# yes94 我再想想,感觉有点眉目,但不确定能做出来。 先谢谢。
yes94 6# 2013-2-24 20:39
本帖最后由 yes94 于 2013-2-24 20:45 编辑 5# abababa 那我写在某处,http://sq.k12.com.cn/discuz/foru ... p;extra=#pid3286979, 等你做完打开链接查看(六楼),看看咱们做的一样不
abababa 7# 2013-2-24 21:06
本帖最后由 abababa 于 2013-2-24 21:08 编辑 6# yes94 $c=m_1m_2>0,b=n_1n_2>0$,然后$m_1+m_2=-b<0,m_1m_2>0$,得出$m_1<0,m_2<0$,同理$n_1<0,n_2<0$ $(m_1+1)(m_2+1)=m_1m_2+m_1+m_2+1=c-b+1$,但是$m_1<0$,而且$m_1$是整数,所以$m_1 \leqslant -1$,然后$m_1+1 \leqslant 0$,同理$m_2+1 \leqslant 0$,所以$c-b+1=(m_1+1)(m_2+1) \geqslant 0$,得到$b-1\leqslant c$ 然后从根是$n_1,n_2$的那个方程得到$0 \leqslant (n_1+1)(n_2+1)=b-c+1$,得到$c \leqslant b+1$,就是有$b-1 \leqslant c \leqslant b+1$ 但是b,c是整数,所以c只有三种可能,$c=b-1,c=b,c=b+1$ $c=b-1$时就有$(n_1+1)(n_2+1)=2$,然后$n_1,n_2$都是负整数,得到$n_1=-2,n_2=-3$,就得到b=6,c=5 $c=b$时就有$(m_1+1)(m_2+1)=1$,再由$m_1,m_2$都是负整数,得到$m_1=m_2=-2$,就得到b=c=4 $c=b+1$时就有$(m_1+1)(m_2+1)=2$,由$m_1,m_2$都是负整数,是到$m_1=-2,m_2=-3$,就得到b=5,c=6 呵呵,关键的一步在于求得b,c的那个$b-1 \leqslant c \leqslant b+1$的关系,这是网友提示让我求的。
abababa 8# 2013-2-24 21:12
6# yes94 看了一下,关键都是用了$(x_1+1)(x_2+1)$这个分解,以前也碰到过这样的,然后联系韦达定理,挺方便的,但以前都太明显了,这次就没想到。
yes94 9# 2013-2-24 21:43
6# yes94 看了一下,关键都是用了$(x_1+1)(x_2+1)$这个分解,以前也碰到过这样的,然后联系韦达定理,挺方便的,但以前都太明显了,这次就没想到。 abababa 发表于 2013-2-24 21:12 你知道为何要用$(x_1+1)(x_2+1)$这个分解吗?你是怎么想到这个分解的?
yes94 10# 2013-2-24 21:45
还是把那边k12的解法复制过来,免得那里被版主删除了 显然,$m_1$,$m_2$,$n_1$,$n_2$都是负数,故$m_1+1\leqslant 0$,$m_2+1\leqslant 0$,$n_1+1\leqslant 0$,$n_2+1\leqslant 0$, 但是,$(m_1+1)(m_2+1)=c-b+1$,$(n_1+1)(n_2+1)=b-c+1$ 故$(m_1+1)(m_2+1)+(n_1+1)(n_2+1)=2$. 而$(m_1+1)(m_2+1)\geqslant 0$,$(m_1+1)(m_2+1)\geqslant 0$ 于是只有以下这三种情况: $(m_1+1)(m_2+1)=0,(n_1+1)(n_2+1)=2$ $(m_1+1)(m_2+1)=(n_1+1)(n_2+1)=1$, $(m_1+1)(m_2+1)=2,(n_1+1)(n_2+1)=0$ 不妨设$m_1\leqslant m_2$,$n_1\leqslant n_2$, 可解得$b=6$,$c=5$,或$b=c=4$,或$b=5,c=6$
abababa 11# 2013-2-24 22:02
9# yes94 我自己得出了b,c都是正整数,然后经网友提示,让我找出b,c的不等式关系,我想了一下肯定就是固定c或b,假设固定c,让b在不等号左右加减一个整数,就是$b-a_i \leqslant c \leqslant b+a_j$,这样因为都在整数范围里,c就一定是$b-a_1,b-a_2,...b,b+a_i$这样,然后就能解了 这样想的话c-b和b-c,就是那些$a_i$肯定就是一个确定的数而不是字母了,c-b就是$m_1m_2+m_1+m_2$,这个形式我很熟悉,加上1就能分解因式,然后后面就都好算了,主要的一步就是开始没想到要确定出b和c的那个不等式关系,通过这个不等式正好得到c-b或b-c是一个能确定的数字
yes94 12# 2013-2-24 23:29
11# abababa 原来是1993年全国初中数学联赛试题,原题还有第一、二问,你直接写第三问了,标准解答和你的做法一样。 http://wenku.baidu.com/view/cf2983dca58da0116c174909.html
abababa 13# 2013-2-25 00:11
12# yes94 原来是有出处的题啊,我拿到的都是复印纸上的,没有前两问,就是求b和c。而且我自己也没完全解出,是网友提示了才解出的,呵呵,我水平不行。
yes94 14# 2013-2-25 21:48
13# abababa 如果不加条件$m_1m_2>0$,$n_1n_2>0$呢?
abababa 15# 2013-2-25 23:09
14# yes94 那样的话就有好多解吧,让c=-b-1,然后取b是任意整数都可以了,就不能列出来了。
thread-1172-1-4.html: [不等式] 这个表达式具有最大值还是最小值?
ccnu_chb_ycb 1# 2013-2-24 21:58

yes94 2# 2013-2-24 22:14
1# ccnu_chb_ycb 你的意思是你已证明了此式子一定有最值?所以你才问最大、最小值。
kuing 3# 2013-2-24 22:58
没锐角?
ccnu_chb_ycb 4# 2013-2-25 16:21
3# kuing 有锐角条件
ccnu_chb_ycb 5# 2013-2-25 16:22
2# yes94 这也是一个需要解决的问题吧 ,我没有证得
kuing 6# 2013-2-25 16:22
4# ccnu_chb_ycb 有锐角条件的话那天不是聊过的么
kuing 7# 2013-2-25 16:25
http://kkkkuingggg.5d6d.net/thread-1146-1-1.html 用这贴里那个恒等式以及那个结论……
ccnu_chb_ycb 8# 2013-2-25 16:38
7# kuing 表示被这个问题自己变来变去变糊涂啦
thread-1173-1-3.html: [数列] 请教一个数列求通项的题
hongxian 1# 2013-2-25 09:56
已知$a_0=1$,$a_1=1$,$a_{n+1}=\begin{cases} 4a_n-2a_{n-1} & n为奇数\\ a_n+3a_{n-1} & n为偶数\end{cases}$,求$a_n$。
realnumber 2# 2013-2-25 12:54
1# hongxian 可以得到$a_{2n+1}=a_{2n}+3a_{2n-1}$,即$a_{2n+1}-3a_{2n-1}=a_{2n}$----(1) $a_{2n}=4a_{2n-1}-2a_{2n-2}$,即$a_{2n}+2a_{2n-2}=4a_{2n-1}$-----(2) 由(1)得到$a_{2n-1}-3a_{2n-3}=a_{2n-2}$----(3) 把(1)(3)代入(2)得到$a_{2n+1}-3a_{2n-1}+2a_{2n-1}-6a_{2n-3}=4a_{2n-1}$---这个可以解了,都是奇数项. 类似地,可以得到偶数项的关系式. ps.题目真要命,好在可以解答了.又,$a_3,a_4$还是要从原关系式计算的.
kuing 3# 2013-2-25 14:49
将奇数项和偶数项看成是两个数列……
realnumber 4# 2013-2-25 15:06
本帖最后由 realnumber 于 2013-2-25 15:22 编辑 3# kuing 如果按$\mod3$分,是不是也可以分三个数列解决? 会不会出现意外? $a_1=1=a_2=a_3$ $a_n=\begin{cases}a_{n-1}+a_{n-2}+a_{n-3},n=3k+3,k\in N \\ a_{n-1}+2a_{n-2}+3a_{n-3},n=3k+1,k\in N\\4a_{n-1}-a_{n-2}-2a_{n-3},n=3k+2,k\in N\end{cases}$ ---似乎很难消去了,刚才2楼方法似乎不好沿用下来. $a_n=\begin{cases}a_{n-1}+a_{n-3},n=3k+3,k\in N \\ a_{n-1}+3a_{n-3},n=3k+1,k\in N\\4a_{n-1}-2a_{n-3},n=3k+2,k\in N\end{cases}$ 这样还是可以沿用2楼办法.
realnumber 5# 2013-2-25 15:26
4# realnumber 觉得都可以,可以解方程组.----好不负责任啊
kuing 6# 2013-2-25 15:40
4# realnumber cases 环境里面,表达式与条件之间加一个 & 就可以隔开并对齐。
kuing 7# 2013-2-25 15:46
已知$a_0=1$,$a_1=1$,$a_{n+1}=\begin{cases} 4a_n-2a_{n-1} & n为奇数\\ a_n+3a_{n-1} & n为偶数\end{cases}$,求$a_n$。 hongxian 发表于 2013-2-25 09:56 令 $b_{n}=a_{2n-1}$, $c_{n}=a_{2n}$,则 \[\left\{\begin{aligned} c_{n}&=4b_{n}-2c_{n-1}, \\ b_{n}&=c_{n-1}+3b_{n-1}, \end{aligned}\right.\] 容易消去。 3# kuing 如果按$\mod3$分,是不是也可以分三个数列解决? 会不会出现意外? $a_1=1=a_2=a_3$ $a_n=\begin{cases}a_{n-1}+a_{n-2}+a_{n-3},n=3k+3,k\in N \\ a_{n-1}+2a_{n-2}+3a_{n-3},n=3k+1,k\in N\\4a_{n-1}-a_{n-2}-2a_{n-3},n=3k+2,k\in N\end{cases}$ ---似乎很难消去了,刚才2楼方法似乎不好沿用下来. realnumber 发表于 2013-2-25 15:06 令 $x_{n}=a_{3n-2}$, $y_{n}=a_{3n-1}$, $z_{n}=a_{3n}$,则 \[\left\{\begin{aligned} z_{n}&=y_{n}+x_{n}+z_{n-1}, \\ x_{n}&=z_{n-1}+2y_{n-1}+3x_{n-1}, \\ y_{n}&=4x_{n}-z_{n-1}-2y_{n-1}, \end{aligned}\right.\] 的确不太好消去……
kuing 8# 2013-2-25 16:14
\[\left\{\begin{aligned} z_{n}&=y_{n}+x_{n}+z_{n-1}, \\ x_{n}&=z_{n-1}+2y_{n-1}+3x_{n-1}, \\ y_{n}&=4x_{n}-z_{n-1}-2y_{n-1}, \end{aligned}\right.\]的确不太好消去…… kuing 发表于 2013-2-25 15:46 第一式加第三式得\[z_{n}=5x_{n}-2y_{n-1},\]代入第二、三式得\[\left\{\begin{aligned} x_{n}&=8x_{n-1}-2y_{n-2}+2y_{n-1}, \\ y_{n}&=4x_{n}-5x_{n-1}+2y_{n-2}-2y_{n-1}, \end{aligned}\right.\]两式相加得\[y_{n}=3x_{n}+3x_{n-1},\]再代回去得\[x_{n}=14x_{n-1}-6x_{n-3},\] ……数据居然有点特殊,意外顺利
hongxian 9# 2013-2-25 19:18
4# realnumber 谢谢了!2#和7#的看明白了,但是$a_n=\begin{cases}a_{n-1}+a_{n-2}+a_{n-3} & n=3k+3,k\in N \\ a_{n-1}+2a_{n-2}+3a_{n-3} & n=3k+1,k\in N\\ 4a_{n-1}-a_{n-2}-2a_{n-3} & n=3k+2,k\in N\end{cases}$和$a_n=\begin{cases}a_{n-1}+a_{n-3} &n=3k+3,k\in N \\ a_{n-1}+3a_{n-3} & n=3k+1,k\in N\\ 4a_{n-1}-2a_{n-3} & n=3k+2,k\in N\end{cases}$还没有看得太明白,谢谢了!
realnumber 10# 2013-2-25 19:54
8# kuing 楼上kuing8楼已经回复 好主意!先换好元,其实kuing,我是瞎闷的,没想到真被你解出来了,那按$\mod n$呢?有没有可能? (也是用连续n项的一次齐次式表示接下来一项,当然也给初始n个值.)
hongxian 11# 2013-2-25 20:10
10# realnumber 原来是两个不同的题,我还以为是一个题变出来的!
realnumber 12# 2013-2-25 20:20
8# kuing 数据不特殊可以解决吗?
kuing 13# 2013-2-25 20:24
12# realnumber 不清楚,有空研究下,现在在搞别的东西
realnumber 14# 2013-2-25 20:41
13# kuing 应该也可以的,从你8楼的解答可以看到,消去$z_n$后,再$x_{n-1}$,这样就把$x_n$用{$y_n$}中项表示出来了. 这样看起来$mod n$也可以依次消元,就是高斯消元法.
yes94 15# 2013-2-25 21:25
看见过这道题: 已知$a_1=1$,$a_2=2$,$a_{n+2}=\begin{cases} 5a_{n+1}-3a_{n}, & 若a_n\cdot a_{n+1}为偶数\\ a_{n+1}-a_n, & 若a_n\cdot a_{n+1}为奇数\end{cases}$ 求{$a_n$}的通项公式。
hongxian 16# 2013-2-26 09:45
本帖最后由 hongxian 于 2013-2-26 09:59 编辑 15# yes94 这样是不是变成了$\left\{\begin{aligned}a_{3k}&=5a_{3k-1}-3a_{3k-2},\\a_{3k+1}&=5a_{3k}-3a_{3k-1},\\a_{3k+2}&=a_{3k+1}-a_{3k}, \end{aligned}\right.$
yes94 17# 2013-2-26 12:48
16# hongxian 不知道啊,
yes94 18# 2013-2-28 19:46
15楼的数列通项公式还不好解决的
hongxian 19# 2013-2-28 23:15
本帖最后由 hongxian 于 2013-2-28 23:18 编辑 18# yes94 象7#那样换元之后应该能够解决,写几步试一下 令$\begin{cases}x_k=a_{3k-2}\\y_k=a_{3k-1}\\z_k=a_{3k}\end{cases}$,则$\begin{cases}z_k=5y_k-3x_k &(1)\\x_{k+1}=5z_k-y_{k} &(2)\\y_{k+1}=x_{k+1}-z_k &(3) \end{cases}$,(1)代入(2)(3)消$z_k$,得$\begin{cases}y_k=\frac{1}{24}\left(x_{k+1}+15x_{k}\right) &(4)\\y_{k+1}=x_{k+1}-5y_{k}+3x_{k} &(5)\end{cases}$,(4)代入(5)好象可得$x_{k+2}-4x_{k+1}+3x_k=0$,不知算错了没有,后面应该就可以做了!
thread-1174-1-4.html: [数列] 数列不等式
guanmo 1# 2013-2-25 11:45
如图
yes94 2# 2013-2-25 12:00
1# guanmo 又来
realnumber 3# 2013-2-25 12:32
本帖最后由 realnumber 于 2013-2-25 12:33 编辑 1# guanmo n=1,2,3实验后发现一个关系式 \[\frac{1}{a_1}+\frac{1}{a_2}+\frac{1}{a_3}+\ldots+\frac{1}{a_n}=1-\frac{1}{a_n(a_n-1)}\]可以用数学归纳法证明. 后面你自己试试,应该可以做出来了.
kuing 4# 2013-2-25 12:48
嘿嘿,FAQ,爪机上给不了链接,搜索下吧,或者按主题分类看看应该能看到类似的题。
yayaweha 5# 2013-2-25 17:49
这个不很简单吗?
yayaweha 6# 2013-2-25 17:50
我当时也做过 http://kkkkuingggg.5d6d.net/thread-562-1-2.html
yayaweha 7# 2013-2-25 17:55
and http://kkkkuingggg.5d6d.net/thread-640-1-13.html
yes94 8# 2013-2-25 21:13
7# yayaweha 所以2楼说,又来这种题目,太流行啦!
thread-1175-1-3.html: [不等式] 有ln的常见不等式
yayaweha 1# 2013-2-25 17:58
本帖最后由 yayaweha 于 2013-2-25 18:20 编辑 $$\sum_{i=1}^{n}\frac{4i}{4i^2-1}>ln(2n+1)$$这个我考试时放不出来,求指导
yayaweha 2# 2013-2-25 18:12
本帖最后由 yayaweha 于 2013-2-25 19:01 编辑 好像也不难!$$\frac{4n}{4n^2-1}=\frac{1}{2n+1}+\frac{1}{2n-1}$$ $$\frac{1}{2n+1}>\frac{1}{2}(ln(2n+3)-ln3),\frac{1}{2n-1}>\frac{1}{2}ln(2n+1)$$
yayaweha 3# 2013-2-25 18:15
怎么考试就想不出来
yayaweha 4# 2013-2-25 19:08
2# yayaweha [ 好像还是不对
realnumber 5# 2013-2-25 20:04
2# yayaweha 按这个拆法 左边你先n=1,2,3,写几个,再化简下,其实都快完成了
yayaweha 6# 2013-2-25 22:21
5# realnumber 什么意思? 你来演示一下
realnumber 7# 2013-2-25 22:53
本帖最后由 realnumber 于 2013-2-26 11:50 编辑 \[\sum_{i=1}^{n}\frac{4i}{4i^2-1}=(\frac{1}{3}+\frac{1}{5}+\ldots+\frac{1}{2n+1})+(1+\frac{1}{3}+\ldots+\frac{1}{2n-1})>1+\frac{1}{2}+\frac{1}{3}+\ldots+\frac{1}{2n}>\ln(2n+1)-\ln1\] 可用导数证明 \[\frac{1}{k}\ge \ln(k+1)-\ln(k)=\ln(1+\frac{1}{k})\iff x\ge \ln(1+x)\] 由9楼提示,已经修改
ccnu_chb_ycb 8# 2013-2-26 10:23
上面的证明应该有问题吧
kuing 9# 2013-2-26 10:45
7# 最后的 $\dfrac1{2n+1}$ 改成  $\dfrac1{2n}$ 就可以了
liaoyouyu07 10# 2013-2-26 14:19
第一个不等式中的1/2是怎么放出来的?
realnumber 11# 2013-2-26 14:49
10# liaoyouyu07 你说得对,7楼还是错了,继续想
kuing 12# 2013-2-26 15:01
才发现……那往后放如何……
realnumber 13# 2013-2-26 15:10
本帖最后由 realnumber 于 2013-2-26 16:51 编辑 \[(\frac{\ln(2x+1)}{2})'=\frac{1}{2x+1}\] \[有\frac{1}{2n+1}\ge \frac{1}{2}\ln(1+\frac{2}{2n+1})=\frac{\ln(2n+3)}{2}-\frac{\ln(2n+1)}{2}\] \[所以1+\frac{1}{3}+\frac{1}{5}+\ldots+\frac{1}{2n-1}>1+\frac{1}{3}+\frac{\ln(2n+1)}{2}-\frac{\ln5}{2}\] \[\frac{1}{3}+\frac{1}{5}+\ldots+\frac{1}{2n+1}>\frac{1}{3}+\frac{\ln(2n+3)}{2}-\frac{\ln5}{2}\] ----两式相加,就可以了.
yayaweha 14# 2013-2-26 18:12
13# realnumber 比我高明,在于前两项没放
liaoyouyu07 15# 2013-2-26 18:24
这个证明方法很不错,赞一个!
realnumber 16# 2013-2-26 18:40
也可以写成这样: \[1+\frac{1}{3}+\frac{1}{5}+\ldots+\frac{1}{2n-1}>1+\frac{1}{3}+\frac{1}{6}+\frac{1}{8}+\ldots+\frac{1}{2n}\] \[那么原式左边>1+\frac{2}{3}+(\frac{1}{5}+\frac{1}{6}+\ldots+\frac{1}{2n}+\frac{1}{2n+1})>1+\frac{2}{3}+\ln\frac{1}{2n+2}-\ln5>\ln(2n+1)\]
第一章 17# 2013-3-1 08:01
用对应项也可以做,不过有点麻烦。 话说标答是怎样的?
abababa 18# 2013-3-1 13:34
发一位网友的解答 $1+2(\sum_{i=3}^{2n-1}\frac{1}{i})+\frac{1}{2n+1}>\\ -\frac{1}{2}+\sum_{i=1}^{2n+1}\frac{1}{i}=-\frac{1}{2}+\ln (2n+2)+\gamma>\ln(2n+2)>\ln(2n+1)$
yes94 19# 2013-3-1 19:24
18# abababa 引用了一个常数$\gamma$
realnumber 20# 2013-3-1 20:38
19# yes94 其实不引用也可以,只需去掉,并把"="修改为">".
thread-1175-2-3.html:
kuing 21# 2013-3-1 21:01
18# abababa 用 $\gamma$ 那个也不是等号吧
abababa 22# 2013-3-1 21:13
本帖最后由 abababa 于 2013-3-1 21:28 编辑 呵呵,其实是昨天问了网友,然后我下线没看到他发的解答,中午收到了离线消息看到了,但他不打latex代码,我匆忙间打的有很多问题,第一个>左边那个求和也错了,是隔1才加的,偶数分母不加,他的解答没这么写,我却都给加上了。 不过觉得那证明还是有问题,当$n \to \infty$时才有那个式子,有限项就不管用了。
yayaweha 23# 2013-3-1 23:28
本帖最后由 yayaweha 于 2013-3-1 23:37 编辑 发个标答吧! 容易证明$$\ln x<\frac{1}{2}(x-\frac{1}{x})$$ 令$x=\frac{2k+1}{2k-1}$有 $$\ln \frac{2k+1}{2k-1}<\frac{1}{2}(\frac{2k+1}{2k-1}-\frac{2k-1}{2k+1})$$ 即$$\ln (2k+1)-\ln (2k-1)<\frac{4k}{4k^2-1}$$ 所以$$\sum_{i=1}^n\frac{4i}{4i^2-1}>\ln (2n+1)$$ PS:也可用数学归纳法
yayaweha 24# 2013-3-1 23:29
$$\gamma$$指欧拉常数吗?
kuing 25# 2013-3-1 23:29
23# yayaweha ln 前面加个反斜杠,thanks
kuing 26# 2013-3-1 23:29
24# yayaweha 是的
yayaweha 27# 2013-3-1 23:33
25# kuing 怎么加?
kuing 28# 2013-3-1 23:35
27# yayaweha
yayaweha 29# 2013-3-1 23:38
发一位网友的解答 $1+2(\sum_{i=3}^{2n-1}\frac{1}{i})+\frac{1}{2n+1}>\\ -\frac{1}{2}+\sum_{i=1}^{2n+1}\frac{1}{i}=-\frac{1}{2}+\ln (2n+2)+\gamma>\ln(2n+2)>\ln(2n+1)$ abababa 发表于 2013-3-1 13:34 这里的常数指欧拉常数?
realnumber 30# 2013-3-1 23:49
应该是
yes94 31# 2013-3-2 13:54
类似的题参考2012天津高考
yes94 32# 2013-3-4 11:38
楼主有不等式:$\sum_{k=1}^{n}\frac{4k}{4k^2-1}>\ln(2n+1)$, 那么下述不等式是否成立(原不等式的反向)?若成立请证明: $\sum_{k=1}^{n}\frac{4k}{4k^2-1}<1+\dfrac12\ln(2n+1)(2n+3)$
thread-1176-1-3.html: [数列] 再问一个求通项的题,先谢谢了!
hongxian 1# 2013-2-26 10:13
2.已知:$x_0=x_1=1$,$x_0x_n+x_1x_{n-1}+\cdots+x_{n-1}x_1+x_nx_0=2^nx_n (n \geqslant 2)$ 求$x_n$
kuing 2# 2013-2-26 11:00
0 和 1 的运算都被定义成 1 的,想到阶乘,再看递推式,又想到二项式,组合数,都是相关的,算了 $x_2$,随即猜到 $x_n=\dfrac1{n!}$,马上变成组合恒等式,归纳之。
hongxian 3# 2013-2-26 11:04
2# kuing 组合恒等式,高!
yes94 4# 2013-2-26 12:28
2# kuing 这个猜想(直觉)丰富啊!
yes94 5# 2013-2-28 19:31
一道类似题: 已知:$x_2=x_3=1$,$x_2x_n+x_3x_{n-1}+x_4x_{n-2}+\cdots+x_{n-1}x_3+x_nx_2=x_{n+1} (n \geqslant 2)$ 求$x_n (n \geqslant 2)$
thread-1177-1-4.html: [不等式] 又见杨学枝猜想,bao力证了先嘿嘿
kuing 1# 2013-2-26 10:16
刚才在粉丝群共享看到的。 还是用之前的老方法了,不过次数有点高,所以有点bao力。 由 Schur 不等式知右边非负,若三边中有两边相等,则左边为 $0$,此时不等式成立,当三角形为正三角形时取等。 下设三边都不相等,由对称性,不妨设 $a<b<c$,则可令 $b=a+t$, $c=a+t+u$, $t$, $u>0$,代入原不等式中等价于 \[\frac{a (a+t) (a+t+u)-(a-u) (a+u) (a+2 t+u)}{4 a (a+t) (a+t+u)+(a-u) (a+u) (a+2 t+u)}\geqslant \frac{t u (t+u)}{a (a+t) (a+t+u)},\] 去分母展开并按 $a$ 整理等价于 \begin{align*} f(a)={}&a^4(t^2+t u+u^2)+a^3(2 t^3-2 t^2 u+2 u^3)+a^2(t^4-8 t^3 u-9 t^2 u^2+u^4)\\ &+a(-4 t^4 u-6 t^3 u^2+2 t u^4)+2 t^3 u^3+3 t^2 u^4+t u^5\geqslant 0, \end{align*} 求导得 \begin{align*} f'(a)={}&4 a^3(t^2+t u+u^2)+3 a^2(2 t^3-2 t^2 u+2 u^3)+2 a(t^4-8 t^3 u-9 t^2 u^2+u^4)\\ &{}-4 t^4 u-6 t^3 u^2+2 t u^4, \end{align*} 由非钝角三角形条件知 \[a^2+b^2\geqslant c^2\iff a^2+(a+t)^2\geqslant (a+t+u)^2\iff a^2\geqslant 2au+2tu+u^2,\] 解得 \[a\geqslant u+\sqrt{2u(t+u)},\] 于是 \begin{align*} f'(a)\geqslant{}& 4 a(2au+2tu+u^2)(t^2+t u+u^2)+3 a^2(2 t^3-2 t^2 u+2 u^3)+2 a(t^4-8 t^3 u-9 t^2 u^2+u^4)\\ &{}-4 t^4 u-6 t^3 u^2+2 t u^4\\ ={}& a^2(6 t^3+2 t^2 u+8 t u^2+14 u^3)+a(2 t^4-8 t^3 u-6 t^2 u^2+12 t u^3+6 u^4)\\ &{}-4 t^4 u-6 t^3 u^2+2 t u^4\\ \geqslant{}& (2au+2tu+u^2)(6 t^3+2 t^2 u+8 t u^2+14 u^3)+a(2 t^4-8 t^3 u-6 t^2 u^2+12 t u^3+6 u^4)\\ &{}-4 t^4 u-6 t^3 u^2+2 t u^4\\ ={}& a(2 t^4+4 t^3 u-2 t^2 u^2+28 t u^3+34 u^4)+8 t^4 u+4 t^3 u^2+18 t^2 u^3+38 t u^4+14 u^5\\ >{}&0, \end{align*} 所以 \[f(a)\geqslant f\bigl(u+\sqrt{2u(t+u)}\bigr),\] 令 \[m=\sqrt{2\left( \frac tu+1 \right)}\iff t=\left( \frac{m^2}2-1 \right)u,\] 代入化简得 \[f\bigl(u+\sqrt{2u(t+u)}\bigr)=\frac{u^6}{16} m^2 (m+1)^2 (m+2)^2 (m^2-2 m-2)^2\geqslant 0,\] 从而原不等式成立。 再看看不等边时的等号成立条件,在上述所设下,取等号当且仅当 $a=u+\sqrt{2u(t+u)}$ 且 $m^2-2 m-2=0$,后者解得 $m=\sqrt3+1$,代入前者化简易得 $t=\bigl(\sqrt3-1\bigr)a$, $u=\bigl(2-\sqrt3\bigr)a$,所以此时 $a:b:c=1:\sqrt3:2$,即三角形为 $90^\circ$, $60^\circ$, $30^\circ$ 角的直角三角形。
thread-1178-1-3.html: [数列] 还来一个求递推通项
hongxian 1# 2013-2-26 18:21
本帖最后由 hongxian 于 2013-3-1 17:25 编辑 已知:$a_0=2$,$a_1=3$,$a_2=6$,且对$n \geqslant 3$有$a_n=(n+4)a_{n-1}-4na_{n-2}+(4n-8)a_{n-3}$ 求:$a_n$
realnumber 2# 2013-2-26 21:35
模仿这类问题做法$a_1=1,a_n=2a_{n-1}+1,n\ge2,n\in N$ 本题可得$a_n-(n+2)a_{n-1}+(2n-2)a_{n-2}=2(a_{n-1}-(n-1+2)a_{n-2}+(2n-2-2)a_{n-3})$ 又$a_2-4a_1+a_0=0$,所以$a_n-(n+2)a_{n-1}+(2n-2)a_{n-2}=0$, 后面你自己来吧.
kuing 3# 2013-2-26 21:37
2# realnumber niubility...我刚才看了好一会都没目测出这个变形……
realnumber 4# 2013-2-26 21:50
本帖最后由 realnumber 于 2013-2-26 21:58 编辑 3# kuing 不是目测的,是计算的. $a_n+xa_{n-1}+ya_{n-2}=k(a_{n-1}+xa_{n-2}+ya_{n-3})$,可以解得$k=2$,但$x=-n-2,y=2n-4$没法用,需要修改. 处理系数中n,然后处理系数中常数,凑巧成了. -----运气成分很大.
yes94 5# 2013-2-27 13:21
4# realnumber 战巡求出:$a_n=2^n+n!$
hongxian 6# 2013-2-27 15:27
本帖最后由 hongxian 于 2013-2-28 11:33 编辑 2# realnumber 谢谢了!好象有点问题, $a_2-4a_1+a_0=0$,好象应该改成$a_2-4a_1+2a_0=-2$,所以$a_n-(n-1)a_{n-1}+(2n-2)a_{n-2}=-2^{n-1}$,$n \geqslant 2$ 所以$a_n-na_{n-1}=2 \left[a_{n-1}-(n-1)a_{n-2}\right]-2^{n-1}$,$n \geqslant 2$ 所以$\frac{a_n-na_{n-1}}{2^{n}}=\frac{a_{n-1}-(n-1)a_{n-2}}{2^{n-1}}-\frac{1}{2}$,$n \geqslant 2$ 所以$\frac{a_n-na_{n-1}}{2^n}=\frac{2-n}{2}$ 所以$a_n-na_{n-1}=2^n-n2^{n-1}  n \in N^*$ 所以$a_n-2^n=n \left(a_{n-1}-2^{n-1}\right)=n(n-1)\left(a_{n-2}-2^{n-2}\right)=n!$ 所以$a_n=2^n+n!$,$n \in N$
hongxian 7# 2013-2-27 16:22
5# yes94 战版主的方法在这里http://bbs.pep.com.cn/forum.php? ... 4&fromuid=56513
realnumber 8# 2013-2-27 16:57
6# hongxian 恩,大意了
yes94 9# 2013-2-28 18:44
把战巡的解法编辑成好看的代码再看看,要不然原解答太难读了! $a_{n}=(n+4)a_{n-1}-4na_{n-2}+4(n-2)a_{n-3}$ $a_{n}-na_{n-1}=4a_{n-1}-4(n-1)a_{n-2}-4a_{n-2}+4(n-2)a_{n-3}$ $a_{n}-na_{n-1}=4[a_{n-1}-(n-1)a_{n-2}]-4[a_{n-2}-(n-2)a_{n-3}]$ 令$a_{n}-na_{n-1}=b_{n}$,就有 $b_{n}=4b_{n-1}-4b_{n-2}$ 这个就好办了吧,剩下楼主自己搞定吧... 最后得到$a_{n}=2^n+n!$ 当然,$Word$的查找与替换功能,还是不错的,省去我编辑的麻烦,
yes94 10# 2013-2-28 18:46
9# yes94 这么多的“4”,都视而不见!
yes94 11# 2013-2-28 19:17
楼主的条件不对,应该改为: 已知:$a_0=2$,$a_1$=3,$a_2=6$,且对$n⩾3$,有$a_n=(n+4)a_{n−1}−4na_{n−2}+(4n−8)a_{n−3}$ 求:$a_n$
hongxian 12# 2013-3-1 17:25
11# yes94 谢谢了,的确,才发现!
thread-1179-1-1.html: 请教怎么给公式整体加一个编号
abababa 1# 2013-2-26 18:47
本帖最后由 abababa 于 2013-2-26 18:50 编辑 比如 $\begin{numcases}x=1\\y=1\\z=1\end{numcases}$ 给每个公式都加上一个编号了,我想要给这一组公式只加一个编号,应该怎么加呢?
kuing 2# 2013-2-26 18:53
\begin{equation} \left\{\begin{aligned} f&=u\\ c+k&=kuing \end{aligned}\right. \end{equation}
abababa 3# 2013-2-26 18:59
2# kuing 谢谢。 之前查过百度,也是这么说的,而且还不能自动加左边那个大括号,主要想问问有没有像numcases那样的简单方式。
kuing 4# 2013-2-26 19:03
不清楚,我一直用这个,嫌输入麻烦的话自定义一个环境就行了。
thread-118-1-1.html: Maple里的判别式是按什么定义的?
kuing 1# 2011-10-18 22:22
以下结果在maple10里输出 二次的时候跟我们用开的一样,但三次和四次似乎不一样,以前用开的三次判别式都是跟何版主在网刊第五期 http://www.pep.com.cn/rjwk/gzsxs ... 0110729_1060568.htm 这里的一样,这跟Maple里的符号相反…… 呃……
①②③④⑤⑥⑦ 2# 2011-10-24 09:20
差个符号又没关系的,能判别就成 http://www.maplesoft.com/support ... w.aspx?path=discrim
①②③④⑤⑥⑦ 3# 2011-10-24 09:27
1# kuing 具体记不清了,高等代数里应该有的吧。 是一个统一的判别式,至于与最初习惯上的三次判别式差一个符号不是什么大不了的事情 Mathematica里也是一样的(Discriminant),不可能为三次弄一个特例
kuing 4# 2011-10-24 10:22
oh thanks 之前只在maple里用过判别式,mathematica里没试过呵呵 刚刚在mathematica里查了下帮助,里面也有讲
thread-1180-1-2.html: [函数] 导数 零点 不等式证明
isea 1# 2013-2-26 22:05
未深入思考,临场被后两问难住了。 出处不知,不过,题目看着像是道陈题。 方法不限,请指点。谢谢! 题:设$a\in \mathbf {R}$,函数$f(x)=\ln x-ax$. (1)若$a=2$,求$y=f(x)$在$P(1,-2)$处的切线方程. (2)若$f(x)$无零点,求实数$a$的取值范围. (3)若$f(x)$有两个相异零点$x_1,x_2$,求证:$x_1\cdot x_2>e^2$.
realnumber 2# 2013-2-26 22:22
本帖最后由 realnumber 于 2013-2-26 22:58 编辑 (2)$\ln(x)-ax=0\iff a=\frac{\ln(x)}{x}=g(x)$,这个函数值域的补集就是所求. 有时不作变形,直接用最大小于0或最小大于0也行. $g(x)$在$x\in (0,e) 递减,x\in (e,+∞)递增$,得本题$a>\frac{1}{e}$ (3)按(2)的办法先得出两根时候,$a$的取值范围$(0,\frac{1}{e})$,再从$\ln x_1=ax_1,\ln x_2=ax_2$得出吧,也许还要反证法配合. 配合$g(x)$图象,不妨设$1<x_1<e<x_2$--不想试了,好象哪里见到过.
isea 3# 2013-2-27 12:03
本帖最后由 isea 于 2013-2-27 23:02 编辑 (2)$\ln(x)-ax=0\iff a=\frac{\ln(x)}{x}=g(x)$,这个函数值域的补集就是所求. 有时不作变形,直接用最大小于0或最小大于0也行. $g(x)$在$x\in (0,e) 递减,x\in (e,+∞)递增$,得本题$a>\frac{1}{e}$ (3)按(2)的办法 ... realnumber 发表于 2013-2-26 22:22 感谢。 第(2)问如果以切线背景来看是显然的。 现在主要是第(3)了,反证法的思路似乎很赞,也好像可行,下午得空时想想,来 ==== 第(3)问,用反证法,也不太好处理。 依题,$0<a<\dfrac1e$,不防设$x_1<x_2$,则有$1<x_1<\dfrac1a<x_2$。 先假设$x_1x_2=e^2$,则由$\ln x_1=ax_1,\ln x_2=ax_2$,消$x_2$有: $ax_1^2-2x_1+ae^2=0$ 对于这个关于$x_1$的一元二次方程,由求根公式得$x_1=\dfrac{1\pm\sqrt{1-a^2e^2}}{a}\notin(1,\dfrac1a)$ ……(目测较小的根并不矛盾) 式子虽不复杂,但往下并不好找矛盾式,卡住
yes94 4# 2013-2-27 13:18
(3)构造不等式$\ln x>\dfrac{2(x-1)}{x+1}(x>1)$,
realnumber 5# 2013-2-27 15:24
4# yes94 这样不好,看不懂
isea 6# 2013-2-27 16:28
本帖最后由 isea 于 2013-2-27 18:23 编辑 (3)构造不等式$\ln x>\dfrac{2(x-1)}{x+1}(x>1)$, yes94 发表于 2013-2-27 13:18 能想到这个函数的,已经对这个题的(代数式变形的)关键点有相当深入的研究了。 至少,这个构造,偶在临场未想到的,其实,现在对这个构造具体如何证明依然不明中。 ================== 按大家的提示,及以往题的启发或者说类推,现给出第2问及第3问具体过程。 第(2)问,分析与解,realnumber 已经在2楼的一种方法与提示,这里从零点定理(另一个)角度考虑。 若$a=0$,虽然不合题设,故$a\ne0$: \begin{align*} f(x)&=\ln x-ax\\ f'(x)&=\dfrac1x-a\\ &=\dfrac{-ax+1}{x},x>0\\ f'(x)&=0\\ x&=\dfrac1a\\ \end{align*} 简单讨论知 $a<0,f(x)$在$(0,+\infty)$上单调递增;                 $a>0,f(x)$在$(0,\dfrac1a)$上单调递增,$f(x)$在$(\dfrac1a,+\infty)$上单调递减。 下面说明当$a<0,f(x)$在$(0,+\infty)$上,只有惟一的零点,也就是: \begin{align*} f(1)&=-a>0\\ f(e^a)&=a-ae^a\\ &=a(1-e^a)<0\\ \end{align*} 从而$a<0$不合题设。 PS:$f(e^a)$这个当时卡着了,今天才想到,其实在本区曾经也有个类似这楼的处理。 下面进一步缩小$a$的范围。 当$a>0$时,$f(x)$无零点$\Leftrightarrow f(x)_{max}<0 \Leftrightarrow f(\dfrac1a)<0 \Leftrightarrow a>\dfrac1e$,这便是第(2)问结果。 PPS:这一点从$\ln x=ax$两图象的交点来看,是极明显的。
realnumber 7# 2013-2-27 16:54
本帖最后由 realnumber 于 2013-2-27 18:42 编辑 (3)因为$1<x_1<e<x_2$,当$x_2\ge e^2$时,$x_1x_2>e^2$显然成立.所以只需要证明$e<x_2<e^2$情景,根据这个范围$x_1$也相应有更小氛围. \[函数g(x)=\frac{\ln x}{x} 在(1,e)递增,在(e,e^2)递减\] \[a=\frac{\ln x_1}{x_1}=\frac{\ln x_2}{x_2}>\frac{2}{e^2}\] 在$x=1$切线展开,和$x≠1$展开,那么会导致$x_1,x_2在x=e$附近时,无法证明$x_1x_2>e^2$.也就是说需要另外想办法.
isea 8# 2013-2-27 17:28
本帖最后由 isea 于 2013-2-27 18:25 编辑 第(3)问,分析与解,仅供参考,偶还未吃透个中原理。 (偶对不等式很弱),由第(2)问,加动笔简单演算便知,此时条件明显的有: $0<a<\dfrac1e,\ln x_1-ax_1=0,\ln x_2-ax_2=0$,而结论可以转化为$a(x_1+x_2)>2$。 这里的$a$处理特别让人头大,条件与结论很难挂上。 临场由于时间限制,直接跳过,未深入思考。 今天下午再提笔,想转化为函数+导数,无果; 后,请教身边朋友,知,此题是2012年广东高三一道模拟题,有标答,便看了一眼,标答关键点便是yes94兄点化的差不多的式子。 当年原标答,最后会附上,为了不影响看帖者的思路,现在不给出; 对标答这种代数转化技巧高;而此题化来化去,似乎都逃不掉$a$!又想从高等数学方面看看,有没直接用的结果,如什么什么展开式之类。 受一陈题的启发,这里从构造函数不等式方面试试,先,不论结果如何。 回到题目:设$a\in \mathbf {R}$,函数$f(x)=\ln x-ax$.(3)若$f(x)$有两个相异零点$x_1,x_2$,求证:$x_1\cdot x_2>e^2$. 下面先证明 $$a>0,0<x<\dfrac1a,f(\dfrac1a+x)>f(\dfrac1a-x)$$ 令 \begin{align*} F(x)&=f(\dfrac1a+x)-f(\dfrac1a-x)\\ &=\ln(\dfrac1a+x)-\ln(\dfrac1a-x)-2ax\\ F'(x)&=\dfrac{2a^3x^2}{(1-ax)(1+ax)}\\ &>0\\ \end{align*} 即 $F(x)$在$(0,\dfrac1a)$单调递增,注意到$F(0)=0$,故$F(x)>0$, 亦即 $a>0,0<x<\dfrac1a,f(\dfrac1a+x)>f(\dfrac1a-x)$。 若 $f(x)$有两个相异零点$x_1,x_2$,则由第(2)问知 $a>0$且$f(\dfrac1a)>0\Leftrightarrow0<a<\dfrac1e,f(x_1)=f(x_2)=0,0<x_1,x_2$。 不防设$x_1<x_2$,则有$0<x_1<\dfrac1a<x_2$,而$0<\dfrac1a-x_1<\dfrac1a$: \begin{align*} f(\dfrac1a+(\dfrac1a-x_1))&>f(\dfrac1a-(\dfrac1a-x_1))=f(x_1)=f(x_2)=0\\ f(\dfrac2a-x_1)&>0;x\in(x_1,x_2),f(x)>0\\ \dfrac2a-x_1&<x_2\\ \dfrac2a&<x_1+x_2\\ ax_1+ax_2&>2\\ \ln x_1+\ln x_2&>2\\ \ln(x_1x_2)&>2\\ x_1x_2&>e^2\\ \end{align*} 嗬,还真绕出来了,$\dfrac2a-x_1<x_2$是关键啊。 PPPS:这里主要考虑是两函数$\ln x,ax$的差,realnumber 用是商的变式,看来是异曲同工。 最后,从这种做法,似乎有函数凹凸性的感觉,因为不熟悉,不知道是否从函数凹凸性手入呢?
yes94 9# 2013-2-27 18:01
第(2)问的解答常规解法应该差不多是都是求导来做的,下面不求导来做它! 搞一个第(2)问的解答:    由于$\ln x\leqslant x-1$,当且仅当$x=1$取等号,故$\ln ax\leqslant ax-1$,当且仅当$ax=1$取等号。    于是$\ln a+\ln x\leqslant ax-1$,即$f(x)=\ln x-ax\leqslant -1-\ln a$,当且仅当$x=\dfrac1a$取等号。     若$f(x)$无零点,则需且只需$-1-\ln a<0$,故$a>\dfrac1e$ 看一哈,对不对?
yes94 10# 2013-2-27 18:04
9# yes94 当然,最好还是讨论一下$a\leqslant0$不符合要求,当然这是显然的了。
yes94 11# 2013-2-27 18:19
8# isea 你喜欢研究第(3)问,那我继续第(2)问:     作函数$y_1=\ln x$的图像,$y_2=ax$的图像,下面求$y_2$和$y_1$的图像相切时的斜率值$k$,则第(2)问答案就是$a>k$。     设切点为$(x_0,y_0)$,因为$(y_1)'=\dfrac1x$,所以切线的斜率$k=\dfrac1{x_0}$,切线方程为$y-y_0=\dfrac1{x_0}(x-x_0)$,因为此切线过原点,于是$y_0=1$,即$\ln x_0=1$,$x_0=e$,$k=\dfrac1{x_0}=\dfrac1{e}$。      故第(2)问答案就是$a>\dfrac1{e}$。
isea 12# 2013-2-27 18:22
11# yes94 你要是有空的话,教教我,怎么用你提到的函数不等式呢?
yes94 13# 2013-2-27 18:34
12# isea 我在导数方面的爱好不是很强烈,这点和kuing一样,所以他基本懒得做导数题,他觉得不等式要过瘾些! 所以我就没的什么教你的啊?估计yezhu对导数情有独钟,要问你就问下他吧。 况且你是初中、高中都通吃(全面发展)的人物,还让教,那不是笑话么? 还有你的那个第(3)问的解析,我也想过的,但最终还是走到另外一条路去了。
abababa 14# 2013-2-27 20:20
本帖最后由 abababa 于 2013-2-27 20:24 编辑 我也试试。 不妨设$x_1>x_2$ $\ln x_1-ax_1=\ln x_2-ax_2$,这就得到了$a=\frac{\ln\frac{x_1}{x_2}}{x_1-x_2}$ 作和得$\ln x_1x_2=a(x_1+x_2)$ 然后由要证的$\ln x_1x_2>\ln e^2=2$得到只要证$a(x_1+x_2)>2$,然后把a代进去,就是要证$(x_1+x_2)\frac{\ln\frac{x_1}{x_2}}{x_1-x_2}>2$,约去$x_2$再把$\frac{x_1}{x_2}$看成变量,然后就是要证$(t+1)\ln t>2(t-1), (t>1)$,$t=1$是$F(t)=(t+1)\ln t-2(t-1)$的零点,只要证明$F(t)$单调,就是证明$F'=\ln t+1/t-1>0$,就是证明$\frac{x}{x+1}+\frac{1}{x+1}-1=0$,最后这个成立
isea 15# 2013-2-27 21:31
14# abababa 13楼的yes94太客气、谦虚。互相学习。 === 楼上的证明方式便偶见到的标答模式,只是这个$a$的代换,实在是巧,得到“齐次”式,化多元为一元,必然。 其次,如果第(3)要用反证法,如何做呢? 顺便附上标答:
realnumber 16# 2013-2-27 21:53
本帖最后由 realnumber 于 2013-2-27 21:58 编辑 (3) \[函数g(x)=\frac{\ln x}{x} 在(1,e)递增,在(e,+∞)递减\] \[1<x_1<e<x_2,a=\frac{\ln x_1}{x_1}=\frac{\ln x_2}{x_2},求证:x_1x_2>e^2\] 换元$t_1=\frac{x_1}{e},t_2=\frac{x_2}{e}$,那么以上结论转化为 \[函数h(t)=\frac{1+\ln t}{t} 在(0,1)递增,在(1,+∞)递减\] \[0<t_1<1<t_2,ae=\frac{1+\ln t_1}{t_1}=\frac{1+\ln t_2}{t_2},求证:t_1t_2>1\] 先用导数证明下列不等式(实为泰勒级数在t=1处展开,保留到2次项,求导二次后可证) 当$t>1$时,$\ln t>(t-1)-0.5(t-1)^2$;当$t<1$时,$\ln t<(t-1)-0.5(t-1)^2$; \[那么\frac{t_1-0.5(t_1-1)^2}{t_1}>\frac{1+\ln t_1}{t_1}=\frac{1+\ln t_2}{t_2}>\frac{t_2-0.5(t_2-1)^2}{t_2}\] 化简结果就是$t_1t_2>1$,完.
abababa 17# 2013-2-27 22:50
15# isea 答案是把t+1除下来了,然后求导就好算了,我做的时候没注意t>1,是直接求的导,最后一步还用了$\frac{x}{1+x}<\ln (x+1)$代换,显得麻烦了。
isea 18# 2013-2-27 23:42
本帖最后由 isea 于 2013-2-28 00:30 编辑 第(2)问的解答常规解法应该差不多是都是求导来做的,下面不求导来做它! 搞一个第(2)问的解答:    由于$\ln x\leqslant x-1$,当且仅当$x=1$取等号,故$\ln ax\leqslant ax-1$,当且仅当$ax=1$取等号。    于 ... yes94 发表于 2013-2-27 18:01 9楼这个写法很流畅啊。 (3) \[函数g(x)=\frac{\ln x}{x} 在(1,e)递增,在(e,+∞)递减\] \[1 realnumber 发表于 2013-2-27 21:53 借用几何画板观察了$g(x)=\dfrac{\ln x}{x}$图象,移除泰勒级数,接16楼前两行,作如下修改成高中方案: $x_1x_2>e^2 \Leftrightarrow x_2>\dfrac{e^2}{x_1}$ 下面证$g(\dfrac{e^2}{x_1})>a=\dfrac{\ln{x_1}}{x_1}$成立. \begin{align*} g(\dfrac{e^2}{x_1})&>\dfrac{\ln{x_1}}{x_1}\\ \Leftrightarrow \dfrac{\ln \dfrac{e^2}{x_1}}{\dfrac{e^2}{x_1}}&>\dfrac{\ln{x_1}}{x_1}\\ \Leftrightarrow\ln x_1-\dfrac{2x_1^2}{x_1^2+e^2}&<0,1<x_1<e\\ \end{align*} 而$F(x)=\ln x-\dfrac{2x^2}{x^2+e^2}$,在$x\in(1,e)$上单调递增,注意到$F(e)=1-1=0$, 故$1<x<e,F(x)=\ln x-\dfrac{2x^2}{x^2+e^2}<0$,即$\ln x_1-\dfrac{2x_1^2}{x_1^2+e^2}<0\Leftrightarrow g(\dfrac{e^2}{x_1})>a=\dfrac{\ln{x_1}}{x_1}$成立。 考虑到 $x\in(x_1,x_2),g(x)>a$,于是$x_1<\dfrac{e^2}{x_1}<x_2\Rightarrow x_1x_2>e^2$
yes94 19# 2013-2-28 19:38
标答出来了啊? 第二问两种方法都是求导。
isea 20# 2013-4-14 21:56
19# yes94 是啊,后来找到出处了。 另外,今天又找到个类题 http://kkkkuingggg.5d6d.net/thread-1360-1-1.html
thread-1180-2-2.html:
转化与化归 21# 2013-4-14 23:31
弄来弄去,到头来还是一个题,综上所述:消参数a
yes94 22# 2013-4-15 21:54
弄来弄去,到头来还是一个题,综上所述:消参数a 转化与化归 发表于 2013-4-14 23:31 看来,万变不离其宗啊! 齐次法,很好的,
hongxian 23# 2013-4-17 18:18
第三问是不是可以加强为求证:$x_1 \cdot x_2>\dfrac{1}{a^2}$
三下五除二 24# 2013-4-24 14:39
不是加强,而是本质上就是你说的
thread-1181-1-3.html: 转,一个初中方程题
realnumber 1# 2013-2-27 08:44
本帖最后由 realnumber 于 2013-2-27 09:07 编辑 $\sqrt{(2010-a)^2}+\sqrt{a-2011}=a$ 求$a-2011^2$的值. 因为条件中有$\sqrt{a-2011}$,所以$a\ge 2011$,那么条件为$a-2010+\sqrt{a-2011}=a$, 化简得$a-2011=2010^2\iff a=2011+2010^2$ 所以$a-2011^2=2011+2010^2-2011^2=2011+(2010-2011)\times(2010+2011)=-2010$
isea 2# 2013-2-27 18:28
常规题中求$a-2010^2$,较大数字随年号变化……
yes94 3# 2013-2-28 19:41
2# isea 以前初中竞赛题的传统:常常有一道题和竞赛题的举行年份有关
kuing 4# 2013-2-28 22:07
3# yes94 这应该不止初中的吧……
yes94 5# 2013-2-28 23:40
4# kuing 高中的? 即便如此,我说的是以前……
thread-1182-1-3.html: [数列] 昨天早上粉丝群这道数列题怎么玩[6#反证应该没问题]
kuing 1# 2013-2-27 08:47
唯因却恩(7643*****)  8:28:35 第二问帮看看 $a_1=1$, $a_{2k}=-a_k$, $a_{2k-1}=(-1)^{k+1}a_k$,证 $S_n\geqslant0$。 昨天看了一下,不会,后来研究别的东西去了,刚才再看,还是不会 感觉有点意思,先转发上来先。
kuing 2# 2013-2-27 08:51
用程序列了一下 an 和 Sn a[1] = 1; Do[Which[EvenQ[n], a[n] = -a[n/2], OddQ[n],   a[n] = (-1)^((n + 3)/2) a[(n + 1)/2]], {n, 2, 1000}] S[n_] := Sum[a[k], {k, 1, n}] Table[a[n], {n, 1, 100}] {1, -1, 1, 1, 1, -1, -1, -1, 1, -1, 1, 1, -1, 1, 1, 1, 1, -1, 1, 1, 1, -1, -1, -1, -1, 1, -1, -1, 1, -1, -1, -1, 1, -1, 1, 1, 1, -1, -1, -1, 1, -1, 1, 1, -1, 1, 1, 1, -1, 1, -1, -1, -1, 1, 1, 1, 1, -1, 1, 1, -1, 1, 1, 1, 1, -1, 1, 1, 1, -1, -1, -1, 1, -1, 1, 1, -1, 1, 1, 1, 1, -1, 1, 1, 1, -1, -1, -1, -1, 1, -1, -1, 1, -1, -1, -1, -1, 1, -1, -1} Table[S[n], {n, 1, 600}] {1, 0, 1, 2, 3, 2, 1, 0, 1, 0, 1, 2, 1, 2, 3, 4, 5, 4, 5, 6, 7, 6, 5, 4, 3, 4, 3, 2, 3, 2, 1, 0, 1, 0, 1, 2, 3, 2, 1, 0, 1, 0, 1, 2, 1, 2, 3, 4, 3, 4, 3, 2, 1, 2, 3, 4, 5, 4, 5, 6, 5, 6, 7, 8, 9, 8, 9, 10, 11, 10, 9, 8, 9, 8, 9, 10, 9, 10, 11, 12, 13, 12, 13, 14, 15, 14, 13, 12, 11, 12, 11, 10, 11, 10, 9, 8, 7, 8, 7, 6, 5, 6, 7, 8, 7, 8, 7, 6, 7, 6, 5, 4, 5, 4, 5, 6, 7, 6, 5, 4, 3, 4, 3, 2, 3, 2, 1, 0, 1, 0, 1, 2, 3, 2, 1, 0, 1, 0, 1, 2, 1, 2, 3, 4, 5, 4, 5, 6, 7, 6, 5, 4, 3, 4, 3, 2, 3, 2, 1, 0, 1, 0, 1, 2, 3, 2, 1, 0, 1, 0, 1, 2, 1, 2, 3, 4, 3, 4, 3, 2, 1, 2, 3, 4, 5, 4, 5, 6, 5, 6, 7, 8, 7, 8, 7, 6, 5, 6, 7, 8, 7, 8, 7, 6, 7, 6, 5, 4, 3, 4, 3, 2, 1, 2, 3, 4, 5, 4, 5, 6, 5, 6, 7, 8, 9, 8, 9, 10, 11, 10, 9, 8, 9, 8, 9, 10, 9, 10, 11, 12, 11, 12, 11, 10, 9, 10, 11, 12, 13, 12, 13, 14, 13, 14, 15, 16, 17, 16, 17, 18, 19, 18, 17, 16, 17, 16, 17, 18, 17, 18, 19, 20, 21, 20, 21, 22, 23, 22, 21, 20, 19, 20, 19, 18, 19, 18, 17, 16, 17, 16, 17, 18, 19, 18, 17, 16, 17, 16, 17, 18, 17, 18, 19, 20, 19, 20, 19, 18, 17, 18, 19, 20, 21, 20, 21, 22, 21, 22, 23, 24, 25, 24, 25, 26, 27, 26, 25, 24, 25, 24, 25, 26, 25, 26, 27, 28, 29, 28, 29, 30, 31, 30, 29, 28, 27, 28, 27, 26, 27, 26, 25, 24, 23, 24, 23, 22, 21, 22, 23, 24, 23, 24, 23, 22, 23, 22, 21, 20, 21, 20, 21, 22, 23, 22, 21, 20, 19, 20, 19, 18, 19, 18, 17, 16, 15, 16, 15, 14, 13, 14, 15, 16, 15, 16, 15, 14, 15, 14, 13, 12, 11, 12, 11, 10, 9, 10, 11, 12, 13, 12, 13, 14, 13, 14, 15, 16, 15, 16, 15, 14, 13, 14, 15, 16, 15, 16, 15, 14, 15, 14, 13, 12, 13, 12, 13, 14, 15, 14, 13, 12, 11, 12, 11, 10, 11, 10, 9, 8, 9, 8, 9, 10, 11, 10, 9, 8, 9, 8, 9, 10, 9, 10, 11, 12, 13, 12, 13, 14, 15, 14, 13, 12, 11, 12, 11, 10, 11, 10, 9, 8, 7, 8, 7, 6, 5, 6, 7, 8, 7, 8, 7, 6, 7, 6, 5, 4, 5, 4, 5, 6, 7, 6, 5, 4, 3, 4, 3, 2, 3, 2, 1, 0, 1, 0, 1, 2, 3, 2, 1, 0, 1, 0, 1, 2, 1, 2, 3, 4, 5, 4, 5, 6, 7, 6, 5, 4, 3, 4, 3, 2, 3, 2, 1, 0, 1, 0, 1, 2, 3, 2, 1, 0, 1, 0, 1, 2, 1, 2, 3, 4, 3, 4, 3, 2, 1, 2, 3, 4, 5, 4, 5, 6, 5, 6, 7, 8, 9, 8, 9, 10, 11, 10, 9, 8, 9, 8, 9, 10, 9, 10, 11, 12, 13, 12, 13, 14, 15, 14, 13, 12}
joatbmon 3# 2013-2-27 14:36
看上去很复杂,我感觉是不是用二进制来解释的?
yes94 4# 2013-2-28 19:57
$a_n$的通项数字全是$1$,$-1$的,realnumber的观察归纳能力强,realnumber出马试试归纳看?
goft 5# 2013-2-28 20:44
本帖最后由 goft 于 2013-2-28 21:18 编辑 搞不定
realnumber 6# 2013-2-28 22:02
本帖最后由 realnumber 于 2013-3-3 08:17 编辑 问题:$a_1=1$, $a_{2k}=-a_k$, $a_{2k-1}=(-1)^{k+1}a_k$,证 $S_n\geqslant0$。 解:$a_{4k}=-a_{2k}=a_k,a_{4k-2}=-a_{2k-1},a_{4k-3}=a_{2k-1},a_{4k-1}=-a_{2k}=a_k$,得到$S_{4k}=2S_k$. 易得$S_1,S_2,S_3,S_4\ge0$. 假设使得$S_n<0$的最小$n=n_0$(容易得出$S_{n_0}=-1,S_{n_0-1}=0,S_{n_0-2}=1,a_{n_0}=-1,a_{n_0-1}=-1$), 1.若$n_0=4k$,由$S_{4k}=2S_k<0$,与$4k$最小矛盾. 2.若$n_0=4k+2$,$S_{4k+2}=S_{4k}+a_{4k+1}+a_{4k+2}=S_{4k}<0$,这与$4k+2$最小矛盾. 3.若$n_0=4k+3$,$S_{4k+3}=S_{4k}+a_{4k+1}+a_{4k+2}+a_{4k+3}=S_{4k}+a_{4k+3}=2S_{k}+a_{k+1}=S_{k+1}<0$,这与$4k+3$最小矛盾.(说明$a_{4k+1}+a_{4k+2}=-a_{2k-1}+a_{2k-1}=0,a_{4k+3}=a_{k+1}=-1,S_{4k}=0=S_k$.) 4.若$n_0=4k+1$,则$a_{4k+1}=-1,S_{4k+1}=-1,S_{4k}=0=S_{k}$,如此,推测出$k$必定为偶数(因为{$a_n$}由$1,-1$组成,若$k$为奇数,则$S_{k}$不为零.ps,也可以用这个办法来处理1.2.两条.),$a_{4k+1}=-1=a_{2k+1}=(-1)^ka_{k+1}=a_{k+1}$,那么$S_{k+1}=S_{k}+a_{k+1}=-1<0$,与$4k+1$最小矛盾. ---又,其实k发的时候就看到,没想到怎么入手.
realnumber 7# 2013-3-2 23:05
4# yes94 终于解决,请检查~~.
kuing 8# 2013-3-3 00:31
初步看了下,没看出问题来。明天再仔细看,先闪了
yes94 9# 2013-3-3 16:22
7# realnumber 虽然看起来吓人,但要赞一个先,这种专研精神值得学习!
kuing 10# 2013-3-3 22:40
9# yes94 你看有没有问题?
thread-1183-1-4.html: 求$2^{sinx}+2^{cosx}$的最值
goft 1# 2013-2-27 19:22
本帖最后由 goft 于 2013-2-27 19:32 编辑 求$ 2^{sinx}+2^{cosx} $的最值. 求不等式解法。 来自:http://tieba.baidu.com/p/2185144500
kuing 2# 2013-2-27 19:27
代码没写对 发错版了
goft 3# 2013-2-27 19:30
代码没写对 发错版了 kuing 发表于 2013-2-27 19:27 想学习下程序,结果发到这里来了……
kuing 4# 2013-2-27 19:31
我移一下 代码应该为 2^{\sin x}+2^{\cos x} 效果 $2^{\sin x}+2^{\cos x}$
goft 5# 2013-2-27 19:33
4# kuing 已经更正,感谢提醒。新手
kuing 6# 2013-2-28 00:47
我能想到的大概在原贴里都有了…… 主要还是最大值,不等式方法,不好弄…… PS、代码其实还是有点不同
yes94 7# 2013-2-28 18:00
6# kuing 原贴里的解法看都看不清楚,不知写的什么天书?
thread-1184-1-4.html: [数列] 继续问数列求通项,谢谢了!
hongxian 1# 2013-2-27 20:55
4.已知:$a_1=1$,$a_{n+1}a_n-2n^2(a_{n+1}-a_n)+1=0$,求$a_n$
kuing 2# 2013-2-27 21:13
楼主那么多数列题 都是来自在哪里的啊?
hongxian 3# 2013-2-27 21:18
2# kuing 有一个学生在外面上竞赛班,在他的试卷上看到的。
realnumber 4# 2013-2-27 22:51
计算了前几项,发现依次是1,3,5,因此猜测$a_n=2n-1$,代入检验成立,可用数学归纳法证明. 或作代换$a_n=b_n+2n-1$,可以用反证法(配合无穷递降),说明任意$n,b_n=0$. ps:楼主对数列题目应该先特殊到一般实验下,...
hongxian 5# 2013-2-27 22:59
4# realnumber 先猜再证,应该的,谢谢了!
yes94 6# 2013-2-28 19:10
5# hongxian 设$a_n=kn+b$,则$a_{n+1}a_n=n^2(a_{n+1}-a_n)^2+k(k+2b)n+b(k+b)$,于是得到下面的题目: (1)已知$a_1=2,a_{n+1}a_n=4n^2(a_{n+1}-a_n)-4$,求$a_n$; (2)已知$a_1=2,a_{n+1}a_n=16n^2-4$,求$a_n$; (3)已知$a_1=2,a_{n+1}a_n=n^2(a_{n+1}-a_n)^2-4$,求$a_n$.
thread-1185-1-4.html: [数论] (zt)是否都为有理数
realnumber 1# 2013-2-27 23:00
本帖最后由 realnumber 于 2013-2-27 23:11 编辑 **(49***54)  22:53:56 应该用反证法,但是我没证出来,请问有人能帮忙做下吗? 是否存在$x$,使得$\tan x+\sqrt 3,\cot x+\sqrt 3$都为有理数.
realnumber 2# 2013-2-27 23:05
1# realnumber $\tan x +\sqrt3=m,\cot x+\sqrt3=n,m,n\in Q$,那么由$\tan x \cot x=1$ 得到$mn+2=\sqrt3(m+n)$,矛盾.
thread-1186-1-3.html: [不等式] 天书问的一条不等式
pxchg1200 1# 2013-2-28 13:05
Let$a,b,c>0$with  $a+b+c=1$,show that: \[ \frac{a^{2}}{b+b^2}+\frac{b^2}{c+c^2}+\frac{c^2}{a+a^2}\geq \frac{3}{4} \] Proof: Rewrite the inequality into: \[ \frac{a^2}{2b^2+ab+bc}+\frac{b^2}{2c^2+ca+cb}+\frac{c^2}{2a^2+ab+ac}\geq \frac{3}{4} \] Using the Awesome CYH skill,we have \[ \left(\sum{\frac{a^2}{2b^2+ab+bc}} \right)\left(\sum{(2a+b)^{2}(2b^2+ab+bc)} \right)\geq \left( 2\sum{a^2}+\sum{ab}  \right)^{2} \] So,just check \[ 4\left( 2\sum{a^2}+\sum{ab}  \right)^{2}\geq 3\left(\sum{(2a+b)^{2}(2b^2+ab+bc)} \right) \] Or \[ 10\sum{a^4}+\sum{a^3b}\geq 11\sum{a^3c} \] Which is obvious true. Done!
kuing 2# 2013-2-28 13:43
跟之前这道题 http://kkkkuingggg.5d6d.net/thread-645-1-1.html 很像,方法也是 你说会不会有 \[\sum\frac{a^2}{b+b^2}\geqslant\sum\frac{a^2}{a+b^2}\geqslant\frac34?\]
goft 3# 2013-2-28 18:27
本题排序不等式较为简便,对称轮换
yes94 4# 2013-2-28 18:37
本题排序不等式较为简便,对称轮换 goft 发表于 2013-2-28 18:27 过程?
kuing 5# 2013-2-28 18:41
4# yes94 大概是 $\{a^2,b^2,c^2\}$ 与 $\left\{\frac1{a+a^2},\frac1{b+b^2},\frac1{c+c^2}\right\}$ 必成反序。
goft 6# 2013-2-28 18:43
程序写不出来啊……,还在练习阶段
kuing 7# 2013-2-28 18:49
4# yes94 大概是 $\{a^2,b^2,c^2\}$ 与 $\left\{\frac1{a+a^2},\frac1{b+b^2},\frac1{c+c^2}\right\}$ 必成反序。 kuing 发表于 2013-2-28 18:41 不过这样下去就会反向了,看来也不是这个意思。 楼上实际操作过没的?
yes94 8# 2013-2-28 18:50
6# goft 一个星期就差不多练会了基本的,
goft 9# 2013-2-28 19:41
本帖最后由 goft 于 2013-2-28 19:55 编辑 7# kuing 晕了,计算错误,代码又弄不起来
thread-1187-1-1.html: px进
kuing 1# 2013-2-28 15:11
\begin{align} f(x)&=ax^2+bx+c\notag\\ &=a(x-x_1)(x-x_2)\notag\\ &=a\left(x-\frac{-b+\sqrt{b^2-4ac}}{2a}\right)\left(x-\frac{-b-\sqrt{b^2-4ac}}{2a}\right). \end{align}
李斌斌755 2# 2013-5-18 15:19
本帖最后由 李斌斌755 于 2013-5-20 14:58 编辑 这个应该不难 \[\begin{align}f(x)&=ax^2+bx+c\notag\\&=a(x-x_1)(x-x_2)\notag\\&=a\left(x-\dfrac{-b+\sqrt{b^2-4ac}}{2a}\right)\left(x-\dfrac{-b-\sqrt{b^2-4ac}}{2a}\right)\end{align}\] \[\begin{align}f(x)&=ax^2+bx+c\notag\\&=a(x-x_1)(x-x_2)\notag\\&=a\left(x-\dfrac{-b+\sqrt{b^2-4ac}}{2a}\right)\left(x-\dfrac{-b-\sqrt{b^2-4ac}}{2a}\right)\end{align}\] 奇怪两代码对了几遍,一样的,怎么一个显示,一个不显示。
kuing 3# 2013-5-20 14:52
\[\begin{align}f(x)&=ax^2+bx+c\notag\\&=a(x-x_1)(x-x_2)\notag\\&=a\left(x-\dfrac{-b+\sqrt{b^2-4ac}}{2a}\right)\left(x-\dfrac{-b-\sqrt{b^2-4ac}}{2a}right)\end{align}\] \[\begin{align}f(x)&=ax^2+bx+c\notag\\&=a(x-x_1)(x-x_2)\notag\\&=a\left(x-\dfrac{-b+\sqrt{b^2-4ac}}{2a}\right)\left(x-\dfrac{-b-\sqrt{b^2-4ac}}{2a}\right)\end{align}\] 奇怪两代码对了几遍,一样的,怎么一个显示,一个不显示。 李斌斌755 发表于 2013-5-18 15:19 明明第一个的 right 前面没有 \
李斌斌755 4# 2013-5-20 15:00
3# kuing 是,刚发现
thread-1188-1-3.html: [不等式] 昨晚天书问那道根式不等式原来还算松动
kuing 1# 2013-3-1 00:34
天书(1846******) 01:22:34 \[\sum\sqrt{\frac{2x(x+y+z)}{(x+y)(x+z)}}\le\sum\sqrt{\frac{3(y+z)}{2x+y+z}}\] 这里当然 $x$, $y$, $z>0$。 由柯西有 \begin{align*} \left( \sum \sqrt{\frac{2x(x+y+z)}{(x+y)(x+z)}} \right)^2&=\frac{2\sum x}{\prod(x+y)}\left( \sum \sqrt{x(y+z)} \right)^2 \\ & \leqslant \frac{6\sum x\sum x(y+z)}{\prod(x+y)} \\ & =\frac{12\sum x\sum xy}{\prod(x+y)}, \end{align*} 由 Holder 有 \[\left( \sum \sqrt{\frac{y+z}{2x+y+z}} \right)^2\sum (y+z)^2(2x+y+z)\geqslant \left( \sum (y+z) \right)^3,\] 即 \[\left( \sum \sqrt{\frac{3(y+z)}{2x+y+z}} \right)^2\geqslant \frac{24\left( \sum x \right)^3}{\sum (y+z)^2(2x+y+z)},\] 因此只要证 \[\frac{2\left( \sum x \right)^2}{\sum (y+z)^2(2x+y+z)}\geqslant \frac{\sum xy}{\prod(x+y)},\] 记 $p=\sum x$, $q=\sum xy$, $r=xyz$,上式可写成 \[f(r)=\frac{2p^2}{2p^3-pq+3r}-\frac q{pq-r}\geqslant 0,\] 显然关于 $r$ 递减,因此由 Schur 不等式有 \[f(r)\geqslant f\left( \frac{4pq-p^3}9 \right)=\frac{3(p^2-3q)(2p^2+q)}{p(5p^2+q)(p^2+5q)}\geqslant 0,\] 从而原不等式成立。
kuing 2# 2013-3-1 00:38
竟然直接用 CS and Holder 就把根号去掉了……汗啊 PS、此不等式等价于三角形中的 \[\sum\cos\frac A2\leqslant\sqrt{\frac32}\sum\sqrt{\frac a{b+c}}.\]
thread-1189-1-1.html: 求验证:薪酬最小二乘法算例
童光红 1# 2013-3-1 09:39
本帖最后由 童光红 于 2013-3-1 09:41 编辑 以下这道题是我自己做的,不知道整个计算过程对不对,请各位老师给予指导一下,谢谢 设岗位评估点数为X,薪酬市场水平为Y,求当岗位评估点数为100的时候,该岗位的薪酬市场水平(即中位值)为多少? -------------------------------------------------------------------------------------------------------------- 岗位        X      Y         XY         X平方 岗位1      50    1000      50000       2500 岗位2     150    2000      300000      22500 N=2     ∑X=200 ∑Y=3000 ∑XY=350000∑X平方=25000 Y=BX+A(A、B为两个实数) ∑Y=NA+B∑X                      (1) ∑XY=A∑Y+B∑X平方               (2) 将上述数据代入方程(1),可得: 3000=2A+200B   A=100B-1500       (3) 将上述数据代入方程代入方程(2)得: 350000=3000A+25000B              (4) 将方程(3)代入方程(4),可得: 350000=3000(100B-1500)+25000B 300000B-4500000+25000B=350000 300000B+25000B=350000+4500000 325000B=4850000 B=14.92=15(14.92的四舍五入)      (5) 将方程(5)代入方程(3),可得: A=100×15-1500 A=0 当岗位评估点数X=100的时候,该岗位的薪酬市场水平就为 Y=BX+A=15×100+0=1500
kuing 2# 2013-3-1 10:18
才两个点?那还用最小二乘?直接过两点的直线……
realnumber 3# 2013-3-1 10:21
1楼,很想知道你做这问题的动机,仅仅是提高数学水平,还是应付考试,还是解决一个实用问题.
童光红 4# 2013-3-1 11:21
我是一位读到高二上学期就辍学的人,当时由于家境不好,所以对高中数学不是很懂,结婚后到江南浙江义乌打工,鬼使神差地干了人力资源管理至今,人力资源管理又错不开员工薪酬设计这一重要环节,近来遇到关于宽带薪酬这一新型薪酬体系,因为宽带薪酬设计里关于曲线方程最小二乘法是怎么也错不过去的一道坎,由于我过去没有接触到这一类的知识,所以就被卡住了,现在我能知道的一点点皮毛,仅限于我自学所得。烦请请各位老师给予指导,辛苦了,谢谢。
童光红 5# 2013-3-1 11:24
我的意思是上述我算的过程对不对?因为自己心里没底,不敢定位,仅此而已。如果我算错了,错在哪个环节,怎么纠正?算式过程是怎么样的?
realnumber 6# 2013-3-1 11:50
是用来分析现有的员工薪酬?还是调整薪酬?还是新岗位确定薪酬?
童光红 7# 2013-3-1 12:28
我是用来分析薪酬市场水平的
realnumber 8# 2013-3-1 13:03
用直线模拟有些奇怪,为什么不是2次函数或指数呢?还有薪资高低与什么量成近似的线形关系.
realnumber 9# 2013-3-1 20:41
岗位评估点,是什么意思,? 估计问题解决了,就不会来看了.
童光红 10# 2013-3-2 11:03
9# realnumber 不会的,我一直在看,一边工作一边自学的。 岗位评估:就是通过对某个岗位(非个人)多方面的分析,评定其对企业价值的大小和重要性的高低。
童光红 11# 2013-3-2 11:05
9# realnumber 岗位价值评估又称职位价值评估或工作评价,是指在工作分析的基础上,采取一定的方法,对岗位在组织中的影响范围、职责大小、工作强度、工作难度、任职条件、岗位工作条件等等特性进行评价,以确定岗位在组织中的相对价值,并据此建立岗位价值序列的过程。 岗位价值评估是指一组评价人员根据岗位价值模型的评价标准,对各岗位完成岗位职责而且对企业贡献价值的大小进行分析和量化评估的一种管理活动。
realnumber 12# 2013-3-2 13:54
10# 童光红 谢谢,希望互补,学数学知道相关应用对教学和视界有帮助. 有用才会去学.虽然有用也是倍受争议.
thread-119-1-1.html: 如何自行删除贴子或附件
kuing 1# 2011-10-18 23:11
一、自行删贴 对于回贴,或者是还没人回复过的主题,均可以自行删除。 方法:对要删的贴,点击编辑,留意右上角,能删的,右上方会有一个 ,勾上“删?”前面的框后,点击“编辑贴子”,OK。 二、删除附件 当然也是先点编辑,进入后留意下图的右方 那些叉叉就是删除的按钮,如果是换新附件,可以点击更新,重新浏览上传附件。
戊概念·五 2# 2011-10-27 21:35
您辛苦了~ Ps:
戊概念·五 3# 2011-11-2 18:38
1# kuing 这帖让我想到一个词: “领教”,就是我这样的
kuing 4# 2012-8-2 16:07
顶一下这基本操作。
thread-1190-1-1.html: Mathtype 6.9
isea 1# 2013-3-1 14:38
2013年2月22日更新的,http://www.dessci.com 可看详细,最大的变化,支持office 2013,偶原用的6.7a不支持。 下载:http://www.dessci.com/en/dl/InstallMTW6.9.exe keygen,见附件(仅keygen )
kuing 2# 2013-3-1 16:15
对于我来说6.0以上就OK了,因为6.0开始就有tex支持,虽然只是一些基本的。
isea 3# 2013-3-8 12:32
自6.8开始,mathtype输入公式退出时,会多出一个空格,这个问题,很烦人啊。 如图,看来,只好退回6.7了。
kuing 4# 2013-3-8 13:37
3# isea 用 latex 吧,这些细节都可以自己控制、设定。
thread-1191-1-1.html: 如何证明$\sin 1^\circ$ 为无理数
isea 1# 2013-3-1 15:07
RT $\cos 1^\circ$ 呢?
kuing 2# 2013-3-1 18:08
先搞 cos 的。 设 $n\in\mbb N$,注意到 $\cos nx=T_n(\cos x)$ 其中 $T_n(x)$ 为第一类切比雪夫多项式,所以假如 $\cos1^\circ$ 为有理数,那么 $\cos n^\circ$ 都为有理数,这显然与事实不符,从而 $\cos1^\circ$ 必无理数。 此外,由 $T_{60}(\cos1^\circ)=\cos60^\circ=1/2$,所以 $\cos1^\circ$ 不是超越数。
yes94 3# 2013-3-1 19:08
2# kuing $\cos1^\circ$是代数数?
kuing 4# 2013-3-1 19:10
3# yes94 是啊,上面不是说了么,是多项式 $2T_{60}(x)-1=0$ 的根,所以是代数数
realnumber 5# 2013-3-1 19:55
本帖最后由 realnumber 于 2013-3-2 07:31 编辑 假设$\cos1^\circ$为有理数,那么由二倍角公式,$\cos2^\circ$也为有理数,由三倍角公式,$\cos6^\circ$也为有理数,依次$\cos12^\circ,\cos36^\circ$都为有理数. 而$\cos36^\circ$是无理数(可百度).如此假设错误,也即$\cos1^\circ,\cos2^\circ$为无理数. 假设$\sin1^\circ$为有理数,那么由二倍角公式,$\cos2^\circ$为有理数,矛盾.所以$\sin1^\circ$也为无理数. 代数数也成立,就三次,二次方程的实数根,依次倒推下来.(另:加个5倍角公式,就可以从$\sin 30^\circ,\cos 60^\circ$倒推下来.)
kuing 6# 2013-3-1 20:13
5# realnumber nice PS、“度”用代码打跟你通过输入法打出来那个有区别 PS2、让我考虑一下要不要新定义一个自定义命令来简化“度”的输入呢?
realnumber 7# 2013-3-1 20:36
6# kuing 修改好了
yes94 8# 2013-3-1 20:39
实际上和数学帝葛军的命制那道高考题有关。
isea 9# 2013-3-1 22:38
这个问题是由$\tan 1^\circ$联想开的。 涉及级数或者高等数学的,这里就略过了。 $\cos36^\circ$是无理数,这里未有过程,也未百度,期待一下,初等方法。
kuing 10# 2013-3-1 23:13
9# isea $\cos36^\circ=2\cos^218^\circ-1$, $\cos54^\circ=4\cos^318^\circ-3\cos18^\circ$ 相加
realnumber 11# 2013-3-2 07:35
本帖最后由 realnumber 于 2013-3-2 09:24 编辑 $36^\circ,72^\circ,72^\circ$,底角作个垂直平分线,如此根据三角形相似,得出底边和腰的比例,再作条底边上的高就得出... 是某一文上看到的,例举了好几个办法,人教的何版应该很熟的,可惜他似乎不来这里.
yes94 12# 2013-3-2 13:22
11# realnumber 那个求$\cos36^\circ$的题方法至少三种: 一种就是你说的构造等腰三角形,用几何方法(和黄金分割有关) 一种是三倍角公式; 一种是$\cos36^\circ+\cos72^\circ$等等的计算吧
kuing 13# 2013-3-2 13:31
...,人教的何版应该很熟的,可惜他似乎不来这里. realnumber 发表于 2013-3-2 07:35 估计他应该在忙着搞研究、写文档以及他自己的事情,偶尔在群里冒泡,已经不见他去哪个论坛了。 PS、我还是决定增加“度”的自定义命令,请留意置顶贴。
yes94 14# 2013-3-2 13:50
葛军的题就是:如果$\cos A$是有理数,那么$\cos nA$是有理数,显然$\cos30\du$是无理数,故$\cos1\du$是无理数,kuing证明了 它是代数数,非超越数。
realnumber 15# 2013-3-2 16:32
$\sin m,\cos m,m\in (0,\frac{\pi}{2}) 且 m\in Q$是无理数,是否成立,怎么证明.
kuing 16# 2013-3-2 16:49
$\sin m,\cos m,m\in (0,\frac{\pi}{2}) 且 m\in Q$是无理数,是否成立,怎么证明. realnumber 发表于 2013-3-2 16:32 语句表达换一下比较好: 设 $m\in (0,\frac\pi2)$ 且 $m\in\mbb Q$,则 $\sin m$ 和 $\cos m$ 都是无理数。是否成立,怎么证明? PS、感觉上成立
yes94 17# 2013-3-2 22:23
$\sin m,\cos m,m\in (0,\frac{\pi}{2}) 且 m\in Q$是无理数,是否成立,怎么证明. realnumber 发表于 2013-3-2 16:32 $m$是角度制(度数)还是弧度制? 猜测你从$\cos 1\du$推广成有理度数的,所以认为你说的$m$是角度制。 但是,你也可能见到我的留言后会说是弧度制。
kuing 18# 2013-3-2 23:05
没所谓啦,kai 放 xing 问题,随意随意,能玩出任何结论来都是好事。
realnumber 19# 2013-3-2 23:08
17# yes94 弧度吧,虽然分数度即使成立,表达也许会烦琐点.
realnumber 20# 2013-3-2 23:13
本帖最后由 realnumber 于 2013-3-2 23:23 编辑 19# realnumber $m\in (0,\frac{\pi}{2}) 且 m\in Q$,则$\sin m,\cos m$是无理数. 猜测个解决方向 设$m=\frac{q}{p},p,q\in N^+$,$\cos 1$与$\cos q$,用$q$倍角联系起来,$\cos q$与$\cos m$用$p$倍角公式联系起来.还要处理$\cos 1$.(这样看起来,倒还是角度可能容易点,还是按k提议,哪个会有什么结论就哪个.)
thread-1191-2-1.html:
realnumber 21# 2013-3-3 08:14
本帖最后由 realnumber 于 2013-3-3 10:23 编辑 17# yes94 $30^\circ$,$60^\circ$还是要去掉,其它不清楚.
abababa 22# 2013-3-3 10:19
发一位网友的解答,他也是先问了下是角度还是弧度,然后说弧度的他不会,给了角度的证明,不过我也没看懂 发一下他的证明 假设$\sin\frac{q\pi}{p}$是有理数,则由 Euler 知$2\sin\frac{q\pi}{p}=e^{\frac{iq\pi}{p}}-e^{\frac{-iq\pi}{p}}=x+y$是有理数 由于$x,y$分别是$x^p,y^p = 1,-1$的根且和为有理数,于是$x+y$是整数 由有界性知$-2 \leqslant x+y \leqslant 2$,于是$x+y=-2,-1,0,1,2$ 即只有$0^\circ,30^\circ,90^\circ$保证其为有理数,其余都是无理数
kuing 23# 2013-3-3 13:58
一同看不懂……
yes94 24# 2013-3-3 16:11
21# realnumber 嗯,$m$是角度制的话,$\cos m$不一定是无理数。 于是你的猜测要成立的话,只能局限于$m$是弧度制。
yes94 25# 2013-3-21 11:23
小资料:李明老师的,应该正确吧,
kuing 26# 2013-3-21 12:11
25# yes94 何版的:http://www.pep.com.cn/rjwk/gzsxs ... 0110516_1041456.htm
realnumber 27# 2013-5-23 10:31

yes94 28# 2013-5-23 19:10
27# realnumber 在某个刊物中有个定理:$\sin1^0$是无理数,得证。
thread-1192-1-3.html: [不等式] Le Hai 不等式
pxchg1200 1# 2013-3-1 16:01
本帖最后由 pxchg1200 于 2013-3-1 16:02 编辑 Let $a,\,b,\,c$ be real numbers satisfying $a+b+c=0$ and $a^2+b^2+c^2=3.$ Prove that \[a^5b+b^5c+c^5a \le -3.\] 来看看Can的神柯西。 Proof. Since $a+b+c=0$ and $a^2+b^2+c^2=3,$ it is easy to obtain the below results: \begin{align*} ab+bc+ca&=-\frac{3}{2}.\\ a^3b+b^3c+c^3a&=-(ab+bc+ca)^2=-\dfrac{9}{4}.\\ ab^2+bc^2+ca^2+3abc&=-(a^2b+b^2c+c^2a).\\ a^3b^3+b^3c^3+c^3a^3&=(ab+bc+ca)^3+3a^2b^2c^2=-\dfrac{27}{8}+3a^2b^2c^2.\\ \sum (4ab+2c^2+6bc+3)^2&=54.\\   \end{align*} With these results, we have \begin{align*} {a^5}b + {b^5}c + {c^5}a& = \sum {{a^5}b}\\ &= \sum {{a^3}b(3 - {b^2} - {c^2})} \\ &= 3\sum {{a^3}b} - \sum {{a^3}{b^3}} - abc\sum {a{b^2}} \\   & = - \frac{{27}}{4} + \frac{{27}}{8} - 3{a^2}{b^2}{c^2} - abc\sum {a{b^2}}\\   &= - \frac{{27}}{8} + abc\sum {{a^2}b} . \end{align*} Therefore, it suffices to prove that \[abc(a^2b+b^2c+c^2a) \le \frac{3}{8}. \qquad  (1) \] On the other hand, using the Cauchy-Schwarz inequality, we have \[{\left[ {\sum {a(4ab + 2{c^2} + 6bc + 3)} } \right]^2} \le \left( {\sum {{a^2}} } \right)\left[ {\sum {{{(4ab + 2{c^2} + 6bc + 3)}^2}} } \right] = 162.\]   From this, it follows that   \[-9\sqrt{2} \le \sum a(4ab+2c^2+6bc+3) \le 9\sqrt{2},\]    or    \[-\frac{3}{\sqrt{2}} \le a^2b+b^2c+c^2a+3abc \le \frac{3}{\sqrt{2}}.\]     The last inequality yields: \[(a^2b+b^2c+c^2a+3abc)^2 \le \frac{9}{2}. \qquad (2)\]     Using (2) and the AM-GM inequality, we have    \begin{align} abc(a^2b+b^2c+c^2a) &=\frac{1}{3}\cdot 3abc\cdot (a^2b+b^2c+c^2a)\\ & \le \frac{1}{3} \left(\frac{3abc+a^2b+b^2c+c^2a}{2}\right)^2\le \frac{3}{8}, \end{align}       which is (1). So, we are done.
kuing 2# 2013-3-2 01:19
果然如标题所示: 厉害不等式
pxchg1200 3# 2013-3-2 12:36
2# kuing 戳中笑点了。哈哈。。。
thread-1193-1-3.html: [几何] 刚才352问的一道立几判断题
kuing 1# 2013-3-1 23:25
我想到的是向量法,易证 $2\vv{MN} = \vv{AC} + \vv{BD}$,于是显然不能平行。 但是那个显然我不知应该怎么表达,用基底来说可以不?实在不懂教…… 以及,不知各位有什么其他方法判断? PS、实际上两个平面不必垂直。
realnumber 2# 2013-3-2 13:09
本帖最后由 realnumber 于 2013-3-2 13:26 编辑 假设$MN\sslash l$ 在平面$\alpha$内作$CE \pqd MN$,在平面$\beta$内作$DF\pqd MN$,则可得$E,M,F$三点共线,且M为EF中点. 又M为AB中点,得到四边形AEBF为平行四边形,有$AE\sslash BF$,得到$AE\sslash l\sslash BF$(这一步推理是课本例题), 如此即$EA\sslash l$,且$EC\sslash l$,所以A,E,C共线,同理F,B,D共线,即$AC\sslash BD$这与条件异面矛盾.
kuing 3# 2013-3-2 13:35
2# realnumber very nice
realnumber 4# 2013-3-2 13:36
1# kuing 向量参与的推理不是很习惯,不清楚下面的表达有没漏洞,甚至错误. 假设平行,得到$2\vv{MN} = \vv{AC} + \vv{BD}=k\vv{n}$,其中$k$是非零实数,$\vv{n}$是直线$l$上一单位向量, 即$ \vv{AC} =- \vv{BD}+k\vv{n}$,说明AC平行$\vv{BD},k\vv{n}$所确定的平面$\beta$.又AC是面$\alpha$内一直线,面$\alpha$与面$\beta$交线为$l$,所以有$AC\sslash l$,同理得$BD\sslash l$,得$AC\sslash BD$与条件异面矛盾.
kuing 5# 2013-3-2 13:41
4# realnumber 应该没问题,就是这里有点小漏洞 说明AC平行$\vv{BD},k\vv{n}$所确定的平面$\beta$ realnumber 发表于 2013-3-2 13:36 还得先说明 $\vv{BD}$ 与 $\vv n$ 不共线,不过这也显然。 再次多谢
yes94 6# 2013-3-2 13:43
反证法,可以证明出AC//MN,导致AB、CD共面,矛盾。
thread-1194-1-1.html: 请教一个沿圆形方向的箭头的画法
abababa 1# 2013-3-2 12:58
如图那样的箭头要用什么软件画出来呢?现在有一个图,里面很多都是沿这种方向的箭头,试了几何画板,没画出来。
kuing 2# 2013-3-2 13:05
$\Huge \circlearrowleft$ 可惜不是闭合的
abababa 3# 2013-3-2 13:19
2# kuing 谢谢。 是我没说清楚,我的图只是一个示例,实际的图是有两个同心圆,不妨叫$O_1,O_2$,然后在$O_1$的圆周上有10个等分点,设为$A_1$到$A_{10}$,然后从$A_i$到$A_{i+1}$这段是要画那个箭头的,$O_2$上有5个等分点,也同样要画5段那样的箭头。
kuing 4# 2013-3-2 13:23
3# abababa 这么复杂……
kuing 5# 2013-3-2 16:42
用 tikz 画吧 \begin{tikzpicture} \newcommand\ra{2} %圆a半径 \newcommand\rb{3} %圆b半径 \newcommand\na{5} %圆a箭头数 \newcommand\nb{10} %圆a箭头数 \newcommand\jtc{0.1} %箭头长度 \newcommand\jtj{45} %箭头角度 \draw (0,0) circle (\ra); \draw (0,0) circle (\rb); \foreach \x in {1,2,...,\na} \draw[rotate={\x*360/\na}] (\ra,0)--+(\jtj:-\jtc) (\ra,0)--+(-\jtj:\jtc); \foreach \x in {1,2,...,\nb} \draw[rotate={\x*360/\nb}] (\rb,0)--+(\jtj:-\jtc) (\rb,0)--+(-\jtj:\jtc); \end{tikzpicture} 调整前面六个参数方可改变大小等。
kuing 6# 2013-3-2 16:46

kuing 7# 2013-3-2 17:07
实际写文档的时候一般就新定义一个命令来用就比较方便了,比如这个附件中那样。 yuanjiantou.tex (576 Bytes) (用 pdflatex 或 xelatex 编译均可) 当然了,以上只是给个思路了,还不是很细致,你甚至可以添加箭头起始位置的角度等等参数进去,看实际需要了。
abababa 8# 2013-3-2 17:55
7# kuing 谢谢,我先试试,现在还一点也不会,需要时间消化。
thread-1195-1-1.html: 有没有人把对数函数y=lnx写错?
xcxsg 1# 2013-3-2 14:46
本帖最后由 xcxsg 于 2013-3-2 14:47 编辑 有没有人把对数函数$y=ln x$写成$y=In x$或者写成$y=Ln x$?
kuing 2# 2013-3-2 15:04
正好当年发过这一贴:http://kkkkuingggg.5d6d.net/thread-132-1-1.html
xcxsg 3# 2013-3-2 19:21
看起来,还真的是有这么写的
kuing 4# 2013-3-2 19:33
3# xcxsg 是的,见过好多次的了。
realnumber 5# 2013-3-19 23:02
盗版模拟卷上偶尔见到,正式点的书籍好象没见过....
kuing 6# 2013-3-20 00:09
5# realnumber 如果正规的也出这样的错那就比较夸张了…… 讲开又讲,还有人会将普通的对数写成这样的 $\log_2^3$
thread-1196-1-3.html: [几何] 来自粉丝群的切点切线圆系方程
kuing 1# 2013-3-2 14:51
唯因却恩(7643*****) 2013-3-2 12:00:28 唯因却恩(7643*****) 2013-3-2 12:06:31 已知切点和切线的圆系方程怎么算呢 直接用普通方法验证它是不难的,这里只是想讲讲它是怎么搞出来的。 首先我们知道,两个不同心的圆的方程,总能通过线性组合得到一条直线方程,就是他们的根轴。 所以,如果两个不同的圆相切于同一个点,能通过线性组合得到切于该点的切线方程。 我们将切点也看成一个圆(没错,这个圆半径为 $0$,方程为 $(x-x_0)^2+(y-y_0)^2=0$),不影响上面的结论,仍然可以通过圆与切点的线性组合得到切线。 因此,反过来,通过切点与切线的线性组合,能得到所有与切线切于该点的圆。 同时应注意上面的 $\lambda$ 不能为 $0$。
kuing 2# 2013-3-2 15:09
已知切线为 $Ax+By+C=0$ 且切点为 $(x_0,y_0)$ 的圆系方程为 \[(x-x_0)^2+(y-y_0)^2+\lambda (Ax+By+C)=0,\] 也可以写成 \[(x-x_0)^2+(y-y_0)^2+\lambda \bigl(A(x-x_0)+B(y-y_0)\bigr)=0,\] 其中 $\lambda \ne0$。
syzychenwj 3# 2013-3-2 18:26
应该是了,thanx!有来路,再证明,ok!
thread-1197-1-1.html: 测试一下输入公式
xcxsg 1# 2013-3-2 15:02
本帖最后由 xcxsg 于 2013-3-2 15:13 编辑 $f(x)=\begin{cases} x+1&x\ge0\\ 1-x^2&x<0 \end{cases}$ $f(x)=\begin{cases} x+1&x>0\\ 1-x^2&x<0\\ \frac{\sin{x}}{x}&x\le1 \end{cases}$ $\begin{cases} 2x+y-1\ge0\\ x^2-2y+3<0\\ x\le1 \end{cases}$
kuing 2# 2013-3-2 15:05
建议到最下面那个版块“公式测试中”里测
xcxsg 3# 2013-3-2 15:07
没注意那个板块
kuing 4# 2013-3-2 15:11
没事,你测试完我晚点移过去那边。以后再想测试的时候再去那里测试
xcxsg 5# 2013-3-2 15:13
我刚刚在那个板块发了几次,都跳回到了这里,而且也没有修改成功。 郁闷啊。
kuing 6# 2013-3-2 15:26
……你就在这贴子里先测试着,移贴要我来。
李斌斌755 7# 2013-5-13 00:39
本帖最后由 李斌斌755 于 2013-5-13 01:05 编辑 日课 \[f(x)=\left\{\begin{aligned}&x+1,&x\geqslant0,\\&1-x^2,&x<0,\end{aligned}\right.\\f(x)=\left\{\begin{aligned}&x+1,&x>1,\\&\dfrac{\sin x}x,&0<x\leqslant1,\\&0,&x=0,\\&\dfrac{\sin x}x,&-1\leqslant x<0,\\&1-x^2,&x<-1,\end{aligned}\right.\]
李斌斌755 8# 2013-5-13 01:09
本帖最后由 李斌斌755 于 2013-5-13 01:28 编辑 \[\left.\begin{aligned}&AB\sslash CD\\&EF\sslash CD\end{aligned}\right\}\riff EF\sslash AB\]
thread-1198-1-3.html: [几何] 一道要求只能用不等式方法解的线性规划题
yes94 1# 2013-3-2 22:32
江苏冯加明(1217***) 15:20:14 江苏冯加明(1217***) 15:20:26 这题用不等式怎么弄啊 湖南-***** 15:22:45 向量夹角 湖南-****) 15:23:10 代端点 江苏冯加明(1217***) 15:28:22 用不等式啊 江苏冯加明(1217***) 15:28:32 不要向量,也不求导 ______kuing edit in $\LaTeX$______ 14. 已知,点 $P(x,y)$ 的坐标满足 $\left\{\begin{aligned} &\sqrt3x-y<0,\\ &x-\sqrt3y+2<0,\\ &y\geqslant0, \end{aligned}\right.$ 则 $\dfrac{\sqrt3x+y}{\sqrt{x^2+y^2}}$ 的取值范围为_____。
kuing 2# 2013-3-2 23:08
兴趣不大……略看应该是点到直线距离搞出来的。 PS、后面号码没打码
yes94 3# 2013-3-3 16:19
2# kuing 好了,qq号码没删完,现在删掉了,上图!谁试一试冯老师的严格要求?
yes94 4# 2013-3-10 13:03
3# yes94 顶上来,看看能否用不等式方法做
yes94 5# 2013-3-10 21:59
本帖最后由 yes94 于 2013-3-10 22:01 编辑 4# yes94 先搞一个最小值: 原式=$\dfrac{\sqrt3x+y}{\sqrt{x^2+y^2}}\geqslant\dfrac{-\sqrt3\abs x+y}{\sqrt{x^2+y^2}}\geqslant\dfrac{-\sqrt3\abs x}{\sqrt{x^2+y^2}}=\dfrac{-\sqrt3}{\sqrt{1+(\dfrac{y}{x})^2}}\geqslant\dfrac{-\sqrt3}{\sqrt{1+0^2}}=-\sqrt3$, 当且仅当$y=0$,$x=-\abs x$即$x<-2$,$y=0$取的最小值$-\sqrt3$。
reny 6# 2013-3-10 22:24
5# yes94 ,用不等式做还不太好做,复杂化了,当然前面老师说的向量就简单化啦。
yes94 7# 2013-3-10 22:51
6# reny 关键是冯加明老师要求:“不要向量,也不求导 ”
第一章 8# 2013-3-10 23:21
其妙太执着了。如果命题人的意图就是与向量结合,那又何苦非得用不等式来解?再说,冯老师的意思会不会是想要通过平移或旋转直线得到最优解?
yes94 9# 2013-3-11 12:22
8# 第一章 和冯老师一样,是有点执着
零定义 10# 2013-3-17 20:27
无聊~那我来玩玩最大值吧...
yes94 11# 2013-3-17 21:36
呵呵 我也给出很久没有写的最大值了。 原式=$\dfrac{\sqrt3x+y}{\sqrt{x^2+y^2}}=\dfrac{\sqrt3(\sqrt3x+y)}{\sqrt{3(x^2+y^2)}}=\dfrac{\sqrt3(\sqrt3x+y)}{\sqrt{3x^2+y^2+2y^2}}<\dfrac{\sqrt3(\sqrt3x+y)}{\sqrt{3x^2+y^2+2\sqrt3xy}}=\dfrac{\sqrt3(\sqrt3x+y)}{\sqrt{(\sqrt3x+y)^2}}\leqslant\sqrt3$, 这里只用了不等式$2y^2>2\sqrt3xy$,这是因为条件$y>\sqrt3x$.
yes94 12# 2013-3-19 20:08
11# yes94 这不是“最大值”,是上确界吧,
thread-1199-1-1.html: 新手上网学数学去处,或问问题,或下资料,或...
realnumber 1# 2013-3-3 09:57
本帖最后由 realnumber 于 2013-3-3 10:23 编辑 陈美葱的BLOG     http://blog.sina.com.cn/meicong0573 休闲数学娱乐论坛        http://kkkkuingggg.5d6d.net/bbs.php 新浪爱问     http://ishare.iask.sina.com.cn/s ... F%BF%BC&format= 数学吧   http://tieba.baidu.com/f?kw=%CA%FD%D1%A7&from=prin   熊昌进数学教育博客  http://blog.sina.com.cn/s/articlelist_1692494035_0_4.html 奥数之家   http://www.aoshoo.com/bbs1/ 彭翕成与数学传播             http://blog.sina.com.cn/pxc417 或一些QQ群,可以直接在QQ搜索里看到  47224687   148443562 == 大家补充~~~
hongxian 2# 2013-3-9 00:27
1# realnumber 在这里看到了一些好的blog,谢谢了!
realnumber 3# 2013-3-9 07:47
2# hongxian 看到的好问题搬到这里来讨论~~~ 也推荐些,地址或问题
thread-12-1-1.html: 求极限 $\lim_{n\to+\infty}\bigl((n+1)^\alpha-n^\alpha\bigr)$
kuing 1# 2011-9-26 18:02
求极限 \[\lim_{n\to+\infty}\bigl((n+1)^\alpha-n^\alpha\bigr)\] 其中 $0<\alpha<1$
kuing 2# 2011-9-26 18:03
用洛必达或者转化为导数的定义不难证明。 主要是还没想到用数列极限的定义来证。。。
海盗船长 3# 2011-9-29 17:17
用二项式定理行不行?
kuing 4# 2011-9-29 17:36
3# 海盗船长 呃,$0<\alpha<1$ 怎么二……
海盗船长 5# 2011-9-29 17:44
哦,我的意思是那种推广到实数的“二项式定理”。。
kuing 6# 2011-9-29 17:45
5# 海盗船长 噢,我瞧瞧看……
图图 7# 2011-10-3 23:13
本帖最后由 图图 于 2011-10-3 23:38 编辑 战巡给的夹逼法 显然$(1+n)^{\alpha}-n^{\alpha}>0$ 又$(1+n)^{\alpha}-n^{\alpha}=n^α((1+\dfrac{1}{n})^{\alpha}-1)<n^{\alpha}((1+\dfrac{1}{n})-1)=n^{\alpha-1}$
kuing 8# 2011-10-3 23:19
7# 图图 有道理,:lol 不错 PS。打公式时请用英文状态输入,括号好难看ing
图图 9# 2011-10-3 23:23
8# kuing 呃,改了... 好不习惯哇
kuing 10# 2011-10-3 23:27
战巡给的夹逼法 显然$(1+n)^α-n^α>0$ 又$(1+n)^α-n^α=n^α((1+\dfrac{1}{n})^α-1)<n^α((1+\dfrac{1}{n})-1)=n^{α-1}$ 图图 发表于 2011-10-3 23:13 呃,你的 $\alpha$ 原来也不是用代码打的(虽然这里能正常显示,是因为 mathjax 竟然能自动转化,这也是 mathjax 强大的地方之一,不过还是建议可以的话尽量用代码) 还有那个减号也是全角的,难怪高度不一样…… 标准输入法 \[(1+n)^\alpha-n^\alpha=n^\alpha\left(\left(1+\frac{1}{n}\right)^\alpha-1\right)<n^\alpha\left(\left(1+\frac{1}{n}\right)-1\right)=n^{\alpha-1}\] 看看有什么不同? PS:右键选 Show... 看我打的代码
图图 11# 2011-10-3 23:47
10# kuing 呃,其实有一个减号是半角的。 唔...
kuing 12# 2011-10-3 23:50
:D 嘿,慢慢来吧 打代码,练耐心、细心
天涯无际 13# 2012-2-21 20:25
直接用拉格朗日中值定理就好了......
叶剑飞Victor 14# 2012-8-23 04:53
本帖最后由 叶剑飞Victor 于 2012-8-23 04:58 编辑 战巡给的夹逼法 显然$(1+n)^{\alpha}-n^{\alpha}>0$ 又$(1+n)^{\alpha}-n^{\alpha}=n^α((1+\dfrac{1}{n})^{\alpha}-1)<n^{\alpha}((1+\dfrac{1}{n})-1)=n^{\alpha-1}$ 图图 发表于 2011-10-3 23:13 这个有问题啊,没法“夹逼”啊! \[ (1+n)^{\alpha}-n^{\alpha}=n^{\alpha}\left[\left(1+\dfrac{1}{n}\right)^{\alpha}-1\right]<n^{\alpha}\left[\left(1+\dfrac{1}{n}\right)-1\right]=n^{\alpha-1} \] 这个公式没啥用吧——难道\( \displaystyle \lim_{n\to +\infty} n^{\alpha-1} = 0 \) 吗?
海盗船长 15# 2012-8-23 10:59
14# 叶剑飞Victor 注意$0<\alpha<1$.
thread-120-1-1.html: 将Mathematica的输出结果复制成latex代码
kuing 1# 2011-10-19 00:23
基本方法: 选择需要复制的部分——右击——Copy As——LaTeX 这样就复制好了,可在其他地方进行粘贴。 不过我们会发现,Mathematica 直接输出的结果通常与我们习惯的写法不一样,这时我们可以用命令:TraditionalForm,使得结果变成传统写法的形式。 所以我个人的操作常规就是这样:在 Mathematica 里算出理想结果之后,输入 TraditionalForm[%],然后再复制出 LaTeX 代码。而且这样复制出来的代码中很少会有多余的东西,很方便好用。
thread-1200-1-1.html: [不等式] 又一ln的数列不等式
yayaweha 1# 2013-3-3 12:03
证明:$$\frac{1}{2^2}\ln 2^2+\frac{1}{3^2}\ln 3^2+\frac{1}{4^2}\ln 4^2+\cdots+\frac{1}{(n+1)^2}\ln (n+1)^2>\frac{n}{2(n+1)(n+2)}$$
yayaweha 2# 2013-3-3 12:24
1# yayaweha 不知用积分能否突破
abababa 3# 2013-3-3 13:10
2# yayaweha 用积分是不是证明左边大于$\int_{1}^{n+1} \frac{2\ln x}{x^2}dx=\frac{2(n-\ln (n+1))}{n+1}$,要证明它大于右边,就是证明 $4(n+2)(n-\ln (n+1))-n>0$ 当$n=1$时成立,然后证明单调吧 求导是$\frac{8x^2+11x-1-4(x+1)\ln (x+1)}{x+1}$,然后用$-\ln (x+1) > -x$代换,就是证明$8x^2+11x-1-4x(x+1)>0$ 化简就是证明$4x^2+7x-1>0$,当$x>1$时总成立,是这样的吗?
kuing 4# 2013-3-3 13:45
2# yayaweha 用积分是不是证明左边大于$\int_{1}^{n+1} \frac{2\ln x}{x^2}dx=\frac{2(n-\ln (n+1))}{n+1}$,要证明它大于右边,就是证明 $4(n+2)(n-\ln (n+1))-n>0$ 当$n=1$时成立,然后证明单调吧 求导是$ ... abababa 发表于 2013-3-3 13:10 没检查你前面的计算,但是证 $4(n+2)(n-\ln (n+1))-n>0$ 可以简单点,事实上有 $4\ln (n+1)<3n$……
realnumber 5# 2013-3-3 13:50
本帖最后由 realnumber 于 2013-3-3 13:57 编辑 应该也是水母,当$n\ge2$时,有$\ln (n+1)>1$ 那么本题,$n=1$单独验证. 当$n\ge2$时,$\frac{1}{2^2}\ln 2^2+\frac{1}{3^2}\ln 3^2+\frac{1}{4^2}\ln 4^2+\cdots+\frac{1}{(n+1)^2}\ln (n+1)^2>\frac{1}{4}+\frac{2}{3^2}+\frac{2}{4^2}+\cdots+\frac{2}{(n+1)^2}=\frac{1}{4}+\frac{2}{3}-\frac{2}{n+2}>\frac{1}{2(n+2)}>\frac{n}{2(n+1)(n+2)}$
kuing 6# 2013-3-3 13:56
5# realnumber 这类题通常逃不过你抓水母的fa眼
realnumber 7# 2013-3-3 13:57
6# kuing 我是海绵宝宝和快乐星
yayaweha 8# 2013-3-3 14:42
本帖最后由 yayaweha 于 2013-3-3 14:44 编辑 5# realnumber 最喜欢看realnumber的解法啦!
yes94 9# 2013-3-3 16:14
5# realnumber 这么说,这个不等式实在是太弱! 放缩误差这么大,还好意思出来考学生?
yayaweha 10# 2013-3-3 23:45
本帖最后由 yayaweha 于 2013-3-3 23:47 编辑 $n\ge2$ 不妨设$a_n=\frac{n}{2(n+1)(n+2)},则a_{n+1}=\frac{n+1}{2(n+2)(n+3)},a_{n+1}-a_n=\frac{1-n}{2(n+1)(n+2)(n+3)}<0$ 只需证$$\frac{1}{2}ln2>\frac{2}{2*3*4}$$ 而$6ln2>1$显然成立
yayaweha 11# 2013-3-3 23:46
10# yayaweha 这样子做不知道对不对
第一章 12# 2013-3-10 08:39
楼上这个证法很好啊。有时命题的人自己走弯路了,还以为这种题很难。下面是另外的一个例子。
yayaweha 13# 2013-3-10 11:59
12# 第一章 这题我当年做过
kuing 14# 2013-3-10 12:03
12# 第一章 跟这个 http://kkkkuingggg.5d6d.net/thread-488-1-1.html 有点像
第一章 15# 2013-3-10 16:59
嗯。都是很简单的问题。
thread-1200-1-3.html: [不等式] 又一ln的数列不等式
yayaweha 1# 2013-3-3 12:03
证明:$$\frac{1}{2^2}\ln 2^2+\frac{1}{3^2}\ln 3^2+\frac{1}{4^2}\ln 4^2+\cdots+\frac{1}{(n+1)^2}\ln (n+1)^2>\frac{n}{2(n+1)(n+2)}$$
yayaweha 2# 2013-3-3 12:24
1# yayaweha 不知用积分能否突破
abababa 3# 2013-3-3 13:10
2# yayaweha 用积分是不是证明左边大于$\int_{1}^{n+1} \frac{2\ln x}{x^2}dx=\frac{2(n-\ln (n+1))}{n+1}$,要证明它大于右边,就是证明 $4(n+2)(n-\ln (n+1))-n>0$ 当$n=1$时成立,然后证明单调吧 求导是$\frac{8x^2+11x-1-4(x+1)\ln (x+1)}{x+1}$,然后用$-\ln (x+1) > -x$代换,就是证明$8x^2+11x-1-4x(x+1)>0$ 化简就是证明$4x^2+7x-1>0$,当$x>1$时总成立,是这样的吗?
kuing 4# 2013-3-3 13:45
2# yayaweha 用积分是不是证明左边大于$\int_{1}^{n+1} \frac{2\ln x}{x^2}dx=\frac{2(n-\ln (n+1))}{n+1}$,要证明它大于右边,就是证明 $4(n+2)(n-\ln (n+1))-n>0$ 当$n=1$时成立,然后证明单调吧 求导是$ ... abababa 发表于 2013-3-3 13:10 没检查你前面的计算,但是证 $4(n+2)(n-\ln (n+1))-n>0$ 可以简单点,事实上有 $4\ln (n+1)<3n$……
realnumber 5# 2013-3-3 13:50
本帖最后由 realnumber 于 2013-3-3 13:57 编辑 应该也是水母,当$n\ge2$时,有$\ln (n+1)>1$ 那么本题,$n=1$单独验证. 当$n\ge2$时,$\frac{1}{2^2}\ln 2^2+\frac{1}{3^2}\ln 3^2+\frac{1}{4^2}\ln 4^2+\cdots+\frac{1}{(n+1)^2}\ln (n+1)^2>\frac{1}{4}+\frac{2}{3^2}+\frac{2}{4^2}+\cdots+\frac{2}{(n+1)^2}=\frac{1}{4}+\frac{2}{3}-\frac{2}{n+2}>\frac{1}{2(n+2)}>\frac{n}{2(n+1)(n+2)}$
kuing 6# 2013-3-3 13:56
5# realnumber 这类题通常逃不过你抓水母的fa眼
realnumber 7# 2013-3-3 13:57
6# kuing 我是海绵宝宝和快乐星
yayaweha 8# 2013-3-3 14:42
本帖最后由 yayaweha 于 2013-3-3 14:44 编辑 5# realnumber 最喜欢看realnumber的解法啦!
yes94 9# 2013-3-3 16:14
5# realnumber 这么说,这个不等式实在是太弱! 放缩误差这么大,还好意思出来考学生?
yayaweha 10# 2013-3-3 23:45
本帖最后由 yayaweha 于 2013-3-3 23:47 编辑 $n\ge2$ 不妨设$a_n=\frac{n}{2(n+1)(n+2)},则a_{n+1}=\frac{n+1}{2(n+2)(n+3)},a_{n+1}-a_n=\frac{1-n}{2(n+1)(n+2)(n+3)}<0$ 只需证$$\frac{1}{2}ln2>\frac{2}{2*3*4}$$ 而$6ln2>1$显然成立
yayaweha 11# 2013-3-3 23:46
10# yayaweha 这样子做不知道对不对
第一章 12# 2013-3-10 08:39
楼上这个证法很好啊。有时命题的人自己走弯路了,还以为这种题很难。下面是另外的一个例子。
yayaweha 13# 2013-3-10 11:59
12# 第一章 这题我当年做过
kuing 14# 2013-3-10 12:03
12# 第一章 跟这个 http://kkkkuingggg.5d6d.net/thread-488-1-1.html 有点像
第一章 15# 2013-3-10 16:59
嗯。都是很简单的问题。
thread-1201-1-3.html: [不等式] a+b+c=3,一个不等式
realnumber 1# 2013-3-3 18:12
本帖最后由 realnumber 于 2013-3-3 18:28 编辑 \[a,b,c\in R^+,a+b+c=3,求\frac{\sqrt{a}}{3-a}+\frac{\sqrt{b}}{3-b}+\frac{\sqrt{c}}{3-c}的最小值.\] 要求主流点,高考难度内的办法. 切线法已经有个,还有别的吗? \[\frac{\sqrt{a}}{3-a}\ge \frac{a}{2}\]
kuing 2# 2013-3-3 18:26
求什么? 主题没分类 楼主喜欢用0.5
realnumber 3# 2013-3-3 18:30
2# kuing 下次看到不顺眼,直接改,不需要客气
goft 4# 2013-3-3 22:04
本帖最后由 goft 于 2013-3-3 22:48 编辑 晕,还是打不出来…… $frac{\sqrt {x}}{3-x}$
kuing 5# 2013-3-3 22:29
4# goft 置顶贴这么难懂么
realnumber 6# 2013-3-3 22:30
4# goft 用2个美元记号,夹住表达式
kuing 7# 2013-3-3 22:37
\[\frac{\sqrt a}{3-a}=\frac{\sqrt a}{b+c}=\frac{\sqrt2a}{\sqrt{2a(b+c)(b+c)}}\geqslant\frac{\sqrt2a}{\sqrt{\left( \frac{2a+b+c+b+c}3\right)^3}}=\frac a2,\] 这算不算是另一种方法
yes94 8# 2013-3-3 23:19
7# kuing 算! 选修的三元均值不等式,只是怎么想到的?学生会问。切线法我觉得也可算高中方法了,
yes94 9# 2013-3-4 00:03
8# yes94 其实切线法知道那个不等式后,用高中的方法还是很容易的: 由均值不等式可知,$a+\dfrac1{\sqrt a}+\dfrac1{\sqrt a}\geqslant3$, 故$3-a\leqslant\dfrac2{\sqrt a}$, 于是,$\dfrac{\sqrt a}{3-a}\geqslant \dfrac a2$
realnumber 10# 2013-3-4 08:33
本帖最后由 realnumber 于 2013-3-4 09:39 编辑 9# yes94 继续你的思路 由均值不等式可知,$(a+1)+\dfrac1{\sqrt a}+\dfrac1{\sqrt a}\ge 2\sqrt a+2\dfrac1{\sqrt a}\geqslant4$, 故$3-a\leqslant\dfrac2{\sqrt a}$ 又,7楼也不错.
yes94 11# 2013-3-4 10:30
10# realnumber 用了两次二元均值, 是否三元均值可用二元均值来证明?你已给出一种, 我来试一试另一种: 因为$a+1\geqslant 2\sqrt a$……(1), 对(1)式乘以$\sqrt a$得:$a\sqrt a+\sqrt a\geqslant 2a$………(2), 对(1)移项得:$a\geqslant 2\sqrt a-1$,代入(2)式得: $a\sqrt a+\sqrt a\geqslant 2a\geqslant 2(2\sqrt a-1)=4\sqrt a-2$, 化简得:$2\geqslant3\sqrt a-a\sqrt a=\sqrt a(3-a)$……(3), 对(3)乘以$\sqrt a$得:$2\sqrt a\geqslant a(3-a)$, 即$\dfrac{\sqrt a}{3-a}\geqslant \dfrac a2$
yes94 12# 2013-3-4 10:35
11# yes94 上面的证法反复用了两次不等式:$a+1\geqslant 2\sqrt a$ 反复乘了两次$\sqrt a$,看来三元均值可用二元均值代替。
ccnu_chb_ycb 13# 2013-3-5 14:55
这个算是初等的证明吧
kuing 14# 2013-3-5 15:20
13# ccnu_chb_ycb 还不是跟7#一样
yes94 15# 2013-3-5 18:21
14# kuing 的确和7楼一样。 下面再来一种三元均值,虽意义不大,且貌似繁琐些: 先证明:$\sqrt a(3-a)\leqslant2$,于是就有$\dfrac{\sqrt a}{3-a}\geqslant\dfrac a2$。 故$\sqrt a(3-a)=\sqrt a(\sqrt3-\sqrt a)(\sqrt3+\sqrt a)=\dfrac1{mn}\sqrt a(\sqrt3m-m\sqrt a)(\sqrt3n+n\sqrt a)\leqslant\dfrac1{mn}(\dfrac{\sqrt3(m+n)+(1-m+n)\sqrt a}3)^3=2$, 其中$m=\dfrac{\sqrt3+1}2$,$n=\dfrac{\sqrt3-1}2$,则$mn=\dfrac12$,且$m-n=1$,$m+n=\sqrt3$,取等号略。证完。
thread-1202-1-3.html: [不等式] 一个不等式
pxchg1200 1# 2013-3-4 15:55
本帖最后由 pxchg1200 于 2013-3-4 15:57 编辑 Let $a,b,c>0$ Prove that: \[ \sqrt{\frac{a}{a+7b+c}}+\sqrt{\frac{b}{b+7c+a}}+\sqrt{\frac{c}{c+7a+b}}\geq 1 \] 弄了半天,Holder不出。。。
yes94 2# 2013-3-4 18:21
1# pxchg1200 转两个类似的不等式(当然简单多了): (1)已知$a、b\in R^+$,$\sqrt{\dfrac{a}{a+3b}}+\sqrt{\dfrac{b}{b+3a}}\geqslant1$; (2)已知$a、b\in R^+$,$\sqrt[3]{\dfrac{a}{a+7b}}+\sqrt[3]{\dfrac{b}{b+7a}}\geqslant1$。 kuing的草稿本真的很有用哦!
zdyzhj 3# 2013-3-5 06:52
这个可爱的不等式昨天晚上解决了。
yes94 4# 2013-3-5 12:04
3# zdyzhj 欢迎县长驾到! 县长是如何解决的呢?
pxchg1200 5# 2013-3-6 16:42
Well,西哥的bao力解答。
pxchg1200 6# 2013-3-6 16:43
3# zdyzhj 另外,麻烦您不要老是说这些没用的话,最好用证明来说话,Ok?
yes94 7# 2013-3-6 19:22
6# pxchg1200 县长的特色
thread-1203-1-1.html: 关于分位数的疑问
童光红 1# 2013-3-4 17:39
各位老师:我在研究宽带薪酬结构的时候,遇到一种关于分位值【即分位数】的问题: 比如: 薪酬分位值反映市场的薪酬水平状态。 25分位值   表示有25%的数据小于此数值,反映市场的较低端水平。 50分位值(中位值)   表示有50%的数据小于此数值,反映市场的中等水平。 75分位值   表示有75%的数据小于此数值,反映市场的较高端水平。 同事解释:假如有100个随机数字,按从小到大排列,排在第25、第50、第75、第90个的数字就是25分位、第50分位、第75分位、第90分位把这些数字改成薪酬水平数,就可以用来比较薪酬的相对水平了。 我对上述解释不太清楚,能否再详细解释一下? 最好举例说明,谢谢!
thread-1204-1-1.html: 发一个公式预览,或者说是发贴草稿本[2013-3-6晚更新]
kuing 1# 2013-3-4 20:25
见附件: 简易LaTeX公式预览(for悠闲数学娱乐论坛)更新日期2013-3-6晚.html (3.2 KB) 这是一个简易的,能随输入代码即时显示公式的html文件,这里我称它为“发贴草稿本”,简称“草稿本”,欢迎大家试用! 之所以弄这个草稿本,是因为本论坛缺少一个发贴预览功能,以至于要发贴后才能看到公式效果,这对于新手来说是个难点,往往要修改好几遍才能写好,而编辑贴子时来回跳转很麻烦。 现在有了草稿本,就可以在发贴前先打开草稿本,打好再复制上来,省了来回跳转的麻烦。 练习打代码的时候也可以用这个来试试。 草稿本中能使用的代码与本论坛一致,所以置顶贴中的自定义代码也是可以用的。 不过由于我其实并不熟悉 html 的东东,因此大概会有不少 bug ,暂时有一个比较大的 bug 就是公式编号问题,所以预览时请先别用自动编号的环境(即用 align* 而别用 align,等等)。 大家先用着看看,有什么问题就在这里留言喔 _____bug's_____ 公式编号问题(未解决) 多个空格只当一个空格(未解决) 小于号问题(已解决2013-3-6) 乱码问题(已解决2013-3-6)
第一章 2# 2013-3-4 20:28
一上来就看到这个了,下来试试
第一章 3# 2013-3-4 20:29
话说我下载了,那个下载次数还是显示0
kuing 4# 2013-3-4 20:30
3# 第一章 论坛问题……嘿嘿,先试试看再说吧
yes94 5# 2013-3-4 21:10
2# 第一章 我都用了一次草稿本发帖,相当好用,所见即所得! 谢谢kuing的细心、贴心!
kuing 6# 2013-3-4 21:22
5# yes94 但是代码还是得自己打的喔,要搞得像mathtype那样有按钮、快捷键这些我就不会弄了,也没打算弄……
yes94 7# 2013-3-4 21:35
6# kuing 试了一下,mathtype也可以搞: 只是要删掉前面很多行的大量乱码(留下关键的两个美元符号及其中间部分全部代码): % MathType!MTEF!2!1!+- % feaagGart1ev2aqatCvAUfeBSjuyZL2yd9gzLbvyNv2CaerbuLwBLn % hiov2DGi1BTfMBaeXatLxBI9gBaerbd9wDYLwzYbItLDharqqtubsr % 4rNCHbWexLMBbXgBd9gzLbvyNv2CaeHbl7mZLdGeaGqiVu0Je9sqqr % pepC0xbbL8F4rqqrFfpeea0xe9Lq-Jc9vqaqpepm0xbba9pwe9Q8fs % 0-yqaqpepae9pg0FirpepeKkFr0xfr-xfr-xb9adbaqaaeGaciGaai % aabeqaamaabaabauqakabaaaaaaaaapeqaamaalaaabaGaeyOeI0Ia % amOyaiabgglaXoaakaaabaGaamOyamaaCaaaleqabaGaaGOmaaaaki % abgkHiTiaaisdacaWGHbGaam4yaaWcbeaaaOqaaiaaikdacaWGHbaa % aiaab2dadaWfqaqaaiGacYgacaGGPbGaaiyBaaWcbaGaamiEaiabgk % ziUkabg6HiLcqabaGcdaGcaaqaaiaadkgadaahaaWcbeqaaiaaikda % aaGccqGHsislcaaI0aGaamyyaiaadogaaSqabaGccaqG9aWaaSaaae % aacaWGUbGaaiyiaaqaaiaadkhacaGGHaWaaeWaaeaacaWGUbGaeyOe % I0IaamOCaaGaayjkaiaawMcaaiaacgcaaaGaaeypamaakaaabaGaam % yyamaaCaaaleqabaGaaGOmaaaakiabgUcaRiaadkgadaahaaWcbeqa % aiaaikdaaaaabeaakiaab2dadaWcaaqaaiaaigdaaeaacaaIYaaaaa % aa!6889! $\dfrac{{ - b \pm \sqrt {{b^2} - 4ac} }}{{2a}}{\rm{ = }}\mathop {\lim }\limits_{x \to \infty } \sqrt {{b^2} - 4ac} {\rm{ = }}\dfrac{{n!}}{{r!\left( {n - r} \right)!}}{\rm{ = }}\sqrt {{a^2} + {b^2}} {\rm{ = }}\dfrac{1}{2}$
kuing 8# 2013-3-4 21:38
7# yes94 mathtype转出来的代码不好。常有很多多余的东西(不是指前面那堆,是指公式中的代码)甚至是错误的东西,要是出了问题也不好找,而且又没有 cases、align 等这些好用的环境,所以我一直提倡能手打就手打
kuing 9# 2013-3-4 21:43
7# yes94 就你这里输出的代码来说 \dfrac{{ - b \pm \sqrt {{b^2} - 4ac} }}{{2a}}{\rm{ = }}\mathop {\lim }\limits_{x \to \infty } \sqrt {{b^2} - 4ac} {\rm{ = }}\dfrac{{n!}}{{r!\left( {n - r} \right)!}}{\rm{ = }}\sqrt {{a^2} + {b^2}} {\rm{ = }}\dfrac{1}{2} 首先,多出了太多的 { } ,还无故地对等号使用了 \rm{ } ,极限那里也有多余的东西,阶乘分母中也无需用 left right。 简化下来,只要这样就可以了 \dfrac{-b\pm\sqrt{b^2-4ac}}{2a}=\lim_{x\to\infty}\sqrt{b^2-4ac}=\dfrac{n!}{r!(n-r)!}=\sqrt{a^2+b^2}=\dfrac12
yes94 10# 2013-3-4 22:16
本帖最后由 yes94 于 2013-3-4 23:16 编辑 9# kuing 哦!学习了!原先\dfrac没有d的,自己加了一个d 主要是mathtype转化成latex2.9版本的,这个latex2.9版本很低级吧 $\dfrac{-b\pm\sqrt{b^2-4ac}}{2a}=\lim_{x\to\infty}\sqrt{b^2-4ac}=\dfrac{n!}{r!(n-r)!}=\sqrt{a^2+b^2}=\dfrac12$ 很奇怪极限的$x\to\infty$的显示没有k12好呢(直接粘贴复制的代码)?没显示在底部。 原来是要用行间公式。 \[\dfrac{-b\pm\sqrt{b^2-4ac}}{2a}=\lim_{x\to\infty}\sqrt{b^2-4ac}=\dfrac{n!}{r!(n-r)!}=\sqrt{a^2+b^2}=\dfrac12\] 再学习一下kuing的首页必读,得知也可以在公式前面加一个\displaystyle,也可以解决上述问题。 $\displaystyle\dfrac{-b\pm\sqrt{b^2-4ac}}{2a}=\lim_{x\to\infty}\sqrt{b^2-4ac}=\dfrac{n!}{r!(n-r)!}=\sqrt{a^2+b^2}=\dfrac12$
kuing 11# 2013-3-4 22:22
10# yes94 不是那个版本低的问题,是mathtype设计转码时不细致,当然有的时候也是人手操作造成的。 PS、那个极限那里,用行间公式就会显示在底了
kuing 12# 2013-3-5 00:44
更新了一下,简化了源码。 欢迎继续测试
╰☆ヾo.海x 13# 2013-3-5 02:14
可以打开可以编辑但是我还没学代码。。。。。。如果能够把mathtype作为插件在这个论坛就好了。。跟word那样
isea 14# 2013-3-5 10:11
发长代码时,这个东西有用 编辑帖跳转是回首页,不能改,这个烦,这个论坛程序
kuing 15# 2013-3-5 13:55
14# isea 是的,所以整这个草稿还是很有必要了,必须减少因代码问题而编辑贴子的麻烦
hongxian 16# 2013-3-5 14:30
15# kuing 些贴应置顶,以方便下载
kuing 17# 2013-3-5 15:22
16# hongxian 有的人没有看置顶的习惯,先放下面一段时间,等沉了再置顶
hongxian 18# 2013-3-6 08:03
17# kuing 有一点小问题,那就是一个公式代码如果太长好象转换不了,只得把它分成两个公式了!
kuing 19# 2013-3-6 08:53
18# hongxian 举个例
hongxian 20# 2013-3-6 12:09
本帖最后由 hongxian 于 2013-3-6 12:11 编辑 19# kuing 看一下这个 $a_{n+1}^2=\dfrac{(1 \times 3)\times(3 \times 5)\times \cdots\times(n-2)\times n \times n}{2^2 \times 4^2 \times \cdots \times(n-1)^2}<$ $n \Longrightarrow a_{n+1}<\sqrt n$ $a_{n+1}^2=\dfrac{(1 \times 3)\times(3 \times 5)\times \cdots\times(n-2)\times n \times n}{2^2 \times 4^2 \times \cdots \times(n-1)^2}<n \Longrightarrow a_{n+1}<\sqrt n$ 还是上一个图吧
thread-1204-2-1.html:
kuing 21# 2013-3-6 12:22
20# hongxian 嗯,我研究一下是什么原因
kuing 22# 2013-3-6 12:29
20# hongxian 发现原因所在了, < 和 n 不能连在一起,只要隔开一下就可以了。 我再研究一下如何解决这个问题
kuing 23# 2013-3-6 13:57
解决了,1楼附件已更新,请重新下载,再测试
kuing 24# 2013-3-6 14:24

hongxian 25# 2013-3-6 14:41
24# kuing 经测试已经解决,谢谢了!
kuing 26# 2013-3-6 15:08
25# hongxian 不客气 顺便说说刚才那bug,其实原因不在公式长短,而在于那个小于号,因为 html 代码里命令都由 < 开始,所以当公式中也有 < 时就容易产生混乱,现在我在内部作了一个些替换操作就不会出现这种混乱了。
kuing 27# 2013-3-6 21:25
刚才有网友告知在 ipad 上打开此文件时乱码,原来是因为文件没指明编码的原因,为此刚才再更新了一次,指定了 gbk 编码。
realnumber 28# 2013-3-9 07:56
修改不够美观,是不是“本帖最后由 goft 于 2013-2-28 21:18 编辑”和正文距离太大,能缩小吗?
kuing 29# 2013-3-9 11:20
28# realnumber 你说的是论坛的设定? 这可没法改喔
Tesla35 30# 2013-3-10 13:06
标题。。。。kuing还活在2012年。。。。。
yes94 31# 2013-3-10 13:14
本帖最后由 yes94 于 2013-3-14 19:34 编辑 30# Tesla35 呵呵 $\text{abcd}\text{efg}\text{hijk}$lmn
kuing 32# 2013-3-10 13:19
标题。。。。kuing还活在2012年。。。。。 Tesla35 发表于 2013-3-10 13:06 还没习惯2013……
kuing 33# 2013-3-10 13:21
汗一下,文件里面也打错了……
kuing 34# 2013-3-10 13:28
看了下之前的版本……由制作开始就打错了一直都是2012…… 附件已更新。
叶剑飞Victor 35# 2013-3-10 14:16
本帖最后由 叶剑飞Victor 于 2013-3-10 14:48 编辑 多个空格只当一个空格 解决方法一: 用<pre>和</pre>把整个预览包起来,删去<br>换行的代码。 在HTML代码中,<pre>标签用于“原样显示”,即有几个空格,就显示几个空格,遇到回车换行符,就换行。 function doPreview() {         var fdsa = document.getElementById("inputText").value;         var asdf = "<pre>";         for (var i = 0; i < fdsa.length; i++)         {                 switch(fdsa.charCodeAt(i))                 {    // 空格和回车换行就不用替换了                 case 60:                         asdf += "<";                         break;                 case 62:                         asdf += ">";                         break;                 case 38:                         asdf += "&";                         break;                 default:                         asdf += fdsa.charAt(i);                         break;                 }         }         asdf += "</pre>";         document.getElementById('output').innerHTML = asdf;         MathJax.Hub.Queue(["Typeset",MathJax.Hub,"output"]); } 复制代码 解决方法二: 用 替换空格。 case 32:  // 空格         asdf += " ";         break; 复制代码
kuing 36# 2013-3-10 14:26
35# 叶剑飞Victor 电脑高手终于来了 方法二我想过,但是这样可能会造成公式里面的空格也变成了真的空格,所以当时没采取。 我先试试方法一。
kuing 37# 2013-3-10 14:36
方法一好像 mathjax 不对它干活了?
kuing 38# 2013-3-10 14:39
难道 mathjax 对 <pre> 也会变成显示源代码
叶剑飞Victor 39# 2013-3-10 14:42
38# kuing 好像是的,我再测试测试。
kuing 40# 2013-3-10 14:45
39# 叶剑飞Victor 方法我已经理解了。 可能 mathjax 为了提供显示源代码的方式,所以对 <pre> 不操作。
thread-1204-3-1.html:
yes94 41# 2013-3-10 14:51
40# kuing 虽然不知道说什么,但知道两大电脑高手双剑合璧,通力合作,天下无敌!
kuing 42# 2013-3-10 14:53
41# yes94 我菜鸟……为搞这个东东也只是临时学了一点点,还东抄西抄……
叶剑飞Victor 43# 2013-3-10 14:56
本帖最后由 叶剑飞Victor 于 2013-3-10 15:04 编辑 38# kuing 那就先MathJax转码,然后再加<pre>。不过MathJax有延时(MathJax.Hub.Queue是异步的),所以要用setTimeOut等一会儿再加<pre> function doPreview() {         var fdsa = document.getElementById("inputText").value;         var asdf = "";         for (var i = 0; i < fdsa.length; i++)         {                 switch(fdsa.charCodeAt(i))                 {                 case 10:                         asdf += "<br>";                         break;                 case 13:                         asdf += "<br>";                         break;                 case 60:                         asdf += "<";                         break;                 case 62:                         asdf += ">";                         break;                 default:                         asdf += fdsa.charAt(i);                         break;                 }         }         document.getElementById('output').innerHTML = asdf;         MathJax.Hub.Queue(["Typeset",MathJax.Hub,"output"]);         setTimeout(function(){                         document.getElementById("output").innerHTML = "<pre>" + document.getElementById("output").innerHTML + "</pre>";                 },1000); }; 复制代码
kuing 44# 2013-3-10 15:09
43# 叶剑飞Victor 可以是可以,但是多做了一步,没那么shuang快了,而且右边的显示有点跳,因为原先设定的样式跟<pre>出来的不一样……
abababa 45# 2013-3-10 16:05
44# kuing 刚问了一位网友,空格是可以显示了,不知道改了些什么。 c.zip (2.02 KB)
kuing 46# 2013-3-10 16:40
45# abababa 这个产生的后果跟我前面说的一样,公式里的空格也会变成真的空格,即使是一个,也产生了距离,所以不行
叶剑飞Victor 47# 2013-3-10 17:00
46# kuing 试试这个,就是反应慢了点,其它都好。
abababa 48# 2013-3-10 17:07
46# kuing 哦,我又让他帮看了看,又发了一个。 c.zip (2.03 KB)
kuing 49# 2013-3-10 17:09
47# 叶剑飞Victor 噢,用了另一个东东将中间的东东临时存放,将最终结果显示出来,所以不会跳…… 不过显示的公式的字体怎么感觉有点怪怪,还有 htm 和 html 有什么不同?
kuing 50# 2013-3-10 17:13
48# abababa 他的意思我懂了         case 36:                 dollar_count++; asdf += "$"; break;         case 32:                 if(dollar_count%2==0) {asdf += " ";}                 break; 这样就可以将两个美元符号之间的部分的空格不作处理。 但是,公式不一定都用美元符号,所以行间公式里的空间肯定还会是空格,环境里肯定也一样。 你可以试试将行间公式里添加空格试试。
kuing 51# 2013-3-10 17:27
50# kuing 这个思路本身是挺好的,可惜实际情况比较复杂,要顾及各种情况的话,整下去可能很麻烦。 any way,感谢大家的支持,至少让我学会一些东东。 其实空格问题并不算是太大的bug,要是不好弄,还是可以不处理的。
abababa 52# 2013-3-10 17:34
51# kuing 不懂编程,基本都看不懂。 下载了#47的那个,挺好用的。
kuing 53# 2013-3-10 17:38
52# abababa 其实我也不太懂,那天临时学了一些,所以看别人的代码,大概能猜到意思。 47#那个我这里测试的效果就是反应稍慢,而且公式字体没原来的好看。我估计还是因为<pre>的字体和mathjax处理的时候生成的字体格式不同所致…… 补充问一下叶:可不可以设置 <pre> 的字体?
叶剑飞Victor 54# 2013-3-10 18:06
本帖最后由 叶剑飞Victor 于 2013-3-10 18:19 编辑 49# kuing 47# 叶剑飞Victor 噢,用了另一个东东将中间的东东临时存放,将最终结果显示出来,所以不会跳…… 不过显示的公式的字体怎么感觉有点怪怪,还有 htm 和 html 有什么不同? kuing 发表于 2013-3-10 17:09 .htm和.html是一回事。这是历史原因,早期DOS操作系统文件名必须遵守“8.3规则”(主文件名不得超过八个字节,扩展名不能超过三个字节),因此 .html 就被写成了.htm,同样还有JPEG格式的图片 .jpeg 写成 .jpg 等等。
叶剑飞Victor 55# 2013-3-10 18:07
本帖最后由 叶剑飞Victor 于 2013-3-10 18:20 编辑 53# kuing 52# abababa 其实我也不太懂,那天临时学了一些,所以看别人的代码,大概能猜到意思。 47#那个我这里测试的效果就是反应稍慢,而且公式字体没原来的好看。我估计还是因为的字体和mathjax处理的时候生成的字体格 ... kuing 发表于 2013-3-10 17:38 可以设置字体的,例如 一、内嵌CSS <pre style="font-family: '宋体';">内容</pre> 复制代码 或者     二、文档内CSS <style type="text/css"> pre {         font-family: "宋体"; } </style> <pre>内容</pre> 复制代码 三、还有外联CSS的,不过要另外一个文件,这里省略了
kuing 56# 2013-3-10 18:13
我知道47#那个为什么字体会有点怪,是因为 <div style="display: none">   <div id="pre-output"></div> </div> 这里的字体大小跟我前面设的输出字体大小不同,我上面设的是 1.2em。 因此,改成 <div style="display: none">   <div id="pre-output" style="font-size:1.2em"></div> </div> 这样效果就OK了
叶剑飞Victor 57# 2013-3-10 18:16
本帖最后由 叶剑飞Victor 于 2013-3-10 18:18 编辑 52# abababa 51# kuing 不懂编程,基本都看不懂。 下载了#47的那个,挺好用的。 abababa 发表于 2013-3-10 17:34 这不能算“编程” 一点点HTML语言和JavaScript语言,这叫“网页设计”。 “网页设计”跟“编程”(正名应为“程序设计”)完全不在一个档次上。 程序设计是用C/C++/Java/VB/C#等程序设计语言写的,难度非常大,很伤脑的。 相较而言,网页设计轻松得多!
kuing 58# 2013-3-10 18:32
57# 叶剑飞Victor 是的,简单得多,连我这种临时学的都能理解个大概……
abababa 59# 2013-3-10 18:38
57# 叶剑飞Victor 哦,听说过那几种,C,VB,Java什么的,不过同样不懂。网页也不懂,只有用别人的了。
kuing 60# 2013-3-10 20:13
我知道47#那个为什么字体会有点怪,是因为 <div style="display: none">   <div id="pre-output"></div> </div> 这里的字体大小跟我前面设的输出字体大小不同,我上面设的是 1.2em。 因此,改成 <div style="display: none">   <div id="pre-output" style="font-size:1.2em"></div> </div> 这样效果就OK了 kuing 发表于 2013-3-10 18:13 刚才实测了下,好像并不是如我所说那样的……
thread-1204-4-1.html:
kuing 61# 2013-3-10 20:41
47#的在我的 IE8 里会出现错乱,不知为什么。 看来不是什么字体问题,而可能是会因浏览器而不同 但是1#的(未处理空格问题版)没问题
yes94 62# 2013-3-14 18:36
61# kuing 打一串英语,用两个美元的话,英语单词之间的空格就没啦,单词全部粘连在一起啦~!
kuing 63# 2013-3-14 18:56
61# kuing 打一串英语,用两个美元的话,英语单词之间的空格就没啦,单词全部粘连在一起啦~! yes94 发表于 2013-3-14 18:36 普通文字本身就不应该放在两美元之间,实在需要的时候还要用 \text{} 将它们括起来。
kuing 64# 2013-3-19 23:33
顶一下,应该差不多要置顶了……
kuing 65# 2013-3-21 17:26
置顶ing
秋风树林 66# 2013-3-22 21:44
噢...这个方便....
yes94 67# 2013-3-25 21:48
发一个公式预览,或者说是发贴草稿本[2013-3-6晚更新] 标题改成: 论坛发贴草稿本(即公式网页预览)下载[2013-3-6晚更新] 看看可以不?
zwl1972 68# 2013-3-28 16:54
7# yes94
zwl1972 69# 2013-3-28 16:59
7# yes94 zwl1972 发表于 2013-3-28 16:54 那些多余东西可以改变设置去掉的 preferences--cut and copy preferences把复选框的那两个对勾去掉
yes94 70# 2013-3-30 18:48
那些多余东西可以改变设置去掉的 preferences--cut and copy preferences把复选框的那两个对勾去掉 zwl1972 发表于 2013-3-28 16:59 谢谢!现在明白了
李斌斌755 71# 2013-4-15 00:04
\sqrt{a}
kuing 72# 2013-4-15 00:07
71# 李斌斌755 把1#的附件(草稿本)下载下来,打开,在里面测试……
李斌斌755 73# 2013-4-15 00:09
thread-1205-1-3.html: (zt)喝到不合格奶粉的概率各为多少?
realnumber 1# 2013-3-5 08:17
小明今天刚出生,妈妈每6天要买一罐奶粉,奶粉合格率为99%。 请问小明在满月前、周岁前与两岁前喝到不合格奶粉的概率各为多少? ----懒得算,有人愿意做吗?似乎很容易喝到...
isea 2# 2013-3-5 09:52
本帖最后由 isea 于 2013-3-5 15:48 编辑 想起微博上的两个等式:$1.01^{365}=37.8,0.99^{365}=0.03$ 简单的算一下,一年12个月,就按一月3罐吧,也得36罐,(看作独立事件) 则(一年)喝到 不合格的概率为$1-0.99^{36}=0.30358678195 \approx 30.3\%$
kuing 3# 2013-3-5 10:03
话说每6天买的话一年都60了……45%
isea 4# 2013-3-5 10:08
这个是看作独立事件的,不过,如果不独立又如何?
realnumber 5# 2013-3-5 11:46
4# isea 恩,开放问题,可以任意改变问题.
thread-1206-1-3.html: [不等式] 三个奇怪的条件不等式
ccnu_chb_ycb 1# 2013-3-5 09:28
这三个条件不等式有些奇怪,现在搜集的证明要么不知是怎么想到的(取等条件怎么自然得到),要么很复杂,希望大家帮忙看看,谢谢
kuing 2# 2013-3-5 09:35
前两题是等价的(只要换个元,就可以转化为另一个不等式),参考链接: http://bbs.pep.com.cn/thread-402061-1-1.html http://bbs.pep.com.cn/thread-351701-1-1.html http://bbs.pep.com.cn/thread-462386-1-1.html 等等…… 第三题应该可以照抄链接中的方法,至于方程组有没有好的解,就不知道了。
ccnu_chb_ycb 3# 2013-3-5 10:04
谢谢kuing
yes94 4# 2013-3-5 12:06
3# ccnu_chb_ycb 楼主昨晚已经知道了解答,又来发帖。 还有个解答就是计神的作差配方法。
kuing 5# 2013-3-5 12:28
噢,不知能不能礼尚往来
ccnu_chb_ycb 6# 2013-3-5 14:51
我昨晚没有得到比较满意的解答啊,或者说我没有弄明白怎么做这类问题,今天明白了,很感谢
kuing 7# 2013-3-6 00:57
其实我意思是我上面提供了链接,你们能不能也提供一些。 没链接截图也行,注明出处
ccnu_chb_ycb 8# 2013-3-6 14:55
这个是长春 成黎明  老师给出的解答,不错系数的配凑过程说是省去了
kuing 9# 2013-3-6 15:09
配凑方法大概也是待定系数,然后解方程了。 PS、4# 说的“计神的作差配方法”有吗?thank you
ccnu_chb_ycb 10# 2013-3-6 15:25
我真的没有啊,我后来又搜集到宋庆老师博客里面的相关的一些资料,你可以看看 http://blog.sina.com.cn/s/blog_4c11310201013vyo.html
kuing 11# 2013-3-6 15:30
oh thanks
yes94 12# 2013-3-6 19:31
9# kuing 陈计:$16[2(a+b+c)^2(2a+4b+7c)-225abc]=(20a+8b+5c)(4b-5c)^2+(16a+64b+11c)(3c-2a)^2+20c(5a-6b)^2≥0$
kuing 13# 2013-3-6 19:39
13# yes94 原来如此……
yes94 14# 2013-3-6 20:12
14# kuing 分析一下他可能是怎么想出来的这个配方?
thread-1207-1-3.html: [几何] 请教一个比较角度大小的问题
abababa 1# 2013-3-5 13:39
本帖最后由 abababa 于 2013-3-5 18:12 编辑 $AB$是$\odot O$内部的线段,圆$AT_1B,AT_2B$和$\odot O$内切于$T_1,T_2$,若圆$AT_1B,AT_2B$的半径分别为$r_1,r_2$,且$r_1>r_2$,能否得出$\angle AT_1B<\angle AT_2B$? 感谢昨天网友帮忙画了图。 我用正弦定理作了,但是钝角的情况考虑不周全,应该怎么弄呢?
abababa 2# 2013-3-6 17:45
先证一下两个角不能都是钝角 设$AB$交$\odot O$于$A',B'$,这样$\angle A'T_1B'+\angle A'T_2B'=\pi$,然后$\angle A'T_1B'>\angle AT_1B,\angle A'T_2B'>\angle AT_2B$,就得到$\angle AT_1B+\angle AT_2B<\pi$,所以不能都是钝角
realnumber 3# 2013-3-9 07:42
两圆相交,那么互相分成两部分,即其中一圆一部分在另一圆内,一部分在外。 弦$AB$把圆$O_1$分成两部分,注意到$DO_2>CO_2$,所以大圆的劣弧在小圆内部,大圆的切点只能在优弧上,即$\angle AT_1B=ADB$为锐角. --ps,平几没看的习惯,教高中,...
abababa 4# 2013-3-9 10:11
3# realnumber 有两个问题,为什么$DO_2>CO_2$呢?当$O_2$在$O_1$右侧时确实是,但怎么证明$O_2$在$O_1$左侧时半径大小出现矛盾呢? 还有为什么由$DO_2>CO_2$,就能得出大圆劣弧在小圆内部呢? 两个问题用几何画板演示都是显然的,但怎么证明呢?
realnumber 5# 2013-3-9 13:31
4# abababa 没你这个情况的, 你先画大圆$O_1$,再添加圆$O_2$,就自然出现大小关系.
abababa 6# 2013-3-9 14:04
5# realnumber 从画图上看确实是没有,但要证明啊,我用几何画板画的也是,让$O_2$在连心线上移动,当移动到$O_1$左边时就出现$\odot O_2$比$\odot O_1$大的情况了,但要怎么证明它呢?不能因为画图这样就行了吧。
realnumber 7# 2013-3-9 19:17
证明:连接$O_1O_2$,交圆$O_1$于两点C,D,记关于点$O_1$,与$O_2$同侧点为C,异侧点为D. 记$\abs{O_1O_2}=m>0$,$\abs{DO_2}=r_1+m>\abs{CO_2}=r_1-m$,其中$r_1$为圆$O_1$半径(其中$r_1>r_2$). ---这个算不算?还是觉得一个作法就够了.
realnumber 8# 2013-3-9 19:24
4# abababa 因为等腰三角形ABD与等腰三角形ABC有公共底边,其中底边上的高较大,则顶角小,而这两个顶角就是弦AB所对的圆周角(注意:同弧所对圆周角相等).那么$T_1$对应角较小.
abababa 9# 2013-3-9 19:59
7# realnumber 如果这样规定$D,O_1$在$O_2$的同侧,$C,O_1$在$O_2$异侧,确实得到了$DO_2>CO_2$,但在这样规定下,为什么就得出了大圆劣弧在小圆内部呢?还是不理解
realnumber 10# 2013-3-9 23:00
楼上图形$r_1>r_2$没体现出来. 由垂径定理,可得出$O_1$离AB比$O_2$离AB远,优弧劣弧说明也是配合这点.既然上面得出一个圆周角是锐角,即对应点在优弧上.
realnumber 11# 2013-3-10 07:48
想到一个办法,假设大圆优弧在小圆内,则大圆的一直径在小圆内,这与$r_1>r_2$矛盾.
abababa 12# 2013-3-10 10:14
11# realnumber 谢谢。 这样就能证明$\angle AT_1B$一定是锐角了,再由正弦定理,另一个不管是锐角还是钝角都有$\angle AT_1B < \angle AT_2B$了。 原题是一个作图题,定圆$\odot O$内给定两定点$A,B$,在$\odot O$圆周上求作一点C使得$\angle ACB$最大 作法是一楼说的那位网友给出的,最后他就说在$\angle AT_1B,\angle AT_2B$里选一个最大的就是,我看图觉得一定是$AT_2B$最大,但就是不知道怎么证明。感谢楼上的证明。
thread-1208-1-3.html: [数列] 试一下草稿本,顺带转一个数列不等式
hongxian 1# 2013-3-5 14:28
本帖最后由 hongxian 于 2013-3-5 14:33 编辑 已知数列$\{a_n\}$满足,$a_1=1$,$a_n \cdot a_{n+1}=n$, (1)求证:$a_{n+2}=\frac{1}{a_{n+1}}+a_n$ (2)求证:$\frac{1}{a_1}+\frac{1}{a_2}+\cdots+\frac{1}{a_n}<3 \sqrt{n}-1$ 来自:http://bbs.pep.com.cn/forum.php?mod=viewthread&tid=2678631
realnumber 2# 2013-3-5 15:25
本帖最后由 realnumber 于 2013-3-5 17:37 编辑 (1)$a_{n+1}=\frac{n}{a_n},a_{n+2}=\frac{n+1}{a_{n+1}}=\frac{1}{a_{n+1}}+a_n$ (2)\[\frac{1}{a_1}+\frac{1}{a_2}+\ldots\+\frac{1}{a_n}=\frac{1}{a_1}+(a_3-a_2)+(a_4-a_3)+\ldots+(a_{n+1}-a_n)=a_{n+1}\] \[而n为奇数时,a_{n+1}=\frac{a_2a_3\ldots a_{n+1}}{a_1a_2\ldots a_n}=\frac{2\times4\times6\times\ldots\times{n}}{1\times3\times5\times\ldots\times{(n-1)}}\] \[而n为偶数时,a_{n+1}=\frac{a_3a_4\ldots a_{n+1}}{a_2a_3\ldots a_n}=\frac{3\times5\times7\times\ldots\times{n}}{2\times4\times6\times\ldots\times{(n-1)}}\] 后面应该可以处理了,只需要平方下,可以利用$\frac{(k-1)(k+1)}{k^2}<1$等处理
kuing 3# 2013-3-5 15:28
2# realnumber 没用草稿本么,又要修改了……
hongxian 4# 2013-3-5 15:32
3# kuing 发现打错了一个字,只有改一下了!
kuing 5# 2013-3-5 15:35
4# hongxian 噢,我不是说你。
realnumber 6# 2013-3-5 17:29
3# kuing en ,当时赶着去坐车,本来就打算修改,后面放缩,如果楼主知道了,就不需要我写了.
hongxian 7# 2013-3-5 19:13
本帖最后由 hongxian 于 2013-3-6 07:25 编辑 2# realnumber 谢谢了!求通项的高手,还是有点小问题 \[\frac{1}{a_1}+\frac{1}{a_2}+\cdots+\frac{1}{a_n}\] 好象应该为\[a_{n+1}+a_{n}-1\] $n$为奇数时需证$\dfrac{1 \times 3 \times 5\times \cdots \times n}{2 \times 4 \times 6\times \cdots \times (n-1)}+\dfrac{2 \times 4 \times 6\times \cdots \times (n-1)}{1 \times 3 \times 5\times \cdots \times (n-2)}<3 \sqrt{n}$ $a_{n+1}^2=\dfrac{(1 \times 3)\times(3 \times 5)\times \cdots\times(n-2)\times n \times n}{2^2 \times 4^2 \times \cdots \times(n-1)^2}<$ $n \Longrightarrow a_{n+1}<\sqrt n$ $a_{n+1}^2=\dfrac{2 \times(2 \times 4)\times(4 \times 6)\times \cdots\times(n-3)\times (n-1) \times (n-1)}{3^2 \times 5^2 \times \cdots \times(n-2)^2}<$ $2(n-1) \Longrightarrow a_{n}<\sqrt{2(n-1)}$ 所以需证$\sqrt n+\sqrt{2(n-1)}<3 \sqrt n \Longleftarrow \sqrt{2(n-1)}<2 \sqrt n \Longleftarrow 2n-2<4n \Longleftarrow 2n-2<4n \Longleftarrow -2<2n$这是明显的 $n$为偶数时$a_{n+1}=\dfrac{2 \times 4 \times\cdots\times n}{1 \times 3\times\cdots\times (n-1)}<\sqrt{2n}$,$a_n=\dfrac{1\times 3\times\cdots\times(n-1)}{2\times4\times\times\cdots\times(n-2)}$ $<\sqrt{n-1}$ 所以需证$\sqrt{2n}+\sqrt{n-1}<3\sqrt n \Longleftarrow \sqrt{n-1}<(3-\sqrt 2)\sqrt n\Longleftarrow -1<(10-6\sqrt 2)n$这也是明显的 不过$n=$1,2好象要单独讨论
realnumber 8# 2013-3-5 23:03
7# hongxian 恩,你是对的
yes94 9# 2013-3-5 23:56
2# realnumber 我想计算一下$a_3$的,结果代realnumber的公式,不知怎么算。
hongxian 10# 2013-3-6 07:42
9# yes94 $a_n=\begin{cases} \dfrac{2\times 4\times\cdots\times(n-1)}{1\times 3\times\cdots\times(n-2)}&n为奇数\\ \dfrac{1\times 3\times\cdots\times(n-1)}{2\times 4\times\times\cdots\times(n-2)}&n为偶数 \end{cases}$  $(n>2)$ $a_1=a_2=1$ $a_3$用上面的公式还是可以求的,$a_3=\dfrac2 1=2$
yes94 11# 2013-3-6 13:54
10# hongxian $a_3$是用你的公式代还是realnumber的公式代合适?
hongxian 12# 2013-3-6 14:35
11# yes94 我只能说我的公式是根据realnumber的思路推出来的。
thread-1209-1-3.html: [函数] 最小值问题
reny 1# 2013-3-5 14:29
本帖最后由 reny 于 2013-3-5 14:33 编辑 已知$a,b,c为常数,且c>b>a$ 对于函数\[y=\frac1 {x-a}+\frac1 {x-b}+\frac1 {c-x} \] 如何取$a,b,c$的大小,使得函数$y在(b,c)$上的最小值容易求呢?
reny 2# 2013-3-5 16:56
本帖最后由 reny 于 2013-3-5 16:57 编辑 刚才找到了一组,$a=1,b=2,c=5$时,可以得到当$x=6-\sqrt{5}$时,函数在(2,5)上取到最小值$ \frac{7+3{\sqrt 5}}{8}.$
yes94 3# 2013-3-5 18:50
2# reny 你在搞命题研究啊? 用柯西不等式就应该可以解决了。 只是待定系数,很难算。
kuing 4# 2013-3-5 20:20
刚才找到了一组,$a=1,b=2,c=5$时,可以得到当$x=6-\sqrt{5}$时,函数在(2,5)上取到最小值$ \frac{7+3{\sqrt 5}}{8}.$ reny 发表于 2013-3-5 16:56 这个数据不对吧,你再算算?
yes94 5# 2013-3-5 20:40
4# kuing 我也觉得, 所以算了一阵,很怕我自己算错了,最终还是没给结果
kuing 6# 2013-3-5 21:05
a=5, b=10, c=37 试试
kuing 7# 2013-3-5 21:38
a=5, b=10, c=37 试试 kuing 发表于 2013-3-5 21:05 讲讲构造过程,其实很简单,就是不断地解。 \[y=\frac1{x-a}+\frac1{x-b}+\frac1{c-x},\] 代入 $a=1$, $b=2$,求导,通分,得 \[y'=-\frac{2 c^2 x^2-6 c^2 x+5 c^2-4 c x^3+12 c x^2-10 c x+x^4-8 x^2+12 x-4}{(x-2)^2 (x-1)^2 (c-x)^2},\] 为求一个好的 $c$,令 $y'=0$,这里不是解出 $x$,而是解出 $c$,得到一个较大根 \[c=\frac{(x-2) (x-1)}{\sqrt{2 x^2-6 x+5}}+x,\] 为使没有根式,希望有 \[2 x^2-6 x+5=m^2,\] 其中 $m$ 和 $x$ 为整数,解 $x$,得较大根 \[x=\frac{3+\sqrt{2m^2-1}}2,\] 因此又需要 $2m^2-1$ 为完全平方数,代入 $m=1$ 发现解出的是 $x=1$, $2$,不行,再代 2, 3, 4, 5 发现 $m=5$ 就可以了,此时 $x=5$,代回去得 $c=37/5$,于是就得到一组好数据 $a=1$, $b=2$, $c=37/5$,因为齐次,同时乘以 $5$,就得到了楼上给出的。
yes94 8# 2013-3-5 22:12
7# kuing 不错啊! 原来和勾股数有关的。 下面就管理员的数据来个柯西不等式: $y=\dfrac1{x-5}+\dfrac1{x-10}+\dfrac1{37-x}=\dfrac9{9(x-5)}+\dfrac{16}{16(x-10)}+\dfrac{25}{25(37-x)}\geqslant\dfrac{(3+4+5)^2}{25\times37-9\times5-16\times10}=\dfrac15$. 当且仅当$x=25$取等号。
kuing 9# 2013-3-5 22:18
算是不小心地研究了一回命题方式……
yes94 10# 2013-3-5 22:23
9# kuing 那你以后就可以不止解题,然后命题, 可是很担心命出来的题很多人都不会做啊?
kuing 11# 2013-3-5 22:27
10# yes94 对于有意识有目标地命题,其实我兴趣不大……倒是有时在研究一些问题时可能会得到副chan品可以拿出来当题目
kuing 12# 2013-3-5 22:30
改编也可以考虑,推广什么的,不过推广其实也可以算是上面说的了
yes94 13# 2013-3-6 20:39
本帖最后由 yes94 于 2013-3-6 21:07 编辑 来三两道题目 ): (1)求$y=\dfrac1{x-1}+\dfrac1{x-6}+\dfrac1{33-x}$($x\in(6,33)$)的最小值。 (2)求$y=\dfrac1{x-2}+\dfrac1{x-7}+\dfrac1{34-x}$($x\in(7,34)$)的最小值。 (3)求$y=\dfrac1{x+90}+\dfrac1{x-1}+\dfrac1{126-x}$($x\in(1,126)$)的最小值。
reny 14# 2013-3-6 23:21
13# yes94 不错!你也是用$kuing$的方法找的吗? PS、对于使得$2m^2-1为完全平方数的m,怎么找? 如果用Mathematic$编程序的话,也不会编.
kuing 15# 2013-3-6 23:39
14# reny 这并不是一个简单问题,印象中好像是 Pell 方程…… 用 Mathematica 找的话,用 Table 列一下就是了……
yes94 16# 2013-3-7 00:07
14# reny 不是king的方法。 你怎么知道我的构造题目是合理的?(合理指的是取等号的$x$值是整数,最小值是有理数),做过我的题目吗?
kuing 17# 2013-3-7 00:28
16# yes94 那大概就是8#柯西的方法了? PS、打错ID了
reny 18# 2013-3-7 11:51
本帖最后由 reny 于 2013-3-7 11:53 编辑 13# yes94 $(1)y=\dfrac1{x-1}+\dfrac1{x-6}+\dfrac1{33-x}=\dfrac9{9(x-1)}+\dfrac{16}{16(x-6)}+\dfrac{25}{25(33-x)}\geqslant\dfrac{(3+4+5)^2}{25\times33-9\times1-16\times6}=\dfrac15$ $(2)y=\dfrac1{x-2}+\dfrac1{x-7}+\dfrac1{34-x}=\dfrac9{9(x-2)}+\dfrac{16}{16(x-7)}+\dfrac{25}{25(34-x)}\geqslant\dfrac{(3+4+5)^2}{25\times34-9\times2-16\times7}=\dfrac15$ $(3)y=\dfrac1{x+90}+\dfrac1{x-1}+\dfrac1{126-x}=\dfrac{25}{25(x+90)}+\dfrac{144}{144(x-1)}+\dfrac{169}{169(126-x)}\geqslant\dfrac{(5+12+13)^2}{169\times126+25\times90-144\times1}=\dfrac1{26}$ 我是用待定系数法硬算的. 总是和勾股数有关,结果总是挺简洁的. 前两个居然系数都没变,最小值也没变. 说下寻找的方法呗.
yes94 19# 2013-3-7 12:48
18# reny 那个ID弄错了成国王了,kuing帮我给改了嘛! 再来四道(可惜后面两道数据过大): (1)求$y=\dfrac1{x-3}+\dfrac1{x-8}+\dfrac1{35-x}$($x\in(8,35)$)的最小值。 (2)求$y=\dfrac1{x-4}+\dfrac1{x-9}+\dfrac1{36-x}$($x\in(9,36)$)的最小值。 (3)求$y=\dfrac1{x+80}+\dfrac1{x-11}+\dfrac1{136-x}$($x\in(11,136)$)的最小值。 (4)求$y=\dfrac1{x+70}+\dfrac1{x-21}+\dfrac1{146-x}$($x\in(21,146)$)的最小值。
reny 20# 2013-3-7 13:24
本帖最后由 reny 于 2013-3-7 13:25 编辑 19# yes94 用什么方法找的这些数据? 如果是这种形式:\[y=\frac{a_1} {x-b_1}+\frac{a_2}{x-b_2}+\frac1 {b_3-x}\], $a_1和a_2$为不相等的正数,这种情况恐怕就有点难找吧!
thread-1209-2-3.html:
yes94 21# 2013-3-7 13:46
20# reny 那肯定麻烦了噻!随便试试看: \[y=\frac{a_1} {x-b_1}+\frac{a_2}{x-b_2}+\frac1 {b_3-x}\], \[y=\frac{1}{\dfrac x{a_1}-\dfrac{b_1}{a_1}}+\frac{1}{\dfrac x{a_2}-\dfrac{b_2}{a_2}}+\frac1{b_3-x}\], 令$x=a_1a_2t$,则 \[y=\frac{1}{a_2t-\dfrac{b_1}{a_1}}+\frac{1}{a_1t-\dfrac{b_1}{a_1}}+\frac1 {b_3-a_1a_2t}\], 作代换后, \[y=\frac{1}{a_2t-a}+\frac{1}{a_1t-b}+\frac1{c-a_1a_2t}\],
thread-121-1-1.html: [地理/物理]是不是有个叫“地转偏向力”的东西?
kuing 1# 2011-10-19 02:00
忽然间隐约想起高中时期学地理,有一个问题总是想不明白,如果没记错,那叫“地转偏向力”,到底是咋回事哩? 有没有懂的讲解一下……是不是要用到物理的非惯性系什么的去扯呢?
戊概念·五 2# 2011-11-2 14:04
1# kuing 不可以是运动的合成么?!
秋风树林 3# 2011-11-2 17:00
你说的应该就是那个非惯性系下的科里奥利力...
kuing 4# 2011-11-2 17:03
3# 秋风树林 oh thanks,拿放大镜先……
thread-1210-1-3.html: [几何] 来自人教论坛的椭圆最大值[未完]
kuing 1# 2013-3-5 16:09
来自:http://bbs.pep.com.cn/forum.php?mod=viewthread&tid=2679466 按原贴回贴中里说的,题目大意是说,给定一个椭圆,任意过椭圆中心的弦AB,以及椭圆上任意点C,求CA+CB的最大值。
kuing 2# 2013-3-5 16:11
下面讨论取最大值时会是怎么样的情形。 来试试物理方法吧,速度分解。 我们不妨先将 C 点固定,让 AB 顺时针旋转,就像图中这样。 显然 A, B 的速度大小相同,方向相反,于是作出速度分解便可知,当那两个角度相等时 CA+CB 取得极值,不难观察出那是最大值。 或者,固定 AB 再让 C 在椭圆上走也差不多,相对来说这个好用些。 当那个速度是那平分线就取最大值了,也就是说,还是当 A-C-B 是光反射的情形取最大值。 待续……
kuing 3# 2013-3-5 17:13
将上述两个结论结合在一起会怎么样? 我们将 C 中心对称过去得 C' ,连成下图 发现了没?A-C-B-C'-A 是一条光反射回路!同时显然可以看出此时那些切线是垂直的,也就是说如果把 C' 处的切线作出来,那么四条切线围成一个矩形。 搞到这些美妙的结论,几乎忘了原题要求什么了 继续待续……
thread-1212-1-3.html: [数列] 接近$\sqrt n$的数列倒数求和
guanmo 1# 2013-3-5 16:52
如图 ______kuing edit in $\LaTeX$______ $b_n$ 为接近 $\sqrt n$ 的正整数,即 $\abs{b_n-\sqrt n}<\dfrac12$,求 $\left\{\dfrac1{b_n}\right\}$ 的前 $1000$ 项和。
kuing 2# 2013-3-5 17:44
先证明,$\{b_n\}$ 中,有 $2k$ 个 $k$($k=1$, $2$, $\ldots$)。 考虑使 $k-0.5<\sqrt{n}<k+0.5$ 成立的 $n$ 的个数。 两边平方得 $k^2-k+0.25<n<k^2+k+0.25$,所以 $k^2-k+1\leqslant n \leqslant k^2+k$,共 $2k$ 个。 于是 $\{1/b_n\}$ 中就有 $2k$ 个 $1/k$,每段加起来的时候都是 $2$,现在要求前 $1000$ 项的和,只要求一下余数就可以算了,这里不再详写。
kuing 3# 2013-3-5 20:13
合并了一下贴子、将多余的回贴删除、编辑了标题以及用 $\LaTeX$ 代码打了一下题目。
thread-1213-1-1.html: 72人记录
realnumber 1# 2013-3-5 23:05
突然明白为什么72记录了,无它,人教论坛那个时间登录不上,就....
kuing 2# 2013-3-5 23:09
1# realnumber 我觉得可能是5d6d的管理团队来xun查
isea 3# 2013-3-6 16:55
哈哈,巡查……
kuing 4# 2013-4-18 02:21
昨天103,没有哪个论坛碰巧昨天不能上吧,可见。。。
李斌斌755 5# 2013-4-18 11:26
昨天家里网络故障,单位里浏览一下
thread-1214-1-1.html: 劝学理由
realnumber 1# 2013-3-6 08:58
1.为维护世界和平--<功夫>. 2.可能要穿越,学点数理化,特别是化学和历史--<玄幻小说>. 3.心理学,概率统计--赌博 ---突然失去兴趣了,写不出了
kuing 2# 2013-3-6 15:15
这是干嘛呢?
realnumber 3# 2013-3-6 20:03
上课看到文科学生学数学毫无动力,胡乱想想,说真的,文科学什么数学呢?动力其实就考试~~
kuing 4# 2013-3-6 21:19
何必劝呢……要劝也应该反向……
realnumber 5# 2013-3-6 22:03
恩,没敢这么想过.实际上也就在课堂上混,老师和学生.
realnumber 6# 2013-3-6 22:04
5# realnumber 当作是春困,情绪低落吧~~
thread-1215-1-3.html: [不等式] 不怕复杂的帮忙弄下这个不等式问题
ccnu_chb_ycb 1# 2013-3-6 18:30

ccnu_chb_ycb 2# 2013-3-6 18:31
也可以借助  数学方面的软件得到结果,非常感谢
ccnu_chb_ycb 3# 2013-3-6 19:19
已经解决,如果没出错的话结果为七分之四
kuing 4# 2013-3-6 22:43
三元齐三次完全对称……只要 Schur 分拆就行了,系数非负是充要的。
thread-1216-1-3.html: [数列] 一道数列填空题(有点竞赛味道)
pengcheng1130 1# 2013-3-7 10:57

pengcheng1130 2# 2013-3-7 11:15
请高手求助!
yes94 3# 2013-3-7 13:02
2# pengcheng1130 先试几个特殊的n玩玩:(与解题毫无关系 ) 假若n=5,则5个数互不相同,所以最小两数之和无法用其余两数之和表示。 若n=6,则有5个数互不相同,这5个数中最小两数之和可能用剩下三数之一与第六个数之和表示。但是这5个数中最大两数之和却再也不能被剩下三数之一与第六个数之和表示了。
kuing 4# 2013-3-7 13:20
我看简直就是竞赛题不会……
yes94 5# 2013-3-7 13:21
本帖最后由 yes94 于 2013-3-7 13:28 编辑 3# yes94 看了v6的还是删掉这段话吧
v6mm131 6# 2013-3-7 13:26
n=13  很容易找到这么13个数字。1,1,1,1,2,2,3,4,4,5,5,5,5。
yes94 7# 2013-3-7 13:30
6# v6mm131 我觉得此题的作用就是反复尝试、探索, 尝试失败了,其实不是真正的失败,是为下一次成功做了很好的铺垫。
v6mm131 8# 2013-3-7 17:21
7# yes94 极端性考虑 最大和最小的值必然最少有4个
yuzi 9# 2013-3-7 19:35
都是些什么题呀。。。高考搞竞赛呀
thread-1217-1-3.html: [几何] C1与O距离
realnumber 1# 2013-3-7 14:22

kuing 2# 2013-3-7 14:30
大概跟这个差不多 http://kkkkuingggg.5d6d.net/thread-366-1-1.html
realnumber 3# 2013-3-7 14:31
本帖最后由 realnumber 于 2013-3-7 14:42 编辑 要使得$C_1$到O的距离最大距离,那么$C_1$也在平面$xoy$内,且$O,C_1$在直线AB两侧.$C_1$到AB为直径的圆上的点,最远就是过圆心. 所以移动AB,使得$O,C_1$,AB中点三点共线,所求距离为4. ,
realnumber 4# 2013-3-7 14:44
2# kuing thanks
thread-1218-1-3.html: [函数] f(x)>0恒成立,求k的取值范围
realnumber 1# 2013-3-7 14:24
本帖最后由 realnumber 于 2013-3-7 14:46 编辑 分子$x^2$与$x$项之间缺了+号.
realnumber 2# 2013-3-7 14:52
本帖最后由 realnumber 于 2013-3-7 15:11 编辑 $k=-1,k=\frac{1}{2}$时,$f(x)>0$不恒成立. 因为$x$趋于无穷,$f(x)>0$成立,那么$\frac{k+1}{2k-1}>0$,又若,分子或分母能分解成一次因式乘积,除非2个因式对应两根一样,否则$f(x)>0$,也不会恒成立. 所以有分子分母的判别式都不大于0.----好象写得乱. \[\frac{k+1}{2k-1}=\frac{k+3}{k+1}=\frac{2k-8}{k-4}=2,解得k=1,也符合题意.\] \begin{cases}\frac{k+1}{2k-1}>0\\ (k+3)^2-4(k+1)(2k-8)\le0\\(k+1)^2-4(2k-1)(k-4)\le0\end{cases}
yes94 3# 2013-3-7 18:11
本帖最后由 yes94 于 2013-3-7 18:12 编辑 2# realnumber 主要是楼上用的是必要条件解题,所以觉得乱。    不过,特殊的、恰如其分的必要条件会成为充要条件(这是需要不断尝试和自身解题功底的)。    好像从图像也可以解释楼上的过程似乎是对的。    如果分子、分母的两个二次函数(楼主已经的出一次函数不可行)的判别式均非负,则需要二次项系数同号(楼主用了必要条件:\[\lim_{x\to\infty}f(x)=\dfrac{k+1}{2k-1}>0\]这个极限来解释,这个必要条件还有点难想到呢)。    如果分子、分母的两个二次函数的判别式一个非负,另一个为正,由分子分母各自图像知,不能保证$f(x)>0$恒成立;     如果分子、分母的两个二次函数的判别式均为正数,则分子分母均和$x$轴有交点,由图像知同样不能保证$f(x)>0$恒成立; 但是极易遗漏$\dfrac{k+1}{2k-1}=\dfrac{k+3}{k+1}=\dfrac{2k-8}{k-4}=2$,解得$k=1$,这也符合题意.     综上所述:$\begin{cases}(k+1)(2k-1)>0\\ (k+3)^2-4(k+1)(2k-8)\leqslant0\\(k+1)^2-4(2k-1)(k-4)\leqslant0\end{cases}$    或$k=1$
realnumber 4# 2013-3-7 18:16
在分子分母是2 次且不能分解的情况下,还可以分类讨论,分子分母都正或都负--解这两类时,注意到是k的一次式,所以可以尝试把k解出来,用处理恒成立的办法处理.这样就不需要极限了.
thread-1219-1-3.html: [不等式] new Vasc inequality
pxchg1200 1# 2013-3-7 14:30
If $a,b,c\ge -3$ such that $a+b+c=3$, then \[\frac 1{a^2}+\frac 1{b^2}+\frac 1{c^2}\ge \frac 1{a}+\frac 1{b}+\frac 1{c}.\]
thread-122-1-1.html: 分式简打
kuing 1# 2011-10-19 02:30
标准分式的 LaTeX 代码格式,是 \frac{分子}{分母} 比如分数 1/2 的标准打法是 \frac{1}{2},分数 (a+b)/(c+d) 的标准打法是 \frac{a+b}{c+d}。 但有些时候可以简化输入,以下举若干例子说明具体规则。 标准输入 简化输入 \frac{1}{2}+\frac{1}{a} \frac12+\frac1a \frac{1}{23}+\frac{2}{a^2} \frac1{23}+\frac2{a^2} \frac{12}{3}+\frac{2a}{b} \frac{12}3+\frac{2a}b \frac{12}{34}+\frac{2a}{3b} 没得简化 \frac{a}{b}+\frac{c}{2} \frac ab+\frac c2 (留意空格) \frac{a+b}{c}+\frac{a}{b+c} \frac{a+b}c+\frac a{b+c} (同样留意空格) \frac{1}{\sqrt{a+b}}+\frac{\sqrt{a+b}}{2} \frac1{\sqrt{a+b}}+\frac{\sqrt{a+b}}2 (经测试,\sqrt{a+b}两边的花括号不能省) \frac{1}{\alpha}+\frac{\pi}{2} \frac1\alpha+\frac\pi2 \frac{\theta}{\pi}+\frac{2}{\sin\pi} \frac\theta\pi+\frac2{\sin\pi} 测试上表效果: \begin{align*} 标准输入的结果&&&&&简化输入的结果\\ \frac{1}{2}+\frac{1}{a}&&&&&\frac12+\frac1a\\ \frac{1}{23}+\frac{2}{a^2}&&&&&\frac1{23}+\frac2{a^2}\\ \frac{12}{3}+\frac{2a}{b}&&&&&\frac{12}3+\frac{2a}b\\ \frac{12}{34}+\frac{2a}{3b}&&&&&没得简化\\ \frac{a}{b}+\frac{c}{2}&&&&&\frac ab+\frac c2 (留意空格)\\ \frac{a+b}{c}+\frac{a}{b+c}&&&&&\frac{a+b}c+\frac a{b+c} (同样留意空格)\\ \frac{1}{\sqrt{a+b}}+\frac{\sqrt{a+b}}{2}&&&&&\frac1{\sqrt{a+b}}+\frac{\sqrt{a+b}}2\\ \frac{1}{\alpha}+\frac{\pi}{2}&&&&&\frac1\alpha+\frac\pi2\\ \frac{\theta}{\pi}+\frac{2}{\sin\pi}&&&&&\frac\theta\pi+\frac2{\sin\pi} \end{align*} 据此,你觉得 \frac123 、\frac{123}456 、\frac abcd 、\frac{ab}cd 、\frac a{bc}def 、 \frac ab{cd} 、\frac1a^2 、 \frac2\sin\pi 会出什么效果呢? 回复可见答案。 本帖隐藏的内容需要回复才可以浏览
kuing 2# 2011-10-19 02:52
差点忘记说: \dfrac 同理。
kuing 3# 2011-10-20 18:17
更新了一下。
贵族风铃 4# 2011-10-20 21:07
让我来瞧一瞧看一看 嘿嘿
kuing 5# 2011-10-20 23:48
4# 贵族风铃
图图 6# 2011-10-24 23:36
.
kuing 7# 2011-10-24 23:43
6# 图图 又是简略得只有一点 要是我就干脆空格
kuing 8# 2011-11-11 11:42
\frac1a^2  与  \frac{1^2}a 的区别: \[\frac1a^2  与  \frac{1^2}a\]
fredjhon 9# 2012-4-2 08:51
本帖最后由 fredjhon 于 2012-4-2 08:53 编辑 试一下看看,\dfrec{a}{b} 怎么打不出来分式啊
thread-1220-1-2.html: [不等式] 刚才352发来的一道简单最值
kuing 1# 2013-3-8 14:17
令 $a=\sin\alpha $, $b=\sin\beta $, $c=\sin\gamma $,则 $a$, $b$, $c\in[-1,1]$,问题等价于求 \[f(a,b,c)=\sqrt{\abs{a-b}}+\sqrt{\abs{b-c}}+\sqrt{\abs{c-a}}\] 的最大值。 由于任意交换两元位置原式的值不变,所以可以不妨设 $1\geqslant a\geqslant b\geqslant c\geqslant -1$,则 \[f(a,b,c)=\sqrt{a-b}+\sqrt{b-c}+\sqrt{a-c},\] 显然关于 $a$ 递增,关于 $c$ 递减,所以 \[f(a,b,c)\leqslant f(1,b,-1)=\sqrt{1-b}+\sqrt{b+1}+\sqrt2 \leqslant \sqrt{(1-b+b+1)(1+1)}+\sqrt2=2+\sqrt2,\] 当 $a=1$, $b=0$, $c=-1$ 时取等。
kuing 2# 2013-3-8 16:16
三元太简单,推广一下试试。 尝试四元,先轮换吧,$a$, $b$, $c$, $d\in[-1,1]$,求 \[f(a,b,c,d)=\sqrt{\abs{a-b}}+\sqrt{\abs{b-c}}+\sqrt{\abs{c-d}}+\sqrt{\abs{d-a}}\] 的最大值。 注意,现在是轮换对称,所以不可以像上面那样不妨设,但是,咳!其实这样更简单,显然 $\sqrt{\abs{a-b}}\leqslant \sqrt2$ 等等,故 $f(a,b,c,d)\leqslant 4\sqrt2$,当 $a=c=1$, $b=d=-1$ 时取等。 还是玩全对称的吧,$a$, $b$, $c$, $d\in[-1,1]$,求 \[f(a,b,c,d)=\sqrt{\abs{a-b}}+\sqrt{\abs{a-c}}+\sqrt{\abs{a-d}}+\sqrt{\abs{b-c}}+\sqrt{\abs{b-d}}+\sqrt{\abs{c-d}}\] 的最大值。 不妨设 $1\geqslant a\geqslant b\geqslant c\geqslant d\geqslant -1$,则 \[f(a,b,c,d)=\sqrt{a-b}+\sqrt{a-c}+\sqrt{a-d}+\sqrt{b-c}+\sqrt{b-d}+\sqrt{c-d},\] 显然关于 $a$ 递增,关于 $d$ 递减,所以 \[f(a,b,c,d)\leqslant f(1,b,c,-1)=\sqrt{1-b}+\sqrt{1-c}+\sqrt{b-c}+\sqrt{b+1}+\sqrt{c+1}+\sqrt2,\] 卡住了,不是简单的事情。 看来元数的推广并不容易哟,待续……
realnumber 3# 2013-3-8 21:25
2# kuing 先换个元,c换为-c,容易得b>0,c>0才可能取最大 固定b+c,对b求导,得到$\frac{1}{\sqrt{1+b}}+\frac{1}{\sqrt{1-c}}=\frac{1}{\sqrt{1-b}}+\frac{1}{\sqrt{1+c}}$,仅b=c时,导数为0(若b>c代入不成立;b<c也不成立),且是取极大值处. 那么问题即为求$f(b)=2\sqrt{1-b}+2\sqrt{1+b}+\sqrt{2b},b\in [0,1]$的最大值.接下来,求导得$\frac{1}{\sqrt{1+b}}+\frac{1}{\sqrt{2b}}=\frac{1}{\sqrt{1-b}}$,在[0,1]有唯一解,去根号后是个b的四次方程. --还是没习惯事先打草稿 .删去重发,好象也和打草稿效果类似.
reny 4# 2013-5-7 21:44
本帖最后由 reny 于 2013-5-7 21:59 编辑 波哥问了这样一题:已知$x,y,z\in[0,1]$,求$x\sqrt{1-y}+y\sqrt{1-z}+z\sqrt{1-x}$的最大值. 不确定论坛出现过没有,没有翻到. 随便链接一个#1推广:http://blog.sina.com.cn/s/blog_4c11310201017ntd.html(大家罗增儒)
kuing 5# 2013-5-7 22:18
波哥问了这样一题:已知$x,y,z\in[0,1]$,求$x\sqrt{1-y}+y\sqrt{1-z}+z\sqrt{1-x}$的最大值. 不确定论坛出现过没有,没有翻到. ... reny 发表于 2013-5-7 21:44 有:http://kkkkuingggg.5d6d.net/view ... &page=1#pid8450
isea 6# 2013-5-8 00:10
竟然把主楼看懂了,鲜见
李斌斌755 7# 2013-5-8 00:28
6# isea 一样,刚能看懂,不知笔下如何生花!
thread-1221-1-3.html: [不等式] 由2次变为3次的不等式
reny 1# 2013-3-9 15:09
若$a,b,c\ge0,a+b+c=1,$我们知道 \[(1-a^2)^2+(1-b^2)^2+(1-c^2)^2\leqslant3(1-\frac1 {3^2})^2\] 而$(1-a^3)^3+(1-b^3)^3+(1-c^3)^3\leqslant3(1-\frac1 {3^3})^3$还成立吗?
reny 2# 2013-3-9 15:22
半凹半凸性没有变,但是最后化简后得到一个不等式$$2(1-\frac{(1-c)^3}{8})^3+(1-c^3)^3\leqslant3(1-\frac1 {3^3})^3$$ 这等价于 $$-\frac1 {1679616 }(-1 + 3 c)^2 (569417 + 523101 c - 332775 c^2 -       327267 c^3 - 503901 c^4 + 40095 c^5 + 130491 c^6 + 185895 c^7)\leqslant0$$
thread-1222-1-3.html: [不等式] 一个轮换不等式
reny 1# 2013-3-10 16:50
本帖最后由 reny 于 2013-3-10 22:08 编辑 前文有http://kkkkuingggg.5d6d.net/viewthread.php?tid=1083(不知有没有更简单的方法),顺便得 已知$a,b,c\in R^+,且abc=1$,证明$$\dfrac1{a^2+2b^2+3}+\dfrac1{b^2+2c^2+3}+\dfrac1{c^2+2a^2+3}\leqslant \dfrac12$$.
pxchg1200 2# 2013-3-10 17:54
这个不等式不成立,请再检查一下。
reny 3# 2013-3-10 22:07
2# pxchg1200 是有问题,没太注意啊. 现在把最初的问题改动了一下
reny 4# 2013-3-10 22:13
1# reny 贴个答案: $LHS\leqslant\dfrac1{2ab+2b+2}+\dfrac1{2bc+2c+2}+\dfrac1{2ca+2a+2}=\dfrac12$
yes94 5# 2013-3-10 22:16
4# reny 又是那个恒等式啊
reny 6# 2013-3-10 22:21
5# yes94 是滴。。。
thread-1223-1-3.html: [不等式] 上海中学的一堆不等式问题
pxchg1200 1# 2013-3-10 18:17

kuing 2# 2013-3-10 18:37
1# pxchg1200 咦?最后一个不是 >2 么
pxchg1200 3# 2013-3-10 18:39
2# kuing 是2,出题人2了
pxchg1200 4# 2013-3-10 18:41
2# kuing 第1题就是我前几天发的那贴,西哥暴力出来了。
yes94 5# 2013-3-10 21:45
4# pxchg1200 以前在百度文库的时候发现有个陆一平文库(命题人)
kuing 6# 2013-3-11 01:00
第三题前两天天书在粉丝群里提到过,当时我们都被四边形和根号吓倒了,其实四边形的条件在本题只是等价于三边之和大于第四边,没其他用处。 所以,我们可以不妨设 $a+b+c+d=1$ 且 $0<a\leqslant b\leqslant c\leqslant d$,则由 $a+b+c>d$ 得 $d<1/2$。 这里为了得到取等条件,我们允许退化情形,即将条件弱化为 $a+b+c+d=1$ 且 $0\leqslant a\leqslant b\leqslant c\leqslant d\leqslant 1/2$。 原不等式等价于 \[f(a,b,c,d)=\sum\sqrt{\frac a{1-a}}\leqslant 1+\frac3{\sqrt5}.\] 先证明如下引理:若 $x$, $y\geqslant 0$, $x+y\leqslant 1/2$,则 \begin{equation}\label{sbxghbdsyls1} \sqrt{\frac x{1-x}}+\sqrt{\frac y{1-y}}\leqslant 2\sqrt{\frac{x+y}{2-x-y}}, \end{equation} 等号成立当且仅当 $x=y$。 令 $x+y=p\leqslant 1/2$, $xy=q$,则 $q\leqslant p^2/4$,将式 \eqref{sbxghbdsyls1} 两边平方整理等价于 \[\frac{4p}{2-p}\geqslant \frac{2-p}{1-p+q}-2+2\sqrt{\frac{q}{1-p+q}},\] 令 \[g(q)=\frac{2-p}{1-p+q}-2+2\sqrt{\frac{q}{1-p+q}},\] 求导得 \begin{align*} g'(q)&=\frac{(1-p)\sqrt{\frac{1-p}{q}+1}-2+p}{(1-p+q)^2} \\ & \geqslant \frac{(1-p)\sqrt{\frac{4(1-p)}{p^2}+1}-2+p}{(1-p+q)^2} \\ & =\frac{(2-p)(1-2p)}{p(p-q-1)^2} \\ & \geqslant 0, \end{align*} 所以 \[g(q)\leqslant g\left( \frac{p^2}4 \right)=\frac{4p}{2-p},\] 故引理得证。 回到原题,由条件易知 $f(a,b,c,d)$ 存在最大值,下面证明 $f(a,b,c,d)$ 取最大值时必有 $a=b=c$。 用反证法,假设 $f(a,b,c,d)$ 取最大值时 $a$, $b$, $c$ 不全相等,则必有 $a<c$。注意到 $a+c\leqslant(a+b+c+d)/2=1/2$,故由引理得 \[f(a,b,c,d)<f\left(\frac{a+c}2,b,\frac{a+c}2,d\right),\] 所以当 $a$, $c$ 都变为其平均数时 $f$ 的值将更大,矛盾。 当 $a=b=c=x$ 时,$d=1-3x$,应有 $x\leqslant 1-3x$ 且 $3x\geqslant 1-3x$,得到 $x\in[1/6,1/4]$,而我们只要求 $h(x)=f(x,x,x,1-3x)$ 的最大值即可。 求导可证 $h(x)$ 递减(过程暂略),所以 $h(x)\leqslant h(1/6)=1+3/\sqrt5$,从而原不等式得证,等号成立当且仅当 $a=b=c=d/3$ 及其轮换,也就是说原题是取不到等号的。
thread-1224-1-3.html: [函数] 一道最值问题
guanmo 1# 2013-3-11 11:51
如图
kuing 2# 2013-3-11 12:20
好像很早前在哪里见过,能提供一下问题的出处吗?
yes94 3# 2013-3-11 12:26
2# kuing 人教的,据说是2009清华自主招生文科第5题 http://bbs.pep.com.cn/forum.php? ... &extra=page%3D3 http://wenku.baidu.com/view/0ee6681b6bd97f192279e9f5.html http://www.jyeoo.com/math2/repor ... 8-bc03-f152eddf71f7 http://www.doc88.com/p-741861720077.html
kuing 4# 2013-3-11 12:31
3# yes94 我以前可能也是在人教见过,但是肯定不是第一个链接那个。现在很少去人教了,那个贴我好像没看过
yes94 5# 2013-3-11 12:45
4# kuing 再看一下,结果不像了!
kuing 6# 2013-3-11 12:54
5# yes94 是喔…… 竟然有不同的题干,但问题那么像的题……各种奇怪
kuing 7# 2013-3-11 13:33
到底哪个才是正版……
realnumber 8# 2013-3-11 13:38
是不是漏了什么条件,即使a为常数也不对, $x_1=a-t=x_2,x_3=x_4=a+t$,当t趋于无穷时,$S_4$的值趋于正无穷. $x_1=a,x_2=a-2t,x_3=a+t=x_4$,当t趋于正无穷或负无穷时,$S_4$的值有一个趋于负无穷.
guanmo 9# 2013-3-11 17:23
解决了吗?
kuing 10# 2013-3-11 17:30
题目都还不知有没有搞错,咋解决……
thread-1225-1-3.html: [不等式] 上确界问题
reny 1# 2013-3-11 16:35
本帖最后由 reny 于 2013-3-11 18:48 编辑 设$a,b,c\geqslant0,且abc=1$,可以证明$$\dfrac1{a^2-a+1}+\dfrac1{b^2-b+1}+\dfrac1{c^2-c+1}\leqslant3$$,下面有这么一个问题: 设$a,b,c\geqslant0,且a+b+c=3$,求$$\dfrac1{a^2-a+1}+\dfrac1{b^2-b+1}+\dfrac1{c^2-c+1}$$的上确界$M,这是安振平老师类比提出的问题,我算了一下,M$取近似值3.0456,是否正确?请教诸位.
kuing 2# 2013-3-11 16:48
调整法呗 不妨设 $a\leqslant b\leqslant c$,则 $a+b\leqslant2$,由此易证 $f(a)+f(b)\leqslant 2f((a+b)/2)$,然后变成单变量函数最大值,会遇到三次方程的。
reny 3# 2013-3-11 18:12
2# kuing 这个函数在[0,3]上的凸凹性不一致,那不是就要分类讨论?   问:上确界会是一个简洁的数吗,我做出来不是滴.
kuing 4# 2013-3-11 18:15
3# reny 无需理会凹凸性,直接作差证。就像昨晚我在这里 http://kkkkuingggg.5d6d.net/view ... &page=1#pid7643 证那个引理那样
kuing 5# 2013-3-11 18:17
我刚才略算了一下后面会遇到了三次方程,就没继续算下去了,如无意外下不会是简单的数
reny 6# 2013-3-11 18:50
我去仔细看一下那种方法。
reny 7# 2013-3-11 21:38
本帖最后由 reny 于 2013-3-11 22:39 编辑 2# kuing $f(a)+f(b)\leqslant 2f((a+b)/2)$在[0,2]上不成立吧,那不就等价于它在[0,2]上是凹函数了? 我算了一下,感觉不对劲
kuing 8# 2013-3-11 21:42
2# kuing f(a)+f(b)\leqslant 2f((a+b)/2)在[0,2]上不成立吧,那不就等价于它在[0,2]上是凹函数了? 我算了一下,感觉不对劲 reny 发表于 2013-3-11 21:38 $f(a)+f(b)\leqslant 2f((a+b)/2)$ 对于 $a$, $b\in[0,2]$ 不成立,但是对 $a$, $b\geqslant0$ 且 $a+b\leqslant2$ 成立。 条件变了,跟凹凸性就没有必然关系了,就像 $\cos x$ 在 $(0,\pi)$ 上有凹有凸,但是对 $\triangle ABC$ 必有 \[\cos A+\cos B=2\cos\frac{A-B}2\cos\frac{A+B}2\leqslant2\cos\frac{A+B}2\] 这其实就是说,条件变强,不等式适用的范围能变广。
thread-1229-1-3.html: [数论] 来自人教群的简单初中数论题
kuing 1# 2013-3-12 18:30
群管-kuing  18:23:17 去分母得 ac=(b+1-a)(a+b) 于是 a=b+1-a, c=a+b 即 b=2a-1, c=3a-1 因为 c>=4,c 为质数,所以 c 为奇数,从而由 c=3a-1 知 a 为偶数,所以 a=2 爱好者-Nash(2770*****)  18:26:08 话说这个a=b+1-a, 可得a能被b+1整除 也可以得出a=2 群管-kuing  18:27:19 嗯,也行 话说我平时是不敢碰数论题的,要不是群里当时说是“八年级竞赛”,我也不会去想,原来还算简单……
yes94 2# 2013-3-12 18:52
最后a、b、c分别为多少?
abababa 3# 2013-3-12 19:40
一到质数不定方程的,多数我都会想到有一个是2,用用奇偶性
yes94 4# 2013-3-12 19:51
3# abababa 很多时候,就是利用了2是唯一的偶质数这一性质,其它质数都是奇数,
yes94 5# 2013-3-13 13:56
4# yes94 最后a、b、c分别为多少?
kuing 6# 2013-3-13 14:02
5# yes94 你不知道吗?
yes94 7# 2013-3-13 18:07
6# kuing 不知道,
thread-123-1-2.html: 我吹我吹我吹吹吹
贵族风铃 1# 2011-10-20 20:13
呼啦啦 呼啦啦 啊呼 唔啦..............
贵族风铃 2# 2011-10-20 20:15
PS:回忆大长今的片头曲....哇哈哈
贵族风铃 3# 2011-10-20 20:15
可 win! KUING!!!come on...
贵族风铃 4# 2011-10-20 20:16
唔唔唔聊....啊偶
kuing 5# 2011-10-20 20:46

贵族风铃 6# 2011-10-20 20:52
o hi o hi..
kuing 7# 2011-10-20 20:55
ohiyo...
贵族风铃 8# 2011-10-20 20:57
咸蛋超人会不会变身?
kuing 9# 2011-10-20 20:58
8# 贵族风铃 问云朵儿
贵族风铃 10# 2011-10-20 21:00
嗖.....~~ 怕不怕? 搜来了它→
贵族风铃 11# 2011-10-20 21:00
9# kuing 云朵儿是谁?        ——小小酥..
kuing 12# 2011-10-20 21:02
11# 贵族风铃 啥时候又多了个“小小酥”出来……
贵族风铃 13# 2011-10-20 21:03
是谁!? 小小酥..... 这广告没看过啊 回忆回忆思过思过
kuing 14# 2011-10-20 21:08
13# 贵族风铃 完全知道是啥 回忆什么?思过?
贵族风铃 15# 2011-10-20 21:10
14# kuing BE..就是让你回忆这个广告啦 。。思过=思考过去(看过的这个广告)
kuing 16# 2011-10-20 21:13
仍然不知是啥
贵族风铃 17# 2011-10-20 21:15
16# kuing 怎么了。。刚刚不是 完全知道是啥嘛,现在怎么还是不知道是啥了。。。 灰过去了耶!又灰过来了耶..
kuing 18# 2011-10-20 21:17
17# 贵族风铃 打少了个至关重要的“不”字
贵族风铃 19# 2011-10-20 21:17
推荐一些歌来。。
贵族风铃 20# 2011-10-20 21:18
18# kuing 哈哈哈 木有关系  啦啦啦种太阳...
thread-123-2-2.html:
kuing 21# 2011-10-20 21:18
19# 贵族风铃 最近听的都是2002年左右的歌
贵族风铃 22# 2011-10-20 21:19
21# kuing 歌,新新与不新,both OK
贵族风铃 23# 2011-10-20 21:20
多打了个无关紧要的字..
kuing 24# 2011-10-20 21:31
在CD里,懒得一一找名字……
贵族风铃 25# 2011-10-20 21:33
24# kuing 额额 那算咯..不然你自己唱一段录音传来啊呵呵
kuing 26# 2011-10-21 00:17
嗖.....~~ 怕不怕? 搜来了它→ 贵族风铃 发表于 2011-10-20 21:00 后面那个表情是搜什么关键词出来的啊?
贵族风铃 27# 2011-10-21 12:04
26# kuing 哈哈
thread-1230-1-2.html: [函数] 求函数$f(x)=|x-1|+|2x-1|+|3x-1|+……+|2011x-1|$的最小值。
yes94 1# 2013-3-12 18:46
求函数$f(x)=|x-1|+|2x-1|+|3x-1|+……+|2011x-1|$的最小值。 多种方法,方法不限,
kuing 2# 2013-3-12 19:27
好像是什么自主招生题还是什么? PS、二元运算符之间的省略号用 \cdots ,不要用中文的省略号……
yes94 3# 2013-3-12 19:37
2# kuing 对,是自主招生题,复制的,忘了改成\cdots和\abs{}
abababa 4# 2013-3-12 19:42
1# yes94 是不是要分区间呀,先判断大于1和小于0都取不到最小值,然后把0到1按1/n分区间讨论?
yes94 5# 2013-3-12 19:52
4# abababa 方法没有界定的,只要言之成理即可
hongxian 6# 2013-3-13 03:50
本帖最后由 hongxian 于 2013-3-13 04:00 编辑 5# yes94 $f(x)=\begin{cases}(1+2+\cdots+2011)x-2011&x \in [1,+\infty)\\ [(-1-2-\cdots-n)+(n+1)+(n+2)+\cdots+2011]x+n-(2011-n)&x\in\left[\frac{1}{n+1},\frac1n\right]\\ -(1+2+\cdots+2011)x+2011&n \in\left(-\infty,\frac{1}{2011}\right] \end{cases}$ $n\in N^*,n\leqslant 2010$再讨论$f(x)$的单调性
realnumber 7# 2013-3-13 13:43
$a$, $b\in\mbb R$, $A>B>0$ 时 $f(x)=A\abs{x-a}+B\abs{x-b}$ 当 $x=a$ 取最小值 $B\abs{a-b}$。 没想到这个过,我是这样 这一步需要事先做个系列,用数轴上的距离解释最好,$│x-a│+│x-b│,在a\le x\le b$取最小. $│x-a│+│x-b│+│x-c│,(a\le b\le c)$在$x=b$最小,可以推广到奇数个绝对值与偶数个
yes94 8# 2013-3-13 13:53
7# realnumber 那个解释很好, 只是万一系数A、B不是整数怎么办?不是有理数怎么办? 另外,不是正数又怎么办?
yes94 9# 2013-3-13 13:54
6# hongxian 这方法还是可以的,
realnumber 10# 2013-3-20 08:39
8# yes94 也许要一步推广,比如这样$y=e\abs{x-1}+\sqrt8\abs{x-2}+\pi\abs{x-3}+\sqrt{10}\abs{x-5}$,用几何画板做些实验,应该有简单规律的.
yes94 11# 2013-3-20 22:53
10# realnumber 有什么规律?
yes94 12# 2013-3-20 22:59
10# realnumber 几个无理数的数据太接近了! 最小值:$2(\sqrt2+\sqrt{10}+e)$
zwl1972 13# 2013-4-24 18:08
本帖最后由 zwl1972 于 2013-4-24 18:14 编辑 \begin{align*}        f(x)= & |x-1|+|2x-1|+|3x-1|+\cdots+|2011x-1| \\        = & \left|x-1\right|+2\left|x-\frac{1}{2}\right|+3\left|x-\frac{1}{3}\right|+\cdots+2011\left|x-\frac{1}{2011}\right|\\        =&\left[2011\left|x-\frac{1}{2011}\right|+2010\left|x-\frac{1}{2010}\right|+\cdots+1423\left|x-\frac{1}{1423}\right|+220\left|x-\frac{1}{1422}\right|\right]+\\         &+\left[1202\left|\frac{1}{1422}-x\right|+1421\left|\dfrac{1}{1421}-x\right|+\cdots+\left|1-x\right|\right]\\     {\color{red} { ( \quad|a|\ge a) }}\ge &\left[2011\left(x-\frac{1}{2011}\right)+2010\left(x-\frac{1}{2010}\right)+\cdots+1423\left(x-\frac{1}{1423}\right)+220\left(x-\frac{1}{1422}\right)\right]\\         &+\left[1202\left(\frac{1}{1422}-x\right)+1421\left(\dfrac{1}{1421}-x\right)+\cdots+\left(1-x\right)\right]\\         =&832\frac{ 491}{711}  {\color{red}{(iff \quad x=\frac{1}{1422}\quad eauality \quad holds)}}      \end{align*}
kuing 14# 2013-4-24 18:11
13# zwl1972 似乎用不了 textcolor 命令,改用 \color{red}{...} 吧
zwl1972 15# 2013-4-24 18:17
13# zwl1972 似乎用不了 textcolor 命令,改用 \color{red}{...} 吧 kuing 发表于 2013-4-24 18:11 kk 速度也太快了
yes94 16# 2013-4-24 21:09
13# zwl1972
李斌斌755 17# 2013-4-24 22:41
北大自主招生题
yes94 18# 2013-4-25 20:41
17# 李斌斌755 对,
thread-1231-1-3.html: [不等式] 一个不等式
FJM 1# 2013-3-12 21:53
这个问题有哪些证法呢,求教 ______kuing edit in $\LaTeX$______ 已知正数 $a$, $b$, $c$,求证 \[\frac{ab}{3a+4b+5c}+\frac{bc}{3b+4c+5a}+\frac{ca}{3c+4a+5b}\leqslant\frac1{12}(a+b+c).\]
yes94 2# 2013-3-13 18:22
一个类似不等式: $Let$ $a、b、c$ $be$ $positive$ $real$ $numbers$ .$Prove$ $the$ $inequality$\[\dfrac{ab}{a+b+2c}+\dfrac{bc}{b+c+2a}+\dfrac{ca}{c+a+2b}\leqslant\dfrac{a+b+c}4\]
yes94 3# 2013-3-13 19:00
本帖最后由 yes94 于 2013-3-14 18:35 编辑 据网友们讨论,可得: \[\dfrac{ab}{3a+4b+5c}\leqslant\dfrac{ab}{36}(\dfrac1{a+b}+\dfrac2{a+c}+\dfrac3{b+c})\], 然后再用排序可得楼主问题。 回楼下:编辑好了,还对不对?
kuing 4# 2013-3-14 13:34
3# yes94 次数都不对?
thread-1232-1-3.html: 网友问的一道似乎有几何背景的三角题[算是解决了]
kuing 1# 2013-3-12 22:57
题目:设 $a+b=60\du$, $c+d=60\du$, $e+f=60\du$, $\displaystyle\frac{\sin a}{\sin b}\cdot\frac{\sin c}{\sin d}\cdot\frac{\sin e}{\sin f}=1$。证明 \[\frac{\sin(2a+f)}{\sin(2f+a)}\cdot\frac{\sin(2e+d)}{\sin(2d+e)}\cdot\frac{\sin(2c+b)}{\sin(2b+c)}=1.\] 想到了角元塞瓦定理……不过 还是不会
kuing 2# 2013-3-13 15:33
通过软件辅助,居然找到恒等式 所以大家可以放心做。
Gauss门徒 3# 2013-3-15 20:39
数吧的家伙秒了
kuing 4# 2013-3-15 20:49
3# Gauss门徒 给个链接呗
kuing 5# 2013-3-15 23:16
3# Gauss门徒 翻到了http://tieba.baidu.com/p/2213327986
kuing 6# 2013-3-16 01:49
构造图形很妙,可惜不足以说明有负数的情形
kuing 7# 2013-3-16 16:41
通过积化和差与和差化积,再加上换元化简,可知本题等价为: 设 $A$, $B$, $C\in\mbb R$ 满足 $\sin A+\sin B+\sin C=0$,求证 \[\cos(A-2B)+\cos(B-2C)+\cos(C-2A)=\cos(2A-B)+\cos(2B-C)+\cos(2C-A).\] 不知这样会不会简单些
kuing 8# 2013-3-31 21:47
通过积化和差与和差化积,再加上换元化简,可知本题等价为: 设 $A$, $B$, $C\in\mbb R$ 满足 $\sin A+\sin B+\sin C=0$,求证 \[\cos(A-2B)+\cos(B-2C)+\cos(C-2A)=\cos(2A-B)+\cos(2B-C)+\cos(2C-A).\] 不知这样会不会简单些 kuing 发表于 2013-3-16 16:41 用一下欧拉公式…… \begin{align*} \cos x&=\frac{e^{ix}+e^{-ix}}2, \\ \sin x&=\frac{i(e^{-ix}-e^{ix})}2. \end{align*} 为方便书写,记 $e^{iA}=a$, $e^{iB}=b$, $e^{iC}=c$,则 \begin{align*} \sum \bigl(\cos (A-2B)-\cos (2A-B)\bigr)&=\frac12\sum (e^{i(A-2B)}+e^{i(2B-A)}-e^{i(2A-B)}-e^{i(B-2A)}) \\ & =\frac12\sum \left( \frac a{b^2}+\frac{b^2}a-\frac{a^2}b-\frac b{a^2} \right) \\ & =\frac12\sum \left( \frac{a^3-b^3}{a^2b^2}+\frac{b^3-a^3}{ab} \right) \\ & =\frac1{2a^2b^2c^2}\sum (a^3-b^3)c^2-\frac1{2abc}\sum (a^3-b^3)c, \end{align*} 因式分解易得 \begin{align*} \sum (a^3-b^3)c^2&=(a-b)(b-c)(c-a)(ab+bc+ca), \\ \sum (a^3-b^3)c&=(a-b)(b-c)(c-a)(a+b+c), \end{align*} 所以 \[\sum (\cos (A-2B)-\cos (2A-B))=\frac{(a-b)(b-c)(c-a)}{2abc}\left( \frac1a+\frac1b+\frac1c-a-b-c \right),\] 而 \[\sin A+\sin B+\sin C=\frac i2\left( \frac1a-a+\frac1b-b+\frac1c-c \right),\] 故此……
kuing 9# 2013-4-1 10:45
至于1#与7#为何等价,我暂时懒得写你们自己可以推一下……
kuing 10# 2013-4-1 21:01
没人鸟……
yes94 11# 2013-4-1 21:16
10# kuing 曲高和寡?还是高处不胜寒啊?
isea 12# 2013-4-1 21:30
http://tt.a.5d6d.com/userdirs/e/3/kkkkuingggg/attachments/forumid_26/13013120425527fe55b24d06d8.jpg没人鸟…… kuing 发表于 2013-4-1 21:01 哈哈,不是了,一是知足了,看到那个图是知足了,二是懒得灌水
yes94 13# 2013-4-1 22:55
12# isea 我喜欢灌水,
thread-1233-1-1.html: [不等式] 一道集训题
reny 1# 2013-3-13 15:02
已知$x_1,x_2,……,x_n为正数,且\prod_{i=1}^{n}x_i=1,k\in N^{+}$,求证 $$\sum_{i=1}^{n}\dfrac{1}{(x_i+1)^k}\geqslant min(\dfrac{n}{2^k},1)$$
yes94 2# 2013-3-13 18:05
1# reny 好像在哪里看过?hjj的书?
reny 3# 2013-3-13 19:23
2# yes94 是的。不知道怎么做。
yes94 4# 2013-3-14 18:37
3# reny hjj的书没答案?
reny 5# 2013-3-14 20:00
4# yes94 只有n=4,k=2的答案. 不知一般地,怎么分类讨论。。。
tan9p 6# 2013-5-10 17:47
做代换 $x_i = \frac{y_i}{y_{i+1}}\quad ( y_{n+1} = y_1)$ 不等式变为$$\sum{\frac{y_i^k}{(y_i+y_{i+1})^k}}\geq\mathrm{min}\{1,\frac{n}{2^k}\}$$ 是不是好做点?
第一章 7# 2013-5-10 19:02
1# reny 好像在哪里看过?hjj的书? yes94 发表于 2013-3-13 18:05 hjj? 就是韩京俊吧? 刚刚看到pinyin,我承认我邪恶了,
kuing 8# 2013-5-11 17:59
7# 第一章 含鸡鸡?
realnumber 9# 2013-5-15 09:20
本帖最后由 realnumber 于 2013-5-15 12:12 编辑 设$e^{t_i}=x_i$,$f(t)=\frac{1}{{(1+e^t)}^k}$, 可得$f'(t)<0;f''(t)>0 \iff e^t>\frac{1}{k} $,即函数单调递减,且在$(-∞,t_0)上凸,(t_0,+∞)下凹,其中ke^{t_0}=1$, 以下用调整法,$t_1+t_2+\cdots+t_n=0$ 若有$t_p,t_q \in [t_0,+∞),$由jessen不等式可得$f(t_p)+f(t_q)\ge 2f(\frac{t_p+t_q}{2})$,可令 $t'_p=t'_q=\frac{t_p+t_q}{2}$, 若有$t_p,t_q \in (-∞,t_0]$,可令$t'_p=t_p+t_q-x_0,t'_q=t_0$,得$f(t_p)+f(t_q) \ge f(t'_p)+f(t'_q)$ 通过这样调整,最后最多有一个$t_i$在$(-∞,t_0)$, 如此,最小值在$t_1= \cdots=t_n=0$或$t_1=(n-1)s,t_2=t_3=\cdots=t_n=s,s<0$ 进一步计算可得$s→-∞$取到.(没验算,猜的)
yes94 10# 2013-5-15 23:13
3# reny 找不到hjj的资料了
thread-1234-1-3.html: [数论] 滚轴上的数字--转自解题群
realnumber 1# 2013-3-14 15:02

kuing 2# 2013-3-14 15:16
我发现我无法理解此题的题意……
yes94 3# 2013-3-14 18:32
2# kuing 我也是,字多的题,眼睛始终要怠工,怎么办?
thread-1235-1-3.html: [函数] 经典一个函数考试题
realnumber 1# 2013-3-14 15:05
广州--陈老师(136*****45) 10:07:00
kuing 2# 2013-3-14 15:23
2008 全国 II 理 《数学空间》总第3期P7~8 http://www.pep.com.cn/rjwk/gzsxs ... 0110516_1041452.htm
yes94 3# 2013-3-14 18:25
来一个命题组的答案:
thread-1236-1-3.html: [数论] 商是整数
Gauss门徒 1# 2013-3-14 15:24
若$\frac{m^3+1}{mn-1}$是整数,求所有满足要求的$(m,n)$
零定义 2# 2013-3-15 17:02
35届IMO?
kuing 3# 2013-3-15 17:23
出处dang牛比
thread-1237-1-3.html: [几何] 12年江苏解几题改成求轨迹,不知有没有纯代数方法?
hongxian 1# 2013-3-14 20:09
已知:$A$,$B$,是椭圆$\dfrac{x^2}{2}+y^2=1$上位于$x$轴同侧的两点,且直线$AF_1$与直线$BF_2$平行,$AF_2$与$BF_1$交于点$P$,求点$P$的轨迹方程。 几何法还是可以做的,用纯代数法消元没成功,方程组列下来,不知有没有高手能消元成功? 解:设$A(x_1,y_1)$,$B(x_2,y_2)$,$P(x,y)$ 则$\displaystyle\begin{cases}\frac{x_1^2}{2}+y_1^2=1\\ \frac{x_2^2}{2}+y_2^2=1\\ \frac{y_1}{x_1+1}=\frac{y_2}{x_2-1}\\ y=\frac{y_2}{x_2+1}(x+1)\\ y=\frac{y_1}{x_1-1}(x-1) \end{cases}\Longrightarrow ?$,后面消元没成功
kuing 2# 2013-3-14 21:32
将后两式代入前三式消去 $y_1$, $y_2$。 由第三式解出 $x_2$ 代入前两式消去 $x_2$。 此时前两式都是关于 $x_1$ 的二次式,消去二次项后再代入任一个即得。
hongxian 3# 2013-3-15 12:00
2# kuing 有点暴力,算到后来还是没有勇气算下去,虽然有时想既然能几何何必代数,但印象当是代数法应该是能够做出来的,晚上再算一下!
kuing 4# 2013-3-15 13:15
我没往下算,只是目测知道可以那样算,计算量可能很大。
hongxian 5# 2013-3-16 03:42
既然消元遇到困难,那就得反思这个方程本身的问题,居然发现一个关键条件($x$轴同侧)没有用到,所以方程的轨迹即使求出来也不是答案中的轨迹方程。看样子有待改进,初步估计极坐标+参数方程应该能够解决。
kuing 6# 2013-3-16 08:16
5# hongxian 其实不必担心这一点。 没有使用同侧的条件,说明上述方程组构成的轨迹包含了同侧或异侧两种情况,而事实上,显然异侧的时候那两条线根本没交点,所以其实“同侧或异侧”也等同于同侧。
hongxian 7# 2013-3-16 16:19
6# kuing 虽说异侧无交点,但毕竟有这种情况存在,有可能反映在方程中是一个恒不为0的因式。顺便发一个解法看行不行? 解:设$AF_1=r_1$,$BF_2=r_2$,$AF_1$,$BF_2$和$x$轴正向的夹角为$\theta$,则 $\displaystyle\begin{cases}r_1=\frac{\sqrt2}{2-\sqrt2\cos\theta}\\ r_2=\frac{\sqrt2}{2+\sqrt2\cos\theta}\\ y=\frac{r_1\sin\theta}{r_1\cos\theta-2}(x-1)\\ y=\frac{r_2\sin\theta}{r_2\cos\theta+2}(x+1)\end{cases}\Longrightarrow \begin{cases}y=\frac{\sqrt2\sin\theta}{3\sqrt2\cos\theta-4}(x-1)\\ y=\frac{\sqrt2\sin\theta}{3\sqrt2\cos\theta+4}(x+1)\end{cases}$ $\displaystyle\Longrightarrow \begin{cases}\cos\theta=\frac{4x}{3\sqrt2}\\ \sin\theta=\frac{4y}{\sqrt2}\end{cases}\Longrightarrow\frac{8}{9}x^2+8y^2=1$
kuing 8# 2013-3-16 16:25
7# hongxian 计算没问题的话就没问题
yes94 9# 2013-3-19 13:42
原高考题标准解答是几何与代数并用的,并不是单纯的几何法啊?
kuing 10# 2013-3-19 13:50
9# yes94 没看过……
thread-1238-1-3.html: 刚才352问的一道向量题
kuing 1# 2013-3-14 21:19
在梯形 $ABCD$ 中,$AD\sslash BC$, $\angle ABC=\pi/3$, $AD=1$, $BC=2$,$P$ 是腰 $AB$ 所在直线上的动点,则 $\bigl|3\vv{PC}+2\vv{PD}\bigr|$ 的最小值为______。 \begin{align*} 3\vv{PC}+2\vv{PD}&=3\bigl(\vv{PB}+\vv{BC}\bigr)+2\bigl(\vv{PA}+\vv{AD}\bigr)\\ &=3\bigl(\vv{AB}-\vv{AP}+2\vv{AD}\bigr)+2\bigl(-\vv{AP}+\vv{AD}\bigr)\\ &=3\vv{AB}-5\vv{AP}+8\vv{AD}, \end{align*} 延长 $AD$ 到 $D'$ 使 $AD'=8AD$,则 $\bigl|3\vv{AB}-5\vv{AP}+8\vv{AD}\bigr|$ 表示的是 $D'$ 到直线 $AB$ 上某个点的距离,而因为 $P$ 是直线 $AB$ 上的动点,所以 $3\vv{AB}-5\vv{AP}$ 可以取遍所有与 $\vv{AB}$ 平行的向量,也就是说那个点也可以取遍直线 $AB$,故此所求的最小值就是 $D'$ 到直线 $AB$ 的距离,亦即是 $D$ 到直线 $AB$ 距离的 $8$ 倍。
三下五除二 2# 2013-3-14 22:04
本帖最后由 三下五除二 于 2013-3-14 22:15 编辑 不得不佩服kuing 这个题我也想过那个系数3和2,应该有些玄机,也想过把2个分解,但我的思路还是固执地走在计算数量积啊,模啊什么的上面
yes94 3# 2013-3-14 23:17
这个几何解释好!
第一章 4# 2013-3-14 23:20
学一下k的分解:
yes94 5# 2013-3-14 23:33
4# 第一章 正准备输入的时候,发现和你的做法大同小异了
yes94 6# 2013-3-14 23:55
5# yes94 还是输入一下,看下latex的“环境”的威力: $\begin{align*} 3\vv{PC}+2\vv{PD}&=3\bigl(\vv{PB}+\vv{BC}\bigr)+2\bigl(\vv{PA}+\vv{AD}\bigr)\\ &=3\bigl(\vv{AB}-\vv{AP}+2\vv{AD}\bigr)+2\bigl(-\vv{AP}+\vv{AD}\bigr)\\ &=3\vv{AB}-5\vv{AP}+8\vv{AD}\\ &=k\vv{AB}+8\vv{AD}(令\vv{AP}=\dfrac{3-k}{5}\vv{AB}), \end{align*}$ 于是, $\begin{align*} \abs{3\vv{PC}+2\vv{PD}}^2 &=\abs{k\vv{AB}+8\vv{AD}}^2\\ &=k^2\abs{\vv{AB}}^2+16k\abs{\vv{AB}}\cdot\abs{\vv{AD}}\cos120\du+64\abs{\vv{AD}}^2\\ &=t^2-8t+64(令t=k\abs{\vv{AB}})\\ &=(t-4)^2+48\\ &\geqslant48, \end{align*}$ 故$\abs{3\vv{PC}+2\vv{PD}}\geqslant4\sqrt3$,等号是可以取到的吧.
kuing 7# 2013-3-15 00:13
6# yes94 环境是可以不用在两边加 $,就会居中,就像草稿本的示例那样
yes94 8# 2013-3-15 00:39
本帖最后由 yes94 于 2013-3-15 00:41 编辑 不得不佩服kuing 这个题我也想过那个系数3和2,应该有些玄机,也想过把2个分解,但我的思路还是固执地走在计算数量积啊,模啊什么的上面 三下五除二 发表于 2013-3-14 22:04 那个系数3和2,的确有些玄机,你这个思路也是可行的, 取点$E$在线段$CD$上,且$DE:EC=3:2$ $\abs{\dfrac35\vv{PC}+\dfrac25\vv{PD}}=\abs{\vv{PE}}\geqslant\dfrac45\sqrt3$, 当且仅当$EP\perp{AB}$上式取等号,此时可以计算出$\abs{\vv{PE}}=\dfrac45\sqrt3$。 故$\abs{3\vv{PC}+2\vv{PD}}=5\abs{\vv{PE}}\geqslant4\sqrt3$, 本方法还找到了真正的$P$点应该在直线$AB$的何处。 搞忘了上图,还不得不重新编辑一下, 回复k:在两边加 $,加惯了,最容易记得这个东东了,其他很难记
yes94 9# 2013-3-15 01:45
2# 三下五除二 “思路还是固执地走在计算数量积啊,模啊什么的上面 ”,其实计算数量积、模啊什么的也是可行的: 如图,记$\alpha=\angle{BPC}$,$\beta=\angle{DPA}$,由正弦定理可得, \[PC=\dfrac{\sqrt3}{\sin\alpha},PD=\dfrac{\sqrt3}{2\sin\beta},\] 于是, \begin{align*} \abs{3\vv{PC}+2\vv{PD}}^2 &=9\abs{\vv{PC}}^2+4\abs{\vv{PD}}^2+12{\vv{PC}}\cdot{\vv{PD}}\\ &=\dfrac{27}{\sin^2\alpha}+\dfrac{3}{\sin^2\beta}+\dfrac{18\cos(\pi-\alpha-\beta)}{\sin\alpha\sin\beta}\\ &=\dfrac{27}{\sin^2\alpha}+\dfrac{3}{\sin^2\beta}-\dfrac{18(\cos\alpha\cos\beta-\sin\alpha\sin\beta)}{\sin\alpha\sin\beta}\\ &=27\csc^2\alpha+3\csc^2\beta-18\cot\alpha\cot\beta+18\\ &=27(1+\cot^2\alpha)+3(1+\cot^2\beta)-18\cot\alpha\cot\beta+18\\ &=48+3(3\cot\alpha-\cot\beta)^2\\ &\geqslant48, \end{align*} 当且仅当$3\cot\alpha=\cot\beta$,即$\tan\alpha=3\tan\beta$时,取等号。 周五下午要出去耍两天周末,论坛就暂别两天啦。
李斌斌755 10# 2013-3-15 02:24
如图,A、B、C、D、P、F、G分别在等距的平行线上滑动。
第一章 11# 2013-3-15 05:33
话说你们都不用睡觉的?
三下五除二 12# 2013-3-15 07:49
都是高手啊,想起古人说的一句话:一切皆有可能!!!
kuing 13# 2013-3-15 08:18
11# 第一章 夜猫不少哇 :D good morning
第一章 14# 2013-3-15 08:29
本帖最后由 第一章 于 2013-3-15 08:37 编辑 最近肾不好,起来夜尿的时候就顺便回了个贴。 PS,现在才发现,那个李斌斌实在太暴力了!少儿不宜啊!
kuing 15# 2013-3-15 09:23
14# 第一章 夜尿回帖牛
isea 16# 2013-3-15 10:54
本帖最后由 isea 于 2013-3-15 10:57 编辑 那个系数3和2,的确有些玄机,你这个思路也是可行的, 取点$E$在线段$CD$上,且$DE:EC=3:2$ $\abs{\dfrac35\vv{PC}+\dfrac25\vv{PD}}=\abs{\vv{PE}}\geqslant\dfrac45\sqrt3$, 当且仅当$EP\perp{AB}$上式取等号 ... yes94 发表于 2013-3-15 00:39 想了一下,原来yes94兄已经给出这个类似的解法了。 借用这个图,只需要E点,其它虚线不需要。 找出 $3\vv {PC}+2\vv {PD}=5[(1-\dfrac25) \vv {PC}+\dfrac25 \vv {PD}]=5\vv {PE},\vv {CE} =\dfrac25 \vv {CD}$ (即图中的E点由来)所表示的向量,(2004年审订,人教B版97页例2是也)。 将最小,再化为几何意义,即图中,$E$到$AB$距离的5倍。 这个,个人觉得自然些。 PS:李斌斌755 构造,思维实在是开阔。 PPS:几天不打代码手就生了。
kuing 17# 2013-3-15 11:06
16# isea 李的方法应该说是最直接切入主题……要多长,延多长
yes94 18# 2013-3-15 12:39
李是几何法专家! 很多题他都喜欢用图形解决,本题他的方法最直截了当,假如把系数3,2改为300,203…… 话说用解析几何方法也是可行的,不动脑,动笔就行了: 以$B$为原点,$BC$为$x$轴建立坐标系,则直线$BA$的方程为$y=\sqrt3x$,于是可设$A(m$,$\sqrt3m)$,$D(m+1$,$\sqrt3m)$,$P(n$,$\sqrt3n)$,所以, \[3\vv{PC}+2\vv{PD}=(2m-5n+8,\sqrt3(2m-5n)),\]故 \begin{align*} \abs{3\vv{PC}+2\vv{PD}}^2&=(u+8)^2+(\sqrt3u)^2\\ &=4u^2+16u+64\\ &=(2u+4)^2+48\\ &\geqslant48 \end{align*} 当且仅当$u=2m-5n=-2$取等号。 周日晚上见!
yes94 19# 2013-3-21 18:41
还想到了一种方法,同乘单位法向量方法,然后用向量内积不等式立得
thread-1239-1-3.html: 昨晚网友问的一道三角无理数问题
kuing 1# 2013-3-15 13:34
昨今明 证明或者证否:当 $n>8$ 时,$\tan(2\pi/n)$ 是无理数。 不知大家有什么简单方法,我用了 n 倍角公式
kuing 2# 2013-3-15 13:44
引理 1 (正切的 $n$ 倍角公式): \[\tan n\theta = \frac{C_n^1 \tan^1 \theta - C_n^3 \tan^3 \theta + C_n^5 \tan^5 \theta - \cdots }{C_n^0 \tan^0 \theta - C_n^2 \tan^2 \theta + C_n^4 \tan ^4 \theta - \cdots }.\] 引理 2 :设 $f(x)=a_nx^n+a_{n-1}x^{n-1}+\cdots+a_1x+a_0$ 是一个整系数多项式,如果有理数 $v/u$ 是它的一个根,其中整数 $u$ 与 $v$ 互素,则 $u\mid a_n$,$v\mid a_0$。特别地,当 $a_n=1$ 时,$f(x)$ 的有理根都是整数,且为常数项 $a_0$ 的因数。 下面证明命题:对于 $n=3$, $5$, $6$, $7$ 以及所有 $\mbb N\ni n>8$,$\tan(2\pi/n)$ 都是无理数。 为方便书写,记 $t=\tan(2\pi/n)$。 (1)当 $n$ 为奇数时,可以直接验证 $n=3$, $5$, $7$ 时 $t$ 是无理数,而对于奇数 $n>8$,由引理 1 得 \[0=\tan\left(n\cdot\frac{2\pi}n\right)=\frac{C_n^1t-C_n^3t^3+C_n^5t^5-\cdots+(-1)^{(n-1)/2}C_n^nt^n}{C_n^0t^0-C_n^2t^2+C_n^4t^4-\cdots},\] 因为 $t\ne0$,所以得到 \[n-C_n^3t^2+C_n^5t^4-\cdots+(-1)^{(n-1)/2}t^{n-1}=0,\] 由引理 2 知,如果 $t$ 为有理数,由 $t$ 必为整数,但当 $n>8$ 时显然有 $0<t<1$,矛盾。 因此,对于所有大于 $1$ 的奇数 $n$,$t$ 都是无理数; (2)当 $n$ 为偶数时,$n$ 总可以表示为 $n=2^k\cdot p$,其中 $k$, $p\in\mbb N^+$ 且 $p$ 为奇数。 (2-1)若 $p=1$,即 $n=2^k$ 时,容易验证当 $n=16$ 时 $t$ 为无理数。 假如存在某个 $n_0=2^{k_0}>16$ 使 $t_0=\tan(2\pi/n_0)$ 为有理数,则由两倍角公式知 $\tan(4\pi/n_0)=2t_0/(1-t_0^2)$ 为有理数,即当 $n=n_0/2$ 时 $t$ 也为有理数,如此类推,最终推出 $n=16$ 时 $t$ 也为有理数,矛盾。 因此,对所有 $n=2^k\geqslant 16$,$t$ 都是无理数。 (2-2)若 $p>1$,假设存在某个 $n_0=2^{k_0}\cdot p$ 使 $t_0=\tan(2\pi/n_0)$ 为有理数,类似地,由两倍角公式最终可推得 $n=p$ 时 $t$ 也为有理数,与(1)的结论矛盾。 因此,对所有 $n=2^k\cdot p$,其中 $k$, $p\in\mbb N^+$ 且 $p$ 为大于 $1$ 的奇数,$t$ 都是无理数。 综上所述,命题获证。
thread-124-1-9.html: [几何] 圆锥曲线顺带测试下公式
GAM 1# 2011-10-20 22:22
本帖最后由 GAM 于 2011-10-20 22:24 编辑 求F1,F2分别是双曲线$\dfrac{X^2}{a^2}-\dfrac{Y^2}{b^2}=1$(a>0,b>0)的左右焦点,具有如下性质,若直线X=T交双曲线于点P,Q,$A_1$、$A_2$为双曲线的顶点,则$A_1P$,$A_2Q$交点的轨迹是椭圆$\dfrac{X^2}{a^2}+\dfrac{Y^2}{b^2}=1$,试对椭圆$\dfrac{X^2}{a^2}+\dfrac{Y^2}{b^2}=1$写出类似的性质,并予以证明
GAM 2# 2011-10-20 22:25
还是比较成功的哈
kuing 3# 2011-10-20 22:41
用 $k_1\cdot k_2$ 为定值那个性质就行了
thread-1240-1-3.html: [不等式] 昨晚网友问的一道三元三参不等式
kuing 1# 2013-3-16 08:52
千变万幻 2013-3-15 23:57:45 $a_1$, $a_2$, $a_3>0$, $a_1+a_2+a_3=1$, $0<\lambda_1<\lambda_2<\lambda_3$,求证 \[(a_1\lambda_1+a_2\lambda_2+a_3\lambda_3)\left(\frac{a_1}{\lambda_1}+\frac{a_2}{\lambda_2}+\frac{a_3}{\lambda_3}\right)\leqslant\frac{(\lambda_1+\lambda_3)^2}{4\lambda_1\lambda_3}.\] 昨晚一开始想到凸性,然后觉得不对头,后来躺床上心算,又以为是错题,结果直到刚才醒来,可能睡一觉头脑清醒了些,才发现昨晚后来还是想错了,其实凸性还是可以的,但是并不是一开始想的用于 $a$,而是用于 $\lambda$ 才对。 我们把左边看成 $\lambda_2$ 的函数,显然是下凸的,所以可以考虑 $\lambda_2$ 为 $\lambda_1$ 或 $\lambda_3$ 时的式子,而这两种情形不难发现等价于完全平方式,再利用定比分点,经过一些计算,最终可以得到如下恒等式: \begin{align*} & \frac{(x+z)^2(a+b+c)^2}{4xz}-(ax+by+cz)\left( \frac ax+\frac by+\frac cz \right) \\ ={}&\frac{(x-z)^2}{4xz}\left( \frac{z-y}{z-x}(a+b-c)^2+\frac{y-x}{z-x}(c+b-a)^2 \right)+\frac{b(z-y)(y-x)(ax+cz)}{xyz} \end{align*} 嗯,我为了方便书写,将 $a_1$, $a_2$, $a_3$, $\lambda_1$, $\lambda_2$, $\lambda_3$ 分别写成了 $a$, $b$, $c$, $x$, $y$, $z$。 是不是有点 JiChen 的感觉 PS、由此可见原题的等号取不到,除非可以让 $a$, $b$, $c$ 为 $0$,那么 $a=c=1/2$, $b=0$ 时取等。
kuing 2# 2013-3-16 13:56
天书(1846******)  13:50:52 http://tieba.baidu.com/p/616153092 就是套了下反向柯西的证明...
Tesla35 3# 2013-3-20 22:30
应该是这个题的离散形式
kuing 4# 2013-3-20 22:38
3# Tesla35 让我也想起了点东西,找到这篇东东
yes94 5# 2013-3-20 22:45
一般形式是反向柯西?那也叫波利亚—舍贵不等式,构造二次函数可证,
thread-1241-1-3.html: [函数] 正弦和
guanmo 1# 2013-3-16 20:28
如图
kuing 2# 2013-3-16 20:33
据说用复数,详情不记得……
kuing 3# 2013-3-16 20:55
抄来的(代码自己打的): 设 $\veps=\cos(2\pi/n)+i\sin(2\pi/n)$,则 $x^n=1$ 的所有根为 $1$, $\veps$, $\veps^2$, $\ldots$, $\veps^{n-1}$,所以 \[x^n-1=(x-1)(x-\veps)(x-\veps^2)\cdots(x-\veps^{n-1}),\] 又 \[x^n-1=(x-1)(x^{n-1}+x^{n-2}+\cdots+1),\] 所以 \[x^{n-1}+x^{n-2}+\cdots+1=(x-\veps)(x-\veps^2)\cdots(x-\veps^{n-1}),\] 上式令 $x=1$ 得 \[(1-\veps)(1-\veps^2)\cdots(1-\veps^{n-1})=n,\] 取模得 \[\abs{1-\veps}\abs{1-\veps^2}\cdots\abs{1-\veps^{n-1}}=n,\] 由隶莫佛公式有 \begin{align*} \abs{1-\veps^k}&=\abs{1-\cos\frac{2k\pi}n-i\sin\frac{2k\pi}n}\\ &=\sqrt{\left(1-\cos\frac{2k\pi}n\right)^2+\sin^2\frac{2k\pi}n}\\ &=\sqrt{2-2\cos\frac{2k\pi}n}\\ &=2\abs{\sin\frac{k\pi}n}, \end{align*} 于是 \[\abs{1-\veps}\abs{1-\veps^2}\cdots\abs{1-\veps^{n-1}}=2^{n-1}\sin\frac\pi n\sin\frac{2\pi}n\cdots\sin\frac{(n-1)\pi}n,\] 即得 \[\sin\frac\pi n\sin\frac{2\pi}n\cdots\sin\frac{(n-1)\pi}n=\frac n{2^{n-1}}.\] PS、明明是正弦积……
guanmo 4# 2013-3-16 20:55
积化和差能不能干?
第一章 5# 2013-3-16 21:21
数归,搞得定吧。
yes94 6# 2013-3-18 11:57
用复数证明,很好! 这样的恒等式(连乘)太多啦! 例如:\[\prod_{k=1}^{n-1}\abs{\cos\dfrac{k\pi}{n}}=\dfrac{1-(-1)^n}{2^n}\], 显然当$n$为偶数时,\[\prod_{k=1}^{n-1}\abs{\cos\dfrac{k\pi}{n}}=0\], 行间公式的效果也不错!
kuing 7# 2013-3-18 11:58
之前何版主在群里发过几个类似的,人教论坛里好像也有,有空翻翻
kuing 8# 2013-3-18 12:08
that's easy to 翻翻:http://bbs.pep.com.cn/forum.php?mod=viewthread&tid=250665
yes94 9# 2013-3-19 13:24
才发现,楼主的标题是正弦和,不是正弦积
thread-1242-1-3.html: [几何] 来自人教群的椭圆短轴圆切线
kuing 1# 2013-3-17 16:30
学生-独孤客(6108*****) 21. 已知椭圆 $E$ 的方程为 $x^2/4+y^2/3=1$,右焦点为 $F$,直线 $l$ 与圆 $x^2+y^2=3$ 相切于点 $Q$,且 $Q$ 在 $y$ 轴的右侧,设直线 $l$ 交椭圆 $E$ 于不同两点 $A(x_1,y_1)$, $B(x_2,y_2)$。 (1)若直线 $l$ 的倾斜角为 $\pi/4$,求直线 $l$ 的方程; (2)求证:$\abs{AF}+\abs{AQ}=\abs{BF}+\abs{BQ}$。 由中线长公式,有 \begin{align*} AF+AQ&=AF+\sqrt{OA^2-OQ^2} \\ &=AF+\sqrt{\frac{2AF^2+2AF_2^2-FF_2^2}4-b^2} \\ &=AF+\sqrt{\frac{2AF^2+2(2a-AF)^2-4c^2}4-b^2} \\ &=AF+\sqrt{AF^2+2a^2-2aAF-c^2-b^2} \\ &=AF+\sqrt{AF^2+a^2-2aAF} \\ &=AF+\sqrt{(a-AF)^2} \\ &=a. \end{align*}
yes94 2# 2013-3-17 21:38
以前有一道题是求三角形$ABF$ 的周长为$2a$,方法还可用参数方程。
第一章 3# 2013-3-18 07:26
回来了?立马赶来回帖?
呆呆 4# 2013-3-18 09:05
本帖最后由 呆呆 于 2013-3-18 10:01 编辑 $AQ^2=OA^2-OQ^2=x^2+y^2-b^2=x^2-\frac{b^2}{a^2}x^2=e^2x^2$
kuing 5# 2013-3-18 09:08
4# 呆呆 有输入错误,but nice! 呆呆
呆呆 6# 2013-3-18 10:01
本帖最后由 呆呆 于 2013-3-18 10:33 编辑 5# kuing 修改了 顺带说一句,此题即: 椭圆上任意一点到内切圆(以短轴为直径的圆)和外切圆(以长轴为直径的圆)的圆幂可用改点坐标表示。
yes94 7# 2013-3-18 11:46
回来了?立马赶来回帖? 第一章 发表于 2013-3-18 07:26 另外,还是把$2$楼说的数方程的过程完整的写出来吧(呆呆省略了开方和代换的过程): 设$A(a\cos\theta,b\sin\theta)$,因为点$Q$在$y$轴的右侧,故$\cos\theta>0$。所以, \begin{align*} \abs{AF}+\abs{AQ}&=a-ea\cos\theta+\sqrt{OA^2-OQ^2}\\ &=a-c\cos\theta+\sqrt{a^2\cos^2\theta+b^2\sin^2\theta-b^2}\\ &=a-c\cos\theta+\sqrt{a^2-b^2}\cos\theta\\ &=a \end{align*} 环境用起来,还真是舒服!
thread-1243-1-3.html: [数列] 来自人教群的一道数列重复无数项题
kuing 1# 2013-3-17 22:42
教师-tan9p(3653*****) $b_1=1$, $b_2=2$, $b_{n+1}b_{n-1}=b_n$, $a_{n+1}-a_n=b_n$。 问:$a_1$ 取何值时,$\{a_n/n\}$ 有重复的无数项? 由递推式易得 \[b_{n+3}=\frac{b_{n+2}}{b_{n+1}}=\frac{\frac{b_{n+1}}{b_n}}{b_{n+1}}=\frac1{b_n}\riff b_{n+6}=b_n,\] 列出 $b_n$ 的前六项为 $1$, $2$, $2$, $1$, $1/2$, $1/2$,于是得到 \[a_{n+6}-a_n=7,\] 写出 $a_n$ 的前六项为 $a_1$, $a_1+1$, $a_1+3$, $a_1+5$, $a_1+6$, $a_1+6.5$,于是得到 \begin{align*} \frac{a_{6k+1}}{6k+1}&=\frac{7k+a_1}{6k+1}=\frac{6a_1-7}{6(6k+1)}+\frac76 ,\\ \frac{a_{6k+2}}{6k+2}&=\frac{7k+a_1+1}{6k+2}=\frac{3a_1-4}{6(3k+1)}+\frac76 ,\\ \frac{a_{6k+3}}{6k+3}&=\frac{7k+a_1+3}{6k+3}=\frac{2a_1-1}{6(2k+1)}+\frac76 ,\\ \frac{a_{6k+4}}{6k+4}&=\frac{7k+a_1+5}{6k+4}=\frac{3a_1+1}{6(3k+2)}+\frac76 ,\\ \frac{a_{6k+5}}{6k+5}&=\frac{7k+a_1+6}{6k+5}=\frac{6a_1+1}{6(6k+5)}+\frac76 ,\\ \frac{a_{6(k+1)}}{6(k+1)}&=\frac{7k+a_1+6.5}{6(k+1)}=\frac{2a_1-1}{12(k+1)}+\frac76, \end{align*} 这样,当 $a_1$ 使以上任意一个式子右边第一项分子为 $0$ 即得无数项重复,这是充分条件。 必要性证明待续…… ____________ 见6#
kuing 2# 2013-3-18 09:20
真不知必要性怎么证……求助帮续……顶一下
realnumber 3# 2013-3-18 13:05
当符合第1部分不为零,又有无数个重复,那么6类项里至少有2个有重复,($C_6^2=15$,要检验15类) 若第一第二类有无数重复项, \[即存在a_1,使得\frac{6a_1-7}{6(6k+1)}=\frac{3a_1-4}{6(3m+1)}中k,m有无数对解.\] \[即\frac{6a_1-7}{3a_1-4}=\frac{6k+1}{3m+1}\] $存在a_1=1,m=2k$ 发现必要性不成立,临时修改,有无数组解.
kuing 4# 2013-3-18 13:31
看来感觉还是靠不住 这样搞下去是不是很复杂了……
kuing 5# 2013-3-18 13:43
等一下,我突然发现我们对题意的理解好像有偏差。 “有重复的无数项” 用数学语言来说到底等价于什么?
realnumber 6# 2013-3-18 14:09
应该是我理解错了?这样题目会太复杂. 如果按kuing的理解--数列中有无数项为同一个值,必要性很好证明啊,6类项第一部分都不为零时,可得都单调,某个值最多出现6次,不会是无数次.
realnumber 7# 2013-3-18 14:17
3楼情况应该说,存在无数对相等的项.也许就这样吧.
kuing 8# 2013-3-18 14:19
6# realnumber 是喔我戆鸠了……
thread-1244-1-3.html: [不等式] 数列不等式
guanmo 1# 2013-3-18 11:55
数列不等式
realnumber 2# 2013-3-18 12:02
$a_1$的值,有吗?
kuing 3# 2013-3-18 12:06
必须要有 $a_1$,否则左边要多大有多大……
guanmo 4# 2013-3-18 13:59
a1=3
第一章 5# 2013-3-18 15:30
这题应该是省略了一个小题:
yes94 6# 2013-3-19 12:40
楼主的类似题是:     已知$a_{n+1}=a_n^2-na_n+1$,当$a_1>3$时,求证:\[\sum_{k=1}^{n}\dfrac{1}{1+a_k}\leqslant\dfrac12\] 楼主的题估计起源于此题。
thread-1245-1-3.html: [几何] 几道平常几何题的逆命题——难
isea 1# 2013-3-18 12:24
本帖最后由 isea 于 2013-3-24 21:07 编辑 就地取材(否则,我一定给 两条内角平分线相等的三角形是等腰三角形 为首题),第一题: 原命题:已知:如图,在正$\triangle ABC$中,若$BD=CE=AF$。求证:$\triangle DEF$是正三角形。 逆命题:已知:如图,在$\triangle ABC$ 中,$BD=CE=AF$。$\triangle DEF$是正三角形。 问:$\triangle ABC$ 是否为正三角形? 如果是,请给出证明;如果不是,请举出反例。 此题部分解决:详细可参考 http://kkkkuingggg.5d6d.net/thread-735-1-1.html
isea 2# 2013-3-18 12:47
本帖最后由 isea 于 2013-3-19 01:26 编辑 第二道,源自人教论坛:http://bbs.pep.com.cn/forum.php?mod=viewthread&tid=2680885 原命题:在$\triangle ABC$中,$\angle BAC=60^\circ$,$\angle C=40^\circ$,$AP$平分$\angle BAC$交$BC$于$P$,$BQ$平分$\angle ABC$交$AC$于$Q$。求证:$AB+BP=BQ+AQ$。 逆命题:在$\triangle ABC$中,$AP$平分$\angle BAC$,交$BC$于$P$,$BQ$平分$\angle ABC$,交$CA$于$Q$,已知$\angle BAC=60^\circ$,且$AB+BP=AQ+QB$。问$\triangle ABC$的各角的度数的可能值是多少? 这个完全解决,因为这是2001年第42届IMO试卷第5题(倒数第二题)! 本楼就来说说此题。 三角法:如图标记角度与线段长度,有 $$AB=c,2\alpha+\beta=120^\circ,\beta=120^\circ-2\alpha$$ 在$\triangle ABQ$中 $$\dfrac c{\sin (\alpha+\beta)}= \dfrac {AQ}{\sin \alpha}=\dfrac {BQ}{\sin 60^\circ}$$ 在$\triangle ABP$中 $$\dfrac c{\sin (30^\circ +\beta)}=\dfrac {BP}{\sin 30^\circ}$$ 于是 \begin{align*} AQ=\dfrac {c \cdot \sin \alpha}{\sin (\alpha+\beta)}, BQ&=\dfrac {c \cdot \sin 60^\circ}{\sin (\alpha+\beta)}, BP=\dfrac {c \cdot \sin 30^\circ}{\sin (30^\circ +\beta)}\\ \\[1ex] AB+BP&=AQ+QB\\ \\[1ex] c+\dfrac {c \cdot \sin 30^\circ}{\sin (30^\circ +\beta)}&=\dfrac {c \cdot \sin \alpha}{\sin (\alpha+\beta)}+\dfrac {c \cdot \sin 60^\circ}{\sin (\alpha+\beta)}\\ \\[1ex] 1+\dfrac {\sin 30^\circ}{\sin (30^\circ +\beta)}&=\dfrac {\sin \alpha+\sin 60^\circ}{\sin (\alpha+\beta)}\\ \\[1ex] 1+\dfrac {\sin 30^\circ}{\sin (30^\circ +\beta)}&=\dfrac {2\sin (\dfrac\alpha2+30^\circ) \cdot \cos (\dfrac\alpha2-30^\circ)}{\sin (\alpha+\beta)}\\ \\[2ex] \sin (\alpha+\beta)&=\sin (120^\circ-\alpha)=\sin (\alpha+60^\circ)\\ \\[2ex] 1+\dfrac {\sin 30^\circ}{\sin (30^\circ +\beta)}&=\dfrac {2\sin (\dfrac\alpha2+30^\circ) \cdot \cos (\dfrac\alpha2-30^\circ)}{\sin (\alpha+60^\circ)}=\dfrac {2\sin (\dfrac\alpha2+30^\circ) \cdot \cos (\dfrac\alpha2-30^\circ)}{2\sin (\dfrac \alpha2+30^\circ) \cdot \cos (\dfrac \alpha2+30^\circ)}\\ \\[1ex] 1+\dfrac {\sin 30^\circ}{\sin (30^\circ +\beta)}&=\dfrac {\cos (\dfrac\alpha2-30^\circ)}{\cos (\dfrac \alpha2+30^\circ)}\\ \\[2ex] \dfrac {\sin 30^\circ}{\sin (30^\circ +\beta)}&=\dfrac {\cos (\dfrac\alpha2-30^\circ)}{\cos (\dfrac \alpha2+30^\circ)}-1=\dfrac {\cos (\dfrac\alpha2-30^\circ)-\cos (\dfrac \alpha2+30^\circ)}{\cos (\dfrac \alpha2+30^\circ)}\\ \\[2ex] \dfrac {\sin 30^\circ}{\sin (30^\circ +\beta)}&=\dfrac {-2\sin \dfrac\alpha2 \cdot \sin (-30^\circ)}{\cos (\dfrac \alpha2+30^\circ)}=\dfrac {2\sin \dfrac\alpha2 \cdot \sin 30^\circ}{\cos (\dfrac \alpha2+30^\circ)}\\ \\[1ex] \dfrac{1}{\sin (30^\circ +\beta)}&=\dfrac {2\sin \dfrac\alpha2}{\cos (\dfrac \alpha2+30^\circ)}\\ \\[5ex] 2\sin \dfrac\alpha2 \cdot \sin (30^\circ +\beta)&=\cos (\dfrac \alpha2+30^\circ)\\ \\[1ex] \sin (30^\circ +\beta)&=\sin (150^\circ-2\alpha)=\sin (30^\circ+2\alpha)\\ \\[1ex] 2\sin \dfrac\alpha2 \cdot \sin (30^\circ+2\alpha)&=\cos (\dfrac \alpha2+30^\circ)\\ \\[1ex] -(\cos (\dfrac{5\alpha}2+30^\circ)-\cos (-\dfrac{3\alpha}2-30^\circ))&=\cos (\dfrac \alpha2+30^\circ)\\ \\[1ex] -\cos (\dfrac{5\alpha}2+30^\circ)+\cos (\dfrac{3\alpha}2+30^\circ)&=\cos (\dfrac \alpha2+30^\circ)\\ \\[1ex] \cos (\dfrac{3\alpha}2+30^\circ)&=\cos (\dfrac \alpha2+30^\circ)+\cos (\dfrac{5\alpha}2+30^\circ)=2\cos (\dfrac {3\alpha}2+30^\circ) \cdot \cos (-\alpha)\\ \\[1ex] \cos (\dfrac{3\alpha}2+30^\circ)&=2\cos (\dfrac {3\alpha}2+30^\circ) \cdot \cos \alpha\\ \\[1ex] \cos (\dfrac{3\alpha}2+30^\circ)(1-2\cos \alpha)&=0\\ \\[1ex] \beta=120^\circ-2\alpha&>0 \Rightarrow 0^\circ < \alpha<60^\circ \Rightarrow 1-2\cos \alpha \ne 0\\ \\[2ex] \cos (\dfrac{3\alpha}2+30^\circ)&=0 \Rightarrow \dfrac{3\alpha}2+30^\circ=90^\circ\\ \alpha&=40^\circ \\[2ex] &\cdots\\ &\cdots \end{align*}
isea 3# 2013-3-19 01:40
本帖最后由 isea 于 2013-3-19 18:52 编辑 以上三角法,和差化积,积化和差,参考于刘文光;过程看上去比较复杂,其实主要就是将钝角化小,观察构成积为零的式子,从而得到结果。 但是,用三角来解此题,真心不好化简。 ============= 此楼介绍纯几何法,其实基本上就是同一法了,特别巧的是,最近对一些以往轴对称求角的题,也碰到了一些用同一法的,也是促成偶特开这一帖的原因之一。 提到纯几何法,那得说说李斌755兄台的同一法,如图: 不过,此解只能说明此解满足条件,是否还另有解,未知。 故此,有以下讨论(按下面图中较大的左图字母说明): 沿$AP$ 将 $AC$ 轴对称,得等边$\triangle ACC'$。 若$BP=BC'$,则由$AQ+QC=AC=AC'=AB+BC'=AB+BP=AQ+QB$,则需$QC=QB$, 亦是 $\dfrac{\angle ABC}2=\angle ACB=40^\circ$,合题…… 若$BP\ne BC'$,则在射线$BC'$上截取$BP'=BP$。 同理截取$QQ’=BQ$,连结$BQ’,PP'$,则等腰$\triangle BPP',\triangle BQQ'$。 由对称性,知$\angle P'=\angle AQ'P$,进一步知图中所有标记点的角的大小均是$\dfrac{\angle ABC}2$,进而有 $\angle PBQ=\angle PQ'B$,于是 $BP=PQ'=PP'=P'B$,等边$\triangle BP'P$,故:$\dfrac{\angle ABC}2=\angle BP'P=60^\circ$,但此时$\triangle ABC$不存在。 也就是说,满中条件的三角形有且只有一个,$\angle ABC=2\angle ACB=40^\circ$。 OK,不保证以上输入,字母之类完全正确,但讨论$BP$与$BC'$是否相等,是纯几入手的关键。
yes94 4# 2013-3-19 12:15
很好!本论坛几何专家之一非IC莫属! 估计很难有人来回你贴,我先来顶一个!
kuing 5# 2013-3-19 12:17
我一直觉得 IC 能将这类大贴整理成文拿去发biao,发哪里应该都没问题,或者IC打算出书?
isea 6# 2013-3-19 17:04
本帖最后由 isea 于 2013-3-19 19:23 编辑 楼上二位,不敢当不敢当,也只是取经及娱乐而已。 有各位,才好玩啊。 ============================ 第三题第1组,前两题,原命题平凡,但逆命题难度实在太大,这里给出相对平易近人一组垂心题(源自单墫),供大家消遣。 原命题:已知,在$\triangle ABC$中,$AD$为高线,$H$为垂心,求证:$\angle BHC = 180^\circ -\angle BAC$。 逆命题:已知,在$\triangle ABC$中,$AD$为高线,若$H$在高线$AD$上,且$\angle BHC = 180^\circ -\angle BAC$,$H$是否为垂心? 第三题第2组,图依然用第1组左图(不防先研究锐角三角形),但相对入手就困难多了—— 原命题:已知,在$\triangle ABC$中,$H$为垂心。 求证: $\triangle AEF, \triangle CDE, \triangle BDF$外接圆相交于$H$, 且$\dfrac{HE}{HF}=\dfrac{EC}{FB},\dfrac{HF}{HD}=\dfrac{FA}{DC},\dfrac{HD}{HE}=\dfrac{BD}{AE}$。 逆命题: 已知,如图,$D,E,F$在$\triangle ABC$分别在其边上, $\triangle AEF, \triangle CDE, \triangle BDF$外接圆相交于$H$, 且 $\dfrac{HE}{HF}=\dfrac{EC}{FB},\dfrac{HF}{HD}=\dfrac{FA}{DC},\dfrac{HD}{HE}=\dfrac{BD}{AE}$ , 是否有$AD,BE,CF$是$\triangle ABC$的三条高,即$H$是垂心?
李斌斌755 7# 2013-3-20 13:45
2# isea 先上一引理
李斌斌755 8# 2013-3-20 13:45
接上楼
李斌斌755 9# 2013-3-20 19:48
7# 李斌斌755 是否需要四边形ABCD为凸四边形?
isea 10# 2013-3-24 21:03
7# 李斌斌755 是否需要四边形ABCD为凸四边形? 李斌斌755 发表于 2013-3-20 19:48 这个引理不成立,其实,这个引理就是说:一且对角相等且一组对边相等的四边形是平行四边形,这是假命题。 这个反例,通常是直接拿一个等腰三角一砍,重新拼接。 人教主页上,有详细的讨论:http://www.pep.com.cn/czsx/jszx/ ... 0110809_1062436.htm 如,用圆的反例: 从全等角度上说,用SSA,必须判定两三角同是锐角或者同是钝角三角形。
isea 11# 2013-3-24 21:20
原命题:略,可参见逆命题。 逆命题: 若圆内接五边形的每个角相等,则它为正边形。 源来:2012年自主招生北约联考倒数第2题,其实$2n+1,n\in \mathbf{Z_+}$均成立。
李斌斌755 12# 2013-3-24 23:11
10# isea 学习
isea 13# 2013-3-24 23:20
10# isea 学习 李斌斌755 发表于 2013-3-24 23:11 共同学习,有“争议”才有进步
isea 14# 2013-3-26 00:12
第六道,又想起一道,想起来就先记下来,免得忘记了: 原命题:$Rt\triangle {ABC}$中,$\angle A=90^\circ,AB$大于$AC$,求证:过点$A$的中线,角平分线,高将$\angle A$四等分。 逆命题:在$\triangle {ABC}$中,$AB$大于$AC$,已知过点$A$的中线,角平分线,高将$\angle A$四等分,求角A的大小。 答案可以参考人教老帖:http://bbs.pep.com.cn/forum.php?mod=viewthread&tid=2597916
李斌斌755 15# 2013-3-29 12:14
本帖最后由 李斌斌755 于 2013-4-13 03:14 编辑 2#isea 再上一引理(引理不成立)
yes94 16# 2013-3-29 12:20
15# 李斌斌755 边边角不能判断两三角形全等的的反例
thread-1246-1-3.html: [数列] 来自粉丝群的一道数列极限
kuing 1# 2013-3-18 14:24
第一问如左边所示变成FAQ,第二问不知怎么做……
yes94 2# 2013-3-19 13:09
1# kuing 是啊,第一问变成:$b_{n+1}=b_n^2+b_n$ 和下面这两个数列都是FAQ吧: $a_{n+1}=a_n^2+a_n-1$ $a_{n+1}=a_n^2-a_n+1$
kuing 3# 2013-3-19 13:11
2# yes94 嗯所以发上来只为第2问
yes94 4# 2013-3-19 13:18
3# kuing Stolz公式呢?
realnumber 5# 2013-3-19 13:38
(2)错误的,因为$a_n>0.1$,所以极限应该趋于正无穷,而不是0.25.
kuing 6# 2013-3-19 13:49
5# realnumber 有道理……
thread-1247-1-1.html: align*环境 中 如何留白即空行
isea 1# 2013-3-19 00:26
本帖最后由 isea 于 2013-3-19 01:28 编辑 align*环境: \begin{align*} k \end{align*} 复制代码 不支持 \vsapce 啊
kuing 2# 2013-3-19 00:28
多打一个  \\  不行么?
kuing 3# 2013-3-19 00:29
\begin{align*} a&=b\\ b&=c\\ \\ c&=d \end{align*}
kuing 4# 2013-3-19 00:31
也可以紧贴在  \\  后面加参数来增加行距,比如  \\[1ex] 、\\[1cm]  等。 \begin{align*} a&=b\\ b&=c\\[1ex] c&=d\\[1cm] e&=f \end{align*}
kuing 5# 2013-3-19 00:32
奇怪,这里怎么不编译
kuing 6# 2013-3-19 00:32
同样的代码在这一楼才正确编译,奇怪奇怪 \begin{align*} a&=b\\ b&=c\\[1ex] c&=d\\[1cm] e&=f \end{align*}
isea 7# 2013-3-19 00:33
本帖最后由 isea 于 2013-3-19 00:34 编辑 \begin{align*} a&=b\\ b&=c\\ \\ c&=d \end{align*} kuing 发表于 2013-3-19 00:29 opera 下,直接 \\ 不行,很奇怪 \begin{align*} 2\\ \\ 3\\ \end{align*} 莫名其妙,这里可以,容我在试下
kuing 8# 2013-3-19 00:46
gather* 环境(多行居中对齐环境)类似  \\[2ex] \begin{gather*} a+b+c=d\\ b+c=d\\[2ex] c=d \end{gather*}
kuing 9# 2013-3-19 00:46
楼上的效果 \begin{gather*} a+b+c=d\\ b+c=d\\[2ex] c=d \end{gather*}
kuing 10# 2013-3-19 00:48
\\[ \begin{gather*} a+b+c=d\\ b+c=d\\[2ex] c=d \end{gather*} 终于知道为什么这样会不编译。
kuing 11# 2013-3-19 00:50
这个贴晚点还是移去测试区吧
isea 12# 2013-3-19 01:29
楼上的效果 \begin{gather*} a+b+c=d\\ b+c=d\\[2ex] c=d \end{gather*} kuing 发表于 2013-3-19 00:46 个人不喜欢一股脑儿的全部中间对齐
kuing 13# 2013-3-19 07:54
12# isea 多数情况用align,但有时可能用gather更合适。 当然,这是个人审美了,也有人喜欢通通左对齐,这里也没什么规定,都随用户喜欢的。
thread-1248-1-3.html: [几何] 深圳一模的解析几何
第一章 1# 2013-3-19 08:40
今天才看到深圳一模的题目。看来我的信息真的不够灵通。 看了解析几何的大题,有点兴趣,随手做一下。 ______kuing edit in $\LaTeX$______ 2003 年深圳一模。 已知两点 $F_1(-1,0)$ 及 $F_2(1,0)$,点 $P$ 在以 $F_1$、$F_2$ 为焦点的椭圆 $C$ 上,且 $\abs{PF_1}$、$\abs{F_1F_2}$、$\abs{PF_2}$ 构成等差数列。 (1)求椭圆 $C$ 的方程; (2)如图 7,动直线 $l:y=kx+m$ 与椭圆 $C$ 有且仅有一个公共点,点 $M$, $N$ 是直线 $l$ 上的两点,且 $F_1M\perp l$, $F_2N\perp l$。求四边形 $F_1MNF_2$ 面积 $S$ 的最大值。
第一章 2# 2013-3-19 08:42
嗯,$F_1M*F_2N=b^2$
第一章 3# 2013-3-19 09:14
本帖最后由 第一章 于 2013-3-19 09:15 编辑 尝试一下草稿纸, 解:设$|F_1M|=m,|F_2N|=n,|MN|=h$ 则$mn=b^2,(m-n)^2+h^2=4c^2$, 可得\[(m+n)^2+h^2=4a^2\]故$F_1MNF_2$的面积$$S=\frac{(m+n)h}{2}\le\frac{(m+n)^2+h^2}{4}=a^2$$仅当$\frac{m+n}{2}=h$时取"=".
kuing 4# 2013-3-19 12:13
$S={}$那个大直角三角形的面积,而它的斜边为定值 $2a$,所以当且仅当两直角边相等时面积取最大值 $a^2$。 PS、其实跟楼上差不多。
yes94 5# 2013-3-19 12:22
一个几何 ,一个代数+结论 ,
第一章 6# 2013-3-19 12:51
强大啊!
yes94 7# 2013-3-19 12:57
关于该图形,似乎还有个什么不等式的,上下界的,忘了
第一章 8# 2013-3-19 12:58
才发现,这个也是深圳一模题。 http://kkkkuingggg.5d6d.net/thread-1175-1-2.html $$\sum_{i=1}^{n}\frac{4i}{4i^2-1}>ln(2n+1)$$ 之前采用对应项+换元,搞定。
yes94 9# 2013-3-19 13:21
8# 第一章 正好那里有个问题没解决:\[\sum_{k=1}^{n}\frac{4k}{4k^2-1}>\ln(2n+1),\] 那么下述不等式是否成立(原不等式的反向)?若成立请证明: \[\sum_{k=1}^{n}\frac{4k}{4k^2-1}<1+\dfrac12\ln(2n+1)(2n+3)\]
第一章 10# 2013-3-19 13:25
对应项证不了, 从第二项开始,左边的每一项都比右边的每一项大,不过误差接近0; 就是左边的第一项<右边的第一项,小太多。 不过我相信这个题目是正确的。
hongxian 11# 2013-3-19 15:52
2# 第一章 $\abs{F_1M} \cdot \abs{F_2N}=b^2$有没有几何解释?
第一章 12# 2013-3-19 16:10
问K吧,纯几何的他搞得比较多。 我只知道这个结论。 很多解析几何的题目,是先有结论,再有题目的。
kuing 13# 2013-3-19 18:30
2# 第一章 $\abs{F_1M} \cdot \abs{F_2N}=b^2$有没有几何解释? hongxian 发表于 2013-3-19 15:52 这样算不算 设 $P$ 为切点,则 \begin{align*} F_1M\cdot F_2N&=PF_1\sin \angle MPF_1\cdot PF_2\sin \angle NPF_2 \\ & =PF_1\cdot PF_2\sin ^2\frac{\pi -\angle F_1PF_2}2 \\ & =PF_1\cdot PF_2\frac{1+\cos \angle F_1PF_2}2 \\ & =\frac12PF_1\cdot PF_2+\frac14(PF_1^2+PF_2^2-F_1F_2^2) \\ & =\frac14\bigl((PF_1+PF_2)^2-F_1F_2^2\bigr) \\ & =a^2-c^2 \\ & =b^2. \end{align*} 注意这里和前面4#都是依靠光学性质的。
hongxian 14# 2013-3-19 18:54
13# kuing 当然算了,也来一个 用一下4#的图,$\displaystyle \begin{cases}\left(F_1M-F_2N\right)^2+h^2=4c^2\\ \left(F_1M+F_2N\right)^2+h^2=4a^2\end{cases}\Longrightarrow F_1M\cdot F_2N=a^2-c^2=b^2$
kuing 15# 2013-3-19 19:16
14# hongxian oh! that's very nice!
yes94 16# 2013-3-19 19:37
14# hongxian 把第一章的解法部分保留,另一部分反推回去就可以啦! 因为第一章的代数解法就是利用结论证明了kuing的几何方法, 所以用kuing的图形,结合第一章的部分运算过程,得到的将是这个(乘积)结论
thread-1249-1-3.html: [组合] 2013广州一模理数13平面分空间
kuing 1# 2013-3-19 14:07
这张试卷的童鞋这题杯具了……其实至少 $f(3)$ 不应该错,不想想空间直角坐标系?显然八个卦限嘛…… 记这 $n$ 个平面都过的点为 $P$。 现在,过 $P$ 再作另一个符合条件的新平面,显然这个新平面与原先的 $n$ 个平面有 $n$ 条过 $P$ 的交线, 易见这 $n$ 条交线将新平面分成 $2n$ 部分,而每一部分将原来的空间多分一部分, 所以新增的空间部分个数就是 $2n$,亦即 $f(n+1)=f(n)+2n$。 其实方法还是老方法,如果你以前研究过 $n$ 条直线最多将平面分成多少个部分,然后拓展到空间什么的,肯定认识这方法,相信很多人读高中就应该研究过。 这类问题大概一搜一大把的文章,这里就不扯下去了……其实本来也不太想扯,只是刚才在某群里回了,所以在这里记录一下,并且改正了当时有部分表达上的不严谨,顺便给这里加点热气,别太冷清了。
yes94 2# 2013-3-19 19:45
赞同,加点火气,可以降点难度,让更多的人参与进来 递推法计数,是一种好方法!
kuing 3# 2013-3-19 20:54
讲开又讲,这道题是在某教师群里看到的,好多老师不会,另外还有一道选择题其实是物理题来的,是船过河的那类问题,也是有些老师不会$^2$ 看来基本上是只要超出常规高中数学的东西都是一点都不看的…… 还记得当时有个老师大概是说,一模考了这道题(1#的)高考就不会考了,言下之意估计就是不用管这题了……
hongxian 4# 2013-3-19 21:09
3# kuing 老师不会,可以原谅,但说“一模考了这道题(1#的)高考就不会考了”就有点问题了!
kuing 5# 2013-3-19 21:54
特意翻了下,的确在高中课本复习题就有这类问题 http://www.pep.com.cn/gzsx/jszx_ ... 0110601_1046941.htm 只不过是圆里的,但是方法显然是一样的,题目也设了后两问,先问线段的,再问分圆,把方法都引出来了的,如果当年有研究过这题(而不是猜了就算了),那么1#的题完全不成问题。
yayaweha 6# 2013-3-19 22:54
我刚考完,8和$n^2-n+2$
kuing 7# 2013-3-19 22:58
6# yayaweha 没有+2
yayaweha 8# 2013-3-19 22:59
说是广州一模,其实是广东省一模。
kuing 9# 2013-3-19 23:00
8# yayaweha 哦,我不懂这些……反正只看题……
yayaweha 10# 2013-3-19 23:00
7# kuing n=3时不是8吗?
yayaweha 11# 2013-3-19 23:02
10# yayaweha 是$n^2-n+2$呀
kuing 12# 2013-3-19 23:03
哦,我傻了
yayaweha 13# 2013-3-19 23:09
船过河,物理老师一定会
kuing 14# 2013-3-19 23:21
13# yayaweha 嗯。那些老师也说学生可能做得比他们更好……
李斌斌755 15# 2013-3-19 23:52
10# yayaweha 不一定
kuing 16# 2013-3-19 23:55
15# 李斌斌755 呃?
yayaweha 17# 2013-3-20 00:01
15# 李斌斌755 任意三个面不经过同一条直线
李斌斌755 18# 2013-3-20 00:08
本帖最后由 李斌斌755 于 2013-3-20 00:10 编辑 晕,忘了3平面不过同一直线,则,1、3;4、6;2、5被分为2。
第一章 19# 2013-3-23 20:43
就题目的条件而言,这个题的模型是$n$棱锥(不考虑底面),答案是2$C_n^2$+2。大家认为呢? ps,如果大胆的把$n$的值从1开始取,貌似也可以得出答案,简单了些。
yayaweha 20# 2013-3-23 21:46
19# 第一章 劲!
thread-1249-2-3.html:
yes94 21# 2013-3-24 22:03
19# 第一章 做等价转化,搞一个模型,便于空间想象,
isea 22# 2013-3-27 18:57
本帖最后由 isea 于 2013-3-27 18:58 编辑 这种递推思考方式,特别受用。 刚开始在竞赛书上看到的, 印象不深,后来海淀考了一次, 好像是见K也是用这种方法, 就记住了,哈哈 ======== 船过河,是这样子的—— 此题的结果是B, 这个相对好算,只要点物理知识即可。如果将水流方向反过来,静水速度又是多少呢?呵呵。 我想问的,一样情况下,是什么结论?
yes94 23# 2013-3-27 21:10
22# isea 解三角形吧?
李斌斌755 24# 2013-3-28 02:41
22# 物理题
thread-125-1-1.html: [数论] 好吧,我承认我衰了(新增题)
GAM 1# 2011-10-20 23:09
本帖最后由 GAM 于 2011-10-23 22:08 编辑 【 转载:出自matrix67.com,matrix67牛淫一枚】 一个小学奥数老师给我讲了一道小学奥数题,这是他在上课时遇到的:从 1 到 4000 中,各位数字之和能被 4 整除的有多少个?     注意,问题可能没有你想的那么简单,满足要求的数分布得并没有那么规则。 1 、 2 、 3 、 4 里有一个满足要求的数, 5 、 6 、 7 、 8 里也有一个满足要求的数,但是 9 、 10 、 11 、 12 里就没有了。     尽管如此,这个问题仍然有一个秒杀解。你能多快想到?         答案就是 1000 。首先, 0 和 4000 都是满足要求的数,因而我们不去看 1 到 4000 中有多少个满足要求的数,转而去看 0 到 3999 中有多少个满足要求的数,这对答案不会有影响。注意到,如果固定了末三位,比如说 618 ,那么在 0618 、 1618 、 2618 、 3618 这四个数中,有且仅有一个数满足,其各位数字之和能被 4 整除。考虑从 000 到 999 这 1000 个可能的末三位组合,每一个组合都唯一地对应了一个满足要求的四位数,因此问题的答案就是 1000 。     真正有趣的事情在后面呢。一个小朋友举手说:“老师,我明白了,按照这个道理,从 1 到 3000 里各位数字之和能被 3 整除的数也是 1000 个。”另一个小朋友说:“废话,各位数字之和能被 3 整除就表明整个数能被 3 整除,在 1 到 3000 里这样的数当然有 1000 个嘛!”全班哄堂大笑。 再加一道吧,考虑到坛子这冷清,不过思维方式都很好的 今天的题目来自这里http://www.cs.cmu.edu/puzzle/puzzle5.html。有三个水桶,它们里面分别装了 a 升的水、 b 升的水和 c 升的水(其中 a 、 b 、 c 都是正整数,桶本身没有容量限制)。你可以把水从一个桶倒进另一个桶,但必须保证让后者的水量刚好变成原来的两倍。证明,不管 a 、 b 、 c 是多少,你总能让其中某一个水桶变空。     例如,假设初始时 (a, b, c) = (3, 2, 1) ,那么你可以先把 (3, 2, 1) 变成 (1, 4, 1) ,再把它变成 (2, 4, 0) ,从而把第三个水桶变空。      让我们先来看一个特殊的情形。假如只有两个水桶,并且其中一个水桶的水量为奇数,另一个水桶的水量为偶数。由于我们只有两个水桶,并且它们的水量也不相等,因此我们只有唯一一种可行的操作:拿起水多的桶,把水倒进水少的桶。注意,每次操作之后,两个桶里的水量仍然是一奇一偶,因此我们还能继续操作下去,永远不会“走死”。     但是,两个桶里的水量只有有限种组合,因而如果我们一直这么走下去,最终必然会和原来的某个状态重合,从而产生循环。事实上,第一个重复的状态一定就是初始时的状态,而不会是半路中的某个状态;换句话说这个循环一定是一个完整的圈,而不是一个 ρ 字形的循环。这是因为,每一个状态都只能由唯一的“前继状态”变过来,即水量为偶数的那个桶其中一半的水在另一个桶中。     这意味着,如果循环的长度是 n ,那么经过 n - 1 步移动之后,我们就能到达初始状态的前继。换句话说,假如 a 是一个偶数, b 是一个奇数,那么我们就有了一种把 (a, b) 变成 (a/2, b + a/2) 的方法。      让我们来看看一些更复杂的情况。假如初始时三个桶的水量分别是奇、偶、偶,下面我们给出一种方法,它能增加“奇数桶”里的水量,同时保持另外两个桶里的水量仍是偶数。把三个桶里的水量分别记作 (a, b, c) ,其中 a 是奇数, b 和 c 都是偶数。如果 b 和 c 都不能被 4 整除的话,可以在它们俩之间倒一次水,让其中一个桶里的水量变成 4 的倍数(同时另一桶水的水量仍是偶数)。现在,无妨假设 b 的水量是 4 的倍数。我们把 (a, b) 变成它的前继,从而让三个桶里的水量分别变成 (a + b/2, b/2, c) 。这样一来,三个桶里的水量仍是奇、偶、偶,但“奇数桶”里的水量变多了。     不断重复上述操作,让“奇数桶”里的水越来越多,同时保持另外两个桶的水量始终是偶数。最终,总有一个“偶数桶”会变空。因此,我们就解决了三个桶分别为奇、偶、偶的情况。      其他情况呢?随便走一步,奇、奇、奇的情况立马就变成了奇、偶、偶,因此奇、奇、奇的情况也就解决了。现在,我们只剩下奇、奇、偶和偶、偶、偶的情况。在两个“奇数桶”之间倒水,可以把奇、奇、偶变成偶、偶、偶。而初始情况为偶、偶、偶,这意味着以后三个桶里的水量也永远都是偶数,因此我们可以把所有桶里的水量都除以 2 ,从而化为规模更小的情况。我们可以用这种方法不断减小问题的规模,直到把问题化为我们已解决的状态为止。至此,这个问题便彻底解决了。      这是一个非常经典的题目了,它也出现在了趣题王 Peter Winkler 的 Mathematical Puzzles 一书中。在书里, Peter Winkler 指出,这道题出自第五届全苏数学奥林匹克竞赛中,随后又出现在了 1993 年的 Putnam 竞赛中。 Peter Winkler 还在书里提到了另一个答案,这是由 Svante Janson 首先给出的。      为了解释清楚,还是让我用一个例子来说明吧。不妨假设这三个桶里的水量分别为 67 升、 10000 升和 12345 升。把三个桶按照水量从少到多排列,依次编号为 A 、 B 、 C 。下面我们给出一个过程,它能把 B 里的水变得比 A 更少。先计算 10000 除以 67 ,结果等于 149 余 17 。把 149 转换为二进制 10010101 。现在,从这个二进制数的最低位开始,从右往左依次查看各个数字:每读到一个 1 ,就把 B 倒进 A ;每读到一个 0 ,就把 C 倒进 A 。容易看出,最后 A 中的水量变成了原来的 28 = 256 倍,其中 B 贡献了 149 倍。因此, B 总共倒出了 149 × 67 升的水,因而 B 里面将会只剩下 17 升的水。由于 17 这个数字是 10000 除以 67 的余数,因此它是一个比 67 小的数。这意味着,我们有了一种刷新水量最小值的方法。不断执行这个过程,最终总能把其中一个桶里的水量变为 0 。     且慢,我们还需要证明,为了完成上述过程, C 里面储备有足够多的水。这很容易看出来,因为在最坏情况下,二进制数恰好为 1000..00 ,但即使是这样, C 需要贡献的水量也不会比 B 更多。然而, C 本来拥有的水量是三个桶里最多的,因此我们永远不用担心 C 里的水不够。
kuing 2# 2011-10-20 23:14
matrix67的blog我以前也看过的说,呵呵,不过话说,小学奥赛题好多我都做不来。。。
yuzi 3# 2011-10-21 10:33
这个解答确实是强呀!
nash 4# 2011-10-22 00:24
我勒个去 这个真是秒杀呀
kuing 5# 2011-10-22 22:32
小学题很多数论或组合的说……这些我都弱
isea 6# 2011-10-26 13:21
5# kuing 这个回答太TMD有意思了。 这个直接看懂了: “老师,我明白了,按照这个道理,从 1 到 3000 里各位数字之和能被 3 整除的数也是 1000 个。” PS:被4整除那个,偶是看不明白了
realnumber 7# 2011-12-15 14:07
5# kuing PS:被4整除那个,偶是看不明白了 isea 发表于 2011-10-26 13:21 注意到,如果固定了末三位,比如说 618 ,那么在 0618 、 1618 、 2618 、 3618 这四个数中,有且仅有一个数满足,其各位数字之和能被 4 整除 就这句话,你不会不明白的。有#312,你说#处填什么?既要0<#312<3999,又要#312=0(mod4)
dahool 8# 2012-1-10 09:22
牛啊,真无语
海盗船长 9# 2012-1-10 11:55
“而初始情况为偶、偶、偶,这意味着以后三个桶里的水量也永远都是偶数,因此我们可以把所有桶里的水量都除以 2 ,从而化为规模更小的情况。” 这个没懂啊,为啥可以都除以“2”?
海盗船长 10# 2012-1-10 14:22
“而初始情况为偶、偶、偶,这意味着以后三个桶里的水量也永远都是偶数,因此我们可以把所有桶里的水量都除以 2 ,从而化为规模更小的情况。” 这个没懂啊,为啥可以都除以“2”? 海盗船长 发表于 2012-1-10 11:55 懂了。。
李斌斌755 11# 2013-5-30 02:13
看了后
thread-1252-1-1.html: 发好后,删去似乎计数不会减少
realnumber 1# 2013-3-19 23:04
如题
kuing 2# 2013-3-19 23:06
是的,自删或他删都不会减少,至少 discuz 的论坛系统一直都是这样的。 所以你可以看到人教论坛那边的新增贴数字总是很大,其实基本上都是广gao贴刷成那样的
kuing 3# 2013-3-19 23:26
去人教看了下,显示“高中数学论坛 (202)”……估计大概有90%是广gao的成果
thread-1253-1-1.html: CTeX 里默认没 \dfrac 呀
isea 1# 2013-3-20 01:12
这个叫代码,命令,还是语言?
kuing 2# 2013-3-20 01:18
大概要加amsmath宏包。 一般来说,写数学的都加这个,还有amssymb。这样才有更全面地使用数学类的命令和符号。
kuing 3# 2013-3-20 01:33
写定理, 证明, 还要加amsthm, 提供定理环境和证明环境。
isea 4# 2013-3-20 01:39
大约知道是啥意思,不过,不懂,现在。 看图,似乎在正文之前要配置声明一些东东? 是这个意思吧?有现成的没?若有,直接丢个数学里最常用来吧,谢谢。
isea 5# 2013-3-20 01:40
汗,原来CTeX里,自带的WinEdt是未注册的啊
kuing 6# 2013-3-20 01:43
呃…我现在爪机,无法丢。 你去外面那个"下载了不知怎么打开"那个帖里找到我发的示例看看。
kuing 7# 2013-3-20 01:44
5# isea 那个未注册无所谓,照用。
isea 8# 2013-3-20 01:44
不着急的,好晚了,睡觉,先。 哈哈,早睡,也,你。
isea 9# 2013-3-20 01:48
果然啊,正文之前要加载宏包,感谢,晚安
kuing 10# 2013-3-20 02:03
今天早上睡得够,所以现在我还很精神啦。 慢慢来,开头可能很艰难。 真正用LaTeX写东西,可不只是像在这里打那点公式那么简单,尢其像你玩几何的,必定很多插图,那是LaTeX的弱项,相当麻烦。
kuing 11# 2013-3-20 23:39
昨晚爪机写不了,现在想起来,再写一下一些常用的包包 \usepackage{amsmath,amssymb,amsthm}  %数学类常用的三个包 \usepackage{esvect}  %向量箭头包,给出 \vv 命令,箭头要比默认的好看很多 \usepackage{stmaryrd}  %为了使用平行符号 \sslash 而加的这个包,当然这个包本身还提供的符号还不止这个,但一般不会用到 \usepackage[top=1in,bottom=1in,left=0.8in,right=0.8in]{geometry}  %页面设置,中括号里的参数是边距,可自行调整,更详细的可参考http://www.ctex.org/documents/packages/layout/geometry.htm \usepackage{fancyhdr}  %页眉、页脚等设置,这个有点复杂,初学的可以先不用,详细可参考http://www.ctex.org/documents/packages/layout/fancyhdr.htm \usepackage{hyperref}  %文档各种链接等等所用,这个包的选择好多,我也没搞太清楚,初学的可以先不用,详细可参考http://zzg34b.w3.c361.com/package/links.htm 另外还有一些插图包,我研究得不多,具体看插图指南。
kuing 12# 2013-3-21 01:14
更新了一下楼上 先打住……
isea 13# 2013-3-26 00:35
昨晚爪机写不了,现在想起来,再写一下一些常用的包包 \usepackage{amsmath,amssymb,amsthm}  %数学类常用的三个包 \usepackage{esvect}  %向量箭头包,给出 \vv 命令,箭头要比默认的好看很多 \usepackage{stm ... kuing 发表于 2013-3-20 23:39 哎呀,这里有大礼包啊
kuing 14# 2013-3-26 01:47
这么晚还没睡,难得喔
thread-1255-1-3.html: [组合] 转一个新题型
hongxian 1# 2013-3-20 11:30
本帖最后由 hongxian 于 2013-3-20 12:11 编辑 把一个正整数n表示为比它小的正整数的和,如3=3;  3=1+2;  3=1+1+1, 共有3种表示法,记f(3)=3, 问:一般地正整数n,  f(n)=? 来自:http://bbs.pep.com.cn/forum.php? ... &extra=page%3D1
kuing 2# 2013-3-20 11:32
是组合题
kuing 3# 2013-3-20 11:37
http://zh.wikipedia.org/wiki/%E6%95%B4%E6%95%B8%E5%88%86%E6%8B%86
kuing 4# 2013-3-20 11:54
也可以放到数论
kuing 5# 2013-3-20 11:57
不过还是觉得组合合适些
hongxian 6# 2013-3-20 12:28
5# kuing 已经改到组合了!
kuing 7# 2013-3-20 13:34
看了下3#wiki的链接……真不是简单的东东……
thread-1256-1-1.html: [几何] 刚才352问的一道平几,两定边,第三边作正三角形
kuing 1# 2013-3-20 18:53
12. 以 $\triangle ABC$ 的边 $BC$ 为边作正三角形 $\triangle BCP$,若 $AB=5$, $AC=2$,则 $AP$ 长度的范围为____。 当时在Q上讲解的时候稍乱(咳,Q上讲,始终不如在论坛上讲那样冷静,所以总是打错东西,连题目都搞错了点,还好影响不大),以下略整理于此。 我们让 $AB$ 固定,那么 $C$ 将在一个圆上动,由对称性,我们只需考虑 $C$ 在直线 $AB$ 的同一侧,再作出两个方向的 $P$(因为原题并没有说要向哪边作,所以两边都要考虑,这在Q上聊的时候我就搞错了),比如说如下图这样。 注意到 $P$ 是 $C$ 绕 $B$ 旋转 $60\du$ 得到的,而 $B$ 是定点,于是 $P$ 的轨迹也就是 $C$ 的轨迹绕 $B$ 旋转 $60\du$ 所得,分两个方向,画出如下。 这样,$AP$ 的长度范围就能看出来了。当然,也可以将 $A$ 也旋转过去,像这样 于是结果就更显然了。下略。 当时,352 还给出了如下代数法的思路 如何进行下去?我也不清楚,或者有没有别的简单的代数方法?
abababa 2# 2013-3-20 19:42
1# kuing 最大值能用托勒密吗?
kuing 3# 2013-3-20 19:45
更新了一下,刚才才发现原来题目并没有说向外作正三角形,所以要考虑两个方向的旋转。
第一章 4# 2013-3-20 20:21
能否考虑让$B$、$C$在$x$轴上,且关于原点对称,则$P$在y轴(负半轴)上,以$B$、$C$为圆心分别作圆,半径是5和2,两圆的交点就是$A$, 观察出最小为3,最大为7.
kuing 5# 2013-3-20 20:36
4# 第一章 可以考虑!
第一章 6# 2013-3-20 20:36
k用了阿氏圆,难道这就是背景?
kuing 7# 2013-3-20 20:37
6# 第一章 跟阿氏圆没关系啊,直接就是 AC 是定值……
第一章 8# 2013-3-20 20:39
晕,以为$CA$跟$CB$是定长了。 做晕头了,都。
kuing 9# 2013-3-20 20:40
淡定…… and,继续期待好的代数方法
kuing 10# 2013-3-20 20:53
1# kuing 最大值能用托勒密吗? abababa 发表于 2013-3-20 19:42 不清楚,你试试看?
yes94 11# 2013-3-20 20:54
1# kuing 最大值能用托勒密吗? abababa 发表于 2013-3-20 19:42 能用! $AP\cdot BC\leqslant5PC+2PB$,故$AP\leqslant5+2=7$, 当且仅当$A,B,P,C$四点共圆时,$AP$取得最大值$7$。
第一章 12# 2013-3-20 20:54
若真是3和7,那刚好是5和2的和、差。 巧合?
yes94 13# 2013-3-20 20:59
12# 第一章 刚刚上来,发完一看,居然有kuing和第一章和我几乎同时发!
第一章 14# 2013-3-20 21:01
晕死,我敢肯定最小值也是四点共圆。托勒密定理已经有十年没用过了。
kuing 15# 2013-3-20 21:03
若真是3和7,那刚好是5和2的和、差。 巧合? 第一章 发表于 2013-3-20 20:54 由1#最后的图也可以看出,必定是和与差……
第一章 16# 2013-3-20 21:07
表示没细看你1#的图,一直沿着自己的思路,,,
yes94 17# 2013-3-20 21:08
15# kuing 那几个图多了,看的眼花缭乱,就没仔细看下去了,估计和第一章感觉一样的
kuing 18# 2013-3-20 21:09
最大最小的情形
kuing 19# 2013-3-20 21:12
60\du 和 120\du 夹角
abababa 20# 2013-3-20 21:25
本帖最后由 abababa 于 2013-3-20 21:27 编辑 发一位网友的解答 $B=(5,\theta_1),C=(2,\theta_2),T=\begin{pmatrix}1/2 & -\sqrt{3}/2\\\sqrt{3}/2 & 1/2\end{pmatrix}$ 得$AP^2=(B-C) \cdot T=29-20\cos(\theta_1-\theta_2)$,得最值为$49,9$ 他没开方,开方就是那个7和3了吧,正好是同向和反向
thread-1256-2-1.html:
yes94 21# 2013-3-20 21:29
20# abababa 感觉是复数的翻版?
kuing 22# 2013-3-20 21:29
看不懂,矩阵都来了
第一章 23# 2013-3-20 21:31
极坐标都派上用场了
abababa 24# 2013-3-20 21:36
其实我也没看懂,而且感觉有问题。
三下五除二 25# 2013-3-20 21:44
本帖最后由 三下五除二 于 2013-3-20 21:49 编辑 我也发一个网友的代数解答,中规中矩的,我开始也这样做的,到最后有点乱。。。。。,
abababa 26# 2013-3-20 21:53
25# 三下五除二 是的,现在我想明白了,我发的网友那个解答就是AB和AC看成向量,然后表示出P,那矩阵貌似旋转矩阵
yes94 27# 2013-3-20 22:05
我也发一个网友的代数解答,中规中矩的,我开始也这样做的,到最后有点乱。。。。。, 三下五除二 发表于 2013-3-20 21:44 主要是你的那个解析式里,既含有$x$,还含有$C$,这两个变量是有关系的(相互制约的),所以你只需保留$C$,消去$x$即可.
kuing 28# 2013-3-20 22:09
不错啊 这楼居然升得这么快
第一章 29# 2013-3-20 22:18
28# kuing 对本网站活跃的会员,k是否考虑发年终奖金?或者在年度考核中评优评先?
kuing 30# 2013-3-20 22:21
29# 第一章 这里是不讲银子的…… 至于什么“年度考核中评优评先”?其实我不懂是啥意思……
yes94 31# 2013-3-20 22:26
30# kuing 没有银子,也可以发点金币噻
yes94 32# 2013-3-21 11:19
20# abababa 感觉是复数的翻版? yes94 发表于 2013-3-20 21:29 结果何版就用了复数!
abababa 33# 2013-3-22 14:59
14# 第一章 确实是啊,最小值就是把那个三角形翻上去,把5变成对角线。 就是这个四边形里再有一个等边三角形的,马上就想到那个经典的用托勒密的题了。
yes94 34# 2013-3-22 18:22
33# abababa 托勒密不等式,可以用复数方法证明,这样就联系起来了
第一章 35# 2013-3-22 22:42
那个托勒密不等式,貌似适用于凸四边形, 而本题,如果点$A$、$P$在$BC$的同侧,且点$A$在$△PBC$内部或边上,怎么处理? 呵呵,勿喷,我疏远托勒密定理已经好多年。
abababa 36# 2013-3-23 19:38
35# 第一章 看看这样行不行 过$A$作$AD // BC$,$D$和$A$关于等边三角形的高对称,然后就有$CD=AB=2$ 此时就有$AD>|AC-CD|=3$,然后在$\triangle PAD$中,因为$\angle APD<60^\circ$,所以$\angle PAD=\angle PDA>60^\circ$,根据大边对大角就有$PA>AD>3$,因为已经算出来$PA$最小是3,这样$A$在内部时就不是最小的了
第一章 37# 2013-3-23 20:14
36# abababa
yes94 38# 2013-3-24 22:27
复数恒等式:$(a-b)(c-d)+(a-d)(b-c)=(a-c)(b-d)$,两边取模,得不等式: $\abs{AC}\cdot\abs{BD}=\abs{(a-c)(b-d)}=\abs{(a-b)(c-d)+(a-d)(b-c)}≤|(a-b)(c-d)|+|(b-c)(a-d)|=\abs{AB}\cdot\abs{CD}+\abs{BC}\cdot\abs{AD}$
yes94 39# 2013-3-25 12:07
38# yes94 根据复数证法,需不需要“凸四边形”这个条件?
isea 40# 2013-3-25 13:47
本帖最后由 isea 于 2013-3-25 14:09 编辑 我看了看,都觉得这个不需要太多的东东,平面几何最大最小的基础题, (自己脑补图)——其实顶楼就是图了 k 的图不好懂(需要细心耐心领会,细看一看,本楼的意思与k 的意思一样),从好懂一点的角度说, 就是将$\triangle {ABC}$跟着旋转, 当$A$与$P$在$BC$同侧,$\angle {ABC}=60^\circ,{AP}_\min=5-2=3$; 当$A$与$P$在$BC$异侧,$\angle {ABC}=120^\circ,AP_\max=5+2=7$。 再换句话说,即$P$在$\triangle {ABC}$外接圆上时,才会取得最大与最小。 这楼这么高,说明还是有难度的,猜测脑补图也不简单的事,示个例子吧: 如图(此图只能取到最小值),将$\triangle {ABC}$绕点$C$顺时针旋转$60^\circ $ ,得到 $\triangle {CPA'}$, 连接$AA'$,则  $AA'=A'C=AC=5,PA'=AB=2$. 在 $\triangle {PAA'}$中,$AC-AB\le\ AP \le AC+AB$,当$A'$落在$PA$上时,取等号。 类似的,不一定作正三角形,作等腰直角三角形啊,甚至于四边形等,都可类似处理——旋转,三角形三边关系。
thread-1256-3-1.html:
yes94 41# 2013-3-25 21:09
40# isea 复数也是处理旋转的利器。
isea 42# 2013-3-25 22:37
41# yes94 我喜欢用向量,偏向量,虽然二者此时在几何意义上已然没分别……
yes94 43# 2013-3-25 23:41
42# isea 我觉得复数和向量也是有区别的, 例如在旋转上,向量比不过复数。 在两向量之和与两复数之和取模时的运算方法有些不同 复数还有共轭复数等等。 但某些地方向量良好的几何意义,又是复数望尘莫及的。 就不再举例了。
李斌斌755 44# 2013-3-28 02:20
本帖最后由 李斌斌755 于 2013-3-28 02:22 编辑 回复40#isea 这样就容易看懂了,当C点绕A点旋转时,P点也绕A1点旋转,两三角形全等,两圆等径。 上下对称,只画一边。
Gauss门徒 45# 2013-5-18 17:02
将A定在原点 把已知长的线段化为圆,在其中一圆上任取一点再固定住。这样这个等边三角形还有两个点,一个是圆,另一个也是圆
thread-1257-1-3.html: [函数] 2013届苏、锡、常、镇、徐、连六市高三第二次模拟考试
pengcheng1130 1# 2013-3-20 21:35
好题欣赏
kuing 2# 2013-3-20 21:40
目测 $\sqrt2$
kuing 3# 2013-3-20 21:43
$a^2+b^2=c^2$, $ab>c$, $a^2b^2>a^2+b^2$, $(a^2-1)(b^2-1)>1$,故此当且仅当 $M\geqslant\sqrt2$
yes94 4# 2013-3-20 22:34
3# kuing 写的稍微简略了些, 如果把充分、必要性写一下的话就很完美了
kuing 5# 2013-3-20 22:40
4# yes94 比较显然,就没写下去了……反正是好题欣赏嘛,又不是求助,能略就略了……
yes94 6# 2013-3-20 23:33
先说必要性: 取直角三角形的三边$t$,$t$,$\sqrt2t$,则$\ln t+\ln t>\ln \sqrt2t$,即$t^2>\sqrt2t$,$t>\sqrt2$,于是$M<\sqrt2$不可行, 所以至少应满足$M\geqslant\sqrt2$。 充分性kuing已给出。
thread-1258-1-3.html: [数列] 2013届苏、锡、常、镇、徐、连六市高三第二次模拟考试数学
pengcheng1130 1# 2013-3-20 21:39
好题欣赏
yes94 2# 2013-3-20 23:21
答案已看,现在没什么新想法的,先顶上来,看看以后还有没有想法
pengcheng1130 3# 2013-3-21 07:11
请高手给出一个解法
kuing 4# 2013-3-21 08:32
3# pengcheng1130 你不是知道答案的吗?
pengcheng1130 5# 2013-3-21 10:29
就是不知道答案啊!
kuing 6# 2013-3-21 12:13
那你怎么知道是好题……
yes94 7# 2013-3-21 12:28
6# kuing 看起来无法下手,就是好题?
goft 8# 2013-3-21 13:30
Sn+m=Sm+n,化简后边的好算了
yes94 9# 2013-3-21 17:27
Sn+m=Sm+n,化简后边的好算了 goft 发表于 2013-3-21 13:30 只给提示,
kuing 10# 2013-3-21 17:28
9# yes94 还是个下标分不清的提示
thread-1259-1-3.html: [函数] 转发3个题
yes94 1# 2013-3-21 18:55

hnsredfox_007 2# 2013-3-21 19:51
本帖最后由 hnsredfox_007 于 2013-3-21 19:54 编辑 1.解:由题意得:$x=\dfrac{x-1}{\sqrt{x-\dfrac{1}{x}}-\sqrt{1-\dfrac{1}{x}}}$, 即$\sqrt{x-\dfrac{1}{x}}-\sqrt{1-\dfrac{1}{x}}=\dfrac{x-1}{x}=1-\dfrac{1}{x}$, 于是$2\sqrt{x-\dfrac{1}{x}}=1+x-\dfrac{1}{x}$,易得$x-\dfrac{1}{x}=1$, 解得$x=\dfrac{1+\sqrt5}{2}$(负值舍去)。
yes94 3# 2013-3-21 22:40
2# hnsredfox_007
abababa 4# 2013-3-22 00:15
本帖最后由 abababa 于 2013-3-22 00:17 编辑 发一位网友的解答,看看对不对 $(x_1,x_2,x_3) \to (x_1,\sqrt{2}x_2,\sqrt{3}x_3)$ $x_1^2+x_2^2+x_3^2=3,\pi:x_1+\sqrt{1/2}x_2+\sqrt{1/3}x_3=1$ 过z的平面束为$x_1+\lambda x_2=0$,在$\pi$上投影使 $1+\sqrt{1/2}\lambda=0$得$\lambda=-\sqrt{2}$ 于是投影为$x_1-\sqrt{2}x_2=0,x_1+\sqrt{1/2}x_2+\sqrt{1/3}x_3=1$ 解得$x_1=\sqrt{2}x_2,x_2=\frac{\sqrt{2}}{3}(1-\sqrt{1/3}x_3)$ 代入球得$11x_3^2-4\sqrt{3}x_3-21=0$解得最小值$x_3=-7\sqrt{3}/11$ 原最小值$x_3=-7/11$
kuing 5# 2013-3-22 01:55
用柯西是不是方便点
kuing 6# 2013-3-22 01:57
第三题求啥? 费马点?
yes94 7# 2013-3-22 12:40
6# kuing 第三题求最(小)值吧,
abababa 8# 2013-3-22 12:55
5# kuing 哦,我不等式好弱啊,都看不出来。版主的强项。
yes94 9# 2013-3-26 21:47
原帖在这里:http://tieba.baidu.com/p/2216307132
yes94 10# 2013-3-27 21:53
总觉得第一问有巧算?
thread-126-1-1.html: 问个小问题~~
贵族风铃 1# 2011-10-21 15:13
word里面怎么画分割线啊。。就是横着一条
kuing 2# 2011-10-21 18:28
英文状态下,连续输入 $\geqslant3$ 个 - 或 _ 或 = 或 * 或 ~ 或 #,回车。
贵族风铃 3# 2011-10-21 18:57
2# kuing 哇哈哈你太有才了。。
①②③④⑤⑥⑦ 4# 2011-10-24 09:47
2# kuing 这是Word的一项自动转换功能,属于“键入时自动套用格式”,这项功能是可以关闭的。 转换出来的线,其实就是段落的“边框线”,大多数时候是转换为上边那个段落的下框线,在“边框和底纹”里面能做的事情更多,如果要删除这条线,往往也要到“边框和底纹”中去操作(如果你不是想要把整个段落都删掉的话)。此外,假如某个段落已经设置了下边框,在这个段落的文字内部打回车分段,生成的两个段落都会有“下边框”的设定,不过连续几个段落都设置了下边框,只有最后一个的下边框会显示出来,当经过很多操作以后,也许会发生一连串的段落都具有“下边框”,如果用户没有察觉,这时候就有趣了,比如针对最后一个段落,去掉了“下边框”,结果发现,那条分割线没有被删掉,而是“逃”到上面去了(其实删掉了,但是前一段落的边框线显现出来了);又比如,在某两段中间回车,输入===,本来是希望增加一条分割线的,结果除了这条双线之外,顶上又多出另外一条单线(Word发现上面的段落已经有单线下边框的设置,它不敢擅自替换为双线下边框,于是空段落保留,并设置双线下边框,而上方段落成为连续一片具有单线下边框最后一段,这条线显现出来),等等……
kuing 5# 2011-10-24 10:26
4# ①②③④⑤⑥⑦ 这么多东西,我对word研究得不多
thread-1260-1-3.html: [不等式] can's old bat dang thuc
pxchg1200 1# 2013-3-21 19:37
本帖最后由 pxchg1200 于 2013-3-21 19:38 编辑 Prove that for any real numbers $ a,b,c$ such that $ a+b+c=3$, then \[ \frac{1}{2a^2+7}+\frac{1}{2b^2+7}+\frac{1}{2c^2+7} \le \frac{1}{3}.\] Equality holds for $ (a,b,c)=1,1,1)$ or $ (a,b,c)=\left( 2,\frac{1}{2},\frac{1}{2}\right).$
thread-1261-1-3.html: [数列] 一个数列题
转化与化归 1# 2013-3-21 22:04
这个题的最后一问能不能放缩成等比数列?
yes94 2# 2013-3-21 23:14
最后一问化下来就是2011重庆高考题吧: 最后一问可以加强为:$0\leqslant a_k\leqslant a_{k+1}\leqslant\dfrac43$, 等比数列怎么放缩啊?
转化与化归 3# 2013-3-21 23:16
我指的是s(n)<1这一问能不能用等比放缩一下
realnumber 4# 2013-3-22 09:52
$a_1=\frac{1}{2},a_2=\frac{1}{3},a_3=\frac{1}{7},a_4=\frac{1}{43},S_2=1-\frac{1}{6},S_3=1-\frac{1}{42},..$ 猜测是很难用等比吧,$S_n$的极限就是1.而利用等比放缩大多是丢掉一部分. 通过上面实验,可以用数学归纳法证明,如下结论后,可证明本题. $S_k=1-\frac{1}{m},k,m\in Z^+,a_{k+1}=\frac{1}{m+1}$,那么$S_{k+1}=1-\frac{1}{m(m+1)},a_{k+2}=\frac{1}{m(m+1)+1}$
转化与化归 5# 2013-3-22 17:24
如果不限制用等比!
yes94 6# 2013-3-22 18:19
5# 转化与化归 这个放缩比较妙!
thread-1262-1-3.html: [数列] 来自人教群的一道两数列相同项
kuing 1# 2013-3-21 23:22
学生-天天在线(3308*****) 14.(理)已知数列 $\{a_n\}$, $\{b_n\}$,且通项公式分别为 $a_n=3n-2$, $b_n=n^2$,现抽出数列 $\{a_n\}$, $\{b_n\}$ 中所有相同的项并按从小到大的顺序排列成一个新的数列 $\{c_n\}$,则可以推断: (1)$c_{50}=$______(填数字); (2)$c_{2k-1}=$______(用 $k$ 表示)。 假设 $a_m=b_n\iff 3m-2=n^2$。 设 $k\in\mbb N$,若 $n=3k+1$,代入解得 $m=3k^2+2k+1\in\mbb N^+$;若 $n=3k+2$,代入解得 $m=3k^2+4k+2\in\mbb N^+$;若 $n=3k+3$,代入解得 $m=3(k+1)^2+2/3\notin\mbb N^+$。 由此可见,相同项构成的新数列为不被 $3$ 整除的正整数的平方,即 $\{1^2,2^2,4^2,5^2,7^2,8^2,10^2,11^2,\ldots\}$, 容易写出其通项公式为 \[c_n=\begin{cases} (3k-2)^2, & n=2k-1,\\ (3k-1)^2, & n=2k. \end{cases}\] 话说,当我写到最后的时候,突然觉得我以前可能写过相同的东西,但是都写完了,算了,不找了……
realnumber 2# 2013-3-22 09:57
没接触竞赛的学生就"特殊到一般"尝试,$1^2,2^2,3^2,4^2,...$,就这个,若$3\mid s,则s^2\equiv0\mod3;3\nmid s,则s^2\equiv1\mod3.$
yes94 3# 2013-3-22 12:33
把$b_n=n^2$改为$b_n=2^n$呢?
realnumber 4# 2013-3-22 13:52
3# yes94 差不多吧,就n分偶数奇数两类
yes94 5# 2013-3-25 00:14
4# realnumber 以前看见过一道 等差数列和等比数列的公共项问题……
thread-1263-1-2.html: [函数] 抽象函数--高中高考难度,收集.
realnumber 1# 2013-3-22 12:09
本帖最后由 realnumber 于 2013-3-22 12:19 编辑 1.已知函数$f(x)$满足:$f(1)=0.25,4f(x)f(y)=f(x+y)+f(x-y),x,y\in R$,求$f(2014)$的值.---不用动手的,就看看好了,一般来自教辅书. 2.定义在R上的函数$f(x)$满足$f(x+y)=f(x)+f(y)+2xy,x,y\in R$,$f(1)=2$,求$f(-2)$的值.
yes94 2# 2013-3-22 12:32
第一题:2010年重庆高考填空题吧,模型:三角函数
hnsredfox_007 3# 2013-3-22 14:28
2.小题的话,选择$f(x)=x^2+x$,则$f(-2)=2$.
kuing 4# 2013-3-22 23:52
1.已知函数$f(x)$满足:$f(1)=0.25,4f(x)f(y)=f(x+y)+f(x-y),x,y\in R$,求$f(2014)$的值.---不用动手的,就看看好了,一般来自教辅书. 2.定义在R上的函数$f(x)$满足$f(x+y)=f(x)+f(y)+2xy,x,y\in R$,$f(1)=2$,求$f(-2) ... realnumber 发表于 2013-3-22 12:09 以下设 $K(x)$ 是任意一个柯西方程的解,即 $K(x)$ 满足对任意 $x$, $y\in\mbb R$ 均有 $K(x)+K(y)=K(x+y)$。 $f(x)=\frac12\cos(K(x))$ 满足 $\forall x$, $y\in\mbb R$ 有 $\abs{f(x)}\leqslant1/2$ 且 $4f(x)f(y)=f(x+y)+f(x-y)$; $f(x)=\frac12\cosh(K(x))$ 满足 $\forall x$, $y\in\mbb R$ 有 $f(x)\geqslant1/2$ 且 $4f(x)f(y)=f(x+y)+f(x-y)$。 $f(x)=K(x)+x^2$ 满足 $\forall x$, $y\in\mbb R$ 有 $f(x+y)=f(x)+f(y)+2xy$。
yes94 5# 2013-3-23 14:07
4# kuing 研究的这么niubility!
kuing 6# 2013-3-23 17:02
5# yes94 给出非连续解的常用办法 上次数学空间11P29后面我也给了个类似的注,昨晚还发现那段里的常数c其实是多余的……
yes94 7# 2013-3-24 22:36
6# kuing 给个链接多好啊
hongxian 8# 2013-4-10 15:11
昨天在群内看到的一个发到这里吧! $f(1)=5$,$f(x)f(y)=f(x+y)+f(x-y)$,$y \ne0$时$f(y)>2$,有理数$a$,$b$满足$\abs{a}<\abs{b}$,比较$f(a)$与$f(b)$的大小
thread-1264-1-3.html: [不等式] 秋风问的一个问题
kuing 1# 2013-3-22 17:57
虽然我并不清楚他原先想构造的东东是啥意思(各种看不懂),但是按照他说的,目前需要解决的问题大意是: 是否存在 $m>0$ 使得当 $x$, $y\in(0,m)$ 且 $x\ne y$ 时恒有 $\dfrac{x^x}{y^y}>\abs{x-y}^\abs{x-y}$? 下面来解决一下。 (1)若 $x>y>0$,则不等式等价于 \[\frac{x^x}{y^y}>(x-y)^{x-y},\] 整理为 \[1>\left(\frac yx\right)^y\left(1-\frac yx\right)^{x-y},\] 显然成立;(这一步由秋风自己给出) (2)若 $0<x<y<m$,则不等式等价于 \[\frac{x^x}{y^y}>(y-x)^{y-x},\] 整理为 \[\left( \frac xy \right)^y>\bigl(x(y-x)\bigr)^{y-x},\] 令 $y=x+tx$,其中 $t>0$,代入上式可以化简为 \[\left( \frac1{1+t} \right)^{1+t}>(tx^2)^t,\] 由 $y<m$ 得 $x<m/(1+t)$,那么 $m$ 只需满足 \[\left( \frac1{1+t} \right)^{1+t}\geqslant t^t\left( \frac m{1+t} \right)^{2t}\] 对 $t>0$ 恒成立即可。继续化简为 \[m^2\leqslant\frac{(1+t)^{1-\frac1t}}t,\] 因此,只要求出上式右边的一个下界,如果这个下界为正,问题便解决。为此,我们令 \[g(t)=\frac{t-1}t\ln (1+t)-\ln t.\] (i)当 $t\leqslant 1$ 时,求导得 \[g'(t)=\frac{(1+t)\ln (1+t)-2t}{t^2(1+t)}\leqslant \frac{(1+t)t-2t}{t^2(1+t)}=\frac{t-1}{t(1+t)}\leqslant 0,\] 从而有 $g(t)\geqslant g(1)=0$; (ii)当 $t>1$,则 \[g(t)>\frac{t-1}t\ln t-\ln t=-\frac{\ln t}t,\] 易证 $(\ln t)/t\leqslant 1/e$,故得到 \[g(t)>-\frac1e.\] 综合(i)(ii)得到对 $t>0$ 总有 \[g(t)>-\frac1e\riff\frac{(1+t)^{1-\frac1t}}t>e^{-1/e},\] 所以,只要 $m\leqslant e^{-1/(2e)}\approx 0.83198$ 就能使原不等式恒成立。 注:本来我心头有点高,想求出 $m$ 的最大值,也就是想直接求出 $g(t)$ 的最小值,可惜是超越的,所以只求了个下界。
秋风树林 2# 2013-3-22 21:50
本帖最后由 秋风树林 于 2013-3-22 22:01 编辑 我来给一下问题的原始背景. 一个函数的连续有许多类型,比如一致连续就是比连续更强的连续. 在微分方程理论中,有两种连续对解的存在唯一性起着至关重要的作用. 先给出两种连续的定义: (1)设$G$是一个区域且$f(x)$为$G$上的连续函数,$L$为一常数,如果$f(x)$满足以下条件: \[\abs{f(x_1)-f(x_2)} \leqslant L\abs{x_1-x_2}\] 则称$f(x)$为$Lipschitz$连续. (2)若对于一个正数$\delta$,和一个在$r>0$上满足$F(r)>0$的连续函数: \[\int_{0}^{\delta}\frac1{F(r)}dr=\infty\] 并且对$f(x)$满足: \[\abs{f(x_1)-f(x_2)} \leqslant F(\abs{x_1-x_2})\] 则称$f(x)$为$Osgood$连续. 显然,$Lipschitz$连续是$Osgood$连续的一种特殊情形. 现在的问题是找出一个函数$f(x)$使其满足$Osgood$连续但不满足$Lipschitz$连续. 现有一函数其导函数不能被直线所控制,但是能被对数曲线所控制,这正好是构造的一种思想. 容易验证$f(x)=x\ln x$不满足$Lipschitz$连续且$F(r)=\abs{r\ln r}$满足$Osgood$连续中所需要的条件. 从而问题转化为是否有: \[\abs{f(x_1)-f(x_2)} \leqslant F(\abs{x_1-x_2})\] 即: \[\abs{x_1\ln x_1-x_2\ln x_2} \leqslant \abs{(x_1-x_2)\ln \abs{x_1-x_2}}\] 等价的,即比较$|\ln \dfrac{x^x}{y^y}|$与$\abs{ln \abs{x-y}^{\abs{x-y}}}$的大小. 进一步地,可考虑$\dfrac{x^x}{y^y}$与$\abs{x-y}^{\abs{x-y}}$的大小关系,这里$x$与$y$的关系任意. 从而引出了现在的问题,背景介绍完毕.
kuing 3# 2013-3-22 21:57
2# 秋风树林 是不是干脆说 $F(r)=\abs{r\ln r}$ 好些……
秋风树林 4# 2013-3-22 21:58
2# 秋风树林 是不是干脆说 $F(r)=\abs{r\ln r}$ 好些…… kuing 发表于 2013-3-22 21:57 嗯,我去改一下
thread-1265-1-1.html: 我只是想用一个贴子试试怎么写...
秋风树林 1# 2013-3-22 21:02
本帖最后由 秋风树林 于 2013-3-22 21:49 编辑 设$G$是一个区域且$f(x)$为$G$上的连续函数,$L$为一常数,如果$f(x)$满足以下条件: \[\abs{f(x_1)-f(x_2)} \leqslant L\abs{x_1-x_2}\] 则称$f(x)$为$Lipschitz$连续. 若对于一个正数$\delta$,和一个在$r>0$上满足$F(r)>0$的连续函数: \[\int_{0}^{\delta}\frac1{F(r)}dr=\infty\] 并且对$f(x)$满足: \[\abs{f(x_1)-f(x_2)} \leqslant F(\abs{x_1-x_2})\] 则称$f(x)$为$Osgood$连续. 显然,$Lipschitz$连续是$Osgood$连续的一种特殊情形. 现在的问题是找出一个函数$f(x)$使其满足$Osgood$连续但不满足$Lipschitz$连续. 现有一函数其导函数不能被直线所控制,但是能被对数曲线所控制,这正好是构造的一种思想. 容易验证$f(x)=x\ln x$不满足$Lipschitz$连续且$F(r)=r\ln r$满足$Osgood$连续中所需要的条件. 从而问题转化为是否有: \[\abs{f(x_1)-f(x_2)} \leqslant F(\abs{x_1-x_2})\] 即: \[\abs{x_1\ln x_1-x_2\ln x_2} \leqslant \abs{(x_1-x_2)\ln \abs{x_1-x_2}}\] 等价的,即比较$|\ln \dfrac{x^x}{y^y}|$与$\abs{ln \abs{x-y}^{\abs{x-y}}}$的大小. 当考虑$0<y<x<1$的条件时,便转化为比较$|\ln \dfrac{x^x}{y^y}|$与$\abs{ln (x-y)^{x-y}}$的大小. 考虑到绝对值对数函数在其小于1的范围的递减性质, 从而问题变为考虑$\dfrac{x^x}{y^y}$与$(x-y)^{x-y}$的大小关系.(*) 其实写到这里我突然发现由于幂指函数的非单调性,其实(*)与原不等式并不等价,汗.... 还是比较$\dfrac{x^x}{y^y}$与$\abs{x-y}^{\abs{x-y}}$好了...
kuing 2# 2013-3-22 21:03
也可以用置顶的草稿本来试玩
秋风树林 3# 2013-3-22 21:04
2# kuing 很久以前就说要学...拖了好久
kuing 4# 2013-3-22 21:05
我也想学很多东西,同拖
kuing 5# 2013-3-22 21:28
来回编辑贴子很麻烦,建议你试试用置顶草稿本……
kuing 6# 2013-3-22 21:34
等价的,即比较$|\ln \dfrac{x^x}{y^y}|$与$|ln |x-y|^{|x-y|}|$的大小. 貌似后者外面并没绝对值? PS、绝对值也可以用 \abs{...} 长度自动适应
秋风树林 7# 2013-3-22 21:36
来回编辑贴子很麻烦,建议你试试用置顶草稿本…… kuing 发表于 2013-3-22 21:28 和这个贴子里编辑有什么区别吗? 还是说一次试验发一次贴?
秋风树林 8# 2013-3-22 21:37
等价的,即比较$|\ln \dfrac{x^x}{y^y}|$与$|ln |x-y|^{|x-y|}|$的大小. 貌似后者外面并没绝对值? PS、绝对值也可以用 \abs{...} 长度自动适应 kuing 发表于 2013-3-22 21:34 对数出来是个负值就没太大意义了
kuing 9# 2013-3-22 21:38
和这个贴子里编辑有什么区别吗? 还是说一次试验发一次贴? 秋风树林 发表于 2013-3-22 21:36 http://kkkkuingggg.5d6d.net/thread-1204-1-1.html 进去看看……
秋风树林 10# 2013-3-22 21:40
和这个贴子里编辑有什么区别吗? 还是说一次试验发一次贴? 秋风树林 发表于 2013-3-22 21:36 突然发现是某个式子忘了加绝对值...
isea 11# 2013-3-26 00:22
来回编辑贴子很麻烦,建议你试试用置顶草稿本…… kuing 发表于 2013-3-22 21:28 那玩意,我在电脑里,有多行公式时,卡得不行…… 所以,我干脆就用CTEX 我试试这里能用 \renewcommand{\baselinestretch}{2.1}不 $\renewcommand{\baselinestretch}{2.1} 12\\34\\56\\78\\9 $
kuing 12# 2013-3-26 01:01
那玩意,我在电脑里,有多行公式时,卡得不行…… 所以,我干脆就用CTEX 我试试这里能用 \renewcommand{\baselinestretch}{2.1}不 $\renewcommand{\baselinestretch}{2.1} 12\\34\\56\\78\\9 $ isea 发表于 2013-3-26 00:22 我这里还好……打比较长篇就是反应会有点点慢,但不至于很卡。 那个命令在这里应该没用。
李斌斌755 13# 2013-4-26 12:11
12# kuing 学latex是件体力活
yes94 14# 2013-4-26 13:33
在5、6行以内还不算很慢的,10行了就很慢了,尤其用了什么begin语句
kuing 15# 2013-4-26 14:17
14# yes94 大概跟浏览器和电脑配置有关……我这里打很长也不觉得很卡
李斌斌755 16# 2013-5-12 22:22
本帖最后由 李斌斌755 于 2013-5-12 22:25 编辑 一举两得(摘录realnumber在“有ln的常见不等式”中的回复) \[(\dfrac{\ln(2x+1)}2)'=\dfrac1{2x+1}\]有\[\dfrac1{2n+1}\geqslant\dfrac12\ln(1+\dfrac2{2n+1})=\dfrac{2n+3}2-\dfrac{2n+1}2\]所以\[1+\dfrac13+\dfrac15+\cdots+\dfrac1{2n-1}>1+\dfrac13+\dfrac{\ln(2n+1)}2-\dfrac{\ln5}2\\\dfrac13+\dfrac15+\cdots+\dfrac1{2n+1}>\dfrac13+\dfrac{\ln(2n+3)}2-\dfrac{\ln5}2\]两式相加
李斌斌755 17# 2013-5-12 23:51
接着练(摘录kuing) 依题意设$c=ub$,则$u\geqslant1,b\geqslant1$,由构成三角形条件知$1+b\geqslant ub$,于是\[1\leqslant u<\dfrac1b+1\]而 \[\max\left\{\dfrac ab,\dfrac bc,\dfrac ca\right\}=\dfrac ca=ub\\\min\left\{\dfrac ab,\dfrac bc,\dfrac ca\right\}=\min\left\{\dfrac ab,\dfrac bc\right\}=\min\left\{\dfrac1b,\dfrac1u\right\}\]于是\[t=ub\min\left\{\dfrac1u,\dfrac1u\right\}=\min\left\{b,u\right\}\]因$b\geqslant1,u\geqslant1$,故$t\geqslant1$,当且仅当$b=u=1$时取等; 又\[t\leqslant u<\dfrac1b+1\leqslant\dfrac1t+1\riff t<\dfrac{\sqrt5+1}2\]当$b=u\to(\sqrt5+1)/2$时$t\to(\sqrt5+1)/2$。
thread-1266-1-3.html: 原来CYH有这样一个意思
kuing 1# 2013-3-23 00:30
话说刚才闲来无事,突然间想到,“CYH技巧”的 C、Y、H 这三个字母到底代表着什么呢? 为此,翻到了这一段 不必看正文,请留意脚注,翻译了中间的 yêu ,原来是“爱”,瞬间感动到想
zdyzhj 2# 2013-3-23 08:05
你现在才知道阿,其实can 是他本人,hang是她老婆,他起了个canhang2007就是二人名字的组合阿,呵呵,爱意浓浓!或惜据说分手了。。
kuing 3# 2013-3-23 10:46
你现在才知道阿,其实can 是他本人,hang是她老婆,他起了个canhang2007就是二人名字的组合阿,呵呵,爱意浓浓!或惜据说分手了。。 zdyzhj 发表于 2013-3-23 08:05 你所说的我当然早就知道的,不然我怎么会只翻译了中间那字就会感动到? 网名取二人组合并不希奇,但是将证明技巧(而且可以说那是他的绝技之一)也这样取名就令我意想不到了。
yes94 4# 2013-3-23 14:09
这么具有传奇色彩?
isea 5# 2013-3-23 23:28
那是什么文字?
kuing 6# 2013-3-23 23:29
5# isea 越南 越南不等式牛……
zdyzhj 7# 2013-3-24 15:09
他的CS技巧,现在看来,我已不觉得多么神奇了!呵可
pxchg1200 8# 2013-3-24 16:54
7# zdyzhj 既然你认为Can的柯西没什么神奇了,那么麻烦你用CS做下这个题看看你的柯西有没有Can的程度再来说这话。 Let $a,b,c>0$ prove that \[ \frac{a^2}{a^2+b^2+ab+ca}+\frac{b^2}{b^2+c^2+bc+ab}+\frac{c^2}{c^2+a^2+ca+bc}\geq \frac{3}{4} \]
yes94 9# 2013-3-24 21:34
8# pxchg1200 县长一贯的作风
pxchg1200 10# 2013-3-24 23:38
9# yes94 这货怎么当上县长的?
yes94 11# 2013-3-24 23:40
10# pxchg1200 搞不清群里为何总叫他县长,
zdyzhj 12# 2013-3-25 07:57
那个小问题,刚才上班,我已轻松搞定了。
力工 13# 2013-3-25 08:41
10# pxchg1200 让子弹飞中的县长,假的,因为嘴大好吹!故名之
yes94 14# 2013-3-25 10:07
10# pxchg1200 让子弹飞中的县长,假的,因为嘴大好吹!故名之 力工 发表于 2013-3-25 08:41 《让子弹飞》
thread-1267-1-3.html: [几何] 一个椭圆类准线与类焦点的结论(资料1)
yes94 1# 2013-3-23 17:19
______kuing edit in $\LaTeX$______ 定理  已知点 $M(m,0)$($0<\abs m<a$)是椭圆 $C: x^2/a^2+y^2/b^2=1$($a>b>0$)内一定点,$P$ 是椭圆 $C$ 上的点,$Q$ 是直线 $x=a^2/m$ 上的点(如图 3)。记直线 $QM$、$PM$ 的斜率为 $k_{QM}$、$k_{PM}$,则直线 $PQ$ 为椭圆 $C$ 的切线的充分必要条件为 \[k_{QM}\cdot k_{PM}=-\frac{b^2}{a^2-m^2}.\]
第一章 2# 2013-3-23 20:17
双曲线、抛物线也应该有类似的结论?!
yes94 3# 2013-3-23 22:22
2# 第一章 应该有吧
isea 4# 2013-3-23 23:26
本帖最后由 isea 于 2013-3-25 11:09 编辑 也添加一个常见的,一般结论: 已知直线$l$与椭圆$C:\dfrac {x^2}{a^2}+\dfrac {y^2}{b^2}=1$交于两点$A,B$。坐标原点$O$到直线$l$的距离为$d>0$, 则$\vv {OA}\perp\vv {OB}\Leftrightarrow d=\dfrac{ab}{\sqrt{a^2+b^2}}$. 然,后一般还有一问,求$S_{\triangle{ABC}}$的面积最大值,其结论为$\left (S_{\triangle{ABC}}\right )_{\max}=\dfrac{ab}2$. =========================== 如果说是资料,最好录成文字,图片,时间一长,服务器必删除
kuing 5# 2013-3-24 00:02
4# isea 代码的各种好处…… 不过其实也不用怕,我偶尔备份纯文字版,也偶尔批量保存图片……
yes94 6# 2013-3-24 23:17
kuing的这道题和该命题有关:http://www.pep.com.cn/rjwk/gzsxs ... 0120724_1133829.htm
kuing 7# 2013-3-24 23:18
6# yes94 昨天就意识到了,1#结论是推广。
yes94 8# 2013-3-24 23:50
现在忽略PQ是否为切线这个问题,问是否有三斜率$k_{PQ}$,$k_{MQ}$,$k_{P'Q}$成等差数列?
kuing 9# 2013-3-25 01:29
6# yes94 昨天就意识到了,1#结论是推广。 kuing 发表于 2013-3-24 23:18 你说能不能通过压缩变换将其转化到。。。
yes94 10# 2013-3-25 10:11
9# kuing 对压缩变换几乎没有研究,因为对压缩变换的不变量不是太清楚,或者压缩系数问题不明白,似乎与 雅克比行列式有关?
kuing 11# 2013-3-25 12:56
10# yes94 作变换 \[\left\{\begin{aligned} X&=x,\\ Y&=\frac{\sqrt{a^2-m^2}}by, \end{aligned}\right.\] 则切线仍为切线,而直线斜率 \[K=\frac{\sqrt{a^2-m^2}}bk,\] 椭圆变成 \[\frac{x^2}{a^2}+\frac{Y^2}{a^2-m^2}=1,\] 注意到 $M(m,0)$ 和直线 $x=a^2/m$ 不变,正好变为椭圆的焦点和准线,所以根据链接中的结论有 \[K_{QM}\cdot K_{PM}=-1\iff\frac{\sqrt{a^2-m^2}}bk_{QM}\cdot \frac{\sqrt{a^2-m^2}}bk_{PM}=-1\iff k_{QM}\cdot k_{PM}=-\frac{b^2}{a^2-m^2}.\]
第一章 12# 2013-3-25 18:17
k可以就此写一篇论文了。 PS,之前看了《空间》的那个结论,命制了一个考题,抛物线的。比较简单(适合高中生做;其实,难度大的题我也命制不了)
kuing 13# 2013-3-25 18:21
12# 第一章 圆锥曲线的论文我可不敢随便写,很容易撞车的,比如说“这不都是几十年前的结论了?”、“我早就推过了!”、“见《数学XX》YY年第ZZ期……”等……
yes94 14# 2013-3-25 21:36
的确,圆锥曲线的性质、知识在很久以前就很成熟啦! 圆锥曲线在约公元前200年时就已被命名和研究了,其发现者为古希腊的数学家阿波罗尼阿斯,那时阿波罗尼阿斯对它们的性质已做了系统性的研究。 http://baike.baidu.com/view/1182424.htm http://zh.wikipedia.org/wiki/%E5%9C%86%E9%94%A5%E6%9B%B2%E7%BA%BF 只是,搞点新意、变通、新应用之类的还是可以的,
kuing 15# 2013-3-25 21:46
阿波罗尼奥斯太牛比,当年压根还没有坐标系这东西,他那真的是完全用纯几何玩的圆锥曲线。 虽然我也经常这样玩,可是我也不敢说我有某次做的东西是新的。
yes94 16# 2013-3-25 21:52
15# kuing 对,他那时还没有坐标系吧?虽然他有些坐标系的思想了,完全是利用定义搞平面几何解法,真的niubility! kuing也喜欢用平面几何解法解决解析几何问题,也相当的niubility!这叫英雄所见略同,(不是抄袭哈)
第一章 17# 2013-3-26 09:42
15# kuing 当年……
yes94 18# 2013-3-26 17:56
17# 第一章 当年叱咤风云?
第一章 19# 2013-3-26 20:56
说的是k,他的那个回复用了“当年”二字,我还以为那个阿波罗尼奥斯生活在当下呢
第一章 20# 2013-3-26 21:02
话说这个阿波罗尼奥斯就是那个阿波罗尼斯吧, 以前因为那个阿氏圆认识他,开始叫阿波罗,后来叫阿波罗尼斯,现在叫阿波罗尼奥斯,下次直接叫阿波罗鸟事。
thread-1267-2-3.html:
yes94 21# 2013-3-26 21:02
19# 第一章 哦,
kuing 22# 2013-3-26 21:13
19# 第一章 “当年”是这样理解的么……我这里指的是他还活着的年代…… 话说距今有多少年?
thread-1268-1-3.html: 来自人教论坛的一道统计(兼复习)
kuing 1# 2013-3-23 19:38
看到这个贴 http://bbs.pep.com.cn/forum.php?mod=viewthread&tid=2703114 让我想复习一下以前高中学过的一点点统计初步,虽然其实从来没认真学过。 复习最好就是自己推一下,如有任何错的请马上指出。 $N$ 个东东里有 $n$ 个 $A$,$m$ 个 $B$,$n+m=N$。 现从中不放回地随机抽取 $k$ 个,则抽到 $x$ 个 $A$ 的概率为 $C_n^x C_m^{k-x}/C_N^k$,于是抽到 $A$ 的个数 $X_A$ 的期望值为 \begin{align*} E(X_A)&=\sum_{x=1}^k\frac{xC_n^x C_m^{k-x}}{C_N^k}\\ &=\frac n{C_N^k}\sum_{x=1}^k C_{n-1}^{x-1} C_m^{k-x}\\ &=\frac{n C_{n-1+m}^{k-1}}{C_N^k}\\ &=\frac{nk}N. \end{align*} (变形过程用到两个组合恒等式,均能在 http://kkkkuingggg.5d6d.net/thread-878-1-1.html 找到,下同) 现在,我们看看放回的情况又如何。有放回地随机抽取 $k$ 个,抽到 $x$ 个 $A$ 的概率为 $C_k^x(n/N)^x(m/N)^{k-x}$,于是抽到 $A$ 的个数 $X_A$ 的期望值为 \begin{align*} E(X_A)&=\sum_{x=1}^k \frac{x C_k^x n^x m^{k-x}}{N^k}\\ &=\frac k{N^k}\sum_{x=1}^k C_{k-1}^{x-1} n^x m^{k-x}\\ &=\frac{nk}{N^k}\sum_{x=1}^k C_{k-1}^{x-1} n^{x-1} m^{k-x}\\ &=\frac{nk}{N^k}(n+m)^{k-1}\\ &=\frac{nk}N. \end{align*} 结果的确是相同的,同理,无论是有放回还是无放回,抽到 $B$ 的个数的期望值都为 $E(X_B)=mk/N$。 于是,可以回到链接的题目中,如果抽到每个 $A$ 记 $p$ 分,每个 $B$ 记 $q$ 分,那么当抽取 $k$ 个时,那么无论是有放回还是无放回,总分 $Y$ 的期望值都是 \[E(Y)=p\cdot E(X_A)+q\cdot E(X_B)=\frac{k(np+mq)}N.\] 也就是说原贴里那两种计算方法结果相同是必然的。
kuing 2# 2013-3-23 19:44
先吃饭,晚点再复习一下方差,and……我突然发现我好像忘记了方差的定义式,看来还得类比一下经典情形……先闪
kuing 3# 2013-3-23 20:46
接楼上,先把定义式类比出来先。 经典情形,设有 $N$ 个数据,其中有 $n_i$ 个 $x_i$, $i=1$, $2$, \ldots, $k$,则这些数据的平均数为 \[\bar x=\sum_{i=1}^k \frac{n_i}Nx_i,\] 方差为 \[ s^2=\sum_{i=1}^k \frac{n_i}N(x_i-\bar x)^2 =\sum_{i=1}^k \frac{n_i}Nx_i^2-2\bar x\sum_{i=1}^k \frac{n_i}Nx_i+\bar x^2\sum_{i=1}^k \frac{n_i}N =\sum_{i=1}^k \frac{n_i}Nx_i^2 - \bar x^2. \] 类比一下,随机变量 $X$,其中 $X=x_i$ 发生的概率为 $p_i$, $i=1$, $2$, \ldots, $k$,则 $X$ 的期望为 \[E(X)=\sum_{i=1}^k p_ix_i,\] 方差为 \[D(X)=\sum_{i=1}^k p_i\bigl(x_i-E(X)\bigr)^2 =\sum_{i=1}^k p_ix_i^2-2E(X)\sum_{i=1}^k p_ix_i+E(X)^2\sum_{i=1}^k p_i =E(X^2)-E(X)^2.\] 这样应该没错吧? 如果没问题的话,$k\to\infty$ 时以上公式应该也适用,而对于连续的随机变量,大概就是要变成积分,等我想想应该怎么写,待续……
第一章 4# 2013-3-23 21:00
超几何分布与二项分布的区别啊。 k推出的那个公式是超几何分布的期望公式吧。记得方差也有公式的。
kuing 5# 2013-3-23 21:14
4# 第一章 嗯,等会也推推,方法应该差不多,以前没认真学,现在几乎是从头开始推,连定义式都要重新想想……慢慢来……
kuing 6# 2013-3-23 21:36
发现有些写法应该改下好些……改了下1# 话说,连续的随机变量还真不知怎么写……貌似没接触过,先放一会……
yayaweha 7# 2013-3-23 21:49
第一次见超几何的期望公式
yayaweha 8# 2013-3-23 21:52
1# kuing 当年我也有这个疑问,现在终于明白了
第一章 9# 2013-3-23 21:55
超几何分布的期望、方差公式其实没多大用处(在高考里面),一方面,高中试卷里面提供的数据不复杂,直接算概率得了;另一方面,绝大多数的老师还真不知道这些公式。
秋风树林 10# 2013-3-23 22:07
其实... $D(X)=E(X-EX)^2=E(X^2-2X*EX+(EX)^2)=E(X^2)-(EX)^2$
kuing 11# 2013-3-23 22:12
10# 秋风树林 还能这样写……
kuing 12# 2013-3-23 22:34
还是推一下方差吧 当组合数上标为负数时定义其值为 $0$,这样对于正整数 $x$ 都有 \[x^2C_n^x=xnC_{n-1}^{x-1}=nC_{n-1}^{x-1}+n(x-1)C_{n-1}^{x-1}=nC_{n-1}^{x-1}+n(n-1)C_{n-2}^{x-2},\] 故 \begin{align*} \sum_{x=1}^k x^2 C_n^x C_m^{k-x} &= n\sum_{x=1}^k C_{n-1}^{x-1} C_m^{k-x} + n(n-1)\sum_{x=2}^k C_{n-2}^{x-2} C_m^{k-x}\\ &=nC_{n-1+m}^{k-1} + n(n-1)C_{n-2+m}^{k-2}\\ &=\frac{nk}NC_N^k + \frac{n(n-1)k(k-1)}{N(N-1)}C_N^k, \end{align*} 于是根据上述公式,对于不放回的方差就是 \[D(X_A)=\frac{nk}N+\frac{n(n-1)k(k-1)}{N(N-1)}-\left(\frac{nk}N\right)^2=\frac{kn(N-k)(N-n)}{N^2(N-1)}.\] 有放回的情形,有 \begin{align*} \sum_{x=1}^k x^2 C_k^x n^x m^{k-x} &= k\sum_{x=1}^k C_{k-1}^{x-1} n^x m^{k-x} + k(k-1)\sum_{x=2}^k C_{k-2}^{x-2} n^x m^{k-x}\\ &=nk\sum_{x=1}^k C_{k-1}^{x-1} n^{x-1} m^{k-x} + n^2k(k-1)\sum_{x=2}^k C_{k-2}^{x-2} n^{x-2} m^{k-x}\\ &=nk(n+m)^{k-1}+n^2k(k-1)(n+m)^{k-2}, \end{align*} 于是根据上述公式,对于有不放回的方差就是 \[D(X_A)=\frac{nk}N+\frac{n^2k(k-1)}{N^2}-\left(\frac{nk}N\right)^2=\frac{nk(N-n)}{N^2}.\]
kuing 13# 2013-3-23 23:03
查了下维基,结果好像一样,不过用的字母不同,看得我的……
kuing 14# 2013-3-23 23:15
顺便地,对于两种情形的方差的差,有 \[\frac{nk(N-n)}{N^2}-\frac{kn(N-k)(N-n)}{N^2(N-1)}=\frac{nk(k-1)(N-n)}{N^2(N-1)}\geqslant 0,\] 所以不方回的方差总是大些。
yes94 15# 2013-3-24 21:51
是的,大学概率论教材详细证明并给出了超几何分布的期望和方差的,特别是期望公式与二项分布的期望相同,方差虽然不同,但是超几何分布取了极限,就和二项分布的方差想通相同了。 证明时要用到一些组合恒等式。
thread-1269-1-3.html: [不等式] 通项与不等式
yayaweha 1# 2013-3-23 21:57
本帖最后由 yayaweha 于 2013-3-23 22:01 编辑 已知$a_n=n$  ,   $b_1=\frac{1}{2}  ,b_{n+1}=\frac{1}{a_k}b_n^2+b_n其中n\le k $ 求证:$$b_n<1$$
yayaweha 2# 2013-3-23 22:01
1# yayaweha 我又遇倒不会的题了
realnumber 3# 2013-3-24 07:27
看到过,~~~
yes94 4# 2013-3-24 21:43
我也来一道类似题: $a_0=\dfrac{1}{2}$,$a_n=a_{n-1}+\dfrac{a_{n-1}^2}{n^2}$,求证:$\dfrac{n+1}{n+2}<a_n<n$。
thread-127-1-8.html: [不等式] 来自pep的弱根式不等式 $\sum\sqrt{ab(a+b)}>\sqrt{\prod(a+b)}$
kuing 1# 2011-10-21 19:37
来自 http://bbs.pep.com.cn/thread-1926228-1-1.html $a,b,c\in\mathbb{R}^+$\[\sqrt{ab(a+b)}+\sqrt{bc(b+c)}+\sqrt{ca(c+a)}>\sqrt{(a+b)(b+c)(c+a)}.\] 两边平方是容易证明的,而且可以加强,详情见:http://www.artofproblemsolving.c ... ?f=151&t=363314 此外,两边除右边,等价于 \[\sum\sqrt{\frac{ab}{(b+c)(c+a)}}>1,\] 这其实等价于三角形不等式 \[\sin\frac A2+\sin\frac B2+\sin\frac C2>1,\] 其三角证法也是熟知的了。 又或者用均值来证,变形为 \[\sum\frac{ab}{\sqrt{a(b+c)}\sqrt{b(c+a)}}>1,\] 由均值有 \[\sum\frac{ab}{\sqrt{a(b+c)}\sqrt{b(c+a)}}\geqslant\sum\frac{2ab}{a(b+c)+b(c+a)}=\sum\frac{2ab}{2ab+bc+ca}>\sum\frac{ab}{ab+bc+ca}=1.\]
pxchg1200 2# 2011-10-23 15:14
我来个稍微有意思点的。呵呵 Let $a,b,c \geq 0 $ prove that: \[ \sqrt{a^{2}+4bc}+\sqrt{b^{2}+4ac}+\sqrt{c^{2}+4ab}\geq \sqrt{15(ab+bc+ca)} \] ( 我暂时还没想到怎么做。。 )
kuing 3# 2011-10-23 21:22
2# pxchg1200 又大变脸……
海盗船长 4# 2011-11-2 18:34
thread-1270-1-3.html: [数列] 讨论数列敛散性
Gauss门徒 1# 2013-3-23 22:34
\[a_1=1, a_2=2, a_{n+1}=\frac{a_na_{n-1}+1}{a_{n-1}}\]
kuing 2# 2013-3-23 23:18
呃,能不能这样,假设收敛于 $A$,则必有 $A=(A^2-1)/A$,无解……所以不收敛
Gauss门徒 3# 2013-3-24 00:35
2# kuing i mean 能不能 估一个下界
kuing 4# 2013-3-24 00:36
3# Gauss门徒 这 mean 跟标题的 mean …………嗯嗯很接近……
kuing 5# 2013-3-24 17:13
不知这里的招式能不能用http://kkkkuingggg.5d6d.net/thread-904-1-1.html
thread-1271-1-1.html: 这是什么样的立体图形
isea 1# 2013-3-23 23:26
本帖最后由 isea 于 2013-3-24 23:15 编辑 特别是那个虚线,看不明白
isea 2# 2013-3-23 23:41
原来是这样的,在人教被秒了 http://bbs.pep.com.cn/forum.php? ... &extra=page%3D1 人多力量大啊
kuing 3# 2013-3-23 23:47
其实应该发到数学区的,这里少人看,我都几乎没留意到……所以慢了……
isea 4# 2013-3-24 23:15
其实应该发到数学区的,这里少人看,我都几乎没留意到……所以慢了…… kuing 发表于 2013-3-23 23:47 没注意 如果能的话,移过去吧
kuing 5# 2013-3-24 23:15
懒了……反正也解决了……
isea 6# 2013-3-24 23:18
懒了……反正也解决了…… kuing 发表于 2013-3-24 23:15 所谓机不可失,失不再来
isea 7# 2013-3-24 23:19
今天网速好像特别快,访问5d6d,北京网通
kuing 8# 2013-3-24 23:21
我电信……一直都快…… PS、是不是有个说法叫南电信北网通……
isea 9# 2013-3-24 23:24
8# kuing 对,垄断的结果
kuing 10# 2013-3-24 23:25
哎,网 fei 好贵……
isea 11# 2013-5-14 22:01
本帖最后由 isea 于 2013-5-14 23:51 编辑 又来这样的变度视图,放一块 == 应该有后加的黑线
kuing 12# 2013-5-14 22:02
11# isea 没看懂,俯视图右边没有线?
李斌斌755 13# 2013-5-14 23:16
11# isea 四面体$A_1-BCD$
李斌斌755 14# 2013-5-14 23:18
13# 李斌斌755 不对,多$CD$
isea 15# 2013-5-14 23:50
应该有,我觉着有,没线,我看不懂。
isea 16# 2013-5-14 23:52
跟主楼一样的玩意
李斌斌755 17# 2013-5-14 23:54
本帖最后由 李斌斌755 于 2013-5-15 00:02 编辑 15# isea 虚线必需有实线围合(有一以上平面盖住它),否则就是实线。
isea 18# 2013-5-14 23:56
17# 李斌斌755 甚有理!
李斌斌755 19# 2013-5-19 16:18
这怎么可能?
kuing 20# 2013-5-19 16:22
19# 李斌斌755 这个太常见了吧,你竟然也不知道原因?
thread-1271-2-1.html:
李斌斌755 21# 2013-5-19 16:26
20# kuing 不知道,猜是斜边不是直线,偷出来的
李斌斌755 22# 2013-5-19 17:31
本帖最后由 李斌斌755 于 2013-5-19 20:16 编辑 你看到什么?怎么上不了图?这下可以了吧。
kuing 23# 2013-5-19 19:44
22# 李斌斌755 估计你的图片过大 最近论坛附件被限制为<=256K
李斌斌755 24# 2013-5-20 00:29
23# kuing 有图了。
kuing 25# 2013-5-20 00:43
嗯,这个也见过不少次了,海豚 or 男女……
李斌斌755 26# 2013-5-20 01:57
25# kuing 厉害,不玩了。
李斌斌755 27# 2013-5-28 02:21
本帖最后由 李斌斌755 于 2013-5-28 02:23 编辑 今天人教群里的,说是模拟题,圆和对角线均为虚线
isea 28# 2013-5-28 02:33
27# 李斌斌755 求啥?看我能不能猜出结果。 一眼看上去,图是看不懂的
李斌斌755 29# 2013-5-28 03:15
28# isea 求是什么样的几何体。
thread-1272-1-1.html: 3道积分计算
╰☆ヾo.海x 1# 2013-3-24 03:05
哎,第一个试着换三角了,还是不行。。第二个分母又不能再分,彻底没思路。。第三个也不知道怎么分了。。
╰☆ヾo.海x 2# 2013-3-24 04:23
1# ╰☆ヾo.海x 在天下的指导下。。都解决了。。。哈哈
╰☆ヾo.海x 3# 2013-3-24 05:33
2# ╰☆ヾo.海x 但是又有新问题了。。哎,本以为这第一个可以用跟那个第一题一样的方法,结果算不出来。。第二个公式怎么证啊。。 例如算1/(x^2+2x+3)dx的积分怎么求?
kuing 4# 2013-3-24 09:34
又见 $\succ$ 当 $>$ 用的……
kuing 5# 2013-3-24 09:51
3# ╰☆ヾo.海x 判别式非负0时其实可以再分,判别式负时分母配方换元。
kuing 6# 2013-3-24 10:28
第一题三角代换后还是可以继续的,不过大概还得再换…… 当 $x\ne -\sqrt2/2$, $-1$, $1$ 时,可令 $x=\cos t$, $t\in (0,3\pi /4)\cup (3\pi /4,\pi )$,则 \[\int\frac{\rmd x}{x+\sqrt{1-x^2}}=\int\frac{-\sin t\rmd t}{\cos t+\sin t}=-\int\frac{\rmd t}{\cot t+1},\] 再令 $t=\arccot u$, $u\in \mbb R\setminus\{-1\}$,则 \[-\int\frac{\rmd t}{\cot t+1}=-\int\frac{\rmd{\arccot u}}{u+1}=\int\frac{\rmd u}{(u+1)(u^2+1)}=\frac12\int\frac{\rmd u}{u+1}-\frac12\int\frac{u-1}{u^2+1}\rmd u,\] 然后代公式,积出后回代,最后验证那几个奇点……挺麻烦
kuing 7# 2013-3-24 10:30
汗哉,居然没有 \arccot 这命令……
kuing 8# 2013-3-24 10:50
爪机拍得太差……
╰☆ヾo.海x 9# 2013-6-5 08:32
8# kuing 太感动了。。。。酷儿还翻书拍照片上传来着....你都要保存下来啊!!!哎但是关键是保存下来后什么时候能正常重见天日不知道啊..
╰☆ヾo.海x 10# 2013-6-5 08:41
6# kuing 哇你第一步真的写的好规范好严格。。。我换元从来都不考虑范围。。。。。 我后来做的还比较简单,直接令x=sint的 然后原式被积部分就变成 1-1/(1+cost),然后再用半角公式变成含(sec(t/2))^2项。。 一定会等到一个全新的 悠闲数学娱乐论坛。。。
kuing 11# 2013-6-5 13:35
现在才来感动……当时看了就过了对不对 ……
╰☆ヾo.海x 12# 2013-6-5 23:25
11# kuing
thread-1273-1-3.html: 当面点师碰到数学
isea 1# 2013-3-24 21:45
本帖最后由 isea 于 2013-3-24 21:49 编辑 源自:2009年上海市普通高等学校春季招生考试 数学卷第11题。 偶以前没注意,这是第一次见到,觉得有点意思—— 以下是面点师一个工作环节的数学模型:如图(图略),在数轴上截取与闭区间$[0,4]$对应的线段,对折后(坐标$4$所对应的点与原点重合) 再均匀地拉成$4$个单位长度的线段,这一过程称为一次操作(例如在第一次操作完成后,原来的坐标$1$、$3$变成$2$,原来的坐标$2$变成$4$,等等)。 那么原闭区间$[0,4]$上(除两个端点外)的点,在第$n$次操作完成后($n \ge 1$),恰好被拉到与$4$重合的点所对应的坐标为$f(n)$,则$f(3)=?$;$f(n)=?$。
yes94 2# 2013-3-24 21:58
标题还很好听,吸引人。 还改遍了原题:http://wenku.baidu.com/view/66ce24c58bd63186bcebbc0e.html
isea 3# 2013-3-24 23:22
2# yes94 这个题似乎还只能通过归纳法,慢慢寻找规律 递推式似乎不好办,似乎,……
李斌斌755 4# 2013-3-25 10:21
本帖最后由 李斌斌755 于 2013-3-25 15:17 编辑 3# isea N次操作后,面条被拉长到2^n倍,每倍段长相当于原面条的4/2^N=1/2^(n-2)长度。看看拉伸 后各段终点对应坐标情况:先看第一段,其终点对应的原坐标为1/2^(n-2);第二段其终点对应 的原坐标为2*1/2^(n-2);第三段其终点对应的原坐标为3*1/2^(n-2)……第2^n段其终点对应的 原坐标为2^n*1/2^(n-2)。奇数段终点落在数轴4点上。      故   f(n)=i/2^(n-2)     (i属于[1,2^n]内奇数)
isea 5# 2013-3-25 14:16
本帖最后由 isea 于 2013-3-25 14:27 编辑 3# isea N次操作后,面条被拉长到2^n倍,每倍段长相当于原面条的4/2^N=1/2^(n-2)长度。看看拉伸 后各段终点对应坐标情况:先看第一段,其终点对应的原坐标为0、1/2^(n-2);第二段其终点对应 的原坐标为2*1/2^( ... 李斌斌755 发表于 2013-3-25 10:21 改下直接的数学公式:(好像翻译得不对劲,楼上似乎不是这个意思) $N$次操作后,面条被拉长到$2^n$倍,每倍段长相当于原面条的$\dfrac4{2^N}=\dfrac{1}{2^{(n-2)}}$长度。看看拉伸 后各段终点对应坐标情况:先看第一段,其终点对应的原坐标为$0、\dfrac1{2^{(n-2)}}$;第二段其终点对应 的原坐标为$2 \cdot \dfrac1{2^{(n-2)}}$;第三段其终点对应的原坐标为$3 \cdot \dfrac{1}{2^{(n-2)}},\cdots$第$2^n$段其终点对应的 原坐标为$\dfrac{2^n \cdot 1}{2^{(n-2)}}$。奇数段终点落在数轴4点上。       故   $f(n)=\dfrac{i}{2^{(n-2)}}$     ($i\in[1,2^n]$内奇数) PS:在数学公式两端加$\$$符号即显示为公式,论坛。 分式复杂一些,分式代码是 \dfrac{113}{355}, 显示出来是:$\dfrac{113}{355}$
kuing 6# 2013-3-25 14:27
5# isea 美元符前加反斜杠就可以生成普通的美元符。
李斌斌755 7# 2013-3-25 15:27
5# isea 谢谢isea,还得学习输入
kuing 8# 2013-3-25 15:29
将过程画一下就很清楚了 这样,结果显然就是 $4\cdot\dfrac k{2^n}$,其中 $k=1$, $3$, $5$, \ldots, $2^n-1$。
kuing 9# 2013-3-25 15:50
上图的 Mathematica 代码 lmcs = 6; f[1] = ListLinePlot[{{0, 0}, {4, 2}, {0, 4}},    PlotRange -> {{0, 4}, {0, 4}}, AspectRatio -> 1]; Do[{asdf1 = Table[{(1 + (-1)^(k + 1)), 4 k/2^n}, {k, 0, 2^n}];   f[n - 1/2] =    ListLinePlot[asdf1, PlotRange -> {{0, 4}, {0, 4}},     AspectRatio -> 1];   asdf2 = Table[{2 (1 + (-1)^(k + 1)), 4 k/2^n}, {k, 0, 2^n}];   f[n] = ListLinePlot[asdf2, PlotRange -> {{0, 4}, {0, 4}},     AspectRatio -> 1];}, {n, 2, lmcs}] lm = Table[f[n], {n, 1, lmcs, 1/2}] Export["lm.gif", lm, ImageSize -> Large] 复制代码 生成的图片默认放在C:\Documents and Settings\Administrator\My Documents,但是打开发现动画太快了,于是我上面的图还经过处理,放慢了速度,优化了大小。
yes94 10# 2013-3-25 21:44
好高级的手段,以后慢慢学学!
kuing 11# 2013-3-26 15:02
10# yes94 我就懂那么点了……昨天群里进的那位问的我看都看不懂
yes94 12# 2013-3-26 17:55
11# kuing 我一点都不懂
thread-1274-1-1.html: [函数] 三次函数仅有两个不同的零点
isea 1# 2013-3-24 22:28
题:已知函数$f(x)=ax^3+bx^2-2(a\ne0)$有且仅有两个不同的零点$x_1,x_2,$则       A. 当$a<0$时,$x_1+x_2<0,x_1x_2>0$        B. 当$a<0$时,$x_1+x_2>0,x_1x_2<0$       C. 当$a>0$时,$x_1+x_2<0,x_1x_2>0$        D. 当$a>0$时,$x_1+x_2>0,x_1x_2<0$ 偶动用韦达定理,硬生生将两根求出来了,超bào力,不知有没轻便之法,多谢。
yes94 2# 2013-3-24 22:32
1# isea 极大、或极小值为0吧?
kuing 3# 2013-3-24 22:33
跟去年山东理数12差不多……我也是把根求出来,其实也不麻烦,见《数学空间》总第9期P22
isea 4# 2013-3-24 22:38
本帖最后由 isea 于 2013-3-24 22:56 编辑 1# isea 极大、或极小值为0吧? yes94 发表于 2013-3-24 22:32 极值肯定是一个方向,多数人的第一方向,哈哈。 3# kuing 好的,我看看去。 网刊最近到第几期了? ==== 看了,这个思路,跟偶一样,呵呵,寻找图象法,乘了个 $\dfrac{1}{x}$ 变成反比例与二次函数,变成你的说的题的原型了,哈哈……
kuing 5# 2013-3-24 22:41
4# isea 准备出12
kuing 6# 2013-3-24 22:46
4# isea 图象法我在评注里有提了一下
isea 7# 2013-3-24 22:51
本帖最后由 isea 于 2013-3-24 22:53 编辑 4# isea 准备出12 kuing 发表于 2013-3-24 22:41 期待一下! 4# isea 图象法我在评注里有提了一下 kuing 发表于 2013-3-24 22:46 等有空的某天,我从极值方向写个详细的过程,三次函数还是值得深入玩味的
kuing 8# 2013-3-24 22:56
7# isea 我估计大概还要等半个月以上才能出12。 其实初稿上星期已经交上了去,现在还没回复,到时会返回一些修改意见,改好后再交一次上去,再等出…… 就是这两等,经常被拖着,很慢……越来越慢
yes94 9# 2013-3-24 22:56
三次函数几乎已经被很多人人重复玩了好几遍了, 不要误会哈, 这句话用什么感*情*色*彩、该怎么说才好呢? 难煞我也。
kuing 10# 2013-3-24 22:57
9# yes94 敏感词尽量用拼音代之,中间加符号不一定能起作用
yes94 11# 2013-3-24 23:25
IC的自编题?还是有出处的题目?还是改编题? IC有改编题目的习惯。
isea 12# 2013-3-24 23:33
本帖最后由 isea 于 2013-3-24 23:35 编辑 三次函数几乎已经被很多人人重复玩了好几遍了, 不要误会哈, 这句话用什么感*情*色*彩、该怎么说才好呢? 难煞我也。 yes94 发表于 2013-3-24 22:56 无所谓了,当不常读书,也不常主动去探究新东西,自己重复发现一下亦有乐趣的 能打出来分享给更多的人,如果能对某个,或者更多,有那么一点点的启迪的话,何乐而不为呢 IC的自编题?还是有出处的题目?还是改编题? IC有改编题目的习惯。 yes94 发表于 2013-3-24 23:25 IC的自编题?还是有出处的题目?还是改编题? IC有改编题目的习惯。 yes94 发表于 2013-3-24 23:25 这里是北京东城区某校零模拟练习题。 PS:小改改顺序,数字之类,不是自编了。
yes94 13# 2013-3-24 23:34
本帖最后由 yes94 于 2013-3-24 23:36 编辑 12# isea 也是哈, 看来我猜的还真准确
yes94 14# 2013-3-25 00:11
有权限下载的话就好了:http://wuxizazhi.cnki.net/Search/ZXSX200710009.html
realnumber 15# 2013-3-26 14:12
选择题方法倒有, $(x-1)^2(x-k)=0$一次项系数为$2k+1=0$,解得$k=-0.5$,即两根为$1,-0.5,a>0$ $-(x-1)^2(x-k)=0$一次项系数为$-2k-1=0$,解得$k=-0.5$,即两根为$1,-0.5,a<0$ 看来与a正负无关,选D
realnumber 16# 2013-3-26 14:15
15# realnumber 似乎也可以改为字母,变为解答题办法,运算难度不知道会怎么样?
isea 17# 2013-3-26 14:26
本帖最后由 isea 于 2013-3-26 14:30 编辑 选择题方法倒有, $(x-1)^2(x-k)=0$一次项系数为$2k+1=0$,解得$k=-0.5$,即两根为$1,-0.5,a>0$ $-(x-1)^2(x-k)=0$一次项系数为$-2k-1=0$,解得$k=-0.5$,即两根为$1,-0.5,a realnumber 发表于 2013-3-26 14:12 我检查了主楼的录入无误,此题的正确选项是 B,realnumber 可能计算有误(未细查,只是感觉,可能)。 $a>0$时,两根和小于零。 ==== 我知道哪儿错了,原函数过$(0,-2)$,而构造的函数不满足此条件
realnumber 18# 2013-3-26 14:31
本帖最后由 realnumber 于 2013-3-26 14:48 编辑 楼上说得对,重新来, $a(x-x_1)^2(x-x_2)=0$,一次项系数为$2ax_1x_2+ax_1^2=0$得$x_1x_2<0$且$x_1=-2x_2$($x_1=0$需另外检验),常数项$-ax_1^2x_2=-2$ 得$ax_1<0$,$x_1+x_2=-x_2$,若$a>0$,则$x_1<0,x_2>0,x_1+x_2<0$,若$a<0$,则$x_1>0,x_2<0,x_1+x_2>0$ 这样就是B了. 又,重新考虑系数问题,3次和2次系数应该没什么大用,而1次和常数已知的量,导致仅用这2个解题,应该很正常.
yes94 19# 2013-3-26 17:53
楼上的是三次函数配方法,和kuing在数学空间里的方法基本一致。 以前证明当$a>0$时,$a^3+2\geqslant3a$,有至少两种方法,其中一种:\[a^3+1+1\geqslant3a\] 另一种是配方法:\[a^3-3a+2=(a-1)^2(a+2)\geqslant0\]
thread-1274-1-3.html: [函数] 三次函数仅有两个不同的零点
isea 1# 2013-3-24 22:28
题:已知函数$f(x)=ax^3+bx^2-2(a\ne0)$有且仅有两个不同的零点$x_1,x_2,$则       A. 当$a<0$时,$x_1+x_2<0,x_1x_2>0$        B. 当$a<0$时,$x_1+x_2>0,x_1x_2<0$       C. 当$a>0$时,$x_1+x_2<0,x_1x_2>0$        D. 当$a>0$时,$x_1+x_2>0,x_1x_2<0$ 偶动用韦达定理,硬生生将两根求出来了,超bào力,不知有没轻便之法,多谢。
yes94 2# 2013-3-24 22:32
1# isea 极大、或极小值为0吧?
kuing 3# 2013-3-24 22:33
跟去年山东理数12差不多……我也是把根求出来,其实也不麻烦,见《数学空间》总第9期P22
isea 4# 2013-3-24 22:38
本帖最后由 isea 于 2013-3-24 22:56 编辑 1# isea 极大、或极小值为0吧? yes94 发表于 2013-3-24 22:32 极值肯定是一个方向,多数人的第一方向,哈哈。 3# kuing 好的,我看看去。 网刊最近到第几期了? ==== 看了,这个思路,跟偶一样,呵呵,寻找图象法,乘了个 $\dfrac{1}{x}$ 变成反比例与二次函数,变成你的说的题的原型了,哈哈……
kuing 5# 2013-3-24 22:41
4# isea 准备出12
kuing 6# 2013-3-24 22:46
4# isea 图象法我在评注里有提了一下
isea 7# 2013-3-24 22:51
本帖最后由 isea 于 2013-3-24 22:53 编辑 4# isea 准备出12 kuing 发表于 2013-3-24 22:41 期待一下! 4# isea 图象法我在评注里有提了一下 kuing 发表于 2013-3-24 22:46 等有空的某天,我从极值方向写个详细的过程,三次函数还是值得深入玩味的
kuing 8# 2013-3-24 22:56
7# isea 我估计大概还要等半个月以上才能出12。 其实初稿上星期已经交上了去,现在还没回复,到时会返回一些修改意见,改好后再交一次上去,再等出…… 就是这两等,经常被拖着,很慢……越来越慢
yes94 9# 2013-3-24 22:56
三次函数几乎已经被很多人人重复玩了好几遍了, 不要误会哈, 这句话用什么感*情*色*彩、该怎么说才好呢? 难煞我也。
kuing 10# 2013-3-24 22:57
9# yes94 敏感词尽量用拼音代之,中间加符号不一定能起作用
yes94 11# 2013-3-24 23:25
IC的自编题?还是有出处的题目?还是改编题? IC有改编题目的习惯。
isea 12# 2013-3-24 23:33
本帖最后由 isea 于 2013-3-24 23:35 编辑 三次函数几乎已经被很多人人重复玩了好几遍了, 不要误会哈, 这句话用什么感*情*色*彩、该怎么说才好呢? 难煞我也。 yes94 发表于 2013-3-24 22:56 无所谓了,当不常读书,也不常主动去探究新东西,自己重复发现一下亦有乐趣的 能打出来分享给更多的人,如果能对某个,或者更多,有那么一点点的启迪的话,何乐而不为呢 IC的自编题?还是有出处的题目?还是改编题? IC有改编题目的习惯。 yes94 发表于 2013-3-24 23:25 IC的自编题?还是有出处的题目?还是改编题? IC有改编题目的习惯。 yes94 发表于 2013-3-24 23:25 这里是北京东城区某校零模拟练习题。 PS:小改改顺序,数字之类,不是自编了。
yes94 13# 2013-3-24 23:34
本帖最后由 yes94 于 2013-3-24 23:36 编辑 12# isea 也是哈, 看来我猜的还真准确
yes94 14# 2013-3-25 00:11
有权限下载的话就好了:http://wuxizazhi.cnki.net/Search/ZXSX200710009.html
realnumber 15# 2013-3-26 14:12
选择题方法倒有, $(x-1)^2(x-k)=0$一次项系数为$2k+1=0$,解得$k=-0.5$,即两根为$1,-0.5,a>0$ $-(x-1)^2(x-k)=0$一次项系数为$-2k-1=0$,解得$k=-0.5$,即两根为$1,-0.5,a<0$ 看来与a正负无关,选D
realnumber 16# 2013-3-26 14:15
15# realnumber 似乎也可以改为字母,变为解答题办法,运算难度不知道会怎么样?
isea 17# 2013-3-26 14:26
本帖最后由 isea 于 2013-3-26 14:30 编辑 选择题方法倒有, $(x-1)^2(x-k)=0$一次项系数为$2k+1=0$,解得$k=-0.5$,即两根为$1,-0.5,a>0$ $-(x-1)^2(x-k)=0$一次项系数为$-2k-1=0$,解得$k=-0.5$,即两根为$1,-0.5,a realnumber 发表于 2013-3-26 14:12 我检查了主楼的录入无误,此题的正确选项是 B,realnumber 可能计算有误(未细查,只是感觉,可能)。 $a>0$时,两根和小于零。 ==== 我知道哪儿错了,原函数过$(0,-2)$,而构造的函数不满足此条件
realnumber 18# 2013-3-26 14:31
本帖最后由 realnumber 于 2013-3-26 14:48 编辑 楼上说得对,重新来, $a(x-x_1)^2(x-x_2)=0$,一次项系数为$2ax_1x_2+ax_1^2=0$得$x_1x_2<0$且$x_1=-2x_2$($x_1=0$需另外检验),常数项$-ax_1^2x_2=-2$ 得$ax_1<0$,$x_1+x_2=-x_2$,若$a>0$,则$x_1<0,x_2>0,x_1+x_2<0$,若$a<0$,则$x_1>0,x_2<0,x_1+x_2>0$ 这样就是B了. 又,重新考虑系数问题,3次和2次系数应该没什么大用,而1次和常数已知的量,导致仅用这2个解题,应该很正常.
yes94 19# 2013-3-26 17:53
楼上的是三次函数配方法,和kuing在数学空间里的方法基本一致。 以前证明当$a>0$时,$a^3+2\geqslant3a$,有至少两种方法,其中一种:\[a^3+1+1\geqslant3a\] 另一种是配方法:\[a^3-3a+2=(a-1)^2(a+2)\geqslant0\]
thread-1275-1-1.html: 来,谁看见修改一下,达到这个效果
isea 1# 2013-3-24 22:32
本帖最后由 isea 于 2013-3-24 22:35 编辑 已知函数$f(x)=ax^3+bx^2-2(a\ne0)$有且仅有两个不同的零点$x_1,x_2$,则 A.当$a<0$时,$x_1+x_2<0,x_1x_2>0$        B.当$a<0$时,$x_1+x_2>0,x_1x_2<0$ C.当$a>0$时,$x_1+x_2<0,x_1x_2>0$      D.当$a>0$时,$x_1+x_2>0,x_1x_2<0$ 复制代码 要的效果:http://kkkkuingggg.5d6d.net/thread-1274-1-1.html ctex里直接是两行…… ========== 算了,不会帖tex代码,自己查查简单的入门排版看看去了。
kuing 2# 2013-3-24 22:38
latex 里空一行(或以上)才算换行(正确说是另起一段)。 于是简单点的处理,你就都多空一行,横向选项之间可以用 \hspace{...} 等来产生距离,但是这样很不方便。 于是高级的做法是专门对选择题选项的排版自定义一个命令,这里先不讲
isea 3# 2013-3-24 22:47
哎呀,空两行(或以上),好使,好使, 这已经够用了,虽其道远远~ PS:难怪,在\begin{}这样的东东下空一行,或几行就编译不了的~
kuing 4# 2013-3-24 22:51
看教程或看别人的代码,就能理解了…… 必须注意的是这里跟 latex 是有区别的,草稿本也是,所以不能随便将这里或草稿本上测试效果用于 latex
isea 5# 2013-3-24 22:55
4# kuing 嗯,实话说,在这里入门latex代码入门是非常棒的一个平台 感谢k带我们一个全新的平台,真好~
kuing 6# 2013-3-24 23:06
5# isea 一般吧,充其量也是熟悉了数学公式代码,但是非数学公式部分其实更难……
isea 7# 2013-3-24 23:09
6# kuing :lol 对我,基本差不多了 我最迷恋是打印出来的数学公式美(特别是字体),其实文档都没这种效果,而且还是非常适合边思考边写
thread-1276-1-1.html: [标题党]能看到这篇文章,这辈子值了。
kuing 1# 2013-3-24 23:04

isea 2# 2013-3-24 23:16
微博上的,流行这种无厘头的东东,我都怀疑这真是学生写的,哈哈
kuing 3# 2013-3-24 23:18
笑笑就好……
thread-1277-1-1.html: 猫和兔
kuing 1# 2013-3-24 23:09
thread-1278-1-1.html: 今天是很难突破七八十帖了
isea 1# 2013-3-24 23:39
晚了晚了,累了累了,睡了睡了。 晚安晚安,回见回见。
kuing 2# 2013-3-25 01:22
上个月好像超过过。 无论如何,感谢大家的支持。
isea 3# 2013-3-25 13:57
找朋友借了一本 LATEX2ε科技排版指南 原来 latex 为是数学 而生的 难怪 那么好看
kuing 4# 2013-3-25 18:26
3# isea 邓、彭、陈 那本?十多年前的了哇,估计会有点 out……
kuing 5# 2013-3-25 20:39
3# isea 网上能找到电子书的较新的中文 latex 教程大概就是这个了: http://ishare.iask.sina.com.cn/f/35749700.html
isea 6# 2013-3-25 22:45
我觉得,这些东东,最精彩,最底层的东西,特别是入门,看那本书都一样。 特别我只是针对数学试卷排版,哈哈,当手册用
kuing 7# 2013-3-25 22:56
6# isea 既然差不多,何不看较新的…… 我看了下那本的前言……那里后面说中文系统还用CCT……这现在应该没什么人用了……
kuing 8# 2013-3-26 00:01
三分钟前截的图 还是没超100……
isea 9# 2013-3-26 00:23
8# kuing 早说,我多发几帖就过了 不过,公式测试区反而来审核啊
kuing 10# 2013-3-26 00:42
9# isea 噢?我去看看什么情况,可能不小心碰到mingan词了
kuing 11# 2013-3-26 00:45
10# kuing 我通过了,没发现什么……奇怪ing
isea 12# 2013-3-26 00:48
好奇怪,看,编辑都不成,测试区, 对不起,管理员设置了需要对发帖进行审核,您无权编辑已通过审核的帖子,请返回。 [ 点击这里返回上一页 ] 不知道是不是英文单词有 每攵感词 啊
kuing 13# 2013-3-26 00:59
12# isea 已经知道,原来那个mingan词库里有“我gan”……我去掉了
thread-1279-1-3.html: [不等式] 解题群看到一个
realnumber 1# 2013-3-25 15:24
作换元$y_i=\log_3x_i,i=1,2,3,4,5$. 问题就为$y_1,y_2,y_3,y_4,y_5\ge0,y_1+y_2+y_3+y_4+y_5=6$,求max{$y_1+y_2,y_2+y_3,y_3+y_4,y_4+y_5$}的最小值. 似乎记得讨论过类似问题.不幸又不会了.
kuing 2# 2013-3-25 15:28
像这样:http://bbs.pep.com.cn/thread-515623-1-1.html PS、\max\{...\}
realnumber 3# 2013-3-25 15:28
直观想想,要使得整体最小,如果$y_1+y_2$比其他大,那么就减小这个量,把减小部分分到其他量上,直至max{}内的4个量值都一样大. 怎么凑个证明呢?
realnumber 4# 2013-3-25 19:26
2楼的解法惊艳, 继续3楼的思路,假设取最小时候,$\max\{\}$内四个变量不等, 若$y_1+y_2>m=\max\{y_2+y_3,y_3+y_4,y_4+y_5\}$          当$y_2>0$时,(略微减小$y_2$的值.)令$y'_2=\min\{\frac{y_2}{2},\frac{y_1+y_2-m}{2}\},记t=y_2-y'_2,y'_i=y_i+0.25t,i=2,3,4,5$,此时,$\max\{y_1+y_2,y_2+y_3,y_3+y_4,y_4+y_5\}>\max\{y'_1+y'_2,y'_2+y'_3,y'_3+y'_4,y'_4+y'_5\}$,这与假设矛盾.          当$y_2=0$时,(略微减小$y_1$的值.) 貌似也可行,但讨论的情况,即使去掉利用对称,还是很多.
realnumber 5# 2013-3-25 19:28
问题如果改为,就为$y_1,y_2,y_3,y_4,y_5\ge0,y_1+y_2+y_3+y_4+y_5=6$,求$\max\{3y_1+y_2,2y_2+4y_3,y_3+3y_4,y_4+5y_5\}$的最小值. 如何?数据胡乱写的,可以按需要修改.
yes94 6# 2013-3-25 21:24
楼主给个原题答案啊?
kuing 7# 2013-3-25 21:52
6# yes94 原题照2#链接的方法照葫芦画瓢…… 系数推广未研究过……
yes94 8# 2013-3-25 21:54
7# kuing 我的意思是,我想给个另解(和你的有些不同),但怕答案错了出丑,现在做题随心而欲,难以静下心来了
kuing 9# 2013-3-25 21:56
8# yes94 ……照写就是了……
isea 10# 2013-3-25 23:00
本帖最后由 isea 于 2013-3-25 23:09 编辑 顶楼这个转化真心赞,这都能想到,2楼链接k的解法,因曾经在这里问过类似的东东,转化为不等式,一点都不奇怪呢 如果不转化,就用原题,是否能从积的不等式出发?随口说说 我按2楼的方法,试试,看看结果,先 $9\cdot1\cdot9\cdot1\cdot9=729,\max\{9,9,9,9,9\}$
kuing 11# 2013-3-25 23:03
顶楼这个转化真心赞,这都能想到,2楼链接k的解法,因曾经在这里问过类似的东东,转化为不等式,一点都不奇怪呢 如果不转化,就用原题,是否能从积的不等式出发?随口说说 我按2楼的方法,试试,看看结果,先 isea 发表于 2013-3-25 23:00 会作那个转化,明显是因为楼主见过和的情形,所以我觉得那很正常。 那个换元不改变本质,所以我可以肯定地说:不换元一定可以(也就是你说的“从积的不等式出发”一定可以),将我链接里那些不等式里全改成积即可,不用试也知道
pengcheng1130 12# 2013-3-25 23:06
实际上也无需换元,一头一尾添个$M$大于等于$x_{1}$,M大于等于$x_{5}$不等式相乘.
pengcheng1130 13# 2013-3-25 23:07
答案因该是9!
isea 14# 2013-3-25 23:08
会作那个转化,明显是因为楼主见过和的情形,所以我觉得那很正常。 那个换元不改变本质,所以我可以肯定地说:不换元一定可以(也就是你说的“从积的不等式出发”一定可以),将我链接里那些不等式里全改成积即 ... kuing 发表于 2013-3-25 23:03 明白了
yes94 15# 2013-3-25 23:13
kuing在这里的解法抄下来(搞成代码):http://bbs.pep.com.cn/thread-515623-1-1.html 非负实数$a,b,c,d,e,f,g$满足$a+b+c+d+e+f+g=1$, 求$\min\{\max\{a+b+c, b+c+d, c+d+e, d+e+f, e+f+g\}\}$. 则记 $K=\max\{a+b+c, b+c+d, c+d+e, d+e+f, e+f+g\}$, 则有 $K\geqslant a$, $K\geqslant a+b$, $K\geqslant a+b+c$, $K\geqslant b+c+d$, $K>=c+d+e$, $K\geqslant d+e+f,$ $K\geqslant e+f+g,$ $K\geqslant f+g,$ $K>=g,$ 将它们全部相加, 得到 $9K\geqslant 3,$ 即 $K\geqslant \dfrac13,$ 而当 $a=d=g=\dfrac13, b=c=e=f=0 $时$K=\dfrac13$, 故$K$的最小值为$\dfrac13$, 即 $\min\{\max\{a+b+c, b+c+d, c+d+e, d+e+f, e+f+g\}\}=\dfrac13$.
yes94 16# 2013-3-25 23:19
15# yes94 kuing用了$9$个不等式相加,肯定把很多人都吓住了,下面只用$3$个不等式就可以了。 记 $K=\max\{a+b+c, b+c+d, c+d+e, d+e+f, e+f+g\}$, 则有 $K\geqslant a+b+c$, $K\geqslant d+e+f,$ $K\geqslant e+f+g\geqslant g,$   将以上三式相加, 得到 $3K\geqslant a+b+c+d+e+f+g=1$ 即 $K\geqslant \dfrac13,$ 而当 $a=d=g=\dfrac13, b=c=e=f=0 $时,$K=\dfrac13$, 故$K$的最小值为$\dfrac13$, 即 $\min\{\max\{a+b+c, b+c+d, c+d+e, d+e+f, e+f+g\}\}=\dfrac13$.
kuing 17# 2013-3-25 23:19
15# yes94 还是有两个 >= 没改过来…… 其实这个情形应该用替换功能嘛……
kuing 18# 2013-3-25 23:23
16# yes94 嗯,很好…… 其实我可不可以说用9个不等式还是更有型
yes94 19# 2013-3-25 23:31
答案因该是9! pengcheng1130 发表于 2013-3-25 23:07 彭老师给出了答案,我就敢放心做啦! 主要是现在做题常常静不下心来了,有时也出错。 设$M=\max\{x_1x_2,x_2x_3,x_3x_4,x_4x_5\}$,则 $M\geqslant x_1x_2$, $M\geqslant x_3x_4$, $M\geqslant x_4x_5\geqslant x_5 $, 以上三式相乘得, $M^3\geqslant x_1x_2x_3x_4x_5=729$, 于是,$M\geqslant9$ 当$x_1=9,x_2=1,x_3=9,x_4=1,x_5=9$可取等号。 的确kuing的9个不等式很大胆!谁有信心搞出9个不等式?只有kuing!独此一家!
kuing 20# 2013-3-25 23:33
... 的确kuing的9个不等式很大胆!谁有信心搞出9个不等式?只有kuing!独此一家! yes94 发表于 2013-3-25 23:31 你又夸张了
thread-1279-2-3.html:
isea 21# 2013-3-25 23:48
本帖最后由 isea 于 2013-3-25 23:49 编辑 19# yes94 大同小异,这只是反思后,优化的书写过程 你又夸张了 kuing 发表于 2013-3-25 23:33 k 的解法绝对独树一帜
kuing 22# 2013-3-25 23:50
其实也没什么特别,也不是大胆,而是那样很对称很好看……
yes94 23# 2013-3-25 23:52
21# isea
李斌斌755 24# 2013-3-27 23:33
这属于什么知识点啊!只能照2#的方法,很难想到 ,16#与2#一样
yes94 25# 2013-3-27 23:39
24# 李斌斌755 这叫加权平均,在16楼里,只是有些权重为0而已。
李斌斌755 26# 2013-3-28 00:53
25# yes94 谢谢yes94
realnumber 27# 2013-3-31 08:59
本帖最后由 realnumber 于 2013-3-31 09:04 编辑 5# realnumber 其实等价于这个问题 $k_1x+k_2y+k_3z=1$,求$\max\{x,y,z\}$的最小值,其中$x,y,z,k_1,k_2,k_3\in R^+,k_1,k_2,k_3为常数$. 如此问题并不复杂. 取等条件看来是$x=y=z$,可以用3楼的思路,并可以推广到n元. 模仿2楼的做法是 $K=\max\{x,y,z\}$,那么$k\ge x,k\ge y,k\ge z$,有$(k_1+k_2+k_3)k\ge k_1x+k_2y+k_3z=1$ 即$k\ge\frac{1}{k_1+k_2+k_3}$,----推广到n元也很简单.(也可以把1,2楼题目利用换元转化到这个形式.)
yes94 28# 2013-3-31 13:50
27# realnumber 也是很早以前说过的加权平均。 例如此贴9楼,http://kkkkuingggg.5d6d.net/thread-1093-1-7.html 是不是又是优化了的解题过程?
realnumber 29# 2013-3-31 16:07
28# yes94 是啊,就是利用换元,变个形式而已.
thread-128-1-2.html: 无无聊聊戆戆鸠鸠,空当接龙暂解烦忧,准备11连胜
kuing 1# 2011-10-21 23:58

kuing 2# 2011-10-22 00:06
像一楼这种情形其实并不常见,因为很多时候解到小点数的基本出来之后一下子就结束了,自动化的。这次到这里停下来是因为还有个 J 在 Q 的下面,将那 Q 拿开就直接赢了。 很应景: 真多1哟
①②③④⑤⑥⑦ 3# 2011-10-25 10:12
XP版的,嗯嗯,这个应该能用选局撤消法解11982局的 :)
kuing 4# 2011-10-25 10:51
3# ①②③④⑤⑥⑦ 选局撤消法解11982局? 没听过,啥意思。。。。
①②③④⑤⑥⑦ 5# 2011-10-25 11:38
11982局是无解的(正常玩法) 不过可以利用程序的一个bug,就是在移动过牌之后,放弃该局重玩或者重新选局后,“撤消”功能仍然可用,安前一次游戏的最后一次移动方式来撤消,利用此bug,可以找一个合适的局,从左数第三列到最右列移动一张牌后,重新选局11982,然后直接“撤消”,这时候红心5到黑桃2的下面,从这里开始,此局易解。(就在11982局,经过很少的操作,就能做到第三列向最后一列移动一张牌,此时重玩,然后撤消也行)。最早见到的说法是,就在11982局,开始后将红心2移动到黑桃3下,直接重玩撤消,黑桃3到了红心2的下面,说从这里开始此局可解,不过还是有难度,看了下确实有,没心思花时间去验证这个说法 此bug在Win7的空档接龙中不复存在。
kuing 6# 2011-10-25 11:41
原来还有无解的局,楼上真是见多识广哟,我只顾玩,完全不知道这些哩
①②③④⑤⑥⑦ 7# 2011-10-25 12:05
11982是自Win98时代就存在的无解局,也是1~32000局中的唯一无解局 XP的空当接龙局数增加,无解局也增加到了8个,另外7个是#146692、#186216、#455889、#495505、#512118、#517776、#781948 此外,隐藏局-1和-2也是无解的,98和XP的空当接龙只有这两个隐藏局,Win7的还有-3和-4两个必胜局 另外,98和XP的空当接龙还有“秒杀”法,按Ctrl+Shift+F10,弹出出错框,点“终止”,然后随意一个操作就能过关
kuing 8# 2011-10-25 12:22
7# ①②③④⑤⑥⑦ 这么多东东…………
海盗船长 9# 2012-1-14 19:34
thread-1280-1-1.html: 请教上边带一横的那个“拔”,要怎么重定义呢?
abababa 1# 2013-3-25 19:59
就是$\bar{xyz}$,变成$\overline{xyz}$ 我想把bar就定义成overline,应该怎么做呢? 我试了\renewcommand{\bar}{\overline},结果出错了,对这些不懂,是哪里弄错了?
kuing 2# 2013-3-25 20:16
\renewcommand{\bar}[1]{\overline{#1}}
abababa 3# 2013-3-25 21:26
2# kuing 谢谢,其实这个形式我也试了,现在我发现我的问题了,把这个放在\begin{document}之内就可以,放在之前就不可以,报告说是Command `\bar' already defined,不知道什么原因。 请问能把它放在\begin{document}之前吗?
abababa 4# 2013-3-25 21:29
3# abababa 太奇怪了,刚才试了一个最简单的文档,放在之前之后都可以 我的那个复杂点的文档就只能放在\begin{document}之后。我再找找原因。
kuing 5# 2013-3-25 21:33
正常来说都可以的,我刚才实测,都可以。
kuing 6# 2013-3-25 21:34
用最简单的文档来测试问题——好习惯
abababa 7# 2013-3-25 21:55
5# kuing \documentclass{article} \usepackage{xeCJK} \renewcommand{\bar}[1]{\overline{#1}} \begin{document} $\bar{xyz}$ \end{document} 复制代码 这里是我的代码,版主试试这个行不行?我发现去掉xeCJK那个包就可以,不去掉就不行。
kuing 8# 2013-3-25 21:58
7# abababa 还真是……
abababa 9# 2013-3-25 22:03
8# kuing xeCJK是不是有好多问题啊?现在最好用的是哪个tex呢?听说有latex3,还有其它的luatex什么的,不明白它们间的关系 xeCJK是支持中文的吧,不加也不行。
kuing 10# 2013-3-25 22:04
《数学空间》也用了 xeCJK 的…… LaTeX3 出来了??
kuing 11# 2013-3-25 22:08
话说,我刚才在用 xeCJK 的情况下在 \begin{document} 前重定义其他命令都没什么问题……
abababa 12# 2013-3-25 22:11
10# kuing 听说的,其实是在网上看到的,也不知道出没出,就知道有这么个名字,而且对于我来说,也不知道这些某某tex都有什么区别啊。最初是来到这论坛看了版主的那个测试帖才学了点tex的代码,再后来下载了ctex,还是网友教我用的xelatex,也就平时打几个公式,很多不常见的还用得着的都不会
abababa 13# 2013-3-25 22:14
11# kuing 是的,我原来写过一个\renewcommand{\proofname}{\bf{证明:}},不知道算不算普通的renewcommand,这个就能用 所以碰上这个以后自然就想在那之下定义,结果不行
kuing 14# 2013-3-25 22:19
13# abababa 没什么普通不普通的啊……都是重定义…… 就这个 bar 不知为什么会这样,或者你去 ctex 论坛问下,那边高手多……
thread-1281-1-3.html: [组合] 请教一个奇偶数排成一圈的问题
abababa 1# 2013-3-25 20:03
50个数排成一圈,其中一半是奇数一半是偶数,求证必有一数,它的两边都是偶数。
yes94 2# 2013-3-25 21:15
1# abababa 像初中奥赛题,
abababa 3# 2013-3-25 21:31
2# yes94 可能是吧,具体来源我也不清楚,都是零散的一些复印纸,还有手抄的。 这题没想到怎么做,发来问问。
realnumber 4# 2013-3-26 08:25
没完善,只是想法,从某个偶数a顺时针开始考察, 1.假定a,偶数b在一起,那么ab或ba前后至少要2个奇数才能继续放一个或2个偶数,然后继续2个或以上奇数, 如果一直2偶2奇数,那么与25个奇数偶数矛盾,如果出现3奇数或一偶数,那么奇数比偶数多,也矛盾. 2.假定a前后是奇数,奇数至少都2个以上,这样奇数比偶数多,也矛盾. 就这么完了?有漏洞吗?
yes94 5# 2013-3-26 18:16
另外一题: 从1-100的自然数中,任意取出51个数,证明其中一定有两个数,它们中的一个是另一个的整数倍。 解答: 因为任何一个正整数都能表示成一个奇数乘2的方幂,并且这种表示方法是唯一的,所以我们可把1-100的正整数分成如下50个抽屉(因为1-100 中 gòng有50个奇数): 1){1,1×2,1×22,1×23,1×24,1×25,1×26}; 2){3,3×2,3×22,3×23,3×24,3×25}; 3){5,5×2,5×22,5×23,5×24}; 4){7,7×2,7×22,7×23}; 5){9,9×2,9×22,9×23}; 6){11,11×2,11×22,11×23}; …… 25){49,49×2}; 26){51}; …… 50){99}。 这样,1-100的正整数就无重复,无遗漏地放进这50个抽屉内了。从这100个数中任取51个数,也即从这50个抽屉内任取51个数,根据抽屉原则,其中必定至少有两个数属于同一个抽屉,即属于(1)-(25)号中的某一个抽屉,显然,在这25个抽屉中的任何同一个抽屉内的两个数中,一个是另一个的整数倍。
kuing 6# 2013-3-26 18:52
5# yes94 审核通过,编辑掉mingan词
abababa 7# 2013-3-26 19:22
5# yes94 是有这个题,这个题我弄出来了。请教一楼的问题。realnumber讲的没大看懂。 比如a是奇数,baab也是最后的ab在一起,它前面只有一个a就可以继续放一个偶数b了,不是必须要有两个奇数。可能是我没理解到位?
realnumber 8# 2013-3-26 19:55
7# abababa 2楼的a也是偶数, 2楼用反证法,意思是至少2个奇数才能隔开偶数,偶数连在一起最多2个;这样导致“奇数比偶数多”或“一样多,但奇数偶数都是偶数个”.
abababa 9# 2013-3-26 20:12
8# realnumber 按照反证法和相邻数关系的提示,假设不存在这样的数,目前得到奇数至少有2个相邻,否则这个奇数两边都是偶数;偶数至多有2个相邻,否则3个偶数相邻,中间的一个满足条件。 最终的证明还没想到,猜是能用奇数和偶数个数相等这个条件?
yes94 10# 2013-3-26 21:04
6# kuing 我觉得好奇怪,我还以为kuing要对我用极刑了?
realnumber 11# 2013-3-26 21:11
9# abababa 我觉得已经完成证明了,这样的话,奇数多于偶数;除非都是2个2个,这样导致奇数偶数相等但不是25个奇数 ,25个偶数,而是2k个.
abababa 12# 2013-3-26 21:26
本帖最后由 abababa 于 2013-3-26 21:29 编辑 11# realnumber 谢谢,我想明白了,如果把相邻的奇数整体看成一个奇数区,相邻的偶数整体看成一个偶数区,重点在于排的这一圈里,奇数区和偶数区必然互相分成相同数量的弧段,只有这样才能说反设后奇数的数量必定不小于偶数的数量,然后排除都是两两一段的可能。
转化与化归 13# 2013-3-28 10:04
我写了一个证明,大家看看
abababa 14# 2013-3-29 19:50
13# 转化与化归 谢谢,用公式表达有时候过程更明白。这个题用公式表达的话分类有点多。
yes94 15# 2013-3-29 20:30
本帖最后由 yes94 于 2013-3-29 20:32 编辑 14# abababa 两个网友证明: 1.反证法:设不存在这样的情形 把所有数左右两边的数放在一起,得到100个数,其中50个奇数,50个偶数(即将原来的50个数每个取了两次); 奇数和偶数相等且不存在一个数左右两侧均为偶数,说明不可能存在某数左右两侧均为奇数的情况,不然奇数肯定比偶数多; 此时考虑左右均为一正一负的情况,简单分析可知这种情况不成立; 由以上,推出矛盾 2.
thread-1282-1-3.html: [函数] 解题群里一个函数方程问题
realnumber 1# 2013-3-25 20:11
______kuing edit in $\LaTeX$______ 记 $\mbb Q^+$ 为正有理数全体的集,$f:\mbb Q^+\to\mbb Q^+$ 满足以下条件: (1)对任何 $x\in\mbb Q^+$,有 $f(x)+f(1/x)=1$; (2)对任何 $x\in\mbb Q^+$,有 $f(2x)=2f(f(x))$,求 $f(2012/2013)$ 的值。
yes94 2# 2013-3-25 21:15
像田开斌的word字迹
第一章 3# 2013-3-25 21:20
这都看出来了?上次那个内外圆环滚珠的?
yes94 4# 2013-3-25 23:49
3# 第一章 果然是他,他给出了解答,相当的长,就不转载了。 本题是1991爱尔兰MO改编,估计是将年份改成2012,2013了 答案是:$f(\dfrac nm)=\dfrac n{m+n}$,故$f(\dfrac {2012}{2013})=\dfrac {2012}{4025}$。
realnumber 5# 2013-3-26 08:26
4# yes94 恩,后来群里说田和严文兰都解答过,估计在他们博客上有.
thread-1283-1-1.html: 南通泰州扬州填空最后一题
转化与化归 1# 2013-3-25 20:37
南通泰州扬州填空最后一题
kuing 2# 2013-3-25 20:41
改编自:http://kkkkuingggg.5d6d.net/thread-930-1-1.html
yes94 3# 2013-3-25 21:13
均改编自:1997MO
kuing 4# 2013-3-25 21:21
出处dǎng牛比
转化与化归 5# 2013-3-25 21:34
计算量不一样啊!
kuing 6# 2013-3-25 21:35
我可没说一样……显然这个还要计算具体,链接里只是说明存在性……
转化与化归 7# 2013-3-25 23:08
6# kuing
kuing 8# 2013-3-25 23:58
7# 转化与化归 结果那么简单,说不定有简单算法……
yes94 9# 2013-3-26 21:25
正想没事做一下,却发现有解答了,那就传上来吧
yes94 10# 2013-3-26 21:27
9# yes94
转化与化归 11# 2013-3-26 22:40
这些解法,没有一个让人满意的!
hongxian 12# 2013-3-27 15:03
11# 转化与化归 要求有点高了,不知楼主有没有好解答!
转化与化归 13# 2013-3-27 15:08
12# hongxian 特别简单的也没有
realnumber 14# 2013-3-27 16:19
本帖最后由 realnumber 于 2013-3-27 16:20 编辑 也想不出好办法,如果是我,就硬算$(a,\frac{1}{a}),(b,\frac{1}{b})$设好后,三边相等列方程组.就看换元法等解方程技巧有没用上了.
isea 15# 2013-3-27 19:00
不愧是IMO试题,真是麻烦
kuing 16# 2013-3-27 19:12
15# isea 不是IMO
isea 17# 2013-3-27 19:17
15# isea 不是IMO kuing 发表于 2013-3-27 19:12 总之,现在的过程就是都麻烦 刚闪了一下,如果补成菱形,从对角形为菱形出发会不会有出路。 也看了那个1997原题,还以为标准会用向量数量积,结果用的斜率
李斌斌755 18# 2013-3-27 23:36
楼主请提供简单方法
沉香陈香 19# 2013-5-29 17:21
提供一种解法
kuing 20# 2013-5-29 17:31
19# 沉香陈香 挺不错喔!
thread-1283-2-1.html:
isea 21# 2013-5-29 20:25
要是19楼会打LaTeX代码就更美了
thread-1284-1-3.html: [不等式] 江苏模考填空压轴题
pengcheng1130 1# 2013-3-25 21:35
请指教
yes94 2# 2013-3-25 21:39
1# pengcheng1130 撞车? http://kkkkuingggg.5d6d.net/thread-1279-1-1.html 不过彭老师很好,给出了题的出处,赞一个!
kuing 3# 2013-3-25 21:51
同一天不同人发同一题,多数就是刚考的而且有一定难度的题……
pengcheng1130 4# 2013-3-25 22:17
此题是今天下午才考的“2013届苏、锡、常、镇、徐、连六市高三第二次模拟考试”题目!
isea 5# 2013-3-25 23:14
k 太懒了,这个应该与另一个合并
kuing 6# 2013-3-25 23:18
5# isea 是的,我很懒…… 不过如果现在合并可能楼层会乱,还是先这样吧……
realnumber 7# 2013-3-26 10:31
现在是联考时间,.....
yes94 8# 2013-3-26 21:08
7# realnumber 联考完毕没有?
thread-1285-1-1.html: M$党,来,公式终极编辑器Aurora——与Mathtype同类 但不同
isea 1# 2013-3-26 02:34
本帖最后由 isea 于 2013-3-27 23:28 编辑 http://kkkkuingggg.5d6d.net/thread-529-1-1.html 20楼——Aurora — 一个在 MSOffice 内输入 LaTeX 公式的很好用插件 包内含破解程序,将Keygen字样的程序复制到其安装目录运行即可。 PS:可能会有部分杀软会报,放心,误报。偶用的MSE未报。
isea 2# 2013-3-26 02:39
本帖最后由 isea 于 2013-3-26 02:47 编辑 哦,Aurora官网:http://elevatorlady.ca/index.html 不过,你还是得先学学LaTeX,至少是其灵魂的数学公式!因为最后的功臣还是LaTeX 如官网给的示例图(其官网的那个小图能点出更多): 03吧?物理? 07吧?化学? 代码转换 这是数学吗?程序算法?
kuing 3# 2013-3-26 02:56
不用装CTeX?
isea 4# 2013-3-26 10:33
3# kuing 内 Micro-MiKTeX 安装程序 所以不必安排CTeX 但如果已经安装CTeX之类,无需要安装 Micro-MiKTeX ,因为有CTeX
kuing 5# 2013-3-26 11:39
那干脆装CTEX
isea 6# 2013-3-26 11:45
对 实际上,可以将Aurora看作是将.tex“显示”在M$里,或者说,Aurora 就相当于这里的草稿本
hnsredfox_007 7# 2013-3-27 14:11
无法安装CTEX2.9,但又想使用这个的中文,怎么办啊
kuing 8# 2013-3-27 14:21
7# hnsredfox_007 表示无能为力……要么换个别的版本再试试……甚至换系统……不过好像夸张了点……
isea 9# 2013-3-27 14:33
本帖最后由 isea 于 2013-3-27 22:18 编辑 如果CTeX都无法正确安装,有一种可能是系统太新, 另一种可能是系统太烂,第三种可能不是 非windows平台,而下载的相应的版本不对(这个可能性很少,因为如果用linux系统,不可能连这个都不知道的)
isea 10# 2013-3-27 19:37
还有一种可能,下载时文件损坏: v2.9.2.164 (203M) 包含 Basic 版 MiKTeX,占用空间小,会根据需要的宏包自动升级。MD5:615C7E9EFC020F5CE81461726D7AC1BE v2.9.2.164 Full (1.31G) 包含完整版 MiKTeX。MD5:D9ED7D4A24861A9A432BB9B5EDD8D9D1
叶剑飞Victor 11# 2013-4-1 00:04
还是直接用 LaTeX 编辑全文舒服些。
yes94 12# 2013-4-2 19:13
已经下载两周了,效果还不错,不知道换一台没安装该软件的电脑能否识别?试了一下好像能行,只是不能编辑,已经变成图片了 就不用latex的那个什么抬头啦,很多前缀,够烦人的。
isea 13# 2013-4-2 23:13
12# yes94 mathtype 也一样,如果电脑里没安装mathtype,也一样转换成图片。 所以,要打印效果最好,其实默认已经够了,将渲染模式改成改成矢量形式,即 vector
dualliot 14# 2013-4-6 14:35
aurora很不错.多年前还用过word2tex,那转化的代码可读性不好.
yes94 15# 2013-4-6 20:07
12# yes94 mathtype 也一样,如果电脑里没安装mathtype,也一样转换成图片。 所以,要打印效果最好,其实默认已经够了,将渲染模式改成改成矢量形式,即 vector isea 发表于 2013-4-2 23:13 可以把用mathtype编辑的公式代码复制到Aurora里编辑后,变成word里的图片,即便打印店的电脑里没安装Aurora,打印效果也会很好?
isea 16# 2013-4-6 22:19
15# yes94 放心,无差的,打印成试卷。想想立体几何是图片格式的图形…… 这个有点像同一个文档,一个用doc格式,一个用pdf打印……
isea 17# 2013-4-9 23:12
这玩间是收费的,这个不爽,其次,好像Aurora的字体还和 LaTeX 数学字体并不是完全一样 打印出来,中文是楷体时,数学字体与汉字似乎更配一些
kuing 18# 2013-4-9 23:20
17# isea 一看就是五笔dang……
isea 19# 2013-4-9 23:54
晕,怎么打成一个一个字了,难道不是词组吗?哈哈
yes94 20# 2013-4-17 23:39
Aurora怎么不能输入中文啊? 好烦!
thread-1285-2-1.html:
kuing 21# 2013-4-17 23:42
20# yes94 肯写代码的还是玩latex吧
thread-1286-1-1.html: 测试一下签名
isea 1# 2013-3-26 14:42
看有没有
kuing 2# 2013-3-26 14:44
新贴有了,旧贴没有……
isea 3# 2013-3-26 14:57
哈哈 成功
thread-1287-1-1.html: 推荐个电影你们
isea 1# 2013-3-26 15:24
本帖最后由 isea 于 2013-3-26 15:25 编辑 被解放的姜戈 【Django.Unchained.2012.720p.BluRay.x264-SPARKS.mkv】【6.6G】 #QQ旋风离线分享# http://urlxf.qq.com/?JniIRjNj #迅雷快传分享# http://kuai.xunlei.com/d/YtvCBgLH9QC0HVFRf4c 简单介绍一下:  本片是一部意大利式的通心粉西部片。从片名看,本片很明显是在向意大利西部片的经典制作《姜戈》致敬。故事中,主人公姜戈是一名获得了自由的奴隶,他在一名德国裔的赏金猎人的指导下成为一名彪悍的、逍遥自在的赏金猎人。他协助那个德国赏金猎人缉拿各种通缉犯以换取报酬,在其帮助下追寻自己的奴隶妻子并试图将她从邪恶的大农场主加尔文·坎迪(Calvin Candie)手里解救出来。 具体:http://movie.douban.com/subject/6307447/
isea 2# 2013-3-26 23:33
小版本 被解放的迪亚戈/铁血枷锁/被解放的姜戈/决杀令(台)/黑杀令(港) 【Django.Unchained.2012.BDRip.XviD-SPARKS.avi】【1.4G】 #QQ旋风离线# http://url.cn/DD4nZu
kuing 3# 2013-3-26 23:35
小心mingan词
thread-1288-1-1.html: 问一个关于引理引用链接的问题
abababa 1# 2013-3-26 19:14
\documentclass[11pt]{book} \usepackage{amsmath} \usepackage[amsmath, thmmarks]{ntheorem} \newtheorem{lemma}{Lemma}[chapter] \usepackage[colorlinks,linkcolor=red,anchorcolor=blue,citecolor=green]{hyperref} \begin{document} \chapter{1} \begin{lemma} Lemma 1 \end{lemma} \chapter{10} \begin{lemma}\label{lemma10} Lemma 10 \end{lemma} Ref the Lemma \ref{lemma10} \end{document} 复制代码 这是我的代码,我按照网上说的,运行了两次xelatex,把那个编号显示出来了,但是点这个编号,却跳到第一页上去了,我定义编号的时候是定义在引理10啊,应该跳到本页才对,怎么回事呢?
kuing 2# 2013-3-26 19:34
还真是…… 但是我刚才将 \usepackage[amsmath, thmmarks]{ntheorem} 换成 \usepackage{amsthm} 就没问题了……
kuing 3# 2013-3-26 19:47
寻找原因对于我来说暂时无能为力……去CTEX论坛问下吧……
thread-1289-1-2.html: [函数] 导数 不等式 求最大整数值
isea 1# 2013-3-27 00:57
本帖最后由 isea 于 2013-3-27 03:26 编辑 题:已知函数$f(x)=(x^3+2x^2+5x+t)e^{-x},t\in \mathbf{R},x\in \mathbf{R} $.     (1)当$t=5 $时,求函数\(y=f(x)\)的单调区间;     (2)若存在实数$t\in [0,1] $,使对任意的$x\in[-4,m] $,不等式$f(x)\leq x $成立,求整数$m $的最大值. 海淀3月部分学校高三联考试题倒数第三题;这题第(2)问有超过北京课标之嫌, 偶对第(2)求得的结果是$0$,但并不能完全肯定。 我觉得试题还是有一定的代表性,若求导,则导数的零点是个难点, 但如2012年广州高三一模也有类似的函数导数不等式综合大题。 故,最终还是发了上来,班门弄斧,抛砖引玉,主要是向大家学习,先谢。 分析与解: (1)增区间$(-\infty,0)$,减区间$(0,+\infty)$; (2)据以往经验,若函数的导函数还要继续求导,往往各自相应的零点或者取最值条件巧同。 而此题不是。 如果我们能在有限的时间里,不借用计算器,大约将函数的图象画出来,数与形结合,将会明朗许多。 而,偶认为求极值,判断与零的大小关系,其本质上就是在作图了。 函数作图,就肯定涉及函数的凹凸性,亦,二次求导,这,超出了高中导数范围, 虽对导函数再求导只是一句话的事,但,如果出现了定义概念之类,那就又带来了许多新的问题。 回到第(2)问,先化简看看再说: \begin{align} f(x)\leq x &\iff (x^3+2x^2+5x+t)e^{-x}\leq x \notag \\ &\iff x^3+2x+5x+t\leq xe^x\notag\\ &\iff t\leq xe^x-x^3-2x^2-5x \end{align} 这里很容易验证,$t\in [0,1]$,当$x=0$(此时有$t=0$)不等式(1)是成立的。 更重要的一方面,这样便将变量与参数分离开了,且不看题中的自变量范围,先。 $$\exists t\in [0,1],\forall x\in \mathbf{R},t\leq xe^x-x^3-2x^2-5x \iff 0\leq xe^x-x^3-2x^2-5x$$ 再结合题设条件$x\in[-4,m] $, 若$x\in[-4,0)$(即$x<0$)则 \begin{align} 0&\leq xe^x-x^3-2x^2-5x \notag \\ \iff 0&\geq e^x-x^2-2x-5 \end{align} 对不等式(2),$x\in[-4,0)$是否成立问题已经很常规了,即讨论函数$F(x)=e^x-x^2-2x-5,x\in[-4,0)$上的最大值与零的大小! 但是这里有个问题,就是导函数的零点无法求出,列表讨论极值出现困难。 不过,一眼看下去,立刻得$x\in[-4,0),F'(x)=e^x-2x-2,(F''(x)=e^x-2<0)$即 此时函数$F'(x)$在$x\in[-4,0)$上单调递减, 故$F'(x)=0$有惟一解, 不防令$F'(x)$零点为$x_0$,(言外之意即是说:$x\in [-4,x_0),F'(x)>0,\cdots$ 即令$F'(x_0)=e^{x_0}-2x_0-2=0 \Rightarrow e^{x_0}=2x_0+2$ 再看$F(x)$,有$F(x)$在$[-4,x_0)$单调递增,在$[x_0,0)$单调递减(有极大值这里即最大值)。 此时可以在稿纸,或者脑袋里画一个$x\in [-4,0),F(x)$草图,再结合要证明的不等式(2), 很自然的计算 $F(x_0)=e^{x_0}-x_0^2-2x_0-5=2x_0+2-x_0^2-2x_0-5=-x_0^2-3<0$,亦不等式(2)成立。 从而$m \geq 0$。 最后,若$x>0$则 \begin{align} 0&\leq xe^x-x^3-2x^2-5x \notag \\ \iff 0&\leq e^x-x^2-2x-5 \end{align} 即讨论函数$F(x)=e^x-x^2-2x-5,x\in(0,+\infty)$上的最小值与零的大小。 讨论方法与$x\in[4,0)$大同小异,不过,这里求整数$m$的最大值,注意到 $F(1)=e-1-2-5<0$,这就意味着$m<1$. 综上,整数$m$的最大值为$0$。
yes94 2# 2013-3-27 11:08
1# isea 思路感觉还不错,
kuing 3# 2013-3-27 11:36
isea 变成夜猫了?
零定义 4# 2013-3-27 15:19
总感觉“整数”这个条件有点多余...
realnumber 5# 2013-3-27 16:13
这样做就不显得多余了,取m=1,令x=1代入可得,$8+t\le e,t\in [0,1]$,显然不成立. 取m=0,以下证明t=0时,恒成立,x=0显然成立,当$-4\le x<0$,即要 证明$x^2+2x+5\ge e^x$ 这个显然成立$x^2+2x+5=(x+1)^2+4\ge4\ge1\ge e^x$
零定义 6# 2013-3-27 16:40
还是多余的吧?只是有了它可以偷偷懒罢了...
yes94 7# 2013-3-27 17:39
6# 零定义 偷偷懒?太形象了!
isea 8# 2013-3-27 18:50
总感觉“整数”这个条件有点多余... 零定义 发表于 2013-3-27 15:19 如果是看了主楼有这种想法的话,不奇怪,因为,主楼我就是从全局考虑的,然后缩小到局部,这样一个思考方式。 而此题如果改成求$m$的范围,结合几何画板之类的软件,观察图象,怕是没结果(或者说没具体初等表达式)。
isea 9# 2013-3-27 19:10
本帖最后由 isea 于 2013-3-27 19:18 编辑 找到来源了,浙江省宁波市十校2012届高三联考数学(理)最后一题,汗。 标答放上来,偶的思想与标答大同小异,标答是从$m$出发的,不知还有是否有他法。
yes94 10# 2013-3-27 21:03
9# isea 所以,2楼说思路感觉还不错,
realnumber 11# 2013-3-28 07:29
9# isea 5楼充分与必要分开做,已经很简洁了.好象没人看到
isea 12# 2013-3-28 11:47
9# isea 5楼充分与必要分开做,已经很简洁了.好象没人看到 realnumber 发表于 2013-3-28 07:29 怎么会呢,第一时间就学习了
yes94 13# 2013-3-28 12:32
9# isea 5楼充分与必要分开做,已经很简洁了.好象没人看到 realnumber 发表于 2013-3-28 07:29 偷偷懒,就意味着……
零定义 14# 2013-3-29 14:44
8# isea 不见得哦~结果依然是0~
yes94 15# 2013-3-29 17:50
9# isea 5楼充分与必要分开做,已经很简洁了.好象没人看到 realnumber 发表于 2013-3-28 07:29 冒险一次成功,就进行第二次,第二次也成功了,……
thread-1289-1-3.html: [函数] 导数 不等式 求最大整数值
isea 1# 2013-3-27 00:57
本帖最后由 isea 于 2013-3-27 03:26 编辑 题:已知函数$f(x)=(x^3+2x^2+5x+t)e^{-x},t\in \mathbf{R},x\in \mathbf{R} $.     (1)当$t=5 $时,求函数\(y=f(x)\)的单调区间;     (2)若存在实数$t\in [0,1] $,使对任意的$x\in[-4,m] $,不等式$f(x)\leq x $成立,求整数$m $的最大值. 海淀3月部分学校高三联考试题倒数第三题;这题第(2)问有超过北京课标之嫌, 偶对第(2)求得的结果是$0$,但并不能完全肯定。 我觉得试题还是有一定的代表性,若求导,则导数的零点是个难点, 但如2012年广州高三一模也有类似的函数导数不等式综合大题。 故,最终还是发了上来,班门弄斧,抛砖引玉,主要是向大家学习,先谢。 分析与解: (1)增区间$(-\infty,0)$,减区间$(0,+\infty)$; (2)据以往经验,若函数的导函数还要继续求导,往往各自相应的零点或者取最值条件巧同。 而此题不是。 如果我们能在有限的时间里,不借用计算器,大约将函数的图象画出来,数与形结合,将会明朗许多。 而,偶认为求极值,判断与零的大小关系,其本质上就是在作图了。 函数作图,就肯定涉及函数的凹凸性,亦,二次求导,这,超出了高中导数范围, 虽对导函数再求导只是一句话的事,但,如果出现了定义概念之类,那就又带来了许多新的问题。 回到第(2)问,先化简看看再说: \begin{align} f(x)\leq x &\iff (x^3+2x^2+5x+t)e^{-x}\leq x \notag \\ &\iff x^3+2x+5x+t\leq xe^x\notag\\ &\iff t\leq xe^x-x^3-2x^2-5x \end{align} 这里很容易验证,$t\in [0,1]$,当$x=0$(此时有$t=0$)不等式(1)是成立的。 更重要的一方面,这样便将变量与参数分离开了,且不看题中的自变量范围,先。 $$\exists t\in [0,1],\forall x\in \mathbf{R},t\leq xe^x-x^3-2x^2-5x \iff 0\leq xe^x-x^3-2x^2-5x$$ 再结合题设条件$x\in[-4,m] $, 若$x\in[-4,0)$(即$x<0$)则 \begin{align} 0&\leq xe^x-x^3-2x^2-5x \notag \\ \iff 0&\geq e^x-x^2-2x-5 \end{align} 对不等式(2),$x\in[-4,0)$是否成立问题已经很常规了,即讨论函数$F(x)=e^x-x^2-2x-5,x\in[-4,0)$上的最大值与零的大小! 但是这里有个问题,就是导函数的零点无法求出,列表讨论极值出现困难。 不过,一眼看下去,立刻得$x\in[-4,0),F'(x)=e^x-2x-2,(F''(x)=e^x-2<0)$即 此时函数$F'(x)$在$x\in[-4,0)$上单调递减, 故$F'(x)=0$有惟一解, 不防令$F'(x)$零点为$x_0$,(言外之意即是说:$x\in [-4,x_0),F'(x)>0,\cdots$ 即令$F'(x_0)=e^{x_0}-2x_0-2=0 \Rightarrow e^{x_0}=2x_0+2$ 再看$F(x)$,有$F(x)$在$[-4,x_0)$单调递增,在$[x_0,0)$单调递减(有极大值这里即最大值)。 此时可以在稿纸,或者脑袋里画一个$x\in [-4,0),F(x)$草图,再结合要证明的不等式(2), 很自然的计算 $F(x_0)=e^{x_0}-x_0^2-2x_0-5=2x_0+2-x_0^2-2x_0-5=-x_0^2-3<0$,亦不等式(2)成立。 从而$m \geq 0$。 最后,若$x>0$则 \begin{align} 0&\leq xe^x-x^3-2x^2-5x \notag \\ \iff 0&\leq e^x-x^2-2x-5 \end{align} 即讨论函数$F(x)=e^x-x^2-2x-5,x\in(0,+\infty)$上的最小值与零的大小。 讨论方法与$x\in[4,0)$大同小异,不过,这里求整数$m$的最大值,注意到 $F(1)=e-1-2-5<0$,这就意味着$m<1$. 综上,整数$m$的最大值为$0$。
yes94 2# 2013-3-27 11:08
1# isea 思路感觉还不错,
kuing 3# 2013-3-27 11:36
isea 变成夜猫了?
零定义 4# 2013-3-27 15:19
总感觉“整数”这个条件有点多余...
realnumber 5# 2013-3-27 16:13
这样做就不显得多余了,取m=1,令x=1代入可得,$8+t\le e,t\in [0,1]$,显然不成立. 取m=0,以下证明t=0时,恒成立,x=0显然成立,当$-4\le x<0$,即要 证明$x^2+2x+5\ge e^x$ 这个显然成立$x^2+2x+5=(x+1)^2+4\ge4\ge1\ge e^x$
零定义 6# 2013-3-27 16:40
还是多余的吧?只是有了它可以偷偷懒罢了...
yes94 7# 2013-3-27 17:39
6# 零定义 偷偷懒?太形象了!
isea 8# 2013-3-27 18:50
总感觉“整数”这个条件有点多余... 零定义 发表于 2013-3-27 15:19 如果是看了主楼有这种想法的话,不奇怪,因为,主楼我就是从全局考虑的,然后缩小到局部,这样一个思考方式。 而此题如果改成求$m$的范围,结合几何画板之类的软件,观察图象,怕是没结果(或者说没具体初等表达式)。
isea 9# 2013-3-27 19:10
本帖最后由 isea 于 2013-3-27 19:18 编辑 找到来源了,浙江省宁波市十校2012届高三联考数学(理)最后一题,汗。 标答放上来,偶的思想与标答大同小异,标答是从$m$出发的,不知还有是否有他法。
yes94 10# 2013-3-27 21:03
9# isea 所以,2楼说思路感觉还不错,
realnumber 11# 2013-3-28 07:29
9# isea 5楼充分与必要分开做,已经很简洁了.好象没人看到
isea 12# 2013-3-28 11:47
9# isea 5楼充分与必要分开做,已经很简洁了.好象没人看到 realnumber 发表于 2013-3-28 07:29 怎么会呢,第一时间就学习了
yes94 13# 2013-3-28 12:32
9# isea 5楼充分与必要分开做,已经很简洁了.好象没人看到 realnumber 发表于 2013-3-28 07:29 偷偷懒,就意味着……
零定义 14# 2013-3-29 14:44
8# isea 不见得哦~结果依然是0~
yes94 15# 2013-3-29 17:50
9# isea 5楼充分与必要分开做,已经很简洁了.好象没人看到 realnumber 发表于 2013-3-28 07:29 冒险一次成功,就进行第二次,第二次也成功了,……
thread-129-1-2.html: 不上Q一样可以看你们在聊啥
kuing 1# 2011-10-22 12:52
最新的 webQQ 不错,登录的时候可以不同时登录QQ,但依然可以看空间、邮箱、Q群等等,就是不能开聊而已。
kuing 2# 2011-10-22 12:58
好多东西 当然,好多对于我来说没用……
kuing 3# 2011-10-22 13:02
大家不妨试试
pxchg1200 4# 2011-10-24 23:14
呵呵,我是一直开隐身。。。 话说那个Google Chrome 浏览器 看起来挺酷的。。
kuing 5# 2011-10-24 23:45
4# pxchg1200 Q我吧党路过 chrome挺爽的
戊概念·五 6# 2011-10-27 21:41
好多东西 55 当然,好多对于我来说没用…… kuing 发表于 2011-10-22 12:58 居然没有JR.....
thread-1291-1-1.html: 突然发现变论坛元老了
realnumber 1# 2013-3-27 16:25
老~~
kuing 2# 2013-3-27 16:37
其实我已经忘记当年那些积分和等级设置是怎么弄的了
isea 3# 2013-3-27 18:40
昨天我就注意到了
yes94 4# 2013-3-27 22:38
建议UID20以前的可以叫元老 ,那样IC也是元老了
isea 5# 2013-3-27 23:29
建议UID20以前的可以叫元老 ,那样IC也是元老了 yes94 发表于 2013-3-27 22:38 咳,这这,还不如直接说你自己得 按你这速度,过不了几天也元老了
isea 6# 2013-3-27 23:30
建议UID20以前的可以叫元老 ,那样IC也是元老了 yes94 发表于 2013-3-27 22:38 晕,原来早是元老了
kuing 7# 2013-3-27 23:32
貌似是按积分划的……不知能不能按时间划,印象中没有那一项……
yes94 8# 2013-4-2 19:17
咳,这这,还不如直接说你自己得 按你这速度,过不了几天也元老了 isea 发表于 2013-3-27 23:29 咳,咳,真的是元老了,IC加油!
isea 9# 2013-4-2 23:18
论坛好像恢复速度了,灌一个,先
kuing 10# 2013-4-2 23:20
9# isea 我发现夜深就不卡了,可能是服务器不太行,支持不住白天的浏览量…… kuing 发表于 2013-4-2 23:15
李斌斌755 11# 2013-5-18 14:59
自己也变元老了,不过是水母元老
yes94 12# 2013-5-23 20:00
11# 李斌斌755 一群元老们在相互吹捧
李斌斌755 13# 2013-5-24 00:00
12# yes94 不能换钱,不犯法。
thread-1292-1-1.html: 被集成在CTeX v2.9.2.164 里的WinEdt自动补齐
isea 1# 2013-3-27 19:32
本帖最后由 isea 于 2013-3-27 20:26 编辑 WinEdt 这个早就知道,小有名气了。 不过,我想 WinEdt 应该有自动补齐,虽然点tool bar上对应命令按钮会自动补齐,但我实在懒得拿鼠标。 现在才知道,原来是来打两个}},如 \begin{gather}},在输入第二}时,就自动变成补齐命令。 再如 \begin{table}},最后效果为——(没那个框,这是论坛MathJax在作怪) \begin{table} * \end{table}
kuing 2# 2013-3-27 19:35
如果你看过这个http://bbs.pep.com.cn/forum.php?mod=viewthread&tid=1731319
isea 3# 2013-3-27 19:38
2# kuing 哈哈 不过,不符合linux命令的习惯,linux都用tab键补齐,打几个字母就可以,怀念啊
isea 4# 2013-3-27 19:43
相对非IT者,这些东东太小众,特别是在人教论坛那边,根本没人看,基本
kuing 5# 2013-3-27 19:46
印象中那个贴最后的回贴里有些地方有错误,不过后来也懒得更新,反正都没人了
isea 6# 2013-3-27 19:47
在tools菜单下,原来是 Ctrl + Enter (补光标当前的东东)
kuing 7# 2013-3-27 20:06
你的 winedt 是什么版本?
isea 8# 2013-3-27 20:19
你的 winedt 是什么版本? kuing 发表于 2013-3-27 20:06 winedt  Build: 20120321  (v. 7.0) CTeX  v2.9.2.164 -- 2012.03.22 集成的
isea 9# 2013-3-27 20:20
原来,选择新写的代码,shift + f9 ,便是编译选择这一部分,只是不显示,需自己去相应目录查看_temp.pdf 效果 == 原来,windows下果然能用dos编译tex文件,latex xx.tex 得相应xx.dvi;pdflatex xx.tex 得相应的xx.pdf文件
kuing 10# 2013-3-27 20:47
8# isea oh,我一直在用 6
thread-1293-1-1.html: [组合] 来自某教师群的一道集合计数
kuing 1# 2013-3-27 23:19
茂名赵老师(3080*****)  21:28:19 请教一题:一个集 合I有1,2,3,4,5五个元素,选择I的两个非空子集A和B,要是B中最小的数大于A中最大的数,则不同的选择方法有多少种? “要是”应该是“要使”。 这里将求解一般情况,记 $f(n)$ 为 $I$ 中有 $n$($n\geqslant 2$)个元素时满足题意的方法数。 显然 $f(2)=1$。当 $n\geqslant 3$ 时, (1)若 $B$ 不包含 $I$ 中的最大元素,则方法数等于 $f(n-1)$; (2)当 $B$ 包含 $I$ 中的最大元素时,     (2-1)若 $B$ 中至少两个元素,则方法数也等于 $f(n-1)$;     (2-2)若 $B$ 中只有这个最大元素,则方法数等于余下 $n-1$ 个元素构成集合的非空子集个数,即 $2^{n-1}-1$。 综上,得到 $f(n)=2f(n-1)+2^{n-1}-1$,于是求通项易得 $f(n)=(n-2)2^{n-1}+1$。 回到原题 $f(5)=49$。
yes94 2# 2013-3-27 23:29
递推法计数搞得炉火纯青啊
kuing 3# 2013-3-27 23:30
2# yes94 又夸张了……这道应该算是比较基础的一类吧……
yes94 4# 2013-3-27 23:35
3# kuing 那道题“$50$个数排成一圈,其中一半是奇数一半是偶数,求证必有一数,它的两边都是偶数。 ”能否递推做? 改成:“$2n$个数排成一圈,其中一半是奇数一半是偶数,求证必有一数,它的两边都是偶数。 ”其中$n$从某个合适的正整数开始。
地狱的死灵 5# 2013-3-27 23:48
A中最大元素为1,A有1种选择,B有2^4-1=15种选择; A中最大元素为2,A有2种选择,B有2^3-1=7种选择; A中最大元素为3,A有2^2=4种选择,B有2^2-1=3种选择; A中最大元素为4,A有2^3=8种选择,B有1种选择; 结果就是1*15+2*7+4*3+8*1=49. 如果改变所给集合的元素个数, 也可以据此算出通项公式
realnumber 6# 2013-3-27 23:52
4# yes94 n是偶数的话结论不成立。反例就是2奇数2偶数依次排成一圈。
isea 7# 2013-3-27 23:52
3# kuing 那道题“$50$个数排成一圈,其中一半是奇数一半是偶数,求证必有一数,它的两边都是偶数。 ”能否递推做? 改成:“$2n$个数排成一圈,其中一半是奇数一半是偶数,求证必有一数,它的两边都是偶数。 ” ... yes94 发表于 2013-3-27 23:35 这个我觉得没法递推,这是明显数论题,感觉和抽屉原则,或者反证法有关。 如果谁去翻一下初中奥林匹克数学竞赛书,应该能得到答案
kuing 8# 2013-3-27 23:57
7# isea 跟数论没啥关系吧……
kuing 9# 2013-3-28 00:03
A中最大元素为1,A有1种选择,B有2^4-1=15种选择; A中最大元素为2,A有2种选择,B有2^3-1=7种选择; A中最大元素为3,A有2^2=4种选择,B有2^2-1=3种选择; A中最大元素为4,A有2^3=8种选择,B有1种选择; 结果就是1*15+2*7+4*3+8*1=49. 如果改变所给集合的元素个数, 也可以据此算出通项公式 地狱的死灵 发表于 2013-3-27 23:48 嗯,这个不错,照你的方法写下。 设 $I=\{1,2,\ldots,n\}$,当 $A$ 最大元素为 $k$ 时,$A$ 有 $2^{k-1}$ 种选择,$B$ 有 $2^{n-k}-1$ 种选择,所以结果为 \[\sum_{k=1}^{n-1}2^{k-1}(2^{n-k}-1)=(n-2)2^{n-1}+1.\]
kuing 10# 2013-5-26 14:37
顺便也把这个链接记上 http://kkkkuingggg.5d6d.net/thread-1596-1-1.html
呆呆 11# 2013-5-30 08:39
本帖最后由 呆呆 于 2013-5-30 09:03 编辑 $\sum_{k = 2}^nC_n^k( k - 1) = \sum_{k = 0}^n C_n^k(k - 1) + 1=(n-2)2^{n-1}+1$
yayaweha 12# 2013-5-30 10:38
1# kuing (2-1)没看懂
realnumber 13# 2013-5-30 13:08
1楼问题穷举也可以,B中最小为1,无 B中最小为2,A={1},符合的B有$2^3$个 B中最小为3....... 似乎也可以推广到n.
kuing 14# 2013-5-30 14:37
$\sum_{k = 2}^nC_n^k( k - 1) = \sum_{k = 0}^n C_n^k(k - 1) + 1=(n-2)2^{n-1}+1$ 呆呆 发表于 2013-5-30 08:39 嗯,10#的链接就是记录了这个方法
thread-1294-1-1.html: 一直在纠结LaTeX 数学模式 下的中文(完全解决于21楼)
isea 1# 2013-3-28 12:09
本帖最后由 isea 于 2013-4-11 21:50 编辑 用CTeX才发现这些东西完全不需要考虑,随便输入中文即可。
kuing 2# 2013-3-28 12:12
我会用 \text{中文}
isea 3# 2013-3-28 12:12
如前几天的导数帖子里的内容,效果不帖了。 大家直接把源代码帖到空白的.tex文件中,自己查看效果。
isea 4# 2013-3-28 12:19
我会用 \text{中文} kuing 发表于 2013-3-28 12:12 我直接给拆开了,各是各的,以后可以用\text{} 不过,这样最麻烦的是中英文的切换,烦得不行 有得有失啊 ==== word + aurora 的好处是不需要切换中英文 虽有所见所得,但不能发挥LaTeX“完整”的功能,如两\begin{}自动公式连号……
isea 5# 2013-3-28 12:24
我会用 \text{中文} kuing 发表于 2013-3-28 12:12 \mbox 好像与 \text 一样,不考虑具体的参数(还不知,有没参数)
kuing 6# 2013-3-28 14:37
一般都是拆开的,不是必要的话我都不会在公式里出现任何全角的东西…… 非要在公式中用中文,也要用 \text{..} 或 \mbox{..}(两者的区别等会再测试),也不能不加任何东西直接打吧?
kuing 7# 2013-3-28 14:49
下载了上面的附件,我不知道你用什么编译的,我用 pdflatex,结果是这样的:
kuing 8# 2013-3-28 15:06
...\text{..} 或 \mbox{..}(两者的区别等会再测试)... kuing 发表于 2013-3-28 14:37 扇形中,$S_{\text{扇形}OAB} \ne S_{\mbox{扇形}OAB}$ 前者用 \text,后者用 \mbox 你也可以在 latex 中测试
isea 9# 2013-3-28 20:41
一般都是拆开的,不是必要的话我都不会在公式里出现任何全角的东西…… 非要在公式中用中文,也要用 \text{..} 或 \mbox{..}(两者的区别等会再测试),也不能不加任何东西直接打吧? kuing 发表于 2013-3-28 14:37 数学模式下必须用代码 \text{} 或者 \mbox{} 我顶楼标题与内容不相符,CTeX套装下非数学模式下直接打。
isea 10# 2013-3-28 20:46
下载了上面的附件,我不知道你用什么编译的,我用 pdflatex,结果是这样的: 1167 1168 1169 1170 kuing 发表于 2013-3-28 14:49 我其实也是一样。 CTeX套装,winedt 7.0,那个逗号是直接从论坛帖里复制了,附件里没改,pdflatex 不支持,所以逗号没了,哈哈 是我的错
kuing 11# 2013-3-28 20:51
估计哪种编译方式都差不多,或者会乱码。 但是这里的mathjax可以,甚至可以对一些全角符号自动变成合适的符号。 其实突然觉得用 xelatex 作一堆设置应该也可以做到那样子,但是似乎没什么必要。
isea 12# 2013-3-28 21:16
估计哪种编译方式都差不多,或者会乱码。 但是这里的mathjax可以,甚至可以对一些全角符号自动变成合适的符号。 其实突然觉得用 xelatex 作一堆设置应该也可以做到那样子,但是似乎没什么必要。 kuing 发表于 2013-3-28 20:51 那是完全没必要的,谁会在纯xyz,abc下输入全角的东东? 那就是切输入法,无意识的情形下
kuing 13# 2013-3-28 21:26
那是完全没必要的,谁会在纯xyz,abc下输入全角的东东? 那就是切输入法,无意识的情形下 isea 发表于 2013-3-28 21:16 那你上面的附件中为什么又会有 [x_0,0) 这样的东东……
isea 14# 2013-3-28 21:47
那你上面的附件中为什么又会有 [x_0,0) 这样的东东…… kuing 发表于 2013-3-28 21:26 不记得了—— 也许,在word里输入数学公式,用全角逗号习惯了。 哈哈
kuing 15# 2013-3-28 22:05
还真让我做到了,在公式中用全角逗号也没问题,还可以作一点距离调整。但是要用 xelatex
isea 16# 2013-3-28 22:12
还真让我做到了,在公式中用全角逗号也没问题,还可以作一点距离调整。但是要用 xelatex kuing 发表于 2013-3-28 22:05 恭喜恭喜
isea 17# 2013-3-28 22:13
还真让我做到了,在公式中用全角逗号也没问题,还可以作一点距离调整。但是要用 xelatex kuing 发表于 2013-3-28 22:05 不过,在英文键盘下输入全角逗号也不容易啊,不过,好处是不会编译出错
kuing 18# 2013-3-28 22:14
呵呵,纯粹无聊试试而已,不打算投入使用,因为其实有点浪费资源,每遇全角逗号都要判断是不是在数学模式里。
yes94 19# 2013-3-30 18:56
10# isea 数学公式终极编辑器:Aurora,基于LaTeX; 与Mathtype最大不同:公式好看,美观;鼠标基本多余 终于用了Aurora,效果还不错!
isea 20# 2013-3-30 20:16
10# isea 数学公式终极编辑器:Aurora,基于LaTeX; 与Mathtype最大不同:公式好看,美观;鼠标基本多余 终于用了Aurora,效果还不错! yes94 发表于 2013-3-30 18:56 同时安装mathtype与aurora ,M$会把快捷键alt+q分配给mathtype。 aurora 是 “alt +u,q”,这个给键距离太大,麻烦,还好,还有第二组ctrl+;; ,即按两次分号。 输入完公式之后,按 ctrl+enter 退出刷新公式;或者 ctrl+r 或者 ctrl+s 刷新(运行,如果有错,还会出现log)不退出aurora。 更多,参见其help
thread-1294-2-1.html:
isea 21# 2013-4-11 21:46
本帖最后由 isea 于 2013-4-11 21:51 编辑 扇形中,$S_{\text{扇形}OAB} \ne S_{\mbox{扇形}OAB}$ 前者用 \text,后者用 \mbox 你也可以在 latex 中测试 kuing 发表于 2013-3-28 15:06 参考自:http://blog.sina.com.cn/s/blog_5e16f1770100gaiw.html 今天了碰到了,\mbox 是LaTeX原有命令,\text是 amsmath宏包 中的命令,它会按照所处位置自动改变字体大小,更强一些。 另外,如果数学模式下的文本较长,且间有数学公式,主要的是,多行公式,即环境下,则有 \intertext 这一命令下方和上方的公式仍处于同一个数学环境之中,因此可以保持上下的对齐关系不变。 如: 故意多的一个(end前的,同理)$\rightarrow$\\begin{align*}    (x + i y)(x - i y) &=x^2 + i xy - i xy -i^2 y^2\\                           &=x^2+y^2\\    \intertext{利用~$i^2=-1$,还可得到}    (x+i y)^2 &=x^2 + 2i xy -y^2\\    (x-i y)^2 &=x^2 - 2i xy -y^2 \\end{align*} 效果
kuing 22# 2013-4-11 21:55
那种情况我一般会拆开来打,所以 \intertext 对于我来说基本多余……事实上我也未曾用过
kuing 23# 2013-4-16 22:50
话说面对全角dang我还是没心思纠正了……
isea 24# 2013-4-16 23:24
23# kuing 可以理解(全角党),可以理解(楼上)
thread-1295-1-1.html: 附件能设置增加对.txt .7z .tex 支持不
isea 1# 2013-3-28 12:21
RT 这样就方便多了
kuing 2# 2013-3-28 12:55
全不限了
isea 3# 2013-3-28 21:25
还是限制一下吧,如果有exe,scr等这样的可执行文件,被恶意传来,染了病毒可不好
kuing 4# 2013-3-28 21:27
这一点限制相对低级的会员就行了,中级会员开始不限制。低级的也可以发 txt,tex,7z
thread-1296-1-1.html: 这两天晚睡了许多
isea 1# 2013-3-28 12:37
本帖最后由 isea 于 2013-3-28 13:37 编辑 奇怪 那些表情 后面的怎么点不上,用的opera
kuing 2# 2013-3-28 12:48
什么点不上?
isea 3# 2013-3-28 13:38
表情 什么 系列 趣味 之类
kuing 4# 2013-3-28 14:19
用 chrome 表示没问题……
isea 5# 2013-3-28 21:48
opera 果然小众   幸好,我不用这些玩意
thread-1297-1-3.html: [不等式] 来自粉丝群的一道幂指轮换最小值[未解决]
kuing 1# 2013-3-28 17:50
$x$, $y$, $z\in\mbb R^+$, $x+y+z=1$ 写大一点那个式子 $\dfrac{x^y}{y^x}+\dfrac{y^z}{z^y}+\dfrac{z^x}{x^z}$ PS、其实本来想将前后的聊天记录也截上来,不过想想还是算了……
thread-1298-1-3.html: [函数] 导数一题,洛比达法则或求导2次?
realnumber 1# 2013-3-29 07:42
本帖最后由 realnumber 于 2013-4-3 14:00 编辑 第2问问题等价于$\forall x,x\ge1, x\ln x+p(x^2-1)\le0$恒成立, 记$h(x)=\ln x+p(x-\frac{1}{x})$,由$f(1)=0$,得存在足够接近1的$ε,y=h(x),x\in[1,ε],$递减,即$h'(x)\le0$,如此得到$p\le -0.5$,再验证下充分性,完.---
realnumber 2# 2013-3-29 10:53
本帖最后由 realnumber 于 2013-3-29 10:56 编辑 群里又看到一题 $\forall x,x\in (0,1)\cup (1,+∞),\frac{2\ln x}{1-x^2}+\frac{1-k}{x}\ge0$恒成立,求k的取值范围.
kuing 3# 2013-3-29 12:17
1# realnumber 在高中大概不能那样玩,似乎需要一些高数定理的支持。
yes94 4# 2013-3-29 12:33
本帖最后由 yes94 于 2013-3-29 12:35 编辑 1172 第2问问题等价于$\forall x,x\ge1, x\ln x+p(x^2-1)\le0$恒成立, 记$h(x)=\ln x+p(x-\frac{1}{x})$,由$f(1)=0$,得存在足够接近1的$ε,y=h(x),x\in[1,ε],$递减,即$h'(x)\le0$,如此得到$p\le -0.5$,再验证下充 ... realnumber 发表于 2013-3-29 07:42 过程太简略的话,会造成读者减少。 这是常数变易法,将$0$变易成$h(1)=0$,其中$h(x)=\ln x+p(x-\dfrac{1}{x})$ 将$h(1)=0$误写为$f(1)=0$吧? 不过确实$f(1)=0$, 尝到了特殊的必要条件会成为充要条件的甜头,成为楼主继续冒险下去的动力。
realnumber 5# 2013-3-29 13:50
3# kuing 应该不需要的,h(0)=0,只有x=0附近递减,才能使得h(x)<0,由图象直观得到的.
kuing 6# 2013-3-29 14:18
5# realnumber 直观不能代替证明……
yes94 7# 2013-3-29 17:38
本帖最后由 yes94 于 2013-3-29 17:39 编辑 本题实际上是abababa以前的一个不等式:当$x>1$时,成立不等式\[\ln x<\frac12(x-\frac1x)\]
yes94 8# 2013-3-29 17:42
3# kuing 应该不需要的,h(0)=0,只有x=0附近递减,才能使得h(x) realnumber 发表于 2013-3-29 13:50 h(0)=0? 你也不想看看并回复4楼
realnumber 9# 2013-3-30 07:26
看到的,yes执著了 8# yes94
yes94 10# 2013-3-30 18:19
9# realnumber 有没有标准答案啊?学习一下
realnumber 11# 2013-3-31 08:24
10# yes94 没有, 又看到一题,2006全国2理 $\forall x\ge0 ,(x+1)\ln(x+1)\ge ax$恒成立,求a的取值范围. 答案$a\le1$
realnumber 12# 2013-3-31 08:27
2012湖南 $\forall x\in R,e^x-ax\ge 1$,求a的取值范围.明显是Taylor级数的痕迹.
yes94 13# 2013-3-31 13:28
2012湖南 $\forall x\in R,e^x-ax\ge 1$,求a的取值范围.明显是Taylor级数的痕迹. realnumber 发表于 2013-3-31 08:27 这是文科的吧, 理科的是:$e^{ax}-x\geqslant1$恒成立,求$a$的取值集合。
yayaweha 14# 2013-4-4 10:43
13# yes94 这个用洛比达就OK了
thread-1299-1-1.html: 幼儿园的题
realnumber 1# 2013-3-29 08:42

abababa 2# 2013-3-29 19:59
1# realnumber 据说那个插值法能行吧。 不过刚才问了一位网友,先说了句横扫幼儿园 ,然后给了35,52,24,48,83,37,76,65,51,14,49,96,68这样的解答,是像那种接龙的东西吧。
isea 3# 2013-3-29 21:52
如果这题有标答,那我真吐血了
thread-13-1-8.html: [不等式] 等比分式不等式(兜了个大弯,新证见5#)
kuing 1# 2011-9-26 18:13
设 $a>0$,$n\in\mathbb{N}^+$,求证 \[\frac{1+a^2+a^4+\cdots+a^{2n}}{a+a^3+\cdots+a^{2n-1}}\geqslant\frac{n+1}{n}\]
kuing 2# 2011-10-1 20:51
想到一个很麻烦的证法,大家看对不对。 超繁证  首先我们发现 $a=1$ 为取等条件,又不难看出将 $a$ 以 $\dfrac1a$ 代入后的不等式与原不等式是等价的,于是我们只需要证明 $a>1$ 时成立即可。 下面我们分 $n$ 的奇偶看一看,先看 $n$ 偶数时,分子分母同是除以 $a^n$,化为\[ \frac{\frac1{a^n} + \frac1{a^{n - 2}} + \cdots + 1 + \cdots + a^{n - 2} + a^n}{\frac1{a^{n - 1}} + \frac1{a^{n - 3}} + \cdots + \frac1a + a + \cdots + a^{n - 3} + a^{n - 1}} \geqslant \frac{n + 1}n \]上式去分母,再两边同时减去 $n(n+1)$,可以整理为\begin{align*} &n\left(\frac1{a^n} + a^n + \frac1{a^{n - 2}} + a^{n - 2} + \cdots + \frac1{a^2} + a^2 - n\right)\\ \geqslant & (n + 1)\left(\frac1{a^{n - 1}} + a^{n - 1} + \frac1{a^{n - 3}} + a^{n - 3} + \cdots + \frac1a + a - n\right) \end{align*}令 $a=b^2,b>1$,上式配方为\begin{align*} &n\left(\left(\frac1{b^n} - b^n\right)^2 + \left(\frac1{b^{n - 2}} - b^{n - 2}\right)^2 + \cdots + \left(\frac1{b^2} - b^2\right)^2\right)\\ \geqslant & (n + 1)\left(\left(\frac1{b^{n - 1}} - b^{n - 1}\right)^2 + \left(\frac1{b^{n - 3}} - b^{n - 3}\right)^2 + \cdots + \left(\frac1b - b\right)^2\right) \end{align*}由此我们不难看出,只需证明\[ \left(\frac1{b^n} - b^n\right)^2 \geqslant \frac{n + 1}n\left(\frac1{b^{n - 1}} - b^{n - 1}\right)^2 \qquad (*) \]即可,皆因如果式 $(*)$ 成立,那么对于其余的对应项,也有\[ \left(\frac1{b^{n - 2}} - b^{n - 2}\right)^2 \geqslant \frac{n - 1}{n - 2}\left(\frac1{b^{n - 3}} - b^{n - 3}\right)^2 \geqslant \frac{n + 1}n\left(\frac1{b^{n - 3}} - b^{n - 3}\right)^2 \]等等,这样,不等式就会成立。 现在暂时不证式 $(*)$,回头看一看 $n$ 为奇数时,用类似的方法及代换,可以将不等式化为\begin{align*} &n\left(\left(\frac1{b^n} - b^n\right)^2 + \left(\frac1{b^{n - 2}} - b^{n - 2}\right)^2 + \cdots + \left(\frac1b - b\right)^2\right)\\ \geqslant & (n + 1)\left(\left(\frac1{b^{n - 1}} - b^{n - 1}\right)^2 + \left(\frac1{b^{n - 3}} - b^{n - 3}\right)^2 + \cdots + \left(\frac1{b^0} - b^0\right)^2\right) \end{align*}所以仍然只需证式 $(*)$ 即可,故此剩下的问题就是证明式 $(*)$。由 $b>1$ 易知式 $(*)$ 等价于\[ \frac{b^{2n} - 1}{b^{2n - 1} - b}\geqslant \sqrt{\frac{n + 1}n}\qquad (*)' \]令\[ f(b)=\frac{b^{2n} - 1}{b^{2n - 1} - b}\qquad b>1 \]则求导得\[ f'(b)=\frac{2nb^{2n - 1} (b^{2n - 1} - b) - (b^{2n} - 1)((2n - 1)b^{2n - 2} - 1)}{(b^{2n - 1} - b)^2} \]上式的分子整理后代回 $a$ 可以化为\[ g(a)=a^{2n - 1} - (2n - 1)(a - 1)a^{n - 1} - 1 \]下面证明 $g(a)>0$,注意到 $g(1)=0$,故只要证明 $g'(a)>0$ 对 $a>1$ 成立。求导得\[ g'(a)=(2 n-1) a^{n-2}(a^n-na+n-1) \]设 $a=1+t$,则\[ a^n-na+n-1=(t+1)^n-nt-1 \]由二项式展开知上式显然为正,故 $g'(a)>0$ 成立,所以 $f(b)$ 在 $(1,+\infty)$ 递增,因此\[ f(b)>\lim_{b\to1+}f(b)=\lim_{b\to1+}\frac{b^{2n} - 1}{b^{2n - 1} - b} \]由洛必达法则有\[ \lim_{b\to1+}\frac{b^{2n} - 1}{b^{2n - 1} - b}=\lim_{b\to1+}\frac{2n b^{2n-1}}{(2n-1)b^{2n-2}-1}=\frac n{n-1} \]即\[ f(b)>\frac n{n-1} \]而显然\[ \frac n{n-1}>\sqrt{\frac n{n-1}}>\sqrt{\frac{n+1}n} \]从而式 $(*)'$ 成立,原不等式获证!
kuing 3# 2011-10-1 21:50
后面求极限那里可以简单点 \[ f(b)>\lim_{b\to1+}f(b)=\lim_{b\to1+}\frac{b^{2n} - 1}{b^{2n - 1} - b}=\lim_{b\to1+}\frac{(b-1)(b^{2n-1}+b^{2n-2}+\cdots+1)}{b(b-1)(b^{2n-3}+b^{2n-4}+\cdots+1)}=\frac n{n-1}>\sqrt{\frac n{n-1}}>\sqrt{\frac{n+1}n} \]
kuing 4# 2011-10-2 01:22
再重新观之,发现二楼的证法前半部分竟然是多余的,why?因为其实上面证式 $(*)'$ 的导数方法,完全适用于原不等式!:L 具体地,利用等比数列求和公式,当 $a>1$ 时,原不等式等价于\[ \frac{1-a^{2n+2}}{a(1-a^{2n})}\geqslant\frac{n+1}n \]这左边跟二楼中的 $f(b)$ 只差指数,将 $n$ 变成 $n+1$ 就是了,所以……oh 但是式 $(*)'$ 比原不等式要弱,由此亦可见,有时候我们通过变形,就可以用较弱的式去证明较强的式,谓之“弱化证法”者也……
kuing 5# 2011-10-2 01:51
于是,我们将多余部分扔掉,再对其余部分加以改良,就得到如下相对简单的证法。 不太繁的证  令\[ K(a)=\frac{1+a^2+a^4+\cdots+a^{2n}}{a+a^3+\cdots+a^{2n-1}}\qquad a\in(0,+\infty) \]则显然 $K(a)$ 在 $(0,+\infty)$ 上连续且可导。 不难发现恒有 $K(a)=K\left(\dfrac1a\right)$,所以我们只需证明原不等式对 $a\geqslant1$ 成立。 又显然 $K(1)=\dfrac{n+1}n$ 为取等条件,故此假如我们能证明 $K(a)$ 在 $(1,+\infty)$ 上单调递增,那么原不等式就成立,下面证之。 由于当 $a>1$ 时,利用等比数列求和公式可将函数化简为\[ K(a)=\frac{1-a^{2n+2}}{a(1-a^{2n})} \]求导得\[ K'(a)=\frac{(2n+2)a^{2n + 1} (a^{2n + 1} - a) - (a^{2n+2} - 1)((2n + 1)a^{2n} - 1)}{(a^{2n + 1} - a)^2} \]令 $a^2=x>1$,则上式的分子可以整理为\[ T(x)=x^{2n+1} - (2n + 1)(x - 1)x^n - 1 \]下面证明当 $x>1$ 时 $T(x)>0$,注意到 $T(1)=0$,故只要证明 $T'(x)>0$ 对 $x>1$ 成立。对 $T(x)$ 求导得\[ T'(x)=(2n+1)x^{n-1}(x^{n+1}-(n+1)x+n) \]由均值不等式,有\[ x^{n+1}+n=x^{n+1}+\underbrace{1+1+\cdots+1}_{n 个}>(n+1)x \]所以 $T'(x)>0$ 成立,从而 $T(x)>0$ 成立,从而 $K'(a)>0$ 成立,所以原不等式获证。
kuing 6# 2011-10-2 02:11
这下可以睡觉了,闪……
yes94 7# 2011-10-2 13:53
新证2:
kuing 8# 2011-10-2 14:00
新证2: yes94 发表于 2011-10-2 13:53 你的意思是有 $a^{2n+1}(na-n-1)\geqslant n-n-1$ ? 这是不是要证一下,似乎并不显然,皆因 $(na-n-1)$ 可以是负的。
kuing 9# 2011-10-2 14:09
事实上 $a^{2n+1}(na-n-1)$ 在 $a=1+\frac1{2n}$ 取最小。 这说明并不是如楼上所说,那最后一步的不等号是因何而来?烦请明示
yes94 10# 2011-10-2 15:01
:L 那个人欺骗了我,我去找他算账。 还有不熟悉这论坛的输入字符,不好说明,
kuing 11# 2011-10-2 15:06
10# yes94 是谁哟?
yes94 12# 2011-10-2 15:17
这论坛打开好慢!终于打开了呢,又是一片空白,又刷新,又等,……,还不能传图片。 估计要用到e=2.71828……,的定义,
kuing 13# 2011-10-2 15:28
为什么我基本上都是飞速打开…… 我印象中,权限我全开放了,应该可以传图片,只不过限了512KB(这是5d6d限的),我用个测试号试一试
kuing 14# 2011-10-2 15:31
为什么我基本上都是飞速打开…… 我印象中,权限我全开放了,应该可以传图片,只不过限了512KB(这是5d6d限的),我用个测试号试一试 kuing 发表于 2011-10-2 15:28 测试成功 http://kkkkuingggg.5d6d.com/thread-39-1-1.html
yes94 15# 2011-10-2 15:46
以前也打不开,或打开慢。或公式只显示上半截(IE8用兼容模式解决了)。 刚才起码等了半个小时才打开论坛, 我上传图片,总说我错误。 中午我上传的图片来自于http://bbs.pep.com.cn/thread-1886518-1-1.html 只能在那个地方先上传。
kuing 16# 2011-10-2 15:48
看来跟IE8“有缘无份”了,呵呵,我用IE6也没事。 我看不会跟地理位置或者网络类型有关吧?甚至是防火墙?杀软?……:L 无能为力了我
yes94 17# 2011-10-2 17:33
哎,来一趟不容易啊!精疲力竭啦,又是刷新,又是等待,出来后又是空白,不知道其他是不是这样的?……, 再看下能不能传图片?好像这次可以了呢? ___________kuing edit in $\LaTeX$___________ $a=1$时,结论显然成立; $a>1$时,原不等式等价于 \begin{align*} \frac{a^{2n+2}-1}{(n+1)(a^2-1)}\geqslant a\frac{a^{2n}-1}{n(a^2-1)}&\iff na^{2n+2}-n-(n+1)a^{2n+1}+(n+1)a\geqslant 0\\ &\iff na^{2n+1}(a-1)+1-a^{2n+1}+(n+1)(a-1)\geqslant 0\\ &\iff na^{2n+1}+n+1-\sum_{i=1}^{2n+1}a_{i-1}\geqslant 0\\ &\iff na^{2n+1}+n-\sum_{i=1}^{2n}a^i\geqslant 0\\ &\iff \sum_{i=1}^{n}(a^{2n+1}+1-a^i-a^{2n+1-i})\geqslant 0, \end{align*} 最后一步因式分解即得。
kuing 18# 2011-10-2 18:40
17# yes94 嗯,这个初步看了下没问题,不错。吃完饭回来再细赏。 至于论坛访问慢之类的问题,暂时还没其他人反映过你所述的问题,isea、yuzi 等在前几天测试公式的时候也很正常地回了贴,不过这几天他们没来,不知是不是仍然正常。
realnumber 19# 2011-10-8 20:58
原不等式,先去分母,再用等比后,构造函数,那样计算更简单,特别是那个求导。
kuing 20# 2011-12-19 13:57
简洁数归见08年人教贴 http://bbs.pep.com.cn/thread-378909-1-1.html 9#
thread-130-1-6.html: 平面上任意六个格点不能连成正六边形
kuing 1# 2011-10-22 15:39
求证:平面上任意六个格点不能连成正六边形。 为方便叙述,我们称所有顶点都在平面格点上的正n边形为“正格点n边形”。 首先有如下显然的事实:以任意两格点的连线为边长向某方向作一正方形,则该正方形的另个两个顶点仍然是格点。 其次,我们假设存在正格点6边形。那么以正格点6边形的某边为边长向内作的正方形两顶点也是格点。 我们依次对6条边都这样操作,得到如下图形 于是显然上图中间连结的6边形仍然是正格点6边形且比原6边形要小,这说明每个正格点6边形总存在比其更小的正格点6边形,这显然矛盾,因为如果存在正格点6边形,必定有最小者。 依此证法,我们还可以证明不存在正格点n边形($n\geqslant5$),而正格点三角形的不存在则可以通过如下图形说明。 由蓝色的正格点三角形通过作正方形得出更小的红色的正格点三角形。 因此综上所述,正格点n边形的解只有 $n=4$。
图图 2# 2011-10-22 15:59
我的考试题,试卷我找到了
kuing 3# 2011-10-22 16:03
2# 图图 是,刚才想起,想到。 不知参考答案是否一样。
图图 4# 2011-10-22 16:07
3# kuing 试卷没有分析
kuing 5# 2011-10-22 16:11
这个证法没问题吧?
hhhzh7241hzh 6# 2011-10-22 16:15
看看
图图 7# 2011-10-22 16:16
5# kuing 应该没有
GAM 8# 2011-10-22 22:01

isea 9# 2011-10-25 11:30
很有意思的题目,曾经有一道题是正三角形三个顶点不能同时落到什么上的,一时忘记了。
kuing 10# 2011-10-25 11:31
9# isea
海盗船长 11# 2011-11-2 18:28
看看
海盗船长 12# 2011-11-2 18:31
无穷递降法
kuing 13# 2011-11-2 18:33
12# 海盗船长 嗯,就是这种思想,不过以前只在数论里听说过。。。。。
哆啦a 14# 2012-8-12 21:50

kuing 15# 2012-8-12 22:00
14# 哆啦a 梦
realnumber 16# 2012-8-21 14:07
看看
realnumber 17# 2012-8-21 14:12
绝妙
三眼娃 18# 2012-8-21 17:18
学习,一定要看
叶剑飞Victor 19# 2012-8-22 15:44
应该可以证明格点无法组成$120^\circ$角吧。
海盗船长 20# 2012-8-22 17:48
19# 叶剑飞Victor 嗯应该可以
thread-130-2-6.html:
hongxian 21# 2012-8-23 04:38
看一下!
kuing 22# 2012-8-23 04:50
21# hongxian 这么晚…
ftg1029 23# 2012-8-23 09:28
我想看看。
abababa 24# 2012-9-6 20:16
试着证明看看,假设能组成正六边形ABCDEF,那么AD都是整数坐标,那么中点O是2分之一个整数,那么等边$\triangle ABO$是一个顶点都是2分之整数的等边$\triangle$,把所有顶点都乘以2,得到一个三个顶点都是整数的等边$\triangle$,和原来的$\triangle$相似,而且三个顶点都是整数,如果设三个点的坐标是$A(0,0)B(m,n)O(x,y)$,那么就能算出来$x=\frac{m}{2}-\frac{\sqrt{3}n}{2},y=\frac{n}{2}+\frac{\sqrt{3}m}{2}$,m、n、x都是整数,那么只有n=0,同理m=0,这时候$ABO$就不是$\triangle$了,所以不能构成正六边形。
kuing 25# 2012-9-6 20:35
24# abababa 看了下应该没问题,证得不错:D
thread-1300-1-3.html: [数列] 数列根号递推求通项(from 粉丝群)
kuing 1# 2013-3-29 14:06
题目:已知数列 $\{a_n\}$ 的前 $n$ 项和为 $S_n$,$a_1=a$($a>0$),且满足 $a_{n+1}=\sqrt{S_n^2+a^2}$,试求数列 $\{a_n\}$ 的通项公式。 由递推关系得 \begin{align*} a_{n+1}^2&=S_n^2+a^2, \\ a_n^2&=S_{n-1}^2+a^2, \end{align*} 相减得 \begin{align*} a_{n+1}^2-a_n^2&=(S_n+S_{n-1})(S_n-S_{n-1}), \\ a_{n+1}^2-a_n^2&=(a_n+2S_{n-1})a_n, \\ a_{n+1}^2-2a_n^2&=2a_nS_{n-1}, \end{align*} 于是有 \begin{align*} \frac{a_{n+1}^2-2a_n^2}{2a_n}&=S_{n-1}, \\ \frac{a_{n+2}^2-2a_{n+1}^2}{2a_{n+1}}&=S_n, \end{align*} 相减得 \[\frac{a_{n+2}^2-2a_{n+1}^2}{2a_{n+1}}-\frac{a_{n+1}^2-2a_n^2}{2a_n}=a_n,\] 可以变形为 \[\left( \frac{a_{n+2}}{a_{n+1}} \right)^2=\frac{a_{n+1}}{a_n}+2,\] 令 $a_{n+1}/a_n=b_n$,则 \[b_{n+1}^2=b_n+2,\] 易知 $a_2=\sqrt2a$,即 $b_1=\sqrt2$,由此易证 $0<b_n<2$,故可以令 $b_n=2\cos c_n$,其中 $c_n\in(0,\pi/2)$,代入上式有 \[(2\cos c_{n+1})^2=2\cos c_n+2 \iff \cos2c_{n+1}=\cos c_n \iff c_{n+1}=\frac12c_n,\] 注意到 $c_1=\pi/4$,于是 \[c_n=\frac\pi{2^{n+1}},\] 即 \[\frac{a_{n+1}}{a_n}=2\cos\frac\pi{2^{n+1}},\] 所以 \[a_n=a\prod_{k=1}^{n-1}\left(2\cos\frac\pi{2^{k+1}}\right)=2^{n-1}a\prod_{k=2}^n\cos\frac\pi{2^k}.\] PS、不知最后结果能不能再化简……
转化与化归 2# 2013-3-29 14:29
其他一样
kuing 3# 2013-3-29 14:30
O,这样的确更简单,我前面做复杂了……
yes94 4# 2013-3-29 18:05
1# kuing 实在要化简,就分子分母乘以某个角度的正弦,然后连锁反应……
kuing 5# 2013-3-29 18:20
4# yes94 OMG!我竟然没想起这个…… 今天不是做题状态……
yes94 6# 2013-3-29 18:23
和切比雪夫多项式有关吧?
kuing 7# 2013-4-1 11:00
化简后结果正好是 \[a_n=a\csc\frac\pi{2^k}.\]
kuing 8# 2013-4-1 11:12
来自人教数学群,战巡 爱好者-战巡(3705*****)  11:02:04 爱好者-战巡(3705*****)  11:03:50 s[n]=a*cot(π/2^(n+1)),a[n]=s[n]-s[n-1]
yes94 9# 2013-4-1 11:28
8# kuing 现在记起了数学通报1998-2000期间某一期有此题的几何解法。
Tesla35 10# 2013-4-1 12:40
收录
thread-1301-1-3.html: [不等式] 闪前发现下午有道题忘了看
kuing 1# 2013-3-30 00:57
$x$, $y$, $z\in[0,1]$,求 $x\sqrt{1-y}+y\sqrt{1-z}+z\sqrt{1-x}$ 的最大值。 又看了看,不会……今天的确不是解题模式……先闪,明天再看
abababa 2# 2013-3-30 16:34
本帖最后由 abababa 于 2013-3-30 16:56 编辑 1# kuing 能不能用排序不等式呢? $x \ge y \ge z$,然后就有$1-z \ge 1-y \ge 1-x$,所给的是乱序,大于逆序吧,$f \ge x\sqrt{1-x}+y\sqrt{1-y}+z\sqrt{1-z}$ 最后变成三个单变量的,求导就能算了,$x=y=z=2/3$时取等号。 我不等式弱,看看有没有错误? 还是错了,求了最小值。 不过用拉格朗日乘数法算,求到的是最小值的那个极值点,另一个应该在端点处吧,如果是x=0,最大值就是$z+y\sqrt{1-z} \le z+\sqrt{1-z} \le 5/4$了,此时$x=0$,$y=1$,$z=\frac{3}{4}$
kuing 3# 2013-3-30 17:25
原来是这样的取等条件……
yes94 4# 2013-3-30 17:53
1# kuing 能不能用排序不等式呢? $x \ge y \ge z$,然后就有$1-z \ge 1-y \ge 1-x$,所给的是乱序,大于逆序吧,$f \ge x\sqrt{1-x}+y\sqrt{1-y}+z\sqrt{1-z}$ 最后变成三个单变量的,求导就能算了,$x=y=z=2 ... abababa 发表于 2013-3-30 16:34 能不能这样设?$x \ge y \ge z$,
yes94 5# 2013-3-30 18:32
某网友的证明:
kuing 6# 2013-3-30 19:49
看了这个,才想起了2009年的这个贴:http://www.irgoc.org/viewtopic.php?f=15&t=9
yes94 7# 2013-3-30 22:09
似乎can_hang们都很在乎copyright呢,我只是没给出是谁答复的,
kuing 8# 2013-3-30 22:18
印象中……题目出处……好像是陈计的……
pxchg1200 9# 2013-4-4 13:42
目测是Vasc的
yes94 10# 2013-4-4 23:39
来一个简单地(两个变量): 当$x,y\in[0,1]$时,求$x\sqrt{1-y}+y\sqrt{1-x}$的最大值。 楼主是三个变量的,能否推广到四个变量?
thread-1302-1-3.html: [不等式] 三边根式不等式(from 粉丝群)
kuing 1# 2013-3-30 14:41
由 \[\left(\frac{a+b+c}{2(a+b)}\right)^2-\frac{(2a+c)(2b+c)}{(3a+b)(3b+a)}=\frac{(3a+3b+c)(a+b-c)(a-b)^2}{4(a+b)^2(3a+b)(3b+a)}\geqslant 0,\] 得到 \[\sum\sqrt{\frac{(2a+c)(2b+c)}{(3a+b)(3b+a)}}\leqslant\frac{a+b+c}2\sum\frac1{a+b},\] 而 \[\frac52-\frac{a+b+c}2\sum\frac1{a+b}=\frac{abc+\sum a^2(b+c-a)}{2(a+b)(b+c)(c+a)}>0,\] 故原不等式得证。
yes94 2# 2013-3-30 22:14
1# kuing 居然还有这种放缩方法! 即便想到了这种放缩,也证明不出来
pxchg1200 3# 2013-3-30 22:16
1# kuing 神奇的局部。。。。
kuing 4# 2013-3-30 23:45
3# pxchg1200 真的是根据你在群里发的那个放过头的放缩再加以收紧得到的……运气可以说是……
thread-1303-1-3.html: [数论] 2012江苏高考附加题的压轴题
pengcheng1130 1# 2013-3-30 21:42
设集合$P_{n}=\{1,2,\cdots,n\},n\in N^{*}$,记$f(n)$为同时满足下列 条件的集合$A$的个数: (1)$A\subseteq P_{n}$; (2)若$x\in A$,则$2x\notin A$; (3)若$x\in \complement_{P_n}A$,则$2x\notin \complement_{P_n}A$. 求$f(n)$的解析式(用$n$表示) 参考答案玄乎玄乎的,哪位能个给出比较自然的思路.
yes94 2# 2013-3-30 22:11
1# pengcheng1130 本来题目就玄乎的玄乎的,…… 似乎是给竞赛党们做的……
realnumber 3# 2013-3-31 10:04
2)(3)的意思可以举这样例子,若$P_n$中出现,1,2,4,8,16,32的等比数列.那么1,4,16必须在一起,2,8,32必须在一起 "1,4,16"与"2,8,32"必须分开. 试试$P_1$,得A为空集或{1},即$f(1)=2$ 试试$P_2$,得A为{1}或{2},即$f(2)=2$ 试试$P_3$,得A为{1}或{2}或{1,3}或{2,3},即$f(3)=4$ 通过$P_3$的例子,考虑,若$P_n$中出现,1,2,4,8,16,32以及3,6,12,24,48,.怎么处理,又冒出新问题"新一个公比为2的等比数列的出现与n的关系"? 若$P_n$可以分割为k个等比数列,那么$f(n)=2^k$, 新的一个奇数出现,可以看作一个新的公比为2的等比数列的首项, 那么就是$k=[\frac{n+1}{2}]$,即问题的答案就是$f(n)=2^{[\frac{n+1}{2}]}$,是不是这样啊?
yes94 4# 2013-3-31 13:35
3# realnumber
realnumber 5# 2013-3-31 16:05
4# yes94 那就是了,[x]取整函数,表示不超过x的最大整数.
isea 6# 2013-3-31 22:06
做过类似的填空题,关注一下
yes94 7# 2013-3-31 23:12
命题$1$:集合$A$若有$k$个元素,则集合$A$有$2^k$个子集。 命题$2$:集合$P_n=\{1,2,3,\cdots,n\}$里奇数的个数是$\dfrac n2$($n$为偶数)或$\dfrac {n+1}2$($n$为奇数)。 命题$3$:若奇数$m\in A$,则$2^2m,2^4m,2^6m,\cdots,2^{2t}m\in A$,其中$2^{2t}m$是属于集合$P_n=\{1,2,3,\cdots,n\}$里最大的元素。 命题$4$:若奇数$m\notin A$,则$2m,2^3m,2^5m,\cdots,2^{2t-1}m\in A$,其中$2^{2t-1}m$是属于集合$P_n=\{1,2,3,\cdots,n\}$里最大的元素。 然后就好办了吧?
thread-1304-1-1.html: 看原始定义文件后测试下
kuing 1# 2013-3-30 23:05
$\wp\ \Re\ \Im\ \top$ $\lnot\ \land\ \lor\ \gets\ \mapstochar\ \mapsto$ \[\intop_a^bf(x)dx=\ointop_a^bf(x)dx\]
thread-1305-1-1.html: 看看\textcircled能不能用
kuing 1# 2013-3-30 23:54
$\textcircled{a}$
kuing 2# 2013-3-31 00:02
果断无法加圈圈…… 再试几个特殊的 $\circledS,\circledR,\copyright$
kuing 3# 2013-4-2 11:49
顺便记录一个链接http://www.latexstudio.net/good-way-to-make-textcircled-numbers/ 关于带圈数字的
isea 4# 2013-4-10 22:43
本帖最后由 isea 于 2013-4-10 22:44 编辑 \usepackage{pifont} \usepackage[perpage,symbol*]{footmisc} \DefineFNsymbols{circled}{{\ding{192}}{\ding{193}}{\ding{194}} {\ding{195}}{\ding{196}}{\ding{197}}{\ding{198}}{\ding{199}}{\ding{200}}{\ding{201}}} \setfnsymbol{circled} 直接将这个代码复制来得了 我想一般情况下,用这个平易近人些 效果图 圈1 对应 \ding{192}
isea 5# 2013-4-20 23:19
本帖最后由 isea 于 2013-4-22 12:50 编辑 何版这一套带圈数字,真漂亮。(不懂原理,但好用,貌似用PGF画的,感觉)。 用xelatex编译,需要tikz与fontspec宏包(调用xeCJK会自动调用fontspec;再如ctexart文档也会自己调用xeCJK)。 \usepackage{xeCJK} \usepackage{tikz}                                                                                                                   %编号圈0样式到20 \newfontfamily\CM{Cambria Math} \newcommand{\cmcirc}[1]{\pgfmathparse{                                                                                %白底黑字 风格1     ifthenelse(#1 > 0 && #1 < 21, Hex(9311+#1), Hex(9450))     }{\CM{\symbol{"\pgfmathresult}}}} \newcommand{\cmcircblk}[1]{\pgfmathparse{                                                                            %黑底白字 风格2     ifthenelse(#1 > 0 && #1 < 11, Hex(10101+#1),         ifthenelse(#1 > 10 && #1 < 21, Hex(9450-10+#1),             Hex(9471)         )     )     }{\CM{\symbol{"\pgfmathresult}}}} 复制代码 用法如圈5: \cmcirc{5}           %风格1 \cmcircblk{5}        %风格2 === 按8楼和10楼已做修改;何版在11楼给出“完整”风格圈,这里只提取我觉得最好看的最小代码。
kuing 6# 2013-4-20 23:34
5# isea 不是用 pgf 画的,那里只不过提供了一个对应关系,方便一点而已,事实上可以不用 pgf
kuing 7# 2013-4-21 00:09
Hex 是 pgf 的一个函数,将十进制数转化为十六进制,\symbol{"十六进制数} 就是引用字体里的某个字符,\pgfmathresult 是 pgf 的输出结果,然后用 ifthenelse 建立一个函数来对应…… 还是不扯太多了,自己理解下……
hnsredfox_007 8# 2013-4-21 09:06
5# isea 为什么我复制粘贴到导言区中,无法编译啊?我的是ctex.
hnsredfox_007 9# 2013-4-21 09:07
4# isea 这个可用!编译通过……
kuing 10# 2013-4-21 09:18
8# hnsredfox_007 他没帖完整,还要加tikz宏包,如果不是用ctex系列文档类或宏包的话还要加fontspec或xecjk。 要用xelatex编译。 如果要整到脚注,还要把脚注的某个参数重定义一下。
hejoseph 11# 2013-4-22 10:44
试了若干个带圈数字的字体,pifont的带圈数字里面的数字形状与默认无圈的数字形状很接近,但太小了,于是提取字体出来放大,做成CircledNum字体,若要使用安装一下就可以了。 pdf里是几种带圈数字及默认无圈的数字的对比。
kuing 12# 2013-4-22 12:55
其实能做,而且有闲情的话,不如整到99
hejoseph 13# 2013-4-22 12:59
unicode里只有0到20带圈数字,21-99都无对应unicode的,无通用性,所以懒得做,20个也足够了
isea 14# 2013-4-22 13:26
本帖最后由 isea 于 2013-4-22 14:00 编辑 安装附带字体,显示7;偶这里还少4和5的字体。 4和5的字体windows下有么?如果我下载安装一下。 ============= 第5种找到windows版了:http://sourceforge.jp/projects/sfnet_junicode/releases/ 维基百科,自由的百科全书 Junicode样字文本 Junicode(全称Junius-Unicode)是一款支援Unicode的复古风格衬线字体,由弗吉尼亚大学Peter S. Baker主持设计。它的设计灵感来自17世纪英国牛津所使用的一种字体。字体囊括了大量特殊字符及合字,以便于中世纪研究使用。具有OpenType特性。 Junicode最新版本为0.7.8,发布于2012年12月30日。 =================== 最后,如果你编译11楼许多错误,建议你把所以有关\CMU行用%标注掉,偶这里CTEX (basic)套装+win7 无法编译出效果来(如下图123数字字哪一行均空白),我想你也应该类似吧。 (即 \newfontfamily\CMU{CMU Serif} {\CMU{0123456789}} )
isea 15# 2013-4-22 14:09
本帖最后由 isea 于 2013-4-22 14:12 编辑 windows 一般用户把11楼(tex文件中的“文字”)代码改成这样,应该只有4个警 告,其他应该能一次性通过 \documentclass{article} \usepackage{fontspec} \usepackage{pifont} \usepackage{tikz} \newcommand{\dingcira}[1]{\pgfmathparse{ int(171+#1) }{\ding{\pgfmathresult}}} \newcommand{\dingcirb}[1]{\pgfmathparse{ int(181+#1) }{\ding{\pgfmathresult}}} \newcommand{\dingcirc}[1]{\pgfmathparse{ int(191+#1) }{\ding{\pgfmathresult}}} \newcommand{\dingcird}[1]{\pgfmathparse{ int(201+#1) }{\ding{\pgfmathresult}}} \newfontfamily\song{SimSun} \newcommand{\songcirc}[1]{\pgfmathparse{     ifthenelse(#1 > 0 && #1 < 21, Hex(9311+#1), Hex(9450))     }{\song{\symbol{"\pgfmathresult}}}} \newcommand{\songcircblk}[1]{\pgfmathparse{     ifthenelse(#1 > 0 && #1 < 11, Hex(10101+#1),         ifthenelse(#1 > 10 && #1 < 21, Hex(9450-10+#1),             Hex(9471)         )     )     }{\song{\symbol{"\pgfmathresult}}}} \newfontfamily\MG{MS Gothic} \newcommand{\mgcirc}[1]{\pgfmathparse{     ifthenelse(#1 > 0 && #1 < 21, Hex(9311+#1), Hex(9450))     }{\MG{\symbol{"\pgfmathresult}}}} \newfontfamily\CM{Cambria Math} \newcommand{\cmcirc}[1]{\pgfmathparse{     ifthenelse(#1 > 0 && #1 < 21, Hex(9311+#1), Hex(9450))     }{\CM{\symbol{"\pgfmathresult}}}} \newcommand{\cmcircblk}[1]{\pgfmathparse{     ifthenelse(#1 > 0 && #1 < 11, Hex(10101+#1),         ifthenelse(#1 > 10 && #1 < 21, Hex(9450-10+#1),             Hex(9471)         )     )     }{\CM{\symbol{"\pgfmathresult}}}} %\newfontfamily\LL{Linux Libertine O}                                                       %Linux字体,windows默认一定没有的 %\newcommand{\libcirc}[1]{\pgfmathparse{ %    ifthenelse(#1 > 0 && #1 < 21, Hex(9311+#1), Hex(9450) %    }{\LL{\symbol{"\pgfmathresult}}}} %\newcommand{\libcircdbl}[1]{\pgfmathparse{Hex(9460+#1)}{\LL{\symbol{"\pgfmathresult}}}} %\newcommand{\libcircblk}[1]{\pgfmathparse{ %    ifthenelse(#1 > 0 && #1 < 11, Hex(10101+#1), %        ifthenelse(#1 > 10 && #1 < 21, Hex(9450-10+#1), %            Hex(9471) %        ) %    ) %    }{\LL{\symbol{"\pgfmathresult}}}} %\newfontfamily\Junicode{Junicode}                                                                                       %Junicode字体,windows下一般默认也没有的 %\newcommand{\juncirc}[1]{{\fontspec[Ligatures=Discretionary]{Junicode}[#1]}} %\newcommand{\juncircdbl}[1]{{\fontspec[Ligatures=Discretionary]{Junicode}[[#1]]}} %\newcommand{\juncircblk}[1]{{\fontspec[Ligatures=Discretionary]{Junicode}<#1>}} \newfontfamily\CircledNum{CircledNum}                                               %安装11楼附带字体后就能正常了 \newcommand{\cncirc}[1]{\pgfmathparse{     ifthenelse(#1 > 0 && #1 < 21, Hex(9311+#1), Hex(9450))     }{\CircledNum{\symbol{"\pgfmathresult}}}} \newcommand{\cncircblk}[1]{\pgfmathparse{     ifthenelse(#1 > 0 && #1 < 11, Hex(10101+#1),         ifthenelse(#1 > 10 && #1 < 21, Hex(9450-10+#1),             Hex(9471)         )     )     }{\CircledNum{\symbol{"\pgfmathresult}}}} \renewcommand{\thefootnote}{\protect\cmcircblk{\value{footnote}}} \setlength{\parindent}{0pt} \begin{document} { \section{PiFont} 0123456789 \foreach \x in {1,...,10} {\dingcira{\x}} \foreach \x in {1,...,10} {\dingcirb{\x}} \foreach \x in {1,...,10} {\dingcirc{\x}} \foreach \x in {1,...,10} {\dingcird{\x}} } { \section{\song{宋体}} 0123456789 \foreach \x in {1,...,10} {\songcirc{\x}} } { \section{\MG{MS Gothic}} 0123456789 \foreach \x in {0,...,20} {\mgcirc{\x}} } %{ %\section{\LL{Linux Libertine}}                                      %Linux字体,windows默认一定没有的 % %0123456789 % %\foreach \x in {0,...,20} {\libcirc{\x}} % %\foreach \x in {1,...,10} {\libcircdbl{\x}} % %\foreach \x in {0,...,20} {\libcircblk{\x}} %} %{ %\section{\Junicode{Junicode}}                           %Junicode字体,windows下一般默认也没有的 % %0123456789 % %\foreach \x in {0,...,20} {\juncirc{\x}} % %\foreach \x in {1,...,10} {\juncircdbl{\x}} % %\foreach \x in {0,...,20} {\juncircblk{\x}} %} { \section{\CM{Cambria Math}} 0123456789 \foreach \x in {0,...,20} {\cmcirc{\x}} \foreach \x in {0,...,20} {\cmcircblk{\x}} } { \section{CircledNum}                     %安装11楼附带字体后就能正常了 0123456789 \foreach \x in {0,...,20} {\cncirc{\x}} \foreach \x in {0,...,20} {\cncircblk{\x}} } \end{document} 复制代码
hejoseph 16# 2013-4-22 14:12
本帖最后由 hejoseph 于 2013-4-22 15:35 编辑 CMU Serif、Linux Libertine、Junicode出问题是因为你还没更新宏包,你把宏包更新完之后就不会有问题了,我的机器也是windows系统
isea 17# 2013-4-22 23:47
CMU Serif、Linux Libertine、Junicode出问题是因为你还没更新宏包,你把宏包更新完之后就不会有问题了,我的机器也是windows系统 hejoseph 发表于 2013-4-22 14:12 升,果断 update MiKTeX updaeable packages
isea 18# 2013-4-23 23:33
安装附带字体,显示7;偶这里还少4和5的字体。 4和5的字体windows下有么?如果我下载安装一下。 ============= 第5种找到windows版了:http://sourceforge.jp/projects/sfnet_junicode/releases/ 维基百 ... isea 发表于 2013-4-22 13:26 ============================== 终于给我找到相应的字体宏包了 Linux Libertine 在最下的 uncategorized (未分类之意)中的 libertine 宏包  (第四种字体,这种字体是多数linux下默认字体,很赞)
isea 19# 2013-4-23 23:37
本帖最后由 isea 于 2013-4-23 23:40 编辑 代码中 \newfontfamily\CMU{CMU Serif} 中的 CMU Serif 是 fonts 类下的  outline fonts ——cm-unicode 宏包,推荐大家安装 同样的,在此分类中,能找到 Junicode 字体
isea 20# 2013-4-23 23:45
差不多,还有 LaTeX Font Warning: Font shape `EU1/SimSun(0)/bx/n' undefined (Font)              using `EU1/SimSun(0)/m/n' instead on input line 101. LaTeX Font Warning: Font shape `EU1/MSGothic(0)/bx/n' undefined (Font)              using `EU1/MSGothic(0)/m/n' instead on input line 109. ************************************************* * fontspec warning: "icu-feature-not-exist-in-font" * * OpenType feature 'Ligatures=Discretionary' (+dlig) not available for font * 'Junicode/B' with script 'Latin' and language 'Default'. ************************************************* ************************************************* * fontspec warning: "icu-feature-not-exist-in-font" * * OpenType feature 'Ligatures=Discretionary' (+dlig) not available for font * 'Junicode/B' with script 'Latin' and language 'Default'. ************************************************* ************************************************* * fontspec warning: "icu-feature-not-exist-in-font" * * OpenType feature 'Ligatures=Discretionary' (+dlig) not available for font * 'Junicode/BI' with script 'Latin' and language 'Default'. ************************************************* LaTeX Font Warning: Font shape `EU1/CambriaMath(0)/bx/n' undefined (Font)              using `EU1/CambriaMath(0)/m/n' instead on input line 141. [1] (C:\Users\iC\Desktop\CircledNum.aux) LaTeX Font Warning: Some font shapes were not available, defaults substituted. ) 这样的警告就无视了,不管了,至此,基本解决能完整编译出11楼
thread-1305-2-1.html: 看看\textcircled能不能用
kuing 1# 2013-3-30 23:54
$\textcircled{a}$
kuing 2# 2013-3-31 00:02
果断无法加圈圈…… 再试几个特殊的 $\circledS,\circledR,\copyright$
kuing 3# 2013-4-2 11:49
顺便记录一个链接http://www.latexstudio.net/good-way-to-make-textcircled-numbers/ 关于带圈数字的
isea 4# 2013-4-10 22:43
本帖最后由 isea 于 2013-4-10 22:44 编辑 \usepackage{pifont} \usepackage[perpage,symbol*]{footmisc} \DefineFNsymbols{circled}{{\ding{192}}{\ding{193}}{\ding{194}} {\ding{195}}{\ding{196}}{\ding{197}}{\ding{198}}{\ding{199}}{\ding{200}}{\ding{201}}} \setfnsymbol{circled} 直接将这个代码复制来得了 我想一般情况下,用这个平易近人些 效果图 圈1 对应 \ding{192}
isea 5# 2013-4-20 23:19
本帖最后由 isea 于 2013-4-22 12:50 编辑 何版这一套带圈数字,真漂亮。(不懂原理,但好用,貌似用PGF画的,感觉)。 用xelatex编译,需要tikz与fontspec宏包(调用xeCJK会自动调用fontspec;再如ctexart文档也会自己调用xeCJK)。 \usepackage{xeCJK} \usepackage{tikz}                                                                                                                   %编号圈0样式到20 \newfontfamily\CM{Cambria Math} \newcommand{\cmcirc}[1]{\pgfmathparse{                                                                                %白底黑字 风格1     ifthenelse(#1 > 0 && #1 < 21, Hex(9311+#1), Hex(9450))     }{\CM{\symbol{"\pgfmathresult}}}} \newcommand{\cmcircblk}[1]{\pgfmathparse{                                                                            %黑底白字 风格2     ifthenelse(#1 > 0 && #1 < 11, Hex(10101+#1),         ifthenelse(#1 > 10 && #1 < 21, Hex(9450-10+#1),             Hex(9471)         )     )     }{\CM{\symbol{"\pgfmathresult}}}} 复制代码 用法如圈5: \cmcirc{5}           %风格1 \cmcircblk{5}        %风格2 === 按8楼和10楼已做修改;何版在11楼给出“完整”风格圈,这里只提取我觉得最好看的最小代码。
kuing 6# 2013-4-20 23:34
5# isea 不是用 pgf 画的,那里只不过提供了一个对应关系,方便一点而已,事实上可以不用 pgf
kuing 7# 2013-4-21 00:09
Hex 是 pgf 的一个函数,将十进制数转化为十六进制,\symbol{"十六进制数} 就是引用字体里的某个字符,\pgfmathresult 是 pgf 的输出结果,然后用 ifthenelse 建立一个函数来对应…… 还是不扯太多了,自己理解下……
hnsredfox_007 8# 2013-4-21 09:06
5# isea 为什么我复制粘贴到导言区中,无法编译啊?我的是ctex.
hnsredfox_007 9# 2013-4-21 09:07
4# isea 这个可用!编译通过……
kuing 10# 2013-4-21 09:18
8# hnsredfox_007 他没帖完整,还要加tikz宏包,如果不是用ctex系列文档类或宏包的话还要加fontspec或xecjk。 要用xelatex编译。 如果要整到脚注,还要把脚注的某个参数重定义一下。
hejoseph 11# 2013-4-22 10:44
试了若干个带圈数字的字体,pifont的带圈数字里面的数字形状与默认无圈的数字形状很接近,但太小了,于是提取字体出来放大,做成CircledNum字体,若要使用安装一下就可以了。 pdf里是几种带圈数字及默认无圈的数字的对比。
kuing 12# 2013-4-22 12:55
其实能做,而且有闲情的话,不如整到99
hejoseph 13# 2013-4-22 12:59
unicode里只有0到20带圈数字,21-99都无对应unicode的,无通用性,所以懒得做,20个也足够了
isea 14# 2013-4-22 13:26
本帖最后由 isea 于 2013-4-22 14:00 编辑 安装附带字体,显示7;偶这里还少4和5的字体。 4和5的字体windows下有么?如果我下载安装一下。 ============= 第5种找到windows版了:http://sourceforge.jp/projects/sfnet_junicode/releases/ 维基百科,自由的百科全书 Junicode样字文本 Junicode(全称Junius-Unicode)是一款支援Unicode的复古风格衬线字体,由弗吉尼亚大学Peter S. Baker主持设计。它的设计灵感来自17世纪英国牛津所使用的一种字体。字体囊括了大量特殊字符及合字,以便于中世纪研究使用。具有OpenType特性。 Junicode最新版本为0.7.8,发布于2012年12月30日。 =================== 最后,如果你编译11楼许多错误,建议你把所以有关\CMU行用%标注掉,偶这里CTEX (basic)套装+win7 无法编译出效果来(如下图123数字字哪一行均空白),我想你也应该类似吧。 (即 \newfontfamily\CMU{CMU Serif} {\CMU{0123456789}} )
isea 15# 2013-4-22 14:09
本帖最后由 isea 于 2013-4-22 14:12 编辑 windows 一般用户把11楼(tex文件中的“文字”)代码改成这样,应该只有4个警 告,其他应该能一次性通过 \documentclass{article} \usepackage{fontspec} \usepackage{pifont} \usepackage{tikz} \newcommand{\dingcira}[1]{\pgfmathparse{ int(171+#1) }{\ding{\pgfmathresult}}} \newcommand{\dingcirb}[1]{\pgfmathparse{ int(181+#1) }{\ding{\pgfmathresult}}} \newcommand{\dingcirc}[1]{\pgfmathparse{ int(191+#1) }{\ding{\pgfmathresult}}} \newcommand{\dingcird}[1]{\pgfmathparse{ int(201+#1) }{\ding{\pgfmathresult}}} \newfontfamily\song{SimSun} \newcommand{\songcirc}[1]{\pgfmathparse{     ifthenelse(#1 > 0 && #1 < 21, Hex(9311+#1), Hex(9450))     }{\song{\symbol{"\pgfmathresult}}}} \newcommand{\songcircblk}[1]{\pgfmathparse{     ifthenelse(#1 > 0 && #1 < 11, Hex(10101+#1),         ifthenelse(#1 > 10 && #1 < 21, Hex(9450-10+#1),             Hex(9471)         )     )     }{\song{\symbol{"\pgfmathresult}}}} \newfontfamily\MG{MS Gothic} \newcommand{\mgcirc}[1]{\pgfmathparse{     ifthenelse(#1 > 0 && #1 < 21, Hex(9311+#1), Hex(9450))     }{\MG{\symbol{"\pgfmathresult}}}} \newfontfamily\CM{Cambria Math} \newcommand{\cmcirc}[1]{\pgfmathparse{     ifthenelse(#1 > 0 && #1 < 21, Hex(9311+#1), Hex(9450))     }{\CM{\symbol{"\pgfmathresult}}}} \newcommand{\cmcircblk}[1]{\pgfmathparse{     ifthenelse(#1 > 0 && #1 < 11, Hex(10101+#1),         ifthenelse(#1 > 10 && #1 < 21, Hex(9450-10+#1),             Hex(9471)         )     )     }{\CM{\symbol{"\pgfmathresult}}}} %\newfontfamily\LL{Linux Libertine O}                                                       %Linux字体,windows默认一定没有的 %\newcommand{\libcirc}[1]{\pgfmathparse{ %    ifthenelse(#1 > 0 && #1 < 21, Hex(9311+#1), Hex(9450) %    }{\LL{\symbol{"\pgfmathresult}}}} %\newcommand{\libcircdbl}[1]{\pgfmathparse{Hex(9460+#1)}{\LL{\symbol{"\pgfmathresult}}}} %\newcommand{\libcircblk}[1]{\pgfmathparse{ %    ifthenelse(#1 > 0 && #1 < 11, Hex(10101+#1), %        ifthenelse(#1 > 10 && #1 < 21, Hex(9450-10+#1), %            Hex(9471) %        ) %    ) %    }{\LL{\symbol{"\pgfmathresult}}}} %\newfontfamily\Junicode{Junicode}                                                                                       %Junicode字体,windows下一般默认也没有的 %\newcommand{\juncirc}[1]{{\fontspec[Ligatures=Discretionary]{Junicode}[#1]}} %\newcommand{\juncircdbl}[1]{{\fontspec[Ligatures=Discretionary]{Junicode}[[#1]]}} %\newcommand{\juncircblk}[1]{{\fontspec[Ligatures=Discretionary]{Junicode}<#1>}} \newfontfamily\CircledNum{CircledNum}                                               %安装11楼附带字体后就能正常了 \newcommand{\cncirc}[1]{\pgfmathparse{     ifthenelse(#1 > 0 && #1 < 21, Hex(9311+#1), Hex(9450))     }{\CircledNum{\symbol{"\pgfmathresult}}}} \newcommand{\cncircblk}[1]{\pgfmathparse{     ifthenelse(#1 > 0 && #1 < 11, Hex(10101+#1),         ifthenelse(#1 > 10 && #1 < 21, Hex(9450-10+#1),             Hex(9471)         )     )     }{\CircledNum{\symbol{"\pgfmathresult}}}} \renewcommand{\thefootnote}{\protect\cmcircblk{\value{footnote}}} \setlength{\parindent}{0pt} \begin{document} { \section{PiFont} 0123456789 \foreach \x in {1,...,10} {\dingcira{\x}} \foreach \x in {1,...,10} {\dingcirb{\x}} \foreach \x in {1,...,10} {\dingcirc{\x}} \foreach \x in {1,...,10} {\dingcird{\x}} } { \section{\song{宋体}} 0123456789 \foreach \x in {1,...,10} {\songcirc{\x}} } { \section{\MG{MS Gothic}} 0123456789 \foreach \x in {0,...,20} {\mgcirc{\x}} } %{ %\section{\LL{Linux Libertine}}                                      %Linux字体,windows默认一定没有的 % %0123456789 % %\foreach \x in {0,...,20} {\libcirc{\x}} % %\foreach \x in {1,...,10} {\libcircdbl{\x}} % %\foreach \x in {0,...,20} {\libcircblk{\x}} %} %{ %\section{\Junicode{Junicode}}                           %Junicode字体,windows下一般默认也没有的 % %0123456789 % %\foreach \x in {0,...,20} {\juncirc{\x}} % %\foreach \x in {1,...,10} {\juncircdbl{\x}} % %\foreach \x in {0,...,20} {\juncircblk{\x}} %} { \section{\CM{Cambria Math}} 0123456789 \foreach \x in {0,...,20} {\cmcirc{\x}} \foreach \x in {0,...,20} {\cmcircblk{\x}} } { \section{CircledNum}                     %安装11楼附带字体后就能正常了 0123456789 \foreach \x in {0,...,20} {\cncirc{\x}} \foreach \x in {0,...,20} {\cncircblk{\x}} } \end{document} 复制代码
hejoseph 16# 2013-4-22 14:12
本帖最后由 hejoseph 于 2013-4-22 15:35 编辑 CMU Serif、Linux Libertine、Junicode出问题是因为你还没更新宏包,你把宏包更新完之后就不会有问题了,我的机器也是windows系统
isea 17# 2013-4-22 23:47
CMU Serif、Linux Libertine、Junicode出问题是因为你还没更新宏包,你把宏包更新完之后就不会有问题了,我的机器也是windows系统 hejoseph 发表于 2013-4-22 14:12 升,果断 update MiKTeX updaeable packages
isea 18# 2013-4-23 23:33
安装附带字体,显示7;偶这里还少4和5的字体。 4和5的字体windows下有么?如果我下载安装一下。 ============= 第5种找到windows版了:http://sourceforge.jp/projects/sfnet_junicode/releases/ 维基百 ... isea 发表于 2013-4-22 13:26 ============================== 终于给我找到相应的字体宏包了 Linux Libertine 在最下的 uncategorized (未分类之意)中的 libertine 宏包  (第四种字体,这种字体是多数linux下默认字体,很赞)
isea 19# 2013-4-23 23:37
本帖最后由 isea 于 2013-4-23 23:40 编辑 代码中 \newfontfamily\CMU{CMU Serif} 中的 CMU Serif 是 fonts 类下的  outline fonts ——cm-unicode 宏包,推荐大家安装 同样的,在此分类中,能找到 Junicode 字体
isea 20# 2013-4-23 23:45
差不多,还有 LaTeX Font Warning: Font shape `EU1/SimSun(0)/bx/n' undefined (Font)              using `EU1/SimSun(0)/m/n' instead on input line 101. LaTeX Font Warning: Font shape `EU1/MSGothic(0)/bx/n' undefined (Font)              using `EU1/MSGothic(0)/m/n' instead on input line 109. ************************************************* * fontspec warning: "icu-feature-not-exist-in-font" * * OpenType feature 'Ligatures=Discretionary' (+dlig) not available for font * 'Junicode/B' with script 'Latin' and language 'Default'. ************************************************* ************************************************* * fontspec warning: "icu-feature-not-exist-in-font" * * OpenType feature 'Ligatures=Discretionary' (+dlig) not available for font * 'Junicode/B' with script 'Latin' and language 'Default'. ************************************************* ************************************************* * fontspec warning: "icu-feature-not-exist-in-font" * * OpenType feature 'Ligatures=Discretionary' (+dlig) not available for font * 'Junicode/BI' with script 'Latin' and language 'Default'. ************************************************* LaTeX Font Warning: Font shape `EU1/CambriaMath(0)/bx/n' undefined (Font)              using `EU1/CambriaMath(0)/m/n' instead on input line 141. [1] (C:\Users\iC\Desktop\CircledNum.aux) LaTeX Font Warning: Some font shapes were not available, defaults substituted. ) 这样的警告就无视了,不管了,至此,基本解决能完整编译出11楼
thread-1306-1-3.html: 这两天论坛有点卡,如果看贴时发现有的回贴不见了,请刷新
kuing 1# 2013-3-31 22:39
如题,若刷新数次仍不见,请在此回贴通知我。
isea 2# 2013-4-2 16:12
还真是卡,挂个电信代理才打得开帖子
kuing 3# 2013-4-2 16:14
2# isea 等不卡了得赶紧存档……
kuing 4# 2013-4-2 23:45
存档完毕……
abababa 5# 2013-4-3 14:31
我刚才才发现卡,原来已经有几天了啊。我就是电信的用户,也还是卡 是不是帖子有点多了啊,觉得应该把测试区的删除几个,还有灌水的,虽然对发帖的有点不公平 不过这是数学论坛,那些帖也确实没什么意义吧
kuing 6# 2013-4-3 15:11
5# abababa 跟这是无关的,我去5d6d官方论坛看过,整个TT组都卡。
kuing 7# 2013-4-5 21:17
顶一下
thread-1307-1-1.html: 昨晚的两个连乘极限(from 人教数学群)随意玩
kuing 1# 2013-4-1 10:28
之所以叫“随意玩”,因为其实我还没什么基础,只是印象中记得一个展开式,也不记得怎么推出来的,也不知能不能用于复数,以及后面的能不能那样乘起来…… 以下过程当时都只在群里发,本来没打算整理上来,因为怕错,但后来看聊天记录似乎说是对的(当然我还是不知道为什么),所以还是决定发上来 入正题,话说昨晚人教群里出现这样一题 爱好者-烟头(2511*****) 22:56:22 给点思路啊 然后聊到收敛问题,接着有人提到求极限,当时我还没怎么理,因为我预计可能没有初等表达式。 然而,当我看到 爱好者-刀客成(8065*****) 23:04:38 精确边界 虽然这显然是Mathematica给出的结果,但是这结果是出乎我意料的,于是我开始认真看了。 突然间,我想起以前见过这样一个展开式 \[\sin(\pi x)=\pi x\prod_{k=1}^{\infty}\left( 1-\frac{x^2}{k^2} \right).\] 但是如果要用于本题,显然差了个符号,那岂不是要令 $x=i$ 才能将减变成加?我并不知道可不可以用于复数,但是也没管那么多,试试会如何。 在展开式中令 $x=i$,得 \[\sin(\pi i)=\pi i\prod_{k=1}^{\infty}\left( 1+\frac1{k^2} \right),\] 由欧拉公式,上式又等价于 \[\frac{i(e^{\pi}-e^{-\pi})}2=2\pi i\prod_{k=2}^{\infty}\left( 1+\frac1{k^2} \right),\] 于是 \[\prod_{k=2}^{\infty}\left( 1+\frac1{k^2} \right)=\frac{(e^{\pi}-e^{-\pi})}{4\pi}.\] 哈,居然就出来了,然后也没管那么多就发到群里了。秋风说 爱好者秋风树林(5221*****) 23:23:39 其实复变里面的三角函数就是从实轴上延拓过去的 感觉上应该行,没严格证过 爱好者秋风树林(5221*****) 23:32:09 其实按全纯函数的唯一性定理和级数的收敛半径来看,应该可以说描述的就是同一个东西,kk的那种方法也许是行得通的 其间,其妙又抛了这个出来 教师-其妙/ka(2360****) 23:24:59 证明; 沿着上面的思路,继续来一个,也不知道接下来的相乘是不是严格的。 仍然由那个展开式,分别令 $x=(i+1)/\sqrt2$ 以及 $x=(i-1)/\sqrt2$ 再两式相乘,得 \[\sin\frac{(i+1)\pi}{\sqrt2}\sin\frac{(i-1)\pi}{\sqrt2}=-\pi^2\prod_{k=1}^{\infty}\left( 1-\frac{(i+1)^2}{2k^2} \right)\left( 1-\frac{(i-1)^2}{2k^2} \right),\] 容易验证 \[\left( 1-\frac{(i+1)^2}{2k^2} \right)\left( 1-\frac{(i-1)^2}{2k^2} \right)=1+\frac1{k^4},\] 由积化和差以及欧拉公式,有 \[\sin\frac{(i+1)\pi}{\sqrt2}\sin\frac{(i-1)\pi}{\sqrt2}=\frac12\Bigl( \cos\bigl(\sqrt2\pi\bigr)-\cos\bigl(\sqrt2i\pi\bigr) \Bigr)=\frac12\cos\bigl(\sqrt2\pi\bigr) -\frac{e^{\sqrt2\pi}+e^{-\sqrt2\pi}}4,\] 于是 \[\prod_{k=1}^{\infty}{\left( 1+\frac1{k^4} \right)}=\frac{e^{\sqrt2\pi}+e^{-\sqrt2\pi}-2\cos\bigl(\sqrt2\pi\bigr)}{4\pi^2},\] 即 \[\prod_{k=2}^{\infty}{\left( 1+\frac1{k^4} \right)}=\frac{e^{\sqrt2\pi}+e^{-\sqrt2\pi}-2\cos\bigl(\sqrt2\pi\bigr)}{8\pi^2}.\] 用软件验证了一下,结果相同,于是也没管三七二十一就发到群里。其妙说 教师-其妙/ka(2360****) 23:46:27 复数是对的 就是秋风说的复变函数续论里的 复变函数续论里有无穷乘积 ……
kuing 2# 2013-4-1 10:39
话说,如果分母换成三次方就不知怎么玩了,但用软件还是能给出结果……
kuing 3# 2013-4-1 20:58
不知有没有一般次数的公式?
yes94 4# 2013-4-2 19:31
《复变函数续论》里找到了kuing的那个结论,该结论在该书P105例二: 《复变函数续论》下载地址:http://ishare.iask.sina.com.cn/d ... php?fileid=10427534
yes94 5# 2013-4-2 19:35
4# yes94 有意思的是那里还定义了正对数的概念,好像是某个抽象函数的实例,这个抽象函数在中学几乎举不出实例出来:
秋风树林 6# 2013-4-2 22:39
《复变函数续论》里找到了kuing的那个结论,该结论在该书P105例二: 11951195 《复变函数续论》下载地址:http://ishare.iask.sina.com.cn/d ... php?fileid=10427534 yes94 发表于 2013-4-2 19:31 又见复旦的....... 怪不得我说为什么我们上学期那本复变这么变 tai....... 张锦豪编的那本也绝对不是省油的灯...
pxchg1200 7# 2013-4-3 02:20
2# kuing 按照约定,我来搞下这个无穷乘积。 \[ S=\prod_{n=1}^{\infty}{\left(1+\frac{1}{n^{3}}\right)}\] 注意到 \begin{align}   S&=\prod_{n=1}^{\infty}{\left(1+\frac{1}{n^{3}}\right)}\\   &=\prod_{n=1}^{\infty}{\left[\frac{(n+1)(n^2-n+1)}{n^3}\right]}\\   &=\prod_{n=1}^{\infty}{\left[\frac{(n+1)(n+x_{1})(n+x_{2})}{n^3}\right]} \end{align} 其中$x_{1}=-\frac{1+\sqrt{3}i}{2},x_{2}=-\frac{1-\sqrt{3}i}{2}$,由韦达定理知$x_{1}+x_{2}=-1,x_{1}\cdot x_{2}=1$. 现在,考虑Gamma函数的无穷乘积 \begin{align} \Gamma{(x)}&=\frac{1}{x}\prod_{n=1}^{\infty}{\frac{\left(1+\frac{1}{n}\right)^{x}}{1+\frac{x}{n}}}\\ &=\lim_{n\rightarrow\infty}{\frac{n!n^{x}}{x(x+1)(x+2)\cdots(x+n)}} \end{align} 则有 \[ \Gamma{(x_{1})}\Gamma{(x_{2})}=\lim_{n\rightarrow\infty}{\frac{(n!)^{2}n^{x_{1}+x_{2}}}{x_{1}x_{2}(x_{1}+1)(x_{2}+1)\cdots(x_{1}+n)(x_{2}+n)}} \] 故 \begin{align}   S&=\prod_{n=1}^{\infty}{\left[\frac{(n+1)(n+x_{1})(n+x_{2})}{n^3}\right]}\\   &=\lim_{n\rightarrow\infty}{\frac{(n!)^{2}(n+1)!}{(n!)^{3}\cdot n\cdot\Gamma{(x_{1})}\Gamma{(x_{1})}}}\\   &=\frac{1}{\Gamma(x_{2})\Gamma(x_{1})} \end{align} 注意到 \begin{align}   \Gamma{(x_{1}+1)}&=x_{1}\Gamma{(x_{1})}\\    \Gamma{(x_{2}+1)}&=x_{1}\Gamma{(x_{2})}   \end{align} 故有 \[ \Gamma{(x_{1}+1)}\Gamma{(x_{2}+1)}=\Gamma{(x_{1})}\Gamma{(x_{2})}\] 剩下的由余元公式就可以得出。
yes94 8# 2013-4-4 14:17
7# pxchg1200 都牛!
wwdwwd2013 9# 2013-5-29 17:34
原题可以这么来,
kuing 10# 2013-5-29 17:42
9# wwdwwd2013 证得不错! PS、“因为”后面第二行后面打少了一个(1+1/n)
isea 11# 2013-5-29 20:32
本帖最后由 isea 于 2013-5-29 20:35 编辑 原题可以这么来, wwdwwd2013 发表于 2013-5-29 17:34 精彩! 我肯定见过,这个式子,但这种放缩,学习了
isea 12# 2013-5-29 20:34
10# kuing 哪儿来的个“神”人wwdwwd2013…… 太犀利了
yayaweha 13# 2013-5-29 23:16
本帖最后由 yayaweha 于 2013-5-29 23:18 编辑 $$\prod_{k=2}^{n} (1+\frac{1}{ k^2})=\prod_{k=2}^{n}(\frac{k^2+1}{k^2})<2$$ $$\iff  \prod_{k=2}^{n} (\frac{k^2}{1+k^2})=\prod_{k=2}^{n} (1-\frac{1}{k^2+1})>\frac{1}{2}$$ $$\prod_{k=2}^{n} (1-\frac{1}{k^2+1})>1-\sum_{k=2}^{n}\frac{1}{1+k^2}$$这样下去能否证出来?
thread-1308-1-3.html: [不等式] 请高手帮忙证明这个不等式,十分感谢!
大程 1# 2013-4-1 16:42
本帖最后由 大程 于 2013-4-1 16:48 编辑 三角形ABC的三边长分别为a,b,c,f(λ)=a/(λa+b+c)+b/(λb+a+c)+c/(λc+a+b)证明:(1)当-1<λ<1时,3/(λ+2)<=f(λ)<2/(λ+1) (2)当λ>1时,2/(λ+1)<f(λ)<=3/(λ+2)
kuing 2# 2013-4-1 17:14
三角形ABC的三边长分别为a,b,c,f(λ)=a/(λa+b+c)+b/(λb+a+c)+c/(λc+a+b)证明:(1)当-1<λ<1时,3/(λ+2)<=f(λ)<2/(λ+1) (2)当λ>1时,2/(λ+1)<f(λ)<=3/(λ+2) 大程 发表于 2013-4-1 16:42 令 $a=y+z$, $b=z+x$, $c=x+y$, $x$, $y$, $z>0$,则 \[f(k)=\sum \frac{y+z}{2x+(k+1)(y+z)}.\] 当 $-1<k<1$,则 \[f(k)<\sum \frac{y+z}{(k+1)x+(k+1)(y+z)}=\frac2{k+1},\] 由柯西不等式及均值不等式,有 \begin{align*} f(k)&\geqslant \frac{\left( \sum (y+z) \right)^2}{\sum (y+z)(2x+(k+1)(y+z))} \\ & =\frac{2\left( \sum x \right)^2}{(k+1)\left( \sum x \right)^2+(1-k)\sum xy} \\ & \geqslant \frac{2\left( \sum x \right)^2}{(k+1)\left( \sum x \right)^2+(1-k)\frac{\left( \sum x \right)^2}3} \\ & =\frac3{k+2}; \end{align*} 当 $k>1$,则 \[f(k)>\sum \frac{y+z}{(k+1)x+(k+1)(y+z)}=\frac2{k+1},\] 由柯西不等式及均值不等式,有 \begin{align*} f(k)&=\frac1{k+1}\sum \left( 1-\frac{2x}{2x+(k+1)(y+z)} \right) \\ & =\frac3{k+1}-\frac2{k+1}\sum \frac x{2x+(k+1)(y+z)} \\ & \leqslant \frac3{k+1}-\frac2{k+1}\cdot \frac{\left( \sum x \right)^2}{\sum x(2x+(k+1)(y+z))} \\ & =\frac3{k+1}-\frac1{k+1}\cdot \frac{\left( \sum x \right)^2}{\left( \sum x \right)^2+(k-1)\sum xy} \\ & \leqslant \frac3{k+1}-\frac1{k+1}\cdot \frac{\left( \sum x \right)^2}{\left( \sum x \right)^2+(k-1)\frac{\left( \sum x \right)^2}3} \\ & =\frac3{k+2}. \end{align*}
yes94 3# 2013-4-1 21:12
搞内切圆代换?
kuing 4# 2013-4-1 21:29
3# yes94 最直接…… 你也可以试试琴生、切线之类的……这种题适合你玩方法dang
yes94 5# 2013-4-1 22:54
4# kuing 这个题还是算了吧,看着有些怕哟,
kuing 6# 2013-4-1 23:03
5# yes94 有什么怕的……强展也不过三次齐次完全对称……怎么玩都行
yes94 7# 2013-4-2 12:25
6# kuing 如果这个条件“三角形$ABC$的三边长分别为$a,b,c,$”没有的话,还敢做一做……
kuing 8# 2013-4-2 12:28
7# yes94 内切圆代换不就去掉了么……
yes94 9# 2013-4-2 13:43
本帖最后由 yes94 于 2013-4-2 13:50 编辑 改一下条件,只证明一下部分结果。看看对不对? 当$-2<λ<0$时,\begin{align*} f(λ)&=\dfrac a{λa+b+c}+\dfrac b{λb+a+c}+\dfrac c{λc+a+b}\\ &=\dfrac{a^2}{λa^2+ab+ac}+\dfrac{b^2}{λb^2+ab+bc}+\dfrac{c^2}{λc^2+ac+bc}\\ &\geqslant\dfrac{(a+b+c)^2}{λ(a^2+b^2+c^2)+2(ab+bc+ac)}\\ &\geqslant\dfrac{(a+b+c)^2}{λ(ab+bc+ac)+2(ab+bc+ac)}\\ &=\dfrac{(a+b+c)^2}{(λ+2)(ab+bc+ac)}\\ &\geqslant\dfrac{3}{λ+2} \end{align*} 对不对啊? 说明:原题条件为$-1<λ<1$,现在改为$-2<λ<0$,左边拓展到了$-2$,右边却缩小到0.
yes94 10# 2013-4-2 13:48
9# yes94 如果$-2<λ<0$,则需加条件分母$λa+b+c>0$等等,而由三角形,当$λ>-1$时,正好得到$λa+b+c>b+c-a>0$,保证分母为正数,所以$λ$不可拓展到-2附近。
thread-131-1-9.html: 几何概率---呵呵,我的娱乐第一帖
yuzi 1# 2011-10-22 20:36
本帖最后由 yuzi 于 2011-10-22 20:49 编辑 已知直线L过点$\left ( -1,0 \right )$,且与圆$C:\left ( x-1 \right )^{2}+y^{2}=3$相交于A、B两点,则弦长$\left | AB \right |\geqslant 2$的概率是多少?
yuzi 2# 2011-10-22 20:52
忘了怎么发帖了。。。
kuing 3# 2011-10-22 21:08
2# yuzi ?现在不是发出来了么 这个题,哪种东西等可能啊? 不交待清楚我怕又遇什么奇论了
明松 4# 2011-10-22 21:20
关注中,kuling,题目所给答案为根号3/3,是用斜率做的,给个说法看看
kuing 5# 2011-10-22 21:21
4# 明松 打多了个 l 。 这个得先交待清楚我3#所说的
nash 6# 2011-10-22 21:22
这个题 有点纠结 答案是用弦长直接上?
nash 7# 2011-10-22 21:25
4# 明松 估计是弦长啦 弦长和斜率不成正比,这个貌似凑巧啦
明松 8# 2011-10-22 21:26
原题就是这样的,我们搞了一会,yuzi说发来给你看看
明松 9# 2011-10-22 21:28
本帖最后由 明松 于 2011-10-22 21:30 编辑 你的意思也就是说可以用面积处理了? 上面的记楼怎么搞出来的?
kuing 10# 2011-10-22 21:38
不交待清楚等可能性,这种题就没法做,跟那个“贝特朗奇论”一样,参考: http://baike.baidu.com/view/3477154.htm http://wenku.baidu.com/view/83c99bd084254b35eefd34cc.html 等等
明松 11# 2011-10-22 21:44
我知道你所说的这个悖论,但是这个题中有个信息把我搞糊涂了,就是直线过定点
nash 12# 2011-10-22 21:48

kuing 13# 2011-10-22 21:49
11# 明松     也类似,比如可以用这几种等可能假设: 1、直线在斜率上均匀分布; 2、直线在倾斜角上均匀分布; 3、直线在到圆心的距离上均匀分布; 4、直线在被圆所的弦长上均匀分布; 5、直线在分割圆所成面积比上均匀分布; ……
明松 14# 2011-10-22 21:51
哦,谢谢,再研究一下,再问,回复的时候楼记怎么搞出来的
kuing 15# 2011-10-22 21:52
14# 明松 点击该楼层左下角的“回复”,小的那个。
GAM 16# 2011-10-22 22:29
长知识了
thread-132-1-1.html: ln 与 In
kuing 1# 2011-10-22 22:39
好奇怪很多人将自然对数打成 In,学生也就算了,连老师都这样。。。
图图 2# 2011-10-23 19:17
$\ln x\ne Inx$
kuing 3# 2011-10-23 21:24
2# 图图
pxchg1200 4# 2011-10-24 14:45
我还看到有 $ Ln x$ 的。。。。
kuing 5# 2011-10-24 14:46
4# pxchg1200 这个还好一点,至少字母算是用对了,只是大小写的问题
pxchg1200 6# 2011-10-24 23:12
5# kuing 这个意思不同的,是复变函数里的符号,考虑的是个多值函数,\ln x 考虑的是主值。。。
kuing 7# 2011-10-24 23:14
6# pxchg1200 噢,有点印象,虽然没正式学过复变,但隐约看过一下
①②③④⑤⑥⑦ 8# 2011-10-25 10:05
Modified Bessel Function of the First Kind   不过n放在下标位置
①②③④⑤⑥⑦ 9# 2011-10-25 10:08
话说,ln到底该怎么念倒是没有见过这方面的规定
kuing 10# 2011-10-25 10:49
8# ①②③④⑤⑥⑦ 嗯?这个是什么?
kuing 11# 2011-10-25 10:49
9# ①②③④⑤⑥⑦ 对哟,我也一直不知道怎么读……
isea 12# 2011-10-25 11:28
ln lg 偶都读log哈哈 复变中的Ln,第一次到时,觉得符号太神奇了。
kuing 13# 2011-10-25 11:32
12# isea 神奇?
isea 14# 2011-10-25 11:45
13# kuing 从l到L就一对多了。。。。。。
kuing 15# 2011-10-25 12:21
14# isea 呃。。。
叶剑飞Victor 16# 2013-3-10 20:28
9# ①②③④⑤⑥⑦ 直接读做“natural logarithm”就行了。
kuing 17# 2013-3-10 20:30
16# 叶剑飞Victor 这么长……
叶剑飞Victor 18# 2013-3-10 20:31
17# kuing 嫌长就读“自然对数”。
kuing 19# 2013-3-10 20:35
还是习惯读“lin”
thread-1320-1-1.html: 回复352的一道集合计数
kuing 1# 2013-4-2 14:04
题目:设集合 $S_n=\{1,2,3,\ldots,n\}$,若 $X \subseteq S_n$,把 $X$ 的所有元素的乘积称为 $X$ 的容量(若 $X$ 中只有一个元素,则该元素的数值即为它的容量,规定空集的容量为 $0$)。若 $X$ 的容量为奇(偶)数,则称 $X$ 为 $S_n$ 的奇(偶)子集。则 $S_{2n}$($n\in\mbb N^+$)的所有奇子集的容量之和为______。 不难发现 $(x+1)(x+3)(x+5)\cdots(x+2n-1)$ 的展开式中,除去最高次项 $x^n$ 外,各项的系数……与那些各类容量的和……实在不知怎么说,请自行理解…… 反正所求的结果就是 $(x+1)(x+3)(x+5)\cdots(x+2n-1)-x^n$ 的系数之和,所以令 $x=1$,得到答案便是 $2^nn!-1$。
kuing 2# 2013-4-2 14:23
如果不理解上面的,也可以用递推去解。 记 $S_{2n}$ 时所求的和为 $f(n)$,那么当 $S_{2(n+1)}$ 时,多了一个元素 $2n+1$ 的选择,多出来的和就是 $2n+1$ 与前面那些奇数的容量和的积,以及它自己(仍然不知怎么表达得更清楚),所以有递推式 \[f(n+1)=f(n)+(2n+1)f(n)+2n+1,\] 变形为 \[f(n+1)+1=(2n+2)(f(n)+1),\] 这样也可以求出楼上的答案。
yes94 3# 2013-4-2 17:51
2# kuing 递推计数又一成功范例! 并且,母函数也派上用场,了不起!
李斌斌755 4# 2013-4-2 19:52
还是2#容易理解
yes94 5# 2013-4-2 21:00
4# 李斌斌755 1楼在哪?
李斌斌755 6# 2013-4-2 22:59
5# yes94 1#仍是kuing
kuing 7# 2013-4-2 23:15
5# yes94 可能刚才你看的时候卡掉了1#吧…… 我发现夜深就不卡了,可能是服务器不太行,支持不住白天的浏览量……
isea 8# 2013-4-6 00:01
集合 相关 mark
kuing 9# 2013-5-13 12:51
刚在人教论坛看到一道类似的: http://bbs.pep.com.cn/forum.php?mod=viewthread&tid=2758890 方法应该一样
isea 10# 2013-5-13 17:06
本帖最后由 isea 于 2013-5-13 17:13 编辑 9# kuing 好巧好巧,偶笨得紧啦。 只能验证k是对的(似懂非懂,这个表达式就是题目的意思,数学真是精练之极) $$f(x)=(x+1)(x+2^2-1)(x+2^3-1)\cdots(x+2^n-1)-x^n,\\f(1)=2\cdot 2^2 \cdots 2^n-1\\=2^{\frac{n(n+1)}2}-1$$
kuing 11# 2013-5-13 17:09
$$f(x)=(x+1)(x+2^2-1)(x+2^3-1)\cdots(x+2^n-1)-x^n,\\f(1)=2\cdot 2^2 \cdots 2^n-1\\=2^{\dfrac{n(n+1)}2-1}$$ isea 发表于 2013-5-13 17:06 不要滥用 \dfrac
isea 12# 2013-5-13 17:13
不鸟你
kuing 13# 2013-5-13 17:15
你不觉得上下标用 \dfrac 后根本不像上下标?$2^{\frac12}\ne2^{\dfrac1{2}}$
isea 14# 2013-5-13 17:17
13# kuing 改改
kuing 15# 2013-5-13 17:25
其实 a/b 这样的形式也不差啊,如果是我就会写成 $2^{n(n+1)/2}$ $2^{\dfrac12}+2^{\frac12}+2^{1/2}$
kuing 16# 2013-5-13 17:39
呵呵,不扯输入了,想想怎么给人教那边的解释吧,我是不会表达的了。
yes94 17# 2013-5-15 23:28
根据n个括号相乘,括号里元素的选取就知道意义啦!
李斌斌755 18# 2013-5-17 00:16
感觉展开式很诡异。明明是一堆数相乘,却变成一堆数相加。类似“积化和差”!
kuing 19# 2013-5-17 00:20
……
李斌斌755 20# 2013-5-17 00:59
19# kuing 我的水平只能这样理解
thread-1321-1-1.html: 博客中如何运行代码
reny 1# 2013-4-2 15:34
看到别人在新浪博客中运行了Latex代码,那是怎么弄的啊? 搞不懂。
kuing 2# 2013-4-2 15:42
用短消息将链接发一下给我,我去看看
kuing 3# 2013-4-2 19:33
看了,那个其实是图片,而且图片上含有链接和标题,是复制mathlinks的,并不是在该博客上生成。
kuing 4# 2013-4-2 19:35
你可以右键点击那些公式,查看属性就可以看出是链接自mathlinks。
reny 5# 2013-4-2 22:31
哦,说明在一般的博客上还是不能运行Latex吧,只好在英文网站上复制啦
kuing 6# 2013-4-2 22:54
要是支持在 <head> 中自己添加东西的就可以像我这里那样用 MathJax,像那个 is-programmer 的 blog 那样。 也有支持 latex 的,像 wordpress,但好像要fanqiang……
thread-1322-1-1.html: 怎么让数学符号变成粗体、黑体或加颜色呢?
abababa 1# 2013-4-3 13:30
比如数学符号$\sin x, \cos x, \ln x$,这里的sin,cos, ln和公式都在一起,想让它们都变成黑体或者加上颜色显示得更清楚,应该怎么做呢? 最好不是一个一个的重定义,而是一次把所有这类符号都弄好。
kuing 2# 2013-4-3 14:09
只改变 sin 不改变 x ?
isea 3# 2013-4-3 14:17
楼主的意思应该是全部黑
abababa 4# 2013-4-3 14:23
不是全部变黑,就是#2的意思,只把sin变黑,x不变黑,或者加颜色也行
isea 5# 2013-4-3 14:46
不会了,我只会全部变 \boldmath 公式全部黑体 \color{blue} 全部变色,若只放\$间,这一段变 楼下继续
isea 6# 2013-4-3 14:57
笨方法,将\sin 之类全部替换成 \boldsymbol\sin
abababa 7# 2013-4-3 15:23
6# isea 单个的新的到是会弄,像\DeclareMathOperator{\lcm}{\bf{lcm}}这样就把\lcm变黑了 要是已经定义好的,比如\sin,我就不会弄了,而且就算会,也是一个一个重新定义,那么多符号也挺麻烦的,不知道有没有直接变黑的
kuing 8# 2013-4-3 15:49
有点麻烦,可能要修改 amsopn 里面的那些相关定义,内部命令不太懂……
kuing 9# 2013-4-3 16:08
找到相关部分了,猜着意思,大概在导言区添加这样一段 \makeatletter \DeclareRobustCommand{\qopname}[3]{%   \mathop{#1\kern\z@\operator@font{\color{red}{#3}}}%   \csname n#2limits@\endcsname} \makeatother 复制代码 就会变红色。 要换其他,修改 \operator@font 后面就行了,原先那里是   \mathop{#1\kern\z@\operator@font#3}%
abababa 10# 2013-4-3 21:43
9# kuing 谢谢。试了下,没编译过去,少什么包吗?还是我写错了? 提示这些: ! Undefined control sequence. \qopname  ... {#1\kern \z@ \operator@font {\color                                                   {red}{#3}}}\csname n#2limi... l.11 $\sin            x, \cos x, \tan x, \ln x$ 我的代码: \documentclass[11pt]{article} \usepackage{amssymb,amsmath,amsthm} \makeatletter \DeclareRobustCommand{\qopname}[3]{% \mathop{#1\kern\z@\operator@font{\color{red}{#3}}}%   \csname n#2limits@\endcsname} \makeatother \begin{document} $\sin x, \cos x, \tan x, \ln x$ \end{document} 复制代码
kuing 11# 2013-4-3 21:49
用了颜色,可能要加 color 宏包,或者 xcolor 宏包。
abababa 12# 2013-4-3 21:56
11# kuing 果然是,谢谢。先用上,虽然还不知道原理。
kuing 13# 2013-4-3 21:59
12# abababa 我也不清楚,全是内部命令…… 暂时还不清楚那样改了会不会有什么“副作用”……
abababa 14# 2013-4-3 22:06
13# kuing 以前一看#1,#2这些就发傻,现在知道点了,是参数的意思,用的时候写什么东西会在里面被替换,现在看圈a又发傻,不懂什么意思。
kuing 15# 2013-4-3 22:13
14# abababa 大概是内部命令才用 @ 这个符号,在内部的时候,@ 就像一般的字母那样。 现在要拿内部的东西出来改,而平常的 @ 有别的意思,所以要先加 \makeatletter,使之变回内部那样,改完后再 \makeatother 恢复正常。
abababa 16# 2013-4-3 22:22
15# kuing 就是说,\operator@font,这整个一长串是一个命令吧?
kuing 17# 2013-4-3 22:26
16# abababa 嗯,如果我没搞错的话。
abababa 18# 2013-4-3 22:52
17# kuing 现在除了那些命令的意义不懂,还有一个是\csname n#2limits@,这一串,中间n前面有个空格,不知是什么意思 按理说应该是\csname是一个命令,和后面那个\endcsname是一起的,中间是n#2limits@,那个#2是要被替换的第二个参数,但这样n和limits@都不带前面的反斜线,应该几是个普通字符而不是命令吧?
kuing 19# 2013-4-3 23:09
\csname 和 \endcsname 貌似就是将中间的字符变成相应的命令,例如 \csname TeX\endcsname 相当于 \TeX 。 之所以要这样做,我想大概就是因为还要把第二个参数代入,就是说如果直接用 \n#2limits@ 估计就会在 # 那里断开了。 而这样做其实只是为了方便后面使用,我看了那段后面,第二个参数不是 o 就是 m,比如 \def\lg{\qopname\relax o{lg}} \def\lim{\qopname\relax m{lim}} 等等,也就是用一个字母来得到不同的类型,\nolimits@ 和 \nmlimits@ ,这两个大概就是不同的上下标设置。
abababa 20# 2013-4-3 23:30
19# kuing 谢谢,经过解释已经看懂了那些字符在写法上的意义,虽然有些命令的意义还不懂,先放一放。 刚才尝试写个\csname没成功,可能是哪里有问题吧,暂时也不打算再弄了,对我来说目前已经满足了。 \newcommand{\mytex}[1]{\csname #1 \endcsname} \mytex{TeX}
thread-1322-2-1.html:
kuing 21# 2013-4-3 23:59
csname 似乎对空格也是有反应的……于是将 #1 后面的空格去掉试试。 PS、以上这些我也是猜着的,自己并没做什么试验,可能还有不准确的地方,或者有些细节还不清楚……还要看的东西实在太多了……
abababa 22# 2013-4-4 09:01
21# kuing 刚试了下不加空格,还真是的,和\endcsname间有空格就不行,确实神奇,呵呵,感谢
kuing 23# 2013-4-4 11:37
22# abababa 8K7 学到新招了吧,虽然暂时应该用不着
abababa 24# 2013-4-4 13:36
23# kuing 确实还没感觉到哪里要用,应该是更基础的层次用的命令吧,像我这样只是写写公式一般用不到,不过学学也好,多知道点
thread-1324-1-3.html: [不等式] 摘两个不等式
reny 1# 2013-4-3 13:32
本帖最后由 reny 于 2013-4-3 15:51 编辑 (1)已知$a,b,c是大于1的实数,且a+b+c=9,$求证$$\sqrt a+\sqrt b+\sqrt c \geqslant \sqrt{ab+bc+ca}$$ ,有木有其它方法 摘自http://blog.sina.com.cn/s/blog_bf493a5c0101i26n.html (2)设$a,b,c>0,且\sqrt a+\sqrt b+\sqrt c \geqslant 3,$求证$$\sqrt{a^2+1}+\sqrt {b^2+1}+\sqrt {c^2+1}\leqslant \sqrt 2(a+b+c)$$,感觉这个不等式好特别啊 摘自安振平的博客http://blog.sina.com.cn/s/blog_5618e6650101jmz5.html
yes94 2# 2013-4-3 18:16
1# reny reny是学生啊?这两个问题有些相似,却远隔千山,但reny捕捉到了,善于总结,
kuing 3# 2013-4-4 12:58
第二题用切线法应该可以
pxchg1200 4# 2013-4-4 13:10
第一题作$a=x^2,b=y^2,c=z^2$后可以变成 \[ \sum{x^4}+2\sum{x^3(y+z)}+2\sum{x^2yz}\geq 7\sum{x^2y^2} \] 然后用4次配方的东东。
pxchg1200 5# 2013-4-4 13:21
3# kuing 第二个好像可以AM-GM Here is my proof. 作$ a=x^2,b=y^2,c=z^2 $ 我们有 \[ x+y+z\geq 3 \] 不等式变成 \[ \sqrt{x^4+1}+\sqrt{y^4+1}+\sqrt{z^4+1}\leq \sqrt{2}(x^2+y^2+z^2) \] 而 \[ x^4+1=(x^2+\sqrt{2}x+1)(x^2-\sqrt{2}x+1) \] 由AM-GM \[ 2\sqrt{2(x^4+1)}=2\sqrt{(2-\sqrt{2})(x^2+\sqrt{2}x+1)\cdot(2+\sqrt{2})(x^2-\sqrt{2}x+1)}\leq 4x^2-4x+4 \] 所以 \[ 2\sum{\sqrt{2(x^4+1)}}\leq 4\sum{x^2}-4\sum{x}+12\leq 4\sum{x^2} \] 上式成立因为\[ x+y+z\geq 3 \] Done
kuing 6# 2013-4-4 15:01
第一题调整也可以 (1)已知 $a$, $b$, $c$ 是大于 $1$ 的实数,且 $a+b+c=9$,求证 \[\sqrt a+\sqrt b+\sqrt c \geqslant \sqrt{ab+bc+ca}.\] 不妨设 $c=\min \{a,b,c\}$,则 $c\in (1,3]$,记 $t=\sqrt{ab}$,则有 $t\leqslant(a+b)/2=(9-c)2<4$ 且 $t>\sqrt{c(a+b-c)}=\sqrt{c(9-2c)}>\sqrt7$。原不等式等价于 \[f(t)=\sqrt{9-c+2t}+\sqrt c-\sqrt{t^2+c(9-c)}\geqslant 0,\] 求导得 \[f'(t)=\frac1{\sqrt{9-c+2t}}-\frac t{\sqrt{t^2+c(9-c)}},\] 由 \[t^2+c(9-c)-t^2(9-c+2t)=-c^2+(t^2+9)c-2t^3-8t^2,\] 上式关于 $c$ 的判别式为 \[\Delta =t^4-8t^3-14t^2+81=-(t-2)^2(34+4t-t^2)-120(t-2)-23<0,\] 从而得到 $f'(t)<0$,所以 \[f(t)\geqslant f\left( \frac{9-c}2 \right)=\sqrt{2(9-c)}+\sqrt c-\frac12\sqrt{3(9-c)(c+3)},\] 故只需证 \[2\sqrt{2(9-c)}+2\sqrt c\geqslant \sqrt{3(9-c)(c+3)},\] 平方整理等价于 \[8\sqrt{2c(9-c)}\geqslant -3c^2+22c+9,\] 由 $c\in(1,3]$ 易知右边为正,故再两边平方整理等价于 \[3(c-3)^2(26c-3c^2-3)\geqslant 0,\] 由 $c\in(1,3]$ 易知成立,从而原不等式得证。
reny 7# 2013-4-4 20:36
本帖最后由 reny 于 2013-4-4 20:41 编辑 4# pxchg1200 刚才在韩京俊那本书第五章找到了第一题的2种解法,很多时候不知道配方怎么配的,只是感觉妙。
reny 8# 2013-4-4 20:38
2# yes94 的确是大四学生,现在没什么事,处于毕业状态,也快要打酱油啦.
reny 9# 2013-4-4 20:54
本帖最后由 reny 于 2013-4-4 21:25 编辑 5# pxchg1200 漂亮简洁的解答啊,用$AM—GM时注意取等条件就ok$ 也学习了调整法,就是有点难算啊
thread-1325-1-3.html: [不等式] 这是均值法吗?
yayaweha 1# 2013-4-4 11:15

yayaweha 2# 2013-4-4 11:18
解答
yayaweha 3# 2013-4-4 11:20
解答中证明$$\abs{ A_n-A_k}<1-\frac{k}{n}$$ 是什么原理,是均值法吗?
kuing 4# 2013-4-4 13:08
什么叫“均值法”? 2#的推理写得很清楚,有什么疑问吗?
yayaweha 5# 2013-4-4 16:13
4# kuing 为什么这么一分组最后能得到两个数组最大值相同
李斌斌755 6# 2013-4-4 18:39
楼不见了
thread-1326-1-1.html: 问一个简单的积分题
abababa 1# 2013-4-4 13:33
本帖最后由 abababa 于 2013-4-4 13:38 编辑 求积分$\int\frac{1}{1+\sqrt{x^2+2x+2}}dx$ 我用三角代换,令$x+1=\tan t$,然后得到$\int\frac{1}{\cos^2 t+\cos t}dt$ 到这一步以后,我用万能公式做代换,令$s=\tan\frac{t}{2}$,最后还要解$s,x$的关系,结果解到的是二次方程$\frac{2s}{1-s^2}=x+1$,积出$s$的表达式后反代回$x$,那个方程的$\Delta$要加减,只有一个是结果 我是不是哪弄复杂了?感觉万能代换就有点复杂
yes94 2# 2013-4-4 14:16
1# abababa 可能是有点复杂:
thread-1327-1-3.html: [不等式] 不等式
huyuhbb 1# 2013-4-4 22:27
新人表示不会打代码...so sorry
pxchg1200 2# 2013-4-4 22:50
1# huyuhbb 这个问题可以看《数学奥林匹克不等式研究》杨学枝著,第67页。
reny 3# 2013-4-4 23:15
1# huyuhbb http://kkkkuingggg.5d6d.net/view ... 1007&highlight=曾经谈过
李斌斌755 4# 2013-4-5 00:27
啥是半凹半凸定理?是高中知识吗!
kuing 5# 2013-4-5 00:30
啥是半凹半凸定理?是高中知识吗! 李斌斌755 发表于 2013-4-5 00:27 高中连凹凸性都没有……
李斌斌755 6# 2013-4-5 00:34
5# kuing 那闪
reny 7# 2013-4-5 09:23
6# 李斌斌755 那就最好参考2楼啦
李斌斌755 8# 2013-4-5 11:20
7# reny 来这是玩,再学习,不了。
huyuhbb 9# 2013-4-5 11:38
主要是不想使用半凸半凹..
yes94 10# 2013-4-5 13:49
9# huyuhbb 能否把半凸半凹的本质领会了,再结合本题实际,将之移植过来,完美的结合在一起,让人丝毫看不出半凸半凹的一点影子?
thread-1328-1-3.html: [数列] 递推数列
转化与化归 1# 2013-4-4 22:40
这个题有什么好的思路
reny 2# 2013-4-4 23:38
本帖最后由 reny 于 2013-4-4 23:40 编辑 1# 转化与化归 这种数列求通项应该是求不出来滴,只好估计看
转化与化归 3# 2013-4-5 08:58
2# reny 能不能谈谈具体的思路
realnumber 4# 2013-4-5 09:09
3# 转化与化归 一般是先做实验嘛,求下$a_2,a_3,a_4$什么的.
huyuhbb 5# 2013-4-5 13:15
[a(n)^2]=n来自命题人讲座数列
yes94 6# 2013-4-5 13:26
[a(n)^2]=n来自命题人讲座数列 huyuhbb 发表于 2013-4-5 13:15 牛!居然知道此题的出处!
kuing 7# 2013-4-5 14:10
5# huyuhbb 嗯,我在《代数不等式》里找到了,不过是在习题里,答案也只是提示,没过程……
yes94 8# 2013-4-5 14:21
[a(n)^2]=n来自命题人讲座数列 huyuhbb 发表于 2013-4-5 13:15 你的意思是$a_n=\sqrt n$?
kuing 9# 2013-4-5 14:24
8# yes94 中括号显然是取整……
转化与化归 10# 2013-4-5 14:32
5# huyuhbb 确实牛!
yes94 11# 2013-4-5 14:53
9# kuing ,还以为,写的是[a(n)]^2=n呢
转化与化归 12# 2013-4-5 15:19
不是我希望看到的解答!
yes94 13# 2013-4-5 15:29
12# 转化与化归 数学归纳法加强了命题,还用了跷跷板归纳法
thread-1329-1-2.html: [组合] 电脑输出2012个数字之和(输出1或2)能被3整除的概率
yes94 1# 2013-4-5 00:05
电脑每秒钟以相同的概率输出数字1或者2,一共输出了2012个数字。则这2012个数字之和被3整除的概率是多少?其概率能大于$\dfrac13$吗?
李斌斌755 2# 2013-4-5 00:21
看不懂题
李斌斌755 3# 2013-4-5 00:33
1# yes94 输出的1和2一样多吗
yes94 4# 2013-4-5 00:36
2# 李斌斌755 电脑每隔一秒钟的时间等可能的输出一个数字1或者2,一共输出了2012次,共2012输出了个数字(1或2)。
李斌斌755 5# 2013-4-5 00:39
4# yes94 还是不明白 坐沙发
yes94 6# 2013-4-5 00:40
3# 李斌斌755 不一定一样多啊? 但是当电脑输出的次数$n$趋向于无穷大的时候,1或者2就基本上是一样多的了 另外,将2012次,改为2013次,其和被3整除的概率能大于$\dfrac13$吗?
kuing 7# 2013-4-5 00:43
设等可能地输出 $n$ 个 1 或 2 之和为 $3k$ 的概率为 $a_n$,为 $3k+1$ 的概率为 $b_n$,为 $3k+2$ 的概率为 $c_n$。 考虑 $a_{n+1}$,即 $n+1$ 个 1 或 2 之和为 $3k$ 的情形发生,这只能有两种“路径”,即 $n$ 个之和为 $3k+1$ 型时再输出 2,以及 $n$ 个之和为 $3k+2$ 型时再输出 1,而输出 1 或 2 等可能,故 $a_{n+1}=0.5b_n+0.5c_n$。同理可得 $b_{n+1}=0.5c_n+0.5a_n$,$c_{n+1}=0.5a_n+0.5b_n$。 通项暂时不求,因为不知这样有没有错……
kuing 8# 2013-4-5 01:03
设等可能地输出 $n$ 个 1 或 2 之和为 $3k$ 的概率为 $a_n$,为 $3k+1$ 的概率为 $b_n$,为 $3k+2$ 的概率为 $c_n$。 考虑 $a_{n+1}$,即 $n+1$ 个 1 或 2 之和为 $3k$ 的情形发生,这只能有两种“路径”,即 $n$ 个之和为 $3k+1$ 型时再输出 2,以及 $n$ 个之和为 $3k+2$ 型时再输出 1,而输出 1 或 2 等可能,故 $a_{n+1}=0.5b_n+0.5c_n$。同理可得 $b_{n+1}=0.5c_n+0.5a_n$,$c_{n+1}=0.5a_n+0.5b_n$。 通项暂时不求,因为不知这样有没有错…… kuing 发表于 2013-4-5 00:43 再想了下,应该没什么漏 dong 吧…… 来求一下通项,首先显然 $a_1=0$, $b_1=c_1=0.5$,后面两式相减得 $b_{n+1}-c_{n+1}=0.5(c_n-b_n)$,故必有 $b_n=c_n$,于是 $a_{n+1}=b_n$ 且 $b_{n+1}=0.5b_n+0.5a_n$,消去 $b_n$ 即得 \[a_{n+2}=\frac12a_{n+1}+\frac12a_n,\] 再由 $a_1=0$, $a_2=b_1=0.5$,即可求得 \[a_n=\frac13\bigl((-1)^n 2^{1-n}+1\bigr).\]
yes94 9# 2013-4-5 13:25
8# kuing 知道你递推法强!这道题似乎是专门为你准备的! 还有没有其他的方法?例如排列组合、母函数方法、等等
yes94 10# 2013-4-8 17:46
9# yes94
kuing 11# 2013-4-8 18:25
还是递推了…… 但是如果将两者输出的概率改变一下,不知还行不行得通 换言之,能否用于推广?……
isea 12# 2013-4-8 19:05
看了两遍题,看不懂,差距
yes94 13# 2013-4-8 21:34
11# kuing 还是k版的递推好,一箭三雕! 并且改变概率(非等可能),kuing的办法仍然吃香! 记得有一年的全国高中数学联赛加试题就是够作多个递推数列、环环相扣做出来的! 但不记得该题题目了  
李斌斌755 14# 2013-4-9 00:20
12# isea 同感
kuing 15# 2013-4-9 00:26
擦,有那么难理解么……
yes94 16# 2013-4-9 21:22

kuing 17# 2013-4-9 21:56
16# yes94 这个也不错
thread-133-1-2.html: 小白问题一个
GAM 1# 2011-10-22 23:27
本帖最后由 GAM 于 2011-10-22 23:42 编辑 如何证明:$$A\Rightarrow B\Leftrightarrow\neg A \vee B$$
GAM 2# 2011-10-22 23:32
没找到交和并的符号
kuing 3# 2011-10-22 23:33
2# GAM 我去补充一下那个表
kuing 4# 2011-10-22 23:37
3# kuing 已补充。见 http://kkkkuingggg.5d6d.com/thread-9-1-1.html 二楼表中的空集附近,由于不知道你想打哪个,所以只能你去看看了
GAM 5# 2011-10-22 23:44
揍是他了 这回好看了
kuing 6# 2011-10-22 23:48
其实我没太懂是啥意思
kuing 7# 2011-10-22 23:51
等价于 $\neg A \vee B$ 为真?
GAM 8# 2011-10-23 00:00
6# kuing
GAM 9# 2011-10-23 00:01
就是这样
kuing 10# 2011-10-23 00:14
这样行不行? 1)证 $(A\Rightarrow B) \implies \neg A \vee B$ 用反证法,假设 $A\Rightarrow B$ 真时 $\neg A \vee B$ 假,则 $\neg A $ 假且 $B$ 假,即 $A$ 真且 $B$ 假,而由 $A\Rightarrow B$ 知 $A$ 真时应有 $B$ 也真,矛盾; 2)证 $\neg A \vee B \implies (A\Rightarrow B)$ 若 $\neg A$ 真,则 $A$ 假,由楼上上的真值表可知无论 $B$ 真假如何都有 $A\Rightarrow B$ 真; 若 $\neg A$ 假,则 $B$ 真,即 $A$ 真且 $B$ 真,此时 $A\Rightarrow B$ 真。
kuing 11# 2011-10-23 00:16
呃,似乎都可以从真值表里直接看出结论。。。
nash 12# 2011-10-23 00:26
以后这个就可以直接应用啦
GAM 13# 2011-10-23 11:46
本帖最后由 GAM 于 2011-10-23 11:48 编辑 10、11# kuing 应该可以,好帅 我当时不知道为啥逻辑上真值表正确,现在也觉得怪怪的
kuing 14# 2011-10-23 12:12
13# GAM 其实我也不太懂这些……
GAM 15# 2011-10-23 12:37
14# kuing 出自卓里奇《数学分析》第四版第一节 ,某白自学中
kuing 16# 2011-10-23 12:41
15# GAM 呃?这不是逻辑题么,数学分析也会玩这些啊?
GAM 17# 2011-10-23 12:48
16# kuing 第一章 一些通用的数学概念与记号 1.逻辑符号 2.集与集的初等运算 3.函数 4.某些补充 就是这样
kuing 18# 2011-10-23 12:50
17# GAM 噢,预备知识…… 睡觉
thread-1330-1-3.html: [不等式] $a^2+b^2=1$,$a^3+b^3+1=m(a+b+1)^3$,求$m$的范围
yes94 1# 2013-4-5 14:58
已知正实数满足:$a^2+b^2=1$,$a^3+b^3+1=m(a+b+1)^3$,求$m$的取值范围。(12湖北)
零定义 2# 2013-4-5 19:09
1# yes94 m∈[3√2/2-2,1/4]?
reny 3# 2013-4-5 20:13
1# yes94
yes94 4# 2013-4-5 23:47
3# reny 不用三角换元,还有没有其他方法?
isea 5# 2013-4-5 23:58
看着像是竞赛的,果然和竞赛相关
kuing 6# 2013-4-6 01:37
这不是很简单么…… 记 $p=a+b$,则 $p\in(1,\sqrt2]$,不难化简得 \[m=\frac3{2(p+1)}-\frac12\]
yes94 7# 2013-4-6 07:26
6# kuing
isea 8# 2013-4-6 22:23
这不是很简单么…… 记 $p=a+b$,则 $p\in(1,\sqrt2]$,不难化简得 \[m=\frac3{2(p+1)}-\frac12\] kuing 发表于 2013-4-6 01:37 拨云见日
thread-1331-1-2.html: [不等式] 2不等式+1组合
huyuhbb 1# 2013-4-5 15:27
组合的第二问..
yes94 2# 2013-4-5 15:41
1# huyuhbb 又一位学生高手光临本论坛,祝贺!
huyuhbb 3# 2013-4-5 15:51
....其实我比较弱的
李斌斌755 4# 2013-4-5 17:12
2# yes94 怎么知道是学生?!
yes94 5# 2013-4-5 17:14
4# 李斌斌755 反正你不是学生
huyuhbb 6# 2013-4-7 12:59
求证明...
kuing 7# 2013-4-7 23:49
都不是简单的东东……三角换元之后也没什么想法……表示还没进入解题状态……
李斌斌755 8# 2013-4-8 11:20
本帖最后由 李斌斌755 于 2013-4-8 11:30 编辑 排梨:把2n个梨按质量大小重新排序,质量小在前,质量一样的相邻。组成一新数列an 设a1=a   有           0  =<an-a(n-1)=<1                             -1=<a1-a2=<0                             0=<a2n-a(2n-1)=<1                            -1=<a2n+a1-[a(2n-1)+a2]=<1 故把新数列中首尾两个依次装袋后排成的数列bn  bn=a(2n+1-k)+ak    (k=1,2,3,…,n) 有    -1=<bn-b(n-1)=<1 即相邻口袋的梨重量相差不超过1克。
huyuhbb 9# 2013-4-8 13:00
8# 李斌斌755 主要是第二问那个环不会
李斌斌755 10# 2013-4-8 14:07
9# huyuhbb 我也不会
kuing 11# 2013-4-8 19:43
PS、为什么都不喜欢回贴呢?每次都是我转上来……
pxchg1200 12# 2013-4-8 20:22
11# kuing Problem: Let $x,y,z,a,b,c\in R^{+}$,with \[ \left\{      \begin{array}{ll} (x+y)^2=4c^2xy+1, &\\ (y+z)^2=4a^2yz+1, & \\ (z+x)^2=4b^2zx+1, & \\        a^2+b^2+c^2+2abc=1, &      \end{array}    \right.\] find the min value of $x+y+z$. Solution: we note $$a=\frac{u}{\sqrt{(u+v)(u+w)}},b=\frac{v}{\sqrt{(v+u)(v+w)}},z=\frac{w}{\sqrt{(w+v)(w+u)}}$$ Hence we have \[ a^2+b^2+c^2=\frac{\sum{u^2(v+w)}}{(u+v)(u+w)(v+w)}\geq \frac{3}{4}\] and by AM-GM we have \[ a+b+c\leq \frac{3}{2} \] Now,due to the condition,we have \[ \sum{z(x+y)^2}=(x+y+z)+4xyz(a^2+b^2+c^2)\geq (x+y+z)+3xyz \] Or \[ xy+yz+xz\geq 1 \] Which leads \[ x+y+z\geq \sqrt{3} \] Done!
Gauss门徒 13# 2013-4-10 10:23
有$2n$个梨排成一行,每相邻的两个梨质量不超过$1$克。证明可以将这些梨装在$n$个口袋里并排成一行,使得每两个相邻口袋里的梨也相差不超过$1$克。
realnumber 14# 2013-4-11 13:47
http://bbs.hupu.com/3421049.html 百度搜索到的2n个梨,不过没看到答案,是2012年9,10年级的莫斯科数学竞赛题.
thread-1332-1-3.html: [数列] 数列证明题
yayaweha 1# 2013-4-5 19:26
本帖最后由 yayaweha 于 2013-4-5 19:27 编辑 已知集合$A=\{a_1,a_2,a_3\cdots,a_n\}$,其中$a_i\in R(1\le i\le n,     n\ge 2)$,$l(A)$表示和$a_i+a_j(1\le i\le j\le n)$中所有不同值的个数。若集合$A=\{2,4,8,\cdots,2_n\}$求证$$l(A)=\frac{n(n-1)}{2}$$
第一章 2# 2013-4-5 20:10
$2^n$吧, 这题不难啊,不就是$C_n^2$?
yayaweha 3# 2013-4-5 20:51
我知道,我想知道怎么证明
第一章 4# 2013-4-5 21:37
本帖最后由 第一章 于 2013-4-5 21:41 编辑 从$A=\{2,4,8,\cdots,2^n\}$任取$2^i,2^j,2^k,2^l$ 只需证明其中任意两个之和不等于其余两个之和 用反证法,不妨设$i<j<k<l$, 若有$2^i+2^l=2^j+2^k$ 则$2^i(1+2^{l-i})=2^j(1+2^{k-l})$ 两边同约去$2^i$,左边为奇数,右边为偶数,矛盾。
第一章 5# 2013-4-5 21:39
印象中今年广州一模也有这种问题,可惜标准答案采用基本不等式证明,走了弯路。
yayaweha 6# 2013-4-5 21:55
5# 第一章 不用基本不等式也用反证法?
yayaweha 7# 2013-4-5 21:57
从$A=\{2,4,8,\cdots,2^n\}$任取$2^i,2^j,2^k,2^l$ 只需证明其中任意两个之和不等于其余两个之和 用反证法,不妨设$i 第一章 发表于 2013-4-5 21:37 为什么这里可已使用不妨设
第一章 8# 2013-4-5 22:20
本帖最后由 第一章 于 2013-4-5 22:31 编辑 6# yayaweha 话说广州那道题的标准答案也是反设,再AM-GM的 我那个证法可能需要修正, 还考虑$2^i+2^k=2^j+2^l$, 其实道理是一样的。
thread-1333-1-3.html: [不等式] 似曾相识的不等式
reny 1# 2013-4-5 19:56
本帖最后由 reny 于 2013-4-5 20:41 编辑 (1)已知$a,b,c,d\in R^+,abcd=1,$求证$$\frac 1{1+a^3}+\frac 1{1+b^3}+\frac 1{1+c^3}+\frac 1{1+d^3}>1$$ (2)已知$a,b\in R^+,$证明$$\frac1 {\sqrt{1+a^2}}+\frac1 {\sqrt{1+b^2}}\geqslant\frac2 {\sqrt{1+ab}}$$对于$ a,b\in(0,1)或者ab\geqslant3$,出自http://ishare.iask.sina.com.cn/f/10736498.html第16页
huyuhbb 2# 2013-4-5 20:06
第一个不成立,a,b,c→无限,d→0 第二个ab<=2时不等号是反的
reny 3# 2013-4-5 20:18
本帖最后由 reny 于 2013-4-5 20:28 编辑 2# huyuhbb 不会吧,(2)没抄错啊。 (1)的确有问题呢,原来证明的时候把右边误看成了$\frac1 2$
huyuhbb 4# 2013-4-5 20:20
3# reny m的值域错了
kuing 5# 2013-4-5 20:31
4# huyuhbb 这才是关键,变成 $m^3/(m^3 + 1) + 3/(m + 1)$ 时 $m\in[1,+\infty)$,当 $m\to+\infty$ 时“取最小值1”,于是得到左>1。
kuing 6# 2013-4-5 20:37
天书说
reny 7# 2013-4-5 20:39
本帖最后由 reny 于 2013-4-5 20:43 编辑 5# kuing 嗯,第(2)问应该没抄错吧
thread-1334-1-1.html: 喔嗬嗬
isea 1# 2013-4-6 00:17
今天的主题加回帖都是偶第一个,嘿嘿 收工,睡觉
thread-1335-1-2.html: [几何] 直线斜率之和为定值与过定点问题
yes94 1# 2013-4-6 07:30
______kuing edit in $\LaTeX$______ 11.(20 分)已知 $E(m,n)$ 为抛物线 $y^2=2px$($p>0$)内一定点,过 $E$ 作斜率分别为 $k_1$、$k_2$ 的两条直线,与抛物线交于点 $A$、$B$、$C$、$D$,且 $M$、$N$ 分别是线段 $AB$、$CD$ 的中点。 (1)当 $n=0$,且 $k_1k_2=-1$ 时,求 $\triangle EMN$ 面积的最小值; (2)若 $k_1+k_2=\lambda$($\lambda\ne0$,$\lambda$ 为常数),证明:直线 $MN$ 过定点。
转化与化归 2# 2013-4-7 07:43
我发一个标答,以引起讨论。
yes94 3# 2013-4-8 17:45
2# 转化与化归 谢谢! 欢迎大家讨论
李斌斌755 4# 2013-4-22 04:12
3# yes94 你自己怎么不讨论讨论
yes94 5# 2013-4-22 12:32
3# yes94 你自己怎么不讨论讨论 李斌斌755 发表于 2013-4-22 04:12 我讨论的还不够多吗? 你也来讨论一下噻!你是几何专家的嘛
李斌斌755 6# 2013-4-22 13:53
5# yes94 我是想讨论,可这论坛上的题会的太少
thread-1339-1-3.html: [不等式] 不等式一题,标题真难取名字啊
realnumber 1# 2013-4-7 10:33
解题群看到的 江苏-周*(1%%%%%905) 2013-4-7 10:02:16 ps:不等式帖子名字怎么取好点?
kuing 2# 2013-4-7 11:57
这个是FAQ,比如说: http://bbs.pep.com.cn/thread-420059-1-1.html http://bbs.pep.com.cn/thread-422829-1-1.html 以及我在《数学空间》总第 4 期 P13 也写过,见 http://www.pep.com.cn/rjwk/gzsxs ... 0110623_1050792.htm
地狱的死灵 3# 2013-4-7 11:58

kuing 4# 2013-4-7 12:04
标题取名其实很容易啊,将题目的特征写上去就可以了。 实在不知怎么描述甚至可以将式子打上标题,就像这样:
yes94 5# 2013-4-7 12:30
标题只要概括全部内容或部分主题、部分特征即可。不一定标题要搞得很好听, 看了kuing的示范,我来给该题目加一个标题: “分母是三元平方和的二次齐次分式的最值问题” 或者直接: “$形如\dfrac{xa_1a_2+ya_2a_3}{a_1^2+a_2^2+a_3^2}$的最大值问题” k版看看可以不?
kuing 6# 2013-4-7 12:37
5# yes94 要是我就选第二种,因为第一种描述起来有点别扭。 不过我建议在标题上的时候打成 (...)/(...) 的形式就好了,免得将主题列表行撑太大
yes94 7# 2013-4-7 12:53
6# kuing 是有点别扭,所以realnumber就难以抉择了,他也是想好了几个标题了的,总觉得别扭,就放弃了吧? 我觉得,管他呢,标题只要能部分反映该题的主要内容,就可以啦! 当然再看看能否将标题搞得好听一点
realnumber 8# 2013-4-7 16:19
我是怕不等式题目标题重复,要不后面跟数字,不等式1,不等式2,很没营养.
kuing 9# 2013-4-7 16:25
所以将式子打上去是个很好的选择
yes94 10# 2013-4-7 23:27
3# 地狱的死灵 死灵的好解法! 死灵终于不再观望而出手了!
kuing 11# 2013-4-7 23:47
10# yes94 嗯,不过又看了下,似乎与“《数学空间》总第 4 期 P13”后面那个注一样了……
isea 12# 2013-4-8 19:45
这个是FAQ,比如说: http://bbs.pep.com.cn/thread-420059-1-1.html http://bbs.pep.com.cn/thread-422829-1-1.html 以及我在《数学空间》总第 4 期 P13 也写过,见 http://www.pep.com.cn/rjwk/gzsxsxkj/2011/s ... kuing 发表于 2013-4-7 11:57 继续学习
thread-134-1-2.html: 来自pep的函数可导求参数值
kuing 1# 2011-10-23 11:10
已知函数 \[f(x)=\cases{x,&x\leqslant 0,\\ \frac{a+b\cos x}x,&x>0,}\] 在 $x=0$ 处可导,则 $a=?,b=?$ 来自 http://bbs.pep.com.cn/thread-1928150-1-1.html 原贴里我一开始目测错了   要可导首先要连续,于是当 $x\to0+$ 时必须是 0/0 型,否则无穷大,即当 $x\to0+$ 时 $a+b\cos x\to0$,于是显然要有 \[a+b=0,\] 又要 \[\lim_{x\to0}\left(\frac{a+b\cos x}x\right)'=1,\] 由以上两点即 \[\lim_{x\to0}\frac{a(x\sin x+\cos x-1)}{x^2}=1 \iff a\lim_{x\to0}\left(\frac{\sin x}x-\frac12\frac{\sin^2\frac{x}{2}}{\bigl(\frac x2\bigr)^2}\right)=1\] 由重要极限知 \[\lim_{x\to0}\left(\frac{\sin x}x-\frac12\frac{\sin^2\frac{x}{2}}{\bigl(\frac x2\bigr)^2}\right)=1-\frac12=\frac12,\] 所以应有 \[a=2.\]
鱼儿 2# 2011-11-17 13:16
本帖最后由 鱼儿 于 2011-11-22 10:07 编辑 kuing兄的解答有误,解答中用到了导函数的连续性,而题设条件仅仅是“$f(x)$在$x=0$处可导”。 下面给出问题的解答。 解  一方面,因为“$f(x)$在$x=0$处可导,所以“$f(x)$在$x=0$处连续,故有$\displaystyle\lim_{x\to 0^+}\frac{a+b\cos x}{x}=\lim_{x\to 0^-}x=0$, 于是知,$\displaystyle\lim_{x\to 0^+}(a+b\cos x)=0$, 即 $a+b=0$               (1) 另一方面,因为“$f(x)$在$x=0$处可导,所以 $f'_-(0)=f'_+(0)$, 而$f'_-(0)=\displaystyle\lim_{x\to 0^-}\frac{f(x)-f(0)}{x-0}=\lim_{x\to 0^-}\frac{x}{x}=\lim_{x\to 0^-}1=1$, 由(1)得 $f'_+(0)=\displaystyle\lim_{x\to 0^+}\frac{f(x)-f(0)}{x-0}=\lim_{x\to 0^+}\frac{\frac{a+b\cos x}{x}-0}{x-0}$ $\displaystyle=\lim_{x\to 0^+}\frac{a+b\cos x}{x^2}=a\lim_{x\to 0^+}\frac{1-\cos x}{x^2}=a\lim_{x\to 0^+}\frac{2\sin^2 \frac{x}{2}}{x^2}$ $\displaystyle=\frac{a}{2}\lim_{x\to 0^+}(\frac{\sin \frac{x}{2}}{\frac{x}{2}})^2=\frac{a}{2}$, 所以 $\displaystyle\frac{a}{2}=1$, 即 $a=2$                  (2) 代入(1)得 $b=-1$, 因此 $a=2, b=-1$.
kuing 3# 2011-11-17 14:03
噢,就是不能直接求导再求极限,而是要用定义式…… PS: 一撇前不用上标,直接先打一撇再用下标即可。即 f'_+(0)=f'_-(0) ,效果 $f'_+(0)=f'_-(0)$ 如果用上标的话就要用\prime,即 f_+^{\prime}(0)=f_-^{\prime}(0) ,效果 $f_+^{\prime}(0)=f_-^{\prime}(0)$
thread-1340-1-3.html: [不等式] 越南和土耳其的不等式赛题
huyuhbb 1# 2013-4-7 22:38
果断还是不会代码..只好传图片了 ________kuing edit in $\LaTeX$________ 一. 2012土耳其 $a,b,c>0,\sum a^3=\sum a^4$,求证 \[\sum\frac{a}{a^2+b^3+c^3}\ge 1.\] 二. 2013越南 求最大正数 $k$,使 \[\frac{1}{a}+\frac{1}{b}+\frac{1}{c}+\frac{k}{1+a+b+c}\ge 3+\frac{k}{4}\] 对满足 $abc=1$ 的正数 $a,b,c$ 恒成立。
yes94 2# 2013-4-7 22:42
1# huyuhbb 题目? 有草稿本和范例,慢慢来就会了
kuing 3# 2013-4-8 01:22
以前研究过分母没有1+的,那时还碰到三次方程。看来加个1数据会变简单? 时间关系,先闪明天再算算看。
pxchg1200 4# 2013-4-8 12:46
4# kuing arqady表示k=13 is the best due to $uvw$ theorm
kuing 5# 2013-4-8 13:31
4# kuing arqady表示k=13 is the best due to $uvw$ theorm pxchg1200 发表于 2013-4-8 12:46 以前研究分母没有1+的那个的时候我也是用 uvw http://www.artofproblemsolving.com/blog/38834
kuing 6# 2013-4-8 16:37
按照楼上链接的方法,我似乎还是遇到三次方程啊……难道计算错了?
kuing 7# 2013-4-8 17:07
仿照楼上上链接的方法,后面的计算略去(当然就不是像链接那样一步步算了,上软件,用结式算……)。 当 $a=b=c=1$ 时不等式恒成立,当 $a$, $b$, $c$ 不全相等时 \[\frac1a+\frac1b+\frac1c+\frac k{1+a+b+c}\geqslant 3+\frac k4\iff ab+bc+ca-3\geqslant k\left( \frac14-\frac1{1+a+b+c} \right),\] 因为 $a+b+c>3$,所以等价于 \[k\leqslant f(a,b,c)=\frac{ab+bc+ca-3}{\frac14-\frac1{1+a+b+c}},\] 所以只要求 $f(a,b,c)$ 的最小值(或下确界)。由 uvw 定理可知只要考虑两元相等,即只要求当 $a>0$ 且 $a\ne1$ 时 \[g(a)=f\left(a,a,\frac1{a^2}\right)\] 的最小值(或下确界)。化简得 \[g(a)=\frac{4 (a+1) (a+2) (2 a^2-a+1)}{a (2 a+1)},\] 显然存在最小值,设其为 $m=g(a_0)$,求导化简得 \[g'(a)=\frac{8 (a^2+a+1) (4 a^3+4 a^2-3 a-1)}{a^2 (2 a+1)^2},\] 则 \[\left\{\begin{aligned} 4 a_0^3+4 a_0^2-3 a_0-1&=0,\\ 4 (a_0+1) (a_0+2) (2 a_0^2-a_0+1)-ma_0 (2 a_0+1)&=0, \end{aligned}\right.\] 消去 $a_0$,即得 \[m^3+46 m^2-699 m-1936=0,\] 只有唯一正数根 $m\approx 13.9676$,这就是 $k$ 的最大值。
kuing 8# 2013-4-8 19:41
天书(1846******) 18:38:45 第一题水,直接乘分子柯西
pxchg1200 9# 2013-4-8 19:53
11# kuing Problem Let $a,b,c>0$ with $ a^3+b^3+c^3=a^4+b^4+c^4 $,Prove that \[ \frac{a}{a^2+b^3+c^3}+\frac{b}{b^2+c^3+a^3}+\frac{c}{c^2+a^3+b^3}\geq 1 \] Proof: by Cauchy-Schwarz,we have \[ \left(\sum{\frac{a}{a^2+b^3+c^3}}\right)\left[\sum{a(a^2+b^3+c^3)}\right]\geq (a+b+c)^2 \] Therefore,it's suffice to prove \[ (a+b+c)^2\geq \sum{a(a^2+b^3+c^3)} \] Or \[ (a+b+c)^2\geq (a+b+c)(a^3+b^3+c^3) \] \[ \Leftrightarrow  (a+b+c)(a^4+b^4+c^4)^2\geq (a^3+b^3+c^3)^3\] Which is true by Holder inequality. Done!
kuing 10# 2013-4-8 20:09
12# pxchg1200 看来的确不是一个难度级别……
kuing 11# 2013-4-8 20:23
顺便将1#用LaTeX输入了下。 To 楼主:你可以编辑1#的贴子,看看代码怎么写
thread-1341-1-2.html: [不等式] 三角形中的$\max \cdot \min$ 能秒不 (2013年4月高三理科北京西城一模)
isea 1# 2013-4-8 10:54
本帖最后由 isea 于 2013-4-8 20:01 编辑 设$\triangle ABC$的三边长分别为$a,b,c$,且$a\le b\le c$. 记$t=\max \left \{\dfrac ab,\dfrac bc,\dfrac ca\right \}\cdot \min\left \{\dfrac ab,\dfrac bc,\dfrac ca\right \}. $ (1)若$\triangle ABC$为等腰三角形,则$t=$_______; (2)设$a=1$,则$t$的取值范围是_______. 北京西城2013年高三一模最一填空题,我只会猜测第一个,第二个都不敢尝试,故发来学习学习。 ==== 以上文字有简写,现附上,图片版原文
李斌斌755 2# 2013-4-8 11:54
蒙,(1)1       (2)[1,2)
yes94 3# 2013-4-8 12:21
3# 李斌斌755 几何高手!
李斌斌755 4# 2013-4-8 12:28
不会删图
李斌斌755 5# 2013-4-8 12:29
4# yes94 对吗?
kuing 6# 2013-4-8 12:35
不会删图 李斌斌755 发表于 2013-4-8 12:28 see http://kkkkuingggg.5d6d.net/thread-119-1-1.html
isea 7# 2013-4-8 12:45
本帖最后由 isea 于 2013-4-8 12:49 编辑 蒙,(1)1       (2)[1,2) 李斌斌755 发表于 2013-4-8 11:54 第二空的上界结果不对,结果是$[1,\dfrac{1+\sqrt5}2)$, 整张试卷:http://pan.baidu.com/share/link?shareid=364288&uk=805596449
yes94 8# 2013-4-8 12:49
6# 李斌斌755 不晓得,总之对你的几何很膜拜,所以没看题 ,先赞一个!
kuing 9# 2013-4-8 13:21
设$\triangle ABC$的三边长分别为$a,b,c$,且$a\le b\le c$.定义$t=\max \left \{\dfrac ab,\dfrac bc,\dfrac ca\right \}\cdot \min\left \{\dfrac ab,\dfrac bc,\dfrac ca\right \}. $ (1)若$\triangle ABC$为等腰三角形,则$t=$_______; (2)设$a=1$,则$t$的取值范围是_______. ... isea 发表于 2013-4-8 10:54 (2)依题意可设 $c=ub$,则 $b\geqslant 1$, $u\geqslant 1$,由构成三角形知 $1+b>ub$,于是 \[1\leqslant u<\frac1b+1,\] 而 \begin{align*} \max \left\{ \frac ab,\frac bc,\frac ca \right\}&=\frac ca=ub, \\ \min \left\{ \frac ab,\frac bc,\frac ca \right\}&=\min \left\{ \frac ab,\frac bc \right\}=\min \left\{ \frac1b,\frac1u \right\}, \end{align*} 于是 \[t=ub\min \left\{ \frac1b,\frac1u \right\}=\min \{b,u\},\] 因 $b\geqslant 1$, $u\geqslant 1$,故 $t\geqslant 1$,当且仅当 $b=u=1$ 时取等; 又 \[t\leqslant u<\frac1b+1\leqslant \frac1t+1 \riff t<\frac{\sqrt5+1}2,\] 当 $b=u\to\bigl(\sqrt5+1\bigr)/2$ 时 $t\to\bigl(\sqrt5+1\bigr)/2$。 综上……
abababa 10# 2013-4-8 13:32
本帖最后由 abababa 于 2013-4-8 13:34 编辑 看看这样行不行 a=1,最大的一定是c,最小的不是$\frac{1}{b}$就是$\frac{b}{c}$ 如果是$\frac{1}{b} \ge \frac{b}{c}$,则$t=c\frac{b}{c}=b$ 所以$\frac{1}{t}\ge\frac{t}{c}$,即$c \ge t^2$ 然后$a+b>c$,就是$1+t>c \ge t^2$,就解出来了 如果是$\frac{1}{b} \le \frac{b}{c}$,则$t=\frac{c}{b}$,即$c=bt$ 然后$a+b>c$,就是$1+b>c=bt$,得$t<1+\frac{1}{b}$,而且$b \le c$,就是$b \le bt$,得$1 \le t$,因为存在t满足$1 \le t < 1+\frac{1}{b}$,所以最小是1
李斌斌755 11# 2013-4-8 13:43
又错了 ,mix不断变换。
kuing 12# 2013-4-8 13:47
12# 李斌斌755 先把图删一个,见7#
kuing 13# 2013-4-8 14:04
嗯?怎么把贴子删了……
李斌斌755 14# 2013-4-8 14:08
13# kuing 操作错了
kuing 15# 2013-4-8 14:27
要我怎么说你呢……
hongxian 16# 2013-4-8 15:02
有点不稳定,刷新后有的贴子没有看到!
kuing 17# 2013-4-8 15:05
16# hongxian 现在应该没问题,原先3#的几何法因为他自己误操作,把贴子删掉了(见上面几层楼)
yes94 18# 2013-4-8 17:43
17# kuing 还可以自己删除自己的整个楼层?
kuing 19# 2013-4-8 18:27
18# yes94 看6#的链接
isea 20# 2013-4-8 18:37
学习了,9楼,10楼,厉害。 这些不等式转来转去就把偶转晕了 9楼,那种不管你是谁,统统一把抓住,这种技巧实在实用,精巧!
thread-1341-2-2.html:
isea 21# 2013-4-8 19:08
稍晚我再想想,能不能转化成线性规划,或者函数的最值……
转化与化归 22# 2013-4-8 23:48
可以转化为规划
isea 23# 2013-4-9 01:11
可以转化为规划 转化与化归 发表于 2013-4-8 23:48
thread-1342-1-2.html: 一道解析几何
转化与化归 1# 2013-4-8 12:46
本帖最后由 转化与化归 于 2013-4-8 21:41 编辑 这道题有没有简洁的方法?
yes94 2# 2013-4-8 12:54
笨方法是可以的吧, 设$MC$的斜率为$k$,$MC$的斜率为$k$,$MD$的斜率为$-\dfrac{1}k$,则$A$、$B$、$C$、$D$四点坐标均可用$k$表示,然后……,然后就是体力活啦!
hnsredfox_007 3# 2013-4-8 14:56
楼主已经试验过什么方法?
kuing 4# 2013-4-8 15:06
题目没交待里面那个圆??
hnsredfox_007 5# 2013-4-8 15:57
2# yes94 这样的处理的确是体力活哦 有相对较简单的方法
yes94 6# 2013-4-8 17:38
楼主已经试验过什么方法? hnsredfox_007 发表于 2013-4-8 14:56 张老师再等丁老师的回话,免得张老师出招的时候,和丁老师的方法雷同了就不好了吧? k版说了,交代过圆这个条件没?以后出题还是要审核一下哦
转化与化归 7# 2013-4-8 21:37
5# hnsredfox_007 期待你的好方法!
转化与化归 8# 2013-4-8 21:43
题目漏了圆的交待,已更改,大家见谅。
yes94 9# 2013-4-8 21:49
8# 转化与化归 你要先说说你知道哪些方法,张老师才好出奇招!免得他的方法和你的的方法雷同! 如果雷同,张老师是不会出招的
转化与化归 10# 2013-4-8 21:52
我没有好方法啊,我的方法就是把点硬算出来。
hnsredfox_007 11# 2013-4-9 08:31
10# 转化与化归 我也是硬算,只是我只硬算一个哦
hnsredfox_007 12# 2013-4-9 09:04
10# 转化与化归 解析:不妨设$l_1$的倾斜角为$\theta$,斜率为$k$,则$l_1$的方程为$y=kx+1$, 代入椭圆方程化简得到$\left(1+4k^2\right)x^2+8kx=0$, 于是得到$x_C=-\dfrac{8k}{1+4k^2}$. 同理,$x_D=\dfrac{8k}{4+k^2}$. 于是由$3\overrightarrow{MA}\cdot\overrightarrow{MC}=4\overrightarrow{MB}\cdot\overrightarrow{MD}$有$3\times2|OM|\sin\theta\cdot|MC|=4\times 2|OM|\cos\theta\cdot|MD|$, 以下不做分析了……
转化与化归 13# 2013-4-9 12:27
12# hnsredfox_007 很好的解法
thread-1343-1-3.html: [不等式] 前晚幻幻问我的连根式不等式
kuing 1# 2013-4-8 14:59
哎,看到连根式我就晕啦,还好刚才终于想到了。 为了能够放缩并且产生“连锁反应”,于是考虑到寻找 $\sqrt[n]{n+c}<c$ 对一定范围的 $n$ 恒成立的正常数 $c$,显然 $c=2$ 就行了,因为 $n+2<2^n$ 对 $n\geqslant3$ 恒成立,这样,就得到 \begin{align*} &\sqrt[n]{n}<\sqrt[n]{n+2}<2 ,\\ &\sqrt[n-1]{n-1+\sqrt[n]{n}}<\sqrt[n-1]{n-1+2}<2 ,\\ &\sqrt[n-2]{n-2+\sqrt[n-1]{n-1+\sqrt[n]{n}}}<\sqrt[n-2]{n-2+2}<2 ,\\ &{}\vdots{}\\ &\sqrt[3]{3+\sqrt[4]{4+\cdots +\sqrt[n-1]{n-1+\sqrt[n]{n}}}}<\sqrt[3]{3+2}<2 ,\\ &\sqrt{2+\sqrt[3]{3+\sqrt[4]{4+\cdots +\sqrt[n-1]{n-1+\sqrt[n]{n}}}}}<\sqrt{2+2}=2. \end{align*}
kuing 2# 2013-4-8 15:26
取别的 $c$ 会得到不同的结果,比如说 $c=1.672$ 时,$n+1.672<(1.672)^n$ 也对 $n\geqslant3$ 恒成立,于是仿上,有 \begin{align*} &\sqrt[n]{n}<\sqrt[n]{n+1.672}<1.672 ,\\ &{}\vdots{}\\ &\sqrt[3]{3+\sqrt[4]{4+\cdots +\sqrt[n-1]{n-1+\sqrt[n]{n}}}}<\sqrt[3]{3+1.672}<1.672 ,\\ &\sqrt{2+\sqrt[3]{3+\sqrt[4]{4+\cdots +\sqrt[n-1]{n-1+\sqrt[n]{n}}}}}<\sqrt{2+1.672}\approx1.916246<1.92. \end{align*}
kuing 3# 2013-4-8 16:07
$f(n)=\sqrt{2+\sqrt[3]{3+\sqrt[4]{4+\cdots +\sqrt[n-1]{n-1+\sqrt[n]{n}}}}}$ 显然是关于 $n$ 递增的,上面已经证明了 $f(n)$ 有上界,即单增有界,必然存在极限,于是继续追问的问题就是 \[\lim_{n\to\infty}f(n)=?\] PS、不知这是不是 old problem?
yes94 4# 2013-4-8 17:34
3# kuing 看起来比较像,李明曾经研究过这个连根式
kuing 5# 2013-4-8 17:38
4# yes94 所以我给李明留了言,他可能不在线
thread-1344-1-1.html: 人说男人像酒[笑话]
isea 1# 2013-4-8 19:10
本帖最后由 isea 于 2013-4-9 01:28 编辑 快300帖啦,真不敢想啊,300帖了,就快。 ====
thread-1345-1-3.html: 不等式
v6mm131 1# 2013-4-8 20:07
求秒
kuing 2# 2013-4-8 20:08
第一行是条件,第二行是求证? PS1、其实全是正数就可以了,同号跟正数没分别,这里。 PS2、标题可以更具体些,and 主题分类
kuing 3# 2013-4-8 22:14
你确定这题正确?
kuing 4# 2013-4-8 22:25
A1 = 2; B1 = 8/3; C1 = 1; A2 = 1; B2 = 1; C2 = 1; 满足 A1^2/(B1^2 + C1^2) < A2^2/(B2^2 + C2^2) 但 (A1 + A2)^2/((B1 + B2)^2 + (C1 + C2)^2) > A2^2/(B2^2 + C2^2) 这样就否定了右边。 左边懒得再构造。
yes94 5# 2013-4-8 22:47
4# kuing 看来v6的题……, 例如该贴:http://kkkkuingggg.5d6d.net/thread-943-1-2.html 如果是自己的猜想,最好要注明,这样就有人要么去证明,要么去否定;
thread-1347-1-1.html: 海淀一模
isea 1# 2013-4-9 00:55
丢空间了 测试 一下 是每攵感词?
isea 2# 2013-4-9 00:55
不是啊,奇怪 b 题:设$l_1,l_2,l_3$为空间三条互相平行且两两间距的距离为4,5,6的直线。给出下列三个结论: (1) $\exists A_i \in l_i(i=1,2,3)$,使得$\triangle A_1A_2A_3$是直角三角形; (2)  $\exists A_i \in l_i(i=1,2,3)$,使得$\triangle_1A_2A_3$是等边三角形; (3) 三条直线上存在四点$A_i(i=1,2,3,4)$,使得四面体$A_1A_2A_3A_4$为在某一个顶点处的三条棱两两互相垂直的四面体。 其中,所有正确结论的序号是____.
kuing 3# 2013-4-9 00:56
我也不知道为什么,这次不是进入审核,是系统直接shan的,我从主题回收站里能看到,看来看去没看出什么来。
kuing 4# 2013-4-9 00:57
这样吧,你直接发图片版好了
isea 5# 2013-4-9 00:57
3# kuing 哈哈,不管了 就这里玩吧 这个前两个一定对,现在就是第三个了
kuing 6# 2013-4-9 00:59
注意cao作者
isea 7# 2013-4-9 00:59
感觉第(3)是错的,少一个直角
kuing 8# 2013-4-9 01:00
5# isea 还是去数学区发一个图片版吧,搞不好这贴也可能被XX
isea 9# 2013-4-9 01:01
1236 注意cao作者 kuing 发表于 2013-4-9 00:59 不管它,谁每攵感一点也好 不过,latex 如何给不是公式编号,像圆卷1之类
isea 10# 2013-4-9 01:02
8# kuing 好吧好吧
kuing 11# 2013-4-9 01:03
9# isea 在真正的 latex 里弄带圈数字的方法也有几种,但是这里都用不了。
isea 12# 2013-4-9 01:04
11# kuing 随便给一个来 我搜索了一下,似乎没看到,基本全是公式的
kuing 13# 2013-4-9 01:06
12# isea http://www.latexstudio.net/good-way-to-make-textcircled-numbers/
kuing 14# 2013-4-9 01:11
13# kuing 链接中的第二个链接错了。
isea 15# 2013-4-9 01:19
啧啧,吓人,弃了 暂时不考虑这个
kuing 16# 2013-4-9 01:37
15# isea 最简单是用那个 \ding 的, 但是数量少。 要是用xelatex的话还可以找一个有足够多这种圈数字的字体然后弄进去,也比较简单。
kuing 17# 2013-4-9 01:42
如果懂得制作字体, 就可以自己设计, 怎么整都行。
isea 18# 2013-4-9 07:45
本帖最后由 isea 于 2013-4-9 07:47 编辑 16# kuing \usepackage{pifont} \usepackage[perpage,symbol*]{footmisc} \DefineFNsymbols{circled}{{\ding{192}}{\ding{193}}{\ding{194}}                                             {\ding{195}}{\ding{196}}{\ding{197}}{\ding{198}}                                             {\ding{199}}{\ding{200}}{\ding{201}}}\setfnsymbol{circled} %编号圈1到圈10 不错,十个,够用了,基本
thread-1348-1-2.html: [几何] 三条平行线从平面到空间
isea 1# 2013-4-9 01:03
这样也只能有两个模型吧,如图所示
kuing 2# 2013-4-9 02:05
这种不用动手只要想象的题留在现在爪机ing的时候做就最好了。 想象了下,1和2考虑特殊位置以及极端情形可由连续变化知都存在; 3通过线面垂直可以证明不存在。
isea 3# 2013-4-9 02:15
嗯,标答是B。 说不一定,大早上就会有 从一点引三棱三夹角关系都能整出来呢 晚安晚安,先
kuing 4# 2013-4-9 02:20
3我是分了两种情况来想的,现在爪机不好说。 哎…我还很精神啦,可是也得睡。
李斌斌755 5# 2013-4-9 10:37
本帖最后由 李斌斌755 于 2013-4-9 23:43 编辑 上个图
isea 6# 2013-4-9 23:08
1239上个图 李斌斌755 发表于 2013-4-9 10:37 211 图(后面那个非等边的),作了个球,没看明白 具体是说直角三角形(我用的三者平行得等腰直角),还是四棱锥?
李斌斌755 7# 2013-4-9 23:26
6# isea (1)抱歉,原图有误。球面上任一点与不经过它的直径的两端点连线为直角三角形,球直径两端点     应分别在两直线上。 (2)思路:分别在三直线上取三点,有三未知数a、x、y,联立三元两次方程,x、y有解,故存在。
isea 8# 2013-4-9 23:51
6# isea (1)抱歉,原图有误。球面上任一点与不经过它的直径的两端点连线为直角三角形,球直径两端点     应分别在两直线上。 (2)思路:分别在三直线上取三点,有三未知数a、x、y,联立三元两次方程,x、y有解 ... 李斌斌755 发表于 2013-4-9 23:26 明白了 直角三角时,当三线共面,直接等腰,直观 进一步,粗略看上去,三线共面时,不存在等边(猜测,未证实)
李斌斌755 9# 2013-4-9 23:59
8# isea 不明白,共线三距离不就四线?
isea 10# 2013-4-10 00:16
8# isea 不明白,共线三距离不就四线? 李斌斌755 发表于 2013-4-9 23:59 倒,真的是! 我都没看题 设,直接默认了,我说怎么这么简单。 幸好看到你的帖子了!
kuing 11# 2013-4-10 00:30
关于1和2的。 比如说我们先将 $A$、$B$、$C$ 按如图所示放置。 容易看出此时 $BC<AB=AC$。 现在,我们将 $A$ 和 $B$ 往上移,并且总保持 $AB=AC$(这是可以做到的,只要 $A$、$B$ 的速度满足一定关系),而当 $A$、$B$ 移得很高很高时,不难想象 $\triangle ABC$ 将会变得很扁,也就是会变成顶角 $A$“非常钝”的一个等腰钝角三角形。于是,在移动过程中,总有一刻,使 $\triangle ABC$ 成为等边三角形,亦总有另一刻,使 $\triangle ABC$ 成为直角三角形(而且还是等腰的)。 这样,一次过就得到1和2都是正确的。 至于3,如图所示。 为方便书写,称三条两两垂直的棱所共的顶点为 $\top$。 假设 $A$ 是 $\top$,那么由 $AD\perp AB$, $AD\perp AC$ 知 $L_3\perp\triangle ABC$,从而 $\triangle ABC$ 三边的长就是三条直线的距离 4、5、6,这就与 $AB\perp AC$ 矛盾。同理可知 $D$ 是 $\top$ 时也矛盾; 假设 $C$ 是 $\top$,那么由 $BC\perp CA$, $BC\perp CD$ 知 $BC\perp\triangle CAD$,而 $L_1\sslash\triangle CAD$,故 $BC\perp L_1$,从而 $BC$ 为 $L_1$ 与 $L_2$ 的距离,于是 $EF\pqd BC$,这样就得到 $EF\perp FG$,矛盾。同理可知 $B$ 是 $\top$ 时也矛盾。 综上,不存在。 以上就是昨晚想象的,即2#所说的过程。 3真是难表达,尽管想的时候只是一瞬间……
kuing 12# 2013-4-10 01:16
更新了一下楼上。 其实还是表达得不好,算了,不玩了,还被说了……闪人,早知不写
李斌斌755 13# 2013-4-10 01:26
11# kuing 如图,四面体三棱i、l、k互相垂直  推出  直线l垂直平面a  推出  平面r垂直平面a  与题意矛盾
kuing 14# 2013-4-10 01:32
13# 李斌斌755 这是其中一种情况。 刚才之所以不知怎么说就是另一种情况难表达些,已经补上。
李斌斌755 15# 2013-4-10 01:38
本帖最后由 李斌斌755 于 2013-4-10 01:41 编辑 14# kuing 回复前没看到,回复后看到了,算插图 另一种情况?什么情况
kuing 16# 2013-4-10 14:14
15# 李斌斌755 11#的“假设 $C$ 是 $\top$ ……”
李斌斌755 17# 2013-4-10 16:01
本帖最后由 李斌斌755 于 2013-4-23 10:09 编辑 16# kuing $BC\perp AC,BC\perp DC\riff BC\perp\alpha$(L2L3组成的平面)$\riff\alpha\perp\beta$(L1L2组成的平面) 与题意矛盾
李斌斌755 18# 2013-4-10 16:08
17# 李斌斌755 三棱必然分布在L1L2L3直线组成的两平面内,三棱互相垂直可推出两平面垂直,与三平面互不垂直矛盾
转化与化归 19# 2013-4-10 17:41
12# kuing 谁说你的呢? 谁说你的呢?
kuing 20# 2013-4-10 17:47
19# 转化与化归 我妈……
thread-1348-2-2.html:
李斌斌755 21# 2013-4-10 20:03
19# 转化与化归
isea 22# 2013-4-10 20:58
17# 李斌斌755 三棱必然分布在L1L2L3直线组成的两平面内,三棱互相垂直可推出两平面垂直,与三平面互不垂直矛盾 李斌斌755 发表于 2013-4-10 16:08 就是这样,不对称的旋转一下(说不对称是4,5,6不等)就同理了 也是在水区说的,少一个垂直
thread-1349-1-2.html: [函数] 三角函数“定弦”长 最大与最小差 单调区间
isea 1# 2013-4-9 09:43
本帖最后由 isea 于 2013-4-9 23:02 编辑 题:已知函数$f(x)=\sin\dfrac \pi 2 x$,任取$t \in \mathbf{R}$,定义集合: $A_t=\{y|y=f(x)$,点$P(t,f(t)),Q(x,f(x))$,满足$ \abs{PQ}\le \sqrt 2\}$。 设$M_t,m_t$分别表示集合$A_t$中元素的最大值和最小值,记$h(t)=M_t-m_t$。则 (1)函数$h(t)$的最大值是______; (2)函数$h(t)$的单调递增区间为______。 ==== 只会画图看看大致走向,然后填空。海淀的题,发来向大家学习学习。 图片版原题
转化与化归 2# 2013-4-9 18:24
三角函数的最值相关题
kuing 3# 2013-4-9 18:30
楼主打题目打错了个字母
转化与化归 4# 2013-4-9 18:33
根本没看打得题,看来还是“有图有真相”
yes94 5# 2013-4-9 19:13
4# 转化与化归 对。有图有真相
isea 6# 2013-4-9 23:04
抱歉抱歉,已修改。 一个个心知肚明…… 不知道这个题会不会有表达式,或者$h(t)$的图象
isea 7# 2013-4-9 23:05
三角函数的最值相关题 转化与化归 发表于 2013-4-9 18:24 填空结果完全正确 感谢
三下五除二 8# 2013-4-15 11:15
感觉,题目里的根号2,换成别的数,题目条件允许的值,2个问题的答案都不变,是不是
yes94 9# 2013-4-15 22:14
把那个集合转换成中文叙述就好理解啦! 然后在几何图像,该函数的周期是$T=4$吧 显然,函数图形在以P点为圆心的圆内点的纵坐标的范围的极端情况,那就得讨论最高点与最低点……………………
isea 10# 2013-4-15 22:53
这题,我讲不清楚,只能理性感受一下 至于那个$\sqrt2$是否是题中的最大,就无法知道了,偶
thread-135-1-6.html: [不等式] 这个够简单了不?(不等式)
pxchg1200 1# 2011-10-23 14:57
Let $a,b,c >0 $ prove that: \[ \frac{a^{2}}{b}+\frac{b^{2}}{c}+\frac{c^{2}}{a}\ge\frac{3(a^{3}+b^{3}+c^{3})}{a^{2}+b^{2}+c^{2}} \] (kk 秒杀了吧 ...)
yizhong 2# 2012-8-23 18:21
本帖最后由 yizhong 于 2012-8-23 18:26 编辑 昨天搞备份的时候才知道原来小PX发了这么多的BAT , 这个 题目可以很简单的用SOS解决,只需要在待证明的不等式两边同时凑上:$-\sum a$即可,接下去就是根据 SOS判定定理解决. 顺便PS一下:这个题目还可以用SOS---Schur解决,过程会比用SOS简洁一些。
kuing 3# 2012-8-23 18:45
2# yizhong 好久没用过 SOS-Schur 了……不太记得怎么配了,写下过程看看
kuing 4# 2012-8-23 19:12
还是配出来了 \[\iff\left(\frac1c+\frac c{ab}-\frac{2(a+b)}{a^2+b^2+c^2}\right)(a-b)^2 + \left(\frac1a+\frac2c-\frac a{bc}-\frac{a+b+2c}{a^2+b^2+c^2}\right)(a-c)(b-c)\geqslant0,\]
kuing 5# 2012-8-23 19:31
4# kuing 配成这样好像判断不出来,得重新配配看。
海盗船长 6# 2012-8-23 20:51
SOS配成这样了,不知道算错没。。 $$\sum \frac{a^2-ab+c^2}{b}(a-b)^2\geqslant 0$$
kuing 7# 2012-8-23 22:12
6# 海盗船长 没错。 但是直接判断不出来,用SOS定理不知是否简单,我懒得试……
海盗船长 8# 2012-8-23 22:24
7# kuing 试了下,,用这个里面第三个判定方法好像可以: http://www.artofproblemsolving.c ... hp?f=55&t=80127
kuing 9# 2012-8-24 00:56
再配了一个 \[\iff \left(\frac{a+b}{ab}-\frac{2(a+b)}{a^2+b^2+c^2}\right)(a-b)^2+\left(\frac{b+c}{ca}-\frac{a+b+2c}{a^2+b^2+c^2}\right)(a-c)(b-c)\geqslant0,\] 这个设 $c=\min\{a,b,c\}$ 应该就可以证出来了。
kuing 10# 2012-8-25 13:59
px 有 ag 或 cs 没?
pxchg1200 11# 2012-8-25 22:42
10# kuing 有是有,就是比较赖皮的那种。。。。明天上
thread-1350-1-2.html: [不等式] 四元的关于abcd的放缩的不等式[未解决]
huyuhbb 1# 2013-4-9 13:10
______kuing edit in $\LaTeX$______ $a+b+c+d=4$, $0\leqslant a$, $b$, $c$, $d\leqslant\dfrac43$,求证 \[(abcd)^3\geqslant(4-3a)(4-3b)(4-3c)(4-3d).\]
reny 2# 2013-4-9 14:44
本帖最后由 reny 于 2013-4-25 22:26 编辑 1# huyuhbb 两边取自然对数,即证$\sum_{cyc}(3lna-ln(4-3a))\geqslant0$, 设$f(x)=3\ln x-\ln(4-3x),x\in(0,\frac43]   (*)$ 易证$当x\in[\frac1 2,\frac4 3],F(x)=3\ln x-\ln(4-3x)-6(x-1)\geqslant0$ 这是因为$F'(x)=\dfrac{6(x-1)(3x-2)}{x(4-3x)}$. 于是当$a,b,c,d\in(\frac1 2,\frac4 3)$时,$\sum_{cyc}(3lna-ln(4-3a))\geqslant \sum_{cyc}6(x-1)=0$. 若存在$a\in(0,\frac1 2)$,怎么办?
kuing 3# 2013-4-9 17:02
2# reny oh,切线法 PS、ln -> \ln (注意空格)
kuing 4# 2013-4-9 17:54
话说,我画图看了看,好像切线并不是恒在下方…… 再看了下,求导求错了
kuing 5# 2013-4-9 18:20
5# reny 还是不对,应为 \[F'(x)=\frac{6 (x-1) (3 x-2)}{x (4-3 x)}\]
kuing 6# 2013-4-9 18:27
其实换个元似乎就是显然的了。 令 $4-3a=x$, $4-3b=y$, $4-3c=z$, $4-3d=w$,则由条件知 $x$, $y$, $z$, $w\in [0,4]$ 且 $x+y+z+w=4$,这样就有 $a=(4-x)/3=(y+z+w)/3$ 等,然后就不用我说了吧
reny 7# 2013-4-9 18:44
8# kuing en,还换元一下就到位了.
huyuhbb 8# 2013-4-9 21:49
8# kuing ....居然换元就证了...
yayaweha 9# 2013-4-9 22:50
9# reny 怎么个到位?
kuing 10# 2013-4-9 23:14
11# yayaweha 还真要写下去啊。。
huamahu 11# 2013-4-12 12:17
左边有个三次,,,,,,,.
kuing 12# 2013-4-12 13:24
13# huamahu 咦,哦,我居然忽略了那个次数 那接下去并不是显然了…… 不过现在没心情做题了……
kuing 13# 2013-4-24 20:07
几乎把这个贴给忘了…… 如果接着8#的代换, 其实换个元似乎就是显然的了。 令 $4-3a=x$, $4-3b=y$, $4-3c=z$, $4-3d=w$,则由条件知 $x$, $y$, $z$, $w\in [0,4]$ 且 $x+y+z+w=4$,这样就有 $a=(4-x)/3=(y+z+w)/3$ 等,然后就不用我说了吧 kuing 发表于 2013-4-9 18:27 齐次化后等价于 \[4^8(x+y+z)^3(y+z+w)^3(z+w+x)^3(w+x+y)^3\geqslant 3^{12}xyzw(x+y+z+w)^8,\] 而这其实并不是显然的(我当时把左边的3次搞丢了)甚至可能是很难进行下去的…… 待续……
yes94 14# 2013-4-24 21:02
见到这些不等式一次,就受虐一次。
李斌斌755 15# 2013-4-25 04:12
16# yes94 不等式与我来说无异于对牛弹琴
yes94 16# 2013-4-25 19:49
17# 李斌斌755 呵呵
thread-1351-1-1.html: 这真是贪吃蛇
isea 1# 2013-4-10 09:55
啧啧
kuing 2# 2013-4-10 13:52
次奥,太牛比了……
isea 3# 2013-4-10 20:27
最后是俄文吗?
thread-1352-1-2.html: [组合] kuing酱那个组合搞定木有?
Gauss门徒 1# 2013-4-10 10:13
棋盘那个~
kuing 2# 2013-4-10 12:40
不会…… 把你所描述的题目打上来: 一个2003*2003的正方形棋盘,第一行有两个缺格A、B,两缺格相距奇数个格,现在准备放入1*2的矩形将棋盘覆盖,但不覆盖其中一个缺格。 证明不覆盖缺格A的方法数=不覆盖缺格B的方法数。 应该没打错吧?
Gauss门徒 3# 2013-4-11 12:49
不会也得会!
realnumber 4# 2013-4-11 13:54
本帖最后由 realnumber 于 2013-4-11 13:57 编辑 两缺格相距,是什么意思呢?还有"奇数格" 比如一个在(1,1),一个(2,2),算奇数? ---ps问题还不知道怎么入手呢... ----在两个对称点,比如角上,应该一样多,算奇数格了
kuing 5# 2013-4-11 13:55
两缺格相距,是什么意思呢?还有"奇数格" 比如一个在(1,1),一个(2,2),算奇数? ---ps问题还不知道怎么入手呢... realnumber 发表于 2013-4-11 13:54 是“第一行有两个缺格A、B,两缺格相距奇数个格”
kuing 6# 2013-4-11 22:02
顶一下继续求助……
Gauss门徒 7# 2013-4-16 01:33
This problem is an old one too. There is a similar one in the book 组合问题的方法与技巧-Mr.Dan Zun
thread-1353-1-2.html: [不等式] 以色列2012不等式赛题
huyuhbb 1# 2013-4-10 13:08

yes94 2# 2013-4-10 19:51
1# huyuhbb 搞点答案来欣赏一下,
huyuhbb 3# 2013-4-10 21:59
2# yes94 不会...第二个找到的答案居然是拉格朗日数乘...
huyuhbb 4# 2013-4-11 22:07
第一个求助。。
yes94 5# 2013-4-12 18:53
4# huyuhbb 那个严文兰搞了4种方法吧,没仔细看,现在没心思看了
reny 6# 2013-4-18 18:55
本帖最后由 reny 于 2013-4-18 18:58 编辑 (2)在http://www.artofproblemsolving.c ... srael+2012#p2884315找到帖子,也没啥好方法
thread-1354-1-1.html: 来个极限吧
pxchg1200 1# 2013-4-10 13:38
证明 \[ \lim_{n\rightarrow\infty}{\left[\sqrt[n+1]{(n+1)!}-\sqrt[n]{n!}\right]}=\frac{1}{e} \]
kuing 2# 2013-4-10 14:26
差点以为是取整……
isea 3# 2013-4-10 20:42
本帖最后由 isea 于 2013-4-10 20:51 编辑 这个方括号的确不太“美”好 不过,命题,美 偶这个都搞不定:$\displaystyle \lim_ {n\to\infty} {\frac {\sqrt[n]{n!}}n}$ n\to\infty下标,汗
地狱的死灵 4# 2013-4-10 21:26
\begin{array}{l} &\mathop {\lim }\limits_{n \to \infty } \frac{{\sqrt[n]{{n!}}}}{n} \\ & = e^{\mathop {\lim }\limits_{n \to \infty } \frac{1}{n}\ln \frac{{n!}}{{n^n }}}  \\ & = e^{\mathop {\lim }\limits_{n \to \infty } \frac{1}{n}\sum\limits_{i = 1}^n {\ln \frac{i}{n}} }  \\ & = e^{\int_0^1 {\ln xdx} }  \\ & = \frac{1}{e} \\ \end{array}
pxchg1200 5# 2013-4-12 13:39
本帖最后由 pxchg1200 于 2013-4-12 13:42 编辑 4# 地狱的死灵 楼上用O.Stolz定理还需要验证那个极限是存在的。 西哥表示有 \[ \left(\frac{n}{n+1}\right)^{n+1}<\sqrt[n+1]{(n+1)!}-\sqrt[n]{n!}<\left(\frac{n}{n+1}\right)^{n} \] 可以AM-GM,可是我只AM-GM出右边那一半,左边弄不出。。
pxchg1200 6# 2013-4-19 23:19
4# 地狱的死灵 TechPaperFurdui.pdf (95.52 KB) 找到篇文章证递减的。
thread-1355-1-2.html: [组合] 边长为3的正三角形(含边界),放入9个点
realnumber 1# 2013-4-10 14:29
西宁-乔**(498*******08)  14:05:47 原题是:边长为3的正三角形(含边界),放入10个点,求证:至少有2个点每两点间的距离不大于1.      有人改编成:边长为3的正三角形(含边界),放入9个点,求证:至少有2个点每两点间的距离不大于1. -----原题很简单,改编题怎么证明?
realnumber 2# 2013-4-10 15:23
应该是正确的,先分割成9个正三角形,同原题,可以得9个三角形分别有一个点,现在考虑,大的三角形顶点ABC所在的三个三角形,里面的那个点如果符合要求,则在A点,也符合要求, 如此只需要把这三点固定在ABC 那么问题等价于6个点放如边长为1的正六边形,(且不能在顶点),两两距离是1。---但是把平面上6个半径为0.5的球挤在一起,直观可以感受到,和7个这样的 球一样,球心会形成6边形。--证明不会,没头绪。
零定义 3# 2013-4-15 14:42
猜测:边长为3的正三角形(含边界),最多能放入7个点,使得每两点间的距离都大于1.
yes94 4# 2013-4-15 22:05
还是原题简单,抽屉原理吧?
thread-1356-1-2.html: 推理
李斌斌755 1# 2013-4-10 20:00
一次期末考试后,甲、乙、丙三人都不知道自己的成绩。但彼此都知道每人知道另两人的成绩。而且三个学生均非常聪明。这天,老师:“你们三个人中有不及格的。”甲说:“不知道有没有我。”乙说:“我也不知道我有没有及格。”丙说:“我原来不知道,现在知道了。”甲说:“现在我也知道了!”最后,乙说:“我也知道了!”   根据上述条件,可以提出()   A  甲不及格,乙不及格,丙及格   B  甲不及格,乙及格,丙不及格   C  甲及格,乙不及格,丙不及格
isea 2# 2013-4-10 20:59
本帖最后由 isea 于 2013-4-10 21:03 编辑 没细看,这个那个经典的帽子颜色一样吧 看题了,不太一样。 不过,这种题的方法是假定某个人的正确的,然后从这个条件发现,如果出现矛盾就说明这个人说谎话了,然后就知道结果了 一般是列个表,理清关系 第一感,也许与此题无关,随便聊一下
李斌斌755 3# 2013-4-10 21:55
本帖最后由 李斌斌755 于 2013-4-11 01:43 编辑 据说是公务员试题,答案是B,可我推出B、C都有可能。故上传请大伙看看。
李斌斌755 4# 2013-4-10 22:18
见   http://tieba.baidu.com/p/2257987974
thread-1357-1-1.html: 查看新帖好像停留在4号了
isea 1# 2013-4-10 22:49
看不到4号以后的帖子了
kuing 2# 2013-4-10 22:59
那个纵横搜索偶尔这样,不是第一次了。 话说之前老卡,不知是不是影响到它了。 先不管,或者自己会好,自己就是这样。
thread-1358-1-2.html: [不等式] $\sum1/(x+y)\geqslant3\sqrt3/2$
reny 1# 2013-4-11 14:11
本帖最后由 reny 于 2013-4-11 14:13 编辑 (1)$已知x,y,z>0,xy+yz+zx=1,$是否成立$$\dfrac1{x+y}+\dfrac1{y+z}+\dfrac1{z+x}\geqslant\dfrac{3\sqrt3}{2}$$(具有三角背景) (2)$已知a,b,c>0,abc=1$,求证$$\dfrac{bc}{a+b}+\dfrac{ac}{b+c}+\dfrac{ab}{c+a}\geqslant\dfrac32$$ 出自http://blog.sina.com.cn/s/blog_c27636ef01019o3c.html
kuing 2# 2013-4-11 14:20
第二题老题,不过没找很早,只找到2010年的贴 http://www.artofproblemsolving.c ... p?f=51&t=362789
kuing 3# 2013-4-11 14:25
第一题如无意外下将会不成立,因为如果成立,陈计当年又何必弄那个 伊  朗 96 呢
kuing 4# 2013-4-11 17:22
事实上,$x$, $y$, $z\geqslant0$, $xy+yz+zx=1$ \[\frac1{x+y}+\frac1{y+z}+\frac1{z+x}\geqslant\frac52\] 是一道熟悉的题
reny 5# 2013-4-11 17:53
4# kuing $e,说明等号是(a,b,c)=(1,1,0)$或其循环时取等
reny 6# 2013-4-14 12:52
4# kuing kk,有没有常规方法的链接
kuing 7# 2013-4-14 13:12
6# reny
reny 8# 2013-4-14 14:11
7# kuing 非常感谢
thread-1359-1-1.html: 酷儿理论
kuing 1# 2013-4-11 17:28

isea 2# 2013-4-11 17:49
酷儿是谁?
kuing 3# 2013-4-11 17:57
最近有人这样称呼我……
isea 4# 2013-4-11 18:11
原来是楼主你啊
isea 5# 2013-4-11 18:12
这书大热啊,翻乱了,都
╰☆ヾo.海x 6# 2013-4-14 22:02
3# kuing ...你自己要的 不关我了啊 哈哈哈
kuing 7# 2013-4-14 22:03
6# ╰☆ヾo.海x I like it!
thread-1360-1-2.html: [函数] 一个函数老题目
转化与化归 1# 2013-4-11 17:54
有没有好的解法?
yes94 2# 2013-4-12 18:56
1# 转化与化归 此方法就是好方法! 这是哪里的题目啊,学习一下,谢谢!
第一章 3# 2013-4-12 19:39
启东中学的考前资料见过,略有不同,
第一章 4# 2013-4-12 19:53
第(1)略; 第(2)题采用两边夹的做法,$k=2,m=-1$;
第一章 5# 2013-4-12 19:55
把第(3)题发一下,其实两个题目是一样的
转化与化归 6# 2013-4-12 20:12
2# yes94 江苏省2013届高三南京、淮安联考3月第二次模拟考试的第20题
转化与化归 7# 2013-4-12 20:13
5# 第一章 这个题考过很多次,可惜每次给出的方法全部是一样的。
依然饭特稀 8# 2013-4-13 12:00
最早是05年湖南考得吧。你自己百度了
转化与化归 9# 2013-4-13 13:05
8# 依然饭特稀 题目最早是哪里的不重要,有没有不一样的方法来解答这个题?
yes94 10# 2013-4-13 22:52
8# 依然饭特稀 范特西好厉害! 出处也很重要!
hongxian 11# 2013-4-14 11:56
感觉一阶导数有零点且二阶导数递减(三阶导数小于0),还停在感觉上说理不清楚!
v6mm131 12# 2013-4-14 12:11
这个题齐次化处理即可啊
hongxian 13# 2013-4-14 12:42
12# v6mm131 愿闻其详!谢谢了!
转化与化归 14# 2013-4-14 13:04
12# v6mm131 有好的方法了?
yes94 15# 2013-4-14 17:16
标答就是齐次化处理
isea 16# 2013-4-14 20:46
本帖最后由 isea 于 2013-4-14 20:54 编辑 齐次齐次,我记得我对这类在这里有帖,先看看能不能用我的方法 == 倒 省略了前两问,要自己动手算一下了,先
isea 17# 2013-4-14 21:45
本帖最后由 isea 于 2013-4-14 21:53 编辑 果然,依葫芦画瓢(极点五笔下不是词组?还是我打错了字?)。 参阅 导数 零点 不等式证明 http://kkkkuingggg.5d6d.net/thread-1180-1-5.html 简单文字说明:有函数对称轴推广的意味。 转化成函数$g(x)=x^2-(a-2)x-a\ln x-c$零点有两相异零点. 易知$a>0$。 进一步:$0<x_1<\dfrac a2<x_2$。 先证明:$0<x<\dfrac a2,g(\dfrac a2+x)<g(\dfrac a2-x)$成立。 从而$g(\dfrac a2+(\dfrac a2-x_1))<g(\dfrac a2-(\dfrac a2-x_1))=g(x_1)=0$。 于是$a-x_1<x_2 \Rightarrow \dfrac a2 <\dfrac {x_1+x_2}2$。 而函数$f(x)$在$\dfrac a2,+\infty)$单调递增…… PS:总喜欢把\dfrac中的f打算,哈哈
isea 18# 2013-4-14 21:48
导数 零点 不等式证明 那帖的16楼还有高等数学下的证明,估计,这题,不这类,对数+二次函数,亦可以类似处理
boysxh 19# 2013-4-15 12:01
果然,依葫芦画瓢(极点五笔下不是词组?还是我打错了字?)。 参阅 导数 零点 不等式证明 http://kkkkuingggg.5d6d.net/thread-1180-1-5.html 简单文字说明:有函数对称轴推广的意味。 转化成函数$g(x)=x^ ... isea 发表于 2013-4-14 21:45 我也这么试过 可以的  但感觉答案更常规 http://blog.sina.com.cn/s/blog_b839fe7a0101hkvv.html
hongxian 20# 2013-4-15 15:34
本帖最后由 hongxian 于 2013-4-16 05:36 编辑 感觉一阶导数有零点且二阶导数递减(三阶导数小于0),还停在感觉上说理不清楚! hongxian 发表于 2013-4-14 11:56 试说一下 要证:$x \in \left(0,\dfrac{a}{2}\right)$时,$f(\dfrac a 2-x)>f(\dfrac a 2+x)$ $\Longleftarrow \displaystyle\int_{0}^{x}-f'(\dfrac a 2-x)\mathrm{d}x>\int_{0}^{x}f'(\dfrac a 2+x)\mathrm{d}x$ $\Longleftarrow -f'(\dfrac a 2-x)>f'(\dfrac a 2+x)$ $\Longleftarrow \displaystyle\int_{0}^{x}f''(\dfrac a 2-x)\mathrm{d}x>\int_{0}^{x}f''(\dfrac a 2+x)\mathrm{d}x$ $\Longleftarrow f''(\dfrac a 2-x)>f''(\dfrac a 2+x)$ $f''(x)$单调递减 所以$f''(\dfrac a 2-x)>f''(\dfrac a 2+x)$成立 所以$x \in \left(0,\dfrac{a}{2}\right)$时,$f(\dfrac a 2-x)>f(\dfrac a 2+x)$成立
thread-1360-2-2.html:
转化与化归 21# 2013-4-15 20:34
20# hongxian 证明的第三行要不要加个负号?
hongxian 22# 2013-4-15 20:44
本帖最后由 hongxian 于 2013-4-16 05:37 编辑 21# 转化与化归 这样说起来,第二行,第三行都要加,已经更改,谢谢了!
第一章 23# 2013-4-16 10:56
9# 转化与化归 确实是2005年的考题,后人把它变了又变,今天又碰到一个,
第一章 24# 2013-4-16 11:00
把2005年的湖南卷(理)贴一下
三下五除二 25# 2013-4-22 23:10
楼上哪个题,我有个简单背景分析 http://blog.sina.com.cn/s/blog_68ef13230101aiq6.html
thread-1361-1-2.html: [数列] 刚才人教群里聊的数列玩tanh
kuing 1# 2013-4-11 18:08
+1-1那个方法就不讲了,我的第一反应是玩 tan,后来发现搞错了,符号不对,于是发现其实应该是玩 tanh,注意到 \[\tanh3x=\frac{\tanh^3x+3\tanh x}{3\tanh^2x+1},\] 这样形式就对上了,不过有个问题,就是在取值范围上,但是如果让 $x$ 取复数那应该没什么问题,时间关系就不扯上去了,煮饭先……
isea 2# 2013-4-11 18:15
双曲函数? 冷门的感觉,还是th x
kuing 3# 2013-4-11 18:16
2# isea 嗯,双曲正切,写作 tanh,也有简写成 th 的,不过我还是不太喜欢简写。
hongxian 4# 2013-4-12 09:58
3# kuing $x_{n+1}-1=\dfrac{(x_n-1)^3}{3x_n^2+1}$ $x_{n+1}+1=\dfrac{(x_n+1)^3}{3x_n^2+1}$ $\dfrac{x_{n+1}-1}{x_{n+1}+1}=\left(\dfrac{x_n-1}{x_n+1}\right)^3$ $\dfrac{x_n-1}{x_n+1}=\left(\dfrac{a-1}{a+1}\right)^{3^{n-1}}$ 不知这样行不行?
hongxian 5# 2013-4-12 10:04
本帖最后由 hongxian 于 2013-4-12 10:05 编辑 4# hongxian 1楼没看清楚,现在才看到!只是双曲函数不是非常清楚,要去了解一下。
thread-1362-1-1.html: 其它 随感
isea 1# 2013-4-11 22:46
LaTeX的标题(\title \maketile)不适合排版一张数学试卷标题,直接将标题当成正文来处理了,想想,在word下也从未用个标题功能……
kuing 2# 2013-4-11 23:08
我几乎没用过 maketitle 这东东 试题版面最好自己慢慢设计……
isea 3# 2013-4-11 23:31
翻了一下胡伟写的那本书,好多命令啊,长路漫漫~
thread-1363-1-1.html: 来来来 求围观 哈哈
isea 1# 2013-4-12 02:05
本帖最后由 isea 于 2013-4-12 02:07 编辑 第一个正式用CTeX (版本2.9.2.164)套装打的简单案例(试卷,题有典型性哦,这么晚整理的东西,一定有质量的) 附件是 tex 源与没CTeX准备的pdf 另外,脚注完全是写得玩玩的,删除,或者用%标注掉即可。
isea 2# 2013-4-12 02:11
超链接为什么是红色?不是blue。 算了,睡觉,先。
kuing 3# 2013-4-12 21:07
2# isea hyperref 宏包有很多选项,自己看一下改哪个,我也没记住。
isea 4# 2013-4-13 00:20
本帖最后由 isea 于 2013-4-13 00:31 编辑 2# isea hyperref 宏包有很多选项,自己看一下改哪个,我也没记住。 kuing 发表于 2013-4-12 21:07 查到了 颜色的应用: 1.红色(Red)用来作为内部的链接 2.红紫色(Magenta)用来作为web或者email地址的链接 3.蓝色(有时候是绿色,还有少数是红色)用来标示强调的词或者说明性的词 默认就是红紫色,原来 和M$及其它略不同,其它基本都是blue ============= 其次,再缺省参数下,那个方框,打印PDF时,是不打印的,摘自 “仍然保留超链接的作用的情况下去掉超链接的下划线或者边框有下划线或者边框的pdf文件打印的时候是不会打印边框以及下划线的, 但是如果是彩色的话,超链接文字是彩色的。colorlinks=true在pdf或者dvi中超链接文字的颜色为彩色(红色), colorlinks=false在pdf中超链接文字的颜色为黑色,而且没有下划线,仍有方框。 在dvi中仍有下划线,但是颜色为黑色”
isea 5# 2013-4-18 12:29
本帖最后由 isea 于 2013-4-19 11:16 编辑 原来paflatex通过的,基本xelatex亦能通过 xelatex 能自动调整中英文的间隔,不过,不过,也有个bug,就是 \( 享受不到这种待遇 第一份xelatex大型练习卷,发来,记念一下 (更新为矢量图,用tikz画的,真强大,太适应数学了,算坐标,标点,哈哈,现学现卖) 另外,下载的顺便看看能不明显的录入错误
kuing 6# 2013-4-18 12:59
5# isea 还有人用 \( 吗?…… xltxtra 宏包用来干嘛的?
isea 7# 2013-4-18 13:07
5# isea 还有人用 \( 吗?…… xltxtra 宏包用来干嘛的? kuing 发表于 2013-4-18 12:59 我就用啊,笔记本就这两个比较近 网上说是 XeLaTeX 相关字体字库 英文xtra 就是 extra (特别,额外,附加) 大约就就那意思了,没啥用,忘记删除
kuing 8# 2013-4-18 13:13
xunicode 呢? 我将 xunicode 和 xltxtra 都去掉了,没发现有什么不同…… 另外,我运行的时候还发现有宏包冲突,原因未明,将 \usepackage{amsmath,amssymb,amsthm} 放到 xecjk 前面才没事,你那里居然没问题?
isea 9# 2013-4-18 23:40
本帖最后由 isea 于 2013-4-19 13:55 编辑 xunicode 呢? 我将 xunicode 和 xltxtra 都去掉了,没发现有什么不同…… 另外,我运行的时候还发现有宏包冲突,原因未明,将 \usepackage{amsmath,amssymb,amsthm} 放到 xecjk 前面才没事,你那里居然没问题 ... kuing 发表于 2013-4-18 13:13 这两包全去了,我觉得也没什么用,另外,enumerate 排版起来,风格虽然一样,但大难看了,还看自己手动打啊,跟word自动编号一样,但enumerate 强多了 tikz真牛,刚入门画了这个矢量图,标签真是麻烦啊,还不知,如果不调坐标,怎么调整位置呢 调整位置,依然可以用   picinpar 宏包 ……
isea 10# 2013-4-18 23:42
本帖最后由 isea 于 2013-4-18 23:43 编辑 5楼更新,画了个立体图 其次,要打印,XeLateX 用什么格式的文件来打印效果最好?PDF相对有点“损失” article 默认10pt ,字体跟word的五号相比有点偏小,不过,与数学公式倒是很匹配 标记一下,下次要改默认字体的大小了 长路漫漫……
kuing 11# 2013-4-18 23:45
这两包全去了,我觉得也没什么用,另外,enumerate 排版起来,风格虽然一样,但大难看了,还看自己手动打啊,跟word自动编号一样,但enumerate 强多了 tikz真牛,刚入门画了这个矢量图,标签真是麻烦啊,还 ... isea 发表于 2013-4-18 23:40 没看懂(指你说enumerate的部分); 暂不聊画图。
kuing 12# 2013-4-18 23:48
不曾打印,PDF也会有损失? 字号的话,ctex 里有设置,其实我还是建议初玩可以用ctexart之类的文档类。
isea 13# 2013-4-18 23:49
本帖最后由 isea 于 2013-4-18 23:56 编辑 没看懂(指你说enumerate的部分); 暂不聊画图。 kuing 发表于 2013-4-18 23:45 对,begin{enumerate} ……\end{enumerate} 对整体效果,就是试卷正文,首是括号时,看起来,并不太美 (但也不差)
kuing 14# 2013-4-18 23:52
其实按道理,开头的全角括号本应被压缩,但不知为何未被压缩,原因待查。
isea 15# 2013-4-18 23:53
不曾打印,PDF也会有损失? 字号的话,ctex 里有设置,其实我还是建议初玩可以用ctexart之类的文档类。 kuing 发表于 2013-4-18 23:48 其实写的时候全是 ctexart ,最后用xelatex 编译,以ctexart 为主了 何况对字体没要求 因为会把中英文略分开,漂亮多了 PDF打印效果不如doc会有损,虽然,如果源文档质量后,两者相差不会太多, 不把两打印文件放一起,不会发现
isea 16# 2013-4-18 23:55
其实按道理,开头的全角括号本应被压缩,但不知为何未被压缩,原因待查。 kuing 发表于 2013-4-18 23:52 这个可能由于偶是全部默认的原因吧,哈哈
kuing 17# 2013-4-18 23:59
16# isea 我这里也一直这样,但是我曾经看过别人编译出来的会压缩,我还以为是我这里的版本旧,所以也没在意,然而你装的应该比较新,但也……所以有空要查查……
isea 18# 2013-4-19 00:05
本帖最后由 isea 于 2013-4-19 00:08 编辑 17# kuing 这个简单,肯定只一个标点符号参数的问题,翻翻xeCJK手册肯定有解 不过,现在不想看了 =========== 不对,括号不是标点符号,哈哈哈。
kuing 19# 2013-4-19 01:40
18# isea 算的; 你的xecjk是什么版本?
isea 20# 2013-4-19 11:15
本帖最后由 isea 于 2013-4-19 11:18 编辑 18# isea 算的; 你的xecjk是什么版本? kuing 发表于 2013-4-19 01:40 不知哪儿看,说明手册上是 2.4.5, 用记事本打开 xeCJK.sty 也有 %% %%  Version 2.4.5 (31-Jan-2012) %% %% 字样 我这里xeCJK宏包顺序放导言区哪儿都行,与amsmath很禾口讠皆。 如果指压缩标点 是 指下图的话,加个 \punctstyle{kaiming} 或者 半角式即可。
thread-1363-2-1.html:
isea 21# 2013-4-19 11:21
其次,原来至于打印,PDF不会不比doc差 打印效果不佳有两个地方 一是打印机打印时,将PDF用的图像形式打印的,其二,还有停留在doc2pdf的时,图被转化为点阵, 但 LaTeX 里画图是矢量…
hejoseph 22# 2013-4-19 11:53
本帖最后由 hejoseph 于 2013-4-19 11:56 编辑 两个tex我都略改写过
isea 23# 2013-4-19 12:31
两个tex我都略改写过 hejoseph 发表于 2013-4-19 11:53 多谢,学习!先。 另外 (1)  99.tex 保存时使用的UTF-8吧?直接winedt v7.0打不开(记事本打开,复制出代码后解决) (2)  (I)(II)后面与文字间的距离太大,个人极不喜欢,看着就想改近些,不知如何调整
hejoseph 24# 2013-4-19 12:38
本帖最后由 hejoseph 于 2013-4-19 12:40 编辑 两个都是utf8格式的,98那个我改了下了,这些都可以修改的,详细说明你自己找enumitem宏包说明文档
isea 25# 2013-4-19 12:38
再次感谢,enumerate 消化,反馈要晚几天了
hejoseph 26# 2013-4-19 12:52
对于item最前面是(,因为不作为句首处理,可以这样写 \hspace{-0.5em}( 试卷里既然i用了正写,为什么e不用正写? \pi的正写可以用CMU Serif里的π,这个字体的风格跟默认公式的风格一致。加下面这段就可以了,需要用正写π就用\uppi(无论是否在数学模式下都可以),其余正写希腊字母类似可调用 \makeatletter \newcommand{\allmodesymb}[2]{\relax\ifmmode{\mathchoice {\mbox{\fontsize{\tf@size}{\tf@size}#1{#2}}} {\mbox{\fontsize{\tf@size}{\tf@size}#1{#2}}} {\mbox{\fontsize{\sf@size}{\sf@size}#1{#2}}} {\mbox{\fontsize{\ssf@size}{\ssf@size}#1{#2}}}} \else \mbox{#1{#2}}\fi} \makeatother \newfontfamily\CMU{CMU Serif} \newcommand{\upalpha}{\allmodesymb{\CMU}{\symbol{"03B1}}} \newcommand{\upbeta}{\allmodesymb{\CMU}{\symbol{"03B2}}} \newcommand{\upgamma}{\allmodesymb{\CMU}{\symbol{"03B3}}} \newcommand{\updelta}{\allmodesymb{\CMU}{\symbol{"03B4}}} \newcommand{\upepsilon}{\allmodesymb{\CMU}{\symbol{"03F5}}} \newcommand{\upzeta}{\allmodesymb{\CMU}{\symbol{"03B6}}} \newcommand{\upeta}{\allmodesymb{\CMU}{\symbol{"03B7}}} \newcommand{\uptheta}{\allmodesymb{\CMU}{\symbol{"03B8}}} \newcommand{\upiota}{\allmodesymb{\CMU}{\symbol{"03B9}}} \newcommand{\upkappa}{\allmodesymb{\CMU}{\symbol{"03BA}}} \newcommand{\uplambda}{\allmodesymb{\CMU}{\symbol{"03BB}}} \newcommand{\upmu}{\allmodesymb{\CMU}{\symbol{"03BC}}} \newcommand{\upnu}{\allmodesymb{\CMU}{\symbol{"03BD}}} \newcommand{\upxi}{\allmodesymb{\CMU}{\symbol{"03BE}}} \newcommand{\upomicron}{\allmodesymb{\CMU}{\symbol{"03BF}}} \newcommand{\uppi}{\allmodesymb{\CMU}{\symbol{"03C0}}} \newcommand{\uprho}{\allmodesymb{\CMU}{\symbol{"03C1}}} \newcommand{\upsigma}{\allmodesymb{\CMU}{\symbol{"03C3}}} \newcommand{\uptau}{\allmodesymb{\CMU}{\symbol{"03C4}}} \newcommand{\upupsilon}{\allmodesymb{\CMU}{\symbol{"03C5}}} \newcommand{\upphi}{\allmodesymb{\CMU}{\symbol{"03D5}}} \newcommand{\upchi}{\allmodesymb{\CMU}{\symbol{"03C7}}} \newcommand{\uppsi}{\allmodesymb{\CMU}{\symbol{"03C8}}} \newcommand{\upomega}{\allmodesymb{\CMU}{\symbol{"03C9}}} \newcommand{\upvarepsilon}{\allmodesymb{\CMU}{\symbol{"03B5}}} \newcommand{\upvartheta}{\allmodesymb{\CMU}{\symbol{"03D1}}} %\newcommand{\upvarpi}{\allmodesymb{\CMU}{\symbol{"03D6}}} \newcommand{\upvarrho}{\allmodesymb{\CMU}{\symbol{"03F1}}} \newcommand{\upvarsigma}{\allmodesymb{\CMU}{\symbol{"03C2}}} \newcommand{\upvarphi}{\allmodesymb{\CMU}{\symbol{"03C6}}}
yes94 27# 2013-4-19 18:48
26# hejoseph
isea 28# 2013-4-27 11:53
本帖最后由 isea 于 2013-4-27 11:56 编辑 对于item最前面是(,因为不作为句首处理,可以这样写 \hspace{-0.5em}( 试卷里既然i用了正写,为什么e不用正写? \pi的正写可以用CMU Serif里的π,这个字体的风格跟默认公式的风格一致。加下面这段就可以了 ... hejoseph 发表于 2013-4-19 12:52 解决了\newfontfamily\CMU{CMU Serif}(如果,有看到的编译出错的,参考这帖19楼)后, 再来看这个 果然 比upgreek 宏包字体更佳强 另外,不直立,那是漏网之鱼
isea 29# 2013-4-27 12:09
26# hejoseph yes94 发表于 2013-4-19 18:48 就是直立的希腊字母,如果不希望导言区很长,把这些另存一个tex文件,如名字upgreek.tex 然后在导言区加 \include{upgreek} 即可。
isea 30# 2013-5-2 22:18
本帖最后由 isea 于 2013-5-4 01:43 编辑 再次感谢,enumerate 消化,反馈要晚几天了 isea 发表于 2013-4-19 12:38 对于item最前面是(,因为不作为句首处理,可以这样写 \hspace{-0.5em}( hejoseph 发表于 2013-4-19 12:52 反馈: 1. enumitem 环境下,如果要嵌套 enumitem,需要对其宏包加参数inline,\usepackage[inline]{enumitem},否则能直接用(不用考虑加载宏包) 2. 反复编译后,适合偶的参数是,嵌套内(其余默认,括号是全角括号,标点模式\punctstyle{kaiming}),[align=left,labelsep=0pt,label= \hspace{-1em}(\Roman*)],效果如图 3. labelindent=0pt ,也许是hejoseph 笔误也许版本不同,此参数偶这里要改成 itemindent=0pt  才有效;enumitem 手册 CTEX 才四页,晕,基本是啥也没用,网上找到邓建松的latex2e科技排版指南 一图: ================== 继续反馈 部分源代码 [precode]\begin{itemize} \item[\Heiti 一.] {\Heiti 选择题: 本大题共8小题,每小题4分,共32 分。在每小题给出的四个选项中,只有一项是符合题目要求的} \begin{enumerate}[leftmargin=*] %\item[\Heiti 一.] {\Heiti 选择题: 本大题共8小题,每小题4分,共32 分。在每小题给出的四个选项中,只有一项是符合题目要求的} \item 已知向量$\vv a=(1,x,-2),b=(2,1,x)$,且$\vv a \perp \vv b$,则$x$ 的值为 \cdots \cdots[/precode]
kuing 31# 2013-5-8 22:14
其实按道理,开头的全角括号本应被压缩,但不知为何未被压缩,原因待查。 kuing 发表于 2013-4-18 23:52 换了 texlive 之后,再编译98.tex,果然会压缩了: 但是又会产生这样的问题: 前面那个并没有当作句首处理,所以没有压缩,而第二行的就是句首,所以也压了。
thread-1364-1-1.html: CTex中数学老师常用的导言区都要写啥啊?
老樊 1# 2013-4-12 18:54
大家好,我也想学习学习LATEX,在写的时候不知道我们经常在高中数学中都用哪些宏包啊?还有就是我也想把我的经常用的一些命令和咱论坛的一直,在哪里定义,文档开始之前还是? 谢谢,期待一些源代码的东西啊!!!
isea 2# 2013-4-12 23:13
本帖最后由 isea 于 2013-4-12 23:24 编辑 老帖的3楼,http://kkkkuingggg.5d6d.net/thread-569-1-1.html 老帖的9楼,http://kkkkuingggg.5d6d.net/thread-620-1-1.html 先来小一点文档,我也刚学,在使用中学,如这帖http://kkkkuingggg.5d6d.net/thread-1289-1-2.html,最简单的源文件在 这里  (虽然有小瑕疵,但基本能看)。 再如,复杂一点的,多些剖填空的括号,选择的横线,一页上按题的数量与距离(其实是全自动的)等等:这个的简单东西,再次强调,LaTeX排版是自动的,CTeX文档模版已经加了一些中文处理的宏包,安装即用。 说白了,使劲正文写东西,编译的结果不满意时,再慢慢排版,不要套用word老思路。 试几个就明白了。
老樊 3# 2013-4-16 18:04
2# isea 你好啊,我用Aurora的时候出现了分式的分母显示不完整,怎么办呢??
isea 4# 2013-4-16 21:38
2# isea 你好啊,我用Aurora的时候出现了分式的分母显示不完整,怎么办呢?? 老樊 发表于 2013-4-16 18:04 这个显示不完整,是word的问题,将行距离固定值改改成单倍并勾上对齐到网格 手上只有word2010,如下图,如果找不到,按ctrl+2调整为2倍行距,一般都可显示完整公式。
老樊 5# 2013-4-16 22:59
4# isea
老樊 6# 2013-4-16 23:00
4# isea
isea 7# 2013-4-16 23:12
1298 4# isea 老樊 发表于 2013-4-16 22:59 没碰到过,这种情况,直接上word附件。只需要这一段。
isea 8# 2013-4-17 12:09
会不会是调用字体宏包引起的冲突?
老樊 9# 2013-4-18 22:39
均值不等式例题选讲.rar (120.55 KB) 8# isea 帮忙看看呀,我换了一个电脑还是这样的啊!!!
isea 10# 2013-4-18 23:33
1315 8# isea 帮忙看看呀,我换了一个电脑还是这样的啊!!! 老樊 发表于 2013-4-18 22:39 我打开时也如你的图,双击打开,刷新一下就OK了 或者全部刷新一下公式,点word 栏的 Aurora  ,第二个,勾全去掉,确定。 你看看,我已经刷新的,如图
老樊 11# 2013-4-19 17:40
本帖最后由 老樊 于 2013-4-29 16:24 编辑 10# isea 我的应该是版本问题还是字体问题吧,我打开之后就是这样的,我运行之后还是一样的,你能不能把你的软件给我传一下,谢谢
isea 12# 2013-4-19 22:12
1327 10# isea 我的应该是版本问题还是字体问题吧,我打开之后就是这样的,我运行之后还是一样的,你能不能把你的关键给我传一下,谢谢 老樊 发表于 2013-4-19 17:40 没看懂  “你的关键”  是指的什么? 如果是软件版本,直接在这里下即可:http://kkkkuingggg.5d6d.net/thread-529-2-1.html  20楼 不过,我建议,将宏包用%标一下,用最小的(基本上是默认,加两最常用的数学包 \usepackage{amsmath} \usepackage{amssymb} % \usepackage{euler}  %字体包,效果也不错 \providecommand{\abs}[1]{\left\lvert#1\right\rvert} \providecommand{\norm}[1]{\left\lVert#1\right\rVert} 即可
老樊 13# 2013-4-29 16:24
12# isea 我这几天还是一直在试的时候还是这个问题,我的分式都是分母显示不全的,不知道为什么?哎,我还是喜欢在word中用的,看见别人的难看的数学符号都想改改,哈哈
老樊 14# 2013-4-29 16:42
10# isea 想知道用Aurora的时候能不能也重新定义向量之类的,有时候写个向量太麻烦了
isea 15# 2013-4-29 21:54
14# 老樊 \usepackage{esvect} \vv
老樊 16# 2013-5-5 08:28
本帖最后由 老樊 于 2013-5-5 09:30 编辑 15# isea 求助一下,我的另一个笔记本用着就没问题,可是我的这个定义了写向量的时候出现了好多的提示信息,但是向量也是对的,不知道怎么回事,下面是代码: 我卸载之后又重装了,不知道怎么了,输出的数学符号是能看见 的,可是总是下面有一大堆的提示信息
isea 17# 2013-5-5 19:57
本帖最后由 isea 于 2013-5-5 20:20 编辑 16# 老樊 没什么错误,看上去,难道是宏包多了引起的?默认设置下应该没这个“log”呀。 这个太专业了,建议去CTeX论坛提问。
老樊 18# 2013-5-5 23:18
17# isea 谢谢啦,我重装了系统,好了,也没有以前的问题了,不过还是谢谢你啊
thread-1365-1-2.html: [几何] 这是2010年某论坛某贴子保存下来的图……
kuing 1# 2013-4-13 09:28
PS、刚才人教群里又提起此题(的类似)故此有所一发。
李斌斌755 2# 2013-4-13 14:06
某年高考题:   求边长为1的正方体,相邻面中心点连线组成的多面体的体积
yes94 3# 2013-4-13 22:48
某年高考题:   求边长为1的正方体,相邻面中心点连线组成的多面体的体积 李斌斌755 发表于 2013-4-13 14:06 今天的公务员考试题之一
李斌斌755 4# 2013-4-14 00:21
3# yes94 公务员考的是哲学
kuing 5# 2013-4-14 00:24
跑题跑得好快……
李斌斌755 6# 2013-4-14 00:53
5# kuing 2003辽宁卷第八题
yes94 7# 2013-4-14 17:19
昨天上午的公务员考试题(《行政职业能力测验》)之一
thread-1367-1-1.html: 无意中又发现一可以自定义header的博客
kuing 1# 2013-4-13 18:32
http://kuingggg.blogbus.com/ 可以自定义 header,意味着可以跟本论坛一样加载 MathJax 用以编译 $\LaTeX$ 公式。
isea 2# 2013-4-13 22:08
呵呵
kuing 3# 2013-4-13 22:11
算是介绍给你们吧,我是不需要用的,我还是喜欢玩论坛,1#的博客如无意外都将会荒废掉……
isea 4# 2013-4-13 22:12
我压根就不注册……
thread-1368-1-2.html: [不等式] 切线?
yayaweha 1# 2013-4-13 20:46
若$a_i>0且\sum_{i=1}^{n} a_i=1$, 证明:$$(a_1+\frac{1}{a_1})(a_2+\frac{1}{a_2}) \cdots  (a_n+\frac{1}{a_n})\ge (\frac{n^2+1}{n})^n$$
kuing 2# 2013-4-13 21:08
holder+A-G 吧
yayaweha 3# 2013-4-13 21:19
this?
kuing 4# 2013-4-13 21:24
也行。 我的意思是 \[\prod_{k=1}^n\left(a_k+\frac1{a_k}\right)\geqslant\left(\sqrt[n]{a_1a_2\cdots a_n}+\frac1{\sqrt[n]{a_1a_2\cdots a_n}}\right)^n\geqslant\cdots\]
kuing 5# 2013-4-13 21:33
括号里是加的情况比较简单,之前研究过减的http://kkkkuingggg.5d6d.net/thread-982-1-1.html
yayaweha 6# 2013-4-13 21:40
4# kuing 这是什么不等式
kuing 7# 2013-4-13 21:45
6# yayaweha 就是我2#说的holder不等式,当然你也可以看成是Carlson不等式,因为次数相同…… 而关于holder不等式的证明可以参考N年前我QQ空间扯过的http://user.qzone.qq.com/249533164/blog/1247282727里面的引理4,还有包括了一些相关不等式的证明,也可以顺便参考下。
yayaweha 8# 2013-4-13 22:13
7# kuing 有用的东西
yayaweha 9# 2013-4-13 22:18
4# kuing $$\sqrt[n]{a_1a_2\cdot a_n}+\frac{1}{\sqrt[n]{a_1+a_2\cdot a_n}}$$ 下面怎么证?
kuing 10# 2013-4-13 22:19
均值+双勾函数单调性……
yayaweha 11# 2013-4-13 22:21
我只知道$\sqrt[n]{a_1a_2\cdot a_n}\le n$ $\frac{1}{\sqrt[n]{a_1+a_2\cdot a_n}}\ge \frac{1}{n}$
yayaweha 12# 2013-4-13 22:29
本帖最后由 yayaweha 于 2013-4-13 22:38 编辑 好像这两个较简单些,对原不等式取对数,用琴生不等式或切线法,求导证明$$ln(x+\frac{1}{x})\ge \frac{n-n^3}{n^2+1}x-\frac{1-n^2}{1+n^2}+ln(n+\frac{1}{n})$$
yayaweha 13# 2013-4-13 22:30
10# kuing 我不懂,写出来吧,K
yes94 14# 2013-4-13 22:35
某省三月二模压轴?
yayaweha 15# 2013-4-13 22:36
不是模拟题,就是作业
yes94 16# 2013-4-13 22:53
某省三月二模压轴也是这题,用的切线法
kuing 17# 2013-4-13 22:59
11# yayaweha 是 $\sqrt[n]{a_1a_2\cdots a_n}\leqslant1/n$
yayaweha 18# 2013-4-13 23:17
17# kuing 怎么双钩+均值?
kuing 19# 2013-4-14 00:13
记 $t=\sqrt[n]{a_1a_2\cdots a_n}$,由均值知 $t\in(0,1/n]$,双勾 $t+1/t$ 在 $(0,1)$ 递减……
thread-1369-1-1.html: 质量守恒……..
isea 1# 2013-4-13 22:06
本帖最后由 isea 于 2013-4-13 22:08 编辑 或曰时代变迁
kuing 2# 2013-4-13 22:08
以前看过……
thread-137-1-2.html: 直纹面
图图 1# 2011-10-24 14:21
证明单叶双曲面的同族的两条直母线异面,且同族的任意三条直母线都不平行于同一平面
kuing 2# 2011-10-24 16:55
第一次玩这种东东,完全不熟行的说,看着显然,就是不会证…… 搞第一个先,单叶双曲面为 \[\frac{x^{2}}{a^{2}}+\frac{y^{2}}{b^{2}}-\frac{z^{2}}{c^{2}}=1,\] 其中 $a,b,c>0$。设其上某直母线 $L_1$ 为 \[\left\{\begin{aligned} \lambda _{1}\left( \frac{x}{a}+\frac{z}{c} \right)&=\lambda _{2}\left( 1+\frac{y}{b} \right), \\ \lambda _{2}\left( \frac{x}{a}-\frac{z}{c} \right)&=\lambda _{1}\left( 1-\frac{y}{b} \right), \end{aligned}\right.\] 其中 $\lambda_1,\lambda_2$ 不同为 0。不妨设 $\lambda_2\ne0$ 时另一同族直母线 $L_2$ 为 \[\left\{\begin{aligned} \lambda _{3}\left( \frac{x}{a}+\frac{z}{c} \right)&=\lambda _{2}\left( 1+\frac{y}{b} \right), \\ \lambda _{2}\left( \frac{x}{a}-\frac{z}{c} \right)&=\lambda _{3}\left( 1-\frac{y}{b} \right), \end{aligned}\right.\] 其中 $\lambda_3\ne\lambda_1$。 令 $z=0$ 可以求出 $L_1$ 过点 $\displaystyle P_1\left(\frac{2 a \lambda_1\lambda_2}{\lambda_1^2+\lambda_2^2},\frac{b(\lambda_1^2-\lambda_2^2)}{\lambda_1^2+\lambda_2^2},0\right)$,$L_2$ 过点 $\displaystyle P_2\left(\frac{2 a \lambda_3\lambda_2}{\lambda_3^2+\lambda_2^2},\frac{b(\lambda_3^2-\lambda_2^2)}{\lambda_3^2+\lambda_2^2},0\right)$,又 $L_1, L_2$ 的方向向量分别为 \begin{align*} \overrightarrow{n_{1}}&=\left( \frac{\lambda _{1}}{a},-\frac{\lambda _{2}}{b},\frac{\lambda _{1}}{c} \right)\times \left( \frac{\lambda _{2}}{a},\frac{\lambda _{1}}{b},-\frac{\lambda _{2}}{c} \right),\\ \overrightarrow{n_{2}}&=\left( \frac{\lambda _{3}}{a},-\frac{\lambda _{2}}{b},\frac{\lambda _{3}}{c} \right)\times \left( \frac{\lambda _{2}}{a},\frac{\lambda _{3}}{b},-\frac{\lambda _{2}}{c} \right), \end{align*} 而直线异面的充要条件是 $\overrightarrow{P_{1}P_{2}}\cdot \left( \overrightarrow{n_{1}}\times \overrightarrow{n_{2}} \right)\ne 0$,代入上面的数据可知 \[\overrightarrow{P_{1}P_{2}}\cdot \left( \overrightarrow{n_{1}}\times \overrightarrow{n_{2}} \right)=-\frac{4\lambda_2^2(\lambda _1-\lambda _3)^2}{abc},\] 的确不为 0。
kuing 3# 2011-10-25 13:52
续第二个 再设有另一直线 $L_3$ 为 \[\left\{\begin{aligned} \lambda _{4}\left( \frac{x}{a}+\frac{z}{c} \right)&=\lambda _{2}\left( 1+\frac{y}{b} \right), \\ \lambda _{2}\left( \frac{x}{a}-\frac{z}{c} \right)&=\lambda _{4}\left( 1-\frac{y}{b} \right), \end{aligned}\right.\] 其中 $\lambda_1,\lambda_3,\lambda_4$ 互不相等。 这样,三条直线的方向向量分别为 \begin{align*} \overrightarrow{n_{1}}&=\left( \frac{\lambda _{1}}{a},-\frac{\lambda _{2}}{b},\frac{\lambda _{1}}{c} \right)\times \left( \frac{\lambda _{2}}{a},\frac{\lambda _{1}}{b},-\frac{\lambda _{2}}{c} \right),\\ \overrightarrow{n_{2}}&=\left( \frac{\lambda _{3}}{a},-\frac{\lambda _{2}}{b},\frac{\lambda _{3}}{c} \right)\times \left( \frac{\lambda _{2}}{a},\frac{\lambda _{3}}{b},-\frac{\lambda _{2}}{c} \right), \\ \overrightarrow{n_{3}}&=\left( \frac{\lambda _{4}}{a},-\frac{\lambda _{2}}{b},\frac{\lambda _{4}}{c} \right)\times \left( \frac{\lambda _{2}}{a},\frac{\lambda _{4}}{b},-\frac{\lambda _{2}}{c} \right), \end{align*} 而 $L_1,L_2,L_3$ 不平行于同一平面等价于 $(\overrightarrow{n_1}\times\overrightarrow{n_2})\times(\overrightarrow{n_1}\times\overrightarrow{n_3})\ne \vec0$,代入以上数据,为 \[(\overrightarrow{n_1}\times\overrightarrow{n_2})\times(\overrightarrow{n_1}\times\overrightarrow{n_3})=\left( \frac{4\lambda _{2}^{3}(\lambda _{1}^{2}-\lambda _{2}^{2})M}{a^{2}b^{3}c^{3}},-\frac{8\lambda _{1}\lambda _{2}^{4}M}{a^{3}b^{2}c^{3}},-\frac{4\lambda _{2}^{3}(\lambda _{1}^{2}+\lambda _{2}^{2})M}{a^{3}b^{3}c^{2}} \right),\] 其中 $M=(\lambda _{1}-\lambda _{3})(\lambda _{1}-\lambda _{4})(\lambda _{3}-\lambda _{4})\ne 0$,可见显然不为 $\vec0$。
thread-1370-1-1.html: 最值问题
reny 1# 2013-4-13 22:37
这个不等式可能成立 我想用Mathematic求下最值,确定下它的正确性,怎么写啊 我写的弄不出来
kuing 2# 2013-4-13 22:56
mathematica也不一定能求出来的,多项式都不一定行,三角弄不出也算是正常……
reny 3# 2013-4-13 23:39
那就只好取几组值试试了
thread-1371-1-2.html: [不等式] 一道n元不等式$\sum(a_i/(1+a_i))^2$
huamahu 1# 2013-4-14 10:56
来自人教论坛。 ______kuing edit in $\LaTeX$______ 已知正数列 $\{a_n\}$ 满足 $a_1+a_2+\cdots+a_n=1$,求证 \[\left(\frac{a_1}{1+a_1}\right)^2+\left(\frac{a_2}{1+a_2}\right)^2+\cdots+\left(\frac{a_n}{1+a_n}\right)^2\geqslant\frac n{(n+1)^2}.\]
kuing 2# 2013-4-14 12:06
切线法大概可行…… PS、这不应该叫数列不等式,因为跟数列没什么关系,建议更改标题。 PS2、把原贴链接贴下http://bbs.pep.com.cn/forum.php?mod=viewthread&tid=2730944原题还要求用数归呢
huamahu 3# 2013-4-14 14:49
切线法好像不行。 切线法好像可以证明当n>4时,是成立的。
kuing 4# 2013-4-14 14:52
3# huamahu oh,那元少的情况另外证一下就好了。 其实用普通方法应该都不难,只不过原贴楼主要求数归倒是不好弄……
yayaweha 5# 2013-4-14 15:35
令$f(x)=(\frac{x}{1+x})^2$                Jensen 不等式
kuing 6# 2013-4-14 15:37
5# yayaweha jensen是不行的,半凹半凸。
kuing 7# 2013-4-14 15:45
虽然Jensen不行,但是调整行。
yayaweha 8# 2013-4-14 15:45
6# kuing 0到1也是半凸半凹吗?
kuing 9# 2013-4-14 15:47
8# yayaweha (0,0.5) 下凸,(0.5,1) 上凸
kuing 10# 2013-4-14 15:50
不过也可以退一步用Jensen,比如说设 $a_1\leqslant a_2\leqslant\cdots\leqslant a_n$,这样必然只有 $a_n$ 才有可能到 $(0.5,1)$ 里面去,这样就可以对前面 $n-1$ 个元用Jensen,最后变成一元函数求最值。
yayaweha 11# 2013-4-14 15:55
10# kuing 那个函数求最值好像不是很好求
kuing 12# 2013-4-14 16:51
11# yayaweha 所以还是不用Jensen了。 这样做就比较简单了: 引理: 设 $f(x)=\bigl(x/(1+x)\bigr)^2$,则当 $x$, $y\ne-1$ 且 $x+y\in[-1,1]$ 时恒有 $f(x)+f(y)\geqslant 2f\bigl((x+y)/2\bigr)$,等号成立当且仅当 $x=y$。 引理的证明: 由条件及均值不等式得 \begin{align*} f(x)+f(y)-2f\left( \frac{x+y}2 \right)&=\frac{\bigl(2-(x+y)^2-2xy(x+y+1)\bigr)(x-y)^2}{(1+x)^2(1+y)^2(2+x+y)^2} \\ & \geqslant \frac{\left( 2-(x+y)^2-\frac{(x+y)^2}2(x+y+1) \right)(x-y)^2}{(1+x)^2(1+y)^2(2+x+y)^2} \\ & =\frac{(1-x-y)(x-y)^2}{2(1+x)^2(1+y)^2} \\ & \geqslant 0. \end{align*} 回到原题,我们让各 $a_i$ 能取 $0$,那么左边必然存在最小值,假设左边取最小值时各 $a_i$ 不全相等,比如说 $a_i\ne a_j$,则由引理知将 $a_i$, $a_j$ 都变成 $(a_i+a_j)/2$ 时左边将更小,从而矛盾,因此左边取最小值时必然各 $a_i$ 都相等,即都为 $1/n$,此时左右相等,故原不等式得证。
kuing 13# 2013-4-14 17:29
10# kuing 那个函数求最值好像不是很好求 yayaweha 发表于 2013-4-14 15:55 不求最值也行,直接与右边作差更方便些,因为知道取等条件,可以预先知道因式。 当 $n\geqslant3$ 时,如10#所说的那样,记 $a_n=x\in(0,1)$,则只要证 \[(n-1)\left(\frac{1-x}{n-x}\right)^2+\left(\frac x{1+x}\right)^2\geqslant\frac n{(n+1)^2},\] 作差并因式分解为 \[\frac{(nx-1)^2\bigl(n(x-1)^2+2x^2+n^2-2n-1\bigr)}{(n+1)^2(n-x)^2(x+1)^2}\geqslant0,\] 显然成立。
thread-1372-1-2.html: 一道求面积的题
转化与化归 1# 2013-4-14 15:52
有没有其他的解法?
kuing 2# 2013-4-14 16:05
你的解法已经很好了啊,定积分之类的其他解法显然更复杂,何必……
转化与化归 3# 2013-4-14 16:41
2# kuing 这是高二希望杯附加题的第二题,所以想知道,对高二学生而言的那种解法是什么?
kuing 4# 2013-4-14 16:44
3# 转化与化归 你的解法高二不能理解?
isea 5# 2013-4-14 19:41
神一般的转化!
转化与化归 6# 2013-4-14 20:45
4# kuing 估计有些学生还不能!主要看看有没有不需变换的初等解法!
kuing 7# 2013-4-14 20:48
6# 转化与化归 用投影什么的来讲可能好理解些……
isea 8# 2013-4-14 21:54
搞竞赛的,必须懂这个变换,否则去不了竞赛
李斌斌755 9# 2013-4-14 22:40
几何画板画出这图,不会玩……
kuing 10# 2013-4-14 23:56
9# 李斌斌755 显然没画对
李斌斌755 11# 2013-4-15 00:21
是这样
kuing 12# 2013-4-15 00:28
11# 李斌斌755 嗯,这个应该正确了
yes94 13# 2013-4-15 22:07
10# kuing 怎么画啊? 用$\pi ab$公式?
isea 14# 2013-4-15 22:57
10# kuing 怎么画啊? 用$\pi ab$公式? yes94 发表于 2013-4-15 22:07 汗啊,汗啊 那个明显是几何画板画的 分段求出y,分段画 分段……
李斌斌755 15# 2013-4-16 02:05
本帖最后由 李斌斌755 于 2013-4-23 09:48 编辑 13# yes94 我都不知道啥是$\pi ab$公式
yes94 16# 2013-4-16 21:51
11# 李斌斌755 切开的苹果
转化与化归 17# 2013-4-16 21:54
16# yes94 苹果曲线?
yes94 18# 2013-4-17 13:16
17# 转化与化归 你觉得像不像?
thread-1373-1-1.html: 怎么觉得我是歌手的歌
isea 1# 2013-4-14 22:00
好多不合太慢,不合我的胃口,听着没感觉呢 但羽泉的老歌新唱,男人哭吧哭吧不是罪,很有感觉,不知道是第几期时原了 百度mp3在线专题里有所有歌曲,有空的可以去听听
kuing 2# 2013-4-14 22:05
表示还是听旧歌比较多……
isea 3# 2013-4-14 22:14
我是歌手 基本全是老歌
isea 4# 2013-4-14 22:15
第一期: 羽 泉 - 心似狂潮 陈 明 - 等你爱我 尚雯婕 - 最终信仰 黄贯中 - 海阔天空 黄绮珊 - 等待 沙宝亮 - 飘 齐 奏 - 夜夜夜夜 第二期: 黄绮珊 - 离不开你 齐 奏 - 用心良苦 尚雯婕 - 你怎么舍得我难过 陈 明 - 当我想你的时候 沙宝亮 - 秋意浓 黄贯中 - 吻别 羽 泉 - 烛光里的妈妈 第三期: 沙宝亮 - 浏阳河 尚雯婕 - Let's Get It Started 羽 泉 - 老男孩 陈 明 - 情人的眼泪 齐 奏 - 张三的歌 黄绮珊 - I Will Always Love You 杨宗纬 - 矜持 黄贯中 - 我终于失去了你 第四期: 齐 奏 - 如果云知道 沙宝亮 - 你把我灌醉 羽 泉 - 热情的沙漠 黄绮珊 - 跟着感觉走 陈 明 - 心痛的感觉 杨宗纬 - 征服 尚雯婕 - 可惜不是你 第五期: 林志炫 - 没离开过 羽 泉 - 再回首 尚雯婕 - Man In The Mirror 黄绮珊 - 牵手 陈 明 - 日不落 沙宝亮 - 女人的选择 周晓鸥 - 爱不爱我 杨宗纬 - 空白格 第六期: 沙宝亮 - 让每个人都心碎 羽 泉 - 我想有个家 尚雯婕 - 王妃 陈 明 - 新不了情 林志炫 - 烟花易冷 周晓鸥 - 天空 黄绮珊 - 回来 第七期: 沙宝亮 - 鸿雁 黄绮珊 - The Power Of Love 尚雯婕 - Moves Like Jagger 羽 泉 - 大中国 林志炫 - Opera 周晓鸥 - 无情的雨无情的你 辛晓琪 - 领悟 陈 明 - 听说爱情回来过 第八期: 沙宝亮 - 你快回来 黄绮珊 - 让我一次爱个够 辛晓琪 - 至少还有你 周晓鸥 - 无地自容 尚雯婕 - Super Star 羽 泉 - 爱 林志炫 - 你的眼神 第九期: 黄绮珊 - 剪爱 林志炫 - Making Love Out Of Nothing At All 羽 泉 - 男人哭吧不是罪 沙宝亮 - 死了都要爱 辛晓琪 - 亲爱的小孩 周晓鸥 - 不再掩饰 彭佳慧 - 走在红毯那一天 尚雯婕 - Dog Days Are Over 第十期: 林志炫 - 夜夜夜夜 彭佳慧 - 残酷的温柔 黄绮珊 - 不让我的眼泪陪我过夜 羽 泉 - 狂流 沙宝亮 - 往事随风 辛晓琪 - 原来的我 周晓鸥 - 九个太阳 齐 奏 - 我只在乎你 第十一期:(复活赛) 第一场: 黄贯中 - 特别的爱给特别的你 尚雯婕 - Love Warrior 战 杨宗纬 - 流浪记 陈 明 - 寂寞让我如此美丽 沙宝亮 - 最初的信仰 第二场: 沙宝亮 - Someone Like You 陈 明 - 梨花又开放 杨宗纬 - 馋 尚雯婕 - 我 黄贯中 - 我就是这样的 第十二期:(半决赛) 周晓鸥 - 流量无悔 黄绮珊 - 一样的月光 彭佳慧 - One Moment In Time 林志炫 - 断了线+回家 辛晓琪 - Memory 羽 泉 - 大地 杨宗纬 - 最爱
kuing 5# 2013-4-14 22:33
哦,我没看过,还以为是那些新秀大赛之类的那些……
李斌斌755 6# 2013-4-14 23:39
3# isea 歌手
isea 7# 2013-4-14 23:51
3# isea 歌手 李斌斌755 发表于 2013-4-14 23:39 这目录是公开的呀,到处有啊 转载而已
isea 8# 2013-4-14 23:52
难得你也来水区,哈哈
kuing 9# 2013-4-15 00:06
7# isea 他可能以为你是歌手,因为你说“我是歌手”……
李斌斌755 10# 2013-4-16 19:06
9# kuing 我知道isea应该是北京老师,我的意思是isea是老师中的歌手。
isea 11# 2013-4-16 21:57
10# 李斌斌755 哈哈,把排列公式的字母写成P,不写成A,这样一看,估计和yes94兄一样,算是“小兄长”了
李斌斌755 12# 2013-4-16 22:59
是“老兄长”了,八十年代高中毕业生
kuing 13# 2013-4-16 23:00
07年高中毕业生lu过……
isea 14# 2013-4-16 23:51
是“老兄长”了,八十年代高中毕业生 李斌斌755 发表于 2013-4-16 22:59 天啊,那学这些LaTeX代码是比较吃力~,会,我想~
isea 15# 2013-4-16 23:54
07年高中毕业生lu过…… kuing 发表于 2013-4-16 23:00 年青,真好! 话说,这个有些偏低,或者说,这个大括号能调整么 cases环境,kuing.
kuing 16# 2013-4-17 00:03
15# isea 行距太大。 如果不想改变整体行距,可以局部改,也就是在那个公式里改。
isea 17# 2013-4-17 00:08
16# kuing 我google一下去
isea 18# 2013-4-17 00:15
15# isea 行距太大。 如果不想改变整体行距,可以局部改,也就是在那个公式里改。 kuing 发表于 2013-4-17 00:03 公式内,\\[+8pt],基本看着居中了,凑合着吧,哈哈
kuing 19# 2013-4-17 00:15
17# isea 简单点就在 \begin{cases} 前面加 \linespread{...}\selectfont 就可以了啊
kuing 20# 2013-4-17 00:22
\[f(x)= \linespread{1.5}\selectfont\begin{cases} ax+1,&-1\le x<0,\\ \dfrac{bx+2}{x+1},&0\le x\le1, \end{cases}\] \[f(x)= \linespread{1}\selectfont\begin{cases} ax+1,&-1\le x<0,\\ \dfrac{bx+2}{x+1},&0\le x\le1, \end{cases}\] \[f(x)= \linespread{0.8}\selectfont\begin{cases} ax+1,&-1\le x<0,\\ \dfrac{bx+2}{x+1},&0\le x\le1, \end{cases}\] 这里用不了,自己复制到 latex 里对比一下吧。 在线预览里面的效果: 你甚至可以新定义一个环境,带个可选参数,代替原先的 cases 。
thread-1373-2-1.html:
isea 21# 2013-4-17 00:45
本帖最后由 isea 于 2013-4-17 00:54 编辑 姜还是老的辣啊 好用 再两行间添 [1em] 好多了 谢过
isea 22# 2013-4-17 00:54
闪人 88
kuing 23# 2013-4-17 00:58
22# isea 今晚闪得还算早啊……
yes94 24# 2013-4-17 23:31
23# kuing 原来在这里吹水啊
李斌斌755 25# 2013-4-25 11:36
今天才知道“我是歌手”是电视节目
hejoseph 26# 2013-4-25 12:38
本帖最后由 hejoseph 于 2013-4-25 12:42 编辑 年青,真好! 话说,这个有些偏低,或者说,这个大括号能调整么 cases环境,kuing. isea 发表于 2013-4-16 23:54 别用cases环境,这样写就好 \[ f(x) = \left\{ \begin{aligned} & ax+1, & & {-}1 \le x < 0, \\ & \frac{bx+2}{x+1}, & & 0 \le x \leqslant 1, \end{aligned} \right. \]
kuing 27# 2013-4-25 12:45
别用cases环境,这样写就好 \[ f(x) = \left\{ \begin{aligned} & ax+1, & & {-}1 \le x < 0, \\ & \frac{bx+2}{x+1}, & & 0 \le x \leqslant 1, \end{aligned} \right. \] hejoseph 发表于 2013-4-25 12:38 在他的行距设置下用这个跟 cases 差不了多少,主要还是行距大,这里显示不出来。
hejoseph 28# 2013-4-25 12:56
本帖最后由 hejoseph 于 2013-4-25 12:57 编辑 我觉得还是用aligned那种好点,起码不用dfrac,公式过大的时候还不会距离过近 \[ f(x)= \begin{cases} \dfrac{ax+1}{x+1},&-1\le x<0,\\ \dfrac{bx+2}{x+1},&0\le x\le1, \end{cases} \] \[ f(x) = \left\{ \begin{aligned} & \frac{ax+1}{x+1}, & & {-}1 \le x < 0, \\ & \frac{bx+2}{x+1}, & & 0 \le x \leqslant 1, \end{aligned} \right. \]
realnumber 29# 2013-4-25 14:52
果然好看点,表示都没尝试去用,91年的高中毕业路过~~~
李斌斌755 30# 2013-4-25 17:24
又一用$P$的
realnumber 31# 2013-4-25 18:50

isea 32# 2013-4-26 14:25
我觉得还是用aligned那种好点,起码不用dfrac,公式过大的时候还不会距离过近 \[ f(x)= \begin{cases} \dfrac{ax+1}{x+1},&-1\le x hejoseph 发表于 2013-4-25 12:56 学习了 & & 加一个& 竟然有空格的效果,结果加了四五个,一看,哈哈,壮观的空格
isea 33# 2013-4-26 14:29
果然好看点,表示都没尝试去用,91年的高中毕业路过~~~ realnumber 发表于 2013-4-25 14:52 又一个辣的老姜
isea 34# 2013-4-26 14:35
本帖最后由 isea 于 2013-4-26 14:43 编辑 类似的 $\left.\begin{aligned} & x+\frac y3=5\\& x=0 \end{aligned}\right\} \riff y=0$ $\left.\begin{aligned} && x+\frac y3=5\\&& x=0 \end{aligned}\right\} \riff y=0$
kuing 35# 2013-4-26 14:49
32# isea \begin{align*} a_1 & a_2 & a_3 & a_4 & a_5 & a_6 & a_7 & a_8\\ abc & def & ghi & jil & mno & pqr & stuv & wxyz \end{align*} \begin{align*} a_1 & a_2 & a_3 & a_4 & a_5 & a_6 & a_7 & a_8\\ abc & def & ghi & jil & mno & pqr & stuv & wxyz \end{align*} 这样看就懂了
isea 36# 2013-4-26 14:56
32# isea \begin{align*} a_1 & a_2 & a_3 & a_4 & a_5 & a_6 & a_7 & a_8\\ abc & def & ghi & jil & mno & pqr & stuv & wxyz \end{align*} \begin{align*} a_1 & a_2 & a_3 & a_4 & a_5 & a_6 & a_7 &  ... kuing 发表于 2013-4-26 14:49 原来如此 &也好玩啊
kuing 37# 2013-4-26 15:12
align 系列环境……还是自己看 mathmode 吧,都有介绍。
yes94 38# 2013-4-26 18:10
align 系列环境……还是自己看 mathmode 吧,都有介绍。 kuing 发表于 2013-4-26 15:12 align 系列环境和array的区别与共同之处? \begin{array}{l} P({x_1},{y_1}),Q({x_2},{y_2}),|PQ| = \sqrt {{{({x_1} - {x_2})}^2} + {{({y_1} - {y_2})}^2}} \\ {y_1} = \frac{1}{2}{e^{{x_1}}} = {e^{ - \ln 2}}{e^{{x_1}}} = {e^{{x_1} - \ln 2}} \geqslant {x_1} - \ln 2 + 1\\ {y_2} = \ln 2x_2 = \ln 2 + \ln {x_2} \leqslant \ln 2 + {x_2} - 1\\ {y_1} - {y_2} \geqslant ({x_1} - \ln 2 + 1) - (\ln 2 + {x_2} - 1) = {x_1} - {x_2} - 2\ln 2 + 2\\ if{\kern 1pt} {\kern 1pt} {\kern 1pt} {\kern 1pt} {x_1} - {x_2} - 2\ln 2 + 2 \leqslant 0,{\kern 1pt} {\kern 1pt} {\kern 1pt} {\kern 1pt} {\kern 1pt} {\kern 1pt} {\kern 1pt} {\kern 1pt} {\kern 1pt} then{\kern 1pt} {\kern 1pt} {\kern 1pt} {\kern 1pt} {\kern 1pt} {\kern 1pt} {\kern 1pt} {\kern 1pt} {\kern 1pt} {\kern 1pt} {x_1} - {x_2} \leqslant 2\ln 2 - 2 < 0 \Rightarrow {({x_1} - {x_2})^2} \geqslant {(2\ln 2 - 2)^2}\\ {\kern 1pt} |PQ| = \sqrt {{{({x_1} - {x_2})}^2} + {{({y_1} - {y_2})}^2}}  > \sqrt {{{({x_1} - {x_2})}^2}}  \geqslant \sqrt {{{(2\ln 2 - 2)}^2}}  = \sqrt 2 (1 - \ln 2),\\ so{\kern 1pt} {\kern 1pt} {\kern 1pt} {\kern 1pt} {\kern 1pt} {\kern 1pt} {\kern 1pt} {\kern 1pt} {\kern 1pt} {y_1} - {y_2} \geqslant {x_1} - {x_2} - 2\ln 2 + 2 > 0\\ |PQ| = \sqrt {{{({x_1} - {x_2})}^2} + {{({y_1} - {y_2})}^2}} \\ \geqslant \sqrt {{{({x_2} - {x_1})}^2} + {{({x_1} - {x_2} - 2\ln 2 + 2)}^2}} \\ \geqslant \sqrt {\frac{{{{({x_2} - {x_1} + {x_1} - {x_2} - 2\ln 2 + 2)}^2}}}{2}} \\ = \sqrt {\frac{{{{(2 - 2\ln 2)}^2}}}{2}} \\ = \sqrt 2 (1 - \ln 2) \end{array}
isea 39# 2013-4-26 18:32
align 系列环境……还是自己看 mathmode 吧,都有介绍。 kuing 发表于 2013-4-26 15:12 又是100多页的东东
yes94 40# 2013-4-26 18:45
38# yes94 begin{array}{l} P({x_1},{y_1}),Q({x_2},{y_2}),|PQ| = \sqrt {{{({x_1} - {x_2})}^2} + {{({y_1} - {y_2})}^2}} \\ {y_1} = \frac{1}{2}{e^{{x_1}}} = {e^{ - \ln 2}}{e^{{x_1}}} = {e^{{x_1} - \ln 2}} \geqslant {x_1} - \ln 2 + 1\\ {y_2} = \ln 2x_2 = \ln 2 + \ln {x_2} \leqslant \ln 2 + {x_2} - 1\\ {y_1} - {y_2} \geqslant ({x_1} - \ln 2 + 1) - (\ln 2 + {x_2} - 1) = {x_1} - {x_2} - 2\ln 2 + 2\\ if{\kern 1pt} {\kern 1pt} {\kern 1pt} {\kern 1pt} {x_1} - {x_2} - 2\ln 2 + 2 \leqslant 0,{\kern 1pt} {\kern 1pt} {\kern 1pt} {\kern 1pt} {\kern 1pt} {\kern 1pt} {\kern 1pt} {\kern 1pt} {\kern 1pt} then{\kern 1pt} {\kern 1pt} {\kern 1pt} {\kern 1pt} {\kern 1pt} {\kern 1pt} {\kern 1pt} {\kern 1pt} {\kern 1pt} {\kern 1pt} {x_1} - {x_2} \leqslant 2\ln 2 - 2 < 0 \Rightarrow {({x_1} - {x_2})^2} \geqslant {(2\ln 2 - 2)^2}\\ {\kern 1pt} |PQ| = \sqrt {{{({x_1} - {x_2})}^2} + {{({y_1} - {y_2})}^2}}  > \sqrt {{{({x_1} - {x_2})}^2}}  \geqslant \sqrt {{{(2\ln 2 - 2)}^2}}  = \sqrt 2 (1 - \ln 2),\\ so{\kern 1pt} {\kern 1pt} {\kern 1pt} {\kern 1pt} {\kern 1pt} {\kern 1pt} {\kern 1pt} {\kern 1pt} {\kern 1pt} {y_1} - {y_2} \geqslant {x_1} - {x_2} - 2\ln 2 + 2 > 0\\ |PQ| = \sqrt {{{({x_1} - {x_2})}^2} + {{({y_1} - {y_2})}^2}} \\ \geqslant \sqrt {{{({x_2} - {x_1})}^2} + {{({x_1} - {x_2} - 2\ln 2 + 2)}^2}} \\ \geqslant \sqrt {\frac{{{{({x_2} - {x_1} + {x_1} - {x_2} - 2\ln 2 + 2)}^2}}}{2}} \\ = \sqrt {\frac{{{{(2 - 2\ln 2)}^2}}}{2}} \\ = \sqrt 2 (1 - \ln 2) end{array}
thread-1373-3-1.html:
kuing 41# 2013-4-26 19:52
40# yes94 恶心的代码就不看了……
李斌斌755 42# 2013-5-11 09:56
试试\[f(x)=\begin{cases}ax+1,&-1\leqslant x<0,\\\dfrac{bx+2}{x+1},&0\leqslant x\leqslant1,\end{cases}\]
李斌斌755 43# 2013-5-11 10:38
本帖最后由 李斌斌755 于 2013-5-11 10:42 编辑 \[f(x)=\left\{\begin{aligned} &ax+1,&-1\leqslant x<0,\\&\dfrac{bx+2}{x+1},&0\leqslant x\leqslant1,\end{aligned}\right.\] \[f(x)=\left\{\begin{aligned} &ax+1,&-1\leqslant x<0,\\&\frac{bx+2}{x+1},&0\leqslant x\leqslant1,\end{aligned}\right.\]
李斌斌755 44# 2013-5-11 10:44
{aligned}比{cases}好看
李斌斌755 45# 2013-5-11 10:51
34# isea 你这左大括号咋整?
thread-1374-1-1.html: 2011年江苏卷 原来在7楼源
isea 1# 2013-4-14 23:48
本帖最后由 isea 于 2013-4-16 00:32 编辑 从人教一个帖里的下载的2011江苏卷数学卷,可惜偶这里的无法通过编译。 我想看看成品效果 如果能的话 THX。 ========= 原来是ctex论坛的东西,原作者用的是 TeXLive
kuing 2# 2013-4-15 00:05
用 xelatex 编译。 PS、不行时可以把错误信息发上来。
kuing 3# 2013-4-15 00:16
其实我也没通过 ! Extended mathchar used as mathchar (71311890). <to be read again>                    \relax l.178 \begin{document}                       ? Process has been terminated ...
kuing 4# 2013-4-15 00:33
去问一下该tex文件的作者吧 我也没搞清楚,有的命令我也不懂
isea 5# 2013-4-15 08:42
算了,可能是编译环境相关距大
kuing 6# 2013-4-15 09:00
5# isea 你用xelatex编译过也不行是吗?
isea 7# 2013-4-15 09:11
本帖最后由 isea 于 2013-4-16 00:23 编辑 5# isea 你用xelatex编译过也不行是吗? kuing 发表于 2013-4-15 09:00 我是basic,xelatex过不去,至少字体都不全,如最后卡住(按enter忽略都无效). 不理它,先,罢了 ============= 原来在ctex 论坛的东西:http://bbs.ctex.org/forum.php?mo ... page%3D2&page=1 源文档能预览其最后状态样子
thread-1375-1-1.html: 关于n次根号的次数位置
kuing 1# 2013-4-15 00:44
这里有两种n次根号的次数位置: 你们喜欢哪种?或者你们可以说说你更加想要的效果。 PS、这里只是问一下大家的喜好,没什么别的意思,各自审美习惯不同。
isea 2# 2013-4-15 08:42
明显第二种啊
isea 3# 2013-4-15 08:45
另外,怎么打出来的?
kuing 4# 2013-4-15 08:55
3# isea 我在尝试重新设计根号。。。
isea 5# 2013-4-15 09:12
本帖最后由 isea 于 2013-4-15 09:14 编辑 $\sqrt[\leftroot{2}\uproot{10}3]{\dfrac 12}$ 原来[n]选项也能加参数啊,smsmath 宏包真强 \sqrt[\leftroot{2}\uproot{10}3]{\dfrac 12} 不过,论坛将3变成斜体了。似乎。
kuing 6# 2013-4-15 09:18
5# isea 这种是手动调整,不太方便,也不能精确地统一,所以我才尝试重定义。
南飞雁 7# 2013-4-15 09:31
我喜欢第一种
nash 8# 2013-4-15 09:31
貌似我的喜好有些另类… 写的时候习惯开方数尽可能靠下些 所以看的时候第一个也更舒服些
山川浮云 9# 2013-4-15 11:14
本帖最后由 山川浮云 于 2013-4-15 11:16 编辑 看错了,是第二种,指数小的
kuing 10# 2013-4-15 11:48
9# 山川浮云 上面两种的指数大小一样大,不同的只是位置。 当然你也可以说说你觉得怎样的指数大小比较合适。
kuing 11# 2013-4-15 13:42
$\sqrt[\leftroot{2}\uproot{10}3]{\dfrac 12}$ 原来[n]选项也能加参数啊,smsmath 宏包真强 \sqrt[\leftroot{2}\uproot{10}3]{\dfrac 12} 不过,论坛将3变成斜体了。似乎。 isea 发表于 2013-4-15 09:12 其实没变斜,你双击放大看就清楚了。
kuing 12# 2013-4-15 14:18
翻了下人教版教材必修一,用的是第二种,见 http://www.pep.com.cn/gzsx/jszx_ ... 0110217_1021408.htm 中间写到:
kuing 13# 2013-4-15 14:25
甲种本教材,中间偏上,算是偏向第二种吧……
kuing 14# 2013-4-15 14:29
《代数不等式》中 《数学奥林匹克不等式研究》里也一样,估计国内的书基本是第二种…… 准确来说,应该是用方正排版的出版社出的书都是第二种,而国内的书基本都是方正排版的,所以…… 而国外的估计是反过来的,用 latex 排版的就不用说了,就说大家都用过的 MathType,是国外的东西,用第一种: Matheiatmca7:
kuing 15# 2013-4-15 15:07
word2003 的 EQ 域: 擦,这个真是另类,好难看……不值一提…… word2007 的自带公式输入: 在中间,略为偏一点点上,但是没甲种本那么上……算是第三种吧……
kuing 16# 2013-4-15 16:01
我是不是太无聊了点……的确是……闲得dan疼……
╰☆ヾo.海x 17# 2013-4-15 17:49
绝对第二种,,写在左上角好些,如果能稍微大些就更好了。。。
kuing 18# 2013-4-15 17:50
17# ╰☆ヾo.海x 你用你电脑的软件输入的是怎么样的
isea 19# 2013-4-15 22:52
哈哈,因为我都是强调偏上 否则会看成积之感 放凹中间不误成积即可
kuing 20# 2013-4-18 02:27
还有木有银??
thread-1375-2-1.html:
hnsredfox_007 21# 2013-4-18 08:06
5# isea 感觉那个根指数离根号的距离有点远哦
李斌斌755 22# 2013-5-11 14:56
也来 \[\sqrt[\leftroot{1}\uproot{8}3]{\dfrac12}\] \[\sqrt[\leftroot{2}\uproot{10}3]{2}\] \[\sqrt[4]{\dfrac12}\]
thread-1376-1-2.html: 甜薯问题
Gauss门徒 1# 2013-4-15 03:58
在$8\times 8$的表格第一行从左到右填上$1-8$,第二行填$9-16$,以此类推填完$64$个数。然后在每个数前面加上正负号使得每行每列恰有$4$个正数。求证所有数的和$=0$.
kuing 2# 2013-4-15 15:32
又是让我无从下手的题…… PS、错字dang……
Gauss门徒 3# 2013-4-15 21:11
2# kuing Wa ta shi ha dai jo bu
yes94 4# 2013-4-15 21:37
我还以为是很甜的红薯呢? 估计用整体思想或者什么不变性吧? 首先,和不可能是奇数,因为正负号不改变奇偶性。故和只可能是偶数。
转化与化归 5# 2013-4-15 21:54
为什么叫“甜薯问题”?
yes94 6# 2013-4-15 22:00
5# 转化与化归 真的是不变性 思路是先找到一种和为零的情况,再证明任意交换两行(或两列)的符号的某两个符号(其余元素符号不交换)……,最后发现涉及到的四个元素之和不变,大概是这样的吧?
kuing 7# 2013-4-15 22:00
牛比……
Gauss门徒 8# 2013-4-16 01:29
填数问题 gou me na sai
李斌斌755 9# 2013-4-16 01:44
厉害。 另楼主怎么老说英语,看不明白
转化与化归 10# 2013-4-16 06:09
8# Gauss门徒 原来如此!
kuing 11# 2013-4-16 11:11
好像不见了一层楼……
hongxian 12# 2013-4-16 12:06
12# kuing 原来这也看得出来!厉害!
isea 13# 2013-4-16 13:05
哎呀,和我那个绝对值求和的最大值差不多啊 这标题太另类了
thread-1377-1-1.html: 此区有个XeLaTeX 听说能直接调用系统字体——
isea 1# 2013-4-15 18:20
本帖最后由 isea 于 2013-4-22 01:24 编辑 但是偶基本没成功过,偶尔成功也是用TeXWorks搞定的 到现在,我终于明白了\usepackage{fontspec}宏包在win7下调用系统字体的! 如下最简单的 %源文件内容从下面开始 %!TEX TS-program = xelatex %~ \XeTeXinputencoding "GB2312" % !Mode:: "TeX:UTF-8" \\documentclass[a4paper]{article}                                             %多了一个\故意的,用时去掉,先 \usepackage[cm-default]{fontspec}                   %no-math %\usepackage{xunicode}                               %不知有什么用,被我标注了             %\usepackage{xltxtra}                                  %不知有什么用,被我标注了       %\usepackage{xeCJK}                                  %不知有什么用,被我标注了   \begin{document}         \fontspec{华文彩云}{这是用 XeLaTex 编译效果  华文彩云}         \fontspec{微软雅黑}{这是用 XeLaTex 编译效果 微软雅黑}          我的中国心 \\end{document}                                                                                  %多了一个\故意的,用时去掉,先 %源文件内容到上面结束 效果图—— 用 TeXWorks 可以看到效果的,但CTeX套安装下的WinEdt就,一般都不会编译成功的,不是乱码,就是其他初学者(如偶),不明白的问题。 到现在,我终于明白了! 最最主要的原因是 网上的教程 基本都是linux下的,所以直接搬到windows下会出错 fontspec 用法太简单了,但网上的介绍用法,却写得邪乎,或者不写,说直接调用;windows 下如何调用?但没有说。 经实验,原来是所以直接调用字体就是(系统盘下)C:\Windows\Fonts 文件夹下的文件名! 如下图(当然,常规两字不要,就是word里看到的字体名称) 非字体文件名!如 \fontspec{微软雅黑} 调用微软雅黑,用 \fontspec{msyh} 是不顶用的,但用英文名 \fontspec{Microsoft Yahei Font} 和\fontspec{微软雅黑}等效。 最后,就是WinEdt 默认不认 UTF-8 (UNICODE里一种)编码,真不可思议! 修改如下,如果不放心,怕出问题,或有乱码,你再改回来即可。 [attach]1282[/attach] 更多且更有效的打开方式:http://kkkkuingggg.5d6d.net/thread-1398-1-1.html
kuing 2# 2013-4-15 18:33
中文的话,用 xeCJK 吧,里面设置了许多中文相关的东西,也提供了一些选项和命令给你用(自己看说明文档,有中文的)。 用 xeCJK 就可以去掉 \usepackage{fontspec},因为 xeCJK 里面会调用它。
isea 3# 2013-4-15 18:42
呵呵,我主要想表达:此时弄明白了 fontspec 在windows 如何直接调用系统字体了 打印试卷我只用三种中文字体,宋体,黑体,楷书 看了楼上,原来xeCJK,这么牛,以后慢慢学吧,闪,先
kuing 4# 2013-4-15 18:53
数学空间早期版本用 CJK,后期也改用了 xeCJK。 当初用 CJK 的时候为了在封面弄那个华文新魏字体都搞得很麻烦,用 xeCJK 就简单多了。
isea 5# 2013-4-16 12:31
本帖最后由 isea 于 2013-4-16 13:03 编辑 嗯,xeCJK 宏包调用字体也方便,不过,要切换字体,必须定义对应字体的命令才行?有直接简单临时切换字体的方式么? 如 \documentclass[a4paper]{article} \usepackage{amsmath,amssymb,amsthm} \usepackage{xeCJK} \setCJKfamilyfont{sls}{STLiti}      \newcommand{\ls}{\CJKfamily{sls}}         %设置华文隶书调用,如{\ls 我的中国心},只有这种临时切换字体方法吗? \setCJKfamilyfont{ads}{Adobe Song Std L}      \newcommand{\ads}{\CJKfamily{ads}} \setCJKmainfont{华文彩云} \begin{document} 我的中国心 {\ls 我的中国心} 同第一个我的中国心 {\ads 我的中国心} \end{document} 对那个xeCJK说明文档看得很迷糊 ctex 下,直接用 \fontspec 调用系统字体,这样看来,ctex 默认处理中文的确很方便新人上手
isea 6# 2013-4-16 13:02
擦 fontspec宏包的说明文档 100多页
叶剑飞Victor 7# 2013-4-19 14:02
本帖最后由 叶剑飞Victor 于 2013-4-20 11:27 编辑 xeCJK能够使中英文使用不同字体。 例如: 中文用“宋体”(SimSun),英文用“Times New Roman”字体 \setmainfont{Times New Roman} \setCJKmainfont{SimSun} 复制代码
isea 8# 2013-4-19 16:49
xeCJK能够使中英文使用不同字体。 例如: 中文用“宋体”,英文用“Times New Roman”字体\setmainfont{Times New Roman} \setCJKmainfont{SimSun} 叶剑飞Victor 发表于 2013-4-19 14:02 对应默认字体大小如何设置呢?
hejoseph 9# 2013-4-19 17:10
对应默认字体大小如何设置呢? isea 发表于 2013-4-19 16:49 类似这样 \renewcommand{\normalsize}{\fontsize{16pt}{\baselineskip}\selectfont}
isea 10# 2013-4-19 22:36
类似这样 \renewcommand{\normalsize}{\fontsize{16pt}{\baselineskip}\selectfont} hejoseph 发表于 2013-4-19 17:10 多谢何版,等稍后几天,有时间了, 测试成熟及心得一块反馈 再次感谢
thread-1378-1-2.html: [组合] 一道组合题
转化与化归 1# 2013-4-15 22:12
本题的证明方法很多,有没有直观模型来求解?
yes94 2# 2013-4-15 22:16
1# 转化与化归 搞一种解法来看看? 建立递推?
转化与化归 3# 2013-4-15 22:25
2# yes94 递推可以的
零定义 4# 2013-4-30 17:23
好高深...弱弱的说个,原题目本身不就是一个模型么...生成函数lu过...
地狱的死灵 5# 2013-4-30 18:46
三元素子集中最小元素为$k$的子集个数为$C_{n - k}^2$ 且$1 \le k \le n - 2$ 所以\[\begin{array}{l} P_n  = \sum\limits_{k = 1}^{n - 2} {kC_{n - k}^2 }  \\   = \sum\limits_{k = 1}^{n - 2} {C_{n - k}^2 }  + \sum\limits_{k = 2}^{n - 2} {C_{n - k}^2 }  + \sum\limits_{k = 3}^{n - 2} {C_{n - k}^2 }  +  \ldots  + C_2^2  \\   = C_n^3  + C_{n - 1}^3  + C_{n - 2}^3  +  \ldots  + C_2^2  \\   = C_{n + 1}^4  \\ \end{array}\]
零定义 6# 2013-4-30 19:24
thread-1379-1-1.html: 绝对值求和的 最大值 及个数
isea 1# 2013-4-15 23:20
本帖最后由 isea 于 2013-4-15 23:26 编辑 题:设$\tau=(x_1,x_2,\cdots,x_{10})$是数$1,2,3,4,5,6,7,8,9,10$的任意一个全排列,定义$S(\tau)={\displaystyle\sum_{k=1}^{10}\lvert 2x_k-3x_{k+1}\rvert}$,其中\(x_{11}=x_1\)。 (Ⅰ)若$\tau=(10,9,8,7,6,5,4,3,2,1)$,求\(S(\tau)\)的值; (Ⅱ)求$S(\tau)$的最大值; (Ⅲ)求使$S(\tau)$达到最大值的所有排列 的个数。 ====== 个人觉得,有点意思,第二问想不明白,特发来向大家学习。 感谢,先,依然附上图片版。
李斌斌755 2# 2013-4-16 01:29
最大值=131 个数720
isea 3# 2013-4-16 11:10
2# 李斌斌755 此是北京朝阳今年理科一模最后一题,标答结果最大值131(楼上完全正确,请教啊!),个数28800(楼上有遗漏); 为不影响大家思路,稍后几天帖标答
李斌斌755 4# 2013-4-16 13:42
本帖最后由 李斌斌755 于 2013-4-16 23:38 编辑 以前见过战巡解过这类题,把绝对值符号去掉,其和$=-55$,即$\sum_{k=1}^{10}(2x_k-3x_{k+1})=-55$ 设\[2x_k-3x_{k+1}=b_k\]                                                                    $(k=1,2,\cdots,10)$ 假设${b_k}$非正数项有\[b_i\] 则${b_k}$正数项只有\[b_j\]其中$\{i\}\cap\{j\}=\varnothing,\{i\}\cup\{j\}=\{1,2,3,4,5,6,7,8,9,10\}$ 得  $\sum_{k=i}b_k+\sum_{k=j}b_k=-55$                              而$(\sum_{k=j}b_k)_{\max}\\=2\times10-3\times1+2\times9-3\times2+2\times8-3\times3+2\times7-3\times4\\=38$          (1) 由(1)有        $(\sum_{k=i}b_k)_{\min}\\=-55-38\\=-93$                      故$(\sum_{k=i}b_k)-(\sum_{k=j}b_k)=-131$
李斌斌755 5# 2013-4-16 14:10
第三问不就是数字(10,1);(9,2);(8,3);(7,4);(5);(6)的排列吗?
李斌斌755 6# 2013-4-16 14:18
5# 李斌斌755 漏了数对可以分列两头。
李斌斌755 7# 2013-4-16 14:19
6# isea 刚学
isea 8# 2013-4-16 14:23
本帖最后由 isea 于 2013-4-16 14:50 编辑 8# 李斌斌755 大家都一样,慢慢来,就是要点耐心,刚输入的公式不要特别长,如果长了,错了的时候刚开始分辨不出哪儿出错了。 kuing 弄的这个,1到2楼已然非常完整了,入门 http://kkkkuingggg.5d6d.net/thread-9-1-1.html === 学会LaTex代码还有一个好处,就是MathType 6.0后支持TeX代码输入,不过,需要手动开启,如图——
李斌斌755 9# 2013-4-16 15:09
本帖最后由 李斌斌755 于 2013-4-16 15:11 编辑 9# isea 谢谢,kuing一直帮忙,进步太慢
李斌斌755 10# 2013-4-16 16:16
本帖最后由 李斌斌755 于 2013-4-16 19:49 编辑 看了标答才知道,数对只要7、8、9、10与1、2、3、4搭配(大数在前)都行, 如 $2\times9-3\times1+2\times10-3\times4+2\times8-3\times3+2\times7-3\times2\\=2\times(7+8+9+10)-3\times(1+2+3+4)\\=38$ 这样一来,这样的数对有$C_4^1C_3^1C_2^1=24$组 一组有$P_6^6+4P_5^5\\=720+480\\=1200$ 共有$1200\times24=28800$
李斌斌755 11# 2013-4-16 16:38
数字相乘怎样操作?
isea 12# 2013-4-16 17:26
本帖最后由 isea 于 2013-4-16 17:42 编辑 数字相乘怎样操作? 李斌斌755 发表于 2013-4-16 16:38 另外 x 乘号 \times ,效果 $2\times 3$。\div 除号,$2\div 3$ ==== 这公式越长,可以用\\人为断行,如 2\times9-3\times1\\+2\times10-3\times4\\+2\times8-3\times3\\+2\times7-3\times2\\=2(7+8+9+10)-3(1+2+3+4)=38 效果 $2\times9-3\times1\\+2\times10-3\times4\\+2\times8-3\times3\\+2\times7-3\times2\\=2(7+8+9+10)-3(1+2+3+4)=38$
李斌斌755 13# 2013-4-16 17:30
13# isea
kuing 14# 2013-4-16 17:31
12# 李斌斌755 置顶写得不够清楚么……
李斌斌755 15# 2013-4-16 17:44
15# kuing 我没看到正负号的情况
kuing 16# 2013-4-16 17:50
17# 李斌斌755 浏览器有种功能叫查找……
李斌斌755 17# 2013-4-16 17:58
本帖最后由 李斌斌755 于 2013-4-16 21:27 编辑 16# isea $b_k$的值可能是正数,、负数或0,加了绝对值后为正数或0,但去掉绝对值后它们的算术和为-55。 如a+5=-3有a=-8有$|a|+|5|=13$ 即$a\ge0,b\ge0$ a-b=定值 求a+b的最大值 由a+b=定值$\implies$  $a=a_{max}$  时  $b=b_{max}$   $\implies$$(a+b)_{max}$=$a_{max}+b_{max}$
yes94 18# 2013-4-16 21:48
10# 李斌斌755 李斌很擅长图形,这次还很擅长抽象的代数!
isea 19# 2013-4-16 21:53
20# yes94 我到是想将$z=2x-3y$转化成为平面内10个整数点$(x,y)(0<x,y\le10,x,y\in \mathbf{N})$到向量$\vv {OA}=(2,-3)$的距离之和,倒是无果。
李斌斌755 20# 2013-4-16 22:27
本帖最后由 李斌斌755 于 2013-4-16 22:30 编辑 23# isea 题设没有大于零啊 笔误 是 \[2x_k-3x_{k+1}=b_k\] 已修改
thread-1379-1-5.html: 绝对值求和的 最大值 及个数
isea 1# 2013-4-15 23:20
本帖最后由 isea 于 2013-4-15 23:26 编辑 题:设$\tau=(x_1,x_2,\cdots,x_{10})$是数$1,2,3,4,5,6,7,8,9,10$的任意一个全排列,定义$S(\tau)={\displaystyle\sum_{k=1}^{10}\lvert 2x_k-3x_{k+1}\rvert}$,其中\(x_{11}=x_1\)。 (Ⅰ)若$\tau=(10,9,8,7,6,5,4,3,2,1)$,求\(S(\tau)\)的值; (Ⅱ)求$S(\tau)$的最大值; (Ⅲ)求使$S(\tau)$达到最大值的所有排列 的个数。 ====== 个人觉得,有点意思,第二问想不明白,特发来向大家学习。 感谢,先,依然附上图片版。
李斌斌755 2# 2013-4-16 01:29
最大值=131 个数720
isea 3# 2013-4-16 11:10
2# 李斌斌755 此是北京朝阳今年理科一模最后一题,标答结果最大值131(楼上完全正确,请教啊!),个数28800(楼上有遗漏); 为不影响大家思路,稍后几天帖标答
李斌斌755 4# 2013-4-16 13:42
本帖最后由 李斌斌755 于 2013-4-16 23:38 编辑 以前见过战巡解过这类题,把绝对值符号去掉,其和$=-55$,即$\sum_{k=1}^{10}(2x_k-3x_{k+1})=-55$ 设\[2x_k-3x_{k+1}=b_k\]                                                                    $(k=1,2,\cdots,10)$ 假设${b_k}$非正数项有\[b_i\] 则${b_k}$正数项只有\[b_j\]其中$\{i\}\cap\{j\}=\varnothing,\{i\}\cup\{j\}=\{1,2,3,4,5,6,7,8,9,10\}$ 得  $\sum_{k=i}b_k+\sum_{k=j}b_k=-55$                              而$(\sum_{k=j}b_k)_{\max}\\=2\times10-3\times1+2\times9-3\times2+2\times8-3\times3+2\times7-3\times4\\=38$          (1) 由(1)有        $(\sum_{k=i}b_k)_{\min}\\=-55-38\\=-93$                      故$(\sum_{k=i}b_k)-(\sum_{k=j}b_k)=-131$
李斌斌755 5# 2013-4-16 14:10
第三问不就是数字(10,1);(9,2);(8,3);(7,4);(5);(6)的排列吗?
李斌斌755 6# 2013-4-16 14:18
5# 李斌斌755 漏了数对可以分列两头。
李斌斌755 7# 2013-4-16 14:19
6# isea 刚学
isea 8# 2013-4-16 14:23
本帖最后由 isea 于 2013-4-16 14:50 编辑 8# 李斌斌755 大家都一样,慢慢来,就是要点耐心,刚输入的公式不要特别长,如果长了,错了的时候刚开始分辨不出哪儿出错了。 kuing 弄的这个,1到2楼已然非常完整了,入门 http://kkkkuingggg.5d6d.net/thread-9-1-1.html === 学会LaTex代码还有一个好处,就是MathType 6.0后支持TeX代码输入,不过,需要手动开启,如图——
李斌斌755 9# 2013-4-16 15:09
本帖最后由 李斌斌755 于 2013-4-16 15:11 编辑 9# isea 谢谢,kuing一直帮忙,进步太慢
李斌斌755 10# 2013-4-16 16:16
本帖最后由 李斌斌755 于 2013-4-16 19:49 编辑 看了标答才知道,数对只要7、8、9、10与1、2、3、4搭配(大数在前)都行, 如 $2\times9-3\times1+2\times10-3\times4+2\times8-3\times3+2\times7-3\times2\\=2\times(7+8+9+10)-3\times(1+2+3+4)\\=38$ 这样一来,这样的数对有$C_4^1C_3^1C_2^1=24$组 一组有$P_6^6+4P_5^5\\=720+480\\=1200$ 共有$1200\times24=28800$
李斌斌755 11# 2013-4-16 16:38
数字相乘怎样操作?
isea 12# 2013-4-16 17:26
本帖最后由 isea 于 2013-4-16 17:42 编辑 数字相乘怎样操作? 李斌斌755 发表于 2013-4-16 16:38 另外 x 乘号 \times ,效果 $2\times 3$。\div 除号,$2\div 3$ ==== 这公式越长,可以用\\人为断行,如 2\times9-3\times1\\+2\times10-3\times4\\+2\times8-3\times3\\+2\times7-3\times2\\=2(7+8+9+10)-3(1+2+3+4)=38 效果 $2\times9-3\times1\\+2\times10-3\times4\\+2\times8-3\times3\\+2\times7-3\times2\\=2(7+8+9+10)-3(1+2+3+4)=38$
李斌斌755 13# 2013-4-16 17:30
13# isea
kuing 14# 2013-4-16 17:31
12# 李斌斌755 置顶写得不够清楚么……
李斌斌755 15# 2013-4-16 17:44
15# kuing 我没看到正负号的情况
kuing 16# 2013-4-16 17:50
17# 李斌斌755 浏览器有种功能叫查找……
李斌斌755 17# 2013-4-16 17:58
本帖最后由 李斌斌755 于 2013-4-16 21:27 编辑 16# isea $b_k$的值可能是正数,、负数或0,加了绝对值后为正数或0,但去掉绝对值后它们的算术和为-55。 如a+5=-3有a=-8有$|a|+|5|=13$ 即$a\ge0,b\ge0$ a-b=定值 求a+b的最大值 由a+b=定值$\implies$  $a=a_{max}$  时  $b=b_{max}$   $\implies$$(a+b)_{max}$=$a_{max}+b_{max}$
yes94 18# 2013-4-16 21:48
10# 李斌斌755 李斌很擅长图形,这次还很擅长抽象的代数!
isea 19# 2013-4-16 21:53
20# yes94 我到是想将$z=2x-3y$转化成为平面内10个整数点$(x,y)(0<x,y\le10,x,y\in \mathbf{N})$到向量$\vv {OA}=(2,-3)$的距离之和,倒是无果。
李斌斌755 20# 2013-4-16 22:27
本帖最后由 李斌斌755 于 2013-4-16 22:30 编辑 23# isea 题设没有大于零啊 笔误 是 \[2x_k-3x_{k+1}=b_k\] 已修改
thread-1379-2-1.html:
李斌斌755 21# 2013-4-16 22:32
本帖最后由 李斌斌755 于 2013-4-16 22:37 编辑 25# isea 抱歉 学习编辑,一直忙着修改帖子,没细看16#
kuing 22# 2013-4-16 22:33
24# 李斌斌755 公式里不要用全角 请用纯英文状态下打代码
李斌斌755 23# 2013-4-16 22:35
本帖最后由 李斌斌755 于 2013-4-16 22:39 编辑 27# kuing 半角的-号太小 回复29#明白
isea 24# 2013-4-16 22:35
本帖最后由 isea 于 2013-4-16 23:10 编辑 24# 李斌斌755 那个减号写在$\$$之间的,要全部要英文状态下,输入,会自动转换成数学的减号,否则就不像减号了。 == 改后,我理解理解 == 25# isea 抱歉 学习编辑,一直忙着修改帖子,没细看16# 李斌斌755 发表于 2013-4-16 22:32 没关系,太理解了,我刚开始都是半夜修改代码的 理解了,这个过程理应没问题,但这(包括标答)里并没有严格证明为什么38是正项和最大。第二问理解了,一会再看第三问。 多谢! thx!
isea 25# 2013-4-16 23:07
以前见过战巡解过这类题,把绝对值符号去掉,其和=-55,即$\sum_{k=1}^{10}(2x_k-3x_{k+1})=-55$ 设\[2x_k-3x_{k+1}=b_k\]                                                                    (k=1,2,$\cdots$, ... 李斌斌755 发表于 2013-4-16 13:42 就本楼而言,对一个全新新手,能打成这样,已经不错了。 至少有些细节还需要修改,如下图所示
李斌斌755 26# 2013-4-16 23:15
24# isea 显然正项最多有四项,余下其它数字无法构成正项,必然最大值只能是 $2\times(10+9+8+7)-3\times(1+2+3+4)=38$
kuing 27# 2013-4-16 23:16
竟然变成了扯公式代码贴…… 咦,好像又少了几层楼,自删的还是被XX?
isea 28# 2013-4-16 23:27
本帖最后由 isea 于 2013-4-16 23:30 编辑 竟然变成了扯公式代码贴…… 咦,好像又少了几层楼,自删的还是被XX? kuing 发表于 2013-4-16 23:16 我删除自己的,无所谓啦,本来就没多少人来人往,兼带扯扯公式,好事啊 == 还有三个地方
李斌斌755 29# 2013-4-16 23:33
28# isea 再次谢谢isea,已修改,看看对吗!
isea 30# 2013-4-16 23:36
28# isea 再次谢谢isea,已修改,看看对吗! 李斌斌755 发表于 2013-4-16 23:33 min是下标_{\min}少 _ 现在,就这个了,呵呵 恭喜,入门
yes94 31# 2013-4-17 13:15
30# isea 在IC的提示下,现在用Aurora用的正欢,省去了latex的编排的麻烦,又占有了latex公式的漂亮! Aurora和word完美结合!
hongxian 32# 2013-5-22 23:08
3# isea 百度了一个答案,传上来
isea 33# 2013-5-27 23:36
32# hongxian 谢谢发解答过程!
thread-138-1-9.html: [不等式] 额,午觉没睡着,爬起来贴个题
pxchg1200 1# 2011-10-24 14:36
Let$x,y,z \geq 0 $ prove that: \[ (x^{2}+y^{2}+z^{2})[\frac{x^{2}}{(x^{2}+yz)^{2}}+\frac{y^{2}}{(y^{2}+xz)^{2}}+\frac{z^{2}}{(z^{2}+xy)^{2}}]\geq \frac{9}{4} \] (和Iran 96 一样的$ \frac{9}{4}$ ,不过SOS计算量大了不是一点点。。。 )另外,为何我的题都没人做啊?!(too easy? or too hard ?)
kuing 2# 2011-10-24 14:44
1# pxchg1200 等我状态好起来,定当奉陪到底……
pxchg1200 3# 2011-10-24 14:50
2# kuing 你怎么也会状态不好啊? 我记得当年kuing在Mathlink上叱咤风云,一天秒个30多题,那个神呐!!!
kuing 4# 2011-10-24 14:50
3# pxchg1200 夸张…………
pxchg1200 5# 2011-10-24 14:53
4# kuing 真的,就一年前我刚上去的时候,第一天看你是Posts:470 的样子,第二天就 500+ 了。 不久就 1000+ 了,篇篇都是excellent。
kuing 6# 2011-10-24 14:54
5# pxchg1200 水得比较多;那个可能是熟人评的分……
kuing 7# 2011-10-24 21:40
作置换 $x\to\dfrac1x, y\to\dfrac1y, z\to\dfrac1z$ 变成 \[(x^2y^2+y^2z^2+z^2x^2)\left( \frac1{(x^2+yz)^2}+\frac1{(y^2+zx)^2}+\frac1{(z^2+xy)^2} \right)\geqslant \frac94.\] 或写成 \[(xy+yz+zx)\left( \frac1{(x+\sqrt{yz})^2}+\frac1{(y+\sqrt{zx})^2}+\frac1{(z+\sqrt{xy})^2} \right)\geqslant \frac94.\] 更像Iran96……不过还没证到
kuing 8# 2011-10-24 22:00
说不定上面这两个已经被研究过了…… 我还是停一下,等出处党看过先
pxchg1200 9# 2011-10-24 22:26
8# kuing 呵呵,我不是出处党。。。 不过那个好像Iran 96 部分调整过的结果。。
pxchg1200 10# 2011-10-24 22:29
本帖最后由 pxchg1200 于 2011-10-24 22:35 编辑 9# pxchg1200 可惜这个放过头了。 \[ (xy+yz+xz)(\frac{1}{(x+\frac{y+z}{2})^{2}}+\frac{1}{(y+\frac{x+z}{2})^{2}}+\frac{1}{(z+\frac{x+y}{2})^{2}})\geq \frac{9}{4} \]
kuing 11# 2011-10-24 22:32
10# pxchg1200 bottema说这个反向了……
pxchg1200 12# 2011-10-24 22:36
11# kuing 呵呵,我也验证了下。。
thread-1380-1-2.html: 一个解三角形的老题
转化与化归 1# 2013-4-16 20:25
看看有没有好方法
转化与化归 2# 2013-4-16 20:30
发一个解答以引起讨论!
李斌斌755 3# 2013-4-16 20:45
这方法不是很好了吗
转化与化归 4# 2013-4-16 21:08
3# 李斌斌755 在讨论一些其他的思路,比如不等式等等的方法
thread-1381-1-2.html: [几何] 圆锥曲线求新解
guanmo 1# 2013-4-16 21:35
见附件,是2005年全国文科第22题 除了坐标整体代换 有其它方法吗
yes94 2# 2013-4-16 21:40
和共轭直径有关吧
guanmo 3# 2013-4-16 21:46
也许,但具体怎么解释呢?
kuing 4# 2013-4-16 21:50
化为圆不就行了么,圆内两垂直直径对应的就是椭圆的共轭直径了,来个勾股定理……
hongxian 5# 2013-4-17 10:44
4# kuing 变圆之后$\vv{O'A'}$和$\vv{O'B'}$刚好垂直,好!
hongxian 6# 2013-4-17 11:15
4# kuing 不过怎样一眼就发现$OA$、$OB$是共轭直径呢?
kuing 7# 2013-4-17 12:14
6# hongxian 其实我没去判断,只是看了yes94说共轭直径,我相信他没判断错,所以就……
isea 8# 2013-4-17 12:47
7# kuing yes94 回帖就是这样子,要么N简洁,要么扩展一堆一堆的
yes94 9# 2013-4-17 13:11
8# isea 以前很喜欢写一大堆,现在做题不在状态了,只是凭记忆和感觉了。
thread-1382-1-1.html: 啧啧
isea 1# 2013-4-16 21:59
这会有12个人在线,这会,同时,竟然 平时没注意,好像是个位数的
kuing 2# 2013-4-16 22:44
1# isea 晚上人多点还是比较正常…… 不过平常的确是个位数的……
李斌斌755 3# 2013-4-16 22:56
可能是这的题难点吧?
kuing 4# 2013-4-16 23:58
3# 李斌斌755 人气要弄起来可不是一件简单的事……像这里这样的小论坛能弄到现在这样我觉得已经不错了……
isea 5# 2013-4-17 00:03
9494
kuing 6# 2013-4-17 00:40
不过,尽管各种论坛都在想方设法搞旺人气,然而有的论坛原本拥有着令人羡慕的人气却不懂得珍惜……
yes94 7# 2013-4-17 23:35
6# kuing 人家是国 企,贷款没压力,还有拨款,现在是民 企没法啊?
kuing 8# 2013-4-17 23:36
7# yes94 表示看不懂……
thread-1383-1-2.html: [数论] 无字天书,虫食算
yes94 1# 2013-4-16 23:16

yes94 2# 2013-4-16 23:19
1# yes94 还有一道: 注意那里有个小数点,*号表示的数字不一定相同
isea 3# 2013-4-16 23:32
小学题啊,第一题是8位循环…
地狱的死灵 4# 2013-4-17 12:29
本帖最后由 地狱的死灵 于 2013-4-17 12:33 编辑 1# yes94 7÷73, 按正常排竖式即对应空格位数 2楼的应该是100÷32=3.125
isea 5# 2013-4-17 12:41
1# yes94 7÷73, 按正常排竖式即对应空格位数 地狱的死灵 发表于 2013-4-17 12:29 厉害啊,我试了好多,就没试出这个来,分母大啊$\huge \dfrac 7{73}=0.\dot{0}958904\dot{1}$
yes94 6# 2013-4-17 13:08
5# isea 据说,要用点数论关于循环节的知识
isea 7# 2013-4-17 13:12
6# yes94 哈哈,用计算器直接算结果试的,没分析结构……
yes94 8# 2013-4-17 13:13
7# isea 充分利用现代工具,就是方便啊
地狱的死灵 9# 2013-4-17 13:21
本帖最后由 地狱的死灵 于 2013-4-17 13:27 编辑 6# yes94 循环节是8位, 除数就必须是99999999的约数(并且是素数), 于是将99999999分解质因数, 得到是73的倍数, 再结合竖式的性质, 知道商的第6位小数应该是0或1, 尝试两次就够了
yes94 10# 2013-4-17 13:31
9# 地狱的死灵
isea 11# 2013-4-17 13:59
6# yes94 循环节是8位, 除数就必须是99999999的约数(并且是素数), 于是将99999999分解质因数, 得到是73的倍数, 再结合竖式的性质, 知道商的第6位小数应该是0或1, 尝试两次就够了 地狱的死灵 发表于 2013-4-17 13:21 同赞!
kuing 12# 2013-4-17 13:59
数论白白lu过膜拜……
yes94 13# 2013-4-17 19:37
发一个循环节理论
kuing 14# 2013-4-17 21:00
13# yes94 突然觉得之前好像在哪里看到过这个图……
yes94 15# 2013-4-17 21:38
14# kuing 严士健《初等数论》
kuing 16# 2013-4-29 18:00
1# yes94 “尽管它连一个数目都没有!”  其实还是有一个的嘛,那个 0.
yes94 17# 2013-4-29 18:23
16# kuing 第二题呢?第二题就真的没有数字了哦!
kuing 18# 2013-4-29 23:44
17# yes94 题目不会…… 倒是让我无无聊聊地尝试在这里实现那个竖式的输入,竟然还勉强整了出来,虽然有点笨重而且效果不尽相同,但已经尽力了暂时(在真 LaTeX 里可以制作得更好,这里能用的命令和环境太少了)。 \begin{array}{cccccccccc} & 0.& \dot\square & \square & \square & \square & \square & \square & \square & \dot\square \\ \hline \smash{\llap{\square\quad\square\ \raise1.5pt\big)}} & \square \\ & \square & \square & \square \\ \hline && \square & \square & \square \\ && \square & \square & \square \\ \hline &&& \square & \square & \square \\ &&& \square & \square & \square \\ \hline &&&& \square & \square & \square \\ &&&& \square & \square & \square \\ \hline &&&&&& \square & \square & \square \\ &&&&&& \square & \square & \square \\ \hline &&&&&&&& \square & \square \\ &&&&&&&& \square & \square \\ \hline &&&&&&&&&\square \end{array}
yes94 19# 2013-4-30 13:58
18# kuing 牛笔啊!
kuing 20# 2013-4-30 14:25
列距居然也没法控制,只能用 \llap 来去掉正方形的宽度以达到缩小列距的效果。 \[\newcommand\resq{\llap{\square}} \begin{array}{cccccccccc} & \llap{0.}& \llap{\dot\square} & \resq & \resq & \resq & \resq & \resq & \resq & \llap{\dot\square} \\ \hline \smash{\llap{\square\ \square\ \raise1.5pt\big)}} & \resq \\ & \resq & \resq & \resq \\ \hline && \resq & \resq & \resq \\ && \resq & \resq & \resq \\ \hline &&& \resq & \resq & \resq \\ &&& \resq & \resq & \resq \\ \hline &&&& \resq & \resq & \resq \\ &&&& \resq & \resq & \resq \\ \hline &&&&&& \resq & \resq & \resq \\ &&&&&& \resq & \resq & \resq \\ \hline &&&&&&&& \resq & \resq \\ &&&&&&&& \resq & \resq \\ \hline &&&&&&&&& \resq \end{array} \] 紧凑一些。
thread-1383-2-2.html:
yes94 21# 2013-4-30 14:38
20# kuing 嗯,这个要紧凑一些
thread-1384-1-2.html: [不等式] 求证:$\ln x-e^{-x}+\frac{2}{ex}>0$
realnumber 1# 2013-4-17 15:20
求证:$\ln x-e^{-x}+\frac{2}{ex}>0$ 暂时没好办法,用了下$-e^{-x}>-\frac{1}{1+x}$还是成立,不过还是不好计算.
kuing 2# 2013-4-17 15:23
是不是跟这个一样 http://kkkkuingggg.5d6d.net/thread-20-1-24.html
isea 3# 2013-4-17 15:37
2# kuing 完全相同
thread-1385-1-2.html: [不等式] $\sum$1/(1+x^2)>=3/2
reny 1# 2013-4-17 21:50
本帖最后由 reny 于 2013-4-18 16:08 编辑 已知$x,y,z$是非负实数,在条件$(1)x+y+z\leqslant3;(2)xy+yz+zx=3$下,分别证明:$$\sum_{cyc}\frac{1}{1+x^2}\geqslant\frac 3 2$$.
yes94 2# 2013-4-17 22:00
1# reny 当$x+y+z=3$时
reny 3# 2013-4-17 22:57
2# yes94 en,取最小值应该就是在$x+y+z=3$时取得,之后还有凸凹不一致
hongxian 4# 2013-4-17 23:05
切线法吧,$x=y=z=1$时取等号。
kuing 5# 2013-4-17 23:07
显然只要考虑 =3 的情形,切线法不是刚刚好吗
hongxian 6# 2013-4-17 23:08
先证明$x>0$时$\dfrac{1}{1+x^2} \ge-\dfrac{1}{2}x+1$
yes94 7# 2013-4-17 23:13
上了kuing的论坛,不等式一直被虐, 这次终于可以轻松一下啦
reny 8# 2013-4-17 23:32
本帖最后由 reny 于 2013-4-17 23:34 编辑 7# yes94 哈哈,条件能否放宽松点
kuing 9# 2013-4-17 23:41
8# reny 新加的条件(2)显然不成,可以让两个元很大另一个很小,左边可以趋向1
reny 10# 2013-4-18 08:41
9# kuing 但是非负实数$x,y,z$中有两个很大的话不满足$xy+yz+zx=3$吧?
kuing 11# 2013-4-18 10:28
10# reny 哦,我又发傻了。。。汗
kuing 12# 2013-4-18 11:45
那样应该是成立的……还两个取等条件……晚点再玩……
yes94 13# 2013-4-18 12:43
8# reny 改条件了嗦? 不让我轻松一下啊? 你的意思是条件(1)(2)是同时成立的? 还是分别成立?
kuing 14# 2013-4-18 12:47
13# yes94 他的意思当然是分别成立,但是在写法是当然不应该那样写。
kuing 15# 2013-4-18 17:34
设 $x$, $y$, $z\geqslant0$, $xy+yz+zx=3$,求证 \[\sum\frac1{1+x^2}\geqslant\frac32.\] 由对称性,不妨设 $x\leqslant y\leqslant z$,记 $p=y+z$, $q=yz$,则由条件易得 $1\leqslant q\leqslant 3$。 条件可以化为 $xp+q=3$,待证不等式容易化为 \[\frac1{1+x^2}+\frac{1-q^2}{p^2+(1-q)^2}\geqslant\frac12,\] 现在,固定 $q$,让 $p$ 减少,则 $x$ 变大,由 $q\geqslant 1$ 知上式左边关于 $p^2$ 递增,关于 $x$ 递减,故此,让 $p$ 减少时,上式左边也减少,从而只需证明 $x\leqslant y=z$ 时原不等式成立即可。 此时 $p^2=4q$, $x^2=(3-q)^2/(4q)$,代入后作差分解等价于 \[\frac{3(3-q)(1-q)^2}{2(q+1)(q^2-2q+9)}\geqslant 0,\] 显然成立,当 $q=1$ 或 $q=3$ 时取等,故此原不等式有两个取等条件,即 $x=y=z=1$ 和 $x=0$, $y=z=\sqrt3$ 及其轮换。
reny 16# 2013-4-18 18:08
15# kuing 这种代换、固定变量、看单调性(减元),学习中...
pxchg1200 17# 2013-4-19 16:09
15# kuing 额,还是来柯西一下吧。 注意到不等式等价于 \[ \frac{x^2}{1+x^2}+\frac{y^2}{1+y^2}+\frac{z^2}{1+z^2}\leq \frac{3}{2}\] 就是 \[ \sum{\frac{3x^2}{3x^2+xy+yz+xz}}\leq \frac{3}{2}\] 由Cauchy-Schwarz \[ \frac{4x^2}{3x^2+xy+yz+xz}\leq \left(\frac{x^2}{2x^2+yz}+\frac{x^2}{x(x+y+z)} \right)\] 所以,只要证明 \[ \sum{\frac{x^2}{2x^2+yz}}\leq 1 \] 也就是 \[ \sum{\frac{yz}{2x^2+yz}}\geq 1 \] 再次运用Cauchy-Schwraz.我们有 \[ \sum{\frac{yz}{2x^2+yz}}\geq \frac{(xy+yz+xz)^2}{\sum{x^2y^2}+2xyz(x+y+z)}=1 \] 由此,不等式得证。
kuing 18# 2013-4-19 16:13
17# pxchg1200 牛比,你用CS比我灵活……
pxchg1200 19# 2013-4-19 16:14
18# kuing 运气好而已。
kuing 20# 2013-4-19 16:22
19# pxchg1200 别谦虚了,这种运气也是需要实力的,感觉越好,运气也越好……
thread-1385-2-2.html:
yes94 21# 2013-4-19 18:39
17# pxchg1200 又被虐了,早点离开是非之地
reny 22# 2013-4-19 19:26
17# pxchg1200 厉害啊,用重要不等式证明很漂亮.
pxchg1200 23# 2013-4-19 22:44
1# reny 对于$x+y+z\leq 3$的这个条件,Cauchy-reverse technique最适合不过了。注意到 \[ \frac{x^2}{1+x^2}\leq \frac{x}{2} \] 就啥都没事了。
reny 24# 2013-4-20 09:54
23# pxchg1200 的确,为了用上条件,柯西求反就秒了,好方法 $\frac{1}{1+x^2}=1-\frac{x^2}{1+x^2}\geqslant1-\frac{x^2}{2x}=1-\frac {x}{ 2}$
thread-1386-1-1.html: 电磁感应
yayaweha 1# 2013-4-17 22:35
U-t的表达式怎么求?
yayaweha 2# 2013-4-17 22:46
本帖最后由 yayaweha 于 2013-4-17 22:50 编辑 我求的是$F-\mu mg-\frac{BLU}{R}=m\cdot \frac{dv}{dt}$ 然后$(F-\mu mg-\frac{BLU}{R})\cdot t=mv$ 又$U=BLv$ 所以$(F-\mu mg-\frac{BLU}{R})\cdot t=m\cdot \frac{U}{BL}$ 不知道这样求对不对?
kuing 3# 2013-4-17 22:48
2# yayaweha \mu \cdot
yayaweha 4# 2013-4-17 22:51
3# kuing 我的表达式求的对吗?
kuing 5# 2013-4-17 22:52
题目好长,我要看十来分钟才能看完
kuing 6# 2013-4-17 23:07
你对 d 的理解还是…………
kuing 7# 2013-4-17 23:33
拉力的功率是什么?P=FV 吗?
kuing 8# 2013-4-18 00:08
7# kuing 如果是这样的话,那比较难积 \[ \frac Pv-\mu mg-\frac{B^2L^2v}R=m\frac{\rmd v}{\rmd t}, \] 分离为 \[\rmd t=\frac{mRv}{PR-\mu mgRv-B^2L^2v^2}\rmd v,\] 两边积分 \[t-t_0=\int_{t_0}^t\rmd t=mR\int_{v_0}^v\frac v{PR-\mu mgRv-B^2L^2v^2}\rmd v,\] 右边积出来那是很复杂的表面式,也无法解出 $v=f(t)$ 这样的形式来。$U$ 和 $v$ 成正比,所以一样。
thread-1387-1-2.html: 树轮问题
Gauss门徒 1# 2013-4-18 04:50
一支数列$a_{n+1}=a_n+S(a_n),a_1=1$,$S(n)$表示$n$各个位数字和($10$禁制不解释)。问题:$123456$是这个数列的项吗? P.S: kuing酱能够首先独立就做出奖励萝莉一只!
Gauss门徒 2# 2013-4-18 05:03
好像太水。。
hnsredfox_007 3# 2013-4-18 08:12
不是,$\{a_n\}$是模周期数列,模9,周期为6
Gauss门徒 4# 2013-4-18 10:27
Kuing没能第一个答出,喝哦嘿
kuing 5# 2013-4-18 10:32
4# Gauss门徒 你你你。。。趁我睡着觉才发题!
kuing 6# 2013-4-18 10:35
还错字dang呢! 又不选主题分类
hnsredfox_007 7# 2013-4-18 10:38

李斌斌755 8# 2013-4-18 11:15
模周期数列
kuing 9# 2013-4-18 11:36
8# 李斌斌755 大概就是除以9的余数是周期数列的意思
李斌斌755 10# 2013-4-18 11:47
9# kuing 谢谢
yes94 11# 2013-4-18 12:48
树伦问题,田鼠问题
hongxian 12# 2013-4-18 15:42
不是,$\{a_n\}$是模周期数列,模9,周期为6 hnsredfox_007 发表于 2013-4-18 08:12 只是猜想?能不能证明?
hnsredfox_007 13# 2013-4-18 18:57
12# hongxian 可以啊 数学归纳法 猜的
hnsredfox_007 14# 2013-4-18 19:03
本帖最后由 hnsredfox_007 于 2013-4-18 19:04 编辑 12# hongxian 首先要知道一个事实:$a_n \equiv S({a_n}) \pmod{9}$
kuing 15# 2013-4-18 19:07
因为 $S(n)\equiv n\pmod9$,故若 $a_n\equiv k\pmod9$ 其中 $k\in\{0,1,2,3,4\}$,则 $a_{n+1}\equiv2k\pmod9$; 若 $a_n\equiv k\pmod9$ 其中 $k\in\{5,6,7,8\}$,则 $a_{n+1}\equiv2k-9\pmod9$。因 $a_1=1$,故 \begin{align*} a_1&\equiv1\pmod9\\ a_2&\equiv2\pmod9\\ a_3&\equiv4\pmod9\\ a_4&\equiv8\pmod9\\ a_5&\equiv7\pmod9\\ a_6&\equiv5\pmod9\\ a_7&\equiv1\pmod9\\ &\vdots \end{align*} 循环……
hnsredfox_007 16# 2013-4-18 19:10
然后由于$a_1 \equiv 1 \pmod{9},a_2 \equiv 2 \pmod{9},a_3 \equiv 4 \pmod{9},a_4 \equiv 8 \pmod{9},a_5 \equiv 7 \pmod{9},a_6 \equiv 5 \pmod{9}$,猜想$a_{6n+k} \equiv a_k \pmod{9},k=1,2,3,4,5,6,n\in \mathbf{N}$。
hongxian 17# 2013-4-18 19:17
16# hnsredfox_007 关键在$a_n \equiv S({a_n}) \pmod{9}$,谢谢了!
kuing 18# 2013-4-18 19:19
17# hongxian 嗯,通常扯到各位数字之和都自然会想到这一点……小学学回来的……
hongxian 19# 2013-4-18 19:22
18# kuing 我可没学过,不过还是想通了!
yes94 20# 2013-4-18 19:53
类似题:
thread-1387-2-2.html:
李斌斌755 21# 2013-4-18 20:09
20# yes94 这题好想,不像楼主的需要“模”
yes94 22# 2013-4-18 20:13
21# 李斌斌755 嗯,你来做做
李斌斌755 23# 2013-4-18 21:27
22# yes94 原来以为$S_n$的值在$10$以内,是$20$以内,有点复杂,现在有事,先闪人……
yes94 24# 2013-4-18 21:33
23# 李斌斌755 没事,然后晚上回来哈
isea 25# 2013-4-19 00:04
楼主不能把标题改改呀,我觉得我没强迫症的,但这个实在太怪异了
李斌斌755 26# 2013-4-19 03:34
本帖最后由 李斌斌755 于 2013-4-19 15:05 编辑 把1至1998分成(1)1至9,(2)10至99(3)100至999,(4)1000至1998四段 (1)  45 (2)个位数上1,2,3,4,5,6,7,8,9出现的次数各为9,和为$9\times{45}=405$        十位数上1,2,3,4,5,6,7,8,9出现的次数各为10,和为$10\times{45}=450$ (3)个位数,、十位数上1,2,3,4,5,6,7,8,9出现的次数各为90,和为$2\times{90}\times{45}=8100$        百位数上1,2,3,4,5,6,7,8,9出现的次数各为100,和为$100\times{45}=4500$ (4)个位数、十位数、百位数上1,2,3,4,5,6,7,8,9出现的次数各为100,和为$3\times{100}\times{45}=13500$        千位数上出现1的出现为999次,和为999 故    $45+405+450+8100+4500+13500+999=27999$
yes94 27# 2013-4-19 12:57
26# 李斌斌755 答案差点点就对了! 你答案的万位,千位、百位都对! 方法稍微复杂了一点
Gauss门徒 28# 2013-4-19 13:45
本质就是模,我相信科学!
kuing 29# 2013-4-19 14:02
摸摸
李斌斌755 30# 2013-4-19 15:04
摸摸,摸着就算 ,摸不着学习,9494公布答案。
李斌斌755 31# 2013-4-19 15:09
27# yes94 错就错了,那有差一点就对了
yes94 32# 2013-4-19 18:41
摸摸,摸着就算 ,摸不着学习,9494公布答案。 李斌斌755 发表于 2013-4-19 15:04 27972
hnsredfox_007 33# 2013-4-19 20:58
20# yes94 $0000,0001,0002,\cdots,0999$, $1000,1001,1002,\cdots,1999$. 以下略
hnsredfox_007 34# 2013-4-19 21:15
33# hnsredfox_007 $1000\times 1+2\times1000\times \dfrac{1}{10} \times(0+1+2+3+4+5+6+7+8+9)\times 3 -28=27972$
yes94 35# 2013-4-19 22:23
34# hnsredfox_007 据说是小学奥数题,原解法比较巧妙,令人赞叹。
李斌斌755 36# 2013-4-20 03:05
$1+1998=1999$ $2+1997=1999$ $3+1996=1999$ $997+1002=1999$ $998+1001=1999$ $999+1000=1999$ $999\times28=27972$
yes94 37# 2013-4-20 12:54
36# 李斌斌755
李斌斌755 38# 2013-4-20 13:28
37# yes94 小学奥数 是利用十进制中个位数可拆分成两个个位数之和不影响结果。
yes94 39# 2013-4-20 23:03
38# 李斌斌755 实际上就是反序相加法,或者叫配对法
thread-139-1-1.html: Maple 中是否有配方软件?
pxchg1200 1# 2011-10-24 14:41
Maple中有类似Matlab的(sostools)配方软件吗? 其实我就是想知道陈计到底怎么搞的。呵呵
kuing 2# 2011-10-24 14:45
1# pxchg1200 不太清楚,sostools 我也没用过,还不会用 matlab。 如果两个软件都玩熟了,估计可以将其转换。
thread-1391-1-2.html: [不等式] 来自三角代换的不等式
reny 1# 2013-4-18 18:34
本帖最后由 reny 于 2013-4-19 20:20 编辑 已知正实数$x,y,z$满足$xy+yz+zx=1$,求证$$\frac{x}{z(x+y)}+\frac{y}{x(y+z)}+\frac{z}{y(z+x)}\geqslant\frac{3\sqrt3}{2}(*)$$ $\color{green}{代换自}$:在锐角$\triangle ABC$中,求证$\sum_{cyc}\frac{\cos A\sin B}{\cos C}\geqslant \frac{3\sqrt3}{2}$ PS、这个不等式不知道出现过没有,是由下面的类比过来的,不过用几组数据测试应该是对的. 在锐角$\triangle ABC$中,成立(已解决) $$\sum_{cyc}\frac{\sin A\cos B}{\cos C}\geqslant\frac{3\sqrt3}{2}  …和(*)类似$$ $$1<\sum_{cyc}\frac{\cos A\sin B}{\sin C}<2、1<\sum_{cyc}\frac{\sin A\cos B}{\sin C}<2$$ $$\sum_{cyc}\frac{\cos A\cos B}{\cos C}\geqslant\frac32$$ $$\sum_{cyc}\frac{\sin A\sin B}{\sin C}>\frac5 2$$ $$\sum_{cyc}\frac{\sin A\sin B}{\cos C}\geqslant\frac9 2$$ $$\sum_{cyc}\frac{\sin A\sin B}{\sin C}\leqslant\frac{\sqrt3}{2}$$
kuing 2# 2013-4-18 18:41
标题是啥意思……
reny 3# 2013-4-18 18:59
2# kuing 本来是个三角不等式的,我用余切代换过来的,觉得这样证明要简单点
reny 4# 2013-4-19 19:44
由CS,$$ \sum_{cyc}\frac{x}{z(x+y)}\geqslant\frac{(x+y+z)^2}{\sum_{cyc}xy^2+3xyz}$$ 需要证明$$\frac{(x+y+z)^2}{\sum_{cyc}xy^2+3xyz}\geqslant\frac{3\sqrt3}{2}$$ 齐次化,即证$4(x+y+z)^4(xy+yz+zx)\geqslant27\left(\sum_{cyc}xy^2+3xyz\right)^2$ 然后,怎么办 高手们,来帮忙
kuing 5# 2013-4-19 19:51
4# reny 昨晚我就这样考虑过,但是由于次数太高而且轮换,还在寻找突破ing……
reny 6# 2013-4-19 19:57
5# kuing 就是啊,如果是对称和就好了
pxchg1200 7# 2013-4-19 21:23
4# reny 继续柯西啊, 我们要证明 \[ 4(x+y+z)^4(xy+yz+xz)\geq 27\left(\sum_{cyc}{xy^2}+3xyz\right)^{2}\] 注意到由Cauchy-Schwarz,我们有 \[\left(\sum_{cyc}{xy^2}+3xyz\right)^{2}=\left(\sum_{cyc}{xy(y+z)}\right)^{2}\leq (xy+yz+xz)\left(\sum{xy(y+z)^2} \right)\] 所以只要证 \[ 4(x+y+z)^4\geq 27\left(\sum{xy(y+z)^2} \right)\] 展开 \[  4\sum{x^4}+16\sum{x^3y}-11\sum{xy^3}+24\sum{x^2y^2}-33\sum{x^2yz}\geq 0 \] 现在套结论 \[ 3m(m+n)-(p^2+pg+g^2)=201>0 \] 其中\[ m=4,n=24,p=16,g=-11 \] 所以不等式得证!
reny 8# 2013-4-19 21:49
7# pxchg1200 $3m(m+n)-(p^2+pg+g^2)$是个结论吗? 怎么来滴
kuing 9# 2013-4-19 21:50
居然套用这个结论……
pxchg1200 10# 2013-4-19 22:31
8# reny Vo Quoc Ba Can的四次配方。 四次配方.pdf (65 KB)
pxchg1200 11# 2013-4-19 22:39
9# kuing 不套也行,用Vasc不等式,可以变成 \[ 4\left[(x^2+y^2+z^2)^2-3(xy^3+yz^3+zx^3)\right]+\sum{xy^3}+16\sum{x^3y}+16\sum{x^2y^2}-33\sum{x^2yz}\geq 0 \] 后面有 \[ \sum{xy^3}\geq xyz(x+y+z) \] 等价于 \[ \frac{y^2}{z}+\frac{z^2}{x}+\frac{x^2}{y}\geq x+y+z \] 同样有 \[ \sum{x^3y}\geq xyz(x+y+z)\] 和 \[ \sum{x^2y^2}\geq xyz(x+y+z) \]
kuing 12# 2013-4-19 23:44
11# pxchg1200 嗯,如果没计算错误的话,这样显然好多了
reny 13# 2013-4-20 09:48
11# pxchg1200 谢谢你的解答和提供的资料
pxchg1200 14# 2013-4-20 12:16
13# reny 不用客气,大家一起讨论学习。
thread-1392-1-1.html: 灌个水 喝点水
isea 1# 2013-4-18 23:59
画个图画累了
isea 2# 2013-4-19 00:09
又12点了,闪人,你们慢慢玩
李斌斌755 3# 2013-4-19 03:39
画啥图
isea 4# 2013-4-19 11:18
http://kkkkuingggg.5d6d.net/view ... &extra=#pid9275 5楼,下PDF,放大个400-800倍的看12题图就明白了 (没锯齿)
isea 5# 2013-4-19 22:28
再灌点水 嘿嘿
isea 6# 2013-4-20 23:50
签到,睡觉
thread-1393-1-2.html: [组合] Another problem for kuing
Gauss门徒 1# 2013-4-19 05:03
$32$ 名学生组成 $33$ 个组,每组人数为 $3$ 且每两组的交集不超过 $2$ 人,求证必有两组的交集是 $1$ 人
Gauss门徒 2# 2013-4-19 05:04
还是奖励萝莉一只~
Gauss门徒 3# 2013-4-19 05:05
其他人不要在kuing回复之前回复
thread-1394-1-2.html: [组合] 一道排列组合题
转化与化归 1# 2013-4-19 07:15
一道排列组合题
yes94 2# 2013-4-19 12:24
1# 转化与化归 (1)两种方法,方法一:求导得,$f'(0)=1+2+3+\cdots+n=\dfrac{n(n+1)}2$ 方法二:当取因式$(1+kx)$里的$kx$时,其余因式只能取1,对$k$求和得,$a_1=1+2+3+\cdots+n=\dfrac{n(n+1)}2$
yes94 3# 2013-4-19 12:50
2# yes94 (2)当取因式$(1+kx)(1+mx)$里的$kx$和$mx$时,其余因式只能取1,对$k、m$求和得,\[a_2=\sum_{1\leqslant k<m\leqslant n}{km}=?\] $2a_2+\dfrac{n(n+1)(2n+1)}{6}=\dfrac{[n(n+1)]^2}{4}$,故$a_2=\dfrac{(n-1)n(n+1)(3n+2)}{24}$,对不对? 其余可仿照这样做?或是建立递推? 中午休息一下,
转化与化归 4# 2013-4-19 18:00
3# yes94 a(2)是对的,这样的递推很难往后推啊!
yes94 5# 2013-4-19 18:36
4# 转化与化归 你有$a_3,a_4$答案没?写出来我看看,我怕算错了
转化与化归 6# 2013-4-19 20:45
5# yes94 没有答案,算到a(2)就结束了
转化与化归 7# 2013-4-19 21:25
本帖最后由 转化与化归 于 2013-4-19 21:27 编辑 5# yes94 刚算了一下a3,a4,供你参考
yes94 8# 2013-4-19 22:20
7# 转化与化归 那说明的确难算,我得到了关系式
thread-1395-1-1.html: 我擦,用word也不至于这样吧,真难想象……
kuing 1# 2013-4-19 16:31
真难想象为什么能排出这样的效果…… PS、这是刚才在人教群里看到的图,这里并不打算讨论原题目,如果想讨论的话可以自行去数学版重新发贴讨论。
isea 2# 2013-4-19 16:48
这一看就知道是复制别人的,自己机器上缺省相应的字体就这样了 word 文档 最难的 就是修改 不规范 的文档,还不如直接再打一遍
kuing 3# 2013-4-19 17:03
2# isea 即使是复制别人的,被复制的原文也一样糟糕。况且我觉得还未必。
老樊 4# 2013-4-20 23:15
我看见这样的文档就不想保存了,哈哈,真的不想看见这样不规范的文档
yes94 5# 2013-4-21 21:51
其实编辑一下,搞点times new man字体就好啦!在加上斜体也行
thread-1396-1-2.html: [不等式] 和伊朗96相似的
pxchg1200 1# 2013-4-19 22:49
设$a,b,c$为非负实数,且有$a+b+c=1$ 证明: \[ \frac{1}{4a+(b-c)^2}+\frac{1}{4b+(c-a)^2}+\frac{1}{4c+(a-b)^2}\geq\frac{9}{4} \]
pxchg1200 2# 2013-4-22 22:45
1# pxchg1200 额,好像很好SOS啊,但除了这个以外有没有AM-GM或CS proof ?
thread-1397-1-2.html: [不等式] 另一个问题
pxchg1200 1# 2013-4-19 22:54
设$a,b,c>0$ 且$a+b+c=3$,证明 \[ \frac{a}{a+bc}+\frac{b}{b+ac}+\frac{c}{c+ab}\geq\frac{3}{2} \] There exist a nice CS proof!
zdyzhj 2# 2013-4-20 16:42
it is not so difficult. i think.
pxchg1200 3# 2013-4-20 16:44
2# zdyzhj 本来就不难,县长同志。
yes94 4# 2013-4-20 22:59
3# pxchg1200 县长难得发一次言啊! 一个类似不等式: \[a,b,c > 0,a + b + c{\rm{ = }}3 \Rightarrow \frac{1}{{a + ab}} + \frac{1}{{b + bc}} + \frac{1}{{c + ca}} \geqslant \frac{3}{2}\]
kuing 5# 2013-4-20 23:31
4# yes94 这也用mathtype copy……一看等号就知道,你不觉得有些多余的代码么 ________ 补充 PS、这个显然简单太多了……
yes94 6# 2013-4-20 23:40
5# kuing 但是还是把$\ge$改成了$\geqslant$的,下次把rm删掉
isea 7# 2013-4-20 23:46
4# yes94 只要最终显示效果的人,我看着这个的数学公式源代码都想吐血
isea 8# 2013-4-20 23:48
3# pxchg1200 县长难得发一次言啊! 一个类似不等式: \[a,b,c > 0,a + b + c{\rm{ = }}3 \Rightarrow \frac{1}{{a + ab}} + \frac{1}{{b + bc}} + \frac{1}{{c + ca}} \geqslant \frac{3}{2}\] yes94 发表于 2013-4-20 22:59 我还以为你把主楼给解了 另外,我看着这些不等式都不一样,反正一个都不会,哈哈 特烦哪些流行的期刊,我的个天的,不知道有多少页都是不等式内容,于是,晕了一圈又一圈
kuing 9# 2013-4-21 00:54
6# yes94 只删 \rm 还不够,两边的 { } 也要删,a=b 与 a{=}b 是不一样的。 PS、还是不扯代码输入了,回正题吧。
地狱的死灵 10# 2013-4-21 01:39
3# pxchg1200 县长难得发一次言啊! 一个类似不等式: \[a,b,c > 0,a + b + c{\rm{ = }}3 \Rightarrow \frac{1}{{a + ab}} + \frac{1}{{b + bc}} + \frac{1}{{c + ca}} \geqslant \frac{3}{2}\] yes94 发表于 2013-4-20 22:59 $a + b + c = 3$ $a^2  + b^2  + c^2  \ge \frac{(a + b + c)^2 }{3} = 3$ $ab + bc + ca = \frac{(a + b + c)^2  - (a^2  + b^2  + c^2 )}{2} \le 3$ $\frac{1}{a + ab} + \frac{1}{b + bc} + \frac{1}{c + ca} \ge \frac{(1 + 1 + 1)^2 }{a + b + c + ab + bc + ca} \ge \frac{3}{2}$
yes94 11# 2013-4-21 13:48
10# 地狱的死灵 再来一个加强的: \[a,b,c,k > 0\Rightarrow \frac1{ka + ab} + \frac1{kb + bc} + \frac1{kc + ca} \geqslant \frac3{k^3 + abc}\]
huyuhbb 12# 2013-4-22 22:15

yes94 13# 2013-4-22 22:26
12# huyuhbb 11楼呢?
pxchg1200 14# 2013-4-22 22:44
13# yes94 11楼不成立,Bottema给出了反例.
kuing 15# 2013-4-22 22:45
12# huyuhbb 突然发现跟这个很像http://kkkkuingggg.5d6d.net/view ... &page=2#pid9342
kuing 16# 2013-4-22 22:45
14# pxchg1200 大概还是有 a+b+c=3 的条件吧
yes94 17# 2013-4-22 22:46
14# pxchg1200 反例是什么?
pxchg1200 18# 2013-4-22 22:47
17# yes94 \[ k=58,a=1,b=\frac{1}{521},c=\frac{1}{9}\]
pxchg1200 19# 2013-4-22 22:48
15# kuing 哈哈,本来就是这样的。Vasc在好久以前就出了这个,最近被 arqady重新提起来了。
yes94 20# 2013-4-22 22:53
18# pxchg1200 但是有人证明出来了,发表在正规刊物上。 我没法算你的反例,数字太大了 能否搞一个小一点的反例?
thread-1397-2-2.html:
pxchg1200 21# 2013-4-22 23:07
20# yes94 好像那个和你的不同啊。
yes94 22# 2013-4-22 23:10
21# pxchg1200 哪里不同啊?
kuing 23# 2013-4-22 23:14
是一样的…… yes 把证明内容截完整上来,看看怎么回事
yes94 24# 2013-4-22 23:23
23# kuing
pxchg1200 25# 2013-4-22 23:27
24# yes94    可是怎么Bottema验证有问题呢。
kuing 26# 2013-4-22 23:32
24# yes94 引理没错,错在这里 第二个 $\iff$ 并不等价,TA将 $\lambda^2-\sqrt[3]{abc}+\sqrt[3]{(abc)^2}$ 当成了 $\lambda^2-\lambda\sqrt[3]{abc}+\sqrt[3]{(abc)^2}$
yes94 27# 2013-4-22 23:50
px看看这个加强有没有反例? \[a,b,c,k > 0\Rightarrow \frac1{ka + ab} + \frac1{kb + bc} + \frac1{kc + ca} \geqslant \frac{3k}{k^3 + abc}\]
kuing 28# 2013-4-22 23:52
27# yes94 软件显示正确。 PS、刚才那回贴咋不见了……
pxchg1200 29# 2013-4-22 23:53
27# yes94 没有,这个是对的。
yes94 30# 2013-4-22 23:54
28# kuing 我自己输入错误,就删掉了,感谢kuing和px
yes94 31# 2013-4-22 23:58
px看看这个加强有没有反例? \[a,b,c,k > 0\Rightarrow \frac1{ka + ab} + \frac1{kb + bc} + \frac1{kc + ca} \geqslant \frac{3k}{k^3 + abc}\] yes94 发表于 2013-4-22 23:50 这个能否有漂亮的CS?
地狱的死灵 32# 2013-4-23 00:45
这个能否有漂亮的CS? yes94 发表于 2013-4-22 23:58 $\begin{array}{l} (k^3  + abc)\sum {\frac{1}{{ka + ab}}}  + 3k \\   = \sum {\frac{{k^3  + abc + k^2 a + kab}}{{ka + ab}}}  \\   = \sum {\frac{{k^2 (k + a)}}{{a(k + b)}}}  + \sum {\frac{{b(k + c)}}{{k + b}}}  \\   \ge 6k \\ \end{array}$
yes94 33# 2013-4-23 12:26
32# 地狱的死灵 没有用CS! 但是漂亮的6元均值不等式! 一个字:妙! 两个字:太妙! 这个添减项、拆项,恰到好处! 死灵是不出手则已,一出手就一鸣惊人!
kuing 34# 2013-4-23 12:50
牛比……
thread-1398-1-1.html: WinEdt 双击UTF-8编码的文件会出错 如果又不想改太多东西 可这样子
isea 1# 2013-4-19 22:54
如图双击打开文档时的错误 怎么办?很容易的办法,先打开WinEdt (或者此时Ctrl+O,即点 打开),选择 :UTF-8(*.*)—— 在这样的状态下打开文件,如图 接下来是选择文件 然后就——该干嘛干嘛 最后是,如果你默认是UTF-8文档,最好加上% !Mode:: "TeX:UTF-8",一般大家只会有中英文吧
isea 2# 2013-4-19 22:59
再或者CTEX套装集成的TeXworks……
kuing 3# 2013-4-19 23:32
大概可以先用记事本之类的东东打开,在前面加一行 % !Mode:: "TeX:UTF-8",保存再双击打开……
thread-1399-1-2.html: [不等式] 来着BQ的一题
pxchg1200 1# 2013-4-19 22:58
设$x,y,z>0$,$xy+yz+xz=1$,证明 \[ \frac{x^2}{1-xy}+\frac{y^2}{1-yz}+\frac{z^2}{1-zx}\geq x^2+y^2+z^2+\frac{1}{2}\]
pxchg1200 2# 2013-4-22 23:05
本帖最后由 pxchg1200 于 2013-4-22 23:09 编辑 唉,没人玩,我来柯西吧。 证明:不等式等价于 \[ \frac{x^2}{z(x+y)}+\frac{y^2}{x(y+z)}+\frac{x^2}{y(x+z)}\geq x^2+y^2+z^2+\frac{1}{2}(xy+yz+xz) \] 由Cauchy-Schwarz不等式,我们有 \[ \left(\sum{\frac{x^2}{z(x+y)}}\right)\left(\sum{(2x+y)^{2}\cdot z(x+y)}\right)\geq \left(2\sum{x^2}+\sum{xy}\right)^{2} \] 故只要证明 \[ 4\left(\sum{x^2}+\frac{1}{2}\sum{xy}\right)(xy+yz+xz)\geq \sum{(2x+y)^{2}\cdot z(x+y)} \] 展开就是 \[  3\sum{xz^3}+2\sum{x^2y^2}\geq 5xyz\sum{x} \] 即 \[ 2\sum{\frac{xy}{z}}+3\sum{\frac{z^2}{y}}\geq 5(x+y+z) \] 由AM-GM和Cauchy-Schwarz显然。 Done!
tan9p 3# 2013-4-23 22:30
$4\left(\sum{x^2}+\frac{1}{2}\sum{xy}\right)(xy+yz+xz)\geq \sum{(2x+y)^{2}\cdot z(x+y)}$ 没看懂,后来懂了。
reny 4# 2013-4-24 13:02
本帖最后由 reny 于 2013-4-24 13:09 编辑 看来2很重要啊,$\left(\sum{\frac{x^2}{z(x+y)}}\right)\left(\sum(\Large{\color{red}{2}}x+y)^{2}\cdot z(x+y)\right)\geqslant \left(2\sum{x^2}+\sum{xy}\right)^{2}$
kuing 5# 2013-4-24 13:33
看来2很重要啊,$\left(\sum{\frac{x^2}{z(x+y)}}\right)\left(\sum(\Large{\color{red}{2}}x+y)^{2}\cdot z(x+y)\right)\geqslant \left(2\sum{x^2}+\sum{xy}\right)^{2}$ reny 发表于 2013-4-24 13:02 \Large{\color{red}{2}}  -->  {\Large\color{red}{2}} $\left(\sum{\frac{x^2}{z(x+y)}}\right)\left(\sum({\Large\color{red}{2}}x+y)^{2}\cdot z(x+y)\right)\geqslant \left(2\sum{x^2}+\sum{xy}\right)^{2}$
yes94 6# 2013-4-24 13:52
5# kuing 什么叫BQ?
kuing 7# 2013-4-24 13:56
6# yes94 一个不等式机器dǎng……
reny 8# 2013-4-24 14:42
5# kuing e,这样啊,我一直没找到哪里错了,
thread-14-1-9.html: [不等式] 三元轮换$\sum{a^4/b^2}+5\sum ab\ldots$
转贴专用 1# 2011-9-26 19:06
已知 $a,b,c>0$,求证 \[\frac{a^4}{b^2}+\frac{b^4}{c^2}+\frac{c^4}{a^2}+5(ab+bc+ca)\geqslant 6(a^2+b^2+c^2)\] 未解决 转自 http://bbs.pep.com.cn/thread-1849766-1-1.html
pxchg1200 2# 2011-10-5 22:35
Can 的证明....
kuing 3# 2011-10-5 22:38
慢慢研究下
pxchg1200 4# 2011-10-5 22:40
主要是SOS-Schur....
thread-140-1-2.html: 再问个小问题!
贵族风铃 1# 2011-10-24 16:47
273→42→8 248→64→24→8 378→???→48→32→6
kuing 2# 2011-10-24 16:58
随意
贵族风铃 3# 2011-10-24 17:03
2# kuing 哎。。。。。
海盗船长 4# 2012-1-14 20:53
各位数字相乘啊
Chetion 5# 2012-1-20 22:28
1# 贵族风铃 呃呃~~~怀旧~关于小学时,按规律写数字的题目。 运算法则是: 二位数中,个位十位相乘。 三位数中,先是个位十位之积,再与百位数字相乘。 所以结果应该是168。 看样子这么多年来,我还没有遗失小学生的想像力。
Chetion 6# 2012-1-20 22:31
上面还有一帖早已揭晓结果。我很冒失呃。。 看来想像力还有点,但是思维的惯性越来越滞重。 惭愧呃。。闪也。
thread-1400-1-2.html: [几何] QQ中看到的一个恒成立问题,先放到几何类中吧!
hongxian 1# 2013-4-20 10:21
求使$\abs{\sin\theta-p\cos\theta-q}\leqslant\dfrac{\sqrt2-1}{2}$对任意$\theta\in\left[0,\dfrac{\pi}{2}\right]$恒成立的实数对$(p,q)$
李斌斌755 2# 2013-4-20 10:48
本帖最后由 李斌斌755 于 2013-4-20 11:51 编辑 $x^2+y^2=1,x\ge0,y\ge0$ 想到直线距离公式 $d=\frac{|x-py-q|}{\sqrt{1+p^2}}$ 想不下去了
李斌斌755 3# 2013-4-20 11:44
本帖最后由 李斌斌755 于 2013-4-22 00:18 编辑 计算太烦,直线l是点B(0,1)到其距离为$\sqrt{2}-1$,原点A到其距离为$1$的直线, 直线k是平行l其过AC中点的直线,数对是直线k的斜率倒数及常数项,还有对应一直线, 共有两个数对
李斌斌755 4# 2013-4-20 11:57
再补个图
转化与化归 5# 2013-4-20 12:10
4# 李斌斌755 应该是一个数对吧!
转化与化归 6# 2013-4-20 12:49
一个代数的解答
yes94 7# 2013-4-20 12:59
352的空间很早就有这个结果,但无解答,做的好!
转化与化归 8# 2013-4-20 13:04
7# yes94 是个好题,大家继续讨论,方法应该很多!
李斌斌755 9# 2013-4-20 13:32
如果有几何意义,我怎么感觉不止一个解
kuing 10# 2013-4-20 13:58
以前我也见过,不过暂时没翻到贴子或聊天记录……
yes94 11# 2013-4-20 14:11
9# 李斌斌755 说说你的答案或者疑惑?
李斌斌755 12# 2013-4-20 14:28
本帖最后由 李斌斌755 于 2013-4-22 00:21 编辑 11# yes94 原式两边同除$\sqrt{1+p^2}$后,其几何意义为单位圆第一象限上的点到某一直线的距离不大于一变数, 存不存在任一$p$值,对应有一个$q$与之相配。 不懂$latex$ 有没有几何意义,高手们帮忙,我想蒙了,……
yes94 13# 2013-4-20 14:47
7# yes94 是个好题,大家继续讨论,方法应该很多! 转化与化归 发表于 2013-4-20 13:04 方法应该很多? 楼主再搞几个方法呢? 下面给出一个方法: 必要性: \[\begin{array}{l} 设f(\theta ) = \sin \theta  - p\cos \theta  - q,\\ 可以验证:\frac{{\sqrt 2 }}{2}f(0) - f(\frac{\pi }{4}) + \frac{{2 - \sqrt 2 }}{2}f(\frac{\pi }{2}) = 1 - \sqrt 2 \\ 由题意,\frac{{\sqrt 2  - 1}}{2} \geqslant |f(0)|,\frac{{\sqrt 2  - 1}}{2} \geqslant |f(\frac{\pi }{4})|,\frac{{\sqrt 2  - 1}}{2} \geqslant |f(\frac{\pi }{2})|\\ 于是,\frac{{\sqrt 2  - 1}}{2}(\frac{{\sqrt 2 }}{2} + 1 + \frac{{2 - \sqrt 2 }}{2})\\ \geqslant |\frac{{\sqrt 2 }}{2}f(0)| + |f(\frac{\pi }{4})| + |\frac{{2 - \sqrt 2 }}{2}f(\frac{\pi }{2})|\\ \geqslant |\frac{{\sqrt 2 }}{2}f(0) - f(\frac{\pi }{4}) + \frac{{2 - \sqrt 2 }}{2}f(\frac{\pi }{2})|\\ = \sqrt 2  - 1\\ \sqrt 2  - 1 \geqslant |\frac{{\sqrt 2 }}{2}f(0)| + |f(\frac{\pi }{4})| + |\frac{{2 - \sqrt 2 }}{2}f(\frac{\pi }{2})| \\\geqslant |\frac{{\sqrt 2 }}{2}f(0) - f(\frac{\pi }{4}) + \frac{{2 - \sqrt 2 }}{2}f(\frac{\pi }{2})| = \sqrt 2  - 1\\ 故|\frac{{\sqrt 2 }}{2}f(0) - f(\frac{\pi }{4}) + \frac{{2 - \sqrt 2 }}{2}f(\frac{\pi }{2})| =\sqrt 2  - 1 \\当且仅当f(0){\rm{ = }} - f(\frac{\pi }{4}){\rm{ = }}f(\frac{\pi }{2}){\rm{ = }}\frac{{1 - \sqrt 2 }}{2}取等号\\ 解得p=-1,q=\frac{\sqrt2+1}2 \end{array}\] 充分性略去(估计原题不是错题吧?$p、q$是存在的吧?所以就略去充分性了)
kuing 14# 2013-4-20 14:49
13# yes94 一看就知道你又用 mathtype copy 出来的 ___________ 补充 PS、有点像那类二次函数绝对值问题的玩法……
yes94 15# 2013-4-20 14:53
14# kuing 本来想用latex的,但是latex的确难打, 最烦的是,汉字不能复制,还得重新在 草稿本上加了汉字。显得不爽
yes94 16# 2013-4-20 16:00
14# kuing 嘻嘻,k一下子就看出了,三次函数、四次函数等高次函数都可以这样玩
转化与化归 17# 2013-4-20 21:56
如果没有几何的解法,本题应该归入代数类型了
isea 18# 2013-4-20 22:04
11# yes94 原式两边同除$sqrt{1+p_2}$后,其几何意义为单位圆第一象限上的点到某一直线的距离不大于一变数, 存不存在任一$p$值,对应有一个$q$与之相配。 不懂$latex$ 有没有几何意义,高手们帮 ... 李斌斌755 发表于 2013-4-20 14:28 都就是少打了个 \   即\sqrt{1+p_2} 所以不显示。一般 根号,分式,函数名,向量等都需要加\表示数学公式开始。 论坛里打的公式就是以LaTeX为底层显示的,没什么的 14# kuing 本来想用latex的,但是latex的确难打, 最烦的是,汉字不能复制,还得重新在 草稿本上加了汉字。显得不爽 yes94 发表于 2013-4-20 14:53 已经安装了CTEX套装的,只需要在以下代码下输入中文与公式即可,在WinEd里,与论坛里帖子一样。 \documentclass{ctexart} \usepackage{amsmath,amssymb,amsthm} \begin{document} 开始输入,汉字就打汉字,数学就打数学公式,就这么简单 \end{document} 复制代码
yes94 19# 2013-4-20 22:48
18# isea 在word里的aurora可以这样吗?aurora在word里不能输入汉字
isea 20# 2013-4-20 23:07
19# yes94 我这里不能,不能加什么宏包,可能是字体设置的问题。 就是能加,也不建议,因为调整大小时麻烦。
thread-1400-2-2.html:
kuing 21# 2013-4-21 16:25
求使$\abs{\sin\theta-p\cos\theta-q}\leqslant\dfrac{\sqrt2-1}{2}$对任意$\theta\in\left[0,\dfrac{\pi}{2}\right]$恒成立的实数对$(p,q)$ hongxian 发表于 2013-4-20 10:21 状态低迷,写个笨方法。 记 $f(\theta )=\abs{\sin \theta -p\cos \theta -q}$ 其中 $\theta \in [0,\pi/2]$。由合一公式 \[\sin \theta -p\cos \theta =\sqrt{1+p^2}\sin (\theta -\arctan p),\] 由于 $\theta -\arctan p\in [-\arctan p,\pi/2-\arctan p]$,为确定 $\sin \theta -p\cos \theta$ 的范围,下面分类讨论。 (1)若 $p<-1$,则 $\pi/4<-\arctan p<\pi/2$, $3\pi/4<\pi/2-\arctan p<\pi$,故此时有 \[1\geqslant \sin (\theta -\arctan p)\geqslant \sin \left( \frac\pi2-\arctan p \right)=\frac1{\sqrt{1+p^2}};\] (2)若 $-1\leqslant p\leqslant 0$,则 $0\leqslant -\arctan p\leqslant \pi/4$, $\pi/2\leqslant \pi/2-\arctan p\leqslant 3\pi/4$,故此时有 \[1\geqslant \sin (\theta -\arctan p)\geqslant \sin (-\arctan p)=-\frac p{\sqrt{1+p^2}};\] (3)若 $p>0$,则 $-\pi/2<-\arctan p<0<\pi/2-\arctan p<\pi/2$,故此时有 \[\frac1{\sqrt{1+p^2}}=\sin \left( \frac\pi2-\arctan p \right)\geqslant \sin (\theta -\arctan p)\geqslant \sin (-\arctan p)=-\frac p{\sqrt{1+p^2}}.\] 综上得 \[\sin \theta -p\cos \theta \in \begin{cases} \bigl[1,\sqrt{1+p^2}\bigr], & p<-1, \\ \bigl[-p,\sqrt{1+p^2}\bigr], & -1\leqslant p\leqslant 0, \\ [-p,1], & p>0. \end{cases}\] 注意到这样一个显然的事实:当 $M\in [a,b]$ 时,当且仅当 $c=(a+b)/2$ 时,$\abs{M-c}$ 的最大值取得最小值,其值为 $(b-a)/2$。 所以,当 $p$ 取定时,若记 $g(p)$ 为 $f(\theta)$ 的最大值的最小值(对于变化的 $q$ 来说),则 \[ g(p)=\begin{cases} \frac{\sqrt{1+p^2}-1}2, & p<-1, \\ \frac{\sqrt{1+p^2}+p}2, & -1\leqslant p\leqslant 0, \\ \frac{1+p}2, & p>0, \end{cases} \] 这样,$g(p)$ 的最小值就是 $f(\theta)$ 的最大值的最小值(对于变化的 $p$, $q$ 来说)。 不难看出 $g(p)$ 是 $\mbb R$ 上的连续函数,由单调性易知当且仅当 $p=-1$ 时 $g(p)$ 取最小值 $\bigl(\sqrt2-1\bigr)/2$,此时相应的 $q$ 应满足 $q=\bigl(-p+\sqrt{1+p^2}\bigr)/2=\bigl(1+\sqrt2\bigr)/2$,也就是说当且仅当 $p=-1$ 且 $q=\bigl(1+\sqrt2\bigr)/2$ 时 $f(\theta)$ 的最大值取得最小值 $\bigl(\sqrt2-1\bigr)/2$,回到原题,所求数对便是它们。
kuing 22# 2013-4-21 16:58
后面可以简化一些,在得到 \[\sin \theta -p\cos \theta \in \begin{cases} \bigl[1,\sqrt{1+p^2}\bigr], & p<-1, \\ \bigl[-p,\sqrt{1+p^2}\bigr], & -1\leqslant p\leqslant 0, \\ [-p,1], & p>0 \end{cases}\] 时,不难发现它实际上等价于 \[\sin \theta -p\cos \theta \in \bigl[-\max\{p,-1\} , \sqrt{1+(\min\{p,0\})^2}\bigr],\] 所以 $g(p)$ 也可以简化为 \[g(p)=\frac{\sqrt{1+(\min\{p,0\})^2}+\max\{p,-1\}}2.\] 不过其实没什么实际作用。
yes94 23# 2013-4-21 17:08
22# kuing 应该有用吧, 改天看看能否借助你的这个结果。
转化与化归 24# 2013-4-21 18:28
希望看到几何的解法
goft 25# 2013-4-21 19:59
$x^2+y^2=1,x\ge0,y\ge0$ 想到直线距离公式 $d=\frac{|x-py-q|}{\sqrt{1+p^2}}$ 想不下去了 李斌斌755 发表于 2013-4-20 10:48 根据平行线夹着圆可知,结果必是l1与l2,或l3与l4,讨论结果可知是l3与l4。 不知道有没有问题,大家看看,理由改怎么讲
yes94 26# 2013-4-21 21:37
25# goft goft终于出马了! 这个月跑哪去了?
李斌斌755 27# 2013-4-21 23:40
本帖最后由 李斌斌755 于 2013-4-22 03:36 编辑 不知这样说不说得过去 \[|\sin\theta-p\cos\theta-q|\leqslant\dfrac{\sqrt2-1}2\\\iff\frac{|\sin\theta-p\cos\theta-q|}{\sqrt{1+p^2}}\leqslant\dfrac{\sqrt2-1}{2\sqrt{1+p^2}}    (1)\] 设\[d=\frac{|\sin\theta-p\cos\theta-q|}{\sqrt{1+p^2}}\]其几何意义为第一象单位圆上的点到直线$x-py-q=0$的距离,令数对$(p,q)$对应一条直线,对于每条直线设\[M=d_{\max}\]则根据图像知\[M_{\min}=\frac{2-\sqrt2}4\]其对应的数对为\[(-1,\frac{\sqrt2+1}2)\]此时直线满足\[M_{\min}=d_{\max}=\frac{\sqrt2-1}{2\sqrt{1+p^2}}=\frac{2-\sqrt2}4\]即数对$(-1,\dfrac {\sqrt2+1}2)$也就是\[p=-1,q_0=OC=\frac{\sqrt2+1}2\] 满足\[|\sin\theta-p\cos\theta-q|\leqslant\frac{\sqrt2-1}2\] 当\[|p|>1\](1)式左边增大,右边减小,不等式不成立,即不存在数对使不等式成立。 当\[p=1\](1)式左边增大,右边不变,不等式不成立,即不存在数对使不等式成立。 当\[|p|<1\]由图像有 \[d_{\max}=\frac{|\sin\theta-p\cos\theta-q|}{\sqrt{1+p^2}}=\frac{|1-q|}{\sqrt{1+p^2}}\] \[q=OD>OC=\frac{\sqrt2+1}2\iff\] \[d_{\max}=\frac{q-1}{\sqrt{1+p^2}}>\frac{\sqrt2-1}{2\sqrt{1+p^2}}\]即(1)式不成立,即不存在数对使不等式成立。 图像只考虑$0<p<1$时情况$-1<p<0$时一样    综上所述只有数对$(1,\dfrac{\sqrt2+1}2)$满足条件 图中两条实直线分别为同颜色两条虚直线的中间线
李斌斌755 28# 2013-4-21 23:48
编辑好累。
yes94 29# 2013-4-21 23:49
27# 李斌斌755 latex进展神速! 没仔细看,但看见部分代码可以修改,例如: 代码M_{min}=d_{man}=\frac{\sqrt2-1}{2\sqrt{1+p^2}}=\frac{2-\sqrt2}4的显示效果是: \[M_{min}=d_{man}=\frac{\sqrt2-1}{2\sqrt{1+p^2}}=\frac{2-\sqrt2}4\] 明显,max写成了man了!min和max要加\!即 代码应该改为:M_{\min}=d_{\max}=\frac{\sqrt2-1}{2\sqrt{1+p^2}}=\frac{2-\sqrt2}4 显示效果是:\[M_{\min}=d_{\max}=\frac{\sqrt2-1}{2\sqrt{1+p^2}}=\frac{2-\sqrt2}4\]
李斌斌755 30# 2013-4-22 00:00
29# yes94 谢谢,已修改。
kuing 31# 2013-4-22 00:03
难得,虽然我还是看不懂……
李斌斌755 32# 2013-4-22 00:05
31# kuing 对不对
李斌斌755 33# 2013-4-22 00:13
本帖最后由 李斌斌755 于 2013-4-22 00:41 编辑 32# 李斌斌755 思路就是先证明$x+y+\dfrac{\sqrt2+1}2=0$(由图像知此时圆弧上的的到直线的最大值最小)时不等式成立,再证明$|p|\ne1,p=1$时不成立。
yes94 34# 2013-4-22 00:33
33# 李斌斌755 明天沿着kuing的那种解法,再给出一个新解法。
李斌斌755 35# 2013-4-22 00:41
34# yes94
hongxian 36# 2013-4-22 02:06
28# 李斌斌755 谢谢了!思路非常好,不过我感觉 $\abs{p}<1$时 $d_{\max}=\left[\dfrac{|\sin\theta-p\cos\theta-q|}{\sqrt{1+p^2}}\right]_{\max}=\dfrac{|1-q|}{\sqrt{1+p^2}}$ 好象还是有点问题,也有可能我自己没有理解清楚!
hongxian 37# 2013-4-22 02:20
27# 李斌斌755 想明白了一点,改成 $\abs{p}<1$时 $d_{\max}=\left[\dfrac{|\sin\theta-p\cos\theta-q|}{\sqrt{1+p^2}}\right]_{\max}\geqslant\dfrac{|1-q|}{\sqrt{1+p^2}}$ 应该就可以了,谢谢了!
李斌斌755 38# 2013-4-22 02:55
本帖最后由 李斌斌755 于 2013-4-22 09:18 编辑 37# hongxian 不客气 当$|p|<1$时\[\left[d_{\max}\right]_{min}\equiv\dfrac{|1-q|}{\sqrt{1+p^2}}=\dfrac{q-1}{\sqrt{1+p^2}}>\dfrac{q_0-1}{\sqrt{1+p^2}}=\dfrac{\sqrt2-1}{\sqrt{1+p^2}}\]($B$点到斜率为$p$且与圆弧相切的直线距离的一半),由图像易知此时的$q>q_0>1$,$q_0=\dfrac{\sqrt2+1}2$为$M_{\min}$的$q$值
李斌斌755 39# 2013-4-22 03:52
本帖最后由 李斌斌755 于 2013-4-22 03:54 编辑 37# hongxian 是的,对于$p$一样,$q$变化时,$d_{\max}$的范围是有下界,无上界。下界为$\dfrac{|1-q|}{\sqrt{1+p^2}}$
李斌斌755 40# 2013-4-22 04:08
37# hongxian 怎样打大中括号
thread-1400-3-2.html:
hongxian 41# 2013-4-22 05:34
本帖最后由 hongxian 于 2013-4-22 05:39 编辑 40# 李斌斌755 \left[中间填内容\right] 例如:\left[\dfrac12\right]的效果就是$\left[\dfrac12\right]$
李斌斌755 42# 2013-4-22 09:14
41# hongxian 谢谢!
李斌斌755 43# 2013-4-22 11:18
本帖最后由 李斌斌755 于 2013-4-22 18:45 编辑 这样是不是好点\[|\sin\theta-p\cos\theta-q|\leqslant\dfrac{\sqrt2-1}2\\\iff\frac{|\sin\theta-p\cos\theta-q|}{\sqrt{1+p^2}}\leqslant\dfrac{\sqrt2-1}{2\sqrt{1+p^2}}    (1)\] 设\[d=\frac{|\sin\theta-p\cos\theta-q|}{\sqrt{1+p^2}}\]其几何意义为第一象单位圆上的点到直线$x-py-q=0$的距离,令数对$(p,q)$对应一条直线. 根据图像分三种情况考虑 1)\[p=0\]\[\left[d_{\max}\right]_{\min}=\dfrac12>\dfrac{\sqrt2-1}2\] 不存在实数对。 2)\[p>0\]当$p$倾向于无穷大或$0$时\[\left[d_{\max}\right]_{\min}>\dfrac12>\dfrac{\sqrt2-1}{2\sqrt{1+p^2}}\]即不存在数对使不等式成立。 3)\[p<0\]由图像有 \[\left[d_{\max}\right]_{\min}=\dfrac{2-\sqrt2}4=\dfrac{2-\sqrt2}4\] 由图像得此时直线方程为\[x+y-\dfrac{\sqrt2+1}2=0\]即$p=-1,q=\dfrac{\sqrt2+1}2$不等式成立,即存在实数对$(-1,\dfrac{\sqrt2+1}2)$使不等式成立。    综上所述只有数对$(-1,\dfrac{\sqrt2+1}2)$满足条件. 不知道上述中的$\left[d_{\max}\right]_{\min}$是否还需要的严格证明。
hongxian 44# 2013-4-22 13:22
43# 李斌斌755 分母中的$\sqrt{1+p^2}$好象都写掉了,$p\geqslant0$时没有什么影响,$p<0$时估计还是要分三类$p\in (-\infty,-1)$,$P=-1$和$p\in(-1,0)$来说。
hongxian 45# 2013-4-22 13:33
43# 李斌斌755 $p<0$时,好象 $\left[d_{\max}\right]_{\min}=\dfrac{\sqrt2-1}{2\sqrt2}$
李斌斌755 46# 2013-4-22 13:47
本帖最后由 李斌斌755 于 2013-4-22 13:49 编辑 回复44#45# 谢谢,43#已修改,当$p<0$时只有$p=-1,q=\dfrac{\sqrt2-1}2$时,$d$才取得最小值(由图像易知,不知需不需要证明),只要$p,q$中有一或两个变化,$d$值就会变大。
李斌斌755 47# 2013-4-22 18:16
本帖最后由 李斌斌755 于 2013-4-23 09:17 编辑 如果设成直线的系数项分别为$sin{\theta},cos{\theta}$就好了,可不分类讨论$p$……
yes94 48# 2013-4-22 18:39
47# 李斌斌755 你43楼的倒数第二行的数对写错啦!是(-1,…),你写成(1,…) 还有,三角函数用\sin,\cos,显示效果:$\sin\theta,\cos\alpha$ 你的sin,cos显示效果:$sin\theta,cos\alpha$
yes94 49# 2013-4-22 21:59
22# kuing 应该有用吧, 改天看看能否借助你的这个结果。 yes94 发表于 2013-4-21 17:08 现在借用kuing的结果: 先定义区间长度函数$length[a,b]=b-a$. \[设x = \sin \theta  - p\cos \theta  \in \left\{ {\begin{array}{*{20}{l}} {[1,\sqrt {1 + {p^2}} ],}&{p <  - 1,}\\ {[ - p,\sqrt {1 + {p^2}} ],}&{ - 1\leqslant p\leqslant 0,}\\ {[ - p,1],}&{p > 0.} \end{array}} \right.\] \[\begin{array}{l} |x - q|\leqslant \frac{{\sqrt 2  - 1}}{2} \Leftrightarrow q - \frac{{\sqrt 2  - 1}}{2}\leqslant x\leqslant q + \frac{{\sqrt 2  - 1}}{2} \Leftrightarrow x \in [q - \frac{{\sqrt 2  - 1}}{2},q + \frac{{\sqrt 2  - 1}}{2}]\\ \Rightarrow length[q - \frac{{\sqrt 2  - 1}}{2},q + \frac{{\sqrt 2  - 1}}{2}] = \sqrt 2  - 1\\ If{\kern 1pt} {\kern 1pt} {\kern 1pt} {\kern 1pt} {\kern 1pt} {\kern 1pt} {\kern 1pt} {\kern 1pt} {\kern 1pt} {\kern 1pt} p > 0 \Rightarrow x \in [ - p,1] \Rightarrow length[ - p,1] = 1 + p > 1 > \sqrt 2  - 1\\ If{\kern 1pt} {\kern 1pt} {\kern 1pt} {\kern 1pt} {\kern 1pt} {\kern 1pt} {\kern 1pt} {\kern 1pt} {\kern 1pt} {\kern 1pt} p <  - 1 \Rightarrow x \in [1,\sqrt {1 + {p^2}} ] \Rightarrow length[1,\sqrt {1 + {p^2}} ] = \sqrt {1 + {p^2}}  - 1 > \sqrt 2  - 1\\ If  {\kern 1pt} {\kern 1pt} {\kern 1pt}  - 1\leqslant p\leqslant0 \Rightarrow x \in [ - p,\sqrt {1 + {p^2}} ] \Rightarrow length[ - p,\sqrt {1 + {p^2}} ] = \sqrt {1+ {p^2}}+ p = \frac{1}{{\sqrt {1+{p^2}}-p}}\geqslant\sqrt 2-1\\ When{\rm{ }}{\kern 1pt} {\kern 1pt} {\kern 1pt} and{\rm{ }}{\kern 1pt} {\kern 1pt} {\kern 1pt} only{\rm{ }}{\kern 1pt} {\kern 1pt} {\kern 1pt} when{\kern 1pt} {\kern 1pt} {\kern 1pt} {\kern 1pt} {\kern 1pt} {\kern 1pt} {\kern 1pt} p = - 1{\kern 1pt} {\kern 1pt} {\kern 1pt} {\kern 1pt} {\kern 1pt} {\kern 1pt} take{\kern 1pt} {\kern 1pt} {\kern 1pt} {\kern 1pt} {\kern 1pt} {\kern 1pt}  "= "\\ So{\kern 1pt} {\kern 1pt} {\kern 1pt} {\kern 1pt} {\kern 1pt} {\kern 1pt} {\kern 1pt} p =  - 1 \Rightarrow x \in [ - p,\sqrt {1 + {p^2}} ] = [1,\sqrt 2 ]\\ Because{\kern 1pt} {\kern 1pt} {\kern 1pt} {\kern 1pt} {\kern 1pt} {\kern 1pt} {\kern 1pt} {\kern 1pt} {\kern 1pt} {\kern 1pt} x \in [q - \frac{{\sqrt 2  - 1}}{2},q + \frac{{\sqrt 2  - 1}}{2}] \Rightarrow length[q - \frac{{\sqrt 2  - 1}}{2},q + \frac{{\sqrt 2  - 1}}{2}] = \sqrt 2  - 1\\ So{\kern 1pt} {\kern 1pt} {\kern 1pt} {\kern 1pt} {\kern 1pt} q - \frac{{\sqrt 2  - 1}}{2} = 1{\kern 1pt} {\kern 1pt} {\kern 1pt} {\kern 1pt} {\kern 1pt} {\kern 1pt} {\kern 1pt} and{\kern 1pt} {\kern 1pt} {\kern 1pt} {\kern 1pt} {\kern 1pt} q + \frac{{\sqrt 2  - 1}}{2} = \sqrt 2  \Rightarrow q = \frac{{\sqrt 2  + 1}}{2} \end{array}\]
李斌斌755 50# 2013-4-22 22:19
49# yes94 中英文结合
isea 51# 2013-4-22 22:28
请问,这么高的楼,有定性的结论了么?
转化与化归 52# 2013-4-22 22:30
再发一个几何解
yes94 53# 2013-4-22 22:37
52# 转化与化归 这个已经想过,正想写一下的,只是和你的大同小异
转化与化归 54# 2013-4-22 22:39
49# yes94 突然发现,和你的解法基本是一样的
yes94 55# 2013-4-22 22:57
49# yes94 突然发现,和你的解法基本是一样的 转化与化归 发表于 2013-4-22 22:39
hongxian 56# 2013-4-23 11:08
52# 转化与化归 好方法!
yes94 57# 2013-4-23 12:34
纵截距之差为定值$\sqrt2-1$,就和我定义的length函数是一样的效果。 可能我的length函数把人吓住了吧? 改天再给出本题的个人第三种解法,希望能有人看懂。
yes94 58# 2013-4-26 00:10
57# yes94 这么晚了,还是明天或者后天吧
转化与化归 59# 2013-4-26 05:32
58# yes94 期待中
李斌斌755 60# 2013-4-26 11:14
47# 李斌斌755 把左边看成直线$x=1$上的点到直线$x\sin\theta-y\cos\theta-q=0$的距离,由题意知,直线\[x\sin\theta-y\cos\theta-q=0\]必须在直线\[x=\dfrac{3-\sqrt2}2\]及直线\[x=\dfrac{\sqrt2+1}2\]范围内(含边界),即\[\dfrac{3-\sqrt2}2\leqslant x=\dfrac{q+p\cos\theta}{\sin\theta}\leqslant\dfrac{\sqrt2+1}2\] 下面就是代数运算,真烦……
thread-1400-4-2.html:
yes94 61# 2013-4-26 11:19
60# 李斌斌755 你很擅长几何
李斌斌755 62# 2013-4-26 11:47
本帖最后由 李斌斌755 于 2013-4-26 11:52 编辑 61# yes94 可这个\[\dfrac{3-\sqrt2}2\leqslant \dfrac{q+p\cos\theta}{\sin\theta}\leqslant\dfrac{\sqrt2+1}2(0\leqslant\theta\leqslant\dfrac{\pi}2)\]不会求呀
yes94 63# 2013-4-26 13:25
62# 李斌斌755 这个楼层盖的很高啦! 是不是最高的楼层? 好的,我来一种方法: \[\begin{array}{l} 设f(\theta ) = \sin \theta  - p\cos \theta ,\\ |f({\theta _1}) - q| \leqslant \dfrac{{\sqrt 2  - 1}}{2},|f({\theta _2}) - q| \leqslant \dfrac{{\sqrt 2  - 1}}{2} \Rightarrow |f({\theta _1}) - f({\theta _2})| \leqslant |f({\theta _1}) - q| + |f({\theta _2}) - q| \leqslant \sqrt 2  - 1\\ 如果{\kern 1pt} {\kern 1pt} {\kern 1pt} {\kern 1pt} {\kern 1pt} |f({\theta _1}) - f({\theta _2})| = |f(\arcsin \dfrac{1}{{\sqrt {1 + {p^2}} }}) - f(\dfrac{\pi }{2})| = |\sqrt {1 + {p^2}}  - 1| = \sqrt {1 + {p^2}}  - 1 \leqslant \sqrt 2  - 1 \Rightarrow {p^2} \leqslant 1\\ 如果{\kern 1pt} {\kern 1pt} {\kern 1pt} {\kern 1pt} {\kern 1pt} |f({\theta _1}) - f({\theta _2})| = |f(\arcsin \dfrac{1}{{\sqrt {1 + {p^2}} }}) - f(0)| = |\sqrt {1 + {p^2}}  + p| = \sqrt {1 + {p^2}}  + p \leqslant \sqrt 2  - 1 \Rightarrow p \leqslant-1\\ So{\kern 1pt} {\kern 1pt} {\kern 1pt} {\kern 1pt} {\kern 1pt} p =-1 \end{array}\]
转化与化归 64# 2013-4-26 19:27
63# yes94 賦值法的一种另一种形式,很好!
李斌斌755 65# 2013-4-26 20:41
本帖最后由 李斌斌755 于 2013-4-27 01:43 编辑 63# yes94 你再来十种八种方法这楼就能盖到$100$层 我再来一种方法看跟你的一样吗。令$M=\sin\theta-p\cos\theta$,对于一取定的$p$值,设$n=M_{\max}-M_{\min}$根据线性规划有 (红色为$p=-1$时$M_{\max},M_{\min}$,蓝色为$p=0$时$M_{\max},M_{\min}$,黑色为$p=2$时$M_{\max},M_{\min}$) \[n\in(\sqrt2-1,1](p\leqslant0),n\in(1,\infty)(p>0)\]对于任一个$p$都有相应的一个$n$与之对应。对于任一数对$(p,q)$若想使$|M-q|_{\max}$有最小值,只有$q=\dfrac n2$时\[|M-q|\leqslant\dfrac n2\]取等条件\[q=\dfrac{M_{\max}+M_{min}}2\] 故原式成立条件为\[n=M_{\max}-M_{\min}=\sqrt2-1,p=-1,q=\dfrac{M_{\max}+M_{min}}2=\dfrac{\sqrt2+1}2\]
yes94 66# 2013-4-26 21:56
65# 李斌斌755 我已证明:$\forall {\theta _1},{\theta _2} \in [0,\dfrac{\pi }{2}] \Rightarrow |f({\theta _1}) - f({\theta _2})| \leqslant |f({\theta _1}) - q| + |f({\theta _2}) - q| \leqslant \sqrt 2  - 1$, 你设$M=\sin\theta-p\cos\theta$,$N=M_{\max}-M_{\min}$,根据线性规划有$N\in(\sqrt2-1,1](p\leqslant0)$,$N\in(1,\infty)(p>0)$ , 但是,\[N=M_{\max}-M_{\min}=|f({\theta _1}) - f({\theta _2})| \leqslant |f({\theta _1}) - q| + |f({\theta _2}) - q| \leqslant \sqrt 2  - 1\]如何解释? 顺便说一下,N\in(\sqrt2-1,1](p\leqslant0),N\in(1,\infty)(p>0)的显示效果: \[N\in(\sqrt2-1,1](p\leqslant0),N\in(1,\infty)(p>0)\]
李斌斌755 67# 2013-4-26 22:22
本帖最后由 李斌斌755 于 2013-4-27 01:58 编辑 66# yes94 $N\in[\sqrt2-1,\infty)\geqslant\sqrt2-1$不是$N\in[\sqrt2-1,\infty)\leqslant\sqrt2-1$ 举个例子说明\[2\leqslant a\leqslant4,b\in R\],当$b$取一定值时,令$M={|a-b|}_{\max}$,求$M_{min}$,只有当$b=\dfrac{4+2}2$时$M$才取得最小值,\[|a-b|\leqslant\dfrac{4-2}2=1\] 比如$p=-1$时,$1\leqslant a=\sin\theta-\cos\theta\leqslant\sqrt2$,\[|a-b|\leqslant\dfrac{\sqrt2-1}2\]当$b=\dfrac{(\sin\theta+\cos\theta)_{\max}+(\sin\theta+\cos\theta)_{\min}}2=\dfrac{\sqrt2+1}2$时取得
kuing 68# 2013-4-26 23:01
epsilon 也能当“属于”用……
李斌斌755 69# 2013-4-26 23:02
68# kuing 我是业余选手
李斌斌755 70# 2013-4-26 23:29
本帖最后由 李斌斌755 于 2013-4-27 02:12 编辑 66# yes94 $M$是一个集合,$n$是个数……不知表达明白没有 唠唠叨叨说了半天,手忙脚乱修改一宿……还是难以表述 干脆说个思路,对于一个指定的$p$,$\sin\theta-p\cos\theta$其值是一闭区间,设该区间中间值为$n$,再对于任意的$q,(q\in R)$,此时构成数对$(p,q)$,若想使$|\sin\theta-p\cos\theta-q|_{\max}$取得最小,必须使$q=n$,且其最小值为$\dfrac{(\sin\theta-p\cos\theta)_{max}-(\sin\theta-p\cos\theta)_{\min}}2$ 注意$n$值只受$p$影响,$q$的变化不影响$n$值。
yes94 71# 2013-4-28 17:56
70# 李斌斌755 是不是和kuing的想法是差不多的?
李斌斌755 72# 2013-4-28 19:32
本帖最后由 李斌斌755 于 2013-4-28 22:05 编辑 71# yes94 是一样的,kuing在考虑$p$时是代数,我的是几何,后面$q$的完全一样。
yes94 73# 2013-4-28 21:55
72# 李斌斌755 8层了啊!
thread-1401-1-1.html: 酒喝多了
isea 1# 2013-4-22 01:26
竟然没睡意,奇了怪了
kuing 2# 2013-4-22 01:44
刚看完片,爪机ing路过。。。
isea 3# 2013-4-22 22:07
啥片,说来听听
kuing 4# 2013-4-22 22:47
2同不宜的,就略去了
isea 5# 2013-4-22 23:31
哈哈。 另外,过百了,不错不错
yes94 6# 2013-4-23 12:52
5# isea 酒喝多就多灌点水,
李斌斌755 7# 2013-4-30 02:55
喝多了,没人无聊。
hnsredfox_007 8# 2013-4-30 07:58
不会喝哦
yes94 9# 2013-4-30 15:01
8# hnsredfox_007 不会吧?张老师,滴酒不沾?
yes94 10# 2013-4-30 15:02
5# isea 酒喝多就多灌点水, yes94 发表于 2013-4-23 12:52 谁把我的字体改成了斜体?
kuing 11# 2013-4-30 15:04
10# yes94 我没编辑过 是不是你回复的时候将第一行最后的 /i 和 /b 删掉了
hnsredfox_007 12# 2013-4-30 15:44
9# yes94 大学毕业散伙饭喝过一杯啤酒…… 工作后聚餐只喝茶水……
kuing 13# 2013-4-30 15:47
12# hnsredfox_007 牛笔……这是真的牛笔……
yes94 14# 2013-4-30 17:17
13# kuing 同感!
yes94 15# 2013-4-30 17:18
11# kuing 我想可能是我寻找表情的时候点错了
李斌斌755 16# 2013-4-30 17:39
10# yes94 有那么敏感吗!
thread-1402-1-2.html: [数列] 求助:一个数列压轴题
weihua97 1# 2013-4-22 10:00
本帖最后由 weihua97 于 2013-4-23 22:47 编辑 已知$a_1=\frac13$,$a_{n+1}=a_n+\frac{a_n^2}{n^2}$,求证:(1)$a_n<a_{n+1}<1$;(2)$a_n>\frac1{2}-\frac1{4n}$
Tesla35 2# 2013-4-22 11:02
收录
yayaweha 3# 2013-4-22 12:08
$a_{n+1}=a_n+\frac{a_n^2}{n^2}$先证明$a_n<a_{n+1}$,然后有$$\frac{1}{n^2}>\frac{1}{a_n}-\frac{1}{a_{n+1}}$$ 有$$2>\sum_{i=1}^{n}\frac{1}{n^2}>\frac{1}{a_1}-\frac{1}{a_{n+1}}$$
yayaweha 4# 2013-4-22 12:09
第2问 用数学归纳法试试看吧!
李斌斌755 5# 2013-4-22 13:24
3# yayaweha 怎样证明\[a_{n+1}>a_n\]
yayaweha 6# 2013-4-22 17:33
5# 李斌斌755 $a_{n+1}=a_n+\frac{a_n^2}{n^2}$ 直接做差,是不是
李斌斌755 7# 2013-4-22 17:45
本帖最后由 李斌斌755 于 2013-4-22 18:29 编辑 6# yayaweha $\dfrac{a_n^2}{n^2}\geqslant0\riff a_n\geqslant0$ 还有等于零情况?
kuing 8# 2013-4-22 18:14
6# yayaweha $\dfrac{{a_n}^2}{n^2}\geqslant0\riff{a_n}\geqslant0$ 还有等于零情况? 李斌斌755 发表于 2013-4-22 17:45 显然不会有0啊 PS、代码可优化, \dfrac{{a_n}^2}{n^2}\geqslant0\riff{a_n}\geqslant0 只要写成 \dfrac{a_n^2}{n^2}\geqslant0\riff a_n\geqslant0 注意 \iff 与 a 之间有空格,否则如果连在一起会被当成另一个未知命令而报错。 还有,{a_n}^2 与 a_n^2 的显示效果会有不同,他们分别显示 ${a_n}^2$ 与 $a_n^2$。
yayaweha 9# 2013-4-22 18:18
直接用数学归纳法证明$a_{n+1}>a_n>0$算了
kuing 10# 2013-4-22 18:20
9# yayaweha 何必哩……一句“显然”就可以带过的事……
李斌斌755 11# 2013-4-22 18:26
8# kuing 谢谢。
hongxian 12# 2013-4-23 14:43
第二问数归好象没有成功,不知是不是算错了!
yes94 13# 2013-4-23 17:42
已知$a_1=\dfrac13$,$a_{n+1}=a_n+\dfrac{a_n^2}{n^2}$,求证:(1)$a_n<a_{n+1}<1;(2)a_n>\dfrac12−\dfrac1{4n−1}$ weihua97 发表于 2013-4-22 10:00 本题目的来源?
weihua97 14# 2013-4-23 22:50
12# hongxian 第二问记错了,已经做了修改,谢谢
weihua97 15# 2013-4-23 22:52
13# yes94 2013安徽儒风教育模拟题九
hongxian 16# 2013-4-24 11:26
15# weihua97 百度没有找到,第二问数归还是没有成功!
yes94 17# 2013-4-24 14:03
15# weihua97 百度没有找到,第二问数归还是没有成功! hongxian 发表于 2013-4-24 11:26 据说不是所有数归都好成功的
hongxian 18# 2013-4-24 14:34
17# yes94 那只有等高手出手了!
kuing 19# 2013-4-24 15:05
其实我觉得……当 $n=1$, $2$, $3$ 时直接验证成立,而 $a_4=30760/59049>1/2$,于是由递增知当 $n\geqslant4$ 时 $a_n>1/2$……
hongxian 20# 2013-4-24 15:08
19# kuing 这样看也是水母了!
thread-1402-2-2.html:
kuing 21# 2013-4-24 15:20
20# hongxian 嗯,倒是第一问还有点意思
第一章 22# 2013-4-24 17:18
不知道原答案是怎样的,我用数学归纳法搞不定。 ps,yayawehe同学的第(1)问做得漂亮,像是两边同除以$a_na_{n+1}$
kuing 23# 2013-4-24 17:28
22# 第一章 放缩然后再除……就因为这个做法所以说第一问有点意思
第一章 24# 2013-4-24 17:32
k是不是考虑把这个题修改修改,不要浪费了……
kuing 25# 2013-4-24 17:55
24# 第一章 暂时没什么 idea,命题一向不是我的强行……或者干脆说是弱项……
hnsredfox_007 26# 2013-4-24 20:53
$a_{n+1}=a_n+\dfrac{a_n^2}{n^2}<a_n+\dfrac{a_n}{n^2}=\dfrac{1+n^2}{n^2}a_n\\ \Longrightarrow a_n>\dfrac{n^2}{1+n^2}a_{n+1} \\ \Longrightarrow a_{n+1}=a_n+\dfrac{a_n^2}{n^2}>a_n+\dfrac{a_n}{n^2}\dfrac{n^2}{1+n^2}a_{n+1}=a_n+\dfrac{1}{1+n^2}a_na_{n+1}>a_n+\dfrac{1}{n+n^2}a_na_{n+1}$ $\Longrightarrow \dfrac{1}{a_n}-\dfrac{1}{a_{n+1}}>\dfrac{1}{n^2+n}=\dfrac{1}{n}-\dfrac{1}{n+1}$ $\displaystyle\Longrightarrow \sum_{i=1}^{n-1}\left(\dfrac{1}{a_i}-\dfrac{1}{a_{i+1}}\right)>\sum_{i=1}^{n-1}\left(\dfrac{1}{i}-\dfrac{1}{i+1}\right)$ $\Longrightarrow 3-\dfrac{1}{a_n}>1-\dfrac{1}{n}$ $\Longrightarrow a_n>\dfrac{n}{2n+1}>\dfrac{2n-1}{4n}=\dfrac12-\dfrac{1}{4n}$
hnsredfox_007 27# 2013-4-24 21:09
26# hnsredfox_007 答案是这么做的吗? 没有看楼上那些讨论,原来还有很简单的方法哦,呵呵
yes94 28# 2013-4-24 21:38
27# hnsredfox_007 某几年的类似于此题的模拟题的放缩方法就是这样的,因此你的可能就是标答! 事实上,这儿也有一题:http://bbs.pep.com.cn/forum.php? ... &extra=page%3D1 $\dfrac{\ln2^2}{2^2}+\dfrac{\ln3^2}{3^2}+\cdots+\dfrac{\ln(n+1)^2}{(n+1)^2}>\dfrac n{2(n+1)(n+2)}$ 标答够造的函数似乎很妙,很时髦,但是由单调性,却发现了那些时髦的构造函数法大材小用了。 因此在命题要防止这种现象,也就是标答不要放缩过度,造成该题有极其简单的方法,失去了压轴题标答的巧妙意味。 楼主是高考命题教师?
第一章 29# 2013-4-24 22:00
19# kuing 话说k的计算没出问题吧? ps,这个论坛,每次关掉页面后,就得重新登录。真心蛋疼,人教那边都没出现这个现象。
yes94 30# 2013-4-24 22:02
29# 第一章 我也没看kk的计算,估计有软件算吧,还会错不成? 你可以再登录时点记住密码就行啦
kuing 31# 2013-4-24 22:10
29# 第一章 我用软件列的。 PS. 我这里没出现过这个问题,各个浏览器都没问题。
yayaweha 32# 2013-4-24 22:56
本帖最后由 yayaweha 于 2013-4-24 22:57 编辑 28# yes94 $$\dfrac{\ln2^2}{2^2}+\dfrac{\ln3^2}{3^2}+\cdots+\dfrac{\ln(n+1)^2}{(n+1)^2}>\dfrac n{2(n+1)(n+2)}$$ 这题当年讨论过的 http://kkkkuingggg.5d6d.net/view ... peid%3D2&page=2 http://kkkkuingggg.5d6d.net/view ... %26amp%3Btypeid%3D2
kuing 33# 2013-4-24 23:06
27# hnsredfox_007 某几年的类似于此题的模拟题的放缩方法就是这样的,因此你的可能就是标答! 事实上,这儿也有一题:http://bbs.pep.com.cn/forum.php? ... &extra=page%3D1 $\dfrac{\l ... yes94 发表于 2013-4-24 21:38 看惯了这里的公式……再看那个链接里的……简直不想看……
yes94 34# 2013-4-25 19:54
33# kuing 我也不想看,所以我还编辑成了latex转过来了!
第一章 35# 2013-4-25 22:31
今天翻了2006年在汕头买的一本书《举一反三——高中数学解题经典》,无意间发现一个类似的题目。 看来,楼主的题也是老题。
第一章 36# 2013-4-25 22:38
本帖最后由 第一章 于 2013-4-25 22:40 编辑 26# hnsredfox_007 书后的解答与张老师的解法完全一样。 奇怪的是$a_0$这东西,貌似高考避开的啊,而一本教辅书,又肯定是照帮原题或网络的题。
yes94 37# 2013-4-26 00:02
26# hnsredfox_007 书后的解答与张老师的解法完全一样。 奇怪的是$a_0$这东西,貌似高考避开的啊,而一本教辅书,又肯定是照帮原题或网络的题。 第一章 发表于 2013-4-25 22:38 我当时就想照搬一下那个放缩的解法了,但是 沉迷于灌水,懒得去想了
hnsredfox_007 38# 2013-4-26 09:34
其实,我也是做过这道题,因为辅导竞赛(好像是在《奥赛经典》一书中)……但是那个好像可以数学归纳法……
李斌斌755 39# 2013-4-26 11:59
38# hnsredfox_007 你们都玩奥数
yes94 40# 2013-4-26 13:28
38# hnsredfox_007 你们都玩奥数 李斌斌755 发表于 2013-4-26 11:59 不算奥数吧
thread-1402-3-2.html:
第一章 41# 2013-4-26 21:18
沉迷于灌水?其妙成水军了?
yes94 42# 2013-4-26 21:39
沉迷于灌水?其妙成水军了? 第一章 发表于 2013-4-26 21:18 嘻嘻, 如果有合适的题的话,还是可以解一下题的,
第一章 43# 2013-4-26 22:14
那我出一个给你,呵呵呵,
yes94 44# 2013-4-26 22:20
那我出一个给你,呵呵呵, 第一章 发表于 2013-4-26 22:14 你要知道什么叫合适哈!
第一章 45# 2013-4-26 22:34
⊙﹏⊙b汗,确实未知。 我什么水平,能出一个让你感兴趣的,确实有难度。
yes94 46# 2013-4-26 22:55
⊙﹏⊙b汗,确实未知。 我什么水平,能出一个让你感兴趣的,确实有难度。 第一章 发表于 2013-4-26 22:34 你理解错了哈! 我做的起的才叫合适。 做不起的叫不合适,就是这个标准。 开玩笑的哈。
yes94 47# 2013-4-30 18:28
2011湖北
李斌斌755 48# 2013-4-30 18:53
45# 第一章 你管他呢,出就是了
thread-1403-1-2.html: 来自人教群的一道看上去不太常规的简单题
kuing 1# 2013-4-22 14:30
教师-齐建民  14:03:14 群管-kuing  14:07:16 分a1=0和2讨论就行了 群管-kuing  14:08:23 目测C [中间省略一段乱七八zao] 群管-kuing  14:16:53 若 a1=0,则 W=a2/3^2+...+a100/3^100<=2/3^2+...+2/3^100<1/3; 若 a1=2,则 W>=2/3。 于是C。 为方便以后存档,打成 $\LaTeX$ 先。 题目:已知集合 $M=\{0,2\}$,数列 $\{a_n\}$ 满足 $a_n\in M$($n=1$, $2$, $3$, \ldots),设 \[W=\frac{a_1}3+\frac{a_2}{3^2}+\cdots+\frac{a_{100}}{3^{100}},\] 则 $W$ 一定不属于区间(  ) A. $[0,1)$    B. $(0,1]$    C. $\left[\dfrac13,\dfrac23\right)$    D. $\left(\dfrac13,\dfrac23\right]$ 解:若 $a_1=0$,则由 $a_i\leqslant2$ 知 $W\leqslant2/3^2+2/3^3+\cdots+2/3^{100}<1/3$; 若 $a_1=2$,则由 $a_i\geqslant0$ 显然 $W\geqslant2/3$。 于是 C。
yes94 2# 2013-4-22 17:49
这种题入口高,先吓唬吓唬人。的确很多人都要被吓住。 一旦入了,就发现好简单! 基于此,k版加了个标题:“一道看上去不太常规的简单题”, 意即本题是常规的!并且本题还是简单的! 自然会思索:本题是怎么编写出来的呢? 从另外一个角度看,这是一个三进制的纯小数:$W=(0.a_1a_2a_3\cdots a_n)_3$,其中$a_k=0或2,k=1,2,3,\cdots n$. 例如:$W_1=(0.022002002)_3$,$W_2=(0.20022002002)_3$之类的, 显然,$W_1=(0.022002002)_3<(0.1)_3=\dfrac13$,$W_2=(0.20022002002)_3\geqslant(0.2)_3=\dfrac23$
kuing 3# 2013-4-22 17:54
2# yes94 是的,看出是3进制,自然就会想到那样做了。
李斌斌755 4# 2013-4-22 18:35
3进制,好好想想
tan9p 5# 2013-4-22 19:59
有高等数学背景: Cantor 集
yes94 6# 2013-4-22 20:50
有高等数学背景: Cantor 集 tan9p 发表于 2013-4-22 19:59 更吓人了!
isea 7# 2013-4-22 21:56
本帖最后由 isea 于 2013-4-22 21:58 编辑 晕,你们说得吓人不轻 极端法,做过广东,武汉模拟卷类似这和式不等式放缩太多,故,若如此,不会陌生, 说那么多,其实就一句话:就像 $$主楼那样 \cdots $$ 不过,这个小题包装得真美,收之 这种题入口高,先吓唬吓唬人。的确很多人都要被吓住。 一旦入了,就发现好简单! 基于此,k版加了个标题:“一道看上去不太常规的简单题”, 意即本题是常规的!并且本题还是简单的! 自然会思索:本题是怎么编写 ... yes94 发表于 2013-4-22 17:49 扑通抱住……
yes94 8# 2013-4-23 17:24
晕,你们说得吓人不轻 极端法,做过广东,武汉模拟卷类似这和式不等式放缩太多,故,若如此,不会陌生, 说那么多,其实就一句话:就像 $$主楼那样 \cdots $$ 不过,这个小题包装得真美,收之 扑通抱住…… isea 发表于 2013-4-22 21:56 包装得真美,收之,扑通抱住…… 抱得美人归?
thread-1404-1-2.html: [数列] 有限个正整数
Gauss门徒 1# 2013-4-22 15:40
设$a,b$是正整数,求证:数列\[f(n)=\left(a+\frac{1}{2}\right)^n+\left(b+\frac{1}{2}\right)^n\]只包含有限多个正整数。
kuing 2# 2013-4-22 16:33
设$a,b$是正整数,求证:数列\[f(n)=\left(a+\frac{1}{2}\right)^n+\left(b+\frac{1}{2}\right)^n\]只包含有限多个正整数。 Gauss门徒 发表于 2013-4-22 15:40 为什么我觉得除了 n=1 之外都不会是正整数?
hejoseph 3# 2013-4-22 17:14
为什么我觉得除了 n=1 之外都不会是正整数? kuing 发表于 2013-4-22 16:33 $a=1$,$b=2$,$f(3)=19$
kuing 4# 2013-4-22 17:20
3# hejoseph oh,是这样,我想错了……状态太差了最近……
yes94 5# 2013-4-22 17:55
4# kuing 没事,这是猜想。 我们需要大胆的猜想。 当然,猜想就有成功的时候,同时也蕴含了失败的时候。 如果不猜想,就无所谓失败,当然也不见成功。 想当年著名费马先生留下两个猜想,有一个正确,另一个错误。
李斌斌755 6# 2013-4-22 18:05
5# yes94 这是大学的东东?
yes94 7# 2013-4-22 18:41
6# 李斌斌755 什么大学的东东?
李斌斌755 8# 2013-4-22 18:47
7# yes94 那是中学的
realnumber 9# 2013-4-24 08:39
本帖最后由 realnumber 于 2013-4-24 14:19 编辑 $f(n)=\frac{p^n+q^n}{2^n}$,其中p,q为正奇数,容易得n为偶数时,$2\mid p^n+q^p ,4\nmid p^n+q^n$, 而n为奇数时,$p^n+q^n=(p+q)(p^{n-1}-p^{n-2}q+p^{n-3}q^2-\cdots +q^{n-1})$,其中$(p^{n-1}-p^{n-2}q+p^{n-3}q^2-\cdots +q^{n-1})$为奇数,所以问题化为对于给定的正整数$a,b,p=2a+1,q=2b+1$,仅有有限个n有$2^n\mid{p+q}$, 显然成立. ---已经改正.按楼下提示
kuing 10# 2013-4-24 13:47
9# realnumber 打错了两个字母,看懂了…… 我的反应好慢……
yes94 11# 2013-4-24 13:50
9# realnumber 打错了两个字母,看懂了…… 我的反应好慢…… kuing 发表于 2013-4-24 13:47 我的反应更慢……,还没看懂
thread-1405-1-1.html: 背水一战
isea 1# 2013-4-22 22:05
英文名:The Last Stand 电影 IMDB,在这特别日期就不发分流链接了 施瓦辛格 的新片,主角,还 好意外,感觉
李斌斌755 2# 2013-4-22 23:42
最后的战争翻译为背水一战
isea 3# 2013-4-23 11:52
2# 李斌斌755 也行
yes94 4# 2013-4-23 12:48
哪里什么新片哦? http://v.youku.com/v_show/id_XNDQwMDE0MDQw.html 订阅 qijiuzhiyue (上传者) 请等待我稍后补充视频描述 评论  转发 8个月前上传
isea 5# 2013-4-23 21:41
4# yes94 你看下时长,那最多是个预告片
yes94 6# 2013-4-23 23:25
5# isea 时长1:30,我以为是一个小时30分钟。 还是在这里发帖安全、轻松、自在些,
isea 7# 2013-4-24 00:00

kuing 8# 2013-4-24 00:17
6# yes94 不一定安全……要防hexie
yes94 9# 2013-4-24 21:16
8# kuing 不是乱发信息哈 指的是这里人少,可以随便灌水
isea 10# 2013-4-24 22:48
这就叫一个舒服啊
thread-1406-1-2.html: [函数] 请教:东北三省四市教研协作体2013年等值诊断联合考试(长春二模)理科第21题第(2)问
shidilin 1# 2013-4-22 22:06
画线部分的根据是什么?谢谢!
isea 2# 2013-4-22 22:19
本帖最后由 isea 于 2013-4-22 22:25 编辑 考虑$ k'(x)=e^x-2x-2$的单调性,然后就知$k'(x)$在$(0,2)$上有极小值(且小于零,还有$k'(0)<0$) 再想想$(x$从$0\rightarrow2)$,就明白了……(其实就是脑补$k'(x)$这一段图象) 可能说得比较抽象,补充一下,具体就是 从 极小值点 到 2(及到正无穷) ,由负变正,且单调,所以就有那个结论了
yes94 3# 2013-4-22 22:41
isea似乎从代数角度来解释, 那我用图像角度来解释,并且把$x_0$的值也近似算出:$x_0\approx1.68$
kuing 4# 2013-4-22 22:43
标题真长……居然能这么长
shidilin 5# 2013-4-22 22:45
感谢指点。 k′(x)=ex−2x−2的单调性,,不需要证明吗? 可是好多类似的题,都是要证明的呢。
shidilin 6# 2013-4-22 22:50
标题真长……居然能这么长 kuing 发表于 2013-4-22 22:43 长乎哉,貌似不长也。
shidilin 7# 2013-4-22 22:57
若允许脑补图象,一开始就补……岂不更简单?
isea 8# 2013-4-22 22:58
本帖最后由 isea 于 2013-4-22 23:03 编辑 感谢指点。 k′(x)=ex−2x−2的单调性,,不需要证明吗? 可是好多类似的题,都是要证明的呢。 shidilin 发表于 2013-4-22 22:45 证,要证,再次求个导啊,这很明显啊 那是标答吗?那个答案连提没没提,直接出结论了。 这个脑补图象不是先知图象,是后知图象
yes94 9# 2013-4-22 23:05
7# shidilin 你这个图像 指数函数与二次函数似乎不好想吧,那个曲率不好解释。 指数函数和直线倒是好想些。用来帮助理解$k'(x)$的(划红线部分),实际上还需二次求导证明
shidilin 10# 2013-4-22 23:11
感谢各位的热心回复! 1.网上下载的,可能是标准答案吧。 2.正因为吃不准,故有此一问。 3.我画的图,仅仅是抬个杠,活跃一下气氛而已。 再次感谢!
kuing 11# 2013-4-22 23:13
网上下的东东不能尽信……我就不太信……信自己……
isea 12# 2013-4-22 23:18
上当了,下次不进来了
isea 13# 2013-4-22 23:18
网上下的东东不能尽信……我就不太信……信自己…… kuing 发表于 2013-4-22 23:13 你还得信5d6d标题长……
isea 14# 2013-4-22 23:43
9# yes94 以切线居多啊 呵呵
thread-1407-1-1.html: 过百过百了
isea 1# 2013-4-22 23:32
几位功不可没啊
李斌斌755 2# 2013-4-22 23:38
做蛋糕
kuing 3# 2013-4-22 23:38
感谢大家的支持啊! 除了感谢,不知说什么好了
isea 4# 2013-4-22 23:42
对每人发个2童不宜来
李斌斌755 5# 2013-4-22 23:43
有玩就好。
isea 6# 2013-4-22 23:50
做蛋糕 李斌斌755 发表于 2013-4-22 23:38 做蛋糕是什么?
isea 7# 2013-4-23 00:01
又是新的一天,准备睡觉了,要 宏包更新得七七八八了
kuing 8# 2013-4-23 00:02
我也得抽空换个新版……
李斌斌755 9# 2013-4-23 00:07
6# isea 众人拾柴火焰高的意思
isea 10# 2013-4-23 00:08
9# 李斌斌755 哪儿的方言?
isea 11# 2013-4-23 00:10
我也得抽空换个新版…… kuing 发表于 2013-4-23 00:02 我是极不喜欢那个 LaTeX Warning: You have requested package `savesym', but the package provide Warning只是,比如我对字体的警告通通无视,但这个看着就不舒服,还不知道怎么回事 反复三次,终于——没新的了。 晚安晚安
kuing 12# 2013-4-23 00:14
11# isea 这个提示是啥意思,我好像没遇到过……
李斌斌755 13# 2013-4-23 00:15
10# isea 自己的
isea 14# 2013-4-23 11:52
11# isea 这个提示是啥意思,我好像没遇到过…… kuing 发表于 2013-4-23 00:14 不知
yes94 15# 2013-4-23 12:50
感谢大家的支持啊! 除了感谢,不知说什么好了 kuing 发表于 2013-4-22 23:38 感谢kuing建立了这么好的论坛!感谢kuing为之付出的努力!
李斌斌755 16# 2013-4-26 12:14
昨天又过了。
李斌斌755 17# 2013-4-26 20:44
16# 李斌斌755 又
yes94 18# 2013-4-30 22:12
今天10点过又要过百了!
李斌斌755 19# 2013-4-30 22:22
18# yes94 现在过了。
kuing 20# 2013-4-30 22:32
thread-1408-1-1.html: 你是否注册了 WinEdt (含破解)
isea 1# 2013-4-22 23:57
顺便看看论坛里有多少人在用CTEX套安装,或者LaTeX 偶个人,用WinEdt v7.0 Build 20120329 覆盖原来默认版本了,用得很顺利
kuing 2# 2013-4-22 23:58
是否注册,影响使用么?我一直没注册,没发现有什么不可用的……
isea 3# 2013-4-22 23:59
也是,只是个人“洁癖”,顺便看看个人口味
hnsredfox_007 4# 2013-4-24 21:01
本帖最后由 hnsredfox_007 于 2013-4-24 21:02 编辑 刚安没多久,没有注册,也没有什么提示,不知道有什么影响吗 用CTEX套装
kuing 5# 2013-4-24 23:02
4# hnsredfox_007 照用就是,要是出问题就不用呗,反正不是必须品……
lxzhang 6# 2013-6-2 15:22
注册当然要比不注册的好,但没有注册也没有发现什么使用中的问题。
thread-1409-1-2.html: [不等式] 2013全国高中数学联赛陕西预赛第10题
转化与化归 1# 2013-4-23 12:13
看了几个解答,好像还可以再讨论一下
yes94 2# 2013-4-23 12:28
1# 转化与化归 把解答都贴出来吧?
转化与化归 3# 2013-4-23 12:31
2# yes94 http://blog.sina.com.cn/s/blog_4c113102010185qq.html
yes94 4# 2013-4-23 12:42
3# 转化与化归 谢谢!你先来一个方法呢? 这么多人给出了方法, 我慢慢学习一下先,
转化与化归 5# 2013-4-23 13:05
暂时没有更好的
yes94 6# 2013-4-23 17:49
5# 转化与化归 次数可以降低成1次或者2次.
goft 7# 2013-4-23 19:14
先令其中一个等于0,再令另一个等于0。 (如x2=0,x4=0)
yes94 8# 2013-4-23 19:44
先令其中一个等于0,再令另一个等于0。 (如x2=0,x4=0) goft 发表于 2013-4-23 19:14 为何要令为0? 令为1不可以?
isea 9# 2013-4-23 22:18
原来 楼主就是时不是地人教论坛资源区时不时发材料的sqing*啊
kuing 10# 2013-4-23 22:29
9# isea 不是吧
goft 11# 2013-4-23 23:14
本帖最后由 goft 于 2013-4-23 23:17 编辑 为何要令为0? 令为1不可以? yes94 发表于 2013-4-23 19:44 为1也可以啊,等价的嘛(将一个x2看成主变量即可)
转化与化归 12# 2013-4-24 06:19
9# isea 你弄错啦!
realnumber 13# 2013-4-24 08:51
连接中同时设$x_1$最大,$x_2$最小,觉得有问题,因为表达式不是全对称的. 可以这样,应该有人写过了吧,用调整法, 不妨设$x_1$最大,先固定$x_1,x_2,x_3,x_4$,可得$x_5=0$ 再重新固定$x_1,x_3,x_4$,可得$x_2=0$ 即求$2x_1^3+x_3^3+x_4^3+\abs{x_3-x_4}^3$的最大值,那么取$x_1=1$ 不妨设$x_3\ge x_4$,固定$x_3$,可得$x_4=0或x_3$,进一步得$x_3=1$,OK了. 按这个办法字母再多也一样.
realnumber 14# 2013-4-24 08:56
13楼一直在用如下结论,给定整数b,$f(x)=x^3+(b-x)^3,b\ge x\ge0$,的最大值,为$f(b)或f(0)$,$b=b-x+x$二项式定理展开易得.
yes94 15# 2013-4-24 13:51
14# realnumber 写的太简略了些,不动笔恐怕难看懂
tan9p 16# 2013-4-25 18:01
设差分别为a,b,c,d,e,满足a+b+c+d+e = 0 一个正的,两个正的,三个正的,四个正的,讨论上去就好了。 幂指数大于等于一都可以,但小于一就悲剧了。
yes94 17# 2013-4-25 19:35
16# tan9p 一般出“幂指数大于等于一的”,不会出幂指数“小于一”的
thread-141-1-8.html: [不等式] Easy or hard?
pxchg1200 1# 2011-10-24 23:35
Let $a,b,c \geq 0 $ prove that: \[ \left.\frac{a^{4}}{b(a^{2}+ab+b^{2})}+\frac{b^{4}}{c(b^{2}+bc+c^{2})}+\frac{c^{4}}{a(c^{2}+ca+a^{2})}+\frac{a+b+c}{3}\geq2\cdot\frac{a^{3}+b^{3}+c^{3}}{a^{2}+b^{2}+c^{2}}\right. \]
kuing 2# 2011-10-24 23:44
1# pxchg1200 一看就觉得hard。。。
pxchg1200 3# 2011-10-24 23:49
本帖最后由 pxchg1200 于 2011-10-24 23:50 编辑 + = ..... 话说kuing啊,怎么不等式只有我们两个在玩,其他人连吭都不吭一声啊?
kuing 4# 2011-10-24 23:54
3# pxchg1200 这里还是冷清啊,会员才50现在,你发的题又难 当然,已注册的不等式高手还是有几个的,只是上线还不多
天涯无际 5# 2012-3-1 20:21
利用 http://kkkkuingggg.5d6d.com/viewthread.php?tid=135&highlight= 然后 \[\frac{a^4}{b(a^2+ab+b^2)}+\frac{a}{3}-\frac{2a^2}{3b}=\frac{a(a+b)(a-b)^2}{3b(a^2+ab+b^2)}\] 即可得证.
thread-1410-1-2.html: [组合] 求助一个6点连线段的染色问题
abababa 1# 2013-4-24 07:30
平面上有6个点,每两点间都连线段并涂红色或蓝色,且要确保任意一点至少引出3条红线段。求证无论怎么连,都必定存在3条红线段,使得这3条中的任意2条都没有公共端点。
李斌斌755 2# 2013-4-24 12:08
这种抽象推理题玩不来,坐沙发
yes94 3# 2013-4-24 13:56
2# 李斌斌755 哈哈!那你怎么找到问题的? Ramsey定理: Ramsey(1903~1930)是英国数理逻辑学家,他把抽屉原理加以推广,得出广义抽屉原理,也称为Ramsey定理。    Ramsey定理(狭义)的内容: 任意六个人中要么至少三个人认识,要么至少三个不认识    证明如下:      首先,把这6个人设为A、B、C、D、E、F六个点。由A点可以引出AB、AC、AD、AE、AF五条线段。 设:如果两个人识,则设这两个人组成的线段为红色;如果两个人不认识,则设这两个人组成的线段为蓝色。     由抽屉原则可知:这五条线段中至少有三条是同色的。     不妨设AB、AC、AD为红色。若BC或CD为红色,则结论显然成立。     若BC和CD均为蓝色,则若BD为红色,则一定有三个人相互认识;      若BD为蓝色,则一定有三个人互相不认识。
abababa 4# 2013-4-24 14:08
3# yes94 没看懂。 这个定理我到是知道,我这有些其它问题也用过这定理,这和我现在的问题有什么联系吗?怎么证明主楼呢?
yes94 5# 2013-4-24 14:17
4# abababa 图论很麻烦,也很难做 ,
abababa 6# 2013-4-24 14:20
5# yes94 你附上这个定理,难道不是提示我要用这个定理去做的意思吗?
李斌斌755 7# 2013-4-24 14:48
5# yes94 叫“图论”
realnumber 8# 2013-4-25 11:00
取一红线2端点,不妨记为A,B; 余下四点,因为每一点至少是三红线段的端点,(除掉A,B以外),至少还要和一点用红线相连---(1), 这四点间若出现四条或以上红线,则问题解决(因为会出现四边形对角线或2条对边,那么就符合题意),那么最多有三条. 由(1),那么只需要考虑以下情况,这四点间恰好有三红线,且按如下连接,CD,CE,CF(四点中某一点必须与其它点连接,否则出现四边形对角线或2条对边);而此时D,E,F与AB都相连,那么出现AD,BE,CF符合题意. -----ps,应该有更简洁的表达,与别的方法.
abababa 9# 2013-4-25 11:58
本帖最后由 abababa 于 2013-4-25 12:16 编辑 8# realnumber 谢谢,不过四点间恰有三红线的情况不是坏情况,这四点间可以只有两条红线,其它都和AB两点连红线,虽然这样最后也能证明,但是分类分得太多了,叙述也麻烦。 证明剩下4点间至少有2条红线:假设4点间的6条线只有0或1条是红线,如果0条,则对C点来说只有AC,BC是红线,与C点至少引出3条红线矛盾 如果4点间只有1条红线,则由于1条线只能覆盖2个点,所以必定还有2点在这4点连线中不连红线,不妨设是E,F,则E,F两点只与A,B连红线,又矛盾 楼上证明了4点间有3条红线的情况,但是没证明有2条红线的情况,有2条红线也可以满足题意,比如就是AB,CD,EF这么连,但是要证明。 刚刚想了下,假设四点CDEF,上面证明了还要考虑这四点间只有2红线的情况,如果这2红线没有公共端点,则满足题意,否则这2红线有公共端点,不妨设CD,CE是红线,此时由于4点间每点至少引出1条红线,所以对于F点又必须引出1条红线,这样就回到楼上证明的四点间恰有3条红线的情况。
李斌斌755 10# 2013-4-25 12:11
这样=写论文
abababa 11# 2013-4-25 12:46
整理一下,叙述还是太长,求助更简明的方法 6点连线中至少有1条红线,否则全是蓝线,与一点至少引出3条红线矛盾 不妨设AB是红线,对于其余四点CDEF,将CD,EF视为对边,CF,DE视为对边,CE,DF视为对边,共3组对边 对于CDEF中任意一点,由于每点必须引出3条红线,除与A,B连线外,至少还有1条红线 以下只考察这四点间的连线 若有4条或4条以上的红线,分配到3组对边中,必有一组对边有两红线,这就满足题意了 若只有0条红线,则C点只与A,B连红线,与C能引出3红线矛盾 若只有1条红线,不妨设CD是红线,则E点只与A,B连红线,与E能引出3红线矛盾 若只有2条红线,若2红线在同一组对边,满足题意,若不在同一组对边,则必有公共端点,不妨设CD,CE是红线,此时对于点F来说,若DF或EF是红线,则分别与CE,CD构成对边,满足题意,否则CF是红线 此时四点间有CD,CE,CF三条红线,若存在第4条红线,则已证明必能满足题意,否则这四点间其它连线都是蓝线,而F只与C连红线,但F能引出3红线,所以FA,FB都是红线,同理EA,EB也都是红线,此时有AE,BF,CD满足题意
yes94 12# 2013-4-25 19:45
交给计算机做的话,只需枚举有限次,瞬间得出结果。
thread-1411-1-1.html: [组合] 画个图来来数数多少条路左下到右上
isea 1# 2013-4-24 19:22
姑且归类为组合。 只会第一空,9,第二空,不会,发过向大家学习。 题如下图,一会在TeX/LaTeX/XeLaTeX区给源代码。
kuing 2# 2013-4-24 19:26
早上就在这里被解决了 http://bbs.pep.com.cn/forum.php?mod=viewthread&tid=2737618
isea 3# 2013-4-24 19:29
呵呵,没事,我主要是尝试学着画个五角星玩玩,有结果更好。 去学习一下
isea 4# 2013-4-24 19:32
只用考虑向上走就行了,第一行有n条路到第二行(每一列都可以往上走),第二行有n条路到第三行...以此类推,而且很容易证明这些路径是唯一的 by 战巡 === 想想果然,向上向上 注:第一行应该是指最后一行,仍将类推
kuing 5# 2013-4-24 19:52
4# isea 想的时候由下往上,自然下面才是第一行……
yes94 6# 2013-4-24 21:05
呵呵,没事,我主要是尝试学着画个五角星玩玩,有结果更好。 去学习一下 isea 发表于 2013-4-24 19:29 latex可以画画个五角星?这么高级!
isea 7# 2013-4-24 21:29
本帖最后由 isea 于 2013-4-24 21:34 编辑 latex可以画画个五角星?这么高级! yes94 发表于 2013-4-24 21:05 这个,太小意思了,只是最初级的(tikz 的应用)。 === tikz 手册 的封面就是用 latex 的宏包tikz (也能独立)画的,看看,多么的华丽
yes94 8# 2013-4-24 21:44
7# isea tikz简单原理能够在5,6句说完么?让我们有个极其初步的印象。 一年前就畏惧latex,时至今日硬着头皮学latex,学的半懂不懂的,好家伙,又更新到tikz,什么时候是个尽头啊? 真的是活到老学到老啊!
kuing 9# 2013-4-24 22:20
8# yes94 可以不学。 制图有很多选择,以你自己的要求选择一个合适的方便的就行。
isea 10# 2013-4-24 22:39
本帖最后由 isea 于 2013-4-25 07:42 编辑 7# isea tikz简单原理能够在5,6句说完么?让我们有个极其初步的印象。 一年前就畏惧latex,时至今日硬着头皮学latex,学的半懂不懂的,好家伙,又更新到tikz,什么时候是个尽头啊? 真的是活到老学到老啊! yes94 发表于 2013-4-24 21:44 TikZ就是 $\LaTeX$众多宏包里一个,(不是$\LaTeX$类的东东),用来绘图(矢量)的,代码式的绘图。 举个例子,如一个顶点在原点,一边在x轴,另一顶点在第一象限的等边三角形(图个方便,可以假设边长是1), 用TikZ作图代码写出来(大约意思)就是三点将这三点 (0,0)--(2,0)--$( \dfrac 12,\dfrac{\sqrt 3}2)$ 依次连接起来,这个封闭的图形便是正三角形。 当然坐标系不画出来的。 正是因为只是$\LaTeX$众多宏包一个,(CTEX默认安装都有这个包),作用是作图。 故,只要有且会某作图的软件即可,学与不学,正如楼下,只图方便。 $\LaTeX$制作试卷在大多数学生眼中都一样,没啥区别,哈哈
李斌斌755 11# 2013-4-24 22:46
这道题曾想死我了。
kuing 12# 2013-4-25 00:32
10# isea latex ($latex$) -> \LaTeX ($\LaTeX$) 正如楼下? 不止是学生,绝大部分人眼中都没啥区别
yes94 13# 2013-4-25 19:52
还是有区别的,因为我排版的卷子,学生都说很爽。 只是学生们一瞬间,就适应了那些歪七八招的排版、乱七八招的公式、字体了。 真佩服学生们的适应能力! 其实学生自己的书写更糟糕!相比之下,电脑打印的丑死都还是矮子当中显人高!
thread-1411-1-2.html: [组合] 画个图来来数数多少条路左下到右上
isea 1# 2013-4-24 19:22
姑且归类为组合。 只会第一空,9,第二空,不会,发过向大家学习。 题如下图,一会在TeX/LaTeX/XeLaTeX区给源代码。
kuing 2# 2013-4-24 19:26
早上就在这里被解决了 http://bbs.pep.com.cn/forum.php?mod=viewthread&tid=2737618
isea 3# 2013-4-24 19:29
呵呵,没事,我主要是尝试学着画个五角星玩玩,有结果更好。 去学习一下
isea 4# 2013-4-24 19:32
只用考虑向上走就行了,第一行有n条路到第二行(每一列都可以往上走),第二行有n条路到第三行...以此类推,而且很容易证明这些路径是唯一的 by 战巡 === 想想果然,向上向上 注:第一行应该是指最后一行,仍将类推
kuing 5# 2013-4-24 19:52
4# isea 想的时候由下往上,自然下面才是第一行……
yes94 6# 2013-4-24 21:05
呵呵,没事,我主要是尝试学着画个五角星玩玩,有结果更好。 去学习一下 isea 发表于 2013-4-24 19:29 latex可以画画个五角星?这么高级!
isea 7# 2013-4-24 21:29
本帖最后由 isea 于 2013-4-24 21:34 编辑 latex可以画画个五角星?这么高级! yes94 发表于 2013-4-24 21:05 这个,太小意思了,只是最初级的(tikz 的应用)。 === tikz 手册 的封面就是用 latex 的宏包tikz (也能独立)画的,看看,多么的华丽
yes94 8# 2013-4-24 21:44
7# isea tikz简单原理能够在5,6句说完么?让我们有个极其初步的印象。 一年前就畏惧latex,时至今日硬着头皮学latex,学的半懂不懂的,好家伙,又更新到tikz,什么时候是个尽头啊? 真的是活到老学到老啊!
kuing 9# 2013-4-24 22:20
8# yes94 可以不学。 制图有很多选择,以你自己的要求选择一个合适的方便的就行。
isea 10# 2013-4-24 22:39
本帖最后由 isea 于 2013-4-25 07:42 编辑 7# isea tikz简单原理能够在5,6句说完么?让我们有个极其初步的印象。 一年前就畏惧latex,时至今日硬着头皮学latex,学的半懂不懂的,好家伙,又更新到tikz,什么时候是个尽头啊? 真的是活到老学到老啊! yes94 发表于 2013-4-24 21:44 TikZ就是 $\LaTeX$众多宏包里一个,(不是$\LaTeX$类的东东),用来绘图(矢量)的,代码式的绘图。 举个例子,如一个顶点在原点,一边在x轴,另一顶点在第一象限的等边三角形(图个方便,可以假设边长是1), 用TikZ作图代码写出来(大约意思)就是三点将这三点 (0,0)--(2,0)--$( \dfrac 12,\dfrac{\sqrt 3}2)$ 依次连接起来,这个封闭的图形便是正三角形。 当然坐标系不画出来的。 正是因为只是$\LaTeX$众多宏包一个,(CTEX默认安装都有这个包),作用是作图。 故,只要有且会某作图的软件即可,学与不学,正如楼下,只图方便。 $\LaTeX$制作试卷在大多数学生眼中都一样,没啥区别,哈哈
李斌斌755 11# 2013-4-24 22:46
这道题曾想死我了。
kuing 12# 2013-4-25 00:32
10# isea latex ($latex$) -> \LaTeX ($\LaTeX$) 正如楼下? 不止是学生,绝大部分人眼中都没啥区别
yes94 13# 2013-4-25 19:52
还是有区别的,因为我排版的卷子,学生都说很爽。 只是学生们一瞬间,就适应了那些歪七八招的排版、乱七八招的公式、字体了。 真佩服学生们的适应能力! 其实学生自己的书写更糟糕!相比之下,电脑打印的丑死都还是矮子当中显人高!
thread-1412-1-1.html: 边学tikz边画了个五角星[画个图来来数数多少条路左下到右上 的源代码]
isea 1# 2013-4-24 19:27
本帖最后由 isea 于 2013-4-24 20:57 编辑 效果在初等数学区的此帖里。 源代码 附件 五边形 纯手工画的 最初等的 最基础的代码 ========== 标记一下,要学交点画法了
isea 2# 2013-4-24 19:28
不知怎么改进这个五角星的画法
yes94 3# 2013-4-24 21:21
代码: \documentclass[11pt,a4paper]{ctexart} \usepackage[margin=2cm]{geometry}             %A4纸张,页边距均为2cm \usepackage{amsmath,amssymb,amsthm}    %最常用的数学宏包 \renewcommand{\baselinestretch}{1.5}         %行距 \usepackage{tikz} \usepackage{picinpar} \begin{document} 2013年深圳高三数学理科二模第13题 \bigskip \begin{window}[0,r,{\begin{tikzpicture}   \draw[line width=1pt,step=1] (0,0) grid (3,3) ;   \draw[line width=0.75] (.5,.5) circle(.2);   \draw[line width=0.75] (2.5,2.5)+(90:.22)--+(126:.084)--+(162:.22)--+(198:.084)--+(234:.22)--                                                                    +(270:.084)--+(306:.22)--+(342:.084)--+(378:.22)--+(414:.084)--cycle;   \draw[dashed,line width=1](.8,.5)--(2.5,.5)--(2.5,1.5)--(1.5,1.5)--(1.5,2.5);   \draw[>=stealth][dashed,line width=1,->](1.5,2.5)--(2.25,2.5); \end{tikzpicture}},{}] 题:在$n\times n$的方格中进行跳棋游戏。规定每跳一步只能向左,或向右,或向上,不能向下,且一次连续行走的路径中不能重复经过同一小方格。设$f(n)$表示从从左下角“$\bigcirc$”位置开始,连续跳到右上角“\begin{tikzpicture} [scale=0.75] \draw (0,0)+(90:.22)--+(126:.084)--+(162:.22)--+(198:.084)--+(234:.22)--+(270:.084)--+(306:.22)--+(342:.084)--+(378:.22)--+(414:.084)--cycle; \end{tikzpicture}”位置结束的所有不同路径的条数。如图,给出了$n=3$的一条路径。则$f(3)=$\underline{\hbox to 15mm{}};$f(n)=$\underline{\hbox to 15mm{}}。 \end{window} \end{document}
isea 4# 2013-4-24 21:26
编译是否通过? PS:你都快800帖了啊,厉害
hejoseph 5# 2013-4-24 21:48
那个圈和五角星有特殊字符就完全可以用字符表示了,不必自己画那么麻烦 \documentclass[11pt,a4paper]{ctexart} \usepackage{fontspec} \usepackage[margin=2cm]{geometry}             %A4纸张,页边距均为2cm \usepackage{amsmath,amssymb,amsthm}    %最常用的数学宏包 \renewcommand{\baselinestretch}{1.5}         %行距 \usepackage{tikz} \usepackage{picinpar} \newfontfamily\hei{SimHei} \begin{document} 2013年深圳高三数学理科二模第13题 \bigskip \begin{window}[0,r,{\begin{tikzpicture}   \draw[line width=1pt,step=1] (0,0) grid (3,3) ;   \draw node at (0.5,0.5) {{\hei ○}};   \draw node at (2.5,2.5) {{\hei ☆}};   \draw[->,-latex,dashed,line width=1](.8,.5)--(2.5,.5)--(2.5,1.5)--(1.5,1.5)--(1.5,2.5)--(2.25,2.5); \end{tikzpicture}},{}] 题:在$n\times n$的方格中进行跳棋游戏。规定每跳一步只能向左,或向右,或向上,不能向下,且一次连续行走的路径中不能重复经过同一小方格。设$f(n)$表示从从左下角“{\hei ○}”位置开始,连续跳到右上角“{\hei ☆}”位置结束的所有不同路径的条数。如图,给出了$n=3$的一条路径。则$f(3)=$\underline{\hbox to 15mm{}};$f(n)=$\underline{\hbox to 15mm{}}。 \end{window} \end{document}
kuing 6# 2013-4-25 00:07
同上。 不过即使自己画,当然也可以简化,但是我也没什么兴趣扯下去……只是隐约记得 tikz 里有 Library 里面就有类似的星可以用
李斌斌755 7# 2013-4-25 04:28
你们说的我都听不懂
isea 8# 2013-4-26 00:10
感谢,何版。 学习了。 \draw node at (2.5,2.5) {{\hei ☆}};   % 这个☆是从哪儿来的?我在邓建松$\LaTeX2e$科技排版指南 附录时找了好久也没找到   \draw[->,-latex,dashed]                   % 这个漂亮
kuing 9# 2013-4-26 00:14
直接用输入法。。。
isea 10# 2013-4-26 00:19
你们说的我都听不懂 李斌斌755 发表于 2013-4-25 04:28 说的是 CTEX 套装下的 $\LaTeX$,具体:http://www.ctex.org/HomePage 不防下载 “v2.9.2.164 (203M) 包含 Basic 版 MiKTeX,占用空间小,会根据需要的宏包自动升级。 ” 这个203M的试试看,入门可看人教那边的:http://bbs.pep.com.cn/forum.php? ... &extra=page%3D2 很简单,说白了,安装后,打开,WinEdt,先输入,这些,然后,按F9就能查看到效果了 \documentclass{ctexart} \usepackage{amsmath,amssymb,amsthm} %最常用的数学宏包,如字体,长公式等等 \begin{document} %在这里输入汉字与数学公式(要加$)跟论坛要求一样 (%符号表注释,生成的PDF中没有)示例 打个公式看看$ a+b=c $ \end{document} ________ kuing edit 1 time
yes94 11# 2013-4-26 00:24
编译是否通过? PS:你都快800帖了啊,厉害 isea 发表于 2013-4-24 21:26 灌水的效果啊! 现在做题是凭目测了,目测能做就做,不能做就暂放一边。
kuing 12# 2013-4-26 01:35
10# isea 编辑了一下你的贴子,自己编辑贴子看看我改了什么。
thread-1413-1-2.html: [几何] 一道解析几何
转化与化归 1# 2013-4-24 22:05
本题有没有好的方法?
kuing 2# 2013-4-24 23:04
09江苏13,我当年写过的 http://bbs.pep.com.cn/forum.php? ... 052&pid=4634432 14# 后面16#也有一个常规解法
转化与化归 3# 2013-4-24 23:23
2# kuing 变换的妙!
李斌斌755 4# 2013-4-25 03:55
标答就是16#
转化与化归 5# 2013-4-25 09:21
4# 李斌斌755 这种题哪里有标答,官方一般是不会给出填空题的解题过程的
kuing 6# 2013-4-25 10:05
完全同意楼上。
kuing 7# 2013-4-25 11:03
选择和填空的那些所谓【解析】都是非官方写的,我曾经也帮别人写过……
李斌斌755 8# 2013-4-25 11:06
7# kuing 原来如此
yes94 9# 2013-4-25 12:10
选择和填空的那些所谓【解析】都是非官方写的,我曾经也帮别人写过…… kuing 发表于 2013-4-25 11:03 然后就变成标答了
yes94 10# 2013-4-25 12:19
9# yes94 设$M(a\cos\theta,b\sin\theta)$,则$T(2a\cos\theta,2b\sin\theta)$在直线$A_1B_2$和$B_1F$的交点上,联立两个方程,于是可以解得$e=2\sqrt7-5$
李斌斌755 11# 2013-4-25 12:21
10# yes94 注意,你这是在倒车
yes94 12# 2013-4-25 12:25
11# 李斌斌755
转化与化归 13# 2013-4-25 17:25
11# 李斌斌755 “倒车”是什么意思?
转化与化归 14# 2013-4-25 17:28
10# yes94 方法不错!
李斌斌755 15# 2013-4-25 18:03
14# 转化与化归 先求$T$坐标,再求$M$,9楼则先$M$,再$T$。
yes94 16# 2013-4-25 19:47
很形象的说法,
转化与化归 17# 2013-4-25 21:52
15# 李斌斌755 原来如此啊!
yes94 18# 2013-4-26 13:29
有没有相似椭圆这个概念?
isea 19# 2013-4-26 13:32
有没有相似椭圆这个概念? yes94 发表于 2013-4-26 13:29 汗。 所有离心率相同的椭圆(双曲线,抛物线)均是相似的。
李斌斌755 20# 2013-4-26 13:37
19# isea 还有这一说。
thread-1413-2-2.html:
isea 21# 2013-4-26 13:39
本帖最后由 isea 于 2013-4-26 13:46 编辑 天啊,不是吧,我还以为这是平常不能再平常的了 如果是人教课本中的标准方程,在原点位似,如,椭圆,由对称性,直接证明第一象限内位似即可(还有四个顶点) === 在新课时,我经常在离心相同的曲线中 用相似求a,b方便简洁
yes94 22# 2013-4-26 18:34
21# isea 还搞位似中心?
thread-1414-1-2.html: [不等式] 用CYH技术弄的一个不等式
pxchg1200 1# 2013-4-24 23:35
设$a,b,c>0$ 且$a+b+c=3 $,证明: \[ \frac{a}{\sqrt{b+c^2}}+\frac{b}{\sqrt{c+a^2}}+\frac{c}{\sqrt{a+b^2}}\geq \frac{3}{\sqrt{2}}\] 证明 由CYH技术。 \[ \left(\sum{\frac{a}{\sqrt{b+c^2}}}\right)^{2}\left(\sum{a(2a+b)^3(b+c^2)}\right)\geq \left(2\sum{a^2}+\sum{ab}  \right)^3 \] 所以只要证明 \[ 2\left(2\sum{a^2}+\sum{ab}\right)^3\geq 9\left(\sum{a(2a+b)^3(b+c^2)}\right)\] 开来后就是 \[ 16\sum{a^6}+21\sum{ab^5}-33\sum{a^4c^2}+33abc\sum{a^2b}-48abc\sum{ab^2}-4\sum{a^3b^3}+21\sum{a^4bc}-18a^2b^2c^2\geq 0\] 不会SOS-Schur.谁来继续下。
kuing 2# 2013-4-25 00:10
1# pxchg1200 又是 2a+b……这会比较强么?
reny 3# 2013-4-25 12:30
1# pxchg1200 $16\sum{a^6}+21\sum{ab^5}-33\sum{a^4c^2}+33abc\sum{a^2b}-48abc\sum{ab^2}-4\sum{a^3b^3}+21\sum{a^4bc}-18a^2b^2c^2\geq 0$ 由$Muirhead$定理,应该能判断吧
pxchg1200 4# 2013-4-25 12:34
3# reny 米尔黑德要求Sym的,对cyclic的无效。
kuing 5# 2013-4-25 12:34
3# reny 那个是轮换求和,miurhead 是处理全对称的
yes94 6# 2013-4-25 19:48
不知在哪里看过这种不齐次的不等式
thread-1415-1-2.html: [不等式] 一个关于4的不等式
pxchg1200 1# 2013-4-24 23:38
设$a,b,c>0$ 证明 \[ \frac{ab}{a^2+4b^2}+\frac{bc}{b^2+4c^2}+\frac{ca}{c^2+4a^2}\leq \frac{3}{5} \] 这个还是可以柯西的,如果把4换成5呢? 我暂时没想到如何柯西了 \[ \frac{ab}{a^2+5b^2}+\frac{bc}{b^2+5c^2}+\frac{ca}{c^2+5a^2}\leq \frac{1}{2} \]
kuing 2# 2013-4-24 23:39
4的总觉得在哪里见过……就是翻不到……
pxchg1200 3# 2013-4-25 12:35
2# kuing 就当做没见过好啦。柯西了吧
tan9p 4# 2013-4-25 17:58
推广上瘾了: $$\frac{ab}{a^2+\lambda b^2}+\frac{bc}{b^2+\lambda c^2}+\frac{ca}{c^2+\lambda a^2}\leq \frac{3}{\lambda+1}$$
yes94 5# 2013-4-25 19:38
4# tan9p 怎么证明?例如$\lambda=5$, 这正是楼主的问题。
tan9p 6# 2013-4-25 22:29
再来一个 $x,y,z>0$,$xyz=1$ 求证: $$\frac{x}{x^2+\lambda}+\frac{y}{y^2+\lambda}+\frac{z}{z^2+\lambda}\leq \frac{3}{1+\lambda}$$
yes94 7# 2013-4-26 00:04
6# tan9p 来一个齐次同除以后再分数置换,
huamahu 8# 2013-4-27 10:26
这两个不等式的推广本质上是一样的,不知道怎么证明。
huamahu 9# 2013-4-28 09:20
不能沉呀
李斌斌755 10# 2013-4-28 09:51
坐沙发
yes94 11# 2013-4-28 12:24
4# tan9p 天书已给出$\lambda$是有范围的,不是恒成立的
李斌斌755 12# 2013-4-28 12:56
11# yes94 怎么求范围?
yes94 13# 2013-4-28 13:17
12# 李斌斌755 软件
kuing 14# 2013-4-28 14:04
px 发一下证明吧,不然这贴都被说是水贴了
kuing 15# 2013-4-28 16:39
设$a,b,c>0$ 证明 \[ \frac{ab}{a^2+4b^2}+\frac{bc}{b^2+4c^2}+\frac{ca}{c^2+4a^2}\leq \frac{3}{5} \] 这个还是可以柯西的,如果把4换成5呢? 我暂时没想到如何柯西了 ... pxchg1200 发表于 2013-4-24 23:38 4的柯西我想到的是这样子: 先化为完全对称,待证的是 \[ \frac{ab}{a^2+4b^2}+\frac{bc}{b^2+4c^2}+\frac{ca}{c^2+4a^2}\leqslant \frac35,\] 我们令 $a/b=t$, $b/c=u$, $c/a=v$,则等价于 $t$, $u$, $v>0$ 且 $tuv=1$ 时 \[ \frac t{t^2+4}+\frac u{u^2+4}+\frac v{v^2+4}\leqslant \frac35,\](这就是4#和6#是等价的原因) 再令 $t=xy/z^2$, $u=yz/x^2$, $v=zx/y^2$, $x$, $y$, $z>0$,则又等价于 \[ \frac{xyz^2}{x^2y^2+4z^4}+\frac{x^2yz}{y^2z^2+4x^4}+\frac{xy^2z}{z^2x^2+4y^4}\leqslant \frac35,\] 再变形一下,等价于 \[ \frac{(2z^2-xy)^2}{x^2y^2+4z^4}+\frac{(2x^2-yz)^2}{y^2z^2+4x^4}+\frac{(2y^2-zx)^2}{z^2x^2+4y^4}\geqslant \frac35,\] 由柯西不等式,只要证 \[5\left(2\sum z^2-\sum xy\right)^2\geqslant 3\left(\sum x^2y^2+4\sum z^4\right),\] 这是成立的,但我懒得想妙法了,因为只是四次齐次完全对称,可以直接tao用各种结论……
pxchg1200 16# 2013-4-29 07:35
本帖最后由 pxchg1200 于 2013-4-29 07:38 编辑 15# kuing @kuing. 这个是我的柯西. \[ \frac{ab}{a^2+4b^2}+\frac{bc}{b^2+4c^2}+\frac{ca}{c^2+4a^2}\leq \frac{3}{5}\] 不等式等价于 \[ \sum{\frac{(2b-a)^2}{a^2+4b^2}}\geq \frac{3}{5}\] 然后用Cauchy-Schwarz \[ \sum{\frac{(2b-a)^2}{a^2+4b^2}}=\sum{\frac{(2b^{2}-ab)^2}{a^2b^2+4b^4}}\geq \frac{(2a^2+2b^2+2c^2-ab-bc-ca)^2}{4\sum{b^4}+\sum{a^2b^2}}\] 这里有一点值得注意就是如果配成 \[ \frac{(2b-a)^4}{(2b-a)^2(a^2+4b^2)}\] 再柯西,运算量明显大一些,而若配成 \[ \frac{(a^2-2ab)^4}{a^4+4b^2a^2} \] 柯西,则会放缩过头。估计是$\sum{a^4}$的系数不够高而导致的。 最后只要验证 \[  \frac{(2a^2+2b^2+2c^2-ab-bc-ca)^2}{4\sum{b^4}+\sum{a^2b^2}}\geq \frac{3}{5}\] 这个是4次对称的,所以。。。
kuing 17# 2013-4-29 10:54
16# pxchg1200 原来如此,嗯,那样柯西,不用换元柯过去后也是对称了,最后要证的式子跟我的完全也一样……
huamahu 18# 2013-4-29 14:02
感谢两位!!!
yes94 19# 2013-4-29 23:00
18# huamahu 怪了? 明明看见县长最后发言,怎么进来发现最后发言是huamahu?
kuing 20# 2013-4-29 23:03
20# yes94 我没删,或是自删,又或是被hx了
thread-1415-2-2.html:
kuing 21# 2013-4-29 23:04
21# kuing 楼层居然变乱了,估计是被hx……
yes94 22# 2013-4-30 13:58
22# kuing 原来如此!
zdyzhj 23# 2013-5-6 23:31
i have a beautiful CS proof for both the imequalities!!!!!!!!!!!!!!!!!!!!!!!!!!!!!!
李斌斌755 24# 2013-5-7 02:01
24# zdyzhj
thread-1416-1-1.html: pre
kuing 1# 2013-4-25 00:49
<pre> test </pre>
kuing 2# 2013-4-25 00:56
\documentclass[11pt,a4paper]{ctexart} \usepackage[margin=2cm]{geometry}             %A4纸张,页边距均为2cm \usepackage{amsmath,amssymb,amsthm}    %最常用的数学宏包 \renewcommand{\baselinestretch}{1.5}         %行距 \usepackage{tikz} \usepackage{picinpar} \begin{document} 2013年深圳高三数学理科二模第13题 \bigskip \begin{window}[0,r,{\begin{tikzpicture}   \draw[line width=1pt,step=1] (0,0) grid (3,3) ;   \draw[line width=0.75] (.5,.5) circle(.2);   \draw[line width=0.75] (2.5,2.5)+(90:.22)--+(126:.084)--+(162:.22)--+(198:.084)--+(234:.22)--                                                                    +(270:.084)--+(306:.22)--+(342:.084)--+(378:.22)--+(414:.084)--cycle;   \draw[dashed,line width=1](.8,.5)--(2.5,.5)--(2.5,1.5)--(1.5,1.5)--(1.5,2.5);   \draw[>=stealth][dashed,line width=1,->](1.5,2.5)--(2.25,2.5); \end{tikzpicture}},{}] 题:在$n\times n$的方格中进行跳棋游戏。规定每跳一步只能向左,或向右,或向上,不能向下,且一次连续行走的路径中不能重复经过同一小方格。设$f(n)$表示从从左下角“$\bigcirc$”位置开始,连续跳到右上角“\begin{tikzpicture} [scale=0.75] \draw (0,0)+(90:.22)--+(126:.084)--+(162:.22)--+(198:.084)--+(234:.22)--+(270:.084)--+(306:.22)--+(342:.084)--+(378:.22)--+(414:.084)--cycle; \end{tikzpicture}”位置结束的所有不同路径的条数。如图,给出了$n=3$的一条路径。则$f(3)=$\underline{\hbox to 15mm{}};$f(n)=$\underline{\hbox to 15mm{}}。 \end{window} \end{document} 复制代码 [precode] \documentclass[11pt,a4paper]{ctexart} \usepackage[margin=2cm]{geometry}             %A4纸张,页边距均为2cm \usepackage{amsmath,amssymb,amsthm}    %最常用的数学宏包 \renewcommand{\baselinestretch}{1.5}         %行距 \usepackage{tikz} \usepackage{picinpar} \begin{document} 2013年深圳高三数学理科二模第13题 \bigskip \begin{window}[0,r,{\begin{tikzpicture}   \draw[line width=1pt,step=1] (0,0) grid (3,3) ;   \draw[line width=0.75] (.5,.5) circle(.2);   \draw[line width=0.75] (2.5,2.5)+(90:.22)--+(126:.084)--+(162:.22)--+(198:.084)--+(234:.22)--                                                                    +(270:.084)--+(306:.22)--+(342:.084)--+(378:.22)--+(414:.084)--cycle;   \draw[dashed,line width=1](.8,.5)--(2.5,.5)--(2.5,1.5)--(1.5,1.5)--(1.5,2.5);   \draw[>=stealth][dashed,line width=1,->](1.5,2.5)--(2.25,2.5); \end{tikzpicture}},{}] 题:在$n\times n$的方格中进行跳棋游戏。规定每跳一步只能向左,或向右,或向上,不能向下,且一次连续行走的路径中不能重复经过同一小方格。设$f(n)$表示从从左下角“$\bigcirc$”位置开始,连续跳到右上角“\begin{tikzpicture} [scale=0.75] \draw (0,0)+(90:.22)--+(126:.084)--+(162:.22)--+(198:.084)--+(234:.22)--+(270:.084)--+(306:.22)--+(342:.084)--+(378:.22)--+(414:.084)--cycle; \end{tikzpicture}”位置结束的所有不同路径的条数。如图,给出了$n=3$的一条路径。则$f(3)=$\underline{\hbox to 15mm{}};$f(n)=$\underline{\hbox to 15mm{}}。 \end{window} \end{document} [/precode] $1234$ [precode]$123[4]$[/precode]
kuing 3# 2013-4-25 01:06
做了一个 precode ,本来想用于源码显示并且公式不会起作用(因为目前的 code 代码里有些公式还是会显示)。 结果有个大缺han,就是中括号会影响 precode 的识别 于是 [precode]\$1234\$[/precode 没问题,但是 [precode]\$123[4]\$[/precode 就会有问题
isea 4# 2013-4-26 14:00
[precode] 原来是这样,标题成 显示源代码,我肯定早细看了
kuing 5# 2013-4-26 14:02
当你确定源码里面没有中括号时,就可以用它来显示。 [precode][/precode]
thread-1417-1-1.html: 灌成金牌
李斌斌755 1# 2013-4-25 12:17
最近多灌了些水,没想成金牌
yes94 2# 2013-4-25 12:37
1# 李斌斌755 多灌一点,
isea 3# 2013-4-25 12:45
没啥用,都一样,基本,权限
thread-1418-1-1.html: 刚才看到yes94说……
kuing 1# 2013-4-25 13:26
刚才看到yes94在这里(http://bbs.pep.com.cn/forum.php? ... 876&pid=7833548)说道 yes94 发表于 2013-4-24 17:51 有几个版主都有解题选集了,k版也搞一个噻 …… 怎么说哩,其实很早就想过这事情,而且两年前还曾经做过一部分。可是我这个人就是懒,这么大的工程,眼看我在网上解的题似乎比他们要多很多,而且我这个人要求又高,肯定不能随便整,至少要像做数学空间那样的要求来做,而且只能是我一个人去做,于是做着做着,就搁置了。至于什么时候会继续?我自己也说不准,得看心情,还得看机缘。或者就在今天,又或者永远都不会。
李斌斌755 2# 2013-4-25 13:46
楼上状况$\iff$完美主义+性情
isea 3# 2013-4-25 20:52
他自己都不搞一个……
李斌斌755 4# 2013-4-25 21:12
3# isea 也是
isea 5# 2013-4-25 22:45
发个帖就一个,比如我
kuing 6# 2013-4-25 22:59
5# isea 没懂。。。
isea 7# 2013-4-25 23:10
6# kuing 灌水就好,还管那么,嘿嘿
kuing 8# 2013-4-26 00:29
话说,当年灌水太多,也在一定程度上加大了做解题集的工作量。。
李斌斌755 9# 2013-4-26 02:00
灌水影响工作
hnsredfox_007 10# 2013-4-26 09:37
灌水就好
yes94 11# 2013-4-26 13:36
灌水就好 hnsredfox_007 发表于 2013-4-26 09:37 张老师也开始灌水了
thread-1419-1-2.html: [数列] 求助:数列不等式的证明
weihua97 1# 2013-4-25 15:58
本帖最后由 weihua97 于 2013-4-25 16:01 编辑 $b_n=2^{n+1}$ 证明:$$\frac{n-1}{2}-\frac17<\sum_{k=2}^{n}\frac{b_{n-1}-1}{b_n-1}<\frac{n-1}{2}$$
realnumber 2# 2013-4-25 16:30
$\frac{2^k-1}{2^{k+1}-1}<\frac{1}{2}$,右边成立了. 左边这样\[\frac{1}{2}-\frac{2^k-1}{2^{k+1}-1}=\frac{1}{2^{k+2}-2}\] 1/7的话多保留几项应该没问题.
yes94 3# 2013-4-25 19:41
好烦! 左右差分居然相同,都是$\dfrac12$,此法暂时失效
零定义 4# 2013-4-25 22:57
左边证明如下:
isea 5# 2013-4-25 23:26
好变态的上下界
yes94 6# 2013-4-25 23:36
4# 零定义 原来是2006福建的翻版,沉醉于灌水啊!
thread-142-1-8.html: [不等式] $ab(4a^{2}+b^{2})$.......
pxchg1200 1# 2011-10-24 23:55
Let $a,b,c \geq 0$ with $ab+bc+ca=2 $ prove that: \[ ab(4a^{2}+b^{2})+bc(4b^{2}+c^{2})+ca(4c^{2}+a^{2})+7abc(a+b+c)\geq 16 \]
pxchg1200 2# 2011-10-25 15:20
any idea?
kuing 3# 2011-10-25 17:29
2# pxchg1200 暂时的idea是齐次化后配成schur形式 \[\sum(5ab+c^2)(a-b)(a-c)\geqslant 0,\] 待续……又或者不续……
pxchg1200 4# 2011-10-25 21:33
3# kuing 这个也不好判别吧。。。
kuing 5# 2011-10-25 23:33
4# pxchg1200 是的,所以暂时只能待续,还可能待不了续……
pxchg1200 6# 2011-10-26 13:05
5# kuing Can 看了这题后笑了,并表示Cauchy-Schwarz毫无压力。。。
kuing 7# 2011-10-26 13:38
我也发现了
kuing 8# 2011-10-26 13:43
\begin{align*} \sum ab(4a^2+b^2)+7abc(a+b+c)\geqslant 16 &\iff \sum ab(4a^2+b^2)+7abc(a+b+c)\geqslant 4(ab+bc+ca)^2\\ &\iff 4\sum a^3b+\sum ab^3 \geqslant 4\sum a^2b^2 +abc(a+b+c)\\ &\iff 4\sum\frac{a^2}c+\sum\frac{b^2}c \geqslant 4\sum\frac{ab}c+\sum a\\ &\iff \sum\frac{(2a-b)^2}c\geqslant\sum a, \end{align*}
pxchg1200 9# 2011-10-26 21:32
8# kuing 嗯,kuing果然犀利!解答基本一致!! 
kuing 10# 2011-10-26 21:38
9# pxchg1200 我也是看了你说柯西没压力才这样想,一除abc之后果然发现玄机
pxchg1200 11# 2011-10-26 21:49
本帖最后由 pxchg1200 于 2011-10-26 22:11 编辑 9# pxchg1200 proof:(can) \[ ab(2a-b)^{2}+bc(2b-c)^{2}+ca(2c-a)^{2}+7abc(a+b+c)\geq 16-4(a^{2}b^{2}+b^{2}c^{2}+c^{2}a^{2}) \] \[ ab(2a-b)^{2}+bc(2b-c)^{2}+ca(2c-a)^{2} \geq abc(a+b+c) \] by CS: \[ (ab(2a-b)^{2}+bc(2b-c)^{2}+ca(2c-a)^{2})(c+a+b)\geq abc(a+b+c)^{2} \] Done!
kuing 12# 2011-10-26 21:57
11# pxchg1200 中间一式左边有个括号变成了上标,右边多一个平方
pxchg1200 13# 2011-10-26 22:12
12# kuing 额,输入得太快了,不过Can的柯西还真是犀利!
thread-1420-1-2.html: [数列] 不能表示为3的幂
reny 1# 2013-4-25 19:09
已知$a_{0} = 1, a_{1} = 3 ,且a_{n+2} - 4a_{n+1} + a_{n} = 0,$ 证明当$n>1$时,不存在正整数$k使得a_{n}=3^k.$
yes94 2# 2013-4-25 19:42
求通项是否上当了?
reny 3# 2013-4-25 20:11
仅凭通项肯定不行的吧,还有无理项.
yes94 4# 2013-4-25 20:31
3# reny
reny 5# 2013-4-25 21:11
4# yes94 这个有特征方程易得,关键是……
零定义 6# 2013-4-25 21:29
本帖最后由 零定义 于 2013-4-25 21:33 编辑 与其说是数列题,不如说是数论题... 本人愚钝的思路: 先考虑mod 2,A(n+2)≡A(n)(mod 2),发现以2为周期; 然后考虑mod 3,3|An; 再考虑mod 9,A(n+2)≡A(n+1)-A(n)(mod 9),发现以6为周期; 最后考虑mod 5,A(n+2)≡-[A(n+1)+A(n)](mod 5),发现以3为周期。 综上,便可证得~ 本人愚钝,并且人懒,望哪位大师写一下证明过程吧~
reny 7# 2013-4-25 21:55
6# 零定义 没有看懂,怎么能证明不可以表示为3的幂?
零定义 8# 2013-4-25 22:17
本帖最后由 零定义 于 2013-4-25 22:19 编辑 现在发现mod 2原来是多余的... 1、3|A(n)可得“A(n+2)≡A(n+1)-A(n)(mod 9)”; 2、A(6n)≡0(mod 9),A(6n+1)≡3(mod 9),A(6n+2)≡3(mod 9),A(6n+3)≡0(mod 9),A(6n+4)≡6(mod 9),A(6n+5)≡6(mod 9)说明“若能表示为3的幂必是3n项”; 3、A(3n)≡0(mod 5),A(3n+1)≡3(mod 5),A(3n+2)≡2(mod 5)说明“若能表示为3的幂必不是3n项”. 综上就得命题成立...
kuing 9# 2013-4-26 01:27
mod 9 似乎不是那么简单吧 我用mathematica列了一下 a[0] = 1; a[1] = 3; Do[a[n + 2] = 4 a[n + 1] - a[n], {n, 0, 100}] Table[Mod[a[n], 9], {n, 0, 100}] 结果是 {1, 3, 2, 5, 0, 4, 7, 6, 8, 8, 6, 7, 4, 0, 5, 2, 3, 1, 1, 3, 2, 5, 0, 4, 7, 6, 8, 8, 6, 7, 4, 0, 5, 2, 3, 1, 1, 3, 2, 5, 0, 4, 7, 6, 8, 8, 6, 7, 4, 0, 5, 2, 3, 1, 1, 3, 2, 5, 0, 4, 7, 6, 8, 8, 6, 7, 4, 0, 5, 2, 3, 1, 1, 3, 2, 5, 0, 4, 7, 6, 8, 8, 6, 7, 4, 0, 5, 2, 3, 1, 1, 3, 2, 5, 0, 4, 7, 6, 8, 8, 6} 周期似乎是18
零定义 10# 2013-4-26 01:37
原来俺一直都在用A0=0来算了...原谅俺吧~~~
李斌斌755 11# 2013-4-26 01:51
又是模周期数列
realnumber 12# 2013-4-26 08:33
9楼的kuing,可以得到$9\mid a_n$,则$n=9m-4,m\in Z$. 要不也罗列下mod2,4;8,10;26,28?分别要被3,9,27整除的话,应该与±1同余.如果有n=9m+1或9m+2什么,那么就 矛盾了. 软件好强大啊.....
yes94 13# 2013-4-26 11:23
12# realnumber 掌握高科技就是好啊
kuing 14# 2013-4-26 14:41
9楼的kuing,可以得到$9\mid a_n$,则$n=9m-4,m\in Z$. 要不也罗列下mod2,4;8,10;26,28?分别要被3,9,27整除的话,应该与±1同余.如果有n=9m+1或9m+2什么,那么就 矛盾了. 软件好强大啊..... realnumber 发表于 2013-4-26 08:33 所有数都是奇数,所以 mod 2 都是 1,于是下面从 mod 3 开始列,列到 mod 30 先,每个列 101 个数(注意是从 a0 开始列的)。 输入: nmax = 100; mmax = 30; a[0] = 1; a[1] = 3; Do[a[n + 2] = 4 a[n + 1] - a[n], {n, 0, nmax}] Do[Print["mod ", k, " = ", Table[Mod[a[n], k], {n, 0, nmax}]], {k, 3, mmax}] 输出: mod 3 = {1,0,2,2,0,1,1,0,2,2,0,1,1,0,2,2,0,1,1,0,2,2,0,1,1,0,2,2,0,1,1,0,2,2,0,1,1,0,2,2,0,1,1,0,2,2,0,1,1,0,2,2,0,1,1,0,2,2,0,1,1,0,2,2,0,1,1,0,2,2,0,1,1,0,2,2,0,1,1,0,2,2,0,1,1,0,2,2,0,1,1,0,2,2,0,1,1,0,2,2,0} mod 4 = {1,3,3,1,1,3,3,1,1,3,3,1,1,3,3,1,1,3,3,1,1,3,3,1,1,3,3,1,1,3,3,1,1,3,3,1,1,3,3,1,1,3,3,1,1,3,3,1,1,3,3,1,1,3,3,1,1,3,3,1,1,3,3,1,1,3,3,1,1,3,3,1,1,3,3,1,1,3,3,1,1,3,3,1,1,3,3,1,1,3,3,1,1,3,3,1,1,3,3,1,1} mod 5 = {1,3,1,1,3,1,1,3,1,1,3,1,1,3,1,1,3,1,1,3,1,1,3,1,1,3,1,1,3,1,1,3,1,1,3,1,1,3,1,1,3,1,1,3,1,1,3,1,1,3,1,1,3,1,1,3,1,1,3,1,1,3,1,1,3,1,1,3,1,1,3,1,1,3,1,1,3,1,1,3,1,1,3,1,1,3,1,1,3,1,1,3,1,1,3,1,1,3,1,1,3} mod 6 = {1,3,5,5,3,1,1,3,5,5,3,1,1,3,5,5,3,1,1,3,5,5,3,1,1,3,5,5,3,1,1,3,5,5,3,1,1,3,5,5,3,1,1,3,5,5,3,1,1,3,5,5,3,1,1,3,5,5,3,1,1,3,5,5,3,1,1,3,5,5,3,1,1,3,5,5,3,1,1,3,5,5,3,1,1,3,5,5,3,1,1,3,5,5,3,1,1,3,5,5,3} mod 7 = {1,3,4,6,6,4,3,1,1,3,4,6,6,4,3,1,1,3,4,6,6,4,3,1,1,3,4,6,6,4,3,1,1,3,4,6,6,4,3,1,1,3,4,6,6,4,3,1,1,3,4,6,6,4,3,1,1,3,4,6,6,4,3,1,1,3,4,6,6,4,3,1,1,3,4,6,6,4,3,1,1,3,4,6,6,4,3,1,1,3,4,6,6,4,3,1,1,3,4,6,6} mod 8 = {1,3,3,1,1,3,3,1,1,3,3,1,1,3,3,1,1,3,3,1,1,3,3,1,1,3,3,1,1,3,3,1,1,3,3,1,1,3,3,1,1,3,3,1,1,3,3,1,1,3,3,1,1,3,3,1,1,3,3,1,1,3,3,1,1,3,3,1,1,3,3,1,1,3,3,1,1,3,3,1,1,3,3,1,1,3,3,1,1,3,3,1,1,3,3,1,1,3,3,1,1} mod 9 = {1,3,2,5,0,4,7,6,8,8,6,7,4,0,5,2,3,1,1,3,2,5,0,4,7,6,8,8,6,7,4,0,5,2,3,1,1,3,2,5,0,4,7,6,8,8,6,7,4,0,5,2,3,1,1,3,2,5,0,4,7,6,8,8,6,7,4,0,5,2,3,1,1,3,2,5,0,4,7,6,8,8,6,7,4,0,5,2,3,1,1,3,2,5,0,4,7,6,8,8,6} mod 10 = {1,3,1,1,3,1,1,3,1,1,3,1,1,3,1,1,3,1,1,3,1,1,3,1,1,3,1,1,3,1,1,3,1,1,3,1,1,3,1,1,3,1,1,3,1,1,3,1,1,3,1,1,3,1,1,3,1,1,3,1,1,3,1,1,3,1,1,3,1,1,3,1,1,3,1,1,3,1,1,3,1,1,3,1,1,3,1,1,3,1,1,3,1,1,3,1,1,3,1,1,3} mod 11 = {1,3,0,8,10,10,8,0,3,1,1,3,0,8,10,10,8,0,3,1,1,3,0,8,10,10,8,0,3,1,1,3,0,8,10,10,8,0,3,1,1,3,0,8,10,10,8,0,3,1,1,3,0,8,10,10,8,0,3,1,1,3,0,8,10,10,8,0,3,1,1,3,0,8,10,10,8,0,3,1,1,3,0,8,10,10,8,0,3,1,1,3,0,8,10,10,8,0,3,1,1} mod 12 = {1,3,11,5,9,7,7,9,5,11,3,1,1,3,11,5,9,7,7,9,5,11,3,1,1,3,11,5,9,7,7,9,5,11,3,1,1,3,11,5,9,7,7,9,5,11,3,1,1,3,11,5,9,7,7,9,5,11,3,1,1,3,11,5,9,7,7,9,5,11,3,1,1,3,11,5,9,7,7,9,5,11,3,1,1,3,11,5,9,7,7,9,5,11,3,1,1,3,11,5,9} mod 13 = {1,3,11,2,10,12,12,10,2,11,3,1,1,3,11,2,10,12,12,10,2,11,3,1,1,3,11,2,10,12,12,10,2,11,3,1,1,3,11,2,10,12,12,10,2,11,3,1,1,3,11,2,10,12,12,10,2,11,3,1,1,3,11,2,10,12,12,10,2,11,3,1,1,3,11,2,10,12,12,10,2,11,3,1,1,3,11,2,10,12,12,10,2,11,3,1,1,3,11,2,10} mod 14 = {1,3,11,13,13,11,3,1,1,3,11,13,13,11,3,1,1,3,11,13,13,11,3,1,1,3,11,13,13,11,3,1,1,3,11,13,13,11,3,1,1,3,11,13,13,11,3,1,1,3,11,13,13,11,3,1,1,3,11,13,13,11,3,1,1,3,11,13,13,11,3,1,1,3,11,13,13,11,3,1,1,3,11,13,13,11,3,1,1,3,11,13,13,11,3,1,1,3,11,13,13} mod 15 = {1,3,11,11,3,1,1,3,11,11,3,1,1,3,11,11,3,1,1,3,11,11,3,1,1,3,11,11,3,1,1,3,11,11,3,1,1,3,11,11,3,1,1,3,11,11,3,1,1,3,11,11,3,1,1,3,11,11,3,1,1,3,11,11,3,1,1,3,11,11,3,1,1,3,11,11,3,1,1,3,11,11,3,1,1,3,11,11,3,1,1,3,11,11,3,1,1,3,11,11,3} mod 16 = {1,3,11,9,9,11,3,1,1,3,11,9,9,11,3,1,1,3,11,9,9,11,3,1,1,3,11,9,9,11,3,1,1,3,11,9,9,11,3,1,1,3,11,9,9,11,3,1,1,3,11,9,9,11,3,1,1,3,11,9,9,11,3,1,1,3,11,9,9,11,3,1,1,3,11,9,9,11,3,1,1,3,11,9,9,11,3,1,1,3,11,9,9,11,3,1,1,3,11,9,9} mod 17 = {1,3,11,7,0,10,6,14,16,16,14,6,10,0,7,11,3,1,1,3,11,7,0,10,6,14,16,16,14,6,10,0,7,11,3,1,1,3,11,7,0,10,6,14,16,16,14,6,10,0,7,11,3,1,1,3,11,7,0,10,6,14,16,16,14,6,10,0,7,11,3,1,1,3,11,7,0,10,6,14,16,16,14,6,10,0,7,11,3,1,1,3,11,7,0,10,6,14,16,16,14} mod 18 = {1,3,11,5,9,13,7,15,17,17,15,7,13,9,5,11,3,1,1,3,11,5,9,13,7,15,17,17,15,7,13,9,5,11,3,1,1,3,11,5,9,13,7,15,17,17,15,7,13,9,5,11,3,1,1,3,11,5,9,13,7,15,17,17,15,7,13,9,5,11,3,1,1,3,11,5,9,13,7,15,17,17,15,7,13,9,5,11,3,1,1,3,11,5,9,13,7,15,17,17,15} mod 19 = {1,3,11,3,1,1,3,11,3,1,1,3,11,3,1,1,3,11,3,1,1,3,11,3,1,1,3,11,3,1,1,3,11,3,1,1,3,11,3,1,1,3,11,3,1,1,3,11,3,1,1,3,11,3,1,1,3,11,3,1,1,3,11,3,1,1,3,11,3,1,1,3,11,3,1,1,3,11,3,1,1,3,11,3,1,1,3,11,3,1,1,3,11,3,1,1,3,11,3,1,1} mod 20 = {1,3,11,1,13,11,11,13,1,11,3,1,1,3,11,1,13,11,11,13,1,11,3,1,1,3,11,1,13,11,11,13,1,11,3,1,1,3,11,1,13,11,11,13,1,11,3,1,1,3,11,1,13,11,11,13,1,11,3,1,1,3,11,1,13,11,11,13,1,11,3,1,1,3,11,1,13,11,11,13,1,11,3,1,1,3,11,1,13,11,11,13,1,11,3,1,1,3,11,1,13} mod 21 = {1,3,11,20,6,4,10,15,8,17,18,13,13,18,17,8,15,10,4,6,20,11,3,1,1,3,11,20,6,4,10,15,8,17,18,13,13,18,17,8,15,10,4,6,20,11,3,1,1,3,11,20,6,4,10,15,8,17,18,13,13,18,17,8,15,10,4,6,20,11,3,1,1,3,11,20,6,4,10,15,8,17,18,13,13,18,17,8,15,10,4,6,20,11,3,1,1,3,11,20,6} mod 22 = {1,3,11,19,21,21,19,11,3,1,1,3,11,19,21,21,19,11,3,1,1,3,11,19,21,21,19,11,3,1,1,3,11,19,21,21,19,11,3,1,1,3,11,19,21,21,19,11,3,1,1,3,11,19,21,21,19,11,3,1,1,3,11,19,21,21,19,11,3,1,1,3,11,19,21,21,19,11,3,1,1,3,11,19,21,21,19,11,3,1,1,3,11,19,21,21,19,11,3,1,1} mod 23 = {1,3,11,18,15,19,15,18,11,3,1,1,3,11,18,15,19,15,18,11,3,1,1,3,11,18,15,19,15,18,11,3,1,1,3,11,18,15,19,15,18,11,3,1,1,3,11,18,15,19,15,18,11,3,1,1,3,11,18,15,19,15,18,11,3,1,1,3,11,18,15,19,15,18,11,3,1,1,3,11,18,15,19,15,18,11,3,1,1,3,11,18,15,19,15,18,11,3,1,1,3} mod 24 = {1,3,11,17,9,19,19,9,17,11,3,1,1,3,11,17,9,19,19,9,17,11,3,1,1,3,11,17,9,19,19,9,17,11,3,1,1,3,11,17,9,19,19,9,17,11,3,1,1,3,11,17,9,19,19,9,17,11,3,1,1,3,11,17,9,19,19,9,17,11,3,1,1,3,11,17,9,19,19,9,17,11,3,1,1,3,11,17,9,19,19,9,17,11,3,1,1,3,11,17,9} mod 25 = {1,3,11,16,3,21,6,3,6,21,3,16,11,3,1,1,3,11,16,3,21,6,3,6,21,3,16,11,3,1,1,3,11,16,3,21,6,3,6,21,3,16,11,3,1,1,3,11,16,3,21,6,3,6,21,3,16,11,3,1,1,3,11,16,3,21,6,3,6,21,3,16,11,3,1,1,3,11,16,3,21,6,3,6,21,3,16,11,3,1,1,3,11,16,3,21,6,3,6,21,3} mod 26 = {1,3,11,15,23,25,25,23,15,11,3,1,1,3,11,15,23,25,25,23,15,11,3,1,1,3,11,15,23,25,25,23,15,11,3,1,1,3,11,15,23,25,25,23,15,11,3,1,1,3,11,15,23,25,25,23,15,11,3,1,1,3,11,15,23,25,25,23,15,11,3,1,1,3,11,15,23,25,25,23,15,11,3,1,1,3,11,15,23,25,25,23,15,11,3,1,1,3,11,15,23} mod 27 = {1,3,11,14,18,4,25,15,8,17,6,7,22,0,5,20,21,10,19,12,2,23,9,13,16,24,26,26,24,16,13,9,23,2,12,19,10,21,20,5,0,22,7,6,17,8,15,25,4,18,14,11,3,1,1,3,11,14,18,4,25,15,8,17,6,7,22,0,5,20,21,10,19,12,2,23,9,13,16,24,26,26,24,16,13,9,23,2,12,19,10,21,20,5,0,22,7,6,17,8,15} mod 28 = {1,3,11,13,13,11,3,1,1,3,11,13,13,11,3,1,1,3,11,13,13,11,3,1,1,3,11,13,13,11,3,1,1,3,11,13,13,11,3,1,1,3,11,13,13,11,3,1,1,3,11,13,13,11,3,1,1,3,11,13,13,11,3,1,1,3,11,13,13,11,3,1,1,3,11,13,13,11,3,1,1,3,11,13,13,11,3,1,1,3,11,13,13,11,3,1,1,3,11,13,13} mod 29 = {1,3,11,12,8,20,14,7,14,20,8,12,11,3,1,1,3,11,12,8,20,14,7,14,20,8,12,11,3,1,1,3,11,12,8,20,14,7,14,20,8,12,11,3,1,1,3,11,12,8,20,14,7,14,20,8,12,11,3,1,1,3,11,12,8,20,14,7,14,20,8,12,11,3,1,1,3,11,12,8,20,14,7,14,20,8,12,11,3,1,1,3,11,12,8,20,14,7,14,20,8} mod 30 = {1,3,11,11,3,1,1,3,11,11,3,1,1,3,11,11,3,1,1,3,11,11,3,1,1,3,11,11,3,1,1,3,11,11,3,1,1,3,11,11,3,1,1,3,11,11,3,1,1,3,11,11,3,1,1,3,11,11,3,1,1,3,11,11,3,1,1,3,11,11,3,1,1,3,11,11,3,1,1,3,11,11,3,1,1,3,11,11,3,1,1,3,11,11,3,1,1,3,11,11,3}
realnumber 15# 2013-4-26 14:56
本帖最后由 realnumber 于 2013-4-26 15:24 编辑 mod 9 mod 26就可以得出矛盾 数列中项$a_n>27$时,若可以写成$a_n=3^k,k\in Z$,那么有$a_n\equiv0 \mod9,a_n\equiv1 \mod26$ 由kuing给的数据可得$n=9m+4,m\in Z,且n=12k或12k+11$,显然$n=12k=9m+4$与$n=9m+4=12k+11$无解,(后者为$9m-12k=7$) ----错了
realnumber 16# 2013-4-26 15:03
本帖最后由 realnumber 于 2013-4-26 15:25 编辑 $\mod9,\mod {10}$,2个就可以了,更简单,$n=9m+4=3k或3k+2,m,k\in Z$----也错了
kuing 17# 2013-4-26 15:16
mod 9 mod 26就可以得出矛盾 数列中项$a_n>27$时,若可以写成$a_n=3^k,k\in Z$,那么有$a_n\equiv0 \mod9,a_n\equiv1 \mod26$ 由kuing给的数据可得$n=9m+4,m\in Z,且n=12k或12k+11$,显然$n=12k=9m+4$与$n=9m+4=12k+ ... realnumber 发表于 2013-4-26 14:56 为什么 $a_n\equiv1 \mod26$? PS、a_n\equiv1 \pmod{26} 显示 $a_n\equiv1 \pmod{26}$
realnumber 18# 2013-4-26 15:20
本帖最后由 realnumber 于 2013-4-26 15:21 编辑 17# kuing 啊,我错了,应该是1,3,9
零定义 19# 2013-4-26 16:12
由kk的列表可得,mod 9和mod 17导致矛盾...哎~不同的首项,观察难度就大了那么的多...
kuing 20# 2013-4-26 16:18
19# 零定义 怎么推的?
thread-1420-2-2.html:
零定义 21# 2013-4-26 16:20
本帖最后由 零定义 于 2013-4-26 16:25 编辑 看你列的模表啊,能被9整除的项,恰好也能被17整除~应该我没数错吧,你数数看... mod 9 = {1,3,2,5,0,4,7,6,8,8,6,7,4,0,5,2,3,1,1,3,2,5,0,4,7,6,8,8,6,7,4,0,5,2,3,1,1,3,2,5,0,4,7,6,8,8,6,7,4,0,5,2,3,1,1,3,2,5,0,4,7,6,8,8,6,7,4,0,5,2,3,1,1,3,2,5,0,4,7,6,8,8,6,7,4,0,5,2,3,1,1,3,2,5,0,4,7,6,8,8,6} mod 17 = {1,3,11,7,0,10,6,14,16,16,14,6,10,0,7,11,3,1,1,3,11,7,0,10,6,14,16,16,14,6,10,0,7,11,3,1,1,3,11,7,0,10,6,14,16,16,14,6,10,0,7,11,3,1,1,3,11,7,0,10,6,14,16,16,14,6,10,0,7,11,3,1,1,3,11,7,0,10,6,14,16,16,14,6,10,0,7,11,3,1,1,3,11,7,0,10,6,14,16,16,14}
kuing 22# 2013-4-26 16:24
看你列的模表啊,能被9整除的项,恰好也能被17整除~应该我没数错吧,你数数看... mod 9 = {1,3,2,5,0,4,7,6,8,8,6,7,4,0,5,2,3,1,1,3,2,5,0,4,7,6,8,8,6,7,4,0,5,2,3,1,1,3,2,5,0,4,7,6,8,8,6,7,4,0,5 ... 零定义 发表于 2013-4-26 16:20 牛笔!还真是……
零定义 23# 2013-4-26 16:27
本帖最后由 零定义 于 2013-4-26 16:28 编辑 还是kk兄牛逼,昨晚给你指出错误后,我找了很久都找不到与9同时整除的...
kuing 24# 2013-4-26 16:30
23# 零定义 是软件牛比,我都没干什么……
零定义 25# 2013-4-26 16:37
本帖最后由 零定义 于 2013-4-26 16:44 编辑 一样的牛逼!这证法坑爹啊~有木有更好的方法呢...感觉这种递推关系的模表有点规律?
kuing 26# 2013-4-26 16:59
既然如此,就让我把这个坑爹的证法写出来 由 $a_{n+2}=4a_{n+1}-a_n$,我们逐步递推下去,可以得到 \[a_{n+9} = 40545 a_{n+1} - 10864 a_n = 265\times9\times17\cdot a_{n+1} - (71\times9\times17+1)\cdot a_n,\] 所以 \begin{align*} a_{n+9} & \equiv -a_n \pmod9,\\ a_{n+9} & \equiv -a_n \pmod{17}, \end{align*} 由此可见,$9\mid a_{n+9}$ 当且仅当 $9\mid a_n$;$17\mid a_{n+9}$ 当且仅当 $17\mid a_n$。 我们分别列出 $a_0$ 到 $a_8$ 除以 $9$ 和除以 $17$ 所得的余数,为 1,3,2,5,0,4,7,6,8 和 1,3,11,7,0,10,6,14,16,正好被整除的位置相同,故此 $9\mid a_n$ 当且仅当 $17\mid a_n$。 易知当 $n>1$ 时 $a_n>9$,所以若 $n>1$ 时 $a_n=3^k$,则必定 $9\mid a_n$,于是又有 $ 17\mid a_n$,矛盾。
kuing 27# 2013-4-26 17:03
好像打反了,a|b 是 a 整除 b 还是 a 被 b 整除?
零定义 28# 2013-4-26 17:05
26# kuing 类似整除符号造反了?
kuing 29# 2013-4-26 17:08
28# 零定义 改了,整除都能打反的,你就知道我数论有多弱……
零定义 30# 2013-4-26 17:22
29# kuing 谦虚了,至少你都给你证明出来了...
yes94 31# 2013-4-26 18:04
楼上两位都是牛笔!
thread-1421-1-1.html: 通关将
李斌斌755 1# 2013-4-25 21:11
看yes的回帖,就想起玩游戏的通关,又想起关羽过五关,斩六将
isea 2# 2013-4-25 22:45
三国杀 杀杀杀
isea 3# 2013-4-25 23:19
yes94 一定是通关通不了了,所以,不来灌水
李斌斌755 4# 2013-4-26 01:58
3# isea
hnsredfox_007 5# 2013-4-26 09:39
三国杀 怎么玩 不会哦 灌水就好
李斌斌755 6# 2013-4-26 11:54
5# hnsredfox_007 家人天天玩
yes94 7# 2013-4-26 13:39
yes94 一定是通关通不了了,所以,不来灌水 isea 发表于 2013-4-25 23:19 是通关通不了了,现在没耐心做题了, 随心所欲的目测,能做则做,不能做就灌水!
李斌斌755 8# 2013-4-26 13:46
7# yes94 四川方言“耍得开心噻”
yes94 9# 2013-4-26 13:49
7# yes94 四川方言“耍得开心噻” 李斌斌755 发表于 2013-4-26 13:46 说的太好啦! 就是要耍的开心! 牛笔!
thread-1422-1-2.html: [几何] 又一道解析几何
yayaweha 1# 2013-4-25 22:19
2013广州二模解析几何,问下第二问的几何证法
isea 2# 2013-4-25 23:35
说怎么这么眼熟,不过,当时也只是闪了一下,角分线定理也好,面积也好,似乎还真不如直接斜率之和为零(到角公式推广),多方便。 当然,现在也并不肯定此题无平几法,也许,类似的有心曲线也有类似的性质,搞不好,来个大统一 楼下继续
isea 3# 2013-4-25 23:45
本帖最后由 isea 于 2013-4-25 23:47 编辑 果然,在圆内为真,而且是下是极显然的,反演变换具有保角性,这样一来,椭圆是成立的。(反演忘得七七八八的,现在只是猜想)
isea 4# 2013-4-26 00:05
几何画板验证,椭圆成立 晚了,明要早起,睡,先 (虚线为验证角相等)
李斌斌755 5# 2013-4-26 02:17
谁有几何证法,发一个。
yes94 6# 2013-4-26 11:27
2# isea 本来想灌水的,但是你写了 楼下继续 才等到现在
李斌斌755 7# 2013-4-26 12:05
2# isea 也认为这题解析比几何简单(若有几何法),不知对不对。
yes94 8# 2013-4-26 12:46
7# 李斌斌755 解析思路是很简单的,尤其是抛物线的斜率非常好表达(点差法)
kuing 9# 2013-4-27 11:55
跟这两个像: http://bbs.pep.com.cn/forum.php?mod=viewthread&tid=669821 http://bbs.pep.com.cn/forum.php?mod=viewthread&tid=2590623 但是抛物弄不成圆……
thread-1423-1-1.html: pdf 2 eps
isea 1# 2013-4-25 22:44
仅纯图 命令行 gswin32c.exe -q -dNOPAUSE -dBATCH -dNOCACHE -sDEVICE=epswrite -sOutputFile=xxx.eps xxx.pdf
李斌斌755 2# 2013-4-25 22:49

isea 3# 2013-4-25 23:08
临时要将pdf纯矢量图转化为图片 那是命令行下的操作 无视吧,灌水就好 看看支持eps格式的图片不
李斌斌755 4# 2013-4-25 23:13
3# isea
kuing 5# 2013-4-25 23:16
3# isea 上次说过啦,除初级的会员外都不限附件格式
isea 6# 2013-4-25 23:18
5# kuing 这个还真是不限制格式,这个我真的懂。 我是想试试DZ是否支持eps格式的图片显示出来。
kuing 7# 2013-4-25 23:24
6# isea 噢…原来如此。 那显然不会显示,印象中wmf之类的都不显示,何况。。。
thread-1424-1-1.html: 这个$\pi$出自哪儿(添加本地图片)
isea 1# 2013-4-25 23:21
本帖最后由 isea 于 2013-4-26 13:02 编辑 kuing的空间签名图
kuing 2# 2013-4-25 23:30
我也不知道,也不知道证明,很早以前看到的,觉得够雄壮,就拿来做签名。。。
isea 3# 2013-4-25 23:37
雄壮?就是霸气!
kuing 4# 2013-4-25 23:45
我词穷。。。
isea 5# 2013-4-25 23:48
谁要是能证明一下,那就牛了
yes94 6# 2013-4-26 00:29
还有那个南飞雁也是用的$\pi$
李斌斌755 7# 2013-4-26 01:56
$\pi,e$是数学中的乾坤!!
kuing 8# 2013-4-26 02:01
7# 李斌斌755 你也能看到楼主发的图吗?
李斌斌755 9# 2013-4-26 09:26
8# kuing 看不到
hnsredfox_007 10# 2013-4-26 09:37
看不到
kuing 11# 2013-4-26 12:51
不知上了QQ的看不看得到
李斌斌755 12# 2013-4-26 13:25
1# isea $e=?$
李斌斌755 13# 2013-4-26 13:26
11# kuing isea已编辑,我看到了
yes94 14# 2013-4-26 13:31
11# kuing 上了qq也看不到 这能看到附件
kuing 15# 2013-4-26 13:39
14# yes94 进我QQ空间、看我日志之后再看能不能……
yes94 16# 2013-4-26 13:48
14# yes94 进我QQ空间、看我日志之后再看能不能…… kuing 发表于 2013-4-26 13:39 刚刚进,还是不能看
kuing 17# 2013-4-26 13:51
看来只有我能直接看到……
isea 18# 2013-4-26 14:04
17# kuing 我没问题,opera
kuing 19# 2013-4-26 14:07
18# isea 清空缓存或许不一定……
hnsredfox_007 20# 2013-4-26 15:36
看看升级不
thread-1425-1-2.html: [函数] 一个函数老题目(2)
转化与化归 1# 2013-4-26 09:17
本帖最后由 转化与化归 于 2013-4-26 19:18 编辑 有没有简洁的思路?
hnsredfox_007 2# 2013-4-26 09:31
1# 转化与化归 个人感觉常规思路就是简洁方法啊! 顺便问一下,你的不简洁方法是什么?
转化与化归 3# 2013-4-26 09:58
2# hnsredfox_007
hnsredfox_007 4# 2013-4-26 10:09
3# 转化与化归 $f'(x)$的单调区间为什么不利用导数求啊
hnsredfox_007 5# 2013-4-26 10:15
3# 转化与化归 利用导数求出$f'(x)$的单调区间为$(0,+\infty)$,以下就考虑$f(x)$在$(0,+\infty)$单调即可啊
yes94 6# 2013-4-26 11:22
丁老师研究问题很深入
kuing 7# 2013-4-26 14:28
1# 注意导数的输入(参考 http://kkkkuingggg.5d6d.net/thread-499-1-1.html);and 主题分类。 3# 排版太难看,不想看…… 对,我就是没什么实际回复……因为这种题实在不感兴趣
yes94 8# 2013-4-26 18:29
7# kuing 看惯了美美的latex字体!
转化与化归 9# 2013-4-26 19:21
7# kuing kuing对分类很敏感啊!已分类。排版是现成的,稍显难看!
thread-1427-1-1.html: 数学空间第12期出了
isea 1# 2013-4-26 13:58
快去围观
kuing 2# 2013-4-26 14:00
前两天我还在想要不要在这里加个“《数学空间》资讯版块”用以宣传……
isea 3# 2013-4-26 14:03
不用加,加了分散人气,随便往哪儿一说,如现在,就成了 不过,好像这期投搞人不多
kuing 4# 2013-4-26 14:05
一直将资讯放在 http://bbs.pep.com.cn/forum.php?mod=forumdisplay&fid=223 ,几乎没人看那个版块。 于是我昨天也决定将置顶的链接汇总复制到了人教高中版。
isea 5# 2013-4-26 14:06
版块太多就有这样的毛病
hnsredfox_007 6# 2013-4-26 15:36
看看升级不
kuing 7# 2013-4-26 17:10
加了分散人气 那个版块根本就没人气……怎么分……
thread-1428-1-1.html: 五佰
isea 1# 2013-4-26 14:59
的确不是伍佰
isea 2# 2013-4-26 15:00
看看升级不升级
kuing 3# 2013-4-26 15:05
是按积分的 具体我也不记得了,当初也是随便设的……反正不要紧
hnsredfox_007 4# 2013-4-26 15:35
看看升级不
yes94 5# 2013-4-26 18:06
4# hnsredfox_007 我也看看还能再升级不?
李斌斌755 6# 2013-4-26 20:47
想升级常来灌水。
hnsredfox_007 7# 2013-4-26 20:54
5# yes94 继续灌……
零定义 8# 2013-4-26 23:14
lulu就升级了
isea 9# 2013-4-27 01:16
这级别真心没用 哈哈 晚安 晚安
hnsredfox_007 10# 2013-4-27 11:35
继续
yes94 11# 2013-4-27 12:42
继续 hnsredfox_007 发表于 2013-4-27 11:35 张老师也来了
thread-1429-1-1.html: 开个新帖吧,关于系统字体的调用 之 XeLaTeX (+ xeCJK + ctex ) 一点体会
isea 1# 2013-4-26 23:52
本帖最后由 isea 于 2013-5-1 21:20 编辑 行文之前,感谢论坛和朋友,特别是kuing及hejoseph,先。 先引用一下,后面具体说明。 其实我还是建议初玩可以用ctexart之类的文档类。 kuing 发表于 2013-4-18 23:48 至现在,我才明白,CTEX 套装,对系统字体的调用,我走了不少弯道。 对最简的这个而已 \documentclass{ctexart}  %使用 ctexart 文档类型排版 begin{document} %从这里输入正文,随便把字可打多点,查看效果 \end{document} 实际上,(这个最简)使用中文已经完全无障碍。从我使用过程,来聊一下,给全新新手一点点新的参考,网上的好多教程说法,一个好多是linux平台,二是,真的好多陈旧了,需要自己去筛选与鉴别。 先说ctex宏包 默认已经给了6种字体,宋体、仿宋、黑体、楷体、隶书、幼圆(在使用XeTEX 时只有前四种)最常见实用的六种; 对应的命令为 \songti 宋体    \heiti 黑体    \fangsong 仿宋   \kaishu 楷书   \lishu 隶书  \youyuan 幼圆。 对字号,已经设置了,小号字体在前面加负号表示,从大到小依次为 \begin{array}{|c|c|c|c|c|c|c|c|} \hline 初号&小初&一号&小一&二号&小二&三号&小三&四号&小四&五号&小五&六号&小六&七号&八号\\ \hline 0& -0& 1 &-1& 2 &-2 &3& -3&4& -4& 5& -5& 6& -6& 7 &8\\ \hline \end{array} 如五号,命令为 \zihao{5}。 然后,你要知道,$\LaTeX$排版效果是全自动的,再人为的加上一些:\\ 是手工换行,\par 是新起一段(或者两次或更多回车) 仅仅这些,然后加入宏包 \usepackage{amsmath,amssymb,amsthm} 就能达到论坛输入的效果了。 实际上更强一些,再了解一下数学模式与中文的混排,老帖一直在纠结LaTeX 数学模式 下的中文(完全解决于21楼),然后,用PDFLaTeX编出这样简短的附件效果(即原老帖3楼的效果 3.pdf (207.88 KB) ),虽然有些瑕疵,且第一次编译有些警告,但基本算是比较成熟了,这便是$\LaTeX$强大排版的功劳。 =======$\LaTeX$老手可以自动绕行,或者无视这些这最最基本东东======== 下面说调用系统字体,也就是回到本文的开始,真正对 ctexart 类型文档的理解,也就是对宏包ctex的学习。(数学常用宏包,要干涉的太少,照打公式即可,不会有大多的障碍,对中学试卷而言,或少急切的应用) 不过,还是从 老帖 此区有个XeLaTeX 听说能直接调用系统字体——说开,相对而言,那时的我比第一帖略“成熟”一些,经过探索(我十分肯定,如果你也新学$\LaTeX$,一定会这样过程,那怕,我这里来说简洁之路),便有了 老帖 来来来 求围观 哈哈 5楼,正式的第一大型试卷,用的 默认 Adobe 宋体(打印效果,真心赞啊!!)。 但大家细看 98.tex 便知文档类用的 article 而非ctexart(其实写的时候用的ctexart,所以有 \newcommand{\heiti}{\CJKfamily{heiti}}  这样,很奇怪的自定义命令)为什么呢?因为xeCJK(且为UTF-8编码这个必须用XeLaTeX编译)与偶有些 ctexart 不和,总是有些字体信息警告,又怕 Adobe 宋体 被冲突了,否则这不是白忙了。 (下面的文字)这是后话(即后来才明白),上面提到,\newcommand{\heiti}{\CJKfamily{heiti}},可这个\heiti 在ctex 是已经存在的命令,无法通过XeLaTeX编译,当时又急着打印,又干脆换成 article ,唉,相安无事,达果效果,其它的先放一边。再表一枝,article,其实也好,至少不与xeJCK 冲突啊。 如果追求完美的话 article 也有些瑕疵,主要有三:一行字比较挤,首行缩进它不是两个中文字(象形文字)的距离,二是字号,这个还真自己再去定义一次。用 article 类型文档里已经成功统一风格,调用系统字体及字号。 那,用ctexart 到底怎么样才能最方便,且没什么字体警告,而通过 xeCJK 调用系统字体 +  XeLaTeX 编译呢?还能直接默认设置,如\zihao,等等 其实非常的简单!在CTEX手册 使用帮助里已经说得非常明白了: 2.1 使用CJK 或xeCJK 这是ctex 宏包的缺省设置。ctex 宏包会自动根据使用的TEX 引擎调用CJK 或者 xeCJK 宏包,你无需再自己调用。 只不过,那时偶看不懂,不知道具体说了什么东西。 那到底什么意思呢? 那就是,ctexart 会自动加载 ctex 宏包,而由上面知道,ctex 会自动调用CJK或者xeCJK!而xeCJK会自动加载fontspec。正是因为如此,此时,你完全不用担心中文下的$\LaTeX$排版,尽管打中文即可,除非你所用的字体不是那六种,且没有定义(或者说指定?指明?)新的字体,不能被$\LaTeX$识别才会出错(当然你一路回车,或者按个s回车,无视这些也成,只是没有你想要的字体而已)。 =======以上闲扯,下面才是标题的回应====ctexart类型文档,一点体会======= 具体如下(以调用系统字体华文彩云为例)(文后的附件smp) \documentclass{ctexart} \setCJKfamilyfont{caiy}{华文彩云}            %含下一行,即把华文彩云简化定义为命令 \cy \newcommand{\cy}{\CJKfamily{caiy}} \begin{document} 注意:UTF-8编码,用XeLaTeX编译,调用系统字体,{\cy 这里示例为华文彩云},或者写全了,{\CJKfamily{caiy}不怕麻烦的话},看两处都是彩云。{\zihao{1} \heiti 把字调大还是黑}体。 对就是这么简单! \end{document} 对,就是这么简单,其实,用了ctex 宏包,要用什么系统字体,自己随便加(导言区),要什么什么字号,自己加,附上源码及效果。 说得比较粗糙,如果有所启发,明白了,不需要此文,那我太高兴了 =======以上方法,下面是补充====xeCJK类型文档,一点体会============= 05.01.2013 增补 \setCJKfamilyfont{caiy}{华文彩云}            %含下一行,即把华文彩云简化定义为命令 \cy \newcommand{\cy}{\CJKfamily{caiy}} 新版 xeCJK (默认安装CTEX 目前非新版,需升级宏包)简化命令(方便多了) \newCJKfontfamily[caiy]\cy{华文彩云} 其中,[caiy] 这个为可省选项,即,下面命令也调用华文彩云 \newCJKfontfamily\cy{华文彩云} 这样一来,就和与偶看到hejoseph用的调用英文字体方法完全一致了 \newfontfamily\CMU{CMU Serif} 查看系统字体列表:参阅老帖:http://kkkkuingggg.5d6d.net/viewthread.php?tid=411
isea 2# 2013-4-27 01:15
OK,新帖开完,睡觉去了
李斌斌755 3# 2013-4-27 01:22
2# isea 跟着慢慢学。
hnsredfox_007 4# 2013-4-27 08:03
慢慢学习中……慢慢学习中……慢慢学习中……
yes94 5# 2013-4-27 13:00
4# hnsredfox_007 还没学, 在word里的?还是论坛上用?
李斌斌755 6# 2013-4-27 21:12
学习,学习,再学习。
yes94 7# 2013-4-28 00:32
悲痛! aurora!要收费啦! 我收到他的一个对话框,可惜没截屏,全是英语的,大约是收费吧 现在出现这样了!公式自动出现红叉叉!
isea 8# 2013-4-28 01:04
悲痛! aurora!要收费啦! 我收到他的一个对话框,可惜没截屏,全是英语的,大约是收费吧 现在出现这样了!公式自动出现红叉叉! 1399 yes94 发表于 2013-4-28 00:32 在水区说这个去 另外,类似这种软件其实还很多,不过,一时名字记不起来了
hnsredfox_007 9# 2013-4-28 08:10
7# yes94 没有吧?我的还正常哎 另外在学latex,特别慢 在word中用aurora……
yes94 10# 2013-4-28 19:33
9# hnsredfox_007 我用的频率太高了吧? 结果就要付费了
李斌斌755 11# 2013-4-29 01:39
10# yes94 用多应该奖励啊
isea 12# 2013-5-1 20:49
05.01.2013 增补 \setCJKfamilyfont{caiy}{华文彩云}            %含下一行,即把华文彩云简化定义为命令 \cy \newcommand{\cy}{\CJKfamily{caiy}} 新版 xeCJK (默认安装CTEX 目前非新版,需升级宏包)等价命令(方便多了) \newCJKfontfamily[caiy]\cy{华文彩云} 其中,[caiy] 这个为可省选项,即,下面命令也调用华文彩云 \newCJKfontfamily\cy{华文彩云} 这样一来,就和与偶看到hejoseph用的调用英文字体方法完全一致了 \newfontfamily\CMU{CMU Serif} 查看系统字体列表:参阅老帖:http://kkkkuingggg.5d6d.net/viewthread.php?tid=411
叶剑飞Victor 13# 2013-6-2 20:06
7# yes94 8# isea 9# hnsredfox_007 Aurora试用期30天,30天之内未输入序列号的就一律画红叉。
thread-143-1-8.html: [不等式] 奥数教程上的题
图图 1# 2011-10-25 00:00
本帖最后由 图图 于 2011-10-25 00:04 编辑 设$x_1,x_2,\cdots,x_n$为正实数,$S=x_1+x_2+\cdots+x_n$求证: \[(1+x_1)(1+x_2)\cdots(1+x_n)\le1+S+\frac{S^2}{2!}+\cdots+\frac{S^n}{n!}\]
pxchg1200 2# 2011-10-25 00:01
What is $ S$ ?
kuing 3# 2011-10-25 00:03
2# pxchg1200 别急,楼主打代码还未习惯,紧张打漏了哟
kuing 4# 2011-10-25 00:10
设$x_1,x_2,\cdots,x_n$为正实数,$S=x_1+x_2+\cdots+x_n$求证: \[(1+x_1)(1+x_2)\cdots(1+x_n)\le1+S+\frac{S^2}{2!}+\cdots+\frac{S^n}{n!}\] 图图 发表于 2011-10-25 00:00 貌似左边直接均值然后用e、泰勒展开什么的...
kuing 5# 2011-10-25 01:06
泰勒似乎米用,还是用数归好了。 由均值有 \[(1+x_1)(1+x_2)\cdots(1+x_n)\leqslant\left(1+\frac Sn\right)^n,\] 只要证 \[\left(1+\frac Sn\right)^n\leqslant1+S+\frac{S^2}{2!}+\cdots+\frac{S^n}{n!}.\] 当 $n=1$ 时不等式显然成立,假设当 $n=k$ 时不等式成立,则当 $n=k+1$ 时,令 \[f(S)=1+S+\frac{S^2}{2!}+\cdots+\frac{S^{k+1}}{(k+1)!}-\left(1+\frac S{k+1}\right)^{k+1},\] 则由归纳假设有 \[f'(S)=1+S+\cdots+\frac{S^k}{k!}-\left(1+\frac S{k+1}\right)^k\geqslant\left(1+\frac Sk\right)^k-\left(1+\frac S{k+1}\right)^k>0,\] 而 \[\lim_{S\to0+}f(S)=1-1=0,\] 从而 $f(S)>0$ 成立,即当 $n=k+1$ 时不等式也成立,由数归知得证。
pxchg1200 6# 2011-10-25 13:15
5# kuing 其实无需数学归纳法,直接二项展开,然后稍微放缩下就好了。。
kuing 7# 2011-10-25 15:23
6# pxchg1200 嗯,即要证 \[\frac{C_{n}^{k}}{n^{k}}\leqslant \frac{1}{k!},\] 代公式化简为 \[n(n-1)\cdots (n-k+1)\leqslant n^{k}.\]
图图 8# 2011-10-25 18:17
5# kuing 是不是应该说明一下f(0)=0呢?
kuing 9# 2011-10-25 18:32
8# 图图 我不是说了 $\lim_{S\to0+}f(S)=1-1=0$ 了么
thread-1430-1-2.html: [数论] 又一道递推数列与数论结合题(来自人教群)
kuing 1# 2013-4-27 00:25
昨天那道也是递推数列与数论结合,该贴楼主分类到“数列”,而我还是觉得更主要是数论吧,这次也是,所以我分类到“数论”。 题目:已知 $a_0=a_1=1$, $a_{n+1}=a_na_{n-1}+1$($n\in\mbb N^+$)。 证明:$n\geqslant2$ 时,$a_n$ 不是完全平方数。 注意到完全平方数除以 $4$ 所得的余数只能是 $0$ 或 $1$,而通过列举发现由 $a_2$ 开始除以 $4$ 所得余数一直是 $2$, $3$, $3$ 循环,于是,我们只要证明的确是循环的就可以了,即我们只要证明: 当 $m\in\mbb N^+$ 时恒有 \begin{align*} a_{3m-1}&\equiv 2\pmod 4, \\ a_{3m}&\equiv 3\pmod 4 ,\\ a_{3m+1}&\equiv 3\pmod 4 . \end{align*} 用数归证之,当 $m=1$ 时,直接验证方知成立,假设当 $m=k$ 时成立,则当 $m=k+1$ 时,有 \begin{align*} a_{3(k+1)-1}&=a_{3k+1}a_{3k}+1\equiv 3\times 3+1\equiv 2\pmod 4 ,\\ a_{3(k+1)}&=a_{3(k+1)-1}a_{3k+1}+1\equiv 2\times 3+1\equiv 3\pmod 4 ,\\ a_{3(k+1)+1}&=a_{3(k+1)}a_{3(k+1)-1}+1\equiv 3\times 2+1\equiv 3\pmod 4, \end{align*} 可见当 $m=k+1$ 时也成立,从而得证。
kuing 2# 2013-4-27 00:31
前近这类题都……又是模周期……
第一章 3# 2013-4-27 06:22
跳跃式数学归纳法?
kuing 4# 2013-4-27 11:42
3# 第一章 不算是吧,将连续三个等式看成一个整体,那就是普通的归纳法……
yes94 5# 2013-4-27 12:09
本质上属于跷跷板归纳法(或者叫螺旋归纳法、多命题相互支持) 一般跷跷板指的是两个命题。广义的跷跷板可以多个命题。 但是此时也恰巧具备跳跃归纳法的一些特征。 kk牛笔!
零定义 6# 2013-4-27 13:48
楼上、楼下的都牛笔~
Tesla35 7# 2013-4-27 21:38
这个也得mark
kuing 8# 2013-4-27 21:41
7# Tesla35 还真是只要出现数列就 mark 。。。
yes94 9# 2013-4-27 21:45
7# Tesla35 还真是只要出现数列就 mark 。。。 kuing 发表于 2013-4-27 21:41
thread-1431-1-2.html: [数列] 徐州一摸20题
boysxh 1# 2013-4-27 14:47
在题中的条件中令k=2n 再和第3问的的条件联立解方程行不行
李斌斌755 2# 2013-4-27 18:54
先做(1)问设$c_n=a_n+b_n$ 1)\[a+b\geqslant0\]\[c_1=a_1+b_1=a+b\geqslant0\\c_{k+1=}a_{k+1}+b_{k+1}=\dfrac{a_k}2-\dfrac{b_k}4+\dfrac{3b_k}4=\dfrac{a_k+b_k}2=\dfrac{c_k}2\\\riff c_n=\dfrac{a+b}{2^{n-1}}\] 2)\[a+b<0\]\[c_1=a_1+b_1=a+b<0\\c_{k+1=}a_{k+1}+b_{k+1}=-\dfrac{a_k}4+\dfrac{b_k}2+\dfrac{3a_k}4=\dfrac{a_k+b_k}2=\dfrac{c_k}2\\\riff c_n=\dfrac{a+b}{2^{n-1}}\] 所以\[a_n+b_n=c_n=\dfrac{a+b}{2^{n-1}}\]
李斌斌755 3# 2013-4-27 19:39
接着来(2)由(1)得\[b_n=\dfrac{a+b}{2^{n-1}}-a_n\\b_{n+1}=\dfrac{a+b}{2^n}-a_{n+1}=\dfrac{a+b}{2^n}-\dfrac{3a_n}4\] 令\[\dfrac{b_{n+1}}{b_n}=q,q\in R\\\dfrac{q(a+b)}{2^{n-1}}-qa_n=\dfrac{a+b}{2^n}-\dfrac{3a_n}4\]卡住
boysxh 4# 2013-4-27 20:21
感觉好像由(1)的通项 再求出an      bn=(an+bn)-an也行
李斌斌755 5# 2013-4-27 21:07
4# boysxh 等高手出手吧
yes94 6# 2013-4-27 21:31
2# 李斌斌755 看了答案,你的是对的http://wenku.baidu.com/view/0760d786cc22bcd126ff0c96.html
Tesla35 7# 2013-4-27 21:32
mark 等待录入
yes94 8# 2013-4-27 21:44
7# Tesla35 不要仅仅录入到你的那个小册子哈 还要粘贴一份答案过来?
李斌斌755 9# 2013-4-27 23:47
8# yes94 我只见其录入
boysxh 10# 2013-4-28 07:42
这样行不行?
yes94 11# 2013-4-28 12:25
10# boysxh 看6楼
boysxh 12# 2013-4-28 14:25
11# yes94 答案我有 感觉弄的太麻烦了而已  而且答案好像有点问题
thread-1432-1-2.html: [不等式] 又摘一不等式(第20届伊朗加强)
reny 1# 2013-4-27 16:29
本帖最后由 reny 于 2013-4-27 23:18 编辑 设$x,y,z>0,x^2+y^2+z^2+xyz=4$,求证$$x+y+z\leqslant\sqrt{2-x}+\sqrt{2-y}+\sqrt{2-z}$$. 摘自:http://blog.sina.com.cn/s/blog_5618e6650101kgc5.html
reny 2# 2013-4-27 23:18
本帖最后由 reny 于 2013-4-27 23:19 编辑 貌似不必限制$x,y,z$的范围. 同时,它也等价于: 在$\triangle ABC$中,成立$$\sin\left(\frac{A}{2} \right)+\sin\left(\frac{B}{2}\right)+\sin\left(\frac{C}{2}\right)\geqslant\cos A+\cos B+\cos C.$$
kuing 3# 2013-4-27 23:21
2# reny 嗯,三角换元后就是这样,我也看到了这点,不过还没证出来,不知是不是已知的不等式,一时也查不到。
reny 4# 2013-4-28 11:48
1# reny 看到一个好的解法: $a+\left(\frac{b+c}{2}\right)^2\leqslant2$(由余弦换元可证) 于是,$\frac{b+c}{2}\leqslant\sqrt{2-a}$,从而$a+b+c\leqslant\sqrt{2-a}+\sqrt{2-b}+\sqrt{2-c}$.
yes94 5# 2013-4-28 12:22
4# reny 这个局部不等式不好找,现在看来,好像这个零件不等式又似乎很简单
thread-1433-1-2.html: [数列] 再请教:数列不等式证明
weihua97 1# 2013-4-27 16:57
本帖最后由 weihua97 于 2013-4-27 16:58 编辑 证明: $$\frac25 <\sum_{k=1}^{n}\frac1{n+k}<\frac45 $$
reny 2# 2013-4-27 17:13
本帖最后由 reny 于 2013-4-27 17:17 编辑 1# weihua97 当然,用积分放缩就挺简单滴. 参考http://www.pep.com.cn/rjwk/gzsxs ... 0110516_1041459.htm
kuing 3# 2013-4-27 17:24
中间那个东西FAQ了……
kuing 4# 2013-4-27 17:33
翻到以前一个贴 http://bbs.pep.com.cn/forum.php?mod=viewthread&tid=456778 居然讨论了五页……
reny 5# 2013-4-27 18:43
又翻到一个帖子http://blog.sina.com.cn/s/blog_a9606fae0101a42l.html
yes94 6# 2013-4-27 22:05
5# reny 你的博客?褚小光?
reny 7# 2013-4-27 22:19
6# yes94 不是,我都是浏览别人的博客而已.
yayaweha 8# 2013-4-27 23:11
记$R_n=\frac{1}{n+1}+\frac{1}{n+2}+\cdots+\frac{1}{2n}$,     $$R_{n+1}-R_n=\frac{1}{2n+1}+\frac{1}{2n+2}-\frac{1}{n+1}$$ 所以数列$\{R_n\}$为増数列,所以$$R_n<\lim_{n\to\infty}R_n=\large\int_{0}^{1}\frac{1}{1+x}dx=ln(1+x)|_{0}^{1}=ln2<\frac{4}{5}$$
零定义 9# 2013-4-28 00:14
本帖最后由 零定义 于 2013-4-28 11:07 编辑 打酱油路过,灌灌水... 哎,纳闷~水平烂,改来又改去...
thread-1434-1-1.html: 两小儿观日
李斌斌755 1# 2013-4-27 17:37
中午也罢,上午也好,距离差不多。上午柔和,只是中午猛烈,犹如论坛上的题,让人吃不消…… 要是调配些小题,似晨曦之日、初春的太阳,而非晌午烈日、盛夏酷暑。那就好了!
yes94 2# 2013-4-27 18:31
1# 李斌斌755 写的好啊!牛笔!
isea 3# 2013-4-27 23:08
本帖最后由 isea 于 2013-4-27 23:10 编辑 在这里小题,基本都是被秒 秒不了,也是非常少见的麻烦题了
李斌斌755 4# 2013-4-27 23:52
3# isea 这样一来人气会更旺,曲高和寡
yes94 5# 2013-4-28 22:00
3楼、4楼说法都有道理 所以可以建议kuing,再设这两个板块: 数学兴趣小组(和同义名词例如竞赛小组、不等式证明等)、中高考试题讨论(或同义名词)
isea 6# 2013-4-28 23:57
别啊,版块多了一点都不好
kuing 7# 2013-4-29 00:11
我就没打算加 当初搞主题分类的时候我就想过,后来还是决定只搞主题分类算了
李斌斌755 8# 2013-4-29 00:46
高度不一样
kuing 9# 2013-5-11 00:46
我突然发现标题有另一种解释,内涵类。
李斌斌755 10# 2013-5-11 00:51
9# kuing
kuing 11# 2013-5-11 01:15
10# 李斌斌755 你也懂了?
thread-1435-1-2.html: [几何] 一道解析几何题(2)
转化与化归 1# 2013-4-27 18:48
有没有好的思路?
kuing 2# 2013-4-27 19:05
以前叫“囧函数”、“囧圆”,现在又叫“莫言函数”、“莫言圆”,又能拿出来当新题了,实在无语,让莫言情何以堪,就这样被硬拉到数学上来了。
kuing 3# 2013-4-27 19:21
http://bbs.pep.com.cn/forum.php?mod=viewthread&tid=1893066 http://wenku.baidu.com/view/4d766384a0116c175f0e48b3.html
转化与化归 4# 2013-4-27 19:52
3# kuing 只提供了配方的一种方法!
李斌斌755 5# 2013-4-27 20:47
3# kuing 有图有真相
yes94 6# 2013-4-27 21:50
下次改为李斌函数。
李斌斌755 7# 2013-4-27 23:56
6# yes94
转化与化归 8# 2013-4-28 11:32
如果a=1,b=2将会怎样?
thread-1436-1-2.html: [不等式] 一道三角形不等式(from 人教数学群)
kuing 1# 2013-4-27 18:49
后面他并没有写出用“亲生”的过程,其实用常规方法就很简单。 下面证明在非钝角 $\triangle ABC$ 中恒有 \[\sin A+\sin B+\sin C\geqslant \cos A+\cos B+\cos C+1.\] 证明: \begin{align*} & \sin A+\sin B-\cos A-\cos B \\ ={}& 2\cos \frac{A-B}2\sin \frac{A+B}2-2\cos \frac{A-B}2\cos \frac{A+B}2 \\ ={}& 2\cos \frac{A-B}2\left( \cos \frac C2-\sin \frac C2 \right), \end{align*} 由 $\triangle ABC$ 为非钝角三角形易知 $\cos(C/2)\geqslant\sin(C/2)$ 以及 $\abs{A-B}\leqslant \pi-A-B=C$,于是得到 \begin{align*} \sin A+\sin B-\cos A-\cos B&\geqslant 2\cos \frac C2\left( \cos \frac C2-\sin \frac C2 \right) \\ & =1+\cos C-\sin C, \end{align*} 即 \[\sin A+\sin B+\sin C\geqslant \cos A+\cos B+\cos C+1.\]
kuing 2# 2013-4-27 19:17
忘了说取等条件,事实上,等号成立当且仅当 $\triangle ABC$ 为直角三角形。
reny 3# 2013-4-27 19:25
赞一个! 由两个角的三角函数变形得出.
thread-1437-1-2.html: [组合] 一个概率题
转化与化归 1# 2013-4-27 22:13
大家看看这个题的答案究竟是什么?
kuing 2# 2013-4-27 22:14
粹文字的题就不能复制文字上来么,非要发图的?还那么小
转化与化归 3# 2013-4-27 22:17
2# kuing 你把题做了,我发个大点的!
kuing 4# 2013-4-27 22:18
坚决不做
转化与化归 5# 2013-4-27 22:19
4# kuing 坚决不换大图
yes94 6# 2013-4-27 23:38

李斌斌755 7# 2013-4-27 23:54

转化与化归 8# 2013-4-28 11:45
有人认为事件总数为5,有人认为事件总数为16
isea 9# 2013-4-28 15:36
上大图
isea 10# 2013-4-28 15:37
上文字版
isea 11# 2013-4-28 15:38
本帖最后由 isea 于 2013-4-28 15:41 编辑 翻页了吧,我说结果,记着了啊,以后忽悠别人去 哈哈哈哈哈哈哈哈哈哈哈哈哈哈哈哈哈哈哈哈哈哈哈哈哈哈哈哈哈哈哈哈哈哈哈哈哈哈哈哈哈哈哈哈哈哈哈哈哈哈哈哈哈哈哈哈哈哈哈哈哈哈哈哈哈哈哈哈哈哈哈哈哈哈哈哈哈哈哈哈哈哈哈哈哈哈哈哈哈哈哈哈哈哈哈哈哈哈哈哈哈哈哈哈哈哈哈哈哈哈哈哈哈哈哈哈哈哈哈哈哈哈哈哈哈哈哈哈哈哈哈哈哈哈哈哈哈哈哈哈哈哈哈哈哈哈哈哈哈哈哈哈哈哈哈哈哈哈哈哈哈哈哈哈哈哈哈哈哈哈哈哈哈哈哈哈哈哈哈哈哈哈哈哈哈哈哈哈哈哈哈哈哈哈哈哈哈哈哈哈哈哈哈哈哈哈哈哈哈哈哈哈哈哈哈哈哈哈哈哈哈哈哈哈哈哈哈哈哈哈哈哈哈哈哈哈哈哈哈哈哈哈哈哈哈哈哈哈哈哈哈哈哈哈哈哈哈哈哈哈哈哈哈哈哈哈哈哈哈哈哈哈哈哈哈哈哈哈哈哈哈哈哈哈哈哈哈哈哈哈哈哈哈哈哈哈哈哈哈哈哈哈哈哈哈哈哈哈哈哈哈哈哈哈哈哈哈哈哈哈哈哈哈哈哈哈哈哈哈哈哈哈哈哈哈哈哈哈哈哈哈哈哈哈哈哈哈哈哈哈哈哈哈哈哈哈哈哈哈哈哈哈哈哈哈哈哈哈哈哈哈哈哈哈哈哈哈哈哈哈哈哈哈哈哈哈哈哈哈哈哈哈哈哈哈哈哈哈哈哈哈哈哈哈哈哈哈哈哈哈哈哈哈哈哈哈哈哈哈哈哈哈哈哈哈哈哈哈哈哈哈哈哈哈哈哈哈哈哈哈哈哈哈哈哈哈哈哈哈哈哈哈哈哈哈哈哈哈哈哈哈哈哈哈哈哈哈哈哈哈哈哈哈哈哈哈哈哈哈哈哈哈哈哈哈哈哈哈哈哈哈哈哈哈哈哈哈哈哈哈哈哈哈哈哈哈哈哈哈哈哈哈哈哈哈哈哈哈哈哈哈哈哈哈哈哈哈哈哈哈哈哈哈哈哈哈哈哈哈哈哈哈哈哈哈哈哈哈哈哈哈哈哈哈哈哈哈哈哈哈哈哈哈哈哈哈哈哈哈哈哈哈哈哈哈哈哈哈哈哈哈哈哈哈哈哈哈哈哈哈哈哈哈哈哈哈哈哈哈哈哈哈哈哈哈哈哈哈哈哈哈哈哈哈哈哈哈哈哈哈哈哈哈哈哈哈哈哈哈哈哈哈哈哈哈哈哈哈哈 == 初中题,不要想多了 列表或者树状图 不是有人什么的,为了分,标答一定是16种 初中教材 人教版 要求如此 可能是为了 等可能 三字 就这p理由 石皮 题
转化与化归 12# 2013-4-28 17:30
11# isea 实在!
thread-1438-1-2.html: [不等式] 求助不等式
goft 1# 2013-4-27 22:29
本帖最后由 goft 于 2013-4-27 22:34 编辑 $ 已知a,b为正实数,ab=2a+2b-3,a^2+b^2的最小值. $
kuing 2# 2013-4-27 23:01
条件化为 $(a-2)(b-2)=1$,故 $a$, $b$ 同时大于 $2$ 或同时小于 $2$,显然要使 $a^2+b^2$ 取最小值应当 $a$, $b<2$,令 $2-a=x$, $2-b=y$, 则易得 $x$, $y\in(1/2,2)$ 且 $xy=1$,而 \[a^2+b^2=(2-x)^2+(2-y)^2=2+(x+y-2)^2\geqslant 2,\] 当且仅当 $a=b=1$ 时取等。
地狱的死灵 3# 2013-4-27 23:02
设a+b=p,ab=q,q=2p-3 a^2+b^2=p^2-2q=p^2-4p+6≥2 a=b=1时取等号
reny 4# 2013-4-27 23:04
1# goft 因为$(a-2)(b-2)=1$,令$a=x-y+2,b=x+y+2$,于是$x^2-y^2=1$ 所以$a^2+b^2=(x-y+2)^2+(x+y+2)^2=2x^2+2y^2+8x+8=4x^2+8x+6\geqslant2.$
goft 5# 2013-4-27 23:08
谢谢大家指导,背景应该是双曲线的性质
reny 6# 2013-4-27 23:11
3# 地狱的死灵 设$a+b=p,ab=q,q=2p-3$ $a^2+b^2=p^2-2q=p^2-4p+6≥2$ $a=b=1$时取等号 解得真干脆啊.
goft 7# 2013-4-27 23:17
设a+b=p,ab=q,q=2p-3 a^2+b^2=p^2-2q=p^2-4p+6≥2 a=b=1时取等号 地狱的死灵 发表于 2013-4-27 23:02 是啊,换得好巧妙
kuing 8# 2013-4-27 23:25
我条件反射地将条件变成了那样……
yes94 9# 2013-4-27 23:48

李斌斌755 10# 2013-4-27 23:50
9# yes94
yes94 11# 2013-4-28 00:00
10# 李斌斌755 你很喜欢几何的嘛,这次怎么不出招?反被我抢去了!
yes94 12# 2013-4-28 00:17
其实2楼、3楼都是一样的!都是下式: \[{a^2} + {b^2} = {(a + b)^2} - 2ab = {(a + b)^2} - 2(2a + 2b - 3) = {(a + b - 2)^2} + 2 \geqslant 2\]
yes94 13# 2013-4-28 00:28
12# yes94 也可以变成这样: \[{a^2} + {b^2} = {(a + b)^2} - 2ab = {(\frac{{ab + 3}}{2})^2} - 2ab = \frac{{{{(ab)}^2}}}{4} - \frac{{ab}}{2} + \frac{9}{4} = \frac{1}{4}{(ab - 1)^2} + 2 \geqslant 2\]
李斌斌755 14# 2013-4-28 01:34
本帖最后由 李斌斌755 于 2013-4-28 19:38 编辑 11# yes94 复杂化,硬来一种。$y=\frac{x^2}2d$上斜率$k=2$的切线方程为$y=2x-2$ 设$a+b=x,ab=y.M=a^2+b^2\riff y=2x-3,M=a^2+b^2=x^2-2y\riff y+\dfrac M2=\dfrac{x^2}2$ 图像整体向下移动一个单位,所以\[\dfrac M2=1\riff M=2\]
yes94 15# 2013-4-28 17:55
14# 李斌斌755 写的神马?
李斌斌755 16# 2013-4-28 19:48
15# yes94 曲线$y=\dfrac{x^2}2+\dfrac M2d$与直线$y=2x-3$有交点,所以$\dfrac M2\geqslant1$
yes94 17# 2013-4-28 21:37
16# 李斌斌755 来一个简单地均值不等式的方法: \[{(a + b)^2}+4 \geqslant 4(a + b) = 2ab + 6 \Rightarrow {a^2} + {b^2} \geqslant 2\]
李斌斌755 18# 2013-4-28 22:10
17# yes94 代数方法好啊
yes94 19# 2013-4-29 11:53
因为\[{a^2}{b^2} + 1 \geqslant 2ab \Rightarrow {a^2}{b^2} + 6ab + 9 \geqslant 8ab + 8 \Rightarrow {(2a + 2b)^2} = {(ab + 3)^2} \geqslant 8ab + 8\] 展开整理得,\[4{a^2} + 4{b^2} \geqslant 8 \Rightarrow {a^2} + {b^2} \geqslant 2\]
yes94 20# 2013-4-29 11:58
19# yes94 或者改写为: \[{(2a + 2b)^2} = {(ab + 3)^2} = ({a^2}{b^2} + 1) + (6ab + 8) \geqslant 2ab +(6ab +8)\]故\[4{a^2} + 8ab + 4{b^2}\geqslant 8ab + 8 \Rightarrow {a^2} + {b^2}\geqslant 2\]
thread-1438-2-2.html:
yes94 21# 2013-4-29 13:16
20# yes94 再修改: 由条件得,$(2a + 2b)^2= (ab + 3)^2$,即$4a^2+4b^2+8ab= a^2b^2+6ab+9 $,故 $4a^2+4b^2= a^2b^2-2ab+9=(ab-1)^2+8\geqslant8$,于是$a^2+b^2\geqslant2$
thread-1439-1-1.html: Aurora 我这里没事啊,破解的
isea 1# 2013-4-28 00:58
http://kkkkuingggg.5d6d.net/thread-529-2-1.html 20楼已经有破解程序啊
isea 2# 2013-4-28 00:59
破解后,有这样的字样
hnsredfox_007 3# 2013-4-28 08:34
2# isea 我说怎么没有事,原来是破解的哦
yes94 4# 2013-4-28 12:47
3# hnsredfox_007 这是什么英语?谁来翻译一下?
yes94 5# 2013-4-28 12:48
1# isea 就是在你的那个链接里下的啊?
yes94 6# 2013-4-28 13:13
5# yes94 先卸载,然后又重新下载了,是不是要点这个啊?Aurora.2.x.Keygen.exe 这次点了它,希望下次不要又不行了
yes94 7# 2013-4-28 13:16

isea 8# 2013-4-28 13:28
OK了 如果真出现,下次直接LaTeX或者其实类似插件
yes94 9# 2013-4-28 21:56
8# isea 真出现,再求助于你哈
isea 10# 2013-4-28 22:13
9# yes94 不会了 破解就行 一看就知道,能用就用的,注册不注册无所谓的主儿,你
thread-144-1-4.html: [不等式] 三元不等式$abc=1,a,b,c>1/4$
图图 1# 2011-10-25 00:13
本帖最后由 图图 于 2011-10-25 00:16 编辑 Let $abc=1,a,b,c>\dfrac14$,prove that: \[\sum\frac1{4a-1}\ge1\]
pxchg1200 2# 2011-10-25 13:12
1# 图图 kuing是故意留下的么?呵呵 in fact,CS kills it proof: by note :$ a=\frac{y}{x},b=\frac{z}{y},c=\frac{x}{z} $ we rewrite the inequality into: \[ \frac{x}{4y-x}+\frac{y}{4z-y}+\frac{z}{4x-z}\geq 1 \] by Cauchy-Schwarz: \[  \frac{x}{4y-x}+\frac{y}{4z-y}+\frac{z}{4x-z}\geq \frac{(x+y+z)^{2}}{4(xy+yz+xz)-(x^{2}+y^{2}+z^{2})} \] Therefore,it suffice to check that: \[  \frac{(x+y+z)^{2}}{4(xy+yz+xz)-(x^{2}+y^{2}+z^{2})} \geq 1 \] Which is \[ a^{2}+b^{2}+c^{2}\geq ab+bc+ca \] Done!
yes94 3# 2013-2-3 22:02
这种方法都成套路了! 可不可以分母换元?
realnumber 4# 2013-2-3 23:11
3# yes94 还好导数也可以,$a=e^x,b=e^y,c=e^z,x+y+z=0$ \[f(x)=\frac{1}{4e^x-1},4e^x>1,f'(x)=\frac{-4e^x}{(4e^x-1)^2},f''(x)=\frac{16e^{2x}+4e^x}{(4e^x-1)^3}>0\] 用下琴生不等式.
yes94 5# 2013-2-3 23:16
4# realnumber 这种代换有时候求导很危险,因为二阶导数不恒正(负),切线法也失效。 但这次运气超好的哦
realnumber 6# 2013-2-3 23:24
5# yes94 是啊,有时还可以缩小范围,比如本题$\frac{1}{4a-1}>1$,可得$a>\frac{1}{2}$,又$abc=1$,所以$a,b,c\in (0.5,4)$,还不行的话,半凹半凸还有机会
thread-1440-1-2.html: [不等式] 不等式
huamahu 1# 2013-4-28 14:34
请教一道不等式。直接柯西没做出来。 ______kuing edit in $\LaTeX$______ Let $a$, $b$, $c$ be non-negative numbers, no two of them are zero. Then, \[\frac{a^2}{a^2+ab+b^2}+\frac{b^2}{b^2+bc+c^2}+\frac{c^2}{c^2+ca+a^2}\geqslant1.\]
kuing 2# 2013-4-28 15:41
这个貌似是熟悉的: $x$, $y$, $z>0$, $xyz=1$ 有 \[\frac1{x^2+x+1}+\frac1{y^2+y+1}+\frac1{z^2+z+1}\geqslant1.\] 证明不太了解,只会bao力展开为 $x^2+y^2+z^2\geqslant xy+yz+zx$…… 问px应该更清楚
reny 3# 2013-4-28 17:39
本帖最后由 reny 于 2013-4-28 17:45 编辑 2# kuing 令$x=\frac{bc}{a^2},y=\frac{ac}{b^2},z=\frac{ab}{c^2}$,于是需要证明 $$\sum\frac{a^4}{a^4+a^2bc+b^2c^2}\geqslant1$$. 由CS,$$LHS\geqslant\frac{\left(\sum a^2\right)^2}{\sum a^4+abc\sum a+\sum b^2c^2}$$ 只需证明$$\left(\sum a^2\right)^2\geqslant\sum a^4+abc\sum a+\sum b^2c^2$$ 它等价于$$\sum a^2(b-c)^2\geqslant0$$,显然. 当且仅当$a=b=c$,即$x=y=z$时取到等号.
kuing 4# 2013-4-28 17:45
oh,原来是这样,汗,我刚才在另一个贴子里用过这代换,怎么这个没考虑……
thread-1441-1-2.html: [不等式] $\sum {\dfrac{1}{{{a^2} + ab}} \geqslant \dfrac{2}{{\sqrt {abcd} }}} $
yes94 1# 2013-4-28 22:07
\[\sum {\frac{1}{{{a^2} + ab}} \geqslant \frac{2}{{\sqrt {abcd} }}} \]
kuing 2# 2013-4-28 22:54
总觉得以前做过,不过一时没翻到…… 还好重新做下也不是很难。 由齐次性,不妨设 $abcd=1$,由此我们可以令 $a=y/x$, $b=z/y$, $c=w/z$, $d=x/w$, $x$, $y$, $z$, $w>0$,则原不等式等价于 \[\frac{x^2}{y^2+zx}+\frac{y^2}{z^2+wy}+\frac{z^2}{w^2+xz}+\frac{w^2}{x^2+yw}\geqslant2,\] 由柯西不等式及均值不等式有 \begin{align*} &\frac{x^2}{y^2+zx}+\frac{y^2}{z^2+wy}+\frac{z^2}{w^2+xz}+\frac{w^2}{x^2+yw}\\ \geqslant{}& \frac{(x^2+y^2+z^2+w^2)^2}{x^2(y^2+zx)+y^2(z^2+wy)+z^2(w^2+xz)+w^2(x^2+yw)}\\ ={}& \frac{(x^2+z^2)^2+(y^2+w^2)^2+2(x^2+z^2)(y^2+w^2)}{xz(x^2+z^2)+yw(y^2+w^2)+(x^2+z^2)(y^2+w^2)}\\ \geqslant{}&2. \end{align*}
kuing 3# 2013-4-28 23:06
翻到我印象中想到的东东了,原来不是同一个题,不过有点相似之处,而且我用的方法也一样。
thread-1442-1-2.html: [数列] $a_1=\dfrac12,a_{n+1}{a_n}-2a_{n+1}+1 = 0$
yes94 1# 2013-4-28 22:14
本帖最后由 yes94 于 2013-4-30 14:01 编辑
yayaweha 2# 2013-4-28 23:10
与第二题是一样的吧?http://kkkkuingggg.5d6d.net/view ... peid%3D2&page=1
reny 3# 2013-4-28 23:32
怎么有点怪,$a_1=\frac1 2,a_2=0,a_3不存在?$
kuing 4# 2013-4-28 23:46
3# reny 确实,估计首项打错了……
reny 5# 2013-4-29 00:26
2# yayaweha 由此看出,$a_n=\dfrac{n}{n+1},a_1=\dfrac1 2没错,估计递推关系错了$
kuing 6# 2013-4-29 00:32
5# reny 噢,那估计是 $a_1=1/2$, $a_{n+1}a_n-2a_{n+1}+1=0$
yes94 7# 2013-4-29 12:13
6# kuing 那就按照这个递推式吧
yes94 8# 2013-4-30 14:01
7# yes94 来来来,好了,题目改好了
Tesla35 9# 2013-4-30 17:06
马了个克
yes94 10# 2013-4-30 17:12
9# Tesla35 还真的来了啊
thread-1443-1-2.html: [数列] ${a_{n + 1}}{a_n} = n$
yes94 1# 2013-4-28 22:19
______kuing edit in $\LaTeX$______ 例 4. 已知数列 $\{a_n\}$ 满足 $a_1=1$, $a_{n+1}a_n=n$, $n\in\mbb N^+$。 (1)求证:$a_{n+2}=\dfrac1{a_{n+1}}+a_n$; (2)求证:$\displaystyle 2\sqrt n-1\leqslant\frac1{a_1}+\frac1{a_2}+\cdots+\frac1{a_n}<\frac52\sqrt n-1$。
零定义 2# 2013-4-30 10:39
本帖最后由 零定义 于 2013-4-30 13:31 编辑 好坑爹...
yes94 3# 2013-4-30 13:51
2# 零定义 牛笔!
李斌斌755 4# 2013-4-30 18:05
数归 ,当年用过两次后就发誓再不用
kuing 5# 2013-4-30 18:07
4# 李斌斌755 你的损失……
yes94 6# 2013-4-30 18:22
数归 ,当年用过两次后就发誓再不用 李斌斌755 发表于 2013-4-30 18:05 我最先是做不起题的时候用,后来坚决不用数归,到后来发现数归的思想在有些题里真的很霸道!现在有时候也会重新拾起数归。
零定义 7# 2013-4-30 18:32
我这烂水平的就只能神马方法都乱用一通了...
kuing 8# 2013-4-30 18:37
7# 零定义 你有一个很大的缺点就是过于谦虚
李斌斌755 9# 2013-4-30 18:43
7# 零定义 我们是没本钱花费
李斌斌755 10# 2013-4-30 18:47
我总感觉数归需要数感,我没有。
零定义 11# 2013-4-30 19:05
本帖最后由 零定义 于 2013-4-30 19:07 编辑 8# kuing 9# 李斌斌755 看到你们做的,我没几个会的...
yes94 12# 2013-4-30 20:16
11# 零定义 玩起移动来了
isea 13# 2013-5-6 00:25
好像在哪儿见过似的,奇了怪了 最后,既然是例题,快发标答看看
thread-1445-1-2.html: [数列] $a_{n + 1}= \frac12(a_n +\frac1{a_n})$
yes94 1# 2013-4-28 22:24

reny 2# 2013-4-28 22:50
(1)当$n\geqslant2,a_n\geqslant1.$ 易得$\frac{a_{n+1}+1}{a_{n+1}-1}=\left(\frac{a_{n}+1}{a_{n}-1}\right)^2.$
kuing 3# 2013-4-28 23:15
2# reny 这种题估计不用求通项…… PS、楼主一下子贴了几道数列,Tesla35 又有得 mark 了
reny 4# 2013-4-28 23:35
人教群里数列题也挺多滴
地狱的死灵 5# 2013-4-29 01:36
$a_n \ge 1$ $a_{n + 1}  - 1 = \frac{1}{2}(a_n  - 1) + \frac{1}{2}(\frac{1}{a_n} - 1) \le \frac{1}{2}(a_n  - 1)$ $a_2  - 1 = \frac{1}{12} < \frac{1}{2^3 }$ 递推得 $a_n  - 1 < \frac{1}{2^{n + 1}}(n \ge 2)$ 所以$\sum\limits_{i = 1}^n {\frac{{a_i }}{{a_{i + 1} }}}  \le \sum\limits_{i = 1}^n {a_i }  < \frac{3}{2} + \sum\limits_{i = 2}^n {(1 + \frac{1}{2^{i + 1} }} ) < n + 1$
Tesla35 6# 2013-4-30 17:29
马克too
thread-1446-1-1.html: 偶又要现学现卖了 :)
isea 1# 2013-4-28 23:08
一会应该能看到简单的题,但是不算简洁的源文件了 期待吧,几何呢
isea 2# 2013-4-28 23:50
来了来了,其实还需要细调整了一下,不过,想睡觉了,再说再说 来来来,秒几何 的源文件,见附件
isea 3# 2013-4-30 00:45
本帖最后由 isea 于 2013-4-30 00:51 编辑 示图 \begin{tikzpicture}[ultra thick,scale=1.5]   \draw (0,0) rectangle (7,8);   \begin{scope}[yshift=3.94cm,scale=2.22,rotate=-25,blue]     \draw (0,0) rectangle(1,1)[yshift=1cm](0,0) rectangle(1,1)   [xshift=1cm,yshift=-1cm](0,0) rectangle(1,1)   [yshift=-1cm](0,0) rectangle(1,1)   [shift={(1cm,1cm)}](0,0) rectangle(1,1);   \end{scope}   \node at (7,4) [right=6.18pt]{8};   \node at (3.5,8) [above=6.18pt]{7};   \draw [very thin,->](3.7,8.3)--(7,8.3);   \draw [very thin,->](3.3,8.3)--(0,8.3);   \draw [very thin,->](7.3,4.2)--(7.3,8); \draw [very thin,<-](7.3,0)--(7.3,3.8);    \draw [thin](0,8.2)--(0,8.4)[xshift=7cm](0,8.2)--(0,8.4);    \draw [thin](7.2,0)--(7.4,0)[yshift=8cm](7.2,0)--(7.4,0); \end{tikzpicture} 复制代码 手工调整的,放大后,能看出小瑕疵,不知如何精确作图?
kuing 4# 2013-4-30 01:00
3# isea 我看大概要先解题再作图……
isea 5# 2013-4-30 21:52
4# kuing 看了那帖,原来误差大啊,哈哈 想精确必须知道各线段长了,或者角度,看来
isea 6# 2013-5-7 14:06
本帖最后由 isea 于 2013-5-7 14:10 编辑 精标标记,如三视图 %开始画标记长5     \coordinate (a) at (1,2.5);       \coordinate (b) at (0.5,3.17);     \coordinate (c) at ($ (a)!0.2! -90:(b) $);     \coordinate (d) at ($ (b)!0.2! 90:(a) $);      \coordinate (c1) at ($ (a)!0.5!0:(c) $);      \coordinate (d1) at ($ (b)!0.5!0:(d) $);    \coordinate (e) at ($ (c1)!0.4! 0:(d1) $);    \coordinate (f) at ($ (c1)!0.6! 0:(d1) $);       \coordinate (m) at ($ (c1)!0.5! 0:(d1) $);    \draw[very thin] (a)--(c);    \draw[very thin] (b)--(d);    \begin{scope}[->,very thin]    \draw (e)--(c1);    \draw (f)--(d1);    \end{scope}   \node  at (m){5};     %画标记长5结束 需要宏包 \usepackage{tikz} \usetikzlibrary{calc,intersections,through,backgrounds} tikz果然对垂直(其实是任意旋转)有算法,好! (b)!0.5!0:(d) 表:线段BD的中点,(b)是B点,0.5表示BD的一半,0是旋转角。
hejoseph 7# 2013-5-13 10:23
本帖最后由 hejoseph 于 2013-5-13 11:38 编辑 你这里有个比较不好的写法: 如图1 \node at (.25,-.75) [below]{图1}; 文档一旦作修改就要重新修改编号。 应该把tikz的图放到figure环境中,加label,然后入如图1那里用ref
isea 8# 2013-5-24 23:35
你这里有个比较不好的写法: 如图1 \node at (.25,-.75) {图1}; 文档一旦作修改就要重新修改编号。 应该把tikz的图放到figure环境中,加label,然后入如图1那里用ref hejoseph 发表于 2013-5-13 10:23 谢谢,这个“样式”真实用!学习中!
isea 9# 2013-5-24 23:37
本帖最后由 isea 于 2013-5-24 23:39 编辑 尝试画了一下,三角形外接圆, http://kkkkuingggg.5d6d.net/view ... amp;extra=#pid11417 \begin{tikzpicture}[line width=1pt] \coordinate [label=below:$B$] (B) at (0,0); \coordinate [label=below:$A$] (A) at (2,0); \coordinate [label=right:$C$] (C) at ($ (A)!0.5! -120:(B) $); \coordinate (C1) at ($ (A)!0.25! -120:(B) $); \path[name path=m](C1)--($(C1)!5!90:(C)$); \path[name path=n](1,0)--($(1,0)!2!90:(A)$); \path[name intersections={of=m and n,by=O}]; \node[draw,circle through=(B)]at (O) {}; \draw(A)--(B)--(C)--cycle; \draw[->](A)--(O)node[above]{O}; \draw[fill] (O) circle (1pt); \end{tikzpicture} 感觉麻烦了,还需要继续学习,先 mark 一下
thread-1448-1-1.html: [几何] 来来来,秒几何
isea 1# 2013-4-28 23:44
本帖最后由 isea 于 2013-4-28 23:51 编辑 源文件在TeX/LaTeX区 题目:如图1,将三角板放在正方形$ABCD$上,使三角板的直角顶点$E$与正方形$ABCD$的顶点 重合.三角板的一边交$CD$于点$F$ ,另一边$CB$ 的延长线于点$G$. (1)求证:$EF=FG$; (2)如图2,移动三角板,使顶点$E$始终在正方形 的对角线 上,其他条件不变,(1)中的结论是否仍然成立?若成立,请给予证明;若不成立,请说明理由; (3)将(2)中的“正方形$ABCD$ ”改为“矩形$ABCD$ ”,且使三角板的一边经过点$B$,其他条件不变,若$AB=a,BC=b$,求$\dfrac{EF}{EG}$ 的值.
kuing 2# 2013-4-28 23:48
初中试题大题的风格……很喜欢玩正方形三角板什么的……
isea 3# 2013-4-28 23:53
本帖最后由 isea 于 2013-4-28 23:56 编辑 初中试题大题的风格……很喜欢玩正方形三角板什么的…… kuing 发表于 2013-4-28 23:48 嗯,第三问一是想睡觉,二是比较简单,虽然也能推广,看看图2就能解,再次,好像比较难画E点 这次学了一下,两直线的交点,复杂一点点的标签,那700页的tikz手册,想想都怕
yes94 4# 2013-4-29 00:20
3# isea 还是windows自带画图软件很好画的 (1)用三角形的旋转很好证明(是AF=AG吧?) (2)图2中,过E向BC做垂线,过E向CD做垂线,再用三角形的旋转全等很好证明,仍然不变
李斌斌755 5# 2013-4-29 01:01
4# yes94 \[\dfrac{EG}{EF}=\dfrac{AB}{BC}=\dfrac ab\]isea是练习编辑后顺手拈过来的
thread-1448-1-2.html: [几何] 来来来,秒几何
isea 1# 2013-4-28 23:44
本帖最后由 isea 于 2013-4-28 23:51 编辑 源文件在TeX/LaTeX区 题目:如图1,将三角板放在正方形$ABCD$上,使三角板的直角顶点$E$与正方形$ABCD$的顶点 重合.三角板的一边交$CD$于点$F$ ,另一边$CB$ 的延长线于点$G$. (1)求证:$EF=FG$; (2)如图2,移动三角板,使顶点$E$始终在正方形 的对角线 上,其他条件不变,(1)中的结论是否仍然成立?若成立,请给予证明;若不成立,请说明理由; (3)将(2)中的“正方形$ABCD$ ”改为“矩形$ABCD$ ”,且使三角板的一边经过点$B$,其他条件不变,若$AB=a,BC=b$,求$\dfrac{EF}{EG}$ 的值.
kuing 2# 2013-4-28 23:48
初中试题大题的风格……很喜欢玩正方形三角板什么的……
isea 3# 2013-4-28 23:53
本帖最后由 isea 于 2013-4-28 23:56 编辑 初中试题大题的风格……很喜欢玩正方形三角板什么的…… kuing 发表于 2013-4-28 23:48 嗯,第三问一是想睡觉,二是比较简单,虽然也能推广,看看图2就能解,再次,好像比较难画E点 这次学了一下,两直线的交点,复杂一点点的标签,那700页的tikz手册,想想都怕
yes94 4# 2013-4-29 00:20
3# isea 还是windows自带画图软件很好画的 (1)用三角形的旋转很好证明(是AF=AG吧?) (2)图2中,过E向BC做垂线,过E向CD做垂线,再用三角形的旋转全等很好证明,仍然不变
李斌斌755 5# 2013-4-29 01:01
4# yes94 \[\dfrac{EG}{EF}=\dfrac{AB}{BC}=\dfrac ab\]isea是练习编辑后顺手拈过来的
thread-1449-1-2.html: [不等式] 我也发个数列
yayaweha 1# 2013-4-28 23:48
本帖最后由 yayaweha 于 2013-4-28 23:51 编辑 设$a_n$是函数$f(x)=x^3+n^2x-1(n\in N^*)$的零点 (1)证明:$$0<a_n<1.$$ (2)证明:$$\frac{n}{n+1}<\sum_{i=1}^n a_i<\frac{3}{2}$$.
kuing 2# 2013-4-29 00:09
印象中几年前见过(或者是类似)没翻到……
地狱的死灵 3# 2013-4-29 00:33
$a_n= \frac{1}{a_n^2+ n^2}$ 显然$0 < a_n  < 1$ $a_n  > \frac{1}{n + n^2} = \frac{1}{n} - \frac{1}{n + 1}$,证明第一个不等号 $a_1  < 1,a_n  < \frac{1}{n^2  - 1}= \frac{1}{2}(\frac{1}{n - 1} - \frac{1}{n + 1})(n > 1)$,证明第二个不等号
yes94 4# 2013-4-29 11:42
(1)因为$f'(x) = 3{x^2} + {n^2} > 0$,故$f(x)$单调递增,$f(0)f(1) =  - {n^2} < 0 \Rightarrow {a_n} \in (0,1)$ (2)数归法:因为\[1 = {a_n}(a_n^2 + {n^2}) < {a_n}(1 + {n^2}) \Rightarrow {a_n} > \frac{1}{{1 + {n^2}}}\] 当$n = 1$ 时,时,$f\left( x \right) = {x^3} + x - 1$,由于$f\left( {\dfrac{2}{3}} \right) = {\left( {\dfrac{2}{3}} \right)^3} + \dfrac{2}{3}-1 = -\dfrac{1}{{27}} < 0$,$f\left( {\dfrac{3}{4}} \right) = {\left( {\dfrac{3}{4}} \right)^3} + \dfrac{3}{4} - 1 = \dfrac{{11}}{{64}} > 0$ , 所以,$\dfrac{1}{2}<\dfrac{2}{3} < {a_1} < \dfrac{3}{4}$ 假设$\sum\limits_{i = 1}^k {{a_i}}  > \dfrac{k}{{k + 1}}$,则当$n =k+1$时,\[\sum\limits_{i = 1}^{k + 1} {{a_i}}  > \frac{k}{{k + 1}} + \frac{1}{{1 + {{(k + 1)}^2}}} > \frac{k}{{k + 1}} + \frac{1}{{(k + 1)(k + 2)}} = \frac{{{k^2} + 2k + 1}}{{(k + 1)(k + 2)}} = \dfrac{{k + 1}}{{k + 2}}\] 再证明右边: ${a_n} = \dfrac{{1 - a_n^3}}{{{n^2}}} < \dfrac{1}{{{n^2}}}{\rm{ = }}\dfrac{4}{{4{n^2}}} < \dfrac{4}{{4{n^2} - 1}} = 2(\dfrac{1}{{2n - 1}} - \dfrac{1}{{2n + 1}}) \Rightarrow \sum\limits_{i = 1}^n {{a_i}}  < \dfrac{3}{4}{\rm{ + }}2(\dfrac{1}{3} - \dfrac{1}{{2n + 1}}) < \dfrac{{17}}{{12}} < \dfrac{3}{2}$
yayaweha 5# 2013-4-29 12:36
本帖最后由 yayaweha 于 2013-4-29 12:39 编辑 $a_n= \frac{1}{a_n^2+ n^2}$ 显然$0 < a_n  < 1$ $a_n  > \frac{1}{n + n^2} = \frac{1}{n} - \frac{1}{n + 1}$,证明第一个不等号 $a_1  < 1,a_n  < \frac{1}{n^2  - 1}= \frac{1}{2}(\frac{1}{n - 1} - \f ... 地狱的死灵 发表于 2013-4-29 00:33 你的方法证不了右边
零定义 6# 2013-4-29 12:55
2013年广州二模压轴题,不解析...
yes94 7# 2013-4-29 12:57
可不可用牛顿切线法?
yayaweha 8# 2013-4-29 13:51
你试一下
yes94 9# 2013-4-29 13:53
8# yayaweha 你试一下
Tesla35 10# 2013-4-30 09:00
来mark了、、、
yes94 11# 2013-4-30 13:57
来mark了、、、 Tesla35 发表于 2013-4-30 09:00 我的那几个数列贴,你怎不mark啊?
thread-145-1-8.html: [函数] 零点问题
图图 1# 2011-10-25 00:31
本帖最后由 图图 于 2011-10-25 00:36 编辑 对任意的实数x,给定区间$\left[k-\dfrac12,k+\dfrac12\right],k\in Z$,设函数f(x)表示实数x与x的给定区间内整数之差的绝对值 当$e^{-\frac12}<a<1$时,求方程$f(x)=\log_a{\sqrt x}$的实根个数,并说明理由(已知$e^{-\frac12}>\dfrac12)$
thread-1450-1-2.html: [几何] 求角度
李斌斌755 1# 2013-4-29 00:37
如图三角形$ABC$中,$AB=AC,\angle A=20^\circ,\angle BDE=20^\circ,\angle ABD=10^\circ$,求$\angle ACE$
kuing 2# 2013-4-29 00:38
这个很著名的FAQ了……
李斌斌755 3# 2013-4-29 00:41
人教初中版题,两天没作出来,刚去百度一下,有题没答案
李斌斌755 4# 2013-4-29 00:42
2# kuing 给个链接。
kuing 5# 2013-4-29 01:04
4# 李斌斌755 http://bbs.pep.com.cn/thread-380504-1-1.html http://www.aoshoo.com/bbs1/dispbbs.asp?boardid=43&id=2035 http://bbs.pep.com.cn/thread-552410-1-1.html ……等等,题目不尽相同,但都是类似的,你可以参考一下。 另外 isea 还有一个大贴: http://bbs.pep.com.cn/thread-668772-1-1.html 里面提及了一大堆求角度的题目……(我看过一点表示头都晕)
李斌斌755 6# 2013-4-29 01:30
5# kuing 谢谢kuing,这贴就是第二个链接楼主几天前在初中版发的贴,第一个链接里图一样,但问的问题不同且英文看不太明白。isea的大帖看过,其他方法多用的是三角、解析,想看看有没有纯几何?现在知道题的背景……免得郁闷
kuing 7# 2013-4-29 01:36
5# kuing 谢谢kuing,这贴就是第二个链接楼主几天前在初中版发的贴,... 李斌斌755 发表于 2013-4-29 01:30 你看错了,不是前几天,是前几年,你再仔细看看那个贴1#的年份。
李斌斌755 8# 2013-4-29 01:44
7# kuing 是$2005$年发的,前几天被翻出来的,还以为……
kuing 9# 2013-4-29 01:46
8# 李斌斌755 嗯,跟第二个链接相差一天,但是第二个链接里的内容更多。
isea 10# 2013-4-29 01:59
本帖最后由 isea 于 2013-5-8 00:53 编辑 原型:(一般中文译为)汤普森问题 自己去 查 几何瑰宝 上册 200 左右($\pm 10$页),具体页数忘记了 追溯到1920年,一堆变式 变来变去都是等边轴对称,共圆,同一法 一般会解的不愿意求,人教几何爱好者都解过,基本 像我即不愿意解,且不又不会解,哈哈 记得人教曾经有几发类似的帖子,偶调侃一下,就引起挑战,然后我灰溜溜的闪了 睡觉睡觉 好晚了 ===================== 现在2013年5月8日更新此楼(补充以往的整理),原型如下 题:$\triangle ABC$中,已知$AB=AC,\angle A=\angle ACD =\angle ABE$,$D$在$AB$上,$E$在$AC$上,求$\angle CDE=?$                                    解法1                                    解法2                                    解法3                                    解法4                                    解法5
李斌斌755 11# 2013-4-29 02:31
10# isea 谢谢isea
isea 12# 2013-5-7 23:22
一会,我来帖帖这个原型
isea 13# 2013-5-8 00:45
本帖最后由 isea 于 2013-5-8 00:52 编辑 继续                                    解法6                                    解法7                                    解法8                                    解法9                                    解法10
isea 14# 2013-5-8 00:53
最后要说的是(当然,解法远不止以上10种),此类型题,有人喜欢坚直图,有人喜欢横向放平的图……
李斌斌755 15# 2013-5-8 01:01
本帖最后由 李斌斌755 于 2013-5-8 01:10 编辑 14# isea 谢谢isea 自己该打,没细看而去做    http://bbs.pep.com.cn/forum.php? ... &extra=page%3D1 上的题,法3与法8构建过,但没证出来。待后慢慢消化。 喜欢1,2,3,4,5,6,8。尤其是1,6.法1全面解剖,法6简洁明朗。
isea 16# 2013-5-8 01:19
我把原人教22楼搬过来得了。 从上面回帖能看出,没消化原22楼的附件,哈哈。 入门足够了(当然也不是入门,是研究了,完全不同境界),这附件其实要更新了,如果从几何变换角度切入的话,少30度角的轴对称,共圆等,不够精练,因为当时是面向初中生而整理的。
isea 17# 2013-5-8 01:21
本帖最后由 isea 于 2013-5-8 01:22 编辑 14# isea 谢谢isea 自己该打,没细看而去做    http://bbs.pep.com.cn/forum.php? ... &extra=page%3D1 上的题,法3与法8构建过,但没证出来。待后慢慢消化。 喜欢1,2,3,4,5,6,8。尤其是 ... 李斌斌755 发表于 2013-5-8 01:01 同感啊,法几我忘记了,反正一个线段旋转60度,然后证正三角顶点落在边上,我都这么处理 其他方法反而不会了,哈哈
isea 18# 2013-5-8 01:23
这帖内容太多且精,平时不怎么研究几何题的,不是马上能读完的
李斌斌755 19# 2013-5-8 03:28
16# isea 收藏了,慢慢研究。
thread-1451-1-1.html: 找贴
李斌斌755 1# 2013-4-29 04:53
本帖最后由 李斌斌755 于 2013-4-29 12:31 编辑 问下kuing,2013年前自己的回复怎么找不着了?
kuing 2# 2013-4-29 12:33
试试在 google 搜索: 李斌斌755 site:kkkkuingggg.5d6d.net 可能能找到
李斌斌755 3# 2013-4-29 14:11
2# kuing 还是找不到
kuing 4# 2013-4-29 14:21
3# 李斌斌755 我试了一下,搜出来好多页,不过比较乱,也有些是重复的,翻它几页应该也能找到你想要找的东东吧
thread-1452-1-2.html: [不等式] 一道不等式
huamahu 1# 2013-4-29 14:30
想好久,不知道怎么做,求教各位! ______kuing edit in $\LaTeX$______ $a$, $b$, $c\in\mbb R^+$, $a^2+b^2+c^2=3$,证明 \[\frac1{1+a^2b^2}+\frac1{1+b^2c^2}+\frac1{1+c^2a^2}\geqslant\frac9{2(a+b+c)}.\]
pxchg1200 2# 2013-5-1 07:03
1# huamahu 没人做? 我来弄下吧。 证明: 首先用3减去两边,不等式等价于 \[ \frac{a^{2}b^{2}}{1+a^2b^2}+\frac{b^{2}c^{2}}{1+b^2c^2}+\frac{c^{2}a^{2}}{1+c^2a^2}\leq 3-\frac{9}{2(a+b+c)} \] 而 \[  \frac{a^{2}b^{2}}{1+a^2b^2}\leq \frac{ab}{2}\] 故只要证明 \[6(a+b+c)\geq 9+(a+b+c)(ab+bc+ca) \] 设$p=a+b+c,q=ab+bc+ca$ \[ p^2-2q=3 \] 不等式变成 \[ f(p)=p^3-15p+18\leq 0\] 注意到$p$的范围是 \[ (\sqrt{3},3] \] 很容易验证那个结论了。 Done。
huamahu 3# 2013-5-1 08:55
谢谢PX
thread-1454-1-1.html: [几何] 不是俄罗斯方块胜似俄罗斯方块
isea 1# 2013-4-30 00:43
本帖最后由 isea 于 2013-5-1 00:19 编辑 源代码 如图,$7\times 8$的矩形,中间5个大小一样的小正方形,组成的整体图形接于矩形。求小正方形的边长。 本帖楼下跟帖解法(非常精彩,不要错过)有纯几,有三角,有软件,其实也都不错,这里给出两兄弟 复数、向量法(写得很自由,随便喷)。 ============================ 题目:如图,$7\times 8$的矩形,中间5个大小一样的小正方形,组成的整体图形接于矩形。求小正方形的边长。 此法源自 江苏 朱丹 文章之 《在概念中发现方法》。 一种解答(写得很自由,随便喷):如图,建直角坐标系$xOy$. 设$\vv {AB}=a+b \mathrm{i},a,b\in \mathbb{R}$,则 \begin{align*} \vv {AD}&=\vv {AB}+\vv {BD}\\ &=(a+b\mathrm{i})+3(a+b\mathrm{i})(-\mathrm{i})\\ &=a+3b+(-3a+b)\mathrm{i}\\ \vv {EC}&=3(a+b\mathrm{i})+2(a+b\mathrm{i})(+\mathrm{i})\\ &=3a-2b+(2a+3b)\mathrm{i} \end{align*} 由$7\times8$的矩形,及向量坐标意义有 $$\left\{\begin{aligned} &a+3b=7\\ &2a+3b=8 \end{aligned}\right.\Rightarrow \left\{\begin{aligned} &a=1\\ &b=2 \end{aligned}\right.$$ 进一步,知小正方形边长$=|\vv {AB}|=\sqrt{1^2+2^2}=\sqrt{5}$.
kuing 2# 2013-4-30 00:48
貌似有道类似的高考题还是什么题……在人教论坛和群都见过的……
isea 3# 2013-4-30 00:50
2# kuing 的确有,但是,我觉得这题难度可能大些,如果方法不当,只是如果,假设,不代表事实,个人观点
李斌斌755 4# 2013-4-30 01:05
好玩,试试!
李斌斌755 5# 2013-4-30 02:44
$AB=BC=CD=EF=GH=x,3x=2x+1\riff x=1\riff CG=5\riff a=\dfrac{5\sqrt3}3$
李斌斌755 6# 2013-4-30 03:10
5# 李斌斌755 应该$a=\sqrt5$
kuing 7# 2013-4-30 11:28
5# 李斌斌755 $AB=BC=CD=EF=GH=x,3x=2x+1\riff x=1\riff CG=5\riff a=\dfrac{5\sqrt3}3$ 李斌斌755 发表于 2013-4-30 02:44 那些线段相等吗? 我觉得不是,那些可不是垂直的
abababa 8# 2013-4-30 12:22
结果确实是$\sqrt{5}$,不过我是列方程后用软件算的,列了三个元,软件解出来6个结果,就一个正的是$\sqrt{5}$,繁琐。
yes94 9# 2013-4-30 13:47
8# abababa 怎么列的?
kuing 10# 2013-4-30 16:35
没翻到以前讨论的贴子,做了一下其实还是挺简单的啊 易见 \[\left\{\begin{aligned} \sqrt{2^2+3^2}x\sin \left( \theta +\arctan \frac32 \right)&=8, \\ \sqrt{1^2+3^2}x\sin \left( \theta +\arctan \frac31 \right)&=7, \end{aligned}\right.\] 展开就是 \[\left\{\begin{aligned} x(2\sin \theta +3\cos \theta )&=8, \\ x(\sin \theta +3\cos \theta )&=7, \end{aligned}\right.\] 减减便是 \[\left\{\begin{aligned} x\sin \theta &=1, \\ x\cos \theta &=2, \end{aligned}\right.\] 于是显然 \[x=\sqrt5,\] 而且还能知道倾斜角 $\theta=\arctan(1/2)$,这样所有的点的坐标就不难得到了。
李斌斌755 11# 2013-4-30 16:37
7# kuing 哪位来证明
yes94 12# 2013-4-30 16:44
楼主该公布解答了
kuing 13# 2013-4-30 17:02
10# kuing 补充说明一下,当 $y>0$ 时 \begin{align*} \sin \left( \arctan \frac xy \right)&=\frac x{\sqrt{x^2+y^2}}, \\ \cos \left( \arctan \frac xy \right)&=\frac y{\sqrt{x^2+y^2}},   \end{align*} 所以10#那个方程组展开的时候刚好根号都没掉了。 而我更怀疑会有更加直接的方式去说明展开后的方程组。
李斌斌755 14# 2013-4-30 17:07
本帖最后由 李斌斌755 于 2013-4-30 17:11 编辑 再上图
yes94 15# 2013-4-30 17:14
13# kuing “而我更怀疑会有更加直接的方式去说明展开后的方程组。” 我也觉得,所以,要求IC给答案。否则我就要给答案了
kuing 16# 2013-4-30 17:16
... 而我更怀疑会有更加直接的方式去说明展开后的方程组。 kuing 发表于 2013-4-30 17:02 的确如此: 这样就直接得到 \[\left\{\begin{aligned} x(2\sin \theta +3\cos \theta )&=8, \\ x(\sin \theta +3\cos \theta )&=7. \end{aligned}\right.\]
kuing 17# 2013-4-30 17:26
14# 李斌斌755 哦,原来那些线的确是垂直的,看来原图还是有挺大的误差……我竟然被受了误差的影响又弱了……
李斌斌755 18# 2013-4-30 17:32
17# kuing 还需要证明啊!
kuing 19# 2013-4-30 18:22
18# 李斌斌755 详细证明就交给你了……我有我那个方法就满足了,而且改一下数据或里面的图形应该还能用…… PS、标题是不是打错了字
李斌斌755 20# 2013-4-30 20:49
本帖最后由 李斌斌755 于 2013-5-2 10:37 编辑 19# kuing 不证明平行,我作平行 易证蓝色矩形为边长为$9$的正方形\[PQ=1,\dfrac{OQ}{OP}=\dfrac{OM}{ON}\riff OQ=2\riff PN=6\riff ON^2=45\riff OG=\sqrt5\]
thread-1454-1-2.html: [几何] 不是俄罗斯方块胜似俄罗斯方块
isea 1# 2013-4-30 00:43
本帖最后由 isea 于 2013-5-1 00:19 编辑 源代码 如图,$7\times 8$的矩形,中间5个大小一样的小正方形,组成的整体图形接于矩形。求小正方形的边长。 本帖楼下跟帖解法(非常精彩,不要错过)有纯几,有三角,有软件,其实也都不错,这里给出两兄弟 复数、向量法(写得很自由,随便喷)。 ============================ 题目:如图,$7\times 8$的矩形,中间5个大小一样的小正方形,组成的整体图形接于矩形。求小正方形的边长。 此法源自 江苏 朱丹 文章之 《在概念中发现方法》。 一种解答(写得很自由,随便喷):如图,建直角坐标系$xOy$. 设$\vv {AB}=a+b \mathrm{i},a,b\in \mathbb{R}$,则 \begin{align*} \vv {AD}&=\vv {AB}+\vv {BD}\\ &=(a+b\mathrm{i})+3(a+b\mathrm{i})(-\mathrm{i})\\ &=a+3b+(-3a+b)\mathrm{i}\\ \vv {EC}&=3(a+b\mathrm{i})+2(a+b\mathrm{i})(+\mathrm{i})\\ &=3a-2b+(2a+3b)\mathrm{i} \end{align*} 由$7\times8$的矩形,及向量坐标意义有 $$\left\{\begin{aligned} &a+3b=7\\ &2a+3b=8 \end{aligned}\right.\Rightarrow \left\{\begin{aligned} &a=1\\ &b=2 \end{aligned}\right.$$ 进一步,知小正方形边长$=|\vv {AB}|=\sqrt{1^2+2^2}=\sqrt{5}$.
kuing 2# 2013-4-30 00:48
貌似有道类似的高考题还是什么题……在人教论坛和群都见过的……
isea 3# 2013-4-30 00:50
2# kuing 的确有,但是,我觉得这题难度可能大些,如果方法不当,只是如果,假设,不代表事实,个人观点
李斌斌755 4# 2013-4-30 01:05
好玩,试试!
李斌斌755 5# 2013-4-30 02:44
$AB=BC=CD=EF=GH=x,3x=2x+1\riff x=1\riff CG=5\riff a=\dfrac{5\sqrt3}3$
李斌斌755 6# 2013-4-30 03:10
5# 李斌斌755 应该$a=\sqrt5$
kuing 7# 2013-4-30 11:28
5# 李斌斌755 $AB=BC=CD=EF=GH=x,3x=2x+1\riff x=1\riff CG=5\riff a=\dfrac{5\sqrt3}3$ 李斌斌755 发表于 2013-4-30 02:44 那些线段相等吗? 我觉得不是,那些可不是垂直的
abababa 8# 2013-4-30 12:22
结果确实是$\sqrt{5}$,不过我是列方程后用软件算的,列了三个元,软件解出来6个结果,就一个正的是$\sqrt{5}$,繁琐。
yes94 9# 2013-4-30 13:47
8# abababa 怎么列的?
kuing 10# 2013-4-30 16:35
没翻到以前讨论的贴子,做了一下其实还是挺简单的啊 易见 \[\left\{\begin{aligned} \sqrt{2^2+3^2}x\sin \left( \theta +\arctan \frac32 \right)&=8, \\ \sqrt{1^2+3^2}x\sin \left( \theta +\arctan \frac31 \right)&=7, \end{aligned}\right.\] 展开就是 \[\left\{\begin{aligned} x(2\sin \theta +3\cos \theta )&=8, \\ x(\sin \theta +3\cos \theta )&=7, \end{aligned}\right.\] 减减便是 \[\left\{\begin{aligned} x\sin \theta &=1, \\ x\cos \theta &=2, \end{aligned}\right.\] 于是显然 \[x=\sqrt5,\] 而且还能知道倾斜角 $\theta=\arctan(1/2)$,这样所有的点的坐标就不难得到了。
李斌斌755 11# 2013-4-30 16:37
7# kuing 哪位来证明
yes94 12# 2013-4-30 16:44
楼主该公布解答了
kuing 13# 2013-4-30 17:02
10# kuing 补充说明一下,当 $y>0$ 时 \begin{align*} \sin \left( \arctan \frac xy \right)&=\frac x{\sqrt{x^2+y^2}}, \\ \cos \left( \arctan \frac xy \right)&=\frac y{\sqrt{x^2+y^2}},   \end{align*} 所以10#那个方程组展开的时候刚好根号都没掉了。 而我更怀疑会有更加直接的方式去说明展开后的方程组。
李斌斌755 14# 2013-4-30 17:07
本帖最后由 李斌斌755 于 2013-4-30 17:11 编辑 再上图
yes94 15# 2013-4-30 17:14
13# kuing “而我更怀疑会有更加直接的方式去说明展开后的方程组。” 我也觉得,所以,要求IC给答案。否则我就要给答案了
kuing 16# 2013-4-30 17:16
... 而我更怀疑会有更加直接的方式去说明展开后的方程组。 kuing 发表于 2013-4-30 17:02 的确如此: 这样就直接得到 \[\left\{\begin{aligned} x(2\sin \theta +3\cos \theta )&=8, \\ x(\sin \theta +3\cos \theta )&=7. \end{aligned}\right.\]
kuing 17# 2013-4-30 17:26
14# 李斌斌755 哦,原来那些线的确是垂直的,看来原图还是有挺大的误差……我竟然被受了误差的影响又弱了……
李斌斌755 18# 2013-4-30 17:32
17# kuing 还需要证明啊!
kuing 19# 2013-4-30 18:22
18# 李斌斌755 详细证明就交给你了……我有我那个方法就满足了,而且改一下数据或里面的图形应该还能用…… PS、标题是不是打错了字
李斌斌755 20# 2013-4-30 20:49
本帖最后由 李斌斌755 于 2013-5-2 10:37 编辑 19# kuing 不证明平行,我作平行 易证蓝色矩形为边长为$9$的正方形\[PQ=1,\dfrac{OQ}{OP}=\dfrac{OM}{ON}\riff OQ=2\riff PN=6\riff ON^2=45\riff OG=\sqrt5\]
thread-1454-2-2.html:
isea 21# 2013-4-30 21:51
13# kuing “而我更怀疑会有更加直接的方式去说明展开后的方程组。” 我也觉得,所以,要求IC给答案。否则我就要给答案了 yes94 发表于 2013-4-30 17:14 结果肯定是$\sqrt 5$,这个没问题的。 给吧,我没意见。 14# 李斌斌755 哦,原来那些线的确是垂直的,看来原图还是有挺大的误差……我竟然被受了误差的影响又弱了…… kuing 发表于 2013-4-30 17:26 示意图,示意图,纯手工调整的,误差当然大了 看了你们的解法,我终于开心了。 我当时想用代数,几何整了一个多小时,解不出结果来 原来真的这么复杂!
isea 22# 2013-4-30 21:59
20# 李斌斌755 扩成正方形,有戏,有空时好好瞧下
yes94 23# 2013-4-30 21:59
21# isea 我说的答案说的是包含过程! 原来你也没有过程啊?
isea 24# 2013-4-30 22:01
21# isea 我说的答案说的是包含过程! 原来你也没有过程啊? yes94 发表于 2013-4-30 21:59 当然有啦 复数加向量 看到这个大家应该都会了
李斌斌755 25# 2013-4-30 22:04
24# isea 太多的知识点交汇,头大
yes94 26# 2013-4-30 22:08
24# isea 想学习一下
kuing 27# 2013-4-30 22:35
21# isea 话说,你说的“纯手工调整”难道是在 tikz 里调一下编译一次?
李斌斌755 28# 2013-4-30 22:41
我的图都是在几何画板里完成的。
isea 29# 2013-4-30 23:54
本帖最后由 isea 于 2013-4-30 23:55 编辑 呵呵,不然怎么办呢,纯手工好像也只能如此罢了 初学嘛,还有很多不了解,纯练手 TikZ 画图而已 几何画板画几何图当然方便啦 对主楼解答,此法源自 江苏 朱丹 文章之 《在概念中发现方法》。 如图,建直角坐标系$xOy$. 设$\vv {AB}=a+b \mathrm{i},a,b\in \mathbb{R}$,则 \begin{align*} \vv {AD}&=\vv {AB}+\vv {BD}\\ &=(a+b\mathrm{i})+3(a+b\mathrm{i})(-\mathrm{i})\\ &=a+3b+(-3a+b)\mathrm{i}\\ \vv {EC}&=3(a+b\mathrm{i})+2(a+b\mathrm{i})(+\mathrm{i})\\ &=3a-2b+(2a+3b)\mathrm{i} \end{align*} 由$7\times8$的矩形,及向量坐标意义有 $$\left\{\begin{aligned} &a+3b=7\\ &2a+3b=8 \end{aligned}\right.\Rightarrow \left\{\begin{aligned} &a=1\\ &b=2 \end{aligned}\right.$$ 进一步,知小正方形边长$=|\vv {AB}|=\sqrt{1^2+2^2}=\sqrt{5}$.
isea 30# 2013-4-30 23:59
本帖最后由 isea 于 2013-5-1 00:01 编辑 OK,复数乘除法几何意义,此时,复数与向量已经“难分难解”了……
kuing 31# 2013-5-1 00:01
29# isea 完全明白,不过,向量跟复数直接用“=”会不会有点…… 其实完全可以不必扯上向量,用复数表示就行了,比如说 $z_B-z_A=a+b\mathrm i$
isea 32# 2013-5-1 00:07
本帖最后由 isea 于 2013-5-1 00:14 编辑 29# isea 完全明白,不过,向量跟复数直接用“=”会不会有点…… 其实完全可以不必扯上向量,用复数表示就行了,比如说 $z_B-z_A=a+b\mathrm i$ kuing 发表于 2013-5-1 00:01 我觉得一回事,竞赛书等经常这样写(如,张景中的绕来绕去的向量),这俩本来就是兄弟,手足,我很少严格区分。而这里,纯是为了方便直观,所以就那样写了,特别是在几何上。 严格规范点就纯复数,体系很完整 另外,你在10楼,也精彩啊,一下子就抓住了实质,完全是对原作者朱丹说的东东具体化。 (这一点,在立体几何,用自由向量,即,不建坐标系,证平行,垂直等的时候,与综合法(纯几何法)相比,辅助线就是图形化,特别明显 不知,明白我表达意思没,说得太乱)
isea 33# 2013-5-1 00:16
本帖最后由 isea 于 2013-5-1 00:21 编辑 的确如此: 1428 这样就直接得到 \[\left\{\begin{aligned} x(2\sin \theta +3\cos \theta )&=8, \\ x(\sin \theta +3\cos \theta )&=7. \end{aligned}\right.\] kuing 发表于 2013-4-30 17:16 原来如此 ================ 李斌斌755 图画得真准,比偶主楼提供不知准多少倍 有空,在 TikZ 里调一下,这个图的精确图
李斌斌755 34# 2013-5-1 01:26
33# isea 我的图是用笨方法(几何画板中9个小正方形组成中正方形,再用平行线画出大正方形)画的。
thread-1455-1-1.html: 五一了
isea 1# 2013-4-30 00:48
休息休息
kuing 2# 2013-4-30 00:51
不是还有一天咩…… 不过对于我来说还是没什么区别……每天都差不多……没有所谓的休息不休息……
isea 3# 2013-4-30 00:53
消失 了 晚安
hnsredfox_007 4# 2013-4-30 07:57
照常上班 明天休息一天
yes94 5# 2013-4-30 15:03
4# hnsredfox_007 高三辛苦啊啦 张老师常年压阵高三吧?
hnsredfox_007 6# 2013-4-30 15:46
5# yes94 是压文科艺术班 3月份临时接班 估计5月中下旬解散 上课真难受啊……
yes94 7# 2013-4-30 17:17
5# yes94 是压文科艺术班 3月份临时接班 估计5月中下旬解散 上课真难受啊…… hnsredfox_007 发表于 2013-4-30 15:46 文科艺术班,那个的确难受,很多他们都不懂。 不过深受学校信任常年压阵也算好事。
李斌斌755 8# 2013-5-1 12:56
都去郊游了。
kuing 9# 2013-5-1 18:55
论坛也五一了……
kuing 10# 2013-5-1 23:15
今日: 24, 昨日: 124
kuing 11# 2013-5-5 02:54
为什么五一会冷清哩
李斌斌755 12# 2013-5-5 15:56
备战高考
thread-1456-1-2.html: [组合] 画个图来来数数多少条路左下到右上
huamahu 1# 2013-4-30 13:17
本帖最后由 huamahu 于 2013-4-30 13:19 编辑 哈哈,这题下面有个帖子完美解决。 http://kkkkuingggg.5d6d.net/thread-1411-1-1.html 我想问下,如果可以向下移动,结果会怎样?
thread-1457-1-2.html: 求$f(n)$
yayaweha 1# 2013-4-30 21:01
题如下,会猜不会证。
李斌斌755 2# 2013-4-30 21:05
本帖最后由 李斌斌755 于 2013-4-30 21:07 编辑 不是这 http://kkkkuingggg.5d6d.net/thread-1411-1-2.html
kuing 3# 2013-4-30 22:44
看来至少会是短期FAQ啊……
thread-1459-1-2.html: [数列] $a_1=1,a_{n + 1}= \frac12(a_n +\frac4{a_n})$
yes94 1# 2013-4-30 21:29
和上次数列相当类似http://kkkkuingggg.5d6d.net/thread-1445-1-1.html
李斌斌755 2# 2013-4-30 21:35
数列不等式专题
李斌斌755 3# 2013-4-30 21:37
把它归拢到一个帖子里
yes94 4# 2013-4-30 22:57
把它归拢到一个帖子里 李斌斌755 发表于 2013-4-30 21:37 归到一个帖子,那么难点就集中了,不利于解决问题
李斌斌755 5# 2013-4-30 23:20
4# yes94 喔
零定义 6# 2013-5-1 00:15
thread-146-1-8.html: [不等式] 继续Tran Quoc Anh。。
pxchg1200 1# 2011-10-25 16:42
1.Let$a,b,c\geq 0 $with $ab+bc+ca=3 $ prove that: \[ 3(a+b+c)+2(\sqrt{a}+\sqrt{b}+\sqrt{c})\geq 15  \] 2.Let$a,b,c \geq 0$ with $a+b+c=3 $ prove that: \[ (1+a^{2})(1+b^2)(1+c^2)\geq (1+a)(1+b)(1+c) \] 3.Let $a,b,c>0$ prove that: \[ (a^2+b^2+c^2)(ab+bc+ca)^2 \geq \frac{27}{64}(a+b)^2(b+c)^{2}(c+a)^2 \]
图图 2# 2011-10-25 17:49
2# pxchg1200 还要上自习啊?你们是硬性规定吗?
pxchg1200 3# 2011-10-25 21:31
3# 图图 不是啊,是作业太多了。。。
thread-1460-1-2.html: [几何] 小球个数
李斌斌755 1# 2013-4-30 21:49
本帖最后由 李斌斌755 于 2013-4-30 22:01 编辑 转自东方论坛     http://bbs.cnool.net/cthread-104238033.html    一个大球内放入6个大小相同小球,当小球体积最大时,求小球的半径;此时空隙可放小球的半径是多少?
李斌斌755 2# 2013-4-30 21:59
本帖最后由 李斌斌755 于 2013-4-30 22:32 编辑 放$2,3,\cdots,n$个相同小球的情况?小球的半径组成的数列有通项吗?
yes94 3# 2013-4-30 22:11
2# 李斌斌755 没事各大论坛闲逛啊?
李斌斌755 4# 2013-4-30 22:16
3# yes94 闲着也是闲着
李斌斌755 5# 2013-5-2 10:51
没人理啊
kuing 6# 2013-5-2 15:34
太难了啊……
realnumber 7# 2013-5-2 16:22
要是可以做实验就好了,证明更是渺茫, 猜是:3个小球两两相切,球心在一个平面内,构成一个正三角形,另外三个也是,且2个面平行,球心在面上的射影构成正六边形,两个面距离适当,使得两组小球相切. 此时,若小球半径为1,则大球半径为$1+\frac{5}{2\sqrt{3}}$
realnumber 8# 2013-5-2 16:28
也即两两相切的三小球,绕三球心正三角形中心旋转60$\du$,再沿着与正三角形所在平面垂直的方向平移$\sqrt3$,这是另外三球位置.假定小球半径为1.
李斌斌755 9# 2013-5-2 16:29
2个,3个的情况,对吗?要怎么证明呢?
realnumber 10# 2013-5-2 16:32
如果六小球球心在一个面个,比如正六边形以及中心7点中的6点,那么大球半径为3,等会再来实验其他位置,
李斌斌755 11# 2013-5-2 16:47
4个情况。A_BCD为球内接正四面体,O为球心。 5个时
李斌斌755 12# 2013-5-2 16:49
10# realnumber 正六面体,妙!
realnumber 13# 2013-5-2 16:56
9# 李斌斌755 应该对吧. 2个情况证明:假定2小球球心距离为d,$d\ge 2r$,那么两小球上分别有一点,距离为2r+d.也即大球要覆盖这2点的话,直径至少要2r+d. 3个也类似.就这样?
realnumber 14# 2013-5-2 17:00
11# 李斌斌755 也是猜测,5个小球时,5个球心是两个底面重合的正四面体的五个顶点.
零定义 15# 2013-5-2 21:23
12# 李斌斌755 5个球心就出现正六面体了?应该是两个共底面的正四面体吧...6个的话,应该是正八面体~表示此题是22届高二希望杯一试的一个填空题
李斌斌755 16# 2013-5-2 22:34
15# 零定义 高二,说说……
kuing 17# 2013-5-2 22:41
15# 零定义 填空题还好了,可以凭感觉来猜…… 其实我前面之所以说难,是因为这些问题若是要严格证明出来的话往往都不是我能干的事……就像等周问题,我一直只会拿来用,要我证明它就真不会……
李斌斌755 18# 2013-5-2 22:59
17# kuing 你们来个高数证明应该可以吧?
零定义 19# 2013-5-2 23:32
由于球具有对称性,不妨从对称性入手,将大球切为半球,再考虑考虑,看看行不行得通...打酱油lu过,继续打酱油去...
李斌斌755 20# 2013-5-3 12:13
谢谢楼上各位,讨论到这本题基本解决!
thread-1460-2-2.html:
realnumber 21# 2013-5-4 07:53
要不还是先玩简单的? 平面内,把$n$个半径为r小圆放入一个半径为1的大圆. $n=2,3,4,..$时,分别求最大的半径r.
abababa 22# 2013-5-4 13:01
21# realnumber 两个小圆应该最简单吧,就是大圆半径R的一半,否则两个小圆外切,过连心线的直线被截成的线段是4r,如果r>R/2就有4r>2R,半径是R的圆就不能覆盖这线段,也不能覆盖小圆。
realnumber 23# 2013-5-4 13:11
22# abababa 请继续...
abababa 24# 2013-5-4 13:21
23# realnumber 往下就难了,刚问了一位网友,他说不会。不过他说放7、8、9个圆的情况是正6、7、8边形中间再放一个,放10个圆的情况就不对称了,是圆周8个小圆连着相切,再在中间放2个。
李斌斌755 25# 2013-5-4 22:46
有通项吗
realnumber 26# 2013-5-5 07:45
通项不知道,看看我们能解决到几个 还可以把圆放在矩形里,而球放在长方体里似乎还有实用价值,相当于包装盒或集装箱什么的装货品. http://sq.k12.com.cn/discuz/thread-445800-1-1.html
李斌斌755 27# 2013-5-5 13:36
26# realnumber 数学+生活=难
thread-1462-1-1.html: 我来告诉大家一个公开的秘密
isea 1# 2013-4-30 22:03
就是积分 3000 就成元老了 帖子大约530左右吧(我这楼当前数减几帖吧)
李斌斌755 2# 2013-4-30 22:30
$530-239=291$
isea 3# 2013-4-30 22:56
其实没用,都一样
李斌斌755 4# 2013-5-2 00:42
打开右上角“个人中心”点击“用户组权限”显示
shidilin 5# 2013-5-2 23:18
就偶是中级职称
李斌斌755 6# 2013-5-3 00:57
5# shidilin 统计数字要有水分才大
hnsredfox_007 7# 2013-5-3 14:15
我差的很远了
nicvic456 8# 2013-5-4 09:29
新人。刚来。
李斌斌755 9# 2013-5-7 12:16
帖子300由“小试牛刀”变“实习记者”
kuing 10# 2013-5-7 12:33
9# 李斌斌755 擦,这个“发贴数级别”我从来没看……也没设置过,全是默认的……
李斌斌755 11# 2013-5-9 04:09
4# 李斌斌755 乞丐,负9个9,快赶上负无穷了,有这会员
thread-1464-1-2.html: [不等式] $\max\{|x-(a-d)|,|y-a|,|z-(a+d)|\}>td$
yes94 1# 2013-4-30 22:07
______kuing edit in $\LaTeX$______ 3. 求所有的正实数 $t$ 满足:存在一个由实数组成的无限集合 $X$,使得对任意的 $x$、$y$、$z\in X$($x$、$y$、$z$ 可相同),及任意实数 $a$ 与正实数 $d$,均有 \[\max\{\abs{x-(a-d)},\abs{y-a},\abs{z-(a+d)}\}>td.\]
李斌斌755 2# 2013-4-30 22:21
看都看不懂题
李斌斌755 3# 2013-5-1 02:07
无限集合$X$与实数集合$R$有区别吗?
yes94 4# 2013-5-1 14:09
3# 李斌斌755 自然数集、有理数集是无限集合,但都不是实数集合,是实数集合的子集
李斌斌755 5# 2013-5-1 14:51
4# yes94 学习了
thread-1465-1-2.html: [不等式] $\min\{\frac x{y(1+z+x)},\frac y{z(1+x+y)},\frac z{x(1+y+z)}\}$的最大值
yes94 1# 2013-4-30 22:33
本帖最后由 yes94 于 2013-4-30 22:54 编辑 那个贴是和最大值有关(要难一些?),http://kkkkuingggg.5d6d.net/thread-1464-1-1.html 这个贴是求最小值的最大值,简单点吧?
李斌斌755 2# 2013-4-30 22:46
我怎么觉得没最小值
kuing 3# 2013-4-30 22:49
核对题目……
yes94 4# 2013-4-30 22:55
3# kuing 已核
李斌斌755 5# 2013-4-30 23:03
本帖最后由 李斌斌755 于 2013-4-30 23:06 编辑 4# yes94 $x=1,y=1,z\to+\infty\riff\dfrac x{y(1+z+x)}\to0$
kuing 6# 2013-4-30 23:06
4# yes94 OK,那就应该挺简单吧,乘起来再用条件 $x$, $y$, $z\geqslant1$ 大概就……
kuing 7# 2013-4-30 23:07
4# yes94 $x=1,y=1,z\to+\infty\riff\dfrac x{y(1+z+x)}\to0$ 李斌斌755 发表于 2013-4-30 23:03 M 能 $\to0$ 没问题啊,现在要求 M 的最大值。
李斌斌755 8# 2013-4-30 23:10
7# kuing 最小值的最大值
地狱的死灵 9# 2013-5-1 01:12
3# kuing 嗯,三个相乘小于等于1/27是显然的, 于是这三个数中至少有一个小于等于1/3, 并且当x,y,z都取1时三个都是1/3, 因此所求的最小值的最大值就是1/3.
thread-1466-1-1.html: 劳模
李斌斌755 1# 2013-4-30 22:37
上月$yes94$的发帖量可当选劳模
kuing 2# 2013-4-30 22:46
又或者叫“闲模”…… PS、其实你应该可以点论坛右下角“论坛统计”里面有个发贴排行看看最近发贴情况……
isea 3# 2013-4-30 22:56
得闲,我先去写那个俄罗斯方块,不然 yes94 嗷嗷叫
李斌斌755 4# 2013-4-30 22:57
2# kuing 刚去看
isea 5# 2013-5-1 00:23
哈哈 闪人,晚安
yes94 6# 2013-5-1 14:13
晕哦! 注册会员 554 发帖会员 185 管理成员 1 未发帖会员 369 新会员 dueufolk 发帖会员占总数 33.39% 今日论坛之星 李斌斌755 (30) 平均每人发帖数 17.33
kuing 7# 2013-5-1 14:18
6# yes94 那是基本概况,你再点右边的发贴排行,还有很多其他排行等数据……
李斌斌755 8# 2013-5-16 03:33
劳模是不是5$\cdot$1领奖归来
thread-1467-1-2.html: [不等式] 一道Abel不等式
tan9p 1# 2013-4-30 23:04
如图. ______kuing edit in $\LaTeX$______ $a_1$, $a_2$, $\ldots$, $a_n$, $b_1$, $b_2$, $\ldots$, $b_n>0$,求证 \[\sum_{k=1}^n\frac{(a_1+a_2+\cdots+a_k)a_k}{b_1+b_2+\cdots+b_k}\leqslant2\sum_{k=1}^n\frac{a_k^2}{b_k}.\]
thread-1469-1-1.html: 奇怪
李斌斌755 1# 2013-5-2 00:08
论坛首页右上角显示今日n人,左下方显示在线m人,n<m
kuing 2# 2013-5-2 00:12
我怎么没看到有“今日n人”?
李斌斌755 3# 2013-5-2 00:18
本帖最后由 李斌斌755 于 2013-5-2 00:21 编辑 2# kuing
kuing 4# 2013-5-2 00:19
无语……
kuing 5# 2013-5-2 01:15
要我怎么说呢……只能无语……
hnsredfox_007 6# 2013-5-2 07:40
3# 李斌斌755 前两个明显是发帖数啊
李斌斌755 7# 2013-5-11 00:31
唉,是发帖数……
thread-147-1-1.html: 真latex在线预览
kuing 1# 2011-10-25 21:58
http://www.tlhiv.org/ltxpreview/ 操作方法就是在左边的框框里打代码,然后点击左下角preview,右边就会显示效果。 这是基于真latex的,但貌似只能玩一页。 允许加载部分宏包,点击Packages便知,只可惜没有cjk,所以没法用中文,但有tikz,也就是可以作图了,又只可惜貌似加不了库,所以作点简单的还可以。 可直接下载,注意右下角的download有几种格式可选。 其余东东,自己玩……
kuing 2# 2011-10-25 22:15
RT所示
kuing 3# 2011-10-25 22:48
那网站里还有一个 PNG/JPEG (Raster) to EPS/PDF (Vector):http://www.tlhiv.org/rast2vec/
kuing 4# 2011-10-26 14:49
一些其他提供在线生成latex公式图片的网站 http://www.imathas.com http://codecogs.izyba.com http://www.forkosh.com/mimetex.html 或 http://www.forkosh.com/mathtex.html http://rogercortesi.com/eqn/index.php
kuing 5# 2011-10-26 16:02
http://rogercortesi.com/eqn/index.php?filename=tempimagedir%2Feqn9161.png&latextext=%5Csum_{cyc}%5Csqrt{%5Cfrac+a{b%2Bc}}%5Cgeqslant2&outtype=png&bgcolor=white&txcolor=blue&res=200&transparent=1&antialias=1
kuing 6# 2012-10-7 00:10
\begin{tikzpicture}[xscale=0.6] \node at (0,0) [above left] {$O$}; \draw [->,>=latex] (-10,0)--(10,0) node[above]{$x$}; \draw [->,>=latex] (0,-1.8)--(0,1.8) node[left]{$y$}; \foreach \x/\xtext in {-540/-3\pi,-450/-\frac{5\pi}2,-360/-2\pi,-270/-\frac{3\pi}2,-180/-\pi,-90/-\frac\pi2,90/\frac\pi2,180/\pi,270/\frac{3\pi}2,360/2\pi,450/\frac{5\pi}2,540/3\pi} { \draw (\x*3.14/180,0) node[below] {$\xtext$} -- +(0,0.1); } \draw (0,-1) node[below left]{$-1$} --+(0.1,0); \draw (0,1) node[above left]{$1$} --+(0.1,0); \draw [domain=-540:540,samples=100,smooth] plot ({\x*3.14/180,sin(\x)}); \foreach \i in {-450,-90,270} { \draw [very thick,blue,domain=\i:(\i+180),samples=50,smooth] plot ({\x*3.14/180,sin(\x)}); } \draw [dashed] (-10,-1)--(10,-1) (-10,1)--(10,1); \end{tikzpicture}
叶剑飞Victor 7# 2013-4-1 00:06
本帖最后由 叶剑飞Victor 于 2013-4-1 00:54 编辑 LaTeX 全文在线编译,支持汉字,可下载编译好的 pdf 文件! http://w3.math.sinica.edu.tw/online_tex/
kuing 8# 2013-4-1 00:36
7# 叶剑飞Victor 谢谢提供,牛比!
叶剑飞Victor 9# 2013-4-1 00:52
8# kuing 可是那个LaTeX只支持繁体字,不支持简体字。
isea 10# 2013-4-1 12:28
\node at (0,0) [above left] {$O$}; \draw [->,>=latex] (-10,0)--(10,0) node[above]{$x$}; \draw [->,>=latex] (0,-1.8)--(0,1.8) node[left]{$y$}; 原来这样标坐标轴的x,y
kuing 11# 2013-4-1 22:22
9# 叶剑飞Victor 还真不支持简体,但是他那里有个列表里也写有简体字体,设成那个还是不行……
叶剑飞Victor 12# 2013-4-20 11:30
ShareLaTeX      https://www.sharelatex.com/      完全支持UTF-8字符,当然包括全部汉字在内。
kuing 13# 2013-4-20 13:56
12# 叶剑飞Victor 这个貌似要翻qiang才能打开……而且要注册?
yes94 14# 2013-4-20 16:05
还叫我捐赠?
kuing 15# 2013-4-20 16:10
14# yes94 捐赠?哪里
yes94 16# 2013-4-20 16:37
15# kuing 你的那个链接的网站的右下方 LaTeX Previewer by Troy Henderson \documentclass{article} \begin{document} \thispagestyle{empty}
kuing 17# 2013-4-20 16:38
16# yes94 O,我没看过那里,看不懂,反正在左边写代码就行了
kuing 18# 2013-4-30 13:27
https://www.writelatex.com/ 这个不知怎样
yes94 19# 2013-4-30 15:10
18# kuing 不能输入中文
thread-1471-1-1.html: 请教一个积分
abababa 1# 2013-5-2 08:42
求一个积分 $\int e^{\sin x}\frac{x\cos^3 x-\sin x}{\cos^2 x} dx$ 软件算的结果挺简单的$e^{\sin x}(x-\sec x)$ 求教怎么算出来的。我想用分部积分和换元,但是肯定哪弄错了,分了几次都没弄出来。
hnsredfox_007 2# 2013-5-2 09:53
1# abababa $\displaystyle \int {\rm{e}}^{\sin x}\frac{x\cos^3x-\sin x}{\cos^2x}{\rm{d}}x=\int {\rm{e}}^{\sin x}x\cos x{\rm{d}}x-\int {\rm{e}}^{\sin x}\frac{\sin x}{\cos^2x}{\rm{d}}x\\ \displaystyle =\int x{\rm{d}}{\rm{e}}^{\sin x}-\int {\rm{e}}^{\sin x}{\rm{d}}\left(\frac{1}{\cos x}\right)=\cdots$ 以下分部积分法即可
hnsredfox_007 3# 2013-5-2 10:01
2# hnsredfox_007 继续 $\displaystyle \int {\rm{e}}^{\sin x}\frac{x\cos^3x-\sin x}{\cos^2x}{\rm{d}}x=\int {\rm{e}}^{\sin x}x\cos x{\rm{d}}x-\int {\rm{e}}^{\sin x}\frac{\sin x}{\cos^2x}{\rm{d}}x\\ \displaystyle =\int x{\rm{d}}{\rm{e}}^{\sin x}-\int {\rm{e}}^{\sin x}{\rm{d}}\left(\frac{1}{\cos x}\right)=\left(x{\rm{e}}^{\sin x}-\int {\rm{e}}^{\sin x}{\rm{d}}x\right)-\left[{\rm{e}}^{\sin x}\left(\frac{1}{\cos x}\right)-\int {\rm{e}}^{\sin x}{\rm{d}}x\right] \\ =\cdots$
abababa 4# 2013-5-2 10:22
2# hnsredfox_007 谢谢,到这就明白了,把不能积的那个$\int e^{\sin x}dx$消掉了,这技巧对我来说目前还有难度,这是怎么想到的呢?
hnsredfox_007 5# 2013-5-2 13:58
4# abababa 试着试着就出来了哦
abababa 6# 2013-5-3 07:12
5# hnsredfox_007 谢谢。 我也曾试过$e^{\sin x}x\cos x$这部分拆出来积,但是碰到$e^{\sin x}$积不出,就没往下做,以为这么拆不对,完全没想到能和后一项消掉。看我来还是弱。
thread-1472-1-1.html: 无聊清理了一下我的网页收藏夹顺便晒晒能晒的东东
kuing 1# 2013-5-2 15:29
由于发太多链接可能会被hx,故此这里不直接将表格贴出来了,只给个链接 http://user.qzone.qq.com/249533164/blog/1366737269 另外顺便将html文件放上来当作是备份。
hnsredfox_007 2# 2013-5-2 15:35
1# kuing 貌似人教论坛打不开吧
kuing 3# 2013-5-2 15:40
2# hnsredfox_007 能啊,我做完那个表之后所有链接都点击过能打开的……
hnsredfox_007 4# 2013-5-2 15:51
3# kuing 难道是我的网络问题?人教网可以打开但人教论坛打不开哦
kuing 5# 2013-5-2 15:54
4# hnsredfox_007 前两天人教群里也有人这样说,而且还说上其他网站都没问题,就人教论坛打不开…… 难道因为论坛被刷广gao太多,被某些网络列为了黑名单?
thread-1473-1-1.html: verbatim?——no
isea 1# 2013-5-2 15:44
\begin{verbatim} \begin{align*} \vv {AD}&=\vv {AB}+\vv {BD}\\ &=(a+b\mathrm{i})+3(a+b\mathrm{i})(-\mathrm{i})\\ &=a+3b+(-3a+b)\mathrm{i}\\ \vv {EC}&=3(a+b\mathrm{i})+2(a+b\mathrm{i})(+\mathrm{i})\\ &=3a-2b+(2a+3b)\mathrm{i} \end{align*} \end{verbatim}
kuing 2# 2013-5-2 15:45
一行内的可以用 \verb
kuing 3# 2013-5-2 15:47
但是这时用 \verb 也不能像真 LaTeX 那样用 \verb"a^2",因为 MathJax 不认,必须放在数学模式,比如 \(\verb"$\sqrt{a^2}=|a|$"\) 显示:\(\verb"$\sqrt{a^2}=|a|$"\) $\verb"\(\sqrt{a^2}=|a|\)"$ 显示:$\verb"\(\sqrt{a^2}=|a|\)"$ 为什么不能全部用 \$ ?原因显然
kuing 4# 2013-5-2 15:57
所以在本论坛显示源码还是比较郁闷的一件事。 没有 verbatim 环境; 用 \verb 又麻烦; 前几天整的 precode 又不能有中括号; ……
kuing 5# 2013-5-2 16:16
MathJax 能认的非数学模式的代码好像就只有 \ref{...} 和 \eqref{...} 了
isea 6# 2013-5-2 23:00
木有关系,能上附件嘛
thread-1474-1-2.html: [不等式] px的一道次数有点高还好对称(from粉丝群)
kuing 1# 2013-5-2 22:13
a,b,c是正的 PX 表示这是个好东东,还说 Michael   22:05:12 Prove it then I will tell you the story 那就写吧,先写成 pqr,就是 \[108r(p^2-2q)^3\leqslant p^6\bigl(p(p^2-3q)+4r\bigr),\] 或 \[4r\bigl((3p^2-6q)^3-p^6\bigr)\leqslant p^7(p^2-3q),\] 左边因式分解,然后约去非负的 $p^2-3q$,即只要证 \[8r(13p^4-42p^2q+36q^2)\leqslant p^7,\] 因为 $3rp\leqslant q^2$,故只要证 \[8q^2(13p^4-42p^2q+36q^2)\leqslant 3p^8,\] 令 $p^2/q=t$,则 $t\geqslant3$,要证的等价于 \[3t^4-8(13t^2-42t+36)\geqslant 0,\] 整理为 \[(t-3)^2(3t^2+18t-23)+36(t-3)+27\geqslant 0,\] 显然成立。
pxchg1200 2# 2013-5-2 22:20
1# kuing 我来告诉你这个故事吧。 晚上想起Can的一个不等式 \[ \frac{a}{b}+\frac{b}{c}+\frac{c}{a}\geq \frac{9(a^2+b^2+c^2)}{(a+b+c)^2} \] 首先怎么弄都弄不出。偶尔发现好像可以利用 \[ (x+y+z)^3\geq \frac{27}{4}(x^2y+y^2z+z^2x+xyz) \] 用 \[ x=\frac{a}{b},y=\frac{b}{c},z=\frac{c}{a} \] 马上变成那个结论了。 我们就把一个轮换的变成了对称的。处理起来好多niao。。
kuing 3# 2013-5-2 22:26
2# pxchg1200 原来如此,完全明白……牛笔的转换……
thread-1475-1-2.html: [不等式] 2013届南京市、盐城市高三三模压轴第三问
pengcheng1130 1# 2013-5-2 23:20
设\{a_n\} 是等差数列,$b_n=a^{a_n}(a>0)$. 证明:$\dfrac{b_1+b_2+\cdots+b_n}{n}\leqslant\dfrac{b_1+b_n}{2}$.
kuing 2# 2013-5-2 23:28
我建议还是将原题发一下,以免漏条件。
pengcheng1130 3# 2013-5-2 23:41
三小问相互独立,我刚才看了,没错误,请放心!
kuing 4# 2013-5-2 23:44
不用了,原来很简单 依题意可设 $b_n=x\cdot y^{n-1}$,其中 $x$, $y>0$,则原不等式等价于 \[2(1+y+y^2+\cdots +y^{n-1})\leqslant n(1+y^{n-1}),\] 易证对 $k=0$, $1$, \ldots, $n-1$ 有 \[1+y^{n-1}\geqslant y^k+y^{n-1-k},\] 求和即得。
pengcheng1130 5# 2013-5-2 23:47
谢谢!
thread-1476-1-2.html: [几何] 小河取水
李斌斌755 1# 2013-5-2 23:56
本帖最后由 李斌斌755 于 2013-5-3 00:03 编辑 从$A$点到河边(河流近似为直线$l$)任意点$P$取水送到B点,求${(AP+PB)}_{\min},{(AP-PB)}_{\max}$ 最小值容易。现在问题是:当$AB\sslash l$时,${(AP-PB)}_{max}=?$,其几何意义?
kuing 2# 2013-5-2 23:59
无穷远呗…… PS、AB\sslash l 得 $AB\sslash l$
李斌斌755 3# 2013-5-3 00:00
本帖最后由 李斌斌755 于 2013-5-3 00:05 编辑 2# kuing 有误,,更正,主要想知道有没有几何意义!
kuing 4# 2013-5-3 00:14
平行话,差的,max 或 min 都是无穷远啦,看哪边而已……几何意义?无穷远去了,也算是退化三角形,不是几何意义么?
李斌斌755 5# 2013-5-3 00:36
本帖最后由 李斌斌755 于 2013-5-3 00:49 编辑 4# kuing 从无穷远$\to P\to$无穷远$\iff$从零到正又到零再到正又到零(不知如何表达 ),最大值不是零。 图中k为AB垂直平分线。
kuing 6# 2013-5-3 00:38
4# kuing 从无穷远$\to P\to$无穷远$\iff0$从零到正又到零再到正又到零(不知如何表达 ),最大值不是零。 李斌斌755 发表于 2013-5-3 00:36 错了,是从 $-\abs{AB}\to0\to+\abs{AB}$ 或反过来,总之,是单调的。
李斌斌755 7# 2013-5-3 00:47
本帖最后由 李斌斌755 于 2013-5-3 00:49 编辑 或者换言之直线$l$平行线段$AB$,能否用规尺作图法在$l$上找到$P$点,使$|AP-PB|$最大。
kuing 8# 2013-5-3 01:05
或者换言之直线$l$平行线段$AB$,能否用规尺作图法在$l$上找到$P$点,使$|AP-PB|$最大。 李斌斌755 发表于 2013-5-3 00:47 怎么又加了绝对值? 绝对值的话,就是由 $\abs{AB}\to0\to\abs{AB}$,“最大值”在无穷远。
李斌斌755 9# 2013-5-3 01:14
8# kuing 无穷远也是零呀,$AP\approx PB$怎么会是最大值
李斌斌755 10# 2013-5-3 01:17
本帖最后由 李斌斌755 于 2013-5-3 01:21 编辑 8# kuing 是$AB$,看来是喝高了
thread-1477-1-2.html: [数列] 昨天一个学弟问我的问题
pxchg1200 1# 2013-5-3 08:34
设$\frac{|b_{n+1}|}{n^2}$,$\frac{|b_{n}|}{n}$,$1+b_{n}^{2}$成等比,且$b_{n}\neq 0$,记 \[ T_{n}=\sum_{k=1}^{n}{\frac{(-1)^k(k+2)}{(k+1)^2}b_k}\] 证明 \[ T_{n}<\frac{5}{4}|b_1| \] 由于昨天晚上在和天书聊bat,所以没空思考,今天特地贴来论坛。
Tesla35 2# 2013-5-3 12:25
数列
零定义 3# 2013-5-3 23:02
玩数列不等式玩上隐了...
pxchg1200 4# 2013-5-3 23:11
3# 零定义 多谢啦,睡神,最近我没状态。。。
零定义 5# 2013-5-3 23:16
px、kk你俩在轮休?这个感觉挺弱的,还可以再加强吧?
realnumber 6# 2013-5-4 07:46
为输入方便,不妨设$b_n\in R^+$ $b_{n+1}=\frac{b_n^2}{1+b_n^2}\le \frac{1}{2}b_n$,而$\frac{k+2}{(k+1)^2}>\frac{k+3}{(k+2)^2}$,楼上的 $\frac{5}{9}$可以修改为$\frac{1}{2}$, 好粗心的我,把$b_2$看作了$b_1$,一直没完成.
零定义 7# 2013-5-4 16:25
“5/9改为1/2”这个这个看到的 了,我是想说将“(k+3)/(k+2)^2<(k+2)/(k+1)^2”再稍微的缩缩~ 顺带的说一个,如果把b(n)看成正的,那么范围就会更小了...
零定义 8# 2013-5-4 16:48
额...经推导,1/2是最佳的了...
realnumber 9# 2013-5-5 07:38
不会是最佳的,如果是$\frac{1}{2}$,那么$b_n=1,\forall n\in N$,但数列$\{ b_n \}$是递减的. 数列递缩的有些快.$n\ge2$时,$\abs{b_n}<1$,$n$足够大时,$b_{n+1}=\frac{b_n^2}{1+b_n^2}\approx b_n^2$
thread-1478-1-2.html: [不等式] 一个不等式
pxchg1200 1# 2013-5-3 08:50
设 $x,y,z>0$.证明 \[ \frac{y}{x}+\frac{z}{y}+\frac{x}{z}\geq 3\left(\frac{x^2}{x^2+2yz}+\frac{y^2}{y^2+2zx}+\frac{z^2}{z^2+2xy}  \right)\] 不知道昨天的结论有没有用。 如果这个成立的话那么 \[ \frac{y}{x}+\frac{z}{y}+\frac{x}{z}\geq 3\left(\frac{x^2}{x^2+2yz}+\frac{y^2}{y^2+2zx}+\frac{z^2}{z^2+2xy}  \right)\geq \left(\sum_{cyc}{\frac{x}{\sqrt{x^2+2yz}}}\right)^{2} \] 右边就是传说中的江西08.
huamahu 2# 2013-5-7 10:35
求解!!!!
thread-1479-1-2.html: [不等式] 关于2013的一个bat
pxchg1200 1# 2013-5-3 08:59
设$a,b,c>0$,且$a+b+c=3$,求证 \[ (a+b)\sqrt[2013]{\frac{ab+1}{c^2+1}}+(b+c)\sqrt[2013]{\frac{bc+1}{a^2+1}}+(c+a)\sqrt[2013]{\frac{ca+1}{b^2+1}}\geq 6\] (来着西西) 明显的,那个2013是故意的,估计改成k也成立。
thread-148-1-2.html: 反函数的二阶导数
kuing 1# 2011-10-26 00:51
设 $y=f(x)$ 的反函数存在,印象中其反函数 $x=f^{-1}(y)$ 的导数为 $\dfrac1{f'(x)}$,那二阶导数如何?今天碰到一问,才试着做了下。 这样? \[ \frac{\mathrm{d}^2x}{\mathrm{d}y^2}=\frac{\mathrm{d}\bigl(\frac{\mathrm{d}x}{\mathrm{d}y}\bigr)}{\mathrm{d}y}=\frac{\mathrm{d}\bigl(\frac1{\mathrm{d}y/\mathrm{d}x}\bigr)}{\mathrm{d}x}\cdot\frac1{\frac{\mathrm{d}y}{\mathrm{d}x}}=\left(\frac1{f'(x)}\right)'\cdot\frac1{f'(x)}=-\frac{f''(x)}{\bigl(f'(x)\bigr)^2}\cdot\frac1{f'(x)}=-\frac{f''(x)}{\bigl(f'(x)\bigr)^3}. \] 又或者或记反函数 $x=g(y)$,用复合函数求导,这样? \begin{align*} g''(y)&=\bigl(g'(y)\bigr)'=\left(\frac1{f'(x)}\right)'=\left(\frac1{f'\bigl(g(y)\bigr)}\right)'\\ &=-\frac1{\Bigl(f'\bigl(g(y)\bigr)\Bigr)^2}\cdot f''\bigl(g(y)\bigr)\cdot g'(y)=-\frac1{\bigl(f'(x)\bigr)^2}\cdot f''(x)\cdot \frac1{f'(x)}=-\frac{f''(x)}{\bigl(f'(x)\bigr)^3}. \end{align*}
kuing 2# 2011-10-26 13:54
1# kuing 还是用 dx, dy 写比较清楚,用 $'$ 这种写法得心里清楚,否则很容易搞错的说。。。
图图 3# 2011-10-27 13:03
2# kuing
thread-1480-1-1.html: 问一个关于定理引用的问题
abababa 1# 2013-5-3 11:13
比如我打了10个定理,都在不同的页里,现在假设第100页要引用一个第5页的定理,如果用\ref的话,点一下那个标号就会跳到第5页了。 但我想有没有可能这样,让它点一下那个标号,就在鼠标点击处的附近弹窗显示这个定理的内容,觉得这样也方便对照。不知道latex能不能弄这个。
kuing 2# 2013-5-3 11:37
这个问题太专业……表示不会
kuing 3# 2013-5-3 14:12
我用 SumatraPDF 来阅读 PDF,对于你说的这种情形(从100页点了链接跳到第5页),可以通过“转到-上一视图”来跳回100页,再看又可以“转到-下一视图”又跳到第5页,通过那两个快捷键,就可以来回切换,勉强能解决对照的问题…… 其他阅读器应该也有类似的跳转功能吧?
isea 4# 2013-5-3 15:22
3# kuing 几乎所有的PDF浏览器 这样的跳转均是 Alt + left arrow 、right arrow
abababa 5# 2013-5-4 12:57
谢谢。来回跳转到是知道,就是有时候定理或者其它引用之类的有点长,需要对照,这时候觉得离得近方便点。 就像论坛那个发帖按钮,鼠标移动到那里就会弹出来四个选择(话题,投票等等)。我以前看latex说是能做交互式的,像什么幻灯片之类的,觉得能不能用这方面的东西实现呢?当然我是不会弄的。
kuing 6# 2013-5-4 14:13
你去 CTeX 论坛问问吧……
thread-1481-1-1.html: [不等式] 请问这个不等式怎么证明?谢谢
weihua97 1# 2013-5-3 11:43

kuing 2# 2013-5-3 11:46
见:http://bbs.pep.com.cn/thread-1446751-1-1.html
kuing 3# 2013-5-3 11:47
怎么这几天这么多人问这道老题……
李斌斌755 4# 2013-5-30 01:25
本帖最后由 李斌斌755 于 2013-5-30 01:26 编辑 没整明白等式$\dfrac b{a+c}=\dfrac a{b+c}+\dfrac c{a+b}$的右边怎么组装出$\dfrac b{a+c}$(什么思路)
keypress 5# 2013-6-1 19:39
这一步的依据是什么: \[\frac{{{a^2}}}{{ab + ac}} + \frac{{{c^2}}}{{ac + bc}} \geqslant \frac{{{{(a + c)}^2}}}{{b(a + c) + 2ca}}\]
kuing 6# 2013-6-1 20:39
5# keypress 柯西不等式变式中最常用的那个。 PS、看来是 mathtype's copy
李斌斌755 7# 2013-6-2 03:58
6# kuing 柯西不等式共有几个变式?
kuing 8# 2013-6-2 16:32
7# 李斌斌755 想怎么变就怎么变,千变万幻
李斌斌755 9# 2013-6-2 19:04
8# kuing
keypress 10# 2013-6-2 20:49
6# kuing 呵呵,这也叫变式?原来是把积变为商,这算跳步吧。 PS:我在mathtype中打\LeTex代码,结果贴出来就有许多多余的{},烦啊,什么时候mathtype能自定义解释器就好了。另外,外面通用的还是word,PDF都吃不开,生存艰难的。
thread-1482-1-1.html: 为何我发的bat总是没啥人碰?
pxchg1200 1# 2013-5-3 23:31
为何我发的bat总是没啥人碰? 有的应该不难吧。难道都吓到了?
kuing 2# 2013-5-3 23:48
吓,就是吓,
李斌斌755 3# 2013-5-4 00:07
1# pxchg1200 我不会不等式。
isea 4# 2013-5-4 01:38
题都看不懂
isea 5# 2013-5-4 01:39
什么是bat
李斌斌755 6# 2013-5-4 02:12
5# isea 是“题”的意思吧?
hnsredfox_007 7# 2013-5-4 09:11
什么东东啊? 批处理文件?
nicvic456 8# 2013-5-4 09:28
没看懂。。
kuing 9# 2013-5-4 13:42
/mg睡神/mg(2067****) 23:49:26 其实...我根本不知道啥叫bat Michael /oh<pxchg1200@****> 23:49:48 bat dang thuc是越南语中不等式的意思 所以我们就叫bat  了 爱咪哆/睡<liuzj288@****> 23:49:54 群里的专用词汇 kuing/yhh/衰/zhh<kuingggg@qq.com> 23:51:37 其实群里也就px你和一中喜欢说bat……我都没这个习惯,因为我总觉得是批处理
kuing 10# 2013-5-5 02:57
其实我宁愿用 ineq
realnumber 11# 2013-5-5 08:04
样子就怕人,第一反应是,又是一个可怕的不等式,然后闪~~. 要不你下次模仿考试题,来个连发,分成2问,前一个简单,后一个才是你要发的难题.
pxchg1200 12# 2013-5-10 23:27
10# kuing 以前Colorfuldreams MM就喜欢用ineq....
李斌斌755 13# 2013-5-14 17:45
11# realnumber 好主意。
thread-1483-1-2.html: [数列] 数列求通项,特征方程什么的.
realnumber 1# 2013-5-4 13:57
本帖最后由 realnumber 于 2013-5-4 15:03 编辑 http://sq.k12.com.cn/discuz/thread-634744-1-1.html 数据改了下,否则三次方程的虚根,要查公式. 已知:$a_1=a_2=a_3=1,a_{n+3}=7a_{n+1}+6a_n$,求此数列的通项公式. 设$a_{n+3}+xa_{n+2}+ya_{n+1}=x(a_{n+2}+xa_{n+1}+ya_n)$ 可得$x^2-y=7,xy=6$,解得三组解$x=3,x=-1,x=-2$(其实直接可列方程$x^3-7x-6=0$)相应的$y=2,-6,-3$ 所以有$a_{n+2}+3a_{n+1}+2a_n=6\times 3^{n-1}$ $a_{n+2}-a_{n+1}-6a_n=-6\times (-1)^{n-1}$ $a_{n+2}-2a_{n+1}-3a_n=-5\times (-2)^{n-1}$ (看作$a_n,a_{n+1},a_{n+2}$的三元一次方程组),解出$a_n$就可以了.
kuing 2# 2013-5-4 15:19
原题的话……直接用三次方程的三个根来表示通项就好了呗……设而不求……
realnumber 3# 2013-5-4 15:23
2# kuing 也是哦,最近没休息好,状态不好~~,其实本来对这类型的有定理,关于$a_k,a_{k+1},a_{k+2},...,a_n$一次齐次方程为递推公式的数列.
Tesla35 4# 2013-5-5 11:46
数列!
kuing 5# 2013-5-5 13:28
4# Tesla35 ……又来……这个也mark?怎么录入……
李斌斌755 6# 2013-5-5 13:37
thread-1484-1-2.html: [不等式] 判断一个不懂事
Gauss门徒 1# 2013-5-5 11:59
设$1\leqslant x\leqslant y\leqslant z$,则\[\sum\sqrt{x-1}\leqslant\sqrt{x+xyz}\]
kuing 2# 2013-5-5 12:47
以前在不等式群里答过
Gauss门徒 3# 2013-5-5 13:12
2# kuing 哈哈这个时候kuing还不会latex么?
Gauss门徒 4# 2013-5-5 13:14
不过kuing还真是v5连韩jj也要膜拜您啊
kuing 5# 2013-5-5 13:21
哈哈这个时候kuing还不会latex么? Gauss门徒 发表于 2013-5-5 13:12 会,但是少用,不是很熟,那时。 大概是07年底初接触latex,玩过一段时间,只是会点皮毛,后来懒了又没怎么用了,直到后来做《数学空间》才算是认真去学去用
kuing 6# 2013-5-5 13:21
不过kuing还真是v5连韩jj也要膜拜您啊 Gauss门徒 发表于 2013-5-5 13:14 显然他没看题,随便说说而已……
李斌斌755 7# 2013-5-6 12:02
1# Gauss门徒 标题
kuing 8# 2013-5-6 12:07
7# 李斌斌755 已经习惯了他的错别字。。。
thread-1485-1-1.html: [几何] 转人教一道轨迹题
李斌斌755 1# 2013-5-5 13:59
本帖最后由 李斌斌755 于 2013-5-6 09:26 编辑 已知凸四边形$ABCD$,若平面内的点$P$满足:\[S_{\triangle PAD}+S_{\triangle PBC}=\dfrac12S _{ABCD}\]求$P$点轨迹。 见 http://bbs.pep.com.cn/forum.php? ... &extra=page%3D1
李斌斌755 2# 2013-5-5 14:05
1# 李斌斌755 在人教那做得云里雾里
kuing 3# 2013-5-5 14:07
原贴里你都解得差不多啦 根据三角形面积公式,轨迹肯定是线性的,找到一些特殊点,结合面积方向,连一下……
李斌斌755 4# 2013-5-5 14:09
3# kuing 就是因为半推半猜,没有严密的推理证明!
isea 5# 2013-5-5 19:40
点幺问的问题没有不麻烦的
hongxian 6# 2013-5-6 09:16
本帖最后由 hongxian 于 2013-5-6 09:18 编辑 原贴中可是$S_{\triangle PAD}+S_{\triangle PBC}=\dfrac12 S_{ABCD}$
李斌斌755 7# 2013-5-6 09:25
6# hongxian 谢谢,已修改
hongxian 8# 2013-5-8 07:04
本帖最后由 hongxian 于 2013-5-8 07:14 编辑 到两条直线$AD$、$BC$距离成比例的点,怎么感觉在两条直线上?
李斌斌755 9# 2013-5-8 09:50
8# hongxian 先判断是$AC,BD$中点连线的直线,再分析到达$AC,BD$时不能再向外走(否则面积大于四边形面积一半),接着考虑向内走时除了该线段的点外,在$AD,BC$的另一侧还有一点(平面上与一定线段面积等于定值的点的轨迹是距该线段距离为定值的两条平行线)……继续推理得一平行四边形。
Zorn 10# 2013-5-8 16:08
一条直线
hongxian 11# 2013-5-8 21:59
本帖最后由 hongxian 于 2013-5-9 08:52 编辑 9# 李斌斌755 谢谢了,有点明白了,首先得要明白轨迹是四条线段,其次得要找到四个关键点。 补个图吧!
thread-1486-1-2.html: [数论] 这道题的解法对吗?
q1946129 1# 2013-5-5 19:55
本帖最后由 q1946129 于 2013-5-5 20:06 编辑 最近翻看一本名为《数学奥林匹克超级题库(初中卷上)》的书,里面有这样一道题:      能不能作一个正方形,它的边长是整数,并且在它所在的平面上能指出一个点,使该点到正方形的四个顶点的距离都可用整数表示?    书中给出的解答是这样的: D24.pdf (64.11 KB) 道题的这种解法对吗?
kuing 2# 2013-5-5 20:42
表示没看懂,为什么会得出“$\Delta=0$”?计算那个东东的判别式干嘛呢?它又不等于0
kuing 3# 2013-5-6 00:13
想来想去,还是不知怎么证……
realnumber 4# 2013-5-6 07:45
本帖最后由 realnumber 于 2013-5-6 07:53 编辑 同意2楼说法,又不是任意的k,表达式$k^2-2bk+l_a^2$为平方数(得出Δ=0,也需要证明;而式子是$k^2-2bk+l_a^2=(k-l_a)^2$,才能直接得到Δ=0);题目是存在k,表达式是某个数的平方,不能得出Δ=0, 比如存在k=2,$k^2-2k+4$为平方数4,但是判别式$Δ=2^2-4\times4<0$
realnumber 5# 2013-5-6 07:59
证明得继续想想,看来1946老师也来了...
kuing 6# 2013-5-6 08:03
5# realnumber 噢? 你们认识喔
realnumber 7# 2013-5-6 08:19
没碰到过,其实我也算宅;只是在人教和k12看到1946老师常发些关于数学普及类或是资料或是数学进展新闻...--后来居然给k12封了ID.
kuing 8# 2013-5-6 08:36
7# realnumber 噢,我不了解,K12也没去了,自从那个古稀老朽把数学版搞乱以后。
李斌斌755 9# 2013-5-6 09:30
7# realnumber 宅男
realnumber 10# 2013-5-6 10:20
k12冷清大部分不是那个老朽问题,不过他确实是战斗机!!
kuing 11# 2013-5-6 12:12
k12冷清大部分不是那个老朽问题,不过他确实是战斗机!! realnumber 发表于 2013-5-6 10:20 那是什么问题呢? 不过我的确是因为他才不去的,看到他没完没了地大扯特扯那些无聊问题就烦。
realnumber 12# 2013-5-6 14:04
我也不知道,有些说法觉得有道理 1.能问数学的地方比以前多,不如百度问问搜搜贴吧什么的,QQ群,博客,还有论坛也多了,一些杂志的论坛,专业论坛,大学校园网,.. 2.新课改等,只会看到非要学好数学的学生减少,也许是好现象.... 3.....干吗在网络上吵架呢,数学版上,还有弱智言论,不过觉得这个影响应该不大,处理不妥才大了,
第一章 13# 2013-5-6 14:16
表示注册了,不过几乎没去过
q1946129 14# 2013-5-6 19:39
我想,解答者是混淆了"完全平方数"和"完全平方式"这两个概念,于是就错误的运用了判别式。 经查,此题出自波兰数学科普作家斯泰因豪斯的《数学万花筒》一书(有中译本),在书中被列为尚未获解的问题之一。我没有看到过它的正确解答,很可能到现在还是一个没有解决的问题。 至于哈工大出版社出版的《奥林匹克数学竞赛超级题库初中卷》一书,质量很差,有的题目有两问居然只有一解;有一个题目的解印的是另外一个题的答案,有好几个题都出现过两三次,雷同的题就更多了。我还只是粗略的翻了一下,估计出错率会大大超过国家规定的万分之一的标准。
kuing 15# 2013-5-6 19:45
14# q1946129 “很可能到现在还是一个没有解决的问题”……看来我这数论菜鸟还是玩不来了……还好昨晚就放弃了…… 那本书我看标题就感觉很那啥…………
thread-1488-1-2.html: [几何] 平行四边形垂直垂直(from 幻)
kuing 1# 2013-5-5 22:17
我的证法很麻烦,不想写了,据说有很多证法,大家都来玩玩……
李斌斌755 2# 2013-5-6 01:29
纯几没想出来
第一章 3# 2013-5-6 07:27
向量应该可以做
kuing 4# 2013-5-6 07:52
2# 李斌斌755 我用了梅氏定理不知算不算纯几?
kuing 5# 2013-5-6 07:53
3# 第一章 噢? 写写看
李斌斌755 6# 2013-5-6 08:29
4# kuing 我不知道梅氏定理
李斌斌755 7# 2013-5-6 09:32
kuing的平几也这么难
isea 8# 2013-5-6 10:15
本帖最后由 isea 于 2013-5-6 10:37 编辑 7# 李斌斌755 昨天第一眼就看到了,觉得向量可能好入手点,竞赛类吧。 == 三线共点问题。 反过来看,以AC为直径的圆,在C的切线与圆内四边形的对角线相交于G,如图能构成平行四边形ABCD。 未想出平几之前,不会简单。 应该是竞赛类(都有Menelaus了),至少和线束交比,完全四边形脱不了干系 == 最后,kuing 平几比不等式是弱点,但还是强的 == 我感觉我见过,至少是道变式,和这道特别的像。 看了下,不是像,是一样,按我的变换命题后,AMDN便是那个四边形。 好像论坛里也有这题,人教一定有的。 又多一变式,经典题就是经典,原型是某一年中学奥林匹克竞赛题,具体忘记了
李斌斌755 9# 2013-5-6 11:52
本帖最后由 李斌斌755 于 2013-5-6 11:53 编辑 8# isea 竞赛题,坐沙发 我也用上圆,可加了轱辘也推不动呀
kuing 10# 2013-5-6 12:20
8# isea 果然是一样的,平几还是isea专业啊。
isea 11# 2013-5-6 12:34
本帖最后由 isea 于 2013-5-6 12:42 编辑 8# isea 果然是一样的,平几还是isea专业啊。 kuing 发表于 2013-5-6 12:20 不敢,此题是碰上了,眼一扫就知竞赛类,然后就不理了。 还有,这题,都被你证几次了,哈哈,我只会这个两次四点共圆,反过来(我发图题的逆命题,即主楼情形),我只会同一法,我多懒啊。 8# isea 竞赛题,坐沙发 我也用上圆,可加了轱辘也推不动呀 李斌斌755 发表于 2013-5-6 11:52 再慢慢推,不急不急
kuing 12# 2013-5-6 14:28
幻后来发来的一个证明(看上去像是命题人讲座系列里的) 牛笔了
李斌斌755 13# 2013-5-6 14:47
本帖最后由 李斌斌755 于 2013-5-6 15:27 编辑 12# kuing 比例线段用得巧啊!! http://bbs.pep.com.cn/forum.php? ... &extra=page%3D1 6#,7#应用过比例线段,可到这
李斌斌755 14# 2013-5-6 19:23
其他证法   http://bbs.pep.com.cn/forum.php? ... &extra=page%3D1
kuing 15# 2013-5-6 19:41
14# 李斌斌755 就平几来说……人教初中区比我们这里专业喔……
李斌斌755 16# 2013-5-6 19:47
15# kuing 那里“砖家”多 ,就法一看半天才明白应用直角三角形斜边高定理
thread-1489-1-1.html: TiKZ处理立体图形与几何画板一般……
isea 1# 2013-5-6 12:37
本帖最后由 isea 于 2013-5-6 12:45 编辑 麻烦…… 如,就画个正方体
kuing 2# 2013-5-6 12:44
tikz-3d, tkz
isea 3# 2013-5-6 12:46
原作者写的,还是? 另外的宏包? 我还是用2D先画着,熟悉熟悉
kuing 4# 2013-5-6 12:49
我也不清楚,没怎么接触过。
hejoseph 5# 2013-5-6 17:11
立体图形还是用asy画比较方便且效果比较好
hejoseph 6# 2013-5-6 17:31
\documentclass{article} \usepackage{tikz} \usepackage{tikz-3dplot} \begin{document} \tdplotsetmaincoords{54.7356}{120} \begin{figure}[!ht] \centering \begin{tikzpicture}[tdplot_main_coords,scale=3] \draw[dashed] (0,0,0) -- (1,0,0) (0,0,0) -- (0,1,0) (0,0,0) -- (0,0,1); \draw (1,1,0) -- (1,1,1) (1,0,1) -- (1,1,1) (0,1,1) -- (1,1,1) (1,0,0) -- (1,1,0) -- (0,1,0) -- (0,1,1) -- (0,0,1) -- (1,0,1) -- cycle; \end{tikzpicture} \end{figure} \end{document} 复制代码 利用tikz-3dplot画的正方体
thread-149-1-7.html: [不等式] 来自pep的四元轮换不等式$\sum ab/d\ge\sum2ab/(b+c)$
kuing 1# 2011-10-26 14:21
来自:http://bbs.pep.com.cn/thread-1936052-1-1.html 对于正数 $a,b,c,d$,有 \[\frac{ab}d+\frac{bc}a+\frac{cd}b+\frac{da}c\geqslant\frac{2ab}{b+c}+\frac{2bc}{c+d}+\frac{2cd}{d+a}+\frac{2da}{a+b}.\] 由均值不等式,我们有 \begin{align*} RHS(右边)&\leqslant\frac{ab}{\sqrt{bc}}+\frac{bc}{\sqrt{cd}}+\frac{cd}{\sqrt{da}}+\frac{da}{\sqrt{ab}}\\ &=a\sqrt{\frac bc}+b\sqrt{\frac cd}+c\sqrt{\frac da}+d\sqrt{\frac ab}\\ &\leqslant \frac a2\left(\frac bd+\frac dc\right) +\frac b2\left(\frac ca+\frac ad\right) +\frac c2\left(\frac db+\frac ba\right) +\frac d2\left(\frac ac+\frac cb\right)\\ &=LHS(左边). \end{align*}
pxchg1200 2# 2011-10-26 21:52
1# kuing 这个分拆很巧妙啊...
尐蒓江华 3# 2011-10-31 22:31
great
kuing 4# 2012-5-20 21:17
原来出自陈计的一般情况: http://room-365.com/bbs/forum.php?mod=viewthread&tid=464
thread-1490-1-2.html: [数列] 转13年浙江省赛一题
realnumber 1# 2013-5-6 16:03
本帖最后由 realnumber 于 2013-5-6 17:12 编辑 答案放在首页提到的群里了,或百度
thread-1491-1-1.html: 一份比较完整的数学试卷(平台Windows + CTeX + XeLaTeX)
isea 1# 2013-5-6 16:10
本帖最后由 isea 于 2013-5-7 23:20 编辑 初稿,试卷内容还未完全校对;无页眉页脚,因为偶从不用,这是主要原因。 网上基本全是Linux下的,这个抛砖引玉。 Windows7,CTeX V2.9.2.164,WinEdt V7.0,XeLaTeX(那必定是UTF8啦)。 其实是2013年海淀高二期中数学理科卷内容以选修2-1为主,就这样子。 感谢论坛各位朋友热情相助。 ========= 楼下的95.tex 即时下的 2013年北京海淀高三二模数学理科试卷
isea 2# 2013-5-7 23:18
此楼有95.tex 即2013年海淀二模理科数学卷 http://kkkkuingggg.5d6d.net/thread-1495-1-1.html 多了表格,及精确定位(tikz) 估计有一段时间不会更新此试卷样式了
李斌斌755 3# 2013-5-11 21:59
1# isea 里面的图是用什么画出来的?
isea 4# 2013-5-11 22:31
3# 李斌斌755 宏包 TiKZ http://kkkkuingggg.5d6d.net/thread-1446-1-1.html
李斌斌755 5# 2013-5-12 03:50
4# isea 看后吧敢学
thread-1492-1-1.html: 展示下asy画的3D图
hejoseph 1# 2013-5-6 17:03
本帖最后由 hejoseph 于 2013-5-6 17:21 编辑 用Adobe Reader 9.0或以上版本可以看到3D效果。
isea 2# 2013-5-6 18:02
强,膜拜
kuing 3# 2013-5-6 18:04
2# isea 需要比较新版本的latex,我这两天也得换个了……
isea 4# 2013-5-7 23:15
哇,这立体图,真不是盖的!! PDF也能看到3D,软件发展真是快
李斌斌755 5# 2013-5-28 01:54
这个能用来推三视图吗?
thread-1493-1-1.html: [几何] 圆外一定点到该圆动直径的张角何时最大?
realnumber 1# 2013-5-7 09:00
连云港-徐*(10*****96) 08:45:35 一个貌似简单的问题:圆外一定点到该圆动直径的张角何时最大?
realnumber 2# 2013-5-7 09:12
本帖最后由 realnumber 于 2013-5-7 09:15 编辑 先苯办法,再考虑有没好办法,圆不妨为圆心在(0,0)半径为1的圆,定点$P(a,0),a>1$,直径两端点$A(\cos x,\sin x),B(-\cos x,-\sin x)$, 那么AP,BP斜率为可得$k_1=\frac{\sin x}{\cos x-a},k_2=\frac{-\sin x}{-\cos x-a}$, 而$\tan{\angle APB}=\abs{\frac{k_1-k_2}{1+k_1k_2}}=\abs{\frac{2a\sin x}{1-a^2}}$,可见$x=\frac{\pi}{2}$,$\angle APB$取最大.
三下五除二 3# 2013-5-7 09:22
设定点P,圆心C,直径端点M,N,用余弦定理计算角MPN的余弦,用到一个公式:PM^2+PN^2=2(PC^2+CM^2),结合基本不等式,可以证明PM=PN时张角最大
realnumber 4# 2013-5-7 09:28
找到几何办法了,可以考虑与垂直时张角相等的点的轨迹,那么是个大圆,.....有调整法味道了
huamahu 5# 2013-5-7 09:57
设定点P,圆心C,直径端点M,N,用余弦定理计算角MPN的余弦,用到一个公式:PM^2+PN^2=2(PC^2+CM^2),结合基本不等式,可以证明PM=PN时张角最大 三下五除二 发表于 2013-5-7 09:22 正解!
李斌斌755 6# 2013-5-7 11:10
可以考虑直径不动,点动。
李斌斌755 7# 2013-5-7 11:50
本帖最后由 李斌斌755 于 2013-5-7 12:03 编辑 设$OP=a,\angle{POB}=\theta,\angle{APB}=\alpha,A(-1,0),B(1,0),P(a\cos\theta,a\sin\theta)$ \[\cos\alpha=\dfrac{(a\cos\theta+1)^2+(a\cos\theta-1)^2+2a^2\sin^2{\theta}-4}{2\sqrt{[(a\cos\theta+1)^2+\sin^2\theta]\times[{(a\cos\theta+1)^2+\sin^2\theta}]}}\]$=\cdots$
李斌斌755 8# 2013-5-7 12:06
7# 李斌斌755 证明真烦 ,选择填空直接由图像得出完了
hejoseph 9# 2013-5-7 13:38
哪里需要这么麻烦,直接用圆周角就得了
kuing 10# 2013-5-8 01:07
表示没看懂何版的…… 倒是想到另外一种方法,如图 设 $\odot O$ 的半径为 $R$,我们在 $PO$ 延长线上取一点 $Q$ 使得 $OQ\cdot OP=R^2$。 这样,根据圆幂定理,可知: $A$、$P$、$B$、$Q$ 四点共圆,记其所共的圆为 $\odot O_1$,其直径为 $d_1$; $C$、$P$、$D$、$Q$ 四点共圆,记其所共的圆为 $\odot O_2$,其直径为 $d_2$。 因为 $AB\perp PQ$,故 $PQ$ 为 $\odot O_1$ 的直径,而 $CD\not\perp PQ$,故 $PQ$ 不为 $\odot O_2$ 的直径,从而必有 $d_1=PQ<d_2$。 根据正弦定理,有 \begin{align*} \sin\angle APB&=\frac{AB}{d_1},\\ \sin\angle CPD&=\frac{CD}{d_2}, \end{align*} 故由 $AB=CD$,$d_1<d_2$ 且由 $P$ 在 $\odot O$ 外可知 $\angle APB$、$\angle CPD$ 均为锐角,即可得到 \[\angle APB>\angle CPD.\]
李斌斌755 11# 2013-5-8 01:26
10# kuing 9#已说。\[\angle BAC=\angle BDC>\angle BEC\]
kuing 12# 2013-5-8 01:29
11# 李斌斌755 噢,原来是延着7#说的,懂了……
李斌斌755 13# 2013-5-8 09:57
10# kuing 弱弱问一下,何版是……
isea 14# 2013-5-8 10:17
10# kuing 弱弱问一下,何版是…… 李斌斌755 发表于 2013-5-8 09:57 9楼 好像是研究几何方面的 在LaTeX方面给偶不少帮助
thread-1494-1-2.html: [函数] 混合组中,变量的范围
realnumber 1# 2013-5-7 09:34
江苏-冯**(12****996)  09:30:51
李斌斌755 2# 2013-5-7 11:06
本帖最后由 李斌斌755 于 2013-5-8 20:59 编辑 要分类讨论
thread-1495-1-1.html: [数列] 2013年海淀二模理科第8题 数列相关 (附全卷及源文件)
isea 1# 2013-5-7 15:38
本帖最后由 isea 于 2013-5-9 22:45 编辑 录试卷呢(源与打印文档均见附件,已校对,改了几个错别字,其他没问题),还没认真看,给大家看看 若数列$\{a_n\}$满足:存在正整数$T$,对任意正整数$n$ 都有$a_{n+T}=a_n$成立,则称数列$\{a_n\}$为周期数列,周期为$T$.已知数列$\{a_n\}$满足$a_1=m(m>0), a_{n+1}=\left\{\begin{aligned}   &a_n-1,&a_n>1,\\&\dfrac 1{a_n},&0<a_n\le 1.\end{aligned}\right.$   则下列结论中错误的是 A. 若$a_3=4$,则$m$可以取3个不同的值 B. 若$m=\sqrt 2$,则数列$\{a_n\}$是周期为3 的数列 C. $\forall T\in \mathbb{N}^*\text{且}T\ge 2,\exists m>1$,使得$\{a_n\}$是周期为$T$ 的数列 D. $\exists m \in \mathbb{Q}\text{且}m\ge 2$,使得数列$\{a_n\}$是周期数列 第8题,原图。
isea 2# 2013-5-7 17:12
选项按排有些不合理,胆子大点,B就是C的一个例子,从而直接选D,而正确结果亦是D 但是对C,还未严格证明 再说
零定义 3# 2013-5-7 19:04
对于C,今晚晚一点我来给出证明...先忙去了...
地狱的死灵 4# 2013-5-7 20:15

零定义 5# 2013-5-7 22:01
4# 地狱的死灵 我也是这样想的~
isea 6# 2013-5-7 22:12
你们都太牛了! 谢谢解答啊,学习学习,先
李斌斌755 7# 2013-5-8 02:04
4# 地狱的死灵 推理能力强
isea 8# 2013-5-8 12:46
本帖最后由 isea 于 2013-5-8 13:21 编辑 1483 地狱的死灵 发表于 2013-5-7 20:15 妙啊,又细细体会一把,标上楼主大名 地狱的死灵 转走了啊 $T^2-2T+5$ 不是完全平方数 正好亦说明D了 === LaTeX 代码(论坛): 8.(2013海淀)若数列$\{a_n\}$满足:存在正整数$T$,对任意正整数$n$ 都有$a_{n+T}=a_n$成立,则称数列$\{a_n\}$为周期数列,周期为$T$.已知数列$\{a_n\}$满足$a_1=m(m>0), a_{n+1}=\left\{\begin{aligned}&a_n-1,&a_n>1,\\&\dfrac 1{a_n},&0<a_n\le 1.\end{aligned}\right.$则下列结论中错误的是      A. 若$a_3=4$,则$m$可以取3个不同的值      B. 若$m=\sqrt 2$,则数列$\{a_n\}$是周期为3 的数列      C. $\forall T\in \mathbb{N}^*$且$T\ge 2,\exists m>1$,使得$\{a_n\}$是周期为$T$ 的数列      D.$\exists m \in \mathbb{Q}$且$m\ge 2$,使得数列$\{a_n\}$是周期数列 分析与解:      如果胆子大点,这题都不用想,直接选择D了,这就是这题结果正是选择D。对于A,按题目所给的递推条件,分二种情况分别计算,如,$3=a_3=a_2-1$,或者$3=a_3=\dfrac 1{a_2}$;B项与A类似处理。下面着重看看选项C(解法来自 地狱的死灵,感谢):      现在只需要证明,当$\forall T\in \mathbb{N}^*,T\ge 2,\exists m\in (T-1,T)$,使得$a_{1+T}=a_1$。 \begin{align*}    \because m\in (T-1,T),T\ge2 ,& \therefore  m-T<0,m>T-1\ge1  \\&\therefore   m-T+1<1\\   &\left\{\begin{aligned}   &a_T=a_{T-1}-1\\   &a_{T-1}=a_{T-2}-1\\   &\ldots\\   &a_3=a_2-1\\   &a_2=a_1-1   \end{aligned}\right\}(T-1)个   \end{align*} 将这个$T-1$等式相加有: \begin{align*}    a_T  &=a_1-(T-1)\\   &=m-T+1\\   &<1\\   &\therefore a_1= a_{T+1}=\dfrac {1}{a_T}=\dfrac 1{m-T+1}=m   \end{align*} 即 \begin{align*}   &m^2-(T-1)m-1=0   \end{align*} 解得 \[ m=\dfrac {T-1\pm\sqrt{T^2-2T+5}}2   \] 而取$m=\dfrac {T-1+\sqrt{T^2-2T+5}}2$时 \begin{align*}   T-1=\dfrac {2(T-1)}2&<m=\dfrac {T-1+\sqrt{T^2-2T+5}}2\\   & <\dfrac {T-1+\sqrt{T^2-T^2+5}}2 \ (\because T\ge2)\\   &<\dfrac {T-1+\sqrt 5}2\\&<T   \end{align*} 即,当$\forall T\in \mathbb{N}^*,T\ge 2,\exists \ m\in (T-1,T)$,使得$a_{1+T}=a_1$。这样,此题不正确的项便是D了。
realnumber 9# 2013-5-8 14:14
8# isea 否定D觉得用反证法,因为不排除还有别的出现周期的方式,比如先减4次,倒数,再减3次,倒数,...构成周期, 假设有理数m存在,设$m=\frac{q}{p},(p,q)=1,p,q\in Z^+,p>2q$, 若$a_k>1,a_{k+1}<1,$($a_k=1或a_{k+1}=1$,容易得到$n\ge k+1,a_n=1,$不合周期定义),记$a_{k+1}=\frac{t}{p},t< p,(t,p)=1,t\in Z^+$,$a_{k+2}=\frac{p}{t}$,...,可见数列中的分式,分母一直变小,一直到1,1,1,..1,不符合周期要求.
isea 10# 2013-5-8 15:46
9# realnumber 正对此严格证明苦恼,先标记
tan9p 11# 2013-5-8 22:32
赞楼主的工作,想问下里面的插图是自己画的还是截的。 另外,我觉得把题目一道一道切割开保存成题库比较好,不知道有没有人有兴趣作这项工作。 我想做,但技术上不行。 想法是->将pdf或doc 版本的自动切割,公式转化 进入数据库。
isea 12# 2013-5-9 00:31
赞楼主的工作,想问下里面的插图是自己画的还是截的。 另外,我觉得把题目一道一道切割开保存成题库比较好,不知道有没有人有兴趣作这项工作。 我想做,但技术上不行。 想法是->将pdf或doc 版本的自动切割,公式转 ... tan9p 发表于 2013-5-8 22:32 LaTeX宏包,用TiKZ自己画的,tex格式就是源文件。 有源文件,想怎么整都行,一点兴趣与爱好,不商用也不感兴趣,随便转载,随便改,但是有了麻烦不要找偶就成,哈哈。
isea 13# 2013-5-9 11:06
8# isea 否定D觉得用反证法,因为不排除还有别的出现周期的方式,比如先减4次,倒数,再减3次,倒数,...构成周期, 假设有理数m存在,设$m=\frac{q}{p},(p,q)=1,p,q\in Z^+,p>2q$, 若$a_k>1,a_{k+1} realnumber 发表于 2013-5-8 14:14 明白大约意思了,但还是理解不了,有些。
Tesla35 14# 2013-5-9 21:30
数列题! 感谢楼主的源文件!!
Tesla35 15# 2013-5-9 21:43
楼主是用CTex编译的么?XeLatex编译不通过啊。。楼主用的什么版本么,,
kuing 16# 2013-5-9 22:04
15# Tesla35 先看看你系统有没有源文件设置里的那些字体
isea 17# 2013-5-9 22:31
楼主是用CTex编译的么?XeLatex编译不通过啊。。楼主用的什么版本么,, Tesla35 发表于 2013-5-9 21:43 嗯,CTeX v2.9.2.164,XeLatex,多半是没字体CMU Serif,见 19楼 http://kkkkuingggg.5d6d.net/thread-1305-2-1.html 然后Adobe Kaiti Std,Adobe Heiti Std,Adobe Song Std 这三种Adobe字体,若没有,网上下之。 最后一种字体是 Cambria Math,这个一般都会有。
kuing 18# 2013-5-9 22:35
17# isea 刚才在群里看到他连 \usepackage[inline]{enumitem} 都报错,提示说未知选项 inline,估计系统太旧了……
isea 19# 2013-5-9 22:43
18# kuing 可能。 这个容易全删除了,只是没标事情而已。 LaTeX排错也是麻烦的,LaTeX的通用性稍差啊。
thread-1496-1-2.html: 2013年海淀二模理科第20题 数表 调正负号
isea 1# 2013-5-7 15:41
本帖最后由 isea 于 2013-5-7 22:59 编辑 这题似乎简单了点,20题的数表倒是有点意思。 ==================================== (前言同 2013年海淀二模理科第8题 数列相关 ) 题: 在平面 直角坐标系中,动点$P(x,y)$到两条坐标轴的距离之和等于它到点$(1,1)$的距离,记点$P$的轨迹为曲线$W$. a)给出下列三个结论:                     (1)曲线$W$关于原点对称;                     (2)曲线$W$关于直线$y=x$对称;                     (3)曲线$W$与$x$轴非负半轴,$y$轴非负半轴围成的封闭图形的面积小于$\dfrac 12$. 其中,所有正确结论的序号是_________. b)曲线$W$上的点到原点距离的最小值为_______. 原题图,14题
isea 2# 2013-5-7 17:10
此题比想像中简单多了,直接列式,平方,用x表示y,OK。 结果第一空 (2)(3);第二问 $2-\sqrt 2$
kuing 3# 2013-5-7 23:45
为什么不用 table、caption 来自动生成“表X”…… PS、要不要那么多回车……
isea 4# 2013-5-8 00:12
3# kuing 因为不会,哈哈,我现在是去学一下
李斌斌755 5# 2013-5-8 00:43
20题看不懂题意
isea 6# 2013-5-8 00:56
本帖最后由 isea 于 2013-5-8 01:00 编辑 20题看不懂题意 李斌斌755 发表于 2013-5-8 00:43 可能打掉某些关键的字,我查一下。 === 第三行的确少了一个字 “使得的数每行”少一个“表”,是“使得的数表每行”
isea 7# 2013-5-8 01:01
本帖最后由 isea 于 2013-5-8 01:06 编辑 有标答了,全部上传了好了
isea 8# 2013-5-8 01:07
好坑爹,论坛写的附件不能超过512K 实际上好像比300K都小,只好分包了
kuing 9# 2013-5-8 01:14
8# isea 是 5d6d 坑我……我最近进后台才发现被砍了一半
isea 10# 2013-5-8 01:25
9# kuing 问题是他不提示你附件超了,直接忽略附件而只发文字 如果不想到是附件超大的话,还真不知道怎么回事呢。
kuing 11# 2013-5-8 01:31
10# isea yeah, that's very keng!
thread-1497-1-2.html: [几何] 等腰求角度
李斌斌755 1# 2013-5-7 22:37
已知$\triangle ABC,AB=AC,\angle BAC=40^\circ,\angle PAB=10^\circ,\angle PBA=40^\circ$,求$\angle PCB$
kuing 2# 2013-5-7 22:47
又是这种类型啊……
isea 3# 2013-5-7 23:20
他最近迷上这些了,这些的确很迷人
零定义 4# 2013-5-7 23:29
本帖最后由 零定义 于 2013-5-7 23:31 编辑 坑爹的题,唯有用坑爹的方法了...
李斌斌755 5# 2013-5-8 00:20
错题一个 ,大伙不管它了
kuing 6# 2013-5-8 00:40
不能叫错题……可以求的,只不过最后的方程估计超越了。
李斌斌755 7# 2013-5-8 00:46
6# kuing 这类几何题要整数解,否则失去其优美性
kuing 8# 2013-5-8 00:47
7# 李斌斌755 那顶多就叫丑题,而不是错题……
thread-1498-1-1.html: 今天终于换成texlive……
kuing 1# 2013-5-7 22:42
RT…… 之前的太老了,越来越多问题,不过由于一向太懒,所以一直没换,直到今天。 texlive并没有配置winedt、sumatrapdf之类的东西,不过自己整一下很简单。 之前一直用winedt6,今天换成了winedt7,为什么不换别的?因为要延用当年自定义的快捷键以及这堆东东: 下一步,配置asy...这样才能编译何版的那些3D图,现在先休息一下。
isea 2# 2013-5-7 22:46
你们先,我垫后
isea 3# 2013-5-7 23:01
\usepackage{asymptote} 在CTEX倒是看到很多人说这个 又一个新东东,不愿意学啊
isea 4# 2013-5-7 23:07
texlive2012-20120701.iso        02-Jul-2012 00:25         2.2G 2.2G 跟影视720P相比只有一半都不到,不过,对偶来说,这个太大了 浪费功能啊
kuing 5# 2013-5-7 23:08
3# isea 对,就是这个东东。 4# isea 对,今天下的就是这个东东。
isea 6# 2013-5-7 23:29
本帖最后由 isea 于 2013-5-7 23:30 编辑 我这个,0day 就是强大。
kuing 7# 2013-5-8 16:34
搞定,asymptote 牛笔 编译了何版的几个文档,都OK
thread-1499-1-1.html: 都差不多
isea 1# 2013-5-7 23:25
本帖最后由 isea 于 2013-5-8 00:24 编辑 本想说点什么的,好像是打小型数学试卷,话到嘴边忘记了 不过,差别也大与风格迥异,要学的东东也多啊,好用,认了 别怪我不告诉你
isea 2# 2013-5-8 01:26
睡觉 晚安
thread-15-1-1.html: M8.0 与 M8.0.1 有何分别?
kuing 1# 2011-9-26 19:24
据说 Mathematica8.0 之后又出了 Mathematica8.0.1 ,有何不同?
①②③④⑤⑥⑦ 2# 2011-9-29 15:32
版本号只动了最后一位,估计没啥大变化,修正一些bug吧,或许枪毙掉一些非法Key
kuing 3# 2011-10-18 14:07
其实我还在用 M7,感觉够用了就懒得升级
thread-150-1-1.html: 大段东东注释
kuing 1# 2011-10-26 14:56
在latex的源文件里,我们都知道可以用 % 来注释一行文字,即该行在 % 后的东东都不予编译,但如果是一大堆东东,每行都在前面加 % 也很麻烦,这时我们可以用 \iffalse …… …… …… \fi 这样被包含的部分也作注释,不予编译。
isea 2# 2013-3-29 15:56
CTeX + WinEdt 可写完之后,选中注释部分,右键——Insert Commont
kuing 3# 2013-3-29 16:02
2# isea oh,这个我还不知道…… 那么恢复不注释自然就是下一个选项了吧,Remove什么
isea 4# 2013-3-29 16:58
3# kuing 对,“望文生义”
yes94 5# 2013-3-30 18:54
还是这个版块热闹
thread-1500-1-1.html: 一亩三分
李斌斌755 1# 2013-5-8 01:33
若有一亩三分薄地,回家玩纯几。
李斌斌755 2# 2013-5-8 03:32
收工, 睡觉。
isea 3# 2013-5-8 13:26
这个都不够吃了,现在
李斌斌755 4# 2013-5-8 13:37
3# isea 我个小,吃不多
isea 5# 2013-5-9 00:34
4# 李斌斌755 我六百帖了,天
kuing 6# 2013-5-9 00:48
5# isea 我哩……看看先
李斌斌755 7# 2013-5-9 04:02
6# kuing 你的不能算,我们的才算
thread-1501-1-1.html: [函数] 两函数上点,距离最小
realnumber 1# 2013-5-8 14:59
浙江叶**(12***81) 14:54:11 请教一下这道题怎么解 ______kuing edit in $\LaTeX$______ 已知 $a>0$,$b>0$,$f_1(x)=\dfrac x{1+ax}$($x<-\dfrac1a$),$f_2(x)=\dfrac x{1+bx}$($x>-\dfrac1b$),$A$、$B$ 分别是 $f_1(x)$、$f_2(x)$ 的两点,求 $\abs{AB}$ 的最小值。
realnumber 2# 2013-5-8 15:22
可能可以这么做,两点连线垂直这两点处的切线.好象计算有些...
realnumber 3# 2013-5-8 15:40
还是表示不会~~`
李斌斌755 4# 2013-5-8 18:54
感觉是同一函数$f(x)=\dfrac x{1+qx}$在不同区间的变化,$a,b$是等价关系……
李斌斌755 5# 2013-5-8 19:04
或者换言之函数可以表示为\[f(x)=\begin{cases}\dfrac x{1+qx}& x<q=-\dfrac1{a}\\\dfrac x{1+qx}& x>q=-\dfrac1{b}\end{cases}\]
kuing 6# 2013-5-8 20:11
4# 李斌斌755 直觉告诉我并不是这样的。
李斌斌755 7# 2013-5-8 20:32
本帖最后由 李斌斌755 于 2013-5-8 20:33 编辑 奇怪,作图$B$点恒过原点,当$a\to\infty$时$A$点可无限接近原点,$|AB|\to0$ 坐沙发
isea 8# 2013-5-9 23:09
我也认为AB趋近于零
kuing 9# 2013-5-9 23:12
题目的意思不是a,b是给定的?
isea 10# 2013-5-9 23:24
9# kuing 嗯,有道理,原来是要用$a,b$来表示这个长度,我让其中一个定下来了,另一个变化了 审题不细啊
kuing 11# 2013-5-9 23:34
10# isea 其实出题的应该尽量避免这些可能的误解发生,只要在前面加“给定”或者“常数”之类的词进去就能避免。
isea 12# 2013-5-9 23:35
本帖最后由 isea 于 2013-5-10 09:03 编辑 好复杂的图象. 还不如直接$a=1,b=2$特殊值看看,先
isea 13# 2013-5-9 23:40
我觉得,这题要么是竞赛题中,要么是题中的一部分,这个相当复杂的感觉。
kuing 14# 2013-5-9 23:41
13# isea 我暂时没打算碰它
李斌斌755 15# 2013-5-10 01:07
本帖最后由 李斌斌755 于 2013-5-10 01:11 编辑 抛砖引玉,代数不会。$f_1(x)=\dfrac x{1+ax}$图像的渐近轴为$y=0,x=-\dfrac1a$,顶点坐标分别为$(-\dfrac2a,\dfrac2a),(0,0),x<-\dfrac1a$时是图像的左支,顶点坐标为$(-\dfrac2a,\dfrac2a)$ $f_1(x)=\dfrac x{1+bx}$图像的渐近轴为$y=0,x=-\dfrac1b$,顶点坐标分别为$(-\dfrac2b,\dfrac2b),(0,0),x>-\dfrac1b$时是图像的右支,顶点坐标为$(0,0)$ 由图像知\[|AB|_{\min}=\dfrac{\sqrt8}a\]与$b$无关
李斌斌755 16# 2013-5-10 01:10
12# isea 我的图像画错了?
isea 17# 2013-5-10 09:04
16# 李斌斌755 我本是把分子分比除了x的,结果忘记把分子变成1 我画错了
李斌斌755 18# 2013-5-10 13:27
楼主该公布答案了。
realnumber 19# 2013-5-14 16:32
我不会,并怀疑有初等解,是群里看到转的,说是今年杭州高复的模拟卷中题目.
李斌斌755 20# 2013-5-14 16:38
19# realnumber 高等解法呢?
thread-1501-2-1.html:
realnumber 21# 2013-5-15 08:51
我也是猜测的,我想表达的是用初等函数表示的解,有限个,有限次复合.
thread-1502-1-1.html: [几何] 圆半径定长 圆上一点到两点距离和最小
isea 1# 2013-5-8 15:50
本帖最后由 isea 于 2013-5-10 21:06 编辑 想了几分便停手了,从几何角度上看,卡住了; 解析方向未想,因为这是海淀初三一模题。 题来了:第三问。请教,感谢先。 注意AB是$y=x+2$,其它的没什么用,可以直接看第三问了。 ================= 解答,高手直接看13楼或18楼即可。
isea 2# 2013-5-8 16:30
Apollonius 圆吧 转化这个$\dfrac{\sqrt 2}2$有得玩了 我就不用电脑画图,不信了
isea 3# 2013-5-8 16:56
本帖最后由 isea 于 2013-5-8 17:04 编辑 问题转化成为: 点$M$到$\triangle BB'M$三个顶点的距离相关,即求$2+QB'+\dfrac{\sqrt 2}{2}QB$最小,其中$QM=2$ 感觉这个方向比阿氏圆好些 人教论坛 好像有讨论过,$\triangle ABC,xPA+yPB+zPC$最小的问题吧,标准竞赛题了 闪,不理,先
kuing 4# 2013-5-8 18:25
你说得没错啊,的确是阿氏圆。 目测+猜测:$A$ 关于 $M$ 的对称点 $A'$ 满足 $QB=\sqrt2 QA'$。 如果成立,就完事了。 我想,也只有存在这样的 $A'$,这题才可做,否则估计就会高次方程。
李斌斌755 5# 2013-5-8 19:33
可这是初中题啊,海淀想干嘛
isea 6# 2013-5-8 19:39
4# kuing 是这样,就是MB和中点 可这是初中题啊,海淀想干嘛 李斌斌755 发表于 2013-5-8 19:33 竞赛的感觉,海淀正统的风格
李斌斌755 7# 2013-5-8 19:43
6# isea 我不知道阿氏圆
李斌斌755 8# 2013-5-8 19:51
7# 李斌斌755 自己科普一下   http://baike.baidu.com/view/1613920.htm   http://wenku.baidu.com/view/df22b8e8172ded630b1cb6ed.html
isea 9# 2013-5-8 23:05
2楼与3楼的想法是最直接第一想法,也应是通法,但从两问题入手都很难,并不通俗。 故从几何动点最值,及特殊的等腰直角三角形入手,并借助于几何画板精确作图,得到下面的方法。 先上图片版 文字版: 解题过程探索及分析与解: 首先,将√(2)QB/2(或√(2)BQ' )构造出来,故以QB以斜边向右下作等腰直角三角形QBQ',这样QB'+√(2)QB/2=QB'+QQ'≥B'Q', 通常情况下B'Q'即为所求,刚开始偶就被这个误导浪费时间了。但Q'在此题中不是定点,由三角形三边关系,这里无法取得 最小。需要转化,需要研究Q' 的轨迹,而这个,偶非常熟悉(唯熟悉尔),Q' 亦定圆上,理由很简单: 以MB再向右下构造等腰直角三角形MDB,由M,B,C三者坐标关系,知D在半径为2的圆M上,且BD,BB' 均为圆M的切线, 这个位置关系下,图形关系就变得十分的“简单”与清晰。 再理顺一下三个等腰直角三角的关系: 他们(△BQQ',△BB'M,△BMD)共45°度的顶点B,同向相似。 考虑到圆M的半径为2(=MQ),于是,将△BQM'绕B点逆时针旋转45°并缩小√(2)倍,即得到△BQ'D,即图中两个阴影三角形 是相似的。 于是DQ'=MQ/√(2)=√(2),换句话说,Q在半径为2圆M上运动,Q' 就在半径为√(2)的圆D上运动。 现在,前面提到的QQ' 再次转化!将Q' 关于QD对称!即图中圆D上的Q''点。 诸位,这个数据真是巧得很啊!DQ''=√(2)=BD/√(2),∠MBD=45°,得到等腰Rt△BQ''D!我怎么这么高兴?Q''是定点啊。 Q运动得到Q' 在圆D上运动,但是Q' 关于QD的对称点Q'' 是定点,MQ的中点是也!(顺便提一下,更巧的是,A点关于M点 的对称点就是Q'' ) 从而:QB'+QB√(2)/2=QB'+QQ'=QB'+QQ''≥B'Q'=√(9+1)=√(10)。 最后,我还是要说一下,没借助高科技(如这里的几何画板作图)情形下,只是想了个七七八八,准确作图后,直接印证 的我的想法与思考过程。 无精确图形,利用复数与向量解决此类题是通法。 =========== PS:懒了很久没动几何画板了
李斌斌755 10# 2013-5-8 23:21
本帖最后由 李斌斌755 于 2013-5-8 23:23 编辑 9# isea 问题的关键就在$Q'$点在定圆$D$上,初中知识能解答
isea 11# 2013-5-8 23:23
本帖最后由 isea 于 2013-5-9 00:18 编辑 下面,来严格证明Q''是定点 其实,发现取MB的中点,完全可以寻找解题过程的写法。 解题过程探索及分析与解续: 正如前,借用电脑高科技手段往往是有效的。但是为什么Q'' 是定点呢?确实,这不是直观的,一眼能简单的看出来的。事实上, 我们取MB的中点q (注意哦,小写字母也是能表示点的,如英国 A. 科克肖特, F. B. 沃尔特斯 写的名著圆锥曲线的几何性质,英 文名 A Treatise on Geometrical Conics)。 则由边角边关系立刻有△QMq≅△BDQ',即图中两个阴影三角形,于是Qq=BQ'=Q'Q;另一方面,D到BM的距离为√(2),即MB为 圆D的切线,再考虑到等腰Rt△BMD,即q为其切点。所以点q 即Q''点。 可能说得太快,是这样的,△QqD≅△QQ''D(SSS),即Q''为MB的中点q(定点)。
李斌斌755 12# 2013-5-9 00:07
11# isea 问题转化为动点$Q$到两定点$B',Q'$的距离和,妙!学习。
isea 13# 2013-5-9 00:24
11# isea 问题转化为动点$Q$到两定点$B',Q'$的距离和,妙!学习。 李斌斌755 发表于 2013-5-9 00:07 简化过程,(初中几何上基本都是这样子啦,只是这题难些,还要再转化一次。 再如:http://kkkkuingggg.5d6d.net/viewthread.php?tid=1256,如也回了的) 回顾一下以上的过程,我们完全可以其书写过程简化。 MQ(^2)=4=√(2)*2√(2)=MQ''*MB,熟悉切割线逆定理的,立刻知道∠MQQ''=∠Q''BQ,有△QMQ''∼△BMQ。 于是QQ''/BQ=QM/BM=2/2√(2)=√(2)/2,即QQ''=√(2)BQ/2, 这样直接就有QB'+QB√(2)/2=QB'+QQ''≥B'Q'=√(9+1)=√(10)。 简单吧,其实思维过程的展现就如上了。转化系数为1,再利用三角形边的关系,化为熟悉的情形。 新课标将切割定理放到了高中,选修4-1,现在初中没这个东东,不知道这个也没关系,直接将积化 比,MQ(^2)=4=√(2)*2√(2)=MQ''*MB,有: (MQ/MQ'')=(MB/MQ),又∠M=∠M,也得到△QMQ''∼△BMQ,以下过程略去。
isea 14# 2013-5-9 00:36
9# isea 问题的关键就在$Q'$点在定圆$D$上,初中知识能解答 李斌斌755 发表于 2013-5-8 23:21 能D为定点,DQ'为定长根2,圆的定义。
李斌斌755 15# 2013-5-9 00:53
本帖最后由 李斌斌755 于 2013-5-9 03:55 编辑 13# isea 这个好理解,但难想到。 9#等价于圆外一点$A,AO=\sqrt2R,P$为圆上一动点,以$P$为圆心,$R_1=\dfrac{\sqrt2}2AP$为半径的圆恒过$AO$中点$D$。 代数证明,圆方程\[x^2+y^2=R^2\]$P(R\cos\theta,R\sin\theta)\\A(R,R)\\D(\dfrac R2,\dfrac R2)$ \[AP=\sqrt{(R\cos\theta-R)^2+(R\sin\theta-R)^2}\\=R\times\sqrt{3-2(\sin\theta-\cos\theta)}\\DP=\sqrt{(R\cos\theta-\dfrac R2)^2+(R\sin\theta-\dfrac R2)^2}\\=\dfrac{\sqrt2}2\times\sqrt{2(R\cos\theta-\dfrac R2)^2+2(R\sin\theta-\dfrac R2)^2}\\=\dfrac{\sqrt2}2R\times\sqrt{3-2(\sin\theta+\cos\theta)}\\=\dfrac{\sqrt2}2AP\] 问题:定圆$O$,半径为$R$,$P$为圆上一动点,圆外一定点$A,AO=k\times R(K>1)$,能否找到一定点$D$,使$AP=t\times PD$($t$为常数),若$D$点存在,$PD$与圆$O$是否有交点。 是不是4#kuing说的只有$k$是特殊值时……否则解高次方程,那么特殊值有那些,除$\sqrt2$外还有没有,可否给个证明。
isea 16# 2013-5-9 01:01
15# 李斌斌755 整个过程就是这样发现的,全写出来了,估计不会有第二次了 主要是正好今天又准备这个试卷,就完整做一次,临场卡住,就没多想。 正好在百度blog有人问这个,so……
李斌斌755 17# 2013-5-9 11:43
本帖最后由 李斌斌755 于 2013-5-9 11:51 编辑 接15# 另一角度$QB'+\dfrac{\sqrt2}{2}QB=\dfrac{\sqrt2}2(QB'+QB)$,是否存在另一定点$E$,使$QE+EB$为定值?
李斌斌755 18# 2013-5-9 12:12
本帖最后由 李斌斌755 于 2013-5-9 12:15 编辑 17# 李斌斌755 $B_1D=\sqrt{10},BE=\sqrt{20}$,$D$在圆$B_1$上,$E$是否在圆$B$上。$E$点的轨迹……
isea 19# 2013-5-10 21:06
把A带上,源自学生的另解
Gauss门徒 20# 2013-5-17 00:35
物吧初三一妹子向量做了30分钟做出
thread-1502-2-1.html:
李斌斌755 21# 2013-5-17 00:50
20# Gauss门徒 初三,向量
Gauss门徒 22# 2013-5-18 15:35
21# 李斌斌755 她自学的。我百度了一下,貌似中考可以使用高中数学??!尼玛!!!!。。。。。
isea 23# 2013-5-18 21:55
22# Gauss门徒 发个链接来看看
thread-1503-1-1.html: 刚没写完
isea 1# 2013-5-9 00:33
现在写完了,写了一个完整的思考过程,很啰嗦的
thread-1504-1-1.html: [不等式] 又一道似曾相识的(from 幻)
kuing 1# 2013-5-9 00:55
少女幻 2013-5-8 11:49:27 下面这个怎么证明? (b+c)^2/(a^2+bc)+(c+a)^2/(b^2+ca)+(a+b)^2/(c^2+ab)>=6 正数 题目:设 $a$, $b$, $c$ 为正数,求证 \[\frac{(b+c)^2}{a^2+bc}+\frac{(c+a)^2}{b^2+ca}+\frac{(a+b)^2}{c^2+ab}\geqslant6.\] 又是似曾相识,又是没翻到贴子…… 可是这次自己证不出了,SOS的话易知等价于 \[\sum(a+b)(a+b-c)(ab+c^2)(a-b)^2\geqslant0,\] 然后也没整出来…… 时间关系,你们先玩,或者给点链接,我明天再瞧瞧……
isea 2# 2013-5-9 00:59
这个真心不认识 看不懂这种简写 晕~
kuing 3# 2013-5-9 01:34
2# isea 在不发生混淆的情况下通常约定 $\sum f(a,b,c)=f(a,b,c)+f(b,c,a)+f(c,a,b)$。
pxchg1200 4# 2013-5-10 01:00
1# kuing 由Cauchy-Schwarz,我们有 \[ \sum{\frac{(b+c)^2}{a^2+bc}}=\sum{\frac{(b+c)^4}{(a^2+bc)(b+c)^2}}\geq \frac{\left(\sum{(a+b)^2} \right)^2}{\sum{(a^2+bc)(b+c)^2}}\] 以下略。
kuing 5# 2013-5-10 13:16
4# pxchg1200 原来是这样柯……我咋没想到哩……
pxchg1200 6# 2013-5-10 23:14
5# kuing 这个还有额外的隐藏取等$a=b,c=0$所以由CYH技术,就是配那个啦。
thread-1505-1-1.html: [几何] 翻人教群聊天记录看到wwd问的这个斜率比为定值
kuing 1# 2013-5-9 01:24
由于这几天人教数学群里扯谈的太多鸟,所以今天几乎没怎么看群。刚才才略翻了一下聊天记录,发现wwd发的这个: 但是群里似乎没人鸟他。 再往上看好像是他从别的题中发现的东东,仍然由于时间关系,你们先玩……
李斌斌755 2# 2013-5-9 02:25
几何画板发现$QP$交$AB$于一定点。
kuing 3# 2013-5-9 07:45
几何画板发现$QP$交$AB$于一定点。 李斌斌755 发表于 2013-5-9 02:25 这一点倒是显然,而且即使换成椭圆,只要$F_1$、$D$为定点(距离不必为半径),那个交点都会是定点,只要用一个过定点的已知结论就可以推出。 但是这对1#原题的求解不知有没有帮助。
isea 4# 2013-5-9 11:08
3# kuing 这两定点到底什么意思? 如果动起来,是函数关系?
kuing 5# 2013-5-9 12:14
4# isea 圆是半径为 $R$ 的定圆,$F_1$、$D$ 为直径 $AB$ 上的定点且 $F_1D=R$,然后其他那些线就如图所示那样连,让 $M$ 动,引起其他那些线也动,证明斜率比为定值。
李斌斌755 6# 2013-5-10 01:58
这个实在想不出几何法
李斌斌755 7# 2013-5-10 14:13
本帖最后由 李斌斌755 于 2013-5-10 14:17 编辑 纳闷……如果图中$CD$是个定值 $ME\perp DF_1,PF\perp BD$,$BD$交$PQ$于$C$\[\triangle MED\sim\triangle PFD\riff\dfrac{ME}{ED}=\dfrac{PF}{FD}\\\dfrac{k_1}{k_2}=\dfrac{\dfrac{ME}{ED}}{\dfrac{PF}{FC}}=\dfrac{\dfrac{PF}{FD}}{\dfrac{PF}{FC}}=\dfrac{FC}{FD}\] $\dfrac{FC}{FD}$是个变量
李斌斌755 8# 2013-5-10 14:26
本帖最后由 李斌斌755 于 2013-5-10 14:27 编辑 7# 李斌斌755 分式的横线粗细不一样?还每次打开都在变化!
kuing 9# 2013-5-10 14:29
7# 李斌斌755 $k_1$ 错 8# 李斌斌755 浏览器问题
李斌斌755 10# 2013-5-10 14:35
本帖最后由 李斌斌755 于 2013-5-10 14:36 编辑 9# kuing 没发现一直想了几天。再想想……
isea 11# 2013-5-11 00:04
查阿氏圆的圆心确定时,看这个,类似,但更几何化些,毕竟斜率不好找几何意义
李斌斌755 12# 2013-5-19 01:11
这个谁给证明或否定!
hejoseph 13# 2013-5-20 13:34
设圆为$x^2+y^2=1$,点$D$为$(x_0,0)$,则点$F_1$坐标为$(x_0-1,0)$,容易得$PQ$过定点$F_2\left(\dfrac{x_0^3-x_0^2-x_0-1}{x_0^2-2x_0-1},0\right)$。设过定点$D$的两直线斜率分别为$k_1$、$k_2$,则过这两条支线与单位圆的四个交点的所有圆锥曲线(包含退化情形)的曲线方程是 \[ x^2+y^2-1+t\left(y-k_1(x-x_0)\right)\left(y-k_2(x-x_0)\right)=0\text{,} \] 即 \begin{equation} (k_1k_2t+1)x^2-(k_1+k_2)txy+(t+1)y^2-2k_1k_2x_0tx+(k_1+k_2)x_0ty+k_1k_2tx_0^2-1=0\text{。}\label{eqn1} \end{equation} 要使其方程表示两直线,并且一直线过点$F_1$,另一直线过点$F_2$,则必定方程可写成 \[ (y-k_3(x-x_0+1))\left(y-k\left(x-\frac{x_0^3-x_0^2-x_0-1}{x_0^2-2x_0-1}\right)\right)=0\text{,} \] 即 \begin{align} & kk_3x^2-(k+k_3)xy+y^2\notag\\ & -\frac{2kk_3(x_0-1)x_0^2}{x_0^2-2x_0-1}x-\frac{(k+k_3)x_0^3-(k+3k_3)x_0^2-(k-k_3)(x_0+1)}{x_0^2-2 x_0-1}y+\frac{kk_3(x_0-1)(x_0^3-x_0^2-x_0-1)}{x_0^2-2x_0-1}\notag\\ ={}&0\text{。}\label{eqn2} \end{align} 由 \[ \frac{k_1k_2t+1}{kk_3}=\frac{t+1}{1} \] 得 \[ t=\frac{kk_3-1}{k_1k_2-kk_3}\text{,} \] 然后计算(\ref{eqn1})与(\ref{eqn2})得到的$x^2$、$xy$、常数项系数比,得 \[ \frac{k_1k_2-1}{k_1k_2-kk_3}=\frac{(k_1+k_2)(kk_3-1)}{(k+k_3)(k_1k_3+kk_3)}=\frac{(x_0^2-2x_0-1)\left(k_1k_2(kk_3-1)x_0^2+kk_3-k_1k_2\right)}{kk_3(k_1k_2+kk_3)(x_0-1)(x_0^3-x_0^2-x_0-1)}\text{,} \] 由 \[ \frac{k_1k_2-1}{k_1k_2-kk_3}=\frac{(k_1+k_2)(kk_3-1)}{(k+k_3)(k_1k_3+kk_3)} \] 得 \[ k=\frac{k_1+k_2-k_3+k_1k_2k_3}{1-k_1k_2+(k_1+k_2)k_3}\text{,} \] 由 \[ \frac{k_1k_2-1}{k_1k_2-kk_3}=\frac{(x_0^2-2x_0-1)\left(k_1k_2(kk_3-1)x_0^2+kk_3-k_1k_2\right)}{kk_3(k_1k_2+kk_3)(x_0-1)(x_0^3-x_0^2-x_0-1)} \] 得 \begin{equation} k=-\frac{(x_0-2) x_0-1) (k_3 (x_0-1) (k_1 k_2-1)+x_0 (k_1+k_2)}{x_0^3 (-k_3 (k_1+k_2)+k_1 k_2-1)+x_0^2 (2 k_3 (k_1+k_2)-k_1 k_2+1)+x_0 (k_3 (k_1+k_2)-k_1 k_2+1)-k_1 k_2+1}\text{,}\label{eqn3} \end{equation} 以上两个得到的$k$值相等得方程 \[ (k_1+k_2)(x_0^2-2x_0-1)k_3^2-2(k_1k_2-1)(x_0^2-x_0-1)k_3+(k_1+k_2)(x_0^2-1)=0\text{,} \] 令$u=\dfrac{k_3}{k}$,则 \begin{equation} (k_1+k_2-k_3+k_1k_2k_3)u+(k_1k_2-1)k_3-(k_1+k_2)k_3^2=0\text{,}\label{eqn4} \end{equation} (\ref{eqn3})、(\ref{eqn4})消去$k_3$,得 \[ (k_1+k_2)(k_1+1)(k_2+1)((x_0^2-2x_0-1)u-x_0^2+1)=0\text{,} \] 由此得 \[ u=\frac{x_0^2-1}{x_0^2-2x_0-1}\text{。} \]
李斌斌755 14# 2013-5-20 14:37
13# hejoseph 天啊
thread-1506-1-1.html: 一又一
李斌斌755 1# 2013-5-9 03:28
修修补补又一夜,昏昏碌碌又一年。
thread-1507-1-1.html: 物理热力学一个小证明
╰☆ヾo.海x 1# 2013-5-9 13:40
哎,海盗给解释了半天,我这个呆脑子还是理解不了。。 粒子撞击一次动能的变化量是2mv, 时间是L/v, 那 根据动量定理出来F=2mv^2/L啊。。但是我们老师讲义上写的F=mv^2/L... 后面怎么把压强联系到动能?那个2/3怎么来的啊。。
kuing 2# 2013-5-9 13:55
看不懂题……
╰☆ヾo.海x 3# 2013-5-10 21:06
1# ╰☆ヾo.海x 好了已经解决了。。。
thread-1508-1-1.html: [几何] 关于替换点……
李斌斌755 1# 2013-5-9 14:12
本帖最后由 李斌斌755 于 2013-5-9 14:19 编辑 由isea的“圆半径定长 圆上一点到两点距离和最小”http://kkkkuingggg.5d6d.net/thread-1502-1-1.html 想到的 1  动点在直线上,一倍距离时定点的替换点(暂且这么叫)就是定点关于直线的对称点。若干倍距离时有没有替换点? 2  动点在圆上,圆外一定点的若干倍距离是否都存在替换点? 3  动点在椭圆、双曲线……
李斌斌755 2# 2013-5-9 14:16
谁有相关资料,给个链接科普
0.1 3# 2013-5-9 19:44
2# 李斌斌755 根据阿氏圆定义知道后两个问题的答案都是否定的。
kuing 4# 2013-5-9 19:48
3# 0.1 0.1大牛也来了欢迎啊!
0.1 5# 2013-5-9 19:51
4# kuing 酷版过奖了,大牛不敢当啊 ,来学习的
isea 6# 2013-5-9 20:42
3# 0.1 0.1大牛也来了欢迎啊! kuing 发表于 2013-5-9 19:48 我发初中版是正确的,至少点幺来了。
isea 7# 2013-5-9 20:48
1弄得像反演变换了,可惜基本都忘记了 其次,那题的不动,完全是巧合点。
0.1 8# 2013-5-9 21:30
6# isea 这里更热闹些
李斌斌755 9# 2013-5-9 22:55
8# 0.1 欢迎。
李斌斌755 10# 2013-5-9 22:58
3# 0.1 但isea帖子里$\dfrac{\sqrt2}2QB$有替换点$D$阿?
isea 11# 2013-5-9 23:17
本帖最后由 isea 于 2013-5-9 23:18 编辑 10# 李斌斌755 我来解释一下吧。 我觉得这应该只是一个巧合。 这图中 QQ''/BQ=√(2)/2 ,即动点Q到两定点Q''与B距离之比为定值,所以动点P的轨迹就是圆M,这就是阿氏圆。 巧的是,这个Q''点也正是BM与圆D的切点。 点幺他主要是否定的3. 其实,你走得更远些了
李斌斌755 12# 2013-5-10 01:55
回复3#,11# 非常感谢两位的热心指导,到论坛来就是学习与玩(寓教于乐 ),之前知道到两定点的距离之比为定值的点轨迹为圆,但不知道其名称(阿氏圆),基本没应用过,现在晓得了。那第一个问题中的“动点到直线上任一点若干倍距离”是否存在替换点呢?
0.1 13# 2013-5-10 07:04
12# 李斌斌755 只要不是1倍均没有替换点,否则直线就成为半径有限的阿氏圆了。
李斌斌755 14# 2013-5-10 13:15
13# 0.1 醍醐灌顶
thread-1509-1-1.html: 我是蝜蝂
李斌斌755 1# 2013-5-9 16:04
在这做题,累死我了。
李斌斌755 2# 2013-5-9 16:06
不要成柳宗元笔下的蝜蝂。
kuing 3# 2013-5-9 17:02
“蝜蝂”这是什么字??
李斌斌755 4# 2013-5-9 19:07
3# kuing 我也不会,百度出来的
isea 5# 2013-5-9 20:47
古老新动物?
thread-151-1-1.html: 两个垂直符号的差别
kuing 1# 2011-10-26 14:57
a \bot b 与 a \perp b 效果: $a \bot b$ 与 $a \perp b$ $\angle ABC=90^\circ \iff AB\bot BC$ $\angle ABC=90^\circ \iff AB\perp BC$
图图 2# 2011-11-28 14:13
1# kuing
icesheep 3# 2011-11-28 14:35
小写、希腊字母用第二个比较好看,大写字母用第一个比较好看
kuing 4# 2012-1-18 17:28
其实本质差别在于 \bot 只是一个普通的符号,没什么特别的属性,而 \perp 则是一个二元关系符,就会在适当的时候产生适当的距离。 ) \perp b: $) \perp b$ ( \perp b: $( \perp b$ 同样是括号后跟\perp,闭括号时就会产生距离,而开括号则不产生。 此外,否定形式命令 \not 也只对这种关系符产生的斜杠的位置才会正确,例如 a \not\perp b 产生 $a \not\perp b$ a \not\bot b 产生 $a \not\bot b$ 后者的位置就不对了
thread-1510-1-1.html: 酷坛
isea 1# 2013-5-9 22:34
这个名字霸气
kuing 2# 2013-5-9 22:36
呃??
isea 3# 2013-5-9 22:51

kuing 4# 2013-5-11 21:31
其实我第一眼看成的是“醋坛”……
李斌斌755 5# 2013-5-11 21:43
4# kuing 好心力
thread-1511-1-1.html: 这两天玩消失了呢
isea 1# 2013-5-9 22:44
本帖最后由 isea 于 2013-5-9 23:38 编辑 yes94
李斌斌755 2# 2013-5-9 23:02
1# isea 是不是抽去命题了。
isea 3# 2013-5-9 23:37

kuing 4# 2013-5-11 21:30
好像有在群里冒泡过……
李斌斌755 5# 2013-5-18 14:46
兜一圈又不知逛哪了
李斌斌755 6# 2013-5-18 14:49
都市侠影也不见踪影?
kuing 7# 2013-5-18 14:52
6# 李斌斌755 刚才还在人教群里聊着……
李斌斌755 8# 2013-5-18 14:55
7# kuing 群里我是不敢去了,会挨香蕉皮的
kuing 9# 2013-5-18 16:25
yes94刚才在群里说 教师-其妙(2360****)  16:24:40 我的电脑每隔一两个月就上不了网,只能上qq
李斌斌755 10# 2013-5-18 17:10
9# kuing 这电脑好阿,可以预防网瘾
yes94 11# 2013-5-23 19:58
10# 李斌斌755 我来了!可惜论坛公示显示的太慢了!等的花儿都谢啦!
kuing 12# 2013-5-23 20:11
11# yes94 是因为 5d6d 的原因,页底的那堆东西老是卡住。 PS、http://kkkkuingggg.5d6d.net/thread-1565-1-1.html 或许有用
isea 13# 2013-5-23 21:40
论坛也开始玩“消失”了,哈哈
李斌斌755 14# 2013-5-24 00:03
13# isea 啥意思
李斌斌755 15# 2013-5-24 00:07
不过我们这里经常线路故障、停电……上不了网络,烦死了。
thread-1512-1-1.html: 级数与数列
Karron_ 1# 2013-5-9 22:51
想了很久想不出。。
Karron_ 2# 2013-5-9 22:59
话说答案有说用这个结论的.不过我没用上只能证明α为正整数是成立的。。
pxchg1200 3# 2013-5-9 23:27
考虑Gamma函数的Euler-Guess公式,我们有 \[ \Gamma{(x)}=\lim_{n\rightarrow\infty}{\frac{n!(n+1)^{x}}{x(x+1)(x+2)\cdots(x+n)}}\] 而递推可得 \[ a_{n+1}=\frac{n!}{(1+\alpha)(2+\alpha)\cdots(n+\alpha)}\] \[ (n+1)^{\alpha}a_{n+1}=\frac{n!(n+1)^{\alpha}}{(1+\alpha)(2+\alpha)\cdots(n+\alpha)}\] 显然 \[ \lim_{n\rightarrow\infty}{n^{\alpha}a_{n}}=\alpha\Gamma{(\alpha)}=\Gamma{(\alpha+1)}\]
Karron_ 4# 2013-5-9 23:35
这是别人提示下做出来的。
pxchg1200 5# 2013-5-9 23:56
2# Karron_ 这个结论不难证明,当$a_{n}$有界的时候只要注意到 \[ 0<\frac{a_{n+1}-a_{n}}{a_{n+1}}<\int_{a_{n}}^{a_{n+1}}{\frac{1}{x}dx}\] 就好了,$a_{n}$ isn't bounded就意味着$a_{n}\rightarrow+\infty$,考虑余项用Cauchy收敛准则就好。
Karron_ 6# 2013-5-10 00:07
5# pxchg1200 你是说答案提示用的结论么? 这个可以用无穷乘积去证明。
pxchg1200 7# 2013-5-10 00:08
本帖最后由 pxchg1200 于 2013-5-10 00:10 编辑 1# Karron_ 至于楼主要求用那个提示,我们可以设$x_{n}=n^{\alpha}a_{n}$,这样由刚刚那个命题,我们只要证级数 \[ \sum_{n=1}^{\infty}{\left(1-\frac{a_{n+1}}{a_{n}}\right)}=\sum_{n=1}^{\infty}{\left(\frac{\left(1+\frac{1}{n}\right)^{\alpha}-\left(1+\frac{\alpha}{n}\right)}{\left(1+\frac{1}{n}\right)^{\alpha}}\right)}\] 收敛即可。 注意到 \[ \left(1+\frac{1}{n}\right)^{\alpha}=e^{\alpha\ln{\left(1+\frac{1}{n}\right)}}\leq e^{\frac{\alpha}{n}}=1+\frac{\alpha}{n}+\frac{1}{2}e^{\frac{\alpha\theta}{n}}\cdot\frac{\alpha^{2}}{n^{2}}\] 其中$\theta\in(0,1)$ 故 \[ \left(\frac{\left(1+\frac{1}{n}  \right)^{\alpha}-\left(1+\frac{\alpha}{n}\right)}{\left(1+\frac{1}{n}\right)^{\alpha}}\right)<\frac{1}{2}e^{\frac{\alpha\theta}{n}}\cdot\frac{\alpha^{2}}{n^{2}}<\frac{1}{2}e^{\alpha}\cdot\frac{\alpha^{2}}{n^2} \] 故级数 \[\sum_{n=1}^{\infty}{\left(\frac{\left(1+\frac{1}{n}  \right)^{\alpha}-\left(1+\frac{\alpha}{n}\right)}{\left(1+\frac{1}{n}\right)^{\alpha}}\right)}<\sum_{n=1}^{\infty}{\frac{1}{2}e^{\alpha}\cdot\frac{\alpha^{2}}{n^2}}\] 收敛。
Karron_ 8# 2013-5-10 00:13
7# pxchg1200   刚才就是卡在怎么证明这个级数是收敛的,没想到用Taylor展开。
thread-1513-1-1.html: 一致收敛
Karron_ 1# 2013-5-9 23:03
摘自梅加强的《数学分析》,316页14题。好像好难。。有没大神帮忙?
pxchg1200 2# 2013-5-10 00:44
1# Karron_ 这个问题也不难,我们先证一个等价的问题。 设$f_{n}(x)$在$[a,b]$上可导,且有$M>0$,使得 \[ |f'_{n}(x)|\leq M \qquad (n\in \mathbf{N},a\leq x\leq b) \] 若$f_{n}(x)$在$[a,b]$上逐点收敛于$f(x)$,则必一致收敛。 证明:对$\forall \varepsilon>0$,作$[a,b]$的分划 \[ \Delta:a=x_{0}<x_{1}<\cdots<x_{k}=b,\qquad \text{分割细度$|\Delta|<\frac{\varepsilon}{3M}$}\] 则对于每个$x_{i}$,由于$f_{n}(x)$逐点收敛,故都存在一个对应的$N_{i},(i=0,1,2,\cdots,k)$,当$m,n>N_{i}$时有 \[ |f_{n}(x_{i})-f_{m}(x_{i})|<\frac{\varepsilon}{3}\] 这样,我们可以找到一个$N=\max\{N_{0},N_{1},\cdots,N_{k}\}$,对任意$n,m>N$有 \[  |f_{n}(x_{i})-f_{m}(x_{i})|<\frac{\varepsilon}{3},\qquad (i=0,1,\cdots,k)\] 现在对任意的$x\in[a,b]$,$x$必落入某个分割区间中,不妨设$x_{i}\leq x\leq x_{i+1}$,则当$n,m>N$时,有 \begin{align*} |f_{n}(x)-f_{m}(x)|&\leq |f_{n}(x)-f_{n}(x_{i})|+|f_{n}(x_{i})-f_{m}(x_{i})|+|f_{m}(x_{i})-f_{m}(x)|\\ &\leq |f'_{n}(c_{1})||x-x_{i}|+|f_{n}(x_{i})-f_{m}(x_{i})|+|f'_{m}(c_{2})||x_{i}-x|\\ &\leq M\cdot\frac{\varepsilon}{3M}+\frac{\varepsilon}{3}+M\cdot\frac{\varepsilon}{3M}=\varepsilon \end{align*} 故$f_{n}(x)\rightrightarrows f(x) $,现在用替换 \[ f_{n}(x)=S_{n}(x) \] 问题显然成立。
Karron_ 3# 2013-5-10 09:58
本帖最后由 Karron_ 于 2013-5-10 10:29 编辑 2# pxchg1200 你看我这样证问题出在哪? 作划分$$a=x_0<x_1<\cdots<x_{k-1}<x_k=b,\vert \Delta \vert< \frac{\varepsilon}{3}$$ $$令F_{m,n}(x)=S_m(x)-S_n(x);m>n$$ 由于$S_n(x)$可导,从而在$[a,b]$上一致连续。从而$F_{m,n}(x)$一致连续。 对于任意$x$必落在某个小区间上,不放设其落在$[x_{k-1},x_k]$ 取$\delta= \frac{\varepsilon}{3}$,则当$\vert x'-x'' \vert <\delta $时, $$\vert F_{m,n}(x')-F_{m,n}(x'') \vert<\frac{\varepsilon}{3}$$则 $$\vert F_{m,n}(x)-F_{m,n}(x_k) \vert<\frac{\varepsilon}{3}$$则 $$\frac{\varepsilon}{3}>\vert F_{m,n}(x)-F_{m,n}(x_k) \vert $$ $$=\vert S_m(x)-S_n(x)+S(x)-S_m(x_k)-S(x)+S_n(x_k) \vert$$ $$>\vert S_m(x)-S_n(x)\vert-\vert S_m(x_k)-S(x_k)\vert-\vert S(x_k)-S_n(x_k)\vert$$ 由于$S_n(x)$点态收敛于$S(x)$,则当存在$N_k$,当$m>n>N_k$时有, $$\vert S_m(x_k)-S(x_k)\vert<\frac{\varepsilon}{3} ;\vert S(x_k)-S_n(x_k)\vert<\frac{\varepsilon}{3}$$ 从而$$\vert S_m(x)-S_n(x)\vert<\varepsilon$$ 现取$N=max{(N_1,N_2,\cdots,N_k)}$,则$m>n>N$ 时,均有$$\vert S_m(x)-S_n(x)\vert<\varepsilon$$ 由Cauch收敛原理知$S_n(x)$一致收敛于$S(x)$ 居然没用到导函数一致有界的条件。。。不知道问题出在哪?难道是这样做区间划分是不可行的?
Karron_ 4# 2013-5-10 10:38
3# Karron_ px 说 一致连续只能说明$\delta$ 存在,不一定跟$\varepsilon$相关。 这就是我证明的致命点了。
Karron_ 5# 2013-5-10 10:48
本帖最后由 Karron_ 于 2013-5-10 11:05 编辑 2# pxchg1200 这是我修改后的证法。 作划分$$a=x_0<x_1<\cdots<x_{k-1}<x_k=b,\vert \Delta \vert< \frac{\varepsilon}{6M}$$ $$令F_{m,n}(x)=S_m(x)-S_n(x);m>n$$ 则$$由\vert S'_n(x)\vert一致有界知\vert F'_{m,n}(x)\vert<2M$$ 对于任意$x$必落在某个分划的区间上,不妨设其落在$[x_{k-1},x_k]$ $$\vert F_{m,n}(x)-F_{m,n}(x_k) \vert=\vert F’_{m,n}(\xi) \vert \vert x-x_k \vert<2M·\frac{\varepsilon}{6M}=\frac{\varepsilon}{3}$$则 $$\vert S_m(x)-S_n(x)\vert-\vert S_m(x_k)-S(x_k)\vert-\vert S(x_k)-S_n(x_k)\vert$$ $$<\vert S_m(x)-S_n(x)+S(x)-S_m(x_k)-S(x)+S_n(x_k) \vert$$ $$=\vert F_{m,n}(x)-F_{m,n}(x_k) \vert<\frac{\varepsilon}{3} $$ 由于$S_n(x)$点态收敛于$S(x)$,则当存在$N_k$,当$m>n>N_k$时有, $$\vert S_m(x_k)-S(x_k)\vert<\frac{\varepsilon}{3} ;\vert S(x_k)-S_n(x_k)\vert<\frac{\varepsilon}{3}$$ 从而$$\vert S_m(x)-S_n(x)\vert<\frac{\varepsilon}{3}+\vert S_m(x_k)-S(x_k)\vert+\vert S(x_k)-S_n(x_k)\vert<\varepsilon$$ 现取$N=max{(N_1,N_2,\cdots,N_k)}$,则$m>n>N$ 时,均有$$\vert S_m(x)-S_n(x)\vert<\varepsilon$$ 由Cauch收敛原理知$S_n(x)$一致收敛于$S(x)$
thread-1514-1-1.html: 不等式
李斌斌755 1# 2013-5-10 01:27
论坛里的不等式看都看不懂,出题人犹如设计院,不等式是图纸,我是农民工
kuing 2# 2013-5-10 01:34
蛋定。。。论坛又不是只有不等式。。。我最近都弱了。。。
李斌斌755 3# 2013-5-10 02:52
瘦死的骆驼比马大
李斌斌755 4# 2013-5-10 03:01
看你们(还有人教那边战巡等)摧枯拉朽,攻城略地……偶
pxchg1200 5# 2013-5-10 23:25
4# 李斌斌755 我也弱了,好多不会。。。
李斌斌755 6# 2013-5-11 00:32
5# pxchg1200 不会吧
kuing 7# 2013-5-11 21:29
gauss门徒(6024*****)  11:23:39 其实大神都说自己是喳喳
thread-1515-1-1.html: 数论
李斌斌755 1# 2013-5-10 04:27
翻看了数论的帖子,不是人做的,是神做的
李斌斌755 2# 2013-5-16 03:29
这个门徒专攻数论
李斌斌755 3# 2013-5-16 03:30
没人喽,睡觉了。
thread-1516-1-1.html: 页面访问量
李斌斌755 1# 2013-5-10 04:31
页面访问量是什么东东
isea 2# 2013-5-10 11:21
这么多
kuing 3# 2013-5-10 12:11
大概就是你的个人资料页面被浏览的次数 比如说我点击一下你的头像或者名字,查看你的个人资料,然后你的页面访问量就会+1
kuing 4# 2013-5-10 12:13
自己点的不知算不算 PS、次奥,10万多……
李斌斌755 5# 2013-5-10 13:30
我还以为是我浏览别人的帖子数量。
hnsredfox_007 6# 2013-5-10 16:56
刷新可以增加页面访问量吗?我喜欢刷新 哦
李斌斌755 7# 2013-5-10 17:19
6# hnsredfox_007 试试,反正没啥用
thread-1517-1-1.html: 雷区
李斌斌755 1# 2013-5-10 13:25
童鞋,千万不要去高等数学啊!小心被击倒
pxchg1200 2# 2013-5-10 23:28
1# 李斌斌755 怎么了?
李斌斌755 3# 2013-5-10 23:49
2# pxchg1200 我不懂高数
thread-1518-1-1.html: MSN
李斌斌755 1# 2013-5-10 17:09
点开个人空间,里面问你还没有开通  MSN,请问什么是 MSN 在线通?有什么作用?
kuing 2# 2013-5-10 18:35
不知道……N年前申请过MSN帐号,然后如无意外地荒废掉了……
thread-1519-1-1.html: [几何] 很另类的几何题
0.1 1# 2013-5-10 18:46
寻求初等证法 ______kuing edit in $\LaTeX$______ 已知:直线 $l\perp AB$ 于 $A$,$E$ 是线段 $AB$ 上不与 $A$、$B$ 重合的一定点,点 $C$ 是以 $AB$ 为直径的半圆上的一动点,$D$ 点在 $l$ 上且与 $C$ 在 $AB$ 同侧并满足 $AD=\text{弧}~AC$。 求证:当 $CB+DE$ 取得最大值时 $BC\sslash DE$。
李斌斌755 2# 2013-5-10 18:59
弧线=直线,初等解法
kuing 3# 2013-5-10 19:16
用速度分解,不难得到……
kuing 4# 2013-5-10 19:29
$BC+DE$ 取最大时 $v\cos x=v\cos y$,故 $x=y$,即得 $\perp$ 及 $\sslash$
kuing 5# 2013-5-10 20:08
4# kuing 此外,如果上述解法没问题,可以看出“弧=线”的条件可以减弱,只要那两点速度相同并且在同侧即可。
李斌斌755 6# 2013-5-10 20:52
$DF=$弧$CM$ $\triangle DQP\cong\triangle MCB\riff PE+BC>EQ>EF\riff DE+BC>BM+EF$ 但怎么证明平行存在或规尺画出平行。
李斌斌755 7# 2013-5-10 20:55
本帖最后由 李斌斌755 于 2013-5-10 20:57 编辑 4# kuing 看不懂 另弧怎么表示?
isea 8# 2013-5-10 21:10
这这,完全是为k 定制的嘛
kuing 9# 2013-5-10 21:31
7# 李斌斌755 这里无法表示弧符号(在真 LaTeX 里也要另外设置),用文字好了。
isea 10# 2013-5-10 21:32
7# 李斌斌755 LateX 巨大的bug
0.1 11# 2013-5-10 22:03
4# kuing 很奇特的思路,有求导的感觉
kuing 12# 2013-5-10 22:06
11# 0.1 是的,考虑的就是变化率。 $v={\rmd s}/{\rmd t}$,本质上是有导数在里面。
李斌斌755 13# 2013-5-10 22:37
本帖最后由 李斌斌755 于 2013-5-10 22:40 编辑 10# isea 明白
kuing 14# 2013-5-11 03:16
LateX 巨大的bug isea 发表于 2013-5-10 21:32 这大概不叫bug。 这只是在于当年设计符号字体的人们认为有没有必要整,或者说按照什么样的标准来编制。
李斌斌755 15# 2013-5-12 13:41
已知条件不变,问题换成:用规尺作图画出$BC\sslash DE$时的$C$点。
kuing 16# 2013-5-12 15:17
已知条件不变,问题换成:用规尺作图画出$BC\sslash DE$时的$C$点。 李斌斌755 发表于 2013-5-12 13:41 估计是尺规不可作的。 我们不妨取一个特殊情况,比如说,设圆半径为 $1$ 并且取 $E$ 为圆心,如图: 于是由“弧=线”应有 $\tan\alpha=2\alpha$,只要能证明 $\alpha$ 为超越数(相信是这样的,虽然暂时不知怎么证),那么 $AD$ 就是超越数,便是尺规不可作的。
李斌斌755 17# 2013-5-12 18:39
16# kuing 喔,超级数$\iff$规尺不可作
kuing 18# 2013-5-12 18:50
16# kuing 喔,超级数$\iff$规尺不可作 李斌斌755 发表于 2013-5-12 18:39 不,超越数是尺规不可作的充分而不必要条件。$\sqrt[3]2$ 也尺规不可作,但它是代数数。 PS、是超越,不是超级
李斌斌755 19# 2013-5-12 19:32
18# kuing 谢谢
thread-152-1-1.html: 下加点
kuing 1# 2011-10-26 15:04
先说明:本贴是为真 LaTeX 而写,暂时不适用于本论坛所用的 MathJax。 首先说英文的下加点 单个字母下加点用 \d{a} 或 \d a 多个时用 \d{a}\d{b}\d{c}\d{d} 或 \d a\d b\d c\d d 若想偷懒用 \d{abcd} ,不是不行,但那个点也会偷懒,它会把 abcd 看作整体,然后在中间下方加且只加一个点 而 \d abcd 则只会对第一个a进行下加点,后边的不鸟你,所以只能打N个,比较麻烦 注意空格问题,如果将 \d a\d b\d c\d d 改成 \d a \d b \d c \d d 那么单词就不连续了,a后面的空格不会忽略的 然后说中文的下加点 需要用宏包 \usepackage{CJKfntef} 用法:\CJKunderdot{需下加点文字,允许有英文,能自动换行,但公式不适用} 宏包 CJKfntef 还提供下划线、波浪线、删除线等: \uline{加下划线,自动换行,允许有英文,对小数学公式也式用(公式较大时会超出下划线)} \uwave{加波浪线,其余同上} \sout{加删除线,其余同上} 注意简单的 \underline{下划线} 遇到换行时容易出问题,所以一般只适用于短短几个字
abababa 2# 2013-2-25 19:35
1# kuing 我试了下加点功能,字母y下的那个点太靠下了,怎么才能弄成和别的点一样齐呢? 还有CJKunderdot的那个点,英文和中文在一起就加不全,应该怎么弄呢?
kuing 3# 2013-2-25 21:45
很少用下加点,也不太清楚,有空研究下。
abababa 4# 2013-2-25 23:12
3# kuing 我觉得latex写公式真方便,但是调这些格式之类的,像我这样学得浅的就麻烦了,弄好久也弄不出。
kuing 5# 2013-4-30 15:34
续 1# 用 \CJKunderanysymbol{下移距离}{加下的符号}{汉字} 这样,就可以自定义汉字下加的东西
abababa 6# 2013-5-3 11:15
试一下,不知道这里能不能用。 $\CJKunderanysymbol{0.5mm}{~}{给字母abcde加波浪线}$
kuing 7# 2013-5-3 11:36
6# abababa 这里自然是不行……1#已经说了……
thread-1520-1-1.html: [函数] 我给市模拟卷命的一个题,不过被刷下来了
第一章 1# 2013-5-10 19:40
这题权当函数题好了。 上次说要出一个题目给其妙的,其实我不会原创题,这是我改编自2012年的一个高考题,改得不好,大家见笑了,希望能提出改进意见…… 已知$x>0$,$n\in N*$,函数$f_n(x)=x^{-n}+x-2$ (1)解方程$f_1(x)=0$; (2)当$n\ge 2$时,证明函数$y=f_n(x)$在区间$(\frac{3}{2},2)$上存在唯一零点; (3)设$x_n$是$f_n(x)$在区间$(\frac{3}{2},2)$上的零点,试探究数列$x_2,x_3,\dots,x_n$的增减性.
依然饭特稀 2# 2013-5-10 22:28
陕西?
第一章 3# 2013-5-10 22:52
是的,陕西卷 本来不想发的,但是看到最近几个二模题都出现这种函数、数列的结合题,故一并发上来。 广州二模21题: 设$a_n$是函数$f(x)=x^3+n^2x-1(n\in N^*)$的零点. (1)证明:$0<a_n<1$; (2)证明:$\frac{n}{n+1}<a_1+a_2+\dots+a_n<\frac{3}{2}$.
第一章 4# 2013-5-10 23:09
还有一题,揭阳二模21题: 设函数$f_n(x)=x^n(1-x)^2$在$[\frac{1}{2},1]$上的最大值为$a_n$,$n\in N^*$. (1)求$a_1$,$a_2$的值; (2)求数列$\{a_n\}$的通项公式; (3)证明:对任意$n\in N^*(n\ge2)$,都有$a_n\le \frac{1}{(n+2)^2}$成立.
thread-1521-1-1.html: [几何] 三角形(或其它多边形)顶点落在一组平行或一组圆上的 几何作图 问题——(在6楼)
isea 1# 2013-5-10 21:18
本帖最后由 isea 于 2013-5-13 12:11 编辑 曾经的题偏简单,均是等距的,没引起重视。 这次海淀高中,初中都涉及这类问题,故找点资料。 不知大家有没类似的资料呢,举了例子:
李斌斌755 2# 2013-5-10 21:39
1# isea 某年高考题考过。
isea 3# 2013-5-13 11:24
选自翟刚的关于几何变换的应用问题
hejoseph 4# 2013-5-13 11:36
平行直线都比较容易,若直线不全是平行的呢?或者把直线变成圆呢?
isea 5# 2013-5-13 11:43
4# hejoseph 直线变圆,可考虑反演变换吧,如果是比较简单的话。 一般的情况,完全超过偶的预期!
hejoseph 6# 2013-5-13 11:47
例如就这个作图: 给定三个圆,求作一个给定边长比是$a:b:c$的三角形,使这个三角形的三个顶点各自落在指定的三个圆上。
李斌斌755 7# 2013-5-13 12:02
这种变化妙
0.1 8# 2013-5-13 14:25
6# hejoseph 在给定的一个圆上任取一点D作为所作三角形的一个顶点,下图所示的是满足条件的三角形有两个的情形:
0.1 9# 2013-5-13 14:29
具体讨论起来太麻烦了
isea 10# 2013-5-13 14:31
具体讨论起来太麻烦了 0.1 发表于 2013-5-13 14:29 厉害,佩服,学习!
hejoseph 11# 2013-5-13 15:00
画图法正确,但很多时候不是每一点都有解的,所以最好能解决哪些点有解,这样就比较完整了
0.1 12# 2013-5-13 16:12
11# hejoseph 讨论才是难点
yes94 13# 2013-5-15 23:40
1# isea 某年高考题考过。 李斌斌755 发表于 2013-5-10 21:39 四川吧?
李斌斌755 14# 2013-5-16 00:39
13# yes94 记不住了。
thread-1522-1-1.html: 我又来了。征求这个不等式的各种证法。
Karron_ 1# 2013-5-10 22:36
本帖最后由 Karron_ 于 2013-5-10 22:39 编辑 今天看到这个不等式$$(\frac{n+1}{e})^n<n!<e(\frac{n+1}{e})^{n+1}$$ 我尝试了下,有下面这个不漂亮的证法。 先证左边:令$$b_n=\frac{(n+1)^n}{e^n·n!}$$ 则$$\frac{b_{n+1}}{b_n}=\frac{(1+\frac{1}{n+1})^{n+1}}{e}$$ 而$$a_n=(1+\frac{1}{n})^n单调递增且趋于e$$ 所以$$\frac{b_{n+1}}{b_n}<1 \rightarrow b_n \leq b_1=\frac{2}{e}<1$$ 再证左边:令$$x_n=\frac{(n+1)^{n+1}}{e^n·n!}$$ $$则\frac{x_{n+1}}{x_n}=\frac{(n+2)^{n+2}}{e(n+1)^{n+2}}=\frac{(1+\frac{1}{n+1})^{n+2}}{e}$$ 而$$ln(1+\frac{1}{n+1})>\frac{1}{n+2} \rightarrow (1+\frac{1}{n+1})^{n+2}>e$$ 即$$\frac{x_{n+1}}{x_n}>1$$ 从而$$x_n \geq x_1=\frac{4}{e}>1$$ 感觉应该还有其他更好的方法。然后还感觉有一些不等式跟这个不等式有关联。。
pxchg1200 2# 2013-5-10 23:17
1# Karron_ 只要注意到 \[ \left(1+\frac{1}{n}\right)^{n}<e<\left(1+\frac{1}{n}\right)^{n+1}\] 就好了,把上面的式子中的$n$从$1$累乘到$n$结论显然。
thread-1523-1-1.html: [函数] 最值问题
Gauss门徒 1# 2013-5-10 23:05
本帖最后由 Gauss门徒 于 2013-5-10 23:07 编辑 设$a+b+c+d=4$ and $a,b,c,d\ge0$,求 \[(a-1)(b-1)+(b-1)(c-1)+(c-1)(d-1)\]的最大值
Gauss门徒 2# 2013-5-10 23:06
@kuing
kuing 3# 2013-5-11 02:44
固定 b 和 c, 当 a 和 d 变化时, 原式关于 a(或 d)是一次函数, 最大值必在端点取得.  而 a 和 d 变化时不会先达到 4, 从而由对称性只要考虑 a=0. 然后再固定 c, 当 b 和 d 变化时原式仍然关于 b(或 d) 是一次函数, 类似地只要考虑 b(或 d)为 0. 变成二元, 怎么玩都行.
kuing 4# 2013-5-11 02:45
爪机打这些真麻烦。。。
kuing 5# 2013-5-15 00:57
跟这道题:http://bbs.pep.com.cn/forum.php?mod=viewthread&tid=2760981 等价
thread-1524-1-1.html: [几何] 转人教求外心轨迹
李斌斌755 1# 2013-5-10 23:22
本帖最后由 李斌斌755 于 2013-5-10 23:27 编辑 已知点$P(-4,0)$,点$A,B$在直线$x=-1$上移动,且满足$\angle APB=60^\circ$  ,求$\triangle APB$外心的轨迹。 原贴见           http://bbs.pep.com.cn/forum.php? ... &extra=page%3D1
地狱的死灵 2# 2013-5-10 23:34
本帖最后由 地狱的死灵 于 2013-5-10 23:41 编辑 容易证明外心到x=-1的距离是OA=OP的一半, 于是所求轨迹就是以x=-1为准线, (-4,0)为焦点的双曲线的一支, 且离心率为2…… 轨迹方程:$\frac{x^2}{4} - \frac{y^2 }{12} = 1$,x<0
isea 3# 2013-5-10 23:41
2# 地狱的死灵 秒了,圆锥曲线第二定义
李斌斌755 4# 2013-5-10 23:41
2# 地狱的死灵 那我转回去
kuing 5# 2013-5-10 23:54
这么简单也转来转去……
李斌斌755 6# 2013-5-10 23:57
5# kuing 我是拉客的
thread-1525-1-1.html: 一个积分不等式(From 西西)
pxchg1200 1# 2013-5-10 23:23
本帖最后由 pxchg1200 于 2013-5-13 00:00 编辑 设$f(x)$是$[0,1]\rightarrow R $上的连续函数,且记$\displaystyle F(x)=\int_{0}^{x}{f(t)dt}$,并有 \[ \int_{0}^{1}{x^2f(x)dx}=-2\int_{\frac{1}{2}}^{1}{F(x)dx}\] 求证 \[ \int_{0}^{1}{f^2(x)dx}\geq 80\left( \int_{0}^{1}{f(x)dx}\right)^2 \]
thread-1526-1-1.html: 马甲
李斌斌755 1# 2013-5-10 23:53
明明是不等式,改函数求最值 ,还是不敢碰。
kuing 2# 2013-5-11 02:48
看情况吧,我刚才回的那个就是用函数思想,的确可以分类到函数。
thread-1527-1-1.html: 休息日
李斌斌755 1# 2013-5-11 15:00
真的是休息日,没人玩,88……嗯跟谁
kuing 2# 2013-5-11 15:03
我也不知道为什么没人,就你在发…… 不过我这两天有点事情搞,整数学空间,改论文……都是代码……
李斌斌755 3# 2013-5-12 15:08
2# kuing 忙完了?
thread-1528-1-1.html: 链接
李斌斌755 1# 2013-5-11 21:53
很多网站度都有友情链接,不知论坛兴不兴。
kuing 2# 2013-5-11 21:54
啥意思,没懂,想表达什么?
李斌斌755 3# 2013-5-11 22:05
2# kuing 能否建立一些友情论坛,可以相互链接,瞎说
thread-1529-1-1.html: [几何] 非角度 求面积 卡了
isea 1# 2013-5-11 22:02
如下图,想整个外接圆来,结果无果,大家看看,有无好办法。 主要想解决第二问,简短有意思。
kuing 2# 2013-5-11 22:16
看着像是那种旋转题……你旋转过没?
isea 3# 2013-5-11 22:20
2# kuing 旋转+相似与作外接圆似乎是同一条路,不太好整
0.1 4# 2013-5-11 23:14
供参考:
isea 5# 2013-5-11 23:38
4# 0.1 原来真是这么奇怪的数字啊! 感谢感谢。
hejoseph 6# 2013-5-12 00:26
第二问用勾股定理建立方程组也容易的,且能解决任意知道直角三角形的边长比和$PA$、$PB$、$PC$长度求直角三角形的边长的问题。 建立方程组 \[ \left\{ \begin{aligned} x^2+y^2&=1 \text{,} \\ (\sqrt 3b-x)^2+y^2&=10 \text{,} \\ (b-y)^2+x^2&=2 \text{。} \end{aligned} \right. \] $(\sqrt 3b-x)^2+y^2=10$展开并利用$x^2+y^2=1$,得 \[ 3b^2-2\sqrt 3bx=9 \text{,} \] 求得 \[ x=\frac{3b^2-9}{2\sqrt 3b} \text{,} \] 由此可得$b>\sqrt 3$。 $(b-y)^2+x^2=2$展开并利用$x^2+y^2=1$,得 \[ b^2-2by=1 \text{,} \] 求得 \[ y=\frac{b^2-1}{2b} \text{。} \] 把上述求得的$x$、$y$的式子代入$x^2+y^2=1$,得 \[ \left(\frac{3b^2-9}{2\sqrt 3b}\right)^2+\left(\frac{b^2-1}{2b}\right)^2=1 \text{,} \] 整理得 \[ b^4-6b^2+7=0 \text{,} \] 解上面的方程得 \[ b=\sqrt{3\pm\sqrt{2}} \text{,} \] 但$b=\sqrt{3-\sqrt{2}}$不满足$b>\sqrt 3$,所以只能$b=\sqrt{3+\sqrt{2}}$。
李斌斌755 7# 2013-5-12 02:51
这题做不来,贴标答
李斌斌755 8# 2013-5-12 02:59
本帖最后由 李斌斌755 于 2013-5-12 03:46 编辑 7# 李斌斌755 特殊直角三角形的个翻折问题+海伦公式($a,b,c$值随意) 而这题的数字根据直角$\triangle ABC,\angle ACB=90^\circ$构建出来的$b=\sqrt{10},a=1\riff c=\sqrt2$
Gauss门徒 9# 2013-5-12 14:04
我以前做过,就是那个对称的方法
Gauss门徒 10# 2013-5-12 14:05
这份模拟卷真是变态
hejoseph 11# 2013-5-12 22:41
本帖最后由 hejoseph 于 2013-5-12 22:58 编辑 令$\angle ACP=\theta$,则 \begin{align*} \cos\theta&=\frac{AC^2+PC^2-PA^2}{2\cdot AC\cdot PC}=\frac{b^2-1}{2b} \text{,} \\ \sin\theta&=\frac{BC^2+PC^2-PB^2}{2\cdot BC\cdot PC}=\frac{3b^2-9}{2\sqrt 3b} \text{,} \end{align*} 由$\sin\theta>0$得$b>\sqrt 3$,再由$\sin^2\theta+\cos^2\theta=1$得 \[ b^4-6b^2+7=0\text{,} \] 后面的跟我上面发的相同了,这个方法对非直角三角形同样适用。
hejoseph 12# 2013-5-13 09:45
本帖最后由 hejoseph 于 2013-5-13 10:04 编辑 下面的是很一般的三角形的例子: 设$P$是$\triangle ABC$内一点,$AB:AC:BC=4:5:6$,$PA=1$,$PB=2$,$PC=3$,求$\triangle ABC$的面积。 设$AB=4t$,$AC=5t$,$BC=6t$,则 \begin{align*} \cos\angle ACB &= \frac{AC^2+BC^2-AB^2}{2\cdot AC\cdot BC}=\frac{3}{4}\text{,}\\ \sin\angle ACB &= \sqrt{1-\cos^2\angle ACB}=\frac{\sqrt 7}{4}\text{,} \end{align*} 由此得 \[ S_{\triangle ABC}=\frac{1}{2}\cdot AC\cdot BC\cdot\sin\angle ACB=\frac{15}{4}\sqrt 7t^2\text{。} \] 设$\angle BPC=\theta$,则 \begin{align*} & \cos\theta=\frac{BC^2+PC^2-PB^2}{2\cdot BC\cdot PC}=\frac{36t^2+5}{36t}\text{,}\\ & \cos(\angle ACB-\theta)=\frac{3}{4}\cos\theta+\frac{\sqrt 7}{4}\sin\theta=\frac{AC^2+PC^2-PA^2}{2\cdot AC\cdot PC}=\frac{20t^2+8}{30t}\text{。} \end{align*} 由$\frac{3}{4}<\cos\theta<1$及$\frac{3}{4}<\cos(\angle ACB-\theta)<1$得$\frac{5}{12}<t<\frac{4}{5}$。由$\cos\theta=\frac{36t^2+5}{36t}$及$\frac{3}{4}\cos\theta+\frac{\sqrt 7}{4}\sin\theta=\frac{20t^2+8}{30t}$得 \[ \sin\theta=\frac{20t^2+39}{60\sqrt 7t}\text{,} \] 由$\sin^2\theta+\cos^2\theta=1$得 \[ \left(\frac{20t^2+39}{60\sqrt 7t}\right)^2+\left(\frac{36t^2+5}{36t}\right)^2=1\text{,} \] 整理得 \[ 14400t^4-9360t^2+1129=0\text{,} \] 解上面的方程,得 \[ t=\frac{1}{2}\sqrt{\frac{39\pm14\sqrt 2}{30}}\text{,} \] 只有$t=\frac{1}{2}\sqrt{\frac{39+14\sqrt 2}{30}}$满足$\frac{5}{12}<t<\frac{4}{5}$,所以 \[ t=\frac{1}{2}\sqrt{\frac{39+14\sqrt 2}{30}}\text{,} \] 所以 \[ S_{\triangle ABC}=\frac{15}{4}\sqrt 7t^2=\frac{39\sqrt 7+14\sqrt{14}}{32}\text{。} \]
李斌斌755 13# 2013-5-13 10:40
12# hejoseph 这个推广好!
isea 14# 2013-5-13 10:45
这真是一道好题啊!
yes94 15# 2013-5-15 23:44
这种题几何做法常常是旋转变换,如果数据特殊,那么会构成特殊角 代数做法就是余弦定理吧
thread-153-1-1.html: [转]中文加粗自动变黑体
kuing 1# 2011-10-26 15:13
与英文习惯不同,中文排版一般没有粗体的概念,而是用宋体、黑体、楷体、仿宋等不同的字体来加强文本效果。在CJK默认的字体定义文件中,宋体对应的粗体是用\CJKbold命令通过微小平移字形来实现的,不仅放大后能看到锯齿,而且也不符合中文的排版习惯。建议将宋体对应的粗体设为黑体,这样文中所有粗体命令作用的环境中加粗的宋体都会自动变成黑体,不需要再用\CJKfamily{hei}来转换。 宋体的字体定义文件C19song.fd里典型的一段代码如下 \DeclareFontFamily{C19}{song}{} \DeclareFontShape{C19}{song}{m}{n}{<-> CJK * gbksong}{} \DeclareFontShape{C19}{song}{bx}{n}{<-> CJKb * gbksong}{\CJKbold} \DeclareFontShape{C19}{song}{m}{it}{<-> CJK * gbksongsl}{} \DeclareFontShape{C19}{song}{bx}{it}{<-> CJKb * gbksongsl}{\CJKbold} \DeclareFontShape{C19}{song}{m}{sl}{<-> CJK * gbksongsl}{} \DeclareFontShape{C19}{song}{bx}{sl}{<-> CJKb * gbksongsl}{\CJKbold} 这里 C19代表GBK字符集的编码方式 m表示字符的粗细程度和宽紧程度均为中等(medium) bx表示字符的属性为粗(bold)和松(expanded) n, it和sl分别表示正常直立体(normal), 意大利斜体(italic)和机械斜体(slanted) \DeclareFontShape的最后两个参数分别是字体定义和命令序列。 显然,上述语句将宋体的粗体定义为用\CJKbold生成。要将其改为黑体,只需将这段代码改为 \DeclareFontFamily{C19}{song}{} \DeclareFontShape{C19}{song}{m}{n}{<-> CJK * gbksong}{} \DeclareFontShape{C19}{song}{bx}{n}{<-> CJKb * gbkhei}{} \DeclareFontShape{C19}{song}{m}{it}{<-> CJK * gbksongsl}{} \DeclareFontShape{C19}{song}{bx}{it}{<-> CJKb * gbkheisl}{} \DeclareFontShape{C19}{song}{m}{sl}{<-> CJK * gbksongsl}{} \DeclareFontShape{C19}{song}{bx}{sl}{<-> CJKb * gbkheisl}{}
kuing 2# 2011-10-26 15:13
又或者改成 \DeclareFontFamily{C19}{song}{} \DeclareFontShape{C19}{song}{m}{n}{<-> CJK * gbksong}{} \DeclareFontShape{C19}{song}{bx}{n}{<-> CJKb * gbkhei}{} \DeclareFontShape{C19}{song}{m}{it}{<-> CJK * gbkkai}{} \DeclareFontShape{C19}{song}{bx}{it}{<-> CJKb * gbkkai}{} \DeclareFontShape{C19}{song}{m}{sl}{<-> CJK * gbkkai}{} \DeclareFontShape{C19}{song}{bx}{sl}{<-> CJKb * gbkkai}{} 即:普通宋体不变,粗宋体变黑体,其它宋体斜体都变楷书 这个应该更标准点
thread-1530-1-1.html: 悲催
李斌斌755 1# 2013-5-12 04:02
坛里逛了一天,没人 好不容易来了一题,竟然不会做
thread-1531-1-1.html: 试一试
wzxsjz 1# 2013-5-12 10:54
求方程sinnx=cosx在[0,π]内解的个数。
李斌斌755 2# 2013-5-12 12:11
数形结合。
kuing 3# 2013-5-12 16:12
2# 李斌斌755 看图容易证明难…… \[f(n)=2\left\lfloor\frac n2\right\rfloor+1\]
李斌斌755 4# 2013-5-12 18:37
3# kuing 我不会证明。
零定义 5# 2013-5-13 18:31

零定义 6# 2013-5-13 19:04
k神说“如果能说明两个“或”的情况不会产生重复,那就更完美些" 那在此随意的补上个说明吧...
kuing 7# 2013-5-13 20:31
5# 零定义 前半没问题,后面有中括号开始还没仔细检验…… 话说那里开始你是不是为了凑成我那个式子而故意整成中括号的啊?
零定义 8# 2013-5-13 20:54
7# kuing 第一:k本身就是整数; 第二:后面的那个式子觉得不够你的简洁,所以就用你那个了.我证明了的,只是人懒,没在这写...
kuing 9# 2013-5-13 20:56
8# 零定义 原来如此
零定义 10# 2013-5-13 21:16
9# kuing 好吧~再补充这个证明...
李斌斌755 11# 2013-5-13 23:05
中括号表示取整吗?
kuing 12# 2013-5-14 00:24
11# 李斌斌755 一般来说是的,不过我更习惯用 $\lfloor x \rfloor$ ,因为它能与向上取整 $\lceil x \rceil$ 形成鲜明对比。
零定义 13# 2013-5-15 15:28
@kuing 如果将sinnx换成tannx,根的个数很容易知道,但能不能算出它的所有根呢?再如果,tan、sin、cos这三个函数随意换呢?
yes94 14# 2013-5-15 21:40
求方程sinnx=cosx在[0,π]内解的个数。 wzxsjz 发表于 2013-5-12 10:54 开头可以这样做啊? \[\begin{array}{l} \sin nx = \cos x \\ \Leftrightarrow \sin nx = \sin (x + \frac{\pi }{2})\\ \Leftrightarrow nx = 2k\pi  + (x + \frac{\pi }{2}),或nx = 2k\pi  + \pi  - (x + \frac{\pi }{2})\\ \Leftrightarrow (n - 1)x = 2k\pi  + \frac{\pi }{2},或(n + 1)x = 2k\pi  + \frac{\pi }{2} \end{array}\] 以下同5楼
yes94 15# 2013-5-15 21:45
14# yes94 开头还可以积化和差来做: \[\begin{array}{l} \sin nx = \cos x \\ \Leftrightarrow \sin nx + \sin (x - \frac{\pi }{2}) = 0 \\ \Leftrightarrow 2\sin \frac{{nx + x - \dfrac{\pi }{2}}}{2}\cos \frac{{nx - x + \dfrac{\pi }{2}}}{2} = 0\\ \Leftrightarrow \sin \dfrac{{nx + x - \dfrac{\pi }{2}}}{2} = 0,或\cos \frac{{nx - x + \dfrac{\pi }{2}}}{2} = 0 \\ \Leftrightarrow \dfrac{{nx + x - \dfrac{\pi }{2}}}{2} = k\pi ,或\frac{{nx - x + \dfrac{\pi }{2}}}{2} = k\pi  + \dfrac{\pi }{2}\\ \Leftrightarrow (n + 1)x = 2k\pi  + \frac{\pi }{2},或(n - 1)x = 2k\pi  + \frac{\pi }{2} \end{array}\] 以下同5楼
thread-1532-1-1.html: [不等式] 昨天晚上天书在kuing粉丝群里发的一个不等式
pxchg1200 1# 2013-5-12 12:52
本帖最后由 pxchg1200 于 2013-5-12 13:19 编辑 设$a,b,c\geq 0$,$a+b+c=ab+bc+ca$,证明 \[ \frac{1}{1+a}+\frac{1}{1+b}+\frac{1}{1+c}\geq \frac{4}{3}\] 其实这个题目就是出题的人玩了点花样,让我们来揭开它神秘的面纱. 证明:kuing 首先注意到取等条件是$a=b=2,c\rightarrow+\infty$,为了摆脱这个$+\infty$,设 \[x=\frac{1}{a},y=\frac{1}{b},z=\frac{1}{c}\] 这样,仍然有$x+y+z=xy+yz+xz$,而不等式变成 \[ \frac{x}{x+1}+\frac{y}{y+1}+\frac{z}{z+1}\geq \frac{4}{3} \] 展开并利用条件,得到 \[ xy+yz+xz+xyz\geq 4 \] 而我们有熟悉恒等式 \[ \frac{1}{x+2}+\frac{1}{y+2}+\frac{1}{z+2}=1 \] 上面的恒等式就是 \[ xy+yz+xz+xyz=4 \] 于是,根据这个思路,我们可以把不等式变成 \[ \frac{1}{x+2}+\frac{1}{y+2}+\frac{1}{z+2}\leq 1\] 就是 \[ \frac{x}{x+2}+\frac{y}{y+2}+\frac{z}{z+2}\geq 1 \] 到这里,不Cauchy-Schwarz还真说不过去。直接 \[ \sum{\frac{x}{x+2}}\geq \frac{(x+y+z)^2}{\sum{x^2}+2\sum{x}}=\frac{(x+y+z)^2}{\sum{x^2}+2\sum{xy}}=1 \] Done!
kuing 2# 2013-5-12 13:47
不是“$a=b=2,c\to+\infty$”,是“当 a=1/2, b=1/2+x, c=1/2+3/(4x) 且 x->0^+ 时” 顺便贴一下当时的聊天记录…… 天书/cy/cy/cy(1846******) 21:34:31 [图片] 疯了,这个到底啥取等??? 天书/cy/cy/cy(1846******) 21:35:14 名侦探柯南——取等条件失踪事件 kuing/yhh/衰/zhh 21:44:29 当 a=1/2, b=1/2+x, c=1/2+3/(4x) 且 x->0^+ 时 天书/cy/cy/cy(1846******) 21:44:51 我勒个去 kuing/yhh/衰/zhh 21:45:36 你都还没验证,怎么就去了…… kuing/yhh/衰/zhh 21:45:55 去了去了……oh yeah! 南飞雁2013(5582****) 21:46:56 这取等条件也太变态了。虽然不知说什么,但也算长见识了 kuing/yhh/衰/zhh 21:47:47 是“取等”条件…… 天书/cy/cy/cy(1846******) 21:50:35 咋构造出来的- - kuing/yhh/衰/zhh 21:52:25 目测1/2,1/2,无穷大,然后将其中一个整个 x 进去,解…… 天书/cy/cy/cy(1846******) 21:57:23 我去 怪物 这都能目测 kuing/yhh/衰/zhh 21:58:21 当然不是一测就好啊 是先目测了一些常规取等,都不对,于是就考虑无穷大 ……
李斌斌755 3# 2013-5-12 13:50
金睛火眼
Gauss门徒 4# 2013-5-12 14:07
这个严格来说不能取等的
kuing 5# 2013-5-13 17:45
1# pxchg1200 话说这样转换之后跟《数学空间》总第12期P25题目3.1.5一个样,条件和结论过来,方法能搬。
thread-1533-1-1.html: [数列] 发个数列题
weihua97 1# 2013-5-12 18:01

kuing 2# 2013-5-12 18:07
群管-kuing  18:03:30 大概像这样做 http://bbs.pep.com.cn/thread-335004-1-1.html 教师--小南海(2660***)  18:04:54 群主厉害啊,这么快就能定位 谢谢 群管-kuing  18:05:44 不完全一样,也不知道那个方法能不能证到这样。
thread-1535-1-1.html:
李斌斌755 1# 2013-5-12 22:59
kuing,图中大括号怎么编辑?
kuing 2# 2013-5-12 23:03
1# 李斌斌755 右键查看代码
李斌斌755 3# 2013-5-12 23:49
2# kuing 谢谢,又学一招。
李斌斌755 4# 2013-5-13 01:30
\[\left.\begin{aligned}&AB\sslash CD\\&EF\sslash CD\end{aligned}\right\}\riff EF\sslash AB\] 括号太大,怎么调整?
kuing 5# 2013-5-13 01:32
4# 李斌斌755 这里不能改行距,用别的方法比较麻烦。 就先这样吧,其实效果还成。
李斌斌755 6# 2013-5-13 01:34
5# kuing 谢谢,将就用
李斌斌755 7# 2013-5-17 01:59
本帖最后由 李斌斌755 于 2013-5-17 02:03 编辑 环境中$\eqref{csineq}% 说明中是引用作用,但不明白怎么个引用法?
李斌斌755 8# 2013-5-17 02:14
本帖最后由 李斌斌755 于 2013-5-17 02:55 编辑 \[\begin{equation}\label{csineq}\left(\sum_{k=1}^n a_kb_k\right)^2\leqslant\left(\sum_{k=1}^n a_k\right)^2\left(\sum_{k=1}^n b_k\right)^2\end{equation}\] 范例中\label{csineq}的作用也是引用吗? \[\eqref{csineq}\]
kuing 9# 2013-5-17 02:25
自己理解,爪机懒得解释,示例应该已经很清楚了。
李斌斌755 10# 2013-5-17 03:18
9# kuing 哈哈,明白了。还是读贴不认真啊!
thread-1536-1-1.html: 开贴练习
李斌斌755 1# 2013-5-13 10:44
本帖最后由 李斌斌755 于 2013-5-15 19:55 编辑 学习LATEX,路慢慢…… 右键查看代码(图中位置点开就是)
李斌斌755 2# 2013-5-13 11:07
本帖最后由 李斌斌755 于 2013-5-13 12:14 编辑 摘录hejoseph一般三角形内一点 记$P$是$\triangle ABC$内一点,$AB:AC:BC=4:5:6$,$PA=1,PB=2,PC=3$,求$\triangle ABC$的面积。 设$AB=4t,AC=5t,BC=6t$,则\[\cos\angle ACB=\dfrac{AC^2+BC^2-AB^2}{2\cdot AC\cdot BC}\\\sin\angle ACB=\sqrt{1-\cos^2\angle ACB}\]由此得\[S_{\triangle ABC}=\dfrac12\cdot AC\cdot BC\cdot\sin\angle ACB\]设$\angle PCB=\theta$,则\[\cos\theta=\dfrac{PC^2+BC^2-PB^2}{2\cdot PC\cdot BC}=\dfrac{30t^2+5}{30t}\\\cos(\angle ACB-\theta)=\dfrac34\cos\theta+\dfrac{\sqrt7}4\sin\theta=\dfrac{AC^2+PC^2-AP^2}{2\cdot AC\cdot PC}=\dfrac{20t^2+8}{30t}\]由$\dfrac34<\cos\theta<1$及$\dfrac34<\cos(\angle ACB-\theta)<1$得$\dfrac5{12<t<\dfrac45}$。由$\cos\theta=\dfrac{30t^2+5}{30t}$及$\dfrac34\cos\theta+\dfrac{\sqrt7}4\sin\theta=\dfrac{20t^2+8}{30t}$得\[\sin\theta=\dfrac{20t^2+39}{60\sqrt7t}\] 由$\sin^2\theta+\cos^2\theta=1$得\[(\dfrac{20t^2+39}{60\sqrt7t})^2+(\dfrac{36t^2+5}{30t})^2=1\]整理得\[14400t^2-9360t+1129=0\]解上面的方程,得\[t=\dfrac12\sqrt{\dfrac{39\pm14\sqrt2}{30}}\]只有$t=\dfrac12\sqrt{\dfrac{39\pm14\sqrt2}{30}}$满足$\dfrac5{12}<t<\dfrac45$,所以 只有$t=\frac12\sqrt{\frac{39\pm14\sqrt2}{30}}$满足$\dfrac5{12}<t<\dfrac45$,所以(两者区别)\[t=\dfrac12\sqrt{\dfrac{39+14\sqrt2}{30}}\]所以\[S_{\triangle ABC}=\dfrac{15}4\sqrt7t^2=\dfrac{39\sqrt7+14\sqrt14}{32}\]
李斌斌755 3# 2013-5-13 12:13
本帖最后由 李斌斌755 于 2013-5-18 01:45 编辑 好累人 首先要熟记常用编码,例如:根号\sqrt;分号\frac;上标^;下标_;相似\sim;全等\cong;平行\sslash;垂直\perp等 \[\cdot\\\cdot\\\cdot\]最后要不耻下问学习过程在贴中是我的一点体会(老手略过)
李斌斌755 4# 2013-5-13 12:17
2# 李斌斌755 打完快一小时,老牛拉破车
isea 5# 2013-5-13 12:29
不要在论坛打,论坛打太慢,偶开始也差不多。 在CTEX套装中用, winedt 下编辑 在CTEX中打代码,按F9查看效果,出错按 e ,回车,返回编辑环境 一般情况下(导言区先加这几个宏包,足够了): \documentclass{ctexart}                %会调用 ctex宏包,中文环境已经设置好 \usepackage{amsmath,amssymb,amsthm}   %最常用的三个数学宏包,一般情况下,基本只使用第一个如\dfrac命令 \pagestyle{empty}                                        %取消页眉页脚 %\usepackage{tikz}                                     %绘图用的,刚开始不必用它,比如偶。 \usepackage{esvect}                                    %向量用法 \vv \usepackage{stmaryrd}                                 %平行用法 \sslash \begin{document} %这里输入正文 \end{document}
isea 6# 2013-5-13 12:34
本帖最后由 isea 于 2013-5-13 12:38 编辑 分式很容易出错,依偶的经验,打复杂分式,是先打简单分式,如(当然CTEX下) $\dfrac{15\sqrt7t^2}4$ 先保证不出错,输入 $\dfrac{15}4$ 加根号(可按F9看编译效果,熟悉后,只要输入代码正确,等整个公式编辑完成后,再查看效果): $\dfrac{15\sqrt7}4$ 加平方 $\dfrac{15\sqrt7t^2}4$ 这类长公式全这么处理 不要一次处理特别大型的文档,出错时,很难找到错误的地方 复杂分式是最容易出错的地方。
kuing 7# 2013-5-13 13:32
5# isea 你确定他装了 CTeX?
kuing 8# 2013-5-13 13:33
6# isea 我在 winedt 里定义了快捷键 `// 自动变成 \frac{}{} 并且光标会移动到第一个 {} 内。
李斌斌755 9# 2013-5-13 15:54
7# kuing 这你也知道
李斌斌755 10# 2013-5-14 00:54
解不下去了,当练习 \[S(x_1,x_2,x_3)=\sum_{1\leqslant i<j\leqslant3}x_ix_j=x_1x_2+x_1x_3+x_2x_3\] 1)当$x_1,x_2,x_3$非负或非正时,即$x_1\leqslant0,x_2\leqslant0,x_3\leqslant0$或$x_1\geqslant0,x_2\geqslant0,x_3\geqslant0$,有 \[0\leqslant x_ix_j\leqslant1\riff 0\leqslant S(x_1,x_2,x_3)\leqslant3\] 2)当$x_1,x_2,x_3$且不全同号时,必有$x_1,x_2,x_3$中同号两同号,设$x_1,x_2$同号,有\[\left.\begin{aligned}&&0<x_1x_2\leqslant1\\&&-1\leqslant x_1x_3<0\\&&-1\leqslant x_2x_3<0\end{aligned}\right\}\riff\]
李斌斌755 11# 2013-5-14 00:56
本帖最后由 李斌斌755 于 2013-5-18 10:58 编辑 8# kuing 还有这一手 符号代码后面是数字可直接跟,字母时要加空格或花括号:$x\ge1,x\ge12,x\ge k,x\le ab$ \frac后面一般带有两个{},否则自动默认其后前两字符为分子分母: \frac{12}{37}显示$\frac{12}{37}$,\frac1237显示$\frac1237$ \frac{ab}{a+b}显示$\frac{ab}{a+b}$,\frac ab{a+b}显示$\frac ab{a+b}$\frac ab(a+b)显示$\frac ab(a+b)$
李斌斌755 12# 2013-5-14 01:20
本帖最后由 李斌斌755 于 2013-5-18 01:48 编辑 用\left(内容\right)调整后的效果 \[(\dfrac12,1)\\\left(\dfrac12,1\right)\]
李斌斌755 13# 2013-5-14 01:30
本帖最后由 李斌斌755 于 2013-5-18 01:56 编辑 花括号:$\{a_n\}$             $\{a_n$             $a_n\}$ {aligned}环境中的右花括号(注意所有的括号要放大都需调整) \[\left.\begin{aligned}&&a+b=5\\&&\dfrac12a+2b=4\end{aligned}\right\}\]
李斌斌755 14# 2013-5-14 01:47
本帖最后由 李斌斌755 于 2013-5-14 13:54 编辑 环境: \[\begin{aligned}f(x)&=ax^2+bx+c\\&=a(x-x_1)(x-x_2)\\&=\cdots\end{aligned}\]
李斌斌755 15# 2013-5-14 01:52
本帖最后由 李斌斌755 于 2013-5-14 15:27 编辑 \[\begin{aligned}f(x)&=ax^2+bx+c\\&=a(x-x_1)(x-x_2)\\&=\cdots\end{aligned}\]
李斌斌755 16# 2013-5-14 01:56
本帖最后由 李斌斌755 于 2013-5-17 10:09 编辑 奇怪,14#15#后面没有出现编号啊 因为14#15#用的是{aligned},只有使用{align}才自动编号。
kuing 17# 2013-5-14 12:26
16# 李斌斌755 仔细看置顶
李斌斌755 18# 2013-5-14 13:25
本帖最后由 李斌斌755 于 2013-5-14 13:54 编辑 17# kuing 再试 \[\begin{align}f(x)&=ax^2+bx+c\\&=a(x-x_1)(x-x_2)\\&=\cdots\end{align}\]
李斌斌755 19# 2013-5-14 13:26
本帖最后由 李斌斌755 于 2013-5-14 16:42 编辑 \begin{align}f(x)&=ax^2+bx+c \notag \\&=a(x-x_1)(x-x_2) \label {asdf01234} \\&= \cdots\end{align} 读了,上面我认为对的不显示,下面故意少个字母倒错误得显示 \[\begin{align}f(x)&=ax^2+bx+c \notag \\&=a(x-x_1)(x-x_2) \abel {asdf01234} \\&= \cdots\end{align}\] 更改数字 \begin{align}f(x)&=ax^2+bx+c \notag \\&=a(x-x_1)(x-x_2) \label {asdf0123} \\&= \cdots\end{align}
kuing 20# 2013-5-14 14:55
19# 李斌斌755 因为 asdf01234 这个 \label 标签上面(15#)已经出现过,同一页面内不能出现重复标签。 你将这个标签改一下,比如改成 asdf012345 就可以了,又或者将上面15#的标签删掉也行。 PS、align 环境不必放在 \ [ \ ] 里
thread-1536-2-1.html:
李斌斌755 21# 2013-5-14 15:29
20# kuing 都是,谢谢kuing
李斌斌755 22# 2013-5-14 16:54
本帖最后由 李斌斌755 于 2013-5-14 17:10 编辑 列表 \begin{array}{|i|c|r|r|}\hline1&2&3&4\\\hline a&b&c&d\\\hline\end{array} \begin{array}{|1|c|c|r|}\hline1&2&3&4\\\hline a&b&c&d\\\hline\end{array} \begin{array}{i|c|r|r|}\hline1&2&3&4\\\hline a&b&c&d\\\hline\end{array}
李斌斌755 23# 2013-5-14 17:14
本帖最后由 李斌斌755 于 2013-5-14 17:15 编辑 \[\left(\begin{array}{c|rr}1&2&3\\\hline a&b&c\end{array}\right)\]
李斌斌755 24# 2013-5-14 17:31
本帖最后由 李斌斌755 于 2013-5-14 17:35 编辑 \begin{equation}a^2\le b^2+c^2\end{equation} \begin{align}a^2\le b^2+c^2\end{align} equation环境与align环境没区别
李斌斌755 25# 2013-5-14 17:36
本帖最后由 李斌斌755 于 2013-5-18 02:01 编辑 align环境 \begin{align}\label{csineq} \left(\sum_{k=1}^na_kb_k\right)^2\le\left(\sum_{k=1}^na_k^2\right)\left(\sum_{k=1}^nb_k^2\right). \end{align} 删掉\label{csineq}的效果 \begin{align} \left(\sum_{k=1}^na_kb_k\right)^2\le\left(\sum_{k=1}^na_k^2\right)\left(\sum_{k=1}^nb_k^2\right). \end{align} \label{csineq}只是为下一步引用作铺垫,有它没它对显示效果没影响。
李斌斌755 26# 2013-5-14 18:24
本帖最后由 李斌斌755 于 2013-5-18 02:03 编辑 关于\left……\right,起调整(放大)作用。 小花括号:$\{\dfrac{\dfrac1a}{\dfrac1b}\}$,放大花括号:$\left\{\dfrac{\dfrac1a}{\dfrac1b}\right\}$   小括号:   $(\dfrac{\dfrac1a}{\dfrac1b})$ , 放大小括号:$\left(\dfrac{\dfrac1a}{\dfrac1b}\right)$
李斌斌755 27# 2013-5-15 00:09
本帖最后由 李斌斌755 于 2013-5-15 00:10 编辑 \[\sum_{k=1}^n\dfrac1{k^3+1}+2\] \[\left(\sum_{k=1}^n\dfrac1{k^3+1}+2\right)^2\]
李斌斌755 28# 2013-5-15 00:27
本帖最后由 李斌斌755 于 2013-5-15 00:56 编辑 分段函数:cases环境与aligned环境区别 cases:(自带花括号) $ f(x)=\begin{cases} x+1, &x>0 \\ 1-x, &x<0 \end{cases}$ aligned:(加花括号)$f(x)=\left\{\begin{aligned}x+1, &x>0 \\ 1-x ,&x<0 \end{aligned}\right.$
李斌斌755 29# 2013-5-15 01:13
本帖最后由 李斌斌755 于 2013-5-15 01:17 编辑 不能这样表示\[x\in R\\x\in\mathbf{R}\]效果是不一样。
李斌斌755 30# 2013-5-15 01:19
本帖最后由 李斌斌755 于 2013-5-15 01:41 编辑 $n\in\mathbf N\\\int_a^b f(x)\mathbf{d}x$ \[\int_a^b f(x)\mathrm{d}x\] \[\left\{\begin{aligned}x=2\\x=3\\x=0\\x=5\end{aligned}\right.\]
李斌斌755 31# 2013-5-15 01:45
本帖最后由 李斌斌755 于 2013-5-15 01:52 编辑 美元个矮, $\int_a^b f(x)\mathrm{d}x$ \[ \int_a^b f(x)\mathrm{d}x\]个高
李斌斌755 32# 2013-5-15 16:05
刚才看讨论的$£$符号,ALT+163=$£$,$£$英镑符号
kuing 33# 2013-5-15 16:18
32# 李斌斌755 真 LaTeX 下用 \pounds 就可以打出来,而且不需要任何宏包,没想到这里竟然用不了。
李斌斌755 34# 2013-5-15 19:41
本帖最后由 李斌斌755 于 2013-5-16 02:30 编辑 再练花括号 不同的代码一样的花括号 $\left\{\begin{aligned}\end{aligned}\right.$                     $\{$ $\left.\begin{aligned}\end{aligned}\right\}$                      $\}$ $\{\}$
李斌斌755 35# 2013-5-16 02:33
本帖最后由 李斌斌755 于 2013-5-16 02:39 编辑 $\left\{\begin{aligned}a+2b=5\\3a+b=6\end{aligned}\right.\\\left.\begin{aligned}a+2b=5\\3a+b=6\end{aligned}\right\}\riff\left\{\begin{aligned}a=\frac75\\b=\frac95\end{aligned}\right.$
李斌斌755 36# 2013-5-16 19:50
本帖最后由 李斌斌755 于 2013-5-16 19:52 编辑 定义: $\newcommand\asdf{a_1+a_2+\cdots+a_n}$ $\newcommand\asd{a_1+a_2+\cdots+a_n}$ $\asdf$ $\asd$ 这样就查不出代码了。
李斌斌755 37# 2013-5-16 19:59
36# 李斌斌755 $\newcommand\asdf[1]{a_1+a_2+\cdots+a_{#1}}$ $\asdf{k}\\\asdf{k+1}\\\asdf{n}\\\asdf{n^2}$
李斌斌755 38# 2013-5-16 20:36
本帖最后由 李斌斌755 于 2013-5-18 02:16 编辑 这个公式好 $\begin{align}\dfrac{a^2+b^2}{a+b}&\geqslant\sqrt{\dfrac{a^2+b^2}2}\\&\geqslant\dfrac{a+b}2\\&\geqslant\sqrt{ab}\\&\geqslant\dfrac2{\frac1a+\frac1b}\end{align}$ $\begin{align}\dfrac{a^2+b^2}{a+b}&\geqslant\sqrt{\dfrac{a^2+b^2}2}\notag\\&\geqslant\dfrac{a+b}2\label{2}\\&\geqslant\sqrt{ab}\\&\geqslant\dfrac2{\frac1a+\frac1b}\notag\end{align}$ $\begin{aligned}\dfrac{a^2+b^2}{a+b}&\geqslant\sqrt{\dfrac{a^2+b^2}2}\\&\geqslant\dfrac{a+b}2\\&\geqslant\sqrt{ab}\\&\geqslant\dfrac2{\frac1a+\frac1b}\end{aligned}$ 由$\eqref{2}$知(这里在(5)式代码后加了\label{},同时启用\ eqref{},{}内符号要对应)
李斌斌755 39# 2013-5-17 01:30
本帖最后由 李斌斌755 于 2013-5-17 03:08 编辑 编辑首页中公式 \[(\sum_{k=1}^n a_kb_k)^2\leqslant(\sum_{k=1}^n a_k^2)(\sum_{k=1}^n b_k^2)\] \[\begin{align}\label{1}\left(\sum_{k=1}^n a_kb_k\right)^2\leqslant\left(\sum_{k=1}^n a_k^2\right)\left(\sum_{k=1}^n b_k^2\right)\end{align}\] $f(x)=\left\{\begin{aligned}&2x+1,&x\geqslant0\\&x-1,&x<0\end{aligned}\right.$ 用\left……\right去扩()时,(及)前都加\,结果错误,原来是模仿花括号时出的错,忘了只有花括号前面才加\ 由$\eqref{1}$得
李斌斌755 40# 2013-5-17 02:06
39# 李斌斌755 范例中\label{csineq}的作用也是引用吗?
thread-1536-3-1.html:
李斌斌755 41# 2013-5-18 00:33
本帖最后由 李斌斌755 于 2013-5-18 10:21 编辑 角度:分别用^\circ和\du表示 $30^\circ=30\du$
李斌斌755 42# 2013-5-18 00:44
本帖最后由 李斌斌755 于 2013-5-18 00:48 编辑 \left,\big,\Big,\bigg,\Bigg $\left(\dfrac12,1\right),(\dfrac12,1),\big(\dfrac12,1\big),\Big(\dfrac12,1\Big),\bigg(\dfrac12,1\bigg),\Bigg(\dfrac12,1\Bigg)$
李斌斌755 43# 2013-5-18 00:58
本帖最后由 李斌斌755 于 2013-5-18 01:00 编辑 话说<和n不能放一起,可以呀! $0<x<n$
kuing 44# 2013-5-18 01:26
话说<和n不能放一起,可以呀! $0<x<n$ 李斌斌755 发表于 2013-5-18 00:58 不要断章取义好吧,当时正在说的是草稿本的 bug,跟本论坛一点关系都没有,后来草稿本也修正了这个 bug。
李斌斌755 45# 2013-5-18 02:18
44# kuing 原来如此
李斌斌755 46# 2013-5-18 11:36
本帖最后由 李斌斌755 于 2013-5-18 11:55 编辑 我常犯的错误(不断更新中……) (1)不全在纯英文状态下输入代码,不全用代码编写字符。 (2)编错代码。 (3)常用的数学符号前漏加\:max,min,sin,cos,…… (4)滥用$$及{}。 (5)
李斌斌755 47# 2013-6-1 01:04
学习柯西不等式 \[(a^2+b^2)(c^2+d^2)\geqslant(ac+bd)^2\]取等条件$ad=bc$ 拓展\[(a_1^2+a_2^2 +a_3^2+\cdots+a_n^2)(b_1^2+b_2^2+b_3^2+\cdots+b_n^2)\\\geqslant(a_1b_1+a_2b_2+a_3b_3+\cdots+a_nb_n)^2\]取等条件$\frac{a_1}{b_1}=\frac{a_2}{b_2}=\frac{a_3}{b_3}=\cdots=\frac{a_n}{b_n}$
thread-1537-1-1.html: 11.11之新解
kuing 1# 2013-5-13 11:33
经查,是真的…… 谢谢幻幻提供此图!
isea 2# 2013-5-13 11:43
这个,哈哈
李斌斌755 3# 2013-5-13 12:04
thread-1538-1-1.html: 数学老师有三宝
kuing 1# 2013-5-13 12:49
爱好者-Tesla35(3705*****)  12:48:30 数学老师有三宝:同理,显然,这不考。
isea 2# 2013-5-13 14:33
还有没有新三宝
thread-1539-1-1.html: [不等式] 人教数学群早上提到的一道几何不等式
kuing 1# 2013-5-13 14:18
教师-穿越时空 2013-5-13 10:47:58 修正一下: 证明 非锐角三角形最大边及其该边的高之和大于另两边之和。    对于哪些锐角三角形最大边及其该边的高之和大于另两边之和? 先证明一下第一个问题,不知下面这样证是否正确? 首先证明直角三角形的情况成立,这是容易的。 不妨设 $\triangle ABC$ 对应三边长为 $a$, $b$, $c$ 且 $a$ 为最大边,其上的高为 $h$,那么当三角形为直角三角形时 \[a+h>b+c \iff a-b>c-h \iff a(1-\sin B)>c(1-\sin B),\] 显然成立; 然后再证明非锐角三角形的情况成立。 此时等价于证明在 $a^2\geqslant b^2+c^2$ 的情况下 \[a+\frac{2S}a>b+c,\] 去分母移项平方等价于 \[16S^2>4a^2(b+c-a)^2,\] 由海伦公式,等价于 \[(a+b+c)\prod(a+b-c)>4a^2(b+c-a)^2,\] 约去一个 $b+c-a$ 后,等价于 \[f(a)=(a+b+c)(a+b-c)(a-b+c)-4a^2(b+c-a)>0,\] 我们对 $a$ 求导,易得 \[f'(a)=2a^2+6a(2a-b-c)+(a+b-c)(a-b+c)>0,\] 因此,我们只需要证明当 $a^2=b^2+c^2$ 时即可,而这就是前面已证的直角三角形的情形,故命题得证。
李斌斌755 2# 2013-5-13 15:05
放个板凳占个位
kuing 3# 2013-5-13 15:13
第二个问题其实没有表达清楚,因为如果你问的是“哪些……”,那么我完全可以说“就是满足 $f(a)>0$ 的那些……”,那样的话…… 可以问得更清楚,而且方式不一,比如说,可以考虑这样的问题: 求最小的正数 $\alpha$,使得当三角形的最大角大于 $\alpha$ 时恒有 $a+h>b+c$($a$, $b$, $c$, $h$ 的意思仍如1#所设,下同); 又或者: 求最小的正数 $k$,使得当三角形的最大边 $a$ 满足 $a>k(b+c)$ 时恒有 $a+h>b+c$; 等等…… 1#的思路还是能使的,感觉第二个问法的会更简单些……但是好像又涉及三次方程,又好像不是,待续……
hejoseph 4# 2013-5-13 15:28
如果是锐角三角形最大角固定的话就不可能恒成立了,但可以讨论哪些角度可使$a+h_a>b+c$能成立
kuing 5# 2013-5-13 15:51
如果是锐角三角形最大角固定的话就不可能恒成立了,但可以讨论哪些角度可使$a+h_a>b+c$能成立 hejoseph 发表于 2013-5-13 15:28 噢对,等腰的时候肯定不能成立……3#可以扔掉了……
kuing 6# 2013-5-13 16:49
换个角度,用坐标来考虑吧。 我们不妨设 $a=2$,令 $B(1,0)$,$C(-1,0)$,设 $A(x,y)$,其中 $y>0$。则 \[a+h>b+c \iff 2+y>\sqrt{(x-1)^2+y^2}+\sqrt{(x+1)^2+y^2},\] 平方移项再平方,整理后最终等价于 \[(4 y + 16) x^2 + 3 y^3 + 8 y^2 - 4 y - 16 < 0,\] 即 \[x^2<\frac{16+4y-8y^2-3y^3}{16+4y},\] 把图作出来 可以看出,曲线包含在两个大圆内部,所以在紫色线里面的点也满足 $a$ 是最大边(其实是废话,因为如果 $a$ 不是最大边,显然 $a+h<b+c$),因此在紫色线内部的点就是所求。 于是只要研究一下那条曲线的特性,大概就能得到一些结论。 ……
李斌斌755 7# 2013-5-13 20:22
直角三角形情况: $\triangle ABC,\angle ACB=90^\circ$,正方形$BCMN$,$\triangle BDC\cong\triangle NLB,LB=DC$,\[MN=NB>NL\riff AL>AM\]
kuing 8# 2013-5-13 20:25
7# 李斌斌755 直角的没难度啦,不过你这个整得也蛮漂亮
李斌斌755 9# 2013-5-13 20:29
8# kuing 试着一步步来,看能走多远
yes94 10# 2013-5-15 22:53
直角的确实没难度,下面写一下试试: \[\begin{array}{l} A \ge B \ge C,2S = ah = bc\sin A,{\kern 1pt} {\kern 1pt} {\kern 1pt} {\kern 1pt} {a^2} = {b^2} + {c^2} - 2bc\cos A,h = \frac{{bc\sin A}}{a}\\ \Rightarrow {(a + h)^2} = {a^2} + 2ah + {h^2} = {b^2} + {c^2} - 2bc\cos A + 2bc\sin A + {(\frac{{bc\sin A}}{a})^2} = {(b + c)^2} + 2bc(\sin A - \cos A - 1) + {(\frac{{bc\sin A}}{a})^2}\\ a + h > b + c \Leftrightarrow {(a + h)^2} > {(b + c)^2} \Leftrightarrow 2bc(\sin A - \cos A - 1) + {(\frac{{bc\sin A}}{a})^2} > 0 \Leftrightarrow 2(\sin A - \cos A - 1) + bc{(\frac{{\sin A}}{a})^2} > 0\\ \Leftrightarrow 2(\sin A - \cos A - 1) + \sin B\sin C > 0 \Leftrightarrow 2(\sin A - \cos A - 1) - \frac{1}{2}[\cos (B + C) - \cos (B - C)] > 0\\ \Leftrightarrow 2\sin A - \frac{3}{2}\cos A{\rm{ + }}\frac{1}{2}\cos (B - C) > 2 \end{array}\]
yes94 11# 2013-5-15 22:54
10# yes94 当$A=\dfrac{\pi}2$时,上式显然成立。
thread-154-1-1.html: [转]LaTeX里使用弧符号
kuing 1# 2011-10-26 15:16
转自 http://bbs.ctex.org/viewthread.php?tid=49447 用这个 \DeclareSymbolFont{ugmL}{OMX}{mdugm}{m}{n} \SetSymbolFont{ugmL}{bold}{OMX}{mdugm}{b}{n} \DeclareMathAccent{\wideparen}{\mathord}{ugmL}{"F3} 确实不错
thread-1540-1-1.html: [转]我饿了
kuing 1# 2013-5-13 14:38
“我饿了”是句很考验人的话。 对妈妈说:我饿了。妈妈就会马上起身说:“我去给你弄吃的。” 对爸爸说:我饿了。爸爸就会笑笑说:“好,我们到外面去吃。” 对男朋友说:我饿了。他就会温柔地说:“想吃什么呢?” 对女朋友说:我饿了。她永远会回答:“我也是!” 因此越来越多的男孩选择找个男朋友。。。
李斌斌755 2# 2013-5-13 15:14

╰☆ヾo.海x 3# 2013-5-14 07:07
我才不会那么说。。我要抢着比他说的早!哈哈哈
李斌斌755 4# 2013-5-15 01:56
3# ╰☆ヾo.海x 女生
thread-1541-1-1.html: [数列] 这种数列形式多元和式的最值 有何好的切入点
isea 1# 2013-5-13 16:37
本帖最后由 isea 于 2013-5-13 16:44 编辑 求助,主要是第(2)问,这种多元的和式最值,经常在高三模拟试卷中出现。 偶基本按答案“宣科”(直接发答案,自学),觉得实在是郁闷啊,请教。 如下实例(答案反白,一般浏览器,全选即可见答案): 题:已知实数$x_1,x_2,\cdots,x_n(n\ge 2,n\in N)$满足$|x_i|\le1$, 记$\displaystyle S(x_1,x_2,\cdots,x_n)=\sum_{1\le i<j\le n}x_ix_j~~(i,j\in N)$. (1)求$S(-1,1,-\dfrac23)$及$S(1,1,-1,-1)$的值;key:$-1;-2$ (2)当$n=3$时,求$s(x_1,x_2,x_3)$的最小值(PS:上界3?);key:$-1$ (3)求$S(x_1,x_2,\cdots,x_n)$的最小值.key:$n$为偶数时,$-\dfrac n2$,奇数时$-\dfrac{n-1}2$
kuing 2# 2013-5-14 00:54
很早很早以前就见过类似的题哩,我还记得是在最早期的数学驿站论坛里解的,考虑完全平方。
李斌斌755 3# 2013-5-14 00:58
刚才一阵好想,无果
李斌斌755 4# 2013-5-14 01:12
与这类,遇到就晕…… http://kkkkuingggg.5d6d.net/view ... p;extra=&page=1
kuing 5# 2013-5-14 01:17
\[\left(\sum_{i=1}^nx_i\right)^2=\sum_{i=1}^nx_i^2+2\sum_{1\le i<j\le n}x_ix_j\]
李斌斌755 6# 2013-5-14 01:59
5# kuing 唉……
isea 7# 2013-5-14 08:25
\[\left(\sum_{i=1}^nx_i\right)^2=\sum_{i=1}^nx_i^2+2\sum_{1\le i kuing 发表于 2013-5-14 01:17 直接被秒了,此式一出
yes94 8# 2013-5-15 23:38
7# isea 怎么秒啊? 朝阳
kuing 9# 2013-5-15 23:51
首先由于每个元相互独立而且 $S$ 对每个元均为线性,故由 $\abs{x_i}\leqslant1$ 知只需考虑 $x_i\in\{-1,1\}$ 时,此时 \[\left(\sum_{i=1}^nx_i\right)^2=\sum_{i=1}^nx_i^2+2\sum_{1\le i<j\le n}x_ix_j=n+2\sum_{1\le i<j\le n}x_ix_j.\] 当 $n$ 为偶数时,$\left(\sum_{i=1}^nx_i\right)^2$ 最小可取到 $0$,故 $\sum_{1\le i<j\le n}x_ix_j$ 的最小值为 $-n/2$; 当 $n$ 为奇数时,$\left(\sum_{i=1}^nx_i\right)^2$ 最小可取到 $1$,故 $\sum_{1\le i<j\le n}x_ix_j$ 的最小值为 $(1-n)/2$。
yes94 10# 2013-5-15 23:57
9# kuing 牛笔!
syzxzxy 11# 2013-5-16 00:36
由于所有变量都是相互独立的,所以用主元法考虑 这样,该式子对每个变量都是线性函数,必在端点取得最值 所以一定是若干个取零,若干个取1时,取得最值 这样是否可以呢? 不会输入数学公式
李斌斌755 12# 2013-5-16 00:41
11# syzxzxy 有这一说,想想……
kuing 13# 2013-5-16 00:41
11# syzxzxy 先看9#
kuing 14# 2013-5-16 00:42
12# 李斌斌755 我9#就已经写了啊
syzxzxy 15# 2013-5-16 07:53
再考虑这样的函数: f(x)=/x-a/+/x-b/+/x-c/+/x-d/+。。。。+/x-z/ 其中a,b,c,.....,z为常数 求这样函数的最值有哪些方法呢?
yes94 16# 2013-5-16 12:59
15# syzxzxy 不是将零点排列起来么?然后绝对值不等式
李斌斌755 17# 2013-5-16 13:31
14# kuing 我也后来看了
thread-1542-1-1.html: 这也行
isea 1# 2013-5-13 17:28
悠闲数学娱乐论坛 百度,goolge都是第一个
kuing 2# 2013-5-13 17:31
那不是很正常么,没别个了啊…… 当初起这个名字的时候,会不会有重复或者类似论坛名字是在考虑之内的……
thread-1543-1-1.html: 第13期初稿交上去了,这次看看要等多久
kuing 1# 2013-5-13 17:34
正常情况下还需要返回修改意见若干,然后改好再交一次上次才能出,看看能不能在高考前出。
kuing 2# 2013-5-13 17:37
这次何版主在封面做了一个真3D图,就像这个贴里的那样子,PDF阅读器支持的话,是能动的。
isea 3# 2013-5-13 22:33
真牛图
kuing 4# 2013-5-24 21:49
十天过去了,连修改意见都还没返回,看来高考前出的可能性不大了
isea 5# 2013-5-24 23:50
一样一样,考前考后,都质量大大地呐~
kuing 6# 2013-5-25 00:00
考后出的话,估计看的人会少些,因为眼球会被高考强力地吸引着……
kuing 7# 2013-5-27 15:18
十天过去了,连修改意见都还没返回,看来高考前出的可能性不大了 kuing 发表于 2013-5-24 21:49 今天终于有反应,返回了修改意见,现已改好再次交了上去,等出了。 离高考还有10天,看看能不能出来。
isea 8# 2013-5-28 01:13
强烈要求出啊,你们几个表这个态就即可 若能,真好
kuing 9# 2013-5-28 01:52
8# isea 出肯定是能出的,时间问题,要看“上头”……
╰☆ヾo.海x 10# 2013-5-28 01:55
6# kuing 高考出来,你会不会继续做题目贴啊。。
kuing 11# 2013-5-28 01:57
10# ╰☆ヾo.海x 还不知道,看心情吧……
李斌斌755 12# 2013-5-28 01:58
10# ╰☆ヾo.海x +幻幻=
╰☆ヾo.海x 13# 2013-5-28 02:30
12# 李斌斌755 。。。。无聊。。
isea 14# 2013-5-28 02:34
没事多等几天也无所谓
kuing 15# 2013-6-6 11:38
今天出了,考前一天出,这样也赶到了,牛笔…… http://www.pep.com.cn/rjwk/gzsxsxkj/ http://www.pep.com.cn/rjwk/gzsxsxkj/sxkj2013/sxkj13/
isea 16# 2013-6-6 18:51
拜读下
thread-1545-1-1.html: 预测
李斌斌755 1# 2013-5-13 20:37
现在会员600+49,过多久达1000. 两月
kuing 2# 2013-5-13 20:43
最新会员的 UID 不一定等于实际会员数。 现时实际的会员数只有 568。 原因是以前发现广gao用户的时候我除了删贴外还会将该帐号也删除,所以实际会员数会减少,但是 UID 是不会变的。 不过现在我最近都变懒了,禁言就算了,懒得删帐号。 PS、编辑了一下1#的数字
kuing 3# 2013-5-13 20:44
就算是UID,我估计要超过1000至少要到2014年
李斌斌755 4# 2013-5-17 00:44
3# kuing 我不管,我按“论坛统计”上的数来算
kuing 5# 2013-5-17 00:47
4# 李斌斌755 论坛统计首页第一行就显示目前是 注册会员        575 是实际数目
kuing 6# 2013-5-17 00:48
咦?这样看来还蛮快……难道真的不用2014?
李斌斌755 7# 2013-5-28 15:51
这两天更快
thread-1546-1-1.html: 哎。。
╰☆ヾo.海x 1# 2013-5-14 07:13
酷儿 ....我觉得我这样下去不行了。。明天开始断网!!!不准开电脑,手机上网不超过半小时每天!! 监督啊....
kuing 2# 2013-5-14 12:25
你又来……
李斌斌755 3# 2013-5-15 01:59
2# kuing 她是…女的
kuing 4# 2013-5-15 02:00
3# 李斌斌755 你怎么看出来的?
李斌斌755 5# 2013-5-15 02:06
4# kuing 见这个贴中她的回复内容   http://kkkkuingggg.5d6d.net/thread-1540-1-1.html
李斌斌755 6# 2013-5-15 02:18
4# kuing 还38的知道在境外
╰☆ヾo.海x 7# 2013-5-15 02:32
6# 李斌斌755 你怎么知道。。
李斌斌755 8# 2013-5-15 02:48
7# ╰☆ヾo.海x 我不知道我怎么知道
isea 9# 2013-5-15 11:42
时差
kuing 10# 2013-5-15 13:38
9# isea 楼主是随机时差……
╰☆ヾo.海x 11# 2013-5-15 18:58
10# kuing
李斌斌755 12# 2013-5-17 00:54
楼主来了
thread-1547-1-1.html: [几何] 又是人教群wwd的解几问题
kuing 1# 2013-5-14 18:08
问题:$y=x^2$ 上,$P$ 为动点,求垂直于过 $P$ 的切线的弦 $PQ$ 的最小值。 这个用解析法应该不难,但是 wwd 说 教师-wwdwwd117(2365*****)  17:45:39 我其实猜出了结果,就是当PQ过(0,2p)时 教师-wwdwwd117(2365*****)  17:46:19 就是证明不出 教师-wwdwwd117(2365*****)  17:47:43 也就是PO垂直于QO时 [中间省略一段] 教师-wwdwwd117(2365*****)  18:04:04 不过呢,看到那么好的几何结论,就想有没有好的几何方法。 我也希望看到几何法,故先转上来。
李斌斌755 2# 2013-5-14 20:17
本帖最后由 李斌斌755 于 2013-5-14 20:21 编辑 先上个图。 $FO=\dfrac12,FO\perp$直线$l$,$M,N$是直线$l$上的两个动点,$PM\sslash OF\sslash QN$,$PC,QD$分别是$MF,NF$的垂直平分线。当$PQ\sslash CF$时,求$PQ$的最小值。
李斌斌755 3# 2013-5-14 20:49
2# 李斌斌755 不好弄,两个动点
李斌斌755 4# 2013-5-14 20:53
2# 李斌斌755 $OF$的中点(原点)使不上劲?
0.1 5# 2013-5-14 21:35
多么美妙的结论啊
isea 6# 2013-5-14 23:55
如果这是真的,那么其他圆锥曲线亦有类似性质
零定义 7# 2013-5-15 01:04
发现“PQ的最小值”等价于“PF+FN的最小值”...如果能证明的话,相信会简单很多...
李斌斌755 8# 2013-5-15 01:11
7# 零定义 $P,F,N$成一条直线,也不好证明啊!
零定义 9# 2013-5-15 01:21
8# 李斌斌755 确实,感觉我有bào力倾向...
李斌斌755 10# 2013-5-15 14:43
本帖最后由 李斌斌755 于 2013-5-17 09:50 编辑 $FO=OE=\dfrac12,FO\perp$直线$l$,$M,N$是直线$l$上的两个动点,$PM\sslash OF\sslash QN,PM=PF,QN=QF,PH\perp QN$,求$PQ$的最小值。 设$MO=a,NO=b$,\[\left.\begin{aligned}\triangle PMF\sim\triangle MEF\\\triangle PNF\sim\triangle NEF\end{aligned}\right\}\riff\left\{\begin{aligned}\dfrac{PM}{MF}=\dfrac{MF}{EF}\\\dfrac{PN}{NF}=\dfrac{NF}{EF}\end{aligned}\right.\riff \left\{\begin{aligned}PM=MF^2=a^2+\dfrac14\\PN=NF^2=b^2+\dfrac14\end{aligned}\right.\\\triangle MOF\sim\triangle PHQ\riff\dfrac{MO}{OF}=\dfrac{PH}{HQ}=\dfrac{a}{\frac12}=\dfrac{b+a}{b^2-a^2}\riff b=a+\dfrac1{2a}\\PQ^2=PH^2+HQ^2=(a+b)^2+(b^2-a^2)^2\\=3a^2+\dfrac3{4a^2}+\dfrac{a^2}2+\dfrac{a^2}2+\dfrac1{16a^4}+3\geqslant3+\dfrac34+3=6\dfrac34\\\riff PQ\geqslant\dfrac{3\sqrt3}2\] 取等条件$a=\dfrac{\sqrt2}2$
李斌斌755 11# 2013-5-15 14:54
10# 李斌斌755 确实过$(0,2p)$点,但与原点……
零定义 12# 2013-5-15 15:11
10# 李斌斌755 牛笔!!!不过类似最后结果...
李斌斌755 13# 2013-5-15 15:15
12# 零定义 答案是多少?
零定义 14# 2013-5-15 15:18
10# 李斌斌755 牛笔!!!昨晚我也类似这样算,可没找到a与b的关系,算不出来... 最后,弱弱的说一个,最后结果... 发现PQ取得最小值时,有很多性质啊,比如:P、F、N三点共线,∠POQ=90°...
零定义 15# 2013-5-15 15:20
13# 李斌斌755 杂么今天论坛卡卡D...你算错数了,答案应该是3√3/2吧...
李斌斌755 16# 2013-5-15 15:28
15# 零定义 谢谢!是算错了$\sqrt{\dfrac{27}4}=\dfrac{3\sqrt3}2$,已修改。
yes94 17# 2013-5-15 21:49
10# 李斌斌755 几何控! 没仔细看,你画的什么啊?
李斌斌755 18# 2013-5-16 00:28
17# yes94 好久不见,抛物线呀
kuing 19# 2013-5-16 17:08
又来试试用速度来看此题。 我们不妨假设 $P$ 在抛物线的右半边上,并且让它在高处沿着抛物线以恒定速率 $v_P$ 下滑,由此引起左边的点 $Q$ 也有一个速率 $v_Q$,如图所示。 由于 $P$ 的速度对于 $l$ 来说始终垂直,而 $Q$ 的速度显然不会与 $l$ 垂直,于是对于 $\abs{PQ}$ 来说,它是在增加还是减少,完全取决于 $Q$ 点的速度方向,即,如果 $Q$ 在下滑,那么 $\abs{PQ}$ 在减少(就像图中显示的那样),如果 $Q$ 在上滑(会的)则 $\abs{PQ}$ 在增加,由此可见,如果 $\abs{PQ}$ 存在最小值,则必然在 $v_Q=0$ 时取得。 而 $v_Q=0$ 的那一刻,我们又可以看成 $P$ 绕 $Q$ 作匀速圆周运动,$\abs{PQ}$ 为其半径,这说明,当 $v_Q=0$ 时,抛物线在 $P$ 处的曲率圆(密切圆)的圆心就是 $Q$。 然后似乎也没什么妙法整下去,如果用曲率圆的相关公式,那又变回代数了(虽然只是要解一个方程而已),咳,待续着先……
沉香陈香 20# 2013-5-29 17:53
这题和论坛上南通的那题类似,提供一种通法
thread-1547-2-1.html:
李斌斌755 21# 2013-5-29 18:47
这个宝莲灯向量熟练
thread-1549-1-1.html: 请教一个集合、几何题
hongxian 1# 2013-5-15 07:02

kuing 2# 2013-5-15 10:07
怎么看都更像数论题。。。 我弱,再睡一会再说
hejoseph 3# 2013-5-15 10:23
$B$就是$y=3x^2+12$,$x^2+\left(3x^2+12\right)^2-108=9x^4+73x^2+36>0$,与$x^2+y^2\leqslant 108$矛盾。 另外,那个空集符号怎么用那个希腊字母?
realnumber 4# 2013-5-15 11:40
3# hejoseph 是点(a,b),不是B中的点.
hejoseph 5# 2013-5-15 11:47
本帖最后由 hejoseph 于 2013-5-15 11:50 编辑 看错了,这样就行: $ax+b=3x^2+12$得$a^2+12b-144\geqslant0$,$a^2+12b-144\geqslant0$,$a^2+b^2\leqslant108$得$a=\pm6\sqrt2$,$b=6$,显然不满足$ax+b=3x^2+12$。
realnumber 6# 2013-5-15 12:04
本帖最后由 realnumber 于 2013-5-15 12:06 编辑 比我简单多了,$ma+b=3m^2+12$,解得$b=3m^2+12-ma$ 即需要满足$a^2+{(3m^2+12-ma)}^2\le 108$,才检验了a=0,a=1不成立,需要检验$\abs a \le10$
hongxian 7# 2013-5-15 12:21
5# hejoseph 谢谢了! 空集符号在mathtype中原来一直都是错用成了$\varphi$,今后会用$\varnothing$了
力工 8# 2013-5-15 13:06
学习,远离参与
kuing 9# 2013-5-15 13:41
原来那么简单……
yes94 10# 2013-5-15 23:10
一个老题的变式: 原来B是$y=3m^2+15$,C是$x^2+y^2\leqslant144$,方法很多,有一种是用点到直线的距离做的
thread-155-1-1.html: 拼音音调的输入
kuing 1# 2011-10-26 15:17
先说明:本贴是为真 LaTeX 而写,暂时不适用于本论坛的 MathJax。 \={a} 第一声 \'{a} 第二声 \v{a} 第三声 \`{a} 第四声 i 要特别点 \={\i} 第一声 \'{\i} 第二声 \v{\i} 第三声 \`{\i} 第四声
thread-1550-1-1.html: [几何] 这道解析几何试题的几何背景是什么?
pengcheng1130 1# 2013-5-15 20:53
求解高手 ______kuing edit in $\LaTeX$______ 椭圆方程:$\dfrac{x^2}{a^2}+\dfrac{y^2}{b^2}=1$($a>b>0$),点 $P$ 是长轴上任意一点,过点 $P$ 作 $CD$ 的平行线交椭圆于 $A$, $B$ 两点,则 $x_A^2+x_B^2$ 为定值。
kuing 2# 2013-5-15 21:03
化成圆,全等,勾股定理。
pengcheng1130 3# 2013-5-15 21:07
这个是不是仿射变换啊!
kuing 4# 2013-5-15 21:21
我不知道这些,只知道可以那样上下拉伸,横坐标不变,相切的依然相切,平行的依然平行
yes94 5# 2013-5-15 22:30
定值为$x_A^2+x_B^2=a^2$,$y_A^2+y_B^2=b^2$?
kuing 6# 2013-5-15 22:31
5# yes94 显然$^2$
isea 7# 2013-5-15 22:32
本帖最后由 isea 于 2013-5-15 23:11 编辑 3# pengcheng1130 就是的。 一般相关数学杂志叫伸缩变换。 受 圆锥曲线的几何性质 的影响,我一般直接叫投影。
yes94 8# 2013-5-15 23:05
和共轭直径有没有关系?
李斌斌755 9# 2013-5-16 00:34
你们说的全不懂
hongxian 10# 2013-5-16 16:56
本帖最后由 hongxian 于 2013-5-16 18:10 编辑 9# 李斌斌755 我来理解一个,不知对不对? 变圆之后 $\abs{x_A}=\abs{DA}$,$\abs{x_B}=\abs{OC}=\abs{CE}=\abs{OD}$ $\therefore x_A^2+x_B^2=\abs{DA}^2+\abs{OD}^2=\abs{OA}^2=a^2$
kuing 11# 2013-5-16 18:36
10# hongxian 大概差不多吧,反正一拉就是显然的了,后面随便证……
thread-1551-1-1.html: [不等式] 编个简单的
pxchg1200 1# 2013-5-15 22:46
本帖最后由 pxchg1200 于 2013-5-15 22:49 编辑 设$a,b,c>0$且有$ab+bc+ca+abc=4$证明 \[ \sqrt[2013]{a+2}+\sqrt[2013]{b+2}+\sqrt[2013]{c+2}\geq 3^{\frac{2014}{2013}}\]
reny 2# 2013-5-23 21:18
本帖最后由 reny 于 2013-5-23 21:27 编辑 这种条件的,貌似和三角有点关系,也可能没关系,说是简单,还是不会证啊.  楼主贴出看. 或证明一般情况:$ \sqrt[n]{a+2}+\sqrt[n]{b+2}+\sqrt[n]{c+2}\geq 3^{\frac{n+1}{n}}.$
reny 3# 2013-5-23 22:12
本帖最后由 reny 于 2013-5-23 22:15 编辑 1# pxchg1200 跟一个相同条件的: 设$a,b,c>0$,且有$ab+bc+ca+abc=4,$试证 $$\dfrac{a^3}{bc}+\dfrac{b^3}{ca}+\dfrac{c^3}{ab}\geqslant3. $$
零定义 4# 2013-5-23 22:46
2013随意装上的?换上n没问题?
reny 5# 2013-5-23 22:58
本帖最后由 reny 于 2013-5-23 23:18 编辑 4# 零定义 只是我的猜测呢,还不知咋证,不过我跟那个可以证(想用均值“秒”应该不行滴).
pxchg1200 6# 2013-5-24 00:16
3# reny 注意到条件就是 $$ \frac{1}{a+2}+\frac{1}{b+2}+\frac{1}{c+2}=1 $$ AM-GM后 \[ a+b+c\geq 3 \] 而由Holder \[ \left(\frac{a^3}{bc}+\frac{b^3}{ac}+\frac{c^3}{ab}\right)(a+b+c)^2\geq (a+b+c)^3 \] 结论显然.
pxchg1200 7# 2013-5-24 00:18
2# reny 很高兴有人弄的我的不等式。哈哈
李斌斌755 8# 2013-5-24 00:20
6# pxchg1200 你的题我们怎作,你扛的武器(用的公式)我们见都没见过
kuing 9# 2013-5-24 00:22
6# pxchg1200 3#那个不用变形也显然能看出a+b+c>=3...
reny 10# 2013-5-24 10:46
本帖最后由 reny 于 2013-5-24 10:48 编辑 2# reny ouch!根据你证我跟那个题,提示我可以这样做: 注意到恒等式$\dfrac{1}{a+2}+\dfrac{1}{b+2}+\dfrac{1}{c+2}=1$(知道她的存在,没想到要用它) 由于$f(x)=x^{-\dfrac1{n}},x\in(0,1)$是凸函数, 所以$\sqrt[n]{a+2}+\sqrt[n]{b+2}+\sqrt[n]{c+2}\geqslant3\left(\dfrac1 3 \sum{\dfrac1{a+2}}\right)^{-\frac1{n}}=3^{\dfrac{n+1}{n}}.$
reny 11# 2013-5-24 13:22
本帖最后由 reny 于 2013-5-25 23:16 编辑 在相同条件下,再跟三道证明: $(1)\sqrt{ab}+\sqrt{bc}+\sqrt{ca}\geqslant\sqrt[4]{ab}+\sqrt[4]{bc}+\sqrt[4]{ca};$(改正) $(2)(2a-bc)(2b-ca)(2c-ab)\geqslant a^2b^2c^2;$ $(3)\abs{a-b}+\abs{a-c}\geqslant\abs{bc-1}+(1-a)(1-b)(1-c).$ 欢迎解答啊!
reny 12# 2013-5-24 16:51
怎么木有人顶起啊
pxchg1200 13# 2013-5-25 11:36
12# reny 做不来。
reny 14# 2013-5-25 22:55
本帖最后由 reny 于 2013-5-25 23:16 编辑 11# reny 先看(1): 令$a=\dfrac{yz}{x},b=\dfrac{xz}{y},a=\dfrac{xy}{z},$则已知条件变为$x^2+y^2+z^2+xyz=4,$ 欲证式$\sqrt{ab}+\sqrt{bc}+\sqrt{ca}\geqslant\sqrt[4]{ab}+\sqrt[4]{bc}+\sqrt[4]{ca}\iff x+y+z\geqslant\sqrt{x}+\sqrt{y}+\sqrt{z}.$ 结合恒等式$\dfrac{x}{2x+yz}+\dfrac{y}{2y+zx}+\dfrac{z}{2z+xy}=1$(和条件等价), 由柯西,$\sum_{cyc}{\dfrac{x}{2x+yz}}.\sum_{cyc}{(2x+yz)}\geqslant\left(\sum_{cyc}{\sqrt{x}}\right)^2$, i.e,$2(x+y+z)+xy+yz+zx\geqslant\left(\sqrt{x}+\sqrt{y}+\sqrt{z}\right)^2.(*)$ 由AMO2001结论——$xy+yz+zx\leqslant 2+xyz$及条件$x^2+y^2+z^2+xyz=4,$有$\left(x+y+z\right)^2\geqslant2(x+y+z)+xy+yz+zx(**).$ 由$(*)和(**)$知,不等式得证.
thread-1552-1-1.html: [数论] 多项式
Gauss门徒 1# 2013-5-12 18:50
求多项式满足首相系数=1其余项系数不是1就是-1并且没有复数根
hejoseph 2# 2013-5-14 09:12
复数是包含实数的,你这种叙述就很不严格了。如果是不含虚数,那么$x^3+x^2-x-1$就是一个例子,但是你找这些多项式有什么用?
Gauss门徒 3# 2013-5-16 00:46
求所有系数非1则-1的多项式并且它的根都是实数
thread-1553-1-1.html: [组合] 转一个概率题,也没答案的
realnumber 1# 2013-5-16 09:53
安徽郑老师(145####81) 2013-05-15 21:52:31 (1994年美国高中数学考试题)掷n次普通骰子得到点数和为1994的概率大于0,且与得到点数和为S的概率相等,则S的最小值是——
地狱的死灵 2# 2013-5-16 10:37
本帖最后由 地狱的死灵 于 2013-5-16 10:47 编辑 1994=6×333-4 即至少“投掷333次骰子点数和为1994,至多4次不是6”. 这与“投掷333次骰子点数和为S,至多4次不是1”的概率相同, S=1×333+4=337
thread-1554-1-1.html: [函数] 方程系数问题
realnumber 1# 2013-5-16 10:54
______kuing edit in $\LaTeX$______ 5、(2011 北大)已知函数 $f(x)=x^2+px+q$,且 $f(f(x))=0$ 仅有一个实根,求证:$p\geqslant 0$, $q\geqslant 0$。
hejoseph 2# 2013-5-16 11:24
这题重根算几个?
李斌斌755 3# 2013-5-16 12:12
2# hejoseph 还真不知道?
isea 4# 2013-5-16 12:43
据求证结果,当p,q同为零时来看,这题重根只算一个
yes94 5# 2013-5-16 12:55
自主招生?保送生测试? 有没有答案或链接啊?
kuing 6# 2013-5-16 13:05
嗯,求原卷确认…… “只有一个实数根”和“只有一个实数解”是不同的…… 按道理,2重根算2个根。
realnumber 7# 2013-5-16 13:53
群里看到转的, 百度了下,http://www.docin.com/p-194795922.html
kuing 8# 2013-5-16 14:34
如果是按“只有一个实数解”来算,也就是说重根只算一个解的话,其实好像也不难吧,就是 $f(x)$ 的最小值等于 $f(x)$ 的较大根,即 \[\frac{4q-p^2}4=\frac{-p+\sqrt{p^2-4q}}2,\] 若 $q<0$,则右边为正,而左边为负,矛盾;另一方面,变形得 \[p=\sqrt{p^2-4q}+\frac{p^2-4q}2\geqslant 0,\] 得证。
kuing 9# 2013-5-16 14:40
根据虚根成对,偶次多项式不可能只有一个实数根
hejoseph 10# 2013-5-16 14:43
题目有许多叙述问题,例如$p$、$q$就是实数了?
kuing 11# 2013-5-16 14:52
10# hejoseph 的确这也是个问题…… 命题的应该尽可能避免这些问题,多说明一句又不费力……
realnumber 12# 2013-5-16 22:31
高中教材没代数基本定理,重根也只是在初中教2次方程时涉及,猜是绝大多数 老师不会推广到一般的重根概念,于学生应该不会误会. 改成解集中有且只有一个实数当然更严密.
yes94 13# 2013-5-16 22:56
能不能找到2个$p>0,q>0$的例子,并且该方程只有唯一的一个实数解?
kuing 14# 2013-5-16 23:06
13# yes94 从 8# 的方程里找就可以了呗,比如说 $p=4$, $q=3$;$p=3/2$, $q=5/16$;等等……
thread-1555-1-1.html: 有约定么
isea 1# 2013-5-16 22:20
最大值,大家写哪个?A.$f(x)_{\max}$     B.$f_{\max}(x)$. 似乎这两都很流行的样子
kuing 2# 2013-5-16 22:25
应该没这类约定,但是如果让我两种选一中,我会用前者。
李斌斌755 3# 2013-5-16 22:44
$\newcommand\asdf[1]{f_1(x)+f_2(x)+\cdots+f_{#1}(x)}$ 第一种易与$\asdf{n}$混淆
thread-1556-1-1.html: logo 太小了
isea 1# 2013-5-17 00:28
不好点上,返回不方便
李斌斌755 2# 2013-5-17 00:46
1# isea logo是啥?
kuing 3# 2013-5-17 00:50
啊,原来你是点LOGO返回的? 我一般是点LOGO下面的链接,或者右上方的“论坛”
thread-1557-1-1.html: 选手
李斌斌755 1# 2013-5-17 00:37
本帖最后由 李斌斌755 于 2013-5-17 00:40 编辑 高等数学讨论$\longleftrightarrow$十米高台,初等数学讨论$\longleftrightarrow$三米跳板。 高台选手不多,跳板难度系数(指出题)最大的选手是px
thread-1558-1-1.html: 两个kuing
李斌斌755 1# 2013-5-17 15:21
你信吗?
kuing 2# 2013-5-17 15:24
很容易办到,只要我用两个浏览器同时登录就行了……
isea 3# 2013-5-17 23:50
快把logo改大
kuing 4# 2013-5-18 00:32
好吧,让我想想换什么先……
thread-1559-1-1.html: 转一个人教保三角形函数
hongxian 1# 2013-5-17 16:01
对于函数 $f(x)$ 定义域内的任意三个不同的数 $a$,$b$,$c$,若这三个数能作为三角形的三边长,则 $f(a)$、$f(b)$、$f(c)$ 也能作为某三角形的三边长,则称这样的函数为三角形函数,以下所给函数中,属于三角形函数的有 1、$f(x)=\sqrt{x}$,$x>0$ 2、$f(x)=\sin x$,$0<x<\pi$    3、$f(x)=1/x$,$x>1$ 4、$f(x)=3^x$,$0<x<1$ 来自:http://bbs.pep.com.cn/forum.php?mod=viewthread&tid=2762403
Tesla35 2# 2013-5-17 16:56
齐建民老师写过几个博客文章: http://blog.sina.com.cn/s/blog_68ef13230101axb0.html http://blog.sina.com.cn/s/blog_68ef13230101axmt.html
kuing 3# 2013-5-17 17:20
1、$\sqrt a+\sqrt b>\sqrt{a+b}>\sqrt c$; 2、反例:$a=\pi/2$, $b=c=\pi-0.1$; 3、反例:$a=1.1$, $b=c=100$; 4、$3^a+3^b\geqslant 2\sqrt{3^{a+b}}>2\sqrt{3^c}$,因为 $c<1$,故 $\sqrt{3^c}<\sqrt3<2$,所以 $2\sqrt{3^c}>3^c$。 PS、相关链接 http://kkkkuingggg.5d6d.net/thread-1098-1-1.html http://bbs.pep.com.cn/thread-433893-1-1.html http://bbs.pep.com.cn/thread-523458-1-1.html ……等等
kuing 4# 2013-5-17 17:24
2# Tesla35 352说的那个结论怎么证?
hongxian 5# 2013-5-19 05:58
4# kuing 什么结论?居然没有看到, 倒是在第三个贴子中看到一个结论改一下写过来:“若$f(x)$为增函数,且$\dfrac{f(x)}{x}$为减函数,则$f(x)$为保三角形函数。”
kuing 6# 2013-5-19 14:50
5# hongxian 2楼第一个链接里面红色字那里……
thread-156-1-1.html: \everymath{\displaystyle}
kuing 1# 2011-10-26 15:33
我们都知道,LaTeX里行内公式里的分式、和式等等会缩小,这是LaTeX为了不影响行距而这样做的,所以其实我们在写文章时,应该尽量避免行内公式中使用大的公式,也不应该用\dfrac或者加个\displaystyle使之变大以致于撑大了该行的行距。在行内写分式时,将分式打成 $1/2$,$a/(b+c)$ 这样子就可以了。 当然,受国内主流排版软件“方正书版”的影响,国内并不太认同上述。所以,在应付“必须要大”的时候,如果很多行内公式都要加\displaystyle才能达到被要求的效果的话,那样是很麻烦的,这里提供一个简便方法: 在导言区加 \everymath{\displaystyle} 这样,行内的公式也默认是 \displaystyle 状态了。 另外,还有\everydisplay,还可以设置颜色。
thread-1560-1-1.html: 三角形轨迹
Gauss门徒 1# 2013-5-17 19:25
在坐标平面内固定一个三角形的顶点不动,另一个点的轨迹是直线,若三点构成的三角形总相似,那么第三个点的轨迹也是直线
kuing 2# 2013-5-17 20:25
位似变换……那不是显然的咩?
Gauss门徒 3# 2013-5-17 21:02
2# kuing 位似你妹妹,直线是任意给的
kuing 4# 2013-5-17 21:21
O,应该是位似旋转变换,$S(O,k,\theta)$……
Gauss门徒 5# 2013-5-18 04:24
4# kuing 你这不负责任的解答,作垂线勾股定理啊。。。
kuing 6# 2013-5-18 08:23
5# Gauss门徒 非常地负责,那的确是旋转位似。 PS、你明知故问又算不算负责哩?
thread-1561-1-1.html: [不等式] 转人教二元不等式
李斌斌755 1# 2013-5-18 00:05
本帖最后由 李斌斌755 于 2013-5-18 02:20 编辑 设$x>0,y>0$,且$(\sqrt{1+x^2}+x-1)(\sqrt{1+y^2}+y-1)\leqslant2$,则$(xy)_{\max}=$   http://bbs.pep.com.cn/forum.php? ... &extra=page%3D1
李斌斌755 2# 2013-5-18 00:20
好弱,该从不等式还是函数方向入手, 一点头绪都没有……
kuing 3# 2013-5-18 01:34
你抄错题了,原贴右边是2。 PS、跟《数学空间》总第 11 期 P16 例 3.2.3 类似,你看看哪个方法能延用。
李斌斌755 4# 2013-5-18 02:20
本帖最后由 李斌斌755 于 2013-5-18 02:46 编辑 3# kuing 赶快修改,去看看! 看了,函数思想、代数变形……看得我面红耳赤 一时还没消化,再想想……
kuing 5# 2013-5-18 10:55
4# 李斌斌755 转化一下呗… 只要证明 \[f(x)=\ln\frac{\sqrt{e^{2x}+1}+e^x-1}{\sqrt2}\] 是单增奇函数即可
李斌斌755 6# 2013-5-18 11:11
5# kuing 对我来说很难了,我连是用函数还是代数变形还没整明白,何况后面还连着两个高难度转换$e^x,\ln$
kuing 7# 2013-5-18 11:22
6# 李斌斌755 为了转化为3楼说的那个题的形式,对所求的那个式子取对数,然后换个元,就成那样子了。 爪机先不说太多了,晚点电脑上再打过程。
李斌斌755 8# 2013-5-18 14:19
本帖最后由 李斌斌755 于 2013-5-18 14:20 编辑 我试写写过程,当学习LaTeX \[(\sqrt{1+x^2}+x-1)(\sqrt{1+y^2}+y-1)\leqslant2\iff\\(\dfrac{\sqrt{1+x^2}+x-1)}{\sqrt2}\dfrac{(\sqrt{1+y^2}+y-1)}{\sqrt2}\leqslant1\\\ln\dfrac{\sqrt{1+x^2}+x-1}{\sqrt2}+\ln\dfrac{\sqrt{1+y^2}+y-1}{\sqrt2}\leqslant0\] 令$x=e^{t_1},y=e^{t_2}$,上式为\[\ln\dfrac{\sqrt{1+e^{2t_1}}+e^{t_1}-1}{\sqrt2}+\ln\dfrac{\sqrt{1+e^{2t_2}}+e^{t_2}-1}{\sqrt2}\leqslant0\] 设$f(t)=\ln\dfrac{\sqrt{1+e^{2t}}+e^t-1}{\sqrt2}$,有\[f(-t)=\ln\dfrac{\sqrt{1+\frac1{e^{2t}}}+\frac1{e^t}-1}{\sqrt2}=\cdots\] 不知路子对不对,到这走不动……唉怎么证明奇偶性、单调性全忘了
kuing 9# 2013-5-18 14:23
设$x>0,y>0$,且$(\sqrt{1+x^2}+x-1)(\sqrt{1+y^2}+y-1)\leqslant2$,则$(xy)_{\max}=$   http://bbs.pep.com.cn/forum.php? ... &extra=page%3D1 李斌斌755 发表于 2013-5-18 00:05 要求 $(xy)_{\max}$,只要求 $(\ln x+\ln y)_{\max}$,令 $\ln x=a$, $\ln y=b$,代入原等式即变为在条件 \[\bigl(\sqrt{1+e^{2a}}+e^a-1\bigr)\bigl(\sqrt{1+e^{2b}}+e^b-1\bigr)\leqslant2\] 下,求 $(a+b)_{\max}$。 令 \[f(x)=\ln\frac{\sqrt{e^{2x}+1}+e^x-1}{\sqrt2},\] 条件又可以变为 $f(a)+f(b)\leqslant0$,于是如果证得 $f(x)$ 为严格递增的奇函数,那么就可以得到 $a+b\leqslant0$,从而搞定。 事实上,$f(x)$ 严格递增是显然的,而 \begin{align*} f(x)+f(-x)&=\ln \frac{\bigl(\sqrt{e^{2x}+1}+e^x-1\bigr)\bigl(\sqrt{e^{-2x}+1}+e^{-x}-1\bigr)}2 \\ & =\ln \frac{\bigl(\sqrt{e^{2x}+1}+e^x-1\bigr)\bigl(\sqrt{1+e^{2x}}+1-e^x\bigr)}{2e^x} \\ & =\ln \frac{e^{2x}+1-(e^x-1)^2}{2e^x} \\ & =0. \end{align*}
kuing 10# 2013-5-18 14:32
上面是沿用了那里所说的辅助函数,而单纯用代数变形的话还不知怎么弄,然而代数变形+辅助函数倒是可以,还更简单些。 对 $\bigl(\sqrt{1+x^2}+x-1\bigr)\bigl(\sqrt{1+y^2}+y-1\bigr)\leqslant2$ 两边乘以 $\sqrt{1+x^2}-x+1$ 有理化后变为 \[x\bigl(\sqrt{1+y^2}+y-1\bigr)\leqslant\sqrt{1+x^2}-x+1,\] 或 \[\sqrt{1+y^2}+y-1\leqslant\sqrt{\frac1{x^2}+1}-1+\frac1x,\] 令 $g(x)=\sqrt{1+x^2}+x-1$,则上式又写成 \[g(y)\leqslant g\left(\frac1x\right),\] 显然 $g(x)$ 在 $(0,+\infty)$ 上递增,故 $y\leqslant1/x$,即 $xy\leqslant1$。
李斌斌755 11# 2013-5-18 14:36
本帖最后由 李斌斌755 于 2013-5-18 14:42 编辑 9# kuing 被我弄复杂了  ,基础不扎实,数学理解能力弱 好笨,要求$a=-b$,非要把$a$硬生生变出$b$,不会证明$a+b=0$
李斌斌755 12# 2013-5-19 00:42
有几何意义吗?
李斌斌755 13# 2013-5-19 00:58
10# kuing 高考题中见过这种应用,即由单调函数值域的大小判断自变量大小,不能活学活用啊
thread-1562-1-1.html: 汗啊
李斌斌755 1# 2013-5-18 02:59
在这混了500贴,代数(特别是不等式)还是一窍不通,当年学的全都还给老师了
李斌斌755 2# 2013-5-18 03:02
哭吧哭吧,晚了晚了,睡了睡了。
thread-1563-1-1.html: 一年一度678
李斌斌755 1# 2013-5-18 13:31
师生们又忙了,没人上论坛……没劲!
kuing 2# 2013-5-18 14:51
我还在想今年要不要还拿些高考题来玩玩……
李斌斌755 3# 2013-5-18 17:11
2# kuing 玩吧,这样也轻松一下。
yes94 4# 2013-5-23 20:02
3# 李斌斌755 李斌是老师吗?
李斌斌755 5# 2013-5-24 00:02
4# yes94 回答过,不是,是爱好者
thread-1564-1-1.html: 转一个人教群看到的题
hongxian 1# 2013-5-18 15:10
设$x=(x_1,x_2,\cdots,x_n)$是前$n$个正数1,2,$\cdots$,$n$的任意次序的排列($n\in N^*$),定义$f(x)$为$x$中每两个相邻元素的差的绝对值的最小值. (1)当$n=5$时,求$f(x)$的取值集合; (2)求$f(x)$的最大值;
kuing 2# 2013-5-18 16:11
第一问略; 第二问:以下设 $k\in\mbb N^+$。 当 $n=2k$ 时,考查数 $k$ 与其相邻的数 $x_i$ 之差的绝对值,显然只有当 $x_i=n$ 时 $\abs{k-x_i}$ 最大,为 $n-k=k$,所以 $f(\boldsymbol x)\leqslant k$,而排列 \[\boldsymbol x=(k, n, k-1, n-1, k-2, n-2, \ldots, 1, k+1)\] 满足 $f(\boldsymbol x)=k$,从而此时 $f(\boldsymbol x)$ 的最大值就是 $k$; 当 $n=2k-1$ 时,考查数 $k$ 与其相邻的数 $x_i$ 之差的绝对值,显然只有当 $x_i=n$ 或 $x_i=1$ 时 $\abs{k-x_i}$ 最大,为 $n-k=k-1$,所以 $f(\boldsymbol x)\leqslant k-1$,而排列 \[\boldsymbol x=(1, k, n, k-1, n-1, k-2, n-2, \ldots, 2, k+1)\] 满足 $f(\boldsymbol x)=k-1$,从而此时 $f(\boldsymbol x)$ 最大值就是 $k-1$。 综上所述,$f(\boldsymbol x)$ 的最大值为 $\lfloor n/2\rfloor$。
李斌斌755 3# 2013-5-18 17:26
本帖最后由 李斌斌755 于 2013-5-18 17:33 编辑 1# hongxian 最小值的最大值,好难懂
李斌斌755 4# 2013-5-18 20:39
本帖最后由 李斌斌755 于 2013-5-18 21:23 编辑 正好当练习 $\newcommand\asdf{|x_1-x_2|+|x_2-x_3|+\cdots+|x_{2k-1}-x_{2k}|}$ $\newcommand\asd{|x_1-x_2|+|x_3-x_3|+\cdots+|x_{2k-1}-x_{2k}|}$ $\newcommand\as{|x_2-x_3|+|x_3-x_3|+\cdots+|x_{2k-2}-x_{2k-1}|}$ 当$n=2k,k\in\mbb N^+$,设$M=\asdf$。从  http://kkkkuingggg.5d6d.net/thread-1379-1-5.html标答中得到启发 \[\begin{aligned}M&=(\overbrace{\asd}^{k~个})+(\overbrace{\as}^{k-1~个})\\&\leqslant[(k+1)+(k+2)+\cdots+2k]-(1+2+\cdots+k)\\&+[(k+1)+(k+2)+\cdots+(2k)]-[1+2+\cdots+(k-1)]\\&=2k^2-k\end{aligned}\] 到这不知道如何表达,总之$M$里面有$2k-1$个$f(x),[f(x)_{\min}]_{max}=\dfrac{2k^2-k}{2k-1}=k=\dfrac n2$(奇怪能正除) 电脑卡,晚些再打$n=2k-1,k\in\mbb N^+$情况 $\newcommand\asdf{|x_1-x_2|+|x_2-x_3|+\cdots+|x_{2k-2}-x_{2k-1}|}$ $\newcommand\asd{|x_1-x_2|+|x_3-x_3|+\cdots+|x_{2k-3}-x_{2k-2}|}$ $\newcommand\as{|x_2-x_3|+|x_3-x_3|+\cdots+|x_{2k-2}-x_{2k-1}|}$ 当$n=2k-1,k\in\mbb N^+$,设$M=\asdf$。 \[\begin{aligned}M&=(\overbrace{\asd}^{k-1~个})+(\overbrace{\as}^{k-1~个})\\&\leqslant[k+(k+1)+\cdots+(2k-2)]-[1+2+\cdots+(k-1)]\\&+[(k+1)+(k+2)+\cdots+(2k-1)]-[1+2+\cdots+(k-1)]\\&=2k^2-3k+1\end{aligned}\] $[f(x)_{\min}]_{max}=\dfrac{2k^2-3k+1}{2k-2}=\dfrac{2k-1}2=\dfrac n2$
thread-1565-1-1.html: 如果论坛卡,可以尝试进Archiver版里面看
kuing 1# 2013-5-18 16:42
所谓“Archiver版”即纯文字版,网址是:http://kkkkuingggg.5d6d.net/archiver/ 当然,Archiver版里不能发贴回贴,而且图片都不显示,还好,公式能照常显示。 所以如果当论坛卡的时候,又只是想看某个不太依赖图片的贴子时,可以尝试进入这里面看。
李斌斌755 2# 2013-5-18 17:19
本帖最后由 李斌斌755 于 2013-5-18 17:21 编辑 1# kuing 我的文字版里只看见文字,代码不显示……(上面已说)那根本看不懂。
kuing 3# 2013-5-18 17:45
2# 李斌斌755 公式是会显示的啊,而且很快
╰☆ヾo.海x 4# 2013-5-26 15:16
哇 这是为了福利我这种百度都永远打不开的鬼地方的朋友的吗?
kuing 5# 2013-5-26 15:20
4# ╰☆ヾo.海x 效果如何?
╰☆ヾo.海x 6# 2013-5-26 15:43
5# kuing 显示公式蛮快的。。
thread-1566-1-1.html: [不等式] 是真是假?
pengcheng1130 1# 2013-5-18 21:27
设$x\in(0,\dfrac{pi}{2}$,若果$x\sin^2x\geqslant1$,那么$xsinx\geqslant1$成立吗?
pengcheng1130 2# 2013-5-18 21:33
设$x\in(0,\dfrac{\pi}{2})$,如果$x\sin^2x<1$,则$x\sin x<1$成立吗?
李斌斌755 3# 2013-5-18 21:42
2# pengcheng1130 \[x\sin^2x<1\iff x\sin x<\dfrac1{\sin x}>1\]
kuing 4# 2013-5-19 01:21
设 $f(x)=x\sin^2x$, $g(x)=x\sin x$,易知它们都在 $(0,\pi/2)$ 上单增,且 $f(0)=g(0)=0$, $f(\pi/2)=g(\pi/2)=\pi/2>1$,从而它们在 $(0,\pi/2)$ 上分别存在唯一的 $a$ 和 $b$ 使得 $f(a)=g(b)=1$,由此可得 $g(b)=f(a)=a\sin^2a<a\sin a=g(a)$,从而 $b<a$。 由此,我们得到: 当 $x\in(0,b)$ 时,$f(x)<1$, $g(x)<1$; 当 $x=b$ 时,$f(x)<1$, $g(x)=1$; 当 $x\in(b,a)$ 时,$f(x)<1$, $g(x)>1$; 当 $x=a$ 时,$f(x)=1$, $g(x)>1$; 当 $x\in(a,\pi/2)$ 时,$f(x)>1$, $g(x)>1$。 这样,便可知 1# 正确,2# 错误。
thread-1567-1-1.html: [组合] 一道组合题
转化与化归 1# 2013-5-18 22:02
本帖最后由 转化与化归 于 2013-5-19 07:43 编辑 一道组合题
李斌斌755 2# 2013-5-18 22:12
本帖最后由 李斌斌755 于 2013-5-18 22:13 编辑 看不懂题 假如$p=1,q=2,r=3$,则集合$\{3+9+27|0\leqslant1<2<3,(1,2,3\in\mbb N^+)\}$是什么
转化与化归 3# 2013-5-18 22:26
2# 李斌斌755 39
李斌斌755 4# 2013-5-18 22:38
3# 转化与化归 不就一个数吗?
tan9p 5# 2013-5-18 23:08
3^4+3^5+3^7 是第 1+3+6+10+15+(10+5) 项 k = 40
零定义 6# 2013-5-18 23:31
木系50么...
李斌斌755 7# 2013-5-19 00:06
本帖最后由 李斌斌755 于 2013-5-19 00:18 编辑 令$b_n=3^{n-1}$,我们发现该数列前$n$项可以组成$C_n^3$项$a_i$的数列,其中$i=C_n^3,a_i=3^{i-2}+3^{i-1}+3^i$ \[\dfrac{2511}{13}\approx193.2<3^5\riff n=8\] $3^4+3^5+3^6=1053$是第$C_7^3$项,$2511-3^7=324=3^4+3^5$,有$C_6^2$ 故$k=C_7^3+C_6^2=50$
零定义 8# 2013-5-19 00:17
7# 李斌斌755 太深奥了,表示看木懂... 类似不需要那么高深的算法吧...
李斌斌755 9# 2013-5-19 00:25
8# 零定义 大师们深入浅出,化腐朽为神奇……吾等小虾米,使尽吃奶之力方想出这等笨方法
李斌斌755 10# 2013-5-19 00:31
1# 转化与化归 应该归入组合吧。
nash 11# 2013-5-19 00:42
10# 李斌斌755 话说这个不是三进制吗? 以前见过一个二进制的,一样一样滴。。。。。。
李斌斌755 12# 2013-5-19 01:01
本帖最后由 李斌斌755 于 2013-5-19 01:06 编辑 11# nash 用进制来解,给启蒙启蒙
零定义 13# 2013-5-19 01:10
本帖最后由 零定义 于 2013-5-19 01:11 编辑 木意思...
李斌斌755 14# 2013-5-19 01:19
本帖最后由 李斌斌755 于 2013-5-19 01:21 编辑 13# 零定义 6,7不能同时取,思路是一样的呀,不过你的解法更简洁 ,11#的3进制能说说嘛!
Tesla35 15# 2013-5-19 05:45
这是03年的高考题吧
转化与化归 16# 2013-5-19 07:28
本帖最后由 转化与化归 于 2013-5-19 07:43 编辑 14# 李斌斌755 写成3进制,若3^7的系数=1,C(6,2),若3^7的系数=0,C(7,3),所以50
李斌斌755 17# 2013-5-19 13:00
16# 转化与化归 谢谢,看不太明白
thread-1568-1-1.html: [数列] 2013上海浦东新区三模14题的问题
longma 1# 2013-5-19 12:26

realnumber 2# 2013-5-21 15:12
本帖最后由 realnumber 于 2013-5-21 15:30 编辑 第1问,不具有变换P,很好说明的. $\{a_n\}$具有P,$a_n=n^2-n$,$1^2,2^2,..n^2$是前n个最小的平方正整数 如此$\{a_n\}$中最大的一项$a_n$,$(n-1)^2< a_n+i\le n^2$,可见$a_n$只能放在最后位置,同理$a_{n-1}$只能放在剩下的最后位置,.... 所以,$\{a_n\}$不具有变换P性质. 第2问,感觉上也没有,怎么说明呢?假设有,数列$\{a_n\}$中至少2项具有这样性质 $a_i+i,a_i+j,i\ne j$,都为平方数,考虑中,....死机....
零定义 3# 2013-5-21 17:47
本帖最后由 零定义 于 2013-5-21 17:50 编辑 存在既是“P性质”,又具有“变换P性质”的数列. 例:${a_n}=n^2-n$,(n≠16);${a_n}$=48,(n=16) 调换第一项与第十六项,它依然是“P性质”数列
realnumber 4# 2013-5-21 18:19
3# 零定义 题目的意思猜是需要每依项都是平方
零定义 5# 2013-5-21 18:28
4# realnumber 不是每项加上它的下标都是平方么...
realnumber 6# 2013-5-22 12:29
3# 零定义 恩,看懂了,确实存在.
thread-157-1-1.html: 中文与公式等之间的“~”
kuing 1# 2011-10-26 15:42
在 LaTeX 中使用中文,我们通常要在中文与公式、数字、英文之间用 ~ 来隔开一个弹性距离(但公式与标点(包括中文标点和英文标点)之间不用加~),具体的标准在 cctspace.exe 里有确定的规则(ctex\ctex\cct\bin 里的 cctspace.exe)。 如果对此还不熟悉或比较混乱,可以先不打 ~ (或者如果已经有一些 ~ 但用得不知正不正确,可以先替换掉),然后再运行 Dos 命令 cctspace -t 原文件 替换后文件 这样 cctspace.exe 就能自动帮你在必要的地方加上 ~ 了。 例如 cctspace -t E:\1.tex E:\2.tex 即是将 E 盘的 1.tex 加上适当的 ~ 后生成 2.tex。 运行前要先定位到 cctspace.exe 文件夹内 比如在目前 kuing 的电脑中: “运行”——“cmd”——“cd C:\CTEX\CTeX\cct\bin”——“cctspace -t E:\1.tex E:\2.tex”
kuing 2# 2012-3-19 18:21
似乎不用定位,直接新建一个 bat 里面写 cctspace -t E:\1.tex E:\2.tex 就行了…………
thread-1570-1-1.html: 审核?
yayaweha 1# 2013-5-19 12:52
现在发帖怎么要审核?
kuing 2# 2013-5-19 14:23
唯一的原因大概只能是触动了 敏gan 词,我已经通了你那个贴子,但是怎么看也看不出到底哪个是 敏gan 词……
thread-1571-1-1.html: 碰撞
yayaweha 1# 2013-5-19 13:05
情景:在光滑的水平面上,质量为$m_1$的小球以速度$v_0$与质量为$m_2$的静止小球发生正碰(对心碰撞),碰后$m_1$的速度为$v_1$,$m_2$的速度为$v_2$。 若$v_1$与$v_2$同向,由CS$$(m_1 v_1^2+m_2 v_2^2)(m_1+m_2)\ge (m_1 v_1+m_2 v_2)^2=(m_1 v_0)^2$$ 知同向时$v_1=v_2$系统损失的能量最多,当$v_1$与$v_2$不同向,什么时候系统损失能量最多,当$m_1$与$m_2$关系为什么时,$v_1$与$v_2$同向,$v_1$与$v_2$不同向。
yayaweha 2# 2013-5-19 17:25
怎么没人
thread-1572-1-1.html: [函数] 一个函数老题目(3)
转化与化归 1# 2013-5-19 14:12
江苏省扬州中学2012—2013学年度第二学期高二期中考试第14题
李斌斌755 2# 2013-5-19 17:09
1# 转化与化归 见过一种方法,看成关于$a,b$的方程,不知行不行。
转化与化归 3# 2013-5-19 17:33
2# 李斌斌755 估计不行
goft 4# 2013-5-19 19:22
这个原题是2012浙江理科高考最后一题的第二小题
yayaweha 5# 2013-5-19 22:05
4# goft 2012课标卷也有这样的类型
kuing 6# 2013-5-19 22:30
由 $f(x)=4ax^3-2bx-a+b$ 容易计算出 $2f(0)+f(1)=a+b$。 由条件知 $-1\leqslant f(0)\leqslant1$, $-1\leqslant f(1)\leqslant1$,得到 $-3\leqslant a+b\leqslant3$。 而当 $a=1$, $b=2$ 时 $f(x)=4x^3-4x+1$ 满足条件;当 $a=-1$, $b=-2$ 时 $f(x)=-4x^3+4x-1$ 也满足条件,从而 $a+b$ 最大值为 $3$,最小值为 $-3$。 估计一般来说解到这里就被结束了,但其实至于中间的值是否完全能取到,还需要进一步论证……
kuing 7# 2013-5-19 22:40
糟糕!看漏了 $a>0$楼上最小值的情况错了……
kuing 8# 2013-5-19 23:05
还好修正也不是难事。 由 $f(x)=4ax^3-2bx-a+b$ 容易计算出 $2f(0)+f(1)=a+b$。 由条件知 $f(0)\leqslant1$, $f(1)\leqslant1$,得到 $a+b\leqslant3$。 而当 $a=1$, $b=2$ 时 $f(x)=4x^3-4x+1$ 满足条件,故 $a+b$ 的最大值为 $3$。 另一方面,容易计算出 $f(0)+f(1)=2a>0$,故 $a+b=2f(0)+f(1)>f(0)$,由条件知 $f(0)\geqslant-1$,故 $a+b>-1$。 而当 $a=\veps$, $b=-1+3\veps$ 并且 $0<\veps<1/3$ 时,$f(x)=4\veps x^3+(2-6\veps)x+2\veps-1$ 显然递增,故当 $x\in[0,1]$ 时 $-1+2\veps=f(0)\leqslant f(x)\leqslant f(1)=1$,即此时 $f(x)$ 满足条件,因此当 $\veps\to0^+$ 时就有 $a+b=-1+4\veps\to-1^+$,这样我们就得到 $a+b$ 的下确界是 $-1$。 中间的值是否能完全取到仍然需要论证,但是由后半部分的证明我们可以看出一个构造的思路,待续……
转化与化归 9# 2013-5-19 23:38
8# kuing 期待中!
kuing 10# 2013-5-20 00:06
原来直接就用回8楼那个构造就可以了,下面来续 8#: 当 $a=\veps$, $b=-1+3\veps$ 时,$f(x)=4\veps x^3+(2-6\veps)x+2\veps-1$,我们将证明此 $f(x)$ 对于所有 $\veps\in(0,1]$ 都是满足题意的。 (1)当 $0<\veps<1/3$ 时 8# 上已经证出是满足题意的; (2)当 $1/3\leqslant\veps\leqslant1$ 时,求导得 $f'(x)=12\veps x^2+2-6\veps$,由 $f'(0)=2-6\veps\leqslant0<f'(1)=2+6\veps$ 知存在 $x_0\in[0,1]$ 使得当 $x=x_0$ 时 $f(x)$ 在 $[0,1]$ 上取最小值,此时 $f'(x_0)=0$,即 $6\veps x_0^2+1-3\veps=0$,于是 \[f(x_0)+1=2x_0+2\veps(2x_0^3-3x_0+1)=2x_0+\frac2{3(1-2x_0^2)}(2x_0^3-3x_0+1) =\frac{2(1-4x_0^3)}{3(1-2x_0^2)},\] 又由 $6\veps x_0^2+1-3\veps=0$ 及 $\veps\leqslant 1$ 得到 \[x_0=\sqrt{\frac12-\frac1{6\veps}}\leqslant\sqrt{\frac12-\frac1{6}}=\frac1{\sqrt3} \riff x_0^3\leqslant \frac1{3\sqrt3}<\frac14,\] 这样我们就得到了 $f(x_0)>-1$,即此时 $f(x)$ 在 $[0,1]$ 上的最小值大于 $-1$。 而由导数的表达式知最大值必在端点处取得,由 $\veps\leqslant 1$ 同样有 $f(0)=-1+2\veps\leqslant 1$ 且 $f(1)=1$,所以此时的 $f(x)$ 都满足题意。 综上所述,当 $\veps$ 取遍 $(0,1]$ 时,$a+b=-1+4\veps$ 取遍 $(-1,3]$。 这样,综合 8# 以及本楼的所有内容,我们可以肯定地说:$a+b$ 的取值范围就是 $(-1,3]$。
李斌斌755 11# 2013-5-20 02:00
总感觉可以线性规划,就是……
thread-1573-1-1.html: [几何] 2013届通、泰、扬、连、淮五市高三三模
pengcheng1130 1# 2013-5-19 14:43
参考答案是硬生生的将四个点的坐标用求根公式给解出处理的。 此题是否有其几何背景,有更简单的解法吗?
kuing 2# 2013-5-19 14:46
斜率相反,四点共圆啊 证明见:http://kkkkuingggg.5d6d.net/thread-1115-1-1.html
李斌斌755 3# 2013-5-19 17:14
斜率的几何意义不敢想了,要人命……
isea 4# 2013-5-29 20:20
本帖最后由 isea 于 2013-5-29 20:22 编辑 江苏那边特别喜欢考这种题,算得人没算下去的欲望, 再如,南通市2013届高三第一次调研测试数学I:第19题,第三问 19. 已知左焦点为$F(-1,0)$的椭圆过点$E(1,\dfrac {2\sqrt3}{3})$.过点$P(1,1)$分别作斜率为$k_1,k_2$的椭圆的动弦$AB,CD$,设$M,N$分别为线段$AB,CD$的中点. (1)求椭圆的标准方程; (2)若$P$为线段$AB$的中点,求$k_1$; (3)若$k_1+k_2=1$,求证直线$MN$恒过定点,并求出定点坐标. 看到点评了: 近几年江苏高考解析几何大题的命题趋势:多考一点“算”,少考一点“想”.正好如某都相反…… 第三问,参考答案
thread-1574-1-1.html: [几何] 一道无图直角三角形中求$\tan B$
isea 1# 2013-5-19 22:59
本帖最后由 isea 于 2013-5-19 23:07 编辑 在$\triangle ABC$中,$\angle ACB=90^\circ,CD\perp AB$于点$D$,若$AB=4CD$。 求$\tan B$。 看到上面的题就想到这个平几法较难的 在$\triangle ABC$中,$D$在$AB$上,$\angle A=75^\circ,CD\perp AB$于点$D$,若$AB=2CD$。 求$\angle B$。
李斌斌755 2# 2013-5-19 23:34
$\triangle CFD$为等边三角形,$\angle ACE=\angle DCB=15\du,\angle B=75\du$
李斌斌755 3# 2013-5-20 00:03
$\triangle BEC$为等边三角形,$\angle ABC=30\du$
李斌斌755 4# 2013-5-20 00:24
3# 李斌斌755 汗,原来在人教初中就有。
isea 5# 2013-5-20 15:52
取E点就成了,F点多余的
isea 6# 2013-5-24 23:51
没人觉得2楼少个解?
李斌斌755 7# 2013-5-25 22:45
6# isea 公布答案。
isea 8# 2013-5-25 23:13
7# 李斌斌755 15度也行啊,没图
李斌斌755 9# 2013-5-25 23:44
8# isea 没反应过来
isea 10# 2013-5-27 23:35
这个,楼上李兄已经解答了,虽然不完整(因为不是正切值),这里略去具体解答了。 如果的确有不理解的,直接跟帖,然后再解答。
thread-1578-1-1.html: 原来也有发广告的
isea 1# 2013-5-20 11:12
无处不在啊
isea 2# 2013-5-20 11:15
本帖最后由 isea 于 2013-5-20 11:18 编辑 随便搜索了一下 数学 logo 还有这种图象 http://uuhy.com/html/14527.html
kuing 3# 2013-5-20 11:45
怪怪地…… 还是现在的好
isea 4# 2013-5-20 11:48
这个不介意,大的就行
kuing 5# 2013-5-20 11:50
thread-1579-1-1.html: [几何] 再转一个人教群内题
hongxian 1# 2013-5-20 15:52
已知:曲线$C:f(x)=x+\dfrac ax$,$(a>0)$与直线$l:y=x$,在曲线$C$上有一个动点$P$,过点$P$分别作直线$l$和$y$轴的垂线,垂足分别为$A$,$B$,再过$P$作曲线$C$的切线,分别与直线$l$和$y$轴交于点$M$,$N$,$O$是坐标原点,若$\triangle APB$的面积为$\dfrac 12$,则$\triangle OMN$的面积为
hongxian 2# 2013-5-20 15:55
1# hongxian 补个图
地狱的死灵 3# 2013-5-20 18:10
1# hongxian 注意到函数图像是以y轴和y=x为渐近线的双曲线, 由双曲线的性质, △APB与△OMN的面积都是定值(与P的选择无关), 于是取P为函数最值点$(\sqrt a ,2\sqrt a )$, 接下来很容易算了……
hongxian 4# 2013-5-20 20:29
3# 地狱的死灵 记起K有一个贴子,对勾函数实质为双曲线.
李斌斌755 5# 2013-5-20 20:37
4# hongxian 可以转换。
kuing 6# 2013-5-20 20:46
首先有一个引理: 双曲线的切线与两渐近线交点的中点为切点。 引理晚点再证,现在先直接拿来用。 如图,那条曲线是任意双曲线,那两条是直线为其渐近线,类似于原题那样作垂线、切线,取 $Q$ 为 $OM$ 的中点。 因为 $PA\perp OA$, $PB\perp OB$,故 $P$, $A$, $O$, $B$ 四点共圆,从而 $\angle ABP=\angle POQ$。 由引理知 $PQ$ 为 $\triangle OMN$ 的中位线,故 $\angle BAP=\angle BOP=\angle OPQ$。 这样就得到了 \[\triangle PAB \sim \triangle QPO,\] 且相似比为 \[\frac{AB}{PO}=\frac{AB}{2R}=\sin\angle AOB,\] 又由中位线易知 $S_{\triangle OMN}=4S_{\triangle OPQ}$,所以 \[\frac{S_{\triangle PAB}}{S_{\triangle OMN}}=\frac{S_{\triangle PAB}}{4S_{\triangle OPQ}}=\frac{\sin^2\angle AOB}4.\] 回到原题,易知 $y=a/x+x$ 为双曲线,$x=0$ 及 $y=x$ 为其两渐近线且夹角为 $45\du$,故 $\sin^2\angle AOB=1/2$,所以 $S_{\triangle PAB}/S_{\triangle OMN}=1/8$。
kuing 7# 2013-5-20 21:29
关于楼上那个引理,其实我也没想到纯几何证法,只想到了下面这样子搞。 先证明 $y=a/x$ 时成立(见http://bbs.pep.com.cn/thread-316595-1-1.html); 然后通过变换 $(x,y)\to(x,y+kx)$,变成一般的 $y=a/x+kx$ 的情形,就像下图那样子,仍然有相切和中点,故成立。 PS、由此还可以看出图中两种情况的那个面积是一样的,再结合前面链接中的内容,也就印证了3#的说法。
李斌斌755 8# 2013-5-20 22:02
链接一个资料 http://www.doc88.com/p-23371520738.html
kuing 9# 2013-5-20 23:22
8# 李斌斌755 太代数了,没心思看
isea 10# 2013-5-23 13:58
看来,在$y=\dfrac 1x$成立的好结论,好多在双曲线标准形式下亦是成立的了
thread-158-1-8.html: [不等式] 贴一段群聊记录
kuing 1# 2011-10-26 17:23
教师-一中(3175*****) 0:32:55 K前阵子耍几何不等式了吧    群管-kuing/hq<kuingggg@qq.com> 0:33:31 有耍过一下 教师-一中(3175*****) 0:36:15 觉得那个拉格朗日插值多项式不仅在解决代数问题,而且在几何不等式中也蛮有用,复数这工具也很强大    群管-kuing/hq<kuingggg@qq.com> 0:37:06 不太懂 教师-一中(3175*****) 0:38:29 比如有些涉及到三角形的不等式,就可以结合复数和那个拉格朗日    教师-一中(3175*****) 0:45:34 我举两个例子:设 $a,b,c$ 为 $\triangle ABC$ 的三边 $BC,CA,AB$ 的长,$P,Q$ 为 $\triangle ABC$ 平面上任意两点,求证: \[a\cdot PA\cdot QA+b\cdot PB\cdot QB+c\cdot PC\cdot QC \geqslant abc.\] 教师-一中(3175*****) 0:48:42 $P$ 为 $\triangle ABC$ 平面上任一点,求证: \[\frac{PA}{BC}+\frac{PB}{CA}+\frac{PC}{AB}\geqslant \sqrt3.\] 教师-一中(3175*****) 0:51:40 以上两题当然证法很多,但是用复数加拉格朗日,几乎就是三四步内解决 …… …… …… 嘿,昨晚由于时间关系没能继续聊下去,现有请一中老师继续聊聊具体咋玩,我是没看过“复数加拉格朗日”咋个玩法的,等待学习中……
kuing 2# 2011-10-26 19:03
看来还是不接受论坛聊,只能还是要我复制上来了 教师-一中(3175*****)  18:31:49 这个题目1复数加拉格朗日插值是这样的: 我们可以设 $P,A,B,C$ 在复平面所对应的复数分被为 $x,x_1,x_2,x_3$,再设多项式 $f(x)=1$,所以在 $x=x_1,x_2,x_3$ 处利用拉格朗日插值得到 \[\frac{(x-x_1)(x-x_2)}{(x_3-x_1)(x_3-x_2)}+\frac{(x-x_2)(x-x_3)}{(x_1-x_2)(x_1-x_3)}+\frac{(x-x_3)(x-x_1)}{(x_2-x_3)(x_2-x_1)}=1,\] 也就是 \[\frac{PA\cdot PB}{BC\cdot CA}+\frac{PB\cdot PC}{CA\cdot AB}+\frac{PC\cdot PA}{AB\cdot BC}\geqslant1,\] 接下去只需要再利用下 $(x+y+z)^2\geqslant3(xy+xz+yz)$ 即可证得。 教师-一中(3175*****)  18:34:42 其中 $\frac{PA\cdot PB}{BC\cdot CA}+\frac{PB\cdot PC}{CA\cdot AB}+\frac{PC\cdot PA}{AB\cdot BC}\geqslant1$ 是用到了复数模不等式。 ____________ 1这里指一楼的第二个题,原聊天记录中此处带了图片,这里就不贴上来了
kuing 3# 2011-10-26 19:09
go on 教师-一中(3175*****)  18:35:36 拉格朗日插值结合复数,或者切比雪夫多项式在处理一些不等式题目威力很大 教师-一中(3175*****)  18:36:57 几何不等式很多可以考虑用复数作为工具来耍,连接他们的桥梁就是复数那个模 然后还再给出了两个题 教师-一中(3175*****)  19:01:42 我以上举的只是想表达一个这样观点:在几何中哪怕是不等式中,尤其是几何不等式中,常常可以去考虑下复数作为工具,而且拉格朗日插值不仅仅是在求解代数题目才能够派上用场,即使是几何的题目也可以
kuing 4# 2011-10-26 22:34
其实当年我也耍过拉格朗日导出类似一些恒等式 http://bbs.pep.com.cn/thread-583379-1-1.html ,只是没往复数和几何不等式那边去想。 现在拿回来玩,就拿贴里的这个结论: \[\sum_{j=1}^n \left( \prod_{1\leqslant i\leqslant n; i\ne j}\frac{x_i}{x_j-x_i} \right)\equiv ( -1 )^{n+1},\] 马上就能得到楼上的题目3(单位圆上n个点那个)的证法了
pxchg1200 5# 2011-10-26 23:00
4# kuing 关于什么同余啊? 右边那个
kuing 6# 2011-10-26 23:01
5# pxchg1200 这里是指恒等
pxchg1200 7# 2011-10-26 23:04
6# kuing 好像很深奥的样子,表示对拉格朗日插值不太了解。。。
kuing 8# 2011-10-27 00:08
嗯,知道咋搞最开始的第一个题目了 设 $a,b,c$ 为 $\triangle ABC$ 的三边 $BC,CA,AB$ 的长,$P,Q$ 为 $\triangle ABC$ 平面上任意两点,求证: \[a\cdot PA\cdot QA+b\cdot PB\cdot QB+c\cdot PC\cdot QC \geqslant abc.\] 格式照抄,改点东西: 我们可以设 $P,Q,A,B,C$ 在复平面所对应的复数分别为 $p,q,x_1,x_2,x_3$,再设多项式 $f(x)=(x-p)(x-q)$,所以在 $x=x_1,x_2,x_3$ 处利用拉格朗日插值得到 \[\frac{(x-x_1)(x-x_2)f(x_3)}{(x_3-x_1)(x_3-x_2)}+\frac{(x-x_2)(x-x_3)f(x_1)}{(x_1-x_2)(x_1-x_3)}+\frac{(x-x_3)(x-x_1)f(x_2)}{(x_2-x_3)(x_2-x_1)}=(x-p)(x-q),\] 对比二次项系数,便有 \[\frac{(x_3-p)(x_3-q)}{(x_3-x_1)(x_3-x_2)}+\frac{(x_1-p)(x_1-q)}{(x_1-x_2)(x_1-x_3)}+\frac{(x_2-p)(x_2-q)}{(x_2-x_3)(x_2-x_1)}=1\] 于是 \begin{align*} \frac{PC\cdot QC}{ab}+\frac{PA\cdot QA}{bc}+\frac{PB\cdot QB}{ca} &=\left|\frac{(x_3-p)(x_3-q)}{(x_3-x_1)(x_3-x_2)}\right|+\left|\frac{(x_1-p)(x_1-q)}{(x_1-x_2)(x_1-x_3)}\right|+\left|\frac{(x_2-p)(x_2-q)}{(x_2-x_3)(x_2-x_1)}\right|\\ &\geqslant \left|\frac{(x_3-p)(x_3-q)}{(x_3-x_1)(x_3-x_2)}+\frac{(x_1-p)(x_1-q)}{(x_1-x_2)(x_1-x_3)}+\frac{(x_2-p)(x_2-q)}{(x_2-x_3)(x_2-x_1)}\right|\\ &=1, \end{align*} 即得 \[a\cdot PA\cdot QA+b\cdot PB\cdot QB+c\cdot PC\cdot QC \geqslant abc.\]
kuing 9# 2011-10-28 23:14
8# kuing 按照这种做法,貌似可以推广,$N$ 边形,$N-1$ 个动点。。。。
海盗船长 10# 2011-11-2 18:15
好方法
力工 11# 2011-11-8 13:27
[$3 + (\sum{\frac {b + c}{b - c}})^2\geq \sum{\frac {(b + c)^2}{(b - c)^2}}\geq3.$BQ
力工 12# 2011-11-8 13:28
11# 力工 多了几个字符。
thread-1580-1-1.html: [数列] 数列的题目
叶文明 1# 2013-5-20 16:08
本帖最后由 叶文明 于 2013-5-20 16:11 编辑 判断下列数列{a_{n}的单调性 $$a_{n+1}=a_{n}+\frac{(2-a_{n}^{2})}{2a_{n}+1}$$
realnumber 2# 2013-5-21 13:00
特殊到一般,计算$a_1,a_2,a_3,$也许你就能找到一般方法了 本题漏了条件$a_1$?
thread-1581-1-1.html: [组合] n个球r个盒
kuing 1# 2013-5-20 16:40
\begin{array}{|c|c|c|c|c|} \hline n~个球&r~个盒&是否允许有空盒&方案数&说明\\ \hline 不同&不同&允许&r^n&乘法原理\\ \hline 不同&不同&不允许&r!S_2(n,r)&\\ \hline 不同&相同&允许&\sum_{i=1}^rS_2(n,i)&仅有空盒若干时转化为下一类故为求和\\ \hline 不同&相同&不允许&S_2(n,r)&第二类~\text{Stirling}~数\\ \hline 相同&不同&允许&\mathrm C_{n+r-1}^{r-1}&隔板法'\\ \hline 相同&不同&不允许&\mathrm C_{n-1}^{r-1}&隔板法\\ \hline 相同&相同&允许&\sum_{i=1}^rP(n,i)&仅有空盒若干时转化为下一类故为求和\\ \hline 相同&相同&不允许&P(n,r)&见~\text{http://zh.wikipedia.org/wiki/整数分拆}\\ \hline \end{array}
kuing 2# 2013-5-20 17:28
最近连续几次看到这类问题,故此刚才无聊就打了这个表。那个 S_2 和 P 我也没怎么研究过,大概在任何一本组合数学的书里面都能找到这些吧。
kuing 3# 2013-5-22 00:00
顺便把 Stirling 数的链接也给一下了 http://zh.wikipedia.org/wiki/Stirling%E6%95%B8 里面有计算公式神马的了。 PS、相对来说还是整数分拆比较难些
thread-1582-1-1.html: [不等式] 一个不等式 在哪见过 想不起来了
boysxh 1# 2013-5-20 21:40
已知$n\geqslant3$且$n\in$$N$ 求证: \[2<(\frac{n+1}{n})^n<3\] 能不能用数学归纳法证明?
kuing 2# 2013-5-20 21:43
参考任何一本高等数学书……讲重要极限那里
realnumber 3# 2013-5-21 12:58
取对数后用导数也可以证明,左边直接就Bonulli不等式 2楼指的是自然底数e
kuing 4# 2013-5-21 13:42
3# realnumber 不建议这样做,容易循环论证
realnumber 5# 2013-5-21 14:08
恩,也是
yayaweha 6# 2013-5-21 22:45
4# kuing 循环论证什么意思?
yayaweha 7# 2013-5-21 22:48
取个对数放缩一下,$$n ln(1+\frac{1}{n})<ln e =1<ln 3$$ 这个显然成立$$ln(1+\frac{1}{n})<\frac{1}{n}$$
kuing 8# 2013-5-22 02:13
循环论证什么意思? yayaweha 发表于 2013-5-21 22:45 百度一下吧。
thread-1583-1-1.html: milksea大牛的书终于要出来了
kuing 1# 2013-5-21 01:30
信息来自:http://bbs.ctex.org/forum.php?mod=viewthread&tid=75863 当当链接:http://product.dangdang.com/product.aspx?product_id=23252576 绝对值强力推荐ing,尽管我还没买到手。
isea 2# 2013-5-21 02:05
本帖最后由 isea 于 2013-5-21 11:30 编辑 $\LaTeX$科技排版指南,借的,粗翻了一下,当手册用,数学部分相对而言比较细,但中文部分的确过时。 $\LaTeX$完全学习手册,第二版,入手,也翻了一遍,与时俱进,大体与前一本相同,手册也,都是好书。 这个,完全不会再考虑入手了 == LeoLiu  milksea 原来是均是刘海洋啊 那真值得借鉴
isea 3# 2013-5-21 11:31
LeoLiu 在北大2013年讲座文档 http://bbs.pku.edu.cn/bbs/bbstco ... p;threadid=14370497
kuing 4# 2013-5-21 14:05
当当那链接里有前两章在线试读
thread-1584-1-1.html: [数论] 睡神的一个猜想
realnumber 1# 2013-5-21 12:14
睡神(20*****7) 给定正整数$p,q,1<p<q$,则不存在无数个正整数n,使得$p^n\mid{q^n-1}$.
kuing 2# 2013-5-21 12:32
"不存在无数个"这样的说法有点怪。 建议改成: 对于任意满足1<p<q的整数p, q,使p^n|(q^n-1)成立的正整数n的个数有限或不存在。
realnumber 3# 2013-5-21 12:42
本帖最后由 realnumber 于 2013-5-21 12:44 编辑 显然若$(p,q)\ne1$,则不存在正整数n,以下有$(p,q)=1$ 假设存在无数个正数n,对给定p,q,有$p^n\mid{q^n-1}$ 取n=m>n=k为其中2个符合$p^n\mid{q^n-1}$的解. 那么$p^k\mid{(q^m-1,q^k-1)}$,而$(q^k-1,q^m-1)=(q^k-1,q^m-q^k)=(q^k-1,q^{m-k}-1)=\cdots=1$,矛盾. 所以猜想成立.
李斌斌755 4# 2013-5-21 13:09
$p^n|(q^n-1)$是啥意思?
realnumber 5# 2013-5-21 13:10
就是整除的意思,比如$4\mid{16},4\nmid{15}$
李斌斌755 6# 2013-5-21 13:11
5# realnumber 谢谢
零定义 7# 2013-5-21 16:38
3# realnumber 我有个疑问,望朱老师解答解答 我们知道,当q为奇数时,(q^m-1,q^k-1)≠1,和你的推导有点矛盾哦~ 或者说,是我理解错了你的解答呢?
零定义 8# 2013-5-21 16:40
2# kuing k神的表达很好,努力向k神学习!!!
realnumber 9# 2013-5-21 16:45
7# 零定义 恩,确实,明天继续了,不知道能不能补好
realnumber 10# 2013-5-21 18:18
本帖最后由 realnumber 于 2013-5-21 20:51 编辑 显然若$(p,q)\ne1$,则不存在正整数n,以下有$(p,q)=1$ 假设存在无数个正数n,对给定p,q,有$p^n\mid{q^n-1}$ 取n=m>n=k为其中2个符合$p^n\mid{q^n-1}$的解. 那么$p^k\mid{(q^m-1,q^k-1)}$,而$(q^k-1,q^m-1)=(q^k-1,q^m-q^k)=(q^k-1,q^{m-k}-1)=\cdots=q-1$,k可以任意大,p-1为常数,矛盾. 所以猜想成立.
零定义 11# 2013-5-21 18:24
10# realnumber 还是有点疑问... 例:取p=2,q=3,m=4,k=2,感觉木有$(q^m-1,q^k-1)$=p-1=1吧?
零定义 12# 2013-5-21 19:26
本帖最后由 零定义 于 2013-5-21 19:38 编辑 10# realnumber 终于看到朱老师的证明问题出在何处了... 1、对于有限问题反证时,应该往上证,而不是向下证; 2、由假设本身就有$(q^k−1,q^m−1)=a*p^k$,毕竟不是证明不存在. 以上两点纯属个人见解,如有不是,望见谅!
realnumber 13# 2013-5-21 20:49
本帖最后由 realnumber 于 2013-5-21 20:51 编辑 $(q^k-1,q^m-1)=(q^k-1,q^m-q^k)=(q^k-1,q^{m-k}-1)=\cdots=q-1$,--这个不是假设,是用(a,b)=(a,b-a)得出来的. k可以任意大,p-1为常数,矛盾. 我还是觉得证明没问题 =1删去了,刚才输入有错误,是q-1,不是p-1---10楼已经修改了
零定义 14# 2013-5-21 21:07
本帖最后由 零定义 于 2013-5-29 19:07 编辑 这个猜想源于在某数学群看到两个数论题. 当时觉得一个北京的老师说话语气有点过,一时气愤,解出了那两个数论题. 下面把原题与个人解答也贴上来吧~ 对于第二个问题,原出题者看了解答后,觉得可以因式分解,太简单了,他便随意的改成了问题3. 各位弟兄,指点指点小弟吧~
零定义 15# 2013-5-21 21:13
13# realnumber 感觉还是有点问题吧? 例:$p=2,q=3,m=4,k=2$,则$(q^m-1,q^k-1)=8≠q-1$
零定义 16# 2013-5-22 22:48
本帖最后由 零定义 于 2013-5-22 22:51 编辑 加强:对于任意满足1<p<q且$p\nmid q-1$的整数p,q,使$p^n|(q^n-1)$成立的正整数n不存在;反之,则正整数n的个数有限. 不知道对与否?
realnumber 17# 2013-5-27 20:05
加强不对的,$4\nmid 15-1$,$4^2\mid 15^2-1$ 原题,不会,继续想
零定义 18# 2013-5-28 12:35
本帖最后由 零定义 于 2013-5-28 12:41 编辑 17# realnumber 原题是错的,例:$q=p^2$,此时有无限个解. 这个是群里改了的,我还是改回我原来的意思吧~ 对于任意满足$1<p<q$且$(p,q)=1$的整数$p,q$,使$p^n|(q^n-1)$成立的正整数$n$的个数有限或不存在.
realnumber 19# 2013-5-28 13:09
本帖最后由 realnumber 于 2013-5-28 13:17 编辑 18# 零定义 不需要条件(p,q)=1,$q=p^2$无限个解好象没那会事.
零定义 20# 2013-5-28 15:00
19# realnumber 噢…的确…自己开始犯傻了…
thread-1584-2-1.html:
realnumber 21# 2013-5-28 22:14
不是无限个,但可以这样做到,不少于给定的k个解, 任意给定p,取$q=p^k+1$,则n=1,2,3,...,k均符合$p^n\mid q^n-1$.
零定义 22# 2013-5-29 00:30
21# realnumber 咋那么少人玩数论的呢...
thread-1585-1-1.html: [几何] 三角形的作图问题
hejoseph 1# 2013-5-21 15:53
给定$\triangle ABC$和三定点$D$、$E$、$F$,求作三直线$PQ$、$QR$、$RP$,使点$P$在直线$BC$上,点$Q$在直线$CA$上,点$R$在直线$AB$上,点$D$在直线$QR$上,点$E$在直线$RP$上,点$F$在直线$PQ$上。
李斌斌755 2# 2013-5-22 03:46
曾见过这题,主要一个$\triangle PRQ$对应无数个$\triangle DEF$,有点蒙……
hejoseph 3# 2013-5-22 14:45
什么意思? 现在是点$D$、$E$、$F$已知求点$P$、$Q$、$R$,不是点$P$、$Q$、$R$已知求点$D$、$E$、$F$。
李斌斌755 4# 2013-5-22 15:40
3# hejoseph 我知道,我的意思是定的$A,B,C$,不同的$D,E,F$,也能做出相同的$P,Q,R$。
hejoseph 5# 2013-5-22 17:59
本帖最后由 hejoseph 于 2013-5-22 19:08 编辑 发个解法,尺规作图法理论上可行,不过我没找到比较好的办法。 在直线$AB$上任取不同于点$A$、$B$的四点$R_1$、$R_2$、$R_3$、$R_4$,作直线$DR_1$、$DR_2$、$DR_3$、$DR_4$分别交直线于点$Q_1$、$Q_2$、$Q_3$、$Q_4$,分别作直线$R_1E$与$Q_1F$、$R_2E$与$Q_2F$、$R_3E$与$Q_3F$、$R_4E$与$Q_4F$的交点$P_1$、$P_2$、$P_3$、$P_4$,过点$A$、$P_1$、$P_2$、$P_3$、$P_4$作圆锥曲线,则这条圆锥曲线与直线$BC$的交点就是所求的点$P$,从而就确定了点$Q$和$R$。
第一章 6# 2013-5-22 21:34
圆锥曲线?何种?是否跟这些点的具体位置有关?
李斌斌755 7# 2013-5-23 01:57
本帖最后由 李斌斌755 于 2013-5-23 01:59 编辑 如图,在$\triangle ABC$三边$AB,AC,BC$上分别各取两点$R,R_1,Q,Q_1,P,P_1$,分别连结$QP,QR,PR,Q_1P_1,Q_1R_1,P_1R_1$。$D,E,F$分别为$RQ$与$Q_1R_1$、$RP$与$R_1P_1$、$QP$与$Q_1P_1$交点。这样一看,反过来,对于该$D,E,F$点来说不就可以做出两组$P,Q,R$了
hejoseph 8# 2013-5-23 16:51
6# 第一章 什么类型的圆锥曲线没关系的,反正没有四点共线的五点就能确定唯一的圆锥曲线。
hejoseph 9# 2013-5-23 16:52
7# 李斌斌755 是啊
李斌斌755 10# 2013-5-24 00:09
9# hejoseph 这样再想想。
thread-1586-1-1.html: winedt8了
kuing 1# 2013-5-22 11:53
see http://bbs.ctex.org/forum.php?mod=viewthread&tid=75809 话说,汗,前些天换texlive时才换成winedt7,现在又说出winedt8了,又要重新配置一次我那堆自定义?
李斌斌755 2# 2013-5-22 15:43
会累死牛的
thread-1587-1-1.html: [不等式] 和为 1 找最大 $\sum (x+1)\sqrt{x}$
Gauss门徒 1# 2013-5-22 16:30
$x,y,z>0, x+y+z=1$ find max \[\sum (x+1)\sqrt{x}\]
kuing 2# 2013-5-22 17:39
PS、标题是我改嘀。 $x,y,z>0, x+y+z=1$ find max \[\sum (x+1)\sqrt{x}\] Gauss门徒 发表于 2013-5-22 16:30 由对称性不妨设 $x\leqslant y\leqslant z$,则 $z\geqslant 1/3$, $x+y\leqslant 2/3$。 记 $f(x)=(x+1)\sqrt{x}$,则 \begin{align*} \left(2f\left(\frac{x+y}2\right)\right)^2-\bigl(f(x)+f(y)\bigr)^2 &=\frac12\bigl(\sqrt x-\sqrt y\bigr)^2\bigl(2-(x+y)^2+2xy-2\sqrt{xy}(2+x+y)\bigr)\\ &\geqslant \frac12\bigl(\sqrt x-\sqrt y\bigr)^2\left(2-\frac49+2xy-2\sqrt{xy}\left(2+\frac23\right)\right)\\ &=\frac19\bigl(\sqrt x-\sqrt y\bigr)^2\bigl(1-3\sqrt{xy}\bigr)\bigl(7-3\sqrt{xy}\bigr), \end{align*} 显然 $\sqrt{xy}\leqslant 1/3$,故我们得到了 \[f(x)+f(y)\leqslant 2f\left(\frac{x+y}2\right)=2f\left(\frac{1-z}2\right),\] 所以只要求 \[g(z)=f(z)+2f\left(\frac{1-z}2\right)\] 在 $[1/3,1]$ 上的最大值。 待续……
零定义 3# 2013-5-22 18:32
一般来说,见到门徒的题就兜路走...
kuing 4# 2013-5-22 18:46
续 2# \[g(z)=(z+1)\sqrt z+\frac{(3-z)\sqrt{1-z}}{\sqrt2},\] 求导得 \[g'(z)=\frac{3z+1}{2\sqrt z}+\frac{3z-5}{2\sqrt2\sqrt{1-z}},\] 有理化右边,有 \[\left(\frac{3z+1}{2\sqrt z}+\frac{5-3z}{2\sqrt2\sqrt{1-z}}\right)g'(z) =\frac{(3z+1)^2}{4z}-\frac{(3z-5)^2}{8(1-z)}=\frac{(2-3z)(3z-1)^2}{8z(1-z)},\] 故由 $z\leqslant1$ 可知,当且仅当 $z=2/3$ 时 $g(z)$ 取最大值,为 \[g\left(\frac23\right)=\frac{17}{3\sqrt6},\] 即为所求。
thread-1588-1-1.html: [几何] 一道向量条件外接圆三角形面积(from 人教数学群)
kuing 1# 2013-5-23 00:06
PS、后面还补充到:$O$ 是 $\triangle ABC$ 外心。 大概应该是 \[2S=\sqrt{1-\biggl( \frac{1+\mu^2-\lambda^2}{2\mu} \biggr)^2}\biggl( 1+\frac{1+\lambda^2-\mu^2}{2\lambda} \biggr)+\sqrt{1-\biggl( \frac{1+\lambda^2-\mu^2}{2\lambda} \biggr)^2}\biggl( 1+\frac{1+\mu^2-\lambda^2}{2\mu} \biggr)\] 这个式子对原题有没有用哩?……
kuing 2# 2013-5-23 00:12
细节不说太多,直接上图解 \begin{align*} 2S&=\sin\alpha+\sin\beta+\sin(\alpha+\beta) \\ &=\sin\alpha(1+\cos\beta)+\sin\beta(1+\cos\alpha) \\ &=\sqrt{1-\cos^2\alpha}(1+\cos\beta)+\sqrt{1-\cos^2\beta}(1+\cos\alpha) \\ &=\sqrt{1-\biggl( \frac{1+\mu^2-\lambda^2}{2\mu} \biggr)^2}\biggl( 1+\frac{1+\lambda^2-\mu^2}{2\lambda} \biggr)+\sqrt{1-\biggl( \frac{1+\lambda^2-\mu^2}{2\lambda} \biggr)^2}\biggl( 1+\frac{1+\mu^2-\lambda^2}{2\mu} \biggr). \end{align*}
李斌斌755 3# 2013-5-23 00:14
一看答案那么复杂就胆怯
kuing 4# 2013-5-23 00:15
3# 李斌斌755 我只是无聊将表达式算出来而已,其实原题的答案不是那个,也大概不需要它。
kuing 5# 2013-5-23 01:11
其实只要用 $2S=\sin\alpha+\sin\beta+\sin(\alpha+\beta)$ 这条式子或者就行了,和差化积,有 \[2S=2\cos\frac{\alpha-\beta}2\sin\frac{\alpha+\beta}2+\sin(\alpha+\beta),\] 因为 \[\cos(\alpha+\beta)=\frac{1-\lambda^2-\mu^2}{2\lambda\mu}=\frac{1-(\lambda-\mu)^2}{2\lambda\mu}-1,\] 故由 $\lambda>1$, $\mu>1$ 且 $\abs{\lambda-\mu}<1$ 可以得到 $\alpha+\beta\in(120\du,180\du)$,即得 $\cos\bigl((\alpha-\beta)/2\bigr)\in(0,1]$,记 $t=(\alpha+\beta)/2\in(60\du,90\du)$,故 \[\sin2t<2S\leqslant2\sin t+\sin2t,\] ……
kuing 6# 2013-5-23 01:21
其实极端考虑也可以比较严格,当然由于时间关系我下面说得比较简略。 首先熟知定圆内接三角形当且仅当正三角形时面积最大,然而由条件易知三角形不可能为正三角形,故必有 $S<3\sqrt3/4$,但是我们让 $\lambda\to1^+$, $\mu\to1^+$ 时能让其无限接近正三角形,即 $S\to\bigl(3\sqrt3/4\bigr)^-$,于是得到 $S$ 的上确界为 $3\sqrt3/4$; 另一方面,我们让 $\lambda\to1^+$, $\mu\to2^-$,易知此时 $S\to0^+$,于是可以得到 $S$ 的下确界为 $0$。 而易见当 $\lambda$, $\mu$ 连续变化时 $S$ 也连续变化,所以 $S$ 的取值范围就是 $\bigl(0,3\sqrt3/4\bigr)$。
yes94 7# 2013-5-23 19:51
6# kuing 照你那样做的话,野猪的题就没什么意思了,猜测是不是他在做某个题时的得到的副产品,然后自己编一个题,还说是选择题,但选项都不给出,条件还少了两个:半径为1,O为圆心,后来经网友质问再加的,说的颠三倒四的。还有呢此题是他在那个简单数列题受阻后抛出的此题,看看谁会这道题。 另外,用结论$S=\dfrac12(\sin2A+\sin2B+\sin2C)=2(\sin A\sin B\sin C)=\dfrac{pr}{2R^2}\leqslant \dfrac{\sqrt3}{4}$,其中p为半周长。根据$\lambda>1,\mu>1$,得$C<60^0$,故上式不可能取等号。 那就说明那个向量的条件用处不大,基本是忽悠人的,此题出的很不好。
nash 8# 2013-5-24 12:12
2# kuing 这个式子估计不是yezhu想要的结果
thread-1589-1-1.html: 数学模式的笑话
isea 1# 2013-5-24 00:10
听说了一個牛掰的公式 : n*AAA+m*ABC+DD就赢了, mn可以等于0, 让我一分钟就学会了麻将。 次公司聚会,酒过三旬,一妹纸溜到我哥们身边,说道:“xx你是不是喜欢我?” 大家都蒙了,这是要表白啊! 我那哥们刚要说话,那妹纸又说道:“你要吱声就是承认你喜欢我,要是不说话就是默认!” 众人纷纷为妹纸的智慧喝彩!那哥们以迅雷不及掩耳之势抱住旁边女同事亲了一口,场面一度失控啊!
isea 2# 2013-5-24 00:11
毕业5年后,我们成婚的一桌,未婚的一桌;   10年后,有孩子的一桌,还没孩子的一桌;   20年后,原配的一桌,二婚的一桌;   25年后,酒量好的一桌,差的一桌;   30年后,国内的一桌,国外的一桌;   35年后,荤的一桌,素的一桌;   40年后,退休的一桌,没退的一桌;   45年后,有牙的一桌,没牙的一桌;   50年后,自己来的一桌,扶着来的一桌;   55年后,说要来也来的一桌,说来却没能来的空一桌;   60年后,能来的一桌,不能来的照片一桌。人生真的很短暂!
kuing 3# 2013-5-24 00:19
1# isea 十三幺就没说……而且东南西北中发白也不能作为ABC……
李斌斌755 4# 2013-5-24 00:26
3# kuing 我没看懂,怪不得老输
thread-159-1-8.html: [不等式] $\sum2a^2/(c^2+3ac+2a^2)\ge1$
图图 1# 2011-10-27 18:24
本帖最后由 图图 于 2011-10-27 18:28 编辑 Let $a,b,c\in R^+$,prove that \[\sum\frac{2a^2}{c^2+3ac+2a^2}\ge1\]
pxchg1200 2# 2011-10-27 20:48
1# 图图 proof: \[ \frac{2a^{2}}{2a^{2}+3ac+c^{2}}=\frac{2}{2+3\frac{c}{a}+\frac{c^{2}}{a^{2}}} \] if we let $ u=\frac{c}{a},v=\frac{a}{b},w=\frac{b}{c} $,we get the result: $uvw=1$ and the original inequality become: \[ \sum{\frac{2}{2+3u+u^{2}}}\geq 1 \] Then,note that: $ u=\frac{yz}{x^{2}},v=\frac{xz}{y^{2}},w=\frac{xy}{z^{2}} $ Our inequality become: \[ \sum{\frac{2x^{4}}{2x^{4}+3x^{2}yz+y^{2}z^{2}}}\geq 1 \] Then,by Cauchy-Schwarz: \[ [ \sum{\frac{2x^{4}}{2x^{4}+3x^{2}yz+y^{2}z^{2}}}\geq \frac{2(x^{2}+y^{2}+z^{2})^{2}}{2\sum{x^{4}}+3\sum{x^{2}yz}+\sum{y^{2}z^{2}}} \] Thus.it suffice to prove that: \[  \frac{2(x^{2}+y^{2}+z^{2})^{2}}{2\sum{x^{4}}+3\sum{x^{2}yz}+\sum{y^{2}z^{2}}}\geq 1 \] Or \[ \sum{x^{2}y^{2}}\geq xyz(x+y+z) \] Which is obvious by AM-GM Done!
kuing 3# 2011-10-28 20:30
http://www.artofproblemsolving.c ... p?f=52&t=370050
thread-1590-1-1.html: [不等式] 转一道人教群线性规划
李斌斌755 1# 2013-5-24 11:33
主要想知道化成斜率方法。
kuing 2# 2013-5-24 11:39
群里不是都讲了吗?
李斌斌755 3# 2013-5-24 11:50
本帖最后由 李斌斌755 于 2013-5-24 12:17 编辑 2# kuing 是这个$u=\dfrac{\dfrac yx}{1+(\dfrac yx)^2}$吗?还有(何版的)过原点直线的斜率,不太明白 把$u$看成关于$\dfrac yx$的函数($\dfrac yx$为斜率)能理解,但把$\dfrac{xy}{x^2+y^2}$直接看成斜率还真不知道如何转换……
hejoseph 4# 2013-5-24 14:45
3# 李斌斌755 不是把$u$看成斜率,是把$\dfrac y x$看成过原点直线的斜率这个信息再化简
李斌斌755 5# 2013-5-24 15:59
4# hejoseph 哦,这样就明白了,谢谢!
hehehe 6# 2013-5-26 19:35
人教群群号是多少,我想加入,谢谢
kuing 7# 2013-5-26 19:38
6# hehehe 11071642
依然饭特稀 8# 2013-5-26 23:41
我出卷子考过,人家都是求交点代入···没人化成斜率搞
thread-1591-1-1.html: [数列] 2011江苏13改为求最大值(from 人教数学群)
kuing 1# 2013-5-24 15:07
2011江苏13是 解答见 http://bbs.pep.com.cn/forum.php? ... 068&pid=6477042 16# 而刚才在人教数学群看到如下的 教师-wwdwwd117(2365*****)  14:40:54 1=a1≤a2≤。。。≤a7,a1,a3,a5,a7成公比为q的等比数列, a2,a4,a6成公差为1的等差数列,则q的最大值? 群管-kuing  14:43:05 改了?原高考题是求最小值…… 教师-wwdwwd117(2365*****)  14:43:24 别人抄过来的题,也可能抄错 我估计不是抄错,而是就是想看看最大值能不能求,这显然可以,方法跟前面链接里的也是一样的,能预计到中间顶住的时候取得最大,于是有 群管-kuing  14:51:55 a3>=a2=a6-2>=a5-2 ==> q>=q^2-2 ==> q<=2,再举一实例 1,2,2,3,4,4,8 满足 q=2 教师-wwdwwd117(2365*****)  14:52:38 唯一的例子吧 群管-kuing  14:53:14 是的 跟原题求最小值时用的方法一样 教师-wwdwwd117(2365*****)  14:53:22 恩,我其实是没看见a1=1,想了一下午了 。。。。
thread-1592-1-1.html: 一道台高中题,多项式(from 人教数学群)
kuing 1# 2013-5-24 22:43
题目:求首项系数为 $2$,且满足 $4f(1)=3f(2)=2f(3)=f(4)$ 的三次多项式 $f(x)$。 以下是部分群聊记录 爱好者-何万程(1785***)  23:19:42 抽空写了下 f(x)=2x^3+bx^2+cx+d 4f(1)=8+4b+4c+4d 3f(2)=48+12b+6c+3d 2f(3)=108+18b+6c+2d f(4)=128+16b+4c+d 4f(1)-3f(2)=-40-8b-2c+d=0 3f(2)-2f(3)=-60-6b+d=0 2f(3)-f(4)=-20+2b+2c+d=0 -40-8b-2c+d-(-60-6b+d)-20+2b+2c+d=d=0 -60-6b=0=>b=-10 -20-20+2c=0=>c=20
kuing 2# 2013-5-24 22:47
我用拉格朗 ri cha 值公式是这样的: 设 $g(x)=f(x)(5-x)$,则由条件可设 $g(1)=g(2)=g(3)=g(4)=t$,又 $g(5)=0$,故由拉格朗 ri cha 值公式,$g(x)$ 可以写为 \begin{align*} g(x)={}&\frac{(x-2)(x-3)(x-4)(x-5)}{(1-2)(1-3)(1-4)(1-5)}t+\frac{(x-1)(x-3)(x-4)(x-5)}{(2-1)(2-3)(2-4)(2-5)}t \\ & +\frac{(x-1)(x-2)(x-4)(x-5)}{(3-1)(3-2)(3-4)(3-5)}t+\frac{(x-1)(x-2)(x-3)(x-5)}{(4-1)(4-2)(4-3)(4-5)}t \\ ={}&\frac{t(x-5)}{4!}[(x-2)(x-3)(x-4)-4(x-1)(x-3)(x-4) \\ & +6(x-1)(x-2)(x-4)-4(x-1)(x-2)(x-3)] \\ ={}&\frac{t(x-5)}{4!}(-x^3+5x^2-10x), \end{align*} 于是 $f(x)=2x^3-10x^2+20x$。
kuing 3# 2013-5-24 23:02
还是不知道用差分是什么意思……
hejoseph 4# 2013-5-25 09:49
本帖最后由 hejoseph 于 2013-5-25 09:53 编辑 若有数列$\{a_n\}$,则令$d_n=a_{n+1}-a_n$,则$\{d_n\}$称为$\{a_n\}$的一阶差分数列,简称差分数列,记为$\Delta\{a_n\}$。 数列$\{a_n\}$的$k$阶差分数列$\Delta^k\{a_n\}$定义为$\Delta\left\{\Delta^{k-1}\{a_n\}\right\}$,$\{a_n\}$的零阶差分数列定义为$\{a_n\}$。 $k$阶差分数列的通项公式为 \[ \sum_{i=0}^k(-1)^kC_k^ia_{n+k-i} \text{。} \] 若$a_n$是关于$n$的$m$次多项式,首项系数为$a$,则$\Delta^m\{a_n\}$是个常数数列$\{m!a\}$。
hejoseph 5# 2013-5-25 10:01
本帖最后由 hejoseph 于 2013-5-25 10:04 编辑 若$\{a_n\}$的各阶差分数列的首项按差分次数(零开始)由小到达排列成数列$\{d_{n+1}\}$,则 \begin{align*} a_n&=\sum_{i=0}^{n-1}C_{n-1}^id_i\text{,}\\ S_n&=\sum_{i=1}^nC_n^id_i\text{。} \end{align*}
零定义 6# 2013-5-28 18:35
本帖最后由 零定义 于 2013-5-28 18:46 编辑 不懂得如何差分运算,贴一个麻烦的差分... 设$f(x)=2x^3+bx^2+cx+d$,则$f^{(3)}(x)=12$ 令 $f(1)=3k,f(2)=4k,f(3)=6k,f(4)=12k$ 则 $\Delta f(1)=f(2)-f(1)=k,\Delta f(2)=f(3)-f(2)=2k,\Delta f(3)=f(4)-f(3)=6k$;      $\Delta^2f(1)=\Delta f(2)-\Delta f(1)=k,\Delta^2f(2)=\Delta f(3)-\Delta f(2)=4k$;      $\Delta^3f(1)=\Delta^2f(2)-\Delta^2f(1)=3k=f^{(3)}(x)=12$. 所以 $k=4$ 所以 $f(2)-f(1)=15+3b+c=k=4,f(3)-f(2)=38+5b+c=2k=8$ 解得 $b=-10,c=20$ 所以 $f(1)=2+b+c+d=3k=12$ 解得 $d=0$ 所以 $f(x)=2x^3-10x^2+20x$
kuing 7# 2013-5-28 18:54
6# 零定义 呃,这样看上去好像还不如直接解方程组……
零定义 8# 2013-5-28 19:02
7# kuing 应该是我不懂运算罢了...
hejoseph 9# 2013-5-28 21:57
本帖最后由 hejoseph 于 2013-5-28 22:10 编辑 令$f(x)=2x^3+bx^2+cx+d$,$4f(1)=3f(2)=2f(3)=f(4)=12t$,代入$f(4)-3f(3)+3f(2)-f(1)=3!\times 2=12$求得$t=4$,接着求得$f(1)=12$,$f(2)=16$,$f(3)=24$,$f(4)=48$,再由$f(3)-3f(2)+3f(1)-f(0)=12$得$f(0)=0$,即$d=0$,由$f(2)-3f(1)+3f(0)-f(-1)=12$得$f(-1)=-32$,结合$f(1)=12$即得$b+c=10$,$b-c=-30$,即$b=-10$,$c=20$,所以$f(x)=2x^2-10x^2+20x$。
isea 10# 2013-5-28 22:03
这题好玩,偶绝对直接硬算
hejoseph 11# 2013-5-28 23:00
差分数列一个重要应用就在于通项为变量的多项式的数列求和,例如$\{a_n\}=\{n^4\}$,$a_1=1$,一至四阶差分首项分别为15、50、60、24,所以 \[ S_n=C_n^1+15C_n^2+50C_n^3+60C_n^4+24C_n^5=\frac{1}{30}n(n+1)(2n+1)(3n^2+3n-1) \]
呆呆 12# 2013-5-30 09:29
我用拉格朗 ri cha 值公式是这样的: 设 $g(x)=f(x)(5-x)$,则由条件可设 $g(1)=g(2)=g(3)=g(4)=t$,又 $g(5)=0$,故由拉格朗 ri cha 值公式,$g(x)$ 可以写为 \begin{align*} g(x)={}&\frac{(x-2)(x-3)(x-4)(x ... kuing 发表于 2013-5-24 22:47 直接写 $(5-x)f(x)-t=-2(x-1)(x-2)(x-3)(x-4)$,令$x=5$
kuing 13# 2013-5-30 14:39
12# 呆呆 噢,这样直接求出 $t$,马上得到 f……nice! 这样简单多了!
hongxian 14# 2013-5-31 15:42
12# 呆呆 妙!
kuing 15# 2013-5-31 16:21
14# hongxian 是啊,的确很妙,我来到了边上都没发现这个玩法,受到了第一感觉拉格想到的朗 ri cha 插的影响了……
thread-1593-1-1.html: [几何] 用三角形所在边的 向量 表示外心到顶点 (方法2楼,过程9楼,推广12楼) 向量
isea 1# 2013-5-24 23:28
本帖最后由 isea 于 2013-5-27 23:30 编辑 也来个向量,相对而言,能秒且出结果,也需要好几秒吧,哈哈 数学通迅上看到的,不满意原作者的解法,故动手自己试了,发现此题有特点,发来,大家欣赏欣赏 题:如图,$\bigodot O$为$\triangle ABC$的外接圆,$AB=2,AC=1,\angle BAC=120^\circ$. 试用$\vv{AB},\vv {AC}$表示$\vv {AO}$. 如,附件(PDF中含作图源代码,供参考)。
nash 2# 2013-5-24 23:43
秒估计有点难度 不过也就是解个方程组的事 设AO=X*AB+Y*AC 然后两边同时点乘AB得出一方程 两边同时点乘AC得出一方程 联立方程组就能求出X,Y 注:上面皆为向量 还是不会打符号
isea 3# 2013-5-24 23:47
本帖最后由 isea 于 2013-5-24 23:54 编辑 2# nash 果然被秒了 这方法其实实用 \$\vv {AB}\$ 向量是 $\vv {AB}$,就这么简单
李斌斌755 4# 2013-5-25 23:45
3# isea 这个有简洁的几何法吗?
kuing 5# 2013-5-26 00:03
见过几次,有点FAQ……不过一时没找到链接
李斌斌755 6# 2013-5-26 00:06
5# kuing 想了好一阵,没想出来
isea 7# 2013-5-26 00:13
2楼是最简洁的解法了吧,加上向量点积几何意义,心算都能 偶用的复数加向量,大同小异
李斌斌755 8# 2013-5-26 00:30
7# isea 一碰向量就犯晕
isea 9# 2013-5-27 23:00
本帖最后由 isea 于 2013-5-28 00:34 编辑 推荐方法(解法1),即2楼具体过程,另外,加数量积几何意义可以避开复杂计算。 解析:记$O$在$AB$,$AC$上的射影分别为$O_1,O_2$,令$\vv {AO}=x \vv {AB}+y\vv {AC}$,于是 \begin{align*} &\left\{\begin{aligned} \vv{AO}\cdot \vv{AB}&=x \vv {AB}\cdot \vv{AB}+y \vv {AC}\cdot \vv{AB}\\ \vv{AO}\cdot \vv{AC}&=x  \vv {AB}\cdot \vv{AC}+y \vv {AC}\cdot \vv{AC} \end{aligned}\right.\\ &\left\{\begin{aligned} AB\cdot AO_1&=x AB^2+ y AC\cdot AB\cdot \cos120^\circ\\ AC\cdot AO_2&=x AB\cdot AC\cdot \cos120^\circ+y AC^2 \end{aligned}\right.\\ &\left\{\begin{aligned} 2&=4x-y\\ \frac 12 &=-x+y \end{aligned}\right.\\ &\left\{\begin{aligned} x&=\frac 56\\ y&=\frac 43 \end{aligned}\right.\\ \therefore \vv {AO}&=\frac 56 \vv {AB}+\frac 43 \vv {AC} \end{align*} 别解:记$\angle OAB=\alpha,\angle OAC=\beta,\alpha+\beta=120^\circ$,(最后代值时,需要先用正弦定理求出AO的长度) \begin{align} \frac {AO}{AB}\vv{AB}&=\vv {AO}\cdot (\cos \alpha + \mathrm{i}\sin \alpha) \label{eq001}\\ \frac {AO}{AC}\vv{AC}&=\vv {AO}\cdot (\cos (-\beta) + \mathrm{i}\sin (-\beta)) \notag\\ & =\vv {AO}\cdot (\cos \beta - \mathrm{i}\sin \beta)\label{eq002} \end{align} 在(\ref{eq001})式两边同乘$\sin \beta$ \begin{align} \frac {AO}{AB}\vv{AB}\sin \beta&=\vv {AO}\cdot (\cos \alpha \sin \beta + \mathrm{i}\sin \alpha \sin \beta)\label{eq003} \end{align} 类似的由(\ref{eq002})可得 \begin{align} \frac {AO}{AC}\vv{AC}\sin \alpha&=\vv {AO}\cdot (\sin \alpha \cos \beta - \mathrm{i}\sin \alpha \sin \beta)\label{eq004} \end{align} (\ref{eq003})+(\ref{eq004}) \begin{align} \frac {AO}{AC}\vv{AC}\sin \alpha+\frac {AO}{AB}\vv{AB}\sin \beta&=\vv {AO}\cdot (\sin \alpha \cos \beta+\cos \alpha \sin \beta)\notag\\ &=\vv {AO} \cdot \sin (\alpha+\beta)\label{eq005} \end{align} 将相应的数据代入(\ref{eq005})并整理就得到此题的结果:$\vv {AO}=\dfrac 56 \vv {AB}+\dfrac 43 \vv {AC}$. 若将(\ref{eq005})式变形为 \begin{align*} \dfrac {\vv {AC}}{AC} \sin \alpha+\dfrac {\vv {AB}} {AB} \sin \beta&=\dfrac{\vv {AO}} {AO} \cdot \sin (\alpha+\beta) \end{align*} 多么美好的式子!
isea 10# 2013-5-27 23:03
坐标法(解法2),如楼上的图,以A为原点,以AB(或者AC)为x轴建议直角坐标系, 待定系数:$\vv {AO}=x \vv {AB}+ y \vv {AC}$,下略
isea 11# 2013-5-27 23:07
本帖最后由 isea 于 2013-5-28 00:35 编辑 解法3,正余弦定理上台即可。 不打字了,自己看看百度知道 别解,向量+旋转(即复数几何意义),这个有点变态,属于个人偏好,不推荐。 合并到9楼了
isea 12# 2013-5-27 23:24
本帖最后由 isea 于 2013-5-27 23:34 编辑 而此题更一般的结论是什么呢? 这也跟以前的老题有关的,向量什么面积之类的。 http://bbs.pep.com.cn/forum.php? ... d=289872&page=1 http://bbs.pep.com.cn/forum.php?mod=viewthread&tid=2570800 http://bbs.pep.com.cn/forum.php? ... d=540072&page=1 http://bbs.pep.com.cn/forum.php? ... d=277670&page=1 (注意第43楼) http://bbs.pep.com.cn/forum.php? ... d=277949&page=1 http://bbs.pep.com.cn/forum.php? ... d=404407&page=1 如果你不愿意细看那些零散的东西,那其结论就是(来自 绕来绕去的向量法,75-76页) 点O是三角形内一点,则$S_{AOB} \vv{OC}+S_{BOC} \vv{OA}+S_{COA} \vv{OB}=\vv 0$。 (这公式其实不用记,具体到具体的题目,在具体的环境下,一般只需要求线段比值即可,如果利用一下向量的叉乘,又会简单许多。
kuing 13# 2013-5-27 23:49
12# isea 这个问题我叫它“超级FAQ”……
李斌斌755 14# 2013-5-28 00:31
我只想用初中知识解
isea 15# 2013-5-28 00:42
本帖最后由 isea 于 2013-5-28 00:47 编辑 14# 李斌斌755 涉及向量数量积,要用初中知识那就要改题,改为 题:如图,$\bigodot O$为$\triangle ABC$的外接圆,$AB=2,AC=1,\angle BAC=120^\circ$. 设$AO$与$BC$交于$D$,求 (1)$\dfrac {AD}{AO}$;(主楼的解法5,先得$\vv {AD}$) (2)$\dfrac {BD}{DC}$.(主要是这个)
李斌斌755 16# 2013-5-28 00:56
15# isea 感觉也不好作,似乎还得求半径。
isea 17# 2013-5-28 01:10
16# 李斌斌755 求肯定可以,不必未必不行,毕竟是向量题啊 我说9楼最后的形式美,主要是因为和 张角公式 特别的像
thread-1594-1-1.html: 抓狂
isea 1# 2013-5-24 23:48
显示数学代码太慢了,还好本地有WinEdt之流
kuing 2# 2013-5-25 00:01
是因为 5d6d 的问题,下面部分的东西老是卡住好一会出不来。 所以前两天我发了这个贴 http://kkkkuingggg.5d6d.net/thread-1565-1-1.html
李斌斌755 3# 2013-5-25 01:09
前两天我还以为是电脑原因。
thread-1595-1-1.html: Archiver 链接呢
isea 1# 2013-5-24 23:54
找不到,没摆出来?
kuing 2# 2013-5-25 00:04
等一下,下面就会有
李斌斌755 3# 2013-5-25 01:12
一直在右下角“论坛统计”右侧啊
isea 4# 2013-5-28 01:15
以前看到过,现在看不到了 也许是opera的问题 不管他
isea 5# 2013-5-28 01:18
本帖最后由 isea 于 2013-5-28 01:20 编辑 果然,换成IE就有了,应该js问题 opera 要好久才显示出来,比显示公式还慢些,或者基本同时显出来
isea 6# 2013-5-28 01:22
我喜欢这个帖数707
thread-1596-1-1.html: [组合] 2009年河南省预赛组合题
pengcheng1130 1# 2013-5-25 14:08
如何计算
kuing 2# 2013-5-25 14:10
见:http://kkkkuingggg.5d6d.net/thread-1293-1-1.html
pengcheng1130 3# 2013-5-25 22:18
参考答案给出的是如下组合数的计算表达式,是否解释一下这是从什么角度计算得啊!
kuing 4# 2013-5-26 01:17
3# pengcheng1130 没看出来。 参考答案只给了这样一条式子?
boysxh 5# 2013-5-26 13:50
参考答案给出的是如下组合数的计算表达式,是否解释一下这是从什么角度计算得啊! pengcheng1130 发表于 2013-5-25 22:18 选取n个元素 按序排列 用隔板法隔开即可
pengcheng1130 6# 2013-5-26 14:34
谢谢,知道了啊!
kuing 7# 2013-5-26 14:37
原来如此……
thread-1597-1-1.html: [转]小明是个善于观察的孩子
kuing 1# 2013-5-25 14:15
thread-1598-1-1.html: [转]什么是USB
kuing 1# 2013-5-25 14:36
PS、字有点小…… PS2、这是什么书?
零定义 2# 2013-5-28 02:43
1# kuing 呢本书叫“圈圈教你玩usb”...
kuing 3# 2013-5-28 02:52
2# 零定义 这你也知道。。。牛笔
kuing 4# 2013-5-29 21:39
群里又再发:
thread-1599-1-1.html: [函数] FAQ
yayaweha 1# 2013-5-25 15:58
本帖最后由 yayaweha 于 2013-5-26 10:43 编辑 若$0<a<b$ 证明:$$a\ln a+b\ln b-(a+b)\ln \frac{a+b}{2}<(b-a)\ln2$$
kuing 2# 2013-5-25 16:06
跟这个 http://bbs.pep.com.cn/thread-375445-1-1.html 很像,说不定是等价的……
yayaweha 3# 2013-5-25 16:11
好像不是等价的,人教那个等价于$$aln a+bln b>(a+b)ln \frac{a+b}{2}$$ 这个直接琴生
yayaweha 4# 2013-5-25 16:15
1# yayaweha 这个用主元思想,把a或b看成变量,另一个看成常数求导即可。 我觉得这个$(b-a)$像中值定理,应该能用积分证明。所以问下
kuing 5# 2013-5-25 16:23
那就是跟这个一样了吧 http://bbs.pep.com.cn/thread-877699-1-1.html
kuing 6# 2013-5-25 16:25
不过楼上贴子的解答还是前面那个,倒是这个贴 http://bbs.pep.com.cn/forum.php? ... 025&pid=3799311 的24#有这个的解答
yayaweha 7# 2013-5-25 16:26
和右边等价
yayaweha 8# 2013-5-25 16:27
我是想问下可不可以用积分证
kuing 9# 2013-5-25 23:24
积分的想不到,不在行…… 倒是想到了下面这个比较无聊的另证。 为方便处理,我们记 $c=(a+b)/2$,并定义 \[g(x)=\begin{cases} x\ln x,&x>0,\\ 0,&x=0, \end{cases}\] 利用极限易证 $g(x)$ 在 $[0,+\infty)$ 上连续。 我们将证明对任意 $0\leqslant a\leqslant b$,总有 \[g(a)+g(b)-2g(c)\leqslant (b-a)\ln2.\] 为此,我们先证明如下引理:若 $f(x)$ 在定义域 $D$ 内可导,$x_0\in D$,设 \[k(x)=\begin{cases} \frac{f(x)-f(x_0)}{x-x_0},&x\in D\setminus\{x_0\},\\ f'(x_0),&x=x_0, \end{cases}\] 若 $f'(x)$ 为增函数,则 $k(x)$ 也为增函数。 引理的证明:由导数定义知 $k(x)$ 连续,当 $x>x_0$ 时,求导并利用拉格朗日中值定理得 \begin{align*} k'(x)&=\frac{(x-x_0)f'(x)-\bigl(f(x)-f(x_0)\bigr)}{(x-x_0)^2} \\ & =\frac1{x-x_0}\left( f'(x)-\frac{f(x)-f(x_0)}{x-x_0} \right) \\ & =\frac1{x-x_0}\bigl( f'(x)-f'(\xi ) \bigr), \end{align*} 其中 $\xi \in (x_0,x)<x$,故由 $f'(x)$ 为增函数有 $f'(\xi )<f'(x)$,从而得到 $k'(x)>0$,同理可证当 $x<x_0$ 时也有 $k'(x)>0$,从而引理得证。 回到原题,当 $a=b$ 时原不等式显然成立,当 $0\leqslant a<b$ 时,原不等式可以整理为 \[\frac{g(b)-g(c)}{b-c}-\frac{g(a)-g(c)}{a-c}\leqslant 2\ln2,\] 现在,我们将 $c$ 固定,让 $b$ 增大,因为 $c=(a+b)/2$ 不变,故此时 $a$ 应减少。 因为 $g'(x)=1+\ln x$ 在 $(0,+\infty)$ 上为增函数,于是由引理可知,此时上式左边将增大。 于是,直到 $a$ 减少到 $0$ 时,上式左边取最大值,此时 $b=2c$,最大值为 \[\frac{g(2c)-g(c)}{2c-c}-\frac{g(0)-g(c)}{0-c},\] 化简后发现恒为 $2\ln2$,所以此时原不等式也成立。 综上所述,原不等式得证,取等条件为 $a=b$ 或 $a=0$。
kuing 10# 2013-5-26 01:23
其实由凸函数的性质也可以类似搞出来……时间关系明天再看看怎么写…… PS、1#   ln -> \ln  ,then $aln a$ -> $a\ln a$
Tesla35 11# 2013-5-26 12:31
这是04年高考全国卷的压轴题吧
yayaweha 12# 2013-5-28 12:40
本帖最后由 yayaweha 于 2013-5-28 12:41 编辑 再问一个FAQ 对任意的$x_1,x_2 \in R$有 $$|\sqrt{1+x_1^2}-\sqrt{1+x_2^2}| \le L|x_1-x_2|$$恒成立,求L得范围
kuing 13# 2013-5-28 13:22
12# yayaweha 这个左边分子有理化应该不难吧
yayaweha 14# 2013-5-28 13:35
有理化完了之后~~
kuing 15# 2013-5-28 13:45
这个左边分子有理化应该不难吧 kuing 发表于 2013-5-28 13:22 类似于《数学空间》总第 3 期 P5 中间那里
yayaweha 16# 2013-5-28 22:31
15# kuing $$\frac{|x_1+x_2|}{\sqrt {x_1^2+1}+\sqrt{x_2^2+1}}$$这个取得到1吗?
kuing 17# 2013-5-28 22:35
16# yayaweha 取不到
thread-16-1-8.html: [不等式] Vasc不等式的另一边,转自何版主
转贴专用 1# 2011-9-26 19:52
本帖最后由 转贴专用 于 2011-9-26 19:54 编辑 $x,y,z$ 为任意实数,有 \[-\frac{\sqrt7}8\left(x^2+y^2+z^2\right)^2\leqslant x^3y+y^3z+z^3x\leqslant\frac13\left(x^2+y^2+z^2\right)^2\] 未解决
pxchg1200 2# 2011-11-1 16:33
1# 转贴专用 那个另一边看见韩京俊的方法好像可以判定。。。。 准备研究下。
kuing 3# 2011-11-1 17:20
2# pxchg1200 表示我还没看那书。。。
pxchg1200 4# 2011-11-5 08:37
3# kuing 那个还真的有判别法的,我表示非常惊讶。Hjj的判别法可以解决三元三次,和三元四次对称和轮换多项式正定的问题。其中三元三次的结论和我的那个简直一模一样。。。。我的那个只是他的一部分推论而已。。。
pxchg1200 5# 2011-11-5 23:18
本帖最后由 pxchg1200 于 2011-11-5 23:22 编辑 1# 转贴专用 今天终于把这个破解了。呵呵 不过不是CS,而是配方。。 CS还在继续开发中。。。。
pxchg1200 6# 2011-11-5 23:42
建议kk先考虑下配方.再从配方去看CS。。
thread-160-1-1.html: 坐标变换
图图 1# 2011-10-27 18:42
将一空间直角坐标系绕方向(a,b,c)右旋$\theta$,原点不动,求坐标变换公式。
kuing 2# 2011-10-28 20:19
记两个链接先 http://zh.wikipedia.org/wiki/%E6%97%8B%E8%BD%AC%E7%9F%A9%E9%98%B5 http://wenku.baidu.com/view/58b1f64cf7ec4afe04a1df73.html
icesheep 3# 2011-11-23 00:24
只要算出基底变成什么,那整个线性变换就确定了。
kuing 4# 2011-11-28 16:22
http://hi.baidu.com/lsjsuper/blo ... 84ef08b051b9bd.html
icesheep 5# 2011-11-28 17:25
不明白这个有什么困难的,肯定是线性变换,而且肯定是正交阵。 只要算出 i,j,k 绕着 (a,b,1) 逆时针转 theta 变成什么,那么任何一个点 (x,y,z) 绕着 (a,b,1) 逆时针转 theta 变成什么也就确定了: \[\pi  \circ \left( {x{\mathbf{i}} + y{\mathbf{j}} + z{\mathbf{k}}} \right) = x\pi \left( {\mathbf{i}} \right) + y\pi \left( {\mathbf{j}} \right) + z\pi \left( {\mathbf{k}} \right)\]
力工 6# 2011-12-3 15:04
最简单的矩阵运算。
realnumber 7# 2011-12-15 08:21
(a,b,c)右旋θ,的意思是不是这样啊?右手捏个拳头,大拇指竖起来,就是向量(a,b,c)的方向,其余四指就是θ的方向,是这样吗?
叶文明 8# 2011-12-20 15:51
参看解析几何中的坐标变换,写出过渡矩阵即可。
thread-1600-1-1.html: [不等式] 一个条件不等式求最值
转化与化归 1# 2013-5-26 13:18
一个条件不等式求最值
kuing 2# 2013-5-26 14:32
见 http://bbs.pep.com.cn/forum.php?mod=viewthread&tid=1751315
转化与化归 3# 2013-5-26 15:08
2# kuing 好题!好解!
thread-1601-1-1.html: 这次终于是原创了(kuing初次尝试自制“QQ默认改版”表情)随时更新
kuing 1# 2013-5-26 14:40
刚才制作的,第一次整,不太容易……效果怎么样 ++  =
kuing 2# 2013-5-26 14:47
之前一直都是到处收集,这次尝试自己制作,大家给点意见或者 idea 啊 注: 制作工具:简单的用 GIF Movie Gear ,复杂的用 Photoshop ,或者一起用……
╰☆ヾo.海x 3# 2013-5-26 14:58
fantastic...wonderful!!! 酷儿is so "COOL"!!!
kuing 4# 2013-5-26 15:02
以后遇到老是发的人就可以这样他了
╰☆ヾo.海x 5# 2013-5-26 15:05
4# kuing 那是。。不过这也表示你自己不会再发这个表情咯?
kuing 6# 2013-5-26 15:28
5# ╰☆ヾo.海x 照发,来敲我吧
kuing 7# 2013-5-26 20:21
再来: 这个做出来的效果没有预想中的好,难度比1#大…… ++  =
Tesla35 8# 2013-5-26 21:26
7# kuing 卧   槽。。。。。
kuing 9# 2013-5-26 22:05
一静,一动(跳右眉) 第一/二帧+第一帧 =
kuing 10# 2013-5-27 16:30
各种 + =
kuing 11# 2013-5-28 00:01
+ =
isea 12# 2013-5-28 01:14
真无聊 楼主
kuing 13# 2013-5-28 01:43
+ = kuing 发表于 2013-5-28 00:01 来个连续变化的组合
李斌斌755 14# 2013-5-28 01:56
12# isea 你入群就知道你老了
kuing 15# 2013-5-29 00:19
* +
kuing 16# 2013-6-2 21:46
+
thread-1602-1-1.html: 省均分0.56的题
yayaweha 1# 2013-5-26 16:35
旧题一道
李斌斌755 2# 2013-5-27 10:26
这题怎么没人解
hongxian 3# 2013-5-27 15:08
1# yayaweha 做了一下第三问不知对不对, $\varphi_{min}=1$,$\varphi_{max}=\dfrac 5 4$
yayaweha 4# 2013-5-27 22:25
3# hongxian 对!可以把解法贴出来吗?
hongxian 5# 2013-5-27 23:04
4# yayaweha 主要是用前两问的结果来做的,可以分析出最值一定是在区域$D$的边界取得,再把边界方程代进去,用一下第二问的结果,就得到的是这个结果。 过程要写的话估计会比较长,特别是第二问。不知题目是来自哪里,省均分是怎么来的?我其实在等楼主贴答案!
yayaweha 6# 2013-5-27 23:44
5# hongxian 2011广东理数
thread-1603-1-1.html: 这个怎么过来的(不等式的秘密)均值证明 (解决)
isea 1# 2013-5-26 20:44
本帖最后由 isea 于 2013-5-26 23:13 编辑 才知道是国外的书,看着不习惯。 画框部分是怎么来的?自己尝试用归纳法证明,才发现,不等式难处理多了,完全卡住 === 去掉网上中文电子版,毕竟人家翻译还是很辛苦的, 而且开篇有说明,若有误,请参考原英文。 == 更新10楼哈工大出版社的这本书质量。
kuing 2# 2013-5-26 20:49
请看原版 这个网络上流传的这个翻译版排版差,也多错。
isea 3# 2013-5-26 21:22
本帖最后由 isea 于 2013-5-26 21:28 编辑 2# kuing 随手拿了这个哈工大的 中文版,纸质的,随便看看就有文字性的错误。(这个感觉给人真的很糟糕) 我发的和 2012年译版(纸质的),这一节是一样的。 刚查了英文版,这个地方看来原文的确是这样。 原作者写这书时,省略的地方很多,两三个小时,才看完12页,而且好多不太明白,的确高人啊
isea 4# 2013-5-26 21:43
4# kuing
kuing 5# 2013-5-26 21:46
现在明白了?
isea 6# 2013-5-26 21:59
5# kuing 还是不会,哈哈 == 我看这个倒是明白了,不等式归纳法还真难处理多了。 ===== 越南等(国外)处理归纳法都倒着来啊,先n后n-1
kuing 7# 2013-5-26 22:17
英文版无误,也看不懂?
isea 8# 2013-5-26 22:40
7# kuing TMD 哈工大出版社,少个s,无语! 大家千万不要入手哈工大出版社的这个。 纯是马扁钱
kuing 9# 2013-5-26 22:43
还有,指数呢,仔细再仔细
isea 10# 2013-5-26 22:45
本帖最后由 isea 于 2013-5-26 22:51 编辑 9# kuing 这个我疑问了很久,当证实少个s后,(我动手算过)就知道那个指数也错了 太气人了 === 看,原书,退了去,反正也看不懂,(哈,这个理由多好)
yayaweha 11# 2013-5-26 22:55
100个初等数学问题 中有一个是讲柯西证明AM-GM
kuing 12# 2013-5-26 22:56
难不成哈工大就是根据那个网络译版来打的?……OMG
kuing 13# 2013-5-26 23:00
其实不等式的文章很多时候并不需要看文字,只看公式,基本就能弄懂意思,真的遇到有部分需要的时候就翻译那部分就可以了。
isea 14# 2013-5-26 23:02
本帖最后由 isea 于 2013-5-26 23:06 编辑 12# kuing 我不知,我不猜 但我坚决退之,主要是我真的看不懂,太难了!这个错误只是导火线。 不过,佩服原作者文思!才子!正是一本好书。
isea 15# 2013-5-26 23:06
本帖最后由 isea 于 2013-5-26 23:11 编辑 哈大工让刘培杰数学工作室的 几何变换与几何证题第2版,就有比较多的文字怕错误, 不过,萧振纲写得真好,能理解,偶完全不介意 看来以后购书,不能只看出版社,还要看作者。 ====== 哦,最后说一下,主要是入手 近代欧氏几何学 单墫译的,凑满99元返20元,看来以后还真不能贪心
isea 16# 2013-5-28 01:12
退了,不过钱会打到amzon帐户了,不知大家有什么推荐的数学书,反正也是要拿书的
isea 17# 2013-5-28 02:35
有没推荐数学书的?
thread-1604-1-1.html: [函数] 一道高考三模三角函数题的困惑
题思思 1# 2013-5-27 14:41

hehehe 2# 2013-5-27 15:05
我觉得答案少考虑了一种情况,(4n-1)T/4=pai/2
kuing 3# 2013-5-27 15:31
“标答”错了,而学生的解法最后也还要注意 $\omega>0$ 这个条件……
kuing 4# 2013-5-27 15:41
2# 是对的,补上那种情况就可以了。 正确答案是 $\omega$ 能取遍正奇数,即 $\omega=2k-1$, $k\in\mbb N^+$ 事实上,满足条件的是如下函数 \[f(x)=A\sin\left((2k-1)\left(x-(-1)^k\frac\pi2\right)\right).\quad(A>0, k\in\mbb N^+)\] 不难验证它恒满足 $f(\pi/2)=0$, $f(\pi)=-1$。
题思思 5# 2013-5-27 15:53
看明白了,多谢各位大神!
isea 6# 2013-5-28 00:56
高三啊,那必须掌握的常规点
thread-1605-1-1.html: [数列] 另一道让我困惑很久的2012年上海高考理科压轴题
题思思 1# 2013-5-27 15:56

kuing 2# 2013-5-27 16:15
你可能对 $B$ 的理解有问题,估计 $B$ 中所用 $s$ 和 $t$ 让你有所误会,它们跟向量中两坐标的位置无关。 或者如果他将 $B$ 写成 $B=\left\{\dfrac ab\Bigm|a\in X,b\in X,\abs a>\abs b\right\}$ 可能你的理解就会不一样。
isea 3# 2013-5-28 00:57
这种数列压轴题,最后一问,只有学习的份了,偶。
kuing 4# 2013-5-28 00:59
这种数列压轴题,最后一问,只有学习的份了,偶。 isea 发表于 2013-5-28 00:57 其实也没什么数列知识点……所以让我发的话我不会将它归为[数列]类……
李斌斌755 5# 2013-5-28 01:04
3# isea 单数列还好,向量 ,最后三角数阵,听都没听过。
isea 6# 2013-5-28 01:16
5# 李斌斌755 所谓 创新题 今年模拟,特别是北京,超级“流行”
nash 7# 2013-5-28 01:39
很久以前做过 不知道是不是又问题
kuing 8# 2013-5-28 01:55
6# isea 这种题挺好,至少算是真正考能力,题海战术破不了。
题思思 9# 2013-5-28 13:19
大家看看我这个解法有问题吗?
thread-1606-1-1.html: 整数规划之钢材利用率最高
tan9p 1# 2013-5-27 22:35
一根2.5米的长条钢材,截0.6米和0.42米的两种规格,怎样截使得钢管利用率最高?
李斌斌755 2# 2013-5-28 00:21
贴人教群答案
isea 3# 2013-5-28 00:51
上面解法怎么就是线性规划,细看楼主的标题,原来就是
李斌斌755 4# 2013-5-28 01:11
3# isea 群里说是“运筹学的线性规划”,如果是350米的钢管呢?
李斌斌755 5# 2013-5-28 01:16
感觉上面的答案是枚举出来的,不能算出来吗?
nash 6# 2013-5-28 01:44
4# 李斌斌755 问,那么350米长的钢管是在哪家工厂生产的…
李斌斌755 7# 2013-5-28 01:45
令直线$0.42x+0.6y=2.5$,把直线向下平移经过的第一个网格点是$(3,2)$,其值为$2.46$
李斌斌755 8# 2013-5-28 01:47
6# nash 脱离现实。
thread-1607-1-1.html: [函数] 肯定会选C
isea 1# 2013-5-28 02:13
本帖最后由 isea 于 2013-5-28 02:23 编辑 一定的,你 题:如图,点A、B、C、D为圆O的四等分点,动点P从圆心O出发,沿O→C→D→O的路线做匀速运动.设运动时间为t(秒),∠APB的度数为y(度),则下列图象中,表示y与t之间函数关系最恰当的是(    ) 这题很经典的,最早看到是2008年江苏盐城中考数学,2009年的兰州中考数学卷等等广泛被引用。 前几天看到 点在半径圆弧上运动引起的角度变化--2012黑龙江绥化(彭翕成)这篇文章,才认真思考图象,严格讲,这是道错题。
kuing 2# 2013-5-28 02:21
除了圆周角相等那里恒定,其余不会是线性
isea 3# 2013-5-28 02:25
2# kuing 可用反正切表示吧,彭翕成弄得比较复杂。
李斌斌755 4# 2013-5-28 02:27
近两年高考题有类似的
isea 5# 2013-5-28 02:29
我更感兴趣的是,有没有一种速度让点在(第一段)线段上运动时,与时间是线性关系?
isea 6# 2013-5-28 02:31
近两年高考题有类似的 李斌斌755 发表于 2013-5-28 02:27 哦,帖个来看看,我见到的似乎不太一样
李斌斌755 7# 2013-5-28 02:34
5# isea 那你把它当问题来解呀?
kuing 8# 2013-5-28 02:37
这种题放在初中的确不会太容易被发现问题,因为初中较少接触复杂的东西,可以说基本上还是处于"线性头脑",所以发现不了很正常。 但是如果放到高中论坛估计就很容易被发现问题了,因为把具体函数关系求出来也没什么难度。 而没想到的是彭大居然这个也拿来说。。。
isea 9# 2013-5-28 02:40
8# kuing 显然他是显摆超级几何画板的 哈哈
isea 10# 2013-5-28 02:46
本帖最后由 isea 于 2013-5-28 22:05 编辑 7# 李斌斌755 脑袋里想了一下,线段下,应该没有,角度了,正切变了,不论速度如何,位置改变以了,so… ==== 有种速度叫变速,有种路程要积分 原观点错误,更新一下。 见第二页讨论。
李斌斌755 11# 2013-5-28 03:09
6# isea 一时找不到,改天找找。
wwdwwd2013 12# 2013-5-28 14:15
5# isea 与PB的平方成正比的速度,就能保证线性
kuing 13# 2013-5-28 20:00
其实还是不能说是错题,因为他并没说 C 是折线,他完全可以说其实两边不是直的,只不过看起来像而已
kuing 14# 2013-5-28 20:55
我更感兴趣的是,有没有一种速度让点在(第一段)线段上运动时,与时间是线性关系? isea 发表于 2013-5-28 02:29 不妨设圆半径为 $1$,点 $P$ 从 $O$ 到 $C$ 所用的时间为 $1$,此段过程中,记点 $P$ 的度速 $v$ 关于时间 $t$ 的函数为 $v=v(t)$($t\in[0,1]$),则有 \[ \cot\angle APB = OP = \int_0^t v(x)\rmd x, \] 若 $\angle APB$ 关于时间 $t$ 为线性,应有 \[\angle APB = \frac\pi2-\frac\pi4t,\] 即 \[ \int_0^t v(x)\rmd x=\cot\left(\frac\pi2-\frac\pi4t\right), \] 两边对 $t$ 求导就得到 \[v(t)=\frac\pi4\sec^2\frac{\pi t}4.\]
isea 15# 2013-5-28 21:57
14# kuing 存在这种速度,晕啊。 果然是直线,孤陋寡闻了 不过,那个 $\angle APB = \frac\pi2-\frac\pi4t$ 理解不了,
kuing 16# 2013-5-28 22:07
15# isea ??线性啊,$t=0$ 时 $\pi/2$,$t=1$ 时 $\pi/4$,于是就……
isea 17# 2013-5-28 22:33
16# kuing 哎呀~嘛呀~
isea 18# 2013-5-28 22:35
5# isea 与PB的平方成正比的速度,就能保证线性 wwdwwd2013 发表于 2013-5-28 14:15 能具体说说吗? 多谢。
wwdwwd2013 19# 2013-5-29 11:30
看图
kuing 20# 2013-5-29 13:45
19# wwdwwd2013 不错,这个结论好看。 另外,沿着这个结论做下去也可以求出 $V$ 的具体表达式,但是不必了,其实注意到了那个角速度恒定,也就可以这样做 \[\frac{OP}{OB}=\tan\angle PBO,\]即\[OP=OB\tan(\omega t),\] 两边对 $t$ 求导即得 \[V=\omega OB\sec^2(\omega t),\] 这样更简单。
thread-1607-2-1.html:
isea 21# 2013-5-29 14:01
19,20楼珠联璧合,这样看,怎么都像是物理题了
kuing 22# 2013-5-29 14:07
21# isea 的确可以考虑拿去当物理题至少简单点可以出成判断速度增加还是减少之类的小题就不错
李斌斌755 23# 2013-5-29 14:12
一些高考数学小题把它当物理题来解就简单多了。
isea 24# 2013-5-29 15:03
联想一道概率题: 在等腰直角三角形$ABC$中,过直角顶点$C$在$\angle ACB$ 内部任作一条射线$CM$,与线段$AB$交于点$M$. 求$AM<AC$的概率. 正好解释了$\dfrac {\sqrt2}2$这个结果不正确的原因。
kuing 25# 2013-5-29 15:06
24# isea 联想得不错……XXX奇论……
thread-1608-1-1.html: 快三点了
isea 1# 2013-5-28 02:34
哎呀,超级晚点了 睡觉睡觉 晚安晚安 拜拜拜拜
isea 2# 2013-5-28 02:38
为什么这帖的帖数显示第一页与第二页不同 真是奇怪啊,与当前数-1不同
kuing 3# 2013-5-28 02:54
明天用电脑再瞧瞧,现在爪机不方便。
李斌斌755 4# 2013-5-28 03:13
2# isea 前两天也发现。
kuing 5# 2013-5-28 13:19
没发现什么……
thread-1609-1-1.html: 还是空间向量
╰☆ヾo.海x 1# 2013-5-28 04:07
本帖最后由 ╰☆ヾo.海x 于 2013-5-28 04:37 编辑 求一平面的方程,which is 与一条直线l1相交,并与另一条直线l2平行。。两直线方程已知,平面方程怎么求啊。。 关键就是不知道平面法向量怎么求。我设法向量的话也有3个未知数,但是我只找到2个方程。。
╰☆ヾo.海x 2# 2013-5-28 04:12
本帖最后由 ╰☆ヾo.海x 于 2013-5-28 04:17 编辑 是不是那个法向量也只要随便令一个轴是1就可以了啊 然后2个方程就可以解出另外2个了。。对吗? 但是我刚试了下代了一个1进去解方程结果delta<0无解。。。
╰☆ヾo.海x 3# 2013-5-28 05:00
2# ╰☆ヾo.海x 才意识到这样的平面可以有无数个。。看来题目本身有问题了。。。
零定义 4# 2013-5-28 12:43
好高深呐...
╰☆ヾo.海x 5# 2013-5-28 20:05
4# 零定义 .......成吉思汗。。你怎么弄的滚动条。。。汗死了
零定义 6# 2013-5-28 23:56
5# ╰☆ヾo.海x I can fly higher!!!
kuing 7# 2013-5-29 00:09
6# 零定义 she is heather
零定义 8# 2013-5-29 00:25
7# kuing 表示神马是"heather"...
零定义 9# 2013-5-29 00:26
原来...表情是飞不起的...
kuing 10# 2013-5-29 00:28
我也不知道 [fly][/fly] 公式能飞 $\sqrt{\dfrac a{b+c}}$
零定义 11# 2013-5-29 00:32
10# kuing 公式也不会飞呐...
kuing 12# 2013-5-29 00:43
11# 零定义 公式要等一下才显示,最近都这样啊
零定义 13# 2013-5-29 01:15
12# kuing 公式显示了就不会飞了...只看到代码在飞...
kuing 14# 2013-5-29 01:20
13# 零定义 原来是浏览器问题,我刚才用IE的确跟你说的一样,但是用 chrome 的话公式也能飞
零定义 15# 2013-5-29 01:27
14# kuing 原来如此...
nash 16# 2013-5-29 03:04
搞的好NB的样子 不会飞的代码还没学的路过
yes94 17# 2013-5-29 21:43
让子弹飞!
╰☆ヾo.海x 18# 2013-5-30 00:52
楼上聊的好欢乐。。。。一群22222
keypress 19# 2013-6-2 21:29
本帖最后由 keypress 于 2013-6-2 21:31 编辑 把此贴移动到水区吧 PS:建议楼主改一下标题,题有误,还是有人会点进来的
thread-161-1-8.html: [不等式] 一个三元FAQ不等式的N元推广(估计早被推广过)
kuing 1# 2011-10-28 22:47
$a,b,c>0,abc=1$,有 \[\frac1{1+a+b}+\frac1{1+a+c}+\frac1{1+b+c}\leqslant1.\] 来自:http://bbs.pep.com.cn/thread-1936695-1-1.html 如标题所示,这题FAQ,easy且推广也一样easy,故以下估计也是早被玩腻了的事,所以请勿计较版权,这里扯这只是为个旺字。 已知 $x_i>0(i=1,2,\ldots,n),x_1x_2\ldots x_n=1$,则 \[\frac1{1+x_2+x_3+x_4+\cdots+x_n}+\frac1{1+x_1+x_3+x_4+\cdots+x_n}+\cdots+\frac1{1+x_1+x_2+x_3+\cdots+x_{n-1}}\leqslant1.\] proof: 令 $x_i=a_i^n,a_i>0,(i=1,2,\ldots,n)$,则 $a_1a_2\ldots a_n=1$,则由均值不等式,有 \begin{align*} &\frac1{1+x_2+x_3+x_4+\cdots+x_n}+\frac1{1+x_1+x_3+x_4+\cdots+x_n}+\cdots+\frac1{1+x_1+x_2+x_3+\cdots+x_{n-1}}\\ =&\frac1{1+a_2^n+a_3^n+a_4^n+\cdots+a_n^n}+\frac1{1+a_1^n+a_3^n+a_4^n+\cdots+a_n^n}+\cdots+\frac1{1+a_1^n+a_2^n+a_3^n+\cdots+a_{n-1}^n}\\ \leqslant&\frac1{1+a_2a_3a_4\ldots a_n(a_2+a_3+a_4+\cdots+a_n)}+\frac1{1+a_1a_3a_4\ldots a_n(a_1+a_3+a_4+\cdots+a_n)}\\ &+\cdots+\frac1{1+a_1a_2a_3\ldots a_{n-1}(a_1+a_2+a_3+\cdots+a_{n-1})}\\ =&\frac{a_1}{a_1+(a_2+a_3+a_4+\cdots+a_n)}+\frac{a_2}{a_2+(a_1+a_3+a_4+\cdots+a_n)}\\ &+\cdots+\frac{a_n}{a_n+(a_1+a_2+a_3+\cdots+a_{n-1})}\\ =&1. \end{align*}
nash 2# 2011-10-30 18:05
这个代换非常犀利
力工 3# 2011-10-30 18:38
请问题目出处,具体记不得了。
力工 4# 2011-10-30 19:11
3# 力工 加权推广:x>0,y>0,sigma(1+xa+yb)^-1<=3/(1+x+y),
thread-1610-1-1.html: 额 到现在也不知道怎么改头像。。
╰☆ヾo.海x 1# 2013-5-28 04:32
同志们,头像咋改啊。。
realnumber 2# 2013-5-28 08:09
右上角,的个人中心点进去就是了
╰☆ヾo.海x 3# 2013-5-28 08:57
2# realnumber 所噶 谢啦
kuing 4# 2013-5-28 13:18
这个也要问……
╰☆ヾo.海x 5# 2013-5-28 20:06
4# kuing 。。。我真没找到改的地方好不好。。
isea 6# 2013-5-28 22:39
http://kkkkuingggg.5d6d.net/memcp.php?action=profile&typeid=2
thread-1611-1-1.html: [数论] 不定方程
realnumber 1# 2013-5-29 13:29
河南张(28^^^^^96)  13:17:27
零定义 2# 2013-5-29 14:35
1# realnumber 这个easy啊~下面是我的愚见哈... 由对称性,不妨设$x≥y>0$ 设$(x,y)=d,x=ad,y=bd$,则$(a,b)=1,a^3+b^3=a^2b^2d$ 所以 $b^3=a^2(b^2d-a)$ 当$a=1$时,有$b^2(d-b)=1$. 此时,$b=1,d=2$,即$x=y=2$ 当$a>1$时,由$(a,b)=1$可知:$(a^2,b^3)=1$. 此时,无解.
yes94 3# 2013-5-29 21:35
2# 零定义 牛笔!
thread-1612-1-1.html: [函数] 恒成立
boysxh 1# 2013-5-29 14:27
对任意的$x\in[-1,1],\abs{4{x^3}+ax^2+bx+c}\leqslant1$恒成立,则$a+b+c=?$
kuing 2# 2013-5-29 15:04
对任意的$x\in[-1,1],\abs{4{x^3}+ax^2+bx+c}\leqslant1$恒成立,则$a+b+c=?$ boysxh 发表于 2013-5-29 14:27 设 $f(x)=4x^3+ax^2+bx+c$,依题意有 \begin{align*} 6 &\geqslant \abs{f(1)}+\abs{f(-1)}+2\left|f\left(\frac12\right)\right|+2\left|f\left(-\frac12\right)\right|\\ & \geqslant \left|f(1)-f(-1)-2f\left(\frac12\right)+2f\left(-\frac12\right)\right|\\ & =6, \end{align*} 故此上述所有用到的不等号应同时取等,即 \[\left\{\begin{aligned} &\abs{f(1)}=\abs{f(-1)}=\left|f\left(\frac12\right)\right|=\left|f\left(-\frac12\right)\right|=1,\\ &f(1)f(-1)<0, \\ &f(-1)f\left(\frac12\right)>0, \\ &f\left(\frac12\right)f\left(-\frac12\right)<0, \end{aligned}\right.\] 解得 $a=c=0$, $b=-3$。
第一章 3# 2013-5-29 15:38
这题也可算FAQ了,很多不等式的数都有类题,二次的 印象中都是用$|f(-1)|,|f(0)|,|f(1)|$,刚刚做了一下,没用$|f(-\frac{1}{2})|$和$|f(\frac{1}{2})|$还真弄不出来。
kuing 4# 2013-5-29 15:55
3# 第一章 二次的才是FAQ,三次玩得并不多,比二次复杂不少。 次数弄下去,其实又会变成切比雪夫多项式……
realnumber 5# 2013-5-29 16:36
想到可能要±0.5什么的,就是没敢代入,怕复杂,怕做无用工.
零定义 6# 2013-5-29 16:38
好高深呐... 填空题的话,还是习惯用特殊值玩玩...
boysxh 7# 2013-5-29 18:04
我也是没敢试0.5和-0.5 一直在1,-1,0里试
kuing 8# 2013-5-29 18:20
$\cos3x=\pm1$ 在 $[0,\pi]$ 上的解为 $k\pi/3$($k=0$, $1$, $2$, $3$),$\cos$ 它们的值就出来四个值 $-1$, $-1/2$, $1/2$, $1$,所以就试这些。
yes94 9# 2013-5-29 21:24
来晚了,这菜,这菜吃过的
thread-1613-1-1.html: 原来好多……
isea 1# 2013-5-29 20:29
初等数学区,解析几何题不少呢 好多都是热点
李斌斌755 2# 2013-5-29 20:46
原来主要是不等式,现在林子大了,什么鸟都有了
isea 3# 2013-5-29 20:50
2# 李斌斌755 不等式,真汉子
李斌斌755 4# 2013-5-29 20:52
3# isea 是不等式,真 啊
thread-1614-1-1.html: [组合] 一个整除题
转化与化归 1# 2013-5-30 10:40
一个整除题
realnumber 2# 2013-5-30 11:03
可以利用它来解,好象也看到过别用递推的解法,忘了.
realnumber 3# 2013-5-30 11:07
本帖最后由 realnumber 于 2013-5-30 11:08 编辑 50!含p=2的次数是$[\frac{50}{2}]+[\frac{50}{2^2}]+[\frac{50}{2^3}]+[\frac{50}{2^4}]+[\frac{50}{2^5}]=47$
转化与化归 4# 2013-5-30 11:30
3# realnumber 搞定!
Tesla35 5# 2013-5-30 12:33
数论导引。。
李斌斌755 6# 2013-5-30 13:45
5# Tesla35 也飞
dualliot 7# 2013-6-1 11:41
难道$C_{50}^1$也能被$2^{47}$整除?
dualliot 8# 2013-6-1 11:44
难道$C_{50}^1$也能被$2^{47}$整除? dualliot 发表于 2013-6-1 11:41 啊,没注意到:分母24!和26!也算出来. 掩面而逃!
thread-1615-1-1.html: 那个飞贴怎么玩啊!
李斌斌755 1# 2013-5-31 02:54
那个飞贴怎么玩啊!
isea 2# 2013-5-31 11:12
16# 李斌斌755 在flash图标与笑脸图标之间的那个A 或者直接打: [fly][\fly] 复制代码
kuing 3# 2013-5-31 11:58
是 [fly][/fly]
李斌斌755 4# 2013-5-31 14:04
本帖最后由 李斌斌755 于 2013-5-31 15:11 编辑 文字 还是不会
kuing 5# 2013-5-31 14:56
这是 discuz 代码(即论坛本身带有的代码),跟 latex、mathjax 无关。 [fly]文字[/fly] 复制代码
李斌斌755 6# 2013-5-31 15:12
本帖最后由 李斌斌755 于 2013-5-31 15:14 编辑 $a+b$ 谢谢kuing
hnsredfox_007 7# 2013-5-31 15:55
原来如此哦
thread-1616-1-1.html: [函数] 一道函数迭代题!
longma 1# 2013-6-1 16:32

longma 2# 2013-6-3 10:25
求指教!!!!!!!!!!!!
kuing 3# 2013-6-3 15:35
哎,这种题,想象倒是容易,写出来太麻烦,所以的确很适合作为填空题,如果是大题的话我宁愿跳过。
kuing 4# 2013-6-3 15:54
$f(x)$ 周期为 4,折线,波浪型,波高为 2。 $g_n(x)$ 只在 $[0,2]$ 里面折,也是波浪型,而且是 $2^{n-1}$ 个波,波高也为 2,很密。 于是想象一下就能想到,$g_n(f(x))$ 就是将密波往两边发射。 然后就跟直线 $x/2014$ 交一下,显然只在 $[0,2\times2014]$ 里面交,想象一下,在这里的拆线的每段都跟直线有一公共点,而 $[0,2\times2014]$ 里共有 $2014\times2^{n-1}$ 个波,每个波有两段,所以共有 $2014\times2^n$ 个公共点,即所求。 其实整个过程在脑子里想象是很快的,但是写上面这个描述用了我十几分钟,而且还不算是一个解题过程,so ...
realnumber 5# 2013-6-3 16:55
n=1,n=2特殊到一般尝试下,
nash 6# 2013-6-6 01:34
无图无真相 求图
thread-1617-1-1.html: [函数] 刚才的一段群聊记录(人教数学群)四次函数非负
kuing 1# 2013-6-2 02:12
________edit in $\LaTeX$________ \begin{align*} \frac{f(x)}{x^2}&=x^2-4x+3+m-\frac{12}x+\frac{12}{x^2} \\ & =x^2-4x+4+12\left( \frac14-\frac1x+\frac1{x^2} \right)+m-4 \\ & =(x-2)^2+12\left( \frac12-\frac1x \right)^2+m-4 \end{align*}
hejoseph 2# 2013-6-2 08:07
本帖最后由 hejoseph 于 2013-6-2 08:10 编辑 $p,q,r$是给定的实常数,$q\neq 0$,$f(x)=x^4+px^2+qx+r$,则 (1)$x$是方程$4t^3+2pt+q=0$中与$q$异号的一根时,$f(x)$在实域内取得最小值; (2)$x$是任意实数时,$f(x)\geqslant 0$恒成立的充要条件是不等式$t^3+(p^2-4r)t^2-2pq^2t+q^4\leqslant0$有正数解。
isea 3# 2013-6-2 19:41
厉害,又一般化了。 仅就主楼的题而,这是导数中常规题,首先参变量分享,有分母有个$x^2$,正好部分分式,搞定,正如主楼。
thread-1618-1-1.html: [转]聪明小孩与笨小孩
kuing 1# 2013-6-2 14:43
thread-1619-1-1.html: [转]piza
kuing 1# 2013-6-2 16:02
thread-162-1-2.html: 独乐乐不如众乐乐O(∩_∩)O~1216在1#有更新
戊概念·五 1# 2011-10-29 13:53
本帖最后由 戊概念·五 于 2012-8-13 23:56 编辑 ·开门帖 我们都是好孩子——一首让许多做了妈妈的歌手无法录制完成的歌:不是实力问题,不是技术问题,因为她们唱到中途就感动得不能自已,所以,最后由创作者自己演唱 快乐练习曲——高三时曾陪我走过一段苦闷时光的歌,再听,又是一番滋味..... 不必知道——当时这首歌刚出来时听着很洒脱的感觉,喜欢,但,最近一次偶然才得以再闻,多轻松的歌唱…… (已改为下载链接) 紫藤花——一直就很痴迷紫色,后来因为朋友,扩为蓝紫系,那种幽静的feeling 因为爱——孙悦最后的高音听着很舒服(发现董赫男的曲子也挺入耳的,嗯。这算是比较经典的一首吧~ ) 心墙——这个是女声版本。在回复的附件里有两版,分别是由JJ(21#)和郭静(22#)演绎的。俊杰那版是自弹吉他的伴奏,郭静那版是录音棚的成品。起初是宿舍里的小六要求我帮忙下载才接触到的,听着很有feeling,后来才知,这也是俊杰的作品~!!! O(∩_∩)O哈哈~两个版本都很有味道呢~ 红日——一首很经典的老歌! 等你爱我——貌似是《将爱》的主题歌,陈明的声线听着蛮好的“可能是我感觉出了错   或许是我要的太多   是否每个人都会像我?” 一直很安静——第一次听到这支歌是那年无意看《仙剑》,凄美的一段爱恋;后来再细听,是得知阿桑离开的时候,多了一层感伤的色调。我想说的是“给你的爱一直很干净”..... 兄弟——龙井的说唱,歌词超赞的(贴在最新回复里)~此生若能得此一帮兄弟,也算没有白走人间这一遭~! 一个像秋天一个像夏天——废话省去,听歌吧~友情,很踏实的的温度 相亲相爱——“天下相亲与相爱 动身千里外 心自成一脉  今夜万家灯火时 或许隔窗望 梦中佳境在” 好难得——我们都爱过某个诗人  可是诗人也是平凡的人  好难得 你遇见了对的人  难得你很认真不想太多去奋不顾身 好难得 你爱上了对的人  可惜Ta不是你的可能  好难得 我们遇见了对的人  难得好几辈子的好运才有机会完成  好难得 我们爱上了对的人  就算Ta不是到最后的人  也祝Ta更勇敢爱别人
kuing 2# 2011-10-30 16:46
乜叉意思?
戊概念·五 3# 2011-10-30 18:27
2# kuing “什么意思”???!!
戊概念·五 4# 2011-10-30 18:28
再: 你问哪里?题目还是歌曲?
kuing 5# 2011-10-30 20:33
4# 戊概念·五 整个贴完全都不懂
戊概念·五 6# 2011-11-1 15:11
5# kuing 好歌大家享
戊概念·五 7# 2011-11-8 11:04
·开门帖 我们都是好孩子——一首让许多做了妈妈的歌手无法录制完成的歌:不是实力问题,不是技术问题,因为她们唱到中途就感动得不能自已,所以,最后由创作者自己演唱 快乐练习曲——高三时曾陪我走过一段苦闷时 ... 戊概念·五 发表于 2011-10-29 13:53 第三首的链接可以直接点击收听,嗯~
戊概念·五 8# 2011-11-9 11:39
本帖最后由 戊概念·五 于 2011-11-9 11:44 编辑 这个只有音频 用搜搜没找到p3 格式的…… 再试一次~
戊概念·五 9# 2011-11-9 11:41
本帖最后由 戊概念·五 于 2011-11-9 11:45 编辑 白白了…… 我分,我分,我分开包~嘎……
kuing 10# 2011-11-9 11:45
http://kkkkuingggg.5d6d.com/thread-52-1-1.html
kuing 11# 2011-11-9 11:46
8# 戊概念·五 “宝贝的主题歌——F4国语版Can't help falling with you” 初中的时候听,当时跟风听F4,现在还有一张CD上有这首。
戊概念·五 12# 2011-11-9 11:47
10# kuing 天~按那个限制,分出几千个文件……
戊概念·五 13# 2011-11-9 11:48
8# 戊概念·五 “宝贝的主题歌——F4国语版Can't help falling with you” 初中的时候听,当时跟风听F4,现在还有一张CD上有这首。 kuing 发表于 2011-11-9 11:46 吔?你加塞了= =
kuing 14# 2011-11-9 11:48
12# 戊概念·五 难道上百M?
戊概念·五 15# 2011-11-9 11:49
14# kuing 呃……没那么大 可能.....是我分卷大小设置错了,看看去===========
kuing 16# 2011-11-9 11:51
15# 戊概念·五 500k一个分卷,一首歌按4~5M算,十个卷够了吧 but,给个链接不就行了么
戊概念·五 17# 2011-11-9 11:52
16# kuing 是,是我设错
戊概念·五 18# 2011-11-9 11:54
16# kuing 链啊,哦:http://video.soso.com/play/%E6%9 ... cid=v.music.vidplay土豆的
戊概念·五 19# 2011-11-9 13:26
15# 戊概念·五 500k一个分卷,一首歌按4~5M算,十个卷够了吧 but,给个链接不就行了么 kuing 发表于 2011-11-9 11:51 分了7个rar
戊概念·五 20# 2011-11-9 13:38
第三首的链接可以直接点击收听,嗯~ 戊概念·五 发表于 2011-11-8 11:04 出错了~
thread-162-2-2.html:
戊概念·五 21# 2011-11-10 12:12
这个是俊杰唱的^_^
戊概念·五 22# 2011-11-10 12:21
这个居然分出四个卷,好像是音质有区别的?立体声?
kuing 23# 2011-11-10 12:23
22# 戊概念·五 比特率差别---kbps
戊概念·五 24# 2011-11-11 13:22
23# kuing 哦,又学到了~!!!!! O(∩_∩)O谢谢哈~!
kuing 25# 2011-11-14 00:27
红日?你听得懂么? 而且你贴的版本。。。。。
戊概念·五 26# 2011-11-14 14:17
25# kuing 版本怎么了?
戊概念·五 27# 2011-11-20 21:08
这个居然分出四个卷,好像是音质有区别的?立体声? 戊概念·五 发表于 2011-11-10 12:21 这个版面……老大更改设置了吗?!
戊概念·五 28# 2011-11-24 22:11
从万民药房出来时,刚好开始放《一直很安静》,还真是巧,昨天才在这里更新了这首,呵呵~这也是缘分么?!“原来缘分是用来说明 你突然不爱我这件事情”…… Ps:“久病成医”,进药房问“奥氮平”,售货员就一束同情的眼光投过来,看来她知道这药是精神用药,呵呵~“精神病患”.....
戊概念·五 29# 2011-12-16 14:57
1# 戊概念·五 《兄弟》的歌词: 你从国外回来 咱就没怎么聊过 一直都在忙着 其实有好多话想说 还没来得及 转眼又要回去了 兄弟 你这一走又是几年 还真舍不得 都是大老爷们儿 说得有点女人了 又要离开了 你得惦记咱这哥几个 我好面儿 这些话太罗嗦 有点说不出口 所以把它写成一首祝福的歌—— 在那边注意身体 有事没事常联系 别老玩神秘 小心回来我跟你急 别老委屈自己 想吃什么吃点什么 要是回来再瘦了 哥几个踹死你 踏踏实实待着 收收你那臭脾气 那边不像咱这边 出事没人真帮你 要是烦了累了 给爷们儿拽一电话 这电话对你没有关机 没有不在服务区 兄弟 你的兄弟就在这里 不管什么时候回来 哥几个等你 兄弟 你的家就在这里 不管你人到了哪儿 哥几个挺你 兄弟 你的心就在这里 不管别人怎么说 哥几个懂你 兄弟 你得赶紧回到这里 不管变成什么样 哥几个陪着你 干杯 一杯接着再来一杯 不醉不归 今晚的任务 哥几个全醉 喝吧 没有人会在乎别的 吐完了再来 不会有人先睡 笑着 也许笑能止住眼泪 在咱们的字典里 哭 可能真的不会 侃吧 口无遮拦 不知疲惫 谁也管不着哥几个爱说谁就说谁 高了 开始在大街上抽风 要走的事全都忘得一干二净 唱着 那首歌名叫龙井 龙井就在这儿 它映在了夜空 走吧 哥几个并肩永远前行 谁也不会落下 这就是龙井 醉了 这回彻底的醉了 什么也不想 这是离别前的安静 兄弟 你的兄弟就在这里 不管什么时候回来 哥几个等你 兄弟 你的家就在这里 不管你人到了哪儿 哥几个挺你 兄弟 你的心就在这里 不管别人怎么说 哥几个懂你 兄弟 你得赶紧回到这里 不管变成什么样 哥几个陪着你 飞机起飞了 没有祝福 没有告别 就这么走了 也是你就这德行 给你准备的东西 你也不说拿走 是怕哥几个把你绑了不让你走是吧 在那边好好的 把自己照顾好了 缺点什么就和哥几个说 你别扛着 要是真想回来你就赶快麻溜回来 小子 这边还有一大堆事等着你弄呢 在这边的家人有事你就言语一声 别让你家老爷子觉得这边没亲人了 还有一大堆话没说你就回去了 啥时候回来给我一准日子 千万别忘了 别的什么话我也不再三八了 电话 给我打电话 咱得联系着 说着这些肉麻的话 直起鸡皮疙瘩 行了 兄弟 咱就这么着吧 兄弟 你的兄弟就在这里 不管什么时候回来 哥几个等你 兄弟 你的家就在这里 不管你人到了哪儿 哥几个挺你 兄弟 你的心就在这里 不管别人怎么说 哥几个懂你 兄弟 你得赶紧回到这里 不管变成什么样 哥几个陪着你
戊概念·五 30# 2011-12-16 14:58
29# 戊概念·五 才发现里面有别字,重发一次~
战巡 31# 2012-1-8 15:27
就听过红日,而且觉得非常捞,每次唱K有人点这首都是唱到一半就被人cut掉
戊概念·五 32# 2012-1-10 19:05
31# 战巡 又看到这句了…… Ps:巡版的UID还真够劲的——110啊
战巡 33# 2012-1-10 20:07
31# 战巡 又看到这句了…… Ps:巡版的UID还真够劲的——110啊 戊概念·五 发表于 2012-1-10 19:05 是你的两倍
戊概念·五 34# 2012-8-13 23:33
33# 战巡 4.。。。。。 2+2=2×2=22 2.。
叶剑飞Victor 35# 2012-8-17 01:47
8# 戊概念·五 自己用“格式工厂”转一下格式就行了。
戊概念·五 36# 2012-8-25 23:58
35# 叶剑飞Victor O(∩_∩)O谢谢哦~问题已经在之后解决了
thread-1620-1-1.html: MathJax 真心慢啊
keypress 1# 2013-6-2 20:34
mathjax依赖于javascript,关了javascript(支持)就没法显示了,而且javascript的通病就是慢,毕竟是客户端调用,啥时弄个服务端转换的就完美了。 老机器伤不起。
kuing 2# 2013-6-2 21:04
其实不是 MathJax 慢,现在之所有显示得慢,是 5d6d 卡出来的结果。 你可以试试进这个页面 http://kkkkuingggg.5d6d.net/archiver/ 来看贴子,也是用 mathjax ,如果在那里的公式显示很快,就足以说明并不是 mathjax 的问题。
keypress 3# 2013-6-2 21:07
本帖最后由 keypress 于 2013-6-2 21:11 编辑 就是,比较快的,看来5d6d慢, 另外,发现公式一多就慢的: http://kkkkuingggg.5d6d.net/archiver/tid-9.html 主要偶机器老了,而且感觉论坛不如AoPS快,有感而发
kuing 4# 2013-6-2 21:15
3# keypress 这个大贴慢点很正常啊,因为的确公式很多,但是对于一般的贴子来说,正常来说还是很快的,5d6d卡也是最近才开始的,以前一直都正常,可能要被 河 蟹 了,连总部都打不开(www.5d6d.net)。
kuing 5# 2013-6-3 00:23
哎,原先以为QQ收购了5d6d会变得更稳定,谁知…… 河 蟹 的威力实在是……
thread-1621-1-1.html: 【水】这个是不是可以说明亚太各国的数学爱好者的活跃度?
keypress 1# 2013-6-2 21:04
看图: 原来我好久前就注册过AoPS的,但是鸟语能看懂,写不成,所以就一直潜了。
thread-1622-1-1.html: [不等式] 求教各位大虾,这个不等式怎么证明
zyz 1# 2013-6-3 08:03

pxchg1200 2# 2013-6-3 12:34
1# zyz 目测应该是 \[ \sqrt{a}+\sqrt{b}+\sqrt{c}=3 \]
kuing 3# 2013-6-3 14:17
2# pxchg1200 是的,肯定是 =3 这题的原型是 a, b, c>0, a+b+c=3,证 $\dfrac1{a^2+b^2+2}+\dfrac1{b^2+c^2+2}+\dfrac1{c^2+a^2+2}\leqslant\frac34$(2009 伊/朗/国/家/队) 后来被改编成一楼的形式,好像是作为某年浙江的初赛题(改编使之变得更简单,直接柯西过去就行了,原题的话还要先变那个型)
zyz 4# 2013-6-3 14:54
不好意思,题目是打错了,是3!谢谢一楼二楼两位大侠!
thread-1623-1-1.html: [几何] 一道等边三角形问题,求几何解法
tan9p 1# 2013-6-3 10:21
正三角形,两个三等分点,求证CF垂直AF.
地狱的死灵 2# 2013-6-3 10:32
连DE,作AC边上的高BH,容易证明DE∥BH,即角DEC=90° 再由△ABD≌△BCE得角ADB=角BEC, 得到F,D,C,E四点共圆 所以角CFD=角DEC=90°
李斌斌755 3# 2013-6-3 13:57
贴个群里的
thread-1624-1-1.html: 中标了
李斌斌755 1# 2013-6-3 15:20
本帖最后由 李斌斌755 于 2013-6-3 15:25 编辑 我的QQ怎么了,首先是鼠标滚动变放缩,再就是回复输入时没反应(按W键则关闭)。
李斌斌755 2# 2013-6-3 15:22
只能看(还得拖着走),不能碰
李斌斌755 3# 2013-6-3 15:29
查看消息记录正常
kuing 4# 2013-6-3 15:36
关了再开
李斌斌755 5# 2013-6-3 19:33
4# kuing 不行,还是老模样。
李斌斌755 6# 2013-6-3 19:47
4# kuing 是恢复,谢谢。
isea 7# 2013-6-3 21:45
看看是按有键在按下状态没有弹起
李斌斌755 8# 2013-6-3 21:56
7# isea 还有这一说,呵呵。
kuing 9# 2013-6-3 22:37
7# isea 当时我也想说这一点,但是想想,如果有键没弹起的话,估计来这里发贴也成问题……
李斌斌755 10# 2013-6-4 14:01
真老了
thread-1625-1-1.html: [数列] 一个数列题
转化与化归 1# 2013-6-3 16:51
本帖最后由 转化与化归 于 2013-6-3 16:52 编辑 一个数列题,过程怎么弄得简短些。
零定义 2# 2013-6-3 19:13
本帖最后由 零定义 于 2013-6-3 21:29 编辑 1# 转化与化归 不知道酱紫弄行不行? $a_1\times(1+q+q^2)=2013=3\times11\times61$ 显然,$a_1\mid3\times11\times61,4\nmid a_1$ $q^2+q+1-\dfrac{2013}{a_1}=0$ $\Delta=1-4\times\left(1-\dfrac{2013}{a_1}\right)=3\times\left(\dfrac{4\times11\times61}{a_1}-1\right)$是一个完全平方数 所以  $\dfrac{4\times11\times67}{a_1}\equiv 1\pmod{3}$ 而  $4\equiv 1\pmod{3},11\equiv 2\pmod{3},61\equiv 1\pmod{3}$ 所以  $a_1=11$  或  $a_1=11\times61=671$ 当$a_1=11$时,$\Delta=3^2\times9^2$,此时 $q=13$,$a_2=11\times13=143$; 当$a_1=671$时,$\Delta=3^2$,此时 $q=1$,$a_2=671\times1=671$. 好吧~不懂咋弄格式,唯有采取空一行的笨方法了...
转化与化归 3# 2013-6-3 20:43
2# 零定义 公比q是有理数啊,公比q未必是整数啊!
零定义 4# 2013-6-3 21:31
3# 转化与化归 有问题么? $q$是有理数,不就是$q^2+q+1-\dfrac{2013}{a_1}=0$这个二次方程的$\Delta$是完全平方数么? 或者是我理解有问题?
转化与化归 5# 2013-6-3 22:08
4# 零定义 a1不一定能被2013整除,你解答的第二行有问题!
零定义 6# 2013-6-3 23:15
5# 转化与化归 噢噢~我知错了... 经程序运算,算得有三组解:(11,143,1859),(528,660,825),(671,671,671)... 至于如何补救,还有待思考...
hejoseph 7# 2013-6-4 09:33
这个之前我也想错了,可以这样做: 设公比为$q=\dfrac tu$,这里$t$、$u$是互素的正整数,不妨设$q\geqslant1$,则 \[ \frac{a_1}{u^2}(t^2+tu+u^2)=2013\text{,} \] 因为$\dfrac{a_1}{u^2}$必定为整数,所以$t^2+tu+u^2$必定是2013其中一个因数,$2013=3\times11\times61$。 若$t^2+tu+u^2=3$,则$(2t+u)^2+3u^2=12$,只能$t=u=1$,此时$a_1=a_2=a_3=671$; 若$t^2+tu+u^2=11$,则$(2t+u)^2+3u^2=44$,无整数解; 若$t^2+tu+u^2=61$,则$(2t+u)^2+3u^2=44$,只能$t=5$,$u=4$,此时$a_1=528$,$a_2=660$,$a_3=825$; 若$t^2+tu+u^2=3\times11$,则$(2t+u)^2+3u^2=132$,无整数解; 若$t^2+tu+u^2=3\times61$,则$(2t+u)^2+3u^2=732$,只能$t=13$,$u=1$,此时$a_1=11$,$a_2=143$,$a_3=1859$; 若$t^2+tu+u^2=11\times61$,则$(2t+u)^2+3u^2=2684$,无整数解; 若$t^2+tu+u^2=3\times11\times61$,则$(2t+u)^2+3u^2=8052$,无整数解。
isea 8# 2013-6-4 18:11
现在倒是显示公式不卡了,晕
yes94 9# 2013-6-6 00:00
公式太卡,搞个图片,齐老师的
kuing 10# 2013-6-6 00:08
9# yes94 这个图片也是何版主发的,齐是转发的,他可不会 latex
yes94 11# 2013-6-6 00:43
10# kuing 呵呵,原来他到处转摘解法哦!没注明出处(谁解的)
转化与化归 12# 2013-6-6 10:52
7# hejoseph 解得好!
thread-1626-1-1.html: [函数] 晚上人教数学群里的一道e^x*lnx不等式
kuing 1# 2013-6-4 00:53
学生—钻研者(5021*****)  22:52:09 求证:e^x*lnx+1/x>1/2(x>0) 求证: \[e^x \ln x + \frac1x > \frac12 \quad(\forall x>0)\] 还真不太易证…… 我的方法是去掉 e^x,后面比较麻烦,明天再写,你们先玩。
hejoseph 2# 2013-6-4 10:03
本帖最后由 hejoseph 于 2013-6-4 12:21 编辑 令 \[ f(x)=\ln x+e^{-x}\left(\frac 1x-\frac 12\right)\text{,} \] 则 \[ f'(x)=\frac{e^{-x}}{2x^2}\left(2xe^x+(x-1)^2-3\right)\text{,} \] 当$x\geqslant1$时$2xe^x+(x-1)^2-3$严格单调增, \[ f(1)=\frac 1e>0\text{,} \] 因此只需要考虑$0<x<1$的情况。以下设$0<x<1$,若$2xe^x+(x-1)^2-3=0$,则 \[ e^x=\frac{-x^2+2x+2}{2x}=1+\frac 1x-\frac x2\text{,} \] 方程左边严格单调增,右边严格减,所以这个方程只有唯一解$x_0$,当$0<x<x_0$时$f(x)$严格单调减,当$x>x_0$时$f(x)$严格单调增。当$x=1$时,方程左边等于$e$,右边等于$\dfrac{3}{2}$,当$x=\dfrac{2}{3}$时,方程左边等于$e^{2/3}$($<\dfrac{13}{6}$),右边等于$\dfrac{13}{6}$,所以$\dfrac{2}{3}<x_0<1$。此时最小值肯定在 \[ 2xe^x+(x-1)^2-3=0 \] 即 \[ e^x=\frac{-x^2+2x+2}{2x} \] 时取得,令 \[ g(x)=\ln x+\frac{2x}{-x^2+2x+2}\left(\frac 1x-\frac 12\right)\text{,} \] 则 \[ g'(x)=\frac{(x-2)(x^2(x-3)-2(x+1))}{x(x^2-2x-2)^2}>0\text{,} \] 即$g(x)$严格单调增,而 \[ g\left(\frac{2}{3}\right)=\frac{6}{13}-\ln\frac{3}{2}>0\text{,} \] 所以$f(x)$的最小值必定大于0,所证明成立。
AAAAA 3# 2013-6-4 10:42
2# hejoseph 倒数第三步貌似有点问题!
kuing 4# 2013-6-4 11:11
3# AAAAA 你意思是指分子分母反了?
hejoseph 5# 2013-6-4 11:43
之前的有错,已修改了
kuing 6# 2013-6-4 12:41
写一下我昨晚的证法。 由不努力不等式,当 $x\geqslant 1$ 时,$e^x=(1+e-1)^x\geqslant1+(e-1)x$ 且 $\ln x\geqslant 0$;当 $0<x<1$ 时,$e^x=(1+e-1)^x<1+(e-1)x$ 且 $\ln x<0$,故此对任意正数 $x$,都有 \[e^x\ln x\geqslant \bigl(1+(e-1)x\bigr)\ln x,\] 所以,欲证原不等式,只要证 \[\bigl(1+(e-1)x\bigr)\ln x+\frac1x>\frac12,\] 上式变形后等价于 \begin{equation}\label{20130603wanjcs} \ln x-\frac{x-2}{2x\bigl(1+(e-1)x\bigr)}>0. \end{equation} (1)当 $x\geqslant 2$ 时,要证式 \eqref{20130603wanjcs} 只要证 \[f(x)=\ln x-\frac{x-2}{2x}>0,\] 求导易得 $f'(x)=1/x-1/x^2>0$,故 $f(x)>f(2)=\ln2>0$,故式 \eqref{20130603wanjcs} 成立; (2)当 $0<x<2$ 时,要证式 \eqref{20130603wanjcs} 只要证 \[g(x)=\ln x-\frac{x-2}{2x(1+2x)}>0,\] 求导并因式分解可得 \[g'(x)=\frac{(4x+1)(x^2+x-1)}{x^2(1+2x)^2},\] 方程 $x^2+x-1=0$ 的正数解为黄金分割数 $x=\varphi=\bigl(\sqrt5-1\bigr)/2$(巧合),从而可知 $g(x)$ 在 $x=\varphi$ 取得最小值,代入化简有 \[g(\varphi)=\ln\varphi-\frac{\varphi-2}{2\varphi(1+2\varphi)} =\ln\varphi-\frac{\varphi-2}{2\varphi+4(1-\varphi)}=\ln\varphi+\frac12,\] 因此 \[g(\varphi)>0 \iff \frac12>\ln\frac1\varphi \iff e>\frac1{\varphi^2} \iff e>\frac{\sqrt5+3}2,\] 在数值上容易验证是成立的,从而式 \eqref{20130603wanjcs} 也成立。 综上所述,式 \eqref{20130603wanjcs} 成立,原不等式得证。
isea 7# 2013-6-4 18:10
看上去,都相当繁,这个应该是压轴题的最后一问,前面应该有铺垫; 其次要么就是改编的
thread-1627-1-1.html: 5d6d要关了,虽然这是意料之中
kuing 1# 2013-6-4 13:22
see: http://backup.5d6d.net/index.php
yes94 2# 2013-6-4 13:29
那我赶快发一帖,以作留恋!
huamahu 3# 2013-6-4 13:46
这么好的交流平台。。。。。。。。。。。。。。
李斌斌755 4# 2013-6-4 13:51
赶紧发帖纪念。
kuing 5# 2013-6-4 14:19
不是应该拍照留念么?
tan9p 6# 2013-6-4 14:46
数据库一定要保留哦,受这个论坛的启发,我觉得大伙可以合起来弄个题库。可惜没有现成的,好的架构。 (有相关贴的东西)
isea 7# 2013-6-4 18:08
本帖最后由 isea 于 2013-6-4 18:17 编辑 可惜了,普通能批量备份就好了 http://www.chinaz.com/news/2013/0604/304861.shtml
kuing 8# 2013-6-5 10:55
1# 的链接又进不了了,数据导出也成问题了。 现在唯一希望是这个通知是假的,是被黑的……
keypress 9# 2013-6-5 14:31
留念
hongxian 10# 2013-6-5 17:33
太惊讶了!这一天还是要来了!
thread-1628-1-1.html: 先拍照留念吧,我会尽量保存所有东西
kuing 1# 2013-6-4 15:21
要蛋定蛋定……
shidilin 2# 2013-6-4 15:38
留个爪
isea 3# 2013-6-4 18:08
看看还有没有免费的地儿,搬个家
李斌斌755 4# 2013-6-5 02:37
再拍几张
╰☆ヾo.海x 5# 2013-6-5 08:29
加油。。我们精神上支持你,行动上追随你!!!
keypress 6# 2013-6-5 14:37
做论坛主机放在国内是有点麻烦的
hongxian 7# 2013-6-5 17:36
一定保存一个带图的!
kuing 8# 2013-6-5 18:26
7# hongxian 有点困难
thread-1629-1-1.html: 到此一游!
南飞雁 1# 2013-6-4 15:41
2013.06.04到此一游。立此为证!
南飞雁 2# 2013-6-4 15:43
水贴1号!O(∩_∩)O哈哈~
thread-1630-1-1.html: [函数] 发几个经典的错题,权当纪念
第一章 1# 2013-6-4 19:59
听说5d6d就要关闭了。无比无语 发几个经典的错题,权当纪念 其实很久之前就想发这贴了,专门收集错题,不过最近孩子出生,比较忙,一拖再拖
第一章 2# 2013-6-4 19:59
第一题: 函数$f(x)=\ln x+ax$的图像存在与直线$2x-y=0$平行的切线,则实数$a$的取值范围是( ) $\color{red}{A.}a\leqslant2$    $\color{red}{B.}a<2$    $\color{red}{C.}a>2$    $\color{red}{D.}a>0$ 原题选$\color{red}{B}$. $\color{blue}{分析:}$由$f'(x)=\frac{1}{x}+a=2$,得$\frac{1}{x}=2-a>0$,故$a<2$. $\color{blue}{思考1:}$导数相等,切线可能与$2x-y=0$重合? $\color{blue}{分析:}$若$2x-y=0$恰为切线,那么切点为$(\frac{1}{2-a},\frac{2}{2-a})$,代入曲线方程,得$a=2-\frac{1}{e}$,故答案应为$a<2$且$a\ne2-\frac{1}{e}$. $\color{blue}{思考2:}$$a=2-\frac{1}{e}$时,会否有两条切线(或多条),一条与$2x-y=0$重合,另一条平行? $\color{blue}{分析:}$$f''(x)=-\frac{1}{x^2}<0$,即$f'(x)$递减,切点不同时,斜率也不同. 综合上述,正确答案为$a<2$且$a\ne2-\frac{1}{e}$.
第一章 3# 2013-6-4 20:00
与第一题类似的还有:(2012年福建理科卷第20题) 已知函数$f(x)=e^x+ax^2-ex$,$a\in R$. (1)若曲线$y=f(x)$在点$(1,f(1))$处的切线平行于$x$轴,求函数$f(x)$的单调区间; (2)试确定$a$的取值范围,使得曲线$y=f(x)$上存在唯一的点$P$,曲线在该点处的切线与曲线只有一个公共点$P$. 详见http://bbs.pep.com.cn/forum.php? ... &extra=page%3D4 第15楼
第一章 4# 2013-6-4 20:00
本帖最后由 第一章 于 2013-6-4 20:14 编辑 第二题:(2005年福建卷) $f(x)$是定义在$R$上的以$3$为周期的奇函数,且$f(x)=0$在区间$(0,6)$内解的个数的最小值是( ) $\color{red}{A.}2$  $\color{red}{B.}3$  $\color{red}{C.}4$ $\color{red}{D.}5$ 分析:∵$f(x)$是奇函数,以$3$为周期,$f(2)=0$, ∴$f(0)=0$, $f(3)=f(0+3)=f(0)=0$, $f(5)=f(2+3)=f(2)=0$    $f(-1)=f(2-3)=f(2)=0$, $f(1)=-f(-1)=0$ $f(4)=f(1+3)=f(1)=0$ 而$f(1.5)=f(1.5-3)=f(-1.5)=-f(1.5)$ 也即$f(1.5)=-f(1.5)$, $f(1.5)=0, f(4.5)=f(1.5+3)=0$ 由此可见,$f(x)=0$在区间$(0,6)$内的解有$7$个,分别是:$1$、$2$、$3$、$4$、$5$、$1.5$、$4.5$ 四个选项中都没有正确答案,说明出题者当时忽视了$f(4.5)=f(1.5)=0$也成立的情况.
第一章 5# 2013-6-4 20:02
第三题: 已知$0<a<1$,关于$x$的方程$a^{|x|}=|\log_ax|$的实数根的个数为( ) $\color{red}{A.}1$    $\color{red}{B.}2$    $\color{red}{C.}3$    $\color{red}{D.}4$ 选$\color{red}{B}$么?看看附件的图,k作的。 实际上,当$a\in[e^{-e},1)$时,交点有2个;当$a\in(0,e^{-e})$时,交点有4个. 相关的讨论见:http://bbs.pep.com.cn/forum.php? ... &extra=page%3D1
第一章 6# 2013-6-4 20:02
第四题:(扬州市2012~2013学年度第一学期期末检测高三数学试卷) 数列$\{a_n\}$满足$a_1>1$,$a_{n+1}-1=a_n(a_n-1)$,($n\in N^*$),且$\frac{1}{a_1}+\frac{1}{a_2}+\dots+\frac{1}{a_{2012}}=2$,  求$a_{2013}-4a_1$的最小值. 命题人的意图可能是把条件$a_{n+1}-1=a_n(a_n-1)$倒过来然后裂项,最后也许就转化为基本不等式的环境,然而…… 数列的每一项由$a_1$唯一确定,所以根据后面的等式,理论上可以将$a_1$解出来,尽管可能不止一个解,但这不会是连续的变量,是离散的一些数值,$a_{2013}$也一样,也就是说,所求的式子 $a_{2013}?4a_1$只能取若干个固定的值(极可能只有一个,感觉上),我想这肯定是出题人并未曾考虑的事情。 这里贴的是k的原话,更详细的见http://kkkkuingggg.5d6d.net/thread-1095-1-10.html
第一章 7# 2013-6-4 20:05
原来还收集了几个,放在另一台电脑里面。不过今天整理桌面的时候居然被我删了。算了,天意如此,也懒得去恢复文件了。 希望大家接着发题
isea 8# 2013-6-4 23:05
学习了,第一题,还真还真没注意过。 第二题,错得太经典了。
零定义 9# 2013-6-5 14:51
4# 第一章 等等...我还没反应过来...$f(2)=0$咋来的? 应该是我笨,看了很久都没看出...
yes94 10# 2013-6-5 23:48
9# 零定义 我也没有反应过来 ,很久没上论坛了,是因为脑子不好使了吧
thread-1631-1-1.html: [数列] 按这样,下一年不好办了啊
isea 1# 2013-6-5 19:56
本帖最后由 isea 于 2013-6-5 20:41 编辑 看看2013年海淀中考数学二模,第12题。 我觉着这个拿到数列中去也算是比较难的题了,与人教高中必修5很贴切,从内容上说。 当然,不了解这些,也没太大影响。或者干脆说:竞赛味道
isea 2# 2013-6-5 20:43
中考高的最后一个选择,填空,最后一题,及部分中间某一题 叫 创新新 创新嘛,无可厚非,对数学有兴趣 且喜欢 的 还真不是无从下手的题
李斌斌755 3# 2013-6-5 20:58
现在是中考考高中题,高考考大学的题
isea 4# 2013-6-5 21:13
本帖最后由 isea 于 2013-6-5 21:40 编辑 呵呵,把第8题也帖了。 (PS:如果要求求CM的表达式……,这个式子可能会比较简单的三角函数) (PPS:求出来了:$y^2=4\sqrt3 \sin (\dfrac{\pi} {6}x-\dfrac {\pi} {3})+7$
yes94 5# 2013-6-5 21:29
1# isea 这题就是先要把人唬住,
isea 6# 2013-6-5 21:51
5# yes94 包装不错的纸老虎
thread-1632-1-1.html: [几何] 其实还是2013年海淀中考二模
isea 1# 2013-6-5 20:29
这个第22题,画框部分,如果单独拿出来,个人觉得,这个小题有点意思,大家先看看。
yes94 2# 2013-6-5 21:32
isea是高中还是初中教师?怎么高初中题目通吃啊!全能型教师!
isea 3# 2013-6-5 21:40
2# yes94 最重要的是几何不分初高中啊
kuing 4# 2013-6-5 21:51
3# isea 分得厉害好吧。。。初中生做不了高中几何题,反过来也是!
isea 5# 2013-6-5 21:53
4# kuing 这个,省略了半句,多数时候
isea 6# 2013-6-5 21:55
本帖最后由 isea 于 2013-6-5 22:14 编辑 7月后,悠闲数学娱乐论坛将变成一个记忆,没这么自由的地方了,不好玩啊 在这里,最大所获是学会用LaTeX,尽量用矢量图,不用再转化成word对象了……
yes94 7# 2013-6-5 22:39
6# isea 万一有转机呢》万一是黑客写的“通告”呢?
thread-1633-1-1.html: [几何] 椭圆内接三角有且仅有三个
isea 1# 2013-6-5 22:07
本帖最后由 isea 于 2013-6-5 22:17 编辑 这题以前应该是博客上看过一眼,就是扫了下,有印象。 具体动手bào力算出长半轴于短半轴比值关系,感觉计算量吓人啊,化简,立方和公式分解等,虽是常规手段,但现在(考试里)用得极少,一般情况下。 特殊情况下也能搞定,将离心率无限的接近于0与1(或者直接考察圆中情况),就能看出结果 题:三个顶点均在椭圆上的三角形称为椭圆的内接三角形。已知点$A$是椭圆的一个短轴端点,如果以$A$为直角顶点的椭圆内接等腰直角三角形有且仅有三个,则椭圆的离心率的取值范围是(  ) A.$(0,\dfrac {\sqrt2}2)$               B.$(\dfrac {\sqrt2}2,\dfrac {\sqrt6}3)$            C.$(\dfrac {\sqrt2}2,1)$                 D.$(\dfrac {\sqrt6}3,1)$
李斌斌755 2# 2013-6-6 01:16
感觉有几何法,一时没想到
李斌斌755 3# 2013-6-6 01:41
没计算。如图,$0<2b<R,2a=\sqrt3R$
thread-1634-1-1.html: 存档正式开始,请勿发贴回贴
kuing 1# 2013-6-6 17:27
记存 header 设置 <script type="text/x-mathjax-config">   MathJax.Hub.Config({     tex2jax: {       inlineMath: [ ['$','$'], ["\\(","\\)"] ],       processEscapes: true       },     TeX: {       equationNumbers: { autoNumber: "AMS" },       Macros: {         mbb: '\\mathbb',         riff: '\\implies',         liff: '\\impliedby',         abs: ['\\left\\lvert #1\\right\\rvert', 1],         rmd: ['\\mathop{\\mathrm{d}#1}', 1],         vv: '\\overrightarrow',         sslash: '\\mathrel{/\\!/}',         pqd: '\\stackrel{\\,\\sslash}{\\raise-.5ex{=\\!\\!\\!\\!=}}',         veps: '\\varepsilon',         du: '^\\circ'       }     },     "HTML-CSS": {       linebreaks: {automatic: true}     },       menuSettings: {         zoom: "Double-Click"     }   }); </script> <script type="text/javascript"   src="http://c328740.r40.cf1.rackcdn.com/mathjax/latest/MathJax.js?config=TeX-AMS-MML_HTMLorMML"> </script>
thread-164-1-8.html: [不等式] $a+b+c=3$轮换
pxchg1200 1# 2011-10-29 19:34
Let$a,b,c>0$ with $a+b+c=3$ prove that: \[ \frac{a}{b^{5}+1}+\frac{b}{c^{5}+1}+\frac{c}{a^{5}+1}\geq \frac{3}{2} \] PS:今天的数学竞赛考衰了。。。 故贴个题来调整下心情。
kuing 2# 2011-10-30 12:09
你说的竞赛是大学生数学竞赛吗?是否跟这个贴里说的一样:http://bbs.pep.com.cn/thread-1948721-1-1.html
pxchg1200 3# 2011-10-30 12:25
2# kuing 是啊,在华南理工考的。题目难死了。两个半小时基本发呆。。。。
pxchg1200 4# 2011-10-30 12:29
本帖最后由 pxchg1200 于 2011-10-30 12:39 编辑 1# pxchg1200 类似的: \begin{align} 2)\frac{a}{b^{6}+1}+\frac{b}{c^{6}+1}+\frac{c​}{a^{6}+1}\geq \frac{3}{2}.\\ 3) \frac{a}{b^{7}+1}+\frac{b}{c^{7}+1}+\frac{c​}{a^{7}+1}\geq \frac{3}{2}.\\ 4) \frac{ab}{b^{3}+1}+\frac{bc}{c^{3}+1}+\frac{ca}{a^{3}+1} \geq \frac{3}{2}.\\ 5) \frac{ab^{2}}{b^{3}+1}+\frac{bc^{2}}{c^{3}+1}+\frac{ca^{2}}{a^{3}+1}\ge​ \frac{3}{2}.\\ 6) \frac{ab}{b^{4}+1}+\frac{bc}{c^{4}+1}+\frac{ca}{a^{4}+1}\geq \frac{3}{2}.\\ \end{align} 这些都简单不到哪去。。。。
kuing 5# 2011-10-30 16:38
4# pxchg1200
pxchg1200 6# 2011-10-30 16:51
5# kuing 先说明那个柯西求反技术没用了,因为下面分母次数太高,命题变强了。。。 直接柯西上的话,运算量不是一般的大,我的电脑都死机。。。
thread-165-1-8.html: [不等式] 来自pep兴趣小组的三角不等式$\sum1/(1+\cos^2A+\cos^2B)\leqslant2$
kuing 1# 2011-10-29 23:38
来自:http://bbs.pep.com.cn/thread-1946686-1-1.html 在任意 $\triangle ABC$ 中,有 \[\frac1{1+\cos^2A+\cos^2B}+\frac1{1+\cos^2B+\cos^2C}+\frac1{1+\cos^2C+\cos^2A}\leqslant2.\] 我们先证明当 $\triangle ABC$ 为直角或钝角三角形时不等式成立。不妨设 $C\geqslant90^\circ$,则 \begin{align*} A+B\leqslant 90^\circ &\implies 0<A\leqslant 90^\circ-B<90^\circ \\ &\implies 1>\cos A \geqslant \cos(90^\circ-B)=\sin B>0\\ &\implies \cos^2A\geqslant\sin^2B>0, \end{align*} 由此可得 \begin{align*} LHS(原不等式左边)&\leqslant\frac1{1+\sin^2B+\cos^2B}+\frac1{1+\cos^2B}+\frac1{1+\sin^2B}\\ &=\frac12+\frac3{2+\sin^2B\cos^2B}\\ &<\frac12+\frac32=2, \end{align*} 即此时不等式成立。 再证当 $\triangle ABC$ 为锐角三有形时不等式成立。令 $2A=\pi-D$,$2B=\pi-E$,$2C=\pi-F$,由 $A,B,C$ 为锐角知 $D,E,F\in(0,\pi)$,且三式相加易得 $D+E+F=\pi$,因此三个角 $D,E,F$ 能构成一个三角形,于是代入即知只要证如下三角形不等式 \[\frac1{4-\cos D-\cos E}+\frac1{4-\cos E-\cos F}+\frac1{4-\cos F-\cos D}\leqslant1,\] 上式采用 $s$-$R$-$r$ 化后等价为 \[(3R-r)s^2-40R^2r+13Rr^2-r^3\geqslant0,\] 由欧拉不等式 $R\geqslant2r$ 及 Gerretsen 不等式 $s^2\geqslant16Rr-5r^2$,得 \begin{align*} (3R-r)s^2-40R^2r+13Rr^2-r^3&\geqslant(3R-r)(16Rr-5r^2)-40R^2r+13Rr^2-r^3\\ &=2r(R-2r)(4R-r)\geqslant0, \end{align*} 即时此不等式成也立。 综上所述,原不等式成立。
力工 2# 2011-10-30 18:20
1# kuing 用cauchy不等式搞掂。sinC^2=(sinAcosB+cosAsinB)^2<=(sina^2+sinB^2)(cosA^2+cosB^2),得cosA^2+cosB^2>=sinC^2/(sinA^2+cosB^2) 以下略
kuing 3# 2011-10-30 20:32
2# 力工 嗯,这个证法好,不过过程打错了一点。 PS,楼上试试用Latex输入
thread-166-1-8.html: 在pep看到的2011全国大学生数学竞赛一道三角函数题要初等解
kuing 1# 2011-10-30 12:15
来自:http://bbs.pep.com.cn/thread-1948721-1-1.html 贴内一楼说到 原帖由 马尾 于 2011-10-29 20:12 发表 今天上午刚考的一道题,应该是用条件极值做吧,我想问一下用初等方法怎们做?已知A,B,C为三角形ABC的三个角求3sinA+4sinB+18sinC的最大值 我在二楼回到 原帖由 kuing 于 2011-10-30 01:08 发表 用加权正弦和不等式,系数解解方程即可,而加权正弦和不等式只需用柯西和嵌入不等式即可证,嵌入不等式又配方可证,所以理论上,算是初等方法。 时间关系,明天有空再详写写。 现详细写于此 在 $\triangle ABC$ 中,求 $3\sin A+4\sin B+18\sin C$ 的最大值。 lemma  (加权正弦和不等式) 对任意实数 $x,y,z$ 及正数 $u,v,w$ 及任意 $\triangle ABC$,有 \[yz\sin A+zx\sin B+xy\sin C\leqslant \left(\frac{x^2}u+\frac{y^2}v+\frac{z^2}w\right)\frac{\sqrt{vw+wu+uv}}2.\] 其中等号成立当且仅当 $x:y:z=\cos A:\cos B:\cos C$ 且 $u:v:w=\cot A:\cot B:\cot C$。 时间关系,引理的证明就不详写了,请自行查阅相关文献,比如小丛书的《几何不等式》里面就有。下面用此引理解决原题: 令 $yz=3$,$zx=4$,$xy=18$,解得一组解为 \[x=2\sqrt6,y=3\sqrt{\frac32},z=\sqrt{\frac23},\] 而对于另外一组参数 $u,v,w$ 的取值,则需要满足取等条件,为此,先求出取等时余弦值的比,由 \[2\sqrt6:3\sqrt{\frac32}:\sqrt{\frac23}=\cos A:\cos B:\cos C,\] 以及三角恒等式 \[\cos^2A+\cos^2B+\cos^2C+2\cos A\cos B\cos C=1,\] 不难解出唯一一组符合条件的解为 \[\cos A = \frac34, \cos B = \frac9{16}, \cos C = \frac18,\] 由此又易得 \[\cot A:\cot B:\cot C=45:27:5,\] 于是取 $u=45$,$v=27$,$w=5$ 即可。 将以上数据代入,就得到 \[3\sin A+4\sin B+18\sin C\leqslant\frac{35 \sqrt{7}}{4},\] 当 $\cos A = \frac34$, $\cos B = \frac9{16}$, $\cos C = \frac18$ 时等号成立。
pxchg1200 2# 2011-10-30 12:27
1# kuing 求解!这题我不会做啊!
kuing 3# 2011-10-30 13:36
2# pxchg1200 刚才写了一半吃饭去了,现在补好了
kuing 4# 2011-10-30 16:33
大学生解法应该是拉格朗日乘数吧,极值点最后要解方程组 \[\left\{\begin{aligned}18 \cos(A+B) + 3 \cos A &= 0,\\ 18 \cos(A+B)+ 4 \cos B &= 0,\end{aligned}\right.\] 只有一个符合条件的解 $\cos A=\dfrac34, \cos B=\dfrac9{16}$
pxchg1200 5# 2011-10-30 16:53
4# kuing 哎呀,考试的时候完全不会做了,本来想用拉格朗日乘数法,可方程解不来,初等的不等式又没几何不等式这方面的研究,结果就挂定了。。。还是你说得对:珍爱生命,远离考试。。。
力工 6# 2011-10-31 09:25
1# kuing 有更简方法的路过。 楼主强壮,这些结论是背下了还是临时翻的?太强壮了。
maweijiao 7# 2011-10-31 09:57
1# kuing 版主好强大啊,考试的时候只算出三个角的余弦之比,之后就不会了,还是知道的太少了,向版主学习~~~~
kuing 8# 2011-10-31 11:32
6# 力工 说说更简单的啊 那个加权正弦和不等式见过一次之后就记得了
海盗船长 9# 2011-11-2 18:09
求引理证明
hhhzh7241hzh 10# 2011-11-2 23:40
http://bbs.pep.com.cn/thread-1948721-1-1.html
thread-167-1-1.html: 【问】每次进数学讨论区看帖,CPU都会狂飙,何故?
戊概念·五 1# 2011-10-30 14:06
RT 求解
戊概念·五 2# 2011-10-30 14:08
还有,勾选“使用个人签名”后,为何不显示?
kuing 3# 2011-10-30 14:18
1# 戊概念·五 可能大概应该估计基本上差不多是因为处理公式,吧
kuing 4# 2011-10-30 14:19
2# 戊概念·五 你签名档写东西了没有
戊概念·五 5# 2011-10-30 18:26
1# 戊概念·五 可能大概应该估计基本上差不多是因为处理公式,吧 kuing 发表于 2011-10-30 14:18 居然用到如此之多的副词..... 是不是使用IE浏览器的问题?
戊概念·五 6# 2011-10-30 18:27
4# kuing 当然有写
kuing 7# 2011-10-30 20:45
那我不知道了
戊概念·五 8# 2011-10-30 20:58
7# kuing
戊概念·五 9# 2011-10-30 20:59
还有,勾选“使用个人签名”后,为何不显示? 戊概念·五 发表于 2011-10-30 14:08 出来了,要点两次复选框才有效
kuing 10# 2011-10-30 21:28
9# 戊概念·五 我选过一次之后就一直是默认选了的。
戊概念·五 11# 2011-11-1 15:18
10# kuing 也许是我的鼠标逗我玩呢,近一年里的表现一直都是时好时坏的 现在好了就ok~!!!!! Ps:这里的设置很显人性化,换头简单易行,攒个~!
thread-168-1-2.html: 等距变换($m=r_\theta s$)
图图 1# 2011-10-30 17:35
证明:m=$r_\theta$s是一个关于过原点且与x轴交角为$\frac12\theta$的直线的反射。
海盗船长 2# 2011-11-5 18:20
不懂,,,
kuing 3# 2011-11-5 21:42
大概意思就是:$A$ 对 $x$ 轴反射后变成 $A'$,然后 $A'$ 绕原点逆时针旋转 $\theta$ 角变成 $A''$,又记 $x$ 轴绕原点逆时针旋转 $\theta/2$ 所成直线为 $L$,证明 $A''$ 与 $A$ 关于 $L$ 对称。
kuing 4# 2011-11-5 21:47
由几何图形看来是显然的,用角度加加减减即可,就是用代数那些坐标变换去写可能麻烦些
kuing 5# 2011-11-5 22:10
设 $A(x,y)$,则 $A'(x,-y)$,则 $A''(x\cos\theta-(-y)\sin\theta,(-y)\cos\theta+x\sin\theta)$。由于显然 $|OA|=|OA''|$ 且 $L$ 过 $O$,于是只要证明 $AA''$ 的中点在 $L$ 上(三线合一),而 $AA''$ 中点坐标为 \[\left(\frac{x+x\cos\theta-(-y)\sin\theta}2,\frac{y+(-y)\cos\theta+x\sin\theta}2\right) = \left(\frac{x(1+\cos\theta)+y\sin\theta}2,\frac{y(1-\cos\theta)+x\sin\theta}2\right),\] 故只要证 \[\frac{y(1-\cos\theta)+x\sin\theta}2\cdot\cos\frac{\theta}2=\frac{x(1+\cos\theta)+y\sin\theta}2\cdot\sin\frac{\theta}2,\] 用两倍角公式可知上式显然成立,即得证。
thread-169-1-8.html: [几何] 来自人教论坛的抛物线求等边三角形斜率问题
nash 1# 2011-10-30 22:00
本帖最后由 nash 于 2011-10-30 22:19 编辑 http://bbs.pep.com.cn/thread-1950655-1-1.html 来自人教论坛的抛物线求等边三角形斜率问题
nash 2# 2011-10-30 22:12
本帖最后由 nash 于 2011-10-30 22:18 编辑 汗 后面的把AB写成BF啦 已改……
thread-17-1-9.html: [不等式] 一元条件不等式$3 - 3x^2 + x^3 >0$(已求得简证)
kuing 1# 2011-9-26 20:39
设实数 $x$ 满足 $x\geqslant0$ 且 $1+4x^2-4x^3\geqslant0$,求证 \[3 - 3x^2 + x^3 >0\]
①②③④⑤⑥⑦ 2# 2011-9-29 17:09
本帖最后由 ①②③④⑤⑥⑦ 于 2011-9-29 17:10 编辑 晕,操作失误,再来一次 $0\leqslant x<1$ 时,$3-3x^2+x^3=3(1-x^2)+x^3>0$ $1\leqslant x<\dfrac{4}{3}$ 时,$3-3x^2+x^3>3+(3x-4)-3x^2+x^3=(x-1)^3\geqslant0$ $x\geqslant\dfrac{4}{3}$ 时,$1+4x^2-4x^3=(1-x^3)+x^2(4-3x)<0$
kuing 3# 2011-9-29 17:25
哟,这个分类讨论,very nice 的说 嘿,谢谢啊,终于不用解三次方程了……
realnumber 4# 2011-10-8 14:10
本帖最后由 realnumber 于 2011-10-8 15:08 编辑 $1≥4x^3-4x^2$,$x^3+x^3+1≥3x^2$ $3+x^3=\frac{9}{4}+\frac{3}{4}+x^3≥\frac{9}{4}+\frac{3}{4}(4x^3-4x^2)+x^3=\frac{1}{4}+(4x^3+2)-3x^2≥\frac{1}{4}+3x^2≥3x^2$
kuing 5# 2011-10-8 22:49
4# realnumber 没看出什么问题,也very nice! 谢谢 PS,公式输入方面,大于等于用 \ge ,第二行可以用行间公式去写,这样分子就不会变小。 \[3+x^3=\frac{9}{4}+\frac{3}{4}+x^3\ge\frac{9}{4}+\frac{3}{4}(4x^3-4x^2)+x^3=\frac{1}{4}+(4x^3+2)-3x^2\ge\frac{1}{4}+3x^2>3x^2\]
thread-170-1-8.html: [不等式] 貌似排序不等式,指数型
realnumber 1# 2011-11-1 10:43
本帖最后由 realnumber 于 2011-11-1 11:46 编辑 a,b为正数,有$a^a+b^b  \ge  a^b+b^a$,几何画板画了下成立,似乎也不强,如果成立也容易推广到n元。 “a>b>0,c>d>0,则$c^a+d^b>c^b+d^a$.   ”似乎要还要加什么条件才成立。
kuing 2# 2011-11-1 21:41
第一个估计已有研究,一时没找到相关文献
kuing 3# 2011-11-1 22:13
找到这个但下不了,看标题和正文快照,估计就是第一个问题 http://epub.cnki.net/grid2008/de ... amp;dbname=CJFD2002
pxchg1200 4# 2011-11-1 23:04
3# kuing 为什么会下不了呢?我毫无压力的就下了。。。
kuing 5# 2011-11-1 23:15
我没帐号 是不是大学里的就可以直接下?
kuing 6# 2011-11-1 23:18
4# pxchg1200 这个好像也是跟这个有关的 http://wuxizazhi.cnki.net/Article/SXTB200909014.html 可以下不?
pxchg1200 7# 2011-11-1 23:18
5# kuing 应该不是,我下的时候它没问我要账号啊...
pxchg1200 8# 2011-11-1 23:20
6# kuing 这个不行了。。。。
kuing 9# 2011-11-1 23:28
7# pxchg1200 那就是你在学校的网络直接通行知网。。。。
图图 10# 2011-11-2 00:38
3# kuing 唔,这个我也能下载,下面那个我有账号,但是下载需要钱
kuing 11# 2011-11-2 01:18
10# 图图 看来大学校园网的都直接通行,这样子以后找论文就方便多哩
kuing 12# 2011-11-2 01:21
我点击下载都提示 “贵单位没有订购该产品,请您与贵单位管理员联系订购!”
鱼儿 13# 2011-11-2 09:13
6# kuing 是这个吗?
kuing 14# 2011-11-2 11:16
13# 鱼儿 下载了打不开,提示载入时发生错误,下了几次也如此。
鱼儿 15# 2011-11-2 14:40
本帖最后由 鱼儿 于 2011-11-2 14:51 编辑 给文本换个格式试试。
kuing 16# 2011-11-2 16:36
15# 鱼儿 oh thanks
海盗船长 17# 2011-11-2 18:08
oh.nice!
djx2011 18# 2011-11-5 22:56
那个下载是有ip限制的,大学里面上网可以直接下,外网需要账号才行的。
kuing 19# 2011-11-5 23:05
18# djx2011 原来如此
thread-171-1-1.html: 制作动态图
kuing 1# 2011-11-1 17:31
用 mathematica 的命令整的……
kuing 2# 2011-11-1 17:51
方法就是用 Table 输出一堆图,然后用 Export["XXX.gif", %] 即可,貌似默认保存到我的文档。
kuing 3# 2011-11-1 18:26
由以下命令得到 Table[Plot3D[Sin[x + a] Sin[y + a], {x, 0, 2 Pi}, {y, 0, 2 Pi}, PlotRange -> 1, PlotPoints -> 50, PlotLabel -> Style[TraditionalForm[z == Sin[x + a] Sin[y + a]], 16, Blue]], {a, 0, Pi, 0.05 Pi}]; Export["sinsin.gif", %]
戊概念·五 4# 2011-11-2 14:16
想到了你以前发的某篇日志,心形方程的那个
kuing 5# 2011-11-2 18:27
多谢提醒 制作中,并遇到困难中
戊概念·五 6# 2011-11-2 20:46
5# kuing 难不倒你的
kuing 7# 2012-9-20 21:53
补充说一句,在输出动态图时,还可以用大尺寸或小尺寸等,方法是在后面加上选项 ImageSize -> 尺寸,尺寸可以用 Large、Medium、Small、Tiny,甚至直接指定大小,比如 ImageSize -> {1024, 768} 即可将输出的图片尺寸指定为 1024*768(原图会放大,但不会拉伸到完全填满尺寸)。
kuing 8# 2012-9-20 21:59
不知有没有选项能调节输出的动态图每帧的时间间隔?
kuing 9# 2012-9-30 14:59
thread-172-1-1.html: 动态参数控制
kuing 1# 2011-11-1 17:58
Animate[带参数的东东,{参数1,初值,终值,步长(可省)},{参数2,初值,终值,步长(可省)}(可省),...,选项(可省)] 很多选项,帮助里各种例子看得我都 简单的例子: Animate[Plot3D[Sin[x + a] Sin[y + b], {x, 0, 2 Pi}, {y, 0, 2 Pi}, PlotRange -> 1, PlotPoints -> 50], {a, 0, Pi}, {b, 0, Pi}] 出来的东东可以点击暂停参数,然后可以手动拖动改变参数值观察变化。
图图 2# 2011-11-28 13:57
1# kuing C语言上机时间,突然发现机房这mathematica居然没有装好....
kuing 3# 2011-11-28 14:00
2# 图图 能不能换机
图图 4# 2011-11-28 14:03
3# kuing 呃,这个应该都一样吧...我再玩会就做C去啦...
kuing 5# 2011-11-28 14:10
4# 图图 嗷嗷
thread-173-1-1.html: [Chemistry]NaHSO3(aq)的各微粒浓度大小。
Chetion 1# 2011-11-2 00:14
本帖最后由 Chetion 于 2011-11-5 08:59 编辑 已知NaHSO3(aq)显酸性。 NaHSO3  ==  Na +   +  HSO3 - HSO3 -  <==>  H +  +  SO3 2-  (电离为主) HSO3 -  +   H2O  <==>  H2SO3  +  OH - (水解很少) 资料书上认为  c(Na +)>c(HSO3 -)>c(H +)>c(SO3 2- )>c(H2SO3)>c(OH -) 请结合上式解释一下为什么c(H2SO3)>c(OH -) (我的化学老师正好认为相反,他认为水的电离仍会产生出OH -。。。) 请指教!
kuing 2# 2011-11-2 01:21
哎,我也不懂哩,五六年没碰过化学了 等高手来。。。
秋风树林 3# 2011-11-2 16:44
本帖最后由 秋风树林 于 2011-11-2 16:50 编辑 自从高考完了之后我就抛弃掉了中学那套判定法则。。。 不清晰的直接用溶液里的平衡常数。。。 当然我还是回忆下好了。。
kuing 4# 2011-11-2 17:04
完全看不懂,闪人。。。
秋风树林 5# 2011-11-2 17:07
4# kuing 你几年没接触很正常嘛。。只要楼主看得懂就好。。。
戊概念·五 6# 2011-11-2 17:41
自从高考完了之后我就抛弃掉了中学那套判定法则。。。 不清晰的直接用溶液里的平衡常数。。。 当然我还是回忆下好了。。 74 秋风树林 发表于 2011-11-2 16:44 可以解释下最后一步“所以”的依据是?
秋风树林 7# 2011-11-2 17:52
6# 戊概念·五 就只需明白为什么H+浓度比亚硫酸根浓度大就行了。。。
戊概念·五 8# 2011-11-2 18:03
7# 秋风树林 啊~!是这样,懂了懂了~!!!!! 大谢大谢~!!!!! Ps:多年不碰化学,退化地真凄惨= =
秋风树林 9# 2011-11-2 18:22
8# 戊概念·五 其实这些东西我真心觉得挺无聊的。。。没啥意义。。。= =
戊概念·五 10# 2011-11-2 18:27
9# 秋风树林 ……确~ 现在这年头,有多少人无奈的无聊着呐~ 比如我
Chetion 11# 2011-11-3 23:00
本帖最后由 Chetion 于 2011-11-3 23:03 编辑 3# 秋风树林 万谢~~~ 4楼的化学功底实在实在忒强nei~~ 由物料、电荷守恒式推出了质子守恒式,然后很直观地证明出来。。 不过LS的这位高手如何果断地选定了质子守恒式? 而且,如何利用平衡常数证明~~(此思路从未出现过脑海。感觉太多平衡太复杂。。。也许高中知识的局限性吧。) 求教ing ......
kuing 12# 2011-11-3 23:14
11# Chetion 好像是三楼才是……
秋风树林 13# 2011-11-4 18:36
11# Chetion 平衡常数算起来确实很麻烦。。。= = 而且我好久没用那些化学热力学的东西都忘得差不多了。。。 质子守恒呃,差不多感觉吧。。。反正都出现了氢离子氢氧根什么的。。。
thread-175-1-1.html: 为何每次浏览有总提示脚本错误?!
戊概念·五 1# 2011-11-2 18:21
不停止运行就会导致停止响应
kuing 2# 2011-11-2 18:26
何种浏览器?
戊概念·五 3# 2011-11-2 18:34
2# kuing IE
kuing 4# 2011-11-2 18:38
何版本?
戊概念·五 5# 2011-11-2 18:38
4# kuing 8
kuing 6# 2011-11-2 18:45
无法测试了,表示我的IE还是IE6,一般也不用了,现在用chrome
戊概念·五 7# 2011-11-2 18:46
无法测试了,表示我的IE还是IE6,一般也不用了,现在用chrome kuing 发表于 2011-11-2 18:45 哦
戊概念·五 8# 2011-11-2 18:47
4# kuing 8 戊概念·五 发表于 2011-11-2 18:38 第25帖留这里了~!
戊概念·五 9# 2011-11-9 16:54
刷了几次提示出现成随机事件 今又出新状况:
kuing 10# 2011-11-9 19:06
9# 戊概念·五 有时候mathjax的公式文件下载或读取不成功就会如此,不过我很少碰到这情况,基本上都很顺
戊概念·五 11# 2011-11-10 21:41
10# kuing 懂了。 不过,又出了点状况:
kuing 12# 2011-11-10 21:44
这不是很正常的事么 所有有公式的标题都应如此
戊概念·五 13# 2011-11-11 11:30
12# kuing 明白。 我是白白= =
thread-176-1-2.html: 话说现在新注册百度文库只有2点财富值了哎
kuing 1# 2011-11-2 18:41
RT,之前是新注册有10点,不知啥时候改的哎,现在下载东东越来越难哟哩……
pxchg1200 2# 2011-11-5 08:33
1# kuing kk有听说过可以免费下载百度文库的么? 我好像看见人人网里某位网友转发过这篇文章来着。。。 PS。今天kk这么早就起来了啊,平时都是吃过午饭才看到上线的。。
kuing 3# 2011-11-5 08:41
2# pxchg1200 怎么下载? 今天有节目嘿所以早起
pxchg1200 4# 2011-11-5 08:45
3# kuing 在百度中搜索“如何免费下载百度文库的文档” 里面都是讲这个的,暂时没试过。。 Ps:我早起是由于宿舍里有such a PSP gamer,一大早起来就打机,被他吵死了,悲剧。
kuing 5# 2011-11-5 08:59
看到有两种,一种是用快照,但是只对纯文字的word才有用,对于我们来说没什么用了,第二种好像很复杂:http://wenku.baidu.com/view/91b8990590c69ec3d5bb7500.html
thread-177-1-1.html: 口畏我是兔兔~
戊概念·五 1# 2011-11-2 18:52
羡慕那些有人陪着一起犯二的人.....
戊概念·五 2# 2011-11-8 10:35
1# 戊概念·五 真的很可爱呐~ Ps:题目借用了小妹的网名.....嘿~
李斌斌755 3# 2013-5-7 02:12
没人做题,也没人吹水。奇怪这贴里的相关帖子怎么是求面积
kuing 4# 2013-5-7 02:31
3# 李斌斌755 你还留意那个相关帖子啊,我基本不怎么看那里。 话说你是什么时区的啊,比我还晚睡。
李斌斌755 5# 2013-5-7 03:04
4# kuing 我是中国最晚时区
thread-178-1-8.html: 一道趣味概率题
nash 1# 2011-11-3 00:26
本帖最后由 nash 于 2011-11-3 00:30 编辑 一个盲人骑马站在悬崖边,如果他向前一步就掉落深渊,已知马向前一步的概率为$\frac{1}{2}$,向后一步的概率为$\frac{1}{2}$,马在悬崖上徘徊,求盲人最后掉落深渊的概率。
kuing 2# 2011-11-3 01:45
必死无颖?
①②③④⑤⑥⑦ 3# 2011-11-3 09:06
2# kuing 0101010101…… 倒也不是必死无疑的 很著名的问题,嗯,相关的 http://bbs.pep.com.cn/viewthread ... d=976841#pid6873706 不过问题编成这样也太生硬了,马向后走和向前走概率一样,这是什么马啊,而且马又不是瞎子,看见悬崖还会往前走……
nash 4# 2011-11-3 11:02
在一个QQ群里面看到的一个问题 有人给出了答案为1 而且给出了一个简单的解释, 用楼上给出的公式求解,需要求一个数列和的极限,昨天就试过了,没算出来… 想知道这个答案是不是就等于1 你们怎么做的?
kuing 5# 2011-11-3 18:56
3# ①②③④⑤⑥⑦ 唔,有空慢慢研究。。。 嘿,那个题设的确是搞笑的……
①②③④⑤⑥⑦ 6# 2011-11-4 09:35
http://oeis.org/A001405
nash 7# 2011-11-4 12:53
看好久才把链接的文章看完了 没看出那个链接对这道题有什么帮助 谢谢楼上的
icesheep 8# 2011-11-17 02:45
本帖最后由 icesheep 于 2011-11-17 13:29 编辑 一个重要结论就是,如果悬崖在马身后,设马走了 m 步向后,n步向前,也即共走了 n+m 步,则不会落下悬崖的走法共有 \[\left( {\begin{array}{*{20}{c}}   {m + n} \\   m \end{array}} \right) - \left( {\begin{array}{*{20}{c}}   {m + n} \\   {m - 1} \end{array}} \right)\]
①②③④⑤⑥⑦ 9# 2011-11-17 10:08
一个重要结论就是,设马走了 m 步向后,n步向前,也即共走了 n+m 步,则不会落下悬崖的走法共有 \[\left( {\begin{array}{*{20}{c}}   {m + n} \\   m \end{array}} \right) - \left( {\begin{array}{*{20}{c}} ... icesheep 发表于 2011-11-17 02:45 只在 m=n 时成立,就是 Catalan 数 前面给出的那个序列,与Dyck paths有关,  去搜索一下就知道是同一个问题,这个序列则是相应的扩展,不要求必须回来,其实相当于楼主的走n步没死 Number of left factors of Dyck paths, consisting of n steps. Example: a(4)=6 because we have UDUD, UDUU, UUDD, UUDU, UUUD and UUUU, where U=(1,1) and D=(1,-1).
icesheep 10# 2011-11-17 12:55
本帖最后由 icesheep 于 2011-11-17 13:04 编辑 9# ①②③④⑤⑥⑦ m不等于n也成立的,你看你的例子中,UUUD,向前走了三步,向后只走了一步,即 n=3,m=1 对应的走法数为 C(4,1)-C(4,0)=3 也即你列出的 UUUD,UUDU,UDUU 这三种
①②③④⑤⑥⑦ 11# 2011-11-17 13:01
9# ①②③④⑤⑥⑦ m不等于n也成立的。 icesheep 发表于 2011-11-17 12:55 $m=2, n=1$ $C_{2+1}^2-C_{2+1}^{2-1}=C_3^2-C_3^1=0$
icesheep 12# 2011-11-17 13:06
确实是零啊,说明 “向后走两步,向前走一步,不管怎么走都会掉下去”。
①②③④⑤⑥⑦ 13# 2011-11-17 13:09
12# icesheep 向后走是活啊,我看反了?
icesheep 14# 2011-11-17 13:13
13# ①②③④⑤⑥⑦ 我看反了。。。原来是头面向悬崖。。。那就是 n,m 表示的意义反一下。。。
①②③④⑤⑥⑦ 15# 2011-11-17 14:06
14# icesheep 嗯,这样应该是对的,最好加上 Max{...,0}
icesheep 16# 2011-11-17 14:57
约定 C(k,-1)=0 即可。我仍然采用m为后退,n为前进的描述方法, 用上述结论分成总步数 k=n+m 为奇数和偶数的情况分开讨论,对 m 从1到 ceil(k/2) 将不会掉落悬崖的路线求和, 最后可以得出结果是 f(k)=C(k,ceil(k/2)) 或者用递推的办法也可以,不过通项就要靠 ”猜测+数归“ 了。 f(1)=1 f(2k)=2f(2k-1) f(2k+1)=2f(2k)-h(2k) 其中 h(2k)=C(2k,k)-C(2k,k-1) ,也就是 Catanlan 数
①②③④⑤⑥⑦ 17# 2011-11-17 15:06
16# icesheep “约定 C(k,-1)=0 即可。”你原来的表达式是减法,不是这个约定能直接解决的。或者你限定一下 m<=n 倒是很容易
icesheep 18# 2011-11-17 15:10
17# ①②③④⑤⑥⑦ m>n 时一定掉下去,所以对活着的路线计数时,只需对 m 从 1 到 ceil[k/2] 求和,这个取整其实就这么来的。 我在想这个要怎么说明: \[\mathop {\lim }\limits_{k \to \infty } \frac{{f\left( k \right)}}{{{2^k}}} = 0\]
①②③④⑤⑥⑦ 19# 2011-11-17 15:43
18# icesheep 你都搞定通项了,这个极限不是很easy嘛……
icesheep 20# 2011-11-17 16:00
19# ①②③④⑤⑥⑦ 我不想用斯特灵公式。。。
thread-178-2-8.html:
lancelot_cc 21# 2011-12-8 21:27
这道题好像很简单的样子哎。。。 只要设掉下去的概率是x, 因为骑士一开始就在崖边,所以跑上来就有1/2的概率掉下去,另有1/2的概率后退一步到距离崖边1单位的地方; 那么就有 x=1/2+1/2*x^2 解得x=1~ 一定死
thread-179-1-8.html: 几何概型问题
世外陶源 1# 2011-11-3 13:30
在线段OA【0,1】上任意投三个点, 问由O至三点的三线段,能构成三角形 的概率?
kuing 2# 2011-11-3 18:53
参考 http://bbs.pep.com.cn/thread-430817-1-1.html http://bbs.pep.com.cn/thread-421274-1-1.html http://bbs.pep.com.cn/thread-167404-1-1.html http://bbs.pep.com.cn/thread-151277-1-773.html http://bbs.pep.com.cn/viewthread.php?tid=159501 http://bbs.pep.com.cn/viewthread ... p;extra=&page=1 http://bbs.pep.com.cn/thread-378689-1-1.html http://bbs.pep.com.cn/viewthread.php?tid=427183 http://bbs.pep.com.cn/viewthread.php?tid=293908 http://bbs.pep.com.cn/viewthread.php?tid=385576 http://bbs.pep.com.cn/viewthread.php?tid=390101 http://bbs.pep.com.cn/thread-355952-1-1.html http://bbs.pep.com.cn/thread-245673-1-1.html http://bbs.pep.com.cn/thread-247132-1-567.html http://bbs.pep.com.cn/thread-387742-1-1.html 总有几个贴跟楼主的题等价
thread-18-1-9.html: [不等式] 一元高次不等式$x^7-x^6-2 x^4-x^3+4 x+2>0$(求简证)
kuing 1# 2011-9-26 20:41
设 $x\in\mathbb{R}^+$,求证 \[x^7-x^6-2 x^4-x^3+4 x+2>0\]
yes94 2# 2011-10-2 14:21
x^7-x^6-2 x^4-x^3+4 x+2=0奇次方程有实数根的嘛!
kuing 3# 2011-10-2 14:22
2# yes94 看清楚条件OK?
yes94 4# 2011-10-2 14:30
∖[x^7-x^6-2 x^4-x^3+4 x+2∖] ,弄错了,x>0.
yes94 5# 2011-10-2 14:31
本帖最后由 yes94 于 2011-10-2 14:37 编辑 这个反斜杠,不好打,复制你的反斜杠也不行啊,公式无法输入, ∖x^7-x^6-2 x^4-x^3+4 x+2 ∖ ∖[x^7-x^6-2 x^4-x^3+4 x+2]∖
kuing 6# 2011-10-2 14:38
:L 难道你的键盘没有反斜杠? 记得是在英文状态下输入 不行的话就先用美元符号,不独行居中罢了
yes94 7# 2011-10-2 14:48
本帖最后由 yes94 于 2011-10-2 14:51 编辑 有反斜杠,但打出来是:#(英文状态),或者是顿号、(中文状态),和3上面的那个键重复了。 $x^7-x^6-2 x^4-x^3+4 x+2$ 看来美元符号行了
kuing 8# 2011-10-2 14:55
有办法,用双重美符号(两边各连续两个的美元符号)就相当于反斜杠那种 $$\sum\frac a{b+c}$$ 这样的话你就可以不打反斜杠了 哈,我去修改一下置顶贴,补充一下这个双美元输入行间……
kuing 9# 2011-10-2 15:13
哎,不过公式里头也常用反斜杠啊,这样你基本上还是打不了公式。。。你的电脑也太奇怪了,竟然会将反斜杠与3上面的#重复:L
kuing 10# 2011-10-4 21:19
其实早前我利用软件作了个图象,然后仅用了一种命令:Factor,得到如下超难看的式: \begin{align*} x^7-x^6-2x^4-x^3+4x+2 & = x\left(x^4+\frac{19}{10}x^3+\frac{1363}{400}x^2+\frac{3887}{1000}x+\frac{99457}{32000}\right)\left(x-\frac{29}{20}\right)^2\\ &\mathrel{\phantom{=}}{}+\frac{1345397}{1600000}\left(x-\frac{32443337}{21526352}\right)^2+\frac{49579106704431}{551074611200000} \end{align*}
pxchg1200 11# 2011-10-4 21:34
10# kuing 不知道kuing看过arqady的方法么?
kuing 12# 2011-10-4 21:35
10# kuing 不知道kuing看过arqady的方法么? pxchg1200 发表于 2011-10-4 21:34 什么方法?
kuing 13# 2011-10-4 21:37
后来倒是知道有个 Sturm 定理可以用上,但是想要用不等式证的
pxchg1200 14# 2011-10-4 21:38
我记得arqady介绍了一个方法可以迅速判别这类型的高次不等式。以补充他的uvw方法,链接很难找。。。
kuing 15# 2011-10-4 21:40
14# pxchg1200 不曾见过的说。。。
realnumber 16# 2011-10-8 14:52
本帖最后由 realnumber 于 2011-10-8 15:07 编辑 即要证明$x(x^6+4-x^4-2x^3-x^2)+2>0$ 而$x^6+x^6+x^6+x^6+2^3+2^3≥12x^4$即$x^6+4≥3x^4$ 只要证明$x(2x^4-2x^3-x^2)+2>0$即$2x^5+2>2x^4+x^3$ 又$x^5+x^5+x^5+1+1≥5x^3$即$\frac{3}{5}x^5+\frac{2}{5}≥x^3$ 只要证明$\frac{7}{5}x^5+\frac{8}{5}≥2x^4$ 而$x^5+x^5+x^5+x^5+2^{2.5}≥5\sqrt{2}x^4$即$\frac{8}{5\sqrt{2}}x^5+\frac{8}{5}≥2x^4$ 而$\frac{8}{5\sqrt2}<\frac{7}{5}$所以原不等式成立
hhhzh7241hzh 17# 2011-10-8 21:14
本帖最后由 hhhzh7241hzh 于 2011-10-8 21:16 编辑 即要证明$x(x^6+4-x^4-2x^3-x^2)+2>0$ 而$x^6+x^6+x^6+x^6+2^3+2^3≥12x^4$即$x^6+4≥3x^4$ 只要证明$x(2x^4-2x^3-x^2)+2>0$即$2x^5+2>2x^4+x^3$ 又$x^5+x^5+x^5+1+1≥5x^3$即$\frac{3}{5}x^5+\frac{2}{5}≥x^3$ ... realnumber 发表于 2011-10-8 14:52 第一步就错了!即要证明x(x^6+4-x^4-2x^3-x^2)+2>0 应该是即要证明x(x^6+4-x^5-2x^3-x^2)+2>0
kuing 18# 2011-10-8 21:55
17# hhhzh7241hzh 嗯,我差点也没发现,谢谢先。 PS,公式中尽量不要用discuz代码,容易造成公式不显示。想加颜色可以用latex里的 \color 代码,比如代码 x(x^6+4-\color{red}{x^5}-2x^3-x^2)+2>0 两边加美元符号后显示 $x(x^6+4-\color{red}{x^5}-2x^3-x^2)+2>0$
tian27546 19# 2011-10-19 14:45
注意到:$168(x^7-x^6-2x^4-x^3+4x+2)=4(17x+9)(x^3-3)^2+15x(x^2-2)^2+x^3(2x-3)^2(25x^2+24x+12)+12$
kuing 20# 2011-10-19 15:01
19# tian27546 oh,配方式正确,挺好看,谢过……
thread-180-1-8.html: [不等式] 睡觉前证一道来自群里的简单不等式
kuing 1# 2011-11-4 02:03
爱好者-Nash(277072575)  1:43:14 问个问题 正实数abc=1 \[ab(a+b)+bc(b+c)+ca(c+a)\geqslant a+b+c+ab+bc+ca.\] 证 \begin{align*} & ab(a+b)+bc(b+c)+ca(c+a)\geqslant a+b+c+ab+bc+ca\\ \iff & ab(a+b+c)+bc(b+c+a)+ca(c+a+b)\geqslant a+b+c+ab+bc+ca+3abc\\ \iff & (ab+bc+ca)(a+b+c)\geqslant a+b+c+ab+bc+ca+3\\ \iff & (ab+bc+ca-1)(a+b+c-1)\geqslant 4, \end{align*} 由均值,显然成立。
kuing 2# 2011-11-4 12:14
Nash 的回复竟然不见了,外面的显示还是应该多出一个回复……
nash 3# 2011-11-4 12:20
百度有时候也会吞贴 也有可能是触碰到敏感词…
thread-181-1-8.html: [不等式] 来自pep的求加强为非严格不等式
kuing 1# 2011-11-4 13:05
来自 http://bbs.pep.com.cn/thread-1965360-1-1.html 谁能把这个严格不等式加强为非严格不等式? 《数学通报》2011年4月号问题 加强为如下不等式,很平凡的 $a=b=c=1/3$ 取等,证法也很平凡。 $a,b,c\in\mathbb{R}^+$,$a+b+c=1$,则 \[\frac{a^2}{b+c^2}+\frac{b^2}{c+a^2}+\frac{c^2}{a+b^2}\geqslant \frac34.\] 证  齐次化为 \[\frac{a^2}{b(a+b+c)+c^2}+\frac{b^2}{c(a+b+c)+a^2}+\frac{c^2}{a(a+b+c)+b^2}\geqslant \frac34,\] 由柯西不等式有 \[\sum{\frac{a^2}{b(a+b+c)+c^2}}\geqslant \frac{\left( \sum{a^2} \right)^2}{\sum{\bigl(a^2b(a+b+c)+a^2c^2\bigr)}},\] 故只要证 \[4\left( \sum{a^2} \right)^2\geqslant 3\sum{\bigl(a^2b(a+b+c)+a^2c^2\bigr)},\] 展开整理为 \[3\left( \sum{a^{4}}-\sum{a^{3}b} \right)+\left( \sum{a^2} \right)^2-abc\sum{a}\geqslant 0,\] 显然成立,得证。
pxchg1200 2# 2011-11-4 16:47
1# kuing \[4\left( \sum{a^2} \right)^2\geqslant 3\sum{\bigl(a^2b(a+b+c)+a^2c^2\bigr)}, \] 其实不用展开了,kk by Vasc inequality: \[ (a^{2}+b^{2}+c^{2})^{2}\geq 3(a^{3}b+b^{3}c+c^{3}a) \] and \[ (a^{2}+b^{2}+c^{2})^{2}\geq 3(a^{2}b^{2}+b^{2}c^{2}+c^{2}a^{2})\geq 2(a^{2}b^{2}+b^{2}c^{2}+c^{2}a^{2})+(a^{2}bc+b^{2}ac+c^{2}ab) \] 第二个乘个3加上第一个得证!
pxchg1200 3# 2011-11-4 16:51
另外,这个呢? 设正数$a,b,c$ 有$a+b+c=3$ 证明: \[ \frac{a}{\sqrt{b+c^{2}}}+\frac{b}{\sqrt{c+a^{2}}}+\frac{c}{\sqrt{a+b^{2}}}\geq \frac{3}{\sqrt{2}} \] 我觉得很像那个啊。 (今年都不去Mathlink了,所以跑这里来玩。)
kuing 4# 2011-11-4 16:51
2# pxchg1200 展开其实也没多少功夫,目测即可 再说,能用简单的不等式就用简单的,说vasc不等式,那贴楼主还不知了不了解,呵呵。
天涯无际 5# 2012-2-21 20:42
本帖最后由 天涯无际 于 2012-2-21 21:07 编辑 3# pxchg1200 设正数$a,b,c$满足$a+b+c=3$,则对任意$k\geq 0$,证明 \[\frac{a}{\sqrt{kb+c^2}}+\frac{b}{\sqrt{kc+a^2}}+\frac{c}{\sqrt{ka+b^2}}\geq \frac{3}{\sqrt{k+1}}\]
thread-182-1-8.html: [不等式] kuing 来玩几个? :D
pxchg1200 1# 2011-11-4 17:21
problem 1: For $a,b,c>0$ show that: \[ \frac{(a+b+c)^{2}}{2(ab+bc+ca)}\geq \frac{a^{2}}{a^{2}+bc}+\frac{b^{2}}{b^{2}+ca}+\frac{c^{2}}{c^{2}+ab} \] Problem 2: For $a,b,c>0$ such that:$ ab+bc+ca=3 $ prove that: \[ (a+2b)(b+2c)(c+2a)\geq 8\] Problem 3: Let $a,b,c\geq 0$ with$ ab+bc+ca+abc=4 $ Show that: \[ \sqrt{a+3}+\sqrt{b+3}+\sqrt{c+3}\geq 6 \] Have fun!
pxchg1200 2# 2011-11-5 23:40
1# pxchg1200 人呢?!
kuing 3# 2011-11-5 23:42
没空啊这几天都
thread-183-1-1.html: [Chemistry]再来溶液浓度问题
Chetion 1# 2011-11-5 08:57
本帖最后由 Chetion 于 2011-11-5 08:58 编辑 教辅书上如是说: 将0.1 mol/L 的NaOH溶液100mL 与0.2mol/L的醋酸溶液100mL混合,过量的 HAc 与生成的 NaAc 的物质的量相等,电离>水解,因而 c(Ac-) >  c(Na+) > c (HAc) > c(H+) > c(OH-) 我列出了三条守恒式。 c(Na+) + c(H+) =c(Ac-) +c(OH-) 2c(Na+) = c (Ac-)+c(HAc) 2c(H+)+ c(HAc) = (Ac-)+2c(OH-) 但是没能证明c (HAc) > c(H+) 。 高手指教~
秋风树林 2# 2011-11-5 19:02
这个只能说用平衡常数最好说明了 但是你需要自己去查下相关的热力学常数和公式。。。 按你们中学的说法就是醋酸大部分未电离,电离度很小,HAc浓度当然远大于H+浓度
thread-184-1-8.html: 夹角和交角有区别么?
kuing 1# 2011-11-5 23:20
RT 交角是否有明确定义?
pxchg1200 2# 2011-11-5 23:22
1# kuing 啥是交角?! 还是第一次听说。。
kuing 3# 2011-11-5 23:23
2# pxchg1200 我记得有一个东西叫“黄赤交角”……
①②③④⑤⑥⑦ 4# 2011-11-8 11:51
天文学名词都来了。 中国的数学书,主要用的是“夹角”吧,日本可能用的是“交角” 无所谓的啦,对于没有方向性的对象,往往取不超过直角的那个,有方向的对象,取不超过平角的,除非有必要扩展角的范围,不然基本就这样了,不管旧的东西,还是新定义的,不太会“别出心裁”的,随便叫。
thread-185-1-2.html: 话说这个论坛怎么没有rating?
pxchg1200 1# 2011-11-6 14:54
本帖最后由 pxchg1200 于 2011-11-6 14:57 编辑 记得AopS都有Rate这个功能的,灌水者基本一分。。。真的很希望看见这个论坛成为中国的AOPS。。。
kuing 2# 2011-11-6 14:58
1# pxchg1200 我看看后台的评分设置如何 至于成为中国的AoPS那是不可能滴。。。
kuing 3# 2011-11-6 15:04
每个回贴的右下角有“评分”,呃,可以试试看
pxchg1200 4# 2011-11-6 15:13
3# kuing 不行啊,说什么无法提交。。。
pxchg1200 5# 2011-11-6 15:15
本帖最后由 pxchg1200 于 2011-11-6 15:17 编辑 其实成为新的AOPS很可能呐,你看,很少有网站支持Latex的,只要人够多,多开几个专区,多upload点资源。完全有可能的。。 越南都有好几个和Aops差不多的网站,如mathscope,math.vn... 都做得很不错。
kuing 6# 2011-11-6 15:55
呃,我评分好像可以,之前也评过的说。。。
海盗船长 7# 2012-1-14 19:37
我也可以评分!!
kuing 8# 2012-1-14 23:23
thread-186-1-1.html: 调节某部分行距
kuing 1# 2011-11-6 20:04
{\renewcommand\baselinestretch{行距}\selectfont 需调节部分 \par}
thread-187-1-1.html: 练习输入公式,兼请教一个问题
第一章 1# 2011-11-6 21:22
已知$f(x)$是$\mathbb{R}$上的奇函数,当$x>0$时,$f(x)={x^2}$; 若对于任意的$x\in{[t,t+2]}$,都有$f(x+t)\ge2f(x)$,求$t$的取值范围。
kuing 2# 2011-11-6 21:31
经典FAQ,见 http://bbs.pep.com.cn/thread-316461-1-1.html 此链接是我第一次看到此题时解的,之后不断见到
第一章 3# 2011-11-6 21:32
答案是$t\ge\sqrt2$, 原题是一个填空题,我的方法比较麻烦, 想问问有没有比较好的方法,同时也问问这题的背景,是不是一个高考题?
第一章 4# 2011-11-6 21:33
厉害!! 学习了
第一章 5# 2011-11-6 21:36
我的方法是 $f(x)=-x|x|$,取$x=t$代入,得出$t\ge0$,往下不用多说
kuing 6# 2011-11-6 21:51
5# 第一章 不是 $f(x)=x|x|$ 么
第一章 7# 2011-11-6 22:12
是,打错了。 多数人会忽视端点值的作用,以下是另外一题: 已知$f(x)=ax^3-3x+1$对于任意的$x\in{[-1,1]}$,总有$f(x)\ge0$成立,则$a=$
kuing 8# 2011-11-6 22:19
7# 第一章 这个之前群里有聊过,还跟三倍角公式扯了下,所以我如果没记错,$a=4$
第一章 9# 2011-11-6 22:20
呵呵。大家莫误会,这题不是来请教的,而是用以说明端点值的作用的
kuing 10# 2011-11-6 22:29
没事,我也是随便扯扯,试贴成功的话这个贴可以移去初等讨论区
第一章 11# 2011-11-6 22:31
呵呵,既然聊开了,再问问, (2010新课标全国卷第21题)设函数$f(x)=e^x-1-x-ax^2$ (1)若$a=0$,求$f(x)$的单调区间; (2)若$x\ge0$时,$f(x)\ge0$,求$a$的取值范围. 对于第(2)问,网上很多资料都说利用$e^x\ge1+x$, 但这很难吧?尽管前一小题有暗示。总觉得他们是已知答案而拼凑出过程。
第一章 12# 2011-11-6 22:40
不知道网上的答案是不是标准答案。 有人切了第2小题来问我,呵呵,有点狼狈,不过第二天就做出来了。 我用的是二阶导数。
只如初见 13# 2011-11-8 11:02

kuing 14# 2011-11-8 11:10
12# 第一章 二次求导 洛必达(野猪在网刊写过) 比较自然
thread-188-1-8.html: [函数] 求助高一函数最值问题
mytestchen 1# 2011-11-7 10:59
函数 $f(x)$ 取 $(x-a)^2,(x+a)^2,(x-2)^2$ 中较大函数的值,其中 $a$ 为非负实数,$f(x)$ 的最小值为 $g(a)$,求 $g(a)$ 的最小值。 __________________ kuing edit
kuing 2# 2011-11-7 13:16
貌似是1,画图解决
mytestchen 3# 2011-11-7 13:17
能否详细解答
kuing 4# 2011-11-7 13:25
当 $0\leqslant a\leqslant2$ 时: 当 $2<a$ 时: 由此不难看出当 $a=0$ 时函数最低点最小,为1
mytestchen 5# 2011-11-7 17:15
谢谢了!
只如初见 6# 2011-11-8 10:44
5# mytestchen 来学习。
kuing 7# 2011-11-8 10:47
6# 只如初见 welcome
thread-189-1-2.html: 积分题($f(x)=\int_1^x\frac{\sin(xt)}tdt$)
海盗船长 1# 2011-11-7 19:20
已知$f\left ( x \right ) = \int_{1}^{x} \frac{\sin\left ( x t \right )}{t} \mathrm{d} t$,求$\int_{0}^{1} x f(x) \mathrm{d} x$.
kuing 2# 2011-11-7 21:03
是否有 $\displaystyle f'(x)=\frac{\sin (x^2)}x ?$
kuing 3# 2011-11-7 22:03
错了,没那么简单
kuing 4# 2011-11-7 22:54
关键就是这个导数了,由于比较长且比较难打,就不在这里输入了,在真 $\LaTeX$ 里写了如附件所示,希望没错。
pxchg1200 5# 2011-11-7 23:00
本帖最后由 pxchg1200 于 2011-11-7 23:20 编辑 3# kuing 首先,\[A=\int_{0}^{1}{xf(x)dx}=\frac{1}{2}x^{2}f(x)|_{0}^{1}-\int_{0}^{1}{\frac{1}{2}x^{2}f'(x)dx}\] \[ f'(x)=\int_{1}^{x}{\cos{xt}dt}+\frac{\sin{x^{2}}}{x}= \frac{2\sin{x^{2}}}{x}-\frac{\sin{x}}{x}\] \[A=\frac{1}{2}(\cos{1}-1)+\frac{1}{2}\int_{0}^{1}{x\sin{x}dx}=\frac{\sin{1}-1}{2}\]
kuing 6# 2011-11-7 23:07
5# pxchg1200 第二行最后少了一个右花括号,修改下我来看看是咋样的
kuing 7# 2011-11-7 23:53
3# kuing 首先,\[A=\int_{0}^{1}{xf(x)dx}=\frac{1}{2}x^{2}f(x)|_{0}^{1}-\int_{0}^{1}{\frac{1}{2}x^{2}f'(x)dx}\] \[ f'(x)=\int_{1}^{x}{\cos{xt}dt}+\frac{\sin{x^{2}}}{x}= \frac{2\sin{x^{2}}}{x}-\frac{\sin{x}}{x}\] \[A=\frac{1}{2}(\cos{1}-1)+\frac{1}{2}\int_{0}^{1}{x\sin{x}dx}=\frac{\sin{1}-1}{2}\] pxchg1200 发表于 2011-11-7 23:00 关键还是那个导数怎么来了 你是根据什么公式得出中间的式子? 我不太了解所以我用定义式去推算。。。
kuing 8# 2011-11-8 12:06
刚才自己试着推导了一下,楼上上上帮忙看看是否正确,详细证明见附件 设二元函数 $g(x,t)$ 在 $[a,b]^2$ 上有定义,且对 $t$ 连续,对 $x$ 可导,令 \[ f(x)=\int_{a}^{x}{g(x,t)\mathrm{d}t},x\in[a,b] \] 则 \[ f'(x)=g(x,x)+\int_{a}^{x}{\frac{\partial g(x,t)}{\partial x}\mathrm{d}t}. \]
pxchg1200 9# 2011-11-8 13:00
本帖最后由 pxchg1200 于 2011-11-8 13:07 编辑 8# kuing 是这么说的: 设$f(x,y),f_{x}(x,y)$在$ R=[a,b]\times[a,b]$上连续,$c(x),d(x)$为定义在$[a,b]$上其值含于$[p,q]$内的可微函数,则函数\\ \[ F(x)=\int_{c(x)}^{d(x)}{f(x,y)dy}\] 在$[a,b]$上可微,且\\ \[ F'(x)=\int_{c(x)}^{d(x)}{f_{x}(x,y)dy}+f(x,d(x))d'(x)-f(x,c(x))c'(x) \] 你那个证明没多大问题,其实只要用复合函数求导就可以了。 把$F(x)$看作复合函数, \[ F(x)=H(x,c,d)=\int_{c}^{d}{f(x,y)dy}\] \[ c=c(x),d=d(x)\] \[ \dfrac{dF}{dx}=\frac{\partial H}{\partial x}+\frac{\partial H}{\partial c}\frac{dc}{dx}+\frac{\partial H}{\partial d}\frac{dd}{dx}=\int_{c(x)}^{d(x)}{f_{x}(x,y)dy}+f(x,d(x))d'(x)-f(x,c(x))c'(x) \]
kuing 10# 2011-11-8 13:36
9# pxchg1200 噢,还要连续可导……
pxchg1200 11# 2011-11-8 15:14
10# kuing 当然,这个条件是很强的,适当减弱是可以的,你可以去看看阿尔泽拉的理论,这方面减弱了不少。。 如果那个积分限还是无穷的话,还要考虑$f_{x}(x,y)$一致收敛的问题。。。。
kuing 12# 2011-11-8 15:16
我还是水手。。这方面
鱼儿 13# 2011-11-9 08:26
本帖最后由 鱼儿 于 2011-11-9 08:30 编辑 楼上两位弄得复杂了,这会让工科学生受不了。也许题目的正解会采用如下如下方法求$f'(x)$: 令$xt=u$,则 $f(x) =\int_{1}^{x} \frac{\sin (xt)}{t} dt=\int_{x}^{x^2} \frac{\sin u}{u} du$, 于是  $f ‘(x) =\frac{\sin (x^2)}{x^2} *2x-\frac{\sin x}{x}=\frac{2\sin (x^2)}{x} -\frac{\sin x}{x}$.
kuing 14# 2011-11-9 10:39
13# 鱼儿 唔,有道理哩
thread-19-1-9.html: [不等式] 2011第十届中国女子数学竞赛不等式加强(求简证)
kuing 1# 2011-9-26 21:06
$a,b,c,d > 0,abcd = 1$,求证 \[\frac{1}{a} + \frac{1}{b} + \frac{1}{c} + \frac{1}{d} + \frac{12}{a + b + c + d} \geqslant 7\] 求简证。
kuing 2# 2011-9-27 23:34
磨光法的证明见: http://bbs.pep.com.cn/thread-1771375-1-1.html
pxchg1200 3# 2011-10-5 22:32
Vo Quoc Ba Can好像有过证明的。。
pxchg1200 4# 2011-10-5 22:45
本帖最后由 pxchg1200 于 2011-10-5 22:46 编辑 also,we have:The positive reals $a,b,c,d $ satisfy $abcd=1$ . Prove that : \[\frac{1}{a}+\frac{1}{b}+\frac{1}{c}+\frac{1}{d}+\frac{9}{{a+b+c+d}}\geqslant\frac{{25}}{4} \] PS:Can 有Cauchy-Schwarz proof!!!
kuing 5# 2011-10-6 00:51
4# pxchg1200 这个是加强前的原题目哟 这个在上面链接里的链接里也有can的证
thread-190-1-2.html: 积分题(2)($\int_0^{+\infty}\frac{\sin x}x$)
pxchg1200 1# 2011-11-8 13:13
计算$\int_{0}^{+\infty}{\frac{\sin{x}}{x}}$
kuing 2# 2011-11-8 13:41
1# pxchg1200 漏了 dx ------ $\displaystyle \int_0^{+\infty} \frac{\sin x}x \mathrm{d}x$
pxchg1200 3# 2011-11-8 15:11
本帖最后由 pxchg1200 于 2011-11-8 15:15 编辑 2# kuing 好吧,算出来了不? (答案 $\frac{\pi}{2}$....)
kuing 4# 2011-11-8 15:23
木有……
pxchg1200 5# 2011-11-8 18:02
4# kuing Hint: 补上收敛因子$e^{-kx}$再对积分号下求导。(注意考虑一致收敛。)
kuing 6# 2011-11-8 20:57
表示不懂……
pxchg1200 7# 2011-11-8 22:10
6# kuing 考虑积分\[ I(a)=\int_{0}^{\infty}{e^{-kx}\frac{\sin{ax}}{x}dx} (a\geq 0 \] 对于$I(a)$在积分号下对$a$求导是可以的,由于$f(x)=e^{-kx}\frac{\sin{ax}}{x}$ 满足,$\int_{0}^{\infty}{\frac{\partial f}{\partial a}}$一致收敛。$f(x),\frac{\partial f}{\partial y}$连续. \[ \Rightarrow \int_{0}^{\infty}{e^{-kx}\cos{ax}dx}=\frac{k}{a^{2}+k^{2}}\] \[ \frac{dI}{da}=\frac{k}{a^{2}+k^{2}}\] 再对$a$求积分,得到: \[ I=\arctan{\frac{a}{k}} \] 令$k\rightarrow 0^{+} $ \[ I_{0}=\lim_{k\rightarrow 0^{+}}{\arctan{\frac{a}{k}}}=\frac{\pi}{2}\]
icesheep 8# 2011-11-10 15:59
本帖最后由 icesheep 于 2011-11-10 16:30 编辑 由 Fourier Core 及其对应的 Fourier 变换: \[\mathfrak{F}\left( {\frac{{\sin {\omega _0}t}}{{\pi t}}} \right) = \Pi \left( {\frac{\omega }{{{\omega _0}}}} \right)\] 所以 \[ \int\limits_{ - \infty }^{ + \infty } {\frac{{\sin {\omega _0}t\cos {\omega _0}t}}{{\pi t}}dt}  = \Pi \left( 1 \right) = \frac{1}{2}\] 下略
kuing 9# 2011-11-10 16:13
完全看不懂ing。。。 慢慢补基础。。。
pxchg1200 10# 2011-11-10 16:16
8# icesheep 终于看到传说中的第三种方法了。。
icesheep 11# 2011-11-10 16:33
10# pxchg1200 开始漏了个2倍,想想还是改积分限看起来自然一些,也不用加那个突兀的2倍了,毕竟 Fourier 变换本就是从 -inf 到 +inf 的积分。 还有一种是复围道积分吧,那个写起来太长了。。。
pxchg1200 12# 2011-11-10 17:10
11# icesheep 我看到的是用了下罗巴切夫斯基的方法,直接积分干了。。
thread-191-1-2.html: 第三帖·改头换面
戊概念·五 1# 2011-11-8 13:34
今儿个喜讯多,心情和刚放晴的天气一样的好~ 吼吼~
戊概念·五 2# 2011-11-8 13:37
本帖最后由 戊概念·五 于 2011-11-8 13:38 编辑 史迪仔
戊概念·五 3# 2011-11-9 12:01
只有主体(英文rap实在跟不上了= =): 你不乖 有时还会作怪 但你不坏 只是不装可爱 冷下来 气氛都被破坏 算你厉害 热带都变寒带 你就是天使 别怀疑 坏脾气 只是没人懂你 好运气 能和你一起 But I can't help falling in love with you 你不乖 有时还会耍赖 但你不坏 其实你很可爱 别打败 你是破坏天才 拒绝贩卖虚伪做作对白 你就是天使 别怀疑 坏脾气 只是没人懂你 好运气 能和你一起 But I can't help falling in love with you 你就是天使 别怀疑 坏脾气 只是没人懂你 好运气 能和你一起 But I can't help falling in love with you
kuing 4# 2011-11-9 12:02
那英文似乎有点出入
戊概念·五 5# 2011-11-9 12:03
4# kuing 那你CD附带的歌词是???
kuing 6# 2011-11-9 12:04
5# 戊概念·五 早就不见了
戊概念·五 7# 2011-11-9 12:07
6# kuing 不过也是,那么久了都 那不妨直接指出感觉哪里不对:
kuing 8# 2011-11-9 12:08
7# 戊概念·五 不了,随意吧。 吃饭
戊概念·五 9# 2011-11-9 12:13
8# kuing
thread-192-1-1.html: 一道作业题$\int_a^bf^2(x)dx\leq\frac{(b-a)^2}2\int_a^bf'^2(x)dx$
pxchg1200 1# 2011-11-8 21:30
设$f(x)$在$[a,b]$上具有一阶连续导数,且$f(a)=0$,证明: \[ \int_{a}^{b}{f^{2}(x)dx}\leq \frac{(b-a)^{2}}{2}\int_{a}^{b}{f'^{2}(x)dx}\]
pxchg1200 2# 2011-11-8 21:49
1# pxchg1200 hint: Cauchy-Schwarz
pxchg1200 3# 2011-11-9 15:56
kuing帮忙做做啊。。
kuing 4# 2011-11-9 15:58
你都有 hint 了,就不是“帮”了吧
pxchg1200 5# 2011-11-9 16:01
4# kuing hint是别人给的,不是我的。我没有solution啊
鱼儿 6# 2011-11-9 16:19
本帖最后由 鱼儿 于 2011-11-14 12:10 编辑 设$f(x)$在$[a,b]$上具有一阶连续导数,且$f(a)=0$,证明: \[ \int_{a}^{b}{f^{2}(x)dx}\leq \frac{(b-a)^{2}}{2}\int_{a}^{b}{f'^{2}(x)dx}\] pxchg1200 发表于 2011-11-8 21:30 证明 设$x\in[a,b]$,因为 $$f(x)= \int_{a}^{x}{f'(t)dt}+f(a)=\int_{a}^{x}{f'(t)dt}$$, 由Cauchy不等式,得 $$f^2(x)= (\int_{a}^{x}{f'(t)dt})^2 \leq \int_{a}^{x}{1^2dt}\int_{a}^{x}{f'^2(t)dt}=(x-a)\int_{a}^{x}{f'^2(t)dt}\leq (x-a)\int_{a}^{b}{f'^2(t)dt}$$, 所以 $$\int_{a}^{b}{f^2(x)dx}\leq \int_{a}^{b}{(x-a)dx}\int_{a}^{b}{f'^2(t)dt}=\frac{(b-a)^{2}}{2}\int_{a}^{b}{f'^{2}(x)dx}$$.
kuing 7# 2011-11-11 11:36
6# 鱼儿 初看了下没问题学习中
pxchg1200 8# 2011-11-11 12:44
6# 鱼儿 Nice solution!
$\TeX$ 9# 2012-2-15 16:44
8# pxchg1200 如果改成$f\left(\frac{a+b}{2}\right)=0$,就有 \[ \int_{a}^{b}{f^{2}(x)dx}\leq \frac{(b-a)^{2}}{8}\int_{a}^{b}{f'^{2}(x)dx}\]
$\TeX$ 10# 2012-2-15 16:47
How about this one? Given $f\in C^{1}[-1,1]$ with $f(0)=0$, prove that \[ \int_{-1}^{1}f'(x)^{2}\mathrm{d}x\geq\frac{3}{2}\left(\int_{-1}^{1}f(x)\mathrm{d}x\right)^{2} \]
Nirvanacs 11# 2012-2-15 19:43
9# $\TeX$ 分成 \(\displaystyle \left[a,\frac{a+b}{2}\right]\)和 \(\displaystyle \left[\frac{a+b}{2},b\right]\)两段之后再用1楼得结论.
thread-193-1-8.html: [不等式] 尽情的配方吧。 :D
pxchg1200 1# 2011-11-8 22:29
1 Let $a, \, b, \, c  \,  \in R$ . Prove that \[(a^2+b^2+c^2)^2-3abc(a+b+c) \ge 3(a+b+c)(a-b)(b-c)(c-a) \] 2.Let $a, \, b, \, c  \,  \in R$ . Prove that \[3(a^4+b^4+c^4)+4(a^3b+b^3c+c^3a) \ge 6abc(a+b+c) \] 3 Let $a, \, b, \, c  \,  \in R$ . Prove that \[(a^2+b^2+c^2)^2+2(a^3b+b^3c+c^3a) \ge 4abc(a+b+c) \] ------------- 4.Let $a, \, b, \, c  \,  \in R$ . Prove that \[(a^2+b^2+c^2)^2+2(a^3b+b^3c+c^3a) \ge 3(ab^3+bc^3+ca^3) \] ------------- 5. Let $a, \, b, \, c  \,  \in R$ . Prove that \[(a^2+b^2+c^2)^2+3(a^3b+b^3c+c^3a) \ge 6abc(a+b+c) \]
thread-194-1-2.html: UID是啥?
pxchg1200 1# 2011-11-8 23:23
看到那个头像下面有个UID,是什么意思啊?
kuing 2# 2011-11-8 23:27
注册号 一般来说按注册顺序排列,所以没特殊情况的话,看UID就可以知道是第几个注册本论坛的
pxchg1200 3# 2011-11-8 23:29
2# kuing 哦,那我是第14个了
图图 4# 2011-11-8 23:35
3# pxchg1200 第二个飘过
kuing 5# 2011-11-8 23:37
4# 图图
图图 6# 2011-11-8 23:39
5# kuing 你不是闪了吗?
kuing 7# 2011-11-9 00:01
6# 图图 我闪得慢,而且老是回来瞧瞧
图图 8# 2011-11-9 00:26
7# kuing 闪得慢,好矛盾的词...
kuing 9# 2011-11-9 00:38
嘿嘿
thread-195-1-2.html: 在Mathlink上看到很搞笑的一幕。。。
pxchg1200 1# 2011-11-8 23:37
http://www.artofproblemsolving.com/Forum/viewtopic.php?f=52&t=443447 注意看4# 和6#。。。
图图 2# 2011-11-8 23:39

pxchg1200 3# 2011-11-8 23:41
2# 图图 真是讽刺啊。。。
kuing 4# 2011-11-9 00:00
表示没看懂
pxchg1200 5# 2011-11-9 07:57
4# kuing 首先是那个Hjj贴上了他的用电脑解答的结果,然后下面的人看不顺眼了,就回了句Beautiful。。。Hjj还以为是夸他。。。
thread-196-1-1.html: 这里不可以直接帖视频?
戊概念·五 1# 2011-11-9 11:26
RT
kuing 2# 2011-11-9 11:32
不可以
戊概念·五 3# 2011-11-9 11:35
2# kuing 哦 那我自己打包.....
kuing 4# 2011-11-9 11:40
http://www.5d6d.com/thread-782113-1-1.html
戊概念·五 5# 2011-11-9 11:56
4# kuing 有劳有劳了~
kuing 6# 2011-11-9 11:59
5# 戊概念·五 汗你的表情………… PS。其实你以前看过那个贴
戊概念·五 7# 2011-11-9 12:02
6# kuing 哪个?
kuing 8# 2011-11-9 12:03
7# 戊概念·五 4#的链接
戊概念·五 9# 2011-11-9 12:05
8# kuing 我还不太正常着了,见谅了~烦请
thread-197-1-2.html: 五的第四水帖·3838
戊概念·五 1# 2011-11-9 12:15
又见此数,命哦~看来这辈子有些缘分是躲不掉了.....
kuing 2# 2011-11-9 12:52
无聊
戊概念·五 3# 2011-11-9 13:12
2# kuing 然也然也~一语中的.....
戊概念·五 4# 2011-11-9 16:27
再来无聊的帖几张为即将到来的某天拍的照
戊概念·五 5# 2011-11-9 16:29
4# 戊概念·五 那三对儿心形的铁盒子都是国庆期间参加婚礼的“证据”
戊概念·五 6# 2011-11-9 16:36
的确无聊呢~
戊概念·五 7# 2011-11-11 11:31
2011.11.11 为逝去的生灵祈祷:愿逝者安息!
kuing 8# 2011-11-11 11:33
7# 戊概念·五 一样无聊
戊概念·五 9# 2011-11-11 11:49
2011.11.11 为逝去的生灵祈祷:愿逝者安息! 戊概念·五 发表于 2011-11-11 11:31 有身在英国的同学证实其真实性:
戊概念·五 10# 2013-1-21 23:53
http://news.163.com/photoview/56 ... ;p=8LOFOPNO56I60001
戊概念·五 11# 2013-2-24 18:56
纯上图:
thread-198-1-2.html: 极限题(单增-数列极限$\to$函数极限)
图图 1# 2011-11-9 22:59
设$f(x)$在$(a,+\infty)$上单调上升,$\lim_{n\to\infty}x_n=+\infty$,若$\lim_{n\to\infty}f(x_n)=A$,求证:$\lim_{x\to+\infty}f(x)=A$.
①②③④⑤⑥⑦ 2# 2011-11-10 11:13
看上去很普通啊,玩定义而已,估计重点也就在于怎么说得滴水不漏了 $x_n$ 是趋于无穷的,肯定某一项起大于a,这样就都落在了f的单调区间里面 在这个范围里就好办了,目标 $A-\epsilon<f(x_N)\le f(x)<f(x_M)\le A$
kuing 3# 2011-11-10 11:16
看上去的确显然但就是不好说
kuing 4# 2011-11-10 11:32
我说得很麻烦: 首先, 由 $\lim_{n\to\infty}x_n=+\infty$ 可知 $\{x_n\}$ 的任意无穷子数列 $\{y_n\}$ 也满足 $\lim_{n\to\infty}y_n=+\infty$, 而在这些 $\{y_n\}$ 里也显然存在单调上升的 $\{y_n\}$, 这是因为 $\{x_n\}$ 里每一项之后总存在比其更大的项, 否则, 如果某项之后找不到更大的项, 那么该数列必有上界, 矛盾. 由于能趋向无穷, 故必能找到在区间 $(a,+\infty)$ 内的单调上升的 $\{y_n\}$, 由 $f(x)$ 单调上升, 则 $f(y_n)$ 也单调上升且 $\lim_{n\to\infty}f(y_n)=A$, 从而必有 $f(y_n)\leqslant A$ 对任意正整数 $n$ 成立. 又显然当 $x\geqslant y_1$ 时, $x$ 总在某个区间 $[y_k,y_{k+1}]$ 内, 从而由单调上升知 $f(x)$ 在区间 $[f(y_k),f(y_{k+1})]$ 内, 即存在 $y_1$ 使当 $x\geqslant y_1$ 时恒有 $f(x)\leqslant A$, 从而 $f(x)$ 在 $[y_1,+\infty)$ 内单调增且有上界 $A$, 从而 $\lim_{x\to+\infty}f(x)$ 存在, 再由 $\lim_{n\to\infty}f(x_n)=A$ 可知此极限值也只能是 $A$.
①②③④⑤⑥⑦ 5# 2011-11-10 12:58
本帖最后由 ①②③④⑤⑥⑦ 于 2011-11-10 13:12 编辑 可以啊,差不多的 对任何 $x>a$,假如 $f(x)>A$, 由于存在一个 $M$,使得 对任何的 $n>M$, $x_n>x$,由单调性,$f(x_n)>f(x)$,因而极限不会小于f(x),与题设矛盾,也就是说对任意的 $x>x_K$,$f(x)\le A$ 单调有界,后面用定理吧
kuing 6# 2011-11-10 13:18
5# ①②③④⑤⑥⑦ 唔,这样简洁很多
kuing 7# 2011-11-10 19:04
一个类似的: $\displaystyle\lim_{x\to a^-}f(x)=A$ 的充要条件是:对于任意的严格单调上升且以 $a$ 为极限的数列 $\{x_n\}$ 均有 $\displaystyle\lim_{n\to\infty}f(x_n)=A$。 PS。有个负号,是左极限 必要性:由 $\displaystyle\lim_{x\to a^-}f(x)=A$,可知对任意 $\varepsilon>0$,存在 $\delta>0$ 使得当 $0<a-x<\delta$ 时有 \[|f(x)-A|<\varepsilon,\] 设 $\{x_n\}$ 是任意的严格单调上升且以 $a$ 为极限的数列,即 $x_n<a$ 且 $\displaystyle\lim_{n\to\infty}x_n=a$,则对上面的 $\delta$,存在 $N>0$ 使得当 $n>N$ 时有 \[0<a-x_n<\delta,\] 从而 \[|f(x_n)-A|<\varepsilon,\] 这样就证明了 $\displaystyle\lim_{n\to\infty}f(x_n)=A$。 充分性:用反证法。假设对于任意的严格单调上升且以 $a$ 为极限的数列 $\{x_n\}$ 均有 $\displaystyle\lim_{n\to\infty}f(x_n)=A$,但 $\displaystyle\lim_{x\to a^-}f(x)\ne A$。 后者说明存在某个 $\varepsilon_0>0$,对任意正数 $\delta$,总存在某个点 $y$ 使得 $0<a-y<\delta$ 但 \[|f(y)-A|\geqslant\varepsilon_0,\] 我们取一列 $\delta_n=1/n,(n=1,2,\ldots)$,那么相应地就有一列 $y_n$ 满足 $0<a-y_n<1/n$ 但 \[|f(y_n)-A|\geqslant\varepsilon_0,\] 下面证明数列 $\{y_n\}$ 内必存在严格单增的无穷子数列 $\{x_n\}$,这只要证明 $\{y_n\}$ 中每一项之后都存在比其大的项。 由 $0<a-y_n<1/n$ 得 $a>y_n>a-1/n$,假设 $y_k$ 之后不存在比其大的项,即对 $n> k$ 都有 $y_n\leqslant y_k$,但实际上,取 \[n=\left[ \frac{1}{a-y_{k}} \right]+1,\] 由 $a>y_k>a-1/k$ 得 \[n\geqslant \left[ \frac1{a-\left( a-\frac1k \right)} \right]+1=k+1>k,\] 且 \[y_n>a-\frac1{\left[ \frac1{a-y_k} \right]+1}>a-\frac1{\frac1{a-y_k}}=y_k,\] 矛盾,所以 $\{y_n\}$ 内存在严格单增的无穷子数列 $\{x_n\}$。设这样的 $\{x_n\}$ 的第 $k$ 项 $x_k$ 对应于 $\{y_n\}$ 的第 $g(k)$ 项 $y_{g(k)}$,由于是子列,显然有 $k\leqslant g(k)$,这样便有 \[0<a-x_n<\frac1{g(n)}\leqslant \frac1n,\] 且 \[|f(x_n)-A|\geqslant\varepsilon_0,\] 令 $n\to\infty$,以上两式结合上述讨论表明该严格上升的 $x_n$ 以 $a$ 为极限且 $\lim_{n\to\infty}f(x_n)\ne A$,矛盾。
thread-199-1-8.html: [不等式] 轮换不等式
pxchg1200 1# 2011-11-12 15:00
Let $a,b,c\geq 0 $ prove that: \[ \frac{a^{2}}{b}+\frac{b^{2}}{c}+\frac{c^{2}}{a}\geq \sqrt{a^{2}-ab+b^{2}}+\sqrt{b^{2}-bc+c^{2}}+\sqrt{c^{2}-ca+a^{2}}\]
天涯无际 2# 2012-2-17 19:10
本帖最后由 天涯无际 于 2012-2-17 19:11 编辑 下面这两个不等式会有用: \[\frac{a^2}{b}+\frac{b^2}{c}+\frac{c^2}{a}\geq \frac{37(a^2+b^2+c^2)-19(ab+bc+ca)}{6(a+b+c)}\] \[\sqrt{a^2-ab+b^2}+\sqrt{b^2-bc+c^2}+\sqrt{c^2-ca+a^2}\leq \frac{10(a^2+b^2+c^2)-(ab+bc+ca)}{3(a+b+c)}\] 第一个没什么意思,不是pqr就是SOS...第二个也很暴力... 当然本题可能有更好的方法,懒得想了...
天涯无际 3# 2012-2-19 19:04
由$Cauchy$不等式有 \[ \begin{align*} \left(\sum_{cyc}{\sqrt{a^2-ab+b^2}}\right)^2 &=\left(\sum_{cyc}{\sqrt{\frac{a^2-ab+b^2}{b}}\cdot\sqrt{b}}\right)^2\\ &\leq \sum_{cyc}{a}\cdot\sum_{cyc}{\frac{a^2-ab+b^2}{b}}\\ &=(a+b+c)\cdot(\frac{a^2}{b}+\frac{b^2}{c}+\frac{c^2}{a}) \end{align*} \] 后面的显然.
thread-2-1-9.html: 本版块讨论初等数学范围内的问题
kuing 1# 2011-9-25 21:39
RT,其实我并不清楚初等与高等是怎么区分的,是否还有中等?:o \[e^{i\pi}+1=0\]
海盗船长 2# 2011-10-3 13:36
《中等数学》:lol
kuing 3# 2011-10-3 14:57
呃,汗…… 杂志名代表不了啥的说。。。
thread-20-1-9.html: [函数] 不等式恒成立 $\ln x>\frac1{e^x}-\frac2{ex}$
转贴专用 1# 2011-9-26 21:16
对 $x>0$ 求证 $\ln x>\dfrac1{e^x}-\dfrac2{ex}$ 转自 http://bbs.pep.com.cn/thread-1877304-1-1.html 事实上\[x\ln x\geqslant -\frac1e \geqslant \frac x{e^x}-\frac2e\]且等号不能同时取。 令 $f(x)=x\ln x$,有 $f'(x)=1+\ln x$,得 \[f(x)\geqslant f\left(\frac1e\right)=-\frac1e\] 令 $g(x)=\dfrac x{e^x}-\dfrac2e$,有 $g'(x)=\dfrac{1-x}{e^x}$,得 \[g(x)\leqslant g\left(1\right)=-\frac1e\]
isea 2# 2011-9-27 19:39
看解决过程,我突然想来了,两边同时乘x便是2011年北京东城区模拟题中的一问而已。 因为是沿题一问一问层进的,所以木感觉有多难……
kuing 3# 2011-9-27 22:10
2# isea 有N问提示这么多,那就毫无难度了……
thread-200-1-8.html: 三次根号有理化
kuing 1# 2011-11-12 16:46
$a,b,c,d,e,f\in\mathbb{Q}$ \begin{align*} \frac{a+b\sqrt[3]{3}+c\sqrt[3]{9}}{d+e\sqrt[3]{3}+f\sqrt[3]{9}}&=\frac{(a+b\sqrt[3]{3}+c\sqrt[3]{9})\bigl(d^{2}+(e\sqrt[3]{3})^{2}+(f\sqrt[3]{9})^{2}-de\sqrt[3]{3}-e\sqrt[3]{3}f\sqrt[3]{9}-f\sqrt[3]{9}d\bigr)}{(d+e\sqrt[3]{3}+f\sqrt[3]{9})\bigl(d^{2}+(e\sqrt[3]{3})^{2}+(f\sqrt[3]{9})^{2}-de\sqrt[3]{3}-e\sqrt[3]{3}f\sqrt[3]{9}-f\sqrt[3]{9}d\bigr)} \\ & =\frac{(a+b\sqrt[3]{3}+c\sqrt[3]{9})\bigl(d^{2}+(e\sqrt[3]{3})^{2}+(f\sqrt[3]{9})^{2}-de\sqrt[3]{3}-e\sqrt[3]{3}f\sqrt[3]{9}-f\sqrt[3]{9}d\bigr)}{d^{3}+(e\sqrt[3]{3})^{3}+(f\sqrt[3]{9})^{3}-3de\sqrt[3]{3}f\sqrt[3]{9}} \\ & =\frac{(a+b\sqrt[3]{3}+c\sqrt[3]{9})\bigl(d^{2}+(e\sqrt[3]{3})^{2}+(f\sqrt[3]{9})^{2}-de\sqrt[3]{3}-e\sqrt[3]{3}f\sqrt[3]{9}-f\sqrt[3]{9}d\bigr)}{d^{3}+3e^{3}+9f^{9}-9def}. \end{align*} 注意到 \[d^{2}+(e\sqrt[3]{3})^{2}+(f\sqrt[3]{9})^{2}-de\sqrt[3]{3}-e\sqrt[3]{3}f\sqrt[3]{9}-f\sqrt[3]{9}d=0\iff d=e\sqrt[3]{3}=f\sqrt[3]{9}\iff d=e=f=0,\] 所以当 $d+e\sqrt[3]{3}+f\sqrt[3]{9}\ne0$ 时必定 $d^{2}+(e\sqrt[3]{3})^{2}+(f\sqrt[3]{9})^{2}-de\sqrt[3]{3}-e\sqrt[3]{3}f\sqrt[3]{9}-f\sqrt[3]{9}d\ne0$,即以上有理化总可行。
图图 2# 2011-11-14 16:03
1# kuing 嘿嘿,记得试试继续啊
thread-201-1-1.html: 含参级数不等式
icesheep 1# 2011-11-12 21:30
p>1 \[\sum\limits_{n = 1}^\infty  {\frac{1}{{\left( {1 + n} \right)\sqrt[p]{n}}}}  < p\]
icesheep 2# 2011-11-15 17:01
\[\int\limits_{\frac{1}{{n + 1}}}^{\frac{1}{n}} {{{\left( {\frac{1}{n}} \right)}^{ - 1 + \frac{1}{p}}}dx}  < \int\limits_{\frac{1}{{n + 1}}}^{\frac{1}{n}} {{x^{ - 1 + \frac{1}{p}}}dx} \]
海盗船长 3# 2011-12-20 12:01
2# icesheep 这样放缩能做出来?
icesheep 4# 2011-12-21 00:58
3# 海盗船长 能的,你失败了?
海盗船长 5# 2011-12-22 13:06
4# icesheep 哦,知道了
thread-202-1-8.html: [数列] 看貌似简单而又复杂的数列通项式题!
wayne 1# 2011-11-13 14:03
回归课本一例题,写出“-1,1,-1,1,····”的通项公式!答案很容易得到! 可是下面这道题貌似不是那么容易吧?今天在此与大家共同探讨,学习中,多多指教!    已知数列:-1,-1,1,1,-1,-1,1,1,······。求该数列的第n项的表达式
icesheep 2# 2011-11-13 16:42
本帖最后由 icesheep 于 2011-11-13 18:59 编辑 \[\left( {\frac{{1 + i}}{2}} \right){\left( { - i} \right)^{n + 1}} + \left( {\frac{{1 - i}}{2}} \right){i^{n+1}}\] \[\cos \frac{{n + 1}}{2}\pi  + \sin \frac{{n + 1}}{2}\pi \] \[{\left( { - 1} \right)^{\left\lceil {\frac{n}{2}} \right\rceil }}\]
wayne 3# 2011-11-13 17:43
2# icesheep 好啊!可是由于时间关系我可能没有按时完成!
力工 4# 2011-11-13 18:15
1# wayne 周期数列,通项有多难,与复数结合还是可以给的。
kuing 5# 2011-11-13 22:29
叹号党
wayne 6# 2011-11-19 20:31
4# 力工 谢谢,可能见识少,所以觉得有点刺手,学习中,多多帮忙!
wayne 7# 2011-11-19 20:32
2# icesheep 非常感谢,学习中!多谢您的指教,互动、互助才让我看到不足!
wayne 8# 2011-11-19 20:33
5# kuing 多谢K哥的参与,学习中,不足之处,望您多多指教!
thread-203-1-1.html: 令人崩溃的迅雷(新问题更新在5#)
戊概念·五 1# 2011-11-13 23:05
本帖最后由 戊概念·五 于 2011-12-18 12:00 编辑 下载视频,要么找不到资源,事实上资源丰富得很;要么停止响应;要么就直接报告崩溃 求救~
戊概念·五 2# 2011-11-14 14:23
1# 戊概念·五 以后改迅崩得了= = 嘛玩意儿都.....
戊概念·五 3# 2011-11-25 21:40
每次崩后 ,桌面上都会出现附件里的俩玩意
戊概念·五 4# 2011-12-18 11:59
3# 戊概念·五 偶然成为迅雷会员后,类似情况就没有再出现了,当是解决了吧= =
戊概念·五 5# 2011-12-18 12:00
4# 戊概念·五 虽然会员身份仅维持了三天,但问题却是没有再出现了,嗯。 新问题的是:打开迅雷后,流量监测显示一直有50kb/S左右的上传速度,不知这迅雷是在上传些嘛玩意呢?!
kuing 6# 2011-12-18 12:14
4# 戊概念·五 免费 vip?怎么弄的
戊概念·五 7# 2011-12-19 12:22
6# kuing 就是每月2号有的一个活动,没记错的话是二号,嗯。下月有活动时我再来通告你吧~ Ps:你怎么知道我是免费VIP?!
pxchg1200 8# 2011-12-19 15:56
迅雷,32级,Vip6 飘过。。。
kuing 9# 2011-12-19 20:41
6# kuing Ps:你怎么知道我是免费VIP?! 戊概念·五 发表于 2011-12-19 12:22 你以为我像你这么笨啊?
戊概念·五 10# 2011-12-20 16:16
迅雷,32级,Vip6 飘过。。。 pxchg1200 发表于 2011-12-19 15:56 有钱银
戊概念·五 11# 2011-12-20 16:18
你以为我像你这么笨啊? kuing 发表于 2011-12-19 20:41 这么明显的事实知道就行了,还说出来= =真锐利 不过说回来了,我5#的问题k兄可否解答呢?!
戊概念·五 12# 2011-12-22 09:15
这么明显的事实知道就行了,还说出来= =真锐利 不过说回来了,我5#的问题k兄可否解答呢?! 戊概念·五 发表于 2011-12-20 16:18 看样子又成悬题了.....木人理啊木人理
戊概念·五 13# 2011-12-22 09:18
今天登录迅雷,领取了迅雷快传的体验资格,看起来还不错:70GB的空间,据说是可以无死链分享的,嗯。 观望中.....
kuing 14# 2011-12-22 10:16
6# kuing 就是每月2号有的一个活动,没记错的话是二号,嗯。下月有活动时我再来通告你吧~ 戊概念·五 发表于 2011-12-19 12:22 不用通知了,刚才已经弄了免费三天vip了 不过其实我根本没什么要下载
戊概念·五 15# 2011-12-22 11:31
14# kuing 你抽中的?! 我倒是觉着开了会员还不如不开来的速度= =
kuing 16# 2011-12-22 14:46
高速通道对于一些没什么资源下载很慢的东西来说还是有用的 以后看来可以找 pxchg1200 帮下载一些。
kuing 17# 2011-12-29 11:08
又有三天vip,这次是抽的
戊概念·五 18# 2012-1-10 19:04
又有三天vip,这次是抽的 kuing 发表于 2011-12-29 11:08 k兄好运气哦~ 真嫉妒呢还.....
thread-204-1-2.html: 大了一点点都传不上,还是要处理了下才行
kuing 1# 2011-11-14 00:31
libertango1.wma (474.77 KB)
thread-205-1-2.html: 得意忘形忘乎所以乐极生悲也……
kuing 1# 2011-11-14 00:37
01234……
kuing 2# 2011-11-14 11:12
还以为顺水推舟,谁知……
图图 3# 2011-11-14 11:58
又没人看得懂
戊概念·五 4# 2011-11-25 21:43
又没人看得懂 图图 发表于 2011-11-14 11:58 原来老大也会说别人看不懂的话
thread-207-1-2.html: 高次方程实根分布问题
icesheep 1# 2011-11-14 07:57
证明:\[{x^n} + {a_{n - 1}}{x^{n - 1}} + ... + {a_1}x + {a_0} = 0\] 全部实根都介于  \[ - \frac{{{a_{n - 1}}}}{n} \pm \frac{{n - 1}}{n}\sqrt {a_{n - 1}^2 - \frac{{2n}}{{n - 1}}{a_{n - 2}}} \]  之间。 据说是用 Cauchy-Schwartz 不等式,没看出来。。
kuing 2# 2011-11-14 11:09
我在 Proofs From The Book 里看到过,的确 CS,还有韦达
kuing 3# 2011-11-14 11:11

pxchg1200 4# 2011-11-14 12:56
2# kuing 艾尔多斯,天书的证明?!
kuing 5# 2011-11-14 12:58
4# pxchg1200
kuing 6# 2011-11-14 13:00
不知道什么时候下的,忘了。。。 比较大,这里传不上
kuing 7# 2011-11-14 13:32
sina 里面还有一个第四版,不过是英文的 http://ishare.iask.sina.com.cn/f/18446958.html
icesheep 8# 2011-11-14 14:15
本帖最后由 icesheep 于 2011-11-14 16:29 编辑 靠原来那家伙漏条件了。。。得是所有根都是实根才能往下做。 实用意义骤减。。。
thread-208-1-2.html: 这个图有意思
kuing 1# 2011-11-14 13:18

戊概念·五 2# 2011-11-14 14:21
1# kuing 少打了一个字
图图 3# 2011-11-27 19:33

戊概念·五 4# 2011-12-16 17:59
要很幸福哦~
内牛满面 5# 2012-7-17 12:43
额。。。。
戊概念·五 6# 2012-8-17 21:24
凑个热闹,上图图: 话说如此下去,女士们该如何是好呢~
戊概念·五 7# 2012-8-18 15:27
6# 戊概念·五 ——现在的好男人,都有男朋友了 ——那坏男人呢? ——坏男人是好男人的男朋友。。。。。
戊概念·五 8# 2012-8-18 15:30
这腰身,估计我一巴掌就能给拍折了。。。。。
戊概念·五 9# 2012-9-17 00:17
此所谓本性难移= =
戊概念·五 10# 2012-9-17 00:18

kuing 11# 2012-9-17 00:25
9# 戊概念·五 为什么只删 yi ben dao 的?难道只下 yi ben dao 的?
戊概念·五 12# 2012-9-17 23:13
11# kuing 表示完全不了解内情,仅仅是看到,而后想起此人在校时的诨号了,而已、、、、、
thread-209-1-8.html: [不等式] CIQ这么有聊?
kuing 1# 2011-11-14 16:23
注意到 \[1+\frac{16}{k^2}+\frac{16}{(k+4)^2}=\left(1+\frac4k-\frac4{k+4}\right)^2,\] 故当 $k>0$ 时,显然就有 \[\sqrt{1+\frac{16}{k^2}+\frac{16}{(k+4)^2}}=1+\frac4k-\frac4{k+4},\] 求和即得 \[\sum_{k=1}^{n}\sqrt{1+\frac{16}{k^2}+\frac{16}{(k+4)^2}}=n-\frac4{n+1}-\frac4{n+2}-\frac4{n+3}-\frac4{n+4}+\frac{25}3,\] 而由 $n\geqslant 1$ 知 \begin{align*} n-\frac4{n+1}-\frac4{n+2}-\frac4{n+3}-\frac4{n+4}+\frac{25}3 &\geqslant n-\frac4{1+1}-\frac4{1+2}-\frac4{1+3}-\frac4{1+4}+\frac{25}3\\ &=n+\frac{16}5>n+3>n+\frac{25}{12}, \end{align*} 故原不等式反向成立。
骨灰级爱好者 2# 2011-11-14 16:59
你轻松搞定啊。我还在挖空心思变换构造呢。
icesheep 3# 2011-11-14 17:37
原来能把和求出来 - -
kuing 4# 2011-11-14 17:44
嗯,故此便有聊……这个“猜测”实在是。。。
pxchg1200 5# 2011-11-14 21:12
4# kuing kk终于霸气登场了。。。
kuing 6# 2011-11-14 21:33
5# pxchg1200 李明在群里叫到我玩所以才扯扯。。。
pxchg1200 7# 2011-11-15 16:07
6# kuing 那个第三题在几年前就被Can做了。。。
kuing 8# 2011-11-15 16:32
7# pxchg1200 那个abc=1的分式不等式?
pxchg1200 9# 2011-11-15 16:43
8# kuing 嗯,就是那个。主要是对$ \sum{\frac{1}{a^{2}+a+1}}$ 这东东的使用。。
kuing 10# 2011-11-15 16:51
嗯,我在不等式群里看到你发了,有空再研究下一般指数
pxchg1200 11# 2011-11-16 22:44
本帖最后由 pxchg1200 于 2011-11-16 22:46 编辑 10# kuing 好吧,现在来扯下那个分式不等式: problem Let $a,b,c>0,abc=1 $ prove that: \[ \dfrac{a^{2}}{a^{3}+2}+\dfrac{b^{2}}{b^{3}+2}+\dfrac{c^{2}}{c^{3}+2}\leq 1 \] proof(Vo Quoc Ba Can) note $ a=\frac{1}{x},b=\frac{1}{y},c=\frac{1}{z}$ we can rewrite our inequality into: \[ \frac{x}{2x^{3}+1}+ \frac{y}{2y^{3}+1}+ \frac{z}{2z^{3}+1}\leq 1 \] noctice that: \[ \frac{2x}{2x^{3}+1}\leq \frac{x^{2}+1}{x^{4}+x^{2}+1}\] Which is \[ x^{2}+1\geq 2x \] so we obtain the well-know result \[  \frac{x^{2}+1}{x^{4}+x^{2}+1}\geq 1 \] Done! 发现很多人问这个 \[ \frac{2x}{2x^{3}+1}\leq \frac{x^{2}+1}{x^{4}+x^{2}+1}\] 是怎么来的?开始我也表示很惊讶。 Can 给出样的解释: 首先我们的原则是用强的不等式去证明弱的不等式,我注意到: $abc=1,a,b,c>0$ \[ \frac{1}{a^{2}+a+1}+ \frac{1}{b^{2}+b+1}+ \frac{1}{c^{2}+c+1}\geq 1 \] 是个很强的不等式(由Vasc发现),尽管用CS很容易证(但这并不代表它很弱!!) 作变换:$  a=\frac{1}{x},b=\frac{1}{y},c=\frac{1}{z}$ 得到: \[ \frac{a^{2}}{a^{2}+a+1}+ \frac{b^{2}}{b^{2}+b+1}+ \frac{c^{2}}{c^{2}+c+1}\geq 1 \] 另外:$abc=1,a,b,c>0$ \[  \frac{1}{a^{2}-a+1}+ \frac{1}{b^{2}-b+1}+ \frac{1}{c^{2}-c+1}\leq 3 \] 也是个很强的不等式,和上面那个配对使用有时能达到很好的效果。 对比分母: \[ (a^{4}+a^{2}+1)=(a^{2}-a+1)(a^{2}+a+1) \] \[ \frac{2(a^{2}+1)}{a^{4}+a^{2}+1}=\frac{1}{a^{2}+a+1}+\frac{1}{a^{2}-a+1} \] 由第一个不等式有 \[ \sum{ \frac{2(a^{2}+1)}{a^{4}+a^{2}+1}}\leq 4 \] 而\[ \sum{\frac{1}{a^{2}+a+1}}\geq 1 \] 故 \[  \frac{1}{a^{2}-a+1}+ \frac{1}{b^{2}-b+1}+ \frac{1}{c^{2}-c+1}\leq 3 \] 作为另一个使用例子,我们来看看下面的不等式: $abc=1,a,b,c>0$  prove that: \[ \frac{1}{\sqrt{4a^{2}+a+4}}+ \frac{1}{\sqrt{4b^{2}+b+4}}+ \frac{1}{\sqrt{4c^{2}+c+4}}\leq 1 \] proof: 显然,要想一下破根号并不是件容易的事,于是我们试下另外的方法去摆脱根号。 回想下著名的Nesbitt inequality: \[ \dfrac{a}{b+c}+\dfrac{b}{a+c}+\dfrac{c}{a+b}\geq \frac{3}{2} \] 我们先正规化$a+b+c=3 $ \[ \Rightarrow \frac{a}{3-a}+ \frac{b}{3-b}+ \frac{c}{3-c}\geq \frac{3}{2}\] 应用切线法求得估计: \[ \dfrac{x}{3-x}\geq \frac{1}{2}+\frac{3}{4}(x-1) \] 我们可以很快证明Nesbitt inequality。 现在,把第一个不等式加载进切线法: \[ \frac{1}{\sqrt{4x^{2}+x+4}}\leq \frac{1}{3}-k(\frac{1}{x^{2m}+x^{m}+1}-\frac{1}{3}) \] 我们要找到合适的$k,m$ 记\[ f(x)=\frac{1}{\sqrt{4x^{2}+x+4}}-\frac{1}{3}+k(\frac{1}{x^{2m}+x^{m}+1}-\frac{1}{3}) \] 求导 \[ f'(x)=-\frac{8x+1}{2(4x^{2}+x+4)^{\frac{3}{2}}}-\frac{k(2mx^{2m-1}+mx^{m-1})}{(x^{2m}+x^{m}+1)^{2}}\] 令$x=1 $ 我们有$ f'(1)=-\frac{1}{6}-\frac{km}{3}$ 因此,为了让$ f'(1)=0$ 和$k>0$ 我们要看看$ k=\frac{-1}{2m},m<0$ \[ \frac{1}{\sqrt{4x^{2}+x+4}}\leq \frac{1}{3}+\frac{1}{2m}(\frac{1}{x^{2m}+x^{m}+1}-\frac{1}{3}) \] 再令$m=-1$ \[\frac{1}{\sqrt{4x^{2}+x+4}}\leq \frac{x+1}{2(x^{2}+x+1)}\] 事实上有 \[ (x+1)^{2}(4x^{2}+x+4)-4(x^{2}+x+1)^{2}=x(x-1)^{2}\] 接下来就是显然了。 Done! 可见,这种方式非常犀利。为我们破根号又提供了种方法。
kuing 12# 2011-11-16 22:52
有空研读下
pxchg1200 13# 2011-11-16 23:38
12# kuing 另外,Can说这个也可以用上面的方法做: \[\sqrt{\frac{2a}{a+b}}+\sqrt{\frac{2b}{b+c}}+\sqrt{\frac{2c}{c+a}}\leq 3 \] 呵呵。。。
鱼儿 14# 2011-11-17 10:32
现在的CIQ,不等式小组成员谁都可以去提,审查也欠严格,有点烂了,没有必要当回事。
thread-21-1-9.html: [函数] 抽象函数选择题 $f(x^y)=yf(x)$
转贴专用 1# 2011-9-27 00:03
设函数 $f(x)$ 在其定义域 $(0,+\infty)$ 上的取值不恒为 0,且 $x>0,y\in\mathbb{R}$ 时,恒有 $f(x^y)=yf(x)$。$a>b>c>1$ 且 $a,b,c$ 成等差数列,则 $f(a)f(c)$ 与 $\bigl(f(b)\bigr)^2$ 的大小关系为 转自 http://bbs.pep.com.cn/thread-1877525-1-1.html 令 $y=0$,则得到 $f(1)=0$,由不恒为 0,即存在 $k\ne1$ 使得 $f(k)=t\ne0$,此时作置换 $x\to k$,$y\to\log_kx$,代入得到\[f(x)=t\cdot\log_kx,\]所以可以确定 $f(x)$ 是对数函数,下略。
thread-210-1-8.html: [不等式] 比较大小
张平 1# 2011-11-14 20:08
在区间(0,π/2)内试比较tan(sinx)与sin(tanx)的大小,并证明你的结论。
icesheep 2# 2011-11-16 15:06
\[\tan \left( {\sin x} \right) = x + \frac{{{x^3}}}{6} - \frac{{{x^5}}}{{40}} - \frac{{107{x^7}}}{{5040}} + o\left( {{x^8}} \right)\] \[\sin \left( {\tan x} \right) = x + \frac{{{x^3}}}{6} - \frac{{{x^5}}}{{40}} - \frac{{55{x^7}}}{{1008}} + o\left( {{x^8}} \right)\]
张平 3# 2011-11-16 21:42
贴答案吧 ___________kuing edit in $\LaTeX$___________ 解  设$f(x)=\tan(\sin x)-\sin(\tan x)$,则 \[f'(x)=\sec^2(\sin x)\cos x-\cos(\tan x)\sec^2x=\frac{\cos^3x-\cos(\tan x)\cos^2(\sin x)}{\cos^2(\sin x)\cos^2x}.\] 当$0<x<\arctan(\pi/2)$时,$0<\tan x<\pi/2$,$0<\sin x<\pi/2$。由余弦函数在$(0,\pi/2)$上的凸性有 \[\sqrt[3]{\cos(\tan x)\cos^2(\sin x)}\leqslant \frac13\bigl(\cos(\tan x)+2\cos(\sin x)\bigr)\leqslant \cos\frac{\tan x+2\sin x}{3}.\] 设$\varphi(x)=\tan x+2\sin x-3x$,则 \[\varphi'(x)=\sec^2x+2\cos x-3=\tan^2x-4\sin^2\frac x2>0,\] 于是 \[\tan x+2\sin x>3x,\] 所以 \[\cos\frac{\tan x+2\sin x}3<\cos x,\] 即 \[\cos(\tan x)\cos^2(\sin x)<\cos^3x.\] 于是,当$x\in(0,\arctan(\pi/2)$时,$f'(x)>0$,又$f(0)=0$,所以$f(x)>0$;当$x\in[\arctan(\pi/2),\pi/2)$时,$\sin\bigl(\arctan(\pi/2)\bigr)<\sin x<1$,由于 \[\sin\left(\arctan\frac\pi2\right)=\frac{\tan\bigl(\arctan\frac\pi2\bigr)}{\sqrt{1+\tan^2\bigl(\arctan\frac\pi2\bigr)}} =\frac{\frac\pi2}{\sqrt{1+\frac{\pi^2}4}}=\frac{\pi}{\sqrt{4+\pi^2}}>\frac\pi4,\] 故$\pi/4<\sin x<1$,于是$1<\tan(\sin x)<\tan1$,所以当$x\in[\arctan(\pi/2),\pi/2)$时,$f(x)>0$。 综上可得,当$x\in(0,\pi/2)$时,$\tan(\sin x)>\sin(\tan x)$。
kuing 4# 2011-11-16 21:48
这么叉难……
icesheep 5# 2011-11-16 22:36
严格地写肯定是麻烦的,直观来看就是2楼那样。
wayne 6# 2011-11-19 21:03
学习了,谢谢!
随便112 7# 2011-11-21 08:20
3# 张平 这个是第18届北京大学生数学竞赛题压轴题
realnumber 8# 2011-12-15 13:45
本帖最后由 realnumber 于 2011-12-15 13:58 编辑 那么难啊,那提供的标答是怎么样解答的? 贴个图,两曲线果然贴得很近.
海盗船长 9# 2012-1-12 10:55
本帖最后由 海盗船长 于 2012-1-12 15:24 编辑 \[ \cos(\cos(x))>\sin(\sin(x)) \] \[\cos(\sin(x))>\sin(\cos(x)) \]
海盗船长 10# 2012-1-12 17:04
thread-211-1-2.html: 发不了视频只能给链接
kuing 1# 2011-11-14 22:15
http://v.youku.com/v_show/id_XMTI2ODM5ODAw.html
pxchg1200 2# 2011-11-16 23:31
1# kuing 哟,kk也听周杰伦?!
戊概念·五 3# 2011-11-20 21:04
Jay的钢琴技艺确实挺赞的
thread-212-1-5.html: 纯粹记录网址-反三角函数
kuing 1# 2011-11-15 11:41
http://zh.wikipedia.org/wiki/%E5 ... 2%E5%87%BD%E6%95%B0
wayne 2# 2011-11-19 21:10
学习了,谢谢!收获不少!
icesheep 3# 2011-11-19 22:59
2# wayne 你是数学研发论坛的 wayne ?
kuing 4# 2012-12-5 16:14
链接里只有公式没有推导,闲来无事,来试试推推先。 设 $\arccos x=t$,则 $\cos t=x$,故 $\sin^2t=1-x^2$,由于 $\arccos x$ 的值域为 $[0,\pi]$,所以 $\sin t$ 一定是非负的,因此可以两边开方得 $\sin t=\sqrt{1-x^2}$。如果 $x\in[0,1]$,则 $t\in[0,\pi/2]$,此时得 $t=\arcsin\sqrt{1-x^2}$;如果 $x\in[-1,0)$,则 $t\in(\pi/2,\pi]$,此时得 $t=\pi-\arcsin\sqrt{1-x^2}$。所以得到公式 \[\arccos x=\begin{cases} \arcsin\sqrt{1-x^2},&x\in[0,1],\\ \pi-\arcsin\sqrt{1-x^2},&x\in[-1,0). \end{cases} \] 设 $\arcsin x=t$,则 $\sin t=x$,故 $\cos^2t=1-x^2$,由于 $\arcsin x$ 的值域为 $[-\pi/2,\pi/2]$,所以 $\cos t$ 一定是非负的,因此可以两边开方得 $\cos t=\sqrt{1-x^2}$。如果 $x\in[0,1]$,则 $t\in[0,\pi/2]$,此时得 $t=\arccos\sqrt{1-x^2}$;如果 $x\in[-1,0)$,则 $t\in[-\pi/2,0)$,此时得 $t=-\arccos\sqrt{1-x^2}$。所以得到公式 \[\arcsin x=\begin{cases} \arccos\sqrt{1-x^2},&x\in[0,1],\\ -\arccos\sqrt{1-x^2},&x\in[-1,0). \end{cases} \] 由上述两公式也就得到恒等式 \[ \sin(\arccos x)=\cos(\arcsin x)=\sqrt{1-x^2}. \]
kuing 5# 2012-12-5 17:06
由上述恒等式,我们有 \[\sin(\arcsin x + \arcsin y) = x \cos(\arcsin y) + y \cos(\arcsin x) = x\sqrt{1-y^2}+y\sqrt{1-x^2},\] 因为 $\arcsin x$ 的值域是 $[-\pi/2,\pi/2]$,所以 $\arcsin x + \arcsin y$ 的取值范围是 $[-\pi,\pi]$,下面分类讨论。 如果 $\abs{\arcsin x + \arcsin y}\leqslant \pi/2$,则 \[\arcsin x + \arcsin y = \arcsin\left(x\sqrt{1-y^2}+y\sqrt{1-x^2}\right);\] 如果 $\arcsin x + \arcsin y > \pi/2$,则 \[\arcsin x + \arcsin y = \pi - \arcsin\left(x\sqrt{1-y^2}+y\sqrt{1-x^2}\right);\] 如果 $\arcsin x + \arcsin y < - \pi/2$,则 \[\arcsin x + \arcsin y = - \pi - \arcsin\left(x\sqrt{1-y^2}+y\sqrt{1-x^2}\right).\] 理论上这样已经足够给出 $\arcsin x + \arcsin y$ 的公式,但是在判断是哪段的时候那个比较大小并不方便操作,有必要再化简一下。 考虑不等式 $\arcsin x + \arcsin y > \pi/2$,由于 $\arcsin x$ 的值域是 $[-\pi/2,\pi/2]$,所以要使不等式成立,$\arcsin x$ 和 $\arcsin y$ 必须都是正的,亦即 $x$, $y$ 都是正数,故此有 \begin{align*} \arcsin x + \arcsin y > \frac\pi2 &\iff \arcsin x > \frac\pi2-\arcsin y \\ &\iff x > \sin\left(\frac\pi2-\arcsin y\right)=\cos(\arcsin y)=\sqrt{1-y^2}\\ &\iff x^2+y^2>1, \end{align*} 同理可证,当 $\arcsin x + \arcsin y < - \pi/2$ 等价于 $x$, $y$ 都是负数且 $x^2+y^2>1$。 故此 $\abs{\arcsin x + \arcsin y}\leqslant \pi/2$ 就是除这两种情况外的情况,即等价于 $xy\leqslant0$ 或 $x^2+y^2\leqslant1$。 综上即得公式 \[\arcsin x + \arcsin y=\begin{cases} \arcsin\left(x\sqrt{1-y^2}+y\sqrt{1-x^2}\right),& xy\leqslant0\vee x^2+y^2\leqslant1,\\ \pi - \arcsin\left(x\sqrt{1-y^2}+y\sqrt{1-x^2}\right),& x>0\wedge y>0\wedge x^2+y^2>1,\\ - \pi - \arcsin\left(x\sqrt{1-y^2}+y\sqrt{1-x^2}\right),& x<0\wedge y<0\wedge x^2+y^2>1. \end{cases}\] 将 $y$ 变成 $-y$ 即得差的公式 \[\arcsin x - \arcsin y=\begin{cases} \arcsin\left(x\sqrt{1-y^2}-y\sqrt{1-x^2}\right),& xy\geqslant0\vee x^2+y^2\leqslant1,\\ \pi - \arcsin\left(x\sqrt{1-y^2}-y\sqrt{1-x^2}\right),& x>0\wedge y<0\wedge x^2+y^2>1,\\ - \pi - \arcsin\left(x\sqrt{1-y^2}-y\sqrt{1-x^2}\right),& x<0\wedge y>0\wedge x^2+y^2>1. \end{cases}\] wiki 那里没标明后面的或和且,最后那条式还写错了。
kuing 6# 2012-12-5 19:21
用类似的方法来推导 $\arccos$ 的和差公式,比 $\arcsin$ 的要简单。 \[\cos(\arccos x - \arccos y) = xy + \sin(\arccos x)\sin(\arccos y) = xy+\sqrt{(1-x^2)(1-y^2)},\] 因为 $\arccos x$ 的值域是 $[0,\pi]$,所以 $\arccos x - \arccos y$ 的取值范围是 $[-\pi,\pi]$,下面分类讨论。 如果 $\arccos x - \arccos y\geqslant 0$,则 \[\arccos x - \arccos y = \arccos\left(xy+\sqrt{(1-x^2)(1-y^2)}\right);\] 如果 $\arccos x - \arccos y<0$,则 \[\arccos x - \arccos y = -\arccos\left(xy+\sqrt{(1-x^2)(1-y^2)}\right).\] 注意到 $\arccos x$ 是单调递减函数,故 $\arccos x - \arccos y\geqslant(<) 0 \iff x\leqslant(>) y$,所以得到公式 \[\arccos x - \arccos y=\begin{cases} \arccos\left(xy+\sqrt{(1-x^2)(1-y^2)}\right),& x\leqslant y,\\ -\arccos\left(xy+\sqrt{(1-x^2)(1-y^2)}\right),& x>y. \end{cases}\] 将 $y$ 变成 $-y$,注意到 $\arccos(-y)=\pi-\arccos y$,得到和的公式 \[\arccos x + \arccos y=\begin{cases} \pi+\arccos\left(-xy+\sqrt{(1-x^2)(1-y^2)}\right),& x+y\leqslant0,\\ \pi-\arccos\left(-xy+\sqrt{(1-x^2)(1-y^2)}\right),& x+y>0. \end{cases}\] 这跟 wiki 中的公式是等价的。
kuing 7# 2012-12-5 20:49
来玩玩 $\arctan$ 的。 设 $\arctan x=t$,则 $t\in(-\pi/2,\pi/2)$ 且 $\tan t=x$,于是由 $x^2=\sin^2t/(1-\sin^2t)$ 解得 $\sin^2t=x^2/(1+x^2)$。如果 $x\geqslant0$,则 $t\in[0,\pi/2)$,故 $\sin t=\sqrt{x^2/(1+x^2)}=x/\sqrt{1+x^2}$;如果 $x<0$,则 $t\in(-\pi/2,0)$,故 $\sin t=-\sqrt{x^2/(1+x^2)}=x/\sqrt{1+x^2}$。所以总有 $\sin t=x/\sqrt{1+x^2}$,再由 $t\in(-\pi/2,\pi/2)$ 即得公式 \[\arctan x=\arcsin\frac x{\sqrt{1+x^2}}.\] 和差就比上面的简单了,因为两角和差正切公式出来的东东还是正切,不像正余弦那么混合,但由于定义域问题,分的类又要多一些。 因为 $\arctan x$ 的值域是 $(-\pi/2,\pi/2)$,所以 $\arctan x+\arctan y$ 的取值范围是 $(-\pi,\pi)$,下面分类讨论。 当 $\abs{\arctan x+\arctan y}\ne\pi/2$ 时,有 \[\tan(\arctan x+\arctan y)=\frac{x+y}{1-xy},\] 如果 $\abs{\arctan x+\arctan y}<\pi/2$,则 \[\arctan x+\arctan y=\arctan\frac{x+y}{1-xy};\] 如果 $\arctan x+\arctan y>\pi/2$,则 \[\arctan x+\arctan y=\arctan\frac{x+y}{1-xy}+\pi;\] 如果 $\arctan x+\arctan y<-\pi/2$,则 \[\arctan x+\arctan y=\arctan\frac{x+y}{1-xy}-\pi.\] 考虑不等式 $\arctan x+\arctan y>\pi/2$,由于 $\arctan x$ 的值域是 $(-\pi/2,\pi/2)$,所以要使不等式成立,$\arctan x$ 和 $\arctan y$ 都必须是正的,亦即 $x$, $y$ 都是正数,故此 \begin{align*} \arctan x+\arctan y>\frac\pi2 &\iff \arctan x>\frac\pi2-\arctan y \\ &\iff x>\tan\left(\frac\pi2-\arctan y\right)=\cot(\arctan y)=\frac1y\\ &\iff xy>1, \end{align*} 同理可证,$\arctan x+\arctan y<-\pi/2$ 等价于 $x$, $y<0$ 且 $xy>1$;$\arctan x+\arctan y=\pi/2$ 等价于 $x$, $y>0$ 且 $xy=1$;$\arctan x+\arctan y=-\pi/2$ 等价于 $x$, $y<0$ 且 $xy=1$。 而 $\abs{\arctan x+\arctan y}<\pi/2$ 就是除以上情况外的情况,即等价于 $xy<1$。 综上即得公式 \[\arctan x + \arctan y=\begin{cases} \frac\pi2,&x>0\wedge y>0\wedge xy=1,\\ -\frac\pi2,&x<0\wedge y<0\wedge xy=1,\\ \arctan\frac{x+y}{1-xy},& xy<1,\\ \arctan\frac{x+y}{1-xy}+\pi,& x>0\wedge y>0\wedge xy>1,\\ \arctan\frac{x+y}{1-xy}-\pi,& x<0\wedge y<0\wedge xy>1. \end{cases}\] 将 $y$ 变成 $-y$,即得差的 \[\arctan x - \arctan y=\begin{cases} \frac\pi2,&x>0\wedge y<0\wedge xy=-1,\\ -\frac\pi2,&x<0\wedge y>0\wedge xy=-1,\\ \arctan\frac{x-y}{1+xy},& xy>-1,\\ \arctan\frac{x-y}{1+xy}+\pi,& x>0\wedge y<0\wedge xy<-1,\\ \arctan\frac{x-y}{1+xy}-\pi,& x<0\wedge y>0\wedge xy<-1. \end{cases}\]
kuing 8# 2012-12-5 22:56
话说……怎么没有 $\arcsin(x+y)$ 的展开什么的?
都市侠影 9# 2012-12-7 20:36
话说……怎么没有 $\arcsin(x+y)$ 的展开什么的? kuing 发表于 2012-12-5 22:56 看嘛,那些公式都不过是把已有的三角公式换个写法,就类似于指数的乘法与对数加法,其实一个鸟样,但 $\arctan{(x+y)}$ 这个似乎就没有对应的三角公式
thread-213-1-1.html: [转]TeX与Word, 从排版软件到中国学术 (shalex)
kuing 1# 2011-11-15 14:12
转于:http://bbs.cnool.net/topic_show. ... 494&flag=topic1 鉴于怕触动敏感词,就不将内容贴于此了,详情看链接吧。
①②③④⑤⑥⑦ 2# 2011-11-17 13:59
说了也没用,我不知道什么学府拒绝TeX,估计一流学府是不太会的。 其实吧,Word没那么差,TeX也没那么好,还是看应用领域。科技论文,尤其是数学公式之类的排版,确实是Word的软肋,不过如果海报之类的也要TeX(当然Word也不怎么样,不过相应来说方便一点),恐怕很多时候是在耽误功夫。 要求所有数学资料都用TeX也不现实,规模不大的时候,Word也还是可以胜任的,公式编辑器也不是什么烂到家的东西,作者自己也说,可以调得好一点的。至于图省事不考虑美观性,也许作者心里根本没有这种美观性,或者不care。没有追求,用TeX照样可以把公式做烂。 其实说白了,不就是字体、大小、位置的调整吗? TeX预先编好程序了(但还是有很多时候要手工慢慢调的,写一点看一下效果是常有的事,TeX也不是背出了命令就完事的),对Word来说,公式编辑器、EQ域、以及2007版之后的公式环境(暂不考虑第三方软件),仅从公式角度来说,和TeX相比,如何?公式编辑器输入不便(对于擅长代码的),新版公式环境对于数学字符的字体被钉死; EQ域是个不错的东西,也是代码式,Word本身就可以以0.5磅为单位调整字体大小和字符位置,结合样式,配上好的字体(我不知道目前怎样配备才能媲美TeX,但理论上可以),定出Small,Large以及上标下标等等很轻松,而且可以一动全文整体调动,然后用EQ域来进一步控制字符的上下偏移(1磅为单位,结合Word的字体属性才能做到0.5磅为单位,所以以字体属性为主),用EQ域及Unicode编码的各种宽度空格(实际依赖字体)来控制水平位置,事情就结了。精细的水平位置的控制,考验美学功力,不管是TeX还是Word。 剩下还有,编号问题,Word的多级编号有时不太好用,个人觉得必须链接样式才好控制,也就用在标题上,其他的,通过Word的各种域,如Listnum、SEQ,配合REF、StyleREF等使用,不太复杂的情况都能搞定。 进一步的话就是模板,也就是那些Small、Large之类的样式,各种预定义的编号列表什么的,弄成模板以便多次调用。 这样,当然不可能完全追上TeX,但是基本搞掂了,输入、搜索、调整都还可以,虽然只能0.5磅为单位,但基本够用了,只不过嘛,如果真的能够做到这一层,TeX早能玩得顺溜了,干嘛折腾Word的弱项,整出来了,真会用Word写长篇数学文章吗?
icesheep 3# 2011-11-17 23:43
要我说这东西迟早被所见即所得淘汰掉的。 完全可以开发一款软件保留 Tex 的优势而做成所见即所得的样子,甚至做得比 Word 还 Word。
pxchg1200 4# 2012-1-24 12:52
哟,又是陈计么。他也不说说他怎么配方的,其实这算啥秘密啊,一看就知道是电脑配的。
thread-214-1-1.html: chinatex的FTP
kuing 1# 2011-11-15 14:51
http://ftp.chinatex.org/ 资料书籍等在 http://ftp.chinatex.org/document/ 随便看了……
thread-215-1-1.html: [转]视频教程-定理环境
kuing 1# 2011-11-15 15:47
http://blog.sina.com.cn/s/blog_5e16f1770102dx36.html
icesheep 2# 2011-11-24 02:53
视频中在作者输入begin{nt}后,他输入了一个指令,一下将 end{nt}给输出了,请问他如何操作的?
kuing 3# 2011-11-24 10:14
2# icesheep http://bbs.pep.com.cn/thread-1731319-1-1.html 1.1
thread-216-1-8.html: [几何] 几何题一个
pxchg1200 1# 2011-11-15 16:11
在四边形$ABCD$中$AB=CD$ ,取$BC$和$AD$ 的中点作直线。并延长$BA,CD$ 交直线于$ E,F$。证明:$ \angle{ADH}=\angle{HFD}$
kuing 2# 2011-11-15 16:37
跟这个是一样的:http://sq.k12.com.cn/discuz/thread-439355-1-1.html
pxchg1200 3# 2011-11-15 16:41
2# kuing 表示木有号,无法看到具体题目和解答。。。
kuing 4# 2011-11-15 16:50
RT
pxchg1200 5# 2011-11-16 12:49
4# kuing 懂了,犀利!
thread-217-1-1.html: 一道有趣的代数题
icesheep 1# 2011-11-18 05:52
a,b 是环 R 中的元素,若 1-ab 可逆,则 1-ba 可逆。 【备注:题和解答都没涉及多少概念,只需注意环中乘法不满足交换律】 【提示:虽然是代数题,不妨用分析的眼光来看】
pxchg1200 2# 2011-11-18 12:50
1# icesheep 个人觉得环R要有单位元e. proof: 设$c=(e-ab)^{-1}$,则 \[ e-ba=e-bc(e-ab)^{-1}a=e-bca+bcaba=e-bca(e-ba). \] \[ (e+bca)(e-ba)=e. \] 又有 \[ (e-ba)(e+bca)=e-ba+bca-babca=e-ba+b(e-ab)ca=e \] 所以$e-ba$可逆 \[ (e-ba)^{-1}=e+bca=e+b(e-ab)^{-1}a \]
icesheep 3# 2011-11-19 15:38
本帖最后由 icesheep 于 2011-11-19 15:45 编辑 题目中出现1了当然说明有乘法单位元啦 关键是 (1+bca) 是怎么猜出来它就是 (1-ba) 的逆元的,一种猜测方法就是如下: \begin{align} \begin{array}{l} \frac{1}{{1 - ba}}& = 1 + ba + baba + bababa + ... \\ & = 1 + b\left( {1 + ab + abab + ...} \right)a \\ & = 1 + b\frac{1}{{1 - ab}}a \\ & = 1 + b{\left( {1 - ab} \right)^{ - 1}}a \\ \end{array} \end{align}
秋风树林 4# 2011-11-19 20:07
3# icesheep 哈哈。。。我们高代老师也是教我们这么猜的。。。
thread-218-1-1.html: 一道积分证明题
pxchg1200 1# 2011-11-18 12:56
设$f(x):[0,+\infty)\to[0,+\infty) $是一个连续函数。且对任意的$x\in[0,+\infty)$满足$ f(f(x))=x^{2} $ 证明: \[ \int_{0}^{1}{f(x)dx}\leq \frac{1}{2} \] (proposed by Mateescu Constantin )
icesheep 2# 2011-11-19 17:25
本帖最后由 icesheep 于 2011-11-19 22:28 编辑 先证单调性,然后应该就好做了 如果一个连续函数非递增也非递减,那么一定存在 $a \ne b$ 使得$f\left( a \right) = f\left( b \right) = L$,于是 ${a^2} = f\left( {f\left( a \right)} \right) = f\left( L \right) = f\left( {f\left( b \right)} \right) = {b^2}$ 与 $a \ne b$ 矛盾! 又若函数递减,则函数有界,而 x^2 无界,故函数递增。
icesheep 3# 2011-11-19 22:38
本帖最后由 icesheep 于 2011-11-19 23:33 编辑 只需证函数不在 [0,1] 的任何子区间上大于 x 即可 (反证法) 若 $\exists \left( {a,b} \right) \subseteq \left[ {0,1} \right]$ 使 $f\left( x \right) > x,\forall x \in \left( {a,b} \right)$ 则对任意的 ${x_0} \in \left( {a,b} \right)$ 或者有 $0 \le f\left( x_0 \right) \le a$ 或者有 $f\left( x_0 \right) \ge b$ 否则 $x_0^2 = f\left( {f\left( {{x_0}} \right)} \right) > f\left( {{x_0}} \right) > {x_0}$ 矛盾! 若 $f\left( {{x_0}} \right) \ge b > {x_0}$ ,则 $x_0^2 = f\left( {f\left( {{x_0}} \right)} \right) < {x_0} < f\left( {{x_0}} \right)$ 和单调性矛盾; 若 $f\left( {{x_0}} \right) \le a < {x_0}$,则和假设矛盾。 故函数不在[0,1]的任何子区间上大于 x
thread-219-1-1.html: 又到求极限时间
kuing 1# 2011-11-19 19:55
学生-小愚(4600*****)    当x趋近于无穷时,求x((1+1/x)^x-e) 洛啊洛: \begin{align*} \lim_{x\to\infty}x\left(\left(1+\frac1x\right)^x-e\right)&=\lim_{t\to0}\frac{(1+t)^{\frac1t}-e}t\\ &=\lim_{t\to0}\left((1+t)^{\frac1t}\right)'\\ &=\lim_{t\to0}\frac{(1+t)^{\frac1t}\bigl(t-(1+t)\ln(1+t)\bigr)}{t^2(1+t)}\\ &=\lim_{t\to0}(1+t)^{\frac1t}\cdot\lim_{t\to0}\frac{\bigl(t-(1+t)\ln(1+t)\bigr)'}{\bigl(t^2(1+t)\bigr)'}\\ &=e\cdot\lim_{t\to0}\frac{-\ln(1+t)}{t(2+3t)}\\ &=-e\cdot\lim_{t\to0}\frac{\bigl(\ln(1+t)\bigr)'}{\bigl(t(2+3t)\bigr)'}\\ &=-e\cdot\lim_{t\to0}\frac1{2(1+t)(1+3t)}\\ &=-\frac e2. \end{align*} 能不能不洛?
鱼儿 2# 2011-11-24 08:05
本帖最后由 鱼儿 于 2011-11-24 08:23 编辑 换个说法而已。 令$x=\frac{1}{t}$,记$f(t)=(1+t)^{\frac{1}{t}}$,则 $$f(0)=\lim_{t\to0}(1+t)^{\frac{1}{t}}=e$$, 利用导数定义,得 $$\lim_{x\to\infty}x((1+\frac{1}{x})^x-e)=\lim_{t\to0}\frac{(1+t)^{\frac{1}{t}} -e}{t}=\lim_{t\to0}\frac{f(t)-f(0)}{t-0}=f'(0)=\cdots$$
战巡 3# 2012-1-7 21:57
本帖最后由 战巡 于 2012-1-7 22:00 编辑 泰勒展开咯..... $$(1+t)^{\frac{1}{t}}$$在原点展开得到 $$e-\frac{et}{2}+o(t^3)$$ $$\lim_{t\to0}\frac{(1+t)^{\frac{1}{t}}-e}{t}=\lim_{t\to0}\frac{-\frac{et}{2}+o(t)}{t}$$ $$=-\frac{e}{2}$$
kuing 4# 2012-1-7 23:02
泰勒展开咯..... $$(1+t)^{\frac{1}{t}}$$在原点展开得到 $$e-\frac{et}{2}+o(t^3)$$ $$\lim_{t\to0}\frac{(1+t)^{\frac{1}{t}}-e}{t}=\lim_{t\to0}\frac{-\frac{et}{2}+o(t)}{t}$$ $$=-\frac{e}{2}$$ 战巡 发表于 2012-1-7 21:57 第二个式应该是 o(t) 吧,不过话说回来,$(1+t)^{\frac1t}$ 在原点展开是怎么展开的?怎样操作?
海盗船长 5# 2012-1-14 21:03
本帖最后由 海盗船长 于 2012-1-14 21:10 编辑 4# kuing $\displaystyle (1+t)^{\frac{1}{t}}=\exp{\left( \frac{\ln(1+t)}{t} \right)}=\exp{\left(1-\frac{t}{2}+o(t)\right)}=\mathrm{e}\left( 1-\frac{t}{2}+o(t) \right)$
kuing 6# 2012-1-14 23:17
5# 海盗船长 噢,如此……
thread-22-1-1.html: 今天论坛不稳定
kuing 1# 2011-9-27 14:49
老是 502 bad 什么的,去5d6d官网看过也有人反映同样的问题,估计是5d6d官方问题,与本论坛的设置无关,故只能静等官方维修了……
kuing 2# 2011-9-28 15:42
今天似乎好了……
海盗船长 3# 2011-10-3 13:39
公式怎么不能显示了。。
kuing 4# 2011-10-3 14:57
我这正常 要么刷新一下?
kuing 5# 2011-10-3 17:35
mathjax 真的卡住了。。。 这个网也显示不来 http://jaxedit.googlecode.com/svn/trunk/jaxedit.html 去 mathjax 官网看看先,希望不是**。。。
kuing 6# 2011-10-3 17:42
:( mathjax 不响应了 http://www.mathjax.org/ mathjax 也不稳定了,汗,希望不会挂掉 不会是在cn才这样吧
thread-220-1-14.html: 三角形的什么心?
realnumber 1# 2011-11-20 07:29
本帖最后由 realnumber 于 2011-11-20 07:39 编辑 已知:P为三角形ABC所在平面上一点,满足$a\overrightarrow{PA}+b\overrightarrow{PB}+c\overrightarrow{PC}=\vec{0}$,其中$a=│BC│,b=│AC│,c=│AB│$ 则P是三角形的____心. 考试猜答案的话,一定是猜内心了。重心向量前的系数都是1,而垂心、外心都可能落到三角形外,那么和不会是$\vec{0}$
图图 2# 2011-11-20 09:56
本帖最后由 图图 于 2011-11-20 10:28 编辑 \begin{align*} &a\overrightarrow{PA}+b\overrightarrow{PB}+c\overrightarrow{PC}=\vec{0}\\\\ \Longleftrightarrow &a\overrightarrow{PA}+b(\overrightarrow{PA}+\overrightarrow{AB})+c(\overrightarrow{PA}+\overrightarrow{AC})=\vec{0} \end{align*} \begin{align*} \Longrightarrow \overrightarrow{AO}&=\frac {b}{a+b+c}\overrightarrow{AB}+\frac {c}{a+b+c}\overrightarrow{AC}\\\\\\ &=\dfrac{\overrightarrow{AB}}{\dfrac{a+b+c}{b}}+\dfrac{\overrightarrow{AC}}{\dfrac{a+b+c}{c}}\\\\\\ &=\dfrac{\overrightarrow{AB}}{\dfrac{a+b+c}{bc}c}+\dfrac{\overrightarrow{AC}}{\dfrac{a+b+c}{bc}b}\\\\\\ &=\frac{\overrightarrow{AB}}{kc}+\frac{\overrightarrow{AC}}{kb}~~~~(let \frac{a+b+c}{bc}=k)\\\\\\ &=\frac1k\left(\frac{\overrightarrow{AB}}{|\overrightarrow{AB}|}+\frac{\overrightarrow{AC}}{|\overrightarrow{AC}|}\right) \end{align*} 所以AP过$\triangle$ABC的内心. 同理,BP,CP过$\triangle$ABC的内心. 所以P是$\triangle$ABC的内心.
realnumber 3# 2011-11-20 19:39
谢了,图图
thread-220-1-8.html: 三角形的什么心?
realnumber 1# 2011-11-20 07:29
本帖最后由 realnumber 于 2011-11-20 07:39 编辑 已知:P为三角形ABC所在平面上一点,满足$a\overrightarrow{PA}+b\overrightarrow{PB}+c\overrightarrow{PC}=\vec{0}$,其中$a=│BC│,b=│AC│,c=│AB│$ 则P是三角形的____心. 考试猜答案的话,一定是猜内心了。重心向量前的系数都是1,而垂心、外心都可能落到三角形外,那么和不会是$\vec{0}$
图图 2# 2011-11-20 09:56
本帖最后由 图图 于 2011-11-20 10:28 编辑 \begin{align*} &a\overrightarrow{PA}+b\overrightarrow{PB}+c\overrightarrow{PC}=\vec{0}\\\\ \Longleftrightarrow &a\overrightarrow{PA}+b(\overrightarrow{PA}+\overrightarrow{AB})+c(\overrightarrow{PA}+\overrightarrow{AC})=\vec{0} \end{align*} \begin{align*} \Longrightarrow \overrightarrow{AO}&=\frac {b}{a+b+c}\overrightarrow{AB}+\frac {c}{a+b+c}\overrightarrow{AC}\\\\\\ &=\dfrac{\overrightarrow{AB}}{\dfrac{a+b+c}{b}}+\dfrac{\overrightarrow{AC}}{\dfrac{a+b+c}{c}}\\\\\\ &=\dfrac{\overrightarrow{AB}}{\dfrac{a+b+c}{bc}c}+\dfrac{\overrightarrow{AC}}{\dfrac{a+b+c}{bc}b}\\\\\\ &=\frac{\overrightarrow{AB}}{kc}+\frac{\overrightarrow{AC}}{kb}~~~~(let \frac{a+b+c}{bc}=k)\\\\\\ &=\frac1k\left(\frac{\overrightarrow{AB}}{|\overrightarrow{AB}|}+\frac{\overrightarrow{AC}}{|\overrightarrow{AC}|}\right) \end{align*} 所以AP过$\triangle$ABC的内心. 同理,BP,CP过$\triangle$ABC的内心. 所以P是$\triangle$ABC的内心.
realnumber 3# 2011-11-20 19:39
谢了,图图
thread-221-1-8.html: [几何] 直线平分两三角形面积的问题
icesheep 1# 2011-11-20 18:31
平面上任给两个三角形,是否存在一条直线,刚好同时平分这两个三角形的面积。
realnumber 2# 2011-11-20 19:52
感觉上存在,而且有无数条,以下只是直观类比,不是证明,平行移动一直线(比如在2三角形同一侧),总可以使得平分一三角形A时,分另一三角形B的比为p(p>1);适当旋转一角度,重新从同一侧平移,平分一三角形A时,分另一三角形B的比为q(q<1);
kuing 3# 2011-11-20 23:04
先考虑一个问题:对于平面上任意给定的三角形,所有平分其面积的直线是否一定覆盖整个平面?
realnumber 4# 2011-11-21 07:27
过平面上一点,应该存在唯一一条平分该三角形,就让直线绕着这点(先假设这点在三角形外)旋转,从开始与三角形有公共点到与三角形没有公共点 ,分三角形的面积比q是(0,+∞),那么应该有个q=1.
thread-222-1-8.html: [不等式] 群里看到的一个不等式
realnumber 1# 2011-11-20 22:51
本帖最后由 realnumber 于 2011-11-20 22:55 编辑 忘了原题在讨论什么,但是印象深刻的一个步骤是$\frac{x^{2}+1}{x^4+x^2+1} \ge \frac{2x}{2x^3+1}$ 改动下,意思就是$\frac{x^2+1}{x^{n+2}+x^{n}+1}  \ge \frac{2x}{2x^{n+1}+1}$,$n \ge 0$,为整数 $\frac{x^4+1}{x^{n+2}+x^{n}+1}  \ge \frac{2x^2}{2x^{n+1}+1}$,$n \ge 0$,为整数 $\frac{x^{2k}+1}{x^{n+2k}+x^{n}+1}  \ge \frac{2x^{k}}{2x^{n+k}+1}$,$n \ge 0$,为整数
kuing 2# 2011-11-20 22:52
原题是 http://kkkkuingggg.5d6d.com/thread-209-2-1.html 里的11楼那个吧
thread-223-1-4.html: [不等式] 不等式放缩
随便112 1# 2011-11-21 08:08
本帖最后由 随便112 于 2011-11-21 08:13 编辑 求证:$\displaystyle \sum_{k=1}^{n} \dfrac{C_{n}^{k}}{1+3^k}\leqslant\dfrac{4^n}{2^n+3^n}$
随便112 2# 2011-11-21 09:07
哦  好像把分母1丢掉就可以了
yes94 3# 2013-2-3 22:26
哦  好像把分母1丢掉就可以了 随便112 发表于 2011-11-21 09:07 什么丢掉了?
realnumber 4# 2013-2-3 22:48
3# yes94 他的意思就是放缩下,把1去掉 得到\[(1+\frac{1}{3})^n-1<\frac{4^n}{2^n+3^n}\] 这个可以用分析法证明.
yes94 5# 2013-2-3 23:07
4# realnumber 哦,那就是变成二项式定理的逆用?,
realnumber 6# 2013-2-3 23:21
5# yes94 是啊,逆用---也听到类似说法,觉得定理本来就是等式,没方向的.
thread-224-1-8.html: [不等式] 不等式求最小值
随便112 1# 2011-11-21 08:18
本帖最后由 随便112 于 2011-11-21 17:59 编辑 若$a,b,c>0,a+b+c=ab+bc+ac$ 求$\dfrac{1}{1+a}+\dfrac{1}{1+b}+\dfrac{1}{1+c}$的最小值
icesheep 2# 2011-11-21 16:51
本帖最后由 icesheep 于 2011-11-25 02:37 编辑 目测不存在啊,取 $a = b = \frac{1}{2} + \varepsilon $ 则 $\mathop {\lim }\limits_{\varepsilon  \to {0^ + }} c =+\infty $ 于是 \[\frac{1}{{1 + a}} + \frac{1}{{1 + b}} + \frac{1}{{1 + c}} \to \frac{4}{3}\]
thread-225-1-8.html: [不等式] 不等式
随便112 1# 2011-11-21 09:13
我们得到不等式链:$tan(tanx)>tan(sinx)>sin(tanx)>sin(sinx)$,是因为$ tanx>sinx$,$x$是锐角 我想这个也是成立的,利用级数得到,请问有什么初等方法呢 $arcsin(arcsinx)>arctan(arcsinx)>arcsin(arctanx)>arctan(arctanx)$,其中$0<x<\dfrac{\pi}{4}$
icesheep 2# 2011-11-21 14:54
超越不等式你求初等方法。。。
realnumber 3# 2011-12-15 13:29
用你提供的不等式,转化下就得到反三角的不等式了(想法来自--类比课本指数3条运算性质,得到对数运算性质,就用定义转换) 第1个:α=arcsinp>β=arctanp等价于p=sinβ/cosβ=sinα>sinβ,则α>β.完 第2个:x=sintanα=tansinβ>sintanβ,所以α>β.完第三个应该也一样吧.不试了
thread-226-1-8.html: [几何] 四面体海伦公式
kuing 1# 2011-11-23 18:40
翻贴偶然看到的
①②③④⑤⑥⑦ 2# 2011-11-24 09:11
1# kuing 唔,肯定不是海伦发现的(海伦公式本身可能也不是海伦发现的),这里用到了角,为什么把它套上海伦的名字呢?
kuing 3# 2011-11-24 10:10
2# ①②③④⑤⑥⑦ 只是因为那个根式跟三角形的海伦公式有点像所以…… 其实应该加个双引号
thread-227-1-1.html: 三个无穷级数的敛散性判别
icesheep 1# 2011-11-24 22:25
\[\sum\limits_{n = 1}^\infty  {\frac{1}{n}\cos \left( {\frac{\pi }{2}\ln n} \right)} \] \[\sum\limits_{n = 1}^\infty  {\frac{1}{{{n^{2 - \sin n}}}}} \] \[\sum\limits_{n = 1}^\infty  {\frac{1}{{{n^a}}}\left( {\begin{array}{*{20}{c}}   {a + n - 1} \\   n \end{array}} \right)} \]
海盗船长 2# 2011-12-9 19:34
第二题求解。
icesheep 3# 2011-12-10 02:05
2# 海盗船长 我也做不来。。。
icesheep 4# 2011-12-10 19:15
今天有人上贴吧问了,收获在此: serie20cosntheta_1281833634.pdf (120.33 KB)
海盗船长 5# 2011-12-13 12:56
是我问的
战巡 6# 2012-1-7 19:37
第二的话,收敛是肯定的,n取整数的时候sin(n)是取不到1的,肯定小于1
海盗船长 7# 2012-1-7 23:07
6# 战巡 那篇文章说第二个是发散的。。
kuing 8# 2012-1-7 23:09
7# 海盗船长 虽然是小于1,但可以趋向1 是不是这个原因
海盗船长 9# 2012-1-7 23:10
8# kuing 应该是的
海盗船长 10# 2012-1-14 14:58
本帖最后由 海盗船长 于 2012-1-14 21:22 编辑 (3)\[ \sum_{n=1}^{\infty} \frac{1}{n^p} {{n+p-1}\choose{n}} \qquad (p > 0)\]    $\displaystyle \frac{a_{n}}{a_{n+1}}=\left( \frac{n+1}{n} \right)^p \frac{n+1}{n+p}$ $\displaystyle \frac{a_{n}}{a_{n+1}}-1-\frac{1}{n}=\frac{\left( 1+\frac{1}{n} \right)^{p+1}-\left(1+\frac{p}{n}\right)}{1+ \frac{p}{n}}-\frac{1}{n}$ $\displaystyle =\frac{1+\frac{1}{n}}{1+\frac{p}{n}}\left(\left(1+\frac{1}{n}\right)^p-\left(1+ \frac{p}{n}\right)\right)$ $\displaystyle =\frac{p(p-1)}{2n^2}+o\left( \frac{1}{n^2} \right)$ 由Gauss判别法知该级数发散!
海盗船长 11# 2012-1-14 16:26
本帖最后由 海盗船长 于 2012-1-14 16:56 编辑 (1)\[ \sum_{n = 1}^{\infty}  {\frac{1}{n} \cos {\left( {\frac{\pi }{2}\ln n} \right)}} \] 取$\displaystyle N_1=\left[ \exp{\left( 2\left( 2k-\frac{1}{3} \right) \right)} \right ]$  $\displaystyle N_2=\left[ \exp{\left( 2\left( 2k+\frac{1}{3} \right) \right)} \right ]$ $(k \in \mathrm{Z^{+}})$, 则当$n \in \{N_1+1,N_1+2, \cdots ,N_2\}$时,有$\displaystyle \cos{\left( \frac{\pi}{2} \ln{n} \right)}>\frac{1}{2}$ 故 $\displaystyle \left| \sum_{n=N_1+1}^{N_2}{\frac{1}{n} \cos {\left( {\frac{\pi }{2}\ln n} \right)}}  \right| > \frac{ \exp {\left(\frac{4}{3}\right)} - 1} {2 \exp{ \left(\frac{4}{3}\right)}} $ 由柯西收敛准则知该级数发散
thread-228-1-8.html: [不等式] 貌似很难的。
pxchg1200 1# 2011-11-25 10:26
Let$a,b,c\geq 0 ,a+b+c=3$ prove that: \[ \frac{ab}{b^{3}+1}+ \frac{bc}{c^{3}+1}+ \frac{ca}{a^{3}+1}\leq \frac{3}{2}\]
pxchg1200 2# 2011-12-6 22:05
1# pxchg1200 First,we build a lemma: lemma: When $x \in (0,3) $ we have: \[ \frac{36}{x^{3}+1}\leq 16x^{2}-59x+61 \] proof of the lemma: \[ \Leftrightarrow (16x^{3}-27x^{2}-9x+25)(x-1)^{2}\geq 0 \] We can easy see that it true,when $x \in (0,3) $. Then,Let us back to the original inequality \[ \frac{ab}{b^{3}+1}+\frac{bc}{c^{3}+1}+\frac{ca}{a^{3}+1}\leq \frac{3}{2}\] Using the lemma: we get \[ \sum{ab(16b^{2}-59b+61)}\leq 54 \] and use our condition $a+b+c=3$ to Homogeneous. after many compute it gives: \[ 6\sum{a^{4}}-4\sum{a^{3}c}-37\sum{a^{3}b}+91\sum{a^{2}b^{2}}-56\sum{a^{2}bc}\geq 0 \] Here,we can use Vo Quoc Ba Can's sum of square skills.we only need to check the following things \begin{equation} m>0 \end{equation} \begin{equation} 3m(m+n)\geq p^{2}+pg+g^{2} \end{equation} When $ m=6, n=91 ,p=-4,g=-37 $ we can find that $ 3m(m+n)=1746>1533=p^{2}+pg+g^{2} $ Hence we are done! \blacksquare
kuing 3# 2011-12-8 20:08
有点暴力 ……
pxchg1200 4# 2011-12-9 21:33
3# kuing 没办法了,那个lemma还是Can说的。另外的证明就是Mix variable了。那个更暴力。
thread-229-1-1.html: 关于参考书的求助
pan135 1# 2011-11-25 18:09
现在大一,在学一元分析学(微积分)和线性代数,各位哥哥姐姐能不能帮忙推荐几本教参,最好基础题与拔高题合理搭配的,先谢谢了!
icesheep 2# 2011-11-25 23:51
数学分析中的典型问题与方法 (裴礼文)
thread-23-1-9.html: [不等式] 常见放缩成等比 $1/(2^{k+1}-1)\leqslant1/(3\cdot2^{k-1})$
kuing 1# 2011-9-27 15:51
求证 \[\frac1{2^2-1}+\frac1{2^3-1}+\frac1{2^4-1}+\cdots+\frac1{2^{n+1}-1}<\frac23\quad (n\in\mathbb{N}^+)\] 由\[\frac1{2^{k + 1} - 1}\leqslant\frac1{3\cdot2^{k - 1}} \iff\frac{2^{k-1} - 1}{3 \cdot 2^{k - 1}\left(2^{k + 1} - 1\right)}\geqslant0 \]对任意 $k\geqslant1$ 成立,得 \begin{align} \frac1{2^2-1}+\frac1{2^3-1}+\frac1{2^4-1}+\cdots+\frac1{2^{n+1}-1} &\leqslant\frac13\left(1+\frac12+\frac14+\cdots+\frac1{2^{n-1}}\right)\\ &=\frac13\left(2-\frac1{2^{n-1}}\right)\\ &<\frac23 \end{align}
thread-230-1-1.html: 群论小题
icesheep 1# 2011-11-27 00:35
G 是一个有限群,f 是 G 上的一个自同构,且 $f \circ f = I$ ,若 f 除了 1 之外没有不动点,证明:$f\left( x \right) = {x^{ - 1}}$
icesheep 2# 2011-11-27 17:57
还是给个 Hint,但是我不知道这个 Hint 怎么想出来的: 考虑 G 中任意一个元素都可以写成 yf(y^(-1)) 。
thread-231-1-1.html: 不定积分$\int 1/(1+x^n) \mathrm{d} x$能不能求出递推公式?
海盗船长 1# 2011-11-27 17:06
本帖最后由 海盗船长 于 2012-1-14 17:09 编辑 RT
icesheep 2# 2011-11-27 18:49
感觉不行
icesheep 3# 2012-1-23 16:41
瞎逛看到一个定积分的式子: \[\int\limits_0^\infty  {\frac{{dx}}{{1 + {x^n}}}}  = \frac{\pi }{n}\csc \left( {\frac{\pi }{n}} \right)\]
Nirvanacs 4# 2012-1-23 18:48
3# icesheep 用留数做.数分方法可参考谢惠明数学分析习题课讲义上册最后一章的参考题
战巡 5# 2012-1-23 19:30
本帖最后由 战巡 于 2012-1-23 20:18 编辑 瞎逛看到一个定积分的式子: \[\int\limits_0^\infty  {\frac{{dx}}{{1 + {x^n}}}}  = \frac{\pi }{n}\csc \left( {\frac{\pi }{n}} \right)\] icesheep 发表于 2012-1-23 16:41 做代换$t=x^n$,得到 $$\displaystyle\int_{0}^{\infty}\frac{1}{1+x^n}dx=\frac{1}{n}\int_{0}^{\infty}\frac{t^{\frac{1}{n}-1}}{1+t}dt$$ 再代换$y=\frac{1}{t}-1$,原式变为 $$\displaystyle\frac{1}{n}\int_{0}^{\infty}\frac{t^{\frac{1}{n}-1}}{1+t}dt=\frac{1}{n}\int_{0}^{1}(1-y)^{\frac{1}{n}-1}y^{-\frac{1}{n}}dy=\frac{1}{n}B(\frac{1}{n},1-\frac{1}{n})$$ 然后由余元公式就得到那个东西了
thread-232-1-1.html: 高数、求导问题
myconger 1# 2011-11-27 22:22
讨论函数y=x|x|在x=0处的可导性
kuing 2# 2011-11-27 22:30
$y=f(x)=x|x|$。当 $x\geqslant0$ 时 $f(x)=x^2$,当 $x<0$ 时 $f(x)=-x^2$,故 \begin{align*} \lim_{x\to0^+}&=\frac{f(x)-f(0)}{x-0}=\frac{x^2}{x}=\lim_{x\to0^+}x=0,\\ \lim_{x\to0^-}&=\frac{f(x)-f(0)}{x-0}=\frac{-x^2}{x}=\lim_{x\to0^-}-x=0, \end{align*} 故 $f'(0)=0$
thread-233-1-8.html: [不等式] 轮换不等式,$a,b,c$
pxchg1200 1# 2011-11-27 23:26
Let $a,b,c >0$ prove that: \[ \frac{a^{3}+bc^{2}}{a^{3}+a^{2}c}+\frac{b^{3}+ca^{2}}{b^{3}+b^{2}a}+\frac{c^{3}+b^{2}a}{c^{3}+c^{2}b}\geq 3 \]
天涯无际 2# 2012-2-12 20:09
本帖最后由 天涯无际 于 2012-2-12 20:12 编辑 由$Cauchy$不等式有:$(a^3+bc^2)(a+b)\geq (a^2+bc)^2$. 所以只需证明 \[\sum_{cyc}{\frac{(a^2+bc)^2}{a^2(a+b)(a+c)}}\geq 3\] 再由$Cauchy$不等式可得 \[\sum_{cyc}{\frac{(a^2+bc)^2}{a^2(a+b)(a+c)}}\geq \frac{(a+b+c+\frac{bc}{a}+\frac{ca}{b}+\frac{ab}{c})^2}{(a+b)(a+c)+(b+c)(b+a)+(c+a)(c+b)}\] 又有$\frac{bc}{a}+\frac{ca}{b}+\frac{ab}{c}\geq a+b+c$, 代入则只需证明 $4(a+b+c)^2\geq 3[(a+b)(a+c)+(b+c)(b+a)+(c+a)(c+b)]$ 展开可知显然成立.
kuing 3# 2012-2-12 20:11
又见楼上,嘿嘿 PS:今天的mathjax公式好像显示不出来,不知啥事情
天涯无际 4# 2012-2-12 20:12
额,闲着没事做,随便写写。。。。。。
kuing 5# 2012-2-17 18:53
今天我这显示也正常了,nice one 啊
thread-234-1-8.html: [不等式] just CS it,(1)
pxchg1200 1# 2011-11-27 23:30
1.Let $a,b,c$ be real numbers.prove that: \[ (a^{2}+3)(b^{2}+3)(c^{2}+3)\geq 4(a+b+c+1)^{2}\] 2.for postive numbers $ x_{1},x_{2},\cdots,x_{n}$ we have \[ x_{1}+x_{2}+\cdots+x_{n}=n \] prove that: \[ \frac{1}{x_{1}^{2}-x_{1}+n}+\frac{1}{x_{2}^{2}-x_{2}+n}+\cdots+\frac{1}{x_{n}^{2}-x_{n}+n}\leq 1 \] Have fun!
pxchg1200 2# 2011-11-28 12:45
咦? 这两个应该不难的啊,怎么没人做?!
pxchg1200 3# 2011-12-1 14:58
Solution 1 : WLOG we can assume that : $ (a^{2}-1)(b^{2}-1)\geq 0 $ Therefore,$ (a^{2}+3)(b^{2}+3)=(a^{2}-1)(b^{2}-1)+4(a^{2}+b^{2}+2) $ by CS $ (a^{2}+b^{2}+1+1)(1+1+c^{2}+1)\geq (a+b+c+1)^{2} $ Done! Solution 2: by CS \[ (x_{1}^{2}+(n-x_{1}))(1+(n-x_{1}))\geq (x_{1}+(n-x_{1}))^{2}=n^{2} \] Therefore: \[ \frac{1}{x_{1}^{2}-x_{1}+n}\leq\frac{n+1-x_{1}}{n^{2}} \] sum it up,the result follows
thread-235-1-1.html: 定积分:$\int\limits_0^{+\infty}{\sin{x^2}dx}$
icesheep 1# 2011-11-28 02:42
求各种方法计算:\[\int\limits_0^{ + \infty } {\sin {x^2}dx} \]
pxchg1200 2# 2011-12-9 21:32
kk有方法计算么? 我表示压力啊。
kuing 3# 2011-12-9 21:53
有的话也回贴了………… 我高等的还很菜……
icesheep 4# 2011-12-10 03:48
本帖最后由 icesheep 于 2011-12-10 03:54 编辑 \[\int\limits_0^\infty  {\cos {x^2}dx}  - i\int\limits_0^\infty  {\sin {x^2}dx}  = \int\limits_0^\infty  {{e^{ - i{x^2}}}dx} \] 作换元 $z = \frac{{1 + i}}{{\sqrt 2 }}x$ \[\int\limits_0^\infty  {{e^{ - i{x^2}}}dx}  = \frac{{1 - i}}{{\sqrt 2 }}\int\limits_0^\infty  {{e^{ - {z^2}}}dz}  = \frac{{1 - i}}{{\sqrt 2 }}\frac{{\sqrt \pi  }}{2}\]
kuing 5# 2011-12-10 12:30
复数也来了,牛了
pxchg1200 6# 2011-12-13 16:18
本帖最后由 pxchg1200 于 2011-12-13 16:19 编辑 5# kuing \[ \Gamma{(\frac{1}{2})}=\sqrt{\pi}=\int_{0}^{\infty}{x^{-\frac{1}{2}}e^{-x}dx} \] \[ \Rightarrow \int_{0}^{\infty}{e^{-x^{2}}dx}=\frac{\sqrt{\pi}}{2} \] (Euler-Poisson) \[ \int_{0}^{\infty}{\sin{t^{2}}dt}=\int_{0}^{\infty}{\frac{\sin{x}}{2\sqrt{x}} dx} \] \[ \int_{0}^{\infty}{e^{-x^{2}}dx}= \sqrt{t} \int_{0}^{\infty}{e^{-u^{2}t}du}=\frac{\sqrt{\pi}}{2}\] (Let $ x=u\sqrt{t}$ ) \[ \frac{1}{\sqrt{x}}=\int_{0}^{\infty}{e^{-u^{2}t}\frac{2}{\sqrt{\pi}}du} \] \[\int_{0}^{\infty}{\frac{\sin{x}}{2\sqrt{x}}dx}=\frac{1}{2}\int_{0}^{\infty}{\sin{t}\int_{0}^{\infty}{\frac{1}{\sqrt{\pi}}e^{-u^{2}t}du}dt}\] 由于一致收敛性,故可以交换积分号 \[ \Leftrightarrow \frac{1}{\sqrt{\pi}}\int_{0}^{\infty}{\int_{0}^{\infty}{\sin{t}e^{-u^{2}t}dt}du} =\frac{1}{\sqrt{\pi}}\int_{0}^{\infty}{\frac{1}{1+u^{4}}du}=\frac{\sqrt{2\pi}}{4} \] Done!
天涯无际 7# 2012-2-21 20:19
这是菲涅尔积分。用柯西积分定理很快就可以算出的。应该是$\sqrt{\frac{\pi}{8}}$。
thread-236-1-1.html: 请教:两个问题,关于切线导数
realnumber 1# 2011-11-29 15:54
111111
kuing 2# 2011-11-29 16:46
算切线; 在实函数范围内,要。
力工 3# 2011-12-3 15:02
切线也在不可导点处。
thread-237-1-6.html: [不等式] 三角函数几何不等式$\sum\cos(A/2)$与$\sum\cos((B-C)/2)$
kuing 1# 2011-12-1 11:13
在 $\triangle ABC$ 中,求证 \[\cos\frac A2+\cos\frac B2+\cos\frac C2\geqslant \frac{\sqrt3}2\left( \cos\frac{A-B}2+\cos\frac{B-C}2+\cos\frac{C-A}2 \right).\]
天涯无际 2# 2012-3-2 10:17
本帖最后由 天涯无际 于 2012-3-2 10:19 编辑 由于有恒等式 \[\sum{\cos{\frac{A-B}{2}}}=2\sum{\sin{\frac{A}{2}}\sin{\frac{B}{2}}}+\sum{\sin{\frac{A}{2}}}\] 故原不等式等价于 \[\sum{\cos{\frac{A}{2}}}\geq \sqrt{3}\sum{\sin{\frac{A}{2}}\sin{\frac{B}{2}}}+\frac{\sqrt{3}}{2}\sum{\sin{\frac{A}{2}}}\] 作变换$A=\pi-2X, B=\pi-2Y, C=\pi-2Z$, 则$\Delta XYZ$为锐角三角形,上式变为 \[\sum{\sin{X}}\geq \sqrt{3}\sum{\cos{X}\cos{Y}}+\frac{\sqrt{3}}{2}\sum{\cos{X}}\] 将上式整理成关于$s,R,r$的表达式,则等价于 \[\frac{s}{R}\geq \sqrt{3}\left(\frac{s^2+r^2}{4R^2}-1\right)+\frac{\sqrt{3}}{2}\left(\frac{r}{R}+1\right)\] 等价于 \[s\leq\frac{2\sqrt{3}R+\sqrt{30R^2-18Rr-9r^2}}{3}\] 由Gerretsen不等式有$s\leq \sqrt{4R^2+4Rr+3r^2}$,只需证明 \[\sqrt{4R^2+4Rr+3r^2}\leq \frac{2\sqrt{3}R+\sqrt{30R^2-18Rr-9r^2}}{3}\] 等价于 \[\frac{4}{3}(R-2r)(6r^3+21r^2R+24rR^2+13R^3)\geq 0\] 上式显然成立.故原不等式得证.由证明过程知当且仅当$\Delta ABC$为正三角形时取得等号.
kuing 3# 2012-3-6 12:40
唔,还是 sRr 有点点暴力鸟…… 有空再验算下
kuing 4# 2012-11-15 19:26
转个简单证法过来 最关键莫过于注意到了第一个恒等式 \[ \sum\cos\frac{B-C}2=\frac12\left(\left(\sum\sin\frac A2\right)^2+\left(\sum\cos\frac A2\right)^2-3\right) \] niubility了
thread-238-1-7.html: [不等式] 高斯函数的三元分式最小值
kuing 1# 2011-12-1 21:31
已知 $a,b,c>0$,求 \[\left[\frac{b+c}a\right]+\left[\frac{c+a}b\right]+\left[\frac{a+b}c\right]\] 的最小值。
kuing 2# 2011-12-1 23:44
其实不难,不过看着感觉有点意思,翻贴的时候意外瞧到的
力工 3# 2011-12-28 07:37
有些意思。易误用均值。
我为中华添光彩 4# 2011-12-28 11:43
1# kuing 请问,最小值是6吗?
我为中华添光彩 5# 2011-12-28 12:29
(b/a+a/b)+(c/b+b/c)+(c/a+a/c) ≥2+2+2=6 (当且仅当a=b=c时取等) (b/a+a/b)+(c/b+b/c)+(c/a+a/c) ≥6 (b/a×a/b×c/b×b/c×c/a×a/c)^(1/6)=6 (当且仅当a=b=c时取等) 这两种算法哪一种是对的?请指教!
killshop 6# 2011-12-28 19:03
从循环不等式角度考虑   或许有些收获
战巡 7# 2012-1-8 14:28
3楼的同志说得对,4楼就一头掉进陷阱里了 由于式子轮换,直接设$a\le{b}\le{c}$ 那么有$$[\frac{b+c}{a}]\ge2,[\frac{a+c}{b}]\ge1$$ 由于整个式子各项都是整数,那么显然加起来至少都是3,此时要求$[\frac{b+c}{a}]=2,[\frac{a+c}{b}]=1,[\frac{a+b}{c}]=0$ 即$b+c<3a,a+c<2b,a+b<c$同时满足 问题是前两式相加就已经有$2c<2a+b,c<a+\frac{b}{2}<a+b$了,第三式无法满足 那么只能退而求其次,整个式子值为4 这个就简单多了,随便举个例子就完了,比如$a=0.8,b=1,c=1.1$,就有最小值4了
kuing 8# 2012-1-8 15:29
7# 战巡 我倒觉得4楼不知道高斯函数 [ ] 是啥,你看他直接忽略了中括号就分开组合了
kuing 9# 2012-1-8 17:53
正确用均值也可以直接证 $\geqslant4$ 由 $[x]>x-1$ 及均值不等式得 \[ \left[\frac{b+c}a\right]+\left[\frac{c+a}b\right]+\left[\frac{a+b}c\right]>\frac{b+c}a-1+\frac{c+a}b-1+\frac{a+b}c-1\geqslant3 \] 从而 \[ \left[\frac{b+c}a\right]+\left[\frac{c+a}b\right]+\left[\frac{a+b}c\right]\geqslant4 \] 举取等条件如战巡。
syzychenwj 10# 2012-3-22 23:47
数学玩家,坚持来学习
thread-239-1-1.html: ff=g 类型问题的常用思路
icesheep 1# 2011-12-3 01:41
即已知 $g\left( x \right)$ 求 $f\left( x \right)$ 使得 $f \circ f = g$ ,有时也许会要求 f 连续/可微
①②③④⑤⑥⑦ 2# 2011-12-5 11:26
本帖最后由 ①②③④⑤⑥⑦ 于 2011-12-15 08:59 编辑 1# icesheep 靠,发帖超时,没得恢复…… 不写了,迭代根问题很麻烦的,很多函数没有连续的两次迭代根,比如严格单调减的函数就一定没有连续迭代根。 即使是看上去最有希望的严格单调增的连续函数,迭代根是肯定有的,但是基本上都是无限个,比如说,定义在$[a,b]$上,$g(a)=a,g(b)=b$ ,且恒有 $g(x)>x$ ,则任意给定定义在 $[x_0,x_1]$ 上的 $f_0(x)$ ,这里 $x0$ 任取,我们要求 $x_0<f_0(x_0)=x_1<f_0(x_1)=g(x_0)$ , 就能拼凑出一个严格单调增的连续函数 $f$ 满足 $f\circ f=g$ 且在 $[x_0,x_1]$ 上 $f=f_0$ 。也就是迭代根无限多个。 (具体表达式不保证能写出来) 所以不太可能见到求迭代根的题目 简单的结论,比如 $ a^2x+b (a\ne0) $ ,有同样为一次函数的两次迭代根 $ax+\dfrac{b}{a+1}$ ,一般是两条, $x+b (b\ne0)$ 则是一条, $f(x)=x$ 则为无穷多条 又比如 $\dfrac{x}{1+2bx}$ 有两次迭代根 $\dfrac{x}{1+bx}$ 等。
kuing 3# 2011-12-5 11:32
1# icesheep 靠,发帖超时,没得恢复…… ①②③④⑤⑥⑦ 发表于 2011-12-5 11:26 呃,工具栏里不是有“恢复数据”的功能么
①②③④⑤⑥⑦ 4# 2011-12-8 09:56
3# kuing 也没啥,具体过程懒得再写了,连续、严格单调增的,两端是不动点,中间没有不动点的,都可以类似的定好初值段,然后构造出一个解(递推+极限过程,解决的是存在性),更多不动点的逐段分别操作就是,两端不是不动点的(当然要求值域不超出定义域),延拓一下
icesheep 5# 2011-12-8 15:28
若g(x)为定义在R上的严格单调递增的连续函数(可以没有不动点,比如 g(x)=e^x),是否一定存在连续 f ? 或者能给一个存在连续 f 的充分条件?
①②③④⑤⑥⑦ 6# 2011-12-14 11:23
本帖最后由 ①②③④⑤⑥⑦ 于 2011-12-14 11:27 编辑 5# icesheep 那我们就对 $g(x)=e^x$ 构造一个二次迭代根,注意到定义域 $R$ 而值域是 $(0,+\infty)$,两者不一样,所以不适合从中间任意取点定初值,我们寻找连续的严格单调增的 $f$,容易知道,这个 $f$ 一定是 $(-\infty,+\infty) \rightarrow (a,+\infty) \rightarrow (0,+\infty)$,这里 $a<0$,下面我们取 $a=-1$, 我们选一个定义在 $(-\infty,-1]$ 上的函数,$f_0(x)=\dfrac{1+x}{1-x}$,它有反函数:$f_0^{-1}(x)=\dfrac{x-1}{x+1},  x\in(-1,0]$。 现在定义 $(-1,0]$ 上的函数 $f_1=g\circ f_0^{-1}$ ,可以具体写出来:$$f_1(x)=e^{\frac{x-1}{x+1}}$$ 由于$f_1(0)=e^{-1}$,接下来我们定义 $(0,e^{-1}]$ 上的函数:$f_2=g\circ f_1^{-1}=g\circ f_0 \circ g^{-1}$ 继续定义 $(e^-1, 1]$ 上的 $f_3=g\circ f_2^{-1}=g^2\circ f_0^{-1} \circ g^{-1}$ $(1,e^{e^{-1}}]$ 上的 $f_4=g\circ f_3^{-1}=g^2\circ f_0 \circ g^{-2}$ …… 我们的到数列:$a_0=-1, a_1=0, a_{n+2}=e^{a_n}$ 区间:$I_0=(-\infty,-1], I_n=(a_{n-1},a_n]$ 定义在各 $I_n$ 上的函数: $f_0, f_{n+1}=g\circ f_n^{-1}$,$f_n$ 的值域是 $I_{n+1}$ (前面已有最初几个的构造,归纳法具体细节略) $a_n$ 无疑是单调增的,趋向无穷大的,将所有这些 $f_n$ 拼接成 $R$ 上的函数 $f$,对任意的实数 $x$,必然有唯一的自然数 $n$ 使得 $x\in I_n$,$$f(f(x))=f_{n+1}(f_n(x))=g(x)$$
icesheep 7# 2011-12-14 16:08
这个有一般性么,无法由此导出一个对5L命题的证明啊,那我换一个 $g\left( x \right) = {x^3} + x$
①②③④⑤⑥⑦ 8# 2011-12-15 09:05
7# icesheep 这个是 $R\rightarrow R$ 的,我觉得把 $\pm\infty$ 视作两个不动点,前面说的那样中间开始定义初值往两边递推应该可以,虽然没实际操作,实在懒得再弄第三遍了(第一遍操作超时毁了,没恢复成功,第二遍我自己先在记事本写的,因为定义域值域不一致又无法延拓,所以实际练手了一次),你有兴趣自己玩吧:)。
①②③④⑤⑥⑦ 9# 2011-12-15 13:11
都搞糊涂了,$g(x)=x^3+x$ 有一个不动点的,用类似的方法得分两段做。 考虑到这是个奇函数,其实只要把 $(0,+\infty)$ 上的搞出来就足够了,这个恰好也是满足 $g(x)>x$ 的,和前面一样,有兴趣你就自己构造一下吧,遇到麻烦再说。
icesheep 10# 2011-12-15 15:07
9# ①②③④⑤⑥⑦ 遇到了啊,第一段【X0,X1】和其上的 f1 就构造不出,要不你给我起个头,我接下来再想想。
①②③④⑤⑥⑦ 11# 2011-12-16 11:41
本帖最后由 ①②③④⑤⑥⑦ 于 2011-12-16 11:54 编辑 10# icesheep 具体形式不重要的啊。你想选怎样的 $x_0$?内部任取的啊,要求是 $0<x_0<f_0(x_0)=x_1<f_0(x_1)=g(x_0)$,那就选 $x_0=2, f_0(x)=x+4$ 好了,$I_0=[2,6]$,往两边推,$f_1=g\circ f_0^{-1},f_{-1}=f_0^{-1}\circ g,\cdots$,$g$的反函数不太好写,但我们本来就没指望这样的方法得到一个形式简单的函数,不需要写出来啊。 示意图: 蓝色实线是 $y=x$,忘了标了。 $f_{-1},f_{-2},...$ 都不是直线段,只是有点接近
thread-24-1-1.html: [函数] 构造一一映射
kuing 1# 2011-9-27 17:12
构造从 (0,1] 到 [0,1] 的一一映射 构造从 [0,1) 到 [0,1] 的一一映射 构造从 (0,1) 到 [0,1] 的一一映射
kuing 2# 2011-9-27 19:06
话说,那个不是破折号,是两个1,早知道用“双射”……
海盗船长 3# 2011-10-3 13:25
http://bbs.pep.com.cn/viewthread.php?tid=1742274&page=1
thread-24-1-9.html: [函数] 构造一一映射
kuing 1# 2011-9-27 17:12
构造从 (0,1] 到 [0,1] 的一一映射 构造从 [0,1) 到 [0,1] 的一一映射 构造从 (0,1) 到 [0,1] 的一一映射
kuing 2# 2011-9-27 19:06
话说,那个不是破折号,是两个1,早知道用“双射”……
海盗船长 3# 2011-10-3 13:25
http://bbs.pep.com.cn/viewthread.php?tid=1742274&page=1
thread-240-1-8.html: [不等式] 【求推广】二阶三角级数的三个小题
icesheep 1# 2011-12-3 02:05
本帖最后由 icesheep 于 2011-12-3 02:40 编辑 设 $T\left( x \right) = {a_1}\cos x + {b_1}\sin x + {a_2}\cos 2x + {b_2}\sin 2x$ 对任意的 x ,满足 $T\left( x \right) \leqslant 1$ 求:$a_1^2 + b_1^2$ 的最大值,$a_2^2 + b_2^2$ 的最大值,$T\left( x \right)$ 的最小值。
icesheep 2# 2011-12-3 02:17
本帖最后由 icesheep 于 2011-12-3 02:40 编辑 前两问变形成 $T\left( x \right) = {A_1}\cos \left( {x - {\varphi _1}} \right) + {A_2}\cos \left( {2\left( {x - {\varphi _2}} \right)} \right)$ 令 $x-{\varphi_1}=\pm\frac{\pi}{4}$,可得 $\frac{{{A_1}}}{{\sqrt 2 }} \pm{A_2}\sin 2\Delta\varphi \leqslant 1$ 于是 ${A_1} \leqslant \sqrt 2 $ 令 $x - {\varphi _2} = \frac{\pi }{2} \pm \frac{\pi }{2}$ ,可得 $\pm {A_1}\cos \Delta \varphi  + {A_2} \leqslant 1$ 于是 ${A_2} \leqslant 1$ 第三问注意到 $f\left( x \right) + f\left( {x + \frac{1}{3}\pi } \right) + f\left( {x + \frac{2}{3}\pi } \right) = 0$ ,从而 $f\left(x\right)\geqslant-2$
thread-241-1-8.html: [不等式] 来自群的简单三元条件$x+y+z=xyz$求最小值
kuing 1# 2011-12-6 14:34
\begin{align*} & x^{2}+y^{2}+z^{2}+\frac{2}{xyz}\\ =&\frac{(x^{2}+y^{2}+z^{2})(x+y+z)}{xyz}+2\sqrt{\frac{xyz}{(x+y+z)^{3}}} \\ \geqslant & \frac{(x+y+z)^{3}}{3xyz}+2\sqrt{\frac{xyz}{(x+y+z)^{3}}} \\ = & \left( \frac{1}{3}-\frac{1}{81\sqrt{3}} \right)\frac{(x+y+z)^{3}}{xyz}+\frac{1}{81\sqrt{3}}\cdot \frac{(x+y+z)^{3}}{xyz}+\sqrt{\frac{xyz}{(x+y+z)^{3}}}+\sqrt{\frac{xyz}{(x+y+z)^{3}}} \\ \geqslant & \left( \frac{1}{3}-\frac{1}{81\sqrt{3}} \right)\cdot 27+3\sqrt[3]{\frac{1}{81\sqrt{3}}} \\ = & 9+\frac{2\sqrt{3}}{9}. \end{align*}
thread-242-1-8.html: [几何] [求助】证等腰
初中生666 1# 2011-12-7 12:10
证等腰 已知三角形ABC,M、N分别为AB、AC上一点,且AM=AN,D为BC中点,∠BDM=∠CDN。求证:三角形ABC为等腰三角形
kuing 2# 2011-12-7 21:01
今天头晕…… 纯平几想不出,反证法也差一点,最后暴力建坐标用解析法证到了
kuing 3# 2011-12-8 12:21
贴一下解析法,没学到的或者不接受的可以无视掉 依题意,不失一般性,设 $B(1,0)$,$C(-1,0)$,则 $D(0,0)$。 由 $\angle BDM=\angle CDN$,可设 $DM$ 和 $DN$ 所在直线分别为 $L_{DM}:y=kx$,$L_{DN}:y=-kx$,其中 $k>0$。 设 $A(a,b)$,其中 $b>0$,由 $A$ 在直线 $DM$ 和 $DN$ 的上方,应有 $b^2>k^2a^2$。 直线 $L_{AB}: (a-1)y=b(x-1)$,直线 $L_{AC}: (a+1)y=b(x+1)$,分别联立 $L_{AB}$、$L_{DM}$ 以及 $L_{AC}$、$L_{DN}$,可以求出 $M$、$N$ 的坐标分别为 \[M\left(\frac{b}{b+k-ak},\frac{bk}{b+k-ak}\right),\quad N\left(-\frac{b}{b+k+ak},\frac{bk}{b+k+ak}\right),\] 由条件 $|AM|=|AN|$,得 \[\left(\frac{b}{b+k-ak}-a\right)^2+\left(\frac{bk}{b+k-ak}-b\right)^2=\left(-\frac{b}{b+k+ak}-a\right)^2+\left(\frac{bk}{b+k+ak}-b\right)^2,\] 化简为 \[\frac{4ab\bigl(-(b+k+bk^2)k^2a^2+(b+k)(b^2+bk+k^2+b^2k^2)\bigr)}{\bigl((b+k)^2-a^2k^2\bigr)^2}=0,\] 由 $k,b>0$,$b^2>k^2a^2$ 知 \begin{align*} -(b+k+bk^2)k^2a^2+(b+k)(b^2+bk+k^2+b^2k^2)&>-(b+k+bk^2)b^2+(b+k)(b^2+bk+k^2+b^2k^2)\\ &=k\bigl((b+k)^2+b^2k^2\bigr)>0,\end{align*} 故只能 $a=0$,可见 $\triangle ABC$ 为等腰三角形。
初中生666 4# 2011-12-8 13:25
非常感谢!顺便说一下此题的来源。11月30号,到北京开会时,茶歇时间,一同事,那了他女儿(初中,初几我忘了问)的这题来问我,结果我想了2天也没想出来。(前几天有人提到反证法,但我想几何问题用反证法,未免太那个了点,嘿嘿);在人教论坛上得帮助,估计为我朋友的朋友在上面发得。见你是解题高手,于是,我就在这里请教了。再次表示感谢!我总算可以交差了! BY THE WAY, 难道这题就没有纯几何证法?
kuing 5# 2011-12-8 13:40
4# 初中生666 我平几水平不怎么样,只会暴力了。 可能有纯平几证法我未曾想到而已。 我会继续关注的。
我为中华添光彩 6# 2011-12-8 22:10
3# kuing 您的解析方法很好,很牛,佩服你。 纠正一个小的笔误。 k>0,应改为k<0。
kuing 7# 2011-12-8 22:14
3# kuing 纠正一个小的笔误。 k>0,应改为k<0。 我为中华添光彩 发表于 2011-12-8 22:10 M 在第一象限的,k>0 没问题吧
我为中华添光彩 8# 2011-12-8 22:19
7# kuing DN在第一象限内,解析式若设成y=kx,那么k>0,但现在您把它设成了y=-kx,所以k应小于0。
kuing 9# 2011-12-8 22:28
8# 我为中华添光彩 M在AB上,我设的是 B(1,0),A(a,b) (b>0)
kuing 10# 2011-12-8 22:49
贴个图
我为中华添光彩 11# 2011-12-8 22:58
10# kuing 明白了,您的ABC表示得同我们一般人的不同。 通常情况,从左到右分别为B、C,您现在的是C、B。
kuing 12# 2011-12-8 23:03
原来还有这习惯,可能是因为我好久没在纸上画过图形,所以没这习惯吧。 我当时先定义B坐标再定义C坐标(按读字母顺序习惯),而又习惯先正后负(正负正负嘛),所以就B在正C在负了……
kuing 13# 2011-12-10 12:28
看了下人教那边,讨论得很热烈哟 有空有精神的时候再慢慢研究一下
李斌斌755 14# 2012-3-5 00:54
据说一些纯几是不能直接用平几证明的,只能反证或解几。 见 http://2666666.blog.163.com/blog ... 0086074082085080071
kuing 15# 2012-3-5 01:11
14# 李斌斌755 链接是 0.1 的博客?(人教论坛的网友 0.1)
thread-243-1-1.html: 积分不等式
kuing 1# 2011-12-8 13:53
设 $f'(x)$ 在 $(a,b)$ 连续且 $f(a)=0$,求证 \[\left|\int_a^bf(x)\text{d}x\right|\leqslant\frac{(b-a)^2}2\max_{a\leqslant x\leqslant b}\left|f'(x)\right|.\]
kuing 2# 2011-12-8 14:05
才发现跟 http://kkkkuingggg.5d6d.com/thread-192-1-1.html 极像的说参考一下先
kuing 3# 2011-12-8 14:09
话说原题是不是需要闭区间呢?否则两端可能不可导,后面的 max 也就。。。。
kuing 4# 2011-12-8 14:44
嗯,借用一下那边的第一步即可了,后面就是注意一下绝对值的东东而已,那边还有柯西,看来还是那个难一点。 记 $\max_{a\leqslant x\leqslant b}\left|f'(x)\right| = k$,设 $x\in[a,b]$,则 \[ f(x)= \int_a^xf'(t)\text dt+f(a)=\int_a^xf'(t)\text dt,\] 于是 \[|f(x)|=\left|\int_a^xf'(t)\text dt\right|\leqslant\int_a^x\left|f'(t)\right|\text dt\leqslant\int_a^xk\text dt=k(x-a),\] 所以 \[\left|\int_a^bf(x)\text dx\right|\leqslant\int_a^b|f(x)|\text dx\leqslant\int_a^bk(x-a)\text dx=\frac{k(b-a)^2}2=\frac{(b-a)^2}2\max_{a\leqslant x\leqslant b}|f'(x)|.\] 其实得到 $|f(x)|\leqslant k(x-a)$ 之后才突然发现几何意义如此明显……
图图 5# 2011-12-9 13:32
3# kuing 嗯,是闭区间..
pxchg1200 6# 2011-12-9 21:30
4# kuing 还是牛顿-莱布尼茨公式重要啊!
thread-244-1-2.html: 无聊也跟着无聊
kuing 1# 2011-12-8 20:58

戊概念·五 2# 2011-12-16 16:22
1# kuing 原来网上的那个被转发若干次的说说是k兄的原创啊~!
kuing 3# 2011-12-16 22:11
2# 戊概念·五 什么说说?
realnumber 4# 2011-12-17 10:57
本帖最后由 realnumber 于 2011-12-17 10:58 编辑 好牛的等式啊,想了半天,没找到漏洞~~ 晕~~本来就是1元=1元,没明白,不是没奇怪的地方吗?
戊概念·五 5# 2011-12-17 17:07
3# kuing 就是截图里你的发言啊~ 我在空间好友更新里见过…… 1元=……=1元
kuing 6# 2011-12-17 17:17
5# 戊概念·五 O
戊概念·五 7# 2011-12-17 17:25
6# kuing 不过,下面明明还开着两个聊天窗口啊.....
kuing 8# 2011-12-17 17:28
7# 戊概念·五 有何关联?
戊概念·五 9# 2011-12-18 11:53
8# kuing 明明就在聊天进行时,怎么还说无聊呢?!
kuing 10# 2011-12-18 12:00
9# 戊概念·五 自己想 自己理解 不理解就算
戊概念·五 11# 2011-12-18 12:02
10# kuing 好一个等差字列
thread-245-1-8.html: [组合] 有意思的二进制问题
lancelot_cc 1# 2011-12-8 21:05
有n张牌垒成一堆,做以下操作: 先将最上面一张放到牌堆最底下,然后将第2张发到桌上, 又将下一张放到牌堆最底下,再将下一张发到桌上, 依次类推,最后牌堆只剩下一张牌,请问是原牌堆的第几张? 现有一种二进制的解法: 1、将n写成二进制数,举例说n=52,则写成110100; 2、将二进制数的第一位挪到最后去生成一个数N,如110100变成101001; 3、这个N对应的十进制数就是答案,101001就是41。 请问怎么证明这个解法?
kuing 2# 2011-12-8 21:12
等价于“约瑟夫环问题”。
lancelot_cc 3# 2011-12-8 21:28
是啊,但是怎么证明这个方法呢
kuing 4# 2011-12-8 21:31
这里有一篇文章
lancelot_cc 5# 2011-12-8 21:34
啊呀~太感谢了!
thread-246-1-8.html: [函数] 曾经被误导,最小中有最大?
realnumber 1# 2011-12-9 15:14
也许这话太形象了,结果~~
thread-247-1-8.html: [不等式] x/y+y/z+z/x的两个下
kuing 1# 2011-12-12 14:48
$x,y,z>0$ \begin{align} \frac xy+\frac yz+\frac zx&\geqslant\frac{9\sqrt3xyz(x^2+y^2+z^2)}{\sqrt{(xy+yz+zx)^5}};\\ \frac xy+\frac yz+\frac zx&\geqslant\sqrt{\frac{3(x+y+z)(x^2y^2+y^2z^2+z^2x^2)}{xyz(xy+yz+zx)}}. \end{align}
pxchg1200 2# 2011-12-12 20:51
这次数这么高来着。。 表示不会做啊。。
kuing 3# 2011-12-12 21:50
我也不会做
thread-248-1-8.html: [函数] 请教一个零值的问题
我为中华添光彩 1# 2011-12-12 20:57
设函数f(x)=x^3-9x^2/2+6x-a (2)若方程f(x)=0有且仅有一个实根,求a的取值范围。 f '(x)=3x^2-9x+6=3(x-1)(x-2) 当x<1时,f '(x)>0;当1<x<2时,f '(x)<0;当x>2时,f '(x)>0 所以x=1时,有最大值;x=2时,有最小值。 f(1)=5/2-a f(2)=2-a 这些都能理解。 下面就不理解了,请大家指教! 故f(2)〉0或者f(1)<0时,方程仅有一个实根,解得a〈2或a〉5/2。
kuing 2# 2011-12-12 21:53
人教的这个贴http://bbs.pep.com.cn/thread-2103459-1-1.html已经讲了我就不重复了。
我为中华添光彩 3# 2011-12-12 22:39
有不少人说了,但都没有说明白。 在(-∞,1)、(1,2)、(2,+∞)三段都有可能获得零点。 必须有一个条件同时否定掉两个区间,只保留一个区间。
kuing 4# 2011-12-12 23:47
有不少人说了,但都没有说明白。 在(-∞,1)、(1,2)、(2,+∞)三段都有可能获得零点。 必须有一个条件同时否定掉两个区间,只保留一个区间。 我为中华添光彩 发表于 2011-12-12 22:39 据单调性,区间 (-∞,1) 以及 (1,2) 内的函数值都小于 f(1),故若 f(1)<0 则这两区间内的函数值都是负的。 同理,若 f(2)>0 则区间 (1,2) 以及 (2,+∞) 内的函数值都是正。 都是同时否定掉两个区间的。
我为中华添光彩 5# 2011-12-13 12:07
您说得很明白,谢谢您的指点!
thread-249-1-1.html: 积分不等式
海盗船长 1# 2011-12-13 13:23
设$f''(x)$在$[0,1]$上连续,$f(0)=f(1)=0$,且$\forall x \in (0,1)$,$f(x) \neq 0$,求证: \[ \int _{0}^{1}\left | \frac{f''(x)}{f(x)} \right |\mathrm{d} x > 4\] 我们去年期末考试题
pxchg1200 2# 2011-12-13 15:54
proof: \\ 由于 $ |f(x)|$ 在$ [0,1]$上连续,故必能取到最大值,设$ y_{0}=|f(c)|$ 为最大值。于是有:\[ \frac{f(c)-f(0)}{c-0}=\frac{f(c)}{c}=f'(x_{1}) \] \[ \frac{f(1)-f(c)}{1-c}=\frac{-f(c)}{1-c}=f'(x_{2}) \] 因此, \[ \int_{0}^{1}{|\frac{f''(x)}{f(x)}| dx}>\int_{x_{1}}^{x_{2}}{|\frac{f''(x)}{f(x)}| dx}\geq \frac{1}{|f(c)|}|f'(x_{2})-f'(x_{1})|= \frac{1}{c(1-c)} \] \\ By AM-GM,we have $ c(1-c) \leq \frac{1}{4} $ \[\int_{0}^{1}{|\frac{f''(x)}{f(x)}| dx}> \frac{1}{4} \] Done!
thread-25-1-6.html: [不等式] 典型的薄马甲吓人题 $\sum1/(1+a^{x-y}+a^{y-z})$
kuing 1# 2011-9-27 17:30
题:已知 $x,y,z$ 为实数,且 $0<a\ne1$,求证 \[\frac1{1+a^{x-y}+a^{y-z}}+\frac1{1+a^{y-z}+a^{z-x}}+\frac1{1+a^{z-x}+a^{x-y}}\leqslant1\] 问题转自 http://user.qzone.qq.com/363215694/blog/1317100708 还红大字双叹号 显然,$a^{x-y}\cdot a^{y-z}\cdot a^{z-x}=1$,所以令 $a^{x-y}=u^3, a^{y-z}=v^3, a^{z-x}=w^3$,问题化为: $u,v,w>0, uvw=1$,有 \[\frac1{1+u^3+v^3}+\frac1{1+v^3+w^3}+\frac1{1+w^3+u^3}\leqslant1\] 这是极为老的问题,由 $u^3+v^3\geqslant uv(u+v)$ 得 \[\frac1{1+u^3+v^3}\leqslant\frac1{1+uv(u+v)}=\frac1{1+\frac{u+v}w}=\frac w{u+v+w}\] 同理得另两个,相加立得。
isea 2# 2011-9-27 19:08
本帖最后由 isea 于 2011-9-27 19:15 编辑 看错了分母,删去原想法。
kuing 3# 2011-9-27 19:09
每一项均为正,且大于1,简单放缩即可,偶都会http://kkkkuingggg.5d6d.com/bbs.php isea 发表于 2011-9-27 19:08 且大于1?
isea 4# 2011-9-27 19:11
3# kuing 小于1,打错字了
kuing 5# 2011-9-27 19:13
4# isea 也没理解上是什么意思,具体写写?
isea 6# 2011-9-27 19:18
5# kuing 看错题啦 对不等式不熟悉啦
kuing 7# 2011-9-27 19:21
6# isea 噢,没事,慢慢熟悉呵呵 可以看看其他贴,有的还没解决的。。。
力工 8# 2011-10-23 20:32
实际上有,a^(x-y).a^(y-z)a^(z-x)=1,换元有,uvw=1.
图图 9# 2011-10-23 21:02
.
kuing 10# 2011-10-23 21:22
9# 图图 真简
我是唐僧 11# 2012-10-3 16:09
想看看答案,哈哈!
thread-250-1-8.html: [不等式] 奇怪的条件
pxchg1200 1# 2011-12-13 16:27
Let $ a,b,c>0$ with \[ a+b+c\le 12\min\left\{\frac{a^{2}}{a+1},\,\frac{b^{2}}{b+6},\,\frac{c^{2}}{c+15}\right\}. \] Determine the minimum value of $ a+b+c$ (Vo Quoc Ba Can)
kuing 2# 2011-12-13 16:31
Potla 也发过这个给我看,完全没头绪……
pxchg1200 3# 2011-12-13 16:32
2# kuing 过了没多久,Potla再次表示毫无压力。。。
realnumber 4# 2011-12-14 15:51
直观想想也许这样,1。假设a+b+c=k是最小,若有a+b+c<12min{f(a),g(b),h(c)},那么c还可以更小点,直至a+b+c=12h(c)(a,b固定的话),导致k更小。(f(a),g(b),h(c)都是增函数,都过(0,0),如果可以令a=0或b=0或c=0的话) 2。可见假设a+b+c=k是最小,有a+b+c≤12min{f(a),g(b),h(c)},且等号成立,若此时k=a+b+c=h(c)=g(a1)<g(a)(或别的如a+b+c=g(b)<h(c)),则固定k, 令c增大,a减小(但始终比a1大)---如此回到1。的情景 由此推测a+b+c=k最小时候有a+b+c=f(a)=f(b)=f(c),三元3方程组,表示还没想出解法。
realnumber 5# 2011-12-14 16:00
本帖最后由 realnumber 于 2011-12-14 16:01 编辑 来个简单的模仿秀,a,b,c都正数,且a+b+c≤12min{$a^2,b^2,c^2$},求a+b+c的最小值.按上面办法可得$a=b=c=\frac{1}{4}$
thread-251-1-8.html: 三角形边长范围a<c,b<c,且1/a+9/b=1
realnumber 1# 2011-12-15 20:03
本帖最后由 realnumber 于 2011-12-15 20:04 编辑 a,b,c为三角形三边,a<c,b<c,且1/a+9/b=1 求c边范围.答案似乎是(10,16)是用两边之和之差猜的.解答题的话,用余弦定理吗?
icesheep 2# 2011-12-16 02:30
1# realnumber (10,+inf) 吧,为什么会有上界呢? c 接近10的时候,就是接近一个正三角形; c 很大的时候,就是一个非常狭长的三角形。 这种题目不会出成解答题的,要回答严密很费事儿。
realnumber 3# 2011-12-16 09:50
恩,应该您是对的,没有上界;开始我还以为(a+b)(1/a+9/b)≥(1+3)(1+3)=16,而c<a+b
thread-252-1-8.html: 概率问题?
realnumber 1# 2011-12-16 14:23
自然数集合中取到偶数概率是多少?0.5? 但是N={0,2,1,4,6,3,8,10,5,....}如此排列呢?概率是2/3? ...
①②③④⑤⑥⑦ 2# 2011-12-16 14:40
1# realnumber 对于样本空间是可列集的情形,不可能有一种概率分布使得每个单元素集所对应的事件等概率发生,你提出取到偶数的概率之前,先要给出明确的概率分布才行,不存在默认的“均匀分布”
kuing 3# 2011-12-16 22:22
话说你列的数有何规律?
realnumber 4# 2011-12-17 10:55
就是2个偶数,一个奇数依次下来,---模仿条件收敛级数----(1-1/2+1/3-1/4+1/5-.....可以交换次序得到极限为任意值) --谢谢2楼,我知道自己一定错了,就是不知道该怎么考虑
thread-253-1-8.html: [几何] 2011年浙江会考第41题6分
realnumber 1# 2011-12-17 13:27
就第2问,已知圆$x^{2}+y^{2}=4$,过(1,0)直线交圆于A、B,C(4,0),求证∠ACE=∠BCE 解几方法知道的,有没平面几何办法?
kuing 2# 2011-12-17 14:05
这是典型的初中题改编成高中题 本来就是平几题,数据可以一般化,只要C、D关于圆互为反演点(也就是说 $EC\cdot ED=R^2$)就有这样的结论。
kuing 3# 2011-12-17 14:58
找到了08年画的一个图,应该就是在初中论坛做题时留下的东东了
kuing 4# 2011-12-17 14:59
还发现原来当年打错字,是关于 PO 对称才对
戊概念·五 5# 2011-12-17 17:12
3# kuing 似曾相识的感觉.....
力工 6# 2011-12-29 14:27
试下,看显示对不对。 不等式$1<1/sqrt(1+a)+1/sqrt(1+b)<=2/sqrt(1+sqrt(ab))$,其中$a,b>0,ab<=4$的一种证明.以下记$t=sqrt(ab),x=a+b>=2t,t<=2$. 证明:1.左边的不等式可加强为$1/sqrt(1+a)+1/sqrt(1+b)>=1,a,b>=0,ab<=9$. 两边平方$1/(1+a)+1/(1+b)+2/sqrt((1+a)(1+b))>=1 iff2sqrt((1+a)(1+b))>=ab-1$.    $ab<=1$情形,上述不等式显然成立;  若$1<ab<=9$,再次平方上述不等式可化为$a^2b^2-6ab-4(a+b)-3<=0,a+b>=2sqrt(ab)$.只需$a^2b^2-6ab-8sqrt(ab)-3<=0iff(t-3)(t+1)^3<=0ifft-3<=0$,显然.    2.右边不等式$1/sqrt(1+a)+1/sqrt(1+b)<=2/sqrt(1+sqrt(ab)),a,b>0,ab<=4$的证明. 将之平方只需证明:$2(1+t)sqrt(1+a+b+t^2)<=4(1+a+b+t^2)-(1+t)(2+a+b)$,再次平方整理为 $(3-t)^2x^2+8(1-t)^3x+4t(3t^3-6t^2+3t-4)>=0$.记其左端为$f(x)$. 而$f^'(x)=2(3-t)^2x+8(1-t)^3>=4t(3-t)^2+8(1-t)^3=(2-t)(1+t)^2>=0$. 于是$f(x)>=f(2t)=0$,得证!(也可以不求导,直接配方
thread-254-1-6.html: [不等式] 杨学枝猜想15(修正,未解决)
realnumber 1# 2011-12-17 20:19
本帖最后由 realnumber 于 2011-12-17 20:21 编辑 大家继续,我看来做不出,解出的话,可直接发给杨老师
kuing 2# 2012-10-7 00:19
最新情况如何?楼主
realnumber 3# 2012-10-14 10:24
本帖最后由 realnumber 于 2012-10-14 10:27 编辑 严文兰老师早已经解决了。虽然还是不明白.
kuing 4# 2012-10-14 14:59
oh,我没太关注那堆猜想,全都不是我在行的东东……
thread-255-1-8.html: [不等式] 嗯,活跃下气氛,来个简单的。(kk来秒杀吧)
pxchg1200 1# 2011-12-18 12:33
Let $a,b,c \geq 0$ prove that: \[ \frac{1}{2a^{2}+bc}+\frac{1}{2b^{2}+ac}+\frac{1}{2c^{2}+ab}\geq \frac{8}{(a+b+c)^2} \]
kuing 2# 2011-12-18 13:25
1# pxchg1200 完全没看出哪里简单了……
pxchg1200 3# 2011-12-18 22:00
2# kuing 嗯,取等条件是$ a=b, c=0 $ 及其轮换。。 应该够简单了吧。
kuing 4# 2011-12-18 22:04
3# pxchg1200 取等条件很容易看出,不过没发现有什么帮助
pxchg1200 5# 2011-12-18 22:25
4# kuing 接着就Cauchy-Schwarz了啊。。
kuing 6# 2011-12-18 23:10
5# pxchg1200 不懂哩 不知怎么柯才不过……
pxchg1200 7# 2011-12-18 23:51
6# kuing Hint: \[ \frac{1}{2a^{2}+bc}+\frac{1}{2b^{2}+ac}+\frac{1}{4c^{2}+2ab}+\frac{1}{4c^{2}+2ab} \geq \cdots \]
kuing 8# 2011-12-19 01:17
7# pxchg1200 这个我也想过刚才,不过好像后面还是反了先睡了明天再玩
pxchg1200 9# 2011-12-19 12:43
8# kuing 另外就是还有这个: \[ \frac{1}{2a^{2}+bc}+\frac{1}{2b^{2}+ca}+\frac{1}{2c^{2}+ba}\ge\frac{1}{ab+bc+ca}+\frac{2}{a^{2}+b^{2}+c^{2}} \]
tian27546 10# 2011-12-28 16:24
http://blog.sina.com.cn/s/blog_62a489bd0100w59b.html
pxchg1200 11# 2011-12-28 22:30
10# tian27546 哟,你也来啦。   不过你那个证明好像太复杂了,其实有个简单的CS proof的。
天涯无际 12# 2012-2-17 19:00
下面这个也是成立的: \[\frac{1}{a^2+2bc}+\frac{1}{b^2+2ca}+\frac{1}{c^2+2ab}\geq \frac{2}{ab+bc+ca}+\frac{1}{a^2+b^2+c^2}\] 不过这个比上面那个简单~~~~~
thread-256-1-1.html: 琴生不等式
海盗船长 1# 2011-12-19 19:01
本帖最后由 海盗船长 于 2011-12-19 22:13 编辑 已知下凸函数$f(x)$在$[a,b]$上可积, 求证: \[ f\left( \frac {a+b} {2} \right) \le \frac {1} {b-a} \int _{a}^{b} f(x) \mathrm{d} x \le \frac {f(a)+f(b)}{2}\]
pxchg1200 2# 2011-12-19 21:42
表示无法显示。。。
kuing 3# 2011-12-19 23:53
2# pxchg1200 这几天mathjax不太稳定 昨天和今天都各有一段时间显示不出来,现在显示OK,不过其实看代码也能知道公式内容了吧应该,这个不复杂
pxchg1200 4# 2011-12-20 16:33
1# 海盗船长 证明: 左边: 用 $x=t+\frac{a+b}{2} $ $ \int_{a}^{b}{f(x)dx}=\int_{\frac{a-b}{2}}^{\frac{b-a}{2}}{f(t+\frac{a+b}{2})dt} $ \[ \int_{\frac{a-b}{2}}^{0}{f(t+\frac{a+b}{2})dt}=\int_{0}^{\frac{b-a}{2}}{f(\frac{a+b}{2}-t)dt}\] (作 $x=-t$) \[ \Rightarrow \int_{a}^{b}{f(x)dx}=\int_{0}^{\frac{b-a}{2}}{f(t+\frac{a+b}{2}+f(\frac{a+b}{2}-t)dt}\geq (b-a)f(\frac{a+b}{2}) \] 右边:对 $x=(1-k)a+kb$ $ dx=(b-a)dk $ \[ \frac{1}{b-a}\int_{a}^{b}{f(x)dx}=\int_{0}^{1}{f((1-k)a+kb)dk}\leq \int_{0}^{1}{(1-k)f(a)+kf(b)dk}=\frac{1}{2}(f(a)+f(b)) \] Done!
海盗船长 5# 2011-12-20 18:33
本帖最后由 海盗船长 于 2011-12-20 18:34 编辑 左边还可以用定积分的定义: 取关于$\dfrac{a+b}{2}$对称的点$\xi _i$做分割,然后对每一对这样的点用凸函数性质。
thread-257-1-1.html: 积分绝对值不等式
海盗船长 1# 2011-12-20 11:27
本帖最后由 海盗船长 于 2011-12-20 11:31 编辑 已知函数$f(x) \in C^{(1)}[a,b] $,求证: (1)对于任意的$x \in [a,b]$,有 \[ |f(x)| \le \left| \frac{1}{b-a} \int_{a}^{b} f(x) \mathrm{d} x\right|+ \int_{a}^{b} |f'(x)| \mathrm{d} x \] (2) \[ \left |f \left ( \frac{a+b}{2} \right ) \right| \le \left|\frac{1}{b-a} \int_{a}^{b} f(x) \mathrm{d} x\right|+ \frac{1}{2} \int_{a}^{b} |f'(x)| \mathrm{d} x \]
icesheep 2# 2011-12-23 23:02
本帖最后由 icesheep 于 2011-12-24 13:36 编辑 ①不妨设 $f\left( \xi \right) = \mathop {\max}\limits_{a \leqslant x \leqslant b} \left| {f\left( x\right)} \right|$,$f\left( \eta\right) = \frac{1}{{b - a}}\int_a^b {f\left( x \right){\text{d}}x} $ \[\int\limits_a^b{\left|{f'\left( x \right)}\right|{\text{d}}x} \geqslant\int\limits_\eta^\xi {\left|{f'\left(x\right)}\right|{\text{d}}x}\geqslant\left|{f\left(\xi\right)-f\left(\eta\right)}\right|\geqslant\left|{f\left(\xi\right)}\right|-\left|{f\left(\eta\right)}\right|\geqslant\left|{f\left(x\right)}\right|-\left|{f\left(\eta\right)}\right|\] ②设 $m = \frac{{a + b}}{2}$,由第一题, \[\left| {f\left( m \right)} \right| \leqslant \left| {\int\limits_a^b {f\left( x \right){\text{d}}x} } \right| + \int\limits_a^m {f'\left( x \right){\text{d}}x} \] \[\left| {f\left( m \right)} \right| \leqslant \left| {\int\limits_a^b {f\left( x \right){\text{d}}x} } \right| + \int\limits_m^b {f'\left( x \right){\text{d}}x} \] 相加,\[\left| {f\left( m \right)} \right| \leqslant \left| {\int\limits_a^b {f\left( x \right){\text{d}}x} } \right| + \frac{1}{2}\int\limits_a^b {f'\left( x \right){\text{d}}x} \]
thread-258-1-1.html: 继续积分绝对值不等式
海盗船长 1# 2011-12-20 15:24
已知函数$f(x) \in C^{(1)}[a,b]$,且$f(a)=0$,求证: \[ \int_{a}^{b} |f(x)f'(x)| \mathrm{d} x \le \frac{b-a}{2} \int_{a}^{b} |f'(x)|^2 \mathrm{d} x\]
pxchg1200 2# 2011-12-20 16:04
1# 海盗船长 证明: 设$ F(x)=|f(x)|-|f(a)|=\int_{a}^{x}{|f(t)|dt} $ 由此\[ |f(x)|=F(x) \] \[ |f'(x)|=F'(x)\] \[ \Rightarrow \int_{a}^{b}{F(x)F'(x)dx}=\frac{1}{2}F^{2}(b) =\frac{1}{2}(\int_{a}^{b}{|f'(t)|dt})^{2}\] Now,Using Cauchy-Schwarz \[ \frac{1}{2}(\int_{a}^{b}{|f'(t)|dt})^{2}\leq \frac{b-a}{2}\int_{a}^{b}{(f'(x))^{2}dx} \] Done!
thread-259-1-1.html: mathjax!正常一点好不……
kuing 1# 2011-12-20 22:12
RT!mathjax!最近你老是不稳定啊不显示啊搞什么呢虽然我听说最近你的总监死了……
kuing 2# 2012-1-12 15:25
又 load 不出来了 哎,到底是mathjax的问题还是****敏感词语呢?
kuing 3# 2012-2-12 15:45
今天又连不上
kuing 4# 2012-2-12 20:35
汗,刚才mathjax主站进不了,用代理竟然进了!还真是*****?
kuing 5# 2012-2-12 20:48
原来有更新 记录两段代码先 <script type="text/javascript"   src="http://cdn.mathjax.org/mathjax/latest/MathJax.js?config=TeX-AMS_HTML"> </script> <script type="text/javascript" src="http://cdn.mathjax.org/mathjax/2.0-beta/MathJax.js?config=TeX-AMS-MML_HTMLorMML-full"> </script>
kuing 6# 2012-2-17 21:58
用 2.0-bate 有一个不好是查看代码时比以前麻烦,以前直接是第一项,现在是第一项里的第二项……
kuing 7# 2012-3-4 13:00
<script type="text/x-mathjax-config">   MathJax.Hub.Config({     tex2jax: {       inlineMath: [ ['$','$'], ["\\(","\\)"] ],       processEscapes: true       }   }); </script> <script type="text/javascript"   src="http://cdn.mathjax.org/mathjax/2.0-latest/MathJax.js?config=TeX-AMS-MML_HTMLorMML"> </script>
kuing 8# 2012-3-4 13:10
<script type="text/javascript">         var _gaq = _gaq || [];         _gaq.push(['_setAccount', 'UA-15609829-1']);         _gaq.push(['_trackPageview']);         _gaq.push(['_trackPageLoadTime']);         (function() {                 var ga = document.createElement('script'); ga.type = 'text/javascript'; ga.async = true;                 ga.src = ('https:' == document.location.protocol ? 'https://ssl' : 'http://www') + '.google-analytics.com/ga.js';                 var s = document.getElementsByTagName('script')[0]; s.parentNode.insertBefore(ga, s);         })(); </script> <script type="text/javascript"   src="http://cdn.mathjax.org/mathjax/2.0-latest/MathJax.js?config=TeX-AMS-MML_HTMLorMML"> </script>
kuing 9# 2012-7-22 16:50
换了个链接(参见http://www.mathjax.org/2012/05/0 ... changes-to-the-cdn/),看会不会稳定些 <script type="text/x-mathjax-config">   MathJax.Hub.Config({     tex2jax: {       inlineMath: [ ['$','$'], ["\\(","\\)"] ],       processEscapes: true       }   }); </script> <script type="text/javascript"   src="http://c328740.r40.cf1.rackcdn.com/mathjax/latest/MathJax.js?config=TeX-AMS_HTML"> </script> <script type="text/x-mathjax-config">   MathJax.Hub.Config({     tex2jax: {       inlineMath: [ ['$','$'], ["\\(","\\)"] ],       processEscapes: true       }   }); </script> <script type="text/javascript"   src="http://cdn.mathjax.org/mathjax/latest/MathJax.js?config=TeX-AMS-MML_HTMLorMML"> </script>
thread-26-1-8.html: [数论] 来自群:数列101,10101,1010101.....的质数个数
kuing 1# 2011-9-27 23:05
在101,10101,1010101………这种数中质数的个数 问题来自超级群 solution 即求 $a_1=101$,$a_{n+1}=100a_n+1$ 的无穷数列 $\{a_n\}$ 中的质数个数。 不难求出\[a_n=\frac{100^{n+1}-1}{99},\] 下面分奇偶项讨论。 若 $n=2k-1,k\in\mathbb{N}^+$,则 \[a_{2k-1}=\frac{100^{2k}-1}{99}=(100^k+1)\cdot\frac{100^k-1}{99},\] 注意到 $k=1$ 时 $a_1=101$ 的确为质数,而当 $k\geqslant2$ 时 $\dfrac{100^k-1}{99}=a_{k-1}$,所以此时 $a_{2k-1}$ 显然不是质数; 若 $n=2k,k\in\mathbb{N}^+$,则 \[a_{2k}=\frac{100^{2k+1}-1}{99}=\frac{(10^{2k+1}+1)(10^{2k+1}-1)}{99},\] 注意到 \begin{align} 10^{2k+1}+1&=10^{2k+1}-(-1)^{2k+1}\\ &=\bigl(10-(-1)\bigr)\bigl(10^{2k}+10^{2k-1}(-1)+10^{2k-2}(-1)^2+\cdots+(-1)^{2k}\bigr)\\ &=11(10^{2k}-10^{2k-1}+10^{2k-2}-\cdots+1),\\ 10^{2k+1}-1&=10^{2k+1}-1^{2k+1}\\ &=(10-1)(10^{2k}+10^{2k-1}+10^{2k-2}+\cdots+1)\\ &=9(10^{2k}+10^{2k-1}+10^{2k-2}+\cdots+1), \end{align} 故 \[a_{2k}=(10^{2k}-10^{2k-1}+10^{2k-2}-\cdots+1)(10^{2k}+10^{2k-1}+10^{2k-2}+\cdots+1),\] 显然上式右边两括号内都 $>1$,故此时 $a_{2k}$ 均不是质数。 综上所述,$\{a_n\}$ 中只有 $a_1=101$ 是质数。
kuing 2# 2012-2-8 14:29
四个多月后的今天,群里再现此题
thread-264-1-1.html: 这里没有设置连发限制?
戊概念·五 1# 2011-12-22 09:23
已经不止一次连发了 最离谱一次一个Ctrl+Enter就出来个三胞胎= =然后还要手动删除后边两个 记得人教那里有个时间限制的,这里k兄没有设置?
kuing 2# 2011-12-22 09:56
1# 戊概念·五 为了大家发贴方便爽快这里完全不限制这些。 而且多发是你(或电脑或网络)的问题。
戊概念·五 3# 2011-12-22 11:33
2# kuing 成,我的电脑也是老人了 我就手动处理她附赠给我的“随机礼物”好了
thread-265-1-7.html: 人教论坛不知怎么的打不开了?
我为中华添光彩 1# 2011-12-22 12:37
如题。 比如 http://bbs.pep.com.cn/forum-38-1.html 。
kuing 2# 2011-12-22 13:21
昨晚开始就进不了了 不知在搞什么,嘿,来这里好了
戊概念·五 3# 2011-12-22 13:41
2# kuing 原来不止我刷不开论坛 还以为又是我的网络出问题了呢
kuing 4# 2011-12-22 13:58
不过最近这里的 mathjax 也有点不稳定,尤其是前几天,有时显示公式比较慢甚至提示错误,今天还好。
戊概念·五 5# 2011-12-22 14:26
4# kuing k兄的帖子巨可爱呀~
我为中华添光彩 6# 2011-12-22 19:44
昨晚开始就进不了了 不知在搞什么,嘿,来这里好了 kuing 发表于 2011-12-22 13:21 来这里是个不错的选择。 可惜的是,人教论坛上有不少现成资源。 由此可见,免费空间是存在数据安全隐患的。
kuing 7# 2011-12-22 20:58
4# kuing k兄的帖子巨可爱呀~ 戊概念·五 发表于 2011-12-22 14:26 胡说
kuing 8# 2011-12-22 21:00
6# 我为中华添光彩 这里主要是玩题研究,没什么资源,更没有人教论坛数学资源区那种资料。
戊概念·五 9# 2011-12-24 12:09
今天还是打不开= =
戊概念·五 10# 2011-12-27 12:01
9# 戊概念·五 今天可以打开了~
kuing 11# 2011-12-27 18:32
的确可以上了。 PS。下面是我在群邮件里面关于这次人教论坛连续N天打不开的事件扯的一些东东,请由下往上读,因为较新回复的邮件会在较上方。 [url=][/url] 15 楼 [url=]群管-kuing/pp[/url]41分钟前 今天停电,刚有电上来就能上了,同时结果也出来了: 一进论坛首先显然是发现论坛并没有升级,猜想二落空; 再进站务和龙门版块,仍然一堆堆的广告贴,猜想一落空; 最后仔细逛了一会,还用马甲测试回贴等等,没发现任何变化,回贴时仍然需要答“九除三等于几”等等…… 而且目前还没有管理人员出来解释原因,没任何公告。 于是……你觉得呢?…… [url=][/url] 14 楼 [url=]爱好者-/yx[/url]昨天 18:54 打电话给人教社吧 [url=][/url] 13 楼 [url=]教师-dilin[/url]昨天 18:31 5 楼 [url=]群管-kuing/pp[/url]12月24日 15:23 回复“教师-dilin”:嗯?这么神奇?难道是缓存?那你上去有没有看到什么特别的东东呢?………… ~~~~~~~~~~~~~~~~~~~~~~~~~ 果然是“缓存”! [url=][/url] 12 楼 [url=]群管-kuing/pp[/url]昨天 18:15 谁有nix电话呢 [url=][/url] 11 楼 [url=]群管-kuing/pp[/url]昨天 13:23 哎……可叹是我依然未存完档,实在有太多需要存档的贴鸟! [url=][/url] 10 楼 [url=]爱好者-动力[/url]昨天 10:27 今天还不行 [url=][/url] 9 楼 [url=]爱好者-yang[/url]昨天 10:02 额,kuing还真能扯,哈哈 [url=][/url] 8 楼 [url=]爱好者-/yx[/url]12月25日 13:32 我觉得也许会是第2点的进一步版:论坛直接被黑了 [url=][/url] 7 楼 [url=]教师-jchentai8[/url]12月25日 9:55 不知道是什么原因,许多天了都上不去。 [url=][/url] 6 楼 [url=]教师-xyz[/url]12月25日 8:11 连版主们都不知道?这看这事有点…… [url=][/url] 5 楼 [url=]群管-kuing/pp[/url]12月24日 15:23 回复“爱好者-/yx”:也不是这么说,虽然是冷但是好贴和资源还是很多。 回复“教师-xyz”:以前论坛有什么事都会有管理员发公告,或者有什么事尽管不公开讲也至少会给版主发短消息通知一下的,有时也会听取一下论坛会员的意见,不过这一两年来就基本完全不理了。。。 回复“教师-dilin”:嗯?这么神奇?难道是缓存?那你上去有没有看到什么特别的东东呢? [url=][/url] 4 楼 [url=]教师-dilin[/url]12月24日 13:59 咦,今天上午在单位,用手机可以上的。 这会儿,在家用电脑,还是上不了,奇怪! [url=][/url] 3 楼 [url=]教师-xyz[/url]12月24日 7:27 分类讨论,呵呵,挺全面的, 不过人教社主页上也没个说明,可见这次肯定是有什么不好说或者不能说的原因。 也许是人教社群管霸道惯了,根本就没想到还得给常来的朋友们说一声。 [url=][/url] 2 楼 [url=]爱好者-/yx[/url]12月24日 1:01 哇靠  太牛太无聊 嗯的确上不了 其实我好久没上  现在论坛不如群了不是么 [url=][/url] 1 楼 [url=]群管-kuing/pp[/url]12月24日 0:57   平时有上人教论坛的网友都应该发现了,人教论坛已经有好几天打不开了,群里好些网友都问及过相关情况。然而到现在也未曾有任何关于此的确切消息,就连我们群里面的人教社bian辑也不清楚具体原因,所以大家也只能众说纷云。   又众所周知我每天都很悠闲,所以我没事就会胡想些乱七八zao的东西,而且还会无聊地写下来,于是就有了这封无聊邮件讲述我对人教论坛连续几天上不了的原因的如下一些绝对值不成shou的猜想: 也许是因为人教论坛的guang告太多,而且有不少是weifa的,甚至是@#$%的,然后被网jing查到或是被举报,结果责令整改,要来一次大扫除或者搞些新措施防guang告等等。 受这几天发生的那个什么C什么N的大网站或大论坛被hei客弄了N多帐hu的mi码资料等的事件的影响,人教论坛可能要搞搞相应的安全方面的东东,也就是可能会升级论坛系统。 这几天可能有些min感zheng治事jian发生,人教论坛为了安全起见,避开min感话题,怕有人发布@#$%而牵连到论坛,所以临时关闭几天避一避风头。   当然,可能的原因绝对还有很多,由于时间关系我暂时就列这三点吧,不知大家怎么看?   说说个人感觉:   第1点的可能xing比较小,因为人教论坛的guang告多并不是一两天的事了,至少这一年半载以来龙门阵版块就天天被guang告爆满。再说,大扫除或什么的估计也不用这么多天。   至于第2、3点,感觉还是第2的可能xing大些,因为论坛上不了和那事件的发生时间很近,而且论坛的确需要升级,实在太老了,Discuz!6.0是07年的东东啊。   最后再扯一下应验,假设过几天论坛能上了,如果: 论坛整洁啦!(看不到guang告贴满天飞而且在发贴回贴时发现防guang告用的新招啦(也就是发贴又更麻烦了)),那很有可能就是应了第1点。 论坛升级啦!那很有可能就是应了第2点。 没发现论坛有任何改变,那很有可能就是应了第3点。   扯完,睡觉,希望明天早上上来论坛就能好起来,不管是以上什么原因……
李斌斌755 12# 2012-5-10 18:47
k哥,人教论坛怎么打不开了
kuing 13# 2012-5-10 18:52
12# 李斌斌755 不知道 而且这个贴已经过时了,还是别顶了。
thread-266-1-8.html: 如何书写公式?
我为中华添光彩 1# 2011-12-22 19:46
我想写带有上下标的数学式,但不知道怎么写。 请版主赐教!
我为中华添光彩 2# 2011-12-22 19:59
本帖最后由 我为中华添光彩 于 2011-12-22 20:01 编辑 刚才看到了教程。 $C_n^m$
我为中华添光彩 3# 2011-12-22 20:02
请教版主 什么叫  行间公式  和 行内公式? 请版主举例说明!
thread-267-1-8.html: [数列] 请教某个数列通项公式
我为中华添光彩 1# 2011-12-22 20:13
本帖最后由 我为中华添光彩 于 2011-12-22 20:29 编辑 $b_1=\dfrac34 , b_{n+1}=\dfrac{1}{2-b_n}$ 求 $b_n$
kuing 2# 2011-12-22 20:30
\begin{align*} b_{n+1}=\frac1{2-b_n}&\implies b_{n+1}-1=\frac1{2-b_n}-1=\frac{b_n-1}{2-b_n}=\frac{b_n-1}{1-(b_n-1)} \\ & \implies\frac1{b_{n+1}-1}=\frac1{b_n-1}-1. \end{align*}
我为中华添光彩 3# 2011-12-22 20:51
2# kuing 谢谢版主! 标准答案也是这样的,不过,我就是想不明白,中间过程中的-1以及后面的颠倒算式是如何想到的。
kuing 4# 2011-12-22 20:56
3# 我为中华添光彩 数学空间第一期P9~10
我为中华添光彩 5# 2011-12-22 21:24
谢谢版主,按照您的指示我找到了这部分内容。
thread-268-1-8.html: [数列] 求教一个数列求和
我为中华添光彩 1# 2011-12-23 12:46
本帖最后由 我为中华添光彩 于 2011-12-23 12:48 编辑 $\dfrac{n^2}{2^n}$
kuing 2# 2011-12-23 13:20
错位相减两次
我为中华添光彩 3# 2011-12-23 13:56
2# kuing 谢谢版主! 我试试看。
我为中华添光彩 4# 2011-12-23 21:51
按照二次错位相减后,结果很复杂。 多次演算,得到的结果都不对。 多次检查,也没发现问题。 先前是S1验算也不对。 现在的情况是S1验算正确,S2就不对了。 真是活见鬼!
我为中华添光彩 5# 2011-12-23 22:28
4# 我为中华添光彩 估计是等比数列的求和错了一位。
icesheep 6# 2011-12-23 23:57
本帖最后由 icesheep 于 2011-12-24 00:07 编辑 4# 我为中华添光彩 每写一个式子,标清楚该式子成立的 n 的范围。 第二次错位相减的时候,n=1 的情况要分开写。
我为中华添光彩 7# 2011-12-24 09:16
本帖最后由 我为中华添光彩 于 2011-12-24 09:19 编辑 谢谢楼上,也谢谢版主,现在对了。 问题出在等比数列求和上。 $\frac12+\frac1{2^2}+\frac1{2^3}+...+\frac1{2^{n-1}}$ $\frac1{2^{n-1}}$不是数列通项。
海盗船长 8# 2011-12-24 15:43
待定系数法: \[ S_n=(An^2+Bn+C) \left( \frac{1}{2} \right)^n+D \]
kuing 9# 2011-12-24 15:44
其实可以裂项
海盗船长 10# 2011-12-24 20:37
怎么裂项啊?和待定系数类似?
kuing 11# 2011-12-24 20:45
人教论坛能上的时候再找贴
海盗船长 12# 2011-12-24 20:56

①②③④⑤⑥⑦ 13# 2011-12-26 13:42
本帖最后由 ①②③④⑤⑥⑦ 于 2011-12-26 13:54 编辑 嗯,错位相减和裂项相消,其本质是差不多的 先来看错位相减: $a_{n+1} - k a_n = b_n$,如果 $b_n$ 容易求和,部分和数列记为 $T_n$,那么前面的式子可以很自然的转化为 $S_{n+1}-a_1-k S_n = T_n$,注意到 $S_{n+1}=S_n+a_{n+1}$,这里不必按一阶递推数列通法解,直接利用已知的 $a_n$ 表达式即可。平时常见的,证明等比数列求和公式的时候, $b_n$ 是常数列(都是0),当 $a_n$ 是等差和等比相乘(指的是对应项相乘得到新数列)的形式,$b_n$ 是等比数列。而这里,一个二阶等差和一个等比相乘,得到的  $b_n$ 是 等差和等比相乘,它的求和,不作为基本结果直接使用,所以要两次错位相减,写起来麻烦一点,思路是清晰的。至于中间关于 $n$ 的范围什么的,其实是一个习惯性问题,按我前面说的来做,其实没有那麻烦的检验首项: $a_n = \dfrac{n^2}{2^n}$,则 $2a_{n+1} - a_n = \dfrac{2n+1}{2^n}$,我们先要搞定 $\dfrac{2n+1}{2^n}$ 的求和, 记$b_n = \dfrac{2n+1}{2^n}$,则 $2b_{n+1} - b_n =\dfrac1{2^{n-1}}$,于是有 $2T_{n+1} -2b_1- T_n =2\left(1-\dfrac1{2^n}\right)$,然后不难得到 $T_n =2\left(1-\dfrac1{2^n}\right)+ 2b_1-2b_{n+1}=2\left(1-\dfrac1{2^n}\right)+ 3-\dfrac{2n+3}{2^n}=5-\dfrac{2n+5}{2^n}$。 现在回到原先的式子,可得到 $2S_{n+1} -2a_1- S_n = 5-\dfrac{2n+5}{2^n}$,整理后可得到 $S_n = 5-\dfrac{2n+5}{2^n}+1-\dfrac{(n+1)^2}{2^n}=6-\dfrac{n^2+4n+6}{2^n}$。 关键是减出一个容易求和的数列,或许你会想到一种扩展,如果另有一个数列 $k_n$ 使得数列 $k_n a_n$ 容易求和,我们可以用 $a_{n+1} - k_n a_n = b_n$ 或其他的扩展形式来求 $S_n$,确实可以,但没想到太好的例子(如果你看下去,相信你也会和我有一样的感觉,这不会有什么典型的例子,或许,到这里你已经识破玄机)。 我们先用一种特殊的格式来求 $a_n=\dfrac{n}{2^n}$ 的和: $2a_{n+1}-\dfrac{a_n}{n}=\dfrac{n+1}{2^n}-\dfrac1{2^n}=a_n$ $2S_n+2a_{n+1}-2a_1 - 1+\dfrac1{2^n} = S_n$ $S_n=1-\dfrac1{2^n}-2a_{n+1}+2a_1=2-\dfrac{n+2}{2^n}$ 现在回过头想想,$k_n a_n$ 自成一个数列,虽然含有 $a_n$,最多也就是一种思路提示,其实就是找到了一个数列 $c_n$ 使得 $a_{n+1}-c_n=b_n$,这里  $b_n$ 和 $c_n$ 都容易求和,正负号不重要,下标有点固定的偏移也不重要,其实我们就是找到了一种等式 $a_n=b_n-c_n$,这是什么?看上去就是裂项啊,只不过最典型的裂项一般是 $a_n=b_{n+1}-b_n$,也就是说,同样是三个数列的线性组合得0,典型的错位相减是其中的两个数列直接来自原数列 $a_n$,此时要求第三个容易求和;而典型的裂项相消是其中两个恰好来自同一个数列的前后项,此时不在乎这个数列有没有什么好的性质,第三个直接来自原数列即能对原数列求和,典型裂项其实一开始就已经找到了 $S_n$ 的表达式的关键部分,就差一点点了。 一般情形,是若干数列的线性组合得到零,其中至少一个直接来自原数列,其他的容易求和,一个原数列,另一个写成了容易求和的形式(其实也就是原数列),那就是典型的裂项,如果两个来自原数列前后项,另一个容易求和,那就是典型错位相减。 回到原题,我们可以对原先的方法略作修改: $a_n = \dfrac{n^2}{2^n}$ $4a_{n+2}-4a_{n+1} + a_n = \dfrac{(n+2)^2-2(n+1)^2+n^2}{2^n} = \dfrac2{2^n}$ $4S_n+4a_{n+2}+4a_{n+1}-4S_n-4a_2-4a_1-4a_{n+1}+4a_1 + S_n = \dfrac{(n+2)^2-2(n+1)^2+n^2}{2^n} = 2-\dfrac2{2^n}$ $S_n = 2-\dfrac2{2^n}-4a_{n+2}+4a_2=2-\dfrac2{2^n}-\dfrac{(n+2)^2}{2^n}+4=6-\dfrac{n^2+4n+6}{2^n}$ 这比两次错位相减或者两次裂项要爽快,为什么配那几个系数?其实首先是确定要三项,只看分子,两项的加减要同时消掉两次项和一次项,一般是做不到的,三项则一定能做到,所以我们要用 $a_{n+2}$,它的系数,就是将分母凑到和 $a_n$ 相同,此题$a_n$的分子只有两次项,所以 $a_{n+1}$ 的系数,一个因子是凑分母,一个因子用于消一次项,是$2\times2=4$,最后消掉两次项就是等比了。一般点的情况,这就是一次待定系数。当然,如果你确定和的形式,也能直接待定系数并装模作样验证一番,看上去更无赖一些。 以上只是随口说说,平时可以玩玩,不针对考试。考场上,重要的是你得算对,用最稳妥的办法,答案要写的明显让阅卷老师看清楚,免得迷失在你天马行空的变换之中,无法保证成功率的,最好还是老老实实用常规的形式,即使是数学归纳法。如果确实有把握,也要写的清晰,可以不厌其烦的带上n的范围,写写检验什么的,以免阅卷老师思维惯性扣本不该扣的小分,当然,即使你自己觉得做得很完美,仍然可能因阅卷老师思维僵化或者同类卷子批多了扫一眼没看到他想看到的字而扣分,一切都是命啊。
我为中华添光彩 14# 2011-12-26 19:43
佩服! 看来你们都是大师,向你们学习!
战巡 15# 2012-1-8 22:36
........ 我早就在网刊写过这东西了.......... http://www.pep.com.cn/rjwk/gzsxs ... 0110728_1060497.htm
thread-271-1-1.html: 有点难度的积分不等式
icesheep 1# 2011-12-23 23:17
$f\left( x \right) \in {C^2}\left[ { - l,l } \right]$,$f\left( 0 \right) = 0$ 证明: \[{\Bigg(  {\int\limits_{ - l}^l {f\left( x \right){\text{d}}x} }  \Bigg)^2} \leqslant \frac{{{l^5}}}{{10}}\int\limits_{ - l}^l {{{\left( {f''\left( x \right)} \right)}^2}{\text{d}}x} \]
海盗船长 2# 2011-12-24 14:59
吧里的题?
icesheep 3# 2012-8-26 18:24
把这个坑给填了,这种问题一般都是用分部积分和柯西不等式,但是具体 g(x) 不一定很好找 试图找 g(x) 使得 \[\int\limits_{ - l}^l {f{\text{d}}x}  = \int\limits_{ - l}^l {gf''{\text{d}}x}  \leqslant \sqrt {\int\limits_{ - l}^l {{g^2}{\text{d}}x} \int\limits_{ - l}^l {f'{'^2}{\text{d}}x} } \] 然后对这个题来说, \[2g\left( x \right) = \left\{ {\begin{array}{*{20}{c}}   {{{\left( {x - l} \right)}^2},0 \leqslant x \leqslant l} \\   {{{\left( {x + l} \right)}^2}, - l \leqslant x \leqslant 0} \end{array}} \right.\]
六月的煜 4# 2012-10-23 18:15
好吧,终于找到这个积分不等式的答案了,西神出过一道题是l=1的特殊情况,国外一本教材上面也有这道题,不过也是l=1的情况(当时没搞到答案)
thread-272-1-8.html: 请问问1+1/2+1/3+...+1/n的总和怎样求啊?
我为中华添光彩 1# 2011-12-24 11:43
$1+\frac12+\frac13+...+\frac1n=?$ $1+\frac1{2^2}+\frac1{3^2}+...+\frac1{n^2}=?$
kuing 2# 2011-12-24 12:27
FAQ:没有初等求和式
pxchg1200 3# 2011-12-24 12:54
1# 我为中华添光彩 第一个不收敛,没有公式。 第二个如果加到正无穷的话是$ \frac{\pi^{2}}{6}$
kuing 4# 2011-12-24 13:00
其实这几点我都在《数学空间》第一期P13最后面都提过,话说我好像前几天才回复过楼主一个贴子提到过此文,看来楼主没看完。
我为中华添光彩 5# 2011-12-24 13:03
这我也就放心了。 要有学生来问,我就告诉他读到大学时就知道了。
我为中华添光彩 6# 2011-12-24 13:27
4# kuing 难怪您这么厉害,原来您就是专家啊! 谢谢您——郭老师!
kuing 7# 2011-12-24 13:56
我不是老师,更不是专家
海盗船长 8# 2011-12-24 20:36

wenshengli 9# 2011-12-26 21:18
本帖最后由 wenshengli 于 2011-12-26 21:26 编辑 1# 我为中华添光彩 上学时学的数学分析忘了?第一个不收敛。 kuing版说的就是这个
我为中华添光彩 10# 2011-12-26 22:45
9# wenshengli 谢谢楼上! 高等数学确实忘了,毕竟多年不玩这个了。
killshop 11# 2011-12-28 19:00
发散数列  没法求和  见数学分析
战巡 12# 2012-1-8 22:25
本帖最后由 战巡 于 2012-1-8 22:28 编辑 我只知道有2个高等的求和式子 $$\displaystyle1+\frac{1}{2}+\frac{1}{3}+\cdots+\frac{1}{n}=\int_{0}^1\frac{x^n-1}{x-1}dx$$ $$\displaystyle1+\frac{1}{2}+\frac{1}{3}+\cdots+\frac{1}{n}=\frac{\Gamma'(n+1)}{\Gamma(n+1)}+c$$ 其中$\displaystyle{c}=\lim_{n\to\infty}[1+\frac{1}{2}+\cdots+\frac{1}{n}-\ln(n)]$为欧拉常数 证明略,太麻烦了,懒得打........
thread-273-1-1.html: 两个零点
海盗船长 1# 2011-12-24 18:09
已知函数$f(x) \in C [0,\pi]$,且满足\[ \int_{0}^{\pi} f(x) \sin x \mathrm{d} x = \int_{0}^{\pi} f(x) \cos x \mathrm{d} x =0\] 求证:$f(x)$在$(0, \pi)$上至少有两个相异零点。
海盗船长 2# 2011-12-24 19:51
已知函数$f(x) \in C [0,\pi]$,且满足\[ \int_{0}^{\pi} f(x) \mathrm{d} x = \int_{0}^{\pi} f(x) \cos x \mathrm{d} x =0\] 求证:$f(x)$在$(0, \pi)$上至少有两个相异零点。
thread-274-1-8.html: [不等式] 请教如何用基本不等式证明另一个不等式?
我为中华添光彩 1# 2011-12-25 11:34
本帖最后由 我为中华添光彩 于 2011-12-25 11:37 编辑 如何用a+b≥2$\sqrt{ab}$, 证明a+b+c≥3 $\sqrt[3]{abc}$?
kuing 2# 2011-12-25 11:46
照搬反向归纳法证n元均值不等式的思路即可 下列各元均非负 由 $a+b\geqslant 2\sqrt{ab}$ 得 \[a+b+c+d\geqslant 2\sqrt{ab}+2\sqrt{cd}\geqslant 2\sqrt{2\sqrt{ab}2\sqrt{cd}}=4\sqrt[4]{abcd},\] 令 $d=\frac{a+b+c}3$,代入上式得 \begin{align*} a+b+c+\frac{a+b+c}3\geqslant 4\sqrt[4]{abc\frac{a+b+c}3} &\iff \frac{a+b+c}3\geqslant \sqrt[4]{abc\frac{a+b+c}3} \\ & \iff \left( \frac{a+b+c}3 \right)^4\geqslant abc\frac{a+b+c}3 \\ & \iff \left( \frac{a+b+c}3 \right)^3\geqslant abc \\ & \iff a+b+c\geqslant 3\sqrt[3]{abc}. \end{align*}
我为中华添光彩 3# 2011-12-25 12:30
2# kuing 谢谢版主! 请问,对于n个非负数,是否也可以这样证明啊?
我为中华添光彩 4# 2011-12-25 17:58
回版主,我找到了一种  n个 非负值 公式的证明方法。 http://wenku.baidu.com/view/2d95fe3a580216fc700afd10.html
thread-275-1-1.html: 积分
海盗船长 1# 2011-12-25 19:25
求证:\[ \int_{0}^{ \pi } \left( \frac{\sin{\frac{nx}{2}}}{\sin{\frac{x}{2}}} \right)^2 \mathrm{d} x = n\pi\]
战巡 2# 2012-1-7 19:31
唔...上次做出来了,结果你跑掉了........
海盗船长 3# 2012-1-7 23:06
2# 战巡 谢啦,不过好长
thread-276-1-1.html: 多项式
海盗船长 1# 2011-12-26 12:17
$f(x)$为$n$阶多项式,且满足$ \displaystyle \int_{0}^{1} x^kf(x) \mathrm{d} x=0 $  $(k=1,2, \cdots ,n)$ 求证:\[ \int_{0}^{1} f^2(x) \mathrm{d} x =(n+1)^2 \left( \int_{0}^{1} f(x) \mathrm{d} x \right)^2\]
战巡 2# 2012-1-7 21:34

海盗船长 3# 2012-1-7 23:08
2# 战巡 谢啦! 原来这题和积分没多大关系额
kuing 4# 2012-1-7 23:12
后面要证的东西似曾相识...
战巡 5# 2012-1-7 23:22
嘿嘿,就是从上次那题幽雅的崩溃的解法里偷学来的
kuing 6# 2012-1-7 23:24
5# 战巡 嗯,想起来了,楼主也问过的那个题
海盗船长 7# 2012-1-8 11:30
哎,问过之后忘了。。
海盗船长 8# 2012-3-5 13:06
http://www.artofproblemsolving.c ... p?f=67&t=467693
syzychenwj 9# 2012-3-7 08:28
kuing&海盗二位小友能否给个联系方式,以能及时联系帮助我解题,我也喜欢数学,但不太擅长,不知方便否?可能会很烦!我的邮箱zychenwj@163.com,QQ也行、电话更好!
kuing 10# 2012-3-7 12:38
9# syzychenwj 电话的话我会考虑收费
syzychenwj 11# 2012-3-7 14:06
9# syzychenwj 电话的话我会考虑收费 kuing 发表于 2012-3-7 12:38 no any problem
thread-277-1-8.html: [函数] 来自群的二次函数与指数函数最多有3个交点
kuing 1# 2011-12-26 15:35
教师-wwd*************  11:40:46 二次函数与指数函数最多有3个交点? 怎么证明的? 证明 假设 $f(x)=ax^2+bx+c\,(a,b,c\in\mathbf R)$ 与 $g(x)=d^x\,(d>0,d\ne1)$ 存在四个不同的共公点,那么 \[h(x)=f(x)-g(x)=ax^2+bx+c-d^x\] 存在四个不等零点,由 $h(x)$ 的连续性以及罗尔定理知 \[h'(x)=2ax+b-d^x\ln d\] 存在三个不等零点,由 $h'(x)$ 的连续性以及罗尔定理知 \[h''(x)=2a-d^x\ln^2d\] 存在两个不等零点,由 $h''(x)$ 的连续性以及罗尔定理知 \[h'''(x)=-d^x\ln^3d\] 存在零点,这显然不可能。
kuing 2# 2011-12-26 16:39
同理可证 n 次函数与指数函数最多有 n+1 个交点
kuing 3# 2011-12-26 17:23
与对数函数应该也一样
thread-278-1-8.html: [几何] 请教两个立体几何问题
我为中华添光彩 1# 2011-12-27 11:08
1、如何证明:三个平面两两相交,三条交线交于一点或互相平行? 2、若三个平面两两相交,且三条交线互相平行,则这三个平面把空间分成几部分? 答案是8部分,但我不能理解。请大家说说为什么是8部分。
①②③④⑤⑥⑦ 2# 2011-12-27 11:54
1# 我为中华添光彩 第一个证明很简单啊。假如 $l_{\gamma\alpha}$ 与 $l_{\alpha\beta}$ 不平行,由于它们是共面直线,也就是它们有交点 $A$。这样 $A$ 就同时属于这三个平面,作为平面 $\beta$ 与  $\gamma$ 的公共点,必在它们的交线 $l_{\beta\gamma}$ 上。 第二个,哪来的8个部分,只有7个,除非三个平面合在一起算一个“部分”
①②③④⑤⑥⑦ 3# 2011-12-28 14:56
3# 我为中华添光彩 那就想想最简单的,空间直角坐标系
我为中华添光彩 4# 2011-12-28 17:06
4# ①②③④⑤⑥⑦ 谢谢你! 不过第一个证明,我有点看不明白。 “假如 lγα 与 lαβ 不平行,由于它们是共面直线,也就是它们有交点 A。这样 A 就同时属于这三个平面,”到这里都好懂。 后面就不太好理解了:“作为平面 β 与 γ 的公共点,必在它们的交线 lβγ 上。” “A”为什么一定要在它们的交线 lβγ 上呢?
①②③④⑤⑥⑦ 5# 2011-12-28 17:49
5# 我为中华添光彩 不然的话,直线和直线外一点确定一个平面,$A$ 与 $l_{\beta\gamma}$ 都是 平面 $\beta$ 和 $\gamma$ 的公共元素,所以 $\beta$ 与 $\gamma$ 重合,这不符合题设(应该默认是两两不重合的平面)
thread-279-1-1.html: 级数收敛
海盗船长 1# 2011-12-27 17:48
本帖最后由 海盗船长 于 2011-12-27 17:49 编辑 已知$\displaystyle \sum_{n=1}^{+\infty} na_{n}$收敛,求证:\[ \lim_{n \to +\infty} n\sum_{i=1}^{+\infty}a_{n+i}=0 \]
icesheep 2# 2012-1-14 08:24
本帖最后由 icesheep 于 2012-1-14 08:36 编辑 记 ${b_n} = n{a_n}$ ,由 Abel 求和公式 \[\begin{align}\sum\limits_{i = 1}^\infty  {\frac{{{b_{n + i}}}}{{n + i}}}  &= \frac{{{B_{n + 1}} - {B_n}}}{{n + 1}} + \frac{{{B_{n + 2}} - {B_{n + 1}}}}{{n + 2}}+ ... \\&=  - \frac{{{B_n}}}{{n + 1}}+\frac{{{B_{n+1}}}}{{\left( {n + 1} \right)\left( {n + 2} \right)}} + \frac{{{B_{n+2}}}}{{\left( {n + 2} \right)\left( {n + 3} \right)}} + ...\end{align}\] 所以这里是不是有点问题??
海盗船长 3# 2012-1-14 14:42
2# icesheep 会不会后面的正项与第一项抵消,最后还是趋于"0"?
icesheep 4# 2012-1-16 14:38
3# 海盗船长 你是对的。。。还是做不出。。。
Nirvanacs 5# 2012-1-16 21:05
额,还在讨论这个,上次提示没做出来吗.利用Cauchy收敛原理加Abel求和,证明任取\(\varepsilon>0\),存在\(N\)使得\(n>N\),都有那个式子\(<\varepsilon\).
海盗船长 6# 2012-1-16 21:57
5# Nirvanacs 提示了还是不会。。
Nirvanacs 7# 2012-1-17 12:26
本帖最后由 Nirvanacs 于 2012-1-17 12:38 编辑 6# 海盗船长 因为\[n\sum_{i=1}^{+\infty}a_{n+i}=\sum_{i=1}^{+\infty}\frac{n}{n+i}(n+i)a_{n+i},\]因此由A-D判别法知,这个式子是收敛的. 任取\(\varepsilon>0\),则由\(\displaystyle \sum_{i=1}ia_i\)收敛知,存在\(N\in \mathbb{Z}^{+}\)使得任取\(m>n>N\)都有\[\left|\sum_{k=n}^m ka_k\right|<\varepsilon\]此时由Abel求和法知,对任意的\(n>N\)及任意的正整数\(m\)有\[\begin{aligned}\left|n\sum_{i=1}^{m}a_{n+i}\right|=&\left|\sum_{i=1}^{m-1}\left[\left(\sum_{k=1}^i (n+k)a_{n+k}\right)\left(\frac{n}{n+i}-\frac{n}{n+i+1}\right)\right]+\sum_{i=1}^m\frac{n}{n+m}(n+i)a_{n+i} \right|\\ \le &\sum_{i=1}^{m}\left[\left|\sum_{k=1}^i (n+k)a_{n+k}\right|\left(\frac{n}{n+i}-\frac{n}{n+i+1}\right)\right]+\left|\sum_{i=1}^m\frac{n}{n+m}(n+i)a_{n+i}\right|\\ \le & \sum_{i=1}^{m-1}\left[\varepsilon\left(\frac{n}{n+i}-\frac{n}{n+i+1}\right)\right]+\frac{n}{n+m}\varepsilon\\ =&\varepsilon\left(\frac{n}{n+1}-\frac{n}{n+m}\right)+\frac{n}{n+m}\varepsilon\\ \le & \varepsilon \end{aligned}\]
海盗船长 8# 2012-1-17 20:22
7# Nirvanacs 懂了,谢啦!
icesheep 9# 2012-1-19 17:15
路萝框好喜欢 epsilon-delta 语言 - - 我总想着用点什么技巧以后就可以直接套定理了
thread-280-1-1.html: 级数收敛性
icesheep 1# 2011-12-27 19:09
正项级数 $\sum\limits_{n = 1}^\infty  {{a_n}} $ 收敛 , 证明: $ \sum\limits_{n = 1}^\infty  {a_n^{1 - 1/n}} $ 收敛
海盗船长 2# 2011-12-28 12:25
本帖最后由 海盗船长 于 2011-12-28 13:02 编辑 能不能这样? 1.如果 $\displaystyle \lim_{n \to \infty} \inf{\sqrt[n]{a_n}} \neq 0$ $\displaystyle \lim_{n \to \infty}\sup \frac{a_n}{\sqrt[n]{a_n}}\frac{1}{a_n}=\lim_{n \to \infty}\sup\frac{1}{\sqrt[n]{a_n}}=\frac{1}{\displaystyle \lim_{n \to \infty} \inf{\sqrt[n]{a_n}} }$ 然后 $\displaystyle 0<\lim_{n \to \infty} \inf{\sqrt[n]{a_n}}\le \lim_{n \to \infty}\sup{\sqrt[n]{a_n}}\le 1$ 由比较判别法知其收敛 2.如果 $\displaystyle \lim_{n \to \infty} \inf{\sqrt[n]{a_n}} = 0$ 然后不会了。。。
tian27546 3# 2011-12-28 16:00
本帖最后由 tian27546 于 2011-12-28 16:09 编辑 由Young不等式得 $ \sqrt[n]{\frac{1}{n}}*{a_{n}}^{1-\frac{1}{n}}\le \frac{1}{n^2}+(1-\frac{1}{n})a_{n}$ 结合 $\sqrt[n]{n}\le \sqrt[e]{e}$,以及$\displaystyle\sum_{n=1}^{\infty}\frac{a_{n}}{n}$收敛。
tian27546 4# 2011-12-28 16:14
本帖最后由 tian27546 于 2011-12-28 16:19 编辑 或者这样: 如果$a_{n}\le \frac{1}{2^n}$,则${a_{n}}^{\frac{n-1}{n}}\le \frac{1}{2^{n-1}}$ 若$ a_{n}>\frac{1}{2^n}$,则 $(a_{n})^{1-\frac{1}{n}}<2a_{n}$
icesheep 5# 2012-1-1 00:22
哈,这做法不错: \[a_n^{1 - 1/n} \leqslant {\left( {{a_n} + \frac{1}{{{n^2}}}} \right)^{1 - 1/n}} = \left( {{a_n} + \frac{1}{{{n^2}}}} \right)\sqrt[n]{{{a_n} + \frac{1}{{{n^2}}}}} \leqslant \left( {{a_n} + \frac{1}{{{n^2}}}} \right)\frac{1}{{\sqrt[n]{{{n^2}}}}}\]
海盗船长 6# 2012-1-4 12:25
原来要用不等式
thread-281-1-7.html: [不等式] 正数$x+y+z=1$证$\sum(2x-x^2)/(2x^2-2x+1)\leqslant3$
kuing 1# 2011-12-29 15:47
已知 $x, y, z>0$ 且 $x+y+z=1 $,求证 \[\frac{2x- x^2}{2x^2-2x+1}+\frac{2y- y^2}{2y^2-2y+1}+\frac{2z- z^2}{2z^2-2z+1}\leqslant3.\]
我为中华添光彩 2# 2011-12-31 12:00
1# kuing 这个不等式看起来简单,可做不起来。 看来,我还得不断学习啊。
pxchg1200 3# 2011-12-31 16:36
2# 我为中华添光彩 我也表示不会用Cauchy-Schwarz证。但SOS还是可以的。。
kuing 4# 2012-1-4 13:14
3# pxchg1200 嗯,懒一点的话用 Schur 分拆(齐次化之后)也可以
pxchg1200 5# 2012-1-17 10:32
4# kuing kk,你说有这个么? \[   \frac{2x-x^{2}}{2x^{2}-2x+1}\leq 3x \]
kuing 6# 2012-1-17 13:50
5# pxchg1200 有的话我就不发上来哩
pxchg1200 7# 2012-1-17 21:19
6# kuing 好吧,看见Can 贴解答了。转载下: We will consider two cases: Case $\min\{x,\,y,\, z \} \ge \frac{1}{4}.$ It is easy to check that for any $a \ge \frac{1}{4},$ we have \[\frac{2a-a^2}{2a^2-2a+1} \le \frac{18a-1}{5}.\] (After expanding, it is equivalent to $(4a-1)(3a-1)^2 \ge 0,$ which is obvious.) So, \[ \frac{2x-x^{2}}{2x^{2}-2x+1}+\frac{2y-y^{2}}{2y^{2}-2y+1}+\frac{2z-z^{2}}{2z^{2}-2z+1} \le \frac{18x-1}{5}+\frac{18y-1}{5}+\frac{18z-1}{5} =3.\] Case $\min\{x,\,y,\,z\} <\frac{1}{4}.$ Without loss of generality, assume that $x<\frac{1}{4}.$ Since $2y-y^2 \ge 0$ and $2y^2-2y+1 =2\left(y-\frac{1}{2}\right)^2+\frac{1}{2} \ge \frac{1}{2},$ we have \[\frac{2y-y^2}{2y^2-2y+1} \le 2(2y-y^2).\] Similarly, we also have \[\frac{2z-z^2}{2z^2-2z+1} \le 2(2z-z^2).\] Using these two inequalities in combination with the Cauchy-Schwarz inequality, we get \[\begin{aligned} \frac{2y-y^2}{2y^2-2y+1}+\frac{2z-z^2}{2z^2-2z+1}& \le 4(y+z) -2(y^2+z^2) \le 4(y+z)-(y+z)^2\\ &=4(1-x)-(1-x)^2=3-2x-x^2.\end{aligned}\] Therefore, it suffices to prove that \[\frac{2x-x^2}{2x^2-2x+1} \le 2x+x^2.\] After expanding, it becomes \[x^2(1-x-x^2) \ge 0,\] which is true because $x^2+x< \frac{1}{16}+\frac{1}{4}<1.$ The proof is completed. $\blacksquare$
kuing 8# 2012-1-17 21:23
原来是分类讨论用切线法 我当时也想过,可惜<1/4时证不出来
kuing 9# 2012-4-11 17:01
http://www.artofproblemsolving.c ... p?f=52&t=458507 这贴里好像说上面那个证法错了
kuing 10# 2012-4-13 01:31
For the first one, I notice that (it is easy to find this identity) \[ {\sum\frac{2x-x^{2}}{2x^{2}-2x+1}-3 =\frac{4(x-y)^{2}(y-z)^{2}(z-x)^{2}}{\prod (2x^{2}-2x+1)}-\sum\frac{(y-z)^{2}(y+z-x)^{2}}{(2y^{2}-2y+1)(2z^{2}-2z+1)},} \] 好一个 easy to find …………
pxchg1200 11# 2012-4-13 22:15
10# kuing 他说他一看就知道是这个。。。   汗。
thread-282-1-8.html: [函数] 过一点作曲线的切线,求m的取值范围
我为中华添光彩 1# 2011-12-29 21:12
若过A(1,m)(m≠-2)可作曲线f(x)=$x^3$-3x的三条切线,求实数m的取值范围。
我为中华添光彩 2# 2011-12-30 11:33
本帖最后由 我为中华添光彩 于 2011-12-30 19:27 编辑 根据导数的切线的性质转化为: $2a^3-3a^2+m+3=0$有三个实数根,求实数m的取值范围。
realnumber 3# 2011-12-30 13:24
$y=f(a)=2a^3-3a^2+3=-m$ 图象,问题即为f极小值<-m<f极大值
我为中华添光彩 4# 2011-12-30 19:26
3# realnumber 谢谢楼上!
thread-283-1-1.html: 积分$\int(1+x^2)/(1+x^4)\rmd x$之类的
kuing 1# 2012-1-4 16:13
\begin{align*} \frac{1+x^{2}}{1+x^{4}}&=\frac{1+x^{2}}{(1+x^{2})^{2}-2x^{2}} \\ & =\frac{1+x^{2}}{\bigl(1+x^{2}+\sqrt{2}x\bigr)\bigl(1+x^{2}-\sqrt{2}x\bigr)} \\ & =\frac{1+x^{2}+\sqrt{2}x+1+x^{2}-\sqrt{2}x}{2\bigl(1+x^{2}+\sqrt{2}x\bigr)\bigl(1+x^{2}-\sqrt{2}x\bigr)} \\ & =\frac{1}{2}\left( \frac{1}{1+x^{2}+\sqrt{2}x}+\frac{1}{1+x^{2}-\sqrt{2}x} \right) \\ & =\frac{1}{2}\left( \frac{1}{\frac{1}{2}+\left( x+\frac{1}{\sqrt{2}} \right)^{2}}+\frac{1}{\frac{1}{2}+\left( x-\frac{1}{\sqrt{2}} \right)^{2}} \right) \end{align*} 故 \begin{align*} \int{\frac{1+x^{2}}{1+x^{4}}\text{d}x}&=\frac{1}{2}\int{\frac{1}{\frac{1}{2}+\left( x+\frac{1}{\sqrt{2}} \right)^{2}}\text{d}x}+\frac{1}{2}\int{\frac{1}{\frac{1}{2}+\left( x-\frac{1}{\sqrt{2}} \right)^{2}}\text{d}x} \\ & =\frac{1}{2}\int{\frac{1}{\left( \frac{1}{\sqrt{2}} \right)^{2}+\left( x+\frac{1}{\sqrt{2}} \right)^{2}}\text{d}\left( x+\frac{1}{\sqrt{2}} \right)}+\frac{1}{2}\int{\frac{1}{\left( \frac{1}{\sqrt{2}} \right)^{2}+\left( x-\frac{1}{\sqrt{2}} \right)^{2}}\text{d}\left( x-\frac{1}{\sqrt{2}} \right)} \\ & =\frac{1}{\sqrt{2}}\arctan \bigl(\sqrt{2}x+1\bigr)+\frac{1}{\sqrt{2}}\arctan \bigl(\sqrt{2}x-1\bigr)+C \end{align*}
海盗船长 2# 2012-1-6 16:30
上下同除$x^2$后凑微分也可以
$\LaTeX$ 3# 2012-2-8 14:36
三角换元可不可以
$TeX$ 4# 2012-2-9 20:25
好像不行
$\LaTeX$ 5# 2012-2-9 21:06
4# $TeX$ 你ID怎么不加个 \ 在前面
$TeX$ 6# 2012-2-9 21:09
5# $\LaTeX$ 忘了。。
kuing 7# 2012-4-5 23:45
今晚tutu又问了一道类似,求 \[\int\frac{\sqrt x}{1+x^2}\text dx.\] 令$t=\sqrt x$,则 \[\int\frac{\sqrt x}{1+x^2}\text dx = \int\frac{t}{1+t^4}\text dt^2=2\int\frac{t^2}{1+t^4}\text dt\] 于是只要求 \[\int\frac{t^2}{1+t^4}\text dt\] 为方便复制上面,这里暂时将t写回x \[\int\frac{x^2}{1+x^4}\text dx\] 这就跟上面的只差了一个$1$,故仍仿上,有 \begin{align*} \frac{x^2}{1+x^{4}}&=\frac{x^{2}}{(1+x^{2})^{2}-2x^{2}} \\   & =\frac{x^{2}}{\bigl(1+x^{2}+\sqrt{2}x\bigr)\bigl(1+x^{2}-\sqrt{2}x\bigr)} \\   & =\frac{x\bigl(1+x^{2}+\sqrt{2}x\bigr)-x\bigl(1+x^{2}-\sqrt{2}x\bigr)}{2\sqrt2\bigl(1+x^{2}+\sqrt{2}x\bigr)\bigl(1+x^{2}-\sqrt{2}x\bigr)} \\   & =\frac{1}{2\sqrt2}\left( \frac{x}{1+x^{2}-\sqrt{2}x}-\frac{x}{1+x^{2}+\sqrt{2}x} \right) , \end{align*} 而$(Mx+N)/(x^2+px+q)$的积分是有公式的,只是我不记得,好像挺长,结果估计比较那个……
kuing 8# 2012-12-7 02:49
晚上看群里提到 $\int\frac1{x^8+1}\rmd x$,想用类似方法玩玩看,结果超级麻烦…… 暂且不考虑定义域什么的问题了,直接令 $x=1/t$,则 \[\int\frac1{x^8+1}\rmd x=\int\frac{t^8}{t^8+1}\rmd{\frac1t}=-\int\frac{t^6}{t^8+1}\rmd t,\] 而 \begin{align*} \frac{t^6}{t^8+1}={}&\frac{t^6}{(t^4+1)^2-2t^4} \\ ={}&\frac{t^6}{(t^4+1+\sqrt2t^2)(t^4+1-\sqrt2t^2)} \\ ={}&\frac1{2\sqrt2}\left( \frac{t^4}{t^4+1-\sqrt2t^2}-\frac{t^4}{t^4+1+\sqrt2t^2} \right) \\ ={}&\frac1{2\sqrt2}\left( \frac{t^4}{(t^2+1)^2-(2+\sqrt2)t^2}-\frac{t^4}{(t^2+1)^2-(2-\sqrt2)t^2} \right) \\ ={}&\frac1{2\sqrt2}\Biggl( \frac{t^4}{(t^2+1+\sqrt{2+\sqrt2}t)(t^2+1-\sqrt{2+\sqrt2}t)}\\ &-\frac{t^4}{(t^2+1+\sqrt{2-\sqrt2}t)(t^2+1-\sqrt{2-\sqrt2}t)} \Biggr) \\ ={}&\frac1{4\sqrt2\sqrt{2+\sqrt2}}\left( \frac{t^3}{t^2+1-\sqrt{2+\sqrt2}t}-\frac{t^3}{t^2+1+\sqrt{2+\sqrt2}t} \right) \\ & -\frac1{4\sqrt2\sqrt{2-\sqrt2}}\left( \frac{t^3}{t^2+1-\sqrt{2-\sqrt2}t}-\frac{t^3}{t^2+1+\sqrt{2-\sqrt2}t} \right), \end{align*} 再利用 \[\frac{t^3}{t^2+1+pt}=\frac{(p^2-1) t+p}{t^2+1+pt}+t-p,\] 便得到 \begin{align*} \frac{t^6}{t^8+1}={}&\frac1{4\sqrt2\sqrt{2+\sqrt2}}\left( \frac{(1+\sqrt2)t-\sqrt{2+\sqrt2}}{t^2+1-\sqrt{2+\sqrt2}t}-\frac{(1+\sqrt2)t+\sqrt{2+\sqrt2}}{t^2+1+\sqrt{2+\sqrt2}t}+2\sqrt{2+\sqrt2} \right) \\ & -\frac1{4\sqrt2\sqrt{2-\sqrt2}}\left( \frac{(1-\sqrt2)t-\sqrt{2-\sqrt2}}{t^2+1-\sqrt{2-\sqrt2}t}-\frac{(1-\sqrt2)t+\sqrt{2-\sqrt2}}{t^2+1+\sqrt{2-\sqrt2}t}+2\sqrt{2-\sqrt2} \right), \end{align*} 然后就可以代公式了,最终结果……算了吧……闪人
q85669551 9# 2012-12-7 13:03
8# kuing   反正算起来好麻烦。。不过理论上是可行的
kuing 10# 2012-12-7 15:32
9# q85669551 怎么了…… 话说将那些根号写成三角函数可能看上去简洁一些……$\sqrt{2+\sqrt2}=2\cos(\pi/8)$,$\sqrt{2-\sqrt2}=2\sin(\pi/8)$
都市侠影 11# 2012-12-7 20:44
本帖最后由 都市侠影 于 2012-12-7 20:49 编辑 似乎我回过这个积分,是一个典型的凑微分的: \[ \int{\frac{1+x^2}{1+x^4}}=\int{\frac{d(x-\frac{1}{x})}{(x-\frac{1}{x})^2+2}} \ = \int{\frac{dz}{z^2+2}} \]
q85669551 12# 2012-12-8 00:35
10# kuing 难算啊。。改成2^n的情形 似乎会出现什么sin((n-1)pi/2^n)之类的数字...
kuing 13# 2012-12-8 00:45
还2^n......有木有递推神马的?
q85669551 14# 2012-12-8 11:21
13# kuing 估计会有表达式... 可以用软件算了 发现规律
reny 15# 2012-12-8 20:04
关键就是因式分解为常见可积基本函数。过程挺复杂啊
realnumber 16# 2012-12-11 08:20
开始自学数学分析?
kuing 17# 2012-12-11 08:25
16# realnumber 偶尔路过玩玩
thread-285-1-8.html: 因式分解
jqjbacon 1# 2012-1-4 18:58
因式分解:$a^3b-ab^3+a^2+b^2+1$
kuing 2# 2012-1-4 22:23
\begin{align*} a^3b-ab^3+a^2+b^2+1&=ab(a-b)(a+b)+a^2+b^2+1\\ &=(a^2-ab)(b^2+ab)+a^2+b^2+1\\ &=(a^2-ab)(b^2+ab)+a^2-ab+b^2+ab+1\\ &=(a^2-ab+1)(b^2+ab+1) \end{align*}
jqjbacon 3# 2012-1-5 08:22
2# kuing 感谢kuing!
thread-286-1-1.html: 在人人上看到的一积分题
kuing 1# 2012-1-4 23:48
设 $f(x)$ 在 $[0,1]$ 上具有连续导数,且 $f(0)=0$。证明:存在 $\xi\in[0,1]$ 使 \[\int_0^1f(x)\text{d}x=\frac12f'(\xi).\] 这里干脆将把上限变一下,一般地,对于正常数 $k$,构造 \[ g(t)=\int_0^t{f(x)\text{d}x}-\frac1{k^2}\int_0^k{f(x)\text{d}x}\int_0^t{2x\text{d}x}, \] 则 \[g'(t)=f(t)-\frac1{k^2}\int_0^k{f(x)\text{d}x}\cdot2t.\] 显然 $g(0)=0$,且 \[ g(k)=\int_0^k{f(x)\text{d}x}-\frac1{k^2}\int_0^k{f(x)\text{d}x}\int_0^k{2x\text{d}x}=0, \] 即 $g(0)=g(k)=0$,故存在 $\xi_1\in(0,k)$ 使 $g'(\xi_1)=0$,即 \[f(\xi_1)-\frac1{k^2}\int_0^k{f(x)\text{d}x}\cdot2\xi_1 =0,\] 由此结合 $f(0)=0$,得到 \[ \int_0^k{f(x)\text{d}x}=\frac{k^2}2\cdot\frac{f(\xi_1)-f(0)}{\xi_1-0}, \] 从而存在 $\xi\in(0,\xi_1)\subsetneqq (0,k)$ 使 \[\int_0^k{f(x)\text{d}x}=\frac{k^2}2f'(\xi).\] 当 $k=1$ 时为上题。
海盗船长 2# 2012-1-6 16:34
这个用泰勒也可以吧
thread-287-1-8.html: [函数] totutu三角函数n阶导数
kuing 1# 2012-1-6 15:00
\begin{align*} (\sin ax)^{(0)}&=\sin ax,\\ (\sin ax)^{(1)}&=a\cos ax=a\sin\left(ax+\frac\pi2\right),\\ (\sin ax)^{(2)}&=-a^2\sin ax=a^2\sin(ax+\pi),\\ (\sin ax)^{(3)}&=-a^3\cos ax=a^3\sin\left(ax+\frac{3\pi}2\right),\\ (\sin ax)^{(4)}&=a^4\sin ax=a^2\sin(ax+2\pi),\\ \ldots&\ldots \end{align*} 归纳出 \begin{equation} (\sin ax)^{(n)}=a^n\sin\left(ax+\frac{n\pi}2\right). \end{equation} 用数学归纳法证之,假设 $n=k$ 成立那么 $n=k+1$ 时 \begin{align*} (\sin ax)^{(k+1)}&=\left(a^k\sin\left(ax+\frac{k\pi}2\right)\right)'\\ &=a^k\cdot a\cos\left(ax+\frac{k\pi}2\right)\\ &=a^{k+1}\sin\left(ax+\frac{k\pi}2+\frac\pi2\right)\\ &=a^{k+1}\sin\left(ax+\frac{(k+1)\pi}2\right), \end{align*} 即得证。由此亦可得 cos 的 \[(\cos ax)^{(n)}=\left(\sin\left(ax+\frac\pi2\right)\right)^{(n)} =a^n\sin\left(ax+\frac\pi2+\frac{n\pi}2\right) =a^n\cos\left(ax+\frac{n\pi}2\right).\]
kuing 2# 2012-1-6 15:35
计算 $(\sin^2x)^{(n)}$ \[(\sin^2x)^{(n)}=\left(\frac{1-\cos2x}2\right)^{(n)}=-\frac12(\cos2x)^{(n)} =-\frac12\cdot2^n\cos\left(2x+\frac{n\pi}2\right)=-2^{n-1}\cos\left(2x+\frac{n\pi}2\right).\] 或 \begin{align*} (\sin^2x)^{(n)}&=(\sin x\cdot\sin x)^{(n)}\\ &=\sum_{k=0}^n C_n^k(\sin x)^{(k)}(\sin x)^{(n-k)}\\ &=\sum_{k=0}^n C_n^k\sin\left(x+\frac{k\pi}2\right)\sin\left(x+\frac{(n-k)\pi}2\right)\\ &=\sum_{k=0}^n \frac{C_n^k}2\left(\cos\left(\frac{k\pi}2-\frac{(n-k)\pi}2\right) -\cos\left(2x+\frac{n\pi}2\right)\right)\\ &=\sum_{k=0}^n \frac{C_n^k}2\left((-1)^k\cos\frac{n\pi}2-\cos\left(2x+\frac{n\pi}2\right)\right)\\ &=\frac{\cos\frac{n\pi}2}2\sum_{k=0}^n C_n^k(-1)^k - \frac{\cos\left(2x+\frac{n\pi}2\right)}2\sum_{k=0}^n C_n^k\\ &=-\frac{\cos\left(2x+\frac{n\pi}2\right)}2\cdot2^n\\ &=-2^{n-1}\cos\left(2x+\frac{n\pi}2\right). \end{align*}
thread-288-1-1.html: 在数学旅程QQ群看到的一题
kuing 1# 2012-1-6 17:39
顾*(1036******) 14:35:36 f(x)=int(cos(1/t),t=0..x);求f'(0) 题目:设 $f(x)=\int_0^x{\cos\frac1t\rmd t}$,求 $f'(0)$。 我们先看 $x>0$ 时,令 $t=1/u$,则 \begin{align*} \int_0^x\cos\frac1t\rmd t&=\int_{+\infty }^{1/x}\cos u\rmd{\frac1u} \\ &=-\int_{1/x}^{+\infty }\cos u\rmd{\frac1u} \\ & =\int_{1/x}^{+\infty }\frac{\cos u}{u^2}\rmd u \\ & =\int_{1/x}^{+\infty }\frac{\rmd{\sin u}}{u^2} \\ & = \left. \frac{\sin u}{u^2} \right|_{1/x}^{+\infty } - \int_{1/x}^{+\infty }\sin u\rmd{\frac1{u^2}}\\ & =2\int_{1/x}^{+\infty }\frac{\sin u}{u^3}\rmd u - x^2\sin \frac1x, \end{align*} 当 $x<0$ 时,利用上述结果,有 \begin{align*} \int_0^x\cos \frac1t\rmd t &=-\int_0^{-x}\cos \frac1t\rmd t\\ &=-2\int_{1/(-x)}^{+\infty }\frac{\sin u}{u^3}\rmd u+(-x)^2\sin \frac1{-x}\\ &=-2\int_{1/(-x)}^{+\infty }\frac{\sin u}{u^3}\rmd u-x^2\sin \frac1x, \end{align*} 于是综合两种情况,当 $x\ne0$ 时我们有 \begin{align*} \int_0^x\cos \frac1t\rmd t&=\frac{x}{\abs{x}}\cdot 2\int_{1/\abs{x}}^{+\infty }\frac{\sin u}{u^3}\rmd u-x^2\sin \frac1x\\ &=x\left(\frac2{\abs{x}}\int_{1/\abs{x}}^{+\infty }\frac{\sin u}{u^3}\rmd u-x\sin \frac1x \right), \end{align*} 因此 \begin{equation} \frac{\int_0^x\cos \frac1t\rmd t}{x}=\frac2{\abs{x}}\int_{1/\abs{x}}^{+\infty }\frac{\sin u}{u^3}\rmd u-x\sin \frac1x, \end{equation} 由此可得 \begin{align*} \left| \frac{\int_0^x\cos \frac1t\rmd t}{x} \right|&=\left|\frac2{\abs{x}}\int_{1/\abs{x}}^{+\infty }\frac{\sin u}{u^3}\rmd u-x\sin \frac1x \right| \\ & \leqslant \frac2{\abs{x}}\left| \int_{1/\abs{x}}^{+\infty }\frac{\sin u}{u^3}\rmd u \right|+\left| x\sin \frac1x \right| \\ & \leqslant \frac2{\abs{x}}\int_{1/\abs{x}}^{+\infty }\frac{\abs{\sin u}}{u^3}\rmd u+\left| x\sin \frac1x \right| \\ & \leqslant \frac2{\abs{x}}\int_{1/\abs{x}}^{+\infty }\frac1{u^3}\rmd u+\left| x\sin \frac1x \right| \\ & =\frac2{\abs{x}}\cdot \left. \left( -\frac1{2u^2} \right) \right|_{1/\abs{x}}^{+\infty }+\left| x\sin \frac1x \right| \\ & =\frac2{\abs{x}}\cdot \frac{x^2}2+\left| x\sin \frac1x \right| \\ & =\abs{x}+\left| x\sin \frac1x \right|, \end{align*} 即 \begin{equation} 0<\left| \frac{\int_0^x\cos \frac1t\rmd t}{x} \right|\leqslant \abs{x}+\left| x\sin \frac1x \right|, \end{equation} 注意到 \[ \lim_{x\to0}\left( \abs{x}+\left| x\sin \frac1x \right| \right)=0, \] 于是由夹逼定理得 \[\lim_{x\to0}\frac{\int_0^x\cos \frac1t\rmd t}{x}=0,\] 从而 \[f'(0)=\lim_{x\to0}\frac{f(x)-f(0)}{x-0}=\lim_{x\to0}\frac{\int_0^x\cos \frac1t\rmd t}{x}=0.\]
kuing 2# 2012-1-12 22:16
咦?我明明回了一个贴怎么没了…………
kuing 3# 2012-1-12 22:21
重新回一次 提问的后来发了下面这个anser 没讨论x>0与<0,不需要吗? 有没有问题?求牛银鉴一鉴ding
thread-29-1-1.html: 动态图演示
kuing 1# 2011-9-28 14:03
行内公式:将公式代码放入两个美元符号中,例:$ax^2+bx+c=0$ 行间公式:将美元符号改一改即可,例:\[ax^2+bx+c=0\] 复杂一点的公式,比如这个方程的根:\[x_{1,2}=\frac{-b\pm\sqrt{b^2-4ac}}{2a}\]
thread-290-1-1.html: 求证存在$f(\xi)+f'(\xi)=0$
kuing 1# 2012-1-7 22:30
设$f(x)$在$[0,1]$上连续,在$(0,1)$内可导,且 \[f(1)=2\int_{0}^{\frac12}e^{x-1}f(x)dx,\] 证明至少存在一点$\xi\in(0,1)$使得$f'(\xi)+f(\xi)=0$。
海盗船长 2# 2012-1-7 23:02
本帖最后由 海盗船长 于 2012-1-7 23:04 编辑 $ef(1)=2 \int_{0}^{\frac{1}{2}} e^x f(x) \mathrm{d} x$ 由积分中值定理得存在$\xi' \in (0,\frac{1}{2}) $使得 $e f(1)=e^{\xi'} f(\xi') $ 设$g(x)=e^x f(x)$ 由罗尔中值定理知存在$\xi \in (\xi',1) $使得 $f(\xi)+f'(\xi)=0$
kuing 3# 2012-1-7 23:15
秒得好快
战巡 4# 2012-1-7 23:19
本帖最后由 战巡 于 2012-1-7 23:21 编辑 其实貌似换成$$f(1)=n\int_{0}^{\frac{1}{n}}e^{x-1}f(x)dx$$都行 令$$g(x)=e^{x-1}f(x)$$ 则$$n\int_{0}^{\frac{1}{n}}e^{x-1}f(x)dx=n\int_{0}^{\frac{1}{n}}g(x)dx$$ $$=\int_{0}^{1}g(\frac{x}{n})dx$$ 由积分中值定理知必然存在$$\xi\in(0,1)$$,使得$$g(\frac{\xi}{n})=\int_{0}^{1}g(\frac{x}{n})dx$$ 那么显然$$\frac{\xi}{n}\in(0,\frac{1}{n})$$,且有$$g(\frac{\xi}{n})=g(1)$$ 由罗尔定理得存在$$\xi'\in(\frac{\xi}{n},1)$$使得$$g'(x)=e^{x-1}(f'(x)+f(x))=0$$
kuing 5# 2012-1-7 23:29
战巡的也OK 公式输入的PS:大公式需要显示在中间的用两个美元,小的公式,比如 \xi\in(0,1) 这种小东东在行内的就用一个美元可以了 再PS一个:后面写 $g'(\xi')=e^{x-1}(f'(\xi')+f(\xi'))=0$ 比较好
thread-291-1-8.html: 给个有意思的问题大家玩玩吧
战巡 1# 2012-1-8 15:41
应该算博弈论之类的题,反正是听别人说的 现在有n个人(n>1),每个人都要从1到k(k为大于1的任意整数)中写下一个整数,他们写的时候是独立的,不能与其他人交流,最后将他们写的所有数求平均值,再乘以(2/3),然后写下的数与这个值相差最小者获胜(有相同的则同时获胜)。这些人都极其聪明且绝对理性,并事先知道规则,他们会怎样写数字以求自己最有可能胜利呢?
海盗船长 2# 2012-1-9 20:58
都写"1"?猜的。。
战巡 3# 2012-1-9 21:51
都写"1"?猜的。。 海盗船长 发表于 2012-1-9 20:58 呵,猜对了,不妨好好想想为什么~~
海盗船长 4# 2012-1-10 11:45
3# 战巡 我这样想的: 由于对称性,所有人都会填一样的数,设其为$m (m \in \{ 1,2,\cdots ,k \})$。但是这样他们就都应该填与$\frac{2}{3}m$最接近的数。假设$m>1$,那么与$\frac{2}{3}m$最接近的数就要小于$m$,只有$m=1$才能满足。
海盗船长 5# 2012-1-10 14:18
本帖最后由 海盗船长 于 2012-1-10 14:21 编辑 如果系数改成$\frac{6}{7}$怎么办? $ \left\{\frac{6}{7} \right\}=1$、 $\left\{ 2*\frac{6}{7} \right\}=2 $ 、 $\left\{ 3*\frac{6}{7} \right\}=3 $ ($\{x\}$表示与$x$最接近的数)
thread-292-1-8.html: 有意思的题
海盗船长 1# 2012-1-10 16:00
有一个正方形的房间,房间的四壁都是镜子。房间里有一个天使和一个恶魔。假设房间是一个单位正方形 [0, 1] × [0, 1] ,那么天使和恶魔便是这个正方形内的两个点 (a, b) 和 (c, d) 。恶魔想要在原地发射致命激光杀死天使(激光可以无限地在镜子间反射)。天使可以根据恶魔的位置,预先在房间里放置一些守卫为自己挡住激光(守卫实际上也是一个个点)。当然,天使可以在自己周围密密麻麻地放一圈守卫,围成一个封闭的圆形,从而让恶魔不管朝什么方向发射激光,最终都无法击中天使。我们的问题是,能把守卫的数量减少到可数个点吗?能把守卫的数量减少到有限个点吗? qq上看到的
战巡 2# 2012-1-10 17:55
可数应该是可以的,如图,恶魔处于B点,天使A点,那么A1到A8为天使镜像(当然,不止这么点,无穷多个,向外还可以无限延伸,但每个区里面只有一个镜像,而且位置是确定的,也就是可数的),在没有阻拦的情况下,恶魔朝着天使本身或任意一个镜像开火,都能打到天使,天使就需要在自身以及每个镜像的开火路径上放个守卫阻拦火力 但似乎要想有限是不可能的,毕竟镜像有无穷多个,路径也有无穷多条,即便天使在墙角都不行,虽然此时大部分镜像重合了,但镜像还是有无穷多个........ 唯一的情况就是天使与恶魔重合(好像太极端了),这时放4个就够了(假设恶魔此时不能直接射击天使,因为它不能射自己).....
thread-293-1-2.html: QQ群是多少啊?
小熊猫 1# 2012-1-10 19:19
QQ群是多少啊?
kuing 2# 2012-1-10 19:59
哪个QQ群?
thread-294-1-1.html: 学习测试
dahool 1# 2012-1-11 10:43
本帖最后由 dahool 于 2012-1-11 10:53 编辑 $f(x)$ $$\displaystyle\sum_{n=1}^{n}\dfrac{1}{n^{n}}$$
kuing 2# 2012-1-11 12:52
用了两个美元符号了就不必加 \displaystyle 了,\dfrac 也可以用回 \frac
thread-295-1-1.html: 极限
海盗船长 1# 2012-1-11 16:46
本帖最后由 海盗船长 于 2012-1-11 16:48 编辑 (1)\[ \lim_{n \to \infty} \frac{n (\sqrt[n] {n}-1)}{\ln{n}}=1 \] 证明: 令$ h_n=\sqrt[n] {n}-1 $ 则有 $n=(1+h_n)^n$ 且 $h_n \to 0$  $(n \to \infty)$ $\displaystyle \lim_{n \to \infty} \frac{n (\sqrt[n] {n}-1)}{\ln{n}} = \lim_{n \to \infty} \frac{n h_n}{n \ln (1+h_n)}=1$
海盗船长 2# 2012-1-11 16:49
本帖最后由 海盗船长 于 2012-1-12 16:48 编辑 (2)\[ \lim_{n \to \infty} \frac{n^2 (\sqrt[n] {n+1}-\sqrt[n+1]{n})}{\ln(n+1)} =1 \] 证明: 先证明\[ \lim_{n \to \infty} \frac{n^2 (\sqrt[n+1] {n+1}-\sqrt[n+1]{n})}{\ln(n+1)} =0 \] $\displaystyle \lim_{n \to \infty} \frac{n^2 (\sqrt[n+1] {n+1}-\sqrt[n+1]{n})}{\ln(n+1)} $ $\displaystyle =\lim_{n \to \infty} \frac{ n^2 \sqrt[n+1]{n+1} \left(1-\left(1-\frac{1}{n+1}\right)^{ \frac{1}{n+1}}\right)}{\ln(n+1)} $ $\displaystyle = \lim_{n \to \infty} \frac{n^2 \left( \left(\frac{1}{(n+1)^2}\right)+o\left( \frac{1}{n^2} \right)\right)}{\ln(n+1)}=0$ 于是,只需证明\[ \lim_{n \to \infty} \frac{n^2 (\sqrt[n] {n+1}-\sqrt[n+1]{n+1})}{\ln(n+1)} =1 \] $\displaystyle \lim_{n \to \infty} \frac{n^2 (\sqrt[n] {n+1}-\sqrt[n+1]{n+1})}{\ln(n+1)} $ $\displaystyle = \lim_{n \to \infty} \frac{ n^2 \sqrt[n+1] {n+1}\left((n+1)^{ \frac{ 1 }{ n(n+1) }}-1\right) } { \ln(n+1) }  $ 类似于(1),令$ h_n = (n+1)^{\frac{1}{n(n+1)}}-1$即得结论。 http://tieba.baidu.com/p/1358122765
海盗船长 3# 2012-1-12 15:18
本帖最后由 海盗船长 于 2012-1-12 16:30 编辑 (3) 数列$\{x_n\}$满足$x_1+x_2+ \cdots +x_n=\dfrac{1}{\sqrt{x_{n+1}}}$ 求极限:\[ \lim_{n \to \infty} \frac{n}{\ln{n}}\left(n^2 x_n^3-\frac{1}{9}\right)\] http://tieba.baidu.com/f?kz=1354206812
thread-296-1-1.html: 数列收敛
海盗船长 1# 2012-1-11 17:33
已知$x_1>0,x_2>0$,$x_{n+2}=\dfrac{2}{x_{n+1}+x_{n}}$ 求证:数列$\{x_n\}$收敛。
icesheep 2# 2012-1-12 22:16
贴吧那个网友的做法我实在不喜欢。。。有空再想想。。。
kuing 3# 2012-1-12 22:17
在哪?
icesheep 4# 2012-1-12 22:24
http://tieba.baidu.com/p/1362751658
海盗船长 5# 2012-1-12 22:25
4# icesheep 找到一个简单的:http://tieba.baidu.com/p/1359456568 不过不是很懂。。
kuing 6# 2012-1-12 22:29
oh 都看不懂
icesheep 7# 2012-1-12 23:24
本帖最后由 icesheep 于 2012-1-13 11:44 编辑 5# 海盗船长 四元数的做法不对吧,最后导不出矛盾啊。 另,我想了一下,如果选出了子列 \[\mathop {\lim }\limits_{k \to \infty } {x_{{n_k} + 2}} = \mathop {\lim }\limits_{n \to \infty } \sup {x_n}\] 则一定有 \[\mathop {\lim }\limits_{k \to \infty } {x_{{n_k} + 1}} = \mathop {\lim }\limits_{k \to \infty } {x_{{n_k}}} = \mathop {\lim }\limits_{n \to \infty } \inf {x_n}\]
海盗船长 8# 2012-1-13 16:31
本帖最后由 海盗船长 于 2012-1-13 16:37 编辑 7# icesheep 哦,你的结论是怎么得到的?
海盗船长 9# 2012-1-13 16:35
本帖最后由 海盗船长 于 2012-1-13 16:38 编辑 7# icesheep 四元数的意思是不是选取适当的e让 a_{n+2}>2/(a+A+2e)>2/(2A-2e)>a+e 或 a_{n+2}<2/(a+A-2e)<2/(2a+2e)<A-e 成立从而导出矛盾?
海盗船长 10# 2012-1-13 16:41
“那么对于任意e>0,必然存在充分大的N,使得n>N时均有 a-e<a_n<a+e,A+e>a_{n+1}>A-e” 这个不懂怎么得到的
icesheep 11# 2012-1-13 18:40
本帖最后由 icesheep 于 2012-1-13 18:55 编辑 不妨设 ${x_{{n_k} + 2}} \to \mathop {\lim }\limits_{n \to \infty } \sup {x_n} = A$ \[\left( {\forall {\varepsilon _1} > 0} \right)\left( {\exists {N_1} > 0} \right)\left( {{n_k} > {N_1} \Rightarrow A - {\varepsilon _1} < {x_{{n_k} + 2}} < A + {\varepsilon _1}} \right)\] 又 $\mathop {\lim }\limits_{n \to \infty } \inf {x_n} = a$ \[\left( {\forall {\varepsilon _2} > 0} \right)\left( {\exists {N_2} > 0} \right)\left( {{n_k} > {N_2} \Rightarrow {x_{{n_k}}} > a - {\varepsilon _2}} \right)\] 于是 ${{x_{{n_k}}} > a - {\varepsilon _2}}$,${{x_{{n_k} + 1}} > a - {\varepsilon _2}}$ 且 ${x_{{n_k}}} + {x_{{n_k} + 1}} < \frac{2}{{A - {\varepsilon _1}}} = 2a + \varepsilon '$ 易得 $a - {\varepsilon _2} < {x_{{n_k}}} < a + \varepsilon ' + {\varepsilon _2}$ , $a - {\varepsilon _2} < {x_{{n_k} + 1}} < a + \varepsilon ' + {\varepsilon _2}$ 由 ${\varepsilon _2}$ 和 $\varepsilon '$ 的任意性,易得 \[\mathop {\lim }\limits_{k \to \infty } {x_{{n_k}}} = \mathop {\lim }\limits_{k \to \infty } {x_{{n_k} + 1}} = a\]
海盗船长 12# 2012-1-13 21:17
哦,懂了
海盗船长 13# 2012-1-13 21:18
终于看懂了嘉威不哭的,,,
海盗船长 14# 2012-7-26 15:04
本帖最后由 海盗船长 于 2012-7-26 15:06 编辑 找到一个简单方法: 考虑映射$F:(0,\infty)\times(0,\infty)\to(0,\infty)\times(0,\infty)$且$F(x,y)=\left(\frac{2}{x+y},x\right)$,则$F(x_{n-1},x_{n-2})=(x_n,x_{n-1})$,且该映射的唯一不动点是$(1,1)$. 构造函数$g(x,y)=\max\left(x,\frac{1}{x},y,\frac{1}{y}\right)$,则有$g(F(x,y))\le g(x,y)$当且仅当$(x,y)=(1,1)$取等号.故数列$g(F(x_{n+1},x_{n}))$单调递减有下界,因而收敛. 在递推式两边取极限就得到$\lim_{n\to \infty}x_n=1$. 参考:http://www.artofproblemsolving.c ... p?f=67&t=488994
thread-297-1-2.html: 求火热!——kuing的心声
贵族风铃 1# 2012-1-11 22:58
求火热! 你的心,就像一把火,燃烧了整个论坛!! 同志们,走过的路过的不要错过,用你们的热情给论坛留下一丝火光,让它燃烧吧!!!
海盗船长 2# 2012-1-11 23:00

kuing 3# 2012-1-11 23:54
thread-298-1-1.html: {物理}关于行车
GAM 1# 2012-1-12 22:44
比较老的题了,一直没想明白。。。 就是一个司机以恒定功率P行驶在阻力一定的路上,此时牵引力为F,速度为v,突然他将功率减小为0.5P,之后汽车逐渐恢复匀速,求牵引力在此过程中变化的图像。 一直没想通F的具体趋势应该是啥样,老师只讲是上凸的。。。
kuing 2# 2012-1-12 23:15
要 F-t 图?
kuing 3# 2012-1-13 00:32
还是要解微分方程哩 动力$F$,阻力$f$恒定,车质量$m$恒定 \[F-f=ma=m\frac{\text{d}v}{\text{d}t}=mP\frac{\text{d}\frac1F}{\text{d}t}=-\frac{mP}{F^2}\frac{\text{d}F}{\text{d}t},\] 故 \[\text{d}t=-\frac{mP}{F^2(F-f)}\text{d}F,\] 设时间由$t_0$到$t$时动力由$F_0$变到$F(t)$,于是 \[t-t_0=\int_{F_0}^{F(t)}-\frac{mP}{F^2(F-f)}\text{d}F,\] ……
GAM 4# 2012-1-16 11:47
2# kuing 是的 3# kuing 这。。。代表的是啥样图像
kuing 5# 2012-1-16 12:21
4# GAM 可以解出这个微分方程,代初始条件得 $t(F)$ 的表达式,但 $F(t)$ 无法解出,不过作图应该不成问题,我有空再试试……
thread-3-1-1.html: 在 IE、chrome 中经已测试成功
kuing 1# 2011-9-25 22:01
RT
kuing 2# 2011-10-4 14:20
相比之下,用 chrome 爽快一些,不知其他浏览器如何
pxchg1200 3# 2011-10-4 22:25
360没问题,完美支持。。
kuing 4# 2011-10-4 22:38
3# pxchg1200 谢谢提供信息
icesheep 5# 2011-11-22 02:24
Firefox 有时可以显示可以,有时容易卡死,此时如果点“继续”就彻底卡死,只能点“禁止脚本”,然后公式就无法显示了。 其实 Firefox 和 Chrome 上都有直接在浏览器端把文本翻译成图片的插件。
icesheep 6# 2011-11-27 03:00
【Firefox】 先装猴子 Greasemonkey : https://addons.mozilla.org/zh-CN/firefox/addon/greasemonkey/ 再装 Tex the world 脚本: http://thewe.net/tex/ 把公式夹在 [; 和 ;] 之间,即可自动转换; 如果需要关闭转换,点击右上角猴子图标关掉脚本即可。 【Chrome】 Render any code between [; and ;] into LaTeX images. https://chrome.google.com/websto ... hkkcgdnmfmhhbjmhggn
kuing 7# 2011-11-27 08:42
6# icesheep 这两个之前我也试过,而且印象中记得是用 http://www.codecogs.com/latex/eqneditor.php 这个网里面的公式编译的 不过这都是需要用户自己装东西,还是mathjax强
thread-30-1-1.html: 支持环境
kuing 1# 2011-9-28 14:17
最常用的多行公式环境:align \begin{align} \sqrt{\frac{a^2+b^2}2}&\ge \frac{a+b}{2}\\ &\ge \sqrt{ab}\\ &\ge \frac2{\frac1a+\frac1b} \end{align}
thread-300-1-1.html: 为啥这个论坛没有版主?
海盗船长 1# 2012-1-14 17:19
本帖最后由 海盗船长 于 2012-1-14 21:25 编辑 RT 申请高数版版主~
海盗船长 2# 2012-1-14 22:22
高级会员了!
yes94 3# 2013-4-28 22:33
2# 海盗船长 金牌会员了!
李斌斌755 4# 2013-5-10 17:48
海盗进城了。
thread-301-1-1.html: [Chemistry]可逆反应的计算题
Chetion 1# 2012-1-14 22:30
Q:将a L的NH3通过灼热的装有铁触煤的硬质玻璃管后,气体体积变为 b L(气体体积在同温同压下测定),求该b L气体中NH3的体积分数. 相关的反应正是工业合成氨的反应。 答案是(2a - b)/b 求高手详解。
秋风树林 2# 2012-1-14 23:31
同学。。。。这种题目做不来就该打PP了。。。 先列出方程 2NH3===N2+3H2     a     2x        x      3x   a-2x       x      3x 然后不用我解释再怎么做以及这么做的原因了吧。。。
Chetion 3# 2012-1-16 21:02
2# 秋风树林 基本方法的确很管用呃~~~不过我用十字相乘法得到的结果不一样。 拜托一下。检查我的错误。 2 NH3         :       a L                             2a-b L                                            b L N2 & 3 H2   :       2a L                           b-a L (当氨全转化为原料时,体积应是2 倍。) 所以氨气体积分数为        (2a-b)/(2a-b+b-a)  =  (2a-b)/a      请分析一下。。
thread-302-1-1.html: f(x)可导$\lim_{x\to\pm\infty}f(x)=a$
kuing 1# 2012-1-15 14:46
已知 $f(x)$ 在 $\mathbf R$ 上可导,且 $\displaystyle\lim_{x\to\pm\infty}f(x)=a$,求证存在 $\xi\in\mathbf R$ 使 $f'(\xi)=0$。
kuing 2# 2012-1-15 14:47
最怕这些看着显然但又不知怎么说明的题哩………… 只想到下面这个比较麻烦的办法: 考虑 $f(0)$ 与 $a$ 的大小关系: 由上下对称性,只要证明当 $f(0)\geqslant a$ 时命题成立即可(这是因为若 $f(0)\leqslant a$,令 $g(x)=2a-f(x)$,则仍有 $\lim_{x\to\pm\infty}g(x)=a$,而且 $g(0)\geqslant a$)。 考虑 $f'(0)$ 与 $0$ 的大小关系: 如果 $f'(0)=0$,命题已经成立,当 $f'(0)\ne0$ 时,由左右对称性,只要证明当 $f'(0)>0$ 时命题成立即可(这是因为若 $f'(0)<0$,令 $h(x)=-f(x)$,则仍有 $\lim_{x\to\pm\infty}h(x)=a$,而且 $h'(0)>0$)。 综合两点,即只要证明当 $f(0)\geqslant a$ 且 $f'(0)>0$ 时命题成立即可。 此时存在 $b>0$ 使 $f(b)>f(0)$,故存在 $c\in(0,b)$ 使 $f(b)>f(c)>f(0)$。 由 $\lim_{x\to+\infty}f(x)=a$ 知,对于正数 $f(c)-a$,存在 $X>0$ 使当 $x>X$ 时有 $|f(x)-a|<f(c)-a$,在 $(X,+\infty)$ 内总能找到一点 $d>b$,则 $f(c)-a>|f(d)-a|\geqslant f(d)-a \implies f(c)>f(d)$,得 $f(b)>f(c)>f(d)$,故存在 $h\in(b,d)$ 使 $f(h)=f(c)$,而 $h>b>c$ 故存在 $\xi\in(c,h)$ 使 $f'(\xi)=0$,命题成立。
kuing 3# 2012-1-15 16:14
求简洁……
秋风树林 4# 2012-1-15 16:34
先这么写着试试。。。有些还是感觉怪怪的
kuing 5# 2012-1-15 16:36
大概懂了
秋风树林 6# 2012-1-15 16:42
大概懂了 kuing 发表于 2012-1-15 16:36 不过还是有点小小的问题 倒数第三行那个不等式跟a的正负性多少会牵扯到一点关系 但是应该可以通过类似的讨论解决 不过直观的讲,因为无穷的极限是a 不是常数函数一定可以用一个领域将x0以后的所有对应函数值给覆盖住 从而上确界或下确界就在这闭区间了 然后Fermat之~
kuing 7# 2012-1-15 16:43
嗯,正想问下那里咋的 当然,如果学我上面那个用对称性什么的搞,或许能去掉一些正负讨论
kuing 8# 2012-1-15 16:48
去后面掉绝对值就行了,也不用什么保序了 \[ \frac{M-a}2>|f(x)-a|\geqslant f(x)-a \implies f(x)<\frac{M+a}2<M \]
海盗船长 9# 2012-1-15 17:16
证明存在不同的两个数$x_1,x_2 \in \mathbf{R}$使得$f(x_1)=f(x_2)$就行了。
kuing 10# 2012-1-15 17:56
呃,其实我也就是为了证明这一点所以才扯出这么多东东…… 有简单的办法就写写吧
海盗船长 11# 2012-1-15 19:12
我错了,回帖之前没看贴,,,
Nirvanacs 12# 2012-1-16 21:09
其实蛮好想的,导函数介值,所以不存在的话,必然单调,然后两边极限相同,容易导出矛盾
kuing 13# 2012-1-16 21:14
12# Nirvanacs 崩溃?
Nirvanacs 14# 2012-1-17 12:39
13# kuing 是的
kuing 15# 2012-1-17 13:52
14# Nirvanacs 又是你,还是你,嘿嘿
叶剑飞Victor 16# 2012-8-23 19:05
本帖最后由 叶剑飞Victor 于 2012-8-23 19:06 编辑 用罗尔中值定理一下子就证明出来了吧。
thread-303-1-1.html: 最近发现用 Firefox 对 Mathjax 公式的读入似乎比较好
kuing 1# 2012-1-16 15:24
如题,有时我用 IE 或 chrome 都读入不了公式,用 Firefox 就多数没问题 不知是我电脑问题还是什么问题……
kuing 2# 2012-1-17 14:55
今天貌似又反过来了
叶剑飞Victor 3# 2012-8-10 00:47
我估计主要原因可能是只有火狐浏览器原生支持MathML,而其他浏览器都不支持MathML(至少不原生支持),所以除了火狐以外都只能用HTML+CSS来显示数学公式。 IE浏览器(以及其他Trident内核浏览器)需要安装MathPlayer插件才能支持MathML,所以不算原生支持MathML
kuing 4# 2012-8-10 00:49
firefox 挺好的,当初存档也用它
叶剑飞Victor 5# 2012-8-10 00:59
本帖最后由 叶剑飞Victor 于 2012-8-10 01:10 编辑 4# kuing http://www.math15.com/是用ASCIIMathML把LaTeX公式转成MathML的,只能使用Firefox浏览器来看数学式子了,IE用户要安装MathPlayer插件才能正确显示数学式子,悲催的Chrome用户更是完全没法看了 ,还有悲催的Opera用户…… 其实我是Chrome用户。由于这个论坛的LaTeX公式是用MathJax处理的,处理出来的HTML+CSS在Chrome中显示效果非常好!P.S.  k12论坛的数学论坛也是用MathJax处理的。
kuing 6# 2012-8-10 02:08
嗯,K12那里我知道,而且是将 mathjax 放在自己的空间上调用的,只可惜那里也很冷清,而且数学论坛那里也变成了那啥“桂林古稀老朽”的“放大缩小”问题专版了,也没人管管。哎,其实K12的论坛管理得比人教好多,可是人气一直就上不来。 看了一下那里的网页代码,加载mathjax的部分是 <script src="http://sq.k12.com.cn/MathJax/MathJax.js?config=TeX-AMS-MML_HTMLorMML" type="text/javascript" type="text/javascript"></script> 复制代码 不知为什么会有两个 type="text/javascript" 。 设置的那部分就不发上来了(因为里面含有美元符号,贴上来会变成公式)
叶剑飞Victor 7# 2012-8-10 09:56
6# kuing 那个MathJax下载下来是一个17.6MB的zip压缩文件,解压出来40.1MB。好占空间的!
thread-304-1-8.html: [不等式] Hard one.
pxchg1200 1# 2012-1-16 21:48
Let $ a,b,c \geq 0$ prove that: \[  \frac{a^{2}+b^{2}+c^{2}}{ab+bc+ca}+\frac{69(a^{3}b+b^{3}c+c^{3}a)}{(a+b+c)^{4}}\ge\frac{32}{9} \] Ps: 好久没上了。busy in school exam...
thread-305-1-2.html: 闪之前乱扯
kuing 1# 2012-1-17 01:12
  话说最近白天似乎都静不下心来做题,白天在各种Q群里看到的未被解决的题都在深夜才着手去做,结果做出来发解答之后提问的人早就走了,第二天上线也不一定看得到了,除非:TA很早上线 或者 早上该群没什么人讲话 又或者 TA有去群社区看群聊记录(或看漫游消息)的习惯。   由此我又想到Q群和论坛的长短,其实以前都扯过,不过懒得找记录,再扯一下也没啥。 Q群的优点在于:讨论气氛好,多人一起交流,讨论可以很激烈,速度够爽快,还可以闲聊,又方便发图,……; 论坛的优点在于:记录的保存,一贴一问题(多数情况下),有条理利于讨论和阅读,不易被刷掉(故此一般不会出现上面说的情况,除非像人教论坛龙门阵版块那样被铺天盖地刷广告),好贴可以收藏,老贴也能被挖出来继续讨论,……。   而两者相应的短处大概就是对方的反面(应该就是除了“多人一起交流”之外了),所以两者可以说是互补的,因此如果能将Q群和论坛的结合,取长补短,整出一个新的东东,那就好玩了,不过目前是否有这种东东存在?   闪!
pxchg1200 2# 2012-1-17 10:29
呵呵,我看你一直都在线来着。
kuing 3# 2012-1-17 14:54
2# pxchg1200 你是说QQ?
pxchg1200 4# 2012-1-17 15:33
3# kuing 是啊。 不就是在Q群里讨论么?
kuing 5# 2012-1-17 15:34
4# pxchg1200 嗯,最近一般是中午到深夜一般都在线
力工 6# 2012-4-10 14:39
关键是如果将群内的问题转入论坛,也是个麻烦,工作量大。
thread-306-1-8.html: [不等式] 谁能解释下Can是怎么配的?
pxchg1200 1# 2012-1-17 10:39
前天在Aops 看到一个题,如下: For $a,b,c \in \mathbb{R}$ for which $(a-b)(a-c)(b-c) \neq 0$, prove: $\sum \frac{a^4}{(a-b)^2(a-c)^2} \ge \frac{1}{2}$ Equality holds when $\{a,b,c\}=\{t,-t,0\}$ for $ t\in \mathbb{R} \backslash \{0\}$ Can给出了他的解答: Proof. Using the Cauchy-Schwarz inequality, we have \[\left[ \sum (2a-b-c)^4\right] \left[ \sum \frac{a^4}{(a-b)^2(a-c)^2}\right] \ge \left[ \sum \frac{a^2(2a-b-c)^2}{(a-b)(a-c)}\right]^2.\] But, it is easy to check that \[\sum (2a-b-c)^4 =18\left( \sum a^2-\sum ab\right)^2\] and \[\begin{aligned}\sum \frac{a^2(2a-b-c)^2}{(a-b)(a-c)} &=4\sum a^2-\sum ab =\left( \sum a\right)^2+3\left( \sum a^2-\sum ab\right) \\ &\ge 3\left( \sum a^2-\sum ab\right) > 0.\end{aligned} \] So we have \[18\left( \sum a^2-\sum ab\right)^2\left[ \sum \frac{a^4}{(a-b)^2(a-c)^2}\right]  \ge \left[ 3\left( \sum a^2-\sum ab\right)\right]^2,\] from which it follows that \[\sum \frac{a^4}{(a-b)^2(a-c)^2} \ge \frac{1}{2},\] as desired. $\blacksquare$ 我想问那个$ \sum (2a-b-c)^4 =18\left( \sum a^2-\sum ab\right)^2$ 是怎么想到的? 问Can, 他说:it's my secret,so I can't tell you now.... Any idea?
kuing 2# 2012-1-17 13:48

kuing 3# 2012-1-17 14:23
我想可能是他早就对四次的恒等式研究得很多了,比如可能推过这个 \begin{align*} \sum{(x-y)^{4}}&=\sum{\left( (x-y)^{2} \right)^{2}} \\ & =\left( \sum{(x-y)^{2}} \right)^{2}-2\sum{(x-y)^{2}(y-z)^{2}} \\ & =\left( 2\sum{(x-y)(y-z)} \right)^{2}-2\sum{(x-y)^{2}(y-z)^{2}} \\ & =4\left( \sum{(x-y)(y-z)} \right)^{2}-2\left( \sum{(x-y)(y-z)} \right)^{2}+4(x-y)(y-z)(z-x)\sum{(x-y)} \\ & =2\left( \sum{(x-y)(y-z)} \right)^{2} \\ & =\frac{1}{2}\left( \sum{(x-y)^{2}} \right)^{2}, \end{align*} 这样若令 $x=a-b,y=b-c,z=c-a$,则 $x+y+z=0$ 且 $\sum{(x-y)^{4}}=\sum{(2a-b-c)^{4}}$,以及 \begin{align*} \frac{1}{2}\left( \sum{(x-y)^{2}} \right)^{2}&=2\left( \left( \sum{x} \right)^{2}-3\sum{xy} \right)^{2} \\ & =18\left( \sum{xy} \right)^{2} \\ & =18\left( \sum{(a-b)(b-c)} \right)^{2} \\ & =18\left( \sum{a^{2}}-\sum{ab} \right)^{2}. \end{align*}
thread-307-1-8.html: [不等式] kk,能给我们讲讲pqr method吗?
pxchg1200 1# 2012-1-17 10:44
最近发现pqr好像很有用。但又没搜索到啥文档,只有uvw的。请kuing介绍下吧,谢谢。
kuing 2# 2012-1-17 13:45
找 On a class of three-variable inequalities(Vo Quoc Ba Can)
kuing 3# 2012-1-17 14:56
其实就是给出 r 的最值 用三次函数搞的
pxchg1200 4# 2012-1-17 14:58
3# kuing 有没有想过把 pqr 和Cauchy-Schwarz 融合在一起? 我觉得那个很有搞头啊。
kuing 5# 2012-1-17 15:04
http://wenku.baidu.com/view/ee67a881d4d8d15abe234e63.html 不过其实我很少这样设 q,而是习惯上直接设成 $ab+bc+ca=q$,此时 r 的范围变成 \[\frac{-2p^3+9pq-2(p^2-3q)\sqrt{p^2-3q}}{27}\leqslant r\leqslant \frac{-2p^3+9pq+2(p^2-3q)\sqrt{p^2-3q}}{27}\]
pxchg1200 6# 2012-1-17 15:08
本帖最后由 pxchg1200 于 2012-1-17 15:10 编辑 5# kuing 我觉得这个文档少了点什么,估计Can没写出来。一个很重要的事实就是任意的三元不等式都可以写成 \[  F(a,b,c)+G(a,b,c)\times p \geq 0 \] 的形式。其中 $ p=a^2b +b^2 c+c^2 a $ 所以,建立对$p$ 的估计是至关重要的。 $ F(a,b,c),G(a,b,c) $都是对称多项式来着。
kuing 7# 2012-1-17 15:08
3# kuing 有没有想过把 pqr 和Cauchy-Schwarz 融合在一起? 我觉得那个很有搞头啊。 pxchg1200 发表于 2012-1-17 14:58 不曾想过,怎么玩?
pxchg1200 8# 2012-1-17 15:12
7# kuing 暂时没有figure out.只是有一次听Can提到过,他说他发明了种新的CS method. 可以解决很多三元3次、4次、5次、6次的cyclic 型不等式。其中第一步就是把不等式改成那种$pqr$的形式。
kuing 9# 2012-1-17 15:16
5# kuing 我觉得这个文档少了点什么,估计Can没写出来。一个很重要的事实就是任意的三元不等式都可以写成 \[  F(a,b,c)+G(a,b,c)\times p \geq 0 \] 的形式。 $\color{red}{其中 p=a^2b+b^2c+c^2a。所以,建立对 p 的估计是至关重要的。}$ pxchg1200 发表于 2012-1-17 15:08 这个我之前也研究过一下,当 $p=a+b+c,q=ab+bc+ca$ 固定时,$t=a^2b+b^2c+c^2a$ 的范围是 \[\left[ \frac19\left( p^3-2(p^2-3q)\sqrt{p^2-3q} \right),\frac19\left( p^3+2(p^2-3q)\sqrt{p^2-3q} \right) \right]\] 但很多时候用不上,因为还会有 r 在。
pxchg1200 10# 2012-1-17 15:18
http://www.artofproblemsolving.com/Forum/viewtopic.php?p=2574568#p2574568 看这里。那个 $ a^{3}b+b^{3}c+c^{3}a\le\frac{p^{4}+9p^{2}q-27q^{2}+2(p^{2}-3q)\sqrt{7p^{2}(p^{2}-3q)}}{27} $ 他们还有对这个$ a^{3}b+b^{3}c+c^{3}a$ 的估计。
kuing 11# 2012-1-17 15:22
8# pxchg1200 噢,他是准备写书介绍这个办法吧?现在不肯透露就
pxchg1200 12# 2012-1-17 15:22
9# kuing 怎么算到的这个范围?!
pxchg1200 13# 2012-1-17 15:27
11# kuing 不知道,我翻遍了他的那本CS book 都没看到一个3元4次的不等式。另外其他几本<old and New inequalities vol 2> 和<Inequalities with beautiful solutions> 也没看见多少关于这类的题。
kuing 14# 2012-1-17 15:27
12# pxchg1200 用另外一个轮换式,加、乘,变成对称,得到一个 pqr 方程,对 r 求偏导………………
pxchg1200 15# 2012-1-17 15:38
本帖最后由 pxchg1200 于 2012-1-17 15:41 编辑 14# kuing 难道是考虑 \[ \sum{a^{2}b}+\sum{ab^{2}}=\sum{ab(a+b)}=x_{1}+x_{2} \] \[ (\sum{a^{2}b})(\sum{ab^{2}})= x_{1}x_{2}\] 其中$ x_{1},x_{2}$为一个一元二次方程的根。。。。 接下来就不知道怎么弄了。
kuing 16# 2012-1-17 15:44
嗯 下面是之前写的(不可能现在这么快写完这么长): 对于实数$a,b,c$,记$p = a + b + c$,$q = ab + bc + ca$,$r = abc$,$t_1 = a^2b + b^2c + c^2a$,$t_2 = ab^2 + bc^2 + ca^2$,直接计算得 \[ \left\{\begin{aligned} t_1+t_2&=pq-3r,\\ t_1t_2&=q^3+p^3r-6pqr+9r^2, \end{aligned}\right. \] 可见$t_1,t_2$乃是如下关于$y$的二次方程 \[ y^2-(pq-3r)y+q^3+p^3r-6pqr+9r^2=0 \] 的两根。当固定$p,q$时,记 \[ F(r,y)=y^2-(pq-3r)y+q^3+p^3r-6pqr+9r^2, \] 则 \[ \frac{\text dy}{\text dr}=-\frac{F_r(r,y)}{F_y(r,y)}=-\frac{3y+p^3-6pq+ 18r}{2y-pq+3r}, \] 联立方程组 \[ \left\{\begin{aligned} 3y+p^3-6pq+18r&=0,\\ y^2-(pq-3r)y+q^3+p^3r-6pqr+9r^2&=0, \end{aligned}\right. \] 解得 \[ \left\{\begin{aligned} r&=\frac{1}{27}\left({-2p^3+9pq+(p^2-3q)\sqrt{p^2-3q}} \right),\\ y&=\frac{1}{9}\left({p^3-2(p^2-3q)\sqrt{p^2-3q}}\right), \end{aligned}\right. \] 及 \[ \left\{\begin{aligned} r&=\frac{1}{27}\left({-2p^3+9pq-(p^2-3q)\sqrt{p^2-3q}} \right),\\ y&=\frac{1}{9}\left({p^3+2(p^2-3q)\sqrt{p^2-3q}}\right), \end{aligned}\right. \] 又熟知当$p,q$固定时$r$的取值范围是 \[ \left[{\frac{1}{27}\left({-2p^3+9pq-2(p^2-3q)\sqrt{p^2-3q}} \right),\frac{1}{27}\left({-2p^3+9pq+2(p^2-3q)\sqrt{p^2-3q}} \right)}\right], \] 显然上述两组解的$r$均在此区间内,又因$F(r,y)=0$是椭圆型曲线,所以取最值的$y$及相应的$r$就是那两组解,也就是说$t_1,t_2$的取值范围就是 \[ \left[{\frac{1}{9}\left({p^3-2(p^2-3q)\sqrt{p^2-3q}} \right),\frac{1}{9}\left({p^3+2(p^2-3q)\sqrt{p^2-3q}}\right)} \right]. \]
kuing 17# 2012-1-17 15:47
$a^3b+b^3c+c^3a$ 应该也可以这样试试
pxchg1200 18# 2012-1-17 15:55
17# kuing 赞一个,   其实$ a^{3}b+b^{3}c+c^{3}a=(a^2b+b^2c+c^2a)(a+b+c)-\sum{a^{2}bc}-\sum{a^{2}b^{2}}$ 所以说得到$ a^2b+b^2c+c^2a $ 就足够了。
kuing 19# 2012-1-17 16:02
18# pxchg1200 不一样的,$\sum{a^2bc}=pr$,有 $r$ 在,$t$ 取最值的条件与 $r$ 取最值的条件不同,所以如果直接代入的话得到的东东是取不到的
kuing 20# 2012-1-17 16:48
可以说,按上面这种方法,只要所研究的对象化出来除了 t 还有 r 在的话就得重新算算了 简单地,可以试试 $a^2b+b^2c+c^2a+abc$ 的取值范围
thread-307-2-8.html:
pxchg1200 21# 2012-1-17 21:14
20# kuing 我只记得有个Well-know的不等式是: \[ a^2b+b^2c+c^2a+abc \leq \frac{4}{27}(a+b+c)^{3} \]
kuing 22# 2012-1-17 21:21
21# pxchg1200 嗯,这个我也知道,不过如果 p,q 固定了的话就不一定能取到等号了
pxchg1200 23# 2012-1-17 21:28
22# kuing 我发现其实那些所谓的pqr,uvw 貌似都是基于一个叫ABC的理论,那个Tran Phuong 在《Diamonds in Mathematical Inequalites》中花了很大篇幅讲这个方法。不过还没看懂。。。。
pxchg1200 24# 2012-1-17 21:42
那个ABC method好像还能证Vasc不等式。 \[ (a^2+b^2+c^2)^2\geq 3(a^3b+b^3c+c^3a) \]
kuing 25# 2012-1-17 21:43
不曾了解ABC
kuing 26# 2012-1-18 01:21
接16楼。 加个$kabc$($k$为常数)试试,令$k_1=t_1+kr$,$k_2=t_2+kr$,利用前面的结果,有 \[ \left\{\begin{aligned} k_1+k_2&=t_1+t_2+2kr=pq+(2k-3)r,\\ k_1k_2&=t_1t_2+kr(t_1+t_2)+k^2r^2=q^3+p^3r+(k-6)pqr+(k^2-3k+9)r^2, \end{aligned}\right. \] 可见$k_1,k_2$乃是如下关于$y$的二次方程 \[ y^2-(pq+(2k-3)r)y+q^3+p^3r+(k-6)pqr+(k^2-3k+9)r^2=0 \] 的两根。当固定$p,q$时,记 \[ F(r,y)=y^2-(pq+(2k-3)r)y+q^3+p^3r+(k-6)pqr+(k^2-3k+9)r^2, \] 则 \[ \frac{\text dy}{\text dr}=-\frac{F_r(r,y)}{F_y(r,y)}=-\frac{-(2k-3)y+(k-6)pq+p^3+2(k^2-3k+9)r}{2y-pq-(2k-3)r}, \] 联立方程组 \[ \left\{\begin{aligned} -(2k-3)y+(k-6)pq+p^3+2(k^2-3k+9)r&=0,\\ y^2-(pq+(2k-3)r)y+q^3+p^3r+(k-6)pqr+(k^2-3k+9)r^2&=0, \end{aligned}\right. \] 解得 \[ \left\{\begin{aligned} r&=\frac{1}{27}\left( -2p^{3}+9pq+\frac{3-2k}{\sqrt{k^{2}-3k+9}}\cdot\sqrt{(p^{2}-3q)^{3}} \right),\\ y&=\frac{1}{27}\left( (3-2k)p^{3}+9kpq-2\sqrt{(k^{2}-3k+9)(p^{2}-3q)^{3}} \right), \end{aligned}\right. \] 及 \[ \left\{\begin{aligned} r&=\frac{1}{27}\left( -2p^{3}+9pq-\frac{3-2k}{\sqrt{k^{2}-3k+9}}\cdot\sqrt{(p^{2}-3q)^{3}} \right),\\ y&=\frac{1}{27}\left( (3-2k)p^{3}+9kpq+2\sqrt{(k^{2}-3k+9)(p^{2}-3q)^{3}} \right), \end{aligned}\right. \] 又熟知当$p,q$固定时$r$的取值范围是 \[ \left[{\frac{1}{27}\left({-2p^3+9pq-2\sqrt{(p^2-3q)^3}} \right),\frac{1}{27}\left({-2p^3+9pq+2\sqrt{(p^2-3q)^3}} \right)}\right], \] 又易证 \[\pm\frac{3-2k}{\sqrt{k^{2}-3k+9}}\in(-2,2),\] 故上述两组解的$r$均在此区间内,又因$F(r,y)=0$是椭圆型曲线,所以取最值的$y$及相应的$r$就是那两组解,也就是说$k_1,k_2$的取值范围就是 \[ \left[ \frac{1}{27}\left( (3-2k)p^{3}+9kpq-2\sqrt{(k^{2}-3k+9)(p^{2}-3q)^{3}} \right), \frac{1}{27}\left( (3-2k)p^{3}+9kpq+2\sqrt{(k^{2}-3k+9)(p^{2}-3q)^{3}} \right) \right]. \]
kuing 27# 2012-1-18 01:54
以上结果用 Mathematica 代入若干数值验证暂时都成立 命令: p = -3; q = 2; k = 1/2; zxz1 = ((3 - 2 k) p^3 + 9 k p q - 2 Sqrt[(k^2 - 3 k + 9) (p^2 - 3 q)^3])/27; zdz1 = ((3 - 2 k) p^3 + 9 k p q + 2 Sqrt[(k^2 - 3 k + 9) (p^2 - 3 q)^3])/27; Minimize[{a^2 b + b^2 c + c^2 a + k a b c, a + b + c == p, a b + b c + c a == q}, {a, b, c}]; zxz2 = Part[%, 1]; Maximize[{a^2 b + b^2 c + c^2 a + k a b c, a + b + c == p, a b + b c + c a == q}, {a, b, c}]; zdz2 = Part[%, 1]; Factor[zxz1 - zxz2] Factor[zdz1 - zdz2] 注:前三行对 p,q,k 赋值,只要填上满足 $p^2\geqslant3q$,k 为任意值都行,填完后运行,只要结果出现两个 0 就是成立的。
kuing 28# 2012-1-19 01:58
好,试水了,$u_1=a^3b+b^3c+c^3a$,$u_2=ab^3+bc^3+ca^3$,计算得 \[\left\{\begin{aligned} u_1+u_2&=p^2q-2q^2-pr,\\ u_1u_2&=q^4+(p^4-5p^2q+q^2)pr+7p^2r^2, \end{aligned}\right.\] 方程 \[y^2-(p^2q-2q^2-pr)y+q^4+(p^4-5p^2q+q^2)pr+7p^2r^2=0,\] 仍然固定p,q \[\frac{\text dy}{\text dr}=-\frac{F_r(r,y)}{F_y(r,y)}=-\frac{p(y+q^2+14pr-5p^2q+p^4)}{2y+2q^2+pr-p^2q},\] 联立 \[\left\{\begin{aligned} p(y+q^2+14pr-5p^2q+p^4)&=0,\\ y^2-(p^2q-2q^2-pr)y+q^4+(p^4-5p^2q+q^2)pr+7p^2r^2&=0, \end{aligned}\right.\] 若$p=0$,第一式恒成立,第二式化为$(y+q^2)^2=0$,从而$y=-q^2$,即$u_1,u_2$为常数$-q^2$; 若$p\ne0$,则解得 …… 时间关系明在再写,反正跟前面提到的那个贴的结果一样。
kuing 29# 2012-1-20 00:35
突然发现有点不同,就是处等条件的问题
pxchg1200 30# 2012-1-20 14:31
29# kuing 上述方法能秒这个么? If $ a,b,c $ are nonnegative real numbers,no two of which are zero,then: \[\frac{a^2+b^2+c^2}{ab+bc+ca}+\frac{69(a^3b+b^3c+c^3a)}{(a+b+c)^4} \ge \frac{32}{9} \] ( quykhtn-qa1)
kuing 31# 2012-1-20 14:38
我还得进一步研究一下未知元是非负的情形,不然如果直接用那个下界(大概都是负的)也没什么用了。
kuing 32# 2012-1-20 23:09
话说还顺便解决了那个vasc的反向,$-\sqrt7/8$ 那个
pxchg1200 33# 2012-1-21 13:47
32# kuing 哇,贴出来看看怎么做的。
kuing 34# 2012-1-21 15:00
33# pxchg1200 呃,就是与下界比较咯,作差移项平方因式分解…………没什么技术含量(暴力分解用软件 )
pxchg1200 35# 2012-1-21 22:44
28# kuing kk继续嘛,写得非常好。 我要看看那个pqr到底有什么用。
pxchg1200 36# 2012-1-21 22:45
34# kuing 先不求技术,做出来再说嘛,能贴下软件的结果么。 求详细过程。。
kuing 37# 2012-1-25 17:51
突然发现其实由 $a^2b+b^2c+c^2a+kabc$ 的范围可以得到 $a^3b+b^3c+c^3a$ 的范围,这是因为 \[a^3b+b^3c+c^3a=p(a^2 b + b^2 c + c^2 a + a b c) - q^2\] 于是当 $p,q$ 固定时,只要分 $p$ 的正负方可由 $a^2b+b^2c+c^2a+kabc$ 取 $k=1$ 时的范围而得到其范围。 咳,原来这个不用重新算,上面白算鸟
kuing 38# 2012-1-25 17:53
这样其它四次轮换式应该也能得到了,哈,没想到这个顺手试试的 +kabc 原来是很有用嘀
pxchg1200 39# 2012-1-27 11:09
38# kuing 表示刚收到你的信息,这几天家里断网了,悲剧。
thread-308-1-7.html: 来自群的一道解方程
kuing 1# 2012-1-17 19:06
学生-废名(3806*****) 在实数范围内解方程 \[\frac{11x^2-6}{7-12x^2}=\sqrt{\frac{7x+6}{12x+11}}.\] (提问者注:两个反函数,解在$y=x$上,然后就变成三次方程了……) 解 首先对这个注明说一下,互为反函数的两函数图象的交点可不一定全在$y=x$上,故此按这样解下去有可能漏解。 事实上,用最原始的方法,两边平方后作差再分解就可以了,不过在因式分解的时候,倒是可以利用一下上面说的这一特点去观察分解。 具体地,我们先两边平方,有 \[\left(\frac{11x^2-6}{7-12x^2}\right)^2=\frac{7x+6}{12x+11},(*)\] 通过各项系数的规律,容易观察到当$x=-1$时两边都是$1$,所以$x+1$是式(*)左右作差后的因式,但剩下的次数还是很高,故此要再从别的方向考虑一下。 根据前面说的,在$y=x$上可能有解,所以我们考虑方程 \[\frac{7x+6}{12x+11}=x^2,\] 同样看到当$x=-1$时两边都是$1$,于是我们对上式作差就可以提出$x+1$得到 \[\frac{7x+6}{12x+11}-x^2=-\frac{(x+1)(12x^2-x-6)}{12x+11},\] 由十字相乘可以看出$12x^2-x-6=(4x-3)(3x+2)$,于是我们再考虑$x=3/4$和$x=-2/3$时式(*)如何,通过代入发现都是其根,从而式(*)左右作差亦有因式$4x-3$及$3x+2$,这样,我们就不难将式(*)化为 \[\left(\frac{11x^2-6}{7-12x^2}\right)^2-\frac{7x+6}{12x+11}= \frac{(x+1)(4x-3)(3x+2)(37x^2+5x-17)}{(12x+11)(12x^2-7)^2}=0,\] 解得 \[x_1=-1,x_2=\frac34,x_3=-\frac23,x_4=\frac{-5+11\sqrt{21}}{74},x_5=\frac{-5-11\sqrt{21}}{74},\] 再通过原方程对$x$的限制$(7x+6)/(12x+1)\geqslant0$且$(11x^2-6)/(7-12x^2)\geqslant0$,不难比较出只有$x_2$和$x_5$是符合的,所以原方程的解为 \[x_2=\frac34,x_5=\frac{-5-11\sqrt{21}}{74}.\] 此最终结果正好说明前面的判断,如果只考虑在$y=x$上的解,那么将会漏掉$x_5$。
dahool 2# 2012-1-18 18:24
我想加群,能发个群号吗?
kuing 3# 2012-1-18 18:45
11071642
dahool 4# 2012-1-18 22:50
谢谢
叶剑飞Victor 5# 2012-8-21 01:58
3# kuing 此群支持$\LaTeX$显示吗?
kuing 6# 2012-8-21 02:11
5# 叶剑飞Victor 不知在未来的多少年内能支持?
thread-309-1-6.html: [不等式] 求CS.
pxchg1200 1# 2012-1-17 21:11
problem: Let $a,b,c \geq 0$ prove that: \[ \sqrt{a(b+1)}+\sqrt{b(c+1)}+\sqrt{c(a+1)}\leq \frac{3}{2}\sqrt{(a+1)(b+1)(c+1)}\]
天涯无际 2# 2012-2-17 19:13
注意到恒等式: \[\frac{9}{4}-\sum{\frac{a}{a+1}}\cdot \sum{\frac{1}{c+1}}=\frac{(ab+bc+ca+3abc-a-b-c-3)^2}{4(a+1)^2(b+1)^2(c+1)^2}\] 然后由Cauchy不等式证毕...
kuing 3# 2012-2-17 19:37
恒等式牛……
pxchg1200 4# 2012-2-19 15:12

pxchg1200 5# 2012-3-12 22:54
2# 天涯无际 我是这样想的。 proof: by Cauchy-Schwar: \[ \sqrt{a(b+1)}+\sqrt{b(c+1)}\leq \sqrt{(a+1)[(b+1)+b(c+1)]} \] Thus,just check \[ \sqrt{2b+bc+1}+\sqrt{c}\leq \sqrt{(b+1)(c+1)}\] by Cauchy-Schwarz again: \[ \sqrt{b(c+2)+1}+\sqrt{c}\leq \sqrt{[b(c+2)+1+(c+1)](1+\frac{c}{c+1})}=\sqrt{\frac{(b+1)(c+2)(2c+1)}{c+1}} \] Or \[ 4(c+2)(2c+1)\leq 9(c+1)^{2} \] Using AM-GM for: \[ 4(c+2)(2c+1)\leq 9(c+1)^{2} \] Hence we obtain the desire result. 楼上应该是Tourish 吧。 (如果我没猜错的话。) :D
kuing 6# 2012-3-13 01:35
你们都好牛……
yizhong 7# 2012-8-23 18:44
本帖最后由 yizhong 于 2012-8-23 23:58 编辑 我来骗下分。本题只需要简单的A-G即可解决,我们两边同时除以$\sqrt{(a+1)(b+1)(c+1)}$ 我们就可得到要证明的不等式变成:$\sqrt{\dfrac{a}{(a+1)(c+1)}}+\sqrt{\dfrac{b}{(b+1)(a+1)}}+\sqrt{\dfrac{c}{(b+1)(c+1)}}\leqslant\dfrac{3}{2}(A)$由于:$\sqrt{\dfrac{a}{(a+1)(c+1)}}\leqslant\dfrac{1}{2}(\dfrac{a}{a+1}+\dfrac{1}{c+1})(1)$ $\sqrt{\dfrac{b}{(a+1)(b+1)}}\leqslant\dfrac{1}{2}(\dfrac{b}{b+1}+\dfrac{1}{a+1})(2)$ $\sqrt{\dfrac{c}{(b+1)(c+1)}}\leqslant\dfrac{1}{2}(\dfrac{c}{c+1}+\dfrac{1}{b+1})(3)$   将(1)  (2)  (3) 叠加即可得到(A) 证毕。
thread-310-1-8.html: [不等式] jensen不等式
yayaweha 1# 2012-1-17 21:15
求jensen不等式加权形式的证明
kuing 2# 2012-1-17 21:27
归纳法
kuing 3# 2012-1-17 21:29
早前在这里 http://bbs.pep.com.cn/viewthread ... id=72072#pid4694276 (3楼) 写过,引理一。
yayaweha 4# 2012-1-19 09:00
3# kuing 问下:jensen给的凸函数的定义是怎么样的?
kuing 5# 2012-1-19 12:34
4# yayaweha 这个我就不是很清楚,得查查书先。 PS。现在一般用的凸函数定义要么是像我给链接里的那种,要么就用f(x1)+f(x2)>=2f(x1/2+x2/2)但是要函数连续,两者好像是等价的,但后者如果没有连续就不等价。
thread-312-1-8.html: 多项式系数绝对值最大值问题!
海盗船长 1# 2012-1-18 11:58
An extremal property of Chebyshev polynomials.PDF (62.41 KB)
kuing 2# 2012-1-18 12:11

icesheep 3# 2012-2-14 00:37
做过关于这个的 http://yjq24.blogbus.com/logs/56793245.html http://yjq24.blogbus.com/logs/75418472.html
thread-313-1-8.html: [几何] 一道圆锥曲线的题
xr5252 1# 2012-1-18 15:30
已知动圆M过定点F(0,-根号2) 且与直线y=根号2相切。 椭圆N的对称轴是坐标轴。一个焦点是F。点A(1,根号2)在椭圆N上 (1)求动圆M的圆心轨迹α的方程和椭圆N的方程 (2)已知与轨迹α在X=-4处的切线平行的直线L与椭圆N交于B C两点。试探究使得△ABC的面积为3/2的直线L是否存在    求各位大神提供思路  本人是一点都没有~!!
kuing 2# 2012-1-18 16:19
第一问会吗?会的话贴贴结果
xr5252 3# 2012-1-18 17:02
一点头绪都木有!!
kuing 4# 2012-1-18 17:43
呃,那看来第二问做不做也罢…… 第一问的话,前者用抛物线定义,后者由 $F(0,-\sqrt2)$ 得另一个焦点 $F'(0,\sqrt2)$,然后计算 $AF+AF'$ 得到长轴,然后也可以算出短轴了
xr5252 5# 2012-1-18 18:09
点到直线的距离方程可以求出动圆的R 但 过定点M的条件有什么用 。。。。 就是圆的不太明白 还是我想错了?动圆圆心的轨迹是直线?不是圆的标准方程?》
xr5252 6# 2012-1-18 18:18
恍然大悟~~~╮(╯▽╰)╭  真挫
海盗船长 7# 2012-1-19 12:27
thread-314-1-8.html: [函数] 还是来自群……
kuing 1# 2012-1-18 23:00
已知函数$f(x)=a\ln x-(x-1)^2-ax$,设$a>0$,如果对于$f(x)$的图象上两点$P_1(x_1,f(x_1)),P_2(x_2,f(x_2))$ $(x_1<x_2)$,存在$x_0\in(x_1,x_2)$,使得$f(x)$的图象在$x=x_0$处的切线$m//P_1P_2$,求证 \[x_0<\frac{x_1+x_2}2.\] 由已知得 \[f'(x_0)=\frac{f(x_1)-f(x_2)}{x_1-x_2},\] 代入数据解得 \[2x_{0}-\frac{a}{x_{0}}=x_{1}+x_{2}-\frac{a(\ln x_{1}-\ln x_{2})}{x_{1}-x_{2}},\] 下面证明 \[\frac{\ln x_{1}-\ln x_{2}}{x_{1}-x_{2}}>\frac{2}{x_{1}+x_{2}},(*)\] 令$\frac{x_{1}}{x_{2}}=t\in(0,1)$,则上式等价于 \[\ln t<2-\frac{4}{t+1},\] 令$g(t)=\ln t-2+\frac{4}{t+1}$,则$g(1)=0$且 \[g'(t)=\frac{(1-t)^{2}}{t(1+t)^{2}}>0,\] 从而当$t\in(0,1)$时必有$g(t)<0$,式(*)得证,于是由$a>0$得到 \[2x_{0}-\frac{a}{x_{0}}=x_{1}+x_{2}-\frac{a(\ln x_{1}-\ln x_{2})}{x_{1}-x_{2}}<2\cdot \frac{x_{1}+x_{2}}{2}-\frac{a}{\frac{x_{1}+x_{2}}{2}},\] 显然当$a>0,x>0$时$h(x)=2x-\frac ax$为增函数,从而由上式有 \[h(x_0)<h\left(\frac{x_1+x_2}2\right) \iff x_0<\frac{x_1+x_2}2.\]
thread-315-1-1.html: 洛必达法则
yayaweha 1# 2012-1-19 19:58
谁能证明一下洛比达法则。要正规一点的证明
kuing 2# 2012-1-19 21:34
翻翻高等数学教材就有了啊
海盗船长 3# 2012-1-19 22:44
thread-316-1-1.html: 其实我一直没搞懂为什么经常看到这样的括号
kuing 1# 2012-1-20 02:05
想不懂他们是怎么输入的 明明就是在公式编辑器里打,而且又不是紧跟着中文,怎么会出来全角的呢?
pxchg1200 2# 2012-1-20 14:29
这个应该是输入法的原因,在搜狗拼音模式下打的括号是( )在英文模式下是(),明显不一样了。
kuing 3# 2012-1-20 14:40
2# pxchg1200 但是那公式没出现中文,按道理说并不需要切换拼音输入法啊
pxchg1200 4# 2012-1-21 10:18
3# kuing 也许不是一口气打出来的,可能打着打着又和别人聊天吧。
kuing 5# 2012-1-21 12:12
4# pxchg1200
kuing 6# 2012-3-26 19:20
又见 不过这个倒是好理解,因为冒号和第一个 ( 跟着中文的后面,所以是文字状态,后面才变回英文,然后又不是继续打,而是复制前面,结果三个都这样…………
kuing 7# 2012-3-29 23:42
又见,http://bbs.pep.com.cn/thread-678165-1-1.html,(...)
kuing 8# 2012-4-2 17:14
又见 这个比较汗 各种乱七八zao
图图 9# 2012-4-2 20:30
8# kuing
kuing 10# 2012-4-8 20:15
又见 又是属于无法理解的
kuing 11# 2012-6-8 23:34
又在群里见到神奇括号
isea 12# 2012-6-27 20:36
很简单,因为有些人的默认输入法是中文! 导致键盘快捷键基本全费!!
kuing 13# 2012-6-27 21:14
很简单,因为有些人的默认输入法是中文! 导致键盘快捷键基本全费!! isea 发表于 2012-6-27 20:36 那为什么往往只有一边括号是全角的,而且有的是左全右半,有的是右全左半,而且括号前后一般都有英文
kuing 14# 2012-11-15 23:22
在群看到的又一极品……括号、字体……
kuing 15# 2013-4-19 17:15
公式输入的问题还是决定移来这边……
yes94 16# 2013-4-27 18:36
10# kuing 2楼说的很对,我经常遇到输入法、括号等问题,但是我及时转换汉字模式,还是英语模式了
thread-317-1-1.html: 讨论数列敛散性
Nirvanacs 1# 2012-1-20 10:33
设 \(\lambda>-4\) , \(\displaystyle x_1=\frac{\lambda}{2},x_{n+1}=\frac{x_n^2}{2}+x_1\) 讨论\(\{x_n\}\)的敛散性.
icesheep 2# 2012-1-21 15:06
本帖最后由 icesheep 于 2012-1-21 17:07 编辑 画图目测是 x1<=1/2 即可,收敛到 1-sqrt(1-2x1)
Nirvanacs 3# 2012-1-21 16:17
2# icesheep 不对,\(\lambda\le 1\)时也有发散的情形.
icesheep 4# 2012-1-21 17:48
又画了个图。。目测 f(x1)>x'' 时,发散,其中 f(x)=x^2/2+x1 , x'' 为 f(x)=x 的较大根。嗯夹在当中的都收敛。。。
海盗船长 5# 2012-1-21 20:25
$\lambda<0$时应该是以$\lambda=-3$为分界
海盗船长 6# 2012-1-21 20:26
但是$\lambda<0$时我只证明到了$\lambda>-2$时收敛
Nirvanacs 7# 2012-1-22 00:20
5# 海盗船长 对,以这个为界
icesheep 8# 2012-1-22 11:17
本帖最后由 icesheep 于 2012-1-22 11:27 编辑 擦,怎么我看图都是收敛的呢? 啊难道x>-1.5 会一直打圈。。。
海盗船长 9# 2012-1-22 18:11
本帖最后由 海盗船长 于 2012-1-22 18:16 编辑 看到一个很有意思的,和这个很类似! ThreeViewsOfTheLogisticMap-sourcecode.rar (7.17 KB) http://demonstrations.wolfram.com/ThreeViewsOfTheLogisticMap/ http://mathworld.wolfram.com/WebDiagram.html
海盗船长 10# 2012-1-27 14:41
不会证明,求解答。
Nirvanacs 11# 2012-1-27 19:49
10# 海盗船长 提示 当 \(\lambda\le 0\) 时 \[\frac{\lambda}{2}\le x_{2k-1}\le 1-\sqrt{1-\lambda}\le x_{2k}\le 0\] 然后再 \(\lambda\ge -3\) 时 , 考虑 \(x_{2k+2}-x_{2k}\) 化成 \(x_{2k}\) 的四次方程 , 这里将会因为 \(\lambda\ge -3\) 导致\(x_{2k}\)的单调性,同样处理奇数项. 对 \(\lambda <-3\) 时 , 我用的反证法 , 假设收敛. 方法很蛋疼 , 不知道有没好解法 .
海盗船长 12# 2012-2-1 20:52
11# Nirvanacs 反证法然后用柯西准则吗?
海盗船长 13# 2012-2-1 21:46
本帖最后由 海盗船长 于 2012-2-1 22:05 编辑 $\lambda<0$时做代换:$\displaystyle y_n= \frac{1}{2} - \frac{x_n}{2(1+\sqrt{1-\lambda})}$ 得到 $\displaystyle y_n=(1+\sqrt{1-\lambda})y_n(1-y_n) \qquad y_0=\frac{1+\sqrt{1-\lambda}}{4}$ 是一个 Logistic Map
thread-318-1-8.html: [数列] 不动点
yayaweha 1# 2012-1-20 20:07
在求一阶递推数列时有一种方法叫不动点法,我想问下为什么这种不动点法可用于求数列通项,它的根源是什么?
icesheep 2# 2012-1-21 15:59
本帖最后由 icesheep 于 2012-1-21 16:06 编辑 我举一个2阶齐次线性递推的例子, \[{a_{n + 2}} = p{a_{n + 1}} + q{a_n}\] 由于这个递推是一个线性齐次的递推,所以递推关系可以写成如下的矩阵形式:\[\left[ {\begin{array}{*{20}{c}}   {{a_{n + 1}}} \\   {{a_n}} \end{array}} \right] = \left[ {\begin{array}{*{20}{c}}   p&q \\   1&0 \end{array}} \right]\left[ {\begin{array}{*{20}{c}}   {{a_n}} \\   {{a_{n - 1}}} \end{array}} \right]\] 我们记${X_{n + 1}} = \left[ {\begin{array}{*{20}{c}}   {{a_{n + 1}}} \\   {{a_n}} \end{array}} \right]$,$A = \left[ {\begin{array}{*{20}{c}}   p&q \\   1&0 \end{array}} \right]$ 那么整个问题就和等比数列 ${X_{n + 1}} = A{X_n}$ 没什么区别了 ,只是我们的公比变成了一个矩阵。 于是,问题变成了求矩阵的幂次,这个问题在线性代数中利用特征值和特征向量会得到解决。
thread-319-1-8.html: [不等式] 不等式$\sum\sqrt{(a^2+2bc)/(b^2+c^2)}\ge 3$
海盗船长 1# 2012-1-21 21:01
本帖最后由 海盗船长 于 2012-1-21 21:07 编辑 $a,b,c>0$,Prove that:\[  \sqrt{\frac{a^2+2bc}{b^2+c^2}}+\sqrt{\frac{b^2+2ca}{c^2+a^2}}+\sqrt{\frac{c^2+2ab}{a^2+b^2}}\ge 3 \] http://tieba.baidu.com/p/1383163620
mathjohnfh 2# 2012-1-21 22:47
本帖最后由 mathjohnfh 于 2012-1-21 22:58 编辑 $\sqrt{\dfrac{a^{2}+2bc}{(b-c)^{2}+2bc}}$ 假设$a^{2}>(b-c)^{2}$ 则$(a+b-c)(a+c-b)>0$ b>c-a,c>b-a $b^{2}>(c-a)^{2}$  ,$c^{2}>(b-a)^{2}$ 以此类推
mathjohnfh 3# 2012-1-21 22:51
本帖最后由 mathjohnfh 于 2012-1-21 22:59 编辑 貌似在那本高中竞赛书上看到过,没大印象了
mathjohnfh 4# 2012-1-21 22:56
相当于三个项都大于一但不严谨
海盗船长 5# 2012-1-22 12:20
看到答案了:http://tieba.baidu.com/p/1383596268
pxchg1200 6# 2012-1-22 13:40
5# 海盗船长 那个sea rover 是你吧!
海盗船长 7# 2012-1-22 13:47
6# pxchg1200 嗯是的
thread-32-1-1.html: Zoom设置
kuing 1# 2011-9-28 14:27
\begin{array}{ccc}   \searrow & \downarrow & \swarrow \\   \rightarrow & 囧 & \leftarrow \\   \nearrow & \uparrow & \nwarrow \end{array}
thread-320-1-4.html: [几何] 解析几何之抛物线。
Chetion 1# 2012-1-23 20:15
已知点P( m, 3 )是抛物线 y =x ^2 + 4 x + n上距点(-2 ,0 )最近的一点。 求证: m + n = 5
yes94 2# 2013-2-3 22:39
很吓人的一道题,
thread-321-1-1.html: 根号求极限!
海盗船长 1# 2012-1-23 22:00
本帖最后由 海盗船长 于 2012-1-23 22:15 编辑 设$\displaystyle x_n=\sqrt{1+2\sqrt{1+3\sqrt{1+4\sqrt{\cdots +n\sqrt{1+(n+1)}}}}} \qquad (n \in \mathbb{N})$ 求证:\[ \lim_{n \to \infty} x_n=3 \] 来源:http://tieba.baidu.com/p/1380265171 参考:http://tieba.baidu.com/p/1238013416
海盗船长 2# 2012-1-27 12:00
答案:http://www.artofproblemsolving.c ... ?f=296&t=460517
GAM 3# 2012-1-27 18:05
长见识
thread-322-1-1.html: 习题1.3.5
GAM 1# 2012-1-26 17:02
题目都来自卓里奇的数学分析,网上没有答案(或者是我没找到),大多评论都觉得这书习题太难,不做也罢, 我扫了一眼前两章的,确实很蛋疼(好像明白又不会整 )。。。 仍觉得扔掉可惜,坛子里学数学的好像很多,就贴在这,觉得好像要总问,就把那个编号也弄上了。。。
GAM 2# 2012-1-26 17:24
都贴上来好了,都云里雾里的。。。
kuing 3# 2012-1-26 23:26

海盗船长 4# 2012-1-27 12:04
一般的数分书上都没有细讲这些……
GAM 5# 2012-1-27 17:29
了解,找了几本,就这个写了,第二章才开始正常起来。。。
pxchg1200 6# 2012-1-27 19:09
  我们都是看的菲赫金哥尔茨的《微积分学教程》,那个更简单点。这个貌似加入了很多集合论的东东。
GAM 7# 2012-1-27 19:32
咳咳。。。那个也在看,传说中教程上例题在卓里奇就被当成习题。。。 就想结合着看,结果相当于生生砍掉一章。。。
Nirvanacs 8# 2012-1-27 19:55
7# GAM 教程上木有这样的题
①②③④⑤⑥⑦ 9# 2012-1-30 17:10
本帖最后由 ①②③④⑤⑥⑦ 于 2012-2-13 11:06 编辑 1a 谢谢秋风树林提醒,这个题目有问题,应该是 $R_1\circ R_2=\Delta_Y$,$R_2\circ R_1=\Delta_X$, 任取 $x\in X$,存在 $y\in Y$ 使得 $(x,y)\in R_1$,且$(y,x)\in R_2$,此时若有 $y'\in Y$使得 $(x,y')\in R_1$,结合 $(y,x)\in R_2$ 可得到 $(y,y')\in R_1\circ R_2=\Delta_Y$,于是 $y'=y$。因此 $R_1$ 是函数。 下面不详细写了,都是类似的,整个1a都很容易 1b 充分性:任取 $(x,x'')\in R\circ R$,按定义,存在 $x'$ 使得 $(x,x')\in R$,$(x',x'')\in R$,根据传递性即有 $(x,x'')\in R$,也就是 $R\circ R\subset R$。 必要性:若存在元素 $x,x',x''$,使得 $(x,x')\in R$,$(x',x'')\in R$,则 $(x,x'')\in R\circ R\subset R$,也就是 $R$ 具有传递性。 1c 充分性:任取 $(x,y)\in R\cap R'$,也就是 $(x,y)\in R$ 且 $(x,y)\in R'$,后者等价于$(y,x)\in R$,根据反对称性必有 $x=y$,因此 $(x,y)\in\Delta_X$,也就是 $R\cap R'\subset\Delta_X$。 必要性:若存在元素 $x,y$ 使得 $xRy$ 与 $yRx$ 同时成立,后者就是 $xR'y$,则 $(x,y)\in R\cap R'\subset\Delta_X$,即有 $x=y$,也就是满足反对称性。 1d 充分性:任取二元素 $x,y$,$xRy$ 与 $yRx$ 至少有一个成立,后者就是 $xR'y$,也就是 $(x,y)\in R\cup R'$,即 $X^2\subset R\cup R'$,而 $R\cup R'\subset X^2$ 是显然的,因此 $R\cup R'=X^2$。 必要性:任取二元素 $x,y\in X$,由于 $R\cup R'=X^2$,我们有 $(x,y)\in R\cup R'$,即 $xRy$ 与 $xR'y$ 至少有一个成立…… 可以用符号表述得更简略一点,懒得弄了,先这样看看吧。其他的以后用空再说。
①②③④⑤⑥⑦ 10# 2012-2-1 13:27
本帖最后由 ①②③④⑤⑥⑦ 于 2012-2-1 13:30 编辑 2# GAM 正好有空,看一下第3题,关键还是一对多和多对一的差别,练习基本的集合关系证明。所有这些命题其实都是显然的。 3a $\forall y\in f(A), \exists x\in A, \textrm{s.t.} f(x)=y$,由于 $A\subset B$,因此 $x\in B$,也就有 $y=f(x)\in f(B)$,这就证明了前半部分。 后半部分举个反例,令 $A=\{-1,1\}, B=\{1,2\}, f(x)=x^2$ 即可。 3b  直接用定义搞定。 3c $\forall y\in f(A\cap B), \exists x\in A\cap B, \textrm{s.t.} f(x)=y$,由  $x\in A$ 可知 $y=f(x)\in f(A)$,同理 $y\in f(B)$,于是  $y\in f(A)\cap f(B)$。 (无法证明两者相等,例如 $A=\{-1,2\},  B=\{1,2\}, f(x)=x^2$) 3d 从3a知道 $f(A)\subset f(A\cup B), f(B)\subset f(A\cup B)$,所以 $f(A)\cup f(B)\subset f(A\cup B)$ 是显然的。 $\forall y\in f(A\cup B), \exists x\in A\cup B, \textrm{s.t.} f(x)=y$,若 $x\in A$ 则 $y=f(x)\in f(A)$,若 $x\notin A$ 则必有  $x\in B$ 从而 $y=f(x)\in f(B)$,综合两方面即得 $y\in f(A)\cup f(B)$,也就是说 $f(A\cup B)\subset f(A)\cup f(B)$。 因此 $f(A\cup B)=f(A)\cup f(B)$ 3e 若 $f^{-1}(A')=\varnothing$,命题显然成立。 否则,$\forall x\in f^{-1}(A')$, 有 $f(x)\in A'\subset B'$,按定义即有 $x\in f^{-1}(B')$。 3f $x\in f^{-1}(A')\cap f^{-1}(B')\iff (x\in f^{-1}(A'))\wedge (x\in f^{-1}(B'))$ $\iff (f(x)\in A')\wedge (f(x)\in B')\iff f(x)\in A'\cap B'\iff x\in f^{-1}(A'\cap B')$ 3g $x\in f^{-1}(A')\cup f^{-1}(B')\iff (x\in f^{-1}(A'))\vee (x\in f^{-1}(B'))$ $\iff (f(x)\in A')\vee (f(x)\in B')\iff f(x)\in A'\cup B'\iff x\in f^{-1}(A'\cup B')$ 3h $f^{-1}(A'\backslash B')\cap f^{-1}(B')=f^{-1}((A'\backslash B')\cap B')=f^{-1}(\varnothing)=\varnothing$ $f^{-1}(A'\backslash B')\cup f^{-1}(B')=f^{-1}((A'\backslash B')\cup B')=f^{-1}(A')$ 原命题得证 3i  上一个的直接推论 3j 若 $A=\varnothing$,命题显然成立。 $x \in A\implies f(x)\in f(A)\iff x\in f^{-1}(f(A))$。 (无法证明两者相等,因为可能多对一,例如 $X=\{1,2\}, A=\{1\}, f(x)=1$) 3k 按定义,若 $x\in f^{-1}(B')$,则必有 $f(x)\in B'$,若 $f^{-1}(B')=\varnothing$,命题显然成立,否则,$\forall y\in f(f^{-1}(B')), \exists x\in f^{-1}(B') \textrm{s.t.} f(x)=y$,而前面又知道 $y=f(x)\in B'$,因此 $f(f^{-1}(B'))\subset B'$。 (无法证明两者相等,因为可能有B'中的元素没有原象,若每一个都有原象,那么两者是相等的)
秋风树林 11# 2012-2-10 18:14
本帖最后由 秋风树林 于 2012-8-8 15:02 编辑 咦?这不是我在群上问过的题么。。。 倒是第一道的第一问我觉得感觉有点问题就改了个条件 其实卓里奇的数分我觉得写得非常好。。。= = 编辑一下,当时做得乱七八糟以致于现在都看不懂的做法。。。
秋风树林 12# 2012-2-10 18:19
9L似乎有点问题啊。。。1a部分 此时若有 y′∈Y使得 (x,y′)∈R1,结合 (y,x)∈R2 可得到 (y,y′)∈R2∘R1=ΔY R2∘R1是先进行R1再有R2怎么来说都不对呃 我倒觉得(y,y')∈R1∘R2
秋风树林 13# 2012-2-10 18:35
要是直接题目条件的话倒是得出这个了,所以觉得题目条件有点问题。。。
①②③④⑤⑥⑦ 14# 2012-2-13 10:59
12# 秋风树林 你说得对,我都没好好看,照着他的结论就直接把各个字母打上去了,复合方向居然反了……
thread-33-1-1.html: 动态图演示公式输入及其他
kuing 1# 2011-9-28 14:35
由于以下几个动态图的尺寸比较大,如出现看不清,请点击图片放大观看,或点击图片下方的链接打开新窗口看。 一、基本输入格式 http://bbs.pep.com.cn/data/attac ... 3a9o2edbCN9CZkp.gif 注:尽管公式中能用中文,但还是建议都在英文状态下输公式,至少美元符号、命令什么的都必须是英文的。 上述动态图之实际效果: 行内公式:将公式代码放入两个美元符号中,例:$ax^2+bx+c=0$ 行间公式:将美元符号改一改即可,例:\[ax^2+bx+c=0\] 复杂一点的公式,比如这个方程的根:\[x_{1,2}=\frac{-b\pm\sqrt{b^2-4ac}}{2a}\]
kuing 2# 2011-9-28 14:36
二、环境及查看公式代码 http://bbs.pep.com.cn/data/attac ... ce0iJMEQUBxEhhT.gif 注: 在 & 处对齐,\\ 为换行符,最后一行的末尾不必加 \\ 。 查看公式代码的具体操作是:右键公式,选 Show Math As -> TeX Commands。 [2012-8-17]新注:后来增加了自动编号功能,故此这里用 align 环境会在每行公式后面自动编号,如果不想要编号,请用 align* 环境。 引用编号时请先在待引用的公式所在行加上一个 label 然后用 eqref 引用,相关内容请自行查阅任何一本 LaTeX 教程。 此外,单个行间公式需编号时,用 equation 环境,引用同上。 上述动态图之实际效果: 最常用的多行公式环境:align \begin{align} \sqrt{\frac{a^2+b^2}2}&\ge \frac{a+b}{2}\\ &\ge \sqrt{ab}\\ &\ge \frac2{\frac1a+\frac1b} \end{align}
kuing 3# 2011-9-28 14:37
三、设置点击公式放大 [[有部分已过时,现在已经默认设成双击放大,无需另外设置]] http://bbs.pep.com.cn/data/attac ... b89njtgXolIAHot.gif 注: 选择 Click 后单击公式就可以放大,而若选 Double-Click 则是双击放大,若选 Hover 则是鼠标移过去停留一下就会放大。 后面选的是放大的百分比,默认是200%。 动态图一开始出现的界面是 winedt,是我预先在那里的打好的代码copy进来这里而已,不必理会。 \begin{array}{ccc}   \searrow & \downarrow & \swarrow \\   \rightarrow & 囧 & \leftarrow \\   \nearrow & \uparrow & \nwarrow \end{array}
骨灰级爱好者 4# 2011-10-15 08:29
我也试试公式输入 $\ 2S_{n+1}=S_{n}^{2}-5
骨灰级爱好者 5# 2011-10-15 08:33
$2S_{n+1}=S_{n}^{2}-5 $
张平 6# 2011-11-8 23:17
$a_n=n^2$
张平 7# 2011-11-8 23:20
$\lim_{h->0}\frac{f(x)-f(x-h)}{h}=f'(x)$
kuing 8# 2011-11-8 23:31
7# 张平 -> 可以用 \to 代替 还可以试试用行间公式模式,这样 $h\to0$ 会显示到 $\lim$ 的正下方。
只如初见 9# 2011-11-9 09:41
\[y=3x^2+5\]
kuing 10# 2011-11-9 10:41
9# 只如初见 再试几个看看
kuing 11# 2011-11-9 12:54
\lim_{h\to0} \frac{f(x)-f(x-h)}h = f'(x) $ \lim_{h\to0} \frac{f(x)-f(x-h)}h = f'(x) $ \[ \lim_{h\to0} \frac{f(x)-f(x-h)}h = f'(x) \]
yayaweha 12# 2012-1-20 18:16
模仿一个看看  $lim_{Δx\to0}\frac{f(x+Δx)-f(x)/Δx$
yayaweha 13# 2012-1-20 18:18
/lim_{Δx\to0}\frac{f(x+Δx)-f(x)/Δx}/
kuing 14# 2012-1-20 18:23
请在英文状态下输入 Δ 要用命令打:\Delta 花括号要成对
ddeyes 15# 2012-1-30 21:49
本帖最后由 ddeyes 于 2012-1-30 21:51 编辑 $\lim_{h \to 0}\frac{f(x)-f(x-h)}{h}=f'(x)$
①②③④⑤⑥⑦ 16# 2012-1-31 08:58
请在英文状态下输入 Δ 要用命令打:\Delta 花括号要成对 kuing 发表于 2012-1-20 18:23 $$lim_{Δx\to0}\frac{f(x+Δx)-f(x)}{Δx}$$ $$lim_{\Delta x\to0}\frac{f(x+\Delta x)-f(x)}{\Delta x}$$
kuing 17# 2012-1-31 13:32
呃,lim 前还少一个 \ ……
①②③④⑤⑥⑦ 18# 2012-1-31 14:11
17# kuing 没注意,我只是看看 Δ
scientist 19# 2012-2-17 13:37
1# kuing 我也来试一下 $ax^2+bx+c=0$ \[ax^2+bx+c=0\]
scientist 20# 2012-2-17 13:39
哈哈,成功了!这个要发表后才看到效果的呀。我原来还以为在编辑帖子的时候就可见的呢。
thread-33-2-1.html:
kuing 21# 2012-2-17 13:48
20# scientist 是的,在编写时所想即所得,发出来才显示。
linzhuqin 22# 2012-3-2 19:36
$\ 2S_{n+1}=S_{n}^{2}-5$
linzhuqin 23# 2012-3-2 20:17
管理,我加你了哦,我的DZ www.hgxyz.com中也想加入这个程序,麻烦指点啊,谢谢。
kuing 24# 2012-3-3 01:11
你查看我这里任意页面上的源代码,将head里面跟MathJax相关的代码抄一下放到你那里的head里就行了。
linzhuqin 25# 2012-3-7 09:11
$\ 2S_{n+1}=S_{n}^{2}-5$
linzhuqin 26# 2012-3-7 09:48
$x_{1,2}=\frac{-b\pm\sqrt{b^2-4ac}}{2a}$
linzhuqin 27# 2012-3-7 10:15
\begin{align} \sqrt{\frac{a^2+b^2}2}&\ge \frac{a+b}{2}\\ &\ge \sqrt{a_1b_1}\\ &\ge \frac2{\frac1a+\frac1b} \end{align}
linzhuqin 28# 2012-3-7 12:11
本帖最后由 linzhuqin 于 2012-3-7 13:30 编辑 $$\lim_{h\to0} \frac{f(x)-f(x-h)}h = f'(x)$$$\lim_{h\to0} \frac{f(x)-f(x-h)}h = f'(x)$\[\lim_{h\to0} \frac{f(x)-f(x-h)}h = f'(x)\] \[a_2c_2\]
kuing 29# 2012-3-7 13:57
我这儿最近看 mathjax 的公式老是不太稳定,不知是不是跟网络有点关系,你那儿看显示正常吗?
linzhuqin 30# 2012-3-8 08:44
本帖最后由 linzhuqin 于 2012-3-8 09:19 编辑 $\Delta{ABC}$$\DeltaABC$ $${\left{frac123{26}}\right}$$ $${\left({\frac{x}{2}}\right)}$$ $${\left{{\frac{x}{2}}\right}}$$ $$\left.{\left \{ (z+0.4)\frac { H(z) }{ z }\right\} } \right_{ z=-0.4 }$$ $$f_{a}(x)=$$$$\{frac{123}{frac{1}{2}+a}}\}$$ $${\left( {\Frac{5*(x+y)}{x^2-y}\right)}*(12-x)$$
linzhuqin 31# 2012-3-8 10:30
本帖最后由 linzhuqin 于 2012-3-8 10:39 编辑 $\angle$  'angle' $\textdegree*$  'textdegree' $\celsius$   'celsius' $\tccentigrade$
linzhuqin 32# 2012-3-8 12:25
$\subseteq$
linzhuqin 33# 2012-3-8 14:29
本帖最后由 linzhuqin 于 2012-3-8 14:39 编辑 $$\lim_{n\to∞}T_n=\lim_{n\to∞}\frac{\left(\frac{1}{q}\right)^n-q}{{\left(\frac{1}{q}\right)^n-1}$$ $$\lim_{n\to0}T_n=\lim_{n\to0}\frac{\left(\frac{1}{q}\right)^n-q}{{\left(\frac{1}{q}\right)^n-1}$$ $$\lim_{n\to0}T_n=\lim_{n\to0}\frac{(\frac{1}{q})^n-q}{(\frac{1}{q})^n-1}}$$ $$lim_{h\to0} \frac{f(x)-f(x-h)}h = f'(x)$$ $$(\frac{1}{q})^n-q$$ $$(\frac{1}{q})^n-1$$ $$\frac{(\frac{1}{q})^n-q}{(\frac{1}{q})^n-1}$$ $$\lim_{n\to0}T_n=\lim_{n\to0}\frac{(\frac{1}{q})^n-q}{(\frac{1}{q})^n-1}$$
linzhuqin 34# 2012-3-8 15:11
$\times$ $\vec{\mathbf{B}}$
linzhuqin 35# 2012-3-8 15:50
$\equiv{x}$ $\hateq{x}$
linzhuqin 36# 2012-3-8 15:51
$\equiv{x}$ $\hateq{x}$ $\uiv{x}$ $\equ{x}$
linzhuqin 37# 2012-3-9 15:11
$\frac{1+4λ}{1+λ}=0$,故:$λ=\frac1{4}$
linzhuqin 38# 2012-3-10 09:54
\ge
linzhuqin 39# 2012-3-10 11:05
本帖最后由 linzhuqin 于 2012-3-10 11:08 编辑 $\pm{a}$
linzhuqin 40# 2012-3-21 17:39
本帖最后由 linzhuqin 于 2012-3-21 17:46 编辑 $\stackrel{a}{\rightarrow}$ $X\stackrel{\text{definition}}{\hbox{\equalsfill}} Y$ $X\stackrel{\text{definition}}{=} Y$
thread-33-3-1.html:
linzhuqin 41# 2012-3-22 08:44
化学方程式中的长等号怎么表示呢? 怎么让等号的上下输入字符串呢? $\Longleftarrow$
linzhuqin 42# 2012-3-22 09:08
本帖最后由 linzhuqin 于 2012-3-24 10:57 编辑 $CO_3^-$ $\_$
linzhuqin 43# 2012-3-23 13:57
本帖最后由 linzhuqin 于 2012-3-23 14:57 编辑 \begin{align} & K=5A \\ & C=25 \end{align} \begin{align} \{ \sqrt{\frac{a^2+b^2}2}&\ge \frac{a+b}{2}\\ &\ge \sqrt{ab}\\ &\ge \frac2{\frac1a+\frac1b} \end{align} $\left.{\left \{ (z+0.4)\frac { H(z) }{ z }\right\} } \right|_{ z=-0.4 }$ $\left \{ \begin{aligned} & a = b \\ & c = d \end{aligned} \right.$ $\left \{ \begin{aligned} a = b \\ c = d \end{aligned} \right\}$ $\left [ \begin{aligned} a = b \\ c = d \end{aligned} \right ]$
xwh19771018 44# 2012-4-30 11:30
本帖最后由 xwh19771018 于 2012-4-30 11:37 编辑 $|x-1|={[sqrt{2x}k]}/2$在区间[k-1,k+1] 上有不等两实数根,k的取值范围
kuing 45# 2012-4-30 13:00
$|x-1|={[sqrt{2x}k]}/2$在区间[k-1,k+1] 上有不等两实数根,k的取值范围 xwh19771018 发表于 2012-4-30 11:30 仔细看另一个置顶贴的代码输入。 打好了再去数学讨论区提问题吧。
jthuang1984 46# 2012-5-25 09:59
$f(x)=4x^3+3tx^2-6t^2x+t-1$
叶剑飞Victor 47# 2012-8-8 18:50
本帖最后由 叶剑飞Victor 于 2012-8-8 18:52 编辑 洛仑兹坐标变换公式 \[ \left\{ \begin{array}{l} x = \frac{x'+vt'}{\sqrt{1-\left(\frac{v}{c}\right)^2}} \\ y = y' \\ z = z' \\ t = \frac{t'+\frac{v}{c^2}x'}{\sqrt{1-\left(\frac{v}{c}\right)^2}} \end{array} \right. \]
叶剑飞Victor 48# 2012-8-8 18:55
本帖最后由 叶剑飞Victor 于 2012-8-17 02:02 编辑 Cardano公式: 一元三次方程\(ax^3+bx^2+cx+d=0 \, (a \ne 0)\)的解是: \begin{align*} x_1 = &-\frac{b}{3 a}\\ &-\frac{1}{3 a} \sqrt[3]{\frac12\left[2 b^3-9 a b c+27 a^2 d+\sqrt{\left(2 b^3-9 a b c+27 a^2 d\right)^2-4 \left(b^2-3 a c\right)^3}\right]}\\ &-\frac{1}{3 a} \sqrt[3]{\frac12\left[2 b^3-9 a b c+27 a^2 d-\sqrt{\left(2 b^3-9 a b c+27 a^2 d\right)^2-4 \left(b^2-3 a c\right)^3}\right]}\\ x_2 = &-\frac{b}{3 a}\\ &+\frac{1+i \sqrt{3}}{6 a} \sqrt[3]{\frac12\left[2 b^3-9 a b c+27 a^2 d+\sqrt{\left(2 b^3-9 a b c+27 a^2 d\right)^2-4 \left(b^2-3 a c\right)^3}\right]}\\ &+\frac{1-i \sqrt{3}}{6 a} \sqrt[3]{\frac12\left[2 b^3-9 a b c+27 a^2 d-\sqrt{\left(2 b^3-9 a b c+27 a^2 d\right)^2-4 \left(b^2-3 a c\right)^3}\right]}\\ x_3 = &-\frac{b}{3 a}\\ &+\frac{1-i \sqrt{3}}{6 a} \sqrt[3]{\frac12\left[2 b^3-9 a b c+27 a^2 d+\sqrt{\left(2 b^3-9 a b c+27 a^2 d\right)^2-4 \left(b^2-3 a c\right)^3}\right]}\\ &+\frac{1+i \sqrt{3}}{6 a} \sqrt[3]{\frac12\left[2 b^3-9 a b c+27 a^2 d-\sqrt{\left(2 b^3-9 a b c+27 a^2 d\right)^2-4 \left(b^2-3 a c\right)^3}\right]} \end{align*}
kuing 49# 2012-8-8 18:58
48# 叶剑飞Victor 根号里面的 1/2 为什么用 \tfrac ?有点小
shidilin 50# 2012-11-27 18:15
我也试试 $f(x)=ax^2(x-2)$
孤星赶月 51# 2012-12-22 13:17
本帖最后由 孤星赶月 于 2012-12-22 13:29 编辑 $q_3q_4q_3...$, 同时$q_3\models\phi$
kuing 52# 2012-12-22 13:47
$q_3q_4q_3...$, 同时$q_3\models\phi$ 孤星赶月 发表于 2012-12-22 13:17 $\models$ 是什么符号?
叶剑飞Victor 53# 2013-4-12 16:45
本帖最后由 叶剑飞Victor 于 2013-4-12 16:47 编辑 来个恶心的: 一元四次方程$ax^4+bx^3+cx^2+dx+e=0 (a \ne 0)$的求根公式: \begin{array}{l} {x_1=-\frac{b}{4a}+\frac{1}{2} \sqrt{\frac{b^2}{4a^2}-\frac{2c}{3a}+\frac{\sqrt[3]{2}\left(c^2-3bd+12ae\right)}{3a\sqrt[3]{2c^3-9bcd+27ad^2+27b^2e-72ace+\sqrt{-4\left(c^2-3bd+12ae\right)^3+\left(2c^3-9bcd+27ad^2+27b^2e-72ace\right)^2}}}+\frac{\sqrt[3]{2c^3-9bcd+27ad^2+27b^2e-72ace+\sqrt{-4\left(c^2-3bd+12ae\right)^3+\left(2c^3-9bcd+27ad^2+27b^2e-72ace\right)^2}}}{3\sqrt[3]{2}a}}-\frac{1}{2} \sqrt{\frac{b^2}{2a^2}-\frac{4c}{3a}-\frac{\sqrt[3]{2}\left(c^2-3bd+12ae\right)}{3a\sqrt[3]{2c^3-9bcd+27ad^2+27b^2e-72ace+\sqrt{-4\left(c^2-3bd+12ae\right)^3+\left(2c^3-9bcd+27ad^2+27b^2e-72ace\right)^2}}}-\frac{\sqrt[3]{2c^3-9bcd+27ad^2+27b^2e-72ace+\sqrt{-4\left(c^2-3bd+12ae\right)^3+\left(2c^3-9bcd+27ad^2+27b^2e-72ace\right)^2}}}{3\sqrt[3]{2}a}+\frac{-b^3+4abc-8a^2d}{4a^3\sqrt{\frac{b^2}{4a^2}-\frac{2c}{3a}+\frac{\sqrt[3]{2}\left(c^2-3bd+12ae\right)}{3a\sqrt[3]{2c^3-9bcd+27ad^2+27b^2e-72ace+\sqrt{-4\left(c^2-3bd+12ae\right)^3+\left(2c^3-9bcd+27ad^2+27b^2e-72ace\right)^2}}}+\frac{\sqrt[3]{2c^3-9bcd+27ad^2+27b^2e-72ace+\sqrt{-4\left(c^2-3bd+12ae\right)^3+\left(2c^3-9bcd+27ad^2+27b^2e-72ace\right)^2}}} {3\sqrt[3]{2}a}}}}}\\\\ {x_2=-\frac{b}{4a}+\frac{1}{2} \sqrt{\frac{b^2}{4a^2}-\frac{2c}{3a}+\frac{\sqrt[3]{2}\left(c^2-3bd+12ae\right)}{3a\sqrt[3]{2c^3-9bcd+27ad^2+27b^2e-72ace+\sqrt{-4\left(c^2-3bd+12ae\right)^3+\left(2c^3-9bcd+27ad^2+27b^2e-72ace\right)^2}}}+\frac{\sqrt[3]{2c^3-9bcd+27ad^2+27b^2e-72ace+\sqrt{-4\left(c^2-3bd+12ae\right)^3+\left(2c^3-9bcd+27ad^2+27b^2e-72ace\right)^2}}}{3\sqrt[3]{2}a}}+\frac{1}{2} \sqrt{\frac{b^2}{2a^2}-\frac{4c}{3a}-\frac{\sqrt[3]{2}\left(c^2-3bd+12ae\right)}{3a\sqrt[3]{2c^3-9bcd+27ad^2+27b^2e-72ace+\sqrt{-4\left(c^2-3bd+12ae\right)^3+\left(2c^3-9bcd+27ad^2+27b^2e-72ace\right)^2}}}-\frac{\sqrt[3]{2c^3-9bcd+27ad^2+27b^2e-72ace+\sqrt{-4\left(c^2-3bd+12ae\right)^3+\left(2c^3-9bcd+27ad^2+27b^2e-72ace\right)^2}}}{3\sqrt[3]{2}a}+\frac{-b^3+4abc-8a^2d}{4a^3\sqrt{\frac{b^2}{4a^2}-\frac{2c}{3a}+\frac{\sqrt[3]{2}\left(c^2-3bd+12ae\right)}{3a\sqrt[3]{2c^3-9bcd+27ad^2+27b^2e-72ace+\sqrt{-4\left(c^2-3bd+12ae\right)^3+\left(2c^3-9bcd+27ad^2+27b^2e-72ace\right)^2}}}+\frac{\sqrt[3]{2c^3-9bcd+27ad^2+27b^2e-72ace+\sqrt{-4\left(c^2-3bd+12ae\right)^3+\left(2c^3-9bcd+27ad^2+27b^2e-72ace\right)^2}}} {3\sqrt[3]{2}a}}}}}\\\\ {x_3=-\frac{b}{4a}-\frac{1}{2} \sqrt{\frac{b^2}{4a^2}-\frac{2c}{3a}+\frac{\sqrt[3]{2}\left(c^2-3bd+12ae\right)}{3a\sqrt[3]{2c^3-9bcd+27ad^2+27b^2e-72ace+\sqrt{-4\left(c^2-3bd+12ae\right)^3+\left(2c^3-9bcd+27ad^2+27b^2e-72ace\right)^2}}}+\frac{\sqrt[3]{2c^3-9bcd+27ad^2+27b^2e-72ace+\sqrt{-4\left(c^2-3bd+12ae\right)^3+\left(2c^3-9bcd+27ad^2+27b^2e-72ace\right)^2}}}{3\sqrt[3]{2}a}}-\frac{1}{2} \sqrt{\frac{b^2}{2a^2}-\frac{4c}{3a}-\frac{\sqrt[3]{2}\left(c^2-3bd+12ae\right)}{3a\sqrt[3]{2c^3-9bcd+27ad^2+27b^2e-72ace+\sqrt{-4\left(c^2-3bd+12ae\right)^3+\left(2c^3-9bcd+27ad^2+27b^2e-72ace\right)^2}}}-\frac{\sqrt[3]{2c^3-9bcd+27ad^2+27b^2e-72ace+\sqrt{-4\left(c^2-3bd+12ae\right)^3+\left(2c^3-9bcd+27ad^2+27b^2e-72ace\right)^2}}}{3\sqrt[3]{2}a}-\frac{-b^3+4abc-8a^2d}{4a^3\sqrt{\frac{b^2}{4a^2}-\frac{2c}{3a}+\frac{\sqrt[3]{2}\left(c^2-3bd+12ae\right)}{3a\sqrt[3]{2c^3-9bcd+27ad^2+27b^2e-72ace+\sqrt{-4\left(c^2-3bd+12ae\right)^3+\left(2c^3-9bcd+27ad^2+27b^2e-72ace\right)^2}}}+\frac{\sqrt[3]{2c^3-9bcd+27ad^2+27b^2e-72ace+\sqrt{-4\left(c^2-3bd+12ae\right)^3+\left(2c^3-9bcd+27ad^2+27b^2e-72ace\right)^2}}} {3\sqrt[3]{2}a}}}}}\\\\ {x_4=-\frac{b}{4a}-\frac{1}{2} \sqrt{\frac{b^2}{4a^2}-\frac{2c}{3a}+\frac{\sqrt[3]{2}\left(c^2-3bd+12ae\right)}{3a\sqrt[3]{2c^3-9bcd+27ad^2+27b^2e-72ace+\sqrt{-4\left(c^2-3bd+12ae\right)^3+\left(2c^3-9bcd+27ad^2+27b^2e-72ace\right)^2}}}+\frac{\sqrt[3]{2c^3-9bcd+27ad^2+27b^2e-72ace+\sqrt{-4\left(c^2-3bd+12ae\right)^3+\left(2c^3-9bcd+27ad^2+27b^2e-72ace\right)^2}}}{3\sqrt[3]{2}a}}+\frac{1}{2} \sqrt{\frac{b^2}{2a^2}-\frac{4c}{3a}-\frac{\sqrt[3]{2}\left(c^2-3bd+12ae\right)}{3a\sqrt[3]{2c^3-9bcd+27ad^2+27b^2e-72ace+\sqrt{-4\left(c^2-3bd+12ae\right)^3+\left(2c^3-9bcd+27ad^2+27b^2e-72ace\right)^2}}}-\frac{\sqrt[3]{2c^3-9bcd+27ad^2+27b^2e-72ace+\sqrt{-4\left(c^2-3bd+12ae\right)^3+\left(2c^3-9bcd+27ad^2+27b^2e-72ace\right)^2}}}{3\sqrt[3]{2}a}-\frac{-b^3+4abc-8a^2d}{4a^3\sqrt{\frac{b^2}{4a^2}-\frac{2c}{3a}+\frac{\sqrt[3]{2}\left(c^2-3bd+12ae\right)}{3a\sqrt[3]{2c^3-9bcd+27ad^2+27b^2e-72ace+\sqrt{-4\left(c^2-3bd+12ae\right)^3+\left(2c^3-9bcd+27ad^2+27b^2e-72ace\right)^2}}}+\frac{\sqrt[3]{2c^3-9bcd+27ad^2+27b^2e-72ace+\sqrt{-4\left(c^2-3bd+12ae\right)^3+\left(2c^3-9bcd+27ad^2+27b^2e-72ace\right)^2}}} {3\sqrt[3]{2}a}}}}}\\\\ {\Delta=256a^3e^3-192a^2bde^2-128a^2c^2e^2+144a^2cd^2e-27a^2d^4+144ab^2ce^2-6ab^2d^2e-80abc^2de+18abcd^3+16ac^4e-4ac^3d^2-27b^4e^2+18b^3 c d e-4 b^3 d^3-4 b^2 c^3 e+b^2 c^2 d^2} \end{array} 好吧,显示得一团糟,还是 http://mathurl.com/c7m26yt 显示得好一些
李斌斌755 54# 2013-5-11 14:20
\[f(x)=\left\{\begin{aligned} &ax+1,&&-1\leqslant x<0,\\&\dfrac{bx+2}{x+1},&&0\leqslant x\leqslant1,\end{aligned}\right.\] \[f(x)=\left\{\begin{align}&ax+1,&-1\leqslant x<1,\\&\dfrac{bx+2}{x+1},&0\leqslant x\leqslant1,\end{align}\right.\] \[f(x)=\begin{align}&ax+1,&-1\leqslant x<1,\\&\dfrac{bx+2}{x+1},&0\leqslant x\leqslant1,\end{align}\] \[f(x)=\begin{cases}&ax+1,&-1\leqslant x<1\\&\frac{bx+2}{x+1},&0\leqslant x\leqslant1\end{cases}\]
李斌斌755 55# 2013-5-11 14:21
54# 李斌斌755 有点意思
李斌斌755 56# 2013-5-11 14:23
还是这样漂亮 \[f(x)=\left\{\begin{aligned} &ax+1,&-1\leqslant x<0,\\&\dfrac{bx+2}{x+1},&0\leqslant x\leqslant1,\end{aligned}\right.\]
wangfb 57# 2013-5-14 17:33
本帖最后由 wangfb 于 2013-5-14 17:36 编辑 \[x^2+y^2=1\]
kuing 58# 2013-5-14 17:43
打开这个页面卡了一阵子,估计是因为53#的公式太长,mathjax 需要点时间才能排好……
kuing 59# 2013-5-14 17:43
赶紧翻个页先
kuing 60# 2013-5-14 17:44
赶紧翻个页先
thread-33-4-1.html:
kuing 61# 2013-5-14 17:44
赶紧翻个页先
kuing 62# 2013-5-14 17:44
终于不卡了
叶剑飞Victor 63# 2013-5-15 23:06
打开这个页面卡了一阵子,估计是因为53#的公式太长,mathjax 需要点时间才能排好…… kuing 发表于 2013-5-14 17:43 是的,一元四次方程的求根公式(http://mathurl.com/c7m26yt)确实太长了。。。。把MathJax卡得半死。
李斌斌755 64# 2013-5-18 14:29
这个叶尽整些生僻的公式
叶剑飞Victor 65# 2013-5-25 14:22
64# 李斌斌755 哪里生僻了?这公式挺著名的啦!
李斌斌755 66# 2013-5-25 15:22
65# 叶剑飞Victor 你自己用过吗
keypress 67# 2013-6-2 21:25
这些公式多是CAS算出来的。 mathematica:
thread-332-1-8.html: 一个很有意思的数学兼哲学问题,请高手解释下
yayaweha 1# 2012-1-28 12:38
在无限大的范围内,究竟是正整数多还是完全平方数多?====================肯定很多人说是正整数多。理由如下:正整数集合包含完全平方数集合且比他大,所以正整数比完全平方数多。但是有一种诡辩的论证证明两者一样多:因为在无限的范围内,一个正整数永远可以对应一个完全平方数,所以两者是一样多的。这种说法乍一看很有道理。到底哪个对!!!!!
GAM 2# 2012-1-28 13:20
康托尔笑了
kuing 3# 2012-1-28 13:28
集合势
①②③④⑤⑥⑦ 4# 2012-1-29 10:34
本帖最后由 ①②③④⑤⑥⑦ 于 2012-1-29 10:38 编辑 我们怎么比较数量的多少? 可以用“对应”,找一个单射,一个对应一个,哪一方有多出来的无法对应的,那就是数量多。如果整体和其中的一部分相比,可以用最自然的和自身对应的方式,这样有了“整体大于部分”,当然还可以发展出更多的方法,如果不涉及无限,无论怎样更换单射的方式,结果不变,多的总是多,少的总是少。 但是一旦涉及到了无限之间的比较,会发现这些规律不是这么好使,自然数和全体平方数比较,当然可以用自然的和自身对应,全体平方数是自然数的一部分,但是我们并不只有这一种对应方式,比如说 $f(n)=n^2$,这就形成了一一对应,表示两者一样多?我们还能这样: $f(n)=4n^2$,这下好,自然数集和平方数集的一个真子集建立一一对应了,那么平方数比自然数更多? 有这些例子,使我们明白了,原来的这些比较的方法不能简单的推广到无限的情形,解决方法无非是:1) 不比较,所有无限都一样。 2) 抛弃某些规则,并尽量采用兼容于有限的比较的法则。 放弃是很容易的,原有规则不成立,我们索性就不要比了,比如复数我们就不谈比较大小。但如果确实有用的话,还是会做些努力的,当然要有用,Cantor就做了这么件事情,尝试对无限进行分类,集合论的产生本来就是想要理清无限之间的关系,当然,同样能适用于有限的情形。没有采用“整体大于部分”,因为这很不好用,如果保留这条而抛弃一一对应的办法,不存在包含关系的就比较不起来,这么做是没有实用性的,数学家们当然不会这么干。他们保留了一一对应的方法,当然,严格来说是定义了集合的势这个新的名词,是可以互相比较,两个集合间只要存在一种一一对应的方法,就说是“等势”的,这种方法兼容有限集之间的比较,对于无限集,也有了一些办法,并且有了很多实际应用,得到了很多好的成果。所以现在,都是采用这些方法。现在把这些再拿出来,不能称之为诡辩什么的了,已经有了统一的规则了。 当然,事情远没有那么简单,建立的集合论,是有缺陷的,比如著名的罗素悖论。朴素集合论过渡到公理集合论,哪些可以作为公理?就像欧氏几何的平行公理会遭人怀疑一样,集合论里面的选择公理,也会令某些人不爽,而这又关系到势的三歧性,也就是我们要比较两个集合的势(通俗地讲,比较两者的多少),除了|A|=|B|,|A|<|B|(从A集合能构造到B的子集的一一对应,但不可能构造到B本身的一一对应,B也不能和A的任何子集建立一一对应),|A|>|B|外,会不会有第四种可能,也就是A、B之间无法建立任何一个一一对应,同时,A可以和B的某个子集一一对应,B也可以和A的某个子集一一对应。选择公理是“构造主义”所极其不喜欢的,因为承认它,就得接受有那种宣告存在但是无论如何都无法精确构造出来的对象,当然,作为构造主义的一类的“直觉主义”更不会喜欢它,因为他们不承认无穷对象的存在。 选择公理这些东西,才真正是数学、逻辑学、哲学的交汇处,事实上有时不用管什么“主义”,对我们构造出来的抽象世界,到底需要哪些规则,各人自然可以有不同的看法,存在性、合理性也都是人为认定的,最终关注的无非是方便、有用、无矛盾(在这片抽象世界中大多数人不能接受的事物)。你可以不接受平行公理,但也不是直接抛弃就完事的,不接受,就要允许各种更复杂的几何的存在,研究更麻烦的问题。同时,在实际应用中,可能在某些时候还是用的欧式几何,当然,使用者可以说这只是近似而图方便什么的(数学家不会那么小气的,合适的地方使用合适的理论)
yayaweha 5# 2012-1-29 13:39
讲的这么深奥
thread-333-1-2.html: 其实我挺喜欢这声线
kuing 1# 2012-1-31 00:32
http://v.youku.com/v_show/id_XMjU4NDQ4NTg4.html 求QQ号……
图图 2# 2012-2-20 10:12

kuing 3# 2012-2-20 14:23

图图 4# 2012-2-20 19:02
3# kuing
thread-335-1-2.html: 关于积分。。
Chetion 1# 2012-2-1 13:25
看完导数,再看积分。听起来循序渐进合情合理,做起来感觉就像从天堂到地狱。。。积分的计算彻底把我打败。有些积分运算的技术含量也实在太高了。。。回头看看导数,有基本公式和求导法则,只要可导,什么样的函数,都可以导出来。导数运算,已经建立了多么朴素而完整系统啊。 可是积分中,稍稍地多了两项的乘除,就没辙了。。。 好吧。以前学习数列时,数学归纳法救我于水火之中;空间几何里,向量曾做过我的救命稻草。积学的学习一定会出现转机。Complaints,到此为止。 大家来讨论讨论吧。。我认为积分运算需要一个完整的体系。既然导数可以。 ·关于积、商、复合函数的积分,可有相应的公式。 ·类比于由导数的定义推出一些常见函数的导数公式的方法,是不是我们也可以由积分的定义来小小地极限一下,推出更多的积分法则。
Chetion 2# 2012-2-1 13:31
看出点眉目了。大家看一下。 http://bbs.kaoyan.com/t3375158p1
海盗船长 3# 2012-2-1 20:29
积分作为导数的逆运算肯定会复杂很多的,不过基本的方法(凑微分、分部积分等)和基本的可积函数系对于一般的积分还是很有效的。
海盗船长 4# 2012-2-1 20:49
至于“积、商、复合函数的积分”,应该不会有普遍的公式(如果有早就应该被前人发现了吧。。)。而且不定积分也不是直接定义在极限的基础上,而是建立在导数之上,想通过求极限来导出不定积分的公式会更加困难。
海盗船长 5# 2012-2-1 20:56
其实一般数分书上的各种求积分的方法就可以解决绝大多数的不定积分了,剩下的就看你的技巧和经验了
thread-339-1-8.html: [数列] 请教与数列结合的概率题
wenshengli 1# 2012-2-2 18:02
___________kuing edit in $\LaTeX$___________ 四楼的正确题目如下: 以掷硬币的结果,决定数轴上两点$A$,$B$的移动情况。 出现正面者:若点$A$的坐标比点$B$的坐标大时,$A$,$B$共同沿正方向移1;不然,$A$向正方向移1。 出现反面者:若点$B$的坐标比点$A$的坐标大时,$A$,$B$共同沿正方向移1;不然,$B$向正方向移1。 最初,$A$,$B$两点均在原点,上述掷币操作$n$次后,$A$,$B$到达点的坐标分别是$a$,$b$。 (1)硬币投掷$n$次,正、反面出现的总情况数为$2^n$,把$a=b$的情况记为$X_n$,试写出$X_n$与$X_{n+1}$之间的关系式; (2)求$X_n$的表达式; (3)硬币投掷$n$次后,正、反面出现的总情况数为$2^n$,求$a$的期望值。
①②③④⑤⑥⑦ 2# 2012-2-3 09:12
1# wenshengli 这道题有点问题 出正面:若A的坐标比B大,两者同时向正方向移动,间距不变,若B的坐标不小于A,B向正方向移动,两者间距增加1 出反面:若B的坐标比A大,两者间距不变,不然,两者间距增加1 除了最开始同在原点,之后再也不会重合的,中间有哪些字抄错了吧
海盗船长 3# 2012-2-3 12:40
说反了吧
wenshengli 4# 2012-2-3 17:15
谢谢,2楼和3楼,两个字母打反了,重新发一个。
①②③④⑤⑥⑦ 5# 2012-2-7 16:38
还是从间距分析,只有以下三种情况 1. 间距不变 2. 落后者向领先者靠近一步 3. 两者在同一位置,其中某一个向前跨一步 我们从A,B在同一位置开始 第一次抛硬币,其中某一个领先一步,此时再抛,如果和之前的是同一面,那么间距不变,如果抛出的结果和最初的不同,两者就再次抵达同一位置。 $X_n$ 到底是什么,抛 $n$ 次后恰好重合的情况数?我想应该是的吧。 抛了n次以后,要么重合,要么有一个领先另一个一步,如果重合,那么抛第 $n+1$ 次后一定不会重合,如果不重合,那么要求下一次抛出和第 $n$ 次的结果不同,两者就能重合,不然就不重合。所以 $X_{n+1}=2^n-X_n$。 显然 $X_1=0$。 $X_n=\dfrac{2^n}3+(-1)^n\dfrac23$ 下面记 $P_n$ 为抛硬币 $n$ 次,所有 $2^n$ 种结果下 $a$ 值的总和,要求的数学期望就是 $\dfrac{P_n}{2^n}$。 我们还是建立递推式,抛了 $n$ 次后,要么重合,要么A领先B一步,要么B领先A一步,且后面两种情况的数量相等(对换序列中的正反面建立一一对应,具体不扯了) 1) $X_n$ 种情况,AB重合,第 $n+1$ 次要抛出正面,$a$ 值增加一,抛出反面 $a$ 值不变。 2) $2^{n-1}-\dfrac{X_n}2$ 种情况,A领先B,若第 $n+1$ 次要抛出正面,$a$ 值增加一,抛出反面 $a$ 值不变。 3) $2^{n-1}-\dfrac{X_n}2$ 种情况,B领先A,此时无论抛出正面还是反面,$a$ 值都会增加一。 也就是说: $P_{n+1}=2P_n+X_n+2^{n-1}-\dfrac{X_n}2+2\left(2^{n-1}-\dfrac{X_n}2\right)=2P_n+2^{n-1}+2^n-\dfrac{X_n}2$ $=2P_n+2^{n-1}+2^n-\dfrac{2^{n-1}}3-(-1)^n\dfrac13=2P_n+\dfrac43\cdot 2^n-(-1)^n\dfrac13$ 又有 $P(1)=1$ 于是可以得到 $P(n)=\dfrac{2^n}9\left(6n-1+\left(-\dfrac12\right)^n\right)$ 所求的数学期望为 $\dfrac19\left(6n-1+\left(-\dfrac12\right)^n\right)$
wenshengli 6# 2012-2-8 11:17
5# ①②③④⑤⑥⑦ 非常感谢!
thread-34-1-4.html: [数列] 不动点求通项,复数特征根照样可行$a_{n+1}=(1+a_n)/(1-a_n)$
kuing 1# 2011-9-28 21:24
问题转自 http://ask.maths168.com/?q-57.html 数列 $\{a_n\}$ 满足 $a_1=2$,$a_{n+1}=\dfrac{1+a_n}{1-a_n}$,则 $\{a_n\}$ 的通项如何? 利用不动点法,$x=\dfrac{1+x}{1-x}$ 的两根为 $\pm i$,尽管为虚数,照板煮碗,得到 \[\frac{a_{n+1}-i}{a_{n+1}+i}=i\cdot\frac{a_n-i}{a_n+i}\] 故 \[\frac{a_n-i}{a_n+i}=i^{n-1}\cdot\frac{2-i}{2+i}\] 解出 $a_n$ 并整理得 \[a_n=\frac{(2-i)i^n+2i-1}{2+i+(2i+1)i^n}\] 由此已经可以看出 4 是 $a_n$ 的周期,分别代入 $n=1,2,3,4$ 也可得到不同的实数,所以尽管有虚数,但同样无碍表示通项。 进一步,对上式分母实数化,再用三角化简,还可以将复数消去,得到 \[a_n=\frac{10\cos\frac{n\pi}{4}}{3\cos\frac{n\pi}{4}-\sin\frac{n\pi}{4}}-3\]
yes94 2# 2013-2-3 21:38
$a_n=\tan(\theta+\dfrac{(n-1)\pi}{4})$,其中$\tan\theta=2$。 由此看出,这个关于正切函数的周期$T=\dfrac{\pi}{\frac{\pi}{4}}=4$。
thread-340-1-1.html: [转]标号填空题
kuing 1# 2012-2-2 20:56
http://bbs.ctex.org/viewthread.php?tid=64792 \newcounter{exer} \renewcommand\theexer{(\arabic{exer})} \newcommand\putblank{\underline{\quad\stepcounter{exer}\theexer\quad}} 值得学习
thread-341-1-2.html: 凌空架杆
kuing 1# 2012-2-3 02:05
http://v.youku.com/v_show/id_XMTU0OTQyODA0.html
海盗船长 2# 2012-2-3 12:38
为啥最后白球还没停就打了?而且最后还弹出去了。。
①②③④⑤⑥⑦ 3# 2012-2-3 12:47
本帖最后由 ①②③④⑤⑥⑦ 于 2012-2-3 12:50 编辑 2# 海盗船长 明显是故意犯规啊,打到粉球后如果差距大于7分该局结果就定下了,黑球打不打无所谓,这种时候随便玩一下裁判不会多说啥的,见过好多次有人故意黑球白球同时进袋的,这个视频里玩得有点过火
kuing 4# 2012-2-3 14:44
嗯,打完绿球就超分了 视频里的奥沙利文打到完全进入状态打到随心所欲最后自然就这样耍了
①②③④⑤⑥⑦ 5# 2012-2-3 16:55
4# kuing 关键是玩凌空架杆的时候还没超分呢,不知啥心态去打哪样的球,架杆在那里抖啊抖的,倒是没影响出球。话说,有没有单手直接捅球的?
kuing 6# 2012-2-3 17:02
5# ①②③④⑤⑥⑦ 那倒是没见过……
kuing 7# 2012-3-3 14:32
4# kuing 话说,有没有单手直接捅球的? ①②③④⑤⑥⑦ 发表于 2012-2-3 16:55 有了:http://v.youku.com/v_show/id_XMzIzMTE1ODg0.html 丁俊辉的 不过那一刻毫无压力
thread-343-1-1.html: 除法竖式等
kuing 1# 2012-2-3 14:36
http://blog.sina.com.cn/s/blog_5e16f1770100nd48.html 中间说的那个 Polynom宏包 够牛,第一次见到 latex 有这样的运算包
thread-344-1-1.html: 矩阵的列数限制
kuing 1# 2012-2-3 16:51
amsmath 里面定义了一个 MaxMatrixCols=10 这样一个数字,限制了每个 matrix 的列数不能超过10,于是当遇到超过10列的矩阵需要输入的时候可以给这个数据加个数,比如 \addtocounter{MaxMatrixCols}{5},这样就变成了 MaxMatrixCols=15。或者直接赋值也行 \setcounter{MaxMatrixCols}{15} 就变成15
thread-345-1-1.html: 斜分数线长度自动适应
kuing 1# 2012-2-3 17:00
\left. 分子 \middle/ 分母 \right. 小缺点是整个分式的两边会多出一点点小距离 集合的竖线也可以这样打 \left\{ x \middle| x>\frac12 \right\} middle 命令在 LaTeX 中其实无需 amsmath 支持,刚才测试过什么宏包都没加也行,现在 mathjax 也支持了,演示一下: \[\left\{ x \middle| x>\frac12 \right\}\] 其实如果是我的话,还是看情况手动打适当长度好了哩 \[\frac12 \bigg/ 3\]\[1 \Big/ \sum a\]还可以自己微调,比如加 \! 什么的\[\frac12 \! \bigg/ \! 3\]\[1 \! \Big/ \! \sum a\]
thread-346-1-1.html: 上次秋风在群里提到的那个字母
kuing 1# 2012-2-3 17:41
是不是这个:$\Huge \aleph$
秋风树林 2# 2012-2-3 17:45
哈哈。。。顺便路过一下。。。
kuing 3# 2012-2-3 17:52
你的头像 $\aleph_0$
秋风树林 4# 2012-2-3 18:07
3# kuing 嘿嘿嘿嘿。。。。
thread-347-1-2.html: BE
贵族风铃 1# 2012-2-3 19:36
哎 我其实 也不知道吹什么好。。
kuing 2# 2012-2-3 22:53
晒晒你的…………
kuing 3# 2012-2-4 01:37
A:老鼠为什么会飞? B:神马,老鼠会飞? A:因为老鼠吃了仙丹。 B:na…ni? A:蛇为什么会飞? B:吃了仙丹了! A:因为蛇吃了老鼠。 B:……… A:老鹰为什么会飞? B:因为吃了蛇! A:傻比,老鹰本来就会飞。
海盗船长 4# 2012-2-4 11:22
哈哈哈哈
海盗船长 5# 2012-2-4 11:24

海盗船长 6# 2012-2-4 11:28

kuing 7# 2012-2-4 12:21
6# 海盗船长 这个看不到
kuing 8# 2012-2-4 18:28
咦,现在能看到了
thread-349-1-2.html: Riemann 猜想漫谈
海盗船长 1# 2012-2-4 23:00
http://www.changhai.org/articles ... ypothesis/index.php
thread-35-1-4.html: [数列] 数列题,来自超级群$a_{n+1}=a_1a_2\cdots a_n-1$
kuing 1# 2011-9-29 14:41
已知数列 $\{a_n\}$ 满足:$a_1=1$,$a_2=2$,$a_3=3$,$a_4=4$,$a_5=5$,且当 $n\geqslant5$ 时,$a_{n+1}=a_1a_2\cdots a_n-1$,若数列 $\{b_n\}$ 满足对任意 $n\in\mathbb{N}^+$,有 $b_n=a_1a_2\cdots a_n-a_1^2-a_2^2-\cdots-a_n^2$,则当 $n\geqslant5$ 时 $b_n=?$ solution 当 $n=5$ 时,$b_5=5!-1^2-2^2-3^2-4^2-5^2=65$; 当 $n\geqslant6$ 时,我们直接计算 $a_6$,为 $a_6=5! - 1 = 119$,并且由递推式,我们有\[ a_{n + 1} + 1 = a_1 a_2 \cdots a_n = a_n (a_1 a_2 \cdots a_{n - 1} ) = a_n (a_n + 1) \]从而得到\[ a_n^2 = a_{n + 1} - a_n + 1 \]由此,我们有\begin{align*} a_1^2 + a_2^2 + \cdots + a_n^2 & = a_1^2 + a_2^2 + a_3^2 + a_4^2 + a_5^2 + a_7 - a_6 + 1 + a_8 - a_7 + 1 + \cdots + a_{n + 1} - a_n + 1 \\ &= 1^2 + 2^2 + 3^2 + 4^2 + 5^2 + a_{n + 1} - a_6 + n - 6 + 1 \\ &= a_{n + 1} + n - 69 \end{align*}所以\[ b_n = a_1 a_2 \cdots a_n - a_1^2 - a_2^2 - \cdots - a_n^2 = a_{n + 1} + 1 - (a_{n + 1} + n - 69) = 70 - n \]又当 $n=5$ 时也适合上式,所以当 $n\geqslant5$ 时 $\{b_n\}$ 的通项为\[ b_n = 70 - n \] [edit 1 time]
①②③④⑤⑥⑦ 2# 2011-9-29 15:52
写成分式形式,"由此,我们有……" 这一部分多余了吧,条件里直接有
kuing 3# 2011-9-29 16:01
2# ①②③④⑤⑥⑦ 嗯,纯粹多余。。。。
kuing 4# 2011-9-29 16:06
删掉了
yes94 5# 2013-2-3 21:23
同一个数列,可以有不同的递推公式
thread-350-1-2.html: 弱弱地问下:吕布的坐骑是赤兔还是貂蝉?
kuing 1# 2012-2-5 02:19
  
pxchg1200 2# 2012-2-5 14:05
你懂的!
海盗船长 3# 2012-2-5 19:27
thread-351-1-8.html: [几何] 一道解析几何题
Chetion 1# 2012-2-5 14:53
本帖最后由 Chetion 于 2012-2-5 15:03 编辑 Q:已知定点M(-1,2),直线$y=k(x+1)$与曲线$y=-\sqrt{x^2+1}$交于A、B两点,N点平分弦AB,试将直线MN在y轴上截距表示为k 的函数,并求出它的值域。 A:$b=2k^2+k$  ( $k\in(-1,-\dfrac{\sqrt{2}}{2})$) 值域为$(1-\dfrac{\sqrt{2}}{2},1)$
thread-352-1-8.html: [几何] 关于圆锥曲线的通式
Chetion 1# 2012-2-5 15:05
本帖最后由 Chetion 于 2012-2-5 15:13 编辑 Q:求过两曲线 $x^2+2y^2-2=0$ 和 $2x^2-y^2-2=0$ 的交点、并且被y轴截得弦长为$\sqrt{13}$的圆锥曲线方程。 圆锥曲线的方程可能不是标准形式,甚至可以可能有过旋转。。。 这样的话,关于平几中圆锥曲线的通式,是不是如下: $Ax^2+By^2+Cxy+Dx+Ey+F=0$($A\cdot B\cdot C\ne0$,当然构成圆锥曲线还有很多条件)
kuing 2# 2012-2-5 15:48
A:用二次曲线系,设 $\lambda_1(x^2+2y^2-2)+\lambda_2(2x^2-y^2-2)=0$,然后……略 习惯上,二次曲线,有的写作 $Ax^2+Bxy+Cy^2+Dx+Ey+F=0$,有的则喜欢写作 $Ax^2+2Bxy+Cy^2+2Dx+2Ey+F=0$(没本质分别,只是在某些结论上显得简洁好看些),限制条件为 $A,B,C$ 不同时为0。 and,二次曲线的分类已经有完整的结论,找本二次曲线的书来看看即可。
thread-353-1-8.html: [函数] 极限问题
syzychenwj 1# 2012-2-5 17:38
(x→0)limf(x)=f(0)=1,f(2x)-f(x)=x^2,求f(x)
kuing 2# 2012-2-5 18:54
人教那边解得快,我不重复了
thread-354-1-8.html: [不等式] 对数—几何平均不等式
海盗船长 1# 2012-2-6 10:58
\[ \frac{\ln{a}-\ln{b}}{a-b}<\frac{1}{\sqrt{ab}} \qquad (a>b>0)\] 等价于$\ln{t}<\frac{1}{2}\left( t-\frac{1}{t} \right) \qquad (t>1)$
kuing 2# 2012-2-6 15:15
呃,印象中还有好多类似的平均,在匡的书里N多,不过我一直没怎么看过……
$\LaTeX$ 3# 2012-2-8 14:22
求导
$\LaTeX$ 4# 2012-2-8 14:26
http://kkkkuingggg.5d6d.com/thread-314-1-1.html 这里也有一个对数平均
$TeX$ 5# 2012-2-9 20:39
呵呵,另一边啊。\[ \frac{2}{a+b}<\frac{\ln{a}-\ln{b}}{a-b} \qquad (a>b>0) \] 等价于$\ln{t}>\frac{2(t-1)}{t+1} \qquad (t>1)$
thread-355-1-1.html: tikz 作图之 fill 的特性
kuing 1# 2012-2-6 15:04
http://bbs.pep.com.cn/thread-2283493-1-1.html \begin{tikzpicture}[draw=red,line width=1.5pt] \draw[fill=white] (0,0) circle (6); \draw[fill=black] (0,-6) arc (-90:90:4) (0,2) arc (-90:-270:2) (0,6) arc (90:-90:6); \draw[fill=gray] (0,-6) arc (-90:90:2) (0,-2) arc (-90:-270:4) (0,6) arc (90:270:2) (0,2) arc (90:-90:4); \end{tikzpicture} \begin{tikzpicture} \draw[fill=white] (0,0) circle (6); \fill[black] (0,-6) arc (-90:90:4) (0,2) arc (-90:-270:2) (0,6) arc (90:-90:6); \fill[gray] (0,-6) arc (-90:90:2) (0,-2) arc (-90:-270:4) (0,6) arc (90:270:2) (0,2) arc (90:-90:4); \end{tikzpicture}
kuing 2# 2012-3-20 13:06
啥时候 MathJax 支持 TikZ 就爽了
thread-356-1-2.html: 开学了。。。
Chetion 1# 2012-2-6 21:13
看来接下来几个月里最后一次上论坛了。。。 作业冇完成呃。。。么办。。。
$\LaTeX$ 2# 2012-2-8 14:33
楼主哪里人啊
$\TeX$ 3# 2012-2-11 20:27
我也开学了,, 不过我们没有作业
thread-357-1-8.html: 向量最值
xr5252 1# 2012-2-9 19:37
已知边长为1的正方形ABCD位于第一象限,A D分别在X Y的正半轴(包括原点)滑动。则向量OB*向量OC的最大值为 ?
$TeX$ 2# 2012-2-9 20:59
1# xr5252 设$\displaystyle \textbf{OA}=(\cos{t},0) \quad \textbf{OD}=(0,\sin{t}) \qquad t \in [0,\frac{\pi}{2}]$ 则 $\displaystyle \textbf{OB}=\textbf{OA}+\textbf{AB}=(\cos{t}+\sin{t},\cos{t})$ $\displaystyle \textbf{OC}=\textbf{OD}+\textbf{DC}=(\sin{t},\cos{t}+\sin{t})$ $\displaystyle \Longrightarrow \textbf{OB} \cdot \textbf{OC}=(\cos{t}+\sin{t})^2 \le 2$
xr5252 3# 2012-2-9 22:08
本帖最后由 xr5252 于 2012-2-9 22:10 编辑 1的活用啊! 谢谢了!
$\TeX$ 4# 2012-2-10 11:12
3# xr5252 不是$1$也可以这样设啊
thread-358-1-1.html: $\int_{0}^{\infty} \frac{\sin{x}}{x} \mathrm{d}x=\frac{\pi}{2}$
$TeX$ 1# 2012-2-9 21:04
本帖最后由 $TeX$ 于 2012-2-9 21:06 编辑 如何利用公式:\[ \frac{\pi}{\sin{\alpha \pi}}=\frac{1}{\alpha}+\sum_{n=1}^{\infty} (-1)^{n}\frac{2 \alpha}{\alpha^2-n^2} \qquad (0< \alpha <1) \] 证明:\[ \int_{0}^{\infty} \frac{\sin{x}}{x} \mathrm{d}x=\frac{\pi}{2} \]
pxchg1200 2# 2012-2-10 10:05
1# $TeX$ 用罗巴切夫斯基的方法就可以了。
$\TeX$ 3# 2012-2-10 11:07
2# pxchg1200 没听说过这个方法,具体是怎么做的啊?
$\TeX$ 4# 2012-2-10 12:46
会了,谢谢
thread-359-1-2.html: 懒羊羊原来是镁铝!
kuing 1# 2012-2-10 00:13
http://v.youku.com/v_show/id_XMzM5MjQxMjUy.html
图图 2# 2012-2-20 10:05

kuing 3# 2012-2-20 14:23
thread-36-1-2.html: 只在一点可导的函数
海盗船长 1# 2011-9-29 17:16
试构造函数$f(x)$,使得$f(x)$在$x=0$处可导,而在其他任意点都不连续。
kuing 2# 2011-9-29 17:31
$f(x)=x\cdot g(x)$ 其中 $g(x)=\begin{cases}0,& x\in\mathbb{Q},\\ 1,& x\in\mathbb{R}\backslash\mathbb{Q}.\end{cases}$ 行不行……
海盗船长 3# 2011-9-29 17:39
好像在$x=0$处不可导吧 改成$f(x)=x^2g(x)$就可以了
kuing 4# 2011-9-29 17:51
3# 海盗船长 噢,没错,刚想漏了点东西。 这么看来,是不是 $x^n,n=2,3,\ldots$ 都行?甚至换成只要是 $h(0)=h'(0)=0$ 且其余地方不恒为 0 的连续函数均可?
①②③④⑤⑥⑦ 5# 2011-9-30 09:17
4# kuing 不恒为零?恒不为零? 有要求“只在”一点可微,其他点要保证不可微 其实给出的那个是典型例子,也说明了函数在一点可微,只保证在这一点连续,不保证在该点的某一领域内连续。 有例子存在即可,把表达式改的复杂点意义不大,可以考虑其他问题,比如在无理点可微而在有理点不可微的连续函数 还有,可以参照,如果g(x)是一个处处连续但处处不可微的函数,则xg(x)就是只在一点可微的连续函数
kuing 6# 2011-9-30 11:33
噢,应该是恒不为0,零点那就连续了
thread-361-1-8.html: [几何] 立体几何,翻折问题,最值问题
scientist 1# 2012-2-10 09:41
在正方形ABCD中,E,F分别为线段AD,BC上的点,角ABE=20度,角CDF=30度,将三角形ABE绕直线BE,三角形CDF绕直线CD各自独立旋转一周,则在所有旋转过程中,直线AB与直线DF所成角的最大值为多少? 请教各位高手,谢谢了!
$\TeX$ 2# 2012-2-10 11:40
没图看得好累。。
$\TeX$ 3# 2012-2-10 11:46

$\TeX$ 4# 2012-2-10 11:48
http://bbs.pep.com.cn/thread-929604-1-1.html
scientist 5# 2012-2-10 12:04
呵呵,看不明白呢,求详解,谢谢!
kuing 6# 2012-2-10 12:44
早前在群里我还画过图形,两个圆锥 群相册里有
kuing 7# 2012-2-10 13:57
群聊记录
scientist 8# 2012-2-11 09:48
7# kuing 请问在哪个群啊?群聊截图的字有点模糊,我琢磨下先。谢谢了!
scientist 9# 2012-2-11 15:59
7# kuing 群聊截图看清楚了,原来这么简单啊,你的图形真厉害!太谢谢了!
thread-362-1-1.html: 转自西西贴吧:数列增长估计
icesheep 1# 2012-2-11 20:44
来源:http://tieba.baidu.com/p/1409012900 ${a_1} = 1$,${a_{n + 1}} = 1 + \frac{n}{{{a_n}}}$ 求证:\[{a_n} = \sqrt n  + \frac{1}{2} - \frac{1}{{8\sqrt n }} + o\left( {\frac{1}{{\sqrt n }}} \right)\]
$\TeX$ 2# 2012-2-11 22:38
不会做,Stolz定理好像也不行
thread-363-1-8.html: 2012自主招生“华约”数学
GAM 1# 2012-2-12 15:39
很凌乱。。。 贴两道大题,求第一道大题的高等解(如果有的话)
thread-365-1-1.html: 极限证明。
Chetion 1# 2012-2-17 10:00
本帖最后由 Chetion 于 2012-2-18 08:58 编辑 两个极限问题。一直想有一个确切的证明。 Q1: 证明: \[\displaystyle\lim_{x\to+\infty}\dfrac{x^n}{a^x}=0  (a>1)\] Q2: 证明: \[\displaystyle\lim_{x\to0^+}x^x=1\] PS:这两个式子是相关联的。不过我希望能从不同的角度来证明它。
kuing 2# 2012-2-17 13:23
呃,怎么都是 $k\to\ldots$ ? 用了行间公式(即用 \ [ 和 \ ] 的那种)就不用加 \displaystyle 了。 说正题: Q1 用$n$次洛必达 Q2 极限好像是1啊
秋风树林 3# 2012-2-17 18:10
第一题n次方只是个幌子。。。 其实和证明x/(a^x)的极限为0没什么区别 而这个极限可以进一步退化为数列极限,一个高斯+夹逼就可以搞定 这个数列从某项开始显然递减有下界。。。。
kuing 4# 2012-2-17 18:12
我比较懒所以喜欢洛…… 第二个也可以略,对数恒等式先再…… 第二题楼上有什么简单方法么
秋风树林 5# 2012-2-17 18:13
我比较懒所以喜欢洛…… 第二题楼上有什么简单方法么 kuing 发表于 2012-2-17 18:12 化为e^(xlnx)再对指数部分洛吧。。。
kuing 6# 2012-2-17 18:14
刚编辑完回贴,看来你引用得比我编辑快嘛
秋风树林 7# 2012-2-17 18:15
刚编辑完回贴,看来你引用得比我编辑快嘛 kuing 发表于 2012-2-17 18:14
kuing 8# 2012-2-17 18:19
换元你看如何,令 x=1/t,最后要弄 $\sqrt[t]t$,这个好像在正整数的时候搞过……
秋风树林 9# 2012-2-17 18:21
换元你看如何,令 x=1/t,最后要弄 $\sqrt[t]t$,这个好像在正整数的时候搞过…… kuing 发表于 2012-2-17 18:19 嗯嗯,确实弄过
kuing 10# 2012-2-17 18:23
不过这里是实数……
秋风树林 11# 2012-2-17 18:25
不过这里是实数…… kuing 发表于 2012-2-17 18:23 可以试试高斯取整和夹逼。。。
kuing 12# 2012-2-17 18:52
11# 秋风树林 唔,还有单调性…… 应该不成问题
thread-366-1-8.html: [几何] 一道立体几何最值问题,解答看不懂,555......
scientist 1# 2012-2-17 16:31
本帖最后由 scientist 于 2012-2-19 16:13 编辑 直线l垂直平面M,垂足为O,正四面体ABCD的棱长为4,C在平面M内,B是直线l上的动点,则当O到AD的距离为最大时,正四面体在平面M上的射影面积为多少? 请各位高手指教,想知道求解过程,谢谢!
kuing 2# 2012-2-17 17:50
设 O 到 AD 的距离为 h,记 BC 的中点为 N,AD 的中点为 Q,则 \[h\leqslant |QN|+|NO|=|QN|+\frac{|BC|}2\] 右边为定值,而取等时即 h 取最大值时,为当 O、N、Q 三点共线且 N 在 OQ 之间时。 然后就要计算此时的射影面积不太想算……
kuing 3# 2012-2-17 18:04
哈哈,群里竟然也问,不会是同一个人吧?或者是同学?
kuing 4# 2012-2-17 18:07
忘记打码了,哎算了,洗个澡先
scientist 5# 2012-2-19 08:48
谢谢kuing !
scientist 6# 2012-2-19 16:13
2# kuing 请教kuing,为什么h<=|QN|+|NO|,能否将此题讲 的详细点,不胜感激!
kuing 7# 2012-2-19 19:06
6# scientist 后面算射影面积我没想到什么好的算法,没目测出来。
scientist 8# 2012-2-20 12:27
平面ABD与投影面的位置关系是什么?投影是三角形吗?我还是不清楚哦。麻烦kuing讲的再详细点,谢谢!无比感谢!
李斌斌755 9# 2012-2-26 05:33
本帖最后由 李斌斌755 于 2013-4-29 02:27 编辑 不懂画图,说个思路。先考虑点$O$何时到$AB$距离最大,我们换个角度看定点$O$与动直线$BC$的空间关系,既定直线$BC$与动点$O$的空间关系,易知点$O$是以$BC$为直径的球面(空间为球面,平面为圆,$\angle BOC=90^\circ$)。可知$O$到$AD$的距离为四面体上以$BC$为直径的球面上的点到$AD$的距离。其最大距离等于$AD$到球心(既$BC$与$AD$的公垂线)+球半径$=\sqrt8+2$.再考虑取得最大距离时四面体的投影情况,此时我们注意到$AD\perp$平面$OBC$,$AD//$平面M,故其投影是以$AD$为底,$O$到$AD$的投影距离($=(\sqrt8+2)\cos45^\circ=2+\sqrt2$)为高的等腰三角形,其面积$=\dfrac{4(\sqrt2+2)}2=4+2\sqrt2$。
kuing 10# 2012-3-3 22:10
点“添加附件”
李斌斌755 11# 2012-3-3 22:41
14# kuing 谢谢
李斌斌755 12# 2012-3-3 22:41
本帖最后由 李斌斌755 于 2012-3-3 22:43 编辑 能直接画图吗
scientist 13# 2012-3-12 12:59
10# 李斌斌755 万分感谢楼上的帮助,有你这个图和详细说明,我就想明白了。学习了,谢谢!
thread-367-1-8.html: [不等式] 不等式问题,解答还是没看懂啊
scientist 1# 2012-2-17 16:38
本帖最后由 scientist 于 2012-2-19 16:07 编辑 若不等式-1<$ax^2+bx+c$<1的解集为(-1,3),则实数a的取值范围是? 请各位指点解题的思路和方法,谢谢!
kuing 2# 2012-2-17 17:24
分类讨论一下吧 如果 $a=0$,则存在 $b=\frac12, c=-\frac12$ 使之符合条件; 如果 $a>0$ 开口向上,由图象可知 $ax^2+bx+c=1$ 的根是 $-1$ 和 $3$,并且 $ax^2+bx+c>-1$ 恒成立,故……; 如果 $a<0$ 开口向下,由图象易见 $ax^2+bx+c=-1$ 的根是 $-1$ 和 $3$,并且 $ax^2+bx+c<1$ 恒成立,故……。
kuing 3# 2012-2-17 17:33
后面自已算算,我就懒得计算了,目测一下估计是\[a\in\left(-\frac12,\frac12\right)\] PS. 一楼公式中 -1<\$ax^2+bx+c\$<1 建议改为 \$-1<ax^2+bx+c<1\$,解集(-1,3)建议改为 \$(-1,3)\$,(注意半角)
scientist 4# 2012-2-19 08:45
谢谢kuing,还有你的建议,我刚开始用,还不顺手哈.
scientist 5# 2012-2-19 16:06
2# kuing 再请教下kuing ,为什么a>0时$ax^2+bx+c=1$的根是-1和3?这么说的意思是不是表示$y=ax^2+bx+c$的对称轴一定在(-1,3),对称轴不会在这个区间外吗?
kuing 6# 2012-2-19 18:57
对称轴必须是x=1 你画个二次函数,用y=-1和y=1去截,解集就是这两直线之间的部分所对应的x
scientist 7# 2012-2-20 12:23
终于想明白了 。移项,让不等式的一边为0,就好了。我算了好一会儿才得出答案,kuing竟然目测一下就得出答案,佩服啊!谢谢了!
thread-368-1-8.html: [函数] 函数问题
scientist 1# 2012-2-17 16:59
已知函数f(x)满足f(x)=2f(1/x),当1<=x<=3,f(x)=lnx,若当1/3<=x<=3时,函数g(x)=f(x)-ax有三个不同零点,则实数a的取值范围是?.答案是ln3/3<=a<1/e. 我的2种做法如下: (1)当1/3<=x<=1时,1<=1/x<=3,所以f(x)=2f(1/x)=2ln(1/x)=  -2lnx. 但是我换个做法: (2)当1<=x<=3时, 1/3<=1/x<=1, 因为f(x)=2f(1/x),所以f(1/x)=(1/2)f(x)=(1/2)lnx, 从而当1/3<=x<=1时,f(x)=(1/2)ln(1/x) 我的问题是:为什么2种做法做出来的答案不一样?是我的哪种做法有问题,还是题目本身有问题,请大家来讲一讲,谢谢!
①②③④⑤⑥⑦ 2# 2012-2-17 17:21
1# scientist 毫无疑问题目有问题,第一个条件必须对x的范围做限制 如果不限制 $$f(x)=2f\left(\frac1x\right)$$ $$f\left(\frac1x\right)=2f(x)$$ 马上得到 $$f(x)=4f(x)$$ ……
kuing 3# 2012-2-17 17:38
这种抽象函数错题实在见得太多了
scientist 4# 2012-2-19 08:46
谢谢两位哈!
thread-369-1-1.html: $\{x\}$函数迭代
$\TeX$ 1# 2012-2-17 21:29
本帖最后由 $\TeX$ 于 2012-2-17 21:31 编辑 设$f(x)=\{10x\}\quad x\in [0,1)$(其中$\{t\}$代表取$t$的小数部分),$f_n(x)=f_{n-1}(x),f_1(x)=f(x)$。 (1)求证:存在$x\in [0,1)$使得$f_n(x)$的闭包为$[0,1]$。 (2)满足(1)条件的$x$的个数怎样?(有限或无限),与其补集(全集为$[0,1)$)相比大小如何? (3)证明:$F(x)=\lim_{n\to \infty}\frac{1}{n}\sum_{i=1}^n \sin {f_i(x)}$几乎处处为零。
海盗船长 2# 2012-3-5 13:08
http://www.artofproblemsolving.c ... p?f=67&t=467656
thread-37-1-1.html: l
kuing 1# 2011-9-30 19:03
$\emph{l}$ $\textrm{l}$ $\textbf{l}$ $\textsf{l}$ $\texttt{l}$ $\textmd{l}$ $\textit{l}$ $\textsc{l}$ $\textsl{l}$ $\mathds{l}$ $\mathbb{l}$ $\mathbf{l}$ $\mathcal{l}$ $\mathfrak{l}$ $\mathrm{l}$ $\ell$
戊概念·五 2# 2011-11-11 11:32
1# kuing 最后一个漂亮~!
kuing 3# 2011-11-11 11:37
2# 戊概念·五 好多字体都用不了 最后那个挺常用的
戊概念·五 4# 2011-11-12 09:56
3# kuing 我喜欢!O(∩_∩)O哈哈~
thread-370-1-1.html: 还是$\{x\}$函数
$\TeX$ 1# 2012-2-17 21:38
(1)求证:存在无理数$\alpha,\beta \in (0,1)$,满足$\lim_{n\to \infty}\inf n\{n\alpha\}\{n\beta\}=0$。 (2)这样的$\alpha,\beta$个数如何(有限或无限),是否所有的无理数$\alpha,\beta \in (0,1)$都能满足(1)的条件?
thread-371-1-1.html: $2^n$的首位
$\TeX$ 1# 2012-2-17 21:41
研究$2^n(n=1,2,3,\cdots)$的十进制形式的首位数的概率分布。
$\TeX$ 2# 2012-2-20 16:00
http://www.artofproblemsolving.c ... ?f=498&t=464969
①②③④⑤⑥⑦ 3# 2012-2-22 09:18
http://oeis.org/A008952
thread-373-1-2.html: 超级玛丽
kuing 1# 2012-2-20 02:04
魂斗罗版:http://v.youku.com/v_show/id_XMTcyMTEyOTQ4.html 其他人物综合:http://v.youku.com/v_show/id_XMTcyMTU1NjE2.html 忍者龙剑传演示版:http://v.youku.com/v_show/id_XMTc2NTMwNDA0.html 搞笑版:http://v.youku.com/v_show/id_XOTE2NTY3MDg=.html
图图 2# 2012-2-20 10:01
发表于 2012-2-20 02:04
kuing 3# 2012-2-20 13:53
2# 图图
kuing 4# 2012-2-20 13:53
继续 bug,技巧等 http://v.youku.com/v_show/id_XMTY1MDgxODg4.html http://v.youku.com/v_show/id_XMTk1NjQzNzgw.html http://v.youku.com/v_show/id_XMjI3NjExMzc2.html
$\TeX$ 5# 2012-2-20 16:03

kuing 6# 2012-8-12 16:29
http://v.youku.com/v_show/id_XNDAwNjc1NDYw.html 这个也不错,中文讲解
kuing 7# 2012-8-12 16:58
顺便发这个 http://v.youku.com/v_show/id_XMzg4MDI5MjI0.html 魂斗罗的,跟上一贴同一个作者制作
thread-374-1-8.html: [不等式] 不等式求最值,用到柯西
scientist 1# 2012-2-20 12:32
本帖最后由 scientist 于 2012-2-20 12:40 编辑 已知$a>0,b>0,c>0,a+b+c=3$,设$S=\dfrac{a^4}{b+c}+\dfrac{b^4}{c+a}+\dfrac{c^4}{a+b}$,求S的最小值. 证明过程中,第一步用到$S=\dfrac{a^4}{b+c}+\dfrac{b^4}{c+a}+\dfrac{c^4}{a+b}\ge\dfrac{(a^2+b^2+c^2)^2}{2(a+b+c)}$,请问这是用了哪个不等式还是...?请指点,谢谢!
kuing 2# 2012-2-20 14:27
柯西不等式的一个常用变式 \[\frac{a_1^2}{b_1}+\frac{a_2^2}{b_2}+\cdots+\frac{a_n^2}{b_n}\geqslant \frac{(a_1+a_2+\cdots+a_n)^2}{b_1+b_2+\cdots+b_n}\] 其中 $b_i>0$ $(i=1,2,\ldots,n)$
scientist 3# 2012-2-20 15:30
2# kuing 明白了!太谢谢了!
thread-375-1-8.html: [不等式] cyc求最小值,原创一个
天涯无际 1# 2012-2-21 15:21
本帖最后由 天涯无际 于 2012-2-21 15:22 编辑 $a,b,c$为非负实数,且没有两个同时为0,求下式的最小值 \[\frac{a}{4b+3c}+\frac{b}{4c+3a}+\frac{c}{4a+3b}+\frac{9(ab+bc+ca)}{2(a+b+c)^2}\] 有bottema的估计30秒就出来了~~ PS:有更一般的形式~~~
随便112 2# 2012-3-4 18:12
a=2b,c=0取等
天涯无际 3# 2012-3-4 18:40
去年国庆节弄的这个不等式,证明很繁琐,记不太清楚了。。。也不完全是原创,由Can的不等式推广而来~~~ http://www.artofproblemsolving.c ... p?f=52&t=433008 http://www.artofproblemsolving.c ... p?f=52&t=434940
thread-377-1-1.html: 悬挂缩进
kuing 1# 2012-2-22 00:38
转自 http://blog.sina.com.cn/s/blog_5e16f1770100gkig.html   方法一   用\hangafter 和 \hangindent来实现。   \hangafter 后面的数字指行数,是从 n+1 行开始有效   \hangindent 后面是尺寸,就是向右移动的距离   这个命令必须放在段的开始,而且有效范围也是当前的段。另起一段后即失效。因此这种方法适用于文章中只有部分段落需要悬挂缩进格式的情况。   举例:    \hangafter 1    \hangindent 1.5em    \noindent    ...
thread-378-1-7.html: [不等式] 一个简单不等式问题
力工 1# 2012-2-22 20:19
已知$a,b,c>=0$ $a^5/(a^3+b^3)+b^5/(b^3+c^3)+c^5/(c^3+a^3)>=(a^2+b^2+c^2)/2$ .
kuing 2# 2012-2-22 22:12
由CS及AG得 \[\sum\frac{a^5}{a^3+b^3}=\sum\frac{a^8}{a^6+a^3b^3}\geqslant\frac{\left(\sum a^4\right)^2}{\sum a^6+\sum a^3b^3}\geqslant\frac{2\left(\sum a^4\right)^2}{2\sum a^6+\sum a^4b^2 + \sum a^2b^4},\] 令 $a^2=x$, $b^2=y$, $c^2=z$,则只要证 \[\frac{2\left(\sum x^2\right)^2}{2\sum x^3+\sum x^2y + \sum xy^2}\geqslant\frac{\sum x}2,\] 上式去分母展开可配方为 \[\sum(x^2-xy+y^2+z^2)(x-y)^2\geqslant0.\]
力工 3# 2012-7-15 15:41
1# 力工 学习下输入. 首先说一下调和级数的一个推广: 设 $z$ 为复数,定义 $H_z=\sum_{k\ge0}(1/k-1/(k+z))$。 则对正整数 $n$,有 $H_n=\sum_{k=1}^n1/k$,因此我们可将上述定义看作是调和级数的推广。 现在,我们将证明 $H_x$ 的渐近估计: $$H_x = \ln x+\gamma+\frac1{2x}+O\left(\frac1{x^2}\right)$$ 其中 $x\to\infty$,$\gamma$ 是 Euler-Mascheroni 常数。 我们利用 Euler-Maclaurin 公式可得 $$H_{n-1}=\sum_{0\lt k\lt n}\frac1k=\ln n+\sum_{k=1}^m\frac{B_k}k(-1)^{k-1}\left(\frac1{n^k}-1\right)-\int_1^n\frac{B_m(\{t\})dt}{t^{m+1}}$$ 因此 $H_{n-1}-\ln n$ 收敛(其实就是 $\gamma$)。 现在我们考虑 $f(t)=1/(k+x)$,由 Euler-Maclaurin 公式, $$\sum_{0\le k\lt n}\frac1{k+x}=\ln(n+x)-\ln x+\sum_{k=1}^m\frac{B_k}k(-1)^{k-1}\left(\frac1{(n+x)^k}-\frac1{x^k}\right)-\int_0^n\frac{B_m(\{t\})dt}{(t+x)^{m+1}}$$ 因此 $$H_x = \lim_{n\to\infty}\left(\sum_{0\lt k\lt n}\frac1k-\sum_{0\lt k\lt n}\frac1{k+x}\right)=\lim_{n\to\infty}\left(H_{n-1}-\ln n+\ln n-\sum_{0\le k\lt n}\frac1{k+x}+\frac1x\right)$$ 其中 $\lim_{n\to\infty}\left(H_{n-1}-\ln n\right) = \gamma$,并且 $\lim_{n\to\infty}\left(\ln n-\ln(n+x)\right)=0$,我们可得 $$H_x=\ln x+\frac1x+\sum_{k=1}^m\frac{(-1)^{k-1}B_k}{kx^k}+\int_0^\infty\frac{B_m(\{t\})dt}{(t+x)^{m+1}}=\ln x+\frac1x+\sum_{k=1}^{m-1}\frac{(-1)^{k-1}B_k}{kx^k}+O\left(\frac1{x^m}\right)$$ 注意到 $\sum_{k=1}^n1/(a+bk)=(H_{n+a/b}-H_{a/b})/b$,因此原式的估计中有 $\ln n$ 也不足为奇了,因为调和级数 $H_x$ 的主部就是 $\ln x$。
thread-379-1-1.html: 位图矢量化程序potrace
kuing 1# 2012-2-23 17:37
主页:http://potrace.sourceforge.net/ 用于将黑白的位图(PBM, PGM, PPM, or BMP format)生成矢量图,可存为eps、pdf等格式 附件中的PDF是这贴(http://kkkkuingggg.5d6d.com/thread-53-1-1.html)中的“大衰”表情通过此程序输出的(原jpg图存档后一些边界细节经过我修补再另存为bmp后再使用该程序) 输出的PDF体积非常小
kuing 2# 2012-2-23 17:57
程序在dos下运行,所以要先“开始-运行-cmd”,再跳转到存放该程序的位置,然后用 potrace X:\YYY\ZZZ.bmp 选项挺多,由于主页是英文,很多还没看懂,不过一般来说,用默认的设置就OK了 例,程序放在 D:\potrace 文件夹下,有一个黑白bmp图 abc.bmp 放在 D:\ 下待处理,则操作为: 开始-运行-cmd D: cd potrace potrace d:\abc.bmp 就OK了,默认是输出eps格式的,如果要输出pdf,则改成 potrace -b pdf d:\abc.bmp 如果要输出的pdf自动清除边界空白,则改成 potrace --tight -b pdf d:\abc.bmp 其它选项中还有可能用上的有 -t $n$           - suppress speckles of up to this size (default 2)  抑制散斑到尺寸$n$(默认是2)? -A $\alpha$  (逆时针旋转$\alpha$角) 有空再慢慢翻译选项并尝试……
kuing 3# 2012-2-23 18:52
注意里面还有一个 mkbitmap 程序(http://potrace.sourceforge.net/mkbitmap.1.html)用于将彩色的位图转化为黑白的位图,用法应该类似,有空再试试
kuing 4# 2012-2-24 01:49
将这贴(http://bbs.pep.com.cn/thread-450519-1-1.html)中的图存档,用画图打开再另存为 ll.bmp 放在D盘,然后用 mkbitmap,命令 mkbitmap -f 4 -s 1 -3 -t 0.4 d:\ll.bmp 生成 ll.pbm,再用 potrace,命令 potrace -b pdf d:\ll.pbm,搞定。 原图: 生成的 PDF: ll.pdf (6.57 KB) 我把PDF的效果截图出来看看:
kuing 5# 2012-2-24 01:52
哈,这样更像是手画的,效果不错我喜欢
最初的梦想 6# 2012-3-9 19:22
将这贴(http://bbs.pep.com.cn/thread-450519-1-1.html)中的图存档,用画图打开再另存为 ll.bmp 放在D盘,然后用 mkbitmap,命令 mkbitmap -f 4 -s 1 -3 -t 0.4 d:\ll.bmp 生成 ll.pbm,再用 potrace,命令 potrac ... kuing 发表于 2012-2-24 01:49 真的很漂亮啊
戊概念·五 7# 2012-8-19 23:07
4# kuing 好有质感
thread-38-1-9.html: [不等式] 简易三角函数不等式
kuing 1# 2011-10-1 13:57
问题来自:http://user.qzone.qq.com/363215694/blog/1317434128 若 $\alpha ,\beta ,\gamma \in\left(0,\dfrac\pi2\right)$,则\[ \frac{\sin\alpha }{1+\sqrt{\sin\beta \sin\gamma }} +\frac{\sin\beta   }{1+\sqrt{\sin\gamma\sin\alpha }} +\frac{\sin\gamma  }{1+\sqrt{\sin\alpha\sin\beta  }}<2. \] solution 由 $\sin\alpha ,\sin\beta ,\sin\gamma \in(0,1)$,可知\[ \frac{\sin\alpha }{1+\sqrt{\sin\beta \sin\gamma }}<\frac{\sin\alpha }{\sin\alpha+\sqrt{\sin\beta \sin\gamma }}=\frac{1}{1+\frac{\sqrt{\sin\beta \sin\gamma }}{\sin\alpha }} \]等三式,分别令 $x,y,z$ 为 $\dfrac{\sqrt{\sin\beta \sin\gamma }}{\sin\alpha }$ 等的轮换三式,即只要证更强式\[ \frac1{1+x}+\frac1{1+y}+\frac1{1+z}<2, \]其中 $x,y,z>0,xyz=1$,此乃显然,皆因上式去分母整理即为\[ xy+yz+zx+1>0.\] remark 右边 2 已是确界,皆因令 $\alpha\to0,\beta=\gamma\to\dfrac\pi2$ 可知原式 $\to2$。
thread-383-1-1.html: 记录个命令,不太记得了
kuing 1# 2012-2-24 22:34

linzhuqin 2# 2012-3-20 13:59
$$in[1]:=reduce[forall][x,equivalent[a x^2+x+b>0,-2<X<3]],{a,b},reals]$$ $$out[1]=a== -1 && b=b$$ 我看看是什么命令
kuing 3# 2012-3-20 14:00
2# linzhuqin ??你在干嘛? 这里是 Mathematica 版块,不是 TeX
thread-384-1-1.html: 进制转化
kuing 1# 2012-2-24 22:57
http://bbs.pep.com.cn/thread-453729-1-1.html 后面那个啥意思……
①②③④⑤⑥⑦ 2# 2012-2-27 16:47
1# kuing IntegerDigits  给出一个整数在指定的进制下,从第一个有效数字起每一个数位上的数依次排列的数列 FromDigits 前者的逆,从一个数列和指定的基数得到十进制数 (另外一种用法可以将罗马数字字符串转为十进制数)
thread-386-1-1.html: JaxEdit已更新到支持一些article和beamer文档类命令
kuing 1# 2012-2-26 14:42
http://jaxedit.googlecode.com/svn/trunk/jaxedit/jaxedit.html 2012年2月26日更新,相对于之前的版本有如下改进: - 调整了外观,根据窗口大小自动调整各区域大小 - 仅重新解析代码的修改部分,适合编写大型的文档 - 增加对一些 article 和 beamer 文档类命令的支持 信息来自 http://bbs.ctex.org/viewthread.php?tid=64185
叶剑飞Victor 2# 2012-8-26 11:26
哇!这网站太棒了!
kuing 3# 2012-8-26 11:33
我也好久没关注了,肯定又有更新了。
thread-388-1-4.html: [不等式] 来自pep的正系数二次方程min、max
kuing 1# 2012-2-27 08:56
原贴地址:http://bbs.pep.com.cn/thread-2363827-1-1.html 原题:设正系数一元二次方程 $ax^2+bx+c=0$ 有实根。证明:       (1)$\max\{a,b,c\}\geqslant \dfrac49(a+b+c)$;   (2)$\min\{a,b,c\}\leqslant \dfrac14(a+b+c)$。 证明 (1)等价于 $a,b,c>0$ 且 $b^2\geqslant 4ac$ 时证 $9\max\{a,b,c\}\geqslant 4(a+b+c)$。 由于 $a, c$ 对称,不妨设 $a\geqslant c$,则 $\max\{a,b,c\}=\max\{a,b\}\geqslant c$,故 \begin{align*} 9\max\{a,b,c\}\geqslant 4(a+b+c) &\iff 9\max\{a,b\}\geqslant 4(\max\{a,b\}+\min\{a,b\}+c)\\ &\iff 5\max\{a,b\}\geqslant 4\min\{a,b\}+4c. \end{align*} 如果 $a\geqslant b$,则 \[5\max\{a,b\}\geqslant 4a+\frac{ab}a\geqslant 4b+\frac{b^2}a\geqslant 4b+\frac{4ac}a=4\min\{a,b\}+4c,\] 此时不等式成立; 如果 $b>a$,当 $b\geqslant4c$ 时不等式显然成立,而当 $b<4c$ 时,我们有 \[5\max\{a,b\}-(4\min\{a,b\}+4c)=5b-\frac{4ac}c-4c\geqslant 5b-\frac{b^2}c-4c=\frac{(b-c)(4c-b)}c>0,\] 故此时不等式也成立。 综上知原不等式得证。 (2)等价于 $a,b,c>0$ 且 $b^2\geqslant 4ac$ 时证 $4\min\{a,b,c\}\leqslant a+b+c$。 由于 $a, c$ 对称,不妨设 $a\geqslant c$,假如 $b<c$,那么 $b<c\leqslant a \implies ac>b^2\geqslant4ac$ 矛盾,所以必定有 $b\geqslant c$,于是 $\min\{a,b,c\}=c$,故 \[4\min\{a,b,c\}\leqslant a+b+c \iff 3c\leqslant a+b,\] 这是显然的, \[3c\leqslant a+\sqrt{4ac}\leqslant a+b,\] 故不等式成立。
力工 2# 2012-4-6 10:31
1# kuing 精彩! 今天终于能正确显示公式了。
kuing 3# 2013-2-1 14:28
“正系数”的条件可以弱化,即有如下命题: 设 $a$, $b$, $c\in\mbb R$ 使得关于 $x$ 的方程 $ax^2+bx+c=0$ 有实根,则有 \begin{align} \max\{a,b,c\}&\geqslant\frac49(a+b+c); \label{abcxfcmax49}\\ \min\{a,b,c\}&\leqslant\frac14(a+b+c). \label{abcxfcmin14} \end{align} 当 $a>0$, $b>0$, $c>0$ 时1楼已证。 当 $a<0$, $b<0$, $c<0$ 时,注意到,若 $a$, $b$, $c$ 使方程 $ax^2+bx+c=0$ 有实根,则 $-a$, $-b$, $-c$ 也能使其有实根,于是由正系数时结论成立可知此时也有 \begin{align*} \max\{-a,-b,-c\}&\geqslant\frac49(-a-b-c),\\ \min\{-a,-b,-c\}&\leqslant\frac14(-a-b-c), \end{align*} 再注意到有恒等式 $\max\{-a,-b,-c\}=-\min\{a,b,c\}$, $\min\{-a,-b,-c\}=-\max\{a,b,c\}$,代入上式即得 \begin{align*} -\min\{a,b,c\}\geqslant\frac49(-a-b-c) &\riff \min\{a,b,c\}\leqslant\frac49(a+b+c)<\frac14(a+b+c),\\ -\max\{a,b,c\}\leqslant\frac14(-a-b-c) &\riff \max\{a,b,c\}\geqslant\frac14(a+b+c)>\frac49(a+b+c), \end{align*} 所以此时式 \eqref{abcxfcmax49}、\eqref{abcxfcmin14} 都成立。 当 $a$, $b$, $c$ 中有非负也有非正时,比如说 $a\geqslant0\geqslant b$ 时,则有 \[\max\{a,b,c\}=\max\{a,c\}\geqslant0\geqslant\min\{b,c\}=\min\{a,b,c\},\] 故 \begin{align*} \max\{a,b,c\}&\geqslant\frac89\max\{a,c\}\geqslant\frac49(a+c)\geqslant\frac49(a+b+c),\\ \min\{a,b,c\}&\leqslant\frac12\min\{b,c\}\leqslant\frac14(b+c)\leqslant\frac14(a+b+c), \end{align*} 所以此时式 \eqref{abcxfcmax49}、\eqref{abcxfcmin14} 也都成立。当 $a\leqslant0\leqslant b$ 或其他情况都是同理的。 综上所述,命题得证。
thread-389-1-8.html: [不等式] 不等式问题
scientist 1# 2012-2-27 14:53
本帖最后由 scientist 于 2012-2-27 15:07 编辑 求证:$$\frac{\ln2}{2}\times\frac{\ln3}{3}\times...\times\frac{\ln n}{n}<\frac{1}{n}$$(n是大于等于2的正整数) 请高手们指教,谢谢!
①②③④⑤⑥⑦ 2# 2012-2-27 16:33
1# scientist $\ln 2<1$ $\ln n<n-1$
①②③④⑤⑥⑦ 3# 2012-2-27 16:33
1# scientist $\ln 2<1$ $\ln n<n-1$
scientist 4# 2012-2-27 17:37
哦,放缩一下,明白了,谢谢!
thread-39-1-1.html: 测试发图片附件
转贴专用 1# 2011-10-2 15:30
上限 512K
转贴专用 2# 2011-10-2 15:41
附件
thread-390-1-1.html: 做题目用命令的例
kuing 1# 2012-2-28 16:37
k 取 0,1,2,3,...,17 t 取 0,1,2,3,...,47 问 8k+3t 可以取多少个不同的数 口袋中有5只球,编号为1,2,3,4,5,从中任取3球,以ζ表示取出的球的最大号码,则Eζ等于
kuing 2# 2012-2-28 16:42
Flatten 是去花括号? Union 是取并集? Length 是数数? Range 是输出一列数? Subsets 是输出所有子集? Map 是某命令作用于一堆东东? Mean 是求算术平均?
①②③④⑤⑥⑦ 3# 2012-2-29 09:18
2# kuing Flatten 可以调整列表的嵌套情况,拼接分块矩阵,甚至砍掉表达式中指定的内层函数名,等等 Union 是取并集,且对元素排序 Range 是指定初值,上界(下界),步长,生成相应的数列(单参数生成指定了上界的自然数列,默认步长是1) Subsets是输出子集,不过还能附加参数限定子集元素数量,或者指定输出第几个符合条件的子集等等 Map[f, {a, b, c, d, e}] 就是 f /@ {a, b, c, d, e},将f作用于后面的参数(是列表的话就逐个作用生成相应列表),Map还能增加参数控制前面的函数作用于嵌套列表的哪一层 话说,你的Mathematica没有帮助文档?
kuing 4# 2012-2-29 13:18
3# ①②③④⑤⑥⑦ 有吧,用 ??XXXX 查看,进去看了几个例子,大概猜测是那种意思所以就那样写之所有后面都带问号就是还不太确定,怕不尽准确。
thread-391-1-1.html: 计算一个积分
天涯无际 1# 2012-2-29 16:23
本帖最后由 天涯无际 于 2012-2-29 16:28 编辑 计算积分$\int_{0}^{1}{\{\frac{1}{x}\}\cdot\{\frac{1}{1-x}\}}dx$, 其中$\{x\}$表示$x$的小数部分.
tian27546 2# 2012-2-29 21:31
本帖最后由 tian27546 于 2012-2-29 21:34 编辑 $2\gamma-1 $ 设F(a,b)=\int_{0}^{1}\{\frac{1}{x}}^a\frac{1}{1-x}}^bdx$ 因为:$ F(1,1)=2\int_{0}^{1}x(\frac{1}{1+x}+\fai'(x)+\frac{1}{(1+x)^2}+\frac{1}{x^2})dx=2\gamma-1 $ 注:有些函数可以参看特殊函数书籍 且$ F(2,2)=4ln(2\pi)-4\gamma-1 $等等。
thread-392-1-8.html: [函数] 导数+不等式
xr5252 1# 2012-3-1 21:41
a》0  f(x)=[x^2+(a-3)x-2a+3]e^x g(x)=2-a-x-4/(x+1) 假设存在x1,x2∈(0,正无穷) 使|f(x1)-g(x2)|<1成立   求a的范围 ___________kuing编辑线____________ 先帮你编辑好看一点 $a>0$ , $f(x)=[x^2+(a-3)x-2a+3]e^x$ , $g(x)=2-a-x-\dfrac{4}{x+1}$ 假设存在 $x_1,x_2 \in (0,+\infty)$ 使 $|f(x_1)-g(x_2)|<1$ 成立,求 $a$ 的范围
kuing 2# 2012-3-1 22:05
$f(x)$ 在 $(0,+\infty)$ 上的值域是 $[e(1-a),+\infty)$, $g(x)$ 在 $(0,+\infty)$ 上的值域是 $(-\infty,-a-1]$, 要满足条件只要 $e(1-a)-(-a-1)<1$ 即可,故 $a>\dfrac{e}{e-1}$
thread-394-1-8.html: 刚学复数遇到的问题
yayaweha 1# 2012-3-3 18:46
复数为什么不不能比较大小,一个一元二次方程的复数根有什么意义,复数在科学中有哪些应用,望高手指点。
①②③④⑤⑥⑦ 2# 2012-3-5 14:16
1. 复数为什么不不能比较大小? 在复数概念产生之前,不等式早就发展起来了,并且在理论以及实际上都有很多应用,这个体系简单的可以看作来自三条基本性质, 1) $a<b,a=b,a>b$ 三者有且只有一者成立 2) 若 $a>b$,则 $a+c>b+c$ 3) 若 $a>b$,$c>0$,则 $ac>bc$, (加法减法乘法以及相等关系都已预先定义,a<b等价于b>a) 在实数范围内上述三条都对,在扩充到复数后,无法保持上述三条同时成立,这个可以证明: 若 $i>0$ ,则 $-1=i^2>0i=0$,$-i=(-1) i > 0$,$0=i+(-i)>0+(-i)=-i>0$ 矛盾。 若 $i<0$ ,则 $0=i+(-i)<0+(-i)=-i$,$-1=(-i)^2>0(-i)=0$,$i=(-1)(-i)>0(-i)=0$,矛盾。 此路不通,我们有两个选择,在复数集上不定义大小关系,或者改动或删去其中的某条基本性质建立一个新的更广泛的序关系。是否要新建一个,看实际需求,事实上没有这方面的迫切需求,所以除非“混论文”,不然没有人去定义这种东西,就算定义了,一般也不会称作“大小关系” 2. 一个一元二次方程的复数根有什么意义 仅就二次方程本身来说,没有太大的意义。但是,对于高次方程来说,有意义,有了复数集,并且把n次根运算扩展到复数,这样一元三次和四次方程才能保证存在“根式解”,一元n次方程能保证有根(代数基本定理),整个多项式理论在处理起来比之没有复数要方便很多。 3. 复数在科学中有哪些应用 在物理学里可能用到,比如力学电学里都有可能,一开始,复数可以用于平面力学系统,不过后来被更为方便的向量替代了。复数和平面向量本来就能一一对应,所以能用平面向量的地方,改用复数也都行得通,就看怎样方便了。现在,复数的作用主要还是体现在数学处理上(不限于多项式、根式,函数理论也需要复数)。
海盗船长 3# 2012-3-6 18:16
复变函数很有用,特别是在物理中
李斌斌755 4# 2012-3-6 23:16
一谈高数,头晕
thread-396-1-8.html: [几何] 一种思路,二面角成60度,过两面外一定点可作几根直线与两面都成30度。
李斌斌755 1# 2012-3-3 23:38
本帖最后由 李斌斌755 于 2012-3-14 19:33 编辑 一种思路,二面角成60度,过两面外一定点可作几根直线与两面都成30度。 如图,二面角分别为a,b.定点为o,过o作平面成30度的园锥oab,反向延长得圆锥ocd,那么圆锥中一无数条母线与a平面成30度,现在只要看圆锥ocd中有几条母线与平面b成30度既可。过o点作平面b’与b平行,由图易知,母线与平面b成角情况为两部分,在面上方角范围为0~30度(30度时为一根,其余为二根);在面下方角范围为0~90度(90度时为一根,其余为二根)。可知共有三根。 补充:是否可以推广到二面角成多少度,过两面外一定点可作几根直线与两面都成多少度。
kuing 2# 2012-3-4 00:17
两对共顶点双圆锥的交线问题 一般结论以前好像总结过,不过一时找不到记录了,估计网上也有。
李斌斌755 3# 2012-3-4 00:58
本帖最后由 李斌斌755 于 2012-3-4 01:05 编辑 如何在电脑上画图?另我在各论坛上见此类题的解答都过于繁琐,不易理解。为啥不用此法?!
kuing 4# 2012-3-4 01:11
下载作图的软件来画图咯,比如几何画板,很多人都在用,使用方法还挺简单,自己摸索下就可以了。
天涯无际 5# 2012-3-4 10:45
记得以前做这题,我是用三余弦定理.....具体的不太记得,年代太久远了~~~
秋风树林 6# 2012-3-5 17:35
本帖最后由 秋风树林 于 2012-3-5 17:37 编辑 两对共顶点双圆锥的交线问题 一般结论以前好像总结过,不过一时找不到记录了,估计网上也有。 kuing 发表于 2012-3-4 00:17 哈哈。。。kk。。。那是我问的问题。。。 后来总结了个一般的。。。
kuing 7# 2012-3-5 17:37
6# 秋风树林 你还画了个图的
kuing 8# 2012-3-5 17:38
刚回复完就发现你编辑上图了
秋风树林 9# 2012-3-5 17:41
刚回复完就发现你编辑上图了 kuing 发表于 2012-3-5 17:38 这叫速度快~
李斌斌755 10# 2012-3-7 02:54
本帖最后由 李斌斌755 于 2012-3-7 02:59 编辑 发此贴的目的只是想与大伙探讨一下圆锥及球在解答某类立几题中的应用。如线面成角·旋转等,可使问题变得简单明了。抛砖引玉,希望与大家共勉! 另为何我无法上图,积分原因吗?还是……
kuing 11# 2012-3-7 12:37
另为何我无法上图,积分原因吗?还是…… 李斌斌755 发表于 2012-3-7 02:54 你一楼已经上过图了,说明你可以上图啊。
李斌斌755 12# 2012-3-7 13:50
你一楼已经上过图了,说明你可以上图啊。 kuing 发表于 2012-3-7 12:37 我想上传几何画板制图,可老提示不支持。应用什么格式?
kuing 13# 2012-3-7 13:56
我想上传几何画板制图,可老提示不支持。应用什么格式? 李斌斌755 发表于 2012-3-7 13:50 几何画板上作好图之后,用 QQ截图(如果没有QQ的话找其他截图软件也可),保存为 jpg 或 png 格式,一般都可以上传
最初的梦想 14# 2012-3-7 15:36
12# 李斌斌755 键盘上的Prt Sc Sys Rq可以直接截图 点过之后进画图,Ctrl+V
李斌斌755 15# 2012-3-7 20:34
谢谢
kuing 16# 2012-3-7 23:26
14# 最初的梦想 嗯,只是按那个键一般是截全屏,放在画图里还要裁剪一下,而且保存的时候我怕楼主没注意格式,印象中默认存为bmp,那个会很大,不一定能传得上来,所以还得选一下格式,故此没提此。
最初的梦想 17# 2012-3-8 15:54
16# kuing 对,还要注意格式。管理员真细心啊!
kuing 18# 2012-3-9 02:35
原来50度和25度那个是2009年高考题 http://bbs.pep.com.cn/thread-511493-1-1.html http://bbs.pep.com.cn/thread-515567-1-1.html
thread-397-1-8.html: [不等式] 柯西证明?
随便112 1# 2012-3-4 15:29
本帖最后由 随便112 于 2012-3-4 15:31 编辑 若  $ a+b+c=3$ 求证:$\dfrac{1}{8+5a^2+5b^2}+\dfrac{1}{8+5b^2+5c^2}+\dfrac{1}{8+5a^2+5c^2}\le \dfrac{1}{6} $
天涯无际 2# 2012-3-4 16:19
这题用Cauchy恐怕也不容易证,注意等号取得条件是$a=b=c=1$或者$a=\dfrac{13}{5},b=c=\dfrac{1}{5}$. ~~~Vasile的好题目~~~
kuing 3# 2012-3-4 17:41
好像有陈计式配方
天涯无际 4# 2012-3-4 18:05
我倒是有一个证明...不过很长~~~
随便112 5# 2012-3-4 18:10
4# 天涯无际 能不能看看你的证明呢?
kuing 6# 2012-3-4 18:29
http://www.artofproblemsolving.c ... ?f=151&t=359320 16楼陈计配方 15楼我那链接里面也有个暴力的……
天涯无际 7# 2012-3-4 18:36
我的倒没有那么暴力~~~都是简单的计算。。。有空我再打出来。。。 还是先用我的软件编辑好再传上来,这里不能预览,编辑起来很费时间~~~
天涯无际 8# 2012-3-6 00:01
本帖最后由 天涯无际 于 2012-3-6 00:05 编辑 改进了原来的证明,显得没有那么长了~~~ 证明:由于原不等式是对称的,可设$a\geq b\geq c$.首先我们来证明 \[\frac{1}{5a^2+5b^2+8}+\frac{1}{5a^2+5c^2+8}\leq \frac{2}{5a^2+5bc+8}\qquad (*)\] 上式等价于 \[\frac{5(b-c)^2(5a^2-5bc+8)}{(5a^2+5b^2+8)(5a^2+5c^2+8)(5a^2+5bc+8)}\geq 0\] 显然成立. 下面,我们来证明 \[\frac{2}{5a^2+5bc+8}+\frac{1}{5b^2+5c^2+8}\leq \frac{8}{20a^2+5(b+c)^2+32}+\frac{2}{5(b+c)^2+16}\qquad (**)\] 而 \[\frac{2}{5(b+c)^2+16}-\frac{1}{5b^2+5c^2+8}=\frac{5(b-c)^2}{[5(b+c)^2+16](5b^2+5c^2+8)}\] \[\frac{8}{20a^2+5(b+c)^2+32}-\frac{2}{5a^2+5bc+8}=-\frac{10(b-c)^2}{[20a^2+5(b+c)^2+32](5a^2+5bc+8)}\] 由此可知,欲证明$(**)$式,只需证明 \[\frac{[20a^2+5(b+c)^2+32](5a^2+5bc+8)}{[5(b+c)^2+16](5b^2+5c^2+8)}\geq 2\] 上式显然成立,因为 \[5a^2+5bc+8\geq 5b^2+5c^2+8\] \[20a^2+5(b+c)^2+32\geq 2\cdot[5(b+c)^2+16]\] 于是,由$(*),(**)$式可知 \begin{align*} &\frac{1}{5a^2+5b^2+8}+\frac{1}{5b^2+5c^2+8}+\frac{1}{5c^2+5a^2+8}\\ \leq &\frac{2}{5a^2+5bc+8}+\frac{1}{5b^2+5c^2+8}\\ \leq &\frac{8}{20a^2+5(b+c)^2+32}+\frac{2}{5(b+c)^2+16}\\ =&\frac{8}{25a^2-30a+77}+\frac{2}{5a^2-30a+61} \end{align*} 于是我们只需证明 \[\frac{8}{25a^2-30a+77}+\frac{2}{5a^2-30a+61}\leq \frac{1}{6}\] 上式等价于 \[\frac{5(a-1)^2(5a-13)^2}{6(25a^2-30a+77)(5a^2-30a+61)}\geq 0\] 显然成立.由此原不等式得证.结合证明过程可知,等号当且仅当$a=b=c=1$或者$a=\dfrac{13}{5},b=c=\dfrac{1}{5}$或者其循环时取得.
kuing 9# 2012-3-6 00:37
有点难想象 得慢慢研究下
天涯无际 10# 2012-3-6 12:43
本帖最后由 天涯无际 于 2012-3-6 12:45 编辑 为什么题目不见了.... 主要思想就是$F(a,b,c)\leq F(a,\dfrac{b+c}{2},\dfrac{b+c}{2})$.但是,有的时候直接证明会很困难,所以不妨尝试减弱这个式子。那么由于$b,c$的位置相等,可以通过$(b-c)^2$项来控制放缩的程度,本题就是在分母中添加$-5(b-c)^2$项. 另外有的时候,$F(a,b,c)\leq F(a,\dfrac{b+c}{2},\dfrac{b+c}{2})$并不成立,同样可以通过此法来减弱放缩程度(例如在分子中添加$k(b-c)^2$)项. 当然,常数$k$的选取则要依题而定了. 表述能力不是很好... ,大概思想就是这样...用这个方法可以证明不少经典的题目~~~~
kuing 11# 2012-3-6 12:48
题目没不见啊 多谢指导
thread-398-1-8.html: [不等式] 不等式
随便112 1# 2012-3-4 15:37
若 $ a,b,c>0$,且 $ a^2+b^2+c^2=3 $,求证: $ 14+13abc\ge 9(ab+bc+ac) $
天涯无际 2# 2012-3-4 16:21
已经说过了,这个题不用p,q,r法很难证明的~~~当然可以用陈计的配方流,反正我是弄不出来~~~
随便112 3# 2012-3-4 18:08
2# 天涯无际 pqr法好像行不通
天涯无际 4# 2012-3-5 10:07
这个用p,q,r法不就是一两步的事情.....怎么会行不通...
随便112 5# 2012-3-5 12:34
是真的,到后面求导无法判断,用三次拆分和四次拆分得到的不等式仍旧无法判断,你不信 你动下笔。。这个不等式比其他(也就是和这个一样,系数不同,在vasil不等式书籍里有类似的,还有越南170.30.24里均有类似的,包括韩京俊里也有类似的,但是都没有这个强)
天涯无际 6# 2012-3-5 14:35
也不知道你p,q,r是否真的掌握了...既然买了韩同学的书(当然了,感谢购买此书),有时间的话好好看看应该很有帮助。对此题而言,即便是最强的系数配置,也很容易证得: 控制$a+b+c,a^2+b^2+c^2$不变,此时也相当于$ab+bc+ca$不变, 那么$abc$的最小值当且仅当$a\leq b=c$或$a=0$时取到. 当$a=0$时,则只需证明$14\geq 9bc$,这由均值不等式显然。 当$a\leq b=c$时,等价于证明当$0\leq a\leq 1$时,有$14+13a\cdot\frac{3-a^2}{2}\geq 9\left(\frac{3-a^2}{2}+2a\sqrt{\frac{3-a^2}{2}}\right)$.我就懒得写了...
随便112 7# 2012-3-6 12:29
为什么可以这样控制呢?求详解,万分感谢,我现在还是高中生。非常想学习更好的不等式,
thread-4-1-1.html: 第二贴开始不用验证码
测试号 1# 2011-9-25 22:03
RT
kuing 2# 2011-9-25 22:05
嗯,注册到发言不用等两天了,不用升级才能发贴了,不用升级才能发附件了……除了第一贴要验证码之外,好像都不用……:lol
thread-40-1-7.html: [不等式] 三元不等式 $\sum\frac{x+y}{xy(4-xy)}\geqslant2$
kuing 1# 2011-10-3 15:05
问题来自 http://zhidao.baidu.com/question/325781708.html 刚无聊上百度看到的。 已知 $x,y,z > 0,x + y + z = 3$,求证\[\frac{x + y}{xy(4 - xy)} + \frac{y + z}{yz(4 - yz)} + \frac{z + x}{zx(4 - zx)} \geqslant 2\] 大家玩玩。 PS:我用切平面成功,嘿,如下: \[ \frac{x + y}{xy(4 - xy)} - \frac{8 - x - y}9 = \frac{\bigl(xy(4 - xy) + 9\bigr)\bigl(\sqrt x - \sqrt y \bigr)^2 + 2\sqrt{xy}\bigl(9 - xy + 2\sqrt{xy}\bigr)\bigl(\sqrt{xy} - 1\bigr)^2}{9xy(4 - xy)}\geqslant0 \] 下略……
转贴专用 2# 2011-10-3 15:06
回贴看看结果
yes94 3# 2011-10-3 15:30
回贴看看结果
yes94 4# 2011-10-3 15:39
还是看不到下半截(看不到分数的分母),刷新,一片空白,再刷新,网页显示:“……来路不正”,我晕!
kuing 5# 2011-10-3 15:41
只能说明您与本论坛“有缘无份”……表示帮不了你
yes94 6# 2011-10-3 15:46

kuing 7# 2011-10-3 15:50
6# yes94 说明问题说一次就够了,发100次截图也一样。
yes94 8# 2011-10-3 15:50

kuing 9# 2011-10-3 16:36
放缩一下再用切线法及均值可能好接受一点。这个例子如果之前知道的话,选入《切线法》一文中的例题也挺不错的 由均值,可知只要证更强式\[ \frac{1}{\sqrt{xy}(4 - xy)} + \frac{1}{\sqrt{yz}(4 - yz)} + \frac{1}{\sqrt{zx}(4 - zx)} \geqslant 1 \]令 $a=\sqrt{xy}, b=\sqrt{yz}, c=\sqrt{zx}$,则由均值知 $0<a\leqslant \dfrac{x+y}2<\dfrac32$ 等以及 $a+b+c\leqslant x+y+z$,即有 $a,b,c\in\left(0,\dfrac32\right)$ 且 $a+b+c\leqslant 3$,而不等式等价于\[ \frac{1}{a(4 - a^2)} + \frac{1}{b(4 - b^2)} + \frac{1}{c(4 - c^2)} \geqslant 1 \]由 $a,b,c$ 的范围,利用切线法,容易得到\[ \frac{1}{a(4 - a^2)}\geqslant\frac{4-a}9\iff\frac{(a-1)^2(9+2a-a^2)}{9a(4-a^2)}\geqslant0 \]成立,故\begin{align*} \frac{1}{a(4 - a^2)} + \frac{1}{b(4 - b^2)} + \frac{1}{c(4 - c^2)} &\geqslant \frac{4-a}9+\frac{4-b}9+\frac{4-c}9\\ &=\frac43-\frac{a+b+c}9\\ &\geqslant \frac43-\frac13\\ &=1 \end{align*}从而原不等式得证。
海盗船长 10# 2011-10-3 16:45
看结果
海盗船长 11# 2011-10-3 16:46
你怎么不回答百度上的提问呢?
kuing 12# 2011-10-3 16:48
11# 海盗船长 没有足够的被采纳的把握,就不发了。要保持采纳率
kuing 13# 2011-10-3 23:20
续楼上
图图 14# 2011-10-3 23:27
.
pxchg1200 15# 2011-10-3 23:30
我也来玩玩。 by AM-GM \[ \sum{\frac{x+y}{xy(4-xy)}}\geq \sum{\frac{2}{\sqrt{xy}(4-xy)}}\geq 2 \] \[ \sum\frac{1}{\sqrt{ab}(4-ab)}\ge 1 \] by Cauchy-Schwarz \[ \sum\frac{1}{\sqrt{ab}(4-ab)}\ge\frac{9}{4\sum\sqrt{ab}-\sum (ab)^{\frac{3}{2}}} \] 所以只要证: \[ 9+\sum (ab)^{\frac{3}{2}}\ge 4\sum\sqrt{ab} \] AM-GM again: \[ \sum\left( (ab)^{\frac{3}{2}}+1+1\right)\ge 3\sum\sqrt{ab} \] 然后用个显然的结果 \[ 3(xy+xz+yz)\le (x+y+z)^{2}\le 3(x^{2}+y^{2}+z^{2}) \] \[ \sum\sqrt{ab}\le\sqrt{3\sum ab}\le\sum a=3 \] Done!
kuing 16# 2011-10-3 23:33
15# pxchg1200 嗯,也是证9楼那个加强式,后面的柯西均值用得不错的,不过怎么x,y写着写着后面变成了a,b又变回去。。。 这个不等式还挺松。
pxchg1200 17# 2011-10-4 12:21
事实上这个是土耳其2007的竞赛题,原题如下: 设$a,b,c \geq 0$,$a+b+c=3$,证明: \[ \sum_{cyc}{\frac{a^{2}+3b^{2}}{ab^{2}(4-ab)}}\geq 4 \]
kuing 18# 2011-10-4 12:36
17# pxchg1200 :o 出处帝来鸟:D
kuing 19# 2011-10-4 12:38
事实上这个是土耳其2007的竞赛题,原题如下: 设$a,b,c \geq 0$,$a+b+c=3$,证明: \[ \sum_{cyc}{\frac{a^{2}+3b^{2}}{ab^{2}(4-ab)}}\geq 4 \] pxchg1200 发表于 2011-10-4 12:21 是不是分子用均值之后就跟前面的一样了
pxchg1200 20# 2011-10-4 12:39
是不是分子用均值之后就跟前面的一样了 kuing 发表于 2011-10-4 12:38 是啊,一模一样
thread-40-2-7.html:
wenshengli 21# 2011-10-4 15:57
学习!
hhhzh7241hzh 22# 2011-10-4 19:09
1# kuing 看看
isea 23# 2011-10-4 20:06
我来学习中、
linzhuqin 24# 2012-3-22 13:58
哈哈,我看看,马上就转了
kuing 25# 2012-3-22 14:22
哈哈,我看看,马上就转了 linzhuqin 发表于 2012-3-22 13:58 转贴时是不是应该在贴里说明一下转自哪?
力工 26# 2012-3-22 16:04
怎么不能显示了?看到的都是符号括号
kuing 27# 2012-3-22 17:04
26# 力工 呃?我这里看很正常……
力工 28# 2012-3-23 10:21
27# kuing 可是,我在其他站又是正常的。 ,怎么有这毛病?
thread-401-1-8.html: [不等式] 一个不等式链
随便112 1# 2012-3-5 12:41
本帖最后由 随便112 于 2012-3-5 12:48 编辑 设$ x,y,z>0 $,且满足 $ x^2+y^2+z^2=1$ 证明: $\dfrac{1+y^2z^2}{(y+z)^2}+\dfrac{1+z^2x^2}{(x+z)^2}+\dfrac{1+x^2y^2}{(x+y)^2} \ge\ frac{5}{2} \ge\dfrac{(yz+xz+xy)^2+y^2z^2}{(y+z)^2}+\dfrac{(yz+xz+xy)^2+z^2x^2}{(z+x)^2}+\dfrac{(yz+xz+xz)^2+X^2Y^2}{(x+y)^2} $ 左边会解答,右边不会了。
天涯无际 2# 2012-3-5 14:44
左边会证,右边不会证 ......这两边难度不是一个级别的,不说啥了~~
随便112 3# 2012-3-6 12:30
求各位大神解答。
kuing 4# 2012-3-6 12:37
  \ 和 frac 不要隔开。 \frac52   左边我也见过不少次,陈计书上也有个配方……右边没见过
pxchg1200 5# 2012-3-12 22:38
4# kuing 左边SOS就可以了。 貌似陈计书上那个SOS配错了。。。   右边还真没见过。
kuing 6# 2012-3-13 01:36
5# pxchg1200 噢?我没验算过呢
thread-402-1-1.html: 为什么现在看帖还要登录?
海盗船长 1# 2012-3-5 13:08
好麻烦啊,,,
kuing 2# 2012-3-5 13:10
紧急通知的原因你懂的,不过别在这儿说
海盗船长 3# 2012-3-5 13:16

kuing 4# 2012-3-8 16:12
http://www.5d6d.com/thread-914058-1-1.html
kuing 5# 2012-3-21 00:04
貌似可以看了现在
海盗船长 6# 2012-3-21 17:43
嗯,好了
thread-403-1-8.html: [不等式] 一个不等式
力工 1# 2012-3-6 14:56
实数$a,b,c>0$, 证明$a^5+b^5+c^5+abc(ab+bc+ca)\ge a^3b^2+a^3c^2+b^3c^2+b^3a^2+c^3a^2+c^3b^2$.
pxchg1200 2# 2012-3-12 22:32
1# 力工 proof: \[ \Leftrightarrow a(a^2-b^2)(a^2-c^2)+b(b^2-a^2)(b^2-c^2)+c(c^2-a^2)(c^2-b^2)\geq 0 \] Which is perfectly true due to Schur inequality.
thread-404-1-8.html: [数论] 请教一个中学数学题,先谢谢了!
hongxian 1# 2012-3-6 17:39

海盗船长 2# 2012-3-6 17:44
1# hongxian $k^2-2008=m^2\;(m\in \mathbb{N})\Longrightarrow (k+m)(k-m)=2008=2^3\cdot 251$
hongxian 3# 2012-3-6 17:49
欲为整数,则应有k^2-2008=n^2 k^2-n^2=2008 (k+n)(k-n)= 2008=8*251 得到:k+n=8*251,k-n=1;① k+n=4*251,k-n=2;② k+n=2*251,k-n=4;③ k+n=251,k-n=8.④ k+n=-8*251,k-n=-1;⑤ k+n=-4*251,k-n=-2;⑥ k+n=-2*251,k-n=-4;⑦ k+n=-251,k-n=-8.⑧ 其中①④⑤⑧无整数解 最小的整数k由⑥得:k=-503 最小正整数k由③得:k=253
海盗船长 4# 2012-3-6 18:13
3# hongxian 显然是正整数啊
hongxian 5# 2012-3-7 11:24
最后结果是-503
thread-405-1-1.html: 单位圆内随机取两点距离小于1/2的概率模拟
kuing 1# 2012-3-8 15:23
问题是前几天在某群里看到的,后来我分段积分计算出结果是 \[\frac{-9 \sqrt{15}+16 \pi +48 \arccos(7/8)}{64 \pi }\approx 0.197282185 \] 昨晚突然想到用 Mathematica 模拟一下,想到了如下 i = 0; j = 0; n = 1000000; Do[x1 = RandomReal[{-1, 1}];   y1 = RandomReal[{-1, 1}];   x2 = RandomReal[{-1, 1}];   y2 = RandomReal[{-1, 1}];   If[x1^2 + y1^2 < 1 && x2^2 + y2^2 < 1, {     j++;     If[(x1 - x2)^2 + (y1 - y2)^2 < 1/4, i++];     }], {n}]; N[i/j, 10] 其中,n 的值越大即投点越多,越精确,当然程序运行也越久,这里我用了 1000000,运行了几次,得到几个结果 0.1974656502 0.1969572882 0.1967015821 0.1973092282 0.1969999043 0.1978208284 0.1971865261 可以看到,跟我算出来的值还是很接近的。 由于不知如何给出单位圆内的随机点,这里我只好用 $[-1,1]^2$ 内的随机点,再进行筛选(将在圆内的筛选出来,在这些里面判断距离再筛选),而这样其实就浪费了很多对点,所以上面这个程序不算好,不知各位有没有更好更优质的程序?
linzhuqin 2# 2012-3-20 13:37
呵呵,原来是你啊,又回到你这儿来了,呵呵。
linzhuqin 3# 2012-3-20 13:39
你用得很熟练了! 有很多字符,编码不太容易找相应的代码,
chinawgp 4# 2012-6-2 17:52
1# kuing 这里有一篇圆内均匀采点的文章
chinawgp 5# 2012-6-2 17:53
1# kuing 这里有一篇圆内均匀采点的文章 http://blog.csdn.net/codeboycjy/article/details/6225886
kuing 6# 2012-6-2 18:01
5# chinawgp 先thanks了,有空仔细看下。
kuing 7# 2012-6-3 18:00
5# chinawgp 前面是弄懂了,原来只要加个根号就行了,我怎么没想到呢 后面的就不太懂了,那啥Jacobian行列式什么的……
都市侠影 8# 2012-7-21 11:22
本帖最后由 都市侠影 于 2012-7-21 11:23 编辑 关于单位圆内的随机点,你可以产生极坐标的嘛,随机产生半径和角度就可以了,然后 x=rcosa,y=rsina
kuing 9# 2012-7-21 12:30
关于单位圆内的随机点,你可以产生极坐标的嘛,随机产生半径和角度就可以了,然后 x=rcosa,y=rsina 都市侠影 发表于 2012-7-21 11:22 关键在于产生出来的是否均匀分布,可以看看5#的链接
都市侠影 10# 2012-7-21 20:24
9# kuing 那篇文章有道理
都市侠影 11# 2012-7-23 12:29
7# kuing 哈哈,我找到为什么加根号的原因了,看这里我写的,实际上是反函数http://kkkkuingggg.5d6d.net/thread-622-1-1.html
thread-406-1-8.html: 求多项式 for tutu(修正版)
kuing 1# 2012-3-10 18:09
求多项式 $f(x)$,使得 $x^2+1\mid f(x)$,且 $x^3+x^2+1\mid f(x)+1$。 设 \begin{align*} f(x)&=p(x)(x^2+1), \\ f(x)+1&=q(x)(x^3+x^2+1), \end{align*} 其中 $p(x)$,$q(x)$ 为多项式。 显然 $p(x)$、$q(x)$ 均不为 $0$,故 $\deg p(x)=\deg q(x) + 1$ ,于是又可设 \[p(x)=axq(x)+r(x),\] 其中 $a\ne0$,且 $\deg r(x)\leqslant\deg q(x)$。 消去 $f(x)$ 得 \begin{align*}   & q(x)(x^3+x^2+1)-1=p(x)(x^2+1) \\ \iff{} & q(x)(x^3+x^2+1)-1=\bigl(axq(x)+r(x)\bigr)(x^2+1) \\ \iff{} & q(x)\bigl(x^3+x^2+1-ax(x^2+1)\bigr)=1+r(x)(x^2+1) \\ \iff{} & q(x)\bigl((1-a)x^3+x^2-ax+1\bigr)=x^2r(x)+r(x)+1, \end{align*} 由于 $\deg x^2r(x) \leqslant \deg x^2q(x) < \deg x^3q(x)$,故显然有 $a=1$,于是 \[(x^2-x+1)q(x)=x^2r(x)+r(x)+1,\] 故 $\deg r(x)=\deg q(x)$,且必有 $r(x)=q(x)+t(x)$,其中 $\deg t(x)<\deg q(x)$,代入得 \[-xq(x)=x^2t(x)+t(x)+1,\] 显然必须 $t(x)=s(x)x-1$,其中 $s(x)$ 是一多项式,代入得 \[-xq(x)=x^3s(x)-x^2+s(x)x\iff q(x)=x-(x^2+1)s(x),\] 故 \begin{align*} p(x)&=xq(x)+r(x)\\ &=(x+1)q(x)+t(x)\\ &=(x+1)\bigl(x-(x^2+1)s(x)\bigr)+s(x)x-1\\ &=x^2+x-1+\bigl(x-(x+1)(x^2+1)\bigr)s(x)\\ &=x^2+x-1-(x^3+x^2+1)s(x), \end{align*} 代入验证有 \begin{align*} f(x)&=\bigl(x^2+x-1-(x^3+x^2+1)s(x)\bigr)(x^2+1),\\ f(x)+1&=\bigl(x-(x^2+1)s(x)\bigr)(x^3+x^2+1), \end{align*} 成立,故所求的所有 $f(x)$ 为 \[f(x)=\bigl(x^2+x-1-(x^3+x^2+1)s(x)\bigr)(x^2+1),\] 其中 $s(x)$ 为任意多项式。
thread-407-1-8.html: [不等式] 群里面看的的。。
pxchg1200 1# 2012-3-12 22:59
Let $ a,b,c>0 $ with $ a+b+c=3 $ prove that: \[ (a^3+1)(b^3+1)(c^3+1)\geq 8 \] 有没有不是MV的证明。
kuing 2# 2012-3-13 01:34
我也是在群里看到的,目前看到最简是下面严文兰这个证法了: 不妨设 $(b-1)(c-1)\geqslant 0$,由holder不等式 $(a^3+1)(1+b^3)(1+c^3)\geqslant (a+bc)^3$,只需证 $a+bc\geqslant 2\iff a+bc\geqslant a+b+c-1 \iff (b-1)(c-1)\geqslant 0$。
pxchg1200 3# 2012-3-13 11:55
2# kuing 这个犀利哦。膜拜大神。。。
thread-408-1-1.html: 英文加四线格
kuing 1# 2012-3-15 14:54
转自 http://bbs.ctex.org/viewthread.php?tid=62984 还没看太懂,有待研究……
thread-409-1-7.html: [几何] 翻旧贴看到的一道解析几何求轨迹
kuing 1# 2012-3-18 20:52
原贴地址:http://bbs.pep.com.cn/thread-552357-1-1.html 题目:已知椭圆$x^2/4+y^2=1$,$N_1(-1,0)$,$N_2(1,0)$,$M(3,0)$,过$M$的直线交椭圆于$P$、$Q$两点,连$N_1P$,$N_2Q$,得交点$R$,求交点$R$的轨迹。
kuing 2# 2012-3-18 21:13
设$R(x,y)$,$QP:PM=t$,则由梅捏劳斯定理,有 \[1=\frac{N_{2}R}{RQ}\cdot \frac{QP}{PM}\cdot \frac{MN_{1}}{N_{1}N_{2}}=\frac{N_{2}R}{RQ}\cdot t\cdot 2,\]于是 \[\frac{QN_{2}}{RN_{2}}=1+\frac{RQ}{N_{2}R}=2t+1,\]从而易得 \[Q\bigl( (2t+1)(x-1)+1,(2t+1)y \bigr),\]再由$QP:PM=t$也易得 \[P\left( \frac{2t+1}{t+1}(x+1)-1,\frac{2t+1}{t+1}y \right),\]而$P$,$Q$在椭圆上,故将两点代入椭圆方程并去分母得 \[\left\{ \begin{aligned} \left( (2t+1)(x+1)-(t+1) \right)^{2}+4\left( (2t+1)y \right)^{2}&=4(t+1)^{2},\\ \left( (2t+1)(x-1)+1 \right)^{2}+4\left( (2t+1)y \right)^{2}&=4, \end{aligned} \right.\]两式相减并因式分解得 \[3t\bigl( (2t+1)\cdot 2x-t \bigr)=4t(t+2),\]由于$t>0$,故约去$t$,再等价整理得 \[12x(2t+1)=7(2t+1)+9\implies 2t+1=\frac{9}{12x-7},\]代回去,即得 \[\left( \frac{9}{12x-7}(x-1)+1 \right)^{2}+4\left( \frac{9}{12x-7}y \right)^{2}=4,\]去分母化简,即得 \[45x^{2}-108y^{2}=20,\]此双曲线方程就是点$R$所在的曲线方程,但结合图形显然应只能取右支且在椭圆内部且应取掉顶点的部分(因$P$,$Q$为椭圆左右顶点时直线$N_1P$,$N_2Q$重合,无法确定交点$R$,故应舍去顶点),故易得最终的轨迹为\[45x^{2}-108y^{2}=20,x\in\left(\frac23,\frac43\right).\]
kuing 3# 2012-3-18 21:21
好久没试过用代数搞解几了,虽然前面还是用了一个纯平几定理,不过总的来说还是代数运算的多一些…… 不知大家有没有简单点的算法?
海盗船长 4# 2012-3-21 17:41
这是什么软件可以算轨迹啊?
kuing 5# 2012-3-21 21:03
4# 海盗船长 我也不知道啊,图是我截取原贴里的图
XDZM 6# 2012-6-11 21:35
用的软件应该是GeoGebra!
thread-41-1-9.html: [数列] 拜读了网刊1,数列微型整理,发现
isea 1# 2011-10-3 20:55
$a_{n+1}=2a_n+3*2^n$ 这类递推式似乎无法用待定系数法^#^
wenshengli 2# 2011-10-3 21:12
本帖最后由 wenshengli 于 2011-10-3 21:13 编辑 网刊1我也看过,不过这个一眼就观察出两边同除以 $2^n$,就成等差数列,然后就解决了。当然等kuing回复。
kuing 3# 2011-10-3 21:16
印象中有的的确待定后发现方程无解 所以还是两边除以 $p^n$ 通用一些
kuing 4# 2011-10-3 21:33
$a_{n+1}=pa_n+k\cdot p^n$ 其中 $k,p$ 为非零常数。 设 $a_{n+1}-x\cdot p^{n+1}=p(a_n-x\cdot p^n)$,展开对比要 $x\cdot p^{n+1}-px\cdot p^n=k\cdot p^n$,左边恒为0,无解。
isea 5# 2011-10-3 22:51
哈哈,是啊,难怪好多题是跟多少的n次幂。 不过,待定系数法好处是能处理大部分数列通项问题。
kuing 6# 2011-10-3 23:05
嗯,待定出来一般比较容易求通项了,所以看着玩咯。 我网刊中那里也只是说对于某些特殊的 $f(n)$ 可以待定。
①②③④⑤⑥⑦ 7# 2011-10-8 10:23
随便聊聊,不是高中的,一阶的非齐次,用齐次通解加上一个非齐次特解 $a_{n+1}=2a_n$ 通解 $C\cdot 2^n$, $a_{n+1}=2a_n+3\times 2^n$ 一个特解 $3\times 2^{n-1}n$ 所以,$a_{n+1}=2a_n+3\times 2^n$ 的通解就是 $C\cdot 2^n+3\times 2^{n-1}n$,令 $n=1$ 有 $a_1=2C+3$ 求得 $C$。 发展到后来也就是这么个样子,高阶的也是这样,不管待定系数凑等比还是高级点用Z变换等,一次齐次的求法就是这样了,带上关于n的基本初等函数(幂、指、对)的非齐次项,特解形式也都有结论(其实是熟知的例子),一般的一次差分方程就是这样搞定的
thread-410-1-1.html: nonumber的反
kuing 1# 2012-3-18 21:40
http://bbs.ctex.org/viewthread.php?tid=70760 \newcommand\addtag{\refstepcounter{equation}\tag{\theequation}} 不知\refstepcounter{equation}啥意思
thread-411-1-1.html: [xelatex]查看系统中文字体列表
kuing 1# 2012-3-19 18:39
新建一个 bat,里面写上 fc-list :lang=zh-cn >zt.txt notepad zt.txt 然后运行就可以将系统的中文字体信息列表放到zt.txt里并自动打开 不过我这里打开后那些换行都变成一个黑矩形,不知为何,但复制粘贴到别的地方倒是正常了 这些中文字体信息可以用于 xeCJK 的中文字体配置。 比如,我这里生成的列表是 YouYuan,幼圆:style=Regular KaiTi_GB2312,楷体_GB2312:style=Regular NSimSun,新宋体:style=Regular FangSong_GB2312,仿宋_GB2312:style=Regular SimSun,宋体:style=Regular STXinwei,华文新魏:style=Regular SimHei,黑体:style=Regular LiSu,隶书:style=Regular (少了点……当初装的那系统也太精简了吧……那个幼圆和隶书还是后来才装的) 然后在 xeCJK 里就可以用 \setCJKmainfont[BoldFont=SimHei,ItalicFont=KaiTi_GB2312]{SimSun} %普通中文用宋体、粗体用黑体、斜体用楷书 \setCJKsansfont{YouYuan} %无衬线字体用幼圆 \setCJKmonofont{FangSong_GB2312} %打印机字体用仿宋 来设置中文字体。另外,也可以用 \setCJKfamilyfont{hwxw}{STXinwei} \newcommand\xw{\CJKfamily{hwxw}} \setCJKfamilyfont{lisu}{LiSu} \newcommand\ls{\CJKfamily{lisu}} 来定义华文新魏字体和隶书字体的命令,正文中用 {\xw 华文新魏} and {\ls 隶书}
kuing 2# 2012-3-19 19:20
又新装了几个字体 现在的列表是 STCaiyun,华文彩云:style=Regular YouYuan,幼圆:style=Regular STHupo,华文琥珀:style=Regular KaiTi_GB2312,楷体_GB2312:style=Regular NSimSun,新宋体:style=Regular FangSong_GB2312,仿宋_GB2312:style=Regular SimSun,宋体:style=Regular STXinwei,华文新魏:style=Regular SimHei,黑体:style=Regular STXingkai,华文行楷:style=Regular LiSu,隶书:style=Regular \setCJKfamilyfont{hwxw}{STXinwei} \newcommand\xw{\CJKfamily{hwxw}} \setCJKfamilyfont{lisu}{LiSu} \newcommand\ls{\CJKfamily{lisu}} \setCJKfamilyfont{hwxk}{STXingkai} \newcommand\xk{\CJKfamily{hwxk}} \setCJKfamilyfont{hwhp}{STHupo} \newcommand\hp{\CJKfamily{hwhp}} \setCJKfamilyfont{hwcy}{STCaiyun} \newcommand\cy{\CJKfamily{hwcy}}
thread-412-1-1.html: winedt的正则表达式
kuing 1# 2012-3-20 21:46
看了一下帮助,似乎跟一般的那些差别比较大,对着英文只能看例子了……
linzhuqin 2# 2012-3-21 17:21
本帖最后由 linzhuqin 于 2012-3-21 17:37 编辑 $\dddtstile{abc}{def}$ $=\vcentcolon=$
kuing 3# 2012-3-21 17:36
2# linzhuqin 试验代码还是去置顶贴后面跟贴试吧
linzhuqin 4# 2012-3-21 17:37
3# kuing 好的,不过论坛设置成了编辑完就跳首页了。
kuing 5# 2012-3-21 21:02
噢,的确是的,那个我也不知道能不能改,是不太方便。 那这样吧,你自己在“公式测试中”里面,自己发一个主题专门自己测试公式的,这样你在里面随便试。
kuing 6# 2012-3-21 21:16
找了一下关于编辑后跳转的问题,这里:http://www.5d6d.com/thread-641990-1-1.html 说没办法通过基本设置实现
thread-413-1-2.html: QQ空间呃夸张了点
kuing 1# 2012-3-21 01:57
临闪前打开QQ空间看一下,结果发现右上角提示:
最初的梦想 2# 2012-3-29 17:34
嘿嘿
kuing 3# 2012-4-22 01:01
开机速度也有点夸
thread-414-1-1.html: 论坛的友情链接又添加了一个,我的新Blog,:D
kuing 1# 2012-3-21 21:21
http://kuing.is-programmer.com/ 前两天才弄的,也支持 MathJax
6yan2 2# 2012-4-6 16:09
贵站怎样设置的支持数字公式?
kuing 3# 2012-4-6 17:56
2# 6yan2 你查看源文件,将与 mathjax 有关的代码 copy 放到 head 里就行了
kuing 4# 2012-10-7 00:45
又添加了个友情链接——在线 $\LaTeX$ 预览
kuing 5# 2012-10-10 23:28
4# kuing 哎,刚加完,结果就发现上不了了,莫非被qiang了?
kuing 6# 2012-10-11 21:28
4# kuing 哎,刚加完,结果就发现上不了了,莫非被qiang了? kuing 发表于 2012-10-10 23:28 还真是:
kuing 7# 2012-10-13 00:46
时而行时而不行……
pxchg1200 8# 2012-12-18 13:06
7# kuing 你那个Gif图片怎么弄的?
kuing 9# 2012-12-18 14:01
8# pxchg1200 用 gggif
thread-415-1-1.html: 公式测试
kuing 1# 2012-3-22 02:27
原本分段函数所用的代码 f(x)=\cases{{x+1}&{x>0}\\{1-x}&{x<0}} 现在效果 $f(x)=\cases{{x+1}&{x>0}\\{1-x}&{x<0}}$ \[f(x)=\cases{{x+1}&{x>0}\\{1-x}&{x<0}}\] 只好在右边那里用 \ (  \ ) 弄起来才正常………… $f(x)=\cases{x+1&\(x>0\)\\1-x&\(x<0\)}$ 好奇怪,2.0之后的\cases右边的东东变样了……你们看着什么效果?
kuing 2# 2012-3-22 02:32
用回环境模式看看 $f(x)=\begin{cases} x+1 & x>0 \\ 1-x & x<0 \end{cases}$ $f(x)=\begin{cases} x+1+2 & x>0 \\ 1-x & x<123 \end{cases}$ 再试试 aligned $f(x)=\left\{\begin{aligned} & x+1 && x>0 \\ & 1-x && x<0 \end{aligned}\right.$ 距离有点大 换 array $f(x)=\left\{\begin{array}{ll} x+1 & x>0 \\ 1-x & x<0 \end{array}\right.$ 还可以,不过这样就比以前麻烦点了 各种代码自己右键看吧
转化与化归 3# 2012-7-23 20:26
公式测试 $\lvert -c\rvert = \sqrt{a^2+b^2}$
转化与化归 4# 2012-7-23 20:27
$|-c|=\sqrt{a^2+b^2}$
kuing 5# 2012-7-24 22:37
(-a+b)    $(-a+b)$ |-a+b|    $|-a+b|$ \lvert-a+b\rvert    $\lvert-a+b\rvert$
v6mm131 6# 2012-8-25 20:55
5# kuing $\ln x\ln(1-x)<\sqrt{x(1-x)}$
thread-416-1-1.html: 关于长等号
kuing 1# 2012-3-22 12:38
网友 linzhuqin 在本论坛上提到: 化学方程式中的长等号怎么表示呢? 怎么让等号的上下输入字符串呢? $\Longleftarrow$ linzhuqin 发表于 2012-3-22 08:44 要长等号而且要根据等号上下字符串自动变长的话,这在目前的MathJax公式系统里似乎没办法做到,在真LaTeX里也需要用宏包或自定义命令来实现 而目前的MathJax公式系统里有amsmath的东西,amsmath里面有xrightarrow和xleftarrow,所以长的箭头倒是可以的,试试: $A\xrightarrow[abcedf]{asdfasdfasdf}B$ 代码:A\xrightarrow[abcedf]{asdfasdfasdf}B $A\xleftarrow[abcedf]{asdfasdfasdf}B$ 代码:A\xleftarrow[abcedf]{asdfasdfasdf}B 除这两个了以外暂时找不到其它可以在这里直接用的了,连双向箭头都没有,如果有发现新的话到时再发贴说。
kuing 2# 2012-3-22 12:49
自动变长实现不了,但手工调整应该是可以的,下面试试手工组合,纯粹是一个尝试 效果: $A\mathrel{\displaystyle\mathop{=\!=\!=\!=\!=\!=\!=\!=}^{asdfasdfasdf}_{asdf}}B$ 代码:A\mathrel{\displaystyle\mathop{=\!=\!=\!=\!=\!=\!=\!=}^{asdfasdfasdf}_{asdf}}B 其中,中间那些 =\! 的数量决定等号的长度,所以不是自动的,因此在打的时候估计一下需要多长来打多少个了……很笨的方法 $A\mathrel{\displaystyle\mathop{=\!=\!=\!=\!=\!=\!=\!=\!=\!=\!=\!=\!=\!=\!=}^{asdfasdfasdfasdfasdfasdf}_{asdf}}B$ 代码:A\mathrel{\displaystyle\mathop{=\!=\!=\!=\!=\!=\!=\!=\!=\!=\!=\!=\!=\!=\!=}^{asdfasdfasdfasdfasdfasdf}_{asdf}}B $A\mathrel{\displaystyle\mathop{=\!=\!=\!=}^{asdfas}_{asdf}}B$ 代码:A\mathrel{\displaystyle\mathop{=\!=\!=\!=}^{asdfas}_{asdf}}B
linzhuqin 3# 2012-3-22 13:07
2# kuing 谢谢咯
kuing 4# 2012-3-22 13:10
用此方法,箭头类的都可以如此手工拼合 $A\mathrel{\displaystyle\mathop{=\!=\!=\!=\!=\!=\!=\!=\!=\!=\!=\!=\!=\!=\!\Rightarrow}^{asdfasdfasdfasdfasdfasdf}_{asdf}}B$ $A\mathrel{\displaystyle\mathop{\Leftarrow\!=\!=\!=\!=\!=\!=\!=\!=\!=\!=\!=\!=\!=\!=}^{asdfasdfasdfasdfasdfasdf}_{asdf}}B$ $A\mathrel{\displaystyle\mathop{\Leftarrow\!=\!=\!=\!=\!=\!=\!=\!=\!=\!=\!=\!=\!=\!\Rightarrow}^{asdfasdfasdfasdfasdfasdf}_{asdf}}B$
kuing 5# 2012-3-22 13:12
在真LaTeX里面的方法见http://blog.sina.com.cn/s/blog_5e16f1770100l6cv.html,需要宏包\usepackage{extarrows}。 什么时候MathJax允许加宏包就好了
kuing 6# 2012-8-28 17:02
$A\xlongequal[dwon]{up}B$ \(\require{extpfeil}\) $A\xlongequal[dwon]{up}B$ $A\xlongequal[.5d6d.net]{kkkkuingggg}B$ 以上的代码: \(\verb"$A\xlongequal[dwon]{up}B$"\) $\verb"\(\require{extpfeil}\)"$ \(\verb"$A\xlongequal[dwon]{up}B$"\) \(\verb"$A\xlongequal[.5d6d.net]{kkkkuingggg}B$"\)
kuing 7# 2012-8-28 17:04
啊哈哈哈,终于找到自动的了 这里的 $\verb"\(\require{extpfeil}\)"$ 就是加载 extpfeil.js,这个 js 里面就有上面这个代码可以用了,这正类似于 LaTeX 中的加载宏包。
kuing 8# 2013-2-5 15:20
试制作平行且等于符号及平行四边形符号 $b\mathrel{\displaystyle\mathop{=\!\!\!=}^{\,\sslash}}p$   $AB\mathrel{\displaystyle\mathop{=\!\!\!=}^{\,\sslash}}CD=AB\stackrel{\,\sslash}{=\!\!\!=}CD=AB\stackrel{\,\sslash}{\raise-.5ex{=\!\!\!=}}CD$ $\raisebox{1ex}{=}=$ $\raise.22ex\hbox{=}=$ $AB\sslash CD = AB\stackrel{\,/\!/}{\raise-.5ex{=\!\!\!\!=}}CD,AB\stackrel{\,/\!/}{\raise-.5ex{=}}CD,b\stackrel{\,/\!/}{\raise-.5ex{=\!\!\!\!=}}p$ \(\newcommand\tmpa{\mathord{\scriptstyle\raise.3ex{/}\kern-.35em\raise-.5ex{{-}\!{-}}\kern-.7em\raise1.1ex{{-}\!{-}}\kern-.35em\raise.3ex{/}}} \newcommand\tmpb{\stackrel{\,/\!/}{\raise-.5ex{=\!\!\!\!=}}}\) 测试一下:在 $\tmpa ABCD$ 中,$AB\tmpb CD$。 再测试一下:在 $S_{\tmpa ABCD}$ 中,$P_{AB\tmpb CD}$。 再测试一下:在 $\tmpa ABCD$ 中,$AB\tmpb CD$。
thread-419-1-2.html: [转]原来它就是一只乌龟...
kuing 1# 2012-3-23 00:27

戊概念·五 2# 2012-8-25 23:27
5只叉烧。。。。。
kuing 3# 2012-8-25 23:38
2# 戊概念·五 因为图片链接人教……
戊概念·五 4# 2012-8-25 23:48
3# kuing 这次进来,连叉烧都木的踪迹了
thread-42-1-1.html: 初次打开本论坛 直接卡死几秒
isea 1# 2011-10-3 20:57
Opera 还是不成啊 机器也破了ing
kuing 2# 2011-10-3 21:19
第一次这样比较正常,主要还是因为要 load 那些 js 文件,所以还要看 mathjax 那边的响应速度,今天下午的时候那边几乎没有响应,以致于卡了好一会,傍晚才好了些,现在可能还不太稳定。
thread-424-1-7.html: [数论] 05江苏预赛一个题
syzychenwj 1# 2012-3-26 18:03
设数列$\{a_n\}$:$a_0=2$,$a_1=16$,$a_{n+2}=16a_{n+1}-63a_n$,$n\in N^*$,则$a_{2005}$被$8^2$除的余数为? ________kuing编辑________ 设数列$\{a_n\}$:$a_0=2$,$a_1=16$,$a_{n+2}=16a_{n+1}-63a_n$,$n\in N^*$,则$a_{2005}$被$8^2$除的余数为? 自己编辑贴子对比一下输入
海盗船长 2# 2012-3-26 18:55
1# syzychenwj 应该有周期性
海盗船长 3# 2012-3-26 18:57
$a_n$取模$8^2$之后取值范围有限,肯定会有两项和开始的两项相同。
kuing 4# 2012-3-26 18:58
数据给得很好啊,直接就是 $a_n=7^n+9^n$
syzychenwj 5# 2012-3-26 19:02
1.多谢kuing指教,我感觉我学的不错似的,差的老远,但比我的能力,我很努力,新东西真的多啊;但课本中N为正粗体; 2.海盗,可求通项,且数列$\{a_n\}$模$8^2$周期为2,怎么证呢?
syzychenwj 6# 2012-3-26 19:07
晕了,{  }中间啥也没有呢?
海盗船长 7# 2012-3-26 19:13
5# syzychenwj $a_n \pmod {8^2}$只能在$0-63$取值,前后两项的组合方式有限,在有限项之后必然会再次出现$2$和$16$,于是$a_n \pmod {8^2}$有周期。
kuing 8# 2012-3-26 19:16
1.多谢kuing指教,我感觉我学的不错似的,差的老远,但比我的能力,我很努力,新东西真的多啊;但课本中N为正粗体; syzychenwj 发表于 2012-3-26 19:02 我是见你用 N* 我才照着打,比较懒。正规一点的应该用 \mathbb{N} 或 \mathbf{N},效果 $n\in\mathbb{N}^+$、$n\in\mathbf{N}^+$
海盗船长 9# 2012-3-26 19:16
$\mathbb{N}$ $\textbf{N}$
kuing 10# 2012-3-26 19:49
?????? 谁封了楼主?
kuing 11# 2012-3-26 19:53
好像知道原因了 这贴有点危险
海盗船长 12# 2012-3-26 19:57
?哪里封了?
海盗船长 13# 2012-3-26 20:00

syzychenwj 14# 2012-3-26 20:02
我正郁闷呢,我还以为嫌弃我的水平差呢! 危险是啥意思?说了不该说的么? 会不会出现无限不循环呢?周期怎么证呢?难道算到$a_2=2(mod8^2)$就说周期是2不成?
海盗船长 15# 2012-3-26 20:08
15# syzychenwj 算到$2$还不能说,需要两项重复出现,因为递推函数是依赖两个初值的,其实是$\{2,16,2,48,2,16,\cdots\}$这样。
syzychenwj 16# 2012-3-26 20:16
我再慢慢学吧!哪里的定理?
海盗船长 17# 2012-3-26 20:18
17# syzychenwj http://ishare.iask.sina.com.cn/f/10767338.html
syzychenwj 18# 2012-3-26 20:20
忘了谢谢了!谢谢二位!以后题目肯定还有很多,二位要费心了!
kuing 19# 2012-3-26 20:25
?哪里封了? 海盗船长 发表于 2012-3-26 19:57 刚才我吃完饭回来发现楼主变成了禁止发言 然后我进后台解封了 原因可能是碰到了 敏gan字符 ------ six four
kuing 20# 2012-3-26 22:15
我正郁闷呢,我还以为嫌弃我的水平差呢! syzychenwj 发表于 2012-3-26 20:02 这不可能,我的人品还不至于这么****吧,呵呵
thread-424-2-7.html:
realnumber 21# 2012-5-26 07:03
本帖最后由 realnumber 于 2012-5-26 07:08 编辑 其实和这个有点类似,$a_1=1,a_n=2a_{n-1}+1$ 设$a_{n+2}+xa_{n+1}=y(a_{n+1}+xa_n)$,对比原题条件可得$y-x=16,and, yx=-63$,那么有2组解$y=9,x=-7 or y=7.x=-9$ 此时数列{$a_{n+2}+xa_{n+1}$}为等比数列,可由刚才2组解,分别解得2个等比数列的通项公式,再解得$a_n$ 看了楼上发言,发现$a_{n+2}=16a_{n+1}+a_n$,也可以变为1楼问题.
thread-425-1-7.html: [函数] 两个FAQ
syzychenwj 1# 2012-3-29 18:28
本帖最后由 syzychenwj 于 2012-3-29 18:41 编辑 1.若关于$x$的方程$\lg (x-1)+\lg (3-x)=\lg (x-a)$只有一解,则实数$a$的取值范围? $x>a$是不是可以不关注? 2.已知$1\leqslant a\leqslant \sqrt{2}$,则方程$\sqrt{a^2-x^2}=\sqrt{2}-|x|$的相异实根个数是? 按$x\leqslant |a|$考虑和按$x\leqslant \sqrt{2}$考虑相同的理由是什么?
syzychenwj 2# 2012-3-29 18:45
今天网站好涩啊!改了那么长时间才差不多像! 很简单的问题,本来不想麻烦二位,人教那边级别又不够,追在人家后边,理睬的又不多,故又拉到这边了。
kuing 3# 2012-3-29 18:54
今天那个公式好像老load不了 不过从代码来看,不像是FAQ吧……
①②③④⑤⑥⑦ 4# 2012-3-30 10:59
1# syzychenwj 第一个 由于 $f(x)=(x-1)(3-x)$ 在 $[1,3]$ 上非负,以及 $g(x)=x-a$ 单调增,$g(a)=0$,交点的 $x>a$ 是必然满足的
syzychenwj 5# 2012-3-30 14:37
哦,按您是这个意思: 1.原方程等价于方程$(x-1)(3-x)=x-a$,且$1<x<3$,因为首先不可能有超出$1<x<3$解,而当$1<x<3$时,$(x-1)(3-x)>0$,若$x-a=(x-1)(3-x)$有解,必有$x>a$;否则就不可能是解; 同理: 2.原方程等价于方程$a^2-x^2=(\sqrt{2}-|x|)^2$,且$|x|\leqslant \sqrt{2}$,因为由$|x|\leqslant \sqrt{2}$,方程不可能有超出$|x|\leqslant \sqrt{2}$的解,而若$a^2-x^2=(\sqrt{2}-|x|)^2$有解,也自然不会出现$x^2>a$的。 故都只需考虑稳定的条件限制就可以了。
syzychenwj 6# 2012-3-30 14:39
谢谢啊!
thread-426-1-7.html: 三次方程的问题
syzychenwj 1# 2012-3-30 21:33
本帖最后由 syzychenwj 于 2012-3-30 22:14 编辑 设$θ$是三次多项式$f(x)=x^3-3x+10$ 的一个根,且$α=\dfrac{θ^2+θ-2}{2} $.若$h(x)$是一个有理系数的二次多项式,满足条件$h(α)=θ$,求$h(0)$ . ______kuing编辑_______ 你的\dfrac{θ^2+θ-2}{2}后面少了一个美元符号,所以后面都乱了,后面就显示不出该变成公式的部分。 设 $\theta$ 是三次多项式 $f(x)=x^3-3x+10$ 的一个根,且 $\alpha=\dfrac{\theta^2+\theta-2}{2}$。若 $h(x)$ 是一个有理系数的二次多项式,满足条件 $h(\alpha)=\theta$,求$h(0)$。
syzychenwj 2# 2012-3-30 21:43
实在不会了,后面的怎么也没打成斜体!
kuing 3# 2012-3-30 21:53
see also http://bbs.pep.com.cn/thread-548287-1-1.html
syzychenwj 4# 2012-3-30 22:20
比数学严谨!题目正在学习!thanx
thread-427-1-7.html: 费解:整系数方程有理根的判定定理
syzychenwj 1# 2012-3-31 19:28
本帖最后由 syzychenwj 于 2012-3-31 19:34 编辑 定理:若形如$a_0x^n+a_1x^{n-1}+…+a_{n-1}x+a_n=0$(其中$a_0,a_1,…,a_n$均为整数)的方程有有理根,则其有理根为有理数$\dfrac{p}{q}$(其中$p$为$a_n$的约数,$q$为$a_0$的约数,且$p,q$互质)。 证明:若方程$a_0x^n+a_1x^{n-1}+…+a_{n-1}x+a_n=0$(其中,$a_0,a_1,…,a_n$均为整数)其有理根$\dfrac{p}{q}$($p,q$互质),则方程一定可分解为如下形式: $(qx-p)(b_1x^{n-1}+…+b_{n-1}x+b_n)=0$(其中,$b_1,b_2,…,b_n$均为整数) 展开后得:$qb_1x^n+(qb_2-pb_1)x^{n-1}+…+(qb_n-pb_{n-1})x-pb_n=0$ 与原方程比较系数,得:$a_0=qb_1,a_n=-pb_n$,因此,$p$为$a_n$的约数,$q$为$a_0$的约数(证毕). 请教之处:$(qx-p)(b_1x^{n-1}+…+b_{n-1}x+b_n)=0$(其中,$b_1,b_2,…,b_n$均为整数?????)为什么$a_0=qb_1$为什么不是$a_0t=qb_1$?
①②③④⑤⑥⑦ 2# 2012-4-1 09:25
1# syzychenwj 可以直接一点嘛 代入易得 \[a_nq^n+a_{n-1}q^{n-1}p+\cdots+a_1qp^{n-1}+a_0p^n=0\] 于是 \[p(a_{n-1}q^{n-1}+\cdots+a_1qp^{n-2}+a_0p^{n-1})=-a_nq^n\] 就有 $p|a_n$ 类似可得  $q|a_0$
syzychenwj 3# 2012-4-1 15:46
嗯,谢谢!那原证明可修复吗?
thread-428-1-7.html: [数论] 请教一个不等式求最值的题目,先谢谢了!
hongxian 1# 2012-4-2 07:06

天涯无际 2# 2012-4-5 10:25
用猥琐的方法算得结果2011~不知道对不对~
thread-429-1-1.html: 手写自动识别生成latex代码
kuing 1# 2012-4-3 00:38
see http://webdemo.visionobjects.com/equation.html?locale=zh_CN 测试效果
kuing 2# 2012-10-14 14:57
顶一下这个
thread-43-1-1.html: 重积分符号测试
kuing 1# 2011-10-3 22:20
\[\int\int f(x) \,\mathrm{d}x\] \[\int f(x) \,\mathrm{d}x \quad \iint f(x) \,\mathrm{d}x \quad \iiint f(x) \,\mathrm{d}x\] \[\iiiint f(x) \,\mathrm{d}x \quad \idotsint f(x) \,\mathrm{d}x \quad \oint f(x) \,\mathrm{d}x\] 奇怪,四重的间距就不会变小了。。。 原来是跟真 latex 有差别
kuing 2# 2011-10-4 11:43
更复杂的重积分号似乎这里打不了,在真 LaTeX 里也需要其他宏包才可
thread-430-1-1.html: 求解一道极限课后习题的解析
fredjhon 1# 2012-4-3 13:31
计算极限$lim_{x\to\+infty}(\sqrt{x+\sqrt{x+\sqrt{x}}}-\sqrt{x+\sqrt{x}})\sqrt{x}$
kuing 2# 2012-4-3 13:51
计算极限$lim_{x\to\+infty}(\sqrt{x+\sqrt{x+\sqrt{x}}}-\sqrt{x+\sqrt{x}})\sqrt{x}$ fredjhon 发表于 2012-4-3 13:31 分子有理化有 \begin{align*}   & \sqrt{x+\sqrt{x+\sqrt{x}}}-\sqrt{x+\sqrt{x}} \\ ={}&\frac{x+\sqrt{x+\sqrt{x}}-\left( x+\sqrt{x} \right)}{\sqrt{x+\sqrt{x+\sqrt{x}}}+\sqrt{x+\sqrt{x}}} \\ ={}&\frac{\sqrt{x+\sqrt{x}}-\sqrt{x}}{\sqrt{x+\sqrt{x+\sqrt{x}}}+\sqrt{x+\sqrt{x}}} \\ ={}&\frac{x+\sqrt{x}-x}{\left( \sqrt{x+\sqrt{x+\sqrt{x}}}+\sqrt{x+\sqrt{x}} \right)\left( \sqrt{x+\sqrt{x}}+\sqrt{x} \right)} \\ ={}&\frac{\sqrt{x}}{\left( \sqrt{x+\sqrt{x+\sqrt{x}}}+\sqrt{x+\sqrt{x}} \right)\left( \sqrt{x+\sqrt{x}}+\sqrt{x} \right)}, \end{align*} 所以 \begin{align*}   & \lim_{x\to +\infty }\left( \sqrt{x+\sqrt{x+\sqrt{x}}}-\sqrt{x+\sqrt{x}} \right)\sqrt{x} \\ ={}&\lim_{x\to +\infty }\frac{x}{\left( \sqrt{x+\sqrt{x+\sqrt{x}}}+\sqrt{x+\sqrt{x}} \right)\left( \sqrt{x+\sqrt{x}}+\sqrt{x} \right)} \\ ={}&\lim_{x\to +\infty }\frac{1}{\left( \sqrt{1+\sqrt{\frac{1}{x}+\frac{1}{x\sqrt{x}}}}+\sqrt{1+\frac{1}{\sqrt{x}}} \right)\left( \sqrt{1+\frac{1}{\sqrt{x}}}+1 \right)} \\ ={}&\frac{1}{(1+1)(1+1)} \\ ={}&\frac{1}{4}. \end{align*}
fredjhon 3# 2012-4-4 11:59
和参考答案一样,我不明白的地方是分子,分母同除以X后,分母根号里的变化,大侠能详细解答下 吗?     没办法我的高数基础实在是太差了!
kuing 4# 2012-4-4 13:31
分母除以x,分给两个括号,即两个括号里分别除以$\sqrt x$,然后放进里面,一层一层放就行了。 PS:这跟高数基础没什么关系,因为这属于基本的指数运算,不曾涉及高数内容。
kuing 5# 2012-4-4 13:34
演示一个,其余自己试试 \begin{align*} \frac{\sqrt{x+\sqrt{x+\sqrt{x}}}}{\sqrt{x}}&=\sqrt{\frac{x+\sqrt{x+\sqrt{x}}}{x}} \\ & =\sqrt{1+\sqrt{\frac{x+\sqrt{x}}{x^{2}}}} \\ & =\sqrt{1+\sqrt{\frac{1}{x}+\frac{1}{x\sqrt{x}}}} \end{align*}
fredjhon 6# 2012-4-5 08:55
谢谢大侠,明白了
thread-431-1-1.html: 一个japan的latex网站
kuing 1# 2012-4-5 00:08
主页 http://www.biwako.shiga-u.ac.jp/sensei/kumazawa/tex.html 数学公式展示 http://www.biwako.shiga-u.ac.jp/ ... index3.html#formula 各种例子包含源码下载,可以慢慢学习。
老樊 2# 2012-7-28 11:35
小日本做的这个还是不错的
kuing 3# 2012-7-28 15:34
我都几乎忘记了还有这个网……
thread-434-1-7.html: [组合] 请教各位老师 涂色问题
独药 1# 2012-4-9 14:16
额 上课没听明白    排列组合中,区域涂色问题 1、为什么要讨论同色和不同色? 2、讨论的标准是什么? 3、为什么有的题讨不讨论结果都一样啊 谢谢!!
海盗船长 2# 2012-4-9 18:28
举一个具体的例子吧
wenshengli 3# 2012-4-9 19:57
1# 独药 这样的问题一般两种方法,一是楼主说的这种叫元素分析法,看几种颜色能满足题意,所以要讨论;另一种方法叫位置分析法。
李斌斌755 4# 2012-4-16 14:19
排列组合,数论,坐沙发听课
thread-435-1-1.html: 积分
图图 1# 2012-4-11 17:56
本帖最后由 图图 于 2012-4-11 18:17 编辑 这个竖线应该用什么命令?
kuing 2# 2012-4-11 18:08
怎样的竖线? 是 \[\left.\frac{dy}{dx}\right|_{x=x_0}\] 还是 \[ \left(\begin{array}{c|cc} 1&2&3\\ \hline 4&5&6\\ \end{array}\right) \]
kuing 3# 2012-4-11 18:09
传不了图片吗?
图图 4# 2012-4-11 18:12
3# kuing 定积分的求出了原函数以后那个竖线 不知道怎么传,我输入了图片在电脑上的地址不行
kuing 5# 2012-4-11 18:14
4# 图图 你是不是点击这里 来上传的?
kuing 6# 2012-4-11 18:15
\[\int_a^b f(x) dx = F(b)-F(a) = \left.F(x)\right|_a^b\] \int_a^b f(x) dx = F(b)-F(a) = \left.F(x)\right|_a^b \[\int_1^2 \frac1{x^2} dx = \left.\frac1x\right|_2^1\] \int_1^2 \frac1{x^2} dx = \left.\frac1x\right|_2^1
图图 7# 2012-4-11 18:18
6# kuing 呃...不是,上面的 额,就是这种线
kuing 8# 2012-4-11 18:21
\left 和 \right 配对使用  只要一边  用 . 来隐藏
图图 9# 2012-4-11 18:21
6# kuing 知道嘞,原来就是最普通的,但是这种怎么判断唉
图图 10# 2012-4-11 18:22
8# kuing 这个你跟我说过嘞 之前用过唉
kuing 11# 2012-4-11 18:22
判断什么?
图图 12# 2012-4-11 18:24
11# kuing 竖线有好多种咩
kuing 13# 2012-4-11 18:25
看情况吧,一时很难说清 你举点例子
图图 14# 2012-4-11 18:25
13# kuing 额,碰到了再说吧
kuing 15# 2012-4-11 18:37
对于绝对值以及楼主这种的竖线,可以统一用 \left| \right|,只需一边的时候用 . 来隐藏即可。比如 \[\left|-2\right|  或  \left|\frac12\right|  或  \left.f(x)\right|_a^b  或  \left.-\frac1x\right|_a^b\] 但不要用 |-2|,原因我之前说过了的。对比一下 \begin{gather} |-2|=2 \\ \left|-2\right|=2 \end{gather} 第一个是 |-2|=2,第二个是 \left|-2\right|=2。 还有 \lvert 和 \rvert 用于最短的且有分左右的竖线,大一点的可以用 \bigl| \bigr| 之类的,但如果不是非常讲究的话,统一用 \left 和 \right 就行了。 而对于表格或矩阵分块之类的竖线就要用别的方式,那些到时再说
图图 16# 2012-4-11 18:55
15# kuing 你上次跟我说标准的用\lvert 嗯,其他的碰到再说,主要是我没在弄几代的,不然也需要了啦,分块
kuing 17# 2012-4-11 18:59
\begin{gather} (a+b)\left|a+b\right|\\ (a+b)\lvert a+b\rvert \end{gather} 第一个是 (a+b)\left|a+b\right| 第二个是 (a+b)\lvert a+b\rvert 这里没分别,但在真 LaTeX 里是有区别的 真 LaTeX 里,\left| XXX \right| 的两边会产生一点小距离,不过好像有的情况又不会,具体规则我不记得了,有空细看再来修改下。
kuing 18# 2012-4-11 19:04
所以我在能直接用 ( ) 的时候就直接 ( ),而不会随意加 \left \right ,需要稍大点的时候用 \bigl  \bigr  \biggl 等
图图 19# 2012-4-11 19:10
17# kuing 嗯,不仔细看看不出来啊,好多细节啊
kuing 20# 2012-4-11 19:16
嗯,所以我说,在不是非常讲究的情形下,可以忽略这些,直接统一用 \left \right 。 吃饭
thread-435-2-1.html:
图图 21# 2012-4-11 19:18
20# kuing 嗯... 我把那个证明结束符改了
kuing 22# 2012-4-11 19:44
21# 图图 难道是抠鼻
图图 23# 2012-4-11 19:57
22# kuing 嘿嘿,不是,换了一个简单清爽一点的
kuing 24# 2012-4-11 20:03
23# 图图 你自己制作的图片么?
图图 25# 2012-4-11 20:08
24# kuing 不是唉,我是觉得那个人人的害羞突然看起来有点脑瘫啊,就换了唉
thread-436-1-1.html: 求级数的和
图图 1# 2012-4-11 18:28
从这个能得到什么唉?
kuing 2# 2012-4-11 18:42
={} & ...
kuing 3# 2012-4-11 18:55
给合上下文…………
图图 4# 2012-4-11 19:07
3# kuing 就是求级数 $\displaystyle\sum_{n=1}^{\infty}r^n\sin nx$ 和 $\displaystyle\sum_{n=1}^{\infty}r^n\cos nx$ 的和唉
icesheep 5# 2012-4-15 01:12
4# 图图 不就是等比级数嘛。你一楼最后一个等号后面的东西错了吧,n 被求和号加掉了,表达式里应该就没有 n 了。
都市侠影 6# 2012-6-5 14:26
本帖最后由 都市侠影 于 2012-6-5 14:29 编辑 用复数试试,令$a=r(\cos{x}+i\sin{x})$,然后求级数$\sum_{i=1}^{\infty}a^{n}$的和 $\sum_{i=1}^{\infty}a^{n}=\lim_{n\to\infty}\frac{a(1-a^{n})}{1-a}$ 复数没学好,这个往下就不会了 思路就是求出来后分离实部和虚部
thread-437-1-1.html: 举反例 $\int_a^{+\infty}f(x)\mathrm{d}x$
图图 1# 2012-4-11 20:43
举例说明由$\int_{a}^{+\infty}f(x)dx$收敛不能得到$\int_{a}^{+\infty}|f(x)|dx$收敛。
kuing 2# 2012-4-11 20:56
f(x)可不可以分段?
图图 3# 2012-4-11 21:11
2# kuing 可以吧,只有满足那个收敛就行了吧
kuing 4# 2012-4-11 21:25
就是可以不连续咯?
kuing 5# 2012-4-11 21:29
\[\Large f(x)=\frac1{[x]\cdot(-1)^{[x]}}\quad x\geqslant 1\] 利用的就是 1-1/2+1/3-1/4+... 收敛而 1+1/2+1/3+1/4+... 不收敛
kuing 6# 2012-4-11 21:35
将上面的例子改成连续也是很容易的,只要将每一段直线两端拉向x轴连着,变成一个个等腰三角形,这样就连续了。 想要光滑也容易,也是拉,但是每一段都变成半个椭圆,这样连接起来就是光滑的,而每个椭圆的面积比没变,所以也符合反例。 但是椭圆情况的话连接处切线斜率垂直于x轴,算不上可导,而要在可导的话暂时还举不来。
图图 7# 2012-4-11 23:02
6# kuing 懂了唉, 刚才跟侄女视频了,嘿嘿
图图 8# 2012-4-11 23:03
6# kuing 我整理一下啊
kuing 9# 2012-4-11 23:18
8# 图图 你看能不能仿着整一个可导的简单的反例出来
海盗船长 10# 2012-4-12 20:06
$\int_0^{+\infty} \frac{\sin x}{x} \mathrm{d}x$ $\int_0^{+\infty} \sin {x^2} \mathrm{d}x $
kuing 11# 2012-4-12 20:36
10# 海盗船长 这个绝对值的时候发散怎么证 昨晚一开始就考虑sinx/x,只是没证出来
图图 12# 2012-4-12 23:26
10# 海盗船长 谢谢海盗
kuing 13# 2012-4-12 23:48
第一个知道了,平方的呢……
kuing 14# 2012-4-12 23:52
噢,好像也差不多,这样看来n次方也一样?
海盗船长 15# 2012-4-13 19:11
嗯,应该是的
pxchg1200 16# 2012-4-13 22:19
11# kuing 绝对值发散很好证明啊。 由于 $ \frac{|\sin{x}|}{x}\geq \frac{\sin^{2}{x}}{x}=\frac{1-\cos{2x}}{2x}=\frac{1}{2x}-\frac{\cos{2x}}{2x} $ 第一部分是发散的,第二部分是收敛的。所以这个差是发散的。
kuing 17# 2012-4-13 22:29
16# pxchg1200 嗯,后来我也知道了,而且平方、n次方也类似,见上面回贴……
秋风树林 18# 2012-4-14 09:28
本帖最后由 秋风树林 于 2012-4-14 22:08 编辑 这就是一个简单的条件收敛和绝对收敛的问题撒 绝对收敛一定收敛,条件收敛不绝对收敛 数分书上的例子很多,做都要做吐了
kuing 19# 2012-4-14 22:22
18# 秋风树林 举多几个例子来瞧瞧
秋风树林 20# 2012-4-15 10:11
19# kuing 这些都是随着p,q取值范围不同而有不同类型的敛散性 当然,本身而言,由着Abel-Dirichlet判别法就可以很容易构造出各种类型的条件收敛积分。。。 __________kuing edit in $\LaTeX$__________ \begin{gather*} \int_{1}^{+\infty}\frac{x^q\sin x}{1+x^p}dx,\\ \int_{1}^{+\infty}\frac{e^{\sin x}\cos x}{x^p}dx,\\ \int_{0}^{+\infty}\frac1{x^p}\cos\frac1{x^2}dx,\\ \int_{1}^{+\infty}\frac{\sin\bigl(x+\frac1x\bigr)}{x^p}dx, \end{gather*}
thread-438-1-3.html: 几个经典问题
kuing 1# 2012-4-11 23:20
http://www.guokr.com/article/60677/
kuing 2# 2013-3-26 21:58
刚才看到了类似于第一个问题的视频版,还连续用了三次…… http://v.youku.com/v_show/id_XNTMyOTQ1MTYw.html
yes94 3# 2013-3-26 22:16
2# kuing 裴波那切数列
yes94 4# 2013-3-26 22:19
该视频能否下载啊?我想要!但下载不了
isea 5# 2013-3-26 23:30
该视频能否下载啊?我想要!但下载不了 yes94 发表于 2013-3-26 22:19 打开页面,正对视频中心向下看,看到“收藏  下载  用手机看  播放:“字样,点下载,就有提示了。 用其专用工具下载,下完,转成MP4之类的格式即可
第一章 6# 2013-3-26 23:34
我已经下载完了。如果要,我传给你们
kuing 7# 2013-3-26 23:36
6# 第一章 放网盘,给链接……
yes94 8# 2013-3-26 23:37
5# isea 我也下载完了,但只能用优酷软件播放,其他软件无法播放,换一台电脑没优酷软件就不行了 如何转成MP4之类的格式?
isea 9# 2013-3-27 00:42
8# yes94 优酷软件 有一个文件转换按钮,点一下就明白,将这种格式转化为MP4非常快。
yes94 10# 2013-3-27 11:14
9# isea 谢谢!终于会了!
yes94 11# 2013-3-27 17:43
6# 第一章 昨晚谢谢啦
第一章 12# 2013-3-27 18:46
这个拍视频的肯定是做了手脚, 看看这图
yes94 13# 2013-3-27 21:13
12# 第一章 为何?以前看过一个裴波那契的
第一章 14# 2013-3-28 08:51
两张图片都是拿出来三小块后的截图, 第一张是装在木匣子里面的,脉络清晰; 第二张是从木匣子里面拿出来的,那些脉络相当的混乱。
thread-44-1-2.html: 水水更健康
kuing 1# 2011-10-4 11:42
本版允许随意聊天吹水,只要不涉及敏感内容即可(否则论坛会危险喔:lol)
图图 2# 2011-10-4 12:38
......跟你的头像一样脑瘫;P
isea 3# 2011-10-4 15:23
话说那图象到底是哪个动漫里的人?
kuing 4# 2011-10-4 15:29
;P 咸蛋超人……
戊概念·五 5# 2011-11-2 14:13
喜欢该主角的身高~ http://sports.qq.com/a/20111101/000400.htm#p=11
thread-440-1-7.html: [不等式] 请教问题
yayaweha 1# 2012-4-13 20:44
2012广州一模理数压轴题,谁有其他解法,我不会用MATHJAX呀!谁能帮我打一下!再解一下
kuing 2# 2012-4-13 20:47
没看过这题,帮你唔到…… 不会打就直接截图发上来吧,如果有电子版的话
yayaweha 3# 2012-4-13 20:55
word发不上来
kuing 4# 2012-4-13 20:58
3# yayaweha 不会截图吗?
yayaweha 5# 2012-4-13 20:58

yayaweha 6# 2012-4-13 20:59
来挑战一下吧
kuing 7# 2012-4-13 21:02
呃,既然是求别的解法,那就把标答发上来先,不然做得一样就白做了
pxchg1200 8# 2012-4-13 22:10
一,二问都很显然啊,第三问用拉格朗日型余项的Taylor公式。
海盗船长 9# 2012-4-15 11:00
5# yayaweha 第三问左边: 只需证明$\left( \frac{2}{n+1} \right)^n\le \frac{1}{n!}$ $\Longleftrightarrow \quad n!\le \left(\frac{n+1}{2}\right)^n$ 用均值就OK了
thread-442-1-7.html: [不等式] 再来$\sum\frac{x+y}{xy(k-xy)}$
kuing 1# 2012-4-14 19:03
$k=4$的时候在这里 http://kkkkuingggg.5d6d.com/thread-40-1-1.html 已经讨论过了,今天在不等式研究群里有人提到“将常数4,改为alfa,则alfa最好的下界会是多少?可以是9/4?” 刚才尝试之下发现的确可以是$9/4$,当然,右边的数也要改一改,即: 已知 $x$, $y$, $z>0$, $x+y+z=3$,则 \[\frac{x+y}{xy\bigl(\frac94-xy\bigr)}+\frac{y+z}{yz\bigl(\frac94-yz\bigr)}+\frac{z+x}{zx\bigl(\frac94-zx\bigr)}\geqslant \frac{24}5.\] 首先由$xy\leqslant(x+y)^2/4<(x+y+z)^2/4=9/4$等知原不等式左边每项的分母均为正。仍然可以用切平面法,有 \begin{align*} &\frac{x+y}{xy\bigl(\frac94-xy\bigr)}-\frac{12x+12y+16}{25}\\ ={}&\frac{\bigl(\sqrt x-\sqrt y\bigr)^2(12x^2y^2-27xy+25)+2\sqrt{xy}\bigl(\sqrt{xy}-1\bigr)^2\bigl(12xy+32\sqrt{xy}+25\bigr)}{25xy\bigl(\frac94-xy\bigr)}\\ \geqslant{}&0, \end{align*} 类似地有另外两式,相加即得证。 但用那贴里9楼的先分子均值再切线法就不行了,用那贴里15楼pxchg1200的方法虽然还行,但后面就不太容易证了,下面详细写写。 这里证明将$4$改成任意的$k\geqslant9/4$也可以,即: 已知 $x$, $y$, $z>0$, $x+y+z=3$, $k\geqslant 9/4$,则 \[\frac{x+y}{xy(k-xy)}+\frac{y+z}{yz(k-yz)}+\frac{z+x}{zx(k-zx)}\geqslant \frac6{k-1}.\] 同样地各项分母为正,对分子用均值,则只需证明如下更强式 \[\frac1{\sqrt{xy}(k-xy)}+\frac1{\sqrt{yz}(k-yz)}+\frac1{\sqrt{zx}(k-zx)}\geqslant \frac3{k-1}.\] 由柯西不等式,有 \[\sum\frac1{\sqrt{xy}(k-xy)}\geqslant \frac9{k\sum\sqrt{xy}-\sum(xy)^{3/2}},\] 故只需证 \[3k-3\geqslant k\sum\sqrt{xy}-\sum(xy)^{3/2},\] 或 \[k\left( 3-\sum\sqrt{xy} \right)+\sum(xy)^{3/2}-3\geqslant 0,\] 注意到$\sum\sqrt{xy}\leqslant \sum(x+y)/2=3$,所以上式关于$k$递减,从而只需证 \[\frac94\left( 3-\sum\sqrt{xy} \right)+\sum(xy)^{3/2}-3\geqslant 0,\] 令$x=a^2$, $y=b^2$, $z=c^2$, $a$, $b$, $c>0$,则$a^2+b^2+c^2=3$,代入上式并齐次化再化简整理等价于 \[5\left( \sum a^2 \right)^3+36\sum a^3b^3\geqslant 9\left( \sum a^2 \right)^2\sum ab.\] 此式用常规方法似乎不太好证,Schur分拆也有负系数,SOS也很复杂,故此这里用pqr算了。 记$p=a+b+c$, $q=ab+bc+ca$, $r=abc$,上式整理为 \[f(r)=108r^2-108pqr+5p^6-39p^4q+96p^2q^2-40q^3\geqslant0,\] 由$pq/2>r$知$f(r)$关于$r$递减,于是只要证明当$r$最大时,即当$a$, $b$, $c$中有两元相等时不等式成立即可,由对称性,不妨设$a=tb=tc$, $t>0$,代入化简知等价于 \[(5t^4-8t^3+8t+40)(t-1)^2\geqslant0,\] 而 \[5t^4-8t^3+8t+40=5t^2(t-1)^2+2t\left(t-\frac54\right)^2+\frac{39}8t+40>40>0,\] 所以原不等式成立。
kuing 2# 2012-4-14 23:58
带$k$来切平面也可以 \[\frac{x+y}{xy(k-xy)}-\frac{4k-8+(3-k)(x+y)}{(k-1)^2}=\frac{2\sqrt{xy}\bigl( \sqrt{xy}-1 \bigr)^2\cdot M + \bigl( \sqrt x-\sqrt y \bigr)^2\cdot N}{xy(k-xy)(k-1)^2},\] 其中 \begin{align*} M&=(k-1)^2+(3-k)xy+2\sqrt{xy},\\ N&=(k-1)^2+(k-3)xy(k-xy) , \end{align*} 下面证明当$k\geqslant 9/4>xy>0$时总有$M>0$且$N\geqslant0$。 (1)若$k>3$,则显然$N>0$,而此时$M$为关于$\sqrt{xy}$的开口向下的二次函数,故由$3/2>\sqrt{xy}>0$得到 \[M>\min\left\{(k-1)^2, (k-1)^2+(3-k)\cdot \left(\frac32\right)^2+2\cdot \frac32\right\}=\min\left\{(k-1)^2,\frac14(4k^2-17k+43)\right\},\] 由$17^2-4\cdot4\cdot43=-399<0$知$4k^2-17k+43>0$,从而$M>0$; (2)若$3\geqslant k\geqslant 9/4$,则显然$M>0$,将$N$整理为$N=(3-k)(xy)^2+k(k-3)xy+(k-1)^2$,故此时$N$为关于$xy$的开口向上的二次函数,计算其判别式得 \[\Delta=k^2(k-3)^2-4(3-k)(k-1)^2=(k-3)(k-2)(k^2+3k-2),\] 故由$3\geqslant k\geqslant 9/4$时显然有$\Delta\leqslant0$,从而$N\geqslant0$。 综上所述,我们得到当$k\geqslant 9/4>xy>0$时有 \[\frac{x+y}{xy(k-xy)}\geqslant\frac{4k-8+(3-k)(x+y)}{(k-1)^2},\] 从而 \[\sum\frac{x+y}{xy(k-xy)}\geqslant\sum\frac{4k-8+(3-k)(x+y)}{(k-1)^2}=\frac6{k-1}.\]
kuing 3# 2012-4-15 01:02
现在就差 \[5\left( \sum a^2 \right)^3+36\sum a^3b^3\geqslant 9\left( \sum a^2 \right)^2\sum ab\] 这一个不等式要是有好证法的话就比较perf...了(又忘记单词怎么拼了)说不准还能发表发表
蒋总裁 4# 2012-4-15 19:59
本帖最后由 蒋总裁 于 2012-4-15 20:01 编辑 $k=\frac{9}{4}$时等价于 证明 $\sum_{cyc}\frac{z(3-z)}{\frac{9}{4}-xy}\ge\frac{24xyz}{5}$ 用切线法不难证明 $\frac{1}{\frac{9}{4}-xy}\ge\frac{16xy+4}{25}$ 于是只需要证明 $\sum_{cyc}\frac{z(3-z)(16xy+4)}{25}\ge\frac{24xyz}{5}$ 等价于$xy+yz+zx\ge3xyz$ 也即$(x+y+z)(xy+yz+zx)\ge9xyz$ $k=4$时,方法一样,不再赘述
kuing 5# 2012-4-16 17:30
这个局部切线法好! 看来这个题的确可以当我那篇切线法的好素材 PS、楼上可以适当将某些公式用行间公式(\ [ 和 \ ] 的那种)来写,这样看得清楚些
thread-443-1-7.html: [不等式] 无人问津呀
yayaweha 1# 2012-4-15 10:49
各位来做做
海盗船长 2# 2012-4-15 11:04
1# yayaweha http://bbs.pep.com.cn/thread-695028-1-1.html
kuing 3# 2012-4-15 11:36
如此名题怎么会无人问津
thread-444-1-6.html: [不等式] 不等式研究群里看到的一道四元不等式
kuing 1# 2012-4-17 19:48
已知 $a$, $b$, $c$, $d>0$ 且 $abcd=1$,求证 \[\frac1{bc+cd+da-1}+\frac1{ab+cd+da-1}+\frac1{ab+bc+da-1}+\frac1{ab+bc+cd-1}\leqslant2.\] 我有暴力证,求简证……
kuing 2# 2012-4-17 22:33
汗,被can一眼看穿,原来只要分母均值就…………咳,主要是没想起 1/(1+x)+1/(1+y)=1 当 xy=1
力工 3# 2012-11-30 17:38
2# kuing ,把你的暴力也贴下吧,记录下,不要失传了。
kuing 4# 2012-11-30 21:26
3# 力工 没找到当时的记录,可能删掉了,也不太记得了
thread-445-1-7.html: [函数] 来自pep的一道分式根号三角最大值
kuing 1# 2012-4-19 22:27
原贴地址:http://bbs.pep.com.cn/thread-2433321-1-1.html 求函数 \[f(x)=\frac{\sqrt{2}\sin x+\cos x}{\sin x+\sqrt{1-\sin x}}\quad (0\leqslant x\leqslant \pi )\] 的最大值。 解: \begin{align*} f(x)&=\frac{\sqrt{2}\sin x+\cos x}{\sin x+\sqrt{1-\sin x}} \\ & \leqslant \frac{\sqrt{2}\sin x+\sqrt{\cos ^{2}x}}{\sin x+\sqrt{1-\sin x}} \\ & =\frac{\sqrt{2}\sin x+\sqrt{(1+\sin x)(1-\sin x)}}{\sin x+\sqrt{1-\sin x}} \\ & \leqslant \frac{\sqrt{2}\sin x+\sqrt{2(1-\sin x)}}{\sin x+\sqrt{1-\sin x}} \\ & =\sqrt{2} , \end{align*} 当$x=\pi/2$时取等。
kuing 2# 2012-4-19 22:30
写完想链接过去,发现已经有类似解了,就免去了。
kuing 3# 2012-4-19 23:17
事实上也可以证明在 $[0,\pi/2]$ 递增,在 $[\pi/2,\pi]$ 递减,得最小值 $-1$。 当然,也可以 \begin{align*} f(x)+1 &=\frac{\bigl(\sqrt{2}+1\bigr)\sin x+\cos x+\sqrt{1-\sin x}}{\sin x+\sqrt{1-\sin x}} \\ & \geqslant \frac{\bigl(\sqrt{2}+1\bigr)\sin x+\cos x+1-\sin x}{\sin x+\sqrt{1-\sin x}} \\ & =\frac{\sqrt{2}\sin x+\cos x+1}{\sin x+\sqrt{1-\sin x}} \\ & \geqslant 0, \end{align*} 当$x=\pi$取等。
kuing 4# 2012-4-20 14:58
另外,这个函数在 $x=\pi/2$ 处连续但不可导原因来自 \[\sqrt{1-\sin x}=\sqrt2\left|\sin\left(\frac x2-\frac\pi4\right)\right|\]
李斌斌755 5# 2012-4-20 23:59
4# kuing 怎么理解连续但不可导?!
kuing 6# 2012-4-21 00:05
5# 李斌斌755 like |x| when x=0
李斌斌755 7# 2012-4-21 02:21
6# kuing 谢谢kuing
jiojio795 8# 2012-4-21 21:41
问下中间那个根号下(1+sinx)(1-sinx)为什么小于等于2(1-sinx)啊? 柯西么?
kuing 9# 2012-4-21 23:00
…… sinx<=1 啊
jiojio795 10# 2012-4-22 12:46
突然发现了 那你是怎么想到的呢? 根据分母结构么?
kuing 11# 2012-4-22 14:04
一眼看下去只有分子有个cos,所以会考虑转化成sin就变成关于sinx的函数再看。而不难发现取最大值的时候 cosx 肯定是正的,所以我一开始就将其变成 $\sqrt{1-\sin^2x}$。变成这样后马上就发现只要再放缩一下就OK了。
力工 12# 2012-5-19 10:11
这类题粗看好象不好处理,细想方法却很多的,以kuing的方法最速。学习
realnumber 13# 2012-5-20 00:04
特别声明,这个不等式应该早就有的,比如学分数后,参加竞赛的同学...
kuing 14# 2012-5-20 00:29
13# realnumber 嗯,$a,b,c,d\in\mathbb{R}^+,a/b\leqslant c/d \implies a/b\leqslant (a+c)/(b+d)\leqslant c/d$ 不错。 点 (b,a) 和 (d,c) 在第一象限,后者与原点连结斜率较大,中间的是两者中点与原点连结斜率,故显然,也能推广系数,定比分点也。
thread-446-1-7.html: [函数] 期中考试题不会呀
yayaweha 1# 2012-4-20 20:13
已知函数f(x)=lnx,g(x)=(1/2)ax^2+bx(a≠0) (I)若a=-2时,函数h(x)=f(x)-g(x)在其定义域内是增函数,求b的取值范围; (II)在(I)的结论下,设φ(x)=e2x+bex,x∈[0,ln2],求函数φ(x)的最小值; (III)设函数f(x)的图象C1与函数g(x)的图象C2交于点P、Q,过线段PQ的中点R作x轴的垂线分别交C1、C2于点M、N,问是否存在点R,使C1在M处的切线与C2在N处的切线平行?若存在,求出R的横坐标;若不存在,请说明理由. 第三问怎么做,求解 能否用凸函数去思考
海盗船长 2# 2012-4-21 13:36
好老的题,人教应该讨论过
thread-447-1-7.html: [函数] [个人第一贴]2011年新课标理压轴题的一个还不错的解法
xiaoyu.watson 1# 2012-4-21 11:21
本帖最后由 xiaoyu.watson 于 2012-5-26 21:05 编辑
xiaoyu.watson 2# 2012-4-21 11:49
感谢教师-郝酒指出了一个小错误。已更正。
kuing 3# 2012-4-21 12:13
看这贴:http://bbs.pep.com.cn/thread-1732075-1-1.html
xiaoyu.watson 4# 2012-4-21 12:27
帅爆了
thread-448-1-7.html: [不等式] 求解三元轮换不等式
yayaweha 1# 2012-4-22 16:37
http://imo.wolfram.com/problemset/IMO2001_solution2.html 期中图片的这个不等式怎么变出来的?
yayaweha 2# 2012-4-22 16:38
难道是利用切线法吗
kuing 3# 2012-4-22 16:58
这种齐次构造也是常用的构造局部不等式的方法之一,与切线法不同。 可以通过待定指数来得到。 \[\frac a{\sqrt{a^2+8bc}}\geqslant \frac{a^r}{a^r+b^r+c^r}\] 根据取等条件等来确定r,然后再证之。 当然,这种方法也是试探方法,并非万试万灵的
yayaweha 4# 2012-4-23 14:53
具有做法是怎么样的 kk一下、、演示一下行吗?然后再给几个例子
thread-449-1-1.html: [转]定义两个可选参数的一种方法
kuing 1# 2012-4-23 17:56
http://bbs.ctex.org/viewthread.php?tid=41496 关于在LaTeX中使用newcommand定义多个可选参数的命令, 除了使用TeX层面的定义外, 还可采用迂回一点的办法, 例如 %% 定义具有两个可选参数的命令, 用法: \range[m][n] or \range[m] or \range \newcommand\range[1][m]{\def\rangeFirstPar{#1}\rangeTemp} \newcommand\rangeTemp[1][n]{\ensuremath{\rangeFirstPar,\ldots,#1}}
thread-45-1-2.html: 用变量替换求极限
图图 1# 2011-10-4 11:45
本帖最后由 图图 于 2011-10-4 11:46 编辑 用变量替换求 $\displaystyle\lim_{x\to+\infty}x^{\frac{1}{x}}$
kuing 2# 2011-10-4 14:04
呃,话说这个用 $\varepsilon$-$N$ 可以证了貌似 限方法我总是不会:sleepy:
图图 3# 2011-10-4 14:27
本帖最后由 图图 于 2011-10-4 15:00 编辑 2# kuing :o 应该叫$\varepsilon$-$\delta$语言吧
kuing 4# 2011-10-4 14:31
呃,都差不多吧。。。好像还是 $\varepsilon$-$X$ 。。。 e 的 。。。how?
海盗船长 5# 2011-10-4 14:54
本帖最后由 海盗船长 于 2011-10-4 14:56 编辑 Let $n^{\frac{1}{n}}=1+a_{n}$ and it's obvious that $a_{n}>0$ we get $ n=(1+a_{n})^n>1+n a_{n}+\frac{n(n-1)}{2} a_{n}^2>\frac{n(n-1)}{2} a_{n}^2$ $\Longrightarrow a_{n}<\sqrt{\frac{2}{n-1}}$ $\Longrightarrow 0<a_{n}<\sqrt{\frac{2}{n-1}}$ Thus we have $\lim_{n \to \infty} n^{\frac{1}{n}}=1$
图图 6# 2011-10-4 16:00
5# 海盗船长 这个我们书上还有一个夹逼的证法。 唔,你们用的应该是南大自己出的书吧?
海盗船长 7# 2011-10-5 15:24
6# 图图 哦,我们用的书是南大编的《高等数学》,但是讲得很粗略,所以自己借了一本数学分析看。
thread-450-1-1.html: [转]如何在LaTeX中制作动画
kuing 1# 2012-4-23 17:57
http://vanabel.sinaapp.com/?p=1632
thread-451-1-1.html: 这些字,你都写对了吗?
kuing 1# 2012-4-25 10:25
话说平时用手机打字时用“笔画”输入的时候就是要按上述正确的笔顺才行。
图图 2# 2012-4-29 10:11
都对的
叶剑飞Victor 3# 2012-8-8 20:22
全写错了。
叶剑飞Victor 4# 2013-5-1 19:14
五笔输入法打“方”字是y(方)y(丶)g(一)n(乙),说明最后一笔是折
isea 5# 2013-5-1 19:51
全对,nokia 时,笔划
kuing 6# 2013-5-1 19:58
4# 叶剑飞Victor 的确如此
isea 7# 2013-5-1 21:04
没细看,但是方法的方,写时怎么样都是最后折
李斌斌755 8# 2013-5-1 23:59
我都对,因为有另一爱好。
isea 9# 2013-5-2 00:49
书法?
李斌斌755 10# 2013-5-2 01:28
9# isea
thread-452-1-1.html: [转]后缀名为XXX.zip.001分卷压缩文件解压方法
kuing 1# 2012-4-25 19:36
下了几个分卷压缩的文件,后缀名为“*.zip.001”、“*.zip.002”、“*.zip.003”、“*.zip.004”,用WINRAR搞不定,装上WINZIP也搞不定,后来搜了一下,找到个办法(只在Windows下搞的): 在windows下“开始-运行-CMD”,在命令行界面进此文件目录下, 然后输入:copy /B *.zip.001+*.zip.002+*.zip.003+*.zip.004 *.zip 等屏幕上慢慢挨个文件名走一遍,出现“已复制 1个文件”后,就可以用WINRAR打开合并后的*.zip 了。 建议事先把文件名字“*”都统一改个简单点的,比如“1”,文件就直接复制在根目录下,省的命令行下输来输去的麻烦。
叶剑飞Victor 2# 2012-8-16 03:04
看来居然每个文件连文件头都没有。对原压缩文件直接二进制拆分。
kuing 3# 2012-8-16 03:14
楼上这么晚…… 其实我并不懂其中的原理
叶剑飞Victor 4# 2012-8-16 11:03
本帖最后由 叶剑飞Victor 于 2012-8-16 11:06 编辑 copy  /b  a+b+c   d 复制代码 这个命令的意思是,对a、b、c这三个文件进行二进制合并后,复制到d文件中。“/b”代表“二进制模式”(相对应的是“/a”,表示“文本模式”) 例如: a文件 10010100 b文件 10010111 c文件 0001010010010111 那么, copy  /b  a+b+c   d 复制代码 之后合并成的d文件就是10010100100101110001010010010111
kuing 5# 2012-8-16 12:39
4# 叶剑飞Victor oh 如此如此……
isea 6# 2013-3-1 23:10
全选,解压,图形界面下……
kuing 7# 2013-3-1 23:15
6# isea 印象中当时这样做是不行的,不然也不用搜到这个方法……
kuing 8# 2013-3-28 16:18
刚才何版主告诉我可以用 7zip 来解压它们
kuing 9# 2013-3-28 18:00
难道 6# 说的就是 7zip?不过好像也不用全选
isea 10# 2013-3-28 21:37
本帖最后由 isea 于 2013-3-28 21:38 编辑 zip 压缩文件 其实很好,至少跨平台相当方便, rar其实是很变态的,至少winrar是收费的,真正的商业软件,在国内这么流行,(我)真的很不解。 当然rar文件有也有自身的优点,并不贬低rar,偶也用winrar,虽用得极少。 基本所以有压缩软件很支持zip格式,zip.002 只是分卷形式, 解压时把所以文件放在同一目录,双击任意一个解压即可。 没什么大不了的。 winrar,winzip无法解压,三种可能,一是文件不完整,如,不是所有的分卷均在同一文件夹下; 二是压缩包不是由winrar,winzip生成,(如winmount,7zip等),或者相应的解压版本过低; 三是文件有包是损坏的。 最后7zip开源,免费,样子一般,但强大好用,我的确用的7z,只有极少由winrar生成的压缩包无法解压。
叶剑飞Victor 11# 2013-4-6 21:07
本帖最后由 叶剑飞Victor 于 2013-4-6 21:15 编辑 10# isea *.zip的格式压缩率实在太低了,没啥意义。我主要都是压缩成*.7z格式的。 zip 压缩文件 其实很好,至少跨平台相当方便 其实zip跨平台一点都不方便,7z跨平台才方便: 在Windows下压缩的汉字文件名文件成*.zip,在Linux下解压就乱码了。 在Linux下压缩的汉字文件名文件成*.zip,在Windows下解压也是乱码。 简体中文Windows文件名默认GBK编码,Linux默认UTF-8编码。 如果是压缩成*.7z格式,则永不会乱码。 *.zip格式不记录字符编码,*.7z记录字符编码。 所以*.zip只能按照目标平台默认的字符编码来压缩和解压,而*.7z可以按照文件中记录的字符编码来解压并转码。
thread-453-1-7.html: 绝对值不等式的证明方法有哪些?
yayaweha 1# 2012-4-29 14:20
下面附上一道题第三问望大家求解
yayaweha 2# 2012-4-30 10:28
怎么没人解答呀!!
yayaweha 3# 2012-4-30 13:26
3# kuing 第2问没撒用
yayaweha 4# 2012-4-30 13:27
求通向先证an在函数的定义域内
kuing 5# 2012-4-30 13:31
噢,我看错了,以为第二问是求 an 的通项……
yayaweha 6# 2012-4-30 13:35
第二问的结论对第三问没用
kuing 7# 2012-4-30 13:39
嗯,但是an可以求出来,你可以试试 不知求出an对后面的证明有没有用
yayaweha 8# 2012-4-30 13:40
an怎么求呀
yayaweha 9# 2012-4-30 13:43
我估计用高中知识通项求不出来,一拿到绝对值不等式就不知该怎么下手!
kuing 10# 2012-4-30 13:53
\begin{align*} 1-a_{n+1}=\frac{(1-a_n)^2}{a_n^2+1},\\ a_{n+1}+1=\frac{(a_n+1)^2}{a_n^2+1}, \end{align*} gives \[\frac{1-a_{n+1}}{a_{n+1}+1}=\left(\frac{1-a_n}{a_n+1}\right)^2\] gives \[\frac{1-a_n}{a_n+1} = \left(\frac{1-a_1}{a_1+1}\right)^{2^{n-1}}\] ...
yayaweha 11# 2012-4-30 15:49
11# kuing 1怎么来的? 不动点吗?
kuing 12# 2012-4-30 16:22
这个观察一下也看得出来了啊,分子跟分母不是刚好是完全平方的东西么…… PS。这类型的确属于特殊的不动点可解情形,一般的二次分式无解。
yayaweha 13# 2012-4-30 17:39
求第三问呀!
海盗船长 14# 2012-4-30 18:08
参考10年的联赛题
都市侠影 15# 2012-6-5 16:00
本帖最后由 都市侠影 于 2012-6-5 16:39 编辑 首先容易证明数列$\{a_{n}\}$满足:$\frac{1}{2}<a_{n}<a_{n+1}<1$ 其次由于 \[ a_{k}=kA_{k}-(k-1)A_{k-1} \] 所以 \begin{align} &|\sum_{i=1}^{n}a_{i}-\sum_{i=1}^{n}A_{i}| \\ =&|\sum_{i=1}^{n}(iA_{i}-(i-1)A_{i-1})-\sum_{i=1}^{n}A_{i}|  \\ =&|\sum_{i=1}^{n}(i-1)(A_{i}-A_{i-1})| \\ \end{align} 接下来对$A_{i}-A_{i-1}$作些估计: \begin{align} &A_{i}-A_{i-1} \\ =& \frac{\sum_{k=1}^{i}a_{k}}{i}-A_{i-1} \\ =&\frac{(i-1)A_{i-1}+a_{i}}{i}-A_{i-1} \\ =&\frac{a_{i}-A_{i-1}}{i} \end{align} 显然$\frac{1}{2}<a_{i}<1$且$\frac{1}{2}<A_{i-1}<1$,所以$0<a_{i}-A_{i-1}<\frac{1}{2}$,从而 \begin{align} 0<A_{i}-A_{i-1}<\frac{1}{2i} \end{align} 于是 \begin{align} &|\sum_{i=1}^{n}(i-1)(A_{i}-A_{i-1})| \\ <&\sum_{i=1}^{n}\frac{i-1}{2i} \\ =&\sum_{i=1}^{n}\Big(\frac{1}{2}-\frac{1}{2i} \Big) \\ =&\frac{1}{2}n-\sum_{i=1}^{n}\frac{1}{2i} \\ <&\frac{1}{2}n-\frac{1}{2} \\ \end{align} 至此证毕,从证明过程中可以看到,该数列的递推式过于强烈,实际只用到了数列单调增加且以$\frac{1}{2}$为下界以1为上界。
yayaweha 16# 2012-6-5 16:21
一般这种估计有什么用
都市侠影 17# 2012-6-5 16:24
写完了,你看看吧
都市侠影 18# 2012-6-5 16:26
累死我了
yayaweha 19# 2012-6-5 16:28
写了30分钟肯定累了
都市侠影 20# 2012-6-5 16:29
还证到了更加强烈的不等式 \begin{align} |\sum_{i=1}^{n}a_{i}-\sum_{i=1}^{n}A_{i}|<\frac{1}{2}n-\sum_{i=1}^{n}\frac{1}{2i} \end{align}
thread-453-2-7.html:
kuing 21# 2012-6-5 16:35
18# 都市侠影 呃,你平时不是写东西都用 latex 的么,写代码应该习惯了吧,怎么会累?
都市侠影 22# 2012-6-5 16:37
21# kuing 我这没有草稿纸,一边心里想着公式的形状一边写LaTeX代码,差点脑袋抽筋了
kuing 23# 2012-6-5 16:40
呃,我几年没用过草稿纸了,打草稿在mathtype上打。
thread-457-1-1.html: 曲线拟合
kuing 1# 2012-4-30 18:13
有下面一堆数据 data = {{1979, 0.15}, {1980, 1.2}, {1981, 3.3}, {1982, 9.5}, {1983,    19}, {1984, 30.61}, {1985, 40.29}, {1986, 42.11}, {1987,    49.84}, {1988, 60}, {1989, 76.78}, {1990, 99.13}, {1991,    153.54}, {1992, 187.8}, {1993, 248.28}, {1994, 318.74}, {1995,    349.99}, {1996, 379.51}, {1997, 418.29}, {1998, 465.73}, {1999,    512.71}, {2000, 576.32}, {2001, 592.53}, {2002, 607.17}, {2003,    627.34}, {2004, 635.67}, {2005, 645.82}, {2006, 674.27}, {2007,    699.99}, {2008, 726.21}, {2009, 753.56}, {2010, 786.17}} 需要用函数曲线 \[y(t) = \frac k{1 + a\cdot e^{-r\cdot t}}\] 来拟合。 但是直接利用 FindFit 的如下命令拟合出来却完全达不到想要的效果: data = {{1979, 0.15}, {1980, 1.2}, {1981, 3.3}, {1982, 9.5}, {1983,     19}, {1984, 30.61}, {1985, 40.29}, {1986, 42.11}, {1987,     49.84}, {1988, 60}, {1989, 76.78}, {1990, 99.13}, {1991,     153.54}, {1992, 187.8}, {1993, 248.28}, {1994, 318.74}, {1995,     349.99}, {1996, 379.51}, {1997, 418.29}, {1998, 465.73}, {1999,     512.71}, {2000, 576.32}, {2001, 592.53}, {2002, 607.17}, {2003,     627.34}, {2004, 635.67}, {2005, 645.82}, {2006, 674.27}, {2007,     699.99}, {2008, 726.21}, {2009, 753.56}, {2010, 786.17}}; model = k/(1 + a*E^(-r*t)) fit = FindFit[data, model, {k, a, r}, t] modelf = Function[{t}, Evaluate[model /. fit]] Plot[{modelf[t]}, {t, 1979, 2010}, Epilog -> Map[Point, data]] 得到的函数是 337.236/(1 + 1. E^(-1. t)) ,而作出的图在该部分几乎是直的。如图。 然而,我们通过将数据平移(将每个横坐标均减去1979)后,却能得到一个很好的拟合 datapy = {{0, 0.15}, {1, 1.2}, {2, 3.3}, {3, 9.5}, {4, 19}, {5,     30.61}, {6, 40.29}, {7, 42.11}, {8, 49.84}, {9, 60}, {10,     76.78}, {11, 99.13}, {12, 153.54}, {13, 187.8}, {14, 248.28}, {15,      318.74}, {16, 349.99}, {17, 379.51}, {18, 418.29}, {19,     465.73}, {20, 512.71}, {21, 576.32}, {22, 592.53}, {23,     607.17}, {24, 627.34}, {25, 635.67}, {26, 645.82}, {27,     674.27}, {28, 699.99}, {29, 726.21}, {30, 753.56}, {31, 786.17}}; model = k/(1 + a*E^(-r*t)) fit = FindFit[datapy, model, {k, a, r}, t] modelf = Function[{t}, Evaluate[model /. fit]] Plot[{modelf[t]}, {t, 0, 31}, Epilog -> Map[Point, datapy]] 得到的函数是 746.207/(1 + 91.4293 E^(-0.26602 t)) ,如图。 这样我们再平移回去,便得到原数据的拟合函数曲线 746.207/(1 + 91.4293 E^(-0.26602 (t - 1979))) ,作图如下。 Plot[746.207/(1 + 91.4293 E^(-0.26602 (t - 1979))), {t, 1979, 2010}, Epilog -> Map[Point, data]] 注意到函数能左右平移,这是因为 $a\cdot e^{-r\cdot (t+c)} = (a\cdot e^{-rc})e^{-r\cdot t}$,所以 746.207/(1 + 91.4293 E^(-0.26602 (t - 1979))) 其实也可以变回原来要求的形式。 于是当时我就奇怪了,既然该函数能左右平移,那为什么一开始不行,要移回来就行了呢? 后来在 http://reference.wolfram.com/legacy/v7/ref/FindFit.html 这里看到后面提到的“参数初始值”部分才知道原因所在。 给定不同的参数初始值,可能会得到不同的拟合曲线,上面的情况就是默认的初始值搜索不到那个很好的拟合。 但是初始值应该定在哪里,真不好说,我试过很多值都还是跟最开始差不多甚至出现了些不知什么意思的信息,直到利用上面的结果的近似值给出的初始值才勉强可以(还会弹出不知什么信息) data = {{1979, 0.15}, {1980, 1.2}, {1981, 3.3}, {1982, 9.5}, {1983,     19}, {1984, 30.61}, {1985, 40.29}, {1986, 42.11}, {1987,     49.84}, {1988, 60}, {1989, 76.78}, {1990, 99.13}, {1991,     153.54}, {1992, 187.8}, {1993, 248.28}, {1994, 318.74}, {1995,     349.99}, {1996, 379.51}, {1997, 418.29}, {1998, 465.73}, {1999,     512.71}, {2000, 576.32}, {2001, 592.53}, {2002, 607.17}, {2003,     627.34}, {2004, 635.67}, {2005, 645.82}, {2006, 674.27}, {2007,     699.99}, {2008, 726.21}, {2009, 753.56}, {2010, 786.17}}; model = k/(1 + a*E^(-r*t)); fit = FindFit[data,   model, {{k, 746.2}, {a, 91.4293*E^(0.266*1979)}, {r, 0.266}}, t] modelf = Function[{t}, Evaluate[model /. fit]] Plot[{modelf[t]}, {t, 1979, 2010}, Epilog -> Map[Point, data]] 所以还是平移好了……
kuing 2# 2012-4-30 18:37
最后顺便计算一下 746.207/(1 + 91.4293 E^(-0.26602 (t - 1979))) 这个结果与原数据的标准差。 asdf = Table[{t, 746.207/(1 + 91.4293 E^(-0.26602 (t - 1979)))}, {t,    1979, 2010}] Sqrt[Total[(asdf - data)^2][[2]]/(2010 - 1979 + 1)] 得到标准差为20.4331
thread-458-1-1.html: 用 chrome 看《数学空间》新一期的网页会有点乱
kuing 1# 2012-5-2 13:50
……用 IE 看就比较正常
6yan2 2# 2012-5-2 19:48
chrome版世界之窗,开完网页过一会才显示
海盗船长 3# 2012-5-2 20:00
找了半天,,怎么没有宣传贴额
海盗船长 4# 2012-5-2 20:08
额,现在文章都好短。。
kuing 5# 2012-5-2 21:36
没动力了
thread-46-1-9.html: 判断三角形形状$(a^2-b^2)/(a^2+b^2)=\sin(A-B)/\sin(A+B)$
kuing 1# 2011-10-4 12:35
在 $\triangle ABC$ 中,若\[ \frac{a^2 - b^2}{a^2 + b^2} = \frac{\sin (A - B)}{\sin (A + B)} \]判断三角形形状。 问题转自某群。 化边化角均可。 大家随便玩玩。
图图 2# 2011-10-4 14:13
呃,直角或等腰,化边做的,感觉化角好麻烦... 过程就不要了吧..
kuing 3# 2011-10-4 14:15
化角用网刊2中提到的“平方差公式”也挺简单
kuing 4# 2011-10-4 15:52
\begin{align*} \frac{a^2 - b^2}{a^2 + b^2} = \frac{\sin (A - B)}{\sin (A + B)}&\iff\frac{\sin^2A - \sin^2B}{\sin^2A + \sin^2B} = \frac{\sin (A - B)}{\sin (A + B)} \\ &\iff \frac{\sin (A - B)\sin (A + B)}{\sin^2A + \sin^2B} = \frac{\sin (A - B)}{\sin (A + B)} \\ &\iff \sin^2A + \sin^2B = \sin^2(A + B)  或  \sin (A - B) = 0 \\ &\iff \sin^2B = \sin^2(A + B) - \sin^2A  或  A = B \\ &\iff \sin^2B = \sin B\sin (A + B + A)  或  A = B \\ &\iff \sin B = \sin (2A + B)  或  A = B \\ &\iff B + 2A + B = \pi  或  A = B \\ &\iff A + B = \frac\pi2  或  A = B \end{align*}
thread-460-1-1.html: amsmath的\dots省略号自动判断
kuing 1# 2012-5-4 16:19
在使用了 amsmath 宏包的前提下,输入 \$x_1+\dots+x_n\$ and \$(x_1,\dots,x_n)\$. 结果: $x_1+\dots+x_n$ and $(x_1,\dots,x_n)$. 汗,我也是今天才知道原来还有这个东东……
thread-461-1-7.html: [组合] 求解一道概率题
jthuang1984 1# 2012-5-4 16:21
在一次智力竞赛中,一共为甲乙丙3名选手准备5道抢答题,在每一个题目的抢答中,每个选手抢到的概率是相等的,问:甲选手抢到的题目多于乙选手而不多于丙选手的概率?
kuing 2# 2012-5-20 17:52
今天进后台才发现这个贴被TX删掉了,现在再反删除。 注意操作者,实在搞不懂触动了什么mingan词语要delete掉
thread-462-1-1.html: \bordermatrix
kuing 1# 2012-5-4 18:02
\[\bordermatrix{ & 0 & 1 & 2\cr 0 & A & B & C\cr 1 & d & e & f\cr 2 & 1 & 2 & 3}\] \bordermatrix{ & 0 & 1 & 2\cr 0 & A & B & C\cr 1 & d & e & f\cr 2 & 1 & 2 & 3} MathJax 还不支持…… 在真 LaTeX 里测试成功 尚未清楚怎么用的……
kuing 2# 2012-5-4 18:30
搜索了一下,功能不怎么样……没什么好玩 玩矩阵输入还是用 blkarray 强
thread-466-1-7.html: 觉得象第一次解这类方程
realnumber 1# 2012-5-12 07:54

realnumber 2# 2012-5-13 22:18
thread-468-1-1.html: 可以用QQ号登录了
kuing 1# 2012-5-17 15:05
可以用QQ号登录了 与帐号绑定
thread-469-1-1.html: @是什么东东?
kuing 1# 2012-5-18 17:40
看帮助时看到一个例子: 第二命令没看懂
kuing 2# 2012-5-19 00:04
总算找到了 跟 Apply 有关,搜这个关键词就能找到相关说明
kuing 3# 2012-5-19 00:06
/@ 和 @@ 是 Map 和 Apply 的简写,@@@ 是 Apply[XXX, YYY, {1}] 的简写,作用于第一层
thread-47-1-3.html: [不等式] 不等式马拉松(inequalites marathon)
pxchg1200 1# 2011-10-4 13:09
Hi,我觉得来个不等式马拉松会很好玩。游戏规则如下: 如果您能解出前面的人出的题,那么您将赢得下一题的出题权,您可以出任何初等不等式的题目。以此往下。 我先开个头,来个简单的: problem 1 设$ x,y,z>0 ,x+y+z=1 $,证明: \[ \frac{1}{\sqrt{x+y}}+ \frac{1}{\sqrt{y+z}}+ \frac{1}{\sqrt{z+x}}\leq \frac{1}{\sqrt{2xyz}} \] Have fun!
kuing 2# 2011-10-4 13:49
:o,只能先顶一下。。。有空再接力:D
pxchg1200 3# 2011-10-4 15:03
kuing 怎么不秒杀来着?
kuing 4# 2011-10-4 15:04
3# pxchg1200 因为我没题供下去……
wenshengli 5# 2011-10-4 16:16
本帖最后由 wenshengli 于 2011-10-4 16:18 编辑 1# pxchg1200 均值定理就可以 过程免了吧 problem2: 已知 $a,b\in R^+$,且$\dfrac1a+\dfrac1b=1$,证明 $\forall n\in N$ $$(a+b)^n-a^n-b^n\ge 2^{2n}-2^{n+1}.$$
图图 6# 2011-10-4 16:49
4# kuing 没题?那一堆呢?
kuing 7# 2011-10-4 17:04
6# 图图 不是自己的不算。。。
图图 8# 2011-10-4 17:07
7# kuing ...那里的题几乎都是你给我的啊
kuing 9# 2011-10-4 17:10
8# 图图 问题是不是我提出的,基本上都是看论坛看到的或者由某些问题遇到的需要解决的问题。。。 PS,版聊了。。。 留位置给大家做题吧
yes94 10# 2013-3-31 14:21
5# wenshengli 均值定理也可以搞定
kuing 11# 2013-3-31 17:57
几乎忘了还有这个贴的存在……
reny 12# 2013-4-1 23:26
本帖最后由 reny 于 2013-4-2 14:50 编辑 1# pxchg1200 我贴一个: 在$\triangle ABC$中,证明:$$\pi (\frac1 A+\frac1 B+\frac1 C)\geqslant (\sin\frac A 2+\sin \frac B 2+\sin \frac C 2) (\frac1{\sin\frac A 2}+\frac1{\sin  \frac B 2}+\frac1{\sin\frac C 2})$$
kuing 13# 2013-4-2 11:00
12# reny Δ -> \triangle sin -> \sin
yes94 14# 2013-4-2 12:22
1# pxchg1200 我贴一个: 在$△ABC$中,证明:$$\pi (\frac1 A+\frac1 B+\frac1 C)\geqslant (sin\frac A 2+sin \frac B 2+sin \frac C 2) (\frac1{sin\frac A 2}+\frac1{sin  \frac B 2}+\frac1{sin\frac C 2}) ... reny 发表于 2013-4-1 23:26 在$\triangle {ABC}$中,证明:\[\pi (\frac1 A+\frac1 B+\frac1 C)\geqslant (\sin\frac A2+\sin\frac B2+\sin\frac C 2) (\frac1{\sin\frac A2}+\frac1{\sin\frac B2}+\frac1{\sin\frac C2})\] 灌一下水,
pxchg1200 15# 2013-4-4 13:40
14# yes94 西哥的答案。
yes94 16# 2013-4-4 14:02
15# pxchg1200 很久没看到西西的活动了!
yes94 17# 2013-4-4 14:11
本帖最后由 yes94 于 2013-4-4 14:26 编辑 也就是说,当$x>a>0,y>b>0,z>c>0$时,是否成立如下不等式? \[(x+y+z)(\dfrac1x+\dfrac1y+\dfrac1z)\geqslant(a+b+c)(\dfrac1a+\dfrac1b+\dfrac1c)\]
kuing 18# 2013-4-4 14:27
17# yes94 $a\to0$
thread-470-1-1.html: [转]矩阵、随机化与分形图形
kuing 1# 2012-5-18 23:26
http://www.51xuewen.com/group/513/topic_18097.htm 里头的代码是 list = {{0, 0}}; last = {{0}, {0}}; For[i = 0, i < 50000, i++, r = Random[];    If[r < 0.85, last = {{0.83, 0.03}, {-0.03, 0.86}}.last + {{0}, {1.5}},      If[r < 0.91, last = {{0.2, -0.25}, {0.21, 0.23}}.last + {{0}, {1.5}},        If[r < 0.99, last = {{-0.15, 0.27}, {0.25, 0.26}}.last + {{0}, {0.45}},          last = {{0, 0}, {0, 0.17}}.last + {{0}, {0}}        ]      ]    ];    list = Append[list, First[Transpose[last]]]; ] ListPlot[list, PlotStyle -> PointSize[0.002]] 复制代码 为防网页丢丢,存档到附件。
thread-471-1-7.html: [几何] 不等式群看到的一道三角题,看上去似乎容易但玩起来还真……
kuing 1# 2012-5-19 23:39
RT,在不等式群看到的一道三角题,看上去似乎容易但玩起来还真不太好解决。 废话少说,先贴题: 辽宁沈阳李明(8753*****)  17:44:55 甘志国老师给我发了一个邮件,要我帮忙求解一个问题!谁能帮算一下?问题如下 请教您一个问题:已知外接圆半径为6的三角形有两边之和为16,求该三角形面积的最大值? 先是有四川蒋淋富用爪机回复如下: 四川蒋淋富(5066*****)  18:55:26 理论上可用xyz=4RS,于是可以设三边分别为x,y,z,x+y=16,R=6,以及面积公式16S^2=x^2y^2+y^2z^2+z^2x^2-x^4-y^4-z^4不难求出z,进而得到面积S=xyz/(4R)得到关于xy的涵数…后面就是求导的事情了 我还没照着试算,具体麻烦程度要看后面得到的那函数到底有多复杂了,有空试试。 后来浙江朱世杰也贴了一个解法,应该是挺正宗的,但我完全没心思看下去。
kuing 2# 2012-5-20 00:01
而我目前的解法并没有简单到哪去。 解  不妨设$a+b=16$,则由正弦定理及$R=6$得 \[\sin A+\sin B=\frac43,\] 那么 \begin{align*} S&=2R^2\sin A\sin B\sin C \\ & =72\sin A\sin B(\sin A\cos B+\cos A\sin B) \\ & \leqslant 72\sin A\sin B\bigl(\sin A\sqrt{1-\sin^2B}+\sqrt{1-\sin^2A}\sin B\bigr), \end{align*} 为方便书写,下面记$x=\sin A$, $y=\sin B$,那么上面得到的结果即为$x$, $y\in (0,1]$, $x+y=4/3$, 且 \[S\leqslant 72xy\bigl(x\sqrt{1-y^2}+y\sqrt{1-x^2}\bigr),\] 到这里想来想去没什么好办法,暂时用切线法先,我们将证明在上述条件下有 \[xy^2\sqrt{1-x^2}\leqslant \frac{4 (28-27 x)}{81 \sqrt5},\] 这里我也没心思想巧妙的证明,消$y$再求导应该也可以,但计算量应该也少不到哪去,故此我还是直接平方作差分解算了,由条件知上式等价于 \[x^2\left(\frac43-x\right)^4(1-x^2)\leqslant \frac{16 (28-27 x)^2}{32805},\] 作差分解等价于 \[\frac{(3 x-2)^2 \Bigl(\bigl(3645 x^4+(7290x^2+11340x+1890) (1-x) x^2+1080\bigr) (x-1)^2+150 (1-x)+16\Bigr)}{32805}\geqslant0,\] 显然成立,故 \[S\leqslant 72xy\bigl(x\sqrt{1-y^2}+y\sqrt{1-x^2}\bigr)\leqslant 72\left(\frac{4 (28-27 x)}{81 \sqrt5}+\frac{4 (28-27 y)}{81 \sqrt5}\right)=\frac{128 \sqrt5}9,\] 当$A=B=\arcsin(2/3)$时$S=128 \sqrt5/9$。 与朱的结果有所不同? 但朱的最后取等条件好像跟我的是一样的,可能是他计算错了。
realnumber 3# 2012-5-20 00:14
恩,/9
kuing 4# 2012-5-20 13:47
\[x\left(\frac43-x\right)^2\sqrt{1-x^2}\] 这个在(0,1]上先上凸后下凸再上凸……没办法直接Jensen,除非分类,但还是比较麻烦
realnumber 5# 2012-5-20 17:33
修改了个错误
kuing 6# 2012-5-20 17:41
oh 我还是没心思细看,呵呵 PS、链接我就懒得发到群里了,因为现在论坛被设置成禁止游客浏览,而多数人看到要注册的时候直接就走人,尽管注册也就十来秒。
kuing 7# 2012-5-20 17:42
咦,刚才再试了一下游客状态,好像解除了…… 那就好了
kuing 8# 2012-5-20 18:45
还是软件牛,我拿朱得到的关于C的函数用软件求导化简一下,本来只是想验证下结果是不是一样,没想到一化简时竟然得到下面这个如此简单的结果 \[S_C' = -9 \cos C+9 \cos2C+8 \sec^2\frac C2\] 再分解一下,亦可得 \[S_C' = \frac14(-9 \cos C+9 \cos2C+9 \cos3C+23) \sec^2\frac C2\] 不知它是怎么搞出来的,反正我一时也没看出来。 这样,照朱的思路,要证$S_C' > 0$,就只要证$-9 \cos C+9 \cos2C+9 \cos3C+23 > 0$,这时我就要人工一下了,利用二、三倍角公式,记$t=\cos C$,则 \[-9 \cos C+9 \cos2C+9 \cos3C+23 = 4 t (3 t-1)^2 + 10 (2 t - 1)^2 + 2 t^2 + 4>4>0.\\ (这里错了,见10楼,修正版见11楼)\] 嘿嘿
kuing 9# 2012-5-20 22:11
原来用一下半角公式就能很简单,注意到 \[\frac{16-9\sin^2C}{1+\cos C}\sin C = (16-9\sin^2C)\tan\frac C2 = \frac12(23+9\cos 2C)\tan\frac C2,\] 故求导便是 \[\frac{23+9\cos 2C}{4\cos^2\frac C2}-9\sin 2C\tan\frac C2,\] 通分为 \[\frac{23+9\cos 2C-18\sin 2C\sin C}{4\cos^2\frac C2},\] 此时如果分子用一下积化和差公式,就能得到上楼分解后的式子了。当然了,到这里其实用上楼的思路的话也不必积化和差了,分子直接化为 \[23+9(2\cos^2C-1)-36(1-\cos^2C)\cos C,\] 然后就可以用上楼的配方了,这样还省回了用三倍角公式。 可见先化简再求导真能省啊
realnumber 10# 2012-5-20 22:36
这样配方还不能说明》4成立,因为t=cosC可能为负,C可能是钝角
kuing 11# 2012-5-20 22:40
10# realnumber 噢对,那我再配一下吧 \[-9 \cos C+9 \cos2C+9 \cos3C+23 = 4 (t + 1) (3 t-1)^2 + 2 (1 - t) (5 - 3 t)>0\] 其中$t=\cos C\in (-1,1)$
realnumber 12# 2012-5-20 22:40
确实这样做是简单多了,我那个凑成,总觉得运气好
realnumber 13# 2012-5-20 22:43
就这么熟练了,kk水平传授下提高水平的经验,
kuing 14# 2012-5-20 22:50
我想求$C$最大估计也可以简单些,因为我二楼得到了$\sin A+\sin B=4/3$,用这个求$C$最大应该是easy的,但三角变换我不是很熟,你试试吧。 我的想法至少可以用反三角做和琴生做,由于如果$A$是钝角,那么将其变为 $\pi-A$ 时也满足上式而$C$变大,所以要求$C$的最大值只要考虑当$A$, $B$都不是钝角的情形。 此时有$A=\arcsin(\sin A)$, $B=\arcsin(\sin B)$,而$\arcsin x$在$x\in(0,1)$为下凸函数,所以 \[C=\pi-A-B=\pi-\arcsin(\sin A)-\arcsin(\sin B)\leqslant \pi-2\arcsin\left(\frac{\sin A+\sin B}2\right)=\pi-2\arcsin\frac23.\] 结果一样。
kuing 15# 2012-5-20 22:53
和差化积是不是可以? \[\frac43=\sin A+\sin B=2\sin\frac{A+B}2\cos\frac{A-B}2\leqslant 2\cos\frac C2 \implies C\leqslant 2\arccos\frac23,\] 汗,看来我转晕了……
kuing 16# 2012-5-20 22:59
13# realnumber 没有了,如果熟练的话,也不会刚才才发现先用半角公式化简,也不会像楼上上那样竟然忘了用和差化积……
kuing 17# 2012-5-21 14:22
一般情况:外接圆半径$R$,两边$a+b=L<4R$,其中$L$, $R$为定值,则面积的最大值如何?
kuing 18# 2012-5-22 16:24
已经得到的结果是当 \[L\geqslant \sqrt{\frac{70}{27}+\frac{26\sqrt{13}}{27}}\cdot R\] 时$S_C'\geqslant0$,其中$\sqrt{\frac{70}{27}+\frac{26\sqrt{13}}{27}}\approx 2.46264186$,一楼的题目中$L/R=16/6=2.666...$大于此值,所以$S_C'>0$
kuing 19# 2012-5-22 16:31
顺便说下$C$的取值范围 当$L<2R$,则 \[C\in \left( 0,\arcsin \frac{L}{2R} \right)\cup \left( \pi -\arcsin \frac{L}{2R},2\arccos \frac{L}{4R} \right];\] 当$4R>L\geqslant 2R$,则 \[C\in \left( 0,2\arccos \frac{L}{4R} \right].\]
realnumber 20# 2012-5-24 14:34
本帖最后由 realnumber 于 2012-5-24 15:20 编辑
thread-471-2-7.html:
realnumber 21# 2012-6-10 19:07
本帖最后由 realnumber 于 2012-6-10 19:12 编辑
kuing 22# 2012-6-10 22:53
有空再慢慢研究
realnumber 23# 2012-6-17 21:31
其实推广2整体思路很简单, 1.用调整法,n边形中分割出4边形, 2.如此需要固定一边,2边和定值,还有一边长度因为内接圆,长度受三条边决定。 3.为达到步骤2,需要引理1,2
thread-472-1-7.html: [不等式] 不等式群里看到的题,群里MS没人鸟
kuing 1# 2012-5-20 14:49

kuing 2# 2012-5-20 15:32
这是昨天的东西,昨天我白天在打牌,晚上看到题的时候我又在搞其他东西,后来就只跟提问人大致说了下做法,现详细写一下如下。 首先如果$a$, $b$, $c$中有两个相等,那么不等式显然对任意实数$k$都成立,下设$a$, $b$, $c$都不相等,由对称性不妨设$a>b>c\geqslant0$,并令$b=c+t$, $a=c+u$, $u>t>0$,则原不等式等价于 \[(3c+t+u)^6\geqslant k\bigl((c+u)^3+(c+t)^3+c^3\bigr)tu(u-t),\] 故要求$k$的最大值,就只要求 \[f(t,u,c)=\frac{(3c+t+u)^6}{\bigl((c+u)^3+(c+t)^3+c^3\bigr)tu(u-t)}\] 的最小值。 下面证明当$t$, $u$固定时$f(t,u,c)$关于$c$是严格递增的。对$c$求导有 \[\frac{\partial f(t,u,c)}{\partial c}=\frac{18(3c+t+u)^5\bigl((c+u)^3+(c+t)^3+c^3\bigr)-3(3c+t+u)^6\bigl((c+u)^2+(c+t)^2+c^2\bigr)}{\bigl((c+u)^3+(c+t)^3+c^3\bigr)^2tu(u-t)},\] 故 \[\frac{\partial f(t,u,c)}{\partial c}> 0 \iff 6\bigl((c+u)^3+(c+t)^3+c^3\bigr)\geqslant (3c+t+u)\bigl((c+u)^2+(c+t)^2+c^2\bigr),\] 事实上由切比雪夫不等式有 \begin{align*} 3\bigl((c+u)^3+(c+t)^3+c^3\bigr)&\geqslant \bigl((c+u)+(c+t)+c\bigr)((c+u)^2+(c+t)^2+c^2)\\ &=(3c+t+u)\bigl((c+u)^2+(c+t)^2+c^2\bigr), \end{align*} 所以显然有$\partial f(t,u,c)/\partial c> 0$成立。 由此有 \[f(t,u,c)\geqslant f(t,u,0),\] 再令$u=xt$, $x>1$,则$f(t,u,0)$可以写成 \[f(t,u,0)=\frac{(x+1)^6}{(x^3+1)x(x-1)}=\frac{(x+1)^5}{x(x-1)(x^2-x+1)}=g(x),\] 那么只要求当$x>1$时$g(x)$的最小值即为$k$的最大值。 显然$g(x)$在$(1,+\infty)$内可导,并且不难看出 \[\lim_{x\to1^+}g(x)=\lim_{x\to+\infty}g(x)=+\infty,\] 所以$g(x)$存在最小值且取最小值处必有$g'(x)=0$,我们设其在$x=x_0$处取得最小值为$m$,那么必有 \[\left\{\begin{aligned} g(x_0)&=m,\\ g'(x_0)&=0, \end{aligned}\right.\] 求导后代入知上述方程组等价于 \[\left\{\begin{aligned} (x_0+1)^5-mx_0(x_0-1)(x_0^2-x_0+1)&=0,\\ x_0^4-8x_0^3+12x_0^2-8x_0+1&=0, \end{aligned}\right.\] 消去$x_0$,得到 \[m^4+3816m^2-1350000=0,\] 由于$m$显然是正的,由上式解得唯一正根 \[m=2\sqrt{ - 477 + 228 \sqrt6 }\approx 18.05366,\] 这就是所求的$k$的最大值。
天涯无际 3# 2012-5-20 19:29
这种类型,上来基本上就可以知道一数为$0$时取得最值......就是算得有些繁琐~~~
kuing 4# 2012-5-20 21:25
3# 天涯无际 基本上是吧,这次这个算是相对复杂点点了,以前见过很多作完增量之后马上就得到c递增,这里还要求导证一下。 不过印象中以前也有见过不是单调的,记不起来具体了。
pxchg1200 5# 2012-5-24 13:06
本帖最后由 pxchg1200 于 2012-5-24 13:17 编辑 4# kuing 那个取等条件太猥琐了。。。。   直接AM-GM还真不好弄。 貌似是$ a=0, b=-1+\sqrt{2\sqrt{6}-3},c=1+\sqrt{2\sqrt{6}-3} $...
pxchg1200 6# 2012-5-24 13:15
Potla同学貌似还给出了以下几个题。 1,Find the max value of $k$,Where $a,b,c\geq 0 $ \[ (a^3+b^3+c^3)^3\geq k(a+b+c)(ab+bc+ca)|(a-b)(b-c)(c-a)| \] here $k=\sqrt{\frac{1}{2}(35+13\sqrt{3})}$ 2.For $a,b,c\geq 0$ find the best $k$: \[ (a^3+b^3+c^3)^2\geq k|(a^2-b^2)(b^2-c^2)(c^2-a^2)| \]
kuing 7# 2012-5-24 14:01
6# pxchg1200 用相同的办法试下能不能凑效?
thread-473-1-1.html: 突然发现网址有点变化
kuing 1# 2012-5-20 17:50
由 .com 变成了 .net 不过如果用旧地址好像也自动跳转到新的上 详情见:http://www.5d6d.net/thread-929495-1-1.html
kuing 2# 2012-5-23 00:44
看了那贴,除了无语,还是无语……
thread-475-1-7.html: [不等式] 请教一个不等式证明,先谢谢了!
hongxian 1# 2012-5-23 09:43
本帖最后由 hongxian 于 2012-5-23 09:45 编辑 已知:$n\in N^*$,且$n\ge 2$,求证:$\frac{1}{\sqrt{n(n+1)}}<\sqrt{n}-\sqrt{n-1}$
kuing 2# 2012-5-23 11:09
very easy 人教论坛上有过程了,我就不写了。
realnumber 3# 2012-6-10 19:18
分析法来写的话,两边倒数,右边分母有理化,两边平方
thread-479-1-7.html: [不等式] 请教一个不等式求最值的题,先谢谢了!
hongxian 1# 2012-5-24 10:29
已知:$a$,$d$为非负实数,$b$,$c$,为正实数, 且$b+c\ge a+d$,求$\frac{b}{c+d}+\frac{c}{a+b}$的最小值;
kuing 2# 2012-5-24 11:12
http://bbs.pep.com.cn/thread-1751315-1-1.html
thread-48-1-1.html: [物理]求电阻
kuing 1# 2011-10-4 13:56
问题来自某网友在QQ上问 求两端总电阻
kuing 2# 2011-10-4 13:58
我用基尔霍夫的那两个定律算得是 51/19,不知有没有算错,物理不太在行了 大家试试
thread-480-1-7.html: [不等式] 看起来很老的题
pxchg1200 1# 2012-5-24 13:23
For $ a,b,c\geq 0$ prove that: \[ \frac{a^4}{a^3+b^3}+\frac{b^4}{b^3+c^3}+\frac{c^4}{c^3+a^3}\geq \frac{a+b+c}{2} \] looking for a CS proof! :D Oh,顺便说下。 那天由于计算错误,我居然误以为这个比下面的不等式强。 \[ \frac{3a^{4}+a^{2}b^{2}}{a^{3}+b^{3}}+\frac{3b^{4}+b^{2}c^{2}}{b^{3}+c^{3}}+\frac{3c^{4}+c^{2}a^{2}}{c^{3}+a^{3}}\ge2(a+b+c) \] 后来发现这两个不等式都不能说明哪个比哪个强。。。 估计是我看错成 \[ \frac{3a^{4}+b^{4}}{a^{3}+b^{3}}+\frac{3b^{4}+c^{4}}{b^{3}+c^{3}}+\frac{3c^{4}+a^{4}}{c^{3}+a^{3}}\ge2(a+b+c) \] 了。。。
kuing 2# 2012-5-24 13:59
第二个比第一个强
pxchg1200 3# 2012-5-24 17:51
2# kuing
kuing 4# 2012-5-24 22:30
目测的,不知是不是这样。 之前研究过一下第二个,没结果
thread-481-1-1.html: 求级数和
pxchg1200 1# 2012-5-24 13:25
For $ x\in(0,\pi) $ \[ S=\sum_{k=1}^{+\infty}{\frac{\sin{kx}}{k}} \] :)
海盗船长 2# 2012-6-23 17:12
用傅立叶级数容易得到\[S=\frac{\pi-x}{2}\]
海盗船长 3# 2012-7-28 22:27
For $x\in (0,\pi)$\[S=\sum_{k=1}^{n}{\frac{\sin{kx}}{k}}< 2\sqrt{\pi}\]
thread-482-1-7.html: 16点问题---推广后呢?
realnumber 1# 2012-5-24 17:17
本帖最后由 realnumber 于 2012-5-24 17:18 编辑 下面是网友:杏坛孔门的解答
realnumber 2# 2012-5-24 17:23
下确界似乎很不好找,几乎是极难级别的. 如此先解决简单的几个问题:1.平面内7个点,任意两点距离小于等于1,若总是存在两点距离小于等于d,求d取值范围. 猜测d≥0.5(d=0.5时,7个点排成正六边形以及一个中心),如果是,怎么证明? 2.:平面内n个点,任意两点距离小于等于1,若总是存在两点距离小于等于d(n),求d(n)取值范围. 那么有d(n)_min≥d(n+1)_min 猜想d(7)≥1/2,d(19)≥1/4,d(37)≥1/6......, d(7),d(19),d(37),..总不能说显然成立 如果数量增多,d(7)问题就变为d(19)问题, -------------------------一个箱子里堆放同一种圆柱形零件似乎又联系.
thread-488-1-7.html: [不等式] 还请教一个不等式证明,先谢谢了!
hongxian 1# 2012-5-25 09:31
本帖最后由 hongxian 于 2012-5-25 14:03 编辑 求证:\((1+1\times 2)\cdot (1+2\times 3)\cdot \cdots \cdot [1+n(n+1)]>{{e}^{2n-3}}\)
kuing 2# 2012-5-25 15:22
太弱了……
realnumber 3# 2012-5-26 12:29
确实,前2项积大于$e^2$,第3项开始每个都大于$e^2$
thread-49-1-1.html: 几个希腊字母变种
kuing 1# 2011-10-4 14:50
epsilon:$\epsilon$ varepsilon:$\varepsilon$ phi:$\phi$ varphi:$\varphi$ pi:$\pi$ varpi:$\varpi$ theta:$\theta$ vartheta:$\vartheta$ sigma:$\sigma$ varsigma:$\varsigma$ 不知还有没有,待完善
kuing 2# 2012-2-3 17:30
偶然在这里 http://blog.sina.com.cn/s/blog_5e16f1770100lxq5.html 还看到了大写的也有 var 还有斜体什么的
海盗船长 3# 2012-2-3 20:38
$\Omicron$   $\omicron$ $O$  $o$
thread-490-1-7.html: [函数] 求解一道函数导数题
jthuang1984 1# 2012-5-25 10:12
已知函数$f(x)=4x^3+3tx^2−6t^2x+t−1$,$x∈R$,$t∈R$, 证明:$∀t∈(0,1)$,$f(x)$在区间$(0,1)$内均有零点。
jthuang1984 2# 2012-5-25 10:19
已知函数$f(x)=4x^3+3tx^2−6t^2x+t−1$,$x∈R$, 证明:$∀t∈(0,+oo)$,$f(x)$在区间$(0,1)&内均有零点。 这才是正确的原题,求高手解答,随带问一下,正无穷的代码是什么?
realnumber 3# 2012-5-25 13:47
计算f(0),f(1),f(0.5)就可以,就零点存在定理, 0<t≤1,f(0)<0,f(1)>0 1<t≤2,f(0.5)<0,f(1)>0 t>2,f(0)>0,f(1)<0 所以,……应该有更简洁的分类,但不去努力找了.
kuing 4# 2012-5-25 14:08
已知函数$f(x)=4x^3+3tx^2−6t^2x+t−1$,$x∈R$, 证明:$∀t∈(0,+oo)$,$f(x)$在区间$(0,1)&内均有零点。 这才是正确的原题,求高手解答,随带问一下,正无穷的代码是什么? jthuang1984 发表于 2012-5-25 10:19 属于用\in   正无穷用+\infty   这些代码其实都能在置顶帖找到
thread-492-1-7.html: [不等式] 请教高考模拟题
yayaweha 1# 2012-5-26 19:47
如图 ______________kuing edit in $\LaTeX$______________ 已知函数 $f(x)=x-x\ln x$,$g(x)=f(x)-xf'(a)$,其中 $f'(a)$ 表示函数 $f(x)$ 在 $x=a$ 处的导数,$a$ 为正常数。 (1)求 $g(x)$ 的单调区间; (2)对任意的正实数 $x_1$, $x_2$ 且 $x_1<x_2$,证明 \[(x_2-x_1)f'(x_2)<f(x_2)-f(x_1)<(x_2-x_1)f'(x_1);\] (3)对任意的 $n\in\mathbf{N}^+$,且 $n\geqslant 2$,证明 \[\frac{1}{\ln2}+\frac{1}{\ln3}+\cdots+\frac{1}{\ln n}<\frac{1-f(n+1)}{\ln2\cdot\ln n}.\]
yayaweha 2# 2012-5-26 19:48
请教第三问,特别难。随便问一下谁能发到人教论坛上
yayaweha 3# 2012-6-2 20:39
怎么没人来呀 这个数列型不等式
kuing 4# 2012-6-2 22:07
我也没什么看法…… 原图片的字有点小,我用$\LaTeX$代码在你一楼把原题重打了一下,看得清楚些。 等数列不等式高手也玩吧……我对这类题现在打不起精神
yayaweha 5# 2012-6-5 12:21
难道真的无人问津吗?
都市侠影 6# 2012-6-5 14:21
请参考我在人教网的回复:http://bbs.pep.com.cn/thread-2521212-1-1.html
kuing 7# 2012-6-5 15:15
6# 都市侠影 其实你可以写在这里,因为既然你用latex写的,把相应代码粘过来直接发贴就行了,省了贴图
都市侠影 8# 2012-6-5 21:14
7# kuing 我是先在人教论坛里看到的,呵呵,早知道就不用这么麻烦帖图了,还看不清楚。
kuing 9# 2012-6-8 00:29
8# 都市侠影 现在知道怎样贴得清楚了吧
thread-494-1-1.html: 请教,DZ 如何安装 MathJax ?
freame 1# 2012-5-27 21:41
上传 MathJax ,然后,写配置;或者直接用  MathJax 官方的服务器,  再写配置。 请教,这个配置写在 DZ 后台的哪个文件的哪个位置? 谢谢
kuing 2# 2012-5-27 22:01
我这里只是简单将一段代码抄进<head>里 查看一下本论坛任意一页面的源代码就可以看到 不知你那边的情况怎样
freame 3# 2012-5-27 22:10
本帖最后由 freame 于 2012-5-27 22:27 编辑 非常感谢!   DZ 安装后,只有 source/archiver/common/  文件夹下,有一个 header.php,一个footer.php 依照我目前掌握的办法,是 把下面代码 <script type=“text/javascript”      src=“http://cdn.mathjax.org/mathjax/latest/MathJax.js?config=TeX-AMS-MML_HTMLorMML”>    </script>   复制代码 放进  header.php 中的 </head> 复制代码 之前,就可以生效的。但是,很奇怪,我在DZ 没有成功。 这个办法,对于 wordpress 是有效的。对 DZ也有类似的办法,差别应该仅仅在于写在哪个文件的哪个位置
kuing 4# 2012-5-27 22:15
我不太了解这些。
kuing 5# 2012-5-27 22:16
把这段 <script type="text/x-mathjax-config">   MathJax.Hub.Config({     tex2jax: {       inlineMath: [ ['$','$'], ["\\(","\\)"] ],       processEscapes: true       }   }); </script> 也加上没?
freame 6# 2012-5-27 22:19
这个不是必须的, 仅仅只是功能增强,比如两个 美元输入行内公式。 那么,麻烦你看看,你把代码插在哪个文件的哪个位置吧? 谢谢 我再试一试
kuing 7# 2012-5-27 22:26
我不懂那些。
freame 8# 2012-5-27 22:30
你不是说,你把代码写在了 head 中吗?麻烦你看看那个文件的详细名称,还有那段代码的下一行是什么? 谢谢
kuing 9# 2012-5-27 22:33
我们的情况是不同的 我做的就是如图所示
freame 10# 2012-5-27 22:50
本帖最后由 freame 于 2012-5-27 22:51 编辑 谢谢! 这样应该就可以 我来试一试 非常感谢
thread-495-1-7.html: [函数] 问一个老题,不会觉得太简单吧?
hongxian 1# 2012-5-28 10:41
1.已知:$a$,$b$,$c\in R$满足$\frac{a}{m+2}+\frac{b}{m+1}+\frac{c}{m}=0$,其中$m>0$,对于$f(x)=a{{x}^{2}}+bx+c$($a\ne 0$) 1)证明:$a\cdot f\left( \frac{m}{m+1} \right)<0$; 2)证明方程$f(x)=0$在(0,1)内有解;
realnumber 2# 2012-6-10 19:15
确实很老了1。就直接检验 2。利用零点存在定理(或2次方程根的分布问题)
thread-496-1-1.html: [求助]鼠标键无法使用
kuing 1# 2012-5-28 16:46
在 控制面板 的 辅助功能选项 里面,在 鼠标 那一栏上,有个叫 鼠标键 的东西,是用小键盘来控制鼠标的。 很早以期用的XP系统中没问题,按 左ALT+左SHIFT+Num Lock 就能出来。 由于换了新机,系统虽然也是XP,但按这个快捷键就没反应了,在控制面板的上述位置里勾选了 使用鼠标键 之后点击确定也没用,再重新打开时又回复到没勾状态。 google过也有人跟我有类似的问题(如 http://bbs.kafan.cn/thread-1011939-1-1.html、http://www.chenggong8.com/computer/12/computer7389.htm 等),但仍未看到有解决方法 求助中……
isea 2# 2012-6-2 17:33
本帖最后由 isea 于 2012-6-2 17:42 编辑 小键盘控制鼠标,哈哈,现在才知道有这功能呢 ============= win7 下,原来在这里:控制面板\所有控制面板项\轻松访问中心\使鼠标更易于使用 还真有..... 你用的多半是精简版的XP吧,所以......
kuing 3# 2012-6-2 17:58
2# isea 嗯,应该是精简的,而且估计是很精简,之前用xelatex时发现连隶书、幼圆这两个常用字体都没有,还有更早前发现那个控制面板的添加或删除组件功能也用不了,后来在网上下载了些文件才能用。 我就是想知道这次鼠标键不能用到底是缺了什么,如果能找到的话就可以补回去
isea 4# 2012-6-2 18:57
快换win7吧,新电脑不用win7浪费.
kuing 5# 2012-6-2 18:58
习惯用叉屁了……
kuing 6# 2013-4-1 21:02
顶一下,鼠标键有时还是挺好用的
thread-498-1-1.html: 函数f(x)=f(x-1)-f(x-2)画图
kuing 1# 2012-6-2 14:47
问题来自 http://bbs.pep.com.cn/thread-2513310-1-1.html 那里说要几何画板画,我不会,用Mathematica倒是容易,就不在那边发了,发在这里 f[x_] := 2^(1 - x) /; x <= 0 f[x_] := f[x - 1] - f[x - 2] /; x > 0 Plot[f[x], {x, -3, 14}] 得到 那些断开的线不知为何会自动连起来,为了去掉它们,注意到都是在非负整数处才出现,故加个选项Exclusions -> Range[0, 14],即 f[x_] := 2^(1 - x) /; x <= 0 f[x_] := f[x - 1] - f[x - 2] /; x > 0 Plot[f[x], {x, -3, 14}, Exclusions -> Range[0, 14]] 得到
kuing 2# 2012-6-2 15:31
至于那些线的端点处是空是实,就得自己去判断了。 软件用的都是采点作图,并不会自动判断奇点并作出虚或实的小圈,目前我玩过的软件都一样。 比如作 $\dfrac{e^x- 1}x$ 的图像,尽管显然 $x=0$时无定义,但由于$x\to0$时左右极限为1(也就是说只要补上$(0,1)$之后函数就完全连续了),所以用各种软件作出来的图像看上去都是连续的。 所以有时候用软件验证问题也得动动脑,不能只看表面。这里也有一个典型例子,就是判断$\dfrac{\tan x}{1-\tan^2x}$的最小正周期。 如果直接用倍角公式化成$\dfrac12\tan2x$那么会以为最小正周期为$\dfrac\pi2$,再用软件直接作$\dfrac{\tan x}{1-\tan^2x}$的图像来验证一下的话,看上去结果的确是$\dfrac\pi2$。 但如果留意定义域,你就会发现其实答案应该是 $\pi$,这是因为原来的函数限制了$x\ne\dfrac\pi2+k\pi$,所以其实在$\dfrac12\tan2x$的图像中挖去$x=\dfrac\pi2+k\pi$的这些点后才是$\dfrac{\tan x}{1-\tan^2x}$的图像。于是和上述类似,用软件作图并没显示出这一差别,所以即使用软件作图验证,还是错了。 完全跑题ing
isea 3# 2012-6-2 17:27
好久没来了,来了,就要支持一个.
chinawgp 4# 2012-6-2 17:43
2# kuing 不错, 哈哈! 好久没来, 帐号要重新激活。 呵呵
kuing 5# 2012-6-2 17:59
4# chinawgp 激活?还有这种事啊? 我也不知道
thread-499-1-1.html: 典型输入之导数和“小于”
kuing 1# 2012-6-2 23:17
这是刚才在人教群里看到的,两个典型的输入问题集中在一起了: 第一:导数。估计TA打导数的时候是先按了上标,然后再打一撇,结果就变成这么小。 实际上,如果是在mathtype输入,在板面上就有导数符号: (1到3阶导数都有)可以点击的。 而latex的输入则是更简单,直接 ' 就行了,无论哪种,都是不需要按上标来打的。 第二:“大于”或“小于”,这种错误更是常见,他们在mathtype的板面上找不到正常的大于号或小于号来点击,便点击了 $\succ$ 和 $\prec$ 当作是大于号和小于号来用,这实在是比较汗颜,难道习惯了用鼠标点击面板,反而就忘记了本来用键盘就能打出大于号和小于号了吗? 再者,符号 $\succ$ 和 $\prec$ 是有别的意思的,在控制里论中常有见。
shidilin 2# 2012-7-17 15:07
感谢K版的科普!
╰☆ヾo.海x 3# 2013-3-24 23:39
1# kuing 我知道怎么用键盘就能打大小号我肯定会打撒。。关键是。。。你教下是什么快捷键啊
kuing 4# 2013-3-24 23:40
3# ╰☆ヾo.海x 不用任何快捷键啊,这里怎么打,那里就怎么打   >    <  最最原始的打法就可以了啊
╰☆ヾo.海x 5# 2013-3-25 00:47
4# kuing 总算眼睛睁开来了。。看到了。。。shift+,  哈哈哈哈哈哈哎。。
kuing 6# 2013-4-19 17:01
前几天收到的一篇稿件又遇这种导数问题: 放大双眼看才看到那一撇,双击编辑,的确与我1#说的一样,用了上标,再打一撇。 顺便将这个贴移来这里……
thread-5-1-9.html: 网刊2期三角公式一文内某段的latex代码直接贴于此看看效果如何
kuing 1# 2011-9-25 22:36
两角和差展开公式: \begin{align*} \sin (\alpha \pm \beta ) & = \sin \alpha \cos \beta \pm \cos \alpha \sin \beta & \cos (\alpha \pm \beta ) & = \cos \alpha \cos \beta \mp \sin \alpha \sin \beta \\ \tan (\alpha \pm \beta ) & = \frac{{\tan \alpha \pm \tan \beta }}{{1 \mp \tan \alpha \tan \beta }} & \cot (\alpha \pm \beta ) & = - \frac{{1 \mp \cot \alpha \cot \beta }}{{\cot \alpha \pm \cot \beta }} \end{align*} 由正余弦的和差展开式可得如下的“积化和差”与“和差化积”公式: \begin{align*} \sin \alpha \cos \beta & = \frac{1}{2}\left( {\sin \left( {\alpha + \beta } \right) + \sin \left( {\alpha - \beta } \right)} \right) & \sin \alpha + \sin \beta & = 2\sin \frac{{\alpha + \beta }}{2}\cos \frac{{\alpha - \beta }}{2}\\ \cos \alpha \sin \beta & = \frac{1}{2}\left( {\sin \left( {\alpha + \beta } \right) - \sin \left( {\alpha - \beta } \right)} \right) & \sin \alpha - \sin \beta & = 2\cos \frac{{\alpha + \beta }}{2}\sin \frac{{\alpha - \beta }}{2}\\ \cos \alpha \cos \beta & = \frac{1}{2}\left( {\cos \left( {\alpha + \beta } \right) + \cos \left( {\alpha - \beta } \right)} \right) & \cos \alpha + \cos \beta & = 2\cos \frac{{\alpha + \beta }}{2}\cos \frac{{\alpha - \beta }}{2}\\ \sin \alpha \sin \beta & = - \frac{1}{2}\left( {\cos \left( {\alpha + \beta } \right) - \cos \left( {\alpha - \beta } \right)} \right) & \cos \alpha - \cos \beta & = - 2\sin \frac{{\alpha + \beta }}{2}\sin \frac{{\alpha - \beta }}{2} \end{align*} 以上的公式属于书本上的公式,故此就不写推导了。“积化和差”和“和差化积”公式虽然据说只要求了解不要求记忆,不过我觉得还是不妨一记,因为挺有用的,至少本文中多次用到。另外扯一题外话,不要总以为学得少或不用记就是减负,其实有时学多点记多点有价值的定理和方法,熟悉运用之后不但不是负担,更会成为你解决问题的兵器,所以换个角度来看,适当学多点也可以是减负。
kuing 2# 2011-9-25 22:38
完全没问题,:victory:
isea 3# 2011-9-25 23:27
这效果真赞,比一些专业数学论坛强多了。 这插件的确厉害。
kuing 4# 2011-9-25 23:30
其实没用插件……5d6d的免费论坛似乎不是那么容易就让你装插件,我只是抄了一点 http://jaxedit.googlecode.com/svn/trunk/jaxedit.html 源文件里的某些代码放到那个头部文件那里,但是由于我其实还没学过HTML和Javascript,所以基本上只是乱撞着研究……
thread-50-1-1.html: 大小
kuing 1# 2011-10-4 15:11
$kuing: \tiny o \small h ~ \large N \LARGE O ~ \Huge !?!?!?$ 测试成功,在公式里,有与真 latex 一样的调整字体大小的方法,此例的代码为 kuing: \tiny o \small h ~ \large N \LARGE O ~ \Huge !?!?!? 其中 ~ 是隔开一个小距离。 如果只想改变一部分字体的大小而不影响其他部分,用花括号在两边括起来即可,比如 kkkk{\Huge uin}gggg 显示为 $kkkk{\Huge uin}gggg$ 字体由小到大的代码,分别是 \tiny \scriptsize \small \normalsize \large \Large \LARGE \huge \Huge 其中 \normalsize 为默认大小。 $\tiny kuing$ $\scriptsize kuing$ $\small kuing$ $\normalsize kuing\leftarrow 默认大小$ $\large kuing$ $\Large kuing$ $\LARGE kuing$ $\huge kuing$ $\Huge kuing$
kuing 2# 2011-10-4 15:19
咦,footnotesize 不行,看来是因为这里不存在脚注之类的东东,所以没有 footnotesize。
kuing 3# 2011-10-4 15:32
测试成功便移来这边算鸟。。。
isea 4# 2013-1-22 01:34
本帖最后由 isea 于 2013-4-17 11:30 编辑 3# kuing 怎么让$$\Huge\pi$$直立不加粗? ======== CTeX 下 \usepackage{upgreek} \pi,\uppi \mu,\upmu 即希腊字母前加up 补充于 04.17.13
kuing 5# 2013-1-22 01:45
4# isea 真正的LaTeX里有办法, 但要在这里实现的话我就不清楚了,恐怕没办法,我明天有空试试看, 现在爪机。
hnsredfox_007 6# 2013-1-22 09:07
$\rm {\pi}$
hnsredfox_007 7# 2013-1-22 09:09
4# isea \rm {\pi}   $\rm {\pi}$
kuing 8# 2013-1-22 12:14
4# isea \rm {\pi}   $\rm {\pi}$ hnsredfox_007 发表于 2013-1-22 09:09 \rm {\pi}   $\rm {\pi}$ \pi   $\pi$ 一样?
hnsredfox_007 9# 2013-1-22 14:05
8# kuing 晕!呵呵,原来$\pi$是直立的啊
kuing 10# 2013-1-22 14:09
9# hnsredfox_007 其实还是斜体,isea 想要的直立 pi 不是这种
kuing 11# 2013-1-22 14:15
\pi abc+\rm{\pi abc}   $\pi abc+\rm{\pi abc}$ \pi abc+\textrm{\pi abc}   $\pi abc+\textrm{\pi abc}$ \rm 对 \pi 没作用; \textrm 更直接把代码也显示出来了?我记得真正的 LaTeX 不是这样的。
thread-500-1-1.html: 将“kuing 状态不佳地 play 一 play 高考(数学) in 2012”复制过来,这里直接显示公式
kuing 1# 2012-6-7 23:54
在这里就直接显示公式了 原贴转自:http://bbs.pep.com.cn/thread-2532549-1-1.html 声 明   还是那句,本人非教师,也非学生,2007年高中毕业=失业,无业游民至今,所以,请不要叫我老师①。而与往年不太一样的是,最近我心情不太好(熟悉我的朋友或者在群里跟我吹得比较多的网友应该知道什么原因,这里就不扯了),状态不佳,所以玩的题可能没往年那么多,质量估计有所下降甚至可能错乱百出,先说一声见谅了,如果发现哪里错了请跟贴提出,同时希望大家一起玩玩。   还有一点不同的是我这次故意用 LaTeX 代码来写题目解答,嘿嘿,估计会被喷,不过我也没所谓了,如果由此没人看就当自拆招牌吧。   最后当然还是要把口号叫一叫:珍爱生命,远离考试。 ___________________   ①这一段略抄自去年的贴子:http://bbs.pep.com.cn/thread-1559068-1-1.html 注:部分题目及解答已被我选入《数学空间》总第9期的《对2012年的几道高考数学题的解答及研究》一文中。见: http://www.pep.com.cn/rjwk/gzsxs ... 0120724_1133829.htm 也可以下载整期的PDF来看,然后找到正文第18页开始。《数学空间》总第9期PDF版下载链接见 http://www.pep.com.cn/rjwk/gzsxsxkj/2012/sxkj9/ 下方。
kuing 2# 2012-6-7 23:54
一、广东文数10 and 理数8 由条件知 \begin{align*} \frac{|\vv a|}{|\vv b|}\cos\theta &= \frac m2,\\ \frac{|\vv b|}{|\vv a|}\cos\theta &= \frac n2, \end{align*} 其中 $m$, $n \in \mathbb Z$,两式相乘得 \[ \cos^2\theta =\frac{mn}4, \] 又由角的范围知 \[ \cos^2\theta \in \left(0,\frac12\right),\] 从而易得 $m=n=1$ 或 $m=n=-1$,后者显然余去,故D。 与文数类似,用回前面所设,得到 \[ \cos^2\theta =\frac{mn}4, \] 此处角的范围不同,有 \[ \cos^2\theta \in \left(\frac12,1\right),\] 从而易得 $m=1$, $n=3$ 或 $m=3$, $n=1$ 或 $m=-1$, $n=-3$ 或 $m=-3$, $n=-1$,显然只有 $m=3$, $n=1$ 符合,故C。
kuing 3# 2012-6-7 23:56
二、安徽理数10 由题意知6位同学之间最多交换$C_6^2=15$次,现在已经交换了13次了,还有两次没交换。 如果这两次涉及同一人,则收到4份纪念品的为2人; 如果这两次不涉及同一人,则收到4份纪念品的为4人。 D
kuing 4# 2012-6-7 23:56
三、安徽理数20 关于这个题的第二问,其实 $P$ 可以一般化,事实上,有如下推广命题: 椭圆上有任意一点$P$(非长轴顶点),焦点$F$相应的准线为$L$,连结$PF$,过$F$作$PF$的垂线交$L$于$Q$,则直线$PQ$与椭圆相切。 双曲线上也类似,抛物线就更不用说了,故此这也提供了作圆锥曲线上的点的切线的一种作法。 下面用几何方法证明推广命题。 假设$PQ$与椭圆交于点$P$以外的另一点$ P' $,作如图所示的辅助线。 则 \[ \frac{PF}{P' F} = \frac{e\cdot PG}{e\cdot P' H} = \frac{PG}{P' H} = \frac{PQ}{P' Q} = \frac{PF}{P' E}, \] 这样得到 \[ P' F = P' E, \] 显然矛盾,故得证。 ______________________ 本题及解答已被我选入《数学空间》总第9期的《对2012年的几道高考数学题的解答及研究》一文中。
kuing 5# 2012-6-7 23:57
四、山东理数12 依题意知关于 $x$ 的多项式 \[ h(x) = ax^3+bx^2-1 \] 有二重根,不妨设二重根为 $x_1$,另一根为 $x_2$,则上述多项式可以分解为 \[ h(x) = a(x-x_1)^2(x-x_2), \] 展开对比 $x$ 的系数得 \[ ax_1(x_1+2x_2)=0, \] 由于 $a \ne 0$ 且显然 $x_1 \ne 0$,故必有 \[ x_1 = -2x_2, \] 又展开对比常数项得 \[ ax_1^2x_2 = 1, \] 从而解得 \[ x_1 = -2x_2 = - \sqrt[3]{\frac2a}, \] 故易得 \[ y_1 = - \frac{y_2}2 = - \sqrt[3]{\frac a2}, \] 从而 \begin{align*} x_1+x_2 &= - \sqrt[3]{\frac1{4a}},\\ y_1+y_2 &= \sqrt[3]{\frac a2}, \end{align*} 选B ______________________ 本题及解答已被我选入《数学空间》总第9期的《对2012年的几道高考数学题的解答及研究》一文中。
kuing 6# 2012-6-7 23:58
五、江西理数6、7 6、由 $a+b=1$, $a^2+b^2=3$ 解得 \[ a = \frac{1\pm\sqrt5}2, b = \frac{1\mp\sqrt5}2, \] 从而 $a^n+b^n$ 符合斐波拉契数列的递推公式。 _____6-10回头更新_____ 更简单的方法是由 $a+b=1$, $a^2+b^2=3$ 得到 $ab=-1$,从而 $a$, $b$ 是方程 $x^2-x-1=0$ 的两根,于是 \[ a^{n+2}+b^{n+2} = a^n(1+a)+b^n(1+b) = a^{n+1}+b^{n+1}+a^n+b^n, \] 所以也得到 $a^n+b^n$ 符合斐波拉契数列的递推公式。 7、由中线长公式,有 \begin{align*} 4PA^2+4PB^2 &= 2AC^2+2AD^2-CD^2 + 2BC^2+2BD^2-CD^2 \\ &= 2(AC^2+BC^2)+2(AD^2+BD^2)-2CD^2 \\ &= 2AB^2+2CD^2 = 10CD^2 = 40PC^2, \end{align*} 故 \[ \frac{PA^2+PB^2}{PC^2}=10. \] _____6-10回头更新_____ 其实直接在 $\triangle PAB$ 时里用中线长公式更好,即 \[ 4PC^2 = 4PD^2 = 2PA^2+2PB^2-AB^2 = 2PA^2+2PB^2-(4PC)^2, \] 从而得到 \[ \frac{PA^2+PB^2}{PC^2}=10. \] ______________________ 本两题及解答已被我选入《数学空间》总第9期的《对2012年的几道高考数学题的解答及研究》一文中。
kuing 7# 2012-6-8 00:00
六、江西理数9、10 9、依题意得 \[ \frac{n \bar x + m \bar y}{n+m} = \alpha \bar x + (1-\alpha) \bar y, \] 作差分解得 \[ \frac{(\bar x - \bar y) \bigl(\alpha m - (1-\alpha)n\bigr)}{m+n} = 0, \] 由 $\bar x \ne \bar y$ 解得 \[ \frac mn = \frac{1-\alpha}{\alpha}>1.\] 10、设截面面积为 $S(x)$,则显然 $V ' (x) = -S(x)$,从而曲线的开始与结尾处的切线斜率必定都为0,只有A。 注:文数10也是这类东西,你们玩吧。
kuing 8# 2012-6-8 00:29
休息一下先
╰☆ヾo.海x 9# 2012-6-8 01:24
我顶我顶我顶顶顶。。。
yayaweha 10# 2012-6-8 10:00
太棒了
kuing 11# 2012-6-8 11:44
七、新课标全国卷理数16 由 \[ a_{n+1} + (-1)^n a_n = 2n-1, \] 得 \begin{align*} a_{n+2} &= (-1)^n a_{n+1}+2n+1 \\ &= (-1)^n \bigl((-1)^{n-1}a_n+2n-1\bigr)+2n+1 \\ &= -a_n+(-1)^n(2n-1)+2n+1, \end{align*} 即 \[ a_{n+2}+a_n = (-1)^n(2n-1)+2n+1, \] 也有 \[ a_{n+3}+a_{n+1} = -(-1)^n(2n+1)+2n+3, \] 两式相加得到 \[ a_n+a_{n+1}+a_{n+2}+a_{n+3} = -2(-1)^n+4n+4, \] 设 $k$ 为整数,则 \[ a_{4k+1}+a_{4k+2}+a_{4k+3}+a_{4k+4} = -2(-1)^{4k+1}+4(4k+1)+4 = 16k+10, \] 于是 $a_n$ 的前60项和便是 \[ S_{60} = \sum_{k=0}^{14}(a_{4k+1}+a_{4k+2}+a_{4k+3}+a_{4k+4}) = \sum_{k=0}^{14}(16k+10) = 1830. \] PS、第12题与个多月前的这个贴http://bbs.pep.com.cn/thread-2457414-1-1.html几乎一样,难道……哼哼 ______________________ 本题及解答已被我选入《数学空间》总第9期的《对2012年的几道高考数学题的解答及研究》一文中。
kuing 12# 2012-6-8 11:44
八、重庆8(文/理) 文8 记 $g(x) = x f '(x)$,则依题意知 $f '(-2) = 0$ 且 $f '' (-2) > 0$,故 $g(-2)=0$,又 $g'(x) = f '(x) + x f '' (x)$,故 $g'(-2) = -2f '' (-2) < 0$,从而只有C。 理8 记 $g(x) = (1-x)f ' (x)$,则 $g'(x) = - f ' (x) + (1-x) f '' (x)$。 由图像知 $g(-2)=0$ 且 $g'(-2)<0$,从而易得 $ f ' (-2) = 0$ 且 $ f '' (-2) < 0$,故 $f(-2)$ 是极大值; 由图知 $g(2)=0$ 且 $g'(2)<0$,从而易得 $ f ' (2) = 0$ 且 $ f '' (2) > 0$,故 $f(2)$ 是极小值。 D PS、重庆9(文/理)跟2010辽宁理数12有点像,但简单多了。
kuing 13# 2012-6-8 11:45
九、安徽15(文/理) 文15 不详讲了,画个图了事: 理15 也不详讲了,①、②都是余弦定理然后将 $c$ 放掉再均值便得;③是FAQ的特例;④、⑤均令 $a=b$ 且 $c \to 0$ 使知不成立。 呃,质量又下降了……
kuing 14# 2012-6-8 18:27
题目出来得太慢了……
wenshengli 15# 2012-6-8 20:19
11# kuing 这个题难倒成片的学生!
kuing 16# 2012-6-9 00:10
十、江苏19 关于本题第(2)问的第(ii)小问,对一般椭圆也成立,对双曲线也也成立。 如图所示 延长 $AF_1$ 交椭圆于 $B ' $,由于平行,易证 $B ' F_1 = BF_2$,设 $B ' F_1 = x$, $AF_1 = y$,则由三角形相似易得 \[ PF_1 = (2a-x)\cdot \frac y{x+y}, PF_2 = (2a-y)\cdot \frac x{x+y}, \] 于是 \[ PF_1+PF_2 = (2a-x)\cdot \frac y{x+y} + (2a-y)\cdot \frac x{x+y} = 2a-\frac2{\frac1x+\frac1y}, \] 由于 $AB ' $ 是焦点弦,故熟知上式最右边那一项的分母为定值,所以得证。 双曲线的类似可证。 ______________________ 本题及解答已被我选入《数学空间》总第9期的《对2012年的几道高考数学题的解答及研究》一文中。
kuing 17# 2012-6-9 00:11
十一、福建理数10 题穷啊,又搞凸函数。 ①错,比如 $f(x)=\begin{cases} 1, & 1<x<3,\\ 2, & x=1~\text{或}~3, \end{cases}$ ②错,比如 $f(x) = -x$ ③④显然正确 事实上,还可以证明 $\displaystyle f\left(\frac{x_1+x_2+x_3}3\right)\leqslant\frac13\bigl(f(x_1)+f(x_2)+f(x_3)\bigr)$,为何不给这个出来判断呢?如果是我出题的话肯定给这个。
kuing 18# 2012-6-9 00:12
十二、湖北理数6 这个按理应该略过的,但突然想起我以前就解过类似的,一找贴之下发现: http://bbs.pep.com.cn/thread-396608-1-1.html 就数字有点不同,方法明显是一样的,大家进去看看便知。 PS、顺手看了下随后的两题,第7题显然①③,第8题看着像小学割补题。
kuing 19# 2012-6-9 02:40
十三、湖南文数16 第一问略,只看下第二问。这个题想是不难想,就是表达有点麻烦。 为方便表达,我们用 $(n)_2$ 来表示 $n$ 的二进制数。 不过还是劲难表达的说。 当某个正整数 $n$ 使得 $b_n=0$ 时 (1)如果 $(n)_2$ 的最后一位为 0(即 $n$ 为偶数)时         (1-1)如果 $(n)_2$ 的倒数第二位也为 0 时,则 $b_{n+1}=b_{n+2}=1$,$b_{n+3}=0$;(例如 $(n)_2 = 1100$)         (1-2)如果 $(n)_2$ 由倒数第二位到倒数第 $p$ 位($p \geqslant 2$)都为 1 而倒数第 $p+1$ 位为 0 时                 (1-2-1)如果 $p$ 为偶数,则 $b_{n+1}=1$,$b_{n+2}=0$;(例如 $(n)_2 = 101110$)                 (1-2-2)如果 $p$ 为奇数,则 $b_{n+1}=b_{n+2}=1$,$b_{n+3}=0$;(例如 $(n)_2 = 11011110$) (2)如果 $(n)_2$ 由最后一位到倒数第 $p$ 位($p \geqslant 1$)都为 1 而倒数第 $p+1$ 位为 0 时         (2-1)如果 $p$ 为奇数,则 $b_{n+1}=0$;(例如 $(n)_2 = 10111$)         (2-2)如果 $p$ 为偶数,则 $b_{n+1}=1$,$b_{n+2}=0$。(例如 $(n)_2 = 11011$) 所有分类讨论完,可见当 $b_n=0$ 后三项之内必再出现 0,再由(1-1)和(1-2-2)的例子可见,$c_m$ 的最大值就是 2。
kuing 20# 2012-6-9 17:21
十五、湖北文数17 既然要算被 5 整除的,那就按 5 为模分类算算。设 $m$ 为非负整数,则 (1)当 $n=5m+1$ 时 \[a_{5m+1}=\frac{(5m+1)(5m+2)}2 = 5m^5 + 15\cdot\frac{m(m+1)}2 + 1,\] 故 $a_{5m+1}$ 除以 5 余 1; (2)当 $n=5m+2$ 时 \[a_{5m+2}=\frac{(5m+2)(5m+3)}2 = 25\cdot\frac{m(m+1)}2 + 3,\] 故 $a_{5m+2}$ 除以 5 余 3; (3)当 $n=5m+3$ 时 \[a_{5m+3}=\frac{(5m+3)(5m+4)}2 = 25\cdot\frac{m(m+1)}2 + 5(m+1) + 1,\] 故 $a_{5m+3}$ 除以 5 余 1; (4)当 $n=5m+4$ 时 \[a_{5m+4}=\frac{(5m+4)(5m+5)}2 = 5\cdot\left(5\cdot\frac{m(m+1)}2+2m+2\right),\] 故 $a_{5m+4}$ 被 5 整除; (5)当 $n=5m+5$ 时 \[a_{5m+5}=\frac{(5m+5)(5m+6)}2 = 5\cdot\left(5\cdot\frac{m(m+1)}2+3m+3\right),\] 故 $a_{5m+5}$ 被 5 整除。 由此可见,$\{b_1,b_2,b_3,b_4,b_5,b_6,\ldots\} = \{a_4,a_5,a_9,a_{10},a_{14},a_{15},\ldots\}$,即 \[\left\{\begin{aligned} b_{2k-1}&=a_{5k-1}=\frac{5k(5k-1)}2,\\ b_{2k}&=a_{5k}=\frac{5k(5k+1)}2, \end{aligned}\right.\]
thread-500-1-3.html: 将“kuing 状态不佳地 play 一 play 高考(数学) in 2012”复制过来,这里直接显示公式
kuing 1# 2012-6-7 23:54
在这里就直接显示公式了 原贴转自:http://bbs.pep.com.cn/thread-2532549-1-1.html 声 明   还是那句,本人非教师,也非学生,2007年高中毕业=失业,无业游民至今,所以,请不要叫我老师①。而与往年不太一样的是,最近我心情不太好(熟悉我的朋友或者在群里跟我吹得比较多的网友应该知道什么原因,这里就不扯了),状态不佳,所以玩的题可能没往年那么多,质量估计有所下降甚至可能错乱百出,先说一声见谅了,如果发现哪里错了请跟贴提出,同时希望大家一起玩玩。   还有一点不同的是我这次故意用 LaTeX 代码来写题目解答,嘿嘿,估计会被喷,不过我也没所谓了,如果由此没人看就当自拆招牌吧。   最后当然还是要把口号叫一叫:珍爱生命,远离考试。 ___________________   ①这一段略抄自去年的贴子:http://bbs.pep.com.cn/thread-1559068-1-1.html 注:部分题目及解答已被我选入《数学空间》总第9期的《对2012年的几道高考数学题的解答及研究》一文中。见: http://www.pep.com.cn/rjwk/gzsxs ... 0120724_1133829.htm 也可以下载整期的PDF来看,然后找到正文第18页开始。《数学空间》总第9期PDF版下载链接见 http://www.pep.com.cn/rjwk/gzsxsxkj/2012/sxkj9/ 下方。
kuing 2# 2012-6-7 23:54
一、广东文数10 and 理数8 由条件知 \begin{align*} \frac{|\vv a|}{|\vv b|}\cos\theta &= \frac m2,\\ \frac{|\vv b|}{|\vv a|}\cos\theta &= \frac n2, \end{align*} 其中 $m$, $n \in \mathbb Z$,两式相乘得 \[ \cos^2\theta =\frac{mn}4, \] 又由角的范围知 \[ \cos^2\theta \in \left(0,\frac12\right),\] 从而易得 $m=n=1$ 或 $m=n=-1$,后者显然余去,故D。 与文数类似,用回前面所设,得到 \[ \cos^2\theta =\frac{mn}4, \] 此处角的范围不同,有 \[ \cos^2\theta \in \left(\frac12,1\right),\] 从而易得 $m=1$, $n=3$ 或 $m=3$, $n=1$ 或 $m=-1$, $n=-3$ 或 $m=-3$, $n=-1$,显然只有 $m=3$, $n=1$ 符合,故C。
kuing 3# 2012-6-7 23:56
二、安徽理数10 由题意知6位同学之间最多交换$C_6^2=15$次,现在已经交换了13次了,还有两次没交换。 如果这两次涉及同一人,则收到4份纪念品的为2人; 如果这两次不涉及同一人,则收到4份纪念品的为4人。 D
kuing 4# 2012-6-7 23:56
三、安徽理数20 关于这个题的第二问,其实 $P$ 可以一般化,事实上,有如下推广命题: 椭圆上有任意一点$P$(非长轴顶点),焦点$F$相应的准线为$L$,连结$PF$,过$F$作$PF$的垂线交$L$于$Q$,则直线$PQ$与椭圆相切。 双曲线上也类似,抛物线就更不用说了,故此这也提供了作圆锥曲线上的点的切线的一种作法。 下面用几何方法证明推广命题。 假设$PQ$与椭圆交于点$P$以外的另一点$ P' $,作如图所示的辅助线。 则 \[ \frac{PF}{P' F} = \frac{e\cdot PG}{e\cdot P' H} = \frac{PG}{P' H} = \frac{PQ}{P' Q} = \frac{PF}{P' E}, \] 这样得到 \[ P' F = P' E, \] 显然矛盾,故得证。 ______________________ 本题及解答已被我选入《数学空间》总第9期的《对2012年的几道高考数学题的解答及研究》一文中。
kuing 5# 2012-6-7 23:57
四、山东理数12 依题意知关于 $x$ 的多项式 \[ h(x) = ax^3+bx^2-1 \] 有二重根,不妨设二重根为 $x_1$,另一根为 $x_2$,则上述多项式可以分解为 \[ h(x) = a(x-x_1)^2(x-x_2), \] 展开对比 $x$ 的系数得 \[ ax_1(x_1+2x_2)=0, \] 由于 $a \ne 0$ 且显然 $x_1 \ne 0$,故必有 \[ x_1 = -2x_2, \] 又展开对比常数项得 \[ ax_1^2x_2 = 1, \] 从而解得 \[ x_1 = -2x_2 = - \sqrt[3]{\frac2a}, \] 故易得 \[ y_1 = - \frac{y_2}2 = - \sqrt[3]{\frac a2}, \] 从而 \begin{align*} x_1+x_2 &= - \sqrt[3]{\frac1{4a}},\\ y_1+y_2 &= \sqrt[3]{\frac a2}, \end{align*} 选B ______________________ 本题及解答已被我选入《数学空间》总第9期的《对2012年的几道高考数学题的解答及研究》一文中。
kuing 6# 2012-6-7 23:58
五、江西理数6、7 6、由 $a+b=1$, $a^2+b^2=3$ 解得 \[ a = \frac{1\pm\sqrt5}2, b = \frac{1\mp\sqrt5}2, \] 从而 $a^n+b^n$ 符合斐波拉契数列的递推公式。 _____6-10回头更新_____ 更简单的方法是由 $a+b=1$, $a^2+b^2=3$ 得到 $ab=-1$,从而 $a$, $b$ 是方程 $x^2-x-1=0$ 的两根,于是 \[ a^{n+2}+b^{n+2} = a^n(1+a)+b^n(1+b) = a^{n+1}+b^{n+1}+a^n+b^n, \] 所以也得到 $a^n+b^n$ 符合斐波拉契数列的递推公式。 7、由中线长公式,有 \begin{align*} 4PA^2+4PB^2 &= 2AC^2+2AD^2-CD^2 + 2BC^2+2BD^2-CD^2 \\ &= 2(AC^2+BC^2)+2(AD^2+BD^2)-2CD^2 \\ &= 2AB^2+2CD^2 = 10CD^2 = 40PC^2, \end{align*} 故 \[ \frac{PA^2+PB^2}{PC^2}=10. \] _____6-10回头更新_____ 其实直接在 $\triangle PAB$ 时里用中线长公式更好,即 \[ 4PC^2 = 4PD^2 = 2PA^2+2PB^2-AB^2 = 2PA^2+2PB^2-(4PC)^2, \] 从而得到 \[ \frac{PA^2+PB^2}{PC^2}=10. \] ______________________ 本两题及解答已被我选入《数学空间》总第9期的《对2012年的几道高考数学题的解答及研究》一文中。
kuing 7# 2012-6-8 00:00
六、江西理数9、10 9、依题意得 \[ \frac{n \bar x + m \bar y}{n+m} = \alpha \bar x + (1-\alpha) \bar y, \] 作差分解得 \[ \frac{(\bar x - \bar y) \bigl(\alpha m - (1-\alpha)n\bigr)}{m+n} = 0, \] 由 $\bar x \ne \bar y$ 解得 \[ \frac mn = \frac{1-\alpha}{\alpha}>1.\] 10、设截面面积为 $S(x)$,则显然 $V ' (x) = -S(x)$,从而曲线的开始与结尾处的切线斜率必定都为0,只有A。 注:文数10也是这类东西,你们玩吧。
kuing 8# 2012-6-8 00:29
休息一下先
╰☆ヾo.海x 9# 2012-6-8 01:24
我顶我顶我顶顶顶。。。
yayaweha 10# 2012-6-8 10:00
太棒了
kuing 11# 2012-6-8 11:44
七、新课标全国卷理数16 由 \[ a_{n+1} + (-1)^n a_n = 2n-1, \] 得 \begin{align*} a_{n+2} &= (-1)^n a_{n+1}+2n+1 \\ &= (-1)^n \bigl((-1)^{n-1}a_n+2n-1\bigr)+2n+1 \\ &= -a_n+(-1)^n(2n-1)+2n+1, \end{align*} 即 \[ a_{n+2}+a_n = (-1)^n(2n-1)+2n+1, \] 也有 \[ a_{n+3}+a_{n+1} = -(-1)^n(2n+1)+2n+3, \] 两式相加得到 \[ a_n+a_{n+1}+a_{n+2}+a_{n+3} = -2(-1)^n+4n+4, \] 设 $k$ 为整数,则 \[ a_{4k+1}+a_{4k+2}+a_{4k+3}+a_{4k+4} = -2(-1)^{4k+1}+4(4k+1)+4 = 16k+10, \] 于是 $a_n$ 的前60项和便是 \[ S_{60} = \sum_{k=0}^{14}(a_{4k+1}+a_{4k+2}+a_{4k+3}+a_{4k+4}) = \sum_{k=0}^{14}(16k+10) = 1830. \] PS、第12题与个多月前的这个贴http://bbs.pep.com.cn/thread-2457414-1-1.html几乎一样,难道……哼哼 ______________________ 本题及解答已被我选入《数学空间》总第9期的《对2012年的几道高考数学题的解答及研究》一文中。
kuing 12# 2012-6-8 11:44
八、重庆8(文/理) 文8 记 $g(x) = x f '(x)$,则依题意知 $f '(-2) = 0$ 且 $f '' (-2) > 0$,故 $g(-2)=0$,又 $g'(x) = f '(x) + x f '' (x)$,故 $g'(-2) = -2f '' (-2) < 0$,从而只有C。 理8 记 $g(x) = (1-x)f ' (x)$,则 $g'(x) = - f ' (x) + (1-x) f '' (x)$。 由图像知 $g(-2)=0$ 且 $g'(-2)<0$,从而易得 $ f ' (-2) = 0$ 且 $ f '' (-2) < 0$,故 $f(-2)$ 是极大值; 由图知 $g(2)=0$ 且 $g'(2)<0$,从而易得 $ f ' (2) = 0$ 且 $ f '' (2) > 0$,故 $f(2)$ 是极小值。 D PS、重庆9(文/理)跟2010辽宁理数12有点像,但简单多了。
kuing 13# 2012-6-8 11:45
九、安徽15(文/理) 文15 不详讲了,画个图了事: 理15 也不详讲了,①、②都是余弦定理然后将 $c$ 放掉再均值便得;③是FAQ的特例;④、⑤均令 $a=b$ 且 $c \to 0$ 使知不成立。 呃,质量又下降了……
kuing 14# 2012-6-8 18:27
题目出来得太慢了……
wenshengli 15# 2012-6-8 20:19
11# kuing 这个题难倒成片的学生!
kuing 16# 2012-6-9 00:10
十、江苏19 关于本题第(2)问的第(ii)小问,对一般椭圆也成立,对双曲线也也成立。 如图所示 延长 $AF_1$ 交椭圆于 $B ' $,由于平行,易证 $B ' F_1 = BF_2$,设 $B ' F_1 = x$, $AF_1 = y$,则由三角形相似易得 \[ PF_1 = (2a-x)\cdot \frac y{x+y}, PF_2 = (2a-y)\cdot \frac x{x+y}, \] 于是 \[ PF_1+PF_2 = (2a-x)\cdot \frac y{x+y} + (2a-y)\cdot \frac x{x+y} = 2a-\frac2{\frac1x+\frac1y}, \] 由于 $AB ' $ 是焦点弦,故熟知上式最右边那一项的分母为定值,所以得证。 双曲线的类似可证。 ______________________ 本题及解答已被我选入《数学空间》总第9期的《对2012年的几道高考数学题的解答及研究》一文中。
kuing 17# 2012-6-9 00:11
十一、福建理数10 题穷啊,又搞凸函数。 ①错,比如 $f(x)=\begin{cases} 1, & 1<x<3,\\ 2, & x=1~\text{或}~3, \end{cases}$ ②错,比如 $f(x) = -x$ ③④显然正确 事实上,还可以证明 $\displaystyle f\left(\frac{x_1+x_2+x_3}3\right)\leqslant\frac13\bigl(f(x_1)+f(x_2)+f(x_3)\bigr)$,为何不给这个出来判断呢?如果是我出题的话肯定给这个。
kuing 18# 2012-6-9 00:12
十二、湖北理数6 这个按理应该略过的,但突然想起我以前就解过类似的,一找贴之下发现: http://bbs.pep.com.cn/thread-396608-1-1.html 就数字有点不同,方法明显是一样的,大家进去看看便知。 PS、顺手看了下随后的两题,第7题显然①③,第8题看着像小学割补题。
kuing 19# 2012-6-9 02:40
十三、湖南文数16 第一问略,只看下第二问。这个题想是不难想,就是表达有点麻烦。 为方便表达,我们用 $(n)_2$ 来表示 $n$ 的二进制数。 不过还是劲难表达的说。 当某个正整数 $n$ 使得 $b_n=0$ 时 (1)如果 $(n)_2$ 的最后一位为 0(即 $n$ 为偶数)时         (1-1)如果 $(n)_2$ 的倒数第二位也为 0 时,则 $b_{n+1}=b_{n+2}=1$,$b_{n+3}=0$;(例如 $(n)_2 = 1100$)         (1-2)如果 $(n)_2$ 由倒数第二位到倒数第 $p$ 位($p \geqslant 2$)都为 1 而倒数第 $p+1$ 位为 0 时                 (1-2-1)如果 $p$ 为偶数,则 $b_{n+1}=1$,$b_{n+2}=0$;(例如 $(n)_2 = 101110$)                 (1-2-2)如果 $p$ 为奇数,则 $b_{n+1}=b_{n+2}=1$,$b_{n+3}=0$;(例如 $(n)_2 = 11011110$) (2)如果 $(n)_2$ 由最后一位到倒数第 $p$ 位($p \geqslant 1$)都为 1 而倒数第 $p+1$ 位为 0 时         (2-1)如果 $p$ 为奇数,则 $b_{n+1}=0$;(例如 $(n)_2 = 10111$)         (2-2)如果 $p$ 为偶数,则 $b_{n+1}=1$,$b_{n+2}=0$。(例如 $(n)_2 = 11011$) 所有分类讨论完,可见当 $b_n=0$ 后三项之内必再出现 0,再由(1-1)和(1-2-2)的例子可见,$c_m$ 的最大值就是 2。
kuing 20# 2012-6-9 17:21
十五、湖北文数17 既然要算被 5 整除的,那就按 5 为模分类算算。设 $m$ 为非负整数,则 (1)当 $n=5m+1$ 时 \[a_{5m+1}=\frac{(5m+1)(5m+2)}2 = 5m^5 + 15\cdot\frac{m(m+1)}2 + 1,\] 故 $a_{5m+1}$ 除以 5 余 1; (2)当 $n=5m+2$ 时 \[a_{5m+2}=\frac{(5m+2)(5m+3)}2 = 25\cdot\frac{m(m+1)}2 + 3,\] 故 $a_{5m+2}$ 除以 5 余 3; (3)当 $n=5m+3$ 时 \[a_{5m+3}=\frac{(5m+3)(5m+4)}2 = 25\cdot\frac{m(m+1)}2 + 5(m+1) + 1,\] 故 $a_{5m+3}$ 除以 5 余 1; (4)当 $n=5m+4$ 时 \[a_{5m+4}=\frac{(5m+4)(5m+5)}2 = 5\cdot\left(5\cdot\frac{m(m+1)}2+2m+2\right),\] 故 $a_{5m+4}$ 被 5 整除; (5)当 $n=5m+5$ 时 \[a_{5m+5}=\frac{(5m+5)(5m+6)}2 = 5\cdot\left(5\cdot\frac{m(m+1)}2+3m+3\right),\] 故 $a_{5m+5}$ 被 5 整除。 由此可见,$\{b_1,b_2,b_3,b_4,b_5,b_6,\ldots\} = \{a_4,a_5,a_9,a_{10},a_{14},a_{15},\ldots\}$,即 \[\left\{\begin{aligned} b_{2k-1}&=a_{5k-1}=\frac{5k(5k-1)}2,\\ b_{2k}&=a_{5k}=\frac{5k(5k+1)}2, \end{aligned}\right.\]
thread-500-2-3.html:
kuing 21# 2012-6-9 17:21
十六、上海理数14 分别过 $B$, $C$ 作 $AD$ 的垂线,由于 $AD\perp BC$,所以两垂线的垂足共点,设其为 $E$。 这里明显有 $EB=EC$,若不等,假设 $EB<EC$,则由勾股定理得 $AB<AC$, $BD<CD$,这与 $AB+BD=AC+CD$ 矛盾。 由此,设 $EB=EC=x$,则易证 $1<x\leqslant \sqrt{a^2-c^2}$,以及 $S_{\triangle EBC}=\sqrt{x^2-1}$,从而 \[V_{ABCD} = \frac{2c}3\sqrt{x^2-1}\leqslant \frac{2c}3\sqrt{a^2-c^2-1}.\] 这题越写越发现没玩头,早知道不选了,不过图都截了也写了大半了,算了哎……
kuing 22# 2012-6-9 17:22
十七、四川10 and 12(文/理) 10(文/理) 由球面直角三角形的某条公式得 \[ \cos\frac{AP}R = \cos60^\circ \cos45^\circ = \frac{\sqrt2}4, \] 故A。 理12 令 \[ g(x) = f \left( x+\frac\pi2 \right) - \pi = 2x+\sin x, \] 则已知等式化为 \[g\left(a_1-\frac\pi2\right)+g\left(a_2-\frac\pi2\right)+\cdots+g\left(a_5-\frac\pi2\right)=0.\] 我们将证明 $a_3=\pi/2$。若不然,假设 $a_3>\pi/2$,那么 \[a_3-\frac\pi2>0,   a_1-\frac\pi2+a_5-\frac\pi2=a_2-\frac\pi2+a_4-\frac\pi2=2a_3-\pi>0,\] 注意到 $g(x)$ 为递增的奇函数,熟知有 $g(x)+g(y)>0 \iff x+y>0$,从而得到 \[g\left(a_3-\frac\pi2\right)>0,   g\left(a_1-\frac\pi2\right)+g\left(a_5-\frac\pi2\right)>0,   g\left(a_2-\frac\pi2\right)+g\left(a_4-\frac\pi2\right)>0,\] 矛盾! 同理可证假设 $a_3<\pi/2$ 时将产生矛盾,故此必有 $a_3=\pi/2$。 从而 $\bigl(f(a_3)\bigr)^2-a_1a_5=\pi^2-(\pi/2+\pi/4)(\pi/2-\pi/4)=13\pi^2/16$。 文12 与理12类似,略。 ______________________ 理12及解答已被我选入《数学空间》总第9期的《对2012年的几道高考数学题的解答及研究》一文中。
kuing 23# 2012-6-9 18:04
十八、四川文数16 ①对,反证一下,假设 $a-b\geqslant 1$,则 $a^2-b^2=(a+b)(a-b)\geqslant a+b>a-b\geqslant 1$,矛盾。 ②错,反例取 $a=2$, $b=2/3$; ③错,事实上,不等式反向成立,即当 $\bigl|\sqrt a - \sqrt b\bigr|=1$ 时有 $|a-b|>1$,我们也用反证法证一下。 假设 $|a-b|\leqslant 1$,则 \[\bigl|\sqrt a - \sqrt b\bigr|=\frac{|a-b|}{\sqrt a+\sqrt b}\leqslant \frac1{\sqrt a+\sqrt b} < \frac1{\bigl|\sqrt a - \sqrt b\bigr|},\] 从而得到 $\bigl|\sqrt a - \sqrt b\bigr|<1$,矛盾; ④对,由 $1=|a^3-b^3|=|a-b|(a^2+ab+b^2)>|a-b|(a^2-2ab+b^2)=|a-b|^3$ 即得 $|a-b|<1$。
kuing 24# 2012-6-9 20:50
十九、四川理数16 这题真不是好对付的。 ①对,直接列出来即知正确,事实上,后面的都是 2; ②错,当 $a=3$ 时 $\{x_n\}$ 为 $\{3,2,1,2,1,2,1,\ldots\}$ 后面一直是 2, 1 循环; ③对,由于对任意整数 $n$,容易证明恒有 \[ \left[ \frac n2 \right]\geqslant \frac{n-1}2,\] 又显然 $x_n+[a/x_n]$ 为整数,于是,由 $[x]>x-1$ 以及均值不等式,我们有 \[ x_{n+1}=\left[ \frac{x_n+\left[\frac a{x_n}\right]}2 \right]\geqslant \frac{x_n+\left[\frac a{x_n}\right]-1}2 > \frac{x_n+\frac a{x_n}-2}2\geqslant \sqrt a-1. \] 所以命题成立; ④对,我们先来证明 $x_n\geqslant \bigl[\sqrt a\bigr]$ 恒成立,若不然,假设存在某个 $x_k<\bigl[\sqrt a\bigr]$,由于 $x_k$ 和 $\bigl[\sqrt a\bigr]$ 均为整数,因此有 \[ x_k\leqslant \bigl[\sqrt a\bigr]-1\leqslant \sqrt a-1, \] 这与③的结论矛盾,从而必有 $x_n\geqslant \bigl[\sqrt a\bigr]$ 恒成立。 接下来证明当 $x_{k+1}\geqslant x_k$ 时必有 $x_k\leqslant \bigl[\sqrt a\bigr]$,若不然,假设 $x_k>\bigl[\sqrt a\bigr]$,由于 $x_k$ 和 $\bigl[\sqrt a\bigr]$ 均为整数,因此有 \[ x_k\geqslant \bigl[\sqrt a\bigr]+1 > \sqrt a, \] 而 \[ x_{k+1}=\left[ \frac{x_k+\left[\frac a{x_k}\right]}2 \right]\leqslant \frac{x_k+\left[\frac a{x_k}\right]}2 \leqslant \frac{x_k+\frac a{x_k}}2, \] 于是有 \[ 2(x_{k+1}-x_k)\leqslant \frac a{x_k}-x_k = \frac{a-x_k^2}{x_k} < 0, \] 矛盾,所以当 $x_{k+1}\geqslant x_k$ 时必有 $x_k\leqslant \bigl[\sqrt a\bigr]$,再由 $x_n\geqslant \bigl[\sqrt a\bigr]$ 恒成立,我们便得到 $x_k=\bigl[\sqrt a\bigr]$。 ______________________ 本题及解答已被我选入《数学空间》总第9期的《对2012年的几道高考数学题的解答及研究》一文中。
blownant 25# 2012-6-10 13:12
thank kuing的解答,学习了 弱弱的问一下,江西省理数第10题怎么理解体积的导数就是截面面积呀
kuing 26# 2012-6-10 13:39
thank kuing的解答,学习了 弱弱的问一下,江西省理数第10题怎么理解体积的导数就是截面面积呀 blownant 发表于 2012-6-10 13:12 通俗说是 $\Delta V \approx S \Delta x$ 用数学语言来说是 $\Delta V = S \Delta x + o(\Delta x)  (\Delta x \to 0)$
谷涵 27# 2012-6-10 14:34
本帖最后由 谷涵 于 2012-6-10 14:35 编辑 四、山东理数12 依题意知关于 $x$ 的多项式 \[ h(x) = ax^3+bx^2-1 \] 有二重根,不妨设二重根为 $x_1$,另 ... kuing 发表于 2012-6-7 23:57 此题,我有一解,您看充分不,可以再加以改善不?
kuing 28# 2012-6-10 14:43
图象虽然是显然的,而且在实际考试中也很好用,不过总是欠缺一点严格,所以对于我这种远离考试者来说,还是喜欢我上面的代数解法,况且也并不麻烦,而且还得到了两根的具体表达式。 PS、本论坛可以直接上传图片,也不会被缩小,故不必在别的地方链接图片过来,外链图片很容易因过期之类的原因而丢失。
╰☆ヾo.海x 29# 2012-6-10 22:43
顶!!!
kuing 30# 2012-6-11 12:35
更正了一下7# 被网友提醒,应为 $V ' (x) = -S(x)$,原先漏掉了负号。
kuing 31# 2012-6-11 12:37
通俗说是 $\Delta V \approx S \Delta x$ 用数学语言来说是 $\Delta V = S \Delta x + o(\Delta x)  (\Delta x \to 0)$ kuing 发表于 2012-6-10 13:39 更正一下,由于计算的是下面部分的体积,而 $x$ 是自上而下的,所以两个 $\Delta$ 是反号的,所以应该是 $\Delta V \approx -S \Delta x$ 或 $\Delta V = -S \Delta x + o(\Delta x)  (\Delta x \to 0)$
XDZM 32# 2012-6-13 18:06
每年高考结束都会关注kuing的play-play,希望将来kuing可以出本书! 安徽理数10:本质应该是 k6完全图去两条边! 江西理科7: 矩形ABCD有性质:OA^2+OC^2=OB^2+OD^2(O为平面内任意点),补成矩形即可! 四川卷那两个12.  两个函数都是中心对称的,对称中心分别为(3,2)和(π/2,π)
kuing 33# 2012-6-13 20:46
每年高考结束都会关注kuing的play-play,希望将来kuing可以出本书! 安徽理数10:本质应该是 k6完全图去两条边! 江西理科7: 矩形ABCD有性质:OA^2+OC^2=OB^2+OD^2(O为平面内任意点),补成矩形即可! 四川卷那两个12.  两个函数都是中心对称的,对称中心分别为(3,2)和(π/2,π) XDZM 发表于 2012-6-13 18:06 出书?我写作水平太差了,不太可能出得了书,自己编着玩倒是可以,出版社估计不会鸟我的。 那个图论不懂, 那个矩形那个,“平面内”可以去掉的,我在数学空间也用过这性质(第4期P10引理2.2.1) 那个就是因为是中心对称所以才去构造那个g(x)。
kuing 34# 2012-6-13 21:17
续楼上:还要单调
q85669551 35# 2012-6-26 20:28
24# kuing 我是被这个题彻底打败了。。考试的时候绝对做不出来,可能还影响后面的心情。。命题2 我想直接搞一般形式证明发散失败,取个特殊值不太恰当,总成了收敛的。。。
yayaweha 36# 2012-7-21 17:03
KK请教11楼那题
yayaweha 37# 2012-7-21 17:05
本帖最后由 yayaweha 于 2012-7-21 17:06 编辑 你怎么想的,把$a_{n+1}$带进去,再来4项一起求和
kuing 38# 2012-7-21 17:06
因为这样就会没了(-1)^k,求和方便,而且60被4整除
yayaweha 39# 2012-7-21 17:07
38# kuing 60也可被2, 6整除
yayaweha 40# 2012-7-21 17:10
本帖最后由 yayaweha 于 2012-7-21 17:15 编辑 消$(-1)^k$应该不止一种方法,能不能直接令$n=2k $或$4k$消
thread-500-3-3.html:
kuing 41# 2012-7-21 17:14
38# kuing 60也可被2, 6整除 yayaweha 发表于 2012-7-21 17:07 两个就有 (-1)^...,四个就会消去,六个就又会有 (-1)^..,所以当然考虑4个而不考虑2或6了。 理论上,两个两个求和当然也可以,分奇偶什么的就行,我只是懒得分,为了方便才考虑4个。
yayaweha 42# 2012-7-21 17:16
明白了,那做这种题的一般做法是怎么样
kuing 43# 2012-7-21 17:22
不知道,我只能说随机应变,常规的那些我都在网刊整理过了……
yayaweha 44# 2012-7-21 17:23
当然,高手都是随机应变
kuing 45# 2012-8-31 16:28
补了一下图,不连去人教了,老是上不了。
yayaweha 46# 2013-3-14 23:46
22# kuing 这个怎么一下子看出$a_3=\frac{π}{2}$
kuing 47# 2013-3-15 00:47
46# yayaweha 对称性
yayaweha 48# 2013-3-15 23:02
47# kuing 能讲具体点吗?
kuing 49# 2013-3-15 23:07
48# yayaweha \[g\left(a_1-\frac\pi2\right)+g\left(a_2-\frac\pi2\right)+\cdots+g\left(a_5-\frac\pi2\right)=0\] 里面等差,由对称性,一边正一边负中间为0时显然符合。
thread-505-1-7.html: [不等式] [原创]08江西卷压轴不等式的清爽解法~
xiaoyu.watson 1# 2012-6-8 13:46
本帖最后由 xiaoyu.watson 于 2012-6-8 13:48 编辑 首先我没学过极限,跟着感觉走的 可能有些(很多)不严谨的地方 所以大家 强烈批评指正!!! 之前看坛子里讨论过。。。着实吓人。。
xiaoyu.watson 2# 2012-6-8 13:47
图片那?????
xiaoyu.watson 3# 2012-6-8 13:49
不会打。。。。。 估计又要麻烦kuing了~。~ pep上还没资格发帖 谁有兴趣就发过去吧
xiaoyu.watson 4# 2012-6-8 13:53
其实好多步都是跟着蒙的 上来就求导函数等于0,解x。其实并没有解,根据第一问蒙出来的 单调性也是根据第一问蒙的 好像都蒙对了。。 整个过程10多分钟吧 不知能得几分 嘿嘿~。~
kuing 5# 2012-6-8 14:01
a<2 时 f(x) 先增后减再增
xiaoyu.watson 6# 2012-6-8 14:08
哪里? 我怎么算的a是多少都先增后减? 求指点
kuing 7# 2012-6-8 14:31
因为你解那个导数为0的方程时漏解 你可以试试代入a=1算算 或者用软件作图看看
thread-507-1-7.html: [不等式] 一个不等式证明题
╰☆ヾo.海x 1# 2012-6-11 02:06
是我们班一个同学自己写的题目...
kuing 2# 2012-6-11 22:35
左 $<\ln3<$ 右 不过证 $<\ln3$ 的话还是涉及点高等的东西,故此可以改用柯西证 左 $<\dfrac2{\sqrt3}<\dfrac43<$ 右 总之这题很……
╰☆ヾo.海x 3# 2012-6-17 00:19
2# kuing 很什么啊?。。。额...他自己说的解法是“前面分母放缩成n+1,后面放缩成等比数列”“前面小于2n/n+1,后面每项换成2的N次项”“就是把2(n-1)小于2的(n-1)次”
kuing 4# 2012-6-17 00:40
2# kuing 很什么啊?。。。 ╰☆ヾo.海x 发表于 2012-6-17 00:19 我上面的回贴表明这个不等式能在中间很容易找到常数将两边分界,而且其实两边不是一个数量级的(左边收敛右边发散),不等式很松,可以尽情放缩,这样一来通常就没什么难度可言了。
kuing 5# 2012-6-17 00:46
现在市面上像这种两边都含n的数列不等式很多都是两边都发散,所以无法像上面那样找到常数去分开两边,或者两边收敛也常常收敛到同一个常数,所以不是那么容易放缩,稍不小心就放过头了
yayaweha 6# 2012-6-17 10:22
2# kuing 用柯西怎么证? 再问下用定积分放缩可否?
kuing 7# 2012-6-17 10:41
6# yayaweha \begin{align*} \left(\frac1{n+1}+\frac1{n+2}+\cdots+\frac1{3n}\right)^2&<(1+1+\cdots+1)\left(\frac1{(n+1)^2}+\frac1{(n+2)^2}+\cdots+\frac1{(3n)^2}\right)\\ &<(1+1+\cdots+1)\left(\frac1{n(n+1)}+\frac1{(n+1)(n+2)}+\cdots+\frac1{(3n-1)(3n)}\right)\\ &=2n\left(\frac1n-\frac1{3n}\right)\\ &=\frac43. \end{align*}
yayaweha 8# 2012-6-17 11:03
6# yayaweha \begin{align*} \left(\frac1{n+1}+\frac1{n+2}+\cdots+\frac1{3n}\right)^2& kuing 发表于 2012-6-17 10:41 这边用定积分也行吧 ln3n-lnn=ln3
yayaweha 9# 2012-6-17 11:04
7# kuing KK着用定积分放缩就是ln3n-lnn=ln3  但是右边呢?
kuing 10# 2012-6-17 11:06
嗯,积分是可以的 但估计楼主不一定懂 柯西应该好接受些
kuing 11# 2012-6-17 11:08
9# yayaweha 右边这么大,不用搞什么放缩啦
yayaweha 12# 2012-6-17 11:12
我是看了数学空间3学到的
yayaweha 13# 2012-6-17 11:13
怎么看不等式强弱
kuing 14# 2012-6-17 11:19
右边只要前两项就已经大于左边了,后面的完全可以不要,而且不要的那些其实还是发散的东西,所以这个不等式很弱……
yayaweha 15# 2012-6-17 11:20
我问的不是这题 是一般怎么看
kuing 16# 2012-6-17 11:30
那就具体情况具体分析了,我不擅长总结这样的东西。
都市侠影 17# 2012-6-18 13:53
本帖最后由 都市侠影 于 2012-6-18 14:00 编辑 用数学归纳法吧,$n=2$是明显成立的吧,假定 \[ \frac{1}{n+1}+\frac{1}{n+1}+\cdots+\frac{1}{3n}<1+\frac{1}{2}+\frac{1}{4}+\cdots+\frac{1}{2(n-1)} \] 那么在$n$换为$n+1$时,左边增加的量是 \[ \frac{1}{3n+1}+\frac{1}{3n+2}+\frac{1}{3n+3}-\frac{1}{n+1} \] 右边增加的量是 \[ \frac{1}{2n} \] 所以只要证明左边的增加量小于右边的增加量就可以了 \[ \frac{1}{3n+1}+\frac{1}{3n+2}+\frac{1}{3n+3}-\frac{1}{n+1}<\frac{1}{2n} \] 证明如下: \begin{align} \frac{1}{3n+1}+\frac{1}{3n+2}+\frac{1}{3n+3}-\frac{1}{n+1}&<\frac{3}{3n+1}-\frac{1}{n+1} \\ &=\frac{2}{(n+1)(3n+1)} \\ &<\frac{2}{2\sqrt{n}\cdot 2\sqrt{3n}} \\ &=\frac{1}{2\sqrt{3}n} \\ &<\frac{1}{2n} \end{align} 证毕。
都市侠影 18# 2012-6-19 08:37
13# yayaweha 所谓不等式的强弱,是指你证出来的不等式与要证的不等式相比,谁是谁的充分条件。 一般而言,你证出来的不等式必须是要证不等式的充分条件,这样证明才能成功,这时你证出来的不等式就是更强的不等式。 举个例,证明不等式 \[ \sum_{k=1}^{n}\frac{1}{k^2}<2 \] 显然你放缩得出来的是 \[ \sum_{k=1}^{n}\frac{1}{k^2}<2-\frac{1}{n} \] 那么自然是你证出来的是更强的不等式。
叶剑飞Victor 19# 2012-8-9 17:20
本帖最后由 叶剑飞Victor 于 2012-8-10 00:01 编辑 求证:$ \displaystyle \frac{1}{n+1}+\frac{1}{n+2}+\cdots+\frac{1}{3n}<1+\frac{1}{2}+\frac{1}{6}+\cdots+\frac{1}{2(n-1)} $ 证明:分类讨论 (1)当$n=2$时, 左边 $ \displaystyle =\frac{1}{3}+\frac{1}{4}+\frac{1}{5}+\frac{1}{6}=\frac{19}{20}<1 $ 右边 $ \displaystyle =1+\frac{1}{2}=\frac{3}{2}>1 $ $\therefore$ 左边<右边 (2)当$n=3$时, 左边 $ \displaystyle = \frac{1}{4}+\frac{1}{5}+\frac{1}{6}+\frac{1}{7}+\frac{1}{8}+\frac{1}{9}<\frac{1}{4}\times 6=\frac{3}{2} $ 右边 $ \displaystyle = 1+\frac{1}{2}+\frac{1}{4}=\frac{7}{4} $ $\therefore$ 左边<右边 (3)当$n=4$时, 左边 $ \displaystyle = \frac{1}{5}+\frac{1}{6}+\frac{1}{7}+\frac{1}{8}+\frac{1}{9}+\frac{1}{10}+\frac{1}{11}+\frac{1}{12}<\frac{1}{5}\times 8=\frac{8}{5} $ 右边 $ \displaystyle = 1+\frac{1}{2}+\frac{1}{4}+\frac{1}{6}=\frac{23}{12} $ $\therefore$ 左边<右边 (4)当$ n\geqslant 5$时, 左边 $ \displaystyle = \frac{1}{n+1}+\frac{1}{n+2}+ \cdots +\frac{1}{3n}< \frac{1}{n+1}\times 2n = \frac{2(n+1)-2}{n+1} = 2 - \frac{2}{n+1} < 2 $ 右边 $ \displaystyle = 1+\frac{1}{2}+\frac{1}{4}+\frac{1}{6}+\cdots+\frac{1}{2(n-1)} \geqslant  1+\frac{1}{2}+\frac{1}{4}+\frac{1}{6}+\frac{1}{8}=\frac{49}{24}>2 $ $\therefore$ 左边<右边 综上所述,左边<右边
thread-508-1-1.html: 表格测试
kuing 1# 2012-6-14 14:54
\[ \begin{tabular}{|l|c|r|r|}   \hline   kui & ng & oh & f... \\   \hline   asdf & 123 & $a^2+b^2$ & $\sum$ \\   \hline   $\frac12 & \sqrt{x} & \sqrt{\frac x{y+z}} & $\dfrac12$ \\   \hline \end{tabular} \] \begin{tabular}{|l|c|r|r|} \hline kui & ng & oh & f... \\ \hline asdf & 123 & 123 & 4567 \\ \hline \end{tabular} \[ \begin{tabular}{|l|c|r|r|} \hline kui & ng & oh & f... \\ \hline asdfadsf & 1234 & 1234 & 45678 \\ \hline \end{tabular} \] \begin{tabular}{|l|c|r|r|} \hline kui & ng & oh & f... \\ \hline asdfadsf & 1234 & 1234 & 45678 \\ \hline \end{tabular} \[ \begin{array}{|l|c|r|r|} \hline kui & ng & oh & f... \\ \hline asdfadsf & 1234 & 1234 & 45678 \\ \hline \end{array} \] \[ \left(\begin{array}{c|cc} 1&2&3\\ \hline 4&5&6\\ \end{array}\right) \]
kuing 2# 2012-6-14 18:05
结果表明这里用不了 tabular,但 array 可以。
thread-51-1-2.html: 太阳神
kuing 1# 2011-10-4 16:04
没错,这个论坛目前的级别星星设置我全部都用太阳了,我喜欢太阳,好多的太阳:loveliness:就像我在人教论坛那堆在线时间太阳那样:victory: (参见http://bbs.pep.com.cn/thread-1174727-1-1.html)
图图 2# 2011-10-4 16:10
1# kuing
kuing 3# 2011-10-4 18:09
原来可以用soso表情
图图 4# 2011-10-4 18:57
3# kuing 哇咔咔
kuing 5# 2011-10-4 21:13
PS:上面这表情不是在soso表情里搜的
thread-52-1-1.html: 注册即可发附件
kuing 1# 2011-10-4 16:19
权限我尽可能开放了 但是附件大小有限制,每个附件不超过512K,这是5d6d官方所限,我也没办法嘀
pxchg1200 2# 2011-11-9 15:55
1# kuing 512KB,太小了吧,稍微大点的PDF都不行。。。
kuing 3# 2011-11-9 16:03
2# pxchg1200 将就下咯,有代码直接打公式已经不错了,所以这里无法做大的。
海盗船长 4# 2012-1-14 22:19
本帖最后由 海盗船长 于 2012-1-14 22:20 编辑 我觉得能发图就够了! PDF一般都是资料什么的。太大的可以发链接
最初的梦想 5# 2012-3-13 16:49
管理员,刚发现我把邮箱写错了,该怎么更正啊?
kuing 6# 2012-3-13 16:58
5# 最初的梦想 这个应该改不了,我也不能……
最初的梦想 7# 2012-3-13 17:06
6# kuing 喵了个咪唔
kuing 8# 2012-3-13 18:06
7# 最初的梦想 改用户名倒是可以(我来改) 所以如果不嫌麻烦,可以先将你现在这个填错邮箱的号改一个名字,然后你用正确的邮箱重新注册一个号,就可以跟用你现在的名字……
最初的梦想 9# 2012-3-15 15:05
8# kuing 那里的邮箱填错会有什么影响么?
kuing 10# 2012-3-15 15:29
其实没什么影响,至少在这里,不会有什么问题…………
最初的梦想 11# 2012-3-15 16:32
10# kuing 既然没什么影响,那就不必烦劳管理员了 谢谢管理员这么有爱的耐心解答
thread-522-1-7.html: 学习你解的题
scgyydf 1# 2012-6-14 16:59
附件 ①对,直接列出来即知正确,事实上,后面的都是 2; ②错,当 $a=3$ 时 $\{x_n\}$ 为 $\{3,2,1,2,1,2,1,\ldots\}$ 后面一直是 2, 1 循环; ③对,由于对任意整数 $n$,容易证明恒有 \[ \left[ \frac n2 \right]\geqslant \frac{n-1}2,\] 又显然 $x_n+[a/x_n]$ 为整数,于是,由 $[x]>x-1$ 以及均值不等式,我们有 \[ x_{n+1}=\left[ \frac{x_n+\left[\frac a{x_n}\right]}2 \right]\geqslant \frac{x_n+\left[\frac a{x_n}\right]-1}2 > \frac{x_n+\frac a{x_n}-2}2\geqslant \sqrt a-1. \] 所以命题成立; ④对,我们先来证明 $x_n\geqslant \bigl[\sqrt a\bigr]$ 恒成立,若不然,假设存在某个 $x_k<\bigl[\sqrt a\bigr]$,由于 $x_k$ 和 $\bigl[\sqrt a\bigr]$ 均为整数,因此有 \[ x_k\leqslant \bigl[\sqrt a\bigr]-1\leqslant \sqrt a-1, \] 这与③的结论矛盾,从而必有 $x_n\geqslant \bigl[\sqrt a\bigr]$ 恒成立。 接下来证明当 $x_{k+1}\geqslant x_k$ 时必有 $x_k\leqslant \bigl[\sqrt a\bigr]$,若不然,假设 $x_k>\bigl[\sqrt a\bigr]$,由于 $x_k$ 和 $\bigl[\sqrt a\bigr]$ 均为整数,因此有 \[ x_k\geqslant \bigl[\sqrt a\bigr]+1 > \sqrt a, \] 而 \[ x_{k+1}=\left[ \frac{x_k+\left[\frac a{x_k}\right]}2 \right]\leqslant \frac{x_k+\left[\frac a{x_k}\right]}2 \leqslant \frac{x_k+\frac a{x_k}}2, \] 于是有 \[ 2(x_{k+1}-x_k)\leqslant \frac a{x_k}-x_k = \frac{a-x_k^2}{x_k} < 0, \] 矛盾,所以当 $x_{k+1}\geqslant x_k$ 时必有 $x_k\leqslant \bigl[\sqrt a\bigr]$,再由 $x_n\geqslant \bigl[\sqrt a\bigr]$ 恒成立,我们便得到 $x_k=\bigl[\sqrt a\bigr]$。 ①对,直接列出来即知正确,事实上,后面的都是 2; ②错,当 $a=3$ 时 $\{x_n\}$ 为 $\{3,2,1,2,1,2,1,\ldots\}$ 后面一直是 2, 1 循环; ③对,由于对任意整数 $n$,容易证明恒有 \[ \left[ \frac n2 \right]\geqslant \frac{n-1}2,\] 又显然 $x_n+[a/x_n]$ 为整数,于是,由 $[x]>x-1$ 以及均值不等式,我们有 \[ x_{n+1}=\left[ \frac{x_n+\left[\frac a{x_n}\right]}2 \right]\geqslant \frac{x_n+\left[\frac a{x_n}\right]-1}2 > \frac{x_n+\frac a{x_n}-2}2\geqslant \sqrt a-1. \] 所以命题成立; ④对,我们先来证明 $x_n\geqslant \bigl[\sqrt a\bigr]$ 恒成立,若不然,假设存在某个 $x_k<\bigl[\sqrt a\bigr]$,由于 $x_k$ 和 $\bigl[\sqrt a\bigr]$ 均为整数,因此有 \[ x_k\leqslant \bigl[\sqrt a\bigr]-1\leqslant \sqrt a-1, \] 这与③的结论矛盾,从而必有 $x_n\geqslant \bigl[\sqrt a\bigr]$ 恒成立。 接下来证明当 $x_{k+1}\geqslant x_k$ 时必有 $x_k\leqslant \bigl[\sqrt a\bigr]$,若不然,假设 $x_k>\bigl[\sqrt a\bigr]$,由于 $x_k$ 和 $\bigl[\sqrt a\bigr]$ 均为整数,因此有 \[ x_k\geqslant \bigl[\sqrt a\bigr]+1 > \sqrt a, \] 而 \[ x_{k+1}=\left[ \frac{x_k+\left[\frac a{x_k}\right]}2 \right]\leqslant \frac{x_k+\left[\frac a{x_k}\right]}2 \leqslant \frac{x_k+\frac a{x_k}}2, \] 于是有 \[ 2(x_{k+1}-x_k)\leqslant \frac a{x_k}-x_k = \frac{a-x_k^2}{x_k} < 0, \] 矛盾,所以当 $x_{k+1}\geqslant x_k$ 时必有 $x_k\leqslant \bigl[\sqrt a\bigr]$,再由 $x_n\geqslant \bigl[\sqrt a\bigr]$ 恒成立,我们便得到 $x_k=\bigl[\sqrt a\bigr]$。 附件
kuing 2# 2012-6-14 17:24
我在 http://kkkkuingggg.5d6d.net/view ... &page=3#pid2660 里也贴过了……
scgyydf 3# 2012-6-15 15:21
2# kuing 就是你的解答,那里是代码,看不清,在这里来学习
scgyydf 4# 2012-6-15 15:21
3# scgyydf ①对,反证一下,假设 $a-b\geqslant 1$,则 $a^2-b^2=(a+b)(a-b)\geqslant a+b>a-b\geqslant 1$,矛盾。 ②错,反例取 $a=2$, $b=2/3$; ③错,事实上,不等式反向成立,即当 $\bigl|\sqrt a - \sqrt b\bigr|=1$ 时有 $|a-b|>1$,我们也用反证法证一下。 假设 $|a-b|\leqslant 1$,则 \[\bigl|\sqrt a - \sqrt b\bigr|=\frac{|a-b|}{\sqrt a+\sqrt b}\leqslant \frac1{\sqrt a+\sqrt b} < \frac1{\bigl|\sqrt a - \sqrt b\bigr|},\] 从而得到 $\bigl|\sqrt a - \sqrt b\bigr|<1$,矛盾; ④对,由 $1=|a^3-b^3|=|a-b|(a^2+ab+b^2)>|a-b|(a^2-2ab+b^2)=|a-b|^3$ 即得 $|a-b|<1$
kuing 5# 2012-6-15 15:34
2# kuing 就是你的解答,那里是代码,看不清,在这里来学习 scgyydf 发表于 2012-6-15 15:21 我二楼的链接不是人教论坛的那个,就是这里的,我已经贴过过来这里了
thread-523-1-1.html: asdf
kuing 1# 2012-6-16 13:30
\[\left( \sum_{k=1}^n a_k b_k \right)^2 \leqslant \left( \sum_{k=1}^n a_k^2 \right) \left( \sum_{k=1}^n b_k^2 \right)\] \[\biggl( \sum_{k=1}^n a_k b_k \biggr)^2 \leqslant \biggl( \sum_{k=1}^n a_k^2 \biggr) \biggl( \sum_{k=1}^n b_k^2 \biggr)\] \[\Bigl( \sum_{k=1}^n a_k b_k \Bigr)^2 \leqslant \Bigl( \sum_{k=1}^n a_k^2 \Bigr) \Bigl( \sum_{k=1}^n b_k^2 \Bigr)\] \[\left( \sum_{k=1}^n a_k b_k \right)^2 \leqslant \left( \sum_{k=1}^n a_k^2 \right) \left( \sum_{k=1}^n b_k^2 \right)\] \[\biggl( \sum_{k=1}^n a_k b_k \biggr)^2 \leqslant \biggl( \sum_{k=1}^n a_k^2 \biggr) \biggl( \sum_{k=1}^n b_k^2 \biggr)\] \begin{gather} \Bigl( \sum_{k=1}^n a_k b_k \Bigr)^2 \leqslant \Bigl( \sum_{k=1}^n a_k^2 \Bigr) \Bigl( \sum_{k=1}^n b_k^2 \Bigr)\\ \lim_{n\to\infty}\sum_{k=1}^n\frac1{k^2}=\frac{\pi^2}6\\ e^{i\pi}+1=0 \end{gather}
kuing 2# 2012-6-16 18:14
\[\frac{a^2+b^2}{a+b}\geqslant\sqrt{\frac{a^2+b^2}2}\geqslant\frac{a+b}2\geqslant\sqrt{ab}\geqslant\frac2{\frac1a+\frac1b}\]
thread-524-1-7.html: [组合] 5人7项目
realnumber 1# 2012-6-16 17:22
本帖最后由 realnumber 于 2012-6-18 08:57 编辑 5个人报名参加7项运动项目,必须满足条件P“每个项目至少1人且每人至少报名1项”,则有几种不同情况? 方法一如下: 设$f(m,n)$表示满足条件P的m人,n个项目的数目,显然$f(1,n)=f(m,1)=1$,$f(m,n)=f(n,m)$. $(2^{m}-1)C^{m}_{m}f(m,n-1)+2^{m-1}C^{m-1}_{m}f(m-1,n-1)+...+2C^{1}_{m}f(1,n-1)=f(m,n)$-----(1) 穷举法得到一般有$f(n,2)=f(2,n)=3^n-2$, 利用上述递推公式和穷举法验证了$f(2,2)=7$,$f(2,3)=f(3,2)=25$,$f(3,3)=265$, 以下都是递推公式得到,但愿没算错, $f(4,2)=79,f(4,3)=2161,f(4,4)=41503$, $f(5,2)=241$,$f(5,3)=16081$,$f(5,4)=693691$, $f(6,2)=727$,$f(6,3)=115465$,$f(6,4)=10924399$, $f(7,2)=2185$.$f(7,3)=816985$,$f(7,4)=167578321$ $f(7,5)=(2^{7}-1)C^{7}_{7}f(7,4)+2^{6}C^{6}_{7}f(6,4)+2^{5}C^{5}_{7}f(5,4)+...+2^{2}C^{2}_{7}f(2,4)+2C^{1}_{7}f(1,4)=26666591281$ 方法二:(逐步淘汰原理) $f(m,n)=f(n,m)=(2^n-1)^m-C^{n-1}_n(2^{n-1}-1)^m+C^{n-2}_n(2^{n-2}-1)^m-C^{n-3}_n(2^{n-3}-1)^m+....$---(2) 那么$f(7,5)=(2^5-1)^7-C^{4}_5(2^{4}-1)^7+C^{3}_5(2^{3}-1)^7-C^{2}_5(2^{2}-1)^7+C^1_5(2-1)^7=26666530801$ 计算错了?还是方法问题? 解释: (1)左边第一项意思是n-1个项目,添一项,这项人数可以为1,2,...,或m;第二项的意思是,添一项,这一项必须有参加0项的1人,再可以添加人数为0,1,2,...或m-1;第三项意思,添一项必须有参加0项的2人,再可以加0,1,2,...,或m-2人;... (2)右边第一项意思是每项目至少1人(但不满足每人至少1项目),这就需要减去其中仅n-1人参加m个项目,(每项目至少1人),继续需要....
kuing 2# 2012-6-16 17:41
前几天我也在某群见有人问过,好像是5个人报7个项目 表示不会……
kuing 3# 2012-6-17 20:48
仍然 ing
realnumber 4# 2012-6-17 21:26
本帖最后由 realnumber 于 2012-6-17 21:42 编辑 我表达能力太差,其实能看到kk这个表情也不错1楼加了注释,希望好点
realnumber 5# 2012-6-18 12:07
本帖最后由 realnumber 于 2012-6-18 12:16 编辑 是西西写的 严老师用math**检验了
kuing 6# 2012-6-18 13:25
mathematica
thread-525-1-1.html: [不等式] 来自pep的几道简单最小值题
kuing 1# 2012-6-16 20:37
原贴链接:http://bbs.pep.com.cn/thread-2558631-1-1.html 一楼楼主: 求不等式的最小值(应该属于竞赛级别的吧) 1.设x,y,z>0,且满足xy+yz+zx=1。求21(x^2+y^2)+z^2的最小值。 2.设x,y,z>0,且满足xy+yz+zx=1。求3x^2+24y^2+z^2的最小值。 3.设实数a,b,c,d 满足ab+bc+cd+da=1。求5x^2+4y^2+5z^2+t^2的最小值。 4.设x,y,z>0,且满足x+y+z=14。求x^2+y^2+z^3的最小值。 三楼我回复: 1、6 \begin{align*} 21(x^2+y^2)+z^2 & = 21(x^2+y^2)+\left( \frac{1-xy}{x+y} \right)^2 \\ & \geqslant \frac{21}2(x+y)^2+\left( \frac{1-\frac{(x+y)^2}4}{x+y} \right)^2 \\ & =\frac{169(x+y)^2}{16}+\frac1{(x+y)^2}-\frac12 \\ & \geqslant 2\sqrt{\frac{169}{16}}-\frac12 \\ & =6 \end{align*} 当$x=y=\frac1{\sqrt{13}}$, $z=\frac6{\sqrt{13}}$时取等; 2、涉及三次方程; 3、字母打错了,后面如果改成$5a^2+4b^2+5c^2+d^2$的话就是$2\sqrt2$ \begin{align*} 5a^2+4b^2+5c^2+d^2 &= 5(a^2+c^2)+\frac45\left( \frac14+1 \right)(4b^2+d^2) \\ & \geqslant \frac52(a+c)^2+\frac45(b+d)^2 \\ & \geqslant 2\sqrt{2(a+c)(b+d)} \\ & =2\sqrt{2(ab+bc+cd+da)} \\ & =2\sqrt2 \end{align*} 当$a=c=\frac1{\sqrt[4]{50}}$, $b=\frac1{\sqrt[4]{200}}$, $d=\sqrt[4]{\frac{32}{25}}$时取等; 4、80 \begin{align*} x^2+y^2+z^3 &\geqslant \frac12(x+y)^2+z^3 \\ & =\frac12(14-z)^2+z^3 \\ & =\frac12(z-2)^2(9+2z)+80 \\ & \geqslant 80 \end{align*} 当$x=y=6$, $z=2$时取等。
thread-525-1-7.html: [不等式] 来自pep的几道简单最小值题
kuing 1# 2012-6-16 20:37
原贴链接:http://bbs.pep.com.cn/thread-2558631-1-1.html 一楼楼主: 求不等式的最小值(应该属于竞赛级别的吧) 1.设x,y,z>0,且满足xy+yz+zx=1。求21(x^2+y^2)+z^2的最小值。 2.设x,y,z>0,且满足xy+yz+zx=1。求3x^2+24y^2+z^2的最小值。 3.设实数a,b,c,d 满足ab+bc+cd+da=1。求5x^2+4y^2+5z^2+t^2的最小值。 4.设x,y,z>0,且满足x+y+z=14。求x^2+y^2+z^3的最小值。 三楼我回复: 1、6 \begin{align*} 21(x^2+y^2)+z^2 & = 21(x^2+y^2)+\left( \frac{1-xy}{x+y} \right)^2 \\ & \geqslant \frac{21}2(x+y)^2+\left( \frac{1-\frac{(x+y)^2}4}{x+y} \right)^2 \\ & =\frac{169(x+y)^2}{16}+\frac1{(x+y)^2}-\frac12 \\ & \geqslant 2\sqrt{\frac{169}{16}}-\frac12 \\ & =6 \end{align*} 当$x=y=\frac1{\sqrt{13}}$, $z=\frac6{\sqrt{13}}$时取等; 2、涉及三次方程; 3、字母打错了,后面如果改成$5a^2+4b^2+5c^2+d^2$的话就是$2\sqrt2$ \begin{align*} 5a^2+4b^2+5c^2+d^2 &= 5(a^2+c^2)+\frac45\left( \frac14+1 \right)(4b^2+d^2) \\ & \geqslant \frac52(a+c)^2+\frac45(b+d)^2 \\ & \geqslant 2\sqrt{2(a+c)(b+d)} \\ & =2\sqrt{2(ab+bc+cd+da)} \\ & =2\sqrt2 \end{align*} 当$a=c=\frac1{\sqrt[4]{50}}$, $b=\frac1{\sqrt[4]{200}}$, $d=\sqrt[4]{\frac{32}{25}}$时取等; 4、80 \begin{align*} x^2+y^2+z^3 &\geqslant \frac12(x+y)^2+z^3 \\ & =\frac12(14-z)^2+z^3 \\ & =\frac12(z-2)^2(9+2z)+80 \\ & \geqslant 80 \end{align*} 当$x=y=6$, $z=2$时取等。
thread-526-1-1.html: 看看
hhhzh7241hzh 1# 2012-6-17 00:16
\begin{align*} x^2+y^2+z^3 &\geqslant \frac12(x+y)^2+z^3 \\ & =\frac12(14-z)^2+z^3 \\ & =\frac12(z-2)^2(9+2z)+80 \\ & \geqslant 80 \end{align*} 当$x=y=6$, $z=2$时取等。
hhhzh7241hzh 2# 2012-6-17 00:19
\begin{align*} 21(x^2+y^2)+z^2 & = 21(x^2+y^2)+\left( \frac{1-xy}{x+y} \right)^2 \\ & \geqslant \frac{21}2(x+y)^2+\left( \frac{1-\frac{(x+y)^2}4}{x+y} \right)^2 \\ & =\frac{169(x+y)^2}{16}+\frac1{(x+y)^2}-\frac12 \\ & \geqslant 2\sqrt{\frac{169}{16}}-\frac12 \\ & =6 \end{align*} 当$x=y=\frac1{\sqrt{13}}$, $z=\frac6{\sqrt{13}}$时取等;
hhhzh7241hzh 3# 2012-6-17 00:23
\begin{align*} 5a^2+4b^2+5c^2+d^2 &= 5(a^2+c^2)+\frac45\left( \frac14+1 \right)(4b^2+d^2) \\ & \geqslant \frac52(a+c)^2+\frac45(b+d)^2 \\ & \geqslant 2\sqrt{2(a+c)(b+d)} \\ & =2\sqrt{2(ab+bc+cd+da)} \\ & =2\sqrt2 \end{align*} 当$a=c=\frac1{\sqrt[4]{50}}$, $b=\frac1{\sqrt[4]{200}}$, $d=\sqrt[4]{\frac{32}{25}}$时取等;
kuing 4# 2012-6-17 00:34
其实我八点多的时候在 http://kkkkuingggg.5d6d.net/thread-525-1-1.html 已经发了一份了……
老樊 5# 2012-7-7 18:09
本帖最后由 老樊 于 2012-7-7 18:11 编辑 $f\left( x\right) =\dfrac {e^{x}} {x}+x^{2}+2$
hnsredfox_007 6# 2013-1-29 16:07
本帖最后由 hnsredfox_007 于 2013-1-29 19:16 编辑 解:不妨设$P \left ( \dfrac{y_1^2}{4},y_1\right),Q\left(\dfrac{y_2^2}{4},y_2\right),M\left(\dfrac{y_0^2}{4},y_0\right)$,依题意可得$$\begin{cases}\frac{y_1}{\dfrac{y_1^2}{4}+1}=\frac{y_0}{\dfrac{y_0^2}{4}+1} \\ \frac{y_2+1}{\dfrac{y_2^2}{4}-1}=\frac{y_0+1}{\dfrac{y_0^2}{4}-1}\end{cases},$$ 解得$$\begin{cases}y_1=\frac{4}{y_0}\\ y_2=-\frac{4+y_0}{1+y_0}\end{cases},$$ 于是直线$PQ$的方程为:$$y-y_1=\frac{y_2-y_1}{\dfrac{y_2^2}{4}-\dfrac{y_1^2}{4}}\left(x-\frac{y_1^2}{4}\right),$$ 整理即得:$$y-\dfrac{4}{y_0}=\dfrac{4y_0(1+y_0)}{4-y_0^2}x-\frac{16(1+y_0)}{(4-y_0^2)y_0},$$ 于是$$y=\dfrac{4y_0(1+y_0)}{4-y_0^2}(x-1)-4,$$ 即直线$PQ$过定点$(1,-4)$.
realnumber 7# 2013-2-1 19:43
本帖最后由 realnumber 于 2013-2-3 12:42 编辑 \begin{array}{|c|c|c|c|c|} \hline\\1 & 2 & 3 & 16 & 17 \\ \hline\\ 4 & 5 & 6 & 18 & 19 \\ \hline\\ 7 & 8 & 6 & 20 & 21 \\ \hline\\ 10 & 11 & 12 & 22 & 23 \\ \hline\\ 13 & 14 & 15 & 24 & 25 \\ \hline \end{array} \begin{cases}1=2 \\ 4-67\\8999\end{cases}
thread-527-1-7.html: 来自群的坐位概率
kuing 1# 2012-6-17 02:43
教师-郝酒(3653*****)  23:16:47 有 $n$ 个人,1 至 n 编号,按编号从小到大进电影院。电影院有 $n$ 把椅子,也用 1 至 n 编号。 第 1 个人随便坐;从第 2 个人开始如果自己的座位空着则必须坐在自己座位上,如果自己的座位被前面的人占了则自己可以随便坐。 问最后一个人能坐在自己座位上的概率是多少? 解:设 $n$($n\geqslant 2$)个人时所求概率为 $P(n)$。当 $n=2$ 时,显然最后一个人坐正确的概率是 $P(2)=1/2$,下设 $n\geqslant 3$。 如果 1 号人坐自己的 1 号位,所有人都将坐正确,这件事发生的概率是 $1/n$; 如果 1 号人不坐自己的位置,则分 $n-1$ 种情况讨论。 (1)如果 1 号人坐了 2 号位。 此时即是 2, 3, 4, ..., n 号人面对着 1, 3, 4, ..., n 号位置,由 2 号人开始并且是随便坐的。 而这相当于 2, 3, 4, ..., n 号人面对着 2, 3, 4, ..., n 号位置,并且第 1 个人(即 2 号人)随便坐一样,这是因为对于 3, 4, ..., n 号人来说,那张不是他们的椅子的编号是 1 或 2 对于他们来说是没分别的,而对于 2 号人来说则无论那张椅子的编号是 1 或 2,只要他坐上去,那么其余的人都将坐正确。 因此,此时相当于 $n-1$ 个人进电影院的情形,即此时最后一个人坐在自己位置上的概率是 $P(n-1)$; (2)如果 1 号人坐了 3 号位。 此时 2 号人要坐 2 号位,剩下即是 3, 4, 5, ..., n 号人面对着 1, 4, 5, ..., n 号位置,由 3 号人开始并且是随便坐的。 与(1)一样,这也相当于 $n-2$ 个人进电影院的情形,即此时最后一个人坐在自己位置上的概率是 $P(n-2)$; (3)…… …… ($n-2$)如果 1 号人坐了 n-1 号位。 此时 2~n-2 号人均坐自己的位置,剩下 n-1 和 n 号人面对着 1 和 n 号位置,由 n-1 号人开始并且是随便坐的。相当于 2 个人进电影院的情形,即此时最后一个人坐在自己位置上的概率是 $P(2)$; ($n-1$)如果 1 号人坐了 n 号位。则无论怎么坐最后那位也坐错,即此时最后一个人坐在自己位置上的概率是 0。 综上所述,对于 $n\geqslant 3$ 时,我们有 \[P(n)=\frac1n+\frac1n P(n-1)+\frac1n P(n-2) + \cdots + \frac1n P(2),\] 上式变形有 \begin{align*} n\cdot P(n)&=1+ P(n-1)+P(n-2) + \cdots + P(2),\\ (n+1)\cdot P(n+1) &=1+P(n)+ P(n-1) + \cdots + P(2), \end{align*} 相减得到 \[P(n+1)=P(n),\] 不难验证$P(3)=P(2)=1/2$,所以对于任意 $n\geqslant 2$ 都有 \[P(n)=\frac12,\] 即无论人数多少,最后的人能坐在自己座位上的概率总是 $1/2$。 这个题由我想通了到表达成上述过程,至少用了一个多小时,真的很难表达,现在这样说不知你们能不能看懂?关键在于理解那个“相当于”。
都市侠影 2# 2012-7-22 14:18
排列组合的东东通常都难以表述清楚,当初那个错位排列问题我用数列递推来解决,也是同样的问题,为了得到递推公式,不得不临时引入了一两个概念才简化了叙述,是改了又改,披阅三载,增删五次,当不为过。
thread-528-1-7.html: [几何] 请教KK
yayaweha 1# 2012-6-17 11:31
有什么简便方法 已知椭圆 $x^2/a^2+y^2/b^2=1$($a>b>0$),$O$ 为坐标原点,$P$、$Q$、$R$ 为椭圆上三动点,且 $OP$、$OR$、$OQ$ 两两的夹角相等,求 $1/\abs{OP}^2+1/\abs{OQ}^2+1/\abs{OR}^2$ 的值。
kuing 2# 2012-6-17 11:37
大概还是用 $P(r_1\cos t, r_1\sin t)$,$Q\bigl(r_2\cos(t+120^\circ), r_2\sin(t+120^\circ)\bigr)$,$R\bigl(r_3\cos(t+240^\circ), r_3\sin(t+240^\circ)\bigr)$ 这种代换?
yayaweha 3# 2012-6-17 11:39
用参数方程接下来怎么算
kuing 4# 2012-6-17 11:44
2# 并不是参数方程…… 人教论坛上好像有类似讨论,我先吃饭。
yayaweha 5# 2012-6-17 11:45
这很像参数方程
kuing 6# 2012-6-17 14:00
将所设代入椭圆方程中求得 \begin{align*} \frac1{|OP|^2}&=\frac1{r_1^2}=\frac{\cos^2t}{a^2}+\frac{\sin^2t}{b^2},\\ \frac1{|OQ|^2}&=\frac1{r_2^2}=\frac{\cos^2(t+120^\circ)}{a^2}+\frac{\sin^2(t+120^\circ)}{b^2},\\ \frac1{|OR|^2}&=\frac1{r_3^2}=\frac{\cos^2(t+240^\circ)}{a^2}+\frac{\sin^2(t+240^\circ)}{b^2}, \end{align*} 相加得 \begin{align*} \frac1{|OP|^2}+\frac1{|OQ|^2}+\frac1{|OR|^2} ={}& \frac{\cos^2t+\cos^2(t+120^\circ)+\cos^2(t+240^\circ)}{a^2}\\ &+\frac{\sin^2t+\sin^2(t+120^\circ)+\sin^2(t+240^\circ)}{b^2}, \end{align*} 熟知有恒等式 \[\cos^2t+\cos^2(t+120^\circ)+\cos^2(t+240^\circ) = \sin^2t+\sin^2(t+120^\circ)+\sin^2(t+240^\circ) = \frac32,\] 所以 \[\frac1{|OP|^2}+\frac1{|OQ|^2}+\frac1{|OR|^2} = \frac32\left(\frac1{a^2}+\frac1{b^2}\right).\] 关键还是那个恒等式,故此如无意外下是可以推广的。
yayaweha 7# 2012-6-17 14:30
6# kuing 不知道这个恒等式怎么办,那要手算怎么算?
realnumber 8# 2012-6-17 21:58
是极坐标,高中选修有
thread-529-1-1.html: latex有没可以在word中直接显示
realnumber 1# 2012-6-17 22:24
或类似word的软件,可以有直接看到公式的,类似mathtype效果 ---不至于论坛发下,自己保存只好图片,以后都没法修改
kuing 2# 2012-6-17 22:38
mathtype 其实也部分支持 tex 代码,但很有限。
kuing 3# 2012-6-17 22:40
当然,要先设置一下,具体可见 http://bbs.pep.com.cn/thread-753654-1-1.html 这样你可以将论坛上的代码在mathtype中粘贴,然后插入到word里。 当然,不一定都可以,前面也说了,支持的有限,而且可能会有很多细节问题。
都市侠影 4# 2012-6-22 21:56
怎么不能修改,你在论坛上编辑帖子不就行了。
kuing 5# 2012-6-23 00:59
4# 都市侠影 其实我也不是十分理解楼主的意思   他可能是要将论坛的帖子移植到word? 或者记录的不是自己的帖子?
叶剑飞Victor 6# 2012-8-9 07:53
3# kuing MathJax支持的LaTeX比MathType更有限。
叶剑飞Victor 7# 2012-8-9 07:57
本帖最后由 叶剑飞Victor 于 2012-8-9 07:59 编辑
kuing 8# 2012-8-9 09:53
3# kuing MathJax支持的LaTeX比MathType更有限。 叶剑飞Victor 发表于 2012-8-9 07:53 请举例
叶剑飞Victor 9# 2012-8-9 10:26
本帖最后由 叶剑飞Victor 于 2012-8-9 11:23 编辑 8# kuing $$ \overarc{AB}  $$ 好吧,看来差不多
kuing 10# 2012-8-9 10:29
9# 叶剑飞Victor mathtype 支持这个吗
叶剑飞Victor 11# 2012-8-9 10:32
10# kuing 嘻嘻,似乎也不支持。只能用鼠标点击“弧”的按钮。
kuing 12# 2012-8-9 10:38
\overarc 在真的 LaTeX 中也需要加载 arcs 宏包才能打出来。 mathtype 上那个直接点击的我想大概是用了另外的方式整了一个弧出来(就像还有平行四边形符号,latex中没这个符号他也有),想复制代码出来也会报错,因此也不算是支持latex上的弧的了……
叶剑飞Victor 13# 2012-8-9 10:45
12# kuing 暂时先这样: $$ \overset{\frown}{AB} $$
kuing 14# 2012-8-9 11:24
相反,mathjax 支持一些 latex 的环境,比如用 align 环境写多行公式可以灵活对齐,各种矩阵环境(vmatrix、bmatrix 等 $\begin{Vmatrix} 1&2\\ a&b \end{Vmatrix}$),如果要分块矩阵也可以用 array 环境来添加竖线,$\left(\begin{array}{c|cc} 1&2&3\\ \hline 4&5&6\\ \end{array}\right)$,因此也可以画表了 $\begin{array}{|l|c|r|r|} \hline kui & ng & oh & f... \\ \hline asdfadsf & 1234 & 1234 & 45678 \\ \hline \end{array}$,这些在 mathtype 上就不支持了吧。
叶剑飞Victor 15# 2012-8-9 11:34
本帖最后由 叶剑飞Victor 于 2012-8-9 11:39 编辑 14# kuing MathType支持Vmatrix,不支持横线和竖线。
kuing 16# 2012-8-9 11:46
15# 叶剑飞Victor oh thanks
kuing 17# 2012-8-9 11:53
15# 叶剑飞Victor 再复制出来的代码好像就变成用 array 的了……
叶剑飞Victor 18# 2012-8-9 12:01
17# kuing 是的
叶剑飞Victor 19# 2012-11-4 18:55
本帖最后由 叶剑飞Victor 于 2012-11-4 18:57 编辑 用Aurora插件不错。用的是真正的LaTeX输出结果,以png图片的格式嵌入到Word中。而且LaTeX代码还是可编辑的。 编辑界面: 结果:
isea 20# 2013-3-26 02:16
本帖最后由 isea 于 2013-3-26 02:26 编辑 真心赞,特别是只是以LaTeX代码为辅助数学公式,太美了,完全可以不用鼠标了。 转载石一歌@doudan:Aurora — 一个在 MSOffice 内输入 LaTeX 公式的很好用插件 工具名称:Aurora2x  (下载),分流 压缩包内有详细的安装说明。 刚才测试了一下,确实能够在 Office 02~03 下(2007没有测试——注:偶已测试,没问题)排版一些标准和非标准(指个人习惯)的数学公式,这归功于使用 MiKTeX 编译环境(请看我在回帖中的注明,特别是机器已经安装有 LaTeX 系统,比如 CTeX 套装)的结果。 Word 内使用其排版公式还可以交叉互引以及更新公式编号等若干强悍功能(方便文章修订后的重新排版),PowerPoint 内使用也方便方便。都可以直接拷贝 TeX 文稿中的代码使用。 安装完以后,在 Word 或 PowerPoint 的菜单栏上会添加一个新的 Aurora 选项卡,和公式编辑器的使用方法一样,点击**之后就会弹出一个输入框,输入想要公式的 TeX 代码,然后点击Refresh,Aurora 就会自动在后台调用 MiKTeX 进行编译,并且将输出转换为 png 格式的图片,**当前位置。Aurora 对话框中的设置允许对导言区进行字体更改,其他宏包加载等多项改动,熟悉 LaTeX 写作的不会陌生。 所以推荐一下,方便用 Office 写论文的朋友,另外也推荐给用 TeX 写文章但喜欢使用PPT 演讲的朋友,话说用 Beamer 挂多张图片也是非常痛苦的事啊! Aurora和各系列TeX版本的兼容性 1) Aurora 在 MikTeX 2.7 和 TeXLive 2009 及以下版本下使用无问题,包括 CTeX 2.7 及以下版本; 2) Aurora 在 MiKTeX 2.8 和 TeXLive 2010 版本下会有问题,包括 CTeX 2.8 ba版本。可能是默认输出 PDF 1.5 造成的,详情可见CTeX论坛讨论帖 http://bbs.ctex.org/viewthread.p ... uid=92282#pid390853 ,解决方法是在导言区中添加“\pdfminorversion=3”,路径是在打开的 Aurora 对话框中选“Properties——>Packages”。 3) Aurora 在 MiKTeX 2.9 版本下无问题,包括 CTeX2.9 版本。 总而言之,可以不必安装软件包内自带的 Micro-MiKTeX 了。 其官网:http://elevatorlady.ca/ 在 Aurora 中如何使用中文 对于CTeX 2.8/2.9 而言,在 Aurora中使用中文需要重新配置字体,在默认为TrueType字体下是不能正确输出中文的。 1) 开始菜单—CTeX—FontSetup—勾选上“生成Type1字库”和“使用Type1字库”这两个选项,重新配置字体,完成安装。 2) 在打开的 Aurora 对话框中选“Properties——>Packages”,添加宏包“\usepackge{ctex}”。 3) 在数学环境中使用“\text{中文}”输入中文。 即可正确输出中文。
thread-529-2-1.html:
isea 21# 2013-3-26 15:06
本帖最后由 isea 于 2013-3-26 15:18 编辑 继续给新手一份不错的入门PDF,转载。 这个是快速上手Aurora及LaTeX两软件。 其实,花点时间看看Aurora自带的帮助文档就可以了。 啊,晕了,rar分卷传上来啊,自己看吧: 在Word下实现基于LaTeX的数学公式的快速输入(实际包含显示结果) http://wenku.baidu.com/view/c241a6ec856a561252d36f6e.html http://ishare.iask.sina.com.cn/f/22815696.html
hnsredfox_007 22# 2013-3-26 21:10
20# isea 无法安装ctex2.9,如何使用?我想输入中文啊
thread-53-1-1.html: 论坛表情
kuing 1# 2011-10-4 18:11
刚刚才在后台发现原来可以用soso表情,发贴时点击表情按钮就可以看到,很多选择呢,还不止默认QQ表情
kuing 2# 2011-10-4 18:57
默认QQ表情里有 搜索 衰 有
图图 3# 2011-10-4 18:59
1# kuing 还是喜欢用它
图图 4# 2011-10-4 19:00
2# kuing
kuing 5# 2011-10-4 19:04

kuing 6# 2011-10-4 19:07
竟然这么少正版的小的那种洋葱头表情
kuing 7# 2011-10-4 19:13

kuing 8# 2011-10-5 14:18

戊概念·五 9# 2011-10-30 14:04
默认QQ表情里有 搜索 衰 有 kuing 发表于 2011-10-4 18:57 最后那个看起来不错
kuing 10# 2012-2-17 17:40
有空试试用 tikz 之类的工具画这个大的衰
thread-530-1-1.html: 奇怪,家里上这个论坛快。学校很慢
realnumber 1# 2012-6-17 22:28
但学校是100m光纤什么的,下载有10mb/s 家里就3m的adsl,下载就300kb/s
kuing 2# 2012-6-17 22:36
不清楚了,没接触过校园的网络
isea 3# 2012-6-28 15:05
教育网访问一些网站是这样子,慢。 反过来也有类似的问题。
thread-531-1-7.html: 2012浙江自选模块数学2题
realnumber 1# 2012-6-18 08:50
本帖最后由 realnumber 于 2012-6-18 08:52 编辑 题号:03  科目:数学 数学史与不等式选讲模块(10分) 已知a∈R,设关于x的不等式$│2x-a│+│x+3│≥2x+4$的解集为A. (1)若a=1,求A;(2)若A=R,求a的取值范围. 题号:04  科目:数学 "矩阵与变换和坐标系与参数方程"模块(10分) 在直角坐标系xOy中,设倾斜角为α的直线l:$x=2+tcosα$,且$y=√3+tsinα$(t为参数)与曲线C: $x=2cosθ$,且$y=sinθ$(θ为参数) 相交于不同两点A,B. (1)若$α=π/3$,求线段AB中点M的坐标; (2)若$│PA│·│PB│=│OP│^2$,其中P(2,√3),求直线l的斜率. ---就给大家看看,不用做的,我有答案的.
kuing 2# 2012-6-18 13:37
∈ 用 \in ≥ 用 \ge  (更标准的用 \geqslant ) sin, cos 用 \sin, \cos θ 用 \theta α 用 \alpha π 用 \pi √ 用 \sqrt
thread-532-1-7.html: [不等式] 征集一个对数不等式的证明
都市侠影 1# 2012-6-18 15:16
本帖最后由 都市侠影 于 2012-6-18 15:17 编辑 对一切正整数$n$成立不等式 \[ 1<\Big(n+\frac{1}{2}\Big)\ln{\Big(1+\frac{1}{n}\Big)}<1+\frac{1}{12n(n+1)} \]
kuing 2# 2012-6-18 18:22
没什么特别方法,还是求导,楼主可能不满意,据我了解楼主喜欢初等证明,不过对于我来说我已经满意了。 令 $x=\frac1n\in(0,1]$,不等式等价于 \[\frac{2x}{x+2}<\ln(x+1)<\frac{x(x^2+12 x+12)}{6 (x+1) (x+2)},\] 令 \begin{align*} f(x)&=\ln(x+1)-\frac{2x}{x+2},\\ g(x)&=\frac{x(x^2+12 x+12)}{6 (x+1) (x+2)}-\ln(x+1), \end{align*} 其中 $x\in(0,1]$。则 \begin{align*} f'(x)&=\frac{x^2}{(x+1) (x+2)^2}>0,\\ g'(x)&=\frac{x^4}{6 (x+1)^2 (x+2)^2}>0, \end{align*} 所以 \begin{align*} f(x)&>\lim_{x\to0^+}f(x)=0,\\ g(x)&>\lim_{x\to0^+}g(x)=0, \end{align*} 即原不等式得证。
都市侠影 3# 2012-6-19 08:32
恩,这个不等式有点太强了,除了求导,就只能是泰勒展开了,汗……
thread-537-1-7.html: [函数] 开个帖子玩玩一个新函数
都市侠影 1# 2012-6-20 13:10
本帖最后由 都市侠影 于 2012-6-20 20:29 编辑 感谢Kuing版的论坛,这里可以随便发帖,没有弱智的验证码,还支持MathJax公式,总之的总之,这是一个适合数学爱好者玩的论坛。 近来对一个函数起了兴趣,开个帖子慢慢研究,为方便发帖,先占了一些楼,哈哈。 ================================================== 方程$x=a^x$的唯一根$R(a)$,这里$0<a<1$,如此得到一个新的函数$R(a),0<a<1$,现在的任务就是研究这个函数的性质。 性质1:函数$R(a)$在定义区间上连续并可导,导函数 \[ R'(a)=\frac{R^2(a)}{a(1-R(a)\ln a)} \] 性质2:两个端点处的极限 \[ \lim_{a\to1^-}R(a)=1,\lim_{a\to0^+}R(a)=0 \] 性质3 ……
都市侠影 2# 2012-6-20 19:57
本帖最后由 都市侠影 于 2012-6-20 21:43 编辑 性质1的证明:函数$R(a)$实际上是由二元方程$F(a,x)=x-a^x=0$所定义的隐函数,根据隐函数理论即可得此结论。
都市侠影 3# 2012-6-20 19:57
本帖最后由 都市侠影 于 2012-6-20 22:00 编辑 性质2的讨论 设$0<a<1$,定义一个函数序列: \[ R_0(a)=1,R_{n+1}(a)=a^{R_n(a)} \] 我的新函数与这个函数序列有着血浓于水的关系,剪不断,理还乱 引理1 对任意正整数$n$,成立不等式 $R_{2n-1}(a)<R(a)<R_{2n}(a)$ 证明:还是用归纳法简单些,因为方程$x=a^x$左边的正比例函数在$x\leqslant0$时值非正,右边的指数函数恒正,所以作为方程根的$R(a)$必然为正值,所以$R(a)>0$,从而$R(a)=a^{R(a)}<a^0=1$,继续下去$R(a)=a^{R(a)}>a^1=a=R_1(a)$,$R(a)=a^{R(a)}<a^{R_1(a)}=R_2(a)$,所以不等式在$n=1$时就成立了,后面的假设递推仍是此法,偷个懒,略去不写了。 引理2:对刚定义的函数序列有 \[ \lim_{n\to0^+}R_{2n-1}(a)=0,\lim_{n\to0^+}R_{2n}(a)=1,\lim_{n\to1^-}R_{n}(a)=1 \] 证明: 引理3:这函数序列收敛到$R(a)$,即是说 \[ \lim_{n\to\infty}R_n(a)=R(a) \] 证明:
都市侠影 4# 2012-6-20 19:57
性质3
都市侠影 5# 2012-6-20 19:58
性质4
都市侠影 6# 2012-6-20 19:58
性质5
都市侠影 7# 2012-6-20 19:58
性质6
都市侠影 8# 2012-6-20 19:59
7
都市侠影 9# 2012-6-20 19:59
8
都市侠影 10# 2012-6-20 19:59
9
都市侠影 11# 2012-6-20 20:00
10
都市侠影 12# 2012-6-20 20:04
11
都市侠影 13# 2012-6-20 20:04
12
kuing 14# 2012-6-20 20:32
感谢Kuing版的论坛,这里可以随便发帖,没有弱智的验证码,还支持MathJax公式,总之的总之,这是一个适合数学爱好者玩的论坛。 都市侠影 发表于 2012-6-20 13:10 哎,可惜冷清,没能力搞起来。正如你上次说的,有XX的不YY,有ZZ的没WW(原句记不清了,大致这样写你应该知道我指哪句吧 )
thread-538-1-1.html: [数论] 来自群的一道数论计数题
kuing 1# 2012-6-20 20:11
据说是苹果的题,不知是不是真的,反正群里的截图就是这样的: 这个题表达得不太好,容易被误解操作,如果不是那句“即排在偶数的灯泡都被关掉”的话,估计我也会理解成每次都把第一个灯开或关。还有一点容易理解错的就是最后说的“第100轮的时候”,这个“时候”到底是指这第100轮操作了没有的呢? 那么这里我重新把题目表达一遍先。 100个灯泡排成一排,依次编号为1~100,原本都是处于关的状态。(其实是不是排成一排没关系,编号位置也没关系) 接下来对这些灯泡进行100轮的操作:第1轮将所有灯泡打开;第2轮将偶数号的灯泡都被关掉;由第3轮开始,第 $k$ 轮对被 $k$ 整除的编号的灯泡进行操作,操作的每个动作是将开着的灯泡关掉,将关掉的灯泡打开。问最终还有哪些灯泡是亮着的? 在群里我发的解答有点简略,这里我也把过程写得好一些。 对于 $n$ 号灯来说,显然最终是开或关就取决于 $n$ 的正约数个数,奇数个则开,偶数个则关。 今设 $n$ 的标准分解式是 \[n=2^{a_1}\cdot 3^{a_2}\cdot 5^{a_3}\cdot 7^{a_4}\cdots ,\] 其中各底数均是质数,各 $a_i$ 是非负整数。 由正约数个数公式 \[d(n)=(1+a_1)(1+a_2)(1+a_3)(1+a_4)\cdots\] 知,要灯泡最终是开的,即是要 $d(n)$ 为奇数,则显然所有的 $a_i$ 都要是偶数,反之亦然。而当所有的 $a_i$ 都要是偶数时显然 $n$ 是完全平方数,因此,有且只有完全平方数号的灯泡最终会开着。 因此,最终亮着的灯的编号是 1, 4, 9, 16, 25, 36, 49, 64, 81, 100,共10个。 嗯,其实也没有详细到哪去,那个正约数个数公式由乘法原理显然可得,这里就不解释太多了。
都市侠影 2# 2012-6-20 20:15
唉,数论没学过,苹果考的也太深了点吧。
都市侠影 3# 2012-6-20 20:16
哦,你一说乘法原理,就明白那个约数公式了。
kuing 4# 2012-6-20 20:18
嗯,不知我现在这样重新对原题目的表述有没有歧义? 先吃饭了
kuing 5# 2012-6-20 22:16
楼上上在群里谈及编程验证此题,可惜C++我不懂,倒是用 Mathematica 编得出来,贴了在 http://kkkkuingggg.5d6d.net/thread-539-1-1.html g[n_, d_] := If[Mod[n, d] == 0, -1, 1]; lst = Table[-k, {k, 1, 100}]; Do[lst = lst*Table[g[k, n], {k, 1, 100}], {n, 1, 100}] lst 得到结果 {1, -2, -3, 4, -5, -6, -7, -8, 9, -10, -11, -12, -13, -14, -15, 16, -17, -18, -19, -20, -21, -22, -23, -24, 25, -26, -27, -28, -29, -30, -31, -32, -33, -34, -35, 36, -37, -38, -39, -40, -41, -42, -43, -44, -45, -46, -47, -48, 49, -50, -51, -52, -53, -54, -55, -56, -57, -58, -59, -60, -61, -62, -63, 64, -65, -66, -67, -68, -69, -70, -71, -72, -73, -74, -75, -76, -77, -78, -79, -80, 81, -82, -83, -84, -85, -86, -87, -88, -89, -90, -91, -92, -93, -94, -95, -96, -97, -98, -99, 100} 可以看出只有完全平方数前面是正的,所以结果符合上面的答案。 链接内后面还有各步状态的数据,由于太长,就不贴过来了。
都市侠影 6# 2012-6-21 09:24
[i=s] 本帖最后由 都市侠影 于 2012-6-21 10:30 编辑 [/i] 我是用C写的,这么小的问题不敢劳烦C++的大驾。检查了下程序,就一个字符打错了, 把19行的a[j]*=-1打成了a[i]*=-1,一字之差啊,修改之后结果是10. 运行结果
kuing 7# 2012-6-21 10:25
oh,C和C++我都不懂,呵呵。 PS。论坛贴里出现 [i] 会变成斜体代码……禁用discuz代码才会正常显示
都市侠影 8# 2012-6-21 10:30
Oh
thread-538-1-7.html: [数论] 来自群的一道数论计数题
kuing 1# 2012-6-20 20:11
据说是苹果的题,不知是不是真的,反正群里的截图就是这样的: 这个题表达得不太好,容易被误解操作,如果不是那句“即排在偶数的灯泡都被关掉”的话,估计我也会理解成每次都把第一个灯开或关。还有一点容易理解错的就是最后说的“第100轮的时候”,这个“时候”到底是指这第100轮操作了没有的呢? 那么这里我重新把题目表达一遍先。 100个灯泡排成一排,依次编号为1~100,原本都是处于关的状态。(其实是不是排成一排没关系,编号位置也没关系) 接下来对这些灯泡进行100轮的操作:第1轮将所有灯泡打开;第2轮将偶数号的灯泡都被关掉;由第3轮开始,第 $k$ 轮对被 $k$ 整除的编号的灯泡进行操作,操作的每个动作是将开着的灯泡关掉,将关掉的灯泡打开。问最终还有哪些灯泡是亮着的? 在群里我发的解答有点简略,这里我也把过程写得好一些。 对于 $n$ 号灯来说,显然最终是开或关就取决于 $n$ 的正约数个数,奇数个则开,偶数个则关。 今设 $n$ 的标准分解式是 \[n=2^{a_1}\cdot 3^{a_2}\cdot 5^{a_3}\cdot 7^{a_4}\cdots ,\] 其中各底数均是质数,各 $a_i$ 是非负整数。 由正约数个数公式 \[d(n)=(1+a_1)(1+a_2)(1+a_3)(1+a_4)\cdots\] 知,要灯泡最终是开的,即是要 $d(n)$ 为奇数,则显然所有的 $a_i$ 都要是偶数,反之亦然。而当所有的 $a_i$ 都要是偶数时显然 $n$ 是完全平方数,因此,有且只有完全平方数号的灯泡最终会开着。 因此,最终亮着的灯的编号是 1, 4, 9, 16, 25, 36, 49, 64, 81, 100,共10个。 嗯,其实也没有详细到哪去,那个正约数个数公式由乘法原理显然可得,这里就不解释太多了。
都市侠影 2# 2012-6-20 20:15
唉,数论没学过,苹果考的也太深了点吧。
都市侠影 3# 2012-6-20 20:16
哦,你一说乘法原理,就明白那个约数公式了。
kuing 4# 2012-6-20 20:18
嗯,不知我现在这样重新对原题目的表述有没有歧义? 先吃饭了
kuing 5# 2012-6-20 22:16
楼上上在群里谈及编程验证此题,可惜C++我不懂,倒是用 Mathematica 编得出来,贴了在 http://kkkkuingggg.5d6d.net/thread-539-1-1.html g[n_, d_] := If[Mod[n, d] == 0, -1, 1]; lst = Table[-k, {k, 1, 100}]; Do[lst = lst*Table[g[k, n], {k, 1, 100}], {n, 1, 100}] lst 得到结果 {1, -2, -3, 4, -5, -6, -7, -8, 9, -10, -11, -12, -13, -14, -15, 16, -17, -18, -19, -20, -21, -22, -23, -24, 25, -26, -27, -28, -29, -30, -31, -32, -33, -34, -35, 36, -37, -38, -39, -40, -41, -42, -43, -44, -45, -46, -47, -48, 49, -50, -51, -52, -53, -54, -55, -56, -57, -58, -59, -60, -61, -62, -63, 64, -65, -66, -67, -68, -69, -70, -71, -72, -73, -74, -75, -76, -77, -78, -79, -80, 81, -82, -83, -84, -85, -86, -87, -88, -89, -90, -91, -92, -93, -94, -95, -96, -97, -98, -99, 100} 可以看出只有完全平方数前面是正的,所以结果符合上面的答案。 链接内后面还有各步状态的数据,由于太长,就不贴过来了。
都市侠影 6# 2012-6-21 09:24
[i=s] 本帖最后由 都市侠影 于 2012-6-21 10:30 编辑 [/i] 我是用C写的,这么小的问题不敢劳烦C++的大驾。检查了下程序,就一个字符打错了, 把19行的a[j]*=-1打成了a[i]*=-1,一字之差啊,修改之后结果是10. 运行结果
kuing 7# 2012-6-21 10:25
oh,C和C++我都不懂,呵呵。 PS。论坛贴里出现 [i] 会变成斜体代码……禁用discuz代码才会正常显示
都市侠影 8# 2012-6-21 10:30
Oh
thread-539-1-1.html: 那个灯泡计数题的编程验证
kuing 1# 2012-6-20 21:58
问题在 http://kkkkuingggg.5d6d.net/thread-538-1-1.html ,这里用 mathematica 编程试一下验证结果 g[n_, d_] := If[Mod[n, d] == 0, -1, 1];    (*用于生成1和-1的“操作”列表*) lst = Table[-k, {k, 1, 100}];    (*初始状态,为方便观察哪个灯亮,这里直接用了灯号,后面看符号即可判定是开还是关*) Do[lst = lst*Table[g[k, n], {k, 1, 100}], {n, 1, 100}]    (*做100轮操作*) lst    (*输出最终结果*) 最终结果为 {1, -2, -3, 4, -5, -6, -7, -8, 9, -10, -11, -12, -13, -14, -15, 16, -17, -18, -19, -20, -21, -22, -23, -24, 25, -26, -27, -28, -29, -30, -31, -32, -33, -34, -35, 36, -37, -38, -39, -40, -41, -42, -43, -44, -45, -46, -47, -48, 49, -50, -51, -52, -53, -54, -55, -56, -57, -58, -59, -60, -61, -62, -63, 64, -65, -66, -67, -68, -69, -70, -71, -72, -73, -74, -75, -76, -77, -78, -79, -80, 81, -82, -83, -84, -85, -86, -87, -88, -89, -90, -91, -92, -93, -94, -95, -96, -97, -98, -99, 100} 可以看出只有完全平方数前面是正的,所以结果符合原贴的答案。
kuing 2# 2012-6-20 22:05
改一改程序还可以直接把每次操作后的结果显示出来 g[n_, d_] := If[Mod[n, d] == 0, -1, 1]; lst = Table[-k, {k, 1, 100}]; Do[{   lst = lst*Table[g[k, n], {k, 1, 100}];   Print[n, "'s step = ", lst]   }, {n, 1, 100}] 得到很长的各步结果: 1's step = {1,2,3,4,5,6,7,8,9,10,11,12,13,14,15,16,17,18,19,20,21,22,23,24,25,26,27,28,29,30,31,32,33,34,35,36,37,38,39,40,41,42,43,44,45,46,47,48,49,50,51,52,53,54,55,56,57,58,59,60,61,62,63,64,65,66,67,68,69,70,71,72,73,74,75,76,77,78,79,80,81,82,83,84,85,86,87,88,89,90,91,92,93,94,95,96,97,98,99,100} 2's step = {1,-2,3,-4,5,-6,7,-8,9,-10,11,-12,13,-14,15,-16,17,-18,19,-20,21,-22,23,-24,25,-26,27,-28,29,-30,31,-32,33,-34,35,-36,37,-38,39,-40,41,-42,43,-44,45,-46,47,-48,49,-50,51,-52,53,-54,55,-56,57,-58,59,-60,61,-62,63,-64,65,-66,67,-68,69,-70,71,-72,73,-74,75,-76,77,-78,79,-80,81,-82,83,-84,85,-86,87,-88,89,-90,91,-92,93,-94,95,-96,97,-98,99,-100} 3's step = {1,-2,-3,-4,5,6,7,-8,-9,-10,11,12,13,-14,-15,-16,17,18,19,-20,-21,-22,23,24,25,-26,-27,-28,29,30,31,-32,-33,-34,35,36,37,-38,-39,-40,41,42,43,-44,-45,-46,47,48,49,-50,-51,-52,53,54,55,-56,-57,-58,59,60,61,-62,-63,-64,65,66,67,-68,-69,-70,71,72,73,-74,-75,-76,77,78,79,-80,-81,-82,83,84,85,-86,-87,-88,89,90,91,-92,-93,-94,95,96,97,-98,-99,-100} 4's step = {1,-2,-3,4,5,6,7,8,-9,-10,11,-12,13,-14,-15,16,17,18,19,20,-21,-22,23,-24,25,-26,-27,28,29,30,31,32,-33,-34,35,-36,37,-38,-39,40,41,42,43,44,-45,-46,47,-48,49,-50,-51,52,53,54,55,56,-57,-58,59,-60,61,-62,-63,64,65,66,67,68,-69,-70,71,-72,73,-74,-75,76,77,78,79,80,-81,-82,83,-84,85,-86,-87,88,89,90,91,92,-93,-94,95,-96,97,-98,-99,100} 5's step = {1,-2,-3,4,-5,6,7,8,-9,10,11,-12,13,-14,15,16,17,18,19,-20,-21,-22,23,-24,-25,-26,-27,28,29,-30,31,32,-33,-34,-35,-36,37,-38,-39,-40,41,42,43,44,45,-46,47,-48,49,50,-51,52,53,54,-55,56,-57,-58,59,60,61,-62,-63,64,-65,66,67,68,-69,70,71,-72,73,-74,75,76,77,78,79,-80,-81,-82,83,-84,-85,-86,-87,88,89,-90,91,92,-93,-94,-95,-96,97,-98,-99,-100} 6's step = {1,-2,-3,4,-5,-6,7,8,-9,10,11,12,13,-14,15,16,17,-18,19,-20,-21,-22,23,24,-25,-26,-27,28,29,30,31,32,-33,-34,-35,36,37,-38,-39,-40,41,-42,43,44,45,-46,47,48,49,50,-51,52,53,-54,-55,56,-57,-58,59,-60,61,-62,-63,64,-65,-66,67,68,-69,70,71,72,73,-74,75,76,77,-78,79,-80,-81,-82,83,84,-85,-86,-87,88,89,90,91,92,-93,-94,-95,96,97,-98,-99,-100} 7's step = {1,-2,-3,4,-5,-6,-7,8,-9,10,11,12,13,14,15,16,17,-18,19,-20,21,-22,23,24,-25,-26,-27,-28,29,30,31,32,-33,-34,35,36,37,-38,-39,-40,41,42,43,44,45,-46,47,48,-49,50,-51,52,53,-54,-55,-56,-57,-58,59,-60,61,-62,63,64,-65,-66,67,68,-69,-70,71,72,73,-74,75,76,-77,-78,79,-80,-81,-82,83,-84,-85,-86,-87,88,89,90,-91,92,-93,-94,-95,96,97,98,-99,-100} 8's step = {1,-2,-3,4,-5,-6,-7,-8,-9,10,11,12,13,14,15,-16,17,-18,19,-20,21,-22,23,-24,-25,-26,-27,-28,29,30,31,-32,-33,-34,35,36,37,-38,-39,40,41,42,43,44,45,-46,47,-48,-49,50,-51,52,53,-54,-55,56,-57,-58,59,-60,61,-62,63,-64,-65,-66,67,68,-69,-70,71,-72,73,-74,75,76,-77,-78,79,80,-81,-82,83,-84,-85,-86,-87,-88,89,90,-91,92,-93,-94,-95,-96,97,98,-99,-100} 9's step = {1,-2,-3,4,-5,-6,-7,-8,9,10,11,12,13,14,15,-16,17,18,19,-20,21,-22,23,-24,-25,-26,27,-28,29,30,31,-32,-33,-34,35,-36,37,-38,-39,40,41,42,43,44,-45,-46,47,-48,-49,50,-51,52,53,54,-55,56,-57,-58,59,-60,61,-62,-63,-64,-65,-66,67,68,-69,-70,71,72,73,-74,75,76,-77,-78,79,80,81,-82,83,-84,-85,-86,-87,-88,89,-90,-91,92,-93,-94,-95,-96,97,98,99,-100} 10's step = {1,-2,-3,4,-5,-6,-7,-8,9,-10,11,12,13,14,15,-16,17,18,19,20,21,-22,23,-24,-25,-26,27,-28,29,-30,31,-32,-33,-34,35,-36,37,-38,-39,-40,41,42,43,44,-45,-46,47,-48,-49,-50,-51,52,53,54,-55,56,-57,-58,59,60,61,-62,-63,-64,-65,-66,67,68,-69,70,71,72,73,-74,75,76,-77,-78,79,-80,81,-82,83,-84,-85,-86,-87,-88,89,90,-91,92,-93,-94,-95,-96,97,98,99,100} 11's step = {1,-2,-3,4,-5,-6,-7,-8,9,-10,-11,12,13,14,15,-16,17,18,19,20,21,22,23,-24,-25,-26,27,-28,29,-30,31,-32,33,-34,35,-36,37,-38,-39,-40,41,42,43,-44,-45,-46,47,-48,-49,-50,-51,52,53,54,55,56,-57,-58,59,60,61,-62,-63,-64,-65,66,67,68,-69,70,71,72,73,-74,75,76,77,-78,79,-80,81,-82,83,-84,-85,-86,-87,88,89,90,-91,92,-93,-94,-95,-96,97,98,-99,100} 12's step = {1,-2,-3,4,-5,-6,-7,-8,9,-10,-11,-12,13,14,15,-16,17,18,19,20,21,22,23,24,-25,-26,27,-28,29,-30,31,-32,33,-34,35,36,37,-38,-39,-40,41,42,43,-44,-45,-46,47,48,-49,-50,-51,52,53,54,55,56,-57,-58,59,-60,61,-62,-63,-64,-65,66,67,68,-69,70,71,-72,73,-74,75,76,77,-78,79,-80,81,-82,83,84,-85,-86,-87,88,89,90,-91,92,-93,-94,-95,96,97,98,-99,100} 13's step = {1,-2,-3,4,-5,-6,-7,-8,9,-10,-11,-12,-13,14,15,-16,17,18,19,20,21,22,23,24,-25,26,27,-28,29,-30,31,-32,33,-34,35,36,37,-38,39,-40,41,42,43,-44,-45,-46,47,48,-49,-50,-51,-52,53,54,55,56,-57,-58,59,-60,61,-62,-63,-64,65,66,67,68,-69,70,71,-72,73,-74,75,76,77,78,79,-80,81,-82,83,84,-85,-86,-87,88,89,90,91,92,-93,-94,-95,96,97,98,-99,100} 14's step = {1,-2,-3,4,-5,-6,-7,-8,9,-10,-11,-12,-13,-14,15,-16,17,18,19,20,21,22,23,24,-25,26,27,28,29,-30,31,-32,33,-34,35,36,37,-38,39,-40,41,-42,43,-44,-45,-46,47,48,-49,-50,-51,-52,53,54,55,-56,-57,-58,59,-60,61,-62,-63,-64,65,66,67,68,-69,-70,71,-72,73,-74,75,76,77,78,79,-80,81,-82,83,-84,-85,-86,-87,88,89,90,91,92,-93,-94,-95,96,97,-98,-99,100} 15's step = {1,-2,-3,4,-5,-6,-7,-8,9,-10,-11,-12,-13,-14,-15,-16,17,18,19,20,21,22,23,24,-25,26,27,28,29,30,31,-32,33,-34,35,36,37,-38,39,-40,41,-42,43,-44,45,-46,47,48,-49,-50,-51,-52,53,54,55,-56,-57,-58,59,60,61,-62,-63,-64,65,66,67,68,-69,-70,71,-72,73,-74,-75,76,77,78,79,-80,81,-82,83,-84,-85,-86,-87,88,89,-90,91,92,-93,-94,-95,96,97,-98,-99,100} 16's step = {1,-2,-3,4,-5,-6,-7,-8,9,-10,-11,-12,-13,-14,-15,16,17,18,19,20,21,22,23,24,-25,26,27,28,29,30,31,32,33,-34,35,36,37,-38,39,-40,41,-42,43,-44,45,-46,47,-48,-49,-50,-51,-52,53,54,55,-56,-57,-58,59,60,61,-62,-63,64,65,66,67,68,-69,-70,71,-72,73,-74,-75,76,77,78,79,80,81,-82,83,-84,-85,-86,-87,88,89,-90,91,92,-93,-94,-95,-96,97,-98,-99,100} 17's step = {1,-2,-3,4,-5,-6,-7,-8,9,-10,-11,-12,-13,-14,-15,16,-17,18,19,20,21,22,23,24,-25,26,27,28,29,30,31,32,33,34,35,36,37,-38,39,-40,41,-42,43,-44,45,-46,47,-48,-49,-50,51,-52,53,54,55,-56,-57,-58,59,60,61,-62,-63,64,65,66,67,-68,-69,-70,71,-72,73,-74,-75,76,77,78,79,80,81,-82,83,-84,85,-86,-87,88,89,-90,91,92,-93,-94,-95,-96,97,-98,-99,100} 18's step = {1,-2,-3,4,-5,-6,-7,-8,9,-10,-11,-12,-13,-14,-15,16,-17,-18,19,20,21,22,23,24,-25,26,27,28,29,30,31,32,33,34,35,-36,37,-38,39,-40,41,-42,43,-44,45,-46,47,-48,-49,-50,51,-52,53,-54,55,-56,-57,-58,59,60,61,-62,-63,64,65,66,67,-68,-69,-70,71,72,73,-74,-75,76,77,78,79,80,81,-82,83,-84,85,-86,-87,88,89,90,91,92,-93,-94,-95,-96,97,-98,-99,100} 19's step = {1,-2,-3,4,-5,-6,-7,-8,9,-10,-11,-12,-13,-14,-15,16,-17,-18,-19,20,21,22,23,24,-25,26,27,28,29,30,31,32,33,34,35,-36,37,38,39,-40,41,-42,43,-44,45,-46,47,-48,-49,-50,51,-52,53,-54,55,-56,57,-58,59,60,61,-62,-63,64,65,66,67,-68,-69,-70,71,72,73,-74,-75,-76,77,78,79,80,81,-82,83,-84,85,-86,-87,88,89,90,91,92,-93,-94,95,-96,97,-98,-99,100} 20's step = {1,-2,-3,4,-5,-6,-7,-8,9,-10,-11,-12,-13,-14,-15,16,-17,-18,-19,-20,21,22,23,24,-25,26,27,28,29,30,31,32,33,34,35,-36,37,38,39,40,41,-42,43,-44,45,-46,47,-48,-49,-50,51,-52,53,-54,55,-56,57,-58,59,-60,61,-62,-63,64,65,66,67,-68,-69,-70,71,72,73,-74,-75,-76,77,78,79,-80,81,-82,83,-84,85,-86,-87,88,89,90,91,92,-93,-94,95,-96,97,-98,-99,-100} 21's step = {1,-2,-3,4,-5,-6,-7,-8,9,-10,-11,-12,-13,-14,-15,16,-17,-18,-19,-20,-21,22,23,24,-25,26,27,28,29,30,31,32,33,34,35,-36,37,38,39,40,41,42,43,-44,45,-46,47,-48,-49,-50,51,-52,53,-54,55,-56,57,-58,59,-60,61,-62,63,64,65,66,67,-68,-69,-70,71,72,73,-74,-75,-76,77,78,79,-80,81,-82,83,84,85,-86,-87,88,89,90,91,92,-93,-94,95,-96,97,-98,-99,-100} 22's step = {1,-2,-3,4,-5,-6,-7,-8,9,-10,-11,-12,-13,-14,-15,16,-17,-18,-19,-20,-21,-22,23,24,-25,26,27,28,29,30,31,32,33,34,35,-36,37,38,39,40,41,42,43,44,45,-46,47,-48,-49,-50,51,-52,53,-54,55,-56,57,-58,59,-60,61,-62,63,64,65,-66,67,-68,-69,-70,71,72,73,-74,-75,-76,77,78,79,-80,81,-82,83,84,85,-86,-87,-88,89,90,91,92,-93,-94,95,-96,97,-98,-99,-100} 23's step = {1,-2,-3,4,-5,-6,-7,-8,9,-10,-11,-12,-13,-14,-15,16,-17,-18,-19,-20,-21,-22,-23,24,-25,26,27,28,29,30,31,32,33,34,35,-36,37,38,39,40,41,42,43,44,45,46,47,-48,-49,-50,51,-52,53,-54,55,-56,57,-58,59,-60,61,-62,63,64,65,-66,67,-68,69,-70,71,72,73,-74,-75,-76,77,78,79,-80,81,-82,83,84,85,-86,-87,-88,89,90,91,-92,-93,-94,95,-96,97,-98,-99,-100} 24's step = {1,-2,-3,4,-5,-6,-7,-8,9,-10,-11,-12,-13,-14,-15,16,-17,-18,-19,-20,-21,-22,-23,-24,-25,26,27,28,29,30,31,32,33,34,35,-36,37,38,39,40,41,42,43,44,45,46,47,48,-49,-50,51,-52,53,-54,55,-56,57,-58,59,-60,61,-62,63,64,65,-66,67,-68,69,-70,71,-72,73,-74,-75,-76,77,78,79,-80,81,-82,83,84,85,-86,-87,-88,89,90,91,-92,-93,-94,95,96,97,-98,-99,-100} 25's step = {1,-2,-3,4,-5,-6,-7,-8,9,-10,-11,-12,-13,-14,-15,16,-17,-18,-19,-20,-21,-22,-23,-24,25,26,27,28,29,30,31,32,33,34,35,-36,37,38,39,40,41,42,43,44,45,46,47,48,-49,50,51,-52,53,-54,55,-56,57,-58,59,-60,61,-62,63,64,65,-66,67,-68,69,-70,71,-72,73,-74,75,-76,77,78,79,-80,81,-82,83,84,85,-86,-87,-88,89,90,91,-92,-93,-94,95,96,97,-98,-99,100} 26's step = {1,-2,-3,4,-5,-6,-7,-8,9,-10,-11,-12,-13,-14,-15,16,-17,-18,-19,-20,-21,-22,-23,-24,25,-26,27,28,29,30,31,32,33,34,35,-36,37,38,39,40,41,42,43,44,45,46,47,48,-49,50,51,52,53,-54,55,-56,57,-58,59,-60,61,-62,63,64,65,-66,67,-68,69,-70,71,-72,73,-74,75,-76,77,-78,79,-80,81,-82,83,84,85,-86,-87,-88,89,90,91,-92,-93,-94,95,96,97,-98,-99,100} 27's step = {1,-2,-3,4,-5,-6,-7,-8,9,-10,-11,-12,-13,-14,-15,16,-17,-18,-19,-20,-21,-22,-23,-24,25,-26,-27,28,29,30,31,32,33,34,35,-36,37,38,39,40,41,42,43,44,45,46,47,48,-49,50,51,52,53,54,55,-56,57,-58,59,-60,61,-62,63,64,65,-66,67,-68,69,-70,71,-72,73,-74,75,-76,77,-78,79,-80,-81,-82,83,84,85,-86,-87,-88,89,90,91,-92,-93,-94,95,96,97,-98,-99,100} 28's step = {1,-2,-3,4,-5,-6,-7,-8,9,-10,-11,-12,-13,-14,-15,16,-17,-18,-19,-20,-21,-22,-23,-24,25,-26,-27,-28,29,30,31,32,33,34,35,-36,37,38,39,40,41,42,43,44,45,46,47,48,-49,50,51,52,53,54,55,56,57,-58,59,-60,61,-62,63,64,65,-66,67,-68,69,-70,71,-72,73,-74,75,-76,77,-78,79,-80,-81,-82,83,-84,85,-86,-87,-88,89,90,91,-92,-93,-94,95,96,97,-98,-99,100} 29's step = {1,-2,-3,4,-5,-6,-7,-8,9,-10,-11,-12,-13,-14,-15,16,-17,-18,-19,-20,-21,-22,-23,-24,25,-26,-27,-28,-29,30,31,32,33,34,35,-36,37,38,39,40,41,42,43,44,45,46,47,48,-49,50,51,52,53,54,55,56,57,58,59,-60,61,-62,63,64,65,-66,67,-68,69,-70,71,-72,73,-74,75,-76,77,-78,79,-80,-81,-82,83,-84,85,-86,87,-88,89,90,91,-92,-93,-94,95,96,97,-98,-99,100} 30's step = {1,-2,-3,4,-5,-6,-7,-8,9,-10,-11,-12,-13,-14,-15,16,-17,-18,-19,-20,-21,-22,-23,-24,25,-26,-27,-28,-29,-30,31,32,33,34,35,-36,37,38,39,40,41,42,43,44,45,46,47,48,-49,50,51,52,53,54,55,56,57,58,59,60,61,-62,63,64,65,-66,67,-68,69,-70,71,-72,73,-74,75,-76,77,-78,79,-80,-81,-82,83,-84,85,-86,87,-88,89,-90,91,-92,-93,-94,95,96,97,-98,-99,100} 31's step = {1,-2,-3,4,-5,-6,-7,-8,9,-10,-11,-12,-13,-14,-15,16,-17,-18,-19,-20,-21,-22,-23,-24,25,-26,-27,-28,-29,-30,-31,32,33,34,35,-36,37,38,39,40,41,42,43,44,45,46,47,48,-49,50,51,52,53,54,55,56,57,58,59,60,61,62,63,64,65,-66,67,-68,69,-70,71,-72,73,-74,75,-76,77,-78,79,-80,-81,-82,83,-84,85,-86,87,-88,89,-90,91,-92,93,-94,95,96,97,-98,-99,100} 32's step = {1,-2,-3,4,-5,-6,-7,-8,9,-10,-11,-12,-13,-14,-15,16,-17,-18,-19,-20,-21,-22,-23,-24,25,-26,-27,-28,-29,-30,-31,-32,33,34,35,-36,37,38,39,40,41,42,43,44,45,46,47,48,-49,50,51,52,53,54,55,56,57,58,59,60,61,62,63,-64,65,-66,67,-68,69,-70,71,-72,73,-74,75,-76,77,-78,79,-80,-81,-82,83,-84,85,-86,87,-88,89,-90,91,-92,93,-94,95,-96,97,-98,-99,100} 33's step = {1,-2,-3,4,-5,-6,-7,-8,9,-10,-11,-12,-13,-14,-15,16,-17,-18,-19,-20,-21,-22,-23,-24,25,-26,-27,-28,-29,-30,-31,-32,-33,34,35,-36,37,38,39,40,41,42,43,44,45,46,47,48,-49,50,51,52,53,54,55,56,57,58,59,60,61,62,63,-64,65,66,67,-68,69,-70,71,-72,73,-74,75,-76,77,-78,79,-80,-81,-82,83,-84,85,-86,87,-88,89,-90,91,-92,93,-94,95,-96,97,-98,99,100} 34's step = {1,-2,-3,4,-5,-6,-7,-8,9,-10,-11,-12,-13,-14,-15,16,-17,-18,-19,-20,-21,-22,-23,-24,25,-26,-27,-28,-29,-30,-31,-32,-33,-34,35,-36,37,38,39,40,41,42,43,44,45,46,47,48,-49,50,51,52,53,54,55,56,57,58,59,60,61,62,63,-64,65,66,67,68,69,-70,71,-72,73,-74,75,-76,77,-78,79,-80,-81,-82,83,-84,85,-86,87,-88,89,-90,91,-92,93,-94,95,-96,97,-98,99,100} 35's step = {1,-2,-3,4,-5,-6,-7,-8,9,-10,-11,-12,-13,-14,-15,16,-17,-18,-19,-20,-21,-22,-23,-24,25,-26,-27,-28,-29,-30,-31,-32,-33,-34,-35,-36,37,38,39,40,41,42,43,44,45,46,47,48,-49,50,51,52,53,54,55,56,57,58,59,60,61,62,63,-64,65,66,67,68,69,70,71,-72,73,-74,75,-76,77,-78,79,-80,-81,-82,83,-84,85,-86,87,-88,89,-90,91,-92,93,-94,95,-96,97,-98,99,100} 36's step = {1,-2,-3,4,-5,-6,-7,-8,9,-10,-11,-12,-13,-14,-15,16,-17,-18,-19,-20,-21,-22,-23,-24,25,-26,-27,-28,-29,-30,-31,-32,-33,-34,-35,36,37,38,39,40,41,42,43,44,45,46,47,48,-49,50,51,52,53,54,55,56,57,58,59,60,61,62,63,-64,65,66,67,68,69,70,71,72,73,-74,75,-76,77,-78,79,-80,-81,-82,83,-84,85,-86,87,-88,89,-90,91,-92,93,-94,95,-96,97,-98,99,100} 37's step = {1,-2,-3,4,-5,-6,-7,-8,9,-10,-11,-12,-13,-14,-15,16,-17,-18,-19,-20,-21,-22,-23,-24,25,-26,-27,-28,-29,-30,-31,-32,-33,-34,-35,36,-37,38,39,40,41,42,43,44,45,46,47,48,-49,50,51,52,53,54,55,56,57,58,59,60,61,62,63,-64,65,66,67,68,69,70,71,72,73,74,75,-76,77,-78,79,-80,-81,-82,83,-84,85,-86,87,-88,89,-90,91,-92,93,-94,95,-96,97,-98,99,100} 38's step = {1,-2,-3,4,-5,-6,-7,-8,9,-10,-11,-12,-13,-14,-15,16,-17,-18,-19,-20,-21,-22,-23,-24,25,-26,-27,-28,-29,-30,-31,-32,-33,-34,-35,36,-37,-38,39,40,41,42,43,44,45,46,47,48,-49,50,51,52,53,54,55,56,57,58,59,60,61,62,63,-64,65,66,67,68,69,70,71,72,73,74,75,76,77,-78,79,-80,-81,-82,83,-84,85,-86,87,-88,89,-90,91,-92,93,-94,95,-96,97,-98,99,100} 39's step = {1,-2,-3,4,-5,-6,-7,-8,9,-10,-11,-12,-13,-14,-15,16,-17,-18,-19,-20,-21,-22,-23,-24,25,-26,-27,-28,-29,-30,-31,-32,-33,-34,-35,36,-37,-38,-39,40,41,42,43,44,45,46,47,48,-49,50,51,52,53,54,55,56,57,58,59,60,61,62,63,-64,65,66,67,68,69,70,71,72,73,74,75,76,77,78,79,-80,-81,-82,83,-84,85,-86,87,-88,89,-90,91,-92,93,-94,95,-96,97,-98,99,100} 40's step = {1,-2,-3,4,-5,-6,-7,-8,9,-10,-11,-12,-13,-14,-15,16,-17,-18,-19,-20,-21,-22,-23,-24,25,-26,-27,-28,-29,-30,-31,-32,-33,-34,-35,36,-37,-38,-39,-40,41,42,43,44,45,46,47,48,-49,50,51,52,53,54,55,56,57,58,59,60,61,62,63,-64,65,66,67,68,69,70,71,72,73,74,75,76,77,78,79,80,-81,-82,83,-84,85,-86,87,-88,89,-90,91,-92,93,-94,95,-96,97,-98,99,100} 41's step = {1,-2,-3,4,-5,-6,-7,-8,9,-10,-11,-12,-13,-14,-15,16,-17,-18,-19,-20,-21,-22,-23,-24,25,-26,-27,-28,-29,-30,-31,-32,-33,-34,-35,36,-37,-38,-39,-40,-41,42,43,44,45,46,47,48,-49,50,51,52,53,54,55,56,57,58,59,60,61,62,63,-64,65,66,67,68,69,70,71,72,73,74,75,76,77,78,79,80,-81,82,83,-84,85,-86,87,-88,89,-90,91,-92,93,-94,95,-96,97,-98,99,100} 42's step = {1,-2,-3,4,-5,-6,-7,-8,9,-10,-11,-12,-13,-14,-15,16,-17,-18,-19,-20,-21,-22,-23,-24,25,-26,-27,-28,-29,-30,-31,-32,-33,-34,-35,36,-37,-38,-39,-40,-41,-42,43,44,45,46,47,48,-49,50,51,52,53,54,55,56,57,58,59,60,61,62,63,-64,65,66,67,68,69,70,71,72,73,74,75,76,77,78,79,80,-81,82,83,84,85,-86,87,-88,89,-90,91,-92,93,-94,95,-96,97,-98,99,100} 43's step = {1,-2,-3,4,-5,-6,-7,-8,9,-10,-11,-12,-13,-14,-15,16,-17,-18,-19,-20,-21,-22,-23,-24,25,-26,-27,-28,-29,-30,-31,-32,-33,-34,-35,36,-37,-38,-39,-40,-41,-42,-43,44,45,46,47,48,-49,50,51,52,53,54,55,56,57,58,59,60,61,62,63,-64,65,66,67,68,69,70,71,72,73,74,75,76,77,78,79,80,-81,82,83,84,85,86,87,-88,89,-90,91,-92,93,-94,95,-96,97,-98,99,100} 44's step = {1,-2,-3,4,-5,-6,-7,-8,9,-10,-11,-12,-13,-14,-15,16,-17,-18,-19,-20,-21,-22,-23,-24,25,-26,-27,-28,-29,-30,-31,-32,-33,-34,-35,36,-37,-38,-39,-40,-41,-42,-43,-44,45,46,47,48,-49,50,51,52,53,54,55,56,57,58,59,60,61,62,63,-64,65,66,67,68,69,70,71,72,73,74,75,76,77,78,79,80,-81,82,83,84,85,86,87,88,89,-90,91,-92,93,-94,95,-96,97,-98,99,100} 45's step = {1,-2,-3,4,-5,-6,-7,-8,9,-10,-11,-12,-13,-14,-15,16,-17,-18,-19,-20,-21,-22,-23,-24,25,-26,-27,-28,-29,-30,-31,-32,-33,-34,-35,36,-37,-38,-39,-40,-41,-42,-43,-44,-45,46,47,48,-49,50,51,52,53,54,55,56,57,58,59,60,61,62,63,-64,65,66,67,68,69,70,71,72,73,74,75,76,77,78,79,80,-81,82,83,84,85,86,87,88,89,90,91,-92,93,-94,95,-96,97,-98,99,100} 46's step = {1,-2,-3,4,-5,-6,-7,-8,9,-10,-11,-12,-13,-14,-15,16,-17,-18,-19,-20,-21,-22,-23,-24,25,-26,-27,-28,-29,-30,-31,-32,-33,-34,-35,36,-37,-38,-39,-40,-41,-42,-43,-44,-45,-46,47,48,-49,50,51,52,53,54,55,56,57,58,59,60,61,62,63,-64,65,66,67,68,69,70,71,72,73,74,75,76,77,78,79,80,-81,82,83,84,85,86,87,88,89,90,91,92,93,-94,95,-96,97,-98,99,100} 47's step = {1,-2,-3,4,-5,-6,-7,-8,9,-10,-11,-12,-13,-14,-15,16,-17,-18,-19,-20,-21,-22,-23,-24,25,-26,-27,-28,-29,-30,-31,-32,-33,-34,-35,36,-37,-38,-39,-40,-41,-42,-43,-44,-45,-46,-47,48,-49,50,51,52,53,54,55,56,57,58,59,60,61,62,63,-64,65,66,67,68,69,70,71,72,73,74,75,76,77,78,79,80,-81,82,83,84,85,86,87,88,89,90,91,92,93,94,95,-96,97,-98,99,100} 48's step = {1,-2,-3,4,-5,-6,-7,-8,9,-10,-11,-12,-13,-14,-15,16,-17,-18,-19,-20,-21,-22,-23,-24,25,-26,-27,-28,-29,-30,-31,-32,-33,-34,-35,36,-37,-38,-39,-40,-41,-42,-43,-44,-45,-46,-47,-48,-49,50,51,52,53,54,55,56,57,58,59,60,61,62,63,-64,65,66,67,68,69,70,71,72,73,74,75,76,77,78,79,80,-81,82,83,84,85,86,87,88,89,90,91,92,93,94,95,96,97,-98,99,100} 49's step = {1,-2,-3,4,-5,-6,-7,-8,9,-10,-11,-12,-13,-14,-15,16,-17,-18,-19,-20,-21,-22,-23,-24,25,-26,-27,-28,-29,-30,-31,-32,-33,-34,-35,36,-37,-38,-39,-40,-41,-42,-43,-44,-45,-46,-47,-48,49,50,51,52,53,54,55,56,57,58,59,60,61,62,63,-64,65,66,67,68,69,70,71,72,73,74,75,76,77,78,79,80,-81,82,83,84,85,86,87,88,89,90,91,92,93,94,95,96,97,98,99,100} 50's step = {1,-2,-3,4,-5,-6,-7,-8,9,-10,-11,-12,-13,-14,-15,16,-17,-18,-19,-20,-21,-22,-23,-24,25,-26,-27,-28,-29,-30,-31,-32,-33,-34,-35,36,-37,-38,-39,-40,-41,-42,-43,-44,-45,-46,-47,-48,49,-50,51,52,53,54,55,56,57,58,59,60,61,62,63,-64,65,66,67,68,69,70,71,72,73,74,75,76,77,78,79,80,-81,82,83,84,85,86,87,88,89,90,91,92,93,94,95,96,97,98,99,-100} 51's step = {1,-2,-3,4,-5,-6,-7,-8,9,-10,-11,-12,-13,-14,-15,16,-17,-18,-19,-20,-21,-22,-23,-24,25,-26,-27,-28,-29,-30,-31,-32,-33,-34,-35,36,-37,-38,-39,-40,-41,-42,-43,-44,-45,-46,-47,-48,49,-50,-51,52,53,54,55,56,57,58,59,60,61,62,63,-64,65,66,67,68,69,70,71,72,73,74,75,76,77,78,79,80,-81,82,83,84,85,86,87,88,89,90,91,92,93,94,95,96,97,98,99,-100} 52's step = {1,-2,-3,4,-5,-6,-7,-8,9,-10,-11,-12,-13,-14,-15,16,-17,-18,-19,-20,-21,-22,-23,-24,25,-26,-27,-28,-29,-30,-31,-32,-33,-34,-35,36,-37,-38,-39,-40,-41,-42,-43,-44,-45,-46,-47,-48,49,-50,-51,-52,53,54,55,56,57,58,59,60,61,62,63,-64,65,66,67,68,69,70,71,72,73,74,75,76,77,78,79,80,-81,82,83,84,85,86,87,88,89,90,91,92,93,94,95,96,97,98,99,-100} 53's step = {1,-2,-3,4,-5,-6,-7,-8,9,-10,-11,-12,-13,-14,-15,16,-17,-18,-19,-20,-21,-22,-23,-24,25,-26,-27,-28,-29,-30,-31,-32,-33,-34,-35,36,-37,-38,-39,-40,-41,-42,-43,-44,-45,-46,-47,-48,49,-50,-51,-52,-53,54,55,56,57,58,59,60,61,62,63,-64,65,66,67,68,69,70,71,72,73,74,75,76,77,78,79,80,-81,82,83,84,85,86,87,88,89,90,91,92,93,94,95,96,97,98,99,-100} 54's step = {1,-2,-3,4,-5,-6,-7,-8,9,-10,-11,-12,-13,-14,-15,16,-17,-18,-19,-20,-21,-22,-23,-24,25,-26,-27,-28,-29,-30,-31,-32,-33,-34,-35,36,-37,-38,-39,-40,-41,-42,-43,-44,-45,-46,-47,-48,49,-50,-51,-52,-53,-54,55,56,57,58,59,60,61,62,63,-64,65,66,67,68,69,70,71,72,73,74,75,76,77,78,79,80,-81,82,83,84,85,86,87,88,89,90,91,92,93,94,95,96,97,98,99,-100} 55's step = {1,-2,-3,4,-5,-6,-7,-8,9,-10,-11,-12,-13,-14,-15,16,-17,-18,-19,-20,-21,-22,-23,-24,25,-26,-27,-28,-29,-30,-31,-32,-33,-34,-35,36,-37,-38,-39,-40,-41,-42,-43,-44,-45,-46,-47,-48,49,-50,-51,-52,-53,-54,-55,56,57,58,59,60,61,62,63,-64,65,66,67,68,69,70,71,72,73,74,75,76,77,78,79,80,-81,82,83,84,85,86,87,88,89,90,91,92,93,94,95,96,97,98,99,-100} 56's step = {1,-2,-3,4,-5,-6,-7,-8,9,-10,-11,-12,-13,-14,-15,16,-17,-18,-19,-20,-21,-22,-23,-24,25,-26,-27,-28,-29,-30,-31,-32,-33,-34,-35,36,-37,-38,-39,-40,-41,-42,-43,-44,-45,-46,-47,-48,49,-50,-51,-52,-53,-54,-55,-56,57,58,59,60,61,62,63,-64,65,66,67,68,69,70,71,72,73,74,75,76,77,78,79,80,-81,82,83,84,85,86,87,88,89,90,91,92,93,94,95,96,97,98,99,-100} 57's step = {1,-2,-3,4,-5,-6,-7,-8,9,-10,-11,-12,-13,-14,-15,16,-17,-18,-19,-20,-21,-22,-23,-24,25,-26,-27,-28,-29,-30,-31,-32,-33,-34,-35,36,-37,-38,-39,-40,-41,-42,-43,-44,-45,-46,-47,-48,49,-50,-51,-52,-53,-54,-55,-56,-57,58,59,60,61,62,63,-64,65,66,67,68,69,70,71,72,73,74,75,76,77,78,79,80,-81,82,83,84,85,86,87,88,89,90,91,92,93,94,95,96,97,98,99,-100} 58's step = {1,-2,-3,4,-5,-6,-7,-8,9,-10,-11,-12,-13,-14,-15,16,-17,-18,-19,-20,-21,-22,-23,-24,25,-26,-27,-28,-29,-30,-31,-32,-33,-34,-35,36,-37,-38,-39,-40,-41,-42,-43,-44,-45,-46,-47,-48,49,-50,-51,-52,-53,-54,-55,-56,-57,-58,59,60,61,62,63,-64,65,66,67,68,69,70,71,72,73,74,75,76,77,78,79,80,-81,82,83,84,85,86,87,88,89,90,91,92,93,94,95,96,97,98,99,-100} 59's step = {1,-2,-3,4,-5,-6,-7,-8,9,-10,-11,-12,-13,-14,-15,16,-17,-18,-19,-20,-21,-22,-23,-24,25,-26,-27,-28,-29,-30,-31,-32,-33,-34,-35,36,-37,-38,-39,-40,-41,-42,-43,-44,-45,-46,-47,-48,49,-50,-51,-52,-53,-54,-55,-56,-57,-58,-59,60,61,62,63,-64,65,66,67,68,69,70,71,72,73,74,75,76,77,78,79,80,-81,82,83,84,85,86,87,88,89,90,91,92,93,94,95,96,97,98,99,-100} 60's step = {1,-2,-3,4,-5,-6,-7,-8,9,-10,-11,-12,-13,-14,-15,16,-17,-18,-19,-20,-21,-22,-23,-24,25,-26,-27,-28,-29,-30,-31,-32,-33,-34,-35,36,-37,-38,-39,-40,-41,-42,-43,-44,-45,-46,-47,-48,49,-50,-51,-52,-53,-54,-55,-56,-57,-58,-59,-60,61,62,63,-64,65,66,67,68,69,70,71,72,73,74,75,76,77,78,79,80,-81,82,83,84,85,86,87,88,89,90,91,92,93,94,95,96,97,98,99,-100} 61's step = {1,-2,-3,4,-5,-6,-7,-8,9,-10,-11,-12,-13,-14,-15,16,-17,-18,-19,-20,-21,-22,-23,-24,25,-26,-27,-28,-29,-30,-31,-32,-33,-34,-35,36,-37,-38,-39,-40,-41,-42,-43,-44,-45,-46,-47,-48,49,-50,-51,-52,-53,-54,-55,-56,-57,-58,-59,-60,-61,62,63,-64,65,66,67,68,69,70,71,72,73,74,75,76,77,78,79,80,-81,82,83,84,85,86,87,88,89,90,91,92,93,94,95,96,97,98,99,-100} 62's step = {1,-2,-3,4,-5,-6,-7,-8,9,-10,-11,-12,-13,-14,-15,16,-17,-18,-19,-20,-21,-22,-23,-24,25,-26,-27,-28,-29,-30,-31,-32,-33,-34,-35,36,-37,-38,-39,-40,-41,-42,-43,-44,-45,-46,-47,-48,49,-50,-51,-52,-53,-54,-55,-56,-57,-58,-59,-60,-61,-62,63,-64,65,66,67,68,69,70,71,72,73,74,75,76,77,78,79,80,-81,82,83,84,85,86,87,88,89,90,91,92,93,94,95,96,97,98,99,-100} 63's step = {1,-2,-3,4,-5,-6,-7,-8,9,-10,-11,-12,-13,-14,-15,16,-17,-18,-19,-20,-21,-22,-23,-24,25,-26,-27,-28,-29,-30,-31,-32,-33,-34,-35,36,-37,-38,-39,-40,-41,-42,-43,-44,-45,-46,-47,-48,49,-50,-51,-52,-53,-54,-55,-56,-57,-58,-59,-60,-61,-62,-63,-64,65,66,67,68,69,70,71,72,73,74,75,76,77,78,79,80,-81,82,83,84,85,86,87,88,89,90,91,92,93,94,95,96,97,98,99,-100} 64's step = {1,-2,-3,4,-5,-6,-7,-8,9,-10,-11,-12,-13,-14,-15,16,-17,-18,-19,-20,-21,-22,-23,-24,25,-26,-27,-28,-29,-30,-31,-32,-33,-34,-35,36,-37,-38,-39,-40,-41,-42,-43,-44,-45,-46,-47,-48,49,-50,-51,-52,-53,-54,-55,-56,-57,-58,-59,-60,-61,-62,-63,64,65,66,67,68,69,70,71,72,73,74,75,76,77,78,79,80,-81,82,83,84,85,86,87,88,89,90,91,92,93,94,95,96,97,98,99,-100} 65's step = {1,-2,-3,4,-5,-6,-7,-8,9,-10,-11,-12,-13,-14,-15,16,-17,-18,-19,-20,-21,-22,-23,-24,25,-26,-27,-28,-29,-30,-31,-32,-33,-34,-35,36,-37,-38,-39,-40,-41,-42,-43,-44,-45,-46,-47,-48,49,-50,-51,-52,-53,-54,-55,-56,-57,-58,-59,-60,-61,-62,-63,64,-65,66,67,68,69,70,71,72,73,74,75,76,77,78,79,80,-81,82,83,84,85,86,87,88,89,90,91,92,93,94,95,96,97,98,99,-100} 66's step = {1,-2,-3,4,-5,-6,-7,-8,9,-10,-11,-12,-13,-14,-15,16,-17,-18,-19,-20,-21,-22,-23,-24,25,-26,-27,-28,-29,-30,-31,-32,-33,-34,-35,36,-37,-38,-39,-40,-41,-42,-43,-44,-45,-46,-47,-48,49,-50,-51,-52,-53,-54,-55,-56,-57,-58,-59,-60,-61,-62,-63,64,-65,-66,67,68,69,70,71,72,73,74,75,76,77,78,79,80,-81,82,83,84,85,86,87,88,89,90,91,92,93,94,95,96,97,98,99,-100} 67's step = {1,-2,-3,4,-5,-6,-7,-8,9,-10,-11,-12,-13,-14,-15,16,-17,-18,-19,-20,-21,-22,-23,-24,25,-26,-27,-28,-29,-30,-31,-32,-33,-34,-35,36,-37,-38,-39,-40,-41,-42,-43,-44,-45,-46,-47,-48,49,-50,-51,-52,-53,-54,-55,-56,-57,-58,-59,-60,-61,-62,-63,64,-65,-66,-67,68,69,70,71,72,73,74,75,76,77,78,79,80,-81,82,83,84,85,86,87,88,89,90,91,92,93,94,95,96,97,98,99,-100} 68's step = {1,-2,-3,4,-5,-6,-7,-8,9,-10,-11,-12,-13,-14,-15,16,-17,-18,-19,-20,-21,-22,-23,-24,25,-26,-27,-28,-29,-30,-31,-32,-33,-34,-35,36,-37,-38,-39,-40,-41,-42,-43,-44,-45,-46,-47,-48,49,-50,-51,-52,-53,-54,-55,-56,-57,-58,-59,-60,-61,-62,-63,64,-65,-66,-67,-68,69,70,71,72,73,74,75,76,77,78,79,80,-81,82,83,84,85,86,87,88,89,90,91,92,93,94,95,96,97,98,99,-100} 69's step = {1,-2,-3,4,-5,-6,-7,-8,9,-10,-11,-12,-13,-14,-15,16,-17,-18,-19,-20,-21,-22,-23,-24,25,-26,-27,-28,-29,-30,-31,-32,-33,-34,-35,36,-37,-38,-39,-40,-41,-42,-43,-44,-45,-46,-47,-48,49,-50,-51,-52,-53,-54,-55,-56,-57,-58,-59,-60,-61,-62,-63,64,-65,-66,-67,-68,-69,70,71,72,73,74,75,76,77,78,79,80,-81,82,83,84,85,86,87,88,89,90,91,92,93,94,95,96,97,98,99,-100} 70's step = {1,-2,-3,4,-5,-6,-7,-8,9,-10,-11,-12,-13,-14,-15,16,-17,-18,-19,-20,-21,-22,-23,-24,25,-26,-27,-28,-29,-30,-31,-32,-33,-34,-35,36,-37,-38,-39,-40,-41,-42,-43,-44,-45,-46,-47,-48,49,-50,-51,-52,-53,-54,-55,-56,-57,-58,-59,-60,-61,-62,-63,64,-65,-66,-67,-68,-69,-70,71,72,73,74,75,76,77,78,79,80,-81,82,83,84,85,86,87,88,89,90,91,92,93,94,95,96,97,98,99,-100} 71's step = {1,-2,-3,4,-5,-6,-7,-8,9,-10,-11,-12,-13,-14,-15,16,-17,-18,-19,-20,-21,-22,-23,-24,25,-26,-27,-28,-29,-30,-31,-32,-33,-34,-35,36,-37,-38,-39,-40,-41,-42,-43,-44,-45,-46,-47,-48,49,-50,-51,-52,-53,-54,-55,-56,-57,-58,-59,-60,-61,-62,-63,64,-65,-66,-67,-68,-69,-70,-71,72,73,74,75,76,77,78,79,80,-81,82,83,84,85,86,87,88,89,90,91,92,93,94,95,96,97,98,99,-100} 72's step = {1,-2,-3,4,-5,-6,-7,-8,9,-10,-11,-12,-13,-14,-15,16,-17,-18,-19,-20,-21,-22,-23,-24,25,-26,-27,-28,-29,-30,-31,-32,-33,-34,-35,36,-37,-38,-39,-40,-41,-42,-43,-44,-45,-46,-47,-48,49,-50,-51,-52,-53,-54,-55,-56,-57,-58,-59,-60,-61,-62,-63,64,-65,-66,-67,-68,-69,-70,-71,-72,73,74,75,76,77,78,79,80,-81,82,83,84,85,86,87,88,89,90,91,92,93,94,95,96,97,98,99,-100} 73's step = {1,-2,-3,4,-5,-6,-7,-8,9,-10,-11,-12,-13,-14,-15,16,-17,-18,-19,-20,-21,-22,-23,-24,25,-26,-27,-28,-29,-30,-31,-32,-33,-34,-35,36,-37,-38,-39,-40,-41,-42,-43,-44,-45,-46,-47,-48,49,-50,-51,-52,-53,-54,-55,-56,-57,-58,-59,-60,-61,-62,-63,64,-65,-66,-67,-68,-69,-70,-71,-72,-73,74,75,76,77,78,79,80,-81,82,83,84,85,86,87,88,89,90,91,92,93,94,95,96,97,98,99,-100} 74's step = {1,-2,-3,4,-5,-6,-7,-8,9,-10,-11,-12,-13,-14,-15,16,-17,-18,-19,-20,-21,-22,-23,-24,25,-26,-27,-28,-29,-30,-31,-32,-33,-34,-35,36,-37,-38,-39,-40,-41,-42,-43,-44,-45,-46,-47,-48,49,-50,-51,-52,-53,-54,-55,-56,-57,-58,-59,-60,-61,-62,-63,64,-65,-66,-67,-68,-69,-70,-71,-72,-73,-74,75,76,77,78,79,80,-81,82,83,84,85,86,87,88,89,90,91,92,93,94,95,96,97,98,99,-100} 75's step = {1,-2,-3,4,-5,-6,-7,-8,9,-10,-11,-12,-13,-14,-15,16,-17,-18,-19,-20,-21,-22,-23,-24,25,-26,-27,-28,-29,-30,-31,-32,-33,-34,-35,36,-37,-38,-39,-40,-41,-42,-43,-44,-45,-46,-47,-48,49,-50,-51,-52,-53,-54,-55,-56,-57,-58,-59,-60,-61,-62,-63,64,-65,-66,-67,-68,-69,-70,-71,-72,-73,-74,-75,76,77,78,79,80,-81,82,83,84,85,86,87,88,89,90,91,92,93,94,95,96,97,98,99,-100} 76's step = {1,-2,-3,4,-5,-6,-7,-8,9,-10,-11,-12,-13,-14,-15,16,-17,-18,-19,-20,-21,-22,-23,-24,25,-26,-27,-28,-29,-30,-31,-32,-33,-34,-35,36,-37,-38,-39,-40,-41,-42,-43,-44,-45,-46,-47,-48,49,-50,-51,-52,-53,-54,-55,-56,-57,-58,-59,-60,-61,-62,-63,64,-65,-66,-67,-68,-69,-70,-71,-72,-73,-74,-75,-76,77,78,79,80,-81,82,83,84,85,86,87,88,89,90,91,92,93,94,95,96,97,98,99,-100} 77's step = {1,-2,-3,4,-5,-6,-7,-8,9,-10,-11,-12,-13,-14,-15,16,-17,-18,-19,-20,-21,-22,-23,-24,25,-26,-27,-28,-29,-30,-31,-32,-33,-34,-35,36,-37,-38,-39,-40,-41,-42,-43,-44,-45,-46,-47,-48,49,-50,-51,-52,-53,-54,-55,-56,-57,-58,-59,-60,-61,-62,-63,64,-65,-66,-67,-68,-69,-70,-71,-72,-73,-74,-75,-76,-77,78,79,80,-81,82,83,84,85,86,87,88,89,90,91,92,93,94,95,96,97,98,99,-100} 78's step = {1,-2,-3,4,-5,-6,-7,-8,9,-10,-11,-12,-13,-14,-15,16,-17,-18,-19,-20,-21,-22,-23,-24,25,-26,-27,-28,-29,-30,-31,-32,-33,-34,-35,36,-37,-38,-39,-40,-41,-42,-43,-44,-45,-46,-47,-48,49,-50,-51,-52,-53,-54,-55,-56,-57,-58,-59,-60,-61,-62,-63,64,-65,-66,-67,-68,-69,-70,-71,-72,-73,-74,-75,-76,-77,-78,79,80,-81,82,83,84,85,86,87,88,89,90,91,92,93,94,95,96,97,98,99,-100} 79's step = {1,-2,-3,4,-5,-6,-7,-8,9,-10,-11,-12,-13,-14,-15,16,-17,-18,-19,-20,-21,-22,-23,-24,25,-26,-27,-28,-29,-30,-31,-32,-33,-34,-35,36,-37,-38,-39,-40,-41,-42,-43,-44,-45,-46,-47,-48,49,-50,-51,-52,-53,-54,-55,-56,-57,-58,-59,-60,-61,-62,-63,64,-65,-66,-67,-68,-69,-70,-71,-72,-73,-74,-75,-76,-77,-78,-79,80,-81,82,83,84,85,86,87,88,89,90,91,92,93,94,95,96,97,98,99,-100} 80's step = {1,-2,-3,4,-5,-6,-7,-8,9,-10,-11,-12,-13,-14,-15,16,-17,-18,-19,-20,-21,-22,-23,-24,25,-26,-27,-28,-29,-30,-31,-32,-33,-34,-35,36,-37,-38,-39,-40,-41,-42,-43,-44,-45,-46,-47,-48,49,-50,-51,-52,-53,-54,-55,-56,-57,-58,-59,-60,-61,-62,-63,64,-65,-66,-67,-68,-69,-70,-71,-72,-73,-74,-75,-76,-77,-78,-79,-80,-81,82,83,84,85,86,87,88,89,90,91,92,93,94,95,96,97,98,99,-100} 81's step = {1,-2,-3,4,-5,-6,-7,-8,9,-10,-11,-12,-13,-14,-15,16,-17,-18,-19,-20,-21,-22,-23,-24,25,-26,-27,-28,-29,-30,-31,-32,-33,-34,-35,36,-37,-38,-39,-40,-41,-42,-43,-44,-45,-46,-47,-48,49,-50,-51,-52,-53,-54,-55,-56,-57,-58,-59,-60,-61,-62,-63,64,-65,-66,-67,-68,-69,-70,-71,-72,-73,-74,-75,-76,-77,-78,-79,-80,81,82,83,84,85,86,87,88,89,90,91,92,93,94,95,96,97,98,99,-100} 82's step = {1,-2,-3,4,-5,-6,-7,-8,9,-10,-11,-12,-13,-14,-15,16,-17,-18,-19,-20,-21,-22,-23,-24,25,-26,-27,-28,-29,-30,-31,-32,-33,-34,-35,36,-37,-38,-39,-40,-41,-42,-43,-44,-45,-46,-47,-48,49,-50,-51,-52,-53,-54,-55,-56,-57,-58,-59,-60,-61,-62,-63,64,-65,-66,-67,-68,-69,-70,-71,-72,-73,-74,-75,-76,-77,-78,-79,-80,81,-82,83,84,85,86,87,88,89,90,91,92,93,94,95,96,97,98,99,-100} 83's step = {1,-2,-3,4,-5,-6,-7,-8,9,-10,-11,-12,-13,-14,-15,16,-17,-18,-19,-20,-21,-22,-23,-24,25,-26,-27,-28,-29,-30,-31,-32,-33,-34,-35,36,-37,-38,-39,-40,-41,-42,-43,-44,-45,-46,-47,-48,49,-50,-51,-52,-53,-54,-55,-56,-57,-58,-59,-60,-61,-62,-63,64,-65,-66,-67,-68,-69,-70,-71,-72,-73,-74,-75,-76,-77,-78,-79,-80,81,-82,-83,84,85,86,87,88,89,90,91,92,93,94,95,96,97,98,99,-100} 84's step = {1,-2,-3,4,-5,-6,-7,-8,9,-10,-11,-12,-13,-14,-15,16,-17,-18,-19,-20,-21,-22,-23,-24,25,-26,-27,-28,-29,-30,-31,-32,-33,-34,-35,36,-37,-38,-39,-40,-41,-42,-43,-44,-45,-46,-47,-48,49,-50,-51,-52,-53,-54,-55,-56,-57,-58,-59,-60,-61,-62,-63,64,-65,-66,-67,-68,-69,-70,-71,-72,-73,-74,-75,-76,-77,-78,-79,-80,81,-82,-83,-84,85,86,87,88,89,90,91,92,93,94,95,96,97,98,99,-100} 85's step = {1,-2,-3,4,-5,-6,-7,-8,9,-10,-11,-12,-13,-14,-15,16,-17,-18,-19,-20,-21,-22,-23,-24,25,-26,-27,-28,-29,-30,-31,-32,-33,-34,-35,36,-37,-38,-39,-40,-41,-42,-43,-44,-45,-46,-47,-48,49,-50,-51,-52,-53,-54,-55,-56,-57,-58,-59,-60,-61,-62,-63,64,-65,-66,-67,-68,-69,-70,-71,-72,-73,-74,-75,-76,-77,-78,-79,-80,81,-82,-83,-84,-85,86,87,88,89,90,91,92,93,94,95,96,97,98,99,-100} 86's step = {1,-2,-3,4,-5,-6,-7,-8,9,-10,-11,-12,-13,-14,-15,16,-17,-18,-19,-20,-21,-22,-23,-24,25,-26,-27,-28,-29,-30,-31,-32,-33,-34,-35,36,-37,-38,-39,-40,-41,-42,-43,-44,-45,-46,-47,-48,49,-50,-51,-52,-53,-54,-55,-56,-57,-58,-59,-60,-61,-62,-63,64,-65,-66,-67,-68,-69,-70,-71,-72,-73,-74,-75,-76,-77,-78,-79,-80,81,-82,-83,-84,-85,-86,87,88,89,90,91,92,93,94,95,96,97,98,99,-100} 87's step = {1,-2,-3,4,-5,-6,-7,-8,9,-10,-11,-12,-13,-14,-15,16,-17,-18,-19,-20,-21,-22,-23,-24,25,-26,-27,-28,-29,-30,-31,-32,-33,-34,-35,36,-37,-38,-39,-40,-41,-42,-43,-44,-45,-46,-47,-48,49,-50,-51,-52,-53,-54,-55,-56,-57,-58,-59,-60,-61,-62,-63,64,-65,-66,-67,-68,-69,-70,-71,-72,-73,-74,-75,-76,-77,-78,-79,-80,81,-82,-83,-84,-85,-86,-87,88,89,90,91,92,93,94,95,96,97,98,99,-100} 88's step = {1,-2,-3,4,-5,-6,-7,-8,9,-10,-11,-12,-13,-14,-15,16,-17,-18,-19,-20,-21,-22,-23,-24,25,-26,-27,-28,-29,-30,-31,-32,-33,-34,-35,36,-37,-38,-39,-40,-41,-42,-43,-44,-45,-46,-47,-48,49,-50,-51,-52,-53,-54,-55,-56,-57,-58,-59,-60,-61,-62,-63,64,-65,-66,-67,-68,-69,-70,-71,-72,-73,-74,-75,-76,-77,-78,-79,-80,81,-82,-83,-84,-85,-86,-87,-88,89,90,91,92,93,94,95,96,97,98,99,-100} 89's step = {1,-2,-3,4,-5,-6,-7,-8,9,-10,-11,-12,-13,-14,-15,16,-17,-18,-19,-20,-21,-22,-23,-24,25,-26,-27,-28,-29,-30,-31,-32,-33,-34,-35,36,-37,-38,-39,-40,-41,-42,-43,-44,-45,-46,-47,-48,49,-50,-51,-52,-53,-54,-55,-56,-57,-58,-59,-60,-61,-62,-63,64,-65,-66,-67,-68,-69,-70,-71,-72,-73,-74,-75,-76,-77,-78,-79,-80,81,-82,-83,-84,-85,-86,-87,-88,-89,90,91,92,93,94,95,96,97,98,99,-100} 90's step = {1,-2,-3,4,-5,-6,-7,-8,9,-10,-11,-12,-13,-14,-15,16,-17,-18,-19,-20,-21,-22,-23,-24,25,-26,-27,-28,-29,-30,-31,-32,-33,-34,-35,36,-37,-38,-39,-40,-41,-42,-43,-44,-45,-46,-47,-48,49,-50,-51,-52,-53,-54,-55,-56,-57,-58,-59,-60,-61,-62,-63,64,-65,-66,-67,-68,-69,-70,-71,-72,-73,-74,-75,-76,-77,-78,-79,-80,81,-82,-83,-84,-85,-86,-87,-88,-89,-90,91,92,93,94,95,96,97,98,99,-100} 91's step = {1,-2,-3,4,-5,-6,-7,-8,9,-10,-11,-12,-13,-14,-15,16,-17,-18,-19,-20,-21,-22,-23,-24,25,-26,-27,-28,-29,-30,-31,-32,-33,-34,-35,36,-37,-38,-39,-40,-41,-42,-43,-44,-45,-46,-47,-48,49,-50,-51,-52,-53,-54,-55,-56,-57,-58,-59,-60,-61,-62,-63,64,-65,-66,-67,-68,-69,-70,-71,-72,-73,-74,-75,-76,-77,-78,-79,-80,81,-82,-83,-84,-85,-86,-87,-88,-89,-90,-91,92,93,94,95,96,97,98,99,-100} 92's step = {1,-2,-3,4,-5,-6,-7,-8,9,-10,-11,-12,-13,-14,-15,16,-17,-18,-19,-20,-21,-22,-23,-24,25,-26,-27,-28,-29,-30,-31,-32,-33,-34,-35,36,-37,-38,-39,-40,-41,-42,-43,-44,-45,-46,-47,-48,49,-50,-51,-52,-53,-54,-55,-56,-57,-58,-59,-60,-61,-62,-63,64,-65,-66,-67,-68,-69,-70,-71,-72,-73,-74,-75,-76,-77,-78,-79,-80,81,-82,-83,-84,-85,-86,-87,-88,-89,-90,-91,-92,93,94,95,96,97,98,99,-100} 93's step = {1,-2,-3,4,-5,-6,-7,-8,9,-10,-11,-12,-13,-14,-15,16,-17,-18,-19,-20,-21,-22,-23,-24,25,-26,-27,-28,-29,-30,-31,-32,-33,-34,-35,36,-37,-38,-39,-40,-41,-42,-43,-44,-45,-46,-47,-48,49,-50,-51,-52,-53,-54,-55,-56,-57,-58,-59,-60,-61,-62,-63,64,-65,-66,-67,-68,-69,-70,-71,-72,-73,-74,-75,-76,-77,-78,-79,-80,81,-82,-83,-84,-85,-86,-87,-88,-89,-90,-91,-92,-93,94,95,96,97,98,99,-100} 94's step = {1,-2,-3,4,-5,-6,-7,-8,9,-10,-11,-12,-13,-14,-15,16,-17,-18,-19,-20,-21,-22,-23,-24,25,-26,-27,-28,-29,-30,-31,-32,-33,-34,-35,36,-37,-38,-39,-40,-41,-42,-43,-44,-45,-46,-47,-48,49,-50,-51,-52,-53,-54,-55,-56,-57,-58,-59,-60,-61,-62,-63,64,-65,-66,-67,-68,-69,-70,-71,-72,-73,-74,-75,-76,-77,-78,-79,-80,81,-82,-83,-84,-85,-86,-87,-88,-89,-90,-91,-92,-93,-94,95,96,97,98,99,-100} 95's step = {1,-2,-3,4,-5,-6,-7,-8,9,-10,-11,-12,-13,-14,-15,16,-17,-18,-19,-20,-21,-22,-23,-24,25,-26,-27,-28,-29,-30,-31,-32,-33,-34,-35,36,-37,-38,-39,-40,-41,-42,-43,-44,-45,-46,-47,-48,49,-50,-51,-52,-53,-54,-55,-56,-57,-58,-59,-60,-61,-62,-63,64,-65,-66,-67,-68,-69,-70,-71,-72,-73,-74,-75,-76,-77,-78,-79,-80,81,-82,-83,-84,-85,-86,-87,-88,-89,-90,-91,-92,-93,-94,-95,96,97,98,99,-100} 96's step = {1,-2,-3,4,-5,-6,-7,-8,9,-10,-11,-12,-13,-14,-15,16,-17,-18,-19,-20,-21,-22,-23,-24,25,-26,-27,-28,-29,-30,-31,-32,-33,-34,-35,36,-37,-38,-39,-40,-41,-42,-43,-44,-45,-46,-47,-48,49,-50,-51,-52,-53,-54,-55,-56,-57,-58,-59,-60,-61,-62,-63,64,-65,-66,-67,-68,-69,-70,-71,-72,-73,-74,-75,-76,-77,-78,-79,-80,81,-82,-83,-84,-85,-86,-87,-88,-89,-90,-91,-92,-93,-94,-95,-96,97,98,99,-100} 97's step = {1,-2,-3,4,-5,-6,-7,-8,9,-10,-11,-12,-13,-14,-15,16,-17,-18,-19,-20,-21,-22,-23,-24,25,-26,-27,-28,-29,-30,-31,-32,-33,-34,-35,36,-37,-38,-39,-40,-41,-42,-43,-44,-45,-46,-47,-48,49,-50,-51,-52,-53,-54,-55,-56,-57,-58,-59,-60,-61,-62,-63,64,-65,-66,-67,-68,-69,-70,-71,-72,-73,-74,-75,-76,-77,-78,-79,-80,81,-82,-83,-84,-85,-86,-87,-88,-89,-90,-91,-92,-93,-94,-95,-96,-97,98,99,-100} 98's step = {1,-2,-3,4,-5,-6,-7,-8,9,-10,-11,-12,-13,-14,-15,16,-17,-18,-19,-20,-21,-22,-23,-24,25,-26,-27,-28,-29,-30,-31,-32,-33,-34,-35,36,-37,-38,-39,-40,-41,-42,-43,-44,-45,-46,-47,-48,49,-50,-51,-52,-53,-54,-55,-56,-57,-58,-59,-60,-61,-62,-63,64,-65,-66,-67,-68,-69,-70,-71,-72,-73,-74,-75,-76,-77,-78,-79,-80,81,-82,-83,-84,-85,-86,-87,-88,-89,-90,-91,-92,-93,-94,-95,-96,-97,-98,99,-100} 99's step = {1,-2,-3,4,-5,-6,-7,-8,9,-10,-11,-12,-13,-14,-15,16,-17,-18,-19,-20,-21,-22,-23,-24,25,-26,-27,-28,-29,-30,-31,-32,-33,-34,-35,36,-37,-38,-39,-40,-41,-42,-43,-44,-45,-46,-47,-48,49,-50,-51,-52,-53,-54,-55,-56,-57,-58,-59,-60,-61,-62,-63,64,-65,-66,-67,-68,-69,-70,-71,-72,-73,-74,-75,-76,-77,-78,-79,-80,81,-82,-83,-84,-85,-86,-87,-88,-89,-90,-91,-92,-93,-94,-95,-96,-97,-98,-99,-100} 100's step = {1,-2,-3,4,-5,-6,-7,-8,9,-10,-11,-12,-13,-14,-15,16,-17,-18,-19,-20,-21,-22,-23,-24,25,-26,-27,-28,-29,-30,-31,-32,-33,-34,-35,36,-37,-38,-39,-40,-41,-42,-43,-44,-45,-46,-47,-48,49,-50,-51,-52,-53,-54,-55,-56,-57,-58,-59,-60,-61,-62,-63,64,-65,-66,-67,-68,-69,-70,-71,-72,-73,-74,-75,-76,-77,-78,-79,-80,81,-82,-83,-84,-85,-86,-87,-88,-89,-90,-91,-92,-93,-94,-95,-96,-97,-98,-99,100}
kuing 3# 2012-6-20 22:07
注意楼上的各列表中有的负号和数字被断开了在两行
thread-54-1-2.html: 开一个影视资源区如何
isea 1# 2011-10-4 20:25
DVDRip BDrip 当前国内外流行影视 网盘分流之类,如何? 如果没人管,偶可自荐
kuing 2# 2011-10-4 20:44
呃,貌似不太合适。。。 这里还是以学科学习研究为主。。。 想推荐电影的话可以在这个吹水版块发发贴就行了,专区就免了
isea 3# 2011-10-4 21:13
呵呵。
thread-540-1-7.html: [不等式] 二元均值不等式的一个变种及其证明
都市侠影 1# 2012-6-21 08:35
本帖最后由 都市侠影 于 2012-6-21 17:08 编辑 昨天群里讨论到一个不等式,设$a$、$b$是两个正实数,那么成立 \[ \frac{a+b}{2}\geqslant \sqrt[a+b]{a^bb^a}\] 昨天讨论时原来是用$m$、$n$来表示两个正实数的,直接导致了我当成正整数进行证明,还是符合约定俗成的好。后来经讨论,发现此不等式还可以加强 \[ \sqrt{ab}\geqslant \sqrt[a+b]{a^bb^a}\] 折腾了下,想了两种证明。 证一:欲证的不等式等价于 \[ \Big(\frac{a}{b}\Big)^{\frac{1}{2}}\geqslant \Big(\frac{a}{b}\Big)^{\frac{b}{a+b}} \] 分 $a\geqslant b$ 和 $a<b$ 稍作讨论即知其成立。 证二:只要证明 \[ \frac{\ln a+\ln b}{2}\geqslant \frac{b}{a+b}\ln a+\frac{a}{a+b}\ln b \] 于是构造一次函数 $f(t)=(1-t)\ln a+t\ln b$,其中 $0<t<1$,根据原不等式的轮换对称特征,引入假定 $a \leqslant b$,那么构造的一次函数必是单调增加的,所以有 \[ f\big(\frac{1}{2}\big) \geqslant f\Big(\frac{a}{a+b}\Big) \] 这意味着要证的不等式成立。
q85669551 2# 2012-6-21 15:18
有点小毛病,最开始你左边写的a、b;右边写的 m、n
kuing 3# 2012-6-21 15:24
2# q85669551 q同学也来了 楼主应该比较喜欢用a,b但又受了原题的影响所以……
q85669551 4# 2012-6-21 15:25
3# kuing   sin 被毒害得深啊
都市侠影 5# 2012-6-21 17:09

yayaweha 6# 2012-6-22 20:04
这题我当时做的时候用取对数的方法
yayaweha 7# 2012-6-23 12:57
$\frac{\ln a+\ln b}{2}\geqslant \frac{b}{a+b}\ln a+\frac{a}{a+b}\ln b$ 由2元加权jensen不等式得 $$\ln \frac{2ab}{a+b}\geqslant \frac{b}{a+b}\ln a+\frac{a}{a+b}\ln b$$
yayaweha 8# 2012-6-23 13:05
本帖最后由 yayaweha 于 2012-6-24 13:15 编辑 再根据 $\sqrt{ab}\geqslant\frac{2ab}{a+b}\ 可得 $$\frac{\ln a+\ln b}{2}\geqslant\ln \frac{2ab}{a+b}\$$ 所以 原不等式 $$\frac{\ln a+\ln b}{2}\geqslant \frac{b}{a+b}\ln a+\frac{a}{a+b}\ln b$$ 成立
yayaweha 9# 2012-6-23 13:19
本帖最后由 yayaweha 于 2012-6-23 13:20 编辑 最后来一个不等式链,前半部份在KK博客看到的 \begin{align}\frac{a^2+b^2}{a+b}&\geqslant\sqrt{\frac{a^2+b^2}2}\\&\geqslant\frac{a+b}2\\&\geqslant\sqrt{ab}\\&\geqslant\frac2{\frac1a+\frac1b}\geqslant\sqrt[a+b]{a^bb^a} \end{align}
yayaweha 10# 2012-6-23 13:20
我打得不好呀
thread-541-1-1.html: 一个数列中的不等式
都市侠影 1# 2012-6-22 12:05
本帖最后由 都市侠影 于 2012-6-22 12:10 编辑 群里冒出来的一个问题: 数列 $ \{x_n\}$ 定义为 \[ x_n=\sqrt{2+\sqrt[3]{3+\cdots+\sqrt[n]{n}}} \] 求证有如下不等式 \[ x_{n+1}-x_n<\frac{1}{n!} \]
都市侠影 2# 2012-6-22 12:06
本帖最后由 都市侠影 于 2012-6-22 12:14 编辑 定义多项式序列 \[ P_2(x)=x^2-2,P_m(x)=(P_{m-1}(x))^m-m \] 那么 $x_n$ 就是多项式 $P_n(x)$ 之一根。期望研究这个多项式序列来得到这一列根的性质。 第一步,每一个多项式的根都不是唯一的,期望把根限制到某个区间上。 以下内容纯属猜测: 1. 数列 $\{x_n\}$单调增加,且极限是 $e$.
海盗船长 3# 2012-6-23 18:55
http://tieba.baidu.com/p/1677332034
q85669551 4# 2012-6-26 19:56
贴吧那个证明我觉得太奇葩了。。求个正常点的。。
thread-55-1-1.html: 秋风扫落叶,换个LOGO
kuing 1# 2011-10-4 20:59

kuing 2# 2011-10-23 21:20

海盗船长 3# 2012-1-14 20:54
该换LOGO了?
kuing 4# 2012-8-19 04:09
换个生日Logo,只用一天
kuing 5# 2012-9-7 00:26
无聊又换
kuing 6# 2013-5-20 01:28

李斌斌755 7# 2013-5-20 01:59
看到了
thread-551-1-7.html: 一个向量选择题
都市侠影 1# 2012-6-22 22:32
本帖最后由 都市侠影 于 2012-6-22 22:41 编辑 人教论坛的帖子http://bbs.pep.com.cn/thread-2561706-1-1.html,为了方便打公式,解答弄到这了。 已知A、B、C是平面上不共线三点,动点P满足向量 \[\overrightarrow{OP}=\frac{1}{3}[(1-\lambda)\overrightarrow{OA}+(1-\lambda)\overrightarrow{OB}+(1+2\lambda)\overrightarrow{OC}](\lambda\in R且\lambda\neq0)\] O为坐标原点,则P的轨迹一定通过△ABC的()。 A.内心   B.垂心   C.重心    D.AB边的中点 该题答案选D,略解如下: 由所给方程可以转化为 \[ \overrightarrow{PA}+\overrightarrow{PB}+\overrightarrow{PC}=\lambda(\overrightarrow{CA}+\overrightarrow{CB}) \] 到此,虽然不能明显看出点 $P$的轨迹,但是 $AB$ 边的中点却是明显的满足条件了,只需要取 $\lambda=-1/2$
kuing 2# 2012-6-23 00:49
以前做过  轨迹是AB边上的中线所在直线除重心外   上不了网 用手机试试回帖 就不打过程了 其实也没什么难度
kuing 3# 2012-6-23 00:51
哈  还真的能回  只可惜手机打字实在麻烦 打代码更加 闪了
q85669551 4# 2012-6-23 14:35
3# kuing 为什么手机会不可以回帖呢??打代码估计够呛。
kuing 5# 2012-6-23 16:13
4# q85669551 因为没试过呗
力工 6# 2012-6-25 21:12
1# 都市侠影 直觉变形利用对称性即可。
thread-552-1-1.html: Latex中小问题
麦田里的守望者 1# 2012-6-25 10:49
请问节标题怎么输入 还有魏国人名之间带有点的怎么输入?谢谢
kuing 2# 2012-6-25 12:13
\section{。。。} 中间的点?直接打 · 可以么?
麦田里的守望者 3# 2012-6-25 14:26
貌似可以哈,谢谢呀!
麦田里的守望者 4# 2012-6-25 14:26
2# kuing 谢谢您!貌似可以哈!
thread-554-1-1.html: latex小问题
麦田里的守望者 1# 2012-6-26 21:37
请问如何让页脚页码在右下方而不是页脚正中央呀?
kuing 2# 2012-6-26 22:59
我一般用 fancyhdr 宏包来设置页眉和页脚这些东东
麦田里的守望者 3# 2012-6-27 02:23
2# kuing 非常感谢版主,版主真是大好人呀,好人一生幸福。可是我还有好多问题要请教,可能太简单了,如果有时间就给我解答一下好吗?谢谢 比如说pdf上的显示的参考文献怎么由“reference” 改成“参考文献”谢谢呀,另外我们不用CJK之类的东西....谢谢啦!
kuing 4# 2012-6-27 02:25
\renewcommand{\refname}{参考文献} 其实常见的问题可以百度一下的
麦田里的守望者 5# 2012-6-28 12:31
4# kuing 是不是还差一个\fontspec{字体}呀
kuing 6# 2012-6-28 13:04
你用 xelatex 编译吗
麦田里的守望者 7# 2012-6-28 23:20
6# kuing 饿 我说初学者不太懂这个单词含义.....
麦田里的守望者 8# 2012-6-29 22:57
2# kuing 版主好 请教你一个问题关于定理 如何将其排版在段落的空两个字处并不是左靠其,往右稍微移动两个字的距离,求指导!
kuing 9# 2012-6-29 23:08
定义新定理的时候在定理名字前加个 \indent (希望没拼错)
麦田里的守望者 10# 2012-6-30 14:40
9# kuing 呵呵可以了 谢谢版主!我想请教一下,latex相比word有什么优缺点呢?latex输入一页资料要耗将近一个小时,就算打字很快的也都把人折腾坏了呀,请问latex有什么实际的用途呀,学好了有什么工作适合做吗?
kuing 11# 2012-6-30 14:59
10# 麦田里的守望者 很懒说这些。 单说效率,大概就用下面这张图说明 美观、功能那些我就不说了。
kuing 12# 2012-6-30 15:23
讲开又讲,经常看到有人为 word 和 latex 争来争去,各种比较,甚至吵得火红火绿的。 其实我觉得真的没必要争,皆因各人的需求、要求、审美观、价值观等等都不一样,找个自己觉得合适的就行了。 我们可以给对方介绍自己喜欢的软件,但不必将自己的观点强加于别人,别人对你介绍的东东喜不喜欢、用不用,那都是别人的事了。 所以其实我很少给别人推荐latex,除非我了解到对方也十分注重美观并且有一定的自学能力把代码耍弄得来。而对于那些连word都玩不来的,各种简单输入都乱七八zao的,绝对不推荐。
麦田里的守望者 13# 2012-7-1 15:42
12# kuing 呵呵 谢谢版主 版主果然有学问!作为学数学的人,我暑假才接触的latex从没有接触过word里面的这类东西。所以对latex比较感兴趣就是输入太耗时间了,您觉得有什么改进或者提高速度的方法吗?我觉得只要能提高办事效率才是好软件
thread-557-1-7.html: 传说这是84年的一道题。。。
q85669551 1# 2012-7-1 02:27
哈哈,大家来玩玩,思路不复杂。可能后面求导数有点难算 。可以使用各种方法,集思广益。。 如图,已知圆心为 $O$、半径为 1 的圆与直线 $l$ 相切于点 $A$,一动点 $P$ 自切点 $A$ 沿直线 $l$ 向右移动时,取弧 $AC$ 的长为 $\frac23AP$,直线 $PC$ 与直线 $AO$ 交于点 $M$。又知当 $AP=\frac34\pi$ 时,点 $P$ 的速度为 $v$,求这时点 $M$ 的速度。
isea 2# 2012-7-1 12:08
怎么么看都觉得是物理题
kuing 3# 2012-7-1 22:58
是不是这样子: 如果这样的话,后面的计算的确有点麻烦 \[ \left\{\begin{aligned} &v_M=x\sec\alpha,\\ &\frac{v\sin\alpha-x}{\frac23v\cos\alpha-x}=\frac{PM}{CM}=\frac{PA}{CO}=\frac34\pi,\\ &\cot\alpha=\frac34\pi-1, \end{aligned}\right. \]
q85669551 4# 2012-7-1 23:02
先回了看。。反正结果是个很丑陋的数字。。
kuing 5# 2012-7-1 23:06
汗,打错三角函数名——“sce”应为“sec”
kuing 6# 2012-7-1 23:07
另一个方法是用 t 来设 P 和 C,计算出 M(t),然后求导,不过估计计算也不会太简单。
q85669551 7# 2012-7-1 23:19
另一个方法是用 t 来设 P 和 C,计算出 M(t),然后求导,不过估计计算也不会太简单。 kuing 发表于 2012-7-1 23:07 我大概是这样做的。。原函数的函数图像看起来还不错。。不过求导真的很麻烦。。注 dx/dt|x=3pi/4=V _____kuing edit in $\LaTeX$_____ \[ \left.\frac{\mathrm dx}{\mathrm dt}\right|_{x=\frac34\pi}=v \]
kuing 8# 2012-7-1 23:26
用软件解一下我上面的方程组,得 后面那个 vM==... 就是答案。 写好一点,就是 \[v_M = \frac{2(3\pi^2-4\pi-8)v}{(3\pi-4)^2}.\]
kuing 9# 2012-7-2 12:01
不过其实出题的算是好心的了,至少那个时候 $OC \perp OA$
thread-559-1-1.html: 平板受热问题
GAM 1# 2012-7-2 17:52
读时没感脚哪里有问题,但是读完后,觉得证明过程和板子没啥关系,那板子不就可以是任意的 ,我就有点乱...
kuing 2# 2012-7-2 18:04
题目都看不懂
GAM 3# 2012-7-2 18:59
是我漏下了神马么
kuing 4# 2012-7-2 19:11
应该是我的理解能力问题…… 现在好像懂他想要表达什么了 不过……不懂
都市侠影 5# 2012-7-4 13:02
本帖最后由 都市侠影 于 2012-7-4 13:06 编辑 题倒是看懂了,不过这线性方程组太庞大了
icesheep 6# 2012-7-11 19:10
连续的情况下就是在给定第一类边界条件下调和方程的解是否存在唯一的问题。
秋风树林 7# 2012-7-24 14:52
这不就是那本代数学引论上的吗 解法是挺精妙的,从极值点开始。。。 是和板子没什么关系,其实就是个齐次线性方程组有唯一零解
thread-56-1-2.html: logo
图图 1# 2011-10-4 21:28
强烈呼吁将logo换成大耳朵图图,呜呜...
kuing 2# 2011-10-4 21:41

图图 3# 2011-10-4 22:00
2# kuing
kuing 4# 2011-10-4 22:10
3# 图图
图图 5# 2011-10-25 01:06
现在的logo 不错哇,过一段时间换成大耳朵图图吧,嘻嘻
kuing 6# 2011-10-25 01:12
5# 图图 还没睡……
thread-561-1-1.html: [Physics]磁带的转动问题。
Chetion 1# 2012-7-4 12:25
本帖最后由 Chetion 于 2012-7-4 21:36 编辑 磁带录音机的空带轴以恒定的角速度转动,重新绕上磁带。绕好后带卷的末半径$r_{末}$为初半径$r_{初}$的3倍。绕带的时间为$t_{1}$。 要在相同的带轴上重新绕上厚度为原磁带一半的薄磁带,问需要多少时间$t_{2}$?
kuing 2# 2012-7-4 14:07
没看出和渐开线有什么关系。 设空带轴的半径为 $r$,磁带总长为 $L$,原磁带的厚度为 $d$,原磁带绕好后的圈数为 $n_1$,则由条件得 \[\left\{\begin{aligned} & \pi (3r)^2-\pi r^2=Ld, \\ & n_1=\frac{3r-r}d, \end{aligned}\right.\] 设换用厚度为原磁带一半的薄磁带后绕好时带卷的末半径为 $R$,圈数为 $n_2$,则 \[\left\{\begin{aligned} & \pi R^2-\pi r^2=L\frac d2, \\ & n_2=\frac{R-r}{\frac d2}, \end{aligned}\right.\] 由于空带轴的角速度恒定,因此 \[\frac{t_1}{t_2}=\frac{n_1}{n_2},\] 解得 \[\left\{\begin{aligned} & R=\sqrt5r, \\ & t_2=\bigl(\sqrt5-1\bigr)t_1. \end{aligned}\right.\]
kuing 3# 2012-7-4 14:20
话说貌似没用到什么物理知识……基本上是数学题……
Chetion 4# 2012-7-4 21:36
3# kuing 。。确实和渐开线无关。天哪,一个暑假还没过完,全忘光了。 不过Kuing列出的面积桓等式还真是没有想到。。相反,我无穷累加的思路,忽视了面积的问题,反而欠妥。 然而,书上给出的答案比你给出的结果多上个二分之一。你的计算没有问题。我正在寻找原因。
kuing 5# 2012-7-4 21:42
把答案打打……
Chetion 6# 2012-7-5 19:54
5# kuing 这个就不用了。。因为不负责的书上只撂下了一个答案。于是参考答案的准确性也令人质疑。 仔细检查了几次。我觉得Kuing没错。
kuing 7# 2012-7-5 20:53
主要是我对我那解答其实也并没有太大信心 到底用 $\pi r_末^2 - \pi r_初^2 = Ld$ 是否合理?我一直在想这个问题,皆因磁带绕起来后是弯的,用Ld算面积似乎有误差,但这个误差到底能否忽略?
thread-562-1-4.html: [数列] 来自群的一道数列不等式
kuing 1# 2012-7-4 18:24
当$n\geqslant2$时,由 \begin{align*} a_1a_2\cdots a_n &= a_{n+1}-1, \\ a_1a_2\cdots a_{n-1} &= a_n-1, \end{align*} 得 \[(a_n-1)a_n=a_{n+1}-1,\] 不难验证当$n=1$时也有$(a_1-1)a_1=a_2-1$,即上式对于任意正整数$n$都成立。显然两边不会出现$0$,故可以取倒数,有 \[(a_n-1)a_n=a_{n+1}-1\iff\frac1{(a_n-1)a_n}=\frac1{a_{n+1}-1}\iff\frac1{a_n}=\frac1{a_n-1}-\frac1{a_{n+1}-1},\] 于是 \[\frac1{a_1}+\frac1{a_2}+\cdots +\frac1{a_n}=\frac1{a_1-1}-\frac1{a_{n+1}-1}=1-\frac1{a_{n+1}-1},\] 而 \[\frac12+\frac14+\frac18+\cdots +\frac1{2^n}=1-\frac1{2^n},\] 故原不等式等价于 \[a_{n+1}\geqslant 2^n+1,\] 下面用数学归纳法证之。当$n=1$时$a_{1+1}=3\geqslant 2^1+1$,不等式成立,假设当$n=k$时成立,即$a_{k+1}\geqslant 2^k+1$,则当$n=k+1$时,有 \[a_{k+1+1}=(a_{k+1}-1)a_{k+1}+1\geqslant (2^k+1-1)(2^k+1)+1=2^{2k}+2^k+1\geqslant 2^{k+1}+1,\] 即当$n=k+1$时不等式也成立,从而由数学归纳法知对任意正整数$n$都有$a_{n+1}\geqslant 2^n+1$,原不等式得证。
wenshengli 2# 2012-7-4 20:11
1# kuing 条件转化的好!牛!!!
q85669551 3# 2012-7-4 21:36
话说这个分式型递推关系好眼熟。。
kuing 4# 2012-7-4 21:52
3# q85669551 显然,因为中间的是FAQ……
q85669551 5# 2012-7-6 00:32
有点点类似08年中科大自招的那道数列题,貌似那道是a1*a2*……*an-1=1-an
yuzi 6# 2012-7-19 19:45
那个裂项常见。。。。佩服。。。
yayaweha 7# 2012-7-21 17:31
顺带问下数学归纳法为什么一般假设$n=k $而有时是假设$n=2^k$
kuing 8# 2012-7-21 17:34
2^k 那种多数是在反向数学归纳法里面用吧?
yayaweha 9# 2012-7-21 17:35
8# kuing [/ 这个不是很明白
kuing 10# 2012-7-21 17:40
呃,百度一下好了……
yes94 11# 2013-2-3 23:03
又是那个数列!
yayaweha 12# 2013-2-9 08:46
本帖最后由 yayaweha 于 2013-2-9 09:38 编辑 练习一下打字  因为$$(a_n-1)a_n=a_{n+1}-1$$ 所以$$a_{n+1}-a_n=a_n^2-2a_n+1=(a_n+1)^2\ge0$$ 数列$\{a_n\}$为增数列,$a_1=2$ 有$$a_{n+1}=a_1a_2\cdots a_n+1\ge2^n+1$$
yayaweha 13# 2013-2-9 09:31
本帖最后由 yayaweha 于 2013-2-9 09:59 编辑 $$\sum_{k=1}^n\frac{1}{a_k}=1-\frac{1}{a_{n+1}-1}\le1$$ $$\Large\sqrt[n]{\prod_{k=1}^n a_k}\geqslant \Large\frac{n}{\sum_{k=1}^n\frac{1}{a_k}}$$ $$a_1a_2\cdots a_n\geqslant  n^n$$ $$a_{n+1}\geqslant n^n+1$$
thread-563-1-1.html: 关于latex中如何插入JPG格式的图片 有人知道吗?
麦田里的守望者 1# 2012-7-5 12:37
请大师指导一下,如何插入JPG格式图片。
kuing 2# 2012-7-5 13:43
貌似得看编译方式,如果没记错,用PDFLATEX编译的话就可以插jpg,具体细节我也记不清了,以前我要插图的时候都先整成eps格式再插,后来学会tikz之后基本上直接用代码画图,不用插。
麦田里的守望者 3# 2012-7-6 19:32
thank you all the same!
叶剑飞Victor 4# 2012-8-8 19:15
首先用这个宏包 \usepackage[pdftex]{color,graphicx} 然后用这个命令插图 \includegraphics[angle=90,width=0.5\textwidth]{test.jpg} 不过这样就不能编译成.dvi文件了(.dvi只支持eps格式图片),只能用pdflatex命令直接编译成.pdf文件。
thread-565-1-1.html: 关于用柯西收敛准则证明自然数平方和数列存在极限
都市侠影 1# 2012-7-6 22:37
本帖最后由 都市侠影 于 2012-7-6 22:48 编辑 今天群里有人讨论如何用柯西收敛准则证明数列 $a_n=\sum_{k=1}^n\frac{1}{k^2}$ 存在极限。 理论上,只要是有极限,都可以用这个准则进行证明,只是有些时候比较麻烦而已。对于这里提到的数列,可以这样来证明: \begin{align} \begin{aligned} a_{n+m}-a_n&=\sum_{k=1}^{m}\frac{1}{(n+k)^2} \\ &<\sum_{k=1}^{m}\frac{1}{(n+k)(n+k-1)} \\ &=\sum_{k=1}^{m}\Big(\frac{1}{n+k-1}-\frac{1}{n+k}\Big) \\ &=\frac{1}{n}-\frac{1}{n+m} \\ &<\frac{1}{n} \end{aligned} \end{align} 如此一来,对于预先给定的任意小的正数 $\varepsilon$,只要取满足 $\frac{1}{n}<\frac{\varepsilon}{2}$ 的正整数 $N$,那么在第 $N$ 项后的任意两项 $a_p$、$a_q$,有 \[ |a_p-a_q|<|(a_p-a_N)+(a_N-a_q)|\leqslant |a_p-a_N|+|a_N-a_q|<\frac{\varepsilon}{2}+\frac{\varepsilon}{2}=\varepsilon \] 证毕。
都市侠影 2# 2012-7-11 18:33
网页公式怎么不显示了,一堆代码?
kuing 3# 2012-7-11 18:49
应该是 mathjax 抽了,你试下进 mathjax 主页看能不能打开
thread-566-1-1.html: 在网页中插入LaTex编码的问题
老樊 1# 2012-7-7 18:02
老板你好,有时候需要在自己的新浪博客中插入数学公式什么的,就不行了,有没有什么办法在这样的网页中插入数学公式,谢谢
kuing 2# 2012-7-7 20:48
没玩过新浪博客,不清楚有没有后台之类的东西进去自己设置网页的头文件,如果没有的话应该就没办法弄成这样。或者看看有没有插件之类的。
叶剑飞Victor 3# 2012-8-17 02:18
本帖最后由 叶剑飞Victor 于 2012-8-17 02:30 编辑 新浪博客博客正文无法编辑HTML源代码。页面设置-->自定义组件-->添加文本组件,虽然可以编辑HTML源代码,但是却过滤掉JavaScript。所以无论如何也无法在新浪博客上使用MathJax。唯一的方法就是把数学公式截图上传。 楼主如果需要发表大量数学论文,建议去“博客园”发表。在博客园的管理页面中,单击“设置”选项卡,找到“页首Html代码”,粘贴如下代码,然后保存即可。 <script type="text/x-mathjax-config">   MathJax.Hub.Config({ extensions: ["tex2jax.js"], jax: ["input/TeX", "output/HTML-CSS"], tex2jax: {   inlineMath: [ [','], ["\\(","\\)"] ],   displayMath: [ [','], ["\\[","\\]"] ],   processEscapes: true }, "HTML-CSS": {   availableFonts: ["TeX"],   minScaleAdjust: 100 }   }); </script> <script type="text/javascript" src="http://sq.k12.com.cn/MathJax/MathJax.js?config=TeX-AMS-MML_HTMLorMML"></script> 复制代码
thread-567-1-7.html: [不等式] 寻初等方法
yayaweha 1# 2012-7-7 18:38
本帖最后由 yayaweha 于 2012-7-7 19:00 编辑 已知f(p+q/2)≤1/2f(p)+1/2f(q) 求证            f(λx1+(1-λ)x2)≤λf(x1)+(1-λ)f(x2)
wenshengli 2# 2012-7-7 18:46
1# yayaweha 下凸函数
yayaweha 3# 2012-7-7 18:48
2# wenshengli 我够知道啦!,我要初等证明方法
都市侠影 4# 2012-7-7 19:32
本帖最后由 都市侠影 于 2012-7-7 19:34 编辑 实际上这个结论是有问题的,还要加上函数连续的条件才能得到那个一般性的结论。 用数学归纳法证明当权重形如 $\frac{m}{2^n}$ 的时候不等式成立,然后再把它视为权重的连续函数,利用连续性证明。
yayaweha 5# 2012-7-7 19:45
4# 都市侠影 怎么搞呀
都市侠影 6# 2012-7-7 19:58
本帖最后由 都市侠影 于 2012-7-7 20:11 编辑 首先用数学归纳法证明当权重 $\lambda$具有形式 $\frac{m}{2^n}$ 的时候不等式成立,其中 $m=0,1,\ldots ,2^n$,也就是 \[ f\big(\big(1-\frac{m}{2^n}\big)a+\frac{m}{2^n}b\big)\leqslant \big(1-\frac{m}{2^n}\big)f(a)+\frac{m}{2^n}f(b) \] 这个没有什么技术性的吧,你试试吧。 有了这个不等式之后,再来考虑证明你的结论,但是要加个条件,就是 $f(x)$ 是连续的。 定义函数 \[ h(\lambda)=f((1-\lambda)a+\lambda b)-[(1-\lambda)f(a)+\lambda f(b)] \] 其中 $0<\lambda<1$,由于 $f(x)$ 是连续的,根据复合函数的连续性可知 $h(\lambda)$ 也是连续函数,而且根据前面所证的,有 \[ h(\frac{m}{2^n})\leqslant 0 \] 现在要说明的是,$\frac{m}{2^n},m=0,1,\ldots ,2^n$ 这样的数是均匀分布在区间 $(0,1)$ 上的,而且随着 $n$ 的增大,它的密度也越来越大,也就是相邻两个数的距离可以任意的小,那么根据连续性的要求,势必有 $h(\lambda)\leqslant 0$,如此即证。 最后这点你也可以用严格的数学语言来写,我觉得通俗的好懂些,你先试着自己写吧。
yayaweha 7# 2012-7-7 20:25
6# 都市侠影 我看不出来哪里运用到了f(p+q/2)≤1/2f(p)+1/2f(q)
都市侠影 8# 2012-7-7 20:33
证前面那个不等式要用,你以为帽子里能跑出兔子来啊。你自己按照那个思路证一下再提问题吧。
yayaweha 9# 2012-7-7 23:08
我知道了 是在数归的过程中运用题设
thread-568-1-7.html: [不等式] 两道不等式。。。
q85669551 1# 2012-7-7 22:51
1.记F(x,y)=(x-y)^2+(x/3+3/y)^2,y≠0,则F(x,y)的最小值是。。 2.x、y∈R,且3|x+1|+2|y-1|≤6 ,则2x-3y的最大值。。。 两个不等式,若是用图像法或线性规划做比较简单。。故希望有不等式解法。。。欢迎各位给出妙方。。 __________kuing edit in $\LaTeX$__________ 1.记 $F(x,y)=(x-y)^2+\left(\dfrac x3+\dfrac3y\right)^2$, $y\ne0$, 则 $F(x,y)$ 的最小值是。。 2. $x$、$y\in \mathbb R$,且 $3|x+1|+2|y-1|\le6$,则 $2x-3y$ 的最大值。。。
q85669551 2# 2012-7-7 22:52
1.是 18/5  2.是 4  。。
kuing 3# 2012-7-8 01:15
第一题展开按 $x$ 整理再重新配方试下? 第二题 \[6\geqslant 3|x+1|+2|y-1|\geqslant \frac43|x+1|+2|y-1|\geqslant \left|\frac43(x+1)-2(y-1)\right|=\frac{2|2x-3y+5|}3,\] 这样得到 \[9\geqslant |2x-3y+5|\iff4\geqslant 2x-3y\geqslant -14.\]
q85669551 4# 2012-7-8 21:55
人教那边看到二次函数展开做法。。没想过展开后上二次函数。。这些不等式知道了取等条件后好构造一些?
kuing 5# 2012-7-9 04:42
oh,我这两天都在搞存档大计,为免扰乱,故没上人教,所以没看到……
q85669551 6# 2012-7-9 12:41
辛苦辛苦  应该很好猜到人教那边这两题是谁最先回复。。
kuing 7# 2012-7-9 15:25
现在能看了,存完99.9%以上的回贴了,那0.1%以内的是有某些不知为什么会卡死的,还有几个手动去掉的资源大贴,目前那个文件夹已经2G多了……
q85669551 8# 2012-7-10 19:47
2G...一个附件算100k的话....
kuing 9# 2012-7-11 00:58
8# q85669551 论坛传图片一般没那么大,平均每个图四五十k左右吧。 其实存档的时候网页的数据也不少的,反正一个普通的十来层楼的贴子也得百多k,如果还要翻页的或者图片多的话就更大了。话说我在人教论坛的回贴列表显示有八千多个贴有我的回复…… 昨天说2G多的时候里头包括 人教论坛数学兴趣小组、本论坛、我在人教论坛的回贴 这三大类的所有贴,目前正在进行的是马甲的存档
thread-569-1-1.html: 能不能发一个高中数学试卷的模板,谢谢
老樊 1# 2012-7-8 14:31
老板,能不能发一个高中数学试卷的模板,谢谢
kuing 2# 2012-7-8 14:54
我没做过这类模板,人教论坛那边好像有人发过
kuing 3# 2012-7-8 14:56
http://bbs.pep.com.cn/thread-2512217-1-1.html
老樊 4# 2012-7-8 15:30
本帖最后由 老樊 于 2012-7-8 15:35 编辑 十分的感谢,我用你说的做分段函数的方法做的分段函数,怎么定义域的x和函数式的x不一样,看看怎么回事 \[f(x)=\cases{{x+1} & x \ge 0\\{1-x} & x<0}\] 还有这个$\ge$在下面单独的时候是可以的,但是写在分段函数中也不行,肿么了!
kuing 5# 2012-7-8 15:36
4# 老樊 前段时间我也发现了,这只能说明 mathjax 下的这个 \cases 的简打有点问题,可能右边默认为文字状态。 我的建议是用环境代替吧 \[ f(x)= \begin{cases} x+1 & x \ge 0 \\ 1-x & x<0 \end{cases} \] 右键自己看代码。
老樊 6# 2012-7-8 15:40
本帖最后由 老樊 于 2012-7-8 15:43 编辑 $y=\{_{-xx < 0}^{xx\geq 0}$ 是滴,谢谢啦,这个是我在Web Equation上面他给出的,真聪明的电脑呀,他好像用的类似于组合数的方式
kuing 7# 2012-7-8 15:42
$y=\{_{-xx < 0}^{xx\geq 0}$ 老樊 发表于 2012-7-8 15:40 这样太小了,不行 置顶贴我也改了下,现在OK了。
thread-57-1-9.html: [函数] 数日前某群的两个抽象函数问题
kuing 1# 2011-10-4 21:32
1. 函数 $f(x)$ 是 $[1,+\infty)$ 上的连续函数且可导,对任意 $x\geqslant1$ 满足 $f(x^2)\geqslant f(2x-1)$。 问题:对任意 $x\geqslant1$ 是否有 $f'(x)\geqslant 0?$ 如果是,请给出证明,如果不是,请说明理由。 2. 函数 $f(x)$ 是 $[1,+\infty)$ 上的连续函数且可导,对任意 $x\geqslant1$ 满足 $\dfrac{f(2x-1)+f(1)}{2}\geqslant f(x)=f\left(\dfrac{2x-1+1}{2}\right)$。 问题:对任意 $x\geqslant1$ 是否有$f''(x)\geqslant 0?$ 如果是,请给出证明,如果不是,请说明理由。
fanwf 2# 2011-10-15 18:23
if \ 2\left ( t-1 \right )^{2}\geq aln\frac{t^{2}}{2t-1} \ for \ all\ t\geq 1,find \the \ range \ of \ a.
kuing 3# 2011-10-15 18:55
2# fanwf 仔细阅读置顶贴学习如何输入数学公式 另,如果跟本贴问题无关请另开新贴发问
thread-575-1-7.html: [数列] 求通项公式。
Chetion 1# 2012-7-12 21:05
本帖最后由 Chetion 于 2012-7-13 17:51 编辑 对于数列$\{a_n\}$,已知 \[a_{n+1}=a_{n}^2+a_n.  a_1=a\]\[求 a_n通项公式。\] 这由一道考卷题目联想的。我只是想探索一下,能不能像二项式一样地,将它的展开式表达出来。毕竟都是多项式,不给出一个通项公式都有些遗憾。。。不过自己确实表示无力。 个人认为,应该先找出递推公式,这指的是通项公式中系数的递推公式。 同时,还有另一道简化很多的题目,不过好像不易转化到此题: 对于数列$\{a_n\}$,已知 \[a_{n+1}=a_{n}^2+2a_n.  a_1=a\]\[求 a_n通项公式。\]
kuing 2# 2012-7-12 21:07
给原题
q85669551 3# 2012-7-12 21:33
还没恢复么。。话说总感觉见过这个题的样子。。如果是a1=a,a[n+1]=a[n]^2+a[n]
kuing 4# 2012-7-12 22:10
3# q85669551 没……希望不是mathjax挂了……更不希望只是我们这里连不上(GFW)…… 原题大概是1/a1+1/a2+...+1/an之类的东东
q85669551 5# 2012-7-13 00:34
现在好没好。。原题难道是那种将最末将下标改了又可以当某年的考题的么   你这么一说感觉有点像了..
Chetion 6# 2012-7-13 18:03
抱歉了。。。昨晚的脚本就是打不开。重启杀毒都不行。不过现在又鬼使神差地好了。
kuing 7# 2012-7-13 19:10
抱歉了。。。昨晚的脚本就是打不开。重启杀毒都不行。不过现在又鬼使神差地好了。 Chetion 发表于 2012-7-13 18:03 昨晚我这里也显示不出来,mathjax偶尔会抽风的。
kuing 8# 2012-7-13 19:11
一般情况来说这种通项无法解
都市侠影 9# 2012-7-14 08:47
本帖最后由 都市侠影 于 2012-7-14 09:00 编辑 你后面那个数列倒是好求: \[ a_{n+1}+1=(a_n+1)^2 \] 剩下的就不用说了吧。 前面这个嘛,按你的思路试了下,应该是可以的,但是通项的复杂程度使得求出来也没有什么意义,还是算了。
thread-58-1-9.html: [不等式] how to CS it?
pxchg1200 1# 2011-10-4 21:32
设$a,b,c \in R $ prove that: \[ a^{4}+b^{4}+c^{4}+a^{3}b+b^{3}c+c^{3}a\geq 2(ab^{3}+bc^{3}+ca^{3}) \]
kuing 2# 2011-10-4 21:34
我大概无能为力了。。。 属于Vasc不等式那系列不等式。。。 貌似都是配方。。。
pxchg1200 3# 2011-10-4 21:36
2# kuing 不要一看就放弃嘛
pxchg1200 4# 2011-10-4 21:50
Ok,最近才发现的: $(b^{2}(b-c)^{2}+c^{2}(c-a)^{2}+a^{2}(a-b)^{2})(c^{2}(c-a)^{2}+a^{2}(a-b)^{2}+b^{2}(b-c)^{2})\geq (\sum{bc(b-c)(c-a)})^{2}=(\sum{b^{2}c^{2}}-\sum{b^{3}c})^{2}$ Done!
kuing 5# 2011-10-4 21:56
了解鸟 这么巧
pxchg1200 6# 2011-10-4 21:58
这么说来的话,那个Vasc 的不等式似乎真的有柯西解呢。 \[ (a^{2}+b^{2}+c^{2})^{2}\geq 3(a^{3}b+b^{3}c+c^{3}a) \] 估计要考虑类似 $ a^{2}(a-b+kc)^{2} $ 之类的式子。。
kuing 7# 2011-10-4 21:59
果然超出我的意料之外最近没什么心思研究高难度东东。。。 PS。这里的公式不会自动换行也是个问题。。。。太长分两行写吧,或者用align
thread-581-1-7.html: [不等式] 不等式
Chetion 1# 2012-7-13 18:00
$α,β\in(0,π/2),x(α+β-π/2)<0$, 求证 $(\dfrac{\cosα}{\sinβ})^x+(\dfrac{\cosβ}{\sinα})^x<2$
kuing 2# 2012-7-13 19:17
分 $\alpha+\beta>\pi/2$ 和 $\alpha+\beta<\pi/2$ 讨论,利用指数函数单调性。
老樊 3# 2012-7-15 14:26
本帖最后由 老樊 于 2012-7-15 14:29 编辑 是滴,这个题就是找那个,我记得那到题目是 $$(\dfrac{b}{a})^x+(\dfrac{a}{b})^x$$ 的形式 哈哈,我也可以用一点点的LaTex了。嘿嘿。
kuing 4# 2012-7-15 14:58
用两个美元就不必 \dfrac 了,可以把 d 省去。
thread-582-1-1.html: 编译问题
老樊 1# 2012-7-16 02:11
我在笔记本装的是最新版的CTex,怎么编译的时候出现: texify: bibtex.exe could not be found. 每个都是,怎么回事呀??老板
kuing 2# 2012-7-16 04:11
没遇到过,不清楚。 你点击什么来编译的?
thread-589-1-2.html: 人教论坛上的几个奇特的ID
kuing 1# 2012-7-17 01:27
粘贴过来这里格式会乱了,给链接算了: http://bbs.pep.com.cn/blog/index ... ce_itemid_9987.html
kuing 2# 2012-7-17 14:55
汗,昨晚一整这个,今天人教就说要升级了,里面的那些链接都看不了了,升完级之后估计这个贴也废了……
戊概念·五 3# 2012-8-4 12:46
1# kuing 第二个ID很亮。。。。。
海盗船长 4# 2012-8-4 13:43
我也要精华溢出!怎么弄啊?
kuing 5# 2012-8-4 13:48
我也不知道怎么做到的
kuing 6# 2012-8-4 13:48
不知论坛升级后会不会修复?(估计不会……)
kuing 7# 2012-8-8 22:12
不知论坛升级后会不会修复?(估计不会……) kuing 发表于 2012-8-4 13:48 果然没,或者说,根本没升级……
戊概念·五 8# 2012-8-9 00:00
7# kuing 论坛可以登录了?!
thread-59-1-9.html: [不等式] 再来一个
pxchg1200 1# 2011-10-4 22:01
$ a,b,c \geq 0, a+b+c=3 $prove that: \[ (ab^{3}+bc^{3}+ca^{3})(ab+bc+ca)\leq 16 \] 这个我证了很久都没搞定。。。技巧不足啊!
pxchg1200 2# 2011-10-4 23:10
kuing 有想法么?
kuing 3# 2011-10-4 23:39
呃暂时没有
pxchg1200 4# 2011-10-5 13:08
膜拜下黄瓜同学神一般的AM-GM kills (Potla 是孟加拉语“黄瓜"的意思) 由AM-GM: \[ \frac{(ab^{3}+bc^{3}+ca^{3})^{2}}{4}(ab+bc+ca)^{2}\leq \frac{1}{27}(ab^{3}+bc^{3}+ca^{3}+(ab+bc+ca)^{2})^{3} \] 只要证: \[ ab^{3}+bc^{3}+ca^{3}+(ab+bc+ca)^{2}\leq \frac{4}{27}(a+b+c)^{4} \] 注意到 \[ ab^{3}+bc^{3}+ca^{3}+(ab+bc+ca)^{2}=(a+b+c)(a^{2}b+b^{2}c+c^{2}a+abc) \] 由: \[ a^{2}b+b^{2}c+c^{2}a+abc\leq \frac{4}{27}(a+b+c)^{3} \] Done!
thread-590-1-7.html: 高考模拟卷上的定积分
hflz01 1# 2012-7-17 11:06
高考模拟卷上的定积分(填空):S=∫(x从0到1)[(1+5*x^2)^100]dx _______kuing edit in $\LaTeX$_______ 高考模拟卷上的定积分(填空): \[ S=\int_0^1(1+5x^2)^{100}dx \]
都市侠影 2# 2012-7-17 12:33
你不看今天群里的讨论结果 \begin{align} &\int_0^1(1+5x^2)^ndx \\ =&\int_0^1\sum_{k=0}^nC_n^k5^kx^{2k}dx \\ =&\sum_{k=0}^nC_n^k5^k\int_0^1x^{2k}dx \\ =&\sum_{k=0}^nC_n^k\frac{5^k}{2k+1} \end{align} 据说这个和式无法化简,只能这样了。
thread-591-1-2.html: 首页问题。。
内牛满面 1# 2012-7-17 12:52
首页建议弄的好看点。。。KK审美有点问题。。。。O(∩_∩)O哈哈哈~
kuing 2# 2012-7-17 13:45
The simpler and sharper, the more beautiful!
thread-598-1-1.html: latex页眉页脚
图图 1# 2012-7-18 23:04
怎么设置能在页眉页脚显示中文?
老樊 2# 2012-7-22 09:36
可以的呀,是在不行你可以做一个页眉和页脚的图片,这样也行的呀,呵呵
thread-6-1-1.html: 连 Archiver 也能显示公式,汗……太强了吧
kuing 1# 2011-9-25 22:43
http://kkkkuingggg.5d6d.com/archiver/tid-1.html
thread-60-1-7.html: [不等式] 我也转一个轮换的$\frac{36}{a^2b+b^2c+c^2a}+\ldots$
kuing 1# 2011-10-4 22:09
取等条件貌似跟Vasc不等式的取等条件之一相同,我暂时还没看到有漂亮证法,了解的吱一声 $a,b,c>0,a+b+c=1$, prove that\[\frac{36}{a^2 b + b^2 c + c^2 a} + \frac1{abc} \geqslant 343\]
pxchg1200 2# 2011-10-4 22:22
这是那个文档上的吧
kuing 3# 2011-10-4 22:24
2# pxchg1200 什么文档?
pxchg1200 4# 2011-10-4 22:27
杨学枝的那个啊,征集手工证明的。貌似又是Vo Quoc Ba Can 出的。。
kuing 5# 2011-10-4 22:29
4# pxchg1200 噢,是那个文档。 我在can的wordpress里看到有这个题不过没证明
pxchg1200 6# 2011-10-4 22:32
本帖最后由 pxchg1200 于 2011-10-4 22:33 编辑 有没有考虑过用: $ a^{2}b+b^{2}c+c^{2}a+abc \leq \frac{4}{27}(a+b+c)^{3} $ 把 $ a^{2}b+b^{2}c+c^{2}a $ 换了,然后用uvw?
kuing 7# 2011-10-4 22:37
6# pxchg1200 满足不了取等条件
yizhong 8# 2012-7-24 16:46
原来小K早就开这个题的贴了啊,这个题目到现在还木有见到A-G,CS这些证法。
thread-603-1-7.html: 最近MathJax老是抽风,不知你们是不是一样的状态
kuing 1# 2012-7-19 18:43
RT,最近MathJax主站(http://www.mathjax.org/)总是间歇性无法访问,以至于公式的显示很不稳定,不知你们所见是不是也这样。 我想应该不是MathJax的问题,总感觉是那个什么墙在搞鬼,郁闷。
都市侠影 2# 2012-7-19 19:43
今天才发现MathJax原来是可以嵌入在网页中的,原来以为只有在MathJax网页上才可以显示,这是个好东西。写数学博客就用它应该可以吧。
kuing 3# 2012-7-20 22:36
今天也好几次无法访问,几分钟前还行的,现在又不行了 ……
都市侠影 4# 2012-7-20 23:41
确实抽了,跳出来这个
kuing 5# 2012-7-21 02:24
看来下次抽的时候要用那个什么门或者什么界试一下,如果正常,说明很可能……有机会试下再说嗯
thread-607-1-2.html: 有点意思
kuing 1# 2012-7-20 14:14
thread-608-1-2.html: 键盘上的字母B键
kuing 1# 2012-7-20 14:28
如图 what about you?
戊概念·五 2# 2012-8-4 12:54
好老的投票。。。。。
kuing 3# 2012-8-4 13:21
嗯,好老了,没事转过来 你选哪个?
海盗船长 4# 2012-8-4 13:43
右手
kuing 5# 2012-8-4 13:46
我是左手 啊对,截图里已经有了
戊概念·五 6# 2012-8-4 15:05
3# kuing 显然是女左 练指法的那些时日还是很有记忆的
叶剑飞Victor 7# 2012-8-8 20:09
我选男生,左手。不过是当时打字训练时,强制纠正过来的。
thread-609-1-2.html: [转]/shuai
kuing 1# 2012-7-20 14:41
工程师认为自己的方程与现实很接近。 物理学家认为现实与自己的方程很接近。 数学家根本不在乎。    一天,数学家觉得自己已受够了数学,于是他跑到消防队去宣布他想当消防员。 消防队长说:“您看上去不错,可是我得先给您一个测试。” 消防队长带数学家到消防队后院小巷,巷子里有一个货栈,一只消防栓和一卷软管。消防队长问:“假设货栈起火,您怎么办?” 数学家回答:“我把消防栓接到软管上,打开水龙,把火浇灭。” 消防队长说:“完全正确!最后一个问题:假设您走进小巷,而货栈没有起火,您怎么办?” 数学家疑惑地思索了半天,终于答道:“我就把货栈点着。” 消防队长大叫起来:“什么?太可怕了!您为什么要把货栈点着?” 数学家回答:“这样我就把问题化简为一个我已经解决过的问题了。”    证明所有大于2的奇数都是质数,不同专业的人给出不同的证明: 数学家:3是质数,5是质数,7是质数,由数学归纳可知,所有大于2的奇数都是质数. 物理学家:3是质数,5是质数,7是质数,9是实验误差,11是质数,...... 工程师:3是质数,5是质数,7是质数,9是质数,11是质数,...... 计算机程序员:3是质数,5是质数,7是质数,7是质数,7是质数,...... 统计学家:让我们来试几个随机抽取的数:17是质数,23是质数,11是质数,......    爱因斯坦和牛顿玩捉迷藏。爱因斯坦先找人。他数完数一转身,发现牛顿就在他身后,爱因斯坦指着他说“抓到了!”牛顿冷笑一声,站在地下自己刚才画的一米见方的格子里说,你找到的不是牛顿......是帕斯卡。帕斯卡一听急了,抱着一平方米的地砖跑出来说:“牛顿在这儿,牛顿在这儿”    常函数和指数函数e^x走在街上,远远看到微分算子,常函数吓得慌忙躲藏,说:“被它微分一下,我就什么都没有啦!”指数函数不慌不忙道:“它可不能把我怎么样,我是e^x!”    物理学家、天文学家和数学家走在苏格兰高原上,碰巧看到一只黑色的羊。 “啊!”天文学家说道,“原来苏格兰的羊是黑色的。” “得了吧,仅凭一次观察你可不能这么说。”物理学家道,“你只能说那只黑色的羊是在苏格兰发现的。” “也不对!”数学家道,“由这次观察你只能说:在这一时刻,这只羊,从我们观察的角度看过去,有一侧表面上是黑色的!”    数学家、生物学家和物理学家坐在露天咖啡座上,悠闲的看着对街商店的人来人往。 首先他们看到两个人走进商店,过了一会儿发现却有三个人走出来。三个朋友就他们的专业发表了彼此的看法: 物理学家: 这证明了测不准原理。 生物学家: 这些人自我繁殖了。 数学家: 若现在再有一人进入此商店则里面将空无一人。
thread-61-1-1.html: 预览按钮问题
pxchg1200 1# 2011-10-4 23:08
为什么发贴时没有那个预览按钮啊? preview 下可以少点错误啊。。
kuing 2# 2011-10-4 23:22
这个我也不知道为什么,还好我打代码基本上很少有错。 可能是这个论坛的版本没有这功能,等会去5d6d官方查查有没有相关说明
kuing 3# 2011-10-4 23:45
只找到了这个:http://www.5d6d.com/thread-814942-1-1.html 这对于我们的情况完全没用,因为公式不会出来。。。 呵呵,看来还是得细心一点了
kuing 4# 2011-10-5 01:38
我看过一些较新版的 discuz 论坛似乎也没预览了
thread-610-1-2.html: [转]在超市买了这么一桶巧克力,结果开罐一看,杯具了。。。
kuing 1# 2012-7-20 14:45
thread-611-1-2.html: [转]猜一句英文
kuing 1# 2012-7-20 14:48
猜一句英文: 「ABABBBAAAAAABBBABAAAABBBBAABBBAAAAA」
thread-612-1-2.html: [转]各种视觉视频
kuing 1# 2012-7-20 14:55
http://player.youku.com/player.php/sid/XMTg5NDM4MDY4/v.swf 长方体, 平方四边形, 平行, 颜色, 平行, 唔知乜, 魔方 http://player.youku.com/player.php/sid/XMTQ0NTM3NTAw/v.swf 斑斑叠合运动 http://player.youku.com/player.php/sid/XMTg4ODgwMzI0/v.swf 巨人错觉 http://player.youku.com/player.php/sid/XMTY5MzA4Mjc2/v.swf 长方体蜡烛 http://player.youku.com/player.php/sid/XMTc5MzUxMjE2/v.swf 转盘 http://player.youku.com/player.php/sid/XMTc5NDEyMzI4/v.swf 若干集合 http://player.youku.com/player.php/sid/XMTc5MzU0Mjk2/v.swf 窗口 http://player.youku.com/player.php/sid/XMTc5OTAzODEy/v.swf 上边集合入边既其中一个 http://player.youku.com/player.php/sid/XMTc5NDA5ODUy/v.swf 又系斑马 食豆都出埋黎 http://player.youku.com/player.php/sid/XMTc5NDAzODI4/v.swf 都系果种原理啦,虽然唔知叫咩
thread-613-1-2.html: [转]解梦
kuing 1# 2012-7-20 15:12
  有位秀才第三次进京赶考,住在一个经常住的店里。考试前两天他做了三个梦:第一个梦是梦到自己在墙上种白菜,第二个梦是下雨天,他戴了斗笠还打着伞,第三个梦是梦到跟心爱的表妹脱光了衣服躺在一起,但是背靠着背。临考之际做此梦,似乎有些深意,秀才第二天去找算命的解梦。算命的一听,连拍大腿说:“你还是回家吧。你想想,高墙上种菜不是白费劲吗?戴斗笠打雨伞不是多此一举吗?跟表妹脱光了衣服躺在一张床上,却背靠背,不是没戏吗?”秀才一听,心灰意冷,回店收拾包裹准备回家。   店老板非常奇怪,问:“不是明天才考试吗?今天怎么就打道回府了?”秀才如此这般说了一番,店老板乐了:“唉,我也会解梦的。我倒觉得,你这次一定能考中。你想想,墙上种菜不是高种吗?戴斗笠打伞不是双保险吗?跟你表妹脱光了背靠背躺在床上,不是说明你翻身的时候就要到了吗?”秀才一听,更有道理,于是精神振奋地参加考试,居然中了个探花。   可见,事物本身并不影响人,人们只受到自己对事物看法的影响,人必须改变被动的思维习惯,养成积极的思维习惯。   怎样才算养成了积极思维的习惯呢?当你在实现目标的过程中,面对具体的工作和任务时,你的大脑里去掉了“不可能”三个字,而代之以“我怎样才能”时,可以说你就养成了积极思维的习惯了。
thread-614-1-7.html: 求必胜策略
kuing 1# 2012-7-20 15:43
现在有两堆石头,一堆50个,一堆100个,两个人轮流取。 每次可以从一堆中取任意多个,或者同时从两堆中取相同多个。 取走最后一个石头的人为输家。 试研究必胜策略
kuing 2# 2012-7-20 16:54
(2, 4) 先行必败?我从4拿走2个变成 (2, 2),就赢了吧?
kuing 3# 2012-7-20 16:55
咦,原2#不见了……看来也发现问题了……不过思路可以参考下
①②③④⑤⑥⑦ 4# 2012-7-20 16:56
必败组合 (0, 1) (2, 2) (3, 5) (4, 7) (6, 10) …… 如果游戏改为取得最后一个石子为胜者,必败组合 (1, 2) (3, 5) (4, 7) (6, 10) …… 从 (3,5) 开始,是两种游戏公共的必败组合……
①②③④⑤⑥⑦ 5# 2012-7-20 16:57
2# kuing 我忘了还有差值为0,所以……
都市侠影 6# 2012-7-26 10:55
本帖最后由 都市侠影 于 2012-7-26 11:15 编辑 我想了个必胜策略,这里题目怎么不见了? 先行者的必胜策略:先假设两堆石头都不会太少,都多于3个。那么先行者可以这样取胜:从其中一堆中取石头,只留下三个,另一堆不动。 这样必胜的理由,慢慢道来,先要说明两点 1.任何情况下任何人如果把一堆石头拿完了,那他也就输定了,当然前提是对手不是笨蛋。 2.如果一方取走石头后两堆石头各剩两个,那他也就赢定了,当然前提是他不是笨蛋。 为什么先行者在其中一堆石头中拿得只剩三个就必胜了呢?显然这时对手如果把两堆中的任何一堆拿完了,那先行者就赢了。假定三个石头的那堆是A堆,另一堆是B堆,按照对手从A堆中拿走的石头数目分别讨论一下。 (1)如果对手在A堆中取走了一块石头(当然可以只取这一块,也可能同时从B堆中取走了一块石头),那么这时A堆只剩两个,先行者只需要在B堆中留下两块石头就赢了。 (2) 如果对手在A堆中取走了两块石头,那么这时A堆只剩一块,先行者只需要把B堆的拿完就行了。 (3)如果对手把A堆拿完了,那先行者自然就更容易取胜了。 (4)如果对手没有在A堆中取,而是仅取了B堆中的石头(当然,他是不会取完的),这时,看看B堆还剩多少块,先行者可以有相应的策略         (a). B堆只有一块,这个先行者是求之不得哇         (b). B堆只有两块,这时A堆还有三块,先行者只要从两堆中各拿两块就赢了。         (c). B堆只有三块,这时A堆还有三块,先行者只要从两堆中各拿走一块就赢了。         (d). B堆还有多于三块,这时A堆还有三块,先行者。。。。。。。。。。。。。又糊涂了,貌似这里有问题
①②③④⑤⑥⑦ 7# 2012-7-26 12:55
6# 都市侠影 当然有问题,看前面的回复,不管取到最后一个是赢还是输,(3,5)都是先行必败的(当然,前提是对手不是笨蛋:-)。 (3,5) (4,7) (6,10) (8,13) …… 每次差值增加1,较小的分量是之前还未出现过的最小自然数。
thread-615-1-6.html: [不等式] 一个似曾相识的不等式
kuing 1# 2012-7-20 21:22
在不等式群看到的:$a$, $b$, $c>0$ \[\left(a^3+\frac1{b^3}-1\right)\left(b^3+\frac1{c^3}-1\right)\left(c^3+\frac1{a^3}-1\right)\leqslant\left(abc+\frac1{abc}-1\right)^3.\] 似曾相识,说不定以前证过,但一时想不起来怎么整了
kuing 2# 2012-7-20 22:30
方法想起来了,还是个暴力的,不满意哟,大家帮忙想个简单的。 我们不妨设 $abc=m>0$,再设 $a^3=xm/y$, $b^3=ym/z$, $c^3=zm/x$, 其中 $x$, $y$, $z>0$,那么原不等式等价于 \[\left( \frac{xm}y+\frac z{ym}-1 \right)\left( \frac{ym}z+\frac x{zm}-1 \right)\left( \frac{zm}x+\frac y{xm}-1 \right)\leqslant\left( m+\frac1m-1 \right)^3,\] 上式展开可以整理为 \[f(m)=p\left(\frac1{m^2}-2m\right)+q\left(m^2-\frac2m\right)+\frac{x^2}{yz}+\frac{y^2}{zx}+\frac{z^2}{xy}-3\geqslant0,\] 其中 $p=x/y+y/z+z/x-3$, $q=y/x+z/y+x/z-3$,显然 $p$, $q$ 要么同时是正的要么同时为 0,而同时为 0 时上式显然成立,故剩下只要考虑当 $p$, $q>0$ 时。对 $f(m)$ 求导得 \[f'(m)=\frac{2(m^3+1)(qm-p)}{m^3},\] 易见 $f(m)$ 取最小值当且仅当 $m=p/q$,即 \[f(m)\geqslant f\left(\frac pq\right) = -\frac{p^2}q-\frac{q^2}p+\frac{x^2}{yz}+\frac{y^2}{zx}+\frac{z^2}{xy}-3,\] 通过暴力因式分解,我们有 \[-\frac{p^2}q-\frac{q^2}p+\frac{x^2}{yz}+\frac{y^2}{zx}+\frac{z^2}{xy}-3 = \frac{(x^2+y^2+z^2-xy-yz-zx)^3}{x^2y^2z^2pq}\geqslant0,\] 于是原不等式成立。
kuing 3# 2012-7-23 21:57
一中,waiting for you
yizhong 4# 2012-8-17 23:26
第三届陈省身杯数学竞赛的第一天第二题
kuing 5# 2012-8-17 23:31
你的解法哩?
海盗船长 6# 2012-8-18 11:02
http://lukang.me/2012-shiing-she ... piad.html#more-3698
yizhong 7# 2012-9-16 11:29
我的方法比萝卜的麻烦些,要不要把过程码上来……
kuing 8# 2012-9-16 12:52
要啊,多多益善啊,嘿嘿
thread-619-1-1.html: 关于mathjax抽风的一个测试
kuing 1# 2012-7-21 16:55
刚才有一段时间又不行,这次我特意找来那个什么门(下称“那啥”)测试了一下,如图: 上图如果看不清,可以点击此链接 http://bbs.pep.com.cn/blog/attac ... 201207211717221.gif 在新窗口看。 注明:我这里如果网页无法访问就会自动跳转到那个114导航。 由图可见,运行那啥之后就能访问mathjax了而且公式也正常显示,一旦退出那啥再看又跳114……这是否足以说明不是mathjax的问题,而是我之前说的……唔……
kuing 2# 2012-7-21 19:34
哎,没办法,谁叫好东西总是外国的
都市侠影 3# 2012-7-21 20:12
原来是被天朝伟大的天网给拦截了,阿弥陀佛。
kuing 4# 2012-7-22 19:16
换了一个链接,希望不会被拦。
kuing 5# 2012-7-23 13:43
换了个链接果然稳定了些,现在mathjax主页又进不了,但是公式还能显示。
kuing 6# 2012-8-27 17:54
移来测试区
kuing 7# 2012-8-27 22:00
PS、可惜此动态图太大,传不上来这里,只好用了外链,结果用了现在上不了的人教真郁闷
thread-62-1-2.html: 利用重要极限求极限
图图 1# 2011-10-5 12:13
本帖最后由 图图 于 2011-10-5 12:25 编辑 \[\lim_{x\to0}\dfrac{\arctan{x}}{x}\]
kuing 2# 2011-10-5 12:29
令 $y=\arctan x$,则 $x=\tan y$ 且当 $x\to0$ 时 $y\to0$,此时\[\lim_{x\to0}\frac{\arctan x}x=\lim_{y\to0}\frac y{\tan y}=\lim_{y\to0}\frac{y\cos y}{\sin y}=1\]
thread-620-1-1.html: 怎么查看所有的宏包和用法
老樊 1# 2012-7-21 17:12
我装的是最新版的full版本,怎么查看每个宏包的内容和用法,这个是英文版的,好难受
kuing 2# 2012-7-21 17:20
我一般是用到的时候再查,一般的宏包都有说明文档,在你的安装目录里就能找到的。再不行就百度google…… 不过这些文档大多都是英文的,特别是较新的那些,中文的说明文档不多,似乎很少人愿意翻译,我想可能是因为大家都觉得能玩latex的都不怕看英文?
kuing 3# 2012-7-21 17:26
http://zzg34b.w3.c361.com/package/font.htm 这个网页里有部分常用宏包的简单介绍,可以参考下了
老樊 4# 2012-7-21 17:41
谢谢了,这段时间安心的学习这个了,呵呵,觉得还是很好的,我觉得很方便,正在学习,希望这个能越来越被更多的人接受
kuing 5# 2012-7-21 17:42
忘了说还有这个 http://ctan.org/ ……当然了,这里更英了…… CTAN是“Comprehensive TeX Archive Network”的首字缩写,为世界上最主要的TeX资源集散网站,搜集了关于TeX的各种文件与软件等等。Perl的资源集散网站CPAN即是基于CTAN的模式诞生与运作。
老樊 6# 2012-7-21 17:54
我们在文章中有选择题的话,有什么现成的宏包没有?还是需要自己按照那个开始敲呀??
kuing 7# 2012-7-21 18:00
宏包我就不清楚了,只记得有人专门为排版数学试卷做过一个文档类,不过我看过里面的排版选择题的方法感觉不怎么样。 如果单单只是搞选择题的话,其实可以自己去设计一些命令就可以的,也不是很难的事,大概不需要另整宏包了。之前人教论坛上也有人分享过方法(那个ID什么c什么math的,你可能看过?),还用了 ifthen 宏包去自动判断单行双行或四行,感觉挺不错的,只是现在上不了人教,转不来这里。
老樊 8# 2012-7-21 18:14
呵呵,就是上不了了,对了,KUING,你的《数学空间》在写的时候我们基本上都用哪些宏包,你能不能把导言区让我看看,如果能看到《数学空间》的TEX版本就更好了,哈哈
kuing 9# 2012-7-21 18:26
呵呵,就是上不了了,对了,KUING,你的《数学空间》在写的时候我们基本上都用哪些宏包,你能不能把导言区让我看看,如果能看到《数学空间》的TEX版本就更好了,哈哈 老樊 发表于 2012-7-21 18:14 数学的肯定要用 amsthm、amsmath、amssymb 作图用 tikz 设置目录标题这些用 titlesec 中文用 xeCJK(所以要用 xelatex 编译了,故此还要用 fontenc 什么的(这个我也不太清楚,何版主更懂些),如果不需要调用系统字体的话只用 CJK 就简单些) 页眉页脚设置用 fancyhdr 页面设置用 geometry 生成PDF各种链接的用 hyperref(这个选项很多,有的我也不太清楚,具体看说明文档) 基本必用的就是上面这些了,别的那些就看情况或个人喜好了,比如想换个好看的向量箭头用 esvect(\vv 命令),更改脚注标号什么的用 footmisc,等等…… 网刊整个导言区挺长的,这里也不便贴出了。
isea 10# 2013-4-10 00:05
看完此帖,还是不明白,在ctex  编译环境下如何查看自带宏包说明? 也就是在运行WinEdt的时候? 我总觉得应该有命令可以查看的,特别是出错的时候。 我现在都是在ctex安装目录搜索*.pdf 一个一个翻……
isea 11# 2013-4-10 00:13
太难看了,特别是放大8倍之类后 \vec \overrightarrow 还真不如宏包 esvect
kuing 12# 2013-4-21 13:56
10# isea 命令行运行 texdoc 宏包名 可以查到一些
isea 13# 2013-5-13 01:04
双击 \usepackage 部分,查看相应宏包文档
kuing 14# 2013-5-13 01:19
13# isea 还真的可以,原来 winedt 有这样的功能……
李斌斌755 15# 2013-5-13 01:39
宏包?改天再学……
thread-622-1-1.html: 从产生单位圆内的随机点联想到的
都市侠影 1# 2012-7-21 20:52
本帖最后由 都市侠影 于 2012-7-23 12:30 编辑 Kuing版的这个帖子http://kkkkuingggg.5d6d.net/thread-405-1-1.html里涉及到要随机产生单位圆内的点,并要符合几何概率,也就是说,点落在任意面积相等区域上的概率都应当是相等的。他那里采用的是从圆外接正方形中随机产生点,若点不在圆内则重新生成点,直到落在圆内为止。我当时给了个建议,说用极坐标来生成,先在0到1上均匀生成r,然后随机生成角度,最后 $x=\cos\theta$,$y=\sin\theta$。这个方法经Kuing指出其实不符合几何概率,详细的认证可以参考这篇文章http://blog.csdn.net/codeboycjy/article/details/6225886,我当时就想改进这个方法,直觉告诉我通过改进是可以达到要求的。 原来的方法问题在于,点落在圆 $x^2+y^2<r^2$ 内的概率是 $r$,跟半径是成正比的,这样一来,落在圆 $x^2+y^2<(\frac{1}{2})^2$ 内的概率是 $\frac{1}{2}$,这个就不符合要求了,那么问题的根源在哪呢,在于随着半径 $r$ 的增大,圆 $x^2+y^2=r^2$ 的周长变长了,但按照原来的方法,半径取到 $r$ 的概率却是一成不变的,如果这个概率能随着 $r$ 增大而增大,那么问题有望解决。下面就按照这个思路来展开。 关键在于半径 $r$ 如何产生,现在把它看成一个连续型随机变量,定义它的分布函数为 \[ F(t)=P(r<t)= \begin{cases} 0 & x \leqslant 0  \\ t^2 & 0<x\leqslant 1 \\ 1 & x>1 \end{cases} \] 这个 $t^2$ 是怎样想到的呢?可以这样,因为我希望这个随机变量 $r$ 取小于 $t$ 的概率,应当等于圆 $x^2+y^2=t^2$ 与单位圆的面积之比,也就是 $t^2$, 这样就得到了它的分布函数,那么它的密度函数自然就是 \[ f(t)= \begin{cases} 2t & 0<t<1 \\ 0 & \textrm{else} \end{cases} \] 这样的分布凭直觉是符合要求的,按照这个分布,随着 $r$ 的增大,点落在圆周 $x^2+y^2=r^2$ 上的概率也会随着变大。可以简单的看一下它落在圆环 $r_1^2<x^2+y^2<r_2^2$ 上的概率: \[ P=\int_{r_1}^{r_2}f(x)dx=r_2^2-r_1^2 \] 显然这个概率正是需要的概率。 当然这并不能说明,按照这个分布,点落在任意等面积的区域上的概率都是相等的,现在就来完成这一点的证明。 在单位圆任意取一个区域 $D$,假如它在极坐标系下可以表示为 \[ \rho_1(\theta)<\rho<\rho_2(\theta) \quad \alpha<\theta<\beta \] 那么点落在这个区域上的概率就是 \begin{align} &\int\!\!\!\int_Df(t)d\sigma \\ &=\frac{1}{2}\int_{\alpha}^{\beta}d\theta\int_{\rho_1(\theta)}^{\rho_2(\theta)}f(t)dt \\ &=\frac{1}{2}\int_{\alpha}^{\beta}(\rho_2^2(\theta)-\rho_1^2(\theta))d\theta \end{align} 这个正好是这个区域的面积,这就说明上面的那个分布的确满足几何概率。 好了,分布有了,但是如何实现呢?现在的计算机都可以容易的实现区间 $[0,1]$ 上的均匀分布,关键如何用这个均匀分布去实现前面所需要的那个分布,也就是说,如何才能让一个过程产生一个数,这个过程必须使得产生的数小于任何数 $t(0<t<1)$ 的概率都是 $t^2$。 更让我感兴趣的问题是,如何实现一个指定分布函数的分布? 一个直观的想法就是利用 $[0,1]$ 上的均匀分布,假如随机变量 $s$ 服从这个均匀分布,然后我们的随机变量就作为一个 $s$ 的函数 $r=h(s)$,为了方便,假定 $h(s)$ 是一个单调增加的函数,那么 \[ F(t)=P(r<t)=P(h(s)<t)=P(s<h^{-1}(t))=h^{-1}(t) \] 这就说明了,要想随机变量 $r$ 的分布函数是 $F(t)$,只要把转换函数 $h(s)$ 取为分布函数 $F(t)$ 的反函数就可以了,至于 $h(s)$ 单调增加,因为分布函数本来就是单调不减的,只要给他加个严格增加的限制就可以了。这样就实现了一个指定分布函数的分布。
都市侠影 2# 2012-7-23 12:24
终于想出来了,不容易哇哇塞
都市侠影 3# 2012-7-24 13:22
好象这个也该放到高等数学版块?用到了分布函数这样的高级概率工具,汗……
kuing 4# 2012-7-26 00:04
3# 都市侠影 嗯,还是决定移过去了。 PS、\int\!\!\!\int 可以用 \iint 代之 \[ \text{\int\!\!\!\int \approx \iint : }\int\!\!\!\int \approx \iint \]
thread-623-1-2.html: no3no4
kuing 1# 2012-7-21 22:55
thread-627-1-7.html: [数论] 一个题目的不科学性
都市侠影 1# 2012-7-22 13:18
本帖最后由 都市侠影 于 2012-7-22 13:27 编辑 群里今天讨论了一个题,原题是这样的 今有一个正偶数 $a$,对任意的正整数 $n$,数 $A=1+a+a^2+\cdots+a^n$ 都是完全平方数,求证 $a$ 是8的倍数。 经过思考,发现题目的条件过于强烈了,事实上,只要 $1+a$ 是完全平方数,数 $a$ 就已经是8的倍数了。原因很简单,显然 $1+a$ 只能是一个奇数的平方:$1+a=(2m+1)^2$,于是 $a=4m(m+1)$ 当然是8的倍数。 这个题目的科学性引起了我的怀疑,凭直觉,感觉题目的条件是不可能的,于是探究发一番,果然证实了我的想法。 把问题变个说法:一个首项为1,公比是正偶数的等比数列,它的每一个前n项和都是完全平方数,这样的数列存在吗? 从最简单的做起,首先这些前n项和当然只能是奇数的平方,设 $S_n=m_n^2$,那么因为 $S_2=1+a=m_2^2$,$S_3=1+a+a^2=S_2+a^2=m_3^2$,所以 $m_2^2+a^2=m_3^2$,这说明 $m_2$、$a$、$m_3$ 是一组勾股数,在网上搜到了一下勾股方程 $x^2+y^2=z^2$ 的正整数解: \begin{align} \begin{cases} x=2rs \\ y=r^2-s^2 \\ z=r^2+s^2 \end{cases} \end{align} 这里 $r>s>0$,一奇一偶并且互素。 利用这个结果,因为勾股方程的左边两个平方和必须是一奇一偶(看那个通解),所以在方程 $m_2^2+a^2=m_3^2$ 中,自然就必须是 \begin{align} \begin{cases} a=2rs \\ m_2=r^2-s^2 \\ m_3=r^2+s^2 \end{cases} \end{align} 但是注意到 $m_2^2=1+a$,所以 \begin{align} \begin{cases} a=2rs \\ 1+a=(r^2-n^2)^2 \end{cases} \end{align} 如此一来得 \[ 1+2rs=(r^2-s^2)^2 \] 这个是不可能的,因为 \begin{align} 1+2rs&=(r^2-s^2)^2 \\ &=(r+s)^2(r-s)^2 \\ &\geqslant (r+s)^2 \end{align} 于是得到 \[ r^2+s^2\leqslant 1 \] 这个可是神仙都做不到啊,所以只前三个和就不可能都是完全平方数,更不用说所有的和都是完全平方数了。
kuing 2# 2012-7-22 13:32
嗯,能使 $1+a+a^2$ 为完全平方数的整数 $a$ 貌似只有 $0$ 和 $-1$
都市侠影 3# 2012-7-22 13:36
啊,对,我搞得太复杂了,当 $a$ 是一个正整数时,$1+a+a^2$ 是不可能为完全平方数的了,因为 \[ a^2<1+a+a^2<(1+a)^2 \]
kuing 4# 2012-7-22 13:45
3# 都市侠影 嗯,是这正整数情况的最简单说明了。 如果考虑 0 和负整数的话,也可以用笨方法证,设 $1+a+a^2=m^2$,其中 $m\in\mathbb N^+$,则 \[a=\frac{-1\pm\sqrt{4m^2-3}}2,\] 那么 $4m^2-3$ 必须为完全平方数,而易证当 $m>1$ 时有 $(2m)^2>4m^2-3>(2m-1)^2$,所以只有 $m=1$ 才符合,此时 $a=0$ 或 $-1$。
都市侠影 5# 2012-7-22 13:50
考虑负整数也可以简单点嘛 \[ (1+a)^2<1+a+a^2\leqslant a^2 \] 注意到这里 $|1+a|<|a|$
kuing 6# 2012-7-22 13:52
我也跳去……
都市侠影 7# 2012-7-22 13:53
thread-63-1-2.html: 低潮期
kuing 1# 2011-10-5 13:34
低潮期相信每个人都会遇到,或多或少,或长或短,难以预计 在谷底的感觉,不动,就不会动,因为势能最小原理,所以可以完全放松,什么都不理——颓废 没有足够能量,是出不去的,正所谓:$E_k\geqslant \Delta E_p + E_f$,所以需要一件不知道是什么也不知道何时会发生的事来打破局面 不过在谷底又不一定什么都干不成,本论坛正是我处于谷底的时候由于无聊而乱逛乱想乱试进而意外顺利地搞了出来的,所以我反而开始欣赏这谷底 然而事件往往总是会出人意料,我还能欣赏多久?根本无法预计,正所谓:一切尽不在掌握之中!
kuing 2# 2011-10-6 14:21
为什么要给我这么好的梦
甄术 3# 2011-10-13 15:14
做了好梦却哭?
kuing 4# 2011-10-13 15:25
3# 甄术 因为只能是梦,现实中没有了
甄术 5# 2011-10-13 19:07
也因为是梦才美好的让人想哭?
戊概念·五 6# 2011-12-24 12:07
为什么要给我这么好的梦 kuing 发表于 2011-10-6 14:21 昨晚定然是能笑着醒来的美梦吧~
kuing 7# 2012-5-28 00:13
这篇东西,也总是偶尔很应景的,所以来顶一顶
kuing 8# 2012-7-31 17:20
这次何以如此久也未遇打破局面的事?
thread-631-1-7.html: [几何] 和垂心与外心有关的平几题
tan9p 1# 2012-7-22 17:23
三角形ABC,H是垂心,O是外心,BD=DP,求证OD 垂直 DQ 多谢。
kuing 2# 2012-7-22 20:32
看着有点想建系……不过这就没什么意思了…… 纯平几暂时没想出来
kuing 3# 2012-7-22 21:17
证到了,不过辅助线有点麻烦。 过程有点略,那些角、等式什么的为何相等自己想想。 有空再想想有没有简单点的。
tan9p 4# 2012-7-22 22:13
多谢! 好像和96年的一道预选题有关。 ABC 是锐角三角形 BC > AC,O是外心,H是垂心。CF是高,过F作PF垂直OF交AC与P,求证角FHP = 角 A. http://mks.mff.cuni.cz/kalva/short/sh96.html
tan9p 5# 2012-7-22 22:15
BTW QQ账号绑定功能是群主自己加的还是Discuz的插件?
kuing 6# 2012-7-22 22:26
4# tan9p 噢,这个就是反过来了?能逆推么?
kuing 7# 2012-7-22 22:28
5# tan9p 不知道,不是我弄的,5d6d不知啥时候有了这个功能
thread-632-1-1.html: MaxValue与Maximize
kuing 1# 2012-7-23 02:25
其实我一直都不知道这两个命令有什么重要区别,除了后者能把取最值时的变量值也输出之外,两者的用法什么的都没什么不同,这样前者岂不是有点多余?
kuing 2# 2012-7-23 02:42
唯一想到的原因大概是由于前者只关注最大值,故算法不同,适用范围可能要大些?或者速度也快些?
①②③④⑤⑥⑦ 3# 2012-7-23 15:05
帮助文档: MaxValue[...] is effectively equivalent to First[Maximize[...]]. 主体算法估计是一样的,效率不清楚。存在意义,我想是便于嵌套在其他运算中。
thread-633-1-7.html: [不等式] 以前的账号忘记了,现在在外地出差ing,重新注册了一个,贴下几个题目
yizhong 1# 2012-7-23 04:49
1, 设 $a,b,c$ 为非负数,并且 $a+b+c=3$, 试求:$(3a^2+bc+3b^2)(3b^2+ca+3c^2)(3c^2+ab+3a^2)$ 的最大值。 2, 设 $x,y,z$ 为非负数,并且 $x+y+z=1$, 证明:$\sqrt{x+y^2}+\sqrt{y+z^2}+\sqrt{z+x^2}\geqslant2$ 3, 设 $x,y,z$ 为非负数,并且 $x+y+z=1$, 证明:$\sqrt[3]{x-y+z^3}+\sqrt[3]{y-z+x^3}+\sqrt[3]{z-x+y^3}\leqslant1$
thread-634-1-7.html: [不等式] can的老问题
yizhong 1# 2012-7-23 08:37
设 $a, b, c>0$, 并且 $a+b+c=1$, 证明:$\dfrac{36}{a^2b+b^2c+c^2a}+\dfrac{1}{abc}\geqslant343$
kuing 2# 2012-7-24 02:58
其实我刚开本坛时也贴过……http://kkkkuingggg.5d6d.net/thread-60-1-1.html
thread-635-1-7.html: [不等式] 一道条件比较诡异的求最大值问题
yizhong 1# 2012-7-23 10:34
试求 $p(a,b,c)=a+b+c+a^2+b^2+c^2$ 在以下条件限制下的最大值: $\left\{ \begin{aligned} &0<c\leqslant b\leqslant a\leqslant 4\\ &3abc\leqslant\min\{6a+8b+12c,72\}\\ &2ab\leqslant\min\{3a+4b,24\} \end{aligned} \right.$
yizhong 2# 2012-7-23 11:59
谢谢K神帮我修改了下,改去了那别扭的中文输入格式
kuing 3# 2012-7-23 12:08
昨天等了你一天都没上来贴题
yizhong 4# 2012-7-23 12:14
我是看好着等你下了线之后才上来开始贴题,先谢谢K神帮我修改原来输入中的不足之处,最后K神还得常常过来这里泡茶,因为接下来我会继续贴,嘿嘿
都市侠影 5# 2012-7-23 12:39
这个问题用初等数学知识的范围恐怕过于复杂,用拉格朗日条件极值处理吧。
kuing 6# 2012-7-23 14:02
4# yizhong 还是别用“神”了受不起……
thread-636-1-7.html: [几何] 一道教材上的平几题
二月鸣 1# 2012-7-23 16:47
本帖最后由 二月鸣 于 2012-7-23 16:48 编辑 AE是三角形ABC的高,点F是AE上任意一点,延长BF交AC于H,CF交AB于G,求证:角GEA=角HEA
kuing 2# 2012-7-23 16:52
FAQ了,最常见的是延长EG、EH交过A与BC的平分线于。。。然后由比例和ceva什么的证相等
kuing 3# 2012-7-23 16:54
isea 在线,他对这个题应该有研究之至了
转化与化归 4# 2012-7-23 20:30
一道教材上的平几题
kuing 5# 2012-7-23 20:33
4# 转化与化归 解析法niubility
isea 6# 2012-7-24 14:38
2# kuing 本质是就是这样子了,教材上就是要用解析法,(解析法,bbs.pep.com.cn 那边是yes94兄写的)。 其实是高等几何的调和点列(和调和线束)内容。 bbs.pep.com.cn 好了后,数学兴趣小组有总结帖。
kuing 7# 2012-7-24 14:44
6# isea 贴子叫什么名字,我可以翻出来
yayaweha 8# 2012-7-24 14:46
顺带问下人教论坛什么时候更新好
kuing 9# 2012-7-24 14:47
8# yayaweha 不知道……
isea 10# 2012-7-24 14:55
本帖最后由 isea 于 2012-7-24 14:58 编辑 6# isea 贴子叫什么名字,我可以翻出来 kuing 发表于 2012-7-24 14:44 "终于找到此题大约源头了——三角形垂足为顶点证角相等"——http://bbs.pep.com.cn/thread-2371085-1-1.html 对于高等几何相关(因为看反演变换时,极线极点也涉及到调和比例...)也翻出来了 这个只是用了下概念,比较详细的证明:(例5) http://wenku.baidu.com/view/3b67483a5727a5e9856a6135.html 直接用内外角平分线与调和比例(充要条件),用得非常彻底:(例7) http://wenku.baidu.com/view/2fa5b444336c1eb91a375dd9.html 不过,我最早看到的,要通俗一些,不是这两个,一时找不到了...... =============================== 此题的推广: http://wenku.baidu.com/view/4c8cc42d2af90242a895e586.html
kuing 11# 2012-7-24 15:01
嗯,找到了,贴上附件。
isea 12# 2012-7-24 15:05
本帖最后由 isea 于 2012-7-24 15:07 编辑 11# kuing 要是能把,这个链接http://bbs.pep.com.cn/thread-840589-1-1.html你的及跟帖的纯几何法也弄过来的话,就非常完整了。就是真正的总结帖。
kuing 13# 2012-7-24 15:10
12# isea 要给标题我才好找,我这里存档的时候已经没了链接地址
isea 14# 2012-7-24 15:13
本帖最后由 isea 于 2012-7-24 15:15 编辑 12# isea 要给标题我才好找,我这里存档的时候已经没了链接地址 kuing 发表于 2012-7-24 15:10 初中数学论坛 » 一道几何题 标题是:一道几何题 baidu缓存 http://cache.baidu.com/c?m=9f65c ... 1709625326&p1=1 PS:k备份真是个好习惯!
kuing 15# 2012-7-24 15:18
14# isea 先回复你的PS,其实我以前就呼吁过大家有空就多存档,因为网络的东东总是很难说的。 事实上,你所有回过的贴的99%都在我手里
kuing 16# 2012-7-24 15:28
14# isea 刚才搞错了,原来存你的那些还是有tid在的(也就是能通过链接地址找到相应存档),只是我自己的那些才是没了的。
thread-637-1-1.html: [物理]来自群的物理题
kuing 1# 2012-7-23 17:35

kuing 2# 2012-7-23 17:38
水平方向只有 $v_0$ 不用看,下面考虑竖直方向,设某时刻线框因磁场力而所受阻为 $F_B$,竖直方向的速度为 $v_z$,则有 \[ F_B = \Delta BIL=\Delta B\frac ERL=\Delta B\frac{\Delta BLv_z}RL=\frac{(\Delta B)^2L^2v_z}R, \] 而 $\Delta B =B_0+k(z+L)-(B_0+kz)=kL$,故 \[F_B = \frac{k^2L^4v_z}R,\] 即是水平方向匀速,竖直方向为阻力正比于速度的“自由”落体。由此得 \[\frac{d v_z}{dt} = g-\frac{F_B}{m}=g-\frac{k^2L^4v_z}{mR},\] 要不要解微分方程?
kuing 3# 2012-7-23 18:12
还是不解微分方程了,突然想起阻力正比于速度是有个关于动量的结论的,于是…… 只考虑竖直方向的动量变化,用 $S_z$ 表示竖直方向上的位移,用 $S$ 表示第三问所求的位移,有 \begin{align*} m\sqrt{v_2^2-v_0^2}&=\int_0^t{(mg-F_B)dt} \\ & =\int_0^t{\left( mg-\frac{k^2L^4v_z}R \right)dt} \\ & =mgt-\int_0^t{\frac{k^2L^4\frac{dS_z}{dt}}Rdt} \\ & =mgt-\int_0^{\sqrt{S^2-(v_0t)^2}}{\frac{k^2L^4}RdS_z} \\ & =mgt-\frac{k^2L^4}R\sqrt{S^2-(v_0t)^2} \end{align*} 这样就可以把 $S$ 解出来……
yayaweha 4# 2012-7-23 20:27
高中的物理题不会用到微分方程吧!
yayaweha 5# 2012-7-23 20:28
顶多用到微元法,根本没用大学的东西
kuing 6# 2012-7-23 20:33
4# yayaweha 所以我并没有解那个微分方程啊,而是考虑动量去了,也就有了后面的解答,只不过还是用了微积分。 5# yayaweha 其实微元法可以看成是不太严格的微积分法,3#后面的过程也可以改写成微元法。
thread-638-1-3.html: [数论] 开个数论的帖子
yizhong 1# 2012-7-23 17:52
1,求方程 $x(x+1)(2x+1)=6y^2$ 的所有正整数组解 $(x,y)$, 使得 $x$ 是偶数。 2,求方程 $x^n+2^{n+1}=y^{n+1}$ 的所有正整数组解 $(x,y,n)$, 满足以下两个条件: (1)$x$ 为奇数,(2)$x$, $n+1$ 互质。 3,试求下述方程的所有有序正整数组解 $(x,y,z)$, 使得 $\Large x^{y^z}y^{z^x}z^{x^y}=1990^{1990}xyz$ ____kuing edit 将第三题后面的式子放大一些,方便看清楚____
yizhong 2# 2012-7-23 20:54
谢谢小K,这些问题陆续将答案敲上来,帖子可不要沉勒,嘿嘿。
kuing 3# 2012-7-23 20:56
慢慢来,答案别发这么快……虽然我估计我做不来……但是还有别的高手……
零定义 4# 2013-3-17 00:04
1、(Lucas猜想)x为偶数时,有且只有一组正整数解(24,70).
kuing 5# 2013-3-17 00:11
4# 零定义 还是知道不少的嘛,睡神
零定义 6# 2013-3-17 18:54
5# kuing 俺只懂得当睡神~
realnumber 7# 2013-3-18 16:27
第3题.两边除以$xyz$ 若x,y,z中有2个1,形如$(1990^{1990},1,1)$的3个解. 若有1个1,这样的解$(1,1990,1990+1),(1,1990^5,199\times2+1),(1,1990^{10},199+1),(1,1990^2,199\times5+1)$24个解. 199为质数,不妨设x=199,y,z不小于2,则容易得左边大于右边,..无解.
yes94 8# 2013-3-18 17:33
慢慢来,答案别发这么快……虽然我估计我做不来……但是还有别的高手…… kuing 发表于 2012-7-23 20:56 这么久了,他连答案也不发!
realnumber 9# 2013-3-19 15:34
第2题(未解决,只是特殊到一般的尝试) 当n=1时,$(y^2-4,y,1),y=2k-1,k\in Z^+$是方程的解. 当n=2时,$x^2=y^3-8=(y-2)(y^2+2y+4),y^3=[(y-2)+2]^3$ \[(y-2,y^2+2y+4)=(y-2,4C_3^1)=(y-2,3),若(y-2,3)=1,得到y-2=s^2,且y^2+2y+4=t^2,s,t\in Z^+ ,\] \[ 得到3=t^2-(y+1)^2=(t-y-1)(t+y+1),t-y-1=1,且t+y+1=3矛盾. \] \[若(y-2,3)=3,得到y-2=3s^2,且y^2+2y+4=3t^2,s,t\in Z^+ ,得到3t^2+1=(3s^2+3)^2,\mod3下矛盾.所以n=2无解.\] 当n=3时,$x^3=y^4-2^4=(y-2)(y^3+2y^2+4y+8),y^4=[(y-2)+2]^4$, \[(y-2,y^3+2y^2+4y+8)=(y-2,8C_4^1)=1,所以y-2=s^3,y^3+2y^2+4y+8=t^3\] \[y,t为正奇数,y^3+2y^2+4y+8<(y+2)^3,y^3+2y^2+4y+8>y^3,所以无解.即n=3无解.\]
realnumber 10# 2013-3-19 16:32
9# realnumber 第三题一般情况如下 $x^n=(y-2)(y^n+2y^{n-1}+\cdots+2^n),(y-2,y^n+2y^{n-1}+\cdots+2^n)=(y-2,2^{n}C_{n+1}^1)=(y-2,n+1)$, 又$(x,n+1)=1$,所以 $y^n+2y^{n-1}+\cdots+2^n=s^n$,s=y,y+2尝试下就可以说明无解.
yes94 11# 2013-3-19 19:43
10# realnumber 专研精神很强!赞一个!
realnumber 12# 2013-3-19 22:51
第一题(只处理了部分) x为偶数,设$x=2t,t\in Z^+$,那么方程为$t(2t+1)(4t+1)=3y^2$, 以下对t按$\mod 3$分类, (1)当$t=3k+1,k\in Z$,$(3k+1)(2k+1)(12k+5)=y^2$,注意到$x,x+1,2x+1$两两互素,那么$3k+1,2k+1,12k+5$也两两互素,即有$3k+1=a^2,2k+1=b^2,12k+5=c^2,a,b,c\in Z$,最后一个为$2\equiv1\mod3 $,矛盾. (2)当$t=3k+2,k\in Z$,$(3k+2)(6k+5)(4k+3)=y^2$,同样有$3k+2,6k+5,4k+3$两两互素,那么$3k+2=a^2,6k+5=b^2,4k+3=c^2,a,b,c\in Z$,第二个为$2\equiv1\mod3 $,矛盾. (3)当$t=3k,k\in Z$,$k(6k+1)(12k+1)=y^2$,那么$k,6k+1,12k+1$也两两互素,有$k=a^2,6k+1=b^2,12k+1=c^2,a,b,c\in Z$ 暂时没想出办法,k=4显然是解,也许会与佩尔方程处理办法有关,$b^2-6a^2=1,c^2-12a^2=1$,明天翻翻那些内容.
yes94 13# 2013-3-19 23:12
13# realnumber 只看最后一段:佩尔方程,有公式的,或者用递推数列来表达
realnumber 14# 2013-3-20 22:00
12# realnumber 也不清楚细节对不对,可得$6a^2=c^2-b^2=(c-b)(c+b)$模仿不定方程$x^2+y^2=z^2$,$可解得a^2=4x^2y^2,b^2=(6x^2-y^2)^2,c^2=(6x^2+y^2)^2,x为正整数,y为正奇数$.代入$6a^2+1=b^2$,化简得到$36x^4-36y^2x^2+y^4-1=0$,看作$x^2$的二次方程,其判别式必须为完全平方,得到$8y^4+1=m^2,m\in Z^+,m为奇数$,即$8y^4=(m+1)(m-1)$,又$(m+1,m-1)=2$,所以有$m-1=2t^4,m+1=4s^4$或$m-1=4t^4,m+1=2s^4$,消去m,得到$1+t^4=2s^4$或$1+2t^4=s^4$,s,t为正整数.(前者有解(1,1),如果能证明就这个唯一解,那么问题解决,当然不成立,也有可能,推理过程不是一直等价的.) $2t^4=(s^2-1)(s^2+1),(s^2-1,s^2+1)=2$,所以$s^2+1=2m^4,s^2-1=n^4或s^2-1=2m^4,s^2+1=n^4,(m,n)=1,mn=t,m,n\in Z^+$,即$±1=s^2-(n^2)^2$这与两不等正整数平方差绝对值最小为3矛盾. 只留下这个问题了$1+t^4=2s^4$是否有唯一解(1,1).
yes94 15# 2013-3-20 22:24
13# realnumber 只看最后一段:佩尔方程,有公式的,或者用递推数列来表达 yes94 发表于 2013-3-19 23:12 佩尔方程$b^2-6a^2=1$的最小正整数解:$a=2,b=5$,根据佩尔方程理论,可得\[a=\dfrac{(5+2\sqrt6)^n-(5-2\sqrt6)^n}{2\sqrt6},\]\[b=\dfrac{(5+2\sqrt6)^n+(5-2\sqrt6)^n}{2},\] 负整数解略去。 对于佩尔方程$c^2-12a^2=1$同理,这里也略去。
realnumber 16# 2013-3-21 20:45
$1+t^4=2s^4$是否有唯一解(1,1).百撕不得骑姐.
零定义 17# 2013-3-21 22:07
16# realnumber Ljunggren方程的一种情形,x^2-2y^4=-1仅有两组正整数解(1,1),(239,13)
realnumber 18# 2013-3-21 22:59
17# 零定义 谢谢,按提示去百度,水很深,看来得放一放了。
yes94 19# 2013-3-22 18:28
$1+t^4=2s^4$是否有唯一解(1,1).百撕不得骑姐. realnumber 发表于 2013-3-21 20:45 要干嘛?撕?骑?
thread-638-2-1.html: [数论] 开个数论的帖子
yizhong 1# 2012-7-23 17:52
1,求方程 $x(x+1)(2x+1)=6y^2$ 的所有正整数组解 $(x,y)$, 使得 $x$ 是偶数。 2,求方程 $x^n+2^{n+1}=y^{n+1}$ 的所有正整数组解 $(x,y,n)$, 满足以下两个条件: (1)$x$ 为奇数,(2)$x$, $n+1$ 互质。 3,试求下述方程的所有有序正整数组解 $(x,y,z)$, 使得 $\Large x^{y^z}y^{z^x}z^{x^y}=1990^{1990}xyz$ ____kuing edit 将第三题后面的式子放大一些,方便看清楚____
yizhong 2# 2012-7-23 20:54
谢谢小K,这些问题陆续将答案敲上来,帖子可不要沉勒,嘿嘿。
kuing 3# 2012-7-23 20:56
慢慢来,答案别发这么快……虽然我估计我做不来……但是还有别的高手……
零定义 4# 2013-3-17 00:04
1、(Lucas猜想)x为偶数时,有且只有一组正整数解(24,70).
kuing 5# 2013-3-17 00:11
4# 零定义 还是知道不少的嘛,睡神
零定义 6# 2013-3-17 18:54
5# kuing 俺只懂得当睡神~
realnumber 7# 2013-3-18 16:27
第3题.两边除以$xyz$ 若x,y,z中有2个1,形如$(1990^{1990},1,1)$的3个解. 若有1个1,这样的解$(1,1990,1990+1),(1,1990^5,199\times2+1),(1,1990^{10},199+1),(1,1990^2,199\times5+1)$24个解. 199为质数,不妨设x=199,y,z不小于2,则容易得左边大于右边,..无解.
yes94 8# 2013-3-18 17:33
慢慢来,答案别发这么快……虽然我估计我做不来……但是还有别的高手…… kuing 发表于 2012-7-23 20:56 这么久了,他连答案也不发!
realnumber 9# 2013-3-19 15:34
第2题(未解决,只是特殊到一般的尝试) 当n=1时,$(y^2-4,y,1),y=2k-1,k\in Z^+$是方程的解. 当n=2时,$x^2=y^3-8=(y-2)(y^2+2y+4),y^3=[(y-2)+2]^3$ \[(y-2,y^2+2y+4)=(y-2,4C_3^1)=(y-2,3),若(y-2,3)=1,得到y-2=s^2,且y^2+2y+4=t^2,s,t\in Z^+ ,\] \[ 得到3=t^2-(y+1)^2=(t-y-1)(t+y+1),t-y-1=1,且t+y+1=3矛盾. \] \[若(y-2,3)=3,得到y-2=3s^2,且y^2+2y+4=3t^2,s,t\in Z^+ ,得到3t^2+1=(3s^2+3)^2,\mod3下矛盾.所以n=2无解.\] 当n=3时,$x^3=y^4-2^4=(y-2)(y^3+2y^2+4y+8),y^4=[(y-2)+2]^4$, \[(y-2,y^3+2y^2+4y+8)=(y-2,8C_4^1)=1,所以y-2=s^3,y^3+2y^2+4y+8=t^3\] \[y,t为正奇数,y^3+2y^2+4y+8<(y+2)^3,y^3+2y^2+4y+8>y^3,所以无解.即n=3无解.\]
realnumber 10# 2013-3-19 16:32
9# realnumber 第三题一般情况如下 $x^n=(y-2)(y^n+2y^{n-1}+\cdots+2^n),(y-2,y^n+2y^{n-1}+\cdots+2^n)=(y-2,2^{n}C_{n+1}^1)=(y-2,n+1)$, 又$(x,n+1)=1$,所以 $y^n+2y^{n-1}+\cdots+2^n=s^n$,s=y,y+2尝试下就可以说明无解.
yes94 11# 2013-3-19 19:43
10# realnumber 专研精神很强!赞一个!
realnumber 12# 2013-3-19 22:51
第一题(只处理了部分) x为偶数,设$x=2t,t\in Z^+$,那么方程为$t(2t+1)(4t+1)=3y^2$, 以下对t按$\mod 3$分类, (1)当$t=3k+1,k\in Z$,$(3k+1)(2k+1)(12k+5)=y^2$,注意到$x,x+1,2x+1$两两互素,那么$3k+1,2k+1,12k+5$也两两互素,即有$3k+1=a^2,2k+1=b^2,12k+5=c^2,a,b,c\in Z$,最后一个为$2\equiv1\mod3 $,矛盾. (2)当$t=3k+2,k\in Z$,$(3k+2)(6k+5)(4k+3)=y^2$,同样有$3k+2,6k+5,4k+3$两两互素,那么$3k+2=a^2,6k+5=b^2,4k+3=c^2,a,b,c\in Z$,第二个为$2\equiv1\mod3 $,矛盾. (3)当$t=3k,k\in Z$,$k(6k+1)(12k+1)=y^2$,那么$k,6k+1,12k+1$也两两互素,有$k=a^2,6k+1=b^2,12k+1=c^2,a,b,c\in Z$ 暂时没想出办法,k=4显然是解,也许会与佩尔方程处理办法有关,$b^2-6a^2=1,c^2-12a^2=1$,明天翻翻那些内容.
yes94 13# 2013-3-19 23:12
13# realnumber 只看最后一段:佩尔方程,有公式的,或者用递推数列来表达
realnumber 14# 2013-3-20 22:00
12# realnumber 也不清楚细节对不对,可得$6a^2=c^2-b^2=(c-b)(c+b)$模仿不定方程$x^2+y^2=z^2$,$可解得a^2=4x^2y^2,b^2=(6x^2-y^2)^2,c^2=(6x^2+y^2)^2,x为正整数,y为正奇数$.代入$6a^2+1=b^2$,化简得到$36x^4-36y^2x^2+y^4-1=0$,看作$x^2$的二次方程,其判别式必须为完全平方,得到$8y^4+1=m^2,m\in Z^+,m为奇数$,即$8y^4=(m+1)(m-1)$,又$(m+1,m-1)=2$,所以有$m-1=2t^4,m+1=4s^4$或$m-1=4t^4,m+1=2s^4$,消去m,得到$1+t^4=2s^4$或$1+2t^4=s^4$,s,t为正整数.(前者有解(1,1),如果能证明就这个唯一解,那么问题解决,当然不成立,也有可能,推理过程不是一直等价的.) $2t^4=(s^2-1)(s^2+1),(s^2-1,s^2+1)=2$,所以$s^2+1=2m^4,s^2-1=n^4或s^2-1=2m^4,s^2+1=n^4,(m,n)=1,mn=t,m,n\in Z^+$,即$±1=s^2-(n^2)^2$这与两不等正整数平方差绝对值最小为3矛盾. 只留下这个问题了$1+t^4=2s^4$是否有唯一解(1,1).
yes94 15# 2013-3-20 22:24
13# realnumber 只看最后一段:佩尔方程,有公式的,或者用递推数列来表达 yes94 发表于 2013-3-19 23:12 佩尔方程$b^2-6a^2=1$的最小正整数解:$a=2,b=5$,根据佩尔方程理论,可得\[a=\dfrac{(5+2\sqrt6)^n-(5-2\sqrt6)^n}{2\sqrt6},\]\[b=\dfrac{(5+2\sqrt6)^n+(5-2\sqrt6)^n}{2},\] 负整数解略去。 对于佩尔方程$c^2-12a^2=1$同理,这里也略去。
realnumber 16# 2013-3-21 20:45
$1+t^4=2s^4$是否有唯一解(1,1).百撕不得骑姐.
零定义 17# 2013-3-21 22:07
16# realnumber Ljunggren方程的一种情形,x^2-2y^4=-1仅有两组正整数解(1,1),(239,13)
realnumber 18# 2013-3-21 22:59
17# 零定义 谢谢,按提示去百度,水很深,看来得放一放了。
yes94 19# 2013-3-22 18:28
$1+t^4=2s^4$是否有唯一解(1,1).百撕不得骑姐. realnumber 发表于 2013-3-21 20:45 要干嘛?撕?骑?
thread-639-1-7.html: 开个多项式帖子
yizhong 1# 2012-7-23 18:32
1,正整数 $n\geqslant3$,$k$是正整数,$p_1,p_2,\ldots,p_k$ 是 $k$ 个不同的素数,求所有整数 $a$, 使得 $f(x)=x^n+ax^{n-1}+p_1p_2\ldots p_k$ 能够分解为两个次数都大于等于1的整系数多项式的乘积。 2,正整数 $m$, $n$ 都大于等于2,并且 $m$, $n$ 互质,证明:$x^{(m-1)n}+x^{(m-2)n}+\cdots+x^{2n}+x^n+1$ 能够分解为 $x^{m-1}+x^{m-2}+\cdots+x^2+x+1$ 及另一个整系数多项式的乘积。 3,求所有正整数 $k\geqslant2$,使得多项式 $x^{2k+1}+x+1$ 能够分解为多项式 $x^k+x+1$ 与一个整系数 多项式的乘积。对每个这样的 $k$,求所有正整数 $n$,使得 $x^n+x+1$ 能够分解为多项式 $x^k+x+1$ 与另一个整系数 多项式的乘积。
thread-64-1-1.html: 粗体测试成功
kuing 1# 2011-10-5 13:45
$\textbf{a}=(a_1,a_2)$ $\bf{b}=(b_1,b_2)$ ${\bf b}=(b_1,b_2)$ 在真 latex 里可以用宏包 \usepackage{bm} \bm a $\boldsymbol{a}=(a_1,a_2)$ $\boldsymbol b=(b_1,b_2)$ 成功!
thread-640-1-7.html: [不等式] 贴道不等式题
yayaweha 1# 2012-7-23 22:19
本帖最后由 yayaweha 于 2012-7-25 18:55 编辑 已知$a_{n+1}=a_n^2-a_n+1$ 求证$$\frac1{a_1}+\frac1{a_2}+\frac1{a_3}+……+\frac1{a_n}>1-\frac1{n^n}$$ 求助呀!! ____kuing edit____ 公式里最好不要有全角字符,省略号用 \cdots 或 \ldots,你的大于号也是全角的,应切换到纯英文的状态下输入公式。 $$\frac1{a_1}+\frac1{a_2}+\frac1{a_3}+\cdots+\frac1{a_n}>1-\frac1{n^n}$$
kuing 2# 2012-7-23 22:22
又是为个递推哟,可以参考这贴 http://kkkkuingggg.5d6d.net/thread-562-1-1.html 的解答的第四行之后的东东
yayaweha 3# 2012-7-23 22:26
最后面不一样 一个是$2^k$ 一个是$n^n$
kuing 4# 2012-7-23 22:27
我没说一样,只是说参考,照着试试好了 对了,你忘记给首项。
yayaweha 5# 2012-7-23 22:28
4# kuing $a_1=2$
yayaweha 6# 2012-7-23 22:30
其实原不等式有两边,另一边就是用这个方法证那一串<1,我觉得现在这个应该要换个递推关系证
kuing 7# 2012-7-23 22:31
学那边那样最后用归纳法可以么?
yayaweha 8# 2012-7-23 22:32
maybe
yayaweha 9# 2012-7-24 08:18
找到原题了 http://wenku.baidu.com/view/0f044c1ca8114431b90dd80a.html 第五题
yayaweha 10# 2012-7-24 08:21
本帖最后由 yayaweha 于 2012-7-24 10:44 编辑 由递推得$a_{n+1}-1=a_n(a_n-1)$一直迭代下去就有$a_{n+1}-1=a_na_{n-1}\cdots a_1(a_1-1)$ 又$a_1=2$所只需证$a_1a_2\cdots a_n > n^n$ 怎么用归纳法证明$a_1a_2\cdots a_n > n^n$
yayaweha 11# 2012-7-24 10:31
本帖最后由 yayaweha 于 2012-7-24 10:45 编辑 仔细想了一下,其实也不用数学归纳法 对$a_1a_2\cdots a_n $用均值不等式$G_n>H_n$几何平均大于调和平均再结合 $$\frac1{a_1}+\frac1{a_2}+\frac1{a_3}+……+\frac1{a_n}<1$$ 直接得到$a_1a_2\cdots a_n > n^n$
都市侠影 12# 2012-7-24 13:33
本帖最后由 都市侠影 于 2012-7-24 13:34 编辑 这还不简单吗 假定 $a_1a_2\cdots a_k>k^k$,那么 \begin{align} &a_1a_2\cdots a_ka_{k+1} \\ >&k^ka_{k+1} \end{align} 所以只要证明 \[ a_{k+1}>\frac{(k+1)^{(k+1)}}{k^k}=(k+1)(1+\frac{1}{k})^k \] 这个更容易了嘛,因为 \[ (1+\frac{1}{k})^k<3 \]
yayaweha 13# 2012-7-24 14:37
接下来要证$a_n>3n$,这个要从$a_4$开始才成立
转化与化归 14# 2012-7-24 15:35
数学归纳
yizhong 15# 2012-7-24 17:03
有一年的IMO SHORTLIST也出现了类似的题,而且题目条件几乎一样。
yayaweha 16# 2012-7-24 18:24
15# yizhong 用$k^{k+1}>(k+1)^k$吗?,这个应该要另外证明,并且不是对任意的正整数,2以上才成立
kuing 17# 2012-7-24 18:30
16# yayaweha 取个对数搞个单调性神马的…… PS、是不是回错楼层……
都市侠影 18# 2012-7-24 20:54
13# yayaweha $a_4$ 成立那就用 $n=4$ 做奠基步嘛,又不是非用 $n=1$ 奠基不可。
thread-641-1-1.html: [不等式] 再问一道
yayaweha 1# 2012-7-23 22:49
本帖最后由 yayaweha 于 2012-7-24 19:31 编辑 $a_{n}$是方程$x^3+\frac xn=1的根$ 求证:$$\left\sum_{i=1}^n\frac1{(i+1)^2{a_i}}\right<a_n$$ _____kuing edit_____ $a_n$ 是方程 $x^3+\frac xn=1$的根 求证:$$\left(\sum_{i=1}^n\frac1{(i+1)^2{a_i}}\right)<a_n$$ \frac1/{{i+1}^2{a_i}} 这个我理解不上,猜测是如此这样子,如果哪里没写对,自己改
yayaweha 2# 2012-7-23 22:51
怎么会这样?
kuing 3# 2012-7-23 23:14
公式打不来可以传图,你那个写法我也很难猜到你想表达的式子,暂时编辑成现在一楼的那样子,如果哪里不一样,你自己修改下吧。
yayaweha 4# 2012-7-23 23:34
没错
kuing 5# 2012-7-24 01:43
4# yayaweha 呃,只是这样的话何必要两边的括号哩…… 话说,没什么想法……
都市侠影 6# 2012-7-24 12:36
本帖最后由 都市侠影 于 2012-7-24 13:19 编辑 这个也不难嘛,因为可以给这个根一个估计 \[ 1-\frac{1}{n+1}<a_n<1 \] 这个证明放在后面,有了这个,那么 \begin{align} &\sum_{i=1}^n\frac{1}{(i+1)^2a_i} \\ <&\sum_{i=1}^n\frac{1}{(i+1)^2(1-\frac{1}{i+1})} \\ =&\sum_{i=1}^n\frac{1}{i(i+1)} \\ =&1-\frac{1}{n+1} \end{align} 这样不就有 \[ \sum_{i=1}^n\frac{1}{(i+1)^2a_i}<a_n \] 了吗。 现在来证明刚开始那个不等式,要先知道 $0<a_n<1$,因为一旦有了这个,立即就有 \[ 1-\frac{a_n}{n}=a_n^3<a_n \] 也就是 \[ 1-\frac{1}{n+1}<a_n<1 \] 而 $0<a_n<1$ 这个就是显然的了,因为方程对应的函数 $f(x)=x^3+\frac{x}{n}-1$ 是增函数,而且 $f(0)<f(a_n)=0<f(1)$,所以有 $0<a_n<1$.阿弥陀佛,善哉善哉。
yizhong 7# 2012-7-24 15:54
本帖最后由 yizhong 于 2012-7-24 15:59 编辑 这个题目几乎可以直接秒了,由于$a_n(a_n^2+\dfrac{1}{n})=1$,所以我们可以得到:$a_n=\dfrac{1}{a_n^2+\dfrac{1}{n}}>\dfrac{1}{1+\dfrac{1}{n}}=\dfrac{n}{n+1}$,注意这里用到了$0<a_n<1$,而这个由已知条件是很容易得到的,以下从略。
kuing 8# 2012-7-24 16:02
7# yizhong 你慢了,你楼上已经写了
yizhong 9# 2012-7-24 16:19
昨天晚上我不知不觉睡着了,不过题目严密一点应该要加上$a_n$是实根这个条件。
thread-641-1-7.html: [不等式] 再问一道
yayaweha 1# 2012-7-23 22:49
本帖最后由 yayaweha 于 2012-7-24 19:31 编辑 $a_{n}$是方程$x^3+\frac xn=1的根$ 求证:$$\left\sum_{i=1}^n\frac1{(i+1)^2{a_i}}\right<a_n$$ _____kuing edit_____ $a_n$ 是方程 $x^3+\frac xn=1$的根 求证:$$\left(\sum_{i=1}^n\frac1{(i+1)^2{a_i}}\right)<a_n$$ \frac1/{{i+1}^2{a_i}} 这个我理解不上,猜测是如此这样子,如果哪里没写对,自己改
yayaweha 2# 2012-7-23 22:51
怎么会这样?
kuing 3# 2012-7-23 23:14
公式打不来可以传图,你那个写法我也很难猜到你想表达的式子,暂时编辑成现在一楼的那样子,如果哪里不一样,你自己修改下吧。
yayaweha 4# 2012-7-23 23:34
没错
kuing 5# 2012-7-24 01:43
4# yayaweha 呃,只是这样的话何必要两边的括号哩…… 话说,没什么想法……
都市侠影 6# 2012-7-24 12:36
本帖最后由 都市侠影 于 2012-7-24 13:19 编辑 这个也不难嘛,因为可以给这个根一个估计 \[ 1-\frac{1}{n+1}<a_n<1 \] 这个证明放在后面,有了这个,那么 \begin{align} &\sum_{i=1}^n\frac{1}{(i+1)^2a_i} \\ <&\sum_{i=1}^n\frac{1}{(i+1)^2(1-\frac{1}{i+1})} \\ =&\sum_{i=1}^n\frac{1}{i(i+1)} \\ =&1-\frac{1}{n+1} \end{align} 这样不就有 \[ \sum_{i=1}^n\frac{1}{(i+1)^2a_i}<a_n \] 了吗。 现在来证明刚开始那个不等式,要先知道 $0<a_n<1$,因为一旦有了这个,立即就有 \[ 1-\frac{a_n}{n}=a_n^3<a_n \] 也就是 \[ 1-\frac{1}{n+1}<a_n<1 \] 而 $0<a_n<1$ 这个就是显然的了,因为方程对应的函数 $f(x)=x^3+\frac{x}{n}-1$ 是增函数,而且 $f(0)<f(a_n)=0<f(1)$,所以有 $0<a_n<1$.阿弥陀佛,善哉善哉。
yizhong 7# 2012-7-24 15:54
本帖最后由 yizhong 于 2012-7-24 15:59 编辑 这个题目几乎可以直接秒了,由于$a_n(a_n^2+\dfrac{1}{n})=1$,所以我们可以得到:$a_n=\dfrac{1}{a_n^2+\dfrac{1}{n}}>\dfrac{1}{1+\dfrac{1}{n}}=\dfrac{n}{n+1}$,注意这里用到了$0<a_n<1$,而这个由已知条件是很容易得到的,以下从略。
kuing 8# 2012-7-24 16:02
7# yizhong 你慢了,你楼上已经写了
yizhong 9# 2012-7-24 16:19
昨天晚上我不知不觉睡着了,不过题目严密一点应该要加上$a_n$是实根这个条件。
thread-642-1-1.html: 不知何时为何多了“相关贴子”在主贴……
kuing 1# 2012-7-24 01:52
比如这贴 http://kkkkuingggg.5d6d.net/thread-641-1-1.html
kuing 2# 2012-7-24 01:54
咦,这贴怎么没有……
kuing 3# 2012-7-26 01:30
呃,有了……
thread-643-1-1.html: 那个搜索功能终于能用了
kuing 1# 2012-7-24 02:57
前段时间一直都搜不出东西来,今天突然发现OK了
thread-644-1-7.html: 一道高次方程
★_/ka_☆ 1# 2012-7-24 14:38
$x^5+x^4-5x^3-2x^2+4x-8=0$
kuing 2# 2012-7-24 14:41
观察法……$x=\pm2$ 是根
yayaweha 3# 2012-7-24 14:41
本帖最后由 yayaweha 于 2012-7-24 14:45 编辑 $x=2$一眼瞄到一个
kuing 4# 2012-7-24 14:53
其实根据那个什么定理,试整数根时也只需试 $\pm1$, $\pm2$, $\pm4$, $\pm8$,别的都不必试了。
yayaweha 5# 2012-7-24 14:55
4# kuing 先用试根法,再作大除法
thread-645-1-7.html: [不等式] 数月前在安zp空间里看到的一道不等式
kuing 1# 2012-7-24 18:39
数月前在安zp空间里(http://user.qzone.qq.com/363215694/blog/1332475335)看到如下不等式题: 不知是不是他原创,而这题难倒了不少高手,我自然也不会了……
kuing 2# 2012-7-24 18:43
后来 can 在 2012-4-18 给出如下证明 然后说“The proof is not very nice :">” 其实我想问,how to get $(2a+c)^2$ 的哩?and,大家有没有别的证法?
tan9p 3# 2012-7-25 19:43
求他的空间地址。
kuing 4# 2012-7-25 19:58
3# tan9p 已在一楼添加链接
thread-646-1-7.html: [函数] 一道函数与数列小题
xulinhe 1# 2012-7-25 13:03
15.设函数, 是公差不为0的等差数列, ,则
xulinhe 2# 2012-7-25 13:07

kuing 3# 2012-7-25 13:35
今年的高考题,参考 http://kkkkuingggg.5d6d.net/view ... &page=3#pid2656 22楼
xulinhe 4# 2012-7-25 14:07
3# kuing 没有   原解答过于繁琐  请教是否有简洁解答
nash 5# 2012-7-25 16:31
貌似这个方法也比较长
kuing 6# 2012-7-25 16:33
参考我所给链接那里自然就得到楼上的解答,楼主不知有没有真的参考过。
nash 7# 2012-7-25 16:40
汗… 我也没注意看 原来是一样的,抄袭~\(≧▽≦)/~啦啦啦
kuing 8# 2012-7-25 16:41
7# nash 不是一样,我那里写的是理科的,那个更复杂一些,不过思路显然是一样的。
thread-647-1-7.html: [函数] 周期函数小题
xulinhe 1# 2012-7-25 13:09
如下回复中的小题
xulinhe 2# 2012-7-25 13:10

yayaweha 3# 2012-7-25 13:39
本帖最后由 yayaweha 于 2012-7-25 13:41 编辑 2# xulinhe 周期为6                    $f(2010)=\frac{1}{2}$ 第一个周期为4 不过怎么f(2)不存在
kuing 4# 2012-7-25 13:43
哈哈,命题的也太大意了,f(2) 居然算不出来,所以 f(n)($n\in\mathbb N, n\geqslant2$)都不能确定了……
kuing 5# 2012-7-25 13:52
我擦,第二个也是错题,直接计算得:$a_2=2 - \sqrt3$, $a_3=1/\sqrt3$, $a_4=1$, $a_5=\sqrt3$, $a_6=2 + \sqrt3$,结果 $a_7$ 不存在,后面还算啥呢……
xulinhe 6# 2012-7-25 14:00
4# kuing 能否联系两角和正切公式?
yayaweha 7# 2012-7-25 14:01
6# xulinhe 可以
xulinhe 8# 2012-7-25 14:03
7# yayaweha 如何做? 能给详解? 谢
yayaweha 9# 2012-7-25 14:13
$ 令f(n)=tanθ_n $ $$tan(45°+θ_n)=\frac{tan45°+tanθ_n}{1-tan45°tanθ_n}=tanθ_{n+1}$$ _____kuing edit in 正确的 $\LaTeX$ 输入_____ 令 $f(n)=\tan\theta_n$ $$\tan(45^\circ+\theta_n)=\frac{\tan45^\circ+\tan\theta_n}{1-\tan45^\circ\tan\theta_n}=\tan\theta_{n+1}$$ (中文在公式外面,公式里面用半角符号(你原来的括号是全角),θ用代码\theta,度用 ^\circ,tan前面加反斜杠。)
kuing 10# 2012-7-25 14:34
9# yayaweha 嗯,由此也可以看出,当 $f(1)=1$,可得 $\theta_1=45^\circ$(懒得 ${}+k\pi$,无关重要),然后 $\tan\theta_2$ 就没了……
thread-648-1-7.html: 概率小题
xulinhe 1# 2012-7-25 13:13
在圆周上任取三点A,B,C, 则 三角形ABC为锐角三角形的概率是----- 在圆周上任取三点A,B,C, 则 三角形ABC为直角三角形的概率是----- 在圆周上任取三点A,B,C, 则 三角形ABC为钝角三角形的概率是-----
yayaweha 2# 2012-7-25 13:22
本帖最后由 yayaweha 于 2012-7-25 13:24 编辑 1# xulinhe 这个简单,几何概型1/2*1/2=1/4(锐角时)
kuing 3# 2012-7-25 13:33
FAQ了,1/4,0,3/4
xulinhe 4# 2012-7-25 14:04
3# kuing 具体的解答?
xulinhe 5# 2012-7-25 14:05
2# yayaweha 直角? 钝角?
thread-649-1-1.html: [不等式] 在数学空间上看到的一道数列不等式
yayaweha 1# 2012-7-25 13:52
本帖最后由 yayaweha 于 2012-7-25 13:57 编辑 设$b_n=\frac{2(n-1)}{n(n+1)}$ 证明:当n≥2时$$b_2^2+b_3^2+\cdots+b_n^2<1$$ 数学空间上讲的是积分放缩,问下有没有一般的方法!
kuing 2# 2012-7-25 13:56
哪一期上的? PS:说一下输入细节: 上下标同时存在时用 b_2^2 这样打就行了(显示为 $b_2^2$),而不必打成 {b_2}^2(显示为 ${b_2}^2$),上下标位置会有不同。 还有就是你那个小于号还是全角的……
yayaweha 3# 2012-7-25 13:57
2# kuing 第三期 战巡的那个
yayaweha 4# 2012-7-25 14:19
怎么没人?
kuing 5# 2012-7-25 14:37
4# yayaweha 我没什么好想法,从他那里的$f(x)$的分部出来的式子可以直接求和得到 \[b_2^2+b_3^2+\cdots+b_n^2=20\sum_{k=3}^n\frac1{k^2}-7+\frac{16}{(n+1)^2}+\frac{16}{1+n}\] 和式保存足够多项,余下放缩裂项理论上总是可行的,不过这样可能比较难看……
yayaweha 6# 2012-7-25 18:40
我也觉得不够清爽,所以来问
yes94 7# 2013-1-20 13:47
6# yayaweha 搞个链接噻,便于他人为你解析
yayaweha 8# 2013-5-10 22:37
本帖最后由 yayaweha 于 2013-5-10 22:39 编辑 $b[n]^2=4(n-1)^2/[n^2(n+1)^2]=4/n^2+16/(n+1)^2-16/n+16/(n+1) \sum[b[n]^2,{n,2,∞}]=\sum[4/n^2,{n,2,∞}]+\sum[16/(n+1)^2,{n,2,∞}]-16\sum[1/n-1/(n+1),{n,2,∞}] =4(π^2/6-1)+16(π^2/6-1-1/4)-16(1/2) =20π^2/6-20-4-8 =10π^2/3-32≈0.8987$
kuing 9# 2013-5-10 23:04
8# yayaweha 战巡写的不是 latex 代码 PS、你的短消息我不知怎么回。
yayaweha 10# 2013-5-11 19:04
9# kuing 随便回,只要能问到答案就行了
yayaweha 11# 2013-5-11 19:04
8# yayaweha 这样子做没有不等式的味道
thread-65-1-9.html: [不等式] Vasc again
pxchg1200 1# 2011-10-6 18:20
If $a,b,c$ are nonnegative real numbers such that $a^2+b^2+c^2=3$, then $(A)  \frac 1{3+\sqrt 2(1-a)}+\frac 1{3+\sqrt 2(1-b)}+\frac 1{3+\sqrt 2(1-c)}\le 1;$ $(B)  \frac 1{3+\sqrt 6(1-a)}+\frac 1{3+\sqrt 6(1-b)}+\frac 1{3+\sqrt 6(1-c)}\ge 1.$ (proposed by Vasc ) Have fun! PS:求犀利的CS证明。。
kuing 2# 2011-10-6 23:44
呃,第二个切线法还要讨论呐。。。 真不好搞。。。
thread-651-1-1.html: 人教升级前看到的一道极限证明题(题目有更新)
kuing 1# 2012-7-25 22:45
在人教论坛升级前我在兴趣小组看到一道极限证明题,如果没记错的话,题目是这样的: 已知实数列 $a_n$ 及非负数列 $p_n$ 分别满足 \begin{gather*} \lim_{n\to\infty}a_n = A, \\ \lim_{n\to\infty}\frac{p_n}{p_1+p_2+\cdots+p_n} = 0, \end{gather*} 求证 \[\lim_{n\to\infty}\frac{a_1p_n+a_2p_{n-1}+\cdots+a_np_1}{p_1+p_2+\cdots+p_n}=A.\] 表示没证出来感觉是挺有意思的一个结论,说不准是个著名定理?(因为各 $p$ 都为 1 时也是个经典结论,这个可以看成更一般的加权情形) _____kuing edit 1 time(见2#)_____
kuing 2# 2012-7-26 22:49
原来看来我记漏了一个条件,$p_n$ 要保号,否则有反例 $a_n=p_n=(-1)^{n+1}\frac1{\sqrt n}$(多谢海盗转发得到此反例),现已在一楼加上“非负”的条件。 大家继续。
海盗船长 3# 2012-7-27 00:07
貌似证明方法和这个类似的
都市侠影 4# 2012-7-27 13:26
本帖最后由 都市侠影 于 2012-7-27 17:13 编辑 我觉得应该这样推广吧: 如果数列 ${a_n}$ 有极限 $A$ (有限的或者无穷的),那么对于任何一个存在有限非零极限的序列 ${p_n}$,都成立 \[ \lim_{n\to\infty}\frac{p_1a_1+p_2a_2+\cdots+p_na_n}{p_1+p_2+\cdots+p_n}=A \]
kuing 5# 2012-7-27 14:48
我觉得应该这样推广吧: 如果数列 ${a_n}$ 有极限 $A$ (有限的或者无穷的),那么对于任何一个存在有限极限的序列 ${p_n}$,都成立 \[ \lim_{n\to\infty}\frac{p_1a_1+p_2a_2+\cdots+p_na_n}{p_1+p_2+\cdots+p_n}=A \] 都市侠影 发表于 2012-7-27 13:26 这个貌似不对吧,$a_n=p_n=\frac{(-1)^{n+1}}n$
①②③④⑤⑥⑦ 6# 2012-7-27 14:51
本帖最后由 ①②③④⑤⑥⑦ 于 2012-7-27 15:01 编辑 我觉得应该这样推广吧: 如果数列 ${a_n}$ 有极限 $A$ (有限的或者无穷的),那么对于任何一个存在有限极限的序列 ${p_n}$,都成立 \[ \lim_{n\to\infty}\frac{p_1a_1+p_2a_2+\cdots+p_na_n}{p_1+p_2+\cdots+p_n ... 都市侠影 发表于 2012-7-27 13:26 $p_n\to 0$ 要排除出去的吧? 原来是卷积形式的,你这个是内积形式的,不太一样哦,这个能直接Stolz了吧?
都市侠影 7# 2012-7-27 17:13
5# kuing 要加个条件:$p_n$ 的极限非零
海盗船长 8# 2012-7-27 22:34
http://ishare.iask.sina.com.cn/f/14112888.html?retcode=0 数学分析中的典型例题和解题方法-孙本旺 第二节,Toeplitz定理
kuing 9# 2012-7-27 22:41
8# 海盗船长 看到了,谢谢提供
海盗船长 10# 2012-7-27 22:46
9# kuing 路箩筐告诉我的
kuing 11# 2012-7-27 23:10
10# 海盗船长 原来还是他……
kuing 12# 2012-8-9 14:16
擦,原来我是没记错的,是那贴的楼主打少了条件 see also: http://bbs.pep.com.cn/thread-2578192-1-1.html
thread-652-1-1.html: 关于“搜索功能”和“相关贴子”
kuing 1# 2012-7-26 01:07
  首先说“搜索功能”,也就是页面左上方的“ ”这个东西。   这个东西相信大家都看到,显示在那里已经有一段时间的了,但是之前一直都搜不到任何东西(不知有没有人试过?)。   但由昨天开始,我发现这搜索功能终于能用了!而且的确能对本论坛作全文搜索,这就相当有用了,特别是对于本论坛来说更是好用!皆因这里的贴子我们基本上都用$\LaTeX$代码来打公式,这样我们完全可以通过搜索公式的关键部分代码来找到你想要的贴子,这是普通的论坛所做不到的事,因为一般的论坛在发题及解答时大都靠贴图,极难搜。嘿嘿,顺便又体现了论坛使用代码打公式的又一好处。   例:今晚我翻看一些旧文件时发现一道当年未解决的几何不等式,但又不记得有没有发过在这里,于是就决定试一下这个搜索功能。   这个不等式是 $\cos\frac A2+\cos\frac B2+\cos\frac C2\geqslant \frac{\sqrt3}2\left( \cos\frac{A-B}2+\cos\frac{B-C}2+\cos\frac{C-A}2 \right)$,可搜的部分很多,这里显然有一个比较特别的关键点就是 $\cos\frac{A-B}2$,于是自然想到搜索 \cos\frac{A-B} 这个关键词应该就能找到比较准确的东西(分母的2就不必搜了,前面应该足够,而且后面可以用2或{2},我也不确定有没有省略),一搜之下,第一个就是了!你试试看?   再说那个出现在贴子一楼的“相关贴子”。   大家应该也看到了,现在很多贴子的一楼都自动冒出了所谓的“相关贴子”这样的一栏东西,本意其实是好的,能将本论坛内一些关键词相同的标题或内容的贴子自动放到那栏上。 [新注] 后来发现这个搜索功能越来越不好用了,主要是因为后台那个同步索引老是停住了,原因不明,以至于搜不到最新的贴子。 现在建议大家采用google搜索,指定本论坛来搜,方法是: 要搜索的关键词 site:kkkkuingggg.5d6d.net
kuing 2# 2012-7-26 12:17
还可以在没输入任何搜索字符时直接点击搜索,打开的页面右边有一个“高级搜索”,里面可以指定版块,指定作者,等等…… 总之,搜索技巧,自己探索
都市侠影 3# 2012-7-26 12:26
网站不错,人气不高,如之奈何?唉……
kuing 4# 2012-7-26 21:50
3# 都市侠影 只能叹了,没办法,人气不是这么容易拉的
yayaweha 5# 2012-7-26 23:15
可能是宣传不够?
isea 6# 2012-7-29 23:35
这 查看新帖 排列也有点意思,有的回复是缩进的,有的却不是
kuing 7# 2012-7-30 01:45
6# isea 这我倒没研究过排列规律,反正一般都能搜出来吧。 不过据说新贴发出后要过一会儿才会被收录数据,所以最最新的贴未必能搜到,可能要待一阵子。
kuing 8# 2012-7-30 12:21
还多了一个“热搜”
kuing 9# 2012-8-24 01:29
移回站务
kuing 10# 2012-12-1 16:27
前段时间这个搜索数据一直停在9月份,今天进后台看发现已经恢复正常。
李斌斌755 11# 2013-4-26 13:53
好好用用。
kuing 12# 2013-4-26 13:55
11# 李斌斌755 最近不太好用,收录老是停滞,现在还停留在4月4日……也就是说搜不到最新贴子
kuing 13# 2013-5-8 22:07
算了,大家以后还是用 google 搜索吧,更新了一下1#
thread-653-1-7.html: test
zmath 1# 2012-7-26 17:25
frac(2,3) 复制代码
kuing 2# 2012-7-26 17:54
看置顶贴先
isea 3# 2012-7-29 23:29
2# kuing 数学与汉字混合输入好麻烦啊,要经常切换中英文输入法,latex里不会也是这样吧?
kuing 4# 2012-7-30 01:39
3# isea 是要切换的,我打文档的时候都要切(ctrl+空格),暂时我还没看到有哪个编辑器能自动实现这些。 tex及各种编辑器都是老外设计的,估计都不会怎么考虑这些。
thread-654-1-2.html: 等到花儿谢了!!!!!
yayaweha 1# 2012-7-26 23:22
数学空间我等你等到花儿谢了呀!!!!!!!
kuing 2# 2012-7-26 23:33
其实制作好的第九期的PDF我已经交给上头一个多月了,不过据闻上头everybody都没空,当然是忙着捞money了,网刊那啥的,能出就不错了……
thread-655-1-7.html: 问一个初中数学题,谢谢了先!
hongxian 1# 2012-7-27 16:45
已知有理数${{x}_{1}}$,${{x}_{2}}$,${{y}_{1}}$,${{y}_{2}}$满足:$x_{1}^{2}+5x_{2}^{2}=10$,${{x}_{2}}{{y}_{1}}-{{x}_{1}}{{y}_{2}}=5$,${{x}_{1}}{{y}_{1}}+5{{x}_{2}}{{y}_{2}}=15$, 求$y_{1}^{2}+5y_{2}^{2}$的值
kuing 2# 2012-7-27 16:51
看上去有点像是由圆锥曲线题整出来的方程组…… PS、那么多 { },其实很多可以省略。
yizhong 3# 2012-7-27 17:05
本帖最后由 yizhong 于 2012-7-27 17:37 编辑 答案35,其实就是构造出一个恒等式。$(x_1^2+5x_2^2)(y_1^2+5y_2^2)=(x_1y_1+5x_2y_2)^2+(\sqrt{5}x_2y_1-\sqrt{5}x_1y_2)^2$,另外PS一下:估计这个题目是给初一党做的,所以加了有理数这个条件。
isea 4# 2012-7-28 14:42
本帖最后由 isea 于 2012-7-28 14:45 编辑 3# yizhong 不算简单啊,变形能力要求很强 哦,我想起来了,这算是竞赛范围了,在竞赛范围里好算有这样的结论
kuing 5# 2012-7-28 14:53
4# isea 熟悉柯西不等式的各种证法可能会想起来,我一开始也想不起,因为我第一感觉如2楼所示,联想到解几去了
yizhong 6# 2012-8-5 12:09
4# isea 据说这个是武汉某中学的分班试题,对于初一党确实是。。。。。。。。套用小K的一句话就是珍惜生命,远离考试。。。。。。。
thread-656-1-1.html: 一道微元法
★_/ka_☆ 1# 2012-7-28 14:15
设在坐标原点有一质量为m的质点,在区间$\[a,a+l],a>0$有一质量为M的均匀细杆,求它们的万有引力.
★_/ka_☆ 2# 2012-7-28 14:27
本帖最后由 ★_/ka_☆ 于 2012-7-28 14:31 编辑 取一段质量微元dM,     $dM=\frac{l}{M}dx$ $\frac{Gm\mathrm{d}m}{(x+a)^2}$ 后面积分区间积出来的答案和标答对不上
kuing 3# 2012-7-28 14:38
以前做过,好像等效于几何平均数那里
kuing 4# 2012-7-28 14:48
在杆上取与质点距离为 $R$ 的一小段,则 \[dF=\frac{G(dM)m}{R^2}=\frac{GMm}{R^2l}dR\] 故 \[F=\int_a^{a+l}\frac{GMm}{R^2l}dR=\frac{GMm}l\int_a^{a+l}{R^{-2}dR}=\frac{GMm}l\left(\frac1a-\frac1{a+l}\right)=\frac{GMm}{a(a+l)}\] 可见此力大小等于两质点质量为 $M$ 和 $m$ 且距离为 $\sqrt{a(a+l)}$ 时的引力,也就是上楼所说的几何平均数那里。
kuing 5# 2012-7-28 20:06
取一段质量微元dM,     $dM=\frac{l}{M}dx$ $\frac{Gm\mathrm{d}m}{(x+a)^2}$ 后面积分区间积出来的答案和标答对不上 ★_/ka_☆ 发表于 2012-7-28 14:27 应该是 $dM=\frac{M}{l}dx$
thread-657-1-7.html: [几何] 请教一个平面几何题,先谢谢了!
hongxian 1# 2012-7-28 19:07

kuing 2# 2012-7-28 20:01
易证 $BF=OE$、$FO=EA$,于是有下图的等角关系,从而 $4(x+y)=180^\circ$,即第二问是 $\angle ACB = 45^\circ$,从而 $OA\perp OB$,即 $AB$ 是直径,第一问就显然了。
都市侠影 3# 2012-7-28 20:20
本帖最后由 都市侠影 于 2012-7-28 20:42 编辑 第一问用外心的性质就可以了: 先证明 $OC\perp EF$,根据外心性质有 $\angle{OCA}=\frac{\pi}{2}-\angle{B}$,而且 $\angle{CEF}=\angle{B}$,所以 $OC \perp EF$ 再证明 $OE \perp CF$,因为 $\angle{OEC}=\angle{ABO}=\frac{\pi}{2}-\angle{ACB}$,所以 $OE \perp CF$ 所以点 $O$ 是三角形 $CEF$ 的垂心。 第二问就是Kuing的那个方法最简单了。
hongxian 4# 2012-7-28 22:43
2# kuing 一个易证,我都看了半天,然来是三角形全等,谢谢了!
kuing 5# 2012-7-28 22:53
一个易证,我都看了半天,然来是三角形全等,谢谢了! hongxian 发表于 2012-7-28 22:43 不用全等啊,由 OB=EF 得 弧OFB=弧EOF,所以 弧BF=弧OE,即 BF=OE,同理 FO=EA
hongxian 6# 2012-7-29 17:47
5# kuing 不好意思,还是我把问题搞复杂了.
thread-658-1-7.html: [几何] [转]看不懂的多面体
kuing 1# 2012-7-28 21:46
 
都市侠影 2# 2012-7-28 22:00
原来Kuing一直在看群里聊天啊,哈哈
kuing 3# 2012-7-28 22:08
2# 都市侠影 其实是先在我的群里发然后有人转到人教那边的
kuing 4# 2012-7-28 22:08
话说在网页上看好像转得比群里快一些。
①②③④⑤⑥⑦ 5# 2012-7-30 09:18
大十二面体? 四种凹正多面体之一吧,刚才误以为是小星形十二面体,不过那个应该是以五角星为面的,脑抽了
shidilin 6# 2012-7-30 12:31
魔方?!
kuing 7# 2012-7-30 12:34
6# shidilin 说不定是,以前见过一些奇特的魔方,好像有类似的。
①②③④⑤⑥⑦ 8# 2012-7-30 13:53
7# kuing Alexander's Star
kuing 9# 2012-7-30 13:59
8# ①②③④⑤⑥⑦ 果然是 http://v.youku.com/v_show/id_XMjg4ODc5OTcy.html
thread-66-1-4.html: 双勾函数乃双曲线是也
kuing 1# 2011-10-6 21:26
\begin{align*} &y = x + \frac1x\\ \iff& xy = x^2 + 1\\ \xrightarrow{\measuredangle = \frac\pi8}&\left(x\cos\frac\pi8 + y\sin\frac\pi8\right)\left(y\cos\frac\pi8 - x\sin\frac\pi8\right) = \left(x\cos\frac\pi8 + y\sin\frac\pi8\right)^2 + 1\\ \iff& \cos \frac{\pi }{8}\sin \frac{\pi }{8}(y^2 - x^2 ) + \left( {\cos ^2 \frac{\pi }{8} - \sin ^2 \frac{\pi }{8}} \right)xy = x^2 \cos ^2 \frac{\pi }{8} + y^2 \sin ^2 \frac{\pi }{8} + 2xy\cos \frac{\pi }{8}\sin \frac{\pi }{8} + 1\\ \iff& \cos \frac{\pi }{8}\sin \frac{\pi }{8}(y^2 - x^2 ) + xy\cos \frac{\pi }{4} = x^2 \cos ^2 \frac{\pi }{8} + y^2 \sin ^2\frac{\pi }{8} + xy\sin \frac{\pi }{4} + 1 \\ \iff& \left( {\cos ^2 \frac{\pi }{8} + \cos \frac{\pi }{8}\sin \frac{\pi }{8}} \right)x^2 + \left( {\sin ^2 \frac{\pi }{8} -\cos \frac{\pi }{8}\sin \frac{\pi }{8}} \right)y^2 + 1 = 0\\ \iff& \left( {\frac{{\cos \frac{\pi }{4} + 1}}{2} + \frac{{\sin \frac{\pi }{4}}}{2}} \right)x^2 + \left( {\frac{{1 - \cos\frac{\pi }{4}}}{2} - \frac{{\sin \frac{\pi }{4}}}{2}} \right)y^2 + 1 = 0\\ \iff& \frac{{1 + \sqrt 2 }}{2}x^2 + \frac{{1 - \sqrt 2 }}{2}y^2 + 1 = 0 \end{align*}
①②③④⑤⑥⑦ 2# 2011-10-8 09:27
其实吧,有那个等价的式子,显然两次曲线,非退化,有两条渐近线,不是双曲线还能是什么(或者用系数判别式),后面的无非是标准化的过程,例行公事 允许仿射变换的话,直接有 $x(y-x)=1$ $x' y' =1$ 后者是双曲线前者也必然是双曲线,不过这不是初等几何了
kuing 3# 2011-10-8 09:31
呃,其实这是回复群里的,他们还不知道这些,倒是知道旋转公式,刚开始还代错了,后来我代正确的就写了下在这。
yes94 4# 2013-2-3 22:13
$y=x+\dfrac1x$是双曲线,离心率$e_1=$? $y=x-\dfrac1x$也是双曲线,离心率$e_2=$?
thread-660-1-1.html: 今年IMC第4题
海盗船长 1# 2012-7-29 13:08
http://tieba.baidu.com/p/1756799529 设 $f : \mathbb R\to\mathbb R$ 为连续可导函数,对任意的实数 $t$,有 $f'(t)>f(f(t))$。证明当 $t\geqslant0$ 时,$f(f(f(t)))\leqslant0$。
thread-661-1-6.html: [几何] 一道解析几何题
hflz01 1# 2012-7-29 20:55
一道解析几何题
isea 2# 2012-7-29 23:32
1# hflz01 硬算不可以吗?想了想,计算量似乎不是特别大。
hflz01 3# 2012-7-29 23:46
也许可以吧,试过,但能力有限,似觉繁琐。期待看到具体过程。
kuing 4# 2012-7-30 03:22
还是极点极线背景
kuing 5# 2012-7-30 03:23
化成圆或许也可以试试,因为这个是椭圆,如果是双曲线什么的就使不得了,时间关系先睡觉觉了,明天再玩……
kuing 6# 2012-8-3 22:24
从极点极线的角度出发,显然有更一般的情况了: 如图,任意一条二次曲线,取其上四点,整成那个完全四边形,然后那两对蓝色切线的交点会在那条粗黑色的线上(即里面那个点对应的极线),楼主的题是这个的特殊情形。
kuing 7# 2012-8-3 23:00
另外,貌似那两对切线的交点还调和分离那两个黑点,如果这是正确的话,那么由于楼主的题中BC过椭圆中心使有一对切线的交点为无穷远点,这将得到G是AD的中点。 看看能不能证证先,如果OK那就是有意外收获了
都市侠影 8# 2012-8-3 23:09
6# kuing 这线多的,像胡子一样
hflz01 9# 2012-8-26 15:22
借助行列式算“三角形”面积为零,搞定。
hflz01 10# 2012-9-3 21:11
证明见附件。
thread-662-1-6.html: 一道数学培训题
转化与化归 1# 2012-7-29 22:14
一道数学培训题 ____kuing edit in $\LaTeX$____ 给定了 $n>1$ 个二次三项式 $x^2-a_1x+b_1$, $\ldots$, $x^2-a_nx+b_n$,其中 $2n$ 个实数 $a_1$, $\ldots$, $a_n$, $b_1$, $\ldots$, $b_n$ 互不相同。试问,是否可能 $a_1$, $\ldots$, $a_n$, $b_1$, $\ldots$, $b_n$ 中的每个数都是其中某个多项式的根?
tan9p 2# 2012-7-31 19:49
似乎可以有 $\sum a_i+b_i = \sum a_i$,$\prod a_ib_i = \prod b_i$得到$\sum b_i=0,\prod a_i=1$ 然后?
kuing 3# 2012-7-31 19:53
2# tan9p 若存在则等价于 \begin{align*} &(x^2-a_1x+b_1)(x^2-a_2x+b_2)\cdots(x^2-a_nx+b_n)\\ ={}&(x-a_1)(x-a_2)\cdots(x-a_n)(x-b_1)(x-b_2)\cdots(x-b_n), \end{align*} 展开,对比常数项得 $\prod b_i=\prod a_ib_i$,对比 $x^{2n-1}$ 的系数得 $\sum a_i=\sum (a_i+b_i)$(即楼上的两式),但仅凭这两式还不够用,估计得把其他的项的系数都也对比上才行(大概就是去证明那堆方程组的解与题设矛盾),但那些项的有点复杂,暂时也没想到什么好的办法。
海盗船长 4# 2012-8-6 14:32
http://www.artofproblemsolving.c ... p?f=36&t=491446
转化与化归 5# 2012-8-7 21:43
牛!
yizhong 6# 2012-9-13 13:57
本帖最后由 yizhong 于 2012-9-13 14:16 编辑 来个比较简洁的证明吧 我们可以用反证法来证明 假设可以做到题目中的要求,则我们就有:由于$a_1,a_2.....a_n,  b_1,b_2....b_n这2n个实数互不相同 所以n个一元二次方程的根的集合刚好是这2n个实数$  我们设每个方程的根为:$c_i  d_i(i=1,2...n) 则我们由韦达定理得到:a_i=c_i +d_i(1)  b_i=c_i.d_i(2)  (i=1,2...n)$ 所以我们由(1)就有:$\sum_ {i=1}^{n}b_i=0(3)$,利用:$c_i ^2+d_i^2=(c_i +d_i)^2-2c_i.d_i=a_i^2-2b_i$ 再结合(3)我们可以得到:$\sum_ {i=1}^{n}b_i^2=0$  所以我们得到所有的$b_i$都为0,矛盾。所以题目的答案是不可能的。
nash 7# 2012-9-13 21:08
6# yizhong
转化与化归 8# 2012-9-13 21:21
6# yizhong 太棒了!
thread-663-1-6.html: [函数] 这道求“复合函数”定义域问题是否无解?
isea 1# 2012-7-29 23:26
若函数$f(|2x-1|)$的定义域$[0,2]$,则函数$f(x)$的定义域是____? 如果将原是中的绝对值去掉,应该大家观点一致,结果是:$[-1,3]$。 但加绝对值后,是否无解?或者说定义域不确定?
kuing 2# 2012-7-30 01:41
这种题通常都有问题。 首先如果这里说的“定义域”是指所谓的“自然定义域”的话,那么条件就已经有问题了,$f(|2x-1|)$ 的定义域不会是 $[0,2]$,因为 $x$ 既然能取 $2$,那么必然也能取 $-1$。而如果不是指“自然定义域”的话,那就是可以自己随意限制,只要使函数有意义就行,那么后面问的定义域我也可以随便说了。 那么如果要使“自然定义域”适用的话,就得改条件,比如我就改成 $f(|2x-1|)$ 的定义域是 $[-1,2]$,就不会出现前面的问题,自然定义域OK了,但后面仍有问题(这才是重点),比如: 设常数 $a<0$,令 \[f(x)=\begin{cases} 0, & x=0~\text{或}~x=3,\\ \ln(x(x-3)(a-x)),& x<a~\text{或}~0<x<3, \end{cases}\] 那么 $f(|2x-1|)$ 的定义域为 $[-1,2]$,而 $f(x)$ 的定义域为 $(-\infty,a)\cup[0,3]$,因 $a$ 而异。 故此……
kuing 3# 2012-7-31 17:52
顺便想起,《数学空间》2011年第2期的第一篇文章也有类似问题,呵呵,惭愧,当时没怎么审稿,随便就上了,可谓问题多多。
第一章 4# 2012-8-29 07:59
看了KK的分析,汗流浃背啊……一直都没有注意到这个问题…… 《数学空间》?就是那个已知$x^2$? 不过KK为什么不构造一个根号的函数,好像更贴合闭区间……
都市侠影 5# 2012-8-29 08:53
弄懂复合函数的内外层函数关系就可以了,至于这道题,完全脑残的人出的题,纠缠它作甚?
kuing 6# 2012-8-29 10:30
4# 第一章 嗯就是那个。 嗯,构造根号应该更简单些,但是我现在另外定义端点也没问题。
第一章 7# 2012-8-30 08:54
5# 都市侠影 这个我懂,但外面很多资料、很多老师还是大搞这类题啊。作为中学教师,提高一些是有必要的。
thread-664-1-1.html: 谈谈导函数的介值性定理
都市侠影 1# 2012-7-30 13:26
本帖最后由 都市侠影 于 2012-9-13 12:39 编辑 在考研论坛看到很多人对导函数的介值性定理很困惑,对于连续函数的介值性倒是比较理解。在我看来,可能是导函数的介值性定理没有连续函数的介值性那样的如雷贯耳,使得很多人一提到介值性就想到了连续函数,事实上,介值性不是连续函数所特有的特性。在本帖子里,详细说一下这个定理和它的证明,看完了应该就没问题了。 导函数介值性定理:对一个在某区间上连续并且可导的函数,它的导函数满足介值性。具体地说,假如 $M$ 与 $N$ 是两个点处的导函数值,那么位于 $M$ 与 $N$ 之间的任何数都能成为某点处的导函数值。 要注意的是这里说的是导函数的介值性,并没有提到导函数是否连续,不要认为导函数是因为连续而具有介值性,事实上,不连续的导函数是存在的,想必例子也是容易找的,这里就不列举了,只是提醒注意这个介值性并不是连续所导致的。 为什么会具有这样的介值性呢,它的本质在于连续函数在闭区间上必有最大值,如果这个最大值不在区间的端点上,那么在函数可导的前提下,这个最大值就成为一个极值,从而在这里的导函数值为零。 证明:为了简化证明,假定函数 $f(x)$ 在区间 $[a,b]$ 上连续,在开区间 $(a,b)$ 上可导(当然在端点处就存在单侧的导数了),任取 $f'(a)<C<f'(b)$,都存在一点 $\xi \in (a,b)$,使得 $f'(\xi)=C$. 为了证明这一点,构造一个函数 $h(x)=f(x)-Cx$,于是 $h(x)$ 也在闭区间上连续,开区间上可导,端点处存在单侧导数,而且 $h'(a)<0<h'(b)$,这说明:两个端点都存在他们各自的一个邻域,使得在这个邻域上,左端点附近有小于左端点函值点的点,右端点附近也有比右端点函数值更小的点,从而左右端点都不可能是函数 $h(x)$ 在闭区间 $[a,b]$ 上的最小值,但是根据 $h(x)$ 的连续性,这个最小值是存在的,所以只能是在开区间内某一点 $\xi$ 处取得,所以 $h'(\xi)=0$,也就是 $f'(\xi)=C$,证毕。
kuing 2# 2012-8-1 23:03
这个定理好像有一个名字叫啥我忘记了,好像是 D 开头的
都市侠影 3# 2012-8-2 08:41
哈哈,达布定理,
秋风树林 4# 2012-8-2 15:59
英文是Darboux
kuing 5# 2012-8-2 16:01
4# 秋风树林 我总是不记得英文名,完全没feel
秋风树林 6# 2012-8-2 16:02
5# kuing 学Riemann积分那里各种Darboux乱跳。。。。- -
kuing 7# 2012-8-2 16:04
6# 秋风树林 神马东东?
秋风树林 8# 2012-8-2 16:07
7# kuing 就是现在一般接触的那个定积分定义要精确化描述,比如那种和式极限定义出的定积分有什么性质等等 其中的和有个上确界和下确界就是所谓的Darboux大和Darboux小和。。。 证明很麻烦。。。用了几页去说明Riemann可积的充要条件。。。- -
kuing 9# 2012-8-2 16:12
飘走……
都市侠影 10# 2012-8-3 13:45
8# 秋风树林 那个的确是有些麻烦,关于定积分的存在条件,有几种不同的描述方法,通过上积分与下积分来讨论是我比较喜欢的一种方法。
地狱的死灵 11# 2012-8-23 22:50
8# 秋风树林 最简单的黎曼可积的充要条件是《实变函数》里提到的“几乎处处连续”
icesheep 12# 2012-9-5 22:01
楼主的证明是错的,导数大于零不能得到该点的某领域内递增。
都市侠影 13# 2012-9-13 12:40
12# icesheep 多谢提醒,已经修正,不需要单调,只要有比端点函数值更小的点就可以了。
thread-665-1-4.html: [函数] 求函数的解析式,有没有高手给一个答案,谢谢了!
hongxian 1# 2012-7-31 19:21
本帖最后由 hongxian 于 2012-7-31 19:31 编辑 已知:$x\in R$,$x\ne 0$,$x\ne 1$时,恒有$f(x)+f\left( \frac{x-1}{x} \right)=1+x$成立,求$f(x)$的解析式 不好意思,把题目$x-1$,写成了$1-x$,麻烦大家了。
kuing 2# 2012-7-31 19:25
\begin{align*} g(x)&=\frac{x-1}x,\\ g(g(x))&=\frac1{1-x},\\ g(g(g(x)))&=x. \end{align*}
kuing 3# 2012-8-1 23:02
不好意思,把题目$x-1$,写成了$1-x$,麻烦大家了。 hongxian 发表于 2012-7-31 19:21 写成 1-x 好像永远迭代不回去,印象中以前还碰到过类似的,迭代的时候还会出现飞波拉鸡数列的东西出来。
yes94 4# 2013-1-29 23:35
2# kuing 迭代周期为3,所以构造三个方程,可解
thread-666-1-1.html: 锐角、钝角三角形集合的势
海盗船长 1# 2012-7-31 21:53
证明或否定$\{锐角三角形\}$和$\{钝角三角形\}$等势。
kuing 2# 2012-7-31 23:29
思路还是沿着我在群里说的“将同周长的拿到一起,再用两个角度对应一下”,只不过后面对应的时候有点麻烦,得注意边界的细节。 首先将 $\{\text{锐角三角形}\}$ 和 $\{\text{钝角三角形}\}$ 分别用别的等势集合写出来。 设某锐角三角形的周长为 $C$,三个内角为 $x$, $y$, $\pi-x-y$,其中 $0<x\leqslant y\leqslant \pi-x-y$ 且 $x + y > \pi/2$,显然 $C$, $x$, $y$ 一一对应着任意锐角三角形,所以 $\{\text{锐角三角形}\}$ 就与如下有序三元数组集合等势: \[\left\{ (C,x,y) \middle| C>0,0<x\leqslant y\leqslant \pi-x-y, x + y > \frac\pi2 \right\}.\] 类似地,与 $\{\text{钝角三角形}\}$ 等势的有序三元数组集合可以写成 \[\left\{ (C,x,y) \middle| C>0,0<x\leqslant y, x + y < \frac\pi2 \right\}.\] 于是我们只要构造这两个集合的一一对应。第一个 $C$ 直接对应就行(也就是最前面说的“将同周长的拿到一起”了),剩下只要将后面的对应好,也就变成平面区域的对应了。 两平面区域 $M=\{(x,y)|0<x\leqslant y\leqslant \pi-x-y, x + y > \pi/2\}$, $N=\{(x,y)|0<x\leqslant y, x + y < \pi/2\}$ 的图形如下图。 本来以为很好办,只要直接反射过去就行,但细想发现麻烦的地方在于边界,直接翻过去边界就对应不上了。 后来想到分割再对应,为方便起见,就以第一个图的 $y=\pi/3$ 切开再分别对应到第二个图上(如图所示),这样就OK了。 具体地,可以将对应关系用如下表达式写出来: 若 $y\geqslant\pi/3$,则 \[M \to N : (x,y) \to \left(-\frac{7\pi}8+\frac{3x}4+\frac{9y}4,\frac{17\pi}8-\frac{9x}4-\frac{15y}4\right);\] 若 $y<\pi/3$,则 \[M \to N : (x,y) \to \left(-\frac{3\pi}8+\frac{3x}4+\frac{3y}4,-\frac{3\pi}8-\frac{9x}4+\frac{15y}4\right).\]
kuing 3# 2012-8-1 01:25
话说能不能证明任意一个平面区域(无论边界情况如何)都和 $[0,1]^2$ 等势?如果成立的话,那么就能直接得到上题的等势了。 又因为 $[0,1]$ 和 $[0,1]^2$ 等势是有现成的构造了,也就是任意线段都和 $[0,1]^2$ 等势(线段无论端点取不取都等势,见),这样还能得到与 $\{\text{直角三角形}\}$ 也等势了。
①②③④⑤⑥⑦ 4# 2012-8-1 09:14
3# kuing (0,1)和整条直线等势很轻松吧,(0,1)×(0,1)和整个平面等势也一样啊 只要包含一条线段,那至少是连续统的势了,而整个平面也不过是连续统的势,你想要的“平面区域”当然没问题了。(这样直接用简单比较而不是构造,需要势的三歧性定理/选择公理,不过一般不特别说明应该是认可选择公理的,这里应该不会去研究换一种选择公理会怎么样吧)
kuing 5# 2012-8-1 10:34
4# ①②③④⑤⑥⑦ 连续统神马的不太了解……
都市侠影 6# 2012-8-1 13:05
本帖最后由 都市侠影 于 2012-8-1 13:07 编辑 aaa
thread-667-1-4.html: [不等式] 函数值域
v6mm131 1# 2012-8-2 11:00
本帖最后由 v6mm131 于 2012-8-2 13:09 编辑 这道题是去年给高一的期中考试试题,是不是难了,征集大家的解法 sqrt(x^2-x^4)+sqrt(2x^2-x^4) 复制代码 的最大值是多少?
kuing 2# 2012-8-2 11:12
看置顶贴学写公式代码先 v6
海盗船长 3# 2012-8-2 11:59
哪有绝对值啊?
kuing 4# 2012-8-2 14:46
高一怎么做啊?除了求导或数形结合(都先换元再说)之外没什么想法
kuing 5# 2012-8-2 15:27
待定系数法也行,先换元 $x^2=t\in[0,1]$,然后待定两个正数 $a$, $b$,有 \begin{align*} \sqrt{x^2-x^4}+\sqrt{2x^2-x^4}&=\sqrt{t(1-t)}+\sqrt{t(2-t)}\\ &=\sqrt{at\cdot \frac1a(1-t)}+\sqrt{bt\cdot \frac1b(2-t)} \\ & \leqslant \frac12\left( at+\frac1a(1-t)+bt+\frac1b(2-t) \right) \\ & =\frac1{2a}+\frac1b+\left( a-\frac1a+b-\frac1b \right)\frac t2, \end{align*} 取等条件为 \[\left\{\begin{aligned} a-\frac1a+b-\frac1b&=0, \\ \frac1a(1-t)&=at, \\ \frac1b(2-t)&=bt, \end{aligned}\right.\] 解得 \[\left\{\begin{aligned} a&=\frac{\sqrt2}2, \\ b&=\sqrt2, \\ t&=\frac23, \end{aligned}\right.\] ……
kuing 6# 2012-8-2 15:38
擦,还是CS好使 \begin{align*} \sqrt{x^2-x^4}+\sqrt{2x^2-x^4}&=\sqrt{x^2}\sqrt{1-x^2}+\sqrt{2-x^2}\sqrt{x^2} \\ & \leqslant \sqrt{(x^2+2-x^2)(1-x^2+x^2)} \\ & =\sqrt2, \end{align*} 取等略……
kuing 7# 2012-8-2 15:41
话说,高一学均值了么?
★_/ka_☆ 8# 2012-8-2 15:54
高一下期才学吧
kuing 9# 2012-8-2 21:26
几何意义(by v6)
yes94 10# 2013-1-29 23:12
这道题当时我也做了的, 幸亏草本(word文档)还在,又找到了…… 只是做的稍微复杂些,管它了,贴出来
thread-669-1-2.html: 吐槽
froglove 1# 2012-8-2 16:37
一天两次重启,先让我吐槽完,再看贴
kuing 2# 2012-8-2 16:41
1# froglove 话说你这种情况我也是第一次听…… 现在是什么情况
froglove 3# 2012-8-2 16:50
现在情况就是战战兢兢看帖,随时准备重启,随时保存网页
kuing 4# 2012-8-2 16:52
擦…… 有没有换别的浏览器试试
froglove 5# 2012-8-2 17:02
不高兴换,还没重启到我无法忍受的地步。。 ps水贴换经验可耻啊。。。
kuing 6# 2012-8-2 17:04
嘿嘿,我电脑上三个浏览器随不同情况看着用哪个 PS、这里积分没什么用,普通会员的权限都差不多。
froglove 7# 2012-8-2 17:07
3个.....还不同情况....干脆弄7个,一个礼拜不重复
海盗船长 8# 2012-8-2 19:16
哪三个啊?
kuing 9# 2012-8-2 19:18
IE、chrome、firefox
thread-67-1-2.html: 今晚群里说的温度曲线与平均温度曲线
kuing 1# 2011-10-7 01:12
当然,原题只是看图说话,这里随便扯扯 设温度曲线 $Q(t)$ 在 $t\in[0,12]$ 内连续,$[0,t]$ 内的平均温度 $G(t)$。 由所设知 $Q(t)$ 可积,故\[G(t)=\frac{\int_0^t Q(x)\mathrm{d}x}t\]从而 $G(t)$ 可导,所以即使温度曲线不光滑,有尖,但只要是连续的,那么平均温度曲线也不会有尖。 再利用求导公式,有\[ G'(t) = \frac{Q(t) \cdot t - \int_0^t Q(x)\mathrm{d}x}{t^2} \]故\[ G'(t) > ( < , = ) 0 \iff Q(t) > ( < , = ) \frac{\int_0^t Q(x)\mathrm{d}x}t=G(t) \]这结果符合一个显然的事实:“瞬时温度高于前一时刻的均温,则均温就该增”(这句话从群里直接复制上来的)
kuing 2# 2011-10-7 01:28
截图里的图画得太烂了,图1纵坐标还标错了,排版又差,连℃都不会输入,烂资料一份啊
nash 3# 2011-10-7 01:34
评论犀利…
isea 4# 2011-10-8 12:53
D?
kuing 5# 2011-10-8 15:35
已知年平均温度为10所以曲线最后不会是0,所以不会是D
海盗船长 6# 2011-10-14 20:18
记得有一个关于股票的类似题
海盗船长 7# 2011-10-14 20:22
http://bbs.pep.com.cn/viewthread ... hlight=%B9%C9%C6%B1
kuing 8# 2011-10-14 21:26
7# 海盗船长 oh,原来我还回复过……
thread-670-1-1.html: [Physics]关于万有引力的一些推论
Chetion 1# 2012-8-2 18:57
求证:质量分布呈球对称的球体产生的万有引力,等效于把球体质量集中于球心处的质点所产生的万有引力。
kuing 2# 2012-8-2 19:07
这好像要另一个质点在球外才适用
海盗船长 3# 2012-8-2 19:13
积个分就行了。或者类比静电场用高斯定理直接此结论
海盗船长 4# 2012-8-2 19:14
嗯,场点还要在球外才行
kuing 5# 2012-8-2 19:25
之前好像在哪看到有个不用积分的方法……
kuing 6# 2012-8-2 19:27
想起来了,http://www.docin.com/p-34619581.html,P55~57(正文页码)
thread-671-1-7.html: [不等式] 数列不等式
随便112 1# 2012-8-2 19:17
设正数序列 $ a_{1},a_{2},\cdots,a_{9}\ge a_{10} $, 且$ a_{1}+a_{2}+a_{3}+\cdots+a_{10}=30$ 和 $ a_{1}a_{2}\cdots a_{10}<21  $. 证明:  $ \frac{1}{a_{1}}+\frac{1}{a_{2}}+\cdots+\frac{1}{a_{9}}>9  $. ______kuing edit in 清楚些______ \[ \frac1{a_1}+\frac1{a_2}+\cdots+\frac1{a_9}>9. \]
kuing 2# 2012-8-2 19:23
貌似跟数列没什么关系的说……${}\geqslant a_{10}$ 的条件可能也是多余的……先吃饭
海盗船长 3# 2012-8-2 20:24
2# kuing 注意下面只加到了$\frac{1}{a_9}$.......
海盗船长 4# 2012-8-2 20:26
http://tieba.baidu.com/p/1405356519#
kuing 5# 2012-8-2 21:27
3# 海盗船长 我是知道只加到 1/a9 的
kuing 6# 2012-8-3 01:41
4# 海盗船长 刚才以为发现反例,才发现原来这个链接里跟楼主的题不一样,那里的是 ${}\leqslant a_{10}$,这就不多余了,也正确了。
thread-672-1-1.html: 试着开了Tag功能,不知会怎么样
kuing 1# 2012-8-2 22:10
RT
kuing 2# 2012-8-23 15:18
有时会影响公式的显示,还是关掉了
thread-673-1-2.html:
froglove 1# 2012-8-3 20:52
除了水不知道该干嘛...
kuing 2# 2012-8-3 20:59
宣传
froglove 3# 2012-8-3 21:22
太有损形象了....让它沉....
kuing 4# 2012-8-3 21:26
求人气^
froglove 5# 2012-8-3 22:00
求沉贴.....kuing你也闲?......
kuing 6# 2012-8-3 22:04
5# froglove 呃,我啥时候不是悠闲的哩?……
froglove 7# 2012-8-3 22:57
这回帖神速啊......我肿么都是过了半小时才回的= =\ kuing不应该做题么....好像还有帖无解中......
kuing 8# 2012-8-3 23:26
7# froglove 最近低潮,没什么做题欲,所以很闲,看心情或看灵感吧,刚才突然发现那道解几的东西想了想,所以这次到我半个小时才回了。
海盗船长 9# 2012-8-4 13:44

froglove 10# 2012-8-7 00:22
半夜三更..木帖.悄悄水个~
kuing 11# 2012-8-7 00:23
10# froglove
froglove 12# 2012-8-7 00:27
大神你多久刷次屏啊==这速度....幸好我没关....我是想悄悄地...
kuing 13# 2012-8-7 00:32
12# froglove 不确定啊,没事干的时候就刷刷……如果你再早一点发我就没那么快了,因为先前在看东西。只是刚才正好刷新就看到你刚发完贴
froglove 14# 2012-8-7 00:35
这次快了 你让我悄悄的水完 ....然后依然沉...等下次水
froglove 15# 2012-8-7 01:11
水贴可耻....不来了
海盗船长 16# 2012-8-7 13:42

kuing 17# 2012-8-7 13:54
16# 海盗船长 第二个图以前也见过,最后一个不知啥意思……
froglove 18# 2012-10-4 22:34
过一个月再来水的心情...很奇妙 还以为帖子没了....找半天...明明就3页嘛....
kuing 19# 2012-10-4 22:36
18# froglove 何以奇妙?
froglove 20# 2012-10-4 22:47
刚刚点开..人教论坛?!怎么变得....好讨厌的感觉.....坚定地不让我发附件嗯哼...
thread-673-2-2.html:
kuing 21# 2012-10-4 22:50
20# froglove 人教我也没怎么去了,你在那边多少分了?据说现在升级比以前还难,呵呵。
froglove 22# 2012-10-4 22:55
多少分?.....六千多吧....还早呢,我死心了.一个月看见一次kk哈,好有喜感
kuing 23# 2012-10-4 23:07
一个月看见一次? 咋回事这是……
froglove 24# 2012-10-5 10:39
诶上大学没带电脑,手机又懒得上qq,所以.....十一回家首次使用电脑..好有喜感
kuing 25# 2012-10-5 12:46
大学怎可没电脑?
thread-674-1-1.html: XeLaTeX运行错误
叶剑飞Victor 1# 2012-8-4 00:17
本帖最后由 叶剑飞Victor 于 2012-8-4 00:20 编辑 文件h.tex,内容 编译命令 xelatex h.tex 结果
kuing 2# 2012-8-4 01:13
那个逗号是半角的还是全角的? 字体大小好像只有 10pt, 11pt, 12pt 怎么正文不写点内容?
叶剑飞Victor 3# 2012-8-4 01:25
本帖最后由 叶剑飞Victor 于 2012-8-4 01:27 编辑 2# kuing 就是英文模式下的逗号,都好后面加了一个空格。 原来是有内容的,但是编译错误后,我就把内容一点一点地删了。没想到都删的不能再删了,还是错误。 另外pdflatex h.tex可以编译通过,xelatex h.tex编译不通过。
kuing 4# 2012-8-4 01:43
可能需要宏包或者设置字体? 其实我也不怎么懂 xelatex,我用 xelatex 的时候一般都用上 xeCJK 宏包(打中文需要),还要设置字体。
kuing 5# 2012-8-4 01:55
我一般就是这样 \documentclass{article} \usepackage{xeCJK} \setCJKmainfont[BoldFont=SimHei,ItalicFont=KaiTi_GB2312]{SimSun} %普通中文用宋体、粗体用黑体、斜体用楷书 然后就begin document开始写东东咯…… 纯英文的没试过。
叶剑飞Victor 6# 2012-8-8 17:44
本帖最后由 叶剑飞Victor 于 2012-8-8 17:46 编辑 问题已解决,版本冲突问题。 运行 xetex -ini -jobname=xelatex -progname=xelatex -etex xelatex.ini 复制代码 然后就可以了
thread-675-1-6.html: [数列] 数列个位数字
海盗船长 1# 2012-8-4 13:33
本帖最后由 海盗船长 于 2012-8-4 13:34 编辑 $\{x_n\}$满足$x_1=1$,$x_{n+1}=4x_n+\big[x_n\sqrt{11}\big]$,求$x_{2013}$的个位数字. 其中$\big[x\big]$,表示对$x$取整。
yizhong 2# 2012-8-17 23:28
oh,第三届陈省身杯数学竞赛第二天的第七题
kuing 3# 2012-8-17 23:30
出处dang又来了
yizhong 4# 2012-9-16 11:12
此题也想到了一个比原解答好的路子,改天一起把那个数列题一起码下
kuing 5# 2012-9-16 12:57
一中niubility,这种题我可一点思路都木有了。
yizhong 6# 2012-9-17 20:23
本帖最后由 yizhong 于 2012-9-17 20:46 编辑 解:首先我们由题目的已知条件可以很容易的知道数列的每项都是为正整数 因为:$0<x_n\sqrt{11}-\big[x_n\sqrt{11}\big]<1,所以:$$\big[x_n\sqrt{11}\big]<x_n\sqrt{11}<\big[x_n\sqrt{11}\big]+1(1)$$ 由题目的条件我们可以得到:$$x_{n+1}-4x_n<x_n\sqrt{11}<x_{n+1}-4x_n+1(2)$$ 由(2)易得:$$(4+\sqrt{11})x_n-1<x_{n+1}<(4+\sqrt{11})x_n(3)$$ 由(3)我们又可以得到:$$4x_{n+1}-5x_n<x_{n+1}\sqrt{11}<4x_{n+1}-5x_n+4-\sqrt{11}(4)$$ 根据取整函数的定义我们由(4)就可以 得到:$$\big[x_{n+1}\sqrt{11}\big]=4x_{n+1}-5x_n(5)$$ 所以在$$n\geqslant2$$,且为整数的时候我们由(5)再结合题目的递推式我们就可以 得到:$$x_{n+1}=8x_n-5x_{n-1}(6)$$  由(6)我们又可以得到:$$x_{n+2}\equiv-x_n (mod10)(7)$$ 由题目的条件我们可得到:$$x_2=7 所以结合(7)式我们很容易得到x_{2013}(原题是x_{2012})的个位数字$    完毕
yizhong 7# 2012-9-17 21:47
晚上比较匆忙,所以暂且就此停笔,另外说明下,根据题目的意思只需要由首项再结合(7)式就可以得到题目所要求的,好了,现在爪机ing就说到这里~~~~
thread-676-1-6.html: [不等式] 一个有难度的不等式,dedicate to 一中
pxchg1200 1# 2012-8-4 22:54
Let$a,b,c$ be postive real numbers,Prove that: \[ \left(\frac{a}{4a+5b+3c}\right)^{\frac{2}{3}}+\left(\frac{b}{4b+5c+3a}\right)^{\frac{2}{3}}+\left(\frac{c}{4c+5a+3b}\right)^{\frac{2}{3}}\leq\frac{3}{12^{\frac{2}{3}}} \] (Tran Quoc Anh)
kuing 2# 2012-8-4 23:04
“dedicate to 一中”?啥意思…… 话说这几天一中不见人了
yizhong 3# 2012-8-5 12:03
现在还在四川ing........今天是一是上来看下小K,二是支持下论坛,三是确定下PX是不是真的被越南MM给拐了
kuing 4# 2012-8-5 12:05
你们都太忙鸟,老是消失
yizhong 5# 2012-8-5 12:07
初看了这个不等式,还是得请小K出手 ,估计我搞不掂
kuing 6# 2012-8-5 12:13
我也不会……指数把我吓着了
kuing 7# 2012-8-7 02:14
不知能不能证这个? \[\left(\sum\frac a{4a+5b+3c}\right)\left(\sum\frac a{5a+4b+3c}\right)\left(\sum\frac{5a+4b+3c}{4a+5b+3c}\right)\leqslant\frac3{16}.\] 挺好看的。
kuing 8# 2012-8-7 19:14
或者这个也不错 \[\left(\sum\frac a{(4a+5b+3c)^2}\right)\left(\sum a(b+c)\right)\left(\sum\frac1{b+c}\right)\leqslant\frac3{16}.\]
kuing 9# 2012-8-8 22:55
Michael <pxchg1200@sina.com>  22:40:10 http://www.artofproblemsolving.c ... p?f=52&t=492017 kuing<kuingggg@qq.com>  22:47:49 后面的还是不知怎么证。不过就前面那个 holder 比我那两个都好
kuing 10# 2012-9-12 01:24
\[ \sqrt{\frac{a}{a+2b+3c}}+\sqrt{\frac{b}{b+2c+3a}}+\sqrt{\frac{c}{c+2a+3b}}\le\sqrt{\frac{3}{2}} \] 来自:http://www.artofproblemsolving.c ... p?f=52&t=345061 arqady 的证法牛比…… 不知有没有类似上面那样的CS或Holder?
thread-677-1-2.html: 找茬
戊概念·五 1# 2012-8-4 23:16
本帖最后由 戊概念·五 于 2012-8-6 21:47 编辑 VS
froglove 2# 2012-8-6 22:03
山寨和正版?......==
kuing 3# 2012-8-6 22:07
除了最下面有点不一样之外看不出什么不同
戊概念·五 4# 2012-8-6 22:20
2# froglove 这是同一家店的两款机械表(至少图片下配的型号说明是不一致的),看起来同样的外观,但是价格却相差了200文,很好奇这200文差在哪里了。。。。。
froglove 5# 2012-8-6 22:26
你们这回帖速度......200文....可能是机芯配置?..
戊概念·五 6# 2012-8-6 22:30
再上去看看,发现、、、、、全部售罄,而且,三个同样的图片,型号说明一致了。。。。。 截图啊截图 忘记PSSR了诶~ 想找块没有镶钻的机械表很难么?????!看得上眼的都是男表,根本戴不住
kuing 7# 2012-8-6 22:33
反相再重叠了一下,的确除了下面之外没有不同
戊概念·五 8# 2012-8-6 22:55
7# kuing 真专业 我当时倒也发现了那里的细微区别,但认为是拍摄角度+展品摆放造成的
戊概念·五 9# 2012-8-6 23:01
7# kuing 走个题:你这是什么先进的玩意儿or技术?!
kuing 10# 2012-8-6 23:03
9# 戊概念·五 PS的几个简单小操作而已,玩找茬可以用来作bi。
戊概念·五 11# 2012-8-6 23:12
本帖最后由 戊概念·五 于 2012-8-10 12:55 编辑 10# kuing 《我笨我开心》,闪人~
kuing 12# 2012-8-6 23:14
11# 戊概念·五 我不是不懂打这个字,而是发这个词语有可能触动敏gan词被自动删除
戊概念·五 13# 2012-8-10 12:30
12# kuing 我也没讲你不懂。。。。。
kuing 14# 2012-8-10 12:34
13# 戊概念·五 那你打这个字出来干嘛
戊概念·五 15# 2012-8-10 12:41
本帖最后由 戊概念·五 于 2012-8-10 12:57 编辑 14# kuing 我傻了
kuing 16# 2012-8-10 12:43
15# 戊概念·五 …………说了“发这个词语有可能触动敏gan词被自动删除”你还发
戊概念·五 17# 2012-8-10 12:58
16# kuing 编了
kuing 18# 2012-8-10 13:06
一个 bi 字可能没什么问题,但跟前面的“作”连起来就有可能有危险
戊概念·五 19# 2012-8-10 14:48
18# kuing 嗯,知道了。
thread-68-1-2.html: 群里一极限题 $\lim_{x\to2}(\cos\frac\pi x)/(2-\sqrt{2x})$
kuing 1# 2011-10-7 12:16
\[ \lim_{x \to 2} \frac{\cos\frac\pi x}{2 - \sqrt {2x}} = \lim_{x \to 2} \frac{(2 + \sqrt {2x} )\sin \frac{\pi (x - 2)}{2x}}{2(2 - x)} = \lim_{x \to 2} \left(\frac{ - \pi (2 + \sqrt {2x} )}{4x} \cdot \frac{\sin \frac{\pi (x - 2)}{2x}}{\frac{\pi (x - 2)}{2x}}\right) = \lim_{x \to 2} \frac{ - \pi (2 + \sqrt {2x} )}{4x} = - \frac\pi2 \]
thread-681-1-7.html: [不等式] $\sum\frac{ab(a^2+b^2)}{\sqrt{(b^2+c^2)(c^2+a^2)}}\ge\sum a^2$
海盗船长 1# 2012-8-6 13:04
$a,b,c\ge 0$ \[\sum_{cyc} \frac{ab(a^2+b^2)}{\sqrt{(b^2+c^2)(c^2+a^2)}}\ge\sum_{cyc} a^2\]
kuing 2# 2012-8-6 14:41
左边看上去有点小,右边比较大,不等式居然成立,看来挺强,而且一个为0也能取等,估计难度不小。
kuing 3# 2012-8-6 17:45
我做得有点复杂,但过程中还算比较顺利,主要是因为平方约去abc后发现竟然也不是很强,总是出乎我意料之外。 \begin{align*}   & \sum \frac{ab(a^2+b^2)}{\sqrt{(b^2+c^2)(c^2+a^2)}}\geqslant \sum a^2 \\ \iff{}&\sum ab(a^2+b^2)\sqrt{a^2+b^2} \geqslant \prod \sqrt{a^2+b^2} \sum a^2 \\ \iff{}&\sum a^2b^2(a^2+b^2)^3+2abc\sum a(a^2+b^2)(a^2+c^2)\sqrt{(a^2+b^2)(a^2+c^2)}\geqslant \prod(a^2+b^2)\left( \sum a^2 \right)^2, \end{align*} 注意到 $a$, $b$, $c$ 中有一个为 $0$ 时不等式取等,所以 $\prod(a^2+b^2)\left( \sum a^2 \right)^2-\sum a^2b^2(a^2+b^2)^3$ 必定有 $a^2b^2c^2$ 的因式,通过待定系数就可以得到 \[\prod(a^2+b^2)\left( \sum a^2 \right)^2-\sum a^2b^2(a^2+b^2)^3=2a^2b^2c^2\left( 3\sum a^4+5\sum a^2b^2 \right),\] 因此,当 $abc=0$ 时原不等式成立,当 $abc\ne0$ 时,原不等式就等价于 \[\sum a(a^2+b^2)(a^2+c^2)\sqrt{(a^2+b^2)(a^2+c^2)}\geqslant abc\left( 3\sum a^4+5\sum a^2b^2 \right).\] 由柯西不等式,要证上式,只要证 \[\sum a(a^2+b^2)(a^2+c^2)(a^2+bc)\geqslant abc\left( 3\sum a^4+5\sum a^2b^2 \right),\] 即 \[\sum a^3(a^2+b^2)(a^2+c^2)\geqslant 2abc\left( \sum a^4+\sum a^2b^2 \right),\] 也即 \[\sum \frac{a^2}{bc(b^2+c^2)}\geqslant \frac{2\left( \sum a^4 +\sum a^2b^2 \right)}{\prod(a^2+b^2)},\] 由均值不等式,要证上式,只要证 \[\sum \frac{a^2}{(b^2+c^2)^2}\geqslant \frac{\sum a^4 +\sum a^2b^2}{\prod(a^2+b^2)},\] 令 $a^2=x$, $b^2=y$, $c^2=z$, $x$, $y$, $z>0$,则上式等价于 \[\sum x(x+y)^2(x+z)^2 \geqslant \left( \sum x^2 +\sum xy \right)\prod(x+y).\] 到这里我也没什么好办法,还是老实展开算了,为方便书写,记 $s=\sum x$, $p=\sum x^2$, $q=\sum xy$,则有 \begin{align*} \sum x(x+y)^2(x+z)^2 &=\sum x\bigl(x(x+y+z)+yz\bigr)^2 \\ & =s^2\sum x^3+2xyzs^2+xyzq \\ & =s^2\bigl((p-q)s+3xyz\bigr)+2xyzs^2+xyzq \\ & =s^3(p-q)+xyz(5s^2+q) \\ & =s(p+2q)(p-q)+xyz(5p+11q), \end{align*} 以及 \[\left( \sum x^2+\sum xy \right)\prod(x+y)=(p+q)(sq-xyz),\] 所以 \begin{align*}   & \sum x(x+y)^2(x+z)^2-\left( \sum x^2+\sum xy \right)\prod(x+y) \\ ={}&s(p+2q)(p-q)+xyz(5p+11q)-(p+q)(sq-xyz) \\ ={}&s(p+2q)(p-q)-sq(p+q)+xyz(6p+12q) \\ ={}&s(p^2-3q^2)+6xyzs^2 \\ ={}&s\left( \sum x^4+2\sum x^2y^2-3\left( \sum x^2y^2+2xyz\sum x \right)+6xyzs \right) \\ ={}&s\left( \sum x^4-\sum x^2y^2 \right) \\ \geqslant{}& 0, \end{align*} 原不等式获证。
kuing 4# 2012-8-6 19:24
有什么链接之类的记得也贴一下
海盗船长 5# 2012-8-6 20:08
4# kuing http://www.artofproblemsolving.c ... ;p=2762252#p2762252
海盗船长 6# 2012-8-6 20:16
http://tieba.baidu.com/p/1773591829
kuing 7# 2012-8-6 20:26
5# 海盗船长 arqady 的思路跟我前面一样了,只是我后面没那么暴力,但也比较麻烦
kuing 8# 2012-8-6 20:29
6# 海盗船长 这个最后那个证明值得看看,有可能是此题命题者的想法
kuing 9# 2012-8-6 20:49
6# 海盗船长 看到链接里的这个链接 http://tieba.baidu.com/p/1215996507 里提到的题目来源“2006年摩尔多瓦,原题是全用边表示的”,才想起原来之前在人教论坛也玩过的(见附件),鱼儿那个证法跟那里也一样。
kuing 10# 2012-8-6 20:52
不过回想不起来也有好处,不然估计就不会有3#的证明。 唔,话太多了……闪走
海盗船长 11# 2012-8-6 22:06
9# kuing 哦想起来了,先在JR里也讨论过
kuing 12# 2012-8-6 22:09
11# 海盗船长 嗯,鱼儿那个回贴我帮贴的图也就是在JR截的
thread-682-1-7.html: [函数] 一中的一个函数方程题
pxchg1200 1# 2012-8-6 20:15
\[ f(x)+f(\frac{x-1}{x})=1+x  (x\neq 0,1) \] 求$f(x)$ 解:以$\frac{t-1}{t}$代$x$,可得 \[ f(\frac{t-1}{t})+f(\frac{-1}{t-1})=\frac{2t-1}{t} \] 另以$ \frac{-1}{t-1}$ 代$x$,可得 \[ f(\frac{-1}{t-1})+f(t)=\frac{t-2}{t-1} \] 从原式加上第3个减去第2个,我们有 \[ f(x)=\frac{(x^3-x^2-1)}{2x(x-1)} \] Done!
kuing 2# 2012-8-6 20:24
汗,一中就是在这里 http://kkkkuingggg.5d6d.net/thread-665-1-1.html 截上群里的……你又转回来这里
thread-684-1-7.html: [不等式] a/b+b/c+c/a,28,12
kuing 1# 2012-8-7 01:22
$a$, $b$, $c>0$ \[\frac ab+\frac bc+\frac ca + \frac{28(ab+bc+ca)}{(a+b+c)^2} \geqslant 12.\] 貌似是道经典,不知这里有没有发过,晚上在不等式群看到,不知最早出自何处?
pxchg1200 2# 2012-8-8 22:01
Proof: Expanding and combining like terms,the inequality turns into \[ \sum{\frac{a^3}{b}}+\sum{\frac{a^2b}{c}}+2\sum{\frac{ab^2}{c}}+7\sum{ab}\geq 10\sum{a^2} \] or,equivalently, \[ \left(\sum{\frac{a^3}{b}}+\sum{\frac{a^2b}{c}}-2\sum{\frac{ab^2}{c}}\right)+\left(4\sum{\frac{ab^2}{c}}-3\sum{ab}\right)\geq 10\sum{a^2}-\sum{ab} \] Notice that: \[ \left(\sum{\frac{a^3}{b}}+\sum{\frac{a^2b}{c}}-2\sum{\frac{ab^2}{c}}\right)=\sum{\frac{b(a-b)^{2}}{c}} \] and \[ \left(4\sum{\frac{ab^2}{c}}-3\sum{ab}\right)=\sum{\frac{c(2a-3b)^{2}}{b}} \] Applying the AM-GM Inequality,we get \[ \sum{\frac{b(a-b)^{2}}{c}} +\sum{\frac{c(2a-3b)^{2}}{b}}\geq 2\sum{(a-b)(2a-3b)} =10\sum{a^2}-10\sum{ab} \] Hence we are done!
kuing 3# 2012-8-8 22:04
niubility的证法,不知是谁最先得到的?
pxchg1200 4# 2012-8-17 20:13
3# kuing 当然是Can他们这群人了。另外,arqady提出 Let $a$, $b$ and $c$ are positive numbers. Prove that: \[\frac{a}{b}+\frac{b}{c}+\frac{c}{a}+2\geq\frac{14(a^2+b^2+c^2)}{(a+b+c)^2}\]
thread-685-1-2.html: [转]逛Q空间看到的
kuing 1# 2012-8-7 14:20
试贴个长图,已去广告
戊概念·五 2# 2012-8-13 00:52
1# kuing HR。。。。。话说,你需要和这种人打交道?!
kuing 3# 2012-8-13 01:05
2# 戊概念·五 HR是什么?打什么交道?
戊概念·五 4# 2012-8-13 22:29
3# kuing HR-Human Resource人力资源,目标是让企业HR更好地进行人力资源的发展和规划。系统重点是实现人力资源部门在员工素质管理、薪酬管理、绩效考核等方面的需求。 你是制度外的人么=。=
kuing 5# 2012-8-13 22:47
4# 戊概念·五 哦 我不了解这些,我只是觉得上面说的有些还是挺有道理的,不管是在什么环境。
戊概念·五 6# 2012-8-26 00:01
5# kuing 大气的道理往往都很简单 试着接接下句?
kuing 7# 2012-8-26 01:06
6# 戊概念·五 接什么?
戊概念·五 8# 2012-8-26 23:30
7# kuing “大道理往往都很简单”“但简单不等同于容易” 自己补起~ 看来我的跳跃度还在,依旧是异于旁人的节奏。。。。。
kuing 9# 2012-8-26 23:47

戊概念·五 10# 2012-8-27 00:10
9# kuing 您就慢慢耗着,精气虚滞之时可再会美丽的天使姐姐
thread-686-1-1.html: 题目简短的难题。。
海盗船长 1# 2012-8-7 16:13
如何放置固定体积的某种不可压缩的均匀液体,使得空间中某给定点上的引力值最大?
秋风树林 2# 2012-11-15 20:13
感觉像一类泛函问题... 我只是随口说说
thread-687-1-2.html: 抠完PP被自己熏倒
kuing 1# 2012-8-7 17:02

叶剑飞Victor 2# 2012-8-8 20:13
thread-688-1-2.html: [转]Google搜索式子可以直接画出3D图了
kuing 1# 2012-8-7 17:06
其实标题所言的内容不是重点,重点在于这个链接里的某些图:http://www.guokr.com/post/132868/ 如果你用 chrome,可以试下点击这个链接:https://www.google.com/#hl=en&am ... f.,cf.osb&cad=b
thread-689-1-7.html: [函数] 群内看到的一个证周期函数的题,不知有没有人做出来,谢谢了!
hongxian 1# 2012-8-7 22:06
本帖最后由 hongxian 于 2012-8-7 22:39 编辑 已知函数$f(x)$(定义域为$R$)满足$f(x)+f(x+2a)=f(x+1)$,证明$f(x)$为周期函数. 看样子是个错题,已经被战巡举出了反例,"这个只要令$f(x)=k^{x}$,其中$k$满足方程$k^{2a}+1=k$即可,然后...$f(x)$不是周期函数" 题目如果改成:已知函数$f(x)$(定义域为$R$)满足$f(x)+f(x+2)=f(x+1)$,证明$f(x)$为周期函数. 则$f(x)$就是以6为周期的周期函数了.
kuing 2# 2012-8-7 22:36
可能那个 a 打多了,或者后面的 +1 其实是 +a 不过讲开又讲,除了 a=1/2 和 a=1 之外还有没有其它的 a 使之能推出周期?
thread-69-1-1.html: 粗体向量
kuing 1# 2011-10-7 14:41
在公式里用 \boldsymbol 命令得到斜粗体。 \boldsymbol{a}=(a_1,a_2,a_3), \boldsymbol b=(b_1,b_2,b_3) 效果:$\boldsymbol{a}=(a_1,a_2,a_3), \boldsymbol b=(b_1,b_2,b_3)$ 叉乘一下,效果:\[\boldsymbol a\times\boldsymbol b = \begin{vmatrix}\boldsymbol i & \boldsymbol j & \boldsymbol k \\ a_1&a_2&a_3\\b_1&b_2&b_3\end{vmatrix}\]
thread-692-1-7.html: [不等式] 网友问的一数列不等式
kuing 1# 2012-8-8 22:22
已知 $a_1=1$, $a_2=2$, $a_{n+2}=a_{n+1}+a_n$,求证 \[\sqrt[n]{a_{n+1}} \geqslant 1 +\frac1{\sqrt[n]{a_n}}.\]
pxchg1200 2# 2012-8-8 22:44
直接把通项求了吧
kuing 3# 2012-8-8 22:54
通项就是斐波拉少了第一个数,不过用通项对后面的证明好像也不太简单吧
yizhong 4# 2012-8-9 01:53
本帖最后由 yizhong 于 2012-8-9 01:57 编辑 基本上可以秒,我们令$a_0=1$,则$a_{k+1}=a_k+a_{k-1}$即:$1=\dfrac{a_k}{a_{k+1}}+\dfrac{a_{k-1}}{a_{k+1}},k=1,2,3....$于是我们有 $n=\sum_{k=1}^{n}\dfrac{a_k}{a_{k+1}}+\sum_{k=1}^{n}\dfrac{a_{k-1}}{a_{k+1}}$,接下去只需要对这个式子使用n元的均值不等式即可得到所要证明的式子。
kuing 5# 2012-8-9 02:01
4# yizhong 好解法 嘿,代码打得还可以,我看到你只修改了一次就没什么错误了。不过我还有建议一些重点公式用行间居中的那种会比较好
yizhong 6# 2012-8-9 02:05
我又回来了,代码是得意于小K的真传
kuing 7# 2012-8-9 02:13
\[n=\sum_{k=1}^n\frac{a_k}{a_{k+1}}+\sum_{k=1}^n\frac{a_{k-1}}{a_{k+1}}\geqslant n\sqrt[n]{\frac{a_1}{a_{n+1}}}+n\sqrt[n]{\frac{a_0a_1}{a_na_{n+1}}} \implies \ldots \] 引用我的贴子也可以看到我的代码。
thread-693-1-2.html: niubility
kuing 1# 2012-8-9 15:51
自从最近我经常说 niubility 之后,貌似越来越多人也说
戊概念·五 2# 2012-8-10 12:34
1# kuing 名人引发的追随流: 通常粉丝会刻意或不自觉的去模仿自己的偶像     or     人难免会受到常接触的人的一些影响,口气、用语、甚至动作气质……
叶剑飞Victor 3# 2012-8-17 01:50
这个Chinglish发明得不错。。。。。
kuing 4# 2012-8-17 01:51
发明得不错是好事嘛,怎么了呢
戊概念·五 5# 2012-8-17 21:19
3# 叶剑飞Victor 隐约记得这个我们老师很早就在课堂上频频提及了。。。。。
thread-694-1-2.html: 话说我进了一个Q群,里面的人……
kuing 1# 2012-8-9 19:42
话说我进了一个Q群,发现这群里的人的群名片如图所示 擦,各种伟人啊! 你们猜猜这是什么群? and,我应该改个什么名字好呢……
海盗船长 2# 2012-8-9 21:49
伟人群。。
海盗船长 3# 2012-8-9 21:51
改成高斯
kuing 4# 2012-8-9 22:55
3# 海盗船长 高斯已经有了,暂时改成了柯西 是什么群的答案改天公布……
海盗船长 5# 2012-8-9 23:44
坐等答案
海盗船长 6# 2012-8-10 00:05

海盗船长 7# 2012-8-10 23:56
一天都过了唉
kuing 8# 2012-8-11 00:21
7# 海盗船长 好吧……都没人看也没人猜(除楼上)……我无聊了 其实大概不会有人猜中 答案如图所示(群头像和群号码部分已作打码处理)
海盗船长 9# 2012-8-11 11:09

叶剑飞Victor 10# 2012-8-11 15:25

戊概念·五 11# 2012-8-11 19:08
本帖最后由 戊概念·五 于 2012-8-11 19:55 编辑 那啥在我们这也是个牛气哄哄的特征符。。。。。基本上能躲就躲= =
kuing 12# 2012-8-11 19:46
11# 戊概念·五 说了多少次不要出现敏感词
戊概念·五 13# 2012-8-11 20:36
12# kuing 已编
pxchg1200 14# 2012-8-11 20:46
我擦,**交流群?!
kuing 15# 2012-8-11 20:49
14# pxchg1200 编辑掉了敏感词。
kuing 16# 2012-8-11 20:50
对了,那个敏感词以后用中文说——“艾威”,这样应该会安全些。
thread-695-1-2.html: 论坛的图片挂掉了
叶剑飞Victor 1# 2012-8-10 00:12
本论坛的图片都不能显示了 实际上是http://tt.a.5d6d.org/网站挂掉了。图片都在这个网站里面。 不过表情符号还是可以用的。
叶剑飞Victor 2# 2012-8-10 00:21
从HTML源代码上看来,用户上传的图片在http://tt.a.5d6d.com/userdirs/e/3/kkkkuingggg/attachments/目录中。而这个目录有被301重定向到http://tt.a.5d6d.org/userdirs/e/3/kkkkuingggg/attachments/的对应目录中。结果http://tt.a.5d6d.org/网址又挂了,然后图片就无法显示了。
kuing 3# 2012-8-10 00:21
没发现?……都还好吧
叶剑飞Victor 4# 2012-8-10 00:22
本帖最后由 叶剑飞Victor 于 2012-8-10 00:24 编辑 3# kuing 自己看吧。 http://kkkkuingggg.5d6d.net/thread-507-1-1.html http://kkkkuingggg.5d6d.net/thread-661-1-1.html
kuing 5# 2012-8-10 00:24
噢,知道了,我把IE缓存清理了之后就看不到了。
叶剑飞Victor 6# 2012-8-10 00:27
本帖最后由 叶剑飞Victor 于 2012-8-10 00:30 编辑 5# kuing 连左边的用户头像都挂了。 用户头像的网址: http://tt.a.5d6d.com/server_5d6d/images/xxxxx.gif 301重定向到 http://tt.a.5d6d.org/server_5d6d/images/xxxxx.gif 可是http://tt.a.5d6d.org/域名解析挂了。然后,用户头像就挂了。
kuing 7# 2012-8-10 00:29
嗯,5d6d官网已经有人在反映了,不止我这里,很多组都有问题,等修复吧。 暂时点击下载好像还可以下载下来。
叶剑飞Victor 8# 2012-8-10 00:34
原来表情符号用的搜搜的表情。HTML源代码中,“”表情的网址分明写着http://cache.soso.com/img/img/e136.gif。话说这表情不侵权吗?
kuing 9# 2012-8-10 00:46
8# 叶剑飞Victor QQ收购了5d6d
kuing 10# 2012-8-10 01:08
技术人员可能睡觉了,估计要明天才修复正常,我也先闪一会了
kuing 11# 2012-8-10 12:38
http://www.5d6d.net/thread-952331-1-1.html
thread-696-1-2.html: 今天google首页的足球
海盗船长 1# 2012-8-10 11:37

海盗船长 2# 2012-8-10 12:10
本帖最后由 海盗船长 于 2012-8-10 23:28 编辑
kuing 3# 2012-8-10 12:45
擦,太慢了……
thread-698-1-7.html: [不等式] 2012中国女子数学奥林匹克第一题送分不等式
kuing 1# 2012-8-10 19:08
已知 $a_1$, $a_2$, \ldots, $a_n$ 均为非负实数,求证 \[\frac1{1+a_1}+\frac{a_1}{(1+a_1)(1+a_2)}+\frac{a_1a_2}{(1+a_1)(1+a_2)(1+a_3)}+\cdots+\frac{a_1a_2\cdots a_{n-1}}{(1+a_1)(1+a_2)\cdots(1+a_n)}\leqslant1.\]
pxchg1200 2# 2012-8-10 21:32
很惭愧的表示这分我收不到。。。 做不来
kuing 3# 2012-8-10 21:50
2# pxchg1200 直接将最后的 $a_n$ 放掉就完事了……
yizhong 4# 2012-8-11 15:48
oh,拿分直接走人
thread-699-1-1.html: zz【更新在8.25】
戊概念·五 1# 2012-8-10 22:15
本帖最后由 戊概念·五 于 2012-8-25 23:47 编辑 → 问:我是已婚MM有套小房,现想换套大房,把小房送父母,但过户费太高。请问怎么减免相关费用? 律师回复:与老公离婚,房给老公,房产证去掉你名字。爸妈离,老公和老妈结婚,房产证加妈名。老公再与老妈离,房给老妈,去老公名,然后各自复婚,房加爸名,共离婚结婚六次,花费共54元!(职场学堂微博) → 一次去洗澡,我对搓澡师傅说:“师傅,我有点脏,最近忙,有阵子没洗澡了。”师傅大义凛然地说:“这都不是事,干搓澡这么多年,多埋汰的我没见过啊!”搓了一会,师傅内伤地说:“小伙子,你过分了啊!”(1******8191) → 学校禁止谈恋爱,但是我们班有两个同学仍在偷偷摸摸地谈,前天被班主任发现后,叫来双方家长。班主任本来是想让家长说说自己的孩子。结果家长们聊了聊,发现对方家庭情况都还不错,然后今天下午突然传出消息说,他们订婚了……
海盗船长 2# 2012-8-10 22:18
离婚不要钱么。。
戊概念·五 3# 2012-8-10 22:27
2# 海盗船长 把“花费”错打成“话费”了呃。。。。。
kuing 4# 2012-8-10 23:23
1、“现象换套大房”…… 2、多埋汰?;内伤地说?;看不明白。 3、以前看过
戊概念·五 5# 2012-8-11 19:14
4# kuing 1. 又被捉误了。。。。。改去~ 2.那个,大概算是北方方言?应该不是太原话.....回头百一下。。 3.那些没有网络的日子,只好看手机报了,上面确实有不少是老段子新发·····
戊概念·五 6# 2012-8-11 19:17
内伤地说? kuing 发表于 2012-8-10 23:23 这个、、、、、古装武打戏里,应该有见过“内伤”的说法吧?!内伤地说,就像是受了内伤似的说。。。。。
戊概念·五 7# 2012-8-11 23:37
4# kuing 1. 又被捉误了。。。。。改去~ 2.那个,大概算是北方方言?应该不是太原话.....回头百一下。。 3.那些没有网络的日子,只好看手机报了,上面确实有不少是老段子新发····· 戊概念·五 发表于 2012-8-11 19:14 那词确实不是太原话——百度百科:http://baike.baidu.com/view/3256.htm?pid=baike.box
戊概念·五 8# 2012-8-13 22:46
→ 几个哥们在一起聊天,有个家伙放了个很臭却不响的屁,刚放完就对大家说:“哎,什么东西糊了?”然后大家使劲吸鼻子……这人简直损透了。 → 爸妈吵架,老妈一怒之下离家出走。期间老爸各种电话、短信道歉,老妈都不理他。我给老妈打电话就说了三个字,老妈立马就回家了。老爸一个劲儿问我说了啥。我无奈说道:“承认吧,老爸,你已经失宠了。我只跟我妈说我饿了。” → 一天,妻子心血来潮,站在镜子前端详自己,然后伤心地哭起来:“原来我的脸这么难看!”一旁的丈夫走过来,拍着她的肩膀安慰道:“你只是偶尔照一次镜子而已,没必要这么伤心。想想我,每天都要看着你,我都没哭……”(找丈夫必须找像这样有强大心理承受能力的人。。。。。嗯!)
海盗船长 9# 2012-8-16 15:21

戊概念·五 10# 2012-8-25 23:38
→ 本人男,昨天和我妈去买衣服,发现试衣间锁不上,就让我妈帮看着点。结果刚把裤子脱了门就开了,一个漂亮的妹子出现啦。我瞬间就震惊了,只穿了一条内裤有木有啊!后来我妈说,看那女孩挺漂亮的就没拦着。我去,真是我亲娘啊! → 家有六岁小女儿,一天和妈妈吵完嘴后,转身对着爷爷奶奶说道:“瞅瞅你俩,怎么给你儿找的媳妇,整天欺负我。” → 有人发状态说:“英国人发明乒乓球时想不到还有中国这个神一般的国度吧!” 这时候神评出现了,“中国蹴鞠的时候一样没有料到今天……” → 中国奥运有三宝:跳水,体操,乒乓好。韩国奥运有三宝:射箭,姓朴,眼睛小。美国奥运有三宝:篮球,田径,家世好。非洲奥运有三宝:人黑,牙白,巨能跑。巴西奥运有三宝:足球,球迷,金牌少。英国奥运有三宝:小贝,女王,观众吵。 → 同事家装修房子,去建材市场转了转,最好的壁纸竟然报价3000/平方米。一看这价格,我们算了下,建议道:“你不如直接用20块钱的纸币糊墙,每平方米不到两千元,不仅可以省一千多块钱,看起来还霸气。”
戊概念·五 11# 2012-8-25 23:46
本帖最后由 戊概念·五 于 2012-9-28 00:25 编辑 → 内涵,原来如此: 我没钱你会爱我吗? ——我没钱,而且以后也不打算努力奋斗,但你必须为我身上并不明显的美好品质义无反顾,否则就是贪财!就是出卖爱情! (呵呵,原来之前是我太过天真了——竟以为这不过是种极端的假设= =) → 那年,他在咖啡店等人,一女孩过来问:“你是王阿姨介绍来相亲的吗?”他抬头打量一下她,正是自己喜欢的类型,心想何不将错就错,于是忙答应道:“对,请坐……” 结婚当天,他坦白当时自己不是去相亲的。老婆笑道:“我也不是去相亲的,只是找个借口和你搭讪。” → 本人呢暗恋一女神,打电话发短信均无果。昨夜再次电话,女神发我一张图,上面有两只鸽子在接吻。 吾窃喜,研究一夜不懂。第二天她告诉我:“鸽吻……滚!” → 辅导班老师给学生布置了不少作业,末了他随口说道:“做错一题罚两题,做对两题奖励一题。”全班同学都不淡定了。
戊概念·五 12# 2012-9-26 01:46
→ 她经常来我这儿借书,有意思的是明明借过的书了,仍像新书似的借走了,还很感兴趣地问我书怎么样。后来我问她:“怎么借书也不会借?一本书借了好几次。”她却振振有词地说:“我是故意的,要不你还真以为我喜欢你的书呀!” → 有人问我,如果你喜欢的一个人不喜欢你,你怎么办? 我说:我从来不会喜欢上不喜欢我的人。没眼光的男人,我干嘛要喜欢呀。 → 也许有人会说男女之间存不存在纯洁的友谊,我不想讨论这个问题。也许异性朋友之间,会偶尔有点心动,甚至有点小冲动,但这都是再正常不过的情感,不能说这样的友谊就肮脏了。 也不要问我,男朋友介意我有异性朋友,我该怎么办?我也不知道。我只遇到过一个这样的男人。那一次,爱情与友谊之间,我毫不犹豫地选择了友谊。我才不会因为一个无法信任我的男人,放弃多年的好哥们儿。 → 爱情从来不是我生活的目标,幸福才是(享乐主义嘴脸暴露无遗╮(╯▽╰)╭),而爱情只是提高我幸福指数的其中一项。所以我对爱情的态度是:管他妈的真爱不真爱,只要让我觉得累,让我爱得不幸福,都给老娘滚蛋。两人互相折磨、要死要活的那种真爱,送我我也不要。让我幸福无比,让我更爱自己,更爱身边的人,让门口的麻辣烫吃起来更香,让纯净水喝起来都有滋有味的爱,就算不是真爱,老娘也要定了。 题外→ 一些人笑起来像哭一样难看,证明这种人警戒心很强,不轻易相信别人,十分谨慎,注意钱财的流向。跟这种人结交一旦取得他的信任,他一定会帮你成事。
戊概念·五 13# 2012-9-28 00:30
→ 我们是动物进化来的,即使高级也还是动物,达尔文主义一直适用。只有把自己训练成更敏捷、更强壮的动物,才能活得好点。所以懒散消极肯定不是长久之计,要是在动物世界,在金字塔底的你还这么耗着,已经死了。 → 如果你想做正确的事,就不要再担心你是否可能做着错误的事! → 儿子不好意思说出你的名字,是因为你的名字的确让他不好意思。 → 中年男性谈论其妻子时的脑电波和正在看恐怖片时的一样 → 真正的陪伴,经得起坎坷,经得起平淡。
戊概念·五 14# 2012-10-2 22:12
→ 人生如果错了方向,停止就是进步。 → 好女人是一所学校,好男人毕业了可以留校任教。 → 每个人的一生都有许多梦想,但如果其中一个不断搅扰着你,剩下的就仅仅是行动了。 → 结论就是你懒得再想下去的地方——路的尽头,仍然是路,只要你愿意走。 → 胆怯的人在危险前被吓住了;懦弱的人在危险中被吓住了;勇敢的人在危险过后被吓住了。 → 失败发生在彻底的放弃之后。
戊概念·五 15# 2012-10-5 23:25
→ 一天女友生气了,大踏步往前走。我在后面边追边喊:“美女,你掉了个男朋友!”GF回过头来恶狠狠的说:“掉了再买一个!”我问:“去哪买?”GF:“怎么,你也要买?”“不,我去卖。” → “老公,我睡不着,给我讲个故事吧。”老公:“好吧。很久很久以前,有个青年,在医学院念书,有一年,他参加了执业医师考试,考试内容包括:生理、生化、病理、药理、免疫、微生物、预防、统计、流病、卫生法、心理、伦理、内科……”还没讲完,老婆就睡着了。 → 当我们喜欢上一个人的时候,她是展现在公众面前的样子:自信、优雅、幽默、体贴、善解人意。当我们爱上了那个人,才发现她身体里那么多喜怒哀乐,她的悲伤、软弱、无理取闹。她的种种,让你觉得你怎么会爱上这样的一个人。这个时候我们总会忘了一件重要的事——你能看到这些,是因为她也爱着你。 → 人生有三样东西是——无法隐瞒的:咳嗽、贫穷和爱(想隐瞒,却欲盖弥彰);不该挥霍的:身体、金钱和爱(想挥霍,却得不偿失);无法挽留的:生命、时间和爱(想挽留,却渐行渐远);不该回忆的:灾难、死亡和爱(想回忆,却苦不堪言) → 面对自己喜欢的女孩子要敢于争取珍惜,不管有什么样的阻力,除非你觉得这个那孩子不值得你爱。一旦失去、你后悔都来不及。
戊概念·五 16# 2012-10-9 23:31
→ 一个男生很喜欢那个女孩,一直都紧追不舍,女孩对男生又不感冒,由于男生死缠烂打,终于有一天女生在上课的时候,转身大力拍了桌子,大声对男生说:“你到底喜欢我什么啊?我改还不行?”男生说:“我就喜欢你不喜欢我,你改啊!” → 昨晚在火车上吃泡面“老坛酸菜面”吃得正香!后座的一个7、8岁的小男孩伸头过来,之后说了一句让我吐血的话:“妈妈,叔叔在吃脑残酸菜面,我也要吃。”孩子你让我怎么往下吃啊 → 中学聚会,女生互相调侃着自己的容貌身材。微胖的小丽对小枫说:“你看我身材怎么样?”小枫打量了一下说:“还好。”小丽紧了一下眉头说:“你骗我,我男朋友说我没有腰。”小枫一听,接话道:“胡说!这么粗的腰还说没有腰。”..... → 一次路考,上路没多久,男教练就靠近她的耳边问:“你是不是很喜欢我?”女孩听了顿时面红耳赤,然后望着教练说:“你怎么知道的?”教练怒道:“教你多少次了,无论别人说什么,你的眼睛都要看着路!” → 有个广告微博发这样的垃圾给我:亲,新浪用户提醒!您有一条私信是来自于微博活动!提示你本人获得二等奖,请登录……我立即举报了它,举报理由我写的是:SB连行骗广告都不给我一等奖,我很受伤!
戊概念·五 17# 2012-11-6 21:52
→ 那些说你做不到的人,也许实在担心你真的会做到。 → 伊拉克战争期间最流行的3个词:和平peace、战争war、发现found,把这三个单词连起来大声朗读三遍,你会有惊天动地的发现:屁是我放的…… → 一个山洞学生和同寝室的东北人一起去小卖店买方便面,路上东北人自言自语:“整个啥味儿的呢?整个葱香牛肉的吧……”山东人不知道整是啥意思,就问东北人,东北人告诉他整就是吃的意思。有一次二人一起去厕所,碰巧厕所停水,里面万两黄金……东北人一拍大腿:“这可咋整啊!~”山东人听罢面如土色,双手扶墙呕吐不止。 → @宋晓军 有人问莫言会不会得奖,旁边有朋友说会的,因为莫言写了中国农民在没有真炸药的情况下,用高粱酿的就发明的“炸药”炸了鬼子。诺贝尔不就是发明炸药的吗? → 爱错过一些人,被一些人错爱过,才慢慢明白了什么适合自己、自己真正想要什么、能切实拥有的又是什么。在明白这些之后,遇上一个对的人,就拥有了对的情感。
戊概念·五 18# 2012-12-3 17:59
http://www.cchhdd.com/player/vid ... dG1sP2ZzPTE%3D.html 真不知是哪里的考场呢,这线路,双倍于考试速度限制的规格跑起还真是有够刺激呐~
戊概念·五 19# 2012-12-27 18:05
→ 马上又周考了,数学老师在办公室里出试卷,他把试卷上选择题的答案设成了AAAAA,AAAAA,AAAAA,AAAAA,然后得意地对另一位老师说:“我要让好学生不敢写,坏学生不敢抄。哥拼的就是心跳!” → 孙云晓教育网: #对话孙云晓#问题孩子多的一个重要原因是问题父亲多。(由此可知,为了下一代考虑,对于孩儿TA爹的挑选,是要慎重的;除非、其实你并不在意TA将来能成为一个怎样的人) → 找不到对的人,其实很可能是,改不掉错的自己。 → 爱,愛。会意字。其本意为亲爱、喜爱,指对人或事物有深厚、真挚的感情。繁体字“愛”的上面是“爪”,中间是“冖”“心”,下面是“攵”。“爪”为手,表示给予、付出,也指收取、获得;“冖”为“冥”字头,意为深奥玄妙;“心”为内心,心灵,是人的心理活动;“攵”为行走、行动。“爪”“冖”“心”“攵”为“愛”,意为将内心深沉的情感化为行动,施与他人,或通过行动赢得他人的情感。 → 熬夜,是因为没有勇气结束这一天;赖床,是因为没有激情开始这一天。。
戊概念·五 20# 2013-1-7 23:10
→ 狡猾和聪明的差距不是在智力上,而是在道德上。 → 上小学时我上课爱睡觉,一次语文课老师布置作业,写一篇题目为《假如我是蜘蛛》的作文,下课后我向同学问了题目。晚上,我在家绞尽脑汁,终于写了篇《假如我是只猪》的作文,后来,我在学校火了 → 大学寒假时,坐火车回家,旁边座位上是一位长相清秀的女孩子,看样子也是学生,我搭讪道:“你好,你是哪个学校的?” 女孩答道:“你好,我是某某医科大学的。” 一看搭讪成功,我赶快套近乎:“哇,原来是白衣天使啊,那我以后看病就找你喽!” 女孩勉强挤出一丝笑容,有些尴尬地应道:“好。” 我继续满怀热情地追问:“那你学的是哪个专业啊?” 女孩顿了一下,答道:“法医……” → 我曾经喜欢上一个学医的女孩子,和她一起去上课,那天不知道老师怎么了,开始讲如何下药让丈夫不明原因猝死:有半年的吃法,1年的吃法,10年的吃法。看着她明眸认真地做笔记,我下定决心、还是和她做好朋友吧! → 一学姐男友劈腿。本来她想就算了,但是他非常贱,还天天拉着新欢特别高调地在她跟前转。 终于有一天,学姐在食堂忍无可忍和他们吵了起来。盛怒之下,她用餐刀捅了前男友20多刀,但竟然都避开了重要内脏和主动脉。 该男受到了巨大的生理心理创伤,还不能算重伤。 学姐因此成为解剖课年年必讲的优秀案例:知识就是力量!
thread-699-2-1.html:
isea 21# 2013-2-28 20:31
可乐可乐啊 有意思
戊概念·五 22# 2013-3-16 22:48
22# isea 是?冰羊吗?!
kuing 23# 2013-3-16 22:54
23# 戊概念·五 不是
戊概念·五 24# 2013-3-16 23:03
24# kuing 呐~ → 春节看到小侄女,我第一句话就是:“考的怎么样啊?”我要把这种痛苦一代一代传承下去!小侄女答道:“我和男朋友都没考好。叔叔你女朋友呢?”我……
戊概念·五 25# 2013-3-19 19:05
→ 今天开玩笑问6岁的表妹:“我们准备养头猪,但是需要安排工作,要选一个人每天给猪喂好吃的,一个人每天给猪打扫房间,一个人每天给猪洗澡,还要选一个人每天陪猪玩儿,请问你要做什么?”她毫不犹豫的答道:“做猪!” → 今天早上坐公交车上班,人不是很多,一老头不停地在车上换位置。只要有人下车离开座位,老头就迅速坐过去。在红绿灯的时候,司机幽幽的回头说:“阿爷,您在我车上玩跳棋呢?” → 一哥们儿买了一枚两块钱的戒指,跑到西湖边表演高富帅,左手捏着这钻戒,右手打电话假装歇斯底里地喊:真的不和我结婚吗!真的不愿意和我在一起吗!那分手吧!永远永远都不要在一起了!然后做了一个极度潇洒的动作……把手机扔进湖里了 → 某工科学校,男女比例7:1,话说新生军训那会儿,晚上男女生拉歌,男生人多声音洪亮,女生劣势非常明显,后来教官教了女生一首歌,直接就把男生秒杀了。只听见一群姑娘大吼:两只老虎,两只老虎,谈恋爱,谈恋爱,两只都是公的,两只都是公的,真..真..! → 上公厕,没带纸。身边只有一百元大钞及一部手机,拨通KFC:“麻烦送个全家桶,特殊要求是男的,多带点纸,位置某街XX男厕二号坑”
戊概念·五 26# 2013-4-26 23:51
→ 真正的朋友在你损他们的时候不会生气,他们只会笑着回敬你更损的话。 → 一人找算命先生问:"先生,我听说左眼跳财右眼跳灾,可是我今天两个眼睛一起跳,到底预示着什么啊?"算命先生捋髯道:"嗯……你的人身意外伤害险快派上用场了。" → 儿子刚学说话时,他天天对儿子说:“叫爸爸。”儿子跟着学,也说:“叫爸爸。”久而久之,儿子养成习惯,见着他便说:“叫爸爸。”他没办法,开始纠正,现在天天对儿子说:"爸爸。"   → 一位同学巨胖,300斤。一日体检,医生看了他的体形说:"不用称了,自己写吧。"那哥们儿拿着笔犹豫半天,觉得写150kg不好看,心一慌写了个0.15吨。 → 我问爸爸,你和妈妈是怎样恋爱的!妈妈抢过话说,当初很多人追我呢,为了娶我甚至他们都打架决斗,其中就有你老爸。爸爸冷冷的说道:"对!有这事。谁输了谁娶她,都玩命的打啊。我当年很瘦,没打赢。"结果,我和我爸晚上没饭吃= =
李斌斌755 27# 2013-4-27 00:07
灌水无敌将。
kuing 28# 2013-4-27 00:32
27# 李斌斌755 人家的水营养丰富,你也得学下……
李斌斌755 29# 2013-4-27 01:00
28# kuing 诗乃与生俱来,学不来啊
yes94 30# 2013-4-27 12:43
29# 李斌斌755 kk不是叫你作诗!
李斌斌755 31# 2013-4-27 13:10
30# yes94 我的意思是灌水的水平与生俱来
thread-7-1-1.html: 测试
yuzi 1# 2011-9-25 22:45
本帖最后由 yuzi 于 2011-9-25 22:52 编辑 已知$a^{2}+b^{2}=3-ab,a>0,b>0$,求$a^{2}+b^{2}$的取值范围。
kuing 2# 2011-9-25 22:53
已知$a^{2}+b^{2}=3-ab,a>0,b>0$,求$a^{2}+b^{2}$的取值范围。 yuzi 发表于 2011-9-25 22:45 由 $a>0,b>0$ 得 $ab>0$ 得 \[a^2+b^2=3-ab<3,\] 当 $a\to0,b\to\sqrt3$ 时 $a^2+b^2\to3$,所以 $a^2+b^2$ 上确界为 3; 另一方面,由均值不等式得 \[a^2+b^2=3-ab \ge 3-\frac{a^2+b^2}{2},\] 得到 $a^2+b^2\ge2$,当 $a=b=1$ 时取等号,所以 $a^2+b^2$ 最小值为 2。
yuzi 3# 2011-9-25 23:31
本帖最后由 yuzi 于 2011-9-26 15:08 编辑 1# yuzi 令$a=r\cos\alpha ,b=r\sin\alpha ,\alpha \in (0,\frac{\pi }{2})$,则 $r^{2}=3-\frac{1}{2}r^{2}\sin2\alpha $ $\Rightarrow r^{2}=\dfrac{6}{2+\sin2\alpha }\in [2,3)$
isea 4# 2011-9-25 23:33
本帖最后由 isea 于 2011-9-25 23:53 编辑 g fg  dg $a^2+b^2=3-ab$ 复制代码 $a^2+b^2=3-ab$ \[a^2+b^2=3-ab\] 复制代码 \[a^2+b^2=3-ab\] $e^\beta=\sin\beta$ $a_b_c$ $a_{b_c}$ ${a_b}_c$
kuing 5# 2011-9-25 23:35
还有,如果嫌分式太小(实际上用美元符号括起来的表示行内(inline)公式,有时是会变小的),那么将 \frac 改成 \dfrac 就会变大。改后效果:$\Rightarrow r^{2}=\dfrac{6}{2+\sin2\alpha }\in [2,3)$
kuing 6# 2011-9-25 23:41
公式两边的 \$ 的前面不必加 \
kuing 7# 2011-9-25 23:42
引用也可以看代码么。。。
kuing 8# 2011-9-25 23:46
行内公式:$\$ $公式代码 $\$ $(分式会变小之类的那种) 行间公式:$\backslash$[公式代码$\backslash$](独占一行且居中的那种,分式等不会变小)
isea 9# 2011-9-25 23:49
哈哈 闪了
zwl1972 10# 2011-10-12 16:12
本帖最后由 zwl1972 于 2011-10-12 16:19 编辑 有用的三元不等式     $ x^{2}+y^{2}+z^{2}\geq xy+yz+zx $    $ a^{2}+b^{2}+c^{2}+d^{2}+e^{2}\ge a(b+c+d+e)$       $(ab+bc+ca)^{2}\ge 3abc(a+b+c)$       $ a^{2}b^{2}+b^{2}c^{2}+c^{2}a^{2}\ge abc(a+b+c)$      $ a^{4}+b^{4}+c^{4}\geq abc(a+b+c)$      $ 2(a^{3}+b^{3}+c^{3})\geq ab(a+b)+bc(b+c)+ca(c+a)$ $ a^{3}b+b^{3}c+c^{3}a\geq abc(a+b+c)$    $ (a+b+c)^{2}\ge 3(ab+bc+ca)$ $ x^{4}+y^{4}+z^{4}\geq (xy)^{2}+(yz)^{2}+(zx)^{2}\geq x^{2}yz+xy^{2}z+xyz^{2}= xyz(x+y+z) $
kuing 11# 2011-10-13 09:10
(1)设 $h(x)=g(x)-f(x)=-8x^2-12x+k+3$,所以问题就转化为对于 $x\in [-3,3]$,都有 $h(x)\ge 0$ 的恒成立问题。则必须 $h(x)_{\min} \ge 0$。因为 $h(x)$ 是二次函数,且开口向下,故最小值必在端点取得,从而容易判断出 $h(x)_{\min}=h(3)=-72-36+k+3\ge 0$,故 $k\ge 105$; (2)根据题意存在 $x\in [-3,3]$,使 $f(x)\le g(x)$ 成立,即只需 $h(x)=g(x)-f(x)=-8x^2-12x+k+3\ge 0$ 在 $x\in [-3,3]$ 内有解,则必须 $h(x)_{\max} \ge 0$,因为 $h(x)$ 是二次函数,且开口向下,对称轴为 $x=-\dfrac34$,从而 $h(x)_{\max} =h\left(-\dfrac34\right)=\dfrac{15}2+k\ge0$,故 $k\ge-\dfrac{15}2$;
kuing 12# 2011-10-13 14:42
$$ \begin{aligned} &\quad 1+1=2,\\ &\quad 2+2=4,\\ &\boxed{\begin{aligned} &3+3=6\\&4+4=8\end{aligned}} \end{aligned} $$
kuing 13# 2011-10-13 14:55
$e=mc^2\tag1$
kuing 14# 2011-10-15 00:42
波浪$~\sim~$号 波浪$~a\sim b~$号 波浪 $a\sim b$ 号 波浪$a\sim b$号
kuing 15# 2011-10-24 13:33
\cases{ax+by+c=0\\c^2=a^2+b^2}     $\cases{ax+by+c=0\\c^2=a^2+b^2}$ $\left\{\begin{aligned}ax+by+c&=0\\c^2&=a^2+b^2\end{aligned}\right.$ 图图图图$\begin{cases}ax+by+c=0\\c^2=a^2+b^2\end{cases}$图图图图$\left\{\begin{aligned}&ax+by+c=0\\&c^2=a^2+b^2\end{aligned}\right.$图图图图$\left\{\begin{aligned}ax+by+c&=0\\c^2&=a^2+b^2\end{aligned}\right.$图图图图$\left\{\begin{aligned}ax+by+c=0\\ c^2=a^2+b^2\end{aligned}\right.$图图 kuing  13:58:10 $\cases{ax+by+c=0\\c^2=a^2+b^2}$ $f(x)=\cases{x+1&x>0\\x-1&x\le0}$
图图 16# 2011-10-24 15:49
15# kuing 嘿嘿
thread-70-1-9.html: [不等式] 晨早流流证个RJ简单不等式$ab+bc+ca=3$
kuing 1# 2011-10-8 10:03
已知 $a,b,c>0,ab+bc+ca=3$,求证\[ \frac1{1+a^2(b+c)}+\frac1{1+b^2(c+a)}+\frac1{1+c^2(a+b)}\leqslant\frac1{abc} \] 问题来自 http://bbs.pep.com.cn/thread-1899855-1-1.html 注意到\[ \frac1{1+a^2(b+c)}=\frac1{1+a(ab+ca)}=\frac1{1+a(3-bc)}=\frac1{1+3a-abc} \]又显然\[ abc=\sqrt{ab\cdot bc\cdot ca}\leqslant\sqrt{\left(\frac{ab+bc+ca}{3}\right)^3}=1 \]所以\[ \frac1{1+a^2(b+c)}=\frac1{1+3a-abc}\leqslant\frac1{3a} \]另外两项同理,故\[ \frac1{1+a^2(b+c)}+\frac1{1+b^2(c+a)}+\frac1{1+c^2(a+b)}\leqslant\frac1{3a}+\frac1{3b}+\frac1{3c} =\frac{ab+bc+ca}{3abc}=\frac1{abc} \]
pxchg1200 2# 2011-10-8 16:02
本帖最后由 pxchg1200 于 2011-10-8 16:04 编辑 那这个呢? For positive real numbers$a,b$  and $c$ we have$a+b+c=3 $ . Prove that: \begin{align} \frac{a}{1+(b+c)^{2}}+\frac{b}{1+(a+c)^{2}}+\frac{c}{1+(a+b)^{2}}\le\frac{3(a^{2}+b^{2}+c^{2})}{a^{2}+b^{2}+c^{2}+12abc} \end{align}
kuing 3# 2011-10-8 16:34
完全改头换面了……
pxchg1200 4# 2011-10-8 20:09
Cauchy-Schwarz kills it!
pxchg1200 5# 2011-10-19 09:39
那这个呢? For positive real numbers$a,b$  and $c$ we have$a+b+c=3 $ . Prove that: \begin{align} \frac{a}{1+(b+c)^{2}}+\frac{b}{1+(a+c)^{2}}+\frac{c}{1+(a+b)^{2}}\le\frac{3(a^{2}+b^{2}+c^{2})}{a^{ ... pxchg1200 发表于 2011-10-8 16:02 (Vo Quoc Ba Can): Write the inequality as \[{\left[ a-\frac{a}{1+(b+c)^2}\right] +\left[b-\frac{b}{1+(c+a)^2}\right]+\left[ c-\frac{c}{1+(a+b)^2}\right] \ge 3-\frac{3(a^2+b^2+c^2)}{a^2+b^2+c^2+12abc},}\] \[\frac{a(b+c)^2}{1+(b+c)^2}+\frac{b(c+a)^2}{1+(c+a)^2}+\frac{c(a+b)^2}{1+(a+b)^2} \ge \frac{36abc}{a^2+b^2+c^2+12abc}.\] Using the Cauchy-Schwarz inequality, we get \[\left[ \sum \frac{a(b+c)^2}{1+(b+c)^2}\right] \left\{ \sum \frac{a\left[1+(b+c)^2\right]}{(b+c)^2}\right\} \ge \left(\sum a\right)^2 =9.\] Therefore, it suffices to prove that \[\frac{9}{\frac{a\left[1+(b+c)^2\right]}{(b+c)^2}+\frac{b\left[1+(c+a)^2\right]}{(c+a)^2}+\frac{c\left[1+(a+b)^2\right]}{(a+b)^2}} \ge \frac{36abc}{a^2+b^2+c^2+12abc},\] or \[\frac{a^2+b^2+c^2+12abc}{abc} \ge 4\left\{\frac{a\left[1+(b+c)^2\right]}{(b+c)^2}+\frac{b\left[1+(c+a)^2\right]}{(c+a)^2}+\frac{c\left[1+(a+b)^2\right]}{(a+b)^2}\right\}.\] Since \[\begin{aligned} \frac{a\left[1+(b+c)^2\right]}{(b+c)^2}+\frac{b\left[1+(c+a)^2\right]}{(c+a)^2}+\frac{c\left[1+(a+b)^2\right]}{(a+b)^2}&=\left[ \frac{a}{(b+c)^2}+\frac{b}{(c+a)^2}+\frac{c}{(a+b)^2}\right]+(a+b+c) \\ &= \frac{a}{(b+c)^2}+\frac{b}{(c+a)^2}+\frac{c}{(a+b)^2}+3, \end{aligned} \] the above inequality can be written as \[\frac{a^2+b^2+c^2}{abc}  \ge 4\left[  \frac{a}{(b+c)^2}+\frac{b}{(c+a)^2}+\frac{c}{(a+b)^2}\right] ,\] which is true because \[4\left[  \frac{a}{(b+c)^2}+\frac{b}{(c+a)^2}+\frac{c}{(a+b)^2}\right] \le 4\left(\frac{a}{4bc}+\frac{b}{4ca}+\frac{c}{4ab}\right) =\frac{a^2+b^2+c^2}{abc}.\] The proof is completed. $\blacksquare$
kuing 6# 2011-10-19 11:08
5# pxchg1200 矮油……关键还是那个柯西了……
thread-700-1-7.html: [不等式] 又是网友问不等式
kuing 1# 2012-8-10 22:56
月中影 08-10 22:18:09 设x,y,z是正数,a^(1/2)=x(y-z)^2,b^(1/2)=y(z-x)^2,c^(1/2)=z(x-y)^2; 求证:a^2+b^2+c^2>=2(ab+bc+ca) 用 $\LaTeX$ 打一下,就是 设 $x$, $y$, $z$ 是正数,$\sqrt a=x(y-z)^2$, $\sqrt b=y(z-x)^2$, $\sqrt c=z(x-y)^2$。求证 \[ a^2+b^2+c^2 \geqslant 2(ab+bc+ca). \]
kuing 2# 2012-8-10 23:10
oh,突然想起以前做过……http://www.artofproblemsolving.c ... p?f=52&t=367372 条件那个根号原来是个提示…… 没事了……
yizhong 3# 2012-8-11 15:51
这个题目貌似是有几何背景的,很大程度上是从三角形中的不等式演变而来,因为从待证明的式子可以看出 从三角形中的那个面积公式:秦九韶-海伦公式可以看出,只是这里次方降为2次
kuing 4# 2012-8-11 15:59
3# yizhong 我那个贴里面的分解就是因为想起了“秦九韶-海伦公式”
yizhong 5# 2012-8-11 16:16
说起这个秦九韶-海伦公式,顺道再PS一下,令我想起了一道骨灰级的88年的CMO中的 一道题目,貌似是第四题,题目中的第一小问就是:设三个正实数a,b,c满足: $(a^2+b^2+c^2)^2>2(a^4+b^4+c^4)$证明:a,b,c一定是某个三角形的三条边长。
thread-701-1-2.html: 无聊看了下人教论坛的发贴量统计
kuing 1# 2012-8-11 00:03
如附件所示
kuing 2# 2012-8-11 00:14
由“每月新增帖子记录”看来: 人教论坛大概是06年6月换成discuz!的,我那时刚混起。 哎,07年比较鼎盛啊,那时我刚当版主。 然后就有点降了,到09年突然又增了?可能是广告刚爆发期。 大概由那个时候开始就是各种“NB”管理手段和设置出来了,人气不断下降的趋势。 不过也仍有几个月出现突然暴高,无法解释。 2012彻底冷清了,对比下前面,哎,嘘唏…… 再看“每日新增帖子记录”,是一个月内的,有点小奇怪,中间那段时间是论坛关闭“升级”的,但每天至少有一贴新增,07-21更是增了15个贴,不知贴子发在哪了?
kuing 3# 2012-8-11 00:26
其实本来这贴想发在人教的,不过想想还是算了,那边太“niubiligy”,没什么发贴欲了。
thread-702-1-1.html: 怎么制作这种底纹
kuing 1# 2012-8-11 16:01

abababa 2# 2012-12-31 22:09
啊,这我在photoshop里见过,有好多滤镜点一下就出这效果
kuing 3# 2012-12-31 23:17
没找到这种效果
abababa 4# 2012-12-31 23:51
我以前在photoshop里见过,就在滤镜那里,可能是装了其它的滤镜吧,不是自带的 刚问了下网友,说是gimp这个软件里自带,在滤镜->艺术->布状效果里,刚试了下,可能是? 发个图看下
thread-703-1-7.html: [不等式] 2012北方数学奥林匹克不等题,再看看今年的女奥那个不等式题,有什么发现木有,嘿嘿
yizhong 1# 2012-8-11 16:28
设n是正整数,证明:$(1+\dfrac{1}{3})(1+\dfrac{1}{3^2})...(1+\dfrac{1}{3^n})<2$
海盗船长 2# 2012-8-11 16:37
水啊。。
海盗船长 3# 2012-8-11 16:38
貌似某道高考题改编的
海盗船长 4# 2012-8-11 16:40
贴个答案 http://lukang.me/2012-china-north-math-olympiad.html
kuing 5# 2012-8-11 16:47
跟 06 江西压轴类似,也可以加强右边,数归
kuing 6# 2012-8-11 16:52
听说可加强 $<\sqrt e$
海盗船长 7# 2012-8-11 17:09
6# kuing 这个取对数再放缩成等比就行了吧
kuing 8# 2012-8-11 17:13
7# 海盗船长
thread-704-1-6.html: [数论] 2012女子最后一题
海盗船长 1# 2012-8-11 18:07
组合数${{2012}\choose{k}}$ $(k=0,1,2,\cdots,2012)$中有多少个可以被$2012$整除。
海盗船长 2# 2012-8-11 22:13
第8题好无聊,就为了考这样一个知识点吗: 对素数p, C_n^k中p的个数等于k+(n-k)在p进制加法下的进位次数。 由于2012=4乘503 先考虑503,2012在503进制下是40,C_2012^k不被503整除当且经当k+(2012-k)不进位, 即 k是503倍数, k=0,503,1006,1509,2012 再考虑2,2012的二进制表示为11111011100, k+(2012-k)不进位, 则k形如xxxxx0xxx00,其中x可以是0或1,有2^8个 恰进一次位, 则形如xxxxx0xx010, 有2^7个,或者xxxx01xxx00,也有2^7个 前面5个数中0和2012重复,故总共不满足要求的k有2^8+2^7+2^7+3=515个, 满足要求的k有2013-515=1498个 http://tieba.baidu.com/p/1783637644?pn=2
kuing 3# 2012-8-11 22:18
完全看不懂
yizhong 4# 2012-9-24 16:08
oh…………kummer定理立刻秒之~~~
thread-705-1-7.html: [不等式] Nice inequality.
pxchg1200 1# 2012-8-11 20:27
For $a,b,c>0$ prove that: \[ \frac{a}{b+c^2}+\frac{b}{c+a^2}+\frac{c}{a+b^2}\geq \frac{9}{a+b+c+3} \]
kuing 2# 2012-8-11 20:28
1# pxchg1200 蔡证过好像
kuing 3# 2012-8-11 20:34
a+b+c=1 时是一道经典题 如果没记错,蔡当时将他那证法发在了不等式小组的论坛上,现在上不了了,没法找出来。 只怪我的存档project太晚开始
pxchg1200 4# 2012-8-11 20:35
本帖最后由 pxchg1200 于 2012-8-11 20:38 编辑 这题很难的。 当然,用所谓的CYH技术就好多了。
kuing 5# 2012-8-11 20:47
这题很难的。 当然,用所谓的CYH技术就好多了。 pxchg1200 发表于 2012-8-11 20:35 什么叫“CYH技术”?
pxchg1200 6# 2012-8-11 20:49
5# kuing 就是带那种特殊式子的Cauchy-Schwarz。。。。
kuing 7# 2012-8-11 20:53
啊哈哈哈!还好我连贴子的标题都记得,利用google的网页快照,把那贴子找出来了!如附件所示。 看了下贴里的证法好像就是你说的那技术?
kuing 8# 2012-8-11 20:55
也有点小佩服我自己的记忆力和搜索能力了
pxchg1200 9# 2012-8-11 21:01
8# kuing 看不到。。。
kuing 10# 2012-8-11 21:03
9# pxchg1200 把rar下载下来解压出来不是一个mht文件吗?用浏览器打开就可以看了吧。 PS、如果看到了的话,注意一下那个过程中的(*)式有输入错误,但后面的应该是正确的。
kuing 11# 2012-8-11 21:19
可能要用 IE 打开,因为我刚才是用 IE 保存的。
pxchg1200 12# 2012-8-11 21:28
proof: by Cauchy-Schwarz inequality, \[ \left(\sum{\frac{a}{b+c^2}}\right)\left[\sum{a(b+c^2)(2a+2b+c)^2}\right]\geq \left(2\sum{a^2}+3\sum{ab}\right)^{2} \] So,it's suffice to prove \[ \left(2\sum{a^2}+3\sum{ab}\right)^{2}\left(3+\sum{a}\right)\geq 9\sum{a(b+c^2)(2a+2b+c)^{2}} \] Or \[ \left(2\sum{a^2}+3\sum{ab}\right)^2\geq 3\sum{ab(2a+2b+c)^{2}} \] and \[ \left(2\sum{a^2}+3\sum{ab}\right)^2\left(\sum{a}\right)\geq 9\sum{c^2a(2a+2b+c)^2} \] For the first inequality,it's \[ 4\sum{a^4}+3\sum{a^2bc}\geq 7\sum{a^2b^2}\] Which is obviously from the Schur in fourth degree. For the second inequality.it's \[ 4\sum{a^5}+7\sum{a^4b}+16\sum{a^4c}+18\sum{a^3bc}\geq 7\sum{a^3(b^2+c^2)}+31\sum{a^2b^2c}\] Now,Using Schur in fifth degree,we have \[ 4\sum{a^5}+4\sum{a^3bc}\geq 4\sum{a^4(b+c)}\geq 4\sum{a^3(b^2+c^2)}\] also \[ 3\sum{a^4(b+c)}\geq 3\sum{a^3(b^2+c^2)} \] \[ 4\sum{a^4b}+4\sum{a^4c}+14\sum{a^3bc}\geq 22\sum{a^2b^2c}\] Thus,just check \[ \sum{a^4c}\geq \sum{a^2b^2c}\] Or \[ \frac{a^3}{b}+\frac{b^3}{c}+\frac{c^3}{a}\geq ab+bc+ca \] Which is obviously by AM-GM inequality. Hence we are done!
pxchg1200 13# 2012-8-11 21:30
10# kuing 依旧无图。。
kuing 14# 2012-8-11 21:32
12# pxchg1200 前面出奇的一样,后面有点不同,不过那不是关键
kuing 15# 2012-8-11 21:35
13# pxchg1200 用 IE 打开,可能还要等待一小会(海盗刚才说的)
kuing 16# 2012-8-11 21:37
不过你在群里也看到了,没所谓了
yizhong 17# 2012-8-14 15:46
本帖最后由 yizhong 于 2012-8-14 15:50 编辑 貌似这样也可以搞出来,$(\sum \dfrac{a}{b+c^2})(\sum a(b+c^2)(2a+2b+c)^2).......$ 后面的证明貌似p,q,r也可以。。。。。
kuing 18# 2012-8-14 16:06
17# yizhong 跟前面一样?
yizhong 19# 2012-8-14 16:13
在回复之前没有看前面的帖子,这个题目在那份越南人的小册子有
╰☆ヾo.海x 20# 2012-8-14 20:40
12# pxchg1200 px....你在国外不?哇,我要向你学习英文答题的感觉啊
thread-705-2-7.html:
kuing 21# 2012-8-14 20:43
20# ╰☆ヾo.海x 在mathlinks的不等式区混一小段日子大概就能用英文写出这种过程……其实来来去去就那若干句常用的……
╰☆ヾo.海x 22# 2012-8-14 20:56
21# kuing mathlink吗?你发个网址。。额,要去看看了。。。
kuing 23# 2012-8-14 21:07
22# ╰☆ヾo.海x 给你发过的,查聊天记录。
thread-706-1-7.html: [不等式] How to holder it?
pxchg1200 1# 2012-8-11 20:42
Let $ a,b,c$ be positive real numbers. Prove that: \[ \left(\dfrac{a}{13a+17b}\right)^{\frac{2}{5}}+\left(\dfrac{b}{13b+17c}\right)^{\frac{2}{5}}+\left(\dfrac{c}{13c+17a}\right)^{\frac{2}{5}}\le\dfrac{3}{30^{\frac{2}{5}}} \]
kuing 2# 2012-8-11 22:06
五次……不知该配啥好了
thread-707-1-7.html: [不等式] Pham kim hung's old result
pxchg1200 1# 2012-8-11 21:43
For $a,b,c,d>0$ with $ (a+b+c+d)(\frac{1}{a}+\frac{1}{b}+\frac{1}{c}+\frac{1}{d})=20 $,prove that \[ (a^2+b^2+c^2+d^2)\left(\frac{1}{a^2}+\frac{1}{b^2}+\frac{1}{c^2}+\frac{1}{d^2}\right)\geq 36 \]
kuing 2# 2012-8-11 22:04
等一中……
pxchg1200 3# 2012-8-12 11:31
What if \[ \left[\sum{(b+c+d-a)^2}\right]\left(\sum{\frac{1}{a^2}}\right)\geq \left(\sum{\frac{b+c+d-a}{a}}\right)^{2} \] ???
yizhong 4# 2012-8-14 15:31
本帖最后由 yizhong 于 2012-8-14 16:07 编辑 根据已知条件和代征式子展开式我们可以定义如下式子:$F(x,y,z)=\dfrac{x}{y}+\dfrac{y}{z}+\dfrac{z}{x},其中这里x,y,z为正数$ 根据这个定义,我们可以容易得到:$F(x,y,z)^2=F(x^2,y^2,z^2)+2F(z,y,x),F(z,y,x)^2=F(z^2,y^2,x^2)+2F(x,y,z)(A)$,所以根据定义式 我们可以将已知条件以及待证明的式子改写成为:$\sum (F(a,b,c)+F(c,b,a))=32(b),\sum (F(a^2,b^2,c^2)+F(c^2,b^2,a^2))\geqslant64(B)$ 利用(A)的结论,我们可以得到要证明(B),相当于证明:$\sum (F(a,b,c)^2+F(c,b,a)^2)\geqslant128$接下去结合条件(b)只需要利用下c-s即可解决。 写得明白些就是:$\sum (F(a,b,c)^2+F(c,b,a)^2)\geqslant\dfrac{1}{8}(\sum (F(a,b,c)+F(c,b,a)))^2=\dfrac{32^2}{8}=128$
kuing 5# 2012-8-14 16:08
4# yizhong 呃,咋四元变三元了……
yizhong 6# 2012-8-14 16:24
此题貌似是VISILE PHAM PHAN 这三个人的杰作,最早出现在06年,在05,06的时候VASILE也 造出了不少类似这样的BAT.
kuing 7# 2012-8-17 01:57
试下编号,引用一中的贴子先,顺便把代码改好一点。 根据已知条件和代征式子展开式我们可以定义如下式子 \[F(x,y,z)=\frac{x}{y}+\frac{y}{z}+\frac{z}{x},\] 其中这里 $x$, $y$, $z$ 为正数。 根据这个定义,我们可以容易得到 \begin{equation}\label{eqA} \begin{aligned} F(x,y,z)^2&=F(x^2,y^2,z^2)+2F(z,y,x),\\ F(z,y,x)^2&=F(z^2,y^2,x^2)+2F(x,y,z), \end{aligned} \end{equation} 所以根据定义式,我们可以将已知条件以及待证明的式子改写成为 \begin{align} \sum (F(a,b,c)+F(c,b,a))&=32,\label{eqb}\\ \sum (F(a^2,b^2,c^2)+F(c^2,b^2,a^2))&\geqslant64,\label{eqB} \end{align} 利用 \eqref{eqA} 的结论,我们可以得到要证明 \eqref{eqB},相当于证明 \[\sum (F(a,b,c)^2+F(c,b,a)^2)\geqslant128,\] 接下去结合条件 \eqref{eqb} 只需要利用下c-s即可解决。 写得明白些就是 \[\sum (F(a,b,c)^2+F(c,b,a)^2)\geqslant\frac18\left(\sum (F(a,b,c)+F(c,b,a))\right)^2=\frac{32^2}{8}=128.\] 效果 very nice!
thread-708-1-2.html: kk.推荐点电影看?
pxchg1200 1# 2012-8-12 21:16
最近没啥好看电影啊,kk推荐点。 (不要那些岛国动作片)...
kuing 2# 2012-8-12 21:33
1# pxchg1200 既然有括号里那个要求,那就不用叫我推荐啦
thread-709-1-2.html: 再次说明存档的重要
kuing 1# 2012-8-12 22:03
话说我之前没去存那里的……
戊概念·五 2# 2012-8-13 22:47
1# kuing 很怀疑:专划出一个盘存档够你用么。。。。。
kuing 3# 2012-8-13 22:52
2# 戊概念·五 够了,我也不是将整个论坛存下来,只是存部分,目前也就存了4G多的空间而已
戊概念·五 4# 2012-8-13 23:41
本帖最后由 戊概念·五 于 2012-8-14 23:54 编辑 3# kuing 看你这段,想起hm之前的一次回帖。。。。。 汗~露肚皮了呃 ******************************************************************************* 回楼下仁兄:刺猬蜷缩身体,似刺球状来护卫自己,露肚皮了那不就是把弱点什么的都暴露了么
kuing 5# 2012-8-14 00:16
4# 戊概念·五 嗯?啥意思
kuing 6# 2012-8-14 13:33
出奇地救回来了 http://www.aoshoo.com/bbs1/index.asp niubility
pxchg1200 7# 2012-8-16 07:57
6# kuing 最后一句亮了!
kuing 8# 2012-8-16 12:40
7# pxchg1200 可能这样会安全些
thread-71-1-5.html: [几何] 对两定圆的幂之比为定值的点的轨迹
kuing 1# 2011-10-8 12:59
设两圆 $O_1$,$O_2$ 的半径分别为 $R$,$r$,圆心距 $2d$,动点 $P$ 对圆 $O_1$,$O_2$ 的幂分别记为 $\rho_1(P)$,$\rho_2(P)$。 今求满足 $\rho_1(P)=k\cdot\rho_2(P)$ 的点的轨迹,其中 $k$ 为常数。 建立坐标系,使 $O_1(-d,0)$,$O_2(d,0)$,设 $P(x,y)$,则由条件得\[ PO_1^2-R^2=k(PO_2^2-r^2) \iff (x+d)^2+y^2-R^2=k\bigl((x-d)^2+y^2-r^2\bigr) \]展开整理为\[ (1-k) x^2+(1-k) y^2+d (2 k+2) x+d^2 (1-k)+k r^2-R^2=0\qquad(*) \]可见要么是圆、要么是直线、要么是个一点、要么没轨迹。取些特殊情况,得到推论: 若 $k=1$,方程为\[4dx+r^2-R^2=0\]为一垂直于两圆心连线的直线,这也就是根轴定理; 若 $R=r=0$ 且 $k>0,k\ne1$,则变成了阿波罗尼斯圆问题,所以这个问题可以看作是阿波罗尼斯圆的推广吧。此时轨迹化为\[x^2+y^2+\frac{2d(k+1)}{(1-k)}\cdot x+d^2=0\]必然是个圆。注意这里的 $k$ 为线段比的平方。 下面考虑一下当 $k\ne1$ 时式 $(*)$ 的相关性质。为此,我们化为标准方程。为\[ \left(x-d\cdot\frac{k+1}{k-1}\right)^2+y^2=\frac{4k}{(k-1)^2}d^2+\frac{k r^2-R^2}{k-1}\qquad(*)' \]故当 $\frac{4k}{(k-1)^2}d^2+\frac{k r^2-R^2}{k-1}>0$ 时式 $(*)'$ 为圆,设圆心为 $K$ 即有\[K\left(d\cdot\frac{k+1}{k-1},0\right)\]在过 $O_1$,$O_2$ 的直线上,并且与两定圆的半径大小无关。考虑该圆心 $K$ 到两定圆圆心的有向长度比,我们有\[ \frac{\overline{O_1K}}{\overline{KO_2}}=\frac{d\cdot\frac{k+1}{k-1}+d}{d-d\cdot\frac{k+1}{k-1}}=-k \]正好等于幂之比的相反数。再考虑半径,设式 $(*)'$ 的圆的半径为 $T$ 则有\[ T=\sqrt{\frac{4k}{(k-1)^2}d^2+\frac{k r^2-R^2}{k-1}} \] 对了,为什么发这贴?其实是因为在 K12 刚才看到这个贴 http://sq.k12.com.cn/discuz/thread-597627-1-1.html 里面的这个题: 该问题可以用上述结论解决。 如果这个定点 $R$ 存在,那么这个定点和圆 $O$ 相当于最开头所设的两定圆的其中一圆半径为 0,而圆 $M$ 则为那轨迹。 不妨令前面的 $O_1$ 为 $O$,$O_2$ 为定点 $R$,两者相距 $2d$,因圆 $M$ 半径为 3,故在 $T$ 的表达式中代入数据 $R=1,r=0$,应有\[ 3=\sqrt{\frac{4k}{(k-1)^2}d^2-\frac{1}{k-1}} \]因显然 $k>0$ 所以由前面有向长度比的式子知 $M$ 在 $OR$ 之外,所以 $R$ 在线段 $OM$ 上,于是有\[ \frac{\left|MO\right|}{\left|MR\right|}=k\iff \frac{\left|MO\right|}{\left|MO\right|-2d}=k\iff \frac{\sqrt5}{\sqrt5-d}=k \]由上面两个方程,解得两组解\[ \left\{ \begin{aligned} d&=\frac{\sqrt5}{10}\\ k&=\frac{10}9 \end{aligned} \right.   或   \left\{ \begin{aligned} d&=\frac{\sqrt5}2\\ k&=2 \end{aligned} \right. \]即\[ \left\{ \begin{aligned} \left|OR\right|&=\frac{\sqrt5}5\\ 定值为&=\frac{\sqrt{10}}3 \end{aligned} \right.   或   \left\{ \begin{aligned} \left|OR\right|&=\sqrt5\\ 定值为&=\sqrt2 \end{aligned} \right. \]由此也容易得到 $R\left(\frac25,\frac15\right)$ 或 $R(2,1)$。 注:切线长比的平方才是圆幂比,所以最后得到的 $k$ 要开方才是线长比。
kuing 2# 2011-10-8 13:43
基本OK鸟,休息一会先
realnumber 3# 2011-10-8 15:20
本帖最后由 realnumber 于 2011-10-8 15:22 编辑 忽然发现,kuing生活好简单,就做数学,自娱自乐~~~ 我就差多了,要么工作,带孩子,玩电子游戏,被孩子抢了电脑看动画,无聊才看数学问题或工作之余禁止游戏网页。
kuing 4# 2013-1-11 12:14
忽然发现,kuing生活好简单,就做数学,自娱自乐~~~ 我就差多了,要么工作,带孩子,玩电子游戏,被孩子抢了电脑看动画,无聊才看数学问题或工作之余禁止游戏网页。 realnumber 发表于 2011-10-8 15:20 其实也没那么简单,还有找片看片呢
第一章 5# 2013-1-21 21:20
今天也碰到这题了。看来这是哪个地方的模拟题,转大发了。 想搞纯几何的,不过没有好的思路。
kuing 6# 2013-1-21 22:16
年多前的东东,自己都不太记得了……
thread-710-1-2.html: 某网站奇葩验证码
叶剑飞Victor 1# 2012-8-13 01:18

叶剑飞Victor 2# 2012-8-13 01:19

叶剑飞Victor 3# 2012-8-13 01:20

kuing 4# 2012-8-13 02:08
第一个以前见过,其实是比较简单的东东了,又对数又反三角神马的粹纯吓人
thread-711-1-7.html: [组合] 某群里看到的一道题
kuing 1# 2012-8-14 01:38
若一个凸四边形被分割成100个边长为1的正三角形,求所有这样的四边形的周长
kuing 2# 2012-8-14 01:40
貌似意思就是100个相同的正三角形能拼成哪些凸四边形,貌似不太容易数的说,顶一下
nash 3# 2012-8-14 02:20
如果存在的话, 那内角只能是60°和120°了,也就只能是等腰梯形或者平行四边形啦, 每一个等边三角形可以构造出四个等边三角形, 如果是等腰梯形,可以分割成等边三角形个数为3*[4^(n-1)]个,貌似不行, 如果是菱形,可以分割成等边三角形个数为2*[4^(n-1)]个,貌似也不行, 那就只能是非菱形的平行四边形了, 可以构造出一种边长分别为1和50的平行四边形(其中一个内角为60°),满足题意, 如果一个边长为2,那么只需要构造25个边长为2的等边三角形,这样的好像构造不出来, 其他的就不知道了。
海盗船长 4# 2012-8-14 10:02
3# nash 等腰梯形的那个有点问题吧,不一定都要是3的倍数
海盗船长 5# 2012-8-14 10:13
而且不一定都是4的幂次吧,,只要完全平方数就可以
nash 6# 2012-8-14 11:22
那个等腰梯形和菱形,我都是从最简单的那种算的,就考虑一种情况 后面的那个平行四边形的,只要边长能被50整除的都可以,(1,50),(2.25),(5,10)
nash 7# 2012-8-14 12:20
5# 海盗船长 为啥只需要平方数就可以
海盗船长 8# 2012-8-14 12:37
边长是2的那个应该是等腰梯形
海盗船长 9# 2012-8-14 12:40
7# nash 我的意思是无论哪种四边形相应的三角形个数都是$k\cdot n^2\quad(k\ge 2)$的形式
海盗船长 10# 2012-8-14 12:44
9# 海盗船长 然后由于$100=2^2\cdot 5^2$就有 1.$n=1,k=100$ 2.$n=2,k=25$ 3.$n=5,k=4$
kuing 11# 2012-8-14 12:51
答案是“百度知道想赚点财富值啊哈哈哈”
海盗船长 12# 2012-8-14 12:59
9# 海盗船长 哦,,好像有问题,应该还有其他的情况
海盗船长 13# 2012-8-14 13:00
11# kuing 你不是kuing吧。。
kuing 14# 2012-8-14 13:13
13# 海盗船长 我不是kuing是谁……
海盗船长 15# 2012-8-14 13:27
12# 海盗船长 如果是等腰梯形应该满足$n^2-m^2$的形式(相当于从一个大的等边三角形中切去一个小的等边三角形) 如果是平行四边形应该满足$n^2-m^2-(n-m)^2$的形式(相当于从等腰梯形中再切去一个等边三角形)
海盗船长 16# 2012-8-14 13:28
14# kuing 哦,感觉不像你说话的语气,,
kuing 17# 2012-8-14 13:36
16# 海盗船长 呃,我是从群里直接复制上来的啊 我的语气是怎么样的
kuing 18# 2012-8-14 13:45
12# 海盗船长 如果是等腰梯形应该满足$n^2-m^2$的形式(相当于从一个大的等边三角形中切去一个小的等边三角形) 如果是平行四边形应该满足$n^2-m^2-(n-m)^2$的形式(相当于从等腰梯形中再切去一个等边三角形 ... 海盗船长 发表于 2012-8-14 13:27 切中要害了
海盗船长 19# 2012-8-14 13:51
17# kuing 感觉那个像海叉的语气啊
kuing 20# 2012-8-14 13:53
19# 海盗船长 我就是复制海叉的啊
thread-711-2-7.html:
海盗船长 21# 2012-8-14 13:56
20# kuing 好吧
kuing 22# 2012-8-14 13:57
21# 海盗船长 难道你没看JR群
海盗船长 23# 2012-8-14 13:57
没有啊,我都没上qq。。。
kuing 24# 2012-8-14 13:59
23# 海盗船长 没上QQ?我好像看到你退了某群
海盗船长 25# 2012-8-14 14:01
24# kuing 就刚刚早上上了下
kuing 26# 2012-8-14 14:02
25# 海盗船长 好吧……先不水了……
╰☆ヾo.海x 27# 2012-8-14 20:35
19# 海盗船长 。。你怎么一感觉就感觉出像我的语气呢?那句话这么具有海x特色啊。。
kuing 28# 2012-8-14 20:38
27# ╰☆ヾo.海x 他懂你
海盗船长 29# 2012-8-14 20:41
27# ╰☆ヾo.海x 只有你会在每句话后面加“啊哈哈哈”。。。
╰☆ヾo.海x 30# 2012-8-14 20:59
29# 海盗船长 。。哪有每句话啊,真是的...  还有酷儿,要点名批评你了。。
kuing 31# 2012-8-14 21:11
感觉都有了还不是懂啊
thread-712-1-7.html: [组合] 一道较弱的离散极值题
nash 1# 2012-8-14 01:54
8位歌手参加艺术节,准备为他们安排M次演出,每次由其中四位登台表演,要求8位歌手中任意两位同时演出的次数一样多,请设计一中方案,使演出次数M最小
kuing 2# 2012-8-14 01:55
没看出哪里弱……表示没秒出来……
nash 3# 2012-8-15 01:04
大牛们也不过来秒一下 不知道是不是下面这样的 共计表演4M人次,每人表演M/2次,对于任意一人甲,与其共同表演的人次为3M/2次,要使得甲与其余7人任意一位同时演出次数一样多,则3M/2能够被7整除,M的最小值为14
海盗船长 4# 2012-8-15 12:19
http://www.wl84.com/forum.php?mod=viewthread&tid=39
kuing 5# 2012-8-15 12:24
4# 海盗船长 啊哈哈哈,路箩筐的网站(准确说是论坛才对,网站好像应该是另外一个)!niubility
nash 6# 2012-8-15 13:10
好简练
thread-713-1-6.html: 这道好像是高考题
kuing 1# 2012-8-14 22:14
晚上在群里有人发这道题: 印象中是高考题,而且是当时我想选上我那贴子里玩结果玩不来的然后就过了,刚才重新想了下,还是不会……
kuing 2# 2012-8-14 22:16
话说上图这排版真差,好难看,有空找找原版图片。 为了看得更清楚,我还是把题目重新打打。 设 $N=2^n$($n\in\mathbb N^+$,$n\geqslant2$),将 $N$ 个数 $x_1, x_2, \ldots, x_n$ 依次放入编号为 $1, 2, \ldots, N$ 的 $N$ 个位置,得到排列 $P_0=x_1x_2\cdots x_n$。将该排列中分别位于奇数与偶数位置的数取出,并按原顺序依次放入对应的前 $\frac N2$ 和后 $\frac N2$ 个位置,得到排列 $P_1=x_1x_3\cdots x_{N-1}x_2x_4\cdots x_N$, 将此操作称为 $C$ 变换。将 $P_1$ 分成两段,每段 $\frac N2$ 个数,并对每段作 $C$ 变换,得到 $P_2$;当 $2\leqslant i\leqslant n-2$ 时,将 $P_i$ 分成 $2^i$ 段,每段 $\frac N{2^i}$ 个数,并对每对作 $C$ 变换,得到 $P_{i+1}$。例如,当 $N=8$ 时,$P_2=x_1x_5x_3x_7x_2x_6x_4x_8$,此时 $x_7$ 位于 $P_2$ 中的第 4 个位置。 (1)当 $N=16$ 时,$x_7$ 位于 $P_2$ 中的第________个位置; (2)当 $N=2^n$($n\geqslant8$)时,$x_{173}$ 位于 $P_4$ 中的第________个位置。
海盗船长 3# 2012-8-14 23:31
这个用二进制应该可以搞定
海盗船长 4# 2012-8-14 23:50
本帖最后由 海盗船长 于 2012-8-14 23:53 编辑 3# 海盗船长 比如判断$7$的位置 把$7$写为二进制:$(111)_{2}$,然后不断将个位抹掉,分组结果就是:第一次左$3$ 第二次右$2$ 。
海盗船长 5# 2012-8-14 23:52
本帖最后由 海盗船长 于 2012-8-14 23:56 编辑 4# 海盗船长 抹去的哪一位$0$或$1$交替代表分组左还是右,剩下的数代表分组后的排位。
kuing 6# 2012-8-15 01:43
看不懂,明天有心思再理解一下……
kuing 7# 2012-8-15 01:58
皮蛋说: 皮蛋(2770*****)  0:45:52 那个P1,P2啥的 分割一次P1 就是把被2除余1的数都搞到左边一堆(1,3,5,……),右边是是偶数(2,4,6,……) 再分割一次P2, 左1为4m-3(1,5,9,……),左2为4m-1(3,7,11,……) P3,P4类似 173被4除余1,被8除余5,16除余3 貌似在P4的左3第11个数 皮蛋(2770*****)  0:48:00 是左4的第11个数 在纸上画的,不知道扔哪里去了
joatbmon 8# 2012-9-27 15:19
我也是用二进制来理解的,还好以前学过点编程
thread-716-1-2.html: 其实我也是特困生
kuing 1# 2012-8-15 14:21
RT,真的很困,很困……
pxchg1200 2# 2012-8-16 07:56
1# kuing 你们夜猫子白天和黑夜都颠倒了吧。
thread-717-1-6.html: [函数] 含有限制条件的绝对值函数最小值
nash 1# 2012-8-15 20:21
本帖最后由 nash 于 2012-8-15 20:32 编辑 已知实数x,y满足$|x|+|y| \leqslant 1$,求f(x,y)=|2x-3y+1.5|+|y-1|+|2y-x-3|的值域
海盗船长 2# 2012-8-15 20:32
类似 http://bbs.pep.com.cn/thread-728057-1-1.html
海盗船长 3# 2012-8-15 20:35
代入端点就OK了
kuing 4# 2012-8-15 20:36
最近没思考力,画图猜答案来了
nash 5# 2012-8-15 20:37
2# 海盗船长 那个帖子以前看过,这种有限制条件的貌似比那个要麻烦的多
海盗船长 6# 2012-8-15 20:44
5# nash 一样可以代入端点吧,不过多了$|x|+|y|\le 1$形成的四条线段也要算进去
nash 7# 2012-8-15 20:51
6# 海盗船长 真心不喜欢暴力代入,有没有其他的方法? 可能这种题就是考察这个的吧 其实后面两个绝对值可以直接去的(估计是凑好的),只需要讨论前面一个绝对值,然后就是线性规划问题了
kuing 8# 2012-8-15 22:00
7# nash 对喔,后面多余,咳,那就没那么好玩了。改系数吧
hongxian 9# 2012-8-16 20:44
用海盗法算了一下,结果好象是$[2.2,11.5]$,在$(0.3,0.7)$处取2.2,在(0,-1)处取11.5,不知对不对。
yizhong 10# 2012-8-23 22:53
我来弄下最大值吧,$|2x-3y+1.5|\leqslant2|x|+3|y|+1.5(1)  |y-1|\leqslant|y|+1 (2) |2y-x-3|\leqslant2|y|+|x|+3(3)$  再将(1) (2) (3)叠加得到:$|2x-3y+1.5|+|y-1|+|2y-x-3|\leqslant\dfrac{11}{2}+3|x|+6|y| \leqslant\dfrac{11}{2}+6|x|+6|y|\leqslant\dfrac{23}{2}$
yes94 11# 2012-8-23 23:27
10# yizhong :victory:
thread-718-1-7.html: [不等式] 网友又问不等式,这次终于分了(秒不成,分也好)
kuing 1# 2012-8-15 22:22
月中影 08-15 21:41:56 x,y,z>=0,∑xy=1,求证:∑x(1-y^2)(1-z^2)<=(4*3^(1/2))/9   用 $\LaTeX$ 打一下,就是 $x$, $y$, $z\geqslant0$, $\sum xy=1$,求证 \[\sum x(1-y^2)(1-z^2)\leqslant \frac{4\sqrt3}9.\] 证明 \begin{align*} \sum x(1-y^2)(1-z^2) &= \sum(x-xy^2-xz^2+xy^2z^2) \\ &= \sum\bigl(x(xy+yz+zx)-xy^2-xz^2\bigr)+xyz\sum xy \\ &= \sum \bigl(xy(x-y)-zx(z-x)+xyz\bigr)+xyz \\ &= \sum xy(x-y) - \sum zx(z-x) + \sum xyz+xyz \\ &= 4xyz, \end{align*} 下略
海盗船长 2# 2012-8-15 23:52
1# kuing 看看怎么分
kuing 3# 2012-8-16 00:11
2# 海盗船长 其实没什么技术,只是骗一点回复
yizhong 4# 2012-8-16 15:23
oh,好老的题目了,是1994香港IMO代表队的选拔考试题目
kuing 5# 2012-8-16 15:29
4# yizhong 你这个出处dang
yizhong 6# 2012-8-16 16:47
本帖最后由 yizhong 于 2012-8-16 16:51 编辑 对于这个题目我给出两个证法:证法(1),其实这个和小K的差不多,我们可以将待证明的不等式改写成为: $x+y+z+xyz(xy+xz+yz)\leqslant\dfrac{4\sqrt{3}}{9}+xy(x+y)+xz(x+z)+yz(y+z)(1)$再利用已知条件 我们又可以将(1)写成:$x+y+z+4xyz\leqslant\dfrac{4\sqrt{3}}{9}+x+y+z$以下从略。 证法(2):齐次化,我们可以将待证明的不等式改写成为:$x(T_2-y^2)(T_2-z^2)+y(T_2-z^2)(T_2-x^2)+z(T_2-x^2)(T_2-y^2) \leqslant\dfrac{4\sqrt{3}}{9}T_2^\dfrac{5}{2}(1)$其中我们这里设:$T_1=x+y+z,  T_2=xy+xz+yz   T_3=xyz$ 接下来我们将(1)展开化简我们得到要证明(1)相当于证明:$4T_2T_3\leqslant\dfrac{4\sqrt{3}}{9}T_2^\dfrac{5}{2}$ 以下从略。 注记:由$xy+xz+yz=1$我们还可以联想到切代换,由于过程比较复杂些,在这里就不再给出。
hongxian 7# 2012-8-16 17:03
让我看一下.
thread-719-1-1.html: 极限。。
pxchg1200 1# 2012-8-16 08:00
\[ \lim_{n\rightarrow\infty}{(1-\frac{1}{1\cdot 2})(1-\frac{1}{2\cdot 3})\cdots(1-\frac{1}{n(n+1)})} \]
海盗船长 2# 2012-8-16 11:07
http://tieba.baidu.com/p/1249815303
海盗船长 3# 2012-8-16 11:11

pxchg1200 4# 2012-8-17 19:42
3# 海盗船长 这么犀利,我用Gamma函数和维尔斯特拉斯公式做的。
海盗船长 5# 2012-8-20 14:00
4# pxchg1200 额,我那个是西西的解答,不过通法应该是用$\Gamma$函数
thread-72-1-1.html: 我的头像、贴里的图片附件是否正常显示?
kuing 1# 2011-10-8 17:11
RT 好奇怪,自从昨天出问题到今天修复好了之后,我用 chrome 浏览器看不到我自己的头像,还有某些图片附件不显示,但用 IE 又能正常显示。。。 不知大家如何
thread-720-1-1.html: [转]简单百年历bat
kuing 1# 2012-8-16 20:36
@echo off color 3F mode con cols=40 lines=20 setlocal enabledelayedexpansion set str=日一二三四五六 set sdate=%date% :Main cls&echo. :: 日期提取、格式化与校验 for /f "tokens=1,2,3 delims=-/: " %%i in ("%sdate%") do (     (set sy=%%i) && (set sm=%%j) && (set sd=%%k) ) if not defined sd set sd=1 if not defined sm set sm=%sy%& set sy=%date:~0,4% (set sy=0000%sy%) && (set sm=00%sm%) && (set sd=00%sd%) (set sy=%sy:~-4%) && (set sm=%sm:~-2%) && (set sd=%sd:~-2%) cd. set /a y=1%sy%-10000, m=1%sm%-100, d=1%sd%-100 2>nul if errorlevel 1 goto Error if %y% lss 100 (     if %y% lss 50 (set /a y+=2000) else (set /a y+=1900)     set sy=!y! ) if %m% lss 13 if %d% lss 32 goto Calc :Error echo.错误的日期. pause>nul set sdate=%date% goto Main :Calc :: 计算每个月的天数 set days=31 for %%i in (4 6 9 11) do if %m% equ %%i set days=30 :: 计算2月份的偏差 set /a leap="^!(y%%4) & ^!(^!(y%%100)) | ^!(y%%400)" if %m% equ 2 set /a days=28+%leap% if %m% leq 2 (set /a y-=1& set /a m+=12) :: 计算指定日期的星期数 set /a w=(d+2*m+3*(m+1)/5+y+y/4-y/100+y/400+1)%%7 echo.  %sy%年%sm%月  查询日:%sy%-%sm%-%sd%,星期!str:~%w%,1! echo. :: 生成月历 set /a wb=(w+35-d) %% 7, we=wb+days+1, day=1 echo.    日   一   二   三   四   五   六 echo. ━━━━━━━━━━━━━━━━━━━ set /p= <nul for /l %%i in (0,1,37) do (     set "temp=  "     if %%i GTR %wb% if %%i LSS %we% (         set temp= !day!         set temp=!temp:~-2!         if !d! EQU !day! set temp=★         set /a day+=1     )     set /p=   !temp!<nul     set /a "wm=(%%i+1)%%7"     if !wm! equ 0 echo.&echo.&set /p= <nul ) echo. echo  ━━━━━━━━━━━━━━━━━━━ echo.  输入日期可查询当日星期并显示当月月历 echo. set sdate= set /p sdate=  格式如:07-02-03,[回车]退出: if defined sdate goto Main 复制代码 将代码保存为 bat 运行即可,可查询 1950-01-01 至 2049-12-31 的日期。
thread-721-1-7.html: 由人教的一个向量及三角形的贴想到的
kuing 1# 2012-8-16 21:23
刚才看了下这个贴 http://bbs.pep.com.cn/thread-2579156-1-1.html,首先可以肯定该贴里 5、6# 说的是正确的。 而这里想说的则是如果按 PA*PB+PA*PC+PB*PC=0 的话,这个 P 会是什么点? 稍想了下,不难想到其实 P 是不确定的,再细想,发现 P 的轨迹反而有趣。 \begin{align*} \sum\overrightarrow{PA}\cdot\overrightarrow{PB}=0&\iff\sum(\overrightarrow{PG}+\overrightarrow{GA})\cdot(\overrightarrow{PG}+\overrightarrow{GB})=0 \\ &\iff 3PG^2+2\overrightarrow{PG}\sum\overrightarrow{GA}+\sum\overrightarrow{GA}\cdot\overrightarrow{GB}=0 \\ &\iff PG^2=-\frac13\sum\overrightarrow{GA}\cdot\overrightarrow{GB}, \end{align*} 其中 $G$ 是 $\triangle ABC$ 的重心,可以证明 $\sum\overrightarrow{GA}\cdot\overrightarrow{GB}<0$,所以右边是正常数,所以 $P$ 的轨迹是个圆且圆心为 $G$。
hongxian 2# 2012-8-16 22:03
强人,不过怎么证明:$\sum\overrightarrow{GA}\cdot\overrightarrow{GB}<0$ 看起来好象是的,但我还不会证呢,见笑了!
kuing 3# 2012-8-16 22:07
2# hongxian 呵呵,我懒了没写,其实也很简单几步 \[\sum\overrightarrow{GA}\cdot\overrightarrow{GB}=\frac12\left(\left( \sum\overrightarrow{GA} \right)^2-\sum GA^2 \right)=-\frac12\sum GA^2=-\frac16(a^2+b^2+c^2)\] 就好了
海盗船长 4# 2012-8-16 22:15
有意思的结论。赞一个
hongxian 5# 2012-8-16 22:16
3# kuing 看明白了,谢谢了!
kuing 6# 2012-8-16 22:23
其实也不限于平面,所以还可以说是球面,半径 $\sqrt{\frac{a^2+b^2+c^2}{18}}$……
kuing 7# 2012-8-17 02:03
用坐标法也很容易得到这些结论。
thread-722-1-7.html: [不等式] 几道不等式问题
ccnu_chb_ycb 1# 2012-8-16 22:53
请教下图片中四道不等式之间有如何联系,本人已经证得第1、2、4个,第3个如何证明?能否找到同一的证法?
ccnu_chb_ycb 2# 2012-8-16 22:54
能否给出第三个的证明,并指出他们之间个人觉得相同的地方……谢谢啦!
kuing 3# 2012-8-16 23:14
以前玩安zp的征解系列的时候写过(2和3),见附件。 后面用了schur分拆,算是有点暴力,估计那时懒得想巧法,这类通常都是先来个柯西或权方和(holder的推论)然后就变成玩多项式神马的。
海盗船长 4# 2012-8-17 10:16
http://www.wl84.com/forum.php?mo ... page%3D1&page=1
ccnu_chb_ycb 5# 2012-8-17 11:21
3# kuing 真心受教了,过程有些麻烦,但是很实用……希望可以给出其余两个的证明,还有就是后面的因式分解问题有什么思路呢
kuing 6# 2012-8-17 12:41
4# 海盗船长 那堆我也解过大部分,不过以前都写在word里,有空再慢慢打上来好了。
thread-723-1-1.html: 发个神奇的无穷乘积。。。
海盗船长 1# 2012-8-17 00:19
\[\prod_{n=1}^{\infty} \left(1 + e^{- \pi n}\right) = 2^{-\frac{1}{8}} e^{\frac{\pi}{24}}\] http://sos440.tistory.com/266
kuing 2# 2012-8-17 01:03
学习一下那个网站里的自动标号设置,我也尝试一下。 以下内容复制自一楼链接,粹纯测试。 Problem. Show the identity \begin{equation} \label{pb:wts} \prod_{n=1}^{\infty} \left(1 + e^{- \pi n}\right) = 2^{-\frac{1}{8}} e^{\frac{\pi}{24}}. \end{equation} Proof. This is related to the famous Dedekind eta function $\eta(s)$, which is defined by \begin{equation*} \eta(\tau) = e^{\frac{\pi i \tau}{12}} \prod_{n=1}^{\infty} \left( 1 - e^{2\pi i n \tau} \right), \end{equation*} although we does not need this relation in this posting. Let us denote \begin{equation} \label{pb:px} P(x) = \prod_{n=1}^{\infty} \left( 1 - e^{-2\pi n x} \right) \end{equation} and prove the identity \begin{equation} \label{pb:iden1} \frac{P(x)}{P\left(\frac{1}{x}\right)} = \frac{e^{\frac{\pi}{12}\left(x - \frac{1}{x}\right)}}{\sqrt{x}}, \end{equation} which holds for $\Re x > 0$. By analytic continuation, it suffices to prove \eqref{pb:iden1} only for positive $x$. Taking logarithm to \eqref{pb:px}, \begin{align*} \log P(x) & = \sum_{n=1}^{\infty} \log \left( 1 - e^{-2\pi n x} \right) \\ & = - \sum_{n=1}^{\infty} \sum_{m=1}^{\infty} \frac{1}{m} e^{-2\pi mn x} \\ & = - \sum_{m=1}^{\infty} \frac{1}{m} \sum_{n=1}^{\infty} e^{-2\pi mn x} \\ & = - \sum_{m=1}^{\infty} \frac{1}{m} \frac{1}{e^{2\pi n x} - 1} \\ & = - \sum_{m=1}^{\infty} \frac{1}{m} \left( \frac{1}{e^{2\pi m x} - 1} + \frac{1}{2} - \frac{1}{2\pi m x} - \frac{1}{2} \right) - \sum_{m=1}^{\infty} \frac{1}{2\pi m^2 x} \\ & = - \sum_{m=1}^{\infty} \frac{1}{m} \left( \frac{1}{2} \frac{e^{2\pi m x} + 1}{e^{2\pi m x} - 1} - \frac{1}{2\pi m x} - \frac{1}{2} \right) - \frac{\zeta(2)}{2\pi x} \\ & = - \frac{1}{2} \sum_{m=1}^{\infty} \frac{1}{m} \left( \coth (\pi m x) - \frac{1}{\pi m x} - 1 \right) - \frac{\pi}{12 x}. \end{align*} Now fix $x$ and define $h(t)$ and $f(t)$ by \begin{equation} \label{pb:hx} h(t) = \begin{cases} 1 & \text{if} \ 1 \leq t \\ t & \text{if} \ -1 < t < 1 \\ -1 & \text{if} \ t \leq -1 \end{cases} \end{equation} and \begin{equation} \label{pb:fx} f(t) = \frac{1}{t} \left( \coth (\pi x t) - \frac{1}{\pi x t} - h(t) \right). \end{equation} Singularity inspection of \eqref{pb:fx} at $t = 0$ and $t = \pm \infty$ shows that \begin{equation*} f(t) = \left(\frac{\pi  x}{3}-1\right) + O \left(t^2 \right) \quad \text{as} \ t \to 0 \end{equation*} and \begin{equation*} f(t) = O \left(t^{-2}\right) \quad \text{as} \ |t| \to \infty. \end{equation*} Thus $f$ is an integrable function whose singularity at the origin cancels out to yield $f(0) = \frac{\pi}{3} - 1$. Since $f$ is even, we obtain \begin{align*} \log P(x) & = - \frac{1}{2} \sum_{m=1}^{\infty} f(m) - \frac{\pi}{12 x} \\ & = - \frac{1}{4} \sum_{-\infty}^{\infty} f(m) + \frac{f(0)}{4} - \frac{\pi}{12 x} \\ & = - \frac{1}{4} \sum_{-\infty}^{\infty} f(m) + \frac{\pi}{12} \left( x - \frac{1}{x} \right) - \frac{1}{4}. \end{align*} To apply Poisson summation formula, we evaluate the Fourier transform $\hat{f}(\xi)$ of $f$. Since $f$ is real-valued and even, the same holds for $\hat{f}(\xi)$. Thus we may assume $\xi > 0$ for technical reason. Then \begin{align*} \hat{f}(\xi) & = \int_{-\infty}^{\infty} \frac{1}{t} \left( \coth (\pi x t) - \frac{1}{\pi x t} - h(t) \right) e^{-2\pi i \xi t} \; dt \\ & = 2 \int_{0}^{\infty} \frac{1}{t} \left( \coth (\pi x t) - \frac{1}{\pi x t} - h(t) \right) \cos (2\pi i \xi t) \; dt \\ & = 2 \int_{0}^{\infty} \frac{1}{u} \left( \coth (\pi u) - \frac{1}{\pi u} \right) \cos \left( \frac{2 \pi \xi u}{x} \right) \; du - 2 \int_{0}^{\infty} \frac{h(t)}{t} \cos (2 \pi \xi t) \; dt. \end{align*} Here we note that \begin{equation*} \frac{1}{t} \left( \coth (\pi  t)-\frac{1}{\pi  t} \right) = \frac{2}{\pi}\sum_{n=1}^{\infty} \frac{1}{t^2 + n^2} \end{equation*} and \begin{equation*} \int_{0}^{\infty} \frac{\cos \alpha u}{u^2 + \beta^2} \; du = \frac{\pi}{2\beta} \, e^{-\alpha \beta} \end{equation*} for $\alpha, \beta > 0$. Then we obtain \begin{align*} 2 \int_{0}^{\infty} \frac{1}{u} \left( \coth (\pi u) - \frac{1}{\pi u} \right) \cos \left( \frac{2 \pi \xi u}{x} \right) \; du \\ & = \frac{4}{\pi} \int_{0}^{\infty} \sum_{n=1}^{\infty} \frac{1}{u^2 + n^2} \cos \left( \frac{2 \pi \xi u}{x} \right) \; du \\ & = 2 \sum_{n=1}^{\infty} \frac{1}{n} \exp \left( -\frac{2 \pi \xi n}{x} \right) \\ & = -2 \log\left( 1 - \exp \left( - \frac{2 \pi \xi}{x} \right) \right), \end{align*} hence we have \begin{equation} \label{pb:fhat} \hat{f}(\xi) = -2 \left( \log\left( 1 - \exp \left( - \frac{2 \pi \xi}{x} \right) \right) + \int_{0}^{\infty} \frac{h(t)}{t} \cos (2 \pi \xi t) \; dt \right). \end{equation} Again, inspecting the singularity of \eqref{pb:fhat} at $\xi = 0$ shows that \begin{align*} \hat{f}(\xi) & = -2 \log\left( \frac{1 - e^{- \frac{2 \pi \xi}{x}}}{\frac{2 \pi \xi}{x}} \right) -2 \log \left( \frac{2 \pi \xi}{x} \right) - \frac{\sin 2 \pi \xi}{\pi \xi} - 2 \int_{2 \pi \xi}^{\infty} \frac{\cos u}{u} \; du \\ & = -2 \log \left( \frac{2 \pi \xi}{x} \right) - 2 - 2 \left( - \log \left( 2\pi \xi \right) - \gamma \right) + o(1)\\ & = 2 \log x - 2 + 2 \gamma + o(1), \end{align*} where we used the identity \begin{equation*} \int_{x}^{\infty} \frac{\cos u}{u} \; du = - \log x - \gamma + o(1) \quad \text{as} \ x \to 0^{+}. \end{equation*} Thus the singularity cancels out and we obtain $\hat{f}(0) = 2 \log x - 2 + 2 \gamma$. Then Poisson summation formula yields \begin{align*} \log P(x) & = - \frac{1}{4} \sum_{-\infty}^{\infty} \hat{f}(\xi) + \frac{\pi}{12} \left( x - \frac{1}{x} \right) - \frac{1}{4} \\ & = - \frac{1}{2} \sum_{\xi = 1}^{\infty} \hat{f}(\xi) - \frac{1}{4} \left( 2 \log x - 2 + 2 \gamma \right) + \frac{\pi}{12} \left( x - \frac{1}{x} \right) - \frac{1}{4}, \end{align*} and plugging \eqref{pb:fhat} yields \begin{align} \log P(x) & = \sum_{\xi = 1}^{\infty} \log\left( 1 - \exp \left( - \frac{2 \pi \xi}{x} \right) \right) - \frac{1}{2} \log x + \frac{\pi}{12} \left( x - \frac{1}{x} \right) + C \nonumber \\ & = \log P\left( \tfrac{1}{x} \right) - \frac{1}{2} \log x + \frac{\pi}{12} \left( x - \frac{1}{x} \right) + C, \label{pb:iden2} \end{align} where $C$ is an absolute constant given by \begin{equation} \label{pb:c} C = \frac{1}{4} - \frac{\gamma}{2} + \sum_{\xi = 1}^{\infty} \int_{0}^{\infty} \frac{h(t)}{t} \cos (2 \pi \xi t) \; dt. \end{equation} Now plugging $x = 1$ to \eqref{pb:iden2} shows that $C = 0$, therefore we have proved \eqref{pb:iden1}. The identity \eqref{pb:iden1} can be used to evaluate some exotic products. Plugging $x = \sqrt{2}$, we have \begin{equation*} \prod_{n=1}^{\infty} \left(1 + e^{- \sqrt{2} \pi n}\right) = 2^{-\frac{1}{4}} e^{\frac{\pi}{12\sqrt{2}}}. \end{equation*} Also, plugging $x = 1+i$ gives \begin{equation*} \prod_{n=1}^{\infty} \left(1 + e^{- \pi (2n-1)}\right) = 2^{\frac{1}{4}} e^{-\frac{\pi}{24}}. \end{equation*} Finally, define \begin{equation} \label{pb:qx} Q(x) = \frac{P(2x)}{P(x)} = \prod_{n=1}^{\infty} \left(1 + e^{-2\pi n x}\right). \end{equation} Then \begin{align*} P\left(\tfrac{1}{2}\right) \prod_{n=1}^{\infty} \left(1 + e^{- \pi (2n-1)}\right) & = \left[ \prod_{n=1}^{\infty} \left(1 - e^{- 2 \pi n} \right) \right] \left[ \prod_{n=1}^{\infty} \left(1 - e^{- \pi (4n-2)}\right) \right] \\ & = \frac{P(1)^2}{P(2)} = 2^{\frac{1}{2}} e^{-\frac{\pi}{8}} \frac{P(1)^2}{P\left(\frac{1}{2}\right)} = 2^{\frac{1}{2}} e^{-\frac{\pi}{8}} P\left(\tfrac{1}{2}\right) Q\left(\tfrac{1}{2}\right)^{2}. \end{align*} Therefore, combining these results together, we have \begin{equation*} \prod_{n=1}^{\infty} \left(1 + e^{- \pi n}\right) = Q(\tfrac{1}{2}) = 2^{-\frac{1}{8}} e^{\frac{\pi}{24}}. \end{equation*}
kuing 3# 2012-8-17 01:05
还真的可以,不错不错,等明天你们看下显示有没有问题。
海盗船长 4# 2012-8-17 10:19
3# kuing 嗯显示的很好,等式的那个链接也不错
kuing 5# 2012-8-17 12:46
4# 海盗船长 ok, that's grate 其实上面的过程我完全看不懂。
pxchg1200 6# 2012-8-17 19:40
5# kuing 没想到棒子的网站也有Latex...
kuing 7# 2012-8-17 19:44
6# pxchg1200 说明 mathjax 的被使用越来越广泛了 我在这里选择 mathjax 看来没选错
thread-724-1-1.html: 公式自动编号测试
kuing 1# 2012-8-17 01:23
\begin{align} \sqrt{\frac{a^2+b^2}2}&\geqslant \frac{a+b}2 \label{pj1}\\ &\geqslant \sqrt{ab} \label{pj2}\\ &\geqslant \frac2{\frac1a+\frac1b} \label{pj3} \end{align} 式 \eqref{pj1} 左边平方平均,右边算术平均;式 \eqref{pj2} 几何平均;式 \eqref{pj3} 调和平均。 而最基本的不等式 \begin{equation}\label{jb} (a-b)^2\geqslant0, \end{equation} 式 \eqref{jb} 都是式 \eqref{pj1}、\eqref{pj2}、\eqref{pj3} 的等价式。 大家看看以上公式的编号是否显示成功,有没有问题都回个贴告诉我一声哈。
kuing 2# 2012-8-17 01:27
试下隔楼层引用。 式 \eqref{pj1} 至式 \eqref{jb} 均等价。 还有 \begin{equation}\label{zl} \frac{a^2+b^2}{a+b}\geqslant\sqrt{\frac{a^2+b^2}2}, \end{equation} 式 \eqref{zl} 也等价于式 \eqref{jb}。 嗯,不过这样的话,你们引用我的编号的时候你要知道我的 label,也就是要右键查看一下了。 记住不要直接打数字,否则如果中间多加一个公式,那就对不上了,必须要用 \lable 和 \eqref 来对应,发完了的就别改了。
kuing 3# 2012-8-17 01:33
\begin{align} |a| &= \sqrt{a^2}\notag\\ &=\sqrt[4]{a^4}\\ &=\sqrt[6]{a^6}\\ &=\sqrt[8]{a^8}\notag\\ &=\sqrt[10]{a^{10}}\\ &=\cdots\notag\\ &=\begin{cases}a&a\geqslant0,\\ -a&a<0.\end{cases}\label{wll} \end{align} 式 \eqref{wll} 的编号对应整个分段嗯嗯 用 \notag 不生成编号
kuing 4# 2012-8-17 01:35
页面有点宽,编号对应哪条式要仔细点看了……
kuing 5# 2012-8-17 01:39
点击编号可以将页面跳至所对应式在顶端。
kuing 6# 2012-8-17 01:50
用了三个浏览器测试都没问题,就等你们测试了。
kuing 7# 2012-8-17 01:51
可惜的一点是,如果翻页了就引用不了了
kuing 8# 2012-8-17 14:51
顺便更新了一下置顶贴关于环境那里。 你们也试试编号and引用啊
thread-725-1-7.html: [不等式] 请教一道不等式问题
ccnu_chb_ycb 1# 2012-8-17 12:38
请高手指教一下这道不等式如何证明,它有个二元的形式,我用函数法证明了,但证法不能平移,忘指教!谢谢
kuing 2# 2012-8-17 12:44
喔,还是安zp的那个系列,恰好我当时又有玩
ccnu_chb_ycb 3# 2012-8-17 16:06
2# kuing 在安振平老师的26个优美的不等式中,您可以把您当初玩过的这些资料给我发一份吗?非常感谢,邮箱:chenghanbo511@sina.com
kuing 4# 2012-8-17 16:08
3# ccnu_chb_ycb 我打算有空统一整理再发上来。
ccnu_chb_ycb 5# 2012-8-17 16:15
4# kuing 恩,最近都在证那26个优美的不等式,现在就剩下:第11个(我的方法太复杂),第16个,第20,22,23,24这六个啦,能否指教下……真心感谢!
thread-726-1-2.html: 这四个webqq有什么不同?
kuing 1# 2012-8-17 14:13
百度搜索 webqq 的时候出来前四个东东如下 分别对应四种网址,不知有何不同? web.qq.com webqq.qq.com web2.qq.com w.qq.com 里面的界面好像没什么区别
戊概念·五 2# 2012-8-17 21:22
咋觉得这么面熟= =0
叶剑飞Victor 3# 2012-8-18 20:47
http://web.qq.com/ 是WebQQ第一版,第二版出来后,自动与第二版合并 http://web2.qq.com/ 这个是WebQQ第二版 http://w.qq.com/  这个原先是精简版的WebQQ,只能显示文字,不能显示图片。现在与 http://web.qq.com/ 合并,没有任何区别了 http://webqq.qq.com/ 这个不知道,从来没用过。
kuing 4# 2012-8-18 21:24
3# 叶剑飞Victor 擦,各种合并……? 搞不懂,为何精简版也要合并,如果还有精简版的话,我肯定用精简版
叶剑飞Victor 5# 2012-8-18 21:59
4# kuing 原先的那个精简版的 http://w.qq.com/ ,其实挺好用的,只显示文字,不显示图片。运行速度非常快,而且省内存。不知怎么的,就和完整版的 http://web.qq.com/ 合并了。
thread-727-1-1.html: 积分不等式
海盗船长 1# 2012-8-17 14:30
本帖最后由 海盗船长 于 2012-8-27 14:16 编辑 若$f(x)$在$(a,b)$上可微,且$|f'(x)|\le M$,则:\[\left|\int_a^b f(x) \mathrm{d}x-\frac{b-a}{2}(f(a)+f(b))\right|\le \frac{(b-a)^2M}{4}-\frac{(f(a)-f(b))^2}{4M}\] http://tieba.baidu.com/p/1797960483
pxchg1200 2# 2012-8-17 19:56
多了个尾巴感觉很诡异。。。
icesheep 3# 2012-8-27 07:14
你把题目打错了。。。
kuing 4# 2012-8-27 12:33
才发现少了一撇
海盗船长 5# 2012-8-27 14:16
改了。。
六月的煜 6# 2012-10-23 18:04
。。。。。。 http://www.wl84.com/forum.php?mo ... &extra=page%3D1
pxchg1200 7# 2012-12-22 09:46
终于会弄了。。。
kuing 8# 2012-12-22 12:42
6#的链接挂了……
pxchg1200 9# 2013-1-14 09:09
8# kuing 好像可以先证明: $g(x)$是$[a,b]$上的可导函数,且$|g'(x)|\leq L$,$ g(a)=g(b)=0$,则: \[ \left| \int_{a}^{b}{g(x)dx}\right|\leq \frac{(b-a)^2L}{4} \] 然后利用这个结论构造辅助函数 \[ F(x)=f(x)-f(a)-\frac{f(b)-f(a)}{b-a}(x-a) \] 估计可以得到结论。 :P
thread-728-1-2.html: pep是不是又上不了了?
海盗船长 1# 2012-8-17 15:55
RT
kuing 2# 2012-8-17 16:00
现在是上不了,等会就不一定了。 不管它咯……以后有什么好题都转过来玩
海盗船长 3# 2012-8-17 16:03
2# kuing 但是以前的老贴看不了了。。
海盗船长 4# 2012-8-17 16:04
有没有什么办法找到给定网址的网页的百度快照?
kuing 5# 2012-8-17 16:05
3# 海盗船长 老贴我有存档整个兴趣小组我都存了下来的
kuing 6# 2012-8-17 16:06
4# 海盗船长 不清楚,直接搜那个网址会不会搜到什么东东出来?
海盗船长 7# 2012-8-17 16:10
好像现在又可以了?
kuing 8# 2012-8-17 16:26
意料之中
kuing 9# 2012-8-23 02:09
听说再次升级
thread-729-1-7.html: [不等式] 几道优美不等式问题
ccnu_chb_ycb 1# 2012-8-17 16:30
在安振平老师的26个优美的不等式中,在下剩下这几个苦思冥想没得到好的解决方法,忘高手指教……万分感谢 有什么好的相关资料可以发本人邮箱:chenghanbo511@sina.com,也可与本人讨论这26个优美的不等式6
kuing 2# 2012-8-17 16:53
第一题我当年也是暴力展开然后凑均值的; 第二题SOS就行了; 第三题其实好像是很老的题目,只要用一个局部不等式\[\frac1{1+x+y}\leqslant\frac{x+y+z^2}{(x+y+z)^2}\]就可以了; 后面两道没查到记录,可能是以前没玩的,等会瞧瞧看……
kuing 3# 2012-8-17 17:28
第五题很简单 在 $\triangle ABC$ 中,$abc=1$,求证 \[\frac1{\sqrt{a(b+c-a)}}+\frac1{\sqrt{b(c+a-b)}}+\frac1{\sqrt{c(a+b-c)}}\geqslant \sqrt{3(a+b+c)}.\] 等价于三角形中证 \[\sum \sqrt{\frac{bc}{b+c-a}}\geqslant \sqrt{3(a+b+c)},\] 作内切圆代换 $a=y+z$, $b=z+x$, $c=x+y$, $x$, $y$, $z>0$,上式等价于 \[\sum \sqrt{\frac{(z+x)(x+y)}{2x}}\geqslant \sqrt{6(x+y+z)},\] 或 \[\sum \sqrt{x+y+z+\frac{yz}x}\geqslant \sqrt{12(x+y+z)},\] 由闵可夫斯基不等式及均值不等式,有 \begin{align*} \sum \sqrt{x+y+z+\frac{yz}x}&\geqslant \sqrt{\left( \sum \sqrt{x+y+z} \right)^2+\biggl( \sum \sqrt{\frac{yz}x} \biggr)^2} \\ & =\sqrt{11(x+y+z)+\sum \frac{yz}x} \\ & =\sqrt{11(x+y+z)+\frac12\sum \left( \frac{yz}x+\frac{xy}z \right)} \\ & \geqslant \sqrt{12(x+y+z)}, \end{align*} 即得证。
kuing 4# 2012-8-17 17:50
第四题就更简单了 在 $\triangle ABC$ 中,求证 \[3\left(\frac1{\cos\frac A2}+\frac1{\cos\frac B2}+\frac1{\cos\frac C2}\right)\tan\frac A2\tan\frac B2\tan\frac C2\leqslant2.\] 令 $x=\tan\frac A2$, $y=\tan\frac B2$, $z=\tan\frac C2$,则 $x$, $y$, $z>0$ 且 $xy+yz+zx=1$。原不等式等价于 \begin{equation}\label{20120817kkkkpro4djs} 3xyz\sum \sqrt{1+x^2}\leqslant 2, \end{equation} 齐次化,有 \begin{align*} \text{式}~\eqref{20120817kkkkpro4djs}&\iff 3xyz\sum \sqrt{xy+yz+zx+x^2}\leqslant 2(xy+yz+zx)^2 \\ & \iff \sum \sqrt{(z+x)(x+y)}\leqslant \frac23\cdot \frac{(xy+yz+zx)^2}{xyz} \\ & \iff \sum \sqrt{(z+x)(x+y)}\leqslant \frac23\left( \sum \frac{xy}z+2\sum x \right), \end{align*} 这很弱,皆因只要均值一下 \begin{gather*} \sum \sqrt{(z+x)(x+y)}\leqslant \sum \frac{2x+y+z}2=2\sum x, \\ \sum \frac{xy}z=\frac12\sum \left( \frac{xy}z+\frac{zx}y \right)\geqslant \sum x, \end{gather*} 就显然成立了。
pxchg1200 5# 2012-8-17 19:22
2# kuing 第二题不需要SOS,just notice that: By Holder inequality \[ \left(\sum{\frac{a^2}{b}}\right)^{2}(\sum{a^2b^2})\geq (\sum{a^2})^3 \] Just check \[ (a^2+b^2+c^2)^2\geq 3(a^2b^2+b^2c^2+c^2a^2) \] Which is obviously true.
kuing 6# 2012-8-17 19:25
5# pxchg1200 嗯,这个方法我也见过,而且还适用于四元的情况。 SOS是我当年的做法,配成 $\displaystyle\sum\left(\frac1b-\frac1{\sqrt{3\sum a^2}+\sum a}\right)(a-b)^2\geqslant0$。 两边平方再凑均值也可以。
pxchg1200 7# 2012-8-17 19:30
本帖最后由 pxchg1200 于 2012-8-17 19:46 编辑 2# kuing 第一题和第二题本是同根,注意到若应用下面结论 \[ (a^2+2)(b^2+2)(c^2+2)\geq 3(a+b+c)^2 \] Let $ a=\frac{x}{\sqrt{y}},b=\frac{y}{\sqrt{z}},c=\frac{z}{\sqrt{x}} $ gives \[ \left(\frac{x^2}{y}+2\right)\left(\frac{y^2}{z}+2\right)\left(\frac{z^2}{x}+2\right)\geq 3\left(\frac{x}{\sqrt{y}}+\frac{y}{\sqrt{z}}+\frac{z}{\sqrt{x}}\right)^{2}\] Just check \[  \left(\frac{x}{\sqrt{y}}+\frac{y}{\sqrt{z}}+\frac{z}{\sqrt{x}}\right)^{2}\geq 3(x+y+z) \] 这正是第二题结论。 done!
kuing 8# 2012-8-17 19:34
7# pxchg1200 oh!原来如此,nice,px好眼力,看来这题就是这样出的,减弱了已知结论,再代换变形…… PS、用用 left right
pxchg1200 9# 2012-8-17 19:45
8# kuing   运气好看到了。。。
ccnu_chb_ycb 10# 2012-8-17 20:02
受教了,好方法……好眼力
ccnu_chb_ycb 11# 2012-8-17 20:03
3# kuing 感谢,我也得闭门思过啦^
ccnu_chb_ycb 12# 2012-8-17 20:05
4# kuing [ 感谢!得下去多练下不等式,可否推荐点资料或书呢?
yizhong 13# 2012-8-17 23:36
小K,小PX,niubility
kuing 14# 2012-8-17 23:41
13# yizhong 我证的都是简单的,px 的那个比较漂亮些。
海盗船长 15# 2012-8-18 00:14
http://www.wl84.com/forum.php?mo ... ptid=45&pid=127
thread-73-1-1.html: [转]bottema2009
kuing 1# 2011-10-8 20:38
http://www.irgoc.org/viewtopic.php?f=10&t=21 内含简易操作指南
thread-730-1-7.html: [不等式] nguoivn's inequality.
pxchg1200 1# 2012-8-17 20:03
Let $a,b,c>0$ prove that: \[ \frac{1}{a}+\frac{1}{b}+\frac{1}{c}+\frac{3}{2a+b}+\frac{3}{2b+c}+\frac{3}{2c+a}\geq 2\sqrt{3}\left(\frac{1}{\sqrt{a(a+2b)}}+\frac{1}{\sqrt{b(b+2c)}}+\frac{1}{\sqrt{c(c+2a)}}\right) \] (表示真心不会做。。。)
yizhong 2# 2012-8-18 10:54
oh,均值基本可秒,$2\sqrt{\dfrac{1}{a}.\dfrac{3}{a+2b}}=2\sqrt{\dfrac{3}{2a+b}.\dfrac{2a+b}{a(a+2b)}} \leqslant\dfrac{3}{2a+b}+\dfrac{2a+b}{a(a+2b)}$同理我们可以得到:$2\sqrt{\dfrac{1}{b}.\dfrac{3}{b+2c}}\leqslant \dfrac{3}{2b+c}+\dfrac{2b+c}{b(b+2c)}$  $2\sqrt{\dfrac{1}{c}.\dfrac{3}{c+2a}}\leqslant\dfrac{3}{2c+a}+\dfrac{2c+a}{c(c+2a)}$ 所以我们只要能证明:$\dfrac{1}{a}+\dfrac{1}{b} +\dfrac{1}{c}\geqslant\dfrac{2a+b}{a(a+2b)}+\dfrac{2b+c}{b(b+2c)}+\dfrac{2c+a}{c(c+2a)}(1)$我们又有:$\dfrac{(a-b)^2}{ab(a+2b)}\geqslant0,也即:\dfrac{2}{a}+\dfrac{1}{b}\geqslant\dfrac{3(2a+b)}{a(a+2b)}$ 同理我们可以得到其余的两个式子,最后叠加即可证得(1).
kuing 3# 2012-8-18 14:39
2# yizhong 好证法! PS1、“均值基本可秒”,但想到这个均值所用的时间……可能要时,或天,甚至 $+\infty$ 呢 PS2、咋不试试用行间公式呢 PS3、试试用 equation 环境来自动编号和引用,这样不用手工打那些(1)什么的。
yizhong 4# 2012-8-18 14:55
确实,我刚刚开始的证法比较拙略 ,后来想用纯A-G,或者CS,一开始试用 均值的时候是约掉:$\sum \dfrac{1}{a}$这一串,但是很快很发现导不出,所以接下去我就想看能 不能约掉:$\sum \dfrac{3}{2a+b}$(因为这个不等式的左边6项,所以我就想想看能不能再化简下)
pxchg1200 5# 2012-8-18 17:21
4# yizhong Nice!
thread-731-1-7.html: [不等式] interesting inequailty.
pxchg1200 1# 2012-8-17 20:05
Let $a,b,c $ are real numbers with $a+b+c=3 $ Prove that: \[ \frac{a^2+b^2c^2}{(b-c)^{2}}+\frac{b^2+c^2a^2}{(c-a)^{2}}+\frac{c^2+a^2b^2}{(a-b)^{2}}\geq 5 \]
thread-732-1-7.html: [不等式] weird one
pxchg1200 1# 2012-8-17 21:49
For $x,y,z>0, c>0$ with $ x+y+z=3 $ find the Min Value of P \[ P=\frac{1}{\sqrt{x^2+y+c}}+\frac{1}{\sqrt{y^2+z+c}}+\frac{1}{\sqrt{z^2+x+c}} \]
thread-733-1-7.html: [不等式] $\sum\frac{1-xy}{y+z+x^2}\ge 0$
海盗船长 1# 2012-8-18 12:52
本帖最后由 海盗船长 于 2012-8-18 20:48 编辑 http://tieba.baidu.com/p/1793749432
thread-734-1-6.html: [不等式] 转 Ji Chen 的一道题
kuing 1# 2012-8-18 17:18
转自:http://www.math.org.cn/forum.php?mod=viewthread&tid=21779 设非负数 $a$, $b$, $c$, $d$ 满足 $a+b+c+d=2$,则 \[\frac{a^2}{(1+a^2)^2}+\frac{b^2}{(1+b^2)^2}+\frac{c^2}{(1+c^2)^2}+\frac{d^2}{(1+d^2)^2}\leqslant\frac{16}{25},\] 等号成立当且仅当 $a=b=c=d=\frac12$。
kuing 2# 2012-8-18 17:57
下面还是运用我在网刊写过的“分类使用切线法”了,这次我在另一个分类时我将再次切线和支撑线。 啊!这个例子要是早点知道,写该文的时候就可以作为一个很好的例题了。 废话少说,开切吧。 由对称性,不妨设 $0\leqslant a\leqslant b\leqslant c\leqslant d$,下面分两种情况。 (1)当 $a\geqslant 1/8$ 时。 我们先证明当 $x\geqslant 1/8$ 时有 \begin{equation}\label{jichenbds20120818qx1} \frac{x^2}{(1+x^2)^2}\leqslant \frac4{125}(12x-1), \end{equation} 上式左右作差分解有 \[\frac4{125}(12x-1)-\frac{x^2}{(1+x^2)^2}=\frac{(2x-1)^2\bigl(12x^3+11x^2+4(8x-1)\bigr)}{125(1+x^2)^2}\geqslant 0,\] 故式 \eqref{jichenbds20120818qx1} 成立,于是有 \[\frac{a^2}{(1+a^2)^2}+\frac{b^2}{(1+b^2)^2}+\frac{c^2}{(1+c^2)^2}+\frac{d^2}{(1+d^2)^2}\leqslant \frac4{125}\bigl(12(a+b+c+d)-4\bigr)=\frac{16}{25},\] 所以此时原不等式成立; (2)当 $0\leqslant a<1/8$ 时。 我们先证明当 $x\geqslant 0$ 时有 \begin{equation}\label{jichenbds20120818qx2} \frac{x^2}{(1+x^2)^2}\leqslant \frac{108(5x+1)}{2197}, \end{equation} 上式左右作差分解有 \[\frac{108(5x+1)}{2197}-\frac{x^2}{(1+x^2)^2}=\frac{(3x-2)^2(60x^3+92x^2+216x+27)}{2197(1+x^2)^2}\geqslant 0,\] 故式 \eqref{jichenbds20120818qx2} 成立。 再证明当 $0\leqslant x<1/8$ 时有 \begin{equation}\label{jichenbds20120818qx3} \frac{x^2}{(1+x^2)^2}\leqslant \frac{512x}{4225}, \end{equation} 上式左右作差分解有 \[\frac{512x}{4225}-\frac{x^2}{(1+x^2)^2}=\frac{x(1-8x)(512-129x-8x^2-64x^3)}{4225(1+x^2)^2},\] 由 $0\leqslant x<1/8$ 显然有 $1-8x>0$ 及 $512-129x-8x^2-64x^3>0$,故式 \eqref{jichenbds20120818qx3} 成立。 于是,由式 \eqref{jichenbds20120818qx2} 和式 \eqref{jichenbds20120818qx3},我们有 \begin{align*} \frac{a^2}{(1+a^2)^2}+\frac{b^2}{(1+b^2)^2}+\frac{c^2}{(1+c^2)^2}+\frac{d^2}{(1+d^2)^2}&\leqslant \frac{512a}{4225}+\frac{108\bigl(5(b+c+d)+3\bigr)}{2197} \\ & =\frac{512a}{4225}+\frac{108\bigl(5(2-a)+3\bigr)}{2197} \\ & =\frac{108}{169}-\frac{6844a}{54925} \\ & \leqslant \frac{108}{169}, \end{align*} 而在数值上,也有 \[\frac{16}{25}-\frac{108}{169}=\frac4{4225}>0,\] 所以此时 \[\frac{a^2}{(1+a^2)^2}+\frac{b^2}{(1+b^2)^2}+\frac{c^2}{(1+c^2)^2}+\frac{d^2}{(1+d^2)^2}<\frac{16}{25},\] 原不等式也成立。 综合(1)(2),原不等式获证。
yizhong 3# 2012-8-21 14:51
小k又发威了,我的切和你的差不多
thread-735-1-4.html: [几何] 一道三角形难题,正三角形证正三角形,题目要看仔细
叶剑飞Victor 1# 2012-8-19 19:55
已知:如图,在 $\triangle ABC$ 中,$BD=CE=AF$。$\triangle DEF$ 是正三角形。 问:$\triangle ABC$ 是否为正三角形? 如果是,请给出证明;如果不是,请举出反例。
海盗船长 2# 2012-8-19 20:44
http://tieba.baidu.com/p/1801314903 http://tieba.baidu.com/p/1803299127 http://tieba.baidu.com/p/1149267764 http://tieba.baidu.com/p/1800632917 http://tieba.baidu.com/p/1802817756 http://tieba.baidu.com/p/1803542290 http://tieba.baidu.com/p/1803391162 http://tieba.baidu.com/p/1801314903 http://tieba.baidu.com/p/1801314903
海盗船长 3# 2012-8-19 20:49
本帖最后由 海盗船长 于 2012-8-19 20:59 编辑 http://tieba.baidu.com/p/1801895331 http://tieba.baidu.com/p/1803245635 http://tieba.baidu.com/p/1153811175 http://tieba.baidu.com/p/1802645340 http://tieba.baidu.com/p/1802907736 http://tieba.baidu.com/p/1802572116 http://tieba.baidu.com/p/1801574213 http://tieba.baidu.com/p/1799099284 http://tieba.baidu.com/p/1801693831 http://tieba.baidu.com/p/1802249685 http://tieba.baidu.com/p/1801633960
海盗船长 4# 2012-8-19 20:54
本帖最后由 海盗船长 于 2012-8-19 21:54 编辑 http://tieba.baidu.com/p/1153520219 http://tieba.baidu.com/p/1724503971 http://tieba.baidu.com/p/1801056619
海盗船长 5# 2012-8-19 21:04
http://www.artofproblemsolving.c ... p?f=46&t=494527
kuing 6# 2012-8-20 01:43
这道题怎么那么热,刚才又有人问我。
海盗船长 7# 2012-8-20 11:15
那个AoPS链接里的解法不错
kuing 8# 2012-8-20 12:07
7# 海盗船长 表示看不懂,复数我也考虑过。 倒是帮我看下上面那个证法有问题没有?
海盗船长 9# 2012-8-20 13:46
8# kuing 后来发现那个解答好像是错的。。。
海盗船长 10# 2012-8-20 14:59
8# kuing 感觉你发的那个也有点问题,,网上有种观点认为此题相当于轮换对称不能这样设序,还要证明另一种情况。
海盗船长 11# 2012-8-20 20:16
http://domino.research.ibm.com/c ... ges/August1998.html
kuing 12# 2012-8-20 22:16
哎,能不能那样设,我也有点晕了不太会判断了心情欠佳状态太差
kuing 13# 2012-8-22 03:12
kuing<kuingggg@qq.com>  17:50:42 本来想了一个三角法, 后来发现有一种情况有问题,没修正成功, 没什么心情就没理了。
kuing 14# 2012-8-22 03:13
话说貌似不少人第一步都不妨设了顺序……
isea 15# 2012-8-23 14:50
这题很早的时候,有人在人教初中数学版问过,好像是0.1,无人问津。 最近很火,可能是因为微博上很流得吧 这题真的不好证, 我回去翻翻帖那些链接,看看有没靠谱的几何证法。
kuing 16# 2012-8-23 14:53
15# isea 0.1 问的那个好像跟这个有点不同,但可能是一个更广的命题。
isea 17# 2012-8-23 14:57
16# kuing 记起来了,是不一样,点么那个或多或少跟Ceva定理有点关系。 海盗所给的那些链接,好多是同质的 没仔细看那些......
kuing 18# 2012-8-23 15:08
17# isea 我也没仔细看那些。 你所指的 0.1 的那个贴是不是如附件所示?
kuing 19# 2012-8-23 15:16
18# kuing 不知那个斜坐标解析法能不能移植来这里?
isea 20# 2012-8-23 15:33
对,0.1的就是那个,原来是M大已经强算出来了啊。
thread-735-2-4.html:
isea 21# 2012-12-3 10:27
研究与总结 两内角平分相等的三角形是等腰三角形 时附带着把这题也解决了 战巡说反证容易,不过,我没看到过完整的过程。 稍后给证法……
kuing 22# 2012-12-3 14:12
21# isea 战巡的好像也是一开始设了大小顺序的,有空找找在哪
isea 23# 2012-12-3 17:13
22# kuing 证法来了,从这证法看,还是有意思的题。 晕,忘记不能传附件,又懒得再敲一次代码,给个PEP的链接图片吧
kuing 24# 2012-12-3 17:40
能传附件啊,谁说不能传?
kuing 25# 2012-12-3 17:45
这里的 y 有范围,恒成立不一定要 $\Delta\le0$ 吧?
kuing 26# 2012-12-3 18:14
战巡说反证容易,不过,我没看到过完整的过程。 isea 发表于 2012-12-3 10:27 找到了记录,如下: 爱好者-战巡(3705*****) 21:54:34 这里AD=BE=CF,不妨设∠A>=∠C>=∠B,则肯定有∠A>=60,∠B<=60,于是作∠EB'D=∠FA'D=60,显然B'在BD上(包括B点),A'在DA延长线上(包括A点) 然后∠B'DE+∠FDA'=120,∠B'ED+∠B'DE=120,可知∠B'ED=∠FDA',可证△B'ED≌△A'FD,有B'E=DA'>=AD=BE,问题是∠BB'E=120,△BB'E中BE是钝角所对边,肯定最大,有BE>=B'E,合起来就是BE=B'E,B与B'重合 正是用了还有争议的不妨设。
isea 27# 2012-12-5 21:56
本帖最后由 isea 于 2012-12-5 21:58 编辑 25# kuing 的确有瑕疵,需要讨论对称轴,如果看成函数的话;如果直接配方 $(y-acosC)^2+a^2cos^2B-a^2cos^2C\ge0$,也不自然…… 如果讨论角的大小,也会用到A、B、C三角的排序 PS:DZ7.0.0原来传附件在下面,晕迷
isea 28# 2012-12-5 22:03
找到了记录,如下: 正是用了还有争议的不妨设。 kuing 发表于 2012-12-3 18:14 这个排序,我是能承认的,在“两内角平分线相等的三角是等腰三角形”中(这里是轮换,可能略有差异),罗增儒老师就是这样讨论的。 战巡这方法还是简洁的,至少目前我觉得算是最完善的,其本质是作两圆(与已知正三角形的外接圆全等),讨论点的位置,从而得到矛盾
kuing 29# 2013-2-6 21:48
难道碰到什么min感词?
海盗船长 30# 2013-2-6 21:57
最近又看到这个题。。 战巡方法如果∠A>=∠B>=∠C貌似就不能用了,怎么破?
kuing 31# 2013-2-6 22:45
最近又看到这个题。。 战巡方法如果∠A>=∠B>=∠C貌似就不能用了,怎么破? 海盗船长 发表于 2013-2-6 21:57 证如我前面所讲,假如换另一种次序发现不能同样地证,那就是不能那样不妨设。 几何上的不妨设,不如不等式中的不妨设那样容易看,但是通过代数化也许也看得更清。 下面把之前说的 kuing  17:50:42 本来想了一个三角法, 后来发现有一种情况有问题,没修正成功, 没什么心情就没理了。 kuing 发表于 2012-8-22 03:12 具体写一写。 如图所示,我们有 $x$, $y$, $z\in (0,120^\circ)$,由正弦定理有 \[\frac{\sin x}{\sin A}=\frac{\sin y}{\sin B}=\frac{\sin z}{\sin C}=\frac ab,\] 又易知 $A=z-x+60^\circ$, $B=x-y+60^\circ$, $C=y-z+60^\circ$,代入即 \[\frac{\sin x}{\sin(z-x+60^\circ )}=\frac{\sin y}{\sin(x-y+60^\circ )}=\frac{\sin z}{\sin(y-z+60^\circ )}=\frac ab,\] 要证的就是 $x=y=z$。 可以发现,最后这个方程组对 $x$, $y$, $z$ 并不是完全对称的,这算不算是从另一个角度说明图形只是轮换对称而非完全对称? 最后这个方程组我现在还没完全解决。
kuing 32# 2013-2-6 22:52
原 29# 又  不  见  了……到底遇到了什么min gan词啊
goft 33# 2013-2-7 14:14
易证⊿BDF≌⊿CF'D',则B=C,同理A=C,故A=B=C
kuing 34# 2013-2-7 14:54
33# goft 如何“易证”?我没看出来……可否详写
yes94 35# 2013-2-7 14:58
34# kuing 易证是他的特色, 他讲题时喜欢给别人思路,不太喜欢给过程,尤其是详细过程。
海盗船长 36# 2013-2-15 19:58
看看这个证明
海盗船长 37# 2013-2-15 20:03
还要说明点DEF都在线段上,不然不对
yes94 38# 2013-2-15 20:19
37# 海盗船长 欢迎海盗回归!
thread-736-1-7.html: 2012北京理科20题第(3)问思路
hflz01 1# 2012-8-19 20:25
附件中的j=1,2,...,t+1怎么来的?
thread-738-1-1.html: Infinite Product
海盗船长 1# 2012-8-20 13:40
本帖最后由 海盗船长 于 2012-8-20 14:31 编辑 $\begin{align*} \prod_{n=2}^{\infty}\left(1-\frac{1}{n^3}\right) & =\prod_{n=2}^{\infty}\frac{(n-1)(n^2+n+1)}{n^3}\\& =\lim_{m\to \infty}\frac{1}{m}\prod_{n=2}^m\frac{(n-\omega)(n-\omega^2)}{n^2}\\& =\lim_{m\to \infty}\frac{\Gamma(m+1-\omega)\Gamma(m+1-\omega^2)}{m(m!)^2\Gamma(-\omega)\Gamma(-\omega^2)(1-\omega)(1-\omega^2)(-\omega)(-\omega^2)}\\& =\frac{1}{3\Gamma(-\omega)\Gamma(-\omega^2)}=\frac{\sin{\pi(-\omega)}}{3\pi}\\& =\frac{\cosh (\frac{ \sqrt{3}\pi}{2} )}{3\pi }\end{align*}$ where $\omega=\frac{-1+\sqrt{3}i}{2}$ we have used the fact that $$\Gamma(m+1-a)=(m-a)(m-a-1)\cdots(1-a)(-a)\Gamma(-a)$$ $$\Gamma(x)\Gamma(1-x)=\frac{\pi}{\sin(\pi x)}$$ and $$\lim_{x\to \infty}\frac{x^a\Gamma(x-a)}{\Gamma(x)}=1$$ http://www.artofproblemsolving.c ... p?f=67&t=494649
海盗船长 2# 2012-8-20 13:57
本帖最后由 海盗船长 于 2012-8-20 14:03 编辑 $$\prod_{n=2}^{\infty}\left(1-\frac{1}{n^2}\right)=\frac{1}{2}$$ $$\prod_{n=2}^{\infty}\left(1-\frac{1}{n(n-1)}\right)=-\frac{\cos\left( \frac{\sqrt{5}\pi }{2} \right)}{\pi }$$
海盗船长 3# 2012-8-20 14:25
本帖最后由 海盗船长 于 2012-8-20 14:30 编辑 $\begin{align*} \prod_{n=2}^{\infty}\left(1-\frac{1}{n^4}\right) & =\prod_{n=2}^{\infty}\frac{(n-1)(n+1)(n^2+1)}{n^4}\\& =\lim_{m\to \infty} \frac{m+1}{2m}\prod_{n=2}^m \left(1+\frac{1}{n^2}\right)\\& =\frac{1}{2}\prod_{n=2}^{\infty}\frac{(n+i)(n-i)}{n^2}\\& =\frac{1}{2} \lim_{n\to \infty}\frac{\Gamma(n+1+i)\Gamma(n+1-i)}{n^2 \Gamma(-i)\Gamma(1+i)(1-i)(1+i)(-i)}\\& = \frac{i\sin(-\pi i)}{4\pi} =\frac{\sinh(\pi)}{4\pi}\end{align*}$
叶剑飞Victor 4# 2012-8-23 04:36
本帖最后由 叶剑飞Victor 于 2012-8-23 04:42 编辑 “$\backslash$begin{align*}”和“$\backslash$end{align*}”的两边不需要再加上“\(\$\)”符号。
kuing 5# 2012-8-23 05:11
4# 叶剑飞Victor 可能他不想要行间  或者一开始打了后来忘记删?  呵呵… PS,如果没记错, 直接反斜杠后跟美元符号就能显示 \$ 。 现在爪机回复ing无法立即看到效果, 睡醒再看。
海盗船长 6# 2012-8-23 10:52
5# kuing 奇怪,,我那天刚写完1楼的时候还可以显示的,现在好像不能显示了
kuing 7# 2012-8-23 12:12
6# 海盗船长 现在可以了,是 TAG 惹的祸,我关闭了 TAG 功能。
thread-740-1-7.html: 推荐一个好软件
hhhzh7241hzh 1# 2012-8-20 17:33
用这个更好!
hhhzh7241hzh 2# 2012-8-20 17:42
(1)当2x-3y+3/2>=0时,f(x,y)=3x-6y+11/2,最小值是f(0.3,0.7)=2.2;最大值是f(0,-1)=11.5。值域是[2.2,11.5]。 (2)当2x-3y+3/2<0时,f(x,y)=-x+5/2,无最小值,但f(x,y) --> f(0.3,0.7)=2.2;最大值是f(-1,0)=3.5。值域是(2.2,3.5]。 综上,f(x,y) 的值域是(2.2,11.5]。
kuing 3# 2012-8-20 18:12
还行 能不能搞点带参数控制神马的?
hhhzh7241hzh 4# 2012-8-20 19:23
具体要联系到软件开发者。
thread-741-1-6.html: [不等式] Vasc‘s new inequality
pxchg1200 1# 2012-8-20 19:21
Let $a,b,c>0$ show that: \[ \frac {a}{\sqrt{a+3b}}+\frac {b}{\sqrt{b+3c}}+\frac {c}{\sqrt{c+3a}}\ge\frac {1}{2}(\sqrt a+\sqrt b+\sqrt c). \]
yizhong 2# 2012-8-25 16:33
本帖最后由 yizhong 于 2012-8-25 16:35 编辑 1# pxchg1200 此题已找到好方法,现在爪机,改天补上。
yizhong 3# 2012-8-30 08:54
本帖最后由 yizhong 于 2012-8-30 09:14 编辑 几天没有上来论坛发帖子了 ,早上抽个空来回下帖子, 。 设:$\sqrt{a}=x, \sqrt{b}=y  ,\sqrt{c}=z$,在这里我们记:$P=\frac{x^2}{\sqrt{x^2+3y^2}}+\frac{y^2}{\sqrt{y^2+3z^2}}+\frac{z^2}{\sqrt{z^2+3x^2}}$由赫尔德不等式我们有:$P^2.(x^4+y^4+z^4+3x^2y^2+3x^2z^2+3y^2z^2)\geqslant(x^2+y^2+z^2)^3$ 如果我们能够证明:$\frac{(x^2+y^2+z^2)^3}{x^4+y^4+z^4+3x^2y^2+3x^2z^2+3y^2z^2}\geqslant\frac{(x+y+z)^2}{4}(A)$则原不等式即可证明。而(A)又相当于证明:$4(x^2+y^2+z^2)^3\geqslant(x+y+z)^2(x^2+y^2+z^2)^2+(x+y+z)^2(x^2y^2+x^2z^2+y^2z^2)(B)$ 而由于:$3(x^2+y^2+z^2)^3\geqslant(x+y+z)^2(x^2+y^2+z^2)^2(容易由CS得到)(1)$由(1)我们又可以得到:$(x^2+y^2+z^2)^3\geqslant(x+y+z)^2(x^2y^2+x^2z^2+y^2z^2)(2)$,所以(B)得证。所以原不等式证毕。
thread-742-1-7.html: [不等式] Nguyễn Huy Hùng's inequality
pxchg1200 1# 2012-8-20 19:23
本帖最后由 pxchg1200 于 2012-8-20 19:24 编辑 Let $a,b,c>0$ prove that: \[ (a^{2}+ab+b^{2})(b^{2}+bc+c^{2})(c^{2}+ca+a^{2})\le\left(\frac{a^{2}+b^{2}+c^{2}+(\sqrt{3}-1)(ab+bc+ca)}{\sqrt{3}}\right)^{3} \] it's weak or not?
thread-743-1-6.html: [不等式] Maybe easy one
pxchg1200 1# 2012-8-20 19:26
Let $a,b,c>0$ with $a+b+c=3 $ prove that: \[ {\frac{{a}^{2}}{b}}+{\frac{{b}^{2}}{c}}+{\frac{{c}^{2}}{a}}+4(ab+bc+ca)\ge 15 \]
yizhong 2# 2012-8-21 15:17
本帖最后由 yizhong 于 2012-8-21 15:55 编辑 一看这个这个题目如果引用一条引理的话(can引理),那么是极其容易,这条引理如下:a,b,c>0 $\sum \dfrac{a^2}{b}\geqslant\dfrac{15(a^2+b^2+c^2)}{2(a+b+c)}-\dfrac{3(a+b+c)}{2}$ 引用这条引理后,待证明的不等式就相当于只需要证明:$5(a^2+b^2+c^2)+8(ab+ac+bc)\geqslant39$ 也就是:$5(a+b+c)^2-10(ab+ac+bc)+8(ab+ac+bc)\geqslant39$ 即:$3\geqslant(ab+ac+bc)$ 而这个是容易得到的,证毕。 另外我们还可以建立这么一条引理:a,b,c>0,$\sum \dfrac{a^2}{b}+(a+b+c)\geqslant\dfrac{6(a^2+b^2+c^2)}{(a+b+c)}$ 这条引理可以用SOS证明,引用这条引理我们可以得到待证明的不等式相当于证明:$6(a^2+b^2+c^2)+12(ab+ac+bc)\geqslant54$ 而:$6(a+b+c)^2=54$  所以原不等式得证。
yizhong 3# 2012-8-21 16:01
今天就先到这里,改天再上来
kuing 4# 2012-8-22 01:56
括号…………
thread-745-1-6.html: [函数] 函数迭代
hflz01 1# 2012-8-21 20:33
如附件。
海盗船长 2# 2012-8-22 00:06
set $\displaystyle f_0(x)=(1+x)^m=\sum_{i=0}^m C_m^i x^i$ then $f_{n+1}(x)=f_n(x)+x f'_n(x)=(x f_n(x))'$ $$\Longrightarrow f_{1}(x)=\frac{\mathrm{d}}{\mathrm{d} x}(x f_0(x))=\sum_{i=0}^m (i+1)C_m^i x^i$$ $$\Longrightarrow f_{n}(1)=\frac{\mathrm{d}}{\mathrm{d} x}(x f_n(x))_{x=1}=\sum_{i=0}^m (i+1)^nC_m^i$$
kuing 3# 2012-8-22 03:08
2# 海盗船长 最后的结果有没有化简的可能?
海盗船长 4# 2012-8-22 11:20
3# kuing 感觉不行,一般化简最后那个式子都是反过来用母函数或者已知的组合恒等式递推出来的,好像没有通式。
hflz01 5# 2012-8-26 09:33
可能是我没问清楚,二楼把问题还原了。本想得到一个多项式的结果,若能得到,就能解决等差等比数列与组合数的恒等式问题。具体猜想见附件,求证明或否定。
kuing 6# 2012-8-26 10:42
还原……
海盗船长 7# 2012-8-27 14:18
thread-746-1-2.html: 原来"姬"和"姫"是两个不同的字
kuing 1# 2012-8-21 23:15
RT 姬姫
叶剑飞Victor 2# 2012-8-23 21:43
又来折腾生僻字。 “姬”读做jī “姫”读做zhěn
海盗船长 3# 2012-8-23 21:46
我们初中数学老师就姓姬 ,刚初一的时候还给我们辨析过这两个字
kuing 4# 2012-8-23 21:49
3# 海盗船长 niubility
thread-747-1-5.html: 突然想起一个老问题,很久以前问过,还没看到过解答,关于覆盖的[已解决]
kuing 1# 2012-8-22 03:19
给出一些面积和为 1 的正方形,证明(或否定):它们能不重叠地放在面积为 2 的正方形里。
海盗船长 2# 2012-8-22 11:23
找到一贴,,不过解答好像有问题,二楼的思路应该可以 http://www.artofproblemsolving.c ... 1&hilit=overlap
kuing 3# 2012-8-22 12:40
2# 海盗船长 这样也能找到,厉害,求翻译……
kuing 4# 2012-8-23 14:51
4# isea 我最早问的时候大概是06年…… 其实我是突然想起来才发贴的,并没有看到相关贴子……这么巧这题最近火了?
kuing 5# 2012-8-23 14:52
……原来 isea 回错贴了 火的是另一个贴
isea 6# 2012-8-23 15:00
5# kuing 哈哈,这样的问题,纯是竞赛题了,关注的人更少。 搞竞赛的人,又基本不上论坛......
海盗船长 7# 2012-8-24 17:31
3# kuing 那贴二楼的大概思路: 首先将这些小正方形由大到小排序,设边长为$a_1,a_2,\cdots$,将$a_1$放在大正方形的左下角然后将剩下的按顺序放在$a_1$上方直到这一列排满,然后另起一列在底部放$a_{k_1}$,进行类似第一列的做法,以此类推。 按这个思路可以将原题化为证明http://www.wl84.com/forum.php?mod=viewthread&tid=62这个命题,这个没做出来。。
realnumber 8# 2012-12-14 20:37
【初等数学小丛书】《覆盖》单墫.pdf 82页,的第44题的证明题,没答案的 自己想段时间,再去看这章内容,似乎有个正方形覆盖例题
kuing 9# 2012-12-14 20:39
8# realnumber 没看过这本书的说…… PS,楼上上的链接似乎挂掉了
realnumber 10# 2012-12-14 21:18
没写连接,可以去新浪爱问下的,数学小丛书
realnumber 11# 2012-12-16 10:17
本帖最后由 realnumber 于 2012-12-16 10:26 编辑 书后面有答案的,
kuing 12# 2012-12-16 13:33
11# realnumber 果然厉害
thread-748-1-2.html: 我想要喝酷儿!!!
kuing 1# 2012-8-22 03:28
NNN年前喝过,现在咋完全不见了哩……
kuing 2# 2012-8-23 05:28
睡觉
海盗船长 3# 2012-8-23 11:02
2# kuing
isea 4# 2012-8-23 15:09
3# 海盗船长 喝什么来着?
kuing 5# 2012-8-23 15:15

海盗船长 6# 2012-8-23 20:59
没喝过。。。
kuing 7# 2012-8-23 21:01
那见过没……
海盗船长 8# 2012-8-23 21:12
很久以前见过。。
kuing 9# 2012-8-23 21:14
我上一次喝估计是初中的时候……
地狱的死灵 10# 2012-8-23 22:00
喝水就好了,反正你这是水帖……
yizhong 11# 2012-8-23 22:01
酷儿,酷儿,酷儿 我以开始还以为是饮料,谁知道原来是小K的昵称
kuing 12# 2012-8-23 22:03
11# yizhong 其实这名字挺不错
thread-749-1-2.html: 边吃边睡
kuing 1# 2012-8-22 13:48
中午群里提到“边吃边睡”,让我想起下面的sp http://www.tudou.com/programs/view/KIF60kKoGHU/
thread-75-1-1.html: 公式中加颜色、加粗、加下划线、调大小等操作汇总
kuing 1# 2011-10-8 22:19
在论坛上,正文普通文字加颜色、加粗、下划线等可以在发贴回贴时通过点击编译界面上的按钮得到,但在公式里似乎不可以这样,否则公式可能会因里头夹杂了Discuz!代码而无法识别,以致于显不出公式。 所以需要用latex里的这些方法,下面一一列出 公式中加颜色: \color{颜色名}{需要加颜色的部分} 例: a+\color{red}{b+c}+\color{yellow}{d}+\color{blue}{e+f+}g\color{gray}{+h}\color{green}{+i+j}\color{orange}{+k}+l+m 效果\[ a+\color{red}{b+c}+\color{yellow}{d}+\color{blue}{e+f+}g\color{gray}{+h}\color{green}{+i+j}\color{orange}{+k}+l+m \]总之,懂一点英文,问题不大。 公式中加粗,用于向量、矩阵: \boldsymbol{需要加的部分} 例: \boldsymbol{a}=(a_1,a_2,a_3), \boldsymbol b=(b_1,b_2,b_3) 效果\[ \boldsymbol{a}=(a_1,a_2,a_3), \boldsymbol b=(b_1,b_2,b_3) \]再输一个叉乘,效果\[ \boldsymbol a\times\boldsymbol b = \begin{vmatrix}\boldsymbol i & \boldsymbol j & \boldsymbol k \\ a_1&a_2&a_3\\b_1&b_2&b_3\end{vmatrix} \]参考:http://kkkkuingggg.5d6d.com/thread-69-1-1.html 另外,如果用 \bf 或 \textbf,则得到的是直立体的加粗。 比如 {\bf a}=(a_1,a_2,a_3) 或 \textbf{a}=(a_1,a_2,a_3) 都将得到 ${\bf a}=(a_1,a_2,a_3)$,文字尚可,但不太适合用于向量或矩阵。 公式中加下划线: \underline{需加下划线部分} 例: a+\underline{b}+c+\underline{d+e}+\underline{f+g+}h 效果\[ a+\underline{b}+c+\underline{d+e}+\underline{f+g+}h \] 公式中调节大小: 内容较多,详情见:http://kkkkuingggg.5d6d.com/thread-50-1-1.html
kuing 2# 2011-10-8 22:25
删除线是否需要?
kuing 3# 2011-11-10 19:11
顶一下
戊概念·五 4# 2011-11-10 19:21
2# kuing 删除线是???
只如初见 5# 2011-11-11 10:13
\colro{green}{kuing} 问题:怎么设置既加色又粗体。
只如初见 6# 2011-11-11 10:14
\color{green}{kuing}
只如初见 7# 2011-11-11 10:17
\color{green}{b+c}   \color{green}{kkkkingggg}
只如初见 8# 2011-11-11 10:18
\color{red}{b+c}
kuing 9# 2011-11-11 10:23
问题:怎么设置既加色又粗体。 只如初见 发表于 2011-11-11 10:13 \[\Large kkkk \color{red}{ kkkk \boldsymbol{kkkk} kkkk} kkkk\]
kuing 10# 2011-11-11 10:33
2# kuing 删除线是??? 戊概念·五 发表于 2011-11-10 19:21 这里没法做出效果 具体地,参考 http://blog.sina.com.cn/s/blog_77f93ba80100vstp.html 中间的图。
戊概念·五 11# 2011-11-11 11:29
10# kuing 懂了,O(∩_∩)O谢谢!
kuing 12# 2012-8-28 19:51
仅用作 MathJax 下的删除线: 先用 $\verb"\(\require{cancel}\)"$ 加载 cancel.js。\(\require{cancel}\) 然后用 \cancel{...}、\bcancel{...}、\xcancel{...}、\cancelto{...}{...} 来在数学公式作画删除线。 比如代码: \(\verb"$\cancel kuing + \cancel{kuing}+\bcancel{kuing}+\xcancel{kuing}"\) \(\verb"+\cancelto{kkkk}{kuing}+\cancelto kuing$"\) 显示 $\cancel kuing + \cancel{kuing}+\bcancel{kuing}+\xcancel{kuing}+\cancelto{kkkk}{kuing}+\cancelto kuing$
kuing 13# 2012-8-28 20:00
\[\xcancel{\sum\sqrt{\frac a{b+c}}>2}\]
thread-750-1-6.html: 请教一个公式
叶剑飞Victor 1# 2012-8-22 15:53
本帖最后由 叶剑飞Victor 于 2012-8-23 04:22 编辑 在$\triangle ABC$中,已知$A$、$B$、$C$的坐标分别是$A(x_1,\,y_1)$、$B(x_2,\,y_2)$、$C(x_3,\,y_3)$。 在$\triangle ABC$ 所在的平面中,有一个点$P$,并已知其坐标为$P(x_0,\,y_0)$。 是否有一个公式,能直接判断$P$点是否在$\triangle ABC$内? 是否有一个公式,能直接判断$P$点是否在$\triangle ABC$的边上? 是否有一个公式,能直接判断$P$点是否在$\triangle ABC$外?
kuing 2# 2012-8-22 16:10
没研究过,想了一下不知用有向面积行不行? PS、不必每处公式都加 \displaystyle 吧……
kuing 3# 2012-8-22 16:15
或者算 XA、XB、XC 两两夹角之和来判断?
海盗船长 4# 2012-8-22 17:51
线性规划
kuing 5# 2012-8-22 18:15
4# 海盗船长 具体怎么操作……
依然饭特稀 6# 2012-8-22 18:44
4# 海盗船长 写出约束条件代入?
都市侠影 7# 2012-8-22 19:41
本帖最后由 都市侠影 于 2012-8-22 21:26 编辑 这个问题有意思,我得到了一个结果,不过不能用一个公式那么简单,而是看三个式子的符号决定。 结果是这样的: ============================结果============================ 三角形的三个顶点假定为 $A(x_1,y_1),B(x_2,y_2),C(x_3,y_3)$ ,要判断的点是 $P(x_0,y_0)$,写出下面的三个表达式: \[ \left| \begin{array}{cc} x_3-x_1 & x_2-x_1 \\ y_3-y_1 & y_2-y_1 \end{array} \right| \cdot \left| \begin{array}{cc} x_0-x_1 & x_2-x_1 \\ y_0-y_1 & y_2-y_1 \end{array} \right| \] 以及 \[ \left| \begin{array}{cc} x_1-x_2 & x_2-x_3 \\ y_1-y_2 & y_2-y_3 \end{array} \right| \cdot \left| \begin{array}{cc} x_0-x_2 & x_2-x_3 \\ y_0-y_2 & y_2-y_3 \end{array} \right| \] 与 \[ \left| \begin{array}{cc} x_2-x_3 & x_1-x_3 \\ y_2-y_3 & y_1-y_3 \end{array} \right| \cdot \left| \begin{array}{cc} x_0-x_3 & x_1-x_3 \\ y_0-y_3 & y_1-y_3 \end{array} \right| \] 这里双竖线是行列式符号,可以简单的认为 \[ \left| \begin{array}{cc} a_{11} & a_{12} \\ a_{21} & a_{22} \end{array} \right| =a_{11}a_{22}-a_{12}a_{21} \] 根据这三个表达式的符号即可判断点 $P$ 的位置: 如果三个表达式全为正数,则点 $P$ 在三角形内 如果三个表达式中有两个为零,则点在三角形的边上,三个为零是不可能的。 如果三个表达式中有且只有一个为零,这时需要判断另外两个表达式的符号,如果均为正,则是在三角形的一条边上,若是一正一负,则是在一边的延长线上,即是在三角形外 如果三个表达式中有两正一负,则为三角形外 如果三个表达式中有两个负数,则为三角形外,不可能三个都是负数的,分析见后。 这就是结果,虽然可以把它们组合成一个式子来进行判别,但我觉得为了把它们组合成一个复杂表达式没啥意思,反而看不出来原理,所以这个结果我就已经满意了。 ============================原理============================ 现在说说它的原理,其实很简单,一条直线把平面会分成两半,两边的点代入直线的方程,它的符号是相反的,而直线上的点代入方程,它是等于0的。 如果点 $P$ 在三角形内,那么点 $P$与点 $A$ 位于直线 $BC$ 的同侧,$P$ 与 $B$ 位于直线 $AC$ 的同侧,$P$ 与 $C$ 位于直线 $AB$ 的同侧。 如果点 $P$ 在三角形上,那么点 $P$ 至少位于三条边中的一条上,当然也可能是两边,但是绝对不会同时位于三条边上,所以上面那三个表达式中最多有两个为零。 如果点 $P$ 在三角形外,那么画个图简单分析一下就可以得到上面的结果。
都市侠影 8# 2012-8-22 20:07
话说,上面这个结果尤其适合于程序实现
叶剑飞Victor 9# 2012-8-22 20:14
本帖最后由 叶剑飞Victor 于 2012-8-22 20:23 编辑 8# 都市侠影 话说,上面这个结果尤其适合于程序实现 话说这题就是编程题。 原题目链接 http://acm.hdu.edu.cn/showproblem.php?pid=4380 输入n,m。 接下来n行,输入n个坐标,表示可选的坐标。 接下来m行,输入m个坐标,表示goldstones的坐标。 从n个可选的点中,选出三个点。这三个点必须围住奇数个goldstones。问点有几种选择法。
都市侠影 10# 2012-8-22 20:25
本帖最后由 都市侠影 于 2012-8-22 21:26 编辑 发现了一个 bug,如果点 $P$ 在三角形外,那三个表达式中也是有可能有一个为零的,这时点在边的延长线上。 已经在楼上修正
海盗船长 11# 2012-8-22 20:32
5# kuing 嗯,就是zhcosin的那个方法
叶剑飞Victor 12# 2012-8-23 04:00
本帖最后由 叶剑飞Victor 于 2012-8-23 04:22 编辑 这题还可以用“有向面积”来计算。 若$\triangle PAB$、$\triangle PBC$、$\triangle PCA$,三个三角形的有向面积同号,则点$P$在$\triangle ABC$内。 否则,若三个三角形的有向面积至少有一个等于0,则,点$P$在$\triangle ABC$的边上。 否则,点$P$在$\triangle ABC$外。 P.S.   有向面积的定义: 已知$A(x_1,\,y_1)$、$B(x_2,\,y_2)$、$C(x_3,\,y_3)$,则$\triangle ABC$的有向面积是: \[ \frac{1}{2}\left|\begin{array}{ccc}x_1 & y_1 & 1 \\x_2 & y_2 & 1 \\x_3 & y_3 & 1\end{array}\right| \] 有向面积的性质: 一个三角形的有向面积的绝对值,等于这个三角形的面积。 若$\triangle ABC$有向面积为正,则$A \longrightarrow B \longrightarrow C$正向绕行(即逆时针方向)。 若$\triangle ABC$有向面积为负,则$A \longrightarrow B \longrightarrow C$负向绕行(即顺时针方向)。
kuing 13# 2012-8-23 04:53
12# 叶剑飞Victor 嘻嘻…我第一反应就是有向面积   只是后来没细想了,懒了
都市侠影 14# 2012-8-23 10:08
12# 叶剑飞Victor 哈哈,你也犯了一个跟我一样的错误,当三个中有一个为零时,这个点也有可能是在边的延长线上,即是在三角形外。
kuing 15# 2012-8-23 14:35
或者算 XA、XB、XC 两两夹角之和来判断? kuing 发表于 2012-8-22 16:15 这个没人写?那我来续一下吧…… 输入 $A$、$B$、$C$、$X$ 的坐标; 计算 $S_{\triangle ABC}$ 并判断是否为 $0$,如果为 $0$,报错重新输入,否则继续; 令 $x=\overrightarrow{XA}$、$y=\overrightarrow{XB}$、$z=\overrightarrow{XC}$; 判断 $x$、$y$、$z$ 中是否有 $\overrightarrow0$,如果有,则报 $X$ 在三角形顶点上并结束,否则继续; 令 $\alpha=\arccos\dfrac{y\cdot z}{|y|\cdot |z|}$、$\beta=\arccos\dfrac{z\cdot x}{|z|\cdot |x|}$、$\gamma=\arccos\dfrac{x\cdot y}{|x|\cdot |y|}$; 判断 $\alpha$、$\beta$、$\gamma$ 中是否有 $0$,如果有,则报 $X$ 在三角形的边的延长线上,否则继续; 判断 $\alpha$、$\beta$、$\gamma$ 中是否有 $\pi$,如果有,则报 $X$ 在三角形的边上(不含顶点),否则继续; 判断 $\alpha+\beta+\gamma$ 是否等于 $\pi$,如果相等,则报 $X$ 在三角形内,否则报 $X$ 在三角形外并结束。 理论上是这样的,但这里用了 $\arccos$ 并且最后判断时还要与 $\pi$ 比较,可能容易产生误差…… 这里其实还可以更细化地将具体在哪个顶点或哪条边上也给出。具体要翻译成计算机语言我就不懂了……
kuing 16# 2012-8-23 15:28
15# kuing 貌似凸N边形也可以这样玩
叶剑飞Victor 17# 2012-8-23 15:55
本帖最后由 叶剑飞Victor 于 2012-8-23 16:13 编辑 15# kuing 令 $x=\overrightarrow{XA}$、$y=\overrightarrow{XB}$、$z=\overrightarrow{XC}$ 单个小写英文字母表示向量的时候,印刷体要用粗体字表示,例如$\boldsymbol{x}$;手写体要在上面写个箭号,例如$\overrightarrow{x}$。
kuing 18# 2012-8-23 16:19
17# 叶剑飞Victor 这我当然是知道的(see http://kkkkuingggg.5d6d.net/thread-69-1-1.html),只是我最近懒么,如果要打粗斜体的话,还不如不令,直接用两个字母的向量还省些输入量……
叶剑飞Victor 19# 2012-8-23 16:50
18# kuing 其实可以打“\vec x”(显示成$\vec x$),三个字母而已。 P.S. “\vec”就是vector(向量)的意思。
kuing 20# 2012-8-23 17:09
19# 叶剑飞Victor 这个我这里有点怪,\vec 在 firefox 里显示正常,用 IE 或 chrome 都会显示有问题,不知为什么。 IE: chrome: 也不知别人的是不是这样,所以我一直就没用它。
thread-750-2-6.html:
叶剑飞Victor 21# 2012-8-23 17:24
本帖最后由 叶剑飞Victor 于 2012-8-23 17:27 编辑 20# kuing 我这里都显示的好好的。 Firefox(HTML+CSS显示) Firefox(MathML显示) Google Chrome(HTML+CSS显示)   P.S. 我这里是Linux操作系统,没有IE浏览器。没法测试IE上的显示效果。
kuing 22# 2012-8-23 17:26
有空更新一下chrome先
叶剑飞Victor 23# 2012-8-23 18:31
本帖最后由 叶剑飞Victor 于 2012-8-23 18:33 编辑 22# kuing 在Windows操作系统中测试了一下: Firefox(HTML+CSS显示): Firefox(MathML显示):   IE(HTML+CSS显示):   IE(MathPlayer插件+MathML显示):   猎豹浏览器,极速(Chrome)模式(HTML+CSS显示):
kuing 24# 2012-8-24 01:45
23# 叶剑飞Victor 刚才更新了一下 chrome,还是一样的显示。 由你上面的测试看来还跟操作系统有关
海盗船长 25# 2012-8-24 11:18
20# kuing 我用ie显示也是这样的效果。。
kuing 26# 2012-8-24 11:28
25# 海盗船长 是哩所以我一直没用它
hhhzh7241hzh 27# 2012-8-25 16:45
本帖最后由 hhhzh7241hzh 于 2012-8-25 17:46 编辑 用向量。先用待定系数法求得(x0, y0)=A*(x1, y1)+B*(x2, y2)+C*(x3, y3),其中A+B+C=1。 若A,B,C均大于0,则在△ABC内; 若两个大于0,一个等于0,则在△ABC的边上; 若两个等于0,一个等于1,则在△ABC的顶点上; 若非上述三种情况,则在△ABC外。
kuing 28# 2012-8-25 17:58
27# hhhzh7241hzh 看着很神奇,但理论一时没反应过来,求原理解释……
hhhzh7241hzh 29# 2012-8-25 18:28
本帖最后由 hhhzh7241hzh 于 2012-8-25 18:32 编辑 原理 这个定理虽然是“空间向量”问题,但也适用于“平面向量”问题。
thread-751-1-6.html: [不等式] 下午一中在群里发的不等式
kuing 1# 2012-8-22 20:22
题目:设 $a$, $b$, $c\geqslant0$,且 $a+b+c=1$,求证 \[\frac1{a+b}+\frac1{b+c}+\frac1{c+a}+6(ab+bc+ca)\geqslant\frac{13}2.\] 以下是部分聊天记录: 爱好者-悠闲数学娱乐论坛?(4396*****)  17:32:05 发论坛看看? 一中(3175*****)  17:47:07 这个题貌似蛮水,所以就没发了 爱好者-悠闲数学娱乐论坛?(4396*****)  17:52:20 水吗?我没看出来 爱好者-悠闲数学娱乐论坛?(4396*****)  18:04:12 可以写成nessbit的加强式 \[\sum\frac c{a+b}\geqslant\frac32+\frac{\sum(a-b)^2}{\left( \sum a \right)^2}\] Albert Du(8635****)  18:05:00 http://www.wl84.com/forum.php?mod=viewthread&tid=63#lastpost 爱好者-悠闲数学娱乐论坛?(4396*****)  18:05:57 这么叉暴力 我还是没看出来怎么水了…… 即使写成那个加强式,也没发现有水的地方 一中(3175*****)  18:07:41 sos下    爱好者-悠闲数学娱乐论坛?(4396*****)  18:08:06 Albert Du(8635****)  18:10:02 我以前好像还问过这个 爱好者-悠闲数学娱乐论坛?(4396*****)  18:14:07 \[\sum(a+b)(a^2+b^2-c^2)(a-b)^2\geqslant0\] 一中(3175*****)  18:16:24 小k发飚了    爱好者-悠闲数学娱乐论坛?(4396*****)  18:17:27 然后我在查SOS定理…… 爱好者-悠闲数学娱乐论坛?(4396*****)  18:23:10 噢,用最简单那个SOS定理好像就OK了,还居然保住了两个取等条件 一中(3175*****)  18:24:35 小k又v5了    爱好者-悠闲数学娱乐论坛?(4396*****)  18:26:06 哎,其实我刚才是觉得SOS可能不容易玩下去,因为这个不等式有两个取等条件,除了全部相等之外,还有一个是0,k,k…… 一中(3175*****)  18:26:38 和为1了塞    恩,两个取等情况    爱好者-悠闲数学娱乐论坛?(4396*****)  18:27:46 先煮饭……回头整理一下。 一中(3175*****)  18:28:17 吃饱塞   
kuing 2# 2012-8-22 20:40
具体地,就是 \begin{align*} \sum \frac1{a+b}+6\sum ab\geqslant \frac{13}2&\iff\sum \frac{a+b+c}{a+b}+\frac{6\sum ab}{\left( \sum a \right)^2}\geqslant \frac{13}2 \\ &\iff\sum \frac c{a+b}\geqslant \frac72-\frac{6\sum ab}{\left( \sum a \right)^2} \\ &\iff\sum \frac c{a+b}\geqslant \frac32+\frac{\sum (a-b)^2}{\left( \sum a \right)^2}, \end{align*} 熟知 \[\sum \frac c{a+b}-\frac32=\sum \frac{(a-b)^2}{2(b+c)(c+a)},\] 所以 \begin{align*} \sum \frac c{a+b}\geqslant \frac32+\frac{\sum (a-b)^2}{\left( \sum a \right)^2}&\iff\sum \frac{(a-b)^2}{2(b+c)(c+a)}\geqslant \frac{\sum (a-b)^2}{\left( \sum a \right)^2} \\ &\iff\sum \left( \frac1{2(b+c)(c+a)}-\frac1{(a+b+c)^2} \right)(a-b)^2\geqslant 0 \\ &\iff\sum \frac{(a^2+b^2-c^2)(a-b)^2}{2(b+c)(c+a)(a+b+c)^2}\geqslant 0 \\ &\iff\sum (a+b)(a^2+b^2-c^2)(a-b)^2\geqslant 0, \end{align*} 不妨设 $a\geqslant b\geqslant c$,则 \begin{align*} &\sum (a+b)(a^2+b^2-c^2)(a-b)^2 \\ \geqslant{}& (b+c)(b^2+c^2-a^2)(b-c)^2+(c+a)(c^2+a^2-b^2)(b-c)^2 \\ ={}&\bigl((a-b)^2(a+b)+(a+b+2c)c^2\bigr)(b-c)^2 \\ \geqslant{}& 0, \end{align*} 等号成立当 $a=b=c=1/3$ 或 $a=b=1/2$, $c=0$。
kuing 3# 2012-8-22 21:30
其实一中刚发题时,我还考虑过下面的证法,当进行到后面的计算时,一中说题目比较水,然后我就停下来重新想其他方向,直到一中说SOS,我就试着SOS出了上面的结果。 现在我还是把刚开始的想法给做完了,顺便发上来。 不妨设 $a\geqslant b\geqslant c$,令 $b+c=x$, $c+a=y$, $a+b=z$,则 $x\leqslant y\leqslant z$, $x+y+z=2$,则 $z\geqslant 2/3$。由 \[\sum (a+b)(b+c)=(a+b+c)^2+ab+bc+ca,\] 得 \[ab+bc+ca=xy+yz+zx-1,\] 于是 \begin{align*} \sum \frac1{a+b}+6\sum ab\geqslant \frac{13}2&\iff\sum \frac1x+6\sum xy\geqslant \frac{25}2 \\ & \iff\left( \frac1{xyz}+6 \right)(xy+yz+zx)\geqslant \frac{25}2 \\ & \iff\left( \frac1{tz}+6 \right)\bigl(t+z(2-z)\bigr)\geqslant \frac{25}2, \end{align*} 其中 \[t=xy\leqslant \left( \frac{x+y}2 \right)^2=\left( \frac{2-z}2 \right)^2,\] 令 \[f(t)=\left( \frac1{tz}+6 \right)\bigl(t+z(2-z)\bigr),\] 则 \[f'(t)=\frac{6t^2+z-2}{t^2},\] 而 \[6t^2+z-2\leqslant 6\left( \frac{2-z}2 \right)^4+z-2=(2-z)\left( 3\left( 1-\frac z2 \right)^3-1 \right),\] 且由 $z\geqslant 2/3$ 得 \[3\left( 1-\frac z2 \right)^3-1\leqslant 3\left( 1-\frac13 \right)^3-1=-\frac19<0,\] 所以 $f'(t)\leqslant 0$,由此得到 \[f(t)\geqslant f\left( \left( \frac{2-z}2 \right)^2 \right)=\left( \frac4{(2-z)^2z}+6 \right)\left( \frac{(2-z)^2}4+z(2-z) \right),\] 又 \[\left( \frac4{(2-z)^2z}+6 \right)\left( \frac{(2-z)^2}4+z(2-z) \right)-\frac{25}2=\frac{(z-1)^2(3z-2)^2}{2z(2-z)}\geqslant 0,\] 所以原不等式成立,等号成立当 $x=y=z=2/3$ 或 $x=y=1/2$, $z=1$,即 $a=b=c=1/3$ 或 $a=b=1/2$, $c=0$。
海盗船长 4# 2012-8-22 22:28
原来我以前问过的类似的另一边
kuing 5# 2012-8-22 22:57
4# 海盗船长 这个网刊第一期我有提及过(P33~34),其实下午我变形到那个加强式之后我就去看了下那里,发现我当时写的时候只是随便放了一下得到了那个加强和反向,比一楼那个弱。
kuing 6# 2012-8-23 00:25
4# 海盗船长 但是这个随手放出来的反向,那个系数竟然没法再小了貌似
yizhong 7# 2012-8-23 19:26
本帖最后由 yizhong 于 2012-8-23 19:43 编辑 这个题还可以用调整法来解决塞 ,不多说了,开调吧,我们设a为a,b,c之中三者最小。 又记:$F(a,b,c)=\sum \dfrac{1}{a+b}+\sum 6ab-\dfrac{13}{2}$,$F(a,b,c)-F(a, \dfrac{b+c}{2}, \dfrac{b+c}{2}) =(b-c)^2(\dfrac{1}{(2a+b+c)(a+c)(a+b)}-\dfrac{1}{4})$,又因为由A-G:$(2a+b+c)(a+c)(a+b)\leqslant(2a+b+c)(\dfrac{(2a+b+c)^2}{2}) =\dfrac{(1+a)^3}{4}<4$ 所以我们就有:$F(a,b,c)\geqslant F(a,t,t)=(3t-1)^2\dfrac{(2t-1)^2}{2t(1-t)}\geqslant0$ 证毕。
kuing 8# 2012-8-23 19:43
7# yizhong 嗯,次数不高,调整还是比较好做的。
kuing 9# 2012-8-23 19:47
顺便说一下,三楼的证法中,换元之后 x, y, z 本应满足构成三角形的条件,但由后面的证明可以看出,x, y, z 只需要是正数而并不需要构成三角形也能使不等式成立,这样看来,原不等式的条件可以弱化,具体地说就是弱化成“设 $a$, $b$, $c\in\mathbb R$ 满足 $a+b+c=1$ 且 $a+b>0$, $b+c>0$, $c+a>0$,求证……”
yizhong 10# 2012-8-23 19:59
本帖最后由 yizhong 于 2012-8-23 20:01 编辑 其实一开始我也木有想到用调整法来做,我一看题目含有:$\sum \dfrac{c}{a+b}, \sum ab$ 再由条件之和为1,可以变出$\sum a^2$出来,所以这才让我坚定用SOS(虽然可以得知题目的取等是两种情况,但是觉得SOS可行) 想到调整法那是因为(有见过调整法解题的就很熟悉,在和为1或者乘积为1的时候常这样做)所以由于题目所给的条件,所以这才用 调整法又做了下。
海盗船长 11# 2012-8-23 20:38
哇,调整法很简洁
thread-752-1-6.html: [函数] 某群的一周期函数小问
kuing 1# 2012-8-23 00:31
如何证明 $\sin(x^2+x)$ 不是周期函数?
kuing 2# 2012-8-23 01:16
哎,这些显然的东西总是不好证明,不知下面这样说行不行? 假设 $f(x)=\sin(x^2+x)$ 为周期函数,设其周期为 $T>0$。 我们来计算 $f(x)$ 的零点,为 \[x^2+x=k\pi\iff x=\frac{\pm\sqrt{4k\pi+1}-1}2,\] 其中 $k\in\mathbb Z$。且考虑 $x\geqslant 0$ 的由小到大排列的所有零点 $x_k=\bigl(\sqrt{4k\pi+1}-1\bigr)/2$($k\in\mathbb N$),考察相邻两零点间的距离,为 \[x_{k+1}-x_k=\frac{\sqrt{4(k+1)\pi+1}-1}2-\frac{\sqrt{4k\pi+1}-1}2=\frac{2\pi}{\sqrt{4(k+1)\pi+1}+\sqrt{4k\pi+1}},\] 可见此距离随 $k$ 的增大而严格减少,因此,必存在正整数 $m$,使 $f(x)$ 在区间 $[0,T]$ 上的零点个数比区间 $[mT,(m+1)T]$ 上的要少,这与 $f(x)$ 以 $T$ 为周期矛盾。 感觉还是缺一点数学语言的刻画。
kuing 3# 2012-8-23 14:12
该群里有人回复如下 其实我看不懂。
nash 4# 2012-8-23 19:07
经常回帖,混个脸熟
kuing 5# 2012-8-23 19:14
4# nash 还是看不懂
kuing 6# 2012-8-23 19:35
“$\sin f(x) = \sin g(x)$ 恒成立” 得到 “$f(x)=g(x)+2k\pi$ 恒成立” 或 “$f(x)=\pi-g(x)+2k\pi$ 恒成立” ?
nash 7# 2012-8-23 20:11
6# kuing 嗯,是呀。
yizhong 8# 2012-8-23 20:16
此类问题的做法大都是反证法(或说通法),还常常用到无理数=有理数的矛盾
kuing 9# 2012-8-23 20:34
7# nash 确信能得到?
海盗船长 10# 2012-8-23 20:36

kuing 11# 2012-8-23 20:46
10# 海盗船长 指点一下啊,就一个表情我无法理解啊……
海盗船长 12# 2012-8-23 21:34
11# kuing 感觉不是很严谨啊,,应该是两个都有可能恒成立,不能直接这样分成两种情形
海盗船长 13# 2012-8-23 21:36
嗯直接取x为有理数好像也可以出矛盾
kuing 14# 2012-8-23 21:47
转群里40的: 地狱的死灵  21:32:18   那题证明不是周期函数可不可以直接证明它的导函数不是周期函数?   直接在导函数中找个点列,   使得导函数在这个点列上是递增的 爱好者-悠闲数学娱乐论坛?  21:34:55   应该可以的导数的想法不错,咋我没想到…… 地狱的死灵  21:36:37   
海盗船长 15# 2012-8-23 21:52
哦,导数方法不错
icesheep 16# 2012-8-27 21:13
一种想法是连续的周期函数是一致连续的,所以只要证明这个函数非一致连续即可。
kuing 17# 2012-8-27 21:19
16# icesheep 连续的周期函数是一致连续的 这个结论怎么证哟?
kuing 18# 2012-8-27 21:25
噢,好像在闭区间连续就…… 我对一致连续不太熟悉的说……
icesheep 19# 2012-8-27 21:28
有界闭区间上的连续函数是一致连续的(有限覆盖原理) 于是该连续函数在两个周期内的闭区间中一定是一致连续的,显然由周期性这个公共的 delta 可以推广到整个 R 上。
thread-753-1-2.html: 原来“压轴”是倒数第二……
kuing 1# 2012-8-23 01:52
RT详情请百度或查阅词典等……
nash 2# 2012-8-23 20:35
以前还以为压轴就是最后一题 和那个“名落孙山”差不多啦…
kuing 3# 2012-8-23 20:49
2# nash 相信绝大部分人这样认为……
海盗船长 4# 2012-8-23 20:54
嗯,高中语文还经常考这种词的意思,,,
thread-754-1-1.html: 看看能不能用LaTeX画图
叶剑飞Victor 1# 2012-8-23 17:18
\[ \setlength{\unitlength}{5cm} \begin{picture}(1,1) \put(0,0){\line(0,1){1}} \put(0,0){\line(1,0){1}} \put(0,0){\line(1,1){1}} \put(0,0){\line(1,2){.5}} \put(0,0){\line(1,3){.3333}} \put(0,0){\line(1,4){.25}} \put(0,0){\line(1,5){.2}} \put(0,0){\line(1,6){.1667}} \put(0,0){\line(2,1){1}} \put(0,0){\line(2,3){.6667}} \put(0,0){\line(2,5){.4}} \put(0,0){\line(3,1){1}} \put(0,0){\line(3,2){1}} \put(0,0){\line(3,4){.75}} \put(0,0){\line(3,5){.6}} \put(0,0){\line(4,1){1}} \put(0,0){\line(4,3){1}} \put(0,0){\line(4,5){.8}} \put(0,0){\line(5,1){1}} \put(0,0){\line(5,2){1}} \put(0,0){\line(5,3){1}} \put(0,0){\line(5,4){1}} \put(0,0){\line(5,6){.8333}} \put(0,0){\line(6,1){1}} \put(0,0){\line(6,5){1}} \end{picture} \]
叶剑飞Victor 2# 2012-8-23 17:19
看来不能。。。。
kuing 3# 2012-8-23 17:33
能就 niubility 了 mathlinks 上能,不过 mathlinks 是生成图片再上来的,跟 mathjax 的处理不一样
thread-755-1-6.html: [组合] 一道排列组合题
nash 1# 2012-8-23 23:15
本帖最后由 nash 于 2012-8-23 23:26 编辑 RT 求前100个正整数构成的集合中,元素和为5的倍数的子集个数
海盗船长 2# 2012-8-24 11:15
令 $$f(x)=2^{20} (1+x)^{40}(1+x^2)^{40}$$ 则有$$N=\frac{1}{5}(f(1)+f(\alpha)+f(\alpha^2)+f(\alpha^3)+f(\alpha^4))-1=253530120045645880299341479935$$个非空子集满足要求。 其中$\alpha=\exp\left(\frac{2\pi i}{5}\right)$
kuing 3# 2012-8-24 11:23
2# 海盗船长 这么大
海盗船长 4# 2012-8-24 11:45
3# kuing 嗯,和$\frac{2^{100}-1}{5}=253530120045645880299340641075$很接近
isea 5# 2012-8-24 14:05
2# 海盗船长 能“初始化”一下么?
kuing 6# 2012-8-24 14:10
5# isea 我也看不懂
海盗船长 7# 2012-8-24 17:41
本帖最后由 海盗船长 于 2012-8-24 17:42 编辑 其实就是用的母函数,先把$1$到$100$按模$5$分成$5$类:$-2,-1,0,1,2\pmod 5$,每一类含有$20$个数后考虑$$g(x)=\left(1+\frac{1}{x^2}\right)^{20}\left(1+\frac{1}{x}\right)^{20}(1+1)^{20}(1+x)^{20}(1+x^2)^{20}$$中$x^{5k}\;(k \in \mathbb{Z})$项系数之和,再用$5$次单位根把这些项分离出来即可。
kuing 8# 2012-8-24 18:06
好像大概基本上差不多应该算是可以说看懂了吧……niubility solution
thread-756-1-5.html: [不等式] 来自群的一道简单最值
kuing 1# 2012-8-24 14:05
爱好者-风度(5120*****) 11:45:17 设实数 $x$, $y\geqslant1/2$,求 $f(x,y)=x^3+y^3+x^2y+xy^2-3(x^2+y^2+xy)+3(x+y)$ 的最小值。 令 $p=x+y$, $q=xy$,则由均值有 $p^2\geqslant4q$,由条件有 $p\geqslant1$, $q\geqslant1/4$,以及 $(2x-1)(2y-1)\geqslant0$ 展开得到 $4q+1\geqslant2p$。 将 $f(x,y)$ 整理为 \[f(x,y)=g(p,q)=p^3-3p^2+3p+(3-2p)q,\] 如果 $p\geqslant3/2$,则 \[g(p,q)\geqslant p^3-3p^2+3p+(3-2p)\frac{p^2}4=\frac14(p-2)^2(2p-1)+1\geqslant1,\] 当 $x=y=1$ 时取等号; 如果 $p<3/2$,则 \[g(p,q)\geqslant p^3-3p^2+3p+(3-2p)\frac{2p-1}4=\frac18(3-2p)\bigl(2(p-1)(3-2p)+1\bigr)+\frac98>\frac98>1.\] 缩上知 $f(x,y)$ 的最小值为 $1$。
yes94 2# 2012-8-25 17:34

kuing 3# 2012-8-25 17:39
2# yes94 这个表情其实可以搜出来不用上传图片的 就是要翻几页 不过前面也有不少选择
Gauss门徒 4# 2012-12-15 06:40
关键还是$p$的上界和下界
thread-757-1-1.html: Mathematica2PDF
骨灰级爱好者 1# 2012-8-24 19:27
Mathematica的代码能不能转换成MathLink格式,直接复制黏贴
kuing 2# 2012-8-24 19:56
什么叫“MathLink格式”? PS、图中的题目在 http://kkkkuingggg.5d6d.net/thread-85-1-1.html 讨论过。
thread-758-1-1.html: 长公式测试
kuing 1# 2012-8-24 20:00
行内:$a=b=c=d=e=f=g=h=i=j=k=l=m=n=o=p=q=r=s=t=u=v=w=x=y=z$,行内:$a=b=c=d=e=f=g=h=i=j=k=l=m=n=o=p=q=r=s=t=u=v=w=x=y=z$,行内:$a=b=c=d=e=f=g=h=i=j=k=l=m=n=o=p=q=r=s=t=u=v=w=x=y=z$, 行内:$a=b=c=d=e=f=g=h=i=j=k=l=m=n=o=p=q=r=s=t=u=v=w=x=y=z$, 行内:$a=b=c=d=e=f=g=h=i=j=k=l=m=n=o=p=q=r=s=t=u=v=w=x=y=z=a=b=c=d=e=f=g=h=i=j=k=l=m=n=o=p=q=r=s=t=u=v=w=x=y=z$。 行间: \[a=b=c=d=e=f=g=h=i=j=k=l=m=n=o=p=q=r=s=t=u=v=w=x=y=z\] 行间: \[a=b=c=d=e=f=g=h=i=j=k=l=m=n=o=p=q=r=s=t=u=v=w=x=y=z=a=b=c=d=e=f=g=h=i=j=k=l=m=n=o=p=q=r=s=t=u=v=w=x=y=z\] 行间: \[\left(\sqrt{\frac a{b+c}}\right)+\left(\sqrt{\frac a{b+c}}\right)+\left(\sqrt{\frac a{b+c}}\right)+\left(\sqrt{\frac a{b+c}}\right)+\left(\sqrt{\frac a{b+c}}\right)+\left(\sqrt{\frac a{b+c}}\right)+\left(\sqrt{\frac a{b+c}}\right)+\left(\sqrt{\frac a{b+c}}\right)+\left(\sqrt{\frac a{b+c}}\right)\] 行间: \[\left(\sqrt{\frac a{b+c}}+\sqrt{\frac a{b+c}}+\sqrt{\frac a{b+c}}\right)+\left(\sqrt{\frac a{b+c}}+\sqrt{\frac a{b+c}}+\sqrt{\frac a{b+c}}\right)+\left(\sqrt{\frac a{b+c}}+\sqrt{\frac a{b+c}}+\sqrt{\frac a{b+c}}\right)\]
kuing 2# 2012-8-24 20:03
断行断行断行断行断行断行断行断行断行断行断行断行断行断行断行断行 $a+b+c+d+e+f+g+h+i$ 断行断行断行断行断行断行断行断行断行断行断行断行断行断行断行断行断行断行断行断行 $a+b+c+d+e+f+g+h+i$
kuing 3# 2012-8-24 20:16
align 的: 右边长: \begin{align*} a+b+c&=d,\\ e&=f+g+h,\\ a&=b+c+d+e+f+g+h+i+j+k+l+m+n+o+p+q+r+s+t+u+v+w+x+y+z,\\ f&=k,\\ a&=b+c+d+e+f+g+h+i+j+k+l+m+n+o+p+q+r+s+t+u+v+w+x+y+z+f**************k,\\ c&=u. \end{align*} 左边长: \begin{align*} a+b+c&=d,\\ e&=f+g+h,\\ a+b+c+d+e+f+g+h+i+j+k+l+m+n+o+p+q+r+s+t+u+v+w+x+y&=z,\\ f&=k,\\ f*********k+a+b+c+d+e+f+g+h+i+j+k+l+m+n+o+p+q+r+s+t+u+v+w+x+y&=z,\\ c&=u. \end{align*} 左右都长: \begin{align*} a+b+c&=d,\\ e&=f+g+h,\\ a+b+c+d+e+f+g+h+i+j+k+l+m+n+o+p+q+r+s+t+u+v+w+x+y&=z,\\ a&=b+c+d+e+f+g+h+i+j+k+l+m+n+o+p+q+r+s+t+u+v+w+x+y+z,\\ f*********k+a+b+c+d+e+f+g+h+i+j+k+l+m+n+o+p+q+r+s+t+u+v+w+x+y&=z, \end{align*}
kuing 4# 2012-8-24 20:23
3# kuing 这个就不太行了
kuing 5# 2012-8-25 14:29
或者说,总的来说就只有单行的行间公式还有点用
thread-759-1-6.html: [不等式] 来自群的简单半对称三角不等式
kuing 1# 2012-8-25 16:49
爱好者-rpgzysb(6492*****) 三角形中 \[(\sin B+\sin C)(\sin A+\sin C)\ge (\sin 2B+\sin A)(\sin 2A+\sin B).\] 等腰取等。 \begin{align*} &\iff (\sin B+\sin (A+B))(\sin A+\sin (A+B))\geqslant (\sin 2B+\sin A)(\sin 2A+\sin B) \\ &\iff 4\cos \frac A2\sin \frac{A+2B}2\cos \frac B2\sin \frac{2A+B}2\geqslant 4\cos \frac{A-2B}2\sin \frac{A+2B}2\cos \frac{2A-B}2\sin \frac{2A+B}2 \\ &\iff 2\cos \frac A2\cos \frac B2\geqslant 2\cos \frac{A-2B}2\cos \frac{2A-B}2 \\ &\iff \cos \frac{A+B}2+\cos \frac{A-B}2\geqslant \cos \frac{3A-3B}2+\cos \frac{A+B}2 \\ &\iff \cos \frac{A-B}2\geqslant 4\cos^3\frac{A-B}2-3\cos \frac{A-B}2 \\ &\iff 4\cos \frac{A-B}2\sin^2\frac{A-B}2\geqslant 0. \end{align*}
kuing 2# 2012-8-25 17:24
最后其实也可以不用三倍角公式,继续又和差化简也是可以做的。 \begin{align*} \cdots & \iff \cos \frac{A-B}2-\cos \frac{3A-3B}2\geqslant 0 \\ & \iff-2\sin (A-B)\sin \frac{-A+B}2\geqslant 0 \\ & \iff4\sin^2\frac{A-B}2\cos \frac{A-B}2\geqslant 0. \end{align*} oh,这个题作为和差化积与积化和差的习题真是个好选择……
yes94 3# 2012-8-25 17:26
thread-76-1-1.html: hspace不知行不行测试一下
kuing 1# 2011-10-8 23:30
$a \hspace{1em} b$ \hspace 行,加上 \underline 作填空用的那种横线: $a^2+b^2 \underline{\hspace{2em}} 2ab$ $a^2+b^2 \underline{\qquad} 2ab$ $a^2+b^2 \underline{        } 2ab$ 可以用,不过目前本论坛中的连续空格并不会自动忽略,所以暂时还不太需要用了。。。 当然,我个人还是建议用,因为这方便直接“移植”到latex中去而不用再修改,另一方面可能以后会有改进至能忽略空格 vspace 呢? $a \vspace{1ex} b$ 也是意料之中,不行
zwl1972 2# 2011-10-12 16:02
本帖最后由 zwl1972 于 2011-10-12 16:14 编辑 测试公式:$\displaystyle\sum_{cyc}a^2\ge\displaystyle\sum_{cyc}bc$.
kuing 3# 2011-10-12 16:06
测试公式:$\displaystyle\sum_{cyc}a^2\ge\displaystyle\sum_{cyc}a$. zwl1972 发表于 2011-10-12 16:02 第二个 \displaystyle 不必加,第一个 \displaystyle 可以影响到后面。在同一个公式中,最前面用一个就够了。
thread-760-1-6.html: [不等式] 一道不等式
v6mm131 1# 2012-8-25 21:00
证明:$\ln x\ln(1-x)<\sqrt{x(1-x)}$ 学会了发题
kuing 2# 2012-8-25 21:10
1# v6mm131 你甚至可以将公式放在贴的标题上,这样从外面就可以看到这个不等式。
kuing 3# 2012-8-25 22:19
理论上可能大概应该估计差不多基本上会有下面这个吧 \[\ln x\ln(1-x)\leqslant\bigl(x(1-x)\bigr)^{-\frac{\ln(\ln2)}{\ln2}}\] 其中,指数 \[-\frac{\ln(\ln2)}{\ln2}\approx0.528766\]
kuing 4# 2012-8-26 02:52
刚才的聊天记录
kuing 5# 2012-8-26 03:31
突然想到了,咳,洗完澡精神了果然灵感也好点。 只要证当 $0<x<1$ 时恒有 \[\ln x>\frac{x-1}{\sqrt x},\] 这用导数证明是极易的,此处从略。再在上式令 $x\to1-x$ 得 \[\ln(1-x)>\frac{-x}{\sqrt{1-x}},\] 由于它们都是负的,所以两式相乘即得到原不等式。 这也让我明白了为什么上面聊天记录中提到原题没给铺垫而显得突兀,显然是因为如果给了铺垫,提示了上面的局部不等式(哪怕是变形后的),那样题目就太简单了,对不住这样一个如此好看的不等式了,呵呵……
kuing 6# 2012-8-26 04:20
好了,原题解决了,顺便回过头来看看我3#随手搞的指数加强。 理论上可能大概应该估计差不多基本上会有下面这个吧 \[\ln x\ln(1-x)\leqslant\bigl(x(1-x)\bigr)^{-\frac{\ln(\ln2)}{\ln2}}\] kuing 发表于 2012-8-25 22:19 可以说,这个加强如果成立,那么该指数是最佳的了,那是因为上述不等式能在 $x=0.5$ 时取等。 为方便书写,我们记那个指数 $-\frac{\ln(\ln2)}{\ln2}=\alpha$,并且以下的 $x$ 默认为 $(0,1)$ 内的变量。 既然有了上面这样的局部不等式思路,即管模仿下,但是指数复杂了,照搬一次和根号大概不行,那我们在局部不等式中应配以什么指数?就待定好了,仿上我们需要适当的 $p$, $q$ 使 \[\ln x\geqslant\frac{-(1-x)^p}{x^q}\] 恒成立,然后令 $x\to1-x$ 并相乘后为 \[\ln x\ln(1-x)\leqslant\bigl(x(1-x)\bigr)^{p-q},\] 故应有 $p-q=\alpha$,亦即需要适当的 $q$ 使 \[f(x)=\ln x+\frac{(1-x)^{q+\alpha}}{x^q}\geqslant0\] 恒成立,注意到 $f(0.5)=0$(由于指数最佳,此等式是必然的),那么我们自然也需要 $f'(0.5)=0$,否则就不会恒成立了,经过具体计算,由 $f'(0.5)=0$ 可以得到 \[q=\frac1{2\ln2}-\frac\alpha2,\] 于是,我们需要的局部不等式是 \[\ln x\geqslant\frac{-(1-x)^{\frac1{2\ln2}+\frac\alpha2}}{x^{\frac1{2\ln2}-\frac\alpha2}},\] 即 \begin{equation}\label{20120826v6lnbdszjxsjb} \ln x\geqslant\frac{-(1-x)^{\frac{1-\ln(\ln2)}{2\ln2}}}{x^{\frac{1+\ln(\ln2)}{2\ln2}}}, \end{equation} 如果式 \eqref{20120826v6lnbdszjxsjb} 成立,那么上述加强式就获证,如果不成立,那么就白干! 时间关系,很晚了,我还是有点撑不住了,明天待续。
thread-761-1-6.html: [不等式] 网友又问不等式,三角形绝对值
kuing 1# 2012-8-25 23:18
月中影  22:36:32 三角形ABC的三条边长为a,b,c,证明:   |a^2-b^2|/c+|b^2-c^2|/a>=|c^2-a^2|/b 打成 $\LaTeX$ 代码先: $\triangle ABC$ 的三条边长为 $a$, $b$, $c$,证明 \[\frac{|a^2-b^2|}c+\frac{|b^2-c^2|}a\geqslant\frac{|c^2-a^2|}b.\] 话说这个题我除了分类讨论之外没什么想法(分三类,分别去绝对值之后因式分解,没什么技术),不知你们有没有一次过能搞定的简洁办法?
kuing 2# 2012-8-26 23:40
分类的具体做法如下。 由于交换 $a$, $c$ 二者的位置,不等式不变,所以不妨设 $a\geqslant c$,然后按 $b$ 与 $a$, $c$ 的大小比较分成三类,分别去绝对值后作因式分解,得到下表。 \begin{array}{|c|c|} \hline 情况 & 原不等式等价于 \\ \hline a\geqslant b\geqslant c & \begin{aligned} & \iff\frac{a^2-b^2}c+\frac{b^2-c^2}a\geqslant \frac{a^2-c^2}b \\ & \iff\frac{(a-b)(b-c)(a-c)(a+b+c)}{abc}\geqslant 0 \end{aligned} \\ \hline b\geqslant a\geqslant c & \begin{aligned} & \iff\frac{b^2-a^2}c+\frac{b^2-c^2}a\geqslant \frac{a^2-c^2}b \\ & \iff\frac{(b-a)(a+c)(b+c)(a+b-c)}{abc}\geqslant 0 \end{aligned} \\ \hline a\geqslant c\geqslant b & \begin{aligned} & \iff\frac{a^2-b^2}c+\frac{c^2-b^2}a\geqslant \frac{a^2-c^2}b \\ & \iff\frac{(c-b)(b+a)(c+a)(b+c-a)}{abc}\geqslant 0 \end{aligned} \\ \hline \end{array} 所以原不等式成立。 其实我觉得后两种情况不需要分开讨论,但不知怎样说明其等价,所以还是分了算了。
thread-762-1-2.html: 救我。。。
╰☆ヾo.海x 1# 2012-8-26 00:44
今天好难过。。救我
kuing 2# 2012-8-26 00:59
1# ╰☆ヾo.海x
icesheep 3# 2012-8-26 01:05
。。。
Tower 4# 2012-8-26 09:31
干嘛了?...
kuing 5# 2012-8-26 10:33
4# Tower /ka  桃子也来了
海盗船长 6# 2012-8-27 14:19
thread-763-1-4.html: [数列] 网友这次问数列
kuing 1# 2012-8-26 23:07
月中影 08-26 22:52:18 数列yn定义如下:y(1)=1,对k>0,y(2k)={2y(k),k为偶数;2y(k )+1,k为奇数},y(2k +1)={2y(k ),k为奇数;2y(k )+1,k为偶数},证明:数列能取遍每个正整数并且恰好一次   打成 $\LaTeX$ 先: 数列 $\{y_n\}$ 定义如下:$y_1=1$,对 $k>0$,$y_{2k}=\begin{cases} 2y_k, & k~\text{为偶数,} \\ 2y_k+1, & k~\text{为奇数,}\end{cases}$$y_{2k+1}=\begin{cases} 2y_k, & k~\text{为奇数,} \\ 2y_k+1, & k~\text{为偶数。}\end{cases}$证明:数列能取遍每个正整数并且恰好一次。 话说这个题目我好像在哪里见过,想不起来,也不知怎么做大家帮瞧瞧。
①②③④⑤⑥⑦ 2# 2012-8-27 09:06
没仔细想,有没有可能证明,当 $i\le 2^n-1$ 时,$y_i\le 2^n-1$,且此数列中没有重复项?
yizhong 3# 2012-9-16 11:05
现在爪机ing,改天我再把过程码下....….
yizhong 4# 2012-9-19 14:42
用进位制可以很容易的解决此问题 ~~~~
yes94 5# 2013-1-26 16:16
用进位制可以很容易的解决此问题 ~~~~ yizhong 发表于 2012-9-19 14:42 一中闪了一下,就不见了,
realnumber 6# 2013-1-26 21:18
1.先证明每个整数$n$,都是数列中的项.以下用反证法证明. 解关于$x$的方程,并设整数$n$是使得$y_x=n$无解的最小的$n$----(1). 若$n=1$,则$x=1$,不合$n$的要求.若$n$为大于1的奇数,考虑关于$t$的方程\[\frac{n-1}{2}=y_t\] 这个方程必定有解,假定解是偶数t,则$y_{2t+1}=n$,假定解是奇数t,则$y_{2t}=n$,都与(1)矛盾. 同样若$n$为大于1的偶数,类似处理. 如此每个整数$n$,总会出现在数列中. 2.证明每个整数$n$都出现一次.同上 解关于$x$的方程,并设整数$n$是使得$y_x=n$多解的最小的$n$----(1). 类似处理.
realnumber 7# 2013-1-28 09:16
4# yizhong 4楼写下过程,以前在k12看到过whitetiger用个这个办法.
yes94 8# 2013-2-19 17:30
用进位制可以很容易的解决此问题 ~~~~ yizhong 发表于 2012-9-19 14:42 一中很久没见来冒泡了! 不过一中的确说得对! 进位制+归纳法可解决该问题
realnumber 9# 2013-2-19 17:33
8# yes94   这下好,你来补完,他在留学(群里看到的),thanks.
yes94 10# 2013-2-19 17:47
9# realnumber 主要是看见你解决的那道题和这道题都是数列分段递推,是否和二进制有关? 所以才联想到这道题,才把它顶上来, 至于过程,有点难打,空了时把它打上或者截图。
thread-765-1-1.html: 临时自定义命令测试
kuing 1# 2012-8-27 01:08
代码: \(\verb"$c>0 \asdf c^2>0$"\) \(\verb"\newcommand\asdf\implies"\) \(\verb"$c>0 \asdf c^2>0$"\) \(\verb"$\newcommand\asdf\implies$"\) \(\verb"$c>0 \asdf c^2>0$"\) 显示: $c>0 \asdf c^2>0$ \newcommand\asdf\implies $c>0 \asdf c^2>0$ $\newcommand\asdf\implies$ $c>0 \asdf c^2>0$ 可以看到,直到 \newcommand\asdf\implies 两边加了美元符号,后面才能正常使用该自定义命令(并且该句不会显示出来),不然 mathjax 不识别,这跟真 LaTeX 不同,真 LaTeX 是不需要加美元的。 新定义的命令仅在此条定义命令所在行之后所在页之内才生效,所以只是临时的自定义命令,并不影响其他贴子。 二楼也证实了隔楼层也没关系,但翻页了的话如无意外就不行。
kuing 2# 2012-8-27 01:10
\(\verb"$c>0\asdf c^2>0$"\):$c>0\asdf c^2>0$
thread-766-1-1.html: \verb测试
kuing 1# 2012-8-27 01:15
$\verb"\frac12, \label{...} and \eqref{...}"$ $\verb"\begin{align*} \sqrt{\frac{a^2+b^2}2} &\ge \frac{a+b}2 \\ &\ge \sqrt{ab} \end{align*}"$ \begin{align*} \sqrt{\frac{a^2+b^2}2} &\ge \frac{a+b}2 \\ &\ge \sqrt{ab} \end{align*} $\verb" \begin{align*} \sqrt{\frac{a^2+b^2}2} &\ge \frac{a+b}2 \\ &\ge \sqrt{ab} \end{align*} "$ \begin{verbatim} \begin{align*} \sqrt{\frac{a^2+b^2}2} &\ge \frac{a+b}2 \\ &\ge \sqrt{ab} \end{align*} \end{verbatim} $\verb"$\frac12$"$ \(\verb"$\frac12$"\)
kuing 2# 2012-8-27 02:47
可惜,verbatim 环境用不了,否则显示环境的代码就简单多了。 现在 verb 虽然可以用,但都只是一行过,无法一次过写多行,只能一行用一个 verb,比较麻烦。
thread-767-1-1.html: mathjax设置添加自定义命令测试
kuing 1# 2012-8-27 02:35
在head里设置mathjax部分加入了 Macros: {         mbb: '\\mathbb',         abs: ['\\lvert #1\\rvert', 1]       } 现测试是否能用: 代码:\(\verb"$\mbb R$"\) 显示:$\mbb R$ 代码:\(\verb"$\mbb RQZN$"\) 显示:$\mbb RQZN$ 代码:\(\verb"$\mbb{RQZN}$"\) 显示:$\mbb{RQZN}$ 代码:\(\verb"$\abs1+\abs{a+b}+\abs{-2}$"\) 显示:$\abs1+\abs{a+b}+\abs{-2}$ 代码:\(\verb"$|-2| \ne \abs{-2}$, $|-a-b| \ne \abs{-a-b}$"\) 显示:$|-2| \ne \abs{-2}$, $|-a-b| \ne \abs{-a-b}$ (知道我为什么要定义这个 abs 了吗?)
kuing 2# 2012-8-27 02:41
测试成功! 太好了,能在整个添加新命令,这样可以设定一些常用的东东方便使用啊。 唔……想想有哪些可以设设先……
kuing 3# 2012-8-27 03:13
暂时设定如下这些       Macros: {         mbb: '\\mathbb',         riff: '\\implies',         liff: '\\impliedby',         abs: ['\\lvert #1\\rvert', 1],         rmd: ['\\mathop{\\mathrm{d}#1}', 1],         vv: '\\overrightarrow',         sslash: '\\mathrel{/\\!/}'       } 其中那个 rmd 是定义微分算子那个直立的 d,该法抄自 milksea。
kuing 4# 2012-8-27 03:17
继续测试: 代码: \(\verb"$a>b \iff b<a$"\) \(\verb"$a>0 \riff a^2>0$"\) \(\verb"$a^2>0 \liff a<0$"\) 分别显示: $a>b \iff b<a$ $a>0 \riff a^2>0$ $a^2>0 \liff a<0$ 微分算子那个直立的 d: 代码: \(\verb"\[\int_a^bf(x)\rmd x=F(b)-F(a)\]"\) 显示: \[\int_a^bf(x)\rmd x=F(b)-F(a)\] 之前的定义法有点问题,现在的不但自动直立而且整个 dx 还会跟前面的 f(x) 产生一点的小距离。
kuing 5# 2012-8-27 03:41
如果你在写某个贴子的时候也想弄一个这里没有的自定义命令来用,那也可以自己临时定义,参见 http://kkkkuingggg.5d6d.net/thread-765-1-1.html 水饺
kuing 6# 2012-8-29 01:31
竟然忘了定个向量命令 \vv,赶紧补上。
kuing 7# 2012-8-29 01:37
OK,\(\verb"$\vv x=\vv{BC}$"\) 显示 $\vv x=\vv{BC}$
孤星赶月 8# 2013-1-12 15:24
本帖最后由 孤星赶月 于 2013-1-12 15:28 编辑 $\top$ $\colon \colon$
thread-768-1-1.html: 试试itemize行不行
kuing 1# 2012-8-27 14:26
\begin{itemize} \item 123 \item asdf \end{itemize} \begin{itemize}\item 123\item asdf\end{itemize} \begin{itemize} \item 123 \item asdf \end{itemize} \begin{itemize} \item 123 \item asdf asdf asdf asdf asdf asdf asdf asdf asdf asdf asdf asdf asdf asdf asdf asdf asdf asdf asdf asdf asdf asdf \item asdf \item 三点一四一五九二六五三五八九七九三二三八四六二六四三三八三二七九五零二八八四一九七一六九三九九三七五一零五八二零九七四九四四五九二三零 \end{itemize}
kuing 2# 2012-8-27 14:29
竟然不行:(
thread-769-1-1.html: 关于查看公式代码
kuing 1# 2012-8-27 18:45
学习写代码的一个重要环节就是要参考一下别人的代码怎么写的,那么在本站,贴子里的公式一般都是瞬间就显示出具体式子来了,那如何看它的代码呢?虽然以前也提到过,但现在还是详细说说。 个人暂时有三种方法: 第一种,就是对着想查看的公式右键,然后就能找到显示代码的选项,该选项的位置可能会因为版本不同而不同,目前的位置是“右键 - Show Math As - TeX Commands”。 此法的缺点在于一次只能查看一个公式的代码,如果你想抄下整个贴子内所有公式的代码,那就得这样操作很多次。 第二种,如果你已经登录了本论坛,你可以利用引用功能,引用你想查看的公式所在贴,点击引用后进入了回贴状态,这时你就可以看到该贴的内容,代码自然也就看到了,而且还连美元符号什么的也有,直接复制就可用。 此法的一大缺点在于如果贴子较长,引用的时候只会将前面一部分引用下来,后面的部分就看不到了。 第三种,就是查看网页源文件(右键 - 查看网页源代码之类的操作就可以做到),然后就能找到整个网页上的所有公式的代码。 此法稍微弥补了前两种方法的缺点,但仍有一大缺点就是网页源文件通常会乱七八zao,不但正文内容不易找,而且还会多出一些 html 代码,复制下来也要进行替换后才能得到真正正确的代码。 综上,仍未有一个完美的方法,暂时来说,看情况地使用第一或第二种方法,第三种方法真的需要时再用。
kuing 2# 2012-8-27 18:52
$\sum\sqrt{\frac a{b+c}}\geqslant2$
realnumber 3# 2013-1-11 21:15
又,怎么才能看到不是自己帖子的代码?--已经知道了,利用QQ截图功能,就是快速按alt_ctrl_A,那么固定住图形就可以看别人的代码了 当然在编译前,已经编译好的话,刷新下就可以.要眼明手快.
kuing 4# 2013-1-11 21:32
嗯,楼上的方法也行,不过如果贴子长,就不太容易准确地截到了
thread-77-1-2.html: 无聊了……
kuing 1# 2011-10-9 00:01
试过这样做的举下手……
甄术 2# 2011-10-13 15:01
俺家木冰箱
kuing 3# 2011-10-13 15:24
2# 甄术 你家每餐饭都清空啊?
甄术 4# 2011-10-13 19:33
3# kuing 不必入冰箱
kuing 5# 2011-10-13 19:33
4# 甄术 你那边很冷?
甄术 6# 2011-10-13 19:35
5# kuing 华北
kuing 7# 2011-10-13 20:45
6# 甄术 呃,地理肓路过……
甄术 8# 2011-10-13 21:32
7# kuing 山西
kuing 9# 2011-10-14 00:39
8# 甄术 太原?
甄术 10# 2011-10-14 10:06
9# kuing yes
kuing 11# 2011-10-14 12:35
没什么,山西我只知道有一个地方叫太原,皆因……
甄术 12# 2011-10-14 12:37
没什么,山西我只知道有一个地方叫太原,皆因…… kuing 发表于 2011-10-14 12:35 省会,果然是地理盲
戊概念·五 13# 2011-11-2 13:59
没什么,山西我只知道有一个地方叫太原,皆因…… kuing 发表于 2011-10-14 12:35 太原是山西省的省会城市 刘是大姓,所以…… 现在三字名都重,难怪有些稀奇古怪的四字人名,比如“张杨果而” 注:本帖纯属天马行空的结果,O(∩_∩)O哈哈~
thread-770-1-1.html: 已默认设置成双击放大公式
kuing 1# 2012-8-27 18:58
指数上指数的公式:$x^{y^z}$,是不是嫌看不清上面的指数?双击公式试试。 如果双击公式没反应,请对着公式“右键 - Math Settings - Zoom Trigger - Double-Click”,然后再双击看看。
thread-771-1-6.html: [数列] 数列求通项
nash 1# 2012-8-27 20:32

海盗船长 2# 2012-8-27 21:39
这么暴力的题目。。有西西的感觉
nash 3# 2012-8-27 22:55
看字体,貌似是很久以前的书本上的 化简到$\a_n*a_n-2_\=(a_n-1_)^2*(1-2*a_n)*(1-2*a_n-2_)$ 貌似方向错了 编辑到现在都没搞好… _______kuing edit in $\LaTeX$_______ 化简到 $a_na_{n-2}=a_{n-1}^2(1-2a_n)(1-2a_{n-2})$
kuing 4# 2012-8-27 22:58
呵呵,我主要是看到“例”字就没什么动力做了。 我刚才其实也目测了一下,化到 \[\left(\frac1{a_n}-2\right)\left(\frac1{a_{n-2}}-2\right)=\frac1{a_{n-1}^2},\] 如果令 $b_n=\frac1{a_n}-2$,就有 $b_nb_{n-2}=(b_{n-1}+2)^2$,看着简洁,但然后想不到就放了……
nash 5# 2012-8-27 23:03
这样后面就可以做了 把n换成n-1再相减就能做了
kuing 6# 2012-8-27 23:13
5# nash 噢?我没看出来,你写写看。
nash 7# 2012-8-28 00:05
本帖最后由 nash 于 2012-8-28 01:06 编辑 4# kuing ______kuing edit in $\LaTeX$______ \begin{align} b_{n+1}b_{n-1}&=b_n^2+4b_n+4,\label{0827slqtxs1}\\ b_nb_{n-2}&=b_{n-1}^2+4b_{n-1}+4,\label{0827slqtxs2} \end{align} $\eqref{0827slqtxs1}-\eqref{0827slqtxs2}$ 得 \begin{align*} b_{n+1}b_{n-1}-b_nb_{n-2}=b_n^2+4b_n-b_{n-1}^2-4b_{n-1} & \riff b_{n-1}(b_{n+1}+b_{n-1}+4)=b_n(b_{n-2}+b_n+4)\\ & \riff \frac{b_{n+1}+b_{n-1}+4}{b_n}=\frac{b_n+b_{n-2}+4}{b_{n-1}}. \end{align*}
nash 8# 2012-8-28 00:10
7# nash 又是编辑好几次
kuing 9# 2012-8-28 00:26
8# nash 来了,刚才有点事闪开了,现在编辑下你的贴子放代码进去,你自己编辑一下看看。
kuing 10# 2012-8-28 00:41
搞定了,你先编辑 3# 的贴子看我怎么打的代码吧,7# 的相对代码复杂些,因为用了多行公式的环境以及编号和引用,你可以先不看。
kuing 11# 2012-8-28 00:44
哎,原来后面这么简单我都没目测出来,竟然目测到 $b_nb_{n-2}=(b_{n-1}+2)^2$ 之后就放了,还不是因为那个“例”……哼哼
kuing 12# 2012-9-1 23:24
这道据说是N年前的集训队测试题
thread-773-1-2.html: 感觉这样聊天不错
叶剑飞Victor 1# 2012-8-27 22:58
本帖最后由 叶剑飞Victor 于 2012-8-27 22:59 编辑 数学公式可不是图片哦!这可是手打的LaTeX哦!
叶剑飞Victor 2# 2012-8-27 23:49
本帖最后由 叶剑飞Victor 于 2012-8-27 23:53 编辑 MathQ聊天感觉也也错。编辑数学公式很方便的。 对方收到的这个数学公式是可以编辑的。
kuing 3# 2012-8-27 23:59
2# 叶剑飞Victor 略有听闻,但我还是没什么兴趣用,除非推广到很多人用。
╰☆ヾo.海x 4# 2012-8-29 23:48
哇。。。还有这种Q啊。。我也要下一个。。哈哈哈
╰☆ヾo.海x 5# 2012-8-29 23:50
1# 叶剑飞Victor 那个mathIM就是专门跟别人聊数学的?那就是说有题目不会直接进这里面问,一定有人回复咯?哇。。这么好啊。。。。
thread-774-1-6.html: [组合] 一个猜想
hflz01 1# 2012-8-28 14:10
____kuing edit in $\LaTeX$____ $n$ 支球队 A1,A2,A3,……,An 进行单循环赛,用 $x_i$, $y_i$ 分别表示球队 Ai 胜的次数与负的次数,若 $\sum x_i^3=\sum y_i^3$,则 $\sum x_i^4=\sum y_i^4$。 (这是原题,今给出猜想:在原题的条件下,若 A1 胜 $x$ 场,负 $y$ 场,则必存在某 Ai,有 Ai 负 $x$ 场,胜 $y$ 场,求证明或否定)
kuing 2# 2012-8-28 15:25
等等……猜想中的“在原题的条件下”是否包括“$\sum x_i^3=\sum y_i^3$”?
hflz01 3# 2012-8-28 16:46
当然包括。
都市侠影 4# 2012-8-29 08:54
不懂单循环比赛的路过.............
hflz01 5# 2012-8-29 10:12
所谓单循环赛,简单地说,是任何两个队之间比赛且仅赛一次。
thread-776-1-1.html: lap测试
kuing 1# 2012-8-28 15:03
$\nearrow$ $\searrow$ $\rlap{\nearrow}\searrow$ $\llap{\nearrow}\searrow$ $\clap{\nearrow}\searrow$ $\rlap XX$ $\llap XX$ $\rlap{\nearrow}\nearrow$ $\rlap{\nearrow}{\nearrow}$ $\rlap{\nearrow}{\searrow}$
kuing 2# 2012-8-28 15:07
OK $\newcommand\jcjt{\rlap{\nearrow}{\searrow}}$ $\newcommand\rto\rightarrow$ $\newcommand\lto\leftarrow$ $\newcommand\uto\uparrow$ $\newcommand\dto\downarrow$ $\newcommand\ruto\nearrow$ $\newcommand\rdto\searrow$ $\newcommand\luto\nwarrow$ $\newcommand\ldto\swarrow$ \[\Large \begin{array}{ccc} A & \!\!\!\rto & \!\!\!B\\ \dto & \!\!\!\jcjt & \!\!\!\dto\\ C & \!\!\!\lto & \!\!\!D \end{array} \]
thread-778-1-1.html: smash test
kuing 1# 2012-8-28 21:28
smash \[ \text{实数} \begin{cases} \text{有理数}\smash[t]{\begin{cases} \text{整数}\smash{\begin{cases} \text{奇数} \\ \text{偶数} \end{cases}}\\ \text{分数} \end{cases}} \\[1ex] \text{无理数}\smash[b]{\begin{cases} \text{代数无理数} \\ \text{超越数} \end{cases}} \end{cases} \]
临时帐号1 2# 2012-9-20 13:01

叶剑飞Victor 3# 2012-11-3 18:28
[i=s] 本帖最后由 叶剑飞Victor 于 2012-11-3 18:43 编辑 [/i] \[ \text{复数}\smash[b] { \begin{cases} \text{实数} \begin{cases} \text{有理数} \smash[t] { \begin{cases} \text{整数} \smash { \begin{cases} \text{奇数} \\ \text{偶数} \end{cases} }\\ \text{分数} \end{cases} } \\[1ex] \text{无理数} \smash[b] { \begin{cases} \text{代数无理数} \\ \text{超越数} \end{cases} } \end{cases}\\ \text{虚数} \begin{cases} \text{纯虚数} \\ \text{非纯虚数} \end{cases} \end{cases} } \]
kuing 4# 2012-11-3 18:35
要禁用discuz代码,不然 [b] 变成了粗体
kuing 5# 2012-11-3 18:39
还有就是那些空格会变成真的空格,这一点我一直也不知怎么通过mathjax设置去解决(http://kkkkuingggg.5d6d.net/viewthread.php?tid=9&page=1#pid31 注意事项第三个),所以在代码上不能缩进……
thread-779-1-2.html: Q群的一个无语的bug
kuing 1# 2012-8-29 03:44
RT,具体地,就是竟然会有两个Q群共用同一个群号码! 不信?一起来试一下,首先进入查找 - 找群。 如果用精确查找,搜索群号“11071642”,只能找到“人教论坛数学官方群”(以下简称“人教群”),如下图: 但!如果用条件查找,搜关键字“中药世家”,这时你就会看到一个头像和群号都跟“人教群”一样的“中药世家”群!如下图: 而更加无语的是,此时如果点击“加入群”想加入这“中药世家”群时,此加群申请竟然会发送到“人教群”里!而我就是这“人教群”的管理员之一 结果就是我现在所遇到的事:保守估计每天至少十来个找中药的人请求加入“人教群”,每次我都只能叉掉,虽然并不是什么废神的事,但长期这样(这现象至少存在几个月了)还是有点郁闷的,而且有几次不小心按了确定,就像今晚那个人进来就找中药师了,搞得又要T人,我郁闷时他估计也郁闷。
海盗船长 2# 2012-8-29 13:09
奇葩
kuing 3# 2012-8-29 17:17
注:本贴粹纯针对事情本身,绝无宣传群之意。
╰☆ヾo.海x 4# 2012-8-29 23:45
我被雷到了。。
isea 5# 2012-8-30 18:40
太神奇了啊!
thread-78-1-3.html: 来自群的三角化简题$\tan(2\pi/13)+\tan(5\pi/13)+\tan(6\pi/13)$
kuing 1# 2011-10-9 17:25
爱好者-Salvation(13058978)  15:10:36 那这个怎么算比较好 tan(2pi/13)+tan(5pi/13)+tan(6pi/13) 求值 $\displaystyle \tan\frac{2\pi}{13}+\tan\frac{5\pi}{13}+\tan\frac{6\pi}{13}$ 注意到三个角之和为 $\pi$,所以\[ \tan \frac{{2\pi }}{{13}} + \tan \frac{{5\pi }}{{13}} + \tan \frac{{6\pi }}{{13}} = \tan \frac{{2\pi }}{{13}}\tan \frac{{5\pi }}{{13}}\tan \frac{{6\pi }}{{13}} \]利用欧拉公式,上式的右边可以写成\[ \frac{{i\left( {e^{ - \frac{{2i\pi }}{{13}}} - e^{\frac{{2i\pi }}{{13}}} } \right)}}{{e^{ - \frac{{2i\pi }}{{13}}} + e^{\frac{{2i\pi }}{{13}}} }} \cdot \frac{{i\left( {e^{ - \frac{{5i\pi }}{{13}}} - e^{\frac{{5i\pi }}{{13}}} } \right)}}{{e^{ - \frac{{5i\pi }}{{13}}} + e^{\frac{{5i\pi }}{{13}}} }} \cdot \frac{{i\left( {e^{ - \frac{{6i\pi }}{{13}}} - e^{\frac{{6i\pi }}{{13}}} } \right)}}{{e^{ - \frac{{6i\pi }}{{13}}} + e^{\frac{{6i\pi }}{{13}}} }} \]展开可整理为\[ \frac{{i\left( {e^{ - \frac{{i\pi }}{{13}}} - e^{\frac{{i\pi }}{{13}}} } \right) + i\left( {e^{ - \frac{{3i\pi }}{{13}}} - e^{\frac{{3i\pi }}{{13}}} } \right) + i\left( {e^{ - \frac{{9i\pi }}{{13}}} - e^{\frac{{9i\pi }}{{13}}} } \right)}}{{ - 2 + e^{ - \frac{{i\pi }}{{13}}} + e^{\frac{{i\pi }}{{13}}} + e^{ - \frac{{3i\pi }}{{13}}} + e^{\frac{{3i\pi }}{{13}}} + e^{ - \frac{{9i\pi }}{{13}}} + e^{\frac{{9i\pi }}{{13}}} }} \]再用欧拉公式化回三角式,为\[ \frac{{\sin \frac{\pi }{{13}} + \sin \frac{{3\pi }}{{13}} + \sin \frac{{9\pi }}{{13}}}}{{ - 1 + \cos \frac{\pi }{{13}} + \cos \frac{{3\pi }}{{13}} + \cos \frac{{9\pi }}{{13}}}} \]分母其实已经早有求过,看看分子如何,平方一下,利用两倍角公式,积化和差之类,有\[ \left( {\sin \frac{\pi }{{13}} + \sin \frac{{3\pi }}{{13}} + \sin \frac{{9\pi }}{{13}}} \right)^2 = \frac{1}{2}\left( {3 + 2\cos \frac{\pi }{{13}} + 2\cos \frac{{3\pi }}{{13}} + 2\cos \frac{{9\pi }}{{13}} + \cos \frac{{2\pi }}{{13}} + \cos \frac{{6\pi }}{{13}} + \cos \frac{{8\pi }}{{13}}} \right) \]注意到\[ \cos \frac{{2\pi }}{{13}} + \cos \frac{{4\pi }}{{13}} + \cos \frac{{6\pi }}{{13}} + \cos \frac{{8\pi }}{{13}} + \cos \frac{{10\pi }}{{13}} + \cos \frac{{12\pi }}{{13}} = - \frac{1}{2} \]于是易得\[ \left( {\sin \frac{\pi }{{13}} + \sin \frac{{3\pi }}{{13}} + \sin \frac{{9\pi }}{{13}}} \right)^2 = \frac{3}{2}\left( {\cos \frac{\pi }{{13}} + \cos \frac{{3\pi }}{{13}} + \cos \frac{{9\pi }}{{13}}} \right) + \frac{5}{4} \]又显然 $\sin \frac{\pi }{{13}} + \sin \frac{{3\pi }}{{13}} + \sin \frac{{9\pi }}{{13}}>0$,所以\[ \sin \frac{\pi }{{13}} + \sin \frac{{3\pi }}{{13}} + \sin \frac{{9\pi }}{{13}} = \sqrt {\frac{3}{2}\left( {\cos \frac{\pi }{{13}} + \cos \frac{{3\pi }}{{13}} + \cos \frac{{9\pi }}{{13}}} \right) + \frac{5}{4}} \]综上,最终得到\[ \tan \frac{{2\pi }}{{13}} + \tan \frac{{5\pi }}{{13}} + \tan \frac{{6\pi }}{{13}} = \frac{{\sqrt {\frac{3}{2}\left( {\cos \frac{\pi }{{13}} + \cos \frac{{3\pi }}{{13}} + \cos \frac{{9\pi }}{{13}}} \right) + \frac{5}{4}} }}{{ - 1 + \cos \frac{\pi }{{13}} + \cos \frac{{3\pi }}{{13}} + \cos \frac{{9\pi }}{{13}}}} \]那 $\cos \frac{\pi }{{13}} + \cos \frac{{3\pi }}{{13}} + \cos \frac{{9\pi }}{{13}}$ 是多少哇?时间关系,抄一段何版主的记录: 因为\[ \cos \frac{2\pi}{13} + \cos \frac{4\pi}{13} + \cos \frac{6\pi}{13} + \cos \frac{8\pi}{13} + \cos \frac{10\pi}{13} + \cos \frac{12\pi}{13} = -\frac{1}{2} \]这样\begin{align*} & \left(\cos \frac{2\pi}{13} + \cos \frac{6\pi}{13} + \cos \frac{8\pi}{13}\right) - \left(\cos \frac{\pi}{13} + \cos \frac{3\pi}{13} + \cos \frac{9\pi}{13}\right) \\ = & \cos \frac{2\pi}{13} + \cos \frac{4\pi}{13} + \cos \frac{6\pi}{13} + \cos \frac{8\pi}{13} + \cos \frac{10\pi}{13} + \cos \frac{12\pi}{13} \\ = & -\frac{1}{2} \end{align*}即\[ \cos \frac{2\pi}{13} + \cos \frac{6\pi}{13} + \cos \frac{8\pi}{13} = \cos \frac{\pi}{13} + \cos \frac{3\pi}{13} + \cos \frac{9\pi}{13} - \frac{1}{2} \]所以\begin{align*} & \left(\cos \frac{\pi}{13} + \cos \frac{3\pi}{13} + \cos \frac{9\pi}{13}\right)^2 \\ = & \cos^2\frac{\pi}{13} + \cos^2\frac{3\pi}{13} + \cos^2\frac{9\pi}{13} + 2\left(\cos \frac{\pi}{13}\cos \frac{3\pi}{13} + \cos \frac{\pi}{13}\cos \frac{9\pi}{13} + \cos \frac{3\pi}{13}\cos \frac{9\pi}{13}\right) \\ = & \frac{1}{2}\left(\cos \frac{2\pi}{13} + \cos \frac{6\pi}{13} + \cos \frac{18\pi}{13} + 3\right) \\ & {}+ \cos \frac{2\pi}{13} + \cos \frac{4\pi}{13} + \cos \frac{6\pi}{13} + \cos \frac{8\pi}{13} + \cos \frac{6\pi}{13} + \cos \frac{10\pi}{13} + \cos \frac{12\pi}{13} \\ = & \frac{1}{2}\left(\cos \frac{2\pi}{13} + \cos \frac{6\pi}{13} + \cos \frac{8\pi}{13}\right) + 1 \\ = & \frac{1}{2}\left(\cos \frac{\pi}{13} + \cos \frac{3\pi}{13} + \cos \frac{9\pi}{13}\right) + \frac{3}{4} \end{align*}而\[ \cos \frac{\pi}{13} + \cos \frac{3\pi}{13} + \cos \frac{9\pi}{13} = \cos \frac{\pi}{13} + \cos \frac{3\pi}{13} - \cos \frac{4\pi}{13} > 0 \]所以 $\cos \dfrac{\pi}{13} + \cos \dfrac{3\pi}{13} + \cos \dfrac{9\pi}{13}$ 是方程 $x^2 = \dfrac{1}{2}x + \dfrac{3}{4}$ 的正根,所以\[ \cos \frac{\pi}{13} + \cos \frac{3\pi}{13} + \cos \frac{9\pi}{13} = \frac{\sqrt{13} + 1}{4} \] 抄完,代入,即得\[ \tan \frac{{2\pi }}{{13}} + \tan \frac{{5\pi }}{{13}} + \tan \frac{{6\pi }}{{13}} = \frac{{\sqrt {\frac{3}{2} \cdot \frac{{\sqrt {13} + 1}}{4} + \frac{5}{4}} }}{{ - 1 + \frac{{\sqrt {13} + 1}}{4}}} = \sqrt {65 + 18\sqrt {13} } \]
yes94 2# 2013-3-19 21:35
1# kuing 太牛了!
kuing 3# 2013-3-19 21:46
当年的转发连码都没打……
thread-781-1-2.html: 这类对话几乎是月经
kuing 1# 2012-8-29 21:33
kuing 18:11:31 hi,你是? **** 21:14:14 我不认识你 是你加我的 kuing 21:19:39 噢?不是你加我的吗? **** 21:19:09 好吧 我搞错了 kuing 21:20:12 没事 **** 21:19:26 我们不认识的 我是在别人的空间里看见你的qq,所以加你了 kuing 21:20:37 352? **** 21:20:01 是的e kuing 21:21:18 猜到了,我在352的QQ空间里也看到你 **** 21:20:57 你们是同事? kuing 21:22:05 不是,只是网友而已 **** 21:21:24 哦 你也是数学老师吧 kuing 21:22:27 不是 我无业游民 **** 21:21:51 不相信 kuing 21:22:44 那你可以问352啊 **** 21:22:13 我从来没有和他聊过天 只是喜欢看他写的东西 kuing 21:23:03 自己说总是没人信,得其他人说才行 **** 21:22:27 那你是做什么的 啊? kuing 21:23:20 无业游民啊 **** 21:22:56 你靠什么生活? kuing 21:24:01 在家 **** 21:23:56 吃什么 kuing 21:24:53 吃饭咯 **** 21:24:53 你还没有毕业? kuing 21:26:59 毕了 **** 21:26:48 好吧,富二代就是不用工作也有饭吃 真好 kuing 21:28:06 不是富二代啊 **** 21:28:28 骗人 只有富二代不用工作也能生活 …………至此表示已经不想再回了,每次被问这些,我都照答,但几乎没人信,哎…… 注:Q名已屏蔽
海盗船长 2# 2012-8-29 21:45
水一下。。
戊概念·五 3# 2012-8-29 23:46
1# kuing 男子明明是遗精好不好= = 其实,你承认就是了嘛:精神上富庶一方的k歌之王
kuing 4# 2012-8-29 23:49
3# 戊概念·五 这种“富”并没有“代”可言
戊概念·五 5# 2012-8-30 23:46
4# kuing 父辈提供的成长环境,对子代的影响是潜移默化且细微知著的。
yizhong 6# 2012-9-17 21:57
第一次进来吹水区,嘿嘿,话说那对话那网友想表达啥主题呢~~~
thread-782-1-6.html: [函数] 三角
yayaweha 1# 2012-8-30 08:02
本帖最后由 yayaweha 于 2013-2-7 22:38 编辑 $y=sinx+sin2x$的值域是多少?怎么求呀?
kuing 2# 2012-8-30 14:02
貌似在某期数学空间我留了这道练习给大家的……
yayaweha 3# 2012-8-30 15:01
是呀!求解
kuing 4# 2012-8-30 15:15
可以直接求导,也可以用万能公式再求导,我个人喜欢用待定系数+均值(更初等且计算更简单些),以前在人教论坛发过,等我找找先。
yayaweha 5# 2012-8-30 15:33
我就是想问均值之类的解法
kuing 6# 2012-8-30 15:44
求教一道三角函数题 - 高中数学论坛 - 中学数学教育论坛 - 人教论坛 - Powered by Discuz!.rar (362.43 KB)
yayaweha 7# 2012-8-30 16:12
蛮复杂的
kuing 8# 2012-8-30 16:26
7# yayaweha 主要是数据不太好计算而已,由于是直接从乘的改编成加,没对数字作精心的设计,所以……
hongxian 9# 2012-8-30 22:52
人教论坛,看样子版主还是可以访问的!
kuing 10# 2012-8-30 22:57
9# hongxian 不能
thread-783-1-6.html: [几何] 一个平面几何题(完全不擅长)
hflz01 1# 2012-8-31 09:21
凸四边形ABCD的两条对角线相交于点O,而△ABO 和△CDO的外接圆圆心分别为点P、点Q.求证:AB+CD≤4PQ.
kuing 2# 2012-9-1 02:00
凸四边形ABCD的两条对角线相交于点O,而△ABO 和△CDO的外接圆圆心分别为点P、点Q.求证:AB+CD≤4PQ. hflz01 发表于 2012-8-31 09:21 我也不擅长平几,想了一晚也没得到几何证法,最后只有硬上代数法搞定了,而且还得到了加强式。 我将证明有更强的\[4PQ>\sqrt{2(AB^2+CD^2)}\]成立,即证 \begin{equation}\label{20120901pingjijiaqiang} 8PQ^2>AB^2+CD^2. \end{equation} 如图所示,以点 $O$ 为原点,以 $AC$ 所在直线为 $x$ 轴,建立平面直角坐标系。 不失一般性,设 $A(-a,0)$, $C(c,0)$, $D(d,1)$, $B(-r\cdot d,-r)$,其中 $a>0$, $c>0$, $r>0$,但 $d$ 的范围是 $\mbb R$。 设 $Q(x_q,y_q)$,则由 $x_q^2+y_q^2=(x_q-c)^2+y_q^2=(x_q-d)^2+(y_q-1)^2$ 求得 $Q\bigl(c/2,(d^2+1-cd)/2\bigr)$,类似地可以求得 $P\bigl(-a/2,-(rd^2+r-ad)/2\bigr)$,于是 \begin{align*} 8PQ^2 &= 8\left(\left(\frac c2+\frac a2\right)^2+\left(\frac{d^2+1-cd}2+\frac{rd^2+r-ad}2\right)^2\right)\\ &=2(a+c)^2+2\bigl((r+1)(d^2+1)-(a+c)d\bigr)^2, \end{align*} 而 \[AB^2+CD^2=(a-rd)^2+r^2+(c-d)^2+1=(r^2+1)(d^2+1)-2(ar+c)d+a^2+c^2,\] 所以,式 \eqref{20120901pingjijiaqiang} 等价于 \begin{equation}\label{20120901pingjijiaqiangdjs} 2(a+c)^2+2\bigl((r+1)(d^2+1)-(a+c)d\bigr)^2>(r^2+1)(d^2+1)-2(ar+c)d+a^2+c^2. \end{equation} 记 $f(d)=2(a+c)^2+2\bigl((r+1)(d^2+1)-(a+c)d\bigr)^2-\bigl((r^2+1)(d^2+1)-2(ar+c)d+a^2+c^2\bigr)$,如果 $d\geqslant 0$,则有\[f(-d)-f(d)=4d\bigl((a+c)(r+1)(d^2+1)-(ar+c)\bigr)\geqslant 0,\]即 $f(-d)\geqslant f(d)$ 对任意 $d\geqslant 0$ 恒成立,这也就是说,我们只要证明当 $d\geqslant 0$ 时式 \eqref{20120901pingjijiaqiangdjs} 恒成立即可。 而当 $d\geqslant 0$ 时,欲证式 \eqref{20120901pingjijiaqiangdjs},只要证明 \begin{equation}\label{20120901pingjijiaqiangdjszjq} (a+c)^2+2\bigl((r+1)(d^2+1)-(a+c)d\bigr)^2>(r^2+1)(d^2+1), \end{equation} 对 $a+c$ 配方,式 \eqref{20120901pingjijiaqiangdjszjq} 等价于 \[(2d^2+1)\left(a+c-\frac{2d(r+1)(d^2+1)}{2d^2+1}\right)^2+\frac{4r(d^2+1)^2+(r^2+1)(d^2+1)}{2d^2+1}>0,\] 从而式 \eqref{20120901pingjijiaqiangdjszjq} 成立,即式 \eqref{20120901pingjijiaqiang} 成立,加强式获证。
kuing 3# 2012-9-1 02:24
郁闷了,刚回完帖,正打算补图,结果就上不了网了,汗哉,也太巧了,现在用手机上,也不知怎么传图,明天再补好了。
yizhong 4# 2012-9-1 12:07
本帖最后由 yizhong 于 2012-9-1 12:09 编辑 3# kuing oh,小K计算niubility,因为平几题如果解析化一不小心就会挂在计算上
kuing 5# 2012-9-1 14:37
OK,可以上网了,已补图。
kuing 6# 2012-9-1 14:44
4# yizhong 其实这计算量比我预料之中要小了。
kuing 7# 2012-9-1 20:37
anyway,期待有漂亮的纯几何法。
kuing 8# 2012-9-1 23:29
不知次数再高点还成不成立,说不准可能还会有 $4PQ>\sqrt[4]{8(AB^4+CD^4)}$ 甚至更高次? 或者再弄点别的形式的加强可能也有机会,因为上面证明中最后的不等式并不是十分强,有空再研究研究。 PS、楼主能不能说说此题的出处?
kuing 9# 2012-9-2 03:09
我擦啊!套用上面的思路发现竟然能证明 $2PQ>AB$ 且 $2PQ>CD$ 而且证明更加简单! 仍然如上所设,可知\[2PQ>AB \iff (a+c)^2+\bigl((r+1)(d^2+1)-(a+c)d\bigr)^2>(a-rd)^2+r^2,\]我们证明更强的\[(a+c)^2+\bigl((r+1)(d^2+1)-(a+c)d\bigr)^2>(a-rd)^2+(c-rd)^2+2ac+r^2,\]仍然对 $a+c$ 配方,上式等价于\[\bigl((r+1)d^2+1-(a+c)d\bigr)^2+2r(d^2+1)>0,\]显然成立,还不用讨论 $d$ 的正负。 同理可得 $2PQ>CD$,这样,不但得到原题的结论,2#的加强以及楼上所指的更高次数的也显然成立了,甚至还能得到另外形式的加强,比如注意到 $2ac=2OA\cdot OC$ 以及 $2r(d^2+1)=2OB\cdot OD$,故由上述证明过程,就可以得到\begin{align*} 2PQ&>\sqrt{AB^2+2(OA\cdot OC+OB\cdot OD)},\\ 2PQ&>\sqrt{CD^2+2(OA\cdot OC+OB\cdot OD)}, \end{align*}若两边平方相加,又有\[8PQ^2>AB^2+CD^2+4(OA\cdot OC+OB\cdot OD),\]一漂亮的加强就得到了,大家还可以试试弄出其他加强来。 不过话说回来,既然能如此简化,是不是该回头想想用平几方法证明 $2PQ>AB$ 且 $2PQ>CD$ 以及这些加强?时间关系,又很晚了,每次都是深夜才有灵感玩题,以至于每次闪人前总是收不成尾……
isea 10# 2012-9-2 17:54
本帖最后由 isea 于 2012-9-2 23:01 编辑 9# kuing 看到此题时,就猜是否有这种可能,原来,是真。 ============================ 由2楼的图 有个粗的想法,记两圆的对应半径为:$r_p$,$r_q$ 分别在两圆中: $\frac {AB}2\le r_p$ $\frac {CD}2\le r_q$ 当两圆心在O点异侧时(P、Q不同在一个圆内),(而两圆至少有一个交点),于是 $r_p < PQ$ (不严谨,此题里看图说话了,需要证明) $r_q < PQ$ (不严谨,此题里看图说话了,需要证明) 于是$\frac {AB}2 + \frac {CD}2 \le r_p+r_q < 2PQ$ 当P、Q落在一个圆内时,不知道了,哈哈......
kuing 11# 2012-9-2 17:57
10# isea 平几高手来了,期待你的平几证法,包括那些加强
isea 12# 2012-9-2 21:45
本帖最后由 isea 于 2012-9-2 22:57 编辑 11# kuing 这题我肯定拿不下来了,看看你的计算就有这种强烈的感觉。 如果点么,游民,四个6,……,等等一时想不起那些ID了,他们过来看看,或许还有可能...... ++++++++++++++++===================++++++++++++++++ 顺着思路粗想了一下,不严谨,10楼成立的情况是在两圆心在O点两侧时 对两圆心在O点同侧时,10楼不成立。
kuing 13# 2012-9-2 23:09
12# isea 嗯,两圆心可能会在同一圆内。 $r_P$ 和 $r_Q$ 均可以远大于 $PQ$:
kuing 14# 2012-9-3 04:05
11# kuing 这题我肯定拿不下来了,看看你的计算就有这种强烈的感觉。 如果点么,游民,四个6,……,等等一时想不起那些ID了,他们过来看看,或许还有可能...... isea 发表于 2012-9-2 21:45 我联系不到他们,你能不能把他们叫来。
kuing 15# 2012-9-3 19:07
楼主不见了
hflz01 16# 2012-9-3 20:52
哈哈,这道题具体来源不明,是在一个大学老师的讲义材料(手写版)上看到的。
kuing 17# 2012-9-3 21:04
16# hflz01 噢?莫非大学也玩这种平几题?
hflz01 18# 2012-9-3 22:12
大学老师当然玩 象中科大数学系还是有相当数量的老师作高中竞赛讲座的。这就是一位大学老师(不是科大的)在给我校学生作竞赛讲座时的题。
kuing 19# 2012-9-3 22:39
18# hflz01 噢,原来如此。 我不了解这些,皆因高中没上过竞赛,也没上过大学。
kuing 20# 2012-9-6 16:09
12# isea 人教论坛可以上了,去初中版叫人吧
thread-784-1-6.html: [不等式] 06罗马尼亚竞赛题的3种漂亮AG证法
yizhong 1# 2012-8-31 16:30
本帖最后由 yizhong 于 2012-8-31 17:00 编辑 题目:$a,b,c>0,a+b+c=3,证明:\frac{1}{a^2}+\frac{1}{b^2}+\frac{1}{c^2}\geqslant a^2+b^2+c^2$ 此题记得小K曾经在数学空间里作为切线法的范例,在这里我就不再给出这种作答。以下是这个题目的另外三种AG做法 证法1:因为a,b,c>0.所以:$\frac{1}{a^2}+\frac{1}{b^2}+\frac{1}{c^2}\geqslant\frac{1}{ab}+\frac{1}{ac}+\frac{1}{bc} =\frac{a+b+c}{abc}=\frac{9}{abc(a+b+c)}\geqslant\frac{27}{(ab+ac+bc)^2}$所以要我们只要证明:$\frac{27}{(ab+ac+bc)^2}\geqslant a^2+b^2+c^2,也就是(a^2+b^2+c^2)(ab+ac+bc)^2\leqslant27$,由三元的AG即可得到。 证法2:题目即证: $\frac{1}{a^2}+ \frac{1}{b^2}+ \frac{1}{c^2}+ 2(ab+ac+bc) \geqslant 9$因为:$\frac{1}{a^2}+\frac{1}{b^2}+\frac{1}{c^2}+2(ab+ac+bc)\geqslant \frac{1}{ab}+\frac{1}{ac}+\frac{1}{bc}+2\sqrt{3abc(a+b+c)}=3(\frac{1}{abc}+\sqrt{abc}+\sqrt{abc}) \geqslant9$ 证法3:题目即证:$\frac{1}{a^2}+\frac{1}{b^2}+\frac{1}{c^2}+2(ab+ac+bc)\geqslant9$由于:$\frac{1}{a^2}+\frac{1}{b^2}+\frac{1}{c^2}+2(ab+ac+bc)\geqslant\frac{1}{3}.(\frac{1}{a}+\frac{1}{b}+\frac{1}{c})^2+2(ab+ac+bc)=\frac{1}{3}.\frac{(ab+ac+bc)^2}{abc} +(ab+ac+bc)+(ab+ac+bc)再利用下三元AG即可证明$
kuing 2# 2012-8-31 17:20
还是我帮你打打好了 以下引用楼主的内容并加以编辑。 题目:$a$, $b$, $c>0$, $a+b+c=3$,证明:\[\frac{1}{a^2}+\frac{1}{b^2}+\frac{1}{c^2}\geqslant a^2+b^2+c^2.\] 此题记得小K曾经在数学空间里作为切线法的范例,在这里我就不再给出这种作答。以下是这个题目的另外三种AG做法 证法1:因为 $a$, $b$, $c>0$,所以\[\frac{1}{a^2}+\frac{1}{b^2}+\frac{1}{c^2}\geqslant\frac{1}{ab}+\frac{1}{ac}+\frac{1}{bc} =\frac{a+b+c}{abc}=\frac{9}{abc(a+b+c)}\geqslant\frac{27}{(ab+ac+bc)^2},\]所以要我们只要证明\[\frac{27}{(ab+ac+bc)^2}\geqslant a^2+b^2+c^2,\]也就是\[(a^2+b^2+c^2)(ab+ac+bc)^2\leqslant27,\]由三元的AG即可得到。 证法2:题目即证\[\frac{1}{a^2}+ \frac{1}{b^2}+ \frac{1}{c^2}+ 2(ab+ac+bc) \geqslant 9,\]因为 \begin{align*} \frac{1}{a^2}+\frac{1}{b^2}+\frac{1}{c^2}+2(ab+ac+bc) &\geqslant \frac{1}{ab}+\frac{1}{ac}+\frac{1}{bc}+2\sqrt{3abc(a+b+c)}\\ &=3\left(\frac{1}{abc}+\sqrt{abc}+\sqrt{abc}\right)\geqslant9. \end{align*} 证法3:题目即证\[\frac{1}{a^2}+\frac{1}{b^2}+\frac{1}{c^2}+2(ab+ac+bc)\geqslant9,\]由于 \begin{align*} \frac{1}{a^2}+\frac{1}{b^2}+\frac{1}{c^2}+2(ab+ac+bc) &\geqslant\frac{1}{3}\cdot\left(\frac{1}{a}+\frac{1}{b}+\frac{1}{c}\right)^2+2(ab+ac+bc)\\ &=\frac{1}{3}\cdot\frac{(ab+ac+bc)^2}{abc}+(ab+ac+bc)+(ab+ac+bc), \end{align*} 再利用下三元AG即可证明。
yizhong 3# 2012-9-1 12:39
本帖最后由 yizhong 于 2012-9-1 12:45 编辑 下面再补充一种做法,这种做法是由萝卜(网名路箩筐)给出的,因为:$a,b,c>0,所以\frac{1}{a^2}+\frac{1}{b^2}+\frac{1}{c^2} \geqslant\frac{1}{ab}+\frac{1}{ac}+\frac{1}{bc}$所以我们只要证明:$\frac{1}{ab}+\frac{1}{ac}+\frac{1}{bc}\geqslant a^2+b^2+c^2$即可。也就是:$abc(a^2+b^2+c^2)\leqslant3$由于$(ab+ac+bc)^2\geqslant3abc(a+b+c)=9abc$,所以我们只要证明: $(ab+ac+bc)^2(a^2+b^2+c^2)\leqslant27$而这个很容易由三元的AG得到,证毕。 另外PS一下:在ji chen 的代数不等式的第六章中,ji chen 刊载了AOPS上arqady的一个均值证法,但是这个均值证法实在是太暴力,用的是68元的均值不等式。 最后谢谢小K帮我重新编辑了下
thread-789-1-6.html: [不等式] 请教一个不等式证明,谢谢了!
hongxian 1# 2012-9-1 14:23
1.求证:$(1+1\times 2)\cdot (1+2\times 3)\cdots [1+n(n+1)]>{{e}^{2n-3}}$
kuing 2# 2012-9-1 14:31
你不是问过了吗 http://kkkkuingggg.5d6d.net/thread-488-1-1.html 连标题都几乎一样
hongxian 3# 2012-9-1 14:33
我记得是问过,但在我的贴子中没有找到,不好意思了!
thread-79-1-2.html: kel……
kuing 1# 2011-10-9 18:45
话说,每次在这里写好题解,然后发链接到群里或论坛里,一下子在线列表就多了N个游客,然而一会之后总是恢复原状
甄术 2# 2011-10-13 15:04
大都不敢班门弄斧了
thread-790-1-6.html: [函数] 一道函数压轴小题
转化与化归 1# 2012-9-2 07:46
一道函数压轴小题,找到答案不难,但是书写过程怎么弄得简洁些?
kuing 2# 2012-9-2 16:12
1# 转化与化归 你书写一下看看,我还不会做呢……
第一章 3# 2012-9-2 17:54
就是跟单调性有关吧。要讨论的
kuing 4# 2012-9-2 18:00
就是跟单调性有关吧。要讨论的 第一章 发表于 2012-9-2 17:54 看到讨论我就不想做了
第一章 5# 2012-9-2 18:01
感觉怪怪的,如果$b=0$,那么$k$?
依然饭特稀 6# 2012-9-3 00:01
先要找到f(x)的极小值-4/27,然后求出f(x)=-4/27的另一个根-4/3,k的最小值是过(0,0),(-4/3,-4/27)两点直线的斜率
第一章 7# 2012-10-3 23:37
我都往单调里做了,也想过直线$y=kx$,只是误以为那个极大值大于零. 顺便问问楼上,那个三次方程怎么解?利用最高次项系数和常数项?
第一章 8# 2012-10-3 23:38
晕,用多项式除法就搞定了。
thread-791-1-6.html: [不等式] 请教一道漂亮优美的三角不等式问题
ccnu_chb_ycb 1# 2012-9-2 14:43
请教这道优美的三角不等式如何证明?谢谢
kuing 2# 2012-9-2 14:54
这么巧,我在某个群里也看到有人问这道题:
kuing 3# 2012-9-2 14:55
不会就是你吧? 不过群里发的截图的排版看上去也是用 mathjax,跟楼主你的不太一样。
ccnu_chb_ycb 4# 2012-9-2 15:04
恩 ,对,就是本人。群里没有人回复,就发在这个论坛里来了。请指教! 还有,那个粉丝群我加了几次怎么都加不进去呢?我是华中师范大学一名在校学子,对于不等式问题很感兴趣,希望今后多多指教!谢谢
kuing 5# 2012-9-2 15:09
原来也不难,只是看上去稍吓人。 由万能公式及柯西不等式,有 \[\sin A\sin B=\frac{4\tan\frac A2\tan\frac B2}{\bigl(1+\tan^2\frac A2\bigr)\bigl(1+\tan^2\frac B2\bigr)} \leqslant \frac{4\tan\frac A2\tan\frac B2}{\bigl(1+\tan\frac A2\tan\frac B2\bigr)^2},\] 所以 \[\sqrt[3]{1-\sin A\sin B}\geqslant \sqrt[3]{1-\frac{4\tan\frac A2\tan\frac B2}{\bigl(1+\tan\frac A2\tan\frac B2\bigr)^2}}=\sqrt[3]{\left(\frac{1-\tan\frac A2\tan\frac B2}{1+\tan\frac A2\tan\frac B2}\right)^2},\] 记 $\tan\frac A2\tan\frac B2=z$, $\tan\frac B2\tan\frac C2=x$, $\tan\frac C2\tan\frac A2=y$,则 $x$, $y$, $z>0$, $x+y+z=1$,且有 \[\sum\sqrt[3]{1-\sin A\sin B}\geqslant\sum\sqrt[3]{\left(\frac{1-x}{1+x}\right)^2}=\sum\sqrt[3]{\left(\frac{y+z}{2x+y+z}\right)^2},\] 于是只要证 \[\sum\sqrt[3]{\left(\frac{y+z}{2x+y+z}\right)^2}\geqslant \frac32\sqrt[3]2,\] 再令 $t=y+z$, $u=z+x$, $v=x+y$,则上式化为已知的不等式 \[\sum\sqrt[3]{\left(\frac t{u+v}\right)^2}\geqslant \frac32\sqrt[3]2,\] 所以原不等式成立。 注:最后所指的已知的不等式的证明见数学空间第一期《由三元循环不等式讲起》的最后一个 Ineq。
kuing 6# 2012-9-2 15:20
恩 ,对,就是本人。群里没有人回复,就发在这个论坛里来了。请指教! 还有,那个粉丝群我加了几次怎么都加不进去呢?我是华中师范大学一名在校学子,对于不等式问题很感兴趣,希望今后多多指教!谢谢 ccnu_chb_ycb 发表于 2012-9-2 15:04 oh,原来还真是同一人。 不过既然你截图里用的是 mathjax,为什么不直接把代码复制上来,这样就可以直接在这里显示公式了,省去重新输入和传图片。 至于粉丝群,主要是快满人了,所以守得紧一些,而且如果只是讨论数学题的话可以在这里发贴就好了。
ccnu_chb_ycb 7# 2012-9-2 15:24
美丽的不等式美丽的证明,佩服!如果还有其他证法也忘指教
ccnu_chb_ycb 8# 2012-9-2 15:41
6# kuing 恩,很感谢你的指教,以后还得多多请教。相信你以后会同以我加进去的。我会努力取得您的认可哟
kuing 9# 2012-9-2 15:53
那就加吧 其实我是想数学题尽量在论坛讨论,群里闲谈吹水为主
kuing 10# 2012-9-2 16:08
7# ccnu_chb_ycb 其他方法恐怕难找,前面不是那样放缩处理的话估计很难搞下去。 所以顶多变一下后面的证明,比如对 $\sum\sqrt[3]{\bigl(\frac{1-x}{1+x}\bigr)^2}$ 用切线法也可以。
thread-792-1-6.html: [不等式] 请教一个优美不等式命题的证明
ccnu_chb_ycb 1# 2012-9-2 23:27
请教这道优美的不等式命题的证明以及解释……谢谢!
kuing 2# 2012-9-3 01:08
老结论了,没记错的话。 现在在外面,回家再给你找找证明。
kuing 3# 2012-9-3 03:55
以前存下来的两个图:
ccnu_chb_ycb 4# 2012-9-3 12:36
很好的解法,结合数列不动点理论给出了巧妙的解答……谢谢啦
kuing 5# 2012-9-4 14:27
3# kuing 后面打错了一个字母。
thread-793-1-6.html: [组合] 集合元素个数最值
hflz01 1# 2012-9-3 15:03
集合元素个数最值
kuing 2# 2012-9-4 12:55
solution by 何版主
kuing 3# 2012-9-4 14:14
何版主还是打错了一点点,下面的红色字是修改了的地方。 设 $A\cup B\cup C=U$,$\complement_U(B\cup C)=A_1$,$\complement_U(A\cup C)=B_1$,$\complement_U(A\cup B)=C_1$,$\complement_U(A_1\cup B_1\cup C)=D$,$\complement_U(A_1\cup B\cup C_1)=\color{red}E$,$\complement_U(A\cup B_1\cup C_1)=\color{red}F$,$A\cap B\cap C=G$,$\abs{A_1}=a$,$\abs{B_1}=b$,$\abs{C_1}=c$,$\abs D=d$,$\abs E=e$,$\abs F=f$,$\abs G=g$,$d+e+f=x$,则 \[\left\{\begin{aligned} a+b+x+g&=20,\\ b+c+x+g&=30,\\ c+a+x+g&=40, \end{aligned}\right.\] 解之得 \[\left\{\begin{aligned} a&=\frac{30-g-x}2,\\ b&=\frac{10-g-x}2,\\ c&=\frac{50-g-x}2, \end{aligned}\right.\] 由 $a\geqslant 0$,$b\geqslant 0$,$c\geqslant 0$ 得 $g\leqslant 10$,因此 $g$ 最大为 $10$,此时有 $a=10$,$b=0$,$c=20$,$x=0$(即 $d=e=f=0$)。 顺便加个示意图。
thread-795-1-6.html: [不等式] 自编的一个有意思的不等式问题
ccnu_chb_ycb 1# 2012-9-3 21:33
这是本人又上次一个不等式自己编的一道试题,想请教高手指教下,我只是部分的解答了,不能形成严密的系统性解答!谢谢
kuing 2# 2012-9-3 21:49
这大概应该叫“改编”好些,准确些讲应该是推广探究,因为你自己还没搞定……
kuing 3# 2012-9-3 22:15
$n=3$ 就是 http://kkkkuingggg.5d6d.net/thread-791-1-1.html; $n=2$ 时仿 $n=3$ 后面变成的是熟知的 nessbit 更简单; $n\geqslant4$ 时,前面与 $n=3$ 类似,得到 $\sum\sqrt[n]{1-\sin A\sin B}\geqslant\sum\sqrt[n]{\bigl(\frac{1-x}{1+x}\bigr)^2}$,这时就跟 $n=3$ 不一样了,这里利用 $\sqrt[n]{\bigl(\frac{1-x}{1+x}\bigr)^2}>1-x$,然后 $\sum\sqrt[n]{1-\sin A\sin B}>\sum(1-x)=2$,当 $A\to0$, $B=C\to90^\circ$ 时 $\sum\sqrt[n]{1-\sin A\sin B}\to2$,所以此时 $2$ 是下确界。
ccnu_chb_ycb 4# 2012-9-3 22:28
对的 ,我由你上次的解法进行的一些探究 ,还想到了一些其他的问题,代数不等式与三角不等式的相互转化问题 。至于n=2,3,4,6的我情形已经解决,其他情形拿不准 ,想请教下您的看法 它和安振平系列的第21个不等式很像,您曾经也解答过
kuing 5# 2012-9-3 22:37
$n\in\mbb N^+$ 容易,上面再补上 $n=1$ 就算是解决了的。 如果是 $n\in\mbb R$ 才麻烦点,大概会跟 $\log_23-1$ 扯上关系,还是要拿些已知结论出来……
ccnu_chb_ycb 6# 2012-9-3 23:03
3# kuing 还有就是当n大于3时的那个不等式是如何想到的呢?
ccnu_chb_ycb 7# 2012-9-3 23:05
大概知道啦,利用了幂函数的单调性就ok啦
ccnu_chb_ycb 8# 2012-9-7 20:16
3# kuing 最后那个2不是下界,因为极限是3,有问题
thread-8-1-1.html: 在 Opera FireFox 下亦显示正常
isea 1# 2011-9-25 23:16
RT,不错。
kuing 2# 2011-9-25 23:18
:lol 多谢支持
isea 3# 2011-9-25 23:19
不过,得普及latex基本语法啊,否则只能看不能用,哈哈。
叶剑飞Victor 4# 2012-8-26 11:30
本帖最后由 叶剑飞Victor 于 2012-8-26 11:37 编辑 3# isea 是的,确实需要普及。 http://filestorage.7host08.com/ftp/一份不太简短的LaTeX2e介绍.pdf 直接看第三章“数学公式”吧。
thread-80-1-8.html: [不等式] How to A-G it?
pxchg1200 1# 2011-10-9 23:31
设$a,b,c \in R^{+} $ 且$a+b+c=3 $,prove that: \[ a\sqrt{a+b}+b\sqrt{b+c}+c\sqrt{c+a}\geq 3\sqrt{2}\]
天涯无际 2# 2012-3-1 20:23
AG的方法没有...不过用别的方法证明也很快的...
天涯无际 3# 2012-3-2 10:25
证明:由于欲证不等式轮换对称,故可设$c=mid(a,b,c)$. 首先证明此时有\\ \[b\sqrt{b+c}+c\sqrt{c+a}\geq b\sqrt{a+b}+c\sqrt{2c}\] 两边平方等价于 \[b^2c+c^2a+2bc\sqrt{(b+c)(c+a)}\geq b^2a+c^3+2bc\sqrt{2c(a+b)}\] \[\Longleftrightarrow (a-c)(c^2-b^2)\geq 2bc\left(\sqrt{2c(a+b)}-\sqrt{(b+c)(c+a)}\right)\] \[\Longleftrightarrow b+c\geq \frac{2bc}{\sqrt{2c(a+b)}+\sqrt{(a+c)(b+c)}}\] 而由均值不等式有$b+c\geq 2\sqrt{bc}$,代入上式则只需证明 \[\sqrt{2c(a+b)}+\sqrt{(a+c)(b+c)}\geq \sqrt{bc}\] 显然成立.由此,结合欲证不等式,只需证明 \[b\sqrt{a+b}+c\sqrt{2c}+a\sqrt{a+b}\geq 3\sqrt{2}\] 也即证明 \[f(c)=(3-c)\sqrt{3-c}+c\sqrt{2c}\geq 3\sqrt{2}\] 而$f'(c)=\frac{3}{2}(\sqrt{2c}-\sqrt{3-c})$,易得到$f'(1)=0$且在$c=1$处取得极小值.于是有 \[f(c)\geq f(1)=3\sqrt{2}\] 于是原不等式得证.当且仅当$a=b=c$时取得等号.
pxchg1200 4# 2012-3-2 18:49
[/b][b] [url=http://kkkkuingggg.5d6d.com/redirect.php?goto=findpost&pid=2070&ptid=80]3#[/url] [i]天涯无际[/i] 赞一个!
thread-801-1-6.html: 请教一个绝对值的问题
abababa 1# 2012-9-4 15:07
本帖最后由 abababa 于 2012-9-4 15:20 编辑 请教一个绝对值的问题: $|ax+by+cz|+|bx+cy+az|+|cx+ay+bz|=|x|+|y|+|z|$,对于任意实数$x,y,z$都成立,求$a,b,c$的值。 我知道答案,$a、b、c$三个中有一个取1或-1,另两个取0就可以,但是为什么呢?
kuing 2# 2012-9-4 15:55
令 $x=y=0$, $z=1$ 得 $\abs a+\abs b+\abs c=1$; 令 $x=y=z=1$ 得 $\abs{a+b+c}=1$,所以 $a$, $b$, $c$ 必定同号(约定 $0$ 与任何实数同号); 令 $x=0$, $y=1$, $z=-1$ 得 $\abs{b-c}+\abs{c-a}+\abs{a-b}=2$,由全对称性,不妨设 $a\geqslant b\geqslant c$,则去绝对值后得 $a-c=1$; 由此,我们有 \[1=\abs a+\abs b+\abs c\geqslant \abs b+\abs{a-c}=\abs b+1,\] 所以必有 $b=0$,从而有 $\abs a+\abs c=1$,如果 $a$, $c$ 同为非负,则 $c=0$,故 $a=1$;如果 $a$, $c$ 同为非正,则 $a=0$,故 $c=-1$。
abababa 3# 2012-9-4 17:22
谢谢。 请问为什么从|a+b+c|=|a|+|b|+|c|=1就能推出abc是同号的呢?请再指点指点好吗?
kuing 4# 2012-9-4 17:31
3# abababa 绝对值不等式的取等条件
abababa 5# 2012-9-4 17:38
4# kuing 谢谢,我看懂了。
thread-802-1-4.html: [组合] 请问一个棋盘的问题
abababa 1# 2012-9-5 20:09
本帖最后由 abababa 于 2013-3-5 18:12 编辑 题和解答是在别处看到的,感觉有问题,请帮助看看。 觉得“据此知,第1列的第2至第n个方格内的各数之和不小于n-(n-m)=m”那里有问题,如果设第一行所有数之和为n-m+x,第一列所有数之和为y,那么有n-m+x+y>=n,这样只能得到y>=m-x,还得不到y>=m,下面那个式子就不一定成立了。是我的理解有问题吗?这个题应该怎么解?
abababa 2# 2012-12-31 18:29
沉了顶一下,谢谢大家关注
realnumber 3# 2013-1-1 08:48
本帖最后由 realnumber 于 2013-1-1 08:49 编辑 想到一个整体办法,整个棋盘有$n^2$个数,设这$n^2$个数的和为$S$,那么$2nS\ge{n^2n}$,也即$S\ge{\frac{n^2}{2}}$. ---好象就这么简单啊.不等式左边是把$a_{ij}$对应的行列都相加$(i=1,2,..n;j=1,2,..n)$,总和就是$2nS$,因为每个$a_{ij}$都被重复加了2n次.
abababa 4# 2013-1-1 10:38
3# realnumber 谢谢。但没看懂,右边的$n^2*n$怎么解释呢?
realnumber 5# 2013-1-1 10:44
$a_{ij}$有$n^2个$啊,题目就有这句话,每个行列和不小于n
abababa 6# 2013-1-1 10:49
本帖最后由 abababa 于 2013-1-1 11:09 编辑 5# realnumber 要是每行每列的和不大于$n$才能用大于等于号吧,题目只说是0时一定不小于$n$,没说不是0的时候元素所在行列的和就一定不大于$n$ 可能理解得不对,我有点绕迷糊了 我再想想,比如设这$n^2$个数里有$x$个是0,那么这$x$个数所在行列所有数之和$X$不小于$xn$,另外$y$个数不是0,$x+y=n^2$ 这$y$个数所在行列所有数之和设为$Y$,这样能得到$2nS=X+Y$,其中$X \geqslant xn$,能得到$Y+xn \geqslant 2nS$,但$Y$和$x$都不一定,还是没法确定,如果设这$y$个不为0的元素所在行列之和也都不小于$n$,这样能得到$Y \geqslant yn$,就得$2nS=X+Y \geqslant (x+y)n=n^3$,但目前不能确定这$y$个不为0的元素所在行列之和都不小于$n$
realnumber 7# 2013-1-1 19:48
恩,看来我错了,果然是幻觉
abababa 8# 2013-2-4 23:32
请教了一位网友,当时他没给出解答,今天又问了一次,说已经解出来一阵了,一直没给我发,呵呵,把答案放上来 设方格中所有数之和为$S$,设每行各数之和为$A_1, A_2, \cdots , A_n$,每列各数之和为$B_1, B_2, \cdots , B_n$ $A_i, B_i$中必有最小的,不妨设$A_1$最小,若$A_1 \geqslant n$,则$S \geqslant nA_1 \geqslant n^2 > \frac{n^2}{2}$ 若$A_1 < n$,设第$1$行有$t$个$0$ 若$t < n - A_1$,则第$1$行所有数之和最小为$1 \cdot (n-t) + 0 \cdot t = n-t > A_1$,与第$1$行所有数之和为$A_1$矛盾 所以$t \geqslant n - A_1$,不妨设第$1$行含有$0$的方格的列为$B_1, B_2, \cdots , B_t$ 由于行列交界处为$0$时行列所有数之和不小于$n$,所以$B_1 + A_1 \geqslant n, B_2 + A_1 \geqslant n, \cdots, B_t + A_1 \geqslant n$ 由于$A_1$是所有行列各数之和中最小的,所以$B_{t+1} \geqslant A_1, B_{t+2} \geqslant A_1, \cdots , B_n \geqslant A_1$ 所以$S - \frac{n^2}{2} = \sum_{i=1}^{t}B_i + \sum_{i=t+1}^{n}B_i - \frac{n^2}{2} \geqslant (n - A_1)t + (n-t)A_1- \frac{n^2}{2} = \frac{(n-2A_1)(2t-n)}{2}$ 由于$t \geqslant n-A_1$,所以$2t-n \geqslant n-2A_1$,所以$S-\frac{n^2}{2} \geqslant \frac{(n-2A_1)^2}{2} \geqslant 0$,即$S \geqslant \frac{n^2}{2}$
thread-803-1-6.html: [不等式] 来自群的一道轮换不等式
kuing 1# 2012-9-5 22:37
提问人“哦”了,但不知发哪去了。 其实我并不是一看就知道怎么证的,但看上去应该不会太难,就直接答应电脑上给回复,其实现在回想下也感觉有点险,一开始想SOS结果不成功,幸好后来还是CS+AG证到了。 不过,到证出来的时候,人又走了哎……只能发到这里先。
kuing 2# 2012-9-5 22:44
\begin{align*} \sum{\frac{a^4}{a^2+ab+b^2}}\geqslant\frac{a^3+b^3+c^3}{a+b+c}&\iff\sum{\frac{a^4}{a^2+ab+b^2}}\geqslant \sum{a^2}-\sum{ab}+\frac{3abc}{a+b+c} \\ & \iff\sum{\left( \frac{a^4}{a^2+ab+b^2}-a^2+ab \right)}\geqslant\frac{3abc}{a+b+c} \\ & \iff\sum{\frac{ab^3}{a^2+ab+b^2}}\geqslant\frac{3abc}{a+b+c} \\ & \iff\sum{\frac{b^2}{c(a^2+ab+b^2)}}\geqslant\frac3{a+b+c}, \end{align*} 由 CS and AG 得 \[\sum{\frac{b^2}{c(a^2+ab+b^2)}}\geqslant\frac{(a+b+c)^2}{\sum{c(a^2+ab+b^2)}}=\frac{a+b+c}{ab+bc+ca}\geqslant \frac3{a+b+c}.\]
thread-804-1-4.html: [不等式] 来自群的似曾相识三角形绝对值最大值
kuing 1# 2012-9-6 02:50

kuing 2# 2012-9-6 02:53
所指的“似曾相识”是这个:http://bbs.pep.com.cn/forum.php?mod=viewthread&tid=503423 但本题有“锐角三角形”条件,故有所不同,有空再玩。
kuing 3# 2013-2-10 21:29
今天人教群里又提起这道题,说来是巧,这两天都在搞这种轮换差式的绝对值问题……不过刚才目测了一下,似乎这道的数据并不好算……
kuing 4# 2013-2-10 23:53
竟然出现了次数很高的东西…… 首先用正弦定理并且通分,有 \[\left| \sum\frac{\sin A-\sin B}{\sin A+\sin B} \right|=\frac{\abs{(a-b)(b-c)(c-a)}}{(a+b)(b+c)(c+a)},\] 若三边中有两边相等则原式为 $0$,下设三边都不相等,由对称性,不妨设 $a<b<c$,则可令 $b=a+t$, $c=a+t+u$, $t$, $u>0$,由锐角三角形条件知 \[a^2+b^2>c^2\iff a^2+(a+t)^2>(a+t+u)^2,\] 解得 \[a>u+\sqrt{2u(t+u)},\] 令 $m=\sqrt{2u(t+u)}$,则 \begin{align*} \left| \sum\frac{\sin A-\sin B}{\sin A+\sin B} \right|&=\frac{tu(t+u)}{(2a+t)(2a+2t+u)(2a+t+u)} \\ & <\frac{tu(t+u)}{(2m+t+2u)(2m+2t+3u)(2m+t+3u)}, \end{align*} 由齐次性,不妨设 $u=1$,则 $m>\sqrt2$ 且 $t=m^2/2-1$,代入上式化简即得 \[\left| \sum\frac{\sin A-\sin B}{\sin A+\sin B} \right|<\frac{m^2(m^2-2)}{(m+1)^2(m+2)^2(m^2+4m+2)}=g(m),\] 下面求 $g(m)$ 在 $\bigl(\sqrt2,+\infty\bigr)$ 上的最大值。 由于 $g(m)$ 在 $\bigl(\sqrt2,+\infty\bigr)$ 上连续,恒正,可导,且易见 \[\lim_{m\to\sqrt2}g(m)=\lim_{m\to+\infty}g(m)=0,\] 所以 $g(m)$ 在 $\bigl(\sqrt2,+\infty\bigr)$ 上存在最大值且取得最大值的点必然为导数为 $0$,设 \[\max_{m>\sqrt2}g(m)=g(m_0)=M,\] 即有方程组 \[\left\{\begin{aligned} g'(m_0)&=0,\\ g(m_0)&=M, \end{aligned}\right.\] 用结式消去 $m_0$,可以得到 $M$ 是如下方程的一个根 \[M^6-161M^4-18625M^2+1=0,\] 这个方程只有两个正数根,其精确解就不写出来了,只写出其近似值分别为 $0.00732743$ 及 $15.4579$,又显然能看出当 $m>\sqrt2$ 时有 $g(m)<1$,所以后者舍去,最终得到 \[M\approx 0.00732743,\] 这就是 $g(m)$ 在 $\bigl(\sqrt2,+\infty\bigr)$ 上的最大值。 再者,由证明过程知当 $a^2+b^2\to c^2$ 且 $u=1$, $m=m_0$ 时原式能趋向 $M$,因此原式实际上无最大值,$M$ 是它的上确界,若要取得最大值,只要将条件改为“非钝角三角形”。
kuing 5# 2013-2-11 00:00
4# kuing 哎,如果将“锐角三角形”去掉就简单多了,小小一个条件就把问题搞得N复杂,不上软件后面都不敢算哩……
thread-805-1-2.html: mat又催第十期了
kuing 1# 2012-9-6 03:02
哎,越整越没意思,评价越来越差,越来越少人看,更越来越没稿子了,这是一个恶性循环…… 没有新生源加入,实在难搞…… 不过话说回来,催什么催!都没心思了还催!每次都催!上次弄好交上去还不是被拖了一个多月才刊出!还好意思催! 水饺
yizhong 2# 2012-9-17 21:59
oh~~~可怜的小k~~~~支持小k的论坛~~~~
kuing 3# 2012-9-17 22:41
前几天上不了网,无聊起来才整了一下这个第十期,连续输入了两篇比较长的东东。 还好有 word2tex,而且我也学会了点 winedt 的正则表达式,所以改起来已经相对简单了不少,但他们稿子的公式输入实在是比较乱,各种麻烦的修改and无语……
kuing 4# 2012-10-10 23:56
大概弄好了,各种拖拉……这期暂定不出能力提升一栏了
thread-806-1-6.html: [数论] 请问一个整数的题
abababa 1# 2012-9-6 19:37
设m,n是正整数,$0<\sqrt{n}-\sqrt{m}<1$,求证不能从区间$[m,n]$中选出四个不同的整数a、b、c、d,使得ab=cd
yizhong 2# 2012-9-12 15:19
此题已想到好的解决方法,现在手机党中,晚上或者明天补上.
yizhong 3# 2012-9-12 15:20
本帖最后由 yizhong 于 2012-9-14 13:49 编辑 证明如下:我们可以设:$m=e^2+f,  n=e^2+f+h     f为自然数,e,h为正整数,并且4\leqslant h<2e+1想想为什么这样设?$   并且我们设:$e^2+f\leqslant a<b<c<d\leqslant e^2+f+h$ 因为:$ab<cd, ac<bd$,所以我们只需要去证明:$ad\ne bc$即可。接下来我们用反证法,证法如下: 假设:$ad=bc$,则我们可设:$\frac{d}{b}=\frac{c}{a}=\frac{p}{q}(1),其中p>q,(p,q)=1,p,q为正整数$ 于是我们就有:$p\geqslant q+1,  \frac{p}{q}\geqslant1+\frac{1}{q}$,  由(1)我们可以知道:a,b是q的倍数 且b>a,则有:$b\geqslant a+q,  \frac{p}{q}=\frac{d}{b}\leqslant\frac{d}{a+q}\leqslant  \frac{e^2+f+h}{e^2+f+q}$ 于是我们就有:$1+\frac{1}{q}\leqslant\frac{p}{q}\leqslant  \frac{e^2+f+h}{e^2+f+q}(2),而我们又有:e^2+q^2+f+q\geqslant 2eq+f+q,  hq<(2e+1)q=2eq+q(这里利用到了题设),所以(2)式是不能够成立的$ 矛盾,故有:$ad\ne bc$
yizhong 4# 2012-9-13 15:09
今天就先到这里吧,昨天由于爪机所以重复发帖了,所以今天的解答就在楼上
abababa 5# 2012-9-13 16:46
3# yizhong 谢谢。 第一步的等价没看懂,如果是$\sqrt{10}-\sqrt{5}<1$,怎么设这两个平方数呢?请再提示一下好吗? 我按3楼的提示做了一下,从“证明如下”开始到“a,b是q的倍数”都是按照3楼的提示,然后是 $\frac{q+1}{q} \leqslant \frac{p}{q} = \frac{d}{b} \leqslant \frac{d}{a+q} \leqslant \frac{n}{a+q} \leqslant \frac{n}{m+q}$,所以得到$q^2+m+q+(m-n)q \leqslant 0$ 然后$1 \leqslant q$,得到$1+m+1+(m-n) \leqslant q^2+m+q+(m-n)q \leqslant 0$,即$2(m+1) \leqslant n$ 所以$\sqrt{2(m+1)}\leqslant\sqrt{n}<\sqrt{m}+1$,两边平方得$m-2\sqrt{m}+1<0$,就是$(\sqrt{m}-1)^2<0$,得出矛盾。
yizhong 6# 2012-9-13 18:46
好吧,今天我比较匆忙,改天再详细写下,其实道理都差不多,我省略了一些
yizhong 7# 2012-9-14 13:54
本帖最后由 yizhong 于 2012-9-14 14:03 编辑 重新写过的证明照旧发在3楼  5楼的朋友你可以再详细看下 另外事实上那个h是可以为2e+1的
abababa 8# 2012-9-14 19:30
7# yizhong 谢谢,我理解了,非常感谢。
thread-807-1-6.html: [组合] 一个正整数集合元素个数的最小值问题
hflz01 1# 2012-9-7 10:22
一个正整数集合元素个数的最小值问题 集 合M由整数组成,其中最小元素为1,最大元素为100,M中除1以外每个数都等于该集 合中两数(允许是相同的)定额管理,求M元素个数最小可能的值。并证明你的结论。
海盗船长 2# 2012-9-7 14:58
本帖最后由 海盗船长 于 2012-9-7 15:14 编辑 $\{1,2,4,8,16,32,36,64,100\}$ $\{1,2,4,8,9,16,25,50,100\}$ 感觉找不到更少的了。。
海盗船长 3# 2012-9-7 15:11
本帖最后由 海盗船长 于 2012-9-7 15:18 编辑 至少有一个数在$[50,100)$中 有一个数在$[25,50)$中 有一个数在$[13,25)$中 有一个数在$[7,13)$中 有一个数在$[4,7)$中 有一个数在$[2,4)$中 然后这个集合至少有$8$个元素,只要证明$8$不可能就行 但是如果上面每个区间里只有一个数的话必然是$2^n$的形式,所以不可能
hflz01 4# 2012-9-7 23:10
船长威武,赞一个!
thread-808-1-6.html: [不等式] 优美的不等式证明
ccnu_chb_ycb 1# 2012-9-7 18:25
这个不等式如何证明?
kuing 2# 2012-9-7 20:03
上次你那帖(http://kkkkuingggg.5d6d.net/thread-795-1-1.html)我说得不够清楚吗?
ccnu_chb_ycb 3# 2012-9-7 20:18
上次的解答有问题,尤其是那个下界2,你再看下你的答案,这个是我研究出来的一个结果,比上次的结果要好
kuing 4# 2012-9-7 20:23
那极限? 你再看清楚那A,B,C分别趋向什么?
kuing 5# 2012-9-10 23:00
\begin{align*} A\to0, B=C\to90^\circ &\riff \sin A\sin B\to0, \sin B\sin C\to1, \sin C\sin A\to0\\ & \riff \sqrt[n]{1-\sin A\sin B}\to1, \sqrt[n]{1-\sin B\sin C}\to0, \sqrt[n]{1-\sin C\sin A}\to1\\ & \riff \sum\sqrt[n]{1-\sin A\sin B}\to1+0+1=2<\frac3{\sqrt[n]4}\text{ (when $n\geqslant4$)} \end{align*} 这样够详细吗?
kuing 6# 2012-9-22 12:50
楼主呢?
kuing 7# 2012-10-5 13:58
喂?
thread-809-1-5.html: [组合] 一道排列组合题
nash 1# 2012-9-10 03:58
在一个4X4的16个小方格中填入2个a和2个b,使得相同的字母既不在同一行也不在同一列,有多少种不同的填法
realnumber 2# 2012-9-12 14:15
本帖最后由 realnumber 于 2012-9-12 14:18 编辑 先解决这个 在一个4X4的16个小方格中填入四个不同字母a,b,c,d,使得任意2个的字母既不在同一行也不在同一列,有多少种不同的填法   依次填入a,b,c,d那么有16X9X4(开始填第一个字母有16个位置可以选择,填好后,按要求,第2个字母只有9个位置了可以选择了,...) 设1楼问题有x种填法,那么4x=16X9X4,所以x=16X9=144
realnumber 3# 2012-9-12 14:21
如果问题为nXm棋盘,有k1个a,k2个b,按1楼要求,应该也可以解决了
nash 4# 2012-9-12 14:33
2# realnumber 谢谢 不过不同的字母可以在同一行或者同一列
realnumber 5# 2012-9-12 18:45
恩。那这样,分类讨论如何,先放置2个a有16×9÷2=72种 如何a,b不同行列,有72×2种 如果仅一个b与a,同行或列,那么有72×6×4(6是直接画图,数出来的) 如果有2个b都与a,同行或列72×(1+)....有些混乱了
realnumber 6# 2013-1-20 09:43
本帖最后由 realnumber 于 2013-1-20 13:11 编辑 如果问题为nXm棋盘,有2个a,2个b,按1楼要求, 运用逐步淘汰原则,\[(2C_n^2C_m^2)^2-C_n^1C_m^1(C_{n-1}^1C_{m-1}^1)^2+2C_n^2C_m^2\] 第一项,行列选出2个后,放置a,b有2种类;第二项,a,b有一个重叠了,先放置重叠的(ab);第三项,都重叠了ab,ab. 1楼问题就是$m=n=4$,答案即为$72^2-36^2+72$.---不清楚有没疏忽的地方.如此三个字母似乎也不是问题. 在一个4X4X4的64个小立方体构成的立方体中填入2个a和2个b,使得相同的字母既不在同一行也不在同一列也不在同一竖,有多少种不同的填法?
thread-81-1-8.html: [不等式] how to CS it? (2)
pxchg1200 1# 2011-10-9 23:41
$a,b,c,x,y,z \geq 0 $ with $x+y+z=a+b+c $ show that: \[ ax^{2}+by^{2}+cz^{2}+xyz\ge 4abc \]
天涯无际 2# 2012-3-2 13:42
本帖最后由 天涯无际 于 2012-3-2 13:44 编辑 证明:假设其中一个数为$0$, 不妨设$z=0$, 于是有$a+b+c=x+y$, 此时注意到 $(ax^2+by^2)(a+b)-ab(x+y)^2=(ax-by)^2\geq 0$, 于是有 \[ax^2+by^2\geq \frac{ab(x+y)^2}{a+b}=\frac{ab(a+b+c)^2}{a+b}\geq 4abc\] 则原不等式成立.所以我们只需考虑$a,b,c,x,y,z$均为正数。 首先证明一个引理:对正数$a,b,c,x,y,z$满足$a+b+c=x+y+z$, 则 \[ax(a+x)+by(b+y)+cz(c+z)\geq 3(abc+xyz)\] 引理证明:由$Cauchy$不等式有 \[a^2x+b^2y+c^2z\geq \frac{(a+b+c)^2}{\frac{1}{x}+\frac{1}{y}+\frac{1}{z}}=\frac{xyz(x+y+z)^2}{xy+yz+zx}\] \[ax^2+by^2+cz^2\geq \frac{(x+y+z)^2}{\frac{1}{a}+\frac{1}{b}+\frac{1}{c}}=\frac{abc(a+b+c)^2}{ab+bc+ca}\] 以上两式相加可得 \[ax(a+x)+by(b+y)+cz(c+z)\geq \frac{xyz(x+y+z)^2}{xy+yz+zx}+\frac{abc(a+b+c)^2}{ab+bc+ca}\geq 3(abc+xyz)\] 则引理得证. 回到原不等式,采用反证法,假设原不等式不成立,则有$ax^2+by^2+cz^2+xyz<4abc$, 则由原不等式的对称性, 将$a,b,c$ 与$x,y,z$置换,则亦有$xa^2+yb^2+zc^2+abc<4xyz$,将这两式相加,即有$ax(a+x)+by(b+y)+cz(c+z)<3(abc+xyz)$,与引理矛盾!故假设不成立.于是原不等式得证.
thread-810-1-4.html: 证明直角三角形(求简便方法)
nash 1# 2012-9-10 05:08
三角形ABC中,sin2A+sin2B=4sinAsinB,证明三角形是直角三角形
kuing 2# 2012-9-10 08:27
以前好像在不知哪个群里聊过,等我能用电脑上再给你找记录。
kuing 3# 2012-9-10 10:00
原来也发过在人教论坛上,还好我有存档,不能上网也能找到帖子,帖的ID是652343,你进人教论坛进随便一个帖子,将链接中的数字换成此数字即可进入我所说的帖。
kuing 4# 2012-10-7 12:56
楼主不见了。 莫非由于没将楼上所说的链接直接贴出就…… 呃,那就贴吧 http://bbs.pep.com.cn/thread-652343-1-1.html
nash 5# 2012-10-7 23:42
4# kuing ,当时这个ID是找过,没找到… 后来那几天比较忙可能,忘记这事了
kuing 6# 2012-10-7 23:45
5# nash 不就是替换一下数字就可以的么……
nash 7# 2012-10-8 00:05
貌似当时的地址栏和这个不一样,然后就杯具了,计算机白痴伤不起呀…
kuing 8# 2012-10-8 00:10
7# nash 没所谓的,只要进随便一个贴(最好是在主题列表里点,不要在最外面点)替换掉差不位数的那个数字就是了。
realnumber 9# 2012-12-2 20:40
本帖最后由 realnumber 于 2012-12-2 21:54 编辑 没简便,可能有人还发过 $\sin2A+\sin2B=2\sin(A+B)\cos(A-B)=4\sin A\sin B=2[\cos(A-B)-\cos(A+B)]$ $\cos(A+B)=\cos(A-B)[1-\sin(A+B)]$ $\sin[0.5\pi-(A+B)]=\cos(A-B)\{1-\cos[0.5\pi-(A+B)]\}$ $\cos[0.25\pi-0.5(A+B)]=\cos(A-B)\sin[0.25\pi-0.5(A+B)]$或$\sin[0.25\pi-0.5(A+B)]=0$ $\frac{1}{\tan[0.25\pi-0.5(A+B)]}=\cos(A-B)$或$A+B=0.5\pi$ $1\ge  \cos(A-B) >-1$,$\frac{1}{\tan[0.25\pi-0.5(A+B)]}>1$或$\frac{1}{\tan[0.25\pi-0.5(A+B)]}< -1$ 即$\frac{1}{\tan[0.25\pi-0.5(A+B)]}=\cos(A-B)$无解
kuing 10# 2012-12-2 21:23
9# realnumber sin -> \sin 等
yes94 11# 2013-2-1 15:51
本帖最后由 yes94 于 2013-2-1 15:57 编辑 9# realnumber 写成0.5,0.25很难看懂,重新写下代码,看看过程, 没简便,可能有人还发过 $\sin2A+\sin2B=2\sin(A+B)\cos(A−B)=4\sin{A}\sin{B}=2[\cos(A−B)−\cos(A+B)]$ $\cos(A+B)=\cos(A−B)[1−\sin(A+B)]$ $\sin[\dfracπ2−(A+B)]=\cos(A−B)[1−\cos(\dfracπ2−A-B)]$ $\cos[\dfracπ4−\dfrac{1}{2}(A+B)]=\cos(A−B)\sin[\dfracπ4−\dfrac{1}{2}(A+B)]$    或$\sin[\dfracπ4−\dfrac{1}{2}(A+B)]=0$ $\dfrac{1}{\tan[\dfracπ4−\dfrac{1}{2}(A+B)]}=\cos(A−B)$或$A+B=\dfracπ2$ $1≥\cos(A−B)>−1$,$\dfrac{1}{\tan[\dfracπ4−\dfrac{1}{2}(A+B)]}>1$    或$\dfrac{1}{\tan[\dfracπ4−\dfrac{1}{2}(A+B)]}<−1$ 即$\dfrac{1}{\tan[\dfracπ4−\dfrac{1}{2}(A+B)]}=\cos(A−B)$无解
thread-811-1-2.html: 关于经典称球问题
kuing 1# 2012-9-10 22:45
题目: 有12个球,形状、颜色、大小等完全一样,看不出区别,其中只有一个球的质量与其他球不同(其他十一个球的质量相同)。给你一架两边是托盘的天平,让你称三次,找出这个球,并且要弄清它的质量比其他球重还是轻。 这个经典问题,我第一次看的时候不会,后来在论坛上看到解答,大致还记得,刚才上Q有人问到,我试着回忆一下: 为方便表达,以下简称那11个质量相同的球为“好球”,另一个为“坏球”。 天平两边各放4个球。(称第1次) ①如果平衡,则所称的8个为好球,坏球在没称的4个中,记这4个球为ABCD。   拿3个好球与ABC称一下。(称第2次)     如果平衡,则D是坏球,再拿一个好球与D称一下便知其是轻还是重。(称第3次,搞定)     如果不平衡,则坏球在ABC中,并且根据偏向已能确定坏球是轻还是重。     A与B称一下。(称第3次)       如果平衡,坏球是C。(搞定)       如果不平衡,根据轻重也能判断哪个是坏球。(搞定) ②如果不平衡,则坏球在所称的8个球中但未能确定在哪一边,记较重一边的四个球为ABCD,另一边的为EFGH。   取下GH,并将CD与F调换位置再称,即ABF与ECD称一下。(称第2次)     如果平衡,则坏球在GH中且必定是较轻,再拿一个好球与G或H称一下便知哪个是坏球。(称第3次,搞定)     如果ABF较重,则CDFGH都是好球1,即ABE中有坏球。     A与B称一下。(称第3次)       如果平衡,坏球是E且较轻。(搞定)       如果不平衡,较重的一边是坏球。(搞定)     如果ABF较轻,则CDF中有坏球2。     C与D称一下。(称第3次)       如果平衡,坏球是F且较轻。(搞定)       如果不平衡,较重的一边是坏球。(搞定) 完。 _____________  1假如GH中有坏球,则此时应该平衡;假如CDF中有坏球,由于CD与F换了位置,则此时应该ABF较轻。  2理由同上。
kuing 2# 2012-9-10 22:48
又印象中记得有人还提到过更一般的推广,具体我实在是想不起来了,哪位知道的烦请告诉我一声。
kuing 3# 2012-9-21 03:30
如果只需找出坏球而无需判断是轻了还是重了,印象中可以增加到13个球,不知还能不能再多,还是要继续求推广结论ing。。。
Gauss门徒 4# 2013-4-15 21:06
$\text{1986 China TST}$
yes94 5# 2013-4-15 21:44
以前程序框图见过这个题,但全忘了怎么称的 但是程序设计的很简单,是枚举法,要循环很多次,但对计算机是相当轻松的,对人就麻烦啦, 说明程序可以优化到使计算机循环3次就可以得到答案。
转化与化归 6# 2013-4-15 22:22
2# kuing 找了找,可能不完整!
yes94 7# 2013-4-17 13:34
6# 转化与化归 资料库啊
thread-812-1-6.html: [不等式] 来自群的一道看上去有点玩头的三角函数不等式
kuing 1# 2012-9-11 03:15
$a$, $b\in(0,\pi/4)$ and $n\in\mbb N$ \[\frac{\sin^na+\sin^nb}{(\sin a+\sin b)^n}\geqslant\frac{\sin^n2a+\sin^n2b}{(\sin2a+\sin2b)^n}.\]
realnumber 2# 2012-9-11 16:47
本帖最后由 realnumber 于 2012-9-11 16:50 编辑 想出来了,不妨设a≥b ,要证明的就这个$x^n+(1-x)^n≥y^n+(1-y)^n$,$x=\frac{sina}{sina+sinb},y=\frac{sin2a}{sin2a+sin2b}$ 而函数$f(x)=x^n+(1-x)^n $,在0<x<0.5单调递减,在1>x>0.5单调递增 如此要证明原不等式成立,只需要证明x≥y,这个简单,化简下,等价于cosb≥cosa,成立
kuing 3# 2012-9-11 16:59
2# realnumber nice! long time no see 啊
realnumber 4# 2012-9-11 17:20
心神不宁,休息段时间
thread-814-1-6.html: [不等式] lbq的三角形三边等差取等的三角函数不等式
kuing 1# 2012-9-11 22:11
在 $\triangle ABC$ 中,求证 \[\cos^3\frac B2\cos^3\frac C2\geqslant\frac{27}4\sin^2\frac A2\sin\frac B2\sin\frac C2.\] 转自 http://www.irgoc.org/viewtopic.php?f=28&t=1064
kuing 2# 2012-9-11 22:44
lbq 的证法有点复杂,其实用积化和差很简单。 记 $t=\cos\frac{B+C}2$, $u=\cos\frac{B-C}2$,则 $t$, $u>0$,且由积化和差公式,有 \begin{align*} \cos\frac B2\cos\frac C2&=\frac{t+u}2,\\ \sin\frac B2\sin\frac C2&=\frac{u-t}2,\\ \sin\frac A2&=t, \end{align*} 所以原不等式等价于 \[\left(\frac{t+u}2\right)^3\geqslant \frac{27}8t^2(u-t)\iff\frac18(2t-u)^2(7t+u)\geqslant 0,\] 显然成立,等号成立当且仅当 \begin{align*} 2t=u &\iff 3t=t+u\\ &\iff 3\sin\frac A2=2\cos\frac B2\cos\frac C2\\ &\iff 3\sin\frac A2\cos\frac A2=2\cos\frac A2\cos\frac B2\cos\frac C2\\ &\iff \frac32\sin A=\frac12(\sin A+\sin B+\sin C)\\ &\iff 2\sin A=\sin B+\sin C\\ &\iff 2a=b+c, \end{align*} 即仅当三角形三边成等差数列时取等号。
thread-815-1-1.html: 求Mm
Chetion 1# 2012-9-12 21:50
今天在网上海翻了一遍。Kuing,能不能共享一下mathematica的资源,或者是各位高手们给出链接。 我承认是自己不愿意花钱买。。。。
kuing 2# 2012-9-12 22:08
当年我搜其于电骡而得
thread-816-1-1.html: 极限题
Chetion 1# 2012-9-12 21:55
本帖最后由 Chetion 于 2012-9-12 21:56 编辑 求证: $\lim_{n\to+\infty}\sqrt[n]{n!},=+\infty $
kuing 2# 2012-9-12 22:24
大概就是倒序相乘,然后放缩(不是均值,是直接放成 $n$)
thread-817-1-6.html: 计算器 abacus 1.0 发布
都市侠影 1# 2012-9-12 22:04
本帖最后由 都市侠影 于 2012-9-29 23:09 编辑 一、简介         abacus 是一个功能强大,扩展性良好的小型开源计算器,得名于中国古代著名的     计算工具----算盘,目前版本 1.0。         软件维护地址:                 http://www.oschina.net/code/snippet_736932_13725         欢迎使用并报告问题或者提出建议,联系方式:                 E-mail: zhoucosin@163.com                 QQ    : 532319166 二、使用方法     + Windows系统:       双击解压出来的 abacus_1.0\windows\vs2008\abacus\Debugabacus.exe 即可运行,       推荐把此文件放到 C:\WINDOWS\system32 目录中去,便可以在命令行终端中直接       通过程序名字 abacus 运行此程序。       【编译】压缩包目录 abacus_1.0\windows\vs2008\abacus 中包含一个windows 下       的 visual studio 2008 的解决方案,可以直接编译。         + linux系统:       【编译】需要自己编译后使用,src 是源文件存放目录,在 linux 目录下有一个       makefile 文件,在终端中切换到此目录,执行如下两条命令:                         make             sudo make install           即可完成编译,将在 abacus_1.0/linux/bin 中生成可执行文件,同时复制到了系       统目录 /bin 下,可以直接作为系统程序运行。 三、功能     1. 支持常规四则混合运算.     2. 支持众多的数学函数,例如三角函数与组合函数,关于函数的列表,请查看软件帮助.     3. 支持符号常量,如圆周率 pi 与自然对数的底数 e.     4. 设计良好,便于添加新的运算符和新的数学函数     5. 帮助信息完善,程序运算时,随时可以输入 h 或者 help 查看帮助     6. 在接下来的 2.0 版本中将支持宏定义功能,例如,用户可以像这样定义一个宏来求解        一元二次方程:                 #define  slove(a,b,c)  (-b+sqrt(b^2-4*a*c))/(2*a)        之后,要求解方程 x^2-3x+2=0,可以直接调用此宏:                 slove(1,-3,2)                程序将输出:                 result=                         2        此外,还将提供对宏的管理功能,即可以自由查看、增删、修改已经定义的宏。 四、设计特色     1. 统一处理运算符和函数,两者在本质上是相同的,即都能对给定的若干个参数求出一        个结果来,例如 2+3 可以写成 +(2,3) 的函数调用形式。     2. 使用运算符前置表达式,常规的数学表达式是很不规范的,比如二元运算符通常是置        于两个操作数之间的,而这对于非二元运算符都不适用,又比如负号是放在被操作数        之前的,而同样作为一元运算符的阶乘符号却是放在被操作数之后的,而函数又采用        了 fun(parm1,parm2,....) 这样的形式,因此,本程序采用运算符前置表达式来规        范数学表达式的形式,例如数学表达式                 2*(3+8)        在规范后将成为                 (* 2 (+ 3 8))        而数学表达式                 (-3+sqrt(3^2-4*1*2))/(2*1)        在规范后将成为                 (/ (+ (- 3) (sqrt (- (^ 3 2) (* (* 4 1) 2)))) (* 2 1)) ========== 最近更新: 2012-09-29 ==========     软件维护地址 http://www.oschina.net/code/snippet_736932_13725     E-mail:    zhoucosin@163.com     QQ:        532319166 下载地址: windows: abacus_1.0_for_windows.rar (68.03 KB) linux : abacus_1.0_for_linux.tar.gz (16.12 KB) 软件截图:
kuing 2# 2012-9-12 22:27
恭喜恭喜 等我能用电脑上再下来玩玩。
都市侠影 3# 2012-9-20 16:30
本帖最后由 都市侠影 于 2012-9-20 19:18 编辑 更新了下,增强了用户体验,允许在表达式中任意使用空格以改良表达式结构层次,但计算器会忽略这些空格。 消除了一个 bug,在只输入一个运算符的情况,程序会给出出错提示而不是直接崩溃掉。
叶剑飞Victor 4# 2012-9-22 01:17
1# 都市侠影 没有abacus.tar.gz吗?
都市侠影 5# 2012-9-22 20:38
4# 叶剑飞Victor 需要的话我可以发给你,这个论坛里的人绝大多数都用的 windows,所以..............
叶剑飞Victor 6# 2012-9-23 00:48
本帖最后由 叶剑飞Victor 于 2012-9-23 00:53 编辑 5# 都市侠影 运行了make.sh 结果,编译错误。
都市侠影 7# 2012-9-24 08:53
本帖最后由 都市侠影 于 2012-9-24 10:45 编辑 在 includes.h 文件中加上头文件: #include <stdio.h> #include <typeinfo> #include <cstring> 我是在 windows 7 下面用 mingw 编译的,gcc 版本是 3.4.5,估计是因为 mingw 自带的 gcc 默认就加入了这些头文件。待我在 linux 下试试.............. 加了上述头文件后,linux 下通过编译:
叶剑飞Victor 8# 2012-9-25 15:59
7# 都市侠影 话说怎么用make.sh了呢?怎么不写Makefile呢?
都市侠影 9# 2012-9-25 16:27
本帖最后由 都市侠影 于 2012-9-25 16:35 编辑 8# 叶剑飞Victor 话说以我目前的水平,还不足以写出一个好的 makefile, 所以将就写了个 make.sh 以解燃眉之急,过几天写一个 makefile,就不用忍受只更新一个文件也不得不全部重新编译的痛苦了。
叶剑飞Victor 10# 2012-9-25 17:15
9# 都市侠影 话说你的make.sh其实可以这么写的 g++   *.cpp  -lm   -o   abacus   -Wall 复制代码
叶剑飞Victor 11# 2012-9-25 17:26
本帖最后由 叶剑飞Victor 于 2012-9-25 17:43 编辑 9# 都市侠影 写个简单的Makefile给你吧 abacus:  Express.o  ExpressPart.o  ExpOperator.o  ExpNumber.o  ExpIdentifier.o  ExpBracket.o  SyntaxParseProcessor.o  WordParseProcessor.o  OperatorFunction.o  Computor.o  demo.o         g++  *.o  -lm  -o  abacus  -Wall Express.o:  Express.cpp         g++  -c  Express.cpp  -Wall ExpressPart.o:  ExpressPart.cpp         g++  -c  ExpressPart.cpp  -Wall ExpOperator.o:  ExpOperator.cpp         g++  -c  ExpOperator.cpp  -Wall ExpNumber.o:  ExpNumber.cpp         g++  -c  ExpNumber.cpp  -Wall ExpIdentifier.o:  ExpIdentifier.cpp         g++  -c  ExpIdentifier.cpp  -Wall ExpBracket.o:  ExpBracket.cpp         g++  -c  ExpBracket.cpp  -Wall SyntaxParseProcessor.o:  SyntaxParseProcessor.cpp         g++  -c  SyntaxParseProcessor.cpp  -Wall WordParseProcessor.o:  WordParseProcessor.cpp         g++  -c  WordParseProcessor.cpp  -Wall OperatorFunction.o:  OperatorFunction.cpp         g++  -c  OperatorFunction.cpp  -Wall Computor.o:  Computor.cpp         g++  -c  Computor.cpp  -Wall demo.o:  demo.cpp         g++  -c  demo.cpp  -Wall clean:         rm  *.o 复制代码
都市侠影 12# 2012-9-25 18:27
本帖最后由 都市侠影 于 2012-9-25 19:12 编辑 10# 叶剑飞Victor 这样的 makefile 我也可以写,只是我不满意这样的方式,因为经常对头文件包含依赖修改的话,反复的修改 makefile 会让人不舒服,我想要的是在 makefile 中自动推导源文件所需的头文件,这样的 makefile 简单的多,只是我还没有去看如何写,这个正是接下来的工作。
叶剑飞Victor 13# 2012-9-25 23:06
demo.cpp 源文件中的 void showOperatorsInfo() 函数 cout<<" <5> a \% b \treturn the remainder of a divided by b."<<endl; “%”不用转义,直接“%”就行了,“\%”是错误的。不过如果用g++编译,如果不加上“-ansi -pedantic-errors”参数,编译也能通过。
都市侠影 14# 2012-9-26 09:00
本帖最后由 都市侠影 于 2012-9-26 09:01 编辑 13# 叶剑飞Victor 刚测试了下,无论是在 win7 下的 MinGW 环境,还是 linux 环境,这个百分号转义与不转义都能正确显示,当然推荐不转义的方式,C 代码写多了的后遗症
都市侠影 15# 2012-9-26 17:59
[i=s] 本帖最后由 都市侠影 于 2012-9-29 11:22 编辑 [/i] 写了个 makefile,是自动推导头文件依赖的。
叶剑飞Victor 16# 2012-9-27 23:48
本帖最后由 叶剑飞Victor 于 2012-9-27 23:49 编辑 百度贴吧C++吧的吧主御坂美琴みさか也做了一个,现在她在召唤你过去讨论。 这里是链接 http://tieba.baidu.com/f?ct=3356 ... 7502425#24649725417
都市侠影 17# 2012-9-28 09:29
本帖最后由 都市侠影 于 2012-9-28 09:50 编辑 16# 叶剑飞Victor ===================================== 关于对变量以及宏的支持的说明 设计的时候没有考虑变量参与运算,当时只想着为一个数学式子计算值,这个式子都提供了,各个变量的值应该都确定了吧,那就不需要变量了。 在什么情况下需要变量呢?我可以把一个含变量的式子存为一个模式,比如把圆的面积公式存起来,以后要求一个圆的面积的时候,我直接调用这个公式就可以了,这本质上是一个用户定义宏的功能(不是在代码中加入函数),这样的话,它就更像是解释器了,这个有意思,待我研究下这个功能,实现这功能之后,用户就可以像下面这样定义宏: #define    AreaOfCycle(r)    pi*r*r 然后就可以在表达式中这样求一个圆环的面积: AreaOfCycle(3)-AreaOfCycle(2) 最后,还要提供对用户自定义宏的管理功能,可以查看、增删、修改已经定义的宏。 感谢网友叶剑飞一直以来的关注,这个计算器必将添加越来越多的功能,越来越完善,特此感谢。 ===================================== 话说,那个链接里面,我只看到一片评论,没看到他写的计算器在哪呀?
叶剑飞Victor 18# 2012-9-28 14:43
本帖最后由 叶剑飞Victor 于 2012-9-28 14:47 编辑 17# 都市侠影 源代码在三楼的链接中 http://ideone.com/o2Ag4 那个之所以写成[dot]是因为百度贴吧会自动吞链接,所以用了这种“反和谐”手段。
都市侠影 19# 2012-10-7 10:38
本帖最后由 都市侠影 于 2012-10-7 11:12 编辑 又有新的想法了,宏这个东西我不是很满意,一是它只是一个单纯的字符串替换,难以进行语法检查,二是它对功能的扩展比较有限,说白了就是已有功能的缩写。现在想实现一种简单的类似 C 语言的编程语言,无须有多少编程知识就能上手,适合于没有计算机编程基础的数学爱好者使用 abacus 编程实现自己的计算需求,具体的想法还不成熟,我打算不区分整数与实数,直接提供一个数类型,具备简单的控制结构等等,使得用户专注于自己的算法,而不是语言的细节。用户只要在配置文件中写上自己的函数,就可以在 abacus 中直接使用这个函数,比如,如果用户可以定义一个分段函数,他就可以在配置文件中加入下面的函数定义:(这里例子是个绝对值函数) num    myfun(num    x) {          num y;          if(x>0 and x<1)   y=sqrt(1-x^2);          else if(x<0 and x>-1)       y=-sqrt(1-x^2);          else y=0;          return y; } 然后在 abacus 中直接输入 sqrt(3)/2+myfun(-19) 即可使用它参与运算。 但目前正在学习编译原理,这个功能是个远期目标,近期还是集中精力实现宏的功能。
叶剑飞Victor 20# 2012-10-7 16:06
19# 都市侠影 楼主的意思是要搞一个类似MatLab的语言吗?
thread-817-2-6.html:
都市侠影 21# 2012-10-7 17:39
有点类似
thread-82-1-9.html: [函数] 来自群的抽象函数问题$f(f(x))=9x+4$(原解二楼有问题暂时关掉,后面有更新)
kuing 1# 2011-10-10 00:00
学生-jhlts(4045*****)  22:58:37 求助啊 若 $f(f(x))=9x+4$,那么 $f(x)$ 一定是一次函数吗?
kuing 2# 2011-10-10 00:32
kuing 3# 发表于 2011-10-10 00:37
错了…………汗
kuing 4# 2011-10-10 02:39
咳,突然想到一个easy的反例 \[f(x)=\begin{cases} 3x+1 & x\in\mathbb{Q}\\ -3x-2 & x\in\mathbb{R}\backslash\mathbb{Q}\\ \end{cases}\]
kuing 5# 2011-10-10 11:35
变式一下,楼上的方法还行不行呢? $f(f(x))=2x+1$,那么 $f(x)$ 是否一定为一次函数? 如下构造行不行? 先定义一个集合:$\mathbb{Q}\bigl(\sqrt2\bigr) = \bigl\{ a + b\sqrt2 \mid a,b\in\mathbb{Q}\bigr\}$ 然后将 $f(x)$ 定义为 \[ f(x) = \begin{cases} \sqrt 2 x + \sqrt 2 - 1 & x \in \mathbb{Q}\bigl(\sqrt2\bigr) \\ - \sqrt 2 x - \sqrt 2 - 1 & x \in \mathbb{R}\backslash\mathbb{Q}\bigl(\sqrt2\bigr) \end{cases} \] 要证明这个 $f(x)$ 满足对 $\forall x \in\mathbb{R}$ 有 $f(f(x))=2x+1$,注意到 $\sqrt 2 x + \sqrt 2 - 1$ 和 $- \sqrt 2 x - \sqrt 2 - 1$ 都是迭代后就是 $2x+1$,故只需证: (一)$\mathbb{Q}\bigl(\sqrt2\bigr)$ 对于映射 $x\to\sqrt 2 x + \sqrt 2 - 1$ 是封闭的; (二)$\mathbb{R}\backslash\mathbb{Q}\bigl(\sqrt2\bigr)$ 对于映射 $x\to-\sqrt 2 x - \sqrt 2 - 1$ 是封闭的。 (一):当 $x\in\mathbb{Q}\bigl(\sqrt2\bigr)$,设 $x=a + b\sqrt2,a,b\in\mathbb{Q}$,则\[ \sqrt 2 x + \sqrt 2 - 1 = \sqrt 2 \bigl(a+b\sqrt2\bigr) + \sqrt 2 - 1=2b-1+(a+1)\sqrt2\in\mathbb{Q}\bigl(\sqrt2\bigr) \]所以成立。 (二):用反证法,当 $x \in \mathbb{R}\backslash\mathbb{Q}\bigl(\sqrt2\bigr)$,假设 $-\sqrt 2 x - \sqrt 2 - 1\notin\mathbb{R}\backslash\mathbb{Q}\bigl(\sqrt2\bigr)$,因为 $-\sqrt 2 x - \sqrt 2 - 1\in\mathbb{R}$,那么必然有 $-\sqrt 2 x - \sqrt 2 - 1\in\mathbb{Q}\bigl(\sqrt2\bigr)$,故可设 $-\sqrt 2 x - \sqrt 2 - 1=a + b\sqrt2,a,b\in\mathbb{Q}$,解得\[ x = \frac{a + b\sqrt 2 + \sqrt 2 + 1}{ - \sqrt 2 } = - b - 1 - \frac{a + 1}{2}\sqrt 2 \in\mathbb{Q}\bigl(\sqrt2\bigr) \]矛盾! 这样就OK了,这样来看似乎对于有两个一次函数解的并且一次项系数为有理数的平方根都可以这样构造,但如果是其他无理数系数,甚至是超越数,会怎样呢?而对于只有一个一次函数解的,又如何呢? 待续…………
kuing 6# 2011-10-10 13:29
也就是比如这两个:$f(f(x))=\pi x+e$、$f(f(x))=x+1$,咋搞?研究中……
kuing 7# 2011-10-10 16:24
$f(f(x))=x+1$ 似乎OK了 也是先记一个集合 $M=\left\{\dfrac x2 \biggm| x\in\mathbb{Z}\right\}$,将 $f(x)$ 定义为 \[ f(x)=\begin{cases} x+1.5 & x\in\mathbb{Z}\\ x-0.5 & x\in M\backslash\mathbb{Z}\\ x+0.5 & x\in\mathbb{R}\backslash M \end{cases} \] 行不行? 如果没问题的话,$f(f(x))=x+k$ 也类似可以构造出来。
thread-822-1-5.html: [不等式] 以前问过的题又再问
kuing 1# 2012-9-15 03:51
话说不等式小组论坛重开了,我重回旧地闲逛时发现当年我在这个帖 http://www.irgoc.org/viewtopic.php?f=28&t=85 里面问的题好像后来在不知什么地方看到过解答,但一时想不起来,也没找到相关记录,而尝试自己重新再证也没证出来,故此又要再次求助了T_T PS. 由于现在是爪机发帖,就暂时不将题目打过来了,爪机打代码超麻烦,大家点击链接进去看吧。
kuing 2# 2012-9-15 04:01
可以看成是Iran96的一个加强噢
kuing 3# 2012-9-16 04:17
有木有银??
yizhong 4# 2012-9-16 11:01
这阵子忙ing......
kuing 5# 2012-9-16 13:03
4# yizhong 忙啥哟?? 这道题你应该也见过吧,知不知道出处??
kuing 6# 2012-12-1 14:24
无意之中找到了 can 的证法,大概是三年前的文件了,可能之前看到的证法就是这个了,见附件。
kuing 7# 2012-12-1 14:25
djvu阅读器自己百度一下吧,我就不传了
pxchg1200 8# 2012-12-3 15:54
7# kuing 就那个文档中截了2页吧。。。
kuing 9# 2012-12-3 15:58
8# pxchg1200 什么文档?
thread-825-1-5.html: 翻群聊记录捞到的一道三角形小题
kuing 1# 2012-9-17 01:38
话说几天上不了网,今晚终于能上之际,指定动作——翻群聊记录,看看有没有讨论什么好题。 结果你应该能想象到,很难捞,因为现在是“diao鱼”热潮,各个群里用口水来“diao鱼”的“zhengzhi家”一堆堆,要从中捞到好的数学题得不断翻页。 言归正传,刚才捞到的是以下题目: $\triangle ABC$ 满足下式,判断三角形的类型。 \[\frac{a\sin A+b\sin B+c\sin C}{a\cos A+b\cos B+c\cos C}=\frac{9R}{2p},\] 其中 $R$, $p$ 分别为外接圆的半径和三角形的半周长。 形式看着不错,难度适中,可以玩玩。 请自己先试试解一解,说不定你解得比我简单。 高手可以略过。 \begin{gather*} \sum a\sin A=\frac{\sum a^2}{2R}, \\ \sum a\cos A=\sum\frac{a(b^2+c^2-a^2)}{2bc}=\frac{\sum a^2(b^2+c^2-a^2)}{2abc}=\frac{2\sum a^2b^2-\sum a^4}{2abc}=\frac{8S^2}{abc}=\frac{abc}{2R^2}, \end{gather*} 故 \[\frac{\sum a\sin A}{\sum a\cos A}=\frac{9R}{2p}\iff\frac{\frac{\sum a^2}{2R}}{\frac{abc}{2R^2}}=\frac{9R}{\sum a}\iff\sum a\sum a^2=9abc,\] 故显然等边。事实上也可以写成不等式的形式,不过左边很大,不等式太弱,好像也没写的必要。
kuing 2# 2012-9-17 01:43
本来还看到一道 分式、高次开根号、阶乘、求和 的不等式,不过怀疑后面有输入错误,就先不发上来了。
joatbmon 3# 2012-9-27 15:20
不会做,回复看答案
wenshengli 4# 2012-12-14 09:07
正三角形?
thread-826-1-1.html: 来自群的二重积分不等式
kuing 1# 2012-9-17 13:42
话说我证出了原式 $\leqslant4/3$,加强了右边,不知对不对…… 吃完饭再发过程上来。
kuing 2# 2012-9-17 14:24
由和差化积有 \[\sin(x^2)+\cos(y^2)=2\sin\left(\frac\pi4+\frac{x^2-y^2}2\right)\sin\left(\frac\pi4+\frac{x^2+y^2}2\right).\] 易证 \[\sin\left(\frac\pi4+t\right)\leqslant \frac1{\sqrt2}(t+1), t\in\left[-\frac\pi2,\frac\pi2\right],\] 显然 $(x^2-y^2)/2$, $(x^2+y^2)/2\in[-\pi/2,\pi/2]$,所以 \[2\sin\left(\frac\pi4+\frac{x^2-y^2}2\right)\sin\left(\frac\pi4+\frac{x^2+y^2}2\right)\leqslant \left(\frac{x^2-y^2}2+1\right)\left(\frac{x^2+y^2}2+1\right)=\frac{x^4}{4}-\frac{y^4}{4}+x^2+1,\] 于是 \[\iint_{[0,1]\times[0,1]}\sin(x^2)+\cos(y^2)\rmd{x}\rmd{y}\leqslant \iint_{[0,1]\times[0,1]}\frac{x^4}{4}-\frac{y^4}{4}+x^2+1\rmd{x}\rmd{y} =\int_0^1x^2+1\rmd{x}=\frac43. \]
kuing 3# 2012-9-17 15:38
原来先利用积分的性质化简原式,会简单很多。 \begin{align*} &\iint_{[0,1]\times[0,1]}\sin(x^2)+\cos(y^2)\rmd{x}\rmd{y}\\ ={}&\iint_{[0,1]\times[0,1]}2\sin\left(\frac\pi4+\frac{x^2-y^2}2\right)\sin\left(\frac\pi4+\frac{x^2+y^2}2\right)\rmd{x}\rmd{y}\\ ={}&\iint_{[0,1]\times[0,1]}\sqrt2\left(\sin\frac{x^2-y^2}2+\cos\frac{x^2-y^2}2\right)\sin\left(\frac\pi4+\frac{x^2+y^2}2\right)\rmd{x}\rmd{y}\\ ={}&\iint_{[0,1]\times[0,1]}\sqrt2\cos\frac{x^2-y^2}2\sin\left(\frac\pi4+\frac{x^2+y^2}2\right)\rmd{x}\rmd{y}\\ ={}&\iint_{[0,1]\times[0,1]}\frac{\sqrt2}2\left(\sin\left(\frac\pi4+x^2\right)+\sin\left(\frac\pi4+y^2\right)\right)\rmd{x}\rmd{y}\\ ={}&\sqrt2\int_0^1\sin\left(\frac\pi4+x^2\right)\rmd{x}, \end{align*} 这样,由 \[\frac{\sqrt2}2\leqslant \sin\left(\frac\pi4+x^2\right)\leqslant \frac1{\sqrt2}(x^2+1),\] 即得 \[1=\sqrt2\int_0^1\frac{\sqrt2}2\rmd x\leqslant \sqrt2\int_0^1\sin\left(\frac\pi4+x^2\right)\rmd{x}\leqslant \sqrt2\int_0^1\frac1{\sqrt2}(x^2+1)\rmd x=\frac43<\sqrt2.\] 如果右边不用切线法,直接用$\sin\bigl(\frac\pi4+x^2\bigr)\leqslant 1$,便出来$\sqrt2$; 如果左边用支撑线法,便会得到更精确的结果,时间关系,就不写了。
都市侠影 4# 2012-9-17 16:06
原来如此暴力,秋风还说要目测,汗.................
kuing 5# 2012-9-17 16:08
显然我也没目测出来………… 不过居然能化成一元函数积分,我也有点意外。 说不定还真有能目测的办法。
kuing 6# 2012-9-17 20:32
pxchg 说: 其实我还在懵ing为什么第一个等号能直接等过去?
kuing 7# 2012-9-17 20:44
好像想懂了,是不是这样推导的?: \begin{align*} \iint_{[a,b]\times[a,b]}f(x)+g(y)\rmd{x}\rmd{y} &=\int_a^b\left(\int_a^bf(x)+g(y)\rmd{x}\right)\rmd{y}\\ &=\int_a^b\left(\int_a^bf(x)\rmd{x}+(b-a)g(y)\right)\rmd{y}\\ &=(b-a)\int_a^bf(x)\rmd{x}+(b-a)\int_a^bg(y)\rmd{y}\\ &=(b-a)\int_a^bf(x)+g(x)\rmd{x}, \end{align*} 这样当 $b-a=1$ 时就有 $\iint_{[a,b]\times[a,b]}f(x)+g(y)\rmd{x}\rmd{y}=\int_a^bf(x)+g(x)\rmd{x}$。 其实我对重积分不太熟悉的说……
kuing 8# 2012-9-19 14:38
经群里 zhcosin 提点,原来直接分开就行了 \begin{align*} \iint_{[a,b]\times[a,b]}f(x)+g(y)\rmd{x}\rmd{y} &=\iint_{[a,b]\times[a,b]}f(x)\rmd{x}\rmd{y} + \iint_{[a,b]\times[a,b]}g(y)\rmd{x}\rmd{y} \\ &=(b-a)\int_a^bf(x)\rmd{x}+(b-a)\int_a^bg(y)\rmd{y}\\ &=(b-a)\int_a^bf(x)+g(x)\rmd{x}, \end{align*} 这样更加显然。
秋风树林 9# 2012-9-19 23:52
经群里 zhcosin 提点,原来直接分开就行了 \begin{align*} \iint_{[a,b]\times[a,b]}f(x)+g(y)\rmd{x}\rmd{y} &=\iint_{[a,b]\times[a,b]}f(x)\rmd{x}\rmd{y} + \iint_{[a,b]\times[a,b]}g(y)\rmd{x}\rmd{y} \\ ... kuing 发表于 2012-9-19 14:38 原来如此 这样果真就是目测。。。
kuing 10# 2012-9-20 00:02
9# 秋风树林 这个是不是就是用了什么线性性质什么的啊?
秋风树林 11# 2012-9-20 09:21
9# 秋风树林 这个是不是就是用了什么线性性质什么的啊? kuing 发表于 2012-9-20 00:02 嗯,线性性 不过这里还是有点二次积分的东西 但是已经能够目测了。。。即使是不知道二次积分的人
thread-83-1-9.html: [不等式] 来自pep的条件简单不等式$x+y+z=xyz,\sum\frac1{1+xy}_\max$
kuing 1# 2011-10-10 12:22
问题来自 http://bbs.pep.com.cn/thread-1903753-1-1.html 已知 $x$、$y$、$z$ 是正实数,且 $x+y+z=xyz$,求 $\dfrac1{1+xy}+\dfrac1{1+yz}+\dfrac1{1+zx}$ 的最大值。 由条件及柯西不等式得\[ \frac1{1+xy}=\frac1{1+\frac{x+y+z}z}=\frac z{z+x+y+z}=\frac z4\cdot\frac4{z+x+y+z}\leqslant\frac z4\left(\frac1{z+x}+\frac1{y+z}\right) \]另外两项同理,故\[ \sum\frac1{1+xy}\leqslant\frac14\sum\left(\frac z{z+x}+\frac z{y+z}\right)=\frac14\sum\left(\frac z{z+x}+\frac x{z+x}\right)=\frac34 \]当 $x=y=z=\sqrt3$ 时取等号。
thread-831-1-6.html: [不等式] 一个复数模不等式(from 秋风)挺好看的
kuing 1# 2012-9-17 22:13
设 $z_1$, $z_2\in\mbb C$ 且 $\abs{z_1}$, $\abs{z_2}<1$,求证 \[\frac{\abs{z_1}-\abs{z_2}}{1-\abs{z_1}\cdot\abs{z_2}}\leqslant \left| \frac{z_1+z_2}{1+\overline{z_1}z_2} \right|\leqslant \frac{\abs{z_1}+\abs{z_2}}{1+\abs{z_1}\cdot\abs{z_2}}.\] 解: 只证右边,左边类似。 注意到 $\abs{z}^2=z\cdot\overline z$,于是 \begin{align*} \left| \frac{z_1+z_2}{1+\overline{z_1}z_2} \right|\leqslant \frac{\abs{z_1}+\abs{z_2}}{1+\abs{z_1}\cdot\abs{z_2}} &\iff \frac{z_1+z_2}{1+\overline{z_1}z_2}\cdot\frac{\overline{z_1}+\overline{z_2}}{1+z_1\overline{z_2}}\leqslant \frac{(\abs{z_1}+\abs{z_2})^2}{(1+\abs{z_1}\cdot\abs{z_2})^2}\\ &\iff \frac{(z_1+z_2)(\overline{z_1}+\overline{z_2})}{(1+\overline{z_1}z_2)(1+z_1\overline{z_2})}-1\leqslant \frac{(\abs{z_1}+\abs{z_2})^2}{(1+\abs{z_1}\cdot\abs{z_2})^2}-1\\ &\iff \frac{z_1\overline{z_1}+z_2\overline{z_2}-1-z_1\overline{z_1}z_2\overline{z_2}}{(1+\overline{z_1}z_2)(1+z_1\overline{z_2})}\leqslant \frac{\abs{z_1}^2+\abs{z_2}^2-1-\abs{z_1}^2\abs{z_2}^2}{(1+\abs{z_1}\cdot\abs{z_2})^2}, \end{align*} 因为 \[z_1\overline{z_1}+z_2\overline{z_2}-1-z_1\overline{z_1}z_2\overline{z_2}=\abs{z_1}^2+\abs{z_2}^2-1-\abs{z_1}^2\abs{z_2}^2 =-(1-\abs{z_1}^2)(1-\abs{z_1}^2)<0,\] 所以只要证 \[(1+\overline{z_1}z_2)(1+z_1\overline{z_2})\leqslant (1+\abs{z_1}\cdot\abs{z_2})^2,\] 即 \[\abs{1+\overline{z_1}z_2}\leqslant 1+\abs{\overline{z_1}}\cdot\abs{z_2},\] 显然成立。
秋风树林 2# 2012-9-17 22:33
先回复一个 但愿我的那个方法能符合糖水不等式的条件。。。
力工 3# 2012-9-21 13:15
1# kuing 让我看看清新淡雅传奇性感的复数不等式,可别告诉我就是三点式啊。
hongxian 4# 2012-9-21 15:48
看一下!
thread-832-1-1.html: 来自群里的一个问题
kuing 1# 2012-9-19 13:36
其实最后还是没研究出什么更好的方法来,代码还是比较糟糕的,详情见附件。 用的是 tikz 宏包及其中的 positioning 库,其中每个公式加 \mathstrut 是为了让各个盒子的高度和深度相同,才能让公式对齐。 改变 A、B 的两个 node 的坐标可以调整两行的距离。
thread-833-1-6.html: 来自pep的一道根式极限
kuing 1# 2012-9-19 14:20
来自 http://bbs.pep.com.cn/thread-2585495-1-1.html 题目:求根极 \[\lim_{x\to\infty}\sqrt[3]{x^3+3x^2}-\sqrt{x^2-2x}.\] 有理化,有 \begin{align*} & \sqrt[3]{x^3+3x^2}-\sqrt{x^2-2x} \\ ={}&\frac{\sqrt[3]{(x^3+3x^2)^2}-(x^2-2x)}{\sqrt[3]{x^3+3x^2}+\sqrt{x^2-2x}} \\ ={}&\frac{(x^3+3x^2)^2-(x^2-2x)^3}{\bigl(\sqrt[3]{x^3+3x^2}+\sqrt{x^2-2x}\bigr)\bigl(\sqrt[3]{(x^3+3x^2)^4}+\sqrt[3]{(x^3+3x^2)^2}(x^2-2x)+(x^2-2x)^2\bigr)} \\ ={}&\frac{x^3(12x^2-3x+8)}{\bigl(\sqrt[3]{x^3+3x^2}+\sqrt{x^2-2x}\bigr)\bigl(\sqrt[3]{(x^3+3x^2)^4}+\sqrt[3]{(x^3+3x^2)^2}(x^2-2x)+(x^2-2x)^2\bigr)} \\ ={}&\frac{12-\frac3x+\frac8{x^2}}{\frac{\sqrt[3]{x^3+3x^2}+\sqrt{x^2-2x}}x\cdot \frac{\sqrt[3]{(x^3+3x^2)^4}+\sqrt[3]{(x^3+3x^2)^2}(x^2-2x)+(x^2-2x)^2}{x^4}} \\ ={}&\frac{12-\frac3x+\frac8{x^2}}{\Bigl(\sqrt[3]{1+\frac3x}+\sqrt{1-\frac2x}\Bigr)\Bigl(\sqrt[3]{\bigl( 1+\frac3x\bigr)^4}+\sqrt[3]{\bigl(1+\frac3x\bigr)^2}\bigl(1-\frac2x\bigr)+\bigl(1-\frac2x\bigr)^2 \Bigr)}, \end{align*} 所以极限显然为$12/(1+1)(1+1+1)=2$
hongxian 2# 2012-9-19 17:04
很长时间没有看到K版在人教哪边发贴了,下次直接发到这边!
kuing 3# 2012-9-19 17:22
2# hongxian 主要是这里打公式方便,就懒得在那边回了。
thread-834-1-6.html: [组合] 请教一个集合的竞赛题,先谢谢了
hongxian 1# 2012-9-21 15:59
1.已知集合$P$是$M=\{x|1\le x\le 2000,x\in N\}$的子集,且$P$中任意两个元素的差都不等于4也不等于7,试问$P$中元素最多可以包含多少个?
nash 2# 2012-9-22 02:42
抽屉原理,考察{1,2…… 11} {1,5}{2,6}{3,7}{4,8}{5,9}{6,10}{7,11}{1,8}{2,9}{3,10}{4,11}这11个子集中,1,2…… 11任意取6个元素,必有2个同属于11个子集中的某一个, 所以{1,2 ……11}中至多有5个符合题意 下面构造{1,3,4,6,9} 然后{11K+1,3,4,6,9;K=0,1…… 181} 所以共有182*5=910个
hongxian 3# 2012-9-22 09:09
2# nash 谢谢了!
thread-835-1-6.html: 一道对数比较大小题
v6mm131 1# 2012-9-21 18:08
近期网络上流传一道比较对数大小的题,请各路高手围攻。。 ________kuing edit in $\LaTeX$________ 求证:\[\log_{\frac14}\frac87>\log_{\frac15}\frac54\]
都市侠影 2# 2012-9-21 20:01
暴力个:
kuing 3# 2012-9-21 20:33
本来没什么兴趣看这种题,不过既然发在这了,现在又闲,还是……: \begin{align*} \log_{\frac14}\frac87>\log_{\frac15}\frac54&\iff\log_4\frac78>\log_5\frac45 \\ &\iff\log_4\frac78+\log_416>\log_5\frac45+\log_525 \\ &\iff\log_414>\log_520, \end{align*} 所以只要证 $f(x)=\log_x(6x-10)$ 在 $[4,+\infty)$ 上递减。求导有 \[f'(x)=-\frac{(3x-5)\ln(6x-10)-3x\ln x}{x(3x-5)\ln^2x},\] 所以只要证 $g(x)=(3x-5)\ln(6x-10)-3x\ln x$ 在 $[4,+\infty)$ 上恒正。求导有 \[g'(x)= 3\ln(6x-10)-3\ln x>0,\] 所以只要证 $g(4)>0$,即 $7\ln14>12\ln4$,也即 $7\ln7>17\ln2$,事实上,有 \[7\ln7=\frac72\ln49>\frac72\ln32=\frac{35}2\ln2>17\ln2,\] 所以原不等式成立。
thread-836-1-1.html: [组合] 请高手指教一道意外得到的排列组合题
ccnu_chb_ycb 1# 2012-9-17 21:36
本帖最后由 ccnu_chb_ycb 于 2012-9-22 10:05 编辑 从1-100里面取两个不同的整数,做乘积,求能被3整除的积的个数
ccnu_chb_ycb 2# 2012-9-22 10:06
学生问到这样一个问题:从1-100里面取两个不同的整数,做乘积,求能被3整除的积的个数
kuing 3# 2012-9-22 12:49
意外得到? 没发现有什么特别的说…… 两数的乘积被3整除,相当于两数至少一个被3整除吧。 100之内的数中有33个被3整除,67个不被3整除,所以结果大概就是 $33\times67+C_{33}^2$?
hongxian 4# 2012-9-22 15:48
2# kuing 这样不知$1\times 27$和$3\times 9$算重复了没有?
kuing 5# 2012-9-22 16:06
3# hongxian 噢,这样也算是同一个?那我上面肯定算多了很多情况了。原来意外在这里……
kuing 6# 2012-9-22 17:34
这样的话我不会了,用程序算了下结果是 1351,希望程序没写错。 先出去吃饭,回来再想想。
ccnu_chb_ycb 7# 2012-9-22 22:57
4# kuing 恩,1*24=2*12=3*8=4*6=24,里面有很多重复的,该如何计算呢?
Gauss门徒 8# 2013-1-4 16:09
本帖最后由 Gauss门徒 于 2013-1-8 16:27 编辑 误会题意了
kuing 9# 2013-1-4 16:17
8# Gauss门徒 怎么算的?
李斌斌755 10# 2013-1-5 00:59
如果积相同算一个,最大的积为9900,在2至9900中只有3300个能被3整除。若不算一个……
kuing 11# 2013-1-5 01:39
10# 李斌斌755 9897能不能取到
thread-836-1-5.html: [组合] 请高手指教一道意外得到的排列组合题
ccnu_chb_ycb 1# 2012-9-17 21:36
本帖最后由 ccnu_chb_ycb 于 2012-9-22 10:05 编辑 从1-100里面取两个不同的整数,做乘积,求能被3整除的积的个数
ccnu_chb_ycb 2# 2012-9-22 10:06
学生问到这样一个问题:从1-100里面取两个不同的整数,做乘积,求能被3整除的积的个数
kuing 3# 2012-9-22 12:49
意外得到? 没发现有什么特别的说…… 两数的乘积被3整除,相当于两数至少一个被3整除吧。 100之内的数中有33个被3整除,67个不被3整除,所以结果大概就是 $33\times67+C_{33}^2$?
hongxian 4# 2012-9-22 15:48
2# kuing 这样不知$1\times 27$和$3\times 9$算重复了没有?
kuing 5# 2012-9-22 16:06
3# hongxian 噢,这样也算是同一个?那我上面肯定算多了很多情况了。原来意外在这里……
kuing 6# 2012-9-22 17:34
这样的话我不会了,用程序算了下结果是 1351,希望程序没写错。 先出去吃饭,回来再想想。
ccnu_chb_ycb 7# 2012-9-22 22:57
4# kuing 恩,1*24=2*12=3*8=4*6=24,里面有很多重复的,该如何计算呢?
Gauss门徒 8# 2013-1-4 16:09
本帖最后由 Gauss门徒 于 2013-1-8 16:27 编辑 误会题意了
kuing 9# 2013-1-4 16:17
8# Gauss门徒 怎么算的?
李斌斌755 10# 2013-1-5 00:59
如果积相同算一个,最大的积为9900,在2至9900中只有3300个能被3整除。若不算一个……
kuing 11# 2013-1-5 01:39
10# 李斌斌755 9897能不能取到
thread-837-1-1.html: 用程序验证一下那个积被3整除的排列组合题
kuing 1# 2012-9-22 17:23
题目来自:http://kkkkuingggg.5d6d.net/thread-836-1-1.html n = 100; asdf = {}; Do[{   Do[{     If[Mod[i j, 3] == 0, asdf = Union[asdf, {i j}]]     }, {j, i, n}]   }, {i, 1, n}] Length[asdf] 输出结果是 1351,希望程序没写错。
thread-838-1-6.html: [不等式] 来自群的指数不等式(from 秋风)
kuing 1# 2012-9-24 16:45
爱好者秋风树林(5221*****)  12:42:48    求证 $yx^y(1-x)<e^{-1}$    区域是 $0<x<1$, $y>0$ proof: 由年轻不等式(Young's inequality,又或者说是加权均值不等式),有 \[x^{\frac y{y+1}}\cdot \bigl(y(1-x)\bigr)^{\frac1{y+1}}\leqslant \frac{xy}{y+1}+\frac{y(1-x)}{y+1}=\frac y{y+1},\] 所以 \[yx^y(1-x) \leqslant \left(\frac y{y+1}\right)^{y+1},\] 于是只要证 \[\left(\frac y{y+1}\right)^{y+1}<e^{-1},\] 即 \[\left(1+\frac1y\right)^{y+1}>e,\] 这是熟知的。 PS、其实有点想放到高等数学版块。
都市侠影 2# 2012-9-24 16:53
果然厉害,我从来没用过这个 $ a^\alpha b^\beta\leqslant a\alpha+b\beta $
秋风树林 3# 2012-9-24 20:23
嗯,我相当于用导数得出了kk第二行的不等式
kuing 4# 2012-9-24 20:25
3# 秋风树林 嗯,一样 年轻不等式可以用导数证
thread-839-1-1.html: 平行符号测试
kuing 1# 2012-9-24 23:49
$AB \sslash CD \iff a \sslash b$
kuing 2# 2012-9-24 23:52
居然没有 sslash 这个平行符号,看来又要自定义了。 $a\mathrel{/\!/}b$ $AB\mathrel{/\!/}CD$
kuing 3# 2012-9-24 23:59
OK,添加平行符号命令 \sslash 成功,一楼也显示了。
thread-84-1-9.html: [不等式] 恒成立
wenshengli 1# 2011-10-10 19:14
本帖最后由 wenshengli 于 2011-10-10 19:16 编辑 已知$f(x)=\sqrt{1+x}+\sqrt{1-x}$,是否存在正常数$\alpha$,使得不等式$f(x)=\sqrt{1+x}+\sqrt{1-x}\le 2-\dfrac{x^2}{\alpha}$对$\forall x \in [0,1]$恒成立?若存在,求出最小的正数$\alpha$,否则说明理由.(求除了换元令$t=\sqrt{1+x}+\sqrt{1-x}$之外的方法)
kuing 2# 2011-10-10 19:38
还是野猪在网刊里写的那种方法方便哟,首先当 $x\ne0$ 时,分离变量为\[ \alpha \geqslant \frac{x^2}{2 - \sqrt {1 + x} - \sqrt {1 - x}} \]可以求出\[ \lim_{x \to 0} \frac{x^2}{2 - \sqrt {1 + x} - \sqrt {1 - x}} = 4 \]所以必要条件是 $\alpha\geqslant4$,再证充分证,当 $\alpha=4$,不等式为\[ \sqrt {1 + x} + \sqrt {1 - x} \leqslant 2 - \frac{x^2}4 \]右边显然为正,故两边平方等价为\[ 2 + 2\sqrt {1 - x^2 } \leqslant 4 - x^2 + \frac{x^4}{16}\iff\sqrt {1 - x^2 } \leqslant 1 - \frac{x^2}2 + \frac{x^4}{32} \]这是显然成立的,因为\[ \sqrt {1 - x^2 } \leqslant \sqrt {1 - x^2 + \frac{x^4}4} = 1 - \frac{x^2}2 \leqslant 1 - \frac{x^2}2 + \frac{x^4}{32} \]
wenshengli 3# 2011-10-10 19:52
2# kuing :D谢了!
kuing 4# 2011-10-10 20:11
补充一下求极限那里,可以不用洛必达,不断分母有理化即可: \begin{align*} \lim_{x \to 0} \frac{{x^2 }}{{2 - \sqrt {1 + x} - \sqrt {1 - x} }} &= \lim_{x \to 0} \frac{{x^2 \bigl(2 + \sqrt {1 + x} + \sqrt {1 - x} \bigr)}}{{4 - \bigl(\sqrt {1 + x} + \sqrt {1 - x} \bigr)^2 }} \\ &= \lim_{x \to 0} \frac{{x^2 \bigl(2 + \sqrt {1 + x} + \sqrt {1 - x} \bigr)}}{{2 - 2\sqrt {1 - x^2 } }} \\ &= \lim_{x \to 0} \frac{{x^2 \bigl(2 + \sqrt {1 + x} + \sqrt {1 - x} \bigr)\bigl(2 + 2\sqrt {1 - x^2 } \bigr)}}{{4 - 4(1 - x^2 )}} \\ &= \lim_{x \to 0} \frac{{\bigl(2 + \sqrt {1 + x} + \sqrt {1 - x} \bigr)\bigl(1 + \sqrt {1 - x^2 } \bigr)}}{2} \\ &= 4 \end{align*}
wenshengli 5# 2011-10-12 18:39
本帖最后由 wenshengli 于 2011-10-12 18:49 编辑 方法2,接kuing四楼,(分离参数法) \begin{align*} \alpha\ge \frac{{x^2 }}{{2 - \sqrt {1 + x} - \sqrt {1 - x} }} &= \frac{{\bigl(2 + \sqrt {1 + x} + \sqrt {1 - x} \bigr)\bigl(1 + \sqrt {1 - x^2 } \bigr)}}{2} \\ \end{align*} \begin{align*} g(x)=\frac{{\bigl(2 + \sqrt {1 + x} + \sqrt {1 - x} \bigr)\bigl(1 + \sqrt {1 - x^2 } \bigr)}}{2} \end{align*} $$g'(x)=\frac{-3x\sqrt{1-x}+(-1-2x)\sqrt{1+x}-4x}{\sqrt{1-x^2}}<0$$     所以,g(x)为[0,1]上的减函数,故,$\alpha \ge4$
wenshengli 6# 2011-10-12 19:05
本帖最后由 wenshengli 于 2011-10-12 19:13 编辑 再发换元$t=\sqrt{1+x}+\sqrt{1-x}$,$t\in [\sqrt{2},2]$,易求得$\alpha \ge\dfrac{(\frac{t^2}{2}-1)^2-1}{t-2}=\dfrac{t^2}{4}(t+2)=h(t)$, $h'(x)=\dfrac{t}{2}(\dfrac{3t}{2}+1)>0$,所以$h(x)$的最大值为4,即证.
kuing 7# 2011-10-12 19:13
5# wenshengli 我写完4楼那里的时候我就想过既然能有理化成这样可能就不必求极限了,不过后来还是懒得去判断单调性,所以就没理了。。。 判断单调性还要求导,还是求个极限简单,充分性也非常容易证。
thread-840-1-6.html: [几何] 在超级群里看到的一道几何题,未证完,还差一点,待续
kuing 1# 2012-9-25 00:28

kuing 2# 2012-9-25 00:42
如图所示,分别记左右两边的圆的半径为 $r$ 和 $R$。 先证明如果 $PQ\sslash CD$ 则也必有 $PH\sslash BE$,反之亦然。这是因为 \[PQ\sslash CD \iff \angle QAC=\angle PAD \iff \angle HAE=\angle PAB \iff PH\sslash BE,\] 因此可以不妨设 $AG$ 与左边的圆的另一交点 $H$ 在 $A$ 与 $Q$ 之间。那么 \begin{align*} PQ\sslash CD \iff{}& PQ\sslash CD~\text{且}~PH\sslash BE\\ \iff{}& \angle Q=\angle AGD~\text{且}~\angle PHQ=\angle AGE\\ \riff{}& \frac{\sin\angle Q}{\sin\angle PHQ}=\frac{\sin\angle AGD}{\sin\angle AGE}\\ \iff{}& \frac{PH}{PQ}=\frac{\sin\angle AGD}{\sin\angle AGE}\\ \iff{}& \frac{2r\cdot\sin\angle PAH}{2R\cdot\sin\angle PAQ}=\frac{\sin\angle AGD}{\sin\angle AGE}\\ \iff{}& \frac{\frac{AB}{\sin\angle AEB}}{\frac{AC}{\sin\angle ADC}}=\frac{\sin\angle AGD}{\sin\angle AGE}\\ \iff{}& \frac{AB}{AC}=\frac{\sin\angle AGD}{\sin\angle ADC}\frac{\sin\angle AEB}{\sin\angle AGE}\\ \iff{}& \frac{AB}{AC}=\frac{AD}{AG}\frac{AG}{AE}\\ \iff{}& \frac{AB}{AC}=\frac{AD}{AE}\\ \iff{}& DE\sslash BC, \end{align*} 这样,只差第二、三行那个反推,也就是说,假如能证明由 $\frac{\sin\angle Q}{\sin\angle PHQ}=\frac{\sin\angle AGD}{\sin\angle AGE}$ 可以推出 $\angle Q=\angle AGD$ 且 $\angle PHQ=\angle AGE$ 就可以了,待续……
kuing 3# 2012-9-25 00:55
时间关系先闪,又要等平几高手出马了……
thread-841-1-6.html: [函数] 又见心形曲线
kuing 1# 2012-9-28 03:52
话说今晚同学在QQ上给我发了下面这张心形函数图 不过我当时正在玩Q版沙罗曼蛇(嗯,最近怀旧嘛,不过这个Q版的比原版难很多,我接了N次机才玩到第6关啊擦……)所以并没细心看。 玩累了之后才回过头来看看,一看之下,咦?奇怪,上面的式子明明是一条函数,怎么会整出一个区域来?而且中间还有这种效果? 仔细看下,再看到解析式中有个 $\cos(200\cdot x)$,噢!原来这并不是一个区域,而是一条很密的上下摆动的函数曲线。 而中间那种效果只是因为线太密了,密到粘在一起,显示出来就会这样,这跟作图的软件有关,用别的软件作出来的效果会不同,比如我用 Mathematica7 作同样的函数得到如下。 好,该琢磨解析式了。想了想,咦,其实这构造原理很简单。 既然“密”是通过 $\cos(200\cdot x)$ 形成的,而且是上下摆动,那么当 $\cos(200\cdot x)$ 取 $\pm1$ 的那些点就是心形轮廓上面的点,因此如果将 $\cos(200\cdot x)$ 分别用 $1$ 和 $-1$ 代替,得到的两条函数便是这个心形的上下两条轮廓,如下图所示。 明白了这一点后,就可以自己重新构造一个出来。具体来说,设上下两条轮廓线分别为 $f(x)$ 和 $g(x)$,那么函数\[y=0.5(1+\cos nx)f(x)+0.5(1-\cos nx)g(x)\]便是所求,其中 $n$ 越大线就越密。 比如我用两个半圆来作 $f(x)$,用两条反三角函数作 $g(x)$(要用到一些绝对值技巧),同样取 $n=200$ 就得到如下的函数及图形。 \[y=\frac12\bigl(1+\cos(200x)\bigr)\sqrt{\frac14-\left(\frac12-\abs x\right)^2}+\frac12\bigl(1-\cos(200x)\bigr)\frac{\arccos(1-2\abs x)-\pi}2\] 其实我个人感觉我整出来的这个比原来的看着还顺眼些 :D 嘿嘿。再华丽一点点,改变 $n$ 的值再加点颜色,可以有如下的效果。 你可以调整 $f(x)$, $g(x)$ 的一些系数改变形状,当然也可以自己去构造,而且也不一定要用 $\cos nx$,还可以搞搞其他样式,比如改为用 $-1$ 的幂结合高斯函数可以得到如下虚线心形。 OK,时间关系,先扯到这里。总之,大家尽情发挥,随便玩。 擦,真的好晚了,不闪就危险了!……
shidilin 2# 2012-9-28 12:39
乖乖,真是有心人呢!
kuing 3# 2012-9-29 19:02
2# shidilin 是无聊的有心人……
戊概念·五 4# 2012-10-2 22:23
1# kuing 个人很喜欢第二、第三个的效果,嗯。。。。。 Ps:都还不到四点诶~不是会熬到五点+了么,这个不算晚吧~
戊概念·五 5# 2012-10-2 22:24
4# 戊概念·五 最后那个虚线的也( ^_^ )不错.....
kuing 6# 2012-10-2 22:31
1# kuing 个人很喜欢第二、第三个的效果,嗯。。。。。 Ps:都还不到四点诶~不是会熬到五点+了么,这个不算晚吧~ 戊概念·五 发表于 2012-10-2 22:23 “第二、三个”是指哪两个啊…… thanks for your 五点+一般是爪机才敢这么晚,电脑状态得早点闪,不然很危险
戊概念·五 7# 2012-10-2 22:50
6# kuing 是指紫色的那两只了——特别是后边的那个,形状看起来更舒服O(∩_∩)O~ 花是送给图图的,吼~不过作为其生父的你,代领就是啦~
thread-842-1-1.html: 看着不难做法很奇怪的题
icesheep 1# 2012-9-28 19:48
本帖最后由 icesheep 于 2012-9-28 19:50 编辑 可导函数 \[\left| {f\left( x \right)} \right| \leqslant 1 , \forall \left| x \right| \leqslant 1\] 证明  \[\exists \xi  \in \left[ {-1,1} \right] \mathrel\backepsilon  \left| {f'\left( \xi  \right)} \right| < 4\]
kuing 2# 2012-9-29 01:34
求证那里的 $\backepsilon$ 是什么东东…… 话说我怎么觉得显然存在 $\xi\in[-1,1]$ 使 $\abs{f'(\xi)}\leqslant1$?……
icesheep 3# 2012-9-29 03:59
本帖最后由 icesheep 于 2012-9-29 04:01 编辑 那个反过来的属于符号,就是“使得” such that 的意思,之前没想到中值定理。 其实原题是二元函数,那样就不能用中值定理了。 $f\left( {x,y} \right)$ 定义在单位圆内,有连续的偏导数且 $\left| {f\left( {x,y} \right)} \right| \leqslant 1$ 证明: \[\exists \left( {{x_0},{y_0}} \right) \mathrel\backepsilon  \left| {\nabla f\left( {{x_0},{y_0}} \right)} \right| < 4\] 不知道右端的4能否改进。
kuing 4# 2012-9-29 13:18
属于符号与反过来的属于符号在代码上也是反过来的 \in $\in$ \ni $\ni$ \backepsilon($\backepsilon$)其实是反过来的 \epsilon($\epsilon$),而 $\epsilon$ 是希腊字母。 回正题:二元函数不懂也……符号都看不懂了
icesheep 5# 2012-9-29 20:48
答案是这样,构造 $g\left( {x,y} \right) = f\left( {x,y} \right) + 2\left( {{x^2} + {y^2}} \right)$ 则 $g\left( {x,y} \right)$ 在单位圆内部必然可以取到最小值, 如若不然则有 g 在边界上某点 P 取到最小为 $g\left( P \right) = f\left( P \right) + 2 \geqslant 1$ 而 $g\left( 0 \right) = f\left( 0 \right) \leqslant 1$ 也即此时仍有 (x,y)=(0,0) 同取到最小值。 所以存在圆内的一点使 g 取到最小值,即有 \[\frac{{\partial g}}{{\partial x}}\left( {{x_0},{y_0}} \right) = 0 = \frac{{\partial f}}{{\partial x}}\left( {{x_0},{y_0}} \right) + 4{x_0}\] \[\frac{{\partial g}}{{\partial y}}\left( {{x_0},{y_0}} \right) = 0 = \frac{{\partial f}}{{\partial y}}\left( {{x_0},{y_0}} \right) + 4{y_0}\] 所以\[\left| {\nabla f} \right| = \left| {\left( {\frac{{\partial f}}{{\partial x}}\left( {{x_0},{y_0}} \right),\frac{{\partial f}}{{\partial y}}\left( {{x_0},{y_0}} \right)} \right)} \right| = 4\left| {\left( {{x_0},{y_0}} \right)} \right| < 4\]
kuing 6# 2012-9-29 21:00
oh,原来那个倒过来的判别式表示梯度……
kuing 7# 2012-9-29 21:50
其实按照这样的方法的话,构造 $h(x,y)=f(x)+2\bigl(\sqrt[3]{x^4}+\sqrt[3]{y^4}\bigr)$ 好像也可以? 如果可以的话,那么令 $\alpha\in(1,2)$,构造 $K(x,y)=f(x)+2({\abs x}^\alpha+{\abs y}^\alpha)$ 就能将右边的 $4$ 改进成了 $2\alpha$? 注意到有不等式 $({\abs x}^\alpha+{\abs y}^\alpha)^{1/\alpha}\geqslant (x^2+y^2)^{1/2}$
kuing 8# 2012-9-29 22:45
好像想错了点东西,貌似最后还要幂平均一下,再想想……
kuing 9# 2012-9-29 22:53
那个应该是改进到 $\dfrac{\alpha}{2^{(\alpha-4)/2}}$,这个式子关于 $\alpha$ 递增,当 $\alpha=2$ 时为 $4$,当 $\alpha=1$ 时为 $2\sqrt2$。
icesheep 10# 2012-9-30 17:10
$({\abs x}^\alpha+{\abs y}^\alpha)^{1/\alpha}\geqslant (x^2+y^2)^{1/2}$ 这个肿么来的?
kuing 11# 2012-9-30 17:53
$({\abs x}^\alpha+{\abs y}^\alpha)^{1/\alpha}\geqslant (x^2+y^2)^{1/2}$ 这个肿么来的? icesheep 发表于 2012-9-30 17:10 \[({\abs x}^\alpha+{\abs y}^\alpha)^{1/\alpha}\geqslant (x^2+y^2)^{1/2} \iff \left(\frac{x^2}{x^2+y^2}\right)^{\alpha/2}+\left(\frac{y^2}{x^2+y^2}\right)^{\alpha/2}\geqslant1\] 故…… 一般来说,$a_i>0$ 时,$f(x)=\sqrt[x]{\sum a_i^x}$ 在 $(-\infty,0)$ 和 $(0,+\infty)$ 上都是递减的,跟幂平均相反。
icesheep 12# 2012-9-30 19:24
右边那个又是肿么得到的呢。。。 另外这样的话是不是严格不等号得换成非严格不等号了。 我试过 $g\left( {x,y} \right) = f\left( {x,y} \right) + 2 - 2\cos \frac{\pi }{2}\left( {{x^2} + {y^2}} \right)$ 结果没有任何改进
kuing 13# 2012-9-30 20:32
你是说 $\displaystyle\left(\frac{x^2}{x^2+y^2}\right)^{\alpha/2}+\left(\frac{y^2}{x^2+y^2}\right)^{\alpha/2}\geqslant1$? 注意 $\displaystyle0<\frac{x^2}{x^2+y^2}<1$, $\alpha/2<1$ 故 $\displaystyle\left(\frac{x^2}{x^2+y^2}\right)^{\alpha/2}\geqslant\frac{x^2}{x^2+y^2}$……
icesheep 14# 2012-9-30 20:37
13# kuing 退化严重
kuing 15# 2012-9-30 22:05
一般情形也这样证。求导也可以。
thread-843-1-6.html: [不等式] 皮蛋深夜发题
kuing 1# 2012-9-29 03:00
实数 $x$, $y$, $z$ 满足 $x+y+z=0$ 且 $x^2+y^2+z^2=1$,记 $m$ 为 $x^2$, $y^2$, $z^2$ 中的最大者,则 $m$ 的最小值为______。 发贴时觉得有玩头,发完才发现……呃: 由于 $x$, $y$, $z$ 中必有两者同号(约定 $0$ 与任何实数同号),由对称性,不妨设 $x$, $y$ 同号,则显然 $m=z^2$,且 $1=2z^2-2xy\leqslant2z^2$,故 $m\geqslant1/2$,当 $x=0$, $y=-1/\sqrt2$, $z=1/\sqrt2$ 时取等。
nash 2# 2012-9-29 13:52
找到取最值的条件就没得搞了…
kuing 3# 2012-9-29 19:42
2# nash 话说你说“类似题”是不是指类似于这个 $\min\bigl\{\max\{a^2, b^2, 1-(a+b)^2\}\bigr\}$ ?
nash 4# 2012-9-29 23:32
3# kuing 不过这个比上面那个药麻烦些
kuing 5# 2012-9-30 00:12
4# nash 都不麻烦,但其实主要是方法不太相同……
力工 6# 2012-10-11 21:13
1# kuing 学习了。几何意义貌似是球与平面的交线(圆)在轴上的投影,这样可直接了。
thread-844-1-1.html: 无聊之作——用TikZ画了个四国象棋,还没想好怎么玩
kuing 1# 2012-9-29 17:45
还没想好要怎么玩才好玩,规则……? PDF版及源码见附件,用 XeLaTeX 编译,系统字体需要有华文新魏,否则棋子上的字可能会用宋体代替。
叶剑飞Victor 2# 2012-11-25 22:40
没听说过这种棋,开眼界了。
thread-845-1-6.html: [组合] 扯一扯简单传球问题的化简公式
kuing 1# 2012-9-30 03:19
这里的“简单传球问题”是指“无线路限制的 $n\geqslant 3$ 人传 $m$ 次球的传球方式计数问题”,之前在《数学空间》第 4 期中,战巡已经给出一个终极解法,除了解决上述简单情形外,还能解决更复杂的有线路限制的情形。 又话说刚才网友“依然饭特稀”给我发了一篇文章(原文见附件),里面提到如下方法但并未给出证明。 $n$ 个人传 $m$ 次球,记 $x=\dfrac{(n-1)^m}n$,则与 $x$ 最接近的整数为传给“非自己的某人”的方法数,与 $x$ 第二接近的整数便是传给自己的方法数。大家牢记一条公式,可以解决此类至少三人传球的所有问题。 初看感觉有点神,因为这个 $x$ 其实就是 $\frac{\text{所有传球方式总数}}{\text{人数}}$,也就是取平均值,竟然对平均值作两边取整就是所求结果。 后来想了想,再看看用常规方法计算出来的公式,马上就知道原由了。 按照以往常规的办法(递推之类的)可以计算出,球传给“非自己的某人”的方法数 $a(n,m)$ 以及传回自己的方法数 $b(n,m)$ 分别是 \begin{align*} a(n,m)&=\frac{(n-1)^m-(-1)^m}n,\\ b(n,m)&=\frac{(n-1)^m+(n-1)(-1)^m}n, \end{align*} 因此 \begin{align*} a(n,m)&=x-\frac{(-1)^m}n,\\ b(n,m)&=x+\frac{(n-1)(-1)^m}n, \end{align*} $a(n,m)$, $b(n,m)$ 都是整数且 $\bigl(a(n,m)-x\bigr)\bigl(b(n,m)-x\bigr)<0$,所以 $x$ 在两结果之间,又由 $n\geqslant3$ 有\[0<\left|\frac{(-1)^m}n\right|<0.5<\left|\frac{(n-1)(-1)^m}n\right|<1,\]所以就有上述结论。 所以,这个方法其实就是对原结果的一种化简公式,而且还方便了记忆,还是有一定益处的。 当然了,如果有线路限制,这种方法就使不得了,所以个人认为原文有标题党成份。 原文附件:
依然饭特稀 2# 2012-9-30 13:11
thread-846-1-6.html: [不等式] 问一道题,看看
yayaweha 1# 2012-10-1 12:58
本帖最后由 yayaweha 于 2013-2-7 22:45 编辑 不等式证明 $a_n=\frac{3^{2^(n-1)}}{3^{2^n}-1}$,$s_n=a_1+a_2+…a_n$, 求证 $$\frac{3}{8}≤s_n≤\frac{1}{2}$$
kuing 2# 2012-10-1 13:27
不等式证明 an=3^2^(n-1)/(3^2^n-1),sn=a1+a2+…an,求证3/8=<sn=<1/2 yayaweha 发表于 2012-10-1 12:58 不会。 PS、题外话一下:
叶剑飞Victor 3# 2012-10-2 00:33
本帖最后由 叶剑飞Victor 于 2012-10-2 00:51 编辑 2# kuing 在数学上,${\left({A}^{B}\right)}^{C}$的括号是必须的。而且数学上规定,${A}^{{B}^{C}}$表示${A}^{\left({B}^{C}\right)}$。 但是数学没有规定A^B^C表示什么,因为数学上根本就不承认“^”表示乘方符号。 另外,据测试: WolframAlpha认为A^B^C表示${A}^{\left({B}^{C}\right)}$。 卡西欧计算器 CASIO fx-82MS 型号认为A^B^C表示${\left({A}^{B}\right)}^{C}$。
thread-847-1-6.html: [函数] 来自群的一道证有界
kuing 1# 2012-10-2 15:13
爱好者-林<wayn*********com>  14:23:01 怎么证明?请教一下 群管-kuing<kuingggg@qq.com>  14:34:41 2 - |f(x)| = ((x^2-1)^2+3x^4+1)/(2(x^4+1)) > 0 爱好者-林<wayn*********com>  14:36:13 k哥是求导? 群管-kuing<kuingggg@qq.com>  14:36:46 你哪里看到有一撇? 爱好者-林<wayn*********com>  14:37:23 呵呵,那小弟不明白了!赐教 群管-kuing<kuingggg@qq.com>  14:37:48 …………那你当我没说过吧 爱好者-林<wayn*********com>  14:38:52 呵呵,说说一下思路!k哥 爱好者-/huaix(4744*****)  14:46:41 把这个函数的值域算出来吧  值域的端点就是上下确界 不过没有必要算出上下确界 爱好者-/huaix(4744*****)  14:47:48 k哥的更简单 群管-kuing<kuingggg@qq.com>  14:49:00 值域也容易,只不过他只是要证上界,就懒得算了,直接目测2就足够配方 “值域也容易”指的是这样: \[t-f(x)=\frac{t x^4-x^2+t-1}{x^4+1},\] 为使分子配成完全平方,我们需要分子关于 $x^2$ 的判别式为 $0$,即 $4t(t-1)=1$,解得 $t=\bigl(\sqrt2+1\bigr)/2$(因为 $t$ 肯定要是正的才能取到等,这里已舍去负根),于是得 \[\frac{\sqrt2+1}2-f(x)=\frac{\bigl(\sqrt2+1\bigr)\bigl(x^2-\sqrt2+1\bigr)^2}{2(x^4+1)},\] 所以最大值就是 $\bigl(\sqrt2+1\bigr)/2$,再由 $f(x)>0$ 及 $\lim_{x\to\infty}f(x)=0$ 知值域是 $\bigl(0,\bigl(\sqrt2+1\bigr)/2\bigr]$。 其实如果只是证有界,而且能用高数的一些什么定理之类的,大概直接由 $\lim_{x\to\infty}f(x)=0$ 以及 $f(x)$ 连续就直接能得到……
第一章 2# 2012-10-9 08:59
学习了不少,k真的厉害! 普通的都是用判别式法,或者用斜率,或者设$x^2+1=t$
realnumber 3# 2012-10-9 13:25
本帖最后由 realnumber 于 2012-10-9 13:26 编辑 又不是求上确界,我来就直接0<f(x)<2 ---原来kk早就说了
thread-848-1-6.html: [不等式] 05集训队不等式
kuing 1# 2012-10-2 15:33
爱好者-\huaix(4744*****)  14:52:14 这个是05年国家集训队的题 群管-kuing<kuingggg@qq.com>  14:52:38 嗯,果然没记错 爱好者-\huaix(4744*****)  14:52:36 看了韩京俊的做法 群管-kuing<kuingggg@qq.com>  14:52:49 用柯西可以做 爱好者-\huaix(4744*****)  14:53:22 pqr也可以 爱好者-\huaix(4744*****)  14:54:32 一不小心就放过头了 群管-kuing<kuingggg@qq.com>  14:55:45 居然通份 爱好者-\huaix(4744*****)  14:56:32 主要是别人发我的时候 我做的放过了头 后来看了韩大神的柯西做法 爱好者-\huaix(4744*****)  14:58:00 群管-kuing<kuingggg@qq.com>  14:58:14 还是用柯西比较常规 噢?他是局部柯西,跟我说的不一样其实 爱好者-\huaix(4744*****)  14:59:47 这个柯西不好想 真是神来之笔 群管-kuing<kuingggg@qq.com>  15:00:00 这个柯西就不太常规了 其实我所指的常规柯西是: \begin{align*} \sum\frac1{a^2-bc+1}&=\sum\frac{3(ab+bc+ca)}{a^2-bc+3(ab+bc+ca)} \\ & =\frac32\sum\frac{2(ab+bc+ca)}{a(a+b+c)+2(ab+bc+ca)} \\ & =\frac32\sum\left( 1-\frac{a(a+b+c)}{a(a+b+c)+2(ab+bc+ca)} \right) \\ & =\frac92-\frac{3(a+b+c)}2\cdot\sum\frac{a}{a(a+b+c)+2(ab+bc+ca)} \\ &\leqslant \frac92-\frac{3(a+b+c)}2\cdot\frac{(a+b+c)^2}{\sum(a^2(a+b+c)+2a(ab+bc+ca))}\\ &=3. \end{align*} 这样就常规多了。
thread-849-1-6.html: [不等式] 来自人教群的四元不等式
kuing 1# 2012-10-3 14:34
群管-kuing<kuingggg@qq.com>  14:31:32 切不出来,但可以柯出来 题目:设 $a\in(-\infty,0)$, $b$, $c$, $d\in(1/2,2)$,且 $a+b+c+d=2$,求证 \[\frac a{a^2-a+1}+\frac b{b^2-b+1}+\frac c{c^2-c+1}+\frac d{d^2-d+1}\leqslant\frac83.\] 证:易见原不等式等价于 \[\frac{(a+1)^2}{3(a^2-a+1)}\leqslant \frac{(b-1)^2}{b^2-b+1}+\frac{(c-1)^2}{c^2-c+1}+\frac{(d-1)^2}{d^2-d+1},\] 由柯西不等式有 \begin{align*} \frac{(b-1)^2}{b^2-b+1}+\frac{(c-1)^2}{c^2-c+1}+\frac{(d-1)^2}{d^2-d+1}&\geqslant \frac{(b+c+d-3)^2}{b^2+c^2+d^2-b-c-d+3} \\ & =\frac{(a+1)^2}{b^2+c^2+d^2-b-c-d+3}, \end{align*} 于是只要证 \[b^2+c^2+d^2-b-c-d+3\leqslant 3(a^2-a+1),\] 将 $a=2-b-c-d$ 代入上式化简,可以化为 \[\frac{(2b+2c+2d-3)^2+(2b-1)(2c-1)+(2b-1)(2d-1)+(2c-1)(2d-1)}2\geqslant0,\] 由条件知成立,故原不等式得证。 PS、由上述证明可见有多余条件,$a$ 无需限制为负,$b$, $c$, $d$ 也无需限制上限。
realnumber 2# 2012-10-22 09:34
本帖最后由 realnumber 于 2012-10-22 09:36 编辑 想用调整处理上面不等式,没成功, 碰到个问题$f(x)=\frac{x}{x^2-x+1}$ 若$x+y=k+1,对任意x,y都有f(x)+f(y)≤f(k)+f(1)$求k的取值范围. 或者找出k的一些范围,也许其他就可以"切"出来了
realnumber 3# 2012-11-9 10:39
应该算是调整出来了
kuing 4# 2012-11-15 19:20
又玩凹凸调整
thread-85-1-1.html: 来自群里的高次求值题 $x^{13}+\frac1{x^{13}}$
kuing 1# 2011-10-10 20:49
为方便书写,记 $T_k=x^k+\dfrac1{x^k}$,不断往上算,有 \begin{align*} T_1&=x + \frac{1}{x} = a \\ T_2&=x^2 + \frac{1}{{x^2 }} = a^2 - 2 \\ T_3&=x^3 + \frac{1}{{x^3 }} = \left( {x + \frac{1}{x}} \right)\left( {x^2 - 1 + \frac{1}{{x^2 }}} \right) = a(a^2 - 3) \\ T_4&=x^4 + \frac{1}{{x^4 }} = (a^2 - 2)^2 - 2 \\ T_5&=x^5 + \frac{1}{{x^5 }} = \left( {x + \frac{1}{x}} \right)\left( {x^4 + \frac{1}{{x^4 }} - x^2 - \frac{1}{{x^2 }} + 1} \right) = a\left( {(a^2 - 2)^2 - 2 - (a^2 - 2) + 1} \right) \\ T_6&=x^6 + \frac{1}{{x^6 }} = a^2 (a^2 - 3)^2 - 2 \end{align*} 由因式分解,并注意到 $T_{2k}=T_k^2-2$,我们有 \begin{align*} T_{13}&=x^{13} + \frac{1}{{x^{13} }}\\ &= \left( {x + \frac{1}{x}} \right)\left( {x^{12} + \frac{1}{{x^{12} }} - x^{10} - \frac{1}{{x^{10} }} + x^8 + \frac{1}{{x^8 }} - x^6 - \frac{1}{{x^6 }} + x^4 + \frac{1}{{x^4 }} - x^2 - \frac{1}{{x^2 }} + 1} \right) \\ &= T_1\left(T_6^2 - T_5^2 + T_4^2 - T_3^2 + T_2^2 - T_1^2 + 1\right) \end{align*} 将前面的值代入展开整理最终得\[ x^{13} + \frac1{x^{13}}=a \left(a^{12}-13 a^{10}+65 a^8-156 a^6+182 a^4-91 a^2+13\right) \]
kuing 2# 2011-10-10 21:27
另解 令 $a=2\cos t$,其中 $t\in\mathbb{C}$,那么由欧拉公式可知方程 $x+\dfrac1x=2\cos t$ 解得\[ x=e^{\pm it} \]于是再由欧拉公式得\[ x^{13}+\frac1{x^{13}}=e^{13it}+e^{-13it}=2\cos 13t \]利用已知的 $\cos n\theta$ 的展开式(切比雪夫多项式),有\[ 2\cos 13t = 8192 \cos ^{13}t-26624 \cos ^{11}t+33280 \cos ^9t-19968 \cos ^7t+5824 \cos ^5t-728 \cos ^3t+26 \cos t \]代回 $\cos t = \dfrac a2$ 化简得\[ 2\cos 13t=a \left(a^{12}-13 a^{10}+65 a^8-156 a^6+182 a^4-91 a^2+13\right) \]即\[ x^{13}+\frac1{x^{13}}=a \left(a^{12}-13 a^{10}+65 a^8-156 a^6+182 a^4-91 a^2+13\right) \]
nash 3# 2011-10-10 21:35
1# kuing 期待个一般性的结论 哈哈
kuing 4# 2011-10-10 21:37
二楼的解法其实已经说明了关键所在:这个问题与 $\cos n\theta$ 的展开式是等价的,而这个展开式问题早就有结论的了,那就是“切比雪夫多项式”。参考 这里
nash 5# 2011-10-10 22:04
不知道n倍角公式和切比雪夫多项式 …… 那一般的也就解决啦
nash 6# 2011-10-10 22:07
刚才没看到你楼上发的帖子
kuing 7# 2011-10-10 22:16
这里回贴快吧
nash 8# 2011-10-10 22:40
相当快 以后就在这常驻啦
kuing 9# 2011-10-10 23:10
8# nash
isea 10# 2011-10-12 20:16
慢慢乘,一般。 来学n次的
thread-85-1-9.html: 来自群里的高次求值题 $x^{13}+\frac1{x^{13}}$
kuing 1# 2011-10-10 20:49
为方便书写,记 $T_k=x^k+\dfrac1{x^k}$,不断往上算,有 \begin{align*} T_1&=x + \frac{1}{x} = a \\ T_2&=x^2 + \frac{1}{{x^2 }} = a^2 - 2 \\ T_3&=x^3 + \frac{1}{{x^3 }} = \left( {x + \frac{1}{x}} \right)\left( {x^2 - 1 + \frac{1}{{x^2 }}} \right) = a(a^2 - 3) \\ T_4&=x^4 + \frac{1}{{x^4 }} = (a^2 - 2)^2 - 2 \\ T_5&=x^5 + \frac{1}{{x^5 }} = \left( {x + \frac{1}{x}} \right)\left( {x^4 + \frac{1}{{x^4 }} - x^2 - \frac{1}{{x^2 }} + 1} \right) = a\left( {(a^2 - 2)^2 - 2 - (a^2 - 2) + 1} \right) \\ T_6&=x^6 + \frac{1}{{x^6 }} = a^2 (a^2 - 3)^2 - 2 \end{align*} 由因式分解,并注意到 $T_{2k}=T_k^2-2$,我们有 \begin{align*} T_{13}&=x^{13} + \frac{1}{{x^{13} }}\\ &= \left( {x + \frac{1}{x}} \right)\left( {x^{12} + \frac{1}{{x^{12} }} - x^{10} - \frac{1}{{x^{10} }} + x^8 + \frac{1}{{x^8 }} - x^6 - \frac{1}{{x^6 }} + x^4 + \frac{1}{{x^4 }} - x^2 - \frac{1}{{x^2 }} + 1} \right) \\ &= T_1\left(T_6^2 - T_5^2 + T_4^2 - T_3^2 + T_2^2 - T_1^2 + 1\right) \end{align*} 将前面的值代入展开整理最终得\[ x^{13} + \frac1{x^{13}}=a \left(a^{12}-13 a^{10}+65 a^8-156 a^6+182 a^4-91 a^2+13\right) \]
kuing 2# 2011-10-10 21:27
另解 令 $a=2\cos t$,其中 $t\in\mathbb{C}$,那么由欧拉公式可知方程 $x+\dfrac1x=2\cos t$ 解得\[ x=e^{\pm it} \]于是再由欧拉公式得\[ x^{13}+\frac1{x^{13}}=e^{13it}+e^{-13it}=2\cos 13t \]利用已知的 $\cos n\theta$ 的展开式(切比雪夫多项式),有\[ 2\cos 13t = 8192 \cos ^{13}t-26624 \cos ^{11}t+33280 \cos ^9t-19968 \cos ^7t+5824 \cos ^5t-728 \cos ^3t+26 \cos t \]代回 $\cos t = \dfrac a2$ 化简得\[ 2\cos 13t=a \left(a^{12}-13 a^{10}+65 a^8-156 a^6+182 a^4-91 a^2+13\right) \]即\[ x^{13}+\frac1{x^{13}}=a \left(a^{12}-13 a^{10}+65 a^8-156 a^6+182 a^4-91 a^2+13\right) \]
nash 3# 2011-10-10 21:35
1# kuing 期待个一般性的结论 哈哈
kuing 4# 2011-10-10 21:37
二楼的解法其实已经说明了关键所在:这个问题与 $\cos n\theta$ 的展开式是等价的,而这个展开式问题早就有结论的了,那就是“切比雪夫多项式”。参考 这里
nash 5# 2011-10-10 22:04
不知道n倍角公式和切比雪夫多项式 …… 那一般的也就解决啦
nash 6# 2011-10-10 22:07
刚才没看到你楼上发的帖子
kuing 7# 2011-10-10 22:16
这里回贴快吧
nash 8# 2011-10-10 22:40
相当快 以后就在这常驻啦
kuing 9# 2011-10-10 23:10
8# nash
isea 10# 2011-10-12 20:16
慢慢乘,一般。 来学n次的
thread-850-1-6.html: [不等式] 再研究$1/a+1/b+1/c+k/(a+b+c)$
kuing 1# 2012-10-4 00:18
话说今晚人教群里网友“爱好者-程汉波(2872*****)”提及了几个形式是 $1/a+1/b+1/c+k/(a+b+c)$ 的不等式(条件是 $a,b,c>0,abc=1$),于是我给他发了这个链接:http://www.artofproblemsolving.com/blog/38834,这是我两年前得到的结果,给出了 $a=b=c$ 取最小值时的 $k$ 的最大值。 但随后该网友继续问“$k$ 不在这个范围内的时候,不等式左边的最值是什么?”,嗯,这的确值得继续探讨的,我不记得当年是否有考虑过,也许是没搞出来。 刚才无聊又试着搞搞,哎,带参数的东西还是不太在行,暂时胡乱搞出了如下猜想: 猜想:已知 $a$, $b$, $c>0$, $abc=1$,记方程 $16x^3+567x^2-3402x-18225=0$ 的唯一正数根为 $K\approx8.1086$,给定 $k\in\mbb R$,记 $1/a+1/b+1/c+k/(a+b+c)$ 的最小值为 $f(k)$,则 \[f(k)=\begin{cases} \displaystyle 3+\frac k3, & k\leqslant K,\\ \displaystyle \sqrt[3]{\frac{27}{32}\bigl(k^2+20k-8-\sqrt{k(k-8)^3}\bigr)}, & k>K. \end{cases}\] 其中前半部分就是链接中的结果了,这里猜想的是后半部分那个。 时间关系,闪一下先。
ccnu_chb_ycb 2# 2012-10-4 00:49
呵呵,明天好好看下这个,谢谢哟
kuing 3# 2012-10-4 00:57
2# ccnu_chb_ycb 先不用谢,我暂时还没完全证出来,只是猜着…… 至于那个 uvw 定理你去 mathlinks 找找相关文章。
thread-851-1-6.html: [不等式] 请教一个优美不等式命题的证明
ccnu_chb_ycb 1# 2012-10-4 19:07
请论坛里的高手们参与本题的讨论,谢谢了……本人随时关注大家的讨论哟,有想法的也可以和我联系; 程汉波:QQ287240279   邮箱:chenghanbo511@sina.com
kuing 2# 2012-10-4 19:19
你的标题真是从一而终 PS1、相关链接还是要给给的:http://kkkkuingggg.5d6d.net/view ... &page=1#pid3744 PS2、怎么不试下用代码?就两个简单公式而已,何必贴图(还贴两个)……
ccnu_chb_ycb 3# 2012-10-4 19:37
恩, ①我想这个标题也许更吸引不等式方面人的注意。 ②这个标题也说明我好记写,而且也不好取其他的标题啊。 ③我不小心贴了两个一样的图上去了,也没改下。 ④我正准备学latex编辑,但是一直没多少时间,呵呵,以后一定会的
ccnu_chb_ycb 4# 2012-10-4 19:39
2# kuing 请指教下还有没有更自然些的解法呢,因为这里用到了两个引理,总感觉有些不自然……应该会有更好的解法吧
kuing 5# 2012-10-4 21:01
恩, ①我想这个标题也许更吸引不等式方面人的注意。 ②这个标题也说明我好记写,而且也不好取其他的标题啊。 ③我不小心贴了两个一样的图上去了,也没改下。 ④我正准备学latex编辑,但是一直没多少时间,呵呵,以后一定会的 ccnu_chb_ycb 发表于 2012-10-4 19:37 回①②:但太千篇一律不方便找,或者你可以将一些关键式子也打上标题。 就像这个贴这样:http://kkkkuingggg.5d6d.net/thread-237-1-7.html; 回③:删除多余附件可参考:http://kkkkuingggg.5d6d.net/thread-119-1-1.html; 回④:在本论坛上对于并不复杂的公式输入,看置顶贴足够应付,并非很复杂,不用花很多时间的。 比如一楼的不等式这样输入 \$(x^2+2y)(y^2+2z)(z^2+2x)\ge9xyz(x+y+z)\$ 就可以了。
kuing 6# 2012-10-4 22:11
想到新方法了,后面发现竟然很弱很弱。 证:首先两边除以 $xyz$,等价于 \[\left( \frac{x^2}z+\frac yz+\frac yz \right)\left( \frac zx+\frac{y^2}x+\frac zx \right)\left( \frac xy+\frac xy+\frac{z^2}y \right)\geqslant 9(x+y+z),\] 由 Holder 不等式之类的,有 \[\left( \frac{x^2}z+\frac yz+\frac yz \right)\left( \frac zx+\frac{y^2}x+\frac zx \right)\left( \frac xy+\frac xy+\frac{z^2}y \right)\geqslant \left( \sqrt[3]{\frac{x^2}y}+\sqrt[3]{\frac{y^2}z}+\sqrt[3]{\frac{z^2}x} \right)^3,\] 为方便处理,换元去掉三次根号,令 $x=a^3$, $y=b^3$, $z=c^3$, $a$, $b$, $c>0$,则要证原不等式,只要证 \begin{equation}\label{20121004jtzzs1} \left( \frac{a^2}b+\frac{b^2}c+\frac{c^2}a \right)^3\geqslant 9(a^3+b^3+c^3). \end{equation} 对于式 \eqref{20121004jtzzs1} 的左边,由柯西不等式,有 \begin{align*} \frac{a^2}b+\frac{b^2}c+\frac{c^2}a&\geqslant \frac{(a^2+b^2+c^2)^2}{a^2b+b^2c+c^2a} \\ & \geqslant \frac{(a^2+b^2+c^2)^2}{\sqrt{(a^2+b^2+c^2)(a^2b^2+b^2c^2+c^2a^2)}} \\ & =\sqrt{\frac{(a^2+b^2+c^2)^3}{a^2b^2+b^2c^2+c^2a^2}}, \end{align*} 即有 \[\left( \frac{a^2}b+\frac{b^2}c+\frac{c^2}a \right)^3\geqslant \sqrt{\frac{(a^2+b^2+c^2)^9}{(a^2b^2+b^2c^2+c^2a^2)^3}},\] 而对于式 \eqref{20121004jtzzs1} 的右边,亦由柯西不等式,有 \[9(a^3+b^3+c^3)\leqslant 9\sqrt{(a^2+b^2+c^2)(a^4+b^4+c^4)},\] 于是,要证式 \eqref{20121004jtzzs1},只要证 \begin{equation}\label{20121004jtzzs2} \frac{(a^2+b^2+c^2)^8}{(a^2b^2+b^2c^2+c^2a^2)^3}\geqslant 81(a^4+b^4+c^4). \end{equation} 令 $a^2+b^2+c^2=p$, $a^2b^2+b^2c^2+c^2a^2=q$,则式 \eqref{20121004jtzzs2} 等价于 \begin{equation}\label{20121004jtzzs3} p^8\geqslant 81q^3(p^2-2q). \end{equation} 由均值不等式,有 $3q\leqslant p^2$ 以及 \[27q^2(p^2-2q)\leqslant 27\left( \frac{2q+p^2-2q}3 \right)^3=p^6,\] 即得式 \eqref{20121004jtzzs3},所以原不等式得证。 由上述证明中的放缩程度可见式 \eqref{20121004jtzzs1} 其实还挺弱的…… 不知这个算不算自然了些?
ccnu_chb_ycb 7# 2012-10-4 23:50
佩服这个解法的同时……还是对于这个解答的过程有些许不满意,总感觉想找个简单优美的解法。呵呵,你对于不等式的功底确实深厚
realnumber 8# 2012-10-5 07:07
先展开,再分成三组,分别用AG,相加--居然也行 $(x^2+2y)(y^2+2z)(z^2+2x)=x^2y^2z^2+2z^3x^2+2y^3z^2+2x^3y^2+4yz^3+4zx^3+4xy^3+8xyz$ 而$\frac{1}{3}x^2y^2z^2+2x^3y^2+4zx^3+\frac{8}{3}xyz \ge \frac{27}{3}x^\frac{54}{27}y^\frac{22}{27}z^\frac{22}{27}$ 同理,类似2个 三个相加,得到 $(x^2+2y)(y^2+2z)(z^2+2x)  \ge 9(xyz)^\frac{22}{27}(x^\frac{32}{27}+y^\frac{32}{27}+z^\frac{32}{27})$
ccnu_chb_ycb 9# 2012-10-5 09:10
8# realnumber 54/27的地方有些问题,应该是(63+1)/27。还有就是证到最后的时候不是和要证明的不等式还有些差距吗?是怎么处理的呢 _____kuing edited 处理敏感数字_____
realnumber 10# 2012-10-5 10:01
54的话,正好用契比雪夫不等式,63+1我不会了,哎~~~,白忙了 要不过会搭配下别的,还是这个办法,先外出买下东西---应该没戏, _____kuing edited 处理敏感数字_____
kuing 11# 2012-10-5 12:57
10# realnumber 展开配均值是可以的,我在http://kkkkuingggg.5d6d.net/view ... &page=1#pid3739也提到过,但是由于真的挺暴力,所以当时就没贴上来。既然如此,还是把当年的word发发好了,见附件。
kuing 12# 2012-10-5 13:01
顺便处理了一下9#和10#的敏感数字。 你懂的
thread-852-1-6.html: [不等式] 来自群的几何不等式,内切圆再来三个圆切着
kuing 1# 2012-10-5 01:02
提问人未配图,其实应该配一个的,否则那三个圆的位置按照原题的叙述其实是不够清晰的。 我顺手画了个,大概是这个样子吧: 我用内切圆代换系统证到了,寻找几何方法ing,或者简单点的代数方法也可以。 时间关系,洗个澡回来再写内切圆代换的证法。
kuing 2# 2012-10-5 02:58
由 \[OA=OO_1+AO_1=R+R_1+R_1\cdot \frac{OA}R,\] 得 \[R_1=R\cdot \frac{OA-R}{OA+R}=R\cdot \frac{(OA-R)^2}{OA^2-R^2}=R\cdot \frac{(OA-R)^2}{AE^2},\] 用内切圆代换系统,设 $AE=AF=x$, $BF=BD=y$, $CD=CE=z$,则有 \begin{align*} R&=\sqrt{\frac{xyz}{x+y+z}}, \\ OA&=\sqrt{\frac{x(x+y)(x+z)}{x+y+z}}, \end{align*} 代入并运用柯西不等式,有 \begin{align*} R_1&=\frac R{x^2}\left( \sqrt{\frac{x(x+y)(x+z)}{x+y+z}}-\sqrt{\frac{xyz}{x+y+z}} \right)^2 \\ & =\frac R{x(x+y+z)}\bigl( \sqrt{(x+y)(x+z)}-\sqrt{yz} \bigr)^2 \\ & \geqslant \frac R{x(x+y+z)}\bigl( x+\sqrt{yz}-\sqrt{yz} \bigr)^2 \\ & =\frac x{x+y+z}\cdot R, \end{align*} 同理有另外两式,三式相加即得 \[R_1+R_2+R_3\geqslant R.\]
kuing 3# 2012-10-5 03:13
( ⊙ o ⊙ )啊!我笨了,其实得到 $R_1$ 的表达式之后应该转化为三角函数更简单。 如上得到 \[R_1=R\cdot \frac{OA-R}{OA+R}=R\cdot \frac{1-\frac R{OA}}{1+\frac R{OA}}=R\cdot \frac{1-\sin\frac A2}{1+\sin\frac A2},\] 于是 \[\frac{R_1+R_2+R_3}R=\frac{1-\sin\frac A2}{1+\sin\frac A2}+\frac{1-\sin\frac B2}{1+\sin\frac B2}+\frac{1-\sin\frac C2}{1+\sin\frac C2},\] 易证函数 $f(x)=(1-\sin x)/(1+\sin x)$ 在 $(0,\pi/2)$ 上是严格下凸的,所以当且仅当 $A=B=C=\pi/3$ 时上式取最小值为 $1$,即得证。 这样就简单多了,当然,最后可能不必用凸函数也能证出来,但时间关系我就懒了。不过2#得到的局部不等式也挺漂亮,也不算完全白干吧……闪了
kuing 4# 2012-10-5 03:59
嗯,最后也可以用柯西加个熟知的三角不等式,爪机就不打代码了,自己试试吧。 水饺…
kuing 5# 2012-10-5 12:49
嗯,最后也可以用柯西加个熟知的三角不等式,爪机就不打代码了,自己试试吧。 水饺… kuing 发表于 2012-10-5 03:59 醒了,哎,反正木银,不写白不写。 \[\sum\frac{1-\sin\frac A2}{1+\sin\frac A2}=-3+2\sum\frac1{1+\sin\frac A2}\geqslant-3+\frac{18}{3+\sum\sin\frac A2}\geqslant-3+\frac{18}{3+\frac32}=1.\]
thread-853-1-1.html: 看到粉丝群里一道向量与三角形的心的题,想起一系列……
kuing 1# 2012-10-5 17:06
已知 $\triangle ABC$ 中,$\abs{BC}=a$, $\abs{CA}=b$, $\abs{AB}=c$,$O$ 为任一定点。 (1)若点 $P$ 满足 \[\vv{OP}=\vv{OA}+\lambda\left(\frac{\vv{AB}}c+\frac{\vv{AC}}b\right),\] 其中 $\lambda\in\mbb R$,则 $P$ 的轨迹一定通过 $\triangle ABC$ 的( )心; (2)若点 $P$ 满足 \[\vv{OP}=\vv{OA}+\lambda\left(\frac{\vv{AB}}{c\sin B}+\frac{\vv{AC}}{b\sin C}\right),\] 其中 $\lambda\in\mbb R$,则 $P$ 的轨迹一定通过 $\triangle ABC$ 的( )心; (3)若点 $P$ 满足 \[\vv{OP}=\vv{OA}+\lambda\left(\frac{\vv{AB}}{c\cos B}+\frac{\vv{AC}}{b\cos C}\right),\] 其中 $\lambda\in\mbb R$,则 $P$ 的轨迹一定通过 $\triangle ABC$ 的( )心; (4)若点 $P$ 满足 \[\vv{OP}=\vv{OA}+\lambda\left(\frac{\vv{AB}}{c\cos C}+\frac{\vv{AC}}{b\cos B}\right),\] 其中 $\lambda\in\mbb R$,则 $P$ 的轨迹一定通过 $\triangle ABC$ 的( )心; 暂时想起这四个,还有没有类似?这当作是收集贴吧 :D 今天群里提到的是第(4)个,也正好是这个我还没找到简洁的方法。
kuing 2# 2012-10-5 19:25
四个的难度可以说由简单到难。 第(1)个显然轨迹是角平分线所在直线就不用说了吧,所以过内心; 第(2)个,由 $c\sin B=b\sin C=2S/a$,得 $\vv{OP}=\vv{OA}+\mu\bigl(\vv{AB}+\vv{AC}\bigr)$,可见 $P$ 的轨迹是中线所在直线,所以过重心; 第(3)个,由 \[\left(\frac{\vv{AB}}{c\cos B}+\frac{\vv{AC}}{b\cos C}\right)\cdot\vv{BC} =-\frac{\vv{BA}\times\vv{BC}}{c\cos B}+\frac{\vv{CA}\times\vv{CB}}{b\cos C} =-a+a=0, \]可见 $P$ 的轨迹是高线所在直线,所以过垂心; 第(4)个,令 $\lambda=bc\cos B\cos C\mu$, $\mu\in\mbb R$, $\vv m = \vv{AB}b\cos B+\vv{AC}c\cos C$,则已知等式化为 \[\vv{OP}=\vv{OA}+\mu\vv m,\] 关键在于 $\vv m$,我们来考查下它的模长,有 \begin{align*} \abs m &=\sqrt{\bigl(\vv{AB}b\cos B+\vv{AC}c\cos C\bigr)^2} \\ & =\sqrt{c^2b^2\cos^2B+b^2c^2\cos^2C+2b^2c^2\cos A\cos B\cos C} \\ & =bc\sqrt{1-\cos^2A} \\ & =bc\sin A, \end{align*} 再考查它与两边的夹角,有 \begin{align*} \cos \bigl\langle \vv m,\vv{AB} \bigr\rangle &=\frac{\bigl(\vv{AB}b\cos B+\vv{AC}c\cos C\bigr)\cdot \vv{AB}}{bc\sin A\cdot c} \\ & =\frac{c^2b\cos B+bc^2\cos A\cos C}{bc^2\sin A} \\ & =\frac{\cos B+\cos A\cos C}{\sin A} \\ & =\sin C, \end{align*} 同理有 $\cos \bigl\langle \vv m,\vv{AC} \bigr\rangle =\sin B$,由此可见 $P$ 的轨迹是过点 $A$ 和外心的直线。 如果最后一行看不懂,请对着这个图再想想,注意一些角度的相等。
kuing 3# 2012-10-5 21:20
继续,来个你们可能不认识的心。 (5)若点 $P$ 满足 \[\vv{OP}=\vv{OA}+\lambda\left(\frac{\vv{AB}}{c\sin C}+\frac{\vv{AC}}{b\sin B}\right),\] 其中 $\lambda\in\mbb R$,则 $P$ 的轨迹一定通过 $\triangle ABC$ 的(   )心。 由于 \[\left|\frac{\vv{AB}}{c\sin C}+\frac{\vv{AC}}{b\sin B}\right|=\left|\frac{\vv{AB}}{c\sin B}+\frac{\vv{AC}}{b\sin C}\right|,\] 且 \[\frac{\vv{AB}}{c\sin C}+\frac{\vv{AC}}{b\sin B} + \frac{\vv{AB}}{c\sin B}+\frac{\vv{AC}}{b\sin C} = \left(\frac1{\sin B}+\frac1{\sin C}\right)\left(\frac{\vv{AB}}c+\frac{\vv{AC}}b\right),\] 结合(1)(2)的证明,可知 $\frac{\vv{AB}}{c\sin C}+\frac{\vv{AC}}{b\sin B}$ 与 $\frac{\vv{AB}}{c\sin B}+\frac{\vv{AC}}{b\sin C}$ 这两个向量关于 $A$ 的内角平分线对称,因此 $P$ 的轨迹是与中线所在直线关于内角平分线对称的直线,也就是类似中线,所以必过 $\triangle ABC$ 的类似重心。 关于类似中线及类似重心的定义可参考: http://zh.wikipedia.org/wiki/%E7%B1%BB%E4%BC%BC%E9%87%8D%E5%BF%83
kuing 4# 2012-10-5 21:31
咦?这样看来,其实(4)也可以仿楼上那样,证明那个向量与(3)的关于内角平分线对称。 这样的话,要证(4)就只需证明:过外心和 $A$ 的直线与过 $A$ 的高线关于 $A$ 的内角平分线对称。 这根据角度应该不难证,不过可能还是要分一下钝角和锐角……
kuing 5# 2012-10-6 00:08
再来自创两个。 (6)若点 $P$ 满足 \[\vv{OP}=\vv{OA}+\lambda\left(\frac{\vv{AB}}{a+b-c}+\frac{\vv{AC}}{a+c-b}\right),\] 其中 $\lambda\in\mbb R$,则 $P$ 的轨迹一定通过 $\triangle ABC$ 的( )心; (7)若点 $P$ 满足 \[\vv{OP}=\vv{OA}+\lambda\left(\frac{\vv{AB}}{a+c-b}+\frac{\vv{AC}}{a+b-c}\right),\] 其中 $\lambda\in\mbb R$,则 $P$ 的轨迹一定通过 $\triangle ABC$ 的( )心。 参考答案:(6)界心;(7)切心。 PS、还是介绍一下相关定义。 三角形的分周线:若一条直线将三角形的周长平分,称此直线为该三角形的一条分周线。 界心:过三角形三个顶点的三条分周线交于一点,称这点为界心。(也有称为“Nagel 点”的) 切心:三角形内切圆与三边的三个切点与其对应顶点的连线交于一点,称这点为切心。(也有称为“Gergonne 点”的)
依然饭特稀 6# 2012-10-6 00:24
第四个有没有更好的方法啊
kuing 7# 2012-10-6 00:26
6# 依然饭特稀 看 3#、4#。 不要急着回贴,看完先。 2#是我原先的证法,直到我在3#创了(5)并利用(2)证出之后,发现(3)和(4)其实也有这样的关系,这已经在4#作了说明,所以(4)的证明已经变得简单,只是我就没详写,只是画了两个图,已经很显然。
kuing 8# 2012-10-6 00:43
洗个澡先,回来再看看有哪些心能构造类似的题出来
Tesla35 9# 2012-10-6 20:28
第一个是2003年的高考题呀。。
realnumber 10# 2012-10-19 09:14
本帖最后由 realnumber 于 2012-10-19 10:48 编辑 (4)也可以这样验证,首先求出λ1,使得P在AB的垂直平分线上;再求出λ2,使得P在AC的垂直平分线上 计算结果发现λ1=λ2,由此可见此时P为外心. $(\vv{AP}-0.5\vv{AB})·0.5\vv{AB}=0$,解得$λ=\frac{RcosBcosC}{sinA}$ 再次计算$(\vv{AP}-0.5\vv{AC})·0.5\vv{AC}=0$,还是有$λ=\frac{RcosBcosC}{sinA}$ 如此,说明$λ=\frac{RcosBcosC}{sinA}$时,点P为外心,完.
kuing 11# 2012-10-19 13:01
10# realnumber 嗯,这个方法也挺好
转化与化归 12# 2012-10-21 15:43
第(4)题 外心的那个题
kuing 13# 2012-10-21 15:44
12# 转化与化归 要不要分圆心在三角形内和外讨论下?
转化与化归 14# 2012-10-21 15:55
呃,要讨论一下,方法是一样的,我就偷懒不写了!
kuing 15# 2012-10-21 16:01
嗯,这个方法很好
realnumber 16# 2013-1-15 08:52
本帖最后由 realnumber 于 2013-1-15 09:09 编辑 http://kkkkuingggg.5d6d.net/thread-220-1-14.html 还有这个,原贴有图图的解答. 已知:P为三角形ABC所在平面上一点,满足$a\overrightarrow{PA}+b\overrightarrow{PB}+c\overrightarrow{PC}=\vec{0}$,其中$a=│BC│,b=│AC│,c=│AB│$ 则P是三角形的____心. 1楼解题群有人在问,就顺便顶了下.
jogger04 17# 2013-5-18 17:09
mark!
李斌斌755 18# 2013-5-18 17:14
9# Tesla35 是高考题,几个心都是用向量做的,真闹心……
thread-855-1-1.html: 弧微分证明
iphone 1# 2012-10-8 11:08
请教各位,如何严格证明弧微分公式,即(dx)^2+(dy)^2=(ds)^2 书上说曲线上两点无限接近,这时两点间弧长和直线距离是等价无穷小,这好像是说明,而不是证明。
kuing 2# 2012-10-8 13:59
我也不是很懂,不知是不是大概这样:\(\require{cancel}\) \[\xcancel{\left(\frac{\Delta s}{\Delta x}\right)^2=\frac{(\Delta x)^2+(\Delta y)^2}{(\Delta x)^2}}=1+\left(\frac{\Delta y}{\Delta x}\right)^2,\] 当 $\rmd y/\rmd x$ 存在时 \[ \left(\frac{\rmd s}{\rmd x}\right)^2=\lim_{\Delta x\to0}\left(\frac{\Delta s}{\Delta x}\right)^2=1+\lim_{\Delta x\to0}\left(\frac{\Delta y}{\Delta x}\right)^2=1+\left(\frac{\rmd y}{\rmd x}\right)^2, \] 所以 $(\rmd s)^2=(\rmd x)^2+(\rmd y)^2$。 也就是说这条公式成立大概需要函数可导? 错了……谢楼下指出,cancel 掉先
iphone 3# 2012-10-8 14:57
(Δs)^2=(Δx)^2+(Δy)^2,why? 我也知几何应用和物理应用中,等价无穷小都是想象出的,不证明。但这个弧长公式属于微积分最简单应用,不知如何从数学上严格证明。是否需构造函数或是极限定义法?
kuing 4# 2012-10-8 15:07
噢,那样的确错了 错得很离谱……叉掉了……
kuing 5# 2012-10-8 16:06
好像要 $y'$ 也连续,由弧长公式推弧微分公式,其中弧长公式用定义推导,这样应该不会循环论证…… 当 $y'$ 连续,有 \[s(x)=\int_{x_0}^x\sqrt{1+(y')^2}\rmd x,\] 故 \[\frac{\rmd s}{\rmd x}=\sqrt{1+(y')^2}=\sqrt{1+\left(\frac{\rmd y}{\rmd x}\right)^2},\] 所以……
秋风树林 6# 2012-10-8 23:16
弧长是有定义的 就是那啥比较麻烦的Darboux和 只有当任意划分都能使得Darboux大和和小和趋于同一个东西才叫可求长 在此过程中就能诱导出一个微分表达式,定义为弧微分 同样,面积也是需要类似的过程来定义的
kuing 7# 2012-10-9 00:03
6# 秋风树林 我不懂了,高等的基础概念的东东还是你比较清楚些。
秋风树林 8# 2012-10-11 15:07
7# kuing 其实就是用许多的折线段来逼近曲线 当这些折线段不管怎么划分,取极限后都为一个值的话,就叫可求长,并定义为曲线的长 这就是我目前学到的了 同样的,面积也是对一个图形作任意划分,其Darboux小和和Darboux大和趋于同一个量,叫可求面积 所以,其实弧微分是定义出来的...
thread-856-1-2.html: 以前睇人玩,而家自己玩
kuing 1# 2012-10-8 18:09

kuing 2# 2012-10-23 16:28
选主角玩
kuing 3# 2012-10-23 16:30
差一丝血也输掉了第一个真是……早知道先爆一个pow可能就赢了
kuing 4# 2012-10-29 03:16
一对一赢一丝
thread-857-1-6.html: [不等式] 来自群的一道整系数二次函数题
kuing 1# 2012-10-11 13:00
题目:已知 $a$, $b$, $c$ 为正整数,方程 $ax^2+bx+c=0$ 的两实根为 $x_1$, $x_2$($x_1\ne x_2$),且 $\abs{x_1}<1$, $\abs{x_2}<1$,求 $a+b+c$ 的最小值。 跟以前的一道FAQ类似,但比那个复杂,吃完饭回来再写解答。 话说绝大多数人还是宁愿把题发在群里尽管没人鸟也不发在这里,只能我见一次转一次好了。
kuing 2# 2012-10-11 14:07
显然两根均为负,所以不妨设 $-1<x_2<x_1<0$,为方便书写,令 $p=-x_1$, $q=-x_2$,则 $0<p<q<1$。 记 $f(x)=ax^2+bx+c$,由于开口向上且两根在 $(-1,0)$ 内,所以有 $f(0)>0$, $f(-1)>0$,又由于 $f(0)$, $f(-1)$ 为整数,所以必有 $f(0)\geqslant 1$, $f(-1)\geqslant 1$。 $f(x)$ 可以表为 $f(x)=a(x-x_1)(x-x_2)=a(x+p)(x+q)$,于是有 \begin{gather*} 1\leqslant f(0)=apq,\\ 1\leqslant f(-1)=a(1-p)(1-q), \end{gather*} 两式相乘,由基本不等式并注意到不能同时取等,有 \[ 1\leqslant a^2p(1-p)q(1-q)<a^2\left(\frac{p+1-p}2\right)^2\left(\frac{q+1-q}2\right)^2=\frac{a^2}{16}, \] 所以 $a>4$,即 $a\geqslant 5$。 再由基本不等式,有 \begin{align*} a+b+c&=f(1)\\ &=a(1+p)(1+q)\\ &=a\left(\frac12+\frac12+p\right)\left(\frac12+\frac12+q\right)\\ &>a\cdot3\sqrt[3]{\frac p4}\cdot3\sqrt[3]{\frac q4}\\ &=9\sqrt[3]{\frac{a^2}{16}f(0)}\\ &\geqslant 9\sqrt[3]{\frac{25}{16}}, \end{align*} 在数值上,我们有 $9^3\cdot25/16>9^2\cdot8\cdot25/16=8100/8>1000$,于是 $a+b+c>9\sqrt[3]{25/16}>10$,所以 $a+b+c\geqslant 11$。 又当 $a=b=5$, $c=1$ 时符合所有条件且 $a+b+c=11$,所以最小值就是 $11$。
kuing 3# 2012-10-11 14:15
以前那道FAQ就是求 $a$ 的最小值,也就是楼上的前半部分解答。现在求 $a+b+c$ 的最小值还是照例地均值,只是后面相对复杂一些。
kuing 4# 2012-10-11 14:36
再来一个更简单的用柯西来证。 前面同2#,后面改用柯西不等式,有 \begin{align*} a+b+c&=f(1)\\ &=a(1+p)(1+q)\\ &>a\bigl(1+\sqrt{pq}\bigr)^2\\ &=\bigl(\sqrt a+\sqrt{f(0)}\bigr)^2\\ &\geqslant \bigl(\sqrt5+1\bigr)^2\\ &=6+2\sqrt5\\ &>6+2\cdot2\\ &=10, \end{align*} 同样得到 $a+b+c>10$ 即 $a+b+c\geqslant11$。
hongxian 5# 2012-10-11 17:55
1# kuing 看样子下次真得记住发到这里了,不过不要嫌太简单了! 人教哪边是越来越冷清了!
kuing 6# 2012-10-11 17:58
5# hongxian 其实我也没怪谁,这里更冷清。 我发来这里主要是这里方便。
thread-858-1-6.html: [几何] 关于 $\angle MBA=k\angle MAB$ 的轨迹
kuing 1# 2012-10-12 17:01
刚才无聊看352的博客,看到此文http://blog.sina.com.cn/s/blog_68ef132301016aqo.html,文中提到的是关于双曲线与 $\angle MBA=2\angle MAB$ 的题目。 话说我对那些题的什么前世今生并不感兴趣,倒是对一般情况更加感兴趣,也就是当更一般的角度比关系 $\angle MBA=k\angle MAB$ 时会形成什么曲线呢? 不失一般性,设 $A(-1,0)$, $B(1,0)$,动点 $M$ 满足 $\angle MBA=k\angle MAB$,$k>1$。 设 $M(x,y)$,由 $k>1$ 可知点 $M$ 在 $y$ 轴右边,即 $x>0$,由对称性,我们先考虑在第一象限内的情形(对称下去就是第四象限内的情形,这也是后面得到的那些方程中的 $y$ 总是偶次的原因)。 当 $\angle MBA$ 不是直角时,有 \begin{align*} \tan\angle MAB&=\frac y{x+1},\\ \tan\angle MBA&=\frac y{1-x}, \end{align*} 解出 $x$, $y$ 并代入 $\angle MBA=k\angle MAB$,有 \begin{equation*} \left\{\begin{aligned} x&=\frac{\tan k\angle MAB-\tan\angle MAB}{\tan k\angle MAB+\tan\angle MAB},\\ y&=\frac{2\tan k\angle MAB\cdot\tan\angle MAB}{\tan k\angle MAB+\tan\angle MAB}, \end{aligned}\right. \end{equation*} 这就是 $M$ 点轨迹的参数方程(严格地说是缺少了一个点的,可以另行补上,这里暂且不提),其中 $\angle MAB$ 为参数,由 $0<\angle MBA+\angle MAB<\pi$ 得 $0<\angle MAB<\dfrac\pi{k+1}$。 再把第四象限也考虑上,那么只需写成 \begin{equation}\label{kpjcsfc} \left\{\begin{aligned} x&=\frac{\tan kt-\tan t}{\tan kt+\tan t},\\ y&=\frac{2\tan kt\cdot\tan t}{\tan kt+\tan t}, \end{aligned}\right. \quad t~\text{为参数,}\abs t<\frac\pi{k+1}. \end{equation} 参数方程有了,那能不能消去参数从而得到一个只有 $x$, $y$ 的方程甚至写成显函数的形式? 首先说明一点,如果定值 $k$ 是个无理数,那显然已经超出我的能力范围(在还没得到上述参数方程之前,我的第一感觉就是如果是无理数比,肯定不会有简单的结果,那大概是数学家才能玩的东西,所以我绝对不会提出352文中的那个猜想,强烈的直觉告诉我不可能有这么简单的结果),因此下面我只研究有理数比的情形。 设 $k=\dfrac nm$,其中 $m$, $n\in\mbb N^+$,互素且 $n>m$。则 $\angle MBA:\angle MAB=n:m$, $\angle MAB<\dfrac{m\pi}{m+n}$,所以 $\dfrac y{x+1}<\tan\dfrac{m\pi}{m+n}$。 由比例条件,可设 $\dfrac{\angle MBA}n=\dfrac{\angle MAB}m=\theta $,则由 $n$ 倍角公式,有 \begin{align} \tan\angle MAB&=\tan m\theta=\frac{C_m^1\tan^1\theta -C_m^3\tan^3\theta +C_m^5\tan^5\theta -\cdots }{C_m^0\tan^0\theta -C_m^2\tan^2\theta +C_m^4\tan^4\theta -\cdots },\\ \tan\angle MBA&=\tan n\theta=\frac{C_n^1\tan^1\theta -C_n^3\tan^3\theta +C_n^5\tan^5\theta -\cdots }{C_n^0\tan^0\theta -C_n^2\tan^2\theta +C_n^4\tan^4\theta -\cdots }, \end{align} 所以,由以下方程组 \begin{equation} \left\{\begin{aligned} \frac y{x+1}&=\frac{C_m^1\tan^1\theta -C_m^3\tan^3\theta +C_m^5\tan^5\theta -\cdots }{C_m^0\tan^0\theta -C_m^2\tan^2\theta +C_m^4\tan^4\theta -\cdots },\\ \frac y{1-x}&=\frac{C_n^1\tan^1\theta -C_n^3\tan^3\theta +C_n^5\tan^5\theta -\cdots }{C_n^0\tan^0\theta -C_n^2\tan^2\theta +C_n^4\tan^4\theta -\cdots }, \end{aligned}\right. \end{equation} 消去 $\tan\theta $ 后得出的方程所确定的曲线就是点 $M$ 所在的曲线。 这个消元在理论上是必定能消去的(用结式之类),但是对于较大的 $m$, $n$,实在非软件做不可,而且可以预见,当 $m$, $n$ 越大,最后出来的方程的次数必定越高,而且还需要排除曲线的一些多余部分才能得到真正的 $M$ 的轨迹,所以对于一般的 $m$, $n$ 还是没什么好搞的了。 下面来点较小的 $m$, $n$ 来试试消元看看是什么方程。 我们让 $m$, $n$ 取最小,即 $m=1$, $n=2$,这就是最简单的情况,此时消去 $\tan\theta $ 后将得到一条二次曲线 $3x^2+2x-y^2-1=0$,也就是352文中的题目的情形,离心率恰好是 $2$。 当 $m=1$, $n=3$,消去 $\tan\theta $ 得到 $2x^3+3x^2-2xy^2-y^2-1$,已经是三次曲线了,不过还算简单,因为 $y$ 都是二次,可以直接写成函数形式,注意限制,最终可以得出此时点 $M$ 的轨迹为\[y=\pm(x+1)\sqrt{\frac{2x-1}{2x+1}} \quad x>\frac12.\]作个图来验证一下: 注意左边的那些曲线不用管它,因为已经限定了 $x>1/2$。这样我们足以整出一道题出来:给出上面这个函数及其上的动点,证明那个三倍角关系。:D 嘿嘿 当 $m=1$, $n=4$,消去 $\tan\theta $ 得到 $5x^4+12x^3-10x^2y^2+6x^2-12xy^2-4x+y^4-2y^2-3=0$,这就复杂一点,虽然也能用求根公式解出函数形式,但判断哪部分曲线才是真正的轨迹时还有点麻烦,具体过程就不写了,只给出结果为\[y=\pm\sqrt{(x+1)\bigl(5x+1-2\sqrt{5x^2+2x+1}\bigr)}\quad x>\frac35.\]作图验证: 当 $m=2$, $n=3$,消去 $\tan\theta $ 得到 $5x^4+4x^3-10x^2y^2-6x^2-4xy^2-4x+y^4+2y^2+1=0$,方程跟上一个居然非常地像!但是排除的时候比上面的要麻烦,结果为\[y=\pm\sqrt{5x^2+2x-1+2x\sqrt{x(5x+4)}}\quad x>\frac15.\]作图验证: 当 $m=1$, $n=5$,消去 $\tan\theta $ 得到 $3x^5+10x^4-10x^3y^2+10x^3-20x^2y^2+3xy^4-10xy^2-5x+2y^4-2=0$,$x$ 的次数上升到五次了,还好 $y$ 的次数还是跟上面一样,可以解出来,经排除,结果为\[y=\pm(x+1)\sqrt{\frac{5x-2\sqrt{4x^2+1}}{3x+2}}\quad x>\frac23.\]作图验证: 当 $m=2$, $n=5$,消去 $\tan\theta $ 得到 $7 x^6+18 x^5-35 x^4 y^2+5 x^4-60 x^3 y^2-20 x^3+21 x^2 y^4-10 x^2 y^2-15 x^2+18 x y^4+20 x y^2+2 x-y^6+y^4+5 y^2+3=0$ oh!六次来了,想解出 $y$?只有卡当公式了,没办法,既然写不成显函数那还是乖乖用回参数方程好了。 先扯到这吧……闪先
三下五除二 2# 2012-10-15 10:01
我也想过一般的情况,但是没想到用参数方程,衰了,从k的这些计算来看,似乎可以推测了,这个轨迹都是双曲线?
kuing 3# 2012-10-15 10:10
2# 三下五除二 ???你没看到后面那些消去 $\tan\theta$ 后都是高次方程么,肯定不是双曲线了啊,只是实际轨迹部分看上去像而已
thread-859-1-6.html: 在群里看到的“拉马努金的恒等式”
kuing 1# 2012-10-13 17:12
老实说我还没听过这个恒等式的名字
kuing 2# 2012-10-13 17:18
在网上搜了一下,第一个链接(http://zhidao.baidu.com/question/399527225.htm)里有这样的一个证明: 3=√(1+8) =√(1+2√(1+3*5)) =√(1+2√(1+3√(1+4*6))) =√(1+2√(1+3√(1+4√(1+5*7)))) =....以此类推=Ramanujan恒等式。 你懂的,亲~ 看上去这证明好像没什么问题而且挺有型,不过细想想,还是觉得有点……不知怎么说,总觉得还差点东西,主要就是最后的那个数的问题。 总之还是不太放心这个证明,决定还是自己证一下,既然也知道结果了,而且又是层层根号,不如就来个“无敌有理化”吧!
kuing 3# 2012-10-13 17:36
记 \begin{align*} a_1&=1, \\ a_2&=\sqrt{1+2}, \\ a_3&=\sqrt{1+2\sqrt{1+3}}, \\ a_4&=\sqrt{1+2\sqrt{1+3\sqrt{1+4}}}, \\ &\vdots \\ a_n&=\sqrt{1+2\sqrt{1+3\sqrt{1+4\sqrt{1+\cdots +(n-2)\sqrt{1+(n-1)\sqrt{1+n}}}}}}, \end{align*} 则原题就是求 $\lim_{n\to\infty}a_n$。 为有理化作准备,再记 \begin{align*} b_1&=a_n, \\ b_2&=\sqrt{1+3\sqrt{1+4\sqrt{1+\cdots +(n-2)\sqrt{1+(n-1)\sqrt{1+n}}}}}, \\ b_3&=\sqrt{1+4\sqrt{1+\cdots +(n-2)\sqrt{1+(n-1)\sqrt{1+n}}}}, \\ &\vdots\\ b_{n-2}&=\sqrt{1+(n-1)\sqrt{1+n}}, \\ b_{n-1}&=\sqrt{1+n}, \end{align*} 那么对于任意满足 $1\leqslant k\leqslant n-2$ 的正整数 $k$,有 \[b_k^2-(k+2)^2=1+(k+1)b_{k+1}-(k+2)^2=(k+1)\bigl(b_{k+1}-(k+3)\bigr)=(k+1)\cdot \frac{b_{k+1}^2-(k+3)^2}{b_{k+1}+k+3},\] 于是 \begin{align*} a_n-3&=\frac{b_1^2-3^2}{b_1+3}=2\cdot \frac{b_2^2-4^2}{(b_1+3)(b_2+4)}=2\cdot 3\cdot \frac{b_3^2-5^2}{(b_1+3)(b_2+4)(b_3+5)} \\ &=\cdots =(n-1)!\cdot \frac{b_{n-1}^2-(n+1)^2}{(b_1+3)(b_2+4)(b_3+5)\cdots (b_{n-1}+n+1)} \\ &=\frac{-(n+1)!}{(b_1+3)(b_2+4)(b_3+5)\cdots (b_{n-1}+n+1)}, \end{align*} 由于各 $b_i$ 都大于 $1$,所以 \begin{align*} \abs{a_n-3}&=\frac{(n+1)!}{(b_1+3)(b_2+4)(b_3+5)\cdots (b_{n-1}+n+1)} \\ &<\frac{(n+1)!}{(1+3)(1+4)(1+5)\cdots (1+n+1)} \\ &=\frac6{n+2}, \end{align*} 这样,当 $n\to\infty$ 时,就自然有 \[\lim_{n\to\infty}a_n=3.\] 中间的过程其实就是不断的有理化,将所有的根号去掉。只不过很不方便表达,所以才引用那些 $b$。
q85669551 4# 2012-10-13 20:53
记 \begin{align*} a_1&=1, \\ a_2&=\sqrt{1+2}, \\ a_3&=\sqrt{1+2\sqrt{1+3}}, \\ a_4&=\sqrt{1+2\sqrt{1+3\sqrt{1+4}}}, \\ &\vdots \\ a_n&=\sqrt{1+2\sqrt{1+3\sqrt{1+4\sqrt{1+\cdots +(n-2)\sqrt{1+(n ... kuing 发表于 2012-10-13 17:36 没看完  但看到令b1=an这步 感觉见过
kuing 5# 2012-10-13 20:57
4# q85669551 那个令纯粹为了方便后面的表达,不然直接有理化的话完全写不下……
q85669551 6# 2012-10-13 21:57
看懂是不费力。但主要是怎么构造bn来有理化,有点意思。
q85669551 7# 2012-10-13 22:00
好吧 我对有理化的理解太狭义。。仅限于 分子/分母有理化了
kuing 8# 2012-10-13 22:18
7# q85669551 现在也是分子有理化啊,不过要N步才到位,不断去根号,自然就会需要那些 b 来简化输入……
q85669551 9# 2012-10-13 23:14
我熟知的分子有理化是例如  Sqrt[x^2+1]-x=1/(Sqrt[x^2+1]+x)这种。。。
thread-86-1-9.html: [函数] 2011天津高考19题(solved)谢谢
wenshengli 1# 2011-10-11 21:43
本帖最后由 wenshengli 于 2011-10-11 22:04 编辑 请教着色部分怎么来的?
kuing 2# 2011-10-11 22:02
右边的图形不就是解释咯。。。
zwl1972 3# 2011-10-12 16:22
形象的表达一下就是使用了"隐身术"
thread-860-1-5.html: [不等式] 来自超级群的要求纯代数解的求最值
kuing 1# 2012-10-13 22:20
爱好者-v6/huaix(4744*****)  21:17:14 爱好者-v6/huaix(4744*****)  21:17:19 做题吧 爱好者-v6/huaix(4744*****)  21:17:39 这个纯代数的解 爱好者-v6/huaix(4744*****)  21:18:24 不要正切换元 题目:已知 $a$, $b$, $c$ 是正实数,且 $abc+a+c=b$。设 \[p=\frac2{a^2+1}-\frac2{b^2+1}+\frac3{c^2+1},\] 则 $p$ 的最大值为______。
kuing 2# 2012-10-13 22:45
由已知等式可设 $b=a+c+t$, $t>0$,则 $ac(a+c+t)=t$,那么 \begin{align*} p&=\frac2{a^2+1}-\frac2{b^2+1}+\frac3{c^2+1} \\ &=\frac2{\frac{ta}{c(a+c+t)}+1}-\frac2{\frac{t(a+c+t)}{ac}+1}+\frac3{\frac{tc}{a(a+c+t)}+1} \\ &=\frac{2c(a+c+t)}{(a+c)(c+t)}-\frac{2ac}{(a+t)(c+t)}+\frac{3a(a+c+t)}{(a+c)(a+t)} \\ &=\frac{3(a+c)(a+t)(c+t)+ct(4a-c-t)}{(a+c)(a+t)(c+t)}, \end{align*} (1)若 $4a-c-t<0$,则 $p<3$; (2)若 $4a-c-t\geqslant 0$,记 \[f(a,c,t)=\frac{3(a+c)(a+t)(c+t)+ct(4a-c-t)}{(a+c)(a+t)(c+t)},\] 则 \[f\left( a,\frac{c+t}2,\frac{c+t}2 \right)-f(a,c,t)=\frac{a(4a-c-t)(a+c+t)(c-t)^2}{(a+c)(a+t)(c+t)(2a+c+t)^2}\geqslant 0,\] 令 $(c+t)/2=ua$, $u>0$,则 \[p\leqslant f(a,ua,ua)=\frac{2u^2+8u+3}{(u+1)^2}=\frac{10}3-3\left( \frac1{u+1}-\frac23 \right)^2\leqslant \frac{10}3,\] 当 $u=1/2$ 时取等号。 综上所述,$p$ 的最大值为 $10/3$,等号成立当且仅当 $a=2c=2t$ 即 $b=2a=4c=\sqrt2$。
kuing 3# 2012-10-13 22:50
如果已经知道或猜到了最大值,也就是只要证明不等式时,作差配方更方便。 \[\frac{10}3-\frac{3(a+c)(a+t)(c+t)+ct(4a-c-t)}{(a+c)(a+t)(c+t)}=\frac{t(a-2c)^2+c(a-2t)^2+a(c-t)^2}{3(a+c)(a+t)(c+t)}\geqslant0.\] 其实我也有点意外的是,齐次后居然会有半对称性。
yes94 4# 2013-1-23 21:45
越南1999MO
pengcheng1130 5# 2013-1-23 22:03
此题,请参考http://blog.sina.com.cn/s/blog_4c1131020101422r.html
yes94 6# 2013-1-23 22:16
5# pengcheng1130 只不过我不是在你的那个博客看见的,宋老师的博客很久没去过了。你的名字的中间再加一个字就是…… 我说的那个证法,和张云华的比较相同(相同度达到90%),只是用柯西不等式而已,而他没有用柯西不等式。
kuing 7# 2013-1-24 10:15
还好,我的证法跟他们没重复(至少看上去没)。
kuing 8# 2013-1-24 12:25
anyway,多谢提供链接
yes94 9# 2013-1-24 15:50
还好,我的证法跟他们没重复(至少看上去没)。 kuing 发表于 2013-1-24 10:15 的确$k$版的证法很独特 ,先猜到去等号条件。再来调整证明,这是$k$版的独门绝技,怎么可能重复? 事实上重复也是很可能的,一种解法,未必然就只有某一个人所独有? 莱比尼兹和牛顿各自独立创立微积分就是这个道理。
kuing 10# 2013-1-24 16:01
的确$k$版的证法很独特 ,先猜到去等号条件。再来调整证明,这是$k$版的独门绝技,怎么可能重复? 事实上重复也是很可能的,一种解法,未必然就只有某一个人所独有? 莱比尼兹和牛顿各自独立创立微积分就是这个道 ... yes94 发表于 2013-1-24 15:50 我也没说不可能重复,也没说只有我才能想到,更不是什么独门绝技,其实取等条件是在我通分化简时才发现的,可以说是意外结果,说不上“先猜”。 看不到重复而觉得欣喜,其实也只是心安点,至少不用太担心会突然跳出一个人来说我抄他,我最怕这样了。
yes94 11# 2013-1-24 16:31
10# kuing ,我们都相信$k$版的实力,的确$k$版的很多种做法视角很新引,他人真的难以复制,所以我才说独门绝学, ,这恐怕不是我一个人的感觉吧。
thread-862-1-6.html: [不等式] 最近貌似挺火的一题
pxchg1200 1# 2012-10-15 22:50
本帖最后由 pxchg1200 于 2012-10-23 19:23 编辑 Let$a,b$  and $c$ be positive real numbers with$abc=1$ . Prove that \[ \sqrt{ 9+16a^{2}}+\sqrt{ 9+16b^{2}}+\sqrt{ 9+16c^{2}}\ge 3+4(a+b+c) \] still alive although nearly die....
kuing 2# 2012-10-16 00:42
挺火?怎么我木见过 最后那句啥意思看不懂求翻译……
都市侠影 3# 2012-10-17 12:48
你把条件打漏了吧,目测条件应该是 $a+b+c=3$
kuing 4# 2012-10-21 22:28
话说既然火,那应该有各种链接吧楼主能不能提供一些参考链接我学习学习
kuing 5# 2012-10-23 16:53
你把条件打漏了吧,目测条件应该是 $a+b+c=3$ 都市侠影 发表于 2012-10-17 12:48 应该不会是 $a+b+c=3$,因为不等式右边就有 $a+b+c$,如果是 $a+b+c=3$ 的话右边应干脆写成 15。
pxchg1200 6# 2012-10-23 19:30
proof: Since \[ \sqrt{9+16a^{2}}-4a=\frac{9}{4a+\sqrt{9+16a^{2}}} \] Using AM-GM inequality,we have \[ \sqrt{9+16a^{2}}\leq \frac{1}{2}\left(\frac{16a^{2}+9}{2a+3}+2a+3 \right) \] Hence \[ \frac{9}{4a+\sqrt{9+16a^{2}}}\geq \frac{2a+3}{2a^{2}+2a+1} \] Using the know result of \[ \boxed{\frac{1}{x^2+x+1}+\frac{1}{y^2+y+1}+\frac{1}{z^2+z+1}\geq 1} \] actually,we have \[ \frac{2a+3}{2a^{2}+2a+1}\geq \frac{3}{a^{\frac{8}{5}}+a^{\frac{4}{5}}+1} \] which can be checked by AM-GM the result follows. Done!
kuing 7# 2012-10-23 19:41
6# pxchg1200 咦,看了这个才想起你前段时间不是在群里发过一个can的类似的一个过程,不知是不是这个题?具体没记起
thread-863-1-6.html: [不等式] 神奇的条件ab+bc+ca = abc+2
pxchg1200 1# 2012-10-15 22:56
For $a,b,c>0$ with $ab+bc+ca=abc+2$ prove that: \[ a^{2}+b^{2}+c^{2}+abc(a+b+c-2)\ge 4 \] (proposed by mudok)
kuing 2# 2012-10-16 02:35
没思路,除了uvw……
pxchg1200 3# 2012-10-16 19:29
2# kuing 只要证明:$a+b+c\geq 3 $
kuing 4# 2012-10-16 20:22
2# kuing 只要证明:$a+b+c\geq 3 $ pxchg1200 发表于 2012-10-16 19:29 这个不成立吧,比如 $a\to0$, $b=c\to\sqrt2$
pxchg1200 5# 2012-10-16 22:32
4# kuing Oh,Schur 放缩过头了。。。
goft 6# 2012-10-18 23:47
对其中的一个字母分三类讨论即可:a>1,a=1,a<1
kuing 7# 2012-10-19 01:08
6# goft 这个提示好简洁……
都市侠影 8# 2012-10-20 18:39
7# kuing 估计这题也是个“非暴力不合作”的..........
kuing 9# 2012-10-21 19:33
证出来没有啊 px
pxchg1200 10# 2012-10-21 19:37
本帖最后由 pxchg1200 于 2012-10-21 19:44 编辑 9# kuing 估计要分类讨论$a+b+c\geq 3$ 或$a+b+c<3 $ 我的想法: $4=2(ab+bc+ca)-2abc $ 所以只要证明 $ a^2+b^2+c^2+abc(a+b+c)\geq 2(ab+bc+ca) $ \[ abc(a+b+c)\geq 2(ab+bc+ca)-(a^2+b^2+c^2) \] 若 $a+b+c\geq 3$ 则由Schur inequality \[ abc(a+b+c)\geq \geq \frac{abc}{a+b+c}\geq 2(ab+bc+ca)-(a^2+b^2+c^2) \] 若 $a+b+c<3 $ ....... 这里不会弄了..
pxchg1200 11# 2012-10-21 20:49
WLOG assume $(a-1)(b-1)\geq 0\implies ab+1\geq a+b;$ and $c+ab\ge2.$ Now note that we have \[\begin{aligned}a^2+b^2+c^2+abc(a+b+c-2)&=a^2+b^2+c^2(1+ab)+abc(a+b)-2abc\\&\geq a^2+b^2+c(c+ab)(a+b)-2abc\\&\geq a^2+b^2+2(ca+bc)-2abc\\&\geq a^2+b^2+2(ca+bc-abc)\\&=a^2+b^2+2(2-ab)\\&=4+(a-b)^2\geq 4.\end{aligned}\] Equality holds iff $a=b=c=1.\Box$ :) Potla 秒杀了。。。。
kuing 12# 2012-10-21 20:54
11# pxchg1200 nice! 其实设 $(a-1)(b-1)\geqslant0$ 这招我也考虑过(以前有类似的题也用过),但是没用成功还是太菜了…… 后来是想换元去了,也没换出来……
thread-864-1-6.html: [不等式] 3元4次?!pqr?!
pxchg1200 1# 2012-10-15 22:59
Let $a,b$ and$c$  be three non-negative real numbers .prove that \[ (a^{2}+b^{2}+c^{2})^{2}\ge 3abc(a+b+c)+4(a+b+c)(a-b)(b-c)(c-a) \]
kuing 2# 2012-10-16 00:53
大概就是只要证 $a\leqslant b\leqslant c$,作差分代换后方知只要证 $a=0$,然后……
pxchg1200 3# 2012-10-21 19:45
2# kuing 弄出来了么?kk
kuing 4# 2012-10-21 20:04
没去弄,用2#的方法如无意外下OK
thread-865-1-5.html: [不等式] 古老的难题
pxchg1200 1# 2012-10-15 23:07
Let $a,b,c\geq 0 $with $a+b+c=1, k=1-\frac{\sqrt{3}}{2}$ prove that: \[ \sqrt{a+k(b-c)^{2}}+\sqrt{b+k(c-a)^2}+\sqrt{c+k(a-b)^2}\leq \sqrt{3} \] (Phan Thanh Nam)
kuing 2# 2012-10-16 00:43
好像在哪里看过,或者是类似的…… 有空找找记录先
yes94 3# 2013-1-24 18:57
咋一看,以为是07女子来了呢
thread-866-1-1.html: 求积分
pxchg1200 1# 2012-10-15 23:09
\[ \int_{0}^{\frac{\pi}{2}}{\ln^{n}{\sin{x}}dx} \] Ps. $n=1,2 $ 能用变换和傅里叶级数做,能否推导出一般公式?
都市侠影 2# 2012-10-18 16:24
本帖最后由 都市侠影 于 2012-10-18 16:30 编辑 你尽找一些完全摸不着边际的问题............ 我给你出个题,你也研究研究: 求积分 \[ \int_0^{\frac{\pi}{2}}\ln{\sin(\cos(xe^x))}dx \]
pxchg1200 3# 2012-10-19 07:38
2# 都市侠影 你这个是随手写的吧,完全没头绪。。。
都市侠影 4# 2012-10-19 13:19
3# pxchg1200 哈哈哈,看见了吧,随便乱写一个,也不是那么容易整的,这就是数学.........
pxchg1200 5# 2012-12-22 09:43
找到答案了
thread-867-1-6.html: 来自群的三角形数个位数有理数
kuing 1# 2012-10-16 20:43
第 $n$ 堆的个数 $f(n)=n(n+1)/2$,要判断 $N$ 是不是有理数,只要看是否存在整数 $k$, 使 $f(n+k)=f(n)+10g(n)$ 恒成立,其中 $g(n)$ 恒为整数。计算得 \[f(n+k)-f(n)=\frac12k(2n+1+k),\] 于是取 $k=20$ 便得 $f(n+20)=f(n)+10(2n+21)$,所以 $N$ 是循环小数,即有理数。
isea 2# 2012-10-17 16:59
这题目有意思,解得k=20也很厉害
kuing 3# 2012-10-17 20:05
2# isea 如果不是从f(n+k)与f(n)的关系出发,而是直接列举找规律的话,那得列二十几个才能看出来……
thread-868-1-1.html: 卡住了,关于e
realnumber 1# 2012-10-16 21:01
本帖最后由 realnumber 于 2013-1-19 15:57 编辑 在证明\[1+\frac{n}{1}+\frac{n^2}{2!}+\frac{n^3}{3!}+...+\frac{n^n}{n!} \ge \frac{e^n}{2}\],n为正整数. 运用数学归纳法过程中 似乎只要证明$\frac{{n}^{n}}{{e}^{n-1}n!} \ge \frac{e-2}{2}$ 就可以了-- 或有没n!上下界近似公式,最好是关于e的指数形式的.
kuing 2# 2012-10-16 21:08
似乎只要证明$\frac{{n}^{n}}{{e}^{n-1}n!} \ge \frac{e-2}{2}$ 就可以了-- realnumber 发表于 2012-10-16 21:01 左边的极限好像是 0 喔 Stirling 公式 $n!\sim\sqrt{2\pi n}n^ne^{-n}$
kuing 3# 2012-10-16 22:37
你那个“只需证……”是怎么得出来的? 归纳法可能加强一下更好证
kuing 4# 2012-10-17 02:36
似乎右边分母那个2还不能减小了,我甚至怀疑 \[\lim_{n\to\infty}\frac{1+\sum_{k=1}^n\frac{n^k}{k!}}{e^n}=2\] 可惜高数不行 ——————2013-2-1更正—————— 右边应该是1/2,当时不知撞什么邪,给写错了……
realnumber 5# 2012-10-17 07:37
这样啊,那我开始就猜错了
yes94 6# 2013-1-24 18:50
左边的极限好像是 0 喔 Stirling 公式 $n!\sim\sqrt{2\pi n}n^ne^{-n}$ kuing 发表于 2012-10-16 21:08 回复时间和楼主的修改时间不一样,常常看不懂
kuing 7# 2013-1-24 19:55
6# yes94 1#应该只是将行内公式改成了行间公式,内容没变。
realnumber 8# 2013-1-27 10:42
似乎是高数的,要不移动到高数区?已经彻底放弃.
kuing 9# 2013-1-27 19:15
用软件画了下似乎有 \[1+\frac n1+\frac{n^2}{2!}+\frac{n^3}{3!}+\cdots+\frac{n^n}{n!}\geqslant\frac12\left(1+\frac1n\right)^{n(n+1/2)}\] 不知会不会容易证一点,不过估计也很难…… 至于移过那边……我暂时不想动……
kuing 10# 2013-1-30 22:49
移过来这边了。 \[1+\frac n1+\frac{n^2}{2!}+\frac{n^3}{3!}+\cdots+\frac{n^n}{n!}\geqslant\frac{e^n}2\] 这个刚才在人教群里看到了一个高等证法,由于我高数弱,看了好一会才看懂了。 感谢 Salvation!
kuing 11# 2013-1-30 23:27
上面过程有点简略(可能因为我弱才觉得),这里顺便补充一下关键几处的推导。 由分部积分有 \[ \int_0^x{t^ne^{-t}\rmd t}=-\int_0^x{t^n\rmd{e^{-t}}}=-t^ne^{-t}|_0^x+\int_0^x{e^{-t}\rmd{t^n}}=-x^ne^{-x}+n\int_0^x{t^{n-1}e^{-t}\rmd t}, \] 记 $\int_0^x{t^ne^{-t}\rmd t}=I_n$, $I_0=\int_0^x{e^{-t}\rmd t}=1-e^{-x}$,上式化为 \[ I_n=-x^ne^{-x}+nI_{n-1}, \] 同除 $n!$ 得 \[ \frac{I_n}{n!}=-\frac{x^n}{n!}e^{-x}+\frac{I_{n-1}}{(n-1)!}, \] 求和得 \[ \frac{I_n}{n!}=-e^{-x}\sum_{i=1}^n{\frac{x^i}{i!}}+I_0=1-e^{-x}s_n(x), \] 由 Gamma 函数的定义知 $n!=\int_0^{+\infty }{t^ne^{-t}\rmd t}$,故 \[ e^{-x}s_n(x)=1-\frac{I_n}{n!}=\frac1{n!}\left( \int_0^{+\infty }{t^ne^{-t}\rmd t}-\int_0^x{t^ne^{-t}\rmd t} \right)=\frac1{n!}\int_x^{+\infty }{t^ne^{-t}\rmd t}, \] 所以 \[ s_n(n)=\frac{e^n}{n!}\int_n^{+\infty }{t^ne^{-t}\rmd t}, \] 于是 \[ s_n(n)>\frac{e^n}2\iff 2\int_n^{+\infty }{t^ne^{-t}\rmd t}>n!=\int_0^{+\infty }{t^ne^{-t}\rmd t}\iff \int_n^{+\infty }{t^ne^{-t}\rmd t}>\int_0^n{t^ne^{-t}\rmd t}. \] 之后的就比较好懂了,最后说的 Taylor 展开也很简单,这里就不详说了。
kuing 12# 2013-2-1 21:30
似乎右边分母那个2还不能减小了,我甚至怀疑 \[\lim_{n\to\infty}\frac{1+\sum_{k=1}^n\frac{n^k}{k!}}{e^n}=2\] 可惜高数不行 kuing 发表于 2012-10-17 02:36 根据上述计算过程,以上猜测等价于 \[\lim_{n\to+\infty}\frac1{n!}\int_n^{+\infty}t^ne^{-t}\rmd t=2,\] 或者能不能将 $n$ 变成 $x\in\mbb R$,不限制在整数,也就是说有没有如下的 \[\lim_{x\to+\infty}\frac{\int_x^{+\infty}t^xe^{-t}\rmd t}{\int_0^{+\infty}t^xe^{-t}\rmd t}=2,\] 能不能证到?继续研究下…… —————————— 更正:右边应为1/2
pxchg1200 13# 2013-2-1 22:09
本帖最后由 pxchg1200 于 2013-2-1 22:11 编辑 10# kuing 我们有 \[\lim_{n\rightarrow\infty}{\frac{1+n+\frac{n^2}{2!}+\cdots+\frac{n^{n}}{n!}}{e^{n}}}=\frac{1}{2} \] 不废话,上图片。
kuing 14# 2013-2-1 22:12
噢,原来我上面都写错了,全将 1/2 写成了 2
pxchg1200 15# 2013-2-1 22:14
14# kuing 这个好像是概率论中的中心极限定理。。。
kuing 16# 2013-2-1 22:15
15# pxchg1200 噢,我看看能不能看懂……
kuing 17# 2013-2-1 22:23
顺便将群里说的转发过来
kuing 18# 2013-2-2 00:54
13# pxchg1200 大概是读懂了,不过细节方面可能还要再想想。 时间关系先闪了
thread-87-1-1.html: 标题也可以用LaTeX代码,比如 $a^x=\log_ax$
kuing 1# 2011-10-12 02:05
在本论坛中,贴子的标题也可以用 $\LaTeX$ 代码,所以我有一个小建议: 可以的话,在发贴时,将问题中关键的或者说有代表性的式子写到标题上,可以不必写完整的式子,简写一下即可(详写可能超字数,也不要搞太高的公式)。 这样做的目的是为了方便看贴和找贴,从外面一下就能看到,就像下面的截图里那样。
kuing 2# 2011-10-29 21:56
补充一句,为了保证点击标题能进入主题贴,请标题里留点非公式的东西,如果标题纯公式的话怕如果公式一时显示不出来就无法点击去了。
thread-870-1-2.html: [数列] 请教一个数列的通项公式的题目,先谢谢了!
hongxian 1# 2012-10-18 18:50
13.对于正整数$n$,最接近$\sqrt{n}$的正整数设为${{a}_{n}}$,又设${{b}_{n}}={{a}_{n}}+n$,从全体正整数中除去所有${{b}_{n}}$($n=$1,2,3,…),余下的正整数按从小到大的顺序排列得$\left\{ {{c}_{n}} \right\}$ 求:$\left\{ {{c}_{n}} \right\}$的通项公式;
kuing 2# 2012-10-18 19:04
这么麻烦……能不能直接用高斯函数或者Round函数之类的东东啊?
hongxian 3# 2012-10-18 19:19
2# kuing 好象和高斯函数没有什么关系
hongxian 4# 2012-10-18 19:20
列举了一下估计是$n^2$,不知怎么证明。
kuing 5# 2012-10-18 19:22
噢?看来有规可循……再瞧瞧先
kuing 6# 2012-10-18 19:58
先证明,$\{a_n\}$ 中,有 $2k$ 个 $k$($k=1$, $2$, $\ldots$) 由于 $\sqrt{n}$ 的小数部分不会是 $0.5$,所以只要考虑使 $k-0.5<\sqrt{n}<k+0.5$ 成立的 $n$ 的个数。 两边平方得 $k^2-k+0.25<n<k^2+k+0.25$,所以 $k^2-k+1\leqslant n \leqslant k^2+k$,共 $2k$ 个。 这也就是说,…… 想是想通了,不好表达……待续
kuing 7# 2012-10-18 20:14
这也就是说,$\{b_n\}$ 中,连续 $2k$ 个顺次整数之后跳一下(+2)然后再接着 $2k+2$ 个顺次整数,…… 注意到$(k+1)^2-k^2=2k+1$,所以相邻两个平方数 $(k+1)^2$ 与 $k^2$ 之间有 $2k$ 个整数,因此不难看出跳过的正好是平方数,所以…… 超越我的表达能力了,先说成这样,大概理解下希望能懂我的意思吧……
hongxian 8# 2012-10-18 22:39
本帖最后由 hongxian 于 2012-10-18 22:41 编辑 好象有点明白了,$k^2-k+1 \le n \le k^2+k$,可得$k^2+1 \le n+k=b_n \le k^2+2k$ 所以$b_n$可以取遍$k^2+1$到$ k^2+2k$之间的所有整数。
q85669551 9# 2012-10-18 23:05
大概想了下...
dualliot 10# 2013-4-9 08:20
$b_n$单调递增没问题,然后$b_n$遍历了除了完全平方数之外的所有数.
thread-876-1-6.html: 终于忍不住,还在把这个题转过来,不知有什么规律?
hongxian 1# 2012-10-19 20:57
Book1.rar (8.94 KB) 定义数列{f(n)}如下: f(0)=f(1)=0 f(n)=n-1-f[f(n-1)+1],n∈N      注: (这里中括号就是中括号,没有特殊含义) 求数列{f(n)}的通项公式。 列了前100项,没有发现什么规律,不知有没有规律。 把我列的也传上来。
kuing 2# 2012-10-19 21:05
495 定义数列{f(n)}如下: f(0)=f(1)=0 f(n)=n-1-f[f(n-1)+1],n∈N      注: (这里中括号就是中括号,没有特殊含义) 求数列{f(n)}的通项公式。 列了前100项,没有发现什么规律,不知有没有规律。 ... hongxian 发表于 2012-10-19 20:57 先扯两个题外: 1、既然需要说明“中括号就是中括号”,何不干脆不用中括号,全用小括号就行了。这种情况我一般都全用小括号,一来避免误解,二来不必担心括号层数太多。 2、那个是不是要offes2007才能打开啊?我这里打不开……
hongxian 3# 2012-10-19 21:09
2# kuing 题目不是我的,是昨天哪个题引出来的YES94的题 excel重新再传一个2003可以打开的。
kuing 4# 2012-10-19 21:11
我用 mathematica 列前100项的结果是: 0, 1, 1, 2, 3, 3, 4, 4, 5, 6, 6, 7, 8, 8, 9, 9, 10, 11, 11, 12, 12, 13, 14, 14, 15, 16, 16, 17, 17, 18, 19, 19, 20, 21, 21, 22, 22, 23, 24, 24, 25, 25, 26, 27, 27, 28, 29, 29, 30, 30, 31, 32, 32, 33, 33, 34, 35, 35, 36, 37, 37, 38, 38, 39, 40, 40, 41, 42, 42, 43, 43, 44, 45, 45, 46, 46, 47, 48, 48, 49, 50, 50, 51, 51, 52, 53, 53, 54, 55, 55, 56, 56, 57, 58, 58, 59, 59, 60, 61, 61
kuing 5# 2012-10-19 21:13
3# hongxian 噢; 嗯,这个可以打开了,跟我上面列的一样(我没列第0项和第101项)
joatbmon 6# 2012-10-19 21:20
本帖最后由 joatbmon 于 2012-10-19 21:26 编辑 光看数据,我猜测跟黄金分割有关系,n和f(n)的比似乎越来越接近黄金分割 我用excel算了一下,应该是f(n)=trunc(n*黄金分割) 经过excel验证,前100数据符合楼主给的数据 然后,证明,我不会 如果我没有记错的话,斐波那契数列跟黄金分割有关的,也许跟这个数列有联系,等待牛人
kuing 7# 2012-10-19 21:27
光看数据,我猜测跟黄金分割有关系,n和f(n)的比似乎越来越接近黄金分割 我用excel算了一下,应该是f(n)=trunc(n*黄金分割) 经过excel验证,前100数据符合楼主给的数据 然后,证明,我不会 如果我没有记错的话,斐波那契数列跟黄金分割有关的,也许跟这个数列有联系,等待牛人 joatbmon 发表于 2012-10-19 21:20 的确如此,好观察力 前100项数据都符合 $\left[n\cdot\dfrac{\sqrt5-1}2\right]$ 这里的中括号就是高斯函数了,下同。
kuing 8# 2012-10-19 21:45
\[\left[ n\cdot \frac{\sqrt{5}-1}{2} \right]=n-1-\left[ \left( \left[ (n-1)\cdot \frac{\sqrt{5}-1}{2} \right]+1 \right)\cdot \frac{\sqrt{5}-1}{2} \right]\] 这条等式验证了1~10000都成立……
kuing 9# 2012-10-19 23:03
\[\left[ n\cdot \frac{\sqrt{5}-1}{2} \right]=n-1-\left[ \left( \left[ (n-1)\cdot \frac{\sqrt{5}-1}{2} \right]+1 \right)\cdot \frac{\sqrt{5}-1}{2} \right]\] 这条等式验证了1~10000都成立…… kuing 发表于 2012-10-19 21:45 证到了,原来也不是很难。 首先,为方便书写,记黄金分割数为 $\varphi =\bigl(\sqrt{5}-1\bigr)/2$,则 $1/\varphi =\varphi +1$,待证的等式为 \begin{equation}\label{20121019czhjfghds} [n\varphi ]=n-1-\bigl[ \bigl( [(n-1)\varphi ]+1 \bigr)\varphi \bigr]. \end{equation} 设 $k$ 为任意整数,我们来求一下使 $[n\varphi ]=k$ 成立的所有整数 $n$。由于 $n\varphi $ 不会是整数,所以 \begin{align*} [ n\varphi ]=k&\iff k<n\varphi <k+1 \\ & \iff k(\varphi +1)<n<(k+1)(\varphi +1) \\ & \iff [k\varphi ]+k+1\leqslant n\leqslant [(k+1)\varphi ]+k+1, \end{align*} 由于 $0<\varphi <1$,所以,要么 $[(k+1)\varphi ]=[k\varphi ]$,要么 $[(k+1)\varphi ]=[k\varphi ]+1$,因此下面分两类讨论。 (1)当 $[(k+1)\varphi ]=[k\varphi ]$ 时,则 $[n\varphi ]=k$ 的解就只有 $n=[k\varphi ]+k+1$,且 $[(n-1)\varphi ]=k-1$,于是 \[n-1-\bigl[ \bigl( [(n-1)\varphi ]+1 \bigr)\varphi \bigr]=[k\varphi ]+k+1-1-[(k-1+1)\varphi ]=k,\] 所以此时式 \eqref{20121019czhjfghds} 成立; (2)当 $[(k+1)\varphi ]=[k\varphi ]+1$ 时,则 $[n\varphi ]=k$ 的解就是 $n=[k\varphi ]+k+1$ 和 $n=[k\varphi ]+k+2$,因此下面再分两小类讨论。     (2-1)当 $n=[k\varphi ]+k+1$ 时,与情况(1)相同,式 \eqref{20121019czhjfghds} 成立;     (2-2)当 $n=[k\varphi ]+k+2$ 时,则 $[(n-1)\varphi ]=[n\varphi ]=k$,于是 \begin{align*} n-1-\bigl[ \bigl( [(n-1)\varphi ]+1 \bigr)\varphi \bigr]&=[k\varphi ]+k+2-1-[(k+1)\varphi] \\ & =[k\varphi ]+k+1-([k\varphi ]+1) \\ & =k, \end{align*} 所以此时式 \eqref{20121019czhjfghds} 也成立。 综上所述,由 $k$ 的任意性,可知式 \eqref{20121019czhjfghds} 对任意整数都成立,得证。
kuing 10# 2012-10-19 23:38
$f(0)=f(1)=0$, $f(n)=n-1-f\bigl(f(n-1)+1\bigr)$, $n\in \mbb N$,求 $f(n)$。 下面利用式 \eqref{20121019czhjfghds} 及第二数学归纳法来证明 $f(n)=[n\varphi ]$, $n\in \mbb N$。 当 $n=0$, $1$ 时显然成立,设 $k$ 正整数,假设 $0\leqslant n\leqslant k$ 时有 $f(n)=[n\varphi ]$,则当 $n=k+1$ 时 \[f(k+1)=k-f\bigl(f(k)+1\bigr)=k-f([k\varphi ]+1),\] 由楼上的证明过程不难看出有 $0\leqslant [k\varphi ]+1\leqslant k$ 成立,由此结合式 \eqref{20121019czhjfghds} 即得 \[k-f([k\varphi ]+1)=k-\bigl[([k\varphi ]+1)\varphi \bigr]=k+1-1-\bigl[\bigl([(k+1-1)\varphi ]+1\bigr)\varphi \bigr]=[(k+1)\varphi ],\] 所以 \[f(k+1)=[(k+1)\varphi ],\] 故由第二数学归纳法知 $f(n)=[n\varphi ]$, $n\in \mbb N$。
kuing 11# 2012-10-20 00:16
其实一开始看着 8# 的等式也有点胆怯,没想到细想了下就意外顺利解决了……最近做题运气不错啊
叶剑飞Victor 12# 2012-10-20 01:53
本帖最后由 叶剑飞Victor 于 2012-10-20 02:38 编辑 这是我算出来的前9999项 前9999项.zip (33.27 KB) 程序如下,用C语言写的: #include <stdio.h> #include <stdlib.h> #define MAX_LENGTH 10000 int f[MAX_LENGTH]; int main ( void ) {         int i;         char file_name[32];         FILE * fp;         sprintf( file_name, "前%d项.csv", MAX_LENGTH - 1 );         fp = (FILE *)fopen( file_name, "w" );         if ( fp == NULL )                 fp = stdout;         f[0] = f[1] = 0;         fprintf( fp, "n,f(n)\n", 0, f[0] );         fprintf( fp, "%d,%d\n", 0, f[0] );         fprintf( fp, "%d,%d\n", 1, f[1] );         for ( i = 2 ; i < MAX_LENGTH ; i ++ )         {                 f[i] = i - 1 - f[f[i - 1]+1];                 fprintf( fp, "%d,%d\n", i, f[i] );         }         fclose( fp );         return EXIT_SUCCESS; } 复制代码
叶剑飞Victor 13# 2012-10-20 02:02
本帖最后由 叶剑飞Victor 于 2012-10-20 02:42 编辑 前99999项 前99999项.zip (330.99 KB) 其实就是把“#define MAX_LENGTH 10000”改成了“#define MAX_LENGTH 100000”而已,多加了一个“0”而已。
hongxian 14# 2012-10-20 12:10
9# kuing 还有个地方没看明白, 为什么$n=\left[ k\varphi  \right]+k+1$时,$\left[ (n-1)\varphi  \right]=k-1$ , 而$n=\left[ k\varphi  \right]+k+2$时,$\left[ (n-1)\varphi  \right]=k$ ?
kuing 15# 2012-10-20 12:46
9# kuing 还有个地方没看明白, 为什么$n=\left[ k\varphi  \right]+k+1$时,$\left[ (n-1)\varphi  \right]=k-1$ , 而$n=\left[ k\varphi  \right]+k+2$时,$\left[ (n-1)\varphi  \right]=k$ ? hongxian 发表于 2012-10-20 12:10 $n=[k\varphi]+k+1$ 是 $[n\varphi]=k$ 的最小的解,所以 $[(n-1)\varphi]$ 就肯定变成 $k-1$ 了。 而当 $[n\varphi]=k$ 有两个解时 $n=[k\varphi]+k+2$ 就是较大的解,所以 $[(n-1)\varphi]$ 仍然是 $k$
都市侠影 16# 2012-10-20 18:37
10# kuing K 版威武,这些题看来也很顽固,真是“非暴力不合作”啊
kuing 17# 2012-10-20 20:43
16# 都市侠影 呵呵嗯,除了归纳猜想,暂时想不到别的证法了……
thread-878-1-6.html: [组合] 收集组合恒等式
kuing 1# 2012-10-21 17:44
刚才在超级群看到有人提到“组合恒等式表”,我手头上没有这类资料,不如就在这里收集一些吧 群里看到的题是: (1)$C_{2n}^0+C_{2n-1}^1+C_{2n-2}^2+\cdots+C_n^n=?$ 右边待填,暂无答案。 再列几个以前看到过的: (2)$C_n^1+2C_n^2+\cdots+nC_n^n=n2^{n-1}$ (3)$(C_n^1)^2+(C_n^2)^2+\cdots+(C_n^n)^2=C_{2n}^n$ (3’)$C_m^0C_n^k+C_m^1C_n^{k-1}+\cdots+C_m^kC_n^0=C_{m+n}^k$ (4)$\displaystyle 1-\frac12C_n^1+\frac13C_n^2+\cdots+(-1)^n\frac1{n+1}C_n^n=\frac1{n+1}$ (5)$\displaystyle C_n^1-\frac{C_n^2}2+\cdots+\frac{(-1)^{n-1}C_n^n}n=1+\frac12+\cdots+\frac1n$ 到你们了
Tesla35 2# 2012-10-25 11:49
K版,等哪天有空我把史济怀那本组合恒等式的附录编辑成latex吧。。
kuing 3# 2012-10-25 15:00
2# Tesla35 直接在这里编辑也行
Tesla35 4# 2012-10-25 17:52
本帖最后由 Tesla35 于 2012-10-25 17:54 编辑 今天先写前十个 $\displaystyle C_n^k=C_n^{n-k}$ $\displaystyle C_n^k=C_{n-1}^k+C_{n-1}^{k-1}$ $\displaystyle C_n^k=\frac{n}{k}C_{n-1}^{k-1}$ $\displaystyle C_n^kC_k^m=C_n^mC_{n-m}^{k-m}=C_n^{k-m}C_{n-k+m}^m(m\leqslant k\leqslant n)$ $\displaystyle \sum_{k=0}^{n}C_n^k=2^n$ $\displaystyle \sum_{k=0}^{n}(-1)^kC_n^k=0$ $\displaystyle \sum_{k=0}^{m}(-1)^kC_n^k=(-1)^mC_{n-1}^m(m<n)$ $\displaystyle \sum_{k=0}^{n}C_{2n}^k=2^{n-1}+\frac{1}{2}C_{2n}^n$ $\displaystyle \sum_{k=0}^{n}\frac{1}{k+1}C_n^k=\frac{1}{n+1}(2^{n+1}-1)$ $\displaystyle \sum_{k=0}^{n}(-1)^k\frac{1}{k+1}C_n^k=\frac{1}{n+1}$ 擦用不了numerate么。。
kuing 5# 2012-10-25 18:21
4# Tesla35 oh,的确用不了,既然如此,还是在真 latex 中写吧
Tesla35 6# 2012-10-26 00:16
前41个恒等式 \item $\displaystyle C_n^k=C_n^{n-k}$ \item $\displaystyle C_n^k=C_{n-1}^k+C_{n-1}^{k-1}$ \item $\displaystyle C_n^k=\frac{n}{k}C_{n-1}^{k-1}$ \item $\displaystyle C_n^kC_k^m=C_n^mC_{n-m}^{k-m}=C_n^{k-m}C_{n-k+m}^m(m\leqslant k\leqslant n)$ \item $\displaystyle \sum_{k=0}^{n}C_n^k=2^n$ \item $\displaystyle \sum_{k=0}^{n}(-1)^kC_n^k=0$ \item $\displaystyle \sum_{k=0}^{m}(-1)^kC_n^k=(-1)^mC_{n-1}^m(m<n)$ \item $\displaystyle \sum_{k=0}^{n}C_{2n}^k=2^{n-1}+\frac{1}{2}C_{2n}^n$ \item $\displaystyle \sum_{k=0}^{n}\frac{1}{k+1}C_n^k=\frac{1}{n+1}(2^{n+1}-1)$ \item $\displaystyle \sum_{k=0}^{n}(-1)^k\frac{1}{k+1}C_n^k=\frac{1}{n+1}$ %10 \item $\displaystyle \sum_{k=1}^{n}k^2C_n^k=n(n+1)2^{n-2}$ \item $\displaystyle \sum_{k=0}^{n}(-1)^kC_n^k\frac{m}{m+k}=(C_{m+n}^n)^{-1}$ \item $\displaystyle \sum_{k=1}^{n}(-1)^{k+1}\frac{1}{k}C_n^k=1+\frac{1}{2}+\cdots+\frac{1}{n}$ \item $\displaystyle \sum_{k=0}^{n}(-1)^k2^{2n-2k}C_{2n-k+1}^k=n+1$ \item $\displaystyle \sum_{k=0}^{n}(-1)^k2^{2n-2k}C_{2n-k}^k=2n+1$ \item $\displaystyle \sum_{k=0}^{n}2^kC_n^k=3^n$ \item $\displaystyle \sum_{k=0}^{n}kC_n^k=n2^{n-1}$ \item $\displaystyle \sum_{k=0}^{n}(k+1)C_n^k=(n+2)2^{n-1}$ \item $\displaystyle \sum_{k=0}^{n}(-1)^kkC_n^k=0(n>1)$ \item $\displaystyle \sum_{k=0}^{n}\frac{1}{k+1}C_n^kx^k=\frac{(1+x)^{n+1}-1}{(n+1)x}$%20 \item $\displaystyle \sum_{k=0}^{n}\frac{(-1)^k}{(k+1)^2}C_n^k=\frac{1}{n+1}\left(1+\frac{1}{2}+\cdots+\frac{1}{n+1}\right)$ \item $\displaystyle \sum_{k=0}^{n-1}C_{2n-1}^k=2^{2n-2}$ \item $\displaystyle \sum_{k=0}^{n}kC_{2n}^k=n2^{2n-1}$ \item $\displaystyle \sum_{k=1}^{n}(-1)^{k+1}\frac{1}{k}C_n^k[1-(1-x)^k]=\sum_{k=1}^{n}\frac{x^k}{k}$ \item $\displaystyle \sum_{k=0}^{n}\frac{(-1)^k}{2k+1}C_n^k=\frac{2^{2n}}{2n+1}(C_{2n}^n)^{-1}$ \item $\displaystyle \sum_{k=0}^{n}\frac{(-1)^k}{1-2k}C_n^k=2^{2n}(C_{2n}^n)^{-1}$ \item $\displaystyle \sum_{k=0}^{n}kC_{2n+1}^k=(2n+1)2^{2n-1}-\frac{2n+1}{2}C_{2n}^n$ \item $\displaystyle \sum_{k=0}^{n}kC_{2n}^{n-k}=nC_{2n-1}^{n}$ \item $\displaystyle \sum_{k=0}^{n}kC_{2n+1}^{n-k}=(2n+1)C_{2n-1}^{n}-2^{2n-1}$ \item $\displaystyle \sum_{k=0}^{n}C_{2n+1}^{2k+1}x^k=2x^{-\frac{1}{2}}[(1+\sqrt{x})^{2n+1}-(1-\sqrt{x})^{2n+1}]$ \item $\displaystyle \sum_{k=0}^{[\frac{n}{2}]}C_{n-k}^k= \frac{1}{\sqrt{5}}\left\{\left(\frac{1+\sqrt{5}}{2}\right)^{n+1}-\left(\frac{1-\sqrt{5}}{2}\right)^{n+1}\right\}$ \item $\displaystyle \sum_{k=0}^{[\frac{n}{2}]}C_{n-k}^kr^k= \frac{1}{2^{n+1}\sqrt{1+4r}}\left\{(1+\sqrt{1+4r})^{n+1}-(1-\sqrt{1+4r})^{n+1}\right\}$ \item $\displaystyle \sum_{k=0}^{[\frac{n}{2}]}C_{n-k}^k2^k=\frac{1}{3}(2^{n+1}+(-1)^n)$ \item $\displaystyle \sum_{k=0}^{[\frac{n}{2}]}C_{n-k}^k6^k=\frac{1}{5}(3^{n+1}+(-1)^n2^{n+1})$ \item $\displaystyle \sum_{k=0}^{[\frac{n}{2}]}(-1)^kC_{n-k}^k=\frac{2}{\sqrt{3}}\sin\frac{n+1}{3}\pi$ \item $\displaystyle \sum_{k=0}^{n-1}C_{n+k}^{2k+1}= \frac{1}{\sqrt{5}}\left\{\left(\frac{3+\sqrt{5}}{2}\right)^{n}-\left(\frac{3-\sqrt{5}}{2}\right)^{n}\right\}$ \item $\displaystyle \sum_{k=0}^{n}C_{n+k}^{2k}= \frac{1}{\sqrt{5}}\left\{\left(\frac{1+\sqrt{5}}{2}\right)^{2n+1}-\left(\frac{1-\sqrt{5}}{2}\right)^{2n+1}\right\}$ \item $\displaystyle \sum_{k=0}^{m}C_{k+n-1}^{n-1}=C_{m+n}^{n}$ \item $\displaystyle \sum_{k=0}^{n}2^{n-k}C_{n+k}^{2k}=\frac{1}{3}(2^{2n+1}+1)$ \item $\displaystyle \sum_{k=0}^{n}(-1)^kC_n^k\left(1+\frac{1}{2}+\cdots+\frac{1}{k}\right)=-\frac{1}{n}$ \item $\displaystyle \sum_{k=0}^{n}(-1)^kC_n^k\left(x+\frac{x^2}{2}+\cdots+\frac{x^k}{k}\right)=-\frac{1}{n}[1-(1-x)^n]$
kuing 7# 2012-10-26 00:19
这么叉多……果然够表……
Tesla35 8# 2012-10-26 12:23
第41~60个 \item $\displaystyle \sum_{k=0}^{n}(-1)^kC_n^k\left(x+\frac{x^2}{2}+\cdots+\frac{x^k}{k}\right)=-\frac{1}{n}[1-(1-x)^n]$ \item $\displaystyle \sum_{k=0}^{n}C_{n+p}^{k+p}x^k=\sum_{k=0}^{n}C_{p+k-1}^n(1+x)^{n-k}$ \item $\displaystyle \sum_{k=0}^{n}(-1)^kC_{n+p}^{k+p}=C_{n+p-1}^n$ \item $\displaystyle \sum_{k=0}^{[\frac{n}{2}]}(-1)^k\frac{n}{n-k}C_{n-k}^kx^{n-2k}= \frac{1}{2^n}\left\{(x+\sqrt{x^2-4})^n+(x-\sqrt{x^2-4})^n\right\}$ \item $\displaystyle \sum_{k=0}^{[\frac{n}{2}]}(-1)^k\frac{n}{n-k}C_{n-k}^k2^{n-2k}=2$ \item $\displaystyle \sum_{k=0}^{[\frac{n}{2}]}(-1)^k\frac{n}{n-k}C_{n-k}^k=2\cos\frac{n\pi}{3},n=1,2,\cdots.$ \item $\displaystyle \sum_{k=0}^{[\frac{n}{2}]}C_n^k\cos\frac{(n-2k)}{3}\pi =\left\{ {\begin{array}{*{20}{c}} {\displaystyle\frac{1}{2},n\text{为奇数}}\\ {\displaystyle\frac{1}{2}(1+C_n^{\frac{n}{2}}),n\text{为偶数}} \end{array}} \right.$ \item $\displaystyle \sum_{k=0}^{[\frac{n}{2}]}\frac{(n+1-2k)^2}{n+1-k}C_n^k=2^n$ \item $\displaystyle \sum_{k=0}^{n}\frac{(-1)^k}{k+1}C_n^k[(1+x)^{k+1}-1]=(-1)^n\frac{x^{n+1}}{n+1}$ \item $\displaystyle \sum_{k=0}^{n}\frac{(-1)^k}{k+1}C_n^k\left(1+\frac{1}{2}+\cdots+\frac{1}{k+1}\right)=\frac{1}{(n+1)^2}$%50 \item $\displaystyle \sum_{k=1}^{n}(-1)^{k+1}C_n^k\sum_{l=1}^k\frac{1}{l}\left(1+\frac{1}{2}+\cdots+\frac{1}{l}\right)=\frac{1}{n^2}$ \item $\displaystyle \sum_{k=0}^{n}\frac{(-1)^{k}}{k+1}C_n^k\sum_{l=0}^k\frac{1}{l+1}\left(1+\frac{1}{2}+\cdots+\frac{1}{l+1}\right)     =\frac{1}{(n+1)^3}$ \item $\displaystyle \sum_{k=0}^{[\frac{n}{2}]}(-1)^kC_{n-k}^k2^{n-2k}=n+1$ \item $\displaystyle \sum_{k=0}^{[\frac{n}{2}]}(-1)^kC_{n-k}^k\frac{(2\cos x)^{n-2k}}{n-k}=\frac{2}{n}\cos nx$ \item $\displaystyle \sum_{k=0}^{[\frac{n}{2}]}C_n^k\cos(n-2k)x =\left\{ {\begin{array}{*{20}{c}} {\displaystyle 2^{n-1}\cos^nx,n\text{为奇数}}\\ {\displaystyle 2^{n-1}\cos^nx+\frac{1}{2}C_n^{\frac{n}{2}},n\text{为偶数}} \end{array}} \right.$ \item $\displaystyle \sum_{k=1}^{n}\frac{(-1)^{k+1}}{k}C_n^k\left(1+\frac{1}{2}+\cdots+\frac{1}{k}\right)=\sum_{k=1}^n\frac{1}{k^2}$ \item $\displaystyle \sum_{k=1}^{n}(-1)^{k}C_n^k\left(1+\frac{1}{2^2}+\cdots+\frac{1}{k^2}\right)=-\frac{1}{n}\sum_{k=1}^n\frac{1}{k}$ \item $\displaystyle \sum_{k=0}^{n}(-1)^kC_n^k(x+n-k)^n=n!$ \item $\displaystyle \sum_{k=0}^{n}(-1)^kP(k)C_n^k=0$这里$P(k)$是$k$的$p(<n)$次多项式 \item $\displaystyle \sum_{k=0}^{n}(-1)^kC_n^k(x-k)^{n+1}=\left(x-\frac{n}{2}\right)(n+1)!$%60 还有100个
kuing 9# 2012-10-26 13:27
niubility
Tesla35 10# 2012-10-27 00:08
本帖最后由 Tesla35 于 2012-10-30 16:35 编辑 1~160:(全) \item $\displaystyle C_n^k=C_n^{n-k}$ \item $\displaystyle C_n^k=C_{n-1}^k+C_{n-1}^{k-1}$ \item $\displaystyle C_n^k=\frac{n}{k}C_{n-1}^{k-1}$ \item $\displaystyle C_n^kC_k^m=C_n^mC_{n-m}^{k-m}=C_n^{k-m}C_{n-k+m}^m(m\leqslant k\leqslant n)$ \item $\displaystyle \sum_{k=0}^{n}C_n^k=2^n$ \item $\displaystyle \sum_{k=0}^{n}(-1)^kC_n^k=0$ \item $\displaystyle \sum_{k=0}^{m}(-1)^kC_n^k=(-1)^mC_{n-1}^m(m<n)$ \item $\displaystyle \sum_{k=0}^{n}C_{2n}^k=2^{n-1}+\frac{1}{2}C_{2n}^n$ \item $\displaystyle \sum_{k=0}^{n}\frac{1}{k+1}C_n^k=\frac{1}{n+1}(2^{n+1}-1)$ \item $\displaystyle \sum_{k=0}^{n}(-1)^k\frac{1}{k+1}C_n^k=\frac{1}{n+1}$ %10 \item $\displaystyle \sum_{k=1}^{n}k^2C_n^k=n(n+1)2^{n-2}$ \item $\displaystyle \sum_{k=0}^{n}(-1)^kC_n^k\frac{m}{m+k}=(C_{m+n}^n)^{-1}$ \item $\displaystyle \sum_{k=1}^{n}(-1)^{k+1}\frac{1}{k}C_n^k=1+\frac{1}{2}+\cdots+\frac{1}{n}$ \item $\displaystyle \sum_{k=0}^{n}(-1)^k2^{2n-2k}C_{2n-k+1}^k=n+1$ \item $\displaystyle \sum_{k=0}^{n}(-1)^k2^{2n-2k}C_{2n-k}^k=2n+1$ \item $\displaystyle \sum_{k=0}^{n}2^kC_n^k=3^n$ \item $\displaystyle \sum_{k=0}^{n}kC_n^k=n2^{n-1}$ \item $\displaystyle \sum_{k=0}^{n}(k+1)C_n^k=(n+2)2^{n-1}$ \item $\displaystyle \sum_{k=0}^{n}(-1)^kkC_n^k=0(n>1)$ \item $\displaystyle \sum_{k=0}^{n}\frac{1}{k+1}C_n^kx^k=\frac{(1+x)^{n+1}-1}{(n+1)x}$%20 \item $\displaystyle \sum_{k=0}^{n}\frac{(-1)^k}{(k+1)^2}C_n^k=\frac{1}{n+1}\left(1+\frac{1}{2}+\cdots+\frac{1}{n+1}\right)$ \item $\displaystyle \sum_{k=0}^{n-1}C_{2n-1}^k=2^{2n-2}$ \item $\displaystyle \sum_{k=0}^{n}kC_{2n}^k=n2^{2n-1}$ \item $\displaystyle \sum_{k=1}^{n}(-1)^{k+1}\frac{1}{k}C_n^k[1-(1-x)^k]=\sum_{k=1}^{n}\frac{x^k}{k}$ \item $\displaystyle \sum_{k=0}^{n}\frac{(-1)^k}{2k+1}C_n^k=\frac{2^{2n}}{2n+1}(C_{2n}^n)^{-1}$ \item $\displaystyle \sum_{k=0}^{n}\frac{(-1)^k}{1-2k}C_n^k=2^{2n}(C_{2n}^n)^{-1}$ \item $\displaystyle \sum_{k=0}^{n}kC_{2n+1}^k=(2n+1)2^{2n-1}-\frac{2n+1}{2}C_{2n}^n$ \item $\displaystyle \sum_{k=0}^{n}kC_{2n}^{n-k}=nC_{2n-1}^{n}$ \item $\displaystyle \sum_{k=0}^{n}kC_{2n+1}^{n-k}=(2n+1)C_{2n-1}^{n}-2^{2n-1}$ \item $\displaystyle \sum_{k=0}^{n}C_{2n+1}^{2k+1}x^k=2x^{-\frac{1}{2}}[(1+\sqrt{x})^{2n+1}-(1-\sqrt{x})^{2n+1}]$%30 \item $\displaystyle \sum_{k=0}^{[\frac{n}{2}]}C_{n-k}^k= \frac{1}{\sqrt{5}}\left\{\left(\frac{1+\sqrt{5}}{2}\right)^{n+1}-\left(\frac{1-\sqrt{5}}{2}\right)^{n+1}\right\}$ \item $\displaystyle \sum_{k=0}^{[\frac{n}{2}]}C_{n-k}^kr^k= \frac{1}{2^{n+1}\sqrt{1+4r}}\left\{(1+\sqrt{1+4r})^{n+1}-(1-\sqrt{1+4r})^{n+1}\right\}$ \item $\displaystyle \sum_{k=0}^{[\frac{n}{2}]}C_{n-k}^k2^k=\frac{1}{3}(2^{n+1}+(-1)^n)$ \item $\displaystyle \sum_{k=0}^{[\frac{n}{2}]}C_{n-k}^k6^k=\frac{1}{5}(3^{n+1}+(-1)^n2^{n+1})$ \item $\displaystyle \sum_{k=0}^{[\frac{n}{2}]}(-1)^kC_{n-k}^k=\frac{2}{\sqrt{3}}\sin\frac{n+1}{3}\pi$ \item $\displaystyle \sum_{k=0}^{n-1}C_{n+k}^{2k+1}= \frac{1}{\sqrt{5}}\left\{\left(\frac{3+\sqrt{5}}{2}\right)^{n}-\left(\frac{3-\sqrt{5}}{2}\right)^{n}\right\}$ \item $\displaystyle \sum_{k=0}^{n}C_{n+k}^{2k}= \frac{1}{\sqrt{5}}\left\{\left(\frac{1+\sqrt{5}}{2}\right)^{2n+1}-\left(\frac{1-\sqrt{5}}{2}\right)^{2n+1}\right\}$ \item $\displaystyle \sum_{k=0}^{m}C_{k+n-1}^{n-1}=C_{m+n}^{n}$ \item $\displaystyle \sum_{k=0}^{n}2^{n-k}C_{n+k}^{2k}=\frac{1}{3}(2^{2n+1}+1)$ \item $\displaystyle \sum_{k=0}^{n}(-1)^kC_n^k\left(1+\frac{1}{2}+\cdots+\frac{1}{k}\right)=-\frac{1}{n}$%40 \item $\displaystyle \sum_{k=0}^{n}(-1)^kC_n^k\left(x+\frac{x^2}{2}+\cdots+\frac{x^k}{k}\right)=-\frac{1}{n}[1-(1-x)^n]$ \item $\displaystyle \sum_{k=0}^{n}C_{n+p}^{k+p}x^k=\sum_{k=0}^{n}C_{p+k-1}^n(1+x)^{n-k}$ \item $\displaystyle \sum_{k=0}^{n}(-1)^kC_{n+p}^{k+p}=C_{n+p-1}^n$ \item $\displaystyle \sum_{k=0}^{[\frac{n}{2}]}(-1)^k\frac{n}{n-k}C_{n-k}^kx^{n-2k}= \frac{1}{2^n}\left\{(x+\sqrt{x^2-4})^n+(x-\sqrt{x^2-4})^n\right\}$ \item $\displaystyle \sum_{k=0}^{[\frac{n}{2}]}(-1)^k\frac{n}{n-k}C_{n-k}^k2^{n-2k}=2$ \item $\displaystyle \sum_{k=0}^{[\frac{n}{2}]}(-1)^k\frac{n}{n-k}C_{n-k}^k=2\cos\frac{n\pi}{3},n=1,2,\cdots.$ \item $\displaystyle \sum_{k=0}^{[\frac{n}{2}]}C_n^k\cos\frac{(n-2k)}{3}\pi =\left\{ {\begin{array}{ll} \displaystyle\frac{1}{2},&n\text{为奇数}\\[1.5ex] \displaystyle\frac{1}{2}(1+C_n^{\frac{n}{2}}),&n\text{为偶数} \end{array}} \right.$ \item $\displaystyle \sum_{k=0}^{[\frac{n}{2}]}\frac{(n+1-2k)^2}{n+1-k}C_n^k=2^n$ \item $\displaystyle \sum_{k=0}^{n}\frac{(-1)^k}{k+1}C_n^k[(1+x)^{k+1}-1]=(-1)^n\frac{x^{n+1}}{n+1}$ \item $\displaystyle \sum_{k=0}^{n}\frac{(-1)^k}{k+1}C_n^k\left(1+\frac{1}{2}+\cdots+\frac{1}{k+1}\right)=\frac{1}{(n+1)^2}$%50 \item $\displaystyle \sum_{k=1}^{n}(-1)^{k+1}C_n^k\sum_{l=1}^k\frac{1}{l}\left(1+\frac{1}{2}+\cdots+\frac{1}{l}\right)=\frac{1}{n^2}$ \item $\displaystyle \sum_{k=0}^{n}\frac{(-1)^{k}}{k+1}C_n^k\sum_{l=0}^k\frac{1}{l+1}\left(1+\frac{1}{2}+\cdots+\frac{1}{l+1}\right)     =\frac{1}{(n+1)^3}$ \item $\displaystyle \sum_{k=0}^{[\frac{n}{2}]}(-1)^kC_{n-k}^k2^{n-2k}=n+1$ \item $\displaystyle \sum_{k=0}^{[\frac{n}{2}]}(-1)^kC_{n-k}^k\frac{(2\cos x)^{n-2k}}{n-k}=\frac{2}{n}\cos nx$ \item $\displaystyle \sum_{k=0}^{[\frac{n}{2}]}C_n^k\cos(n-2k)x =\left\{ {\begin{array}{ll} \displaystyle 2^{n-1}\cos^nx,&n\text{为奇数}\\[1.5ex] \displaystyle 2^{n-1}\cos^nx+\frac{1}{2}C_n^{\frac{n}{2}},&n\text{为偶数} \end{array}} \right.$ \item $\displaystyle \sum_{k=1}^{n}\frac{(-1)^{k+1}}{k}C_n^k\left(1+\frac{1}{2}+\cdots+\frac{1}{k}\right)=\sum_{k=1}^n\frac{1}{k^2}$ \item $\displaystyle \sum_{k=1}^{n}(-1)^{k}C_n^k\left(1+\frac{1}{2^2}+\cdots+\frac{1}{k^2}\right)=-\frac{1}{n}\sum_{k=1}^n\frac{1}{k}$ \item $\displaystyle \sum_{k=0}^{n}(-1)^kC_n^k(x+n-k)^n=n!$ \item $\displaystyle \sum_{k=0}^{n}(-1)^kP(k)C_n^k=0$这里$P(k)$是$k$的$p(<n)$次多项式 \item $\displaystyle \sum_{k=0}^{n}(-1)^kC_n^k(x-k)^{n+1}=\left(x-\frac{n}{2}\right)(n+1)!$%60 \item $\displaystyle \sum_{k=1}^{n}(-1)^kC_n^k\left(1+\frac{1}{3}+\cdots+\frac{1}{2k-1}\right)=-\frac{2^{2n-2}}{2n-1}(C_{2n-2}^{n-1})^{-1}$ \item $\displaystyle \sum_{k=1}^{n}k^4C_n^k=C_n^12^{n-1}+14C_n^22^{n-2}+36C_n^32^{n-3}+24C_n^42^{n-4}$ \item $\displaystyle \sum_{k=0}^{n}(-1)^k(k+1)^nC_{n+1}^{k+1}=0$ \item $\displaystyle \sum_{k=0}^{n}\sum_{l=0}^{k}(-1)^lC_k^l(l+1)^n=0$\\ 在65-73的等式中,$\{a_n\}$是$p$阶等差数列,$\Delta^ka_0$是$\{a_n\}$的$k$阶差分数列首项,$(n\geqslant p)$ \item $\displaystyle \sum_{k=0}^{n}a_kC_n^k=\sum_{k=0}^{p}2^{n-k}C_n^k\Delta^ka_0$ \item $\displaystyle \sum_{k=0}^{n}(-1)^ka_kC_n^k=0(n>p)$ \item $\displaystyle \sum_{k=0}^{n}a_kC_n^kx^k(1-x)^{n-k}=\sum_{k=0}^{p}C_n^k\Delta^ka_0x^k$ \item $\displaystyle \sum_{k=0}^{n}a_kC_n^kt^k=\sum_{k=0}^{p}C_n^kt^k(1+t)^{n-k}\Delta^ka_0$ \item $\displaystyle \sum_{k=0}^{n}(-1)^ka_kC_n^kt^k=\sum_{k=0}^{p}(-1)^kC_n^kt^k(1-t)^{n-k}\Delta^ka_0$ \item $\displaystyle \sum_{k=0}^{n}a_kC_n^k\cos kx= \sum_{k=0}^{p}C_n^k2^{n-k}\left(\cos \frac{x}{2}\right)^{n-k}\cos\frac{n+k}{2}x\Delta^ka_0$%70 \item $\displaystyle \sum_{k=0}^{n}a_kC_n^k\sin kx= \sum_{k=0}^{p}C_n^k2^{n-k}\left(\cos \frac{x}{2}\right)^{n-k}\sin\frac{n+k}{2}x\Delta^ka_0$ \item $\displaystyle \sum_{k=0}^{n}(-1)^ka_kC_n^k\cos kx= (-1)^n\sum_{k=0}^{p}C_n^k2^{n-k}\cos \frac{1}{2}[n(x+\pi)+k(x-\pi)]\left(\sin \frac{x}{2}\right)^{n-k}\Delta^ka_0$ \item $\displaystyle \sum_{k=0}^{n}(-1)^ka_kC_n^k\sin kx= (-1)^n\sum_{k=0}^{p}C_n^k2^{n-k}\sin \frac{1}{2}[n(x+\pi)+k(x-\pi)]\left(\sin \frac{x}{2}\right)^{n-k}\Delta^ka_0$ \item $\displaystyle \sum_{k=0}^{n}C_n^k\cos kx=2^n\left(\cos\frac{x}{2}\right)^n\cos\frac{x}{2}x$ \item $\displaystyle \sum_{k=0}^{n}C_n^k\sin kx=2^n\left(\cos\frac{x}{2}\right)^n\sin\frac{x}{2}x$ \item $\displaystyle \sum_{k=0}^{n}(-1)^kC_n^k\cos kx=(-1)^n2^n\cos\frac{n}{2}(x+\pi)\left(\sin\frac{x}{2}\right)^n$ \item $\displaystyle \sum_{k=0}^{n}(-1)^kC_n^k\sin kx=(-1)^n2^n\sin\frac{n}{2}(x+\pi)\left(\sin\frac{x}{2}\right)^n$ \item $\displaystyle \sum_{k=0}^{n}kC_n^k\cos kx=n2^{n-1}\left(\cos\frac{x}{2}\right)^{n-1}\cos\frac{n+1}{2}x$ \item $\displaystyle \sum_{k=0}^{n}kC_n^k\sin kx=n2^{n-1}\left(\cos\frac{x}{2}\right)^{n-1}\sin\frac{n+1}{2}x$ \item $\displaystyle \sum_{k=0}^{n}(-1)^kkC_n^k\cos kx=(-1)^nn2^{n-1}\cos \frac{1}{2}[(n+1)x+(n-1)\pi]\left(\sin \frac{x}{2}\right)^{n-1}$%80 \item $\displaystyle \sum_{k=0}^{n}(-1)^kkC_n^k\sin kx=(-1)^nn2^{n-1}\sin \frac{1}{2}[(n+1)x+(n-1)\pi]\left(\sin \frac{x}{2}\right)^{n-1}$ \item $\displaystyle \sum_{k=0}^{n}(-1)^kC_n^k2^k\left(\cos\frac{x}{2}\right)^k\cos\frac{k}{2}x=(-1)^n\cos nx$ \item $\displaystyle \sum_{k=0}^{n}(-1)^kC_n^k2^k\left(\cos\frac{x}{2}\right)^k\sin\frac{k}{2}x=(-1)^n\sin nx$ \item $\displaystyle \sum_{k=0}^{n}C_n^k2^k\cos\frac{k}{2}(x+\pi)\left(\sin\frac{x}{2}\right)^k=\cos nx$ \item $\displaystyle \sum_{k=0}^{n}C_n^k2^k\sin\frac{k}{2}(x+\pi)\left(\sin\frac{x}{2}\right)^k=\sin nx$ \item $\displaystyle \sum_{k=0}^{n}(-1)^k2^{\frac{k}{2}}C_n^k\cos\frac{\pi}{4}=\cos\frac{n\pi}{2}$ \item $\displaystyle \sum_{k=0}^{n}(-1)^{k+1}2^{\frac{k}{2}}C_n^k\sin\frac{\pi}{4}=\sin\frac{n\pi}{2}$ \item $\displaystyle \sum_{k=0}^{[\frac{n-r}{m}]}C_n^{r+km}=     \frac{1}{m}\sum_{k=0}^{m-1}\left(2\cos\frac{k\pi}{m}\right)\cos\frac{(n-2r)k\pi}{m},r\leqslant m$ \item $\displaystyle \sum_{k=0}^{[\frac{n}{2}]}C_n^{2k}=2^{n-1}$ \item $\displaystyle \sum_{k=0}^{[\frac{n}{4}]}C_n^{4k}=\frac{1}{4}\left(2^n+2(\sqrt{n})^n\cos\frac{n\pi}{4}\right)$%90 \item $\displaystyle \sum_{k=0}^{n}C_{4n}^{4k}=\frac{1}{4}(2^{4n}+(-1)^n2^{2n+1})$ \item $\displaystyle \sum_{k=0}^{[\frac{n-1}{3}]}C_n^{1+3k}=\frac{1}{3}\left(2^n+2\cos\frac{(n-2)}{2}\pi\right)$ \item $\displaystyle \sum_{k=0}^{[\frac{n-2}{3}]}C_n^{2+3k}=\frac{1}{3}\left(2^n+2\cos\frac{(n-4)}{2}\pi\right)$ \item $\displaystyle \sum_{k=0}^{n-1}(-1)^k\left(\cos\frac{k\pi}{n}\right)^n=\frac{n}{2^{n-1}}$ \item $\displaystyle \sum_{k=0}^{[\frac{n-r}{m}]}C_n^{r+km}\cos(r+km)x =\frac{2^n}{m}\sum_{k=0}^{m-1}\cos\left[\frac{nx}{2}+\frac{k\pi}{m}(n-2r)\right]\cos^n\left(\frac{x}{2}+\frac{k\pi}{m}\right)$ \item $\displaystyle \sum_{k=0}^{[\frac{n-r}{m}]}C_n^{r+km}\sin(r+km)x =\frac{2^n}{m}\sum_{k=0}^{m-1}\sin\left[\frac{nx}{2}+\frac{k\pi}{m}(n-2r)\right]\cos^n\left(\frac{x}{2}+\frac{k\pi}{m}\right)$ \item $\displaystyle \sum_{k=0}^{[\frac{n}{m}]}C_n^{km}\cos kmx =\frac{2^n}{m}\sum_{k=0}^{m-1}\cos n\left(\frac{x}{2}+\frac{k\pi}{m}\right)\cos^n\left(\frac{x}{2}+\frac{k\pi}{m}\right)$ \item $\displaystyle \sum_{k=0}^{[\frac{n}{m}]}C_n^{km}\sin kmx =\frac{2^n}{m}\sum_{k=0}^{m-1}\sin n\left(\frac{x}{2}+\frac{k\pi}{m}\right)\cos^n\left(\frac{x}{2}+\frac{k\pi}{m}\right)$ \item $\displaystyle \sum_{k=0}^{[\frac{n}{2}]}C_n^{2k}\cos 2k=2^{n-1} \left\{\cos\frac{nx}{2}\cos^n{\frac{x}{2}}+\cos\frac{x+\pi}{2}\left(\cos\frac{x+\pi}{2}\right)^n\right\}$ \item $\displaystyle \sum_{k=0}^{[\frac{n}{2}]}C_n^{2k}\sin 2k=2^{n-1} \left\{\sin\frac{nx}{2}\cos^n{\frac{x}{2}}+\sin\frac{x+\pi}{2}\left(\cos\frac{x+\pi}{2}\right)^n\right\}$%100 \item $\displaystyle \sum_{k=0}^n\left(\frac{k}{n}-a\right)^2C_n^kx^k(1-x)^{n-k}=(x-a)^2+\frac{x(1-x)}{n}$ \item $\displaystyle \sum_{k=0}^{[\frac{n}{2}]}(-1)^kC_{n-k}^k(2\cos x)^{n-2k}=\frac{\sin(n+1)x}{\sin x}$ \item $\displaystyle \sum_{k=0}^n(-1)^kk2^{k-1}C_n^k\left(\cos\frac{x}{2}\right)^{k-1}\cos\frac{k+1}{2}x=(-1)^nn\cos nx$ \item $\displaystyle \sum_{k=0}^n(-1)^kk2^{k-1}C_n^k\left(\cos\frac{x}{2}\right)^{k-1}\sin\frac{k+1}{2}x=(-1)^nn\sin nx$ \item $\displaystyle \sum_{k=0}^nk2^{k-1}C_n^k\cos\frac{1}{2}[(k+1)x+(k-1)\pi]\left(\sin\frac{x}{2}\right)^{k-1}=n\cos nx$ \item $\displaystyle \sum_{k=0}^nk2^{k-1}C_n^k\sin\frac{1}{2}[(k+1)x+(k-1)\pi]\left(\sin\frac{x}{2}\right)^{k-1}=n\sin nx$ \item $\displaystyle \sum_{k=0}^n(-1)^kk2^{k-1}C_n^k2^{\frac{k-1}{2}}\sin\frac{k+1}{4}\pi=(-1)^nn\sin\frac{n\pi}{2}$ \item $\displaystyle \sum_{k=0}^{[\frac{n-1}{2}]}C_n^{2k+1}\cos (2k+1)x=2^{n-1}\left\{\cos^n\frac{x}{2}\cos\frac{nx}{2} -(-1)^n\sin^n\frac{x}{2}\cos\frac{n(x+\pi)}{2}\right\}$ \item $\displaystyle \sum_{k=0}^{[\frac{n-1}{2}]}C_n^{2k+1}\sin (2k+1)x=2^{n-1}\left\{\cos^n\frac{x}{2}\sin\frac{nx}{2} -(-1)^n\sin^n\frac{x}{2}\sin\frac{n(x+\pi)}{2}\right\}$ \item $\displaystyle \sum_{k=0}^{n}C_n^k(x+k)^{k-1}(y+n-k)^{n-k}=x^{-1}(x+y+n)^n$%110 \item $\displaystyle \sum_{k=0}^{n}C_n^k(x+k)^{k-1}(y+n-k)^{n-k-1}=(x^{-1}+y^{-1})(x+y+n)^{n-1}$ \item $\displaystyle \sum_{k=0}^{n}C_n^k(x+k)^{k-2}(y+n-k)^{n-k}=x^{-2}(x+1)^{-1}[(x+1)(x+y+n)^n-nx(x+y+n)^{n-1}]$ \item $\displaystyle \sum_{k=0}^{n}C_n^k(x+k)^k(y+n-k)^{n-k}=\sum_{k=0}^{n}C_n^kk!(x+y+n)^{n-k}$ \item $\displaystyle \sum_{k=1}^{n}C_{n-1}^{k-1}n^{-k}k!=1$ \item $\displaystyle \sum_{k=0}^{n}C_n^k(x+k)(x+n)^{n-k-1}k!=(x+n)^n$ \item $\displaystyle \sum_{k=0}^{n}C_n^kx(x+k)^{k-1}(y-k)^{n-k}=(x+y)^n$ \item $\displaystyle \sum_{k=0}^{n}C_n^k(x-k)^{n-k}y(y+k)^{k-1}=(x+y)^n$ \item $\displaystyle \sum_{k=0}^{n}C_n^k(x+k)^k(y+n-k)^{n-k-1}=y^{-1}(x+y+n)^n$ \item $\displaystyle \sum_{k=0}^{n}C_n^kk^k(n-k+1)^{n-k-1}=(n+1)^n$ \item $\displaystyle \sum_{k=0}^{n}(-1)^kC_n^k(n-k-1)^{n-k-1}(k+1)^k=-n^n$%120 \item $\displaystyle \sum_{k=0}^{n}C_n^k(x^2-k)(-x+k)^{k-2}(x+y+n-k)^{n-k}=y(y+n)^{n-1}$ \item $\displaystyle \sum_{k=1}^{n}C_n^kk^{k-1}(n-k+1)^{n-k}=n(n+1)^{n-1}$ \item $\displaystyle \sum_{k=m}^{n}(-1)^kC_n^kC_k^m=(-1)^m\delta_{mn}$ \item $\displaystyle \sum_{k=m}^{n}C_n^kC_k^m=2^{n-m}C_n^m$ \item $\displaystyle \sum_{k=1}^{n-1}\frac{1}{k(n-k)}C_{2(k-1)}^{k-1}C_{2(n-k-1)}^{n-k-1}=\frac{1}{n}C_{2(n-1)}^{n-1}$ \item $\displaystyle \sum_{k=l}^{n}(-1)^k\frac{1}{k+1}C_n^kC_k^l=\frac{(-1)^l}{n+1}$ \item $\displaystyle \sum_{k=m}^{n}C_n^kC_k^mx^k(1-x)^{n-k}=C_n^mx^m$ \item $\displaystyle \sum_{k=0}^{n}(C_n^k)^2=C_{2n}^n$ \item $\displaystyle \sum_{k=0}^{n}(-1)^k(C_n^k)^2= \left\{ {\begin{array}{ll} \displaystyle 0,&n\text{为奇数}\\[1.5ex] \displaystyle(-1)^\frac{n}{2}C_n^{\frac{n}{2}},&n\text{为偶数} \end{array}} \right.$ \item $\displaystyle \sum_{k=0}^{n}C_{p+k}^pC_{q+n-k}^q=C_{p+q+1}^{p+q+n+1}$%130 \item $\displaystyle \sum_{k=0}^{q}C_n^kC_m^{q-k}=C_{m+n}^q$ \item $\displaystyle \sum_{k=0}^{n}(-1)^kC_n^kC_{m+k}^q=(-1)^nC_m^{q-n}$ \item $\displaystyle \sum_{k=0}^{n}C_{2k}^kC_{2n-2k}^{n-k}=2^{2n}$ \item $\displaystyle \sum_{k=0}^{n-1}(C_n^0+C_n^1+\cdots+C_n^k)(C_n^{k+1}+C_n^{k+2}+\cdots+C_n^n)=\frac{n}{2}C_{2n}^n$ \item $\displaystyle \sum_{k=1}^{n}C_n^{k-1}C_n^k=\frac{(2n)!}{(n-1)!(n+1)!}$ \item $\displaystyle \sum_{k=0}^{2n}C_m^kC_m^{2n-k}=(-1)^nC_m^n$ \item $\displaystyle \sum_{k=0}^{n}kC_n^kC_m^k=nC_{n+m-1}^n$ \item $\displaystyle \sum_{k=0}^{n}(-1)^kC_n^kC_{m+k}^{q+k}=(-1)^nC_m^{q+n}$ \item $\displaystyle \sum_{k=0}^{n}kC_{2k}^kC_{2n-2k}^{n-k}=n2^{2n-1}$ \item $\displaystyle \sum_{k=0}^{n}(-1)^kC_n^kC_{m+n+k}^m=\delta_{mn}(m\leqslant n)$%140 \item $\displaystyle \sum_{k=0}^{n-1}(C_{2n}^{2k+1})^2=\frac{1}{2}(C_{4n}^{2n}+(-1)^{n-1}C_{2n}^n)$ \item $\displaystyle \sum_{k=0}^{[\frac{n}{2}]}(-1)^kC_{n+1}^kC_{2n-2k}^n=n+1$ \item $\displaystyle \sum_{k=0}^{[\frac{n}{2}]}(-1)^kC_n^kC_{2n-2k}^n=2^n$ \item $\displaystyle \sum_{k=0}^{[\frac{n-1}{2}]}\left(1-\frac{2k}{n}\right)^2(C_n^k)^2=\frac{1}{n}C_{2n-1}^{n-1}$ \item $\displaystyle \sum_{k=q}^{r}(-1)^kC_k^qC_n^{r-k}= \left\{ {\begin{array}{ll} \displaystyle (-1)^qC_{n-q-1}^{r-q},&r<n\\[1.5ex] \displaystyle 0,&r\geqslant n \end{array}} \right.$ \item $\displaystyle \sum_{k=0}^{2n}(-1)^kC_{2k}^kC_{4n-2k}^{2n-k}=2^{2n}C_{2n}^n$ \item $\displaystyle \sum_{k=0}^{2n+1}(-1)^kC_{2k}^kC_{4n+2-2k}^{2n+1-k}=0$ \item $\displaystyle \sum_{k=0}^{n}C_{2n-2k}^{n-k}C_{2k}^k\frac{1}{2k-1}=0$ \item $\displaystyle \sum_{k=0}^{n-1}(C_n^0-C_n^1+C_n^2-\cdots+(-1)^kC_n^k) ((-1)^{k+1}C_n^{k+1}+(-1)^{k+2}C_n^{k+2}+\cdots+(-1)^nC_n^n)=-C_{2n-2}^{n-1}$ \item $\displaystyle \sum_{k=0}^{n}(-1)^kC_{n+p}^{k+p}C_{p+k-1}^k=1$%150 \item $\displaystyle \sum_{k=0}^{n}(-1)^kkC_n^kC_{n+m-1}^k=(-1)^nnC_m^n$ \item $\displaystyle \sum_{k=0}^{n}(-1)^kC_{p_1+k}^{p_1}C_{p_2+k}^{p_2}\cdots C_{p_r+k}^{p_r}C_n^k=0, n>p_1+p_2+\cdots+p_r$ \item $\displaystyle \sum_{k=0}^{n}(-1)^k(C_{p+k}^p)^rC_n^k=0,n>rp$ \item $\displaystyle \sum_{k=0}^{n}(-1)^kC_n^kC_{m-2k}^{n-1}=0,m\geqslant 2n$ \item $\displaystyle \sum_{k=0}^{n}(-1)^k(C_n^k)^{-1}=\frac{n+1}{n+2}(1+(-1)^n)$ \item $\displaystyle \sum_{k=0}^{n}(-1)^k\frac{n-k}{C_{2n}^k}=0$ \item $\displaystyle \sum_{k=0}^{n}(-1)^kC_n^k(C_{m+k}^k)^{-1}=\frac{m}{m+n}$ \item $\displaystyle \sum_{k=0}^{n}(-1)^kC_n^k2^{2k}(C_{2k}^k)^{-1}=\frac{1}{1-2n}$ \item $\displaystyle \sum_{k=0}^{n}\frac{(-1)^k}{2k+1}C_n^k(C_{2k}^k)^{-1}2^{2k}=\frac{1}{2n+1}$ \item $\displaystyle \sum_{k=0}^{n}(-1)^{k-1}\frac{2^{2k}}{2k-1}C_n^k(C_{2k-2}^{k-1})^{-1}= 4\left(1+\frac{1}{3}+\cdots+\frac{1}{2n-1}\right)$%160
Tesla35 11# 2012-10-30 16:37
终于完结了
kuing 12# 2012-10-30 16:42
11# Tesla35 辛苦晒,加个威望以表谢意
thread-879-1-6.html: [不等式] 发个简单的
pxchg1200 1# 2012-10-21 19:51
For$a,b,c>0$ prove that: $ a\sqrt{4a^2+5bc}+b\sqrt{4b^2+5ca}+c\sqrt{4c^2+5ab}\ge (a+b+c)^2 $
thread-88-1-2.html: 填空:寻寻觅觅,$\underline{\hspace{4em}}$,凄凄惨惨戚戚
kuing 1# 2011-10-12 15:07
RT
isea 2# 2011-10-13 23:05
1# kuing 李清照的词,应该谁都知道
kuing 3# 2011-10-14 00:40
2# isea 醉翁之意不在酒……
甄术 4# 2011-10-14 10:43
2# isea 醉翁之意不在酒…… kuing 发表于 2011-10-14 00:40 论坛,冷冷清清
thread-882-1-6.html: [数列] 来自人教论坛的要求用数学归纳法证数列不等式
kuing 1# 2012-10-22 14:38
原贴地址:http://bbs.pep.com.cn/forum.php?mod=viewthread&tid=282365 请帮忙用数学归纳法证明 \[\frac{a_1}{a_2}+\frac{a_2}{a_3}+\frac{a_3}{a_4}+\cdots+\frac{a_n}{a_{n+1}}>\frac n2-\frac13\] 其中 $a_n=2^n-1$ 像原贴3#那样直接归不行,而像原贴4#那样的加强显然也是一样(只改变常数显然没用,大概他自己也没动手去做),原贴后面12#的提示也没看懂(goft的提示总是超级简洁)。 其实加强命题也很容易想到,下面将其加强成 \[\frac{a_1}{a_2}+\frac{a_2}{a_3}+\frac{a_3}{a_4}+\cdots+\frac{a_n}{a_{n+1}}>\frac n2-\frac13+\frac1{2^{n+1}}.\] 其中 $n\geqslant4$。($n=1$, $2$, $3$ 时不加强,直接用原不等式验证) 这样就可以直接数归了,但是要先验证 $n=4$ 总感觉不太满意,不知有没有个更好的加强方法?
shidilin 2# 2012-10-22 17:22
http://bbs.pep.com.cn/forum.php?mod=viewthread&tid=621446 9楼,10楼。 ………………………………………………………………………… 我只是想问4楼的加强,如何用数归法操作。谢谢KK!
kuing 3# 2012-10-22 21:37
要不要再强一点 \[\frac{a_1}{a_2}+\frac{a_2}{a_3}+\frac{a_3}{a_4}+\cdots+\frac{a_n}{a_{n+1}}>\frac n2-\frac{27}{89}\] 比那个13/42强
kuing 4# 2012-10-23 13:36
数值上较强了,用相同的方法可能就要从比较后面的项开始用归纳法才行,也就是说要验证前面很多个n……超麻烦
thread-883-1-6.html: 大家来找碴,经典问题A,B是两个村子,使得AB+AC最小?
realnumber 1# 2012-10-23 09:14
经典问题:A,B是两个村子,需要在河边修个码头C,使得AB+AC最小? 1.        河流比较理想化,一般随地形形成,不会总是直线. 2.        在一些极端情况 才是合适的,或 其中D是三角形ABC的费马点. 这样的路和常识有冲突. 3.        如果A村子有100人,B有200人,问题似乎进一步复杂.在l上选点C的话,可以用费马原理解决,就是光线折射啦. 大家继续....
都市侠影 2# 2012-10-23 12:39
你这完全是瞎扯蛋嘛,你那问题都定死了是 $AC+BC$,你那下面搞成啥东东了?
kuing 3# 2012-10-23 13:24
2# 都市侠影 ?哪里定了?
kuing 4# 2012-10-23 13:27
1# realnumber 话说《数学空间》总第9期(2012年第2期)最后一篇文章就有提到这个问题的
realnumber 5# 2012-10-23 15:41
本帖最后由 realnumber 于 2013-1-30 15:50 编辑 你就当推广问题好了;其实本意是给学生看,实际情况远比理想化后的复杂
kuing 6# 2012-10-23 16:25
http://www.pep.com.cn/rjwk/gzsxs ... 0120724_1133856.htm
thread-885-1-4.html: [数列] 请教一个数列题,先谢谢了!
hongxian 1# 2012-10-23 18:47
1.各项均为正数的数列$\left\{ {{a}_{n}} \right\}$,${{a}_{1}}=a$,${{a}_{2}}=b$,且对满足$m+n=p+q$的正整数$m$,$n$,$p$,$q$都有$\frac{{{a}_{m}}+{{a}_{n}}}{(1+{{a}_{m}})(1+{{a}_{n}})}=\frac{{{a}_{p}}+{{a}_{q}}}{(1+{{a}_{p}})(1+{{a}_{q}})}$ 证明:对任意$a$,存在与$a$有关的常数$\lambda $,使得对于每个正整数$n$,都有$\frac{1}{\lambda }\le {{a}_{n}}\le \lambda $
hongxian 2# 2012-10-23 19:03
2009江西高考22题,看了一下答案感觉有点难想到!
kuing 3# 2012-10-23 19:42
2# hongxian 把答案贴上来瞧瞧……
hongxian 4# 2012-10-23 19:47
本帖最后由 hongxian 于 2012-10-23 20:00 编辑 http://www.jyeoo.com/math2/ques/ ... e-b712-bbcca8d3791d
kuing 5# 2012-10-24 21:23
那种解法比较巧,我只会笨方法了: 当 $a\ne1$ 且 $b\ne1$ 时,先求通项,不难求得 \[a_n=\frac{1-\bigl( \frac{1+a}{1-a} \bigr)^{n-2}\bigl( \frac{1-b}{1+b} \bigr)^{n-1}}{1+\bigl( \frac{1+a}{1-a} \bigr)^{n-2}\bigl( \frac{1-b}{1+b} \bigr)^{n-1}},\] 为使 $a_n$ 恒为正,需 \[ \left| \left( \frac{1+a}{1-a} \right)^{n-2}\left( \frac{1-b}{1+b} \right)^{n-1} \right|<1 \] 对正整数 $n$ 恒成立,两边平方等价于 \[\left( \frac{1-b}{1+b} \right)^2\left( \left( \frac{1+a}{1-a} \right)^2\left( \frac{1-b}{1+b} \right)^2 \right)^{n-2}<1,\] 假如 $\bigl( \frac{1+a}{1-a} \bigr)^2\bigl( \frac{1-b}{1+b} \bigr)^2>1$,那么当 $n$ 充分大时上式左边必然不小于 $1$,所以必需且只需有 \[\left( \frac{1+a}{1-a} \right)^2\left( \frac{1-b}{1+b} \right)^2\leqslant1 \iff -\left| \frac{1-a}{1+a} \right|\leqslant \frac{1-b}{1+b}\leqslant \left| \frac{1-a}{1+a} \right|.\] (1)若 $n$ 为奇数,则 $0<\bigl( \frac{1-b}{1+b} \bigr)^{n-1}\leqslant \bigl( \frac{1-a}{1+a} \bigr)^{n-1}$,当 $\bigl( \frac{1-b}{1+b} \bigr)^{n-1}\to0$ 时 $a_n\to1$,当 $\bigl( \frac{1-b}{1+b} \bigr)^{n-1}=\bigl( \frac{1-a}{1+a} \bigr)^{n-1}$ 时 $a_n=a$,注意到 $a_n$ 关于 $\bigl( \frac{1-b}{1+b} \bigr)^{n-1}$ 总是单调的,所以有 \[\min\{1,a\}\leqslant a_n\leqslant \max\{1,a\};\] (2)若 $n$ 为偶数,则 $-\left| \frac{1-a}{1+a} \right|^{n-1}\leqslant \bigl( \frac{1-b}{1+b} \bigr)^{n-1}\leqslant \left| \frac{1-a}{1+a} \right|^{n-1}$,当 $\bigl( \frac{1-b}{1+b} \bigr)^{n-1}=-\left| \frac{1-a}{1+a} \right|^{n-1}$ 时 $a_n=\frac{1+a+\abs{1-a}}{1+a-\abs{1-a}}$,当 $\bigl( \frac{1-b}{1+b} \bigr)^{n-1}=\left| \frac{1-a}{1+a} \right|^{n-1}$ 时 $a_n=\frac{1+a-\abs{1-a}}{1+a+\abs{1-a}}$,注意到 $a_n$ 关于 $\bigl( \frac{1-b}{1+b} \bigr)^{n-1}$ 是单调递减的,所以有 \[\frac{1+a-\abs{1-a}}{1+a+\abs{1-a}}\leqslant a_n\leqslant \frac{1+a+\abs{1-a}}{1+a-\abs{1-a}}.\] 注意到 $\frac{1+a-\abs{1-a}}{1+a+\abs{1-a}}<1$, $\frac{1+a+\abs{1-a}}{1+a-\abs{1-a}}>1$ 且 \begin{align*} a-\frac{1+a-\abs{1-a}}{1+a+\abs{1-a}}&=\frac{(a+1)\bigl(\abs{1-a}-(1-a)\bigr)}{1+a+\abs{1-a}}\geqslant 0,\\ \frac{1+a+\abs{1-a}}{1+a-\abs{1-a}}-a&=\frac{(a+1)\bigl(\abs{1-a}+(1-a)\bigr)}{1+a-\abs{1-a}}\geqslant 0, \end{align*} 所以综合上述两点,对于任意正整数 $n$,都有 \[\frac{1+a-\abs{1-a}}{1+a+\abs{1-a}}\leqslant a_n\leqslant \frac{1+a+\abs{1-a}}{1+a-\abs{1-a}},\] 于是,取 \[\lambda = \frac{1+a+\abs{1-a}}{1+a-\abs{1-a}},\] 就有 \[\frac1\lambda \leqslant a_n\leqslant \lambda\] 恒成立。 容易验证此结果也适合 $a=1$ 或 $b=1$ 的情形,所以此结果就是所求。 不知这样对不对?
kuing 6# 2012-10-24 23:06
用一些数字验证过,应该是正确的。
hongxian 7# 2012-10-24 23:18
5# kuing 笨办法见真功夫,光一个通项就够我一求的了!
kuing 8# 2012-10-24 23:29
7# hongxian 求通项很简单啊 \[\frac{a_m+a_n}{(1+a_m)(1+a_n)}=\frac12\cdot \frac{(1+a_m)(1+a_n)-(1-a_m)(1-a_n)}{(1+a_m)(1+a_n)}=\frac12-\frac12\cdot \frac{1-a_m}{1+a_m}\cdot \frac{1-a_n}{1+a_n},\] 令 $b_n=\frac{1-a_n}{1+a_n}$,就有 $b_mb_n=b_pb_q$,令 $m=2$, $p=n+1$, $q=1$(这里昨晚打错了),然后就得到 $b_n$ 等比,求出后解出 $a_n$
kuing 9# 2012-10-25 00:01
怎样,这个过程应该比标答好想一些吧?至少思路上……
hongxian 10# 2012-10-25 08:47
9# kuing 思路的确要顺得多,不过计算量也够呛,也种计算量也只有想k版这样功力深厚的才能算得出来,而且所有的情况都能考虑到!佩服!
kuing 11# 2012-10-25 09:25
10# hongxian 我怎么没觉得计算量哪里大了……
kuing 12# 2012-10-25 09:33
突然发现最后的结果可以更加简单。 在得到 \[\frac{1+a-\abs{1-a}}{1+a+\abs{1-a}}\leqslant a_n\leqslant \frac{1+a+\abs{1-a}}{1+a-\abs{1-a}}\] 之后,如果 $a>1$,可以化简为 \[\frac1a\leqslant a_n\leqslant a;\] 如果 $a<1$,可以化简为 \[a\leqslant a_n\leqslant\frac1a.\] 因此,结果可以写成 \[\min\left\{a,\frac1a\right\}\leqslant a_n\leqslant\max\left\{a,\frac1a\right\},\] 当 $a=1$ 时 $a_n$ 恒为 $1$ 也符合上式,所以,取 \[\lambda=\max\left\{a,\frac1a\right\}\] 便是所求。 这样一来,不但结果更加简单,而且可以看出,这个 $\lambda$ 已经是最佳的结果,不能再小了。
hongxian 13# 2012-10-25 09:45
12# kuing 好结果!
kuing 14# 2012-10-25 17:29
再来分析一下单调性。 当 $a\ne1$ 且 $b\ne1$ 时,为方便书写,记 $A=\frac{1+a}{1-a}$, $B=\frac{1-b}{1+b}$,前面已经得到 $a_n$ 的通项为 \[a_n=\frac{1-A^{n-2}B^{n-1}}{1+A^{n-2}B^{n-1}},\quad\text{其中}~A^2B^2\leqslant 1,\] 则 \[a_{n+1}-a_n=\frac{1-A^{n-1}B^n}{1+A^{n-1}B^n}-\frac{1-A^{n-2}B^{n-1}}{1+A^{n-2}B^{n-1}}=\frac{2A^{n-2}B^{n-1}(1-AB)}{(1+A^{n-1}B^n)(1+A^{n-2}B^{n-1})},\] 如果 $1-AB=0$ 即 $a=b$ 时,则 $a_n$ 恒为 $a$,下设 $1-AB\ne0$,因为 $A^2B^2\leqslant 1$ 所以 $1-AB>0$,故此 $a_{n+1}-a_n$ 的正负取决于 $A^{n-2}B^{n-1}$ 的符号,下面分类讨论。 (1)当 $A>0$, $B>0$ 即 $a<1$, $b<1$ 时,显然 $A^{n-2}B^{n-1}>0$,故 $a_{n+1}>a_n$,即 $a_n$ 为递增数列,再注意到 $\lim_{n\to\infty}a_n=1$,所以此时有 $a\leqslant a_n<1$; (2)当 $A<0$, $B<0$ 即 $a>1$, $b>1$ 时,显然 $A^{n-2}B^{n-1}<0$,故 $a_{n+1}<a_n$,即 $a_n$ 为递减数列,再注意到 $\lim_{n\to\infty}a_n=1$,所以此时有 $1<a_n\leqslant a$; (3)当 $A>0$, $B<0$ 即 $a<1$, $b>1$ 时,若 $n$ 为奇数,则 $A^{n-2}B^{n-1}>0$,若 $n$ 为偶数,$A^{n-2}B^{n-1}<0$,所以此时 $a_n$ 为摆动数列; (4)当 $A<0$, $B>0$ 即 $a>1$, $b<1$ 时,若 $n$ 为奇数,则 $A^{n-2}B^{n-1}<0$,若 $n$ 为偶数,$A^{n-2}B^{n-1}>0$,所以此时 $a_n$ 也为摆动数列。 再者,当 $a_n$ 为摆动数列时,即 $(a-1)(b-1)<0$ 时,对于 $n\geqslant 3$,有 \begin{align*} (a-a_n)(b-a_n)&=\left( -\frac{1-A}{1+A}-\frac{1-A^{n-2}B^{n-1}}{1+A^{n-2}B^{n-1}} \right)\left( \frac{1-B}{1+B}-\frac{1-A^{n-2}B^{n-1}}{1+A^{n-2}B^{n-1}} \right) \\ & =\frac{2(A^{n-1}B^{n-1}-1)}{(1+A)(1+A^{n-2}B^{n-1})}\cdot \frac{2B(A^{n-2}B^{n-2}-1)}{(1+B)(1+A^{n-2}B^{n-1})} \\ & =\frac{(a-1)(b-1)(1-A^{n-2}B^{n-2})(1-A^{n-1}B^{n-1})}{(1+A^{n-2}B^{n-1})^2}\\ &\leqslant 0, \end{align*} 等号成立当且仅当 $AB=-1$。 综上所述,我们得到以下结论: 当 $AB=1$ 时 $a_n=a$; 当 $1>AB>0$ 时 $a_n$ 为单调增或单调减数列,并且 $\min\{1,a\}\leqslant a_n\leqslant \max\{1,a\}$; 当 $0>AB>-1$ 时 $a_n$ 为摆动数列,并且当 $n\geqslant 3$ 时 $\min\{a,b\}<a_n<\max\{a,b\}$; 当 $AB=-1$ 时 $a_n$ 为摆动数列,并且 $a_{2k-1}=a$, $a_{2k}=b$,其中 $k\in\mbb N^+$。 此外,由 \begin{align*} A^2B^2\leqslant 1&\iff (1-a)^2(1+b)^2\geqslant (1+a)^2(1-b)^2\\ & \iff (a-b)(ab-1)\geqslant 0\\ &\iff(b-a)\left(b-\frac1a\right)\leqslant 0, \end{align*} 得到 \[\min \left\{ a,\frac1a \right\}\leqslant b\leqslant \max \left\{ a,\frac1a \right\},\] 再结合上述结论,也不难看出取 $\lambda =\max\left\{ a,\frac1a \right\}$ 是合理的,于是这也算是本题的另一种方法。
kuing 15# 2012-10-25 17:44
加点修饰,大概几乎可以成文了,有机有机
hongxian 16# 2012-10-25 18:17
15# kuing 结果比标答更优,期待大作!
Gauss门徒 17# 2012-12-25 14:15
佩服!
kuing 18# 2012-12-25 14:31
16# hongxian 准备放在下一期数学空间里,跟这里所发的内容大致一样,除了后面考虑摆动的时候有点点不同,证出了摆动的幅度越来越小
yayaweha 19# 2012-12-29 22:47
感觉与拉格朗日插值公式有关
hongxian 20# 2013-1-21 18:50
18# kuing 不会是指第三期吧!好象没有看到!
thread-885-2-4.html:
kuing 21# 2013-1-21 18:52
20# hongxian 还没出呢,上头还在拖着,估计过年前都不一定能出得来
kuing 22# 2013-1-21 18:53
其实对于你来说也没什么所谓了吧,反正主要内容是一样的。
kuing 23# 2013-2-6 20:45
2013第1期已经出了,关于本题的文章见 http://www.pep.com.cn/rjwk/gzsxs ... 0130130_1148978.htm
yayaweha 24# 2013-2-8 18:55
官方的标准答案“巧”在哪里?我没看出来
kuing 25# 2013-2-8 19:24
24# yayaweha 这……无法表达 ……大概就是所谓“只可意会不可言传”?……
yes94 26# 2013-2-8 21:45
25# kuing 应该是,“只可意会,不好言传”,一字之差而已
thread-886-1-2.html: [转]英语也形象
kuing 1# 2012-10-24 20:16
哈,这样记不错
戊概念·五 2# 2012-10-24 21:50
1# kuing
戊概念·五 3# 2012-10-24 23:16
Love me or hate me, both are in my favor. If you love me, I'll always be in your heart. If you hete me, I'll always be in your mind. 试试这个?!
kuing 4# 2012-10-24 23:18
你那微博?
戊概念·五 5# 2012-10-24 23:19
还是送这个给你—— Never frown, even when you are sad, because you never know who is falling in love with your smile. 纵然伤心,也不要愁眉不展,因为你不知是谁会爱上你的笑容。
戊概念·五 6# 2012-10-24 23:21
4# kuing 被你说中了,蛮厉害的嘛~
kuing 7# 2012-10-24 23:30
6# 戊概念·五
戊概念·五 8# 2012-11-6 21:04
7# kuing 有进步~
thread-887-1-6.html: 群聊记录$\cos\frac\pi7-\cos\frac{2\pi}7+\cos\frac{3\pi}7$
kuing 1# 2012-10-27 18:09
thread-888-1-6.html: [不等式] 来自群的简单三元不等式
kuing 1# 2012-10-27 22:33
let $a=y/x$, $b=z/y$, $c=x/z$, $x$, $y$, $z>0$, using CS, Vasc's inequality and AG, we have \begin{align*} \sum\frac1{a(a+1)+ab(ab+1)}&=\sum\frac{x^2}{xy+y^2+zx+z^2}\\ &\geqslant \frac{\left(\sum x^2\right)^2}{\sum x^2(xy+y^2+zx+z^2)}\\ &=\frac{\left(\sum x^2\right)^2}{\sum x^3y+\sum xy^3+2\sum x^2y^2}\\ &\geqslant \frac{\left(\sum x^2\right)^2}{\frac13\left(\sum x^2\right)^2+\frac13\left(\sum x^2\right)^2+\frac23\left(\sum x^2\right)^2}\\ &=\frac34. \end{align*}
kuing 2# 2012-10-27 22:57
不用 Vasc 不等式也行,配方易得 \[4\left(\sum x^2\right)^2-3\left(\sum x^3y+\sum xy^3+2\sum x^2y^2\right)=\sum(x^2+y^2-xy)(x-y)^2+\sum(x^2-y^2)^2.\]
力工 3# 2012-10-28 16:30
2# kuing 忒强了吧,kk.我想不到这么多。
thread-89-1-9.html: [组合] (转)圆排列?m个红,n个绿围成一圈,有几种排法?
realnumber 1# 2011-10-12 15:41
http://bbs.cnool.net/topic_show. ... 494&flag=topic1 莫年:“红色,绿色,白色各两枚棋子围成一圈,不同的围法种数多少种? 请问是否有简单的计数方式,或者来个推广什么的,最好有个公式。” 确实是18种=3×c(4,2)×c(2,2)。先放2个红有3种,再余下4个空位放2个绿,再..。 更进一步m个红,n个绿,s个白在桌面上排成一圈,有几种排法?
realnumber 2# 2011-10-12 15:45
本帖最后由 realnumber 于 2011-10-12 16:14 编辑 研究问题还是从特殊到一般处理 1红+5绿组成一圈,有6种,(穷举,以下同) 2红+4绿组成一圈,有3种 3红+3绿组成一圈,有4种-----圆排列理解的话,公式是5!/(3!3!)=10/3--看来不能把圆排列套用过来,更简单反例是2红+2绿有2种,但3!/(2!2!)=1.5。 4红+2绿组成一圈,有3种 5红+1绿组成一圈,有6种(红,绿可以交换,看来只要研究前3种) n≥6 1红+(n-1)绿组成一圈,有n种 2红+(n-2)绿组成一圈,有n/2种(n为偶数),或(n-1)/2种(n为奇数) 3红+(n-3)绿组成一圈,有?种,想不清楚了,先留个位置。 ---- ---- 3红+3绿组成一圈,有4种 3红+4绿组成一圈,有6种 3红+5绿组成一圈,有7种 3红+6绿组成一圈,有10种
kuing 3# 2011-10-12 16:04
这个贴里好像有相关内容…… http://bbs.pep.com.cn/thread-671809-1-1.html
thread-890-1-6.html: [数列] 来自粉丝群的一道等差数列题
kuing 1# 2012-10-28 16:11
中间略省一段…… 如图所示,这里暂且按照第一种的理解。 依题意知 $a_n$ 递减,且 $a_1$, $a_2$, $\ldots$, $a_8>0$, $a_9$, $a_{10}$, $\ldots <0$,故此 $\displaystyle \frac{S_1}{a_1}$, $\displaystyle \frac{S_2}{a_2}$, $\ldots $, $\displaystyle \frac{S_8}{a_8}>0$, $\displaystyle \frac{S_9}{a_9}$, $\displaystyle \frac{S_{10}}{a_{10}}$, $\ldots $, $\displaystyle \frac{S_{15}}{a_{15}}<0$, $\displaystyle \frac{S_{16}}{a_{16}}$, $\displaystyle \frac{S_{17}}{a_{17}}$, $\ldots >0$,且由于 $S_1$ 到 $S_8$ 递增而 $a_1$ 到 $a_8$ 递减,所以 $\displaystyle \frac{S_1}{a_1}$ 到 $\displaystyle \frac{S_8}{a_8}$ 递增,所以如果存在正整数 $m$ 使 $\displaystyle \frac{S_m}{a_m}>\frac{S_8}{a_8}$ 则必然 $m\geqslant 16$。 设 $S_n=An^2+Bn$,则 $a_n=S_n-S_{n-1}=A(2n-1)+B$,依题意知 $A<0$, $B>0$, $\displaystyle \frac{15}2<-\frac B{2A}<8$,令 $\displaystyle \frac BA=u$,则 $-15>u>-16$,于是 \[\frac{S_n}{a_n}=\frac{An^2+Bn}{A(2n-1)+B}=\frac{n^2+un}{2n-1+u},\] 设 $m\geqslant 16$ 使 $\displaystyle \frac{S_m}{a_m}>\frac{S_8}{a_8}$,则 \[\frac{m^2+um}{2m-1+u}>\frac{64+8 u}{15+u},\] 作差分解等价于 \[\frac{(m-8)(m u+15 m+u^2+7 u-8)}{(u+15)(2m-1+u)}>0,\] 由 $-15>u>-16$, $m\geqslant 16$ 知 $\displaystyle \frac{m-8}{(u+15)(2m-1+u)}<0$,所以上式等价于 \[m u+15 m+u^2+7 u-8<0,\] 即 \[m>\frac{u^2+7u-8}{-(u+15)},\] 不难求得当 $-15>u>-16$ 时有 $\displaystyle \frac{u^2+7u-8}{-(u+15)}$ 的取值范围是 $(136,+\infty)$,所以至少要 $m=137$ 才存在 $u$ 使 $\displaystyle \frac{S_{137}}{a_{137}}>\frac{S_8}{a_8}$,也就是说只要 $t\leqslant 136$,则满足条件的所有数列都会有 $\displaystyle \frac{S_1}{a_1}$, $\displaystyle \frac{S_2}{a_2}$, $\ldots$, $\displaystyle \frac{S_t}{a_t}$ 中必定 $\displaystyle \frac{S_8}{a_8}$ 最大。
joatbmon 2# 2012-10-29 23:39
本帖最后由 joatbmon 于 2012-10-29 23:53 编辑 我不太习惯这种思考方式,能不能找到原题及命题人给的做法 我是这么想的,$\frac{S_n}{a_n}$看成一个函数,分别用求和公式和通项公式代入,得到$f(n)$,这个题不妨取$d=-1$,相当于$a_1$要大于7小于7.5,然后用几何画板把函数画出来,找到$f(8)$,平行于x轴的直线画过去跟函数右边去相交,结果发现右边横坐标最大大概130到140之间,此时a取的是7.499 还有一个问题,取a=7.4会发现最大值大于150,因此我觉得如果是楼主的答案,是不是意思是对于所有满足题意的等差数列必须要恒成立的最大t 看来楼主答案是对的,但是感觉做法不好看
kuing 3# 2012-10-30 02:38
2# joatbmon 题意的不清在截图里面已经提及
thread-891-1-6.html: [不等式] 来自人教数学群的一道三元条件为$2(a+b+c)^2+3(bc+ca+ab)\leqslant 9$的不等式
kuing 1# 2012-10-29 00:26
“学生-天书”说“把11年美国条件改了”,如果是真的话,又不知是谁改的呢?(其实看样子我个人觉得有点像JiChen改的,如果有知情的麻烦说一声。) 题目:已知 $a$, $b$, $c>0$ 满足 $2(a+b+c)^2+3(bc+ca+ab)\leqslant 9$,求证 \begin{equation}\label{sytjbds20121029ys} \frac{bc+1}{(b+c)^2}+\frac{ca+1}{(c+a)^2}+\frac{ab+1}{(a+b)^2}\geqslant 3. \end{equation} 证明:由条件知,要证式 \eqref{sytjbds20121029ys},只需证 \begin{equation}\label{sytjbds20121029zyz1} \sum{\frac{bc}{(b+c)^2}}+\frac{2(a+b+c)^2+3(bc+ca+ab)}9\sum{\frac1{(b+c)^2}}\geqslant 3, \end{equation} 由著名的伊朗96不等式,有 \[\sum{\frac1{(b+c)^2}}\geqslant \frac9{4(bc+ca+ab)},\] 因此,要证式 \eqref{sytjbds20121029zyz1},只需证 \begin{equation}\label{sytjbds20121029zyz2} \sum{\frac{bc}{(b+c)^2}}+\frac{2(a+b+c)^2+3(bc+ca+ab)}{4(bc+ca+ab)}\geqslant 3, \end{equation} 作等价变形,有 \begin{align*} \eqref{sytjbds20121029zyz2} &\iff \sum{\left( \frac{bc}{(b+c)^2}-\frac14 \right)}+\frac{2(a+b+c)^2-6(bc+ca+ab)}{4(bc+ca+ab)}\geqslant 0 \\ &\iff \sum{\frac{-(b-c)^2}{4(b+c)^2}}+\frac{\sum{(b-c)^2}}{4(bc+ca+ab)}\geqslant 0 \\ &\iff \sum{\left( 1-\frac{bc+ca+ab}{(b+c)^2} \right)(b-c)^2}\geqslant 0, \end{align*} 由对称性,不妨设 $a\geqslant b\geqslant c$,则易知 $\displaystyle \frac{a-c}{b-c}\geqslant \frac ab$,记 $\displaystyle S_a=1-\frac{bc+ca+ab}{(b+c)^2}$, $\displaystyle S_b=1-\frac{bc+ca+ab}{(c+a)^2}$, $\displaystyle S_c=1-\frac{bc+ca+ab}{(a+b)^2}$,则易知 $S_b\geqslant 0$, $S_c\geqslant 0$,下面证明 $a^2S_b+b^2S_a>0$。事实上 \begin{align*} a^2S_b+b^2S_a&=a^2+b^2-(bc+ca+ab)\left( \frac{a^2}{(c+a)^2}+\frac{b^2}{(b+c)^2} \right) \\ & >2ab-(bc+ca+ab)\left( \frac a{a+2c}+\frac b{b+2c} \right) \\ & =2ab-\frac{2(bc+ca+ab)^2}{(a+2c)(b+2c)} \\ & =2ab-\frac{2a^2b^2+4abc(a+b)+8abc^2+2c^2(a+b)^2-8abc^2}{ab+2c(a+b)+4c^2} \\ & =\frac{2c^2(a-b)^2}{ab+2c(a+b)+4c^2} \\ & \geqslant 0, \end{align*} 所以 $a^2S_b+b^2S_a>0$ 成立。由此,我们得到 \begin{align*} \sum{\left( 1-\frac{bc+ca+ab}{(b+c)^2} \right)(b-c)^2}&\geqslant S_a(b-c)^2+S_b(c-a)^2 \\ & =(b-c)^2\left( S_a+\left( \frac{a-c}{b-c} \right)^2S_b \right) \\ & \geqslant (b-c)^2\left( S_a+\frac{a^2}{b^2}S_b \right) \\ & =\frac{(b-c)^2(a^2S_b+b^2S_a)}{b^2} \\ & \geqslant 0, \end{align*} 所以式 \eqref{sytjbds20121029zyz2} 成立,故原不等式得证。
kuing 2# 2012-10-29 03:11
果然出自 JiChen 之手,见: http://bbs.cnool.net/cthread-65628662.html 百度搜不出来,还是 google 好使啊
thread-892-1-6.html: [函数] 三次函数问题
nash 1# 2012-10-30 04:04
实数a满足,有且仅有一个正方形四个顶点均在函数y=x^3+ax上,求正方形的边长
kuing 2# 2012-10-30 04:11
似曾相识,可能是老竞赛题。 爪机ing
nash 3# 2012-10-30 04:19
听说是20年前的题目
kuing 4# 2012-10-30 15:29
3# nash 果然没印象错……不过解法显然没印象……
shidilin 5# 2012-10-30 15:49
正方形的中心,是三次曲线的对称中心?
kuing 6# 2012-10-30 15:58
5# shidilin 直觉上如无意外是这样的,不过要证明……
nash 7# 2012-10-31 00:31
假设正方形ABCD的中心不是坐标原点,则A、B、C、D关于原点对称的点A1、B1、C1、D1构成的正方形在函数图像上,矛盾…
kuing 8# 2012-10-31 01:12
7# nash 我倒是想知道如果不是限制了“有且仅有一个”的话,还有没有中心不在原点的这样的正方形?
nash 9# 2012-10-31 01:58
8# kuing 看图像想不出来如何构造,如果去掉“仅有一个”而求a的取值范围,不知道是否可解
realnumber 10# 2012-12-1 12:23
记$f(x)=y=x^3+ax$ 正方形因为对角线互相平分,四个顶点横坐标为c-s,c-t,c+t,c+s,(其中s>0,t>0) ,那么纵坐标$f(c-s)+f(c+s)=f(c-t)+f(c+t)$,化简得c=0,(s=t舍去) 后面就简单了,对角线垂直相等用下好了
kuing 11# 2012-12-1 13:49
10# realnumber 那看来正方形中心只能在原点……
thread-893-1-5.html: [数论] 一道数论题
nash 1# 2012-10-30 04:16

kuing 2# 2012-10-30 04:51
这道也是老题, 我印象中第一次见的时候好像是捕狐犬解过, 何版主还给过一般结论, 明天有空翻翻帖看看。
realnumber 3# 2012-12-1 17:23
本帖最后由 realnumber 于 2012-12-1 17:37 编辑 怎么证的 PS、∈ 是 \in ,花括号是 \{ 和 \} ,max 前可加 \ 使之直立。 kuing 发表于 2012-12-1 16:07 这也被看出来了啊,试下$\{$,$\in$    $\out$ $\up$  $\down$ 忧闷,分子分母看错, ,重新考虑,前面删了
kuing 4# 2012-12-1 19:02
3# realnumber 那我前面的也删了。 out? up? down? 想打什么……
realnumber 5# 2012-12-1 21:12
胡乱打的,就想看看是什么?结果没什么
kuing 6# 2012-12-2 15:51
5# realnumber 有 \in 没有 \out ,但是有 \notin ,即 $\notin$ 没有 \up 和 \down ,但有 \uparrow 和 \downarrow ,即 $\uparrow$ 和 $\downarrow$
realnumber 7# 2012-12-8 16:50
本帖最后由 realnumber 于 2013-1-26 12:49 编辑 以下字母一般都是正整数 \[\frac{a}{a+m}+\frac{c}{c+n} <1 -------------(1)\] 不妨设$a \ge c$,m,n最小可以取1, m=1时,可解得对应的$n=ac+1$,使得(1)成立,且(1)左边最大。 又\[1-\frac{a}{a+m}-\frac{c}{c+n} =\frac{mn-ac}{mn+cm+an+cm},-------(2)\] $mn-ac \ge 0$,固定$mn=ac+k_{0}$,那么$m=1,n=ac+k_{0}$时,(2)分母最大,(2)的值最小 如此可得对于固定的$a,c (a \ge c)$,当$m=1,n=ac+1$,(1)成立,且左边最大。 要解决1楼问题,只需要解答如下问题:$a,c (a \ge c   ,a+c=20),$\[\frac{a}{a+1}+\frac{c}{c+1+ac} <1 \]成立,且使得不等式左边最大 令$a=10+x,c=10-x, 0 \le x \le 10,$ 那么\[1- (\frac{a}{a+1}+\frac{c}{c+1+ac}) =\frac{1}{(a+1)(c+ac+1)}=\frac{1}{(11+x)(111-x-x^2)}\] 设$f(x)=(11+x)(111-x-x^2),   0 \le x  \le 10$, $f′(x)=(111-x-x^2)+(11+x)(-1-2x)=-3x^2-24x+100$,2次函数的对称轴在y轴左侧,$f′(3) \ge 0$,$f′(4)  \le 0$ 经计算f(3)=1386>f(4)=1365,所以a=13,b=14,c=7,d=99符合要求,完--- 另外得到的性质 固定a,c,m,则存在唯一$n_{0}$,当$n \ge n_{0}$时,(1)均成立,显然$n=n_{0}$时,$\frac{a}{b}+\frac{c}{d}$最接近1;当$n < n_{0}$时,(1)均不成立. 对于固定的a,c,$m_{1},m_{2}$($m_{1} \ge m_{2}$),设$m_{1}$对应着$n_{1}$,$m_{2}$对应着$n_{2}$,那么当$n \ge n_{2}  $时候,(1)成立,即$\frac{a}{a+m_{2}}+\frac{c}{c+n} <1 $,而此时$\frac{a}{a+m_{1}}+\frac{c}{c+n} <1 $也成立,说明$n_{1} \le n_{2}$ 1楼条件或类似 推广1:\[\frac{a}{sa+m}+\frac{c}{sc+n} <\frac{1}{s} ,s\in N\],  也可以用同样的办法得到 推广2:\[\frac{a}{a+m}+\frac{c}{c+n} <s ,s\in N ,s \ge 2\],  s给出具体数值比如s=2,似乎还是可以解? 推广3:\[\frac{a}{a+m}+\frac{c}{c+n} +\frac{d}{d+p}<1 \],  一点都不会 ---ps.似乎总是代具体数值开始做实验,走了n多弯路......特殊到一般
wenshengli 8# 2012-12-10 20:43
以下字母一般都是正整数 $\frac{a}{a+m}+\frac{c}{c+n} realnumber 发表于 2012-12-8 16:50 you are great!!!
realnumber 9# 2012-12-12 15:22
本帖最后由 realnumber 于 2013-1-26 21:57 编辑 不清楚是否可解 提问:$a,b,c,d\in N,a+c=20,$\[\frac{a}{b}+\frac{c}{d}<\frac{5}{7},求\frac{a}{b}+\frac{c}{d}的最大值.\]
thread-894-1-6.html: [不等式] 来自人教数学群的三元不等式想切线
kuing 1# 2012-10-30 15:36
群里后面提到,想先均值放缩再切线,结果会反向,而用切平面也失败。 后来我想到用局部切线,先不管 $ab$,只切 $1/(7+2c^2)$,结果切直线和抛物线均失败,然后就睡觉去了,决定睡醒后上软件暴力切三次函数试试,结果刚才居然切成功了,不过真的挺暴力,不用软件我不敢算。 题目:已知 $a$, $b$, $c>0$ 且 $a+b+c=3$,求证 \[\frac{ab}{7+2c^2}+\frac{bc}{7+2a^2}+\frac{ca}{7+2b^2}\leqslant \frac13.\] 证明:我们先证明对任意的 $x\geqslant 0$ 都有 \begin{equation}\label{20121030baoliqie} \frac{567}{7+2x^2}\leqslant K(x), \end{equation} 其中 \[K(x)=4x^3-18x^2-4x+81+9\sqrt{\frac27}(x-1)^2x,\] 事实上,作差因式分解有 \[K(x)-\frac{567}{7+2x^2}=\frac{\bigl( 28+9\sqrt{14} \bigr)\bigl( 9\sqrt{14}-28-10x \bigr)^2(x-1)^2x}{350(2x^2+7)}\geqslant 0,\] 所以式 \eqref{20121030baoliqie} 成立。 由此可见,要证原不等式,只要证明 \[abK(c)+bcK(a)+caK(b)\leqslant 189,\] 即 \begin{equation}\label{20121030zyzs1} abc\sum\left( 4a^2-18a-4+9\sqrt{\frac27}(a-1)^2 \right)+81\sum ab\leqslant 189, \end{equation} 由于 $0<\sum ab\leqslant \left(\sum a\right)^2/3=3$,我们可以设 \[\sum ab=3-\frac{q^2}3,\] 其中 $3>q\geqslant 0$,那么式 \eqref{20121030zyzs1} 可以化简为 \begin{equation}\label{20121030zyzs2} 2abc\left( 3q^2\sqrt{\frac27}+\frac{4q^2}3-27 \right)+27(2-q^2)\leqslant 0, \end{equation} 易证 $3q^2\sqrt{2/7}+4q^2/3-27<0$,于是由 pqr 定理,有 \begin{align*} & 2abc\left( 3q^2\sqrt{\frac27}+\frac{4q^2}3-27 \right)+27(2-q^2) \\ \leqslant{}& \frac{2(3+q)^2(3-2q)}{27}\left( 3q^2\sqrt{\frac27}+\frac{4q^2}3-27 \right)+27(2-q^2) \\ ={}&\frac{-q^2}{567}\left( \bigl( 112+36\sqrt{14} \bigr)q^3+\bigl( 504+162\sqrt{14} \bigr)q^2-2268q-486\sqrt{14}+3591 \right), \end{align*} 可以证明 $\bigl( 112+36\sqrt{14} \bigr)q^3+\bigl( 504+162\sqrt{14} \bigr)q^2-2268q-486\sqrt{14}+3591>0$,所以式 \eqref{20121030zyzs2} 成立,原不等式得证。
都市侠影 2# 2012-10-30 16:16
好暴力........又是个非暴力不合作的 不愧是K版,连函数都是 $K(x)$..........
pxchg1200 3# 2012-10-30 16:16
1# kuing 好暴力啊,有没有简单点的?
kuing 4# 2012-10-30 16:17
好暴力........又是个非暴力不合作的 不愧是K版,连函数都是 $K(x)$.......... 都市侠影 发表于 2012-10-30 16:16 故意用K的
kuing 5# 2012-10-30 16:19
1# kuing 好暴力啊,有没有简单点的? pxchg1200 发表于 2012-10-30 16:16 暂时没想到,CS、AG什么的技术靠你们了,那些我还没掌握。
kuing 6# 2012-10-30 17:38
补充最后“可以证明”的证明 \begin{align*} &\bigl( 112+36\sqrt{14} \bigr)q^3+\bigl( 504+162\sqrt{14} \bigr)q^2-2268q-486\sqrt{14}+3591\\ ={}&\left(4 \bigl(28+9 \sqrt{14}\bigr) q+\frac65 \bigl(28+9 \sqrt{14}\bigr) \left(5+2 \sqrt{54 \sqrt{14}-143}\right)\right) \left(q-\frac3{10} \left(\sqrt{54 \sqrt{14}-143}-5\right)\right)^2\\ &+\frac{27}5 \left(1435-45 \sqrt{14}-\sqrt{450324 \sqrt{14}-431858}\right), \end{align*} 其中 $1435-45 \sqrt{14}-\sqrt{450324 \sqrt{14}-431858}\approx 147.20587$,所以……
thread-895-1-6.html: [几何] 来自人教数学群的抛物线角度相等
kuing 1# 2012-10-30 16:48
设 $A(x_1,y_1)$, $B(x_2,y_2)$,联立方程组,有 \[\left\{\begin{aligned} y &=k(x-a), \\ y^2 &=2px, \end{aligned}\right.\riff k^2x^2-(2p+2k^2a)x+k^2a^2=0,\] 于是由韦达定理,有 \[x_1x_2=a^2,\] 不妨设 $A$ 在 $x$ 轴上方,则 \begin{align*} \tan \angle APF-\tan \angle BPF&=\frac{y_1}{x_1+a}+\frac{y_2}{x_2+a} \\ & =\frac{k(x_1-a)}{x_1+a}+\frac{k(x_2-a)}{x_2+a} \\ & =\frac{2k(x_1x_2-a^2)}{(x_1+a)(x_2+a)} \\ & =0, \end{align*} 所以 $\angle APF=\angle BPF$。
kuing 2# 2012-10-30 16:53
这种题出在选择题感觉浪费了点,就像上面聊天记录里的,有一定经验的都能目测出C了,即使不会证的,要碰运气时估计也猜C的多……咳
kuing 3# 2012-10-30 16:59
搜了一下,原来上面群聊记录中提到的“阿基米德三角形”就是抛物线焦点弦的那堆熟知的常用结论
thread-896-1-1.html: [不等式] 《数学通报》上未给出证明的猜想.看不懂啊……
ccnu_chb_ycb 1# 2012-10-30 20:37
今晚看《数学通报》,看到这个没有证明的数学结论,想起以前就遇到过好几个类似的问题,方法和思维都很凌乱,今天的算是一般性的结果啦,请高人赐教,谢谢
kuing 2# 2012-10-30 22:05
(1)求正实数 $\lambda $ 的范围使不等式链 \[\sqrt{\frac a{\lambda a+b}}+\sqrt{\frac b{\lambda b+a}}\leqslant \sqrt{\frac a{a+\lambda b}}+\sqrt{\frac b{b+\lambda a}}\leqslant \frac2{\sqrt{\lambda +1}} \]对于任意 $a$, $b>0$ 恒成立。 先考虑左边的不等式,由排序不等式知,当 $\lambda \geqslant 1$ 时有 \[\sqrt{\frac a{\lambda a+b}}+\sqrt{\frac b{\lambda b+a}}\leqslant \sqrt{\frac a{a+\lambda b}}+\sqrt{\frac b{b+\lambda a}},\] 当 $0<\lambda<1$ 时反向,所以必先有 $\lambda \geqslant 1$。 再考虑右边的不等式,当 $b=1$, $a\to+\infty$ 时显然 $\sqrt{\frac a{a+\lambda b}}+\sqrt{\frac b{b+\lambda a}}\to1$,所以应有 $\frac2{\sqrt{\lambda +1}}\geqslant 1$,即 $\lambda \leqslant 3$。 令 $a/b=x$, $b/a=y$,则 $x$, $y>0$, $xy=1$,且 \begin{align*} \sqrt{\frac a{a+\lambda b}}+\sqrt{\frac b{b+\lambda a}}\leqslant \frac2{\sqrt{\lambda +1}}&\iff\frac1{1+\lambda x}+\frac1{1+\lambda y}+\frac2{\sqrt{(1+\lambda x)(1+\lambda y)}}\leqslant \frac4{\lambda +1} \\ & \iff\frac{2+\lambda (x+y)}{1+\lambda (x+y)+\lambda^2}+\frac2{\sqrt{1+\lambda (x+y)+\lambda^2}}\leqslant \frac4{\lambda +1} \\ & \iff\frac2{\sqrt{1+\lambda (x+y)+\lambda^2}}\leqslant \frac{3-\lambda }{\lambda +1}+\frac{\lambda^2-1}{1+\lambda (x+y)+\lambda^2} \\ & \iff2\leqslant \frac{3-\lambda }{\lambda +1}\sqrt{1+\lambda (x+y)+\lambda^2}+\frac{\lambda^2-1}{\sqrt{1+\lambda (x+y)+\lambda^2}}, \end{align*} 令 $t=\sqrt{1+\lambda (x+y)+\lambda^2}$,则 $x+y\geqslant 2\sqrt{xy}=2$ 知 $t\in [1+\lambda ,+\infty )$,记 \[f(t)=\frac{3-\lambda }{\lambda +1}\cdot t+\frac{\lambda^2-1}t,\] 若 $\lambda =3$,则 \[\lim_{t\to+\infty}f(t)=0<2,\] 不符合题意; 若 $\lambda =1$,则 $f(t)$ 在 $[1+\lambda ,+\infty )$ 递增,得 \[f(t)\geqslant f(1+\lambda )=2,\] 符合题意; 当 $3>\lambda >1$ 时,$f(t)$ 在 $\left( 0,(1+\lambda )\sqrt{\frac{\lambda -1}{3-\lambda }} \right)$ 递减,在 $\left( (1+\lambda )\sqrt{\frac{\lambda -1}{3-\lambda }},+\infty \right)$ 递增。 若 $3>\lambda >2$,则 $(1+\lambda )\sqrt{\frac{\lambda -1}{3-\lambda }}>1+\lambda$,故此时有 \[f(t)_{\min }=f\left( (1+\lambda )\sqrt{\frac{\lambda -1}{3-\lambda }} \right)=2\sqrt{(3-\lambda )(\lambda -1)}<2,\] 不符合题意; 若 $2\geqslant \lambda >1$,则 $(1+\lambda )\sqrt{\frac{\lambda -1}{3-\lambda }}\leqslant 1+\lambda$,故此时也有 \[f(t)_{\min }=f(1+\lambda )=2,\] 符合题意。 综上所述,所求的 $\lambda $ 的范围为 $[1,2]$。
kuing 3# 2012-10-30 22:59
(2)求正实数 $\lambda $ 的范围使不等式链 \[\sqrt{\frac a{\lambda a+b}}+\sqrt{\frac b{\lambda b+a}}\leqslant \frac2{\sqrt{\lambda +1}}\leqslant \sqrt{\frac a{a+\lambda b}}+\sqrt{\frac b{b+\lambda a}} \]对于任意 $a$, $b>0$ 恒成立。 令 $b=1$, $a\to +\infty $,得到 \[\frac1{\sqrt{\lambda }}\leqslant \frac2{\sqrt{\lambda +1}}\leqslant 1\riff\lambda \geqslant 3.\] 先考虑左边的不等式,令 $\lambda =1/\mu$,则 $0<\mu\leqslant 1/3$,且 \[\sqrt{\frac a{\lambda a+b}}+\sqrt{\frac b{\lambda b+a}}\leqslant \frac2{\sqrt{\lambda +1}} \iff\sqrt{\frac a{a+\mu b}}+\sqrt{\frac b{b+\mu a}}\leqslant \frac2{\sqrt{1+\mu }},\] 在形式上,这与(1)中右边的不等式一样,但是参数的前提范围不一样,不能直接引用其结果,但能类似地做,也就是作同样的代换及等价变形,得到 \[2\leqslant \frac{3-\mu }{\mu +1}\sqrt{1+\mu (x+y)+\mu^2}+\frac{\mu^2-1}{\sqrt{1+\mu (x+y)+\mu^2}},\] 此时由 $0<\mu\leqslant 1/3$ 知上式右边关于 $x+y$ 递增,所以当 $x+y=2$ 时取最小值 $2$,故此对所有的 $\lambda \geqslant 3$ 左边的不等式都成立。 再考虑右边的不等式,与(1)的右边仅是反向,故可以类似地作同样的代换及等价变形,得到 \[2\geqslant \frac{3-\lambda }{\lambda +1}\sqrt{1+\lambda (x+y)+\lambda^2}+\frac{\lambda^2-1}{\sqrt{1+\lambda (x+y)+\lambda^2}},\] 此时由 $\lambda \geqslant 3$ 知上式右边关于 $x+y$ 递减,所以当 $x+y=2$ 时取最大值 $2$,故此对所有的 $\lambda \geqslant 3$ 右边的不等式也都成立。 综上所述,所求的 $\lambda $ 的范围为 $[3,+\infty)$。
reny 4# 2013-5-23 19:40
本帖最后由 reny 于 2013-5-23 21:33 编辑 有个疑问:不妨取$\lambda=7,$那么由#1知,$\sqrt{\dfrac{a}{a+7b}}+\sqrt{\dfrac{b}{7a+b}}\geqslant\dfrac1{\sqrt2}$,而$\sqrt{\dfrac{a}{a+7b}}+\sqrt{\dfrac{b}{7a+b}}$的最大值又如何求呢?
kuing 5# 2013-5-23 20:33
有个疑问:不妨取$\lambda=7,$那么由#1知,$\sqrt{\dfrac{a}{a+7b}}+\sqrt{\dfrac{b}{7a+b}}\geqslant1$,而$\sqrt{\dfrac{a}{a+7b}}+\sqrt{\dfrac{b}{7a+b}}$的最大值又如何求呢? reny 发表于 2013-5-23 19:40 根据3#的结论,当 $\lambda=7$ 时应为 \[\sqrt{\frac a{a+7b}}+\sqrt{\frac b{7a+b}}\geqslant\frac1{\sqrt2}.\] 最大值也不会难到哪去,其实方法在2#已经展示了,你自己可以仿着试试。 PS、要不是你顶起这个贴,我也不知道原来之前我用了两层楼回楼主这个题但是楼主却影都不见……
reny 6# 2013-5-23 21:32
本帖最后由 reny 于 2013-5-23 21:51 编辑 5# kuing 其实,主要是想知道$\sqrt[n]{\dfrac{a}{a+\left(2^n-1\right)b}}+\sqrt[n]{\dfrac{b}{\left(2^n-1\right)a+b}}$的最大值. 取$\lambda=3$时,$1\leqslant\sqrt{\dfrac{a}{a+3b}}+\sqrt{\dfrac{b}{3a+b}}\leqslant\dfrac{3\sqrt2}{4}.$ 当$\dfrac{a}{a+3b}=\dfrac{7+3\sqrt5}{16}或\dfrac{7-3\sqrt5}{16}$时取到最大值,是不是有点特别啊.   可见$n$次方的最大值是有难度的.
thread-897-1-6.html: 今天存档了本论坛所有贴子(Archiver版)更新至V1.1
kuing 1# 2012-11-1 19:49
鉴于这个贴子 http://www.5d6d.net/thread-958579-1-1.html 所说的,为保险起见,今天我特意对本论坛的所有贴子进行了简单存档。 为方便处理和测试,此次的存档暂时只存了Archiver版,也就是纯文字版,无图无附件,不过本论坛中大部分贴子都不需要图片,因为公式可以直接显示出来。 现将本次存档的文件打包传了上网,下载链接是: http://www.kuaipan.cn/file/id_56999842425077798.htm 下载后解压到任何位置均可,可以离线浏览,无需联网也能显示公式,更多细节参见压缩包里的说明文档。 kuing 2012年中光棍节
kuing 2# 2012-11-1 21:30
V1.1更新:原Archiver版字体太小,现已将其改成较大字号,看着舒服点。
kuing 3# 2012-11-2 15:46
终于有 1次下载 了……
thread-898-1-5.html: [不等式] $ \sum_{i=1}^na_i^{\sum\limits_{j\neq i}a_j}>1.$
freame 1# 2012-11-2 13:34
$ a_1,a_2,\cdots,a_n>0, n= 2,3,4,5 $, 那么 $$ \sum_{i=1}^na_i^{\sum\limits_{j\neq i}a_j}>1.$$ 如何证明 $n=4,5$ 的情况? 据说人教论坛研究过这个问题,请教
kuing 2# 2012-11-2 13:46
$ a_1,a_2,\cdots,a_n>0, n= 2,3,4,5 $, 那么 $$ \sum_{i=1}^na_i^{\sum\limits_{j\neq i}a_j}>1.$$ 如何证明 $n=4,5$ 的情况? 据说人教论坛研究过这个问题,请教 freame 发表于 2012-11-2 13:34 经典不等式 http://bbs.pep.com.cn/forum.php?mod=viewthread&tid=323853 是在这里研究过,后面有个解答,不过我一直没看懂
freame 3# 2012-11-2 13:50
谢谢,我先看看去。 人教论坛限制不少,我很久的ID,级别低,现在不能发帖
kuing 4# 2012-11-2 13:55
3# freame n=4在贴里24楼,印象中某期的《中国初等数学研究》里面收录过他的这个证明,可能还不止那里贴的那些,具体你可以问问那贴的楼主,也就是在本论坛上的 realnumber。 人教论坛只会搞限制,呵呵,低级会员在那里很难混的,所以也很冷清了……
realnumber 5# 2012-12-23 10:49
本帖最后由 realnumber 于 2013-1-6 18:04 编辑 我只是证明了 n=3,4,主要方法是分类,a,b,c,d字母范围足够小 至于n=5没证明,猜测类似办法也行,但没精力去分了,n=4已经分得倒胃口了,如果哪天有更好的设想也许会再试试.具体证明kuing的连接就有. 分类设想来自这个:也许有如下类似的原理,f(x)≥g(x)恒成立,夹在虚线间的部分可以看到此时f(x)min>g(x)max 以往也有类似的经验,
realnumber 6# 2012-12-23 10:52
只是分得够小,至于为什么分在那里可以,我也觉得是运气.
realnumber 7# 2013-1-7 20:26
5楼附件加了word版,n=3,4;人教论坛那边似乎看不清晰
thread-9-1-1.html: 本站的 $\LaTeX$ 公式的基本输入[2013-3-30, 2#表格少量更新]
kuing 1# 2011-9-26 09:16
  首先说说什么叫“行内公式”和“行间公式”。   所谓“行内公式”,就是镶嵌在普通文段内的公式,就像 $c^2=a^2+b^2$ 这样,跟文字排在一起。   而“行间公式”则是另起一行并且以居中的形式来显示公式,就像下面这个公式这样 \[c^2=a^2+b^2\]   行间公式通常用来显示文中相对重点的或者较为复杂的公式,同时还可以使其自动编号,以方便其他地方进行引用,就像下面这样 \begin{equation}\label{bzdlatexgsdjbsrzdbh1} c^2=a^2+b^2 \end{equation}   式 \eqref{bzdlatexgsdjbsrzdbh1} 叫勾股定理。   下面开始讲具体输入。   1、行内公式的输入格式:     \(\Large\verb"$公式代码$"\)  或者  $\Large\verb"\(公式代码\)"$   通常我们用前者。   例 1.1:代码 \(\verb"$c^2=a^2+b^2$"\) 显示 $c^2=a^2+b^2$   例 1.2:代码 \(\verb"$\sum_{k=1}^n\frac{1}{k^2}$"\) 显示 $\sum_{k=1}^n\frac{1}{k^2}$   2、行间公式的输入格式:     \(\Large\verb"\[公式代码\]"\)  或者  \(\Large\verb"$$公式代码$$"\)  或者  使用“环境”(\(\verb"\begin{...}...\end{...}"\))   例 2.1:代码 \(\verb"\[c^2=a^2+b^2\]"\) 显示 \[c^2=a^2+b^2\]  例 2.2:代码 \(\verb"\[\sum_{k=1}^n\frac{1}{k^2}\]"\) 显示 \[\sum_{k=1}^n\frac{1}{k^2}\]   至于公式代码部分,常用的代码已经在二楼的列表里给出,对于一般的题目基本能应付了,如果想看更全的代码表大家可以在网上查找相关文档(有些较偏门的代码在这里可能用不了,因为这里毕竟不是真 LaTeX,而目前 MathJax v2.0 所支持的代码可以参考这里:http://www.mathjax.org/docs/2.0/tex.html,网页里头还讲了一些拓展,有兴趣的可以研究下)。   一些说明:   1、美元符号 \(\verb"$"\) 为纯英文状态下按 Shift+4 所得;   2、反斜杠在台式机键盘在F11下边,也要在纯英文状态下输入;   3、“使用‘环境’”的部分留到后面再讲;   *4、行内公式的分式、和式等具有一定高度的公式将会显示得比行间公式(标准大小)的要小,并且一些本应在正上或正下方的上下标将被放到右边(对比“例 1.2”与“例 2.2”的显示效果便知)。这是 TeX 设计者之意,既然在行内,就不应该把行距撑大,故对超过一定高度的公式作这样的处理以减小高度。   如果你不在意行距,要用行内公式但又要使其总以标准大小显示的话,可以公式前面加一个 \displaystyle,比如 \(\verb"$\displaystyle\sum_{k=1}^n\frac{1}{k^2}$"\) 将显示 $\displaystyle\sum_{k=1}^n\frac{1}{k^2}$,而如果只想要分式不变小,可以将 \frac 改成 \dfrac,比如 \(\verb"$\frac12=\dfrac12$"\) 将显示 $\frac12=\dfrac12$。看到行距被撑大了没?   事实上这种情况我会用 $a/b$、$(a+b)/(c+d)$ 这种写法,既不会显小也不撑大行距,只要写法正确(必要的括号加好)我想不会有人看不懂吧。   一些建议:   1、虽然在 MathJax 里,公式中可以包含中文或其他全角符号(比如 α、Δ、π 这些直接能在某些输入法里打出来的全角符号),但我还是建议尽可能都在纯英文状态下输入代码来写公式,尽量不要让公式代码里出现中文或全角符号(经常看到半角全角交替的括号让我比较恼火,必须强调公式中要用纯英文的输入状态以杜绝这一现象);   2、对于一些重点或关键的公式,以及一些较复杂的公式,建议采用行间公式的模式去输入,除了方便观看之外,还方便输入(你试多了自然会知道,复杂的公式用行内公式来写的话可能很难看,你可能经常需要用 \dfrac 代替 \frac,或者经常 \displaystyle 才能达到你想要的效果),而且还能编号和引用,这一点要用到环境,也留到后面讲。
kuing 2# 2011-9-26 10:18
基本常用符号对应代码表: 上下标:$x^n$,$x^{123}$,$x_n$,$x_{123}$,$C_n^m$,$C_{100}^{50}$ x^n,x^{123},x_n,x_{123},C_n^m,C_{100}^{50} 注意:当上标或下标非单个数字或字母时要加花括号括起整体,否则只会对首个数字或字母生效(比如 x^10 会显示 $x^10$),下同。 花括号:$\{a_n\}$ \{a_n\} 在各种括号的输入中,只有花括号才需要加反斜杠,其余直接输入即可。 分式:$\frac{a}{b}$,$\frac12$,$\frac1{23}$ \frac{a}{b},\frac12,\frac1{23} 后面的两个简写自己理解。 根式:$\sqrt{a}$,$\sqrt[n]{a}$,$\sqrt2$,$\sqrt[3]2$ \sqrt{a},\sqrt[n]{a},\sqrt2,\sqrt[3]2 后面的两个简写自己理解。 对数:$\log_ab$,$\ln a$,$\lg10$ \log_ab,\ln a,\lg10 中间的 ln 和 a 之间的空格必不可少。 简言之,若命令后紧跟的是英文字母,就必须用空格隔开,否则 \lna 这样会被系统理解为一个新的未知命令而报错,若紧跟数字则可以不空,下同。 和式:$\sum_{k=1}^{n}f(k)$ \sum_{k=1}^{n}f(k) 左边是行内公式的显示结果,在行间公式才显示标准的:\[\sum_{k=1}^{n}f(k)\] 积式:$\prod_{k=1}^{n}f(k)$ \prod_{k=1}^{n}f(k) 与和式类似。 极限:$\lim_{k\to\infty}k^{-1}=0$ \lim_{k\to\infty}k^{-1}=0 与和式类似,此外,\to 是 $\to$,\infty 是 $\infty$ 积分:$\int_{a}^{b}f(x)dx$ \int_{a}^{b}f(x)dx 与和式类似。 正负、负正:$\pm1$,$\mp1$,$\pm x$,$\mp x$ \pm1,\mp1,\pm x,\mp x 后面两个记得不要少了的空格。 同余: 不带括号:$a\equiv b\mod cd$ 带括号:$a\equiv b\pmod{cd}$ 不带括号:a\equiv b\mod cd 带括号:a\equiv b\pmod{cd} 后者的括号是自动加的,不需要手打,但注意花括号不要少,除非是单个字符 短的左推出、右推出、等价于:$\Rightarrow$,$\Leftarrow$,$\Leftrightarrow$ \Rightarrow,\Leftarrow,\Leftrightarrow 若首字母不大写则变成 $\rightarrow$,$\leftarrow$,$\leftrightarrow$。 另外,$\to$ 和 $\gets$ 也可分别用 \to 和 \gets 得到。 长的左推出、右推出、等价于:$\Longrightarrow$,$\Longleftarrow$,$\Longleftrightarrow$ \Longrightarrow,\Longleftarrow,\Longleftrightarrow 也有类似的首字母不大写的情况。 另外,还有简写: \iff 得 $\iff $,\riff 得 $\riff$,\liff 得 $\liff$ 注意,后面两个是本论坛自定义的,推荐使用这三个简写。 大小关系:$\ge$,$\le$,$\ne$,$\equiv$,$\approx$ \ge,\le,\ne,\equiv,\approx 对于前两个,后加 qslant 则变成更好看的 $\geqslant$,$\leqslant$ 相似(或等价量)、全等:$\sim$,$\cong$ \sim,\cong 导数:$f'(x)$,$f''(x)$,$f'''(x)$ f'(x),f''(x),f'''(x) 注意:撇 ' 由右单引号键打出(台式机键盘在中括号下面,必须要在纯英文状态下输入),二阶就打两撇,而不是打双引号,三阶就三撇,如此类推。 常用希腊字母: $\pi$,$\alpha$,$\beta$,$\gamma$,$\theta$,$\rho$, $\lambda$,$\mu$,$\Delta$,$\xi$,$\omega$, $\phi$($\varphi$),$\epsilon$($\varepsilon$) \pi,\alpha,\beta,\gamma,\theta,\rho, \lambda,\mu,\Delta,\xi,\omega, \phi(\varphi),\epsilon(\varepsilon) 三角函数、度:$\sin x$,$\cos^2x$,$\tan30^\circ$,$\cot^245^\circ$ \sin x,\cos^2x,\tan30^\circ,\cot^245^\circ 有些空格能省,有些不能,自己理解。 三角形、圆、角、垂直:$\triangle$,$\odot$,$\angle$,$\perp$ \triangle,\odot,\angle,\perp 注意:不要用 $\Delta$(\Delta) 来表示三角形,判别式才是 $\Delta$ 点乘、叉乘、除以:$a\cdot b$,$1\times2$,$355\div113$ a\cdot b,1\times2,355\div113 存在、任意、非、或、且:$\exists$,$\forall$,$\neg$,$\vee$,$\wedge$ \exists,\forall,\neg(或 \lnot),\vee(或 \lor),\wedge(或 \land) 补集、空集:$\complement$,$\varnothing$ \complement,\varnothing 空集不推荐用那个 \emptyset(效果:$\emptyset$)更不应该用 \phi(效果:$\phi$) 并、交、包含、真包含:$\cup$,$\cap$,$\subseteq$,$\supseteq$,$\subsetneqq$,$\supsetneqq$ \cup,\cap,\subseteq,\supseteq,\subsetneqq,\supsetneqq 如果习惯 $\subset$,$\supset$ 的就用 \subset,\supset 属于、不属于:$\in$,$\notin$ \in,\notin 另外,还有 \ni 得到 $\ni$ 组合数:${n\choose m}$ {n\choose m} 两边花括号不要少,注意空格。 分段函数:$f(x)=\begin{cases} x+1 & x>0 \\ 1-x & x<0 \end{cases}$ 以前用 \cases,后来发现有点问题,现改用cases环境: \(\verb"$f(x)=\begin{cases}"\) \(\verb"x+1 & x>0 \\"\) \(\verb"1-x & x<0 \end{cases}$"\) 向量:$\vec a,\overrightarrow{a},\overrightarrow{AB}$ \vec a,\overrightarrow{a},\overrightarrow{AB} 其中 \vec 只适用于单个字母的向量。 另外,为简化输入,本论坛已自定义了 \vv 命令,只要输入 \vv a,\vv{AB} 即得 $\vv a$,$\vv{AB}$,这里建议统一使用它。 上下括号:$n=\underbrace{1+1+\cdots+1}_{n~个}=\overbrace{1+1+\cdots+1}^{n~个}$ n=\underbrace{1+1+\cdots+1}_{n~个}=\overbrace{1+1+\cdots+1}^{n~个} 那个 ~ 是为了在 $n$ 和“个”字之间隔开一小距离,此乃中英混排的习惯
kuing 3# 2011-9-26 10:34
环境 环境是指 \(\verb"\begin{...} ... \end{...}"\) 这样的东西,环境在真 LaTeX 中不一定要用来打公式,但这里我们只讲跟公式输入有关的常用环境。 多行公式输入环境 最常用的:align 或 align* 或 aligned 第一个会对每行公式自动编号,第二个则不编号,第三个属于“次环境”暂时不介绍(其实是不知怎么说)。 例:如下代码 \(\verb"\begin{align}"\) \(\verb"f(x) & = ax^2+bx+c\\"\) \(\verb"& = a(x-x_1)(x-x_2)\\"\) \(\verb"& = \cdots"\) \(\verb"\end{align}"\) 将显示 \begin{align} f(x) & = ax^2+bx+c\\ & = a(x-x_1)(x-x_2)\\ & = \cdots \end{align} 注:在 & 处对齐,\\ 为换行,最后一行后面不必再换行。 如果不想某些编号,可以在该行的公式后 \\ 前加 \notag;如果想引用编号,要先在该行公式中加入标签 \(\verb"\label{...}"\) 然后引用时用 \(\verb"\eqref{...}"\),花号里可填任意的英文或数字(冒号什么的好像也可以,但建议不要用别的符号以免出问题)。下面也演示一下: 代码: \(\verb"\begin{align}"\) \(\verb"f(x) & = ax^2+bx+c \notag \\"\) \(\verb"& = a(x-x_1)(x-x_2) \label{asdf01234} \\"\) \(\verb"& = \cdots"\) \(\verb"\end{align}"\) 将显示 \begin{align} f(x) & = ax^2+bx+c \notag \\ & = a(x-x_1)(x-x_2) \label{asdf01234} \\ & = \cdots \end{align} 可以看到,第一行没编号了,现在要引用第二行的编号,只要输入 \(\verb"\eqref{asdf01234}"\) 即得 \eqref{asdf01234}。 这种引用的好处在于那些号码是自动编的,也就是说即使前面突然增加或减少一些公式的编号,编号数字变化时引用也跟着自动变更,不需要手动去改那数字。而且还可以点击跳转,大家不妨试试。 此外,单个行间公式需编号时,可以用 equation 环境,引用方法同上。 \(\verb"\begin{equation}\label{csineq}"\) \(\verb"\left(\sum_{k=1}^na_kb_k\right)^2\le\left(\sum_{k=1}^na_k^2\right)\left("\) \(\verb"\sum_{k=1}^nb_k^2\right)."\) \(\verb"\end{equation}"\) 显示 \begin{equation}\label{csineq} \left(\sum_{k=1}^na_kb_k\right)^2\le\left(\sum_{k=1}^na_k^2\right)\left(\sum_{k=1}^nb_k^2\right). \end{equation} 用 \(\verb"\eqref{csineq}"\) 来引用:式 \eqref{csineq} 称为 Cauchy-Schwarz 不等式(不知有没有拼错,我总是不记得这些极难记的英文名字)。 多行公式全部居中:gather 或 gather* 或 gathered 跟 align 基本类似,但是全部居中对齐,所以里面无需用 & 。 例:如下代码 \(\verb"\begin{gather}"\) \(\verb"f(x) = ax^2+bx+c+00\\"\) \(\verb"= a(x-x_1)(x-x_2)\\"\) \(\verb"= \cdots"\) \(\verb"\end{gather}"\) 将显示 \begin{gather} f(x) = ax^2+bx+c+00\\ = a(x-x_1)(x-x_2)\\ = \cdots \end{gather} 矩阵环境系列 matrix: 代码: \(\verb"\begin{matrix}"\) \(\verb"k & u \\"\) \(\verb"i & n"\) \(\verb"\end{matrix}"\) 显示: \begin{matrix} k & u \\ i & n \end{matrix} 将 matrix 改成 bmatrix、pmatrix、vmatrix、Vmatrix 分别得不同的矩阵显示,如下表: \begin{array}{|c|c|c|c|} \hline \text{bmatrix} & \text{pmatrix} & \text{vmatrix} & \text{Vmatrix}\\ \hline \begin{bmatrix} k & u \\ i & n \end{bmatrix} & \begin{pmatrix} k & u \\ i & n \end{pmatrix} & \begin{vmatrix} k & u \\ i & n \end{vmatrix} & \begin{Vmatrix} k & u \\ i & n \end{Vmatrix} \\ \hline \end{array} 咦?你可能会发现,这个表是怎么做出来的?这里我用的是 array 环境,这也是一个很常用的环境。 随便来个简单点的 \(\verb"\begin{array}{|l|c|r|r|}"\) \(\verb"\hline"\) \(\verb"kui & ng & oh & f... \\"\) \(\verb"\hline"\) \(\verb"asdfadsf & 1234 & 1234 & 45678 \\"\) \(\verb"\hline"\) \(\verb"\end{array}"\) 显示 \begin{array}{|l|c|r|r|} \hline kui & ng & oh & f... \\ \hline asdfadsf & 1234 & 1234 & 45678 \\ \hline \end{array} 关于 array 环境的更详细使用方法请参考任意一本 LaTeX 教程。 注:tabular 环境这里暂时用不了。 再来一个利用 array 输入的分块矩阵: \(\verb"\[\left(\begin{array}{c|cc}"\) \(\verb"1&2&3\\"\) \(\verb"\hline"\) \(\verb"4&5&6\\"\) \(\verb"\end{array}\right)\]"\) 显示 \[\left(\begin{array}{c|cc} 1&2&3\\ \hline 4&5&6\\ \end{array}\right)\]
kuing 4# 2011-9-26 10:47
注意事项 可以的话,公式中不要使用 Discuz! 代码,容易造成公式不识别。如需对公式加粗或加颜色,可用 latex 代码去加,比如 kkkk\color{red}{uin}gggg 显示 $kkkk\color{red}{uin}gggg$,详情参考:http://kkkkuingggg.5d6d.com/thread-75-1-1.html 第一次来本站看公式时显示可能会慢一点,如果出不来可以刷新一下,load 完第一次之后就会快了,网速慢的话显示可能也会慢些,总来上来看目前速度还算可以。 由于初步搞,又不懂 HTML、javascript 之类的,暂时还没太完善,所以在打公式代码的时候不要连续输入多个空格,否则真的会空开,原因这里的 MathJax 并不会自动将多个空格变为一个空格,如果哪位有办法实现这一点,麻烦告之 尽管自从 mathtype6.0 及其打后的版本都有转换输出 LaTeX 代码的功能,但这里还是建议能手打就手打,因为从 mathtype 输出的代码中经常有多余的东西,未必能得到你想要的效果。 对于一些公式比较多的贴,可能由打开页面到显示完毕需要一点点时间,页面也会有点跳动,故此遇到此情形时请稍等几秒钟等显示完毕稳定后再进行操作。
kuing 5# 2011-9-26 14:01
一些细节 关于括号: 建议在有一定高度的公式内使用 \left 和 \right 进行自动调整适当的括号高度。 例如,在行间公式的情况下,我们要输入 1/2 到 1 的开区间,如果只用普通括号输入 (\frac12,1),则得到\[(\frac12,1)\]我们建议用 \left 和 \right 将括号调大到适当大小,改为输入 \left(\frac12,1\right),得到\[\left(\frac12,1\right)\]中括号、大括号同理。 而如果觉得 \left 和 \right 出来的效果不满意,还可以自行选择有固定大小的括号,以左小括号为例,由大到小为 (,\big(,\Big(,\bigg(,\Bigg( 分别套用这些括号出来的效果分别为\[(\frac12,1),\big(\frac12,1\big),\Big(\frac12,1\Big),\bigg(\frac12,1\bigg),\Bigg(\frac12,1\Bigg)\]如果要更细节专业一点,左右还应该要区分,左边要加 l,右边要加 r,即 \Bigl(\frac12,1\Bigr) 等,效果\[(\frac12,1),\bigl(\frac12,1\bigr),\Bigl(\frac12,1\Bigr),\biggl(\frac12,1\biggr),\Biggl(\frac12,1\Biggr)\]具体原因参考 TheTeXBook。 关于公式中的省略号: 在公式中,建议用 \cdots、\ldots 等等命令来输入省略号,而不要直接打三个点 ... 甚至随手打 n 个点。 对比一下: \(\verb"$x_1+x_2+...+x_n$"\) 得 $x_1+x_2+...+x_n$ \(\verb"$x_1+x_2+\cdots+x_n$"\) 则得 $x_1+x_2+\cdots+x_n$ 至于什么时候用 \cdots 什么时候用 \ldots ,按照 TheTeXBook 里的标准,二元运算符或关系符之间的省略号用 \cdots,其余用 \ldots。 例如 \(\verb"\[f(x_1,x_2,\ldots,x_n)=x_1+x_2+\cdots+x_n\]"\),效果 \[f(x_1,x_2,\ldots,x_n)=x_1+x_2+\cdots+x_n\]但这似乎与国内普遍的使用方法有所不同,国内似乎全都用 \cdots 呃。 建议中文与行内公式之间空一格会好看一点。(在真 LaTeX 里一般用 ~ ,这里用不了) 关于特定数集、微积分中的 d: 建议对实数集、自然数集之类这些特定的数集使用 \mathbb 或 \mathbf 之类字体来表示,以区别普通字母。 例如,用 x\in\mathbb{R} 和 x\in\mathbf{R} 分别得到\[x\in\mathbb{R}  和  x\in\mathbf{R}\]微积分中的 d 建议用 \mathrm{d} 变成直立体,以区别普通字母。 例如二楼中微积分那段按上述改后得\[\int_{a}^{b}f(x)\mathrm{d}x\] (2012-8-27注:本论坛已设计了自定义命令 \mbb 和 \rmd,用法是 \mbb{R},\mbb{N}^+,\rmd{x},\rmd{x^2} 等)
kuing 6# 2011-9-26 14:44
一些相关贴子汇总: 动态图演示公式输入及其他: http://kkkkuingggg.5d6d.net/thread-33-1-1.html 关于查看公式代码: http://kkkkuingggg.5d6d.net/thread-769-1-1.html 已默认设置成双击放大公式 http://kkkkuingggg.5d6d.net/thread-770-1-1.html 公式中加颜色、加粗、加下划线、调大小等操作汇总: http://kkkkuingggg.5d6d.net/thread-75-1-1.html 本论坛的自定义命令: http://kkkkuingggg.5d6d.net/thread-1030-1-1.html
hhhzh7241hzh 7# 2011-9-26 16:09
本帖最后由 hhhzh7241hzh 于 2011-9-26 16:16 编辑 看看行不行: $f(x_1,x_2,\ldots,x_n)=x_1+x_2+\cdots+x_n$
kuing 8# 2011-9-26 16:11
看看行不行: \sum_{k=1}^{n}f(k) hhhzh7241hzh 发表于 2011-9-26 16:09 两边要加美元符号才显示啊 如果直接复制下来就能显示的话,那我那里也全变成公式啦 美元符 f(x...... 美元符 加个 \s 干嘛呢
hhhzh7241hzh 9# 2011-9-26 16:18
本帖最后由 hhhzh7241hzh 于 2011-9-26 16:20 编辑 8# kuing 这下对了,谢谢!以后慢慢学呗。:) 还有个问题,就是网页更新时跳得挺厉害的,有没有改进的方法?
kuing 10# 2011-9-26 16:31
8# kuing 这下对了,谢谢!以后慢慢学呗。:) 还有个问题,就是网页更新时跳得挺厉害的,有没有改进的方法? hhhzh7241hzh 发表于 2011-9-26 16:18 这个贴集公式多,所以出现这种情况在所难免了。。。等待一会等公式都显示完就好了。。。普通贴子公式不太多时应该不会跳这么厉害
isea 11# 2011-9-26 22:47
经测试 最后一行,向量,\vec a 即可
kuing 12# 2011-9-26 22:52
经测试 最后一行,向量,\vec a 即可 isea 发表于 2011-9-26 22:47 嗯,不过这个只适用于单个字母的向量,比较小
wenshengli 13# 2011-9-27 14:54
本帖最后由 wenshengli 于 2011-9-28 14:15 编辑 证明:$\forall x\in(0,+\infty)$,都有$\displaystyle lnx>\frac{1}{e^x}-\frac{2}{ex}$ 按kuing说法,再弄一次: 证明:$\forall x\in(0,+\infty)$,都有$\ \ln x>\dfrac{1}{e^x}-\dfrac{2}{ex}$
kuing 14# 2011-9-27 16:52
证明:$\forall x\in(0,+\infty)$,都有$\displaystyle lnx>\frac{1}{e^x}-\frac{2}{ex}$ wenshengli 发表于 2011-9-27 14:54 嗯,基本可以,只是有一处,lnx 可以改成 \ln x ,这样 ln 就会直立起来,但记得 ln 与 x 之间要有空格。 还有,如果觉得加 \displaystyle 太麻烦,可以将 \frac 换成 \dfrac,也可以干脆换成行间公式去写,如果你不介意独行居中的话。
wenshengli 15# 2011-9-28 14:05
谢谢,四年前上学时用TEX,现在忘的差不多了。
cgj1982 16# 2011-10-17 15:45
10# kuing 实验:$a_n=n^2+2$
kuing 17# 2011-10-18 15:57
16# cgj1982 可以了
魔幻水果 18# 2011-10-18 21:16
本帖最后由 魔幻水果 于 2011-10-18 21:20 编辑 我来试试:$\sum_{k=1}^{n}f(k)$              $ \displaystyle\sum_{k=1}^{n}f(k)$
魔幻水果 19# 2011-10-18 21:22
ok,不容易啊,三试成功。
kuing 20# 2011-10-19 01:41
19# 魔幻水果 慢慢来哈。 一开始可以输入一些简单的。 较复杂的那些,或公式高度比较高的那些,建议用行间公式模式,这样不用加\displaystyle
thread-9-2-1.html:
tianxiahao 21# 2011-10-24 18:15
$8x^2=0$
第一章 22# 2011-11-6 19:29
$\frac{a+b}{2}\ge \sqrt{ab}$
第一章 23# 2011-11-6 20:03
我来试试 $\sum_{k=1}^{n}f(k)$
mathjohnfh 24# 2012-1-20 18:16
本帖最后由 mathjohnfh 于 2012-1-20 19:20 编辑 ${{n}\choose{m}}$ $\prod_{k=1}^{n}f(k)$ $\sum_{k=1}^{n}f(k)$ $$\displaystyle\in$$
kuing 25# 2012-1-20 20:55
24# mathjohnfh 用两边各两个美元符号就不必加 \displaystyle 了
海盗船长 26# 2012-1-23 22:18
\[\sqrt{1+\sqrt{1+\sqrt{1+\sqrt{1+\sqrt{1+\sqrt{1+\sqrt{1+\sqrt{1+\sqrt{1+\sqrt{1+\sqrt{1+\sqrt{1+\sqrt{1}}}}}}}}}}}}}\]
kuing 27# 2012-1-23 22:34
26# 海盗船长
海盗船长 28# 2012-1-27 12:07
$\sqrt{1+\sqrt{1+\sqrt{1+\sqrt{1+\sqrt{1+\sqrt{1+\sqrt{1+\sqrt{1+\sqrt{1+\sqrt{1+\sqrt{1+\sqrt{1+\sqrt{1}}}}}}}}}}}}}$
海盗船长 29# 2012-1-27 14:39
呵呵开始几个根号是斜的,后来就成直的了
linzhuqin 30# 2012-3-22 16:17
$2Al(OH)_3 \mathrel{\displaystyle\mathop{=\!=\!=\!=}^{加热}} Al2_O_3+3H_2O$
linzhuqin 31# 2012-3-22 16:18
本帖最后由 linzhuqin 于 2012-3-25 09:27 编辑 $2Al(OH)_3 \mathrel{\displaystyle\mathop{=\!=\!=\!=}^{加热} }Al2_O_3+3H_2O$ $2AlOH)_3 \mathrel{\displaystyle\mathop{=\!=\!=\!=}^{高温} }Al_2O_3+3H_2O$ 上下语法都一样,为什么上面的不显示?错在哪里? $x^x$
kuing 32# 2012-3-22 17:06
31# linzhuqin 错在 “ Al2_O_3 ”
fredjhon 33# 2012-4-2 09:03
$\fracax^2+bx+c=0$
fredjhon 34# 2012-4-3 12:49
我也来试下 $lin_{x\to\infty}\left(\sqrt{x+\sqrt{x+\sqrt{x}}
fredjhon 35# 2012-4-3 12:50
$lin_{x\to\infty}\left(\sqrt{x+\sqrt{x+\sqrt{x}}$
kuing 36# 2012-4-3 12:58
$lin_{x\to\infty}\left(\sqrt{x+\sqrt{x+\sqrt{x}}$ fredjhon 发表于 2012-4-3 12:50 极限是 \lim \left( 多余 右边缺少花括号 } 写正确是 $\lim_{x\to\infty}\sqrt{x+\sqrt{x+\sqrt{x}}}$
fredjhon 37# 2012-4-3 13:16
\lim {x\rightarrow+\infty}(\sqrt{x+\sqrt{x+\sqrt{x}}}-\sqrt{x+\sqrt{x}}\right)
fredjhon 38# 2012-4-3 13:18
这个是从Web Equation上抄的代码,怎么也不对啊。希望大侠能指正
kuing 39# 2012-4-3 13:19
38# fredjhon 你两边不加美元符号他怎么会显示呢
fredjhon 40# 2012-4-3 13:24
lim_{x\to\infty}(\sqrt{x+\sqrt{x+\sqrt{x}}}-\sqrt{x+\sqrt{x}})\sqrt{x}
thread-9-3-1.html:
fredjhon 41# 2012-4-3 13:25
$lim_{x\to\infty}(\sqrt{x+\sqrt{x+\sqrt{x}}}-\sqrt{x+\sqrt{x}})\sqrt{x}$
fredjhon 42# 2012-4-3 13:26
谢谢这位大侠了,明白了
ytxiehua2008 43# 2012-5-18 16:30
本帖最后由 ytxiehua2008 于 2012-5-18 16:31 编辑 $\frac{2}{\frac{1}{a}+\frac{1}{b}}\le\sqrt{ab}\le\frac{a+b}{2}\le\sqrt{\frac{a^2+b^2}{2}}$ $x_{1,2}=\frac{-b\pm\sqrt{b^2-4ac}}{2a}$
yayaweha 44# 2012-6-7 20:59
由条件知 \begin{align*} \frac{|\vv a|}{|\vv b|}\cos\theta &= \frac m2,\\ \frac{|\vv b|}{|\vv a|}\cos\theta &= \frac n2, \end{align*} 其中 $m$, $n \in \mathbb Z$,两式相乘得 \[ \cos^2\theta =\frac{mn}4, \] 又由角的范围知 \[ \cos^2\theta \in \left(0,\frac12\right),\] 从而易得 $m=n=1$ 或 $m=n=-1$,后者显然余去,故D。
yayaweha 45# 2012-6-7 21:00
与文数类似,用回前面所设,得到 \[ \cos^2\theta =\frac{mn}4, \] 此处角的范围不同,有 \[ \cos^2\theta \in \left(\frac12,1\right),\] 从而易得 $m=1$, $n=3$ 或 $m=3$, $n=1$ 或 $m=-1$, $n=-3$ 或 $m=-3$, $n=-1$,显然只有 $m=3$, $n=1$ 符合,故C。   在人教论坛看不懂 在这看
kuing 46# 2012-6-8 00:21
回楼上:我开了个贴子专门将那里的复制过来这里显示:http://kkkkuingggg.5d6d.net/thread-500-1-1.html
yayaweha 47# 2012-6-23 12:50
\frac{\ln a+\ln b}{2}\geqslant \frac{b}{a+b}\ln a+\frac{a}{a+b}\ln b
yayaweha 48# 2012-6-23 12:51
$\frac{\ln a+\ln b}{2}\geqslant \frac{b}{a+b}\ln a+\frac{a}{a+b}\ln b$
急性子小蜗牛 49# 2012-6-23 20:40
$x^2+y^2=z^2$
老樊 50# 2012-6-29 18:11
本帖最后由 老樊 于 2012-6-29 18:13 编辑 我也试试,$$f(x)=x^2$$ $$a_n=2^n$$
yayaweha 51# 2012-7-1 10:43
\[ |PA|^2+|PB|^2+|PC|^2+|PD|^2 = 4|PO|^2+|OA|^2+|OB|^2+|OC|^2+|OD|^2, \]
老樊 52# 2012-7-8 10:04
本帖最后由 老樊 于 2012-7-8 10:06 编辑 1# kuing \[x^2+y^2=1\] \[f(x)=\frac{e^x}{x}\]
老樊 53# 2012-7-8 11:58
本帖最后由 老樊 于 2012-7-8 15:29 编辑 \[2AlOH)_3 \mathrel{\displaystyle\mathop{=\!=}^{高温} }Al_2O_3+3H_2O\] \[f(x)=\cases{{x+1} & {x>0}\\{1-x} & {x<0}}\]
kuing 54# 2012-7-8 13:42
53# 老樊 打化学式还是比较麻烦的事,真LaTeX里似乎要用到某些宏包才简单些,具体不了解没输过化学的东东,忘得差不多了。
isea 55# 2012-7-24 13:46
子集 \subseteq   \supseteq   $\rightarrow$ $\subseteq$   $\supseteq$ 真子集 \subsetneqq  \supsetneqq $\rightarrow$ $\subsetneqq$  $\supsetneqq$
kuing 56# 2012-7-24 13:57
$\LaTeX$数学公式表
★_/ka_☆ 57# 2012-7-24 14:49
本帖最后由 ★_/ka_☆ 于 2012-7-24 14:50 编辑 $\int_\sec^3xdx$
kuing 58# 2012-7-24 14:55
57# ★_/ka_☆ 不需要加 _ $\int\sec^3xdx$
距_离 59# 2012-7-29 23:22
想要输入多行的公式,居中并且每行需要和等号对齐,如:a=b+c+d+1                                                                           =c+d+c+1                                                                           =1+d+b+c                                                                           =e 应该如何控制???
kuing 60# 2012-7-30 01:49
想要输入多行的公式,居中并且每行需要和等号对齐,如:a=b+c+d+1                                                                           =c+d+c+1                                                          ... 距_离 发表于 2012-7-29 23:22 对着 3楼 的第一个公式,右键(如图所示那样),可以看到原代码,仿着试试。
thread-9-4-1.html:
Candice 61# 2012-8-1 22:25
可以复制吗?
kuing 62# 2012-8-1 22:29
61# Candice 右击 - Show Math As - TeX Commands,可以查看代码,即可复制代码。
叶剑飞Victor 63# 2012-8-10 23:57
本帖最后由 叶剑飞Victor 于 2012-8-11 14:39 编辑 53# 老樊 $$ 2\textrm{Al(OH)}_3 \mathrel{\mathop{=\!\!\!\!=\!\!\!\!=\!\!\!\!=\!\!\!\!=\!\!\!\!=\!\!\!\!=}^{\textrm{高温}}} \textrm{Al}_2\textrm{O}_3+3\textrm{H}_2\textrm{O} $$
v6mm131 64# 2012-8-25 19:34
本帖最后由 v6mm131 于 2012-8-25 19:42 编辑 \ln x\ 复制代码
sign008 65# 2012-10-28 12:54
\(\sum_{i=0}^{c-1}C_{num[i]}^2=\sum_{i=0}^{c-1}\frac{1}{2}num[i]\cdot (num[i]-1)\) 试一下看看
Min 66# 2012-12-3 10:36
本帖最后由 Min 于 2012-12-3 10:44 编辑 好难的样子。。。试试。。。 $\int_{0}^{\frac{\pi}{2}}(\int_{x}^{\frac{\pi}{2}}\frac{sint}{t}dt)dx$
realnumber 67# 2012-12-3 13:14
卸掉mathplay后>,<显示正常了,不再是?了
kuing 68# 2012-12-3 14:08
67# realnumber mathplayer? 竟然有冲突?
kuing 69# 2012-12-3 14:26
好难的样子。。。试试。。。 $\int_{0}^{\frac{\pi}{2}}(\int_{x}^{\frac{\pi}{2}}\frac{sint}{t}dt)dx$ Min 发表于 2012-12-3 10:36 \[\int_0^{\frac\pi2}\left(\int_x^{\frac\pi2}\frac{\sin t}t\rmd t\right)\rmd x\] 怎么做?
realnumber 70# 2012-12-3 15:13
今天才发现的,我就奇怪学校的电脑老是显示问题(以前装了mathplayer),而回家一看又好好的,试着卸掉,ok了
Gauss门徒 71# 2012-12-15 06:21
本帖最后由 Gauss门徒 于 2012-12-15 06:24 编辑 嗯嗯,试试 \[\sum_{i=1}^{n}a_{i}b_{i}\le (\sum_{i=1}^{n}a_{i}^{p})^{\frac{1}{p}}(\sum_{i=1}^{n}b_{i}^{q})^{\frac{1}{q}}\]
kuing 72# 2012-12-15 10:35
71# Gauss门徒 有的花括号可以省去。 括号可以试试加 \left 和 \right
pengcheng1130 73# 2013-1-6 17:28
$S_n^{123}=S_{n-1}^2-32$
kuing 74# 2013-1-8 22:54
增加了一个置顶
yes94 75# 2013-1-24 19:16
在k12看到这个帖子的输入 (1-\frac{1}{4})(1-\frac{1}{4^2})(1-\frac{1}{4^3})...(1-\frac{1}{4^n})>1-\frac{1}{4}-\frac{1}{4^2}-...\frac{1}{4^n}>1-\frac{1}{3}=\frac{2}{3} 其中省略号没用那个是么英文字母呢? 试一试能否显示成功: $(1-\frac{1}{4})(1-\frac{1}{4^2})(1-\frac{1}{4^3})...(1-\frac{1}{4^n})>1-\frac{1}{4}-\frac{1}{4^2}-...\frac{1}{4^n}>1-\frac{1}{3}=\frac{2}{3}$
yes94 76# 2013-1-24 19:19
75# yes94 再试一试: (1-\frac14)(1-\frac14^2)(1-\frac14^3)...(1-\frac14^n)>1-\frac14-\frac14^2-...\frac14^n>1-\frac13=\frac23 $(1-\frac14)(1-\frac14^2)(1-\frac14^3)...(1-\frac14^n)>1-\frac14-\frac14^2-...\frac14^n>1-\frac13=\frac23$
yes94 77# 2013-1-24 19:23
76# yes94 晕! (1-\dfrac14)(1-\dfrac1{4^2})(1-\dfrac1{4^3})...(1-\dfrac1{4^n})>1-\dfrac1{4}-\dfrac1{4^2}-...\dfrac1{4^n}>1-\dfrac13=\dfrac23 显示: $(1-\dfrac14)(1-\dfrac1{4^2})(1-\dfrac1{4^3})...(1-\dfrac1{4^n})>1-\dfrac1{4}-\dfrac1{4^2}-...\dfrac1{4^n}>1-\dfrac13=\dfrac23$
yes94 78# 2013-1-25 19:40
本帖最后由 yes94 于 2013-1-25 19:50 编辑 三阶矩阵如何输入? \begin{pmatrix} 3 & 2 & 1 \\ 1 & -1 & 1 \\ 1 & 1 & -1 \end{pmatrix} 已知空间的一个基底为{$\vec a ,\vec b,\vec c$},$\vec p=3\vec a+2\vec b+\vec c$,$\vec m=\vec a-\vec b+\vec c$,$\vec n=\vec a+\vec b-\vec c$, 试判断$\vec p、\vec m、\vec n$是否共面,此题是水题,练习代码输入 谢谢版主!已修改
yes94 79# 2013-1-27 20:11
对\varphi 表示的效果$\varphi$不太满意 ,有没有上封口的$\varphi$? 怎么输入?
kuing 80# 2013-1-27 20:28
79# yes94 更新了。
thread-9-5-1.html:
yes94 81# 2013-1-27 20:57
本帖最后由 yes94 于 2013-1-27 21:11 编辑 谢谢楼上! \ne,\equiv分别表示$\ne$,$\equiv$, 但那个“不同于”,“不恒等于”怎么表示? \nequiv?$\nequiv $ $\not\equiv $ 谢谢楼下!
kuing 82# 2013-1-27 21:00
二元关系符前面加 \not 一般都可以变成其否定。 \not\equiv  $\not\equiv$
kuing 83# 2013-1-27 22:30
82# kuing 对了,同余那里我更新了一下。
kuing 84# 2013-1-27 23:39
话说,我还以为你们会比较喜欢这种 $\varphi$,所以当时只写了这个…… 顺便再加上了 $\epsilon$($\varepsilon$)
yayaweha 85# 2013-2-8 14:20
什么叫做使用‘环境’,另外什么是环境
kuing 86# 2013-2-8 14:23
85# yayaweha 3# ……
hongxian 87# 2013-3-8 10:40
本帖最后由 hongxian 于 2013-3-8 10:41 编辑 总感觉极限符号$\lim_{x \to \infty}f(x)$和书本上的不太一样,能不能实现这种 效果?
kuing 88# 2013-3-8 11:40
\[\lim_{x \to \infty}f(x)\] 认真看行内、行间公式的区别
hongxian 89# 2013-3-8 13:14
88# kuing 已经解决,谢谢!$\displaystyle\lim_{x \to \infty}f(x)$
pengcheng1130 90# 2013-3-30 20:59
设集合$P_{n}=\{1,2,\ldots,n\},n\inN^*
pengcheng1130 91# 2013-3-30 21:00
设集合$P_{n}=\{1,2,\ldots,n\},n\inN^{*}$
kuing 92# 2013-3-30 21:17
91# pengcheng1130 如果命令接下来的是英文字母,必须空开。 比如你的 \inN 就一定要写成 \in N ,否则它会将 \inN 看成一个命令,以至于报错
pengcheng1130 93# 2013-3-30 21:37
设集合$P_{n}=\{1,2,\cdots,n\},n\in N^{*}$,记$f(n)$为同时满足下列 条件的集合$A$的个数: (1)$A\subseteq P_{n}$; (2)若$x\in A$,则$2x\notin A$; (3)若$x\in complement_{P_n}A$,则$2x\notin \complement_{P_n}A$. 求$f(n)$的解析式(用$n$表示
pengcheng1130 94# 2013-3-30 21:38
设集合$P_{n}=\{1,2,\cdots,n\},n\in N^{*}$,记$f(n)$为同时满足下列 条件的集合$A$的个数: (1)$A\subseteq P_{n}$; (2)若$x\in A$,则$2x\notin A$; (3)若$x\in \complement_{P_n}A$,则$2x\notin \complement_{P_n}A$. 求$f(n)$的解析式(用$n$表示
kuing 95# 2013-3-30 21:40
94# pengcheng1130 也可以用http://kkkkuingggg.5d6d.net/thread-1204-1-1.html中的草稿本来测试公式
pengcheng1130 96# 2013-3-30 23:08
我发的那个补集那个符号,感觉不太齐,有什么方法处理?
pengcheng1130 97# 2013-3-30 23:10
我们学习\[a^2+b^2=c^2\]这叫勾股定理。
kuing 98# 2013-3-30 23:14
96# pengcheng1130 补集感觉还好啊
pengcheng1130 99# 2013-4-2 10:41
$\frac34$,$\displaystyle\frac34$
kuing 100# 2013-4-2 11:01
99# pengcheng1130 只是分式,还可以 \dfrac34
thread-9-6-1.html:
yes94 101# 2013-4-2 17:56
$\frac34$,$\displaystyle\frac34$ pengcheng1130 发表于 2013-4-2 10:41 用这里的草稿本写公式很方便的:http://kkkkuingggg.5d6d.net/thread-1204-1-1.html 然后在复制到本论坛粘贴,就OK啦
yes94 102# 2013-4-4 18:09
k版,公式里想搞粗体怎么办? 准备在word里用latex公式的粗体
kuing 103# 2013-4-4 18:31
102# yes94 4# 或 6# 均有相关链接
kuing 104# 2013-4-4 19:29
2013-3-30, 2#表格少量更新
isea 105# 2013-4-8 19:58
k版,公式里想搞粗体怎么办? 准备在word里用latex公式的粗体 yes94 发表于 2013-4-4 18:09 如果用的Aurora插件,只需要在电脑上看上去醒目一些,只需把图中的选项勾上即可。不影响打印。即打印不是粗体。 或者在首先加\boldmath (在第一个\$前),这样只是所有的公式一次性加粗。
李斌斌755 106# 2013-4-14 23:47
试试 $c^2=a^2+b^2$
李斌斌755 107# 2013-4-14 23:54
本帖最后由 李斌斌755 于 2013-4-14 23:55 编辑 $sqrt{a}$
kuing 108# 2013-4-14 23:58
107# 李斌斌755 用草稿本试吧。
李斌斌755 109# 2013-4-15 00:03
好的
hongxian 110# 2013-5-15 06:46
$\pounds$符号能不能用?看样子不能!
kuing 111# 2013-5-15 13:36
110# hongxian 直接打 £
hongxian 112# 2013-5-15 15:40
本帖最后由 hongxian 于 2013-5-15 15:53 编辑 111# kuing 在word中的**符号中找到  £  ;换成symbol字体变成了$\le$了。 £
hongxian 113# 2013-5-15 15:43
112# hongxian word中复制过来,居然成了这样!
kuing 114# 2013-5-15 15:43
££ 我从word复制过来没问题……
kuing 115# 2013-5-15 15:45
ALT+163 也可以 £
hongxian 116# 2013-5-15 15:45
114# kuing 你用的是什么字体?
kuing 117# 2013-5-15 15:47
我就打开word,什么都没设置,然后点击 cha入 - 符号,然后找到这个符号,点 cha入,然后复制过来就是 £
hongxian 118# 2013-5-15 15:52
117# kuing 有可能和上下文有关我前后加空格后就ok了
hongxian 119# 2013-5-15 15:55
本帖最后由 hongxian 于 2013-5-15 15:57 编辑 117# kuing 还有不明白的是“cha 入”两个字为什么变成了星号了?
kuing 120# 2013-5-15 15:57
119# hongxian 大概是min感词自动被替换……
thread-9-7-1.html:
叶剑飞Victor 121# 2013-5-25 14:25
120# kuing 扌臿    入
kuing 122# 2013-5-25 14:33
插$\lambda$
thread-90-1-1.html: 又一群里极限题
kuing 1# 2011-10-12 15:55
最近群里时兴极限? \begin{align*} \lim_{x \to 0} \frac{{\sqrt {1 + \tan x} - \sqrt {1 + \sin x} }}{{x\sqrt {1 + \sin ^2 x} - x}}& = \lim_{x \to 0} \frac{{(\tan x - \sin x)(\sqrt {1 + \sin ^2 x} + 1)}}{{x\sin ^2 x(\sqrt {1 + \tan x} + \sqrt {1 + \sin x} )}} \\ &= \lim_{x \to 0} \frac{{1 - \cos x}}{{x\sin x\cos x}} \cdot \lim_{x \to 0} \frac{{\sqrt {1 + \sin ^2 x} + 1}}{{\sqrt {1 + \tan x} + \sqrt {1 + \sin x} }} \\ &= \lim_{x \to 0} \frac{{\sin \frac{x}{2}}}{{x\cos \frac{x}{2}\cos x}} \cdot \lim_{x \to 0} \frac{{\sqrt {1 + \sin ^2 x} + 1}}{{\sqrt {1 + \tan x} + \sqrt {1 + \sin x} }} \\ &= \lim_{x \to 0} \frac{{\sin \frac{x}{2}}}{{\frac{x}{2}}} \cdot \lim_{x \to 0} \frac{1}{{2\cos \frac{x}{2}\cos x}} \cdot \lim_{x \to 0} \frac{{\sqrt {1 + \sin ^2 x} + 1}}{{\sqrt {1 + \tan x} + \sqrt {1 + \sin x} }} \\ &= 1 \cdot \frac{1}{{2 \cdot 1 \cdot 1}} \cdot \frac{{\sqrt 1 + 1}}{{\sqrt 1 + \sqrt 1 }} \\ &= \frac{1}{2} \end{align*}
海盗船长 2# 2011-10-14 20:15
是不是因为很多人都大一了?
kuing 3# 2011-10-14 21:27
2# 海盗船长 呃,不知道
kuing 4# 2012-10-1 20:24
又一道类似,还是来自群的 分母有理化,有 \begin{align*} & \lim_{x\to0}\frac{\tan x-\sin x}{\bigl(\sqrt[3]{1+x^2}-1\bigr)\bigl(\sqrt{1+\sin x}-1\bigr)} \\ ={}&\lim_{x\to0}\frac{(\tan x-\sin x)\bigl(\sqrt{1+\sin x}+1\bigr)\bigl(\sqrt[3]{(1+x^2)^2}+\sqrt[3]{1+x^2}+1\bigr)}{x^2\sin x} \\ ={}&\lim_{x\to0}\frac{(1-\cos x)\bigl(\sqrt{1+\sin x}+1\bigr)\bigl(\sqrt[3]{(1+x^2)^2}+\sqrt[3]{1+x^2}+1\bigr)}{x^2\cos x} \\ ={}&\lim_{x\to0}\left( \frac{\sin \frac x2}{\frac x2} \right)^2\frac{\bigl(\sqrt{1+\sin x}+1\bigr)\bigl(\sqrt[3]{(1+x^2)^2}+\sqrt[3]{1+x^2}+1\bigr)}{2\cos x} \\ ={}&3. \end{align*}
六月的煜 5# 2012-10-25 17:26
本帖最后由 六月的煜 于 2012-10-25 17:57 编辑 话说哪个群? 1. \begin{align*} \lim_{x \to 0} \frac{{\sqrt {1 + \tan x} - \sqrt {1 + \sin x} }}{{x\sqrt {1 + \sin ^2 x} - x}} &=\lim_{x \to 0} \frac{\frac{1}{2\sqrt{\xi}}\sin x(\frac{1}{\cos x}-1)}{x\frac{1}{2}ln(1+\sin^2x)}\\ &=\lim_{x \to 0} \frac{1-\cos x}{x^2}=\frac{1}{2} \end{align*} 2. \begin{align*} \lim_{x\to0}\frac{\tan x-\sin x}{\bigl(\sqrt[3]{1+x^2}-1\bigr)\bigl(\sqrt{1+\sin x}-1\bigr)}&= \lim_{x\to 0}\frac{x(1-\cos x)}{\frac{1}{3}\ln(1+x^2)\frac{1}{2}\ln(1+\sin x)}\\ &=\lim_{x\to 0}\frac{x\frac{x^2}{2}}{\frac{1}{3}x^2\frac{1}{2}x}\\ &=3 \end{align*}
kuing 6# 2012-10-25 17:47
话说哪个群? 六月的煜 发表于 2012-10-25 17:26 人教论坛的数学群,友情链接里能找到其群号 话说我对等价量替换还不太熟,所以习惯用有理化了 PS、那些函数名前面加个 \ 会变正体,正规些。
六月的煜 7# 2012-10-25 18:00
已改正,群已申请,谢谢!
kuing 8# 2012-10-25 18:24
7# 六月的煜 刚才在煮饭,已通过,不客气
thread-900-1-2.html: 求window98的开机和关机音乐(wav格式)
kuing 1# 2012-11-2 19:38
近期一直都在怀旧 求window98的开机和关机音乐(wav格式)
win98 2# 2012-11-5 01:04
本帖最后由 win98 于 2012-11-5 01:57 编辑 不用求了,简直装了win98,于是易得:
win98 3# 2012-11-5 02:03
话说刚才装的那win98各种问题…… 首先是无法识别显示器,可能是因为在笔记本上安装的原因,以至于颜色最多只有16色 然后声卡驱动好像又有问题,以至于没声音^2 上网倒是没问题,上本论坛的公式显示不出来也意料之中,但是竟然连贴子都发不成功,一直提示发贴超时^3 最后看能不能装个QQ,已经预料到装较新版的应该不行,所以整了个2008版的来装,结果也不行,还竟然提示要我装2006版^$\infty$ …… 所以基本上玩不了什么了,怀念完之后还是换回了叉屁。
kuing 4# 2012-11-5 02:10
换回大号,发几个在win98上的截图,包括上述的一些情形,还有最后一个用爪机拍的。 win98桌面 里面有个 word2000,进去后其实没发现跟03有什么区别 能上网,但有点点乱而且左下角总提示有错误 上论坛,公式如无意外地不显示 要我装2006…… 这个只能用爪机拍了
叶剑飞Victor 5# 2012-11-25 22:31
3# win98 安装 Windows 98 之后需要安装显卡驱动程序,才能支持真彩色;还要安装声卡驱动程序,才能播放声音。还有Windows 98的默认浏览器是IE4,可以尝试安装MathPlayer,看看能否显示公式。
pxchg1200 6# 2012-12-18 13:04
4# kuing 目测是Ghost版的win 98,其实你可以用虚拟机试试的,那样不要系统。先装个Virtual Box就Ok,驱动什么的都能搞定。。。
kuing 7# 2012-12-18 14:15
6# pxchg1200 嗯。 有空研究下虚拟机
thread-901-1-6.html: 请教一个三角函数求取值范围的问题
hongxian 1# 2012-11-3 06:46
2.锐角$x$、$y$、$z$满足${{\cos }^{2}}x+{{\cos }^{2}}y+{{\cos }^{2}}z=1$,试求$x+y+z$的取值范围。
kuing 2# 2012-11-3 11:05
当年扯过一下,但没细说http://sq.k12.com.cn/discuz/thread-346574-1-1.html
hongxian 3# 2012-11-3 11:24
2# kuing 居然是个切线法,好!
kuing 4# 2012-11-3 11:30
3# hongxian 没搞切线啊,印象中是切不出来的,后面用的是凹凸性+调整法
kuing 5# 2012-11-3 15:59
上面给出的链接里我没有详细讲,因为我知道要讲很多东东,可见当时很懒。哎,现在其实也懒,不过实在是闲着没事干,下面还是稍讲讲是怎么个“凹凸调整”法。 先给出一个引理(来源于 zhaobin 的半凹半凸定理,此处只是个特例) 若 $f(x)$ 在区间 $[a,b]$ 内下凸,在区间 $[b,c]$ 内上凸,变量 $x$, $y$, $z\in[a,c]$ 满足 $x\leqslant y\leqslant z$ 且 $x+y+z$ 为定值。 则:$f(x)+f(y)+f(z)$ 取最小值时必有 $x=y$ 或 $z=c$;$f(x)+f(y)+f(z)$ 取最大值时必有 $x=a$ 或 $y=z$。 注: 此处使用的凸函数定义是:设 $f(x)$ 在 $D$ 上有定义,若对任意的 $0<\lambda <1$, $a$, $b\in D$ 都有 $f\bigl(\lambda a+(1-\lambda )b\bigr)\leqslant \lambda f(x)+(1-\lambda )f(b)$,则称 $f(x)$ 在 $D$ 上为下凸函数,简称下凸,若不等号反向,则称为上凸。 在此定义下,若 $f(x)$ 二阶可导,则下凸等价于 $f''(x)\geqslant0$,上凸则反号。 接下来的证明将用到以下两个常用性质: 性质一、若 $x$, $y\in D$ 且 $f(x)$ 下凸,则 $f(x)+f(y)\geqslant 2f\bigl((x+y)/2\bigr)$; 性质二、若 $x$, $y\in D$, $x\leqslant y$ 且 $f(x)$ 上凸,设 $h>0$ 使得 $x-h$, $y+h\in D$,则 $f(x)+f(y)\geqslant f(x-h)+f(y+h)$。 性质一由定义令 $\lambda =1/2$ 立得,性质二由以下两式相加即得。 \begin{align*} f(x)&\geqslant \frac{y-x+h}{y-x+2h}f(x-h)+\frac h{y-x+2h}f(y+h),\\ f(y)&\geqslant \frac h{y-x+2h}f(x-h)+\frac{y-x+h}{y-x+2h}f(y+h). \end{align*} 这两个性质说明,当两个变量保持其和不变时,对于下凸函数,将两个变量拉到一起时函数值之和最小,对于上凸函数,将两个变量越往两边拉,函数值之和越小。 引理的证明 我们只证最小值的情形,最大值的类似。 (1)若 $y\leqslant b$,则固定 $z$,此时即 $x$, $y\in[a,b]$ 且 $x+y$ 为定值,故由性质一知 $f(x)+f(y)+f(z)$ 取最小值时有 $x=y$; (2)若 $y>b$,分两小类讨论。   (2-1)若 $x\leqslant b$,则固定 $x$,此时即 $y$, $z\in[b,c]$ 且 $y+z$ 为定值,再分两小类讨论。     (2-1-1)若 $y+z\leqslant b+c$,则 $y+z-b\in[b,c]$,故由性质二有 $f(x)+f(y)+f(z)\geqslant f(x)+f(b)+f(y+z-b)$,注意到此时 $x$, $b\in[a,b]$,将 $x$ 重获ziyou,由性质一,又有 $f(x)+f(b)+f(y+z-b)\geqslant 2f\bigl((x+b)/2\bigr)+f(y+z-b)$,由此可见 $f(x)+f(y)+f(z)$ 取最小值时有 $x=y$;     (2-1-2)若 $y+z>b+c$,则 $y+z-c\in[b,c]$,故由性质二有 $f(x)+f(y)+f(z)\geqslant f(x)+f(y+z-c)+f(c)$,由此可见 $f(x)+f(y)+f(z)$ 取最小值时有 $z=c$。   (2-2)若 $x>b$,则固定 $y$,此时即 $x$, $z\in[b,c]$ 且 $x+z$ 为定值,再分两小类讨论。     (2-2-1)若 $x+z\leqslant b+c$,则 $x+z-b\in[b,c]$,故由性质二有 $f(x)+f(y)+f(z)\geqslant f(b)+f(y)+f(x+z-b)$,即转化为情形(2-1);     (2-2-2)若 $x+z>b+c$,则 $x+z-c\in[b,c]$,故由性质二有 $f(x)+f(y)+f(z)\geqslant f(x+z-c)+f(y)+f(c)$,由此可见 $f(x)+f(y)+f(z)$ 取最小值时有 $z=c$。 综上,引理得证。 将此方法推广到多元便得到半凹半凸定理。你能在脑海里动态地实现上述调整过程吗?如果实现了,那你已经理解了这个方法,并且也知道会明白多元的情形如何。 回到原题,为了方便处理,我将“锐角”的条件改为 $x$, $y$, $z\in[0,\pi/2]$,令 $\cos^2x=a$, $\cos^2y=b$, $\cos^2z=c$,则 $a$, $b$, $c\geqslant 0$ 且 $a+b+c=1$,令 $f(x)=\arccos\sqrt x$,则问题化为求 $f(a)+f(b)+f(c)$ 的取值范围。 求二阶导数易知 $f(x)$ 在 $[0,1]$ 上先下凸再上凸,即满足引理的条件。 由对称性,不妨设 $a\leqslant b\leqslant c$,先求最小值,由引理知,只要考虑 $a=b$ 与 $c=1$ 两种情况,后者显然只能得到 $f(a)+f(b)+f(c)=\pi$,而前者则化为求 $g(t)=2f(t)+f(1-2t)$ 的最小值,其中 $t\in[0,1/3]$。求导易得 \[g'(t)=\frac{3t-1}{\sqrt{t(1-t)(1-2t)}\bigl(2\sqrt{1-2t}+\sqrt{2(1-t)}\bigr)},\] 故当 $t=1/3$ 时 $g(t)$ 取最小值 $3\arccos(1/\sqrt3)$,所以 $f(a)+f(b)+f(c)$ 的最小值就是 $3\arccos(1/\sqrt3)$; 再求最大值,由引理知,只要考虑 $a=0$ 与 $b=c$ 两种情况,前者化为求 $\pi/2+f(t)+f(1-t)$ 的最大值,其中 $t\in[0,1/2]$,事实上,有恒等式 $\arccos\sqrt x+\arccos\sqrt{1-x}=\pi/2$,所以前者只能得到 $f(a)+f(b)+f(c)=\pi$,而后者仍然化为求上述 $g(t)$ 的最大值,只是此时 $t$ 的范围变成 $t\in[1/3,1/2]$,显然当 $t=1/2$ 时 $g(t)$ 取最大值 $\pi$,所以 $f(a)+f(b)+f(c)$ 的最大值就是 $\pi$。 综上,由连续性,所求的取值范围为 $[3\arccos(1/\sqrt3),\pi]$。而在原题的“锐角”情形下,取不到 $\pi$,但可以趋向它,所以原题的答案是 $[3\arccos(1/\sqrt3),\pi)$。
kuing 6# 2012-11-3 16:07
小修改了一下。
hongxian 7# 2012-11-3 20:49
本帖最后由 hongxian 于 2012-11-3 20:56 编辑 6# kuing 谢谢K版了,半凹半凸定理的确还是第一次接触,感觉不错谢谢了!那是不是还有一个先上凸再下凸的结论呢?
kuing 8# 2012-11-3 21:02
7# hongxian 显然是有的,结论类似,我懒得写而已。 这个方法是很好用的,很多问题都能解决,不过似乎了解这个方法并且接受在解题中使用的人并不多,所以我也很少用,不到万不得已时都不出这招。
hongxian 9# 2012-11-3 21:57
8# kuing 证明(2-1)中$f(x)+f(b)+f(y+z-b)\geqslant 2f\bigl((x+b)/2\bigr)+f(y+z-b)$,由此可见$f(x)+f(y)+f(z)$取最小值时有$x=y$; 似乎用到了$f(x)$是单调递减。不知我的理解对不对?
kuing 10# 2012-11-3 22:44
9# hongxian 没用到单调性,你自己再理解下。
hongxian 11# 2012-11-4 09:44
本帖最后由 hongxian 于 2012-11-4 09:46 编辑 10# kuing 又理解了一个,个人感觉应该分$x>b$和$x \leqslant b$两种情况来理解, $x>b$时,$y=x>b$时取最小,再让$x$重获自由; $x \leqslant b$时,令$x+y+z=t$为定值,$f(x)+f(b)+f(y+z-b)=f(x)+f(b)+f(t-b-x)$为关于$x$的下凸函数,再用性质一得$x=y=b$时取最小值。不知还有什么问题?
kuing 12# 2012-11-4 12:28
后面那个理解有点问题,不过你倒是提醒了我漏掉了 $x>b$ 没讨论。 我再修改一下。
kuing 13# 2012-11-4 12:48
$f(x)+f(y)+f(z)\geqslant f(x)+f(b)+f(y+z-b)$ 这个的意思是,$x$ 固定,$y$ 被调整到 $b$(凹凸分界处)时和变小,这里的 $f(b)$ 应该看成 $y$ 被调整成 $b$,它还可以继续参与调整。 然后将 $x$ 重获ziyou $f(x)+f(b)+f(y+z-b)\geqslant 2f\bigl((x+b)/2\bigr)+f(y+z-b)$ 意思就是将在凹凸分界处的 $y$ 与 $x$ 拉到一起,具体就是拉到了 $(x+b)/2$ 处,所以也有 $x=y$ 取更小。
hongxian 14# 2012-11-4 22:45
13# kuing 似乎有点明白了,看样子是先固定$x$调整$y$和$z$,再固定$z$调整$x$和$y$,也就是说这种情况一定可以调整到$y<b$, 再分析一下我的问题出在哪里吧,先固定$x$调整$y$和$z$是一样的,再固定$y$调整$x$和$z$,$x$调整到$x=y=b$却有可能还没有取到最小值。还是思维被$x\leqslant b \leqslant y\leqslant z$限制住了! 谢谢了!
kuing 15# 2012-11-4 23:10
其实用反证法可能更容易理解,就是说假设取最小值时 $x\ne y$ 且 $z<c$,然后运用上面的各种情况,总可以通过调整使之更小,从而矛盾。
thread-903-1-6.html: [数列] 求数列通项(不用数学归纳法)
转化与化归 1# 2012-11-3 16:57
求数列通项(不用数学归纳法)
kuing 2# 2012-11-3 16:59
话说让我用数归我可能也不会……
kuing 3# 2012-11-3 16:59
只给了 a4, a5 会不会不够条件确定 an ?
转化与化归 4# 2012-11-3 17:08
3# kuing 肯定可以的
kuing 5# 2012-11-3 17:12
4# 转化与化归 哦,是的,令 k=3,4... 可以得到 a1,a2... 我再瞧瞧
kuing 6# 2012-11-3 17:28
这样的话数归倒是很简单……但是如果改变 a4,a5 的值可能变化会很大,有可能只有特殊数据才能简单……不过也可能只是我目光短浅……暂时一点头绪都木有……先煮饭去了
转化与化归 7# 2012-11-3 18:48
6# kuing 吃饭是最重要的!
thread-904-1-6.html: [数列] 一个数列无界性的证明
都市侠影 1# 2012-11-3 19:42
本帖最后由 都市侠影 于 2012-11-3 19:54 编辑 近日群里出现一个数列,要证明它的无界性。数列由 $a_{n+1}=a_n+\frac{1}{a_n^2}$ 来确定,并且 $a_1>1$。 略证如下:假定对于某个 $n\geqslant 5$ 成立不等式 $a_n<\sqrt{a_1^2+n}$,那么由于函数 $f(x)=x+\frac{1}{x^2}$ 在 $x\geqslant 2$ 时是增函数,有: \[ a_{n+1}=a_n+\frac{1}{a_n^2} <\sqrt{a_1^2+n}+\frac{1}{a_1^2+n} <\sqrt{a_1^2+(n+1)} \] 这就是说,根据数学归纳法,对于此项后面的所有项,就都成立这个不等式了。于是就有: \[ a_{n+1}-a_n=\frac{1}{a_n^2}>\frac{1}{a_1^2+n} \] 取 $M$ 为不小于 $a_1^2$ 的任何一个正整数,有: \[ a_{n+1}>a_1+\sum_{k=1}^n\frac{1}{a_1^2+k}>a_1+\sum_{k=1}^n\frac{1}{M+k} \] 众所周知,自然数的倒数和是无界的,因此 $a_n$ 也是无界的。 至于不存在 $n\geqslant 5$ 使得 $a_n<\sqrt{a_1^2+n}$ 成立的情况,这时数列明显就是无界的,证毕。
pxchg1200 2# 2012-11-3 20:54
本帖最后由 pxchg1200 于 2012-11-3 22:04 编辑 1# 都市侠影 ah,若$a_{n}$ 有界,不妨设$ 0<|a_{n}|\leq M$ \[ a_{n+1}=a_{n}+\frac{1}{a_{n}^{2}} \] gives \[ a_{n+1}=a_{1}+\sum_{k=1}^{n}{\frac{1}{a^{2}_{k}}}\geq a_{1}+\frac{n^{2}}{\sum_{k}{a^{2}_{k}}}\geq a_{1}+\frac{n}{M^{2}} \] 数列显然无界,所以矛盾。
kuing 3# 2012-11-3 20:57
2# pxchg1200 昨天群聊记录也提到这个了,不过没你写得详细。 群管-kuing/bb/jy/cd<kuingggg@qq.com> 15:25:02 \begin{align*} a_n&=(a_n-a_{n-1})+(a_{n-1}-a_{n-2})+\cdots+(a_2-a_1)+a_1\\ &=\frac1{a_{n-1}^2}+\frac1{a_{n-2}^2}+\cdots+\frac1{a_1^2}+a_1, \end{align*} 假设 $a_n$ 有界,那么 $\{1/a_n^2\}$ 不趋向 0,所以当 $n\to\infty$ 时后面的求和无穷,得 $a_n$ 无界,矛盾…… 爱好者-战巡(3705*****) 15:27:08 或者之前说的,有界递增必有极限,此时a[n+1]=a[n],无解 群管-kuing/bb/jy/cd<kuingggg@qq.com> 15:30:32 严格点说应该是此时 lim a[n+1] = lim a[n],于是得到 lim 1/a[n]^2=0,即 a[n] 无界,矛盾
kuing 4# 2012-11-5 14:12
$a_1>1$ 其实可以弱化为 $a_1>0$,下面用数学归纳法证明当 $n\geqslant 2$ 时恒有 $a_n>\sqrt[3]{3n}$。 先验证 $a_2$ 成立,由均值有 \[a_2=\frac{a_1}2+\frac{a_1}2+\frac1{a_1^2}\geqslant \frac3{\sqrt[3]4}>\sqrt[3]6,\] 所以 $n=2$ 时成立,假设当 $n=k$ 时成立,即有 $a_k>\sqrt[3]{3k}$,当 $n=k+1$ 时,因为函数 $f(x)=x+1/x^2$ 在 $\bigl(\sqrt[3]2,+\infty \bigr)$ 上单调增,而 $\sqrt[3]{3k}>\sqrt[3]2$,所以 \[a_{k+1}=a_k+\frac1{a_k^2}>\sqrt[3]{3k}+\frac1{\bigl(\sqrt[3]{3k}\bigr)^2}=\sqrt[3]{3k+\frac1k+\frac1{(3k)^2}+3}>\sqrt[3]{3(k+1)},\] 于是当 $n=k+1$ 时也成立,故由数学归纳法知对任意 $n\geqslant 2$ 时都有 $a_n>\sqrt[3]{3n}$。 这样也算是从初等方法得到无\界吧
yfgkey 5# 2012-11-5 15:23
两位解的不错。 上次没注意到放缩那步,样如果两边直接求极限就搞定了。
kuing 6# 2012-11-5 15:26
5# yfgkey 其实如果不是你做错了,大概就不会有这个贴以及这些解法了
thread-905-1-1.html: 判断数列的敛散性
叶文明 1# 2012-11-4 14:54
$x_{n+1}=\sqrt[2]{x_{n}}+\sqrt[2]{x_{n-1}}$ $ x_{0}>0,x_{1}>0$
kuing 2# 2012-11-4 20:47
\begin{align*} \abs{x_{n+1}-4}&=\left|\sqrt{x_n}-2+\sqrt{x_{n-1}}-2\right|\\ &\leqslant \left|\sqrt{x_n}-2\right|+\left|\sqrt{x_{n-1}}-2\right|\\ &=\frac{\abs{x_n-4}}{\sqrt{x_n}+2}+\frac{\abs{x_{n-1}-4}}{\sqrt{x_{n-1}}+2}\\ &<\frac12(\abs{x_n-4}+\abs{x_{n-1}-4}) \end{align*} 这样不知有帮助没?
都市侠影 3# 2012-11-4 21:06
本帖最后由 都市侠影 于 2012-11-4 21:25 编辑 2# kuing 帮助是大大的有啊,不过需要加强一下:-) 首先需要证明对于数 $M(0<M<1)$,从某一项开始,所有的项都将大于 $M$,这个放在后面,有了这个就可以作估计: \begin{align*} \abs{x_{n+1}-4}&=\left|\sqrt{x_n}-2+\sqrt{x_{n-1}}-2\right|\\ &\leqslant \left|\sqrt{x_n}-2\right|+\left|\sqrt{x_{n-1}}-2\right|\\ &=\frac{\abs{x_n-4}}{\sqrt{x_n}+2}+\frac{\abs{x_{n-1}-4}}{\sqrt{x_{n-1}}+2}\\ &<\frac{1}{\sqrt{M}+2}(\abs{x_n-4}+\abs{x_{n-1}-4}) \\ &\leqslant\frac{2}{\sqrt{M}+2}max(\abs{x_n-4},\abs{x_{n-1}-4}) \end{align*} 由于那个比例因子是小于1的,这个收敛就顺理成章了。 回头说说那个下界 $M$,如果数列的两个初值中有一个不小于1或者两个都不小于1,那么根据归纳法,后面所有的项都大于1,那 $M$ 也自然就是下界了。如果两个初值都小于1,那么由于 $x_{n+1}>\sqrt{x_n}>x_n$ ,只要取正整数 $m$ 使 \begin{equation*} x[0]^{\frac{1}{2^m}}>M \end{equation*} 也就是取 \begin{equation*} m>\log_2{\log_Mx[0]} \end{equation*} 就能保证从第 m 项后面所有的项都大于 $M$.
kuing 4# 2012-11-4 21:12
噢,原来还要加强下,刚才没细想,在整电脑……
yxsx99 5# 2012-11-5 06:44
有界数列有收敛子列,所以存在子列 |xn_k -4| -> m, 进一步xn有子列xt 收敛于a,代入原递推式知a=4。故任给e,充分大t有|xt-4|<e,从而任给n>t后 |xn-4|<e。即xn收敛于4。 3# 都市侠影
thread-906-1-5.html: [函数] 一道非线性目标函数问题
hflz01 1# 2012-11-5 14:35
一道非线性目标函数问题 _____kuing edit in $\LaTeX$_____ 15. 已知实数 $x$, $y$ 满足 $\left\{\begin{aligned} &4x-3y+18\geqslant 0,\\ &y\geqslant 5,\\ &5x-y-5\leqslant 0, \end{aligned}\right.$则 $z=\dfrac{\sqrt{x^2+(y-2)^2}}y$ 的取值范围为_____
kuing 2# 2012-11-5 14:52
重复了一个附件,我删掉了。 群里其妙已经说了一个不错的方法,我就不重复说了,下面再给另一个思路,先画个图 当 $y$ 固定时显然 $z$ 关于 $x^2$ 递增; 当 $x$ 固定时,对 $y$ 求导得 $z'_y=\dfrac{2y-x^2-4}{y^2\sqrt{x^2+(y-2)^2}}$,再作出 $2y=x^2+4$ 的图象不难发现在给定区域内有 $2y>x^2+4$,所以 $z$ 关于 $y$ 递增。 综合两点,取最大值时为最上面的顶点 $(3,10)$,即最大值为 $\dfrac{\sqrt{73}}{10}$,取最小值时为下面的直线与 $y$ 轴的交点 $(0,5)$,即最小值为 $\dfrac35$。
kuing 3# 2012-11-5 15:11
顺便将那个“简单线性规划解题系统.exe”发上来……早前下载的小工具,忘了在哪里下的了 简单线性规划解题系统.rar (305.14 KB)
力工 4# 2012-11-5 19:49
3# kuing 这个系统是个好家伙,呵呵,谢谢了
kuing 5# 2012-11-5 19:59
4# 力工 只能处理不带参数的简单的线性规划问题……
imkeke 6# 2012-11-26 23:42
能把群里的做法发上来学习下么?谢谢
kuing 7# 2012-11-26 23:58
6# imkeke 摘录群聊记录如下: 教师-其妙/ka(2360****) 14:32:08 y钻到根号里? 爱好者-风度(5120*****) 14:32:32 然后呢 教师-其妙/ka(2360****) 14:32:42 令x/y=a,1/y=b 就是距离模型 条件变成: 4a+18b>=3 5a-5b<=1 b>0 b<=1/5 教师-其妙/ka(2360****) 14:35:21 令2/y=b较好吧 爱好者-风度(5120*****) 14:35:27 嗯 其妙老师得到的答案是多少 教师-其妙/ka(2360****) 14:35:58 没算了 要画图, 下了, PS、这里指的群是人教数学群,不是我那里。
imkeke 8# 2012-11-27 16:41
7# kuing 非常感谢
kuing 9# 2013-1-19 18:24
顺便将那个“简单线性规划解题系统.exe”发上来……早前下载的小工具,忘了在哪里下的了 kuing 发表于 2012-11-5 15:11 刚才做主题分类才发现原来就是在本论坛下载的 见 http://kkkkuingggg.5d6d.net/thread-740-1-1.html
thread-907-1-6.html: [不等式] 一个无理不等式
hflz01 1# 2012-11-5 18:19
一个无理不等式
kuing 2# 2012-11-5 18:45
跟这个贴 http://bbs.pep.com.cn/forum.php? ... age%3D1&page=55 的1076楼的题目差不多,应该可以类似做,后面有反证法。
kuing 3# 2012-11-5 19:35
照板煮碗 \begin{align*} \sqrt{x+\sqrt{y+\sqrt z}}&=\sqrt{x+\sqrt{\frac13\cdot 3y+\frac23\cdot \frac32z^{1/2}}} \\ & \geqslant \sqrt{x+\sqrt{(3y)^{1/3}\cdot \left( \frac32z^{1/2} \right)^{2/3}}} \\ & =\sqrt{x+3^{1/2}\cdot 2^{-1/3}\cdot (yz)^{1/6}} \\ & =\sqrt{\frac17\cdot 7x+\frac67\cdot \frac76\cdot 3^{1/2}\cdot 2^{-1/3}\cdot (yz)^{1/6}} \\ & \geqslant \sqrt{(7x)^{1/7}\cdot \left( \frac76\cdot 3^{1/2}\cdot 2^{-1/3}\cdot (yz)^{1/6} \right)^{6/7}} \\ & =7^{1/2}\cdot 3^{-3/14}\cdot 2^{-4/7}\cdot \sqrt[14]{xyz}, \end{align*} 其中 $7^{1/2}\cdot 3^{-3/14}\cdot 2^{-4/7}\approx 1.40699$。
hflz01 4# 2012-11-6 08:51
人教那个链接打开后,明明我登陆了,但还是看不到附件,求助。
kuing 5# 2012-11-6 13:12
4# hflz01 我这里可以看啊 换新版后有的大图可能显示不出来,但点击小图应该都可以看的吧?
thread-908-1-1.html: 用latex作图
叶文明 1# 2012-11-5 22:00
$如何用latex作图像y=\sin{x^{2}}$
kuing 2# 2012-11-5 22:31
我只会用 tikz 画 \begin{tikzpicture} \draw[->](-3,0)--(3,0); \draw[->](0,-2)--(0,2); \draw[domain=-3:3,smooth,samples=100] plot (\x,{sin(\x*\x r)}); \end{tikzpicture} 前面那两个是画坐标系,不用管它,最长那行就是画 $\sin x^2$
叶文明 3# 2012-11-6 12:36
这么厉害!
孤星赶月 4# 2012-12-21 19:32
本帖最后由 孤星赶月 于 2012-12-21 19:33 编辑 $\f(x)=x^2$
thread-909-1-1.html: 求积分
叶文明 1# 2012-11-6 13:10
$ \int x^x .dx $
kuing 2# 2012-11-6 13:11
这个能积出来?
叶文明 3# 2012-11-6 16:11
$ \int_{0}^{1} x^x .dx 是可以的$
kuing 4# 2012-11-6 16:48
3# 叶文明 这个我也不会……
叶文明 5# 2012-11-8 15:17
将$x^{x}$变为$e^{x\cdot\ln{x}}$再用级数展开就行
都市侠影 6# 2012-11-22 08:45
将$x^{x}$变为$e^{x\cdot\ln{x}}$再用级数展开就行 叶文明 发表于 2012-11-8 15:17 知道你还问?
thread-91-1-9.html: [不等式] SOS1_E.pdf中最后一例(伊朗96)中的问题
kuing 1# 2011-10-13 11:20
附件中的文档没记错的话是在mathlinks里早前讲sos方法的文档,刚才与何版主聊的时候何版主提到最后一例,有点莫名其妙,的确,那里跟前面列的几种情况中不同,而且即使所用的结论仍然正确但也少了些说明,这里扯一下。 sos1_e.pdf (54.6 KB) 首先文档中前面提到: 后面的例子是这样写的: 很明显,后面这跟前面的结论不同,还好所用的结论还是成立的,这里简单扯一下。 事实上,我们有如下结论: 若 $S=S_{a}(b-c)^{2}+S_{b}(c-a)^{2}+S_{c}(a-b)^{2}$,当 \begin{align*} &a\geqslant b\geqslant c\geqslant 0 \\ &S_{a}\geqslant 0, S_{b}\geqslant 0 \\ &b^{2}S_{c}+c^{2}S_{b}\geqslant 0 \end{align*} 那么有 $S\geqslant0$ 成立。 与原文对比,原文少了 $S_b\geqslant0$ 的证明,还好这是正确的因为由所设知 $a,b,c$ 是三角形三边,故易得 $S_b\geqslant \dfrac2{c(b+c)}-\dfrac1{b^2}=\dfrac{(2b+c)(b-c)}{b^2c(b+c)}\geqslant0$。 而至于上述结论的证明,这里用最简单的放缩方法,由 $b^{2}S_{c}+c^{2}S_{b}\geqslant 0$ 得 $S_{c}\geqslant -\dfrac{c^{2}S_{b}}{b^{2}}$,于是 \begin{align*} S&\geqslant S_{b}(c-a)^{2}+S_{c}(a-b)^{2} \\ & \geqslant S_{b}(c-a)^{2}-\frac{c^{2}S_{b}}{b^{2}}(a-b)^{2} \\ & =\frac{a(b-c)(b(a-c)+c(a-b))S_{b}}{b^{2}} \\ & \geqslant 0 \end{align*} 在推的过程中还顺便得出了另一个类似的结论: 若 $S=S_{a}(b-c)^{2}+S_{b}(c-a)^{2}+S_{c}(a-b)^{2}$,当 \begin{align*} &b+c\geqslant a\geqslant b\geqslant c\\ &S_{a}\geqslant 0, S_{b}\geqslant 0 \\ &b^{2}S_{b}+c^{2}S_{c}\geqslant 0 \end{align*} 那么有 $S\geqslant0$ 成立。 证明:由 $b+c\geqslant a\geqslant b\geqslant c$ 可得 $c(a-c)\geqslant b(a-b)\geqslant0$,于是 \begin{align*} c^{2}S&= c^{2}S_{a}(b-c)^{2}+S_{b}(c(a-c))^{2}+c^{2}S_{c}(a-b)^{2} \\ & \geqslant S_{b}(b(a-b))^{2}+c^{2}S_{c}(a-b)^{2} \\ & \geqslant (b^{2}S_{b}+c^{2}S_{c})(a-b)^{2} \\ & \geqslant 0 \end{align*} 后面这个结论适用于三角形三边,而这两种结论对于该题都是适用的。 这类结论其实可以自己再推一些总结一些,不过我觉得,其实在实战中看具体情况具体分析去放缩更加实际一些,不等式始终是需要灵活玩的。
pxchg1200 2# 2011-10-13 21:09
围观下。。。。
thread-910-1-1.html: 求极限
叶文明 1# 2012-11-6 13:14
$ u_n=n^{\alpha}.sin n $ $with\alpha>0$
kuing 2# 2012-11-6 13:17
直觉上发散(不是无穷大),具体证不会…… PS: sin -> \sin \$with\alpha>0\$ -> with \$\alpha>0\$
都市侠影 3# 2012-11-22 08:48
本帖最后由 都市侠影 于 2012-11-22 08:52 编辑 明显是发散的,存在任意大的正整数 $n$,使得 $\sin{n}>k$,这里 $k$ 是一个0与1之间的某个正实数。 道理也很简单,正弦函数在每一个 $x$ 轴上方的波峰部分所对应的横坐标范围是 $\pi$ 个单位长度,在这个范围内是存在2到3个正整数的,剩下的就很简单了。
thread-911-1-6.html: [不等式] 论坛关闭前发的最后一题
pxchg1200 1# 2012-11-6 16:12
For$a,b,c\geq 0$ prove that: \[ \sqrt[3]{a^2-ab+b^2}+\sqrt[3]{b^2-bc+c^2}+\sqrt[3]{c^2-ca+a^2}\geq \sqrt[3]{8(a^2+b^2+c^2)+ab+bc+ca} \]
kuing 2# 2012-11-6 16:50
嗯,明天就要关了,苦b的论坛…… 不知会要开多久?N天之后重开,人气估计又再减少……
shidilin 3# 2012-11-6 17:15
吓人一跳,以为论坛挂了呢。开16+2大了,党的光荣传统惯例了。
kuing 4# 2012-11-6 17:17
3# shidilin 嗯,所以我特意制作了这个http://kkkkuingggg.5d6d.net/thread-897-1-1.html,期间如果想看某些贴的话可以用它来看看,不过就只能看了,如果有啥想回复的等重开后再回好了……
thread-913-1-6.html: [函数] 发一个题试一下!
hongxian 1# 2012-11-8 07:57

hongxian 2# 2012-11-8 07:58
看样子,能发!
第一章 3# 2012-11-9 22:12
k被骗了
第一章 4# 2012-11-10 18:31
复合函数的单调性吧,好题!结果是[-1,0];对不? 论坛好像能上,但页面关掉就再也上不了。
第一章 5# 2012-11-10 18:32
过一会就会被关页面了
kuing 6# 2012-11-10 21:15
4# 第一章 间歇xing能上 但一般不到几分钟就不行,就算不关页面,刷新一下也会变成斯巴达
kuing 7# 2012-11-10 21:16
昨晚就想回复,结果打好之后一点回复就斯巴达
第一章 8# 2012-11-10 22:18
看来这是纯数学和政治的不兼容
kuing 9# 2012-11-15 19:38
那天在群里发的,忘记存代码了,懒得重打了……
第一章 10# 2012-11-16 13:06
求导吗?
kuing 11# 2012-11-16 13:08
嗯,我求导做的
第一章 12# 2012-11-16 13:12
我那天用复合函数做的,还把题目收录了,看来又得写多一种解法。
kuing 13# 2012-11-16 13:24
12# 第一章 收录?收到哪里去了?
第一章 14# 2012-11-16 17:48
我自己收集的一些题目啊
kuing 15# 2012-11-16 17:53
thread-914-1-1.html: 物理
yayaweha 1# 2012-11-9 22:54
可在速度的某一方向上使用动能定理吗?最好有证明?
kuing 2# 2012-11-15 19:19
我看不行吧,动能是能量,有方向吗?
秋风树林 3# 2012-11-15 20:12
动能定理其实是个标量积分...没方向这一说法
kuing 4# 2012-11-15 20:14
刚才我打错字了
thread-915-1-6.html: [几何] 前几天粉丝群里的一几何题
kuing 1# 2012-11-15 19:23
题目:$BM$ 是锐角 $\triangle ABC$ 的中线,$\triangle ABM$ 的外接圆在 $A$ 点的切线与 $\triangle BCM$ 的外接圆在 $C$ 点的切线交于 $D$。 求证:$D$ 关于直线 $AC$ 的对称点位于直线 $BM$ 上。 证明:在 $BM$ 上取一点 $E$,使 $ME\cdot MB=(AC/2)^2$,此时易证 \begin{align*} \triangle MEA &\sim \triangle MAB,\\ \triangle MEC &\sim \triangle MCB, \end{align*} 所以有 \begin{align*} \angle MAE &= \angle MBA = \angle MAD,\\ \angle MCE &= \angle MBC = \angle MCD, \end{align*} 可见 $E$ 和 $D$ 关于 $AC$ 对称,故得证。
isea 2# 2012-11-16 10:21
ME⋅MB=(AC/2)^2 看到这个就想到反演,不过,都快忘记了……
thread-916-1-1.html: 电!!!!
yayaweha 1# 2012-11-15 23:16
本帖最后由 yayaweha 于 2012-11-15 23:21 编辑 如图所示,沿水平方向放置一条平直光滑槽,它垂直穿过开有小孔的两平行薄板,板相距3.5L。槽内有两个质量均为m的小球A和B,球A带电量为+2q,球B带电量为-3q,两球由长为2L的轻杆相连,组成一带电系统。最初A和B分别静止于左板的两侧,离板的距离均为L。若视小球为质点,不计轻杆的质量,在两板间加上与槽平行向右的匀强电场E后(设槽和轻杆由特殊绝缘材料制成,不影响电场的分布),求: (1)球B刚进入电场时,带电系统的速度大小; (2)带电系统从开始运动到速度第一次为零所需的时间及球A相对右板的位置。 不用解题~ 就问下像这个题中进入电场后A球对B球做不做功?就是说若求进入电场后B球电势能的变化,算不算A球对它库仑力做的功?
kuing 2# 2012-11-15 23:29
用杆连着成了系统就不用算内部的相互作用了吧?
yayaweha 3# 2012-11-15 23:37
只关于B球要考虑吧!
kuing 4# 2012-11-15 23:40
3# yayaweha 那也只是考虑系统外部对 B 的作用就行了,内部已经有杆对 B 的作用力抵消了 A 对 B 的力。
yayaweha 5# 2012-11-15 23:41
4# kuing 电势能变化与杆的力无关
kuing 6# 2012-11-15 23:52
A 对 B 做功,杆对 B 也做功,抵消
yayaweha 7# 2012-11-15 23:54
6# kuing 电势能的变化量只与电场力做的功有关
kuing 8# 2012-11-16 07:59
呃……哎物理我也不太懂了,反正感觉这里不用考虑那些,或者说是只需用经典力学那些东东去弄就行了的
海盗船长 9# 2013-2-18 22:49
如果取地面为参考系并且计入a对b的库仑力的话,由于a在运动,b所处的电场就不是静电场不能定义电势,电势能也没有意义
thread-919-1-6.html: 请教一个三角形中的最值问题,先谢谢了!
hongxian 1# 2012-11-16 11:02
10.在$\triangle ABC$中,满足$A=2B$,$C$是钝角,三边长$a$、$b$、$c$均为整数,求$\triangle ABC$周长的最小值
kuing 2# 2012-11-16 11:15
又要重新打翻译…… $a$, $b$, $c\in\mbb N^+$, $(a+b)^2>c>a^2+b^2$, $a^2=b(b+c)$,求 $a+b+c$ 最小值。
kuing 3# 2012-11-16 11:18
只会笨方法 \[a^2=b(b+c) \riff \left\{\begin{aligned} &c=\frac{a^2}b-b,\\ & b\mid a^2, \end{aligned}\right.\] 代入 $(a+b)^2>c^2>a^2+b^2$ 中化简可以得到 \[2>\frac ab>\sqrt 3,\] 通过由小到大寻找,可以找到同时满足 $2>a/b>\sqrt 3$ 且 $b\mid a^2$ 的最小的正整数 $a$ 是 $28$(其实也不是很麻烦,因为有很多数很容易排除,比如质数显然不符合可以直接跳过),所以 \[a+b+c=a+\frac{a^2}b>a+\sqrt3a\geqslant 28+\sqrt3\cdot28\approx 76.4974,\] 所以有 \[a+b+c\geqslant 77,\] 当 $a=28$, $b=16$, $c=33$ 时满足所有条件且 $a+b+c=77$,所以 $a+b+c$ 的最小值为 $77$。
hongxian 4# 2012-11-21 19:19
本帖最后由 hongxian 于 2012-11-21 19:22 编辑 把答案整理一下发上来看一下! 10.在$\triangle ABC$中,满足$A=2B$,$C$是钝角,三边长$a$、$b$、$c$均为整数,求$\triangle ABC$周长的最小值。 解:因为$C=\pi -A-B=\pi -3B>\frac{\pi }{2}$,所以$B<\frac{\pi }{6}$,$\cos B>\frac{\sqrt{3}}{2}$且$\cos B$为有理数 令$\cos B=\frac{n}{m}$,$m>n$,$m$,$n\in N^*$,且$(m,\ n)=1$ 由$\left\{ \begin{matrix}    \frac{n}{m}>\frac{\sqrt{3}}{2}  \\    \frac{n}{m}<1  \\ \end{matrix} \right.\Rightarrow \frac{\sqrt{3}}{2}m<n<m$,又$m,n\in N^*$所以$m-\frac{\sqrt{3}}{2}m>1\Rightarrow m>4+2\sqrt{3}>7\Rightarrow m\geqslant 8$ 又$c=\frac{b}{\sin B}\cdot \sin 3B=b(3-4{{\sin }^{2}}B)=b(4{{\cos }^{2}}B-1)=b\left( \frac{4{{n}^{2}}}{{{m}^{2}}}-1 \right)$ 故$\frac{4bm}{{{m}^{2}}}$为整数,又$(m,\ n)=1$,则$\frac{4b}{{{m}^{2}}}$为整数。由$m\geqslant 8$,故$b\geqslant 16$ 又$\cos B>\frac{\sqrt{3}}{2}$,所以$c>16\left[ 4{{\left( \frac{\sqrt{3}}{2} \right)}^{2}}-1 \right]=32$,所以$c\geqslant 33$ $a=\frac{b\sin 2B}{\sin B}=2b\cos B\ge 2\times 16\times \frac{\sqrt{3}}{2}=16\sqrt{3}>27$,所以$a\geqslant 28$ 即$a+b+c\geqslant 28+16+33=77$ 且当$a=28$,$b=16$,$c=33$时,由余弦定理得:$\cos A=\frac{17}{32}$,$\cos B=\frac{7}{8}$ 故$\cos A=\cos 2B=2{{\cos }^{2}}B-1$,$A=2B$,$\cos B=\frac{7}{8}>\frac{\sqrt{3}}{2}$,$B<\frac{\pi }{6}$,$A<\frac{\pi }{3}$,$C>\pi -\frac{\pi }{6}-\frac{\pi }{3}=\frac{\pi }{2}$ 即$a=28$,$b=16$,$c=33$符合条件,$\triangle ABC$周长的最小值为77. .
kuing 5# 2012-11-21 20:34
原来 $a\geqslant28$ 的证明可以这么简单,那样将两个解法结合起来就可以更简单了。
力工 6# 2012-11-22 07:54
1# hongxian 呵呵,18D不是关了吗?怎么16号可以发帖?
hongxian 7# 2012-11-22 09:30
6# 力工 这可是18D之后的第一贴,之前还被shan了一次!
thread-92-1-1.html: 公式标签功能\tag测试
kuing 1# 2011-10-13 15:00
公式中出现 \tag{内容},无论是行内或行间模式,都会变成行间公式,且会在公式右边标一个 (内容) 。 测试: 美元符号 \tag{123} a=b+c 美元符号,效果: $\tag{123} a=b+c$ 美元符号 a=b+c \tag{123} 美元符号,效果: $a=b+c \tag{123}$ 美元符号 a=b\tag{123}+c 美元符号,效果: $a=b\tag{123}+c$ 美元符号 a=b+c \tag{123}\tag{456} 美元符号,效果: $a=b+c \tag{123}\tag{456}$ 也就是说 \tag 并不限于位置,但在同一公式中 \tag 只能出现一个 反斜杠左中括号 \tag{123} a=b+c 反斜杠右中括号,效果: \[\tag{123} a=b+c\] 环境之中也可使用,在需要标签的公式行中加就行了 \begin{align*} a&=b+c\tag{oh}\\ &=k+u\\ \tag{no!}&=i+n+g \end{align*} 加个星号,变成 \tag* ,与不加星号的区别,就是没了个括号。 试试:反斜杠左中括号 \tag*{123} a=b+c 反斜杠右中括号,效果: \[\tag*{123} a=b+c\] 环境之中也可使用,在需要标签的公式行中加就行了 \begin{align*} a&=b+c\tag*{oh}\\ &=k+u\\ &=i+\tag*{no!}n+g \end{align*}
kuing 2# 2011-10-14 13:59
原来这个tag会令部分浏览器看不到公式还是用不了可惜
thread-920-1-1.html: mathematica中的一些符号的输入
kuing 1# 2012-11-16 11:04
圆周率 $\pi$(Pi):Esc p Esc 自然对数的底 $\mathrm e$(E):Esc ee Esc 虚数单位 $\mathrm i$(I):Esc ii Esc 无穷 $\infty$(Infinity):Esc inf Esc 度 $^\circ$(Degree):Esc deg Esc 并 $\cup$(Union):Esc un Esc 交 $\cap$(Intersection): Esc inter Esc 且 $\wedge$(And):Esc and Esc 或者直接 && 或 $\vee$(Or):Esc or Esc 或者直接 || 非 $\neg$(Not):Esc not Esc 或者直接 ! 属于 $\in$(Element):Esc elem Esc 待查待续……
yes94 2# 2013-4-6 20:12
1# kuing Esc是神马?电脑键盘左上角的那个键?
kuing 3# 2013-4-6 20:22
2# yes94 是
thread-921-1-1.html: [不等式] 宋庆老师15年前的不等式
ccnu_chb_ycb 1# 2012-11-17 20:23
这是宋庆老师15年前提出的一个不等式,最近看到kuing构造切线法证明不等式的延伸——构造平面发证明不等式,感觉对此不等式充满了好奇,如下问题: (1)此不等式如何简单的证明,方法当然越多越好; (2)此不等式好像在老高中教材中出现过二元的情形,记得当初是利用分析法证明的; (3)这个好像和两角和的正切公式很像,但是又不一样?能否提供一些思考方向呢? (4)当然的此不等式能够进行推广?一般情形大致什么样?证法能够平移当然最好;                                                             ——欢迎高手们进行交流        ccnu-chb-ycb     邮箱:chenghanbo511@sina.com
kuing 2# 2012-11-18 04:01
有人证出过没?
ccnu_chb_ycb 3# 2012-11-18 18:50
这个宋庆老师本人证明出来了,两次用到柯西不等式然后进行了换元,想找下其他的解法试下……
kuing 4# 2012-11-18 19:38
方便贴出来欣赏下不?
ccnu_chb_ycb 5# 2012-11-20 21:45
4# kuing kuing,最近都在忙实习的事,那个解答我有空贴出来哟,在《数学通讯》2012年第9期上面有,有空贴出来哦……
kuing 6# 2012-11-20 22:01
十五年后才解答
reny 7# 2013-5-23 10:51
没人贴,我将答案贴出来吧。由于过程有点长,所以上的附件。
kuing 8# 2013-5-23 11:13
oh,我几乎忘了这一贴的存在…… 这样看来其实很松?
reny 9# 2013-5-23 13:21
的确也是,还两次用了柯西放缩.   像这种方法,难道就是试出来滴。
thread-922-1-1.html: [函数] 一个函数题
转化与化归 1# 2012-11-17 21:07
一个函数题 ___________kuing edit in $\LaTeX$___________ 若不存在正实数 $m$, $n$,使函数 $f(x)=a-\abs{x^3}$ 在区间 $[m,n]$ 上的值域为 $[m,n]$,则实数 $a$ 的取值范围为?
realnumber 2# 2012-12-3 22:09
也不知道是不是,配合图象,也许有没考虑到的 $a \le -\frac{2\sqrt{3}}{9}$或$a > \sqrt{2}$
kuing 3# 2012-12-3 23:11
2# realnumber “配合图象”的意思是不是 and 这两个临界点?
kuing 4# 2012-12-4 02:20
严格证明的话估计少不免要分类讨论……还是不想做了……
realnumber 5# 2012-12-4 07:36
恩,是这样,还好看来没错;一开始是分类讨论的,还是配合图象来的快多了 什么m,n同号异号什么的.
thread-922-1-5.html: [函数] 一个函数题
转化与化归 1# 2012-11-17 21:07
一个函数题 ___________kuing edit in $\LaTeX$___________ 若不存在正实数 $m$, $n$,使函数 $f(x)=a-\abs{x^3}$ 在区间 $[m,n]$ 上的值域为 $[m,n]$,则实数 $a$ 的取值范围为?
realnumber 2# 2012-12-3 22:09
也不知道是不是,配合图象,也许有没考虑到的 $a \le -\frac{2\sqrt{3}}{9}$或$a > \sqrt{2}$
kuing 3# 2012-12-3 23:11
2# realnumber “配合图象”的意思是不是 and 这两个临界点?
kuing 4# 2012-12-4 02:20
严格证明的话估计少不免要分类讨论……还是不想做了……
realnumber 5# 2012-12-4 07:36
恩,是这样,还好看来没错;一开始是分类讨论的,还是配合图象来的快多了 什么m,n同号异号什么的.
thread-925-1-1.html: 求积分
pxchg1200 1# 2012-11-19 15:09
本帖最后由 pxchg1200 于 2012-11-19 15:15 编辑 \[ \int_{0}^{\pi}{\frac{x\cos{x}}{1+\sin^{2}{x}}dx} \] \[ \int_{-\infty}^{+\infty}{\frac{\cos{x}}{1+x^4}dx} \] \[ \int_{0}^{\infty}{e^{-x}\ln{x}dx} \]
hnsredfox_007 2# 2012-11-22 11:16
第三个不会了
kuing 3# 2012-11-22 14:38
第一个懂,第二个不懂。。。Res是神马…… PS、上标字体小得有点看不清
hnsredfox_007 4# 2012-11-22 14:43
3# kuing Res f(z) 是残数(留数) ,复变函数里面的.
kuing 5# 2012-11-22 14:45
噢,原来这么高森……没玩过……只能潜水了
pxchg1200 6# 2012-11-25 18:02
本帖最后由 pxchg1200 于 2012-11-25 18:05 编辑 5# kuing 第一题做错了,答案应该是 \[ \ln^{2}{(\sqrt{2}+1)}-\frac{\pi^{2}}{4} \] 第三个答案是 \[ -C \] C表示Euler常数。 PS:考虑\[ \Gamma^{'}{(1)} \]
hnsredfox_007 7# 2012-11-26 10:23
是的 第一个化简时符号弄错了
thread-926-1-6.html: [不等式] 来自群的三次根式不等式,分类切线成功
kuing 1# 2012-11-19 21:17
题目:设 $a$, $b$, $c>0$, $abc=1$,求证 \[\sqrt[3]{a^3-a+1}+\sqrt[3]{b^3-b+1}+\sqrt[3]{c^3-c+1}\geqslant a+b+c.\] 分类切线又成功了。 先做点准备工作,证明以下两式成立。 \begin{align} \sqrt[3]{x^3-x+1}&>x-\frac16\quad\forall x\in\mbb R;\label{20121119abc1flqxs1}\\ \sqrt[3]{x^3-x+1}&\geqslant x-\frac13\ln x\quad\forall x\geqslant \frac12.\label{20121119abc1flqxs2} \end{align} 先证式 \eqref{20121119abc1flqxs1}。 若 $x<1$,则显然 $\sqrt[3]{x^3-x+1}>x$,此时式 \eqref{20121119abc1flqxs1} 成立; 若 $x\geqslant 1$,则 \begin{align*} \sqrt[3]{x^3-x+1}-x+\frac16&=\frac{1-x}{\sqrt[3]{(x^3-x+1)^2}+x\sqrt[3]{x^3-x+1}+x^2}+\frac16 \\ & \geqslant \frac{1-x}{1+x+x^2}+\frac16 \\ & =\frac{7-5x+x^2}{6(1+x+x^2)} \\ & >0, \end{align*} 故此时式 \eqref{20121119abc1flqxs1} 也成立,所以式 \eqref{20121119abc1flqxs1} 得证。 再证式 \eqref{20121119abc1flqxs2}。令 \[f(x)=\sqrt[3]{x^3-x+1}-x+\frac13\ln x,\] 求导得 \[f'(x)=\frac{3x^3-x-(3x-1)\sqrt[3]{(x^3-x+1)^2}}{3x\sqrt[3]{(x^3-x+1)^2}},\] 当 $1/2\leqslant x\leqslant 1$ 时,有 \begin{align*} 3x^3-x-(3x-1)\sqrt[3]{(x^3-x+1)^2}&\leqslant 3x^3-x-(3x-1)(x^3-x+1) \\ & =(1-x)\bigl((3x-1)(x^2-1)-x\bigr) \\ & \leqslant 0, \end{align*} 所以此时 $f'(x)\leqslant 0$; 当 $x\geqslant 1$ 时,有 \[3x^3-x-(3x-1)\sqrt[3]{(x^3-x+1)^2}\geqslant 3x^3-x-(3x-1)x^2=x(x-1)\geqslant 0,\] 所以此时 $f'(x)\geqslant 0$。 综上知当 $x\geqslant 1/2$ 时恒有 $f(x)\geqslant f(1)=0$,所以式 \eqref{20121119abc1flqxs2} 得证。 回到原题,由对称性,不妨设 $a=\min\{a,b,c\}$,分两类讨论。 (1)若 $a\geqslant 1/2$,则由式 \eqref{20121119abc1flqxs2} 即得 \[\sum{\sqrt[3]{a^3-a+1}}\geqslant \sum{\left( a-\frac13\ln a \right)}=\sum a-\frac13\ln abc=\sum a,\] 原不等式成立; (2)若 $0<a<1/2$,则由式 \eqref{20121119abc1flqxs1} 得 \[\sum{\sqrt[3]{a^3-a+1}}>\sqrt[3]{a^3-a+1}+b+c-\frac13,\] 所以只要证明 \[\sqrt[3]{a^3-a+1}\geqslant a+\frac13,\] 上式两边立方并化简整理等价于 \[26-36a-27a^2\geqslant 0,\] 由 $0<a<1/2$ 易知成立,所以原不等式也成立。 综上所述,原不等式获证。
pxchg1200 2# 2012-11-20 12:36
1# kuing 莫切了,我们有: \[ \sqrt[3]{x^3-x+1}-x\geq \frac{1}{3}\left(\frac{3}{x^2+x+1}-1 \right) \] Done!
kuing 3# 2012-11-20 13:19
1# kuing 莫切了,我们有: \[ \sqrt[3]{x^3-x+1}-x\geq \frac{1}{3}\left(\frac{3}{x^2+x+1}-1 \right) \] Done! pxchg1200 发表于 2012-11-20 12:36 这个之前也见过,还有推广的:http://tieba.baidu.com/p/1960866492,那里后面好像打少了个 -x,不知 $n$ 的局部怎么证?
pxchg1200 4# 2012-11-20 13:22
3# kuing Vasc 给的,具体不知道
kuing 5# 2012-11-20 13:25
4# pxchg1200 这些想法太牛比,我高攀不起,所以还是玩切的……
kuing 6# 2012-11-20 13:25
对了,有没有 Vasc 关于此不等式的相关链接?
pxchg1200 7# 2012-11-20 13:27
6# kuing 找不到鸟。。。。
thread-927-1-4.html: [不等式] 自以为简洁的证明呢,但也有说错证
realnumber 1# 2012-11-21 14:57
有老师说有问题,似乎集中在x+y能否固定,以及z=0可不可以
kuing 2# 2012-11-21 15:41
前半部分(直到“都是 z 的增函数”)都没问题,固定 $x+y$ 是可以的,后面不好说……
realnumber 3# 2012-11-21 15:55
baoshisun4  15:44:11 x=y=z=sqtr3/3,x+y<2, x=2,y=1/2,z=0,x+y>2. 确实有点问题。 Real  15:49:11 满足x+y=2sqtr3/3,又xy+xz+yz=1,得不到,z=0
kuing 4# 2012-11-21 16:00
要么就这样: 我们得到了当 $x\geqslant y\geqslant z\geqslant 0$ 且 $xy+yz+zx=1$ 并固定 $x+y$ 时原不等式左边总是关于 $z$ 递增,而约束条件可以看成 $xy+z(x+y)=1$,因此也可以说是关于 $xy$ 递减,于是要原不等式左边变小,只要将 $x$, $y$“拉近”,而在“拉近”的同时 $z$ 变小,所以当 $x+y$ 固定时,原不等式左边取最小值要么 $x=y$,要么 $z=0$,也就是只要证明这两种情况下原不等式成立即可。
hnsredfox_007 5# 2012-11-22 09:47
附件
kuing 6# 2012-11-22 13:32
5# hnsredfox_007 welcome 嗯,相当于将 4# 的想法表达了出来,本质一样。 x+y 的固定值决定了 z 能不能取0,而不能取 0 时便是 x=y 时取最小,此时 z 就是那个 1/k-k/4
pxchg1200 7# 2012-11-25 18:09
额,都在搞什么啊,平方然后上Iran 96啊。。。
kuing 8# 2012-11-25 18:35
7# pxchg1200 言下之意是不是有了简单方法就不应该再讨论别的方法了?
realnumber 9# 2013-2-22 08:23
7# pxchg1200 Iran 96不知道,一个教高三的同事问的问题,似乎在高考模拟卷上.
yes94 10# 2013-2-22 14:38
9# realnumber $x+y$都固定了,你还“$z=0$时,$x+y\geqslant2\sqrt{xy}=2$” 万一你固定的$x+y=1$,怎办? 万一你固定的$x+y=3$,怎办? 此时$x+y=3$会造成你的“$f(0)=\dfrac1{x+y}+x+y\geqslant\dfrac52$”就无法取等号,造成不等式也无法取等号。 据说,有时候要慎用不妨设$x\geqslant y\geqslant z$, 还有最好也要慎用固定某些字母(据说此方法常常要搞调整法的本质)。
yes94 11# 2013-2-22 14:59
5# hnsredfox_007 做的的确很妙!只是红狐老师是“空谈误guo,实干兴bang”的那种,从不多言多语,哪怕是一个字! 不来则罢,一来就是做题的那种, kuing的评述也很强大!
realnumber 12# 2013-2-22 18:01
11# yes94 2楼k的评论和你前面的矛盾.
yes94 13# 2013-2-22 18:36
12# realnumber 矛盾在哪呢? 再看一遍都没看出呢
realnumber 14# 2013-2-22 20:00
10# yes94 其实我没明白你10楼的说法.
yes94 15# 2013-2-22 20:58
14# realnumber     那估计你没看懂红狐的证明,他分两种情况讨论(分$k\geqslant2$和$k<2$讨论),可是他搞了一个大括号,很容易使人看不懂,建议他分开写吧。     还有他一上来就“$z\geqslant\cdots\dfrac1{x+y}-\dfrac{x+y}4$”,建议他把这个“$z\geqslant\cdots\dfrac1{x+y}-\dfrac{x+y}4=\dfrac1{k}-\dfrac{k}4$”写在后面(即写在第二种情况$k<2$后面),免得混淆。    还有,$2$楼kuing这样说的:“前半部分(直到“都是$z$的增函数”)都没问题,固定$x+y$是可以的,后面不好说……”    而$10$楼的叙述正好就是从kuing叙述的这个地方开始的呀? 那我把把10楼再解释一遍吧:    如果你预先固定的$x+y=1$时(有没有此种可能?),$x+y\geqslant2\sqrt{xy}=2$就不成立,所以需要讨论$x+y\geqslant2$和$x+y\le2$,红狐做到了。    如果你预先固定的$x+y=3$时(有没有此种可能?),那么你写的不等式$\dfrac1{x+y}+x+y\geqslant\dfrac52$”就无法取等号(只有$x+y=2$才取等号),    我的意思就是这样的,不知对不对?
realnumber 16# 2013-2-22 23:47
本帖最后由 realnumber 于 2013-2-22 23:52 编辑 15# yes94 你对比下3楼,是不是和你一样. ps,有时觉得1楼说不定还是可以,虽然取不到2.5,但是比2.5大,就不影响最小值2.5.也就是说扩大范围情况下的最小值都是2.5,那么在原范围情况下就不会小于2.5. 在条件"满足x+y=2sqtr3/3,又xy+xz+yz=1,得不到,z=0 "下能不能添加z=0?
realnumber 17# 2013-2-22 23:53
16# realnumber kk出马?我有些犯迷糊了,最近状态实在欠佳.....
kuing 18# 2013-2-23 00:02
我今天也没什么状态……刚刚才在群里说我今天一道题都没做……好颓废的一天…… 哎,看到上面一堆的,就看不下去了……
realnumber 19# 2013-2-23 00:11
18# kuing 碰到长的,别人的,与自己理解有出入的,加上状态不够好,基本当机,.. 高中就意识到的老毛病,非得自己慢慢领会,或实在避不过去....
realnumber 20# 2013-2-23 10:08
16# realnumber 答案对的,也仅仅是凑巧答案对而已.我还是继续想,能不能改成更严密的表达办法,如果也可以的话.
thread-927-2-4.html:
realnumber 21# 2013-2-23 11:07
本帖最后由 realnumber 于 2013-2-23 11:51 编辑 条件$xy+yz+zx=1$且$x\ge y\ge z \ge0$下,记$f(x,y,z)=\frac{1}{x+y}+\frac{1}{x+z}+\frac{1}{y+z}$. 那么可以证明$f(x,y,z)\ge f(x,y,0)$,当且仅当$z=0$取等号. 证明:\[f(x,y,z)\ge f(x,y,0)\iff   \frac{1}{x+y}+\frac{x+y+2z}{1+z^2}\ge \frac{1}{x+y}+x+y\] \[\iff x+y+2z\ge (1+z^2)(x+y)\iff 2z\ge z^2(x+y)\], 而$2>1\ge z(x+y)$,显然成立,完. -------还是需要考虑$z≠0$情况.--好吧,就这样了,虽然还是有疑惑的地方.
yes94 22# 2013-2-23 11:59
21# realnumber 这个就有点像调整法或者叫磨光变换的意味了,
yes94 23# 2013-2-23 12:21
条件$xy+yz+zx=1$且$x\ge y\ge z \ge0$下,记$f(x,y,z)=\frac{1}{x+y}+\frac{1}{x+z}+\frac{1}{y+z}$. 那么可以证明$f(x,y,z)\ge f(x,y,0)$,当且仅当$z=0$取等号. 证明:\[f(x,y,z)\ge f(x,y,0)\iff   \frac{1}{x+y} ... realnumber 发表于 2013-2-23 11:07 有两点疑惑,谁解释一下呢,       疑惑$1$:的确,$z=0$时,根据条件可得$xy=1$,于是$f(x,y,0)=\dfrac{1}{x+y}+\dfrac{1}{x}+\dfrac{1}{y}=\dfrac{1}{x+y}+\dfrac{x+y}{xy}=\dfrac{1}{x+y}+x+y$,     但是$f(x,y,z)\geqslant f(x,y,0)$的意思是否是保证了不等式$f(x,y,z)\geqslant f(x,y,0)$两边的$x,y$的取值一样了呢? 但是显然,不等式$f(x,y,z)\geqslant f(x,y,0)$右边的$x、y$是有限制条件的,其限制条件是$xy=1$(因为$z=0$) 而不等式$f(x,y,z)\geqslant f(x,y,0)$左边的$x、y$的限制条件却是原汁原味的$xy+yz+zx=1$! 换句话说,不等式$f(x,y,z)\geqslant f(x,y,0)$两边的$x,y$似乎不一样!对不对?      疑惑$2$:如果是保持不等式两边$x,y$的值不变,不等式$f(x,y,z)\geqslant f(x,y,0)$的右边,我们根据分母变小,分数值应该变大的原理, 似乎$f(x,y,z)=\frac{1}{x+y}+\frac{1}{x+z}+\frac{1}{y+z}$的后两个加数的分母应该变小后导致$f(x,y,z)$的值变大?即应该是$f(x,y,0)\geqslant f(x,y,z)$呢?这样不就导致矛盾了么?该如何理解呢?
realnumber 24# 2013-2-23 12:45
23# yes94 ,疑惑1,两边x,y一样;又难点在$z≠0$.具体看火狐老师证明.
yes94 25# 2013-2-23 13:34
24# realnumber 唉! 不说了吧,
thread-928-1-6.html: [不等式] 来自群的n个分数的题
kuing 1# 2012-11-24 15:57
其实就是加权平均,注意到 \[\frac{a_1+a_2+\cdots +a_n}{b_1+b_2+\cdots +b_n}=\frac{b_1}{\sum{b_i}}\cdot \frac{a_1}{b_1}+\frac{b_2}{\sum{b_i}}\cdot \frac{a_2}{b_2}+\cdots +\frac{b_n}{\sum{b_i}}\cdot \frac{a_n}{b_n},\] 这里 $\sum{b_i}$ 表示 $b_1+b_2+\cdots+b_n$,由于各 $b_i$ 都是正的,所以显然。
realnumber 2# 2012-11-30 22:15
也可以这样,先证明$\frac{a}{b} \ge \frac{c}{d}$,那么有$\frac{a}{b} \ge \frac{a+c}{b+d}   \ge \frac{c}{d}$ 其中b>0,d>0;然后就是数学归纳法了
kuing 3# 2012-11-30 22:21
2# realnumber 嗯,也不错……
thread-929-1-6.html: [几何] 来自群的简单平几凸四边形
kuing 1# 2012-11-24 16:29
准教师-咪哆<1067******>  16:07:08 凸四边形ABCD中,AB=BC=CD,∠A:∠B:∠C=1:1:2,求∠D hint: 作角 C 的平分线,然后连那啥,大概就能证到全等了 示意图: 当然,不能说 SSA,因为没这条定理
第一章 2# 2012-11-24 21:48
为什么不是作角B的平分线?
kuing 3# 2012-11-24 22:36
2# 第一章 也可以啊,都是可以推出三个三角形都全等的。
isea 4# 2012-11-24 23:07
3# kuing 他的意思可能是,那样的话,两个全等都直接SAS
kuing 5# 2012-11-24 23:32
4# isea 噢…嗯,那的确是。
thread-93-1-2.html: 2
甄术 1# 2011-10-13 15:07
本帖最后由 甄术 于 2011-10-14 10:05 编辑 终于可以传图了! 是不是很二啊?
kuing 2# 2011-10-13 15:25
能的
甄术 3# 2011-10-13 19:08
再试试 被提示:无效的图片文件。
kuing 4# 2011-10-13 19:15
什么格式?
甄术 5# 2011-10-13 19:17
4# kuing png
kuing 6# 2011-10-13 19:27
png是可以发的,我之前也发过,用测试号也发过。 估计是跟浏览器有关,之前野猪在某些论坛也遇到过这种情况,但有时照发也能发出来,我也不知道怎么解决这个问题
kuing 7# 2011-10-13 19:31
搜索了一下,找到了这个:http://www.cnbeta.com/articles/53436.htm
甄术 8# 2011-10-13 19:32
6# kuing 我用的是IE7 您的是?
kuing 9# 2011-10-13 19:32
8# 甄术 我用 chrome
甄术 10# 2011-10-13 19:34
9# kuing 头次听说,露怯了
kuing 11# 2011-10-13 19:36
10# 甄术 chrome是google的浏览器 你先看看7#那个链接吧
甄术 12# 2011-10-13 19:39
7# kuing 等下再试试
甄术 13# 2011-10-13 21:34
改了设置,尝试下 依然无效
kuing 14# 2011-10-13 21:47
改完设置后重启过没
kuing 15# 2011-10-13 22:20
我把这个方法给了野猪看,他也没问题了。。。
甄术 16# 2011-10-14 10:03
14# kuing 好像没
甄术 17# 2011-10-14 10:04
再试 嗯,好了\(^o^)/~
kuing 18# 2011-10-14 12:34
那就行了 多谢
甄术 19# 2011-10-14 12:36
18# kuing 怎么谢我呢?因该是我表示感谢啊!
kuing 20# 2011-10-14 12:41
19# 甄术 因为当初野猪跟我说这个事的时候我并没去搜解决方法,全靠你这次再提起我才搜。
thread-93-2-2.html:
甄术 21# 2011-10-14 12:56
20# kuing 巧合巧合了!
kuing 22# 2011-10-19 01:53
21# 甄术 你再回一贴就四个2了 PS:有空seeseehttp://kkkkuingggg.5d6d.com/thread-119-1-1.html
甄术 23# 2011-10-31 11:39
21# 甄术 你再回一贴就四个2了 PS:有空seeseehttp://kkkkuingggg.5d6d.com/thread-119-1-1.html kuing 发表于 2011-10-19 01:53 这帖发表就可以成双的二了
thread-930-1-1.html: [几何] 来自群的曲线两点与定点构成正三角形
kuing 1# 2012-11-24 16:48
学生-独孤客(6108*****) 18. 已知点 $A(-1,-1)$。若曲线 $G$ 上存在两点 $B$, $C$,使 $\triangle ABC$ 为正三角形,则称 $G$ 为 $\Gamma$ 型曲线。 问:$y=-\dfrac1x$($x>0$)是不是。 群管-kuing  16:34:38 将曲线绕A旋转60度看跟原来的曲线有没有交点…… 目测是有的 学生--独孤客(6108*****)  16:39:56 逆时针还是顺时针 群管-kuing  16:40:42 都可以 如果顺时针有交点,则逆时针也一定有 学生--独孤客(6108*****)  16:41:01 哦 群管-kuing  16:41:44 有交点就是存在了
kuing 2# 2012-11-24 16:55
示意图: 当然可以具体求出旋转后的曲线计算交点坐标从而得到具体的B和C,不过我就懒得算了。 用此法可以将此题改编,比如研究存在更一般的等腰三角形情况的顶角的范围等……
转化与化归 3# 2012-11-24 18:24
我好像都是老老实实的用切线做的。
kuing 4# 2012-11-24 18:27
3# 转化与化归 切线?怎么做
转化与化归 5# 2012-11-24 18:29
过点的两条切线,看看夹角和60度的关系,当然具体算的时候也可以考虑一些特殊位置。
kuing 6# 2012-11-24 18:31
5# 转化与化归 这里也就一条切线吧……
转化与化归 7# 2012-11-24 18:32
这不就更好了!
isea 8# 2013-3-26 02:41
哎呀,今天,不,昨天碰到这题了,没细想
yes94 9# 2013-3-26 17:58
http://wenku.baidu.com/view/b85c1223192e45361066f578.html
kuing 10# 2013-3-26 18:00
9# yes94 出处dǎng牛比
thread-930-1-3.html: [几何] 来自群的曲线两点与定点构成正三角形
kuing 1# 2012-11-24 16:48
学生-独孤客(6108*****) 18. 已知点 $A(-1,-1)$。若曲线 $G$ 上存在两点 $B$, $C$,使 $\triangle ABC$ 为正三角形,则称 $G$ 为 $\Gamma$ 型曲线。 问:$y=-\dfrac1x$($x>0$)是不是。 群管-kuing  16:34:38 将曲线绕A旋转60度看跟原来的曲线有没有交点…… 目测是有的 学生--独孤客(6108*****)  16:39:56 逆时针还是顺时针 群管-kuing  16:40:42 都可以 如果顺时针有交点,则逆时针也一定有 学生--独孤客(6108*****)  16:41:01 哦 群管-kuing  16:41:44 有交点就是存在了
kuing 2# 2012-11-24 16:55
示意图: 当然可以具体求出旋转后的曲线计算交点坐标从而得到具体的B和C,不过我就懒得算了。 用此法可以将此题改编,比如研究存在更一般的等腰三角形情况的顶角的范围等……
转化与化归 3# 2012-11-24 18:24
我好像都是老老实实的用切线做的。
kuing 4# 2012-11-24 18:27
3# 转化与化归 切线?怎么做
转化与化归 5# 2012-11-24 18:29
过点的两条切线,看看夹角和60度的关系,当然具体算的时候也可以考虑一些特殊位置。
kuing 6# 2012-11-24 18:31
5# 转化与化归 这里也就一条切线吧……
转化与化归 7# 2012-11-24 18:32
这不就更好了!
isea 8# 2013-3-26 02:41
哎呀,今天,不,昨天碰到这题了,没细想
yes94 9# 2013-3-26 17:58
http://wenku.baidu.com/view/b85c1223192e45361066f578.html
kuing 10# 2013-3-26 18:00
9# yes94 出处dǎng牛比
thread-932-1-6.html: [几何] 来自群的椭圆乘积定值与平方和定值
kuing 1# 2012-11-26 18:45
题目:椭圆 $x^2/16+y^2/4=1$ 上有两点 $P$, $Q$,$O$ 为原点,若 $OP$, $OQ$ 斜率之积为 $-1/4$,则 $\abs{OP}^2+\abs{OQ}^2$ 的值为(  ) 解:作圆 $x^2+y^2=16$,分别过 $P$, $Q$ 作 $y$ 轴的平行线交圆于 $P'$, $Q'$,交 $x$ 轴于 $A$, $B$,连结 $OA$, $OB$, $OP'$, $OQ'$,所图所示。 对比椭圆与圆的方程可知 $P$, $Q$ 分别为 $AP'$, $BQ'$ 的中点,于是由 $OP$, $OQ$ 斜率之积为 $-1/4$ 可知 $OP'$, $OQ'$ 斜率之积为 $-1$,也就是说 $OP'\perp OQ'$,从而 $\triangle OAP'\cong\triangle Q'BO$,所以 \begin{align*} OP^2+OQ^2&=OA^2+PA^2+OB^2+QB^2\\ &=OA^2+\biggl(\frac{P'A}2\biggr)^2+OB^2+\biggl(\frac{Q'B}2\biggr)^2\\ &=OA^2+\biggl(\frac{P'A}2\biggr)^2+P'A^2+\left(\frac{OA}2\right)^2\\ &=\frac54P'O^2\\ &=20. \end{align*}
kuing 2# 2012-11-26 18:47
其他解法
isea 3# 2012-11-26 20:39
主楼这种构造实在令人叹为观止 碰到这烂题,看来都向这个方向试试
kuing 4# 2012-11-26 21:25
3# isea 主要是以前碰到过垂直时候的平方和定值问题,所以试图转化到垂直上,于是自然想到拉伸,将 -1/4 拉到 -1,正好椭圆也变成了圆,平方和才能化过去,所以这个题的数据不能乱给。
第一章 5# 2012-11-30 18:05
叹,围观之。
thread-933-1-1.html: 一个加权型的中值定理证明。。
q85669551 1# 2012-11-26 20:37

kuing 2# 2012-11-26 22:17
1# q85669551 “在 (a,b) 内导数存在且不为 0” 是指 “不恒为0” 还是 “恒不为0”?
q85669551 3# 2012-11-27 00:31
恒不为0。。神奇,我的一楼不见了。。刚才战巡说了说,明白了。。kk有什么方法,我是拿反函数的导数绕的。。
q85669551 4# 2012-11-27 00:32
2# kuing 我一回完 一楼就出现了。。
kuing 5# 2012-11-27 01:12
群里看不懂……也没什么法子……高数我弱
地狱的死灵 6# 2012-11-27 10:03
考研后好久都没撸过这种题了……
q85669551 7# 2012-11-27 12:57
6# 地狱的死灵 额。。比较简洁。。这种思路第一次见。。。
q85669551 8# 2012-11-27 12:58
5# kuing 6楼给的那个似乎还不错。。
kuing 9# 2012-11-27 14:50
6# 地狱的死灵 高,实在是高
、独木桥 10# 2012-12-2 22:52
这个问题还可以推广的。话说,LZ是不是厦大的?
地狱的死灵 11# 2012-12-3 19:49
10# 、独木桥 一样的, 就是把区间[a,b]分成m:n的点作为介值点, 用两次拉格朗日
kuing 12# 2012-12-3 20:11
m,n 要不要正数?
、独木桥 13# 2012-12-3 21:54
m,n为正数。没补充。证明如下,与上面的思路一样,忘记发上来了。
q85669551 14# 2012-12-4 19:42
10# 、独木桥 因何得此结论?是.
、独木桥 15# 2012-12-5 19:25
本帖最后由 、独木桥 于 2012-12-5 19:27 编辑 14# q85669551 因为这是厦大高数A期中考的附加题。  由此就可推出你应该是厦大的了。
q85669551 16# 2012-12-5 19:46
= = 问题关键是我是问你怎么知道此题来源。。貌似这个试卷都还没出官方的pdf,你也是厦大的?哪级什么学院?
、独木桥 17# 2012-12-5 19:59
16# q85669551 嗯,我也是厦大的。12级物理系,你呢?
q85669551 18# 2012-12-5 20:02
17# 、独木桥 软院的。。我倒很好奇你怎么来这儿的...
、独木桥 19# 2012-12-5 20:05
18# q85669551   我也是个数学爱好者,上网浏览网页的时候就发现这个论坛了.貌似以前Kuing帮我证明过一个很麻烦的不等式,有点印象然后就进来了。
q85669551 20# 2012-12-5 20:06
19# 、独木桥 加QQ 加QQ..
thread-933-2-1.html:
、独木桥 21# 2012-12-5 20:07
本帖最后由 、独木桥 于 2012-12-5 20:09 编辑 20# q85669551 告诉我你的吧。我加你。难道qq就是是你的ID名字?
kuing 22# 2012-12-5 20:09
19# 、独木桥 什么不等式……
q85669551 23# 2012-12-5 20:11
21# 、独木桥 显然不是啊。。2428921608 这才是.
、独木桥 24# 2012-12-5 20:18
22# kuing 就是这个了,以前在那个不等式爱好者那个群发出来的。不过现在我没在那个群了。
kuing 25# 2012-12-5 20:28
24# 、独木桥 噢,原来是这个,我好像还把指数弄更一般化了点的,不限整数。 那个群现在很少讨论不等式了,大多数都在讨论高等数学,所以群名也变了。
、独木桥 26# 2012-12-5 20:37
25# kuing 那群现在还收人吗?群号多少?
kuing 27# 2012-12-5 20:59
26# 、独木桥 好像满人了
thread-934-1-1.html: 定积分
froglove 1# 2012-11-26 22:43
∫ 0-4 x√(1-x^2)/(1+x^2) 不太会用那个代码。。。
kuing 2# 2012-11-26 23:01
∫ 0-4 x√(1-x^2)/(1+x^2) 不太会用那个代码。。。 froglove 发表于 2012-11-26 22:43 根号的范围到哪里?是 $\frac{x\sqrt{1-x^2}}{1+x^2}$ 还是 $x\sqrt{\frac{1-x^2}{1+x^2}}$? PS1、还是试着看置顶写代码吧,不然容易像上面那样有歧义不知根号到哪里。 PS2、如果是前者三角代换有机会,后者不太会…… PS3、积分上限?4?
froglove 3# 2012-11-27 19:10
本帖最后由 froglove 于 2012-11-27 19:11 编辑 分子分母都根号。。不幸是后者。。宿舍网慢。。那个太多图的教程帖好难打开。。。。
kuing 4# 2012-11-27 19:43
那那个上限怎么回事哩?你打的“∫ 0-4”是否表示从0到4的积分?但定义域也就 [0,1]……
kuing 5# 2012-11-27 19:59
换元居然恰好去根号 令 \[\sqrt{\frac{1-x^2}{1+x^2}}=t,\] 可以解得 \begin{align*} x&=\sqrt{\frac{1-t^2}{1+t^2}},\\ \rmd x&=\rmd{\sqrt{\frac{1-t^2}{1+t^2}}}=-\frac{2t}{(1+t^2)^2}\sqrt{\frac{1+t^2}{1-t^2}}\rmd t, \end{align*} 得到 \[x\rmd x=-\frac{2t}{(1+t^2)^2}\rmd t,\] 所以 \[\int x\sqrt{\frac{1-x^2}{1+x^2}}\rmd x=-2\int\frac{t^2}{(1+t^2)^2}\rmd t,\] 然后应该有固定套路了……
froglove 6# 2012-11-27 21:21
上限为1,打错了。。居然是这样暴力解得。。o_O
kuing 7# 2012-11-27 21:31
6# froglove 也许有简单方法吧……我不在行……O_o
froglove 8# 2012-11-27 21:42
这大小眼。。好对称==
kuing 9# 2012-11-27 21:49
啊!突然想起半角公式,有更简单解法了 令 $x=\sqrt{\cos t}$,由半角公式,有 \begin{align*} \int x\sqrt{\frac{1-x^2}{1+x^2}}\rmd x&=\int\sqrt{\cos t}\cdot \sqrt{\frac{1-\cos t}{1+\cos t}}\rmd{\sqrt{\cos t}} \\ & =\int\sqrt{\cos t}\cdot \tan\frac t2\cdot \frac{-\sin t}{2\sqrt{\cos t}}\rmd t \\ & =-\int\sin^2\frac t2\rmd t\\ & =\cdots \end{align*}
froglove 10# 2012-11-27 21:53
目测这个像正解。。为毛我只想到万能公式还把自己绕进去了。。
thread-935-1-6.html: [函数] 请教:辽宁本溪一中高三第二次月考第16题
shidilin 1# 2012-11-26 23:01
请教:辽宁本溪一中2013届上学期高三年级第二次月考数学(理)试题第16题
kuing 2# 2012-11-26 23:43
依题意不妨设 $f(x)=ax^2(x-b)$,其中 $a$, $b\ne0$,则 $f'(x)=ax(3x-2b)$,设 $Q(m,n)$,其中 $0<m\leqslant 2$,则依题意有 \[\left\{\begin{aligned} am^2(m-b)&=n,\\ 3m-2b&=0,\\ 1+\sqrt{2m-m^2}&=n, \end{aligned}\right.\] 解得 \[\left\{\begin{aligned} a&=-\frac{2\sqrt{2m-m^2}+2}{m^3},\\ b&=\frac{3m}2, \end{aligned}\right.\] 从而 \[f'(x)=\frac{6x(m-x)\bigl(\sqrt{2m-m^2}+1\bigr)}{m^3},\] 因此曲线 $y=f(x)$ 的切线斜率的最大值为 \[g(m)=\frac{3\sqrt{2m-m^2}+3}{2m},\] 求导可证 $g(m)$ 在 $(0,2]$ 上递减,所以所求的最小值就是 $g(2)=3/4$。
kuing 3# 2012-11-26 23:53
最后也可以不求导,搞个三角代换,或者变成两点连线斜率什么的都可以。
shidilin 4# 2012-11-27 18:22
谢谢kk版主! 看来玄机都在设  $f(x)=ax^2(x-b)$ 我设成了  $f(x)=ax^3+bx^2$   到后面运算很繁,只好作罢。 是否一开始设成 $f(x)=ax^2(2x-3b)$ 效果更好些?
kuing 5# 2012-11-27 18:37
4# shidilin 理论上这些都行,就看哪个解出来的样子简洁些,本质没变
hongxian 6# 2012-11-29 20:02
本帖最后由 hongxian 于 2012-11-30 08:35 编辑 好题,不过作一个填空题有点浪费了, 设$f'(x)=ax(x-m)$,则$f(x)=\frac{ax^3}{3}-\frac{amx^2}{2}$ 好象也可以完成,剩下的和k版的方法没有什么区别
kuing 7# 2012-11-29 20:23
6# hongxian 嗯,完全可以放在大题……
第一章 8# 2012-11-30 07:57
这种题,难道不是数形结合? 那个曲线显然是半圆,而三次函数的导数是开口向下的抛物线啊, 就是在$0<b<=-3a$的条件下,求$-b^2/3a$的最小值, 还没想清楚。
thread-936-1-6.html: [几何] 最多可作几条大于90°的射线--以及怎么推广?
realnumber 1# 2012-11-27 12:00
本帖最后由 realnumber 于 2012-12-1 10:23 编辑 从空间一点最多可作几条两两夹角大于90°的射线? 5点不可以 证明:选定2点以及球心,把球面剖成2部分,易得有个半球面上至少有4个点(注意:边界上点,可以认为同在两个半球面上) 再过这2点中一点,把这个半球分割成4块1/8球面,容易得某一块有2点,这2点所成角不超过直角,矛盾,完 四点可以显然了
realnumber 2# 2012-11-27 12:35
本帖最后由 realnumber 于 2012-11-27 12:46 编辑 先放向量a,b,c,d 沈-- 12:21:08 两两钝角 沈-- 12:21:13 然后放向量e 沈-- 12:21:56 e必然在某三条射线 内部,不妨a,b,c 沈-- 12:22:28 分解e=xa+yb+zc  x,y,z均为正 Real 12:22:55 没问题,继续 沈-- 12:23:01 考虑e乘a e乘b  e乘c 沈-- 12:23:09 使得均为负 沈-- 12:23:23 可以得矛盾 沈-- 12:24:56 传统解法 从空间一点最多可作几条两两夹角大于90°的射线? 分析:注意到三面角的三个面角两两之和小于四个直角,故可用三个钝角为面角做成一个三面角进行考察. 解:做一个三个面角均为钝角的三面角V-ABC,取VA=VB=VC.过A,B,C作平面ABC,作VO⊥平面ABC,垂足为O.显然后∠AVO=∠BVO=∠CVO<90°. 取VO的反向延长线VD,则四射线VA,VB,VC,VD两两夹角均大于90°. 如果这样的直线至少有5条,设为OA,OB,OC,OD,OE,过O作平面α⊥OE,则其余四条射线必与OE不在平面α的同侧(否则该射线与OE所成角将小于90°),而O-ABCD成四面角. 同理过O作其余四条射线中任意一条的垂面β,则其余三条射线也必在垂面β的另一侧. 由此可断定O-ABCD为凸四面角,从而四个面角之和小于360°,因而这四个角中至少有一个面角小于90°,与题设四个面角均大于90°矛盾. 故这样的射线最多只能引4条
realnumber 3# 2012-11-27 12:37
推广: 最少几条射线同一起点,则存在2条成60度? 改成70度又如何?
thread-937-1-6.html: [几何] 轨迹形状 伴随圆
v6mm131 1# 2012-11-28 19:57
不妨试试这题
kuing 2# 2012-11-28 20:29
伴随圆那时候是切线垂直,现在相当于是法线垂直,解法大概也类似了,等会抽空写写
kuing 3# 2012-11-28 21:28
才发现比切线复杂多了,方程次数高了…… $a^4 b^4 x^4 - 2 a^2 b^6 x^4 + b^8 x^4 - a^2 b^4 x^6 - b^6 x^6 - 2 a^6 b^2 x^2 y^2 + 4 a^4 b^4 x^2 y^2 - 2 a^2 b^6 x^2 y^2 - 2 a^4 b^2 x^4 y^2 - 3 a^2 b^4 x^4 y^2 - b^6 x^4 y^2 + a^8 y^4 - 2 a^6 b^2 y^4 + a^4 b^4 y^4 - a^6 x^2 y^4 - 3 a^4 b^2 x^2 y^4 - 2 a^2 b^4 x^2 y^4 - a^6 y^6 - a^4 b^2 y^6 = 0.$ 代入 $a=4$, $b=3$ 后得到方程为 \[3969 x^4 - 2025 x^6 - 14112 x^2 y^2 - 9225 x^4 y^2 + 12544 y^4 - 13600 x^2 y^4 - 6400 y^6=0.\] 作图得D
yuzi 4# 2012-11-29 09:39
。。。要命
abababa 5# 2012-11-29 09:56
能不能设参数$(\cos \theta, \sin \theta)$,然后切点弦好求,用韦达定理求切点弦的中点M,M也是PC的中点,C就求出来了
kuing 6# 2012-11-29 15:00
5# abababa 这方向也不错,等会试试
kuing 7# 2012-11-29 20:32
椭圆 $x^2/a^2+y^2/b^2=1$,其伴随圆 $x^2+y^2=a^2+b^2$ 上有动点 $P(x_p,y_p)$,相应的与椭圆的两切线 $PA$, $PB$,其中 $A$, $B$ 为切点且这两点处的法线交于 $C(x_c,y_c)$,线段 $AB$ 的中点为 $D(x_d,y_d)$。 利用切点弦结论,有 \[l_{AB}:\frac{x_px}{a^2}+\frac{y_py}{b^2}=1,\] 另一方面,利用中点弦结论,有 \[l_{AB}:\frac{x_dx}{a^2}+\frac{y_dy}{b^2}=\frac{x_d^2}{a^2}+\frac{y_d^2}{b^2},\] 于是对比得 \[\frac{x_d}{x_p}=\frac{y_d}{y_p}=\frac{x_d^2}{a^2}+\frac{y_d^2}{b^2},\] 由于 $P$ 在伴随圆上,即有 $PA\perp PB$,于是四边形 $PACB$ 为矩形,所以 $D$ 也是线段 $PC$ 的中点,即有 $x_p+x_c=2x_d$ 且 $y_p+y_c=2y_d$,从而 \[\frac{2(x_p+x_c)}{x_p}=\frac{2(y_p+y_c)}{y_p}=\frac{(x_p+x_c)^2}{a^2}+\frac{(y_p+y_c)^2}{b^2},\] 解这个方程组,得 \[\left\{\begin{aligned} x_c&=-x_p,\\ y_c&=-y_p, \end{aligned}\right.~\text{或}~\left\{\begin{aligned} x_c&=x_p\left( \frac2{x_p^2/a^2+y_p^2/b^2}-1 \right),\\ y_c&=y_p\left( \frac2{x_p^2/a^2+y_p^2/b^2}-1 \right), \end{aligned}\right.\] 如果 $x_c=-x_p$ 且 $y_c=-y_p$ 意味着 $D$ 为原点,这显然不可能,因此可以排除前者。再对后者令 $x_p=\sqrt{a^2+b^2}\cos \theta $ 且 $y_p=\sqrt{a^2+b^2}\sin \theta$ 即得 \[\left\{\begin{aligned} x_c&=\frac{\cos \theta }{\sqrt{a^2+b^2}}\left( \frac2{\cos^2\theta/a^2+\sin^2\theta/b^2}-a^2-b^2 \right),\\ y_c&=\frac{\sin \theta }{\sqrt{a^2+b^2}}\left( \frac2{\cos^2\theta/a^2+\sin^2\theta/b^2}-a^2-b^2 \right), \end{aligned}\right.\] 这就是点 $C$ 的轨迹的参数方程。
kuing 8# 2012-11-29 20:50
经检验,上述参数方程满足3#的方程。 感谢楼上上提供的思路 此外,由证明过程可以看出,直线PD必过原点,并且半不限于伴随圆。
力工 9# 2012-11-30 17:28
7# kuing 晕题了,看着头晕,只有kuing有这功力。
kuing 10# 2012-11-30 17:58
9# 力工 不不,思路是 abababa 提供的,我是沿着这方向玩,不过我没用韦达定理,而是直接用两个已知结论对比得到弦中点的表示,相对简单了些。
kuing 11# 2012-12-1 14:36
话说我想知道楼主的图是用什么作图工具画的,还有楼主的解法如何
thread-938-1-1.html: 无穷乘积的一致收敛
秋风树林 1# 2012-11-29 09:11
本帖最后由 秋风树林 于 2012-11-29 09:13 编辑 做倒是做出来了,不过有些小地方还没完善,所以就先不发了 中间那个花写体表示全纯函数列 并且没有一个函数恒等于0 区域是复平面的区域
秋风树林 2# 2012-11-29 13:38
对于这个题目,我有两种思路 1.利用两边同时取log再交换求导与极限的顺序,形式上看起来是很一致的 但是其一是log的单值支问题,我觉得这个不构成太大问题,只要log f的这个级数一致收敛 那么在一个小圆盘上能做到一致收敛同时小圆盘单连通故可取到单值支,由紧集的性质可知在整个紧集上都能一致收敛 问题我觉得出现在两个方面,一个就是log f 的一致收敛暂时还没去想过怎么证 第二个致命的地方在于log(a*b)不一定就等于log a+log b,事实上大部分情况是不等的 2.利用余项,证余项一致收敛于0 余项的形式倒是很简单,比如令g_n(z)为从第n项开始后的连乘积 这家伙全纯并且与f(z)的导数有个很简单的递推关系 余项则为g_n'(z)/g_n(z),而下面这个一致趋于1,上面一致收敛于0(交换求导与极限顺序) 当然,我在证的时候感觉有些比如一致有界这个东西没去仔细说清楚,就不发仔细的证明了 这就是我的想法 帮忙看看
kuing 3# 2012-11-29 15:01
看都看不懂鸟……
秋风树林 4# 2012-11-29 15:34
看都看不懂鸟…… kuing 发表于 2012-11-29 15:01 kk和我的方向不一样嘛... 你的很多东西我也看不懂啊...
都市侠影 5# 2012-11-30 13:40
复变的不懂.........
thread-939-1-6.html: [不等式] inequality with a+b+c=3
pxchg1200 1# 2012-11-29 18:27
Let $a,b,c$ be nonnegative real numbers such that$a+b+c=3 $ . Prove that \[ \frac{1}{2+a^3b}+\frac{1}{2+b^3c}+\frac{1}{2+c^3a}\geq 1. \]
thread-94-1-2.html: 极限 $(\sin x)^{\tan x},x\to\pi/2$
kuing 1# 2011-10-13 15:56
求极限 \[\lim_{x\to\pi/2}(\sin x)^{\tan x} \qquad\cdots(1)\] 这类我通常都取个对数再说,为求式 $(1)$ 先求 \[ \lim_{x\to\pi/2}\tan x\ln\sin x \qquad\cdots(2) \] 为了使用 $e$ 的那个重要极限,令 $\sin x=1-\dfrac1t$,我们先算由左侧极限,当 $x\to\dfrac{\pi}2-$ 时 $t\to+\infty$,且有 $\tan x=\dfrac{t-1}{\sqrt{2t-1}}$,那么 \[ \lim_{x\to\pi/2-}\tan x\ln\sin x = \lim_{t\to+\infty}\dfrac{t-1}{\sqrt{2t-1}}\ln\left(1-\frac1t\right) =\lim_{t\to+\infty}\dfrac{t-1}{-t\sqrt{2t-1}}\ln\left(1+\frac1{-t}\right)^{-t} \] 于是显然为0,右侧极限类似,故式 $(2)$ 为0,即式 $(1)$ 为1
kuing 2# 2011-10-13 16:00
这样看来取对数其实还是挺多余的……
①②③④⑤⑥⑦ 3# 2011-10-14 09:42
我看到的是这样的 是行间公式\[前没换行导致的?
kuing 4# 2011-10-14 12:29
3# ①②③④⑤⑥⑦ 呃,奇怪,这个贴不止你一个反映这情况,难道是因为我用了\tag?但是我用IE6和chrome看都正常。 我试试全部换行你再看看
kuing 5# 2011-10-14 12:30
修改了,麻烦再看看
①②③④⑤⑥⑦ 6# 2011-10-14 12:59
5# kuing 还是看不了,看来是tag的问题
kuing 7# 2011-10-14 13:37
6# ①②③④⑤⑥⑦ 噢,那我不用tag了,再修改一下先
海盗船长 8# 2011-10-14 20:11
$\lim_{x\to\pi/2}(\sin x)^{\tan x}$ $=\lim_{x\to 0}(\cos x)^{-\cot x}$ $=\lim_{x\to 0}(\cos x)^{-\frac{\cos x}{\sin x}}$ $=\lim_{x\to 0}((1+\cos x-1)^{\frac{1}{\cos x-1}\left(-\frac{\cos x (\cos x-1)}{\sin x}\right)}$ $=\lim_{x\to 0}((1+\cos x-1)^{\frac{1}{\cos x-1}\left(-\left(\frac{2\sin^2{\frac{x}{2}}}{\sin x}-\sin x\right)\right)}$ $=1$
kuing 9# 2011-10-14 21:36
8# 海盗船长 嗯,可以。要是我的话那个负号就不要了,这样可以打少些括号。PS:后面两行多了左括号……
鱼儿 10# 2011-10-20 15:38
本帖最后由 鱼儿 于 2011-10-24 12:56 编辑 另解  令$x=\frac{\pi}{2}-t$,则 $\lim_{x\to\frac{\pi}{2}}(\sin x)^{\tan x}=\lim_{t\to0}(\cos t)^{\cot t}$ $=\lim_{t\to0}(\cos^2t)^\frac{\cot t}{2}$ $=\lim_{t\to0}\frac{1}{(1+\tan^2t)^\frac{\cot t}{2}}$ $=\frac{1}{\lim_{t\to0}[(1+\tan^2t)^{\cot^2t}]^{\frac{\tan t}{2}}}$ $=\frac{1}{e^0}$ $=1$
kuing 11# 2011-10-20 17:36
10# 鱼儿 开头是否应为 $x=\dfrac\pi2-t$ PS, 可以试试用align环境去输入这种多行等号的公式。
鱼儿 12# 2011-10-24 12:59
11# kuing 是的,已经在帖子中修改了。谢谢指正!
thread-940-1-6.html: [不等式] 最近比较流行的几个Vasc不等式
pxchg1200 1# 2012-11-29 18:49
本帖最后由 pxchg1200 于 2012-11-29 18:51 编辑 1.If$a,b,c$   are nonnegative real numbers such that$ab+bc+ca=4$ , then \[ \sqrt{a^2+9bc}+\sqrt{b^2+9ca}+\sqrt{c^2+9ab}\ge 10. \] 2. If$a,b,c$   are nonnegative real numbers such that$ab+bc+ca=3$ , then \[ \sqrt{a^2+4bc}+\sqrt{b^2+4ca}+\sqrt{c^2+4ab}\ge\sqrt{a^2+b^2+c^2+42}. \] 3. This inequality inspires other similar nice inequalities for $a,b,c\ge 0$. $(A) \ \ \ \  \sum\sqrt{(ab+c^2)(ac+b^2)}\le \frac 3{4}(a+b+c)^2;$ $(B) \ \ \ \  \sum\sqrt{(a^2+ab+b^2)(a^2+ac+c^2)}\ge (a+b+c)^2;$ $(C) \ \ \ \  \sum\sqrt{(9ab+c^2)(9ac+b^2)}\ge 7(ab+bc+ca);$ $(D) \ \ \ \  \sum\sqrt{(a^2+7ab+b^2)(a^2+7ac+c^2)}\ge 7(ab+bc+ca).$ Also, the cyclic inequality holds: $(B1) \ \ \ \  \sum\sqrt{(a^2+b^2+bc)(b^2+c^2+ca)}\ge (a+b+c)^2;$
都市侠影 2# 2012-11-30 13:45
有何流行之处?
thread-941-1-6.html: [不等式] 这次有没算错?一个组合不等式
realnumber 1# 2012-11-30 10:20
本帖最后由 realnumber 于 2012-12-28 11:06 编辑 http://tieba.baidu.com/p/1231401354?pn=1 解答: _________kuing edited in $\LaTeX$_________ 正项数列 $\{a_n\}$,$a_1 \leqslant a_2 \leqslant \cdots \leqslant a_n $,$0 < m < n$,$m$、$n \in \mbb N^ + $,证明: \begin{align*} &(C_m^m a_m + C_{m + 1}^m a_{m + 1} + \cdots + C_n^m a_n )^2\\ \geqslant{}& (C_{m - 1}^{m - 1} a_{m - 1} + C_m^{m - 1} a_m + \cdots + C_n^{m - 1} a_n )(C_{m + 1}^{m + 1} a_{m + 1} + \cdots + C_n^{m + 1} a_n ). \end{align*} 以下用数学归纳法证明。 \begin{align*} \text{左边} = {}&(C_m^m a_m + \cdots + C_{n - 1}^m a_{n - 1} )^2 + 2C_n^m a_n (C_m^m a_m + \cdots + C_{n - 1}^m a_{n - 1} ) + (C_n^m a_n)^2,\\ \text{右边} = {}&(C_{m - 1}^{m - 1} a_{m - 1} + \cdots + C_{n - 1}^{m - 1} a_{n - 1} )(C_{m + 1}^{m + 1} a_{m + 1} + \cdots + C_{n - 1}^{m + 1} a_{n - 1} )\\ & + C_n^{m + 1} a_n (C_{m - 1}^{m - 1} a_{m - 1} + \cdots + C_{n - 1}^{m - 1} a_{n - 1} ) + C_n^{m - 1} a_n (C_{m + 1}^{m + 1} a_{m + 1} + \cdots + C_{n - 1}^{m + 1} a_{n - 1} )\\ & + C_n^{m - 1} C_n^{m + 1} a_n^2, \end{align*}由归纳假设 \[ (C_m^m a_m + \cdots + C_{n - 1}^m a_{n - 1} )^2 \geqslant (C_{m - 1}^{m - 1} a_{m - 1} + \cdots + C_{n - 1}^{m - 1} a_{n - 1} )(C_{m + 1}^{m + 1} a_{m + 1} + \cdots + C_{n - 1}^{m + 1} a_{n - 1} ), \]所以只需要证明: \begin{align*} & 2C_n^m a_n (C_m^m a_m + \cdots + C_{n - 1}^m a_{n - 1} ) + (C_n^m a_n )^2\\ \geqslant{}& C_n^{m + 1} a_n (C_{m - 1}^{m - 1} a_{m - 1} + \cdots + C_{n - 1}^{m - 1} a_{n - 1} ) + C_n^{m - 1} a_n (C_{m + 1}^{m + 1} a_{m + 1} + \cdots + C_{n - 1}^{m + 1} a_{n - 1} )\\ & + C_n^{m - 1} C_n^{m + 1} a_n^2, \end{align*}两边除以 $a_n$,即 \begin{align} &2C_n^m (C_m^m a_m + \cdots + C_{n - 1}^m a_{n - 1} ) + (C_n^m )^2a_n\notag\\ \geqslant{}& C_n^{m + 1} (C_{m - 1}^{m - 1} a_{m - 1} + \cdots + C_{n - 1}^{m - 1} a_{n - 1} ) + C_n^{m - 1} (C_{m + 1}^{m + 1} a_{m + 1} + \cdots + C_{n - 1}^{m + 1} a_{n - 1} )\notag\\ & + C_n^{m - 1} C_n^{m + 1} a_n,\label{zhbds20121130s1} \end{align}以上是 $a_n $ 的一次不等式,只需要证明一次项系数为正,且 $a_n = a_{n - 1} $ 成立即可。 \[ (C_n^m )^2 > C_n^{m - 1} C_n^{m + 1} \iff n - m + 1 > n - m \iff 1 > 0, \]又 $a_n = a_{n - 1} $ 时,又可以看作 $a_{n - 1} $ 的一次不等式,只需要证明一次项系数为正,且 $a_{n - 1} = a_{n - 2} $ 成立即可,……,如此要使得式 \eqref{zhbds20121130s1} 成立,只需要证明,对任意正整数 $k$($m \leqslant k \leqslant n - 1$),有 \begin{align*} &2C_n^m (C_k^m + \cdots + C_{n - 1}^m ) + (C_n^m )^2 - C_n^{m - 1} C_n^{m + 1}\\ &- C_n^{m + 1} (C_k^{m - 1} + \cdots + C_{n - 1}^{m - 1} ) - C_n^{m - 1} (C_k^{m + 1} + \cdots + C_{n - 1}^{m + 1} ) > 0, \end{align*}以及 $a_1 = a_2 = \cdots = a_n$ 时,式 \eqref{zhbds20121130s1} 成立。 $a_1 = a_2 = \cdots = a_n$ 时,式 \eqref{zhbds20121130s1} 为 \begin{align*} &2C_n^m C_n^{m + 1} + (C_n^m )^2 \geqslant C_n^{m + 1} C_n^m + C_n^{m - 1} C_n^{m + 2} + C_n^{m - 1} C_n^{m + 1}\\ \iff{}& C_n^m C_n^{m + 1} + (C_n^m )^2 \geqslant C_n^{m - 1} C_n^{m + 2} + C_n^{m - 1} C_n^{m + 1}\\ \iff{}& C_n^m C_{n + 1}^{m + 1} \geqslant C_n^{m - 1} C_{n + 1}^{m + 2}\\ \iff{}& \frac{m + 2}m \cdot \frac{n - m + 1}{n - m} > 1, \end{align*}以下证明:对任意正整数 $k$($m \leqslant k \leqslant n - 1$),有 \begin{align*} & 2C_n^m (C_k^m + \cdots + C_{n - 1}^m ) + (C_n^m )^2 - C_n^{m - 1} C_n^{m + 1}\\ &- C_n^{m + 1} (C_k^{m - 1} + \cdots + C_{n - 1}^{m - 1} ) - C_n^{m - 1} (C_k^{m + 1} + \cdots + C_{n - 1}^{m + 1} ) > 0\\ \iff{}& 2C_n^m (C_n^{m + 1} - C_k^{m + 1} ) + (C_n^m )^2 - C_n^{m - 1} C_n^{m + 1} - C_n^{m + 1} (C_n^m - C_k^m ) - C_n^{m - 1} (C_n^{m + 2} - C_k^{m + 2} ) > 0\\ \iff{}& C_n^m C_{n + 1}^{m + 1} - C_n^{m - 1} C_{n + 1}^{m + 2} > 2C_n^m C_k^{m + 1} - C_n^{m + 1} C_k^m - C_n^{m - 1} C_k^{m + 2}\\ \iff{}& C_{n + 1}^{m + 1} \left(1 - \frac m{m + 2} \cdot \frac{n - m}{n - m + 1}\right) > C_k^{m + 1} \left(2 - \frac{n - m}{k - m} - \frac m{m + 2} \cdot \frac{k - m - 1}{n + 1 - m}\right), \end{align*}又 \begin{align*} &\frac{C_{n + 1}^{m + 1} }{C_k^{m + 1} } \geqslant \frac{C_{n + 1}^{m + 1} }{C_{n - 1}^{m + 1} } = \frac{n(n + 1)}{(n - m)(n - m - 1)}\\ \liff{}& \frac{n(n + 1)}{(n - m)(n - m - 1)}\left(1 - \frac m{m + 2} \cdot \frac{n - m}{n - m + 1}\right) > 2 - \frac{n - m}{k - m} - \frac m{m + 2} \cdot \frac{k - m - 1}{n + 1 - m} = f(k - m), \end{align*}(注:$f( x ) = - \frac m{m + 2} \cdot \frac x{n + 1 - m} - \frac{n - m}x + C_0$ 类型),在 $m \leqslant k \leqslant n - 1$ 单调递增。 当 $k = n - 1$ 时, \begin{align*} & \liff \frac{n(n + 1)}{(n - m)(n - m - 1)}\left(1 - \frac m{m + 2} \cdot \frac{n - m}{n - m + 1}\right) > 2 - \frac{n - m}{n - 1 - m} - \frac m{m + 2} \cdot \frac{n - m - 2}{n + 1 - m}\\ & \iff \frac{n(n + 1)}{(n - m)(n - m - 1)}\left(1 - \frac m{m + 2} \cdot \frac{n - m}{n - m + 1}\right) > \frac{n - m - 2}{n - 1 - m} - \frac m{m + 2} \cdot \frac{n - m - 2}{n + 1 - m}\\ & \begin{aligned} \liff \frac{(n + 1)}{(n - m)(n - m - 1)}\left(1 - \frac m{m + 2}\right) &\geqslant \frac1{n - 1 - m} - \frac m{m + 2} \cdot \frac1{n + 1 - m}\\ & = \frac1{n + 1 - m}\left(\frac2{n - 1 - m} + 1 - \frac m{m + 2}\right) \end{aligned}\\ & \iff \frac{(n + 1)}{(n - m)(n - m - 1)} \cdot \frac1{m + 2} \geqslant \frac1{n + 1 - m} \cdot \frac{n + 1}{(n - 1 - m)(m + 2)}\\ & \iff (n + 1 - m) \geqslant (n - m) \iff 1 \geqslant 0. \end{align*}
都市侠影 2# 2012-11-30 13:42
题也不打上来,解答也不打上来,略过.......
realnumber 3# 2012-11-30 13:56
题目有连接,解答在附件里
kuing 4# 2012-11-30 15:49
等我把楼主的解答敲上来吧。
kuing 5# 2012-11-30 16:13
总算敲好了,已编辑在一楼,还没细看内容。 PS、式子都有点长,不太好打,编辑了好几次,不过应该没有打错,楼主可以核对一下。
第一章 6# 2012-11-30 18:03
k敲这个用了多长时间? 15:49----16:13?
kuing 7# 2012-11-30 18:10
6# 第一章 差不多就这个时间吧,连同后来小修改几次的时间,主要是公式长,断行麻烦……
thread-942-1-5.html: 怎么证明运输的效率最高?
realnumber 1# 2012-11-30 21:59
本帖最后由 realnumber 于 2012-12-1 09:44 编辑 一个商人骑一头驴要穿越1000公里长的沙漠,去卖3000根胡萝卜。已知驴一次性可驮1000根胡萝卜,但每走1公里又要吃掉1根胡萝卜。问:商人最多可卖出多少胡萝卜?---可以百度“3000胡萝卜” 这样可以得到最佳(是这样吗?) 答案:1.运到离起点x公里A处,那么“来回来回来”共有胡萝卜$2(1000-2x)+1000-x=2000$,解得$x=200$ 2.再离A地y公里的地方B,“来回来",共有胡萝卜$1000-2y+1000-y=1000$,解得$y=\frac{1000}{3}$, 3.那么此时带着1000根胡萝卜,还要走$1000-x-y$的路,那么有胡萝卜$x+y$根。 看起来是最优解答,我的问题是,怎么证明它是最佳的? 这样可以吗?设起点有1000n根胡萝卜,走x公里到A处(0<x<1000),那么全部运输到A处时,A处理有1000n-2nx+x根 消耗2nx-x,有x公里,每公里消耗率(?)为2n-1,这个数据与x无关,说明满载的话,走第一次与距离无关,但与总数有关 分2次走的话,第1次运到越多,接下来的消耗率越高,但似乎距离就短了------有些凌乱了-----
realnumber 2# 2012-12-4 07:45
大致有点明白了,如果满载所有货物运到某处效率仅与总量有关(2n-1),(距离越短越好,但要保证满载的话,所有货物运输到下一地点,正好消耗一次满载最优),
realnumber 3# 2012-12-22 07:51
本帖最后由 realnumber 于 2012-12-22 08:30 编辑 推广问题:有1000n+r根,n,r为正整数,0<r<1000,可以走多远? 1.丢弃不成立的,y+x,可以消耗x,把y向前搬动. 猜测是:$\frac{r}{2n+1}+\frac{1000}{2n-1}+\frac{1000}{2n-3}+...+\frac{1000}{1}$ 证明:其他走法$\frac{x_{1}}{2n+1}+\frac{x_{2}}{2n-1}+\frac{x_{3}}{2n-3}+...+\frac{x_{2n-1}}{1}$ 其中$Σx_{i}≤1000n+r,且0≤x_{i}≤1000$.显然是猜测的那个最大(说明:分母是每段来和回的次数) 2.各类花样,比如来回中途多次续传,可以等价于某个依次传输.若运输次数相等,则可以合并 3.问题推广到多条线路或平面,立体模式,最好与加油站运输油料接近(就这条未完成了).
thread-943-1-2.html: [函数] 两函数最短距离
v6mm131 1# 2012-12-1 17:48
两函数最短距离
kuing 2# 2012-12-1 17:59
为什么不改成 $y=\sqrt x$
realnumber 3# 2012-12-1 21:17
本帖最后由 realnumber 于 2012-12-1 22:36 编辑 好象超越方程解不了,假设$A(a,a^2),B(b,lnb)$两点距离最近,那么A,B处的切线平行,且和AB垂直 那么有$2a=\frac{1}{b}$且$2a \times  \frac{a^2-\ln b}{a-b}=-1$
第一章 4# 2012-12-1 21:40
这题改编的吧?选择题?
kuing 5# 2012-12-1 22:28
3# realnumber oh,好像真是超越了 PS、lnb 应为 \ln b ,× 代码 \times
yes94 6# 2013-2-1 16:07
[quote]为什么不改成 $y=\sqrt x$ kuing 发表于 2012-12-1 17:59 [/quo 为什么不改成$y=e^x$
kuing 7# 2013-2-1 16:08
6# yes94 因为原先就是 $e^x$……
yes94 8# 2013-2-1 16:09
6# yes94 因为原先就是 $e^x$…… kuing 发表于 2013-2-1 16:08 改来改去,就还原了!
kuing 9# 2013-2-1 16:11
嗯,还不能乱改,一下就超越了
yes94 10# 2013-3-26 00:10
2010年或2012年高考选择题,记不得了。 设直线$x=t$与函数$f(x)=x^2$,$g(x)=\ln x$的图象分别交于点$M,N$,则当$|MN|$达到最小时$t$的值为_______
zwl1972 11# 2013-3-28 17:14
本帖最后由 zwl1972 于 2013-3-28 17:15 编辑 最近的版本是:$~e^x+x=a~$的零点为$~x_1~$,$~\ln x+x=a~$的零点是$~x_2,$则$~|x_1-x_2|_{min}=$____.
yes94 12# 2013-3-28 17:59
11# zwl1972 直线$y=x$与$y=a-x$垂直,反函数
yes94 13# 2013-3-28 18:01
最近的版本是:$~e^x+x=a~$的零点为$~x_1~$,$~\ln x+x=a~$的零点是$~x_2,$则$~|x_1-x_2|_{min}=$____. zwl1972 发表于 2013-3-28 17:14 还可以求$x_1+x_2$的值吧?其中$a$是常数。
kuing 14# 2013-4-21 13:06
顺便记录刚才搜贴搜到的链接:http://bbs.pep.com.cn/forum.php?mod=viewthread&tid=2457414
yes94 15# 2013-4-21 13:54
14# kuing 把题弄上来:
yes94 16# 2013-4-21 23:40
15# yes94 搞一个不等式的解答,不用反函数 \[\begin{array}{l} P({x_1},{y_1}),Q({x_2},{y_2}),|PQ| = \sqrt {{{({x_1} - {x_2})}^2} + {{({y_1} - {y_2})}^2}} \\ {y_1} = \frac{1}{2}{e^{{x_1}}} = {e^{ - \ln 2}}{e^{{x_1}}} = {e^{{x_1} - \ln 2}} \geqslant {x_1} - \ln 2 + 1\\ {y_2} = \ln 2x_2 = \ln 2 + \ln {x_2} \leqslant \ln 2 + {x_2} - 1\\ {y_1} - {y_2} \geqslant ({x_1} - \ln 2 + 1) - (\ln 2 + {x_2} - 1) = {x_1} - {x_2} - 2\ln 2 + 2\\ If{\kern 1pt} {\kern 1pt} {\kern 1pt} {\kern 1pt} {x_1} - {x_2} - 2\ln 2 + 2 \leqslant 0,{\kern 1pt} {\kern 1pt} {\kern 1pt} {\kern 1pt} {\kern 1pt} {\kern 1pt} {\kern 1pt} {\kern 1pt} {\kern 1pt} then{\kern 1pt} {\kern 1pt} {\kern 1pt} {\kern 1pt} {\kern 1pt} {\kern 1pt} {\kern 1pt} {\kern 1pt} {\kern 1pt} {\kern 1pt} {x_1} - {x_2} \leqslant 2\ln 2 - 2 < 0 \Rightarrow {({x_1} - {x_2})^2} \geqslant {(2\ln 2 - 2)^2}\\ {\kern 1pt} |PQ| = \sqrt {{{({x_1} - {x_2})}^2} + {{({y_1} - {y_2})}^2}}  > \sqrt {{{({x_1} - {x_2})}^2}}  \geqslant \sqrt {{{(2\ln 2 - 2)}^2}}  = \sqrt 2 (1 - \ln 2),\\ If{\kern 1pt} {\kern 1pt} {\kern 1pt} {\kern 1pt} {\kern 1pt} {\kern 1pt} {\kern 1pt} {\kern 1pt} {\kern 1pt} {y_1} - {y_2} \geqslant {x_1} - {x_2} - 2\ln 2 + 2 > 0\\ |PQ| = \sqrt {{{({x_1} - {x_2})}^2} + {{({y_1} - {y_2})}^2}} \\ \geqslant \sqrt {{{({x_2} - {x_1})}^2} + {{({x_1} - {x_2} - 2\ln 2 + 2)}^2}} \\ \geqslant \sqrt {\frac{{{{({x_2} - {x_1} + {x_1} - {x_2} - 2\ln 2 + 2)}^2}}}{2}} \\ = \sqrt {\frac{{{{(2 - 2\ln 2)}^2}}}{2}} \\ = \sqrt 2 (1 - \ln 2) \end{array}\]
kuing 17# 2013-4-21 23:53
16# yes94 好多 {\kern 1pt} …………
yes94 18# 2013-4-22 00:29
17# kuing 为了空格!latex空格被忽略
kuing 19# 2013-4-22 00:33
18# yes94 说明不应该那样打,那些应该被分开,文字一般都不放在数学模式下。
thread-944-1-5.html: [几何] 来自人教群的四个直角的四边形是平面图形
kuing 1# 2012-12-1 19:14
教师-红米饭(1571******)  18:38:52 有四个角是直角的四边形一定是平面图形对吗? 教师-红米饭(1571******)  18:54:08 网上说法不一 教师-红米饭(1571******)  18:54:29 我证明是对的,但是网上怎么还有人说不一定呢 群管-kuing  18:55:03 我们这里也是网上,我们说对还是不对,结果还是网上说法不一 设 $\vv{AB}=\vv a$, $\vv{BC}=\vv b$, $\vv{CD}=\vv c$, $\vv{DA}=\vv d$,则 \[\vv a+\vv b+\vv c+\vv d=\vv0,\] 由四个直角得 \[\vv a\cdot\vv b=\vv b\cdot\vv c=\vv c\cdot\vv d=\vv d\cdot\vv a=0,\] 将第一式两边平方并代入第二式得 \[\bigl|\vv a\bigr|^2+\bigl|\vv b\bigr|^2+\bigl|\vv c\bigr|^2+\bigl|\vv d\bigr|^2+2\vv a\cdot\vv c+2\vv b\cdot\vv d=0,\] 即 \[\bigl(\vv a+\vv c\bigr)^2+\bigl(\vv b+\vv d\bigr)^2=0,\] 从而 $\vv a=-\vv c$ 且 $\vv b=-\vv d$。 让那些说不一定的人来挑一挑这个证明有没有毛病吧。
kuing 2# 2012-12-1 19:25
爱好者-何万程(1785***)  19:23:49 反证也容易的,若AD、BC异面,则AB、CD都是AD、BC的公垂线,矛盾
realnumber 3# 2012-12-1 22:12
本帖最后由 realnumber 于 2012-12-1 22:14 编辑 2楼反证简明,1楼也是. 新教材删了异面直线距离,模仿下,假设AB,CD异面,过B作直线m平行CD,记AB、m确定的平面为α,那么由AD、BC都垂直面α 可得AD∥BC,即AD,BC共面,即AB、CD共面,矛盾.
thread-945-1-5.html: [不等式] (转)一个4次的不等式
realnumber 1# 2012-12-3 18:13
$a、b、c、d \in R$,$a+b+c+d=0$,证明:$7(a^2+b^2+c^2+d^2)^{2}  \ge 12(a^4+b^4+c^4+d^4)$
kuing 2# 2012-12-3 18:19
似曾相识,JiChen 好像给过配方,我看能不能找到记录
kuing 3# 2012-12-3 18:23
有了 见:http://www.artofproblemsolving.c ... p?f=51&t=317535
realnumber 4# 2012-12-3 18:24
不用,随手发了下,还以为睡仙??很想解决,而我又没思路。结果他习惯自己考虑。
realnumber 5# 2012-12-3 18:25
果然,kk越来越专业了
kuing 6# 2012-12-3 18:27
“睡仙??”是? 呃,专业啥,只是记得,找链接而已,那些配方都不知怎么得到的,后面hedeng那些倒是知道是机器。
realnumber 7# 2012-12-3 19:20
是不等式群里的,我也不知道是谁,睡仙14129510
kuing 8# 2012-12-3 20:12
O,想起来了,不过群我退了不少了
pxchg1200 9# 2012-12-5 13:00
8# kuing ah,There exist a nice CS proof! :D
pxchg1200 10# 2012-12-14 12:58
本帖最后由 pxchg1200 于 2012-12-14 12:59 编辑 9# pxchg1200 Okay,上CS proof! 话说这个是Vasile Cirtoaje的一个inequality. proof: Notice that: \[ \sum{[b^2+c^2+d^2+3(bc+cd+db)]}=3(a+b+c+d)^2= 0 \] We can assume \[ b^2+c^2+d^2+3(bc+cd+db)\geq 0 \] and \[ \sum{a^4}=(\sum{a^2})^2-2a^2(b^2+c^2+d^2)-2(b^2c^2+c^2d^2+d^2b^2) \] so,it's suffices to check \[  24(b^2c^2+c^2d^2+d^2b^2)+24a^2(b^2+c^2+d^2)\geq 5(a^2+b^2+c^2+d^2)^2 \] Now,Using Cauchy-Schwarz,gives \[ b^2c^2+c^2d^2+d^2b^2\geq \frac{(bc+cd+db)^2}{3} \] it's suffice to check \[ 8(bc+cd+db)^2+24a^2(b^2+c^2+d^2)\geq 5(\sum{a^2})^2 \] set \[ x=b^2+c^2+d^2, y=bc+cd+db \] we have \[ x+3y\geq 0 \] \[ a^2=x+2y \] Hence \[ 8y^2+24x(x+2y)\geq 5(2x+2y)^2 \] Or \[ 4(x-y)(x+3y)\geq 0 \] Done! Equality Occurs when $a=-3b=-3c=-3d $ :D Ps: 发现字体好看多了,kk。  :P
kuing 11# 2012-12-14 14:12
10# pxchg1200 漂亮 其实先看后面再看前面会比较流畅,我估计想的时候也是这样的顺序……蛮自然的,怎么我就没往这里想过呢?
thread-946-1-5.html: [函数] 范围
reny 1# 2012-12-3 19:37
本帖最后由 reny 于 2013-3-30 13:08 编辑 在$△ABC中,a^3+b^3=c^3,求角C$范围.
realnumber 2# 2012-12-3 19:58
本帖最后由 realnumber 于 2012-12-4 07:37 编辑 不清楚是否为下确界 $a+b>c$,$c^3=a^3+b^3=(a+b)(a^2-ab+b^2)>c(a^2-ab+b^2)$ $\cos C=\frac{a^2+b^2-c^2}{2ab} < \frac{1}{2}$ $C > \frac{\pi}{3}$ 既然你已经得到不超过直角. -----看来果然要函数做,看楼下
kuing 3# 2012-12-3 20:59
由条件得 \[\cos C=\frac{a^2+b^2-c^2}{2ab}=\frac{a^2+b^2-\sqrt[3]{(a^3+b^3)^2}}{2ab},\] 记 $t=(a+b)^2/(ab)\in [4,+\infty )$,则 \[\cos C=\frac{t}2-1-\frac12\sqrt[3]{t(t-3)^2}=f(t),\] 求导得 \[f'(t)=\frac12-\frac{t-1}{2\sqrt[3]{t^2(t-3)}}<\frac12-\frac{t-1}{2\cdot \frac{t+t+t-3}3}=0,\] 从而 \[f(4)=1-\frac1{\sqrt[3]2}\geqslant f(t)>\lim_{t\to+\infty}f(t)=0,\] 即得 \[C\in \left[ \arccos \left( 1-\frac1{\sqrt[3]2} \right),\frac\pi2 \right).\]
reny 4# 2012-12-4 17:30
这种代数换元太好了,不过很难想到,谢谢你的答案。补充问一下:如果是a^n+b^n=c^n(n>=3),这时能够求出角C下界的一般式呢?
kuing 5# 2012-12-4 21:40
4# reny 有点麻烦,要用别的代换了,等会写写。
kuing 6# 2012-12-4 22:09
这种代数换元太好了,不过很难想到,谢谢你的答案。补充问一下:如果是a^n+b^n=c^n(n>=3),这时能够求出角C下界的一般式呢? reny 发表于 2012-12-4 17:30 $n=3$ 的情况前面已经解决了,下面设 $n\geqslant 4$。 \[2\cos C=\frac{a^2+b^2-c^2}{ab}=\frac{a^2+b^2-\sqrt[n]{(a^n+b^n)^2}}{ab},\] 由对称性,不妨设 $a\geqslant b$,并令 $a/b=x\in[1,\infty)$,则 \[2\cos C=x+\frac1x-\frac{\sqrt[n]{(x^n+1)^2}}x=g(x),\] 下面证明当 $x\geqslant 1$ 时 $g(x)$ 递减。求导有 \begin{align*} g'(x)&=1-\frac1{x^2}-\frac{2x^n(x^n+1)^{\frac2n-1}-\sqrt[n]{(x^n+1)^2}}{x^2} \\ & =\frac{x^2-1-(x^n+1)^{\frac2n-1}(x^n-1)}{x^2}, \end{align*} 即要证明对任意 $x\geqslant1$ 有 \[h(x)=(x^n+1)^{\frac2n-1}(x^n-1)-x^2+1\geqslant 0,\] 易见 $h(1)=0$,所以只要证明当 $x\geqslant 1$ 时 $h(x)$ 递增。求导有 \begin{align*} h'(x)&=\left( \frac2n-1 \right)(x^n+1)^{\frac2n-2}nx^{n-1}(x^n-1)+(x^n+1)^{\frac2n-1}nx^{n-1}-2x \\ & =2x\left( (x^n+1)^{\frac2n-2}x^{n-2}(x^n+n-1)-1 \right), \end{align*} 即要证明对任意 $x\geqslant1$ 有 \[(x^n+1)^{\frac2n-2}x^{n-2}(x^n+n-1)\geqslant 1,\] 由 $n\geqslant 4$,只需证明更强式 \[(x^n+1)^{\frac2n-2}x^{n-2}(x^n+3)\geqslant 1,\] 上式可以整理为 \[1+\frac3{x^n}\geqslant \left( 1+\frac1{x^n} \right)^{2-\frac2n},\] 于是只需证明 \[1+\frac3{x^n}\geqslant \left( 1+\frac1{x^n} \right)^2,\] 展开等价于显然成立的 $1/x^n\geqslant1/x^{2n}$。这样,当 $x\geqslant 1$ 时 $g(x)$ 递减就得到了证明,从而 \[g(1)=2-\sqrt[n]4\geqslant2\cos C>\lim_{x\to+\infty}g(x)=0,\] 即得 \[C\in\left[\arccos\left(1-\frac12\sqrt[n]4\right),\frac\pi2\right).\] 当 $a=b$ 时取得区间左端点。
reny 7# 2012-12-5 16:54
本帖最后由 reny 于 2012-12-5 17:14 编辑 g(x)的导函数的分子应该是稍复杂点的x^2-1-(x^n+1)^(2/n-1)(2x^(n-1)-x^n-1)吧
kuing 8# 2012-12-5 17:28
7# reny 你再仔细算下
reny 9# 2012-12-5 18:25
是的,没错。我少乘了一个x,你这解法计算量不小,最终做出来了,太强了。
kuing 10# 2012-12-5 18:34
9# reny 暂时没想到更好的办法,那些计算我也费了点神才搞了出来
thread-947-1-5.html: [不等式] 怎么猜到a=2c?
realnumber 1# 2012-12-4 10:48
本帖最后由 realnumber 于 2012-12-4 10:50 编辑 偶是猜的a=b,而a=2c是求导的,哪帮牛人怎么得出a=2c的呢?
kuing 2# 2012-12-4 14:41
说到讲解?那些解法似乎都不太适合,至少对初学的不适合。 个人认为,猜 a=b 最正常,因为只有左边有 ab 而且在分子,其余都是 a+b,所以必然 a=b 取等,表达成不等式就是先对左边用 $ab\leqslant(a+b)^2/4$。 然后可以换元什么的变成二元,而如果是我就直接由齐次性先令 a+b+c=1 结果变成一元函数求最值,求导搞定。
Gauss门徒 3# 2013-1-8 16:33
A,b对称
thread-948-1-4.html: [数论] 请教一个整除的题
abababa 1# 2012-12-4 20:42
设$A_1=0, A_2=1$,当$n>2$时,定义$A_n$为把$A_{n-1}, A_{n-2}$的十进制展开式按从左到右的次序连接起来得到的数,例如$A_3=A_2A_1=10, A_4=A_3A_2=101, A_5=A_4A_3=10110$,求所有的自然数$n$使$A_n$能被$11$整除
kuing 2# 2012-12-4 20:55
十进制还是二进制?
abababa 3# 2012-12-4 21:16
2# kuing 是10进制,可能因为前两个正好是1,0看起来像二进制。这个题一位网友说已经解出来了,但是他说他解得太复杂了,他得的结果是n=6k+1满足。
abababa 4# 2013-1-25 19:23
顶一下
realnumber 5# 2013-1-25 21:29
本帖最后由 realnumber 于 2013-1-25 22:11 编辑 被11整除的数字(10进制),"偶数位和"与"奇数位和"相等,$10^n\equiv1 \mod11$,$n$为偶数;$10^n\equiv-1 \mod11$,$n$为奇数. 如此以下只关心$A_n$的是几位数,取$\mod11$情况下. $A_3,A_4,A_5,A_6,A_7,...$依次是2,3,5,8,13,21,...位数,即偶,奇,奇,偶,奇,奇,....位数,---3为周期 (偶)$A_3\equiv-1$,(奇)$A_4\equiv2$,(奇)$A_5=A_4A_3\equiv2-1\equiv1$, (偶)$A_6=A_5A_4\equiv10+2\equiv-1$,(奇)$A_7=A_6A_5=10+1\equiv2$,(奇)$A_8=A_7A_6\equiv1$, $A_9=-1$,$A_{10}=2$开始出现周期了,...所以$n=6n+1$. ---我不知道怎么表达更清楚点.这个需要写吗? $A_n=A_{n-1}A_{n-2}\equiv10A_{n-1}+A_{n-2}  \mod11$($A_{n-2}$为奇位数); $A_n=A_{n-1}A_{n-2}\equiv A_{n-1}+A_{n-2}  \mod11$($A_{n-2}$为偶位数).
abababa 6# 2013-1-26 08:59
5# realnumber 和那位网友说的差不多,都是先得到这个数的位数是Fibonacci数列,再得到$A_n \equiv (-1)^{d_{n-2}}A_{n-1} + A_{n-2} \pmod{11}$,$d$是位数 把n模3分类,然后用数学归纳法分别证这三类模2的余数是0,1,1 这就用了三次数学归纳法,但还不是最麻烦的,最麻烦的是他下面的证明,把n模6分类了,分别求了$A_n$模11的余数,用了6次数学归纳法,他说自己证明麻烦了,也没想出简单的方法。 楼上得出的周期最后也要用什么方法证明一下吧?不能就这么说是周期就证明了。
realnumber 7# 2013-1-26 09:13
本帖最后由 realnumber 于 2013-1-26 09:21 编辑 6# abababa 个人觉得还是我这样写法更适合交流,刚刚学自这个帖子,里面模周期也没证明啊.没参与竞赛,不清楚要求. 写成数学归纳法自然可以 先验证6个成立,假设6个,根据递推关系得出6个,就好了.
abababa 8# 2013-1-26 09:23
7# realnumber 确实模余数是有规律,以前也总这么用,就是一个数的多少次方,求尾数,$m^{4p+q} \equiv m^q \pmod{10}$,但后来就觉得这么弄不严密,用了数学归纳法证明它以后才这么用的。 我在想的是,是不是这个题还有其它的方向入手,能不用这么多次归纳。
abababa 9# 2013-1-26 09:25
7# realnumber 我也不是搞竞赛的,只是有点兴趣,碰到一些问题就弄一下,但我水平不行,经常求教别人,呵呵
realnumber 10# 2013-1-26 09:31
9# abababa 水平我也水,只能去解决觉得能做的,好在只是业余爱好,不靠它混工资.
realnumber 11# 2013-1-26 09:48
6# abababa 前面部分,也不需要那么多次数学归纳法的, 前3项,偶奇奇,假设3项,推出3项.$\mod11$的递推关系直接写 ,应该不需要归纳法说明(已经证明了$A_{n-2}$是偶位数还是奇位数).
abababa 12# 2013-1-26 10:00
本帖最后由 abababa 于 2013-1-26 10:05 编辑 11# realnumber 确实是,都是递推的,后一项是前两项的和,模2的话必定是0,1,1 其实所有的常数系数的齐次线性递推数列都是模周期的数列,Fibonacci就是这样的数列,所以肯定是模周期的。
thread-95-1-1.html: 来自pep的阶乖不等式
kuing 1# 2011-10-13 18:03
http://bbs.pep.com.cn/thread-1908938-1-1.html $n\geqslant6$,证\[ \left(\frac n2\right)^n>n!>\left(\frac n3\right)^n. \hspace{5em}(1) \] 以前好像就见过,不过没想起来当时具体怎么证,刚才试了一下,发现还是没能秒,用了好多分了。。。。 对于式 $(1)$ 左边,由均值不等式,我们有 \begin{align*} n! &=2\cdot 3\cdots (n-2)\cdot (n-1)n \\ & <\left( \frac{2+3+\cdots +(n-2)}{n-3} \right)^{n-3}\cdot (n-1)n \\ & =\left( \frac{n}{2} \right)^{n-3}\cdot (n-1)n, \end{align*}而\[ \left( \frac{n}{2} \right)^{3}-(n-1)n=\frac18n(n^{2}-8n+8), \]当 $n$ 为大于 6 的整数时上式大于 0,此时即得式 $(1)$ 左边,而 $n=6$ 可以直接验证不等式成立,所以式 $(1)$ 左边得证; 对于式 $(1)$ 右边,我们将证明更强式\[ n!>\left( \frac{n+1}e \right)^n. \hspace{5em}(2) \]熟知对任意正整数 $n$ 有 $\left( 1+\dfrac1n \right)^n<e$,于是得\[ \prod_{k=1}^n \left( \frac{k+1}k \right)^k < e^n, \]注意左边可以约掉很多东东,化简后,等价于\[ \frac{(n+1)^n}{n!}<e^n, \]即式 $(2)$。
海盗船长 2# 2011-10-14 20:23
哈哈,好多大一的数分题。。
kuing 3# 2011-10-14 21:25
2# 海盗船长 这个也是数分题?
pxchg1200 4# 2011-10-20 23:56
Stirling公式秒之。。。
kuing 5# 2011-10-20 23:59
4# pxchg1200 嗯,原贴也有提及这个。。。
kuing 6# 2012-3-25 00:37
左边这个更强些 http://bbs.pep.com.cn/thread-585932-1-1.html
海盗船长 7# 2012-3-26 17:52
阶乖。。。
kuing 8# 2012-3-26 17:59
7# 海盗船长
pxchg1200 9# 2012-3-27 22:44
貌似是谢惠明的《数学分析习题课讲义》上的题目来着。
yayaweha 10# 2012-7-30 10:59
这道题是零几年的上海交大自主招生题,用数学归纳法比较简单,最后后就变得只需证$2<(1+\frac{1}{k})^k<3$
都市侠影 11# 2012-7-30 12:38
本帖最后由 都市侠影 于 2012-7-30 12:52 编辑 有更好的结论: \[ \lim_{n\to\infty}\frac{\sqrt[n]{n!}}{n}=\frac{1}{e} \] 如果用上积分,这个极限可以简单的证明: \begin{align} &\lim_{n\to\infty}\frac{\sqrt[n]{n!}}{n} \\ =&\lim_{n\to\infty}\sqrt[n]{\frac{1}{n}\cdot\frac{2}{n}\cdots\frac{n}{n}} \end{align} 取对数后,是 \begin{align} &\lim_{n\to\infty}\frac{1}{n}\sum_{k=1}^n\ln{\frac{k}{n}} \\ =&\int_0^1\ln x \textrm{d}x \\ =& -1 \end{align} 所以就得上面的极限。
thread-950-1-5.html: [几何] AB是否过定点?
realnumber 1# 2012-12-6 12:56
本帖最后由 realnumber 于 2012-12-6 13:01 编辑 椭圆$\frac{x^2}{a^2}+\frac{y^2}{b^2}=1$上一定点P,以及椭圆的弦PA,PB;若两弦的斜率之积为常数,问AB是否过定点? 可以用换元法变为圆的问题,可是依然不会,真要下决心硬算?
realnumber 2# 2012-12-6 14:01
本帖最后由 realnumber 于 2012-12-6 14:10 编辑 是《中学数学参考》2012,1~2上旬97页 答案是定点,没过程 $P(x_{0},y_{0})$,$k_{PA}k_{PB}=k$;定点坐标就是$(\frac{ka^{2}+b^{2}}{ka^{2}-b^{2}}x_{0},-\frac{ka^{2}+b^{2}}{ka^{2}-b^{2}}y_{0})$ 要不别算它了,----
kuing 3# 2012-12-6 14:16
这类定点结论好多了,都是早有研究的,不过基本上都是用代数玩的,很少能玩成纯平几。
凌群 4# 2012-12-7 16:20
本帖最后由 凌群 于 2012-12-7 16:21 编辑 你去百度文库里面搜索《圆锥曲线内接三角形的又一组性质》(蒋利敏) 这里面有证明的 或者用坐标平移,将原点平移到点A处
都市侠影 5# 2012-12-7 20:28
这类定点结论好多了,都是早有研究的,不过基本上都是用代数玩的,很少能玩成纯平几。 kuing 发表于 2012-12-6 14:16 很正常,因为斜率之积为定值这个条件,实在找不到一个相应的几何意义。
thread-951-1-5.html: 请教几道题的解答?
一枝海 1# 2012-12-6 17:41
http://bbs.pep.com.cn/forum.php?mod=viewthread&tid=2605816&extra=page%3D1
kuing 2# 2012-12-6 23:45
我把题目粘过来了,方便看。 第一题大概就分段画出图形就基本OK了吧 第四题或者可以说是物理题?不过数理不分家 \[I_R=k\cdot \frac{\sin \alpha }{(R\sec \alpha )^2}=\frac k{R^2}\cdot \sin \alpha \cos^2\alpha,\] 然后最值随便求…… 最后一题像小学题?依题意知不会全天下雨,所以 \[没下雨的天数=\frac{上午晴天的天数+下午晴天的天数-下雨的天数}2=\frac{9+12-11}2=5.\] 或者看成集合,用容斥原理神马来看? 由于不会全天下雨,所以,$上午晴天\cup 下午晴天=全集$,$上午晴天\cap 下午晴天=全天晴天$,故此 \begin{align*} \abs{全天晴天}&=\abs{全集}-\abs{有下雨}\\ &=\abs{上午晴天\cup 下午晴天}-\abs{有下雨}\\ &=\abs{上午晴天}+\abs{下午晴天}-\abs{上午晴天\cap 下午晴天}-\abs{有下雨}\\ &=\abs{上午晴天}+\abs{下午晴天}-\abs{全天晴天}-\abs{有下雨}, \end{align*} 同样也得到 \[没下雨的天数=\frac{上午晴天的天数+下午晴天的天数-下雨的天数}2.\]
kuing 3# 2012-12-7 00:44
第二题好像有点无聊?反正我是懒得一个个算了……略过 似乎就第三题在这些里面最难……好像以前见过,至少是类似的,不过这类题我几乎都没弄懂过
realnumber 4# 2012-12-10 22:07
本帖最后由 realnumber 于 2012-12-10 22:11 编辑 3.提案有p个,那么国家最多是$2^{p-1}$个,(我是先做实验,p=2,p=3,p=4后,猜的) 存在这样的情景,所有国家都含有1号提案,那么从余下的p-1个方案中挑不同的方案办法有$2^{p-1}$种.(自然这不是唯一的一个办法,可以试下p=3,或4就会发现) 以下证明大于等于$2^{p-1}+1$的国家数目是不可能的, (1)若所有国家有1个公共提案,比如1号,那么余下的p-1个提案最多有$2^{p-1}$个选择. (2)若所有的国家没有1个公共的提案,对于含有1号提案的国家不作变化, 对于不含1号提案的国家逐个做如下处理,把它的提案变为全体提案构成集合的补集,(每改变一个国家,考察任意两个国家,此时依然满足题中所给条件,如果变化前符合的话),经过如此处理,问题转化为(1) 的情景,完毕.
realnumber 5# 2012-12-11 11:58
本帖最后由 realnumber 于 2012-12-11 12:37 编辑 看来是这个没错了,群里有个高手的解
thread-952-1-1.html: 群里那个爪机解锁题
kuing 1# 2012-12-7 23:27
\begin{align*} F(x)&=\int_0^x{(x^2-t^2)f(t)\rmd t}=x^2\int_0^x{f(t)\rmd t}-\int_0^x{t^2f(t)\rmd t}, \\ F'(x)&=2x\int_0^x{f(t)\rmd t}+x^2f(x)-x^2f(x)=2x\int_0^x{f(t)\rmd t}, \\ F''(x)&=2\int_0^x{f(t)\rmd t}-2xf(x), \\ F'''(x)&=2f(x)-2f(x)-2xf'(x)=-2xf'(x), \end{align*} 而 $(x^k)''=k(k-1)x^{k-2}$,所以只要 $k=3$,那么洛完就能约掉 $x$ 了 \[\lim _{x\to 0}\frac{F'(x)}{x^3}=\lim _{x\to 0}\frac{F'''(x)}{6x}=\lim _{x\to 0}\frac{-f'(x)}{3}=-\frac13f'(0)\ne0.\] 不过总感觉好像差了点东东
realnumber 2# 2012-12-11 08:19
哈哈, 唉,我可解不开,只能看解答了
thread-953-1-1.html: 求最大公约数的一道题,只需要思路。
q85669551 1# 2012-12-8 00:33
...简言之就是从一个正整数的连续有界子集中选n个数,求这个n个数的最大的最大公约数 比如3,4,5,6,7,8,9当中选2个数 最大的最大公约数为4 选3个数 最大的最大公约数为3...
kuing 2# 2012-12-8 01:08
好难...懂 借贴测试飞公式 $\frac{a+b}2\geqslant\sqrt{ab}$ $$\frac{a+b}2\geqslant\sqrt{ab}$$ \begin{equation}\frac{a+b}2\geqslant\sqrt{ab}\end{equation}
q85669551 3# 2012-12-8 03:36
不知道这样对不对。。
Gauss门徒 4# 2013-5-12 18:33
2# kuing 其实通显然的。
thread-954-1-4.html: [不等式] 一个三角不等式,已经解决,见5楼
realnumber 1# 2012-12-8 21:09
本帖最后由 realnumber 于 2013-1-28 09:02 编辑 \[\abs{\cos{x}}+\abs{\cos{2x} }+\abs{\cos{2^2x}}+···+\abs{\cos{2^nx}} \ge \frac{n}{2},\]其中$n \in N_{+}$. ---便于阅读,把本贴以及回帖错误都删了.
kuing 2# 2012-12-8 22:53
没看懂……
kuing 3# 2012-12-9 14:25
3# realnumber 2项配的话不是要证其>=1么
kuing 4# 2012-12-9 19:23
5# realnumber 大于1/2相加后不是n/4吗
hnsredfox_007 5# 2013-1-28 08:50
本帖最后由 hnsredfox_007 于 2013-1-28 08:52 编辑 用数学归纳法证明。 (1)显然$n=0,1$时成立(楼上已证,咱不再絮叨了,呵呵); (2)假设$n=k,k+1$时成立,即$$\sum_{i=0}^k|\cos 2^ix| \geqslant\frac{k}{2} (\ast),$$ $$\sum_{i=0}^{k+1}|\cos 2^ix| \geqslant\frac{k+1}{2} (\ast \ast);$$ 则$n=k+1$时,若$$|\cos x|\geqslant \frac12,$$则由$(\ast)$得: $$\sum_{i=0}^{k+2}|\cos 2^ix| = |\cos x|+\sum_{i=1}^{k+2}|\cos 2^ix|= |\cos x|+\sum_{i=0}^{k+1}|\cos 2^i(2x)|\geqslant \frac12+\frac{k+1}{2}=\frac{k+2}{2};$$ 若$$|\cos x|\leqslant \frac12,$$则由$(\ast\ast)$得: $$\sum_{i=0}^{k+2}|\cos 2^ix| = |\cos x|+|\cos(2x)| +\sum_{i=2}^{k+2}|\cos 2^ix|= |\cos x|+|\cos(2x)|+\sum_{i=0}^{k}|\cos 2^i(2^2x)|\geqslant 1+\frac{k}{2}=\frac{k+2}{2}.$$ 由(1)(2)可知,结论对所有的自然数成立。证毕。
hnsredfox_007 6# 2013-1-28 08:56
上楼最后一步,若$$|\cos x|\leqslant \frac12,$$则$$|\cos x|+|\cos(2x)|\geqslant 1$$易证吧
realnumber 7# 2013-1-28 08:59
5# hnsredfox_007 ,学了一招.
realnumber 8# 2013-1-28 09:32
本帖最后由 realnumber 于 2013-1-28 10:10 编辑 西西(24#####63)  09:26:29 角度还成自然等差结论仍然成立 难度就不是一个档次 正弦仍然成立 ,取x为1,下界仍然为你的那个 大概意思是 存在$x\ge 1$,其中$n \in N_{+}$, \[\abs{\cos{x}}+\abs{\cos{2x} }+\abs{\cos{3x}}+···+\abs{\cos{nx}} \ge \frac{n}{2},\] \[\abs{\sin{x}}+\abs{\sin{2x} }+\abs{\sin{3x}}+···+\abs{\sin{nx}} \ge \frac{n}{2},\]
hnsredfox_007 9# 2013-1-28 09:40
8# realnumber 正弦那个易否定 取$x=k\pi$即可
hnsredfox_007 10# 2013-1-28 09:47
本帖最后由 hnsredfox_007 于 2013-1-28 09:54 编辑 8# realnumber 余弦那个,$n$为奇数时,取$x=\dfrac{\pi}{2}$.否定之
realnumber 11# 2013-1-28 10:09
10# hnsredfox_007 你的对,应该是西西没说清楚或我没领会,需要条件是"存在x$\ge1$",我去修改
hnsredfox_007 12# 2013-1-28 14:33
11# realnumber 那么 余弦那个,取$x=\pi$,显然成立 正弦那个,$n$为偶数时,取$x=\dfrac{\pi}{2}$,显然成立。奇数时,还没想好……
realnumber 13# 2013-1-28 19:11
本帖最后由 realnumber 于 2013-1-28 19:14 编辑 12# hnsredfox_007 模仿你的做法$x=\frac{\pi}{4}$,那么\[\sin{\frac{\pi}{4}}+...+\sin{\frac{n\pi}{4}}≈{\frac{4n}{8}\frac{\sqrt{2}}{2}+\frac{2n}{8}1+\frac{2n}{8}0}\ge\frac{n}{2}\] 当$n=8k,k\in N$取等号. 这样看起来题目有些水....
realnumber 14# 2013-1-28 19:18
12# hnsredfox_007 奇数$\frac{\pi}{2}$也对
thread-955-1-5.html: [不等式] 求最大值
转化与化归 1# 2012-12-9 08:46
求最大值
转化与化归 2# 2012-12-15 11:25
一个不等式的证明
kuing 3# 2012-12-15 13:37
??x, y 不是对所有的 a,b,c 都能取到 0 吧……
转化与化归 4# 2012-12-15 16:22
abz+bcx+cay<=abz'  加个一撇可能容易理解些
thread-957-1-5.html: [不等式] 来自群的一个简单三元不等式
kuing 1# 2012-12-12 23:11
首先由于不等式是齐次不等式,所以其实无论是否成立,那个条件“$abc\geqslant1$”都是多余的,只要是正数就够了。 展开易得 \begin{align*} \sum{\frac ab}\cdot \sum{\frac ba}\geqslant \sum{a}\cdot \sum{\frac1{a}}&\iff\sum{\frac{a^2}{bc}}+\sum{\frac{bc}{a^2}}\geqslant \sum{\frac ab}+\sum{\frac ba} \\ & \iff\frac12\sum{\left( \frac{a^2}{bc}+\frac{ca}{b^2} \right)}+\frac12\sum{\left( \frac{bc}{a^2}+\frac{b^2}{ca} \right)}\geqslant \sum{\frac ab}+\sum{\frac ba}, \end{align*} 由均值不等式有 \[\frac12\sum{\left( \frac{a^2}{bc}+\frac{ca}{b^2} \right)}+\frac12\sum{\left( \frac{bc}{a^2}+\frac{b^2}{ca} \right)}\geqslant \sum{\left( \frac ab \right)^{3/2}}+\sum{\left( \frac ba \right)^{3/2}},\] 所以只要证明 \begin{equation}\label{20121212bdsgqs} \sum{\left( \frac ab \right)^{3/2}}+\sum{\left( \frac ba \right)^{3/2}}\geqslant \sum{\frac ab}+\sum{\frac ba}. \end{equation} 上式利用如下命题即可证得。 命题    对于任意 $a_1$, $a_2$, \ldots, $a_n>0$ 且 $a_1a_2\cdots a_n=1$。 若 $p\geqslant q \geqslant 0$,则 \begin{equation}\label{zhtg1} a_1^p + a_2^p + \cdots + a_n^p \geqslant a_1^q + a_2^q + \cdots + a_n^q; \end{equation} 若 $0\geqslant p \geqslant q$,则 \begin{equation}\label{zhtg2} a_1^p + a_2^p + \cdots + a_n^p \leqslant a_1^q + a_2^q + \cdots + a_n^q. \end{equation} 这个命题我其实已经在《数学空间》总第4期P11里提到过了,只不过当时限于篇幅没写证明,下面写一写。 不失一般性,设 $a_1\geqslant a_2\geqslant \cdots\geqslant a_n>0$,令 $f(t)= a_1^t + a_2^t + \cdots + a_n^t $,则 \[ f'(t)= a_1^t \ln a_1 + a_2^t \ln a_2 + \cdots + a_n^t \ln a_n, \] 当 $t\geqslant 0$ 时,则显然有 $a_1^t \geqslant a_2^t \geqslant \cdots\geqslant a_n^t$ 及 $\ln a_1 \geqslant \ln a_2 \geqslant \cdots \geqslant \ln a_n $,于是由切比雪夫不等式,有 \begin{align*} a_1^t \ln a_1 + a_2^t \ln a_2 + \cdots + a_n^t \ln a_n & \geqslant \frac1{n}\left(a_1^t + a_2^t + \cdots + a_n^t \right)\left(\ln a_1 + \ln a_2 + \cdots + \ln a_n \right) \\ & = \frac1{n}\left(a_1^t + a_2^t + \cdots + a_n^t \right)\ln \left(a_1a_2\cdots a_n\right) = 0, \end{align*} 即当 $t\geqslant 0$ 时 $f'(t)\geqslant0$,同理可证当 $t\leqslant 0$ 时 $f'(t)\leqslant0$,故命题得证。 在命题中取 $n=3$, $p=3/2$, $q=1$,并分别令 $a_1=a/b$, $a_2=b/c$, $a_3=c/a$ 及 $a_1=b/a$, $a_2=c/b$, $a_3=a/c$ 得到的两式相加即得式 \eqref{20121212bdsgqs},从而原不等式得证。
kuing 2# 2012-12-12 23:39
擦,我还是搞复杂了,只展开右边更简单 \[\sum{\frac ab}\cdot \sum{\frac ba}\geqslant \sum{a}\cdot \sum{\frac 1a}\iff\sum{\frac ab}\cdot \sum{\frac ba}\geqslant 3+\sum{\frac ab}+\sum{\frac ba},\] 由均值不等式有 \begin{align*} \frac 13\sum{\frac ab}\cdot \sum{\frac ba}&\geqslant 3 ,\\ \frac 13\sum{\frac ab}\cdot \sum{\frac ba}&\geqslant \sum{\frac ba} ,\\ \frac 13\sum{\frac ba}\cdot \sum{\frac ab}&\geqslant \sum{\frac ab} , \end{align*} 三式相加即得。 原来这么弱,谁来加强一下玩玩。
pxchg1200 3# 2012-12-14 12:47
2# kuing 貌似还是搞复杂了。 注意到: 只要证明: \[ (\sum{a^2b})(\sum{ab^2})\geq abc(a+b+c)(ab+bc+ca) \] by Holder \[ 3 (\sum{a^2b})(\sum{ab^2})\geq (ab+bc+ca)^3 \geq 3abc(a+b+c)(ab+bc+ca) \] Done!
kuing 4# 2012-12-14 13:21
3# pxchg1200 嗯,反正不等式蛮弱,证法肯定很多
reny 5# 2012-12-15 00:36
加强为(a/b+b/c+c/a)(b/a+c/b+a/c)+18>=3(a+b+c)(1/a+1/b+1/c)
kuing 6# 2012-12-15 01:07
5# reny 嗯,这样用类似于2#那样之后分解为 $\left(\sum(a/b)-3\right)\left(\sum(b/a)-3\right)\geqslant0$
thread-958-1-5.html: [不等式] 来自群的一道看上去不易证其实有点弱的轮换不等式
kuing 1# 2012-12-13 11:58
爱好者荆楚(7048*****) 07:38:26 由柯西不等式有 \begin{align*} \sum{\frac{2x^2+xy}{(y+\sqrt{zx}+z)^2}}&\geqslant \sum{\frac{2x^2+xy}{(y+z+z)(y+x+z)}} \\ & =\frac1{x+y+z}\sum{\left( \frac{2x^2+xy-x(y+2z)}{y+2z}+x \right)} \\ & =1+\frac2{x+y+z}\sum{\frac{x(x-z)}{y+2z}} \\ & =1+\frac2{x+y+z}\sum{\left( \frac{x(x-z)^2}{(y+2z)(x+y+z)}+\frac{x(x-z)}{x+y+z} \right)} \\ & \geqslant 1+\frac2{x+y+z}\sum{\frac{x(x-z)}{x+y+z}} \\ & =1+\frac{\sum{(x-y)^2}}{(x+y+z)^2} \\ & \geqslant 1. \end{align*}
kuing 2# 2012-12-13 18:45
刚才又看到了一个类似: 由均值不等式得 \begin{align*} \sum{\frac{2x^2+xy}{(x+2y+3z)^2}}&=\sum{\frac{2x(2x+y)^2}{(4x+2y)(x+2y+3z)(x+2y+3z)}} \\ & \geqslant \sum{\frac{2x(2x+y)^2}{\left( \frac{4x+2y+x+2y+3z+x+2y+3z}3 \right)^3}} \\ & =\frac{\sum{x(2x+y)^2}}{4(x+y+z)^3} \\ & =\frac{4\sum{x^3}+4\sum{x^2y}+\sum{xy^2}}{4(x+y+z)^3} \\ & \geqslant \frac{\sum{x^3}+3\sum{x^2y}+3\sum{xy^2}+6xyz}{4(x+y+z)^3} \\ & =\frac14. \end{align*}
kuing 3# 2012-12-13 22:55
2# kuing 这题再来个局部切线法。 由齐次性,不妨设 $x+y+z=1$,则 \[\frac{2x^2+xy}{(x+2y+3z)^2}=x\cdot \frac{2x+y}{(3-2x-y)^2},\] 只对 $\frac{2x+y}{(3-2x-y)^2}$ 使用切线法,注意使用时可将 $2x+y$ 看成单变量来切,可得 \[\frac{2x+y}{(3-2x-y)^2}\geqslant \frac12(2x+y)-\frac14\iff\frac{(2x+y-1)^2(9-4x-2y)}{4(3-2x-y)^2}\geqslant 0,\] 于是 \begin{align*} \sum{\frac{2x^2+xy}{(x+2y+3z)^2}}&\geqslant \frac12\sum{x(2x+y)}-\frac14\sum{x} \\ & =\sum{x^2}+\frac12\sum{xy}-\frac14 \\ & \geqslant \frac12\sum{x^2}+\sum{xy}-\frac14 \\ & =\frac14. \end{align*}
kuing 4# 2012-12-13 23:19
3# kuing 再来个琴生不等式的。 仍然不妨设 $x+y+z=1$,记 \[f(t)=\frac{t}{(3-t)^2},\] 其中 $t\in(0,3)$,则求导有 \begin{align*} f'(t)&=\frac{t+3}{(3-t)^3}>0, \\ f''(t)&=\frac{2(t+6)}{(3-t)^4}>0, \end{align*} 所以 $f(t)$ 在 $(0,3)$ 单增且下凸,于是由琴生不等式有 \begin{align*} \sum{\frac{2x^2+xy}{(x+2y+3z)^2}}&=\sum{(x\cdot f(2x+y))} \\ & \geqslant f\left( \sum{x(2x+y)} \right) \\ & =f\left( 2\sum{x^2}+\sum{xy} \right) \\ & \geqslant f\left( \sum{x^2}+2\sum{xy} \right) \\ & =f(1) \\ & =\frac14. \end{align*}
kuing 5# 2012-12-13 23:25
由上面两个证明可以看出,同样有以下不等式成立。 \[\sum\frac{2xy+y^2}{(x+2y+3z)^2}\geqslant\frac14.\] 而上式跟2#的不等式相比,哪个强呢?猜测这个更强些,因为在同样证法时可少放缩一步,故有理由相信这个更紧。当然了,还是要证过才知道。时间关系,明天再玩……
Gauss门徒 6# 2012-12-14 12:42
第二个不等式是直接由第一个去根号得来的= =
kuing 7# 2012-12-14 13:17
6# Gauss门徒 oh, 才看出来
kuing 8# 2012-12-14 17:35
5# kuing 续 5#,我昨晚的猜测即 \begin{equation}\label{20121213wcx} \sum{\frac{2x^2+xy}{(x+2y+3z)^2}}\geqslant \sum{\frac{2xy+y^2}{(x+2y+3z)^2}} \end{equation} 对任意正数 $x$, $y$, $z$ 成立,下面证之。 左右作差可知式 \eqref{20121213wcx} 等价于 \begin{equation}\label{20121213wcxdjs} \sum{\frac{(x-y)(2x+y)}{(x+2y+3z)^2}}\geqslant 0, \end{equation} 换元,令 $2x+y=a$, $2y+z=b$, $2z+x=c$,则式 \eqref{20121213wcxdjs} 化为 \begin{equation}\label{20121213wcxdjshy} \sum{\frac{a(a-2b+c)}{(b+c)^2}}\geqslant 0, \end{equation} 下面证明式 \eqref{20121213wcxdjshy} 对任意正数 $a$, $b$, $c$ 成立。 变形有 \begin{align*} \text{式}~\eqref{20121213wcxdjshy} & \iff\sum{\left( \frac{a(a-b)}{(b+c)^2}+\frac{a(c-b)}{(b+c)^2} \right)}\geqslant 0 \\ & \iff\sum{\left( \frac{a(a-b)}{(b+c)^2}+\frac{c(b-a)}{(a+b)^2} \right)}\geqslant 0 \\ & \iff\sum{\frac{\bigl((a+b+c)^2-ac\bigr)(a-b)(a-c)}{(a+b)^2(b+c)^2}}\geqslant 0 \\ & \iff\sum{(c+a)^2\bigl((a+b+c)^2-ca\bigr)(a-b)(a-c)}\geqslant 0, \end{align*} 为方便书写,记 \begin{align*} A&=(c+a)^2\bigl((a+b+c)^2-ca\bigr), \\ B&=(a+b)^2\bigl((a+b+c)^2-ab\bigr), \\ C&=(b+c)^2\bigl((a+b+c)^2-bc\bigr), \end{align*} 即 \[\text{式}~\eqref{20121213wcxdjshy}\iff f(a,b,c)=A(a-b)(a-c)+B(b-a)(b-c)+C(c-a)(c-b)\geqslant 0,\] 由式 \eqref{20121213wcxdjshy} 是轮换对称式,不失一般性,可设 $(b-a)(b-c)\leqslant 0$,则由均值不等式有 \begin{align*} f(a,b,c)&=A(a-b)^2+(A-B+C)(a-b)(b-c)+C(b-c)^2 \\ & \geqslant (A-B+C+2\sqrt{AC})(a-b)(b-c) \\ & =\left( \bigl(\sqrt{A}+\sqrt{C}\bigr)^2-B \right)(a-b)(b-c) \\ & =\bigl( \sqrt{A}+\sqrt{C}-\sqrt{B} \bigr)\bigl( \sqrt{A}+\sqrt{C}+\sqrt{B} \bigr)(a-b)(b-c), \end{align*} 于是只需证明 \[\sqrt{A}+\sqrt{C}\geqslant \sqrt{B},\] 事实上 \begin{align*} \sqrt{A}+\sqrt{C}&=(c+a)\sqrt{(a+b+c)^2-ca}+(b+c)\sqrt{(a+b+c)^2-bc} \\ & =(c+a)\sqrt{(a+b)^2+c^2+2bc+ca}+(b+c)\sqrt{(a+b)^2+c^2+2ca+bc} \\ & >(c+a)(a+b)+(b+c)(a+b) \\ & >(a+b+c)(a+b) \\ & >(a+b)\sqrt{(a+b+c)^2-ab} \\ & =\sqrt{B}, \end{align*} 所以式 \eqref{20121213wcxdjshy} 成立,原不等式 \eqref{20121213wcx} 得证。
kuing 9# 2012-12-14 17:52
从楼上的证法可以归结出一个判断Schur型不等式的方法,即 设 $A$, $B$, $C$ 为关于 $a$, $b$, $c$ 的非负变量,且 \[f(a,b,c)=A(a-b)(a-c)+B(b-a)(b-c)+C(c-a)(c-b)\] 是轮换对称式。如果当 $(b-a)(b-c)\leqslant0$ 时恒有 $\sqrt A+\sqrt C\geqslant\sqrt B$ 成立,则 $f(a,b,c)\geqslant0$ 恒成立。
thread-959-1-5.html: [函数] 求一个最小值
reny 1# 2012-12-14 11:05
本帖最后由 reny 于 2012-12-14 11:09 编辑 求f(x)=sqrt((x+1)^2+4)+2sqrt((x-2)^2+1)的最小值
Gauss门徒 2# 2012-12-14 12:51
求导吧,这个就是费马光学原理。。柯西算得蛋疼取等还算不到= =
realnumber 3# 2012-12-14 13:55
求导后得,4次方程,看来要套4次方程求根公式
kuing 4# 2012-12-14 13:59
数据没设好,四次方程分解不了只能套公式……
kuing 5# 2012-12-14 14:38
像这贴 http://bbs.pep.com.cn/forum.php?mod=viewthread&tid=1343676 那样,数据设计得好就有简单解
reny 6# 2012-12-14 21:31
5# kuing 调整一下数据,就像人教论坛那个例子柯西不等式能解决不?
kuing 7# 2012-12-15 01:04
6# reny 那大概要看调整成怎样了……
thread-96-1-2.html: 单侧极限时,左侧趋向与右边趋向到底怎么写?
kuing 1# 2011-10-13 21:12
随便翻了几本高等数学或数学分析的电子书(新旧不一),找到了三种写法 1:$\color{red}{\Large x\to a+}$ 或 $\color{red}{\Large x\to a-}$ 型: 2:$\color{red}{\Large x\to a^+}$ 或 $\color{red}{\Large x\to a^-}$ 型: 3:$\color{red}{\Large x\to a+0}$ 或 $\color{red}{\Large x\to a-0}$ 型:
kuing 2# 2011-10-13 22:51
其实早前就想搞清楚哪个正规一些,国际上是怎么规定的?虽然是有点无关紧要,不过我学latex排版之后对排版的要求高了,还是想搞清楚一下。
①②③④⑤⑥⑦ 3# 2011-10-14 09:25
英国标准 BS ISO 80000-2 2009中,a+,a- 国内标准,只找到93年的:GB3102.11-93物理科学和技术中使用的数学符号,+和-放在上标位置,不过很多大学数学书都不遵从的吧
kuing 4# 2011-10-14 12:31
3# ①②③④⑤⑥⑦ 谢谢先,不过我想问下这些标准是怎么找的?
①②③④⑤⑥⑦ 5# 2011-10-14 12:58
4# kuing 我也想找几份标准,查ISO, 得知最新版本为ISO 80000,要购买的,不过网上找到了BS的PDF,就看这个了 国家标准要好几个网站能查,然后找到了93年那个,有PDF,早期的六几年的那个找不到,93年的这个是跟着ISO的92年的标准的(也是编号31),现在ISO有09年的那个了,但没看到有新的国标文件代替93版的
kuing 6# 2011-10-14 19:48
我暂时的习惯是类型1
海盗船长 7# 2011-10-14 20:16
类型二见得最多
thread-960-1-1.html: 积分不等式
pxchg1200 1# 2012-12-14 13:04
本帖最后由 pxchg1200 于 2012-12-14 13:06 编辑 For $a\geq 2 $show that: \[ \int_{0}^{+\infty}{\abs{\frac{\sin{x}}{x}}^{a}}\leq \frac{\pi}{\sqrt{2a}} \]
kuing 2# 2012-12-14 18:47
1# pxchg1200 你漏了 dx 没打(可以用 \rmd{x})
Tesla35 3# 2012-12-15 22:11
这个不等式已经有一些说明了: http://www.mysanco.com/wenda/ind ... amp;questionid=1326
╰☆ヾo.海x 4# 2012-12-16 12:54
这个假期我要自学微积分了。。大家多多关照啊,我可能会经常提问哦。。特别是弱智的问题。。。
thread-961-1-5.html: [不等式] 看kk兴致这么高,再弄一个题
pxchg1200 1# 2012-12-14 13:10
Let $a,b,c$ be postive real numbers,with $a^2+b^2+c^2=3 $, Show that: \[ \frac{a}{ab+3}+\frac{b}{bc+3}+\frac{c}{ca+3}\leq \frac{3}{4} \]
pxchg1200 2# 2012-12-18 13:11
1# pxchg1200 kk人呢?
kuing 3# 2012-12-18 14:01
不会……
thread-962-1-5.html: [几何] 椭圆、圆的几个最值
v6mm131 1# 2012-12-15 08:14
如题 突然发现ID是250
yes94 2# 2013-1-10 21:44
自编的啊?好难算哦
thread-963-1-1.html: 来自人教群的一道分式两个根号积分
kuing 1# 2012-12-15 14:47
\[\int\frac{\rmd x}{1+\sqrt{x+1}+\sqrt{x+3}}\] 我的想法是分母有理化,有 \[\frac1{1+\sqrt{x+1}+\sqrt{x+3}}=\frac{2x+3+3\sqrt{x+1}-\sqrt{x+3}-2\sqrt{x+1}\sqrt{x+3}}{4x+3}\] 然后分开,逐个积,前面的好办,就是最后一个有点麻烦,即 \[\int\frac{\sqrt{x+1}\sqrt{x+3}}{4x+3}\rmd x\] 怎么积简单?求教……
kuing 2# 2012-12-15 14:56
群里还提及一个牛逼换元 照我看应该是 \[\sqrt{x+3}=\frac1{\sqrt2}\left(t+\frac1t\right)\text{,}\sqrt{x+1}=\frac1{\sqrt2}\left(t-\frac1t\right)\] 其中 $t\geqslant1$。
kuing 3# 2012-12-15 15:06
或者说更一般地,设常数 $a<b$,对于含 $\sqrt{x+a}$ 和 $\sqrt{x+b}$ 并且 $x$ 能取遍 $[-a,+\infty)$ 的,可作换元 \[\sqrt{x+a}=\frac{\sqrt{b-a}}2\left( t-\frac1t \right)\text{,}\sqrt{x+b}=\frac{\sqrt{b-a}}2\left( t+\frac1t \right)\] 其中 $t\in[1,+\infty)$。
kuing 4# 2012-12-15 15:31
续1#:根号下的二次函数原来有个叫欧拉代换的东东……
椰乡故有情 5# 2012-12-15 15:59
3# kuing 怎么能想得到的?
kuing 6# 2012-12-15 16:02
5# 椰乡故有情 我是想不到的……
kuing 7# 2012-12-15 16:28
倒是让我想到另外的换元方法 令 $\sqrt{x+a}=q$, $\sqrt{x+b}=p$,则 $p\geqslant\sqrt{b-a}$, $q\geqslant0$ 且 $p^2-q^2=b-a>0$,所以由双曲线的参数方程,可令 \[p=\frac{\sqrt{b-a}}{\cos \theta }\text{,}q=\sqrt{b-a}\tan \theta\] 其中 $\theta \in [0,\pi/2)$。 若再用万能代换,设 $t=\tan(\theta/2)$,则 \[p=\sqrt{b-a}\cdot \frac{1+t^2}{1-t^2}\text{,}q=\sqrt{b-a}\cdot \frac{2t}{1-t^2}\] 其中 $t\in[0,1)$。 这样就得到了三角代换和另一种代数代换了
kuing 8# 2012-12-15 16:44
\[p=\sqrt{b-a}\cdot \frac{1+t^2}{1-t^2}\text{,}q=\sqrt{b-a}\cdot \frac{2t}{1-t^2}\] 其中 $t\in[0,1)$。 kuing 发表于 2012-12-15 16:28 这个跟前面3#那个代换可以相通,只要作置换 $t\to 1-\frac2{t+1}$ 即可化简得到。
thread-964-1-1.html: 来自群的一道分式根号指数积分
kuing 1# 2012-12-15 18:16
令 $u=\sqrt{e^x-1}$,则 $x=\ln (u^2+1)$,故 \begin{align*} \int\frac{xe^x}{\sqrt{e^x-1}}\rmd x&=\int\frac{(u^2+1)\ln (u^2+1)}u\rmd{\ln (u^2+1)} \\ & =2\int\ln (u^2+1)\rmd u \\ & =2u\ln (u^2+1)-2\int u\rmd{\ln (u^2+1)} \\ & =2u\ln (u^2+1)-4\int\frac{u^2}{u^2+1}\rmd u \\ & =\cdots \end{align*}
thread-965-1-5.html: 初中什么竞赛啊,有些难度
realnumber 1# 2012-12-16 18:12
--平几我不会. http://sq.k12.com.cn/discuz/thread-627003-1-1.html
realnumber 2# 2012-12-16 18:23
本帖最后由 realnumber 于 2012-12-16 18:31 编辑 5.苯办法是穷举,四个顶点为直角顶点,应该一样多的,最靠近顶点的8个也应该是一样的,...、 4。不是很明白,m=5.n=3似乎也符合,但没答案啊
kuing 3# 2012-12-17 16:20
那道平几倒不是难 如图,$\triangle ABC$ 的内切圆 $I$ 在边 $AB$、$BC$、$CA$ 上的切点分别为 $D$、$E$、$F$,直线 $EF$ 分别与 $AI$、$BI$、$DI$ 交于点 $M$、$N$、$K$。求证:$DM\cdot KE=DN\cdot KF$。 连结 $IE$,$IF$,显然 $\triangle ADM \cong \triangle AFM$,于是 $\triangle MID\cong\triangle MIF$,所以 $\angle IDM=\angle IFM=\angle IEM$,从而 $D$、$I$、$M$、$E$ 四点共圆。 同理可得 $\angle IDN=\angle IEN=\angle IFN$,从而 $D$、$I$、$N$、$F$ 四点共圆,并且可得 $\angle IDM=\angle IDN$,即 $DK$ 为 $\triangle DMN$ 的角平分线,从而 \[\frac{DM}{DN}=\frac{KM}{KN},\] 由共圆得 \[KM\cdot KE=KI\cdot KD=KN\cdot KF,\] 即得 \[\frac{KE}{KF}=\frac{KN}{KM}=\frac{DN}{DM},\] 即得要证的结论。
kuing 4# 2012-12-17 16:42
3# kuing 其实还有 $KE/KF=CA/CB$
Gauss门徒 5# 2012-12-17 16:49
我怎么觉得这平几好难
realnumber 6# 2012-12-17 20:07
平几,基本略过,从没处理过那么复杂图形.
kuing 7# 2012-12-17 22:48
严格来说似乎还应该考虑这种情况 不过也差不多一样。
thread-967-1-1.html: 证明积分不等式
reny 1# 2012-12-16 20:58
设 $f(x)$ 在 $[0,1]$ 上非负递增,求证:存在 $[0,1]$ 上的非负凸函数 $g(x)$ 满足 $g(x)\leqslant f(x)$ 且 \[\int_0^1g(x)\rmd{x}\geqslant \frac12\int_0^1f(x)\rmd{x}.\] _____kuing edit in $\LaTeX$_____
都市侠影 2# 2012-12-18 13:49
不成立吧,当 $f(x)$ 非常极端的时候,是不存在这样的函数的。有空写个例上来。
都市侠影 3# 2012-12-18 13:51
$f(x)$ 有多变态呢,如果 $f(x)$ 凹的太厉害,图像非常靠近 $(1,0)$ 点,类似于反比例图像的时候,符合条件的 $g(x)$ 怕是不存在的
kuing 4# 2012-12-18 14:14
3# 都市侠影 题目中的“凸函数”应该是指下凸吧
kuing 5# 2012-12-18 14:45
直观上是能想象到应该如何构造 $g(x)$ 的,比如这样的 $f(x)$ 那么 $g(x)$ 如下图蓝色线所示。 就是不知如何用数学语言去表达及证明。 那个 $f(x)$ 可以不连续,反正下面的线就是这样绕。 于是从面积来看结果是成立的,线重合的地方积分的面积相同,而不重合的地方,直线积分那面积的两倍肯定不小于上方曲线那积分的面积。 极端的情况,如果允许非严格递增的话,那么比如当 $f(x)=\begin{cases} 0,&x\in[0,0.5],\\ 1,&x\in(0.5,1], \end{cases}$ 时,$g(x)$ 只能是 $g(x)=\begin{cases} 0,&x\in[0,0.5],\\ 2x-1,&x\in(0.5,1]. \end{cases}$ 也符合上面的蓝色绕法。
kuing 6# 2012-12-18 20:59
这样描述 $g(x)$ 行不行呢? 对于每个非负实数 $k$,使 $f(x)\geqslant kx+b$ 在 $[0,1]$ 上恒成立的最大的 $b$ 记为 $b(k)$,那么曲线 $y=g(x)$ 为直线系 $y=kx+b(k)$ 当 $k$ 取遍非负实数时在 $x\in[0,1]$ 上的包络线? 其实我对“包络”的概念不清楚,不知这样说有没有问题……反正 $g(x)$ 就是通过那系列直线围出来的应该是没错的
kuing 7# 2013-1-14 01:24
顶一下, 有答案吗楼主
kuing 8# 2013-4-2 11:42
thread-968-1-4.html: 好久没见“折线和”问题了,这次改编得看上去比之前的难些……
kuing 1# 2012-12-17 19:42
问题:已知 $\theta \in\mbb R$,实数 $x_1$, $x_2$, $x_3$, $x_4$ 满足 $\cos\theta \leqslant x_1\leqslant 2\cos\theta$, $\sin\theta \leqslant x_2\leqslant 2\sin\theta$, $2x_3+x_4-6=0$,则 $\abs{x_1-x_3}+\abs{x_2-x_4}$ 的最小值为 首先显然 $\theta$ 只能是第一象限角,然后画图完事吧……有 \[\abs{x_1-x_3}+\abs{x_2-x_4}=\color{blue}{蓝线}\geqslant\color{red}{红线}\geqslant\color{green}{绿线}=3-\sqrt5\]
realnumber 2# 2012-12-17 20:05
这样也可以,先固定$x_{1},x_{2}$ $\abs{x_{1}-x_{3}}+\abs{x_{2}-6+2x_{3}}=\abs{x_{3}-x_{1}}+\abs{0.5x_{2}-3+x_{3}}+\abs{0.5x_{2}-3+x_{3}}$ 在$x_{3}=3-0.5x_{2}$取最小值,然后就是圆环内动点问题了.
kuing 3# 2012-12-17 21:23
2# realnumber 嗯,帮你把过程写完吧。 易证当 $a$, $b\in\mbb R$, $A>B>0$ 时 $f(x)=A\abs{x-a}+B\abs{x-b}$ 当 $x=a$ 取最小值 $B\abs{a-b}$。 于是 \[\abs{x_1-x_3}+\abs{x_2-x_4}=\abs{x_3-x_1}+2\abs{x_3+\frac12x_2-3}\geqslant\abs{3-\frac12x_2-x_1},\] 而 \[x_1+\frac12x_2\leqslant 2\cos\theta+\sin\theta \leqslant \sqrt5,\] 所以 \[\abs{3-\frac12x_2-x_1}\geqslant3-\sqrt5,\] 当 $x_1=4/\sqrt5$, $x_3=4/\sqrt5+3-\sqrt5$, $x_2=x_4=2/\sqrt5$ 时等号成立。
kuing 4# 2012-12-17 21:48
这样就成功地将图形语言翻译为严格的代数证明鸟
realnumber 5# 2012-12-17 21:51
$a$, $b\in\mbb R$, $A>B>0$ 时 $f(x)=A\abs{x-a}+B\abs{x-b}$ 当 $x=a$ 取最小值 $B\abs{a-b}$。 没想到这个过,我是这样 这一步需要事先做个系列,用数轴上的距离解释最好,$│x-a│+│x-b│,在a\le x\le b$取最小 $│x-a│+│x-b│+│x-c│,(a\le b\le c)$在$x=b$最小,可以推广到奇数个绝对值与偶数个
kuing 6# 2012-12-17 21:56
5# realnumber 折线函数的一般情况何版主在数学空间第1期写过……
yes94 7# 2013-2-1 16:19
$a$, $b\in\mbb R$, $A>B>0$ 时 $f(x)=A\abs{x-a}+B\abs{x-b}$ 当 $x=a$ 取最小值 $B\abs{a-b}$ 写写代码玩玩(基本都是复制的): $f(x)=A\abs{x-a}+B\abs{x-b}\geqslant B\abs{x-a}+B\abs{x-b}=B[\abs{x-a}+\abs{b-x}]\geqslant B[\abs{x-a+b-x}]=B\abs{a-b}$ 当且仅当$A\abs{x-a}=B\abs{x-a}$,且$(x-a)(b-x)\geqslant0$,即$x=a$取等号。
yes94 8# 2013-2-3 11:28
本帖最后由 yes94 于 2013-2-3 12:00 编辑 没事练习下代码玩玩: 上面的思路是消元法,即先消去$x_4$,然后绝对值不等式再消去$x_3$,下面直接消去,其实都一样。 $\abs{x_1-x_3}+\abs{x_2-x_4}\geqslant\abs{x_1-x_3}+\dfrac12\abs{x_2-x_4}\geqslant\abs{x_1-x_3+\dfrac12x_2-\dfrac12x_4}=\abs{x_1+\dfrac12x_2-3}$, 由柯西不等式可得,$(x_1+\dfrac12x_2)^2\leqslant (x_1^2+x_2^2)(1+\dfrac14)\leqslant 4\cdot\dfrac54\leqslant 5$, 于是,$x_1+\dfrac12x_2\leqslant \sqrt5<3$, 故$\abs{\dfrac12x_2+x_1-3}=3-(\dfrac12x_2+x_1)\geqslant3-\sqrt5$. 当且仅当$x_2=x_4$,$(x_1-x_3)(x_2-x_4)\geqslant0$,$x_3+\dfrac12x_4=3$,$x_1=2x_2$,$x_1^2+x_2^2=4$取等号,见3楼。
thread-969-1-1.html: 请教一个求极限的题$\lim_{n \to \infty} \sum_{k=1}^{n} \frac{k}{(k+1)!}$
abababa 1# 2012-12-17 20:31
请教一个求极限的题$\lim_{n \to \infty} \sum_{k=1}^{n} \frac{k}{(k+1)!}$ 感觉和e有关,但是我想不出来,请大家帮帮忙
kuing 2# 2012-12-17 20:37
其实还是高中题吧 \[\frac{k}{(k+1)!}=\frac{k+1-1}{(k+1)!}=\frac1{k!}-\frac1{(k+1)!}\] 故 \[\sum_{k=1}^n\frac{k}{(k+1)!}=1-\frac1{(n+1)!}\]
abababa 3# 2012-12-17 20:42
谢谢 原来这么做的,一开始看到阶乘,就去想e,想错方向了。
thread-97-1-2.html: [网盘HTTP分流]赛车总动员2/汽车总动员2 ||功夫熊猫2(4楼)||美国队长(楼)
isea 1# 2011-10-13 22:58
本帖最后由 isea 于 2011-10-14 23:00 编辑 Cars.2.DVDRip.XviD-iMBT   2CD 转存115网盘于没有水的魚,感谢。 http://115.com/file/bh05imzs# 赛车总动员2.Cars.2.DVDRip.XviD-iMBT-cd1.rar http://115.com/file/dn9c1tvr#  赛车总动员2.Cars.2.DVDRip.XviD-iMBT-cd2.rar QQ离线 Cars.2.DVDRip.XviD-iMBT/CD1, http://url.cn/0oGFUX Cars.2.DVDRip.XviD-iMBT/CD2, http://url.cn/1qmLxA 迅雷文件邮(转载于没有水的鱼) http://wjy.xunlei.com/d/NXZIUNLHNRDY
isea 2# 2011-10-13 23:00
Cars.2.720p.BluRay.x264-METiS  4.4G   转存于115网盘,感谢没有水的鱼 http://115.com/file/cloguelu# http://115.com/file/aqazi08l# http://115.com/file/bh0z69fd# 迅雷文件邮(转载于没有水的鱼) http://wjy.xunlei.com/d/NUHAAJENEGMM
isea 3# 2011-10-13 23:01
本帖最后由 isea 于 2011-10-14 23:01 编辑 简单介绍 《赛车总动员2》主要讲述了闪电麦坤和板牙在世界各地的精彩遭遇。为了决出谁是世界上最快的赛车,他们参加了“世界杯大赛”。 海报:http://images3.static-bluray.com/movies/covers/25785_front.jpg
isea 4# 2011-10-14 10:44
功夫熊猫2 Kung.Fu.Panda.2.DVDRip.XviD-TARGET 115网盘 http://115.com/file/clogs9xy# QQ离线 Kung.Fu.Panda.2.DVDRip.XviD-TARGET,http://url.cn/1nSDrR
isea 5# 2011-10-14 22:59
美国队长/美国队长:复仇者先锋(港) Captain.America.The.First.Avenger.DVDRip.XviD-TARGET 115网盘(转存于馒头@YDY) http://115.com/file/e6rk0ei3# target-catfa-xvid-cd1.rar http://115.com/file/e6rk0m09#  target-catfa-xvid-cd2.rar QQ离线 提醒:将分享到微博的勾勾去掉 http://url.cn/1Hxn6E target-catfa-xvid-cd1.avi http://url.cn/35FUVE target-catfa-xvid-cd2.avi 迅雷文件邮(转载于没有水的鱼) http://wjy.xunlei.com/d/NMJSTRPWNYVT
thread-970-1-5.html: 转发一个问题,关于多项式
realnumber 1# 2012-12-18 08:56
Joseph2338 :有一个$n$次多项式$f(x)$,当$x$取任何整数时$f(x)$的值都是整数的$n$次方,那么是否一定有$f(x)=(ax+b)^n$的形式?其中$a$、$b$都是整数 http://sq.k12.com.cn/discuz/foru ... p;page=1#pid2600039
thread-971-1-5.html: [不等式] 百度贴吧防水题
Gauss门徒 1# 2012-12-19 05:52
\[a,b,c>0\] 证明: \[\frac{a}{b}+\frac{b}{c}+\frac{c}{a}+\frac{3\sqrt[3]{abc}}{a+b+c}\ge 4\]
kuing 2# 2012-12-19 12:10
易证 \[\frac{a}{b}+\frac{b}{c}+\frac{c}{a}\geqslant\frac{a+b+c}{\sqrt[3]{abc}}\]
kuing 3# 2012-12-19 12:13
话说为什么叫防水题??
kuing 4# 2012-12-19 12:52
根据2#的式子,原不等式至少可以加强为 \[\frac ab+\frac bc+\frac ca+\frac{9\sqrt[3]{abc}}{a+b+c}\geqslant6\] 但是我相信 \[\frac ab+\frac bc+\frac ca+\frac{k\sqrt[3]{abc}}{a+b+c}\geqslant3+\frac k3\] 的最大 $k$ 应该不止 9,可能比较难,有空再研究下。
kuing 5# 2012-12-19 14:00
修改了一下楼上,刚才写错了。 PS、bottema 给出的最大 $k$ 是一个七次方程的根,近似值是 24.9238,好大,不过证明我就无能为力了…
thread-972-1-5.html: [函数] 证明f(x)恒为0
reny 1# 2012-12-19 14:36
已知对于任意的x,y都有f(xy)=xf(y)+yf(x),且对任意x,|f(x)|小于等于1,求证f(x)=0恒成立 。
kuing 2# 2012-12-19 15:25
可归纳证明对任意 $n\in\mbb N^+$ 有 $f(x^n)=nx^{n-1}f(x)$,如果存在 $x_0\ne0$ 使 $f(x_0)\ne0$,则令 $n\to\infty$ 便知与 $\abs{f(x)}\leqslant1$ 矛盾。
reny 3# 2012-12-19 22:21
如果是$0<x_{0}<1$,取极限的话,导不出矛盾吧?
kuing 4# 2012-12-19 22:54
3# reny 用 \[0=f(1)=xf\left(\frac1x\right)+\frac1xf(x)\]
hnsredfox_007 5# 2012-12-20 08:24
本帖最后由 hnsredfox_007 于 2012-12-20 08:26 编辑 易得:$(1)f(0)=f(1)=f(-1)=0;(2)f(x)$是奇函数,$(3)f(x^n)=nx^{n-1}f(x), n \in \mathbf N^*$。 由$(2)$可知,只需要证明$f(x)=0,\forall x \in [0,+\infty)$。 当$t\in (0,1)$时,$|f(t)|=|f(x_0^n)|=|nx_0^{n-1}f(x_0)|\leqslant |nx_0^{n-1}|$,取极限得,$f(t)=0$。 当$t\in (1,+\infty)$时,$|f(t^n)|=|nt^{n-1}f(t)|$,于是$|f(t)|=\left |\dfrac{f(t^n)}{nt^{n-1}}\right | \leqslant \left |\dfrac{1}{nt^{n-1}}\right |$,取极限得,$f(t)=0$。 以下略
kuing 6# 2012-12-20 10:04
易得:$(1)f(0)=f(1)=f(-1)=0;(2)f(x)$是奇函数,$(3)f(x^n)=nx^{n-1}f(x), n \in \mathbf N^*$。 由$(2)$可知,只需要证明$f(x)=0,\forall x \in [0,+\infty)$。 当$t\in (0,1)$时,$|f(t)|=|f(x_0^n)|=|nx_0^{n- ... hnsredfox_007 发表于 2012-12-20 08:24 $(0,1)$的证明确问题, 那个$x_0$跟$n$有关, 取极限得不到零。
hnsredfox_007 7# 2012-12-20 10:10
6# kuing 的确是
kuing 8# 2012-12-20 11:37
7# hnsredfox_007 嗯,$(1,+\infty)$ 的没问题,所以也可以拿 4# 的式子来推到 $(0,1)$ 上。
reny 9# 2012-12-20 11:38
本帖最后由 reny 于 2012-12-20 11:39 编辑 用解柯西函数方程得方法不行?易得$\frac{f(x)}{xy}=\frac{f(x)}x+\frac{f(y)}y$,因为是$\abs{f(x)}\le1$,不是$f(x)$连续或单调.
kuing 10# 2012-12-20 11:58
用解柯西函数方程得方法不行?易得$\frac{f(x)}{xy}=\frac{f(x)}x+\frac{f(y)}y$,因为是$\abs{f(x)}\le1$,不是$f(x)$连续或单调. reny 发表于 2012-12-20 11:38 用柯西方程的理论肯定可以,因为柯西方程的非连续解在任何区间内都是无界的
thread-973-1-5.html: [几何] 来自群的焦点弦与抛物线围成面积
kuing 1# 2012-12-19 21:56
爱好/kel/yb緈諨(3977*****)  21:43:55 抛物线y^2=4ax,过焦点的弦与抛物线围成面积的最小值 这道题怎么做简单 群管-kuing/bb/jy/cd  21:45:09 目测 爱好者-林  21:46:17 是不是垂直?面积最小? 群管-kuing/bb/jy/cd  21:47:26 显然的 这样就行了 群管-kuing/bb/jy/cd  21:48:17 一倾斜,左边上升的面积比右边下沉的面积要大
kuing 2# 2012-12-19 22:33
代数法没想到简单方法,用常规的韦达定理…… 不妨设抛物线为 $y=4ax^2$,其中 $a>0$,焦点弦为 $y=kx+a$,与抛物线两交点为 $(x_1,4ax_1^2)$, $(x_2,4ax_2^2)$,其中 $x_1<x_2$,联立方程组后根据韦达定理可得 \[x_1+x_2=\frac{k}{4a}, x_1x_2=-\frac14,x_2-x_1=\sqrt{(x_1+x_2)^2-4x_1x_2}=\frac{\sqrt{k^2+16a^2}}{4a},\] 于是 \begin{align*} S&=\frac12(4ax_1^2-a+4ax_2^2-a)(x_2-x_1)-\int_{x_1}^{x_2}4ax^2\rmd x\\ &=\frac16(x_2-x_1)\bigl(3k(x_1+x_2)-8a(x_1+x_2)^2+8ax_1x_2+6a\bigr)\\ &=\frac16\cdot\frac{\sqrt{k^2+16a^2}}{4a}\left(3k\left(\frac{k}{4a}\right)-8a\left(\frac{k}{4a}\right)^2+4a\right)\\ &=\frac{\sqrt{(k^2+16a^2)^3}}{96a^2}, \end{align*} 所以当且仅当 $k=0$ 时那面积取最小值 $2a/3$。
thread-974-1-5.html: [不等式] 三角不等式
reny 1# 2012-12-19 23:21
本帖最后由 reny 于 2012-12-19 23:30 编辑 已知A ,B ,C 为锐角,$tanAtanBtanC=16\sqrt{2}$,证明:$\frac1{cosA}+\frac1{cosB}+\frac1{cosC}\ge1$
kuing 2# 2012-12-19 23:34
用 \[\frac1{\cos A}=\sqrt{1+\tan^2A}\] 然后均值什么的随便就行了吧
kuing 3# 2012-12-19 23:42
话说目测了一下完全可以证 $\geqslant9$ ?
reny 4# 2012-12-20 12:00
3# kuing 搞错了,是证明$cosA+cosB+cosC\ge1$
kuing 5# 2012-12-20 12:03
4# reny 这样的话好像转化为一个被研究过的结论的特例……
kuing 6# 2012-12-20 12:41
参考这篇东东http://www.docin.com/p-7311951.html
kuing 7# 2012-12-20 13:39
oh,原来数据刚好取的等价于IMO42-2(楼上链接中的文章也有提及) 由条件可令 $\tan A=\frac{\sqrt{8yz}}{x}$, $\tan B=\frac{\sqrt{8zx}}{y}$, $\tan C=\frac{\sqrt{8xy}}{z}$,其中 $x$, $y$, $z>0$,则 \[\sum\cos A=\sum\frac1{\sqrt{1+\tan^2A}}=\sum\frac{x}{\sqrt{x^2+8yz}}\]
thread-975-1-5.html: [不等式] 又一个三角不等式
reny 1# 2012-12-21 10:58
本帖最后由 reny 于 2012-12-21 11:04 编辑 $在△ABC中,求证\sum{tan^{2}}\frac{A}{2}\ge\frac{16}{9}(\sum{sin^{2}}\frac{A}{2})^2$
kuing 2# 2012-12-21 14:35
在 $\triangle ABC$ 中,求证 \[\sum\tan^2\frac A2\geqslant \frac{16}9\left(\sum\sin^2\frac A2\right)^2.\] 记 $\tan \frac A2=x$, $\tan \frac B2=y$, $\tan \frac C2=z$,则 $x$, $y$, $z>0$ 且 $xy+yz+zx=1$,于是 \begin{align*} \sum \sin^2\frac A2&=3-\sum \cos^2\frac A2 \\ & =3-\sum \frac1{1+x^2} \\ & =3-\sum \frac1{xy+yz+zx+x^2} \\ & =3-\sum \frac1{(x+y)(z+x)} \\ & =3-\frac{2\sum x}{\prod(x+y)} \\ & =3-\frac{2\sum x}{\sum x-xyz}, \end{align*} 记 $p=\sum x$,由均值知 $p^2\geqslant 3$,则原不等式等价于 \[p^2-2\geqslant \frac{16}9\left( 3-\frac{2p}{p-xyz} \right)^2.\] 由 Schur 不等式的等价形式 $xyz\geqslant \frac{4p-p^3}9$ 知 \[0<3-\frac{2p}{p-xyz}\leqslant 3-\frac{2p}{p-\frac{4p-p^3}9}=\frac{3(p^2-1)}{p^2+5},\] 故 \[\frac{16}9\left( 3-\frac{2p}{p-xyz} \right)^2\leqslant\frac{16(p^2-1)^2}{(p^2+5)^2},\] 而 \[p^2-2-\frac{16(p^2-1)^2}{(p^2+5)^2}=\frac{(p^2-3)(p^4-5p^2+22)}{(p^2+5)^2}\geqslant 0,\] 所以原不等式成立。
kuing 3# 2012-12-21 16:07
突然想起 G-B 不等式 \[\tan^2\frac A2+\tan^2\frac B2+\tan^2\frac C2\geqslant2-8\sin\frac A2\sin\frac B2\sin\frac C2=2-\frac{2r}R,\] 而 \[\sin^2\frac A2+\sin^2\frac B2+\sin^2\frac C2=1-\frac r{2R},\] 所以只要证 \[2-\frac{2r}R\geqslant \frac{16}9\left(1-\frac r{2R}\right)^2,\] 左右作差知上式等价于 \[\frac{2(R-2r)(R+r)}{9R^2}\geqslant0,\] 显然成立。
reny 4# 2012-12-21 17:23
你的两种解法都用到很多三角结论、恒等变形,以及因式分解。因为对有些结论不熟悉,我看了好一会儿才看明白呢,谢谢你的解答。 问一下:有点儿复杂的因式分解,你是凭什么技巧来分解的呢?
reny 5# 2012-12-21 17:30
本帖最后由 reny 于 2012-12-21 17:58 编辑 在图书馆找到了答案,给出了下面的方法: \begin{align} \sum{tan}^2\frac{A}{2}&\ge\frac49(\sum{sin}^2A)(\sum{tan}^2\frac{A}{2})\notag\\ &=\frac{16}{9}(\sum{sin}^2\frac{A}{2}{cos}^2\frac{A}{2})(\sum\frac{{sin}^2\frac{A}{2}}{{cos}^2\frac{A}{2}})\notag\\ &\ge\frac{16}{9}(\sum{sin}^2\frac{A}{2})^2\notag \end{align} 这种解法用到了$\sum{sin}^2A=3-\sum{cos}^2A=2+2\prod{cosA}\le\frac94$ 编辑这个解答,也顺便$\color{red}{学习}$一下使用$\color{green}{LaTex}$
kuing 6# 2012-12-21 18:21
oh,原来命题的直接舍掉了 $\sum\sin^2A$ ……真简单 那些恒等变形玩多了就会了,甚至条件反射了。因式分解的话有时可以根据取等条件知道会有什么因式。 PS、在代码的输入上你那有不少要改进的,你可以引用我前面的贴子参考一下怎么输入。
reny 7# 2012-12-21 22:04
本帖最后由 reny 于 2012-12-21 22:07 编辑 像这个不等式$\prod(-a+b+c)\le\prod a$是不是很简单得出?
kuing 8# 2012-12-21 22:08
7# reny 这个等价于 Schur 不等式哟
reny 9# 2012-12-21 22:26
本帖最后由 reny 于 2012-12-21 22:27 编辑 刚才查了它是$r=1$时的几种变形,它和$s_1^3-4s_1s_2+9s_3\ge 0$等价,需要$a\ge0,b\ge0,c\ge0$,不等式的一些重要结论用的少,在看书的时候,那些方法中用的结论有时候就不知道怎么来的。
thread-976-1-1.html: 人教群里看到的一道美妙的导数不等式
kuing 1# 2012-12-21 18:06
图片里的斜体太斜了点,以至于导数都看不太清,重新用 $\LaTeX$ 输入一下先。 如果对 $x\in(0,+\infty)$ 都有 $\abs{f(x)}\leqslant A$, $\abs{f''(x)}\leqslant B$,其中 $A$, $B$ 为正常数,证明:$\abs{f'(x)}\leqslant \sqrt{AB}$。 结论真美哟,不过我倒是觉得她更像柯西 怎么证呢?一时没想到
kuing 2# 2012-12-22 01:26
越想越觉得这命题有可能是错的,$\abs{f'(x)}$ 的上界可能要更大些,时间关系明天再研究研究。 大家也可以试试从反方向想,看能不能找到反例。
kuing 3# 2012-12-22 23:11
整理一下,其中用到粉丝群里 px 所说的部分内容。 命题 1  设常数 $a\in\mbb R$,$f(x)$ 定义在 $(a,+\infty)$ 上且二阶可导,若对 $x\in(a,+\infty)$ 有 $\sup\abs{f(x)}=M_0$, $\sup\abs{f'(x)}=M_1$, $\sup\abs{f''(x)}=M_2$,其中 $M_0$, $M_1$, $M_2$ 均为正常数,则有 $M_1\leqslant2\sqrt{M_0M_2}$。 证命题 1  对于每个 $x\in(a,+\infty)$,我们在 $x+2\sqrt{M_0/M_2}$ 处作泰勒展开,有 \[f\left( x+2\sqrt{\frac{M_0}{M_2}} \right)=f(x)+f'(x)\cdot 2\sqrt{\frac{M_0}{M_2}}+\frac{f''(\xi )}2\cdot \left( 2\sqrt{\frac{M_0}{M_2}} \right)^2,\] 其中 $\xi \in \left( x,x+2\sqrt{M_0/M_2} \right)$,于是 \begin{align*} 2\sqrt{\frac{M_0}{M_2}}\cdot \abs{f'(x)}&=\left| f\left( x+2\sqrt{\frac{M_0}{M_2}} \right)-f(x)-\frac{f''(\xi )}2\cdot \left( 2\sqrt{\frac{M_0}{M_2}} \right)^2 \right| \\ & \leqslant \left| f\left( x+2\sqrt{\frac{M_0}{M_2}} \right) \right|+\abs{f(x)}+\frac{2M_0}{M_2}\cdot \abs{f''(\xi )} \\ & \leqslant 4M_0, \end{align*} 所以 \[ \abs{f'(x)}\leqslant 2\sqrt{M_0M_2}, \] 即得 \[M_1\leqslant2\sqrt{M_0M_2}.\] 说明  命题 1 的结论 $M_1\leqslant2\sqrt{M_0M_2}$ 中的等号是可以取到的,比如当 $a=0$ 时,取 \[f(x)=\begin{cases} 2(x-1)^2-1, & x\in (0,1], \\ \frac{x(x-2)}{(x-1)^2+1}, & x\in (1,+\infty ), \end{cases}\] 则可以证明 $f(x)$ 二阶可导,具体地,有 \begin{align*} f'(x)&=\begin{cases} 4(x-1), & x\in (0,1], \\ \frac{4(x-1)}{((x-1)^2+1)^2}, & x\in (1,+\infty ), \end{cases}\\ f''(x)&=\begin{cases} 4, & x\in (0,1], \\ -\frac{4(3x^2-6x+2)}{((x-1)^2+1)^3}, & x\in (1,+\infty ), \end{cases} \end{align*} 此时易证 $M_0=1$, $M_1=M_2=4$,满足 $M_1=2\sqrt{M_0M_2}$。 还有一点要注意的是,此例子中并不存在 $\abs{f'(x)}=4$ 的 $x$,但是 $M_1=4$,这是因为 $\lim_{x\to0^+}\abs{f'(x)}=4$。 而我个人猜测总有 $\abs{f'(x)}<2\sqrt{M_0M_2}$,待研究。 命题 2  (据 px 在群里所给的提示下证出)$f(x)$ 定义在 $\mbb R$ 上且二阶可导,若对 $x\in\mbb R$ 有 $\sup\abs{f(x)}=M_0$, $\sup\abs{f'(x)}=M_1$, $\sup\abs{f''(x)}=M_2$,其中 $M_0$, $M_1$, $M_2$ 均为正常数,则有 $M_1\leqslant \sqrt{2M_0M_2}$。 证命题 2  设任意 $h\ne0$,对于每个 $x\in\mbb R$,分别在 $x+h$ 及 $x-h$ 处作泰勒展开,有 \begin{align*} f(x+h)&=f(x)+f'(x)h+\frac{f''(\xi_1)}2h^2,\\ f(x-h)&=f(x)-f'(x)h+\frac{f''(\xi_2)}2h^2, \end{align*} 其中 $\xi_1\in(\min\{x,x+h\},\max\{x,x+h\})$, $\xi_2\in(\min\{x,x-h\},\max\{x,x-h\})$,两式相减得 \[f(x+h)-f(x-h)=2hf'(x)+\frac{h^2}2\bigl(f''(\xi_1)-f''(\xi_2)\bigr),\] 故此 \begin{align*} 2h\abs{f'(x)}&\leqslant \abs{2hf'(x)}\\ &=\abs{f(x+h)-f(x-h)-\frac{h^2}2\bigl(f''(\xi_1)-f''(\xi_2)\bigr)}\\ &\leqslant \abs{f(x+h)}+\abs{f(x-h)}+\frac{h^2}2\bigl(\abs{f''(\xi_1)}+\abs{f''(\xi_2)}\bigr)\\ &\leqslant 2M_0+h^2M_2, \end{align*} 所以 \[h^2M_2-2h\abs{f'(x)}+2M_0\geqslant 0,\] 上式对任意 $h\ne0$ 恒成立,必然有 \[\Delta=4\abs{f'(x)}^2-8M_0M_2\leqslant 0,\] 所以 \[\abs{f'(x)}\leqslant \sqrt{2M_0M_2},\] 即得 \[M_1\leqslant\sqrt{2M_0M_2}.\] 说明  命题 2 的结论 $M_1\leqslant\sqrt{2M_0M_2}$ 中的等号我猜测是取不了的,待研究。(px 在群里发的例子只定义了在 $(-1,+\infty)$ 上,是用于命题 1 取等反例才对) 但是系数是最佳的,比如说,我们让 $f(x)$ 是一个周期为 $4$ 的函数,其中一个周期 $[0,4)$ 内的表达式如下 \[f(x)=\begin{cases} \frac{x^{\alpha +2}-(a+2)x}{(\alpha +1)(\alpha +2)},&x\in [0,1), \\ \frac{(a+2)(x-2)+(2-x)^{\alpha +2}}{(\alpha +1)(\alpha +2)},&x\in [1,2), \\ \frac{(a+2)(x-2)-(x-2)^{\alpha +2}}{(\alpha +1)(\alpha +2)},&x\in [2,3), \\ \frac{(a+2)(4-x)-(4-x)^{\alpha +2}}{(\alpha +1)(\alpha +2)},&x\in [3,4), \end{cases}\] 其中 $\alpha>0$,那么我们可以证明 $f(x)$ 二阶可导,且它的一阶和二阶导数也以 4 为周期,可以写出它们在 $[0,4)$ 的表达式如下 \begin{align*} f'(x)&=\begin{cases} \frac{x^{\alpha +1}-1}{\alpha +1},&x\in [0,1), \\ \frac{1-(2-x)^{\alpha +1}}{\alpha +1},&x\in [1,2), \\ \frac{1-(x-2)^{\alpha +1}}{\alpha +1},&x\in [2,3), \\ \frac{(4-x)^{\alpha +1}-1}{\alpha +1},&x\in [3,4), \end{cases}\\ f''(x)&=\begin{cases} x^{\alpha },&x\in [0,1), \\ (2-x)^{\alpha },&x\in [1,2), \\ -(x-2)^{\alpha },&x\in [2,3), \\ -(4-x)^{\alpha },&x\in [3,4), \end{cases} \end{align*} 于是此时易证 $M_0=1/(\alpha +2)$, $M_1=1/(\alpha +1)$, $M_2=1$,而 \[2M_0M_2-M_1^2=\frac2{\alpha +2}-\frac1{(\alpha +1)^2}=\frac{\alpha (2\alpha +3)}{(\alpha +2)(\alpha +1)^2},\] 可见当 $\alpha \to 0^+$ 时 $M_1\to\sqrt{2M_0M_2}$,所以系数是最佳的。
思念的温度 4# 2012-12-22 23:56
其实这题是我给天书看的。。。 这个命题有关landua不等式,即函数跟二阶导能够控制一阶导的范围。。。 之前查过资料,sqrt2这个下界是在l_(无穷)空间成立 而这里系数最小为1是在L_2空间成立,但是关于这个证明其实没找到。。。 不知道老师是否有兴趣把证明找出来,或者证明一下。。。
kuing 5# 2012-12-23 00:06
不懂了,什么叫“l_(无穷)空间”和“L_2 空间”?,我对高等的东东了解不多……
thread-977-1-5.html: [几何] 来自人教群的一道椭圆平几题
kuing 1# 2012-12-21 20:03
如图,不用解释了吧……
老人与海 2# 2012-12-23 10:28
将F2点转化为关于角平分线对称的点F2'.那么F2'到F1的距离为2a,那么方程可求得:$(x-2)^2+y^2=a^2$
第一章 3# 2012-12-26 21:04
可以肯定的是,在双曲线中, 则应使AP为内角平分线,同样得点P的轨迹是以原点为圆心,实半轴为半径的圆。 不过,如果考虑椭圆中,AP为内角平分线(或者双曲线中,AP为外角平分线),则轨迹又是什么?
kuing 4# 2012-12-26 23:06
可以肯定的是,在双曲线中, 则应使AP为内角平分线,同样得点P的轨迹是以原点为圆心,实半轴为半径的圆。 不过,如果考虑椭圆中,AP为内角平分线(或者双曲线中,AP为外角平分线),则轨迹又是什么? 第一章 发表于 2012-12-26 21:04 不是常见轨迹,不过倒也不是难算,仍以右焦点为例。 我们延用1#的结论,如图所示。 显然 $APF_2Q$ 是矩形,故易证 $O$, $P$, $Q$ 三点共线。 设椭圆方程为 $x^2/a^2+y^2/b^2=1$, $a>b>0$, $F_2(c,0)$, $c=\sqrt{a^2-b^2}$,再设 $P(a\cos\theta,a\sin\theta)$,则容易求出直线 $OP$ 和 $F_2Q$ 的方程分别为 \begin{align*} L_{OP}&:\sin \theta \cdot x-\cos \theta \cdot y=0, \\ L_{F_2Q}&:(c-a\cos \theta )(x-c)-a\sin \theta \cdot y=0, \end{align*} 于是联立解得 $Q$ 点轨迹的参数方程为 \[\left\{\begin{aligned} x&=\frac{c(c-a\cos \theta )\cos \theta }{c\cos \theta -a}, \\ y&=\frac{c(c-a\cos \theta )\sin \theta }{c\cos \theta -a}, \end{aligned}\right.\] 其中 $\theta$ 为参数且 $\theta\in[0,2\pi)$。
第一章 5# 2012-12-27 10:12
本帖最后由 第一章 于 2012-12-27 10:19 编辑 求Q的时候,想过用参数方程,但孤立了,没想过利用点P,更没想到OPQ共线; 顺便问下,这种曲线有没有一个名称啥的(双曲线的情况,貌似不是封闭曲线了)?
kuing 6# 2012-12-27 10:37
5# 第一章 应该没什么名称吧,暂时也没见过求这种轨迹的题目 现在要出门,晚点回来试试双曲线
kuing 7# 2012-12-27 15:31
看了一下,仍然有共线什么的,左支右支都是,推导是完全一样的,即方程不用变,照代双曲线的 a 和 c 就行。 此时的确不是闭合曲线,因为这时分母能够趋向 0,也就是双曲线上的当动点无穷远时。
thread-978-1-5.html: [组合] 任意抽取三个人,必有一道题的答案互不相同(未解决)
realnumber 1# 2012-12-21 21:35
http://www.aoshoo.com/bbs1/dispb ... ;ID=8170&skin=0 有一个班级学生参加考试,试卷只有六道选择题,每道选择题有三个备选答案。阅卷老师发现任意抽取三个人,必有一道题的答案互不相同。请问该班级最多有多少人? 假设有x人,一个必要条件是,那么第一题选择AB(假定选C的人最少)的人至少有$\frac{2x}{3}$ 第1,2,3,4,5,6题(选较多2个答案的人有$x(\frac{2}{3})^6 \le 2$,解得$x\le \frac{729}{32}$, 这个必要条件略改进点就这样,倒推, 第6题2个,第五题3个,第四题5个,第三题8个,第二题12个,第一题18个.人数就是不超过26人. 继续改进这个办法 第六题1,1,1个(两项和最大的不能达到3或以上,余下类似),第五题2,1,1,个(其中较多两项和为3,不能出现4,余下类似),第四题3,1,1,或2,2,2个,第三题5,1,1或3,3,3个等,第二题5,4,4个等,第一题7,6,6个等.人数就是不超过19人.
realnumber 2# 2012-12-24 15:21
本帖最后由 realnumber 于 2012-12-26 14:24 编辑 1题3人:依次为A;B;C 2题4人:依次为AA;AB;BC;CC; 3题6人:依次为 AAA;                     ABB;                     BCB;                     CCA.                     CBC;                     BAC;--检查了2边应该没问题                    4题9人:依次为  AAAA;                      ABBB;                      BCBA;                      CCAB.                      CBCA;                      BACB;                      BBAC;                      CABC;                      ACCC;--也检查了2边 5题难道也开始手动?,瞎凑了2天,无果,....
thread-979-1-1.html: 公式编辑器
reny 1# 2012-12-21 21:36
Mathematic 5.0 英文版 的公式编辑器关了,怎么调出来啊?
kuing 2# 2012-12-21 21:46
没用过这么低的版本…… 我用的是7.0,是在这里调出来的
reny 3# 2012-12-21 22:00
通过找单词Palettes找到啦,
kuing 4# 2012-12-21 22:11
嗯,有空还是换个新版吧,现在都出到8了。 较新版功能强些,作出来的图也好看很多的。
reny 5# 2012-12-21 22:29
主要是这个软件,网上不容易搞到。
hnsredfox_007 6# 2012-12-25 14:13
4# kuing 嗯,有空还是换个新版吧,现在都出到 9 了。 较新版功能强些,作出来的图也好看很多的。
kuing 7# 2012-12-25 14:29
6# hnsredfox_007 噢?9都有了啊?我又out了
yes94 8# 2013-4-6 20:10
6# hnsredfox_007 噢?9都有了啊?我又out了 kuing 发表于 2012-12-25 14:29 我更out,还不会!
thread-98-1-2.html: [数列] 来自群的数列题,没事求求通项 $x_{n+1}=1+x_n/(p+x_n)$
kuing 1# 2011-10-14 15:59
\[x_{1}=1,x_{n+1}=1+\frac{x_{n}}{p+x_{n}},p>0\] 由 $p>0$ 可令 $p=2\tan t$,其中 $t$ 为锐角,不动点方程 \[x=1+\frac{x}{2\tan t+x}\] 的解为 \[x=1-\tan t\pm \sec t,\] 从而易得 \[\frac{x_{n+1}-\left( 1-\tan t+\sec t \right)}{x_{n+1}-\left( 1-\tan t-\sec t \right)}=\tan \frac{t}{2}\cot \left( \frac{t}{2}+\frac{\pi }{4} \right)\cdot \frac{x_{n}-\left( 1-\tan t+\sec t \right)}{x_{n}-\left( 1-\tan t-\sec t \right)},\] 于是由等比及 $x_1=1$ 得 \[\frac{x_{n}-\left( 1-\tan t+\sec t \right)}{x_{n}-\left( 1-\tan t-\sec t \right)}=\left( \tan \frac{t}{2}\cot \left( \frac{t}{2}+\frac{\pi }{4} \right) \right)^{n-1}\cdot \left( -\cot ^{2}\left( \frac{t}{2}+\frac{\pi }{4} \right) \right),\] 将 $x_n$ 解出并化简整理为 \[x_{n}=\frac{2}{\frac{2\csc t}{\left( \cot \frac{t}{2}\tan \left( \frac{t}{2}+\frac{\pi }{4} \right) \right)^{n}-1}+1+\tan \frac{t}{2}}.\] 由此也不难发现 $x_n$ 是个递增数列,极限为 \[\lim_{n\to+\infty}x_{n}=\frac{2}{1+\tan \frac{t}{2}}=1-\tan t+\sec t,\] 正是不动点的正根。换回 $p$,为 \[1-\frac{p}{2}+\sqrt{1+\frac{p^{2}}{4}},\] 原题中第一问证 $p=2$ 时 $x_n<\sqrt2$,而上式代 $p=2$ 正是 $\sqrt2$,说明要证的这个界就是极限,所以不可再小了。
kuing 2# 2011-10-14 23:08
补发原题: 已知数列$\{x_n\}$中,$x_1=1$,$x_{n+1}=1+\frac{x_n}{p+x_n}$($n\in\mathbf N^+$,$p$是正常数)。 (1)当$p=2$时,用数学归纳法证明$x_n<\sqrt2$; (2)是否存在正整数$M$,使得对于任意正整数$n$,都有$x_M>x_n$?
Tesla35 3# 2013-4-11 22:50
这题貌似没有收录。mark之
hongxian 4# 2013-4-12 09:28
2# kuing 三角代换求不动点,好! 单就原题来说,求通项好象有点不划算!
李斌斌755 5# 2013-4-12 12:09
kuing是出题人而非解题者
hongxian 6# 2013-4-12 18:26
5# 李斌斌755 出题有可能是先有通项再出题了!
kuing 7# 2013-4-12 18:28
我不会出题……
yes94 8# 2013-4-13 22:54
话说本题还有3种方法?
thread-980-1-5.html: [不等式] 不等式证明
reny 1# 2012-12-22 11:34
本帖最后由 reny 于 2012-12-22 16:35 编辑 博客中看到的不等式: $(1)$设$x_1,x_2,……,x_n\in R^{+}$,求证:$$\sum_{i=1}^{n}(\frac{\sum_{j=1}^{i}x_j}{i})^3\le\frac{27}{8}\sum_{i=1}^{n}x_{i}^3$$ $(2)$已知$数列\{a_n\}满足a_n=\frac{1}{(n+1)^2}\sqrt{\frac{n+2}{n+1}},求证$$$\sum_{i=1}^{n}a_i<2(\sqrt{2}-1)$$
kuing 2# 2012-12-22 14:06
第二个 \[ \frac1{(n+1)^2}\sqrt{\frac{n+2}{n+1}}\leqslant \frac{2(n+1)}{3n}\sqrt{\frac{n+1}n}-\frac{2(n+2)}{3(n+1)}\sqrt{\frac{n+2}{n+1}}, \] 从第三项开始放缩。 PS1、这是积分放出来的; PS2、像第一题这么复杂的式子其实可以试试用行间公式……见置顶贴
reny 3# 2012-12-22 16:26
本帖最后由 reny 于 2012-12-22 17:34 编辑 2# kuing 不清楚积分怎么放缩的 \begin{align*} \sum_{i=1}^{n}a_i&\le\frac14\sqrt{\frac32}+\frac19\sqrt{\frac43}+\frac89\sqrt{\frac43}-\frac{2(n+2)}{3(n+1)}\sqrt{\frac{n+2}{n+1}}\\ &<\frac14\sqrt{\frac32}+\frac19\sqrt{\frac43}+\frac89\sqrt{\frac43}-\frac23\\ &<2(\sqrt{2}-1) \end{align*} 积分不等式对于复杂点儿的数列不等式确实管用,主要是求积分。 我觉得右边的值和$\frac{\pi}{12}$比较接近,用$\sum_{i=1}^{n}\frac{1}{2n^2}$来逼近,但是失败了。
kuing 4# 2012-12-22 16:34
参考《数学空间》第3期《数列不等式的定积分解法》: http://www.pep.com.cn/rjwk/gzsxs ... 0110516_1041459.htm
kuing 5# 2012-12-22 17:01
第一个不等式想来想去没证出来,哪里来的? PS、align 环境默认自动编号,如果不想编号可以加星号,即 \(\verb"\begin{align*}...\end{align*}"\)
reny 6# 2012-12-22 17:16
5# kuing http://blog.sina.com.cn/s/blog_4b27095d01017fxt.html 也没有答案 上面那个积分你是不是用换元$t=\sqrt{\frac{x+2}{x+1}}$
kuing 7# 2012-12-22 17:27
当时懒了,用了软件 现在在研究第一个加强,不加强很难数归……
reny 8# 2012-12-22 17:35
7# kuing 哦,换元也很容易求出来的。 用什么软件求的积分?
kuing 9# 2012-12-22 17:50
你不是有 mathematica 么……
reny 10# 2012-12-24 13:06
$\color{red}{不等式(1)有没有人能找到证明方法?}$
kuing 11# 2012-12-26 16:27
10# reny 查到了,第一个不等式是哈代不等式(http://en.wikipedia.org/wiki/Hardy%27s_inequality)当 $p=3$ 时的有限项情形……而且那个系数最佳,难怪一直加强不来,不太易玩……
kuing 12# 2012-12-26 19:12
以下证明是按照 Elliott 对哈代不等式的证明改写的。 对所有 $k\geqslant2$,由均值,有 \begin{align*} & \left( \frac{\sum_{i=1}^kx_i}k \right)^3-\frac32\left( \frac{\sum_{i=1}^kx_i}k \right)^2x_k \\ ={}&\left( \frac{\sum_{i=1}^kx_i}k \right)^3-\frac32\left( k\cdot \frac{\sum_{i=1}^kx_i}k-(k-1)\cdot \frac{\sum_{i=1}^{k-1}x_i}{k-1} \right)\left( \frac{\sum_{i=1}^kx_i}k \right)^2 \\ ={}&\left( \frac{\sum_{i=1}^kx_i}k \right)^3\left( 1-\frac{3k}2 \right)+\frac{3(k-1)}2\left( \frac{\sum_{i=1}^kx_i}k \right)^2\frac{\sum_{i=1}^{k-1}x_i}{k-1} \\ \leqslant{}& \left( \frac{\sum_{i=1}^kx_i}k \right)^3\left( 1-\frac{3k}2 \right)+\frac{k-1}2\left( 2\left( \frac{\sum_{i=1}^kx_i}k \right)^3+\left( \frac{\sum_{i=1}^{k-1}x_i}{k-1} \right)^3 \right) \\ ={}&\frac12\left( (k-1)\left( \frac{\sum_{i=1}^{k-1}x_i}{k-1} \right)^3-k\left( \frac{\sum_{i=1}^kx_i}k \right)^3 \right), \end{align*} 于是上式取 $k$ 从 $2$ 到 $n$ 求和得 \[\sum_{k=2}^n{\left( \frac{\sum_{i=1}^kx_i}k \right)^3}-\frac32\sum_{k=2}^n{\left( \frac{\sum_{i=1}^kx_i}k \right)^2x_k}\leqslant \frac12\left( x_1^3-n\left( \frac{\sum_{i=1}^nx_i}n \right)^3 \right),\] 即 \[ \sum_{k=1}^n{\left( \frac{\sum_{i=1}^kx_i}k \right)^3}-\frac32\sum_{k=1}^n{\left( \frac{\sum_{i=1}^kx_i}k \right)^2x_k}\leqslant -\frac n2\left( \frac{\sum_{i=1}^nx_i}n \right)^3\leqslant 0, \] 故由 Holder 不等式,有 \begin{align*} \sum_{k=1}^n{\left( \frac{\sum_{i=1}^kx_i}k \right)^3}&\leqslant \frac32\sum_{k=1}^n{\left( \frac{\sum_{i=1}^kx_i}k \right)^2x_k} \\ & \leqslant \frac32\left( \sum_{k=1}^n{\left( \frac{\sum_{i=1}^kx_i}k \right)^3} \right)^{2/3}\left( \sum_{k=1}^n{x_k^3} \right)^{1/3}, \end{align*} 所以 \[ \left( \sum_{k=1}^n{\left( \frac{\sum_{i=1}^kx_i}k \right)^3} \right)^{1/3}\leqslant \frac32\left( \sum_{k=1}^n{x_k^3} \right)^{1/3}, \] 即 \[ \sum_{k=1}^n{\left( \frac{\sum_{i=1}^kx_i}k \right)^3}\leqslant \frac{27}8\sum_{k=1}^n{x_k^3}. \] 等号取不了,除非 $x_i$ 全为 $0$。
reny 13# 2012-12-27 08:15
结果是一个挺有难度的不等式,感谢找到了答案
thread-981-1-4.html: [不等式] 翻人教群聊记录发现的一道题(幂指不等式)
kuing 1# 2012-12-23 17:59
题目中那个 $a_1^{a_1}+a_2^{a_2}\geqslant a_1^{a_2}+a_2^{a_1}$ 不知有没有人研究过其 $n$ 元推广?
kuing 2# 2012-12-23 20:33
你说会不会有以下这个成立呢? \[a^a+b^b\geqslant 2\left( \frac{a+b}2 \right)^{\frac{a+b}2}\geqslant a^b+b^a,\] 其中 $a$, $b\in (0,1)$。 又会不会有这个成立呢?:$f(\lambda)=a^{\lambda a+(1-\lambda )b}+b^{\lambda b+(1-\lambda )a}$ 在 $[0,1]$ 上递增? 如果成立,则由 $f(1)\geqslant f(0)$ 即得1#的问题。
kuing 3# 2012-12-23 20:49
又会不会有这个成立呢?:$f(\lambda)=a^{\lambda a+(1-\lambda )b}+b^{\lambda b+(1-\lambda )a}$ 在 $[0,1]$ 上递增? kuing 发表于 2012-12-23 20:33 可惜,这个单调性竟然是不成立的,比如当 $a=0.01$, $b=0.5$ 时,$f(0)\approx 1.09309$, $f(0.2)\approx 1.08491$, 所以并不是恒递增的。
realnumber 4# 2012-12-24 13:09
本帖最后由 realnumber 于 2012-12-24 13:14 编辑 我发过 http://kkkkuingggg.5d6d.net/viewthread.php?tid=170&highlight= 就是为了处理这个,可惜没好思路 http://kkkkuingggg.5d6d.net/thread-898-1-1.html ---对“a>b>0,c>d>0,则$c^a+d^b>c^b+d^a$.   ”给出怎么样关系或限制,就可以成立?
kuing 5# 2012-12-24 13:43
4# realnumber oh,我竟然忘了原来就在本论坛就有讨论让我昨晚还老在各大论坛网站上搜,却没在这儿搜一下…… 哎记xing越来越差了……
realnumber 6# 2012-12-24 21:23
本帖最后由 realnumber 于 2012-12-24 21:24 编辑 不妨设$x \ge y$ $x^x-y^x \ge x^y-y^y$ 等价于$x^x(1-(\frac{y}{x})^x) \ge y^y((\frac{x}{y})^y-1)$ 而$(\frac{y}{x})^x =[1-\frac{x-y}{x}]^x \le 1-x+y$,$(\frac{x}{y})^y=[1+\frac{x-y}{y}]^y \le 1+x-y$ 即$x^x(1-(\frac{y}{x})^x) \ge x^x(x-y)\ge y^y(x-y)\ge y^y((\frac{x}{y})^y-1)$
kuing 7# 2012-12-24 21:34
不妨设$x \ge y$ $x^x-y^x \ge x^y-y^y$ 等价于$x^x(1-(\frac{y}{x})^x) \ge y^y((\frac{x}{y})^y-1)$ 而$(\frac{y}{x})^x =[1-\frac{x-y}{x}]^x \le 1-x+y$,$(\frac{x}{y})^y=[1+\frac{x-y}{y}]^y \le 1+x-y$ 即$x^x(1-(\frac{y}{x})^x) \ge x^x(x-y)\ge y^y(x-y)\ge y^y((\frac{x}{y})^y-1)$ realnumber 发表于 2012-12-24 21:23 $x^x\geqslant y^y$?
realnumber 8# 2012-12-24 21:41
晕,出错,还以为找到简便办法
realnumber 9# 2012-12-24 22:22
本帖最后由 realnumber 于 2012-12-25 15:24 编辑 错的,还在修改, $x,y\in (0,1)$,证明:$x^x +y^y \ge x^y+y^x$. 不妨设$x \ge y$ 记$f(x)=x^x +y^y- x^y-y^x$,$f′(x)=x^x(1+lnx)-yx^{y-1}-y^xlny$, 而$x^xlnx-y^xlny\ge 0$,等价于$(\frac{x}{y})^xlnx\ge lny$,等价于 $x^x\ge yx^{y-1}$等价于$x^{1+x-y}\ge x \ge y$ 所以$f′(x) \ge 0$,而$f(y)=0$,那么当$x\ge y$时,$f(x)\ge 0$,即原不等式成立.---错的,还在修改, 后面是张小明老师发过来的原题配套答案
kuing 10# 2012-12-25 16:58
2#那个加强有没有人证过?
reny 11# 2012-12-25 22:38
9# realnumber $(\frac{x}{y})^xlnx\ge lny等价于x^x\ge yx^{y-1}$怎么来的?
realnumber 12# 2012-12-26 08:07
不等价的;因为证明出错,删去了中间部分,就变这个样子
v6mm131 13# 2012-12-26 08:53
哦 这个韩神的证法最好 韩神的方法可以推广到n元  走向IMO2008给出了牟晓生的一个导数加伯努利的证明
hnsredfox_007 14# 2012-12-26 08:55

realnumber 15# 2012-12-26 15:54
本帖最后由 realnumber 于 2012-12-28 23:24 编辑 几何画板试了下,似乎这个也成立$t,a,b\in R+$,则$a^{ta}+b^{tb}\ge a^{tb}+b^{ta}$,-----(1)如果成立,很容易推广到n元. 不妨设$a\ge b$ 1.a,b至少有一个为1时,很容易验证. 2.$a>1,0<b<1$,那么$a^{ta}> a^{tb},b^{tb}>b^{ta}$,也有(1)成立. 3.$a\ge b>1$,模仿14楼,详细见17楼,$b^{(a-b)t}\ge 1\ge \frac{b}{a}$,所以(1)也成立. 4.$1>a\ge{b}>0,0<t<1$,见17楼也有(1)成立 5.$1>a\ge{b}>0,t>1$不一定成立,比如$a=0.5,b=0.01,t=100$,不等式(1)反向了.
kuing 16# 2012-12-26 17:02
15# realnumber 韩京俊的证法就是证了这一点 在他书上P160
realnumber 17# 2012-12-26 20:01
本帖最后由 realnumber 于 2012-12-28 12:44 编辑 那么这个呢?几何画板试了下,似乎这个也成立$t,a,b\in R+$,则$a^{a^t}+b^{b^t}\ge a^{b^t}+b^{a^t}$ 满足什么条件的正的增函数$y=f(x)$?,$a,b\in (0,1)$,则$a^{f(a)}+b^{f(b)}\ge a^{f(b)}+b^{f(a)}$--成立的一个充分非必要条件是 $b\ge f(b)$, 模仿14楼做法,记$h(x)=x^{f(a)}-x^{f(b)},b\le x\le a\le 1$,不等式成立的一个充分条件是$h′(x)=f(a)x^{f(a)-1}-f(b)x^{f(b)-1}\ge 0$(先增后略微减下也可以啊,就不知道怎么处理) 即$x^{f(a)-f(b)}\ge \frac{f(b)}{f(a)}$,因为$x\in [b,a]$只要$b^{f(a)-f(b)}\ge \frac{f(b)}{f(a)}$,又$b^{f(a)-f(b)}\ge \frac{b}{b+f(a)-f(b)}\ge \frac{f(b)}{f(a)}$(又放了下),得到$(b-f(b))(f(a)-f(b))\ge 0$.完 这样不等式成立 $a,b\in (0,1)$,则$a^{\sin{a}}+b^{\sin{b}}\ge a^{\sin{b}}+b^{\sin{a}}$ 或$a,b\in (0,1)$,则$a^{a^2}+b^{b^2}\ge a^{b^2}+b^{a^2}$ 但是不符合$b\ge f(b)$的,几何画板试了下也成立,看来没找到更一般的结论 比如 $a,b\in (0,1)$,则$a^{\tan{a}}+b^{\tan{b}}\ge a^{\tan{b}}+b^{\tan{a}}$
realnumber 18# 2012-12-26 22:06
本帖最后由 realnumber 于 2012-12-26 22:52 编辑 14楼的这个错的吧 14楼方法行的,这一步$x^{a_1-a_2}-\frac{a_2}{a_1} \ge 0$,只需要证明$x=a_2$即可,而这是成立的即$a^b\ge \frac{a}{a+b}$,见连接http://bbs.pep.com.cn/forum.php? ... p;extra=&page=1
yayaweha 19# 2012-12-26 23:00
插一楼! 这题的条件挺像的!这两题能不能串起来?
kuing 20# 2012-12-26 23:11
19# yayaweha 这个就是加权均值,比1#的应该简单,难度大概不是一个级别的吧……
thread-981-2-4.html:
realnumber 21# 2013-1-6 15:46
本帖最后由 realnumber 于 2013-4-8 15:04 编辑 2楼kuing猜想的证明,见附件. 已知$a,b\in{(0,1)}$,求证:$a^a+b^b\ge{2({\frac{a+b}{2}})^{\frac{a+b}{2}}}\ge{a^b+b^a}$. 证明:前一个用Jensen不等式易证,以下证明后一个.--朱世杰
realnumber 22# 2013-1-6 19:50
可惜韩京俊(92#####95)说了  19:02:00这道陈题了,指上楼的
kuing 23# 2013-1-6 20:19
22# realnumber 意料之中,不过你的证明并没有白干的。暂时没发现有什么错误。
realnumber 24# 2013-1-6 22:42
未证明呢,
realnumber 25# 2013-1-7 15:56
本帖最后由 realnumber 于 2013-1-26 17:10 编辑 没头绪,暂时放弃一段时间 几何画板试了这个$a,b,c\in{(0,1)},3(\frac{a+b+c}{3})^{\frac{a+b+c}{3}}\ge{a^{\frac{b+c}{2}}+b^{\frac{a+c}{2}}+c^{\frac{a+b}{2}}}$,似乎也成立,但依然没找到北.
realnumber 26# 2013-1-30 15:50
本帖最后由 realnumber 于 2013-4-8 14:01 编辑 24# realnumber 走运了,又证明了一个,声明:引用证明请跟上论坛地址,前段时间上面21楼证明被一博客发布,居然去掉了"kuing"的猜想以及帖子地址. \[以下证明              2{\sqrt{ab}}^{\sqrt{ab}}\ge{a^b+b^a},a,b\in(0,1)\] 不妨设$a\ge b$,记$m=\sqrt{ab},t=\sqrt{\frac{a}{b}},a\ge{b},t\ge1,0<m<1$ \[以下证明                 f(t)=(mt)^\frac{m}{t}+{(\frac{m}{t})}^{mt}为减函数.\] \[f'(t)=(mt)^\frac{m}{t}(\frac{m}{t^2}-\frac{m}{t^2}\ln(mt))+{(\frac{m}{t})}^{mt}(-m+m\ln(\frac{m}{t}))\] \[而                (mt)^\frac{m}{t}\frac{m}{t^2}\le{(\frac{m}{t})}^{mt}m\] 等价于$a^bb\le{b^aa}$等价于$b^{1-a}\le a^{1-b}$,等价于$b^{\frac{1}{1-b}}\le{a^{\frac{1}{1-a}}}$等价于$\frac{\ln(b)}{1-b}\le\frac{\ln(a)}{1-a}$ 容易用导数证明$g(x)=\frac{\ln(x)}{1-x},x\ge0$为增函数. \[又               m\ln(\frac{m}{t}){(\frac{m}{t})}^{mt}\le (mt)^\frac{m}{t}\frac{m}{t^2}\ln(mt)\] 等价于$ab^a\ln(b)\le ba^b\ln(a)$等价于\[a^{1-b}\ln(\frac{1}{b})\ge{b^{1-a}\ln(\frac{1}{a})}\] 而之前已经证明$b^{1-a}\ge a^{1-b}>0$,又$\ln(\frac{1}{b})\ge \ln(\frac{1}{a})>0$显然也成立,完.--朱世杰
realnumber 27# 2013-1-30 16:04
本帖最后由 realnumber 于 2013-1-30 17:00 编辑 26# realnumber 难道有这个$a\ge m \ge b$,那么$2m^m\ge a^b+b^a$,这样的话,四个不是一起证明了啊.好象机会....有吗? ---几何画板实验显示,仅有些情况下会成立.
yes94 28# 2013-1-30 16:52
$a^a+b^b\geqslant a^b+b^a>1$成立不? 其中$a,b\in (0,1)$
realnumber 29# 2013-1-30 16:57
28# yes94 前面好几个地方证明了.9楼图片.17楼模仿14楼.21楼也算吧.
realnumber 30# 2013-1-30 20:36
本帖最后由 realnumber 于 2013-2-1 14:04 编辑 26# realnumber 用这个办法对付$a=\sqrt2m\cos{x},b=\sqrt2m\sin{x},m=\sqrt{\frac{a^2+b^2}{2}}$,再次失效. 对付$a=\frac{m}{1+\cos{2α}},b=\frac{m}{1-\cos{2α}},m=\frac{2ab}{a+b}$也失效(就是调和平均).
thread-982-1-5.html: [不等式] 求值域
reny 1# 2012-12-23 21:55
本帖最后由 reny 于 2012-12-23 22:24 编辑 $已知a,b,c\in{R^+},且a+b+c=1,求$$$(\frac{1}{a}-a)(\frac{1}{b}-b)(\frac{1}{c}-c)$$的值域。
kuing 2# 2012-12-23 22:14
将每个分子的 1 都用 $a+b+c$ 代一下就好了呀
reny 3# 2012-12-23 22:26
2# kuing $$\color{red}{原来有点错误,现在改了.}$$
老人与海 4# 2012-12-23 22:50
看这里,有详细解析。
reny 5# 2012-12-23 23:02
4# 老人与海
kuing 6# 2012-12-23 23:13
以下命题已有文献证过 $a_i \in \mbb R^+$($i=1$, $2$, \ldots, $n$, $n \geqslant 3$),$\sum_{i=1}^n a_i = s \leqslant 1$,$k \in \mbb N^+$,则 \[\prod_{i=1}^n \left(\frac1{a_i^k}-a_i^k\right) \geqslant \left(\frac{n^k}{s^k}-\frac{s^k}{n^k}\right)^n.\]
reny 7# 2012-12-23 23:23
6# kuing $\color{green}{这个解答复杂不?}$
kuing 8# 2012-12-23 23:37
7# reny 忘了,6# 的内容其实是我几年前在某论坛里说的,当时也没把具体文献贴出来,一时也没找到放在哪了……
kuing 9# 2012-12-24 15:31
以下命题已有文献证过 $a_i \in \mbb R^+$($i=1$, $2$, \ldots, $n$, $n \geqslant 3$),$\sum_{i=1}^n a_i = s \leqslant 1$,$k \in \mbb N^+$,则 \[\prod_{i=1}^n \left(\frac1{a_i^k}-a_i^k\right) \geqslant \left(\frac{n^k}{s^k}-\frac{s^k}{n^k}\right)^n.\] kuing 发表于 2012-12-23 23:13 先考虑 $s=1$ 时。 下面先证 $k=1$ 的情形。为此,先证明如下引理: 设 $x$, $y>0$, $x+y\leqslant \lambda $,其中 $\lambda $ 为方程 $\lambda^4+16\lambda^2-16=0$ 的唯一正根 $\lambda =2\sqrt{\sqrt5-2}\approx 0.971737$,则有 \[\left( \frac1x-x \right)\left( \frac1y-y \right)\geqslant \left( \frac2{x+y}-\frac{x+y}2 \right)^2.\] 引理的证明:作差有 \begin{align*} & \left( \frac1x-x \right)\left( \frac1y-y \right)-\left( \frac2{x+y}-\frac{x+y}2 \right)^2 \\ ={}&\frac1{xy}-\frac4{(x+y)^2}+xy-\frac{(x+y)^2}4-\frac{x}{y}-\frac{y}{x}+2 \\ ={}&\frac{(x-y)^2}{xy(x+y)^2}-\frac{(x-y)^2}4-\frac{(x-y)^2}{xy} \\ \geqslant {}&\frac{(x-y)^2}{xy\lambda^2}-\frac{(x-y)^2\lambda^2}{4(x+y)^2}-\frac{(x-y)^2}{xy} \\ ={}&(x-y)^2\frac{4(1-\lambda^2)(x+y)^2-\lambda^4xy}{4\lambda^2xy(x+y)^2} \\ ={}&(x-y)^2\frac{4(1-\lambda^2)(x+y)^2-16(1-\lambda^2)xy}{4\lambda^2xy(x+y)^2} \\ ={}&(x-y)^2\frac{4(1-\lambda^2)(x-y)^2}{4\lambda^2xy(x+y)^2} \\ \geqslant {}&0, \end{align*} 故引理得证,等号成立当且仅当 $x=y$。 回到原不等式,由对称性,不妨设 $a_1\leqslant a_2\leqslant \cdots \leqslant a_n$。 (1)若 $a_n<\lambda $,则原不等式左边取最小值时必定是 $a_1=a_2=\cdots=a_n=1/n$ 时,若不然,假设取小值时存在 $a_i\ne a_j$,那么由引理可知,以 $a_i'=a_j'=(a_i+a_j)/2$ 代之可令其更小,矛盾。而当 $a_1=a_2=\cdots=a_n=1/n$ 时正是原不等式取等时,从而原不等式成立; (2)若 $a_n\geqslant \lambda $,则 $a_1\leqslant a_2\leqslant \cdots \leqslant a_{n-1}<1-\lambda$,此时,固定 $a_n$,类似于(1)中的处理方法,可得不等式左边取最小值必有 $a_1=a_2=\cdots =a_{n-1}=(1-a_n)/(n-1)$,所以 \[\prod\limits_{i=1}^n{\left( \frac1{a_i}-a_i \right)}\geqslant \left( \frac{n-1}{1-a_n}-\frac{1-a_n}{n-1} \right)^{n-1}\left( \frac1{a_n}-a_n \right),\] 因为 $1-a_n<1-\lambda <0.1$ 且 $n\geqslant 3$,易证 \[\frac{n-1}{1-a_n}-\frac{1-a_n}{n-1}>10(n-1)-\frac1{10(n-1)}>6\left( n-\frac1n \right),\] 且 \[\left( \frac{n-1}{1-a_n}-\frac{1-a_n}{n-1} \right)\left( \frac1{a_n}-a_n \right)>2,\] 所以 \[\left( \frac{n-1}{1-a_n}-\frac{1-a_n}{n-1} \right)^{n-1}\left( \frac1{a_n}-a_n \right)>2\cdot 6^{n-2}\left( n-\frac1n \right)^{n-2},\] 故此要证原不等式,就只要证 \[2\cdot 6^{n-2}\geqslant \left( n-\frac1n \right)^2,\] 容易验证上式对 $n\geqslant 3$ 恒成立,所以原不等式也成立。 综合(1)(2)可知 $k=1$ 的情形得证。 再证 $k\geqslant 2$ 的情形。由 Carlson 不等式及 $k=1$ 的情形,有 \begin{align*} \prod\limits_{i=1}^n{\left( \frac1{a_i^k}-a_i^k \right)}&=\prod\limits_{i=1}^n{\left( \frac1{a_i}-a_i \right)}\cdot \prod\limits_{i=1}^n{(a_i^{k-1}+a_i^{k-2}+\cdots +a_i^{1-k})} \\ & \geqslant \left( n-\frac1n \right)^n\cdot (t^{k-1}+t^{k-2}+\cdots +t^{1-k})^n, \end{align*} 其中 $t=\sqrt[n]{a_1a_2\cdots a_n}\in (0,1/n]$,而 \begin{align*} f(t)&=t^{k-1}+t^{k-2}+\cdots +t^{1-k} \\ & =t^{k-1}+\frac1{t^{k-1}}+t^{k-2}+\frac1{t^{k-2}}+\cdots, \end{align*} 上式最后一项为 $1$ 或 $t+1/t$,总之,由双勾函数可知 $f(t)$ 在 $(0,1)$ 上递减,从而当 $t=1/n$ 时 $f(t)$ 取最小值,故 \begin{align*} \left( n-\frac1n \right)^n\cdot (t^{k-1}+t^{k-2}+\cdots +t^{1-k})^n&\geqslant \left( n-\frac1n \right)^n\cdot \left( \frac1{n^{k-1}}+\frac1{n^{k-2}}+\cdots +\frac1{n^{1-k}} \right)^n \\ & =\left( n^k-\frac1{n^k} \right)^n, \end{align*} 所以 $k\geqslant 2$ 的情形也得证,即 $s=1$ 时原命题获证。
kuing 10# 2012-12-24 15:57
接楼上,当 $s\leqslant \lambda$ 时只剩情况(1),而 $\lambda<s<1$ 时的情况(2)完全类似可证,所以原命题成立。 其实我怀疑 $k$ 不限制正整数也可能成立,甚至只要正数?再研究一下先。
reny 11# 2012-12-24 19:19
10# kuing 确实有点儿难度,这种方法可以学学。
thread-983-1-5.html: [不等式] sqing老师的2个不等式
realnumber 1# 2012-12-24 08:42
本帖最后由 realnumber 于 2012-12-24 09:02 编辑 http://blog.sina.com.cn/s/blog_4c11310201014paw.html 第2个$a=\cos α,b=\cos β, α,β为锐角$,那么问题等价于 $(\sin β-\sin α)(\sin (β-α)) \ge 0$,总算找到个会的.
kuing 2# 2012-12-24 09:40
第一个难道不是排序不等式一步到位??
kuing 3# 2012-12-24 09:43
第二个其实也是
realnumber 4# 2012-12-24 10:37
就这样被你秒了?可那边似乎很复杂,瞄了下略过.
realnumber 5# 2012-12-24 12:11
kk第一个写写看,没想出你说的办法
reny 6# 2012-12-24 12:22
本帖最后由 reny 于 2012-12-24 12:25 编辑 5# realnumber 不妨设$a\ge b\ge c>0$,则$$a\sqrt{1+a^2}\ge b\sqrt{1+b^2}\ge c\sqrt{1+c^2},\frac{1}{\sqrt{1+a^2}}\le\frac{1}{\sqrt{1+b^2}}\le\frac{1}{\sqrt{1+c^2}}$$ 故由排序不等式($乱序\ge逆序$),$$LHS\ge a+b+c.$$
realnumber 7# 2012-12-24 12:43
本帖最后由 realnumber 于 2012-12-24 12:46 编辑 恩,...明白了,没看仔细,那边附2也有排序
kuing 8# 2012-12-24 12:57
6# reny 这样写不妨设是不好的,因为不等式只是轮换对称,不完全对称。 其实也不必设序,因为由单调性显然无论 $a$, $b$, $c$ 顺序如何,序列 $\bigl\{a\sqrt{1+a^2},b\sqrt{1+b^2},c\sqrt{1+c^2}\bigr\}$ 与 $\Bigl\{\frac{1}{\sqrt{1+a^2}},\frac{1}{\sqrt{1+b^2}},\frac{1}{\sqrt{1+c^2}}\Bigr\}$ 总成反序,所以由乱序和$\geqslant$反序和即得原不等式,这样就可以避免别人揪住你那个不妨设。
reny 9# 2012-12-24 13:03
8# kuing 嗯,是这样的,更严谨些。
realnumber 10# 2013-1-16 20:50
是不是也是排序不等式?那么第一个不等号其实反向了? 若$x\ge y\ge z$,那么$\frac{x}{x+a}\ge\frac{y}{y+a}\ge\frac{z}{z+a}\ge$,以及$x+a\ge y+a \ge z+a$
kuing 11# 2013-1-16 21:22
是不是也是排序不等式?那么第一个不等号其实反向了? 若$x\ge y\ge z$,那么$\frac{x}{x+a}\ge\frac{y}{y+a}\ge\frac{z}{z+a}\ge$,以及$x+a\ge y+a \ge z+a$ realnumber 发表于 2013-1-16 20:50 是的,一样道理,无论 $x$, $y$, $z$ 是何种顺序,由单调性知序列 $\{x+a, y+a, z+a\}$ 与 $\bigl\{\frac x{x+a}, \frac y{y+a}, \frac z{z+a}\bigr\}$ 总是同序的,所以那三个东东 $x+y+z$ 最大。
kuing 12# 2013-1-16 21:45
也有可能是他把题目打错了,因为只要换一下分子分母就有 \[\frac{a+x}{a+y}x+\frac{a+y}{a+z}y+\frac{a+z}{a+x}x\geqslant x+y+z.\]
thread-984-1-1.html: 只显示1页,还是最近1年?
realnumber 1# 2012-12-24 13:07
本帖最后由 realnumber 于 2012-12-24 13:08 编辑 选择自己的主题或回复,只能看到最近1页,去年发的没显示,能否开下,为了查找方便, 我搞错了,从名字点进去,可以看全部
kuing 2# 2012-12-24 13:54
1# realnumber 嗯 用那个纵横搜索还可以搜到别人的主题
realnumber 3# 2013-3-25 15:10
点名字,似乎只能看主题,怎么才能看到全部的回复帖子? 记得回复过一个问题,但不是我自己开的主题.
kuing 4# 2013-3-25 15:37
个人中心里面“我的贴子”里有主题和回复,能显示一些,不全,我也不知为什么
thread-985-1-5.html: [不等式] 横刀夺三角爱
isea 1# 2012-12-24 18:03
本帖最后由 isea 于 2012-12-24 18:49 编辑 经典题目:在三角形$ABC$中,求证$$\cos A+\cos B+\cos C\le \dfrac{3}{2}.$$ 横刀夺爱: (在脑子想个三角形,先), 设$\vv {AB},\vv {BC},\vv {CA}$的单位向量为$\vv a ,\vv b,\vv c$, 则 $\vv a\vv b=-\cos (\pi-B)=-\cos B,$ 同理 $\vv b\vv c=-\cos C,\vv c \vv a=-\cos A,$ 于是 $\cos A+\cos B+\cos C=-\vv a\vv b-\vv b\vv c-\vv c\vv a\\ \cos A+\cos B+\cos C=(\vv a+\vv b+\vv c)^2-3+3(-\vv a\vv b-\vv b\vv c-\vv c\vv a)\\ \cos A+\cos B+\cos C=(\vv a+\vv b+\vv c)^2-3+3(-\cos A+\cos B+\cos C)\\ -2\cos A+\cos B+\cos C=(\vv a+\vv b+\vv c)^2-3\ge-3\\ \cos A+\cos B+\cos C\le \dfrac32$ 取$“="$时当且仅当$\vv a+\vv b+\vv c=\vv 0$, $AB=BC=CA$,亦即三角形此时为等边三角形. 有点意思,来自张景中院士. 顺便打打公式,对齐真不容易啊
kuing 2# 2012-12-24 18:05
这种证法太多的经典题通常我只是当看客的
isea 3# 2012-12-24 18:06
2# kuing 嗯,那个3/2 怎么变大? 一会横刀一下
kuing 4# 2012-12-24 18:09
看置顶中的“一些说明”中的第*4点 PS、 向量可以用 \vv{AB}、\vv a 等,这是我为方便输入而定义的简洁命令; cosA -> \cos A 点乘 \cdot
isea 5# 2012-12-24 18:15
4# kuing 好看多了,不过,中英文混排(公式输入时),痛苦啊,这玩意毕竟是外国人发明的……
kuing 6# 2012-12-24 18:20
5# isea 没办法,我输入的时候也要来回切换。
reny 7# 2012-12-24 18:44
这种方法在彭翕成与张景中老师的《绕来绕去的向量法》$P224$中可以看到,挨着还有用向量法证明$\sum cos^2{A}\ge\frac34$
isea 8# 2012-12-24 18:52
呵呵,的确是 绕来绕去向量法 一书中的,不过,偶只是看到的节选,一会找本全书看看,有点意思 对于∑那样的符号,看着就觉着不懂了.......
kuing 9# 2012-12-24 19:01
话说我翻到两年前这贴 http://bbs.pep.com.cn/forum.php? ... page%3D1&page=3 嵌入不等式也可以这样做
reny 10# 2012-12-24 19:17
8# isea 这本书网上也找得到电子版的,用向量法证明有些题确实有优势,不过对于证明不等式,我个人认为还不是很好。
isea 11# 2012-12-24 19:35
本帖最后由 isea 于 2012-12-24 19:37 编辑 sigh~ 经典题就是不一样,我还以为这个是神来之笔,原来向量这种方法也是经典中的经典 要多“读书”啊 人教链接那个帖子,老早也见过,当时也就一闪而过,现在的高一,正在进行向量的应用,偶见这种不等式的证明,还真是觉得有点意思
thread-986-1-5.html: [不等式] 关于三角函数的值域
reny 1# 2012-12-24 19:44
有没有见过下面这个题的解答方法: $在△ABC中,求$$$y=\frac{sinA+sinB+sinC}{cosA+cosB+cosC}$$的值域。
kuing 2# 2012-12-24 20:12
\[\frac{\sin A+\sin B+\sin C}{\cos A+\cos B+\cos C}=\frac s{R+r}\] $s$ 是半周长,$R$ 和 $r$ 是外接圆和内切圆半径。
reny 3# 2012-12-24 20:51
2# kuing 知道关系$R\ge 2r,但是s怎么办?$
kuing 4# 2012-12-24 20:58
大概要用那个Ger什么什么不等式…… 还是别sRr了,要么化边试试看…… 下确界显然是0(当 $A=B\to0$, $C\to\pi$)所以只要管上确界
reny 5# 2012-12-24 21:36
4# kuing 我曾经搜到了答案,但是挺复杂的啊。希望给出简单的严谨的方法!
kuing 6# 2012-12-24 23:27
5# reny 后面没看懂,我的话后面还是用分类 Schur 吧。 延用楼上前半的结果,直到写出 $y$ 的表达式的第一行,分母根号内的每个 1 用 $uv+vw+wu=1$ 代入,可以化简为 \[y=\frac4{4uvw+(u+v)(v+w)(w+u)}=\frac4{3uvw+p},\] 其中 $p=u+v+w\geqslant\sqrt3$。 (1)若 $p>2$,则显然 $y<4/(3uvw+2)<2$; (2)若 $\sqrt3\leqslant p\leqslant 2$,则由 Schur 不等式有 \[y\leqslant \frac4{\frac{4p-p^3}3+p}=\frac{12}{p(7-p^2)}=f(p),\] 求导得 \[f'(p)=\frac{12(3p^2-7)}{p^2(7-p^2)^2}\geqslant 0,\] 所以 $y\leqslant f(2)=2$,但是等号取不了,因为需要 $p=2$,此时 $4p-p^3=0$,所以 $3uvw\geqslant(4p-p^3)/3$ 无法取等。 综上可知 $y<2$,当 $u$, $v\to1$, $w\to0$ 时 $y\to2$,所以 2 是上确界。
kuing 7# 2012-12-25 00:08
查了一下,刚才4#想说的不等式的名字叫 Gerretsen 不等式,真长,所以一直记不住,只记得前几个字母,对E文真是没感觉。 Gerretsen 不等式的具体表达式是 $4R^2+4Rr+3r^2\geqslant s^2\geqslant 16Rr-5r^2$,是个挺好用的不等式,这里用左边那个,有 \[\frac s{R+r}\leqslant \frac{\sqrt{4R^2+4Rr+3r^2}}{R+r}<\frac{\sqrt{4R^2+8Rr+4r^2}}{R+r}=2,\] 当三角形为等腰三角形且顶角趋向 0 时 $r\to0$, $s\to2R\not\to0$,故 $s/(R+r)\to2$。
reny 8# 2012-12-25 15:20
本帖最后由 reny 于 2012-12-25 15:22 编辑 正如你在前文http://kkkkuingggg.5d6d.net/viewthread.php?tid=975&highlight=中的证明方法, $用Schur不等式证明这类问题很管用呢,第二种方法用到Gerretsen $不等式,还是我初次见到,增长了见识, 你知道的结论也确实非常多!
kuing 9# 2012-12-25 15:25
8# reny 几何不等式的经典结论非常多,几何不等式的发展在中国应该是比较领先的,牛人很多,我也只是略懂一二……
转化与化归 10# 2012-12-29 20:30
三角函数
kuing 11# 2012-12-29 20:42
10# 转化与化归 oh,这样也可以。 不过最好补充一下确界的说明。
转化与化归 12# 2012-12-29 20:51
确界是应该说一下的!
reny 13# 2012-12-30 00:43
这种方法也挺好的,只是对于解答题的话,不太容易想到,因为担心放缩大了。
thread-987-1-5.html: 来自人教论坛的一道空间四边形对角线向量点乘
kuing 1# 2012-12-24 23:51
来源:http://bbs.pep.com.cn/forum.php?mod=viewthread&tid=2628443 这个题的数据太特殊了,$3^2+11^2=7^2+9^2$,竟然刚好是垂直,其实一般情况也能解的。 记 $\vv{AB}=\vv a$, $\vv{BC}=\vv b$, $\vv{CD}=\vv c$, $\vv{DA}=\vv d$,则 $\vv a+\vv b+\vv c+\vv d=\vv0$,且 \begin{align*} 4\vv{AC}\cdot\vv{BD}&=\bigl(\vv{AB}+\vv{BC}+\vv{AD}+\vv{DC}\bigr)\cdot\bigl(\vv{BC}+\vv{CD}+\vv{BA}+\vv{AD}\bigr)\\ &=\bigl(\vv a+\vv b-\vv d-\vv c\bigr)\cdot\bigl(\vv b+\vv c-\vv a-\vv d\bigr)\\ &= -|\vv a|^2+|\vv b|^2-|\vv c|^2+|\vv d|^2+2\vv a\cdot\vv c-2\vv b\cdot\vv d\\ &= -|\vv a|^2+|\vv b|^2-|\vv c|^2+|\vv d|^2+2\vv a\cdot\vv c+2\vv b\cdot\bigl(\vv a+\vv b+\vv c\bigr)\\ &= -|\vv a|^2+|\vv b|^2-|\vv c|^2+|\vv d|^2+2\bigl(\vv a+\vv b\bigr)\cdot\bigl(\vv b+\vv c\bigr)\\ &= -|\vv a|^2+|\vv b|^2-|\vv c|^2+|\vv d|^2+2\vv{AC}\cdot\vv{BD}, \end{align*} 从而解得 \[\vv{AC}\cdot\vv{BD}=\frac12\bigl(-|\vv a|^2+|\vv b|^2-|\vv c|^2+|\vv d|^2\bigr).\]
kuing 2# 2012-12-25 00:20
感觉我这个推导好像还是繁琐了点,不知有没有更简单的?
hongxian 3# 2012-12-25 09:16
本帖最后由 hongxian 于 2012-12-25 11:18 编辑 $\vv{AC} \cdot \vv{BD}=(\vv{AB}+\vv{BC}) \cdot (\vv{BC}+\vv{CD})=\vv{AB} \cdot \vv{BC}+\vv{AB} \cdot \vv{CD}+\vv{BC} \cdot \vv{CD}+\vv{BC}^2$ $\vv{AD}^2=(\vv{AB}+\vv{BC}+\vv{CD})^2=\vv{AB}^2+\vv{BC}^2+\vv{CD}^2+2(\vv{AB} \cdot \vv{BC}+\vv{AB} \cdot \vv{CD}+\vv{BC} \cdot \vv{CD})$ 所以$\vv{AB} \cdot \vv{BC}+\vv{AB} \cdot \vv{CD}+\vv{BC} \cdot \vv{CD}=\dfrac{1}{2}\bigl (\vv{AD}^2-\vv{AB}^2-\vv{BC}^2-\vv{CD}^2)$ 代入上式有$\vv{AC} \cdot \vv{BD}=\dfrac{1}{2}(\vv{AD}^2-\vv{AB}^2-\vv{BC}^2-\vv{CD}^2)+\vv{BC}^2=\dfrac{1}{2} (\vv{AD}^2-\vv{AB}^2+\vv{BC}^2-\vv{CD}^2)$
realnumber 4# 2012-12-25 09:26
本帖最后由 realnumber 于 2012-12-25 13:14 编辑 就是第一步不知道怎么严格点, 引入平行六面体,使得1楼四点是平行六面体其中四个顶点,以同一顶点D为起点的向量设为$\vv{a} \vv{b}\vv{c}$ 那么已知条件是$\abs{\vv{a}+\vv{b}}=11$,$\abs{\vv{a}-\vv{b}}=3$,$\abs{\vv{c}+\vv{b}}=9$,$\abs{\vv{b}-\vv{c}}=7$, 求$(\vv{c}+\vv{a})(\vv{c}-\vv{a})$. 只需要四个等式2个平方和与另外2个平方和的差就可以. 这样就可以,平移DB,使得和AC互相平分,那么六面体就出现了.
kuing 5# 2012-12-25 12:28
3# hongxian 嗯,这个推导也不错
kuing 6# 2012-12-25 13:14
4# realnumber 补上图终于看懂了,也挺牛!
isea 7# 2012-12-28 22:09
感觉我这个推导好像还是繁琐了点,不知有没有更简单的? kuing 发表于 2012-12-25 00:20 可能用两次余弦定理算比较简洁的吧 我来试写一下
isea 8# 2012-12-28 22:13
本帖最后由 isea 于 2013-1-23 22:43 编辑 $\vv {AC}\cdot\vv {BD}=\\ \vv {AB}\cdot\vv {BD}+\vv {BC}\cdot\vv {BD}\\ =-\dfrac{{AB}^2+{BD}^2-{AD}^2}{2}+\dfrac{{BD}^2+{BC}^2-{CD}^2}{2}\\ =…… $ 应该可以化了…… ================== 2013年1月23日 默默更新一个应用。
kuing 9# 2012-12-28 23:46
8# isea oh,牛…… PS、多行公式的输入其实可以试下 align* 环境
isea 10# 2012-12-29 01:20
8# isea oh,牛…… PS、多行公式的输入其实可以试下 align* 环境 kuing 发表于 2012-12-28 23:46 其实一回事,只不过是三个三个分开写的,看起来少一些“字”而已。这个一般结论的推论就是立几里常说的:正四面体的对棱相互垂直。 下次试试 align*
yes94 11# 2012-12-29 22:13
这是斯坦纳四面体公式,可以构造一个四棱锥证明的,四棱锥的底面是一个平行四边形,也就是原来三棱锥的底面三角形补成的平行四边形, 也可以用向量方法:
yes94 12# 2012-12-29 22:21
设O为任一点,这样就毫无技巧啦! 无需动用脑筋,只需不断运算即可,
kuing 13# 2012-12-29 22:23
证法dang来了
yes94 14# 2012-12-29 22:25
证法dang来了 kuing 发表于 2012-12-29 22:23 顺便说下,斯坦纳四面体公式原来是求异面直线的夹角和异面直线的距离的,
thread-988-1-5.html: [不等式] a+b+c+d+e=1
老人与海 1# 2012-12-25 12:43
已知$a+b+c+d+e=1,a,b,c,d,e$全为正数,求$abc+abd+abe+acd+ace+ade+bcd+bce+cde+bde$的取值范围
kuing 2# 2012-12-25 13:30
为方便书写,将五个元记为 $a_i$, $i=1$, $2$, $3$, $4$, $5$,由马克劳林不等式,有 \[\sqrt[3]{\frac{\sum_{1\leqslant i_1<\cdots<i_3\leqslant5}a_{i_1}a_{i_2}a_{i_3}}{C_5^3}}\leqslant \frac{\sum_{i=1}^5a_i}{C_5^1},\] 即得 \[\sum_{1\leqslant i_1<\cdots<i_3\leqslant5}a_{i_1}a_{i_2}a_{i_3}\leqslant \frac2{25}.\] 另一方面,当 $a$, $b$, $c$, $d\to0$, $e\to1$ 时 $\sum_{1\leqslant i_1<\cdots<i_3\leqslant5}a_{i_1}a_{i_2}a_{i_3}\to0$,所以所求的取值范围是 $(0,2/25]$。
老人与海 3# 2012-12-25 14:28
哦,
thread-989-1-1.html: 因式分解
reny 1# 2012-12-26 22:42
本帖最后由 reny 于 2012-12-26 22:44 编辑 $Mathematic中能够因式分解含有除x以外还有其它字母的多项式吗?如x^2+ax+b$
kuing 2# 2012-12-26 23:57
那也要看具体的多项式是什么情况了,你那个不行,但 x^2-a^2 肯定行……
thread-99-1-9.html: [不等式] 来自pep的简单根式不等式求最值
kuing 1# 2011-10-15 16:07
原贴:http://bbs.pep.com.cn/thread-1913050-1-1.html Let $x,y,z\in(0,1)$, if \[\sqrt{\frac{1-x}{yz}}+\sqrt{\frac{1-y}{xz}}+\sqrt{\frac{1-z}{xy}}=2,\] find the maximum of $xyz$. 其实我年多前就解过,马上翻翻记录贴上来 设 $xyz=t$,则由条件得 \[ 2\sqrt t = \sum {\sqrt {x(1 - x)} } = \sqrt 3 \sum {\sqrt {\frac{x} {3}(1 - x)} } \leqslant \sqrt 3 \sum {\frac{{\frac{x} {3} + 1 - x}} {2}} = \sqrt 3 \left( {\frac{3} {2} - \frac{1} {3}\sum x } \right) \leqslant \sqrt 3 \left( {\frac{3} {2} - \sqrt[3]{t}} \right), \] 由此得到 \[ 4t \leqslant 3\left( {\frac{3} {2} - \sqrt[3]{t}} \right)^2, \] 再令 $t=u^3$,代入得 \[ 4u^3 \leqslant 3\left( {\frac{3} {2} - u} \right)^2 \iff \frac{1} {4}(4u - 3)(4u^2 + 9) \leqslant 0 \iff u\leqslant\frac34, \] 所以 \[xyz=t=u^3\leqslant\frac{27}{64},\] 当 $x=y=z=\dfrac34$ 时取等号,故 $xyz$ 的最大值就是 $\dfrac{27}{64}$。
fanwf 2# 2011-10-15 16:16
网页看不清哎,
kuing 3# 2011-10-15 16:17
2# fanwf 嗯?公式显示不出么?
fanwf 4# 2011-10-15 16:30
2# fanwf ok了,搞定了,THANKS kk,
kuing 5# 2011-10-15 17:34
you're welcome
thread-990-1-5.html: [几何] 一道平几,条件很多,能用多少种方法弄出来?
duck 1# 2012-12-26 23:06
ABCD是正方形,BE//AC且CE=AC,延长EC交BA延长线于F,求证AF=AE.
kuing 2# 2012-12-27 15:56
把你知道的方法贴贴撇
duck 3# 2012-12-27 19:41
建系硬上弓、方程解边长。(这两种应该算一种方法) 证线三等分线,即证角是15度,构造30度角的直角三角形就可以了。
kuing 4# 2012-12-27 21:15
3# duck 过 C 作 BE 垂线?
duck 5# 2012-12-27 21:38
4# kuing 嗯,就是这种方法。
kuing 6# 2012-12-27 23:34
5# duck O,那其实如果只是要证角 E 是30度的话用正弦定理也可以,还不用作辅助线。 有了30度,剩下都显然。
duck 7# 2012-12-28 14:13
6# kuing 是的,正弦定理也可,多谢!
thread-991-1-1.html: 级数
froglove 1# 2012-12-28 21:11
本帖最后由 froglove 于 2012-12-28 21:54 编辑 重来
kuing 2# 2012-12-28 21:26
无法理解
froglove 3# 2012-12-28 21:55
等着
kuing 4# 2012-12-28 22:21
右边两个下标都是 n-1 ?
froglove 5# 2012-12-28 23:01
是的。。。。
kuing 6# 2012-12-28 23:27
那我还是不敢做了,因为如果是我出题目,我会将右边两项合并起来。 况且题目还给了前两项
froglove 7# 2012-12-28 23:44
被你一问翻了练习册。。。真的没打错
realnumber 8# 2012-12-28 23:59
估计后一个是n-2,印刷错误也常见啊
froglove 9# 2012-12-29 00:02
有可能。。。。
Zorn 10# 2013-5-6 09:39
用微分方程的方法:即有     f(x)=7.5[x+1]^(2/3)-4.5
thread-992-1-1.html: 级数
froglove 1# 2012-12-28 21:15
题目
kuing 2# 2012-12-28 23:42
分母很大啊 \[\frac{n^{n-1}}{(2n^2+\ln n+1)^{\frac{n+1}2}}<\frac{n^{n-1}}{(2n^2)^{\frac{n+1}2}}<\frac1{n^2}\]
froglove 3# 2012-12-28 23:49
直接被诡异的次方晃花眼了
kuing 4# 2012-12-28 23:49
3# froglove 嗯,这种题比较有聊……
thread-993-1-5.html: [几何] 来自粉丝群的一道椭圆外切四边形证角相等
kuing 1# 2012-12-29 14:42
具体地,如图所示 那么 \begin{align} \angle PFQ=\angle MFN & \iff \angle PFN=\angle QFM \notag \\ & \iff \angle PFA+\angle AFN=\angle QFB+\angle BFM, \label{20121229tywqsbxdjs1} \end{align} 根据已知结论(上面的截图中说的不太清楚,具体地说,就是过椭圆外一点 $X$ 作椭圆两切线,切点分别为 $Y$, $Z$,椭圆某焦点为 $F$,则 $\angle XFY=\angle XFZ$),有 \begin{align*} \angle PFA&=\angle PFD, \\ \angle AFN&=\angle CFN, \\ \angle QFB&=\angle QFD, \\ \angle BFM&=\angle CFM, \end{align*} 所以 \begin{align*} \text{式}~\eqref{20121229tywqsbxdjs1} & \iff \frac12(\angle PFA+\angle PFD+\angle AFN+\angle CFN)=\frac12(\angle QFB+\angle QFD+\angle BFM+\angle CFM) \\ & \iff \frac12(360^\circ -\angle DFC)=\frac12(360^\circ -\angle DFC), \end{align*} 得证。
kuing 2# 2012-12-29 15:04
哎,那个已知结论本来想在人教论坛上找个参考链接的,结果今天人教偏偏上不了,真是……
yes94 3# 2012-12-29 21:27
谢谢版主!
kuing 4# 2013-1-6 21:28
哎,那个已知结论本来想在人教论坛上找个参考链接的,结果今天人教偏偏上不了,真是…… kuing 发表于 2012-12-29 15:04 几乎忘了找链接 http://bbs.pep.com.cn/thread-287613-1-1.html
thread-994-1-5.html: 求角C的范围
老人与海 1# 2012-12-29 16:20
本帖最后由 老人与海 于 2012-12-30 00:33 编辑 $在三角形ABC中,已知ab<\lambda c^2(\lambda \in R^+),\lambda 为常数,求角C的取值范围.$
kuing 2# 2012-12-29 16:46
想象了下应该能取遍 $(0,\pi)$,想一下怎么表达……
kuing 3# 2012-12-29 17:14
如图 作线段 $AB=c$,过 $A$ 作半径为 $r$ 的圆,其中 $r$ 满足 \[r<\min \left\{ c,\frac{-c+c\sqrt{1+4\lambda}}2 \right\},\] 那么该圆与线段 $AB$ 必有交点,且交点在线段 $AB$ 内部。 在圆上任取一点 $C$,则 \[CA\cdot CB\leqslant r\cdot (r+c)<\min \left\{ c,\frac{-c+c\sqrt{1+4\lambda }}2 \right\}\cdot \left( \min \left\{ c,\frac{-c+c\sqrt{1+4\lambda }}2 \right\}+c \right),\] (1)若 $\frac{-c+c\sqrt{1+4\lambda }}2>c$,可以解出 $\lambda >2$,则此时有 \[\min \left\{ c,\frac{-c+c\sqrt{1+4\lambda }}2 \right\}\cdot \left( \min \left\{ c,\frac{-c+c\sqrt{1+4\lambda }}2 \right\}+c \right)=c\cdot 2c<\lambda c^2;\] (2)若 $\frac{-c+c\sqrt{1+4\lambda }}2\leqslant c$,则此时有 \begin{align*} & \min \left\{ c,\frac{-c+c\sqrt{1+4\lambda }}2 \right\}\cdot \left( \min \left\{ c,\frac{-c+c\sqrt{1+4\lambda }}2 \right\}+c \right) \\ ={}&\frac{-c+c\sqrt{1+4\lambda }}2\cdot \left( \frac{-c+c\sqrt{1+4\lambda }}2+c \right) \\ ={}&\lambda c^2. \end{align*} 综合(1)(2)可知,无论 $\lambda $ 取任何正数,总有 \[CA\cdot CB < \lambda c^2,\] 所以,只要 $C$ 不与 $A$, $B$ 共线,那么 $\triangle ABC$ 都满足条件。从而当 $C$ 取遍圆上与 $A$, $B$ 不共线的所有点时,$\angle C$ 取遍 $(0,\pi)$。
老人与海 4# 2012-12-29 18:53
本帖最后由 老人与海 于 2012-12-29 18:57 编辑 $总是感觉角C的范围应该与\lambda有关。当\lambda=1时,若角C等于90度,则c^2=a^2+b^2>ab,矛盾$
kuing 5# 2012-12-29 19:38
$总是感觉角C的范围应该与\lambda有关。当\lambda=1时,若角C等于90度,则c^2=a^2+b^2>ab,矛盾$ 老人与海 发表于 2012-12-29 18:53 矛盾?这不恒成立么?
老人与海 6# 2012-12-29 20:37
5# kuing 我说的矛盾是指与已知条件$ab<c^2矛盾。$
kuing 7# 2012-12-29 20:40
5# kuing 我说的矛盾是指与已知条件$ab<c^2矛盾。$ 老人与海 发表于 2012-12-29 20:37 $a^2+b^2=c^2$ 与 $ab<c^2$ 矛盾?
isea 8# 2012-12-29 22:13
这题目为什么我看上去觉得这么奇怪,怪得都不像是数学题呢
kuing 9# 2012-12-29 22:16
8# isea 呃,难道我理解错题意?
reny 10# 2012-12-29 23:30
$不妨我们就假设\lambda=1,对于ab>c^2,容易求出C\in(0,\frac \pi 3),那对于ab<c^2,角C的范围是多少?$
kuing 11# 2012-12-29 23:34
10# reny 呃,你也看不懂3#……?
reny 12# 2012-12-30 00:31
本帖最后由 reny 于 2012-12-30 00:45 编辑 11# kuing 看懂了过程,只是觉得有点怪,因为当$\lambda=1,对于ab>c^2,$容易求出$C\in(0,\frac \pi 3),而对于ab<c^2$,按你的结果就是$C\in(0, \pi).$  此题其实是个存在性问题,就是求出所有符合$ab<\lambda c^2的以a,b,c为边的三角形的角C的$取值范围
kuing 13# 2012-12-30 01:01
11# kuing 看懂了过程,只是觉得有点怪,因为当$\lambda=1,对于ab>c^2,$容易求出$C\in(0,\frac \pi 3),而对于ab<c^2$,按你的结果就是$C\in(0, \pi).$  ... reny 发表于 2012-12-30 00:31 这并不矛盾的,条件反过来不代表求出的范围不能相容。 比如说 $a=2$, $b=\sqrt3$, $c=1$ 满足 $ab>c^2$ 且 $C=30^\circ$; 而 $a=\dfrac{\sqrt3+\sqrt{15}}4$, $b=\dfrac12$, $c=1$ 满足 $ab<c^2$ 且也有 $C=30^\circ$。
isea 14# 2012-12-30 02:00
8# isea 呃,难道我理解错题意? kuing 发表于 2012-12-29 22:16 我想表达的主要意思是我看不懂这样的题。 k 你应该没偏离题意
thread-995-1-1.html: 看代码
yayaweha 1# 2012-12-29 22:44
 == 证明 ==   ===[[数学归纳法]]===   当<math>n=1</math>;;,   :<math> (a+b)^1 = \sum_{k=0}^1 { 1 \choose k } a^{1-k}b^k = { 1 \choose 0 }a^1b^0+{ 1 \choose 1 }a^0b^1 = a+b</math>   假设二项展开式在 <math>n=m</math> 时成立。若<math>n=m+1</math>;;,   :<math> (a+b)^{m+1}</math> <math> =</math> <math> a(a+b)^m + b(a+b)^m</math>   ::<math> =</math> <math> a \sum_{k=0}^m { m \choose k } a^{m-k} b^k + b \sum_{j=0}^m { m \choose j } a^{m-j} b^j</math>   ::<math> =</math> <math> \sum_{k=0}^m { m \choose k } a^{m-k+1} b^k + \sum_{j=0}^m { m \choose j } a^{m-j} b^{j+1}</math> 将<math>a</math>;;、<math>b</math>;;乘入   ::<math> =</math> <math> a^{m+1} + \sum_{k=1}^m { m \choose k } a^{m-k+1} b^k + \sum_{j=0}^m { m \choose j } a^{m-j} b^{j+1}</math> 取出<math>k=0</math>;;的项   ::<math> =</math> <math> a^{m+1} + \sum_{k=1}^m { m \choose k } a^{m-k+1} b^k + \sum_{k=1}^{m+1} { m \choose k-1 }a^{m-k+1}b^{k}</math> 设<math> j = k-1</math>   ::<math> =</math> <math> a^{m+1} + \sum_{k=1}^m { m \choose k } a^{m-k+1}b^k + \sum_{k=1}^{m} { m \choose k-1 }a^{m+1-k}b^{k} + b^{m+1}</math> 取出<math>k=m+1</math>;;项   ::<math> =</math> <math> a^{m+1} + b^{m+1} + \sum_{k=1}^m \left[ { m \choose k } + { m \choose k-1 } \right] a^{m+1-k}b^k</math> 两者加起   ::<math> =</math> <math> a^{m+1} + b^{m+1} + \sum_{k=1}^m { m+1 \choose k } a^{m+1-k}b^k</math> 套用[[帕斯卡法则]]   ::<math> =</math> <math> \sum_{k=0}^{m+1} { m+1 \choose k } a^{m+1-k}b^k</math>   ===一般形式的证明===   通常二项式定理可以直接使用[[泰勒公式]]进行证明. 下面的方法不使用泰勒公式   <blockquote>   令   <math>f(x)=(1+x)^\alpha</math>,<math>g(x)=\sum_{k=0}^{\infty}{a \choose k}x^k</math>. 注意只有当 <math>|x|<1</math>;;时上述两个函数才收敛   * 首先证明 <math>f(x)</math>;;收敛于1. 这里省略   * 之后,易得<math>f(x)</math>;;满足微分方程:<math>;(1+x)y'(x) = \alpha y(x)</math>. 用求导的一般方法就能得到这个结论,这里省略   * 再证明 <math>g(x)</math>;;亦满足上述微分方程:   <blockquote>   <math>   \begin{align}   g'(x) & = \sum_{k=0}^{\infty}{a\choose k}k x^{k-1}\\   & = \sum_{k=-1}^{\infty}{a\choose (k+1)}(k+1)x^{k} \\   & = {a \choose 0} 0 x^{-1} + \sum_{k=0}^{\infty}{a\choose {k+1}}(k+1) x^{k} \\   & = \sum_{k=0}^{\infty}{a\choose {k+1}}(k+1) x^{k} \\   & = \sum_{k=0}^{\infty}{a \choose k}(a-k) x^k \\   \end{align}   </math>   <math>   \begin{align}   {a \choose {k+1} }(k+1) & = \frac{(a)(a-1)\cdots(a - k + 1)(a - k)}{(k+1)!}(k+1) \\   & = \frac{(a)(a-1)\cdots(a - k + 1)(a - k)}{k!} \\   & = {a \choose k}(a-k)   \end{align}   </math>   </blockquote>   于是 <math>;(1+x)g'(x) = g'(x) + \sum_{k=0}^{\infty}{a \choose k}kx^k = ag(x)</math>   </blockquote>   * 令<math>H(x) = \frac{g(x)}{f(x)}</math>;;,由于<math>g(x)</math>;;和<math>f(x)</math>;;满足同样的微分方程,<math>H'(x) = 0</math>;;,于是<math>H(x)</math>;;是一个常数,即<math>f(x) = ag(x)</math>   * 代入<math>x = 0</math>;;的情况,证明<math>a = 1</math>
yayaweha 2# 2012-12-29 22:45
还是看不懂
kuing 3# 2012-12-29 22:46
??哪里复制过来的?
yayaweha 4# 2012-12-29 23:07
3# kuing 在百度百科看来的,看不懂
kuing 5# 2012-12-29 23:09
4# yayaweha 那估计百科也不知道是复制哪里的代码了,参杂了一些其它代码,而且公式中的括号也不统一,有全角有半角。
kuing 6# 2012-12-29 23:15
实在是太乱了点,本来想试着改一下,还是不了……
kuing 7# 2012-12-29 23:29
原来是复制维基的,看:http://zh.wikipedia.org/wiki/%E4 ... F%E5%AE%9A%E7%90%86 也不知道他是怎么复制的,维基里的明明没有全角的括号,为什么百度百科里的却有呢?不解
yayaweha 8# 2012-12-29 23:42
这下看懂了
thread-996-1-5.html: [几何] 关于线段比例的几何题,提供一种纯几何法 供参考
isea 1# 2012-12-30 19:49
本帖最后由 isea 于 2012-12-31 23:22 编辑 想了好久,纯几何,总觉得差那么一步。 题目:已知A,B为两条定直线AX,BY上的定点,P和R为射线AX上两点,Q和S为射线BY上的两点, $\dfrac{AP}{BQ}=\dfrac{AR}{BS}$为定比,M,N,T分别为线段AB,PQ,RS上的点, $\dfrac{AM}{MB}=\dfrac{PN}{NQ}=\dfrac{RT}{TS}$为另一定比,试问M,N,T三点的位置关系如何?证明你的结论.
kuing 2# 2012-12-30 19:56
我想用复数的说…… 纯平几……玩面积?
isea 3# 2012-12-30 21:00
2# kuing 这个题应该是出自一本叫 高中数学竞赛标准教材 中的习题,网上一搜,什么第八章,其实就这本书。 这题一眼望过去,复数或者向量都比较基础,也“受”用。 94年就有这题了吧,广思集益一下……点共线,我想也许有办法吧。
kuing 4# 2012-12-30 21:02
感觉面积法应该也有机,不过没想出来…… 平几我小弱,先潜水……
isea 5# 2012-12-31 23:09
本帖最后由 isea 于 2012-12-31 23:26 编辑 感觉面积法应该也有机,不过没想出来…… 平几我小弱,先潜水…… kuing 发表于 2012-12-30 21:02 原是在任取一点为原点,用向量可以证出; 后来,又换了一种向量证明,先给个过程。 参考这种向量法,想想其几何意义,似乎搞定了,如图—— 图中,新产生的线,平行而来。 先证两对(现看着像的)“8字型”的三角形分别相似,出C,N,D共线,E,T,F共线。 进而(由红色线段上比值知)CD与EF平行,而两分点N,T的分比(分蓝色线段)是相等的。 于是在三角形MEF中证得M、N、T共线。
kuing 6# 2012-12-31 23:16
有点立体感
isea 7# 2012-12-31 23:18
以复数或向量来看,在空间的确是成立的。
kuing 8# 2013-1-1 15:32
用速度神马的行不行
thread-997-1-5.html: [函数] 貌似不是初中生能理解的问题
realnumber 1# 2012-12-30 21:41
本帖最后由 realnumber 于 2012-12-30 22:28 编辑 北京王**(3762***34) 20:46:56 各位老师,今天一个初中孩子问了一道题我觉得比较有意思,我不知道有哪位老师研究过没有? 在正数域中,问是否存在这样一个正数$m$,对于所有的正数$x$都有$m^x≥x^m$?如果存在请求出符合条件的m的值;如果不存在请证明。 北京王**(3762***34) 20:57:07 此题已有答案,但是没有简要的中学生能看明白的解法。 几何画板探索了下,似乎是m=e就符合.然后是苯办法求了4次导数 $y''''=e^x-e(e-1)(e-2)(e-3)x^{e-4}\ge 0$,说明y'''是增函数 $y'''=e^x-e(e-1)(e-2)x^{e-3}$,y'''符号从负到正,说明y''先减后增 $y''=e^x-e(e-1)x^{e-2}$,y''符号从负到正,说明y'先减后增 $y'=e^x-ex^{e-1}$,y'符号从0,到负再到正,说明y'先减后增 $y=e^x-x^e,x\ge 1$,且x=e处,导数为0,是唯一的极小值,也是最小值,完.---哎呀,这个证明.
kuing 2# 2012-12-30 22:06
初中就算了吧……反正高中就会变得很简单,取个对数,求一次导就行了
realnumber 3# 2012-12-30 22:17
哦,我再来一次,按2楼提示 $f(x)=x-e\ln{x}$,那么$f′(x)=1-\frac{e}{x}$,果然简单, 汗1楼的证明
kuing 4# 2012-12-30 22:20
3# realnumber 导数符号不要用全角的(即中文输入法状态下的)一撇来打,直接在纯英文状态下按回车键左边的那个单引号 ' 就行了
thread-998-1-5.html: [不等式] 几过好多次的四元根式轮换不等式$\sum\sqrt{a/(a+b+c)}$
kuing 1# 2012-12-30 22:15
题目:设 $a$, $b$, $c$, $d>0$,求证 \[\sqrt{\frac a{a+b+c}}+\sqrt{\frac b{b+c+d}}+\sqrt{\frac c{c+d+a}}+\sqrt{\frac d{d+a+b}}\leqslant\frac4{\sqrt3}.\] 这道题见过很多次,不过一时没想起也没查到好的证法,只查到了陈计书上的那个“极品”局部不等式,那个就不贴上来了。 求个好证法,或者相关链接,thank you。
thread-999-1-5.html: [不等式] 求积分证明
依然饭特稀 1# 2012-12-30 22:17
求积分证明方法
kuing 2# 2012-12-30 22:53
用积分证 $1+1/2+1/3+\cdots+1/n>\ln(n+1)$ 然后再证 $\ln(n+1)\geqslant$ 右边,算不算“积分证明方法”?
kuing 3# 2012-12-30 23:24
或者这样吧 当 $n=1$, $2$ 自行验证,当 $n\geqslant3$,则由 \[ \ln \frac{(2e)^n}{n!}=n\ln (2e)-\ln (n!)=n(\ln 2+1)-\sum_{k=1}^n{\ln k}, \] 可知原不等式等价于 \[ \sum_{k=1}^n{\left( \frac2k+\ln k \right)}\geqslant n(\ln 2+1), \] 易证 $f(x)=2/x+\ln x$ 在 $[2,+\infty )$ 上递增,故 \begin{align*} \sum_{k=1}^n{\left( \frac2k+\ln k \right)}&=3+\ln 2+\sum_{k=3}^n{f(k)} \\ & >3+\ln 2+\int_2^n{f(x)\rmd x} \\ & =3+\ln 2+(n+2)\ln n-n-4\ln 2+2 \\ & =5-3\ln 2-n+(n+2)\ln n, \end{align*} 从而只需证 \[ 5-3\ln 2-n+(n+2)\ln n\geqslant n(\ln 2+1), \] 即只需证当 $n\geqslant 3$ 时恒有 \[ g(n)=\ln n-\frac{n(\ln 2+2)-5+3\ln 2}{n+2}\geqslant 0, \] 求导得 \[ g'(n)=\frac{n^2+(\ln 2-5)n+4}{n(n+2)^2}\geqslant \frac{9+3(\ln 2-5)+4}{n(n+2)^2}=\frac{\ln (8/e^2)}{n(n+2)^2}>0, \] 从而 \[ g(n)\geqslant g(3)=\ln 3-\frac{6\ln 2+1}5=\frac15\ln \frac{243}{64e}>0, \] 得证。
hnsredfox_007 4# 2012-12-31 07:50
不用积分方法更简单哦
kuing 5# 2012-12-31 09:53
4# hnsredfox_007 你想说数归?
hnsredfox_007 6# 2012-12-31 10:00
本帖最后由 hnsredfox_007 于 2012-12-31 10:02 编辑 易证$f(x)=\frac{2}{x}+\ln x$在$[1,2]$上单调递减,在$[2,+\infty)$上单调递增,从而$f(n)\geqslant f(2)=1+\ln 2=\ln (2\rm e)$,然后有$$\sum_{i=1}^n f(i)\geqslant n\ln(2\rm e)$$,但是等号不能成立哦
hnsredfox_007 7# 2012-12-31 10:00
5# kuing 不是
kuing 8# 2012-12-31 10:03
oh,原来如此……
依然饭特稀 9# 2012-12-31 11:42
我就是六楼的做法,想寻求积分证法
kuing 10# 2012-12-31 12:36
既然如此何必积分? 不如加强一下再说